Location via proxy:   [ UP ]  
[Report a bug]   [Manage cookies]                
Download as pdf or txt
Download as pdf or txt
You are on page 1of 1492

CHEMISTRY

(OPENSTAX)

Flowers, Theopold and Langley


Chemistry

Flowers, Theopold and Langley


OpenStax
This text is disseminated via the Open Education Resource (OER) LibreTexts Project (https://LibreTexts.org) and like the hundreds
of other texts available within this powerful platform, it is freely available for reading, printing and "consuming." Most, but not all,
pages in the library have licenses that may allow individuals to make changes, save, and print this book. Carefully
consult the applicable license(s) before pursuing such effects.
Instructors can adopt existing LibreTexts texts or Remix them to quickly build course-specific resources to meet the needs of their
students. Unlike traditional textbooks, LibreTexts’ web based origins allow powerful integration of advanced features and new
technologies to support learning.

The LibreTexts mission is to unite students, faculty and scholars in a cooperative effort to develop an easy-to-use online platform
for the construction, customization, and dissemination of OER content to reduce the burdens of unreasonable textbook costs to our
students and society. The LibreTexts project is a multi-institutional collaborative venture to develop the next generation of open-
access texts to improve postsecondary education at all levels of higher learning by developing an Open Access Resource
environment. The project currently consists of 14 independently operating and interconnected libraries that are constantly being
optimized by students, faculty, and outside experts to supplant conventional paper-based books. These free textbook alternatives are
organized within a central environment that is both vertically (from advance to basic level) and horizontally (across different fields)
integrated.
The LibreTexts libraries are Powered by MindTouch® and are supported by the Department of Education Open Textbook Pilot
Project, the UC Davis Office of the Provost, the UC Davis Library, the California State University Affordable Learning Solutions
Program, and Merlot. This material is based upon work supported by the National Science Foundation under Grant No. 1246120,
1525057, and 1413739. Unless otherwise noted, LibreTexts content is licensed by CC BY-NC-SA 3.0.
Any opinions, findings, and conclusions or recommendations expressed in this material are those of the author(s) and do not
necessarily reflect the views of the National Science Foundation nor the US Department of Education.
Have questions or comments? For information about adoptions or adaptions contact info@LibreTexts.org. More information on our
activities can be found via Facebook (https://facebook.com/Libretexts), Twitter (https://twitter.com/libretexts), or our blog
(http://Blog.Libretexts.org).

This text was compiled on 12/05/2022


TABLE OF CONTENTS
Licensing

1: Essential Ideas of Chemistry


1.1: Chemistry in Context
1.2: Phases and Classi cation of Matter
1.3: Physical and Chemical Properties
1.4: Measurements
1.5: Measurement Uncertainty, Accuracy, and Precision
1.6: Mathematical Treatment of Measurement Results
1.E: Essential Ideas of Chemistry (Exercises)

2: Atoms, Molecules, and Ions


2.0: Prelude to Atoms
2.1: Early Ideas in Atomic Theory
2.2: Evolution of Atomic Theory
2.3: Atomic Structure and Symbolism
2.4: Chemical Formulas
2.5: The Periodic Table
2.6: Molecular and Ionic Compounds
2.7: Chemical Nomenclature
2.E: Atoms, Molecules, and Ions (Exercises)

3: Composition of Substances and Solutions


3.1: Formula Mass and the Mole Concept
3.2: Determining Empirical and Molecular Formulas
3.3: Molarity
3.4: Other Units for Solution Concentrations
3.E: Composition of Substances and Solutions (Exercises)

4: Stoichiometry of Chemical Reactions


4.0: Prelude to Stoichiometry
4.1: Writing and Balancing Chemical Equations
4.2: Classifying Chemical Reactions
4.3: Reaction Stoichiometry
4.4: Reaction Yields
4.5: Quantitative Chemical Analysis
4.E: Stoichiometry of Chemical Reactions (Exercises)

5: Thermochemistry
5.0: Prelude to Thermochemistry
5.1: Energy Basics
5.2: Calorimetry
5.3: Enthalpy
5.E: Thermochemistry (Exercises)

1 https://chem.libretexts.org/@go/page/182306
6: Electronic Structure and Periodic Properties
6.1: Electromagnetic Energy
6.2: The Bohr Model
6.3: Development of Quantum Theory
6.4: Electronic Structure of Atoms (Electron Con gurations)
6.5: Periodic Variations in Element Properties
6.E: Electronic Structure and Periodic Properties (Exercises)

7: Chemical Bonding and Molecular Geometry


7.0: Prelude to Chemical Bonding and Molecular Geometry
7.1: Ionic Bonding
7.2: Covalent Bonding
7.3: Lewis Symbols and Structures
7.4: Formal Charges and Resonance
7.5: Strengths of Ionic and Covalent Bonds
7.6: Molecular Structure and Polarity
7.E: Chemical Bonding and Molecular Geometry (Exercises)

8: Advanced Theories of Covalent Bonding


8.0: Prelude to Covalent Bonding
8.1: Valence Bond Theory
8.2: Hybrid Atomic Orbitals
8.3: Multiple Bonds
8.4: Molecular Orbital Theory
8.E: Advanced Theories of Covalent Bonding (Exercises)

9: Gases
9.1: Gas Pressure
9.2: Relating Pressure, Volume, Amount, and Temperature - The Ideal Gas Law
9.3: Stoichiometry of Gaseous Substances, Mixtures, and Reactions
9.4: Effusion and Diffusion of Gases
9.5: The Kinetic-Molecular Theory
9.6: Non-Ideal Gas Behavior
9.E: Gases (Exercises)

10: Liquids and Solids


10.0: Prelude to Liquids and Solids
10.1: Intermolecular Forces
10.2: Properties of Liquids
10.3: Phase Transitions
10.4: Phase Diagrams
10.5: The Solid State of Matter
10.6: Lattice Structures in Crystalline Solids
10.E: Liquids and Solids (Exercises)

11: Solutions and Colloids


11.0: Prelude to Solutions and Colloids
11.1: The Dissolution Process
11.2: Electrolytes

2 https://chem.libretexts.org/@go/page/182306
11.3: Solubility
11.4: Colligative Properties
11.5: Colloids
11.E: Solutions and Colloids (Exercises)

12: Kinetics
12.0: Prelude to Kinetics
12.1: Chemical Reaction Rates
12.2: Factors Affecting Reaction Rates
12.3: Rate Laws
12.4: Integrated Rate Laws
12.5: Collision Theory
12.6: Reaction Mechanisms
12.7: Catalysis
12.E: Kinetics (Exercises)

13: Fundamental Equilibrium Concepts


13.0: Prelude to Equilibrium
13.1: Chemical Equilibria
13.2: Equilibrium Constants
13.3: Shifting Equilibria - Le Chatelier’s Principle
13.4: Equilibrium Calculations
13.E: Fundamental Equilibrium Concepts (Exercises)

14: Acid-Base Equilibria


14.1: Brønsted-Lowry Acids and Bases
14.2: pH and pOH
14.3: Relative Strengths of Acids and Bases
14.4: Hydrolysis of Salt Solutions
14.5: Polyprotic Acids
14.6: Buffers
14.7: Acid-Base Titrations
14.E: Acid-Base Equilibria (Exercises)

15: Equilibria of Other Reaction Classes


15.1: Precipitation and Dissolution
15.2: Lewis Acids and Bases
15.3: Coupled Equilibria
15.E: Equilibria of Other Reaction Classes (Exercises)

16: Thermodynamics
16.1: Spontaneity
16.2: Entropy
16.3: The Second and Third Laws of Thermodynamics
16.4: Gibbs Energy
16.E: Thermodynamics (Exercises)

3 https://chem.libretexts.org/@go/page/182306
17: Electrochemistry
17.1: Balancing Oxidation-Reduction Reactions
17.2: Galvanic Cells
17.3: Standard Reduction Potentials
17.4: The Nernst Equation
17.5: Batteries and Fuel Cells
17.6: Corrosion
17.7: Electrolysis
17.E: Electrochemistry (Exercises)

18: Representative Metals, Metalloids, and Nonmetals


18.1: Periodicity
18.2: Occurrence and Preparation of the Representative Metals
18.3: Structure and General Properties of the Metalloids
18.4: Structure and General Properties of the Nonmetals
18.5: Occurrence, Preparation, and Compounds of Hydrogen
18.6: Occurrence, Preparation, and Properties of Carbonates
18.7: Occurrence, Preparation, and Properties of Nitrogen
18.8: Occurrence, Preparation, and Properties of Phosphorus
18.9: Occurrence, Preparation, and Compounds of Oxygen
18.10: Occurrence, Preparation, and Properties of Sulfur
18.11: Occurrence, Preparation, and Properties of Halogens
18.12: Occurrence, Preparation, and Properties of the Noble Gases
18.E: Representative Metals, Metalloids, and Nonmetals (Exercises)

19: Transition Metals and Coordination Chemistry


19.1: Properties of Transition Metals and Their Compounds
19.2: Coordination Chemistry of Transition Metals
19.3: Optical and Magnetic Properties of Coordination Compounds
19.E: Transition Metals and Coordination Chemistry (Exercises)

20: Organic Chemistry


20.0: Prelude to Organic Chemistry
20.1: Hydrocarbons
20.2: Alcohols and Ethers
20.3: Aldehydes, Ketones, Carboxylic Acids, and Esters
20.4: Amines and Amides
20.E: Organic Chemistry (Exercises)

21: Nuclear Chemistry


21.1: Nuclear Structure and Stability
21.2: Nuclear Equations
21.3: Radioactive Decay
21.4: Transmutation and Nuclear Energy
21.5: Uses of Radioisotopes
21.6: Biological Effects of Radiation
21.E: Nuclear Chemistry (Exercises)

4 https://chem.libretexts.org/@go/page/182306
Appendices
Composition of Commercial Acids and Bases
Essential Mathematics
Formation Constants for Complex Ions
Fundamental Physical Constants
Ionization Constants of Weak Acids
Ionization Constants of Weak Bases
Solubility Products
Standard Electrode (Half-Cell) Potentials
Standard Thermodynamic Properties for Selected Substances
The Periodic Table
Units and Conversion Factors
Water Properties

Index

Glossary

Detailed Licensing

5 https://chem.libretexts.org/@go/page/182306
Licensing
A detailed breakdown of this resource's licensing can be found in Back Matter/Detailed Licensing.

1 https://chem.libretexts.org/@go/page/417133
CHAPTER OVERVIEW
1: Essential Ideas of Chemistry

A general chemistry Libretexts Textbook remixed and remastered from


OpenStax's textbook:
General Chemistry
Most everything you do and encounter during your day involves chemistry. Making coffee, cooking eggs, and toasting bread
involve chemistry. The products you use—like soap and shampoo, the fabrics you wear, the electronics that keep you connected to
your world, the gasoline that propels your car—all of these and more involve chemical substances and processes. Whether you are
aware or not, chemistry is part of your everyday world. In this course, you will learn many of the essential principles underlying the
chemistry of modern-day life.
1.1: Chemistry in Context
1.2: Phases and Classification of Matter
1.3: Physical and Chemical Properties
1.4: Measurements
1.5: Measurement Uncertainty, Accuracy, and Precision
1.6: Mathematical Treatment of Measurement Results
1.E: Essential Ideas of Chemistry (Exercises)

This page titled 1: Essential Ideas of Chemistry is shared under a CC BY 4.0 license and was authored, remixed, and/or curated by OpenStax via
source content that was edited to the style and standards of the LibreTexts platform; a detailed edit history is available upon request.

1
1.1: Chemistry in Context
 Learning Objectives
Outline the historical development of chemistry
Provide examples of the importance of chemistry in everyday life
Describe the scientific method and informs the scientific process
Differentiate among hypotheses, theories, and laws
Provide examples illustrating macroscopic, microscopic, and symbolic domains

Throughout human history, people have tried to convert matter into more useful forms. Our Stone Age ancestors chipped pieces of
flint into useful tools and carved wood into statues and toys. These endeavors involved changing the shape of a substance without
changing the substance itself. But as our knowledge increased, humans began to change the composition of the substances as well
—clay was converted into pottery, hides were cured to make garments, copper ores were transformed into copper tools and
weapons, and grain was made into bread.
Humans began to practice chemistry when they learned to control fire and use it to cook, make pottery, and smelt metals.
Subsequently, they began to separate and use specific components of matter. A variety of drugs such as aloe, myrrh, and opium
were isolated from plants. Dyes, such as indigo and Tyrian purple, were extracted from plant and animal matter. Metals were
combined to form alloys—for example, copper and tin were mixed together to make bronze—and more elaborate smelting
techniques produced iron. Alkalis were extracted from ashes, and soaps were prepared by combining these alkalis with fats.
Alcohol was produced by fermentation and purified by distillation.
Attempts to understand the behavior of matter extend back for more than 2500 years. As early as the sixth century BC, Greek
philosophers discussed a system in which water was the basis of all things. You may have heard of the Greek postulate that matter
consists of four elements: earth, air, fire, and water. Subsequently, an amalgamation of chemical technologies and philosophical
speculations were spread from Egypt, China, and the eastern Mediterranean by alchemists, who endeavored to transform “base
metals” such as lead into “noble metals” like gold, and to create elixirs to cure disease and extend life (Figure 1.1.1).

Figure 1.1.1 : This portrayal shows an alchemist’s workshop circa 1580. Although alchemy made some useful contributions to how
to manipulate matter, it was not scientific by modern standards. (credit: Chemical Heritage Foundation).
A sketch depicts 4 people stirring and handling chemicals. The chemicals are held in a variety of barrels and large cylinders.
Several of the containers are being heated over burning embers. A large stove in the laboratory is filled with burning embers. There
is also a large chest in the corner that is producing steam.
From alchemy came the historical progressions that led to modern chemistry: the isolation of drugs from natural sources,
metallurgy, and the dye industry. Today, chemistry continues to deepen our understanding and improve our ability to harness and
control the behavior of matter. This effort has been so successful that many people do not realize either the central position of
chemistry among the sciences or the importance and universality of chemistry in daily life.

Access for free at OpenStax 1.1.1 https://chem.libretexts.org/@go/page/39654


1.1.1: Chemistry: The Central Science
Chemistry is sometimes referred to as “the central science” due to its interconnectedness with a vast array of other STEM
disciplines (STEM stands for areas of study in the science, technology, engineering, and math fields). Chemistry and the language
of chemists play vital roles in biology, medicine, materials science, forensics, environmental science, and many other fields (Figure
1.1.2). The basic principles of physics are essential for understanding many aspects of chemistry, and there is extensive overlap

between many subdisciplines within the two fields, such as chemical physics and nuclear chemistry. Mathematics, computer
science, and information theory provide important tools that help us calculate, interpret, describe, and generally make sense of the
chemical world. Biology and chemistry converge in biochemistry, which is crucial to understanding the many complex factors and
processes that keep living organisms (such as us) alive. Chemical engineering, materials science, and nanotechnology combine
chemical principles and empirical findings to produce useful substances, ranging from gasoline to fabrics to electronics.
Agriculture, food science, veterinary science, and brewing and wine making help provide sustenance in the form of food and drink
to the world’s population. Medicine, pharmacology, biotechnology, and botany identify and produce substances that help keep us
healthy. Environmental science, geology, oceanography, and atmospheric science incorporate many chemical ideas to help us better
understand and protect our physical world. Chemical ideas are used to help understand the universe in astronomy and cosmology.

Figure 1.1.2 : Knowledge of chemistry is central to understanding a wide range of scientific disciplines. This diagram shows just
some of the interrelationships between chemistry and other fields.
A flowchart shows a box containing chemistry at its center. Chemistry is connected to geochemistry, nuclear chemistry, chemical
physics, nanoscience and nanotechnology, materials science, chemical engineering, biochemistry and molecular biology,
environmental science, agriculture, and mathematics. Each of these disciplines is further connected to other related fields including
medicine, biology, food science, geology earth sciences, toxicology, physics, and computer science.
What are some changes in matter that are essential to daily life? Digesting and assimilating food, synthesizing polymers that are
used to make clothing, containers, cookware, and credit cards, and refining crude oil into gasoline and other products are just a few
examples. As you proceed through this course, you will discover many different examples of changes in the composition and
structure of matter, how to classify these changes and how they occurred, their causes, the changes in energy that accompany them,
and the principles and laws involved. As you learn about these things, you will be learning chemistry, the study of the composition,
properties, and interactions of matter. The practice of chemistry is not limited to chemistry books or laboratories: It happens
whenever someone is involved in changes in matter or in conditions that may lead to such changes.

1.1.2: The Scientific Method


Chemistry is a science based on observation and experimentation. Doing chemistry involves attempting to answer questions and
explain observations in terms of the laws and theories of chemistry, using procedures that are accepted by the scientific community.
There is no single route to answering a question or explaining an observation, but there is an aspect common to every approach:
Each uses knowledge based on experiments that can be reproduced to verify the results. Some routes involve a hypothesis, a
tentative explanation of observations that acts as a guide for gathering and checking information. We test a hypothesis by
experimentation, calculation, and/or comparison with the experiments of others and then refine it as needed.
Some hypotheses are attempts to explain the behavior that is summarized in laws. The laws of science summarize a vast number of
experimental observations, and describe or predict some facet of the natural world. If such a hypothesis turns out to be capable of

Access for free at OpenStax 1.1.2 https://chem.libretexts.org/@go/page/39654


explaining a large body of experimental data, it can reach the status of a theory. Scientific theories are well-substantiated,
comprehensive, testable explanations of particular aspects of nature. Theories are accepted because they provide satisfactory
explanations, but they can be modified if new data become available. The path of discovery that leads from question and
observation to law or hypothesis to theory, combined with experimental verification of the hypothesis and any necessary
modification of the theory, is called the scientific method (Figure 1.1.3).

In this flowchart, the observation and curiosity box has an arrow pointing to a box labeled form hypothesis; make prediction. A
curved arrow labeled next connects this box to a box labeled perform experiment; make more observations. Another arrow points
back to the box that says form hypothesis; make prediction. This arrow is labeled results not consistent with prediction. Another
arrow, labeled results are consistent with prediction points from the perform experiment box to a box labeled contributes to body of
knowledge. However, an arrow also points from contributes to body of knowledge back to the form hypothesis; make prediction
box. This arrow is labeled further testing does not support hypothesis. There are also two other arrows leading out from contributes
to body of knowledge. One arrow is labeled much additional testing yields constant observations. This leads to the observation
becomes law box. The other arrow is labeled much additional testing supports hypothesis. This arrow leads to the hypothesis
becomes theory box.
Figure 1.1.3 : The scientific method follows a process similar to the one shown in this diagram. All the key components are shown,
in roughly the right order. Scientific progress is seldom neat and clean: It requires open inquiry and the reworking of questions and
ideas in response to findings.

1.1.3: The Domains of Chemistry


Chemists study and describe the behavior of matter and energy in three different domains: macroscopic, microscopic, and
symbolic. These domains provide different ways of considering and describing chemical behavior.
Macro is a Greek word that means “large.” The macroscopic domain is familiar to us: It is the realm of everyday things that are
large enough to be sensed directly by human sight or touch. In daily life, this includes the food you eat and the breeze you feel on
your face. The macroscopic domain includes everyday and laboratory chemistry, where we observe and measure physical and
chemical properties, or changes such as density, solubility, and flammability.
The microscopic domain of chemistry is almost always visited in the imagination. Micro also comes from Greek and means
“small.” Some aspects of the microscopic domains are visible through a microscope, such as a magnified image of graphite or
bacteria. Viruses, for instance, are too small to be seen with the naked eye, but when we’re suffering from a cold, we’re reminded
of how real they are.
However, most of the subjects in the microscopic domain of chemistry—such as atoms and molecules—are too small to be seen
even with standard microscopes and often must be pictured in the mind. Other components of the microscopic domain include ions
and electrons, protons and neutrons, and chemical bonds, each of which is far too small to see. This domain includes the individual
metal atoms in a wire, the ions that compose a salt crystal, the changes in individual molecules that result in a color change, the
conversion of nutrient molecules into tissue and energy, and the evolution of heat as bonds that hold atoms together are created.
The symbolic domain contains the specialized language used to represent components of the macroscopic and microscopic
domains. Chemical symbols (such as those used in the periodic table), chemical formulas, and chemical equations are part of the
symbolic domain, as are graphs and drawings. We can also consider calculations as part of the symbolic domain. These symbols

Access for free at OpenStax 1.1.3 https://chem.libretexts.org/@go/page/39654


play an important role in chemistry because they help interpret the behavior of the macroscopic domain in terms of the components
of the microscopic domain. One of the challenges for students learning chemistry is recognizing that the same symbols can
represent different things in the macroscopic and microscopic domains, and one of the features that makes chemistry fascinating is
the use of a domain that must be imagined to explain behavior in a domain that can be observed.
A helpful way to understand the three domains is via the essential and ubiquitous substance of water. That water is a liquid at
moderate temperatures, will freeze to form a solid at lower temperatures, and boil to form a gas at higher temperatures (Figure
1.1.4) are macroscopic observations. But some properties of water fall into the microscopic domain—what we cannot observe with
the naked eye. The description of water as comprised of two hydrogen atoms and one oxygen atom, and the explanation of freezing
and boiling in terms of attractions between these molecules, is within the microscopic arena. The formula H2O, which can describe
water at either the macroscopic or microscopic levels, is an example of the symbolic domain. The abbreviations (g) for gas, (s) for
solid, and (l) for liquid are also symbolic.

Figure 1.1.4 : (a) Moisture in the air, icebergs, and the ocean represent water in the macroscopic domain. (b) At the molecular level
(microscopic domain), gas molecules are far apart and disorganized, solid water molecules are close together and organized, and
liquid molecules are close together and disorganized. (c) The formula H2O symbolizes water, and (g), (s), and (l) symbolize its
phases. Note that clouds are actually comprised of either very small liquid water droplets or solid water crystals; gaseous water in
our atmosphere is not visible to the naked eye, although it may be sensed as humidity. (credit a: modification of work by
“Gorkaazk”/Wikimedia Commons).
Figure A shows a photo of an iceberg floating in a sea has three arrows. Each arrow points to figure B, which contains three
diagrams showing how the water molecules are organized in the air, ice, and sea. In the air, which contains the gaseous form of
water, H subscript 2 O gas, the water molecules are disconnected and widely spaced. In the ice, which is the solid form of water, H
subscript 2 O solid, the water molecules are bonded together into rings, with each ring containing six water molecules. Three of
these rings are connected to each other. In the sea, which is the liquid form of water, H subscript 2 O liquid, the water molecules are
very densely packed. The molecules are not bonded together.

1.1.4: Key Concepts and Summary


Chemistry deals with the composition, structure, and properties of matter, and the ways by which various forms of matter may be
interconverted. Thus, it occupies a central place in the study and practice of science and technology. Chemists use the scientific
method to perform experiments, pose hypotheses, and formulate laws and develop theories, so that they can better understand the
behavior of the natural world. To do so, they operate in the macroscopic, microscopic, and symbolic domains. Chemists measure,
analyze, purify, and synthesize a wide variety of substances that are important to our lives.

Glossary
chemistry
study of the composition, properties, and interactions of matter

hypothesis
tentative explanation of observations that acts as a guide for gathering and checking information

law
statement that summarizes a vast number of experimental observations, and describes or predicts some aspect of the natural
world

Access for free at OpenStax 1.1.4 https://chem.libretexts.org/@go/page/39654


macroscopic domain
realm of everyday things that are large enough to sense directly by human sight and touch

microscopic domain
realm of things that are much too small to be sensed directly

scientific method
path of discovery that leads from question and observation to law or hypothesis to theory, combined with experimental
verification of the hypothesis and any necessary modification of the theory

symbolic domain
specialized language used to represent components of the macroscopic and microscopic domains, such as chemical symbols,
chemical formulas, chemical equations, graphs, drawings, and calculations

theory
well-substantiated, comprehensive, testable explanation of a particular aspect of nature

This page titled 1.1: Chemistry in Context is shared under a CC BY 4.0 license and was authored, remixed, and/or curated by OpenStax via
source content that was edited to the style and standards of the LibreTexts platform; a detailed edit history is available upon request.

Access for free at OpenStax 1.1.5 https://chem.libretexts.org/@go/page/39654


1.2: Phases and Classification of Matter
 Learning Objectives
Describe the basic properties of each physical state of matter: solid, liquid, and gas.
Define and give examples of atoms and molecules.
Classify matter as an element, compound, homogeneous mixture, or heterogeneous mixture with regard to its physical state
and composition.
Use symbolic, particulate, or macroscopic representations to describe or classify the different types of matter.
Distinguish between mass and weight.
Apply the law of conservation of matter.

Matter is defined as anything that occupies space and has mass, and it is all around us. Solids and liquids are more obviously
matter: We can see that they take up space, and their weight tells us that they have mass. Gases are also matter; if gases did not take
up space, a balloon would stay collapsed rather than inflate when filled with gas.
Solids, liquids, and gases are the three states of matter commonly found on earth (Figure 1.2.1). A solid is rigid and possesses a
definite shape. A liquid flows and takes the shape of a container, except that it forms a flat or slightly curved upper surface when
acted upon by gravity. (In zero gravity, liquids assume a spherical shape.) Both liquid and solid samples have volumes that are very
nearly independent of pressure. A gas takes both the shape and volume of its container.

Figure 1.2.1 : The three most common states or phases of matter are solid, liquid, and gas.
A beaker labeled solid contains a cube of red matter and says has fixed shape and volume. A beaker labeled liquid contains a
brownish-red colored liquid. This beaker says takes shape of container, forms horizontal surfaces, has fixed volume. The beaker
labeled gas is filled with a light brown gas. This beaker says expands to fill container.
A fourth state of matter, plasma, occurs naturally in the interiors of stars. A plasma is a gaseous state of matter that contains
appreciable numbers of electrically charged particles (Figure 1.2.2). The presence of these charged particles imparts unique
properties to plasmas that justify their classification as a state of matter distinct from gases. In addition to stars, plasmas are found
in some other high-temperature environments (both natural and man-made), such as lightning strikes, certain television screens,
and specialized analytical instruments used to detect trace amounts of metals.

Access for free at OpenStax 1.2.1 https://chem.libretexts.org/@go/page/39648


Figure 1.2.2 : A plasma torch can be used to cut metal. (credit: “Hypertherm”/Wikimedia Commons)
A cutting torch is being used to cut a piece of metal. Bright, white colored plasma can be seen near the tip of the torch, where it is
contacting the metal.

What is Plasma?

Video 1.2.1 : In a tiny cell in a plasma television, the plasma emits ultraviolet light, which in turn causes the display at that
location to appear a specific color. The composite of these tiny dots of color makes up the image that you see. Watch this video to
learn more about plasma and the places you encounter it.
Some samples of matter appear to have properties of solids, liquids, and/or gases at the same time. This can occur when the sample
is composed of many small pieces. For example, we can pour sand as if it were a liquid because it is composed of many small
grains of solid sand. Matter can also have properties of more than one state when it is a mixture, such as with clouds. Clouds appear
to behave somewhat like gases, but they are actually mixtures of air (gas) and tiny particles of water (liquid or solid).
The mass of an object is a measure of the amount of matter in it. One way to measure an object’s mass is to measure the force it
takes to accelerate the object. It takes much more force to accelerate a car than a bicycle because the car has much more mass. A
more common way to determine the mass of an object is to use a balance to compare its mass with a standard mass.
Although weight is related to mass, it is not the same thing. Weight refers to the force that gravity exerts on an object. This force is
directly proportional to the mass of the object. The weight of an object changes as the force of gravity changes, but its mass does
not. An astronaut’s mass does not change just because she goes to the moon. But her weight on the moon is only one-sixth her
earth-bound weight because the moon’s gravity is only one-sixth that of the earth’s. She may feel “weightless” during her trip when
she experiences negligible external forces (gravitational or any other), although she is, of course, never “massless.”
The law of conservation of matter summarizes many scientific observations about matter: It states that there is no detectable
change in the total quantity of matter present when matter converts from one type to another (a chemical change) or changes
among solid, liquid, or gaseous states (a physical change). Brewing beer and the operation of batteries provide examples of the
conservation of matter (Figure 1.2.4). During the brewing of beer, the ingredients (water, yeast, grains, malt, hops, and sugar) are
converted into beer (water, alcohol, carbonation, and flavoring substances) with no actual loss of substance. This is most clearly
seen during the bottling process, when glucose turns into ethanol and carbon dioxide, and the total mass of the substances does not
change. This can also be seen in a lead-acid car battery: The original substances (lead, lead oxide, and sulfuric acid), which are

Access for free at OpenStax 1.2.2 https://chem.libretexts.org/@go/page/39648


capable of producing electricity, are changed into other substances (lead sulfate and water) that do not produce electricity, with no
change in the actual amount of matter.

Figure 1.2.3 : (a) The mass of beer precursor materials is the same as the mass of beer produced: Sugar has become alcohol and
carbonation. (b) The mass of the lead, lead oxide plates, and sulfuric acid that goes into the production of electricity is exactly
equal to the mass of lead sulfate and water that is formed.
Diagram A shows a beer bottle containing pre-beer and sugar. An arrow points from this bottle to a second bottle. This second
bottle contains the same volume of liquid, however, the sugar has been converted into ethanol and carbonation as beer was made.
Diagram B shows a car battery that contains sheets of P B and P B O subscript 2 along with H subscript 2 S O subscript 4. After the
battery is used, it contains an equal mass of P B S O subscript 4 and H subscript 2 O.
Although this conservation law holds true for all conversions of matter, convincing examples are few and far between because,
outside of the controlled conditions in a laboratory, we seldom collect all of the material that is produced during a particular
conversion. For example, when you eat, digest, and assimilate food, all of the matter in the original food is preserved. But because
some of the matter is incorporated into your body, and much is excreted as various types of waste, it is challenging to verify by
measurement.

1.2.1: Atoms and Molecules


An atom is the smallest particle of an element that has the properties of that element and can enter into a chemical combination.
Consider the element gold, for example. Imagine cutting a gold nugget in half, then cutting one of the halves in half, and repeating
this process until a piece of gold remained that was so small that it could not be cut in half (regardless of how tiny your knife may
be). This minimally sized piece of gold is an atom (from the Greek atomos, meaning “indivisible”) (Figure 1.2.4). This atom would
no longer be gold if it were divided any further.

Figure 1.2.4 : (a) This photograph shows a gold nugget. (b) A scanning-tunneling microscope (STM) can generate views of the
surfaces of solids, such as this image of a gold crystal. Each sphere represents one gold atom. (credit a: modification of work by
United States Geological Survey; credit b: modification of work by “Erwinrossen”/Wikimedia Commons)
Figure A shows a gold nugget as it would appear to the naked eye. The gold nugget is very irregular, with many sharp edges. It
appears gold in color. The microscope image of a gold crystal shows many similarly sized gold stripes that are separated by dark
areas. Looking closely, one can see that the gold stripes are made of many, tiny, circular atoms.
The first suggestion that matter is composed of atoms is attributed to the Greek philosophers Leucippus and Democritus, who
developed their ideas in the 5th century BCE. However, it was not until the early nineteenth century that John Dalton (1766–1844),
a British schoolteacher with a keen interest in science, supported this hypothesis with quantitative measurements. Since that time,
repeated experiments have confirmed many aspects of this hypothesis, and it has become one of the central theories of chemistry.

Access for free at OpenStax 1.2.3 https://chem.libretexts.org/@go/page/39648


Other aspects of Dalton’s atomic theory are still used but with minor revisions (details of Dalton’s theory are provided in the
chapter on atoms and molecules).
An atom is so small that its size is difficult to imagine. One of the smallest things we can see with our unaided eye is a single thread
of a spider web: These strands are about 1/10,000 of a centimeter (0.0001 cm) in diameter. Although the cross-section of one strand
is almost impossible to see without a microscope, it is huge on an atomic scale. A single carbon atom in the web has a diameter of
about 0.000000015 centimeter, and it would take about 7000 carbon atoms to span the diameter of the strand. To put this in
perspective, if a carbon atom were the size of a dime, the cross-section of one strand would be larger than a football field, which
would require about 150 million carbon atom “dimes” to cover it. (Figure 1.2.5) shows increasingly close microscopic and atomic-
level views of ordinary cotton.

Figure 1.2.5 : These images provide an increasingly closer view: (a) a cotton boll, (b) a single cotton fiber viewed under an optical
microscope (magnified 40 times), (c) an image of a cotton fiber obtained with an electron microscope (much higher magnification
than with the optical microscope); and (d and e) atomic-level models of the fiber (spheres of different colors represent atoms of
different elements). (credit c: modification of work by “Featheredtar”/Wikimedia Commons)
Figure A shows a puffy white cotton boll growing on a brown twig. Figure B shows a magnified cotton strand. The strand appears
transparent but contains dark areas within its interior. Figure C shows the surface of several crisscrossing and overlapping cotton
fibers. Its surface is rough along the edges but smooth near the center of each strand. Figure D shows three strands of molecules
connected into three vertical chains. Each strand contains about five molecules. Figure E shows that the cotton molecule contains
about a dozen atoms. The black carbon atoms form rings that are connected by red oxygen atoms. Many of the carbon atoms are
also bonded to hydrogen atoms, shown as white balls, or other oxygen atoms.
An atom is so light that its mass is also difficult to imagine. A billion lead atoms (1,000,000,000 atoms) weigh about 3 × 10 −13

grams, a mass that is far too light to be weighed on even the world’s most sensitive balances. It would require over
300,000,000,000,000 lead atoms (300 trillion, or 3 × 1014) to be weighed, and they would weigh only 0.0000001 gram.
It is rare to find collections of individual atoms. Only a few elements, such as the gases helium, neon, and argon, consist of a
collection of individual atoms that move about independently of one another. Other elements, such as the gases hydrogen, nitrogen,
oxygen, and chlorine, are composed of units that consist of pairs of atoms (Figure 1.2.6). One form of the element phosphorus
consists of units composed of four phosphorus atoms. The element sulfur exists in various forms, one of which consists of units
composed of eight sulfur atoms. These units are called molecules. A molecule consists of two or more atoms joined by strong
forces called chemical bonds. The atoms in a molecule move around as a unit, much like the cans of soda in a six-pack or a bunch
of keys joined together on a single key ring. A molecule may consist of two or more identical atoms, as in the molecules found in
the elements hydrogen, oxygen, and sulfur, or it may consist of two or more different atoms, as in the molecules found in water.
Each water molecule is a unit that contains two hydrogen atoms and one oxygen atom. Each glucose molecule is a unit that
contains 6 carbon atoms, 12 hydrogen atoms, and 6 oxygen atoms. Like atoms, molecules are incredibly small and light. If an
ordinary glass of water were enlarged to the size of the earth, the water molecules inside it would be about the size of golf balls.

Access for free at OpenStax 1.2.4 https://chem.libretexts.org/@go/page/39648


Figure 1.2.6 : The elements hydrogen, oxygen, phosphorus, and sulfur form molecules consisting of two or more atoms of the same
element. The compounds water, carbon dioxide, and glucose consist of combinations of atoms of different elements.
The hydrogen molecule, H subscript 2, is shown as two small, white balls bonded together. The oxygen molecule O subscript 2, is
shown as two red balls bonded together. The phosphorous molecule, P subscript 4, is shown as four orange balls bonded tightly
together. The sulfur molecule, S subscript 8, is shown as 8 yellow balls linked together. Water molecules, H subscript 2 O, consist
of one red oxygen atom bonded to two smaller white hydrogen atoms. The hydrogen atoms are at an angle on the oxygen molecule.
Carbon dioxide, C O subscript 2, consists of one carbon atom and two oxygen atoms. One oxygen atom is bonded to the carbon’s
right side and the other oxygen is bonded to the carbon’s left side. Glucose, C subscript 6 H subscript 12 O subscript 6, contains a
chain of carbon atoms that have attached oxygen or hydrogen atoms.

1.2.2: Classifying Matter


We can classify matter into several categories. Two broad categories are mixtures and pure substances. A pure substance has a
constant composition. All specimens of a pure substance have exactly the same makeup and properties. Any sample of sucrose
(table sugar) consists of 42.1% carbon, 6.5% hydrogen, and 51.4% oxygen by mass. Any sample of sucrose also has the same
physical properties, such as melting point, color, and sweetness, regardless of the source from which it is isolated.
We can divide pure substances into two classes: elements and compounds. Pure substances that cannot be broken down into simpler
substances by chemical changes are called elements. Iron, silver, gold, aluminum, sulfur, oxygen, and copper are familiar examples
of the more than 100 known elements, of which about 90 occur naturally on the earth, and two dozen or so have been created in
laboratories.
Pure substances that can be broken down by chemical changes are called compounds. This breakdown may produce either elements
or other compounds, or both. Mercury(II) oxide, an orange, crystalline solid, can be broken down by heat into the elements
mercury and oxygen (Figure 1.2.7). When heated in the absence of air, the compound sucrose is broken down into the element
carbon and the compound water. (The initial stage of this process, when the sugar is turning brown, is known as caramelization—
this is what imparts the characteristic sweet and nutty flavor to caramel apples, caramelized onions, and caramel). Silver(I) chloride
is a white solid that can be broken down into its elements, silver and chlorine, by absorption of light. This property is the basis for
the use of this compound in photographic films and photochromic eyeglasses (those with lenses that darken when exposed to light).

Figure 1.2.7 : (a)The compound mercury(II) oxide, (b)when heated, (c) decomposes into silvery droplets of liquid mercury and
invisible oxygen gas. (credit: modification of work by Paul Flowers)
This figure shows a series of three photos labeled a, b, and c. Photo a shows the bottom of a test tube that is filled with an orange-
red substance. A slight amount of a silver substance is also visible. Photo b shows the substance in the test tube being heated over a
flame. Photo c shows a test tube that is not longer being heated. The orange-red substance is almost completely gone, and small,
silver droplets of a substance are left.
The properties of combined elements are different from those in the free, or uncombined, state. For example, white crystalline
sugar (sucrose) is a compound resulting from the chemical combination of the element carbon, which is a black solid in one of its
uncombined forms, and the two elements hydrogen and oxygen, which are colorless gases when uncombined. Free sodium, an
element that is a soft, shiny, metallic solid, and free chlorine, an element that is a yellow-green gas, combine to form sodium
chloride (table salt), a compound that is a white, crystalline solid.

Access for free at OpenStax 1.2.5 https://chem.libretexts.org/@go/page/39648


A mixture is composed of two or more types of matter that can be present in varying amounts and can be separated by physical
changes, such as evaporation (you will learn more about this later). A mixture with a composition that varies from point to point is
called a heterogeneous mixture. Italian dressing is an example of a heterogeneous mixture (Figure 1.2.1a). Its composition can
vary because we can make it from varying amounts of oil, vinegar, and herbs. It is not the same from point to point throughout the
mixture—one drop may be mostly vinegar, whereas a different drop may be mostly oil or herbs because the oil and vinegar
separate and the herbs settle. Other examples of heterogeneous mixtures are chocolate chip cookies (we can see the separate bits of
chocolate, nuts, and cookie dough) and granite (we can see the quartz, mica, feldspar, and more).
A homogeneous mixture, also called a solution, exhibits a uniform composition and appears visually the same throughout. An
example of a solution is a sports drink, consisting of water, sugar, coloring, flavoring, and electrolytes mixed together uniformly
(Figure 1.2.1b). Each drop of a sports drink tastes the same because each drop contains the same amounts of water, sugar, and other
components. Note that the composition of a sports drink can vary—it could be made with somewhat more or less sugar, flavoring,
or other components, and still be a sports drink. Other examples of homogeneous mixtures include air, maple syrup, gasoline, and a
solution of salt in water.

Figure 1.2.7 : (a) Oil and vinegar salad dressing is a heterogeneous mixture because its composition is not uniform throughout. (b)
A commercial sports drink is a homogeneous mixture because its composition is uniform throughout. (credit a “left”: modification
of work by John Mayer; credit a “right”: modification of work by Umberto Salvagnin; credit b “left: modification of work by Jeff
Bedford)
Diagram A shows a glass containing a red liquid with a layer of yellow oil floating on the surface of the red liquid. A zoom in box
is magnifying a portion of the red liquid that contains some of the yellow oil. The zoomed in image shows that oil is forming round
droplets within the red liquid. Diagram B shows a photo of Gatorade G 2. A zoom in box is magnifying a portion of the Gatorade,
which is uniformly red.
Although there are just over 100 elements, tens of millions of chemical compounds result from different combinations of these
elements. Each compound has a specific composition and possesses definite chemical and physical properties by which we can
distinguish it from all other compounds. And, of course, there are innumerable ways to combine elements and compounds to form
different mixtures. A summary of how to distinguish between the various major classifications of matter is shown in (Figure 1.2.8).

Figure 1.2.8 : Depending on its properties, a given substance can be classified as a homogeneous mixture, a heterogeneous mixture,
a compound, or an element.
This flow chart begins with matter at the top and the question: does the matter have constant properties and composition? If no,
then it is a mixture. This leads to the next question: is it uniform throughout? If no, it is heterogeneous. If yes, it is homogenous. If
the matter does have constant properties and composition, it is a pure substance. This leads to the next question: can it be simplified
chemically? If no, it is an element. If yes, then it is a compound.
Eleven elements make up about 99% of the earth’s crust and atmosphere (Table 1.2.1). Oxygen constitutes nearly one-half and
silicon about one-quarter of the total quantity of these elements. A majority of elements on earth are found in chemical
combinations with other elements; about one-quarter of the elements are also found in the free state.
Table 1.2.1 : Elemental Composition of Earth
Element Symbol Percent Mass Element Symbol Percent Mass

Access for free at OpenStax 1.2.6 https://chem.libretexts.org/@go/page/39648


Element Symbol Percent Mass Element Symbol Percent Mass

oxygen O 49.20 chlorine Cl 0.19

silicon Si 25.67 phosphorus P 0.11

aluminum Al 7.50 manganese Mn 0.09

iron Fe 4.71 carbon C 0.08

calcium Ca 3.39 sulfur S 0.06

sodium Na 2.63 barium Ba 0.04

potassium K 2.40 nitrogen N 0.03

magnesium Mg 1.93 fluorine F 0.03

hydrogen H 0.87 strontium Sr 0.02

titanium Ti 0.58 all others - 0.47

1.2.3: Decomposition of Water / Production of Hydrogen


Water consists of the elements hydrogen and oxygen combined in a 2 to 1 ratio. Water can be broken down into hydrogen and
oxygen gases by the addition of energy. One way to do this is with a battery or power supply, as shown in (Figure 1.2.9).

Figure 1.2.9 : The decomposition of water is shown at the macroscopic, microscopic, and symbolic levels. The battery provides an
electric current (microscopic) that decomposes water. At the macroscopic level, the liquid separates into the gases hydrogen (on the
left) and oxygen (on the right). Symbolically, this change is presented by showing how liquid H2O separates into H2 and O2 gases.
A rectangular battery is immersed in a beaker filled with liquid. Each of the battery terminals are covered by an overturned test
tube. The test tubes each contain a bubbling liquid. Zoom in areas indicate that the liquid in the beaker is water, 2 H subscript 2 O
liquid. The bubbles in the test tube over the negative terminal are hydrogen gas, 2 H subscript 2 gas. The bubbles in the test tube
over the positive terminal are oxygen gas, O subscript 2 gas.
The breakdown of water involves a rearrangement of the atoms in water molecules into different molecules, each composed of two
hydrogen atoms and two oxygen atoms, respectively. Two water molecules form one oxygen molecule and two hydrogen
molecules. The representation for what occurs, 2 H O(l) → 2 H (g) + O (g) , will be explored in more depth in later chapters.
2 2 2

The two gases produced have distinctly different properties. Oxygen is not flammable but is required for combustion of a fuel, and
hydrogen is highly flammable and a potent energy source. How might this knowledge be applied in our world? One application
involves research into more fuel-efficient transportation. Fuel-cell vehicles (FCV) run on hydrogen instead of gasoline (Figure
1.2.10). They are more efficient than vehicles with internal combustion engines, are nonpolluting, and reduce greenhouse gas

emissions, making us less dependent on fossil fuels. FCVs are not yet economically viable, however, and current hydrogen
production depends on natural gas. If we can develop a process to economically decompose water, or produce hydrogen in another
environmentally sound way, FCVs may be the way of the future.

Access for free at OpenStax 1.2.7 https://chem.libretexts.org/@go/page/39648


Figure 1.2.10 : A fuel cell generates electrical energy from hydrogen and oxygen via an electrochemical process and produces only
water as the waste product.
The fuel cell consists of a proton exchange membrane sandwiched between an anode and a cathode. Hydrogen gas enters the
battery near the anode. Oxygen gas enters the battery near the cathode. The entering hydrogen gas is broken up into single white
spheres that each have a positive charge. These are protons. The protons repel negatively-charged electrons within the anode. These
electrons travel through a circuit, providing electricity to anything attached to the battery. The protons continue through the proton
exchange membrane and through the cathode to reach the oxygen gas molecules at the opposite end of the battery. There, the
oxygen atoms split up into single red spheres. Each oxygen atom takes on two of the incoming protons to form a water molecule.

1.2.4: Chemistry of Cell Phones


Imagine how different your life would be without cell phones (Figure 1.2.11) and other smart devices. Cell phones are made from
numerous chemical substances, which are extracted, refined, purified, and assembled using an extensive and in-depth
understanding of chemical principles. About 30% of the elements that are found in nature are found within a typical smart phone.
The case/body/frame consists of a combination of sturdy, durable polymers comprised primarily of carbon, hydrogen, oxygen, and
nitrogen [acrylonitrile butadiene styrene (ABS) and polycarbonate thermoplastics], and light, strong, structural metals, such as
aluminum, magnesium, and iron. The display screen is made from a specially toughened glass (silica glass strengthened by the
addition of aluminum, sodium, and potassium) and coated with a material to make it conductive (such as indium tin oxide). The
circuit board uses a semiconductor material (usually silicon); commonly used metals like copper, tin, silver, and gold; and more
unfamiliar elements such as yttrium, praseodymium, and gadolinium. The battery relies upon lithium ions and a variety of other
materials, including iron, cobalt, copper, polyethylene oxide, and polyacrylonitrile.

Figure 1.2.11 : Almost one-third of naturally occurring elements are used to make a modern cell phone. (credit: modification of
work by John Taylor)
A cell phone is labeled to show what its components are made of. The case components are made of polymers such as A B S and or
metals such as aluminum, iron, and magnesium. The processor components are made of silicon, common metals such as copper, tin
and gold, and uncommon elements such as yttrium and gadolinium. The screen components are made of silicon oxide, also known
as glass. The glass is strengthened by the addition of aluminum, sodium, and potassium. The battery components contain lithium
combined with other metals such as cobalt, iron, and copper.

Access for free at OpenStax 1.2.8 https://chem.libretexts.org/@go/page/39648


Summary
Matter is anything that occupies space and has mass. The basic building block of matter is the atom, the smallest unit of an element
that can enter into combinations with atoms of the same or other elements. In many substances, atoms are combined into
molecules. On earth, matter commonly exists in three states: solids, of fixed shape and volume; liquids, of variable shape but fixed
volume; and gases, of variable shape and volume. Under high-temperature conditions, matter also can exist as a plasma. Most
matter is a mixture: It is composed of two or more types of matter that can be present in varying amounts and can be separated by
physical means. Heterogeneous mixtures vary in composition from point to point; homogeneous mixtures have the same
composition from point to point. Pure substances consist of only one type of matter. A pure substance can be an element, which
consists of only one type of atom and cannot be broken down by a chemical change, or a compound, which consists of two or more
types of atoms.

Glossary
atom
smallest particle of an element that can enter into a chemical combination

compound
pure substance that can be decomposed into two or more elements

element
substance that is composed of a single type of atom; a substance that cannot be decomposed by a chemical change

gas
state in which matter has neither definite volume nor shape

heterogeneous mixture
combination of substances with a composition that varies from point to point

homogeneous mixture
(also, solution) combination of substances with a composition that is uniform throughout

liquid
state of matter that has a definite volume but indefinite shape

law of conservation of matter


when matter converts from one type to another or changes form, there is no detectable change in the total amount of matter
present

mass
fundamental property indicating amount of matter

matter
anything that occupies space and has mass

mixture
matter that can be separated into its components by physical means

molecule
bonded collection of two or more atoms of the same or different elements

plasma
gaseous state of matter containing a large number of electrically charged atoms and/or molecules

pure substance
homogeneous substance that has a constant composition

Access for free at OpenStax 1.2.9 https://chem.libretexts.org/@go/page/39648


solid
state of matter that is rigid, has a definite shape, and has a fairly constant volume

weight
force that gravity exerts on an object

This page titled 1.2: Phases and Classification of Matter is shared under a CC BY 4.0 license and was authored, remixed, and/or curated by
OpenStax via source content that was edited to the style and standards of the LibreTexts platform; a detailed edit history is available upon request.

Access for free at OpenStax 1.2.10 https://chem.libretexts.org/@go/page/39648


1.3: Physical and Chemical Properties
 Learning Objectives
Describe the difference between physical and chemical properties or changed.
Identify a property or transformation as either physical or chemical using symbolic, particulate, or macroscopic
representations.
Identify the properties of matter as extensive or intensive.
Recognize and describe the parts of the NFPA hazard diamond.

The characteristics that enable us to distinguish one substance from another are called properties. A physical property is a
characteristic of matter that is not associated with a change in its chemical composition. Familiar examples of physical properties
include density, color, hardness, melting and boiling points, and electrical conductivity. We can observe some physical properties,
such as density and color, without changing the physical state of the matter observed. Other physical properties, such as the melting
temperature of iron or the freezing temperature of water, can only be observed as matter undergoes a physical change. A physical
change is a change in the state or properties of matter without any accompanying change in its chemical composition (the identities
of the substances contained in the matter). We observe a physical change when wax melts, when sugar dissolves in coffee, and
when steam condenses into liquid water (Figure 1.3.1). Other examples of physical changes include magnetizing and
demagnetizing metals (as is done with common antitheft security tags) and grinding solids into powders (which can sometimes
yield noticeable changes in color). In each of these examples, there is a change in the physical state, form, or properties of the
substance, but no change in its chemical composition.

Figure 1.3.1 : (a) Wax undergoes a physical change when solid wax is heated and forms liquid wax. (b) Steam condensing inside a
cooking pot is a physical change, as water vapor is changed into liquid water. (credit a: modification of work by
“95jb14”/Wikimedia Commons; credit b: modification of work by “mjneuby”/Flickr).

The change of one type of matter into another type (or the inability to change) is a chemical property. Examples of chemical
properties include flammability, toxicity, acidity, reactivity (many types), and heat of combustion. Iron, for example, combines with
oxygen in the presence of water to form rust; chromium does not oxidize (Figure 1.3.2). Nitroglycerin is very dangerous because it
explodes easily; neon poses almost no hazard because it is very unreactive.

Figure 1.3.2 : (a) One of the chemical properties of iron is that it rusts; (b) one of the chemical properties of chromium is that it
does not. (credit a: modification of work by Tony Hisgett; credit b: modification of work by “Atoma”/Wikimedia Commons)
Comparison of two metal equipments. The first image shows a rusted brown appearance and the following image shows a shiny
metallic appearance.

Access for free at OpenStax 1.3.1 https://chem.libretexts.org/@go/page/39649


To identify a chemical property, we look for a chemical change. A chemical change always produces one or more types of matter
that differ from the matter present before the change. The formation of rust is a chemical change because rust is a different kind of
matter than the iron, oxygen, and water present before the rust formed. The explosion of nitroglycerin is a chemical change because
the gases produced are very different kinds of matter from the original substance. Other examples of chemical changes include
reactions that are performed in a lab (such as copper reacting with nitric acid), all forms of combustion (burning), and food being
cooked, digested, or rotting (Figure 1.3.3).

Figure 1.3.3 : (a) Copper and nitric acid undergo a chemical change to form copper nitrate and brown, gaseous nitrogen dioxide. (b)
During the combustion of a match, cellulose in the match and oxygen from the air undergo a chemical change to form carbon
dioxide and water vapor. (c) Cooking red meat causes a number of chemical changes, including the oxidation of iron in myoglobin
that results in the familiar red-to-brown color change. (d) A banana turning brown is a chemical change as new, darker (and less
tasty) substances form. (credit b: modification of work by Jeff Turner; credit c: modification of work by Gloria Cabada-Leman;
credit d: modification of work by Roberto Verzo)
Four pictures. A. a flask with copper wires placed in a blue solution and also the presence of a gas. B. A lighted match. C. meat
cubes being cooked on a pan. D. Bananas forming black spots on its peel.
Properties of matter fall into one of two categories. If the property depends on the amount of matter present, it is an extensive
property. The mass and volume of a substance are examples of extensive properties; for instance, a gallon of milk has a larger mass
and volume than a cup of milk. The value of an extensive property is directly proportional to the amount of matter in question. If
the property of a sample of matter does not depend on the amount of matter present, it is an intensive property. Temperature is an
example of an intensive property. If the gallon and cup of milk are each at 20 °C (room temperature), when they are combined, the
temperature remains at 20 °C. As another example, consider the distinct but related properties of heat and temperature. A drop of
hot cooking oil spattered on your arm causes brief, minor discomfort, whereas a pot of hot oil yields severe burns. Both the drop
and the pot of oil are at the same temperature (an intensive property), but the pot clearly contains much more heat (extensive
property).

1.3.1: Hazard Diamond


You may have seen the symbol shown in Figure 1.3.4 on containers of chemicals in a laboratory or workplace. Sometimes called a
“fire diamond” or “hazard diamond,” this chemical hazard diamond provides valuable information that briefly summarizes the
various dangers of which to be aware when working with a particular substance.

Access for free at OpenStax 1.3.2 https://chem.libretexts.org/@go/page/39649


Figure 1.3.4 : The National Fire Protection Agency (NFPA) hazard diamond summarizes the major hazards of a chemical
substance.
The National Fire Protection Agency (NFPA) 704 Hazard Identification System was developed by NFPA to provide safety
information about certain substances. The system details flammability, reactivity, health, and other hazards. Within the overall
diamond symbol, the top (red) diamond specifies the level of fire hazard (temperature range for flash point). The blue (left)
diamond indicates the level of health hazard. The yellow (right) diamond describes reactivity hazards, such as how readily the
substance will undergo detonation or a violent chemical change. The white (bottom) diamond points out special hazards, such as if
it is an oxidizer (which allows the substance to burn in the absence of air/oxygen), undergoes an unusual or dangerous reaction with
water, is corrosive, acidic, alkaline, a biological hazard, radioactive, and so on. Each hazard is rated on a scale from 0 to 4, with 0
being no hazard and 4 being extremely hazardous.
While many elements differ dramatically in their chemical and physical properties, some elements have similar properties. We can
identify sets of elements that exhibit common behaviors. For example, many elements conduct heat and electricity well, whereas
others are poor conductors. These properties can be used to sort the elements into three classes: metals (elements that conduct
well), nonmetals (elements that conduct poorly), and metalloids (elements that have properties of both metals and nonmetals).
The periodic table is a table of elements that places elements with similar properties close together (Figure 1.3.5 ). You will learn
more about the periodic table as you continue your study of chemistry.

Access for free at OpenStax 1.3.3 https://chem.libretexts.org/@go/page/39649


Figure 1.3.5 : The periodic table shows how elements may be grouped according to certain similar properties. Note the background
color denotes whether an element is a metal, metalloid, or nonmetal, whereas the element symbol color indicates whether it is a
solid, liquid, or gas.

Summary
All substances have distinct physical and chemical properties, and may undergo physical or chemical changes. Physical properties,
such as hardness and boiling point, and physical changes, such as melting or freezing, do not involve a change in the composition
of matter. Chemical properties, such flammability and acidity, and chemical changes, such as rusting, involve production of matter
that differs from that present beforehand.
Measurable properties fall into one of two categories. Extensive properties depend on the amount of matter present, for example,
the mass of gold. Intensive properties do not depend on the amount of matter present, for example, the density of gold. Heat is an
example of an extensive property, and temperature is an example of an intensive property.

Glossary
chemical change
change producing a different kind of matter from the original kind of matter

chemical property
behavior that is related to the change of one kind of matter into another kind of matter

extensive property
property of a substance that depends on the amount of the substance

intensive property
property of a substance that is independent of the amount of the substance

physical change
change in the state or properties of matter that does not involve a change in its chemical composition

physical property

Access for free at OpenStax 1.3.4 https://chem.libretexts.org/@go/page/39649


characteristic of matter that is not associated with any change in its chemical composition

This page titled 1.3: Physical and Chemical Properties is shared under a CC BY 4.0 license and was authored, remixed, and/or curated by
OpenStax via source content that was edited to the style and standards of the LibreTexts platform; a detailed edit history is available upon request.

Access for free at OpenStax 1.3.5 https://chem.libretexts.org/@go/page/39649


1.4: Measurements
 Learning Objectives
Explain the process of measurement and describe the three basic parts of a quantity.
Describe the properties and units of length, mass, volume, density, temperature, and time.
Recognize the common unit prefixes and use them to describe the magnitude of a measurement.
Describe and calculate the density of a substance.
Perform basic unit calculations and conversions in the metric and other unit systems.

Measurements provide the macroscopic information that is the basis of most of the hypotheses, theories, and laws that describe the
behavior of matter and energy in both the macroscopic and microscopic domains of chemistry. Every measurement provides three
kinds of information: the size or magnitude of the measurement (a number); a standard of comparison for the measurement (a unit);
and an indication of the uncertainty of the measurement. While the number and unit are explicitly represented when a quantity is
written, the uncertainty is an aspect of the measurement result that is more implicitly represented and will be discussed later.
The number in the measurement can be represented in different ways, including decimal form and scientific notation. For example,
the maximum takeoff weight of a Boeing 777-200ER airliner is 298,000 kilograms, which can also be written as 2.98 × 105 kg.
The mass of the average mosquito is about 0.0000025 kilograms, which can be written as 2.5 × 10−6 kg.
Units, such as liters, pounds, and centimeters, are standards of comparison for measurements. When we buy a 2-liter bottle of a soft
drink, we expect that the volume of the drink was measured, so it is two times larger than the volume that everyone agrees to be 1
liter. The meat used to prepare a 0.25-pound hamburger is measured so it weighs one-fourth as much as 1 pound. Without units, a
number can be meaningless, confusing, or possibly life threatening. Suppose a doctor prescribes phenobarbital to control a patient’s
seizures and states a dosage of “100” without specifying units. Not only will this be confusing to the medical professional giving
the dose, but the consequences can be dire: 100 mg given three times per day can be effective as an anticonvulsant, but a single
dose of 100 g is more than 10 times the lethal amount.
We usually report the results of scientific measurements in SI units, an updated version of the metric system, using the units listed
in Table 1.4.1. Other units can be derived from these base units. The standards for these units are fixed by international agreement,
and they are called the International System of Units or SI Units (from the French, Le Système International d’Unités). SI units
have been used by the United States National Institute of Standards and Technology (NIST) since 1964.
Table 1.4.1 : Base Units of the SI System
Property Measured Name of Unit Symbol of Unit

length meter m

mass kilogram kg

time second s

temperature kelvin K

electric current ampere A

amount of substance mole mol

luminous intensity candela cd

Sometimes we use units that are fractions or multiples of a base unit. Ice cream is sold in quarts (a familiar, non-SI base unit), pints
(0.5 quart), or gallons (4 quarts). We also use fractions or multiples of units in the SI system, but these fractions or multiples are
always powers of 10. Fractional or multiple SI units are named using a prefix and the name of the base unit. For example, a length
of 1000 meters is also called a kilometer because the prefix kilo means “one thousand,” which in scientific notation is 103 (1
kilometer = 1000 m = 103 m). The prefixes used and the powers to which 10 are raised are listed in Table 1.4.2.
NG, equals 4 times ten to the negative 9, or 0.000000004 g. The prefix micro has the greek letter mu as its symbol and a factor of
10 to the negative sixth power. Therefore, 1 microliter, or mu L, is equal to one times ten to the negative 6 or 0.000001 L. The
prefix milli has a lowercase M as its symbol and a factor of 10 to the negative third power. Therefore, 2 millimoles, or M mol, are
equal to two times ten to the negative 3 or 0.002 mol. The prefix centi has a lowercase C as its symbol and a factor of 10 to the

Access for free at OpenStax 1.4.1 https://chem.libretexts.org/@go/page/39650


negative second power. Therefore, 7 centimeters, or C M, are equal to seven times ten to the negative 2 meters or 0.07 M O L. The
prefix deci has a lowercase D as its symbol and a factor of 10 to the negative first power. Therefore, 1 deciliter, or lowercase D
uppercase L, are equal to one times ten to the negative 1 meters or 0.1 L. The prefix kilo has a lowercase K as its symbol and a
factor of 10 to the third power. Therefore, 1 kilometer, or K M, is equal to one times ten to the third meters or 1000 M. The prefix
mega has an uppercase M as its symbol and a factor of 10 to the sixth power. Therefore, 3 megahertz, or M H Z, are equal to three
times 10 to the sixth hertz, or 3000000 H Z. The prefix giga has an uppercase G as its symbol and a factor of 10 to the ninth power.
Therefore, 8 gigayears, or G Y R, are equal to eight times 10 to the ninth years, or 800000000 G Y R. The prefix tera has an
uppercase T as its symbol and a factor of 10 to the twelfth power. Therefore, 5 terawatts, or T W, are equal to five times 10 to the
twelfth watts, or 5000000000000 W." data-quail-id="64" data-mt-width="1076">
Table 1.4.2 : Common Unit Prefixes
Prefix Symbol Factor Example

1 femtosecond (fs) = 1 × 10−15 s


femto f 10−15
(0.000000000000001 s)
1 picometer (pm) = 1 × 10−12 m
pico p 10−12
(0.000000000001 m)
4 nanograms (ng) = 4 × 10−9 g
nano n 10−9
(0.000000004 g)
1 microliter (μL) = 1 × 10−6 L
micro µ 10−6
(0.000001 L)
2 millimoles (mmol) = 2 × 10−3
milli m 10−3
mol (0.002 mol)
7 centimeters (cm) = 7 × 10−2 m
centi c 10−2
(0.07 m)
1 deciliter (dL) = 1 × 10−1 L (0.1
deci d 10−1
L)
1 kilometer (km) = 1 × 103 m
kilo k 103
(1000 m)
3 megahertz (MHz) = 3 × 106 Hz
mega M 106
(3,000,000 Hz)
8 gigayears (Gyr) = 8 × 109 yr
giga G 109
(8,000,000,000 Gyr)
5 terawatts (TW) = 5 × 1012 W
tera T 1012
(5,000,000,000,000 W)

1.4.1: SI Base Units


The initial units of the metric system, which eventually evolved into the SI system, were established in France during the French
Revolution. The original standards for the meter and the kilogram were adopted there in 1799 and eventually by other countries.
This section introduces four of the SI base units commonly used in chemistry. Other SI units will be introduced in subsequent
chapters.

1.4.1.1: Length
The standard unit of length in both the SI and original metric systems is the meter (m). A meter was originally specified as
1/10,000,000 of the distance from the North Pole to the equator. It is now defined as the distance light in a vacuum travels in
1/299,792,458 of a second. A meter is about 3 inches longer than a yard (Figure 1.4.1); one meter is about 39.37 inches or 1.094
yards. Longer distances are often reported in kilometers (1 km = 1000 m = 103 m), whereas shorter distances can be reported in
centimeters (1 cm = 0.01 m = 10−2 m) or millimeters (1 mm = 0.001 m = 10−3 m).

Access for free at OpenStax 1.4.2 https://chem.libretexts.org/@go/page/39650


Figure 1.4.1 : The relative lengths of 1 m, 1 yd, 1 cm, and 1 in. are shown (not actual size), as well as comparisons of 2.54 cm and 1
in., and of 1 m and 1.094 yd.
A ruler is shown with various lengths of black line shown above it to compare the relative lengths of 1 inch, meter, centimeter, and
yard.

1.4.1.2: Mass
The standard unit of mass in the SI system is the kilogram (kg). A kilogram was originally defined as the mass of a liter of water (a
cube of water with an edge length of exactly 0.1 meter). It is now defined by a certain cylinder of platinum-iridium alloy, which is
kept in France (Figure 1.4.2). Any object with the same mass as this cylinder is said to have a mass of 1 kilogram. One kilogram is
about 2.2 pounds. The gram (g) is exactly equal to 1/1000 of the mass of the kilogram (10−3 kg).

Figure 1.4.2 : This replica prototype kilogram is housed at the National Institute of Standards and Technology (NIST) in Maryland.
(credit: National Institutes of Standards and Technology).

1.4.1.3: Temperature
Temperature is an intensive property. The SI unit of temperature is the kelvin (K). The IUPAC convention is to use kelvin (all
lowercase) for the word, K (uppercase) for the unit symbol, and neither the word “degree” nor the degree symbol (°). The degree
Celsius (°C) is also allowed in the SI system, with both the word “degree” and the degree symbol used for Celsius measurements.
Celsius degrees are the same magnitude as those of kelvin, but the two scales place their zeros in different places. Water freezes at
273.15 K (0 °C) and boils at 373.15 K (100 °C) by definition, and normal human body temperature is approximately 310 K (37
°C). The conversion between these two units and the Fahrenheit scale will be discussed later in this chapter.

1.4.1.4: Time
The SI base unit of time is the second (s). Small and large time intervals can be expressed with the appropriate prefixes; for
example, 3 microseconds = 0.000003 s = 3 × 10−6 and 5 megaseconds = 5,000,000 s = 5 × 106 s. Alternatively, hours, days, and
years can be used.

1.4.2: Derived SI Units


We can derive many units from the seven SI base units. For example, we can use the base unit of length to define a unit of volume,
and the base units of mass and length to define a unit of density.

Access for free at OpenStax 1.4.3 https://chem.libretexts.org/@go/page/39650


1.4.2.1: Volume
Volume is the measure of the amount of space occupied by an object. The standard SI unit of volume is defined by the base unit of
length (Figure 1.4.3). The standard volume is a cubic meter (m3), a cube with an edge length of exactly one meter. To dispense a
cubic meter of water, we could build a cubic box with edge lengths of exactly one meter. This box would hold a cubic meter of
water or any other substance.

Figure 1.4.3 : (a) The relative volumes are shown for cubes of 1 m3, 1 dm3 (1 L), and 1 cm3 (1 mL) (not to scale). (b) The diameter
of a dime is compared relative to the edge length of a 1-cm3 (1-mL) cube.
A more commonly used unit of volume is derived from the decimeter (0.1 m, or 10 cm). A cube with edge lengths of exactly one
decimeter contains a volume of one cubic decimeter (dm3). A liter (L) is the more common name for the cubic decimeter. One liter
is about 1.06 quarts. A cubic centimeter (cm3) is the volume of a cube with an edge length of exactly one centimeter. The
abbreviation cc (for cubic centimeter) is often used by health professionals. A cubic centimeter is also called a milliliter (mL) and is
1/1000 of a liter.

1.4.2.2: Density
We use the mass and volume of a substance to determine its density. Thus, the units of density are defined by the base units of mass
and length.
The density of a substance is the ratio of the mass of a sample of the substance to its volume. The SI unit for density is the kilogram
per cubic meter (kg/m3). For many situations, however, this as an inconvenient unit, and we often use grams per cubic centimeter
(g/cm3) for the densities of solids and liquids, and grams per liter (g/L) for gases. Although there are exceptions, most liquids and
solids have densities that range from about 0.7 g/cm3 (the density of gasoline) to 19 g/cm3 (the density of gold). The density of air
is about 1.2 g/L. Table 1.4.3 shows the densities of some common substances.
Table 1.4.3 : Densities of Common Substances
Solids Liquids Gases (at 25 °C and 1 atm)

ice (at 0 °C) 0.92 g/cm3 water 1.0 g/cm3 dry air 1.20 g/L

oak (wood) 0.60–0.90 g/cm3 ethanol 0.79 g/cm3 oxygen 1.31 g/L

iron 7.9 g/cm3 acetone 0.79 g/cm3 nitrogen 1.14 g/L

copper 9.0 g/cm3 glycerin 1.26 g/cm3 carbon dioxide 1.80 g/L

lead 11.3 g/cm3 olive oil 0.92 g/cm3 helium 0.16 g/L

silver 10.5 g/cm3 gasoline 0.70–0.77 g/cm3 neon 0.83 g/L

gold 19.3 g/cm3 mercury 13.6 g/cm3 radon 9.1 g/L

While there are many ways to determine the density of an object, perhaps the most straightforward method involves separately
finding the mass and volume of the object, and then dividing the mass of the sample by its volume. In the following example, the
mass is found directly by weighing, but the volume is found indirectly through length measurements.
mass
density =
volume

Access for free at OpenStax 1.4.4 https://chem.libretexts.org/@go/page/39650


 Example 1.4.1

Calculation of Density Gold—in bricks, bars, and coins—has been a form of currency for centuries. In order to swindle people
into paying for a brick of gold without actually investing in a brick of gold, people have considered filling the centers of
hollow gold bricks with lead to fool buyers into thinking that the entire brick is gold. It does not work: Lead is a dense
substance, but its density is not as great as that of gold, 19.3 g/cm3. What is the density of lead if a cube of lead has an edge
length of 2.00 cm and a mass of 90.7 g?

Solution
The density of a substance can be calculated by dividing its mass by its volume. The volume of a cube is calculated by cubing
the edge length.
3
volume of lead cube = 2.00 cm × 2.00 cm × 2.00 cm = 8.00 cm

mass 90.7 g 11.3 g 3


density = = = = 11.3 g/cm
3
volume 8.00 cm 1.00 cm3

(We will discuss the reason for rounding to the first decimal place in the next section.)

 Exercise 1.4.1
a. To three decimal places, what is the volume of a cube (cm3) with an edge length of 0.843 cm?
b. If the cube in part (a) is copper and has a mass of 5.34 g, what is the density of copper to two decimal places?

Answer a
0.599 cm3;
Answer b
8.91 g/cm3

 Example 1.4.2: Using Displacement of Water to Determine Density

This PhET simulation illustrates another way to determine density, using displacement of water. Determine the density of the
red and yellow blocks.

Solution
When you open the density simulation and select Same Mass, you can choose from several 5.00-kg colored blocks that you can
drop into a tank containing 100.00 L water. The yellow block floats (it is less dense than water), and the water level rises to
105.00 L. While floating, the yellow block displaces 5.00 L water, an amount equal to the weight of the block. The red block
sinks (it is more dense than water, which has density = 1.00 kg/L), and the water level rises to 101.25 L.
The red block therefore displaces 1.25 L water, an amount equal to the volume of the block. The density of the red block is:
mass 5.00 kg
density = = = 4.00 kg/L
volume 1.25 L

Note that since the yellow block is not completely submerged, you cannot determine its density from this information. But if
you hold the yellow block on the bottom of the tank, the water level rises to 110.00 L, which means that it now displaces 10.00
L water, and its density can be found:
mass 5.00 kg
density = = = 0.500 kg/L
volume 10.00 L

Access for free at OpenStax 1.4.5 https://chem.libretexts.org/@go/page/39650


 Exercise 1.4.1

Remove all of the blocks from the water and add the green block to the tank of water, placing it approximately in the middle of
the tank. Determine the density of the green block.

Answer
2.00 kg/L

Summary
Measurements provide quantitative information that is critical in studying and practicing chemistry. Each measurement has an
amount, a unit for comparison, and an uncertainty. Measurements can be represented in either decimal or scientific notation.
Scientists primarily use the SI (International System) or metric systems. We use base SI units such as meters, seconds, and
kilograms, as well as derived units, such as liters (for volume) and g/cm3 (for density). In many cases, we find it convenient to use
unit prefixes that yield fractional and multiple units, such as microseconds (10−6 seconds) and megahertz (106 hertz), respectively.

1.4.3: Key Equations


mass
density =
volume

Glossary
Celsius (°C)
unit of temperature; water freezes at 0 °C and boils at 100 °C on this scale

cubic centimeter (cm3 or cc)


volume of a cube with an edge length of exactly 1 cm

cubic meter (m3)


SI unit of volume

density
ratio of mass to volume for a substance or object

kelvin (K)
SI unit of temperature; 273.15 K = 0 ºC

kilogram (kg)
standard SI unit of mass; 1 kg = approximately 2.2 pounds

length
measure of one dimension of an object

liter (L)
(also, cubic decimeter) unit of volume; 1 L = 1,000 cm3

meter (m)
standard metric and SI unit of length; 1 m = approximately 1.094 yards

milliliter (mL)
1/1,000 of a liter; equal to 1 cm3

second (s)
SI unit of time

SI units (International System of Units)

Access for free at OpenStax 1.4.6 https://chem.libretexts.org/@go/page/39650


standards fixed by international agreement in the International System of Units (Le Système International d’Unités)

unit
standard of comparison for measurements

volume
amount of space occupied by an object

This page titled 1.4: Measurements is shared under a CC BY 4.0 license and was authored, remixed, and/or curated by OpenStax via source
content that was edited to the style and standards of the LibreTexts platform; a detailed edit history is available upon request.

Access for free at OpenStax 1.4.7 https://chem.libretexts.org/@go/page/39650


Access for free at OpenStax 1.4.8 https://chem.libretexts.org/@go/page/39650
Access for free at OpenStax 1.4.9 https://chem.libretexts.org/@go/page/39650
1.5: Measurement Uncertainty, Accuracy, and Precision
 Learning Objectives
Compare and contrast exact and uncertain numbers.
Correctly represent uncertainty in quantities using significant figures.
Identify the number of significant figures in value.
Solve problems that involve various calculations and report the results with the appropriate number of significant figures.
Apply proper rounding rules to computed quantities
Define accuracy and precision, and use accuracy and precision to describe data sets.

Counting is the only type of measurement that is free from uncertainty, provided the number of objects being counted does not
change while the counting process is underway. The result of such a counting measurement is an example of an exact number. If
we count eggs in a carton, we know exactly how many eggs the carton contains. The numbers of defined quantities are also exact.
By definition, 1 foot is exactly 12 inches, 1 inch is exactly 2.54 centimeters, and 1 gram is exactly 0.001 kilogram. Quantities
derived from measurements other than counting, however, are uncertain to varying extents due to practical limitations of the
measurement process used.

1.5.1: Significant Figures in Measurement


The numbers of measured quantities, unlike defined or directly counted quantities, are not exact. To measure the volume of liquid
in a graduated cylinder, you should make a reading at the bottom of the meniscus, the lowest point on the curved surface of the
liquid.

Figure 1.5.1 : To measure the volume of liquid in this graduated cylinder, you must mentally subdivide the distance between the 21
and 22 mL marks into tenths of a milliliter, and then make a reading (estimate) at the bottom of the meniscus.
A 25 millilter graduated cylinder filled with liquid. The markings on the cylinders is zoomed in to show the bottom of the meniscus
between 21 and 22 milliliters.
Refer to the illustration in Figure 1.5.1. The bottom of the meniscus in this case clearly lies between the 21 and 22 markings,
meaning the liquid volume is certainly greater than 21 mL but less than 22 mL. The meniscus appears to be a bit closer to the 22-
mL mark than to the 21-mL mark, and so a reasonable estimate of the liquid’s volume would be 21.6 mL. In the number 21.6, then,
the digits 2 and 1 are certain, but the 6 is an estimate. Some people might estimate the meniscus position to be equally distant from
each of the markings and estimate the tenth-place digit as 5, while others may think it to be even closer to the 22-mL mark and
estimate this digit to be 7. Note that it would be pointless to attempt to estimate a digit for the hundredths place, given that the
tenths-place digit is uncertain. In general, numerical scales such as the one on this graduated cylinder will permit measurements to
one-tenth of the smallest scale division. The scale in this case has 1-mL divisions, and so volumes may be measured to the nearest
0.1 mL.

Access for free at OpenStax 1.5.1 https://chem.libretexts.org/@go/page/39651


This concept holds true for all measurements, even if you do not actively make an estimate. If you place a quarter on a standard
electronic balance, you may obtain a reading of 6.72 g. The digits 6 and 7 are certain, and the 2 indicates that the mass of the
quarter is likely between 6.71 and 6.73 grams. The quarter weighs about 6.72 grams, with a nominal uncertainty in the
measurement of ± 0.01 gram. If we weigh the quarter on a more sensitive balance, we may find that its mass is 6.723 g. This means
its mass lies between 6.722 and 6.724 grams, an uncertainty of 0.001 gram. Every measurement has some uncertainty, which
depends on the device used (and the user’s ability). All of the digits in a measurement, including the uncertain last digit, are called
significant figures or significant digits. Note that zero may be a measured value; for example, if you stand on a scale that shows
weight to the nearest pound and it shows “120,” then the 1 (hundreds), 2 (tens) and 0 (ones) are all significant (measured) values.
Whenever you make a measurement properly, all the digits in the result are significant. But what if you were analyzing a reported
value and trying to determine what is significant and what is not? Well, for starters, all nonzero digits are significant, and it is only
zeros that require some thought. We will use the terms “leading,” “trailing,” and “captive” for the zeros and will consider how to
deal with them.

The left diagram uses the example of 3090. The zero in the hundreds place is labeled “captive” and the zero in the ones place is
labeled trailing. The right diagram uses the example 0.008020. The three zeros in the ones, tenths, and hundredths places are
labeled “leading.” The zero in the ten-thousandths place is labeled “captive” and the zero in the millionths place is labeled
“trailing.”
Starting with the first nonzero digit on the left, count this digit and all remaining digits to the right. This is the number of
significant figures in the measurement unless the last digit is a trailing zero lying to the left of the decimal point.

The left diagram uses the example of 1267 meters. The number 1 is the first nonzero figure on the left. 1267 has 4 significant
figures in total. The right diagram uses the example of 55.0 grams. The number 5 in the tens place is the first nonzero figure on the
left. 55.0 has 3 significant figures. Note that the 0 is to the right of the decimal point and therefore is a significant figure.
Captive zeros result from measurement and are therefore always significant. Leading zeros, however, are never significant—they
merely tell us where the decimal point is located.

The left diagram uses the example of 70.607 milliliters. The number 7 is the first nonzero figure on the left. 70.607 has 5
significant figures in total, as all figures are measured including the 2 zeros. The right diagram uses the example of 0.00832407 M
L. The number 8 is the first nonzero figure on the left. 0.00832407 has 6 significant figures.
The leading zeros in this example are not significant. We could use exponential notation (as described in Appendix B) and express
the number as 8.32407 × 10−3; then the number 8.32407 contains all of the significant figures, and 10−3 locates the decimal point.
The number of significant figures is uncertain in a number that ends with a zero to the left of the decimal point location. The zeros
in the measurement 1,300 grams could be significant or they could simply indicate where the decimal point is located. The
ambiguity can be resolved with the use of exponential notation: 1.3 × 103 (two significant figures), 1.30 × 103 (three significant
figures, if the tens place was measured), or 1.300 × 103 (four significant figures, if the ones place was also measured). In cases
where only the decimal-formatted number is available, it is prudent to assume that all trailing zeros are not significant.

Access for free at OpenStax 1.5.2 https://chem.libretexts.org/@go/page/39651


This figure uses the example of 1300 grams. The one and the 3 are significant figures as they are clearly the result of measurement.
The 2 zeros could be significant if they were measured or they could be placeholders.
When determining significant figures, be sure to pay attention to reported values and think about the measurement and significant
figures in terms of what is reasonable or likely—that is, whether the value makes sense. For example, the official January 2014
census reported the resident population of the US as 317,297,725. Do you think the US population was correctly determined to the
reported nine significant figures, that is, to the exact number of people? People are constantly being born, dying, or moving into or
out of the country, and assumptions are made to account for the large number of people who are not actually counted. Because of
these uncertainties, it might be more reasonable to expect that we know the population to within perhaps a million or so, in which
case the population should be reported as 317 million, or 3.17 × 10 people.
8

1.5.2: Significant Figures in Calculations


A second important principle of uncertainty is that results calculated from a measurement are at least as uncertain as the
measurement itself. We must take the uncertainty in our measurements into account to avoid misrepresenting the uncertainty in
calculated results. One way to do this is to report the result of a calculation with the correct number of significant figures, which is
determined by the following three rules for rounding numbers:
1. When we add or subtract numbers, we should round the result to the same number of decimal places as the number with the
least number of decimal places (the least precise value in terms of addition and subtraction).
2. When we multiply or divide numbers, we should round the result to the same number of digits as the number with the least
number of significant figures (the least precise value in terms of multiplication and division).
3. If the digit to be dropped (the one immediately to the right of the digit to be retained) is less than 5, we “round down” and leave
the retained digit unchanged; if it is more than 5, we “round up” and increase the retained digit by 1; if the dropped digit is 5,
we round up or down, whichever yields an even value for the retained digit. (The last part of this rule may strike you as a bit
odd, but it’s based on reliable statistics and is aimed at avoiding any bias when dropping the digit “5,” since it is equally close to
both possible values of the retained digit.)
The following examples illustrate the application of this rule in rounding a few different numbers to three significant figures:
0.028675 rounds “up” to 0.0287 (the dropped digit, 7, is greater than 5)
18.3384 rounds “down” to 18.3 (the dropped digit, 3, is less than 5)
6.8752 rounds “up” to 6.88 (the dropped digit is 5, and the retained digit is even)
92.85 rounds “down” to 92.8 (the dropped digit is 5, and the retained digit is even)
Let’s work through these rules with a few examples.

 Example 1.5.1: Rounding Numbers

Round the following to the indicated number of significant figures:


a. 31.57 (to two significant figures)
b. 8.1649 (to three significant figures)
c. 0.051065 (to four significant figures)
d. 0.90275 (to four significant figures)

Solution
a. 31.57 rounds “up” to 32 (the dropped digit is 5, and the retained digit is even)
b. 8.1649 rounds “down” to 8.16 (the dropped digit, 4, is less than 5)
c. 0.051065 rounds “down” to 0.05106 (the dropped digit is 5, and the retained digit is even)
d. 0.90275 rounds “up” to 0.9028 (the dropped digit is 5, and the retained digit is even)

Access for free at OpenStax 1.5.3 https://chem.libretexts.org/@go/page/39651


 Exercise 1.5.1
Round the following to the indicated number of significant figures:
a. 0.424 (to two significant figures)
b. 0.0038661 (to three significant figures)
c. 421.25 (to four significant figures)
d. 28,683.5 (to five significant figures)

Answer a
0.42
Answer b
0.00387
Answer c
421.2
Answer d
28,684

 Example 1.5.2: Addition and Subtraction with Significant Figures Rule:

When we add or subtract numbers, we should round the result to the same number of decimal places as the number with the
least number of decimal places (i.e., the least precise value in terms of addition and subtraction).
a. Add 1.0023 g and 4.383 g.
b. Subtract 421.23 g from 486 g.

Solution
(a)
1.0023 g

+ 4.383 g
––––––––
5.3853 g

Answer is 5.385 g (round to the thousandths place; three decimal places)


(b)
486 g

− 421.23 g
––––––––
64.77 g

Answer is 65 g (round to the ones place; no decimal places)

Access for free at OpenStax 1.5.4 https://chem.libretexts.org/@go/page/39651


 Exercise 1.5.2
a. Add 2.334 mL and 0.31 mL.
b. Subtract 55.8752 m from 56.533 m.

Answer a
2.64 mL
Answer b
0.658 m

 Example 1.5.3: Multiplication and Division with Significant Figures

Rule: When we multiply or divide numbers, we should round the result to the same number of digits as the number with the
least number of significant figures (the least precise value in terms of multiplication and division).
a. Multiply 0.6238 cm by 6.6 cm.
b. Divide 421.23 g by 486 mL.

Solution
(a)
2 2
0.6238 cm × 6.6 cm = 4.11708 cm → result is 4.1 cm (round to two significant figures)

four significant figures × two significant figures → two significant figures answer

(b)
421.23 g
= 0.86728... g/mL → result is 0.867 g/mL (round to three significant figures)
486 mL

f ive signif icant f igures


→ three signif icant f igures answer
three signif icant f igures

 Exercise 1.5.3
a. Multiply 2.334 cm and 0.320 cm.
b. Divide 55.8752 m by 56.53 s.

Answer a
0.747 cm2
Answer b
0.9884 m/s

In the midst of all these technicalities, it is important to keep in mind the reason why we use significant figures and rounding rules
—to correctly represent the certainty of the values we report and to ensure that a calculated result is not represented as being more
certain than the least certain value used in the calculation.

 Example 1.5.4: Calculation with Significant Figures

One common bathtub is 13.44 dm long, 5.920 dm wide, and 2.54 dm deep. Assume that the tub is rectangular and calculate its
approximate volume in liters.

Access for free at OpenStax 1.5.5 https://chem.libretexts.org/@go/page/39651


Solution
V = l×w ×d

= 13.44 dm × 5.920 dm × 2.54 dm


3
= 202.09459...dm (value from calculator)

3
= 202 dm , or 202 L (answer rounded to three significant figures)

 Exercise 1.5.4: Determination of Density Using Water Displacement

What is the density of a liquid with a mass of 31.1415 g and a volume of 30.13 cm3?

Answer
1.034 g/mL

 Example 1.5.4

A piece of rebar is weighed and then submerged in a graduated cylinder partially filled with water, with results as shown.

A graduated cylinder filled with liquid is shown. One shows the level before the rebar is added and the other shows the level
with the rebar submerged in the liquid. Rebar mass is 69.658 grams, final volume is 22.4 milliliters, and initial volume is 13.5
milliliters.
a. Use these values to determine the density of this piece of rebar.
b. Rebar is mostly iron. Does your result in (a) support this statement? How?

Solution
The volume of the piece of rebar is equal to the volume of the water displaced:
3
volume = 22.4 mL − 13.5 mL = 8.9 mL = 8.9 cm

(rounded to the nearest 0.1 mL, per the rule for addition and subtraction)
The density is the mass-to-volume ratio:
mass 69.658 g
3
density = = = 7.8 g/cm
3
volume 8.9 cm

(rounded to two significant figures, per the rule for multiplication and division)
The density of iron is 7.9 g/cm3, very close to that of rebar, which lends some support to the fact that rebar is mostly iron.

Access for free at OpenStax 1.5.6 https://chem.libretexts.org/@go/page/39651


 Exercise 1.5.4
An irregularly shaped piece of a shiny yellowish material is weighed and then submerged in a graduated cylinder, with results
as shown.

A graduated cylinder filled with liquid is shown. One shows the level before the material is added and the other shows the level
with the material submerged in the liquid. Mass is 51.842 grams, final volume is 19.8 milliliters, and initial volume is 17.1
milliliters.
a. Use these values to determine the density of this material.
b. Do you have any reasonable guesses as to the identity of this material? Explain your reasoning.

Answer a
19 g/cm3
Answer b
It is likely gold; it has the right appearance for gold and very close to the density given for gold.

1.5.3: Accuracy and Precision


Scientists typically make repeated measurements of a quantity to ensure the quality of their findings and to know both the precision
and the accuracy of their results. Measurements are said to be precise if they yield very similar results when repeated in the same
manner. A measurement is considered accurate if it yields a result that is very close to the true or accepted value. Precise values
agree with each other; accurate values agree with a true value. These characterizations can be extended to other contexts, such as
the results of an archery competition (Figure 1.5.2).

Figure 1.5.2 : (a) These arrows are close to both the bull’s eye and one another, so they are both accurate and precise. (b) These
arrows are close to one another but not on target, so they are precise but not accurate. (c) These arrows are neither on target nor
close to one another, so they are neither accurate nor precise.
Suppose a quality control chemist at a pharmaceutical company is tasked with checking the accuracy and precision of three
different machines that are meant to dispense 10 ounces (296 mL) of cough syrup into storage bottles. She proceeds to use each

Access for free at OpenStax 1.5.7 https://chem.libretexts.org/@go/page/39651


machine to fill five bottles and then carefully determines the actual volume dispensed, obtaining the results tabulated in Table
1.5.2.

Table 1.5.2 : Volume (mL) of Cough Medicine Delivered by 10-oz (296 mL) Dispensers
Dispenser #1 Dispenser #2 Dispenser #3

283.3 298.3 296.1

284.1 294.2 295.9

283.9 296.0 296.1

284.0 297.8 296.0

284.1 293.9 296.1

Considering these results, she will report that dispenser #1 is precise (values all close to one another, within a few tenths of a
milliliter) but not accurate (none of the values are close to the target value of 296 mL, each being more than 10 mL too low).
Results for dispenser #2 represent improved accuracy (each volume is less than 3 mL away from 296 mL) but worse precision
(volumes vary by more than 4 mL). Finally, she can report that dispenser #3 is working well, dispensing cough syrup both
accurately (all volumes within 0.1 mL of the target volume) and precisely (volumes differing from each other by no more than 0.2
mL).

Summary
Quantities can be exact or measured. Measured quantities have an associated uncertainty that is represented by the number of
significant figures in the measurement. The uncertainty of a calculated value depends on the uncertainties in the values used in the
calculation and is reflected in how the value is rounded. Measured values can be accurate (close to the true value) and/or precise
(showing little variation when measured repeatedly).

Glossary
uncertainty
estimate of amount by which measurement differs from true value

significant figures
(also, significant digits) all of the measured digits in a determination, including the uncertain last digit

rounding
procedure used to ensure that calculated results properly reflect the uncertainty in the measurements used in the calculation

precision
how closely a measurement matches the same measurement when repeated

exact number
number derived by counting or by definition

accuracy
how closely a measurement aligns with a correct value

This page titled 1.5: Measurement Uncertainty, Accuracy, and Precision is shared under a CC BY 4.0 license and was authored, remixed, and/or
curated by OpenStax via source content that was edited to the style and standards of the LibreTexts platform; a detailed edit history is available
upon request.

Access for free at OpenStax 1.5.8 https://chem.libretexts.org/@go/page/39651


1.6: Mathematical Treatment of Measurement Results
 Learning Objectives
Explain the dimensional analysis (factor label) approach to mathematical calculations involving quantities.
Describe how to use dimensional analysis to carry out unit conversions for a given property and computations involving two or more properties.
Convert between the three main temperature units: Fahrenheit, Celsius, and Kelvin.

It is often the case that a quantity of interest may not be easy (or even possible) to measure directly but instead must be calculated from other directly measured properties and appropriate
mathematical relationships. For example, consider measuring the average speed of an athlete running sprints. This is typically accomplished by measuring the time required for the athlete to run from
the starting line to the finish line, and the distance between these two lines, and then computing speed from the equation that relates these three properties:
distance
speed =
time

An Olympic-quality sprinter can run 100 m in approximately 10 s, corresponding to an average speed of


100 m
= 10 m/s
10 s

Note that this simple arithmetic involves dividing the numbers of each measured quantity to yield the number of the computed quantity (100/10 = 10) and likewise dividing the units of each measured
quantity to yield the unit of the computed quantity (m/s = m/s). Now, consider using this same relation to predict the time required for a person running at this speed to travel a distance of 25 m. The
same relation between the three properties is used, but in this case, the two quantities provided are a speed (10 m/s) and a distance (25 m). To yield the sought property, time, the equation must be
rearranged appropriately:
distance
time =
speed

The time can then be computed as:


25 m
= 2.5 s
10 m/s

Again, arithmetic on the numbers (25/10 = 2.5) was accompanied by the same arithmetic on the units (m/m/s = s) to yield the number and unit of the result, 2.5 s. Note that, just as for numbers, when
a unit is divided by an identical unit (in this case, m/m), the result is “1”—or, as commonly phrased, the units “cancel.”
These calculations are examples of a versatile mathematical approach known as dimensional analysis (or the factor-label method). Dimensional analysis is based on this premise: the units of quantities
must be subjected to the same mathematical operations as their associated numbers. This method can be applied to computations ranging from simple unit conversions to more complex, multi-step
calculations involving several different quantities.

1.6.1: Conversion Factors and Dimensional Analysis


A ratio of two equivalent quantities expressed with different measurement units can be used as a unit conversion factor. For example, the lengths of 2.54 cm and 1 in. are equivalent (by definition),
and so a unit conversion factor may be derived from the ratio,
2.54 cm cm
(2.54 cm = 1 in. ) or 2.54
1 in. in.

Several other commonly used conversion factors are given in Table 1.6.1.
Table 1.6.1 : Common Conversion Factors
Length Volume Mass

1 m = 1.0936 yd 1 L = 1.0567 qt 1 kg = 2.2046 lb

1 in. = 2.54 cm (exact) 1 qt = 0.94635 L 1 lb = 453.59 g

1 km = 0.62137 mi 1 ft3 = 28.317 L 1 (avoirdupois) oz = 28.349 g

1 mi = 1609.3 m 1 tbsp = 14.787 mL 1 (troy) oz = 31.103 g

When we multiply a quantity (such as distance given in inches) by an appropriate unit conversion factor, we convert the quantity to an equivalent value with different units (such as distance in
centimeters). For example, a basketball player’s vertical jump of 34 inches can be converted to centimeters by:
2.54 cm
34 in. × = 86 cm
1 in.

Since this simple arithmetic involves quantities, the premise of dimensional analysis requires that we multiply both numbers and units. The numbers of these two quantities are multiplied to yield the
number of the product quantity, 86, whereas the units are multiplied to yield
in. ×cm
.
in.

Just as for numbers, a ratio of identical units is also numerically equal to one,
in.
=1
in.

and the unit product thus simplifies to cm. (When identical units divide to yield a factor of 1, they are said to “cancel.”) Using dimensional analysis, we can determine that a unit conversion factor has
been set up correctly by checking to confirm that the original unit will cancel, and the result will contain the sought (converted) unit.

 Example 1.6.1: Using a Unit Conversion Factor


The mass of a competition Frisbee is 125 g. Convert its mass to ounces using the unit conversion factor derived from the relationship 1 oz = 28.349 g (Table 1.6.1).

Solution
If we have the conversion factor, we can determine the mass in kilograms using an equation similar the one used for converting length from inches to centimeters.

x oz = 125 g × unit conversion f actor

We write the unit conversion factor in its two forms:

Access for free at OpenStax 1.6.1 https://chem.libretexts.org/@go/page/39652


1 oz 28.349 g
and
28.349 g 1 oz

The correct unit conversion factor is the ratio that cancels the units of grams and leaves ounces.
1 oz
x oz = 125 g ×
28.349 g

125
=( ) oz
28.349

= 4.41 oz (three signif icant f igures)

 Exercise 1.6.1
Convert a volume of 9.345 qt to liters.

Answer
8.844 L

Beyond simple unit conversions, the factor-label method can be used to solve more complex problems involving computations. Regardless of the details, the basic approach is the same—all the
factors involved in the calculation must be appropriately oriented to insure that their labels (units) will appropriately cancel and/or combine to yield the desired unit in the result. This is why it is
referred to as the factor-label method. As your study of chemistry continues, you will encounter many opportunities to apply this approach.

 Example 1.6.2: Computing Quantities from Measurement Results

What is the density of common antifreeze in units of g/mL? A 4.00-qt sample of the antifreeze weighs 9.26 lb.

Solution
mass
Since density = , we need to divide the mass in grams by the volume in milliliters. In general: the number of units of B = the number of units of A × unit conversion factor. The
volume
necessary conversion factors are given in Table 1.7.1: 1 lb = 453.59 g; 1 L = 1.0567 qt; 1 L = 1,000 mL. We can convert mass from pounds to grams in one step:
453.59 g
3
9.26 lb × = 4.20 × 10 g
1 lb

We need to use two steps to convert volume from quarts to milliliters.


1. Convert quarts to liters.
1 L
4.00 qt × = 3.78 L
1.0567 qt

2. Convert liters to milliliters.


1000 mL 3
3.78 L × = 3.78 × 10 mL
1 L

Then,
3
4.20 × 10 g
density = = 1.11 g/mL
3
3.78 × 10 mL

Alternatively, the calculation could be set up in a way that uses three unit conversion factors sequentially as follows:

9.26 lb 1.0567 qt 1 L
453.59 g
× × × = 1.11 g/mL
4.00 qt 1 L 1000 mL
1 lb

 Exercise 1.6.2
What is the volume in liters of 1.000 oz, given that 1 L = 1.0567 qt and 1 qt = 32 oz (exactly)?

Answer
−2
2.956 × 10 L

 Example 1.6.3: Computing Quantities from Measurement Results

While being driven from Philadelphia to Atlanta, a distance of about 1250 km, a 2014 Lamborghini Aventador Roadster uses 213 L gasoline.
a. What (average) fuel economy, in miles per gallon, did the Roadster get during this trip?
b. If gasoline costs $3.80 per gallon, what was the fuel cost for this trip?

Solution
(a) We first convert distance from kilometers to miles:
0.62137 mi
1250 km × = 777 mi
1 km

and then convert volume from liters to gallons:

1.0567 qt
1 gal
213 L × × = 56.3 gal
1 L 4 qt

Then,

Access for free at OpenStax 1.6.2 https://chem.libretexts.org/@go/page/39652


777 mi
(average) mileage = = 13.8 miles/gallon = 13.8 mpg
56.3 gal

Alternatively, the calculation could be set up in a way that uses all the conversion factors sequentially, as follows:

1250 km 0.62137 mi 1 L 4 qt
× × × = 13.8 mpg
213 L 1 km 1.0567 qt 1 gal

(b) Using the previously calculated volume in gallons, we find:


$3.80
56.3 gal × = $214
1 gal

 Exercise 1.6.3

A Toyota Prius Hybrid uses 59.7 L gasoline to drive from San Francisco to Seattle, a distance of 1300 km (two significant digits).
a. What (average) fuel economy, in miles per gallon, did the Prius get during this trip?
b. If gasoline costs $3.90 per gallon, what was the fuel cost for this trip?

Answer a
51 mpg
Answer b
$62

1.6.2: Conversion of Temperature Units


We use the word temperature to refer to the hotness or coldness of a substance. One way we measure a change in temperature is to use the fact that most substances expand when their temperature
increases and contract when their temperature decreases. The mercury or alcohol in a common glass thermometer changes its volume as the temperature changes. Because the volume of the liquid
changes more than the volume of the glass, we can see the liquid expand when it gets warmer and contract when it gets cooler.
To mark a scale on a thermometer, we need a set of reference values: Two of the most commonly used are the freezing and boiling temperatures of water at a specified atmospheric pressure. On the
Celsius scale, 0 °C is defined as the freezing temperature of water and 100 °C as the boiling temperature of water. The space between the two temperatures is divided into 100 equal intervals, which
we call degrees. On the Fahrenheit scale, the freezing point of water is defined as 32 °F and the boiling temperature as 212 °F. The space between these two points on a Fahrenheit thermometer is
divided into 180 equal parts (degrees).
Defining the Celsius and Fahrenheit temperature scales as described in the previous paragraph results in a slightly more complex relationship between temperature values on these two scales than for
different units of measure for other properties. Most measurement units for a given property are directly proportional to one another (y = mx). Using familiar length units as one example:
1 ft
length in f eet = ( ) × length in inches
12 in.

where
y = length in feet,
x = length in inches, and
the proportionality constant, m, is the conversion factor.
The Celsius and Fahrenheit temperature scales, however, do not share a common zero point, and so the relationship between these two scales is a linear one rather than a proportional one (
y = mx + b ). Consequently, converting a temperature from one of these scales into the other requires more than simple multiplication by a conversion factor, m, it also must take into account

differences in the scales’ zero points (b ).


The linear equation relating Celsius and Fahrenheit temperatures is easily derived from the two temperatures used to define each scale. Representing the Celsius temperature as x and the Fahrenheit
temperature as y , the slope, m, is computed to be:
Δy
m =
Δx

∘ ∘
212 F − 32 F
=
∘ ∘
100 C −0 C


180 F
=

100 C


9 F
=

5 C

The y-intercept of the equation, b, is then calculated using either of the equivalent temperature pairs, (100 °C, 212 °F) or (0 °C, 32 °F), as:
b = y − mx



9 F ∘
= 32 F− ×0 C

5 C


= 32 F

The equation relating the temperature scales is then:



9 F ∘
T∘ F = ( × T∘ C ) + 32 C

5 C

An abbreviated form of this equation that omits the measurement units is:
9
T∘ F = × T∘ C + 32
5

Rearrangement of this equation yields the form useful for converting from Fahrenheit to Celsius:
5
T∘ C = (T∘ F + 32)
9

As mentioned earlier in this chapter, the SI unit of temperature is the kelvin (K). Unlike the Celsius and Fahrenheit scales, the kelvin scale is an absolute temperature scale in which 0 (zero) K
corresponds to the lowest temperature that can theoretically be achieved. The early 19th-century discovery of the relationship between a gas's volume and temperature suggested that the volume of a

Access for free at OpenStax 1.6.3 https://chem.libretexts.org/@go/page/39652


gas would be zero at −273.15 °C. In 1848, British physicist William Thompson, who later adopted the title of Lord Kelvin, proposed an absolute temperature scale based on this concept (further
treatment of this topic is provided in this text’s chapter on gases).
The freezing temperature of water on this scale is 273.15 K and its boiling temperature 373.15 K. Notice the numerical difference in these two reference temperatures is 100, the same as for the
K
Celsius scale, and so the linear relation between these two temperature scales will exhibit a slope of 1 ∘
. Following the same approach, the equations for converting between the kelvin and Celsius
C
temperature scales are derived to be:

TK = T∘ C + 273.15

T∘ C = TK − 273.15

The 273.15 in these equations has been determined experimentally, so it is not exact. Figure 1.6.1 shows the relationship among the three temperature scales. Recall that we do not use the degree sign
with temperatures on the kelvin scale.

Figure 1.6.1 : The Fahrenheit, Celsius, and kelvin temperature scales are compared.
Although the kelvin (absolute) temperature scale is the official SI temperature scale, Celsius is commonly used in many scientific contexts and is the scale of choice for nonscience contexts in almost
all areas of the world. Very few countries (the U.S. and its territories, the Bahamas, Belize, Cayman Islands, and Palau) still use Fahrenheit for weather, medicine, and cooking.

 Example 1.6.4: Conversion from Celsius

Normal body temperature has been commonly accepted as 37.0 °C (although it varies depending on time of day and method of measurement, as well as among individuals). What is this
temperature on the kelvin scale and on the Fahrenheit scale?

Solution

K= C + 273.15 = 37.0 + 273.2 = 310.2 K

9 9
∘ ∘ ∘
F = C + 32.0 = ( × 37.0) + 32.0 = 66.6 + 32.0 = 98.6 F
5 5

 Exercise 1.6.4
Convert 80.92 °C to K and °F.

Answer
354.07 K, 177.7 °F

 Example 1.6.5: Conversion from Fahrenheit

Baking a ready-made pizza calls for an oven temperature of 450 °F. If you are in Europe, and your oven thermometer uses the Celsius scale, what is the setting? What is the kelvin temperature?

Solution
5 5 5
∘ ∘ ∘ ∘
C = ( F − 32) = (450 − 32) = × 418 = 232 C → set oven to 230 C (two significant figures)
9 9 9

∘ 2
K= C + 273.15 = 230 + 273 = 503 K → 5.0 × 10 K (two signif icant f igures)

 Exercise 1.6.5

Convert 50 °F to °C and K.

Answer
10 °C, 280 K

Summary
Measurements are made using a variety of units. It is often useful or necessary to convert a measured quantity from one unit into another. These conversions are accomplished using unit conversion
factors, which are derived by simple applications of a mathematical approach called the factor-label method or dimensional analysis. This strategy is also employed to calculate sought quantities using
measured quantities and appropriate mathematical relations.

Access for free at OpenStax 1.6.4 https://chem.libretexts.org/@go/page/39652


1.6.3: Key Equations
5
T∘ C = × T∘ F − 32
9
9
T∘ F = × T∘ C + 32
5

TK = C + 273.15

T∘ C = K − 273.15

Glossary
dimensional analysis
(also, factor-label method) versatile mathematical approach that can be applied to computations ranging from simple unit conversions to more complex, multi-step calculations involving several
different quantities

Fahrenheit
unit of temperature; water freezes at 32 °F and boils at 212 °F on this scale

unit conversion factor


ratio of equivalent quantities expressed with different units; used to convert from one unit to a different unit

This page titled 1.6: Mathematical Treatment of Measurement Results is shared under a CC BY 4.0 license and was authored, remixed, and/or curated by OpenStax via source content that was edited to the style and
standards of the LibreTexts platform; a detailed edit history is available upon request.

Access for free at OpenStax 1.6.5 https://chem.libretexts.org/@go/page/39652


Access for free at OpenStax 1.6.6 https://chem.libretexts.org/@go/page/39652
1.E: Essential Ideas of Chemistry (Exercises)
1.E.1: 1.1: Chemistry in Context
1.E.1.1: Q1.1.1
Explain how you could experimentally determine whether the outside temperature is higher or lower than 0 °C (32 °F) without
using a thermometer.

1.E.1.2: S1.1.1
Place a glass of water outside. It will freeze if the temperature is below 0 °C.

1.E.1.3: Q1.1.2
Identify each of the following statements as being most similar to a hypothesis, a law, or a theory. Explain your reasoning.
a. Falling barometric pressure precedes the onset of bad weather.
b. All life on earth has evolved from a common, primitive organism through the process of natural selection.
c. My truck’s gas mileage has dropped significantly, probably because it’s due for a tune-up.

1.E.1.4: Q1.1.3
Identify each of the following statements as being most similar to a hypothesis, a law, or a theory. Explain your reasoning.
a. The pressure of a sample of gas is directly proportional to the temperature of the gas.
b. Matter consists of tiny particles that can combine in specific ratios to form substances with specific properties.
c. At a higher temperature, solids (such as salt or sugar) will dissolve better in water.

1.E.1.5: S1.1.3
(a) law (states a consistently observed phenomenon, can be used for prediction); (b) theory (a widely accepted explanation of the
behavior of matter); (c) hypothesis (a tentative explanation, can be investigated by experimentation)
Identify each of the underlined items as a part of either the macroscopic domain, the microscopic domain, or the symbolic domain
of chemistry. For any in the symbolic domain, indicate whether they are symbols for a macroscopic or a microscopic feature.
a. (a) The mass of a lead pipe is 14 lb.
b. (b) The mass of a certain chlorine atom is 35 amu.
c. (c) A bottle with a label that reads Al contains aluminum metal.
d. (d) Al is the symbol for an aluminum atom.
Identify each of the underlined items as a part of either the macroscopic domain, the microscopic domain, or the symbolic domain
of chemistry. For those in the symbolic domain, indicate whether they are symbols for a macroscopic or a microscopic feature.
a. (a) A certain molecule contains one H atom and one Cl atom.
b. (b) Copper wire has a density of about 8 g/cm3.
c. (c) The bottle contains 15 grams of Ni powder.
d. (d) A sulfur molecule is composed of eight sulfur atoms.
(a) symbolic, microscopic; (b) macroscopic; (c) symbolic, macroscopic; (d) microscopic
According to one theory, the pressure of a gas increases as its volume decreases because the molecules in the gas have to move a
shorter distance to hit the walls of the container. Does this theory follow a macroscopic or microscopic description of chemical
behavior? Explain your answer.
The amount of heat required to melt 2 lbs of ice is twice the amount of heat required to melt 1 lb of ice. Is this observation a
macroscopic or microscopic description of chemical behavior? Explain your answer.
Macroscopic. The heat required is determined from macroscopic properties.

1.E.2: 1.2: Phases and Classification of Matter

Access for free at OpenStax 1.E.1 https://chem.libretexts.org/@go/page/68815


1.E.2.1: Questions
1. Why do we use an object's mass, rather than its weight, to indicate the amount of matter it contains?
2. What properties distinguish solids from liquids? Liquids from gases? Solids from gases?
3. How does a heterogeneous mixture differ from a homogeneous mixture? How are they similar?
4. How does a homogeneous mixture differ from a pure substance? How are they similar?
5. How does an element differ from a compound? How are they similar?
6. How do molecules of elements and molecules of compounds differ? In what ways are they similar?
7. How does an atom differ from a molecule? In what ways are they similar?
8. Many of the items you purchase are mixtures of pure compounds. Select three of these commercial products and prepare a list
of the ingredients that are pure compounds.
9. Classify each of the following as an element, a compound, or a mixture:
a. copper
b. water
c. nitrogen
d. sulfur
e. air
f. sucrose
g. a substance composed of molecules each of which contains two iodine atoms
h. gasoline
10. Classify each of the following as an element, a compound, or a mixture:
a. iron
b. oxygen
c. mercury oxide
d. pancake syrup
e. carbon dioxide
f. a substance composed of molecules each of which contains one hydrogen atom and one chlorine atom
g. baking soda
h. baking powder
11. A sulfur atom and a sulfur molecule are not identical. What is the difference?
12. How are the molecules in oxygen gas, the molecules in hydrogen gas, and water molecules similar? How do they differ?
13. We refer to astronauts in space as weightless, but not without mass. Why?
14. As we drive an automobile, we don't think about the chemicals consumed and produced. Prepare a list of the principal
chemicals consumed and produced during the operation of an automobile.
15. Matter is everywhere around us. Make a list by name of fifteen different kinds of matter that you encounter every day. Your list
should include (and label at least one example of each) the following: a solid, a liquid, a gas, an element, a compound, a
homogenous mixture, a heterogeneous mixture, and a pure substance.
16. When elemental iron corrodes it combines with oxygen in the air to ultimately form red brown iron(III) oxide which we call
rust. (a) If a shiny iron nail with an initial mass of 23.2 g is weighed after being coated in a layer of rust, would you expect the
mass to have increased, decreased, or remained the same? Explain. (b) If the mass of the iron nail increases to 24.1 g, what
mass of oxygen combined with the iron?
17. As stated in the text, convincing examples that demonstrate the law of conservation of matter outside of the laboratory are few
and far between. Indicate whether the mass would increase, decrease, or stay the same for the following scenarios where
chemical reactions take place:
a. Exactly one pound of bread dough is placed in a baking tin. The dough is cooked in an oven at 350 °F releasing a wonderful
aroma of freshly baked bread during the cooking process. Is the mass of the baked loaf less than, greater than, or the same as
the one pound of original dough? Explain.
b. When magnesium burns in air a white flaky ash of magnesium oxide is produced. Is the mass of magnesium oxide less than,
greater than, or the same as the original piece of magnesium? Explain.
c. Antoine Lavoisier, the French scientist credited with first stating the law of conservation of matter, heated a mixture of tin
and air in a sealed flask to produce tin oxide. Did the mass of the sealed flask and contents decrease, increase, or remain the
same after the heating?

Access for free at OpenStax 1.E.2 https://chem.libretexts.org/@go/page/68815


18. Yeast converts glucose to ethanol and carbon dioxide during anaerobic fermentation as depicted in the simple chemical equation
here:
glucose → ethanol + carbon dioxide (1.E.1)

a. If 200.0 g of glucose is fully converted, what will be the total mass of ethanol and carbon dioxide produced?
b. If the fermentation is carried out in an open container, would you expect the mass of the container and contents after
fermentation to be less than, greater than, or the same as the mass of the container and contents before fermentation?
Explain.
c. If 97.7 g of carbon dioxide is produced, what mass of ethanol is produced?

1.E.2.2: Solutions
2 Liquids can change their shape (flow); solids can’t. Gases can undergo large volume changes as pressure changes; liquids do not.
Gases flow and change volume; solids do not.
4.The mixture can have a variety of compositions; a pure substance has a definite composition. Both have the same composition
from point to point.
6 Molecules of elements contain only one type of atom; molecules of compounds contain two or more types of atoms. They are
similar in that both are comprised of two or more atoms chemically bonded together.
8. Answers will vary. Sample answer: Gatorade contains water, sugar, dextrose, citric acid, salt, sodium chloride, monopotassium
phosphate, and sucrose acetate isobutyrate.
11. (a) element; (b) element; (c) compound; (d) mixture, (e) compound; (f) compound; (g) compound; (h) mixture
12. In each case, a molecule consists of two or more combined atoms. They differ in that the types of atoms change from one
substance to the next.
14. Gasoline (a mixture of compounds), oxygen, and to a lesser extent, nitrogen are consumed. Carbon dioxide and water are the
principal products. Carbon monoxide and nitrogen oxides are produced in lesser amounts.
16. (a) Increased as it would have combined with oxygen in the air thus increasing the amount of matter and therefore the mass. (b)
0.9 g
18. (a) 200.0 g; (b) The mass of the container and contents would decrease as carbon dioxide is a gaseous product and would leave
the container. (c) 102.3 g

1.E.3: 1.3: Physical and Chemical Properties


Classify the six underlined properties in the following paragraph as chemical or physical:
Fluorine is a pale yellow gas that reacts with most substances. The free element melts at −220 °C and boils at −188 °C. Finely
divided metals burn in fluorine with a bright flame. Nineteen grams of fluorine will react with 1.0 gram of hydrogen.
Classify each of the following changes as physical or chemical:
a. condensation of steam
b. burning of gasoline
c. souring of milk
d. dissolving of sugar in water
e. melting of gold
(a) physical; (b) chemical; (c) chemical; (d) physical; (e) physical
Classify each of the following changes as physical or chemical:
a. coal burning
b. ice melting
c. mixing chocolate syrup with milk
d. explosion of a firecracker
e. magnetizing of a screwdriver

Access for free at OpenStax 1.E.3 https://chem.libretexts.org/@go/page/68815


The volume of a sample of oxygen gas changed from 10 mL to 11 mL as the temperature changed. Is this a chemical or physical
change?
physical
A 2.0-liter volume of hydrogen gas combined with 1.0 liter of oxygen gas to produce 2.0 liters of water vapor. Does oxygen
undergo a chemical or physical change?
Explain the difference between extensive properties and intensive properties.
The value of an extensive property depends upon the amount of matter being considered, whereas the value of an intensive
property is the same regardless of the amount of matter being considered.
Identify the following properties as either extensive or intensive.
a. volume
b. temperature
c. humidity
d. heat
e. boiling point
The density (d) of a substance is an intensive property that is defined as the ratio of its mass (m) to its volume (V).
mass m
density = d=
volume V

Considering that mass and volume are both extensive properties, explain why their ratio, density, is intensive.
Being extensive properties, both mass and volume are directly proportional to the amount of substance under study. Dividing one
extensive property by another will in effect “cancel” this dependence on amount, yielding a ratio that is independent of amount (an
intensive property).

1.E.4: 1.4: Measurements


Is one liter about an ounce, a pint, a quart, or a gallon?
Is a meter about an inch, a foot, a yard, or a mile?
about a yard
Indicate the SI base units or derived units that are appropriate for the following measurements:
1. (a) the length of a marathon race (26 miles 385 yards)
2. (b) the mass of an automobile
3. (c) the volume of a swimming pool
4. (d) the speed of an airplane
5. (e) the density of gold
6. (f) the area of a football field
7. (g) the maximum temperature at the South Pole on April 1, 1913
Indicate the SI base units or derived units that are appropriate for the following measurements:
1. (a) the mass of the moon
2. (b) the distance from Dallas to Oklahoma City
3. (c) the speed of sound
4. (d) the density of air
5. (e) the temperature at which alcohol boils
6. (f) the area of the state of Delaware
7. (g) the volume of a flu shot or a measles vaccination
(a) kilograms; (b) meters; (c) kilometers/second; (d) kilograms/cubic meter; (e) kelvin; (f) square meters; (g) cubic meters
Give the name and symbol of the prefixes used with SI units to indicate multiplication by the following exact quantities.
1. (a) 103
2. (b) 10−2

Access for free at OpenStax 1.E.4 https://chem.libretexts.org/@go/page/68815


3. (c) 0.1
4. (d) 10−3
5. (e) 1,000,000
6. (f) 0.000001
Give the name of the prefix and the quantity indicated by the following symbols that are used with SI base units.
1. (a) c
2. (b) d
3. (c) G
4. (d) k
5. (e) m
6. (f) n
7. (g) p
8. (h) T
(a) centi-, × 10−2; (b) deci-, × 10−1; (c) Giga-, × 109; (d) kilo-, × 103; (e) milli-, × 10−3; (f) nano-, × 10−9; (g) pico-, × 10−12; (h)
tera-, × 1012
A large piece of jewelry has a mass of 132.6 g. A graduated cylinder initially contains 48.6 mL water. When the jewelry is
submerged in the graduated cylinder, the total volume increases to 61.2 mL.
1. (a) Determine the density of this piece of jewelry.
2. (b) Assuming that the jewelry is made from only one substance, what substance is it likely to be? Explain.
Visit this PhET density simulation and select the Same Volume Blocks.
1. (a) What are the mass, volume, and density of the yellow block?
2. (b) What are the mass, volume and density of the red block?
3. (c) List the block colors in order from smallest to largest mass.
4. (d) List the block colors in order from lowest to highest density.
5. (e) How are mass and density related for blocks of the same volume?
(a) 8.00 kg, 5.00 L, 1.60 kg/L; (b) 2.00 kg, 5.00 L, 0.400 kg/L; (c) red < green < blue < yellow; (d) If the volumes are the same,
then the density is directly proportional to the mass.
Visit this PhET density simulation and select Custom Blocks and then My Block.
1. (a) Enter mass and volume values for the block such that the mass in kg is less than the volume in L. What does the block do?
Why? Is this always the case when mass < volume?
2. (b) Enter mass and volume values for the block such that the mass in kg is more than the volume in L. What does the block do?
Why? Is this always the case when mass > volume?
3. (c) How would (a) and (b) be different if the liquid in the tank were ethanol instead of water?
4. (d) How would (a) and (b) be different if the liquid in the tank were mercury instead of water?
Visit this PhET density simulation and select Mystery Blocks.
1. (a) Pick one of the Mystery Blocks and determine its mass, volume, density, and its likely identity.
2. (b) Pick a different Mystery Block and determine its mass, volume, density, and its likely identity.
3. (c) Order the Mystery Blocks from least dense to most dense. Explain.
(a) (b) Answer is one of the following. A/yellow: mass = 65.14 kg, volume = 3.38 L, density = 19.3 kg/L, likely identity = gold.
B/blue: mass = 0.64 kg, volume = 1.00 L, density = 0.64 kg/L, likely identity = apple. C/green: mass = 4.08 kg, volume = 5.83 L,
density = 0.700 kg/L, likely identity = gasoline. D/red: mass = 3.10 kg, volume = 3.38 L, density = 0.920 kg/L, likely identity =
ice; and E/purple: mass = 3.53 kg, volume = 1.00 L, density = 3.53 kg/L, likely identity = diamond. (c) B/blue/apple (0.64 kg/L) <
C/green/gasoline (0.700 kg/L) < D/red/ice (0.920 kg/L) < E/purple/diamond (3.53 kg/L) < A/yellow/gold (19.3 kg/L)

1.E.5: 1.5: Measurement Uncertainty, Accuracy, and Precision


Express each of the following numbers in scientific notation with correct significant figures:
1. (a) 711.0

Access for free at OpenStax 1.E.5 https://chem.libretexts.org/@go/page/68815


2. (b) 0.239
3. (c) 90743
4. (d) 134.2
5. (e) 0.05499
6. (f) 10000.0
7. (g) 0.000000738592
Express each of the following numbers in exponential notation with correct significant figures:
1. (a) 704
2. (b) 0.03344
3. (c) 547.9
4. (d) 22086
5. (e) 1000.00
6. (f) 0.0000000651
7. (g) 0.007157
(a) 7.04 × 102; (b) 3.344 × 10−2; (c) 5.479 × 102; (d) 2.2086 × 104; (e) 1.00000 × 103; (f) 6.51 × 10−8; (g) 7.157 × 10−3
Indicate whether each of the following can be determined exactly or must be measured with some degree of uncertainty:
1. (a) the number of eggs in a basket
2. (b) the mass of a dozen eggs
3. (c) the number of gallons of gasoline necessary to fill an automobile gas tank
4. (d) the number of cm in 2 m
5. (e) the mass of a textbook
6. (f) the time required to drive from San Francisco to Kansas City at an average speed of 53 mi/h
Indicate whether each of the following can be determined exactly or must be measured with some degree of uncertainty:
1. (a) the number of seconds in an hour
2. (b) the number of pages in this book
3. (c) the number of grams in your weight
4. (d) the number of grams in 3 kilograms
5. (e) the volume of water you drink in one day
6. (f) the distance from San Francisco to Kansas City
(a) exact; (b) exact; (c) uncertain; (d) exact; (e) uncertain; (f) uncertain
How many significant figures are contained in each of the following measurements?
1. (a) 38.7 g
2. (b) 2 × 1018 m
3. (c) 3,486,002 kg
4. (d) 9.74150 × 10−4 J
5. (e) 0.0613 cm3
6. (f) 17.0 kg
7. (g) 0.01400 g/mL
How many significant figures are contained in each of the following measurements?
1. (a) 53 cm
2. (b) 2.05 × 108 m
3. (c) 86,002 J
4. (d) 9.740 × 104 m/s
5. (e) 10.0613 m3
6. (f) 0.17 g/mL
7. (g) 0.88400 s
(a) two; (b) three; (c) five; (d) four; (e) six; (f) two; (g) five

Access for free at OpenStax 1.E.6 https://chem.libretexts.org/@go/page/68815


The following quantities were reported on the labels of commercial products. Determine the number of significant figures in each.
1. (a) 0.0055 g active ingredients
2. (b) 12 tablets
3. (c) 3% hydrogen peroxide
4. (d) 5.5 ounces
5. (e) 473 mL
6. (f) 1.75% bismuth
7. (g) 0.001% phosphoric acid
8. (h) 99.80% inert ingredients
Round off each of the following numbers to two significant figures:
1. (a) 0.436
2. (b) 9.000
3. (c) 27.2
4. (d) 135
5. (e) 1.497 × 10−3
6. (f) 0.445
(a) 0.44; (b) 9.0; (c) 27; (d) 140; (e) 1.5 × 10−3; (f) 0.44
Round off each of the following numbers to two significant figures:
1. (a) 517
2. (b) 86.3
3. (c) 6.382 × 103
4. (d) 5.0008
5. (e) 22.497
6. (f) 0.885
Perform the following calculations and report each answer with the correct number of significant figures.
1. (a) 628 × 342
2. (b) (5.63 × 102) × (7.4 × 103)
3. (c)
28.0
4.
13.483
5. (d) 8119 × 0.000023
6. (e) 14.98 + 27,340 + 84.7593
7. (f) 42.7 + 0.259
(a) 2.15 × 105; (b) 4.2 × 106; (c) 2.08; (d) 0.19; (e) 27,440; (f) 43.0
Perform the following calculations and report each answer with the correct number of significant figures.
1. (a) 62.8 × 34
2. (b) 0.147 + 0.0066 + 0.012
3. (c) 38 × 95 × 1.792
4. (d) 15 – 0.15 – 0.6155
0.0500
5. (e) 8.78 × ( )
0.478

6. (f) 140 + 7.68 + 0.014


7. (g) 28.7 – 0.0483
(88.5 − 87.57)
8. (h)
45.13

Consider the results of the archery contest shown in this figure.


1. (a) Which archer is most precise?
2. (b) Which archer is most accurate?

Access for free at OpenStax 1.E.7 https://chem.libretexts.org/@go/page/68815


3. (c) Who is both least precise and least accurate?

(a) Archer X; (b) Archer W; (c) Archer Y


Classify the following sets of measurements as accurate, precise, both, or neither.
1. (a) Checking for consistency in the weight of chocolate chip cookies: 17.27 g, 13.05 g, 19.46 g, 16.92 g
2. (b) Testing the volume of a batch of 25-mL pipettes: 27.02 mL, 26.99 mL, 26.97 mL, 27.01 mL
3. (c) Determining the purity of gold: 99.9999%, 99.9998%, 99.9998%, 99.9999%

1.E.6: 1.6: Mathematical Treatment of Measurement Results


Write conversion factors (as ratios) for the number of:
a. yards in 1 meter
b. liters in 1 liquid quart
c. pounds in 1 kilogram
1.0936 yd 0.94635 L 2.2046 lb
(a) ; (b) ; (c)
1 m 1 qt 1 kg

Write conversion factors (as ratios) for the number of:


1. (a) kilometers in 1 mile
2. (b) liters in 1 cubic foot
3. (c) grams in 1 ounce
The label on a soft drink bottle gives the volume in two units: 2.0 L and 67.6 fl oz. Use this information to derive a conversion
factor between the English and metric units. How many significant figures can you justify in your conversion factor?
2.0 L 0.030 L
=
67.6 f l oz 1 f l oz

Only two significant figures are justified.


The label on a box of cereal gives the mass of cereal in two units: 978 grams and 34.5 oz. Use this information to find a conversion
factor between the English and metric units. How many significant figures can you justify in your conversion factor?
Soccer is played with a round ball having a circumference between 27 and 28 in. and a weight between 14 and 16 oz. What are
these specifications in units of centimeters and grams?
68–71 cm; 400–450 g
A woman's basketball has a circumference between 28.5 and 29.0 inches and a maximum weight of 20 ounces (two significant
figures). What are these specifications in units of centimeters and grams?
How many milliliters of a soft drink are contained in a 12.0-oz can?

Access for free at OpenStax 1.E.8 https://chem.libretexts.org/@go/page/68815


355 mL
A barrel of oil is exactly 42 gal. How many liters of oil are in a barrel?
The diameter of a red blood cell is about 3 × 10−4 in. What is its diameter in centimeters?
8 × 10−4 cm
The distance between the centers of the two oxygen atoms in an oxygen molecule is 1.21 × 10−8 cm. What is this distance in
inches?
Is a 197-lb weight lifter light enough to compete in a class limited to those weighing 90 kg or less?
yes; weight = 89.4 kg
A very good 197-lb weight lifter lifted 192 kg in a move called the clean and jerk. What was the mass of the weight lifted in
pounds?
Many medical laboratory tests are run using 5.0 μL blood serum. What is this volume in milliliters?
5.0 × 10−3 mL
If an aspirin tablet contains 325 mg aspirin, how many grams of aspirin does it contain?
Use scientific (exponential) notation to express the following quantities in terms of the SI base units in [link]:
1. (a) 0.13 g
2. (b) 232 Gg
3. (c) 5.23 pm
4. (d) 86.3 mg
5. (e) 37.6 cm
6. (f) 54 μm
7. (g) 1 Ts
8. (h) 27 ps
9. (i) 0.15 mK
(a) 1.3 × 10−4 kg; (b) 2.32 × 108 kg; (c) 5.23 × 10−12 m; (d) 8.63 × 10−5 kg; (e) 3.76 × 10−1 m; (f) 5.4 × 10−5 m; (g) 1 × 1012 s;
(h) 2.7 × 10−11 s; (i) 1.5 × 10−4 K
Complete the following conversions between SI units.
1. (a) 612 g = ________ mg
2. (b) 8.160 m = ________ cm
3. (c) 3779 μg = ________ g
4. (d) 781 mL = ________ L
5. (e) 4.18 kg = ________ g
6. (f) 27.8 m = ________ km
7. (g) 0.13 mL = ________ L
8. (h) 1738 km = ________ m
9. (i) 1.9 Gg = ________ g
Gasoline is sold by the liter in many countries. How many liters are required to fill a 12.0-gal gas tank?
45.4 L
Milk is sold by the liter in many countries. What is the volume of exactly 1/2 gal of milk in liters?
A long ton is defined as exactly 2240 lb. What is this mass in kilograms?
1.0160 × 103 kg
Make the conversion indicated in each of the following:
1. (a) the men’s world record long jump, 29 ft 4¼ in., to meters
2. (b) the greatest depth of the ocean, about 6.5 mi, to kilometers
3. (c) the area of the state of Oregon, 96,981 mi2, to square kilometers

Access for free at OpenStax 1.E.9 https://chem.libretexts.org/@go/page/68815


4. (d) the volume of 1 gill (exactly 4 oz) to milliliters
5. (e) the estimated volume of the oceans, 330,000,000 mi3, to cubic kilometers.
6. (f) the mass of a 3525-lb car to kilograms
7. (g) the mass of a 2.3-oz egg to grams
Make the conversion indicated in each of the following:
1. (a) the length of a soccer field, 120 m (three significant figures), to feet
2. (b) the height of Mt. Kilimanjaro, at 19,565 ft the highest mountain in Africa, to kilometers
3. (c) the area of an 8.5 t 11-inch sheet of paper in cm2
4. (d) the displacement volume of an automobile engine, 161 in.3, to liters
5. (e) the estimated mass of the atmosphere, 5.6 t 1015 tons, to kilograms
6. (f) the mass of a bushel of rye, 32.0 lb, to kilograms
7. (g) the mass of a 5.00-grain aspirin tablet to milligrams (1 grain = 0.00229 oz)
1. (a) 394 ft
2.
3. (b) 5.9634 km
4.
5. (c) 6.0 × 102
6.
7. (d) 2.64 L
8.
9. (e) 5.1 × 1018 kg
10.
11. (f) 14.5 kg
12.
13. (g) 324 mg
Many chemistry conferences have held a 50-Trillion Angstrom Run (two significant figures). How long is this run in kilometers
and in miles? (1 Å = 1 × 10−10 m)
A chemist’s 50-Trillion Angstrom Run (see Exercise) would be an archeologist’s 10,900 cubit run. How long is one cubit in meters
and in feet? (1 Å = 1 × 10−8 cm)
0.46 m; 1.5 ft/cubit
The gas tank of a certain luxury automobile holds 22.3 gallons according to the owner’s manual. If the density of gasoline is 0.8206
g/mL, determine the mass in kilograms and pounds of the fuel in a full tank.
As an instructor is preparing for an experiment, he requires 225 g phosphoric acid. The only container readily available is a 150-
mL Erlenmeyer flask. Is it large enough to contain the acid, whose density is 1.83 g/mL?
Yes, the acid's volume is 123 mL.
To prepare for a laboratory period, a student lab assistant needs 125 g of a compound. A bottle containing 1/4 lb is available. Did
the student have enough of the compound?
A chemistry student is 159 cm tall and weighs 45.8 kg. What is her height in inches and weight in pounds?
62.6 in (about 5 ft 3 in.) and 101 lb
In a recent Grand Prix, the winner completed the race with an average speed of 229.8 km/h. What was his speed in miles per hour,
meters per second, and feet per second?
Solve these problems about lumber dimensions.
(a) To describe to a European how houses are constructed in the US, the dimensions of “two-by-four” lumber must be converted
into metric units. The thickness × width × length dimensions are 1.50 in. × 3.50 in. × 8.00 ft in the US. What are the dimensions
in cm × cm × m?
(b) This lumber can be used as vertical studs, which are typically placed 16.0 in. apart. What is that distance in centimeters?

Access for free at OpenStax 1.E.10 https://chem.libretexts.org/@go/page/68815


(a) 3.81 cm × 8.89 cm × 2.44 m; (b) 40.6 cm
The mercury content of a stream was believed to be above the minimum considered safe—1 part per billion (ppb) by weight. An
analysis indicated that the concentration was 0.68 parts per billion. What quantity of mercury in grams was present in 15.0 L of the
1 ng Hg
water, the density of which is 0.998 g/ml? (1 ppb Hg = )
1 g water

Calculate the density of aluminum if 27.6 cm3 has a mass of 74.6 g.


2.70 g/cm3
Osmium is one of the densest elements known. What is its density if 2.72 g has a volume of 0.121 cm3?
Calculate these masses.
(a) What is the mass of 6.00 cm3 of mercury, density = 13.5939 g/cm3?
(b) What is the mass of 25.0 mL octane, density = 0.702 g/cm3?
(a) 81.6 g; (b) 17.6 g
Calculate these masses.
1. (a) What is the mass of 4.00 cm3 of sodium, density = 0.97 g/cm?
2. (b) What is the mass of 125 mL gaseous chlorine, density = 3.16 g/L?
Calculate these volumes.
1. (a) What is the volume of 25 g iodine, density = 4.93 g/cm3?
2. (b) What is the volume of 3.28 g gaseous hydrogen, density = 0.089 g/L?
(a) 5.1 mL; (b) 37 L
Calculate these volumes.
1. (a) What is the volume of 11.3 g graphite, density = 2.25 g/cm3?
2. (b) What is the volume of 39.657 g bromine, density = 2.928 g/cm3?
Convert the boiling temperature of gold, 2966 °C, into degrees Fahrenheit and kelvin.
5371 °F, 3239 K
Convert the temperature of scalding water, 54 °C, into degrees Fahrenheit and kelvin.
Convert the temperature of the coldest area in a freezer, −10 °F, to degrees Celsius and kelvin.
−23 °C, 250 K
Convert the temperature of dry ice, −77 °C, into degrees Fahrenheit and kelvin.
Convert the boiling temperature of liquid ammonia, −28.1 °F, into degrees Celsius and kelvin.
−33.4 °C, 239.8 K
The label on a pressurized can of spray disinfectant warns against heating the can above 130 °F. What are the corresponding
temperatures on the Celsius and kelvin temperature scales?
The weather in Europe was unusually warm during the summer of 1995. The TV news reported temperatures as high as 45 °C.
What was the temperature on the Fahrenheit scale?
113 °F

1.E.7: Contributors and Attributions


Paul Flowers (University of North Carolina - Pembroke), Klaus Theopold (University of Delaware) and Richard Langley
(Stephen F. Austin State University) with contributing authors. Textbook content produced by OpenStax College is licensed
under a Creative Commons Attribution License 4.0 license. Download for free at http://cnx.org/contents/85abf193-
2bd...a7ac8df6@9.110).

Access for free at OpenStax 1.E.11 https://chem.libretexts.org/@go/page/68815


This page titled 1.E: Essential Ideas of Chemistry (Exercises) is shared under a CC BY 4.0 license and was authored, remixed, and/or curated by
OpenStax via source content that was edited to the style and standards of the LibreTexts platform; a detailed edit history is available upon request.

Access for free at OpenStax 1.E.12 https://chem.libretexts.org/@go/page/68815


CHAPTER OVERVIEW
2: Atoms, Molecules, and Ions

A general chemistry Libretexts Textbook remixed and remastered from


OpenStax's textbook:
General Chemistry
This chapter will describe some of the fundamental chemical principles related to the composition of matter, including those central
to the concept of molecular identity.
2.0: Prelude to Atoms
2.1: Early Ideas in Atomic Theory
2.2: Evolution of Atomic Theory
2.3: Atomic Structure and Symbolism
2.4: Chemical Formulas
2.5: The Periodic Table
2.6: Molecular and Ionic Compounds
2.7: Chemical Nomenclature
2.E: Atoms, Molecules, and Ions (Exercises)

Thumbnail: Spinning Buckminsterfullerene (C ). (CC BY-SA 3.0; unported; Sponk).


60

This page titled 2: Atoms, Molecules, and Ions is shared under a CC BY 4.0 license and was authored, remixed, and/or curated by OpenStax via
source content that was edited to the style and standards of the LibreTexts platform; a detailed edit history is available upon request.

1
2.0: Prelude to Atoms
Your overall health and susceptibility to disease depends upon the complex interaction between your genetic makeup and
environmental exposure, with the outcome difficult to predict. Early detection of biomarkers, substances that indicate an organism’s
disease or physiological state, could allow diagnosis and treatment before a condition becomes serious or irreversible. Recent
studies have shown that your exhaled breath can contain molecules that may be biomarkers for recent exposure to environmental
contaminants or for pathological conditions ranging from asthma to lung cancer.

Figure 2.0.1 : Analysis of molecules in an exhaled breath can provide valuable information, leading to early diagnosis of diseases or
detection of environmental exposure to harmful substances. (credit: modification of work by Paul Flowers)
A person is shown blowing into a tube connected to a plastic bag. There is a computer screen displaying data that reads “Mass
Spectral Breath Analysis.” An arrow from the plastic bag points to an illustration of different molecular compounds contained in
the person’s exhalation
Scientists are working to develop biomarker “fingerprints” that could be used to diagnose a specific disease based on the amounts
and identities of certain molecules in a patient’s exhaled breath. An essential concept underlying this goal is that of a molecule’s
identity, which is determined by the numbers and types of atoms it contains, and how they are bonded together. This chapter will
describe some of the fundamental chemical principles related to the composition of matter, including those central to the concept of
molecular identity.

This page titled 2.0: Prelude to Atoms is shared under a CC BY 4.0 license and was authored, remixed, and/or curated by OpenStax via source
content that was edited to the style and standards of the LibreTexts platform; a detailed edit history is available upon request.

Access for free at OpenStax 2.0.1 https://chem.libretexts.org/@go/page/38137


2.1: Early Ideas in Atomic Theory
 Learning Objectives

By the end of this section, you will be able to:


State the postulates of Dalton’s atomic theory
Use postulates of Dalton’s atomic theory to explain the laws of definite and multiple proportions

The language used in chemistry is seen and heard in many disciplines, ranging from medicine to engineering to forensics to art.
The language of chemistry includes its own vocabulary as well as its own form of shorthand. Chemical symbols are used to
represent atoms and elements. Chemical formulas depict molecules as well as the composition of compounds. Chemical equations
provide information about the quality and quantity of the changes associated with chemical reactions.
This chapter will lay the foundation for our study of the language of chemistry. The concepts of this foundation include the atomic
theory, the composition and mass of an atom, the variability of the composition of isotopes, ion formation, chemical bonds in ionic
and covalent compounds, the types of chemical reactions, and the naming of compounds. We will also introduce one of the most
powerful tools for organizing chemical knowledge: the periodic table.

2.1.1: Atomic Theory through the Nineteenth Century


The earliest recorded discussion of the basic structure of matter comes from ancient Greek philosophers, the scientists of their day.
In the fifth century BC, Leucippus and Democritus argued that all matter was composed of small, finite particles that they called
atomos, a term derived from the Greek word for “indivisible.” They thought of atoms as moving particles that differed in shape and
size, and which could join together. Later, Aristotle and others came to the conclusion that matter consisted of various
combinations of the four “elements”—fire, earth, air, and water—and could be infinitely divided. Interestingly, these philosophers
thought about atoms and “elements” as philosophical concepts, but apparently never considered performing experiments to test
their ideas.
The Aristotelian view of the composition of matter held sway for over two thousand years, until English schoolteacher John Dalton
helped to revolutionize chemistry with his hypothesis that the behavior of matter could be explained using an atomic theory. First
published in 1807, many of Dalton’s hypotheses about the microscopic features of matter are still valid in modern atomic theory.
Here are the postulates of Dalton’s atomic theory.
1. Matter is composed of exceedingly small particles called atoms. An atom is the smallest unit of an element that can participate
in a chemical change.
2. An element consists of only one type of atom, which has a mass that is characteristic of the element and is the same for all
atoms of that element (Figure 2.1.1). A macroscopic sample of an element contains an incredibly large number of atoms, all of
which have identical chemical properties.

Figure 2.1.1 : A pre-1982 copper penny (left) contains approximately 3 × 1022 copper atoms (several dozen are represented as
brown spheres at the right), each of which has the same chemical properties. (credit: modification of work by “slgckgc”/Flickr)
The left image shows a photograph of a stack of pennies. The right image calls out an area of one of the pennies, which is made up
of many sphere-shaped copper atoms. The atoms are densely organized.
3. Atoms of one element differ in properties from atoms of all other elements.
4. A compound consists of atoms of two or more elements combined in a small, whole-number ratio. In a given compound, the
numbers of atoms of each of its elements are always present in the same ratio (Figure 2.1.2).

Access for free at OpenStax 2.1.1 https://chem.libretexts.org/@go/page/38138


Figure 2.1.2 : Copper(II) oxide, a powdery, black compound, results from the combination of two types of atoms—copper (brown
spheres) and oxygen (red spheres)—in a 1:1 ratio. (credit: modification of work by “Chemicalinterest”/Wikimedia Commons)
The left image shows a container with a black, powdery compound. The right image calls out the molecular structure of the powder
which contains copper atoms that are clustered together with an equal number of oxygen atoms.
5. Atoms are neither created nor destroyed during a chemical change, but are instead rearranged to yield substances that are
different from those present before the change (Figure 2.1.3).

Figure 2.1.3 : When the elements copper (a shiny, red-brown solid, shown here as brown spheres) and oxygen (a clear and colorless
gas, shown here as red spheres) react, their atoms rearrange to form a compound containing copper and oxygen (a powdery, black
solid). (credit copper: modification of work by Copper [images-of-elements.com]).
The left stoppered bottle contains copper and oxygen. There is a callout which shows that copper is made up of many sphere-
shaped atoms. The atoms are densely organized. The open space of the bottle contains oxygen gas, which is made up of bonded
pairs of oxygen atoms that are evenly spaced. The right stoppered bottle shows the compound copper two oxide. A callout from the
powder shows a molecule of copper two oxide, which contains copper atoms that are clustered together with an equal number of
oxygen atoms.
Dalton’s atomic theory provides a microscopic explanation of the many macroscopic properties of matter that you’ve learned about.
For example, if an element such as copper consists of only one kind of atom, then it cannot be broken down into simpler
substances, that is, into substances composed of fewer types of atoms. And if atoms are neither created nor destroyed during a
chemical change, then the total mass of matter present when matter changes from one type to another will remain constant (the law
of conservation of matter).

 Example 2.1.1: Testing Dalton’s Atomic Theory

In the following drawing, the green spheres represent atoms of a certain element. The purple spheres represent atoms of
another element. If the spheres touch, they are part of a single unit of a compound. Does the following chemical change
represented by these symbols violate any of the ideas of Dalton’s atomic theory? If so, which one?

Access for free at OpenStax 2.1.2 https://chem.libretexts.org/@go/page/38138


This equation shows that the starting materials of the reaction are two bonded, green spheres, which are being combined with
two smaller, bonded purple spheres. The product of the change is one purple sphere that is bonded to one green sphere.

Solution
The starting materials consist of two green spheres and two purple spheres. The products consist of only one green sphere and
one purple sphere. This violates Dalton’s postulate that atoms are neither created nor destroyed during a chemical change, but
are merely redistributed. (In this case, atoms appear to have been destroyed.)

 Exercise 2.1.1

In the following drawing, the green spheres represent atoms of a certain element. The purple spheres represent atoms of
another element. If the spheres touch, they are part of a single unit of a compound. Does the following chemical change
represented by these symbols violate any of the ideas of Dalton’s atomic theory? If so, which one

This equation shows that the starting materials of the reaction are two sets of bonded, green spheres which are each being
combined with two smaller, bonded purple spheres. The products of the change are two molecules that each contain one purple
sphere bonded between two green spheres.

Answer
The starting materials consist of four green spheres and two purple spheres. The products consist of four green spheres and
two purple spheres. This does not violate any of Dalton’s postulates: Atoms are neither created nor destroyed, but are
redistributed in small, whole-number ratios.

Dalton knew of the experiments of French chemist Joseph Proust, who demonstrated that all samples of a pure compound contain
the same elements in the same proportion by mass. This statement is known as the law of definite proportions or the law of constant
composition. The suggestion that the numbers of atoms of the elements in a given compound always exist in the same ratio is
consistent with these observations. For example, when different samples of isooctane (a component of gasoline and one of the
standards used in the octane rating system) are analyzed, they are found to have a carbon-to-hydrogen mass ratio of 5.33:1, as
shown in Table 2.1.1.
Table 2.1.1 : Constant Composition of Isooctane
Sample Carbon Hydrogen Mass Ratio
14.82 g carbon 5.33 g carbon
A 14.82 g 2.78 g =
2.78 g hydrogen 1.00 g hydrogen

22.33 g carbon 5.33 g carbon


B 22.33 g 4.19 g =
4.19 g hydrogen 1.00 g hydrogen

19.40 g carbon 5.33 g carbon


C 19.40 g 3.64 g =
3.63 g hydrogen 1.00 g hydrogen

It is worth noting that although all samples of a particular compound have the same mass ratio, the converse is not true in general.
That is, samples that have the same mass ratio are not necessarily the same substance. For example, there are many compounds
other than isooctane that also have a carbon-to-hydrogen mass ratio of 5.33:1.00.
Dalton also used data from Proust, as well as results from his own experiments, to formulate another interesting law. The law of
multiple proportions states that when two elements react to form more than one compound, a fixed mass of one element will react

Access for free at OpenStax 2.1.3 https://chem.libretexts.org/@go/page/38138


with masses of the other element in a ratio of small, whole numbers. For example, copper and chlorine can form a green, crystalline
solid with a mass ratio of 0.558 g chlorine to 1 g copper, as well as a brown crystalline solid with a mass ratio of 1.116 g chlorine to
1 g copper. These ratios by themselves may not seem particularly interesting or informative; however, if we take a ratio of these
ratios, we obtain a useful and possibly surprising result: a small, whole-number ratio.
1.116 g Cl

1 g Cu 2
=
0.558 g Cl 1

1 g Cu

This 2-to-1 ratio means that the brown compound has twice the amount of chlorine per amount of copper as the green compound.
This can be explained by atomic theory if the copper-to-chlorine ratio in the brown compound is 1 copper atom to 2 chlorine atoms,
and the ratio in the green compound is 1 copper atom to 1 chlorine atom. The ratio of chlorine atoms (and thus the ratio of their
masses) is therefore 2 to 1 (Figure 2.1.4).

Figure 2.1.4 : Compared to the copper chlorine compound in (a), where copper is represented by brown spheres and chlorine by
green spheres, the copper chlorine compound in (b) has twice as many chlorine atoms per copper atom. (credit a: modification of
work by “Benjah-bmm27”/Wikimedia Commons; credit b: modification of work by “Walkerma”/Wikimedia Commons)
Figure A shows a pile of green powder. A callout shows that the green powder is made up of a lattice of copper atoms interspersed
with chlorine atoms. The atoms are color coded brown for copper and green for chlorine. The number of copper atoms is equal to
the number of chlorine atoms in the molecule. Figure B shows a pile of brown powder. A callout shows that the brown powder is
also made up of copper and chlorine atoms similar to the molecule shown in figure A.

 Example 2.1.2: Laws of Definite and Multiple Proportions

A sample of compound A (a clear, colorless gas) is analyzed and found to contain 4.27 g carbon and 5.69 g oxygen. A sample
of compound B (also a clear, colorless gas) is analyzed and found to contain 5.19 g carbon and 13.84 g oxygen. Are these data
an example of the law of definite proportions, the law of multiple proportions, or neither? What do these data tell you about
substances A and B?

Solution
In compound A, the mass ratio of carbon to oxygen is:
1.33 g O

1 g C

In compound B, the mass ratio of carbon to oxygen is:


2.67 g O

1 g C

The ratio of these ratios is:

Access for free at OpenStax 2.1.4 https://chem.libretexts.org/@go/page/38138


1.33 g O

1 g C 1
=
2.67 g O 2

1 g C

This supports the law of multiple proportions. This means that A and B are different compounds, with A having one-half as
much carbon per amount of oxygen (or twice as much oxygen per amount of carbon) as B. A possible pair of compounds that
would fit this relationship would be A = CO2 and B = CO.

 Exercise 2.1.1

A sample of compound X (a clear, colorless, combustible liquid with a noticeable odor) is analyzed and found to contain 14.13
g carbon and 2.96 g hydrogen. A sample of compound Y (a clear, colorless, combustible liquid with a noticeable odor that is
slightly different from X’s odor) is analyzed and found to contain 19.91 g carbon and 3.34 g hydrogen. Are these data an
example of the law of definite proportions, the law of multiple proportions, or neither? What do these data tell you about
substances X and Y?

Answer
14.13 g C
In compound X, the mass ratio of carbon to hydrogen is .
2.96 g H

19.91 g C
In compound Y, the mass ratio of carbon to oxygen is .
3.34 g H

The ratio of these ratios is


14.13 g C

2.96 g H 4.77 g C/g H 4


= = 0.800 = .
19.91 g C 5.96 g C/g H 5

3.34 g H

This small, whole-number ratio supports the law of multiple proportions. This means that X and Y are different
compounds.

Summary
The ancient Greeks proposed that matter consists of extremely small particles called atoms. Dalton postulated that each element has
a characteristic type of atom that differs in properties from atoms of all other elements, and that atoms of different elements can
combine in fixed, small, whole-number ratios to form compounds. Samples of a particular compound all have the same elemental
proportions by mass. When two elements form different compounds, a given mass of one element will combine with masses of the
other element in a small, whole-number ratio. During any chemical change, atoms are neither created nor destroyed.

Glossary
Dalton’s atomic theory
set of postulates that established the fundamental properties of atoms

law of constant composition


(also, law of definite proportions) all samples of a pure compound contain the same elements in the same proportions by mass

law of multiple proportions


when two elements react to form more than one compound, a fixed mass of one element will react with masses of the other
element in a ratio of small whole numbers

law of definite proportions

Access for free at OpenStax 2.1.5 https://chem.libretexts.org/@go/page/38138


(also, law of constant composition) all samples of a pure compound contain the same elements in the same proportions by mass

Paul Flowers (University of North Carolina - Pembroke), Klaus Theopold (University of Delaware) and Richard Langley
(Stephen F. Austin State University) with contributing authors. Textbook content produced by OpenStax College is licensed
under a Creative Commons Attribution License 4.0 license. Download for free at http://cnx.org/contents/85abf193-
2bd...a7ac8df6@9.110).

This page titled 2.1: Early Ideas in Atomic Theory is shared under a CC BY 4.0 license and was authored, remixed, and/or curated by OpenStax
via source content that was edited to the style and standards of the LibreTexts platform; a detailed edit history is available upon request.

Access for free at OpenStax 2.1.6 https://chem.libretexts.org/@go/page/38138


2.2: Evolution of Atomic Theory
 Learning Objectives
Outline milestones in the development of modern atomic theory
Summarize and interpret the results of the experiments of Thomson, Millikan, and Rutherford
Describe the three subatomic particles that compose atoms
Introduce the term isotopes

In the two centuries since Dalton developed his ideas, scientists have made significant progress in furthering our understanding of
atomic theory. Much of this came from the results of several seminal experiments that revealed the details of the internal structure
of atoms. Here, we will discuss some of those key developments, with an emphasis on application of the scientific method, as well
as understanding how the experimental evidence was analyzed. While the historical persons and dates behind these experiments
can be quite interesting, it is most important to understand the concepts resulting from their work.

2.2.1: Atomic Theory after the Nineteenth Century


If matter were composed of atoms, what were atoms composed of? Were they the smallest particles, or was there something
smaller? In the late 1800s, a number of scientists interested in questions like these investigated the electrical discharges that could
be produced in low-pressure gases, with the most significant discovery made by English physicist J. J. Thomson using a cathode
ray tube. This apparatus consisted of a sealed glass tube from which almost all the air had been removed; the tube contained two
metal electrodes. When high voltage was applied across the electrodes, a visible beam called a cathode ray appeared between them.
This beam was deflected toward the positive charge and away from the negative charge, and was produced in the same way with
identical properties when different metals were used for the electrodes. In similar experiments, the ray was simultaneously
deflected by an applied magnetic field, and measurements of the extent of deflection and the magnetic field strength allowed
Thomson to calculate the charge-to-mass ratio of the cathode ray particles. The results of these measurements indicated that these
particles were much lighter than atoms (Figure 2.2.1).

Figure 2.2.1 : (a) J. J. Thomson produced a visible beam in a cathode ray tube. (b) This is an early cathode ray tube, invented in
1897 by Ferdinand Braun. (c) In the cathode ray, the beam (shown in yellow) comes from the cathode and is accelerated past the
anode toward a fluorescent scale at the end of the tube. Simultaneous deflections by applied electric and magnetic fields permitted
Thomson to calculate the mass-to-charge ratio of the particles composing the cathode ray. (credit a: modification of work by Nobel
Foundation; credit b: modification of work by Eugen Nesper; credit c: modification of work by “Kurzon”/Wikimedia Commons).
Based on his observations, here is what Thomson proposed and why: The particles are attracted by positive (+) charges and
repelled by negative (−) charges, so they must be negatively charged (like charges repel and unlike charges attract); they are less
massive than atoms and indistinguishable, regardless of the source material, so they must be fundamental, subatomic constituents
of all atoms. Although controversial at the time, Thomson’s idea was gradually accepted, and his cathode ray particle is what we

Access for free at OpenStax 2.2.1 https://chem.libretexts.org/@go/page/38139


now call an electron, a negatively charged, subatomic particle with a mass more than one thousand-times less that of an atom. The
term “electron” was coined in 1891 by Irish physicist George Stoney, from “electric ion.”
In 1909, more information about the electron was uncovered by American physicist Robert A. Millikan via his “oil drop”
experiments. Millikan created microscopic oil droplets, which could be electrically charged by friction as they formed or by using
X-rays. These droplets initially fell due to gravity, but their downward progress could be slowed or even reversed by an electric
field lower in the apparatus. By adjusting the electric field strength and making careful measurements and appropriate calculations,
Millikan was able to determine the charge on individual drops (Figure 2.2.2).

Figure 2.2.2 : Millikan’s experiment measured the charge of individual oil drops. The tabulated data are examples of a few possible
values.
Looking at the charge data that Millikan gathered, you may have recognized that the charge of an oil droplet is always a multiple of
a specific charge, 1.6 × 10−19 C. Millikan concluded that this value must therefore be a fundamental charge—the charge of a single
electron—with his measured charges due to an excess of one electron (1 times 1.6 × 10−19 C), two electrons (2 times 1.6 × 10−19
C), three electrons (3 times 1.6 × 10−19 C), and so on, on a given oil droplet. Since the charge of an electron was now known due
to Millikan’s research, and the charge-to-mass ratio was already known due to Thomson’s research (1.759 × 1011 C/kg), it only
required a simple calculation to determine the mass of the electron as well.

−19
1 kg −31
Mass of electron = 1.602 × 10 C × = 9.107 × 10 kg (2.3.1)
11
1.759 × 10 C

Scientists had now established that the atom was not indivisible as Dalton had believed, and due to the work of Thomson, Millikan,
and others, the charge and mass of the negative, subatomic particles—the electrons—were known. However, the positively charged
part of an atom was not yet well understood. In 1904, Thomson proposed the “plum pudding” model of atoms, which described a
positively charged mass with an equal amount of negative charge in the form of electrons embedded in it, since all atoms are
electrically neutral. A competing model had been proposed in 1903 by Hantaro Nagaoka, who postulated a Saturn-like atom,
consisting of a positively charged sphere surrounded by a halo of electrons (Figure 2.2.3).

Figure 2.2.3 : (a) Thomson suggested that atoms resembled plum pudding, an English dessert consisting of moist cake with
embedded raisins (“plums”). (b) Nagaoka proposed that atoms resembled the planet Saturn, with a ring of electrons surrounding a
positive “planet.” (credit a: modification of work by “Man vyi”/Wikimedia Commons; credit b: modification of work by
“NASA”/Wikimedia Commons).

Access for free at OpenStax 2.2.2 https://chem.libretexts.org/@go/page/38139


The next major development in understanding the atom came from Ernest Rutherford, a physicist from New Zealand who largely
spent his scientific career in Canada and England. He performed a series of experiments using a beam of high-speed, positively
charged alpha particles (α particles) that were produced by the radioactive decay of radium; α particles consist of two protons and
two neutrons (you will learn more about radioactive decay in the chapter on nuclear chemistry). Rutherford and his colleagues
Hans Geiger (later famous for the Geiger counter) and Ernest Marsden aimed a beam of α particles, the source of which was
embedded in a lead block to absorb most of the radiation, at a very thin piece of gold foil and examined the resultant scattering of
the α particles using a luminescent screen that glowed briefly where hit by an α particle.
What did they discover? Most particles passed right through the foil without being deflected at all. However, some were diverted
slightly, and a very small number were deflected almost straight back toward the source (Figure 2.2.4). Rutherford described
finding these results: “It was quite the most incredible event that has ever happened to me in my life. It was almost as incredible as
if you fired a 15-inch shell at a piece of tissue paper and it came back and hit you”1 (p. 68).

Figure 2.2.4 : Geiger and Rutherford fired α particles at a piece of gold foil and detected where those particles went, as shown in
this schematic diagram of their experiment. Most of the particles passed straight through the foil, but a few were deflected slightly
and a very small number were significantly deflected.
Here is what Rutherford deduced: Because most of the fast-moving α particles passed through the gold atoms undeflected, they
must have traveled through essentially empty space inside the atom. Alpha particles are positively charged, so deflections arose
when they encountered another positive charge (like charges repel each other). Since like charges repel one another, the few
positively charged α particles that changed paths abruptly must have hit, or closely approached, another body that also had a highly
concentrated, positive charge. Since the deflections occurred a small fraction of the time, this charge only occupied a small amount
of the space in the gold foil. Analyzing a series of such experiments in detail, Rutherford drew two conclusions:
1. The volume occupied by an atom must consist of a large amount of empty space.
2. A small, relatively heavy, positively charged body, the nucleus, must be at the center of each atom.
This analysis led Rutherford to propose a model in which an atom consists of a very small, positively charged nucleus, in which
most of the mass of the atom is concentrated, surrounded by the negatively charged electrons, so that the atom is electrically neutral
(Figure 2.2.5).

Access for free at OpenStax 2.2.3 https://chem.libretexts.org/@go/page/38139


Figure 2.2.5 : The α particles are deflected only when they collide with or pass close to the much heavier, positively charged gold
nucleus. Because the nucleus is very small compared to the size of an atom, very few α particles are deflected. Most pass through
the relatively large region occupied by electrons, which are too light to deflect the rapidly moving particles.
After many more experiments, Rutherford also discovered that the nuclei of other elements contain the hydrogen nucleus as a
“building block,” and he named this more fundamental particle the proton, the positively charged, subatomic particle found in the
nucleus. With one addition, which you will learn next, this nuclear model of the atom, proposed over a century ago, is still used
today.
PhET: Rutherford Scattering
Another important finding was the discovery of isotopes. During the early 1900s, scientists identified several substances that
appeared to be new elements, isolating them from radioactive ores. For example, a “new element” produced by the radioactive
decay of thorium was initially given the name mesothorium. However, a more detailed analysis showed that mesothorium was
chemically identical to radium (another decay product), despite having a different atomic mass. This result, along with similar
findings for other elements, led the English chemist Frederick Soddy to realize that an element could have types of atoms with
different masses that were chemically indistinguishable. These different types are called isotopes—atoms of the same element that
differ in mass. Soddy was awarded the Nobel Prize in Chemistry in 1921 for this discovery.
One puzzle remained: The nucleus was known to contain almost all of the mass of an atom, with the number of protons only
providing half, or less, of that mass. Different proposals were made to explain what constituted the remaining mass, including the
existence of neutral particles in the nucleus. As you might expect, detecting uncharged particles is very challenging, and it was not
until 1932 that James Chadwick found evidence of neutrons, uncharged, subatomic particles with a mass approximately the same as
that of protons. The existence of the neutron also explained isotopes: They differ in mass because they have different numbers of
neutrons, but they are chemically identical because they have the same number of protons. This will be explained in more detail
later in this chapter.

Summary
Although no one has actually seen the inside of an atom, experiments have demonstrated much about atomic structure. Thomson’s
cathode ray tube showed that atoms contain small, negatively charged particles called electrons. Millikan discovered that there is a
fundamental electric charge—the charge of an electron. Rutherford’s gold foil experiment showed that atoms have a small, dense,
positively charged nucleus; the positively charged particles within the nucleus are called protons. Chadwick discovered that the
nucleus also contains neutral particles called neutrons. Soddy demonstrated that atoms of the same element can differ in mass;
these are called isotopes.

Access for free at OpenStax 2.2.4 https://chem.libretexts.org/@go/page/38139


Footnotes
1. Ernest Rutherford, “The Development of the Theory of Atomic Structure,” ed. J. A. Ratcliffe, in Background to Modern
Science, eds. Joseph Needham and Walter Pagel, (Cambridge, UK: Cambridge University Press, 1938), 61–74. Accessed
September 22, 2014, https://ia600508.us.archive.org/3/it...e032734mbp.pdf.

Glossary
alpha particle (α particle)
positively charged particle consisting of two protons and two neutrons

electron
negatively charged, subatomic particle of relatively low mass located outside the nucleus

isotopes
atoms that contain the same number of protons but different numbers of neutrons

neutron
uncharged, subatomic particle located in the nucleus

proton
positively charged, subatomic particle located in the nucleus

nucleus
massive, positively charged center of an atom made up of protons and neutrons

This page titled 2.2: Evolution of Atomic Theory is shared under a CC BY 4.0 license and was authored, remixed, and/or curated by OpenStax via
source content that was edited to the style and standards of the LibreTexts platform; a detailed edit history is available upon request.

Access for free at OpenStax 2.2.5 https://chem.libretexts.org/@go/page/38139


2.3: Atomic Structure and Symbolism
 Learning Objectives
Write and interpret symbols that depict the atomic number, mass number, and charge of an atom or ion
Define the atomic mass unit and average atomic mass
Calculate average atomic mass and isotopic abundance

The development of modern atomic theory revealed much about the inner structure of atoms. It was learned that an atom contains a
very small nucleus composed of positively charged protons and uncharged neutrons, surrounded by a much larger volume of space
containing negatively charged electrons. The nucleus contains the majority of an atom’s mass because protons and neutrons are
much heavier than electrons, whereas electrons occupy almost all of an atom’s volume. The diameter of an atom is on the order of
10−10 m, whereas the diameter of the nucleus is roughly 10−15 m—about 100,000 times smaller. For a perspective about their
relative sizes, consider this: If the nucleus were the size of a blueberry, the atom would be about the size of a football stadium
(Figure 2.3.1).

Figure 2.3.1 : If an atom could be expanded to the size of a football stadium, the nucleus would be the size of a single blueberry.
(credit middle: modification of work by “babyknight”/Wikimedia Commons; credit right: modification of work by Paxson
Woelber).
The diagram on the left shows a picture of an atom with a nucleus. The central figure shows a photograph of an American football
stadium. The figure on the right shows a photograph of a handful of blueberries.
Atoms—and the protons, neutrons, and electrons that compose them—are extremely small. For example, a carbon atom weighs
less than 2 × 10−23 g, and an electron has a charge of less than 2 × 10−19 C (coulomb). When describing the properties of tiny
objects such as atoms, we use appropriately small units of measure, such as the atomic mass unit (amu) and the fundamental unit of
charge (e). The amu was originally defined based on hydrogen, the lightest element, then later in terms of oxygen. Since 1961, it
has been defined with regard to the most abundant isotope of carbon, atoms of which are assigned masses of exactly 12 amu. (This
isotope is known as “carbon-12” as will be discussed later in this module.) Thus, one amu is exactly 1/12 of the mass of one
carbon-12 atom: 1 amu = 1.6605 × 10−24 g. (The Dalton (Da) and the unified atomic mass unit (u) are alternative units that are
equivalent to the amu.) The fundamental unit of charge (also called the elementary charge) equals the magnitude of the charge of an
electron (e) with e = 1.602 × 10−19 C.
A proton has a mass of 1.0073 amu and a charge of 1+. A neutron is a slightly heavier particle with a mass 1.0087 amu and a
charge of zero; as its name suggests, it is neutral. The electron has a charge of 1− and is a much lighter particle with a mass of
about 0.00055 amu (it would take about 1800 electrons to equal the mass of one proton. The properties of these fundamental
particles are summarized in Table 2.3.1. (An observant student might notice that the sum of an atom’s subatomic particles does not
equal the atom’s actual mass: The total mass of six protons, six neutrons, and six electrons is 12.0993 amu, slightly larger than the
12.00 amu of an actual carbon-12 atom. This “missing” mass is known as the mass defect, and you will learn about it in the chapter
on nuclear chemistry.)
Table 2.3.1 : Properties of Subatomic Particles
Name Location Charge (C) Unit Charge Mass (amu) Mass (g)

electron outside nucleus −1.602 × 10


−19
1− 0.00055 0.00091 × 10
−24

proton nucleus 1.602 × 10


−19
1+ 1.00727 1.67262 × 10
−24

Access for free at OpenStax 2.3.1 https://chem.libretexts.org/@go/page/38140


Name Location Charge (C) Unit Charge Mass (amu) Mass (g)

neutron nucleus 0 0 1.00866 1.67493 × 10


−24

The number of protons in the nucleus of an atom is its atomic number (Z ). This is the defining trait of an element: Its value
determines the identity of the atom. For example, any atom that contains six protons is the element carbon and has the atomic
number 6, regardless of how many neutrons or electrons it may have. A neutral atom must contain the same number of positive and
negative charges, so the number of protons equals the number of electrons. Therefore, the atomic number also indicates the number
of electrons in an atom. The total number of protons and neutrons in an atom is called its mass number (A). The number of
neutrons is therefore the difference between the mass number and the atomic number: A – Z = number of neutrons.
atomic number (Z) = number of protons

mass number (A) = number of protons + number of neutrons

A−Z = number of neutrons

Atoms are electrically neutral if they contain the same number of positively charged protons and negatively charged electrons.
When the numbers of these subatomic particles are not equal, the atom is electrically charged and is called an ion. The charge of an
atom is defined as follows:
Atomic charge = number of protons − number of electrons
As will be discussed in more detail later in this chapter, atoms (and molecules) typically acquire charge by gaining or losing
electrons. An atom that gains one or more electrons will exhibit a negative charge and is called an anion. Positively charged atoms
called cations are formed when an atom loses one or more electrons. For example, a neutral sodium atom (Z = 11) has 11 electrons.
If this atom loses one electron, it will become a cation with a 1+ charge (11 − 10 = 1+). A neutral oxygen atom (Z = 8) has eight
electrons, and if it gains two electrons it will become an anion with a 2− charge (8 − 10 = 2−).

 Example 2.3.1: Composition of an Atom

Iodine is an essential trace element in our diet; it is needed to produce thyroid hormone. Insufficient iodine in the diet can lead
to the development of a goiter, an enlargement of the thyroid gland (Figure 2.3.2).

Figure 2.3.2 : (a) Insufficient iodine in the diet can cause an enlargement of the thyroid gland called a goiter. (b) The addition
of small amounts of iodine to salt, which prevents the formation of goiters, has helped eliminate this concern in the US where
salt consumption is high. (credit a: modification of work by “Almazi”/Wikimedia Commons; credit b: modification of work by
Mike Mozart)
Figure A shows a photo of a person who has a very swollen thyroid in his or her neck. Figure B shows a photo of a canister of
Morton iodized salt.
The addition of small amounts of iodine to table salt (iodized salt) has essentially eliminated this health concern in the United
States, but as much as 40% of the world’s population is still at risk of iodine deficiency. The iodine atoms are added as anions,
and each has a 1− charge and a mass number of 127. Determine the numbers of protons, neutrons, and electrons in one of these
iodine anions.

Access for free at OpenStax 2.3.2 https://chem.libretexts.org/@go/page/38140


Solution
The atomic number of iodine (53) tells us that a neutral iodine atom contains 53 protons in its nucleus and 53 electrons outside
its nucleus. Because the sum of the numbers of protons and neutrons equals the mass number, 127, the number of neutrons is
74 (127 − 53 = 74). Since the iodine is added as a 1− anion, the number of electrons is 54 [53 – (1–) = 54].

 Exercise 2.3.1

An ion of platinum has a mass number of 195 and contains 74 electrons. How many protons and neutrons does it contain, and
what is its charge?

Answer
78 protons; 117 neutrons; charge is 4+

2.3.1: Chemical Symbols


A chemical symbol is an abbreviation that we use to indicate an element or an atom of an element. For example, the symbol for
mercury is Hg (Figure 2.3.3). We use the same symbol to indicate one atom of mercury (microscopic domain) or to label a
container of many atoms of the element mercury (macroscopic domain).

Figure 2.3.3 : The symbol Hg represents the element mercury regardless of the amount; it could represent one atom of mercury or a
large amount of mercury. from Wikipedia (user: Materialscientist).
The symbols for several common elements and their atoms are listed in Table 2.3.2. Some symbols are derived from the common
name of the element; others are abbreviations of the name in another language. Symbols have one or two letters, for example, H for
hydrogen and Cl for chlorine. To avoid confusion with other notations, only the first letter of a symbol is capitalized. For example,
Co is the symbol for the element cobalt, but CO is the notation for the compound carbon monoxide, which contains atoms of the
elements carbon (C) and oxygen (O). All known elements and their symbols are in the periodic table.
Table 2.3.2 : Some Common Elements and Their Symbols
Element Symbol Element Symbol

aluminum Al iron Fe (from ferrum)

bromine Br lead Pb (from plumbum)

calcium Ca magnesium Mg

carbon C mercury Hg (from hydrargyrum)

chlorine Cl nitrogen N

chromium Cr oxygen O

cobalt Co potassium K (from kalium)

copper Cu (from cuprum) silicon Si

fluorine F silver Ag (from argentum)

Access for free at OpenStax 2.3.3 https://chem.libretexts.org/@go/page/38140


Element Symbol Element Symbol

gold Au (from aurum) sodium Na (from natrium)

helium He sulfur S

hydrogen H tin Sn (from stannum)

iodine I zinc Zn

Traditionally, the discoverer (or discoverers) of a new element names the element. However, until the name is recognized by the
International Union of Pure and Applied Chemistry (IUPAC), the recommended name of the new element is based on the Latin
word(s) for its atomic number. For example, element 106 was called unnilhexium (Unh), element 107 was called unnilseptium
(Uns), and element 108 was called unniloctium (Uno) for several years. These elements are now named after scientists or locations;
for example, element 106 is now known as seaborgium (Sg) in honor of Glenn Seaborg, a Nobel Prize winner who was active in
the discovery of several heavy elements.

2.3.2: Isotopes
The symbol for a specific isotope of any element is written by placing the mass number as a superscript to the left of the element
symbol (Figure 2.3.4). The atomic number is sometimes written as a subscript preceding the symbol, but since this number defines
the element’s identity, as does its symbol, it is often omitted. For example, magnesium exists as a mixture of three isotopes, each
with an atomic number of 12 and with mass numbers of 24, 25, and 26, respectively. These isotopes can be identified as 24Mg,
25
Mg, and 26Mg. These isotope symbols are read as “element, mass number” and can be symbolized consistent with this reading.
For instance, 24Mg is read as “magnesium 24,” and can be written as “magnesium-24” or “Mg-24.” 25Mg is read as “magnesium
25,” and can be written as “magnesium-25” or “Mg-25.” All magnesium atoms have 12 protons in their nucleus. They differ only
because a 24Mg atom has 12 neutrons in its nucleus, a 25Mg atom has 13 neutrons, and a 26Mg has 14 neutrons.

Figure 2.3.4 : The symbol for an atom indicates the element via its usual two-letter symbol, the mass number as a left superscript,
the atomic number as a left subscript (sometimes omitted), and the charge as a right superscript.
This diagram shows the symbol for helium, “H e.” The number to the upper left of the symbol is the mass number, which is 4. The
number to the upper right of the symbol is the charge which is positive 2. The number to the lower left of the symbol is the atomic
number, which is 2. This number is often omitted. Also shown is “M g” which stands for magnesium It has a mass number of 24, a
charge of positive 2, and an atomic number of 12.
Information about the naturally occurring isotopes of elements with atomic numbers 1 through 10 is given in Table 2.3.2. Note that
in addition to standard names and symbols, the isotopes of hydrogen are often referred to using common names and accompanying
symbols. Hydrogen-2, symbolized 2H, is also called deuterium and sometimes symbolized D. Hydrogen-3, symbolized 3H, is also
called tritium and sometimes symbolized T.
Table 2.3.2 : Nuclear Compositions of Atoms of the Very Light Elements
Number of Number of % Natural
Element Symbol Atomic Number Mass (amu)
Protons Neutrons Abundance
1
H
1
1 1 0 1.0078 99.989
(protium)
2
H
hydrogen 1
1 1 1 2.0141 0.0115
(deuterium)
3
H
1
1 1 2 3.01605 — (trace)
(tritium)
3
2
He 2 2 1 3.01603 0.00013
helium
4
2
He 2 2 2 4.0026 100

Access for free at OpenStax 2.3.4 https://chem.libretexts.org/@go/page/38140


Number of Number of % Natural
Element Symbol Atomic Number Mass (amu)
Protons Neutrons Abundance
6
3
Li 3 3 3 6.0151 7.59
lithium
7
3
Li 3 3 4 7.0160 92.41

beryllium 9
4
Be 4 4 5 9.0122 100
10
5
B 5 5 5 10.0129 19.9
boron
11
5
B 5 5 6 11.0093 80.1
12
6
C 6 6 6 12.0000 98.89

carbon 13
6
C 6 6 7 13.0034 1.11
14
6
C 6 6 8 14.0032 — (trace)
14
7
N 7 7 7 14.0031 99.63
nitrogen
15
7
N 7 7 8 15.0001 0.37
16
8
O 8 8 8 15.9949 99.757

oxygen 17
8
O 8 8 9 16.9991 0.038
18
8
O 8 8 10 17.9992 0.205

fluorine 19
9
F 9 9 10 18.9984 100
20
10
Ne 10 10 10 19.9924 90.48

neon 21
10
Ne 10 10 11 20.9938 0.27
22
10
Ne 10 10 12 21.9914 9.25

2.3.3: Atomic Mass


Because each proton and each neutron contribute approximately one amu to the mass of an atom, and each electron contributes far
less, the atomic mass of a single atom is approximately equal to its mass number (a whole number). However, the average masses
of atoms of most elements are not whole numbers because most elements exist naturally as mixtures of two or more isotopes.
The mass of an element shown in a periodic table or listed in a table of atomic masses is a weighted, average mass of all the
isotopes present in a naturally occurring sample of that element. This is equal to the sum of each individual isotope’s mass
multiplied by its fractional abundance.

average mass = ∑(f ractional abundance × isotopic mass)i

For example, the element boron is composed of two isotopes: About 19.9% of all boron atoms are 10B with a mass of 10.0129 amu,
and the remaining 80.1% are 11B with a mass of 11.0093 amu. The average atomic mass for boron is calculated to be:

boron average mass = (0.199 × 10.0129 amu) + (0.801 × 11.0093 amu)

= 1.99 amu + 8.82 amu

= 10.81 amu

It is important to understand that no single boron atom weighs exactly 10.8 amu; 10.8 amu is the average mass of all boron atoms,
and individual boron atoms weigh either approximately 10 amu or 11 amu.

 Example 2.3.2: Calculation of Average Atomic Mass

A meteorite found in central Indiana contains traces of the noble gas neon picked up from the solar wind during the meteorite’s
trip through the solar system. Analysis of a sample of the gas showed that it consisted of 91.84% 20Ne (mass 19.9924 amu),
0.47% 21Ne (mass 20.9940 amu), and 7.69% 22Ne (mass 21.9914 amu). What is the average mass of the neon in the solar
wind?

Access for free at OpenStax 2.3.5 https://chem.libretexts.org/@go/page/38140


Solution
average mass = (0.9184 × 19.9924 amu) + (0.0047 × 20.9940 amu) + (0.0769 × 21.9914 amu)

= (18.36 + 0.099 + 1.69) amu

= 20.15 amu

The average mass of a neon atom in the solar wind is 20.15 amu. (The average mass of a terrestrial neon atom is 20.1796 amu.
This result demonstrates that we may find slight differences in the natural abundance of isotopes, depending on their origin.)

 Exercise 2.3.2

A sample of magnesium is found to contain 78.70% of 24Mg atoms (mass 23.98 amu), 10.13% of 25
Mg atoms (mass 24.99
amu), and 11.17% of 26Mg atoms (mass 25.98 amu). Calculate the average mass of a Mg atom.

Answer
24.31 amu

We can also do variations of this type of calculation, as shown in the next example.

 Example 2.3.3: Calculation of Percent Abundance

Naturally occurring chlorine consists of 35Cl (mass 34.96885 amu) and 37Cl (mass 36.96590 amu), with an average mass of
35.453 amu. What is the percent composition of Cl in terms of these two isotopes?

Solution
The average mass of chlorine is the fraction that is 35Cl times the mass of 35Cl plus the fraction that is 37Cl times the mass of
37Cl.

35 35 37 37
average mass = (f raction of Cl × mass of Cl) + (f raction of Cl × mass of Cl)

If we let x represent the fraction that is 35Cl, then the fraction that is 37Cl is represented by 1.00 − x.
(The fraction that is 35Cl + the fraction that is 37Cl must add up to 1, so the fraction of 37Cl must equal 1.00 − the fraction of
35Cl.)

Substituting this into the average mass equation, we have:


35.453 amu = (x × 34.96885 amu) + [(1.00 − x) × 36.96590 amu]

35.453 = 34.96885x + 36.96590 − 36.96590x

1.99705x = 1.513

1.513
x = = 0.7576
1.99705

35
So solving yields: x = 0.7576, which means that 1.00 − 0.7576 = 0.2424. Therefore, chlorine consists of 75.76% Cl and
24.24% 37Cl.

 Exercise 2.3.3
Naturally occurring copper consists of 63Cu (mass 62.9296 amu) and 65Cu (mass 64.9278 amu), with an average mass of
63.546 amu. What is the percent composition of Cu in terms of these two isotopes?

Answer
69.15% Cu-63 and 30.85% Cu-65

Access for free at OpenStax 2.3.6 https://chem.libretexts.org/@go/page/38140


Figure 2.3.5 : Analysis of zirconium in a mass spectrometer produces a mass spectrum with peaks showing the different isotopes of
Zr.
The left diagram shows how a mass spectrometer works. The graph to the right of the spectrometer shows a mass spectrum of
zirconium. The relative abundance, as a percentage from 0 to 100, is graphed on the y axis, and the mass to charge ratio is graphed
on the x axis. The sample contains five different isomers of zirconium.

The occurrence and natural abundances of isotopes can be experimentally determined using an instrument called a mass
spectrometer. Mass spectrometry (MS) is widely used in chemistry, forensics, medicine, environmental science, and many other
fields to analyze and help identify the substances in a sample of material. In a typical mass spectrometer (Figure 2.3.5), the sample
is vaporized and exposed to a high-energy electron beam that causes the sample’s atoms (or molecules) to become electrically
charged, typically by losing one or more electrons. These cations then pass through a (variable) electric or magnetic field that
deflects each cation’s path to an extent that depends on both its mass and charge (similar to how the path of a large steel ball
bearing rolling past a magnet is deflected to a lesser extent that that of a small steel BB). The ions are detected, and a plot of the
relative number of ions generated versus their mass-to-charge ratios (a mass spectrum) is made. The height of each vertical feature
or peak in a mass spectrum is proportional to the fraction of cations with the specified mass-to-charge ratio. Since its initial use
during the development of modern atomic theory, MS has evolved to become a powerful tool for chemical analysis in a wide range
of applications.

Mass Spectrometry MS

Video 2.3.1 : Watch this video from the Royal Society for Chemistry for a brief description of the rudiments of mass spectrometry.

Summary
An atom consists of a small, positively charged nucleus surrounded by electrons. The nucleus contains protons and neutrons; its
diameter is about 100,000 times smaller than that of the atom. The mass of one atom is usually expressed in atomic mass units
(amu), which is referred to as the atomic mass. An amu is defined as exactly 1/12 of the mass of a carbon-12 atom and is equal to
1.6605 × 10−24 g.

Access for free at OpenStax 2.3.7 https://chem.libretexts.org/@go/page/38140


Protons are relatively heavy particles with a charge of 1+ and a mass of 1.0073 amu. Neutrons are relatively heavy particles with no
charge and a mass of 1.0087 amu. Electrons are light particles with a charge of 1− and a mass of 0.00055 amu. The number of
protons in the nucleus is called the atomic number (Z) and is the property that defines an atom’s elemental identity. The sum of the
numbers of protons and neutrons in the nucleus is called the mass number and, expressed in amu, is approximately equal to the
mass of the atom. An atom is neutral when it contains equal numbers of electrons and protons.
Isotopes of an element are atoms with the same atomic number but different mass numbers; isotopes of an element, therefore, differ
from each other only in the number of neutrons within the nucleus. When a naturally occurring element is composed of several
isotopes, the atomic mass of the element represents the average of the masses of the isotopes involved. A chemical symbol
identifies the atoms in a substance using symbols, which are one-, two-, or three-letter abbreviations for the atoms.

2.3.4: Key Equations


average mass = ∑ (f ractional abundance × isotopic mass)i
i

Glossary
anion
negatively charged atom or molecule (contains more electrons than protons)

atomic mass
average mass of atoms of an element, expressed in amu

atomic mass unit (amu)


1
(also, unified atomic mass unit, u, or Dalton, Da) unit of mass equal to of the mass of a 12C atom
12

atomic number (Z)


number of protons in the nucleus of an atom

cation
positively charged atom or molecule (contains fewer electrons than protons)

chemical symbol
one-, two-, or three-letter abbreviation used to represent an element or its atoms

Dalton (Da)
alternative unit equivalent to the atomic mass unit

fundamental unit of charge


(also called the elementary charge) equals the magnitude of the charge of an electron (e) with e = 1.602 × 10−19 C

ion
electrically charged atom or molecule (contains unequal numbers of protons and electrons)

mass number (A)


sum of the numbers of neutrons and protons in the nucleus of an atom
unified atomic mass unit (u)
alternative unit equivalent to the atomic mass unit

Paul Flowers (University of North Carolina - Pembroke), Klaus Theopold (University of Delaware) and Richard Langley
(Stephen F. Austin State University) with contributing authors. Textbook content produced by OpenStax College is licensed
under a Creative Commons Attribution License 4.0 license. Download for free at http://cnx.org/contents/85abf193-
2bd...a7ac8df6@9.110).

Access for free at OpenStax 2.3.8 https://chem.libretexts.org/@go/page/38140


This page titled 2.3: Atomic Structure and Symbolism is shared under a CC BY 4.0 license and was authored, remixed, and/or curated by
OpenStax via source content that was edited to the style and standards of the LibreTexts platform; a detailed edit history is available upon request.

Access for free at OpenStax 2.3.9 https://chem.libretexts.org/@go/page/38140


2.4: Chemical Formulas
 Learning Objectives
Symbolize the composition of molecules using molecular formulas and empirical formulas
Represent the bonding arrangement of atoms within molecules using structural formulas

A molecular formula is a representation of a molecule that uses chemical symbols to indicate the types of atoms followed by
subscripts to show the number of atoms of each type in the molecule. (A subscript is used only when more than one atom of a given
type is present.) Molecular formulas are also used as abbreviations for the names of compounds.
The structural formula for a compound gives the same information as its molecular formula (the types and numbers of atoms in the
molecule) but also shows how the atoms are connected in the molecule. The structural formula for methane contains symbols for
one C atom and four H atoms, indicating the number of atoms in the molecule (Figure 2.4.1). The lines represent bonds that hold
the atoms together. (A chemical bond is an attraction between atoms or ions that holds them together in a molecule or a crystal.) We
will discuss chemical bonds and see how to predict the arrangement of atoms in a molecule later. For now, simply know that the
lines are an indication of how the atoms are connected in a molecule. A ball-and-stick model shows the geometric arrangement of
the atoms with atomic sizes not to scale, and a space-filling model shows the relative sizes of the atoms.

Figure 2.4.1 : A methane molecule can be represented as (a) a molecular formula, (b) a structural formula, (c) a ball-and-stick
model, and (d) a space-filling model. Carbon and hydrogen atoms are represented by black and white spheres, respectively.
Figure A shows C H subscript 4. Figure B shows a carbon atom that is bonded to four hydrogen atoms at right angles: one above,
one to the left, one to the right, and one below. Figure C shows a 3-D, ball-and-stick model of the carbon atom bonded to four
hydrogen atoms. Figure D shows a space-filling model of a carbon atom with hydrogen atoms partially embedded into the surface
of the carbon atom.
Although many elements consist of discrete, individual atoms, some exist as molecules made up of two or more atoms of the
element chemically bonded together. For example, most samples of the elements hydrogen, oxygen, and nitrogen are composed of
molecules that contain two atoms each (called diatomic molecules) and thus have the molecular formulas H2, O2, and N2,
respectively. Other elements commonly found as diatomic molecules are fluorine (F2), chlorine (Cl2), bromine (Br2), and iodine
(I2). The most common form of the element sulfur is composed of molecules that consist of eight atoms of sulfur; its molecular
formula is S8 (Figure 2.4.2).

Figure 2.4.2 : A molecule of sulfur is composed of eight sulfur atoms and is therefore written as S8. It can be represented as (a) a
structural formula, (b) a ball-and-stick model, and (c) a space-filling model. Sulfur atoms are represented by yellow spheres.
Figure A shows eight sulfur atoms, symbolized with the letter S, that are bonded to each other to form an octagon. Figure B shows
a 3-D, ball-and-stick model of the arrangement of the sulfur atoms. The shape is clearly not octagonal as it is represented in the
structural formula. Figure C is a space-filling model that shows each sulfur atom is partially embedded into the sulfur atom it bonds
with.
It is important to note that a subscript following a symbol and a number in front of a symbol do not represent the same thing; for
example, H2 and 2H represent distinctly different species. H2 is a molecular formula; it represents a diatomic molecule of
hydrogen, consisting of two atoms of the element that are chemically bonded together. The expression 2H, on the other hand,

Access for free at OpenStax 2.4.1 https://chem.libretexts.org/@go/page/38141


indicates two separate hydrogen atoms that are not combined as a unit. The expression 2H2 represents two molecules of diatomic
hydrogen (Figure 2.4.3).

Figure 2.4.3 : The symbols H, 2H, H2, and 2H2 represent very different entities.
This figure shows four diagrams. The diagram for H shows a single, white sphere and is labeled one H atom. The diagram for 2 H
shows two white spheres that are not bonded together. It is labeled 2 H atoms. The diagram for H subscript 2 shows two white
spheres bonded together. It is labeled one H subscript 2 molecule. The diagram for 2 H subscript 2 shows two sets of bonded, white
spheres. It is labeled 2 H subscript 2 molecules.
Compounds are formed when two or more elements chemically combine, resulting in the formation of bonds. For example,
hydrogen and oxygen can react to form water, and sodium and chlorine can react to form table salt. We sometimes describe the
composition of these compounds with an empirical formula, which indicates the types of atoms present and the simplest whole-
number ratio of the number of atoms (or ions) in the compound. For example, titanium dioxide (used as pigment in white paint and
in the thick, white, blocking type of sunscreen) has an empirical formula of TiO2. This identifies the elements titanium (Ti) and
oxygen (O) as the constituents of titanium dioxide, and indicates the presence of twice as many atoms of the element oxygen as
atoms of the element titanium (Figure 2.4.4).

Figure 2.4.4 : (a) The white compound titanium dioxide provides effective protection from the sun. (b) A crystal of titanium
dioxide, TiO2, contains titanium and oxygen in a ratio of 1 to 2. The titanium atoms are gray and the oxygen atoms are red. (credit
a: modification of work by “osseous”/Flickr).
Figure A shows a photo of a person applying suntan lotion to his or her lower leg. Figure B shows a 3-D ball-and-stick model of
the molecule titanium dioxide, which involves a complicated interlocking of many titanium and oxygen atoms. The titanium atoms
in the molecule are shown as silver spheres and the oxygen atoms are shown as red spheres. There are twice as many oxygen atoms
as titanium atoms in the molecule.
As discussed previously, we can describe a compound with a molecular formula, in which the subscripts indicate the actual
numbers of atoms of each element in a molecule of the compound. In many cases, the molecular formula of a substance is derived
from experimental determination of both its empirical formula and its molecular mass (the sum of atomic masses for all atoms
composing the molecule). For example, it can be determined experimentally that benzene contains two elements, carbon (C) and
hydrogen (H), and that for every carbon atom in benzene, there is one hydrogen atom. Thus, the empirical formula is CH. An
experimental determination of the molecular mass reveals that a molecule of benzene contains six carbon atoms and six hydrogen
atoms, so the molecular formula for benzene is C6H6 (Figure 2.4.5).

Access for free at OpenStax 2.4.2 https://chem.libretexts.org/@go/page/38141


Figure 2.4.5 : Benzene, C6H6, is produced during oil refining and has many industrial uses. A benzene molecule can be represented
as (a) a structural formula, (b) a ball-and-stick model, and (c) a space-filling model. (d) Benzene is a clear liquid. (credit d:
modification of work by Sahar Atwa).
Figure A shows that benzene is composed of six carbons shaped like a hexagon. Every other bond between the carbon atoms is a
double bond. Each carbon also has a single bonded hydrogen atom. Figure B shows a 3-D, ball-and-stick drawing of benzene. The
six carbon atoms are black spheres while the six hydrogen atoms are smaller, white spheres. Figure C is a space-filling model of
benzene which shows that most of the interior space is occupied by the carbon atoms. The hydrogen atoms are embedded in the
outside surface of the carbon atoms. Figure d shows a small vial filled with benzene which appears to be clear.
If we know a compound’s formula, we can easily determine the empirical formula. (This is somewhat of an academic exercise; the
reverse chronology is generally followed in actual practice.) For example, the molecular formula for acetic acid, the component
that gives vinegar its sharp taste, is C2H4O2. This formula indicates that a molecule of acetic acid (Figure 2.4.6) contains two
carbon atoms, four hydrogen atoms, and two oxygen atoms. The ratio of atoms is 2:4:2. Dividing by the lowest common
denominator (2) gives the simplest, whole-number ratio of atoms, 1:2:1, so the empirical formula is CH2O. Note that a molecular
formula is always a whole-number multiple of an empirical formula.

Figure 2.4.6 : (a) Vinegar contains acetic acid, C2H4O2, which has an empirical formula of CH2O. It can be represented as (b) a
structural formula and (c) as a ball-and-stick model. (credit a: modification of work by “HomeSpot HQ”/Flickr)
Figure A shows a jug of distilled, white vinegar. Figure B shows a structural formula for acetic acid which contains two carbon
atoms connected by a single bond. The left carbon atom forms single bonds with three hydrogen atoms. The right carbon atom
forms a double bond with an oxygen atom. The right carbon atom also forms a single bond with an oxygen atom. This oxygen
forms a single bond with a hydrogen atom. Figure C shows a 3-D ball-and-stick model of acetic acid.

 Example 2.4.1: Empirical and Molecular Formulas

Molecules of glucose (blood sugar) contain 6 carbon atoms, 12 hydrogen atoms, and 6 oxygen atoms. What are the molecular
and empirical formulas of glucose?

Solution
The molecular formula is C6H12O6 because one molecule actually contains 6 C, 12 H, and 6 O atoms. The simplest whole-
number ratio of C to H to O atoms in glucose is 1:2:1, so the empirical formula is CH2O.

 Exercise 2.4.1

A molecule of metaldehyde (a pesticide used for snails and slugs) contains 8 carbon atoms, 16 hydrogen atoms, and 4 oxygen
atoms. What are the molecular and empirical formulas of metaldehyde?

Answer
Molecular formula, C8H16O4; empirical formula, C2H4O

Access for free at OpenStax 2.4.3 https://chem.libretexts.org/@go/page/38141


It is important to be aware that it may be possible for the same atoms to be arranged in different ways: Compounds with the same
molecular formula may have different atom-to-atom bonding and therefore different structures. For example, could there be another
compound with the same formula as acetic acid, C2H4O2? And if so, what would be the structure of its molecules?
If you predict that another compound with the formula C2H4O2 could exist, then you demonstrated good chemical insight and are
correct. Two C atoms, four H atoms, and two O atoms can also be arranged to form methyl formate, which is used in
manufacturing, as an insecticide, and for quick-drying finishes. Methyl formate molecules have one of the oxygen atoms between
the two carbon atoms, differing from the arrangement in acetic acid molecules. Acetic acid and methyl formate are examples of
isomers—compounds with the same chemical formula but different molecular structures (Figure 2.4.7). Note that this small
difference in the arrangement of the atoms has a major effect on their respective chemical properties. You would certainly not want
to use a solution of methyl formate as a substitute for a solution of acetic acid (vinegar) when you make salad dressing.

Figure 2.4.7 : Molecules of (a) acetic acid and methyl formate (b) are structural isomers; they have the same formula (C2H4O2) but
different structures (and therefore different chemical properties).
Figure A shows a structural diagram of acetic acid, C subscript 2 H subscript 4 O subscript 2. Acetic acid contains two carbon
atoms connected by a single bond. The left carbon atom forms single bonds with three hydrogen atoms. The carbon on the right
forms a double bond with an oxygen atom. The right carbon atom also forms a single bond to an oxygen atom which forms a single
bond with a hydrogen atom. Figure B shows a structural diagram of methyl formate, C subscript 2 H subscript 4 O subscript 2. This
molecule contains a carbon atom which forms single bonds with three hydrogen atoms, and a single bond with an oxygen atom.
The oxygen atom forms a single bond with another carbon atom which forms a double bond with another oxygen atom and a single
bond with a hydrogen atom.
Many types of isomers exist (Figure 2.4.8). Acetic acid and methyl formate are structural isomers, compounds in which the
molecules differ in how the atoms are connected to each other. There are also various types of spatial isomers, in which the relative
orientations of the atoms in space can be different. For example, the compound carvone (found in caraway seeds, spearmint, and
mandarin orange peels) consists of two isomers that are mirror images of each other. S-(+)-carvone smells like caraway, and R-(−)-
carvone smells like spearmint.

Access for free at OpenStax 2.4.4 https://chem.libretexts.org/@go/page/38141


Figure 2.4.8 : Molecules of carvone are spatial isomers; they only differ in the relative orientations of the atoms in space. (credit
bottom left: modification of work by “Miansari66”/Wikimedia Commons; credit bottom right: modification of work by Forest &
Kim Starr)

Summary
A molecular formula uses chemical symbols and subscripts to indicate the exact numbers of different atoms in a molecule or
compound. An empirical formula gives the simplest, whole-number ratio of atoms in a compound. A structural formula indicates
the bonding arrangement of the atoms in the molecule. Ball-and-stick and space-filling models show the geometric arrangement of
atoms in a molecule. Isomers are compounds with the same molecular formula but different arrangements of atoms.

Glossary
empirical formula
formula showing the composition of a compound given as the simplest whole-number ratio of atoms

isomers
compounds with the same chemical formula but different structures

molecular formula
formula indicating the composition of a molecule of a compound and giving the actual number of atoms of each element in a
molecule of the compound.

spatial isomers
compounds in which the relative orientations of the atoms in space differ

structural isomer
one of two substances that have the same molecular formula but different physical and chemical properties because their atoms
are bonded differently

structural formula
shows the atoms in a molecule and how they are connected

Access for free at OpenStax 2.4.5 https://chem.libretexts.org/@go/page/38141


This page titled 2.4: Chemical Formulas is shared under a CC BY 4.0 license and was authored, remixed, and/or curated by OpenStax via source
content that was edited to the style and standards of the LibreTexts platform; a detailed edit history is available upon request.

Access for free at OpenStax 2.4.6 https://chem.libretexts.org/@go/page/38141


2.5: The Periodic Table
 Learning Objectives
State the periodic law and explain the organization of elements in the periodic table
Predict the general properties of elements based on their location within the periodic table
Identify metals, nonmetals, and metalloids by their properties and/or location on the periodic table

As early chemists worked to purify ores and discovered more elements, they realized that various elements could be grouped
together by their similar chemical behaviors. One such grouping includes lithium (Li), sodium (Na), and potassium (K): These
elements all are shiny, conduct heat and electricity well, and have similar chemical properties. A second grouping includes calcium
(Ca), strontium (Sr), and barium (Ba), which also are shiny, good conductors of heat and electricity, and have chemical properties
in common. However, the specific properties of these two groupings are notably different from each other. For example: Li, Na,
and K are much more reactive than are Ca, Sr, and Ba; Li, Na, and K form compounds with oxygen in a ratio of two of their atoms
to one oxygen atom, whereas Ca, Sr, and Ba form compounds with one of their atoms to one oxygen atom. Fluorine (F), chlorine
(Cl), bromine (Br), and iodine (I) also exhibit similar properties to each other, but these properties are drastically different from
those of any of the elements above.
Dimitri Mendeleev in Russia (1869) and Lothar Meyer in Germany (1870) independently recognized that there was a periodic
relationship among the properties of the elements known at that time. Both published tables with the elements arranged according
to increasing atomic mass. But Mendeleev went one step further than Meyer: He used his table to predict the existence of elements
that would have the properties similar to aluminum and silicon, but were yet unknown. The discoveries of gallium (1875) and
germanium (1886) provided great support for Mendeleev’s work. Although Mendeleev and Meyer had a long dispute over priority,
Mendeleev’s contributions to the development of the periodic table are now more widely recognized (Figure 2.5.1).

Access for free at OpenStax 2.5.1 https://chem.libretexts.org/@go/page/38142


Figure 2.5.1 : (a) Dimitri Mendeleev is widely credited with creating (b) the first periodic table of the elements. (credit a:
modification of work by Serge Lachinov; credit b: modification of work by “Den fjättrade ankan”/Wikimedia Commons)
Figure A shows a photograph of Dimitri Mendeleev. Figure B shows the first periodic table developed by Mendeleev, which had
eight groups and twelve periods. In the first group (—, R superscript plus sign 0) is the following information: H = 1, L i = 7, N a =
23, K = 39, (C u = 63), R b = 85, (A g = 108), C a = 183, (—),—, (A u = 199) —. Note that each of these entries corresponds to
one of the twelve periods respectively. The second group (—, R 0) contains the following information: (not entry for period 1) B o
= 9, 4, M g = 24, C a = 40, Z n = 65, S r = 87, C d = 112, B a = 187, —, —, H g = 200, —. Note the ach of these entries
corresponds to one of the twelve periods respectively. Group three (—, R superscript one 0 superscript nine) contains the
information: (no entry for period 1), B = 11, A l = 27, 8. — = 44, — = 68, ? Y t = 88, I n = 113, ? D I = 138, —, ? E r = 178, T l =
204, —. Note that each of these entries corresponds to one of the twelve periods respectively. Group four (RH superscript four, R0
superscript eight) contains the following information: (no entry for period 1), C = 12, B i = 28, T i = 48, — = 72, Z r = 90, S n =
118, ? C o = 140, ? L a = 180, P b = 207, T h = 231. Note that each of these entries corresponds to one of the twelve periods
respectively. Group five (R H superscript two, R superscript two 0 superscript five) contains the following information: (no entry
for period 1), N = 14, P = 31, V = 51, A s = 75, N b = 94, S b = 122, —, —, T a = 182, B l = 208, —. Note that each of these
entries corresponds to one of the twelve periods respectively. Group six (R H superscript two, R 0 superscript three) contains the
following information: (no entry for period 1), O = 16, S = 32, C r = 52, S o = 78, M o = 96, T o = 125, —, —, W = 184, —, U =
240. Note that each of these entries corresponds to one of the twelve periods respectively. Group seven (R H , R superscript plus
sing, 0 superscript 7) contains the following information: (no entry for period 1), F = 19, C l = 35, 5, M n = 55, B r = 80, — = 100,
J = 127, —, —, —, —, —. Note that each of these entries corresponds to one of the twelve periods respectively. Group 8 (—, R 0
superscript four) contains the following information: (no entry for periods 1, 2, 3), in period 4: F o = 56, C o = 59, N i = 59, C u =
63, no entry for period five, in period 6: R u = 104, R h = 104, P d = 106, A g = 108, no entries for periods 7, 8 , or 9, in period 10:
O s = 195, I r = 197, P t = 198, A u = 199, no entries for periods 11 or 12.
By the twentieth century, it became apparent that the periodic relationship involved atomic numbers rather than atomic masses. The
modern statement of this relationship, the periodic law, is as follows: the properties of the elements are periodic functions of their
atomic numbers. A modern periodic table arranges the elements in increasing order of their atomic numbers and groups atoms with
similar properties in the same vertical column (Figure 2.5.2). Each box represents an element and contains its atomic number,
symbol, average atomic mass, and (sometimes) name. The elements are arranged in seven horizontal rows, called periods or series,
and 18 vertical columns, called groups. Groups are labeled at the top of each column. In the United States, the labels traditionally
were numerals with capital letters. However, IUPAC recommends that the numbers 1 through 18 be used, and these labels are more
common. For the table to fit on a single page, parts of two of the rows, a total of 14 columns, are usually written below the main
body of the table.

Access for free at OpenStax 2.5.2 https://chem.libretexts.org/@go/page/38142


Figure 2.5.2 : Elements in the periodic table are organized according to their properties.
The Periodic Table of Elements is shown. The 18 columns are labeled “Group” and the 7 rows are labeled “Period.” Below the
table to the right is a box labeled “Color Code” with different colors for metals, metalloids, and nonmetals, as well as solids,
liquids, and gases. To the left of this box is an enlarged picture of the upper-left most box on the table. The number 1 is in its upper-
left hand corner and is labeled “Atomic number.” The letter “H” is in the middle in red indicating that it is a gas. It is labeled
“Symbol.” Below that is the number 1.008 which is labeled “Atomic Mass.” Below that is the word hydrogen which is labeled
“name.” The color of the box indicates that it is a nonmetal. Each element will be described in this order: atomic number; name;
symbol; whether it is a metal, metalloid, or nonmetal; whether it is a solid, liquid, or gas; and atomic mass. Beginning at the top left
of the table, or period 1, group 1, is a box containing “1; hydrogen; H; nonmetal; gas; and 1.008.” There is only one other element
box in period 1, group 18, which contains “2; helium; H e; nonmetal; gas; and 4.003.” Period 2, group 1 contains “3; lithium; L i;
metal; solid; and 6.94” Group 2 contains “4; beryllium; B e; metal; solid; and 9.012.” Groups 3 through 12 are skipped and group
13 contains “5; boron; B; metalloid; solid; 10.81.” Group 14 contains “6; carbon; C; nonmetal; solid; and 12.01.” Group 15
contains “7; nitrogen; N; nonmetal; gas; and 14.01.” Group 16 contains “8; oxygen; O; nonmetal; gas; and 16.00.” Group 17
contains “9; fluorine; F; nonmetal; gas; and 19.00.” Group 18 contains “10; neon; N e; nonmetal; gas; and 20.18.” Period 3, group
1 contains “11; sodium; N a; metal; solid; and 22.99.” Group 2 contains “12; magnesium; M g; metal; solid; and 24.31.” Groups 3
through 12 are skipped again in period 3 and group 13 contains “13; aluminum; A l; metal; solid; and 26.98.” Group 14 contains
“14; silicon; S i; metalloid; solid; and 28.09.” Group 15 contains “15; phosphorous; P; nonmetal; solid; and 30.97.” Group 16
contains “16; sulfur; S; nonmetal; solid; and 32.06.” Group 17 contains “17; chlorine; C l; nonmetal; gas; and 35.45.” Group 18
contains “18; argon; A r; nonmetal; gas; and 39.95.” Period 4, group 1 contains “19; potassium; K; metal; solid; and 39.10.” Group
2 contains “20; calcium; C a; metal; solid; and 40.08.” Group 3 contains “21; scandium; S c; metal; solid; and 44.96.” Group 4
contains “22; titanium; T i; metal; solid; and 47.87.” Group 5 contains “23; vanadium; V; metal; solid; and 50.94.” Group 6
contains “24; chromium; C r; metal; solid; and 52.00.” Group 7 contains “25; manganese; M n; metal; solid; and 54.94.” Group 8
contains “26; iron; F e; metal; solid; and 55.85.” Group 9 contains “27; cobalt; C o; metal; solid; and 58.93.” Group 10 contains
“28; nickel; N i; metal; solid; and 58.69.” Group 11 contains “29; copper; C u; metal; solid; and 63.55.” Group 12 contains “30;
zinc; Z n; metal; solid; and 65.38.” Group 13 contains “31; gallium; G a; metal; solid; and 69.72.” Group 14 contains “32;
germanium; G e; metalloid; solid; and 72.63.” Group 15 contains “33; arsenic; A s; metalloid; solid; and 74.92.” Group 16 contains
“34; selenium; S e; nonmetal; solid; and 78.97.” Group 17 contains “35; bromine; B r; nonmetal; liquid; and 79.90.” Group 18
contains “36; krypton; K r; nonmetal; gas; and 83.80.” Period 5, group 1 contains “37; rubidium; R b; metal; solid; and 85.47.”
Group 2 contains “38; strontium; S r; metal; solid; and 87.62.” Group 3 contains “39; yttrium; Y; metal; solid; and 88.91.” Group 4
contains “40; zirconium; Z r; metal; solid; and 91.22.” Group 5 contains “41; niobium; N b; metal; solid; and 92.91.” Group 6
contains “42; molybdenum; M o; metal; solid; and 95.95.” Group 7 contains “43; technetium; T c; metal; solid; and 97.” Group 8
contains “44; ruthenium; R u; metal; solid; and 101.1.” Group 9 contains “45; rhodium; R h; metal; solid; and 102.9.” Group 10
contains “46; palladium; P d; metal; solid; and 106.4.” Group 11 contains “47; silver; A g; metal; solid; and 107.9.” Group 12
contains “48; cadmium; C d; metal; solid; and 112.4.” Group 13 contains “49; indium; I n; metal; solid; and 114.8.” Group 14
contains “50; tin; S n; metal; solid; and 118.7.” Group 15 contains “51; antimony; S b; metalloid; solid; and 121.8.” Group 16
contains “52; tellurium; T e; metalloid; solid; and 127.6.” Group 17 contains “53; iodine; I; nonmetal; solid; and 126.9.” Group 18
contains “54; xenon; X e; nonmetal; gas; and 131.3.” Period 6, group 1 contains “55; cesium; C s; metal; solid; and 132.9.” Group
2 contains “56; barium; B a; metal; solid; and 137.3.” Group 3 breaks the pattern. The box has a large arrow pointing to a row of
elements below the table with atomic numbers ranging from 57-71. In sequential order by atomic number, the first box in this row

Access for free at OpenStax 2.5.3 https://chem.libretexts.org/@go/page/38142


contains “57; lanthanum; L a; metal; solid; and 138.9.” To its right, the next is “58; cerium; C e; metal; solid; and 140.1.” Next is
“59; praseodymium; P r; metal; solid; and 140.9.” Next is “60; neodymium; N d; metal; solid; and 144.2.” Next is “61;
promethium; P m; metal; solid; and 145.” Next is “62; samarium; S m; metal; solid; and 150.4.” Next is “63; europium; E u; metal;
solid; and 152.0.” Next is “64; gadolinium; G d; metal; solid; and 157.3.” Next is “65; terbium; T b; metal; solid; and 158.9.” Next
is “66; dysprosium; D y; metal; solid; and 162.5.” Next is “67; holmium; H o; metal; solid; and 164.9.” Next is “68; erbium; E r;
metal; solid; and 167.3.” Next is “69; thulium; T m; metal; solid; and 168.9.” Next is “70; ytterbium; Y b; metal; solid; and 173.1.”
The last in this special row is “71; lutetium; L u; metal; solid; and 175.0.” Continuing in period 6, group 4 contains “72; hafnium;
H f; metal; solid; and 178.5.” Group 5 contains “73; tantalum; T a; metal; solid; and 180.9.” Group 6 contains “74; tungsten; W;
metal; solid; and 183.8.” Group 7 contains “75; rhenium; R e; metal; solid; and 186.2.” Group 8 contains “76; osmium; O s; metal;
solid; and 190.2.” Group 9 contains “77; iridium; I r; metal; solid; and 192.2.” Group 10 contains “78; platinum; P t; metal; solid;
and 195.1.” Group 11 contains “79; gold; A u; metal; solid; and 197.0.” Group 12 contains “80; mercury; H g; metal; liquid; and
200.6.” Group 13 contains “81; thallium; T l; metal; solid; and 204.4.” Group 14 contains “82; lead; P b; metal; solid; and 207.2.”
Group 15 contains “83; bismuth; B i; metal; solid; and 209.0.” Group 16 contains “84; polonium; P o; metal; solid; and 209.”
Group 17 contains “85; astatine; A t; metalloid; solid; and 210.” Group 18 contains “86; radon; R n; nonmetal; gas; and 222.”
Period 7, group 1 contains “87; francium; F r; metal; solid; and 223.” Group 2 contains “88; radium; R a; metal; solid; and 226.”
Group 3 breaks the pattern much like what occurs in period 6. A large arrow points from the box in period 7, group 3 to a special
row containing the elements with atomic numbers ranging from 89-103, just below the row which contains atomic numbers 57-71.
In sequential order by atomic number, the first box in this row contains “89; actinium; A c; metal; solid; and 227.” To its right, the
next is “90; thorium; T h; metal; solid; and 232.0.” Next is “91; protactinium; P a; metal; solid; and 231.0.” Next is “92; uranium;
U; metal; solid; and 238.0.” Next is “93; neptunium; N p; metal; solid; and N p.” Next is “94; plutonium; P u; metal; solid; and
244.” Next is “95; americium; A m; metal; solid; and 243.” Next is “96; curium; C m; metal; solid; and 247.” Next is “97;
berkelium; B k; metal; solid; and 247.” Next is “98; californium; C f; metal; solid; and 251.” Next is “99; einsteinium; E s; metal;
solid; and 252.” Next is “100; fermium; F m; metal; solid; and 257.” Next is “101; mendelevium; M d; metal; solid; and 258.” Next
is “102; nobelium; N o; metal; solid; and 259.” The last in this special row is “103; lawrencium; L r; metal; solid; and 262.”
Continuing in period 7, group 4 contains “104; rutherfordium; R f; metal; solid; and 267.” Group 5 contains “105; dubnium; D b;
metal; solid; and 270.” Group 6 contains “106; seaborgium; S g; metal; solid; and 271.” Group 7 contains “107; bohrium; B h;
metal; solid; and 270.” Group 8 contains “108; hassium; H s; metal; solid; and 277.” Group 9 contains “109; meitnerium; M t; not
indicated; solid; and 276.” Group 10 contains “110; darmstadtium; D s; not indicated; solid; and 281.” Group 11 contains “111;
roentgenium; R g; not indicated; solid; and 282.” Group 12 contains “112; copernicium; C n; metal; liquid; and 285.” Group 13
contains “113; ununtrium; U u t; not indicated; solid; and 285.” Group 14 contains “114; flerovium; F l; not indicated; solid; and
289.” Group 15 contains “115; ununpentium; U u p; not indicated; solid; and 288.” Group 16 contains “116; livermorium; L v; not
indicated; solid; and 293.” Group 17 contains “117; ununseptium; U u s; not indicated; solid; and 294.” Group 18 contains “118;
ununoctium; U u o; not indicated; solid; and 294.”
Many elements differ dramatically in their chemical and physical properties, but some elements are similar in their behaviors. For
example, many elements appear shiny, are malleable (able to be deformed without breaking) and ductile (can be drawn into wires),
and conduct heat and electricity well. Other elements are not shiny, malleable, or ductile, and are poor conductors of heat and
electricity. We can sort the elements into large classes with common properties: metals (elements that are shiny, malleable, good
conductors of heat and electricity—shaded yellow); nonmetals (elements that appear dull, poor conductors of heat and electricity—
shaded green); and metalloids (elements that conduct heat and electricity moderately well, and possess some properties of metals
and some properties of nonmetals—shaded purple).
The elements can also be classified into the main-group elements (or representative elements) in the columns labeled 1, 2, and 13–
18; the transition metals in the columns labeled 3–12; and inner transition metals in the two rows at the bottom of the table (the top-
row elements are called lanthanides and the bottom-row elements are actinides; Figure 2.5.3). The elements can be subdivided
further by more specific properties, such as the composition of the compounds they form. For example, the elements in group 1 (the
first column) form compounds that consist of one atom of the element and one atom of hydrogen. These elements (except
hydrogen) are known as alkali metals, and they all have similar chemical properties. The elements in group 2 (the second column)
form compounds consisting of one atom of the element and two atoms of hydrogen: These are called alkaline earth metals, with
similar properties among members of that group. Other groups with specific names are the pnictogens (group 15), chalcogens
(group 16), halogens (group 17), and the noble gases (group 18, also known as inert gases). The groups can also be referred to by
the first element of the group: For example, the chalcogens can be called the oxygen group or oxygen family. Hydrogen is a unique,
nonmetallic element with properties similar to both group 1 and group 17 elements. For that reason, hydrogen may be shown at the
top of both groups, or by itself.

Access for free at OpenStax 2.5.4 https://chem.libretexts.org/@go/page/38142


Figure 2.5.3 : The periodic table organizes elements with similar properties into groups.
This diagram combines the groups and periods of the periodic table based on their similar properties. Group 1 contains the alkali
metals, group 2 contains the earth alkaline metals, group 15 contains the pnictogens, group 16 contains the chalcogens, group 17
contains the halogens and group 18 contains the noble gases. The main group elements consist of groups 1, 2, and 12 through 18.
Therefore, most of the transition metals, which are contained in groups 3 through 11, are not main group elements. The lanthanides
and actinides are called out at the bottom of the periodic table.

 Example 2.5.1: Naming Groups of Elements

Atoms of each of the following elements are essential for life. Give the group name for the following elements:
a. chlorine
b. calcium
c. sodium
d. sulfur

Solution
The family names are as follows:
a. halogen
b. alkaline earth metal
c. alkali metal
d. chalcogen

 Exercise 2.5.1

Give the group name for each of the following elements:


a. krypton
b. selenium
c. barium
d. lithium

Answer a

Access for free at OpenStax 2.5.5 https://chem.libretexts.org/@go/page/38142


noble gas
Answer b
chalcogen
Answer c
alkaline earth metal
Answer d
alkali metal

In studying the periodic table, you might have noticed something about the atomic masses of some of the elements. Element 43
(technetium), element 61 (promethium), and most of the elements with atomic number 84 (polonium) and higher have their atomic
mass given in square brackets. This is done for elements that consist entirely of unstable, radioactive isotopes (you will learn more
about radioactivity in the nuclear chemistry chapter). An average atomic weight cannot be determined for these elements because
their radioisotopes may vary significantly in relative abundance, depending on the source, or may not even exist in nature. The
number in square brackets is the atomic mass number (and approximate atomic mass) of the most stable isotope of that element.

Summary
The discovery of the periodic recurrence of similar properties among the elements led to the formulation of the periodic table, in
which the elements are arranged in order of increasing atomic number in rows known as periods and columns known as groups.
Elements in the same group of the periodic table have similar chemical properties. Elements can be classified as metals, metalloids,
and nonmetals, or as a main-group elements, transition metals, and inner transition metals. Groups are numbered 1–18 from left to
right. The elements in group 1 are known as the alkali metals; those in group 2 are the alkaline earth metals; those in 15 are the
pnictogens; those in 16 are the chalcogens; those in 17 are the halogens; and those in 18 are the noble gases.

Glossary
actinide
inner transition metal in the bottom of the bottom two rows of the periodic table

alkali metal
element in group 1

alkaline earth metal


element in group 2

chalcogen
element in group 16

group
vertical column of the periodic table

halogen
element in group 17

inert gas
(also, noble gas) element in group 18

inner transition metal


(also, lanthanide or actinide) element in the bottom two rows; if in the first row, also called lanthanide, or if in the second row,
also called actinide

Access for free at OpenStax 2.5.6 https://chem.libretexts.org/@go/page/38142


lanthanide
inner transition metal in the top of the bottom two rows of the periodic table

main-group element
(also, representative element) element in columns 1, 2, and 12–18

metal
element that is shiny, malleable, good conductor of heat and electricity

metalloid
element that conducts heat and electricity moderately well, and possesses some properties of metals and some properties of
nonmetals

noble gas
(also, inert gas) element in group 18

nonmetal
element that appears dull, poor conductor of heat and electricity

period
(also, series) horizontal row of the periodic table

periodic law
properties of the elements are periodic function of their atomic numbers.

periodic table
table of the elements that places elements with similar chemical properties close together

pnictogen
element in group 15

representative element
(also, main-group element) element in columns 1, 2, and 12–18

transition metal
element in columns 3–11

series
(also, period) horizontal row of the period table

This page titled 2.5: The Periodic Table is shared under a CC BY 4.0 license and was authored, remixed, and/or curated by OpenStax via source
content that was edited to the style and standards of the LibreTexts platform; a detailed edit history is available upon request.

Access for free at OpenStax 2.5.7 https://chem.libretexts.org/@go/page/38142


2.6: Molecular and Ionic Compounds
 Learning Objectives
Define ionic and molecular (covalent) compounds
Predict the type of compound formed from elements based on their location within the periodic table
Determine formulas for simple ionic compounds

In ordinary chemical reactions, the nucleus of each atom (and thus the identity of the element) remains unchanged. Electrons,
however, can be added to atoms by transfer from other atoms, lost by transfer to other atoms, or shared with other atoms. The
transfer and sharing of electrons among atoms govern the chemistry of the elements. During the formation of some compounds,
atoms gain or lose electrons, and form electrically charged particles called ions (Figure 2.6.1).

Figure 2.6.1 : (a) A sodium atom (Na) has equal numbers of protons and electrons (11) and is uncharged. (b) A sodium cation (Na+)
has lost an electron, so it has one more proton (11) than electrons (10), giving it an overall positive charge, signified by a
superscripted plus sign.
Figure A shows a sodium atom which has a nucleus containing 11 protons, 12 neutrons, and 11 electrons. Figure B shows a sodium
ion. Its nucleus contains 11 protons, 12 neutrons, and 11 electrons.
You can use the periodic table to predict whether an atom will form an anion or a cation, and you can often predict the charge of the
resulting ion. Atoms of many main-group metals lose enough electrons to leave them with the same number of electrons as an atom
of the preceding noble gas. To illustrate, an atom of an alkali metal (group 1) loses one electron and forms a cation with a 1+
charge; an alkaline earth metal (group 2) loses two electrons and forms a cation with a 2+ charge, and so on. For example, a neutral
calcium atom, with 20 protons and 20 electrons, readily loses two electrons. This results in a cation with 20 protons, 18 electrons,
and a 2+ charge. It has the same number of electrons as atoms of the preceding noble gas, argon, and is symbolized Ca2+. The name
of a metal ion is the same as the name of the metal atom from which it forms, so Ca2+ is called a calcium ion.
When atoms of nonmetal elements form ions, they generally gain enough electrons to give them the same number of electrons as an
atom of the next noble gas in the periodic table. Atoms of group 17 gain one electron and form anions with a 1− charge; atoms of
group 16 gain two electrons and form ions with a 2− charge, and so on. For example, the neutral bromine atom, with 35 protons
and 35 electrons, can gain one electron to provide it with 36 electrons. This results in an anion with 35 protons, 36 electrons, and a
1− charge. It has the same number of electrons as atoms of the next noble gas, krypton, and is symbolized Br−. (A discussion of the
theory supporting the favored status of noble gas electron numbers reflected in these predictive rules for ion formation is provided
in a later chapter of this text.)
Note the usefulness of the periodic table in predicting likely ion formation and charge (Figure 2.6.2). Moving from the far left to
the right on the periodic table, main-group elements tend to form cations with a charge equal to the group number. That is, group 1
elements form 1+ ions; group 2 elements form 2+ ions, and so on. Moving from the far right to the left on the periodic table,
elements often form anions with a negative charge equal to the number of groups moved left from the noble gases. For example,
group 17 elements (one group left of the noble gases) form 1− ions; group 16 elements (two groups left) form 2− ions, and so on.
This trend can be used as a guide in many cases, but its predictive value decreases when moving toward the center of the periodic
table. In fact, transition metals and some other metals often exhibit variable charges that are not predictable by their location in the
table. For example, copper can form ions with a 1+ or 2+ charge, and iron can form ions with a 2+ or 3+ charge.

Access for free at OpenStax 2.6.1 https://chem.libretexts.org/@go/page/38143


Figure 2.6.2 : Some elements exhibit a regular pattern of ionic charge when they form ions.
This periodic table shows the charges of ions corresponding with the group number. Starting with a charge of positive one for
group 1, positive 2 for group 2, and negative 4, 3, 2, and 1 for groups 14, 15, 16, and 17 respectively. Noble gases on the far right
do not have any charges. Some of the ions of transition metals are shown, with multiple charges existing for certain metals.

 Example 2.6.1: Composition of Ions

An ion found in some compounds used as antiperspirants contains 13 protons and 10 electrons. What is its symbol?

Solution
Because the number of protons remains unchanged when an atom forms an ion, the atomic number of the element must be 13.
Knowing this lets us use the periodic table to identify the element as Al (aluminum). The Al atom has lost three electrons and
thus has three more positive charges (13) than it has electrons (10). This is the aluminum cation, Al3+.

 Exercise 2.6.1

Give the symbol and name for the ion with 34 protons and 36 electrons.

Answer
Se2−, the selenide ion

 Example 2.6.2: Formation of Ions

Magnesium and nitrogen react to form an ionic compound. Predict which forms an anion, which forms a cation, and the
charges of each ion. Write the symbol for each ion and name them.

Solution
Magnesium’s position in the periodic table (group 2) tells us that it is a metal. Metals form positive ions (cations). A
magnesium atom must lose two electrons to have the same number electrons as an atom of the previous noble gas, neon. Thus,
a magnesium atom will form a cation with two fewer electrons than protons and a charge of 2+. The symbol for the ion is
Mg2+, and it is called a magnesium ion.

Access for free at OpenStax 2.6.2 https://chem.libretexts.org/@go/page/38143


Nitrogen’s position in the periodic table (group 15) reveals that it is a nonmetal. Nonmetals form negative ions (anions). A
nitrogen atom must gain three electrons to have the same number of electrons as an atom of the following noble gas, neon.
Thus, a nitrogen atom will form an anion with three more electrons than protons and a charge of 3−. The symbol for the ion is
N3−, and it is called a nitride ion.

 Exercise 2.6.2

Aluminum and carbon react to form an ionic compound. Predict which forms an anion, which forms a cation, and the charges
of each ion. Write the symbol for each ion and name them.

Answer
Al will form a cation with a charge of 3+: Al3+, an aluminum ion. Carbon will form an anion with a charge of 4−: C4−, a
carbide ion.

The ions that we have discussed so far are called monatomic ions, that is, they are ions formed from only one atom. We also find
many polyatomic ions. These ions, which act as discrete units, are electrically charged molecules (a group of bonded atoms with an
overall charge). Some of the more important polyatomic ions are listed in Table 2.6.1. Oxyanions are polyatomic ions that contain
one or more oxygen atoms. At this point in your study of chemistry, you should memorize the names, formulas, and charges of the
most common polyatomic ions. Because you will use them repeatedly, they will soon become familiar.
Table 2.6.1 : Common Polyatomic Ions
Name Formula Related Acid Formula

ammonium NH
+

hydronium H O
3
+

oxide O
2 −

peroxide O
2 −
2

hydroxide OH

acetate CH COO
3

acetic acid CH COOH
3

cyanide CN

hydrocyanic acid HCN

azide N

3
hydrazoic acid HN
3

carbonate CO
2 −
3
carbonic acid H CO
2 3

bicarbonate HCO

3

nitrate NO

3
nitric acid HNO
3

nitrite NO

2
nitrous acid HNO
2

sulfate SO
2 −
4
sulfuric acid H SO
2 4

hydrogen sulfate HSO



4

sulfite SO
2 −
3
sulfurous acid H SO
2 3

hydrogen sulfite HSO



3

phosphate PO
3 −
4
phosphoric acid H PO
3 4

hydrogen phosphate HPO


2 −
4

dihydrogen phosphate H PO
2

4

perchlorate ClO

4
perchloric acid HClO
4

chlorate ClO

3
chloric acid HClO
3

Access for free at OpenStax 2.6.3 https://chem.libretexts.org/@go/page/38143


Name Formula Related Acid Formula

chlorite ClO

2
chlorous acid HClO
2

hypochlorite ClO

hypochlorous acid HClO

chromate CrO
2 −
4
chromic acid H CrO
2 4

dichromate Cr O
2
2 −
7
dichromic acid H Cr O
2 2 7

permanganate MnO

4
permanganic acid HMnO
4

Note that there is a system for naming some polyatomic ions; -ate and -ite are suffixes designating polyatomic ions containing
more or fewer oxygen atoms. Per- (short for “hyper”) and hypo- (meaning “under”) are prefixes meaning more oxygen atoms than
-ate and fewer oxygen atoms than -ite, respectively. For example, perchlorate is ClO , chlorate is ClO , chlorite is ClO and

4

3

2

hypochlorite is ClO−. Unfortunately, the number of oxygen atoms corresponding to a given suffix or prefix is not consistent; for
example, nitrate is NO while sulfate is SO . This will be covered in more detail in the next module on nomenclature.

3
2 −
4

The nature of the attractive forces that hold atoms or ions together within a compound is the basis for classifying chemical bonding.
When electrons are transferred and ions form, ionic bonds result. Ionic bonds are electrostatic forces of attraction, that is, the
attractive forces experienced between objects of opposite electrical charge (in this case, cations and anions). When electrons are
“shared” and molecules form, covalent bonds result. Covalent bonds are the attractive forces between the positively charged nuclei
of the bonded atoms and one or more pairs of electrons that are located between the atoms. Compounds are classified as ionic or
molecular (covalent) on the basis of the bonds present in them.

2.6.1: Ionic Compounds


When an element composed of atoms that readily lose electrons (a metal) reacts with an element composed of atoms that readily
gain electrons (a nonmetal), a transfer of electrons usually occurs, producing ions. The compound formed by this transfer is
stabilized by the electrostatic attractions (ionic bonds) between the ions of opposite charge present in the compound. For example,
when each sodium atom in a sample of sodium metal (group 1) gives up one electron to form a sodium cation, Na+, and each
chlorine atom in a sample of chlorine gas (group 17) accepts one electron to form a chloride anion, Cl−, the resulting compound,
NaCl, is composed of sodium ions and chloride ions in the ratio of one Na+ ion for each Cl− ion. Similarly, each calcium atom
(group 2) can give up two electrons and transfer one to each of two chlorine atoms to form CaCl2, which is composed of Ca2+ and
Cl− ions in the ratio of one Ca2+ ion to two Cl− ions.
A compound that contains ions and is held together by ionic bonds is called an ionic compound. The periodic table can help us
recognize many of the compounds that are ionic: When a metal is combined with one or more nonmetals, the compound is usually
ionic. This guideline works well for predicting ionic compound formation for most of the compounds typically encountered in an
introductory chemistry course. However, it is not always true (for example, aluminum chloride, AlCl3, is not ionic).
You can often recognize ionic compounds because of their properties. Ionic compounds are solids that typically melt at high
temperatures and boil at even higher temperatures. For example, sodium chloride melts at 801 °C and boils at 1413 °C. (As a
comparison, the molecular compound water melts at 0 °C and boils at 100 °C.) In solid form, an ionic compound is not electrically
conductive because its ions are unable to flow (“electricity” is the flow of charged particles). When molten, however, it can conduct
electricity because its ions are able to move freely through the liquid (Figure 2.6.3).

Access for free at OpenStax 2.6.4 https://chem.libretexts.org/@go/page/38143


Figure 2.6.3 : Sodium chloride melts at 801 °C and conducts electricity when molten. (credit: modification of work by Mark Blaser
and Matt Evans)
This figure shows three photos connected by right-facing arrows. The first shows a light bulb as part of a complex lab equipment
setup. The light bulb is not lit. The second photo shows a substances being heated or set on fire. The third shows the light bulb
again which is lit.
In every ionic compound, the total number of positive charges of the cations equals the total number of negative charges of the
anions. Thus, ionic compounds are electrically neutral overall, even though they contain positive and negative ions. We can use this
observation to help us write the formula of an ionic compound. The formula of an ionic compound must have a ratio of ions such
that the numbers of positive and negative charges are equal.

 Example 2.6.3: Predicting the Formula of an Ionic Compound

The gemstone sapphire (Figure 2.6.4) is mostly a compound of aluminum and oxygen that contains aluminum cations, Al3+,
and oxygen anions, O2−. What is the formula of this compound?

Figure 2.6.4 : Although pure aluminum oxide is colorless, trace amounts of iron and titanium give blue sapphire its
characteristic color. (credit: modification of work by Stanislav Doronenko)
Solution Because the ionic compound must be electrically neutral, it must have the same number of positive and negative
charges. Two aluminum ions, each with a charge of 3+, would give us six positive charges, and three oxide ions, each with a
charge of 2−, would give us six negative charges. The formula would be Al2O3.

 Exercise 2.6.3

Predict the formula of the ionic compound formed between the sodium cation, Na+, and the sulfide anion, S2−.

Answer
Na2S

Many ionic compounds contain polyatomic ions (Table 2.6.1) as the cation, the anion, or both. As with simple ionic compounds,
these compounds must also be electrically neutral, so their formulas can be predicted by treating the polyatomic ions as discrete
units. We use parentheses in a formula to indicate a group of atoms that behave as a unit. For example, the formula for calcium
phosphate, one of the minerals in our bones, is Ca3(PO4)2. This formula indicates that there are three calcium ions (Ca2+) for every
two phosphate (PO ) groups. The PO
3 −
4
groups are discrete units, each consisting of one phosphorus atom and four oxygen
3 −
4

Access for free at OpenStax 2.6.5 https://chem.libretexts.org/@go/page/38143


atoms, and having an overall charge of 3−. The compound is electrically neutral, and its formula shows a total count of three Ca,
two P, and eight O atoms.

 Example 2.6.4: Predicting the Formula of a Compound with a Polyatomic Anion

Baking powder contains calcium dihydrogen phosphate, an ionic compound composed of the ions Ca2+ and H 2

PO4 . What is
the formula of this compound?

Solution
The positive and negative charges must balance, and this ionic compound must be electrically neutral. Thus, we must have two
negative charges to balance the 2+ charge of the calcium ion. This requires a ratio of one Ca2+ ion to two H PO ions. We
2

4

designate this by enclosing the formula for the dihydrogen phosphate ion in parentheses and adding a subscript 2. The formula
is Ca(H2PO4)2.

 Exercise 2.6.4

Predict the formula of the ionic compound formed between the lithium ion and the peroxide ion, O 2−
2
(Hint: Use the periodic
table to predict the sign and the charge on the lithium ion.)

Answer
Li2O2

Because an ionic compound is not made up of single, discrete molecules, it may not be properly symbolized using a molecular
formula. Instead, ionic compounds must be symbolized by a formula indicating the relative numbers of its constituent ions. For
compounds containing only monatomic ions (such as NaCl) and for many compounds containing polyatomic ions (such as CaSO4),
these formulas are just the empirical formulas introduced earlier in this chapter. However, the formulas for some ionic compounds
containing polyatomic ions are not empirical formulas. For example, the ionic compound sodium oxalate is comprised of Na+ and
C O
2
2−
4
ions combined in a 2:1 ratio, and its formula is written as Na2C2O4. The subscripts in this formula are not the smallest-
possible whole numbers, as each can be divided by 2 to yield the empirical formula, NaCO2. This is not the accepted formula for
sodium oxalate, however, as it does not accurately represent the compound’s polyatomic anion, C O .2
2−

2.6.2: Molecular Compounds


Many compounds do not contain ions but instead consist solely of discrete, neutral molecules. These molecular compounds
(covalent compounds) result when atoms share, rather than transfer (gain or lose), electrons. Covalent bonding is an important and
extensive concept in chemistry, and it will be treated in considerable detail in a later chapter of this text. We can often identify
molecular compounds on the basis of their physical properties. Under normal conditions, molecular compounds often exist as
gases, low-boiling liquids, and low-melting solids, although many important exceptions exist.
Whereas ionic compounds are usually formed when a metal and a nonmetal combine, covalent compounds are usually formed by a
combination of nonmetals. Thus, the periodic table can help us recognize many of the compounds that are covalent. While we can
use the positions of a compound’s elements in the periodic table to predict whether it is ionic or covalent at this point in our study
of chemistry, you should be aware that this is a very simplistic approach that does not account for a number of interesting
exceptions. Shades of gray exist between ionic and molecular compounds, and you’ll learn more about those later.

 Example 2.6.5: Predicting the Type of Bonding in Compounds


Predict whether the following compounds are ionic or molecular:
a. KI, the compound used as a source of iodine in table salt
b. H2O2, the bleach and disinfectant hydrogen peroxide
c. CHCl3, the anesthetic chloroform

Access for free at OpenStax 2.6.6 https://chem.libretexts.org/@go/page/38143


d. Li2CO3, a source of lithium in antidepressants

Solution
a. Potassium (group 1) is a metal, and iodine (group 17) is a nonmetal; KI is predicted to be ionic.
b. Hydrogen (group 1) is a nonmetal, and oxygen (group 16) is a nonmetal; H2O2 is predicted to be molecular.
c. Carbon (group 14) is a nonmetal, hydrogen (group 1) is a nonmetal, and chlorine (group 17) is a nonmetal; CHCl3 is
predicted to be molecular.
d. Lithium (group 1) is a metal, and carbonate is a polyatomic ion; Li2CO3 is predicted to be ionic.

 Exercise 2.6.5
Using the periodic table, predict whether the following compounds are ionic or covalent:
a. SO2
b. CaF2
c. N2H4
d. Al2(SO4)3

Answer a
molecular
Answer b
ionic
Answer c
molecular
Answer d
ionic

Summary
Metals (particularly those in groups 1 and 2) tend to lose the number of electrons that would leave them with the same number of
electrons as in the preceding noble gas in the periodic table. By this means, a positively charged ion is formed. Similarly, nonmetals
(especially those in groups 16 and 17, and, to a lesser extent, those in Group 15) can gain the number of electrons needed to
provide atoms with the same number of electrons as in the next noble gas in the periodic table. Thus, nonmetals tend to form
negative ions. Positively charged ions are called cations, and negatively charged ions are called anions. Ions can be either
monatomic (containing only one atom) or polyatomic (containing more than one atom).
Compounds that contain ions are called ionic compounds. Ionic compounds generally form from metals and nonmetals.
Compounds that do not contain ions, but instead consist of atoms bonded tightly together in molecules (uncharged groups of atoms
that behave as a single unit), are called covalent compounds. Covalent compounds usually form from two or more nonmetals.

Glossary
covalent bond
attractive force between the nuclei of a molecule’s atoms and pairs of electrons between the atoms

covalent compound
(also, molecular compound) composed of molecules formed by atoms of two or more different elements

ionic bond
electrostatic forces of attraction between the oppositely charged ions of an ionic compound

Access for free at OpenStax 2.6.7 https://chem.libretexts.org/@go/page/38143


ionic compound
compound composed of cations and anions combined in ratios, yielding an electrically neutral substance

molecular compound
(also, covalent compound) composed of molecules formed by atoms of two or more different elements

monatomic ion
ion composed of a single atom

polyatomic ion
ion composed of more than one atom

oxyanion
polyatomic anion composed of a central atom bonded to oxygen atoms

This page titled 2.6: Molecular and Ionic Compounds is shared under a CC BY 4.0 license and was authored, remixed, and/or curated by
OpenStax via source content that was edited to the style and standards of the LibreTexts platform; a detailed edit history is available upon request.

Access for free at OpenStax 2.6.8 https://chem.libretexts.org/@go/page/38143


2.7: Chemical Nomenclature
 Learning Objectives
Derive names for common types of inorganic compounds using a systematic approach.
Describe how to name binary covalent compounds including acids and oxyacids.

Nomenclature, a collection of rules for naming things, is important in science and in many other situations. This module describes
an approach that is used to name simple ionic and molecular compounds, such as NaCl, CaCO3, and N2O4. The simplest of these
are binary compounds, those containing only two elements, but we will also consider how to name ionic compounds containing
polyatomic ions, and one specific, very important class of compounds known as acids (subsequent chapters in this text will focus
on these compounds in great detail). We will limit our attention here to inorganic compounds, compounds that are composed
principally of elements other than carbon, and will follow the nomenclature guidelines proposed by IUPAC. The rules for organic
compounds, in which carbon is the principle element, will be treated in a later chapter on organic chemistry.

2.7.1: Ionic Compounds


To name an inorganic compound, we need to consider the answers to several questions. First, is the compound ionic or molecular?
If the compound is ionic, does the metal form ions of only one type (fixed charge) or more than one type (variable charge)? Are the
ions monatomic or polyatomic? If the compound is molecular, does it contain hydrogen? If so, does it also contain oxygen? From
the answers we derive, we place the compound in an appropriate category and then name it accordingly. We will begin with the
nomenclature rules for ionic compounds.

2.7.2: Compounds Containing Only Monatomic Ions


The name of a binary compound containing monatomic ions consists of the name of the cation (the name of the metal) followed by
the name of the anion (the name of the nonmetallic element with its ending replaced by the suffix –ide). Some examples are given
in Table 2.7.2.
Table 2.7.1 : Names of Some Ionic Compounds
NaCl, sodium chloride Na2O, sodium oxide

KBr, potassium bromide CdS, cadmium sulfide

CaI2, calcium iodide Mg3N2, magnesium nitride

CsF, cesium fluoride Ca3P2, calcium phosphide

LiCl, lithium chloride Al4C3, aluminum carbide

2.7.3: Compounds Containing Polyatomic Ions


Compounds containing polyatomic ions are named similarly to those containing only monatomic ions, except there is no need to
change to an –ide ending, since the suffix is already present in the name of the anion. Examples are shown in Table 2.7.2.
CL, ammonium chloride, C a S O subscript 4 calcium sulfate, and M g subscript 3 ( P O subscript 4 ) subscript 2 magnesium
phosphate." data-quail-id="54" data-mt-width="1246">
Table 2.7.2 : Names of Some Polyatomic Ionic Compounds
KC2H3O2, potassium acetate (NH4)Cl, ammonium chloride

NaHCO3, sodium bicarbonate CaSO4, calcium sulfate

Al2(CO3)3, aluminum carbonate Mg3(PO4)2, magnesium phosphate

Access for free at OpenStax 2.7.1 https://chem.libretexts.org/@go/page/38144


 Ionic Compounds in Your Cabinets

Every day you encounter and use a large number of ionic compounds. Some of these compounds, where they are found, and
what they are used for are listed in Table 2.7.3. Look at the label or ingredients list on the various products that you use during
the next few days, and see if you run into any of those in this table, or find other ionic compounds that you could now name or
write as a formula.
Table 2.7.3 : Everyday Ionic Compounds
Ionic Compound Name Use

NaCl sodium chloride ordinary table salt

KI potassium iodide added to “iodized” salt for thyroid health

NaF sodium fluoride ingredient in toothpaste

baking soda; used in cooking (and in


NaHCO3 sodium bicarbonate
antacids)

Na2CO3 sodium carbonate washing soda; used in cleaning agents

NaOCl sodium hypochlorite active ingredient in household bleach

CaCO3 calcium carbonate ingredient in antacids

Mg(OH)2 magnesium hydroxide ingredient in antacids

Al(OH)3 aluminum hydroxide ingredient in antacids

NaOH sodium hydroxide lye; used as drain cleaner

K3PO4 potassium phosphate food additive (many purposes)

MgSO4 magnesium sulfate added to purified water

Na2HPO4 sodium hydrogen phosphate anti-caking agent; used in powdered products

Na2SO3 sodium sulfite preservative

2.7.4: Compounds Containing a Metal Ion with a Variable Charge


Most of the transition metals can form two or more cations with different charges. Compounds of these metals with nonmetals are
named with the same method as compounds in the first category, except the charge of the metal ion is specified by a Roman
numeral in parentheses after the name of the metal. The charge of the metal ion is determined from the formula of the compound
and the charge of the anion. For example, consider binary ionic compounds of iron and chlorine. Iron typically exhibits a charge of
either 2+ or 3+, and the two corresponding compound formulas are FeCl2 and FeCl3. The simplest name, “iron chloride,” will, in
this case, be ambiguous, as it does not distinguish between these two compounds. In cases like this, the charge of the metal ion is
included as a Roman numeral in parentheses immediately following the metal name. These two compounds are then
unambiguously named iron(II) chloride and iron(III) chloride, respectively. Other examples are provided in Table 2.7.4.
Table 2.7.4 : Names of Some Transition Metal Ionic Compounds
Transition Metal Ionic Compound Name

FeCl3 iron(III) chloride

Hg2O mercury(I) oxide

HgO mercury(II) oxide

Cu3(PO4)2 copper(II) phosphate

Out-of-date nomenclature used the suffixes –ic and –ous to designate metals with higher and lower charges, respectively: Iron(III)
chloride, FeCl3, was previously called ferric chloride, and iron(II) chloride, FeCl2, was known as ferrous chloride. Though this
naming convention has been largely abandoned by the scientific community, it remains in use by some segments of industry. For
example, you may see the words stannous fluoride on a tube of toothpaste. This represents the formula SnF2, which is more

Access for free at OpenStax 2.7.2 https://chem.libretexts.org/@go/page/38144


properly named tin(II) fluoride. The other fluoride of tin is SnF4, which was previously called stannic fluoride but is now named
tin(IV) fluoride.

 Example 2.7.1: Naming Ionic Compounds

Name the following ionic compounds, which contain a metal that can have more than one ionic charge:
a. Fe2S3
b. CuSe
c. GaN
d. CrCl3
e. Ti2(SO4)3

Solution
The anions in these compounds have a fixed negative charge (S2−, Se2− , N3−, Cl−, and SO ), and the compounds must be
2−

neutral. Because the total number of positive charges in each compound must equal the total number of negative charges, the
positive ions must be Fe3+, Cu2+, Ga3+, Cr3+, and Ti3+. These charges are used in the names of the metal ions:
a. iron(III) sulfide
b. copper(II) selenide
c. gallium(III) nitride
d. chromium(III) chloride
e. titanium(III) sulfate

 Exercise 2.7.1

Write the formulas of the following ionic compounds:


a. chromium(III) phosphide
b. mercury(II) sulfide
c. manganese(II) phosphate
d. copper(I) oxide
e. chromium(VI) fluoride

Answer a
CrP
Answer b
HgS
Answer c
Mn3(PO4)2
Answer d
Cu2O
Answer e
CrF6

 Erin Brokovich and Chromium Contamination

In the early 1990s, legal file clerk Erin Brockovich (Figure 2.7.2) discovered a high rate of serious illnesses in the small town
of Hinckley, California. Her investigation eventually linked the illnesses to groundwater contaminated by Cr(VI) used by
Pacific Gas & Electric (PG&E) to fight corrosion in a nearby natural gas pipeline. As dramatized in the film Erin Brokovich
(for which Julia Roberts won an Oscar), Erin and lawyer Edward Masry sued PG&E for contaminating the water near Hinckley

Access for free at OpenStax 2.7.3 https://chem.libretexts.org/@go/page/38144


in 1993. The settlement they won in 1996—$333 million—was the largest amount ever awarded for a direct-action lawsuit in
the US at that time.

Figure 2.7.2: (a) Erin Brockovich found that Cr(VI), used by PG&E, had contaminated the Hinckley, California, water supply.
(b) The Cr(VI) ion is often present in water as the polyatomic ions chromate, CrO (left), and dichromate, Cr O (right).
2−

4 2
2−

Chromium compounds are widely used in industry, such as for chrome plating, in dye-making, as preservatives, and to prevent
corrosion in cooling tower water, as occurred near Hinckley. In the environment, chromium exists primarily in either the
Cr(III) or Cr(VI) forms. Cr(III), an ingredient of many vitamin and nutritional supplements, forms compounds that are not very
soluble in water, and it has low toxicity. Cr(VI), on the other hand, is much more toxic and forms compounds that are
reasonably soluble in water. Exposure to small amounts of Cr(VI) can lead to damage of the respiratory, gastrointestinal, and
immune systems, as well as the kidneys, liver, blood, and skin.
Despite cleanup efforts, Cr(VI) groundwater contamination remains a problem in Hinckley and other locations across the
globe. A 2010 study by the Environmental Working Group found that of 35 US cities tested, 31 had higher levels of Cr(VI) in
their tap water than the public health goal of 0.02 parts per billion set by the California Environmental Protection Agency.

2.7.5: Molecular (Covalent) Compounds


The bonding characteristics of inorganic molecular compounds are different from ionic compounds, and they are named using a
different system as well. The charges of cations and anions dictate their ratios in ionic compounds, so specifying the names of the
ions provides sufficient information to determine chemical formulas. However, because covalent bonding allows for significant
variation in the combination ratios of the atoms in a molecule, the names for molecular compounds must explicitly identify these
ratios.

2.7.6: Compounds Composed of Two Elements


When two nonmetallic elements form a molecular compound, several combination ratios are often possible. For example, carbon
and oxygen can form the compounds CO and CO2. Since these are different substances with different properties, they cannot both
have the same name (they cannot both be called carbon oxide). To deal with this situation, we use a naming method that is
somewhat similar to that used for ionic compounds, but with added prefixes to specify the numbers of atoms of each element. The
name of the more metallic element (the one farther to the left and/or bottom of the periodic table) is first, followed by the name of
the more nonmetallic element (the one farther to the right and/or top) with its ending changed to the suffix –ide. The numbers of
atoms of each element are designated by the Greek prefixes shown in Table 2.7.5.
Table 2.7.5 : Nomenclature Prefixes
Number Prefix Number Prefix

1 (sometimes omitted) mono- 6 hexa-

2 di- 7 hepta-

3 tri- 8 octa-

4 tetra- 9 nona-

5 penta- 10 deca-

Access for free at OpenStax 2.7.4 https://chem.libretexts.org/@go/page/38144


When only one atom of the first element is present, the prefix mono- is usually deleted from that part. Thus, CO is named carbon
monoxide, and CO is called carbon dioxide. When two vowels are adjacent, the a in the Greek prefix is usually dropped. Some
2

other examples are shown in Table 2.7.6.


Table 2.7.6 : Names of Some Molecular Compounds Composed of Two Elements
Compound Name Compound Name

SO2 sulfur dioxide BCl3 boron trichloride

SO3 sulfur trioxide SF6 sulfur hexafluoride

NO2 nitrogen dioxide PF5 phosphorus pentafluoride

N2O4 dinitrogen tetroxide P4O10 tetraphosphorus decaoxide

N2O5 dinitrogen pentoxide IF7 iodine heptafluoride

There are a few common names that you will encounter as you continue your study of chemistry. For example, although NO is
often called nitric oxide, its proper name is nitrogen monoxide. Similarly, N2O is known as nitrous oxide even though our rules
would specify the name dinitrogen monoxide. (And H2O is usually called water, not dihydrogen monoxide.) You should commit to
memory the common names of compounds as you encounter them.

 Example 2.7.2: Naming Covalent Compounds

Name the following covalent compounds:


a. SF6
b. N2O3
c. Cl2O7
d. P4O6

Solution
Because these compounds consist solely of nonmetals, we use prefixes to designate the number of atoms of each element:
a. sulfur hexafluoride
b. dinitrogen trioxide
c. dichlorine heptoxide
d. tetraphosphorus hexoxide

 Exercise 2.7.2

Write the formulas for the following compounds:


a. phosphorus pentachloride
b. dinitrogen monoxide
c. iodine heptafluoride
d. carbon tetrachloride

Answer a
PCl5
Answer b
N2O
Answer c
IF7
Answer d
CCl4

Access for free at OpenStax 2.7.5 https://chem.libretexts.org/@go/page/38144


2.7.7: Binary Acids
Some compounds containing hydrogen are members of an important class of substances known as acids. The chemistry of these
compounds is explored in more detail in later chapters of this text, but for now, it will suffice to note that many acids release
hydrogen ions, H+, when dissolved in water. To denote this distinct chemical property, a mixture of water with an acid is given a
name derived from the compound’s name. If the compound is a binary acid (comprised of hydrogen and one other nonmetallic
element):
1. The word “hydrogen” is changed to the prefix hydro-
2. The other nonmetallic element name is modified by adding the suffix -ic
3. The word “acid” is added as a second word
For example, when the gas HCl (hydrogen chloride) is dissolved in water, the solution is called hydrochloric acid. Several other
examples of this nomenclature are shown in Table 2.7.7.
Table 2.7.7 : Names of Some Simple Acids
Name of Gas Name of Acid

HF(g), hydrogen fluoride HF(aq), hydrofluoric acid

HCl(g), hydrogen chloride HCl(aq), hydrochloric acid

HBr(g), hydrogen bromide HBr(aq), hydrobromic acid

HI(g), hydrogen iodide HI(aq), hydroiodic acid

H2S(g), hydrogen sulfide H2S(aq), hydrosulfuric acid

2.7.8: Oxyacids
Many compounds containing three or more elements (such as organic compounds or coordination compounds) are subject to
specialized nomenclature rules that you will learn later. However, we will briefly discuss the important compounds known as
oxyacids, compounds that contain hydrogen, oxygen, and at least one other element, and are bonded in such a way as to impart
acidic properties to the compound (you will learn the details of this in a later chapter). Typical oxyacids consist of hydrogen
combined with a polyatomic, oxygen-containing ion. To name oxyacids:
1. Omit “hydrogen”
2. Start with the root name of the anion
3. Replace –ate with –ic, or –ite with –ous
4. Add “acid”
For example, consider H2CO3 (which you might be tempted to call “hydrogen carbonate”). To name this correctly, “hydrogen” is
omitted; the –ate of carbonate is replace with –ic; and acid is added—so its name is carbonic acid. Other examples are given in
Table 2.7.8. There are some exceptions to the general naming method (e.g., H2SO4 is called sulfuric acid, not sulfic acid, and
H2SO3 is sulfurous, not sulfous, acid).
Table 2.7.8 : Names of Common Oxyacids
Formula Anion Name Acid Name

HC2H3O2 acetate acetic acid

HNO3 nitrate nitric acid

HNO2 nitrite nitrous acid

HClO4 perchlorate perchloric acid

H2CO3 carbonate carbonic acid

H2SO4 sulfate sulfuric acid

H2SO3 sulfite sulfurous acid

Access for free at OpenStax 2.7.6 https://chem.libretexts.org/@go/page/38144


Formula Anion Name Acid Name

H3PO4 phosphate phosphoric acid

Summary
Chemists use nomenclature rules to clearly name compounds. Ionic and molecular compounds are named using somewhat-different
methods. Binary ionic compounds typically consist of a metal and a nonmetal. The name of the metal is written first, followed by
the name of the nonmetal with its ending changed to –ide. For example, K2O is called potassium oxide. If the metal can form ions
with different charges, a Roman numeral in parentheses follows the name of the metal to specify its charge. Thus, FeCl2 is iron(II)
chloride and FeCl3 is iron(III) chloride. Some compounds contain polyatomic ions; the names of common polyatomic ions should
be memorized. Molecular compounds can form compounds with different ratios of their elements, so prefixes are used to specify
the numbers of atoms of each element in a molecule of the compound. Examples include SF6, sulfur hexafluoride, and N2O4,
dinitrogen tetroxide. Acids are an important class of compounds containing hydrogen and having special nomenclature rules.
Binary acids are named using the prefix hydro-, changing the –ide suffix to –ic, and adding “acid;” HCl is hydrochloric acid.
Oxyacids are named by changing the ending of the anion (-ate to –ic, and -ite to -ous) and adding “acid;” H2CO3 is carbonic acid.

Glossary
binary acid
compound that contains hydrogen and one other element, bonded in a way that imparts acidic properties to the compound
(ability to release H+ ions when dissolved in water)

binary compound
compound containing two different elements.

oxyacid
compound that contains hydrogen, oxygen, and one other element, bonded in a way that imparts acidic properties to the
compound (ability to release H+ ions when dissolved in water)

nomenclature
system of rules for naming objects of interest

This page titled 2.7: Chemical Nomenclature is shared under a CC BY 4.0 license and was authored, remixed, and/or curated by OpenStax via
source content that was edited to the style and standards of the LibreTexts platform; a detailed edit history is available upon request.

Access for free at OpenStax 2.7.7 https://chem.libretexts.org/@go/page/38144


2.E: Atoms, Molecules, and Ions (Exercises)
2.E.1: 2.1: Early Ideas in Atomic Theory
In the following drawing, the green spheres represent atoms of a certain element. The purple spheres represent atoms of another
element. If the spheres of different elements touch, they are part of a single unit of a compound. The following chemical change
represented by these spheres may violate one of the ideas of Dalton’s atomic theory. Which one?

The starting materials consist of one green sphere and two purple spheres. The products consist of two green spheres and two
purple spheres. This violates Dalton’s postulate that that atoms are not created during a chemical change, but are merely
redistributed.
Which postulate of Dalton’s theory is consistent with the following observation concerning the weights of reactants and products?
When 100 grams of solid calcium carbonate is heated, 44 grams of carbon dioxide and 56 grams of calcium oxide are produced.
Identify the postulate of Dalton’s theory that is violated by the following observations: 59.95% of one sample of titanium dioxide is
titanium; 60.10% of a different sample of titanium dioxide is titanium.
This statement violates Dalton’s fourth postulate: In a given compound, the numbers of atoms of each type (and thus also the
percentage) always have the same ratio.
Samples of compound X, Y, and Z are analyzed, with results shown here.

Compound Description Mass of Carbon Mass of Hydrogen

clear, colorless, liquid with strong


X 1.776 g 0.148 g
odor

clear, colorless, liquid with strong


Y 1.974 g 0.329 g
odor

clear, colorless, liquid with strong


Z 7.812 g 0.651 g
odor

Do these data provide example(s) of the law of definite proportions, the law of multiple proportions, neither, or both? What do
these data tell you about compounds X, Y, and Z?

2.E.2: 2.2: Evolution of Atomic Theory

2.E.3: Exercises
1. The existence of isotopes violates one of the original ideas of Dalton’s atomic theory. Which one?
2. How are electrons and protons similar? How are they different?
3. How are protons and neutrons similar? How are they different?
4. Predict and test the behavior of α particles fired at a “plum pudding” model atom.
a. Predict the paths taken by α particles that are fired at atoms with a Thomson’s plum pudding model structure. Explain why
you expect the α particles to take these paths.
b. If α particles of higher energy than those in (a) are fired at plum pudding atoms, predict how their paths will differ from the
lower-energy α particle paths. Explain your reasoning.
c. Now test your predictions from (a) and (b). Open the Rutherford Scattering simulation and select the “Plum Pudding Atom”
tab. Set “Alpha Particles Energy” to “min,” and select “show traces.” Click on the gun to start firing α particles. Does this
match your prediction from (a)? If not, explain why the actual path would be that shown in the simulation. Hit the pause
button, or “Reset All.” Set “Alpha Particles Energy” to “max,” and start firing α particles. Does this match your prediction
from (b)? If not, explain the effect of increased energy on the actual paths as shown in the simulation.
5. Predict and test the behavior of α particles fired at a Rutherford atom model.

Access for free at OpenStax 2.E.1 https://chem.libretexts.org/@go/page/77590


a. (a) Predict the paths taken by α particles that are fired at atoms with a Rutherford atom model structure. Explain why you
expect the α particles to take these paths.
b. (b) If α particles of higher energy than those in (a) are fired at Rutherford atoms, predict how their paths will differ from the
lower-energy α particle paths. Explain your reasoning.
c. (c) Predict how the paths taken by the α particles will differ if they are fired at Rutherford atoms of elements other than gold.
What factor do you expect to cause this difference in paths, and why?
d. (d) Now test your predictions from (a), (b), and (c). Open the Rutherford Scattering simulation and select the “Rutherford
Atom” tab. Due to the scale of the simulation, it is best to start with a small nucleus, so select “20” for both protons and
neutrons, “min” for energy, show traces, and then start firing α particles. Does this match your prediction from (a)? If not,
explain why the actual path would be that shown in the simulation. Pause or reset, set energy to “max,” and start firing α
particles. Does this match your prediction from (b)? If not, explain the effect of increased energy on the actual path as
shown in the simulation. Pause or reset, select “40” for both protons and neutrons, “min” for energy, show traces, and fire
away. Does this match your prediction from (c)? If not, explain why the actual path would be that shown in the simulation.
Repeat this with larger numbers of protons and neutrons. What generalization can you make regarding the type of atom and
effect on the path of α particles? Be clear and specific.

2.E.3.1: Solutions
1 Dalton originally thought that all atoms of a particular element had identical properties, including mass. Thus, the concept of
isotopes, in which an element has different masses, was a violation of the original idea. To account for the existence of isotopes, the
second postulate of his atomic theory was modified to state that atoms of the same element must have identical chemical properties.
2 Both are subatomic particles that reside in an atom’s nucleus. Both have approximately the same mass. Protons are positively
charged, whereas neutrons are uncharged.
3 Both are subatomic particles that reside in an atom’s nucleus. Both have approximately the same mass. Protons are positively
charged, whereas neutrons are uncharged.
4. (a) The plum pudding model indicates that the positive charge is spread uniformly throughout the atom, so we expect the α
particles to (perhaps) be slowed somewhat by the positive-positive repulsion, but to follow straight-line paths (i.e., not to be
deflected) as they pass through the atoms. (b) Higher-energy α particles will be traveling faster (and perhaps slowed less) and will
also follow straight-line paths through the atoms. (c) The α particles followed straight-line paths through the plum pudding atom.
There was no apparent slowing of the α particles as they passed through the atoms.
5. (a) The Rutherford atom has a small, positively charged nucleus, so most α particles will pass through empty space far from the
nucleus and be undeflected. Those α particles that pass near the nucleus will be deflected from their paths due to positive-positive
repulsion. The more directly toward the nucleus the α particles are headed, the larger the deflection angle will be. (b) Higher-energy
α particles that pass near the nucleus will still undergo deflection, but the faster they travel, the less the expected angle of
deflection. (c) If the nucleus is smaller, the positive charge is smaller and the expected deflections are smaller—both in terms of
how closely the α particles pass by the nucleus undeflected and the angle of deflection. If the nucleus is larger, the positive charge
is larger and the expected deflections are larger—more α particles will be deflected, and the deflection angles will be larger. (d) The
paths followed by the α particles match the predictions from (a), (b), and (c).

2.E.4: 2.3: Atomic Structure and Symbolism


In what way are isotopes of a given element always different? In what way(s) are they always the same?
Write the symbol for each of the following ions:
1. (a) the ion with a 1+ charge, atomic number 55, and mass number 133
2. (b) the ion with 54 electrons, 53 protons, and 74 neutrons
3. (c) the ion with atomic number 15, mass number 31, and a 3− charge
4. (d) the ion with 24 electrons, 30 neutrons, and a 3+ charge
1.
(a) 133Cs+; (b) 127I−; (c) 31P3−; (d) 57Co3+
Write the symbol for each of the following ions:
1. (a) the ion with a 3+ charge, 28 electrons, and a mass number of 71

Access for free at OpenStax 2.E.2 https://chem.libretexts.org/@go/page/77590


2. (b) the ion with 36 electrons, 35 protons, and 45 neutrons
3. (c) the ion with 86 electrons, 142 neutrons, and a 4+ charge
4. (d) the ion with a 2+ charge, atomic number 38, and mass number 87
Open the Build an Atom simulation and click on the Atom icon.
1. (a) Pick any one of the first 10 elements that you would like to build and state its symbol.
2. (b) Drag protons, neutrons, and electrons onto the atom template to make an atom of your element. State the numbers of
protons, neutrons, and electrons in your atom, as well as the net charge and mass number.
3. (c) Click on “Net Charge” and “Mass Number,” check your answers to (b), and correct, if needed.
4. (d) Predict whether your atom will be stable or unstable. State your reasoning.
5. (e) Check the “Stable/Unstable” box. Was your answer to (d) correct? If not, first predict what you can do to make a stable atom
of your element, and then do it and see if it works. Explain your reasoning.
(a) Carbon-12, 12C; (b) This atom contains six protons and six neutrons. There are six electrons in a neutral 12C atom. The net
charge of such a neutral atom is zero, and the mass number is 12. (c) The preceding answers are correct. (d) The atom will be stable
since C-12 is a stable isotope of carbon. (e) The preceding answer is correct. Other answers for this exercise are possible if a
different element of isotope is chosen.
Open the Build an Atom simulation
(a) Drag protons, neutrons, and electrons onto the atom template to make a neutral atom of Oxygen-16 and give the isotope symbol
for this atom.
(b) Now add two more electrons to make an ion and give the symbol for the ion you have created.
Open the Build an Atom simulation
(a) Drag protons, neutrons, and electrons onto the atom template to make a neutral atom of Lithium-6 and give the isotope symbol
for this atom.
(b) Now remove one electron to make an ion and give the symbol for the ion you have created.
(a) Lithium-6 contains three protons, three neutrons, and three electrons. The isotope symbol is 6Li or 6
3
Li . (b) 6Li+ or 6
3
Li
+

Determine the number of protons, neutrons, and electrons in the following isotopes that are used in medical diagnoses:
(a) atomic number 9, mass number 18, charge of 1−
(b) atomic number 43, mass number 99, charge of 7+
(c) atomic number 53, atomic mass number 131, charge of 1−
(d) atomic number 81, atomic mass number 201, charge of 1+
(e) Name the elements in parts (a), (b), (c), and (d).
The following are properties of isotopes of two elements that are essential in our diet. Determine the number of protons, neutrons
and electrons in each and name them.
(a) atomic number 26, mass number 58, charge of 2+
(b) atomic number 53, mass number 127, charge of 1−
(a) Iron, 26 protons, 24 electrons, and 32 neutrons; (b) iodine, 53 protons, 54 electrons, and 74 neutrons
Give the number of protons, electrons, and neutrons in neutral atoms of each of the following isotopes:
(a) 10
5
B

(b) 199
80
Hg

(c) 63
29
Cu

(d) 13
6
C

(e) 77
34
Se

Give the number of protons, electrons, and neutrons in neutral atoms of each of the following isotopes:

Access for free at OpenStax 2.E.3 https://chem.libretexts.org/@go/page/77590


(a) 7
3
Li

(b) 125
52
Te

(c) 109
47
Ag

(d) 15
7
N

(e) 31
15
P

(a) 3 protons, 3 electrons, 4 neutrons; (b) 52 protons, 52 electrons, 73 neutrons; (c) 47 protons, 47 electrons, 62 neutrons; (d) 7
protons, 7 electrons, 8 neutrons; (e) 15 protons, 15 electrons, 16 neutrons
Click on the site and select the “Mix Isotopes” tab, hide the “Percent Composition” and “Average Atomic Mass” boxes, and then
select the element boron.
(a) Write the symbols of the isotopes of boron that are shown as naturally occurring in significant amounts.
(b) Predict the relative amounts (percentages) of these boron isotopes found in nature. Explain the reasoning behind your choice.
(c) Add isotopes to the black box to make a mixture that matches your prediction in (b). You may drag isotopes from their bins or
click on “More” and then move the sliders to the appropriate amounts.
(d) Reveal the “Percent Composition” and “Average Atomic Mass” boxes. How well does your mixture match with your
prediction? If necessary, adjust the isotope amounts to match your prediction.
(e) Select “Nature’s” mix of isotopes and compare it to your prediction. How well does your prediction compare with the naturally
occurring mixture? Explain. If necessary, adjust your amounts to make them match “Nature’s” amounts as closely as possible.
Repeat Exercise using an element that has three naturally occurring isotopes.
Let us use neon as an example. Since there are three isotopes, there is no way to be sure to accurately predict the abundances to
make the total of 20.18 amu average atomic mass. Let us guess that the abundances are 9% Ne-22, 91% Ne-20, and only a trace of
Ne-21. The average mass would be 20.18 amu. Checking the nature’s mix of isotopes shows that the abundances are 90.48% Ne-
20, 9.25% Ne-22, and 0.27% Ne-21, so our guessed amounts have to be slightly adjusted.
An element has the following natural abundances and isotopic masses: 90.92% abundance with 19.99 amu, 0.26% abundance with
20.99 amu, and 8.82% abundance with 21.99 amu. Calculate the average atomic mass of this element.
Average atomic masses listed by IUPAC are based on a study of experimental results. Bromine has two isotopes 79Br and 81Br,
whose masses (78.9183 and 80.9163 amu) and abundances (50.69% and 49.31%) were determined in earlier experiments.
Calculate the average atomic mass of bromine based on these experiments.
79.904 amu
Variations in average atomic mass may be observed for elements obtained from different sources. Lithium provides an example of
this. The isotopic composition of lithium from naturally occurring minerals is 7.5% 6Li and 92.5% 7Li, which have masses of
6.01512 amu and 7.01600 amu, respectively. A commercial source of lithium, recycled from a military source, was 3.75% 6Li (and
the rest 7Li). Calculate the average atomic mass values for each of these two sources.
The average atomic masses of some elements may vary, depending upon the sources of their ores. Naturally occurring boron
consists of two isotopes with accurately known masses (10B, 10.0129 amu and 11B, 11.0931 amu). The actual atomic mass of boron
can vary from 10.807 to 10.819, depending on whether the mineral source is from Turkey or the United States. Calculate the
percent abundances leading to the two values of the average atomic masses of boron from these two countries.
Turkey source: 0.2649 (of 10.0129 amu isotope); US source: 0.2537 (of 10.0129 amu isotope)
The 18O:16O abundance ratio in some meteorites is greater than that used to calculate the average atomic mass of oxygen on earth.
Is the average mass of an oxygen atom in these meteorites greater than, less than, or equal to that of a terrestrial oxygen atom?

2.E.5: 2.4: Chemical Formulas


Explain why the symbol for an atom of the element oxygen and the formula for a molecule of oxygen differ.
The symbol for the element oxygen, O, represents both the element and one atom of oxygen. A molecule of oxygen, O2, contains
two oxygen atoms; the subscript 2 in the formula must be used to distinguish the diatomic molecule from two single oxygen atoms.

Access for free at OpenStax 2.E.4 https://chem.libretexts.org/@go/page/77590


Explain why the symbol for the element sulfur and the formula for a molecule of sulfur differ.
Write the molecular and empirical formulas of the following compounds:
(a)

(b)

(c)

(d)

(a) molecular CO2, empirical CO2; (b) molecular C2H2, empirical CH; (c) molecular C2H4, empirical CH2; (d) molecular H2SO4,
empirical H2SO4
Write the molecular and empirical formulas of the following compounds:
(a)

(b)

(c)

(d)

Access for free at OpenStax 2.E.5 https://chem.libretexts.org/@go/page/77590


Determine the empirical formulas for the following compounds:
1. (a) caffeine, C8H10N4O2
2. (b) fructose, C12H22O11
3. (c) hydrogen peroxide, H2O2
4. (d) glucose, C6H12O6
5. (e) ascorbic acid (vitamin C), C6H8O6
(a) C4H5N2O; (b) C12H22O11; (c) HO; (d) CH2O; (e) C3H4O3
Determine the empirical formulas for the following compounds:
1. (a) acetic acid, C2H4O2
2. (b) citric acid, C6H8O7
3. (c) hydrazine, N2H4
4. (d) nicotine, C10H14N2
5. (e) butane, C4H10
Write the empirical formulas for the following compounds:
(a)

(b)

(a) CH2O; (b) C2H4O


Open the Build a Molecule simulation and select the “Larger Molecules” tab. Select an appropriate atoms “Kit” to build a molecule
with two carbon and six hydrogen atoms. Drag atoms into the space above the “Kit” to make a molecule. A name will appear when
you have made an actual molecule that exists (even if it is not the one you want). You can use the scissors tool to separate atoms if
you would like to change the connections. Click on “3D” to see the molecule, and look at both the space-filling and ball-and-stick
possibilities.
1. (a) Draw the structural formula of this molecule and state its name.
2. (b) Can you arrange these atoms in any way to make a different compound?
Use the Build a Molecule simulation to repeat Exercise, but build a molecule with two carbons, six hydrogens, and one oxygen.
1. (a) Draw the structural formula of this molecule and state its name.
2. (b) Can you arrange these atoms to make a different molecule? If so, draw its structural formula and state its name.
3. (c) How are the molecules drawn in (a) and (b) the same? How do they differ? What are they called (the type of relationship
between these molecules, not their names).
(a) ethanol

Access for free at OpenStax 2.E.6 https://chem.libretexts.org/@go/page/77590


(b) methoxymethane, more commonly known as dimethyl ether

(c) These molecules have the same chemical composition (types and number of atoms) but different chemical structures. They are
structural isomers.
Use the Build a Molecule simulation to repeat Exercise, but build a molecule with three carbons, seven hydrogens, and one
chlorine.
a. Draw the structural formula of this molecule and state its name.
b. Can you arrange these atoms to make a different molecule? If so, draw its structural formula and state its name.
c. How are the molecules drawn in (a) and (b) the same? How do they differ? What are they called (the type of relationship
between these molecules, not their names)?

2.E.6: 2.5: The Periodic Table


Using the periodic table, classify each of the following elements as a metal or a nonmetal, and then further classify each as a main-
group (representative) element, transition metal, or inner transition metal:
a. uranium
b. bromine
c. strontium
d. neon
e. gold
f. americium
g. rhodium
h. sulfur
i. carbon
j. potassium
(a) metal, inner transition metal; (b) nonmetal, representative element; (c) metal, representative element; (d) nonmetal,
representative element; (e) metal, transition metal; (f) metal, inner transition metal; (g) metal, transition metal; (h) nonmetal,
representative element; (i) nonmetal, representative element; (j) metal, representative element
Using the periodic table, classify each of the following elements as a metal or a nonmetal, and then further classify each as a main-
group (representative) element, transition metal, or inner transition metal:
1. (a) cobalt
2. (b) europium
3. (c) iodine
4. (d) indium
5. (e) lithium
6. (f) oxygen
7. (g) cadmium
8. (h) terbium
9. (i) rhenium
Using the periodic table, identify the lightest member of each of the following groups:

Access for free at OpenStax 2.E.7 https://chem.libretexts.org/@go/page/77590


1. (a) noble gases
2. (b) alkaline earth metals
3. (c) alkali metals
4. (d) chalcogens
(a) He; (b) Be; (c) Li; (d) O
Using the periodic table, identify the heaviest member of each of the following groups:
1. (a) alkali metals
2. (b) chalcogens
3. (c) noble gases
4. (d) alkaline earth metals
1. Use the periodic table to give the name and symbol for each of the following elements:
2. (a) the noble gas in the same period as germanium
3. (b) the alkaline earth metal in the same period as selenium
4. (c) the halogen in the same period as lithium
5. (d) the chalcogen in the same period as cadmium
(a) krypton, Kr; (b) calcium, Ca; (c) fluorine, F; (d) tellurium, Te
Use the periodic table to give the name and symbol for each of the following elements:
1. (a) the halogen in the same period as the alkali metal with 11 protons
2. (b) the alkaline earth metal in the same period with the neutral noble gas with 18 electrons
3. (c) the noble gas in the same row as an isotope with 30 neutrons and 25 protons
4. (d) the noble gas in the same period as gold
Write a symbol for each of the following neutral isotopes. Include the atomic number and mass number for each.
1. (a) the alkali metal with 11 protons and a mass number of 23
2. (b) the noble gas element with and 75 neutrons in its nucleus and 54 electrons in the neutral atom
3. (c) the isotope with 33 protons and 40 neutrons in its nucleus
4. (d) the alkaline earth metal with 88 electrons and 138 neutrons
(a) 23
11
Na ; (b) 129
54
Xe ; (c) 73
33
As ; (d) 226
88
Ra

Write a symbol for each of the following neutral isotopes. Include the atomic number and mass number for each.
1. (a) the chalcogen with a mass number of 125
2. (b) the halogen whose longest-lived isotope is radioactive
3. (c) the noble gas, used in lighting, with 10 electrons and 10 neutrons
4. (d) the lightest alkali metal with three neutrons

2.E.7: 2.6: Molecular and Ionic Compounds


Using the periodic table, predict whether the following chlorides are ionic or covalent: KCl, NCl3, ICl, MgCl2, PCl5, and CCl4.
Ionic: KCl, MgCl2; Covalent: NCl3, ICl, PCl5, CCl4
Using the periodic table, predict whether the following chlorides are ionic or covalent: SiCl4, PCl3, CaCl2, CsCl, CuCl2, and CrCl3.
For each of the following compounds, state whether it is ionic or covalent. If it is ionic, write the symbols for the ions involved:
1. (a) NF3
2. (b) BaO,
3. (c) (NH4)2CO3
4. (d) Sr(H2PO4)2
5. (e) IBr
6. (f) Na2O
(a) covalent; (b) ionic, Ba2+, O2−; (c) ionic, NH , CO ; (d) ionic, Sr2+, H
+

4
2−
3 2
PO

4
; (e) covalent; (f) ionic, Na+, O2−

Access for free at OpenStax 2.E.8 https://chem.libretexts.org/@go/page/77590


For each of the following compounds, state whether it is ionic or covalent, and if it is ionic, write the symbols for the ions involved:
1. (a) KClO4
2. (b) MgC2H3O2
3. (c) H2S
4. (d) Ag2S
5. (e) N2Cl4
6. (f) Co(NO3)2
For each of the following pairs of ions, write the symbol for the formula of the compound they will form:
1. (a) Ca2+, S2−
2. (b) NH , SO
+

4
2−

3. (c) Al3+, Br−


4. (d) Na+, HPO 2−
4

5. (e) Mg2+, PO 3−
4

(a) CaS; (b) (NH4)2CO3; (c) AlBr3; (d) Na2HPO4; (e) Mg3 (PO4)2
For each of the following pairs of ions, write the symbol for the formula of the compound they will form:
1. (a) K+, O2−
2. (b) NH , PO
+
4
3−
4

3. (c) Al3+, O2−


4. (d) Na+, CO 2−

5. (e) Ba2+, PO 3−

2.E.8: 2.7: Chemical Nomenclature


Name the following compounds:
1. (a) CsCl
2. (b) BaO
3. (c) K2S
4. (d) BeCl2
5. (e) HBr
6. (f) AlF3
(a) cesium chloride; (b) barium oxide; (c) potassium sulfide; (d) beryllium chloride; (e) hydrogen bromide; (f) aluminum fluoride
Name the following compounds:
1. (a) NaF
2. (b) Rb2O
3. (c) BCl3
4. (d) H2Se
5. (e) P4O6
6. (f) ICl3
Write the formulas of the following compounds:
1. (a) rubidium bromide
2. (b) magnesium selenide
3. (c) sodium oxide
4. (d) calcium chloride
5. (e) hydrogen fluoride
6. (f) gallium phosphide
7. (g) aluminum bromide
8. (h) ammonium sulfate
(a) RbBr; (b) MgSe; (c) Na2O; (d) CaCl2; (e) HF; (f) GaP; (g) AlBr3; (h) (NH4)2SO4

Access for free at OpenStax 2.E.9 https://chem.libretexts.org/@go/page/77590


Write the formulas of the following compounds:
1. (a) lithium carbonate
2. (b) sodium perchlorate
3. (c) barium hydroxide
4. (d) ammonium carbonate
5. (e) sulfuric acid
6. (f) calcium acetate
7. (g) magnesium phosphate
8. (h) sodium sulfite
Write the formulas of the following compounds:
1. (a) chlorine dioxide
2. (b) dinitrogen tetraoxide
3. (c) potassium phosphide
4. (d) silver(I) sulfide
5. (e) aluminum nitride
6. (f) silicon dioxide
(a) ClO2; (b) N2O4; (c) K3P; (d) Ag2S; (e) AlN; (f) SiO2
Write the formulas of the following compounds:
1. (a) barium chloride
2. (b) magnesium nitride
3. (c) sulfur dioxide
4. (d) nitrogen trichloride
5. (e) dinitrogen trioxide
6. (f) tin(IV) chloride
Each of the following compounds contains a metal that can exhibit more than one ionic charge. Name these compounds:
1. (a) Cr2O3
2. (b) FeCl2
3. (c) CrO3
4. (d) TiCl4
5. (e) CoO
6. (f) MoS2
(a) chromium(III) oxide; (b) iron(II) chloride; (c) chromium(VI) oxide; (d) titanium(IV) chloride; (e) cobalt(II) oxide; (f)
molybdenum(IV) sulfide
Each of the following compounds contains a metal that can exhibit more than one ionic charge. Name these compounds:
1. (a) NiCO3
2. (b) MoO3
3. (c) Co(NO3)2
4. (d) V2O5
5. (e) MnO2
6. (f) Fe2O3
The following ionic compounds are found in common household products. Write the formulas for each compound:
1. (a) potassium phosphate
2. (b) copper(II) sulfate
3. (c) calcium chloride
4. (d) titanium dioxide
5. (e) ammonium nitrate
6. (f) sodium bisulfate (the common name for sodium hydrogen sulfate)

Access for free at OpenStax 2.E.10 https://chem.libretexts.org/@go/page/77590


(a) K3PO4; (b) CuSO4; (c) CaCl2; (d) TiO2; (e) NH4NO3; (f) NaHSO4
The following ionic compounds are found in common household products. Name each of the compounds:
1. (a) Ca(H2PO4)2
2. (b) FeSO4
3. (c) CaCO3
4. (d) MgO
5. (e) NaNO2
6. (f) KI
What are the IUPAC names of the following compounds?
1. (a) manganese dioxide
2. (b) mercurous chloride (Hg2Cl2)
3. (c) ferric nitrate [Fe(NO3)3]
4. (d) titanium tetrachloride
5. (e) cupric bromide (CuBr2)
(a) manganese(IV) oxide; (b) mercury(I) chloride; (c) iron(III) nitrate; (d) titanium(IV) chloride; (e) copper(II) bromide

This page titled 2.E: Atoms, Molecules, and Ions (Exercises) is shared under a CC BY 4.0 license and was authored, remixed, and/or curated by
OpenStax via source content that was edited to the style and standards of the LibreTexts platform; a detailed edit history is available upon request.

Access for free at OpenStax 2.E.11 https://chem.libretexts.org/@go/page/77590


CHAPTER OVERVIEW
3: Composition of Substances and Solutions

A general chemistry Libretexts Textbook remixed and remastered from


OpenStax's textbook:
General Chemistry
Quantitative aspects of the composition of substances and mixtures are the subject of this chapter.
3.1: Formula Mass and the Mole Concept
3.2: Determining Empirical and Molecular Formulas
3.3: Molarity
3.4: Other Units for Solution Concentrations
3.E: Composition of Substances and Solutions (Exercises)

This page titled 3: Composition of Substances and Solutions is shared under a CC BY 4.0 license and was authored, remixed, and/or curated by
OpenStax via source content that was edited to the style and standards of the LibreTexts platform; a detailed edit history is available upon request.

1
3.1: Formula Mass and the Mole Concept
 Learning Objectives
Calculate formula masses for covalent and ionic compounds
Define the amount unit mole and the related quantity Avogadro’s number
Explain the relation between mass, moles, and numbers of atoms or molecules, and perform calculations deriving these
quantities from one another

We can argue that modern chemical science began when scientists started exploring the quantitative as well as the qualitative
aspects of chemistry. For example, Dalton’s atomic theory was an attempt to explain the results of measurements that allowed him
to calculate the relative masses of elements combined in various compounds. Understanding the relationship between the masses of
atoms and the chemical formulas of compounds allows us to quantitatively describe the composition of substances.

3.1.1: Formula Mass


In an earlier chapter, we described the development of the atomic mass unit, the concept of average atomic masses, and the use of
chemical formulas to represent the elemental makeup of substances. These ideas can be extended to calculate the formula mass of a
substance by summing the average atomic masses of all the atoms represented in the substance’s formula.

3.1.1.1: Formula Mass for Covalent Substances


For covalent substances, the formula represents the numbers and types of atoms composing a single molecule of the substance;
therefore, the formula mass may be correctly referred to as a molecular mass. Consider chloroform (CHCl3), a covalent compound
once used as a surgical anesthetic and now primarily used in the production of tetrafluoroethylene, the building block for the “anti-
stick” polymer, Teflon. The molecular formula of chloroform indicates that a single molecule contains one carbon atom, one
hydrogen atom, and three chlorine atoms. The average molecular mass of a chloroform molecule is therefore equal to the sum of
the average atomic masses of these atoms. Figure 3.1.1 outlines the calculations used to derive the molecular mass of chloroform,
which is 119.37 amu.

Figure 3.1.1 : The average mass of a chloroform molecule, CHCl3, is 119.37 amu, which is the sum of the average atomic masses of
each of its constituent atoms. The model shows the molecular structure of chloroform.
A table and diagram are shown. The table is made up of six columns and five rows. The header row reads: “Element,” “Quantity,”
a blank space, “Average atomic mass (a m u),” a blank space, and “Subtotal (a m u).” The first column contains the symbols “C,”
“H,” “C l” and a blank, merged cell that runs the width of the first five columns. The second column contains the numbers “1,” “1,”
and “3” as well as the merged cell. The third column contains the multiplication symbol in each cell except for the last, merged
cell. The fourth column contains the numbers “12.01,” “1.008,” and “35.45” as well as the merged cell. The fifth column contains
the symbol “=” in each cell except for the last, merged cell. The sixth column contains the values “12.01,” “1.008,” “106.35,” and
“119.37.” There is a thick black line below the number 106.35. The merged cell under the first five columns reads “Molecular
mass.” To the left of the table is a diagram of a molecule. Three green spheres are attached to a slightly smaller black sphere, which
is also attached to a smaller white sphere. The green spheres lie beneath and to the sides of the black sphere while the white sphere
is located straight up from the black sphere.
Likewise, the molecular mass of an aspirin molecule, C9H8O4, is the sum of the atomic masses of nine carbon atoms, eight
hydrogen atoms, and four oxygen atoms, which amounts to 180.15 amu (Figure 3.1.2).

Access for free at OpenStax 3.1.1 https://chem.libretexts.org/@go/page/38147


Figure 3.1.2 : The average mass of an aspirin molecule is 180.15 amu. The model shows the molecular structure of aspirin,
C9H8O4.
A table and diagram are shown. The table is made up of six columns and five rows. The header row reads: “Element,” “Quantity,”
a blank space, “Average atomic mass (a m u),” a blank space, and “Subtotal (a m u).” The first column contains the symbols “C,”
“H,” “O,” and a merged cell. The merged cell runs the length of the first five columns. The second column contains the numbers
“9,” “8,” and “4” as well as the merged, cell. The third column contains the multiplication symbol in each cell except for the last,
merged cell. The fourth column contains the numbers “12.01,” “1.008,” and “16.00” as well as the merged cell. The fifth column
contains the symbol “=” in each cell except for the last, merged cell. The sixth column contains the values: “108.09,” “8.064,”
“64.00,” and “180.15.” There is a thick black line below the number 64.00. The merged cell under the first five columns reads
“Molecular mass.” To the left of the table is a diagram of a molecule. Six black spheres are located in a six-sided ring and
connected by alternating double and single black bonds. Attached to each of the four black spheres is one smaller white sphere.
Attached to the farthest right black sphere is a red sphere, connected to two more black spheres, all in a row. Attached to the last
black sphere of that row are two more white spheres. Attached to the first black sphere of that row is another red sphere. A black
sphere, attached to two red spheres and a white sphere is attached to the black sphere on the top right of the six-sided ring.

 Example 3.1.1: Computing Molecular Mass for a Covalent Compound

Ibuprofen, C13H18O2, is a covalent compound and the active ingredient in several popular nonprescription pain medications,
such as Advil and Motrin. What is the molecular mass (amu) for this compound?

Solution
Molecules of this compound are comprised of 13 carbon atoms, 18 hydrogen atoms, and 2 oxygen atoms. Following the
approach described above, the average molecular mass for this compound is therefore:

 Exercise 3.1.1

Acetaminophen, C8H9NO2, is a covalent compound and the active ingredient in several popular nonprescription pain
medications, such as Tylenol. What is the molecular mass (amu) for this compound?

Answer
151.16 amu

3.1.1.2: Formula Mass for Ionic Compounds


Ionic compounds are composed of discrete cations and anions combined in ratios to yield electrically neutral bulk matter. The
formula mass for an ionic compound is calculated in the same way as the formula mass for covalent compounds: by summing the
average atomic masses of all the atoms in the compound’s formula. Keep in mind, however, that the formula for an ionic compound
does not represent the composition of a discrete molecule, so it may not correctly be referred to as the “molecular mass.”

Access for free at OpenStax 3.1.2 https://chem.libretexts.org/@go/page/38147


As an example, consider sodium chloride, NaCl, the chemical name for common table salt. Sodium chloride is an ionic compound
composed of sodium cations, Na+, and chloride anions, Cl−, combined in a 1:1 ratio. The formula mass for this compound is
computed as 58.44 amu (Figure 3.1.3).

Figure 3.1.3 : Table salt, NaCl, contains an array of sodium and chloride ions combined in a 1:1 ratio. Its formula mass is 58.44
amu.
A table and diagram are shown. The table is made up of six columns and four rows. The header row reads: “Element,” “Quantity,”
a blank space, “Average atomic mass (a m u),” a blank space and “Subtotal (a m u).” The first column contains the symbols “N a”,
“C l,” and a merged cell. The merged cell runs the length of the first five columns. The second column contains the numbers “1”
and “1” as well as the merged cell. The third column contains the multiplication symbol in each cell except for the last, merged
cell. The fourth column contains the numbers “22.99” and “35.45” as well as the merged cell. The fifth column contains the symbol
“=” in each cell except for the last, merged cell. The sixth column contains the values “22.99,” “35.45,” and “58.44.” There is a
thick black line below the number “35.45.” The merged cell under the first five columns reads “Formula mass.” To the left of the
table is a diagram of a chemical structure. The diagram shows green and purple spheres placed in an alternating pattern, making up
the corners of eight stacked cubes to form one larger cube. The green spheres are slightly smaller than the purple spheres.
Note that the average masses of neutral sodium and chlorine atoms were used in this computation, rather than the masses for
sodium cations and chlorine anions. This approach is perfectly acceptable when computing the formula mass of an ionic
compound. Even though a sodium cation has a slightly smaller mass than a sodium atom (since it is missing an electron), this
difference will be offset by the fact that a chloride anion is slightly more massive than a chloride atom (due to the extra electron).
Moreover, the mass of an electron is negligibly small with respect to the mass of a typical atom. Even when calculating the mass of
an isolated ion, the missing or additional electrons can generally be ignored, since their contribution to the overall mass is
negligible, reflected only in the nonsignificant digits that will be lost when the computed mass is properly rounded. The few
exceptions to this guideline are very light ions derived from elements with precisely known atomic masses.

 Example 3.1.2: Computing Formula Mass for an Ionic Compound

Aluminum sulfate, Al2(SO4)3, is an ionic compound that is used in the manufacture of paper and in various water purification
processes. What is the formula mass (amu) of this compound?

Solution
The formula for this compound indicates it contains Al3+ and SO42− ions combined in a 2:3 ratio. For purposes of computing a
formula mass, it is helpful to rewrite the formula in the simpler format, Al2S3O12. Following the approach outlined above, the
formula mass for this compound is calculated as follows:

 Exercise 3.1.2

Calcium phosphate, Ca (PO ) , is an ionic compound and a common anti-caking agent added to food products. What is the
3 4 2

formula mass (amu) of calcium phosphate?

Answer
310.18 amu

Access for free at OpenStax 3.1.3 https://chem.libretexts.org/@go/page/38147


3.1.2: The Mole
The identity of a substance is defined not only by the types of atoms or ions it contains, but by the quantity of each type of atom or
ion. For example, water, H O , and hydrogen peroxide, H O , are alike in that their respective molecules are composed of
2 2 2

hydrogen and oxygen atoms. However, because a hydrogen peroxide molecule contains two oxygen atoms, as opposed to the water
molecule, which has only one, the two substances exhibit very different properties. Today, we possess sophisticated instruments
that allow the direct measurement of these defining microscopic traits; however, the same traits were originally derived from the
measurement of macroscopic properties (the masses and volumes of bulk quantities of matter) using relatively simple tools
(balances and volumetric glassware). This experimental approach required the introduction of a new unit for amount of substances,
the mole, which remains indispensable in modern chemical science.
The mole is an amount unit similar to familiar units like pair, dozen, gross, etc. It provides a specific measure of the number of
atoms or molecules in a bulk sample of matter. A mole is defined as the amount of substance containing the same number of
discrete entities (such as atoms, molecules, and ions) as the number of atoms in a sample of pure 12C weighing exactly 12 g. One
Latin connotation for the word “mole” is “large mass” or “bulk,” which is consistent with its use as the name for this unit. The
mole provides a link between an easily measured macroscopic property, bulk mass, and an extremely important fundamental
property, number of atoms, molecules, and so forth.
The number of entities composing a mole has been experimentally determined to be 6.02214179 × 10 , a fundamental constant
23

named Avogadro’s number (N ) or the Avogadro constant in honor of Italian scientist Amedeo Avogadro. This constant is
A

properly reported with an explicit unit of “per mole,” a conveniently rounded version being 6.022 × 10 /mol. 23

Consistent with its definition as an amount unit, 1 mole of any element contains the same number of atoms as 1 mole of any other
element. The masses of 1 mole of different elements, however, are different, since the masses of the individual atoms are drastically
different. The molar mass of an element (or compound) is the mass in grams of 1 mole of that substance, a property expressed in
units of grams per mole (g/mol) (Figure 3.1.4).

Figure 3.1.4 : Each sample contains 6.022 × 10 atoms —1.00 mol of atoms. From left to right (top row): 65.4 g zinc, 12.0 g
23

carbon, 24.3 g magnesium, and 63.5 g copper. From left to right (bottom row): 32.1 g sulfur, 28.1 g silicon, 207 g lead, and 118.7 g
tin. (credit: modification of work by Mark Ott).
This figure contains eight different substances displayed on white circles. The amount of each substance is visibly different.
Because the definitions of both the mole and the atomic mass unit are based on the same reference substance, 12C, the molar mass
of any substance is numerically equivalent to its atomic or formula weight in amu. Per the amu definition, a single 12C atom weighs
12 amu (its atomic mass is 12 amu). According to the definition of the mole, 12 g of 12C contains 1 mole of 12C atoms (its molar
mass is 12 g/mol). This relationship holds for all elements, since their atomic masses are measured relative to that of the amu-
reference substance, 12C. Extending this principle, the molar mass of a compound in grams is likewise numerically equivalent to its
formula mass in amu (Figure 3.1.5).

Access for free at OpenStax 3.1.4 https://chem.libretexts.org/@go/page/38147


<div data-mt-source="1"
&quot;This
" height="292" width="437" src="/@api/deki/files/56173/CNX_Chem_03_02_compound.jpg">
Figure 3.1.5 : Each sample contains 6.022 × 10 molecules or formula units—1.00 mol of the compound or element. Clock-wise
23

from the upper left: 130.2 g of C8H17OH (1-octanol, formula mass 130.2 amu), 454.4 g of HgI2 (mercury(II) iodide, formula mass
454.4 amu), 32.0 g of CH3OH (methanol, formula mass 32.0 amu) and 256.5 g of S8 (sulfur, formula mass 256.5 amu). (credit:
Sahar Atwa).
Table 3.1.1 : Mass of one mole of elements
Element Average Atomic Mass (amu) Molar Mass (g/mol) Atoms/Mole

C 12.01 12.01 6.022 × 10


23

H 1.008 1.008 6.022 × 10


23

O 16.00 16.00 6.022 × 10


23

Na 22.99 22.99 6.022 × 10


23

Cl 33.45 35.45 6.022 × 10


23

While atomic mass and molar mass are numerically equivalent, keep in mind that they are vastly different in terms of scale, as
represented by the vast difference in the magnitudes of their respective units (amu versus g). To appreciate the enormity of the
mole, consider a small drop of water after a rainfall. Although this represents just a tiny fraction of 1 mole of water (~18 g), it
contains more water molecules than can be clearly imagined. If the molecules were distributed equally among the roughly seven
billion people on earth, each person would receive more than 100 billion molecules.

How big is a mole? (Not the animal, the …

Video 3.1.1 : The mole is used in chemistry to represent 6.022 × 10 of something, but it can be difficult to conceptualize such a
23

large number. Watch this video and then complete the “Think” questions that follow. Explore more about the mole by reviewing the
information under “Dig Deeper.”
The relationships between formula mass, the mole, and Avogadro’s number can be applied to compute various quantities that
describe the composition of substances and compounds. For example, if we know the mass and chemical composition of a
substance, we can determine the number of moles and calculate number of atoms or molecules in the sample. Likewise, if we know
the number of moles of a substance, we can derive the number of atoms or molecules and calculate the substance’s mass.

 Example 3.1.3: Deriving Moles from Grams for an Element

According to nutritional guidelines from the US Department of Agriculture, the estimated average requirement for dietary
potassium is 4.7 g. What is the estimated average requirement of potassium in moles?

Solution
The mass of K is provided, and the corresponding amount of K in moles is requested. Referring to the periodic table, the
atomic mass of K is 39.10 amu, and so its molar mass is 39.10 g/mol. The given mass of K (4.7 g) is a bit more than one-tenth
the molar mass (39.10 g), so a reasonable “ballpark” estimate of the number of moles would be slightly greater than 0.1 mol.

Access for free at OpenStax 3.1.5 https://chem.libretexts.org/@go/page/38147


The molar amount of a substance may be calculated by dividing its mass (g) by its molar mass (g/mol):

The factor-label method supports this mathematical approach since the unit “g” cancels and the answer has units of “mol:”

mol K
4.7 g K( ) = 0.12 mol K
39.10 g

The calculated magnitude (0.12 mol K) is consistent with our ballpark expectation, since it is a bit greater than 0.1 mol.

 Exercise 3.1.3: Beryllium

Beryllium is a light metal used to fabricate transparent X-ray windows for medical imaging instruments. How many moles of
Be are in a thin-foil window weighing 3.24 g?

Answer
0.360 mol

 Example 3.1.4: Deriving Grams from Moles for an Element

A liter of air contains 9.2 × 10


−4
mol argon. What is the mass of Ar in a liter of air?

Solution
The molar amount of Ar is provided and must be used to derive the corresponding mass in grams. Since the amount of Ar is
less than 1 mole, the mass will be less than the mass of 1 mole of Ar, approximately 40 g. The molar amount in question is
approximately one-one thousandth (~10−3) of a mole, and so the corresponding mass should be roughly one-one thousandth of
the molar mass (~0.04 g):

In this case, logic dictates (and the factor-label method supports) multiplying the provided amount (mol) by the molar mass
(g/mol):

39.95 g
−4
9.2 × 10 mol Ar ( ) = 0.037 g Ar
mol Ar

The result is in agreement with our expectations, around 0.04 g Ar.

 Exercise 3.1.4

What is the mass of 2.561 mol of gold?

Answer
504.4 g

Access for free at OpenStax 3.1.6 https://chem.libretexts.org/@go/page/38147


 Example 3.1.6: Deriving Number of Atoms from Mass for an Element
Copper is commonly used to fabricate electrical wire (Figure 3.1.6). How many copper atoms are in 5.00 g of copper wire?

Figure 3.1.6 : Copper wire is composed of many, many atoms of Cu. (credit: Emilian Robert Vicol)

Solution
The number of Cu atoms in the wire may be conveniently derived from its mass by a two-step computation: first calculating
the molar amount of Cu, and then using Avogadro’s number (NA) to convert this molar amount to number of Cu atoms:

Considering that the provided sample mass (5.00 g) is a little less than one-tenth the mass of 1 mole of Cu (~64 g), a
reasonable estimate for the number of atoms in the sample would be on the order of one-tenth NA, or approximately 1022 Cu
atoms. Carrying out the two-step computation yields:
23
mol Cu 6.022 × 10 atoms
22
5.00 g Cu ( )( ) = 4.74 × 10 atoms of copper
63.55 g mol

The factor-label method yields the desired cancellation of units, and the computed result is on the order of 1022 as expected.

 Exercise 3.1.6

A prospector panning for gold in a river collects 15.00 g of pure gold. How many Au atoms are in this quantity of gold?

Answer
4.586 × 10
22
Au atoms

 Example 3.1.7: Deriving Moles from Grams for a Compound

Our bodies synthesize protein from amino acids. One of these amino acids is glycine, which has the molecular formula
C2H5O2N. How many moles of glycine molecules are contained in 28.35 g of glycine?

Solution
We can derive the number of moles of a compound from its mass following the same procedure we used for an element in
Example 3.1.6:

Access for free at OpenStax 3.1.7 https://chem.libretexts.org/@go/page/38147


The molar mass of glycine is required for this calculation, and it is computed in the same fashion as its molecular mass. One
mole of glycine, C2H5O2N, contains 2 moles of carbon, 5 moles of hydrogen, 2 moles of oxygen, and 1 mole of nitrogen:

A table is shown that is made up of six columns and six rows. The header row reads: “Element,” “Quantity (mol element / mol
compound,” a blank space, “Molar mass (g / mol element),” a blank space, and “Subtotal (a m u).” The first column contains
the symbols “C,” “H,” “O,” “N,” and a merged cell. The merged cell runs the width of the first five columns. The second
column contains the numbers “2,” “5,” “2,” and “1” as well as the merged cell. The third column contains the multiplication
symbol in each cell except for the last, merged cell. The fourth column contains the numbers “12.01,” “1.008,” “16.00,” and
“14.007” as well as the merged cell. The fifth column contains the symbol “=” in each cell except for the last, merged cell. The
sixth column contains the values “24.02,” “5.040,” “32.00,” “14.007,” and “75.07.” There is a thick black line under the
number 14.007. The merged cell under the first five columns reads “Molar mass (g / mol compound). There is a ball-and-stick
drawing to the right of this table. It shows a black sphere that forms a double bond with a slightly smaller red sphere, a single
bond with another red sphere, and a single bond with another black sphere. The red sphere that forms a single bond with the
black sphere also forms a single bond with a smaller, white sphere. The second black sphere forms a single bond with a
smaller, white sphere and a smaller blue sphere. The blue sphere forms a single bond with two smaller, white spheres each.
The provided mass of glycine (~28 g) is a bit more than one-third the molar mass (~75 g/mol), so we would expect the
computed result to be a bit greater than one-third of a mole (~0.33 mol). Dividing the compound’s mass by its molar mass
yields:

mol glycine
28.35 g glycine ( ) = 0.378 mol glycine
75.07 g

This result is consistent with our rough estimate.

 Exercise 3.1.7

How many moles of sucrose, C 12 H22 O11 , are in a 25-g sample of sucrose?

Answer
0.073 mol

 Example 3.1.8: Deriving Grams from Moles for a Compound

Vitamin C is a covalent compound with the molecular formula C6H8O6. The recommended daily dietary allowance of vitamin
C for children aged 4–8 years is 1.42 × 10−4 mol. What is the mass of this allowance in grams?

Solution
As for elements, the mass of a compound can be derived from its molar amount as shown:

The molar mass for this compound is computed to be 176.124 g/mol. The given number of moles is a very small fraction of a
mole (~10−4 or one-ten thousandth); therefore, we would expect the corresponding mass to be about one-ten thousandth of the
molar mass (~0.02 g). Performing the calculation, we get:

Access for free at OpenStax 3.1.8 https://chem.libretexts.org/@go/page/38147


176.124 g
−4
1.42 × 10 mol vitamin C ( ) = 0.0250 g vitamin C
mol vitamin C

This is consistent with the anticipated result.

 Exercise 3.1.8

What is the mass of 0.443 mol of hydrazine, N 2 H4 ?

Answer
14.2 g

 Example 3.1.9: Deriving the Number of Molecules from the Compound Mass

A packet of an artificial sweetener contains 40.0 mg of saccharin (C7H5NO3S), which has the structural formula:

Given that saccharin has a molar mass of 183.18 g/mol, how many saccharin molecules are in a 40.0-mg (0.0400-g) sample of
saccharin? How many carbon atoms are in the same sample?

Solution
The number of molecules in a given mass of compound is computed by first deriving the number of moles, as demonstrated in
Example 3.1.8, and then multiplying by Avogadro’s number:

Using the provided mass and molar mass for saccharin yields:
23
mol C H NO S 6.022 × 10 C H NO S molecules
7 5 3 7 5 3
0.0400 g C H NO S ( )( )
7 5 3
183.18 g C H NO S 1 mol C H NO S
7 5 3
7 5 3

20
= 1.31 × 10 C H NO S molecules
7 5 3

The compound’s formula shows that each molecule contains seven carbon atoms, and so the number of C atoms in the
provided sample is:

20
7 C atoms 21
1.31 × 10 C H NO S molecules ( ) = 9.20 × 10 C atoms
7 5 3
1 C H NO S molecule
7 5 3

 Exercise 3.1.9

How many C 4 H10 molecules are contained in 9.213 g of this compound? How many hydrogen atoms?

Answer

Access for free at OpenStax 3.1.9 https://chem.libretexts.org/@go/page/38147


22
9.545 × 10 molecules C4 H10
23
9.545 × 10 atoms H

Summary
The formula mass of a substance is the sum of the average atomic masses of each atom represented in the chemical formula and is
expressed in atomic mass units. The formula mass of a covalent compound is also called the molecular mass. A convenient amount
unit for expressing very large numbers of atoms or molecules is the mole. Experimental measurements have determined the number
of entities composing 1 mole of substance to be 6.022 × 1023, a quantity called Avogadro’s number. The mass in grams of 1 mole of
substance is its molar mass. Due to the use of the same reference substance in defining the atomic mass unit and the mole, the
formula mass (amu) and molar mass (g/mol) for any substance are numerically equivalent (for example, one H2O molecule weighs
approximately18 amu and 1 mole of H2O molecules weighs approximately 18 g).

Footnotes
1. 1 Omiatek, Donna M., Amanda J. Bressler, Ann-Sofie Cans, Anne M. Andrews, Michael L. Heien, and Andrew G. Ewing. “The
Real Catecholamine Content of Secretory Vesicles in the CNS Revealed by Electrochemical Cytometry.” Scientific Report 3
(2013): 1447, accessed January 14, 2015, doi:10.1038/srep01447.

Glossary
Avogadro’s number (NA)
experimentally determined value of the number of entities comprising 1 mole of substance, equal to 6.022 × 1023 mol−1

formula mass
sum of the average masses for all atoms represented in a chemical formula; for covalent compounds, this is also the molecular
mass

mole
amount of substance containing the same number of atoms, molecules, ions, or other entities as the number of atoms in exactly
12 grams of 12C

molar mass
mass in grams of 1 mole of a substance

This page titled 3.1: Formula Mass and the Mole Concept is shared under a CC BY 4.0 license and was authored, remixed, and/or curated by
OpenStax via source content that was edited to the style and standards of the LibreTexts platform; a detailed edit history is available upon request.

Access for free at OpenStax 3.1.10 https://chem.libretexts.org/@go/page/38147


3.2: Determining Empirical and Molecular Formulas
 Learning Objectives
Compute the percent composition of a compound
Determine the empirical formula of a compound
Determine the molecular formula of a compound

In the previous section, we discussed the relationship between the bulk mass of a substance and the number of atoms or molecules
it contains (moles). Given the chemical formula of the substance, we were able to determine the amount of the substance (moles)
from its mass, and vice versa. But what if the chemical formula of a substance is unknown? In this section, we will explore how to
apply these very same principles in order to derive the chemical formulas of unknown substances from experimental mass
measurements.

3.2.1: Percent Composition


The elemental makeup of a compound defines its chemical identity, and chemical formulas are the most succinct way of
representing this elemental makeup. When a compound’s formula is unknown, measuring the mass of each of its constituent
elements is often the first step in the process of determining the formula experimentally. The results of these measurements permit
the calculation of the compound’s percent composition, defined as the percentage by mass of each element in the compound. For
example, consider a gaseous compound composed solely of carbon and hydrogen. The percent composition of this compound could
be represented as follows:
mass H
%H = × 100%
mass compound

mass C
%C = × 100%
mass compound

If analysis of a 10.0-g sample of this gas showed it to contain 2.5 g H and 7.5 g C, the percent composition would be calculated to
be 25% H and 75% C:
2.5 g H
%H = × 100% = 25%
10.0 g compound

7.5 g C
%C = × 100% = 75%
10.0 g compound

 Example 3.2.1: Calculation of Percent Composition


Analysis of a 12.04-g sample of a liquid compound composed of carbon, hydrogen, and nitrogen showed it to contain 7.34 g C,
1.85 g H, and 2.85 g N. What is the percent composition of this compound?

Solution
To calculate percent composition, we divide the experimentally derived mass of each element by the overall mass of the
compound, and then convert to a percentage:
7.34 g C
%C = × 100% = 61.0%
12.04 g compound

1.85 g H
%H = × 100% = 15.4%
12.04 g compound

2.85 g N
%N = × 100% = 23.7%
12.04 g compound

The analysis results indicate that the compound is 61.0% C, 15.4% H, and 23.7% N by mass.

Access for free at OpenStax 3.2.1 https://chem.libretexts.org/@go/page/38148


 Exercise 3.2.1

A 24.81-g sample of a gaseous compound containing only carbon, oxygen, and chlorine is determined to contain 3.01 g C, 4.00
g O, and 17.81 g Cl. What is this compound’s percent composition?

Answer
12.1% C, 16.1% O, 71.8% Cl

3.2.2: Determining Percent Composition from Formula Mass


Percent composition is also useful for evaluating the relative abundance of a given element in different compounds of known
formulas. As one example, consider the common nitrogen-containing fertilizers ammonia (NH3), ammonium nitrate (NH4NO3),
and urea (CH4N2O). The element nitrogen is the active ingredient for agricultural purposes, so the mass percentage of nitrogen in
the compound is a practical and economic concern for consumers choosing among these fertilizers. For these sorts of applications,
the percent composition of a compound is easily derived from its formula mass and the atomic masses of its constituent elements.
A molecule of NH3 contains one N atom weighing 14.01 amu and three H atoms weighing a total of (3 × 1.008 amu) = 3.024 amu.
The formula mass of ammonia is therefore (14.01 amu + 3.024 amu) = 17.03 amu, and its percent composition is:
14.01 amu N
%N = × 100% = 82.27%
17.03 amu NH3

3.024 amu N
%H = × 100% = 17.76%
17.03 amu NH3

This same approach may be taken considering a pair of molecules, a dozen molecules, or a mole of molecules, etc. The latter
amount is most convenient and would simply involve the use of molar masses instead of atomic and formula masses, as
demonstrated Example 3.2.2. As long as we know the chemical formula of the substance in question, we can easily derive percent
composition from the formula mass or molar mass.

 Example 3.2.2: Determining Percent Composition from a Molecular Formula

Aspirin is a compound with the molecular formula C9H8O4. What is its percent composition?

Solution
To calculate the percent composition, we need to know the masses of C, H, and O in a known mass of C9H8O4. It is convenient
to consider 1 mol of C9H8O4 and use its molar mass (180.159 g/mole, determined from the chemical formula) to calculate the
percentages of each of its elements:

9 mol C × molar mass C 9 × 12.01 g/mol 108.09 g/mol


%C = × 100 = 180.159 g/mol × 100 = × 100
molar mass C H O 180.159 g/mol
9 18 4

%C = 60.00 % C

8 mol H × molar mass H 8 × 1.008 g/mol 8.064 g/mol


%H = × 100 = 180.159 g/mol × 100 = × 100
molar mass C H O 180.159 g/mol
9 18 4

%H = 4.476 % H

4 mol O × molar mass O 4 × 16.00 g/mol 64.00 g/mol


%O = × 100 = 180.159 g/mol × 100 = × 100
molar mass C H O 180.159 g/mol
9 18 4

%O = 35.52%

Note that these percentages sum to equal 100.00% when appropriately rounded.

Access for free at OpenStax 3.2.2 https://chem.libretexts.org/@go/page/38148


 Exercise 3.2.2

To three significant digits, what is the mass percentage of iron in the compound F e 2 O3 ?

Answer
69.9% Fe

3.2.3: Determination of Empirical Formulas


As previously mentioned, the most common approach to determining a compound’s chemical formula is to first measure the
masses of its constituent elements. However, we must keep in mind that chemical formulas represent the relative numbers, not
masses, of atoms in the substance. Therefore, any experimentally derived data involving mass must be used to derive the
corresponding numbers of atoms in the compound. To accomplish this, we can use molar masses to convert the mass of each
element to a number of moles. We then consider the moles of each element relative to each other, converting these numbers into a
whole-number ratio that can be used to derive the empirical formula of the substance. Consider a sample of compound determined
to contain 1.71 g C and 0.287 g H. The corresponding numbers of atoms (in moles) are:
1 mol C
1.71 g C × = 0.142 mol C
12.01 g C

1 mol H
0.287 g H × = 0.284 mol H
1.008 g H

Thus, we can accurately represent this compound with the formula C0.142H0.284. Of course, per accepted convention, formulas
contain whole-number subscripts, which can be achieved by dividing each subscript by the smaller subscript:

C 0.142 H 0.284 or CH
2
0.142 0.142

(Recall that subscripts of “1” are not written, but rather assumed if no other number is present.)
The empirical formula for this compound is thus CH2. This may or not be the compound’s molecular formula as well; however, we
would need additional information to make that determination (as discussed later in this section).
Consider as another example a sample of compound determined to contain 5.31 g Cl and 8.40 g O. Following the same approach
yields a tentative empirical formula of:

C l0.150O0.525 = C l 0.150 O 0.525 = Cl O3.5


0.150 0.150

In this case, dividing by the smallest subscript still leaves us with a decimal subscript in the empirical formula. To convert this into
a whole number, we must multiply each of the subscripts by two, retaining the same atom ratio and yielding Cl2O7 as the final
empirical formula.

 Procedure

In summary, empirical formulas are derived from experimentally measured element masses by:
1. Deriving the number of moles of each element from its mass
2. Dividing each element’s molar amount by the smallest molar amount to yield subscripts for a tentative empirical formula
3. Multiplying all coefficients by an integer, if necessary, to ensure that the smallest whole-number ratio of subscripts is
obtained

Figure 3.2.1 outlines this procedure in flow chart fashion for a substance containing elements A and X.

Access for free at OpenStax 3.2.3 https://chem.libretexts.org/@go/page/38148


Figure 3.2.1 : The empirical formula of a compound can be derived from the masses of all elements in the sample.
A flow chart is shown that is composed of six boxes, two of which are connected together by a right facing arrow and located
above two more that are also connected by a right-facing arrow. These two rows of boxes are connected vertically by a line that
leads to a right-facing arrow and the last two boxes, connected by a final right facing arrow. The first two upper boxes have the
phrases, “Mass of A atoms” and “Moles of A atoms” respectively, while the arrow that connects them has the phrase, “Divide by
molar mass,” written below it. The second two bottom boxes have the phrases, “Mass of X atoms” and “Moles of X atoms”
respectively, while the arrow that connects them has the phrase, “Divide by molar mass” written below it. The arrow that connects
the upper and lower boxes to the last two boxes has the phrase “Divide by lowest number of moles” written below it. The last two
boxes have the phrases, “A to X mole ratio” and “Empirical formula” respectively, while the arrow that connects them has the
phrase, “Convert ratio to lowest whole numbers” written below it.

 Example 3.2.3: Determining an Empirical Formula from Masses of Elements

A sample of the black mineral hematite (Figure 3.2.2), an oxide of iron found in many iron ores, contains 34.97 g of iron and
15.03 g of oxygen. What is the empirical formula of hematite?

Figure 3.2.2 : Hematite is an iron oxide that is used in jewelry. (credit: Mauro Cateb)

Solution
For this problem, we are given the mass in grams of each element. Begin by finding the moles of each:
mol Fe
34.97 g Fe ( ) = 0.6261 mol Fe
55.85 g

mol O
15.03 g O ( ) = 0.9394 mol O
16.00 g

Next, derive the iron-to-oxygen molar ratio by dividing by the lesser number of moles:
0.6261
= 1.000 mol Fe
0.6261

0.9394
= 1.500 mol O
0.6261

The ratio is 1.000 mol of iron to 1.500 mol of oxygen (Fe1O1.5). Finally, multiply the ratio by two to get the smallest possible
whole number subscripts while still maintaining the correct iron-to-oxygen ratio:

2(F e1 O1.5 ) = F e2 O3

The empirical formula is F e 2 O3 .

Access for free at OpenStax 3.2.4 https://chem.libretexts.org/@go/page/38148


 Exercise 3.2.3

What is the empirical formula of a compound if a sample contains 0.130 g of nitrogen and 0.370 g of oxygen?

Answer
N2 O5

Calculating Percent Composition and E…


E…

Video 3.2.1 : Additional worked examples illustrating the derivation of empirical formulas are presented in the brief video clip.

3.2.4: Deriving Empirical Formulas from Percent Composition


Finally, with regard to deriving empirical formulas, consider instances in which a compound’s percent composition is available
rather than the absolute masses of the compound’s constituent elements. In such cases, the percent composition can be used to
calculate the masses of elements present in any convenient mass of compound; these masses can then be used to derive the
empirical formula in the usual fashion.

 Example 3.2.4: Determining an Empirical Formula from Percent Composition

The bacterial fermentation of grain to produce ethanol forms a gas with a percent composition of 27.29% C and 72.71% O
(Figure 3.2.3). What is the empirical formula for this gas?

Figure 3.2.3 : An oxide of carbon is removed from these fermentation tanks through the large copper pipes at the top. (credit:
“Dual Freq”/Wikimedia Commons)
A picture is shown of four copper-colored industrial containers with a large pipe connecting to the top of each one.

Solution
Since the scale for percentages is 100, it is most convenient to calculate the mass of elements present in a sample weighing 100
g. The calculation is “most convenient” because, per the definition for percent composition, the mass of a given element in
grams is numerically equivalent to the element’s mass percentage. This numerical equivalence results from the definition of the

Access for free at OpenStax 3.2.5 https://chem.libretexts.org/@go/page/38148


“percentage” unit, whose name is derived from the Latin phrase per centum meaning “by the hundred.” Considering this
definition, the mass percentages provided may be more conveniently expressed as fractions:
27.29 g C
27.29 % C =
100 g compound

72.71 g O
72.71 % O =
100 g compound

The molar amounts of carbon and hydrogen in a 100-g sample are calculated by dividing each element’s mass by its molar mass:
mol C
27.29 g C ( ) = 2.272 mol C
12.01 g

mol O
72.71 g O ( ) = 4.544 mol O
16.00 g

Coefficients for the tentative empirical formula are derived by dividing each molar amount by the lesser of the two:
2.272 mol C
=1
2.272

4.544 mol O
=2
2.272

Since the resulting ratio is one carbon to two oxygen atoms, the empirical formula is CO2.

 Exercise 3.2.4

What is the empirical formula of a compound containing 40.0% C, 6.71% H, and 53.28% O?

Answer
C H2 O

3.2.5: Derivation of Molecular Formulas


Recall that empirical formulas are symbols representing the relative numbers of a compound’s elements. Determining the absolute
numbers of atoms that compose a single molecule of a covalent compound requires knowledge of both its empirical formula and its
molecular mass or molar mass. These quantities may be determined experimentally by various measurement techniques. Molecular
mass, for example, is often derived from the mass spectrum of the compound (see discussion of this technique in the previous
chapter on atoms and molecules). Molar mass can be measured by a number of experimental methods, many of which will be
introduced in later chapters of this text.
Molecular formulas are derived by comparing the compound’s molecular or molar mass to its empirical formula mass. As the name
suggests, an empirical formula mass is the sum of the average atomic masses of all the atoms represented in an empirical formula.
If we know the molecular (or molar) mass of the substance, we can divide this by the empirical formula mass in order to identify
the number of empirical formula units per molecule, which we designate as n:
g
molecular or molar mass (amu or )
mol
= n f ormula units/molecule
g
empirical f ormula mass (amu or )
mol

The molecular formula is then obtained by multiplying each subscript in the empirical formula by n, as shown by the generic
empirical formula AxBy:

(Ax By )n = Anx Bnx

Access for free at OpenStax 3.2.6 https://chem.libretexts.org/@go/page/38148


For example, consider a covalent compound whose empirical formula is determined to be CH2O. The empirical formula mass for
this compound is approximately 30 amu (the sum of 12 amu for one C atom, 2 amu for two H atoms, and 16 amu for one O atom).
If the compound’s molecular mass is determined to be 180 amu, this indicates that molecules of this compound contain six times
the number of atoms represented in the empirical formula:
180 amu/molecule
= 6 f ormula units/molecule
amu
30
f ormula unit

Molecules of this compound are then represented by molecular formulas whose subscripts are six times greater than those in the
empirical formula:

(CH O) =C H O
2 6 6 12 6

Note that this same approach may be used when the molar mass (g/mol) instead of the molecular mass (amu) is used. In this case,
we are merely considering one mole of empirical formula units and molecules, as opposed to single units and molecules.

 Example 3.2.5: Determination of the Molecular Formula for Nicotine

Nicotine, an alkaloid in the nightshade family of plants that is mainly responsible for the addictive nature of cigarettes,
contains 74.02% C, 8.710% H, and 17.27% N. If 40.57 g of nicotine contains 0.2500 mol nicotine, what is the molecular
formula?

Solution
Determining the molecular formula from the provided data will require comparison of the compound’s empirical formula mass
to its molar mass. As the first step, use the percent composition to derive the compound’s empirical formula. Assuming a
convenient, a 100-g sample of nicotine yields the following molar amounts of its elements:
1 mol C
(74.02 g C) ( ) = 6.163 mol C (3.2.1)
12.01 g C

1 mol H
(8.710 g H) ( ) = 8.624 mol H (3.2.2)
1.01 g H

1 mol N
(17.27 g N) ( ) = 1.233 mol N (3.2.3)
14.01 g N

Next, we calculate the molar ratios of these elements relative to the least abundant element, N .
6.163 mol C / 1.233 mol N = 5
8.264 mol H / 1.233 mol N = 7
1.233 mol N / 1.233 mol N = 1
1.233/1.233 = 1.000 mol N
6.163/1.233 = 4.998 mol C
8.624/1.233 = 6.994 mol H
The C-to-N and H-to-N molar ratios are adequately close to whole numbers, and so the empirical formula is C5H7N. The
empirical formula mass for this compound is therefore 81.13 amu/formula unit, or 81.13 g/mol formula unit.
We calculate the molar mass for nicotine from the given mass and molar amount of compound:
40.57 g nicotine 162.3 g
=
0.2500 mol nicotine mol

Comparing the molar mass and empirical formula mass indicates that each nicotine molecule contains two formula units:
162.3 g/mol
= 2 f ormula units/molecule
g
81.13
f ormula unit

Access for free at OpenStax 3.2.7 https://chem.libretexts.org/@go/page/38148


Thus, we can derive the molecular formula for nicotine from the empirical formula by multiplying each subscript by two:

(C H N) =C H N
5 7 2 10 14 2

 Exercise 3.2.5

What is the molecular formula of a compound with a percent composition of 49.47% C, 5.201% H, 28.84% N, and 16.48% O,
and a molecular mass of 194.2 amu?

Answer
C8H10N4O2

Summary
The chemical identity of a substance is defined by the types and relative numbers of atoms composing its fundamental entities
(molecules in the case of covalent compounds, ions in the case of ionic compounds). A compound’s percent composition provides
the mass percentage of each element in the compound, and it is often experimentally determined and used to derive the compound’s
empirical formula. The empirical formula mass of a covalent compound may be compared to the compound’s molecular or molar
mass to derive a molecular formula.

3.2.6: Key Equations


mass X
%X = × 100%
mass compound
g
molecular or molar mass (amu or )
mol
= n f ormula units/molecule
g
empirical f ormula mass (amu or )
mol
(AxBy)n = AnxBny

Glossary
percent composition
percentage by mass of the various elements in a compound

empirical formula mass


sum of average atomic masses for all atoms represented in an empirical formula

This page titled 3.2: Determining Empirical and Molecular Formulas is shared under a CC BY 4.0 license and was authored, remixed, and/or
curated by OpenStax via source content that was edited to the style and standards of the LibreTexts platform; a detailed edit history is available
upon request.

Access for free at OpenStax 3.2.8 https://chem.libretexts.org/@go/page/38148


3.3: Molarity
 Learning Objectives
Describe the fundamental properties of solutions
Calculate solution concentrations using molarity
Perform dilution calculations using the dilution equation

In preceding sections, we focused on the composition of substances: samples of matter that contain only one type of element or
compound. However, mixtures—samples of matter containing two or more substances physically combined—are more commonly
encountered in nature than are pure substances. Similar to a pure substance, the relative composition of a mixture plays an
important role in determining its properties. The relative amount of oxygen in a planet’s atmosphere determines its ability to sustain
aerobic life. The relative amounts of iron, carbon, nickel, and other elements in steel (a mixture known as an “alloy”) determine its
physical strength and resistance to corrosion. The relative amount of the active ingredient in a medicine determines its effectiveness
in achieving the desired pharmacological effect. The relative amount of sugar in a beverage determines its sweetness (Figure
3.3.1). In this section, we will describe one of the most common ways in which the relative compositions of mixtures may be

quantified.

Figure 3.3.1 : Sugar is one of many components in the complex mixture known as coffee. The amount of sugar in a given amount
of coffee is an important determinant of the beverage’s sweetness. (credit: Jane Whitney)

3.3.1: Solutions
We have previously defined solutions as homogeneous mixtures, meaning that the composition of the mixture (and therefore its
properties) is uniform throughout its entire volume. Solutions occur frequently in nature and have also been implemented in many
forms of manmade technology. We will explore a more thorough treatment of solution properties in the chapter on solutions and
colloids, but here we will introduce some of the basic properties of solutions.
The relative amount of a given solution component is known as its concentration. Often, though not always, a solution contains one
component with a concentration that is significantly greater than that of all other components. This component is called the solvent
and may be viewed as the medium in which the other components are dispersed, or dissolved. Solutions in which water is the
solvent are, of course, very common on our planet. A solution in which water is the solvent is called an aqueous solution.
A solute is a component of a solution that is typically present at a much lower concentration than the solvent. Solute concentrations
are often described with qualitative terms such as dilute (of relatively low concentration) and concentrated (of relatively high
concentration).
Concentrations may be quantitatively assessed using a wide variety of measurement units, each convenient for particular
applications. Molarity (M) is a useful concentration unit for many applications in chemistry. Molarity is defined as the number of
moles of solute in exactly 1 liter (1 L) of the solution:
mol solute
M = (3.3.1)
L solution

Access for free at OpenStax 3.3.1 https://chem.libretexts.org/@go/page/38149


 Example 3.3.1: Calculating Molar Concentrations

A 355-mL soft drink sample contains 0.133 mol of sucrose (table sugar). What is the molar concentration of sucrose in the
beverage?

Solution
Since the molar amount of solute and the volume of solution are both given, the molarity can be calculated using the definition
of molarity. Per this definition, the solution volume must be converted from mL to L:
mol solute
M =
L solution

0.133 mol
=
1 L
355 mL ×
1000 mL

= 0.375 M

 Exercise 3.3.1

A teaspoon of table sugar contains about 0.01 mol sucrose. What is the molarity of sucrose if a teaspoon of sugar has been
dissolved in a cup of tea with a volume of 200 mL?

Answer
0.05 M

 Example 3.3.2: Deriving Moles and Volumes from Molar Concentrations

How much sugar (mol) is contained in a modest sip (~10 mL) of the soft drink from Example 3.3.1?

Solution
In this case, we can rearrange the definition of molarity to isolate the quantity sought, moles of sugar. We then substitute the
value for molarity that we derived in Example 3.4.2, 0.375 M:
mol solute
M = (3.3.2)
L solution

mol solute = M × L solution

mol sugar 1 L
mol solute = 0.375 × (10 mL × ) = 0.004 mol sugar
L 1000 mL

 Exercise 3.3.2
What volume (mL) of the sweetened tea described in Example 3.3.1 contains the same amount of sugar (mol) as 10 mL of the
soft drink in this example?

Answer
80 mL

Access for free at OpenStax 3.3.2 https://chem.libretexts.org/@go/page/38149


 Example 3.3.3: Calculating Molar Concentrations from the Mass of Solute

Distilled white vinegar (Figure 3.3.2) is a solution of acetic acid, C H C O H, in water. A 0.500-L vinegar solution contains
3 2

25.2 g of acetic acid. What is the concentration of the acetic acid solution in units of molarity?

Figure 3.3.3 : Distilled white vinegar is a solution of acetic acid in water.


A label on a container is shown. The label has a picture of a salad with the words “Distilled White Vinegar,” and, “Reduced
with water to 5% acidity,” written above it.

Solution
As in previous examples, the definition of molarity is the primary equation used to calculate the quantity sought. In this case,
the mass of solute is provided instead of its molar amount, so we must use the solute’s molar mass to obtain the amount of
solute in moles:
1 mol CH CO H
3 2
25.2 g CH CO H ×
3 2
mol solute 60.052 g CH CO H
3 2
M = = = 0.839 M (3.3.3)
L solution 0.500 L solution

0.839 mol solute


M =
1.00 L solution

Nov 29, 2019, 5:24 PM


mol solute
M = = 0.839 M (3.3.4)
L solution

 Exercise 3.3.3

Calculate the molarity of 6.52 g of C oC l (128.9 g/mol) dissolved in an aqueous solution with a total volume of 75.0 mL.
2

Answer
0.674 M

 Example 3.3.4: Determining the Mass of Solute in a Given Volume of Solution

How many grams of NaCl are contained in 0.250 L of a 5.30-M solution?

Solution
The volume and molarity of the solution are specified, so the amount (mol) of solute is easily computed as demonstrated in
Example 3.3.3:

Access for free at OpenStax 3.3.3 https://chem.libretexts.org/@go/page/38149


mol solute
M = (3.3.5)
L solution

mol solute = M × L solution (3.3.6)

mol NaCl
mol solute = 5.30 × 0.250 L = 1.325 mol NaCl (3.3.7)
L

Finally, this molar amount is used to derive the mass of NaCl:


58.44 g NaCl
1.325 mol NaCl × = 77.4 g NaCl (3.3.8)
mol NaCl

 Exercise 3.3.4

How many grams of C aC l (110.98 g/mol) are contained in 250.0 mL of a 0.200-M solution of calcium chloride?
2

Answer
5.55 g C aC l 2

When performing calculations stepwise, as in Example 3.3.3, it is important to refrain from rounding any intermediate calculation
results, which can lead to rounding errors in the final result. In Example 3.3.4, the molar amount of NaCl computed in the first
step, 1.325 mol, would be properly rounded to 1.32 mol if it were to be reported; however, although the last digit (5) is not
significant, it must be retained as a guard digit in the intermediate calculation. If we had not retained this guard digit, the final
calculation for the mass of NaCl would have been 77.1 g, a difference of 0.3 g.
In addition to retaining a guard digit for intermediate calculations, we can also avoid rounding errors by performing computations
in a single step (Example 3.3.5). This eliminates intermediate steps so that only the final result is rounded.

 Example 3.3.5: Determining the Volume of Solution

In Example 3.3.3 , we found the typical concentration of vinegar to be 0.839 M. What volume of vinegar contains 75.6 g of
acetic acid?

Solution
First, use the molar mass to calculate moles of acetic acid from the given mass:
mol solute
g solute × = mol solute (3.3.9)
g solute

Then, use the molarity of the solution to calculate the volume of solution containing this molar amount of solute:
L solution
mol solute × = L solution (3.3.10)
mol solute

Combining these two steps into one yields:


mol solute L solution
g solute × × = L solution (3.3.11)
g solute mol solute

mol CH CO H L solution
3 2
75.6 g CH CO H ( )( ) = 1.50 L solution (3.3.12)
3 2
60.05 g 0.839 mol CH CO H
3 2

 Exercise 3.3.5:

What volume of a 1.50-M KBr solution contains 66.0 g KBr?

Answer

Access for free at OpenStax 3.3.4 https://chem.libretexts.org/@go/page/38149


0.370 L

3.3.2: Dilution of Solutions


Dilution is the process whereby the concentration of a solution is lessened by the addition of solvent. For example, we might say
that a glass of iced tea becomes increasingly diluted as the ice melts. The water from the melting ice increases the volume of the
solvent (water) and the overall volume of the solution (iced tea), thereby reducing the relative concentrations of the solutes that
give the beverage its taste (Figure 3.3.2).

Figure 3.3.2 : Both solutions contain the same mass of copper nitrate. The solution on the right is more dilute because the copper
nitrate is dissolved in more solvent. (credit: Mark Ott).
This figure shows two graduated cylinders side-by-side. The first has about half as much blue liquid as the second. The blue liquid
is darker in the first cylinder than in the second.
Dilution is also a common means of preparing solutions of a desired concentration. By adding solvent to a measured portion of a
more concentrated stock solution, we can achieve a particular concentration. For example, commercial pesticides are typically sold
as solutions in which the active ingredients are far more concentrated than is appropriate for their application. Before they can be
used on crops, the pesticides must be diluted. This is also a very common practice for the preparation of a number of common
laboratory reagents (Figure 3.3.3).

Figure 3.3.3 : A solution of KM nO is prepared by mixing water with 4.74 g of KMnO4 in a flask. (credit: modification of work
4

by Mark Ott)
This figure shows two photos. In the first, there is an empty glass container, 4.75 g of K M n O subscript 4 powder on a white
circle, and a bottle of distilled water. In the second photo the powder and about half the water have been added to the glass
container. The liquid in the glass container is almost black in color.
A simple mathematical relationship can be used to relate the volumes and concentrations of a solution before and after the dilution
process. According to the definition of molarity, the molar amount of solute in a solution is equal to the product of the solution’s
molarity and its volume in liters:

n = ML

Expressions like these may be written for a solution before and after it is diluted:

Access for free at OpenStax 3.3.5 https://chem.libretexts.org/@go/page/38149


n1 = M1 L1

n2 = M2 L2

where the subscripts “1” and “2” refer to the solution before and after the dilution, respectively. Since the dilution process does not
change the amount of solute in the solution,n1 = n2. Thus, these two equations may be set equal to one another:

M1 L1 = M2 L2

This relation is commonly referred to as the dilution equation. Although we derived this equation using molarity as the unit of
concentration and liters as the unit of volume, other units of concentration and volume may be used, so long as the units properly
cancel per the factor-label method. Reflecting this versatility, the dilution equation is often written in the more general form:

C1 V1 = C2 V2

where C and V are concentration and volume, respectively.

 Example 3.3.6: Determining the Concentration of a Diluted Solution

If 0.850 L of a 5.00-M solution of copper nitrate, Cu(NO3)2, is diluted to a volume of 1.80 L by the addition of water, what is
the molarity of the diluted solution?

Solution
We are given the volume and concentration of a stock solution, V1 and C1, and the volume of the resultant diluted solution, V2.
We need to find the concentration of the diluted solution, C2. We thus rearrange the dilution equation in order to isolate C2:

C1 V1 = C2 V2

C1 V1
C2 =
V2

Since the stock solution is being diluted by more than two-fold (volume is increased from 0.85 L to 1.80 L), we would expect
the diluted solution’s concentration to be less than one-half 5 M. We will compare this ballpark estimate to the calculated result
to check for any gross errors in computation (for example, such as an improper substitution of the given quantities).
Substituting the given values for the terms on the right side of this equation yields:
mol
0.850 L × 5.00
L
C2 = = 2.36 M
1.80 L

This result compares well to our ballpark estimate (it’s a bit less than one-half the stock concentration, 5 M).

 Exercise 3.3.6

What is the concentration of the solution that results from diluting 25.0 mL of a 2.04-M solution of CH3OH to 500.0 mL?

Answer
0.102 M C H 3 OH

 Example 3.3.7: Volume of a Diluted Solution

What volume of 0.12 M HBr can be prepared from 11 mL (0.011 L) of 0.45 M HBr?

Solution

Access for free at OpenStax 3.3.6 https://chem.libretexts.org/@go/page/38149


We are given the volume and concentration of a stock solution, V1 and C1, and the concentration of the resultant diluted
solution, C2. We need to find the volume of the diluted solution, V2. We thus rearrange the dilution equation in order to isolate
V2:

C1 V1 = C2 V2

C1 V1
V2 =
C2

Since the diluted concentration (0.12 M) is slightly more than one-fourth the original concentration (0.45 M), we would expect
the volume of the diluted solution to be roughly four times the original volume, or around 44 mL. Substituting the given values
and solving for the unknown volume yields:
(0.45 M )(0.011 L)
V2 =
(0.12 M )

V2 = 0.041 L

The volume of the 0.12-M solution is 0.041 L (41 mL). The result is reasonable and compares well with our rough estimate.

 Exercise 3.3.7

A laboratory experiment calls for 0.125 M H N O . What volume of 0.125 M H N O can be prepared from 0.250 L of 1.88 M
3 3

HN O ?3

Answer
3.76 L

 Example 3.3.8: Volume of a Concentrated Solution Needed for Dilution

What volume of 1.59 M KOH is required to prepare 5.00 L of 0.100 M KOH?

Solution
We are given the concentration of a stock solution, C1, and the volume and concentration of the resultant diluted solution, V2
and C2. We need to find the volume of the stock solution, V1. We thus rearrange the dilution equation in order to isolate V1:

C1 V1 = C2 V2

C2 V2
V1 =
C1

Since the concentration of the diluted solution 0.100 M is roughly one-sixteenth that of the stock solution (1.59 M), we would
expect the volume of the stock solution to be about one-sixteenth that of the diluted solution, or around 0.3 liters. Substituting
the given values and solving for the unknown volume yields:
(0.100 M )(5.00 L)
V1 =
1.59 M

V1 = 0.314 L

Thus, we would need 0.314 L of the 1.59-M solution to prepare the desired solution. This result is consistent with our rough
estimate.

Access for free at OpenStax 3.3.7 https://chem.libretexts.org/@go/page/38149


 Exercise 3.3.8
What volume of a 0.575-M solution of glucose, C6H12O6, can be prepared from 50.00 mL of a 3.00-M glucose solution?

Answer
0.261

Summary
Solutions are homogeneous mixtures. Many solutions contain one component, called the solvent, in which other components,
called solutes, are dissolved. An aqueous solution is one for which the solvent is water. The concentration of a solution is a measure
of the relative amount of solute in a given amount of solution. Concentrations may be measured using various units, with one very
useful unit being molarity, defined as the number of moles of solute per liter of solution. The solute concentration of a solution may
be decreased by adding solvent, a process referred to as dilution. The dilution equation is a simple relation between concentrations
and volumes of a solution before and after dilution.

3.3.3: Key Equations


mol solute
M =
L solution
C1V1 = C2V2

Glossary
aqueous solution
solution for which water is the solvent

concentrated
qualitative term for a solution containing solute at a relatively high concentration

concentration
quantitative measure of the relative amounts of solute and solvent present in a solution

dilute
qualitative term for a solution containing solute at a relatively low concentration

dilution
process of adding solvent to a solution in order to lower the concentration of solutes

dissolved
describes the process by which solute components are dispersed in a solvent

molarity (M)
unit of concentration, defined as the number of moles of solute dissolved in 1 liter of solution

solute
solution component present in a concentration less than that of the solvent
solvent
solution component present in a concentration that is higher relative to other components

This page titled 3.3: Molarity is shared under a CC BY 4.0 license and was authored, remixed, and/or curated by OpenStax via source content that
was edited to the style and standards of the LibreTexts platform; a detailed edit history is available upon request.

Access for free at OpenStax 3.3.8 https://chem.libretexts.org/@go/page/38149


3.4: Other Units for Solution Concentrations
 Learning Objectives
Define the concentration units of mass percentage, volume percentage, mass-volume percentage, parts-per-million (ppm),
and parts-per-billion (ppb)
Perform computations relating a solution’s concentration and its components’ volumes and/or masses using these units

In the previous section, we introduced molarity, a very useful measurement unit for evaluating the concentration of solutions.
However, molarity is only one measure of concentration. In this section, we will introduce some other units of concentration that
are commonly used in various applications, either for convenience or by convention.

3.4.1: Mass Percentage


Earlier in this chapter, we introduced percent composition as a measure of the relative amount of a given element in a compound.
Percentages are also commonly used to express the composition of mixtures, including solutions. The mass percentage of a
solution component is defined as the ratio of the component’s mass to the solution’s mass, expressed as a percentage:
mass of component
mass percentage = × 100% (3.4.1)
mass of solution

We are generally most interested in the mass percentages of solutes, but it is also possible to compute the mass percentage of
solvent.
Mass percentage is also referred to by similar names such as percent mass, percent weight, weight/weight percent, and other
variations on this theme. The most common symbol for mass percentage is simply the percent sign, %, although more detailed
symbols are often used including %mass, %weight, and (w/w)%. Use of these more detailed symbols can prevent confusion of
mass percentages with other types of percentages, such as volume percentages (to be discussed later in this section).
Mass percentages are popular concentration units for consumer products. The label of a typical liquid bleach bottle (Figure 3.4.1)
cites the concentration of its active ingredient, sodium hypochlorite (NaOCl), as being 7.4%. A 100.0-g sample of bleach would
therefore contain 7.4 g of NaOCl.

Figure 3.4.1 : Liquid bleach is an aqueous solution of sodium hypochlorite (NaOCl). This brand has a concentration of 7.4%
NaOCl by mass.
The sides of two cylindrical containers are shown. Each container’s label is partially visible. The left container’s label reads
“Bleach.” The right label contains more information about the product including the phrase, “Contains: Sodium hypochlorite 7.4
%.”

 Example 3.4.1: Calculation of Percent by Mass


A 5.0-g sample of spinal fluid contains 3.75 mg (0.00375 g) of glucose. What is the percent by mass of glucose in spinal fluid?

Solution
The spinal fluid sample contains roughly 4 mg of glucose in 5000 mg of fluid, so the mass fraction of glucose should be a bit
less than one part in 1000, or about 0.1%. Substituting the given masses into the equation defining mass percentage yields:

Access for free at OpenStax 3.4.1 https://chem.libretexts.org/@go/page/38150


1 g
3.75 mg glucose ×
1000 mg
% glucose = = 0.075%
5.0 g spinal f luid

The computed mass percentage agrees with our rough estimate (it’s a bit less than 0.1%).
Note that while any mass unit may be used to compute a mass percentage (mg, g, kg, oz, and so on), the same unit must be
used for both the solute and the solution so that the mass units cancel, yielding a dimensionless ratio. In this case, we converted
the units of solute in the numerator from mg to g to match the units in the denominator. We could just as easily have converted
the denominator from g to mg instead. As long as identical mass units are used for both solute and solution, the computed mass
percentage will be correct.

 Exercise 3.4.1

A bottle of a tile cleanser contains 135 g of HCl and 775 g of water. What is the percent by mass of HCl in this cleanser?

Answer
14.8%

 Example 3.4.2: Calculations using Mass Percentage

“Concentrated” hydrochloric acid is an aqueous solution of 37.2% HCl that is commonly used as a laboratory reagent. The
density of this solution is 1.19 g/mL. What mass of HCl is contained in 0.500 L of this solution?

Solution
The HCl concentration is near 40%, so a 100-g portion of this solution would contain about 40 g of HCl. Since the solution
density isn’t greatly different from that of water (1 g/mL), a reasonable estimate of the HCl mass in 500 g (0.5 L) of the
solution is about five times greater than that in a 100 g portion, or 5 × 40 = 200 g . To derive the mass of solute in a solution
from its mass percentage, we need to know the corresponding mass of the solution. Using the solution density given, we can
convert the solution’s volume to mass, and then use the given mass percentage to calculate the solute mass. This mathematical
approach is outlined in this flowchart:

A diagram of three boxes connected by a right-facing arrow in between each is shown. The box on the left contains the phrase,
“Volume of solution ( m L ),” the middle box reads, “Mass of solution ( g ),” while the one on the right contains the phrase,
“Mass of H C l ( g ).” There is a phrase under the left arrow that says, “Multiply by density ( g / m L )” and under the right
arrow it states, “Multiply by mass percent as ratio ( g H C l / g solution ).”
For proper unit cancellation, the 0.500-L volume is converted into 500 mL, and the mass percentage is expressed as a ratio,
37.2 g HCl/g solution:
1.19 g solution 37.2 g HCl
500 mL solution ( )( ) = 221 g HCl
mL solution 100 g solution

This mass of HCl is consistent with our rough estimate of approximately 200 g.

 Exercise 3.4.2
What volume of concentrated HCl solution contains 125 g of HCl?

Answer

Access for free at OpenStax 3.4.2 https://chem.libretexts.org/@go/page/38150


282 mL

3.4.2: Volume Percentage


Liquid volumes over a wide range of magnitudes are conveniently measured using common and relatively inexpensive laboratory
equipment. The concentration of a solution formed by dissolving a liquid solute in a liquid solvent is therefore often expressed as a
volume percentage, %vol or (v/v)%:
volume solute
volume percentage = × 100% (3.4.2)
volume solution

 Example 3.4.3: Calculations using Volume Percentage

Rubbing alcohol (isopropanol) is usually sold as a 70%vol aqueous solution. If the density of isopropyl alcohol is 0.785 g/mL,
how many grams of isopropyl alcohol are present in a 355 mL bottle of rubbing alcohol?

Solution
Per the definition of volume percentage, the isopropanol volume is 70% of the total solution volume. Multiplying the
isopropanol volume by its density yields the requested mass:
70 mL isopropyl alcohol 0.785 g isopropyl alcohol
355 mL solution( )( ) = 195 g isopropyl alcohol
100 mL solution 1 mL isopropyl alcohol

 Exercise 3.4.3

Wine is approximately 12% ethanol (CH CH OH ) by volume. Ethanol has a molar mass of 46.06 g/mol and a density 0.789
3 2

g/mL. How many moles of ethanol are present in a 750-mL bottle of wine?

Answer
1.5 mol ethanol

3.4.3: Mass-Volume Percentage


“Mixed” percentage units, derived from the mass of solute and the volume of solution, are popular for certain biochemical and
medical applications. A mass-volume percent is a ratio of a solute’s mass to the solution’s volume expressed as a percentage. The
specific units used for solute mass and solution volume may vary, depending on the solution. For example, physiological saline
solution, used to prepare intravenous fluids, has a concentration of 0.9% mass/volume (m/v), indicating that the composition is 0.9
g of solute per 100 mL of solution. The concentration of glucose in blood (commonly referred to as “blood sugar”) is also typically
expressed in terms of a mass-volume ratio. Though not expressed explicitly as a percentage, its concentration is usually given in
milligrams of glucose per deciliter (100 mL) of blood (Figure 3.4.2).

Access for free at OpenStax 3.4.3 https://chem.libretexts.org/@go/page/38150


Figure 3.4.2 : “Mixed” mass-volume units are commonly encountered in medical settings. (a) The NaCl concentration of
physiological saline is 0.9% (m/v). (b) This device measures glucose levels in a sample of blood. The normal range for glucose
concentration in blood (fasting) is around 70–100 mg/dL. (credit a: modification of work by “The National Guard”/Flickr; credit b:
modification of work by Biswarup Ganguly).
Two pictures are shown labeled a and b. Picture a depicts a clear, colorless solution in a plastic bag being held in a person’s hand.
Picture b shows a person’s hand holding a detection meter with a digital readout screen while another hand holds someone’s finger
up to the end of the meter. The meter is pressed to the drop of blood that is at the end of the person’s finger.

3.4.4: Parts per Million and Parts per Billion


Very low solute concentrations are often expressed using appropriately small units such as parts per million (ppm) or parts per
billion (ppb). Like percentage (“part per hundred”) units, ppm and ppb may be defined in terms of masses, volumes, or mixed
mass-volume units. There are also ppm and ppb units defined with respect to numbers of atoms and molecules.
The mass-based definitions of ppm and ppb are given here:
mass solute
6
ppm = × 10 ppm (3.4.3)
mass solution

mass solute
9
ppb = × 10 ppb (3.4.4)
mass solution

Both ppm and ppb are convenient units for reporting the concentrations of pollutants and other trace contaminants in water.
Concentrations of these contaminants are typically very low in treated and natural waters, and their levels cannot exceed relatively
low concentration thresholds without causing adverse effects on health and wildlife. For example, the EPA has identified the
maximum safe level of fluoride ion in tap water to be 4 ppm. Inline water filters are designed to reduce the concentration of
fluoride and several other trace-level contaminants in tap water (Figure 3.4.3).

Figure 3.4.3 : (a) In some areas, trace-level concentrations of contaminants can render unfiltered tap water unsafe for drinking and
cooking. (b) Inline water filters reduce the concentration of solutes in tap water. (credit a: modification of work by Jenn Durfey;
credit b: modification of work by “vastateparkstaff”/Wikimedia commons).
Two pictures are shown labeled a and b. Picture a depicts a clear, colorless solution in a plastic bag being held in a person’s hand.
Picture b shows a person’s hand holding a detection meter with a digital readout screen while another hand holds someone’s finger
up to the end of the meter. The meter is pressed to the drop of blood that is at the end of the person’s finger.

Access for free at OpenStax 3.4.4 https://chem.libretexts.org/@go/page/38150


 Example 3.4.4: Parts per Million and Parts per Billion Concentrations

According to the EPA, when the concentration of lead in tap water reaches 15 ppb, certain remedial actions must be taken.
What is this concentration in ppm? At this concentration, what mass of lead (μg) would be contained in a typical glass of water
(300 mL)?

Solution
The definitions of the ppm and ppb units may be used to convert the given concentration from ppb to ppm. Comparing these
two unit definitions shows that ppm is 1000 times greater than ppb (1 ppm = 103 ppb). Thus:
1 ppm
15 ppb × = 0.015 ppm
3
10 ppb

The definition of the ppb unit may be used to calculate the requested mass if the mass of the solution is provided. However,
only the volume of solution (300 mL) is given, so we must use the density to derive the corresponding mass. We can assume
the density of tap water to be roughly the same as that of pure water (~1.00 g/mL), since the concentrations of any dissolved
substances should not be very large. Rearranging the equation defining the ppb unit and substituting the given quantities yields:
mass solute
9
ppb = × 10 ppb
mass solution

ppb × mass solution


mass solute =
9
10 ppb

1.00 g
15 ppb × 300 mL ×
mL −6
mass solute = = 4.5 × 10 g
9
10 ppb

Finally, convert this mass to the requested unit of micrograms:


1 μg
−6
4.5 × 10 g× = 4.5 μg
−6
10 g

 Exercise 3.4.4

A 50.0-g sample of industrial wastewater was determined to contain 0.48 mg of mercury. Express the mercury concentration of
the wastewater in ppm and ppb units.

Answer
9.6 ppm, 9600 ppb

Summary
In addition to molarity, a number of other solution concentration units are used in various applications. Percentage concentrations
based on the solution components’ masses, volumes, or both are useful for expressing relatively high concentrations, whereas lower
concentrations are conveniently expressed using ppm or ppb units. These units are popular in environmental, medical, and other
fields where mole-based units such as molarity are not as commonly used.

Glossary
mass percentage
ratio of solute-to-solution mass expressed as a percentage

mass-volume percent

Access for free at OpenStax 3.4.5 https://chem.libretexts.org/@go/page/38150


ratio of solute mass to solution volume, expressed as a percentage

parts per billion (ppb)


ratio of solute-to-solution mass multiplied by 109

parts per million (ppm)


ratio of solute-to-solution mass multiplied by 106

volume percentage
ratio of solute-to-solution volume expressed as a percentage

This page titled 3.4: Other Units for Solution Concentrations is shared under a CC BY 4.0 license and was authored, remixed, and/or curated by
OpenStax via source content that was edited to the style and standards of the LibreTexts platform; a detailed edit history is available upon request.

Access for free at OpenStax 3.4.6 https://chem.libretexts.org/@go/page/38150


3.E: Composition of Substances and Solutions (Exercises)
3.E.1: 3.1: Formula Mass and the Mole Concept
What is the total mass (amu) of carbon in each of the following molecules?
1. (a) CH4
2. (b) CHCl3
3. (c) C12H10O6
4. (d) CH3CH2CH2CH2CH3
(a) 12.01 amu; (b) 12.01 amu; (c) 144.12 amu; (d) 60.05 amu
What is the total mass of hydrogen in each of the molecules?
1. (a) CH4
2. (b) CHCl3
3. (c) C12H10O6
4. (d) CH3CH2CH2CH2CH3
Calculate the molecular or formula mass of each of the following:
(a) P4
(b) H2O
(c) Ca(NO3)2
(d) CH3CO2H (acetic acid)
(e) C12H22O11 (sucrose, cane sugar).
(a) 123.896 amu; (b) 18.015 amu; (c) 164.086 amu; (d) 60.052 amu; (e) 342.297 amu
Determine the molecular mass of the following compounds:
(a)

(b)

(c)

(d)

Determine the molecular mass of the following compounds:

Access for free at OpenStax 3.E.1 https://chem.libretexts.org/@go/page/74180


(a)

(b)

(c)

(d)

1. (a) 56.107 amu;


2. (b) 54.091 amu;
3. (c) 199.9976 amu;
4. (d) 97.9950 amu
Which molecule has a molecular mass of 28.05 amu?
(a)

(b)

(c)

Write a sentence that describes how to determine the number of moles of a compound in a known mass of the compound if we
know its molecular formula.

Access for free at OpenStax 3.E.2 https://chem.libretexts.org/@go/page/74180


Use the molecular formula to find the molar mass; to obtain the number of moles, divide the mass of compound by the molar mass
of the compound expressed in grams.
Compare 1 mole of H2, 1 mole of O2, and 1 mole of F2.
1. (a) Which has the largest number of molecules? Explain why.
2. (b) Which has the greatest mass? Explain why.
Which contains the greatest mass of oxygen: 0.75 mol of ethanol (C2H5OH), 0.60 mol of formic acid (HCO2H), or 1.0 mol of water
(H2O)? Explain why.
Formic acid. Its formula has twice as many oxygen atoms as the other two compounds (one each). Therefore, 0.60 mol of formic
acid would be equivalent to 1.20 mol of a compound containing a single oxygen atom.
Which contains the greatest number of moles of oxygen atoms: 1 mol of ethanol (C2H5OH), 1 mol of formic acid (HCO2H), or 1
mol of water (H2O)? Explain why.
How are the molecular mass and the molar mass of a compound similar and how are they different?
The two masses have the same numerical value, but the units are different: The molecular mass is the mass of 1 molecule while the
molar mass is the mass of 6.022 × 1023 molecules.
Calculate the molar mass of each of the following compounds:
1. (a) hydrogen fluoride, HF
2. (b) ammonia, NH3
3. (c) nitric acid, HNO3
4. (d) silver sulfate, Ag2SO4
5. (e) boric acid, B(OH)3
Calculate the molar mass of each of the following:
1. (a) S8
2. (b) C5H12
3. (c) Sc2(SO4)3
4. (d) CH3COCH3 (acetone)
5. (e) C6H12O6 (glucose)
(a) 256.528 g/mol; (b) 72.150 g mol−1; (c) 378.103 g mol−1; (d) 58.080 g mol−1; (e) 180.158 g mol−1
Calculate the empirical or molecular formula mass and the molar mass of each of the following minerals:
1. (a) limestone, CaCO3
2. (b) halite, NaCl
3. (c) beryl, Be3Al2Si6O18
4. (d) malachite, Cu2(OH)2CO3
5. (e) turquoise, CuAl6(PO4)4(OH)8(H2O)4
Calculate the molar mass of each of the following:
1. (a) the anesthetic halothane, C2HBrClF3
2. (b) the herbicide paraquat, C12H14N2Cl2
3. (c) caffeine, C8H10N4O2
4. (d) urea, CO(NH2)2
5. (e) a typical soap, C17H35CO2Na
(a) 197.382 g mol−1; (b) 257.163 g mol−1; (c) 194.193 g mol−1; (d) 60.056 g mol−1; (e) 306.464 g mol−1
Determine the number of moles of compound and the number of moles of each type of atom in each of the following:
1. (a) 25.0 g of propylene, C3H6
2. (b) 3.06 × 10−3 g of the amino acid glycine, C2H5NO2
3. (c) 25 lb of the herbicide Treflan, C13H16N2O4F (1 lb = 454 g)
4. (d) 0.125 kg of the insecticide Paris Green, Cu4(AsO3)2(CH3CO2)2

Access for free at OpenStax 3.E.3 https://chem.libretexts.org/@go/page/74180


5. (e) 325 mg of aspirin, C6H4(CO2H)(CO2CH3)
Determine the mass of each of the following:
1. (a) 0.0146 mol KOH
2. (b) 10.2 mol ethane, C2H6
3. (c) 1.6 × 10−3 mol Na2 SO4
4. (d) 6.854 × 103 mol glucose, C6 H12 O6
5. (e) 2.86 mol Co(NH3)6Cl3
1. (a) 0.819 g;
2. (b) 307 g;
3. (c) 0.23 g;
4. (d) 1.235 × 106 g (1235 kg);
5. (e) 765 g
Determine the number of moles of the compound and determine the number of moles of each type of atom in each of the following:
1. (a) 2.12 g of potassium bromide, KBr
2. (b) 0.1488 g of phosphoric acid, H3PO4
3. (c) 23 kg of calcium carbonate, CaCO3
4. (d) 78.452 g of aluminum sulfate, Al2(SO4)3
5. (e) 0.1250 mg of caffeine, C8H10N4O2
Determine the mass of each of the following:
1. (a) 2.345 mol LiCl
2. (b) 0.0872 mol acetylene, C2H2
3. (c) 3.3 × 10−2 mol Na2 CO3
4. (d) 1.23 × 103 mol fructose, C6 H12 O6
5. (e) 0.5758 mol FeSO4(H2O)7
1. (a) 99.41 g;
2. (b) 2.27 g;
3. (c) 3.5 g;
4. (d) 222 kg;
5. (e) 160.1 g
The approximate minimum daily dietary requirement of the amino acid leucine, C6H13NO2, is 1.1 g. What is this requirement in
moles?
Determine the mass in grams of each of the following:
1. (a) 0.600 mol of oxygen atoms
2. (b) 0.600 mol of oxygen molecules, O2
3. (c) 0.600 mol of ozone molecules, O3
1.
(a) 9.60 g; (b) 19.2 g; (c) 28.8 g
A 55-kg woman has 7.5 × 10−3 mol of hemoglobin (molar mass = 64,456 g/mol) in her blood. How many hemoglobin molecules is
this? What is this quantity in grams?
Determine the number of atoms and the mass of zirconium, silicon, and oxygen found in 0.3384 mol of zircon, ZrSiO4, a
semiprecious stone.
zirconium: 2.038 × 1023 atoms; 30.87 g; silicon: 2.038 × 1023 atoms; 9.504 g; oxygen: 8.151 × 1023 atoms; 21.66 g
Determine which of the following contains the greatest mass of hydrogen: 1 mol of CH4, 0.6 mol of C6H6, or 0.4 mol of C3H8.
Determine which of the following contains the greatest mass of aluminum: 122 g of AlPO4, 266 g of Al2Cl6, or 225 g of Al2S3.
AlPO4: 1.000 mol

Access for free at OpenStax 3.E.4 https://chem.libretexts.org/@go/page/74180


Al2Cl6: 1.994 mol
Al2S3: 3.00 mol
Diamond is one form of elemental carbon. An engagement ring contains a diamond weighing 1.25 carats (1 carat = 200 mg). How
many atoms are present in the diamond?
The Cullinan diamond was the largest natural diamond ever found (January 25, 1905). It weighed 3104 carats (1 carat = 200 mg).
How many carbon atoms were present in the stone?
3.113 × 1025 C atoms
One 55-gram serving of a particular cereal supplies 270 mg of sodium, 11% of the recommended daily allowance. How many
moles and atoms of sodium are in the recommended daily allowance?
A certain nut crunch cereal contains 11.0 grams of sugar (sucrose, C12H22O11) per serving size of 60.0 grams. How many servings
of this cereal must be eaten to consume 0.0278 moles of sugar?
0.865 servings, or about 1 serving.
A tube of toothpaste contains 0.76 g of sodium monofluorophosphate (Na2PO3F) in 100 mL.
a. What mass of fluorine atoms in mg was present?
b. How many fluorine atoms were present?
Which of the following represents the least number of molecules?
a. 20.0 g of H2O (18.02 g/mol)
b. 77.0 g of CH4 (16.06 g/mol)
c. 68.0 g of CaH2 (42.09 g/mol)
d. 100.0 g of N2O (44.02 g/mol)
e. 84.0 g of HF (20.01 g/mol)
20.0 g H2O represents the least number of molecules since it has the least number of moles.

3.E.2: 3.2: Determining Empirical and Molecular Formulas


What information do we need to determine the molecular formula of a compound from the empirical formula?
Calculate the following to four significant figures:
a. (a) the percent composition of ammonia, NH3
b. (b) the percent composition of photographic “hypo,” Na2S2O3
c. (c) the percent of calcium ion in Ca3(PO4)2
(a) % N = 82.24%
% H = 17.76%;
(b) % Na = 29.08%
% S = 40.56%
% O = 30.36%;
(c) % Ca2+ = 38.76%
Determine the following to four significant figures:
a. the percent composition of hydrazoic acid, HN3
b. the percent composition of TNT, C6H2(CH3)(NO2)3
c. the percent of SO42– in Al2(SO4)3
Determine the percent ammonia, NH3, in Co(NH3)6Cl3, to three significant figures.
% NH3 = 38.2%
Determine the percent water in CuSO4·5H2O to three significant figures.

Access for free at OpenStax 3.E.5 https://chem.libretexts.org/@go/page/74180


Determine the empirical formulas for compounds with the following percent compositions:
(a) 15.8% carbon and 84.2% sulfur
(b) 40.0% carbon, 6.7% hydrogen, and 53.3% oxygen
(a) CS2
(b) CH2O
Determine the empirical formulas for compounds with the following percent compositions:
(a) 43.6% phosphorus and 56.4% oxygen
(b) 28.7% K, 1.5% H, 22.8% P, and 47.0% O
A compound of carbon and hydrogen contains 92.3% C and has a molar mass of 78.1 g/mol. What is its molecular formula?
C6H6
Dichloroethane, a compound that is often used for dry cleaning, contains carbon, hydrogen, and chlorine. It has a molar mass of 99
g/mol. Analysis of a sample shows that it contains 24.3% carbon and 4.1% hydrogen. What is its molecular formula?
Determine the empirical and molecular formula for chrysotile asbestos. Chrysotile has the following percent composition: 28.03%
Mg, 21.60% Si, 1.16% H, and 49.21% O. The molar mass for chrysotile is 520.8 g/mol.
Mg3Si2H3O8 (empirical formula), Mg6Si4H6O16 (molecular formula)
Polymers are large molecules composed of simple units repeated many times. Thus, they often have relatively simple empirical
formulas. Calculate the empirical formulas of the following polymers:
a. Lucite (Plexiglas); 59.9% C, 8.06% H, 32.0% O
b. Saran; 24.8% C, 2.0% H, 73.1% Cl
c. polyethylene; 86% C, 14% H
d. polystyrene; 92.3% C, 7.7% H
e. Orlon; 67.9% C, 5.70% H, 26.4% N
A major textile dye manufacturer developed a new yellow dye. The dye has a percent composition of 75.95% C, 17.72% N, and
6.33% H by mass with a molar mass of about 240 g/mol. Determine the molecular formula of the dye.
C15H15N3

3.E.3: 3.3: Molarity


3.E.3.1: Questions
Explain what changes and what stays the same when 1.00 L of a solution of NaCl is diluted to 1.80 L.
What information do we need to calculate the molarity of a sulfuric acid solution?
We need to know the number of moles of sulfuric acid dissolved in the solution and the volume of the solution.
What does it mean when we say that a 200-mL sample and a 400-mL sample of a solution of salt have the same molarity? In what
ways are the two samples identical? In what ways are these two samples different?
Determine the molarity for each of the following solutions:
a. 0.444 mol of CoCl2 in 0.654 L of solution
b. 98.0 g of phosphoric acid, H3PO4, in 1.00 L of solution
c. 0.2074 g of calcium hydroxide, Ca(OH)2, in 40.00 mL of solution
d. 10.5 kg of Na2SO4·10H2O in 18.60 L of solution
e. 7.0 × 10−3 mol of I2 in 100.0 mL of solution
f. 1.8 × 104 mg of HCl in 0.075 L of solution
a. (a) 0.679 M;
b. (b) 1.00 M;
c. (c) 0.06998 M;

Access for free at OpenStax 3.E.6 https://chem.libretexts.org/@go/page/74180


d. (d) 1.75 M;
e. (e) 0.070 M;
f. (f) 6.6 M
Determine the molarity of each of the following solutions:
a. 1.457 mol KCl in 1.500 L of solution
b. 0.515 g of H2SO4 in 1.00 L of solution
c. 20.54 g of Al(NO3)3 in 1575 mL of solution
d. 2.76 kg of CuSO4·5H2O in 1.45 L of solution
e. 0.005653 mol of Br2 in 10.00 mL of solution
f. 0.000889 g of glycine, C2H5NO2, in 1.05 mL of solution
Answers:
a.) 0.9713 M
b.) 5.25x10-3 M
c.) 6.122x10-2 M
d.) 7.62 M
e.) 0.5653 M
f.) 1.13x10-2 M
Consider this question: What is the mass of the solute in 0.500 L of 0.30 M glucose, C6H12O6, used for intravenous injection?
(a) Outline the steps necessary to answer the question.
(b) Answer the question.
(a) determine the number of moles of glucose in 0.500 L of solution; determine the molar mass of glucose; determine the mass of
glucose from the number of moles and its molar mass; (b) 27 g
Consider this question: What is the mass of solute in 200.0 L of a 1.556-M solution of KBr?
1. (a) Outline the steps necessary to answer the question.
2. (b) Answer the question.
Answer:
(a)
1. Calculate to moles of KBr by multiplying the Molarity by the amount of solution (200.0 L)
2. Find the Molar Mass of KBr and convert moles of solute to grams
(b)
1.556 moles KBr
× 200.0 L = 311.2 moles KBr
1 L

119.0 g KBr
311.2 moles KBr × = 37, 030 g
1 mole KBr

37,030g; 37.03 kg
Calculate the number of moles and the mass of the solute in each of the following solutions:
1. (a) 2.00 L of 18.5 M H2SO4, concentrated sulfuric acid
2. (b) 100.0 mL of 3.8 × 10−5 M NaCN, the minimum lethal concentration of sodium cyanide in blood serum
3. (c) 5.50 L of 13.3 M H2CO, the formaldehyde used to “fix” tissue samples
4. (d) 325 mL of 1.8 × 10−6 M FeSO4, the minimum concentration of iron sulfate detectable by taste in drinking water
(a) 37.0 mol H2SO4;
3.63 × 103 g H2SO4;

Access for free at OpenStax 3.E.7 https://chem.libretexts.org/@go/page/74180


(b) 3.8 × 10−6 mol NaCN;
1.9 × 10−4 g NaCN;
(c) 73.2 mol H2CO;
2.20 kg H2CO;
(d) 5.9 × 10−7 mol FeSO4;
8.9 × 10−5 g FeSO4
Calculate the number of moles and the mass of the solute in each of the following solutions:
a. 325 mL of 8.23 × 10−5 M KI, a source of iodine in the diet
b. 75.0 mL of 2.2 × 10−5 M H2SO4, a sample of acid rain
c. 0.2500 L of 0.1135 M K2CrO4, an analytical reagent used in iron assays
d. 10.5 L of 3.716 M (NH4)2SO4, a liquid fertilizer
Answers:
a. 2.67x10-5 moles KI; 4.44x10-3g KI
b. 1.7x10-6 moles H2SO4 ; 1.6x10-4 g H2SO4
c. 2.838x10-2 moles K2CrO4 ; 5.510g K2CrO4
d. 39.0 moles (NH4)2SO4 ; 5,160 g (NH4)2SO4
Consider this question: What is the molarity of KMnO4 in a solution of 0.0908 g of KMnO4 in 0.500 L of solution?
1. (a) Outline the steps necessary to answer the question.
2. (b) Answer the question.
(a) Determine the molar mass of KMnO4; determine the number of moles of KMnO4 in the solution; from the number of moles and
the volume of solution, determine the molarity; (b) 1.15 × 10−3 M
Consider this question: What is the molarity of HCl if 35.23 mL of a solution of HCl contain 0.3366 g of HCl?
1. (a) Outline the steps necessary to answer the question.
2. (b) Answer the question.
Answer:
(a)
1. Convert g of HCl to moles of HCl and convert mL of solution to L of solution
2. Divide moles of HCl by L of solution
(b)
1 mole HCl −3
0.3366 g HCl × = 9.232 × 10 moles HCl
36.46 g HCl

35.23 mL = 0.03523 L

−3
9.232 × 10 moles HCl
= 0.2621 M HCl
0.03523 L

0.2621 M ;
Calculate the molarity of each of the following solutions:
(a) 0.195 g of cholesterol, C27H46O, in 0.100 L of serum, the average concentration of cholesterol in human serum
(b) 4.25 g of NH3 in 0.500 L of solution, the concentration of NH3 in household ammonia
(c) 1.49 kg of isopropyl alcohol, C3H7OH, in 2.50 L of solution, the concentration of isopropyl alcohol in rubbing alcohol
(d) 0.029 g of I2 in 0.100 L of solution, the solubility of I2 in water at 20 °C
(a) 5.04 × 10−3 M;

Access for free at OpenStax 3.E.8 https://chem.libretexts.org/@go/page/74180


(b) 0.499 M;
(c) 9.92 M;
(d) 1.1 × 10−3 M
Calculate the molarity of each of the following solutions:
a. 293 g HCl in 666 mL of solution, a concentrated HCl solution
b. 2.026 g FeCl3 in 0.1250 L of a solution used as an unknown in general chemistry laboratories
c. 0.001 mg Cd2+ in 0.100 L, the maximum permissible concentration of cadmium in drinking water
d. 0.0079 g C7H5SNO3 in one ounce (29.6 mL), the concentration of saccharin in a diet soft drink.
There is about 1.0 g of calcium, as Ca2+, in 1.0 L of milk. What is the molarity of Ca2+ in milk?
0.025 M
What volume of a 1.00-M Fe(NO3)3 solution can be diluted to prepare 1.00 L of a solution with a concentration of 0.250 M?
If 0.1718 L of a 0.3556-M C3H7OH solution is diluted to a concentration of 0.1222 M, what is the volume of the resulting solution?
0.5000 L
If 4.12 L of a 0.850 M-H3PO4 solution is be diluted to a volume of 10.00 L, what is the concentration the resulting solution?
What volume of a 0.33-M C12H22O11 solution can be diluted to prepare 25 mL of a solution with a concentration of 0.025 M?
1.9 mL
What is the concentration of the NaCl solution that results when 0.150 L of a 0.556-M solution is allowed to evaporate until the
volume is reduced to 0.105 L?
What is the molarity of the diluted solution when each of the following solutions is diluted to the given final volume?
a. (a) 1.00 L of a 0.250-M solution of Fe(NO3)3 is diluted to a final volume of 2.00 L
b. (b) 0.5000 L of a 0.1222-M solution of C3H7OH is diluted to a final volume of 1.250 L
c. (c) 2.35 L of a 0.350-M solution of H3PO4 is diluted to a final volume of 4.00 L
d. (d) 22.50 mL of a 0.025-M solution of C12H22O11 is diluted to 100.0 mL
1. (a) 0.125 M;
2. (b) 0.04888 M;
3. (c) 0.206 M;
4. (e) 0.0056 M
What is the final concentration of the solution produced when 225.5 mL of a 0.09988-M solution of Na2CO3 is allowed to
evaporate until the solution volume is reduced to 45.00 mL?
A 2.00-L bottle of a solution of concentrated HCl was purchased for the general chemistry laboratory. The solution contained 868.8
g of HCl. What is the molarity of the solution?
11.9 M
An experiment in a general chemistry laboratory calls for a 2.00-M solution of HCl. How many mL of 11.9 M HCl would be
required to make 250 mL of 2.00 M HCl?
What volume of a 0.20-M K2SO4 solution contains 57 g of K2SO4?
1.6 L
The US Environmental Protection Agency (EPA) places limits on the quantities of toxic substances that may be discharged into the
sewer system. Limits have been established for a variety of substances, including hexavalent chromium, which is limited to 0.50
mg/L. If an industry is discharging hexavalent chromium as potassium dichromate (K2Cr2O7), what is the maximum permissible
molarity of that substance?

3.E.4: 3.4: Other Units for Solution Concentrations

Access for free at OpenStax 3.E.9 https://chem.libretexts.org/@go/page/74180


3.E.4.1: Questions
1. Consider this question: What mass of a concentrated solution of nitric acid (68.0% HNO3 by mass) is needed to prepare 400.0 g
of a 10.0% solution of HNO3 by mass?
a. Outline the steps necessary to answer the question.
b. Answer the question.
2. What mass of a 4.00% NaOH solution by mass contains 15.0 g of NaOH?
3. What mass of solid NaOH (97.0% NaOH by mass) is required to prepare 1.00 L of a 10.0% solution of NaOH by mass? The
density of the 10.0% solution is 1.109 g/mL.
4. What mass of HCl is contained in 45.0 mL of an aqueous HCl solution that has a density of 1.19 g cm–3 and contains 37.21%
HCl by mass?
5. The hardness of water (hardness count) is usually expressed in parts per million (by mass) of CaCO , which is equivalent to
3

milligrams of CaCO per liter of water. What is the molar concentration of Ca2+ ions in a water sample with a hardness count
3

of 175 mg CaCO3/L?
6. The level of mercury in a stream was suspected to be above the minimum considered safe (1 part per billion by weight). An
analysis indicated that the concentration was 0.68 parts per billion. Assume a density of 1.0 g/mL and calculate the molarity of
mercury in the stream.
7. In Canada and the United Kingdom, devices that measure blood glucose levels provide a reading in millimoles per liter. If a
measurement of 5.3 mM is observed, what is the concentration of glucose (C6H12O6) in mg/dL?
8. A throat spray is 1.40% by mass phenol, C H OH , in water. If the solution has a density of 0.9956 g/mL, calculate the
6 5

molarity of the solution.


9. Copper(I) iodide (CuI) is often added to table salt as a dietary source of iodine. How many moles of CuI are contained in 1.00
lb (454 g) of table salt containing 0.0100% CuI by mass?
10. A cough syrup contains 5.0% ethyl alcohol, C2H5OH, by mass. If the density of the solution is 0.9928 g/mL, determine the
molarity of the alcohol in the cough syrup.
11. D5W is a solution used as an intravenous fluid. It is a 5.0% by mass solution of dextrose (C H O ) in water. If the density of
6 12 6

D5W is 1.029 g/mL, calculate the molarity of dextrose in the solution.


12. Find the molarity of a 40.0% by mass aqueous solution of sulfuric acid, H SO , for which the density is 1.3057 g/mL.
2 4

3.E.4.2: Solutions
1
(a) The dilution equation can be used, appropriately modified to accommodate mass-based concentration units:

% mass1 × mass1 = % mass2 × mass2 (3.E.1)

This equation can be rearranged to isolate mass and the given quantities substituted into this equation.
1

(b) 58.8 g
3. 114 g 5. 1.75 × 10 −3
M 7 95 mg/dL9 2.38 × 10 −4
mol 11 0.29mol

This page titled 3.E: Composition of Substances and Solutions (Exercises) is shared under a CC BY 4.0 license and was authored, remixed, and/or
curated by OpenStax via source content that was edited to the style and standards of the LibreTexts platform; a detailed edit history is available
upon request.

Access for free at OpenStax 3.E.10 https://chem.libretexts.org/@go/page/74180


CHAPTER OVERVIEW
4: Stoichiometry of Chemical Reactions

A general chemistry Libretexts Textbook remixed and remastered from


OpenStax's textbook:
General Chemistry
This chapter will describe how to symbolize chemical reactions using chemical equations, how to classify some common chemical
reactions by identifying patterns of reactivity, and how to determine the quantitative relations between the amounts of substances
involved in chemical reactions—that is, the reaction stoichiometry.
4.0: Prelude to Stoichiometry
4.1: Writing and Balancing Chemical Equations
4.2: Classifying Chemical Reactions
4.3: Reaction Stoichiometry
4.4: Reaction Yields
4.5: Quantitative Chemical Analysis
4.E: Stoichiometry of Chemical Reactions (Exercises)

This page titled 4: Stoichiometry of Chemical Reactions is shared under a CC BY 4.0 license and was authored, remixed, and/or curated by
OpenStax via source content that was edited to the style and standards of the LibreTexts platform; a detailed edit history is available upon request.

1
4.0: Prelude to Stoichiometry
Solid-fuel rockets are a central feature in the world’s space exploration programs, including the new Space Launch System being
developed by the National Aeronautics and Space Administration (NASA) to replace the retired Space Shuttle fleet (Figure 4.0.1).
The engines of these rockets rely on carefully prepared solid mixtures of chemicals combined in precisely measured amounts.
Igniting the mixture initiates a vigorous chemical reaction that rapidly generates large amounts of gaseous products. These gases
are ejected from the rocket engine through its nozzle, providing the thrust needed to propel heavy payloads into space. Both the
nature of this chemical reaction and the relationships between the amounts of the substances being consumed and produced by the
reaction are critically important considerations that determine the success of the technology. This chapter will describe how to
symbolize chemical reactions using chemical equations, how to classify some common chemical reactions by identifying patterns
of reactivity, and how to determine the quantitative relations between the amounts of substances involved in chemical reactions—
that is, the reaction stoichiometry.

Figure 4.0.1 : Many modern rocket fuels are solid mixtures of substances combined in carefully measured amounts and ignited to
yield a thrust-generating chemical reaction. (credit: modification of work by NASA)
An image is shown of a rocket that appears to have just passed through a layer of clouds as it travels skyward. A bright white light
is seen in the upper right corner of the image. To the lower left appears the layer of clouds and the bottom of the rocket with fire
projecting from the fuel cones at its base.

This page titled 4.0: Prelude to Stoichiometry is shared under a CC BY 4.0 license and was authored, remixed, and/or curated by OpenStax via
source content that was edited to the style and standards of the LibreTexts platform; a detailed edit history is available upon request.

Access for free at OpenStax 4.0.1 https://chem.libretexts.org/@go/page/38157


4.1: Writing and Balancing Chemical Equations
 Learning Objectives
Derive chemical equations from narrative descriptions of chemical reactions.
Write and balance chemical equations in molecular, total ionic, and net ionic formats.

The preceding chapter introduced the use of element symbols to represent individual atoms. When atoms gain or lose electrons to
yield ions, or combine with other atoms to form molecules, their symbols are modified or combined to generate chemical formulas
that appropriately represent these species. Extending this symbolism to represent both the identities and the relative quantities of
substances undergoing a chemical (or physical) change involves writing and balancing a chemical equation. Consider as an
example the reaction between one methane molecule (CH4) and two diatomic oxygen molecules (O2) to produce one carbon
dioxide molecule (CO2) and two water molecules (H2O). The chemical equation representing this process is provided in the upper
half of Figure 4.1.1, with space-filling molecular models shown in the lower half of the figure.

Figure 4.1.1 : The reaction between methane and oxygen to yield carbon dioxide and water (shown at bottom) may be represented
by a chemical equation using formulas (top).
This figure shows a balanced chemical equation followed below by a representation of the equation using space-filling models. The
equation reads C H subscript 4 plus 2 O subscript 2 arrow C O subscript 2 plus 2 H subscript 2 O. Under the C H subscript 4, the
molecule is shown with a central black sphere, representing a C atom, to which 4 smaller white spheres, representing H atoms, are
distributed evenly around. All four H atoms are bonded to the central black C atom. This is followed by a plus sign. Under the 2 O
subscript 2, two molecules are shown. The molecules are each composed of two red spheres bonded together. The red spheres
represent O atoms. To the right of an arrow and under the C O subscript 2, appears a single molecule with a black central sphere
with two red spheres bonded to the left and right. Following a plus sign and under the 2 H subscript 2 O, are two molecules, each
with a central red sphere and two smaller white spheres attached to the lower right and lower left sides of the central red sphere.
Note that in space filling models of molecules, spheres appear slightly compressed in regions where there is a bond between two
atoms.
This example illustrates the fundamental aspects of any chemical equation:
1. The substances undergoing reaction are called reactants, and their formulas are placed on the left side of the equation.
2. The substances generated by the reaction are called products, and their formulas are placed on the right sight of the equation.
3. Plus signs (+) separate individual reactant and product formulas, and an arrow (⟶) separates the reactant and product (left and
right) sides of the equation.
4. The relative numbers of reactant and product species are represented by coefficients (numbers placed immediately to the left of
each formula). A coefficient of 1 is typically omitted.
It is common practice to use the smallest possible whole-number coefficients in a chemical equation, as is done in this example.
Realize, however, that these coefficients represent the relative numbers of reactants and products, and, therefore, they may be
correctly interpreted as ratios. Methane and oxygen react to yield carbon dioxide and water in a 1:2:1:2 ratio. This ratio is satisfied
if the numbers of these molecules are, respectively, 1-2-1-2, or 2-4-2-4, or 3-6-3-6, and so on (Figure 4.1.2). Likewise, these
coefficients may be interpreted with regard to any amount (number) unit, and so this equation may be correctly read in many ways,
including:
One methane molecule and two oxygen molecules react to yield one carbon dioxide molecule and two water molecules.
One dozen methane molecules and two dozen oxygen molecules react to yield one dozen carbon dioxide molecules and two
dozen water molecules.

Access for free at OpenStax 4.1.1 https://chem.libretexts.org/@go/page/38156


One mole of methane molecules and 2 moles of oxygen molecules react to yield 1 mole of carbon dioxide molecules and 2
moles of water molecules.

Figure 4.1.2 : Regardless of the absolute number of molecules involved, the ratios between numbers of molecules of each species
that react (the reactants) and molecules of each species that form (the products) are the same and are given in the chemical
equation.

4.1.1: Balancing Equations


When a chemical equation is balanced it means that equal numbers of atoms for each element involved in the reaction are
represented on the reactant and product sides. This is a requirement the equation must satisfy to be consistent with the law of
conservation of matter. It may be confirmed by simply summing the numbers of atoms on either side of the arrow and comparing
these sums to ensure they are equal. Note that the number of atoms for a given element is calculated by multiplying the coefficient
of any formula containing that element by the element’s subscript in the formula. If an element appears in more than one formula
on a given side of the equation, the number of atoms represented in each must be computed and then added together. For example,
both product species in the example reaction, CO and H O , contain the element oxygen, and so the number of oxygen atoms on
2 2

the product side of the equation is

⎛ 2 O atoms ⎞ ⎛ 1 O atom ⎞
1 CO molecule × + 2 H O molecule × = 4 O atoms
2 2
⎝ CO molecule ⎠ ⎝ H O molecule ⎠
2 2

The equation for the reaction between methane and oxygen to yield carbon dioxide and water is confirmed to be balanced per this
approach, as shown here:

CH +2 O → CO +2 H O
4 2 2 2

Equation for the reaction


CH +2O → CO +2H O
4 2 2 2

Element Reactants Products Balanced?

C 1×1=1 1×1=1 1 = 1, yes

H 4×1=4 2×2=4 4 = 4, yes

O 2×2=4 (1 × 2) + (2 × 1) = 4 4 = 4, yes

A balanced chemical equation often may be derived from a qualitative description of some chemical reaction by a fairly simple
approach known as balancing by inspection. Consider as an example the decomposition of water to yield molecular hydrogen and
oxygen. This process is represented qualitatively by an unbalanced chemical equation:
H O→ H +O (unbalanced)
2 2 2

Comparing the number of H and O atoms on either side of this equation confirms its imbalance:
Comparisons between H and O atoms
Element Reactants Products Balanced?

Access for free at OpenStax 4.1.2 https://chem.libretexts.org/@go/page/38156


Element Reactants Products Balanced?

H 1×2=2 1×2=2 2 = 2, yes

O 1×1=1 1×2=2 1 ≠ 2, no

The numbers of H atoms on the reactant and product sides of the equation are equal, but the numbers of O atoms are not. To
achieve balance, the coefficients of the equation may be changed as needed. Keep in mind, of course, that the formula subscripts
define, in part, the identity of the substance, and so these cannot be changed without altering the qualitative meaning of the
equation. For example, changing the reactant formula from H2O to H2O2 would yield balance in the number of atoms, but doing so
also changes the reactant’s identity (it’s now hydrogen peroxide and not water). The O atom balance may be achieved by changing
the coefficient for H2O to 2.

2H O→ H +O (unbalanced)
2 2 2

O atom balance may be achieved by changing the coefficient for H2O to 2


Element Reactants Products Balanced?

H 2×2=4 1×2=2 4 ≠ 2, no

O 2×1=2 1×2=2 2 = 2, yes

The H atom balance was upset by this change, but it is easily reestablished by changing the coefficient for the H2 product to 2.
2H O→ 2H +O (balanced)
2 2 2

H atom balance upset but easily reestablished by changing the coefficient for the H2 product to 2.
Element Reactants Products Balanced?

H 2×2=4 2×2=2 4 = 4, yes

O 2×1=2 1×2=2 2 = 2, yes

These coefficients yield equal numbers of both H and O atoms on the reactant and product sides, and the balanced equation is,
therefore:

2H O→ 2H +O
2 2 2

 Example 4.1.1: Balancing Chemical Equations

Write a balanced equation for the reaction of molecular nitrogen (N2) and oxygen (O2) to form dinitrogen pentoxide.

Solution
First, write the unbalanced equation.
N +O → N O (unbalanced)
2 2 2 5

Next, count the number of each type of atom present in the unbalanced equation.
Unbalanced Equation
Element Reactants Products Balanced?

N 1×2=2 1×2=2 2 = 2, yes

O 1×2=2 1×5=5 2 ≠ 5, no

Though nitrogen is balanced, changes in coefficients are needed to balance the number of oxygen atoms. To balance the
number of oxygen atoms, a reasonable first attempt would be to change the coefficients for the O2 and N2O5 to integers that
will yield 10 O atoms (the least common multiple for the O atom subscripts in these two formulas).

Access for free at OpenStax 4.1.3 https://chem.libretexts.org/@go/page/38156


N +5 O → 2N O (unbalanced)
2 2 2 5

N +5O → 2N O (unbalanced)
2 2 2 5

Element Reactants Products Balanced?

N 1×2=2 2×2=4 2 ≠ 4, no

O 5 × 2 = 10 2 × 5 = 10 10 = 10, yes

The N atom balance has been upset by this change; it is restored by changing the coefficient for the reactant N2 to 2.

2N +5 O → 2N O
2 2 2 5

N atom balance upset but restored by changing the coefficient for the reactant N2 to 2.
Element Reactants Products Balanced?

N 2×2=4 2×2=4 4 = 4, yes

O 5 × 2 = 10 2 × 5 = 10 10 = 10, yes

The numbers of N and O atoms on either side of the equation are now equal, and so the equation is balanced.

 Exercise 4.1.1

Write a balanced equation for the decomposition of ammonium nitrate to form molecular nitrogen, molecular oxygen, and
water. (Hint: Balance oxygen last, since it is present in more than one molecule on the right side of the equation.)

Answer
2 NH NO → 2N +O +4 H O
4 3 2 2 2

Balancing Reactions Which Contain Polyatomics: Balancing Reactions Which Contain


Polyatomics(opens in new window) [youtu.be]

It is sometimes convenient to use fractions instead of integers as intermediate coefficients in the process of balancing a chemical
equation. When balance is achieved, all the equation’s coefficients may then be multiplied by a whole number to convert the
fractional coefficients to integers without upsetting the atom balance. For example, consider the reaction of ethane (C2H6) with
oxygen to yield H2O and CO2, represented by the unbalanced equation:
C H +O → H O + CO (unbalanced)
2 6 2 2 2

Following the usual inspection approach, one might first balance C and H atoms by changing the coefficients for the two product
species, as shown:
C H +O → 3 H O + 2 CO (unbalanced)
2 6 2 2 2

This results in seven O atoms on the product side of the equation, an odd number—no integer coefficient can be used with the O2
reactant to yield an odd number, so a fractional coefficient, , is used instead to yield a provisional balanced equation:
7

7
C H + O → 3 H O + 2 CO
2 6 2 2 2 2

A conventional balanced equation with integer-only coefficients is derived by multiplying each coefficient by 2:

2C H +7 O → 6 H O + 4 CO
2 6 2 2 2

Finally with regard to balanced equations, recall that convention dictates use of the smallest whole-number coefficients. Although
the equation for the reaction between molecular nitrogen and molecular hydrogen to produce ammonia is, indeed, balanced,

Access for free at OpenStax 4.1.4 https://chem.libretexts.org/@go/page/38156


3N +9 H → 6 NH
2 2 3

the coefficients are not the smallest possible integers representing the relative numbers of reactant and product molecules. Dividing
each coefficient by the greatest common factor, 3, gives the preferred equation:

N +3 H → 2 NH
2 2 3

 Phet Simulation
Use this interactive tutorial for additional practice balancing equations.

4.1.2: Additional Information in Chemical Equations


The physical states of reactants and products in chemical equations very often are indicated with a parenthetical abbreviation
following the formulas. Common abbreviations include s for solids, l for liquids, g for gases, and aq for substances dissolved in
water (aqueous solutions, as introduced in the preceding chapter). These notations are illustrated in the example equation here:

2 Na(s) + 2 H O(l) → 2 NaOH(aq) + H (g)


2 2

This equation represents the reaction that takes place when sodium metal is placed in water. The solid sodium reacts with liquid
water to produce molecular hydrogen gas and the ionic compound sodium hydroxide (a solid in pure form, but readily dissolved in
water).
Special conditions necessary for a reaction are sometimes designated by writing a word or symbol above or below the equation’s
arrow. For example, a reaction carried out by heating may be indicated by the uppercase Greek letter delta (Δ) over the arrow.
Δ

CaCO (s) −
−→ CaO(s) + CO (g)
3 2

Other examples of these special conditions will be encountered in more depth in later chapters.

4.1.3: Equations for Ionic Reactions


Given the abundance of water on earth, it stands to reason that a great many chemical reactions take place in aqueous media. When
ions are involved in these reactions, the chemical equations may be written with various levels of detail appropriate to their
intended use. To illustrate this, consider a reaction between ionic compounds taking place in an aqueous solution. When aqueous
solutions of CaCl and AgNO are mixed, a reaction takes place producing aqueous Ca(NO ) and solid AgCl:
2 3 3 2

CaCl (aq) + 2 AgNO (aq) → Ca (NO ) (aq) + 2 AgCl(s)


2 3 3 2

This balanced equation, derived in the usual fashion, is called a molecular equation because it doesn’t explicitly represent the ionic
species that are present in solution. When ionic compounds dissolve in water, they may dissociate into their constituent ions, which
are subsequently dispersed homogenously throughout the resulting solution (a thorough discussion of this important process is
provided in the chapter on solutions). Ionic compounds dissolved in water are, therefore, more realistically represented as
dissociated ions, in this case:
2+ −
CaCl (aq) → Ca (aq) + 2 Cl (aq)
2

+ −
2 AgNO (aq) → 2 Ag (aq) + 2 NO3 (aq)
3

2+ −
Ca (NO ) (aq) → Ca (aq) + 2 NO (aq)
3 2 3

Unlike these three ionic compounds, AgCl does not dissolve in water to a significant extent, as signified by its physical state
notation, (s).
Explicitly representing all dissolved ions results in a complete ionic equation. In this particular case, the formulas for the dissolved
ionic compounds are replaced by formulas for their dissociated ions:
2+ − + − 2+ −
Ca (aq) + 2 Cl (aq) + 2 Ag (aq) + 2 NO (aq) → Ca (aq) + 2 NO (aq) + 2 Ag Cl(s)
3 3

Access for free at OpenStax 4.1.5 https://chem.libretexts.org/@go/page/38156


Examining this equation shows that two chemical species are present in identical form on both sides of the arrow, Ca (aq) and 2 +

NO (aq) . These spectator ions—ions whose presence is required to maintain charge neutrality—are neither chemically nor

3

physically changed by the process, and so they may be eliminated from the equation to yield a more succinct representation called a
net ionic equation:
2+ − + − 2+ −
Ca (aq) + 2 Cl (aq) + 2 Ag (aq) + 2 NO (aq) → Ca (aq) + 2 NO (aq) + 2 AgCl(s)
3 3

− +
2 Cl (aq) + 2 Ag (aq) → 2 AgCl(s)

Following the convention of using the smallest possible integers as coefficients, this equation is then written:
− +
Cl (aq) + Ag (aq) → AgCl(s)

This net ionic equation indicates that solid silver chloride may be produced from dissolved chloride and silver(I) ions, regardless of
the source of these ions. These molecular and complete ionic equations provide additional information, namely, the ionic
compounds used as sources of Cl and Ag .
− +

 Example 4.1.2: Molecular and Ionic Equations

When carbon dioxide is dissolved in an aqueous solution of sodium hydroxide, the mixture reacts to yield aqueous sodium
carbonate and liquid water. Write balanced molecular, complete ionic, and net ionic equations for this process.

Solution
Begin by identifying formulas for the reactants and products and arranging them properly in chemical equation form:
CO (aq) + NaOH(aq) → Na CO (aq) + H O(l) (unbalanced)
2 2 3 2

Balance is achieved easily in this case by changing the coefficient for NaOH to 2, resulting in the molecular equation for this
reaction:
+
CO (aq) NaOH(aq) → Na CO (aq) + H O(l)
2 2 2 3 2

The two dissolved ionic compounds, NaOH and Na2CO3, can be represented as dissociated ions to yield the complete ionic
equation:
+ − + 2 −
CO (aq) + 2 Na (aq) + 2 OH (aq) → 2 Na (aq) + CO3 (aq) + H O(l)
2 2

+
Finally, identify the spectator ion(s), in this case Na (aq), and remove it from each side of the equation to generate the net ionic
equation:
+ − + 2−
CO (aq) + 2 Na (aq) + 2 OH (aq) → 2 Na (aq) + CO (aq) + H O(l)
2 3 2

− 2−
CO (aq) + 2 OH (aq) → CO (aq) + H O(l)
2 3 2

 Exercise 4.1.2
Diatomic chlorine and sodium hydroxide (lye) are commodity chemicals produced in large quantities, along with diatomic
hydrogen, via the electrolysis of brine, according to the following unbalanced equation:
electricity

NaCl(aq) + H O(l) −−−−−→ NaOH(aq) + H (g) + Cl (g)


2 2 2

Write balanced molecular, complete ionic, and net ionic equations for this process.

Answer
Balanced molecular equation:

2 NaCl(aq) + 2 H O(l) ⟶ 2 NaOH(aq) + H (g) + Cl (g)


2 2 2

Access for free at OpenStax 4.1.6 https://chem.libretexts.org/@go/page/38156


Balanced ionic equation:
+ − + −
2 Na (aq) + 2 Cl (aq) + 2 H O(l) ⟶ 2 Na (aq) + 2 OH (aq) + H (g) + Cl (g)
2 2 2

Balanced net ionic equation:


− −
2 Cl (aq) + 2 H O(l) ⟶ 2 OH (aq) + H (g) + Cl (g)
2 2 2

4.1.4: Key Concepts and Summary


Chemical equations are symbolic representations of chemical and physical changes. Formulas for the substances undergoing the
change (reactants) and substances generated by the change (products) are separated by an arrow and preceded by integer
coefficients indicating their relative numbers. Balanced equations are those whose coefficients result in equal numbers of atoms for
each element in the reactants and products. Chemical reactions in aqueous solution that involve ionic reactants or products may be
represented more realistically by complete ionic equations and, more succinctly, by net ionic equations.

Glossary
balanced equation
chemical equation with equal numbers of atoms for each element in the reactant and product

chemical equation
symbolic representation of a chemical reaction

coefficient
number placed in front of symbols or formulas in a chemical equation to indicate their relative amount

complete ionic equation


chemical equation in which all dissolved ionic reactants and products, including spectator ions, are explicitly represented by
formulas for their dissociated ions

molecular equation
chemical equation in which all reactants and products are represented as neutral substances

net ionic equation


chemical equation in which only those dissolved ionic reactants and products that undergo a chemical or physical change are
represented (excludes spectator ions)

product
substance formed by a chemical or physical change; shown on the right side of the arrow in a chemical equation

reactant
substance undergoing a chemical or physical change; shown on the left side of the arrow in a chemical equation

spectator ion
ion that does not undergo a chemical or physical change during a reaction, but its presence is required to maintain charge
neutrality

This page titled 4.1: Writing and Balancing Chemical Equations is shared under a CC BY 4.0 license and was authored, remixed, and/or curated
by OpenStax via source content that was edited to the style and standards of the LibreTexts platform; a detailed edit history is available upon
request.

Access for free at OpenStax 4.1.7 https://chem.libretexts.org/@go/page/38156


4.2: Classifying Chemical Reactions
 Learning Objectives
Define three common types of chemical reactions (precipitation, acid-base, and oxidation-reduction)
Classify chemical reactions as one of these three types given appropriate descriptions or chemical equations
Identify common acids and bases
Predict the solubility of common inorganic compounds by using solubility rules
Compute the oxidation states for elements in compounds

Humans interact with one another in various and complex ways, and we classify these interactions according to common patterns
of behavior. When two humans exchange information, we say they are communicating. When they exchange blows with their fists
or feet, we say they are fighting. Faced with a wide range of varied interactions between chemical substances, scientists have
likewise found it convenient (or even necessary) to classify chemical interactions by identifying common patterns of reactivity.
This module will provide an introduction to three of the most prevalent types of chemical reactions: precipitation, acid-base, and
oxidation-reduction.

4.2.1: Precipitation Reactions and Solubility Rules


A precipitation reaction is one in which dissolved substances react to form one (or more) solid products. Many reactions of this
type involve the exchange of ions between ionic compounds in aqueous solution and are sometimes referred to as double
displacement, double replacement, or metathesis reactions. These reactions are common in nature and are responsible for the
formation of coral reefs in ocean waters and kidney stones in animals. They are used widely in industry for production of a number
of commodity and specialty chemicals. Precipitation reactions also play a central role in many chemical analysis techniques,
including spot tests used to identify metal ions and gravimetric methods for determining the composition of matter (see the last
module of this chapter).
The extent to which a substance may be dissolved in water, or any solvent, is quantitatively expressed as its solubility, defined as
the maximum concentration of a substance that can be achieved under specified conditions. Substances with relatively large
solubilities are said to be soluble. A substance will precipitate when solution conditions are such that its concentration exceeds its
solubility. Substances with relatively low solubilities are said to be insoluble, and these are the substances that readily precipitate
from solution. More information on these important concepts is provided in the text chapter on solutions. For purposes of
predicting the identities of solids formed by precipitation reactions, one may simply refer to patterns of solubility that have been
observed for many ionic compounds (Table 4.2.1).
Table 4.2.1 : Solubilities of Common Ionic Compounds in Water
Soluble compounds contain Exceptions to these solubility rules include

group 1 metal cations (Li+, Na+, K+, Rb+, and Cs+) and ammonium
ion (NH )
+
4

the halide ions (Cl−, Br−, and I−) halides of Ag+, Hg , and Pb2+
2+
2

the acetate (C H O ) , bicarbonate (HCO ) , nitrate (NO ), and


2 3

2

3

3
sulfates of Ag+, Ba2+, Ca2+, Hg , Pb2+, and Sr2+
2+
2

chlorate (ClO ) ions



3

the sulfate (SO ) ion



4

Insoluble compounds contain Exceptions to these insolubility rules include

carbonate (CO ) , chromate (CrO


2−
3
2−
4
) , phosphate (PO
3−
4
) , and compounds of these anions with group 1 metal cations and
sulfide (S2−) ions ammonium ion
hydroxide ion (OH−) hydroxides of group 1 metal cations and Ba2+

A vivid example of precipitation is observed when solutions of potassium iodide and lead nitrate are mixed, resulting in the
formation of solid lead iodide:

2 KI(aq) + Pb(NO ) (aq) → PbI (s) + 2 KNO (aq)


3 2 2 3

4.2.1 https://chem.libretexts.org/@go/page/38155
This observation is consistent with the solubility guidelines: The only insoluble compound among all those involved is lead iodide,
one of the exceptions to the general solubility of iodide salts.
The net ionic equation representing this reaction is:
2+ −
Pb (aq) + 2 I (aq) → PbI (s)
2

Lead iodide is a bright yellow solid that was formerly used as an artist’s pigment known as iodine yellow (Figure 4.2.1 ). The
properties of pure PbI2 crystals make them useful for fabrication of X-ray and gamma ray detectors.

Figure 4.2.1 : A precipitate of PbI2 forms when solutions containing Pb2+ and I− are mixed. (credit: Der Kreole/Wikimedia
Commons)
A photograph is shown of a yellow green opaque substance swirled through a clear, colorless liquid in a test tube.
The solubility guidelines in Table 4.2.1 may be used to predict whether a precipitation reaction will occur when solutions of
soluble ionic compounds are mixed together. One merely needs to identify all the ions present in the solution and then consider if
possible cation/anion pairing could result in an insoluble compound. For example, mixing solutions of silver nitrate and sodium
fluoride will yield a solution containing Ag+, NO , Na+, and F− ions. Aside from the two ionic compounds originally present in

3

the solutions, AgNO3 and NaF, two additional ionic compounds may be derived from this collection of ions: NaNO3 and AgF. The
solubility guidelines indicate all nitrate salts are soluble but that AgF is one of the exceptions to the general solubility of fluoride
salts. A precipitation reaction, therefore, is predicted to occur, as described by the following equations:

NaF(aq) + AgNO (aq) → AgF(s) + NaNO (aq) (molecular)


3 3

+ −
Ag (aq) + F (aq) → AgF(s) (net ionic)

 Example 4.2.1: Predicting Precipitation Reactions


Predict the result of mixing reasonably concentrated solutions of the following ionic compounds. If precipitation is expected,
write a balanced net ionic equation for the reaction.
a. potassium sulfate and barium nitrate
b. lithium chloride and silver acetate
c. lead nitrate and ammonium carbonate

Solution
(a) The two possible products for this combination are KNO3 and BaSO4. The solubility guidelines indicate BaSO4 is
insoluble, and so a precipitation reaction is expected. The net ionic equation for this reaction, derived in the manner detailed in
the previous module, is
2+ 2−
Ba (aq) + SO (aq) → BaSO (s)
4 4

4.2.2 https://chem.libretexts.org/@go/page/38155
(b) The two possible products for this combination are LiC2H3O2 and AgCl. The solubility guidelines indicate AgCl is
insoluble, and so a precipitation reaction is expected. The net ionic equation for this reaction, derived in the manner detailed in
the previous module, is
+ −
Ag (aq) + Cl (aq) → AgCl(s)

(c) The two possible products for this combination are PbCO3 and NH4NO3. The solubility guidelines indicate PbCO3 is
insoluble, and so a precipitation reaction is expected. The net ionic equation for this reaction, derived in the manner detailed in
the previous module, is
2+ 2−
Pb (aq) + CO3 (aq) → PbCO (s)
3

 Exercise 4.2.1

Which solution could be used to precipitate the barium ion, Ba2+, in a water sample: sodium chloride, sodium hydroxide, or
sodium sulfate? What is the formula for the expected precipitate?

Answer
sodium sulfate, BaSO4

4.2.2: Acid-Base Reactions


An acid-base reaction is one in which a hydrogen ion, H+, is transferred from one chemical species to another. Such reactions are of
central importance to numerous natural and technological processes, ranging from the chemical transformations that take place
within cells and the lakes and oceans, to the industrial-scale production of fertilizers, pharmaceuticals, and other substances
essential to society. The subject of acid-base chemistry, therefore, is worthy of thorough discussion, and a full chapter is devoted to
this topic later in the text.
For purposes of this brief introduction, we will consider only the more common types of acid-base reactions that take place in
aqueous solutions. In this context, an acid is a substance that will dissolve in water to yield hydronium ions, H3O+. As an example,
consider the equation shown here:
− +
HCl(aq) + H O(aq) → Cl (aq) + H O (aq)
2 3

The process represented by this equation confirms that hydrogen chloride is an acid. When dissolved in water, H3O+ ions are
produced by a chemical reaction in which H+ ions are transferred from HCl molecules to H2O molecules (Figure 4.2.2).

4.2.3 https://chem.libretexts.org/@go/page/38155
Figure 4.2.2 : When hydrogen chloride gas dissolves in water, (a) it reacts as an acid, transferring protons to water molecules to
yield (b) hydronium ions (and solvated chloride ions)
This figure shows two flasks, labeled a and b. The flasks are both sealed with stoppers and are nearly three-quarters full of a liquid.
Flask a is labeled H C l followed by g in parentheses. In the liquid there are approximately twenty space-filling molecular models
composed of one red sphere and two smaller attached white spheres. The label H subscript 2 O followed by a q in parentheses is
connected with a line to one of these models. In the space above the liquid in the flask, four space filling molecular models
composed of one larger green sphere to which a smaller white sphere is bonded are shown. To one of these models, the label H C l
followed by g in parentheses is attached with a line segment. An arrow is drawn from the space above the liquid pointing down into
the liquid below. Flask b is labeled H subscript 3 O superscript positive sign followed by a q in parentheses. This is followed by a
plus sign and C l superscript negative sign which is also followed by a q in parentheses. In this flask, no molecules are shown in the
open space above the liquid. A label, C l superscript negative sign followed by a q in parentheses, is connected with a line segment
to a green sphere. This sphere is surrounded by four molecules composed each of one red sphere and two white smaller spheres. A
few of these same molecules appear separate from the green spheres in the liquid. A line segment connects one of them to the label
H subscript 2 O which is followed by l in parentheses. There are a few molecules formed from one central larger red sphere to
which three smaller white spheres are bonded. A line segment is drawn from one of these to the label H subscript 3 O superscript
positive sign, followed by a q in parentheses.
The nature of HCl is such that its reaction with water as just described is essentially 100% efficient: Virtually every HCl molecule
that dissolves in water will undergo this reaction. Acids that completely react in this fashion are called strong acids, and HCl is one
among just a handful of common acid compounds that are classified as strong (Table 4.2.1). A far greater number of compounds
behave as weak acids and only partially react with water, leaving a large majority of dissolved molecules in their original form and
generating a relatively small amount of hydronium ions. Weak acids are commonly encountered in nature, being the substances
partly responsible for the tangy taste of citrus fruits, the stinging sensation of insect bites, and the unpleasant smells associated with
body odor. A familiar example of a weak acid is acetic acid, the main ingredient in food vinegars:
− +
CH CO H(aq) + H O(l) ⇌ CH CO (aq) + H O (aq)
3 2 2 3 2 3

When dissolved in water under typical conditions, only about 1% of acetic acid molecules are present in the ionized form,
CH CO
3
(Figure 4.2.3). (The use of a double-arrow in the equation above denotes the partial reaction aspect of this process, a

concept addressed fully in the chapters on chemical equilibrium.)

4.2.4 https://chem.libretexts.org/@go/page/38155
Figure 4.2.3 : (a) Fruits such as oranges, lemons, and grapefruit contain the weak acid citric acid. (b) Vinegars contain the weak
acid acetic acid. (credit a: modification of work by Scott Bauer; credit b: modification of work by Brücke-Osteuropa/Wikimedia
Commons)
This figure contains two images, each with an associated structural formula provided in the lower left corner of the image. The first
image is a photograph of a variety of thinly sliced, circular cross sections of citrus fruits ranging in color for green to yellow, to
orange and reddish-orange. The slices are closely packed on a white background. The structural formula with this picture shows a
central chain of five C atoms. The leftmost C atom has an O atom double bonded above and to the left and a singly bonded O atom
below and to the left. This single bonded O atom has an H atom indicated in red on its left side which is highlighted in pink. The
second C atom moving to the right has H atoms bonded above and below. The third C atom has a single bonded O atom above
which has an H atom on its right. This third C atom has a C atom bonded below it which has an O atom double bonded below and
to the left and a singly bonded O atom below and to the right. An H atom appears in red and is highlighted in pink to the right of
the singly bonded O atom. The fourth C atom has H atoms bonded above and below. The fifth C atom is at the right end of the
structure. It has an O atom double bonded above and to the right and a singly bonded O atom below and to the right. This single
bonded O atom has a red H atom on its right side which is highlighted in pink. The second image is a photograph of bottles of
vinegar. The bottles are labeled, “Balsamic Vinegar,” and appear to be clear and colorless. The liquid in this bottle appears to be
brown. The structural formula that appears with this image shows a chain of two C atoms. The leftmost C atom has H atoms
bonded above, below, and to the left. The C atom on the right has a doubly bonded O atom above and to the right and a singly
bonded O atom below and to the right. This O atom has an H atom bonded to its right which is highlighted in pink.
Table 4.2.2 : Common Strong Acids
Compound Formula Name in Aqueous Solution

HBr hydrobromic acid

HCl hydrochloric acid

HI hydroiodic acid

HNO3 nitric acid

HClO4 perchloric acid

H2SO4 sulfuric acid

A base is a substance that will dissolve in water to yield hydroxide ions, OH−. The most common bases are ionic compounds
composed of alkali or alkaline earth metal cations (groups 1 and 2) combined with the hydroxide ion—for example, NaOH and
Ca(OH)2. When these compounds dissolve in water, hydroxide ions are released directly into the solution. For example, KOH and
Ba(OH)2 dissolve in water and dissociate completely to produce cations (K+ and Ba2+, respectively) and hydroxide ions, OH−.
These bases, along with other hydroxides that completely dissociate in water, are considered strong bases.
Consider as an example the dissolution of lye (sodium hydroxide) in water:
+ −
NaOH(s) → Na (aq) + OH (aq)

This equation confirms that sodium hydroxide is a base. When dissolved in water, NaOH dissociates to yield Na+ and OH− ions.
This is also true for any other ionic compound containing hydroxide ions. Since the dissociation process is essentially complete
when ionic compounds dissolve in water under typical conditions, NaOH and other ionic hydroxides are all classified as strong
bases.

4.2.5 https://chem.libretexts.org/@go/page/38155
Unlike ionic hydroxides, some compounds produce hydroxide ions when dissolved by chemically reacting with water molecules. In
all cases, these compounds react only partially and so are classified as weak bases. These types of compounds are also abundant in
nature and important commodities in various technologies. For example, global production of the weak base ammonia is typically
well over 100 metric tons annually, being widely used as an agricultural fertilizer, a raw material for chemical synthesis of other
compounds, and an active ingredient in household cleaners (Figure 4.2.4). When dissolved in water, ammonia reacts partially to
yield hydroxide ions, as shown here:
+ −
NH (aq) + H O(l) ⇌ NH (aq) + OH (aq)
3 2 4

This is, by definition, an acid-base reaction, in this case involving the transfer of H+ ions from water molecules to ammonia
molecules. Under typical conditions, only about 1% of the dissolved ammonia is present as NH ions. +

Figure 4.2.4 : Ammonia is a weak base used in a variety of applications. (a) Pure ammonia is commonly applied as an agricultural
fertilizer. (b) Dilute solutions of ammonia are effective household cleansers. (credit a: modification of work by National Resources
Conservation Service; credit b: modification of work by pat00139)
This photograph shows a large agricultural tractor in a field pulling a field sprayer and a large, white cylindrical tank which is
labeled “Caution Ammonia.”
The chemical reactions described in which acids and bases dissolved in water produce hydronium and hydroxide ions, respectively,
are, by definition, acid-base reactions. In these reactions, water serves as both a solvent and a reactant. A neutralization reaction is
a specific type of acid-base reaction in which the reactants are an acid and a base, the products are often a salt and water, and
neither reactant is the water itself:

acid + base → salt + water

To illustrate a neutralization reaction, consider what happens when a typical antacid such as milk of magnesia (an aqueous
suspension of solid Mg(OH)2) is ingested to ease symptoms associated with excess stomach acid (HCl):

Mg (OH) (s) + 2 HCl(aq) → MgCl (aq) + 2 H O(l).


2 2 2

Note that in addition to water, this reaction produces a salt, magnesium chloride.

 Example 4.2.2: Writing Equations for Acid-Base Reactions


Write balanced chemical equations for the acid-base reactions described here:
a. the weak acid hydrogen hypochlorite reacts with water
b. a solution of barium hydroxide is neutralized with a solution of nitric acid

Solution
(a) The two reactants are provided, HOCl and H2O. Since the substance is reported to be an acid, its reaction with water will
involve the transfer of H+ from HOCl to H2O to generate hydronium ions, H3O+ and hypochlorite ions, OCl−.
− +
HOCl(aq) + H O(l) ⇌ OCl (aq) + H O (aq)
2 3

A double-arrow is appropriate in this equation because it indicates the HOCl is a weak acid that has not reacted completely.
(b) The two reactants are provided, Ba(OH)2 and HNO3. Since this is a neutralization reaction, the two products will be water
and a salt composed of the cation of the ionic hydroxide (Ba2+) and the anion generated when the acid transfers its hydrogen
ion (NO ).

3

4.2.6 https://chem.libretexts.org/@go/page/38155
Ba (OH) (aq) + 2 HNO (aq) → Ba (NO ) (aq) + 2 H O(l)
2 3 3 2 2

 Exercise 4.2.21

Write the net ionic equation representing the neutralization of any strong acid with an ionic hydroxide. (Hint: Consider the ions
produced when a strong acid is dissolved in water.)

Answer
+ −
H O (aq) + OH (aq) → 2 H O(l)
3 2

Explore the microscopic view of strong and weak acids and bases.

4.2.3: Oxidation-Reduction Reactions


Earth’s atmosphere contains about 20% molecular oxygen, O2, a chemically reactive gas that plays an essential role in the
metabolism of aerobic organisms and in many environmental processes that shape the world. The term oxidation was originally
used to describe chemical reactions involving O2, but its meaning has evolved to refer to a broad and important reaction class
known as oxidation-reduction (redox) reactions. A few examples of such reactions will be used to develop a clear picture of this
classification.
Some redox reactions involve the transfer of electrons between reactant species to yield ionic products, such as the reaction
between sodium and chlorine to yield sodium chloride:

2 Na(s) + Cl (g) → 2 NaCl(s)


2

It is helpful to view the process with regard to each individual reactant, that is, to represent the fate of each reactant in the form of
an equation called a half-reaction:
+ −
2 Na(s) → 2 Na (s) + 2 e

− −
Cl (g) + 2 e → 2 Cl (s)
2

These equations show that Na atoms lose electrons while Cl atoms (in the Cl2 molecule) gain electrons, the “s” subscripts for the
resulting ions signifying they are present in the form of a solid ionic compound. For redox reactions of this sort, the loss and gain
of electrons define the complementary processes that occur:
oxidation = loss of electrons (4.2.1)

reduction = gain of electrons (4.2.2)

In this reaction, then, sodium is oxidized and chlorine undergoes reduction. Viewed from a more active perspective, sodium
functions as a reducing agent (reductant), since it provides electrons to (or reduces) chlorine. Likewise, chlorine functions as an
oxidizing agent (oxidant), as it effectively removes electrons from (oxidizes) sodium.
reducing agent = species that is oxidized (4.2.3)

oxidizing agent = species that is reduced (4.2.4)

Some redox processes, however, do not involve the transfer of electrons. Consider, for example, a reaction similar to the one
yielding NaCl:

H (g) + Cl (g) → 2 HCl(g)


2 2

The product of this reaction is a covalent compound, so transfer of electrons in the explicit sense is not involved. To clarify the
similarity of this reaction to the previous one and permit an unambiguous definition of redox reactions, a property called oxidation
number has been defined. The oxidation number (or oxidation state) of an element in a compound is the charge its atoms would
possess if the compound was ionic. The following guidelines are used to assign oxidation numbers to each element in a molecule or
ion.
1. The oxidation number of an atom in an elemental substance is zero.
2. The oxidation number of a monatomic ion is equal to the ion’s charge.
3. Oxidation numbers for common nonmetals are usually assigned as follows:

4.2.7 https://chem.libretexts.org/@go/page/38155
Hydrogen: +1 when combined with nonmetals, −1 when combined with metals
1
Oxygen: −2 in most compounds, sometimes −1 (so-called peroxides, O 2−
2
), very rarely − (so-called superoxides, O ),−
2
2
positive values when combined with F (values vary)
Halogens: −1 for F always, −1 for other halogens except when combined with oxygen or other halogens (positive oxidation
numbers in these cases, varying values)
4. The sum of oxidation numbers for all atoms in a molecule or polyatomic ion equals the charge on the molecule or ion.
Note: The proper convention for reporting charge is to write the number first, followed by the sign (e.g., 2+), while oxidation
number is written with the reversed sequence, sign followed by number (e.g., +2). This convention aims to emphasize the
distinction between these two related properties.

 Example 4.2.3: Assigning Oxidation Numbers

Follow the guidelines in this section of the text to assign oxidation numbers to all the elements in the following species:
a. H2S
b. SO 2−
3

c. Na2SO4

Solution
(a) According to guideline 1, the oxidation number for H is +1.
Using this oxidation number and the compound’s formula, guideline 4 may then be used to calculate the oxidation number for
sulfur:
charge on H S = 0 = (2 × +1) + (1 × x)
2

x = 0 − (2 × +1) = −2

(b) Guideline 3 suggests the oxidation number for oxygen is −2.


Using this oxidation number and the ion’s formula, guideline 4 may then be used to calculate the oxidation number for sulfur:
2−
charge on SO = −2 = (3 × −2) + (1 × x)
3

x = −2 − (3 × −2) = +4

(c) For ionic compounds, it’s convenient to assign oxidation numbers for the cation and anion separately.
According to guideline 2, the oxidation number for sodium is +1.
Assuming the usual oxidation number for oxygen (−2 per guideline 3), the oxidation number for sulfur is calculated as directed
by guideline 4:
2−
charge on SO = −2 = (4 × −2) + (1 × x)
4

x = −2 − (4 × −2) = +6

 Exercise 4.2.3

Assign oxidation states to the elements whose atoms are underlined in each of the following compounds or ions:
a. KNO3
b. AlH3
c. –
NH

+
4

d. H –
2 PO
– 4

Answer a
N, +5
Answer b

4.2.8 https://chem.libretexts.org/@go/page/38155
Al, +3
Answer c
N, −3
Answer d
P, +5

Using the oxidation number concept, an all-inclusive definition of redox reaction has been established. Oxidation-reduction (redox)
reactions are those in which one or more elements involved undergo a change in oxidation number. While the vast majority of
redox reactions involve changes in oxidation number for two or more elements, a few interesting exceptions to this rule do exist as
shown below\). Definitions for the complementary processes of this reaction class are correspondingly revised as shown here:
oxidation = increase in oxidation number (4.2.5)

reduction = decrease in oxidation number (4.2.6)

Returning to the reactions used to introduce this topic, they may now both be identified as redox processes. In the reaction between
sodium and chlorine to yield sodium chloride, sodium is oxidized (its oxidation number increases from 0 in Na to +1 in NaCl) and
chlorine is reduced (its oxidation number decreases from 0 in Cl2 to −1 in NaCl). In the reaction between molecular hydrogen and
chlorine, hydrogen is oxidized (its oxidation number increases from 0 in H2 to +1 in HCl) and chlorine is reduced (its oxidation
number decreases from 0 in Cl2 to −1 in HCl).
Several subclasses of redox reactions are recognized, including combustion reactions in which the reductant (also called a fuel) and
oxidant (often, but not necessarily, molecular oxygen) react vigorously and produce significant amounts of heat, and often light, in
the form of a flame. Solid rocket-fuel reactions such as the one depicted below are combustion processes. A typical propellant
reaction in which solid aluminum is oxidized by ammonium perchlorate is represented by this equation:

10 Al(s) + 6 NH ClO (s) → 4 Al O (s) + 2 AlCl (s) + 12 H O(g) + 3 N (g)


4 4 2 3 3 2 2

Watch a brief video showing the test firing of a small-scale, prototype, hybrid rocket engine planned for use in the new Space
Launch System being developed by NASA. The first engines firing at 3 s (green flame) use a liquid fuel/oxidant mixture, and
the second, more powerful engines firing at 4 s (yellow flame) use a solid mixture.

NASA Tests Model of Powerful New R…


R…

Single-displacement (replacement) reactions are redox reactions in which an ion in solution is displaced (or replaced) via the
oxidation of a metallic element. One common example of this type of reaction is the acid oxidation of certain metals:

Zn(s) + 2 HCl(aq) → ZnCl (aq) + H (g)


2 2

Metallic elements may also be oxidized by solutions of other metal salts; for example:

Cu(s) + 2 AgNO (aq) → Cu(NO ) (aq) + 2 Ag(s)


3 3 2

4.2.9 https://chem.libretexts.org/@go/page/38155
This reaction may be observed by placing copper wire in a solution containing a dissolved silver salt. Silver ions in solution are
reduced to elemental silver at the surface of the copper wire, and the resulting Cu2+ ions dissolve in the solution to yield a
characteristic blue color (Figure 4.2.4).

Figure 4.2.4 : (a) A copper wire is shown next to a solution containing silver(I) ions. (b) Displacement of dissolved silver ions by
copper ions results in (c) accumulation of gray-colored silver metal on the wire and development of a blue color in the solution, due
to dissolved copper ions. (credit: modification of work by Mark Ott)
This figure contains three photographs. In a, a coiled copper wire is shown beside a test tube filled with a clear, colorless liquid. In
b, the wire has been inserted into the test tube with the clear, colorless liquid. In c, the test tube contains a light blue liquid and the
coiled wire appears to have a fuzzy silver gray coating.

 Example 4.2.4: Describing Redox Reactions

Identify which equations represent redox reactions, providing a name for the reaction if appropriate. For those reactions
identified as redox, name the oxidant and reductant.
a. ZnCO (s) → ZnO(s) + CO (g)
3 2

b. 2 Ga(l) + 3 Br (l) → 2 GaBr (s)


2 3

c. 2 H O (aq) → 2 H O(l) + O (g)


2 2 2 2

d. BaCl (aq) + K SO (aq) → BaSO (s) + 2 KCl(aq)


2 2 4 4

e. C H (g) + 3 O (g) → 2 CO (g) + 2 H O(l)


2 4 2 2 2

Solution
Redox reactions are identified per definition if one or more elements undergo a change in oxidation number.
a. This is not a redox reaction, since oxidation numbers remain unchanged for all elements.
b. This is a redox reaction. Gallium is oxidized, its oxidation number increasing from 0 in Ga(l) to +3 in GaBr3(s). The
reducing agent is Ga(l). Bromine is reduced, its oxidation number decreasing from 0 in Br2(l) to −1 in GaBr3(s). The
oxidizing agent is Br2(l).
c. This is a redox reaction. It is a particularly interesting process, as it involves the same element, oxygen, undergoing both
oxidation and reduction (a so-called disproportionation reaction). Oxygen is oxidized, its oxidation number increasing from
−1 in H2O2(aq) to 0 in O2(g). Oxygen is also reduced, its oxidation number decreasing from −1 in H2O2(aq) to −2 in
H2O(l). For disproportionation reactions, the same substance functions as an oxidant and a reductant.
d. This is not a redox reaction, since oxidation numbers remain unchanged for all elements.
e. This is a redox reaction (combustion). Carbon is oxidized, its oxidation number increasing from −2 in C2H4(g) to +4 in
CO2(g). The reducing agent (fuel) is C2H4(g). Oxygen is reduced, its oxidation number decreasing from 0 in O2(g) to −2 in
H2O(l). The oxidizing agent is O2(g).

 Exercise 4.2.4

This equation describes the production of tin(II) chloride:

Sn(s) + 2 HCl(g) → SnCl (s) + H (g)


2 2

Is this a redox reaction? If so, provide a more specific name for the reaction if appropriate, and identify the oxidant and
reductant.

Answer

4.2.10 https://chem.libretexts.org/@go/page/38155
Yes, a single-replacement reaction. Sn(s) is the reductant, HCl(g) is the oxidant.

4.2.3.1: Balancing Redox Reactions via the Half-Reaction Method


Redox reactions that take place in aqueous media often involve water, hydronium ions, and hydroxide ions as reactants or products.
Although these species are not oxidized or reduced, they do participate in chemical change in other ways (e.g., by providing the
elements required to form oxyanions). Equations representing these reactions are sometimes very difficult to balance by inspection,
so systematic approaches have been developed to assist in the process. One very useful approach is to use the method of half-
reactions, which involves the following steps:
1. Write the two half-reactions representing the redox process.
2. Balance all elements except oxygen and hydrogen.
3. Balance oxygen atoms by adding H2O molecules.
4. Balance hydrogen atoms by adding H+ ions.
5. Balance charge1 by adding electrons.
6. If necessary, multiply each half-reaction’s coefficients by the smallest possible integers to yield equal numbers of electrons in
each.
7. Add the balanced half-reactions together and simplify by removing species that appear on both sides of the equation.
8. For reactions occurring in basic media (excess hydroxide ions), carry out these additional steps:
Add OH− ions to both sides of the equation in numbers equal to the number of H+ ions.
On the side of the equation containing both H+ and OH− ions, combine these ions to yield water molecules.
Simplify the equation by removing any redundant water molecules.
9. Finally, check to see that both the number of atoms and the total charges2 are balanced.

 Example 4.2.5: Balancing Redox Reactions in Acidic Solution

Write a balanced equation for the reaction between dichromate ion and iron(II) to yield iron(III) and chromium(III) in acidic
solution.
2− 2+ 3+ 3+
Cr O + Fe → Cr + Fe
2 7

Solution
Write the two half-reactions.
Each half-reaction will contain one reactant and one product with one element in common.
2+ 3+
Fe → Fe
2− 3+
Cr O7 → Cr
2

Balance all elements except oxygen and hydrogen. The iron half-reaction is already balanced, but the chromium half-reaction
shows two Cr atoms on the left and one Cr atom on the right. Changing the coefficient on the right side of the equation to 2
achieves balance with regard to Cr atoms.
2+ 3+
Fe → Fe
2− 3+
Cr O7 → 2 Cr
2

Balance oxygen atoms by adding H2O molecules. The iron half-reaction does not contain O atoms. The chromium half-reaction
shows seven O atoms on the left and none on the right, so seven water molecules are added to the right side.
2+ 3+
Fe → Fe
2− 3+
Cr O → 2 Cr +7 H O
2 7 2

+
Balance hydrogen atoms by adding H ions. The iron half-reaction does not contain H atoms. The chromium half-reaction shows 14
H atoms on the right and none on the left, so 14 hydrogen ions are added to the left side.

4.2.11 https://chem.libretexts.org/@go/page/38155
2+ 3+
Fe → Fe
2− + 3+
Cr O + 14 H → 2 Cr +7 H O
2 7 2

Balance charge by adding electrons. The iron half-reaction shows a total charge of 2+ on the left side (1 Fe2+ ion) and 3+ on the
right side (1 Fe3+ ion). Adding one electron to the right side bring that side’s total charge to (3+) + (1−) = 2+, and charge balance is
achieved.
The chromium half-reaction shows a total charge of (1 × 2−) + (14 × 1+) = 12+ on the left side (1 Cr O ion and 14 H+ ions). 2
2−

The total charge on the right side is (2 × 3+) = 6 + (2 Cr3+ ions). Adding six electrons to the left side will bring that side’s total
charge to (12+ + 6−) = 6+, and charge balance is achieved.

2+ 3+ −
Fe → Fe +e
2− + − 3+
Cr O + 14 H +6 e → 2 Cr +7 H O
2 7 2

Multiply the two half-reactions so the number of electrons in one reaction equals the number of electrons in the other reaction. To
be consistent with mass conservation, and the idea that redox reactions involve the transfer (not creation or destruction) of
electrons, the iron half-reaction’s coefficient must be multiplied by 6.

2+ 3+ −
6 Fe → 6 Fe +6 e
2− − + 3+
Cr O +6 e + 14 H → 2 Cr +7 H O
2 7 2

Add the balanced half-reactions and cancel species that appear on both sides of the equation.
2+ 2− − + 3+ − 3+
6 Fe + Cr O +6 e + 14 H → 6 Fe +6 e + 2 Cr +7 H O
2 7 2

Only the six electrons are redundant species. Removing them from each side of the equation yields the simplified, balanced
equation here:
2+ 2− + 3+ 3+
6 Fe + Cr O + 14 H → 6 Fe + 2 Cr +7 H O
2 7 2

A final check of atom and charge balance confirms the equation is balanced.
Final check of atom and charge balance confirms the equation is balanced.
Reactants Products

Fe 6 6

Cr 2 2

O 7 7

H 14 14

charge 24+ 24+

 Exercise 4.2.5
In acidic solution, hydrogen peroxide reacts with Fe2+ to produce Fe3+ and H2O. Write a balanced equation for this reaction.

Answer
+ 2+ 3+
H O (aq) + 2 H (aq) + 2 Fe → 2 H O(l) + 2 Fe
2 2 2

Summary
Chemical reactions are classified according to similar patterns of behavior. A large number of important reactions are included in
three categories: precipitation, acid-base, and oxidation-reduction (redox). Precipitation reactions involve the formation of one or
more insoluble products. Acid-base reactions involve the transfer of hydrogen ions between reactants. Redox reactions involve a

4.2.12 https://chem.libretexts.org/@go/page/38155
change in oxidation number for one or more reactant elements. Writing balanced equations for some redox reactions that occur in
aqueous solutions is simplified by using a systematic approach called the half-reaction method.

4.2.3.1: Footnotes
1. 1 The requirement of “charge balance” is just a specific type of “mass balance” in which the species in question are electrons.
An equation must represent equal numbers of electrons on the reactant and product sides, and so both atoms and charges must
be balanced.
2. 2 The requirement of “charge balance” is just a specific type of “mass balance” in which the species in question are electrons.
An equation must represent equal numbers of electrons on the reactant and product sides, and so both atoms and charges must
be balanced.

4.2.3.2: Glossary

acid
substance that produces H3O+ when dissolved in water

acid-base reaction
reaction involving the transfer of a hydrogen ion between reactant species

base
substance that produces OH− when dissolved in water

combustion reaction
vigorous redox reaction producing significant amounts of energy in the form of heat and, sometimes, light

half-reaction
an equation that shows whether each reactant loses or gains electrons in a reaction.

insoluble
of relatively low solubility; dissolving only to a slight extent

neutralization reaction
reaction between an acid and a base to produce salt and water

oxidation
process in which an element’s oxidation number is increased by loss of electrons

oxidation-reduction reaction
(also, redox reaction) reaction involving a change in oxidation number for one or more reactant elements

oxidation number
(also, oxidation state) the charge each atom of an element would have in a compound if the compound were ionic

oxidizing agent
(also, oxidant) substance that brings about the oxidation of another substance, and in the process becomes reduced

precipitate
insoluble product that forms from reaction of soluble reactants

precipitation reaction
reaction that produces one or more insoluble products; when reactants are ionic compounds, sometimes called double-
displacement or metathesis

reduction
process in which an element’s oxidation number is decreased by gain of electrons

4.2.13 https://chem.libretexts.org/@go/page/38155
reducing agent
(also, reductant) substance that brings about the reduction of another substance, and in the process becomes oxidized

salt
ionic compound that can be formed by the reaction of an acid with a base that contains a cation and an anion other than
hydroxide or oxide

single-displacement reaction
(also, replacement) redox reaction involving the oxidation of an elemental substance by an ionic species

soluble
of relatively high solubility; dissolving to a relatively large extent

solubility
the extent to which a substance may be dissolved in water, or any solvent

strong acid
acid that reacts completely when dissolved in water to yield hydronium ions

strong base
base that reacts completely when dissolved in water to yield hydroxide ions

weak acid
acid that reacts only to a slight extent when dissolved in water to yield hydronium ions
weak base
base that reacts only to a slight extent when dissolved in water to yield hydroxide ions

4.2: Classifying Chemical Reactions is shared under a CC BY 4.0 license and was authored, remixed, and/or curated by LibreTexts.

4.2.14 https://chem.libretexts.org/@go/page/38155
4.3: Reaction Stoichiometry
 Learning Objectives
Explain the concept of stoichiometry as it pertains to chemical reactions
Use balanced chemical equations to derive stoichiometric factors relating amounts of reactants and products
Perform stoichiometric calculations involving mass, moles, and solution molarity

A balanced chemical equation provides a great deal of information in a very succinct format. Chemical formulas provide the
identities of the reactants and products involved in the chemical change, allowing classification of the reaction. Coefficients
provide the relative numbers of these chemical species, allowing a quantitative assessment of the relationships between the
amounts of substances consumed and produced by the reaction. These quantitative relationships are known as the reaction’s
stoichiometry, a term derived from the Greek words stoicheion (meaning “element”) and metron (meaning “measure”). In this
module, the use of balanced chemical equations for various stoichiometric applications is explored.
The general approach to using stoichiometric relationships is similar in concept to the way people go about many common
activities. Cooking, for example, offers an appropriate comparison. Suppose a recipe for making eight pancakes calls for 1 cup
3
pancake mix, cup milk, and one egg. The “equation” representing the preparation of pancakes per this recipe is
4

3
1 cup mix + cup milk + 1 egg → 8 pancakes (4.3.1)
4

If two dozen pancakes are needed for a big family breakfast, the ingredient amounts must be increased proportionally according to
the amounts given in the recipe. For example, the number of eggs required to make 24 pancakes is
1 egg
24 pancakes × = 3 eggs (4.3.2)
8 pancakes

Balanced chemical equations are used in much the same fashion to determine the amount of one reactant required to react with a
given amount of another reactant, or to yield a given amount of product, and so forth. The coefficients in the balanced equation are
used to derive stoichiometric factors that permit computation of the desired quantity. To illustrate this idea, consider the production
of ammonia by reaction of hydrogen and nitrogen:
N (g) + 3 H (g) → 2 NH (g) (4.3.3)
2 2 3

This equation shows that ammonia molecules are produced from hydrogen molecules in a 2:3 ratio, and stoichiometric factors may
be derived using any amount (number) unit:
2 NH molecules 2 doz NH molecules 2 mol NH molecules
3 3 3
or or (4.3.4)
3H molecules 3 doz H molecules 3 mol H molecules
2 2 2

These stoichiometric factors can be used to compute the number of ammonia molecules produced from a given number of
hydrogen molecules, or the number of hydrogen molecules required to produce a given number of ammonia molecules. Similar
factors may be derived for any pair of substances in any chemical equation.

 Example 4.3.1: Moles of Reactant Required in a Reaction


How many moles of I2 are required to react with 0.429 mol of Al according to the following equation (see Figure 4.3.2)?

2 Al + 3 I → 2 AlI (4.3.5)
2 3

Access for free at OpenStax 4.3.1 https://chem.libretexts.org/@go/page/38160


Figure 4.3.1 : Aluminum and iodine react to produce aluminum iodide. The heat of the reaction vaporizes some of the solid
iodine as a purple vapor. (credit: modification of work by Mark Ott)
Three pictures showing the progression of the reaction between iodine and aluminum. The first diagram shows only a pile of
silver powder while the second diagram shows small amount of vapor forming. The final picture shows a large release of
purple vapor.

Solution
3 mol I
Referring to the balanced chemical equation, the stoichiometric factor relating the two substances of interest is 2
. The
2 mol Al
molar amount of iodine is derived by multiplying the provided molar amount of aluminum by this factor:

3 mol I2
mol I2 = 0.429 mol Al ×
2 mol Al

= 0.644 mol I2

 Exercise 4.3.1

How many moles of Ca(OH)2 are required to react with 1.36 mol of H3PO4 to produce Ca3(PO4)2 according to the equation
3 Ca (OH) + 2 H PO → Ca (PO ) + 6 H O ?
2 3 4 3 4 2 2

Answer
2.04 mol

 Example 4.3.2: Number of Product Molecules Generated by a Reaction

How many carbon dioxide molecules are produced when 0.75 mol of propane is combusted according to this equation?
C H +5 O → 3 CO +4 H O (4.3.6)
3 8 2 2 2

Solution
The approach here is the same as for Example 4.3.1, though the absolute number of molecules is requested, not the number of
moles of molecules. This will simply require use of the moles-to-numbers conversion factor, Avogadro’s number.
The balanced equation shows that carbon dioxide is produced from propane in a 3:1 ratio:
\[\ce{\dfrac{3\: mol\: CO2}{1\: mol\: C3H8}} \label{4.4.7} \]
Using this stoichiometric factor, the provided molar amount of propane, and Avogadro’s number,

Access for free at OpenStax 4.3.2 https://chem.libretexts.org/@go/page/38160


3 mol CO2 23
6.022 × 10 C O2 molecules 24
0.75 mol C3 H8 × × = 1.4 × 10 C O2 molecules (4.3.7)
1 mol C3 H8 mol CO2

 Exercise 4.3.1

How many NH3 molecules are produced by the reaction of 4.0 mol of Ca(OH)2 according to the following equation:
(NH ) SO + Ca (OH) → 2 NH + CaSO +2 H O (4.3.8)
4 2 4 2 3 4 2

Answer
4.8 × 1024 NH3 molecules

These examples illustrate the ease with which the amounts of substances involved in a chemical reaction of known stoichiometry
may be related. Directly measuring numbers of atoms and molecules is, however, not an easy task, and the practical application of
stoichiometry requires that we use the more readily measured property of mass.

 Example 4.3.3: Relating Masses of Reactants and Products

What mass of sodium hydroxide, NaOH, would be required to produce 16 g of the antacid milk of magnesia [magnesium
hydroxide, Mg(OH)2] by the following reaction?
MgCl (aq) + 2 NaOH(aq) → Mg (OH) (s) + 2 NaCl(aq)
2 2

Solution
The approach used previously in Examples 4.3.1 and 4.3.2 is likewise used here; that is, we must derive an appropriate
stoichiometric factor from the balanced chemical equation and use it to relate the amounts of the two substances of interest. In this
case, however, masses (not molar amounts) are provided and requested, so additional steps of the sort learned in the previous
chapter are required. The calculations required are outlined in this flowchart:

1 mol Mg(OH)2 2 mol NaOH 40.0 g NaOH


16 g Mg(OH)2 × × × = 22 g NaOH
58.3 g Mg(OH)2 1 mol Mg(OH)2 mol NaOH

 Exercise 4.3.3

What mass of gallium oxide, Ga2O3, can be prepared from 29.0 g of gallium metal? The equation for the reaction is
4 Ga + 3 O → 2 Ga O .
2 2 3

Answer
39.0 g

Access for free at OpenStax 4.3.3 https://chem.libretexts.org/@go/page/38160


 Example 4.3.4: Relating Masses of Reactants
What mass of oxygen gas, O2, from the air is consumed in the combustion of 702 g of octane, C8H18, one of the principal
components of gasoline?

2C H + 25 O → 16 CO + 18 H O
8 18 2 2 2

Solution
The approach required here is the same as for the Example 4.3.3, differing only in that the provided and requested masses are
both for reactant species.

1 mol C H 25 mol O
8 18 2 32.00 g O
2 3
702 g C H × × × = 2.46 × 10 g O
8 18 2
114.23 g C H 2 mol C H mol O
8 18 8 18 2

 Exercise 4.3.4

What mass of CO is required to react with 25.13 g of Fe2O3 according to the equation Fe 2
O
3
+ 3 CO → 2 Fe + 3 CO
2
?

Answer
13.22 g

These examples illustrate just a few instances of reaction stoichiometry calculations. Numerous variations on the beginning and
ending computational steps are possible depending upon what particular quantities are provided and sought (volumes, solution
concentrations, and so forth). Regardless of the details, all these calculations share a common essential component: the use of
stoichiometric factors derived from balanced chemical equations. Figure 4.3.2 provides a general outline of the various
computational steps associated with many reaction stoichiometry calculations.

Access for free at OpenStax 4.3.4 https://chem.libretexts.org/@go/page/38160


Figure 4.3.2 : The flowchart depicts the various computational steps involved in most reaction stoichiometry calculations.
Schematic flowchart showing how density is used for converting volume of pure substance to mass, molar mass for mass and
moles, molarity for moles and volume of solution, Avogadro's number for moles and number of particles, and stoichiometric factor
for relating moles of one substance to another.

 Airbags

Airbags (Figure 4.3.3) are a safety feature provided in most automobiles since the 1990s. The effective operation of an airbag
requires that it be rapidly inflated with an appropriate amount (volume) of gas when the vehicle is involved in a collision. This
requirement is satisfied in many automotive airbag systems through use of explosive chemical reactions, one common choice
being the decomposition of sodium azide, NaN3. When sensors in the vehicle detect a collision, an electrical current is passed
through a carefully measured amount of NaN3 to initiate its decomposition:

2 NaN (s) → 3 N (g) + 2 Na(s)


3 2

This reaction is very rapid, generating gaseous nitrogen that can deploy and fully inflate a typical airbag in a fraction of a second
(~0.03–0.1 s). Among many engineering considerations, the amount of sodium azide used must be appropriate for generating
enough nitrogen gas to fully inflate the air bag and ensure its proper function. For example, a small mass (~100 g) of NaN3 will
generate approximately 50 L of N2.

Access for free at OpenStax 4.3.5 https://chem.libretexts.org/@go/page/38160


Figure 4.3.3 : Airbags deploy upon impact to minimize serious injuries to passengers. (credit: Jon Seidman)

Summary
A balanced chemical equation may be used to describe a reaction’s stoichiometry (the relationships between amounts of reactants
and products). Coefficients from the equation are used to derive stoichiometric factors that subsequently may be used for
computations relating reactant and product masses, molar amounts, and other quantitative properties.

4.3.0.1: Glossary

stoichiometric factor
ratio of coefficients in a balanced chemical equation, used in computations relating amounts of reactants and products
stoichiometry
relationships between the amounts of reactants and products of a chemical reaction

This page titled 4.3: Reaction Stoichiometry is shared under a CC BY 4.0 license and was authored, remixed, and/or curated by OpenStax via
source content that was edited to the style and standards of the LibreTexts platform; a detailed edit history is available upon request.

Access for free at OpenStax 4.3.6 https://chem.libretexts.org/@go/page/38160


4.4: Reaction Yields
 Learning Objectives
Explain the concepts of theoretical yield and limiting reactants/reagents.
Derive the theoretical yield for a reaction under specified conditions.
Calculate the percent yield for a reaction.

The relative amounts of reactants and products represented in a balanced chemical equation are often referred to as stoichiometric
amounts. All the exercises of the preceding module involved stoichiometric amounts of reactants. For example, when calculating
the amount of product generated from a given amount of reactant, it was assumed that any other reactants required were available
in stoichiometric amounts (or greater). In this module, more realistic situations are considered, in which reactants are not present in
stoichiometric amounts.

4.4.1: Limiting Reactant


Consider another food analogy, making grilled cheese sandwiches (Figure 4.4.1):
1 slice of cheese + 2 slices of bread → 1 sandwich (4.4.1)

Stoichiometric amounts of sandwich ingredients for this recipe are bread and cheese slices in a 2:1 ratio. Provided with 28 slices of
bread and 11 slices of cheese, one may prepare 11 sandwiches per the provided recipe, using all the provided cheese and having six
slices of bread left over. In this scenario, the number of sandwiches prepared has been limited by the number of cheese slices, and
the bread slices have been provided in excess.

Figure 4.4.1 : Sandwich making can illustrate the concepts of limiting and excess reactants.
This figure has three rows showing the ingredients needed to make a sandwich. The first row reads, “1 sandwich = 2 slices of bread
+ 1 slice of cheese.” Two slices of bread and one slice of cheese are shown. The second row reads, “Provided with: 28 slices of
bread + 11 slices of cheese.” There are 28 slices of bread and 11 slices of cheese shown. The third row reads, “We can make: 11
sandwiches + 6 slices of bread left over.” 11 sandwiches are shown with six extra slices of bread.
Consider this concept now with regard to a chemical process, the reaction of hydrogen with chlorine to yield hydrogen chloride:

H (g) + Cl (g) → 2 HCl(g)


2 2

The balanced equation shows the hydrogen and chlorine react in a 1:1 stoichiometric ratio. If these reactants are provided in any
other amounts, one of the reactants will nearly always be entirely consumed, thus limiting the amount of product that may be
generated. This substance is the limiting reactant, and the other substance is the excess reactant. Identifying the limiting and excess
reactants for a given situation requires computing the molar amounts of each reactant provided and comparing them to the
stoichiometric amounts represented in the balanced chemical equation.
For example, imagine combining 6 moles of H2 and 4 moles of Cl2. Identifying the limiting reactant involves comparing the
amount of product expected for the complete reaction of each reactant. Each reactant amount is used to separately calculate the

Access for free at OpenStax 4.4.1 https://chem.libretexts.org/@go/page/38159


amount of product that would be formed per the reaction’s stoichiometry. The reactant yielding the lesser amount of product is the
limiting reactant.
For the example in the previous paragraph, the complete reaction of the hydrogen would yield
2 mol HCl
mol HCl produced = 6 mol H2 × = 12 mol HCl
1 mol H2

The complete reaction of the provided chlorine would produce


2 mol HCl
mol HCl produced = 4 mol C l2 × = 8 mol HCl
1 mol Cl2

The chlorine will be completely consumed once 8 moles of HCl have been produced. Since enough hydrogen was provided to yield
12 moles of HCl, there will be non-reacted hydrogen remaining once this reaction is complete. Chlorine, therefore, is the limiting
reactant and hydrogen is the excess reactant (Figure 4.4.2). To determine the amount of excess reactant that remains, the amount
of hydrogen consumed in the reaction can be subtracted from the starting quantity of hydrogen.
The amount of hydrogen consumed is

1 mol H2
mol H2 produced = 8 mol HCl × = 4 mol H2
2 mol HCl

Subtract the hydrogen consumed from the starting quantity

mole of excess H2 = 6 mol H2 starting − 4 mol H2 consumed = 2 mol H2 excess

Figure 4.4.2 : When H2 and Cl2 are combined in nonstoichiometric amounts, one of these reactants will limit the amount of HCl
that can be produced. This illustration shows a reaction in which hydrogen is present in excess and chlorine is the limiting reactant.
The figure shows a space-filling molecular models reacting. There is a reaction arrow pointing to the right in the middle. To the left
of the reaction arrow there are three molecules each consisting of two green spheres bonded together. There are also five molecules
each consisting of two smaller, white spheres bonded together. Above these molecules is the label, “Before reaction,” and below
these molecules is the label, “6 H subscript 2 and 4 C l subscript 2.” To the right of the reaction arrow, there are eight molecules
each consisting of one green sphere bonded to a smaller white sphere. There are also two molecules each consisting of two white
spheres bonded together. Above these molecules is the label, “After reaction,” and below these molecules is the label, “8 H C l and
2 H subscript 2.”

 Example 4.4.1: Identifying the Limiting Reactant


Silicon nitride is a very hard, high-temperature-resistant ceramic used as a component of turbine blades in jet engines. It is
prepared according to the following equation:

3 Si(s) + 2 N (g) → Si N (s)


2 3 4

Which is the limiting reactant when 2.00 g of Si and 1.50 g of N2 react?

Solution

Access for free at OpenStax 4.4.2 https://chem.libretexts.org/@go/page/38159


Compute the provided molar amounts of reactants, and then compare these amounts to the balanced equation to identify the
limiting reactant.
1 mol Si
mol Si = 2.00 g Si × = 0.0712 mol Si
28.09 g Si

1 mol N2
mol N2 = 1.50 g N2 × = 0.0535 mol N2
28.02 g N2

The provided Si:N2 molar ratio is:


0.0712 mol Si 1.33 mol Si
=
0.0535 mol N2 1 mol N2

The stoichiometric Si:N2 ratio is:


3 mol Si 1.5 mol Si
=
2 mol N2 1 mol N2

Comparing these ratios shows that Si is provided in a less-than-stoichiometric amount, and so is the limiting reactant.
Alternatively, compute the amount of product expected for complete reaction of each of the provided reactants. The 0.0712
moles of silicon would yield
1 mol Si3 N4
mol Si3 N4 produced = 0.0712 mol Si × = 0.0237 mol Si3 N4
3 mol Si

while the 0.0535 moles of nitrogen would produce


1 mol Si3 N4
mol Si3 N4 produced = 0.0535 mol N2 × = 0.0268 mol Si3 N4
2 mol N2

Since silicon yields the lesser amount of product, it is the limiting reactant.

 Exercise 4.4.1

Which is the limiting reactant when 5.00 g of H2 and 10.0 g of O2 react and form water?

Answer
O2

4.4.2: Percent Yield


The amount of product that may be produced by a reaction under specified conditions, as calculated per the stoichiometry of an
appropriate balanced chemical equation, is called the theoretical yield of the reaction. In practice, the amount of product obtained is
called the actual yield, and it is often less than the theoretical yield for a number of reasons. Some reactions are inherently
inefficient, being accompanied by side reactions that generate other products. Others are, by nature, incomplete (consider the
partial reactions of weak acids and bases discussed earlier in this text). Some products are difficult to collect without some loss, and
so less than perfect recovery will reduce the actual yield. The extent to which a reaction’s theoretical yield is achieved is commonly
expressed as its percent yield:
actual yield
percent yield = × 100%
theoretical yield

Actual and theoretical yields may be expressed as masses or molar amounts (or any other appropriate property; e.g., volume, if the
product is a gas). As long as both yields are expressed using the same units, these units will cancel when percent yield is calculated.

Access for free at OpenStax 4.4.3 https://chem.libretexts.org/@go/page/38159


 Example 4.4.2: Calculation of Percent Yield
Upon reaction of 1.274 g of copper sulfate with excess zinc metal, 0.392 g copper metal was obtained according to the
equation:

CuSO (aq) + Zn(s) → Cu(s) + ZnSO (aq)


4 4

What is the percent yield?

Solution
The provided information identifies copper sulfate as the limiting reactant, and so the theoretical yield is found by the approach
illustrated in the previous module, as shown here:

1 mol CuSO4 1 mol Cu 63.55 g Cu


1.274 g CuS O4 × × × = 0.5072 g Cu
159.62 g CuSO4 1 mol CuSO4 1 mol Cu

Using this theoretical yield and the provided value for actual yield, the percent yield is calculated to be
actual yield
percent yield = ( ) × 100
theoretical yield

0.392 g Cu
percent yield = ( ) × 100
0.5072 g Cu

= 77.3%

 Exercise 4.4.2

What is the percent yield of a reaction that produces 12.5 g of the gas Freon CF2Cl2 from 32.9 g of CCl4 and excess HF?

CCl + 2 HF → CF Cl + 2 HCl
4 2 2

Answer
48.3%

 Green Chemistry and Atom Economy


The purposeful design of chemical products and processes that minimize the use of environmentally hazardous substances and
the generation of waste is known as green chemistry. Green chemistry is a philosophical approach that is being applied to many
areas of science and technology, and its practice is summarized by guidelines known as the “Twelve Principles of Green
Chemistry”. One of the 12 principles is aimed specifically at maximizing the efficiency of processes for synthesizing chemical
products. The atom economy of a process is a measure of this efficiency, defined as the percentage by mass of the final product
of a synthesis relative to the masses of all the reactants used:
mass of product
atom economy = × 100%
mass of reactants

Though the definition of atom economy at first glance appears very similar to that for percent yield, be aware that this property
represents a difference in the theoretical efficiencies of different chemical processes. The percent yield of a given chemical
process, on the other hand, evaluates the efficiency of a process by comparing the yield of product actually obtained to the
maximum yield predicted by stoichiometry.
The synthesis of the common nonprescription pain medication, ibuprofen, nicely illustrates the success of a green chemistry
approach (Figure 4.4.3). First marketed in the early 1960s, ibuprofen was produced using a six-step synthesis that required 514
g of reactants to generate each mole (206 g) of ibuprofen, an atom economy of 40%. In the 1990s, an alternative process was

Access for free at OpenStax 4.4.4 https://chem.libretexts.org/@go/page/38159


developed by the BHC Company (now BASF Corporation) that requires only three steps and has an atom economy of ~80%,
nearly twice that of the original process. The BHC process generates significantly less chemical waste; uses less-hazardous and
recyclable materials; and provides significant cost-savings to the manufacturer (and, subsequently, the consumer). In
recognition of the positive environmental impact of the BHC process, the company received the Environmental Protection
Agency’s Greener Synthetic Pathways Award in 1997.

Figure 4.4.3 : (a) Ibuprofen is a popular nonprescription pain medication commonly sold as 200 mg tablets. (b) The BHC
process for synthesizing ibuprofen requires only three steps and exhibits an impressive atom economy. (credit a: modification
of work by Derrick Coetzee)

Summary
When reactions are carried out using less-than-stoichiometric quantities of reactants, the amount of product generated will be
determined by the limiting reactant. The amount of product generated by a chemical reaction is its actual yield. This yield is often
less than the amount of product predicted by the stoichiometry of the balanced chemical equation representing the reaction (its
theoretical yield). The extent to which a reaction generates the theoretical amount of product is expressed as its percent yield.

4.4.3: Key Equations


actual yield
percent yield = ( ) × 100
theoretical yield

Glossary
actual yield
amount of product formed in a reaction

excess reactant
reactant present in an amount greater than required by the reaction stoichiometry

limiting reactant
reactant present in an amount lower than required by the reaction stoichiometry, thus limiting the amount of product generated

percent yield
measure of the efficiency of a reaction, expressed as a percentage of the theoretical yield

theoretical yield
amount of product that may be produced from a given amount of reactant(s) according to the reaction stoichiometry

This page titled 4.4: Reaction Yields is shared under a CC BY 4.0 license and was authored, remixed, and/or curated by OpenStax via source
content that was edited to the style and standards of the LibreTexts platform; a detailed edit history is available upon request.

Access for free at OpenStax 4.4.5 https://chem.libretexts.org/@go/page/38159


4.5: Quantitative Chemical Analysis
 Learning Objectives
Describe the fundamental aspects of titrations and gravimetric analysis.
Perform stoichiometric calculations using typical titration and gravimetric data.

In the 18th century, the strength (actually the concentration) of vinegar samples was determined by noting the amount of potassium
carbonate, K2CO3, which had to be added, a little at a time, before bubbling ceased. The greater the weight of potassium carbonate
added to reach the point where the bubbling ended, the more concentrated the vinegar.
We now know that the effervescence that occurred during this process was due to reaction with acetic acid, CH3CO2H, the
compound primarily responsible for the odor and taste of vinegar. Acetic acid reacts with potassium carbonate according to the
following equation:

2 CH CO H(aq) + K CO (s) → 2 KCH CO (aq) + CO (g) + H O(l)


3 2 2 3 3 3 2 2

The bubbling was due to the production of CO2.


The test of vinegar with potassium carbonate is one type of quantitative analysis—the determination of the amount or concentration
of a substance in a sample. In the analysis of vinegar, the concentration of the solute (acetic acid) was determined from the amount
of reactant that combined with the solute present in a known volume of the solution. In other types of chemical analyses, the
amount of a substance present in a sample is determined by measuring the amount of product that results.

4.5.1: Titration
The described approach to measuring vinegar strength was an early version of the analytical technique known as titration analysis.
A typical titration analysis involves the use of a buret (Figure 4.5.1) to make incremental additions of a solution containing a
known concentration of some substance (the titrant) to a sample solution containing the substance whose concentration is to be
measured (the analyte). The titrant and analyte undergo a chemical reaction of known stoichiometry, and so measuring the volume
of titrant solution required for complete reaction with the analyte (the equivalence point of the titration) allows calculation of the
analyte concentration. The equivalence point of a titration may be detected visually if a distinct change in the appearance of the
sample solution accompanies the completion of the reaction. The halt of bubble formation in the classic vinegar analysis is one
such example, though, more commonly, special dyes called indicators are added to the sample solutions to impart a change in color
at or very near the equivalence point of the titration. Equivalence points may also be detected by measuring some solution property
that changes in a predictable way during the course of the titration. Regardless of the approach taken to detect a titration’s
equivalence point, the volume of titrant actually measured is called the end point. Properly designed titration methods typically
ensure that the difference between the equivalence and end points is negligible. Though any type of chemical reaction may serve as
the basis for a titration analysis, the three described in this chapter (precipitation, acid-base, and redox) are most common.
Additional details regarding titration analysis are provided in the chapter on acid-base equilibria.

Access for free at OpenStax 4.5.1 https://chem.libretexts.org/@go/page/38158


Figure 4.5.1 : (a) A student fills a buret in preparation for a titration analysis. (b) A typical buret permits volume measurements to
the nearest 0.1 mL. (credit a: modification of work by Mark Blaser and Matt Evans; credit b: modification of work by Mark Blaser
and Matt Evans)
Two pictures are shown. In a, a person is shown pouring a liquid from a small beaker into a buret. The person is wearing goggles
and gloves as she transfers the solution into the buret. In b, a close up view of the markings on the side of the buret is shown. The
markings for 10, 15, and 20 are clearly shown with horizontal rings printed on the buret. Between each of these whole number
markings, half markings are also clearly shown with horizontal line segment markings.

 Example 4.5.1: Titration Analysis

The end point in a titration of a 50.00-mL sample of aqueous HCl was reached by addition of 35.23 mL of 0.250 M NaOH
titrant. The titration reaction is:
HCl(aq) + NaOH(aq) → NaCl(aq) + H O(l)
2

What is the molarity of the HCl?

Solution
As for all reaction stoichiometry calculations, the key issue is the relation between the molar amounts of the chemical species
of interest as depicted in the balanced chemical equation. The approach outlined in previous modules of this chapter is
followed, with additional considerations required, since the amounts of reactants provided and requested are expressed as
solution concentrations.
For this exercise, the calculation will follow the following outlined steps:

Access for free at OpenStax 4.5.2 https://chem.libretexts.org/@go/page/38158


This figure shows four rectangles. The first is shaded lavender and is labeled, “Volume of N a O H.” This rectangle is followed
by an arrow pointing right which is labeled, “Molar concentration,” to a second rectangle. This second rectangle is shaded pink
and is labeled, “Moles of N a O H.” This rectangle is followed by an arrow pointing right which is labeled, “Stoichiometric
factor,” to a third rectangle which is shaded pink and is labeled, “Moles of H C l.” This rectangle is followed by an arrow
labeled, “Solution volume,” which points right to a fourth rectangle. This fourth rectangle is shaded lavender and is labeled,
“Concentration of H C l.”
The molar amount of HCl is calculated to be:

1 L 0.250 mol NaOH 1 mol HCl −3


35.23 mL NaOH × × × = 8.81 × 10 mol HCl
1000 mL 1 L 1 mol NaOH

Using the provided volume of HCl solution and the definition of molarity, the HCl concentration is:
mol HCl
M =
L solution
−3
8.81 × 10 mol HCl
M =
1 L
50.00 mL ×
1000 mL

M = 0.176 M

Note: For these types of titration calculations, it is convenient to recognize that solution molarity is also equal to the number of
millimoles of solute per milliliter of solution:
3
10 mmol

mol solute mol mmol solute


M = × =
3
L solution 10 mL mL solution

Using this version of the molarity unit will shorten the calculation by eliminating two conversion factors:
0.250 mmol NaOH 1 mmol HCl
35.23 mL NaOH × ×
mL NaOH 1 mmol NaOH
= 0.176 M HCl
50.00 mL solution

 Exercise 4.5.1

A 20.00-mL sample of aqueous oxalic acid, H2C2O4, was titrated with a 0.09113-M solution of potassium permanganate,
KMnO4.
− + 2+
2 MnO (aq) + 5 H C O (aq) + 6 H (aq) → 10 CO (g) + 2 Mn (aq) + 8 H O(l)
4 2 2 4 2 2

A volume of 23.24 mL was required to reach the end point. What is the oxalic acid molarity?

Answer
0.2648 M

Access for free at OpenStax 4.5.3 https://chem.libretexts.org/@go/page/38158


4.5.2: Gravimetric Analysis
A gravimetric analysis is one in which a sample is subjected to some treatment that causes a change in the physical state of the
analyte that permits its separation from the other components of the sample. Mass measurements of the sample, the isolated
analyte, or some other component of the analysis system, used along with the known stoichiometry of the compounds involved,
permit calculation of the analyte concentration. Gravimetric methods were the first techniques used for quantitative chemical
analysis, and they remain important tools in the modern chemistry laboratory.
The required change of state in a gravimetric analysis may be achieved by various physical and chemical processes. For example,
the moisture (water) content of a sample is routinely determined by measuring the mass of a sample before and after it is subjected
to a controlled heating process that evaporates the water. Also common are gravimetric techniques in which the analyte is subjected
to a precipitation reaction of the sort described earlier in this chapter. The precipitate is typically isolated from the reaction mixture
by filtration, carefully dried, and then weighed (Figure 4.5.2). The mass of the precipitate may then be used, along with relevant
stoichiometric relationships, to calculate analyte concentration.

Figure 4.5.2 : Precipitate may be removed from a reaction mixture by filtration.


"A photo is shown of a flask and funnel used for filtration. The flask contains a slightly opaque liquid filtrate with a slight yellow
tint. A funnel, which contains a bright yellow and orange material, sits atop the flask. The flask is held in place by a clamp and is
connected to a vacuum line. The connection between the funnel and flask is sealed with a rubber bung or gasket."

 Example 4.5.2: Gravimetric Analysis

A 0.4550-g solid mixture containing MgSO4 is dissolved in water and treated with an excess of Ba(NO3)2, resulting in the
precipitation of 0.6168 g of BaSO4.

MgSO (aq) + Ba (NO ) (aq) → BaSO (s) + Mg (NO ) (aq)


4 3 2 4 3 2

What is the concentration (percent) of MgSO4 in the mixture?

Solution
The plan for this calculation is similar to others used in stoichiometric calculations, the central step being the connection
between the moles of BaSO4 and MgSO4 through their stoichiometric factor. Once the mass of MgSO4 is computed, it may be
used along with the mass of the sample mixture to calculate the requested percentage concentration.

Access for free at OpenStax 4.5.4 https://chem.libretexts.org/@go/page/38158


"This figure shows five rectangles. The first is shaded yellow and is labeled “Mass of B a S O subscript 4.” This rectangle is
followed by an arrow pointing right to a second rectangle. The arrow is labeled, “Molar mass.” The second rectangle is shaded
pink and is labeled, “Moles of B a S O subscript 4.” This rectangle is followed by an arrow pointing right to a third rectangle.
The arrow is labeled, “Stoichiometric factor.” This third rectangle is shaded pink and is labeled, “Moles of M g S O subscript
4.” This rectangle is followed by an arrow labeled, “Molar mass,” which points downward to a fourth rectangle. This fourth
rectangle is shaded yellow and is labeled, “Mass of M g S O subscript 4.” This rectangle is followed by an arrow labeled,
“Sample mass,” which points left to a fifth rectangle. This fifth rectangle is shaded lavender and is labeled, “Percent M g S O
subscript 4."

The mass of MgSO4 that would yield the provided precipitate mass is

1 mol BaSO4 1 mol MgSO4 120.37 g MgSO4


0.6168 g BaSO4 × × × = 0.3181 g MgSO4
233.43 g BaSO4 1 mol BaSO4 1 mol MgSO4

The concentration of MgSO4 in the sample mixture is then calculated to be


mass MgSO
4
percent MgSO = × 100%
4
mass sample

0.3181 g
× 100% = 69.91%
0.4550 g

 Exercise 4.5.2

What is the percent of chloride ion in a sample if 1.1324 g of the sample produces 1.0881 g of AgCl when treated with excess
Ag+?
+ −
Ag (aq) + Cl (aq) → AgCl(s)

Answer
23.76%

4.5.3: Combustion Analysis


The elemental composition of hydrocarbons and related compounds may be determined via a gravimetric method known as
combustion analysis. In a combustion analysis, a weighed sample of the compound is heated to a high temperature under a stream
of oxygen gas, resulting in its complete combustion to yield gaseous products of known identities. The complete combustion of
hydrocarbons, for example, will yield carbon dioxide and water as the only products. The gaseous combustion products are swept
through separate, preweighed collection devices containing compounds that selectively absorb each product (Figure 4.5.3). The
mass increase of each device corresponds to the mass of the absorbed product and may be used in an appropriate stoichiometric
calculation to derive the mass of the relevant element.

Access for free at OpenStax 4.5.5 https://chem.libretexts.org/@go/page/38158


Figure 4.5.3 : This schematic diagram illustrates the basic components of a combustion analysis device for determining the carbon
and hydrogen content of a sample.
This diagram shows an arrow pointing from O subscript 2 into a tube that leads into a vessel containing a red material, labeled
“Sample.” This vessel is inside a blue container with a red inner lining which is labeled “Furnace.” An arrow points from the tube
to the right into the vessel above the red sample material. An arrow leads out of this vessel through a tube into a second vessel
outside the furnace. An line points from this tube to a label above the diagram that reads “C O subscript 2, H subscript 2 O, O
subscript 2, and other gases.” Many small green spheres are visible in the second vessel which is labeled below, “H subscript 2 O
absorber such as M g ( C l O subscript 4 ) subscript 2.” An arrow points to the right through the vessel, and another arrow points
right heading out of the vessel through a tube into a third vessel. The third vessel contains many small blue spheres. It is labeled “C
O subscript 2 absorber such as N a O H.” An arrow points right through this vessel, and a final arrow points out of a tube at the
right end of the vessel. Outside the end of this tube at the end of the arrow is the label, “O subscript 2 and other gases.”

 Example 4.5.3: Combustion Analysis

Polyethylene is a hydrocarbon polymer used to produce food-storage bags and many other flexible plastic items. A combustion
analysis of a 0.00126-g sample of polyethylene yields 0.00394 g of CO2 and 0.00161 g of H2O. What is the empirical formula
of polyethylene?

Solution
The primary assumption in this exercise is that all the carbon in the sample combusted is converted to carbon dioxide, and all
the hydrogen in the sample is converted to water:
y
Cx Hy (s) + excess O (g) → x CO (g) + H O(g)
2 2 2
2

Note that a balanced equation is not necessary for the task at hand. To derive the empirical formula of the compound, only the
subscripts x and y are needed.
First, calculate the molar amounts of carbon and hydrogen in the sample, using the provided masses of the carbon dioxide and
water, respectively. With these molar amounts, the empirical formula for the compound may be written as described in the
previous chapter of this text. An outline of this approach is given in the following flow chart:

Access for free at OpenStax 4.5.6 https://chem.libretexts.org/@go/page/38158


This figure shows two flowcharts. The first row is a single flow chart. In this row, a rectangle at the left is shaded yellow and is
labeled, “Mass of C O subscript 2.” This rectangle is followed by an arrow pointing right to a second rectangle. The arrow is
labeled, “Molar mass.” The second rectangle is shaded pink and is labeled, “Moles of C O subscript 2.” This rectangle is
followed by an arrow pointing right to a third rectangle. The arrow is labeled, “Stoichiometric factor.” The third rectangle is
shaded pink and is labeled, “Moles of C.” This rectangle is followed by an arrow labeled “Molar mass” which points right to a
fourth rectangle. The fourth rectangle is shaded yellow and is labeled “Mass of C.” Below, is a second flowchart. It begins with
a yellow shaded rectangle on the left which is labeled, “Mass of H subscript 2 O.” This rectangle is followed by an arrow
labeled, “Molar mass,” which points right to a second rectangle. The second rectangle is shaded pink and is labeled, “Moles of
H subscript 2 O.” This rectangle is followed by an arrow pointing right to a third rectangle. The arrow is labeled,
“Stoichiometric factor.” The third rectangle is shaded pink and is labeled “Moles of H.” This rectangle is followed to the right
by an arrow labeled, “Molar mass,” which points to a fourth rectangle. The fourth rectangle is shaded yellow and is labeled
“Mass of H.” An arrow labeled, “Sample mass” points down beneath this rectangle to a green shaded rectangle. This rectangle
is labeled, “Percent composition.” An arrow extends beneath the pink rectangle labeled, “Moles of H,” to a green shaded
rectangle labeled, “C to H mole ratio.” Beneath this rectangle, an arrow extends to a second green shaded rectangle which is
labeled, “Empirical formula.”
1 mol CO2 1 mol C −5
mol C = 0.00394 g C O2 × × = 8.95 × 10 mol C
44.01 g/mol 1 mol CO2

1 mol H2 O 2 mol H
−4
mol H = 0.00161 g H2 O × × = 1.79 × 10 mol H
18.02 g/mol 1 mol H2 O

The empirical formula for the compound is then derived by identifying the smallest whole-number multiples for these molar
amounts. The H-to-C molar ratio is
−4
mol H 1.79 × 10 mol H 2 mol H
= =
−5
mol C 8.95 × 10 mol C 1 mol C

and the empirical formula for polyethylene is CH2.

 Exercise 4.5.3

A 0.00215-g sample of polystyrene, a polymer composed of carbon and hydrogen, produced 0.00726 g of CO2 and 0.00148 g
of H2O in a combustion analysis. What is the empirical formula for polystyrene?

Answer
CH

Access for free at OpenStax 4.5.7 https://chem.libretexts.org/@go/page/38158


Summary
The stoichiometry of chemical reactions may serve as the basis for quantitative chemical analysis methods. Titrations involve
measuring the volume of a titrant solution required to completely react with a sample solution. This volume is then used to
calculate the concentration of analyte in the sample using the stoichiometry of the titration reaction. Gravimetric analysis involves
separating the analyte from the sample by a physical or chemical process, determining its mass, and then calculating its
concentration in the sample based on the stoichiometry of the relevant process. Combustion analysis is a gravimetric method used
to determine the elemental composition of a compound by collecting and weighing the gaseous products of its combustion.

Glossary
analyte
chemical species of interest

buret
device used for the precise delivery of variable liquid volumes, such as in a titration analysis

combustion analysis
gravimetric technique used to determine the elemental composition of a compound via the collection and weighing of its
gaseous combustion products

end point
measured volume of titrant solution that yields the change in sample solution appearance or other property expected for
stoichiometric equivalence (see equivalence point)

equivalence point
volume of titrant solution required to react completely with the analyte in a titration analysis; provides a stoichiometric amount
of titrant for the sample’s analyte according to the titration reaction

gravimetric analysis
quantitative chemical analysis method involving the separation of an analyte from a sample by a physical or chemical process
and subsequent mass measurements of the analyte, reaction product, and/or sample

indicator
substance added to the sample in a titration analysis to permit visual detection of the end point

quantitative analysis
the determination of the amount or concentration of a substance in a sample

titrant
solution containing a known concentration of substance that will react with the analyte in a titration analysis

titration analysis
quantitative chemical analysis method that involves measuring the volume of a reactant solution required to completely react
with the analyte in a sample

This page titled 4.5: Quantitative Chemical Analysis is shared under a CC BY 4.0 license and was authored, remixed, and/or curated by OpenStax
via source content that was edited to the style and standards of the LibreTexts platform; a detailed edit history is available upon request.

Access for free at OpenStax 4.5.8 https://chem.libretexts.org/@go/page/38158


4.E: Stoichiometry of Chemical Reactions (Exercises)
4.E.1: 4.1: Writing and Balancing Chemical Equations
4.E.1.1: Q4.1.1
What does it mean to say an equation is balanced? Why is it important for an equation to be balanced?

4.E.1.2: S4.1.1
An equation is balanced when the same number of each element is represented on the reactant and product sides. Equations must be
balanced to accurately reflect the law of conservation of matter.

4.E.1.3: Q4.1.2
Consider molecular, complete ionic, and net ionic equations.
a. What is the difference between these types of equations?
b. In what circumstance would the complete and net ionic equations for a reaction be identical?

4.E.1.4: Q4.1.3
Balance the following equations:
a. PCl (s) + H O(l) → POCl (l) + HCl(aq)
5 2 3

b. Cu(s) + HNO (aq) → Cu(NO ) (aq) + H O(l) + NO(g)


3 3 2 2

c. H (g) + I (s) → HI(s)


2 2

d. Fe(s) + O (g) → Fe O (s) 2 2 3

e. Na(s) + H O(l) → NaOH(aq) + H (g)


2 2

f. (NH ) Cr O (s) → Cr O (s) + N (g) + H O(g)


4 2 2 7 2 3 2 2

g. P (s) + Cl (g) → PCl (l)


4 2 3

h. PtCl (s) → Pt(s) + Cl (g)


4 2

4.E.1.5: S4.1.3
a. PCl (s) + H O(l) → POCl (l) + 2 HCl(aq) ;
5 2 3

b. 3 Cu(s) + 8 HNO (aq) → 3 Cu(NO ) (aq) + 4 H O(l) + 2 NO(g) ;


3 3 2 2

c. H (g) + I (s) → 2 HI(s) ;


2 2

d. 4 Fe(s) + 3 O (g) → 2 Fe O (s) ; 2 2 3

e. 2 Na(s) + 2 H O(l) → 2 NaOH(aq) + H (g) ;


2 2

f. (NH ) Cr O (s) → Cr O (s) + N (g) + 4 H O(g) ;


4 2 52 7 2 3 2 2

g. P (s) + 6 Cl (g) → 4 PCl (l) ;


4 2 3

h. PtCl (s) → Pt(s) + 2 Cl (g)


4 2

4.E.2: Q4.1.4
Balance the following equations:
a. Ag(s) + H S(g) + O (g) → Ag S(s) + H O(l)
2 2 2 2

b. P (s) + O (g) → P O (s)


4 2 4 10

c. Pb(s) + H O(l) + O (g) → Pb(OH) (s)


2 2 2

d. Fe(s) + H O(l) → Fe O (s) + H (g)


2 3 4 2

e. Sc O (s) + SO (l) → Sc (SO ) (s)


2 3 3 2 4 3

f. Ca (PO ) (aq) + H PO (aq) → Ca(H PO )


3 4 2 3 4 2 4 2
(aq)

g. Al(s) + H SO (aq) → Al (SO ) (s) + H (g)


2 4 2 4 3 2

h. TiCl (s) + H O(g) → TiO (s) + HCl(g)


4 2 2

4.E.2.1: S4.1.4
a. 4 Ag(s) + 2 H S(g) + O (g) → 2 Ag S(s) + 2 H
2 2 2 2
O(l)

b. P (s) + 5 O (g) → P O (s)


4 2 4 10

c. 2 Pb(s) + 2 H O(l) + O (g) → 2 Pb(OH) (s)


2 2 2

d. 3 Fe(s) + 4 H O(l) → Fe O (s) + 4 H (g)


2 3 4 2

Access for free at OpenStax 4.E.1 https://chem.libretexts.org/@go/page/58067


e. Sc O (s) + 3 SO (l) → Sc (SO ) (s)
2 3 3 2 4 3

f. Ca (PO ) (aq) + 4 H PO (aq) → 3 Ca(H PO ) (aq)


3 4 2 3 4 2 4 2

g. 2 Al(s) + 3 H SO (aq) → Al (SO ) (s) + 3 H (g)


2 4 2 4 3 2

h. TiCl (s) + 2 H O(g) → TiO (s) + 4 HCl(g)


4 2 2

4.E.2.2: Q4.1.5
Write a balanced molecular equation describing each of the following chemical reactions.
a. Solid calcium carbonate is heated and decomposes to solid calcium oxide and carbon dioxide gas.
b. Gaseous butane, C4H10, reacts with diatomic oxygen gas to yield gaseous carbon dioxide and water vapor.
c. Aqueous solutions of magnesium chloride and sodium hydroxide react to produce solid magnesium hydroxide and aqueous sodium
chloride.
d. Water vapor reacts with sodium metal to produce solid sodium hydroxide and hydrogen gas.

4.E.2.3: S4.1.5
a. CaCO (s) → CaO(s) + CO (g) ;
3 2

b. 2 C H (g) + 13 O (g) → 8 CO (g) + 10 H O(g) ;


4 10 2 2 2

c. MgCl (aq) + 2 NaOH(aq) → Mg(OH) (s) + 2 NaCl(aq) ;


2 2

d. 2 H O(g) + 2 Na(s) → 2 NaOH(s) + H (g)


2 2

4.E.2.4: Q4.1.6
Write a balanced equation describing each of the following chemical reactions.
a. Solid potassium chlorate, KClO3, decomposes to form solid potassium chloride and diatomic oxygen gas.
b. Solid aluminum metal reacts with solid diatomic iodine to form solid Al2I6.
c. When solid sodium chloride is added to aqueous sulfuric acid, hydrogen chloride gas and aqueous sodium sulfate are produced.
d. Aqueous solutions of phosphoric acid and potassium hydroxide react to produce aqueous potassium dihydrogen phosphate and
liquid water.

4.E.2.5: Q4.1.7
Colorful fireworks often involve the decomposition of barium nitrate and potassium chlorate and the reaction of the metals
magnesium, aluminum, and iron with oxygen.
a. Write the formulas of barium nitrate and potassium chlorate.
b. The decomposition of solid potassium chlorate leads to the formation of solid potassium chloride and diatomic oxygen gas. Write
an equation for the reaction.
c. The decomposition of solid barium nitrate leads to the formation of solid barium oxide, diatomic nitrogen gas, and diatomic
oxygen gas. Write an equation for the reaction.
d. Write separate equations for the reactions of the solid metals magnesium, aluminum, and iron with diatomic oxygen gas to yield
the corresponding metal oxides. (Assume the iron oxide contains Fe3+ ions.)

4.E.2.6: Q4.1.7
a. Ba(NO3)2, KClO3;
b. 2 KClO (s) → 2 KCl(s) + 3 O (g) ;
3 2

c. 2 Ba(NO ) (s) → 2 BaO(s) + 2 N (g) + 5 O (g) ;


3 2 2 2

d. 2 Mg(s) + O (g) → 2 MgO(s) ; 4 Al(s) + 3 O (g) → 2 Al


2 2 2
O (g)
3
; 4 Fe(s) + 3 O
2
(g) → 2 Fe O (s)
2 3

4.E.2.7: Q4.1.8
Fill in the blank with a single chemical formula for a covalent compound that will balance the equation:

Access for free at OpenStax 4.E.2 https://chem.libretexts.org/@go/page/58067


4.E.2.8: Q4.1.9
Aqueous hydrogen fluoride (hydrofluoric acid) is used to etch glass and to analyze minerals for their silicon content. Hydrogen
fluoride will also react with sand (silicon dioxide).
a. Write an equation for the reaction of solid silicon dioxide with hydrofluoric acid to yield gaseous silicon tetrafluoride and liquid
water.
b. The mineral fluorite (calcium fluoride) occurs extensively in Illinois. Solid calcium fluoride can also be prepared by the reaction of
aqueous solutions of calcium chloride and sodium fluoride, yielding aqueous sodium chloride as the other product. Write complete
and net ionic equations for this reaction.

4.E.2.9: S4.1.9
a. 4 HF(aq) + SiO (s) → SiF (g) + 2 H O(l) ;
2 4 2

b. complete ionic equation: 2 Na (aq) + 2 F (aq) + Ca + − 2+


(aq) + 2 Cl

(aq) → CaF (s) + 2 Na
2
+
(aq) + 2 Cl

(aq) , net ionic
equation: 2 F (aq) + Ca (aq) → CaF (s)
− 2+

4.E.2.10: Q4.1.10
A novel process for obtaining magnesium from sea water involves several reactions. Write a balanced chemical equation for each step
of the process.
a. The first step is the decomposition of solid calcium carbonate from seashells to form solid calcium oxide and gaseous carbon
dioxide.
b. The second step is the formation of solid calcium hydroxide as the only product from the reaction of the solid calcium oxide with
liquid water.
c. Solid calcium hydroxide is then added to the seawater, reacting with dissolved magnesium chloride to yield solid magnesium
hydroxide and aqueous calcium chloride.
d. The solid magnesium hydroxide is added to a hydrochloric acid solution, producing dissolved magnesium chloride and liquid
water.
e. Finally, the magnesium chloride is melted and electrolyzed to yield liquid magnesium metal and diatomic chlorine gas.

4.E.2.11: Q4.1.11
From the balanced molecular equations, write the complete ionic and net ionic equations for the following:
a. K C O (aq) + Ba(OH) (aq) → 2 KOH(aq) + BaC O (s)
2 2 4 2 2 2

b. Pb(NO ) (aq) + H SO (aq) → PbSO (s) + 2 HNO (aq)


3 2 2 4 4 3

c. CaCO (s) + H SO (aq) → CaSO (s) + CO (g) + H O(l)


3 2 4 4 2 2

4.E.2.12: S4.1.11
a. 2K
+
(aq) + C O
2
2−

4
(aq) + Ba
2+
(aq) + 2 OH

(aq) → 2 K
+
(aq) + 2 OH

(aq) + BaC O (s)
2 4
(complete) (4.E.1)

2+ 2−
Ba (aq) + C O (aq) → BaC O (s) (net) (4.E.2)
2 4 2 4

b. Pb
2+
(aq) + 2 NO3 (aq) + 2 H
− +
(aq) + SO
2−

4
(aq) → PbSO (s) + 2 H
4
+
(aq) + 2 NO3 (aq)

(complete) (4.E.3)

2+ 2−
Pb (aq) + SO (aq) → PbSO (s) (net) (4.E.4)
4 4

c. CaCO (s) + 2 H
3
+
(aq) + SO
2−

4
(aq) → CaSO (s) + CO (g) + H O(l)
4 2 2
(complete) (4.E.5)

+ 2−
CaCO (s) + 2 H (aq) + SO 4 (aq) → CaSO (s) + CO (g) + H O(l) (net) (4.E.6)
3 4 2 2

4.E.3: 4.2: Classifying Chemical Reactions


4.E.3.1: Q4.2.1
Use the following equations to answer the next five questions:
i. H O(s) → H O(l)
2 2

ii. Na (aq) + Cl (aq)Ag (aq) + NO


+ − + −

3
(aq) → AgCl(s) + Na
+
(aq) + NO

3
(aq)

iii. CH OH(g) + O (g) → CO (g) + H


3 2 2 2
O(g)

iv. 2 H O(l) → 2 H (g) + O (g)


2 2 2

v. H (aq) + OH (aq) → H O(l)


+ −

Access for free at OpenStax 4.E.3 https://chem.libretexts.org/@go/page/58067


a. Which equation describes a physical change?
b. Which equation identifies the reactants and products of a combustion reaction?
c. Which equation is not balanced?
d. Which is a net ionic equation?

4.E.3.2: S4.2.1
a.) i. H 2 O(solid) → H2 O(liquid)

b.) iii.
c.) iii. 2 CH 3
OH(g) + 3 O (g) → 2 CO (g) + 4 H O(g)
2 2 2

d.) v.

4.E.3.3: Q4.2.2
Indicate what type, or types, of reaction each of the following represents:
a. Ca(s) + Br (l) → CaBr (s) 2 2

b. Ca(OH) (aq) + 2 HBr(aq) → CaBr (aq) + 2 H


2 2 2
O(l)

c. C H (l) + 9 O (g) → 6 CO (g) + 6 H O(g)


6 12 2 2 2

4.E.3.4: S4.2.2
oxidation-reduction (addition); acid-base (neutralization); oxidation-reduction (combustion)
<

4.E.3.5: Q4.2.3
Indicate what type, or types, of reaction each of the following represents:
a. H O(g) + C(s) → CO(g) + H (g)
2 2

b. 2 KClO (s) → 2 KCl(s) + 3 O (g)


3 2

c. Al(OH) (aq) + 3 HCl(aq) → AlBr (aq) + 3 H O(l)


3 3 2

d. Pb(NO ) (aq) + H SO (aq) → PbSO (s) + 2 HNO


3 2 2 4 4 3
(aq)

4.E.3.6: Q4.2.4
Silver can be separated from gold because silver dissolves in nitric acid while gold does not. Is the dissolution of silver in nitric acid
an acid-base reaction or an oxidation-reduction reaction? Explain your answer.

4.E.3.7: S4.2.4
It is an oxidation-reduction reaction because the oxidation state of the silver changes during the reaction.

4.E.3.8: Q4.2.5
Determine the oxidation states of the elements in the following compounds:
a. NaI
b. GdCl3
c. LiNO3
d. H2Se
e. Mg2Si
f. RbO2, rubidium superoxide
g. HF

4.E.3.9: Q4.2.6
Determine the oxidation states of the elements in the compounds listed. None of the oxygen-containing compounds are peroxides or
superoxides.
a. H3PO4
b. Al(OH)3
c. SeO2
d. KNO2

Access for free at OpenStax 4.E.4 https://chem.libretexts.org/@go/page/58067


e. In2S3
f. P4O6

4.E.3.10: S4.2.6
H +1, P +5, O −2; Al +3, H +1, O −2; Se +4, O −2; K +1, N +3, O −2; In +3, S −2; P +3, O −2

4.E.3.11: Q4.2.7
Determine the oxidation states of the elements in the compounds listed. None of the oxygen-containing compounds are peroxides or
superoxides.
a. H2SO4
b. Ca(OH)2
c. BrOH
d. ClNO2
e. TiCl4
f. NaH

4.E.3.12: S4.2.7
a. H1+, O2-, S6+
b. H1+, O2-, Ca+2
c. H1+, O2-, Br1+
d. O2-, Cl1-, N5+
e. Cl1-, Ti4+
f. H1+, Na1-

4.E.3.13: Q4.2.8
Classify the following as acid-base reactions or oxidation-reduction reactions:
a. Na S(aq) + 2 HCl(aq) → 2 NaCl(aq) + H S(g)
2 2

b. 2 Na(s) + 2 HCl(aq) → 2 NaCl(aq) + H (g) 2

c. Mg(s) + Cl (g) → MgCl (s)2 2

d. MgO(s) + 2 HCl(aq) → MgCl (aq) + H O(l) 2 2

e. K P(s) + 2 O (g) → K PO (s)


3 2 3 4

f. 3 KOH(aq) + H PO (aq) → K PO (aq) + 3 H 3 4 3 4 2


O(l)

4.E.3.14: S4.2.9
acid-base; oxidation-reduction: Na is oxidized, H+ is reduced; oxidation-reduction: Mg is oxidized, Cl2 is reduced; acid-base;
oxidation-reduction: P3− is oxidized, O2 is reduced; acid-base

4.E.3.15: Q4.2.10
Identify the atoms that are oxidized and reduced, the change in oxidation state for each, and the oxidizing and reducing agents in each
of the following equations:
a. Mg(s) + NiCl (aq) → MgCl (aq) + Ni(s)
2 2

b. PCl (l) + Cl (g) → PCl (s)


3 2 5

c. C H (g) + 3 O (g) → 2 CO (g) + 2 H O(g)


2 4 2 2 2

d. Zn(s) + H SO (aq) → ZnSO (aq) + H (g)


2 4 4 2

e. 2 K S O (s) + I (s) → K S O (s) + 2 KI(s)


2 2 3 2 2 4 6

f. 3 Cu(s) + 8 HNO (aq) → 3 Cu(NO ) (aq) + 2 NO(g) + 4 H


3 3 2 2
O(l)

4.E.3.16: Q4.2.11
Complete and balance the following acid-base equations:
a. HCl gas reacts with solid Ca(OH)2(s).
b. A solution of Sr(OH)2 is added to a solution of HNO3.

Access for free at OpenStax 4.E.5 https://chem.libretexts.org/@go/page/58067


4.E.3.17: S4.2.11
a. 2 HCl(g) + Ca(OH) (s) → CaCl (s) + 2 H O(l) ;
2 2 2

b. Sr(OH) (aq) + 2 HNO (aq) → Sr(NO ) (aq) + 2 H


2 3 3 2 2
O(l)

4.E.3.18: Q4.2.12
Complete and balance the following acid-base equations:
a. A solution of HClO4 is added to a solution of LiOH.
b. Aqueous H2SO4 reacts with NaOH.
c. Ba(OH)2 reacts with HF gas.

4.E.3.19: Q4.2.13
Complete and balance the following oxidation-reduction reactions, which give the highest possible oxidation state for the oxidized
atoms.
a. Al(s) + F (g) →
2

b. Al(s) + CuBr (aq) → (single displacement)


2

c. P (s) + O (g) →
4 2

d. Ca(s) + H O(l) → (products are a strong base and a diatomic gas)


2

4.E.3.20: S4.2.13
a. 2 Al(s) + 3 F (g) → 2 AlF (s) ;
2 3

b. 2 Al(s) + 3 CuBr (aq) → 3 Cu(s) + 2 AlBr


2 3
(aq) ;
c. P (s) + 5 O (g) → P O (s) ; Ca(s) + 2 H
4 2 4 10 2
O(l) → Ca (OH) (aq) + H (g)
2 2

4.E.3.21: Q4.2.14
Complete and balance the following oxidation-reduction reactions, which give the highest possible oxidation state for the oxidized
atoms.
a. K(s) + H O(l) →
2

b. Ba(s) + HBr(aq) →
c. Sn(s) + I (s) →
2

4.E.3.22: Q4.2.15
Complete and balance the equations for the following acid-base neutralization reactions. If water is used as a solvent, write the
reactants and products as aqueous ions. In some cases, there may be more than one correct answer, depending on the amounts of
reactants used.
a. Mg(OH) (s) + HClO (aq) →
2 4

b. SO (g) + H O(l) → (assume an excess of water and that the product dissolves)
3 2

c. SrO(s) + H SO (l) → 2 4

4.E.3.23: S4.2.15
a. Mg(OH) (s) + 2 HClO (aq) → Mg (aq) + 2 ClO (aq) + 2 H O(l) ;
2 4
2+ −
4 2

b. SO (g) + 2 H O(l) → H O (aq) + HSO (aq) , (a solution of H2SO4);


3 2 3
+ −
4

c. SrO(s) + H SO (l) → SrSO (s) + H O


2 4 4 2

4.E.3.24: Q4.2.16
When heated to 700–800 °C, diamonds, which are pure carbon, are oxidized by atmospheric oxygen. (They burn!) Write the balanced
equation for this reaction.

4.E.3.25: Q4.2.17
The military has experimented with lasers that produce very intense light when fluorine combines explosively with hydrogen. What is
the balanced equation for this reaction?

Access for free at OpenStax 4.E.6 https://chem.libretexts.org/@go/page/58067


4.E.3.26: S4.2.17
H (g) + F (g) → 2 HF(g)
2 2

4.E.3.27: Q4.2.18
Write the molecular, total ionic, and net ionic equations for the following reactions:
a. Ca(OH) (aq) + HC H O (aq) →
2 2 3 2

b. H PO (aq) + CaCl (aq) →


3 4 2

4.E.3.28: Q4.2.19
Great Lakes Chemical Company produces bromine, Br2, from bromide salts such as NaBr, in Arkansas brine by treating the brine with
chlorine gas. Write a balanced equation for the reaction of NaBr with Cl2.

4.E.3.29: S4.2.19
2 NaBr(aq) + Cl (g) → 2 NaCl(aq) + Br (l)
2 2

4.E.3.30: Q4.2.20
In a common experiment in the general chemistry laboratory, magnesium metal is heated in air to produce MgO. MgO is a white solid,
but in these experiments it often looks gray, due to small amounts of Mg3N2, a compound formed as some of the magnesium reacts
with nitrogen. Write a balanced equation for each reaction.

4.E.3.31: Q4.2.21
Lithium hydroxide may be used to absorb carbon dioxide in enclosed environments, such as manned spacecraft and submarines. Write
an equation for the reaction that involves 2 mol of LiOH per 1 mol of CO2. (Hint: Water is one of the products.)

4.E.3.32: S4.2.21
2 LiOH(aq) + CO (g) → Li CO (aq) + H O(l)
2 2 3 2

4.E.3.33: Q4.2.22
Calcium propionate is sometimes added to bread to retard spoilage. This compound can be prepared by the reaction of calcium
carbonate, CaCO3, with propionic acid, C2H5CO2H, which has properties similar to those of acetic acid. Write the balanced equation
for the formation of calcium propionate.

4.E.3.34: Q4.2.23
Complete and balance the equations of the following reactions, each of which could be used to remove hydrogen sulfide from natural
gas:
a. Ca(OH) (s) + H 2 2
S(g) →

b. Na CO (aq) + H
2 3 2
S(g) →

4.E.3.35: S4.2.23
a. Ca(OH) (s) + H 2 2
S(g) → CaS(s) + 2 H O(l)
2
;
b. Na CO (aq) + H
2 3 2
S(g) → Na S(aq) + CO (g) + H O(l)
2 2 2

4.E.3.36: Q4.2.24
Copper(II) sulfide is oxidized by molecular oxygen to produce gaseous sulfur trioxide and solid copper(II) oxide. The gaseous product
then reacts with liquid water to produce liquid hydrogen sulfate as the only product. Write the two equations which represent these
reactions.

4.E.3.37: Q4.2.25
Write balanced chemical equations for the reactions used to prepare each of the following compounds from the given starting
material(s). In some cases, additional reactants may be required.
a. solid ammonium nitrate from gaseous molecular nitrogen via a two-step process (first reduce the nitrogen to ammonia, then
neutralize the ammonia with an appropriate acid)
b. gaseous hydrogen bromide from liquid molecular bromine via a one-step redox reaction

Access for free at OpenStax 4.E.7 https://chem.libretexts.org/@go/page/58067


c. gaseous H2S from solid Zn and S via a two-step process (first a redox reaction between the starting materials, then reaction of the
product with a strong acid)

4.E.3.38: S4.2.25
a. step 1: N (g) + 3 H (g) → 2 NH (g) , step 2: NH (g) + HNO (aq) → NH
2 2 3 3 3 4
NO (aq) → NH NO (s)(after drying)
3 4 3
;
b. H (g) + Br (l) → 2 HBr(g) ;
2 2

c. Zn(s) + S(s) → ZnS(s) and ZnS(s) + 2 HCl(aq) → ZnCl (aq) + H S(g) 2 2

4.E.3.39: Q4.2.26
Calcium cyclamate Ca(C6H11NHSO3)2 is an artificial sweetener used in many countries around the world but is banned in the United
States. It can be purified industrially by converting it to the barium salt through reaction of the acid C6H11NHSO3H with barium
carbonate, treatment with sulfuric acid (barium sulfate is very insoluble), and then neutralization with calcium hydroxide. Write the
balanced equations for these reactions.

4.E.3.40: Q4.2.27
Complete and balance each of the following half-reactions (steps 2–5 in half-reaction method):
a. Sn (aq) → Sn (aq)
4+ 2+

+
b. [Ag(NH ) ] (aq) → Ag(s) + NH (aq)
3 2 3

c. Hg Cl (s) → Hg(l) + Cl (aq)


2 2

d. H O(l) → O (g) (in acidic solution)


2 2

e. IO (aq) → I (s)

3 2

f. SO (aq) → SO (aq) (in acidic solution)


2−
3
2−
4

g. MnO (aq) → Mn (aq) (in acidic solution)


4
2+

h. Cl (aq) → ClO (aq) (in basic solution)


− −
3

4.E.3.41: S4.2.27
a. Sn (aq) + 2 e → Sn (aq) ,
4+ − 2+

b. [Ag(NH ) ] (aq) + e → Ag(s) + 2 NH (aq) ;


3 2
+ −
3

c. Hg Cl (s) + 2 e → 2 Hg(l) + 2 Cl (aq) ;


2 2
− −

d. 2 H O(l) → O (g) + 4 H (aq) + 4 e ;


2 2
+ −

e. 6 H O(l) + 2 IO (aq) + 10 e → I (s) + 12 OH (aq) ;


2

3

2

f. H O(l) + SO (aq) → SO (aq) + 2 H (aq) + 2 e ;


2
2−

3
2−

4
+ −

g. (g) 8 H (aq) + MnO (aq) + 5 e → Mn (aq) + 4 H O(l) ;


+ −
4
− 2+

h. (h) Cl (aq) + 6 OH (aq) → ClO (aq) + 3 H O(l) + 6 e


− − −
3 2

4.E.3.42: Q4.2.28
Complete and balance each of the following half-reactions (steps 2–5 in half-reaction method):
a. Cr (aq) → Cr (aq)
2+ 3+

b. Hg(l) + Br (aq) → HgBr (aq)


− 2−
4

c. ZnS(s) → Zn(s) + S (aq) 2−

d. H (g) → H O(l) (in basic solution)


2 2

e. H (g) → H O (aq) (in acidic solution)


2 3
+

f. NO (aq) → HNO (aq) (in acidic solution)



3 2

g. MnO (s) → MnO (aq) (in basic solution)


2

h. Cl (aq) → ClO (aq) (in acidic solution)


− −
3

4.E.3.43: Q4.2.29
Balance each of the following equations according to the half-reaction method:
a. Sn (aq) + Cu (aq) → Sn (aq) + Cu (aq)
2+ 2+ 4+ +

b. H S(g) + Hg (aq) → Hg(l) + S(s) (in acid)


2
2+
2

c. CN (aq) + ClO (aq) → CNO (aq) + Cl (aq) (in acid)


2
− −

d. Fe (aq) + Ce (aq) → Fe (aq) + Ce (aq)


2+ 4+ 3+ 3+

e. HBrO(aq) → Br (aq) + O (g) (in acid) −


2

Access for free at OpenStax 4.E.8 https://chem.libretexts.org/@go/page/58067


4.E.3.44: S4.2.29
a. Sn (aq) + 2 Cu (aq) → Sn (aq) + 2 Cu (aq) ;
2+ 2+ 4+ +

b. H S(g) + Hg (aq) + 2 H O(l) → 2 Hg(l) + S(s) + 2 H O (aq) ;


2
2+
2 2 3
+

c. 5 CN (aq) + 2 ClO (aq) + 3 H O(l) → 5 CNO (aq) + 2 Cl (aq) + 2 H


2 2
− −

3
O
+
(aq) ;
d. Fe (aq) + Ce (aq) → Fe (aq) + Ce (aq) ;
2+ 4+ 3+ 3+

e. 2 HBrO(aq) + 2 H O(l) → 2 H O (aq) + 2 Br (aq) + O (g)


2 3
+ −
2

4.E.3.45: Q4.2.30
Balance each of the following equations according to the half-reaction method:
a. Zn(s) + NO (aq) → Zn (aq) + N (g) (in acid)

3
2+

b. Zn(s) + NO (aq) → Zn (aq) + NH (aq) (in base)


3
2+
3

c. CuS(s) + NO (aq) → Cu (aq) + S(s) + NO(g) (in acid)


3
2+

d. NH (aq) + O (g) → NO (g) (gas phase)


3 2 2

e. Cl (g) + OH (aq) → Cl (aq) + ClO (aq) (in base)


2
− − −

f. H O (aq) + MnO (aq) → Mn (aq) + O (g) (in acid)


2 2

4
2+
2

g. NO (g) → NO (aq) + NO (aq) (in base)


2

3

2

h. Fe (aq) + I (aq) → Fe (aq) + I (aq)


3+ − 2+

4.E.3.46: Q4.2.31
Balance each of the following equations according to the half-reaction method:
a. MnO (aq) + NO (aq) → MnO (s) + NO (aq) (in base)

4

2 2

b. MnO (aq) → MnO (aq) + MnO (s) (in base)


2−
4

4 2

c. Br (l) + SO (g) → Br (aq) + SO (aq) (in acid)


2 2
− 2−
4

4.E.3.47: S4.2.31
a. 2 MnO (aq) + 3 NO (aq) + H O(l) → 2 MnO (s) + 3 NO (aq) + 2 OH (aq) ;

4

2 2 2

3

b. 3 MnO (aq) + 2 H O(l) → 2 MnO (aq) + 4 OH (aq) + MnO (s) (in base) ;
2−
4 2

4

2

c. Br (l) + SO (g) + 2 H O(l) → 4 H (aq) + 2 Br (aq) + SO (aq)


2 2 2
+ − 2−

4.E.4: 4.3: Reaction Stoichiometry


4.E.4.1: Q4.3.1
Write the balanced equation, then outline the steps necessary to determine the information requested in each of the following:
a. The number of moles and the mass of chlorine, Cl2, required to react with 10.0 g of sodium metal, Na, to produce sodium chloride,
NaCl.
b. The number of moles and the mass of oxygen formed by the decomposition of 1.252 g of mercury(II) oxide.
c. The number of moles and the mass of sodium nitrate, NaNO3, required to produce 128 g of oxygen. (NaNO2 is the other product.)
d. The number of moles and the mass of carbon dioxide formed by the combustion of 20.0 kg of carbon in an excess of oxygen.
e. The number of moles and the mass of copper(II) carbonate needed to produce 1.500 kg of copper(II) oxide. (CO2 is the other
product.)

f.

4.E.4.2: Q4.3.2
Determine the number of moles and the mass requested for each reaction in Exercise.

4.E.4.3: S4.3.2
0.435 mol Na, 0.217 mol Cl2, 15.4 g Cl2; 0.005780 mol HgO, 2.890 × 10−3 mol O2, 9.248 × 10−2 g O2; 8.00 mol NaNO3, 6.8 × 102 g
NaNO3; 1665 mol CO2, 73.3 kg CO2; 18.86 mol CuO, 2.330 kg CuCO3; 0.4580 mol C2H4Br2, 86.05 g C2H4Br2

Access for free at OpenStax 4.E.9 https://chem.libretexts.org/@go/page/58067


4.E.4.4: Q4.3.3
Write the balanced equation, then outline the steps necessary to determine the information requested in each of the following:
a. The number of moles and the mass of Mg required to react with 5.00 g of HCl and produce MgCl2 and H2.
b. The number of moles and the mass of oxygen formed by the decomposition of 1.252 g of silver(I) oxide.
c. The number of moles and the mass of magnesium carbonate, MgCO3, required to produce 283 g of carbon dioxide. (MgO is the
other product.)
d. The number of moles and the mass of water formed by the combustion of 20.0 kg of acetylene, C2H2, in an excess of oxygen.
e. The number of moles and the mass of barium peroxide, BaO2, needed to produce 2.500 kg of barium oxide, BaO (O2 is the other
product.)

f.

4.E.4.5: Q4.3.4
Determine the number of moles and the mass requested for each reaction in Exercise.

4.E.4.6: S4.3.4
0.0686 mol Mg, 1.67 g Mg; 2.701 × 10−3 mol O2, 0.08644 g O2; 6.43 mol MgCO3, 542 g MgCO3 713 mol H2O, 12.8 kg H2O; 16.31
mol BaO2, 2762 g BaO2; 0.207 mol C2H4, 5.81 g C2H4

4.E.4.7: Q4.3.5
H2 is produced by the reaction of 118.5 mL of a 0.8775-M solution of H3PO4 according to the following equation:
2 Cr + 2 H PO → 3 H + 2 CrPO .
3 4 2 4

a. Outline the steps necessary to determine the number of moles and mass of H2.
b. Perform the calculations outlined.

4.E.4.8: S4.3.5
a.)
1. Convert mL to L
2. Multiply L by the molarity to determine moles of H3PO4
3. Convert moles of H3PO4 to moles of H2
4. Multiply moles of H2 by the molar mass of H2 to get the answer in grams
b.)
1 L
1. 118.5 mL × = 0.1185 L
1000 mL

0.8775 moles H PO
2. 0.1185 L × 3 4
= 0.1040 moles H PO
3 4
1 L

3 moles H
2
3. 0.1040 moles H 3
PO
4
× = 0.1560 moles H
2
2 moles H PO
3 4

2.02g
4. 0.1560 moles H 2
× = 0.3151g H
2
1 mole

4.E.4.9: Q4.3.6
Gallium chloride is formed by the reaction of 2.6 L of a 1.44 M solution of HCl according to the following equation:
2 Ga + 6 HCl → 2 GaCl + 3 H .
3 2

a. Outline the steps necessary to determine the number of moles and mass of gallium chloride.
b. Perform the calculations outlined.

Access for free at OpenStax 4.E.10 https://chem.libretexts.org/@go/page/58067


4.E.4.10: S4.3.6
volume HCl solution → mol HCl → mol GaCl3 ; 1.25 mol GaCl3, 2.2 × 102 g GaCl3

4.E.4.11: Q4.3.7
I2 is produced by the reaction of 0.4235 mol of CuCl2 according to the following equation: 2 CuCl 2
+ 4 KI → 2 CuI + 4 KCl + I
2
.
a. How many molecules of I2 are produced?
b. What mass of I2 is produced?

4.E.4.12: Q4.3.8
Silver is often extracted from ores as K[Ag(CN)2] and then recovered by the reaction
2 K[Ag (CN) ](aq) + Zn(s) → 2 Ag(s) + Zn(CN) (aq) + 2 KCN(aq)
2 2

a. How many molecules of Zn(CN)2 are produced by the reaction of 35.27 g of K[Ag(CN)2]?
b. What mass of Zn(CN)2 is produced?

4.E.4.13: S4.3.8
5.337 × 1022 molecules; 10.41 g Zn(CN)2

4.E.4.14: Q4.3.9
What mass of silver oxide, Ag2O, is required to produce 25.0 g of silver sulfadiazine, AgC10H9N4SO2, from the reaction of silver
oxide and sulfadiazine?
2C H N SO + Ag O → 2 AgC H N SO +H O
10 10 4 2 2 10 9 4 2 2

4.E.4.15: Q4.3.10
Carborundum is silicon carbide, SiC, a very hard material used as an abrasive on sandpaper and in other applications. It is prepared by
the reaction of pure sand, SiO2, with carbon at high temperature. Carbon monoxide, CO, is the other product of this reaction. Write the
balanced equation for the reaction, and calculate how much SiO2 is required to produce 3.00 kg of SiC.

4.E.4.16: S4.3.10
SiO
2
+ 3 C → SiC + 2 CO , 4.50 kg SiO2

4.E.4.17: Q4.3.11
Automotive air bags inflate when a sample of sodium azide, NaN3, is very rapidly decomposed.
2 NaN (s) → 2 Na(s) + 3 N (g)
3 2

What mass of sodium azide is required to produce 2.6 ft3 (73.6 L) of nitrogen gas with a density of 1.25 g/L?

4.E.4.18: S4.3.11
142g NaN3

4.E.4.19: Q4.3.12
Urea, CO(NH2)2, is manufactured on a large scale for use in producing urea-formaldehyde plastics and as a fertilizer. What is the
maximum mass of urea that can be manufactured from the CO2 produced by combustion of 1.00×103 kg of carbon followed by the
reaction?
CO (g) + 2 NH (g) → CO(NH ) (s) + H O(l) (4.E.7)
2 3 2 2 2

4.E.4.20: S4.3.12
5.00 × 103 kg

4.E.4.21: Q4.3.13
In an accident, a solution containing 2.5 kg of nitric acid was spilled. Two kilograms of Na2CO3 was quickly spread on the area and
CO2 was released by the reaction. Was sufficient Na2CO3 used to neutralize all of the acid?

Access for free at OpenStax 4.E.11 https://chem.libretexts.org/@go/page/58067


4.E.4.22: Q4.3.14
A compact car gets 37.5 miles per gallon on the highway. If gasoline contains 84.2% carbon by mass and has a density of 0.8205
g/mL, determine the mass of carbon dioxide produced during a 500-mile trip (3.785 liters per gallon).

4.E.4.23: S4.3.14
1.28 × 105 g CO2

4.E.4.24: Q4.3.15
What volume of a 0.750 M solution of hydrochloric acid, a solution of HCl, can be prepared from the HCl produced by the reaction of
25.0 g of NaCl with an excess of sulfuric acid?
NaCl(s) + H SO (l) → HCl(g) + NaHSO (s) (4.E.8)
2 4 4

4.E.4.25: Q4.3.16
What volume of a 0.2089 M KI solution contains enough KI to react exactly with the Cu(NO3)2 in 43.88 mL of a 0.3842 M solution of
Cu(NO3)2?
2 Cu(NO ) + 4 KI → 2 CuI + I + 4 KNO (4.E.9)
3 2 2 3

4.E.4.26: S4.3.16
161.40 mL KI solution

4.E.4.27: Q4.3.17
A mordant is a substance that combines with a dye to produce a stable fixed color in a dyed fabric. Calcium acetate is used as a
mordant. It is prepared by the reaction of acetic acid with calcium hydroxide.

2 CH CO H + Ca (OH) → Ca (CH CO ) +2 H O (4.E.10)


3 2 2 3 2 2 2

What mass of Ca(OH)2 is required to react with the acetic acid in 25.0 mL of a solution having a density of 1.065 g/mL and containing
58.0% acetic acid by mass?

4.E.4.28: Q4.3.18
The toxic pigment called white lead, Pb3(OH)2(CO3)2, has been replaced in white paints by rutile, TiO2. How much rutile (g) can be
prepared from 379 g of an ore that contains 88.3% ilmenite (FeTiO3) by mass?
2 FeTiO + 4 HCl + Cl → 2 FeCl + 2 TiO +2 H O (4.E.11)
3 2 3 2 2

4.E.4.29: S4.3.18
176 g TiO2

4.E.5: 4.4: Reaction Yields


4.E.5.1: Q4.4.1
The following quantities are placed in a container: 1.5 × 1024 atoms of hydrogen, 1.0 mol of sulfur, and 88.0 g of diatomic oxygen.
a. What is the total mass in grams for the collection of all three elements?
b. What is the total number of moles of atoms for the three elements?
c. If the mixture of the three elements formed a compound with molecules that contain two hydrogen atoms, one sulfur atom, and
four oxygen atoms, which substance is consumed first?
d. How many atoms of each remaining element would remain unreacted in the change described in ?

4.E.5.2: Q4.4.2
What is the limiting reactant in a reaction that produces sodium chloride from 8 g of sodium and 8 g of diatomic chlorine?

4.E.5.3: S4.4.2
The limiting reactant is Cl2.

Access for free at OpenStax 4.E.12 https://chem.libretexts.org/@go/page/58067


4.E.5.4: Q4.4.3
Which of the postulates of Dalton's atomic theory explains why we can calculate a theoretical yield for a chemical reaction?

4.E.5.5: Q4.4.4
A student isolated 25 g of a compound following a procedure that would theoretically yield 81 g. What was his percent yield?

4.E.5.6: S4.4.4
Percent yield = 31%

4.E.5.7: Q4.4.5
A sample of 0.53 g of carbon dioxide was obtained by heating 1.31 g of calcium carbonate. What is the percent yield for this reaction?
CaCO (s) → CaO(s) + CO (s) (4.E.12)
3 2

4.E.5.8: Q4.4.6
Freon-12, CCl2F2, is prepared from CCl4 by reaction with HF. The other product of this reaction is HCl. Outline the steps needed to
determine the percent yield of a reaction that produces 12.5 g of CCl2F2 from 32.9 g of CCl4. Freon-12 has been banned and is no
longer used as a refrigerant because it catalyzes the decomposition of ozone and has a very long lifetime in the atmosphere. Determine
the percent yield.

4.E.5.9: S4.4.6
g CCl → mol CCl → mol CCl F → g CCl F , percent yield = 48.3%
4 4 2 2 2 2

4.E.5.10: Q4.4.7
Citric acid, C6H8O7, a component of jams, jellies, and fruity soft drinks, is prepared industrially via fermentation of sucrose by the
mold Aspergillus niger. The equation representing this reaction is
C H O +H O+3 O → 2C H O +4 H O (4.E.13)
12 22 11 2 2 6 8 7 2

What mass of citric acid is produced from exactly 1 metric ton (1.000 × 103 kg) of sucrose if the yield is 92.30%?

4.E.5.11: Q4.4.8
Toluene, C6H5CH3, is oxidized by air under carefully controlled conditions to benzoic acid, C6H5CO2H, which is used to prepare the
food preservative sodium benzoate, C6H5CO2Na. What is the percent yield of a reaction that converts 1.000 kg of toluene to 1.21 kg
of benzoic acid?
2 C H CH +3 O → 2 C H CO H + 2 H O (4.E.14)
6 5 3 2 6 5 2 2

4.E.5.12: S4.4.8
percent yield = 91.3%

4.E.5.13: Q4.4.9
In a laboratory experiment, the reaction of 3.0 mol of H2 with 2.0 mol of I2 produced 1.0 mol of HI. Determine the theoretical yield in
grams and the percent yield for this reaction.

4.E.5.14: Q4.4.10
Outline the steps needed to solve the following problem, then do the calculations. Ether, (C2H5)2O, which was originally used as an
anesthetic but has been replaced by safer and more effective medications, is prepared by the reaction of ethanol with sulfuric acid.
2C2H5OH + H2SO4 ⟶ (C2H5)2 + H2SO4·H2O

4.E.5.15: Q4.4.11
What is the percent yield of ether if 1.17 L (d = 0.7134 g/mL) is isolated from the reaction of 1.500 L of C2H5OH (d = 0.7894 g/mL)?

4.E.5.16: S4.4.11
Convert mass of ethanol to moles of ethanol; relate the moles of ethanol to the moles of ether produced using the stoichiometry of the
balanced equation. Convert moles of ether to grams; divide the actual grams of ether (determined through the density) by the
theoretical mass to determine the percent yield; 87.6%

Access for free at OpenStax 4.E.13 https://chem.libretexts.org/@go/page/58067


4.E.5.17: Q4.4.12
Outline the steps needed to determine the limiting reactant when 30.0 g of propane, C3H8, is burned with 75.0 g of oxygen.
0.8347 g
percent yield = × 100% = 87.6% (4.E.15)
0.9525 g

Determine the limiting reactant.

4.E.5.18: Q4.4.13
Outline the steps needed to determine the limiting reactant when 0.50 g of Cr and 0.75 g of H3PO4 react according to the following
chemical equation?
2 Cr + 2 H PO → 2 CrPO +3 H (4.E.16)
3 4 4 2

Determine the limiting reactant.

4.E.5.19: S4.4.13
The conversion needed is mol Cr → mol H PO
2 4
. Then compare the amount of Cr to the amount of acid present. Cr is the limiting
reactant.

4.E.5.20: Q4.4.14
What is the limiting reactant when 1.50 g of lithium and 1.50 g of nitrogen combine to form lithium nitride, a component of advanced
batteries, according to the following unbalanced equation?
Li + N → Li N (4.E.17)
2 3

4.E.5.21: S4.4.14
6 Li + N → 2 Li N (4.E.18)
2 3

1 mole Li 2 mole Li N
3
1.50g Li × × = 0.0720 moles Li N (4.E.19)
3
6.94g Li 6 mole Li

1 mole N 2 mole Li N
2 3
1.50g N × × = 0.107 moles Li N (4.E.20)
2 3
28.02g N 1 mole N
2 2

Li is the limiting reactant

4.E.5.22: Q4.4.15
Uranium can be isolated from its ores by dissolving it as UO2(NO3)2, then separating it as solid UO2(C2O4)·3H2O. Addition of 0.4031
g of sodium oxalate, Na2C2O4, to a solution containing 1.481 g of uranyl nitrate, UO2(NO2)2, yields 1.073 g of solid
UO2(C2O4)·3H2O.

Na C O + UO (NO ) + 3 H O ⟶ UO (C O ) ⋅ 3 H O + 2 NaNO (4.E.21)


2 2 4 2 3 2 2 2 2 4 2 3

Determine the limiting reactant and the percent yield of this reaction.

4.E.5.23: S4.4.15
Na2C2O4 is the limiting reactant. percent yield = 86.6%

4.E.5.24: Q4.4.16
How many molecules of C2H4Cl2 can be prepared from 15 C2H4 molecules and 8 Cl2 molecules?

4.E.5.25: Q4.4.17
How many molecules of the sweetener saccharin can be prepared from 30 C atoms, 25 H atoms, 12 O atoms, 8 S atoms, and 14 N
atoms?

Access for free at OpenStax 4.E.14 https://chem.libretexts.org/@go/page/58067


4.E.5.26: S4.4.17
Only four molecules can be made.

4.E.5.27: Q4.4.18
The phosphorus pentoxide used to produce phosphoric acid for cola soft drinks is prepared by burning phosphorus in oxygen.
a. What is the limiting reactant when 0.200 mol of P4 and 0.200 mol of O2 react according to

P +5 O → P O (4.E.22)
4 2 4 10

b. Calculate the percent yield if 10.0 g of P4O10 is isolated from the reaction.

4.E.5.28: Q4.4.19
Would you agree to buy 1 trillion (1,000,000,000,000) gold atoms for $5? Explain why or why not. Find the current price of gold at
http://money.cnn.com/data/commodities/ (1 troy ounce = 31.1 g)

4.E.5.29: S4.4.19
This amount cannot be weighted by ordinary balances and is worthless.

4.E.6: 4.5: Quantitative Chemical Analysis


4.E.6.1: Q4.5.1
What volume of 0.0105-M HBr solution is be required to titrate 125 mL of a 0.0100-M Ca(OH)2 solution?
Ca (OH) (aq) + 2 HBr(aq) → CaBr (aq) + 2 H O(l) (4.E.23)
2 2 2

4.E.6.2: Q4.5.2
Titration of a 20.0-mL sample of acid rain required 1.7 mL of 0.0811 M NaOH to reach the end point. If we assume that the acidity of
the rain is due to the presence of sulfuric acid, what was the concentration of sulfuric acid in this sample of rain?

4.E.6.3: S4.5.2
3.4 × 10−3 M H2SO4

4.E.6.4: Q4.5.3
What is the concentration of NaCl in a solution if titration of 15.00 mL of the solution with 0.2503 M AgNO3 requires 20.22 mL of the
AgNO3 solution to reach the end point?
AgNO (aq) + NaCl(aq) → AgCl(s) + NaNO (aq) (4.E.24)
3 3

4.E.6.5: Q4.5.4
In a common medical laboratory determination of the concentration of free chloride ion in blood serum, a serum sample is titrated
with a Hg(NO3)2 solution.
− −
2 Cl (aq) + Hg (NO ) (aq) → 2 NO3 (aq) + HgCl (s) (4.E.25)
3 2 2

What is the Cl− concentration in a 0.25-mL sample of normal serum that requires 1.46 mL of 5.25 × 10−4 M Hg(NO3)2(aq) to reach
the end point?

4.E.6.6: S4.5.4
9.6 × 10−3 M Cl−

Access for free at OpenStax 4.E.15 https://chem.libretexts.org/@go/page/58067


4.E.6.7: Q4.5.5
Potatoes can be peeled commercially by soaking them in a 3-M to 6-M solution of sodium hydroxide, then removing the loosened
skins by spraying them with water. Does a sodium hydroxide solution have a suitable concentration if titration of 12.00 mL of the
solution requires 30.6 mL of 1.65 M HCI to reach the end point?

4.E.6.8: Q4.5.6
A sample of gallium bromide, GaBr2, weighing 0.165 g was dissolved in water and treated with silver nitrate, AgNO3, resulting in the
precipitation of 0.299 g AgBr. Use these data to compute the %Ga (by mass) GaBr2.

4.E.6.9: S4.5.6
22.4%

4.E.6.10: Q4.5.7
The principal component of mothballs is naphthalene, a compound with a molecular mass of about 130 amu, containing only carbon
and hydrogen. A 3.000-mg sample of naphthalene burns to give 10.3 mg of CO2. Determine its empirical and molecular formulas.

4.E.6.11: Q4.5.8
A 0.025-g sample of a compound composed of boron and hydrogen, with a molecular mass of ~28 amu, burns spontaneously when
exposed to air, producing 0.063 g of B2O3. What are the empirical and molecular formulas of the compound.

4.E.6.12: S4.5.8
The empirical formula is BH3. The molecular formula is B2H6.

4.E.6.13: Q4.5.9
Sodium bicarbonate (baking soda), NaHCO3, can be purified by dissolving it in hot water (60 °C), filtering to remove insoluble
impurities, cooling to 0 °C to precipitate solid NaHCO3, and then filtering to remove the solid, leaving soluble impurities in solution.
Any NaHCO3 that remains in solution is not recovered. The solubility of NaHCO3 in hot water of 60 °C is 164 g L. Its solubility in
cold water of 0 °C is 69 g/L. What is the percent yield of NaHCO3 when it is purified by this method?

4.E.6.14: Q4.5.10
What volume of 0.600 M HCl is required to react completely with 2.50 g of sodium hydrogen carbonate?
NaHCO (aq) + HCl(aq) → NaCl(aq) + CO (g) + H O(l) (4.E.26)
3 2 2

4.E.6.15: S4.5.10
49.6 mL

4.E.6.16: Q4.5.11
What volume of 0.08892 M HNO3 is required to react completely with 0.2352 g of potassium hydrogen phosphate?

2 HNO (aq) + K HPO (aq) → H PO (aq) + 2 KNO (aq) (4.E.27)


3 2 4 2 4 3

4.E.6.17: Q4.5.12
What volume of a 0.3300-M solution of sodium hydroxide would be required to titrate 15.00 mL of 0.1500 M oxalic acid?
C O H (aq) + 2 NaOH(aq) → Na C O (aq) + 2 H O(l) (4.E.28)
2 4 2 2 2 4 2

4.E.6.18: S4.5.12
13.64 mL

4.E.6.19: Q4.5.13
What volume of a 0.00945-M solution of potassium hydroxide would be required to titrate 50.00 mL of a sample of acid rain with a
H2SO4 concentration of 1.23 × 10−4 M.
H SO (aq) + 2 KOH(aq) → K SO (aq) + 2 H O(l) (4.E.29)
2 4 2 4 2

Access for free at OpenStax 4.E.16 https://chem.libretexts.org/@go/page/58067


4.E.6.20: S4.5.13
1.30 mL

4.E.6.21: Q4.5.14
A sample of solid calcium hydroxide, Ca(OH)2, is allowed to stand in water until a saturated solution is formed. A titration of 75.00
mL of this solution with 5.00 × 10−2 M HCl requires 36.6 mL of the acid to reach the end point.

Ca (OH) (aq) + 2 HCl(aq) → CaCl (aq) + 2 H O(l) (4.E.30)


2 2 2

What is the molarity?

4.E.6.22: S4.5.14
1.22 M

4.E.6.23: Q4.5.15
What mass of Ca(OH)2 will react with 25.0 g of propionic acid to form the preservative calcium propionate according to the equation?

4.E.6.24: Q4.5.16
How many milliliters of a 0.1500-M solution of KOH will be required to titrate 40.00 mL of a 0.0656-M solution of H3PO4?
H PO (aq) + 2 KOH(aq) → K HPO (aq) + 2 H O(l) (4.E.31)
3 4 2 4 2

4.E.6.25: S4.5.16
34.99 mL KOH

4.E.6.26: Q4.5.17
Potassium acid phthalate, KHC6H4O4, or KHP, is used in many laboratories, including general chemistry laboratories, to standardize
solutions of base. KHP is one of only a few stable solid acids that can be dried by warming and weighed. A 0.3420-g sample of
KHC6H4O4 reacts with 35.73 mL of a NaOH solution in a titration. What is the molar concentration of the NaOH?

KHC H O (aq) + NaOH(aq) → KNaC H O (aq) + H O(aq) (4.E.32)


6 4 4 6 4 4 2

4.E.6.27: Q4.5.18
The reaction of WCl6 with Al at ~400 °C gives black crystals of a compound containing only tungsten and chlorine. A sample of this
compound, when reduced with hydrogen, gives 0.2232 g of tungsten metal and hydrogen chloride, which is absorbed in water.
Titration of the hydrochloric acid thus produced requires 46.2 mL of 0.1051 M NaOH to reach the end point. What is the empirical
formula of the black tungsten chloride?

4.E.6.28: S4.5.19
The empirical formula is WCl4.

This page titled 4.E: Stoichiometry of Chemical Reactions (Exercises) is shared under a CC BY 4.0 license and was authored, remixed, and/or
curated by OpenStax via source content that was edited to the style and standards of the LibreTexts platform; a detailed edit history is available upon
request.

Access for free at OpenStax 4.E.17 https://chem.libretexts.org/@go/page/58067


CHAPTER OVERVIEW
5: Thermochemistry

A general chemistry Libretexts Textbook remixed and remastered from


OpenStax's textbook:
General Chemistry
Useful forms of energy are also available from a variety of chemical reactions other than combustion. For example, the energy
produced by the batteries in a cell phone, car, or flashlight results from chemical reactions. This chapter introduces many of the
basic ideas necessary to explore the relationships between chemical changes and energy, with a focus on thermal energy.
5.0: Prelude to Thermochemistry
5.1: Energy Basics
5.2: Calorimetry
5.3: Enthalpy
5.E: Thermochemistry (Exercises)

Thumbnail: Flames of charcoal. (CC BY-SA 3.0; Oscar via Wikipedia)

This page titled 5: Thermochemistry is shared under a CC BY 4.0 license and was authored, remixed, and/or curated by OpenStax via source
content that was edited to the style and standards of the LibreTexts platform; a detailed edit history is available upon request.

1
5.0: Prelude to Thermochemistry
Chemical reactions, such as those that occur when you light a match, involve changes in energy as well as matter. Societies at all
levels of development could not function without the energy released by chemical reactions. In 2012, about 85% of US energy
consumption came from the combustion of petroleum products, coal, wood, and garbage. We use this energy to produce electricity
(38%); to transport food, raw materials, manufactured goods, and people (27%); for industrial production (21%); and to heat and
power our homes and businesses (10%).1 While these combustion reactions help us meet our essential energy needs, they are also
recognized by the majority of the scientific community as a major contributor to global climate change.

Figure 5.0.1 : Sliding a match head along a rough surface initiates a combustion reaction that produces energy in the form of heat
and light. (credit: modification of work by Laszlo Ilyes).
Useful forms of energy are also available from a variety of chemical reactions other than combustion. For example, the energy
produced by the batteries in a cell phone, car, or flashlight results from chemical reactions. This chapter introduces many of the
basic ideas necessary to explore the relationships between chemical changes and energy, with a focus on thermal energy.

Footnotes
1. US Energy Information Administration, Primary Energy Consumption by Source and Sector, 2012, Total Energy
[www.eia.gov]. Data derived from US Energy Information Administration, Monthly Energy Review (January 2014).

This page titled 5.0: Prelude to Thermochemistry is shared under a CC BY 4.0 license and was authored, remixed, and/or curated by OpenStax via
source content that was edited to the style and standards of the LibreTexts platform; a detailed edit history is available upon request.

Access for free at OpenStax 5.0.1 https://chem.libretexts.org/@go/page/38164


5.1: Energy Basics
 Learning Objectives
Define energy, distinguish types of energy, and describe the nature of energy changes that accompany chemical and
physical changes
Distinguish the related properties of heat, thermal energy, and temperature
Define and distinguish specific heat and heat capacity, and describe the physical implications of both
Perform calculations involving heat, specific heat, and temperature change

Chemical changes and their accompanying changes in energy are important parts of our everyday world (Figure 5.1.1). The
macronutrients in food (proteins, fats, and carbohydrates) undergo metabolic reactions that provide the energy to keep our bodies
functioning. We burn a variety of fuels (gasoline, natural gas, coal) to produce energy for transportation, heating, and the
generation of electricity. Industrial chemical reactions use enormous amounts of energy to produce raw materials (such as iron and
aluminum). Energy is then used to manufacture those raw materials into useful products, such as cars, skyscrapers, and bridges.

Figure 5.1.1 : The energy involved in chemical changes is important to our daily lives: (a) A cheeseburger for lunch provides the
energy you need to get through the rest of the day; (b) the combustion of gasoline provides the energy that moves your car (and
you) between home, work, and school; and (c) coke, a processed form of coal, provides the energy needed to convert iron ore into
iron, which is essential for making many of the products we use daily. (credit a: modification of work by “Pink Sherbet
Photography”/Flickr; credit b: modification of work by Jeffery Turner).
Three pictures are shown and labeled a, b, and c. Picture a is a cheeseburger. Picture b depicts a highway that is full of traffic.
Picture c is a view into an industrial metal furnace. The view into the furnace shows a hot fire burning inside.
Over 90% of the energy we use comes originally from the sun. Every day, the sun provides the earth with almost 10,000 times the
amount of energy necessary to meet all of the world’s energy needs for that day. Our challenge is to find ways to convert and store
incoming solar energy so that it can be used in reactions or chemical processes that are both convenient and nonpolluting. Plants
and many bacteria capture solar energy through photosynthesis. We release the energy stored in plants when we burn wood or plant
products such as ethanol. We also use this energy to fuel our bodies by eating food that comes directly from plants or from animals
that got their energy by eating plants. Burning coal and petroleum also releases stored solar energy: These fuels are fossilized plant
and animal matter.
This chapter will introduce the basic ideas of an important area of science concerned with the amount of heat absorbed or released
during chemical and physical changes—an area called thermochemistry. The concepts introduced in this chapter are widely used in
almost all scientific and technical fields. Food scientists use them to determine the energy content of foods. Biologists study the
energetics of living organisms, such as the metabolic combustion of sugar into carbon dioxide and water. The oil, gas, and
transportation industries, renewable energy providers, and many others endeavor to find better methods to produce energy for our
commercial and personal needs. Engineers strive to improve energy efficiency, find better ways to heat and cool our homes,
refrigerate our food and drinks, and meet the energy and cooling needs of computers and electronics, among other applications.
Understanding thermochemical principles is essential for chemists, physicists, biologists, geologists, every type of engineer, and
just about anyone who studies or does any kind of science.

5.1.1: Energy
Energy can be defined as the capacity to supply heat or do work. One type of work (w) is the process of causing matter to move
against an opposing force. For example, we do work when we inflate a bicycle tire—we move matter (the air in the pump) against
the opposing force of the air surrounding the tire.

Access for free at OpenStax 5.1.1 https://chem.libretexts.org/@go/page/38165


Like matter, energy comes in different types. One scheme classifies energy into two types: potential energy, the energy an object
has because of its relative position, composition, or condition, and kinetic energy, the energy that an object possesses because of its
motion. Water at the top of a waterfall or dam has potential energy because of its position; when it flows downward through
generators, it has kinetic energy that can be used to do work and produce electricity in a hydroelectric plant (Figure 5.1.2). A
battery has potential energy because the chemicals within it can produce electricity that can do work.

Figure 5.1.2 : (a) Water that is higher in elevation, for example, at the top of Victoria Falls, has a higher potential energy than water
at a lower elevation. As the water falls, some of its potential energy is converted into kinetic energy. (b) If the water flows through
generators at the bottom of a dam, such as the Hoover Dam shown here, its kinetic energy is converted into electrical energy.
(credit a: modification of work by Steve Jurvetson; credit b: modification of work by “curimedia”/Wikimedia commons).
Two pictures are shown and labeled a and b. Picture a shows a large waterfall with water falling from a high elevation at the top of
the falls to a lower elevation. The second picture is a view looking down into the Hoover Dam. Water is shown behind the high
wall of the dam on one side and at the base of the dam on the other.
Energy can be converted from one form into another, but all of the energy present before a change occurs always exists in some
form after the change is completed. This observation is expressed in the law of conservation of energy: during a chemical or
physical change, energy can be neither created nor destroyed, although it can be changed in form. (This is also one version of the
first law of thermodynamics, as you will learn later.)
When one substance is converted into another, there is always an associated conversion of one form of energy into another. Heat is
usually released or absorbed, but sometimes the conversion involves light, electrical energy, or some other form of energy. For
example, chemical energy (a type of potential energy) is stored in the molecules that compose gasoline. When gasoline is
combusted within the cylinders of a car’s engine, the rapidly expanding gaseous products of this chemical reaction generate
mechanical energy (a type of kinetic energy) when they move the cylinders’ pistons.
According to the law of conservation of matter (seen in an earlier chapter), there is no detectable change in the total amount of
matter during a chemical change. When chemical reactions occur, the energy changes are relatively modest and the mass changes
are too small to measure, so the laws of conservation of matter and energy hold well. However, in nuclear reactions, the energy
changes are much larger (by factors of a million or so), the mass changes are measurable, and matter-energy conversions are
significant. This will be examined in more detail in a later chapter on nuclear chemistry. To encompass both chemical and nuclear
changes, we combine these laws into one statement: The total quantity of matter and energy in the universe is fixed.

5.1.2: Thermal Energy, Temperature, and Heat


Thermal energy is kinetic energy associated with the random motion of atoms and molecules. Temperature is a quantitative
measure of “hot” or “cold.” When the atoms and molecules in an object are moving or vibrating quickly, they have a higher
average kinetic energy (KE), and we say that the object is “hot.” When the atoms and molecules are moving slowly, they have
lower KE, and we say that the object is “cold” (Figure 5.1.3). Assuming that no chemical reaction or phase change (such as
melting or vaporizing) occurs, increasing the amount of thermal energy in a sample of matter will cause its temperature to increase.
And, assuming that no chemical reaction or phase change (such as condensation or freezing) occurs, decreasing the amount of
thermal energy in a sample of matter will cause its temperature to decrease.

Access for free at OpenStax 5.1.2 https://chem.libretexts.org/@go/page/38165


Figure 5.1.3 : (a) The molecules in a sample of hot water move more rapidly than (b) those in a sample of cold water.
Two molecular drawings are shown and labeled a and b. Drawing a is a box containing fourteen red spheres that are surrounded by
lines indicating that the particles are moving rapidly. This drawing has a label that reads “Hot water.” Drawing b depicts another
box of equal size that also contains fourteen spheres, but these are blue. They are all surrounded by smaller lines that depict some
particle motion, but not as much as in drawing a. This drawing has a label that reads “Cold water.”
Most substances expand as their temperature increases and contract as their temperature decreases. This property can be used to
measure temperature changes, as shown in Figure 5.1.4. The operation of many thermometers depends on the expansion and
contraction of substances in response to temperature changes.

A picture labeled a is shown as well as a pair of drawings labeled b. Picture a shows the lower portion of an alcohol thermometer.
The thermometer has a printed scale to the left of the tube in the center that reads from negative forty degrees at the bottom to forty
degrees at the top. It also has a scale printed to the right of the tube that reads from negative thirty degrees at the bottom to thirty
five degrees at the top. On both scales, the volume of the alcohol in the tube reads between nine and ten degrees. The two images
labeled b both depict a metal strip coiled into a spiral and composed of brass and steel. The left coil, which is loosely coiled, is
labeled along its upper edge with the 30 degrees C and 10 degrees C. The end of the coil is near the 30 degrees C label. The right
hand coil is much more tightly wound and the end is near the 10 degree C label.

Figure 5.1.4 : (a) In an alcohol or mercury thermometer, the liquid (dyed red for visibility) expands when heated and contracts
when cooled, much more so than the glass tube that contains the liquid. (b) In a bimetallic thermometer, two different metals (such
as brass and steel) form a two-layered strip. When heated or cooled, one of the metals (brass) expands or contracts more than the
other metal (steel), causing the strip to coil or uncoil. Both types of thermometers have a calibrated scale that indicates the
temperature. (credit a: modification of work by “dwstucke”/Flickr). (c) The demonstration allows one to view the effects of heating
and cooling a coiled bimetallic strip.A bimetallic coil from a thermometer reacts to the heat from a lighter, by uncoiling and then
coiling back up when the lighter is removed. Animation used with permission from Hustvedt (via Wikipedia).
Heat (q) is the transfer of thermal energy between two bodies at different temperatures. Heat flow (a redundant term, but one
commonly used) increases the thermal energy of one body and decreases the thermal energy of the other. Suppose we initially have
a high temperature (and high thermal energy) substance (H) and a low temperature (and low thermal energy) substance (L). The
atoms and molecules in H have a higher average KE than those in L. If we place substance H in contact with substance L, the
thermal energy will flow spontaneously from substance H to substance L. The temperature of substance H will decrease, as will the
average KE of its molecules; the temperature of substance L will increase, along with the average KE of its molecules. Heat flow
will continue until the two substances are at the same temperature (Figure 5.1.5).

Access for free at OpenStax 5.1.3 https://chem.libretexts.org/@go/page/38165


Figure 5.1.5 : (a) Substances H and L are initially at different temperatures, and their atoms have different average kinetic energies.
(b) When they are put into contact with each other, collisions between the molecules result in the transfer of kinetic (thermal)
energy from the hotter to the cooler matter. (c) The two objects reach “thermal equilibrium” when both substances are at the same
temperature, and their molecules have the same average kinetic energy.
Three drawings are shown and labeled a, b, and c, respectively. The first drawing labeled a depicts two boxes, with a space in
between and the pair is captioned “Different temperatures.” The left hand box is labeled H and holds fourteen well-spaced red
spheres with lines drawn around them to indicate rapid motion. The right hand box is labeled L and depicts fourteen blue spheres
that are closer together than the red spheres and have smaller lines around them showing less particle motion. The second drawing
labeled b depicts two boxes that are touching one another. The left box is labeled H and contains fourteen maroon spheres that are
spaced evenly apart. There are tiny lines around each sphere depicting particle movement. The right box is labeled L and holds
fourteen purple spheres that are slightly closer together than the maroon spheres. There are also tiny lines around each sphere
depicting particle movement. A black arrow points from the left box to the right box and the pair of diagrams is captioned
“Contact.” The third drawing labeled c, is labeled “Thermal equilibrium.” There are two boxes shown in contact with one another.
Both boxes contain fourteen purple spheres with small lines around them depicting moderate movement. The left box is labeled H
and the right box is labeled L.

Matter undergoing chemical reactions and physical changes can release or absorb heat. A change that releases heat is called an
exothermic process. For example, the combustion reaction that occurs when using an oxyacetylene torch is an exothermic process
—this process also releases energy in the form of light as evidenced by the torch’s flame (Figure 5.1.6a). A reaction or change that
absorbs heat is an endothermic process. A cold pack used to treat muscle strains provides an example of an endothermic process.
When the substances in the cold pack (water and a salt like ammonium nitrate) are brought together, the resulting process absorbs
heat, leading to the sensation of cold.

Figure 5.1.6 : (a) An oxyacetylene torch produces heat by the combustion of acetylene in oxygen. The energy released by this
exothermic reaction heats and then melts the metal being cut. The sparks are tiny bits of the molten metal flying away. (b) A cold
pack uses an endothermic process to create the sensation of cold. (credit a: modification of work by “Skatebiker”/Wikimedia
commons).
Two pictures are shown and labeled a and b. Picture a shows a metal railroad tie being cut with the flame of an acetylene torch.
Picture b shows a chemical cold pack containing ammonium nitrate.

5.1.3: Measuring Energy and Heat Capacity


Historically, energy was measured in units of calories (cal). A calorie is the amount of energy required to raise one gram of water
by 1 degree C (1 kelvin). However, this quantity depends on the atmospheric pressure and the starting temperature of the water.
The ease of measurement of energy changes in calories has meant that the calorie is still frequently used. The Calorie (with a
capital C), or large calorie, commonly used in quantifying food energy content, is a kilocalorie. The SI unit of heat, work, and
energy is the joule. A joule (J) is defined as the amount of energy used when a force of 1 newton moves an object 1 meter. It is
named in honor of the English physicist James Prescott Joule. One joule is equivalent to 1 kg m2/s2, which is also called 1 newton–
meter. A kilojoule (kJ) is 1000 joules. To standardize its definition, 1 calorie has been set to equal 4.184 joules.
We now introduce two concepts useful in describing heat flow and temperature change. The heat capacity (C) of a body of matter is
the quantity of heat (q) it absorbs or releases when it experiences a temperature change (ΔT) of 1 degree Celsius (or equivalently, 1

Access for free at OpenStax 5.1.4 https://chem.libretexts.org/@go/page/38165


kelvin)
q
C = (5.1.1)
ΔT

Heat capacity is determined by both the type and amount of substance that absorbs or releases heat. It is therefore an extensive
property—its value is proportional to the amount of the substance.
For example, consider the heat capacities of two cast iron frying pans. The heat capacity of the large pan is five times greater than
that of the small pan because, although both are made of the same material, the mass of the large pan is five times greater than the
mass of the small pan. More mass means more atoms are present in the larger pan, so it takes more energy to make all of those
atoms vibrate faster. The heat capacity of the small cast iron frying pan is found by observing that it takes 18,150 J of energy to
raise the temperature of the pan by 50.0 °C
18, 140 J
Csmall pan = = 363 J/°C (5.1.2)
50.0 °C

The larger cast iron frying pan, while made of the same substance, requires 90,700 J of energy to raise its temperature by 50.0 °C.
The larger pan has a (proportionally) larger heat capacity because the larger amount of material requires a (proportionally) larger
amount of energy to yield the same temperature change:
90, 700 J
Clarge pan = = 1814 J/°C (5.1.3)
50.0 °C

The specific heat capacity (c) of a substance, commonly called its “specific heat,” is the quantity of heat required to raise the
temperature of 1 gram of a substance by 1 degree Celsius (or 1 kelvin):
q
c = (5.1.4)
mΔT

Specific heat capacity depends only on the kind of substance absorbing or releasing heat. It is an intensive property—the type, but
not the amount, of the substance is all that matters. For example, the small cast iron frying pan has a mass of 808 g. The specific
heat of iron (the material used to make the pan) is therefore:
18, 140 J
ciron = = 0.449 J/g °C (5.1.5)
(808 g)(50.0 °C)

The large frying pan has a mass of 4040 g. Using the data for this pan, we can also calculate the specific heat of iron:
90, 700 J
ciron = = 0.449 J/g °C (5.1.6)
(4, 040 g)(50.0 °C)

Although the large pan is more massive than the small pan, since both are made of the same material, they both yield the same
value for specific heat (for the material of construction, iron). Note that specific heat is measured in units of energy per temperature
per mass and is an intensive property, being derived from a ratio of two extensive properties (heat and mass). The molar heat
capacity, also an intensive property, is the heat capacity per mole of a particular substance and has units of J/mol °C (Figure 5.1.7).

Figure 5.1.7 : Due to its larger mass, a large frying pan has a larger heat capacity than a small frying pan. Because they are made of
the same material, both frying pans have the same specific heat. (credit: Mark Blaser).
The picture shows two black metal frying pans sitting on a flat surface. The left pan is about half the size of the right pan.

Access for free at OpenStax 5.1.5 https://chem.libretexts.org/@go/page/38165


Liquid water has a relatively high specific heat (about 4.2 J/g °C); most metals have much lower specific heats (usually less than 1
J/g °C). The specific heat of a substance varies somewhat with temperature. However, this variation is usually small enough that we
will treat specific heat as constant over the range of temperatures that will be considered in this chapter. Specific heats of some
common substances are listed in Table 5.1.1.
Table 5.1.1 : Specific Heats of Common Substances at 25 °C and 1 bar
Substance Symbol (state) Specific Heat (J/g °C)

helium He(g) 5.193

water H2O(l) 4.184

ethanol C2H6O(l) 2.376

ice H2O(s) 2.093 (at −10 °C)

water vapor H2O(g) 1.864

nitrogen N2(g) 1.040

air 1.007

oxygen O2(g) 0.918

aluminum Al(s) 0.897

carbon dioxide CO2(g) 0.853

argon Ar(g) 0.522

iron Fe(s) 0.449

copper Cu(s) 0.385

lead Pb(s) 0.130

gold Au(s) 0.129

silicon Si(s) 0.712

If we know the mass of a substance and its specific heat, we can determine the amount of heat, q, entering or leaving the substance
by measuring the temperature change before and after the heat is gained or lost:
q = (specific heat) × (mass of substance) × (temperature change)

q = c × m × ΔT

= c × m × (Tfinal − Tinitial)

In this equation, c is the specific heat of the substance, m is its mass, and ΔT (which is read “delta T”) is the temperature change,
Tfinal − Tinitial. If a substance gains thermal energy, its temperature increases, its final temperature is higher than its initial
temperature, Tfinal − Tinitial has a positive value, and the value of q is positive. If a substance loses thermal energy, its temperature
decreases, the final temperature is lower than the initial temperature, Tfinal − Tinitial has a negative value, and the value of q is
negative.

 Example 5.1.1: Measuring Heat

A flask containing 8.0 × 10 2


g of water is heated, and the temperature of the water increases from 21 °C to 85 °C. How much
heat did the water absorb?

Solution
To answer this question, consider these factors:
the specific heat of the substance being heated (in this case, water)

Access for free at OpenStax 5.1.6 https://chem.libretexts.org/@go/page/38165


the amount of substance being heated (in this case, 800 g)
the magnitude of the temperature change (in this case, from 21 °C to 85 °C).
The specific heat of water is 4.184 J/g °C, so to heat 1 g of water by 1 °C requires 4.184 J. We note that since 4.184 J is
required to heat 1 g of water by 1 °C, we will need 800 times as much to heat 800 g of water by 1 °C. Finally, we observe that
since 4.184 J are required to heat 1 g of water by 1 °C, we will need 64 times as much to heat it by 64 °C (that is, from 21 °C to
85 °C).
This can be summarized using the equation:
q = c × m × ΔT

= c × m × (Tfinal − Tinitial)

= (4.184 J/ g °C) × (800 g ) × (85 − 21)°C

= (4.184 J/ g ° C ) × (800 g ) × (64)° C

= 210, 000 J(= 210 kJ)

Because the temperature increased, the water absorbed heat and q is positive.

 Exercise 5.1.1

How much heat, in joules, must be added to a 5.00 × 10 2


g iron skillet to increase its temperature from 25 °C to 250 °C? The
specific heat of iron is 0.451 J/g °C.
Answer
4
5.05 × 10 J

Note that the relationship between heat, specific heat, mass, and temperature change can be used to determine any of these
quantities (not just heat) if the other three are known or can be deduced.

 Example 5.1.2: Determining Other Quantities

A piece of unknown metal weighs 348 g. When the metal piece absorbs 6.64 kJ of heat, its temperature increases from 22.4 °C
to 43.6 °C. Determine the specific heat of this metal (which might provide a clue to its identity).

Solution
Since mass, heat, and temperature change are known for this metal, we can determine its specific heat using the relationship:
q = c × m × ΔT

= c × m × (Tfinal − Tinitial)

Substituting the known values:

6, 640 J = c × (348 g) × (43.6 − 22.4) °C

Solving:
6, 640 J
c = = 0.900 J/g °C
(348 g) × (21.2°C)

Comparing this value with the values in Table 5.1.1, this value matches the specific heat of aluminum, which suggests that the
unknown metal may be aluminum.

Access for free at OpenStax 5.1.7 https://chem.libretexts.org/@go/page/38165


 Exercise 5.1.2
A piece of unknown metal weighs 217 g. When the metal piece absorbs 1.43 kJ of heat, its temperature increases from 24.5 °C
to 39.1 °C. Determine the specific heat of this metal, and predict its identity.

Answer
c = 0.45 J/g °C ; the metal is likely to be iron from checking Table 5.1.1.

5.1.4: Solar Thermal Energy Power Plants


The sunlight that reaches the earth contains thousands of times more energy than we presently capture. Solar thermal systems
provide one possible solution to the problem of converting energy from the sun into energy we can use. Large-scale solar thermal
plants have different design specifics, but all concentrate sunlight to heat some substance; the heat “stored” in that substance is then
converted into electricity.
The Solana Generating Station in Arizona’s Sonora Desert produces 280 megawatts of electrical power. It uses parabolic mirrors
that focus sunlight on pipes filled with a heat transfer fluid (HTF) (Figure 5.1.8). The HTF then does two things: It turns water into
steam, which spins turbines, which in turn produces electricity, and it melts and heats a mixture of salts, which functions as a
thermal energy storage system. After the sun goes down, the molten salt mixture can then release enough of its stored heat to
produce steam to run the turbines for 6 hours. Molten salts are used because they possess a number of beneficial properties,
including high heat capacities and thermal conductivities.

Figure 5.1.8 : This solar thermal plant uses parabolic trough mirrors to concentrate sunlight. (credit a: modification of work by
Bureau of Land Management)
This figure has two parts labeled a and b. Part a shows rows and rows of trough mirrors. Part b shows how a solar thermal plant
works. Heat transfer fluid enters a tank via pipes. The tank contains water which is heated. As the heat is exchanged from the pipes
to the water, the water becomes steam. The steam travels to a steam turbine. The steam turbine begins to turn which powers a
generator. Exhaust steam exits the steam turbine and enters a cooling tower.
The 377-megawatt Ivanpah Solar Generating System, located in the Mojave Desert in California, is the largest solar thermal power
plant in the world (Figure 5.1.9). Its 170,000 mirrors focus huge amounts of sunlight on three water-filled towers, producing steam
at over 538 °C that drives electricity-producing turbines. It produces enough energy to power 140,000 homes. Water is used as the
working fluid because of its large heat capacity and heat of vaporization.

Access for free at OpenStax 5.1.8 https://chem.libretexts.org/@go/page/38165


Figure 5.1.9 : (a) The Ivanpah solar thermal plant uses 170,000 mirrors to concentrate sunlight on water-filled towers. (b) It covers
4000 acres of public land near the Mojave Desert and the California-Nevada border. (credit a: modification of work by Craig
Dietrich; credit b: modification of work by “USFWS Pacific Southwest Region”/Flickr)
Two pictures are shown and labeled a and b. Picture a shows a thermal plant with three tall metal towers. Picture b is an arial
picture of the mirrors used at the plant. They are arranged in rows.

Summary
Energy is the capacity to do work (applying a force to move matter). Kinetic energy (KE) is the energy of motion; potential energy
is energy due to relative position, composition, or condition. When energy is converted from one form into another, energy is
neither created nor destroyed (law of conservation of energy or first law of thermodynamics). Matter has thermal energy due to the
KE of its molecules and temperature that corresponds to the average KE of its molecules. Heat is energy that is transferred between
objects at different temperatures; it flows from a high to a low temperature. Chemical and physical processes can absorb heat
(endothermic) or release heat (exothermic). The SI unit of energy, heat, and work is the joule (J). Specific heat and heat capacity
are measures of the energy needed to change the temperature of a substance or object. The amount of heat absorbed or released by
a substance depends directly on the type of substance, its mass, and the temperature change it undergoes.

5.1.5: Key Equations


q = c × m × ΔT = c × m × (Tfinal − Tinitial)

Glossary
calorie (cal)
unit of heat or other energy; the amount of energy required to raise 1 gram of water by 1 degree Celsius; 1 cal is defined as
4.184 J

endothermic process
chemical reaction or physical change that absorbs heat

energy
capacity to supply heat or do work

exothermic process
chemical reaction or physical change that releases heat

heat (q)
transfer of thermal energy between two bodies

heat capacity (C)


extensive property of a body of matter that represents the quantity of heat required to increase its temperature by 1 degree
Celsius (or 1 kelvin)

joule (J)
SI unit of energy; 1 joule is the kinetic energy of an object with a mass of 2 kilograms moving with a velocity of 1 meter per
second, 1 J = 1 kg m2/s and 4.184 J = 1 cal

Access for free at OpenStax 5.1.9 https://chem.libretexts.org/@go/page/38165


kinetic energy
1
energy of a moving body, in joules, equal to 2
mv (where m = mass and v = velocity)
2

potential energy
energy of a particle or system of particles derived from relative position, composition, or condition

specific heat capacity (c)


intensive property of a substance that represents the quantity of heat required to raise the temperature of 1 gram of the substance
by 1 degree Celsius (or 1 kelvin)

temperature
intensive property of matter that is a quantitative measure of “hotness” and “coldness”

thermal energy
kinetic energy associated with the random motion of atoms and molecules

thermochemistry
study of measuring the amount of heat absorbed or released during a chemical reaction or a physical change

work (w)
energy transfer due to changes in external, macroscopic variables such as pressure and volume; or causing matter to move
against an opposing force

This page titled 5.1: Energy Basics is shared under a CC BY 4.0 license and was authored, remixed, and/or curated by OpenStax via source
content that was edited to the style and standards of the LibreTexts platform; a detailed edit history is available upon request.

Access for free at OpenStax 5.1.10 https://chem.libretexts.org/@go/page/38165


5.2: Calorimetry
 Learning Objectives
Explain the technique of calorimetry
Calculate and interpret heat and related properties using typical calorimetry data

One technique we can use to measure the amount of heat involved in a chemical or physical process is known as calorimetry. Calorimetry is used
to measure amounts of heat transferred to or from a substance. To do so, the heat is exchanged with a calibrated object (calorimeter). The change
in temperature of the measuring part of the calorimeter is converted into the amount of heat (since the previous calibration was used to establish
its heat capacity). The measurement of heat transfer using this approach requires the definition of a system (the substance or substances
undergoing the chemical or physical change) and its surroundings (the other components of the measurement apparatus that serve to either
provide heat to the system or absorb heat from the system). Knowledge of the heat capacity of the surroundings, and careful measurements of the
masses of the system and surroundings and their temperatures before and after the process allows one to calculate the heat transferred as
described in this section.
A calorimeter is a device used to measure the amount of heat involved in a chemical or physical process. For example, when an exothermic
reaction occurs in solution in a calorimeter, the heat produced by the reaction is absorbed by the solution, which increases its temperature. When
an endothermic reaction occurs, the heat required is absorbed from the thermal energy of the solution, which decreases its temperature (Figure
5.2.1). The temperature change, along with the specific heat and mass of the solution, can then be used to calculate the amount of heat involved

in either case.

Figure 5.2.1 : In a calorimetric determination, either (a) an exothermic process occurs and heat, q, is negative, indicating that thermal energy is
transferred from the system to its surroundings, or (b) an endothermic process occurs and heat, q, is positive, indicating that thermal energy is
transferred from the surroundings to the system.
Two diagrams labeled a and b are shown. Each is made up of two rectangular containers with a thermometer inserted into the top right and
extending inside. There is a right facing arrow connecting each box in each diagram. The left container in diagram a depicts a pink and green
swirling solution with the terms “Exothermic process” and “System” written in the center with arrows facing away from the terms pointing to
“q.” The labels “Solution” and “Surroundings” are written at the bottom of the container. The right container in diagram a has the term “Solution”
written at the bottom of the container and a red arrow facing up near the thermometer with the phrase “Temperature increased” next to it. The
pink and green swirls are more blended in this container. The left container in diagram b depicts a purple and blue swirling solution with the
terms “Endothermic process” and “System” written in the center with arrows facing away from the terms and “Solution” and “Surroundings”
written at the bottom. The arrows point away from the letter “q.” The right container in diagram b has the term “Solution” written at the bottom
and a red arrow facing down near the thermometer with the phrase “Temperature decreased” next to it. The blue and purple swirls are more
blended in this container.

By convention, q is given a negative (-) sign when the system releases heat to the surroundings (exothermic); q is given a positive (+) sign
when the system absorbs heat from the surroundings (endothermic).

Scientists use well-insulated calorimeters that all but prevent the transfer of heat between the calorimeter and its environment. This enables the
accurate determination of the heat involved in chemical processes, the energy content of foods, and so on. General chemistry students often use
simple calorimeters constructed from polystyrene cups (Figure 5.2.2). These easy-to-use “coffee cup” calorimeters allow more heat exchange
with their surroundings, and therefore produce less accurate energy values.

Access for free at OpenStax 5.2.1 https://chem.libretexts.org/@go/page/38166


Figure 5.2.2 : A simple calorimeter can be constructed from two polystyrene cups. A thermometer and stirrer extend through the cover into the
reaction mixture.
Two Styrofoam cups are shown nested in one another with a cover over the top. A thermometer and stirring rod are inserted through the cover
and into the solution inside the cup, which is shown as a cut-away. The stirring rod has a double headed arrow next to it facing up and down. The
liquid mixture inside the cup is labeled “Reaction mixture.”
Commercial solution calorimeters are also available. Relatively inexpensive calorimeters often consist of two thin-walled cups that are nested in a
way that minimizes thermal contact during use, along with an insulated cover, handheld stirrer, and simple thermometer. More expensive
calorimeters used for industry and research typically have a well-insulated, fully enclosed reaction vessel, motorized stirring mechanism, and a
more accurate temperature sensor (Figure 5.2.3).

Figure 5.2.3 : Commercial solution calorimeters range from (a) simple, inexpensive models for student use to (b) expensive, more accurate
models for industry and research.
Two diagrams are shown and labeled a and b. Diagram a depicts a thermometer which passes through a disk-like insulating cover and into a metal
cylinder which is labeled “metal inner vessel,” which is in turn nested in a metal cylinder labeled “metal outer vessel.” The inner cylinder rests on
an insulating support ring. A stirrer passes through the insulating cover and into the inner cylinder as well. Diagram b shows an inner metal vessel
half full of liquid resting on an insulating support ring and nested in a metal outer vessel. A precision temperature probe and motorized stirring
rod are placed into the solution in the inner vessel and connected by wires to equipment exterior to the set-up.
Before we practice calorimetry problems involving chemical reactions, consider a simple example that illustrates the core idea behind
calorimetry. Suppose we initially have a high-temperature substance, such as a hot piece of metal (M), and a low-temperature substance, such as
cool water (W). If we place the metal in the water, heat will flow from M to W. The temperature of M will decrease, and the temperature of W
will increase, until the two substances have the same temperature—that is, when they reach thermal equilibrium (Figure 5.2.4). If this occurs in a
calorimeter, ideally all of this heat transfer occurs between the two substances, with no heat gained or lost by either the calorimeter or the
calorimeter’s surroundings. Under these ideal circumstances, the net heat change is zero:

q substance M +q substance W =0 (5.2.1)

This relationship can be rearranged to show that the heat gained by substance M is equal to the heat lost by substance W:

q substance M = −q substance W (5.2.2)

Access for free at OpenStax 5.2.2 https://chem.libretexts.org/@go/page/38166


The magnitude of the heat (change) is therefore the same for both substances, and the negative sign merely shows that q and substance M

q are opposite in direction of heat flow (gain or loss) but does not indicate the arithmetic sign of either q value (that is determined by
substance W

whether the matter in question gains or loses heat, per definition). In the specific situation described, q is a negative value and
substance M

q is positive, since heat is transferred from M to W.


substance W

Figure 5.2.4 : In a simple calorimetry process, (a) heat, q, is transferred from the hot metal, M, to the cool water, W, until (b) both are at the same
temperature.
Two diagrams are shown and labeled a and b. Each diagram is composed of a rectangular container with a thermometer inserted inside from the
top right corner. Both containers are connected by a right-facing arrow. Both containers are full of water, which is depicted by the letter “W,” and
each container has a square in the middle which represents a metal which is labeled with a letter “M.” In diagram a, the metal is drawn in brown
and has three arrows facing away from it. Each arrow has the letter “q” at its end. The metal is labeled “system” and the water is labeled
“surroundings.” The thermometer in this diagram has a relatively low reading. In diagram b, the metal is depicted in purple and the thermometer
has a relatively high reading.

 Example 5.2.1: Heat Transfer between Substances at Different Temperatures

A hot 360-g piece of rebar (a steel rod used for reinforcing concrete) is dropped into 425 mL of water at 24.0 °C. The final temperature of the
water is measured as 42.7 °C. Calculate the initial temperature of the piece of rebar. Assume the specific heat of steel is approximately the
same as that for iron (Table T4), and that all heat transfer occurs between the rebar and the water (there is no heat exchange with the
surroundings).

Solution
The temperature of the water increases from 24.0 °C to 42.7 °C, so the water absorbs heat. That heat came from the piece of rebar, which
initially was at a higher temperature. Assuming that all heat transfer was between the rebar and the water, with no heat “lost” to the
surroundings, then heat given off by rebar = − heat taken in by water, or:

qrebar = −qwater

Since we know how heat is related to other measurable quantities, we have:

(c × m × ΔT )rebar = −(c × m × ΔT )water

Letting f = final and i = initial, in expanded form, this becomes:

crebar × mrebar × (Tf ,rebar − Ti,rebar) = −cwater × mwater × (Tf ,water − Ti,water)

The density of water is 1.0 g/mL, so 425 mL of water = 425 g. Noting that the final temperature of both the rebar and water is 42.7 °C,
substituting known values yields:

(0.449 J/g °C)(360g)(42.7°C − Ti,rebar) = −(4.184 J/g °C)(425 g)(42.7°C − 24.0°C)

(4.184 J/g °C)(425 g)(42.7°C − 24.0°C)


Ti,rebar = + 42.7°C
(0.449 J/g °C)(360 g)

Solving this gives Ti,rebar= 248 °C, so the initial temperature of the rebar was 248 °C.

Access for free at OpenStax 5.2.3 https://chem.libretexts.org/@go/page/38166


 Exercise 5.2.1A
A 248-g piece of copper is dropped into 390 mL of water at 22.6 °C. The final temperature of the water was measured as 39.9 °C. Calculate
the initial temperature of the piece of copper. Assume that all heat transfer occurs between the copper and the water.

Answer
The initial temperature of the copper was 335.6 °C.

 Exercise 5.2.1B

A 248-g piece of copper initially at 314 °C is dropped into 390 mL of water initially at 22.6 °C. Assuming that all heat transfer occurs
between the copper and the water, calculate the final temperature.

Answer
The final temperature (reached by both copper and water) is 38.7 °C.

This method can also be used to determine other quantities, such as the specific heat of an unknown metal.

 Example 5.2.2: Identifying a Metal by Measuring Specific Heat

A 59.7 g piece of metal that had been submerged in boiling water was quickly transferred into 60.0 mL of water initially at 22.0 °C. The final
temperature is 28.5 °C. Use these data to determine the specific heat of the metal. Use this result to identify the metal.

Solution
Assuming perfect heat transfer, heat given off by metal = −heat taken in by water, or:

qmetal = −qwater

In expanded form, this is:

cmetal × mmetal × (Tf ,metal − Ti,metal) = −cwater × mwater × (Tf ,water − Ti,water)

Noting that since the metal was submerged in boiling water, its initial temperature was 100.0 °C; and that for water, 60.0 mL = 60.0 g; we
have:

(cmetal )(59.7 g)(28.5°C − 100.0°C) = −(4.18 J/g °C)(60.0 g)(28.5°C − 22.0°C)

Solving this:
−(4.184 J/g °C)(60.0 g)(6.5°C)
cmetal = = 0.38 J/g °C
(59.7 g)(−71.5°C)

Comparing this with values in Table T4, our experimental specific heat is closest to the value for copper (0.39 J/g °C), so we identify the
metal as copper.

 Exercise 5.2.2

A 92.9-g piece of a silver/gray metal is heated to 178.0 °C, and then quickly transferred into 75.0 mL of water initially at 24.0 °C. After 5
minutes, both the metal and the water have reached the same temperature: 29.7 °C. Determine the specific heat and the identity of the metal.
(Note: You should find that the specific heat is close to that of two different metals. Explain how you can confidently determine the identity
of the metal).

Answer
cmetal = 0.13 J/g °C

This specific heat is close to that of either gold or lead. It would be difficult to determine which metal this was based solely on the
numerical values. However, the observation that the metal is silver/gray in addition to the value for the specific heat indicates that the
metal is lead.

Access for free at OpenStax 5.2.4 https://chem.libretexts.org/@go/page/38166


When we use calorimetry to determine the heat involved in a chemical reaction, the same principles we have been discussing apply. The amount
of heat absorbed by the calorimeter is often small enough that we can neglect it (though not for highly accurate measurements, as discussed later),
and the calorimeter minimizes energy exchange with the surroundings. Because energy is neither created nor destroyed during a chemical
reaction, there is no overall energy change during the reaction. The heat produced or consumed in the reaction (the “system”), qreaction, plus the
heat absorbed or lost by the solution (the “surroundings”), q , must add up to zero:
solution

qreaction + qsolution = 0  (5.2.3)

This means that the amount of heat produced or consumed in the reaction equals the amount of heat absorbed or lost by the solution:
qreaction = −qsolution (5.2.4)

This concept lies at the heart of all calorimetry problems and calculations.

 Example 5.2.3: Heat Produced by an Exothermic Reaction

When 50.0 mL of 0.10 M HCl(aq) and 50.0 mL of 1.00 M NaOH(aq), both at 22.0 °C, are added to a coffee cup calorimeter, the temperature
of the mixture reaches a maximum of 28.9 °C. What is the approximate amount of heat produced by this reaction?

HCl(aq) + NaOH(aq) ⟶ NaCl(aq) + H O(l)


2

Solution
To visualize what is going on, imagine that you could combine the two solutions so quickly that no reaction took place while they mixed;
then after mixing, the reaction took place. At the instant of mixing, you have 100.0 mL of a mixture of HCl and NaOH at 22.0 °C. The HCl
and NaOH then react until the solution temperature reaches 28.9 °C.
The heat given off by the reaction is equal to that taken in by the solution. Therefore:

qreaction = −qsolution

(It is important to remember that this relationship only holds if the calorimeter does not absorb any heat from the reaction, and there is no
heat exchange between the calorimeter and its surroundings.)
Next, we know that the heat absorbed by the solution depends on its specific heat, mass, and temperature change:

qsolution = (c × m × ΔT )solution

To proceed with this calculation, we need to make a few more reasonable assumptions or approximations. Since the solution is aqueous, we
can proceed as if it were water in terms of its specific heat and mass values. The density of water is approximately 1.0 g/mL, so 100.0 mL has
a mass of about 1.0 × 102 g (two significant figures). The specific heat of water is approximately 4.18 J/g °C, so we use that for the specific
heat of the solution. Substituting these values gives:
2 3
qsolution = (4.184 J/g °C)(1.0 × 10 g)(28.9°C − 22.0°C) = 2.89 × 10 J

Finally, since we are trying to find the heat of the reaction, we have:
3
qreaction = −qsolution = −2.89 × 10 J

The negative sign indicates that the reaction is exothermic. It produces 2.89 kJ of heat.

 Exercise 5.2.3

When 100 mL of 0.200 M NaCl(aq) and 100 mL of 0.200 M AgNO3(aq), both at 21.9 °C, are mixed in a coffee cup calorimeter, the
temperature increases to 23.5 °C as solid AgCl forms. How much heat is produced by this precipitation reaction? What assumptions did you
make to determine your value?

Answer
J; assume no heat is absorbed by the calorimeter, no heat is exchanged between the calorimeter and its surroundings, and that
3
1.34 × 10

the specific heat and mass of the solution are the same as those for water

 Thermochemistry of Hand Warmers

When working or playing outdoors on a cold day, you might use a hand warmer to warm your hands (Figure 5.2.5). A common reusable
hand warmer contains a supersaturated solution of NaC2H3O2 (sodium acetate) and a metal disc. Bending the disk creates nucleation sites

Access for free at OpenStax 5.2.5 https://chem.libretexts.org/@go/page/38166


around which the metastable NaC2H3O2 quickly crystallizes (a later chapter on solutions will investigate saturation and supersaturation in
more detail).
The process NaC H O (aq) ⟶ NaC H O (s) is exothermic, and the heat produced by this process is absorbed by your hands, thereby
2 3 2 2 3 2

warming them (at least for a while). If the hand warmer is reheated, the NaC2H3O2 redissolves and can be reused.

Figure 5.2.5 : Chemical hand warmers produce heat that warms your hand on a cold day. In this one, you can see the metal disc that initiates
the exothermic precipitation reaction. (credit: modification of work by Science Buddies TV/YouTube)
A series of three photos is shown. There are two right-facing arrows connecting one photo to the next. The first photo shows a chemical hand
warmer. It is a bag that contains a clear, colorless liquid. There is a white disk located to the right inside the bag. The second photo shows the
same thing, except the white disc has become a white, cloudy substance. The third photo shows the entire bag filled with this white
substance.
Another common hand warmer produces heat when it is ripped open, exposing iron and water in the hand warmer to oxygen in the air. One
simplified version of this exothermic reaction is
3
2 Fe(s) + O (g) ⟶ Fe O (s).  n
2 2 2 3

Salt in the hand warmer catalyzes the reaction, so it produces heat more rapidly; cellulose, vermiculite, and activated carbon help distribute
the heat evenly. Other types of hand warmers use lighter fluid (a platinum catalyst helps lighter fluid oxidize exothermically), charcoal
(charcoal oxidizes in a special case), or electrical units that produce heat by passing an electrical current from a battery through resistive
wires.

 Example 5.2.4: Heat Flow in an Instant Ice Pack

When solid ammonium nitrate dissolves in water, the solution becomes cold. This is the basis for an “instant ice pack” (Figure 5.2.5). When
3.21 g of solid NH4NO3 dissolves in 50.0 g of water at 24.9 °C in a calorimeter, the temperature decreases to 20.3 °C.
Calculate the value of q for this reaction and explain the meaning of its arithmetic sign. State any assumptions that you made.

Figure 5.2.5 : An instant cold pack consists of a bag containing solid ammonium nitrate and a second bag of water. When the bag of water is
broken, the pack becomes cold because the dissolution of ammonium nitrate is an endothermic process that removes thermal energy from the
water. The cold pack then removes thermal energy from your body.

Solution
We assume that the calorimeter prevents heat transfer between the solution and its external environment (including the calorimeter itself), in
which case:

qrxn = −qsoln

with “rxn” and “soln” used as shorthand for “reaction” and “solution,” respectively.
Assuming also that the specific heat of the solution is the same as that for water, we have:

Access for free at OpenStax 5.2.6 https://chem.libretexts.org/@go/page/38166


qrxn = −qsoln = −(c × m × ΔT )soln

= −[(4.184J/g °C) × (53.2 g) × (20.3°C − 24.9°C)]

= −[(4.184J/g °C) × (53.2 g) × (−4.6°C)]

3
+ 1.0 × 10 J = +1.0 kJ

The positive sign for q indicates that the dissolution is an endothermic process.

 Exercise 5.2.4

When a 3.00-g sample of KCl was added to 3.00 × 102 g of water in a coffee cup calorimeter, the temperature decreased by 1.05 °C. How
much heat is involved in the dissolution of the KCl? What assumptions did you make?

Answer
1.33 kJ; assume that the calorimeter prevents heat transfer between the solution and its external environment (including the calorimeter
itself) and that the specific heat of the solution is the same as that for water.

If the amount of heat absorbed by a calorimeter is too large to neglect or if we require more accurate results, then we must take into account the
heat absorbed both by the solution and by the calorimeter.

Figure 5.2.6 : (a) A bomb calorimeter is used to measure heat produced by reactions involving gaseous reactants or products, such as combustion.
(b) The reactants are contained in the gas-tight “bomb,” which is submerged in water and surrounded by insulating materials. (credit a:
modification of work by “Harbor1”/Wikimedia commons)
A picture and a diagram are shown, labeled a and b, respectively. Picture a depicts a bomb calorimeter. It is a cube-shaped machine with a cavity
in the top, a metal cylinder that is above the cavity, and a read-out panel attached to the top-right side. Diagram b depicts a cut away figure of a
cube with a cylindrical container full of water in the middle of it. Another container, labeled “bomb,” sits inside of a smaller cylinder which holds
a sample cup and is nested in the cylindrical container surrounded by the water. A black line extends into the water and is labeled “Precision
thermometer.” Two wires labeled “Electrodes” extend away from a cover that sits on top of the interior container. A read-out panel is located at
the top right of the cube.
The calorimeters described are designed to operate at constant (atmospheric) pressure and are convenient to measure heat flow accompanying
processes that occur in solution. A different type of calorimeter that operates at constant volume, colloquially known as a bomb calorimeter, is
used to measure the energy produced by reactions that yield large amounts of heat and gaseous products, such as combustion reactions. (The term
“bomb” comes from the observation that these reactions can be vigorous enough to resemble explosions that would damage other calorimeters.)
This type of calorimeter consists of a robust steel container (the “bomb”) that contains the reactants and is itself submerged in water (Figure
5.2.6). The sample is placed in the bomb, which is then filled with oxygen at high pressure. A small electrical spark is used to ignite the sample.

The energy produced by the reaction is trapped in the steel bomb and the surrounding water. The temperature increase is measured and, along
with the known heat capacity of the calorimeter, is used to calculate the energy produced by the reaction. Bomb calorimeters require calibration
to determine the heat capacity of the calorimeter and ensure accurate results. The calibration is accomplished using a reaction with a known q,
such as a measured quantity of benzoic acid ignited by a spark from a nickel fuse wire that is weighed before and after the reaction. The
temperature change produced by the known reaction is used to determine the heat capacity of the calorimeter. The calibration is generally
performed each time before the calorimeter is used to gather research data.

Access for free at OpenStax 5.2.7 https://chem.libretexts.org/@go/page/38166


Physical Chemistry iBook - Bomb Calorimetry

Video 5.2.1 : Video of view how a bomb calorimeter is prepared for action.

 Example 5.2.5: Bomb Calorimetry

When 3.12 g of glucose, C6H12O6, is burned in a bomb calorimeter, the temperature of the calorimeter increases from 23.8 °C to 35.6 °C.
The calorimeter contains 775 g of water, and the bomb itself has a heat capacity of 893 J/°C. How much heat was produced by the
combustion of the glucose sample?

Solution
The combustion produces heat that is primarily absorbed by the water and the bomb. (The amounts of heat absorbed by the reaction products
and the unreacted excess oxygen are relatively small and dealing with them is beyond the scope of this text. We will neglect them in our
calculations.)
The heat produced by the reaction is absorbed by the water and the bomb:

qrxn = −(qwater + qbomb )

= −[(4.184 J/g °C) × (775 g) × (35.6°C − 23.8°C) + 893 J/°C × (35.6°C − 23.8°C)]

= −(38, 300 J + 10, 500 J)

= −48, 800 J = −48.8 kJ

This reaction released 48.7 kJ of heat when 3.12 g of glucose was burned.

 Exercise 5.2.5
When 0.963 g of benzene, C6H6, is burned in a bomb calorimeter, the temperature of the calorimeter increases by 8.39 °C. The bomb has a
heat capacity of 784 J/°C and is submerged in 925 mL of water. How much heat was produced by the combustion of the glucose sample?
Answer

39.0 kJ

Since the first one was constructed in 1899, 35 calorimeters have been built to measure the heat produced by a living person.1 These whole-body
calorimeters of various designs are large enough to hold an individual human being. More recently, whole-room calorimeters allow for relatively
normal activities to be performed, and these calorimeters generate data that more closely reflect the real world. These calorimeters are used to
measure the metabolism of individuals under different environmental conditions, different dietary regimes, and with different health conditions,
such as diabetes. In humans, metabolism is typically measured in Calories per day. A nutritional calorie (Calorie) is the energy unit used to
quantify the amount of energy derived from the metabolism of foods; one Calorie is equal to 1000 calories (1 kcal), the amount of energy needed
to heat 1 kg of water by 1 °C.

Access for free at OpenStax 5.2.8 https://chem.libretexts.org/@go/page/38166


 Measuring Nutritional Calories
In your day-to-day life, you may be more familiar with energy being given in Calories, or nutritional calories, which are used to quantify the
amount of energy in foods. One calorie (cal) = exactly 4.184 joules, and one Calorie (note the capitalization) = 1000 cal, or 1 kcal. (This is
approximately the amount of energy needed to heat 1 kg of water by 1 °C.)
The macronutrients in food are proteins, carbohydrates, and fats or oils. Proteins provide about 4 Calories per gram, carbohydrates also
provide about 4 Calories per gram, and fats and oils provide about 9 Calories/g. Nutritional labels on food packages show the caloric content
of one serving of the food, as well as the breakdown into Calories from each of the three macronutrients (Figure 5.2.7).

Figure 5.2.7 : (a) Macaroni and cheese contain energy in the form of the macronutrients in the food. (b) The food’s nutritional information is
shown on the package label. In the US, the energy content is given in Calories (per serving); the rest of the world usually uses kilojoules.
(credit a: modification of work by “Rex Roof”/Flickr)
For the example shown in (b), the total energy per 228-g portion is calculated by:
(5 g protein × 4 Calories/g) + (31 g carb × 4 Calories/g) + (12 g f at × 9 Calories/g) = 252 Calories (5.2.5)

So, you can use food labels to count your Calories. But where do the values come from? And how accurate are they? The caloric content of
foods can be determined by using bomb calorimetry; that is, by burning the food and measuring the energy it contains. A sample of food is
weighed, mixed in a blender, freeze-dried, ground into powder, and formed into a pellet. The pellet is burned inside a bomb calorimeter, and
the measured temperature change is converted into energy per gram of food.
Today, the caloric content on food labels is derived using a method called the Atwater system that uses the average caloric content of the
different chemical constituents of food, protein, carbohydrate, and fats. The average amounts are those given in the equation and are derived
from the various results given by bomb calorimetry of whole foods. The carbohydrate amount is discounted a certain amount for the fiber
content, which is indigestible carbohydrate. To determine the energy content of a food, the quantities of carbohydrate, protein, and fat are
each multiplied by the average Calories per gram for each and the products summed to obtain the total energy.

Summary
Calorimetry is used to measure the amount of thermal energy transferred in a chemical or physical process. This requires careful measurement of
the temperature change that occurs during the process and the masses of the system and surroundings. These measured quantities are then used to
compute the amount of heat produced or consumed in the process using known mathematical relations. Calorimeters are designed to minimize
energy exchange between the system being studied and its surroundings. They range from simple coffee cup calorimeters used by introductory
chemistry students to sophisticated bomb calorimeters used to determine the energy content of food.

Footnotes
1. 1 Francis D. Reardon et al. “The Snellen human calorimeter revisited, re-engineered and upgraded: Design and performance characteristics.”
Medical and Biological Engineering and Computing 8 (2006)721–28, The Snellen human calorimeter revisited, re-engineered and upgraded:
design and performance characteristics [link.springer.com].

Glossary
bomb calorimeter
device designed to measure the energy change for processes occurring under conditions of constant volume; commonly used for reactions
involving solid and gaseous reactants or products

Access for free at OpenStax 5.2.9 https://chem.libretexts.org/@go/page/38166


calorimeter
device used to measure the amount of heat absorbed or released in a chemical or physical process

calorimetry
process of measuring the amount of heat involved in a chemical or physical process

nutritional calorie (Calorie)


unit used for quantifying energy provided by digestion of foods, defined as 1000 cal or 1 kcal

surroundings
all matter other than the system being studied

system
portion of matter undergoing a chemical or physical change being studied

This page titled 5.2: Calorimetry is shared under a CC BY 4.0 license and was authored, remixed, and/or curated by OpenStax via source content that was edited
to the style and standards of the LibreTexts platform; a detailed edit history is available upon request.

Access for free at OpenStax 5.2.10 https://chem.libretexts.org/@go/page/38166


5.3: Enthalpy
 Learning Objectives
State the first law of thermodynamics
Define enthalpy and explain its classification as a state function
Write and balance thermochemical equations
Calculate enthalpy changes for various chemical reactions
Explain Hess’s law and use it to compute reaction enthalpies

Thermochemistry is a branch of chemical thermodynamics, the science that deals with the relationships between heat, work, and
other forms of energy in the context of chemical and physical processes. As we concentrate on thermochemistry in this chapter, we
need to consider some widely used concepts of thermodynamics.
Substances act as reservoirs of energy, meaning that energy can be added to them or removed from them. Energy is stored in a
substance when the kinetic energy of its atoms or molecules is raised. The greater kinetic energy may be in the form of increased
translations (travel or straight-line motions), vibrations, or rotations of the atoms or molecules. When thermal energy is lost, the
intensities of these motions decrease and the kinetic energy falls. The total of all possible kinds of energy present in a substance is
called the internal energy (U), sometimes symbolized as E.
As a system undergoes a change, its internal energy can change, and energy can be transferred from the system to the surroundings,
or from the surroundings to the system. Energy is transferred into a system when it absorbs heat (q) from the surroundings or when
the surroundings do work (w) on the system. For example, energy is transferred into room-temperature metal wire if it is immersed
in hot water (the wire absorbs heat from the water), or if you rapidly bend the wire back and forth (the wire becomes warmer
because of the work done on it). Both processes increase the internal energy of the wire, which is reflected in an increase in the
wire’s temperature. Conversely, energy is transferred out of a system when heat is lost from the system, or when the system does
work on the surroundings.
The relationship between internal energy, heat, and work can be represented by the equation:
ΔU = q + w (5.3.1)

as shown in Figure 5.3.1. This is one version of the first law of thermodynamics, and it shows that the internal energy of a system
changes through heat flow into or out of the system (positive q is heat flow in; negative q is heat flow out) or work done on or by
the system. The work, w, is positive if it is done on the system and negative if it is done by the system.

Figure 5.3.1 : The internal energy, U, of a system can be changed by heat flow and work. If heat flows into the system, qin, or work
is done on the system, won, its internal energy increases, ΔU > 0. If heat flows out of the system, qout, or work is done by the
system, wby, its internal energy decreases, ΔU < 0.
A type of work called expansion work (or pressure-volume work) occurs when a system pushes back the surroundings against a
restraining pressure, or when the surroundings compress the system. An example of this occurs during the operation of an internal
combustion engine. The reaction of gasoline and oxygen is exothermic. Some of this energy is given off as heat, and some does
work pushing the piston in the cylinder. The substances involved in the reaction are the system, and the engine and the rest of the

Access for free at OpenStax 5.3.1 https://chem.libretexts.org/@go/page/38167


universe are the surroundings. The system loses energy by both heating and doing work on the surroundings, and its internal energy
decreases. (The engine is able to keep the car moving because this process is repeated many times per second while the engine is
running.) We will consider how to determine the amount of work involved in a chemical or physical change in the chapter on
thermodynamics.
As discussed, the relationship between internal energy, heat, and work can be represented as ΔU = q + w. Internal energy is a type
of quantity known as a state function (or state variable), whereas heat and work are not state functions. The value of a state function
depends only on the state that a system is in, and not on how that state is reached. If a quantity is not a state function, then its value
does depend on how the state is reached. An example of a state function is altitude or elevation. If you stand on the summit of Mt.
Kilimanjaro, you are at an altitude of 5895 m, and it does not matter whether you hiked there or parachuted there. The distance you
traveled to the top of Kilimanjaro, however, is not a state function. You could climb to the summit by a direct route or by a more
roundabout, circuitous path (Figure 5.3.2). The distances traveled would differ (distance is not a state function) but the elevation
reached would be the same (altitude is a state function).

Figure 5.3.2 : Paths X and Y represent two different routes to the summit of Mt. Kilimanjaro. Both have the same change in
elevation (altitude or elevation on a mountain is a state function; it does not depend on path), but they have very different distances
traveled (distance walked is not a state function; it depends on the path). (credit: modification of work by Paul Shaffner)
An aerial photo depicts a view of Mount Kilimanjaro. A straight, green arrow labeled X is drawn from the term “base,” written at
the bottom of the mountain, to the term “Summit,” written at the top of the mountain. Another arrow labeled Y is draw from the
base to the summit alongside the green arrow, but this arrow is pink and has three large S-shaped curves along its length.
Chemists ordinarily use a property known as enthalpy (H ) to describe the thermodynamics of chemical and physical processes.
Enthalpy is defined as the sum of a system’s internal energy (U ) and the mathematical product of its pressure (P ) and volume (V ):
H = U +PV (5.3.2)

Since it is derived from three state functions (U , P , and V ), enthalpy is also a state function. Enthalpy values for specific
substances cannot be measured directly; only enthalpy changes for chemical or physical processes can be determined. For
processes that take place at constant pressure (a common condition for many chemical and physical changes), the enthalpy change (
ΔH ) is:

ΔH = ΔU + P ΔV (5.3.3)

The mathematical product P ΔV represents work (w), namely, expansion or pressure-volume work as noted. By their definitions,
the arithmetic signs of ΔV and w will always be opposite:
P ΔV = −w (5.3.4)

Substituting Equation 5.3.4 and the definition of internal energy (Equation 5.3.1) into Equation 5.3.3 yields:
ΔH = ΔU + P ΔV (5.3.5)

= qp + w − w (5.3.6)

= qp (5.3.7)

where q is the heat of reaction under conditions of constant pressure.


p

And so, if a chemical or physical process is carried out at constant pressure with the only work done caused by expansion or
contraction, then the heat flow (q ) and enthalpy change (ΔH ) for the process are equal.
p

Access for free at OpenStax 5.3.2 https://chem.libretexts.org/@go/page/38167


The heat given off when you operate a Bunsen burner is equal to the enthalpy change of the methane combustion reaction that takes
place, since it occurs at the essentially constant pressure of the atmosphere. On the other hand, the heat produced by a reaction
measured in a bomb calorimeter is not equal to ΔH because the closed, constant-volume metal container prevents expansion work
from occurring. Chemists usually perform experiments under normal atmospheric conditions, at constant external pressure with
q = ΔH , which makes enthalpy the most convenient choice for determining heat.

The following conventions apply when we use ΔH :


1. Chemists use a thermochemical equation to represent the changes in both matter and energy. In a thermochemical equation, the
enthalpy change of a reaction is shown as a ΔH value following the equation for the reaction. This ΔH value indicates the
amount of heat associated with the reaction involving the number of moles of reactants and products as shown in the chemical
equation. For example, consider this equation:
1
H (g) + O (g) ⟶ H O(l) ΔH = −286 kJ (5.3.8)
2 2 2 2

This equation indicates that when 1 mole of hydrogen gas and 12 mole of oxygen gas at some temperature and pressure change
to 1 mole of liquid water at the same temperature and pressure, 286 kJ of heat are released to the surroundings. If the
coefficients of the chemical equation are multiplied by some factor, the enthalpy change must be multiplied by that same factor
(ΔH is an extensive property).
(two-fold increase in amounts)

2 H (g) + O (g) ⟶ 2 H O(l) ΔH = 2 × (−286 kJ) = −572 kJ


2 2 2

(two-fold decrease in amounts)

1 1 1 1
H (g) + O (g) ⟶ H O(l) ΔH = × (−286 kJ) = −143 kJ
2 2 2
2 4 2 2

2. The enthalpy change of a reaction depends on the physical state of the reactants and products of the reaction (whether we have
gases, liquids, solids, or aqueous solutions), so these must be shown. For example, when 1 mole of hydrogen gas and 12 mole
of oxygen gas change to 1 mole of liquid water at the same temperature and pressure, 286 kJ of heat are released. If gaseous
water forms, only 242 kJ of heat are released.
1
H (g) + O (g) ⟶ H O(g) ΔH = −242 kJ (5.3.9)
2 2 2 2

3. A negative value of an enthalpy change, ΔH, indicates an exothermic reaction; a positive value of ΔH indicates an endothermic
reaction. If the direction of a chemical equation is reversed, the arithmetic sign of its ΔH is changed (a process that is
endothermic in one direction is exothermic in the opposite direction).

 Example 5.3.1: Measurement of an Enthalpy Change

When 0.0500 mol of HCl(aq) reacts with 0.0500 mol of NaOH(aq) to form 0.0500 mol of NaCl(aq), 2.9 kJ of heat are
produced. What is ΔH, the enthalpy change, per mole of acid reacting, for the acid-base reaction run under the conditions
described ?

HCl(aq) + NaOH(aq) → NaCl(aq) + H O(l)


2

Solution
For the reaction of 0.0500 mol acid (HCl), q = −2.9 kJ. This ratio
−2.9 kJ

0.0500 mol HCl

can be used as a conversion factor to find the heat produced when 1 mole of HCl reacts:
−2.9 kJ
ΔH = 1 mol HCl × = −58 kJ
0.0500 mol HCl

Access for free at OpenStax 5.3.3 https://chem.libretexts.org/@go/page/38167


The enthalpy change when 1 mole of HCl reacts is −58 kJ. Since that is the number of moles in the chemical equation, we
write the thermochemical equation as:

HCl(aq) + NaOH(aq) ⟶ NaCl(aq) + H O(l) ΔH = −58 kJ


2

 Exercise 5.3.1

When 1.34 g Zn(s) reacts with 60.0 mL of 0.750 M HCl(aq), 3.14 kJ of heat are produced. Determine the enthalpy change per
mole of zinc reacting for the reaction:

Zn(s) + 2 HCl(aq) ⟶ ZnCl(aq) + H2(g)

Answer
ΔH = −153 kJ

Be sure to take both stoichiometry and limiting reactants into account when determining the ΔH for a chemical reaction.

 Example 5.3.2: Another Example of the Measurement of an Enthalpy Change


A gummy bear contains 2.67 g sucrose, C12H22O11. When it reacts with 7.19 g potassium chlorate, KClO3, 43.7 kJ of heat are
produced. Determine the enthalpy change for the reaction

C H O (aq) + 8 KClO (aq) ⟶ 12 CO (g) + 11 H O(l) + 8 KCl(aq)


12 22 11 3 2 2

Solution
1 mol
We have 2.67 g × = 0.00780 mol C12 H22 O11 available, and
342.3 g

1 mol
7.19 g × = 0.0587 mol KCl O3 available.
122.5 g

Since
1 mol C H O
12 22 11
0.0587 mol KCl O3 × = 0.00734 mol C H O
12 22 11
8 mol KClO3

is needed, C12H22O11 is the excess reactant and KClO3 is the limiting reactant.
−43.7 kJ
The reaction uses 8 mol KClO3, and the conversion factor is , so we have
0.0587 mol KClO3
−43.7 kJ
ΔH = 8 mol × = −5960 kJ . The enthalpy change for this reaction is −5960 kJ, and the
0.0587 mol KClO3

thermochemical equation is:

C H O + 8 KClO ⟶ 12 CO + 11 H O + 8 KCl ΔH = −5960 kJ


12 22 11 3 2 2

 Exercise 5.3.2
When 1.42 g of iron reacts with 1.80 g of chlorine, 3.22 g of FeCl 2(s)
and 8.60 kJ of heat is produced. What is the enthalpy
change for the reaction when 1 mole of FeCl (s) is produced? 2

Answer
ΔH = −338 kJ

Access for free at OpenStax 5.3.4 https://chem.libretexts.org/@go/page/38167


Enthalpy changes are typically tabulated for reactions in which both the reactants and products are at the same conditions. A
standard state is a commonly accepted set of conditions used as a reference point for the determination of properties under other
different conditions. For chemists, the IUPAC standard state refers to materials under a pressure of 1 bar and solutions at 1 M, and
does not specify a temperature (it used too). Many thermochemical tables list values with a standard state of 1 atm. Because the
ΔH of a reaction changes very little with such small changes in pressure (1 bar = 0.987 atm), ΔH values (except for the most
precisely measured values) are essentially the same under both sets of standard conditions. We will include a superscripted “o” in
the enthalpy change symbol to designate standard state. Since the usual (but not technically standard) temperature is 298.15 K, we
will use a subscripted “298” to designate this temperature. Thus, the symbol (ΔH ) is used to indicate an enthalpy change for a
298

process occurring under these conditions. (The symbol ΔH is used to indicate an enthalpy change for a reaction occurring under
nonstandard conditions.)
The enthalpy changes for many types of chemical and physical processes are available in the reference literature, including those
for combustion reactions, phase transitions, and formation reactions. As we discuss these quantities, it is important to pay attention
to the extensive nature of enthalpy and enthalpy changes. Since the enthalpy change for a given reaction is proportional to the
amounts of substances involved, it may be reported on that basis (i.e., as the ΔH for specific amounts of reactants). However, we
often find it more useful to divide one extensive property (ΔH) by another (amount of substance), and report a per-amount intensive
value of ΔH, often “normalized” to a per-mole basis. (Note that this is similar to determining the intensive property specific heat
from the extensive property heat capacity, as seen previously.)

5.3.1: Enthalpy of Combustion


Standard enthalpy of combustion (ΔH ) is the enthalpy change when 1 mole of a substance burns (combines vigorously with
C

oxygen) under standard state conditions; it is sometimes called “heat of combustion.” For example, the enthalpy of combustion of
ethanol, −1366.8 kJ/mol, is the amount of heat produced when one mole of ethanol undergoes complete combustion at 25 °C and 1
atmosphere pressure, yielding products also at 25 °C and 1 atm.

C H OH(l) + 3 O (g) ⟶ 2 CO + 3 H O(l) ΔH = −1366.8 kJ (5.3.10)
2 5 2 2 2 298

Enthalpies of combustion for many substances have been measured; a few of these are listed in Table 5.3.1. Many readily available
substances with large enthalpies of combustion are used as fuels, including hydrogen, carbon (as coal or charcoal), and
hydrocarbons (compounds containing only hydrogen and carbon), such as methane, propane, and the major components of
gasoline.
Table 5.3.1 : Standard Molar Enthalpies of Combustion
Enthalpy of Combustion
Substance Combustion Reaction ∘
kJ
ΔHc ( at 25°C)
mol

carbon C(s) + O (g) ⟶ CO (g)


2 2
−393.5

hydrogen H (g) +
2
1

2
O (g) ⟶ H O(l)
2 2
−285.8

magnesium Mg(s) +
1

2
O (g) ⟶ MgO(s)
2
−601.6

sulfur S(s) + O (g) ⟶ SO (g)


2 2
−296.8

carbon monoxide CO(g) +


1

2
O (g) ⟶ CO (g)
2 2
−283.0

methane CH (g) + 2 O (g) ⟶ CO (g) + 2 H O(l)


4 2 2 2
−890.8
5
acetylene C H (g) +
2 2
O (g) ⟶ 2 CO (g) + H O(l)
2 2 2
−1301.1
2

ethanol C H OH(l) + 3 O (g) ⟶ CO (g) + 3 H O(l)


2 5 2 2 2
−1366.8
3
methanol CH OH(l) +
3
O (g) ⟶ CO (g) + 2 H O(l)
2 2 2
−726.1
2

25
isooctane C H
8 18
(l) + O (g) ⟶ 8 CO (g) + 9 H O(l)
2 2 2
−5461
2

Access for free at OpenStax 5.3.5 https://chem.libretexts.org/@go/page/38167


 Example 5.3.3: Using Enthalpy of Combustion

As Figure 5.3.3 suggests, the combustion of gasoline is a highly exothermic process. Let us determine the approximate amount
of heat produced by burning 1.00 L of gasoline, assuming the enthalpy of combustion of gasoline is the same as that of
isooctane, a common component of gasoline. The density of isooctane is 0.692 g/mL.

Figure 5.3.3 : The combustion of gasoline is very exothermic. (credit: modification of work by “AlexEagle”/Flickr)

Solution
Starting with a known amount (1.00 L of isooctane), we can perform conversions between units until we arrive at the desired
amount of heat or energy. The enthalpy of combustion of isooctane provides one of the necessary conversions. Table 5.3.1
gives this value as −5460 kJ per 1 mole of isooctane (C8H18).
Using these data,

1000 mL C H 0.692 g C H 1 mol C H


8 18 8 18 8 18 −5460 kJ
4
1.00 LC H × × × × = −3.31 × 10 kJ
8 18
1 LC H 1 mL C H 114 g C H 1 mol C H
8 18 8 18 8 18 8 18

The combustion of 1.00 L of isooctane produces 33,100 kJ of heat. (This amount of energy is enough to melt 99.2 kg, or about
218 lbs, of ice.)
Note: If you do this calculation one step at a time, you would find:
3
1.00 L C H ⟶ 1.00 × 10 mL C H
8 18 8 18

3
1.00 × 10 mL C H ⟶ 692 g C H
8 18 8 18

692 g C H ⟶ 6.07 mol C H


8 18 8 18

4
692 g C H ⟶ −3.31 × 10 kJ
8 18

 Exercise 5.3.3
How much heat is produced by the combustion of 125 g of acetylene?

Answer
6.25 × 103 kJ

 Emerging Algae-Based Energy Technologies (Biofuels)

As reserves of fossil fuels diminish and become more costly to extract, the search is ongoing for replacement fuel sources for
the future. Among the most promising biofuels are those derived from algae (Figure 5.3.4). The species of algae used are
nontoxic, biodegradable, and among the world’s fastest growing organisms. About 50% of algal weight is oil, which can be
readily converted into fuel such as biodiesel. Algae can yield 26,000 gallons of biofuel per hectare—much more energy per
acre than other crops. Some strains of algae can flourish in brackish water that is not usable for growing other crops. Algae can
produce biodiesel, biogasoline, ethanol, butanol, methane, and even jet fuel.

Access for free at OpenStax 5.3.6 https://chem.libretexts.org/@go/page/38167


Figure 5.3.4 : (a) Tiny algal organisms can be (b) grown in large quantities and eventually (c) turned into a useful fuel such as
biodiesel. (credit a: modification of work by Micah Sittig; credit b: modification of work by Robert Kerton; credit c:
modification of work by John F. Williams)
According to the US Department of Energy, only 39,000 square kilometers (about 0.4% of the land mass of the US or less than
1
of the area used to grow corn) can produce enough algal fuel to replace all the petroleum-based fuel used in the US. The
7
cost of algal fuels is becoming more competitive—for instance, the US Air Force is producing jet fuel from algae at a total cost
of under $5 per gallon. The process used to produce algal fuel is as follows: grow the algae (which use sunlight as their energy
source and CO2 as a raw material); harvest the algae; extract the fuel compounds (or precursor compounds); process as
necessary (e.g., perform a transesterification reaction to make biodiesel); purify; and distribute (Figure 5.3.5).

Figure 5.3.5 : Algae convert sunlight and carbon dioxide into oil that is harvested, extracted, purified, and transformed into a
variety of renewable fuels.

5.3.2: Standard Enthalpy of Formation


A standard enthalpy of formation ΔH is an enthalpy change for a reaction in which exactly 1 mole of a pure substance is formed

f

from free elements in their most stable states under standard state conditions. These values are especially useful for computing or
predicting enthalpy changes for chemical reactions that are impractical or dangerous to carry out, or for processes for which it is
difficult to make measurements. If we have values for the appropriate standard enthalpies of formation, we can determine the
enthalpy change for any reaction, which we will practice in the next section on Hess’s law.
The standard enthalpy of formation of CO2(g) is −393.5 kJ/mol. This is the enthalpy change for the exothermic reaction:
∘ ∘
C(s) + O (g) ⟶ CO (g) ΔH = ΔH = −393.5 kJ (5.3.11)
2 2 f 298

starting with the reactants at a pressure of 1 atm and 25 °C (with the carbon present as graphite, the most stable form of carbon
under these conditions) and ending with one mole of CO2, also at 1 atm and 25 °C. For nitrogen dioxide, NO , ΔH is 33.2 2(g)

f

kJ/mol. This is the enthalpy change for the reaction:


1
∘ ∘
N (g) + O (g) ⟶ NO (g) ΔH = ΔH = +33.2 kJ (5.3.12)
2 2 2 f 298
2

A reaction equation with mole of N2 and 1 mole of O2 is correct in this case because the standard enthalpy of formation always
1

refers to 1 mole of product, NO2(g).


You will find a table of standard enthalpies of formation of many common substances in Tables T1 and T2. These values indicate
that formation reactions range from highly exothermic (such as −2984 kJ/mol for the formation of P4O10) to strongly endothermic
(such as +226.7 kJ/mol for the formation of acetylene, C2H2). By definition, the standard enthalpy of formation of an element in its
most stable form is equal to zero under standard conditions, which is 1 atm for gases and 1 M for solutions.

Access for free at OpenStax 5.3.7 https://chem.libretexts.org/@go/page/38167


 Example 5.3.4: Evaluating an Enthalpy of Formation
Ozone, O3(g), forms from oxygen, O2(g), by an endothermic process. Ultraviolet radiation is the source of the energy that
drives this reaction in the upper atmosphere. Assuming that both the reactants and products of the reaction are in their standard
states, determine the standard enthalpy of formation, ΔH of ozone from the following information:

f


3 O (g) ⟶ 2 O (g) ΔH = +286 kJ
2 3 298

Solution ΔH is the enthalpy change for the formation of one mole of a substance in its standard state from the elements in
f

their standard states. Thus, ΔH for O3(g) is the enthalpy change for the reaction:

f

3
O (g) ⟶ O (g)
2 3
2

286 kJ
For the formation of 2 mol of O3(g), ΔH ∘
298
= +286 kJ . This ratio, ( ), can be used as a conversion factor to find
2 mol O3

the heat produced when 1 mole of O3(g) is formed, which is the enthalpy of formation for O3(g):
286 kJ

ΔH for 1 mole of O (g) = 1 mol O3 × = 143 kJ
3
2 mol O3

Therefore, ΔH f

[ O (g)] = +143 kJ/mol
3
.

 Exercise 5.3.4
Hydrogen gas, H2, reacts explosively with gaseous chlorine, Cl2, to form hydrogen chloride, HCl(g). What is the enthalpy
change for the reaction of 1 mole of H2(g) with 1 mole of Cl2(g) if both the reactants and products are at standard state
conditions? The standard enthalpy of formation of HCl(g) is −92.3 kJ/mol.

Answer
For the reaction

H (g) + Cl (g) ⟶ 2 HCl(g) ΔH = −184.6 kJ
2 2 298

 Example 5.3.5: Writing Reaction Equations for ΔH f


Write the heat of formation reaction equations for:


a. C H OH
2 5 (l)

b. Ca (PO )
3 4 2(s)

Solution
Remembering that ΔH reaction equations are for forming 1 mole of the compound from its constituent elements under

f

standard conditions, we have:


a. 2 C(s, graphite) + 3 H (g) + O (g) ⟶ C H OH(l)
2
1

2 2 2 5

b. 3 Ca(s) + P (s) + 4 O (g) ⟶ Ca (PO ) (s)


1

2 4 2 3 4 2

Note: The standard state of carbon is graphite, and phosphorus exists as P . 4

 Exercise 5.3.5
Write the heat of formation reaction equations for:
a. C H OC
2 5 2
H5(l)

b. Na CO2 3(s)

Access for free at OpenStax 5.3.8 https://chem.libretexts.org/@go/page/38167


Answer a
4 C(s, graphite) + 5 H (g) +
2
1

2
O (g) ⟶ C H OC H (l)
2 2 5 2 5
;
Answer b
3
2 Na(s) + C(s, graphite) + O (g) ⟶ Na CO (s)
2 2 3
2

5.3.3: Hess’s Law


There are two ways to determine the amount of heat involved in a chemical change: measure it experimentally, or calculate it from
other experimentally determined enthalpy changes. Some reactions are difficult, if not impossible, to investigate and make accurate
measurements for experimentally. And even when a reaction is not hard to perform or measure, it is convenient to be able to
determine the heat involved in a reaction without having to perform an experiment.
This type of calculation usually involves the use of Hess’s law, which states: If a process can be written as the sum of several
stepwise processes, the enthalpy change of the total process equals the sum of the enthalpy changes of the various steps. Hess’s law
is valid because enthalpy is a state function: Enthalpy changes depend only on where a chemical process starts and ends, but not on
the path it takes from start to finish. For example, we can think of the reaction of carbon with oxygen to form carbon dioxide as
occurring either directly or by a two-step process. The direct process is written:

C(s) + O2(g) ⟶ CO2(g) ΔH = −394 kJ (5.3.13)
298

In the two-step process, first carbon monoxide is formed:


1

C(s) + O2(g) ⟶ CO(g) ΔH = −111 kJ (5.3.14)
298
2

Then, carbon monoxide reacts further to form carbon dioxide:


1

CO(g) + O (g) ⟶ CO2 (g) ΔH = −283 kJ (5.3.15)
2 298
2

The equation describing the overall reaction is the sum of these two chemical changes:
1
Step 1: C(s) + O (g) ⟶ CO(g)
2
2
1
Step 2: CO(g) + O (g) ⟶ CO (g)
2 2
2
––––––––––––––––––––––––––––––––––––
1 1
Sum: C(s) + O (g) + CO(g) + O (g) ⟶ CO(g) + CO (g)
2 2 2
2 2

Because the CO produced in Step 1 is consumed in Step 2, the net change is:
C(s) + O2(g) ⟶ CO2(g) (5.3.16)

According to Hess’s law, the enthalpy change of the reaction will equal the sum of the enthalpy changes of the steps. We can apply
the data from the experimental enthalpies of combustion in Table 5.3.1 to find the enthalpy change of the entire reaction from its
two steps:
1

C(s) + O (g) ⟶ CO(g) ΔH = −111 kJ
2 298
2
1

CO(g) + O (g) ⟶ CO (g) ΔH = −283 kJ
2 2 298
2
¯
¯¯¯¯¯¯¯¯¯¯¯¯¯¯¯¯¯¯¯¯¯¯¯¯¯¯¯¯¯¯¯¯¯¯¯¯¯¯¯¯¯¯¯¯¯¯¯¯¯¯¯¯¯¯¯¯¯¯¯
¯ ¯
¯¯¯¯¯¯¯¯¯¯¯¯¯¯¯¯¯¯¯¯¯¯¯¯¯¯¯¯¯¯¯¯¯¯¯¯
¯

C(s) + O (g) ⟶ CO (g) ΔH = −394 kJ
2 2 298

The result is shown in Figure 5.3.6. We see that ΔH of the overall reaction is the same whether it occurs in one step or two. This
finding (overall ΔH for the reaction = sum of ΔH values for reaction “steps” in the overall reaction) is true in general for chemical
and physical processes.

Access for free at OpenStax 5.3.9 https://chem.libretexts.org/@go/page/38167


Figure 5.3.6 : The formation of CO2(g) from its elements can be thought of as occurring in two steps, which sum to the overall
reaction, as described by Hess’s law. The horizontal blue lines represent enthalpies. For an exothermic process, the products are at
lower enthalpy than are the reactants.
Before we further practice using Hess’s law, let us recall two important features of ΔH.
1. ΔH is directly proportional to the quantities of reactants or products. For example, the enthalpy change for the reaction forming
1 mole of NO2(g) is +33.2 kJ:
1
N (g) + O (g) ⟶ NO (g) ΔH = +33.2 kJ (5.3.17)
2 2 2
2

When 2 moles of NO2 (twice as much) are formed, the ΔH will be twice as large:
N (g) + 2 O (g) ⟶ 2 NO (g) ΔH = +66.4 kJ (5.3.18)
2 2 2

In general, if we multiply or divide an equation by a number, then the enthalpy change should also be multiplied or divided by
the same number.
2. ΔH for a reaction in one direction is equal in magnitude and opposite in sign to ΔH for the reaction in the reverse direction. For
example, given that:
H (g) + Cl (g) ⟶ 2 HCl(g) ΔH = −184.6 kJ (5.3.19)
2 2

Then, for the “reverse” reaction, the enthalpy change is also “reversed”:
2 HCl(g) ⟶ H (g) + Cl (g) ΔH = +184.6 kJ (5.3.20)
2 2

 Example 5.3.6: Stepwise Calculation of ΔH ∘


f

Using Hess’s Law Determine the enthalpy of formation, ΔH


f

, of FeCl3(s) from the enthalpy changes of the following two-
step process that occurs under standard state conditions:

Fe(s) + Cl (g) ⟶ FeCl (s) ΔH ° = −341.8 kJ


2 2

1
FeCl (s) + Cl (g) ⟶ FeCl (s) ΔH ° = −57.7 kJ
2 2 3
2

Solution
We are trying to find the standard enthalpy of formation of FeCl3(s), which is equal to ΔH° for the reaction:
3

Fe(s) + Cl (g) ⟶ FeCl (s) ΔH =?
2 3 f
2

Looking at the reactions, we see that the reaction for which we want to find ΔH° is the sum of the two reactions with known
ΔH values, so we must sum their ΔHs:

Access for free at OpenStax 5.3.10 https://chem.libretexts.org/@go/page/38167


Fe(s) + Cl (g) ⟶ FeCl (s) ΔH ° = −341.8 kJ
2 2

1
FeCl (s) +Cl (g) ⟶ FeCl (s) ΔH ° = −57.7 kJ
2 2 3
2
––––––––––––––––––––––––––––––––––––––––––––––––––––
1
Fe(s) + Cl (g) ⟶ FeCl (s) ΔH ° = −399.5 kJ
2 3
2

The enthalpy of formation, ΔH , of FeCl3(s) is −399.5 kJ/mol.



f

 Exercise 5.3.6

Calculate ΔH for the process:

N (g) + 2 O (g) ⟶ 2 NO (g)


2 2 2

from the following information:

N (g) + O (g) ⟶ 2 NO(g) ΔH = 180.5 kJ


2 2

1
NO(g) + O (g) ⟶ NO (g) ΔH = −57.06 kJ
2 2
2

Answer
66.4 kJ

Here is a less straightforward example that illustrates the thought process involved in solving many Hess’s law problems. It shows
how we can find many standard enthalpies of formation (and other values of ΔH) if they are difficult to determine experimentally.

 Example 5.3.7: A More Challenging Problem

Using Hess’s Law Chlorine monofluoride can react with fluorine to form chlorine trifluoride:
(i) ClF(g) + F 2
(g) ⟶ ClF (g)
3
ΔH ° = ?

Use the reactions here to determine the ΔH° for reaction (i):
(ii) 2 OF 2
(g) ⟶ O (g) + 2 F (g)
2 2
ΔH

(ii)
= −49.4 kJ

(iii) 2 ClF(g) + O 2
(g) ⟶ Cl O(g) + OF (g)
2 2
ΔH

(iii)
= +205.6 kJ

3
(iv) ClF 3
(g) + O (g) ⟶
2
1

2
Cl O(g) +
2
OF (g)
2
ΔH

(iv)
= +266.7 kJ
2

Solution
Our goal is to manipulate and combine reactions (ii), (iii), and (iv) such that they add up to reaction (i). Going from left to right
in (i), we first see that ClF is needed as a reactant. This can be obtained by multiplying reaction (iii) by , which means that
(g)
1

the ΔH° change is also multiplied by : 1

1 1 1 1
ClF(g) + O (g) ⟶ Cl O(g) + OF (g) ΔH ° = (205.6) = +102.8 kJ
2 2 2
2 2 2 2

Next, we see that F is also needed as a reactant. To get this, reverse and halve reaction (ii), which means that the ΔH° changes
2

sign and is halved:


1
O (g) + F (g) ⟶ OF (g) ΔH ° = +24.7 kJ
2 2 2
2

To get ClF3 as a product, reverse (iv), changing the sign of ΔH°:


1 3
Cl O(g) + OF (g) ⟶ ClF (g) + O (g) ΔH ° = −266.7 kJ
2 2 3 2
2 2

Access for free at OpenStax 5.3.11 https://chem.libretexts.org/@go/page/38167


Now check to make sure that these reactions add up to the reaction we want:
1 1 1
ClF(g) + O (g) ⟶ Cl O(g) + OF (g) ΔH ° = +102.8 kJ
2 2 2
2 2 2
1
O (g) + F (g) ⟶ OF (g) ΔH ° = +24.7 kJ
2 2 2
2
1 3
Cl O(g) + OF (g) ⟶ ClF (g) + O (g) ΔH ° = −266.7 kJ
2 2 3 2
2 2
¯¯¯¯¯¯¯¯¯¯¯¯¯¯¯¯¯¯¯¯¯¯¯¯¯¯¯¯¯¯¯¯¯¯¯¯¯¯¯¯¯¯¯¯¯¯¯¯¯¯¯¯¯¯¯¯¯¯¯¯¯¯¯¯¯¯¯¯¯¯¯¯¯¯¯¯¯¯¯¯¯¯¯¯¯¯
¯ ¯
¯¯¯¯¯¯¯¯¯¯¯¯¯¯¯¯¯¯¯¯¯¯¯¯¯¯¯¯¯¯¯¯¯¯¯¯¯
¯
ClF(g) + F ⟶ ClF (g) ΔH ° = −139.2 kJ
2 3

3
Reactants 1

2
O
2
and 1

2
O
2
cancel out product O2; product 1

2
Cl O
2
cancels reactant 1

2
Cl O
2
; and reactant OF
2
is cancelled by
2
products OF and OF2. This leaves only reactants ClF(g) and F2(g) and product ClF3(g), which are what we want. Since
1

2 2

summing these three modified reactions yields the reaction of interest, summing the three modified ΔH° values will give the
desired ΔH°:
ΔH ° = (+102.8 kJ) + (24.7 kJ) + (−266.7 kJ) = −139.2 kJ

 Exercise 5.3.7

Aluminum chloride can be formed from its elements:


(i) 2 Al(s) + 3 Cl 2
(g) ⟶ 2 AlCl (s)
3
ΔH ° = ?

Use the reactions here to determine the ΔH° for reaction (i):
(ii) HCl(g) ⟶ HCl(aq) ΔH

(ii)
= −74.8 kJ

(iii) H 2
(g) + Cl (g) ⟶ 2 HCl(g)
2
ΔH

(iii)
= −185 kJ

(iv) AlCl 3
(aq) ⟶ AlCl (s)
3
ΔH

(iv)
= +323 kJ/mol

(v) 2 Al(s) + 6 HCl(aq) ⟶ 2 AlCl 3


(aq) + 3 H (g)
2
ΔH

(v)
= −1049 kJ

Answer
−1407 kJ

We also can use Hess’s law to determine the enthalpy change of any reaction if the corresponding enthalpies of formation of the
reactants and products are available. The stepwise reactions we consider are: (i) decompositions of the reactants into their
component elements (for which the enthalpy changes are proportional to the negative of the enthalpies of formation of the
reactants), followed by (ii) re-combinations of the elements to give the products (with the enthalpy changes proportional to the
enthalpies of formation of the products). The standard enthalpy change of the overall reaction is therefore equal to: (ii) the sum of
the standard enthalpies of formation of all the products plus (i) the sum of the negatives of the standard enthalpies of formation of
the reactants. This is usually rearranged slightly to be written as follows, with ∑ representing “the sum of” and n standing for the
stoichiometric coefficients:
∘ ∘ ∘
ΔH = ∑ n × ΔH (products) − ∑ n × ΔH (reactants) (5.3.21)
reaction f f

The following example shows in detail why this equation is valid, and how to use it to calculate the enthalpy change for a reaction
of interest.

 Example 5.3.8: Using Hess’s Law


What is the standard enthalpy change for the reaction:

3 NO (g) + H O(l) ⟶ 2 HNO (aq) + NO(g) ΔH ° = ?


2 2 3

Solution 1: Using the Equation

Access for free at OpenStax 5.3.12 https://chem.libretexts.org/@go/page/38167


Alternatively, we could use the special form of Hess’s law given previously:
∘ ∘ ∘
ΔH = ∑ n × ΔH (products) − ∑ n × ΔH (reactants)
reaction f f

⎡ −207.4 kJ +90.2 kJ ⎤
= 2 mol HNO3 × +1 mol NO (g) ×
⎣ mol HNO3 (aq) mol NO (g) ⎦

⎡ +33.2 kJ −285.8 kJ ⎤
− 3 mol NO2 (g) × +1 mol H2 O (l) ×
⎣ mol NO2 (g) mol H2 O (l) ⎦

= 2(−207.4 kJ) + 1(+90.2 kJ) − 3(+33.2 kJ) − 1(−285.8 kJ)

= −138.4 kJ

Solution 2: Supporting Why the General Equation Is Valid


We can write this reaction as the sum of the decompositions of 3NO2(g) and 1H2O(l) into their constituent elements, and the
formation of 2 HNO3(aq) and 1 NO(g) from their constituent elements. Writing out these reactions, and noting their
relationships to the ΔH values for these compounds (from Tables T1 and T2 ), we have:

f

3

3 NO (g) ⟶ N (g) + 3 O (g) ΔH = −99.6 kJ
2 2 2 1
2

1
∘ ∘
H O(l) ⟶ H (g) + O (g) ΔH = +285.8 kJ [−1 × ΔH (H O)]
2 2 2 2 f 2
2

∘ ∘
H (g) + N (g) + 3 O (g) ⟶ 2 HNO (aq) ΔH = −414.8 kJ [2 × ΔH (HNO )]
2 2 2 3 3 f 3

1 1

N (g) + O (g) ⟶ NO(g) ΔH = +90.2 kJ [1 × (NO)]
2 2 4
2 2

Summing these reaction equations gives the reaction we are interested in:

3 NO (g) + H O(l) ⟶ 2 HNO (aq) + NO(g)


2 2 3

Summing their enthalpy changes gives the value we want to determine:


∘ ∘ ∘ ∘ ∘
ΔHrxn = ΔH + ΔH + ΔH + ΔH = (−99.6 kJ) + (+285.8 kJ) + (−414.8 kJ) + (+90.2 kJ)
1 2 3 4

= −138.4 kJ

So the standard enthalpy change for this reaction is ΔH° = −138.4 kJ.
Note that this result was obtained by:
1. multiplying the ΔH of each product by its stoichiometric coefficient and summing those values,

f

2. multiplying the ΔH of each reactant by its stoichiometric coefficient and summing those values, and then

f

3. subtracting the result found in step 2 from the result found in step 1.
This is also the procedure in using the general equation, as shown.

 Exercise 5.3.8

Calculate the heat of combustion of 1 mole of ethanol, C2H5OH(l), when H2O(l) and CO2(g) are formed. Use the following
enthalpies of formation: C2H5OH(l), −278 kJ/mol; H2O(l), −286 kJ/mol; and CO2(g), −394 kJ/mol.

Answer
−1368 kJ/mol

Access for free at OpenStax 5.3.13 https://chem.libretexts.org/@go/page/38167


Summary
If a chemical change is carried out at constant pressure and the only work done is caused by expansion or contraction, q for the
change is called the enthalpy change with the symbol ΔH, or ΔH for reactions occurring under standard state conditions. The

298

value of ΔH for a reaction in one direction is equal in magnitude, but opposite in sign, to ΔH for the reaction in the opposite
direction, and ΔH is directly proportional to the quantity of reactants and products. Examples of enthalpy changes include enthalpy
of combustion, enthalpy of fusion, enthalpy of vaporization, and standard enthalpy of formation. The standard enthalpy of
formation, ΔH , is the enthalpy change accompanying the formation of 1 mole of a substance from the elements in their most

f

stable states at 1 bar (standard state). Many of the processes are carried out at 298.15 K. If the enthalpies of formation are available
for the reactants and products of a reaction, the enthalpy change can be calculated using Hess’s law: If a process can be written as
the sum of several stepwise processes, the enthalpy change of the total process equals the sum of the enthalpy changes of the
various steps.

5.3.4: Key Equations


ΔU = q + w
∘ ∘ ∘
ΔH = ∑ n × ΔH (products) − ∑ n × ΔH (reactants)
reaction f f

Footnotes
1. 1 For more on algal fuel, see www.theguardian.com/environme...n-fuel-problem.

Glossary
chemical thermodynamics
area of science that deals with the relationships between heat, work, and all forms of energy associated with chemical and
physical processes

enthalpy (H)
sum of a system’s internal energy and the mathematical product of its pressure and volume

enthalpy change (ΔH)


heat released or absorbed by a system under constant pressure during a chemical or physical process

expansion work (pressure-volume work)


work done as a system expands or contracts against external pressure

first law of thermodynamics


internal energy of a system changes due to heat flow in or out of the system or work done on or by the system

Hess’s law
if a process can be represented as the sum of several steps, the enthalpy change of the process equals the sum of the enthalpy
changes of the steps

hydrocarbon
compound composed only of hydrogen and carbon; the major component of fossil fuels

internal energy (U)


total of all possible kinds of energy present in a substance or substances

standard enthalpy of combustion (ΔH ) ∘


c

heat released when one mole of a compound undergoes complete combustion under standard conditions

standard enthalpy of formation (ΔH ) ∘


f

enthalpy change of a chemical reaction in which 1 mole of a pure substance is formed from its elements in their most stable
states under standard state conditions

Access for free at OpenStax 5.3.14 https://chem.libretexts.org/@go/page/38167


standard state
set of physical conditions as accepted as common reference conditions for reporting thermodynamic properties; 1 bar of
pressure, and solutions at 1 molar concentrations, usually at a temperature of 298.15 K
state function
property depending only on the state of a system, and not the path taken to reach that state

This page titled 5.3: Enthalpy is shared under a CC BY 4.0 license and was authored, remixed, and/or curated by OpenStax via source content that
was edited to the style and standards of the LibreTexts platform; a detailed edit history is available upon request.

Access for free at OpenStax 5.3.15 https://chem.libretexts.org/@go/page/38167


5.E: Thermochemistry (Exercises)
5.1: Energy Basics
Q5.1.1
A burning match and a bonfire may have the same temperature, yet you would not sit around a burning match on a fall evening to
stay warm. Why not?

S5.1.1
The temperature of 1 gram of burning wood is approximately the same for both a match and a bonfire. This is an intensive property
and depends on the material (wood). However, the overall amount of produced heat depends on the amount of material; this is an
extensive property. The amount of wood in a bonfire is much greater than that in a match; the total amount of produced heat is also
much greater, which is why we can sit around a bonfire to stay warm, but a match would not provide enough heat to keep us from
getting cold.

Q5.1.2
Prepare a table identifying several energy transitions that take place during the typical operation of an automobile.

Q5.1.3
Explain the difference between heat capacity and specific heat of a substance.

S5.1.3
Heat capacity refers to the heat required to raise the temperature of the mass of the substance 1 degree; specific heat refers to the
heat required to raise the temperature of 1 gram of the substance 1 degree. Thus, heat capacity is an extensive property, and specific
heat is an intensive one.

Q5.1.4
Calculate the heat capacity, in joules and in calories per degree, of the following:
a. 28.4 g of water
b. 1.00 oz of lead

Q5.1.5
Calculate the heat capacity, in joules and in calories per degree, of the following:
a. 45.8 g of nitrogen gas
b. 1.00 pound of aluminum metal

S5.1.5
a. 47.6 J/°C; 11.38 cal °C−1;
b. 407 J/°C; 97.3 cal °C−1

Q5.1.6
How much heat, in joules and in calories, must be added to a 75.0–g iron block with a specific heat of 0.449 J/g °C to increase its
temperature from 25 °C to its melting temperature of 1535 °C?

S5.1.6
q = mC Δ°T

0.449 J
q = (75.0g) × ( ) × (1, 510°K) = 50, 800J
g °C

50,800J ; 12,200cal

Access for free at OpenStax 5.E.1 https://chem.libretexts.org/@go/page/42017


Q5.1.7
How much heat, in joules and in calories, is required to heat a 28.4-g (1-oz) ice cube from −23.0 °C to −1.0 °C?

S5.1.7
1310 J; 313 cal

Q5.1.8
How much would the temperature of 275 g of water increase if 36.5 kJ of heat were added?

S5.1.8
ΔT° = 31.7° C

Q5.1.9
If 14.5 kJ of heat were added to 485 g of liquid water, how much would its temperature increase?

S5.1.9
7.15 °C

Q5.1.10
A piece of unknown substance weighs 44.7 g and requires 2110 J to increase its temperature from 23.2 °C to 89.6 °C.
a. What is the specific heat of the substance?
b. If it is one of the substances found in Table, what is its likely identity?

S5.1.10
a.) Solve for the specific heat C and compare the values with the chart
q = mC Δ°T

2110J = (44.7 g)(C )(66.4°C )

2110 J
C =
2970 g °C

0.711 J
C =
g °C

b.) Silicon

Q5.1.11
A piece of unknown solid substance weighs 437.2 g, and requires 8460 J to increase its temperature from 19.3 °C to 68.9 °C.
a. What is the specific heat of the substance?
b. If it is one of the substances found in Table, what is its likely identity?

S5.1.11
a. 0.390 J/g °C;
b. Copper is a likely candidate.

Q5.1.12
An aluminum kettle weighs 1.05 kg.
a. What is the heat capacity of the kettle?
b. How much heat is required to increase the temperature of this kettle from 23.0 °C to 99.0 °C?
c. How much heat is required to heat this kettle from 23.0 °C to 99.0 °C if it contains 1.25 L of water (density of 0.997 g/mL and a
specific heat of 4.184 J/g °C)?

Access for free at OpenStax 5.E.2 https://chem.libretexts.org/@go/page/42017


Q5.1.13
Most people find waterbeds uncomfortable unless the water temperature is maintained at about 85 °F. Unless it is heated, a
waterbed that contains 892 L of water cools from 85 °F to 72 °F in 24 hours. Estimate the amount of electrical energy required over
24 hours, in kWh, to keep the bed from cooling. Note that 1 kilowatt-hour (kWh) = 3.6 × 106 J, and assume that the density of
water is 1.0 g/mL (independent of temperature). What other assumptions did you make? How did they affect your calculated result
(i.e., were they likely to yield “positive” or “negative” errors)?

S5.1.13
We assume that the density of water is 1.0 g/cm3(1 g/mL) and that it takes as much energy to keep the water at 85 °F as to heat it
from 72 °F to 85 °F. We also assume that only the water is going to be heated. Energy required = 7.47 kWh

5.2: Calorimetry
Q5.2.1
A 500-mL bottle of water at room temperature and a 2-L bottle of water at the same temperature were placed in a refrigerator. After
30 minutes, the 500-mL bottle of water had cooled to the temperature of the refrigerator. An hour later, the 2-L of water had cooled
to the same temperature. When asked which sample of water lost the most heat, one student replied that both bottles lost the same
amount of heat because they started at the same temperature and finished at the same temperature. A second student thought that
the 2-L bottle of water lost more heat because there was more water. A third student believed that the 500-mL bottle of water lost
more heat because it cooled more quickly. A fourth student thought that it was not possible to tell because we do not know the
initial temperature and the final temperature of the water. Indicate which of these answers is correct and describe the error in each
of the other answers.

Q5.2.2
Would the amount of heat measured for the reaction in Example be greater, lesser, or remain the same if we used a calorimeter that
was a poorer insulator than a coffee cup calorimeter? Explain your answer.

S5.2.2
lesser; more heat would be lost to the coffee cup and the environment and so ΔT for the water would be lesser and the calculated q
would be lesser

Q5.2.3
Would the amount of heat absorbed by the dissolution in Example appear greater, lesser, or remain the same if the experimenter
used a calorimeter that was a poorer insulator than a coffee cup calorimeter? Explain your answer.

Q5.2.4
Would the amount of heat absorbed by the dissolution in Example appear greater, lesser, or remain the same if the heat capacity of
the calorimeter were taken into account? Explain your answer.

S5.2.4
greater, since taking the calorimeter’s heat capacity into account will compensate for the thermal energy transferred to the solution
from the calorimeter; this approach includes the calorimeter itself, along with the solution, as “surroundings”: qrxn = −(qsolution +
qcalorimeter); since both qsolution and qcalorimeter are negative, including the latter term (qrxn) will yield a greater value for the heat of
the dissolution

Q5.2.5
How many milliliters of water at 23 °C with a density of 1.00 g/mL must be mixed with 180 mL (about 6 oz) of coffee at 95 °C so
that the resulting combination will have a temperature of 60 °C? Assume that coffee and water have the same density and the same
specific heat.

Q5.2.6
How much will the temperature of a cup (180 g) of coffee at 95 °C be reduced when a 45 g silver spoon (specific heat 0.24 J/g °C)
at 25 °C is placed in the coffee and the two are allowed to reach the same temperature? Assume that the coffee has the same density

Access for free at OpenStax 5.E.3 https://chem.libretexts.org/@go/page/42017


and specific heat as water.

S5.2.6
The temperature of the coffee will drop 1 degree.

Q5.2.7
A 45-g aluminum spoon (specific heat 0.88 J/g °C) at 24 °C is placed in 180 mL (180 g) of coffee at 85 °C and the temperature of
the two become equal.
a. What is the final temperature when the two become equal? Assume that coffee has the same specific heat as water.
b. The first time a student solved this problem she got an answer of 88 °C. Explain why this is clearly an incorrect answer.

Q5.2.8
The temperature of the cooling water as it leaves the hot engine of an automobile is 240 °F. After it passes through the radiator it
has a temperature of 175 °F. Calculate the amount of heat transferred from the engine to the surroundings by one gallon of water
with a specific heat of 4.184 J/g °C.

S5.2.8
2
5.7 × 10 kJ

Q5.2.8
A 70.0-g piece of metal at 80.0 °C is placed in 100 g of water at 22.0 °C contained in a calorimeter like that shown in Figure. The
metal and water come to the same temperature at 24.6 °C. How much heat did the metal give up to the water? What is the specific
heat of the metal?

Q5.2.9
If a reaction produces 1.506 kJ of heat, which is trapped in 30.0 g of water initially at 26.5 °C in a calorimeter like that in Figure,
what is the resulting temperature of the water?

S5.2.9
38.5 °C

Q5.2.10
A 0.500-g sample of KCl is added to 50.0 g of water in a calorimeter (Figure). If the temperature decreases by 1.05 °C, what is the
approximate amount of heat involved in the dissolution of the KCl, assuming the heat capacity of the resulting solution is 4.18 J/g
°C? Is the reaction exothermic or endothermic?

Q5.2.11
Dissolving 3.0 g of CaCl2(s) in 150.0 g of water in a calorimeter (Figure) at 22.4 °C causes the temperature to rise to 25.8 °C. What
is the approximate amount of heat involved in the dissolution, assuming the heat capacity of the resulting solution is 4.18 J/g °C? Is
the reaction exothermic or endothermic?

S5.2.11
2.2 kJ; The heat produced shows that the reaction is exothermic.

Q5.2.12
When 50.0 g of 0.200 M NaCl(aq) at 24.1 °C is added to 100.0 g of 0.100 M AgNO3(aq) at 24.1 °C in a calorimeter, the
temperature increases to 25.2 °C as AgCl(s) forms. Assuming the specific heat of the solution and products is 4.20 J/g °C, calculate
the approximate amount of heat in joules produced.

Q5.2.13
The addition of 3.15 g of Ba(OH)2•8H2O to a solution of 1.52 g of NH4SCN in 100 g of water in a calorimeter caused the
temperature to fall by 3.1 °C. Assuming the specific heat of the solution and products is 4.20 J/g °C, calculate the approximate
amount of heat absorbed by the reaction, which can be represented by the following equation:

Access for free at OpenStax 5.E.4 https://chem.libretexts.org/@go/page/42017


Ba(OH )2 ⋅ 8 H2 O(s) + 2N H4 SC N(aq) → Ba(SC N )2(aq) + 2N H3(aq) + 10 H2 O(l) (5.E.1)

S5.2.13
1.4 kJ

Q5.2.14
The reaction of 50 mL of acid and 50 mL of base described in Example increased the temperature of the solution by 6.9 degrees.
How much would the temperature have increased if 100 mL of acid and 100 mL of base had been used in the same calorimeter
starting at the same temperature of 22.0 °C? Explain your answer.

Q5.2.15
If the 3.21 g of NH4NO3 in Example were dissolved in 100.0 g of water under the same conditions, how much would the
temperature change? Explain your answer.

S5.2.15
22.6. Since the mass and the heat capacity of the solution is approximately equal to that of the water, the two-fold increase in the
amount of water leads to a two-fold decrease of the temperature change.

Q5.2.16
When 1.0 g of fructose, C6H12O6(s), a sugar commonly found in fruits, is burned in oxygen in a bomb calorimeter, the temperature
of the calorimeter increases by 1.58 °C. If the heat capacity of the calorimeter and its contents is 9.90 kJ/°C, what is q for this
combustion?

Q5.2.17
When a 0.740-g sample of trinitrotoluene (TNT), C7H5N2O6, is burned in a bomb calorimeter, the temperature increases from 23.4
°C to 26.9 °C. The heat capacity of the calorimeter is 534 J/°C, and it contains 675 mL of water. How much heat was produced by
the combustion of the TNT sample?

S5.2.17
11.7 kJ

Q5.2.18
One method of generating electricity is by burning coal to heat water, which produces steam that drives an electric generator. To
determine the rate at which coal is to be fed into the burner in this type of plant, the heat of combustion per ton of coal must be
determined using a bomb calorimeter. When 1.00 g of coal is burned in a bomb calorimeter, the temperature increases by 1.48 °C.
If the heat capacity of the calorimeter is 21.6 kJ/°C, determine the heat produced by combustion of a ton of coal (2.000 × 103
pounds).

Q5.2.19
The amount of fat recommended for someone with a daily diet of 2000 Calories is 65 g. What percent of the calories in this diet
would be supplied by this amount of fat if the average number of Calories for fat is 9.1 Calories/g?

S5.2.19
30%

Q5.2.20
A teaspoon of the carbohydrate sucrose (common sugar) contains 16 Calories (16 kcal). What is the mass of one teaspoon of
sucrose if the average number of Calories for carbohydrates is 4.1 Calories/g?

Q5.2.21
What is the maximum mass of carbohydrate in a 6-oz serving of diet soda that contains less than 1 Calorie per can if the average
number of Calories for carbohydrates is 4.1 Calories/g?

Access for free at OpenStax 5.E.5 https://chem.libretexts.org/@go/page/42017


S5.2.21
0.24 g

Q5.2.22
A pint of premium ice cream can contain 1100 Calories. What mass of fat, in grams and pounds, must be produced in the body to
store an extra 1.1 × 103 Calories if the average number of Calories for fat is 9.1 Calories/g?

Q5.2.23
A serving of a breakfast cereal contains 3 g of protein, 18 g of carbohydrates, and 6 g of fat. What is the Calorie content of a
serving of this cereal if the average number of Calories for fat is 9.1 Calories/g, for carbohydrates is 4.1 Calories/g, and for protein
is 4.1 Calories/g?

S5.2.23
1.4 × 102 Calories

Q5.2.24
Which is the least expensive source of energy in kilojoules per dollar: a box of breakfast cereal that weighs 32 ounces and costs
$4.23, or a liter of isooctane (density, 0.6919 g/mL) that costs $0.45? Compare the nutritional value of the cereal with the heat
produced by combustion of the isooctane under standard conditions. A 1.0-ounce serving of the cereal provides 130 Calories.

5.3: Enthalpy
Q5.3.1
Explain how the heat measured in [link] differs from the enthalpy change for the exothermic reaction described by the following
equation:

HCl(aq) + NaOH(aq) ⟶ NaCl(aq) + H O(l) (5.E.2)


2

The enthalpy change of the indicated reaction is for exactly 1 mol HCL and 1 mol NaOH; the heat in the example is produced by
0.0500 mol HCl and 0.0500 mol NaOH. Using the data in the check your learning section of [link], calculate ΔH in kJ/mol of
AgNO3(aq) for the reaction:

NaCl(aq) + AgNO (aq) ⟶ AgCl(s) + NaNO (aq) (5.E.3)


3 3

Q5.3.2
Calculate the enthalpy of solution (ΔH for the dissolution) per mole of NH4NO3 under the conditions described in [link].

S5.3.2
25 kJ mol−1

Q5.3.3
Calculate ΔH for the reaction described by the equation.
Ba (OH) ⋅ 8 H O(s) + 2 NH SCN(aq) ⟶ Ba (SCN) (aq) + 2 NH (aq) + 10 H O(l)
2 2 4 2 3 2

Q5.3.4
Calculate the enthalpy of solution (ΔH for the dissolution) per mole of CaCl2.

S5.3.4
81 kJ mol−1

Q5.3.5
Although the gas used in an oxyacetylene torch is essentially pure acetylene, the heat produced by combustion of one mole of
acetylene in such a torch is likely not equal to the enthalpy of combustion of acetylene listed in Table. Considering the conditions
for which the tabulated data are reported, suggest an explanation.

Access for free at OpenStax 5.E.6 https://chem.libretexts.org/@go/page/42017


Q5.3.6
How much heat is produced by burning 4.00 moles of acetylene under standard state conditions?

S5.3.6
5204.4 kJ

Q5.3.7
How much heat is produced by combustion of 125 g of methanol under standard state conditions?

Q5.3.8
How many moles of isooctane must be burned to produce 100 kJ of heat under standard state conditions?

S5.3.8
1.83 × 10−2 mol

Q5.3.9
What mass of carbon monoxide must be burned to produce 175 kJ of heat under standard state conditions?

Q5.3.10
When 2.50 g of methane burns in oxygen, 125 kJ of heat is produced. What is the enthalpy of combustion per mole of methane
under these conditions?

S5.3.10
802 kJ mol−1

Q5.3.11
How much heat is produced when 100 mL of 0.250 M HCl (density, 1.00 g/mL) and 200 mL of 0.150 M NaOH (density, 1.00
g/mL) are mixed?

HCl(aq) + NaOH(aq) ⟶ NaCl(aq) + H O(l) ΔH = −58 kJ (5.E.4)
2 298

If both solutions are at the same temperature and the heat capacity of the products is 4.19 J/g °C, how much will the temperature
increase? What assumption did you make in your calculation?

Q5.3.12
A sample of 0.562 g of carbon is burned in oxygen in a bomb calorimeter, producing carbon dioxide. Assume both the reactants
and products are under standard state conditions, and that the heat released is directly proportional to the enthalpy of combustion of
graphite. The temperature of the calorimeter increases from 26.74 °C to 27.93 °C. What is the heat capacity of the calorimeter and
its contents?

S5.3.12
15.5 kJ/ºC

Q5.3.13
Before the introduction of chlorofluorocarbons, sulfur dioxide (enthalpy of vaporization, 6.00 kcal/mol) was used in household
refrigerators. What mass of SO2 must be evaporated to remove as much heat as evaporation of 1.00 kg of CCl2F2 (enthalpy of
vaporization is 17.4 kJ/mol)?
The vaporization reactions for SO2 and CCl2F2 are
SO (l) ⟶ SO (g)
2 2
and CCl 2
F(l) ⟶ CCl F (g)
2 2
, respectively.

Q5.3.14
Homes may be heated by pumping hot water through radiators. What mass of water will provide the same amount of heat when
cooled from 95.0 to 35.0 °C, as the heat provided when 100 g of steam is cooled from 110 °C to 100 °C.

Access for free at OpenStax 5.E.7 https://chem.libretexts.org/@go/page/42017


S5.3.14
7.43 g

Q5.3.15
Which of the enthalpies of combustion in Table the table are also standard enthalpies of formation?

Q5.3.16
Does the standard enthalpy of formation of H2O(g) differ from ΔH° for the reaction 2 H 2
(g) + O (g) ⟶ 2 H O(g)
2 2
?

S5.3.16
No.

Q5.3.17
Joseph Priestly prepared oxygen in 1774 by heating red mercury(II) oxide with sunlight focused through a lens. How much heat is
required to decompose exactly 1 mole of red HgO(s) to Hg(l) and O2(g) under standard conditions?

Q5.3.18
How many kilojoules of heat will be released when exactly 1 mole of manganese, Mn, is burned to form Mn3O4(s) at standard state
conditions?

S5.3.18
459.6 kJ

Q5.3.19
How many kilojoules of heat will be released when exactly 1 mole of iron, Fe, is burned to form Fe2O3(s) at standard state
conditions?

Q5.3.20
The following sequence of reactions occurs in the commercial production of aqueous nitric acid:
4 NH (g) + 5 O (g) ⟶ 4 NO(g) + 6 H O(l) ΔH = −907 kJ
3 2 2

2 NO(g) + O (g) ⟶ 2 NO (g) ΔH = −113 kJ


2 2

3 NO + H O(l) ⟶ 2 HNO (aq) + NO(g) ΔH = −139 kJ


2 2 2

Determine the total energy change for the production of one mole of aqueous nitric acid by this process.

S5.3.20
495 kJ/mol

Q5.3.21
Both graphite and diamond burn.
C(s, diamond) + O (g) ⟶ CO (g)
2 2

For the conversion of graphite to diamond:



C(s, graphite) ⟶ C(s, diamond) ΔH = 1.90 kJ
298

Which produces more heat, the combustion of graphite or the combustion of diamond?

Q5.3.22
From the molar heats of formation in Appendix G, determine how much heat is required to evaporate one mole of water:
H O(l) ⟶ H O(g)
2 2

S5.3.22
44.01 kJ/mol

Access for free at OpenStax 5.E.8 https://chem.libretexts.org/@go/page/42017


Q5.3.23
Which produces more heat?
Os(s) ⟶ 2 O (g) ⟶ OsO (s)
2 4

or
Os(s) ⟶ 2 O (g) ⟶ OsO (g)
2 4

for the phase change OsO 4


(s) ⟶ OsO (g)
4
ΔH = 56.4 kJ

Q5.3.24
Calculate ΔH ∘
298
for the process
5
Sb(s) + Cl (g) ⟶ SbCl (g)
2 5
2

from the following information:


3

Sb(s) + Cl (g) ⟶ SbCl (g) ΔH = −314 kJ
2 3 298
2


SbCl (s) + Cl (g) ⟶ SbCl (g) ΔH = −80 kJ
3 2 5 298

S5.3.24
394 kJ

Q5.3.25
Calculate ΔH ∘
298
for the process Zn(s) + S(s) + 2 O 2
(g) ⟶ ZnSO (s)
4

from the following information:



Zn(s) + S(s) ⟶ ZnS(s) ΔH = −206.0 kJ
298


ZnS(s) + 2 O (g) ⟶ ZnSO (s) ΔH = −776.8 kJ
2 4 298

Q5.3.26
Calculate ΔH for the process
Hg Cl (s) ⟶ 2 Hg(l) + Cl (g)
2 2 2

from the following information:


Hg(l) + Cl (g) ⟶ HgCl (s) ΔH = −224 kJ
2 2

Hg(l) + HgCl (s) ⟶ Hg Cl (s) ΔH = −41.2 kJ


2 2 2

S5.3.26
265 kJ

Q5.3.27
Calculate ΔH ∘
298
for the process
Co O (s) ⟶ 3 Co(s) + 2 O (g)
3 4 2

from the following information:


1

Co(s) + O (g) ⟶ CoO(s) ΔH = −237.9 kJ
2 298
2


3 Co(s) + O (g) ⟶ Co O (s) ΔH = −177.5 kJ
2 3 4 298

Q5.3.28
Calculate the standard molar enthalpy of formation of NO(g) from the following data:

N (g) + 2 O ⟶ 2 NO (g) ΔH = 66.4 kJ
2 2 2 298

Access for free at OpenStax 5.E.9 https://chem.libretexts.org/@go/page/42017



2 NO(g) + O ⟶ 2 NO (g) ΔH = −114.1 kJ
2 2 298

S5.3.28
90.3 mol−1 of NO

Q5.3.29
Using the data in Appendix G, calculate the standard enthalpy change for each of the following reactions:
a. N (g) + O (g) ⟶ 2 NO(g)
2 2

b. Si(s) + 2 Cl (g) ⟶ SiCl (g)


2 4

c. Fe O (s) + 3 H (g) ⟶ 2 Fe(s) + 3 H O(l)


2 3 2 2

d. 2 LiOH(s) + CO (g) ⟶ Li CO (s) + H O(g) 2 2 3 2

Q5.3.30
Using the data in Appendix G, calculate the standard enthalpy change for each of the following reactions:
a. Si(s) + 2 F (g) ⟶ SiF (g)
2 4

b. 2 C(s) + 2 H (g) + O (g) ⟶ CH CO H(l)


2 2 3 2

c. CH (g) + N (g) ⟶ HCN(g) + NH (g) ;


4 2 3

d. CS (g) + 3 Cl (g) ⟶ CCl (g) + S Cl (g)


2 2 4 2 2

S5.3.30
a. −1615.0 kJ mol−1;
b. −484.3 kJ mol−1;
c. 164.2 kJ;
d. −232.1 kJ

Q5.3.31
The following reactions can be used to prepare samples of metals. Determine the enthalpy change under standard state conditions
for each.
a. 2 Ag O(s) ⟶ 4 Ag(s) + O (g)
2 2

b. SnO(s) + CO(g) ⟶ Sn(s) + CO (g) 2

c. Cr O (s) + 3 H (g) ⟶ 2 Cr(s) + 3 H O(l)


2 3 2 2

d. 2 Al(s) + Fe O (s) ⟶ Al O (s) + 2 Fe(s)


2 3 2 3

Q5.3.32
The decomposition of hydrogen peroxide, H2O2, has been used to provide thrust in the control jets of various space vehicles. Using
the data in Appendix G, determine how much heat is produced by the decomposition of exactly 1 mole of H2O2 under standard
conditions.
2 H O (l) ⟶ 2 H O(g) + O (g)
2 2 2 2

S5.3.32
−54.04 kJ mol−1

Q5.3.33
Calculate the enthalpy of combustion of propane, C3H8(g), for the formation of H2O(g) and CO2(g). The enthalpy of formation of
propane is −104 kJ/mol.

Q5.3.34
Calculate the enthalpy of combustion of butane, C4H10(g) for the formation of H2O(g) and CO2(g). The enthalpy of formation of
butane is −126 kJ/mol.

Access for free at OpenStax 5.E.10 https://chem.libretexts.org/@go/page/42017


S5.3.34
2660 kJ mol−1

Q5.3.35
Both propane and butane are used as gaseous fuels. Which compound produces more heat per gram when burned?

Q5.3.36
The white pigment TiO2 is prepared by the reaction of titanium tetrachloride, TiCl4, with water vapor in the gas phase:
TiCl (g) + 2 H O(g) ⟶ TiO (s) + 4 HCl(g)
4 2 2
.
How much heat is evolved in the production of exactly 1 mole of TiO2(s) under standard state conditions?

S5.3.36
67.1 kJ

Q5.3.37
Water gas, a mixture of H2 and CO, is an important industrial fuel produced by the reaction of steam with red hot coke, essentially
pure carbon:
C(s) + H O(g) ⟶ CO(g) + H (g)
2 2
.
a. Assuming that coke has the same enthalpy of formation as graphite, calculate ΔH for this reaction.∘
298

b. Methanol, a liquid fuel that could possibly replace gasoline, can be prepared from water gas and additional hydrogen at high
temperature and pressure in the presence of a suitable catalyst:
2 H (g) + CO(g) ⟶ CH OH(g). (5.E.5)
2 3

Under the conditions of the reaction, methanol forms as a gas. Calculate ΔH for this reaction and for the condensation of

298

gaseous methanol to liquid methanol.


c. Calculate the heat of combustion of 1 mole of liquid methanol to H2O(g) and CO2(g).

Q5.3.38
In the early days of automobiles, illumination at night was provided by burning acetylene, C2H2. Though no longer used as auto
headlamps, acetylene is still used as a source of light by some cave explorers. The acetylene is (was) prepared in the lamp by the
reaction of water with calcium carbide, CaC2:
CaC (s) + H O(l) ⟶ Ca (OH) (s) + C H (g)
2 2 2 2 2
.
Calculate the standard enthalpy of the reaction. The ΔH of CaC2 is −15.14 kcal/mol.
f

S5.3.38
122.8 kJ

Q5.3.39
From the data in Table, determine which of the following fuels produces the greatest amount of heat per gram when burned under
standard conditions: CO(g), CH4(g), or C2H2(g).

Q5.3.40
The enthalpy of combustion of hard coal averages −35 kJ/g, that of gasoline, 1.28 × 105 kJ/gal. How many kilograms of hard coal
provide the same amount of heat as is available from 1.0 gallon of gasoline? Assume that the density of gasoline is 0.692 g/mL (the
same as the density of isooctane).

S5.3.40
3.7 kg

Access for free at OpenStax 5.E.11 https://chem.libretexts.org/@go/page/42017


Q5.3.41
Ethanol, C2H5OH, is used as a fuel for motor vehicles, particularly in Brazil.
a. Write the balanced equation for the combustion of ethanol to CO2(g) and H2O(g), and, using the data in Appendix G, calculate
the enthalpy of combustion of 1 mole of ethanol.
b. The density of ethanol is 0.7893 g/mL. Calculate the enthalpy of combustion of exactly 1 L of ethanol.
c. Assuming that an automobile’s mileage is directly proportional to the heat of combustion of the fuel, calculate how much
farther an automobile could be expected to travel on 1 L of gasoline than on 1 L of ethanol. Assume that gasoline has the heat
of combustion and the density of n–octane, C8H18 (ΔH = −208.4 kJ/mol; density = 0.7025 g/mL).

f

Q5.3.42
Among the substances that react with oxygen and that have been considered as potential rocket fuels are diborane [B2H6, produces
B2O3(s) and H2O(g)], methane [CH4, produces CO2(g) and H2O(g)], and hydrazine [N2H4, produces N2(g) and H2O(g)]. On the
basis of the heat released by 1.00 g of each substance in its reaction with oxygen, which of these compounds offers the best
possibility as a rocket fuel? The ΔH of B2H6(g), CH4(g), and N2H4(l) may be found in Appendix G.
f

S5.3.42
On the assumption that the best rocket fuel is the one that gives off the most heat, B2H6 is the prime candidate.

Q5.3.43
How much heat is produced when 1.25 g of chromium metal reacts with oxygen gas under standard conditions?
Ethylene, C2H2, a byproduct from the fractional distillation of petroleum, is fourth among the 50 chemical compounds produced
commercially in the largest quantities. About 80% of synthetic ethanol is manufactured from ethylene by its reaction with water in
the presence of a suitable catalyst.
C H (g) + H O(g) ⟶ C H OH(l)
2 4 2 2 5

Using the data in the table in Appendix G, calculate ΔH° for the reaction.

S5.3.43
88.2 kJ

Q5.3.44
The oxidation of the sugar glucose, C6H12O6, is described by the following equation:
C H O (s) + 6 O (g) ⟶ 6 CO (g) + 6 H O(l) ΔH = −2816 kJ
6 12 6 2 2 2

The metabolism of glucose gives the same products, although the glucose reacts with oxygen in a series of steps in the body.
a. How much heat in kilojoules can be produced by the metabolism of 1.0 g of glucose?
b. How many Calories can be produced by the metabolism of 1.0 g of glucose?

Q5.3.45
Propane, C3H8, is a hydrocarbon that is commonly used as a fuel.
a. Write a balanced equation for the complete combustion of propane gas.
b. Calculate the volume of air at 25 °C and 1.00 atmosphere that is needed to completely combust 25.0 grams of propane. Assume
that air is 21.0 percent O2 by volume. (Hint: we will see how to do this calculation in a later chapter on gases—for now use the
information that 1.00 L of air at 25 °C and 1.00 atm contains 0.275 g of O2 per liter.)
c. The heat of combustion of propane is −2,219.2 kJ/mol. Calculate the heat of formation, ΔH of propane given that ΔH of

f f

H2O(l) = −285.8 kJ/mol and ΔH of CO2(g) = −393.5 kJ/mol.



f

d. Assuming that all of the heat released in burning 25.0 grams of propane is transferred to 4.00 kilograms of water, calculate the
increase in temperature of the water.

S5.3.45
a. C H (g) + 5 O
3 8 2
(g) ⟶ 3 CO (g) + 4 H O(l)
2 2
;
b. 330 L;

Access for free at OpenStax 5.E.12 https://chem.libretexts.org/@go/page/42017


c. −104.5 kJ mol−1;
d. 75.4 °C

Q5.3.46
During a recent winter month in Sheboygan, Wisconsin, it was necessary to obtain 3500 kWh of heat provided by a natural gas
furnace with 89% efficiency to keep a small house warm (the efficiency of a gas furnace is the percent of the heat produced by
combustion that is transferred into the house).
a. Assume that natural gas is pure methane and determine the volume of natural gas in cubic feet that was required to heat the
house. The average temperature of the natural gas was 56 °F; at this temperature and a pressure of 1 atm, natural gas has a
density of 0.681 g/L.
b. How many gallons of LPG (liquefied petroleum gas) would be required to replace the natural gas used? Assume the LPG is
liquid propane [C3H8: density, 0.5318 g/mL; enthalpy of combustion, 2219 kJ/mol for the formation of CO2(g) and H2O(l)] and
the furnace used to burn the LPG has the same efficiency as the gas furnace.
c. What mass of carbon dioxide is produced by combustion of the methane used to heat the house?
d. What mass of water is produced by combustion of the methane used to heat the house?
e. What volume of air is required to provide the oxygen for the combustion of the methane used to heat the house? Air contains
23% oxygen by mass. The average density of air during the month was 1.22 g/L.
f. How many kilowatt–hours (1 kWh = 3.6 × 106 J) of electricity would be required to provide the heat necessary to heat the
house? Note electricity is 100% efficient in producing heat inside a house.
g. Although electricity is 100% efficient in producing heat inside a house, production and distribution of electricity is not 100%
efficient. The efficiency of production and distribution of electricity produced in a coal-fired power plant is about 40%. A
certain type of coal provides 2.26 kWh per pound upon combustion. What mass of this coal in kilograms will be required to
produce the electrical energy necessary to heat the house if the efficiency of generation and distribution is 40%?

This page titled 5.E: Thermochemistry (Exercises) is shared under a CC BY 4.0 license and was authored, remixed, and/or curated by OpenStax
via source content that was edited to the style and standards of the LibreTexts platform; a detailed edit history is available upon request.

Access for free at OpenStax 5.E.13 https://chem.libretexts.org/@go/page/42017


CHAPTER OVERVIEW
6: Electronic Structure and Periodic Properties

A general chemistry Libretexts Textbook remixed and remastered from


OpenStax's textbook:
General Chemistry
The study of chemistry must at some point extend to the molecular level, for the physical and chemical properties of a substance
are ultimately explained in terms of the structure and bonding of molecules. This module introduces some basic facts and principles
that are needed for a discussion of organic molecules.
6.1: Electromagnetic Energy
6.2: The Bohr Model
6.3: Development of Quantum Theory
6.4: Electronic Structure of Atoms (Electron Configurations)
6.5: Periodic Variations in Element Properties
6.E: Electronic Structure and Periodic Properties (Exercises)

This page titled 6: Electronic Structure and Periodic Properties is shared under a CC BY 4.0 license and was authored, remixed, and/or curated by
OpenStax via source content that was edited to the style and standards of the LibreTexts platform; a detailed edit history is available upon request.

1
6.1: Electromagnetic Energy
 Learning Objectives
Explain the basic behavior of waves, including traveling waves and standing waves
Describe the wave nature of light
Use appropriate equations to calculate related light-wave properties such as period, frequency, wavelength, and energy
Distinguish between line and continuous emission spectra
Describe the particle nature of light

The nature of light has been a subject of inquiry since antiquity. In the seventeenth century, Isaac Newton performed experiments
with lenses and prisms and was able to demonstrate that white light consists of the individual colors of the rainbow combined
together. Newton explained his optics findings in terms of a "corpuscular" view of light, in which light was composed of streams of
extremely tiny particles travelling at high speeds according to Newton's laws of motion. Others in the seventeenth century, such as
Christiaan Huygens, had shown that optical phenomena such as reflection and refraction could be equally well explained in terms
of light as waves travelling at high speed through a medium called "luminiferous aether" that was thought to permeate all space.
Early in the nineteenth century, Thomas Young demonstrated that light passing through narrow, closely spaced slits produced
interference patterns that could not be explained in terms of Newtonian particles but could be easily explained in terms of waves.
Later in the nineteenth century, after James Clerk Maxwell developed his theory of electromagnetic radiation and showed that light
was the visible part of a vast spectrum of electromagnetic waves, the particle view of light became thoroughly discredited. By the
end of the nineteenth century, scientists viewed the physical universe as roughly comprising two separate domains: matter
composed of particles moving according to Newton's laws of motion, and electromagnetic radiation consisting of waves governed
by Maxwell's equations. Today, these domains are referred to as classical mechanics and classical electrodynamics (or classical
electromagnetism). Although there were a few physical phenomena that could not be explained within this framework, scientists at
that time were so confident of the overall soundness of this framework that they viewed these aberrations as puzzling paradoxes
that would ultimately be resolved somehow within this framework. As we shall see, these paradoxes led to a contemporary
framework that intimately connects particles and waves at a fundamental level called wave-particle duality, which has superseded
the classical view.
Visible light and other forms of electromagnetic radiation play important roles in chemistry, since they can be used to infer the
energies of electrons within atoms and molecules. Much of modern technology is based on electromagnetic radiation. For example,
radio waves from a mobile phone, X-rays used by dentists, the energy used to cook food in your microwave, the radiant heat from
red-hot objects, and the light from your television screen are forms of electromagnetic radiation that all exhibit wavelike behavior.

6.1.1: Waves
A wave is an oscillation or periodic movement that can transport energy from one point in space to another. Common examples of
waves are all around us. Shaking the end of a rope transfers energy from your hand to the other end of the rope, dropping a pebble
into a pond causes waves to ripple outward along the water's surface, and the expansion of air that accompanies a lightning strike
generates sound waves (thunder) that can travel outward for several miles. In each of these cases, kinetic energy is transferred
through matter (the rope, water, or air) while the matter remains essentially in place. An insightful example of a wave occurs in
sports stadiums when fans in a narrow region of seats rise simultaneously and stand with their arms raised up for a few seconds
before sitting down again while the fans in neighboring sections likewise stand up and sit down in sequence. While this wave can
quickly encircle a large stadium in a few seconds, none of the fans actually travel with the wave-they all stay in or above their
seats.
Waves need not be restricted to travel through matter. As Maxwell showed, electromagnetic waves consist of an electric field
oscillating in step with a perpendicular magnetic field, both of which are perpendicular to the direction of travel. These waves can
travel through a vacuum at a constant speed of 2.998 × 108 m/s, the speed of light (denoted by c).
All waves, including forms of electromagnetic radiation, are characterized by, a wavelength (denoted by λ, the lowercase Greek
letter lambda), a frequency (denoted by ν, the lowercase Greek letter nu), and an amplitude. As can be seen in Figure 6.1.1, the
wavelength is the distance between two consecutive peaks or troughs in a wave (measured in meters in the SI system).
Electromagnetic waves have wavelengths that fall within an enormous range-wavelengths of kilometers (103 m) to picometers
(10−12 m) have been observed. The frequency is the number of wave cycles that pass a specified point in space in a specified

Access for free at OpenStax 6.1.1 https://chem.libretexts.org/@go/page/38174


amount of time (in the SI system, this is measured in seconds). A cycle corresponds to one complete wavelength. The unit for
frequency, expressed as cycles per second [s−1], is the hertz (Hz). Common multiples of this unit are megahertz, (1 MHz = 1 × 106
Hz) and gigahertz (1 GHz = 1 × 109 Hz). The amplitude corresponds to the magnitude of the wave's displacement and so, in Figure,
this corresponds to one-half the height between the peaks and troughs. The amplitude is related to the intensity of the wave, which
for light is the brightness, and for sound is the loudness.

Figure 6.1.1 : One-dimensional sinusoidal waves show the relationship among wavelength, frequency, and speed. The wave with
the shortest wavelength has the highest frequency. Amplitude is one-half the height of the wave from peak to trough. (CC by 4.0;
OpenStax)
This figure includes 5 one-dimensional sinusoidal waves in two columns. The column on the left includes three waves, and the
column on the right includes two waves. In each column, dashed vertical line segments extend down the left and right sides of the
column. A right pointing arrow extends from the left dashed line to the right dashed line in both columns and is labeled, “Distance
traveled in 1 second.” The waves all begin on the left side at a crest. The wave at the upper left shows 3 peaks to the right of the
starting point. A bracket labeled, “lambda subscript 1,” extends upward from the second and third peaks. Beneath this wave is the
label, “nu subscript 1 equals 4 cycles per second equals 3 hertz.” The wave below has six peaks to the right of the starting point
with a bracket similarly connecting the third and fourth peaks which is labeled, “lambda subscript 2.” Beneath this wave is the
label, “nu subscript 2 equals 8 cycles per second equals 6 hertz” The third wave in the column has twelve peaks to the right of the
starting point with a bracket similarly connecting the seventh and eighth peaks which is labeled, “lambda subscript 3.” Beneath this
wave is the label, “nu subscript 3 equals 12 cycles per second equals 12 hertz.” All waves in this column appear to have the same
vertical distance from peak to trough. In the second column, the two waves are similarly shown, but lack the lambda labels. The top
wave in this column has a greater vertical distance between the peaks and troughs and is labeled, “Higher amplitude.” The wave
beneath it has a lesser distance between the peaks and troughs and is labeled, “Lower amplitude.”
The product of a wave's wavelength (λ) and its frequency (ν), λν, is the speed of the wave. Thus, for electromagnetic radiation in a
vacuum:
8 −1
c = 2.998 × 10 ms = λν (6.1.1)

Wavelength and frequency are inversely proportional: As the wavelength increases, the frequency decreases. The inverse
proportionality is illustrated in Figure 6.1.2. This figure also shows the electromagnetic spectrum, the range of all types of
electromagnetic radiation. Each of the various colors of visible light has specific frequencies and wavelengths associated with
them, and you can see that visible light makes up only a small portion of the electromagnetic spectrum. Because the technologies
developed to work in various parts of the electromagnetic spectrum are different, for reasons of convenience and historical legacies,
different units are typically used for different parts of the spectrum. For example, radio waves are usually specified as frequencies
(typically in units of MHz), while the visible region is usually specified in wavelengths (typically in units of nm or angstroms).

Access for free at OpenStax 6.1.2 https://chem.libretexts.org/@go/page/38174


Figure 6.1.2 : Portions of the electromagnetic spectrum are shown in order of decreasing frequency and increasing wavelength.
Examples of some applications for various wavelengths include positron emission tomography (PET) scans, X-ray imaging, remote
controls, wireless Internet, cellular telephones, and radios. (credit “Cosmic ray": modification of work by NASA; credit “PET
scan": modification of work by the National Institute of Health; credit “X-ray": modification of work by Dr. Jochen Lengerke;
credit “Dental curing": modification of work by the Department of the Navy; credit “Night vision": modification of work by the
Department of the Army; credit “Remote": modification of work by Emilian Robert Vicol; credit “Cell phone": modification of
work by Brett Jordan; credit “Microwave oven": modification of work by Billy Mabray; credit “Ultrasound": modification of work
by Jane Whitney; credit “AM radio": modification of work by Dave Clausen)

 Example 6.1.1: Determining the Frequency and Wavelength of Radiation

A sodium streetlight gives off yellow light that has a wavelength of 589 nm (1 nm = 1 × 10−9 m). What is the frequency of this
light?

Solution
We can rearrange the Equation 6.1.1 to solve for the frequency:
c
ν =
λ

Since c is expressed in meters per second, we must also convert 589 nm to meters.
8 −1 9
2.998 × 10 m s 1 × 10 nm
14 −1
ν =( )( ) = 5.09 × 10 s
589 nm 1 m

 Exercise 6.1.1

One of the frequencies used to transmit and receive cellular telephone signals in the United States is 850 MHz. What is the
wavelength in meters of these radio waves?

Answer
0.353 m = 35.3 cm

6.1.2: Wireless Communication


Many valuable technologies operate in the radio (3 kHz-300 GHz) frequency region of the electromagnetic spectrum. At the low
frequency (low energy, long wavelength) end of this region are AM (amplitude modulation) radio signals (540-2830 kHz) that can
travel long distances. FM (frequency modulation) radio signals are used at higher frequencies (87.5-108.0 MHz). In AM radio, the
information is transmitted by varying the amplitude of the wave (Figure 6.1.5). In FM radio, by contrast, the amplitude is constant
and the instantaneous frequency varies.

Access for free at OpenStax 6.1.3 https://chem.libretexts.org/@go/page/38174


Figure 6.1.3 : Radio and cell towers are typically used to transmit long-wavelength electromagnetic radiation. Increasingly, cell
towers are designed to blend in with the landscape, as with the Tucson, Arizona, cell tower (right) disguised as a palm tree. (credit
left: modification of work by Sir Mildred Pierce; credit middle: modification of work by M.O. Stevens)

Other technologies also operate in the radio-wave portion of the electromagnetic spectrum. For example, 4G cellular telephone
signals are approximately 880 MHz, while Global Positioning System (GPS) signals operate at 1.228 and 1.575 GHz, local area
wireless technology (Wi-Fi) networks operate at 2.4 to 5 GHz, and highway toll sensors operate at 5.8 GHz. The frequencies
associated with these applications are convenient because such waves tend not to be absorbed much by common building materials.

Figure 6.1.4 : This schematic depicts how amplitude modulation (AM) and frequency modulation (FM) can be used to transmit a
radio wave.
One particularly characteristic phenomenon of waves results when two or more waves come into contact: They interfere with each
other. Figure 6.1.5 shows the interference patterns that arise when light passes through narrow slits closely spaced about a
wavelength apart. The fringe patterns produced depend on the wavelength, with the fringes being more closely spaced for shorter
wavelength light passing through a given set of slits. When the light passes through the two slits, each slit effectively acts as a new
source, resulting in two closely spaced waves coming into contact at the detector (the camera in this case). The dark regions in
Figure 6.1.5 correspond to regions where the peaks for the wave from one slit happen to coincide with the troughs for the wave
from the other slit (destructive interference), while the brightest regions correspond to the regions where the peaks for the two
waves (or their two troughs) happen to coincide (constructive interference). Likewise, when two stones are tossed close together
into a pond, interference patterns are visible in the interactions between the waves produced by the stones. Such interference
patterns cannot be explained by particles moving according to the laws of classical mechanics.

Access for free at OpenStax 6.1.4 https://chem.libretexts.org/@go/page/38174


Figure 6.1.5 : Interference fringe patterns are shown for light passing through two closely spaced, narrow slits. The spacing of the
fringes depends on the wavelength, with the fringes being more closely spaced for the shorter-wavelength blue light. (credit:
PASCO)

 Dorothy Hodgkin

Because the wavelengths of X-rays (10-10,000 picometers [pm]) are comparable to the size of atoms, X-rays can be used to
determine the structure of molecules. When a beam of X-rays is passed through molecules packed together in a crystal, the X-
rays collide with the electrons and scatter. Constructive and destructive interference of these scattered X-rays creates a specific
diffraction pattern. Calculating backward from this pattern, the positions of each of the atoms in the molecule can be
determined very precisely. One of the pioneers who helped create this technology was Dorothy Crowfoot Hodgkin.
She was born in Cairo, Egypt, in 1910, where her British parents were studying archeology. Even as a young girl, she was
fascinated with minerals and crystals. When she was a student at Oxford University, she began researching how X-ray
crystallography could be used to determine the structure of biomolecules. She invented new techniques that allowed her and
her students to determine the structures of vitamin B12, penicillin, and many other important molecules. Diabetes, a disease that
affects 382 million people worldwide, involves the hormone insulin. Hodgkin began studying the structure of insulin in 1934,
but it required several decades of advances in the field before she finally reported the structure in 1969. Understanding the
structure has led to better understanding of the disease and treatment options.

Not all waves are travelling waves. Standing waves (also known as stationary waves) remain constrained within some region of
space. As we shall see, standing waves play an important role in our understanding of the electronic structure of atoms and
molecules. The simplest example of a standing wave is a one-dimensional wave associated with a vibrating string that is held fixed
at its two end points. Figure 6.1.6 shows the four lowest-energy standing waves (the fundamental wave and the lowest three
harmonics) for a vibrating string at a particular amplitude. Although the string's motion lies mostly within a plane, the wave itself is
considered to be one dimensional, since it lies along the length of the string. The motion of string segments in a direction
perpendicular to the string length generates the waves and so the amplitude of the waves is visible as the maximum displacement of
the curves seen in Figure 6.1.6. The key observation from the figure is that only those waves having an integer number, n, of half-
wavelengths between the end points can form. A system with fixed end points such as this restricts the number and type of the
possible waveforms. This is an example of quantization, in which only discrete values from a more general set of continuous values
of some property are observed. Another important observation is that the harmonic waves (those waves displaying more than one-
half wavelength) all have one or more points between the two end points that are not in motion. These special points are nodes. The
energies of the standing waves with a given amplitude in a vibrating string increase with the number of half-wavelengths n. Since
the number of nodes is n – 1, the energy can also be said to depend on the number of nodes, generally increasing as the number of
nodes increases.

Figure 6.1.6 : A vibrating string shows some one-dimensional standing waves. Since the two end points of the string are held fixed,
only waves having an integer number of half-wavelengths can form. The points on the string between the end points that are not
moving are called the nodes.

Access for free at OpenStax 6.1.5 https://chem.libretexts.org/@go/page/38174


An example of two-dimensional standing waves is shown in Figure 6.1.7 which shows the vibrational patterns on a flat surface.
Although the vibrational amplitudes cannot be seen like they could in the vibrating string, the nodes have been made visible by
sprinkling the drum surface with a powder that collects on the areas of the surface that have minimal displacement. For one-
dimensional standing waves, the nodes were points on the line, but for two-dimensional standing waves, the nodes are lines on the
surface (for three-dimensional standing waves, the nodes are two-dimensional surfaces within the three-dimensional volume).
Because of the circular symmetry of the drum surface, its boundary conditions (the drum surface being tightly constrained to the
circumference of the drum) result in two types of nodes: radial nodes that sweep out all angles at constant radii and, thus, are seen
as circles about the center, and angular nodes that sweep out all radii at constant angles and, thus, are seen as lines passing through
the center. The upper left image in Figure 6.1.7 shows two radial nodes, while the image in the lower right shows the vibrational
pattern associated with three radial nodes and two angular nodes.

Figure 6.1.7 : Two-dimensional standing waves can be visualized on a vibrating surface. The surface has been sprinkled with a
powder that collects near the nodal lines. There are two types of nodes visible: radial nodes (circles) and angular nodes (radii).
For a more animated video, check this link out.

6.1.3: Blackbody Radiation and the Ultraviolet Catastrophe


The last few decades of the nineteenth century witnessed intense research activity in commercializing newly discovered electric
lighting. This required obtaining a better understanding of the distributions of light emitted from various sources being considered.
Artificial lighting is usually designed to mimic natural sunlight within the limitations of the underlying technology. Such lighting
consists of a range of broadly distributed frequencies that form a continuous spectrum. Figure 6.1.8 shows the wavelength
distribution for sunlight. The most intense radiation is in the visible region, with the intensity dropping off rapidly for shorter
wavelength ultraviolet (UV) light, and more slowly for longer wavelength infrared (IR) light.

Access for free at OpenStax 6.1.6 https://chem.libretexts.org/@go/page/38174


Figure 6.1.8 : The spectral distribution (light intensity vs. wavelength) of sunlight reaches the Earth's atmosphere as UV light,
visible light, and IR light. The unabsorbed sunlight at the top of the atmosphere has a distribution that approximately matches the
theoretical distribution of a blackbody at 5250 °C, represented by the blue curve. (credit: modification of work by American
Society for Testing and Materials (ASTM) Terrestrial Reference Spectra for Photovoltaic Performance Evaluation)
In Figure 6.1.8, the solar distribution is compared to a representative distribution, called a blackbody spectrum, that corresponds to
a temperature of 5250 °C. The blackbody spectrum matches the solar spectrum quite well. A blackbody is a convenient, ideal
emitter that approximates the behavior of many materials when heated. It is “ideal” in the same sense that an ideal gas is a
convenient, simple representation of real gases that works well, provided that the pressure is not too high nor the temperature too
low. A good approximation of a blackbody that can be used to observe blackbody radiation is a metal oven that can be heated to
very high temperatures. The oven has a small hole allowing for the light being emitted within the oven to be observed with a
spectrometer so that the wavelengths and their intensities can be measured. Figure 6.1.8 shows the resulting curves for some
representative temperatures. Each distribution depends only on a single parameter: the temperature. The maxima in the blackbody
curves, λmax, shift to shorter wavelengths as the temperature increases, reflecting the observation that metals being heated to high
temperatures begin to glow a darker red that becomes brighter as the temperature increases, eventually becoming white hot at very
high temperatures as the intensities of all of the visible wavelengths become appreciable. This common observation was at the heart
of the first paradox that showed the fundamental limitations of classical physics that we will examine.
Physicists derived mathematical expressions for the blackbody curves using well-accepted concepts from the theories of classical
mechanics and classical electromagnetism. The theoretical expressions as functions of temperature fit the observed experimental
blackbody curves well at longer wavelengths, but showed significant discrepancies at shorter wavelengths. Not only did the
theoretical curves not show a peak, they absurdly showed the intensity becoming infinitely large as the wavelength became smaller,
which would imply that everyday objects at room temperature should be emitting large amounts of UV light. This became known
as the “ultraviolet catastrophe” because no one could find any problems with the theoretical treatment that could lead to such
unrealistic short-wavelength behavior. Finally, around 1900, Max Planck derived a theoretical expression for blackbody radiation
that fit the experimental observations exactly (within experimental error). Planck developed his theoretical treatment by extending
the earlier work that had been based on the premise that the atoms composing the oven vibrated at increasing frequencies (or
decreasing wavelengths) as the temperature increased, with these vibrations being the source of the emitted electromagnetic
radiation. But where the earlier treatments had allowed the vibrating atoms to have any energy values obtained from a continuous
set of energies (perfectly reasonable, according to classical physics), Planck found that by restricting the vibrational energies to
discrete values for each frequency, he could derive an expression for blackbody radiation that correctly had the intensity dropping
rapidly for the short wavelengths in the UV region.

E = nhν, n = 1, 2, 3, . . .

The quantity h is a constant now known as Planck's constant, in his honor. Although Planck was pleased he had resolved the
blackbody radiation paradox, he was disturbed that to do so, he needed to assume the vibrating atoms required quantized energies,

Access for free at OpenStax 6.1.7 https://chem.libretexts.org/@go/page/38174


which he was unable to explain. The value of Planck's constant is very small, 6.626 × 10−34 joule seconds (J s), which helps explain
why energy quantization had not been observed previously in macroscopic phenomena.

Figure 6.1.9 : Blackbody spectral distribution curves are shown for some representative temperatures.
A graph is shown with a horizontal axis labeled, “Wavelength lambda (micrometers)” and a vertical axis labeled, “Intensity I (a r b.
units).” The horizontal axis begins at 0 and extends to 3.0 with markings provided every 0.1 micrometer. Similarly, the vertical axis
begins at 0 and extends to 10 with markings every 1 unit. Two vertical dashed lines are drawn. The first appears at about 0.39
micrometers and the second at about 0.72 micrometers. To the left of the first of these lines, the label, “Ultraviolet,” appears at the
top of the graph. Between these lines, the label, “Visible,” appears at the top of the graph. To the right of the second of these lines,
the label, “Infrared,” appears at the top of the graph. To the far right of the graph in open space a purple dot is placed which is
labeled, “lambda maximum.” A “Temperature” label is located in a central region of the graph. A blue curve begins on the
horizontal axis at about 0.05 micrometers. This curve increases steeply to a maximum value between the two vertical line segments
of approximately 9.5 at about 0.55 micrometers. This curve decreases rapidly at first, then tapers off to reach a value of about 1.5 at
the far right end of the graph. This blue curve is labeled 6000 K beneath the “Temperature” label. Curves are similarly drawn in
green for 5000 K, orange for 4000 K, and red for 3000 K. As the temperature decreases, the height of the peak is lower and shifted
right on the graph. The maximum value for the green curve is around 4.5 at 7.2 micrometers. This curve tapers at the right end of
the graph to a value around 0.6. The maximum for the orange curve is around 2 at about 0.9 micrometers. This curve tapers at the
right end of the graph to a value around 0.2. The maximum for the red curve is around 0.7 at about 1.2 micrometers. This curve
tapers at the right end of the graph to a value around 0.1. The entire region under the blue curve that is between the two dashed
lines, indicating the visible region, is shaded with vertical bands of color. The colors extending left to right across this region are
violet, indigo, blue, green, yellow, orange, and red. A purple dot is placed at the peak of each of the four colored curves. These
peaks are connected with a dashed curve.

6.1.4: The Photoelectric Effect


The next paradox in the classical theory to be resolved concerned the photoelectric effect (Figure 6.1.10). It had been observed that
electrons could be ejected from the clean surface of a metal when light having a frequency greater than some threshold frequency
was shone on it. Surprisingly, the kinetic energy of the ejected electrons did not depend on the brightness of the light, but increased
with increasing frequency of the light. Since the electrons in the metal had a certain amount of binding energy keeping them there,
the incident light needed to have more energy to free the electrons. According to classical wave theory, a wave's energy depends on
its intensity (which depends on its amplitude), not its frequency. One part of these observations was that the number of electrons
ejected within in a given time period was seen to increase as the brightness increased. In 1905, Albert Einstein was able to resolve
the paradox by incorporating Planck's quantization findings into the discredited particle view of light (Einstein actually won his
Nobel prize for this work, and not for his theories of relativity for which he is most famous).
Einstein argued that the quantized energies that Planck had postulated in his treatment of blackbody radiation could be applied to
the light in the photoelectric effect so that the light striking the metal surface should not be viewed as a wave, but instead as a
stream of particles (later called photons) whose energy depended on their frequency, according to Planck's formula, E = hν (or, in
hc
terms of wavelength using c = νλ, E = ). Electrons were ejected when hit by photons having sufficient energy (a frequency
λ
greater than the threshold). The greater the frequency, the greater the kinetic energy imparted to the escaping electrons by the
collisions. Einstein also argued that the light intensity did not depend on the amplitude of the incoming wave, but instead

Access for free at OpenStax 6.1.8 https://chem.libretexts.org/@go/page/38174


corresponded to the number of photons striking the surface within a given time period. This explains why the number of ejected
electrons increased with increasing brightness, since the greater the number of incoming photons, the greater the likelihood that
they would collide with some of the electrons.
With Einstein's findings, the nature of light took on a new air of mystery. Although many light phenomena could be explained
either in terms of waves or particles, certain phenomena, such as the interference patterns obtained when light passed through a
double slit, were completely contrary to a particle view of light, while other phenomena, such as the photoelectric effect, were
completely contrary to a wave view of light. Somehow, at a deep fundamental level still not fully understood, light is both wavelike
and particle-like. This is known as wave-particle duality.

Figure 6.1.10 : Photons with low frequencies do not have enough energy to cause electrons to be ejected via the photoelectric
effect. For any frequency of light above the threshold frequency, the kinetic energy of an ejected electron will increase linearly with
the energy of the incoming photon.
The figure includes three diagrams of waves approaching a flat, horizontal surface that is labeled, “Metal,” from an angle around
45 degrees above and to the left relative to the surface. At the top of the diagram at the center is the label, “E equals h nu.” At the
left, a sinusoidal wave reaches the surface and stops. The portion of the diagram near the flat metal surface is labeled, “No
electrons ejected,” and the wave is labeled, “700 n m.” To the right, a second similar, more compressed wave, which is labeled,
“550 n m,” reaches the flat surface. This time, an arrow extends up and to the right at an angle of approximately 45 degrees. A tiny
yellow circle with a negative sign in it is at the center of the arrow shaft. Above this arrow is the equation, “v subscript max equals
2.96 times 10 superscript 5 m per s.” To the far right, a third similar, even more compressed wave, which is labeled “400 n m”
reaches the flat surface. This time, an arrow extends up and to the right at an angle of approximately 45 degrees. A tiny yellow
circle with a negative sign in it is at the center of the arrow shaft. Above this arrow is the equation “v subscript max equals 6.22
times 10 superscript 5 m per s.”

 Example 6.1.2: Calculating the Energy of Radiation

When we see light from a neon sign, we are observing radiation from excited neon atoms. If this radiation has a wavelength of
640 nm, what is the energy of the photon being emitted?

Solution
We use the part of Planck's equation that includes the wavelength, λ, and convert units of nanometers to meters so that the units
of λ and c are the same.
hc
E =
λ

−34 8 −1
(6.626 × 10 J s )(2.998 × 10 m s )
=

1 m
(640 nm ) ( )
9
10 nm

−19
= 3.10 × 10 J

 Exercise 6.1.2

The microwaves in an oven are of a specific frequency that will heat the water molecules contained in food. (This is why most
plastics and glass do not become hot in a microwave oven-they do not contain water molecules.) This frequency is about 3 ×
109 Hz. What is the energy of one photon in these microwaves?

Access for free at OpenStax 6.1.9 https://chem.libretexts.org/@go/page/38174


Answer
2 × 10−24 J

 Example 6.1.3: Photoelectric Effect

Identify which of the following statements are false and, where necessary, change the italicized word or phrase to make them
true, consistent with Einstein's explanation of the photoelectric effect.
a. Increasing the brightness of incoming light increases the kinetic energy of the ejected electrons.
b. Increasing the wavelength of incoming light increases the kinetic energy of the ejected electrons.
c. Increasing the brightness of incoming light increases the number of ejected electrons.
d. Increasing the frequency of incoming light can increase the number of ejected electrons.

Solution
a. False. Increasing the brightness of incoming light has no effect on the kinetic energy of the ejected electrons. Only energy,
not the number or amplitude, of the photons influences the kinetic energy of the electrons.
b. False. Increasing the frequency of incoming light increases the kinetic energy of the ejected electrons. Frequency is
proportional to energy and inversely proportional to wavelength. Frequencies above the threshold value transfer the excess
energy into the kinetic energy of the electrons.
c. True. Because the number of collisions with photons increases with brighter light, the number of ejected electrons
increases.
d. True with regard to the threshold energy binding the electrons to the metal. Below this threshold, electrons are not emitted
and above it they are. Once over the threshold value, further increasing the frequency does not increase the number of
ejected electrons

 Exercise 6.1.3
Calculate the threshold energy in kJ/mol of electrons in aluminum, given that the lowest frequency photon for which the
photoelectric effect is observed is 9.87 × 10 H z .
14

Answer
3.94 kJ/mol

6.1.5: Line Spectra


Another paradox within the classical electromagnetic theory that scientists in the late nineteenth century struggled with concerned
the light emitted from atoms and molecules. When solids, liquids, or condensed gases are heated sufficiently, they radiate some of
the excess energy as light. Photons produced in this manner have a range of energies, and thereby produce a continuous spectrum in
which an unbroken series of wavelengths is present. Most of the light generated from stars (including our sun) is produced in this
fashion. You can see all the visible wavelengths of light present in sunlight by using a prism to separate them. As can be seen in
Figure 6.1.8, sunlight also contains UV light (shorter wavelengths) and IR light (longer wavelengths) that can be detected using
instruments but that are invisible to the human eye. Incandescent (glowing) solids such as tungsten filaments in incandescent lights
also give off light that contains all wavelengths of visible light. These continuous spectra can often be approximated by blackbody
radiation curves at some appropriate temperature, such as those shown in Figure 6.1.9.
In contrast to continuous spectra, light can also occur as discrete or line spectra having very narrow line widths interspersed
throughout the spectral regions such as those shown in Figure 6.1.5. Exciting a gas at low partial pressure using an electrical
current, or heating it, will produce line spectra. Fluorescent light bulbs and neon signs operate in this way (Figure 6.1.11). Each
element displays its own characteristic set of lines, as do molecules, although their spectra are generally much more complicated.

Access for free at OpenStax 6.1.10 https://chem.libretexts.org/@go/page/38174


Figure 6.1.11 : Neon signs operate by exciting a gas at low partial pressure using an electrical current. This sign shows the elaborate
artistic effects that can be achieved. (credit: Dave Shaver)
Each emission line consists of a single wavelength of light, which implies that the light emitted by a gas consists of a set of discrete
energies. For example, when an electric discharge passes through a tube containing hydrogen gas at low pressure, the H2 molecules
are broken apart into separate H atoms, and we see a blue-pink color. Passing the light through a prism produces a line spectrum,
indicating that this light is composed of photons of four visible wavelengths, as shown in Figure 6.1.12.

Figure 6.1.12 : Line spectra of select gas. Compare the two types of emission spectra: continuous spectrum of white light (top) and
the line spectra of the light from excited sodium, hydrogen, calcium, and mercury atoms.
An image is shown with 5 rows. Across the top and bottom of the image is a scale that begins at 4000 angstroms at the left and
extends to 740 angstroms at the far right. The top row is a continuous band of the visible spectrum, showing the colors from violet
at the far left through indigo, blue, green, yellow, orange, and red at the far right. The second row, labeled, “N a,” shows the
emission spectrum for the element sodium, which includes two narrow vertical bands in the blue range, two narrow bands in the
yellow-green range, two narrow bands in the yellow range, and one narrow band in the orange range. The third row, labeled, “H,”
shows the emission spectrum for hydrogen. This spectrum shows single bands in the violet, indigo, blue, and orange regions. The
fourth row, labeled, “C a,” shows the emission spectrum for calcium. This spectrum shows bands in the following colors and
frequencies; one violet, five indigo, one blue, two green, two yellow-green, one yellow, two yellow-orange, one orange, and one
red. The fifth row, labeled, “H g,” shows the emission spectrum for mercury. This spectrum shows bands in the following colors
and frequencies; two violet, one indigo, two blue, one green, two yellow, two orange, and one orange-red. It is important to note
that each of the color bands for the emission spectra of the elements matches to a specific wavelength of light. Extending a vertical
line from the bands to the scale above or below the diagram will match the band to a specific measurement on the scale.
The origin of discrete spectra in atoms and molecules was extremely puzzling to scientists in the late nineteenth century, since
according to classical electromagnetic theory, only continuous spectra should be observed. Even more puzzling, in 1885, Johann

Access for free at OpenStax 6.1.11 https://chem.libretexts.org/@go/page/38174


Balmer was able to derive an empirical equation that related the four visible wavelengths of light emitted by hydrogen atoms to
whole integers. That equation is the following one, in which k is a constant:
1 1 1
=k( − ) , n = 3, 4, 5, 6
2
λ 4 n

Other discrete lines for the hydrogen atom were found in the UV and IR regions. Johannes Rydberg generalized Balmer's work and
developed an empirical formula that predicted all of hydrogen's emission lines, not just those restricted to the visible range, where,
n1 and n2 are integers, n1 < n2, and R is the Rydberg constant (1.097 × 107 m−1).

1 1 1
= R∞ ( − )
2 2
λ n n
1 2

Even in the late nineteenth century, spectroscopy was a very precise science, and so the wavelengths of hydrogen were measured to
very high accuracy, which implied that the Rydberg constant could be determined very precisely as well. That such a simple
formula as the Rydberg formula could account for such precise measurements seemed astounding at the time, but it was the
eventual explanation for emission spectra by Neils Bohr in 1913 that ultimately convinced scientists to abandon classical physics
and spurred the development of modern quantum mechanics.

Summary
Light and other forms of electromagnetic radiation move through a vacuum with a constant speed, c, of 2.998 × 108 m s−1. This
radiation shows wavelike behavior, which can be characterized by a frequency, ν, and a wavelength, λ, such that c = λν. Light is an
example of a travelling wave. Other important wave phenomena include standing waves, periodic oscillations, and vibrations.
Standing waves exhibit quantization, since their wavelengths are limited to discrete integer multiples of some characteristic lengths.
Electromagnetic radiation that passes through two closely spaced narrow slits having dimensions roughly similar to the wavelength
will show an interference pattern that is a result of constructive and destructive interference of the waves. Electromagnetic radiation
also demonstrates properties of particles called photons. The energy of a photon is related to the frequency (or alternatively, the
hc
wavelength) of the radiation as E = hν (or E = ), where h is Planck's constant. That light demonstrates both wavelike and
λ
particle-like behavior is known as wave-particle duality. All forms of electromagnetic radiation share these properties, although
various forms including X-rays, visible light, microwaves, and radio waves interact differently with matter and have very different
practical applications. Electromagnetic radiation can be generated by exciting matter to higher energies, such as by heating it. The
emitted light can be either continuous (incandescent sources like the sun) or discrete (from specific types of excited atoms).
Continuous spectra often have distributions that can be approximated as blackbody radiation at some appropriate temperature. The
line spectrum of hydrogen can be obtained by passing the light from an electrified tube of hydrogen gas through a prism. This line
spectrum was simple enough that an empirical formula called the Rydberg formula could be derived from the spectrum. Three
historically important paradoxes from the late 19th and early 20th centuries that could not be explained within the existing
framework of classical mechanics and classical electromagnetism were the blackbody problem, the photoelectric effect, and the
discrete spectra of atoms. The resolution of these paradoxes ultimately led to quantum theories that superseded the classical
theories.

6.1.6: Key Equations


c = λν
hc
E = hν = , where h = 6.626 × 10−34 J s
λ

1 1 1
= R∞ ( − )
2 2
λ n n
1 2

Glossary
amplitude
extent of the displacement caused by a wave (for sinusoidal waves, it is one-half the difference from the peak height to the
trough depth, and the intensity is proportional to the square of the amplitude)

blackbody

Access for free at OpenStax 6.1.12 https://chem.libretexts.org/@go/page/38174


idealized perfect absorber of all incident electromagnetic radiation; such bodies emit electromagnetic radiation in characteristic
continuous spectra called blackbody radiation

continuous spectrum
electromagnetic radiation given off in an unbroken series of wavelengths (e.g., white light from the sun)

electromagnetic radiation
energy transmitted by waves that have an electric-field component and a magnetic-field component

electromagnetic spectrum
range of energies that electromagnetic radiation can comprise, including radio, microwaves, infrared, visible, ultraviolet, X-
rays, and gamma rays; since electromagnetic radiation energy is proportional to the frequency and inversely proportional to the
wavelength, the spectrum can also be specified by ranges of frequencies or wavelengths

frequency (ν )
number of wave cycles (peaks or troughs) that pass a specified point in space per unit time

hertz (Hz)
the unit of frequency, which is the number of cycles per second, s−1

intensity
property of wave-propagated energy related to the amplitude of the wave, such as brightness of light or loudness of sound

interference pattern
pattern typically consisting of alternating bright and dark fringes; it results from constructive and destructive interference of
waves

line spectrum
electromagnetic radiation emitted at discrete wavelengths by a specific atom (or atoms) in an excited state

node
any point of a standing wave with zero amplitude

photon
smallest possible packet of electromagnetic radiation, a particle of light

quantization
occurring only in specific discrete values, not continuous

standing wave
(also, stationary wave) localized wave phenomenon characterized by discrete wavelengths determined by the boundary
conditions used to generate the waves; standing waves are inherently quantized

wave
oscillation that can transport energy from one point to another in space

wavelength (λ)
distance between two consecutive peaks or troughs in a wave

wave-particle duality
term used to describe the fact that elementary particles including matter exhibit properties of both particles (including localized
position, momentum) and waves (including nonlocalization, wavelength, frequency)

This page titled 6.1: Electromagnetic Energy is shared under a CC BY 4.0 license and was authored, remixed, and/or curated by OpenStax via
source content that was edited to the style and standards of the LibreTexts platform; a detailed edit history is available upon request.

Access for free at OpenStax 6.1.13 https://chem.libretexts.org/@go/page/38174


6.2: The Bohr Model
 Learning Objectives
Describe the Bohr model of the hydrogen atom
Use the Rydberg equation to calculate energies of light emitted or absorbed by hydrogen atoms

Following the work of Ernest Rutherford and his colleagues in the early twentieth century, the picture of atoms consisting of tiny
dense nuclei surrounded by lighter and even tinier electrons continually moving about the nucleus was well established. This
picture was called the planetary model, since it pictured the atom as a miniature “solar system” with the electrons orbiting the
nucleus like planets orbiting the sun. The simplest atom is hydrogen, consisting of a single proton as the nucleus about which a
single electron moves. The electrostatic force attracting the electron to the proton depends only on the distance between the two
particles.
m1 m2
Fgravity = G
2
r

with
G is a gravitational constant
m1 and m are the masses of particle 1 and 2, respectively
2

r is the distance between the two particles

The electrostatic force has the same form as the gravitational force between two mass particles except that the electrostatic force
depends on the magnitudes of the charges on the particles (+1 for the proton and −1 for the electron) instead of the magnitudes of
the particle masses that govern the gravitational force.
m1 m2
Felectrostatic = k
2
r

with
k is a constant
m1 and m are the masses of particle 1 and 2, respectively
2

r is the distance between the two particles

Since forces can be derived from potentials, it is convenient to work with potentials instead, since they are forms of energy. The
electrostatic potential is also called the Coulomb potential. Because the electrostatic potential has the same form as the gravitational
potential, according to classical mechanics, the equations of motion should be similar, with the electron moving around the nucleus
in circular or elliptical orbits (hence the label “planetary” model of the atom). Potentials of the form V(r) that depend only on the
radial distance r are known as central potentials. Central potentials have spherical symmetry, and so rather than specifying the
position of the electron in the usual Cartesian coordinates (x, y, z), it is more convenient to use polar spherical coordinates centered
at the nucleus, consisting of a linear coordinate r and two angular coordinates, usually specified by the Greek letters theta (θ) and
phi (Φ). These coordinates are similar to the ones used in GPS devices and most smart phones that track positions on our (nearly)
spherical earth, with the two angular coordinates specified by the latitude and longitude, and the linear coordinate specified by sea-
level elevation. Because of the spherical symmetry of central potentials, the energy and angular momentum of the classical
hydrogen atom are constants, and the orbits are constrained to lie in a plane like the planets orbiting the sun. This classical
mechanics description of the atom is incomplete, however, since an electron moving in an elliptical orbit would be accelerating (by
changing direction) and, according to classical electromagnetism, it should continuously emit electromagnetic radiation. This loss
in orbital energy should result in the electron’s orbit getting continually smaller until it spirals into the nucleus, implying that atoms
are inherently unstable.
In 1913, Niels Bohr attempted to resolve the atomic paradox by ignoring classical electromagnetism’s prediction that the orbiting
electron in hydrogen would continuously emit light. Instead, he incorporated into the classical mechanics description of the atom
Planck’s ideas of quantization and Einstein’s finding that light consists of photons whose energy is proportional to their frequency.
Bohr assumed that the electron orbiting the nucleus would not normally emit any radiation (the stationary state hypothesis), but it
would emit or absorb a photon if it moved to a different orbit. The energy absorbed or emitted would reflect differences in the
orbital energies according to this equation:

Access for free at OpenStax 6.2.1 https://chem.libretexts.org/@go/page/38175


hc
|ΔE| = | Ef − Ei | = hu = (6.2.1)
λ

In this equation, h is Planck’s constant and Ei and Ef are the initial and final orbital energies, respectively. The absolute value of the
energy difference is used, since frequencies and wavelengths are always positive. Instead of allowing for continuous values for the
angular momentum, energy, and orbit radius, Bohr assumed that only discrete values for these could occur (actually, quantizing any
one of these would imply that the other two are also quantized). Bohr’s expression for the quantized energies is:
k
En = − (6.2.2)
2
n

with n = 1, 2, 3, . . .
In this expression, k is a constant comprising fundamental constants such as the electron mass and charge and Planck’s constant.
Inserting the expression for the orbit energies into the equation for ΔE gives

1 1 hc
ΔE = k ( − ) = (6.2.3)
2 2
n n λ
1 2

or

1 k 1 1
= ( − ) (6.2.4)
2 2
λ hc n n
1 2

The lowest few energy levels are shown in Figure 6.2.1. One of the fundamental laws of physics is that matter is most stable with
the lowest possible energy. Thus, the electron in a hydrogen atom usually moves in the n = 1 orbit, the orbit in which it has the
lowest energy. When the electron is in this lowest energy orbit, the atom is said to be in its ground electronic state (or simply
ground state). If the atom receives energy from an outside source, it is possible for the electron to move to an orbit with a higher n
value and the atom is now in an excited electronic state (or simply an excited state) with a higher energy. When an electron
transitions from an excited state (higher energy orbit) to a less excited state, or ground state, the difference in energy is emitted as a
photon. Similarly, if a photon is absorbed by an atom, the energy of the photon moves an electron from a lower energy orbit up to a
more excited one.

Figure 6.2.1 : Quantum numbers and energy levels in a hydrogen atom. The more negative the calculated value, the lower the
energy.
We can relate the energy of electrons in atoms to what we learned previously about energy. The law of conservation of energy says
that we can neither create nor destroy energy. Thus, if a certain amount of external energy is required to excite an electron from one
energy level to another, that same amount of energy will be liberated when the electron returns to its initial state (Figure 6.2.2). In
effect, an atom can “store” energy by using it to promote an electron to a state with a higher energy and release it when the electron
returns to a lower state. The energy can be released as one quantum of energy, as the electron returns to its ground state (say, from

Access for free at OpenStax 6.2.2 https://chem.libretexts.org/@go/page/38175


n =5 to n = 1 ), or it can be released as two or more smaller quanta as the electron falls to an intermediate state, then to the
ground state (say, from n = 5 to n = 4 , emitting one quantum, then to n = 1 , emitting a second quantum).

Figure 6.2.2 : The horizontal lines show the relative energy of orbits in the Bohr model of the hydrogen atom, and the vertical
arrows depict the energy of photons absorbed (left) or emitted (right) as electrons move between these orbits.
The figure includes a diagram representing the relative energy levels of the quantum numbers of the hydrogen atom. An upward
pointing arrow at the left of the diagram is labeled, “E.” A grey shaded vertically oriented rectangle is placed just right of the arrow.
The rectangle height matches the arrow length. Colored, horizontal line segments are placed inside the rectangle and labels are
placed to the right of the box, arranged in a column with the heading, “Energy, n.” At the very base of the rectangle, a purple
horizontal line segment is drawn. A black line extends to the right to the label, “1.” At a level approximately three-quarters of the
distance to the top of the rectangle, a blue horizontal line segment is drawn. A black line extends to the right to the label, “2.” At a
level approximately seven-eighths the distance from the base of the rectangle, a green horizontal line segment is drawn. A black
line extends to the right to the label, “3.” Just a short distance above this segment, an orange horizontal line segment is drawn. A
black line segment extends to the right to the label, “4.” Just above this segment, a red horizontal line segment is drawn. A black
line extends to the right to the label, “5.” Just a short distance above this segment, a brown horizontal line segment is drawn. A
black line extends to the right to the label, “infinity.” Arrows are drawn to depict energies of photons absorbed, as shown by
upward pointing arrows on the left, or released as shown by downward pointing arrows on the right side of the diagram between
the colored line segments. The label, “Electron moves to higher energy as light is absorbed,” is placed beneath the upward pointing
arrows. Similarly, the label, “Electron moves to lower energy as light is emitted,” appears beneath the downward pointing arrows.
Moving left to right across the diagram, arrows extend from one colored line segment to the next in the following order: purple to
blue, purple to green, purple to orange, purple to red, purple to brown, blue to green, blue to orange, and blue to red. The arrows
originating from the same colored segment are grouped together by close placement of the arrows. Similarly, the downward arrows
follow in this sequence; brown to purple, red to purple, orange to purple, green to purple, blue to purple, red to blue, orange to blue,
and green to blue. Arrows are again grouped by close placement according to the color at which the arrows end.
Since Bohr’s model involved only a single electron, it could also be applied to the single electron ions He+, Li2+, Be3+, and so
forth, which differ from hydrogen only in their nuclear charges, and so one-electron atoms and ions are collectively referred to as
hydrogen-like or hydrogenic atoms. The energy expression for hydrogen-like atoms is a generalization of the hydrogen atom
energy, in which Z is the nuclear charge (+1 for hydrogen, +2 for He, +3 for Li, and so on) and k has a value of 2.179 × 10 J . –18

2
kZ
En = − (6.2.5)
2
n

The sizes of the circular orbits for hydrogen-like atoms are given in terms of their radii by the following expression, in which a is o

a constant called the Bohr radius, with a value of 5.292 × 10 m:


−11

2
n
r = a0 (6.2.6)
Z

The equation also shows us that as the electron’s energy increases (as n increases), the electron is found at greater distances from
the nucleus. This is implied by the inverse dependence on r in the Coulomb potential, since, as the electron moves away from the
nucleus, the electrostatic attraction between it and the nucleus decreases, and it is held less tightly in the atom. Note that as n gets
larger and the orbits get larger, their energies get closer to zero, and so the limits n ⟶ ∞ and r ⟶ ∞ imply that E = 0

Access for free at OpenStax 6.2.3 https://chem.libretexts.org/@go/page/38175


corresponds to the ionization limit where the electron is completely removed from the nucleus. Thus, for hydrogen in the ground
state n = 1 , the ionization energy would be:
ΔE = En⟶∞ − E1 = 0 + k = k (6.2.7)

With three extremely puzzling paradoxes now solved (blackbody radiation, the photoelectric effect, and the hydrogen atom), and all
involving Planck’s constant in a fundamental manner, it became clear to most physicists at that time that the classical theories that
worked so well in the macroscopic world were fundamentally flawed and could not be extended down into the microscopic domain
of atoms and molecules. Unfortunately, despite Bohr’s remarkable achievement in deriving a theoretical expression for the Rydberg
constant, he was unable to extend his theory to the next simplest atom, He, which only has two electrons. Bohr’s model was
severely flawed, since it was still based on the classical mechanics notion of precise orbits, a concept that was later found to be
untenable in the microscopic domain, when a proper model of quantum mechanics was developed to supersede classical
mechanics.

 Example 6.2.1: Calculating the Energy of an Electron in a Bohr Orbit

Early researchers were very excited when they were able to predict the energy of an electron at a particular distance from the
nucleus in a hydrogen atom. If a spark promotes the electron in a hydrogen atom into an orbit with n = 3 , what is the
calculated energy, in joules, of the electron?

Solution
The energy of the electron is given by Equation 6.2.5:
2
−kZ
E =
2
n

The atomic number, Z , of hydrogen is 1; k = 2.179 × 10 –18


J ; and the electron is characterized by an n value of 3. Thus,
−18 2
−(2.179 × 10 J) × (1 )
−19
E = = −2.421 × 10 J
2
(3)

 Exercise 6.2.1

The electron in Example 6.2.1 in the n = 3 state is promoted even further to an orbit with n = 6 . What is its new energy?

Answer
TBD

 Example 6.2.2: Calculating Electron Transitions in a One–electron System

What is the energy (in joules) and the wavelength (in meters) of the line in the spectrum of hydrogen that represents the
movement of an electron from Bohr orbit with n = 4 to the orbit with n = 6? In what part of the electromagnetic spectrum do
we find this radiation?

Solution
In this case, the electron starts out with n = 4 , so n = 4 . It comes to rest in the
1 n =6 orbit, so n2 = 6 . The difference in
energy between the two states is given by this expression:

1 1
−18
ΔE = E1 − E2 = 2.179 × 10 ( − )
2 2
n n
1 2

1 1
−18
ΔE = 2.179 × 10 ( − ) J
2 2
4 6

Access for free at OpenStax 6.2.4 https://chem.libretexts.org/@go/page/38175


1 1
−18
ΔE = 2.179 × 10 ( − ) J
16 36

−20
ΔE = 7.566 × 10 J

This energy difference is positive, indicating a photon enters the system (is absorbed) to excite the electron from the n = 4 orbit
up to the n = 6 orbit. The wavelength of a photon with this energy is found by the expression E = hcλ . Rearrangement gives:
hc
λ =
E

From the figure of electromagnetic radiation, we can see that this wavelength is found in the infrared portion of the
electromagnetic spectrum.

 Exercise 6.2.2

What is the energy in joules and the wavelength in meters of the photon produced when an electron falls from the n = 5 to the
n = 3 level in a H e ion (Z = 2 for H e )?
+ +

Answer
6.198 × 10
–19
J and 3.205 × 10−7
m

Bohr’s model of the hydrogen atom provides insight into the behavior of matter at the microscopic level, but it is does not account
for electron–electron interactions in atoms with more than one electron. It does introduce several important features of all models
used to describe the distribution of electrons in an atom. These features include the following:
The energies of electrons (energy levels) in an atom are quantized, described by quantum numbers: integer numbers having only
specific allowed value and used to characterize the arrangement of electrons in an atom.
An electron’s energy increases with increasing distance from the nucleus.
The discrete energies (lines) in the spectra of the elements result from quantized electronic energies.
Of these features, the most important is the postulate of quantized energy levels for an electron in an atom. As a consequence, the
model laid the foundation for the quantum mechanical model of the atom. Bohr won a Nobel Prize in Physics for his contributions
to our understanding of the structure of atoms and how that is related to line spectra emissions.

Summary
Bohr incorporated Planck’s and Einstein’s quantization ideas into a model of the hydrogen atom that resolved the paradox of atom
stability and discrete spectra. The Bohr model of the hydrogen atom explains the connection between the quantization of photons
and the quantized emission from atoms. Bohr described the hydrogen atom in terms of an electron moving in a circular orbit about
a nucleus. He postulated that the electron was restricted to certain orbits characterized by discrete energies. Transitions between
these allowed orbits result in the absorption or emission of photons. When an electron moves from a higher-energy orbit to a more
stable one, energy is emitted in the form of a photon. To move an electron from a stable orbit to a more excited one, a photon of
energy must be absorbed. Using the Bohr model, we can calculate the energy of an electron and the radius of its orbit in any one-
electron system.

Glossary
Bohr’s model of the hydrogen atom
structural model in which an electron moves around the nucleus only in circular orbits, each with a specific allowed radius; the
orbiting electron does not normally emit electromagnetic radiation, but does so when changing from one orbit to another.

excited state
state having an energy greater than the ground-state energy

Access for free at OpenStax 6.2.5 https://chem.libretexts.org/@go/page/38175


ground state
state in which the electrons in an atom, ion, or molecule have the lowest energy possible

quantum number
integer number having only specific allowed values and used to characterize the arrangement of electrons in an atom

This page titled 6.2: The Bohr Model is shared under a CC BY 4.0 license and was authored, remixed, and/or curated by OpenStax via source
content that was edited to the style and standards of the LibreTexts platform; a detailed edit history is available upon request.

Access for free at OpenStax 6.2.6 https://chem.libretexts.org/@go/page/38175


6.3: Development of Quantum Theory
 Learning Objectives
Extend the concept of wave–particle duality that was observed in electromagnetic radiation to matter as well
Understand the general idea of the quantum mechanical description of electrons in an atom, and that it uses the notion of
three-dimensional wave functions, or orbitals, that define the distribution of probability to find an electron in a particular
part of space
List and describe traits of the four quantum numbers that form the basis for completely specifying the state of an electron in
an atom

Bohr’s model explained the experimental data for the hydrogen atom and was widely accepted, but it also raised many questions.
Why did electrons orbit at only fixed distances defined by a single quantum number n = 1, 2, 3, and so on, but never in between?
Why did the model work so well describing hydrogen and one-electron ions, but could not correctly predict the emission spectrum
for helium or any larger atoms? To answer these questions, scientists needed to completely revise the way they thought about
matter.

6.3.1: Behavior in the Microscopic World


We know how matter behaves in the macroscopic world—objects that are large enough to be seen by the naked eye follow the rules
of classical physics. A billiard ball moving on a table will behave like a particle: It will continue in a straight line unless it collides
with another ball or the table cushion, or is acted on by some other force (such as friction). The ball has a well-defined position and
velocity (or a well-defined momentum, p = mv , defined by mass m and velocity v ) at any given moment. In other words, the ball
is moving in a classical trajectory. This is the typical behavior of a classical object.
When waves interact with each other, they show interference patterns that are not displayed by macroscopic particles such as the
billiard ball. For example, interacting waves on the surface of water can produce interference patters similar to those shown on
Figure 6.3.1. This is a case of wave behavior on the macroscopic scale, and it is clear that particles and waves are very different
phenomena in the macroscopic realm.

Figure 6.3.1 : An interference pattern on the water surface is formed by interacting waves. The waves are caused by reflection of
water from the rocks. (credit: modification of work by Sukanto Debnath)
A photograph is shown of ripples in water. The ripples display an interference pattern with each other.
As technological improvements allowed scientists to probe the microscopic world in greater detail, it became increasingly clear by
the 1920s that very small pieces of matter follow a different set of rules from those we observe for large objects. The
unquestionable separation of waves and particles was no longer the case for the microscopic world.
One of the first people to pay attention to the special behavior of the microscopic world was Louis de Broglie. He asked the
question: If electromagnetic radiation can have particle-like character, can electrons and other submicroscopic particles exhibit
wavelike character? In his 1925 doctoral dissertation, de Broglie extended the wave–particle duality of light that Einstein used to
resolve the photoelectric-effect paradox to material particles. He predicted that a particle with mass m and velocity v (that is, with
linear momentum p) should also exhibit the behavior of a wave with a wavelength value λ, given by this expression in which h is
the familiar Planck’s constant
h h
λ = = (6.3.1)
mv p

Access for free at OpenStax 6.3.1 https://chem.libretexts.org/@go/page/38176


This is called the de Broglie wavelength. Unlike the other values of λ discussed in this chapter, the de Broglie wavelength is a
characteristic of particles and other bodies, not electromagnetic radiation (note that this equation involves velocity [v, m/s], not
frequency [ν, Hz]. Although these two symbols are identical, they mean very different things). Where Bohr had postulated the
electron as being a particle orbiting the nucleus in quantized orbits, de Broglie argued that Bohr’s assumption of quantization can
be explained if the electron is considered not as a particle, but rather as a circular standing wave such that only an integer number
of wavelengths could fit exactly within the orbit (Figure 6.3.2).

Figure 6.3.1 : If an electron is viewed as a wave circling around the nucleus, an integer number of wavelengths must fit into the
orbit for this standing wave behavior to be possible.
This figure includes a circle formed from a dashed line. A sinusoidal wave pattern indicated with a solid red line is wrapped around
the circle, centered about the edge of the circle. Line segments extend outward from the circle extending through 2 wave crests
along the circle. A double ended arrow is drawn between these segments and is labeled, “wavelength, lambda.” A dashed double
headed arrow is drawn from the center to the edge of the circle and is labeled, “radius r.”
For a circular orbit of radius r, the circumference is 2πr, and so de Broglie’s condition is:
2πr = nλ (6.3.2)

with n = 1, 2, 3, . . .
Since the de Broglie expression relates the wavelength to the momentum and, hence, velocity, this implies:
nh nh nhr nhr
2πr = nλ = = = = (6.3.3)
p mv mvr L

This expression can be rearranged to give Bohr’s formula for the quantization of the angular momentum:
nh
L = = nℏ (6.3.4)

Classical angular momentum L for a circular motion is equal to the product of the radius of the circle and the momentum of the
moving particle p.
L = rp = rmv (for a circular motion) (6.3.5)

Figure 6.3.3 : The diagram shows angular momentum for a circular motion.
Shortly after de Broglie proposed the wave nature of matter, two scientists at Bell Laboratories, C. J. Davisson and L. H. Germer,
demonstrated experimentally that electrons can exhibit wavelike behavior by showing an interference pattern for electrons
travelling through a regular atomic pattern in a crystal. The regularly spaced atomic layers served as slits, as used in other
interference experiments. Since the spacing between the layers serving as slits needs to be similar in size to the wavelength of the
tested wave for an interference pattern to form, Davisson and Germer used a crystalline nickel target for their “slits,” since the
spacing of the atoms within the lattice was approximately the same as the de Broglie wavelengths of the electrons that they used.
Figure 6.3.4 shows an interference pattern.

Access for free at OpenStax 6.3.2 https://chem.libretexts.org/@go/page/38176


Figure 6.3.4 : (a) The interference pattern for electrons passing through very closely spaced slits demonstrates that quantum
particles such as electrons can exhibit wavelike behavior. (b) The experimental results illustrated here demonstrate the wave–
particle duality in electrons. The electrons pass through very closely spaced slits, forming an interference pattern, with increasing
numbers of electrons being recorded from the left image to the right. With only a few electrons recorded, it is clear that the
electrons arrive as individual localized “particles,” but in a seemingly random pattern. As more electrons arrive, a wavelike
interference pattern begins to emerge. Note that the probability of the final electron location is still governed by the wave-type
distribution, even for a single electron, but it can be observed more easily if many electron collisions have been recorded.

The wave–particle duality of matter can be seen by observing what happens if electron collisions are recorded over a long period of
time. Initially, when only a few electrons have been recorded, they show clear particle-like behavior, having arrived in small
localized packets that appear to be random. As more and more electrons arrived and were recorded, a clear interference pattern that
is the hallmark of wavelike behavior emerged. Thus, it appears that while electrons are small localized particles, their motion does
not follow the equations of motion implied by classical mechanics, but instead it is governed by some type of a wave equation that
governs a probability distribution even for a single electron’s motion. Thus the wave–particle duality first observed with photons is
actually a fundamental behavior intrinsic to all quantum particles.

DR. QUANTUM - DOUBLE SLIT EXPERI…


EXPERI…

Video 6.3.1 : View the Dr. Quantum – Double Slit Experiment cartoon for an easy-to-understand description of wave–particle
duality and the associated experiments.

 Example 6.3.1: Calculating the Wavelength of a Particle

If an electron travels at a velocity of 1.000 × 107 m s–1 and has a mass of 9.109 × 10–28 g, what is its wavelength?

Solution

Access for free at OpenStax 6.3.3 https://chem.libretexts.org/@go/page/38176


We can use de Broglie’s equation to solve this problem, but we first must do a unit conversion of Planck’s constant. You
learned earlier that 1 J = 1 kg m2/s2. Thus, we can write h = 6.626 × 10–34 J s as 6.626 × 10–34 kg m2/s.
h
λ =
mv

−34 2
6.626 × 10 kg m /s
=
−31 7
(9.109 × 10 kg)(1.000 × 10 m/s)

−11
= 7.274 × 10 m

This is a small value, but it is significantly larger than the size of an electron in the classical (particle) view. This size is the
same order of magnitude as the size of an atom. This means that electron wavelike behavior is going to be noticeable in an
atom.

 Exercise 6.3.1

Calculate the wavelength of a softball with a mass of 100 g traveling at a velocity of 35 m s–1, assuming that it can be modeled
as a single particle.

Answer
1.9 × 10–34 m.

We never think of a thrown softball having a wavelength, since this wavelength is so small it is impossible for our senses or any
known instrument to detect (strictly speaking, the wavelength of a real baseball would correspond to the wavelengths of its
constituent atoms and molecules, which, while much larger than this value, would still be microscopically tiny). The de Broglie
wavelength is only appreciable for matter that has a very small mass and/or a very high velocity.
Werner Heisenberg considered the limits of how accurately we can measure properties of an electron or other microscopic particles.
He determined that there is a fundamental limit to how accurately one can measure both a particle’s position and its momentum
simultaneously. The more accurately we measure the momentum of a particle, the less accurately we can determine its position at
that time, and vice versa. This is summed up in what we now call the Heisenberg uncertainty principle: It is fundamentally
impossible to determine simultaneously and exactly both the momentum and the position of a particle. For a particle of mass m
moving with velocity vx in the x direction (or equivalently with momentum px), the product of the uncertainty in the position, Δx,
ℏ h
and the uncertainty in the momentum, Δpx , must be greater than or equal to (recall that ℏ = , the value of Planck’s constant
2 2π
divided by 2π).

Δx × Δpx = (Δx)(mΔv) ≥
2

This equation allows us to calculate the limit to how precisely we can know both the simultaneous position of an object and its
momentum. For example, if we improve our measurement of an electron’s position so that the uncertainty in the position (Δx) has a
value of, say, 1 pm (10–12 m, about 1% of the diameter of a hydrogen atom), then our determination of its momentum must have an
uncertainty with a value of at least
−34 2
h (1.055 × 10 kg m /s)
−23
[Δp = mΔv = ] = = 5 × 10 kg m/s.
−12
(2Δx) (2 × 1 × 10 m)

The value of ħ is not large, so the uncertainty in the position or momentum of a macroscopic object like a baseball is too
insignificant to observe. However, the mass of a microscopic object such as an electron is small enough that the uncertainty can be
large and significant.
It should be noted that Heisenberg’s uncertainty principle is not just limited to uncertainties in position and momentum, but it also
links other dynamical variables. For example, when an atom absorbs a photon and makes a transition from one energy state to

another, the uncertainty in the energy and the uncertainty in the time required for the transition are similarly related, as ΔE Δt ≥ .
2
As will be discussed later, even the vector components of angular momentum cannot all be specified exactly simultaneously.

Access for free at OpenStax 6.3.4 https://chem.libretexts.org/@go/page/38176


Heisenberg’s principle imposes ultimate limits on what is knowable in science. The uncertainty principle can be shown to be a
consequence of wave–particle duality, which lies at the heart of what distinguishes modern quantum theory from classical
mechanics. Recall that the equations of motion obtained from classical mechanics are trajectories where, at any given instant in
time, both the position and the momentum of a particle can be determined exactly. Heisenberg’s uncertainty principle implies that
such a view is untenable in the microscopic domain and that there are fundamental limitations governing the motion of quantum
particles. This does not mean that microscopic particles do not move in trajectories, it is just that measurements of trajectories are
limited in their precision. In the realm of quantum mechanics, measurements introduce changes into the system that is being
observed.

6.3.2: The Quantum–Mechanical Model of an Atom


Shortly after de Broglie published his ideas that the electron in a hydrogen atom could be better thought of as being a circular
standing wave instead of a particle moving in quantized circular orbits, as Bohr had argued, Erwin Schrödinger extended de
Broglie’s work by incorporating the de Broglie relation into a wave equation, deriving what is today known as the Schrödinger
equation. When Schrödinger applied his equation to hydrogen-like atoms, he was able to reproduce Bohr’s expression for the
energy and, thus, the Rydberg formula governing hydrogen spectra, and he did so without having to invoke Bohr’s assumptions of
stationary states and quantized orbits, angular momenta, and energies; quantization in Schrödinger’s theory was a natural
consequence of the underlying mathematics of the wave equation. Like de Broglie, Schrödinger initially viewed the electron in
hydrogen as being a physical wave instead of a particle, but where de Broglie thought of the electron in terms of circular stationary
waves, Schrödinger properly thought in terms of three-dimensional stationary waves, or wavefunctions, represented by the Greek
letter psi, ψ. A few years later, Max Born proposed an interpretation of the wavefunction ψ that is still accepted today: Electrons
are still particles, and so the waves represented by ψ are not physical waves but, instead, are complex probability amplitudes. The
square of the magnitude of a wavefunction ∣ψ∣ describes the probability of the quantum particle being present near a certain
2

location in space. This means that wavefunctions can be used to determine the distribution of the electron’s density with respect to
the nucleus in an atom. In the most general form, the Schrödinger equation can be written as:
^
H ψ = Eψ

^
H is the Hamiltonian operator, a set of mathematical operations representing the total energy of the quantum particle (such as an
electron in an atom), ψ is the wavefunction of this particle that can be used to find the special distribution of the probability of
finding the particle, and E is the actual value of the total energy of the particle.
Schrödinger’s work, as well as that of Heisenberg and many other scientists following in their footsteps, is generally referred to as
quantum mechanics.

Schrödinger's Cat

Video 6.3.2 : You may also have heard of Schrödinger because of his famous thought experiment. This story explains the concepts
of superposition and entanglement as related to a cat in a box with poison (Schrödinger's Cat [www.youtube.com]).

Access for free at OpenStax 6.3.5 https://chem.libretexts.org/@go/page/38176


6.3.3: Understanding Quantum Theory of Electrons in Atoms
The goal of this section is to understand the electron orbitals (location of electrons in atoms), their different energies, and other
properties. The use of quantum theory provides the best understanding to these topics. This knowledge is a precursor to chemical
bonding.
As was described previously, electrons in atoms can exist only on discrete energy levels but not between them. It is said that the
energy of an electron in an atom is quantized, that is, it can be equal only to certain specific values and can jump from one energy
level to another but not transition smoothly or stay between these levels.
The energy levels are labeled with an n value, where n = 1, 2, 3, …. Generally speaking, the energy of an electron in an atom is
greater for greater values of n. This number, n, is referred to as the principal quantum number. The principal quantum number
defines the location of the energy level. It is essentially the same concept as the n in the Bohr atom description. Another name for
the principal quantum number is the shell number. The shells of an atom can be thought of concentric circles radiating out from the
nucleus. The electrons that belong to a specific shell are most likely to be found within the corresponding circular area. The further
we proceed from the nucleus, the higher the shell number, and so the higher the energy level (Figure 6.3.5). The positively charged
protons in the nucleus stabilize the electronic orbitals by electrostatic attraction between the positive charges of the protons and the
negative charges of the electrons. So the further away the electron is from the nucleus, the greater the energy it has.

Figure 6.3.5 : Different shells are numbered by principal quantum numbers.


This quantum mechanical model for where electrons reside in an atom can be used to look at electronic transitions, the events when
an electron moves from one energy level to another. If the transition is to a higher energy level, energy is absorbed, and the energy
change has a positive value. To obtain the amount of energy necessary for the transition to a higher energy level, a photon is
absorbed by the atom. A transition to a lower energy level involves a release of energy, and the energy change is negative. This
process is accompanied by emission of a photon by the atom. The following equation summarizes these relationships and is based
on the hydrogen atom:
ΔE = Efinal − Einitial

1 1
−18
= −2.18 × 10 ( − ) J
2 2
n n
f i

The values nf and ni are the final and initial energy states of the electron.
The principal quantum number is one of three quantum numbers used to characterize an orbital. An atomic orbital, which is distinct
from an orbit, is a general region in an atom within which an electron is most probable to reside. The quantum mechanical model
specifies the probability of finding an electron in the three-dimensional space around the nucleus and is based on solutions of the
Schrödinger equation. In addition, the principal quantum number defines the energy of an electron in a hydrogen or hydrogen-like
atom or an ion (an atom or an ion with only one electron) and the general region in which discrete energy levels of electrons in a
multi-electron atoms and ions are located.
Another quantum number is l, the angular momentum quantum number. It is an integer that defines the shape of the orbital, and
takes on the values, l = 0, 1, 2, …, n – 1. This means that an orbital with n = 1 can have only one value of l, l = 0, whereas n = 2
permits l = 0 and l = 1, and so on. The principal quantum number defines the general size and energy of the orbital. The l value
specifies the shape of the orbital. Orbitals with the same value of l form a subshell. In addition, the greater the angular momentum
quantum number, the greater is the angular momentum of an electron at this orbital.

Access for free at OpenStax 6.3.6 https://chem.libretexts.org/@go/page/38176


Orbitals with l = 0 are called s orbitals (or the s subshells). The value l = 1 corresponds to the p orbitals. For a given n, p orbitals
constitute a p subshell (e.g., 3p if n = 3). The orbitals with l = 2 are called the d orbitals, followed by the f-, g-, and h-orbitals for l =
3, 4, 5, and there are higher values we will not consider.
There are certain distances from the nucleus at which the probability density of finding an electron located at a particular orbital is
zero. In other words, the value of the wavefunction ψ is zero at this distance for this orbital. Such a value of radius r is called a
radial node. The number of radial nodes in an orbital is n – l – 1.

Figure 6.3.6 : The graphs show the probability (y axis) of finding an electron for the 1s, 2s, 3s orbitals as a function of distance
from the nucleus.
Consider the examples in Figure 6.3.7. The orbitals depicted are of the s type, thus l = 0 for all of them. It can be seen from the
graphs of the probability densities that there are 1 – 0 – 1 = 0 places where the density is zero (nodes) for 1s (n = 1), 2 – 0 – 1 = 1
node for 2s, and 3 – 0 – 1 = 2 nodes for the 3s orbitals.
The s subshell electron density distribution is spherical and the p subshell has a dumbbell shape. The d and f orbitals are more
complex. These shapes represent the three-dimensional regions within which the electron is likely to be found.

Access for free at OpenStax 6.3.7 https://chem.libretexts.org/@go/page/38176


Figure 6.3.7 : Shapes of s, p, d, and f orbitals.

If an electron has an angular momentum (l ≠ 0), then this vector can point in different directions. In addition, the z component of
the angular momentum can have more than one value. This means that if a magnetic field is applied in the z direction, orbitals with
different values of the z component of the angular momentum will have different energies resulting from interacting with the field.
The magnetic quantum number, called ml, specifies the z component of the angular momentum for a particular orbital. For
example, for an s orbital, l = 0, and the only value of ml is zero. For p orbitals, l = 1, and ml can be equal to –1, 0, or +1. Generally
speaking, ml can be equal to –l, –(l – 1), …, –1, 0, +1, …, (l – 1), l. The total number of possible orbitals with the same value of l (a
subshell) is 2l + 1. Thus, there is one s-orbital for ml = 0, there are three p-orbitals for ml = 1, five d-orbitals for ml = 2, seven f-
orbitals for ml = 3, and so forth. The principal quantum number defines the general value of the electronic energy. The angular
momentum quantum number determines the shape of the orbital. And the magnetic quantum number specifies orientation of the
orbital in space, as can be seen in Figure 6.3.7.

Access for free at OpenStax 6.3.8 https://chem.libretexts.org/@go/page/38176


Figure 6.3.8 : The chart shows the energies of electron orbitals in a multi-electron atom.
Figure 6.3.8 illustrates the energy levels for various orbitals. The number before the orbital name (such as 2s, 3p, and so forth)
stands for the principal quantum number, n. The letter in the orbital name defines the subshell with a specific angular momentum
quantum number l = 0 for s orbitals, 1 for p orbitals, 2 for d orbitals. Finally, there are more than one possible orbitals for l ≥ 1,
each corresponding to a specific value of ml. In the case of a hydrogen atom or a one-electron ion (such as He+, Li2+, and so on),
energies of all the orbitals with the same n are the same. This is called a degeneracy, and the energy levels for the same principal
quantum number, n, are called degenerate energy levels. However, in atoms with more than one electron, this degeneracy is
eliminated by the electron–electron interactions, and orbitals that belong to different subshells have different energies. Orbitals
within the same subshell (for example ns, np, nd, nf, such as 2p, 3s) are still degenerate and have the same energy.
While the three quantum numbers discussed in the previous paragraphs work well for describing electron orbitals, some
experiments showed that they were not sufficient to explain all observed results. It was demonstrated in the 1920s that when
hydrogen-line spectra are examined at extremely high resolution, some lines are actually not single peaks but, rather, pairs of
closely spaced lines. This is the so-called fine structure of the spectrum, and it implies that there are additional small differences in
energies of electrons even when they are located in the same orbital. These observations led Samuel Goudsmit and George
Uhlenbeck to propose that electrons have a fourth quantum number. They called this the spin quantum number, or ms.
The other three quantum numbers, n, l, and ml, are properties of specific atomic orbitals that also define in what part of the space an
electron is most likely to be located. Orbitals are a result of solving the Schrödinger equation for electrons in atoms. The electron
spin is a different kind of property. It is a completely quantum phenomenon with no analogues in the classical realm. In addition, it
cannot be derived from solving the Schrödinger equation and is not related to the normal spatial coordinates (such as the Cartesian
x, y, and z). Electron spin describes an intrinsic electron “rotation” or “spinning.” Each electron acts as a tiny magnet or a tiny
rotating object with an angular momentum, even though this rotation cannot be observed in terms of the spatial coordinates.
The magnitude of the overall electron spin can only have one value, and an electron can only “spin” in one of two quantized states.
One is termed the α state, with the z component of the spin being in the positive direction of the z axis. This corresponds to the spin
1 1
quantum number ms = . The other is called the β state, with the z component of the spin being negative and ms = − . Any
2 2
electron, regardless of the atomic orbital it is located in, can only have one of those two values of the spin quantum number. The
1 1
energies of electrons having m s =− and m s = are different if an external magnetic field is applied.
2 2

Figure 6.3.9 : Electrons with spin values \(±\ce{1/2}\) in an external magnetic field.

Access for free at OpenStax 6.3.9 https://chem.libretexts.org/@go/page/38176


Figure 6.3.9 illustrates this phenomenon. An electron acts like a tiny magnet. Its moment is directed up (in the positive direction of
1
the z axis) for the spin quantum number and down (in the negative z direction) for the spin quantum number of − . A magnet 1

2
2
has a lower energy if its magnetic moment is aligned with the external magnetic field (the left electron) and a higher energy for the
1
magnetic moment being opposite to the applied field. This is why an electron with ms = has a slightly lower energy in an
2
1
external field in the positive z direction, and an electron with m s =− has a slightly higher energy in the same field. This is true
2
even for an electron occupying the same orbital in an atom. A spectral line corresponding to a transition for electrons from the
same orbital but with different spin quantum numbers has two possible values of energy; thus, the line in the spectrum will show a
fine structure splitting.

6.3.4: The Pauli Exclusion Principle


An electron in an atom is completely described by four quantum numbers: n, l, ml, and ms. The first three quantum numbers define
the orbital and the fourth quantum number describes the intrinsic electron property called spin. An Austrian physicist Wolfgang
Pauli formulated a general principle that gives the last piece of information that we need to understand the general behavior of
electrons in atoms. The Pauli exclusion principle can be formulated as follows: No two electrons in the same atom can have exactly
the same set of all the four quantum numbers. What this means is that electrons can share the same orbital (the same set of the
quantum numbers n, l, and ml), but only if their spin quantum numbers ms have different values. Since the spin quantum number
1
can only have two values (± ) , no more than two electrons can occupy the same orbital (and if two electrons are located in the
2

same orbital, they must have opposite spins). Therefore, any atomic orbital can be populated by only zero, one, or two electrons.
The properties and meaning of the quantum numbers of electrons in atoms are briefly summarized in Table 6.3.1.
Table 6.3.1 : Quantum Numbers, Their Properties, and Significance
Name Symbol Allowed values Physical meaning

shell, the general region for the


principal quantum number n 1, 2, 3, 4, …. value of energy for an electron on
the orbital

angular momentum or azimuthal


l 0≤l≤n–1 subshell, the shape of the orbital
quantum number

magnetic quantum number ml – l ≤ ml ≤ l orientation of the orbital

1 1 direction of the intrinsic quantum


spin quantum number ms , −
2 2 “spinning” of the electron

 Example 6.3.2: Working with Shells and Subshells

Indicate the number of subshells, the number of orbitals in each subshell, and the values of l and ml for the orbitals in the n = 4
shell of an atom.

Solution
For n = 4, l can have values of 0, 1, 2, and 3. Thus, s, p, d, and f subshells are found in the n = 4 shell of an atom. For l = 0 (the
s subshell), ml can only be 0. Thus, there is only one 4s orbital. For l = 1 (p-type orbitals), m can have values of –1, 0, +1, so
we find three 4p orbitals. For l = 2 (d-type orbitals), ml can have values of –2, –1, 0, +1, +2, so we have five 4d orbitals. When
l = 3 (f-type orbitals), ml can have values of –3, –2, –1, 0, +1, +2, +3, and we can have seven 4f orbitals. Thus, we find a total
of 16 orbitals in the n = 4 shell of an atom.

Access for free at OpenStax 6.3.10 https://chem.libretexts.org/@go/page/38176


 Exercise 6.3.2

Identify the subshell in which electrons with the following quantum numbers are found:
a. n = 3, l = 1;
b. n = 5, l = 3;
c. n = 2, l = 0.

Answer a
3p
Answer b
5f
Answer c
2s

 Example 6.3.3: Maximum Number of Electrons

Calculate the maximum number of electrons that can occupy a shell with (a) n = 2, (b) n = 5, and (c) n as a variable. Note you
are only looking at the orbitals with the specified n value, not those at lower energies.

Solution
(a) When n = 2, there are four orbitals (a single 2s orbital, and three orbitals labeled 2p). These four orbitals can contain eight
electrons.
(b) When n = 5, there are five subshells of orbitals that we need to sum:
1 orbital labeled 5s

3 orbitals labeled 5p

5 orbitals labeled 5d

7 orbitals labeled 5f

+9 orbitals labeled 5g
–––––––––––––––––––––
25 orbitals total

Again, each orbital holds two electrons, so 50 electrons can fit in this shell.
(c) The number of orbitals in any shell n will equal n2. There can be up to two electrons in each orbital, so the maximum number of
electrons will be 2 × n2

 Exercise 6.3.3

If a shell contains a maximum of 32 electrons, what is the principal quantum number, n?

Answer
n=4

 Example 6.3.4: Working with Quantum Numbers


Complete the following table for atomic orbitals:
Table for Atomic Orbitals
Orbital n l ml degeneracy Radial nodes (no.)

4f

Access for free at OpenStax 6.3.11 https://chem.libretexts.org/@go/page/38176


Orbital n l ml degeneracy Radial nodes (no.)

4 1

7 7 3

5d

Solution
The table can be completed using the following rules:
The orbital designation is nl, where l = 0, 1, 2, 3, 4, 5, … is mapped to the letter sequence s, p, d, f, g, h, …,
The ml degeneracy is the number of orbitals within an l subshell, and so is 2l + 1 (there is one s orbital, three p orbitals, five
d orbitals, seven f orbitals, and so forth).
The number of radial nodes is equal to n – l – 1.
Solution to Example 6.3.4
Orbital n l ml degeneracy Radial nodes (no.)

4f 4 3 7 0

4p 4 1 3 2

7f 7 3 7 3

5d 5 2 5 2

 Exercise 6.3.4

How many orbitals have l = 2 and n = 3?

Answer
The five degenerate 3d orbitals

Summary
Macroscopic objects act as particles. Microscopic objects (such as electrons) have properties of both a particle and a wave. Their
exact trajectories cannot be determined. The quantum mechanical model of atoms describes the three-dimensional position of the
electron in a probabilistic manner according to a mathematical function called a wavefunction, often denoted as ψ. Atomic
wavefunctions are also called orbitals. The squared magnitude of the wavefunction describes the distribution of the probability of
finding the electron in a particular region in space. Therefore, atomic orbitals describe the areas in an atom where electrons are
most likely to be found.
An atomic orbital is characterized by three quantum numbers. The principal quantum number, n, can be any positive integer. The
general region for value of energy of the orbital and the average distance of an electron from the nucleus are related to n. Orbitals
having the same value of n are said to be in the same shell. The angular momentum quantum number, l, can have any integer value
from 0 to n – 1. This quantum number describes the shape or type of the orbital. Orbitals with the same principal quantum number
and the same l value belong to the same subshell. The magnetic quantum number, ml, with 2l + 1 values ranging from –l to +l,
1
describes the orientation of the orbital in space. In addition, each electron has a spin quantum number, ms, that can be equal to ± .
2
No two electrons in the same atom can have the same set of values for all the four quantum numbers.

Glossary
angular momentum quantum number (l)
quantum number distinguishing the different shapes of orbitals; it is also a measure of the orbital angular momentum

Access for free at OpenStax 6.3.12 https://chem.libretexts.org/@go/page/38176


atomic orbital
mathematical function that describes the behavior of an electron in an atom (also called the wavefunction), it can be used to find
the probability of locating an electron in a specific region around the nucleus, as well as other dynamical variables

d orbital
region of space with high electron density that is either four lobed or contains a dumbbell and torus shape; describes orbitals
with l = 2. An electron in this orbital is called a d electron

electron density
a measure of the probability of locating an electron in a particular region of space, it is equal to the squared absolute value of
the wave function ψ

f orbital
multilobed region of space with high electron density, describes orbitals with l = 3. An electron in this orbital is called an f
electron

Heisenberg uncertainty principle


rule stating that it is impossible to exactly determine both certain conjugate dynamical properties such as the momentum and the
position of a particle at the same time. The uncertainty principle is a consequence of quantum particles exhibiting wave–particle
duality

magnetic quantum number (ml)


quantum number signifying the orientation of an atomic orbital around the nucleus; orbitals having different values of ml but the
same subshell value of l have the same energy (are degenerate), but this degeneracy can be removed by application of an
external magnetic field

p orbital
dumbbell-shaped region of space with high electron density, describes orbitals with l = 1. An electron in this orbital is called a p
electron

Pauli exclusion principle


specifies that no two electrons in an atom can have the same value for all four quantum numbers

principal quantum number (n)


quantum number specifying the shell an electron occupies in an atom

quantum mechanics
field of study that includes quantization of energy, wave-particle duality, and the Heisenberg uncertainty principle to describe
matter

s orbital
spherical region of space with high electron density, describes orbitals with l = 0. An electron in this orbital is called an s
electron

shell
set of orbitals with the same principal quantum number, n

spin quantum number (ms)


1 1
number specifying the electron spin direction, either + or −
2 2

subshell
set of orbitals in an atom with the same values of n and l

wavefunction (ψ)

Access for free at OpenStax 6.3.13 https://chem.libretexts.org/@go/page/38176


mathematical description of an atomic orbital that describes the shape of the orbital; it can be used to calculate the probability of
finding the electron at any given location in the orbital, as well as dynamical variables such as the energy and the angular
momentum

This page titled 6.3: Development of Quantum Theory is shared under a CC BY 4.0 license and was authored, remixed, and/or curated by
OpenStax via source content that was edited to the style and standards of the LibreTexts platform; a detailed edit history is available upon request.

Access for free at OpenStax 6.3.14 https://chem.libretexts.org/@go/page/38176


6.4: Electronic Structure of Atoms (Electron Configurations)
 Learning Objectives
Derive the predicted ground-state electron configurations of atoms
Identify and explain exceptions to predicted electron configurations for atoms and ions
Relate electron configurations to element classifications in the periodic table

Having introduced the basics of atomic structure and quantum mechanics, we can use our understanding of quantum numbers to
determine how atomic orbitals relate to one another. This allows us to determine which orbitals are occupied by electrons in each
atom. The specific arrangement of electrons in orbitals of an atom determines many of the chemical properties of that atom.

6.4.1: Orbital Energies and Atomic Structure


The energy of atomic orbitals increases as the principal quantum number, n , increases. In any atom with two or more electrons, the
repulsion between the electrons makes energies of subshells with different values of l differ so that the energy of the orbitals
increases within a shell in the order s < p < d < f. Figure 6.4.1 depicts how these two trends in increasing energy relate. The 1s
orbital at the bottom of the diagram is the orbital with electrons of lowest energy. The energy increases as we move up to the 2s and
then 2p, 3s, and 3p orbitals, showing that the increasing n value has more influence on energy than the increasing l value for small
atoms. However, this pattern does not hold for larger atoms. The 3d orbital is higher in energy than the 4s orbital. Such overlaps
continue to occur frequently as we move up the chart.

Figure 6.4.1 : Generalized energy-level diagram for atomic orbitals in an atom with two or more electrons (not to scale).
Electrons in successive atoms on the periodic table tend to fill low-energy orbitals first. Thus, many students find it confusing that,
for example, the 5p orbitals fill immediately after the 4d, and immediately before the 6s. The filling order is based on observed
experimental results, and has been confirmed by theoretical calculations. As the principal quantum number, n, increases, the size of
the orbital increases and the electrons spend more time farther from the nucleus. Thus, the attraction to the nucleus is weaker and
the energy associated with the orbital is higher (less stabilized). But this is not the only effect we have to take into account. Within
each shell, as the value of l increases, the electrons are less penetrating (meaning there is less electron density found close to the
nucleus), in the order s > p > d > f. Electrons that are closer to the nucleus slightly repel electrons that are farther out, offsetting the
more dominant electron–nucleus attractions slightly (recall that all electrons have −1 charges, but nuclei have +Z charges). This
phenomenon is called shielding and will be discussed in more detail in the next section. Electrons in orbitals that experience more
shielding are less stabilized and thus higher in energy. For small orbitals (1s through 3p), the increase in energy due to n is more
significant than the increase due to l; however, for larger orbitals the two trends are comparable and cannot be simply predicted. We
will discuss methods for remembering the observed order.
The arrangement of electrons in the orbitals of an atom is called the electron configuration of the atom. We describe an electron
configuration with a symbol that contains three pieces of information ( Figure 6.4.2):
1. The number of the principal quantum shell, n,

Access for free at OpenStax 6.4.1 https://chem.libretexts.org/@go/page/38177


2. The letter that designates the orbital type (the subshell, l), and
3. A superscript number that designates the number of electrons in that particular subshell.
For example, the notation 2p4 (read "two–p–four") indicates four electrons in a p subshell (l = 1) with a principal quantum number
(n) of 2. The notation 3d8 (read "three–d–eight") indicates eight electrons in the d subshell (i.e., l = 2) of the principal shell for
which n = 3.

Figure 6.4.2 : The diagram of an electron configuration specifies the subshell (n and l value, with letter symbol) and superscript
number of electrons.

6.4.2: The Aufbau Principle


To determine the electron configuration for any particular atom, we can “build” the structures in the order of atomic numbers.
Beginning with hydrogen, and continuing across the periods of the periodic table, we add one proton at a time to the nucleus and
one electron to the proper subshell until we have described the electron configurations of all the elements. This procedure is called
the Aufbau principle, from the German word Aufbau (“to build up”). Each added electron occupies the subshell of lowest energy
available (in the order shown in Figure 6.4.3), subject to the limitations imposed by the allowed quantum numbers according to the
Pauli exclusion principle. Electrons enter higher-energy subshells only after lower-energy subshells have been filled to capacity.
Figure 6.4.3 illustrates the traditional way to remember the filling order for atomic orbitals.

Figure 6.4.3 : The arrow leads through each subshell in the appropriate filling order for electron configurations. This chart is
straightforward to construct. Simply make a column for all the s orbitals with each n shell on a separate row. Repeat for p, d, and f.
Be sure to only include orbitals allowed by the quantum numbers (no 1p or 2d, and so forth). Finally, draw diagonal lines from top
to bottom as shown.
Since the arrangement of the periodic table is based on the electron configurations, Figure 6.4.4 provides an alternative method for
determining the electron configuration. The filling order simply begins at hydrogen and includes each subshell as you proceed in
increasing Z order. For example, after filling the 3p block up to Ar, we see the orbital will be 4s (K, Ca), followed by the 3d
orbitals.

Access for free at OpenStax 6.4.2 https://chem.libretexts.org/@go/page/38177


Figure 6.4.4 : This periodic table shows the electron configuration for each subshell. By “building up” from hydrogen, this table
can be used to determine the electron configuration for any atom on the periodic table.

We will now construct the ground-state electron configuration and orbital diagram for a selection of atoms in the first and second
periods of the periodic table. Orbital diagrams are pictorial representations of the electron configuration, showing the individual
orbitals and the pairing arrangement of electrons. We start with a single hydrogen atom (atomic number 1), which consists of one
proton and one electron. Referring to either Figure 6.4.3 or 6.4.4, we would expect to find the electron in the 1s orbital. By
1
convention, the m s =+ value is usually filled first. The electron configuration and the orbital diagram are:
2

Following hydrogen is the noble gas helium, which has an atomic number of 2. The helium atom contains two protons and two
1
electrons. The first electron has the same four quantum numbers as the hydrogen atom electron (n = 1, l = 0, ml = 0, ms = + ).
2
The second electron also goes into the 1s orbital and fills that orbital. The second electron has the same n, l, and ml quantum
1
numbers, but must have the opposite spin quantum number, ms = − . This is in accord with the Pauli exclusion principle: No
2
two electrons in the same atom can have the same set of four quantum numbers. For orbital diagrams, this means two arrows go in
each box (representing two electrons in each orbital) and the arrows must point in opposite directions (representing paired spins).
The electron configuration and orbital diagram of helium are:

The n = 1 shell is completely filled in a helium atom.

Access for free at OpenStax 6.4.3 https://chem.libretexts.org/@go/page/38177


The next atom is the alkali metal lithium with an atomic number of 3. The first two electrons in lithium fill the 1s orbital and have
the same sets of four quantum numbers as the two electrons in helium. The remaining electron must occupy the orbital of next
lowest energy, the 2s orbital (Figure 6.4.3 or 6.4.4). Thus, the electron configuration and orbital diagram of lithium are:

An atom of the alkaline earth metal beryllium, with an atomic number of 4, contains four protons in the nucleus and four electrons
surrounding the nucleus. The fourth electron fills the remaining space in the 2s orbital.

An atom of boron (atomic number 5) contains five electrons. The n = 1 shell is filled with two electrons and three electrons will
occupy the n = 2 shell. Because any s subshell can contain only two electrons, the fifth electron must occupy the next energy level,
which will be a 2p orbital. There are three degenerate 2p orbitals (ml = −1, 0, +1) and the electron can occupy any one of these p
orbitals. When drawing orbital diagrams, we include empty boxes to depict any empty orbitals in the same subshell that we are
filling.

Carbon (atomic number 6) has six electrons. Four of them fill the 1s and 2s orbitals. The remaining two electrons occupy the 2p
subshell. We now have a choice of filling one of the 2p orbitals and pairing the electrons or of leaving the electrons unpaired in two
different, but degenerate, p orbitals. The orbitals are filled as described by Hund’s rule: the lowest-energy configuration for an atom
with electrons within a set of degenerate orbitals is that having the maximum number of unpaired electrons. Thus, the two electrons
in the carbon 2p orbitals have identical n, l, and ms quantum numbers and differ in their ml quantum number (in accord with the
Pauli exclusion principle). The electron configuration and orbital diagram for carbon are:

Nitrogen (atomic number 7) fills the 1s and 2s subshells and has one electron in each of the three 2p orbitals, in accordance with
Hund’s rule. These three electrons have unpaired spins. Oxygen (atomic number 8) has a pair of electrons in any one of the 2p
orbitals (the electrons have opposite spins) and a single electron in each of the other two. Fluorine (atomic number 9) has only one
2p orbital containing an unpaired electron. All of the electrons in the noble gas neon (atomic number 10) are paired, and all of the
orbitals in the n = 1 and the n = 2 shells are filled. The electron configurations and orbital diagrams of these four elements are:

Access for free at OpenStax 6.4.4 https://chem.libretexts.org/@go/page/38177


Figure 6.4.5 : Since the core electron shells correspond to noble gas electron configurations, we can abbreviate electron
configurations by writing the noble gas that matches the core electron configuration, along with the valence electrons in a
condensed format. For our sodium example, the symbol [Ne] represents core electrons, (1s22s22p6) and our abbreviated or
condensed configuration is [Ne]3s1.
The alkali metal sodium (atomic number 11) has one more electron than the neon atom. This electron must go into the lowest-
energy subshell available, the 3s orbital, giving a 1s22s22p63s1 configuration. The electrons occupying the outermost shell orbital(s)
(highest value of n) are called valence electrons, and those occupying the inner shell orbitals are called core electrons ( Figure
\PageIndex5\PageIndex5). Since the core electron shells correspond to noble gas electron configurations, we can abbreviate
electron configurations by writing the noble gas that matches the core electron configuration, along with the valence electrons in a
condensed format. For our sodium example, the symbol [Ne] represents core electrons, (1s22s22p6) and our abbreviated or
condensed configuration is [Ne]3s1.

Figure 6.4.5 : A core-abbreviated electron configuration (right) replaces the core electrons with the noble gas symbol whose
configuration matches the core electron configuration of the other element.

Similarly, the abbreviated configuration of lithium can be represented as [He]2s1, where [He] represents the configuration of the
helium atom, which is identical to that of the filled inner shell of lithium. Writing the configurations in this way emphasizes the
similarity of the configurations of lithium and sodium. Both atoms, which are in the alkali metal family, have only one electron in a
valence s subshell outside a filled set of inner shells.
1
Li : [He] 2s

1
Na : [Ne] 3s

The alkaline earth metal magnesium (atomic number 12), with its 12 electrons in a [Ne]3s2 configuration, is analogous to its family
member beryllium, [He]2s2. Both atoms have a filled s subshell outside their filled inner shells. Aluminum (atomic number 13),
with 13 electrons and the electron configuration [Ne]3s23p1, is analogous to its family member boron, [He]2s22p1.

Access for free at OpenStax 6.4.5 https://chem.libretexts.org/@go/page/38177


The electron configurations of silicon (14 electrons), phosphorus (15 electrons), sulfur (16 electrons), chlorine (17 electrons), and
argon (18 electrons) are analogous in the electron configurations of their outer shells to their corresponding family members
carbon, nitrogen, oxygen, fluorine, and neon, respectively, except that the principal quantum number of the outer shell of the
heavier elements has increased by one to n = 3. Figure 6.4.6 shows the lowest energy, or ground-state, electron configuration for
these elements as well as that for atoms of each of the known elements.

Figure 6.4.6 : This version of the periodic table shows the outer-shell electron configuration of each element. Note that down each
group, the configuration is often similar.
When we come to the next element in the periodic table, the alkali metal potassium (atomic number 19), we might expect that we
would begin to add electrons to the 3d subshell. However, all available chemical and physical evidence indicates that potassium is
like lithium and sodium, and that the next electron is not added to the 3d level but is, instead, added to the 4s level (Figure 6.4.3 or
6.4.4). As discussed previously, the 3d orbital with no radial nodes is higher in energy because it is less penetrating and more

shielded from the nucleus than the 4s, which has three radial nodes. Thus, potassium has an electron configuration of [Ar]4s1.
Hence, potassium corresponds to Li and Na in its valence shell configuration. The next electron is added to complete the 4s
subshell and calcium has an electron configuration of [Ar]4s2. This gives calcium an outer-shell electron configuration
corresponding to that of beryllium and magnesium.
Beginning with the transition metal scandium (atomic number 21), additional electrons are added successively to the 3d subshell.
This subshell is filled to its capacity with 10 electrons (remember that for l = 2 [d orbitals], there are 2l + 1 = 5 values of ml,
meaning that there are five d orbitals that have a combined capacity of 10 electrons). The 4p subshell fills next. Note that for three
series of elements, scandium (Sc) through copper (Cu), yttrium (Y) through silver (Ag), and lutetium (Lu) through gold (Au), a
total of 10 d electrons are successively added to the (n – 1) shell next to the n shell to bring that (n – 1) shell from 8 to 18 electrons.
For two series, lanthanum (La) through lutetium (Lu) and actinium (Ac) through lawrencium (Lr), 14 f electrons (l = 3, 2l + 1 = 7
ml values; thus, seven orbitals with a combined capacity of 14 electrons) are successively added to the (n – 2) shell to bring that
shell from 18 electrons to a total of 32 electrons.

Access for free at OpenStax 6.4.6 https://chem.libretexts.org/@go/page/38177


 Example 6.4.1: Quantum Numbers and Electron Configurations
What is the electron configuration and orbital diagram for a phosphorus atom? What are the four quantum numbers for the last
electron added?
Solution
The atomic number of phosphorus is 15. Thus, a phosphorus atom contains 15 electrons. The order of filling of the energy
levels is 1s, 2s, 2p, 3s, 3p, 4s, . . . The 15 electrons of the phosphorus atom will fill up to the 3p orbital, which will contain
three electrons:

The last electron added is a 3p electron. Therefore, n = 3 and, for a p-type orbital, l = 1. The ml value could be –1, 0, or +1. The
1
three p orbitals are degenerate, so any of these ml values is correct. For unpaired electrons, convention assigns the value of +
2
1
for the spin quantum number; thus, m s =+ .
2

 Exercise 6.4.1

Identify the atoms from the electron configurations given:


a. [Ar]4s23d5
b. [Kr]5s24d105p6

Answer a
Mn
Answer b
Xe

The periodic table can be a powerful tool in predicting the electron configuration of an element. However, we do find exceptions to
the order of filling of orbitals that are shown in Figure 6.4.3 or 6.4.4. For instance, the electron configurations of the transition
metals chromium (Cr; atomic number 24) and copper (Cu; atomic number 29), among others, are not those we would expect. In
general, such exceptions involve subshells with very similar energy, and small effects can lead to changes in the order of filling.
In the case of Cr and Cu, we find that half-filled and completely filled subshells apparently represent conditions of preferred
stability. This stability is such that an electron shifts from the 4s into the 3d orbital to gain the extra stability of a half-filled 3d
subshell (in Cr) or a filled 3d subshell (in Cu). Other exceptions also occur. For example, niobium (Nb, atomic number 41) is
predicted to have the electron configuration [Kr]5s24d3. Experimentally, we observe that its ground-state electron configuration is
actually [Kr]5s14d4. We can rationalize this observation by saying that the electron–electron repulsions experienced by pairing the
electrons in the 5s orbital are larger than the gap in energy between the 5s and 4d orbitals. There is no simple method to predict the
exceptions for atoms where the magnitude of the repulsions between electrons is greater than the small differences in energy
between subshells.

6.4.3: Electron Configurations and the Periodic Table


As described earlier, the periodic table arranges atoms based on increasing atomic number so that elements with the same chemical
properties recur periodically. When their electron configurations are added to the table (Figure 6.4.6), we also see a periodic
recurrence of similar electron configurations in the outer shells of these elements. Because they are in the outer shells of an atom,
valence electrons play the most important role in chemical reactions. The outer electrons have the highest energy of the electrons in

Access for free at OpenStax 6.4.7 https://chem.libretexts.org/@go/page/38177


an atom and are more easily lost or shared than the core electrons. Valence electrons are also the determining factor in some
physical properties of the elements.
Elements in any one group (or column) have the same number of valence electrons; the alkali metals lithium and sodium each have
only one valence electron, the alkaline earth metals beryllium and magnesium each have two, and the halogens fluorine and
chlorine each have seven valence electrons. The similarity in chemical properties among elements of the same group occurs
because they have the same number of valence electrons. It is the loss, gain, or sharing of valence electrons that defines how
elements react.
It is important to remember that the periodic table was developed on the basis of the chemical behavior of the elements, well before
any idea of their atomic structure was available. Now we can understand why the periodic table has the arrangement it has—the
arrangement puts elements whose atoms have the same number of valence electrons in the same group. This arrangement is
emphasized in Figure 6.4.6, which shows in periodic-table form the electron configuration of the last subshell to be filled by the
Aufbau principle. The colored sections of Figure 6.4.6 show the three categories of elements classified by the orbitals being filled:
main group, transition, and inner transition elements. These classifications determine which orbitals are counted in the valence
shell, or highest energy level orbitals of an atom.
1. Main group elements (sometimes called representative elements) are those in which the last electron added enters an s or a p
orbital in the outermost shell, shown in blue and red in Figure 6.4.6. This category includes all the nonmetallic elements, as
well as many metals and the intermediate semimetallic elements. The valence electrons for main group elements are those with
the highest n level. For example, gallium (Ga, atomic number 31) has the electron configuration [Ar]4s23d104p1, which contains
three valence electrons (underlined). The completely filled d orbitals count as core, not valence, electrons.
2. Transition elements or transition metals. These are metallic elements in which the last electron added enters a d orbital. The
valence electrons (those added after the last noble gas configuration) in these elements include the ns and (n – 1) d electrons.
The official IUPAC definition of transition elements specifies those with partially filled d orbitals. Thus, the elements with
completely filled orbitals (Zn, Cd, Hg, as well as Cu, Ag, and Au in Figure 6.4.6) are not technically transition elements.
However, the term is frequently used to refer to the entire d block (colored yellow in Figure 6.4.6), and we will adopt this usage
in this textbook.
3. Inner transition elements are metallic elements in which the last electron added occupies an f orbital. They are shown in green
in Figure 6.4.6. The valence shells of the inner transition elements consist of the (n – 2)f, the (n – 1)d, and the ns subshells.
There are two inner transition series:
1. The lanthanide series: lanthanide (La) through lutetium (Lu)
2. The actinide series: actinide (Ac) through lawrencium (Lr)
Lanthanum and actinium, because of their similarities to the other members of the series, are included and used to name the series,
even though they are transition metals with no f electrons.

6.4.4: Electron Configurations of Ions


We have seen that ions are formed when atoms gain or lose electrons. A cation (positively charged ion) forms when one or more
electrons are removed from a parent atom. For main group elements, the electrons that were added last are the first electrons
removed. For transition metals and inner transition metals, however, electrons in the s orbital are easier to remove than the d or f
electrons, and so the highest ns electrons are lost, and then the (n – 1)d or (n – 2)f electrons are removed. An anion (negatively
charged ion) forms when one or more electrons are added to a parent atom. The added electrons fill in the order predicted by the
Aufbau principle.

 Example 6.4.2: Predicting Electron Configurations of Ions


What is the electron configuration and orbital diagram of:
a. Na+
b. P3–
c. Al2+
d. Fe2+
e. Sm3+
Solution

Access for free at OpenStax 6.4.8 https://chem.libretexts.org/@go/page/38177


First, write out the electron configuration for each parent atom. We have chosen to show the full, unabbreviated configurations
to provide more practice for students who want it, but listing the core-abbreviated electron configurations is also acceptable.
Next, determine whether an electron is gained or lost. Remember electrons are negatively charged, so ions with a positive
charge have lost an electron. For main group elements, the last orbital gains or loses the electron. For transition metals, the last
s orbital loses an electron before the d orbitals.
a. Na: 1s22s22p63s1. Sodium cation loses one electron, so Na+: 1s22s22p63s1 = Na+: 1s22s22p6.
b. P: 1s22s22p63s23p3. Phosphorus trianion gains three electrons, so P3−: 1s22s22p63s23p6.
c. Al: 1s22s22p63s23p1. Aluminum dication loses two electrons Al2+: 1s22s22p63s23p1 = Al2+: 1s22s22p63s1.
d. Fe: 1s22s22p63s23p64s23d6. Iron(II) loses two electrons and, since it is a transition metal, they are removed from the 4s
orbital Fe2+: 1s22s22p63s23p64s23d6 = 1s22s22p63s23p63d6.
e. Sm: 1s22s22p63s23p64s23d104p65s24d105p66s24f6. Samarium trication loses three electrons. The first two will be lost from
the 6s orbital, and the final one is removed from the 4f orbital. Sm3+: 1s22s22p63s23p64s23d104p65s24d105p66s24f6 =
1s22s22p63s23p64s23d104p65s24d105p64f5.

 Exercise 6.4.2
a. Which ion with a +2 charge has the electron configuration 1s22s22p63s23p63d104s24p64d5?
b. Which ion with a +3 charge has this configuration?

Answer a
Tc2+
Answer b
Ru3+

Summary
The relative energy of the subshells determine the order in which atomic orbitals are filled (1s, 2s, 2p, 3s, 3p, 4s, 3d, 4p, and so on).
Electron configurations and orbital diagrams can be determined by applying the Pauli exclusion principle (no two electrons can
have the same set of four quantum numbers) and Hund’s rule (whenever possible, electrons retain unpaired spins in degenerate
orbitals).
Electrons in the outermost orbitals, called valence electrons, are responsible for most of the chemical behavior of elements. In the
periodic table, elements with analogous valence electron configurations usually occur within the same group. There are some
exceptions to the predicted filling order, particularly when half-filled or completely filled orbitals can be formed. The periodic table
can be divided into three categories based on the orbital in which the last electron to be added is placed: main group elements (s
and p orbitals), transition elements (d orbitals), and inner transition elements (f orbitals).

Glossary
Aufbau principle
procedure in which the electron configuration of the elements is determined by “building” them in order of atomic numbers,
adding one proton to the nucleus and one electron to the proper subshell at a time

core electron
electron in an atom that occupies the orbitals of the inner shells

electron configuration
electronic structure of an atom in its ground state given as a listing of the orbitals occupied by the electrons

Hund’s rule

Access for free at OpenStax 6.4.9 https://chem.libretexts.org/@go/page/38177


every orbital in a subshell is singly occupied with one electron before any one orbital is doubly occupied, and all electrons in
singly occupied orbitals have the same spin

orbital diagram
pictorial representation of the electron configuration showing each orbital as a box and each electron as an arrow

valence electrons
electrons in the outermost or valence shell (highest value of n) of a ground-state atom; determine how an element reacts

valence shell
outermost shell of electrons in a ground-state atom; for main group elements, the orbitals with the highest n level (s and p
subshells) are in the valence shell, while for transition metals, the highest energy s and d subshells make up the valence shell
and for inner transition elements, the highest s, d, and f subshells are included

This page titled 6.4: Electronic Structure of Atoms (Electron Configurations) is shared under a CC BY 4.0 license and was authored, remixed,
and/or curated by OpenStax via source content that was edited to the style and standards of the LibreTexts platform; a detailed edit history is
available upon request.

Access for free at OpenStax 6.4.10 https://chem.libretexts.org/@go/page/38177


6.5: Periodic Variations in Element Properties
 Learning Objectives
Describe and explain the observed trends in atomic size, ionization energy, and electron affinity of the elements

The elements in groups (vertical columns) of the periodic table exhibit similar chemical behavior. This similarity occurs because
the members of a group have the same number and distribution of electrons in their valence shells. However, there are also other
patterns in chemical properties on the periodic table. For example, as we move down a group, the metallic character of the atoms
increases. Oxygen, at the top of Group 16 (6A), is a colorless gas; in the middle of the group, selenium is a semiconducting solid;
and, toward the bottom, polonium is a silver-grey solid that conducts electricity.
As we go across a period from left to right, we add a proton to the nucleus and an electron to the valence shell with each successive
element. As we go down the elements in a group, the number of electrons in the valence shell remains constant, but the principal
quantum number increases by one each time. An understanding of the electronic structure of the elements allows us to examine
some of the properties that govern their chemical behavior. These properties vary periodically as the electronic structure of the
elements changes. They are (1) size (radius) of atoms and ions, (2) ionization energies, and (3) electron affinities.

6.5.1: Variation in Covalent Radius


The quantum mechanical picture makes it difficult to establish a definite size of an atom. However, there are several practical ways
to define the radius of atoms and, thus, to determine their relative sizes that give roughly similar values. We will use the covalent
radius (Figure 6.5.1), which is defined as one-half the distance between the nuclei of two identical atoms when they are joined by a
covalent bond (this measurement is possible because atoms within molecules still retain much of their atomic identity).

Access for free at OpenStax 6.5.1 https://chem.libretexts.org/@go/page/38178


Figure 6.5.1 : (a) The radius of an atom is defined as one-half the distance between the nuclei in a molecule consisting of two
identical atoms joined by a covalent bond. The atomic radius for the halogens increases down the group as n increases. (b)
Covalent radii of the elements are shown to scale. The general trend is that radii increase down a group and decrease across a
period.
This figure has two parts: a and b. In figure a, 4 diatomic molecules are shown to illustrate the method of determining the atomic
radius of an atom. The first model, in light green, is used to find the F atom radius. Two spheres are pushed very tightly together.
The distance between the centers of the two atoms is indicated above the diagram with a double headed arrow labeled, “128 p m.”
The endpoints of this arrow connect to line segments that extend to the atomic radii below. Beneath the molecule is the label, “F
radius equals 128 p m divided by 2 equals 64 p m.” The next three models are similarly used to show the atomic radii of additional
atoms. The second diatomic molecule is in a darker shade of green. The distance between the radii is 198 p m. Beneath the
molecule is the label, “C l radius equals 198 p m divided by 2 equals 99 pm.” The third diatomic molecule is in red. The distance
between the radii is 228 p m. Beneath the molecule is the label, “B r radius equals 228 p m divided by 2 equals 114 pm.” The
fourth diatomic molecule is in purple. The distance between the radii is 266 p m. Beneath the molecule is the label, “I radius equals
266 p m divided by 2 equals 133 p m.” In figure b, a periodic table layout is used to compare relative sizes of atoms using green
spheres. No spheres are provided for the noble or inert gas, group 18 elements. General trends noted are increasing circle size
moving from top to bottom in a group, with a general tendency toward increasing atomic radii toward the lower left corner of the
periodic table.
We know that as we scan down a group, the principal quantum number, n, increases by one for each element. Thus, the electrons
are being added to a region of space that is increasingly distant from the nucleus. Consequently, the size of the atom (and its
covalent radius) must increase as we increase the distance of the outermost electrons from the nucleus. This trend is illustrated for
the covalent radii of the halogens in Table 6.5.1 and Figure 6.5.1. The trends for the entire periodic table can be seen in Figure
6.5.2.

Table 6.5.1 : Covalent Radii of the Halogen Group Elements


Atom Covalent radius (pm) Nuclear charge

F 64 +9

Cl 99 +17

Br 114 +35

I 133 +53

At 148 +85

Access for free at OpenStax 6.5.2 https://chem.libretexts.org/@go/page/38178


As shown in Figure 6.5.2, as we move across a period from left to right, we generally find that each element has a smaller covalent
radius than the element preceding it. This might seem counterintuitive because it implies that atoms with more electrons have a
smaller atomic radius. This can be explained with the concept of effective nuclear charge, Z . This is the pull exerted on a
ef f

specific electron by the nucleus, taking into account any electron–electron repulsions. For hydrogen, there is only one electron and
so the nuclear charge (Z) and the effective nuclear charge (Zeff) are equal. For all other atoms, the inner electrons partially shield the
outer electrons from the pull of the nucleus, and thus:

Zeff = Z − shielding

Shielding is determined by the probability of another electron being between the electron of interest and the nucleus, as well as by
the electron–electron repulsions the electron of interest encounters. Core electrons are adept at shielding, while electrons in the
same valence shell do not block the nuclear attraction experienced by each other as efficiently. Thus, each time we move from one
element to the next across a period, Z increases by one, but the shielding increases only slightly. Thus, Zeff increases as we move
from left to right across a period. The stronger pull (higher effective nuclear charge) experienced by electrons on the right side of
the periodic table draws them closer to the nucleus, making the covalent radii smaller.

Figure 6.5.2 : Within each period, the trend in atomic radius decreases as Z increases; for example, from K to Kr. Within each group
(e.g., the alkali metals shown in purple), the trend is that atomic radius increases as Z increases.
This graph is labeled Atomic Number on the horizontal axis and Radius (p m) on the vertical axis. A black jagged line connects the
radii values for elements with atomic numbers 1 through 60 on the graph. Peaks are evident at the locations of the alkali metals: L i,
N a, K, R b, and C s. Similarly, minima exist at the locations of noble or inert gases: H e, N e, A r, K r, X e, and R n. The locations
of period 4 and period 5 transition elements are provided with green dots.
Thus, as we would expect, the outermost or valence electrons are easiest to remove because they have the highest energies, are
shielded more, and are farthest from the nucleus. As a general rule, when the representative elements form cations, they do so by
the loss of the ns or np electrons that were added last in the Aufbau process. The transition elements, on the other hand, lose the ns
electrons before they begin to lose the (n – 1)d electrons, even though the ns electrons are added first, according to the Aufbau
principle.

 Example 6.5.1: Sorting Atomic Radii

Predict the order of increasing covalent radius for Ge, Fl, Br, Kr.

Solution

Access for free at OpenStax 6.5.3 https://chem.libretexts.org/@go/page/38178


Radius increases as we move down a group, so Ge < Fl (Note: Fl is the symbol for flerovium, element 114, NOT fluorine).
Radius decreases as we move across a period, so Kr < Br < Ge. Putting the trends together, we obtain Kr < Br < Ge < Fl.

 Exercise 6.5.1

Give an example of an atom whose size is smaller than fluorine.

Answer
Ne or He

6.5.2: Variation in Ionic Radii


Ionic radius is the measure used to describe the size of an ion. A cation always has fewer electrons and the same number of protons
as the parent atom; it is smaller than the atom from which it is derived (Figure 6.5.3). For example, the covalent radius of an
aluminum atom (1s22s22p63s23p1) is 118 pm, whereas the ionic radius of an Al3+ (1s22s22p6) is 68 pm. As electrons are removed
from the outer valence shell, the remaining core electrons occupying smaller shells experience a greater effective nuclear charge
Zeff (as discussed) and are drawn even closer to the nucleus.

Figure 6.5.3 : The radius for a cation is smaller than the parent atom (Al), due to the lost electrons; the radius for an anion is larger
than the parent (S), due to the gained electrons.
The figure includes spheres in green to represent the relative sizes of A l and S atoms. Beneath each of these spheres is a red
sphere. The red sphere represents the ionic states of aluminum and sulfur respectively. While radius for the aluminum cation is
smaller than aluminum, the sulfur anion has a greater radius than sulfur.
Cations with larger charges are smaller than cations with smaller charges (e.g., V2+ has an ionic radius of 79 pm, while that of V3+
is 64 pm). Proceeding down the groups of the periodic table, we find that cations of successive elements with the same charge
generally have larger radii, corresponding to an increase in the principal quantum number, n.
An anion (negative ion) is formed by the addition of one or more electrons to the valence shell of an atom. This results in a greater
repulsion among the electrons and a decrease in Z per electron. Both effects (the increased number of electrons and the
ef f

decreased Zeff) cause the radius of an anion to be larger than that of the parent atom ( Figure 6.5.3). For example, a sulfur atom
([Ne]3s23p4) has a covalent radius of 104 pm, whereas the ionic radius of the sulfide anion ([Ne]3s23p6) is 170 pm. For consecutive
elements proceeding down any group, anions have larger principal quantum numbers and, thus, larger radii.
Atoms and ions that have the same electron configuration are said to be isoelectronic. Examples of isoelectronic species are N3–,
O2–, F–, Ne, Na+, Mg2+, and Al3+ (1s22s22p6). Another isoelectronic series is P3–, S2–, Cl–, Ar, K+, Ca2+, and Sc3+ ([Ne]3s23p6). For
atoms or ions that are isoelectronic, the number of protons determines the size. The greater the nuclear charge, the smaller the
radius in a series of isoelectronic ions and atoms.

6.5.3: Variation in Ionization Energies


The amount of energy required to remove the most loosely bound electron from a gaseous atom in its ground state is called its first
ionization energy (IE1). The first ionization energy for an element, X, is the energy required to form a cation with +1 charge:
+ −
X(g) ⟶ X (g) + e IE
1

The energy required to remove the second most loosely bound electron is called the second ionization energy (IE2).
+ 2+ −
X (g) ⟶ X (g) + e IE
2

The energy required to remove the third electron is the third ionization energy, and so on. Energy is always required to remove
electrons from atoms or ions, so ionization processes are endothermic and IE values are always positive. For larger atoms, the most
loosely bound electron is located farther from the nucleus and so is easier to remove. Thus, as size (atomic radius) increases, the

Access for free at OpenStax 6.5.4 https://chem.libretexts.org/@go/page/38178


ionization energy should decrease. Relating this logic to what we have just learned about radii, we would expect first ionization
energies to decrease down a group and to increase across a period.

Figure 6.5.4 : The first ionization energy of the elements in the first five periods are plotted against their atomic number.
This figure includes Atomic Number on the horizontal axis and, Ionization Energy (k J divided by mol) on the vertical axis. A red
jagged line connects the ionization energies for elements with atomic numbers 1 through 86 on the graph. Peaks are evident at the
locations of the noble or inert gases: H e, N e, A r, K r, X e, and R n. Similarly, minima exist at the locations of the alkali metals: L
i, N a, K, R b, and C s.
Figure 6.5.4 graphs the relationship between the first ionization energy and the atomic number of several elements. Within a
period, the values of first ionization energy for the elements (IE1) generally increases with increasing Z. Down a group, the IE1
value generally decreases with increasing Z. There are some systematic deviations from this trend, however. Note that the
ionization energy of boron (atomic number 5) is less than that of beryllium (atomic number 4) even though the nuclear charge of
boron is greater by one proton. This can be explained because the energy of the subshells increases as l increases, due to
penetration and shielding (as discussed previously in this chapter). Within any one shell, the s electrons are lower in energy than the
p electrons. This means that an s electron is harder to remove from an atom than a p electron in the same shell. The electron
removed during the ionization of beryllium ([He]2s2) is an s electron, whereas the electron removed during the ionization of boron
([He]2s22p1) is a p electron; this results in a lower first ionization energy for boron, even though its nuclear charge is greater by one
proton. Thus, we see a small deviation from the predicted trend occurring each time a new subshell begins.

Figure 6.5.5 : This version of the periodic table shows the first ionization energy of (IE: 1), in kJ/mol, of selected elements.
The trends for first ionization energies across periods and down groups are shown in this version of the periodic table.
Another deviation occurs as orbitals become more than one-half filled. The first ionization energy for oxygen is slightly less than
that for nitrogen, despite the trend in increasing IE1 values across a period. Looking at the orbital diagram of oxygen, we can see

Access for free at OpenStax 6.5.5 https://chem.libretexts.org/@go/page/38178


that removing one electron will eliminate the electron–electron repulsion caused by pairing the electrons in the 2p orbital and will
result in a half-filled orbital (which is energetically favorable). Analogous changes occur in succeeding periods (note the dip for
sulfur after phosphorus in Figure 6.5.4.

The electronic configuration for oxygen is 1s superscript 2, 2s superscript, 2p superscript 4. An orbital diagram shows 2 squares
each filled with a pair of opposite pointing arrows to represent the 1s ans 2s orbitals. Each of the three connected square which
represents the 2p orbitals are filled with one arrow respectively with the addition of 1 red arrow in the first of the three squares.
Removing an electron from a cation is more difficult than removing an electron from a neutral atom because of the greater
electrostatic attraction to the cation. Likewise, removing an electron from a cation with a higher positive charge is more difficult
than removing an electron from an ion with a lower charge. Thus, successive ionization energies for one element always increase.
As seen in Table 6.5.2, there is a large increase in the ionization energies (color change) for each element. This jump corresponds
to removal of the core electrons, which are harder to remove than the valence electrons. For example, Sc and Ga both have three
valence electrons, so the rapid increase in ionization energy occurs after the third ionization.
Table 6.5.2: Successive Ionization Energies for Selected Elements (kJ/mol)
Element IE1 IE2 IE3 IE4 IE5 IE6 IE7

K 418.8 3051.8 4419.6 5876.9 7975.5 9590.6 11343

Ca 589.8 1145.4 4912.4 6490.6 8153.0 10495.7 12272.9

Sc 633.1 1235.0 2388.7 7090.6 8842.9 10679.0 13315.0

Ga 578.8 1979.4 2964.6 6180 8298.7 10873.9 13594.8

Ge 762.2 1537.5 3302.1 4410.6 9021.4 Not available Not available

As 944.5 1793.6 2735.5 4836.8 6042.9 12311.5 Not available

 Example 6.5.2: Ranking Ionization Energies


Predict the order of increasing energy for the following processes: IE1 for Al, IE1 for Tl, IE2 for Na, IE3 for Al.

Solution
Removing the 6p1 electron from Tl is easier than removing the 3p1 electron from Al because the higher n orbital is farther from
the nucleus, so IE1(Tl) < IE1(Al). Ionizing the third electron from
2+ 3+ −
Al (Al ⟶ Al +e )

requires more energy because the cation Al2+ exerts a stronger pull on the electron than the neutral Al atom, so IE1(Al) <
IE3(Al). The second ionization energy for sodium removes a core electron, which is a much higher energy process than
removing valence electrons. Putting this all together, we obtain:
IE1(Tl) < IE1(Al) < IE3(Al) < IE2(Na).

 Exercise 6.5.2
Which has the lowest value for IE1: O, Po, Pb, or Ba?

Answer
Ba

Access for free at OpenStax 6.5.6 https://chem.libretexts.org/@go/page/38178


6.5.4: Variation in Electron Affinities
The electron affinity [EA] is the energy change for the process of adding an electron to a gaseous atom to form an anion (negative
ion).
− −
X(g) + e ⟶ X (g) EA
1

This process can be either endothermic or exothermic, depending on the element. The EA of some of the elements is given in
Figure 6.5.6. You can see that many of these elements have negative values of EA, which means that energy is released when the
gaseous atom accepts an electron. However, for some elements, energy is required for the atom to become negatively charged and
the value of their EA is positive. Just as with ionization energy, subsequent EA values are associated with forming ions with more
charge. The second EA is the energy associated with adding an electron to an anion to form a –2 ion, and so on.

Figure 6.5.6 : This version of the periodic table displays the electron affinity values (in kJ/mol) for selected elements.:
The trend for electron affinity values across periods and down groups is shown in this version of the periodic table. Values are not
shown for groups 3 to 12 as well as period 7.
As we might predict, it becomes easier to add an electron across a series of atoms as the effective nuclear charge of the atoms
increases. We find, as we go from left to right across a period, EAs tend to become more negative. The exceptions found among the
elements of group 2 (2A), group 15 (5A), and group 18 (8A) can be understood based on the electronic structure of these groups.
The noble gases, group 18 (8A), have a completely filled shell and the incoming electron must be added to a higher n level, which
is more difficult to do. Group 2 (2A) has a filled ns subshell, and so the next electron added goes into the higher energy np, so,
again, the observed EA value is not as the trend would predict. Finally, group 15 (5A) has a half-filled np subshell and the next
electron must be paired with an existing np electron. In all of these cases, the initial relative stability of the electron configuration
disrupts the trend in EA.
We also might expect the atom at the top of each group to have the largest EA; their first ionization potentials suggest that these
atoms have the largest effective nuclear charges. However, as we move down a group, we see that the second element in the group
most often has the greatest EA. The reduction of the EA of the first member can be attributed to the small size of the n = 2 shell and
the resulting large electron–electron repulsions. For example, chlorine, with an EA value of –348 kJ/mol, has the highest value of
any element in the periodic table. The EA of fluorine is –322 kJ/mol. When we add an electron to a fluorine atom to form a fluoride
anion (F–), we add an electron to the n = 2 shell. The electron is attracted to the nucleus, but there is also significant repulsion from
the other electrons already present in this small valence shell. The chlorine atom has the same electron configuration in the valence
shell, but because the entering electron is going into the n = 3 shell, it occupies a considerably larger region of space and the
electron–electron repulsions are reduced. The entering electron does not experience as much repulsion and the chlorine atom
accepts an additional electron more readily.
The properties discussed in this section (size of atoms and ions, effective nuclear charge, ionization energies, and electron
affinities) are central to understanding chemical reactivity. For example, because fluorine has an energetically favorable EA and a
large energy barrier to ionization (IE), it is much easier to form fluorine anions than cations. Metallic properties including

Access for free at OpenStax 6.5.7 https://chem.libretexts.org/@go/page/38178


conductivity and malleability (the ability to be formed into sheets) depend on having electrons that can be removed easily. Thus,
metallic character increases as we move down a group and decreases across a period in the same trend observed for atomic size
because it is easier to remove an electron that is farther away from the nucleus.

Summary
Electron configurations allow us to understand many periodic trends. Covalent radius increases as we move down a group because
the n level (orbital size) increases. Covalent radius mostly decreases as we move left to right across a period because the effective
nuclear charge experienced by the electrons increases, and the electrons are pulled in tighter to the nucleus. Anionic radii are larger
than the parent atom, while cationic radii are smaller, because the number of valence electrons has changed while the nuclear
charge has remained constant. Ionization energy (the energy associated with forming a cation) decreases down a group and mostly
increases across a period because it is easier to remove an electron from a larger, higher energy orbital. Electron affinity (the energy
associated with forming an anion) is more favorable (exothermic) when electrons are placed into lower energy orbitals, closer to
the nucleus. Therefore, electron affinity becomes increasingly negative as we move left to right across the periodic table and
decreases as we move down a group. For both IE and electron affinity data, there are exceptions to the trends when dealing with
completely filled or half-filled subshells.

Glossary
covalent radius
one-half the distance between the nuclei of two identical atoms when they are joined by a covalent bond

effective nuclear charge


charge that leads to the Coulomb force exerted by the nucleus on an electron, calculated as the nuclear charge minus shielding

electron affinity
energy required to add an electron to a gaseous atom to form an anion

ionization energy
energy required to remove an electron from a gaseous atom or ion. The associated number (e.g., second ionization energy)
corresponds to the charge of the ion produced (X2+)

isoelectronic
group of ions or atoms that have identical electron configurations

This page titled 6.5: Periodic Variations in Element Properties is shared under a CC BY 4.0 license and was authored, remixed, and/or curated by
OpenStax via source content that was edited to the style and standards of the LibreTexts platform; a detailed edit history is available upon request.

Access for free at OpenStax 6.5.8 https://chem.libretexts.org/@go/page/38178


6.E: Electronic Structure and Periodic Properties (Exercises)
6.1: Electromagnetic Energy
Q6.1.1
The light produced by a red neon sign is due to the emission of light by excited neon atoms. Qualitatively describe the spectrum
produced by passing light from a neon lamp through a prism.

S6.1.1
The spectrum consists of colored lines, at least one of which (probably the brightest) is red.

Q6.1.2
An FM radio station found at 103.1 on the FM dial broadcasts at a frequency of 1.031 × 108 s−1 (103.1 MHz). What is the
wavelength of these radio waves in meters?

S6.1.2
c
λ =
ν

m
8
2.998 × 10
s
λ = = 2.908 m
1
8
1.031 × 10
s

Q6.1.3
FM-95, an FM radio station, broadcasts at a frequency of 9.51 × 107 s−1 (95.1 MHz). What is the wavelength of these radio waves
in meters?

S6.1.3
c
λ =
ν

m
8
2.998 × 10
s
λ = = 3.15 m
1
7
9.51 × 10
s

Q6.1.4
A bright violet line occurs at 435.8 nm in the emission spectrum of mercury vapor. What amount of energy, in joules, must be
released by an electron in a mercury atom to produce a photon of this light?

S6.1.4
hc
E =
λ

8
m −34
(2.998 × 10 ) (6.6262 × 10 Js)
s
E =
−7
4.358 × 10 m

−19
= 4.56 × 10 J

Q6.1.5
Light with a wavelength of 614.5 nm looks orange. What is the energy, in joules, per photon of this orange light? What is the
energy in eV (1 eV = 1.602 × 10−19 J)?

S6.1.5
1.) First convert 614.5 nm into meters
6.145nm = 6.145 × 10 −7
m

Access for free at OpenStax 6.E.1 https://chem.libretexts.org/@go/page/77623


2.) Then calculate the amount of energy this wavelength of light contains using the equations:
c hc
E = hν and ν = which can be manipulated algebraically into: E =
λ λ

h = Plancks constant → 6.6262 × 10 −34


Js
m
c = Speed of light → 2.998 × 10 8

s
λ = wavelength of photon → 6.145 × 10 −7
m

−34 8
m
(6.6262 × 10 Js) (2.998 × 10 )
s −19
= = 3.233 × 10 J (6.E.1)
−7
6.145 × 10 m 

3.) Then convert Joules into eV:

−19
1eV
= (3.233 × 10 J) × (6.E.2)
−19
1.602 × 10 J

= 2.018eV (6.E.3)

Q6.1.6
Heated lithium atoms emit photons of light with an energy of 2.961 × 10−19 J. Calculate the frequency and wavelength of one of
these photons. What is the total energy in 1 mole of these photons? What is the color of the emitted light?

S6.1.6
E
1.) E = hν and ν =
h

h = Plancks constant → 6.6262 × 10 −34


Js

−19
2.961 × 10 J
Frequency: ν = −34
  = 4.469 × 10
14
Hz
6.6262 × 10 Js

c
2.) λ =
ν
m
c = Speed of light → 2.998 × 10 8

m
8
2.998 × 10
s
Wavelength: λ =   = 6.709 × 10
−7
m = Red Light
1
14
4.469 × 10
s

Total energy:
−19 23
2.961 × 10 J 6.022 × 10 photons
5
E = × = 1.783 × 10 J
1 photon 1 mole

Q6.1.7
A photon of light produced by a surgical laser has an energy of 3.027 × 10−19 J. Calculate the frequency and wavelength of the
photon. What is the total energy in 1 mole of photons? What is the color of the emitted light?

S6.1.7
ν = 4.568 × 1014 s; λ = 656.3 nm; Energy mol−1 = 1.823 × 105 J mol−1; red

Q6.1.8
When rubidium ions are heated to a high temperature, two lines are observed in its line spectrum at wavelengths (a) 7.9 × 10−7 m
and (b) 4.2 × 10−7 m. What are the frequencies of the two lines? What color do we see when we heat a rubidium compound?

Access for free at OpenStax 6.E.2 https://chem.libretexts.org/@go/page/77623


Q6.1.9
The emission spectrum of cesium contains two lines whose frequencies are (a) 3.45 × 1014 Hz and (b) 6.53 × 1014 Hz. What are the
wavelengths and energies per photon of the two lines? What color are the lines?

S6.1.9
(a) λ = 8.69 × 10−7 m; E = 2.29 × 10−19 J; (b) λ = 4.59 × 10−7 m; E = 4.33 × 10−19 J; The color of (a) is red; (b) is blue.

Q6.1.10
Photons of infrared radiation are responsible for much of the warmth we feel when holding our hands before a fire. These photons
will also warm other objects. How many infrared photons with a wavelength of 1.5 × 10−6 m must be absorbed by the water to
warm a cup of water (175 g) from 25.0 °C to 40 °

S6.1.10
1.) First we must use the equation: q=mCΔT° to calculate the amount of Energy in Joules (J) to warm 175g of H2O a total of 15°
Celsius
m = mass in grams(g) → 175g
4.184J
C = Specific heat of H2O(l) →
g°C

ΔT° = Difference in temperature → 40 - 25 =15°


4.184J
q = 175g × × 15°C (6.E.4)
g°C

q= 11,000 J
2.) Now we need to calculate the amount of energy in Joules (J) that one Photon with a wavelength of 1.5x10-6 m contains. We will
use the equations:
c hc
E = hν and ν = which can be manipulated algebraically into: E =
λ λ

h = Plancks constant → 6.6262 × 10 −34


Js
m
c = Speed of light → 2.998 × 10 8

s
λ = wavelength of photon → 1.5 × 10 −6
m

m
−34 8
(6.6262 × 10 Js) (2.998 × 10 )
s −19
= = 1.3 × 10 J (6.E.5)
−6
1.5 × 10 m 

This is the energy in one photon, so now we have to see how many times 1.3x10-19 J fits into the 11,000 J from our first calculation:
11, 000 J
22
= = 8.3 × 10 photons (6.E.6)
J
−19
1.3 × 10
photon

22
= 8.3 × 10 photons (6.E.7)

Q6.1.11
One of the radiographic devices used in a dentist's office emits an X-ray of wavelength 2.090 × 10−11 m. What is the energy, in
joules, and frequency of this X-ray?

S6.1.11
E = 9.502 × 10−15 J; ν = 1.434 × 1019 s−1

Q6.1.12
The eyes of certain reptiles pass a single visual signal to the brain when the visual receptors are struck by photons of a wavelength
of 850 nm. If a total energy of 3.15 × 10−14 J is required to trip the signal, what is the minimum number of photons that must strike

Access for free at OpenStax 6.E.3 https://chem.libretexts.org/@go/page/77623


the receptor?

S6.1.12
1.) First we need to convert nanometers to meters
850nm = 8.5 x 10-7 m
2. Then calculate Energy in Joules a photon of this wavelength (λ) produces:
m
−34 8
(6.6262 × 10 Js) (2.998 × 10 )
hc s −19
J
(E = ) = = 2.3 × 10 (6.E.8)
−7
λ 8.5 × 10 m  photon

3.) Then we need to find out how many of these photons it will take to trip the visual signal to the brain:
−14
3.15 × 10 J
= (6.E.9)
J
−19
2.3 × 10
photon

5
= 1.3 × 10 photons (6.E.10)

Q6.1.13
RGB color television and computer displays use cathode ray tubes that produce colors by mixing red, green, and blue light. If we
look at the screen with a magnifying glass, we can see individual dots turn on and off as the colors change. Using a spectrum of
visible light, determine the approximate wavelength of each of these colors. What is the frequency and energy of a photon of each
of these colors?

S6.1.13
Red: 660 nm; 4.54 × 1014 Hz; 3.01 × 10−19 J.
Green: 520 nm; 5.77 × 1014 Hz; 3.82 × 10−19 J.
Blue: 440 nm; 6.81 × 1014 Hz; 4.51 × 10−19 J.
Somewhat different numbers are also possible.

Q6.1.14
Answer the following questions about a Blu-ray laser:
a. The laser on a Blu-ray player has a wavelength of 405 nm. In what region of the electromagnetic spectrum is this radiation?
What is its frequency?
b. A Blu-ray laser has a power of 5 milliwatts (1 watt = 1 J s−1). How many photons of light are produced by the laser in 1 hour?
c. The ideal resolution of a player using a laser (such as a Blu-ray player), which determines how close together data can be stored
on a compact disk, is determined using the following formula: Resolution = 0.60(λ/NA), where λ is the wavelength of the laser
and NA is the numerical aperture. Numerical aperture is a measure of the size of the spot of light on the disk; the larger the NA,
the smaller the spot. In a typical Blu-ray system, NA = 0.95. If the 405-nm laser is used in a Blu-ray player, what is the closest
that information can be stored on a Blu-ray disk?
d. The data density of a Blu-ray disk using a 405-nm laser is 1.5 × 107 bits mm−2. Disks have an outside diameter of 120 mm and
a hole of 15-mm diameter. How many data bits can be contained on the disk? If a Blu-ray disk can hold 9,400,000 pages of text,
how many data bits are needed for a typed page? (Hint: Determine the area of the disk that is available to hold data. The area
inside a circle is given by A = πr2, where the radius r is one-half of the diameter.)

Q6.1.15
What is the threshold frequency for sodium metal if a photon with frequency 6.66 × 1014 s−1 ejects a photon with 7.74 × 10−20 J
kinetic energy? Will the photoelectric effect be observed if sodium is exposed to orange light?

S6.1.15
5.49 × 1014 s−1; no

Access for free at OpenStax 6.E.4 https://chem.libretexts.org/@go/page/77623


6.2: The Bohr Model
Q6.2.1
Why is the electron in a Bohr hydrogen atom bound less tightly when it has a quantum number of 3 than when it has a quantum
number of 1?

Q6.2.2
What does it mean to say that the energy of the electrons in an atom is quantized?

S6.2.2
Quantized energy means that the electrons can possess only certain discrete energy values; values between those quantized
values are not permitted.

Q6.2.3
Using the Bohr model, determine the energy, in joules, necessary to ionize a ground-state hydrogen atom. Show your
calculations.

Q6.2.4
The electron volt (eV) is a convenient unit of energy for expressing atomic-scale energies. It is the amount of energy that an
electron gains when subjected to a potential of 1 volt; 1 eV = 1.602 × 10 J . Using the Bohr model, determine the energy,
−19

in electron volts, of the photon produced when an electron in a hydrogen atom moves from the orbit with n = 5 to the orbit
with n = 2 . Show your calculations.

S6.2.4
1 1
−18
E = E2 − E5 = 2.179 × 10 ( − ) J (6.E.11)
2 2
n n
2 5

1 1
−18 −19
= 2.179 × 10 ( − ) = 4.576 × 10 J (6.E.12)
2 2
2 5

−19
4.576 × 10 J
= = 2.856 eV (6.E.13)
−19 −1
1.602 × 10 J eV

Q6.2.5
Using the Bohr model, determine the lowest possible energy for the electron in the H e ion. +

Q6.2.6
Using the Bohr model, determine the energy of an electron with n = 6 in a hydrogen atom.

S6.2.6
−8.716 × 10−18 J

Q6.2.7
Using the Bohr model, determine the energy of an electron with n = 8 in a hydrogen atom.

Q6.2.8
How far from the nucleus in angstroms (1 angstrom = 1 × 10
–10
m ) is the electron in a hydrogen atom if it has an energy of
J?
−20
−8.72 × 10

S6.2.8
−20
−3.405 × 10 J

Access for free at OpenStax 6.E.5 https://chem.libretexts.org/@go/page/77623


Q6.2.9
What is the radius, in angstroms, of the orbital of an electron with n = 8 in a hydrogen atom?

Q6.2.10
Using the Bohr model, determine the energy in joules of the photon produced when an electron in a He ion moves from the
+

orbit with n = 5 to the orbit with n = 2.

Q6.2.11
Using the Bohr model, determine the energy in joules of the photon produced when an electron in a Li2+ ion moves from the
orbit with n = 2 to the orbit with n = 1.

S6.2.11
1.471 × 10−17 J

Q6.2.12
Consider a large number of hydrogen atoms with electrons randomly distributed in the n = 1, 2, 3, and 4 orbits
a. How many different wavelengths of light are emitted by these atoms as the electrons fall into lower-energy orbitals?
b. Calculate the lowest and highest energies of light produced by the transitions described in part (a).
c. Calculate the frequencies and wavelengths of the light produced by the transitions described in part (b).

Q6.2.13
How are the Bohr model and the Rutherford model of the atom similar? How are they different?

Q6.2.14
The spectra of hydrogen and of calcium are shown in[link]. What causes the lines in these spectra? Why are the colors of the
lines different? Suggest a reason for the observation that the spectrum of calcium is more complicated than the spectrum of
hydrogen

S6.2.14
Both involve a relatively heavy nucleus with electrons moving around it, although strictly speaking, the Bohr model works only for
one-electron atoms or ions. According to classical mechanics, the Rutherford model predicts a miniature “solar system” with
electrons moving about the nucleus in circular or elliptical orbits that are confined to planes. If the requirements of classical
electromagnetic theory that electrons in such orbits would emit electromagnetic radiation are ignored, such atoms would be stable,
having constant energy and angular momentum, but would not emit any visible light (contrary to observation). If classical
electromagnetic theory is applied, then the Rutherford atom would emit electromagnetic radiation of continually increasing
frequency (contrary to the observed discrete spectra), thereby losing energy until the atom collapsed in an absurdly short time
(contrary to the observed long-term stability of atoms). The Bohr model retains the classical mechanics view of circular orbits
confined to planes having constant energy and angular momentum, but restricts these to quantized values dependent on a single
quantum number, n. The orbiting electron in Bohr’s model is assumed not to emit any electromagnetic radiation while moving
about the nucleus in its stationary orbits, but the atom can emit or absorb electromagnetic radiation when the electron changes from
one orbit to another. Because of the quantized orbits, such “quantum jumps” will produce discrete spectra, in agreement with
observations.

6.3: Development of Quantum Theory


Q6.3.1
How are the Bohr model and the quantum mechanical model of the hydrogen atom similar? How are they different?

S6.3.1
Both models have a central positively charged nucleus with electrons moving about the nucleus in accordance with the Coulomb
electrostatic potential. The Bohr model assumes that the electrons move in circular orbits that have quantized energies, angular
momentum, and radii that are specified by a single quantum number, n = 1, 2, 3, …, but this quantization is an ad hoc assumption
made by Bohr to incorporate quantization into an essentially classical mechanics description of the atom. Bohr also assumed that

Access for free at OpenStax 6.E.6 https://chem.libretexts.org/@go/page/77623


electrons orbiting the nucleus normally do not emit or absorb electromagnetic radiation, but do so when the electron switches to a
different orbit. In the quantum mechanical model, the electrons do not move in precise orbits (such orbits violate the Heisenberg
uncertainty principle) and, instead, a probabilistic interpretation of the electron’s position at any given instant is used, with a
mathematical function ψ called a wavefunction that can be used to determine the electron’s spatial probability distribution. These
wavefunctions, or orbitals, are three-dimensional stationary waves that can be specified by three quantum numbers that arise
naturally from their underlying mathematics (no ad hoc assumptions required): the principal quantum number, n (the same one used
by Bohr), which specifies shells such that orbitals having the same n all have the same energy and approximately the same spatial
extent; the angular momentum quantum number l, which is a measure of the orbital’s angular momentum and corresponds to the
orbitals’ general shapes, as well as specifying subshells such that orbitals having the same l (and n) all have the same energy; and
the orientation quantum number m, which is a measure of the z component of the angular momentum and corresponds to the
orientations of the orbitals. The Bohr model gives the same expression for the energy as the quantum mechanical expression and,
hence, both properly account for hydrogen’s discrete spectrum (an example of getting the right answers for the wrong reasons,
something that many chemistry students can sympathize with), but gives the wrong expression for the angular momentum (Bohr
orbits necessarily all have non-zero angular momentum, but some quantum orbitals [s orbitals] can have zero angular momentum).

Q6.3.2
What are the allowed values for each of the four quantum numbers: n, l, ml, and ms?

Q6.3.3
Describe the properties of an electron associated with each of the following four quantum numbers: n, l, ml, and ms.

S6.3.3
n determines the general range for the value of energy and the probable distances that the electron can be from the nucleus. l
determines the shape of the orbital. ml determines the orientation of the orbitals of the same l value with respect to one another. ms
determines the spin of an electron.

Q6.3.4
Answer the following questions:
a. Without using quantum numbers, describe the differences between the shells, subshells, and orbitals of an atom.
b. How do the quantum numbers of the shells, subshells, and orbitals of an atom differ?

Q6.3.5
Identify the subshell in which electrons with the following quantum numbers are found:
a. n = 2, l = 1
b. n = 4, l = 2
c. n = 6, l = 0

S6.3.5
(a) 2p; (b) 4d; (c) 6s

Q6.3.6
Which of the subshells described in Question 5 contain degenerate orbitals? How many degenerate orbitals are in each?

Q6.3.7
Identify the subshell in which electrons with the following quantum numbers are found:
a. n = 3, l = 2
b. n = 1, l = 0
c. n = 4, l = 3

S6.3.7
(a) 3d; (b) 1s; (c) 4f

Access for free at OpenStax 6.E.7 https://chem.libretexts.org/@go/page/77623


Q6.3.8
Which of the subshells described in Question 7 contain degenerate orbitals? How many degenerate orbitals are in each?

Q6.3.9
Sketch the boundary surface of a d 2
x −y
2 and a py orbital. Be sure to show and label the axes.

Q6.3.10
Sketch the px and dxz orbitals. Be sure to show and label the coordinates.

Q6.3.11
Consider the orbitals shown here in outline.

a. What is the maximum number of electrons contained in an orbital of type (x)? Of type (y)? Of type (z)?
b. How many orbitals of type (x) are found in a shell with n = 2? How many of type (y)? How many of type (z)?
c. Write a set of quantum numbers for an electron in an orbital of type (x) in a shell with n = 4. Of an orbital of type (y) in a shell
with n = 2. Of an orbital of type (z) in a shell with n = 3.
d. What is the smallest possible n value for an orbital of type (x)? Of type (y)? Of type (z)?
e. What are the possible l and ml values for an orbital of type (x)? Of type (y)? Of type (z)?

S6.3.11
1 1 1
(a) x. 2, y. 2, z. 2; (b) x. 1, y. 3, z. 0; (c) x. 4 0 0 , y. 2 1 0 , z. 3 2 0 ; (d) x. 1, y. 2, z. 3; (e) x. l = 0, ml = 0, y. l = 1, ml = –1, 0,
2 2 2
or +1, z. l = 2, ml = –2, –1, 0, +1, +2

Q6.3.12
State the Heisenberg uncertainty principle. Describe briefly what the principle implies.

Q6.3.13
How many electrons could be held in the second shell of an atom if the spin quantum number ms could have three values instead of
just two? (Hint: Consider the Pauli exclusion principle.)

S6.3.13
12

Q6.3.14
Which of the following equations describe particle-like behavior? Which describe wavelike behavior? Do any involve both types of
behavior? Describe the reasons for your choices.

Access for free at OpenStax 6.E.8 https://chem.libretexts.org/@go/page/77623


a. c = λν
2

b. E =
2
2
n a0
c. r =
Z
d. E = hν
h
e. λ =

Q6.3.15
Write a set of quantum numbers for each of the electrons with an n of 4 in a Se atom.

S6.3.15
n l ml s
1
4 0 0 +
2

1
4 0 0 −
2

1
4 1 −1 +
2

1
4 1 0 +
2

1
4 1 +1 +
2

1
4 1 −1 −
2

6.4: Electronic Structure of Atoms (Electron Configurations)


Q6.4.1
Read the labels of several commercial products and identify monatomic ions of at least four transition elements contained in the
products. Write the complete electron configurations of these cations.

Q6.4.2
Read the labels of several commercial products and identify monatomic ions of at least six main group elements contained in the
products. Write the complete electron configurations of these cations and anions.

S6.4.2
For example, Na+: 1s22s22p6; Ca2+: 1s22s22p6; Sn2+: 1s22s22p63s23p63d104s24p64d105s2; F–: 1s22s22p6; O2–: 1s22s22p6; Cl–:
1s22s22p63s23p6.

Q6.4.3
Using complete subshell notation (not abbreviations, 1s22s22p6, and so forth), predict the electron configuration of each of the
following atoms:
a. C
b. P
c. V
d. Sb
e. Sm

S6.4.3
a.) 1s22s22p2
b.) 1s22s22p63s23p3
c.) 1s22s22p63s23p63d34s2

Access for free at OpenStax 6.E.9 https://chem.libretexts.org/@go/page/77623


d.) 1s22s22p63s23p63d104s24p64d105s25p3
e.) 1s22s22p63s23p63d104s24p64d104f65s25p66s2

Q6.4.4
Using complete subshell notation (1s22s22p6, and so forth), predict the electron configuration of each of the following atoms:
a. N
b. Si
c. Fe
d. Te
e. Tb

S6.4.4
a. 1s22s22p3;
b. 1s22s22p63s23p2;
c. 1s22s22p63s23p64s23d6;
d. 1s22s22p63s23p64s23d104p65s24d105p4;
e. 1s22s22p63s23p64s23d104p65s24d105p66s24f9

Q6.4.5
Is 1s22s22p6 the symbol for a macroscopic property or a microscopic property of an element? Explain your answer.

Q6.4.6
What additional information do we need to answer the question “Which ion has the electron configuration 1s22s22p63s23p6”?

S6.4.6
The charge on the ion.

Q6.4.7
Draw the orbital diagram for the valence shell of each of the following atoms:
a. C
b. P
c. V
d. Sb
e. Ru

Q6.4.8
Use an orbital diagram to describe the electron configuration of the valence shell of each of the following atoms:
a. N
b. Si
c. Fe
d. Te
e. Mo

S6.4.8
(a)

(b)

Access for free at OpenStax 6.E.10 https://chem.libretexts.org/@go/page/77623


(c)

(d)

(e)

Q6.4.9
Using complete subshell notation (1s22s22p6, and so forth), predict the electron configurations of the following ions.
a. N3–
b. Ca2+
c. S–
d. Cs2+
e. Cr2+
f. Gd3+

Q6.4.10
Which atom has the electron configuration: 1s22s22p63s23p64s23d104p65s24d2?

S6.4.10
Zr

Q6.4.11
Which atom has the electron configuration: 1s22s22p63s23p63d74s2?

S6.4.11
Co ; Cobalt

Q6.4.12
Which ion with a +1 charge has the electron configuration 1s22s22p63s23p63d104s24p6? Which ion with a –2 charge has this
configuration?

S6.4.12
Rb+, Se2−

Access for free at OpenStax 6.E.11 https://chem.libretexts.org/@go/page/77623


Q6.4.13
Which of the following atoms contains only three valence electrons: Li, B, N, F, Ne?

S6.4.13
B ; Boron

Q6.4.14
Which of the following has two unpaired electrons?
a. Mg
b. Si
c. S
d. Both Mg and S
e. Both Si and S.

S6.4.14
Although both (b) and (c) are correct, (e) encompasses both and is the best answer.

Q6.4.15
Which atom would be expected to have a half-filled 6p subshell?

S6.4.15
Bi ; Bismuth

Q6.4.16
Which atom would be expected to have a half-filled 4s subshell?

S6.4.16
K

Q6.4.17
In one area of Australia, the cattle did not thrive despite the presence of suitable forage. An investigation showed the cause to be
the absence of sufficient cobalt in the soil. Cobalt forms cations in two oxidation states, Co2+ and Co3+. Write the electron structure
of the two cations.

Q6.4.18
Thallium was used as a poison in the Agatha Christie mystery story “The Pale Horse.” Thallium has two possible cationic forms,
+1 and +3. The +1 compounds are the more stable. Write the electron structure of the +1 cation of thallium.

S6.4.18
1s22s22p63s23p63d104s24p64d105s25p66s24f145d10

Q6.4.19
Write the electron configurations for the following atoms or ions:
a. B3+
b. O–
c. Cl3+
d. Ca2+
e. Ti

Q6.4.20
Cobalt–60 and iodine–131 are radioactive isotopes commonly used in nuclear medicine. How many protons, neutrons, and
electrons are in atoms of these isotopes? Write the complete electron configuration for each isotope.

Access for free at OpenStax 6.E.12 https://chem.libretexts.org/@go/page/77623


S6.4.20
Co has 27 protons, 27 electrons, and 33 neutrons: 1s22s22p63s23p64s23d7.
I has 53 protons, 53 electrons, and 78 neutrons: 1s22s22p63s23p63d104s24p64d105s25p5.

Q6.4.21
Write a set of quantum numbers for each of the electrons with an n of 3 in a Sc atom.

6.5: Periodic Variations in Element Properties


Q6.5.1
Based on their positions in the periodic table, predict which has the smallest atomic radius: Mg, Sr, Si, Cl, I.

S6.5.1
Cl

Q6.5.2
Based on their positions in the periodic table, predict which has the largest atomic radius: Li, Rb, N, F, I.

Q6.5.3
Based on their positions in the periodic table, predict which has the largest first ionization energy: Mg, Ba, B, O, Te.

S6.5.3
O

Q6.5.4
Based on their positions in the periodic table, predict which has the smallest first ionization energy: Li, Cs, N, F, I.

Q6.5.5
Based on their positions in the periodic table, rank the following atoms in order of increasing first ionization energy: F, Li, N, Rb

S6.5.5
Rb < Li < N < F

Q6.5.6
Based on their positions in the periodic table, rank the following atoms or compounds in order of increasing first ionization energy:
Mg, O, S, Si

Q6.5.7
Atoms of which group in the periodic table have a valence shell electron configuration of ns2np3?

S6.5.7
15 (5A)

Q6.5.8
Atoms of which group in the periodic table have a valence shell electron configuration of ns2?

Q6.5.9
Based on their positions in the periodic table, list the following atoms in order of increasing radius: Mg, Ca, Rb, Cs.

S6.5.9
Mg < Ca < Rb < Cs

Access for free at OpenStax 6.E.13 https://chem.libretexts.org/@go/page/77623


Q6.5.10
Based on their positions in the periodic table, list the following atoms in order of increasing radius: Sr, Ca, Si, Cl.

Q6.5.11
Based on their positions in the periodic table, list the following ions in order of increasing radius: K+, Ca2+, Al3+, Si4+.

S6.5.11
Si4+ < Al3+ < Ca2+ < K+

Q6.5.12
List the following ions in order of increasing radius: Li+, Mg2+, Br–, Te2–.

Q6.5.13
Which atom and/or ion is (are) isoelectronic with Br+: Se2+, Se, As–, Kr, Ga3+, Cl–?

S6.5.13
Se, As−

Q6.5.14
Which of the following atoms and ions is (are) isoelectronic with S2+: Si4+, Cl3+, Ar, As3+, Si, Al3+?

Q6.5.15
Compare both the numbers of protons and electrons present in each to rank the following ions in order of increasing radius: As3–,
Br–, K+, Mg2+.

S6.5.15
Mg2+ < K+ < Br– < As3–

Q6.5.16
Of the five elements Al, Cl, I, Na, Rb, which has the most exothermic reaction? (E represents an atom.) What name is given to the
energy for the reaction? Hint: note the process depicted does not correspond to electron affinity
+ −
E (g) + e ⟶ E(g)

Q6.5.17
Of the five elements Sn, Si, Sb, O, Te, which has the most endothermic reaction? (E represents an atom.) What name is given to the
energy for the reaction?
+ −
E(g) ⟶ E (g) + e (6.E.14)

S6.5.17
O, IE1

Q6.5.18
The ionic radii of the ions S2–, Cl–, and K+ are 184, 181, 138 pm respectively. Explain why these ions have different sizes even
though they contain the same number of electrons.

Q6.5.19
Which main group atom would be expected to have the lowest second ionization energy?

S6.5.19
Ra

Access for free at OpenStax 6.E.14 https://chem.libretexts.org/@go/page/77623


Q6.5.20
Explain why Al is a member of group 13 rather than group 3?

This page titled 6.E: Electronic Structure and Periodic Properties (Exercises) is shared under a CC BY 4.0 license and was authored, remixed,
and/or curated by OpenStax via source content that was edited to the style and standards of the LibreTexts platform; a detailed edit history is
available upon request.

Access for free at OpenStax 6.E.15 https://chem.libretexts.org/@go/page/77623


CHAPTER OVERVIEW
7: Chemical Bonding and Molecular Geometry

A general chemistry Libretexts Textbook remixed and remastered from


OpenStax's textbook:
General Chemistry
A chemical bond is an attraction between atoms that allows the formation of chemical substances that contain two or more atoms.
The bond is caused by the electrostatic force of attraction between opposite charges, either between electrons and nuclei, or as the
result of a dipole attraction. All bonds can be explained by quantum theory, but, in practice, simplification rules allow chemists to
predict the strength, directionality, and polarity of bonds. The octet rule and VSEPR theory are two examples. More sophisticated
theories are valence bond theory which includes orbital hybridization and resonance, and the linear combination of atomic orbitals
molecular orbital method. Electrostatics are used to describe bond polarities and the effects they have on chemical substances.
7.0: Prelude to Chemical Bonding and Molecular Geometry
7.1: Ionic Bonding
7.2: Covalent Bonding
7.3: Lewis Symbols and Structures
7.4: Formal Charges and Resonance
7.5: Strengths of Ionic and Covalent Bonds
7.6: Molecular Structure and Polarity
7.E: Chemical Bonding and Molecular Geometry (Exercises)

Thumbnail: Covalently bonded hydrogen and carbon in a w:molecule of methane. (CC BY-SA 2.5; DynaBlast via Wikipedia)

This page titled 7: Chemical Bonding and Molecular Geometry is shared under a CC BY 4.0 license and was authored, remixed, and/or curated by
OpenStax via source content that was edited to the style and standards of the LibreTexts platform; a detailed edit history is available upon request.

1
7.0: Prelude to Chemical Bonding and Molecular Geometry
It has long been known that pure carbon occurs in different forms (allotropes) including graphite and diamonds. But it was not until
1985 that a new form of carbon was recognized: buckminsterfullerene, commonly known as a “buckyball.” This molecule was
named after the architect and inventor R. Buckminster Fuller (1895–1983), whose signature architectural design was the geodesic
dome, characterized by a lattice shell structure supporting a spherical surface. Experimental evidence revealed the formula, C60,
and then scientists determined how 60 carbon atoms could form one symmetric, stable molecule. They were guided by bonding
theory—the topic of this chapter—which explains how individual atoms connect to form more complex structures.

Figure 7.0.1 : Nicknamed “buckyballs,” buckminsterfullerene molecules (C60) contain only carbon atoms. Here they are shown in a
ball-and-stick model (left). These molecules have single and double carbon-carbon bonds arranged to form a geometric framework
of hexagons and pentagons, similar to the pattern on a soccer ball (center). This unconventional molecular structure is named after
architect R. Buckminster Fuller, whose innovative designs combined simple geometric shapes to create large, strong structures such
as this weather radar dome near Tucson, Arizona (right). (credit middle: modification of work by “Petey21”/Wikimedia Commons;
credit right: modification of work by Bill Morrow).
Three figures are shown. The left figure is a many-sides spherical ball composed of hexagonal rings which have carbon atoms at
each corner. The center picture shows a soccer ball. The right picture shown as water tower with sides shaped like hexagonal rings.

This page titled 7.0: Prelude to Chemical Bonding and Molecular Geometry is shared under a CC BY 4.0 license and was authored, remixed,
and/or curated by OpenStax via source content that was edited to the style and standards of the LibreTexts platform; a detailed edit history is
available upon request.

Access for free at OpenStax 7.0.1 https://chem.libretexts.org/@go/page/38182


7.1: Ionic Bonding
 Learning Objectives
Explain the formation of cations, anions, and ionic compounds
Predict the charge of common metallic and nonmetallic elements, and write their electron configurations

As you have learned, ions are atoms or molecules bearing an electrical charge. A cation (a positive ion) forms when a neutral atom
loses one or more electrons from its valence shell, and an anion (a negative ion) forms when a neutral atom gains one or more
electrons in its valence shell.
Compounds composed of ions are called ionic compounds (or salts), and their constituent ions are held together by ionic bonds:
electrostatic forces of attraction between oppositely charged cations and anions. The properties of ionic compounds shed some light
on the nature of ionic bonds. Ionic solids exhibit a crystalline structure and tend to be rigid and brittle; they also tend to have high
melting and boiling points, which suggests that ionic bonds are very strong. Ionic solids are also poor conductors of electricity for
the same reason—the strength of ionic bonds prevents ions from moving freely in the solid state. Most ionic solids, however,
dissolve readily in water. Once dissolved or melted, ionic compounds are excellent conductors of electricity and heat because the
ions can move about freely.
Neutral atoms and their associated ions have very different physical and chemical properties. Sodium atoms form sodium metal, a
soft, silvery-white metal that burns vigorously in air and reacts explosively with water. Chlorine atoms form chlorine gas, Cl2, a
yellow-green gas that is extremely corrosive to most metals and very poisonous to animals and plants. The vigorous reaction
between the elements sodium and chlorine forms the white, crystalline compound sodium chloride, common table salt, which
contains sodium cations and chloride anions (Figure Figure 7.1.1). The compound composed of these ions exhibits properties
entirely different from the properties of the elements sodium and chlorine. Chlorine is poisonous, but sodium chloride is essential
to life; sodium atoms react vigorously with water, but sodium chloride simply dissolves in water.

Figure 7.1.1 : (a) Sodium is a soft metal that must be stored in mineral oil to prevent reaction with air or water. (b) Chlorine is a
pale yellow-green gas. (c) When combined, they form white crystals of sodium chloride (table salt). (credit a: modification of work
by “Jurii”/Wikimedia Commons)

7.1.1: The Formation of Ionic Compounds


Binary ionic compounds are composed of just two elements: a metal (which forms the cations) and a nonmetal (which forms the
anions). For example, NaCl is a binary ionic compound. We can think about the formation of such compounds in terms of the
periodic properties of the elements. Many metallic elements have relatively low ionization potentials and lose electrons easily.
These elements lie to the left in a period or near the bottom of a group on the periodic table. Nonmetal atoms have relatively high
electron affinities and thus readily gain electrons lost by metal atoms, thereby filling their valence shells. Nonmetallic elements are
found in the upper-right corner of the periodic table.
As all substances must be electrically neutral, the total number of positive charges on the cations of an ionic compound must equal
the total number of negative charges on its anions. The formula of an ionic compound represents the simplest ratio of the numbers
of ions necessary to give identical numbers of positive and negative charges. For example, the formula for aluminum oxide, Al2O3,
indicates that this ionic compound contains two aluminum cations, Al3+, for every three oxide anions, O2− [thus, (2 × +3) + (3 × –
2) = 0].
It is important to note, however, that the formula for an ionic compound does not represent the physical arrangement of its ions. It
is incorrect to refer to a sodium chloride (NaCl) “molecule” because there is not a single ionic bond, per se, between any specific
pair of sodium and chloride ions. The attractive forces between ions are isotropic—the same in all directions—meaning that any

Access for free at OpenStax 7.1.1 https://chem.libretexts.org/@go/page/38183


particular ion is equally attracted to all of the nearby ions of opposite charge. This results in the ions arranging themselves into a
tightly bound, three-dimensional lattice structure. Sodium chloride, for example, consists of a regular arrangement of equal
numbers of Na+ cations and Cl– anions (Figure Figure 7.1.2).

Figure 7.1.2 : The atoms in sodium chloride (common table salt) are arranged to (a) maximize opposite charges interacting. The
smaller spheres represent sodium ions, the larger ones represent chloride ions. In the expanded view (b), the geometry can be seen
more clearly. Note that each ion is “bonded” to all of the surrounding ions—six in this case.
Two diagrams are shown and labeled “a” and “b.” Diagram a shows a cube made up of twenty-seven alternating purple and green
spheres. The purple spheres are smaller than the green spheres. Diagram b shows the same spheres, but this time, they are spread
out and connected in three dimensions by white rods. The purple spheres are labeled “N superscript postive sign” while the green
are labeled “C l superscript negative sign.”
The strong electrostatic attraction between Na+ and Cl– ions holds them tightly together in solid NaCl. It requires 769 kJ of energy
to dissociate one mole of solid NaCl into separate gaseous Na+ and Cl– ions:
+ −
NaCl(s) ⟶ Na (g) + Cl (g) ΔH = 769 kJ

7.1.2: Electronic Structures of Cations


When forming a cation, an atom of a main group element tends to lose all of its valence electrons, thus assuming the electronic
structure of the noble gas that precedes it in the periodic table. For groups 1 (the alkali metals) and 2 (the alkaline earth metals), the
group numbers are equal to the numbers of valence shell electrons and, consequently, to the charges of the cations formed from
atoms of these elements when all valence shell electrons are removed. For example, calcium is a group 2 element whose neutral
atoms have 20 electrons and a ground state electron configuration of 1s22s22p63s23p64s2. When a Ca atom loses both of its valence
electrons, the result is a cation with 18 electrons, a 2+ charge, and an electron configuration of 1s22s22p63s23p6. The Ca2+ ion is
therefore isoelectronic with the noble gas Ar.
For groups 12–17, the group numbers exceed the number of valence electrons by 10 (accounting for the possibility of full d
subshells in atoms of elements in the fourth and greater periods). Thus, the charge of a cation formed by the loss of all valence
electrons is equal to the group number minus 10. For example, aluminum (in group 13) forms 3+ ions (Al3+).
Exceptions to the expected behavior involve elements toward the bottom of the groups. In addition to the expected ions Tl3+, Sn4+,
Pb4+, and Bi5+, a partial loss of these atoms’ valence shell electrons can also lead to the formation of Tl+, Sn2+, Pb2+, and Bi3+ ions.
The formation of these 1+, 2+, and 3+ cations is ascribed to the inert pair effect, which reflects the relatively low energy of the
valence s-electron pair for atoms of the heavy elements of groups 13, 14, and 15. Mercury (group 12) also exhibits an unexpected
behavior: it forms a diatomic ion, Hg (an ion formed from two mercury atoms, with an Hg-Hg bond), in addition to the expected
2+

monatomic ion Hg2+ (formed from only one mercury atom).


Transition and inner transition metal elements behave differently than main group elements. Most transition metal cations have 2+
or 3+ charges that result from the loss of their outermost s electron(s) first, sometimes followed by the loss of one or two d
electrons from the next-to-outermost shell. For example, iron (1s22s22p63s23p63d64s2) forms the ion Fe2+ (1s22s22p63s23p63d6) by
the loss of the 4s electrons and the ion Fe3+ (1s22s22p63s23p63d5) by the loss of the 4s electrons and one of the 3d electrons.
Although the d orbitals of the transition elements are—according to the Aufbau principle—the last to fill when building up electron
configurations, the outermost s electrons are the first to be lost when these atoms ionize. When the inner transition metals form
ions, they usually have a 3+ charge, resulting from the loss of their outermost s electrons and a d or f electron.

Access for free at OpenStax 7.1.2 https://chem.libretexts.org/@go/page/38183


 Example 7.1.1: Determining the Electronic Structures of Cations

There are at least 14 elements categorized as “essential trace elements” for the human body. They are called “essential”
because they are required for healthy bodily functions, “trace” because they are required only in small amounts, and
“elements” in spite of the fact that they are really ions. Two of these essential trace elements, chromium and zinc, are required
as Cr3+ and Zn2+. Write the electron configurations of these cations.

Solution
First, write the electron configuration for the neutral atoms:
Zn: [Ar]3d104s2
Cr: [Ar]3d54s1
Next, remove electrons from the highest energy orbital. For the transition metals, electrons are removed from the s orbital first
and then from the d orbital. For the p-block elements, electrons are removed from the p orbitals and then from the s orbital.
Zinc is a member of group 12, so it should have a charge of 2+, and thus loses only the two electrons in its s orbital. Chromium
is a transition element and should lose its s electrons and then its d electrons when forming a cation. Thus, we find the
following electron configurations of the ions:
Zn2+: [Ar]3d10
Cr3+: [Ar]3d3

 Exercise 7.1.1

Potassium and magnesium are required in our diet. Write the electron configurations of the ions expected from these elements.

Answer
K+: [Ar], Mg2+: [Ne]

7.1.3: Electronic Structures of Anions


Most monatomic anions form when a neutral nonmetal atom gains enough electrons to completely fill its outer s and p orbitals,
thereby reaching the electron configuration of the next noble gas. Thus, it is simple to determine the charge on such a negative ion:
The charge is equal to the number of electrons that must be gained to fill the s and p orbitals of the parent atom. Oxygen, for
example, has the electron configuration 1s22s22p4, whereas the oxygen anion has the electron configuration of the noble gas neon
(Ne), 1s22s22p6. The two additional electrons required to fill the valence orbitals give the oxide ion the charge of 2– (O2–).

 Example 7.1.2: Determining the Electronic Structure of Anions

Selenium and iodine are two essential trace elements that form anions. Write the electron configurations of the anions.

Solution
Se2–: [Ar]3d104s24p6
I–: [Kr]4d105s25p6

 Exercise 7.1.2

Write the electron configurations of a phosphorus atom and its negative ion. Give the charge on the anion.

Answer

Access for free at OpenStax 7.1.3 https://chem.libretexts.org/@go/page/38183


P: [Ne]3s23p3
P3–: [Ne]3s23p6

Summary
Atoms gain or lose electrons to form ions with particularly stable electron configurations. The charges of cations formed by the
representative metals may be determined readily because, with few exceptions, the electronic structures of these ions have either a
noble gas configuration or a completely filled electron shell. The charges of anions formed by the nonmetals may also be readily
determined because these ions form when nonmetal atoms gain enough electrons to fill their valence shells.

Glossary
inert pair effect
tendency of heavy atoms to form ions in which their valence s electrons are not lost
ionic bond
strong electrostatic force of attraction between cations and anions in an ionic compound

This page titled 7.1: Ionic Bonding is shared under a CC BY 4.0 license and was authored, remixed, and/or curated by OpenStax via source
content that was edited to the style and standards of the LibreTexts platform; a detailed edit history is available upon request.

Access for free at OpenStax 7.1.4 https://chem.libretexts.org/@go/page/38183


7.2: Covalent Bonding
 Learning Objectives
Describe the formation of covalent bonds
Define electronegativity and assess the polarity of covalent bonds

In ionic compounds, electrons are transferred between atoms of different elements to form ions. But this is not the only way that
compounds can be formed. Atoms can also make chemical bonds by sharing electrons between each other. Such bonds are called
covalent bonds. Covalent bonds are formed between two atoms when both have similar tendencies to attract electrons to
themselves (i.e., when both atoms have identical or fairly similar ionization energies and electron affinities). For example, two
hydrogen atoms bond covalently to form an H2 molecule; each hydrogen atom in the H2 molecule has two electrons stabilizing it,
giving each atom the same number of valence electrons as the noble gas He.
Compounds that contain covalent bonds exhibit different physical properties than ionic compounds. Because the attraction between
molecules, which are electrically neutral, is weaker than that between electrically charged ions, covalent compounds generally have
much lower melting and boiling points than ionic compounds. In fact, many covalent compounds are liquids or gases at room
temperature, and, in their solid states, they are typically much softer than ionic solids. Furthermore, whereas ionic compounds are
good conductors of electricity when dissolved in water, most covalent compounds, being electrically neutral, are poor conductors
of electricity in any state.

7.2.1: Formation of Covalent Bonds


Nonmetal atoms frequently form covalent bonds with other nonmetal atoms. For example, the hydrogen molecule, H2, contains a
covalent bond between its two hydrogen atoms. Figure 7.2.1 illustrates why this bond is formed. Starting on the far right, we have
two separate hydrogen atoms with a particular potential energy, indicated by the red line. Along the x-axis is the distance between
the two atoms. As the two atoms approach each other (moving left along the x-axis), their valence orbitals (1s) begin to overlap.
The single electrons on each hydrogen atom then interact with both atomic nuclei, occupying the space around both atoms. The
strong attraction of each shared electron to both nuclei stabilizes the system, and the potential energy decreases as the bond
distance decreases. If the atoms continue to approach each other, the positive charges in the two nuclei begin to repel each other,
and the potential energy increases. The bond length is determined by the distance at which the lowest potential energy is achieved.

Access for free at OpenStax 7.2.1 https://chem.libretexts.org/@go/page/38184


Figure 7.2.1 : The potential energy of two separate hydrogen atoms (right) decreases as they approach each other, and the single
electrons on each atom are shared to form a covalent bond. The bond length is the internuclear distance at which the lowest
potential energy is achieved.
It is essential to remember that energy must be added to break chemical bonds (an endothermic process), whereas forming chemical
bonds releases energy (an exothermic process). In the case of H2, the covalent bond is very strong; a large amount of energy, 436
kJ, must be added to break the bonds in one mole of hydrogen molecules and cause the atoms to separate:

H (g) ⟶ 2 H(g) ΔH = 436 kJ


2

Conversely, the same amount of energy is released when one mole of H2 molecules forms from two moles of H atoms:

2 H(g) ⟶ H (g) ΔH = −436 kJ


2

7.2.2: Pure vs. Polar Covalent Bonds


If the atoms that form a covalent bond are identical, as in H2, Cl2, and other diatomic molecules, then the electrons in the bond
must be shared equally. We refer to this as a pure covalent bond. Electrons shared in pure covalent bonds have an equal probability
of being near each nucleus. In the case of Cl2, each atom starts off with seven valence electrons, and each Cl shares one electron
with the other, forming one covalent bond:

Cl + Cl ⟶ Cl
2

The total number of electrons around each individual atom consists of six nonbonding electrons and two shared (i.e., bonding)
electrons for eight total electrons, matching the number of valence electrons in the noble gas argon. Since the bonding atoms are
identical, Cl2 also features a pure covalent bond.
When the atoms linked by a covalent bond are different, the bonding electrons are shared, but no longer equally. Instead, the
bonding electrons are more attracted to one atom than the other, giving rise to a shift of electron density toward that atom. This
unequal distribution of electrons is known as a polar covalent bond, characterized by a partial positive charge on one atom and a
partial negative charge on the other. The atom that attracts the electrons more strongly acquires the partial negative charge and vice
versa. For example, the electrons in the H–Cl bond of a hydrogen chloride molecule spend more time near the chlorine atom than
near the hydrogen atom. Thus, in an HCl molecule, the chlorine atom carries a partial negative charge and the hydrogen atom has a
partial positive charge. Figure 7.2.2 shows the distribution of electrons in the H–Cl bond. Note that the shaded area around Cl is
much larger than it is around H. Compare this to Figure 7.2.1, which shows the even distribution of electrons in the H2 nonpolar
bond.

Access for free at OpenStax 7.2.2 https://chem.libretexts.org/@go/page/38184


Figure 7.2.2 : (a) The distribution of electron density in the HCl molecule is uneven. The electron density is greater around the
chlorine nucleus. The small, black dots indicate the location of the hydrogen and chlorine nuclei in the molecule. (b) Symbols δ+
and δ– indicate the polarity of the H–Cl bond.
We sometimes designate the positive and negative atoms in a polar covalent bond using a lowercase Greek letter “delta,” δ, with a
plus sign or minus sign to indicate whether the atom has a partial positive charge (δ+) or a partial negative charge (δ–). This
symbolism is shown for the H–Cl molecule in Figure 7.2.2b.

7.2.3: Electronegativity
Whether a bond is nonpolar or polar covalent is determined by a property of the bonding atoms called electronegativity.
Electronegativity is a measure of the tendency of an atom to attract electrons (or electron density) towards itself. It determines how
the shared electrons are distributed between the two atoms in a bond. The more strongly an atom attracts the electrons in its bonds,
the larger its electronegativity. Electrons in a polar covalent bond are shifted toward the more electronegative atom; thus, the more
electronegative atom is the one with the partial negative charge. The greater the difference in electronegativity, the more polarized
the electron distribution and the larger the partial charges of the atoms.
Figure 7.2.3 shows the electronegativity values of the elements as proposed by one of the most famous chemists of the twentieth
century: Linus Pauling. In general, electronegativity increases from left to right across a period in the periodic table and decreases
down a group. Thus, the nonmetals, which lie in the upper right, tend to have the highest electronegativities, with fluorine the most
electronegative element of all (EN = 4.0). Metals tend to be less electronegative elements, and the group 1 metals have the lowest
electronegativities. Note that noble gases are excluded from this figure because these atoms usually do not share electrons with
others atoms since they have a full valence shell. (While noble gas compounds such as XeO2 do exist, they can only be formed
under extreme conditions, and thus they do not fit neatly into the general model of electronegativity.)

Figure 7.2.3 : The electronegativity values derived by Pauling follow predictable periodic trends with the higher electronegativities
toward the upper right of the periodic table.

 Linus Pauling

Linus Pauling is the only person to have received two unshared (individual) Nobel Prizes: one for chemistry in 1954 for his
work on the nature of chemical bonds and one for peace in 1962 for his opposition to weapons of mass destruction. He
developed many of the theories and concepts that are foundational to our current understanding of chemistry, including
electronegativity and resonance structures.

Access for free at OpenStax 7.2.3 https://chem.libretexts.org/@go/page/38184


Linus Pauling (1901–1994) made many important contributions to the field of chemistry. He was also a prominent activist,
publicizing issues related to health and nuclear weapons.
Pauling also contributed to many other fields besides chemistry. His research on sickle cell anemia revealed the cause of the
disease—the presence of a genetically inherited abnormal protein in the blood—and paved the way for the field of molecular
genetics. His work was also pivotal in curbing the testing of nuclear weapons; he proved that radioactive fallout from nuclear
testing posed a public health risk.

7.2.4: Electronegativity versus Electron Affinity


We must be careful not to confuse electronegativity and electron affinity. The electron affinity of an element is a measurable
physical quantity, namely, the energy released or absorbed when an isolated gas-phase atom acquires an electron, measured in
kJ/mol. Electronegativity, on the other hand, describes how tightly an atom attracts electrons in a bond. It is a dimensionless
quantity that is calculated, not measured. Pauling derived the first electronegativity values by comparing the amounts of energy
required to break different types of bonds. He chose an arbitrary relative scale ranging from 0 to 4.

7.2.5: Electronegativity and Bond Type


The absolute value of the difference in electronegativity (ΔEN) of two bonded atoms provides a rough measure of the polarity to be
expected in the bond and, thus, the bond type. When the difference is very small or zero, the bond is covalent and nonpolar. When
it is large, the bond is polar covalent or ionic. The absolute values of the electronegativity differences between the atoms in the
bonds H–H, H–Cl, and Na–Cl are 0 (nonpolar), 0.9 (polar covalent), and 2.1 (ionic), respectively. The degree to which electrons are
shared between atoms varies from completely equal (pure covalent bonding) to not at all (ionic bonding). Figure 7.2.4 shows the
relationship between electronegativity difference and bond type.

Figure 7.2.4 : As the electronegativity difference increases between two atoms, the bond becomes more ionic.
A rough approximation of the electronegativity differences associated with covalent, polar covalent, and ionic bonds is shown in
Figure 7.2.4. This table is just a general guide, however, with many exceptions. For example, the H and F atoms in HF have an
electronegativity difference of 1.9, and the N and H atoms in NH3 a difference of 0.9, yet both of these compounds form bonds that
are considered polar covalent. Likewise, the Na and Cl atoms in NaCl have an electronegativity difference of 2.1, and the Mn and I
atoms in MnI2 have a difference of 1.0, yet both of these substances form ionic compounds.
The best guide to the covalent or ionic character of a bond is to consider the types of atoms involved and their relative positions in
the periodic table. Bonds between two nonmetals are generally covalent; bonding between a metal and a nonmetal is often ionic.
Some compounds contain both covalent and ionic bonds. The atoms in polyatomic ions, such as OH–, NO , and NH , are held

3
+

together by polar covalent bonds. However, these polyatomic ions form ionic compounds by combining with ions of opposite

Access for free at OpenStax 7.2.4 https://chem.libretexts.org/@go/page/38184


charge. For example, potassium nitrate, KNO3, contains the K+ cation and the polyatomic NO anion. Thus, bonding in potassium

nitrate is ionic, resulting from the electrostatic attraction between the ions K+ and NO , as well as covalent between the nitrogen

and oxygen atoms in NO . −

 Example 7.2.1: Electronegativity and Bond Polarity

Bond polarities play an important role in determining the structure of proteins. Using the electronegativity values in Table A2,
arrange the following covalent bonds—all commonly found in amino acids—in order of increasing polarity. Then designate the
positive and negative atoms using the symbols δ+ and δ–:
C–H, C–N, C–O, N–H, O–H, S–H

Solution
The polarity of these bonds increases as the absolute value of the electronegativity difference increases. The atom with the δ–
designation is the more electronegative of the two. Table 7.2.1 shows these bonds in order of increasing polarity.
Table 7.2.1 : Bond Polarity and Electronegativity Difference
Bond ΔEN Polarity
δ− δ+
C–H 0.4 C − H

δ− δ+
S–H 0.4 S − H

δ+ δ−
C–N 0.5 C − N

δ− δ+
N–H 0.9 N − H

δ+ δ−
C–O 1.0 C − O

δ− δ+
O–H 1.4 O − H

 Exercise 7.2.1

Silicones are polymeric compounds containing, among others, the following types of covalent bonds: Si–O, Si–C, C–H, and
C–C. Using the electronegativity values in Figure 7.2.3, arrange the bonds in order of increasing polarity and designate the
positive and negative atoms using the symbols δ+ and δ–.

Answer
Answer to Exercise 7.2.1
Bond Electronegativity Difference Polarity

C–C 0.0 nonpolar


δ− δ+
C–H 0.4 C − H

δ+ δ−
Si–C 0.7 Si − C

δ+ δ−
Si–O 1.7 Si − O

Summary
Covalent bonds form when electrons are shared between atoms and are attracted by the nuclei of both atoms. In pure covalent
bonds, the electrons are shared equally. In polar covalent bonds, the electrons are shared unequally, as one atom exerts a stronger
force of attraction on the electrons than the other. The ability of an atom to attract a pair of electrons in a chemical bond is called its
electronegativity. The difference in electronegativity between two atoms determines how polar a bond will be. In a diatomic

Access for free at OpenStax 7.2.5 https://chem.libretexts.org/@go/page/38184


molecule with two identical atoms, there is no difference in electronegativity, so the bond is nonpolar or pure covalent. When the
electronegativity difference is very large, as is the case between metals and nonmetals, the bonding is characterized as ionic.

Glossary
bond length
distance between the nuclei of two bonded atoms at which the lowest potential energy is achieved

covalent bond
bond formed when electrons are shared between atoms

electronegativity
tendency of an atom to attract electrons in a bond to itself

polar covalent bond


covalent bond between atoms of different electronegativities; a covalent bond with a positive end and a negative end
pure covalent bond
(also, nonpolar covalent bond) covalent bond between atoms of identical electronegativities

This page titled 7.2: Covalent Bonding is shared under a CC BY 4.0 license and was authored, remixed, and/or curated by OpenStax via source
content that was edited to the style and standards of the LibreTexts platform; a detailed edit history is available upon request.

Access for free at OpenStax 7.2.6 https://chem.libretexts.org/@go/page/38184


7.3: Lewis Symbols and Structures
 Learning Objectives
Write Lewis symbols for neutral atoms and ions
Draw Lewis structures depicting the bonding in simple molecules

Thus far, we have discussed the various types of bonds that form between atoms and/or ions. In all cases, these bonds involve the
sharing or transfer of valence shell electrons between atoms. In this section, we will explore the typical method for depicting
valence shell electrons and chemical bonds, namely Lewis symbols and Lewis structures.

7.3.1: Lewis Symbols


We use Lewis symbols to describe valence electron configurations of atoms and monatomic ions. A Lewis symbol consists of an
elemental symbol surrounded by one dot for each of its valence electrons:

Figure 7.3.1 shows the Lewis symbols for the elements of the third period of the periodic table. Electron dots are typically arranged
in four pairs located on the four "sides" of the atomic symbol.

Figure 7.3.1 : Lewis symbols illustrating the number of valence electrons for each element in the third period of the periodic table.

Lewis symbols can be used to illustrate the formation of cations from atoms, as shown here for sodium and calcium:

Likewise, they can be used to show the formation of anions from atoms, as shown here for chlorine and sulfur:

Figure 7.3.2 demonstrates the use of Lewis symbols to show the transfer of electrons during the formation of ionic compounds.

Access for free at OpenStax 7.3.1 https://chem.libretexts.org/@go/page/38185


Figure 7.3.2 : Cations are formed when atoms lose electrons, represented by fewer Lewis dots, whereas anions are formed by atoms
gaining electrons. The total number of electrons does not change.

7.3.2: Lewis Structures


We also use Lewis symbols to indicate the formation of covalent bonds, which are shown in Lewis structures, drawings that
describe the bonding in molecules and polyatomic ions. For example, when two chlorine atoms form a chlorine molecule, they
share one pair of electrons:

The Lewis structure indicates that each Cl atom has three pairs of electrons that are not used in bonding (called lone pairs) and one
shared pair of electrons (written between the atoms). A dash (or line) is usually used to indicate a shared pair of electrons:

In the Lewis model, a single shared pair of electrons is a single bond. Each Cl atom interacts with eight valence electrons total: the
six in the lone pairs and the two in the single bond.

7.3.3: The Octet Rule


The other halogen molecules (F2, Br2, I2, and At2) form bonds like those in the chlorine molecule: one single bond between atoms
and three lone pairs of electrons per atom. This allows each halogen atom to have a noble gas electron configuration. The tendency
of main group atoms to form enough bonds to obtain eight valence electrons is known as the octet rule.
The number of bonds that an atom can form can often be predicted from the number of electrons needed to reach an octet (eight
valence electrons); this is especially true of the nonmetals of the second period of the periodic table (C, N, O, and F). For example,
each atom of a group 14 element has four electrons in its outermost shell and therefore requires four more electrons to reach an
octet. These four electrons can be gained by forming four covalent bonds, as illustrated here for carbon in CCl4 (carbon
tetrachloride) and silicon in SiH4 (silane). Because hydrogen only needs two electrons to fill its valence shell, it is an exception to
the octet rule and only needs to form one bond. The transition elements and inner transition elements also do not follow the octet
rule since they have d and f electrons involved in their valence shells.

Access for free at OpenStax 7.3.2 https://chem.libretexts.org/@go/page/38185


Group 15 elements such as nitrogen have five valence electrons in the atomic Lewis symbol: one lone pair and three unpaired
electrons. To obtain an octet, these atoms form three covalent bonds, as in NH3 (ammonia). Oxygen and other atoms in group 16
obtain an octet by forming two covalent bonds:

7.3.4: Double and Triple Bonds


As previously mentioned, when a pair of atoms shares one pair of electrons, we call this a single bond. However, a pair of atoms
may need to share more than one pair of electrons in order to achieve the requisite octet. A double bond forms when two pairs of
electrons are shared between a pair of atoms, as between the carbon and oxygen atoms in CH2O (formaldehyde) and between the
two carbon atoms in C2H4 (ethylene):

A triple bond forms when three electron pairs are shared by a pair of atoms, as in carbon monoxide (CO) and the cyanide ion
(CN–):

7.3.5: Writing Lewis Structures with the Octet Rule


For very simple molecules and molecular ions, we can write the Lewis structures by merely pairing up the unpaired electrons on
the constituent atoms. See these examples:

For more complicated molecules and molecular ions, it is helpful to follow the step-by-step procedure outlined here:
1. Determine the total number of valence (outer shell) electrons among all the atoms. For cations, subtract one electron for each
positive charge. For anions, add one electron for each negative charge.
2. Draw a skeleton structure of the molecule or ion, arranging the atoms around a central atom. (Generally, the least
electronegative element should be placed in the center.) Connect each atom to the central atom with a single bond (one electron
pair).
3. Distribute the remaining electrons as lone pairs on the terminal atoms (except hydrogen), completing an octet around each
atom.

Access for free at OpenStax 7.3.3 https://chem.libretexts.org/@go/page/38185


4. Place all remaining electrons on the central atom.
5. Rearrange the electrons of the outer atoms to make multiple bonds with the central atom in order to obtain octets wherever
possible.
Let us determine the Lewis structures of SiH4, CHO , NO+, and OF2 as examples in following this procedure:

2

1. Determine the total number of valence (outer shell) electrons in the molecule or ion.
For a molecule, we add the number of valence electrons on each atom in the molecule:
SiH (7.3.1)
4

Si: 4 valence electrons/atom × 1 atom = 4 (7.3.2)

+H: 1 valence electron/atom × 4 atoms = 4 (7.3.3)


––––––––––––––––––––––––––––––––––––––––––
= 8 valence electrons (7.3.4)

For a negative ion, such as CHO , we add the number of valence electrons on the atoms to the number of negative charges

on the ion (one electron is gained for each single negative charge):

CHO
2

C: 4 valence electrons/atom × 1 atom = 4

H: 1 valence electron/atom × 1 atom = 1

O: 6 valence electrons/atom × 2 atoms = 12

+ 1 additional electron = 1
––––––––––––––––––––––––––––––––––––––––
= 18 valence electrons
For a positive ion, such as NO+, we add the number of valence electrons on the atoms in the ion and then subtract the
number of positive charges on the ion (one electron is lost for each single positive charge) from the total number of valence
electrons:
+
NO

N: 5 valence electrons/atom × 1 atom = 5

O: 6 valence electron/atom × 1 atom = 6

+ −1 electron (positive charge) = −1


––––––––––––––––––––––––––––––––––––––––––
= 10 valence electrons
Since OF2 is a neutral molecule, we simply add the number of valence electrons:
OF
2

O: 6 valence electrons/atom × 1 atom =6

+ F: 7 valence electrons/atom × 2 atoms = 14


–––––––––––––––––––––––––––––––––––––––––––––
= 20 valence electrons

2. Draw a skeleton structure of the molecule or ion, arranging the atoms around a central atom and connecting each atom to the
central atom with a single (one electron pair) bond. (Note that we denote ions with brackets around the structure, indicating the
charge outside the brackets:)

3. When several arrangements of atoms are possible, as for CHO , we must use experimental evidence to choose the correct one.

In general, the less electronegative elements are more likely to be central atoms. In CHO , the less electronegative carbon

atom occupies the central position with the oxygen and hydrogen atoms surrounding it. Other examples include P in POCl3, S in
SO2, and Cl in ClO . An exception is that hydrogen is almost never a central atom. As the most electronegative element,

fluorine also cannot be a central atom.


4. Distribute the remaining electrons as lone pairs on the terminal atoms (except hydrogen) to complete their valence shells with
an octet of electrons.
There are no remaining electrons on SiH4, so it is unchanged:

Access for free at OpenStax 7.3.4 https://chem.libretexts.org/@go/page/38185


5. Place all remaining electrons on the central atom.
For SiH4, CHO , and NO+, there are no remaining electrons; we already placed all of the electrons determined in Step 1.

For OF2, we had 16 electrons remaining in Step 3, and we placed 12, leaving 4 to be placed on the central atom:

6. Rearrange the electrons of the outer atoms to make multiple bonds with the central atom in order to obtain octets wherever
possible.
SiH4: Si already has an octet, so nothing needs to be done.
CHO : We have distributed the valence electrons as lone pairs on the oxygen atoms, but the carbon atom lacks an octet:

NO+: For this ion, we added eight outer electrons, but neither atom has an octet. We cannot add any more electrons since we
have already used the total that we found in Step 1, so we must move electrons to form a multiple bond:

This still does not produce an octet, so we must move another pair, forming a triple bond:

In OF2, each atom has an octet as drawn, so nothing changes.

 Example 7.3.1: Writing Lewis Structures

NASA’s Cassini-Huygens mission detected a large cloud of toxic hydrogen cyanide (HCN) on Titan, one of Saturn’s moons.
Titan also contains ethane (H3CCH3), acetylene (HCCH), and ammonia (NH3). What are the Lewis structures of these
molecules?
Solution
Calculate the number of valence electrons.
HCN: (1 × 1) + (4 × 1) + (5 × 1) = 10
H3CCH3: (1 × 3) + (2 × 4) + (1 × 3) = 14
HCCH: (1 × 1) + (2 × 4) + (1 × 1) = 10
NH3: (5 × 1) + (3 × 1) = 8
Draw a skeleton and connect the atoms with single bonds. Remember that H is never a central atom:

Where needed, distribute electrons to the terminal atoms:

HCN: six electrons placed on N


H3CCH3: no electrons remain

Access for free at OpenStax 7.3.5 https://chem.libretexts.org/@go/page/38185


HCCH: no terminal atoms capable of accepting electrons
NH3: no terminal atoms capable of accepting electrons
Where needed, place remaining electrons on the central atom:

HCN: no electrons remain


H3CCH3: no electrons remain
HCCH: four electrons placed on carbon
NH3: two electrons placed on nitrogen
Where needed, rearrange electrons to form multiple bonds in order to obtain an octet on each atom:
HCN: form two more C–N bonds
H3CCH3: all atoms have the correct number of electrons
HCCH: form a triple bond between the two carbon atoms
NH3: all atoms have the correct number of electrons

 Exercise 7.3.1

Both carbon monoxide, CO, and carbon dioxide, CO2, are products of the combustion of fossil fuels. Both of these gases also
cause problems: CO is toxic and CO2 has been implicated in global climate change. What are the Lewis structures of these two
molecules?

Answer

 Fullerene Chemistry
Carbon soot has been known to man since prehistoric times, but it was not until fairly recently that the molecular structure of
the main component of soot was discovered. In 1996, the Nobel Prize in Chemistry was awarded to Richard Smalley, Robert
Curl, and Harold Kroto for their work in discovering a new form of carbon, the C60 buckminsterfullerene molecule. An entire
class of compounds, including spheres and tubes of various shapes, were discovered based on C60. This type of molecule,
called a fullerene, consists of a complex network of single- and double-bonded carbon atoms arranged in such a way that each
carbon atom obtains a full octet of electrons. Because of their size and shape, fullerenes can encapsulate other molecules, so
they have shown potential in various applications from hydrogen storage to targeted drug delivery systems. They also possess
unique electronic and optical properties that have been put to good use in solar powered devices and chemical sensors.

7.3.6: Exceptions to the Octet Rule


Many covalent molecules have central atoms that do not have eight electrons in their Lewis structures. These molecules fall into
three categories:
Odd-electron molecules have an odd number of valence electrons, and therefore have an unpaired electron.
Electron-deficient molecules have a central atom that has fewer electrons than needed for a noble gas configuration.
Hypervalent molecules have a central atom that has more electrons than needed for a noble gas configuration.

Access for free at OpenStax 7.3.6 https://chem.libretexts.org/@go/page/38185


7.3.6.1: Odd-electron Molecules
We call molecules that contain an odd number of electrons free radicals. Nitric oxide, NO, is an example of an odd-electron
molecule; it is produced in internal combustion engines when oxygen and nitrogen react at high temperatures.
To draw the Lewis structure for an odd-electron molecule like NO, we follow the same five steps we would for other molecules,
but with a few minor changes:
1. Determine the total number of valence (outer shell) electrons. The sum of the valence electrons is 5 (from N) + 6 (from O) = 11.
The odd number immediately tells us that we have a free radical, so we know that not every atom can have eight electrons in its
valence shell.
2. Draw a skeleton structure of the molecule. We can easily draw a skeleton with an N–O single bond: N–O
3. Distribute the remaining electrons as lone pairs on the terminal atoms. In this case, there is no central atom, so we distribute the
electrons around both atoms. We give eight electrons to the more electronegative atom in these situations; thus oxygen has the
filled valence shell:

4. Place all remaining electrons on the central atom. Since there are no remaining electrons, this step does not apply.
5. Rearrange the electrons to make multiple bonds with the central atom in order to obtain octets wherever possible. We know that
an odd-electron molecule cannot have an octet for every atom, but we want to get each atom as close to an octet as possible. In
this case, nitrogen has only five electrons around it. To move closer to an octet for nitrogen, we take one of the lone pairs from
oxygen and use it to form a NO double bond. (We cannot take another lone pair of electrons on oxygen and form a triple bond
because nitrogen would then have nine electrons:)

7.3.6.2: Electron-deficient Molecules


We will also encounter a few molecules that contain central atoms that do not have a filled valence shell. Generally, these are
molecules with central atoms from groups 2 and 13 and outer atoms that are hydrogen or other atoms that do not form multiple
bonds. For example, in the Lewis structures of beryllium dihydride, BeH2, and boron trifluoride, BF3, the beryllium and boron
atoms each have only four and six electrons, respectively. It is possible to draw a structure with a double bond between a boron
atom and a fluorine atom in BF3, satisfying the octet rule, but experimental evidence indicates the bond lengths are closer to that
expected for B–F single bonds. This suggests the best Lewis structure has three B–F single bonds and an electron deficient boron.
The reactivity of the compound is also consistent with an electron deficient boron. However, the B–F bonds are slightly shorter
than what is actually expected for B–F single bonds, indicating that some double bond character is found in the actual molecule.

An atom like the boron atom in BF3, which does not have eight electrons, is very reactive. It readily combines with a molecule
containing an atom with a lone pair of electrons. For example, NH3 reacts with BF3 because the lone pair on nitrogen can be shared
with the boron atom:

7.3.6.3: Hypervalent Molecules


Elements in the second period of the periodic table (n = 2) can accommodate only eight electrons in their valence shell orbitals
because they have only four valence orbitals (one 2s and three 2p orbitals). Elements in the third and higher periods (n ≥ 3) have
more than four valence orbitals and can share more than four pairs of electrons with other atoms because they have empty d orbitals

Access for free at OpenStax 7.3.7 https://chem.libretexts.org/@go/page/38185


in the same shell. Molecules formed from these elements are sometimes called hypervalent molecules.Table 7.3.5 shows the Lewis
structures for two hypervalent molecules, PCl5 and SF6.

Table 7.3.5: In PCl5, the central atom phosphorus shares five pairs of electrons. In SF6, sulfur shares six pairs of electrons.
In some hypervalent molecules, such as IF5 and XeF4, some of the electrons in the outer shell of the central atom are lone pairs:

When we write the Lewis structures for these molecules, we find that we have electrons left over after filling the valence shells of
the outer atoms with eight electrons. These additional electrons must be assigned to the central atom.

 Example 7.3.2: Octet Rule Violations

Xenon is a noble gas, but it forms a number of stable compounds. We examined XeF earlier. What are the Lewis structures of
4

XeF and XeF ?


2 6

Solution
We can draw the Lewis structure of any covalent molecule by following the six steps discussed earlier. In this case, we can
condense the last few steps, since not all of them apply.
Step 1: Calculate the number of valence electrons:
XeF
2
: 8 + (2 × 7) = 22

XeF
6
: 8 + (6 × 7) = 50
Step 2: Draw a skeleton joining the atoms by single bonds. Xenon will be the central atom because fluorine cannot be a
central atom:

Step 3: Distribute the remaining electrons.


XeF2: We place three lone pairs of electrons around each F atom, accounting for 12 electrons and giving each F atom 8 electrons.
Thus, six electrons (three lone pairs) remain. These lone pairs must be placed on the Xe atom. This is acceptable because Xe atoms
have empty valence shell d orbitals and can accommodate more than eight electrons. The Lewis structure of XeF2 shows two
bonding pairs and three lone pairs of electrons around the Xe atom:

XeF6: We place three lone pairs of electrons around each F atom, accounting for 36 electrons. Two electrons remain, and this lone
pair is placed on the Xe atom:

Access for free at OpenStax 7.3.8 https://chem.libretexts.org/@go/page/38185


 Exercise 7.3.2: interhalogens

The halogens form a class of compounds called the interhalogens, in which halogen atoms covalently bond to each other. Write
the Lewis structures for the interhalogens BrCl and ICl .
3

4

Answer

Summary
Valence electronic structures can be visualized by drawing Lewis symbols (for atoms and monatomic ions) and Lewis structures
(for molecules and polyatomic ions). Lone pairs, unpaired electrons, and single, double, or triple bonds are used to indicate where
the valence electrons are located around each atom in a Lewis structure. Most structures—especially those containing second row
elements—obey the octet rule, in which every atom (except H) is surrounded by eight electrons. Exceptions to the octet rule occur
for odd-electron molecules (free radicals), electron-deficient molecules, and hypervalent molecules.

Glossary
double bond
covalent bond in which two pairs of electrons are shared between two atoms

free radical
molecule that contains an odd number of electrons

hypervalent molecule
molecule containing at least one main group element that has more than eight electrons in its valence shell

Lewis structure
diagram showing lone pairs and bonding pairs of electrons in a molecule or an ion

Lewis symbol
symbol for an element or monatomic ion that uses a dot to represent each valence electron in the element or ion

lone pair
two (a pair of) valence electrons that are not used to form a covalent bond

octet rule
guideline that states main group atoms will form structures in which eight valence electrons interact with each nucleus, counting
bonding electrons as interacting with both atoms connected by the bond

single bond

Access for free at OpenStax 7.3.9 https://chem.libretexts.org/@go/page/38185


bond in which a single pair of electrons is shared between two atoms
triple bond
bond in which three pairs of electrons are shared between two atoms

This page titled 7.3: Lewis Symbols and Structures is shared under a CC BY 4.0 license and was authored, remixed, and/or curated by OpenStax
via source content that was edited to the style and standards of the LibreTexts platform; a detailed edit history is available upon request.

Access for free at OpenStax 7.3.10 https://chem.libretexts.org/@go/page/38185


7.4: Formal Charges and Resonance
 Learning Objectives
Compute formal charges for atoms in any Lewis structure
Use formal charges to identify the most reasonable Lewis structure for a given molecule
Explain the concept of resonance and draw Lewis structures representing resonance forms for a given molecule

Previously, we discussed how to write Lewis structures for molecules and polyatomic ions. In some cases, however, there is
seemingly more than one valid structure for a molecule. We can use the concept of formal charges to help us predict the most
appropriate Lewis structure when more than one is reasonable.

7.4.1: Calculating Formal Charge


The formal charge of an atom in a molecule is the hypothetical charge the atom would have if we could redistribute the electrons in
the bonds evenly between the atoms. Another way of saying this is that formal charge results when we take the number of valence
electrons of a neutral atom, subtract the nonbonding electrons, and then subtract the number of bonds connected to that atom in the
Lewis structure.
Thus, we calculate formal charge as follows:
1
formal charge = # valence shell electrons (free atom) − # lone pair electrons −   # bonding electrons
2

We can double-check formal charge calculations by determining the sum of the formal charges for the whole structure. The sum of
the formal charges of all atoms in a molecule must be zero; the sum of the formal charges in an ion should equal the charge of the
ion.
We must remember that the formal charge calculated for an atom is not the actual charge of the atom in the molecule. Formal
charge is only a useful bookkeeping procedure; it does not indicate the presence of actual charges.

 Example 7.4.1: Calculating Formal Charge from Lewis Structures


Assign formal charges to each atom in the interhalogen ion ICl . −
4

Solution
We divide the bonding electron pairs equally for all I– Cl bonds:

We assign lone pairs of electrons to their atoms. Each Cl atom now has seven electrons assigned to it, and the I atom has eight.
Subtract this number from the number of valence electrons for the neutral atom:
I: 7 – 8 = –1
Cl: 7 – 7 = 0
The sum of the formal charges of all the atoms equals –1, which is identical to the charge of the ion (–1).

Access for free at OpenStax 7.4.1 https://chem.libretexts.org/@go/page/38186


 Exercise 7.4.1

Calculate the formal charge for each atom in the carbon monoxide molecule:

Answer
C −1, O +1

 Example 7.4.2: Calculating Formal Charge from Lewis Structures

Assign formal charges to each atom in the interhalogen molecule BrCl . 3

Solution
Assign one of the electrons in each Br–Cl bond to the Br atom and one to the Cl atom in that bond:

Assign the lone pairs to their atom. Now each Cl atom has seven electrons and the Br atom has seven electrons.
Subtract this number from the number of valence electrons for the neutral atom. This gives the formal charge:
Br: 7 – 7 = 0
Cl: 7 – 7 = 0
All atoms in BrCl
3
have a formal charge of zero, and the sum of the formal charges totals zero, as it must in a neutral
molecule.

 Exercise 7.4.2

Determine the formal charge for each atom in NCl . 3

Answer
N: 0; all three Cl atoms: 0

7.4.2: Using Formal Charge to Predict Molecular Structure


The arrangement of atoms in a molecule or ion is called its molecular structure. In many cases, following the steps for writing
Lewis structures may lead to more than one possible molecular structure—different multiple bond and lone-pair electron
placements or different arrangements of atoms, for instance. A few guidelines involving formal charge can be helpful in deciding
which of the possible structures is most likely for a particular molecule or ion.

Access for free at OpenStax 7.4.2 https://chem.libretexts.org/@go/page/38186


 Predicting Molecular Structure Guidelines
1. A molecular structure in which all formal charges are zero is preferable to one in which some formal charges are not zero.
2. If the Lewis structure must have nonzero formal charges, the arrangement with the smallest nonzero formal charges is
preferable.
3. Lewis structures are preferable when adjacent formal charges are zero or of the opposite sign.
4. When we must choose among several Lewis structures with similar distributions of formal charges, the structure with the
negative formal charges on the more electronegative atoms is preferable.

To see how these guidelines apply, let us consider some possible structures for carbon dioxide, CO . We know from our previous
2

discussion that the less electronegative atom typically occupies the central position, but formal charges allow us to understand why
this occurs. We can draw three possibilities for the structure: carbon in the center and double bonds, carbon in the center with a
single and triple bond, and oxygen in the center with double bonds:

Three Lewis structures are shown. The left and right structures show a carbon atom double bonded to two oxygen atoms, each of
which has two lone pairs of electrons. The center structure shows a carbon atom that is triple bonded to an oxygen atom with one
lone pair of electrons and single bonded to an oxygen atom with three lone pairs of electrons. The third structure shows an oxygen
atom double bonded to another oxygen atom with to lone pairs of electrons. The first oxygen atom is also double bonded to a
carbon atom with two lone pairs of electrons.
Comparing the three formal charges, we can definitively identify the structure on the left as preferable because it has only formal
charges of zero (Guideline 1).
As another example, the thiocyanate ion, an ion formed from a carbon atom, a nitrogen atom, and a sulfur atom, could have three
different molecular structures: CNS , NCS , or CSN . The formal charges present in each of these molecular structures can help
– – –

us pick the most likely arrangement of atoms. Possible Lewis structures and the formal charges for each of the three possible
structures for the thiocyanate ion are shown here:

Two rows of structures and numbers are shown. The top row is labeled, “Structure” and depicts three Lewis structures and the
bottom row is labeled, “Formal charge.” The left structure shows a carbon atom double bonded to a nitrogen atom with two lone
electron pairs on one side and double bonded to a sulfur atom with two lone electron pairs on the other. The structure is surrounded
by brackets and has a superscripted negative sign. Below this structure are the numbers negative one, zero, and zero. The middle
structure shows a carbon atom with two lone pairs of electrons double bonded to a nitrogen atom that is double bonded to a sulfur
atom with two lone electron pairs. The structure is surrounded by brackets and has a superscripted negative sign. Below this
structure are the numbers negative two, positive one, and zero. The right structure shows a carbon atom with two lone electron
pairs double bonded to a sulfur atom that is double bonded to a nitrogen atom with two lone electron pairs. The structure is
surrounded by brackets and has a superscripted negative sign. Below this structure are the numbers negative two, positive two, and
one.
Note that the sum of the formal charges in each case is equal to the charge of the ion (–1). However, the first arrangement of atoms
is preferred because it has the lowest number of atoms with nonzero formal charges (Guideline 2). Also, it places the least
electronegative atom in the center, and the negative charge on the more electronegative element (Guideline 4).

 Example 7.4.3: Using Formal Charge to Determine Molecular Structure

Nitrous oxide, N2O, commonly known as laughing gas, is used as an anesthetic in minor surgeries, such as the routine
extraction of wisdom teeth. Which is the likely structure for nitrous oxide?

Access for free at OpenStax 7.4.3 https://chem.libretexts.org/@go/page/38186


Solution Determining formal charge yields the following:

The structure with a terminal oxygen atom best satisfies the criteria for the most stable distribution of formal charge:

The number of atoms with formal charges are minimized (Guideline 2), and there is no formal charge larger than one
(Guideline 2). This is again consistent with the preference for having the less electronegative atom in the central position.

 Exercise 7.4.3
Which is the most likely molecular structure for the nitrite (NO ) ion?

Answer

ONO

7.4.3: Resonance
You may have noticed that the nitrite anion in Example 7.4.3 can have two possible structures with the atoms in the same positions.
The electrons involved in the N–O double bond, however, are in different positions:

Two Lewis structures are shown. The left structure shows an oxygen atom with three lone pairs of electrons single bonded to a
nitrogen atom with one lone pair of electrons that is double bonded to an oxygen with two lone pairs of electrons. Brackets
surround this structure, and there is a superscripted negative sign. The right structure shows an oxygen atom with two lone pairs of
electrons double bonded to a nitrogen atom with one lone pair of electrons that is single bonded to an oxygen atom with three lone
pairs of electrons. Brackets surround this structure, and there is a superscripted negative sign.
If nitrite ions do indeed contain a single and a double bond, we would expect for the two bond lengths to be different. A double
bond between two atoms is shorter (and stronger) than a single bond between the same two atoms. Experiments show, however,
that both N–O bonds in NO have the same strength and length, and are identical in all other properties.

2

It is not possible to write a single Lewis structure for NO in which nitrogen has an octet and both bonds are equivalent. Instead,

we use the concept of resonance: if two or more Lewis structures with the same arrangement of atoms can be written for a molecule
or ion, the actual distribution of electrons is an average of that shown by the various Lewis structures. The actual distribution of
electrons in each of the nitrogen-oxygen bonds in NO is the average of a double bond and a single bond. We call the individual

Lewis structures resonance forms. The actual electronic structure of the molecule (the average of the resonance forms) is called a
resonance hybrid of the individual resonance forms. A double-headed arrow between Lewis structures indicates that they are
resonance forms. Thus, the electronic structure of the NO ion is shown as:

Access for free at OpenStax 7.4.4 https://chem.libretexts.org/@go/page/38186


Two Lewis structures are shown with a double headed arrow drawn between them. The left structure shows an oxygen atom with
two lone pairs of electrons double bonded to a nitrogen atom with one lone pair of electrons that is single bonded to an oxygen
atom with three lone pairs of electrons. Brackets surround this structure, and there is a superscripted negative sign. The right
structure shows an oxygen atom with three lone pairs of electrons single bonded to a nitrogen atom with one lone pair of electrons
that is double bonded to an oxygen atom with two lone pairs of electrons. Brackets surround this structure, and there is a
superscripted negative sign.
We should remember that a molecule described as a resonance hybrid never possesses an electronic structure described by either
resonance form. It does not fluctuate between resonance forms; rather, the actual electronic structure is always the average of that
shown by all resonance forms. George Wheland, one of the pioneers of resonance theory, used a historical analogy to describe the
relationship between resonance forms and resonance hybrids. A medieval traveler, having never before seen a rhinoceros, described
it as a hybrid of a dragon and a unicorn because it had many properties in common with both. Just as a rhinoceros is neither a
dragon sometimes nor a unicorn at other times, a resonance hybrid is neither of its resonance forms at any given time. Like a
rhinoceros, it is a real entity that experimental evidence has shown to exist. It has some characteristics in common with its
resonance forms, but the resonance forms themselves are convenient, imaginary images (like the unicorn and the dragon).
The carbonate anion, CO , provides a second example of resonance:
2−
3

Three Lewis structures are shown with double headed arrows in between. Each structure is surrounded by brackets, and each has a
superscripted two negative sign. The left structure depicts a carbon atom bonded to three oxygen atoms. It is single bonded to two
of these oxygen atoms, each of which has three lone pairs of electrons, and double bonded to the third, which has two lone pairs of
electrons. The double bond is located between the lower left oxygen atom and the carbon atom. The central and right structures are
the same as the first, but the position of the double bonded oxygen has moved to the lower right oxygen in the central structure and
to the top oxygen in the right structure.
One oxygen atom must have a double bond to carbon to complete the octet on the central atom. All oxygen atoms, however, are
equivalent, and the double bond could form from any one of the three atoms. This gives rise to three resonance forms of the
carbonate ion. Because we can write three identical resonance structures, we know that the actual arrangement of electrons in the
carbonate ion is the average of the three structures. Again, experiments show that all three C–O bonds are exactly the same.

Summary
In a Lewis structure, formal charges can be assigned to each atom by treating each bond as if one-half of the electrons are assigned
to each atom. These hypothetical formal charges are a guide to determining the most appropriate Lewis structure. A structure in
which the formal charges are as close to zero as possible is preferred. Resonance occurs in cases where two or more Lewis
structures with identical arrangements of atoms but different distributions of electrons can be written. The actual distribution of
electrons (the resonance hybrid) is an average of the distribution indicated by the individual Lewis structures (the resonance forms).

7.4.4: Key Equations


1
formal charge = # valence shell electrons (free atom) − # one pair electrons −   # bonding electrons
2

Glossary
formal charge
charge that would result on an atom by taking the number of valence electrons on the neutral atom and subtracting the
nonbonding electrons and the number of bonds (one-half of the bonding electrons)

Access for free at OpenStax 7.4.5 https://chem.libretexts.org/@go/page/38186


molecular structure
arrangement of atoms in a molecule or ion

resonance
situation in which one Lewis structure is insufficient to describe the bonding in a molecule and the average of multiple
structures is observed

resonance forms
two or more Lewis structures that have the same arrangement of atoms but different arrangements of electrons

resonance hybrid
average of the resonance forms shown by the individual Lewis structures

This page titled 7.4: Formal Charges and Resonance is shared under a CC BY 4.0 license and was authored, remixed, and/or curated by OpenStax
via source content that was edited to the style and standards of the LibreTexts platform; a detailed edit history is available upon request.

Access for free at OpenStax 7.4.6 https://chem.libretexts.org/@go/page/38186


Access for free at OpenStax 7.4.7 https://chem.libretexts.org/@go/page/38186
7.5: Strengths of Ionic and Covalent Bonds
 Learning Objectives
Describe the energetics of covalent and ionic bond formation and breakage
Use the Born-Haber cycle to compute lattice energies for ionic compounds
Use average covalent bond energies to estimate enthalpies of reaction

A bond’s strength describes how strongly each atom is joined to another atom, and therefore how much energy is required to break
the bond between the two atoms. In this section, you will learn about the bond strength of covalent bonds, and then compare that to
the strength of ionic bonds, which is related to the lattice energy of a compound.

7.5.1: Bond Strength: Covalent Bonds


Stable molecules exist because covalent bonds hold the atoms together. We measure the strength of a covalent bond by the energy
required to break it, that is, the energy necessary to separate the bonded atoms. Separating any pair of bonded atoms requires
energy; the stronger a bond, the greater the energy required to break it. The energy required to break a specific covalent bond in
one mole of gaseous molecules is called the bond energy or the bond dissociation energy. The bond energy for a diatomic molecule,
D X–Y , is defined as the standard enthalpy change for the endothermic reaction:
X Y(g) ⟶ X(g) + Y(g) DX−Y = ΔH ° (7.5.1)

For example, the bond energy of the pure covalent H–H bond, Δ H –H , is 436 kJ per mole of H–H bonds broken:

H2(g) ⟶ 2 H(g) DH −H = ΔH ° = 436kJ (7.5.2)

Breaking a bond always require energy to be added to the molecule. Correspondingly,


making a bond always releases energy.
Molecules with three or more atoms have two or more bonds. The sum of all bond energies in such a molecule is equal to the
standard enthalpy change for the endothermic reaction that breaks all the bonds in the molecule. For example, the sum of the four
C–H bond energies in CH4, 1660 kJ, is equal to the standard enthalpy change of the reaction:

A reaction is shown with Lewis structures. The first structure shows a carbon atom single bonded to four hydrogen atoms with the
symbol, “( g )” written next to it. A right-facing arrow points to the letter “C” and the symbol “( g ),” which is followed by a plus
sign. Next is the number 4, the letter “H” and the symbol, “( g ).” To the right of this equation is another equation: capital delta H
superscript degree symbol equals 1660 k J.
The average C–H bond energy, D , is 1660/4 = 415 kJ/mol because there are four moles of C–H bonds broken per mole of the
C –H

reaction. Although the four C–H bonds are equivalent in the original molecule, they do not each require the same energy to break;
once the first bond is broken (which requires 439 kJ/mol), the remaining bonds are easier to break. The 415 kJ/mol value is the
average, not the exact value required to break any one bond.
The strength of a bond between two atoms increases as the number of electron pairs in the bond increases. Generally, as the bond
strength increases, the bond length decreases. Thus, we find that triple bonds are stronger and shorter than double bonds between
the same two atoms; likewise, double bonds are stronger and shorter than single bonds between the same two atoms. Average bond
energies for some common bonds appear in Table 7.5.2, and a comparison of bond lengths and bond strengths for some common
bonds appears in Table 7.5.2. When one atom bonds to various atoms in a group, the bond strength typically decreases as we move
down the group. For example, C–F is 439 kJ/mol, C–Cl is 330 kJ/mol, and C–Br is 275 kJ/mol.
Table 7.5.1 : Bond Energies (kJ/mol)
Bond Bond Energy Bond Bond Energy Bond Bond Energy

H–H 436 C–S 260 F–Cl 255

Access for free at OpenStax 7.5.1 https://chem.libretexts.org/@go/page/38187


Bond Bond Energy Bond Bond Energy Bond Bond Energy

H–C 415 C–Cl 330 F–Br 235

H–N 390 C–Br 275 Si–Si 230

H–O 464 C–I 240 Si–P 215

H–F 569 N–N 160 Si–S 225

H–Si 395 N = N 418 Si–Cl 359

H–P 320 N ≡ N 946 Si–Br 290

H–S 340 N–O 200 Si–I 215

H–Cl 432 N–F 270 P–P 215

H–Br 370 N–P 210 P–S 230

H–I 295 N–Cl 200 P–Cl 330

C–C 345 N–Br 245 P–Br 270

C = C 611 O–O 140 P–I 215

C ≡ C 837 O = O 498 S–S 215

C–N 290 O–F 160 S–Cl 250

C = N 615 O–Si 370 S–Br 215

C ≡ N 891 O–P 350 Cl–Cl 243

C–O 350 O–Cl 205 Cl–Br 220

C = O 741 O–I 200 Cl–I 210

C ≡ O 1080 F–F 160 Br–Br 190

C–F 439 F–Si 540 Br–I 180

C–Si 360 F–P 489 I–I 150

C–P 265 F–S 285

Table 7.5.2 : Average Bond Lengths and Bond Energies for Some Common Bonds
Bond Bond Length (Å) Bond Energy (kJ/mol)

C–C 1.54 345

C = C 1.34 611

C ≡ C 1.20 837

C–N 1.43 290

C = N 1.38 615

C ≡ N 1.16 891

C–O 1.43 350

C = O 1.23 741

C ≡ O 1.13 1080

We can use bond energies to calculate approximate enthalpy changes for reactions where enthalpies of formation are not available.
Calculations of this type will also tell us whether a reaction is exothermic or endothermic.

Access for free at OpenStax 7.5.2 https://chem.libretexts.org/@go/page/38187


An exothermic reaction (ΔH negative, heat produced) results when the bonds in the products are stronger than the bonds
in the reactants.
An endothermic reaction (ΔH positive, heat absorbed) results when the bonds in the products are weaker than those in the
reactants.

The enthalpy change, ΔH, for a chemical reaction is approximately equal to the sum of the energy required to break all bonds in the
reactants (energy “in”, positive sign) plus the energy released when all bonds are formed in the products (energy “out,” negative
sign). This can be expressed mathematically in the following way:

ΔH = ∑ Dbonds broken − ∑ Dbonds formed (7.5.3)

In this expression, the symbol Σ means “the sum of” and D represents the bond energy in kilojoules per mole, which is always a
positive number. The bond energy is obtained from a table and will depend on whether the particular bond is a single, double, or
triple bond. Thus, in calculating enthalpies in this manner, it is important that we consider the bonding in all reactants and products.
Because D values are typically averages for one type of bond in many different molecules, this calculation provides a rough
estimate, not an exact value, for the enthalpy of reaction.
Consider the following reaction:
H + Cl ⟶ 2 HCl (7.5.4)
2 (g) 2 (g) (g)

or
H– H + Cl– Cl ⟶ 2 H– Cl (7.5.5)
(g) (g) (g)

To form two moles of HCl, one mole of H–H bonds and one mole of Cl–Cl bonds must be broken. The energy required to break
these bonds is the sum of the bond energy of the H–H bond (436 kJ/mol) and the Cl–Cl bond (243 kJ/mol). During the reaction,
two moles of H–Cl bonds are formed (bond energy = 432 kJ/mol), releasing 2 × 432 kJ; or 864 kJ. Because the bonds in the
products are stronger than those in the reactants, the reaction releases more energy than it consumes:

ΔH = ∑ Dbonds broken − ∑ Dbonds f ormed

= [ DH−H + DCl−Cl ] − 2 DH−Cl

= [436 + 243] − 2(432) = −185 kJ

This excess energy is released as heat, so the reaction is exothermic. Table T2 gives a value for the standard molar enthalpy of
formation of HCl(g), ΔH , of –92.307 kJ/mol. Twice that value is –184.6 kJ, which agrees well with the answer obtained earlier

f

for the formation of two moles of HCl.

 Example 7.5.1: Using Bond Energies to Approximate Enthalpy Changes

Methanol, CH3OH, may be an excellent alternative fuel. The high-temperature reaction of steam and carbon produces a
mixture of the gases carbon monoxide, CO, and hydrogen, H2, from which methanol can be produced. Using the bond energies
in Table 7.5.2, calculate the approximate enthalpy change, ΔH, for the reaction here:

C O(g) + 2H 2(g) ⟶ C H3 OH(g)

Solution
First, we need to write the Lewis structures of the reactants and the products:

Access for free at OpenStax 7.5.3 https://chem.libretexts.org/@go/page/38187


A set of Lewis diagrams show a chemical reaction. The first structure shows a carbon atom with a lone pair of electrons triple
bonded to an oxygen with a lone pair of electrons. To the right of this structure is a plus sign, then the number 2 followed by a
hydrogen atom single bonded to a hydrogen atom. To the right of this structure is a right-facing arrow followed by a hydrogen
atom single bonded to a carbon atom that is single bonded to two hydrogen atoms and an oxygen atom with two lone pairs of
electrons. The oxygen atom is also single bonded to a hydrogen atom.
From this, we see that ΔH for this reaction involves the energy required to break a C–O triple bond and two H–H single bonds,
as well as the energy produced by the formation of three C–H single bonds, a C–O single bond, and an O–H single bond. We
can express this as follows (via Equation 7.5.3):

ΔH = ∑ Dbonds broken − ∑ Dbonds f ormed

ΔH = [ DC≡O + 2(DH−H )] − [3(DC−H ) + DC−O + DO−H ]

Using the bond energy values in Table 7.5.2, we obtain:

ΔH = [1080 + 2(436)] − [3(415) + 350 + 464]

= −107 kJ

We can compare this value to the value calculated based on ΔH data from Appendix G:

f

∘ ∘ ∘
ΔH = [ΔH CH OH(g)] − [ΔH CO(g) + 2 × ΔH H ]
f 3 f f 2

= [−201.0] − [−110.52 + 2 × 0]

= −90.5 kJ

Note that there is a fairly significant gap between the values calculated using the two different methods. This occurs because D
values are the average of different bond strengths; therefore, they often give only rough agreement with other data.

 Exercise 7.5.1

Ethyl alcohol, CH3CH2OH, was one of the first organic chemicals deliberately synthesized by humans. It has many uses in
industry, and it is the alcohol contained in alcoholic beverages. It can be obtained by the fermentation of sugar or synthesized
by the hydration of ethylene in the following reaction:

A set of Lewis structures show a chemical reaction. The first structure shows two carbon atoms that are double bonded
together and are each single bonded to two hydrogen atoms. This structure is followed by a plus sign, then an oxygen atom
with two lone pairs of electrons single bonded to two hydrogen atoms. A right-facing arrow leads to a carbon atom single
bonded to three hydrogen atoms and a second carbon atom. The second carbon atom is single bonded to two hydrogen atoms
and an oxygen atom with two lone pairs of electrons. The oxygen atom is single bonded to a hydrogen atom as well.
Using the bond energies in Table 7.5.2, calculate an approximate enthalpy change, ΔH, for this reaction.

Answer
–35 kJ

7.5.2: Ionic Bond Strength and Lattice Energy


An ionic compound is stable because of the electrostatic attraction between its positive and negative ions. The lattice energy of a
compound is a measure of the strength of this attraction. The lattice energy (ΔH ) of an ionic compound is defined as the
lattice

energy required to separate one mole of the solid into its component gaseous ions. For the ionic solid MX, the lattice energy is the
enthalpy change of the process:

Access for free at OpenStax 7.5.4 https://chem.libretexts.org/@go/page/38187


+ n−
M X(s) ⟶ M n +X ΔHlattice (7.5.6)
(g) (g)

Note that we are using the convention where the ionic solid is separated into ions, so our lattice energies will be endothermic
(positive values). Some texts use the equivalent but opposite convention, defining lattice energy as the energy released when
separate ions combine to form a lattice and giving negative (exothermic) values. Thus, if you are looking up lattice energies in
another reference, be certain to check which definition is being used. In both cases, a larger magnitude for lattice energy indicates a
more stable ionic compound. For sodium chloride, ΔHlattice = 769 kJ. Thus, it requires 769 kJ to separate one mole of solid NaCl
into gaseous Na+ and Cl– ions. When one mole each of gaseous Na+ and Cl– ions form solid NaCl, 769 kJ of heat is released.
The lattice energy ΔH of an ionic crystal can be expressed by the following equation (derived from Coulomb’s law,
lattice

governing the forces between electric charges):


+ −
C (Z )(Z )
ΔHlattice = (7.5.7)
Ro

in which
C is a constant that depends on the type of crystal structure;
Z and Z are the charges on the ions; and
+ –

R is the interionic distance (the sum of the radii of the positive and negative ions).
o

Thus, the lattice energy of an ionic crystal increases rapidly as the charges of the ions increase and the sizes of the ions decrease.
When all other parameters are kept constant, doubling the charge of both the cation and anion quadruples the lattice energy. For
example, the lattice energy of LiF (Z+ and Z– = 1) is 1023 kJ/mol, whereas that of MgO (Z+ and Z– = 2) is 3900 kJ/mol (Ro is
nearly the same—about 200 pm for both compounds).
Different interatomic distances produce different lattice energies. For example, we can compare the lattice energy of MgF2 (2957
kJ/mol) to that of MgI2 (2327 kJ/mol) to observe the effect on lattice energy of the smaller ionic size of F– as compared to I–.

 Example 7.5.2: Lattice Energy Comparisons

The precious gem ruby is aluminum oxide, Al2O3, containing traces of Cr3+. The compound Al2Se3 is used in the fabrication of
some semiconductor devices. Which has the larger lattice energy, Al2O3 or Al2Se3?

Solution
In these two ionic compounds, the charges Z+ and Z– are the same, so the difference in lattice energy will mainly depend upon
Ro. The O2– ion is smaller than the Se2– ion. Thus, Al2O3 would have a shorter interionic distance than Al2Se3, and Al2O3
would have the larger lattice energy.

 Exercise 7.5.2

Zinc oxide, ZnO, is a very effective sunscreen. How would the lattice energy of ZnO compare to that of NaCl?

Answer
ZnO would have the larger lattice energy because the Z values of both the cation and the anion in ZnO are greater, and the
interionic distance of ZnO is smaller than that of NaCl.

7.5.3: The Born-Haber Cycle


It is not possible to measure lattice energies directly. However, the lattice energy can be calculated using the equation given in the
previous section or by using a thermochemical cycle. The Born-Haber cycle is an application of Hess’s law that breaks down the
formation of an ionic solid into a series of individual steps:
ΔH , the standard enthalpy of formation of the compound

f

IE, the ionization energy of the metal


EA, the electron affinity of the nonmetal

Access for free at OpenStax 7.5.5 https://chem.libretexts.org/@go/page/38187


ΔHs , the enthalpy of sublimation of the metal

D, the bond dissociation energy of the nonmetal


ΔHlattice, the lattice energy of the compound
Figure 7.5.1 diagrams the Born-Haber cycle for the formation of solid cesium fluoride.

Figure 7.5.1 : The Born-Haber cycle shows the relative energies of each step involved in the formation of an ionic solid from the
necessary elements in their reference states.
A diagram is shown. An upward facing arrow is drawn to the far left of the chart and is labeled “H increasing.” A horizontal line is
drawn at the bottom of the chart. A downward-facing, vertical arrow to the left side of this line is labeled, “Overall change.” Beside
this arrow is another label, “capital delta H subscript f, equals negative 553.5 k J per mol, ( Enthalpy of formation ).” Three
horizontal lines, one above the other, and all above the bottom line, are labeled, from bottom to top, as: “C s ( s ), plus sign, one
half F subscript 2, ( g ),” “C s ( g ), plus sign, one half F subscript 2, ( g ),” and “C s, superscript positive sign, ( g ), plus sign, one
half F subscript 2, ( g ).” Each of these lines is connected by an upward-facing vertical arrow. Each arrow is labeled, “capital delta
H subscript 1, equals 76.5 k J per mol, ( Enthalpy of sublimation ),” “capital delta H subscript 2, equals 375.7 k J per mol, (
ionization energy ),” and “capital delta H subscript 3 equals 79.4 k J / mol ( one half dissociation energy ).” Another horizontal line
is drawn in the center top portion of the diagram and is labeled “C s, superscript positive sign, ( g ), plus sign, F, ( g ).” There is one
more horizontal line drawn to the right of the overall diagram and located halfway down the image. An arrow connects the top line
to this line and is labeled, “capital delta H equals negative 328.2 k J / mol ( electron affinity ).” The line is labeled, “C s superscript
positive sign ( g ) plus F superscript negative sign ( g ).” The arrow connecting this line to the bottom line is labeled, “negative
capital delta H subscript lattice equals negative 756.9 k J / mol.” The arrow points to a label on the bottom line which reads, “C s F
( s ).”
We begin with the elements in their most common states, Cs(s) and F2(g). The ΔH represents the conversion of solid cesium into

s

a gas, and then the ionization energy converts the gaseous cesium atoms into cations. In the next step, we account for the energy
required to break the F–F bond to produce fluorine atoms. Converting one mole of fluorine atoms into fluoride ions is an
exothermic process, so this step gives off energy (the electron affinity) and is shown as decreasing along the y-axis. We now have
one mole of Cs cations and one mole of F anions. These ions combine to produce solid cesium fluoride. The enthalpy change in
this step is the negative of the lattice energy, so it is also an exothermic quantity. The total energy involved in this conversion is
equal to the experimentally determined enthalpy of formation, ΔH , of the compound from its elements. In this case, the overall
f

change is exothermic.
Hess’s law can also be used to show the relationship between the enthalpies of the individual steps and the enthalpy of formation.
Table 7.5.3 shows this for cesium fluoride, CsF.
Table 7.5.3 : Enthalpies of Select Transitions
Enthalpy of sublimation of Cs(s) Cs(s) ⟶ Cs(g)

ΔH = ΔHs = 77 kJ/mol

1 1
One-half of the bond energy of F2 F (g) ⟶ F(g)
2
ΔH = D = 79 kJ/mol
2 2

Ionization energy of Cs(g) Cs(g) ⟶ Cs


+
(g) + e

ΔH = I E = 376 kJ/mol

Negative of the electron affinity of F F(g) + e



⟶ F

(g) ΔH = −EA = −328 kJ/mol

Negative of the lattice energy of CsF(s) Cs


+
(g) + F

(g) ⟶ CsF(s) ΔH = −ΔHlattic e = ?

Access for free at OpenStax 7.5.6 https://chem.libretexts.org/@go/page/38187


1
∘ ∘
ΔH = ΔH = ΔHs + D + I E + (−EA) + (−ΔHlattic e )
f
Enthalpy of formation of CsF(s), add steps 1–5 1
2

Cs(s) + F (g) ⟶ CsF(s) = −554 kJ/mol


2
2

Thus, the lattice energy can be calculated from other values. For cesium chloride, using this data, the lattice energy is:

ΔHlattice = (411 + 109 + 122 + 496 + 368) kJ = 770 kJ

The Born-Haber cycle may also be used to calculate any one of the other quantities in the equation for lattice energy, provided that
the remainder is known. For example, if the relevant enthalpy of sublimation ΔH , ionization energy (IE), bond dissociation

s

enthalpy (D), lattice energy ΔHlattice, and standard enthalpy of formation ΔH are known, the Born-Haber cycle can be used to

f

determine the electron affinity of an atom.


Lattice energies calculated for ionic compounds are typically much larger than bond dissociation energies measured for covalent
bonds. Whereas lattice energies typically fall in the range of 600–4000 kJ/mol (some even higher), covalent bond dissociation
energies are typically between 150–400 kJ/mol for single bonds. Keep in mind, however, that these are not directly comparable
values. For ionic compounds, lattice energies are associated with many interactions, as cations and anions pack together in an
extended lattice. For covalent bonds, the bond dissociation energy is associated with the interaction of just two atoms.

Summary
The strength of a covalent bond is measured by its bond dissociation energy, that is, the amount of energy required to break that
particular bond in a mole of molecules. Multiple bonds are stronger than single bonds between the same atoms. The enthalpy of a
reaction can be estimated based on the energy input required to break bonds and the energy released when new bonds are formed.
For ionic bonds, the lattice energy is the energy required to separate one mole of a compound into its gas phase ions. Lattice energy
increases for ions with higher charges and shorter distances between ions. Lattice energies are often calculated using the Born-
Haber cycle, a thermochemical cycle including all of the energetic steps involved in converting elements into an ionic compound.

7.5.3.1: Key Equations


Bond energy for a diatomic molecule: XY(g) ⟶ X(g) + Y(g) D
X–Y
= ΔH °

Enthalpy change: ΔH = ƩDbonds broken – ƩDbonds formed


Lattice energy for a solid MX: MX(s) ⟶ M (g) + X (g) n+ n−
ΔHlattice
+ −
C(Z )(Z )
Lattice energy for an ionic crystal: ΔH lattice =
Ro

Footnotes
1. This question is taken from the Chemistry Advanced Placement Examination and is used with the permission of the Educational
Testing Service.

Glossary
bond energy
(also, bond dissociation energy) energy required to break a covalent bond in a gaseous substance

Born-Haber cycle
thermochemical cycle relating the various energetic steps involved in the formation of an ionic solid from the relevant elements
lattice energy (ΔHlattice)
energy required to separate one mole of an ionic solid into its component gaseous ions

This page titled 7.5: Strengths of Ionic and Covalent Bonds is shared under a CC BY 4.0 license and was authored, remixed, and/or curated by
OpenStax via source content that was edited to the style and standards of the LibreTexts platform; a detailed edit history is available upon request.

Access for free at OpenStax 7.5.7 https://chem.libretexts.org/@go/page/38187


7.6: Molecular Structure and Polarity
 Learning Objectives
Predict the structures of small molecules using valence shell electron pair repulsion (VSEPR) theory
Explain the concepts of polar covalent bonds and molecular polarity
Assess the polarity of a molecule based on its bonding and structure

Thus far, we have used two-dimensional Lewis structures to represent molecules. However, molecular structure is actually three-
dimensional, and it is important to be able to describe molecular bonds in terms of their distances, angles, and relative
arrangements in space (Figure 7.6.1). A bond angle is the angle between any two bonds that include a common atom, usually
measured in degrees. A bond distance (or bond length) is the distance between the nuclei of two bonded atoms along the straight
line joining the nuclei. Bond distances are measured in Ångstroms (1 Å = 10–10 m) or picometers (1 pm = 10–12 m, 100 pm = 1 Å).

Figure 7.6.1 : Bond distances (lengths) and angles are shown for the formaldehyde molecule, H2CO.
A pair of images are shown. The left image shows a carbon atom with three atoms bonded in a triangular arrangement around it.
There are two hydrogen atoms bonded on the left side of the carbon and the angle between them is labeled, “118 degrees” and,
“Bond angle.” The carbon is also double bonded to an oxygen atom. The double bond is shaded and there is a bracket which labels
the bond, “Bond length ( angstrom ), ( center to center ),” and, “1.21 angstrom.” The right image shows a ball-and-stick model of
the same elements. The hydrogen atoms are white, the carbon atom is black, and the oxygen atom is red."

7.6.1: VSEPR Theory


Valence shell electron-pair repulsion theory (VSEPR theory) enables us to predict the molecular structure, including approximate
bond angles around a central atom, of a molecule from an examination of the number of bonds and lone electron pairs in its Lewis
structure. The VSEPR model assumes that electron pairs in the valence shell of a central atom will adopt an arrangement that
minimizes repulsions between these electron pairs by maximizing the distance between them. The electrons in the valence shell of
a central atom form either bonding pairs of electrons, located primarily between bonded atoms, or lone pairs. The electrostatic
repulsion of these electrons is reduced when the various regions of high electron density assume positions as far from each other as
possible.
VSEPR theory predicts the arrangement of electron pairs around each central atom and, usually, the correct arrangement of atoms
in a molecule. We should understand, however, that the theory only considers electron-pair repulsions. Other interactions, such as
nuclear-nuclear repulsions and nuclear-electron attractions, are also involved in the final arrangement that atoms adopt in a
particular molecular structure.
As a simple example of VSEPR theory, let us predict the structure of a gaseous BeF2 molecule. The Lewis structure of BeF2
(Figure 7.6.2) shows only two electron pairs around the central beryllium atom. With two bonds and no lone pairs of electrons on
the central atom, the bonds are as far apart as possible, and the electrostatic repulsion between these regions of high electron
density is reduced to a minimum when they are on opposite sides of the central atom. The bond angle is 180° (Figure 7.6.2).

Access for free at OpenStax 7.6.1 https://chem.libretexts.org/@go/page/38188


Figure 7.6.2 : The BeF2 molecule adopts a linear structure in which the two bonds are as far apart as possible, on opposite sides of
the Be atom.
A Lewis structure is shown. A fluorine atom with three lone pairs of electrons is single bonded to a beryllium atom which is single
bonded to a fluorine atom with three lone pairs of electrons. The angle of the bonds between the two fluorine atoms and the
beryllium atom is labeled, “180 degrees.”
Figure 7.6.3 illustrates this and other electron-pair geometries that minimize the repulsions among regions of high electron density
(bonds and/or lone pairs). Two regions of electron density around a central atom in a molecule form a linear geometry; three
regions form a trigonal planar geometry; four regions form a tetrahedral geometry; five regions form a trigonal bipyramidal
geometry; and six regions form an octahedral geometry.

Access for free at OpenStax 7.6.2 https://chem.libretexts.org/@go/page/38188


Figure 7.6.3 : The basic electron-pair geometries predicted by VSEPR theory maximize the space around any region of electron
density (bonds or lone pairs).
A table with four rows and six columns is shown. The header column contains the phrases, “Number of regions,” “Spatial
arrangement,” “Wedge/dash Notation,” and “Electron pair Geometry.” The first row reads: “Two regions of high electron density (
bonds and/or unshared pairs )”, “Three regions of high electron density ( bonds and/or unshared pairs ),” “Four regions of high
electron density ( bonds and/or unshared pairs ),” “Five regions of high electron density ( bonds and/or unshared pairs ),” and “Six
regions of high electron density ( bonds and/or unshared pairs ).” The second row shows diagrams of orbitals. The first image
shows two oval-shaped orbs with an arrow indicating an angle of 180 degrees. The second image shows three oval-shaped orbs
with an arrow indicating an angle of 120 degrees. The third image shows four oval-shaped orbs with an arrow indicating an angle
of 109.5 degrees. The fourth image shows five oval-shaped orbs with an arrow indicating an angle of 90 and 120 degrees. The fifth
image shows six oval-shaped orbs with an arrow indicating an angle of 90 degrees. The third row contains Lewis structures. The
first structure shows a beryllium atom single bonded to two hydrogen atoms. The second structure shows a boron atom single
bonded to three hydrogen atoms. The third structure shows a carbon atom single bonded to four hydrogen atoms. The fourth
structure shows a phosphorus atom single bonded to five fluorine atoms. The fifth structure shows a sulfur atom single bonded to
six fluorine atoms. The fourth row contains the phrases “Linear; 180 degree angle,” Trigonal Planar; all angles 120 degrees,”
“Tetrahedral; all angles 109.5 degrees,” “Trigonal bipyramidal; angles of 90 degrees and 120 degrees. An attached atom may be
equatorial, ( in the plane of the triangle ), or axial, ( above the plane of the triangle ),” and “Octahedral; 90 degrees or 180 degrees.”

7.6.2: Electron-pair Geometry versus Molecular Structure


It is important to note that electron-pair geometry around a central atom is not the same thing as its molecular structure. The
electron-pair geometries shown in Figure 7.6.3 describe all regions where electrons are located, bonds as well as lone pairs.
Molecular structure describes the location of the atoms, not the electrons.
We differentiate between these two situations by naming the geometry that includes all electron pairs the electron-pair geometry.
The structure that includes only the placement of the atoms in the molecule is called the molecular structure. The electron-pair
geometries will be the same as the molecular structures when there are no lone electron pairs around the central atom, but they will
be different when there are lone pairs present on the central atom.

Access for free at OpenStax 7.6.3 https://chem.libretexts.org/@go/page/38188


Figure 7.6.4 : The molecular structure of the methane molecule, CH4, is shown with a tetrahedral arrangement of the hydrogen
atoms. VSEPR structures like this one are often drawn using the wedge and dash notation, in which solid lines represent bonds in
the plane of the page, solid wedges represent bonds coming up out of the plane, and dashed lines represent bonds going down into
the plane.
A Lewis structure shows a carbon atom single bonded to four hydrogen atoms. This structure uses wedges and dashes to give it a
three dimensional appearance.
For example, the methane molecule, CH4, which is the major component of natural gas, has four bonding pairs of electrons around
the central carbon atom; the electron-pair geometry is tetrahedral, as is the molecular structure (Figure 7.6.4). On the other hand,
the ammonia molecule, NH3, also has four electron pairs associated with the nitrogen atom, and thus has a tetrahedral electron-pair
geometry. One of these regions, however, is a lone pair, which is not included in the molecular structure, and this lone pair
influences the shape of the molecule (Figure 7.6.5).

Figure 7.6.5 : (a) The electron-pair geometry for the ammonia molecule is tetrahedral with one lone pair and three single bonds. (b)
The trigonal pyramidal molecular structure is determined from the electron-pair geometry. (c) The actual bond angles deviate
slightly from the idealized angles because the lone pair takes up a larger region of space than do the single bonds, causing the HNH
angle to be slightly smaller than 109.5°.
Three images are shown and labeled, “a,” “b,” and “c.” Image a shows a nitrogen atom single bonded to three hydrogen atoms.
There are four oval-shaped orbs that surround each hydrogen and one facing away from the rest of the molecule. These orbs are
located in a tetrahedral arrangement. Image b shows a ball-and-stick model of the nitrogen single bonded to the three hydrogen
atoms. Image c is the same as image a, but there are four curved, double headed arrows that circle the molecule and are labeled,
“106.8 degrees.”
Small distortions from the ideal angles in Figure 7.6.5 can result from differences in repulsion between various regions of electron
density. VSEPR theory predicts these distortions by establishing an order of repulsions and an order of the amount of space
occupied by different kinds of electron pairs. The order of electron-pair repulsions from greatest to least repulsion is:
lone pair-lone pair > lone pair-bonding pair > bonding pair-bonding pair
This order of repulsions determines the amount of space occupied by different regions of electrons. A lone pair of electrons
occupies a larger region of space than the electrons in a triple bond; in turn, electrons in a triple bond occupy more space than those
in a double bond, and so on. The order of sizes from largest to smallest is:
lone pair > triple bond > double bond > single bond
Consider formaldehyde, H2CO, which is used as a preservative for biological and anatomical specimens. This molecule has regions
of high electron density that consist of two single bonds and one double bond. The basic geometry is trigonal planar with 120°
bond angles, but we see that the double bond causes slightly larger angles (121°), and the angle between the single bonds is slightly
smaller (118°).
In the ammonia molecule, the three hydrogen atoms attached to the central nitrogen are not arranged in a flat, trigonal planar
molecular structure, but rather in a three-dimensional trigonal pyramid (Figure 7.6.6) with the nitrogen atom at the apex and the
three hydrogen atoms forming the base. The ideal bond angles in a trigonal pyramid are based on the tetrahedral electron pair
geometry. Again, there are slight deviations from the ideal because lone pairs occupy larger regions of space than do bonding
electrons. The H–N–H bond angles in NH3 are slightly smaller than the 109.5° angle in a regular tetrahedron (Figure 7.6.6)
because the lone pair-bonding pair repulsion is greater than the bonding pair-bonding pair repulsion. The ideal molecular structures
are predicted based on the electron-pair geometries for various combinations of lone pairs and bonding pairs.

Access for free at OpenStax 7.6.4 https://chem.libretexts.org/@go/page/38188


Figure 7.6.6 : The molecular structures are identical to the electron-pair geometries when there are no lone pairs present (first
column). For a particular number of electron pairs (row), the molecular structures for one or more lone pairs are determined based
on modifications of the corresponding electron-pair geometry.
A table is shown that is comprised of six rows and six columns. The header row reads: “Number of Electron Pairs,” “Electron pair
geometries; 0 lone pair,” “1 lone pair,” “2 lone pairs,” “3 lone pairs,” and “4 lone pairs.” The first column contains the numbers 2,
3, 4, 5, and 6. The first space in the second column contains a structure in which the letter E is single bonded to the letter X on each
side. The angle of the bonds is labeled with a curved, double headed arrow and the value, “180 degrees.” The structure is labeled,
“Linear.” The second space in the second column contains a structure in which the letter E is single bonded to the letter X on three
sides. The angle between the bonds is labeled with a curved, double headed arrow and the value, “120 degrees.” The structure is
labeled, “Trigonal planar.” The third space in the second column contains a structure in which the letter E is single bonded to the
letter X four times. The angle between the bonds is labeled with a curved, double headed arrow and the value, “109 degrees.” The
structure is labeled, “Tetrahedral.” The fourth space in the second column contains a structure in which the letter E is single bonded
to the letter X on five sides. The angle between the bonds is labeled with a curved, double headed arrow and the values “90 and 120
degrees.” The structure is labeled, “Trigonal bipyramid.” The fifth space in the second column contains a structure in which the
letter E is single bonded to the letter X on six sides. The angle between the bonds is labeled with a curved, double headed arrow
and the value, “90 degrees.” The structure is labeled, “Octahedral.” The first space in the third column is empty while the second
contains a structure in which the letter E is single bonded to the letter X on each side and has a lone pair of electrons. The angle
between the bonds is labeled with a curved, double headed arrow and the value, “less than 120 degrees.” The structure is labeled,
“Bent or angular.” The third space in the third column contains a structure in which the letter E is single bonded to the letter X three
times and to a lone pair of electrons. It is labeled with a curved, double headed arrow and the value, “less than 109 degrees.” The
structure is labeled, “Trigonal pyramid.” The fourth space in the third column contains a structure in which the letter E is single
bonded to the letter X on four sides and has a lone pair of electrons. The bond angle is labeled with a curved, double headed arrow
and the values, “less than 90 and less than 120 degrees.” The structure is labeled, “Sawhorse or seesaw.” The fifth space in the third
column contains a structure in which the letter E is single bonded to the letter X on five sides and has a lone pair of electrons. The
bond angle is labeled with a curved, double headed arrow and the value, “less than 90 degrees.” The structure is labeled, “Square
pyramidal.” The first and second spaces in the fourth column are empty while the third contains a structure in which the letter E is
single bonded to the letter X on each side and has two lone pairs of electrons. The bond angle is labeled with a curved, double
headed arrow and the value, “less than less than 109 degrees.” The structure is labeled, “Bent or angular.” The fourth space in the
fourth column contains a structure in which the letter E is single bonded to the letter X three times and to two lone pairs of
electrons. The bond angle is labeled with a curved, double headed arrow and the value, “less than 90 degrees.” The structure is
labeled, “T - shape.” The fifth space in the fourth column contains a structure in which the letter E is single bonded to the letter X
on four sides and has two lone pairs of electrons. The bond angle is labeled with a curved, double headed arrow and the value “90
degrees.” The structure is labeled, “Square planar.” The first, second and third spaces in the fifth column are empty while the fourth
contains a structure in which the letter E is single bonded to the letter X on each side and has three lone pairs of electrons. The

Access for free at OpenStax 7.6.5 https://chem.libretexts.org/@go/page/38188


bond angle is labeled with a curved, double headed arrow and the value, “180 degrees.” The structure is labeled, “Linear.” The fifth
space in the fifth column contains a structure in which the letter E is single bonded to the letter X three times and to three lone pairs
of electrons. The bond angle is labeled with a curved, double headed arrow and the value, “less than 90 degrees.” The structure is
labeled, “T - shape.” The first, second, third, and fourth spaces in the sixth column are empty while the fifth contains a structure in
which the letter E is single bonded to the letter X on each side and has four lone pairs of electrons. The bond angle is labeled with a
curved, double headed arrow and the value “180 degrees.” The structure is labeled, “Linear.” All the structures use wedges and
dashes to give them three dimensional appearances.

According to VSEPR theory, the terminal atom locations (Xs in Figure 7.6.7) are equivalent within the linear, trigonal planar, and
tetrahedral electron-pair geometries (the first three rows of the table). It does not matter which X is replaced with a lone pair
because the molecules can be rotated to convert positions. For trigonal bipyramidal electron-pair geometries, however, there are
two distinct X positions (Figure 7.6.7a): an axial position (if we hold a model of a trigonal bipyramid by the two axial positions,
we have an axis around which we can rotate the model) and an equatorial position (three positions form an equator around the
middle of the molecule). The axial position is surrounded by bond angles of 90°, whereas the equatorial position has more space
available because of the 120° bond angles. In a trigonal bipyramidal electron-pair geometry, lone pairs always occupy equatorial
positions because these more spacious positions can more easily accommodate the larger lone pairs.
Theoretically, we can come up with three possible arrangements for the three bonds and two lone pairs for the ClF3 molecule
(Figure 7.6.7). The stable structure is the one that puts the lone pairs in equatorial locations, giving a T-shaped molecular structure.

Figure 7.6.7 : (a) In a trigonal bipyramid, the two axial positions are located directly across from one another, whereas the three
equatorial positions are located in a triangular arrangement. (b–d) The two lone pairs (red lines) in ClF3 have several possible
arrangements, but the T-shaped molecular structure (b) is the one actually observed, consistent with the larger lone pairs both
occupying equatorial positions.
Four sets of images are shown and labeled, “a,” “b,” “c,” and “d.” Each image is separated by a dashed vertical line. Image a shows
a six-faced, bi-pyramidal structure where the central vertical axis is labeled, “Axial,” and the horizontal plane is labeled,
“Equatorial.” Image b shows a pair of diagrams in the same shape as image a, but in these diagrams, the left has a chlorine atom in
the center while the right has a chlorine atom in the center, two fluorine atoms on the upper and lower ends, and one fluorine in the
left horizontal position. Image c shows a pair of diagrams in the same shape as image a, but in these diagrams, the left has a
chlorine atom in the center while the right has a chlorine atom in the center and three fluorine atoms in each horizontal position.
Image d shows a pair of diagrams in the same shape as image a, but in these diagrams, the left has a chlorine atom in the center
while the right has a chlorine atom in the center, two fluorine atoms in the horizontal positions, and one in the axial bottom
position.
When a central atom has two lone electron pairs and four bonding regions, we have an octahedral electron-pair geometry. The two
lone pairs are on opposite sides of the octahedron (180° apart), giving a square planar molecular structure that minimizes lone pair-
lone pair repulsions.

 Predicting Electron Pair Geometry and Molecular Structure


The following procedure uses VSEPR theory to determine the electron pair geometries and the molecular structures:
1. Write the Lewis structure of the molecule or polyatomic ion.
2. Count the number of regions of electron density (lone pairs and bonds) around the central atom. A single, double, or triple
bond counts as one region of electron density.
3. Identify the electron-pair geometry based on the number of regions of electron density: linear, trigonal planar, tetrahedral,
trigonal bipyramidal, or octahedral (Figure 7.6.7, first column).
4. Use the number of lone pairs to determine the molecular structure (Figure 7.6.7 ). If more than one arrangement of lone
pairs and chemical bonds is possible, choose the one that will minimize repulsions, remembering that lone pairs occupy
more space than multiple bonds, which occupy more space than single bonds. In trigonal bipyramidal arrangements,
repulsion is minimized when every lone pair is in an equatorial position. In an octahedral arrangement with two lone pairs,
repulsion is minimized when the lone pairs are on opposite sides of the central atom.

Access for free at OpenStax 7.6.6 https://chem.libretexts.org/@go/page/38188


The following examples illustrate the use of VSEPR theory to predict the molecular structure of molecules or ions that have no
lone pairs of electrons. In this case, the molecular structure is identical to the electron pair geometry.

 Example 7.6.1: Predicting Electron-pair Geometry and Molecular Structure

Predict the electron-pair geometry and molecular structure for each of the following:
a. carbon dioxide, CO2, a molecule produced by the combustion of fossil fuels
b. boron trichloride, BCl3, an important industrial chemical

Solution
(a) We write the Lewis structure of CO2 as:

This shows us two regions of high electron density around the carbon atom—each double bond counts as one region, and there
are no lone pairs on the carbon atom. Using VSEPR theory, we predict that the two regions of electron density arrange
themselves on opposite sides of the central atom with a bond angle of 180°. The electron-pair geometry and molecular
structure are identical, and CO2 molecules are linear.
(b) We write the Lewis structure of BCl3 as:

Thus we see that BCl3 contains three bonds, and there are no lone pairs of electrons on boron. The arrangement of three
regions of high electron density gives a trigonal planar electron-pair geometry. The B–Cl bonds lie in a plane with 120° angles
between them. BCl3 also has a trigonal planar molecular structure.

The electron-pair geometry and molecular structure of BCl3 are both trigonal planar. Note that the VSEPR geometry indicates
the correct bond angles (120°), unlike the Lewis structure shown above.

 Exercise 7.6.1

Carbonate, CO , is a common polyatomic ion found in various materials from eggshells to antacids. What are the electron-
2−

pair geometry and molecular structure of this polyatomic ion?

Answer
The electron-pair geometry is trigonal planar and the molecular structure is trigonal planar. Due to resonance, all three C–O
bonds are identical. Whether they are single, double, or an average of the two, each bond counts as one region of electron
density.

 Example 7.6.2: Predicting Electron-pair Geometry and Molecular Structure

Two of the top 50 chemicals produced in the United States, ammonium nitrate and ammonium sulfate, both used as fertilizers,
contain the ammonium ion. Predict the electron-pair geometry and molecular structure of the NH cation.
+

Access for free at OpenStax 7.6.7 https://chem.libretexts.org/@go/page/38188


Solution
We write the Lewis structure of NH as: +
4

Figure 7.6.7 ).

Figure 7.6.8 : The ammonium ion displays a tetrahedral electron-pair geometry as well as a tetrahedral molecular structure.

 Exercise 7.6.2

Identify a molecule with trigonal bipyramidal molecular structure.

Answer
Any molecule with five electron pairs around the central atoms including no lone pairs will be trigonal bipyramidal. PF is 5

a common example

The next several examples illustrate the effect of lone pairs of electrons on molecular structure.

 Example 7.6.3: Lone Pairs on the Central Atom

Predict the electron-pair geometry and molecular structure of a water molecule.

Solution
The Lewis structure of H2O indicates that there are four regions of high electron density around the oxygen atom: two lone
pairs and two chemical bonds:

Figure 7.6.9 . Thus, the electron-pair geometry is tetrahedral and the molecular structure is bent with an angle slightly less than
109.5°. In fact, the bond angle is 104.5°.

Access for free at OpenStax 7.6.8 https://chem.libretexts.org/@go/page/38188


Figure 7.6.9 : (a) H2O has four regions of electron density around the central atom, so it has a tetrahedral electron-pair
geometry. (b) Two of the electron regions are lone pairs, so the molecular structure is bent.

 Exercise 7.6.3

The hydronium ion, H3O+, forms when acids are dissolved in water. Predict the electron-pair geometry and molecular structure
of this cation.

Answer
electron pair geometry: tetrahedral; molecular structure: trigonal pyramidal

 Example 7.6.4: SF4 Sulfur tetrafluoride,

Predicting Electron-pair Geometry and Molecular Structure: SF4, is extremely valuable for the preparation of fluorine-
containing compounds used as herbicides (i.e., SF4 is used as a fluorinating agent). Predict the electron-pair geometry and
molecular structure of a SF4 molecule.

Solution
The Lewis structure of SF4 indicates five regions of electron density around the sulfur atom: one lone pair and four bonding
pairs:

Figure 7.6.10 ).

Figure 7.6.10 : (a) SF4 has a trigonal bipyramidal arrangement of the five regions of electron density. (b) One of the regions is a
lone pair, which results in a seesaw-shaped molecular structure.

Access for free at OpenStax 7.6.9 https://chem.libretexts.org/@go/page/38188


 Exercise 7.6.4
Predict the electron pair geometry and molecular structure for molecules of XeF2.

Answer
The electron-pair geometry is trigonal bipyramidal. The molecular structure is linear.

 Example 7.6.4: XeF4

Of all the noble gases, xenon is the most reactive, frequently reacting with elements such as oxygen and fluorine. Predict the
electron-pair geometry and molecular structure of the XeF4 molecule.

Solution
The Lewis structure of XeF4 indicates six regions of high electron density around the xenon atom: two lone pairs and four
bonds:

Figure 7.6.11 : The five atoms are all in the same plane and have a square planar molecular structure.

Figure 7.6.11 : (a) XeF4 adopts an octahedral arrangement with two lone pairs (red lines) and four bonds in the electron-pair
geometry. (b) The molecular structure is square planar with the lone pairs directly across from one another.

 Exercise 7.6.4

In a certain molecule, the central atom has three lone pairs and two bonds. What will the electron pair geometry and molecular
structure be?

Answer
electron pair geometry: trigonal bipyramidal; molecular structure: linear

7.6.3: Molecular Structure for Multicenter Molecules


When a molecule or polyatomic ion has only one central atom, the molecular structure completely describes the shape of the
molecule. Larger molecules do not have a single central atom, but are connected by a chain of interior atoms that each possess a
“local” geometry. The way these local structures are oriented with respect to each other also influences the molecular shape, but
such considerations are largely beyond the scope of this introductory discussion. For our purposes, we will only focus on
determining the local structures.

Access for free at OpenStax 7.6.10 https://chem.libretexts.org/@go/page/38188


 Example 7.6.5: Predicting Structure in Multicenter Molecules

The Lewis structure for the simplest amino acid, glycine, H2NCH2CO2H, is shown here. Predict the local geometry for the
nitrogen atom, the two carbon atoms, and the oxygen atom with a hydrogen atom attached:

Solution

Consider each central atom independently. The electron-pair geometries:


nitrogen––four regions of electron density; tetrahedral
carbon (CH2)––four regions of electron density; tetrahedral
carbon (CO2)—three regions of electron density; trigonal planar
oxygen (OH)—four regions of electron density; tetrahedral
The local structures:
nitrogen––three bonds, one lone pair; trigonal pyramidal
carbon (CH2)—four bonds, no lone pairs; tetrahedral
carbon (CO2)—three bonds (double bond counts as one bond), no lone pairs; trigonal planar
oxygen (OH)—two bonds, two lone pairs; bent (109°)

 Exercise 7.6.5
Another amino acid is alanine, which has the Lewis structure shown here. Predict the electron-pair geometry and local
structure of the nitrogen atom, the three carbon atoms, and the oxygen atom with hydrogen attached:

Answer
electron-pair geometries: nitrogen––tetrahedral; carbon (CH)—tetrahedral; carbon (CH3)—tetrahedral; carbon (CO2)—
trigonal planar; oxygen (OH)—tetrahedral; local structures: nitrogen—trigonal pyramidal; carbon (CH)—tetrahedral;
carbon (CH3)—tetrahedral; carbon (CO2)—trigonal planar; oxygen (OH)—bent (109°)

 Example 7.6.6: Molecular Simulation


Using this molecular shape simulator allows us to control whether bond angles and/or lone pairs are displayed by checking or
unchecking the boxes under “Options” on the right. We can also use the “Name” checkboxes at bottom-left to display or hide
the electron pair geometry (called “electron geometry” in the simulator) and/or molecular structure (called “molecular shape”
in the simulator).
Build the molecule HCN in the simulator based on the following Lewis structure:

Access for free at OpenStax 7.6.11 https://chem.libretexts.org/@go/page/38188


H– C ≡ N

Click on each bond type or lone pair at right to add that group to the central atom. Once you have the complete molecule, rotate it
to examine the predicted molecular structure. What molecular structure is this?
Solution
The molecular structure is linear.

 Exercise 7.6.6

Build a more complex molecule in the simulator. Identify the electron-group geometry, molecular structure, and bond angles.
Then try to find a chemical formula that would match the structure you have drawn.

Answer
Answers will vary. For example, an atom with four single bonds, a double bond, and a lone pair has an octahedral electron-
group geometry and a square pyramidal molecular structure. XeOF4 is a molecule that adopts this structure.

7.6.4: Molecular Polarity and Dipole Moment


As discussed previously, polar covalent bonds connect two atoms with differing electronegativities, leaving one atom with a partial
positive charge (δ+) and the other atom with a partial negative charge (δ–), as the electrons are pulled toward the more
electronegative atom. This separation of charge gives rise to a bond dipole moment. The magnitude of a bond dipole moment is
represented by the Greek letter mu (µ) and is given by

μ = Qr (7.6.1)

where
Q is the magnitude of the partial charges (determined by the electronegativity difference) and
r is the distance between the charges:
This bond moment can be represented as a vector, a quantity having both direction and magnitude (Figure 7.6.12). Dipole vectors
are shown as arrows pointing along the bond from the less electronegative atom toward the more electronegative atom. A small
plus sign is drawn on the less electronegative end to indicate the partially positive end of the bond. The length of the arrow is
proportional to the magnitude of the electronegativity difference between the two atoms.

Figure 7.6.12 : (a) There is a small difference in electronegativity between C and H, represented as a short vector. (b) The
electronegativity difference between B and F is much larger, so the vector representing the bond moment is much longer.
A whole molecule may also have a separation of charge, depending on its molecular structure and the polarity of each of its bonds.
If such a charge separation exists, the molecule is said to be a polar molecule (or dipole); otherwise the molecule is said to be
nonpolar. The dipole moment measures the extent of net charge separation in the molecule as a whole. We determine the dipole
moment by adding the bond moments in three-dimensional space, taking into account the molecular structure.
For diatomic molecules, there is only one bond, so its bond dipole moment determines the molecular polarity. Homonuclear
diatomic molecules such as Br2 and N2 have no difference in electronegativity, so their dipole moment is zero. For heteronuclear
molecules such as CO, there is a small dipole moment. For HF, there is a larger dipole moment because there is a larger difference
in electronegativity.
When a molecule contains more than one bond, the geometry must be taken into account. If the bonds in a molecule are arranged
such that their bond moments cancel (vector sum equals zero), then the molecule is nonpolar. This is the situation in CO2 (Figure
7.6.13A). Each of the bonds is polar, but the molecule as a whole is nonpolar. From the Lewis structure, and using VSEPR theory,

we determine that the CO2 molecule is linear with polar C=O bonds on opposite sides of the carbon atom. The bond moments
cancel because they are pointed in opposite directions. In the case of the water molecule (Figure 7.6.13B), the Lewis structure

Access for free at OpenStax 7.6.12 https://chem.libretexts.org/@go/page/38188


again shows that there are two bonds to a central atom, and the electronegativity difference again shows that each of these bonds
has a nonzero bond moment. In this case, however, the molecular structure is bent because of the lone pairs on O, and the two bond
moments do not cancel. Therefore, water does have a net dipole moment and is a polar molecule (dipole).

Figure 7.6.13 : The overall dipole moment of a molecule depends on the individual bond dipole moments and how they are
arranged. (a) Each CO bond has a bond dipole moment, but they point in opposite directions so that the net CO2 molecule is
nonpolar. (b) In contrast, water is polar because the OH bond moments do not cancel out.
The OCS molecule has a structure similar to CO2, but a sulfur atom has replaced one of the oxygen atoms. To determine if this
molecule is polar, we draw the molecular structure. VSEPR theory predicts a linear molecule:

The C–O bond is considerably polar. Although C and S have very similar electronegativity values, S is slightly more
electronegative than C, and so the C-S bond is just slightly polar. Because oxygen is more electronegative than sulfur, the oxygen
end of the molecule is the negative end.
Chloromethane, CH3Cl, is another example of a polar molecule. Although the polar C–Cl and C–H bonds are arranged in a
tetrahedral geometry, the C–Cl bonds have a larger bond moment than the C–H bond, and the bond moments do not completely
cancel each other. All of the dipoles have a upward component in the orientation shown, since carbon is more electronegative than
hydrogen and less electronegative than chlorine:

When we examine the highly symmetrical molecules BF3 (trigonal planar), CH4 (tetrahedral), PF5 (trigonal bipyramidal), and SF6
(octahedral), in which all the polar bonds are identical, the molecules are nonpolar. The bonds in these molecules are arranged such
that their dipoles cancel. However, just because a molecule contains identical bonds does not mean that the dipoles will always
cancel. Many molecules that have identical bonds and lone pairs on the central atoms have bond dipoles that do not cancel.
Examples include H2S and NH3. A hydrogen atom is at the positive end and a nitrogen or sulfur atom is at the negative end of the
polar bonds in these molecules:

To summarize, to be polar, a molecule must:


1. Contain at least one polar covalent bond.
2. Have a molecular structure such that the sum of the vectors of each bond dipole moment does not cancel.

7.6.5: Properties of Polar Molecules


Polar molecules tend to align when placed in an electric field with the positive end of the molecule oriented toward the negative
plate and the negative end toward the positive plate (Figure 7.6.14). We can use an electrically charged object to attract polar

Access for free at OpenStax 7.6.13 https://chem.libretexts.org/@go/page/38188


molecules, but nonpolar molecules are not attracted. Also, polar solvents are better at dissolving polar substances, and nonpolar
solvents are better at dissolving nonpolar substances.

Figure 7.6.14 : (a) Molecules are always randomly distributed in the liquid state in the absence of an electric field. (b) When an
electric field is applied, polar molecules like HF will align to the dipoles with the field direction.

 Example 7.6.7: Polarity Simulations

Open the molecule polarity simulation and select the “Three Atoms” tab at the top. This should display a molecule ABC with
three electronegativity adjustors. You can display or hide the bond moments, molecular dipoles, and partial charges at the right.
Turning on the Electric Field will show whether the molecule moves when exposed to a field, similar to Figure 7.6.14.
Use the electronegativity controls to determine how the molecular dipole will look for the starting bent molecule if:
a. A and C are very electronegative and B is in the middle of the range.
b. A is very electronegative, and B and C are not.

Solution
a. Molecular dipole moment points immediately between A and C.
b. Molecular dipole moment points along the A–B bond, toward A.

 Exercise 7.6.7

Determine the partial charges that will give the largest possible bond dipoles.

Answer
The largest bond moments will occur with the largest partial charges. The two solutions above represent how unevenly the
electrons are shared in the bond. The bond moments will be maximized when the electronegativity difference is greatest.
The controls for A and C should be set to one extreme, and B should be set to the opposite extreme. Although the
magnitude of the bond moment will not change based on whether B is the most electronegative or the least, the direction of
the bond moment will.

Access for free at OpenStax 7.6.14 https://chem.libretexts.org/@go/page/38188


Summary
VSEPR theory predicts the three-dimensional arrangement of atoms in a molecule. It states that valence electrons will assume an
electron-pair geometry that minimizes repulsions between areas of high electron density (bonds and/or lone pairs). Molecular
structure, which refers only to the placement of atoms in a molecule and not the electrons, is equivalent to electron-pair geometry
only when there are no lone electron pairs around the central atom. A dipole moment measures a separation of charge. For one
bond, the bond dipole moment is determined by the difference in electronegativity between the two atoms. For a molecule, the
overall dipole moment is determined by both the individual bond moments and how these dipoles are arranged in the molecular
structure. Polar molecules (those with an appreciable dipole moment) interact with electric fields, whereas nonpolar molecules do
not.

Glossary
axial position
location in a trigonal bipyramidal geometry in which there is another atom at a 180° angle and the equatorial positions are at a
90° angle

bond angle
angle between any two covalent bonds that share a common atom

bond distance
(also, bond length) distance between the nuclei of two bonded atoms

bond dipole moment


separation of charge in a bond that depends on the difference in electronegativity and the bond distance represented by partial
charges or a vector

dipole moment
property of a molecule that describes the separation of charge determined by the sum of the individual bond moments based on
the molecular structure

electron-pair geometry
arrangement around a central atom of all regions of electron density (bonds, lone pairs, or unpaired electrons)

equatorial position
one of the three positions in a trigonal bipyramidal geometry with 120° angles between them; the axial positions are located at a
90° angle

linear
shape in which two outside groups are placed on opposite sides of a central atom

molecular structure
structure that includes only the placement of the atoms in the molecule

octahedral
shape in which six outside groups are placed around a central atom such that a three-dimensional shape is generated with four
groups forming a square and the other two forming the apex of two pyramids, one above and one below the square plane

polar molecule
(also, dipole) molecule with an overall dipole moment

tetrahedral
shape in which four outside groups are placed around a central atom such that a three-dimensional shape is generated with four
corners and 109.5° angles between each pair and the central atom

trigonal bipyramidal

Access for free at OpenStax 7.6.15 https://chem.libretexts.org/@go/page/38188


shape in which five outside groups are placed around a central atom such that three form a flat triangle with 120° angles
between each pair and the central atom, and the other two form the apex of two pyramids, one above and one below the
triangular plane

trigonal planar
shape in which three outside groups are placed in a flat triangle around a central atom with 120° angles between each pair and
the central atom

valence shell electron-pair repulsion theory (VSEPR)


theory used to predict the bond angles in a molecule based on positioning regions of high electron density as far apart as
possible to minimize electrostatic repulsion

vector
quantity having magnitude and direction

This page titled 7.6: Molecular Structure and Polarity is shared under a CC BY 4.0 license and was authored, remixed, and/or curated by
OpenStax via source content that was edited to the style and standards of the LibreTexts platform; a detailed edit history is available upon request.

Access for free at OpenStax 7.6.16 https://chem.libretexts.org/@go/page/38188


7.E: Chemical Bonding and Molecular Geometry (Exercises)
7.E.1: 7.1: Ionic Bonding
7.E.1.1: Q7.1.1
Does a cation gain protons to form a positive charge or does it lose electrons?

7.E.1.2: S7.1.1
The protons in the nucleus do not change during normal chemical reactions. Only the outer electrons move. Positive charges form
when electrons are lost.

7.E.1.3: Q7.1.2
Iron(III) sulfate [Fe2(SO4)3] is composed of Fe3+ and SO 2−
4
ions. Explain why a sample of iron(III) sulfate is uncharged.

7.E.1.4: Q7.1.3
Which of the following atoms would be expected to form negative ions in binary ionic compounds and which would be expected to
form positive ions: P, I, Mg, Cl, In, Cs, O, Pb, Co?

7.E.1.5: S7.1.3
P, I, Cl, and O would form anions because they are nonmetals. Mg, In, Cs, Pb, and Co would form cations because they are metals.

7.E.1.6: Q7.1.4
Which of the following atoms would be expected to form negative ions in binary ionic compounds and which would be expected to
form positive ions: Br, Ca, Na, N, F, Al, Sn, S, Cd?

7.E.1.7: Q7.1.5
Predict the charge on the monatomic ions formed from the following atoms in binary ionic compounds:
a. P
b. Mg
c. Al
d. O
e. Cl
f. Cs

7.E.1.8: S7.1.5
P3–; Mg2+; Al3+; O2–; Cl–; Cs+

7.E.1.9: Q7.1.6
Predict the charge on the monatomic ions formed from the following atoms in binary ionic compounds:
1. I
2. Sr
3. K
4. N
5. S
6. In

7.E.1.10: S7.1.6
1. I-
2. Sr2+
3. K+
4. N3-
5. S2-

Access for free at OpenStax 7.E.1 https://chem.libretexts.org/@go/page/77587


6. In3+

7.E.1.11: Q7.1.7
Write the electron configuration for each of the following ions:
1. As3–
2. I–
3. Be2+
4. Cd2+
5. O2–
6. Ga3+
7. Li+
8. (h) N3–
9. (i) Sn2+
10. (j) Co2+
11. (k) Fe2+
12. (l) As3+

7.E.1.12: S7.1.7
[Ar]4s23d104p6; [Kr]4d105s25p6 1s2 [Kr]4d10; [He]2s22p6; [Ar]3d10; 1s2 (h) [He]2s22p6 (i) [Kr]4d105s2 (j) [Ar]3d7 (k) [Ar]3d6, (l)
[Ar]3d104s2

7.E.1.13: Q7.1.8
Write the electron configuration for the monatomic ions formed from the following elements (which form the greatest
concentration of monatomic ions in seawater):
1. Cl
2. Na
3. Mg
4. Ca
5. K
6. Br
7. Sr
8. (h) F

7.E.1.14: Q7.1.9
Write out the full electron configuration for each of the following atoms and for the monatomic ion found in binary ionic
compounds containing the element:
a. Al
b. Br
c. Sr
d. Li
e. As
f. S

7.E.1.15: S7.1.9
1s22s22p63s23p1; Al3+: 1s22s22p6; 1s22s22p63s23p63d104s24p5; 1s22s22p63s23p63d104s24p6; 1s22s22p63s23p63d104s24p65s2;
Sr2+: 1s22s22p63s23p63d104s24p6; 1s22s1;
Li+: 1s2; 1s22s22p63s23p63d104s24p3; 1s22s22p63s23p63d104s24p6; 1s22s22p63s23p4; 1s22s22p63s23p6

7.E.1.16: Q7.1.10
From the labels of several commercial products, prepare a list of six ionic compounds in the products. For each compound, write
the formula. (You may need to look up some formulas in a suitable reference.)

Access for free at OpenStax 7.E.2 https://chem.libretexts.org/@go/page/77587


7.E.2: 7.3: Covalent Bonding
Why is it incorrect to speak of a molecule of solid NaCl?
NaCl consists of discrete ions arranged in a crystal lattice, not covalently bonded molecules.
What information can you use to predict whether a bond between two atoms is covalent or ionic?
Predict which of the following compounds are ionic and which are covalent, based on the location of their constituent atoms in the
periodic table:
1. Cl2CO
2. MnO
3. NCl3
4. CoBr2
5. K2S
6. CO
7. CaF2
8. (h) HI
9. (i) CaO
10. (j) IBr
11. (k) CO2
ionic: (b), (d), (e), (g), and (i); covalent: (a), (c), (f), (h), (j), and (k)
Explain the difference between a nonpolar covalent bond, a polar covalent bond, and an ionic bond.
From its position in the periodic table, determine which atom in each pair is more electronegative:
1. Br or Cl
2. N or O
3. S or O
4. P or S
5. Si or N
6. Ba or P
7. N or K
Cl; O; O; S; N; P; N
From its position in the periodic table, determine which atom in each pair is more electronegative:
1. N or P
2. N or Ge
3. S or F
4. Cl or S
5. H or C
6. Se or P
7. C or Si
From their positions in the periodic table, arrange the atoms in each of the following series in order of increasing electronegativity:
1. C, F, H, N, O
2. Br, Cl, F, H, I
3. F, H, O, P, S
4. Al, H, Na, O, P
5. Ba, H, N, O, As
H, C, N, O, F; H, I, Br, Cl, F; H, P, S, O, F; Na, Al, H, P, O; Ba, H, As, N, O
From their positions in the periodic table, arrange the atoms in each of the following series in order of increasing electronegativity:
1. As, H, N, P, Sb
2. Cl, H, P, S, Si

Access for free at OpenStax 7.E.3 https://chem.libretexts.org/@go/page/77587


3. Br, Cl, Ge, H, Sr
4. Ca, H, K, N, Si
5. Cl, Cs, Ge, H, Sr
Which atoms can bond to sulfur so as to produce a positive partial charge on the sulfur atom?
N, O, F, and Cl
Which is the most polar bond?
1. C–C
2. C–H
3. N–H
4. O–H
5. Se–H
Identify the more polar bond in each of the following pairs of bonds:
1. HF or HCl
2. NO or CO
3. SH or OH
4. PCl or SCl
5. CH or NH
6. SO or PO
7. CN or NN
HF; CO; OH; PCl; NH; PO; CN
Which of the following molecules or ions contain polar bonds?
1. O3
2. S8
3. O2−
2

4. NO −
3

5. CO2
6. H2S
7. BH −
4

7.E.3: 7.4: Lewis Symbols and Structures


7.E.3.1: Q7.4.1
Write the Lewis symbols for each of the following ions:
a. As3–
b. I–
c. Be2+
d. O2–
e. Ga3+
f. Li+
g. N3–

7.E.3.2: S7.4.1
eight electrons:

eight electrons:

Access for free at OpenStax 7.E.4 https://chem.libretexts.org/@go/page/77587


no electrons
Be2+;
eight electrons:

no electrons
Ga3+;
no electrons
Li+;
eight electrons:

7.E.3.3: Q7.4.2
Many monatomic ions are found in seawater, including the ions formed from the following list of elements. Write the Lewis
symbols for the monatomic ions formed from the following elements:
a. Cl
b. Na
c. Mg
d. Ca
e. K
f. Br
g. Sr
h. F

7.E.3.4: Q7.4.3
Write the Lewis symbols of the ions in each of the following ionic compounds and the Lewis symbols of the atom from which they
are formed:
1. MgS
2. Al2O3
3. GaCl3
4. K2O
5. Li3N
6. KF
(a)

;
(b)

;
(c)

Access for free at OpenStax 7.E.5 https://chem.libretexts.org/@go/page/77587


;
(d)

;
(e)

;
(f)

In the Lewis structures listed here, M and X represent various elements in the third period of the periodic table. Write the formula
of each compound using the chemical symbols of each element:
(a)

(b)

(c)

(d)

Write the Lewis structure for the diatomic molecule P2, an unstable form of phosphorus found in high-temperature phosphorus
vapor.

Write Lewis structures for the following:


a. H2
b. HBr
c. PCl3
d. SF2
e. H2CCH2
f. HNNH
g. H2CNH
h. (h) NO–
i. (i) N2

Access for free at OpenStax 7.E.6 https://chem.libretexts.org/@go/page/77587


j. (j) CO
k. (k) CN–
Write Lewis structures for the following:
a. O2
b. H2CO
c. AsF3
d. ClNO
e. SiCl4
f. H3O+
g. NH +

h. (h) BF −

i. (i) HCCH
j. (j) ClCN
k. (k) C 2+
2

(a)

In this case, the Lewis structure is inadequate to depict the fact that experimental studies have shown two unpaired electrons in
each oxygen molecule.
(b)

;
(c)

;
(d)

;
(e)

;
(f)

;
(g)

Access for free at OpenStax 7.E.7 https://chem.libretexts.org/@go/page/77587


;
(h)

;
(i)

;
(j)

;
(k)

Write Lewis structures for the following:


1. ClF3
2. PCl5
3. BF3
4. PF −
6

Write Lewis structures for the following:


1. SeF6
2. XeF4
3. SeCl +
3

4. Cl2BBCl2 (contains a B–B bond)


SeF6:

;
XeF4:

Access for free at OpenStax 7.E.8 https://chem.libretexts.org/@go/page/77587


SeCl
+

3
:

;
Cl2BBCl2:

Write Lewis structures for:


1. PO 3−

2. ICl −

3. SO 2−
3

4. HONO
Correct the following statement: “The bonds in solid PbCl2 are ionic; the bond in a HCl molecule is covalent. Thus, all of the
valence electrons in PbCl2 are located on the Cl– ions, and all of the valence electrons in a HCl molecule are shared between the H
and Cl atoms.”
Two valence electrons per Pb atom are transferred to Cl atoms; the resulting Pb2+ ion has a 6s2 valence shell configuration. Two of
the valence electrons in the HCl molecule are shared, and the other six are located on the Cl atom as lone pairs of electrons.
Write Lewis structures for the following molecules or ions:
1. SbH3
2. XeF2
3. Se8 (a cyclic molecule with a ring of eight Se atoms)
Methanol, H3COH, is used as the fuel in some race cars. Ethanol, C2H5OH, is used extensively as motor fuel in Brazil. Both
methanol and ethanol produce CO2 and H2O when they burn. Write the chemical equations for these combustion reactions using
Lewis structures instead of chemical formulas.

Many planets in our solar system contain organic chemicals including methane (CH4) and traces of ethylene (C2H4), ethane (C2H6),
propyne (H3CCCH), and diacetylene (HCCCCH). Write the Lewis structures for each of these molecules.
Carbon tetrachloride was formerly used in fire extinguishers for electrical fires. It is no longer used for this purpose because of the
formation of the toxic gas phosgene, Cl2CO. Write the Lewis structures for carbon tetrachloride and phosgene.

Access for free at OpenStax 7.E.9 https://chem.libretexts.org/@go/page/77587


Identify the atoms that correspond to each of the following electron configurations. Then, write the Lewis symbol for the common
ion formed from each atom:
1. 1s22s22p5
2. 1s22s22p63s2
3. 1s22s22p63s23p64s23d10
4. 1s22s22p63s23p64s23d104p4
5. 1s22s22p63s23p64s23d104p1
The arrangement of atoms in several biologically important molecules is given here. Complete the Lewis structures of these
molecules by adding multiple bonds and lone pairs. Do not add any more atoms.
the amino acid serine:

urea:

pyruvic acid:

uracil:

carbonic acid:

Access for free at OpenStax 7.E.10 https://chem.libretexts.org/@go/page/77587


(a)

;
(b)

;
(c)

;
(d)

;
(e)

A compound with a molar mass of about 28 g/mol contains 85.7% carbon and 14.3% hydrogen by mass. Write the Lewis structure
for a molecule of the compound.
A compound with a molar mass of about 42 g/mol contains 85.7% carbon and 14.3% hydrogen by mass. Write the Lewis structure
for a molecule of the compound.

Access for free at OpenStax 7.E.11 https://chem.libretexts.org/@go/page/77587


Two arrangements of atoms are possible for a compound with a molar mass of about 45 g/mol that contains 52.2% C, 13.1% H, and
34.7% O by mass. Write the Lewis structures for the two molecules.
How are single, double, and triple bonds similar? How do they differ?
Each bond includes a sharing of electrons between atoms. Two electrons are shared in a single bond; four electrons are shared in a
double bond; and six electrons are shared in a triple bond.

7.E.4: 7.5: Formal Charges and Resonance


Write resonance forms that describe the distribution of electrons in each of these molecules or ions.
1. selenium dioxide, OSeO
2. nitrate ion, NO−
3

3. nitric acid, HNO3 (N is bonded to an OH group and two O atoms)


4. benzene, C6H6:

the formate ion:

Write resonance forms that describe the distribution of electrons in each of these molecules or ions.
1. sulfur dioxide, SO2
2. carbonate ion, CO 2−

3. hydrogen carbonate ion, HCO (C is bonded to an OH group and two O atoms)


4. pyridine:

the allyl ion:

Access for free at OpenStax 7.E.12 https://chem.libretexts.org/@go/page/77587


(a)

;
(b)

;
(c)

;
(d)

;
(e)

Write the resonance forms of ozone, O3, the component of the upper atmosphere that protects the Earth from ultraviolet radiation.
Sodium nitrite, which has been used to preserve bacon and other meats, is an ionic compound. Write the resonance forms of the
nitrite ion, NO .

Access for free at OpenStax 7.E.13 https://chem.libretexts.org/@go/page/77587


In terms of the bonds present, explain why acetic acid, CH3CO2H, contains two distinct types of carbon-oxygen bonds, whereas the
acetate ion, formed by loss of a hydrogen ion from acetic acid, only contains one type of carbon-oxygen bond. The skeleton
structures of these species are shown:

Write the Lewis structures for the following, and include resonance structures where appropriate. Indicate which has the strongest
carbon-oxygen bond.
1. CO2
2. CO
(a)

(b)

CO has the strongest carbon-oxygen bond because there is a triple bond joining C and O. CO2 has double bonds.
Toothpastes containing sodium hydrogen carbonate (sodium bicarbonate) and hydrogen peroxide are widely used. Write Lewis
structures for the hydrogen carbonate ion and hydrogen peroxide molecule, with resonance forms where appropriate.
Determine the formal charge of each element in the following:
1. HCl
2. CF4
3. PCl3
4. PF5
H: 0, Cl: 0; C: 0, F: 0; P: 0, Cl 0; P: 0, F: 0
Determine the formal charge of each element in the following:
1. H3O+
2. SO 2−
4

3. NH3
4. O 2−

5. H2O2
Calculate the formal charge of chlorine in the molecules Cl2, BeCl2, and ClF5.
Cl in Cl2: 0; Cl in BeCl2: 0; Cl in ClF5: 0
Calculate the formal charge of each element in the following compounds and ions:
1. F2CO
2. NO–
3. BF −

4. SnCl −
3

5. H2CCH2
6. ClF3
7. SeF6
8. (h) PO 3−
4

Access for free at OpenStax 7.E.14 https://chem.libretexts.org/@go/page/77587


Draw all possible resonance structures for each of these compounds. Determine the formal charge on each atom in each of the
resonance structures:
1. O3
2. SO2
3. NO −
2

4. NO −
3

;
(b)

;
(c)

;
(d)

Based on formal charge considerations, which of the following would likely be the correct arrangement of atoms in nitrosyl
chloride: ClNO or ClON?
Based on formal charge considerations, which of the following would likely be the correct arrangement of atoms in hypochlorous
acid: HOCl or OClH?
HOCl
Based on formal charge considerations, which of the following would likely be the correct arrangement of atoms in sulfur dioxide:
OSO or SOO?
Draw the structure of hydroxylamine, H3NO, and assign formal charges; look up the structure. Is the actual structure consistent
with the formal charges?
The structure that gives zero formal charges is consistent with the actual structure:

Iodine forms a series of fluorides (listed here). Write Lewis structures for each of the four compounds and determine the formal
charge of the iodine atom in each molecule:

Access for free at OpenStax 7.E.15 https://chem.libretexts.org/@go/page/77587


1. IF
2. IF3
3. IF5
4. IF7
Write the Lewis structure and chemical formula of the compound with a molar mass of about 70 g/mol that contains 19.7%
nitrogen and 80.3% fluorine by mass, and determine the formal charge of the atoms in this compound.
NF3;

Which of the following structures would we expect for nitrous acid? Determine the formal charges:

Sulfuric acid is the industrial chemical produced in greatest quantity worldwide. About 90 billion pounds are produced each year in
the United States alone. Write the Lewis structure for sulfuric acid, H2SO4, which has two oxygen atoms and two OH groups
bonded to the sulfur.

7.E.5: 7.6: Strengths of Ionic and Covalent Bonds


Which bond in each of the following pairs of bonds is the strongest?
1. C–C or C = C
2. C–N or C ≡ N
3. C ≡ O or C = O
4. H–F or H–Cl
5. C–H or O–H
6. C–N or C–O
Using the bond energies in Table, determine the approximate enthalpy change for each of the following reactions:
1. H (g) + Br (g) ⟶ 2 HBr(g)
2 2

2. CH (g) + I (g) ⟶ CH I(g) + HI(g)


4 2 3

3. (c) C H (g) + 3 O (g) ⟶ 2 CO (g) + 2 H


2 4 2 2 2
O(g)

1. −114 kJ;
2. 30 kJ;
3. (c) −1055 kJ
Using the bond energies in Table, determine the approximate enthalpy change for each of the following reactions:
1. Cl (g) + 3 F (g) ⟶ 2 ClF (g)
2 2 3

2. H C = C H (g) + H (g) ⟶ H CCH (g)


2 2 2 3 3

3. (c) 2 C H (g) + 7 O (g) ⟶ 4 CO (g) + 6 H


2 6 2 2 2
O(g)

Access for free at OpenStax 7.E.16 https://chem.libretexts.org/@go/page/77587


When a molecule can form two different structures, the structure with the stronger bonds is usually the more stable form. Use bond
energies to predict the correct structure of the hydroxylamine molecule:

The greater bond energy is in the figure on the left. It is the more stable form.
How does the bond energy of HCldiffer from the standard enthalpy of formation of HCl(g)?
Using the standard enthalpy of formation data in Appendix G, show how the standard enthalpy of formation of HCl(g) can be used
to determine the bond energy.
1 1
∘ ∘
HCl(g) ⟶ H (g) + Cl (g) ΔH = −ΔH
2 2 1 f[HCl(g)]
2 2

1
∘ ∘
H (g) ⟶ H(g) ΔH = ΔH
2 2 f[H(g)]
2

1
∘ ∘
Cl (g) ⟶ Cl(g) ΔH = ΔH
2 3 f[Cl(g)]
2
–––––––––––––––––––––––––––––––––––––––––––––––
∘ ∘ ∘ ∘
HCl(g) ⟶ H(g) + Cl(g) ΔH = ΔH + ΔH + ΔH
298 1 2 3

∘ ∘ ∘ ∘
DHCl = ΔH = ΔH + ΔH + ΔH (7.E.1)
298 f[HCl(g)] f[H(g)] f[Cl(g)]

= −(−92.307 kJ) + 217.97 kJ + 121.3 kJ (7.E.2)

= 431.6 kJ (7.E.3)

Using the standard enthalpy of formation data in Appendix G, calculate the bond energy of the carbon-sulfur double bond in CS2.
Using the standard enthalpy of formation data in Appendix G, determine which bond is stronger: the S–F bond in SF4(g) or in
SF6(g)?
The S–F bond in SF4 is stronger.
Using the standard enthalpy of formation data in Appendix G, determine which bond is stronger: the P–Cl bond in PCl3(g) or in
PCl5(g)?
Complete the following Lewis structure by adding bonds (not atoms), and then indicate the longest bond:

The C–C single bonds are longest.


Use the bond energy to calculate an approximate value of ΔH for the following reaction. Which is the more stable form of FNO2?

Access for free at OpenStax 7.E.17 https://chem.libretexts.org/@go/page/77587


Use principles of atomic structure to answer each of the following:1
1. The radius of the Ca atom is 197 pm; the radius of the Ca2+ ion is 99 pm. Account for the difference.
2. The lattice energy of CaO(s) is –3460 kJ/mol; the lattice energy of K2O is –2240 kJ/mol. Account for the difference.
3. (c) Given these ionization values, explain the difference between Ca and K with regard to their first and second ionization
energies.

Element First Ionization Energy (kJ/mol) Second Ionization Energy (kJ/mol)

K 419 3050

Ca 590 1140

The first ionization energy of Mg is 738 kJ/mol and that of Al is 578 kJ/mol. Account for this difference.
When two electrons are removed from the valence shell, the Ca radius loses the outermost energy level and reverts to the lower n =
3 level, which is much smaller in radius. The +2 charge on calcium pulls the oxygen much closer compared with K, thereby
increasing the lattice energy relative to a less charged ion. (c) Removal of the 4s electron in Ca requires more energy than removal
of the 4s electron in K because of the stronger attraction of the nucleus and the extra energy required to break the pairing of the
electrons. The second ionization energy for K requires that an electron be removed from a lower energy level, where the attraction
is much stronger from the nucleus for the electron. In addition, energy is required to unpair two electrons in a full orbital. For Ca,
the second ionization potential requires removing only a lone electron in the exposed outer energy level. In Al, the removed
electron is relatively unprotected and unpaired in a p orbital. The higher energy for Mg mainly reflects the unpairing of the 2s
electron.
The lattice energy of LiF is 1023 kJ/mol, and the Li–F distance is 200.8 pm. NaF crystallizes in the same structure as LiF but with a
Na–F distance of 231 pm. Which of the following values most closely approximates the lattice energy of NaF: 510, 890, 1023,
1175, or 4090 kJ/mol? Explain your choice.
For which of the following substances is the least energy required to convert one mole of the solid into separate ions?
1. MgO
2. SrO
3. (c) KF
4. CsF
5. MgF2
(d)
The reaction of a metal, M, with a halogen, X2, proceeds by an exothermic reaction as indicated by this equation:
M(s) + X (g) ⟶ MX (s) . For each of the following, indicate which option will make the reaction more exothermic. Explain
2 2

your answers.
1. a large radius vs. a small radius for M+2
2. a high ionization energy vs. a low ionization energy for M
3. (c) an increasing bond energy for the halogen
4. a decreasing electron affinity for the halogen
5. an increasing size of the anion formed by the halogen
The lattice energy of LiF is 1023 kJ/mol, and the Li–F distance is 201 pm. MgO crystallizes in the same structure as LiF but with a
Mg–O distance of 205 pm. Which of the following values most closely approximates the lattice energy of MgO: 256 kJ/mol, 512
kJ/mol, 1023 kJ/mol, 2046 kJ/mol, or 4008 kJ/mol? Explain your choice.
4008 kJ/mol; both ions in MgO have twice the charge of the ions in LiF; the bond length is very similar and both have the same
structure; a quadrupling of the energy is expected based on the equation for lattice energy
Which compound in each of the following pairs has the larger lattice energy? Note: Mg2+ and Li+ have similar radii; O2– and F–
have similar radii. Explain your choices.
1. MgO or MgSe
2. LiF or MgO
3. (c) Li2O or LiCl

Access for free at OpenStax 7.E.18 https://chem.libretexts.org/@go/page/77587


4. Li2Se or MgO
Which compound in each of the following pairs has the larger lattice energy? Note: Ba2+ and
K+ have similar radii; S2– and Cl– have similar radii. Explain your choices.
1. K2O or Na2O
2. K2S or BaS
3. (c) KCl or BaS
4. BaS or BaCl2
Na2O; Na+ has a smaller radius than K+; BaS; Ba has a larger charge than K; (c) BaS; Ba and S have larger charges; BaS; S has a
larger charge
Which of the following compounds requires the most energy to convert one mole of the solid into separate ions?
1. MgO
2. SrO
3. (c) KF
4. CsF
5. MgF2
Which of the following compounds requires the most energy to convert one mole of the solid into separate ions?
1. K2S
2. K2O
3. (c) CaS
4. Cs2S
5. CaO
(e)
The lattice energy of KF is 794 kJ/mol, and the interionic distance is 269 pm. The Na–F
distance in NaF, which has the same structure as KF, is 231 pm. Which of the following values is the closest approximation of the
lattice energy of NaF: 682 kJ/mol, 794 kJ/mol, 924 kJ/mol, 1588 kJ/mol, or 3175 kJ/mol? Explain your answer.

7.E.6: 7.7: Molecular Structure and Polarity


Explain why the HOH molecule is bent, whereas the HBeH molecule is linear.
The placement of the two sets of unpaired electrons in water forces the bonds to assume a tetrahedral arrangement, and the
resulting HOH molecule is bent. The HBeH molecule (in which Be has only two electrons to bond with the two electrons from the
hydrogens) must have the electron pairs as far from one another as possible and is therefore linear.
What feature of a Lewis structure can be used to tell if a molecule’s (or ion’s) electron-pair geometry and molecular structure will
be identical?
Explain the difference between electron-pair geometry and molecular structure.
Space must be provided for each pair of electrons whether they are in a bond or are present as lone pairs. Electron-pair geometry
considers the placement of all electrons. Molecular structure considers only the bonding-pair geometry.
Why is the H–N–H angle in NH3 smaller than the H–C–H bond angle in CH4? Why is the H–N–H angle in NH
+

4
identical to the
H–C–H bond angle in CH4?
Explain how a molecule that contains polar bonds can be nonpolar.
As long as the polar bonds are compensated (for example. two identical atoms are found directly across the central atom from one
another), the molecule can be nonpolar.
As a general rule, MXn molecules (where M represents a central atom and X represents terminal atoms; n = 2 – 5) are polar if there
is one or more lone pairs of electrons on M. NH3 (M = N, X = H, n = 3) is an example. There are two molecular structures with
lone pairs that are exceptions to this rule. What are they?
Predict the electron pair geometry and the molecular structure of each of the following molecules or ions:

Access for free at OpenStax 7.E.19 https://chem.libretexts.org/@go/page/77587


1. SF6
2. PCl5
3. (c) BeH2
4. CH +

1. Both the electron geometry and the molecular structure are octahedral.
2. Both the electron geometry and the molecular structure are trigonal bipyramid.
3. (c) Both the electron geometry and the molecular structure are linear.
4. Both the electron geometry and the molecular structure are trigonal planar.
Identify the electron pair geometry and the molecular structure of each of the following molecules or ions:
1. IF+
6

2. CF4
3. (c) BF3
4. SiF −

5. BeCl2
What are the electron-pair geometry and the molecular structure of each of the following molecules or ions?
1. ClF5
2. ClO −

3. (c) TeCl 2−

4. PCl3
5. SeF4
6. PH −

electron-pair geometry: octahedral, molecular structure: square pyramidal; electron-pair geometry: tetrahedral, molecular structure:
bent; (c) electron-pair geometry: octahedral, molecular structure: square planar; electron-pair geometry: tetrahedral, molecular
structure: trigonal pyramidal; electron-pair geometry: trigonal bypyramidal, molecular structure: seesaw; electron-pair geometry:
tetrahedral, molecular structure: bent (109°)
Predict the electron pair geometry and the molecular structure of each of the following ions:
1. H3O+
2. PCl −

3. (c) SnCl −

4. BrCl −

5. ICl3
6. XeF4
7. (g) SF2
Identify the electron pair geometry and the molecular structure of each of the following molecules:
1. ClNO (N is the central atom)
2. CS2
3. (c) Cl2CO (C is the central atom)
4. Cl2SO (S is the central atom)
5. SO2F2 (S is the central atom)
6. XeO2F2 (Xe is the central atom)
7. (g) ClOF (Cl is the central atom)
+
2

electron-pair geometry: trigonal planar, molecular structure: bent (120°); electron-pair geometry: linear, molecular structure: linear;
(c) electron-pair geometry: trigonal planar, molecular structure: trigonal planar; electron-pair geometry: tetrahedral, molecular
structure: trigonal pyramidal; electron-pair geometry: tetrahedral, molecular structure: tetrahedral; electron-pair geometry: trigonal
bipyramidal, molecular structure: seesaw; (g) electron-pair geometry: tetrahedral, molecular structure: trigonal pyramidal
Predict the electron pair geometry and the molecular structure of each of the following:
1. IOF5 (I is the central atom)

Access for free at OpenStax 7.E.20 https://chem.libretexts.org/@go/page/77587


2. POCl3 (P is the central atom)
3. (c) Cl2SeO (Se is the central atom)
4. ClSO+ (S is the central atom)
5. F2SO (S is the central atom)
6. NO −

7. (g) SiO 4−

Which of the following molecules and ions contain polar bonds? Which of these molecules and ions have dipole moments?
1. ClF5
2. ClO −
2

3. (c) TeCl 2−
4

4. PCl3
5. SeF4
6. PH −
2

7. (g) XeF2
All of these molecules and ions contain polar bonds. Only ClF5, ClO , PCl3, SeF4, and PH have dipole moments.

2

Which of the molecules and ions in Exercise contain polar bonds? Which of these molecules and ions have dipole moments?
1. H3O+
2. PCl−
4

3. (c) SnCl −
3

4. BrCl −
4

5. ICl3
6. XeF4
7. (g) SF2
Which of the following molecules have dipole moments?
1. CS2
2. SeS2
3. (c) CCl2F2
4. PCl3 (P is the central atom)
5. ClNO (N is the central atom)
SeS2, CCl2F2, PCl3, and ClNO all have dipole moments.
Identify the molecules with a dipole moment:
1. SF4
2. CF4
3. (c) Cl2CCBr2
4. CH3Cl
5. H2CO
The molecule XF3 has a dipole moment. Is X boron or phosphorus?
P
The molecule XCl2 has a dipole moment. Is X beryllium or sulfur?
Is the Cl2BBCl2 molecule polar or nonpolar?
nonpolar
There are three possible structures for PCl2F3 with phosphorus as the central atom. Draw them and discuss how measurements of
dipole moments could help distinguish among them.
Describe the molecular structure around the indicated atom or atoms:
1. the sulfur atom in sulfuric acid, H2SO4 [(HO)2SO2]

Access for free at OpenStax 7.E.21 https://chem.libretexts.org/@go/page/77587


2. the chlorine atom in chloric acid, HClO3 [HOClO2]
3. (c) the oxygen atom in hydrogen peroxide, HOOH
4. the nitrogen atom in nitric acid, HNO3 [HONO2]
5. the oxygen atom in the OH group in nitric acid, HNO3 [HONO2]
6. the central oxygen atom in the ozone molecule, O3
7. (g) each of the carbon atoms in propyne, CH3CCH
8. (h) the carbon atom in Freon, CCl2F2
9. (i) each of the carbon atoms in allene, H2CCCH2
tetrahedral; trigonal pyramidal; (c) bent (109°); trigonal planar; bent (109°); bent (109°); (g) CH3CCH tetrahedral, CH3CCH linear;
(h) tetrahedral; (i) H2CCCH2 linear; H2CCCH2 trigonal planar
Draw the Lewis structures and predict the shape of each compound or ion:
1. CO2
2. NO −

3. (c) SO3
4. SO2−

A molecule with the formula AB2, in which A and B represent different atoms, could have one of three different shapes. Sketch and
name the three different shapes that this molecule might have. Give an example of a molecule or ion for each shape.

A molecule with the formula AB3, in which A and B represent different atoms, could have one of three different shapes. Sketch and
name the three different shapes that this molecule might have. Give an example of a molecule or ion that has each shape.
Draw the Lewis electron dot structures for these molecules, including resonance structures where appropriate:
1. CS2−
3

2. CS2
3. (c) CS
predict the molecular shapes for CS2−
3
and CS2 and explain how you arrived at your predictions
(a)

;
(b)

;
(c)

Access for free at OpenStax 7.E.22 https://chem.libretexts.org/@go/page/77587


CS3
2−
includes three regions of electron density (all are bonds with no lone pairs); the shape is trigonal planar; CS2 has only two
regions of electron density (all bonds with no lone pairs); the shape is linear
What is the molecular structure of the stable form of FNO2? (N is the central atom.)
A compound with a molar mass of about 42 g/mol contains 85.7% carbon and 14.3% hydrogen. What is its molecular structure?
The Lewis structure is made from three units, but the atoms must be rearranged:

Use the simulation to perform the following exercises for a two-atom molecule:
1. Adjust the electronegativity value so the bond dipole is pointing toward B. Then determine what the electronegativity values
must be to switch the dipole so that it points toward A.
2. With a partial positive charge on A, turn on the electric field and describe what happens.
3. (c) With a small partial negative charge on A, turn on the electric field and describe what happens.
4. Reset all, and then with a large partial negative charge on A, turn on the electric field and describe what happens.
Use the simulation to perform the following exercises for a real molecule. You may need to rotate the molecules in three
dimensions to see certain dipoles.
1. Sketch the bond dipoles and molecular dipole (if any) for O3. Explain your observations.
2. Look at the bond dipoles for NH3. Use these dipoles to predict whether N or H is more electronegative.
3. (c) Predict whether there should be a molecular dipole for NH3 and, if so, in which direction it will point. Check the molecular
dipole box to test your hypothesis.
The molecular dipole points away from the hydrogen atoms.
Use the Molecule Shape simulator to build a molecule. Starting with the central atom, click on the double bond to add one double
bond. Then add one single bond and one lone pair. Rotate the molecule to observe the complete geometry. Name the electron group
geometry and molecular structure and predict the bond angle. Then click the check boxes at the bottom and right of the simulator to
check your answers.
Use the Molecule Shape simulator to explore real molecules. On the Real Molecules tab, select H2O. Switch between the “real”
and “model” modes. Explain the difference observed.
The structures are very similar. In the model mode, each electron group occupies the same amount of space, so the bond angle is
shown as 109.5°. In the “real” mode, the lone pairs are larger, causing the hydrogens to be compressed. This leads to the smaller
angle of 104.5°.
Use the Molecule Shape simulator to explore real molecules. On the Real Molecules tab, select “model” mode and S2O. What is
the model bond angle? Explain whether the “real” bond angle should be larger or smaller than the ideal model angle.

This page titled 7.E: Chemical Bonding and Molecular Geometry (Exercises) is shared under a CC BY 4.0 license and was authored, remixed,
and/or curated by OpenStax via source content that was edited to the style and standards of the LibreTexts platform; a detailed edit history is
available upon request.

Access for free at OpenStax 7.E.23 https://chem.libretexts.org/@go/page/77587


CHAPTER OVERVIEW
8: Advanced Theories of Covalent Bonding

A general chemistry Libretexts Textbook remixed and remastered from


OpenStax's textbook:
General Chemistry
We have examined the basic ideas of bonding, showing that atoms share electrons to form molecules with stable Lewis structures
and that we can predict the shapes of those molecules by valence shell electron pair repulsion (VSEPR) theory. These ideas provide
an important starting point for understanding chemical bonding. But these models sometimes fall short in their abilities to predict
the behavior of real substances. How can we reconcile the geometries of s, p, and d atomic orbitals with molecular shapes that
show angles like 120° and 109.5°? Furthermore, we know that electrons and magnetic behavior are related through electromagnetic
fields.
8.0: Prelude to Covalent Bonding
8.1: Valence Bond Theory
8.2: Hybrid Atomic Orbitals
8.3: Multiple Bonds
8.4: Molecular Orbital Theory
8.E: Advanced Theories of Covalent Bonding (Exercises)

This page titled 8: Advanced Theories of Covalent Bonding is shared under a CC BY 4.0 license and was authored, remixed, and/or curated by
OpenStax via source content that was edited to the style and standards of the LibreTexts platform; a detailed edit history is available upon request.

1
8.0: Prelude to Covalent Bonding
We have examined the basic ideas of bonding, showing that atoms share electrons to form molecules with stable Lewis structures
and that we can predict the shapes of those molecules by valence shell electron pair repulsion (VSEPR) theory. These ideas provide
an important starting point for understanding chemical bonding. But these models sometimes fall short in their abilities to predict
the behavior of real substances. How can we reconcile the geometries of s, p, and d atomic orbitals with molecular shapes that
show angles like 120° and 109.5°? Furthermore, we know that electrons and magnetic behavior are related through electromagnetic
fields. Both N2 and O2 have fairly similar Lewis structures that contain lone pairs of electrons.

Two Lewis diagrams are shown. The diagram on the left shows two nitrogen atoms, represented by the letter N connected by three
lines and with a lone pair of electrons on each end of the structure. The diagram on the right shows two oxygen atoms, depicted by
the letter O, connected by two lines. Two pairs of electrons surround each oxygen to the top and ends of the structure.
Yet oxygen demonstrates very different magnetic behavior than nitrogen. We can pour liquid nitrogen through a magnetic field with
no visible interactions, while liquid oxygen is attracted to the magnet and floats in the magnetic field. We need to understand the
additional concepts of valence bond theory, orbital hybridization, and molecular orbital theory to understand these observations.

Figure 8.0.1 : Oxygen molecules orient randomly most of the time, as shown in the top magnified view. However, when we pour
liquid oxygen through a magnet, the molecules line up with the magnetic field, and the attraction allows them to stay suspended
between the poles of the magnet where the magnetic field is strongest. Other diatomic molecules (like N2) flow past the magnet.
The detailed explanation of bonding described in this chapter allows us to understand this phenomenon. (credit: modification of
work by Jefferson Lab)
A pitcher is shown pouring liquid oxygen through the gap between two magnets, where it has formed a solid disk. A call out box
near the stream of liquid oxygen shows an image of six pairs of spheres, spread apart from one another. Another call out box near
the solid disk shows ten pairs of spheres much closer together.

This page titled 8.0: Prelude to Covalent Bonding is shared under a CC BY 4.0 license and was authored, remixed, and/or curated by OpenStax
via source content that was edited to the style and standards of the LibreTexts platform; a detailed edit history is available upon request.

Access for free at OpenStax 8.0.1 https://chem.libretexts.org/@go/page/38191


8.1: Valence Bond Theory
 Learning Objectives
Describe the formation of covalent bonds in terms of atomic orbital overlap
Define and give examples of σ and π bonds

As we know, a scientific theory is a strongly supported explanation for observed natural laws or large bodies of experimental data.
For a theory to be accepted, it must explain experimental data and be able to predict behavior. For example, VSEPR theory has
gained widespread acceptance because it predicts three-dimensional molecular shapes that are consistent with experimental data
collected for thousands of different molecules. However, VSEPR theory does not provide an explanation of chemical bonding.
There are successful theories that describe the electronic structure of atoms. We can use quantum mechanics to predict the specific
regions around an atom where electrons are likely to be located: A spherical shape for an s orbital, a dumbbell shape for a p orbital,
and so forth. However, these predictions only describe the orbitals around free atoms. When atoms bond to form molecules, atomic
orbitals are not sufficient to describe the regions where electrons will be located in the molecule. A more complete understanding
of electron distributions requires a model that can account for the electronic structure of molecules. One popular theory holds that a
covalent bond forms when a pair of electrons is shared by two atoms and is simultaneously attracted by the nuclei of both atoms. In
the following sections, we will discuss how such bonds are described by valence bond theory and hybridization.
Valence bond theory describes a covalent bond as the overlap of half-filled atomic orbitals (each containing a single electron) that
yield a pair of electrons shared between the two bonded atoms. We say that orbitals on two different atoms overlap when a portion
of one orbital and a portion of a second orbital occupy the same region of space. According to valence bond theory, a covalent bond
results when two conditions are met:
1. an orbital on one atom overlaps an orbital on a second atom and
2. the single electrons in each orbital combine to form an electron pair.
The mutual attraction between this negatively charged electron pair and the two atoms’ positively charged nuclei serves to
physically link the two atoms through a force we define as a covalent bond. The strength of a covalent bond depends on the extent
of overlap of the orbitals involved. Orbitals that overlap extensively form bonds that are stronger than those that have less overlap.
The energy of the system depends on how much the orbitals overlap. Figure 8.1.1 illustrates how the sum of the energies of two
hydrogen atoms (the colored curve) changes as they approach each other. When the atoms are far apart there is no overlap, and by
convention we set the sum of the energies at zero. As the atoms move together, their orbitals begin to overlap. Each electron begins
to feel the attraction of the nucleus in the other atom. In addition, the electrons begin to repel each other, as do the nuclei. While the
atoms are still widely separated, the attractions are slightly stronger than the repulsions, and the energy of the system decreases. (A
bond begins to form.) As the atoms move closer together, the overlap increases, so the attraction of the nuclei for the electrons
continues to increase (as do the repulsions among electrons and between the nuclei). At some specific distance between the atoms,
which varies depending on the atoms involved, the energy reaches its lowest (most stable) value. This optimum distance between
the two bonded nuclei is the bond distance between the two atoms. The bond is stable because at this point, the attractive and
repulsive forces combine to create the lowest possible energy configuration. If the distance between the nuclei were to decrease
further, the repulsions between nuclei and the repulsions as electrons are confined in closer proximity to each other would become
stronger than the attractive forces. The energy of the system would then rise (making the system destabilized), as shown at the far
left of Figure 8.1.1.

Access for free at OpenStax 8.1.1 https://chem.libretexts.org/@go/page/38192


Figure 8.1.1 : The interaction of two hydrogen atoms changes as a function of distance. The energy of the system changes as the
atoms interact. The lowest (most stable) energy occurs at a distance of 74 pm, which is the bond length observed for the H2
molecule.
A pair of diagrams are shown and labeled “a” and “b”. Diagram a shows three consecutive images. The first image depicts two
separated blurry circles, each labeled with a positive sign and the term “H atom.” The phrase written under them reads,
“Sufficiently far apart to have no interaction.” The second image shows the same two circles, but this time they are much closer
together and are labeled, “Atoms begin to interact as they move closer together.” The third image shows the two circles
overlapping, labeled, “H subscript 2,” and, “Optimum distance to achieve lowest overall energy of system.” Diagram b shows a
graph on which the y-axis is labeled “Energy ( J ),” and the x-axis is labeled, “Internuclear distance ( p m ).” The midpoint of the y-
axis is labeled as zero. The curve on the graph begins at zero p m and high on the y-axis. The graph slopes downward steeply to a
point far below the zero joule line on the y-axis and the lowest point reads “0.74 p m” and “H bonded to H bond length.” It is also
labeled “ negative 7.24 times 10 superscript negative 19 J.” The graph then rises again to zero J. The graph is accompanied by the
same images from diagram a; the first image correlates to the point in the graph where it crosses the zero point on the y-axis, the
third image where the graph is lowest.
The bond energy is the difference between the energy minimum (which occurs at the bond distance) and the energy of the two
separated atoms. This is the quantity of energy released when the bond is formed. Conversely, the same amount of energy is
required to break the bond. For the H molecule shown in Figure 8.1.1, at the bond distance of 74 pm the system is
2

J lower in energy than the two separated hydrogen atoms. This may seem like a small number. However, we know
−19
7.24 × 10

from our earlier description of thermochemistry that bond energies are often discussed on a per-mole basis. For example, it requires
J to break one H–H bond, but it takes 4.36 × 10 J to break 1 mole of H–H bonds. A comparison of some bond
−19 5
7.24 × 10

lengths and energies is shown in Table 8.1.1. We can find many of these bonds in a variety of molecules, and this table provides
average values. For example, breaking the first C–H bond in CH4 requires 439.3 kJ/mol, while breaking the first C–H bond in
H– CH C H (a common paint thinner) requires 375.5 kJ/mol.
2 6 5

Table 8.1.1 : Representative Bond Energies and Lengths


Bond Length (pm) Energy (kJ/mol) Bond Length (pm) Energy (kJ/mol)

H–H 74 436 C–O 140.1 358

H–C 106.8 413 C = O 119.7 745

H–N 101.5 391 C ≡ O 113.7 1072

H–O 97.5 467 H–Cl 127.5 431

C–C 150.6 347 H–Br 141.4 366

C = C 133.5 614 H–I 160.9 298

Access for free at OpenStax 8.1.2 https://chem.libretexts.org/@go/page/38192


Bond Length (pm) Energy (kJ/mol) Bond Length (pm) Energy (kJ/mol)

C ≡ C 120.8 839 O–O 148 146

C–N 142.1 305 O = O 120.8 498

C = N 130.0 615 F–F 141.2 159

C ≡ N 116.1 891 Cl–Cl 198.8 243

In addition to the distance between two orbitals, the orientation of orbitals also affects their overlap (other than for two s orbitals,
which are spherically symmetric). Greater overlap is possible when orbitals are oriented such that they overlap on a direct line
between the two nuclei. Figure 8.1.2 illustrates this for two p orbitals from different atoms; the overlap is greater when the orbitals
overlap end to end rather than at an angle.

Figure 8.1.2 : (a) The overlap of two p orbitals is greatest when the orbitals are directed end to end. (b) Any other arrangement
results in less overlap. The dots indicate the locations of the nuclei.
The overlap of two s orbitals (as in H2), the overlap of an s orbital and a p orbital (as in HCl), and the end-to-end overlap of two p
orbitals (as in Cl2) all produce sigma bonds (σ bonds), as illustrated in Figure 8.1.3. A σ bond is a covalent bond in which the
electron density is concentrated in the region along the internuclear axis; that is, a line between the nuclei would pass through the
center of the overlap region. Single bonds in Lewis structures are described as σ bonds in valence bond theory.

Figure 8.1.3 : Sigma (σ) bonds form from the overlap of the following: (a) two s orbitals, (b) an s orbital and a p orbital, and (c) two
p orbitals. The dots indicate the locations of the nuclei.
Three diagrams are shown and labeled “a,” “b,” and “c.” Diagram a shows two spherical orbitals lying side by side and
overlapping. Diagram b shows one spherical and one peanut-shaped orbital lying near one another so that the spherical orbital
overlaps with one end of the peanut-shaped orbital. Diagram c shows two peanut-shaped orbitals lying end to end so that one end
of each orbital overlaps the other.
A pi bond (π bond) is a type of covalent bond that results from the side-by-side overlap of two p orbitals, as illustrated in Figure
8.1.4. In a π bond, the regions of orbital overlap lie on opposite sides of the internuclear axis. Along the axis itself, there is a node,

that is, a plane with no probability of finding an electron.


<div data-mt-source="1"
&quot;&quot;
" style="width: 325px; height: 115px;" width="325px" height="115px" src="/@api/deki/files/56400/CNX_Chem_08_01_pi.jpg">
Figure 8.1.4 : Pi (π) bonds form from the side-by-side overlap of two p orbitals. The dots indicate the location of the nuclei.
While all single bonds are σ bonds, multiple bonds consist of both σ and π bonds. As the Lewis structures suggest, O2 contains a
double bond, and N2 contains a triple bond. The double bond consists of one σ bond and one π bond, and the triple bond consists of
one σ bond and two π bonds. Between any two atoms, the first bond formed will always be a σ bond, but there can only be one σ
bond in any one location. In any multiple bond, there will be one σ bond, and the remaining one or two bonds will be π bonds.
These bonds are described in more detail later in this chapter.

Access for free at OpenStax 8.1.3 https://chem.libretexts.org/@go/page/38192


A diagram contains three Lewis structures. The left most structure shows an H atom bonded to a C l atom by a single bond. The C l
atom has three lone pairs of electrons. The phrase “One sigma bond No pi bonds” is written below the drawing. The center
structure shows two O atoms bonded by a double bond. The O atoms each have two lone pairs of electrons. The phrase “One sigma
bond One pi bond” is written below the drawing. The right most structure shows two N atoms bonded by a triple bond. Each N
atom has a lone pairs of electrons. The phrase “One sigma bond Two pi bonds” is written below the drawing.
As seen in Table 8.1.1, an average carbon-carbon single bond is 347 kJ/mol, while in a carbon-carbon double bond, the π bond
increases the bond strength by 267 kJ/mol. Adding an additional π bond causes a further increase of 225 kJ/mol. We can see a
similar pattern when we compare other σ and π bonds. Thus, each individual π bond is generally weaker than a corresponding σ
bond between the same two atoms. In a σ bond, there is a greater degree of orbital overlap than in a π bond.

 Example 8.1.1: Counting σ and π Bonds

Butadiene, C4H6, is used to make synthetic rubber. Identify the number of σ and π bonds contained in this molecule.

Butadiene

Solution
There are six σ C–H bonds and one σ C–C bond, for a total of seven from the single bonds. There are two double bonds that
each have a π bond in addition to the σ bond. This gives a total nine σ and two π bonds overall.

 Exercise 8.1.1

Identify each illustration as depicting a σ or π bond:


a. side-by-side overlap of a 4p and a 2p orbital
b. end-to-end overlap of a 4p and 4p orbital
c. end-to-end overlap of a 4p and a 2p orbital

Answer
(a) is a π bond with a node along the axis connecting the nuclei while (b) and (c) are σ bonds that overlap along the axis.

Summary
Valence bond theory describes bonding as a consequence of the overlap of two separate atomic orbitals on different atoms that
creates a region with one pair of electrons shared between the two atoms. When the orbitals overlap along an axis containing the

Access for free at OpenStax 8.1.4 https://chem.libretexts.org/@go/page/38192


nuclei, they form a σ bond. When they overlap in a fashion that creates a node along this axis, they form a π bond.

Glossary
overlap
coexistence of orbitals from two different atoms sharing the same region of space, leading to the formation of a covalent bond

node
plane separating different lobes of orbitals, where the probability of finding an electron is zero

pi bond (π bond)
covalent bond formed by side-by-side overlap of atomic orbitals; the electron density is found on opposite sides of the
internuclear axis

sigma bond (σ bond)


covalent bond formed by overlap of atomic orbitals along the internuclear axis

valence bond theory


description of bonding that involves atomic orbitals overlapping to form σ or π bonds, within which pairs of electrons are
shared

This page titled 8.1: Valence Bond Theory is shared under a CC BY 4.0 license and was authored, remixed, and/or curated by OpenStax via
source content that was edited to the style and standards of the LibreTexts platform; a detailed edit history is available upon request.

Access for free at OpenStax 8.1.5 https://chem.libretexts.org/@go/page/38192


8.2: Hybrid Atomic Orbitals
 Learning Objectives
Explain the concept of atomic orbital hybridization
Determine the hybrid orbitals associated with various molecular geometries

Thinking in terms of overlapping atomic orbitals is one way for us to explain how chemical bonds form in diatomic molecules.
However, to understand how molecules with more than two atoms form stable bonds, we require a more detailed model. As an
example, let us consider the water molecule, in which we have one oxygen atom bonding to two hydrogen atoms. Oxygen has the
electron configuration 1s22s22p4, with two unpaired electrons (one in each of the two 2p orbitals). Valence bond theory would
predict that the two O–H bonds form from the overlap of these two 2p orbitals with the 1s orbitals of the hydrogen atoms. If this
were the case, the bond angle would be 90°, as shown in Figure 8.2.1, because p orbitals are perpendicular to each other.
Experimental evidence shows that the bond angle is 104.5°, not 90°. The prediction of the valence bond theory model does not
match the real-world observations of a water molecule; a different model is needed.

Figure 8.2.1 : The hypothetical overlap of two of the 2p orbitals on an oxygen atom (red) with the 1s orbitals of two hydrogen
atoms (blue) would produce a bond angle of 90°. This is not consistent with experimental evidence.1
Two peanut-shaped orbitals lie perpendicular to one another. They overlap with spherical orbitals to the left and top of the diagram.
Quantum-mechanical calculations suggest why the observed bond angles in H2O differ from those predicted by the overlap of the
1s orbital of the hydrogen atoms with the 2p orbitals of the oxygen atom. The mathematical expression known as the wave
function, ψ, contains information about each orbital and the wavelike properties of electrons in an isolated atom. When atoms are
bound together in a molecule, the wave functions combine to produce new mathematical descriptions that have different shapes.
This process of combining the wave functions for atomic orbitals is called hybridization and is mathematically accomplished by the
linear combination of atomic orbitals, LCAO, (a technique that we will encounter again later). The new orbitals that result are
called hybrid orbitals. The valence orbitals in an isolated oxygen atom are a 2s orbital and three 2p orbitals. The valence orbitals in
an oxygen atom in a water molecule differ; they consist of four equivalent hybrid orbitals that point approximately toward the
corners of a tetrahedron (Figure 8.2.2). Consequently, the overlap of the O and H orbitals should result in a tetrahedral bond angle
(109.5°). The observed angle of 104.5° is experimental evidence for which quantum-mechanical calculations give a useful
explanation: Valence bond theory must include a hybridization component to give accurate predictions.

Figure 8.2.2 : (a) A water molecule has four regions of electron density, so VSEPR theory predicts a tetrahedral arrangement of
hybrid orbitals. (b) Two of the hybrid orbitals on oxygen contain lone pairs, and the other two overlap with the 1s orbitals of
hydrogen atoms to form the O–H bonds in H2O. This description is more consistent with the experimental structure.
Two diagrams are shown and labeled “a” and “b.” Diagram a shows two peanut-shaped orbitals lying in a tetrahedral arrangement
around the letter “O.” Diagram b shows the same two orbitals, but they now overlap to the top and to the left with two spherical
orbitals, each labeled “H.” A pair of electrons occupies each lobe of the peanut-shaped orbitals.

Access for free at OpenStax 8.2.1 https://chem.libretexts.org/@go/page/38193


The following ideas are important in understanding hybridization:
1. Hybrid orbitals do not exist in isolated atoms. They are formed only in covalently bonded atoms.
2. Hybrid orbitals have shapes and orientations that are very different from those of the atomic orbitals in isolated atoms.
3. A set of hybrid orbitals is generated by combining atomic orbitals. The number of hybrid orbitals in a set is equal to the number
of atomic orbitals that were combined to produce the set.
4. All orbitals in a set of hybrid orbitals are equivalent in shape and energy.
5. The type of hybrid orbitals formed in a bonded atom depends on its electron-pair geometry as predicted by the VSEPR theory.
6. Hybrid orbitals overlap to form σ bonds. Unhybridized orbitals overlap to form π bonds.
In the following sections, we shall discuss the common types of hybrid orbitals.

8.2.1: sp Hybridization
The beryllium atom in a gaseous BeCl2 molecule is an example of a central atom with no lone pairs of electrons in a linear
arrangement of three atoms. There are two regions of valence electron density in the BeCl2 molecule that correspond to the two
covalent Be–Cl bonds. To accommodate these two electron domains, two of the Be atom’s four valence orbitals will mix to yield
two hybrid orbitals. This hybridization process involves mixing of the valence s orbital with one of the valence p orbitals to yield
two equivalent sp hybrid orbitals that are oriented in a linear geometry (Figure 8.2.3). In this figure, the set of sp orbitals appears
similar in shape to the original p orbital, but there is an important difference. The number of atomic orbitals combined always
equals the number of hybrid orbitals formed. The p orbital is one orbital that can hold up to two electrons. The sp set is two
equivalent orbitals that point 180° from each other. The two electrons that were originally in the s orbital are now distributed to the
two sp orbitals, which are half filled. In gaseous BeCl2, these half-filled hybrid orbitals will overlap with orbitals from the chlorine
atoms to form two identical σ bonds.

Figure 8.2.3 : Hybridization of an s orbital (blue) and a p orbital (red) of the same atom produces two sp hybrid orbitals (yellow).
Each hybrid orbital is oriented primarily in just one direction. Note that each sp orbital contains one lobe that is significantly larger
than the other. The set of two sp orbitals are oriented at 180°, which is consistent with the geometry for two domains.
A series of three diagrams connected by a right-facing arrow that is labeled, “Hybridization,” and a downward-facing arrow
labeled, “Gives a linear arrangement,” are shown. The first diagram shows a blue spherical orbital and a red, peanut-shaped orbital,
each placed on an X, Y, Z axis system. The second diagram shows the same two orbitals, but they are now purple and have one
enlarged lobe and one smaller lobe. Each lies along the x-axis in the drawing. The third diagram shows the same two orbitals, but
their smaller lobes now overlap along the x-axis while their larger lobes are located at and labeled as “180 degrees” from one
another.
We illustrate the electronic differences in an isolated Be atom and in the bonded Be atom in the orbital energy-level diagram in
Figure 8.2.4. These diagrams represent each orbital by a horizontal line (indicating its energy) and each electron by an arrow.
Energy increases toward the top of the diagram. We use one upward arrow to indicate one electron in an orbital and two arrows (up
and down) to indicate two electrons of opposite spin.

Access for free at OpenStax 8.2.2 https://chem.libretexts.org/@go/page/38193


Figure 8.2.4 : This orbital energy-level diagram shows the sp hybridized orbitals on Be in the linear BeCl2 molecule. Each of the
two sp hybrid orbitals holds one electron and is thus half filled and available for bonding via overlap with a Cl 3p orbital.
A diagram is shown in two parts, connected by a right facing arrow labeled, “Hybridization.” The left diagram shows an up-facing
arrow labeled, “E.” To the lower right of the arrow is a short, horizontal line labeled, “2 s,” that has two vertical half-arrows facing
up and down on it. To the upper right of the arrow are a series of three short, horizontal lines labeled, “2 p.” Above these two sets
of lines is the phrase, “Orbitals in an isolated B e atom.” The right side of the diagram shows two short, horizontal lines placed
halfway up the space and each labeled, “s p.” An upward-facing half arrow is drawn vertically on each line. Above these lines are
two other short, horizontal lines, each labeled, “2 p.” Above these two sets of lines is the phrase, “Orbitals in the s p hybridized B e
in B e C l subscript 2.”

When atomic orbitals hybridize, the valence electrons occupy the newly created orbitals. The Be atom had two valence electrons,
so each of the sp orbitals gets one of these electrons. Each of these electrons pairs up with the unpaired electron on a chlorine atom
when a hybrid orbital and a chlorine orbital overlap during the formation of the Be–Cl bonds. Any central atom surrounded by just
two regions of valence electron density in a molecule will exhibit sp hybridization. Other examples include the mercury atom in the
linear HgCl2 molecule, the zinc atom in Zn(CH3)2, which contains a linear C–Zn–C arrangement, and the carbon atoms in HCCH
and CO2.

8.2.2: sp2 Hybridization


The valence orbitals of a central atom surrounded by three regions of electron density consist of a set of three sp2 hybrid orbitals
and one unhybridized p orbital. This arrangement results from sp2 hybridization, the mixing of one s orbital and two p orbitals to
produce three identical hybrid orbitals oriented in a trigonal planar geometry (Figure 8.2.5).

Access for free at OpenStax 8.2.3 https://chem.libretexts.org/@go/page/38193


Figure 8.2.5 : The hybridization of an s orbital (blue) and two p orbitals (red) produces three equivalent sp2 hybridized orbitals
(yellow) oriented at 120° with respect to each other. The remaining unhybridized p orbital is not shown here, but is located along
the z axis.
A series of three diagrams connected by a right-facing arrow that is labeled, “Hybridization,” and a downward-facing arrow
labeled, “Gives a trigonal planar arrangement,” are shown. The first diagram shows a blue spherical orbital and two red, peanut-
shaped orbitals, each placed on an X, Y, Z axis system. The two red orbitals are located on the x and z axes, respectively. The
second diagram shows the same three orbitals, but they are now purple and have one enlarged lobe and one smaller lobe. Each lies
in a different axis in the drawing. The third diagram shows the same three orbitals, but their smaller lobes now overlap while their
larger lobes are located at and labeled as “120 degrees” from one another.
Although quantum mechanics yields the “plump” orbital lobes as depicted in Figure 8.2.5, sometimes for clarity these orbitals are
drawn thinner and without the minor lobes, as in Figure 8.2.6, to avoid obscuring other features of a given illustration. We will use
these “thinner” representations whenever the true view is too crowded to easily visualize.

Figure 8.2.6 : This alternate way of drawing the trigonal planar sp2 hybrid orbitals is sometimes used in more crowded figures.
Three balloon-like orbitals are shown, and connect together near their narrower ends in one plane. The angle between a pair of
lobes is labeled, “120 degrees.”
The observed structure of the borane molecule, BH3, suggests sp2 hybridization for boron in this compound. The molecule is
trigonal planar, and the boron atom is involved in three bonds to hydrogen atoms ( Figure 8.2.7).

Figure 8.2.7 : BH3 is an electron-deficient molecule with a trigonal planar structure.


A boron atom is shown connected to three hydrogen atoms, which are arranged around it like a pyramid. The angle from one line
connecting the boron atom to a hydrogen atom to another line connecting the boron atom to a hydrogen atom is labeled, “120
degrees.”
We can illustrate the comparison of orbitals and electron distribution in an isolated boron atom and in the bonded atom in BH3 as
shown in the orbital energy level diagram in Figure 8.2.8. We redistribute the three valence electrons of the boron atom in the three

Access for free at OpenStax 8.2.4 https://chem.libretexts.org/@go/page/38193


sp2 hybrid orbitals, and each boron electron pairs with a hydrogen electron when B–H bonds form.

Figure 8.2.8 : In an isolated B atom, there are one 2s and three 2p valence orbitals. When boron is in a molecule with three regions
of electron density, three of the orbitals hybridize and create a set of three sp2 orbitals and one unhybridized 2p orbital. The three
half-filled hybrid orbitals each overlap with an orbital from a hydrogen atom to form three σ bonds in BH3.
A diagram is shown in two parts, connected by a right facing arrow labeled, “Hybridization.” The left diagram shows an up-facing
arrow labeled “E.” To the lower right of the arrow is a short, horizontal line labeled, “2 s,” that has two vertical half-arrows facing
up and down on it. To the upper right of the arrow are a series of three short, horizontal lines labeled, “2 p.” Above both sets of
these lines is the phrase, “Orbitals in an isolated B atom.” One of the lines has a vertical, up-facing arrow drawn on it. The right
side of the diagram shows three short, horizontal lines placed halfway up the space and each labeled, “s p superscript 2.” An
upward-facing half arrow is drawn vertically on each line. Above these lines is one other short, horizontal line, labeled, “2 p.”
Above both sets of lines is the phrase, “Orbitals in the s p superscript 2 hybridized B atom in B H subscript 3.”
Any central atom surrounded by three regions of electron density will exhibit sp2 hybridization. This includes molecules with a
lone pair on the central atom, such as ClNO (Figure 8.2.9), or molecules with two single bonds and a double bond connected to the
central atom, as in formaldehyde, CH2O, and ethene, H2CCH2.

Figure 8.2.9 : The central atom(s) in each of the structures shown contain three regions of electron density and are sp2 hybridized.
As we know from the discussion of VSEPR theory, a region of electron density contains all of the electrons that point in one
direction. A lone pair, an unpaired electron, a single bond, or a multiple bond would each count as one region of electron density.
Three Lewis structures are shown. The left-hand structure shows a chlorine atom surrounded by three lone pairs of electrons single
bonded to a nitrogen atom with one lone pair of electrons and double bonded to an oxygen atom with two lone pairs of electrons.
The middle structure shows a carbon atom single bonded to two hydrogen atoms and double bonded to an oxygen atom that has
two lone pairs of electrons. The right-hand structure shows two carbon atoms, double bonded to one another and each single
bonded to two hydrogen atoms.

8.2.3: sp3 Hybridization


The valence orbitals of an atom surrounded by a tetrahedral arrangement of bonding pairs and lone pairs consist of a set of four sp3
hybrid orbitals. The hybrids result from the mixing of one s orbital and all three p orbitals that produces four identical sp3 hybrid
orbitals (Figure 8.2.10). Each of these hybrid orbitals points toward a different corner of a tetrahedron.

Access for free at OpenStax 8.2.5 https://chem.libretexts.org/@go/page/38193


Figure 8.2.10 : The hybridization of an s orbital (blue) and three p orbitals (red) produces four equivalent sp3 hybridized orbitals
(yellow) oriented at 109.5° with respect to each other.
A series of three diagrams connected by a right-facing arrow that is labeled, “Hybridization,” and a downward-facing arrow
labeled, “Gives a tetrahedral arrangement,” are shown. The first diagram shows a blue spherical orbital and three red, peanut-
shaped orbitals, each placed on an x, y, z axis system. The three red orbitals are located on the x , y and z axes, respectively. The
second diagram shows the same four orbitals, but they are now purple and have one enlarged lobe and one smaller lobe. Each lies
in a different axis in the drawing. The third diagram shows the same four orbitals, but their smaller lobes now overlap to form a
tetrahedral structure.
A molecule of methane, CH4, consists of a carbon atom surrounded by four hydrogen atoms at the corners of a tetrahedron. The
carbon atom in methane exhibits sp3 hybridization. We illustrate the orbitals and electron distribution in an isolated carbon atom
and in the bonded atom in CH4 in Figure 8.2.11. The four valence electrons of the carbon atom are distributed equally in the hybrid
orbitals, and each carbon electron pairs with a hydrogen electron when the C–H bonds form.

Access for free at OpenStax 8.2.6 https://chem.libretexts.org/@go/page/38193


Figure 8.2.11 : The four valence atomic orbitals from an isolated carbon atom all hybridize when the carbon bonds in a molecule
like CH4 with four regions of electron density. This creates four equivalent sp3 hybridized orbitals. Overlap of each of the hybrid
orbitals with a hydrogen orbital creates a C–H σ bond.
A diagram is shown in two parts, connected by a right facing arrow labeled, “Hybridization.” The left diagram shows an up-facing
arrow labeled “E.” To the lower right of the arrow is a short, horizontal line labeled, “2 s,” that has two vertical half-arrows facing
up and down on it. To the upper right of the arrow are a series of three short, horizontal lines labeled, “2 p.” Two of the lines have a
vertical, up-facing arrow drawn on them. Above both sets of lines is the phrase, “Orbitals in an isolated C atom.” The right side of
the diagram shows four short, horizontal lines placed halfway up the space and each labeled, “s p superscript 3.” An upward-facing
half arrow is drawn vertically on each line. Above these lines is the phrase, “Orbitals in the s p superscript 3 hybridized C atom in
C H subscript 4.”
In a methane molecule, the 1s orbital of each of the four hydrogen atoms overlaps with one of the four sp3 orbitals of the carbon
atom to form a sigma (σ) bond. This results in the formation of four strong, equivalent covalent bonds between the carbon atom and
each of the hydrogen atoms to produce the methane molecule, CH4.
The structure of ethane, C2H6, is similar to that of methane in that each carbon in ethane has four neighboring atoms arranged at the
corners of a tetrahedron—three hydrogen atoms and one carbon atom (Figure 8.2.10). However, in ethane an sp3 orbital of one
carbon atom overlaps end to end with an sp3 orbital of a second carbon atom to form a σ bond between the two carbon atoms. Each
of the remaining sp3 hybrid orbitals overlaps with an s orbital of a hydrogen atom to form carbon–hydrogen σ bonds. The structure
and overall outline of the bonding orbitals of ethane are shown in Figure 8.2.12. The orientation of the two CH3 groups is not fixed
relative to each other. Experimental evidence shows that rotation around σ bonds occurs easily.

Figure 8.2.12 : (a) In the ethane molecule, C2H6, each carbon has four sp3 orbitals. (b) These four orbitals overlap to form seven σ
bonds.
Two diagrams are shown and labeled “a” and “b.” Diagram a shows two carbon atoms, each surrounded by their four s p subscript
three hybridized orbitals in a three dimensional arrangement. Each of the orbitals is shown overlapping with a spherical hydrogen
atom. Diagram b shows the same general arrangement, but the hydrogen atoms are just represented by an, “H” and their spherical
orbitals are not shown.
An sp3 hybrid orbital can also hold a lone pair of electrons. For example, the nitrogen atom in ammonia is surrounded by three
bonding pairs and a lone pair of electrons directed to the four corners of a tetrahedron. The nitrogen atom is sp3 hybridized with
one hybrid orbital occupied by the lone pair.
The molecular structure of water is consistent with a tetrahedral arrangement of two lone pairs and two bonding pairs of electrons.
Thus we say that the oxygen atom is sp3 hybridized, with two of the hybrid orbitals occupied by lone pairs and two by bonding
pairs. Since lone pairs occupy more space than bonding pairs, structures that contain lone pairs have bond angles slightly distorted
from the ideal. Perfect tetrahedra have angles of 109.5°, but the observed angles in ammonia (107.3°) and water (104.5°) are
slightly smaller. Other examples of sp3 hybridization include CCl4, PCl3, and NCl3.

Access for free at OpenStax 8.2.7 https://chem.libretexts.org/@go/page/38193


8.2.4: sp3d and sp3d2 Hybridization
To describe the five bonding orbitals in a trigonal bipyramidal arrangement, we must use five of the valence shell atomic orbitals
(the s orbital, the three p orbitals, and one of the d orbitals), which gives five sp3d hybrid orbitals. With an octahedral arrangement
of six hybrid orbitals, we must use six valence shell atomic orbitals (the s orbital, the three p orbitals, and two of the d orbitals in its
valence shell), which gives six sp3d2 hybrid orbitals. These hybridizations are only possible for atoms that have d orbitals in their
valence subshells (that is, not those in the first or second period).

Figure 8.2.13 : The three compounds pictured exhibit sp3d hybridization in the central atom and a trigonal bipyramid form. SF4 and
C lF
4
+
have one lone pair of electrons on the central atom, and ClF3 has two lone pairs giving it the T-shape shown.
Three Lewis structures are shown along with designations of molecular shape. The left image shows a sulfur atom singly bonded to
four fluorine atoms. The sulfur atom has one lone pair of electrons while each fluorine has three. Two fluorine atoms are drawn
vertically up and down from the sulfur while the other two are shown going into and out of the page. The second structure shows
one chlorine atom singly bonded to three fluorine atoms. The chlorine has two lone pairs of electrons while each fluorine has three.
Two fluorine atoms are drawn vertically up and down from the sulfur while the other is shown horizontally. The right structure
shows a chlorine atom singly bonded to four fluorine atoms. The chlorine atom has one lone pair of electrons and a superscript plus
sign, while each fluorine has three lone pairs of electrons. Two fluorine atoms are drawn vertically up and down from the sulfur
while the other two are shown going into and out of the page.
In a molecule of phosphorus pentachloride, PCl5, there are five P–Cl bonds (thus five pairs of valence electrons around the
phosphorus atom) directed toward the corners of a trigonal bipyramid. We use the 3s orbital, the three 3p orbitals, and one of the 3d
orbitals to form the set of five sp3d hybrid orbitals (Figure 8.2.13) that are involved in the P–Cl bonds. Other atoms that exhibit
sp3d hybridization include the sulfur atom in SF4 and the chlorine atoms in ClF3 and in ClF . (The electrons on fluorine atoms are
+
4

omitted for clarity.)

Figure 8.2.14 : (a) The five regions of electron density around phosphorus in PCl5 require five hybrid sp3d orbitals. (b) These
orbitals combine to form a trigonal bipyramidal structure with each large lobe of the hybrid orbital pointing at a vertex. As before,
there are also small lobes pointing in the opposite direction for each orbital (not shown for clarity).
Two images are shown and labeled “a” and “b.” Image a depicts a ball-and-stick model in a trigonal bipyramidal arrangement.
Image b depicts the hybrid orbitals in the same arrangement and each is labeled, “s p superscript three d.”
The sulfur atom in sulfur hexafluoride, SF6, exhibits sp3d2 hybridization. A molecule of sulfur hexafluoride has six bonding pairs
of electrons connecting six fluorine atoms to a single sulfur atom. There are no lone pairs of electrons on the central atom. To bond
six fluorine atoms, the 3s orbital, the three 3p orbitals, and two of the 3d orbitals form six equivalent sp3d2 hybrid orbitals, each
directed toward a different corner of an octahedron. Other atoms that exhibit sp3d2 hybridization include the phosphorus atom in
PCl , the iodine atom in the interhalogens IF , IF5, ICl , IF and the xenon atom in XeF4.
− + − −
6 6 4 4

Access for free at OpenStax 8.2.8 https://chem.libretexts.org/@go/page/38193


Figure 8.2.15 : (a) Sulfur hexafluoride, SF6, has an octahedral structure that requires sp3d2 hybridization. (b) The six sp3d2 orbitals
form an octahedral structure around sulfur. Again, the minor lobe of each orbital is not shown for clarity.
Two images are shown and labeled “a” and “b.” Image a depicts a ball-and-stick model in an octahedral arrangement. Image b
depicts the hybrid orbitals in the same arrangement and each is labeled, “s p superscript three d superscript two.”

8.2.5: Assignment of Hybrid Orbitals to Central Atoms


The hybridization of an atom is determined based on the number of regions of electron density that surround it. The geometrical
arrangements characteristic of the various sets of hybrid orbitals are shown in Figure 8.2.16. These arrangements are identical to
those of the electron-pair geometries predicted by VSEPR theory. VSEPR theory predicts the shapes of molecules, and hybrid
orbital theory provides an explanation for how those shapes are formed. To find the hybridization of a central atom, we can use the
following guidelines:
1. Determine the Lewis structure of the molecule.
2. Determine the number of regions of electron density around an atom using VSEPR theory, in which single bonds, multiple
bonds, radicals, and lone pairs each count as one region.
3. Assign the set of hybridized orbitals from Figure 8.2.16 that corresponds to this geometry.

Access for free at OpenStax 8.2.9 https://chem.libretexts.org/@go/page/38193


Figure 8.2.16 : The shapes of hybridized orbital sets are consistent with the electron-pair geometries. For example, an atom
surrounded by three regions of electron density is sp2 hybridized, and the three sp2 orbitals are arranged in a trigonal planar fashion.
A table is shown that is composed of five columns and six rows. The header row contains the phrases, “Regions of electron
density,” “Arrangement,” (which has two columns below it), and “Hybridization,” (which has two columns below it). The first
column contains the numbers “2,” “3,” “4,” “5,” and “6.” The second column contains images of a line, a triangle, a three sided
pyramid, a trigonal bipyramid, and an eight-faced ocatahedron. The third column contains the terms, “Linear,” “Trigonal planar,”
“Tetrahedral,” “Trigonal bipyramidal,” and “Octahedral.” The fourth column contains the terms “s p,” “s p superscript 2,” “s p
superscript 3,” “s p superscript 3 d,” and “s p superscript 3 d superscript 2.” The last column contains drawings of the molecules
beginning with a peanut-shaped structure marked with an angle of “180 degrees.” The second structure is made up of three equal-
sized, rounded structures connected at one point with an angle of “120 degrees,” while the third structure is a three-dimensional
arrangement of four equal-sized, rounded structures labeled as “109.5 degrees.” The fourth structure is made up of five equal-sized,
rounded structures connected at “120 and 90 degrees,” while the fifth structure has six equal-sized, rounded structures connected at
“90 degrees.”

It is important to remember that hybridization was devised to rationalize experimentally


observed molecular geometries, not the other way around.
The model works well for molecules containing small central atoms, in which the valence electron pairs are close together in space.
However, for larger central atoms, the valence-shell electron pairs are farther from the nucleus, and there are fewer repulsions.
Their compounds exhibit structures that are often not consistent with VSEPR theory, and hybridized orbitals are not necessary to
explain the observed data.

Access for free at OpenStax 8.2.10 https://chem.libretexts.org/@go/page/38193


Three Lewis structures are shown. The left structure shows an oxygen atom with two lone pairs of electrons single bonded to two
hydrogen atoms. The middle structure is made up of a sulfur atom with two lone pairs of electrons single bonded to two hydrogen
atoms. The right structure is made up of a tellurium atom with two lone pairs of electrons single bonded to two hydrogen atoms.
From left to right, the bond angles of each molecule decrease.
For example, we have discussed the H–O–H bond angle in H2O, 104.5°, which is more consistent with sp3 hybrid orbitals (109.5°)
on the central atom than with 2p orbitals (90°). Sulfur is in the same group as oxygen, and H2S has a similar Lewis structure.
However, it has a much smaller bond angle (92.1°), which indicates much less hybridization on sulfur than oxygen. Continuing
down the group, tellurium is even larger than sulfur, and for H2Te, the observed bond angle (90°) is consistent with overlap of the
5p orbitals, without invoking hybridization. We invoke hybridization where it is necessary to explain the observed structures.

 Example 8.2.1: Assigning Hybridization

Ammonium sulfate is important as a fertilizer. What is the hybridization of the sulfur atom in the sulfate ion, SO 2−
4
?

Solution
The Lewis structure of sulfate shows there are four regions of electron density.

Figure 8.2.16 ).
A structure is shown in which a sulfur atom is bonded to four oxygen atoms in a tetrahedral arrangement. Two of the oxygen
atoms have a negative charge.

 Exercise 8.2.1
What is the hybridization of the selenium atom in SeF4?

A Lewis structure is shown in which four fluorine atoms are each attached to one sulfur atom. Two of the attached fluorine
atoms are vertically attached up and down, while two are attached into and out of the page to the right. The sulfur also has one
lone pair of electrons attached to the left of the structure.

Answer
The selenium atom is sp3d hybridized.

Access for free at OpenStax 8.2.11 https://chem.libretexts.org/@go/page/38193


 Example 8.2.2: Assigning Hybridization

Urea, NH2C(O)NH2, is sometimes used as a source of nitrogen in fertilizers. What is the hybridization of each nitrogen and
carbon atom in urea?

Solution
The Lewis structure of urea is

Figure 8.2.16 ). This is the hybridization of the nitrogen atoms in urea.


A Lewis structure is shown in which a carbon atom is double bonded to an oxygen atom that has two lone pairs of electrons.
The carbon atom forms single bonds to two nitrogen atoms. Each nitrogen is single bonded to two hydrogen atoms, and each
nitrogen atoms has one lone pair of electrons.
The carbon atom is surrounded by three regions of electron density, positioned in a trigonal planar arrangement. The
hybridization in a trigonal planar electron pair geometry is sp2 (Figure 8.2.16), which is the hybridization of the carbon atom in
urea.

 Exercise 8.2.1

Acetic acid, H3CC(O)OH, is the molecule that gives vinegar its odor and sour taste. What is the hybridization of the two
carbon atoms in acetic acid?

A Lewis structure is shown in which a carbon atom is double bonded to an oxygen atom that has two lone pairs of electrons
and single bonded to another oxygen atom that is single boned to a hydrogen atom. This second oxygen atom has two lone
pairs of electrons. The carbon is also single bonded to a carbon atom that is single bonded to three hydrogen atoms.

Answer
H3C, sp3; C(O)OH, sp2

Summary
We can use hybrid orbitals, which are mathematical combinations of some or all of the valence atomic orbitals, to describe the
electron density around covalently bonded atoms. These hybrid orbitals either form sigma (σ) bonds directed toward other atoms of
the molecule or contain lone pairs of electrons. We can determine the type of hybridization around a central atom from the
geometry of the regions of electron density about it. Two such regions imply sp hybridization; three, sp2 hybridization; four, sp3
hybridization; five, sp3d hybridization; and six, sp3d2 hybridization. Pi (π) bonds are formed from unhybridized atomic orbitals (p
or d orbitals).

Access for free at OpenStax 8.2.12 https://chem.libretexts.org/@go/page/38193


8.2.5.1: Footnotes
1. Note that orbitals may sometimes be drawn in an elongated “balloon” shape rather than in a more realistic “plump” shape in
order to make the geometry easier to visualize.

8.2.5.2: Glossary

hybrid orbital
orbital created by combining atomic orbitals on a central atom

hybridization
model that describes the changes in the atomic orbitals of an atom when it forms a covalent compound

sp hybrid orbital
one of a set of two orbitals with a linear arrangement that results from combining one s and one p orbital

sp2 hybrid orbital


one of a set of three orbitals with a trigonal planar arrangement that results from combining one s and two p orbitals

sp3 hybrid orbital


one of a set of four orbitals with a tetrahedral arrangement that results from combining one s and three p orbitals

sp3d hybrid orbital


one of a set of five orbitals with a trigonal bipyramidal arrangement that results from combining one s, three p, and one d orbital

sp3d2 hybrid orbital


one of a set of six orbitals with an octahedral arrangement that results from combining one s, three p, and two d orbitals

This page titled 8.2: Hybrid Atomic Orbitals is shared under a CC BY 4.0 license and was authored, remixed, and/or curated by OpenStax via
source content that was edited to the style and standards of the LibreTexts platform; a detailed edit history is available upon request.

Access for free at OpenStax 8.2.13 https://chem.libretexts.org/@go/page/38193


8.3: Multiple Bonds
 Learning Objectives
Describe multiple covalent bonding in terms of atomic orbital overlap
Relate the concept of resonance to π-bonding and electron delocalization

The hybrid orbital model appears to account well for the geometry of molecules involving single covalent bonds. Is it also capable
of describing molecules containing double and triple bonds? We have already discussed that multiple bonds consist of σ and π
bonds. Next we can consider how we visualize these components and how they relate to hybrid orbitals. The Lewis structure of
ethene, C2H4, shows us that each carbon atom is surrounded by one other carbon atom and two hydrogen atoms.

Figure 8.3.2 ).
A Lewis structure is shown in which two carbon atoms are bonded together by a double bond. Each carbon atom is bonded to two
hydrogen atoms by a single bond.

Figure 8.3.1 : In ethene, each carbon atom is sp2 hybridized, and the sp2 orbitals and the p orbital are singly occupied. The hybrid
orbitals overlap to form σ bonds, while the p orbitals on each carbon atom overlap to form a π bond.
A diagram is shown in two parts, connected by a right facing arrow labeled, “Hybridization.” The left diagram shows an up-facing
arrow labeled, “E.” To the lower right of the arrow is a short, horizontal line labeled, “2 s,” that has two vertical half-arrows facing
up and down on it. To the upper right of the arrow are a series of three short, horizontal lines labeled, “2 p.” Above both sets of
lines is the phrase, “Orbitals in an isolated C atom.” Two of the lines have vertical, up-facing arrows drawn on them. The right side
of the diagram shows three short, horizontal lines placed halfway up the space and each labeled, “s p superscript 2.” An upward-
facing half arrow is drawn vertically on each line. Above these lines is one other short, horizontal line, labeled, “p.” Above both
sets of lines is the phrase, “Orbitals in the s p superscript 2 hybridized C atom in C subscript 2 H subscript 4.”
The π bond in the C=C double bond results from the overlap of the third (remaining) 2p orbital on each carbon atom that is not
involved in hybridization. This unhybridized p orbital (lobes shown in red and blue in Figure 8.3.2) is perpendicular to the plane of
the sp2 hybrid orbitals. Thus the unhybridized 2p orbitals overlap in a side-by-side fashion, above and below the internuclear axis
and form a π bond.

Access for free at OpenStax 8.3.1 https://chem.libretexts.org/@go/page/38194


Figure 8.3.2 : In the ethene molecule, C2H4, there are (a) five σ bonds. One C–C σ bond results from overlap of sp2 hybrid orbitals
on the carbon atom with one sp2 hybrid orbital on the other carbon atom. Four C–H bonds result from the overlap between the C
atoms' sp2 orbitals with s orbitals on the hydrogen atoms. (b) The π bond is formed by the side-by-side overlap of the two
unhybridized p orbitals in the two carbon atoms. The two lobes of the π bond are above and below the plane of the σ system.
Two diagrams are shown labeled, “a” and “b.” Diagram a shows two carbon atoms with three purple balloon-like orbitals arranged
in a plane around them and two red balloon-like orbitals arranged vertically and perpendicularly to the plane. There is an overlap of
two of the purple orbitals in between the two carbon atoms, and the other four purple orbitals that face the outside of the molecule
are shown interacting with spherical blue orbitals from four hydrogen atoms. Diagram b depicts a similar image to diagram a, but
the red, vertical orbitals are interacting above and below the plane of the molecule to form two areas labeled, “One pi bond.”
In an ethene molecule, the four hydrogen atoms and the two carbon atoms are all in the same plane. If the two planes of sp2 hybrid
orbitals tilted relative to each other, the p orbitals would not be oriented to overlap efficiently to create the π bond. The planar
configuration for the ethene molecule occurs because it is the most stable bonding arrangement. This is a significant difference
between σ and π bonds; rotation around single (σ) bonds occurs easily because the end-to-end orbital overlap does not depend on
the relative orientation of the orbitals on each atom in the bond. In other words, rotation around the internuclear axis does not
change the extent to which the σ bonding orbitals overlap because the bonding electron density is symmetric about the axis.
Rotation about the internuclear axis is much more difficult for multiple bonds; however, this would drastically alter the off-axis
overlap of the π bonding orbitals, essentially breaking the π bond.

A diagram of a carbon atom with two balloon-like purple orbitals labeled, “sp” arranged in a linear fashion around it is shown. Four
red balloon-like orbitals are aligned in pairs in the y and z axes around the carbon and are labeled, “unhybridized p orbital,” and,
“Second unhybridized p orbital.”
Figure 8.3.3 : Diagram of the two linear sp hybrid orbitals of a carbon atom, which lie in a straight line, and the two unhybridized p
orbitals at perpendicular angles.
In molecules with sp hybrid orbitals, two unhybridized p orbitals remain on the atom (Figure 8.3.3). We find this situation in
acetylene, H−C≡C−H, which is a linear molecule. The sp hybrid orbitals of the two carbon atoms overlap end to end to form a σ
bond between the carbon atoms (Figure 8.3.4). The remaining sp orbitals form σ bonds with hydrogen atoms. The two
unhybridized p orbitals per carbon are positioned such that they overlap side by side and, hence, form two π bonds. The two carbon
atoms of acetylene are thus bound together by one σ bond and two π bonds, giving a triple bond.

Access for free at OpenStax 8.3.2 https://chem.libretexts.org/@go/page/38194


Figure 8.3.4 : (a) In the acetylene molecule, C2H2, there are two C–H σ bonds and a C≡C triple bond involving one C–C σ bond and
two C–C π bonds. The dashed lines, each connecting two lobes, indicate the side-by-side overlap of the four unhybridized p
orbitals. (b) This shows the overall outline of the bonds in C2H2. The two lobes of each of the π bonds are positioned across from
each other around the line of the C–C σ bond.
Two diagrams are shown and labeled, “a” and “b.” Diagram a shows two carbon atoms with two purple balloon-like orbitals
arranged in a plane around each of them, and four red balloon-like orbitals arranged along the y and z axes perpendicular to the
plane of the molecule. There is an overlap of two of the purple orbitals in between the two carbon atoms. The other two purple
orbitals that face the outside of the molecule are shown interacting with spherical blue orbitals from two hydrogen atoms. Diagram
b depicts a similar image to diagram a, but the red, vertical orbitals are interacting above and below and to the front and back of the
plane of the molecule to form two areas labeled, “One pi bond,” and, “Second pi bond,” each respectively.
Hybridization involves only σ bonds, lone pairs of electrons, and single unpaired electrons (radicals). Structures that account for
these features describe the correct hybridization of the atoms. However, many structures also include resonance forms. Remember
that resonance forms occur when various arrangements of π bonds are possible. Since the arrangement of π bonds involves only the
unhybridized orbitals, resonance does not influence the assignment of hybridization.
For example, molecule benzene has two resonance forms (Figure 8.3.5). We can use either of these forms to determine that each of
the carbon atoms is bonded to three other atoms with no lone pairs, so the correct hybridization is sp2. The electrons in the
unhybridized p orbitals form π bonds. Neither resonance structure completely describes the electrons in the π bonds. They are not
located in one position or the other, but in reality are delocalized throughout the ring. Valence bond theory does not easily address
delocalization. Bonding in molecules with resonance forms is better described by molecular orbital theory.

A diagram is shown that is made up of two Lewis structures connected by a double ended arrow. The left image shows six carbon
atoms bonded together with alternating double and single bonds to form a six-sided ring. Each carbon is also bonded to a hydrogen
atom by a single bond. The right image shows the same structure, but the double and single bonds in between the carbon atoms
have changed positions.
Figure 8.3.5 : Each carbon atom in benzene, C6H6, is sp2 hybridized, independently of which resonance form is considered. The
electrons in the π bonds are not located in one set of p orbitals or the other, but rather delocalized throughout the molecule.

 Example 8.3.1: Assignment of Hybridization Involving Resonance

Some acid rain results from the reaction of sulfur dioxide with atmospheric water vapor, followed by the formation of sulfuric
acid. Sulfur dioxide, SO , is a major component of volcanic gases as well as a product of the combustion of sulfur-containing
2

coal. What is the hybridization of the S atom in SO ? 2

Solution
The resonance structures of SO are 2

Access for free at OpenStax 8.3.3 https://chem.libretexts.org/@go/page/38194


Two Lewis structures connected by a double-ended arrow are shown. The left structure shows a sulfur atom with one lone pair
of electrons and a positive sign which is single bonded on one side to an oxygen atom with three lone pairs of electrons and a
negative sign. The sulfur atom is double bonded on the other side to another oxygen atom with two lone pairs of electrons. The
right-hand structure is the same as the left except that the position of the double bonded oxygen atom is switched. In both
structures the attached oxygen atoms form an acute angle in terms of the sulfur atom.
The sulfur atom is surrounded by two bonds and one lone pair of electrons in either resonance structure. Therefore, the
electron-pair geometry is trigonal planar, and the hybridization of the sulfur atom is sp2.

 Exercise 8.3.1

Another acid in acid rain is nitric acid, HNO3, which is produced by the reaction of nitrogen dioxide, NO2, with atmospheric
water vapor. What is the hybridization of the nitrogen atom in NO2? (Note: the lone electron on nitrogen occupies a hybridized
orbital just as a lone pair would.)

Answer
sp2

Summary
Multiple bonds consist of a σ bond located along the axis between two atoms and one or two π bonds. The σ bonds are usually
formed by the overlap of hybridized atomic orbitals, while the π bonds are formed by the side-by-side overlap of unhybridized
orbitals. Resonance occurs when there are multiple unhybridized orbitals with the appropriate alignment to overlap, so the
placement of π bonds can vary.

This page titled 8.3: Multiple Bonds is shared under a CC BY 4.0 license and was authored, remixed, and/or curated by OpenStax via source
content that was edited to the style and standards of the LibreTexts platform; a detailed edit history is available upon request.

Access for free at OpenStax 8.3.4 https://chem.libretexts.org/@go/page/38194


8.4: Molecular Orbital Theory
 Learning Objectives
Outline the basic quantum-mechanical approach to deriving molecular orbitals from atomic orbitals
Describe traits of bonding and antibonding molecular orbitals
Calculate bond orders based on molecular electron configurations
Write molecular electron configurations for first- and second-row diatomic molecules
Relate these electron configurations to the molecules’ stabilities and magnetic properties

For almost every covalent molecule that exists, we can now draw the Lewis structure, predict the electron-pair geometry, predict
the molecular geometry, and come close to predicting bond angles. However, one of the most important molecules we know, the
oxygen molecule O2, presents a problem with respect to its Lewis structure. We would write the following Lewis structure for O2:

A Lewis structure is shown. It is made up of two oxygen atoms, each with two lone pairs of electrons, bonded together with a
double bond.
This electronic structure adheres to all the rules governing Lewis theory. There is an O=O double bond, and each oxygen atom has
eight electrons around it. However, this picture is at odds with the magnetic behavior of oxygen. By itself, O2 is not magnetic, but it
is attracted to magnetic fields. Thus, when we pour liquid oxygen past a strong magnet, it collects between the poles of the magnet
and defies gravity. Such attraction to a magnetic field is called paramagnetism, and it arises in molecules that have unpaired
electrons. And yet, the Lewis structure of O2 indicates that all electrons are paired. How do we account for this discrepancy?
Magnetic susceptibility measures the force experienced by a substance in a magnetic field. When we compare the weight of a
sample to the weight measured in a magnetic field (Figure 8.4.1), paramagnetic samples that are attracted to the magnet will appear
heavier because of the force exerted by the magnetic field. We can calculate the number of unpaired electrons based on the increase
in weight.

Figure 8.4.1 : A Gouy balance compares the mass of a sample in the presence of a magnetic field with the mass with the
electromagnet turned off to determine the number of unpaired electrons in a sample.
Experiments show that each O2 molecule has two unpaired electrons. The Lewis-structure model does not predict the presence of
these two unpaired electrons. Unlike oxygen, the apparent weight of most molecules decreases slightly in the presence of an
inhomogeneous magnetic field. Materials in which all of the electrons are paired are diamagnetic and weakly repel a magnetic
field. Paramagnetic and diamagnetic materials do not act as permanent magnets. Only in the presence of an applied magnetic field
do they demonstrate attraction or repulsion.

Access for free at OpenStax 8.4.1 https://chem.libretexts.org/@go/page/38195


levitating frog

Video 8.4.1 : Water, like most molecules, contains all paired electrons. Living things contain a large percentage of water, so they
demonstrate diamagnetic behavior. If you place a frog near a sufficiently large magnet, it will levitate. You can see videos of
diamagnetic floating frogs, strawberries, and more (levitating frog [www.youtube.com])
Molecular orbital theory (MO theory) provides an explanation of chemical bonding that accounts for the paramagnetism of the
oxygen molecule. It also explains the bonding in a number of other molecules, such as violations of the octet rule and more
molecules with more complicated bonding (beyond the scope of this text) that are difficult to describe with Lewis structures.
Additionally, it provides a model for describing the energies of electrons in a molecule and the probable location of these electrons.
Unlike valence bond theory, which uses hybrid orbitals that are assigned to one specific atom, MO theory uses the combination of
atomic orbitals to yield molecular orbitals that are delocalized over the entire molecule rather than being localized on its constituent
atoms. MO theory also helps us understand why some substances are electrical conductors, others are semiconductors, and still
others are insulators. Table 8.4.1 summarizes the main points of the two complementary bonding theories. Both theories provide
different, useful ways of describing molecular structure.
Table \(\PageIndex{1}\): Comparison of Bonding Theories
Valence Bond Theory Molecular Orbital Theory

considers bonds as localized between one pair of atoms considers electrons delocalized throughout the entire molecule

creates bonds from overlap of atomic orbitals (s, p, d…) and hybrid
combines atomic orbitals to form molecular orbitals (σ, σ*, π, π*)
orbitals (sp, sp2, sp3…)

creates bonding and antibonding interactions based on which orbitals


forms σ or π bonds
are filled

predicts molecular shape based on the number of regions of electron


predicts the arrangement of electrons in molecules
density

needs multiple structures to describe resonance

Molecular orbital theory describes the distribution of electrons in molecules in much the same way that the distribution of electrons
in atoms is described using atomic orbitals. Using quantum mechanics, the behavior of an electron in a molecule is still described
by a wave function, Ψ, analogous to the behavior in an atom. Just like electrons around isolated atoms, electrons around atoms in
molecules are limited to discrete (quantized) energies. The region of space in which a valence electron in a molecule is likely to be
found is called a molecular orbital (Ψ2). Like an atomic orbital, a molecular orbital is full when it contains two electrons with
opposite spin.
We will consider the molecular orbitals in molecules composed of two identical atoms (H2 or Cl2, for example). Such molecules
are called homonuclear diatomic molecules. In these diatomic molecules, several types of molecular orbitals occur.
The mathematical process of combining atomic orbitals to generate molecular orbitals is called the linear combination of atomic
orbitals (LCAO). The wave function describes the wavelike properties of an electron. Molecular orbitals are combinations of
atomic orbital wave functions. Combining waves can lead to constructive interference, in which peaks line up with peaks, or
destructive interference, in which peaks line up with troughs (Figure 8.4.2). In orbitals, the waves are three dimensional, and they
combine with in-phase waves producing regions with a higher probability of electron density and out-of-phase waves producing
nodes, or regions of no electron density.

Access for free at OpenStax 8.4.2 https://chem.libretexts.org/@go/page/38195


Figure 8.4.2 : (a) When in-phase waves combine, constructive interference produces a wave with greater amplitude. (b) When out-
of-phase waves combine, destructive interference produces a wave with less (or no) amplitude.
A pair of diagrams are shown and labeled, “a” and “b.” Diagram a shows two identical waves with two crests and two troughs.
They are drawn one above the other with a plus sign in between and an equal sign to the right. To the right of the equal sign is a
much taller wave with a same number of troughs and crests. Diagram b shows two waves with two crests and two troughs, but they
are mirror images of one another rotated over a horizontal axis. They are drawn one above the other with a plus sign in between
and an equal sign to the right. To the right of the equal sign is a flat line.
There are two types of molecular orbitals that can form from the overlap of two atomic s orbitals on adjacent atoms. The two types
are illustrated in Figure 8.4.3. The in-phase combination produces a lower energy σs molecular orbital (read as "sigma-s") in which
most of the electron density is directly between the nuclei. The out-of-phase addition (which can also be thought of as subtracting
the wave functions) produces a higher energy σ molecular orbital (read as "sigma-s-star") molecular orbital in which there is a

s

node between the nuclei. The asterisk signifies that the orbital is an antibonding orbital. Electrons in a σs orbital are attracted by
both nuclei at the same time and are more stable (of lower energy) than they would be in the isolated atoms. Adding electrons to
these orbitals creates a force that holds the two nuclei together, so we call these orbitals bonding orbitals. Electrons in the σ ∗
s

orbitals are located well away from the region between the two nuclei. The attractive force between the nuclei and these electrons
pulls the two nuclei apart. Hence, these orbitals are called antibonding orbitals. Electrons fill the lower-energy bonding orbital
before the higher-energy antibonding orbital, just as they fill lower-energy atomic orbitals before they fill higher-energy atomic
orbitals.

Figure 8.4.3 : Sigma (σ) and sigma-star (σ*) molecular orbitals are formed by the combination of two s atomic orbitals. The plus
(+) signs indicate the locations of nuclei.
A diagram is shown that depicts a vertical upward-facing arrow that lies to the left of all the other portions of the diagram and is
labeled, “E.” To the immediate right of the midpoint of the arrow are two circles each labeled with a positive sign, the letter S, and
the phrase, “Atomic orbitals.” These are followed by a right-facing horizontal arrow that points to the same two circles labeled with
plus signs, but they are now touching and are labeled, “Combine atomic orbitals.” Two right-facing arrows lead to the last portion
of the diagram, one facing upward and one facing downward. The upper arrow is labeled, “Subtract,” and points to two oblong
ovals labeled with plus signs, and the phrase, “Antibonding orbitals sigma subscript s superscript asterisk.” The lower arrow is
labeled, “Add,” and points to an elongated oval with two plus signs that is labeled, “Bonding orbital sigma subscript s.” The
heading over the last section of the diagram are the words, “Molecular orbitals.”
In p orbitals, the wave function gives rise to two lobes with opposite phases, analogous to how a two-dimensional wave has both
parts above and below the average. We indicate the phases by shading the orbital lobes different colors. When orbital lobes of the
same phase overlap, constructive wave interference increases the electron density. When regions of opposite phase overlap, the
destructive wave interference decreases electron density and creates nodes. When p orbitals overlap end to end, they create σ and
σ* orbitals (Figure 8.4.4). If two atoms are located along the x-axis in a Cartesian coordinate system, the two px orbitals overlap
end to end and form σpx (bonding) and σ (antibonding) (read as "sigma-p-x" and "sigma-p-x star," respectively). Just as with s-

px

orbital overlap, the asterisk indicates the orbital with a node between the nuclei, which is a higher-energy, antibonding orbital.

Access for free at OpenStax 8.4.3 https://chem.libretexts.org/@go/page/38195


Two horizontal rows of diagrams are shown. The upper diagram shows two equally-sized peanut-shaped orbitals with a plus sign in
between them connected to a merged orbital diagram by a right facing arrow. The merged diagram has a much larger oval at the
center and much smaller ovular orbitals on the edge. It is labeled, “sigma subscript p x.” The lower diagram shows two equally-
sized peanut-shaped orbitals with a plus sign in between them connected to a split orbital diagram by a right facing arrow. The split
diagram has a much larger oval at the outer ends and much smaller ovular orbitals on the inner edges. It is labeled, “sigma subscript
p x superscript asterisk”.
Figure 8.4.4 : Combining wave functions of two p atomic orbitals along the internuclear axis creates two molecular orbitals, σp and
σ .

p

The side-by-side overlap of two p orbitals gives rise to a pi (π) bonding molecular orbital and a \(π^*\) antibonding molecular
orbital, as shown in Figure 8.4.5. In valence bond theory, we describe π bonds as containing a nodal plane containing the
internuclear axis and perpendicular to the lobes of the p orbitals, with electron density on either side of the node. In molecular
orbital theory, we describe the π orbital by this same shape, and a π bond exists when this orbital contains electrons. Electrons in
this orbital interact with both nuclei and help hold the two atoms together, making it a bonding orbital. For the out-of-phase
combination, there are two nodal planes created, one along the internuclear axis and a perpendicular one between the nuclei.

Figure 8.4.5 : Side-by-side overlap of each two p orbitals results in the formation of two π molecular orbitals. Combining the out-
of-phase orbitals results in an antibonding molecular orbital with two nodes. One contains the internuclear axis, and one is
perpendicular to the axis. Combining the in-phase orbitals results in a bonding orbital. There is a node (blue plane) containing the
internuclear axis with the two lobes of the orbital located above and below this node.
Two horizontal rows of diagrams are shown. The upper and lower diagrams both begin with two vertical peanut-shaped orbitals
with a plus sign in between followed by a right-facing arrow. The upper diagram shows the same vertical peanut orbitals bending
slightly away from one another and separated by a dotted line. It is labeled, “pi subscript p superscript asterisk.” The lower diagram
shows the horizontal overlap of the two orbitals and is labeled, “pi subscript p.”
In the molecular orbitals of diatomic molecules, each atom also has two sets of p orbitals oriented side by side (py and pz), so these
four atomic orbitals combine pairwise to create two π orbitals and two π orbitals. The π and π orbitals are oriented at right

py

py

angles to the π and π orbitals. Except for their orientation, the πpy and πpz orbitals are identical and have the same energy; they
pz

pz

are degenerate orbitals. The π and π antibonding orbitals are also degenerate and identical except for their orientation. A total

py

pz

of six molecular orbitals results from the combination of the six atomic p orbitals in two atoms: σ and σ , π and π , π and
px

px py

py pz

π .

pz

Access for free at OpenStax 8.4.4 https://chem.libretexts.org/@go/page/38195


 Example 8.4.1: Molecular Orbitals

Predict what type (if any) of molecular orbital would result from adding the wave functions so each pair of orbitals shown
overlap. The orbitals are all similar in energy.

Three diagrams are shown and labeled “a,” “b,” and “c.” Diagram a shows two horizontal peanut-shaped orbitals laying side-
by-side. They are labeled, “3 p subscript x and 3 p subscript x.” Diagram b shows one vertical and one horizontal peanut-
shaped orbital which are at right angles to one another. They are labeled, “3 p subscript x and 3 p subscript y.” Diagram c
shows two vertical peanut-shaped orbitals laying side-by-side and labeled, “3 p subscript y and 3 p subscript y.”

Solution
a. This is an in-phase combination, resulting in a σ3p orbital
b. This will not result in a new orbital because the in-phase component (bottom) and out-of-phase component (top) cancel out.
Only orbitals with the correct alignment can combine.
c. This is an out-of-phase combination, resulting in a π orbital.

3p

 Exercise 8.4.1

Label the molecular orbital shown as σ or π, bonding or antibonding and indicate where the node occurs.

Two orbitals are shown lying end-to-end. Each has one enlarged and one small side. The small sides are facing one another

Answer
The orbital is located along the internuclear axis, so it is a σ orbital. There is a node bisecting the internuclear axis, so it is
an antibonding orbital.

Two orbitals are shown lying end-to-end. Each has one enlarged and one small side. The small sides are facing one another and are
separated by a vertical dotted line.

 Application: Computational Chemistry in Drug Design

While the descriptions of bonding described in this chapter involve many theoretical concepts, they also have many practical,
real-world applications. For example, drug design is an important field that uses our understanding of chemical bonding to
develop pharmaceuticals. This interdisciplinary area of study uses biology (understanding diseases and how they operate) to
identify specific targets, such as a binding site that is involved in a disease pathway. By modeling the structures of the binding
site and potential drugs, computational chemists can predict which structures can fit together and how effectively they will
bind (Figure 8.4.6). Thousands of potential candidates can be narrowed down to a few of the most promising candidates. These

Access for free at OpenStax 8.4.5 https://chem.libretexts.org/@go/page/38195


candidate molecules are then carefully tested to determine side effects, how effectively they can be transported through the
body, and other factors. Dozens of important new pharmaceuticals have been discovered with the aid of computational
chemistry, and new research projects are underway.

Figure 8.4.6 : The molecule shown, HIV-1 protease, is an important target for pharmaceutical research. By designing molecules
that bind to this protein, scientists are able to drastically inhibit the progress of the disease.
A diagram of a molecule is shown. The image shows a tangle of ribbon-like, intertwined, pink and green curling lines with a
complex ball and stick model in the center.

8.4.1: Molecular Orbital Energy Diagrams


The relative energy levels of atomic and molecular orbitals are typically shown in a molecular orbital diagram (Figure 8.4.7). For a
diatomic molecule, the atomic orbitals of one atom are shown on the left, and those of the other atom are shown on the right. Each
horizontal line represents one orbital that can hold two electrons. The molecular orbitals formed by the combination of the atomic
orbitals are shown in the center. Dashed lines show which of the atomic orbitals combine to form the molecular orbitals. For each
pair of atomic orbitals that combine, one lower-energy (bonding) molecular orbital and one higher-energy (antibonding) orbital
result. Thus we can see that combining the six 2p atomic orbitals results in three bonding orbitals (one σ and two π) and three
antibonding orbitals (one σ* and two π*).

Access for free at OpenStax 8.4.6 https://chem.libretexts.org/@go/page/38195


Figure 8.4.7 : This is the molecular orbital diagram for the homonuclear diatomic Be , showing the molecular orbitals of the
+

valence shell only. The molecular orbitals are filled in the same manner as atomic orbitals, using the Aufbau principle and Hund’s
rule.
A diagram is shown that has an upward-facing vertical arrow running along the left side labeled, “E.” At the bottom center of the
diagram is a horizontal line labeled, “sigma subscript 2 s,” that has two vertical half arrows drawn on it, one facing up and one
facing down. This line is connected to the right and left by upward-facing, dotted lines to two more horizontal lines, each labeled,
“2 s.” The line on the left has two vertical half arrows drawn on it, one facing up and one facing down while the line of the right
has one half arrow facing up drawn on it. These two lines are connected by upward-facing dotted lines to another line in the center
of the diagram, but further up from the first. It is labeled, “sigma subscript 2 s superscript asterisk.” This horizontal line has one
upward-facing vertical half-arrow drawn on it. Moving farther up the center of the diagram is a long horizontal line labeled, “sigma
subscript 2 p subscript x,” which lies below two horizontal lines. These two horizontal lines lie side-by-side, and labeled, “pi
subscript 2 p subscript y,” and, “pi subscript 2 p subscript z.” Both the bottom and top lines are connected to the right and left by
upward-facing, dotted lines to three more horizontal lines, each labeled, “2 p.” These sets of lines are connected by upward-facing
dotted lines to another single line and then pair of double lines in the center of the diagram, but farther up from the lower lines.
They are labeled, “sigma subscript 2 p subscript x superscript asterisk,” and, ““pi subscript 2 p subscript y superscript asterisk,”
and, “pi subscript 2 p subscript z superscript asterisk,” respectively. The left and right sides of the diagram have headers that read,
”Atomic orbitals,” while the center is header reads “Molecular orbitals”.
We predict the distribution of electrons in these molecular orbitals by filling the orbitals in the same way that we fill atomic
orbitals, by the Aufbau principle. Lower-energy orbitals fill first, electrons spread out among degenerate orbitals before pairing,
and each orbital can hold a maximum of two electrons with opposite spins (Figure 8.4.7). Just as we write electron configurations
for atoms, we can write the molecular electronic configuration by listing the orbitals with superscripts indicating the number of
electrons present. For clarity, we place parentheses around molecular orbitals with the same energy. In this case, each orbital is at a
different energy, so parentheses separate each orbital. Thus we would expect a diatomic molecule or ion containing seven electrons
(such as Be ) would have the molecular electron configuration (σ ) (σ ) (σ ) (σ ) . It is common to omit the core electrons
+
2 1s
2 ∗
1s
2
2s
2 ∗
2s
1

from molecular orbital diagrams and configurations and include only the valence electrons.

8.4.2: Bond Order


The filled molecular orbital diagram shows the number of electrons in both bonding and antibonding molecular orbitals. The net
contribution of the electrons to the bond strength of a molecule is identified by determining the bond order that results from the
filling of the molecular orbitals by electrons.
When using Lewis structures to describe the distribution of electrons in molecules, we define bond order as the number of bonding
pairs of electrons between two atoms. Thus a single bond has a bond order of 1, a double bond has a bond order of 2, and a triple
bond has a bond order of 3. We define bond order differently when we use the molecular orbital description of the distribution of
electrons, but the resulting bond order is usually the same. The MO technique is more accurate and can handle cases when the
Lewis structure method fails, but both methods describe the same phenomenon.
In the molecular orbital model, an electron contributes to a bonding interaction if it occupies a bonding orbital and it contributes to
an antibonding interaction if it occupies an antibonding orbital. The bond order is calculated by subtracting the destabilizing

Access for free at OpenStax 8.4.7 https://chem.libretexts.org/@go/page/38195


(antibonding) electrons from the stabilizing (bonding) electrons. Since a bond consists of two electrons, we divide by two to get the
bond order. We can determine bond order with the following equation:
(number of bonding electrons) − (number of antibonding electrons)
bond order =
2

The order of a covalent bond is a guide to its strength; a bond between two given atoms becomes stronger as the bond order
increases. If the distribution of electrons in the molecular orbitals between two atoms is such that the resulting bond would have a
bond order of zero, a stable bond does not form. We next look at some specific examples of MO diagrams and bond orders.

8.4.3: Bonding in Diatomic Molecules


A dihydrogen molecule (H2) forms from two hydrogen atoms. When the atomic orbitals of the two atoms combine, the electrons
occupy the molecular orbital of lowest energy, the σ1s bonding orbital. A dihydrogen molecule, H2, readily forms because the
energy of a H2 molecule is lower than that of two H atoms. The σ1s orbital that contains both electrons is lower in energy than
either of the two 1s atomic orbitals.
A molecular orbital can hold two electrons, so both electrons in the H2 molecule are in the σ1s bonding orbital; the electron
configuration is (σ ) . We represent this configuration by a molecular orbital energy diagram (Figure 8.4.8) in which a single
1s
2

upward arrow indicates one electron in an orbital, and two (upward and downward) arrows indicate two electrons of opposite spin.

Figure 8.4.8 : The molecular orbital energy diagram predicts that H2 will be a stable molecule with lower energy than the separated
atoms.
A diagram is shown that has an upward-facing vertical arrow running along the left side labeled “E.” At the bottom center of the
diagram is a horizontal line labeled, “sigma subscript 1 s,” that has two vertical half arrows drawn on it, one facing up and one
facing down. This line is connected to the right and left by upward-facing, dotted lines to two more horizontal lines, each labeled,
“1 s,” and each with one vertical half-arrow facing up drawn on it. These two lines are connected by upward-facing dotted lines to
another line in the center of the diagram, but farther up from the first, and labeled, “sigma subscript 1 s superscript asterisk.” The
left and right sides of the diagram have headers that read, ”Atomic orbitals,” while the center header reads, “Molecular orbitals.”
The bottom left and right are labeled “H” while the center is labeled “H subscript 2.”
A dihydrogen molecule contains two bonding electrons and no antibonding electrons so we have
(2 − 0)
bond order in H = =1
2
2

Because the bond order for the H–H bond is equal to 1, the bond is a single bond.
A helium atom has two electrons, both of which are in its 1s orbital. Two helium atoms do not combine to form a dihelium
molecule, He2, with four electrons, because the stabilizing effect of the two electrons in the lower-energy bonding orbital would be
offset by the destabilizing effect of the two electrons in the higher-energy antibonding molecular orbital. We would write the
hypothetical electron configuration of He2 as (σ ) (σ ) as in Figure 8.4.9. The net energy change would be zero, so there is no
1s
2 ∗
1s
2

driving force for helium atoms to form the diatomic molecule. In fact, helium exists as discrete atoms rather than as diatomic
molecules. The bond order in a hypothetical dihelium molecule would be zero.
(2 − 2)
bond order in He = =0
2
2

A bond order of zero indicates that no bond is formed between two atoms.

Access for free at OpenStax 8.4.8 https://chem.libretexts.org/@go/page/38195


Figure 8.4.9 : The molecular orbital energy diagram predicts that He2 will not be a stable molecule, since it has equal numbers of
bonding and antibonding electrons.
A diagram is shown that has an upward-facing vertical arrow running along the left side labeled, “E.” At the bottom center of the
diagram is a horizontal line labeled, “sigma subscript 1 s,” that has two vertical half arrows drawn on it, one facing up and one
facing down. This line is connected to the right and left by upward-facing, dotted lines to two more horizontal lines, each labeled,
“1 s,” and each with one vertical half-arrow facing up and one facing down drawn on it. These two lines are connected by upward-
facing dotted lines to another line in the center of the diagram, but farther up from the first, and labeled, “sigma subscript 1 s
superscript asterisk.” This line has one upward-facing and one downward-facing vertical arrow drawn on it. The left and right sides
of the diagram have headers that read, “Atomic orbitals,” while the center header reads, “Molecular orbitals.” The bottom left and
right are labeled, “H e,” while the center is labeled, “H e subscript 2.”

8.4.4: The Diatomic Molecules of the Second Period


Eight possible homonuclear diatomic molecules might be formed by the atoms of the second period of the periodic table: Li2, Be2,
B2, C2, N2, O2, F2, and Ne2. However, we can predict that the Be2 molecule and the Ne2 molecule would not be stable. We can see
this by a consideration of the molecular electron configurations (Table 8.4.1).
Table 8.4.1 : Electron Configuration and Bond Order for Molecular Orbitals in Homonuclear Diatomic Molecules of Period Two Elements
Molecule Electron Configuration Bond Order

Li2 2
(σ2s ) 1

Be2 (unstable) 2
(σ2s ) (σ

2s
)
2
0

B2 2
(σ2s ) (σ

2s
2
) (π2py , π2pz )
2
1

C2 2
(σ2s ) (σ

2s
2
) (π2py , π2pz )
4
2

N2 2
(σ2s ) (σ

2s
2 4 2
) (π2py , π2pz ) (σ2px ) 3

O2 2
(σ2s ) (σ

2s
2 2 4
) (σ2px ) (π2py , π2pz ) (π

2py


2pz
)
2
2

F2 2
(σ2s ) (σ

2s
2 2 4
) (σ2px ) (π2py , π2pz ) (π

2py
, π

2pz
)
4
1

Ne2 (unstable) 2
(σ2s ) (σ

2s
2 2 4
) (σ2px ) (π2py , π2pz ) (π

2py


2pz
4
) (σ

2px
2
) 0

We predict valence molecular orbital electron configurations just as we predict electron configurations of atoms. Valence electrons
are assigned to valence molecular orbitals with the lowest possible energies. Consistent with Hund’s rule, whenever there are two
or more degenerate molecular orbitals, electrons fill each orbital of that type singly before any pairing of electrons takes place.

Access for free at OpenStax 8.4.9 https://chem.libretexts.org/@go/page/38195


Figure 8.4.10 : This shows the MO diagrams for each homonuclear diatomic molecule in the second period. The orbital energies
decrease across the period as the effective nuclear charge increases and atomic radius decreases. Between N2 and O2, the order of
the orbitals changes.
A graph is shown in which the y-axis is labeled, “E,” and appears as a vertical, upward-facing arrow. Across the top, the graph
reads, “L i subscript 2,” “B e subscript 2,” “B subscript 2,” “C subscript 2,” “N subscript 2,” “O subscript 2,” “F subscript 2,” and
“Ne subscript 2.” Directly below each of these element terms is a single pink line, and all lines are connected to one another by a
dashed line, to create an overall line that decreases in height as it moves from left to right across the graph. This line is labeled,
“sigma subscript 2 p x superscript asterisk”. Directly below each of these lines is a set of two pink lines, and all lines are connected
to one another by a dashed line, to create an overall line that decreases in height as it moves from left to right across the graph. It is
consistently lower than the first line. This line is labeled, “pi subscript 2 p y superscript asterisk,” and, “pi subscript 2 p z
superscript asterisk.” Directly below each of these double lines is a single pink line, and all lines are connected to one another by a
dashed line, to create an overall line that decreases in height as it moves from left to right across the graph. It has a distinctive drop
at the label, “O subscript 2.” This line is labeled, “sigma subscript 2 p x.” Directly below each of these lines is a set of two pink
lines, and all lines are connected to one another by a dashed line to create an overall line that decreases very slightly in height as it
moves from left to right across the graph. It is consistently lower than the third line until it reaches the point labeled, “O subscript
2.” This line is labeled, “pi subscript 2 p y,” and, “pi subscript 2 p z.” Directly below each of these lines is a single blue line, and all
lines are connected to one another by a dashed line to create an overall line that decreases in height as it moves from left to right
across the graph. This line is labeled, “sigma subscript 2 s superscript asterisk.” Finally, directly below each of these lines is a
single blue line, and all lines are connected to one another by a dashed line to create an overall line that decreases in height as it
moves from left to right across the graph. This line is labeled. “sigma subscript 2 s.”

As we saw in valence bond theory, σ bonds are generally more stable than π bonds formed from degenerate atomic orbitals.
Similarly, in molecular orbital theory, σ orbitals are usually more stable than π orbitals. However, this is not always the case. The
MOs for the valence orbitals of the second period are shown in Figure 8.4.10. Looking at Ne2 molecular orbitals, we see that the
order is consistent with the generic diagram shown in the previous section. However, for atoms with three or fewer electrons in the
p orbitals (Li through N) we observe a different pattern, in which the σp orbital is higher in energy than the πp set. Obtain the
molecular orbital diagram for a homonuclear diatomic ion by adding or subtracting electrons from the diagram for the neutral
molecule.
This switch in orbital ordering occurs because of a phenomenon called s-p mixing. s-p mixing does not create new orbitals; it
merely influences the energies of the existing molecular orbitals. The σs wavefunction mathematically combines with the σp
wavefunction, with the result that the σs orbital becomes more stable, and the σp orbital becomes less stable (Figure 8.4.11).
Similarly, the antibonding orbitals also undergo s-p mixing, with the σs* becoming more stable and the σp* becoming less stable.

Access for free at OpenStax 8.4.10 https://chem.libretexts.org/@go/page/38195


Figure 8.4.11 : Without mixing, the MO pattern occurs as expected, with the σp orbital lower in energy than the σp orbitals. When s-
p mixing occurs, the orbitals shift as shown, with the σp orbital higher in energy than the πp orbitals.
A diagram is shown. At the bottom left of the diagram is a horizontal line that is connected to the right and left by upward-facing,
dotted lines to two more horizontal lines. Those two lines are connected by upward-facing dotted lines to another line in the center
of the diagram but farther up from the first. Each of the bottom two central lines has a vertical downward-facing arrow. Above this
structure is a horizontal line that is connected to the right and left by upward-facing, dotted lines to two sets of three horizontal
lines and those two lines are connected by upward-facing dotted lines to another line in the center of the diagram, but further up
from the first. In between the horizontal lines of this structure are two pairs of horizontal lines that are above the first line but below
the second and connected by dotted lines to the side horizontal lines. The bottom and top central lines each have an upward-facing
vertical arrow. These two structures are redrawn on the right side of the diagram, but this time, the central lines of the bottom
structure are moved downward in relation to the side lines. The upper portion of the structure has its central lines shifted upward in
relation to the side lines. This structure also shows the bottom line appearing above the set of two lines.
s-p mixing occurs when the s and p orbitals have similar energies. The energy difference between 2s and 2p orbitals in O, F, and Ne
is greater than that in Li, Be, B, C, and N. Because of this, O2, F2, and Ne exhibit negligible s-p mixing (not sufficient to change the
energy ordering), and their MO diagrams follow the normal pattern, as shown in Figure 8.4.7. All of the other period 2 diatomic
molecules do have s-p mixing, which leads to the pattern where the σp orbital is raised above the πp set.
Using the MO diagrams shown in Figure 8.4.11, we can add in the electrons and determine the molecular electron configuration
and bond order for each of the diatomic molecules. As shown in Table 8.4.1, Be2 and Ne2 molecules would have a bond order of 0,
and these molecules do not exist.
The combination of two lithium atoms to form a lithium molecule, Li2, is analogous to the formation of H2, but the atomic orbitals
involved are the valence 2s orbitals. Each of the two lithium atoms has one valence electron. Hence, we have two valence electrons
available for the σ2s bonding molecular orbital. Because both valence electrons would be in a bonding orbital, we would predict the
Li2 molecule to be stable. The molecule is, in fact, present in an appreciable concentration in lithium vapor at temperatures near the
boiling point of the element. All of the other molecules in Table 8.4.1 with a bond order greater than zero are also known.
The O2 molecule has enough electrons to half fill the (π , π ) level. We expect the two electrons that occupy these two

2py

2pz

degenerate orbitals to be unpaired, and this molecular electronic configuration for O2 is in accord with the fact that the oxygen
molecule has two unpaired electrons ( Figure 8.4.10). The presence of two unpaired electrons has proved to be difficult to explain
using Lewis structures, but the molecular orbital theory explains it quite well. In fact, the unpaired electrons of the oxygen
molecule provide a strong piece of support for the molecular orbital theory.

 Application: Band Theory in Extended Systems

When two identical atomic orbitals on different atoms combine, two molecular orbitals result (e.g., H in Figure 8.4.8). The
2

bonding orbital is lower in energy than the original atomic orbitals because the atomic orbitals are in-phase in the molecular
orbital. The antibonding orbital is higher in energy than the original atomic orbitals because the atomic orbitals are out-of-
phase.

Access for free at OpenStax 8.4.11 https://chem.libretexts.org/@go/page/38195


In a solid, similar things happen, but on a much larger scale. Remember that even in a small sample there are a huge number of
atoms (typically > 1023 atoms), and therefore a huge number of atomic orbitals that may be combined into molecular orbitals.
When N valence atomic orbitals, all of the same energy and each containing one (1) electron, are combined, N/2 (filled)
bonding orbitals and N/2 (empty) antibonding orbitals will result. Each bonding orbital will show an energy lowering as the
atomic orbitals are mostly in-phase, but each of the bonding orbitals will be a little different and have slightly different
energies. The antibonding orbitals will show an increase in energy as the atomic orbitals are mostly out-of-phase, but each of
the antibonding orbitals will also be a little different and have slightly different energies. The allowed energy levels for all the
bonding orbitals are so close together that they form a band, called the valence band. Likewise, all the antibonding orbitals are
very close together and form a band, called the conduction band. Figure 8.4.12) shows the bands for three important classes of
materials: insulators, semiconductors, and conductors.

Figure 8.4.12 : Molecular orbitals in solids are so closely spaced that they are described as bands. The valence band is lower in
energy and the conduction band is higher in energy. The type of solid is determined by the size of the “band gap” between the
valence and conduction bands. Only a very small amount of energy is required to move electrons from the valence band to the
conduction band in a conductor, and so they conduct electricity well. In an insulator, the band gap is large, so that very few
electrons move, and they are poor conductors of electricity. Semiconductors are in between: they conduct electricity better than
insulators, but not as well as conductors.
This figure shows three diagrams. The first is labeled, “Insulator,” and it consists of two boxes. The “conduction” box is above
and the “valence” box is below. A large gap marked by 4 dashed lines contains a double-headed arrow. One head pointing
towards the “conduction box” and the other towards the “valence” box. The arrow is labeled, “Band gap.” The second diagram
is similar to the first, but the band gap is about half as large. This diagram is labeled, “Semiconductor.” The third diagram is
similar to the other two, but the band gap is about a fifth that of the “Semiconductor” diagram. This diagram is labeled,
“Conductor.”
In order to conduct electricity, electrons must move from the filled valence band to the empty conduction band where they can
move throughout the solid. The size of the band gap, or the energy difference between the top of the valence band and the
bottom of the conduction band, determines how easy it is to move electrons between the bands. Only a small amount of energy
is required in a conductor because the band gap is very small. This small energy difference is “easy” to overcome, so they are
good conductors of electricity. In an insulator, the band gap is so “large” that very few electrons move into the conduction
band; as a result, insulators are poor conductors of electricity. Semiconductors conduct electricity when “moderate” amounts of
energy are provided to move electrons out of the valence band and into the conduction band. Semiconductors, such as silicon,
are found in many electronics.
Semiconductors are used in devices such as computers, smartphones, and solar cells. Solar cells produce electricity when light
provides the energy to move electrons out of the valence band. The electricity that is generated may then be used to power a
light or tool, or it can be stored for later use by charging a battery. As of December 2014, up to 46% of the energy in sunlight
could be converted into electricity using solar cells.

 Example 8.4.2: Molecular Orbital Diagrams, Bond Order, and Number of Unpaired Electrons

Draw the molecular orbital diagram for the oxygen molecule, O2. From this diagram, calculate the bond order for O2. How
does this diagram account for the paramagnetism of O2?

Solution

Access for free at OpenStax 8.4.12 https://chem.libretexts.org/@go/page/38195


We draw a molecular orbital energy diagram similar to that shown in Figure 8.4.7. Each oxygen atom contributes six electrons,
so the diagram appears as shown in Figure 8.4.7.

Figure 8.4.13 : The molecular orbital energy diagram for O2 predicts two unpaired electrons.
We calculate the bond order as
(8 − 4)
O = =2
2
2

Oxygen's paramagnetism is explained by the presence of two unpaired electrons in the (π2py, π2pz)* molecular orbitals.

 Exercise 8.4.2

The main component of air is N2. From the molecular orbital diagram of N2, predict its bond order and whether it is
diamagnetic or paramagnetic.

Answer
N2 has a bond order of 3 and is diamagnetic.

 Example 8.4.3: Ion Predictions with MO Diagrams

Give the molecular orbital configuration for the valence electrons in C 2−

2
. Will this ion be stable?

Solution
Looking at the appropriate MO diagram, we see that the π orbitals are lower in energy than the σp orbital. The valence electron
configuration for C2 is
2
(σ2s ) (σ

2s
2
) (π2py , π2pz )
4
.
Adding two more electrons to generate the C 2−
2
anion will give a valence electron configuration of
2 ∗ 2 4 2
(σ2s ) (σ ) (π2py , π2pz ) (σ2px )
2s

Since this has six more bonding electrons than antibonding, the bond order will be 3, and the ion should be stable.

Access for free at OpenStax 8.4.13 https://chem.libretexts.org/@go/page/38195


 Exercise 8.4.3

How many unpaired electrons would be present on a Be 2−

2
ion? Would it be paramagnetic or diamagnetic?

Answer
two, paramagnetic

8.4.5: Key Concepts and Summary


Molecular orbital (MO) theory describes the behavior of electrons in a molecule in terms of combinations of the atomic wave
functions. The resulting molecular orbitals may extend over all the atoms in the molecule. Bonding molecular orbitals are formed
by in-phase combinations of atomic wave functions, and electrons in these orbitals stabilize a molecule. Antibonding molecular
orbitals result from out-of-phase combinations of atomic wave functions and electrons in these orbitals make a molecule less stable.
Molecular orbitals located along an internuclear axis are called σ MOs. They can be formed from s orbitals or from p orbitals
oriented in an end-to-end fashion. Molecular orbitals formed from p orbitals oriented in a side-by-side fashion have electron
density on opposite sides of the internuclear axis and are called π orbitals.
We can describe the electronic structure of diatomic molecules by applying molecular orbital theory to the valence electrons of the
atoms. Electrons fill molecular orbitals following the same rules that apply to filling atomic orbitals; Hund’s rule and the Aufbau
principle tell us that lower-energy orbitals will fill first, electrons will spread out before they pair up, and each orbital can hold a
maximum of two electrons with opposite spins. Materials with unpaired electrons are paramagnetic and attracted to a magnetic
field, while those with all-paired electrons are diamagnetic and repelled by a magnetic field. Correctly predicting the magnetic
properties of molecules is in advantage of molecular orbital theory over Lewis structures and valence bond theory.

This page titled 8.4: Molecular Orbital Theory is shared under a CC BY 4.0 license and was authored, remixed, and/or curated by OpenStax via
source content that was edited to the style and standards of the LibreTexts platform; a detailed edit history is available upon request.

Access for free at OpenStax 8.4.14 https://chem.libretexts.org/@go/page/38195


8.E: Advanced Theories of Covalent Bonding (Exercises)
8.E.0.1: 8.1: Valence Bond Theory
8.E.0.1: Chapter Exercises
1. Explain how σ and π bonds are similar and how they are different.
2. Draw a curve that describes the energy of a system with H and Cl atoms at varying distances. Then, find the minimum energy of
this curve two ways.
a. Use the bond energy found in Table 8.2.1 to calculate the energy for one single HCl bond (Hint: How many bonds are in a
mole?)
b. Use the enthalpy of reaction and the bond energies for H and C l to solve for the energy of one mole of HCl bonds.
2 2


H2(g) + C l2(g) ⇌ 2H C l(g) ΔHrxn = −184.7 kJ/mol (8.E.1)

3. Explain why bonds occur at specific average bond distances instead of the atoms approaching each other infinitely close.
4. Use valence bond theory to explain the bonding in F2, HF, and ClBr. Sketch the overlap of the atomic orbitals involved in the
bonds.
5. Use valence bond theory to explain the bonding in O2. Sketch the overlap of the atomic orbitals involved in the bonds in O2.
6. How many σ and π bonds are present in the molecule HCN?
7. A friend tells you N2 has three π bonds due to overlap of the three p-orbitals on each N atom. Do you agree?
8. Draw the Lewis structures for CO2 and CO, and predict the number of σ and π bonds for each molecule.
a. CO2
b. CO

8.E.0.1: Solutions
1. Similarities: Both types of bonds result from overlap of atomic orbitals on adjacent atoms and contain a maximum of two
electrons. Differences: σ bonds are stronger and result from end-to-end overlap and all single bonds are σ bonds; π bonds between
the same two atoms are weaker because they result from side-by-side overlap, and multiple bonds contain one or more π bonds (in
addition to a σ bond).
2

When H and Cl are separate (the x axis) the energy is at a particular value. As they approach, it decreases to a minimum at 127 pm
(the bond distance), and then it increases sharply as you get closer.
a. (a) H–Cl431 kJ/mol 427kJmol×mol6.022×1023bonds×1000 JkJ=7.09×10−19
b. (b) We know Hess’s law related to bond energies: ΔH°=ƩΔH∘BDE(broken)−ƩΔH∘BDE(formed) We are given the enthalpy of
reaction
−184.7kJ/mol = (ΔH ∘ BDE(H – H ) + ΔH ∘ BDE(C l– C l)) − (2ΔH ∘ BDE(H – C l)) (8.E.2)

H – H is436kJ/molandC l– C lis243 (8.E.3)

– 184.7kJ/mol = (436 + 243)– 2x = 679– 2x (8.E.4)

2x = 863.7kJ/mol (8.E.5)

Access for free at OpenStax 8.E.1 https://chem.libretexts.org/@go/page/46472


x = 432 kJ/mol (8.E.6)

This is very close to the value from part (a).


3. The specific average bond distance is the distance with the lowest energy. At distances less than the bond distance, the positive
charges on the two nuclei repel each other, and the overall energy increases.
4. The single bond present in each molecule results from overlap of the relevant orbitals: F 2p orbitals in F2, the H 1s and F 2p
orbitals in HF, and the Cl 3p orbital and Br 4p orbital in ClBr.
5. Bonding: One σ bond and one π bond. The s orbitals are filled and do not overlap. The p orbitals overlap along the axis to form a
σ bond and side by side to form the π bond.

6. H– C ≡ N has two σ (H–C and C–N) and two π (making the CN triple bond).

7. No, two of the p orbitals (one on each N) will be oriented end-to-end and will form a σ bond.

8. (a) 2 σ 2 π;

(b) 1 σ 2 π;

8.E.0.1: 8.2: Hybrid Atomic Orbitals

8.E.1: Chemistry End of Chapter Exercises


Why is the concept of hybridization required in valence bond theory?
Hybridization is introduced to explain the geometry of bonding orbitals in valance bond theory.
Give the shape that describes each hybrid orbital set:
(a) sp2
(b) sp3d
(c) sp
(d) sp3d2
Explain why a carbon atom cannot form five bonds using sp3d hybrid orbitals.
There are no d orbitals in the valence shell of carbon.
What is the hybridization of the central atom in each of the following?
(a) BeH2

Access for free at OpenStax 8.E.2 https://chem.libretexts.org/@go/page/46472


(b) SF6
(c) PO 3−

(d) PCl5
A molecule with the formula AB3 could have one of four different shapes. Give the shape and the hybridization of the central A
atom for each.
trigonal planar, sp2; trigonal pyramidal (one lone pair on A) sp3; T-shaped (two lone pairs on A sp3d, or (three lone pairs on A)
sp3d2
Methionine, CH3SCH2CH2CH(NH2)CO2H, is an amino acid found in proteins. Draw a Lewis structure of this compound. What is
the hybridization type of each carbon, oxygen, the nitrogen, and the sulfur?

Sulfuric acid is manufactured by a series of reactions represented by the following equations:


S (s) + 8 O (g) ⟶ 8 SO (g)
8 2 2

2 SO (g) + O (g) ⟶ 2 SO (g)


2 2 3

SO (g) + H O(l) ⟶ H SO (l)


3 2 2 4

Draw a Lewis structure, predict the molecular geometry by VSEPR, and determine the hybridization of sulfur for the following:
(a) circular S8 molecule
(b) SO2 molecule
(c) SO3 molecule
(d) H2SO4 molecule (the hydrogen atoms are bonded to oxygen atoms)
(a) Each S has a bent (109°) geometry, sp3

(b) Bent (120°), sp2

(c) Trigonal planar, sp2

Access for free at OpenStax 8.E.3 https://chem.libretexts.org/@go/page/46472


(d) Tetrahedral, sp3

Two important industrial chemicals, ethene, C2H4, and propene, C3H6, are produced by the steam (or thermal) cracking process:
2 C H (g) ⟶ C H (g) + C H (g) + CH (g) + H (g)
3 8 2 4 3 6 4 2

For each of the four carbon compounds, do the following:


(a) Draw a Lewis structure.
(b) Predict the geometry about the carbon atom.
(c) Determine the hybridization of each type of carbon atom.
For many years after they were discovered, it was believed that the noble gases could not form compounds. Now we know that
belief to be incorrect. A mixture of xenon and fluorine gases, confined in a quartz bulb and placed on a windowsill, is found to
slowly produce a white solid. Analysis of the compound indicates that it contains 77.55% Xe and 22.45% F by mass.
(a) What is the formula of the compound?
(b) Write a Lewis structure for the compound.
(c) Predict the shape of the molecules of the compound.
(d) What hybridization is consistent with the shape you predicted?
(a) XeF2
(b)

(c) linear (d) sp3d


Consider nitrous acid, HNO2 (HONO).
(a) Write a Lewis structure.
(b) What are the electron pair and molecular geometries of the internal oxygen and nitrogen atoms in the HNO2 molecule?
(c) What is the hybridization on the internal oxygen and nitrogen atoms in HNO2?
Strike-anywhere matches contain a layer of KClO3 and a layer of P4S3. The heat produced by the friction of striking the match
causes these two compounds to react vigorously, which sets fire to the wooden stem of the match. KClO3 contains the ClO ion.−

P4S3 is an unusual molecule with the skeletal structure.

Access for free at OpenStax 8.E.4 https://chem.libretexts.org/@go/page/46472


(a) Write Lewis structures for P4S3 and the ClO ion.

(b) Describe the geometry about the P atoms, the S atom, and the Cl atom in these species.
(c) Assign a hybridization to the P atoms, the S atom, and the Cl atom in these species.
(d) Determine the oxidation states and formal charge of the atoms in P4S3 and the ClO ion.

(a)

(b) P atoms, trigonal pyramidal; S atoms, bent, with two lone pairs; Cl atoms, trigonal pyramidal; (c) Hybridization about P, S, and
1
Cl is, in all cases, sp3; (d) Oxidation states P +1, S −1 , Cl +5, O –2. Formal charges: P 0; S 0; Cl +2: O –1
3

Identify the hybridization of each carbon atom in the following molecule. (The arrangement of atoms is given; you need to
determine how many bonds connect each pair of atoms.)

Write Lewis structures for NF3 and PF5. On the basis of hybrid orbitals, explain the fact that NF3, PF3, and PF5 are stable
molecules, but NF5 does not exist.

Phosphorus and nitrogen can form sp3 hybrids to form three bonds and hold one lone pair in PF3 and NF3, respectively. However,
nitrogen has no valence d orbitals, so it cannot form a set of sp3d hybrid orbitals to bind five fluorine atoms in NF5. Phosphorus has
d orbitals and can bind five fluorine atoms with sp3d hybrid orbitals in PF5.
In addition to NF3, two other fluoro derivatives of nitrogen are known: N2F4 and N2F2. What shapes do you predict for these two
molecules? What is the hybridization for the nitrogen in each molecule?

8.E.1.1: 8.3: Multiple Bonds

Access for free at OpenStax 8.E.5 https://chem.libretexts.org/@go/page/46472


8.E.1.1: Chemistry End of Chapter Exercises
The bond energy of a C–C single bond averages 347 kJ mol−1; that of a C≡C triple bond averages 839 kJ mol−1. Explain why the
triple bond is not three times as strong as a single bond.
A triple bond consists of one σ bond and two π bonds. A σ bond is stronger than a π bond due to greater overlap.
For the carbonate ion, CO , draw all of the resonance structures. Identify which orbitals overlap to create each bond.
2−

A useful solvent that will dissolve salts as well as organic compounds is the compound acetonitrile, H3CCN. It is present in paint
strippers.
(a) Write the Lewis structure for acetonitrile, and indicate the direction of the dipole moment in the molecule.
(b) Identify the hybrid orbitals used by the carbon atoms in the molecule to form σ bonds.
(c) Describe the atomic orbitals that form the π bonds in the molecule. Note that it is not necessary to hybridize the nitrogen atom.
(a)

(b) The terminal carbon atom uses sp3 hybrid orbitals, while the central carbon atom is sp hybridized. (c) Each of the two π bonds
is formed by overlap of a 2p orbital on carbon and a nitrogen 2p orbital.
For the molecule allene, H C = C = C H , give the hybridization of each carbon atom. Will the hydrogen atoms be in the same
2 2

plane or perpendicular planes?


Identify the hybridization of the central atom in each of the following molecules and ions that contain multiple bonds:
(a) ClNO (N is the central atom)
(b) CS2
(c) Cl2CO (C is the central atom)
(d) Cl2SO (S is the central atom)
(e) SO2F2 (S is the central atom)
(f) XeO2F2 (Xe is the central atom)
(g) ClOF (Cl is the central atom)
+
2

(a) sp ; (b) sp; (c) sp2; (d) sp3; (e) sp3; (f) sp3d; (g) sp3
2

Describe the molecular geometry and hybridization of the N, P, or S atoms in each of the following compounds.
(a) H3PO4, phosphoric acid, used in cola soft drinks
(b) NH4NO3, ammonium nitrate, a fertilizer and explosive
(c) S2Cl2, disulfur dichloride, used in vulcanizing rubber
(d) K4[O3POPO3], potassium pyrophosphate, an ingredient in some toothpastes
For each of the following molecules, indicate the hybridization requested and whether or not the electrons will be delocalized:
(a) ozone (O3) central O hybridization
(b) carbon dioxide (CO2) central C hybridization
(c) nitrogen dioxide (NO2) central N hybridization
(d) phosphate ion (PO 3−
4
) central P hybridization
(a) sp , delocalized; (b) sp, localized; (c) sp2, delocalized; (d) sp3, delocalized
2

Access for free at OpenStax 8.E.6 https://chem.libretexts.org/@go/page/46472


For each of the following structures, determine the hybridization requested and whether the electrons will be delocalized:
(a) Hybridization of each carbon

(b) Hybridization of sulfur

(c) All atoms

Draw the orbital diagram for carbon in CO2 showing how many carbon atom electrons are in each orbital.

Each of the four electrons is in a separate orbital and overlaps with an electron on an oxygen atom.

8.E.1.1: 8.4: Molecular Orbital Theory

8.E.2: Chemistry End of Chapter Exercises


Sketch the distribution of electron density in the bonding and antibonding molecular orbitals formed from two s orbitals and from
two p orbitals.
How are the following similar, and how do they differ?
(a) σ molecular orbitals and π molecular orbitals
(b) ψ for an atomic orbital and ψ for a molecular orbital
(c) bonding orbitals and antibonding orbitals

Access for free at OpenStax 8.E.7 https://chem.libretexts.org/@go/page/46472


(a) Similarities: Both are bonding orbitals that can contain a maximum of two electrons. Differences: σ orbitals are end-to-end
combinations of atomic orbitals, whereas π orbitals are formed by side-by-side overlap of orbitals. (b) Similarities: Both are
quantum-mechanical constructs that represent the probability of finding the electron about the atom or the molecule. Differences: ψ
for an atomic orbital describes the behavior of only one electron at a time based on the atom. For a molecule, ψ represents a
mathematical combination of atomic orbitals. (c) Similarities: Both are orbitals that can contain two electrons. Differences:
Bonding orbitals result in holding two or more atoms together. Antibonding orbitals have the effect of destabilizing any bonding
that has occurred.
If molecular orbitals are created by combining five atomic orbitals from atom A and five atomic orbitals from atom B combine,
how many molecular orbitals will result?
Can a molecule with an odd number of electrons ever be diamagnetic? Explain why or why not.
An odd number of electrons can never be paired, regardless of the arrangement of the molecular orbitals. It will always be
paramagnetic.
Can a molecule with an even number of electrons ever be paramagnetic? Explain why or why not.
Why are bonding molecular orbitals lower in energy than the parent atomic orbitals?
Bonding orbitals have electron density in close proximity to more than one nucleus. The interaction between the bonding positively
charged nuclei and negatively charged electrons stabilizes the system.
Calculate the bond order for an ion with this configuration:
2 ∗ 2 2 4 ∗ ∗ 3
(σ2s ) (σ ) (σ2px ) (π2py , π2pz ) (π , π )
2s 2py 2pz

Explain why an electron in the bonding molecular orbital in the H2 molecule has a lower energy than an electron in the 1s atomic
orbital of either of the separated hydrogen atoms.
The pairing of the two bonding electrons lowers the energy of the system relative to the energy of the nonbonded electrons.
Predict the valence electron molecular orbital configurations for the following, and state whether they will be stable or unstable
ions.
(a) Na 2+
2

(b) Mg 2+
2

(c) Al 2+

(d) Si 2+
2

(e) P 2+
2

(f) S 2+
2

(g) F 2+
2

(h) Ar 2+
2

Determine the bond order of each member of the following groups, and determine which member of each group is predicted by the
molecular orbital model to have the strongest bond.
(a) H2, H , H +
2

2

(b) O2, O 2+
2
,O 2−
2

(c) Li2, Be , Be2 +

(d) F2, F , F +
2

2

(e) N2, N , N +

2

(a) H2 bond order = 1, H bond order = 0.5, H bond order = 0.5, strongest bond is H2; (b) O2 bond order = 2, O bond order =
+

2

2
2+
2

3; O bond order = 1, strongest bond is O ; (c) Li2 bond order = 1, Be bond order = 0.5, Be2 bond order = 0, strongest bond is
2−
2
2+
2
+
2

Li ;(d) F2 bond order = 1, F bond order = 1.5, F bond order = 0.5, strongest bond is F ; (e) N2 bond order = 3, N bond order
+ − + +

2 2 2 2 2

= 2.5, N bond order = 2.5, strongest bond is N2



2

Access for free at OpenStax 8.E.8 https://chem.libretexts.org/@go/page/46472


For the first ionization energy for an N2 molecule, what molecular orbital is the electron removed from?
Compare the atomic and molecular orbital diagrams to identify the member of each of the following pairs that has the highest first
ionization energy (the most tightly bound electron) in the gas phase:
(a) H and H2
(b) N and N2
(c) O and O2
(d) C and C2
(e) B and B2
(a) H2; (b) N2; (c) O; (d) C2; (e) B2
Which of the period 2 homonuclear diatomic molecules are predicted to be paramagnetic?
A friend tells you that the 2s orbital for fluorine starts off at a much lower energy than the 2s orbital for lithium, so the resulting σ2s
molecular orbital in F2 is more stable than in Li2. Do you agree?
Yes, fluorine is a smaller atom than Li, so atoms in the 2s orbital are closer to the nucleus and more stable.
True or false: Boron contains 2s22p1 valence electrons, so only one p orbital is needed to form molecular orbitals.
What charge would be needed on F2 to generate an ion with a bond order of 2?
2+
Predict whether the MO diagram for S2 would show s-p mixing or not.
Explain why N 2+

2
is diamagnetic, while O
4+
2
, which has the same number of valence electrons, is paramagnetic.
N2 has s-p mixing, so the π orbitals are the last filled in N
2+

2
. O2 does not have s-p mixing, so the σp orbital fills before the π
orbitals.
Using the MO diagrams, predict the bond order for the stronger bond in each pair:
(a) B2 or B +
2

(b) F2 or F +

(c) O2 or O 2+

(d) C or C
+
2

2

This page titled 8.E: Advanced Theories of Covalent Bonding (Exercises) is shared under a CC BY 4.0 license and was authored, remixed, and/or
curated by OpenStax via source content that was edited to the style and standards of the LibreTexts platform; a detailed edit history is available
upon request.

Access for free at OpenStax 8.E.9 https://chem.libretexts.org/@go/page/46472


CHAPTER OVERVIEW
9: Gases

A general chemistry Libretexts Textbook remixed and remastered from


OpenStax's textbook:
General Chemistry
In this chapter, we examine the relationships between gas temperature, pressure, amount, and volume. We will study a simple
theoretical model and use it to analyze the experimental behavior of gases. The results of these analyses will show us the
limitations of the theory and how to improve on it.
9.1: Gas Pressure
9.2: Relating Pressure, Volume, Amount, and Temperature - The Ideal Gas Law
9.3: Stoichiometry of Gaseous Substances, Mixtures, and Reactions
9.4: Effusion and Diffusion of Gases
9.5: The Kinetic-Molecular Theory
9.6: Non-Ideal Gas Behavior
9.E: Gases (Exercises)

Paul Flowers (University of North Carolina - Pembroke), Klaus Theopold (University of Delaware) and Richard Langley
(Stephen F. Austin State University) with contributing authors. Textbook content produced by OpenStax College is licensed
under a Creative Commons Attribution License 4.0 license. Download for free at http://cnx.org/contents/85abf193-
2bd...a7ac8df6@9.110).
Thumbnail: As long as black-body radiation (not shown) doesn't escape a system, atoms in thermal agitation undergo essentially
elastic collisions. On average, two atoms rebound from each other with the same kinetic energy as before a collision. Five
atoms are colored red so their paths of motion are easier to see. (Public Domain; Greg L via Wikipedia)

This page titled 9: Gases is shared under a CC BY 4.0 license and was authored, remixed, and/or curated by OpenStax via source content that was
edited to the style and standards of the LibreTexts platform; a detailed edit history is available upon request.

1
9.1: Gas Pressure
 Learning Objectives
Define the property of pressure
Define and convert among the units of pressure measurements
Describe the operation of common tools for measuring gas pressure
Calculate pressure from manometer data

The earth’s atmosphere exerts a pressure, as does any other gas. Although we do not normally notice atmospheric pressure, we are
sensitive to pressure changes—for example, when your ears “pop” during take-off and landing while flying, or when you dive
underwater. Gas pressure is caused by the force exerted by gas molecules colliding with the surfaces of objects (Figure 9.1.1).
Although the force of each collision is very small, any surface of appreciable area experiences a large number of collisions in a
short time, which can result in a high pressure. In fact, normal air pressure is strong enough to crush a metal container when not
balanced by equal pressure from inside the container.

Figure 9.1.1 : The atmosphere above us exerts a large pressure on objects at the surface of the earth, roughly equal to the weight of
a bowling ball pressing on an area the size of a human thumbnail.
Diagram of Earth with a square inch column of air molecules extending to the atmosphere. This column points to an arrow pointing
down on a bowling ball resting on a human thumbnail placed on top of a table.
Atmospheric pressure is caused by the weight of the column of air molecules in the atmosphere above an object, such as the tanker
car. At sea level, this pressure is roughly the same as that exerted by a full-grown African elephant standing on a doormat, or a
typical bowling ball resting on your thumbnail. These may seem like huge amounts, and they are, but life on earth has evolved
under such atmospheric pressure. If you actually perch a bowling ball on your thumbnail, the pressure experienced is twice the
usual pressure, and the sensation is unpleasant.

A dramatic illustration of atmospheric pressure is provided in this brief video, which shows a railway tanker car imploding when
its internal pressure is decreased.
Pressure is defined as the force exerted on a given area:
F
P = (9.1.1)
A

Access for free at OpenStax 9.1.1 https://chem.libretexts.org/@go/page/38201


Since pressure is directly proportional to force and inversely proportional to area (Equation 9.1.1), pressure can be increased
either by either increasing the amount of force or by decreasing the area over which it is applied. Correspondingly, pressure
can be decreased by either decreasing the force or increasing the area.

Let’s apply the definition of pressure (Equation 9.1.1 ) to determine which would be more likely to fall through thin ice in Figure
9.1.2.—the elephant or the figure skater?

Figure 9.1.2 : Although (a) an elephant’s weight is large, creating a very large force on the ground, (b) the figure skater exerts a
much higher pressure on the ice due to the small surface area of her skates. (credit a: modification of work by Guido da Rozze;
credit b: modification of work by Ryosuke Yagi).
Figure a is a photo of a large gray elephant on grassy, beige terrain. Figure b is a photo of a figure skater with her right skate on the
ice, upper torso lowered, arms extended upward behind her chest, and left leg extended upward behind her.
A large African elephant can weigh 7 tons, supported on four feet, each with a diameter of about 1.5 ft (footprint area of 250 in2),
so the pressure exerted by each foot is about 14 lb/in2:
lb 1 elephant 1 f oot
2
pressure per elephant f oot = 14, 000 × × = 14 lb/i n (9.1.2)
2
elephant 4 f eet 250 in

The figure skater weighs about 120 lbs, supported on two skate blades, each with an area of about 2 in2, so the pressure exerted by
each blade is about 30 lb/in2:
lb 1 skater 1 blade 2
pressure per skate blade = 120 × × = 30 lb/i n (9.1.3)
2
skater 2 blades 2 in

Even though the elephant is more than one hundred times heavier than the skater, it exerts less than one-half of the pressure and
would therefore be less likely to fall through thin ice. On the other hand, if the skater removes her skates and stands with bare feet
(or regular footwear) on the ice, the larger area over which her weight is applied greatly reduces the pressure exerted:
lb 1 skater 1 f oot
2
pressure per human f oot = 120 × × = 2 lb/i n (9.1.4)
2
skater 2 f eet 30 in

The SI unit of pressure is the pascal (Pa), with 1 Pa = 1 N/m2, where N is the newton, a unit of force defined as 1 kg m/s2. One
pascal is a small pressure; in many cases, it is more convenient to use units of kilopascal (1 kPa = 1000 Pa) or bar (1 bar = 100,000
Pa). In the United States, pressure is often measured in pounds of force on an area of one square inch—pounds per square inch (psi)
—for example, in car tires. Pressure can also be measured using the unit atmosphere (atm), which originally represented the
average sea level air pressure at the approximate latitude of Paris (45°). Table 9.1.1 provides some information on these and a few
other common units for pressure measurements
Table 9.1.1 : Pressure Units
Unit Name and Abbreviation Definition or Relation to Other Unit Comment

pascal (Pa) 1 Pa = 1 N/m2 recommended IUPAC unit

kilopascal (kPa) 1 kPa = 1000 Pa

pounds per square inch (psi) air pressure at sea level is ~14.7 psi

atmosphere (atm) 1 atm = 101,325 Pa air pressure at sea level is ~1 atm

bar (bar, or b) 1 bar = 100,000 Pa (exactly) commonly used in meteorology

Access for free at OpenStax 9.1.2 https://chem.libretexts.org/@go/page/38201


Unit Name and Abbreviation Definition or Relation to Other Unit Comment

millibar (mbar, or mb) 1000 mbar = 1 bar

used by aviation industry, also some weather


inches of mercury (in. Hg) 1 in. Hg = 3386 Pa
reports
1 named after Evangelista Torricelli, inventor of
torr 1 torr = atm
760 the barometer

millimeters of mercury (mm Hg) 1 mm Hg ~1 torr

 Example 9.1.1: Conversion of Pressure Units

The United States National Weather Service reports pressure in both inches of Hg and millibars. Convert a pressure of 29.2 in.
Hg into:
a. torr
b. atm
c. kPa
d. mbar

Solution
This is a unit conversion problem. The relationships between the various pressure units are given in Table 9.2.1.
25.4 mm 1 torr
a. 29.2 in Hg × × = 742 torr
1 in 1 mm Hg

1 atm
b. 742 torr × = 0.976 atm
760 torr

101.325 kPa
c. 742 torr × = 98.9 kPa
760 torr

1000 Pa 1 bar 1000 mbar


d. 98.9 kPa × × × = 989 mbar
1 kPa 100, 000 Pa 1 bar

 Exercise 9.1.1

A typical barometric pressure in Kansas City is 740 torr. What is this pressure in atmospheres, in millimeters of mercury, in
kilopascals, and in bar?

Answer
0.974 atm; 740 mm Hg; 98.7 kPa; 0.987 bar

We can measure atmospheric pressure, the force exerted by the atmosphere on the earth’s surface, with a barometer (Figure 9.1.3).
A barometer is a glass tube that is closed at one end, filled with a nonvolatile liquid such as mercury, and then inverted and
immersed in a container of that liquid. The atmosphere exerts pressure on the liquid outside the tube, the column of liquid exerts
pressure inside the tube, and the pressure at the liquid surface is the same inside and outside the tube. The height of the liquid in the
tube is therefore proportional to the pressure exerted by the atmosphere.

Access for free at OpenStax 9.1.3 https://chem.libretexts.org/@go/page/38201


Figure 9.1.3 : In a barometer, the height, h, of the column of liquid is used as a measurement of the air pressure. Using very dense
liquid mercury (left) permits the construction of reasonably sized barometers, whereas using water (right) would require a
barometer more than 30 feet tall.
Two barometers are in vacuum. One utilizes mercury while the other uses water in the capillary tube. Both barometers are exposed
to atmospheric pressure. The barometer with mercury shows mercury levels of 2.49 feet. The barometer with water has a much
greater level of 33.9 feet.
If the liquid is water, normal atmospheric pressure will support a column of water over 10 meters high, which is rather inconvenient
for making (and reading) a barometer. Because mercury (Hg) is about 13.6-times denser than water, a mercury barometer only
1
needs to be as tall as a water barometer—a more suitable size. Standard atmospheric pressure of 1 atm at sea level (101,325
13.6
Pa) corresponds to a column of mercury that is about 760 mm (29.92 in.) high. The torr was originally intended to be a unit equal
to one millimeter of mercury, but it no longer corresponds exactly. The pressure exerted by a fluid due to gravity is known as
hydrostatic pressure, p:
p = hρg (9.1.5)

where
h is the height of the fluid,
ρ is the density of the fluid, and
g is acceleration due to gravity.

 Example 9.1.2: Calculation of Barometric Pressure

Show the calculation supporting the claim that atmospheric pressure near sea level corresponds to the pressure exerted by a
column of mercury that is about 760 mm high. The density of mercury = 13.6 g/cm . 3

Solution
The hydrostatic pressure is given by Equation 9.1.5, with h = 760 mm, ρ = 13.6 g/cm , and g = 9.81 m/s . Plugging these
3 2

values into the Equation 9.1.5 and doing the necessary unit conversions will give us the value we seek. (Note: We are
expecting to find a pressure of ~101,325 Pa:)
2
kg ⋅ m/s kg
2
101, 325 N/ m = 101, 325 = 101, 325
2 2
m m⋅s

Access for free at OpenStax 9.1.4 https://chem.libretexts.org/@go/page/38201


3
1 m 13.6 g 1 kg (100 cm) 9.81 m
p = (760 mm × ) ×( × × ) ×( )
3 3 2
1000 mm 1 cm 1000 g (1 m) 1 s

3 2 5 2 5 2
= (0.760 m)(13, 600 kg/ m )(9.81 m/ s ) = 1.01 × 10 kg/ms = 1.01 × 10 N/ m

5
= 1.01 × 10 Pa

 Exercise 9.1.2

Calculate the height of a column of water at 25 °C that corresponds to normal atmospheric pressure. The density of water at
this temperature is 1.0 g/cm3.

Answer
10.3 m

A manometer is a device similar to a barometer that can be used to measure the pressure of a gas trapped in a container. A closed-
end manometer is a U-shaped tube with one closed arm, one arm that connects to the gas to be measured, and a nonvolatile liquid
(usually mercury) in between. As with a barometer, the distance between the liquid levels in the two arms of the tube (h in the
diagram) is proportional to the pressure of the gas in the container. An open-end manometer (Figure 9.1.3) is the same as a closed-
end manometer, but one of its arms is open to the atmosphere. In this case, the distance between the liquid levels corresponds to the
difference in pressure between the gas in the container and the atmosphere.

Figure 9.1.4 : A manometer can be used to measure the pressure of a gas. The (difference in) height between the liquid levels (h) is
a measure of the pressure. Mercury is usually used because of its large density.
The first manometer is closed end. The gas in the bulb exerts a certain pressure on the liquid in the tube so that the height, h,
between the two levels of liquid on both sides of the U tube is proportional to the pressure. The equation is P subscript gas equals to
h rho g. The second manometer has an open end. The equation for P subscript gas is equals to P subscript atm minus h rho g. The
final manometer is also open ended and has equation of P subscript gas equals to P subscript atm plus h rho g for cases where
pressure of the gas is greater than atmospheric pressure.

 Example 9.1.3: Calculation of Pressure Using an Open-End Manometer

The pressure of a sample of gas is measured at sea level with an open-end Hg (mercury) manometer, as shown below.
Determine the pressure of the gas in:
a. mm Hg
b. atm
c. kPa

Access for free at OpenStax 9.1.5 https://chem.libretexts.org/@go/page/38201


The height is the difference between the two levels of mercury on each side of the U tube and has a value of 13.7 centimeters.
The level on the right side is higher than the left.

Solution
The pressure of the gas equals the hydrostatic pressure due to a column of mercury of height 13.7 cm plus the pressure of the
atmosphere at sea level. (The pressure at the bottom horizontal line is equal on both sides of the tube. The pressure on the left
is due to the gas and the pressure on the right is due to 13.7 cm of Hg plus atmospheric pressure.)
a. In mm Hg, this is: 137 mm Hg + 760 mm Hg = 897 mm Hg
1 atm
b. 897 mmHg × = 1.18 atm
760 mmHg

101.325 kPa
c. 1.18 atm × = 1.20 × 10
2
kPa
1 atm

 Exercise 9.1.3

The pressure of a sample of gas is measured at sea level with an open-end Hg manometer, as shown below Determine the
pressure of the gas in:
a. mm Hg
b. atm
c. kPa

The height is the difference between the two levels of mercury on each side of the U tube and has a value of 4.63 inches. The
level on the left side is higher than the right.

Answer a
642 mm Hg

Access for free at OpenStax 9.1.6 https://chem.libretexts.org/@go/page/38201


Answer b
0.845 atm
Answer c
85.6 kPa

 Application: Measuring Blood Pressure

Blood pressure is measured using a device called a sphygmomanometer (Greek sphygmos = “pulse”). It consists of an
inflatable cuff to restrict blood flow, a manometer to measure the pressure, and a method of determining when blood flow
begins and when it becomes impeded (Figure 9.1.5). Since its invention in 1881, it has been an essential medical device. There
are many types of sphygmomanometers: manual ones that require a stethoscope and are used by medical professionals;
mercury ones, used when the most accuracy is required; less accurate mechanical ones; and digital ones that can be used with
little training but that have limitations. When using a sphygmomanometer, the cuff is placed around the upper arm and inflated
until blood flow is completely blocked, then slowly released. As the heart beats, blood forced through the arteries causes a rise
in pressure. This rise in pressure at which blood flow begins is the systolic pressure—the peak pressure in the cardiac cycle.
When the cuff’s pressure equals the arterial systolic pressure, blood flows past the cuff, creating audible sounds that can be
heard using a stethoscope. This is followed by a decrease in pressure as the heart’s ventricles prepare for another beat. As cuff
pressure continues to decrease, eventually sound is no longer heard; this is the diastolic pressure—the lowest pressure (resting
phase) in the cardiac cycle. Blood pressure units from a sphygmomanometer are in terms of millimeters of mercury (mm Hg).

Figure 9.1.5 : (a) A medical technician prepares to measure a patient’s blood pressure with a sphygmomanometer. (b) A typical
sphygmomanometer uses a valved rubber bulb to inflate the cuff and a diaphragm gauge to measure pressure. (credit a:
modification of work by Master Sgt. Jeffrey Allen)

9.1.1: Meteorology, Climatology, and Atmospheric Science


Throughout the ages, people have observed clouds, winds, and precipitation, trying to discern patterns and make predictions: when
it is best to plant and harvest; whether it is safe to set out on a sea voyage; and much more. We now face complex weather and
atmosphere-related challenges that will have a major impact on our civilization and the ecosystem. Several different scientific
disciplines use chemical principles to help us better understand weather, the atmosphere, and climate. These are meteorology,
climatology, and atmospheric science. Meteorology is the study of the atmosphere, atmospheric phenomena, and atmospheric
effects on earth’s weather. Meteorologists seek to understand and predict the weather in the short term, which can save lives and
benefit the economy. Weather forecasts (Figure 9.1.5) are the result of thousands of measurements of air pressure, temperature, and
the like, which are compiled, modeled, and analyzed in weather centers worldwide.

Access for free at OpenStax 9.1.7 https://chem.libretexts.org/@go/page/38201


Figure 9.1.6 : Meteorologists use weather maps to describe and predict weather. Regions of high (H) and low (L) pressure have
large effects on weather conditions. The gray lines represent locations of constant pressure known as isobars. (credit: modification
of work by National Oceanic and Atmospheric Administration)
A weather map of the United States is shown which points out areas of high and low pressure with the letters H in blue and L in
red. There are curved grey lines throughout the United States region as well as beyond it around area of Canada and the oceans.
In terms of weather, low-pressure systems occur when the earth’s surface atmospheric pressure is lower than the surrounding
environment: Moist air rises and condenses, producing clouds. Movement of moisture and air within various weather fronts
instigates most weather events.
The atmosphere is the gaseous layer that surrounds a planet. Earth’s atmosphere, which is roughly 100–125 km thick, consists of
roughly 78.1% nitrogen and 21.0% oxygen, and can be subdivided further into the regions shown in Figure 9.1.7: the exosphere
(furthest from earth, > 700 km above sea level), the thermosphere (80–700 km), the mesosphere (50–80 km), the stratosphere
(second lowest level of our atmosphere, 12–50 km above sea level), and the troposphere (up to 12 km above sea level, roughly 80%
of the earth’s atmosphere by mass and the layer where most weather events originate). As you go higher in the troposphere, air
density and temperature both decrease.

Figure 9.1.7 : Earth’s atmosphere has five layers: the troposphere, the stratosphere, the mesosphere, the thermosphere, and the
exosphere.
The different layers of the atmosphere is illustrated as a cross sectional slice of the Earth's atmosphere. The different thickness of
each layer is shown. The thermosphere has the largest portion, followed by the exosphere, stratosphere, mesosphere, and
troposphere.
Climatology is the study of the climate, averaged weather conditions over long time periods, using atmospheric data. However,
climatologists study patterns and effects that occur over decades, centuries, and millennia, rather than shorter time frames of hours,
days, and weeks like meteorologists. Atmospheric science is an even broader field, combining meteorology, climatology, and other
scientific disciplines that study the atmosphere.

Summary
Gases exert pressure, which is force per unit area. The pressure of a gas may be expressed in the SI unit of pascal or kilopascal, as
well as in many other units including torr, atmosphere, and bar. Atmospheric pressure is measured using a barometer; other gas
pressures can be measured using one of several types of manometers.

Access for free at OpenStax 9.1.8 https://chem.libretexts.org/@go/page/38201


9.1.2: Key Equations
F
P =
A
p = hρg

Glossary
atmosphere (atm)
unit of pressure; 1 atm = 101,325 Pa

bar
(bar or b) unit of pressure; 1 bar = 100,000 Pa

barometer
device used to measure atmospheric pressure

hydrostatic pressure
pressure exerted by a fluid due to gravity

manometer
device used to measure the pressure of a gas trapped in a container

pascal (Pa)
SI unit of pressure; 1 Pa = 1 N/m2

pounds per square inch (psi)


unit of pressure common in the US

pressure
force exerted per unit area

torr
1
unit of pressure; 1 torr = atm
760

This page titled 9.1: Gas Pressure is shared under a CC BY 4.0 license and was authored, remixed, and/or curated by OpenStax via source content
that was edited to the style and standards of the LibreTexts platform; a detailed edit history is available upon request.

Access for free at OpenStax 9.1.9 https://chem.libretexts.org/@go/page/38201


9.2: Relating Pressure, Volume, Amount, and Temperature - The Ideal Gas Law
 Learning Objectives
Identify the mathematical relationships between the various properties of gases
Use the ideal gas law, and related gas laws, to compute the values of various gas properties under specified conditions

During the seventeenth and especially eighteenth centuries, driven both by a desire to understand nature and a quest to make balloons in which they could fly (Figure 9.2.1), a number of scientists
established the relationships between the macroscopic physical properties of gases, that is, pressure, volume, temperature, and amount of gas. Although their measurements were not precise by
today’s standards, they were able to determine the mathematical relationships between pairs of these variables (e.g., pressure and temperature, pressure and volume) that hold for an ideal gas—a
hypothetical construct that real gases approximate under certain conditions. Eventually, these individual laws were combined into a single equation—the ideal gas law—that relates gas quantities
for gases and is quite accurate for low pressures and moderate temperatures. We will consider the key developments in individual relationships (for pedagogical reasons not quite in historical order),
then put them together in the ideal gas law.

Figure 9.2.1 : In 1783, the first (a) hydrogen-filled balloon flight, (b) manned hot air balloon flight, and (c) manned hydrogen-filled balloon flight occurred. When the hydrogen-filled balloon
depicted in (a) landed, the frightened villagers of Gonesse reportedly destroyed it with pitchforks and knives. The launch of the latter was reportedly viewed by 400,000 people in Paris.
This figure includes three images. Image a is a black and white image of a hydrogen balloon apparently being deflated by a mob of people. In image b, a blue, gold, and red balloon is being held to
the ground with ropes while positioned above a platform from which smoke is rising beneath the balloon. In c, an image is shown in grey on a peach-colored background of an inflated balloon with
vertical striping in the air. It appears to have a basket attached to its lower side. A large stately building appears in the background.

9.2.1: Pressure and Temperature: Amontons’s Law


Imagine filling a rigid container attached to a pressure gauge with gas and then sealing the container so that no gas may escape. If the container is cooled, the gas inside likewise gets colder and its
pressure is observed to decrease. Since the container is rigid and tightly sealed, both the volume and number of moles of gas remain constant. If we heat the sphere, the gas inside gets hotter (Figure
9.2.2) and the pressure increases.

Figure 9.2.2 : The effect of temperature on gas pressure: When the hot plate is off, the pressure of the gas in the sphere is relatively low. As the gas is heated, the pressure of the gas in the sphere
increases.
This figure includes three similar diagrams. In the first diagram to the left, a rigid spherical container of a gas to which a pressure gauge is attached at the top is placed in a large beaker of water,
indicated in light blue, atop a hot plate. The needle on the pressure gauge points to the far left on the gauge. The diagram is labeled “low P” above and “hot plate off” below. The second similar
diagram also has the rigid spherical container of gas placed in a large beaker from which light blue wavy line segments extend from the top of the liquid in the beaker. The beaker is situated on top
of a slightly reddened circular area. The needle on the pressure gauge points straight up, or to the middle on the gauge. The diagram is labeled “medium P” above and “hot plate on medium” below.
The third diagram also has the rigid spherical container of gas placed in a large beaker in which bubbles appear near the liquid surface and several wavy light blue line segments extend from the
surface out of the beaker. The beaker is situated on top of a bright red circular area. The needle on the pressure gauge points to the far right on the gauge. The diagram is labeled “high P” above and
“hot plate on high” below.
This relationship between temperature and pressure is observed for any sample of gas confined to a constant volume. An example of experimental pressure-temperature data is shown for a sample
of air under these conditions in Figure 9.2.3. We find that temperature and pressure are linearly related, and if the temperature is on the kelvin scale, then P and T are directly proportional (again,
when volume and moles of gas are held constant); if the temperature on the kelvin scale increases by a certain factor, the gas pressure increases by the same factor.

Figure 9.2.3 : For a constant volume and amount of air, the pressure and temperature are directly proportional, provided the temperature is in kelvin. (Measurements cannot be made at lower
temperatures because of the condensation of the gas.) When this line is extrapolated to lower pressures, it reaches a pressure of 0 at –273 °C, which is 0 on the kelvin scale and the lowest possible
temperature, called absolute zero.
This figure includes a table and a graph. The table has 3 columns and 7 rows. The first row is a header, which labels the columns “Temperature, degrees C,” “Temperature, K,” and “Pressure, k P a.”
The first column contains the values from top to bottom negative 150, negative 100, negative 50, 0, 50, and 100. The second column contains the values from top to bottom 173, 223, 273, 323, 373,
and 423. The third column contains the values 36.0, 46.4, 56.7, 67.1, 77.5, and 88.0. A graph appears to the right of the table. The horizontal axis is labeled “Temperature ( K ).” with markings and
labels provided for multiples of 100 beginning at 0 and ending at 500. The vertical axis is labeled “Pressure ( k P a )” with markings and labels provided for multiples of 10, beginning at 0 and
ending at 100. Six data points from the table are plotted on the graph with black dots. These dots are connected with a solid black line. A dashed line extends from the data point furthest to the left
to the origin. The graph shows a positive linear trend.
Guillaume Amontons was the first to empirically establish the relationship between the pressure and the temperature of a gas (~1700), and Joseph Louis Gay-Lussac determined the relationship
more precisely (~1800). Because of this, the P-T relationship for gases is known as either Amontons’s law or Gay-Lussac’s law. Under either name, it states that the pressure of a given amount of
gas is directly proportional to its temperature on the kelvin scale when the volume is held constant. Mathematically, this can be written:

Access for free at OpenStax 9.2.1 https://chem.libretexts.org/@go/page/38202


P ∝ T or P = constant × T or P = k × T

where ∝ means “is proportional to,” and k is a proportionality constant that depends on the identity, amount, and volume of the gas.
P P
For a confined, constant volume of gas, the ratio is therefore constant (i.e., =k ). If the gas is initially in “Condition 1” (with P = P1 and T = T1), and then changes to “Condition 2” (with P =
T T
P1 P2 P1 P2
P2 and T = T2), we have that =k and =k , which reduces to = . This equation is useful for pressure-temperature calculations for a confined gas at constant volume. Note that
T1 T2 T1 T2

temperatures must be on the kelvin scale for any gas law calculations (0 on the kelvin scale and the lowest possible temperature is called absolute zero). (Also note that there are at least three ways
we can describe how the pressure of a gas changes as its temperature changes: We can use a table of values, a graph, or a mathematical equation.)

 Example 9.2.1: Predicting Change in Pressure with Temperature

A can of hair spray is used until it is empty except for the propellant, isobutane gas.
a. On the can is the warning “Store only at temperatures below 120 °F (48.8 °C). Do not incinerate.” Why?
b. The gas in the can is initially at 24 °C and 360 kPa, and the can has a volume of 350 mL. If the can is left in a car that reaches 50 °C on a hot day, what is the new pressure in the can?

Solution
a. The can contains an amount of isobutane gas at a constant volume, so if the temperature is increased by heating, the pressure will increase proportionately. High temperature could lead to
high pressure, causing the can to burst. (Also, isobutane is combustible, so incineration could cause the can to explode.)
b. We are looking for a pressure change due to a temperature change at constant volume, so we will use Amontons’s/Gay-Lussac’s law. Taking P1 and T1 as the initial values, T2 as the
temperature where the pressure is unknown and P2 as the unknown pressure, and converting °C to K, we have:
P1 P2 360 kPa P2
=  which means that  =
T1 T2 297 K 323 K

Rearranging and solving gives:


360 kPa × 323 K
P2 = = 390 kPa
297 K

 Exercise 9.2.1

A sample of nitrogen, N2, occupies 45.0 mL at 27 °C and 600 torr. What pressure will it have if cooled to –73 °C while the volume remains constant?

Answer
400 torr

9.2.2: Volume and Temperature: Charles’s Law


If we fill a balloon with air and seal it, the balloon contains a specific amount of air at atmospheric pressure, let’s say 1 atm. If we put the balloon in a refrigerator, the gas inside gets cold and the
balloon shrinks (although both the amount of gas and its pressure remain constant). If we make the balloon very cold, it will shrink a great deal, and it expands again when it warms up.

Liquid Nitrogen Experiments: The Balloon

Video 9.2.1 : This video shows how cooling and heating a gas causes its volume to decrease or increase, respectively.
These examples of the effect of temperature on the volume of a given amount of a confined gas at constant pressure are true in general: The volume increases as the temperature increases, and
decreases as the temperature decreases. Volume-temperature data for a 1-mole sample of methane gas at 1 atm are listed and graphed in Figure 9.2.2.

Access for free at OpenStax 9.2.2 https://chem.libretexts.org/@go/page/38202


Figuire 9.2.4 : The volume and temperature are linearly related for 1 mole of methane gas at a constant pressure of 1 atm. If the temperature is in kelvin, volume and temperature are directly
proportional. The line stops at 111 K because methane liquefies at this temperature; when extrapolated, it intersects the graph’s origin, representing a temperature of absolute zero.
This figure includes a table and a graph. The table has 3 columns and 6 rows. The first row is a header, which labels the columns “Temperature, degrees C,” “Temperature, K,” and “Pressure, k P a.”
The first column contains the values from top to bottom negative 100, negative 50, 0, 100, and 200. The second column contains the values from top to bottom 173, 223, 273, 373, and 473. The
third column contains the values 14.10, 18.26, 22.40, 30.65, and 38.88. A graph appears to the right of the table. The horizontal axis is labeled “Temperature ( K ).” with markings and labels
provided for multiples of 100 beginning at 0 and ending at 300. The vertical axis is labeled “Volume ( L )” with marking and labels provided for multiples of 10, beginning at 0 and ending at 30.
Five data points from the table are plotted on the graph with black dots. These dots are connected with a solid black line. The graph shows a positive linear trend.
The relationship between the volume and temperature of a given amount of gas at constant pressure is known as Charles’s law in recognition of the French scientist and balloon flight pioneer
Jacques Alexandre César Charles. Charles’s law states that the volume of a given amount of gas is directly proportional to its temperature on the kelvin scale when the pressure is held constant.
Mathematically, this can be written as:

V αT or V = constant ⋅ T or V = k ⋅ T

with k being a proportionality constant that depends on the amount and pressure of the gas.
V V1 V2
For a confined, constant pressure gas sample, is constant (i.e., the ratio = k), and as seen with the P-T relationship, this leads to another form of Charles’s law: = .
T T1 T2

 Example 9.2.2: Predicting Change in Volume with Temperature

A sample of carbon dioxide, CO2, occupies 0.300 L at 10 °C and 750 torr. What volume will the gas have at 30 °C and 750 torr?

Solution
Because we are looking for the volume change caused by a temperature change at constant pressure, this is a job for Charles’s law. Taking V1 and T1 as the initial values, T2 as the temperature at
which the volume is unknown and V2 as the unknown volume, and converting °C into K we have:
V1 V2 0.300 L V2
=  which means that  =
T1 T2 283 K 303 K

0.300 L × 303 K
Rearranging and solving gives: V 2 = = 0.321 L
283 K

This answer supports our expectation from Charles’s law, namely, that raising the gas temperature (from 283 K to 303 K) at a constant pressure will yield an increase in its volume (from 0.300 L to
0.321 L).

 Exercise 9.2.2

A sample of oxygen, O2, occupies 32.2 mL at 30 °C and 452 torr. What volume will it occupy at –70 °C and the same pressure?

Answer
21.6 mL

 Example 9.2.3: Measuring Temperature with a Volume

Change Temperature is sometimes measured with a gas thermometer by observing the change in the volume of the gas as the temperature changes at constant pressure. The hydrogen in a
particular hydrogen gas thermometer has a volume of 150.0 cm3 when immersed in a mixture of ice and water (0.00 °C). When immersed in boiling liquid ammonia, the volume of the
hydrogen, at the same pressure, is 131.7 cm3. Find the temperature of boiling ammonia on the kelvin and Celsius scales.

Solution
A volume change caused by a temperature change at constant pressure means we should use Charles’s law. Taking V1 and T1 as the initial values, T2 as the temperature at which the volume is
unknown and V2 as the unknown volume, and converting °C into K we have:
3 3
V1 V2 150.0 cm 131.7 cm
=  which means that  =
T1 T2 273.15 K T2

3
131.7 cm × 273.15 K
Rearrangement gives T 2 =
3
= 239.8 K
150.0 cm

Subtracting 273.15 from 239.8 K, we find that the temperature of the boiling ammonia on the Celsius scale is –33.4 °C.

 Exercise 9.2.3
What is the volume of a sample of ethane at 467 K and 1.1 atm if it occupies 405 mL at 298 K and 1.1 atm?

Answer
635 mL

Access for free at OpenStax 9.2.3 https://chem.libretexts.org/@go/page/38202


9.2.2.0.1: Volume and Pressure: Boyle’s Law
If we partially fill an airtight syringe with air, the syringe contains a specific amount of air at constant temperature, say 25 °C. If we slowly push in the plunger while keeping temperature constant,
the gas in the syringe is compressed into a smaller volume and its pressure increases; if we pull out the plunger, the volume increases and the pressure decreases. This example of the effect of
volume on the pressure of a given amount of a confined gas is true in general. Decreasing the volume of a contained gas will increase its pressure, and increasing its volume will decrease its
pressure. In fact, if the volume increases by a certain factor, the pressure decreases by the same factor, and vice versa. Volume-pressure data for an air sample at room temperature are graphed in
Figure 9.2.5.

Figure 9.2.5 : When a gas occupies a smaller volume, it exerts a higher pressure; when it occupies a larger volume, it exerts a lower pressure (assuming the amount of gas and the temperature do not
1
change). Since P and V are inversely proportional, a graph of vs. V is linear.
P

This figure contains a diagram and two graphs. The diagram shows a syringe labeled with a scale in m l or c c with multiples of 5 labeled beginning at 5 and ending at 30. The markings halfway
between these measurements are also provided. Attached at the top of the syringe is a pressure gauge with a scale marked by fives from 40 on the left to 5 on the right. The gauge needle rests
between 10 and 15, slightly closer to 15. The syringe plunger position indicates a volume measurement about halfway between 10 and 15 m l or c c. The first graph is labeled “V ( m L )” on the
horizontal axis and “P ( p s i )” on the vertical axis. Points are labeled at 5, 10, 15, 20, and 25 m L with corresponding values of 39.0, 19.5, 13.0, 9.8, and 6.5 p s i. The points are connected with a
smooth curve that is declining at a decreasing rate of change. The second graph is labeled “V ( m L )” on the horizontal axis and “1 divided by P ( p s i )” on the vertical axis. The horizontal axis is
labeled at multiples of 5, beginning at zero and extending up to 35 m L. The vertical axis is labeled by multiples of 0.02, beginning at 0 and extending up to 0.18. Six points indicated by black dots
on this graph are connected with a black line segment showing a positive linear trend.
Unlike the P-T and V-T relationships, pressure and volume are not directly proportional to each other. Instead, P and V exhibit inverse proportionality: Increasing the pressure results in a decrease
of the volume of the gas. Mathematically this can be written:
1
P ∝
V

or
1
P =k⋅
V

or

PV = k

or

P1 V1 = P2 V2

1
with k being a constant. Graphically, this relationship is shown by the straight line that results when plotting the inverse of the pressure ( ) versus the volume (V), or the inverse of volume
P

1
( ) versus the pressure (P ). Graphs with curved lines are difficult to read accurately at low or high values of the variables, and they are more difficult to use in fitting theoretical equations and
V

parameters to experimental data. For those reasons, scientists often try to find a way to “linearize” their data. If we plot P versus V, we obtain a hyperbola (Figure 9.2.6).

1
Figure 9.2.6 : The relationship between pressure and volume is inversely proportional. (a) The graph of P vs. V is a hyperbola, whereas (b) the graph of ( ) vs. V is linear.
P

This diagram shows two graphs. In a, a graph is shown with volume on the horizontal axis and pressure on the vertical axis. A curved line is shown on the graph showing a decreasing trend with a
decreasing rate of change. In b, a graph is shown with volume on the horizontal axis and one divided by pressure on the vertical axis. A line segment, beginning at the origin of the graph, shows a
positive, linear trend.
The relationship between the volume and pressure of a given amount of gas at constant temperature was first published by the English natural philosopher Robert Boyle over 300 years ago. It is
summarized in the statement now known as Boyle’s law: The volume of a given amount of gas held at constant temperature is inversely proportional to the pressure under which it is measured.

 Example 9.2.4: Volume of a Gas Sample


The sample of gas has a volume of 15.0 mL at a pressure of 13.0 psi. Determine the pressure of the gas at a volume of 7.5 mL, using:
a. the P-V graph in Figure 9.2.6a

Access for free at OpenStax 9.2.4 https://chem.libretexts.org/@go/page/38202


1
b. the vs. V graph in Figure 9.2.6b
P
c. the Boyle’s law equation
Comment on the likely accuracy of each method.

Solution
a. Estimating from the P-V graph gives a value for P somewhere around 27 psi.
1
b. Estimating from the versus V graph give a value of about 26 psi.
P
c. From Boyle’s law, we know that the product of pressure and volume (PV) for a given sample of gas at a constant temperature is always equal to the same value. Therefore we have P1V1 = k
and P2V2 = k which means that P1V1 = P2V2.
Using P1 and V1 as the known values 13.0 psi and 15.0 mL, P2 as the pressure at which the volume is unknown, and V2 as the unknown volume, we have:

P1 V1 = P2 V2 or 13.0 psi × 15.0 mL = P2 × 7.5 mL

Solving:

13.0 psi × 15.0 mL


P2 = = 26 psi
7.5 mL

It was more difficult to estimate well from the P-V graph, so (a) is likely more inaccurate than (b) or (c). The calculation will be as accurate as the equation and measurements allow.

 Exercise 9.2.4

The sample of gas has a volume of 30.0 mL at a pressure of 6.5 psi. Determine the volume of the gas at a pressure of 11.0 psi, using:
a. the P-V graph in Figure 9.2.6a
1
b. the vs. V graph in Figure 9.2.6b
P
c. the Boyle’s law equation
Comment on the likely accuracy of each method.

Answer a
about 17–18 mL
Answer b
~18 mL
Answer c
17.7 mL; it was more difficult to estimate well from the P-V graph, so (a) is likely more inaccurate than (b); the calculation will be as accurate as the equation and measurements allow

 Breathing and Boyle’s Law

What do you do about 20 times per minute for your whole life, without break, and often without even being aware of it? The answer, of course, is respiration, or breathing. How does it work? It
turns out that the gas laws apply here. Your lungs take in gas that your body needs (oxygen) and get rid of waste gas (carbon dioxide). Lungs are made of spongy, stretchy tissue that expands
and contracts while you breathe. When you inhale, your diaphragm and intercostal muscles (the muscles between your ribs) contract, expanding your chest cavity and making your lung volume
larger. The increase in volume leads to a decrease in pressure (Boyle’s law). This causes air to flow into the lungs (from high pressure to low pressure). When you exhale, the process reverses:
Your diaphragm and rib muscles relax, your chest cavity contracts, and your lung volume decreases, causing the pressure to increase (Boyle’s law again), and air flows out of the lungs (from
high pressure to low pressure). You then breathe in and out again, and again, repeating this Boyle’s law cycle for the rest of your life (Figure 9.2.7).

Figure 9.2.7 : Breathing occurs because expanding and contracting lung volume creates small pressure differences between your lungs and your surroundings, causing air to be drawn into and
forced out of your lungs.
This figure contains two diagrams of a cross section of the human head and torso. The first diagram on the left is labeled “Inspiration.” It shows curved arrows in gray proceeding through the
nasal passages and mouth to the lungs. An arrow points downward from the diaphragm, which is relatively flat, just beneath the lungs. This arrow is labeled “Diaphragm contracts.” At the
entrance to the mouth and nasal passages, a label of P subscript lungs equals 1 dash 3 torr lower” is provided. The second, similar diagram, which is labeled “Expiration,” reverses the direction
of both arrows. Arrows extend from the lungs out through the nasal passages and mouth. Similarly, an arrow points up to the diaphragm, showing a curved diaphragm and lungs reduced in size
from the previous image. This arrow is labeled “Diaphragm relaxes.” At the entrance to the mouth and nasal passages, a label of P subscript lungs equals 1 dash 3 torr higher” is provided.

Access for free at OpenStax 9.2.5 https://chem.libretexts.org/@go/page/38202


9.2.3: Moles of Gas and Volume: Avogadro’s Law
The Italian scientist Amedeo Avogadro advanced a hypothesis in 1811 to account for the behavior of gases, stating that equal volumes of all gases, measured under the same conditions of
temperature and pressure, contain the same number of molecules. Over time, this relationship was supported by many experimental observations as expressed by Avogadro’s law: For a confined
gas, the volume (V) and number of moles (n) are directly proportional if the pressure and temperature both remain constant.
In equation form, this is written as:
V1 V2
V ∝ n or V = k × n or  =
n1 n2

Mathematical relationships can also be determined for the other variable pairs, such as P versus n, and n versus T.

Visit this interactive PhET simulation to investigate the relationships between pressure, volume, temperature, and amount of gas. Use the
simulation to examine the effect of changing one parameter on another while holding the other parameters constant (as described in the
preceding sections on the various gas laws).

9.2.4: The Ideal Gas Law


To this point, four separate laws have been discussed that relate pressure, volume, temperature, and the number of moles of the gas:
Boyle’s law: PV = constant at constant T and n
P
Amontons’s law: = constant at constant V and n
T
V
Charles’s law: = constant at constant P and n
T
V
Avogadro’s law: = constant at constant P and T
n

Combining these four laws yields the ideal gas law, a relation between the pressure, volume, temperature, and number of moles of a gas:

P V = nRT

where P is the pressure of a gas, V is its volume, n is the number of moles of the gas, T is its temperature on the kelvin scale, and R is a constant called the ideal gas constant or the universal gas
constant. The units used to express pressure, volume, and temperature will determine the proper form of the gas constant as required by dimensional analysis, the most commonly encountered
values being 0.08206 L atm mol–1 K–1 and 8.3145 kPa L mol–1 K–1.

The ideal gas law is easy to remember and apply in solving problems, as long as you use the proper values and units for the gas constant, R.
Gases whose properties of P, V, and T are accurately described by the ideal gas law (or the other gas laws) are said to exhibit ideal behavior or to approximate the traits of an ideal gas. An ideal gas
is a hypothetical construct that may be used along with kinetic molecular theory to effectively explain the gas laws as will be described in a later module of this chapter. Although all the calculations
presented in this module assume ideal behavior, this assumption is only reasonable for gases under conditions of relatively low pressure and high temperature. In the final module of this chapter, a
modified gas law will be introduced that accounts for the non-ideal behavior observed for many gases at relatively high pressures and low temperatures.
The ideal gas equation contains five terms, the gas constant R and the variable properties P, V, n, and T. Specifying any four of these terms will permit use of the ideal gas law to calculate the fifth
term as demonstrated in the following example exercises.

 Example 9.2.5: Using the Ideal Gas Law


Methane, CH4, is being considered for use as an alternative automotive fuel to replace gasoline. One gallon of gasoline could be replaced by 655 g of CH4. What is the volume of this much
methane at 25 °C and 745 torr?

Solution

 Exercise 9.2.5

Calculate the pressure in bar of 2520 moles of hydrogen gas stored at 27 °C in the 180-L storage tank of a modern hydrogen-powered car.

Answer
350 bar

P1 V1 P2 V2
If the number of moles of an ideal gas are kept constant under two different sets of conditions, a useful mathematical relationship called the combined gas law is obtained: = using
T1 T2

units of atm, L, and K. Both sets of conditions are equal to the product of n × R (where n = the number of moles of the gas and R is the ideal gas law constant).

 Example 9.2.6: Using the Combined Gas Law

When filled with air, a typical scuba tank with a volume of 13.2 L has a pressure of 153 atm (Figure 9.2.8). If the water temperature is 27 °C, how many liters of air will such a tank provide to a
diver’s lungs at a depth of approximately 70 feet in the ocean where the pressure is 3.13 atm?

Figure 9.2.8 : Scuba divers use compressed air to breathe while underwater. (credit: modification of work by Mark Goodchild)
This photograph shows a scuba diver underwater with a tank on his or her back and bubbles ascending from the breathing apparatus.
Letting 1 represent the air in the scuba tank and 2 represent the air in the lungs, and noting that body temperature (the temperature the air will be in the lungs) is 37 °C, we have:

Access for free at OpenStax 9.2.6 https://chem.libretexts.org/@go/page/38202


P1 V1 P2 V2 (153 atm)(13.2 L) (3.13 atm)(V2 )
= ⟶ =
T1 T2 (300 K) (310 K)

Solving for V2:

(153 atm )(13.2 L)(310 K )


V2 = = 667 L
(300 K )(3.13 atm )

(Note: Be advised that this particular example is one in which the assumption of ideal gas behavior is not very reasonable, since it involves gases at relatively high pressures and low
temperatures. Despite this limitation, the calculated volume can be viewed as a good “ballpark” estimate.)

 Exercise 9.2.6

A sample of ammonia is found to occupy 0.250 L under laboratory conditions of 27 °C and 0.850 atm. Find the volume of this sample at 0 °C and 1.00 atm.

Answer
0.193 L

 The Interdependence between Ocean Depth and Pressure in Scuba Diving


Whether scuba diving at the Great Barrier Reef in Australia (shown in Figure 9.2.9) or in the Caribbean, divers must understand how pressure affects a number of issues related to their comfort
and safety.

Figure 9.2.9 : Scuba divers, whether at the Great Barrier Reef or in the Caribbean, must be aware of buoyancy, pressure equalization, and the amount of time they spend underwater, to avoid the
risks associated with pressurized gases in the body. (credit: Kyle Taylor).
This picture shows colorful underwater corals and anemones in hues of yellow, orange, green, and brown, surrounded by water that appears blue in color.
Pressure increases with ocean depth, and the pressure changes most rapidly as divers reach the surface. The pressure a diver experiences is the sum of all pressures above the diver (from the
water and the air). Most pressure measurements are given in units of atmospheres, expressed as “atmospheres absolute” or ATA in the diving community: Every 33 feet of salt water represents
1 ATA of pressure in addition to 1 ATA of pressure from the atmosphere at sea level. As a diver descends, the increase in pressure causes the body’s air pockets in the ears and lungs to
compress; on the ascent, the decrease in pressure causes these air pockets to expand, potentially rupturing eardrums or bursting the lungs. Divers must therefore undergo equalization by adding
air to body airspaces on the descent by breathing normally and adding air to the mask by breathing out of the nose or adding air to the ears and sinuses by equalization techniques; the corollary
is also true on ascent, divers must release air from the body to maintain equalization. Buoyancy, or the ability to control whether a diver sinks or floats, is controlled by the buoyancy
compensator (BCD). If a diver is ascending, the air in his BCD expands because of lower pressure according to Boyle’s law (decreasing the pressure of gases increases the volume). The
expanding air increases the buoyancy of the diver, and she or he begins to ascend. The diver must vent air from the BCD or risk an uncontrolled ascent that could rupture the lungs. In
descending, the increased pressure causes the air in the BCD to compress and the diver sinks much more quickly; the diver must add air to the BCD or risk an uncontrolled descent, facing much
higher pressures near the ocean floor. The pressure also impacts how long a diver can stay underwater before ascending. The deeper a diver dives, the more compressed the air that is breathed
because of increased pressure: If a diver dives 33 feet, the pressure is 2 ATA and the air would be compressed to one-half of its original volume. The diver uses up available air twice as fast as
at the surface.

9.2.5: Standard Conditions of Temperature and Pressure


We have seen that the volume of a given quantity of gas and the number of molecules (moles) in a given volume of gas vary with changes in pressure and temperature. Chemists sometimes make
comparisons against a standard temperature and pressure (STP) for reporting properties of gases: 273.15 K (0.00 °C) and 1 atm (101.325 kPa). At STP, an ideal gas has a volume of about 22.4 L—
this is referred to as the standard molar volume (Figure 9.2.10).

Figure 9.2.10 : Since the number of moles in a given volume of gas varies with pressure and temperature changes, chemists use standard temperature and pressure (273.15 K and 1 atm or 101.325
kPa) to report properties of gases.
This figure shows three balloons each filled with H e, N H subscript 2, and O subscript 2 respectively. Beneath the first balloon is the label “4 g of He” Beneath the second balloon is the label, “15 g
of N H subscript 2.” Beneath the third balloon is the label “32 g of O subscript 2.” Each balloon contains the same number of molecules of their respective gases.

Summary
The behavior of gases can be described by several laws based on experimental observations of their properties. The pressure of a given amount of gas is directly proportional to its absolute
temperature, provided that the volume does not change (Amontons’s law). The volume of a given gas sample is directly proportional to its absolute temperature at constant pressure (Charles’s law).

Access for free at OpenStax 9.2.7 https://chem.libretexts.org/@go/page/38202


The volume of a given amount of gas is inversely proportional to its pressure when temperature is held constant (Boyle’s law). Under the same conditions of temperature and pressure, equal
volumes of all gases contain the same number of molecules (Avogadro’s law).
The equations describing these laws are special cases of the ideal gas law, PV = nRT, where P is the pressure of the gas, V is its volume, n is the number of moles of the gas, T is its kelvin
temperature, and R is the ideal (universal) gas constant.

9.2.6: Key Equations


PV = nRT

Summary
absolute zero
temperature at which the volume of a gas would be zero according to Charles’s law.

Amontons’s law
(also, Gay-Lussac’s law) pressure of a given number of moles of gas is directly proportional to its kelvin temperature when the volume is held constant

Avogadro’s law
volume of a gas at constant temperature and pressure is proportional to the number of gas molecules

Boyle’s law
volume of a given number of moles of gas held at constant temperature is inversely proportional to the pressure under which it is measured

Charles’s law
volume of a given number of moles of gas is directly proportional to its kelvin temperature when the pressure is held constant

ideal gas
hypothetical gas whose physical properties are perfectly described by the gas laws

ideal gas constant (R)


constant derived from the ideal gas equation R = 0.08226 L atm mol–1 K–1 or 8.314 L kPa mol–1 K–1

ideal gas law


relation between the pressure, volume, amount, and temperature of a gas under conditions derived by combination of the simple gas laws

standard conditions of temperature and pressure (STP)


273.15 K (0 °C) and 1 atm (101.325 kPa)

standard molar volume


volume of 1 mole of gas at STP, approximately 22.4 L for gases behaving ideally

This page titled 9.2: Relating Pressure, Volume, Amount, and Temperature - The Ideal Gas Law is shared under a CC BY 4.0 license and was authored, remixed, and/or curated by OpenStax via source content that was
edited to the style and standards of the LibreTexts platform; a detailed edit history is available upon request.

Access for free at OpenStax 9.2.8 https://chem.libretexts.org/@go/page/38202


Access for free at OpenStax 9.2.9 https://chem.libretexts.org/@go/page/38202
Access for free at OpenStax 9.2.10 https://chem.libretexts.org/@go/page/38202
Access for free at OpenStax 9.2.11 https://chem.libretexts.org/@go/page/38202
Access for free at OpenStax 9.2.12 https://chem.libretexts.org/@go/page/38202
9.3: Stoichiometry of Gaseous Substances, Mixtures, and Reactions
 Learning Objectives
Use the ideal gas law to compute gas densities and molar masses
Perform stoichiometric calculations involving gaseous substances
State Dalton’s law of partial pressures and use it in calculations involving gaseous mixtures

The study of the chemical behavior of gases was part of the basis of perhaps the most fundamental chemical revolution in history.
French nobleman Antoine Lavoisier, widely regarded as the “father of modern chemistry,” changed chemistry from a qualitative to
a quantitative science through his work with gases. He discovered the law of conservation of matter, discovered the role of oxygen
in combustion reactions, determined the composition of air, explained respiration in terms of chemical reactions, and more. He was
a casualty of the French Revolution, guillotined in 1794. Of his death, mathematician and astronomer Joseph-Louis Lagrange said,
“It took the mob only a moment to remove his head; a century will not suffice to reproduce it."
As described in an earlier chapter of this text, we can turn to chemical stoichiometry for answers to many of the questions that ask
“How much?” We can answer the question with masses of substances or volumes of solutions. However, we can also answer this
question another way: with volumes of gases. We can use the ideal gas equation to relate the pressure, volume, temperature, and
number of moles of a gas. Here we will combine the ideal gas equation with other equations to find gas density and molar mass.
We will deal with mixtures of different gases, and calculate amounts of substances in reactions involving gases. This section will
not introduce any new material or ideas, but will provide examples of applications and ways to integrate concepts we have already
discussed.

9.3.1: Density of a Gas


m
Recall that the density of a gas is its mass to volume ratio, ρ = . Therefore, if we can determine the mass of some volume of a
V
gas, we will get its density. The density of an unknown gas can used to determine its molar mass and thereby assist in its
identification. The ideal gas law, PV = nRT, provides us with a means of deriving such a mathematical formula to relate the density
of a gas to its volume in the proof shown in Example 9.3.1.

 Example 9.3.1: Derivation of a Density Formula from the Ideal Gas Law
Use PV = nRT to derive a formula for the density of gas in g/L

Solution
P V = nRT

Rearrange to get (mol/L):


n P
=
v RT

Multiply each side of the equation by the molar mass, ℳ. When moles are multiplied by ℳ in g/mol, g are obtained:
n P
(M) ( ) =( ) (M)
V RT

PM
M/V = ρ =
RT

 Exercise 9.3.1
A gas was found to have a density of 0.0847 g/L at 17.0 °C and a pressure of 760 torr. What is its molar mass? What is the gas?

Answer

Access for free at OpenStax 9.3.1 https://chem.libretexts.org/@go/page/38203


PM
ρ =
RT

1 atm M
0.0847 g/L = 760 torr × × × 290 K
760 torr 0.0821 L atm /mol K

ℳ = 2.02 g/mol; therefore, the gas must be hydrogen (H2, 2.02 g/mol)

We must specify both the temperature and the pressure of a gas when calculating its density because the number of moles of a gas
(and thus the mass of the gas) in a liter changes with temperature or pressure. Gas densities are often reported at STP.

 Example 9.3.2: Empirical/Molecular Formula Problems

Using the Ideal Gas Law and Density of a Gas Cyclopropane, a gas once used with oxygen as a general anesthetic, is
composed of 85.7% carbon and 14.3% hydrogen by mass. Find the empirical formula. If 1.56 g of cyclopropane occupies a
volume of 1.00 L at 0.984 atm and 50 °C, what is the molecular formula for cyclopropane?

Solution
Strategy:
First solve the empirical formula problem using methods discussed earlier. Assume 100 g and convert the percentage of each
element into grams. Determine the number of moles of carbon and hydrogen in the 100-g sample of cyclopropane. Divide by
the smallest number of moles to relate the number of moles of carbon to the number of moles of hydrogen. In the last step,
realize that the smallest whole number ratio is the empirical formula:
1 mol C 7.136
85.7 g C × = 7.136 mol C = 1.00 mol C
12.01 g C 7.136

1 mol H 14.158
14.3 g H × = 14.158 mol H = 1.98 mol H
1.01 g H 7.136

Empirical formula is CH2 [empirical mass (EM) of 14.03 g/empirical unit].


Next, use the density equation related to the ideal gas law to determine the molar mass:
PM 1.56 g M
d = = 0.984 atm × × 323 K
RT 1.00 L 0.0821 L atm/mol K

M 42.0
ℳ = 42.0 g/mol, = = 2.99 , so (3)(CH2) = C3H6 (molecular formula)
EM 14.03

 Exercise 9.3.2

Acetylene, a fuel used welding torches, is comprised of 92.3% C and 7.7% H by mass. Find the empirical formula. If 1.10 g of
acetylene occupies of volume of 1.00 L at 1.15 atm and 59.5 °C, what is the molecular formula for acetylene?

Answer
Empirical formula, CH; Molecular formula, C2H2

9.3.1.0.1: Molar Mass of a Gas


Another useful application of the ideal gas law involves the determination of molar mass. By definition, the molar mass of a
substance is the ratio of its mass in grams, m, to its amount in moles, n:
grams of substance m
M= =
moles of substance n

The ideal gas equation can be rearranged to isolate n:

Access for free at OpenStax 9.3.2 https://chem.libretexts.org/@go/page/38203


PV
n =
RT

and then combined with the molar mass equation to yield:


mRT
M=
PV

This equation can be used to derive the molar mass of a gas from measurements of its pressure, volume, temperature, and mass.

 Example 9.3.3: Determining the Molar Mass of a Volatile Liquid

The approximate molar mass of a volatile liquid can be determined by:


1. Heating a sample of the liquid in a flask with a tiny hole at the top, which converts the liquid into gas that may escape
through the hole
2. Removing the flask from heat at the instant when the last bit of liquid becomes gas, at which time the flask will be filled
with only gaseous sample at ambient pressure
3. Sealing the flask and permitting the gaseous sample to condense to liquid, and then weighing the flask to determine the
sample’s mass (Figure 9.3.1)

Figure 9.3.1 : When the volatile liquid in the flask is heated past its boiling point, it becomes gas and drives air out of the flask.
At tl ⟶g , the flask is filled with volatile liquid gas at the same pressure as the atmosphere. If the flask is then cooled to room
temperature, the gas condenses and the mass of the gas that filled the flask, and is now liquid, can be measured. (credit:
modification of work by Mark Ott)
Using this procedure, a sample of chloroform gas weighing 0.494 g is collected in a flask with a volume of 129 cm3 at 99.6 °C
when the atmospheric pressure is 742.1 mm Hg. What is the approximate molar mass of chloroform?

Solution
Since
m
M=
n

and
PV
n =
RT

substituting and rearranging gives


mRT
M=
PV

then
mRT (0.494 g) × 0.08206 L ⋅ atm/mol K × 372.8 K
M= = = 120 g/mol
PV 0.976 atm × 0.129 L

Access for free at OpenStax 9.3.3 https://chem.libretexts.org/@go/page/38203


 Exercise 9.3.3
A sample of phosphorus that weighs 3.243 × 10−2 g exerts a pressure of 31.89 kPa in a 56.0-mL bulb at 550 °C. What are the
molar mass and molecular formula of phosphorus vapor?

Answer
124 g/mol P4

9.3.1.0.1: The Pressure of a Mixture of Gases: Dalton’s Law


Unless they chemically react with each other, the individual gases in a mixture of gases do not affect each other’s pressure. Each
individual gas in a mixture exerts the same pressure that it would exert if it were present alone in the container ( Figure 9.3.2). The
pressure exerted by each individual gas in a mixture is called its partial pressure. This observation is summarized by Dalton’s law
of partial pressures: The total pressure of a mixture of ideal gases is equal to the sum of the partial pressures of the component
gases:

PT otal = PA + PB + PC +. . . = ∑ Pi

In the equation PTotal is the total pressure of a mixture of gases, PA is the partial pressure of gas A; PB is the partial pressure of gas
B; PC is the partial pressure of gas C; and so on.

Figure 9.3.2 : If equal-volume cylinders containing gas A at a pressure of 300 kPa, gas B at a pressure of 600 kPa, and gas C at a
pressure of 450 kPa are all combined in the same-size cylinder, the total pressure of the mixture is 1350 kPa.
This figure includes images of four gas-filled cylinders or tanks. Each has a valve at the top. The interior of the first cylinder is
shaded blue. This region contains 5 small blue circles that are evenly distributed. The label “300 k P a” is on the cylinder. The
second cylinder is shaded lavender. This region contains 8 small purple circles that are evenly distributed. The label “600 k P a” is
on the cylinder. To the right of these cylinders is a third cylinder. Its interior is shaded pale yellow. This region contains 12 small
yellow circles that are evenly distributed. The label “450 k P a” is on this region of the cylinder. An arrow labeled “Total pressure
combined” appears to the right of these three cylinders. This arrow points to a fourth cylinder. The interior of this cylinder is
shaded a pale green. It contains evenly distributed small circles in the following quantities and colors; 5 blue, 8 purple, and 12
yellow. This cylinder is labeled “1350 k P a.”
The partial pressure of gas A is related to the total pressure of the gas mixture via its mole fraction (X), a unit of concentration
defined as the number of moles of a component of a solution divided by the total number of moles of all components:
nA
PA = XA × PT otal where XA =
nT otal

where PA, XA, and nA are the partial pressure, mole fraction, and number of moles of gas A, respectively, and nTotal is the number of
moles of all components in the mixture.

 Example 9.3.2: The Pressure of a Mixture of Gases


A 10.0-L vessel contains 2.50 × 10−3 mol of H2, 1.00 × 10−3 mol of He, and 3.00 × 10−4 mol of Ne at 35 °C.
a. What are the partial pressures of each of the gases?
b. What is the total pressure in atmospheres?

Access for free at OpenStax 9.3.4 https://chem.libretexts.org/@go/page/38203


Solution
The gases behave independently, so the partial pressure of each gas can be determined from the ideal gas equation, using
nRT
P = :
V

−3 −1 −1
(2.50 × 10 mol)(0.08206 L atm mol K )(308 K )
−3
PH2 = = 6.32 × 10 atm
10.0 L

−3 −1 −1
(1.00 × 10 mol )(0.08206 L atm mol K )(308 K )
−3
PHe = = 2.53 × 10 atm
10.0 L

−4 −1 −1
(3.00 × 10 mol )(0.08206 L atm mol K )(308 K )
−4
PNe = = 7.58 × 10 atm
10.0 L

The total pressure is given by the sum of the partial pressures:


−3
PT = PH2 + PHe + PNe = (0.00632 + 0.00253 + 0.00076) atm = 9.61 × 10 atm

 Exercise 9.3.2

A 5.73-L flask at 25 °C contains 0.0388 mol of N2, 0.147 mol of CO, and 0.0803 mol of H2. What is the total pressure in the
flask in atmospheres?

Answer
1.137 atm

Here is another example of this concept, but dealing with mole fraction calculations.

 Example 9.3.3: The Pressure of a Mixture of Gases

A gas mixture used for anesthesia contains 2.83 mol oxygen, O2, and 8.41 mol nitrous oxide, N2O. The total pressure of the
mixture is 192 kPa.
a. What are the mole fractions of O2 and N2O?
b. What are the partial pressures of O2 and N2O?

Solution
The mole fraction is given by
nA
XA =
nT otal

and the partial pressure is

PA = XA × PT otal

For O2,
nO2 2.83mol
XO2 = = = 0.252
nT otal (2.83 + 8.41) mol

and

PO = XO × PT otal = 0.252 × 192 kPa = 48.4 kPa


2 2

For N2O,

Access for free at OpenStax 9.3.5 https://chem.libretexts.org/@go/page/38203


nN O 8.41 mol
2
XN2 O = = = 0.748
nT otal (2.83 + 8.41) mol

and

PN2 O = XN2 O × PT otal = (0.748) × 192 kPa = 143.6 kPa

 Exercise 9.3.3

What is the pressure of a mixture of 0.200 g of H2, 1.00 g of N2, and 0.820 g of Ar in a container with a volume of 2.00 L at 20
°C?

Answer
1.87 atm

9.3.1.0.1: Collection of Gases over Water


A simple way to collect gases that do not react with water is to capture them in a bottle that has been filled with water and inverted
into a dish filled with water. The pressure of the gas inside the bottle can be made equal to the air pressure outside by raising or
lowering the bottle. When the water level is the same both inside and outside the bottle (Figure 9.3.3), the pressure of the gas is
equal to the atmospheric pressure, which can be measured with a barometer.

Figure 9.3.3 : When a reaction produces a gas that is collected above water, the trapped gas is a mixture of the gas produced by the
reaction and water vapor. If the collection flask is appropriately positioned to equalize the water levels both within and outside the
flask, the pressure of the trapped gas mixture will equal the atmospheric pressure outside the flask (see the earlier discussion of
manometers).
This figure shows a diagram of equipment used for collecting a gas over water. To the left is an Erlenmeyer flask. It is
approximately two thirds full of a lavender colored liquid. Bubbles are evident in the liquid. The label “Reaction Producing Gas”
appears below the flask. A line segment connects this label to the liquid in the flask. The flask has a stopper in it through which a
single glass tube extends from the open region above the liquid in the flask up, through the stopper, to the right, then angles down
into a pan that is nearly full of light blue water. This tube again extends right once it is well beneath the water’s surface. It then
bends up into an inverted flask which is labeled “Collection Flask.” This collection flask is positioned with its mouth beneath the
surface of the light blue water and appears approximately half full. Bubbles are evident in the water in the inverted flask. The open
space above the water in the inverted flask is labeled “collected gas.”
However, there is another factor we must consider when we measure the pressure of the gas by this method. Water evaporates and
there is always gaseous water (water vapor) above a sample of liquid water. As a gas is collected over water, it becomes saturated
with water vapor and the total pressure of the mixture equals the partial pressure of the gas plus the partial pressure of the water
vapor. The pressure of the pure gas is therefore equal to the total pressure minus the pressure of the water vapor—this is referred to
as the “dry” gas pressure, that is, the pressure of the gas only, without water vapor.

Access for free at OpenStax 9.3.6 https://chem.libretexts.org/@go/page/38203


Figure 9.3.4 : This graph shows the vapor pressure of water at sea level as a function of temperature.
A graph is shown. The horizontal axis is labeled “Temperature ( degrees C )” with markings and labels provided for multiples of 20
beginning at 0 and ending at 100. The vertical axis is labeled “Vapor pressure ( torr )” with marking and labels provided for
multiples of 200, beginning at 0 and ending at 800. A smooth solid black curve extends from the origin up and to the right across
the graph. The graph shows a positive trend with an increasing rate of change. On the vertical axis is ( 7 60) and an arrow pointing
to it. The arrow is labeled, “Vapor pressure at ( 100 degrees C ).”

The vapor pressure of water, which is the pressure exerted by water vapor in equilibrium with liquid water in a closed container,
depends on the temperature (Figure 9.3.4); more detailed information on the temperature dependence of water vapor can be found
in Table 9.3.1, and vapor pressure will be discussed in more detail in the next chapter on liquids.
Table 9.3.1 : Vapor Pressure of Ice and Water in Various Temperatures at Sea Level
Temperature Temperature Temperature
Pressure (torr) Pressure (torr) Pressure (torr)
(°C) (°C) (°C)

–10 1.95 18 15.5 30 31.8

–5 3.0 19 16.5 35 42.2

–2 3.9 20 17.5 40 55.3

0 4.6 21 18.7 50 92.5

2 5.3 22 19.8 60 149.4

4 6.1 23 21.1 70 233.7

6 7.0 24 22.4 80 355.1

8 8.0 25 23.8 90 525.8

10 9.2 26 25.2 95 633.9

12 10.5 27 26.7 99 733.2

14 12.0 28 28.3 100.0 760.0

16 13.6 29 30.0 101.0 787.6

 Example 9.3.4: Pressure of a Gas Collected Over Water

If 0.200 L of argon is collected over water at a temperature of 26 °C and a pressure of 750 torr in a system like that shown in
Figure 9.3.3, what is the partial pressure of argon?

Solution
According to Dalton’s law, the total pressure in the bottle (750 torr) is the sum of the partial pressure of argon and the partial
pressure of gaseous water:

PT = PAr + PH2 O

Access for free at OpenStax 9.3.7 https://chem.libretexts.org/@go/page/38203


Rearranging this equation to solve for the pressure of argon gives:

PAr = PT − PH2 O

The pressure of water vapor above a sample of liquid water at 26 °C is 25.2 torr (Appendix E), so:

PAr = 750 torr − 25.2 torr = 725 torr

 Exercise 9.3.4
A sample of oxygen collected over water at a temperature of 29.0 °C and a pressure of 764 torr has a volume of 0.560 L. What
volume would the dry oxygen have under the same conditions of temperature and pressure?

Answer
0.583 L

9.3.1.0.1: Chemical Stoichiometry and Gases


Chemical stoichiometry describes the quantitative relationships between reactants and products in chemical reactions. We have
previously measured quantities of reactants and products using masses for solids and volumes in conjunction with the molarity for
solutions; now we can also use gas volumes to indicate quantities. If we know the volume, pressure, and temperature of a gas, we
can use the ideal gas equation to calculate how many moles of the gas are present. If we know how many moles of a gas are
involved, we can calculate the volume of a gas at any temperature and pressure.
9.3.1.0.1: Avogadro’s Law Revisited
Sometimes we can take advantage of a simplifying feature of the stoichiometry of gases that solids and solutions do not exhibit: All
gases that show ideal behavior contain the same number of molecules in the same volume (at the same temperature and pressure).
Thus, the ratios of volumes of gases involved in a chemical reaction are given by the coefficients in the equation for the reaction,
provided that the gas volumes are measured at the same temperature and pressure.
We can extend Avogadro’s law (that the volume of a gas is directly proportional to the number of moles of the gas) to chemical
reactions with gases: Gases combine, or react, in definite and simple proportions by volume, provided that all gas volumes are
measured at the same temperature and pressure. For example, since nitrogen and hydrogen gases react to produce ammonia gas
according to

N (g) + 3 H (g) ⟶ 2 NH (g)


2 2 3

a given volume of nitrogen gas reacts with three times that volume of hydrogen gas to produce two times that volume of ammonia
gas, if pressure and temperature remain constant.
The explanation for this is illustrated in Figure 9.3.4. According to Avogadro’s law, equal volumes of gaseous N2, H2, and NH3, at
the same temperature and pressure, contain the same number of molecules. Because one molecule of N2 reacts with three
molecules of H2 to produce two molecules of NH3, the volume of H2 required is three times the volume of N2, and the volume of
NH3 produced is two times the volume of N2.

Access for free at OpenStax 9.3.8 https://chem.libretexts.org/@go/page/38203


Figure 9.3.5 : One volume of N2 combines with three volumes of H2 to form two volumes of NH3.
This diagram provided models the chemical reaction written with formulas across the bottom of the figure. The reaction is written;
N subscript 2 plus 3 H subscript 2 followed by an arrow pointing right to N H subscript 3. Just above the formulas, space-filling
models are provided. Above N H subscript 2, two blue spheres are bonded. Above 3 H subscript 2, three pairs of two slightly
smaller white spheres are bonded. Above N H subscript 3, two molecules are shown composed each of a central blue sphere to
which three slightly smaller white spheres are bonded. Across the top of the diagram, the reaction is illustrated with balloons. To
the left is a light blue balloon which is labeled “N subscript 2”. This balloon contains a single space-filling model composed of two
bonded blue spheres. This balloon is followed by a plus sign, then three grey balloons which are each labeled “H subscript 2.” Each
of these balloons similarly contain a single space-filling model composed of two bonded white spheres. These white spheres are
slightly smaller than the blue spheres. An arrow follows which points right to two light green balloons which are each labeled “N H
subscript 3.” Each light green balloon contains a space-filling model composed of a single central blue sphere to which three
slightly smaller white spheres are bonded.

 Example 9.3.5: Reaction of Gases

Propane, C3H8(g), is used in gas grills to provide the heat for cooking. What volume of O2(g) measured at 25 °C and 760 torr is
required to react with 2.7 L of propane measured under the same conditions of temperature and pressure? Assume that the
propane undergoes complete combustion.

Solution
The ratio of the volumes of C3H8 and O2 will be equal to the ratio of their coefficients in the balanced equation for the reaction:
C H (g) + 5 O (g) ⟶ 3 CO (g) + 4 H O(l) (9.3.1)
3 8 2 2 2

1 volume + 5 volumes 3 volumes + 4 volumes (9.3.2)

From the equation, we see that one volume of C3H8 will react with five volumes of O2:
5 LO
2
2.7 L C3 H8 × = 13.5 L O
2
1 L C3 H8

A volume of 13.5 L of O2 will be required to react with 2.7 L of C3H8.

 Exercise 9.3.5

An acetylene tank for an oxyacetylene welding torch provides 9340 L of acetylene gas, C2H2, at 0 °C and 1 atm. How many
tanks of oxygen, each providing 7.00 × 103 L of O2 at 0 °C and 1 atm, will be required to burn the acetylene?

2C H +5 O ⟶ 4 CO +2 H O
2 2 2 2 2

Answer
3.34 tanks (2.34 × 104 L)

Access for free at OpenStax 9.3.9 https://chem.libretexts.org/@go/page/38203


 Example 9.3.6: Volumes of Reacting Gases

Ammonia is an important fertilizer and industrial chemical. Suppose that a volume of 683 billion cubic feet of gaseous
ammonia, measured at 25 °C and 1 atm, was manufactured. What volume of H2(g), measured under the same conditions, was
required to prepare this amount of ammonia by reaction with N2?

N (g) + 3 H (g) ⟶ 2 NH (g)


2 2 3

Solution
Because equal volumes of H2 and NH3 contain equal numbers of molecules and each three molecules of H2 that react produce
two molecules of NH3, the ratio of the volumes of H2 and NH3 will be equal to 3:2. Two volumes of NH3, in this case in units
of billion ft3, will be formed from three volumes of H2:
3
3 billion f t H2
3 3 3
683 billion f t NH3 × = 1.02 × 10 billion f t H2
3
2 billion f t NH3

The manufacture of 683 billion ft3 of NH3 required 1020 billion ft3 of H2. (At 25 °C and 1 atm, this is the volume of a cube
with an edge length of approximately 1.9 miles.)

 Exercise 9.3.6
What volume of O2(g) measured at 25 °C and 760 torr is required to react with 17.0 L of ethylene, C2H4(g), measured under
the same conditions of temperature and pressure? The products are CO2 and water vapor.

Answer
51.0 L

 Example 9.3.7: Volume of Gaseous Product

What volume of hydrogen at 27 °C and 723 torr may be prepared by the reaction of 8.88 g of gallium with an excess of
hydrochloric acid?

2 Ga(s) + 6 HCl(aq) ⟶ 2 GaCl (aq) + 3 H (g)


3 2

Solution
To convert from the mass of gallium to the volume of H2(g), we need to do something like this:

The first two conversions are:

1 mol Ga 3 mol H2
8.88 g Ga × × = 0.191 mol H2
69.723 g Ga 2 mol Ga

Finally, we can use the ideal gas law:


−1 −1
nRT 0.191 mol × 0.08206 L atm mol K × 300 K
VH2 = ( ) = = 4.94 L
P 0.951 atm
H2

Access for free at OpenStax 9.3.10 https://chem.libretexts.org/@go/page/38203


 Exercise 9.3.7

Sulfur dioxide is an intermediate in the preparation of sulfuric acid. What volume of SO2 at 343 °C and 1.21 atm is produced
by burning l.00 kg of sulfur in oxygen?

Answer
1.30 × 103 L

 Greenhouse Gases and Climate Change

The thin skin of our atmosphere keeps the earth from being an ice planet and makes it habitable. In fact, this is due to less than
1
0.5% of the air molecules. Of the energy from the sun that reaches the earth, almost is reflected back into space, with the
3
rest absorbed by the atmosphere and the surface of the earth. Some of the energy that the earth absorbs is re-emitted as infrared
(IR) radiation, a portion of which passes back out through the atmosphere into space. However, most of this IR radiation is
absorbed by certain substances in the atmosphere, known as greenhouse gases, which re-emit this energy in all directions,
trapping some of the heat. This maintains favorable living conditions—without atmosphere, the average global average
temperature of 14 °C (57 °F) would be about –19 °C (–2 °F). The major greenhouse gases (GHGs) are water vapor, carbon
dioxide, methane, and ozone. Since the Industrial Revolution, human activity has been increasing the concentrations of GHGs,
which have changed the energy balance and are significantly altering the earth’s climate (Figure 9.3.6).

Figure 9.3.6 : Greenhouse gases trap enough of the sun’s energy to make the planet habitable—this is known as the greenhouse
effect. Human activities are increasing greenhouse gas levels, warming the planet and causing more extreme weather events.
There is strong evidence from multiple sources that higher atmospheric levels of CO2 are caused by human activity, with fossil
3
fuel burning accounting for about of the recent increase in CO2. Reliable data from ice cores reveals that CO2 concentration
4
in the atmosphere is at the highest level in the past 800,000 years; other evidence indicates that it may be at its highest level in
20 million years. In recent years, the CO2 concentration has increased from historical levels of below 300 ppm to almost 400
ppm today (Figure 9.3.7).

Access for free at OpenStax 9.3.11 https://chem.libretexts.org/@go/page/38203


Figure 9.3.7 : Figure CO2 levels over the past 700,000 years were typically from 200–300 ppm, with a steep, unprecedented
increase over the past 50 years.

Summary
The ideal gas law can be used to derive a number of convenient equations relating directly measured quantities to properties of
interest for gaseous substances and mixtures. Appropriate rearrangement of the ideal gas equation may be made to permit the
calculation of gas densities and molar masses. Dalton’s law of partial pressures may be used to relate measured gas pressures for
gaseous mixtures to their compositions. Avogadro’s law may be used in stoichiometric computations for chemical reactions
involving gaseous reactants or products.
9.3.1.0.1: Key Equations
PTotal = PA + PB + PC + … = ƩiPi
PA = XA PTotal
nA
XA =
nT otal

9.3.1.0.1: Footnotes
1. “Quotations by Joseph-Louis Lagrange,” last modified February 2006, accessed February 10, 2015, www-history.mcs.st-
andrews.ac.../Lagrange.html

Summary
Dalton’s law of partial pressures
total pressure of a mixture of ideal gases is equal to the sum of the partial pressures of the component gases.

mole fraction (X)


concentration unit defined as the ratio of the molar amount of a mixture component to the total number of moles of all mixture
components

partial pressure
pressure exerted by an individual gas in a mixture

vapor pressure of water


pressure exerted by water vapor in equilibrium with liquid water in a closed container at a specific temperature

This page titled 9.3: Stoichiometry of Gaseous Substances, Mixtures, and Reactions is shared under a CC BY 4.0 license and was authored,
remixed, and/or curated by OpenStax via source content that was edited to the style and standards of the LibreTexts platform; a detailed edit
history is available upon request.

Access for free at OpenStax 9.3.12 https://chem.libretexts.org/@go/page/38203


Access for free at OpenStax 9.3.13 https://chem.libretexts.org/@go/page/38203
Access for free at OpenStax 9.3.14 https://chem.libretexts.org/@go/page/38203
Access for free at OpenStax 9.3.15 https://chem.libretexts.org/@go/page/38203
Access for free at OpenStax 9.3.16 https://chem.libretexts.org/@go/page/38203
Access for free at OpenStax 9.3.17 https://chem.libretexts.org/@go/page/38203
9.4: Effusion and Diffusion of Gases
 Learning Objectives
Define and explain effusion and diffusion
State Graham’s law and use it to compute relevant gas properties

If you have ever been in a room when a piping hot pizza was delivered, you have been made aware of the fact that gaseous
molecules can quickly spread throughout a room, as evidenced by the pleasant aroma that soon reaches your nose. Although
gaseous molecules travel at tremendous speeds (hundreds of meters per second), they collide with other gaseous molecules and
travel in many different directions before reaching the desired target. At room temperature, a gaseous molecule will experience
billions of collisions per second. The mean free path is the average distance a molecule travels between collisions. The mean free
path increases with decreasing pressure; in general, the mean free path for a gaseous molecule will be hundreds of times the
diameter of the molecule
In general, we know that when a sample of gas is introduced to one part of a closed container, its molecules very quickly disperse
throughout the container; this process by which molecules disperse in space in response to differences in concentration is called
diffusion (shown in Figure 9.4.1). The gaseous atoms or molecules are, of course, unaware of any concentration gradient, they
simply move randomly—regions of higher concentration have more particles than regions of lower concentrations, and so a net
movement of species from high to low concentration areas takes place. In a closed environment, diffusion will ultimately result in
equal concentrations of gas throughout, as depicted in Figure 9.4.1. The gaseous atoms and molecules continue to move, but since
their concentrations are the same in both bulbs, the rates of transfer between the bulbs are equal (no net transfer of molecules
occurs).

Figure 9.4.1 : (a) Two gases, H2 and O2, are initially separated. (b) When the stopcock is opened, they mix together. The lighter gas,
H2, passes through the opening faster than O2, so just after the stopcock is opened, more H2 molecules move to the O2 side than O2
molecules move to the H2 side. (c) After a short time, both the slower-moving O2 molecules and the faster-moving H2 molecules
have distributed themselves evenly on both sides of the vessel.
In this figure, three pairs of gas filled spheres or vessels are shown connected with a stopcock between them. In a, the figure is
labeled, “Stopcock closed.” Above, the left sphere is labeled, “H subscript 2.” It contains approximately 30 small, white, evenly
distributed circles. The sphere to its right is labeled, “O subscript 2.” It contains approximately 30 small red evenly distributed
circles. In b, the figure is labeled, “Stopcock open.” The stopcock valve handle is now parallel to the tube connecting the two
spheres. On the left, approximately 9 small, white circles and 4 small, red circles are present, with the red spheres appearing
slightly closer to the stopcock. On the right side, approximately 25 small, red spheres and 21 small, white spheres are present, with
the concentration of white spheres slightly greater near the stopcock. In c, the figure is labeled “Some time after Stopcock open.” In
this situation, the red and white spheres appear evenly mixed and uniformly distributed throughout both spheres.
We are often interested in the rate of diffusion, the amount of gas passing through some area per unit time:
amount of gas passing through an area
rate of diffusion =
unit of time

The diffusion rate depends on several factors: the concentration gradient (the increase or decrease in concentration from one point
to another); the amount of surface area available for diffusion; and the distance the gas particles must travel. Note also that the time
required for diffusion to occur is inversely proportional to the rate of diffusion, as shown in the rate of diffusion equation.
A process involving movement of gaseous species similar to diffusion is effusion, the escape of gas molecules through a tiny hole
such as a pinhole in a balloon into a vacuum (Figure 9.4.1). Although diffusion and effusion rates both depend on the molar mass
of the gas involved, their rates are not equal; however, the ratios of their rates are the same.

Access for free at OpenStax 9.4.1 https://chem.libretexts.org/@go/page/38204


Figure 9.4.2 : Diffusion occurs when gas molecules disperse throughout a container. Effusion occurs when a gas passes through an
opening that is smaller than the mean free path of the particles, that is, the average distance traveled between collisions. Effectively,
this means that only one particle passes through at a time.
This figure contains two cylindrical containers which are oriented horizontally. The first is labeled “Diffusion.” In this container,
approximately 25 purple and 25 green circles are shown, evenly distributed throughout the container. “Trails” behind some of the
circles indicate motion. In the second container, which is labeled “Effusion,” a boundary layer is evident across the center of the
cylindrical container, dividing the cylinder into two halves. A black arrow is drawn pointing through this boundary from left to
right. To the left of the boundary, approximately 16 green circles and 20 purple circles are shown again with motion indicated by
“trails” behind some of the circles. To the right of the boundary, only 4 purple and 16 green circles are shown.

If a mixture of gases is placed in a container with porous walls, the gases effuse through the small openings in the walls. The lighter
gases pass through the small openings more rapidly (at a higher rate) than the heavier ones (Figure 9.4.2). In 1832, Thomas
Graham studied the rates of effusion of different gases and formulated Graham’s law of effusion: The rate of effusion of a gas is
inversely proportional to the square root of the mass of its particles:
1
rate of effusion ∝ −−
√M

This means that if two gases A and B are at the same temperature and pressure, the ratio of their effusion rates is inversely
proportional to the ratio of the square roots of the masses of their particles:
−−−
rate of effusion of B √MA
= −−−
rate of effusion of A √MB

Figure 9.4.3 : A balloon filled with air (the blue one) remains full overnight. A balloon filled with helium (the green one) partially
deflates because the smaller, light helium atoms effuse through small holes in the rubber much more readily than the heavier
molecules of nitrogen and oxygen found in air. (credit: modification of work by Mark Ott)
This figure shows two photos. The first photo shows a blue balloon which floats above a green balloon. The green balloon is
resting on a surface. Both balloons are about the same size. The second photo shows the same two balloons, but the blue one is now
smaller than the green one. Both are resting on a surface.

 Example 9.4.1: Applying Graham’s Law to Rates of Effusion


Calculate the ratio of the rate of effusion of hydrogen to the rate of effusion of oxygen.

Solution
From Graham’s law, we have:

Access for free at OpenStax 9.4.2 https://chem.libretexts.org/@go/page/38204


−−−−−−−−−
√1.43 g L−1
rate of effusion of hydrogen 1.20 4
= = =
−−−−−−−−−−−
rate of effusion of oxygen 0.300 1
√0.0899 g L−1

Using molar masses:


−1 −−
32 g mol √16
rate of effusion of hydrogen 4
= = – =
rate of effusion of oxygen −1 √1 1
2 g mol

Hydrogen effuses four times as rapidly as oxygen.

 Exercise 9.4.1

At a particular pressure and temperature, nitrogen gas effuses at the rate of 79 mL/s. Using the same apparatus at the same
temperature and pressure, at what rate will sulfur dioxide effuse?

Answer
52 mL/s

Here’s another example, making the point about how determining times differs from determining rates.

 Example 9.4.2: Effusion Time Calculations

It takes 243 s for 4.46 × 10−5 mol Xe to effuse through a tiny hole. Under the same conditions, how long will it take 4.46 ×
10−5 mol Ne to effuse?

Solution
It is important to resist the temptation to use the times directly, and to remember how rate relates to time as well as how it
relates to mass. Recall the definition of rate of effusion:
amount of gas transferred
rate of effusion =
time

and combine it with Graham’s law:


−−−−
rate of effusion of gas Xe √MNe
= −−−−
rate of effusion of gas Ne √MXe

To get:
amount of Xe transferred
−−−−
√MNe
time for Xe
= −−−−
amount of Ne transferred √MXe

time for Ne

Noting that amount of A = amount of B, and solving for time for Ne:

amount of Xe
−−−− −−−−
time for Ne √MNe √MNe
time for Xe
= = =
−−−− −−−−
time for Xe √MXe √MXe
amount of Ne

time for Ne

and substitute values:

Access for free at OpenStax 9.4.3 https://chem.libretexts.org/@go/page/38204


−−−−−−−−− −

time f or Ne  20.2 g mol
= = 0.392
243 s ⎷
131.3 g mol

Finally, solve for the desired quantity:

time f or Ne = 0.392 × 243 s = 95.3 s

Note that this answer is reasonable: Since Ne is lighter than Xe, the effusion rate for Ne will be larger than that for Xe, which
means the time of effusion for Ne will be smaller than that for Xe.

 Exercise 9.4.2
2
A party balloon filled with helium deflates to of its original volume in 8.0 hours. How long will it take an identical balloon
3
1
filled with the same number of moles of air (ℳ = 28.2 g/mol) to deflate to of its original volume?
2

Answer
32 h

Finally, here is one more example showing how to calculate molar mass from effusion rate data.

 Example 9.4.3: Determining Molar Mass Using Graham’s Law

An unknown gas effuses 1.66 times more rapidly than CO2. What is the molar mass of the unknown gas? Can you make a
reasonable guess as to its identity?

Solution
From Graham’s law, we have:
−−−−−
rate of ef f usion of Unknown √MCO2
= −−−−−−−−
rate of ef f usion of CO2 √MUnknown

Plug in known data:


−−−−− −−−−
1.66 √44.0 g/mol
=
−−−−−−−−
1 √MUnknown

Solve:
44.0 g/mol
MUnknown = = 16.0 g/mol
2
(1.66)

The gas could well be CH4, the only gas with this molar mass.

 Exercise
Hydrogen gas effuses through a porous container 8.97-times faster than an unknown gas. Estimate the molar mass of the
unknown gas.

Answer
163 g/mol

Access for free at OpenStax 9.4.4 https://chem.libretexts.org/@go/page/38204


 Application: Use of Diffusion for Uranium Enrichment

Gaseous diffusion has been used to produce enriched uranium for use in nuclear power plants and weapons. Naturally
occurring uranium contains only 0.72% of 235U, the kind of uranium that is “fissile,” that is, capable of sustaining a nuclear
fission chain reaction. Nuclear reactors require fuel that is 2–5% 235U, and nuclear bombs need even higher concentrations.
One way to enrich uranium to the desired levels is to take advantage of Graham’s law. In a gaseous diffusion enrichment plant,
uranium hexafluoride (UF6, the only uranium compound that is volatile enough to work) is slowly pumped through large
cylindrical vessels called diffusers, which contain porous barriers with microscopic openings. The process is one of diffusion
because the other side of the barrier is not evacuated. The 235UF6 molecules have a higher average speed and diffuse through
the barrier a little faster than the heavier 238UF6 molecules. The gas that has passed through the barrier is slightly enriched in
235UF and the residual gas is slightly depleted. The small difference in molecular weights between 235UF and 238UF only
6 6 6
about 0.4% enrichment, is achieved in one diffuser (Figure 9.4.4). But by connecting many diffusers in a sequence of stages
(called a cascade), the desired level of enrichment can be attained.

Figure 9.4.4 : In a diffuser, gaseous UF6 is pumped through a porous barrier, which partially separates 235UF6 from 238UF6 The
UF6 must pass through many large diffuser units to achieve sufficient enrichment in 235U.
This figure shows a large cylindrical container oriented horizontally. A narrow tube or pipe which is labeled “porous barrier”
runs horizontally through the center of the tube and extends a short distance out from the left and right ends of the cylinder. At
the far left, an arrow points right into the tube. This arrow is labele, “Uranium hexafluoride ( U F subscript 6 ).” A line segment
connects the label, “High pressure feed tube,” to the tube where it enters the cylinder. In the short region of tube outside the
cylinder, 5 small, purple circles and 4 small, green circles are present. Inside the cylinder, an arrow points right through the
tube which contains many evenly distributed, purple circles and a handful of green circles which decrease in quantity moving
left to right through the cylinder. Curved arrows extend from the inner area of the tube into the outer region of the cylinder.
Three of these arrows point into the area above the tube and three point into the area below. Two line segments extend from the
label, “Higher speed superscript 235 U F subscript 6 diffuses through barrier faster than superscript 238 U F subscript 6,” to
two green circles in the space above the tube. In the short section of tubing just outside the cylinder, 8 small, purple circles are
present. An arrow labeled, “Depleted superscript 238 U F subscript 6,” points right extending from the end of this tube. The
larger space outside the tube contains approximately 100 evenly distributed small green circles and only 5 purple circles. Eight
of the purple circles appear at the left end of the cylinder. A tube exits the lower right end of the cylinder. It has 5 green circles
followed by a right pointing arrow and the label, “Enriched superscript 235 U F subscript 6.”
The large scale separation of gaseous 235UF6 from 238UF6 was first done during the World War II, at the atomic energy
installation in Oak Ridge, Tennessee, as part of the Manhattan Project (the development of the first atomic bomb). Although
the theory is simple, this required surmounting many daunting technical challenges to make it work in practice. The barrier
must have tiny, uniform holes (about 10–6 cm in diameter) and be porous enough to produce high flow rates. All materials (the
barrier, tubing, surface coatings, lubricants, and gaskets) need to be able to contain, but not react with, the highly reactive and
corrosive UF6.
Because gaseous diffusion plants require very large amounts of energy (to compress the gas to the high pressures required and
drive it through the diffuser cascade, to remove the heat produced during compression, and so on), it is now being replaced by
gas centrifuge technology, which requires far less energy. A current hot political issue is how to deny this technology to Iran, to
prevent it from producing enough enriched uranium for them to use to make nuclear weapons.

Access for free at OpenStax 9.4.5 https://chem.libretexts.org/@go/page/38204


Summary
Gaseous atoms and molecules move freely and randomly through space. Diffusion is the process whereby gaseous atoms and
molecules are transferred from regions of relatively high concentration to regions of relatively low concentration. Effusion is a
similar process in which gaseous species pass from a container to a vacuum through very small orifices. The rates of effusion of
gases are inversely proportional to the square roots of their densities or to the square roots of their atoms/molecules’ masses
(Graham’s law).
9.4.0.0.1: Key Equations
amount of gas passing through an area
rate of diffusion =
unit of time
−−
− −− −
rate of effusion of gas A √mB √MB
= = −− −
−−

rate of effusion of gas B √mA √M A

Summary
diffusion
movement of an atom or molecule from a region of relatively high concentration to one of relatively low concentration
(discussed in this chapter with regard to gaseous species, but applicable to species in any phase)

effusion
transfer of gaseous atoms or molecules from a container to a vacuum through very small openings

Graham’s law of effusion


rates of diffusion and effusion of gases are inversely proportional to the square roots of their molecular masses

mean free path


average distance a molecule travels between collisions

rate of diffusion
amount of gas diffusing through a given area over a given time

This page titled 9.4: Effusion and Diffusion of Gases is shared under a CC BY 4.0 license and was authored, remixed, and/or curated by OpenStax
via source content that was edited to the style and standards of the LibreTexts platform; a detailed edit history is available upon request.

Access for free at OpenStax 9.4.6 https://chem.libretexts.org/@go/page/38204


Access for free at OpenStax 9.4.7 https://chem.libretexts.org/@go/page/38204
Access for free at OpenStax 9.4.8 https://chem.libretexts.org/@go/page/38204
9.5: The Kinetic-Molecular Theory
 Learning Objectives
State the postulates of the kinetic-molecular theory
Use this theory’s postulates to explain the gas laws

The gas laws that we have seen to this point, as well as the ideal gas equation, are empirical, that is, they have been derived from
experimental observations. The mathematical forms of these laws closely describe the macroscopic behavior of most gases at
pressures less than about 1 or 2 atm. Although the gas laws describe relationships that have been verified by many experiments,
they do not tell us why gases follow these relationships.
The kinetic molecular theory (KMT) is a simple microscopic model that effectively explains the gas laws described in previous
modules of this chapter. This theory is based on the following five postulates described here. (Note: The term “molecule” will be
used to refer to the individual chemical species that compose the gas, although some gases are composed of atomic species, for
example, the noble gases.)
1. Gases are composed of molecules that are in continuous motion, travelling in straight lines and changing direction only when
they collide with other molecules or with the walls of a container.
2. The molecules composing the gas are negligibly small compared to the distances between them.
3. The pressure exerted by a gas in a container results from collisions between the gas molecules and the container walls.
4. Gas molecules exert no attractive or repulsive forces on each other or the container walls; therefore, their collisions are elastic
(do not involve a loss of energy).
5. The average kinetic energy of the gas molecules is proportional to the kelvin temperature of the gas.
The test of the KMT and its postulates is its ability to explain and describe the behavior of a gas. The various gas laws can be
derived from the assumptions of the KMT, which have led chemists to believe that the assumptions of the theory accurately
represent the properties of gas molecules. We will first look at the individual gas laws (Boyle’s, Charles’s, Amontons’s,
Avogadro’s, and Dalton’s laws) conceptually to see how the KMT explains them. Then, we will more carefully consider the
relationships between molecular masses, speeds, and kinetic energies with temperature, and explain Graham’s law.

9.5.1: The Kinetic-Molecular Theory Explains the Behavior of Gases, Part I


Recalling that gas pressure is exerted by rapidly moving gas molecules and depends directly on the number of molecules hitting a
unit area of the wall per unit of time, we see that the KMT conceptually explains the behavior of a gas as follows:
Amontons’s law. If the temperature is increased, the average speed and kinetic energy of the gas molecules increase. If the
volume is held constant, the increased speed of the gas molecules results in more frequent and more forceful collisions with the
walls of the container, therefore increasing the pressure (Figure 9.5.1a).
Charles’s law. If the temperature of a gas is increased, a constant pressure may be maintained only if the volume occupied by
the gas increases. This will result in greater average distances traveled by the molecules to reach the container walls, as well as
increased wall surface area. These conditions will decrease both the frequency of molecule-wall collisions and the number of
collisions per unit area, the combined effects of which balance the effect of increased collision forces due to the greater kinetic
energy at the higher temperature.
Boyle’s law. If the gas volume is decreased, the container wall area decreases and the molecule-wall collision frequency
increases, both of which increase the pressure exerted by the gas (Figure 9.5.1b).
Avogadro’s law. At constant pressure and temperature, the frequency and force of molecule-wall collisions are constant. Under
such conditions, increasing the number of gaseous molecules will require a proportional increase in the container volume in
order to yield a decrease in the number of collisions per unit area to compensate for the increased frequency of collisions
(Figure 9.5.1c).
Dalton’s Law. Because of the large distances between them, the molecules of one gas in a mixture bombard the container walls
with the same frequency whether other gases are present or not, and the total pressure of a gas mixture equals the sum of the
(partial) pressures of the individual gases.

Access for free at OpenStax 9.5.1 https://chem.libretexts.org/@go/page/38205


Figure 9.5.1 : (a) When gas temperature increases, gas pressure increases due to increased force and frequency of molecular
collisions. (b) When volume decreases, gas pressure increases due to increased frequency of molecular collisions. (c) When the
amount of gas increases at a constant pressure, volume increases to yield a constant number of collisions per unit wall area per unit
time.
This figure shows 3 pairs of pistons and cylinders. In a, which is labeled, “Charles’s Law,” the piston is positioned for the first
cylinder so that just over half of the available volume contains 6 purple spheres with trails behind them. The trails indicate
movement. Orange dashes extend from the interior surface of the cylinder where the spheres have collided. This cylinder is labeled,
“Baseline.” In the second cylinder, the piston is in the same position, and the label, “Heat” is indicated in red capitalized text. Four
red arrows with wavy stems are pointing upward to the base of the cylinder. The six purple spheres have longer trails behind them
and the number of orange dashes indicating points of collision with the container walls has increased. A rectangle beneath the
diagram states, “Temperature increased, Volume constant equals Increased pressure.” In b, which is labeled, “Boyle’s Law,” the
first, baseline cylinder shown is identical to the first cylinder in a. In the second cylinder, the piston has been moved, decreasing the
volume available to the 6 purple spheres to half of the initial volume. The six purple spheres have longer trails behind them and the
number of orange dashes indicating points of collision with the container walls has increased. This second cylinder is labeled,
“Volume decreased.” A rectangle beneath the diagram states, “Volume decreased, Wall area decreased equals Increased pressure.”
In c, which is labeled “Avogadro’s Law,” the first, baseline cylinder shown is identical to the first cylinder in a. In the second
cylinder, the number of purple spheres has changed from 6 to 12 and volume has doubled. This second cylinder is labeled
“Increased gas.” A rectangle beneath the diagram states, “At constant pressure, More gas molecules added equals Increased
volume.”

9.5.2: Molecular Velocities and Kinetic Energy


The previous discussion showed that the KMT qualitatively explains the behaviors described by the various gas laws. The
postulates of this theory may be applied in a more quantitative fashion to derive these individual laws. To do this, we must first
look at velocities and kinetic energies of gas molecules, and the temperature of a gas sample.
In a gas sample, individual molecules have widely varying speeds; however, because of the vast number of molecules and
collisions involved, the molecular speed distribution and average speed are constant. This molecular speed distribution is known as
a Maxwell-Boltzmann distribution, and it depicts the relative numbers of molecules in a bulk sample of gas that possesses a given
speed (Figure 9.5.2).

Access for free at OpenStax 9.5.2 https://chem.libretexts.org/@go/page/38205


Figure 9.5.2 : The molecular speed distribution for oxygen gas at 300 K is shown here. Very few molecules move at either very low
or very high speeds. The number of molecules with intermediate speeds increases rapidly up to a maximum, which is the most
probable speed, then drops off rapidly. Note that the most probable speed, νp, is a little less than 400 m/s, while the root mean
square speed, urms, is closer to 500 m/s.
The kinetic energy (KE) of a particle of mass (m) and speed (u) is given by:
1
2
KE = mu
2

Expressing mass in kilograms and speed in meters per second will yield energy values in units of joules (J = kg m2 s–2). To deal
with a large number of gas molecules, we use averages for both speed and kinetic energy. In the KMT, the root mean square
velocity of a particle, urms, is defined as the square root of the average of the squares of the velocities with n = the number of
particles:
−−−−−−−−−−−−−−−−−−−
−− 2 2 2 2
¯
¯¯¯
¯ u +u +u +u +…
√ 2 1 2 3 4
urms = u =√
n

The average kinetic energy, KEavg, is then equal to:


1
2
KEavg = m urms
2

The KEavg of a collection of gas molecules is also directly proportional to the temperature of the gas and may be described by the
equation:
3
KEavg = RT
2

where R is the gas constant and T is the kelvin temperature. When used in this equation, the appropriate form of the gas constant is
8.314 J mol-1K-1 (8.314 kg m2s–2mol-1K–1). These two separate equations for KEavg may be combined and rearranged to yield a
relation between molecular speed and temperature:
1 3
2
m urms = RT
2 2

−−−−−
3RT
urms = √ (9.5.1)
m

 Example 9.5.1: Calculation of urms

Calculate the root-mean-square velocity for a nitrogen molecule at 30 °C.

Solution
Convert the temperature into Kelvin:

30°C + 273 = 303 K

Determine the mass of a nitrogen molecule in kilograms:

Access for free at OpenStax 9.5.3 https://chem.libretexts.org/@go/page/38205


28.0 g
1 kg
× = 0.028 kg/mol
1 mol 1000 g

Replace the variables and constants in the root-mean-square velocity formula (Equation 9.5.1 ), replacing Joules with the
equivalent kg m2s–2:
−−−−−
3RT
urms = √
m
−−−−−−−−−−−−−−−−−−−−−
3(8.314 J/mol K)(303 K)
urms = √
(0.028 kg/mol)

− −−−− −−−− −−− −−


5 2 −2
= √ 2.70 × 10 m s

= 519 m/s

 Exercise 9.5.1

Calculate the root-mean-square velocity for an oxygen molecule at –23 °C.

Answer
441 m/s

If the temperature of a gas increases, its KEavg increases, more molecules have higher speeds and fewer molecules have lower
speeds, and the distribution shifts toward higher speeds overall, that is, to the right. If temperature decreases, KEavg decreases, more
molecules have lower speeds and fewer molecules have higher speeds, and the distribution shifts toward lower speeds overall, that
is, to the left. This behavior is illustrated for nitrogen gas in Figure 9.5.3.

Figure 9.5.3 : The molecular speed distribution for nitrogen gas (N2) shifts to the right and flattens as the temperature increases; it
shifts to the left and heightens as the temperature decreases.
A graph with four positively or right-skewed curves of varying heights is shown. The horizontal axis is labeled, “Velocity v ( m
divided by s ).” This axis is marked by increments of 500 beginning at 0 and extending up to 1500. The vertical axis is labeled,
“Fraction of molecules.” The label, “N subscript 2,” appears in the open space in the upper right area of the graph. The tallest and
narrowest of these curves is labeled, “100 K.” Its right end appears to touch the horizontal axis around 700 m per s. It is followed
by a slightly wider curve which is labeled, “200 K,” that is about three quarters of the height of the initial curve. Its right end
appears to touch the horizontal axis around 850 m per s. The third curve is significantly wider and only about half the height of the
initial curve. It is labeled, “500 K.” Its right end appears to touch the horizontal axis around 1450 m per s. The final curve is only
about one third the height of the initial curve. It is much wider than the others, so much so that its right end has not yet reached the
horizontal axis. This curve is labeled, “1000 K.”
At a given temperature, all gases have the same KEavg for their molecules. Gases composed of lighter molecules have more high-
speed particles and a higher urms, with a speed distribution that peaks at relatively higher velocities. Gases consisting of heavier
molecules have more low-speed particles, a lower urms, and a speed distribution that peaks at relatively lower velocities. This trend
is demonstrated by the data for a series of noble gases shown in Figure 9.5.4.

Access for free at OpenStax 9.5.4 https://chem.libretexts.org/@go/page/38205


Figure 9.5.4 : Molecular velocity is directly related to molecular mass. At a given temperature, lighter molecules move faster on
average than heavier molecules.
A graph is shown with four positively or right-skewed curves of varying heights. The horizontal axis is labeled, “Velocity v ( m
divided by s ).” This axis is marked by increments of 500 beginning at 0 and extending up to 3000. The vertical axis is labeled,
“Fraction of molecules.” The tallest and narrowest of these curves is labeled, “X e.” Its right end appears to touch the horizontal
axis around 600 m per s. It is followed by a slightly wider curve which is labeled, “A r,” that is about half the height of the initial
curve. Its right end appears to touch the horizontal axis around 900 m per s. The third curve is significantly wider and just over a
third of the height of the initial curve. It is labeled, “N e.” Its right end appears to touch the horizontal axis around 1200 m per s.
The final curve is only about one fourth the height of the initial curve. It is much wider than the others, so much so that its right
reaches the horizontal axis around 2500 m per s. This curve is labeled, “H e.”

The gas simulator may be used to examine the effect of temperature on molecular
velocities. Examine the simulator’s “energy histograms” (molecular speed distributions)
and “species information” (which gives average speed values) for molecules of different
masses at various temperatures.

9.5.3: The Kinetic-Molecular Theory Explains the Behavior of Gases, Part II


According to Graham’s law, the molecules of a gas are in rapid motion and the molecules themselves are small. The average
distance between the molecules of a gas is large compared to the size of the molecules. As a consequence, gas molecules can move
past each other easily and diffuse at relatively fast rates.
The rate of effusion of a gas depends directly on the (average) speed of its molecules:

effusion rate ∝ urms

Using this relation, and the equation relating molecular speed to mass, Graham’s law may be easily derived as shown here:
−−−−−
3RT
urms = √
m

3RT 3RT
m = =
2 2
urms ¯¯
u
¯

−−−−−
3RT

−−−

ef f usion rate A urms A
mA mB
= = −−−−− =√
ef f usion rate B urms B 3RT mA

mB

The ratio of the rates of effusion is thus derived to be inversely proportional to the ratio of the square roots of their masses. This is
the same relation observed experimentally and expressed as Graham’s law.

Summary
The kinetic molecular theory is a simple but very effective model that effectively explains ideal gas behavior. The theory assumes
that gases consist of widely separated molecules of negligible volume that are in constant motion, colliding elastically with one
another and the walls of their container with average velocities determined by their absolute temperatures. The individual

Access for free at OpenStax 9.5.5 https://chem.libretexts.org/@go/page/38205


molecules of a gas exhibit a range of velocities, the distribution of these velocities being dependent on the temperature of the gas
and the mass of its molecules.

9.5.4: Key Equations


−−−−−−−−−−−−−−−−−−−
−− 2 2 2 2
¯
¯¯¯
¯
u +u +u +u +…
1 2 3 4
√ 2
urms = u =√
n

3
KEavg = RT
2
−−−−−
3RT
urms =√
m

Summary
kinetic molecular theory
theory based on simple principles and assumptions that effectively explains ideal gas behavior

root mean square velocity (urms)


measure of average velocity for a group of particles calculated as the square root of the average squared velocity

This page titled 9.5: The Kinetic-Molecular Theory is shared under a CC BY 4.0 license and was authored, remixed, and/or curated by OpenStax
via source content that was edited to the style and standards of the LibreTexts platform; a detailed edit history is available upon request.

Access for free at OpenStax 9.5.6 https://chem.libretexts.org/@go/page/38205


9.6: Non-Ideal Gas Behavior
 Learning Objectives
Describe the physical factors that lead to deviations from ideal gas behavior
Explain how these factors are represented in the van der Waals equation
Define compressibility (Z) and describe how its variation with pressure reflects non-ideal behavior
Quantify non-ideal behavior by comparing computations of gas properties using the ideal gas law and the van der Waals
equation

Thus far, the ideal gas law, PV = nRT, has been applied to a variety of different types of problems, ranging from reaction
stoichiometry and empirical and molecular formula problems to determining the density and molar mass of a gas. As mentioned in
the previous modules of this chapter, however, the behavior of a gas is often non-ideal, meaning that the observed relationships
between its pressure, volume, and temperature are not accurately described by the gas laws. In this section, the reasons for these
deviations from ideal gas behavior are considered.
One way in which the accuracy of PV = nRT can be judged is by comparing the actual volume of 1 mole of gas (its molar volume,
Vm) to the molar volume of an ideal gas at the same temperature and pressure. This ratio is called the compressibility factor (Z)
with:
molar volume of gas at same T and P P Vm
Z = =( )
molar volume of ideal gas at same T and P RT
measured

Ideal gas behavior is therefore indicated when this ratio is equal to 1, and any deviation from 1 is an indication of non-ideal
behavior. Figure 9.6.1 shows plots of Z over a large pressure range for several common gases.

Figure 9.6.1 : A graph of the compressibility factor (Z) vs. pressure shows that gases can exhibit significant deviations from the
behavior predicted by the ideal gas law.
A graph is shown. The horizontal axis is labeled, “P ( a t m ).” Its scale begins at zero with markings provided by multiples of 200
up to 1000. The vertical axis is labeled, “Z le( k P a ).” This scale begins at zero and includes multiples of 0.5 up to 2.0. Six curves
are drawn of varying colors. One of these curves is a horizontal, light purple line extending right from 1.0 k P a on the vertical axis,
which is labeled “Ideal gas.” The region of the graph beneath this line is shaded tan. The remaining curves also start at the same
point on the vertical axis. An orange line extends to the upper right corner of the graph, reaching a value of approximately 1.7 k P a
at 1000 a t m. This orange curve is labeled, “H subscript 2.” A blue curve dips below the horizontal ideal gas line initially, then
increases to cross the line just past 200 a t m. This curve reaches a value of nearly 2.0 k P a at about 800 a t m. This curve is
labeled, “N subscript 2.” A red curve dips below the horizontal ideal gas line initially, then increases to cross the line just past 400 a
t m. This curve reaches a value of nearly 1.5 k P a at about 750 a t m. This curve is labeled, “O subscript 2.” A purple curve dips
below the horizontal ideal gas line, dipping even lower than the O subscript 2 curve initially, then increases to cross the ideal gas
line at about 400 a t m. This curve reaches a value of nearly 2.0 k P a at about 850 a t m. This curve is labeled, “C H subscript 4.” A
yellow curve dips below the horizontal ideal gas line, dipping lower than the other curves to a minimum of about 0.4 k P a at about
0.75 a t m, then increases to cross the ideal gas line at about 500 a t m. This curve reaches a value of about 1.6 k P a at about 900 a t
m. This curve is labeled, “C O subscript 2.”
As is apparent from Figure 9.6.1, the ideal gas law does not describe gas behavior well at relatively high pressures. To determine
why this is, consider the differences between real gas properties and what is expected of a hypothetical ideal gas.

Access for free at OpenStax 9.6.1 https://chem.libretexts.org/@go/page/38206


Particles of a hypothetical ideal gas have no significant volume and do not attract or repel each other. In general, real gases
approximate this behavior at relatively low pressures and high temperatures. However, at high pressures, the molecules of a gas are
crowded closer together, and the amount of empty space between the molecules is reduced. At these higher pressures, the volume
of the gas molecules themselves becomes appreciable relative to the total volume occupied by the gas (Figure 9.6.2). The gas
therefore becomes less compressible at these high pressures, and although its volume continues to decrease with increasing
pressure, this decrease is not proportional as predicted by Boyle’s law.

Figure 9.6.2 : Raising the pressure of a gas increases the fraction of its volume that is occupied by the gas molecules and makes the
gas less compressible.
This figure includes three diagrams. In a, a cylinder with 9 purple spheres with trails indicating motion are shown. Above the
cylinder, the label, “Particles ideal gas,” is connected to two of the spheres with line segments extending into the square. The label
“Assumes” is beneath the square. In b, a cylinder and piston is shown. A relatively small open space is shaded lavender with 9
purple spheres packed close together. No motion trails are present on the spheres. Above the piston, a downward arrow labeled
“Pressure” is directed toward the enclosed area. In c, the cylinder is exactly the same as the first, but the number of molecules has
doubled.
At relatively low pressures, gas molecules have practically no attraction for one another because they are (on average) so far apart,
and they behave almost like particles of an ideal gas. At higher pressures, however, the force of attraction is also no longer
insignificant. This force pulls the molecules a little closer together, slightly decreasing the pressure (if the volume is constant) or
decreasing the volume (at constant pressure) (Figure 9.6.3). This change is more pronounced at low temperatures because the
molecules have lower KE relative to the attractive forces, and so they are less effective in overcoming these attractions after
colliding with one another.

Figure 9.6.3 : (a) Attractions between gas molecules serve to decrease the gas volume at constant pressure compared to an ideal gas
whose molecules experience no attractive forces. (b) These attractive forces will decrease the force of collisions between the
molecules and container walls, therefore reducing the pressure exerted compared to an ideal gas.
This figure includes two diagrams. Each involves two lavender shaded boxes that contain 14 relatively evenly distributed, purple
spheres. In the first box in a, a nearly centrally located purple sphere has 6 double-headed arrows extending outward from it to
nearby spheres. A single purple arrow is pointing right into open space. This box is labeled, “real.” There is a second box that looks
slightly larger than the first box in a. It has the same number of particles but no arrows. This box is labeled, “ideal.” In b, the first
box has a purple sphere at the right side which has 4 double-headed arrows radiating out to the top, bottom, and left to other
spheres. A single purple arrow points right through open space to the edge of the box. This box has no spheres positioned near its
right edge This box is labeled, “real.” The second box is the same size as the first box and contains the same number of particles.
There are no arrows in it, except for the purple arrow which appears to be bigger and bolder. This box is labeled, “ideal.”

Access for free at OpenStax 9.6.2 https://chem.libretexts.org/@go/page/38206


There are several different equations that better approximate gas behavior than does the ideal gas law. The first, and simplest, of
these was developed by the Dutch scientist Johannes van der Waals in 1879. The van der Waals equation improves upon the ideal
gas law by adding two terms: one to account for the volume of the gas molecules and another for the attractive forces between
them.

This figure shows the equation P V equals n R T, with the P in blue text and the V in red text. This equation is followed by a right
pointing arrow. Following this arrow, to the right in blue text appears the equation ( P minus a n superscript 2 divided by V squared
),” which is followed by the red text ( V minus n b ). This is followed in black text with equals n R T. Beneath the second equation
appears the label, “Correction for molecular attraction” which is connected with a line segment to V squared. A second label,
“Correction for volume of molecules,” is similarly connected to n b which appears in red.
The constant a corresponds to the strength of the attraction between molecules of a particular gas, and the constant b corresponds to
2
n a
the size of the molecules of a particular gas. The “correction” to the pressure term in the ideal gas law is 2
, and the “correction”
V
to the volume is nb. Note that when V is relatively large and n is relatively small, both of these correction terms become negligible,
and the van der Waals equation reduces to the ideal gas law, PV = nRT. Such a condition corresponds to a gas in which a relatively
low number of molecules is occupying a relatively large volume, that is, a gas at a relatively low pressure. Experimental values for
the van der Waals constants of some common gases are given in Table 9.6.1.
Table 9.6.1 : Values of van der Waals Constants for Some Common Gases
Gas a (L2 atm/mol2) b (L/mol)

N2 1.39 0.0391

O2 1.36 0.0318

CO2 3.59 0.0427

H2O 5.46 0.0305

He 0.0342 0.0237

CCl4 20.4 0.1383

At low pressures, the correction for intermolecular attraction, a, is more important than the one for molecular volume, b. At high
pressures and small volumes, the correction for the volume of the molecules becomes important because the molecules themselves
are incompressible and constitute an appreciable fraction of the total volume. At some intermediate pressure, the two corrections
have opposing influences and the gas appears to follow the relationship given by PV = nRT over a small range of pressures. This
behavior is reflected by the “dips” in several of the compressibility curves shown in Figure 9.6.1. The attractive force between
molecules initially makes the gas more compressible than an ideal gas, as pressure is raised (Z decreases with increasing P). At
very high pressures, the gas becomes less compressible (Z increases with P), as the gas molecules begin to occupy an increasingly
significant fraction of the total gas volume.
Strictly speaking, the ideal gas equation functions well when intermolecular attractions between gas molecules are negligible and
the gas molecules themselves do not occupy an appreciable part of the whole volume. These criteria are satisfied under conditions
of low pressure and high temperature. Under such conditions, the gas is said to behave ideally, and deviations from the gas laws are
small enough that they may be disregarded—this is, however, very often not the case.

 Example 9.6.1: Comparison of Ideal Gas Law and van der Waals Equation

A 4.25-L flask contains 3.46 mol CO2 at 229 °C. Calculate the pressure of this sample of CO2:
a. from the ideal gas law
b. from the van der Waals equation

Access for free at OpenStax 9.6.3 https://chem.libretexts.org/@go/page/38206


c. Explain the reason(s) for the difference.

Solution
(a) From the ideal gas law:
−1 −1
nRT 3.46 mol × 0.08206 L atm mol K × 502 K
P = = = 33.5 atm
V 4.25 L

(b) From the van der Waals equation:


2 2
n a nRT n a
(P + ) × (V − nb) = nRT ⟶ P = −
2 2
V (V − nb) V

−1 −1 2 2 2
3.46 mol × 0.08206 L atm mo l K × 502 K (3.46 mol ) × 3.59 L atm mo l
P = −
−1 2
(4.25 L − 3.46 mol × 0.0427 L mo l ) (4.25 L)

This finally yields P = 32.4 atm.


(c) This is not very different from the value from the ideal gas law because the pressure is not very high and the temperature is not
very low. The value is somewhat different because CO2 molecules do have some volume and attractions between molecules, and
the ideal gas law assumes they do not have volume or attractions.

 Exercise 9.6.1

A 560-mL flask contains 21.3 g N2 at 145 °C. Calculate the pressure of N2:
a. from the ideal gas law
b. from the van der Waals equation
c. Explain the reason(s) for the difference.

Answer a
46.562 atm
Answer b
46.594 atm
Answer c
The van der Waals equation takes into account the volume of the gas molecules themselves as well as intermolecular
attractions.

Summary
Gas molecules possess a finite volume and experience forces of attraction for one another. Consequently, gas behavior is not
necessarily described well by the ideal gas law. Under conditions of low pressure and high temperature, these factors are negligible,
the ideal gas equation is an accurate description of gas behavior, and the gas is said to exhibit ideal behavior. However, at lower
temperatures and higher pressures, corrections for molecular volume and molecular attractions are required to account for finite
molecular size and attractive forces. The van der Waals equation is a modified version of the ideal gas law that can be used to
account for the non-ideal behavior of gases under these conditions.

9.6.1: Key Equations


molar volume of gas at same T and P P × Vm
Z = =( )
molar volume of ideal gas at same T and P R×T
measured
2
n a
(P + ) × (V − nb) = nRT
2
V

Access for free at OpenStax 9.6.4 https://chem.libretexts.org/@go/page/38206


Glossary
compressibility factor (Z)
ratio of the experimentally measured molar volume for a gas to its molar volume as computed from the ideal gas equation

van der Waals equation


modified version of the ideal gas equation containing additional terms to account for non-ideal gas behavior

This page titled 9.6: Non-Ideal Gas Behavior is shared under a CC BY 4.0 license and was authored, remixed, and/or curated by OpenStax via
source content that was edited to the style and standards of the LibreTexts platform; a detailed edit history is available upon request.

Access for free at OpenStax 9.6.5 https://chem.libretexts.org/@go/page/38206


9.E: Gases (Exercises)
9.1: Gas Pressure
Q9.1.1
Why are sharp knives more effective than dull knives (Hint: think about the definition of pressure)?

S9.1.1
The cutting edge of a knife that has been sharpened has a smaller surface area than a dull knife. Since pressure is force per unit
area, a sharp knife will exert a higher pressure with the same amount of force and cut through material more effectively.

Q9.1.2
Why do some small bridges have weight limits that depend on how many wheels or axles the crossing vehicle has?

Q9.1.3
Why should you roll or belly-crawl rather than walk across a thinly-frozen pond?

S9.1.3
Lying down distributes your weight over a larger surface area, exerting less pressure on the ice compared to standing up. If you
exert less pressure, you are less likely to break through thin ice.

Q9.1.4
A typical barometric pressure in Redding, California, is about 750 mm Hg. Calculate this pressure in atm and kPa.

Q9.1.5
A typical barometric pressure in Denver, Colorado, is 615 mm Hg. What is this pressure in atmospheres and kilopascals?

S9.1.5
0.809 atm; 82.0 kPa

Q9.1.6
A typical barometric pressure in Kansas City is 740 torr. What is this pressure in atmospheres, in millimeters of mercury, and in
kilopascals?

Q9.1.7
Canadian tire pressure gauges are marked in units of kilopascals. What reading on such a gauge corresponds to 32 psi?

S9.1.7
2.2 × 102 kPa

Q9.1.8
During the Viking landings on Mars, the atmospheric pressure was determined to be on the average about 6.50 millibars (1 bar =
0.987 atm). What is that pressure in torr and kPa?

Q9.1.9
The pressure of the atmosphere on the surface of the planet Venus is about 88.8 atm. Compare that pressure in psi to the normal
pressure on earth at sea level in psi.

S9.1.9
Earth: 14.7 lb in–2; Venus: 13.1× 103 lb in−2

Q9.1.10
A medical laboratory catalog describes the pressure in a cylinder of a gas as 14.82 MPa. What is the pressure of this gas in
atmospheres and torr?

Access for free at OpenStax 9.E.1 https://chem.libretexts.org/@go/page/44130


Q9.1.11
Consider this scenario and answer the following questions: On a mid-August day in the northeastern United States, the following
information appeared in the local newspaper: atmospheric pressure at sea level 29.97 in., 1013.9 mbar.
a. What was the pressure in kPa?
b. The pressure near the seacoast in the northeastern United States is usually reported near 30.0 in. Hg. During a hurricane, the
pressure may fall to near 28.0 in. Hg. Calculate the drop in pressure in torr.

S9.1.11
(a) 101.5 kPa; (b) 51 torr drop

Q9.1.12
Why is it necessary to use a nonvolatile liquid in a barometer or manometer?

Q9.1.13
The pressure of a sample of gas is measured at sea level with a closed-end manometer. The liquid in the manometer is mercury.
Determine the pressure of the gas in:
a. torr
b. Pa
c. bar

S9.1.13
(a) 264 torr; (b) 35,200 Pa; (c) 0.352 bar

Q9.1.14
The pressure of a sample of gas is measured with an open-end manometer, partially shown to the right. The liquid in the
manometer is mercury. Assuming atmospheric pressure is 29.92 in. Hg, determine the pressure of the gas in:
a. torr
b. Pa
c. bar

Access for free at OpenStax 9.E.2 https://chem.libretexts.org/@go/page/44130


Q9.1.15
The pressure of a sample of gas is measured at sea level with an open-end mercury manometer. Assuming atmospheric pressure is
760.0 mm Hg, determine the pressure of the gas in:
a. mm Hg
b. atm
c. kPa

S9.1.15
(a) 623 mm Hg; (b) 0.820 atm; (c) 83.1 kPa

Q9.1.16
The pressure of a sample of gas is measured at sea level with an open-end mercury manometer. Assuming atmospheric pressure is
760 mm Hg, determine the pressure of the gas in:
a. mm Hg
b. atm
c. kPa

Q9.1.17
How would the use of a volatile liquid affect the measurement of a gas using open-ended manometers vs. closed-end manometers?

S9.1.17
With a closed-end manometer, no change would be observed, since the vaporized liquid would contribute equal, opposing pressures
in both arms of the manometer tube. However, with an open-ended manometer, a higher pressure reading of the gas would be
obtained than expected, since Pgas = Patm + Pvol liquid.

Access for free at OpenStax 9.E.3 https://chem.libretexts.org/@go/page/44130


9.2: Relating Pressure, Volume, Amount, and Temperature: The Ideal Gas Law
Q9.2.1
Sometimes leaving a bicycle in the sun on a hot day will cause a blowout. Why?

Q9.2.2
Explain how the volume of the bubbles exhausted by a scuba diver (Figure) change as they rise to the surface, assuming that they
remain intact.

S9.2.2
As the bubbles rise, the pressure decreases, so their volume increases as suggested by Boyle’s law.

Q9.2.3
One way to state Boyle’s law is “All other things being equal, the pressure of a gas is inversely proportional to its volume.” (a)
What is the meaning of the term “inversely proportional?” (b) What are the “other things” that must be equal?

Q9.2.4
An alternate way to state Avogadro’s law is “All other things being equal, the number of molecules in a gas is directly proportional
to the volume of the gas.”
a. What is the meaning of the term “directly proportional?”
b. What are the “other things” that must be equal?

S9.2.4
(a) The number of particles in the gas increases as the volume increases. (b) temperature, pressure

Q9.2.5
How would the graph in Figure change if the number of moles of gas in the sample used to determine the curve were doubled?

The volume and temperature are linearly related for 1 mole of methane gas at a constant pressure of 1 atm. If the temperature is in
kelvin, volume and temperature are directly proportional. The line stops at 111 K because methane liquefies at this temperature;
when extrapolated, it intersects the graph’s origin, representing a temperature of absolute zero.

Q9.2.6
How would the graph in Figure change if the number of moles of gas in the sample used to determine the curve were doubled?

Access for free at OpenStax 9.E.4 https://chem.libretexts.org/@go/page/44130


When a gas occupies a smaller volume, it exerts a higher pressure; when it occupies a larger volume, it exerts a lower pressure
(assuming the amount of gas and the temperature do not change). Since P and V are inversely proportional, a graph of 1/P vs. \
(V\) is linear.

S9.2.6
The curve would be farther to the right and higher up, but the same basic shape.

Q9.2.7
In addition to the data found in Figure, what other information do we need to find the mass of the sample of air used to determine
the graph?

Q9.2.8
Determine the volume of 1 mol of CH4 gas at 150 K and 1 atm, using Figure.

S9.2.8
16.3 to 16.5 L

Q9.2.9
Determine the pressure of the gas in the syringe shown in Figure when its volume is 12.5 mL, using:
a. the appropriate graph
b. Boyle’s law

Q9.2.10
A spray can is used until it is empty except for the propellant gas, which has a pressure of 1344 torr at 23 °C. If the can is thrown
into a fire (T = 475 °C), what will be the pressure in the hot can?

S9.2.10
3.40 × 103 torr

Access for free at OpenStax 9.E.5 https://chem.libretexts.org/@go/page/44130


Q9.2.11
What is the temperature of an 11.2-L sample of carbon monoxide, CO, at 744 torr if it occupies 13.3 L at 55 °C and 744 torr?

S9.2.11
P1 V1 P2 V2
we must use = and solve for T1
T1 T2

P1 V1 T2
T1 =
P2 V2

Where:
P1 = 744 torr

V1 = 11.2 L

P2 = 744 torr

V2 = 13.3 L

T2 = 328.15° K

(744 torr)(11.2 L)(328.15° K)


= 276° K
(744 torr)(13.3 L)

276°K ; 3°C

Q9.2.12
A 2.50-L volume of hydrogen measured at –196 °C is warmed to 100 °C. Calculate the volume of the gas at the higher
temperature, assuming no change in pressure.

S9.2.13
12.1 L

Q9.2.14
A balloon inflated with three breaths of air has a volume of 1.7 L. At the same temperature and pressure, what is the volume of the
balloon if five more same-sized breaths are added to the balloon?

Q9.2.15
A weather balloon contains 8.80 moles of helium at a pressure of 0.992 atm and a temperature of 25 °C at ground level. What is the
volume of the balloon under these conditions?

S9.2.15
217 L

Q9.2.16
The volume of an automobile air bag was 66.8 L when inflated at 25 °C with 77.8 g of nitrogen gas. What was the pressure in the
bag in kPa?

Access for free at OpenStax 9.E.6 https://chem.libretexts.org/@go/page/44130


Q9.2.17
How many moles of gaseous boron trifluoride, BF3, are contained in a 4.3410-L bulb at 788.0 K if the pressure is 1.220 atm? How
many grams of BF3?

S9.2.17
8.190 × 10–2 mol; 5.553 g

Q9.2.18
Iodine, I2, is a solid at room temperature but sublimes (converts from a solid into a gas) when warmed. What is the temperature in a
73.3-mL bulb that contains 0.292 g of I2 vapor at a pressure of 0.462 atm?

S9.2.18
1.) Use the equation P V = nRT and solve for T
PV
T =
nR

2.) convert grams of I2 to moles of I2 and convert mL to L


1 mole I
2 −3
0.292g I × = 1.15 × 10 moles I
2 2
253.8g I
2

73.3 mL = 0.0733 L

atm L
3.) Use these values along with R = 0.08206 to solve for T
mole °K

(0.462 atm )(0.0733 L )


T = = 359 °K
atm L
−3
(1.15 × 10 moles )(0.08206 )
mole °K

359°K ; 86°C

Q9.2.19
How many grams of gas are present in each of the following cases?
a. 0.100 L of CO2 at 307 torr and 26 °C
b. 8.75 L of C2H4, at 378.3 kPa and 483 K
c. 221 mL of Ar at 0.23 torr and –54 °C

S9.2.19
(a) 7.24 × 10–2 g; (b) 23.1 g; (c) 1.5 × 10–4 g

Q9.2.20
A high altitude balloon is filled with 1.41 × 104 L of hydrogen at a temperature of 21 °C and a pressure of 745 torr. What is the
volume of the balloon at a height of 20 km, where the temperature is –48 °C and the pressure is 63.1 torr?

Q9.2.21
A cylinder of medical oxygen has a volume of 35.4 L, and contains O2 at a pressure of 151 atm and a temperature of 25 °C. What
volume of O2 does this correspond to at normal body conditions, that is, 1 atm and 37 °C?

S9.2.21
5561 L

Q9.2.22
A large scuba tank (Figure) with a volume of 18 L is rated for a pressure of 220 bar. The tank is filled at 20 °C and contains enough
air to supply 1860 L of air to a diver at a pressure of 2.37 atm (a depth of 45 feet). Was the tank filled to capacity at 20 °C?

Access for free at OpenStax 9.E.7 https://chem.libretexts.org/@go/page/44130


Q9.2.23
A 20.0-L cylinder containing 11.34 kg of butane, C4H10, was opened to the atmosphere. Calculate the mass of the gas remaining in
the cylinder if it were opened and the gas escaped until the pressure in the cylinder was equal to the atmospheric pressure, 0.983
atm, and a temperature of 27 °C.

S9.2.23
46.4 g

Q9.2.24
While resting, the average 70-kg human male consumes 14 L of pure O2 per hour at 25 °C and 100 kPa. How many moles of O2
are consumed by a 70 kg man while resting for 1.0 h?

Q9.2.25
For a given amount of gas showing ideal behavior, draw labeled graphs of:
a. the variation of P with V
b. the variation of V with T
c. the variation of P with T
1
d. the variation of with V
P

Q9.2.26
For a gas exhibiting ideal behavior:

Q9.2.27
A liter of methane gas, CH4, at STP contains more atoms of hydrogen than does a liter of pure hydrogen gas, H2, at STP. Using
Avogadro’s law as a starting point, explain why.

Access for free at OpenStax 9.E.8 https://chem.libretexts.org/@go/page/44130


Q9.2.28
The effect of chlorofluorocarbons (such as CCl2F2) on the depletion of the ozone layer is well known. The use of substitutes, such
as CH3CH2F(g), for the chlorofluorocarbons, has largely corrected the problem. Calculate the volume occupied by 10.0 g of each
of these compounds at STP:
a. CCl2F2(g)
b. CH3CH2F(g)

S9.2.28
(a) 1.85 L CCl2F2; (b) 4.66 L CH3CH2F

Q9.2.29
As 1 g of the radioactive element radium decays over 1 year, it produces 1.16 × 1018 alpha particles (helium nuclei). Each alpha
particle becomes an atom of helium gas. What is the pressure in pascal of the helium gas produced if it occupies a volume of 125
mL at a temperature of 25 °C?

Q9.2.30
A balloon that is 100.21 L at 21 °C and 0.981 atm is released and just barely clears the top of Mount Crumpet in British Columbia.
If the final volume of the balloon is 144.53 L at a temperature of 5.24 °C, what is the pressure experienced by the balloon as it
clears Mount Crumpet?

S9.2.30
0.644 atm

Q9.2.31
If the temperature of a fixed amount of a gas is doubled at constant volume, what happens to the pressure?

Q9.2.32
If the volume of a fixed amount of a gas is tripled at constant temperature, what happens to the pressure?

S9.2.32
The pressure decreases by a factor of 3.

9.3: Stoichiometry of Gaseous Substances, Mixtures, and Reactions


Q9.3.1
What is the density of laughing gas, dinitrogen monoxide, N2O, at a temperature of 325 K and a pressure of 113.0 kPa?

S9.3.1
1.) First convert kPa to atm
1 atm
113.0 kP a × = 1.115 atm
101.325 kP a

PM
2.) The use the equation d = where d = density in g L-1 and M = molar mass in g mol-1
RT

g
(1.115 atm)(44.02 )
mol g
d = = 1.84
atm L L
(0.08206 )(325 °K )
mole °K

Q9.3.2
Calculate the density of Freon 12, CF2Cl2, at 30.0 °C and 0.954 atm.

Access for free at OpenStax 9.E.9 https://chem.libretexts.org/@go/page/44130


S9.3.2
4.64 g L−1

Q9.3.3
Which is denser at the same temperature and pressure, dry air or air saturated with water vapor? Explain.

Q9.3.4
A cylinder of O2(g) used in breathing by emphysema patients has a volume of 3.00 L at a pressure of 10.0 atm. If the temperature
of the cylinder is 28.0 °C, what mass of oxygen is in the cylinder?

S9.3.4
38.8 g

Q9.3.5
What is the molar mass of a gas if 0.0494 g of the gas occupies a volume of 0.100 L at a temperature 26 °C and a pressure of 307
torr?

S9.3.5
1.) convert torr to atm and °C to °K
307 torr = 0.404atm

26°C = 300.°K

2.) Use the equation P V = nRT and solve for n


PV
n =
RT

(0.404 atm )(0.100 L )


n = = 0.00165 moles
atm L
(0.08206 )(300. °K )
mol °K

3.) Then divide grams by the number of moles to obtain the molar mass:
0.0494g g
= 30.0
0.00165 moles mole

Q9.3.6
What is the molar mass of a gas if 0.281 g of the gas occupies a volume of 125 mL at a temperature 126 °C and a pressure of 777
torr?

S9.3.6
72.0 g mol−1

Q9.3.7
How could you show experimentally that the molecular formula of propene is C3H6, not CH2?

Q9.3.8
The density of a certain gaseous fluoride of phosphorus is 3.93 g/L at STP. Calculate the molar mass of this fluoride and determine
its molecular formula.

S9.3.8
88.1 g mol−1; PF3

Q9.3.9
Consider this question: What is the molecular formula of a compound that contains 39% C, 45% N, and 16% H if 0.157 g of the
compound occupies 125 mL with a pressure of 99.5 kPa at 22 °C?

Access for free at OpenStax 9.E.10 https://chem.libretexts.org/@go/page/44130


a. Outline the steps necessary to answer the question.
b. Answer the question.

Q9.3.10
A 36.0–L cylinder of a gas used for calibration of blood gas analyzers in medical laboratories contains 350 g CO2, 805 g O2, and
4,880 g N2. At 25 degrees C, what is the pressure in the cylinder in atmospheres?

S9.3.10
141 atm

Q9.3.11
A cylinder of a gas mixture used for calibration of blood gas analyzers in medical laboratories contains 5.0% CO2, 12.0% O2, and
the remainder N2 at a total pressure of 146 atm. What is the partial pressure of each component of this gas? (The percentages given
indicate the percent of the total pressure that is due to each component.)

Q9.3.12
A sample of gas isolated from unrefined petroleum contains 90.0% CH4, 8.9% C2H6, and 1.1% C3H8 at a total pressure of 307.2
kPa. What is the partial pressure of each component of this gas? (The percentages given indicate the percent of the total pressure
that is due to each component.)

S9.3.12
CH4: 276 kPa; C2H6: 27 kPa; C3H8: 3.4 kPa

Q9.3.13
A mixture of 0.200 g of H2, 1.00 g of N2, and 0.820 g of Ar is stored in a closed container at STP. Find the volume of the container,
assuming that the gases exhibit ideal behavior.

Q9.3.14
Most mixtures of hydrogen gas with oxygen gas are explosive. However, a mixture that contains less than 3.0 % O2 is not. If
enough O2 is added to a cylinder of H2 at 33.2 atm to bring the total pressure to 34.5 atm, is the mixture explosive?

S9.3.14
Yes

Q9.3.15
A commercial mercury vapor analyzer can detect, in air, concentrations of gaseous Hg atoms (which are poisonous) as low as 2 ×
10−6 mg/L of air. At this concentration, what is the partial pressure of gaseous mercury if the atmospheric pressure is 733 torr at 26
°C?

Q9.3.16
A sample of carbon monoxide was collected over water at a total pressure of 756 torr and a temperature of 18 °C. What is the
pressure of the carbon monoxide? (See Table for the vapor pressure of water.)

S9.3.16
740 torr

Q9.3.17
In an experiment in a general chemistry laboratory, a student collected a sample of a gas over water. The volume of the gas was 265
mL at a pressure of 753 torr and a temperature of 27 °C. The mass of the gas was 0.472 g. What was the molar mass of the gas?

Q9.3.18
Joseph Priestley first prepared pure oxygen by heating mercuric oxide, HgO:
2 HgO(s) ⟶ 2 Hg(l) + O (g)
2

Access for free at OpenStax 9.E.11 https://chem.libretexts.org/@go/page/44130


a. Outline the steps necessary to answer the following question: What volume of O2 at 23 °C and 0.975 atm is produced by the
decomposition of 5.36 g of HgO?
b. Answer the question.

S9.3.18
(a) Determine the moles of HgO that decompose; using the chemical equation, determine the moles of O2 produced by
decomposition of this amount of HgO; and determine the volume of O2 from the moles of O2, temperature, and pressure. (b) 0.308
L

Q9.3.19
Cavendish prepared hydrogen in 1766 by the novel method of passing steam through a red-hot gun barrel:
4 H O(g) + 3 Fe(s) ⟶ Fe O (s) + 4 H (g) (9.E.1)
2 3 4 2

a. Outline the steps necessary to answer the following question: What volume of H2 at a pressure of 745 torr and a temperature of
20 °C can be prepared from the reaction of 15.O g of H2O?
b. Answer the question.

Q9.3.20
The chlorofluorocarbon CCl2F2 can be recycled into a different compound by reaction with hydrogen to produce CH2F2(g), a
compound useful in chemical manufacturing:
CCl F (g) + 4 H (g) ⟶ CH F (g) + 2 HCl(g) (9.E.2)
2 2 2 2 2

a. Outline the steps necessary to answer the following question: What volume of hydrogen at 225 atm and 35.5 °C would be
required to react with 1 ton (1.000 × 103 kg) of CCl2F2?
b. Answer the question.

S9.3.20
a. Determine the molar mass of CCl2F2. From the balanced equation, calculate the moles of H2 needed for the complete reaction.
From the ideal gas law, convert moles of H2 into volume.
b. 3.72 × 103 L

Q9.3.21
Automobile air bags are inflated with nitrogen gas, which is formed by the decomposition of solid sodium azide (NaN3). The other
product is sodium metal. Calculate the volume of nitrogen gas at 27 °C and 756 torr formed by the decomposition of 125 g of
sodium azide.

Q9.3.22
Lime, CaO, is produced by heating calcium carbonate, CaCO3; carbon dioxide is the other product.
a. Outline the steps necessary to answer the following question: What volume of carbon dioxide at 875° and 0.966 atm is
produced by the decomposition of 1 ton (1.000 × 103 kg) of calcium carbonate?
b. Answer the question.

S9.3.22
(a) Balance the equation. Determine the grams of CO2 produced and the number of moles. From the ideal gas law, determine the
volume of gas. (b) 7.43 × 105 L

Q9.3.23
Before small batteries were available, carbide lamps were used for bicycle lights. Acetylene gas, C2H2, and solid calcium
hydroxide were formed by the reaction of calcium carbide, CaC2, with water. The ignition of the acetylene gas provided the light.
Currently, the same lamps are used by some cavers, and calcium carbide is used to produce acetylene for carbide cannons.
a. Outline the steps necessary to answer the following question: What volume of C2H2 at 1.005 atm and 12.2 °C is formed by the
reaction of 15.48 g of CaC2 with water?
b. Answer the question.

Access for free at OpenStax 9.E.12 https://chem.libretexts.org/@go/page/44130


Q9.3.24
Calculate the volume of oxygen required to burn 12.00 L of ethane gas, C2H6, to produce carbon dioxide and water, if the volumes
of C2H6 and O2 are measured under the same conditions of temperature and pressure.

S9.3.24
42.00 L

Q9.3.25
What volume of O2 at STP is required to oxidize 8.0 L of NO at STP to NO2? What volume of NO2 is produced at STP?

Q9.3.26
Consider the following questions:
a. What is the total volume of the CO2(g) and H2O(g) at 600 °C and 0.888 atm produced by the combustion of 1.00 L of C2H6(g)
measured at STP?
b. What is the partial pressure of H2O in the product gases?

S9.3.26
(a) 18.0 L; (b) 0.533 atm

Q9.3.27
Methanol, CH3OH, is produced industrially by the following reaction:
 copper catalyst 300 °C, 300 atm 

CO(g) + 2 H (g) −−−−−−−−−−−−−−−−−−→ CH OH(g) (9.E.3)


2 3

Q9.3.28
Assuming that the gases behave as ideal gases, find the ratio of the total volume of the reactants to the final volume.

Q9.3.29
What volume of oxygen at 423.0 K and a pressure of 127.4 kPa is produced by the decomposition of 129.7 g of BaO2 to BaO and
O2?

S9.3.29
10.57 L O2

Q9.3.30
A 2.50-L sample of a colorless gas at STP decomposed to give 2.50 L of N2 and 1.25 L of O2 at STP. What is the colorless gas?

Q9.3.31
Ethanol, C2H5OH, is produced industrially from ethylene, C2H4, by the following sequence of reactions:
3C H + 2 H SO ⟶ C H HSO + (C H ) SO (9.E.4)
2 4 2 4 2 5 4 2 5 2 4

C H HSO + (C H ) SO + 3 H O ⟶ 3 C H OH + 2 H SO (9.E.5)
2 5 4 2 5 2 4 2 2 5 2 4

What volume of ethylene at STP is required to produce 1.000 metric ton (1000 kg) of ethanol if the overall yield of ethanol is
90.1%?

S9.3.31
5.40 × 105 L

Q9.3.32
One molecule of hemoglobin will combine with four molecules of oxygen. If 1.0 g of hemoglobin combines with 1.53 mL of
oxygen at body temperature (37 °C) and a pressure of 743 torr, what is the molar mass of hemoglobin?

Access for free at OpenStax 9.E.13 https://chem.libretexts.org/@go/page/44130


Q9.3.33
A sample of a compound of xenon and fluorine was confined in a bulb with a pressure of 18 torr. Hydrogen was added to the bulb
until the pressure was 72 torr. Passage of an electric spark through the mixture produced Xe and HF. After the HF was removed by
reaction with solid KOH, the final pressure of xenon and unreacted hydrogen in the bulb was 36 torr. What is the empirical formula
of the xenon fluoride in the original sample? (Note: Xenon fluorides contain only one xenon atom per molecule.)

S9.3.33
XeF2

Q9.3.34
One method of analyzing amino acids is the van Slyke method. The characteristic amino groups (−NH2) in protein material are
allowed to react with nitrous acid, HNO2, to form N2 gas. From the volume of the gas, the amount of amino acid can be
determined. A 0.0604-g sample of a biological sample containing glycine, CH2(NH2)COOH, was analyzed by the van Slyke
method and yielded 3.70 mL of N2 collected over water at a pressure of 735 torr and 29 °C. What was the percentage of glycine in
the sample?
CH (NH )CO H + HNO ⟶ CH (OH)CO H + H O + N
2 2 2 2 2 2 2 2

9.4: Effusion and Diffusion of Gases


Q9.4.1
A balloon filled with helium gas is found to take 6 hours to deflate to 50% of its original volume. How long will it take for an
identical balloon filled with the same volume of hydrogen gas (instead of helium) to decrease its volume by 50%?

S9.4.1
4.2 hours

Q9.4.2
Explain why the numbers of molecules are not identical in the left- and right-hand bulbs shown in the center illustration of Figure.

Q9.4.3
Starting with the definition of rate of effusion and Graham’s finding relating rate and molar mass, show how to derive the Graham’s
law equation, relating the relative rates of effusion for two gases to their molecular masses.

S9.4.3
Effusion can be defined as the process by which a gas escapes through a pinhole into a vacuum. Graham’s law states that with a
1/2
rate A molar mass of B
mixture of two gases A and B: ( ) =( ) . Both A and B are in the same container at the same
rate B molar mass of A

temperature, and therefore will have the same kinetic energy:


1/2
2 2 1/2 1/2
1 1 1 v mB v mB vA mB
KEA = KEB KE = mv
2
Therefore, 2
mA v
A
= mB v
2
B
A
= (
A
) =( ) =( )
2 2
2 2 2 v mA v mA vB mA
B B

Q9.4.4
Heavy water, D2O (molar mass = 20.03 g mol–1), can be separated from ordinary water, H2O (molar mass = 18.01), as a result of
the difference in the relative rates of diffusion of the molecules in the gas phase. Calculate the relative rates of diffusion of H2O and
D2O.

Q9.4.5
Which of the following gases diffuse more slowly than oxygen? F2, Ne, N2O, C2H2, NO, Cl2, H2S

S9.4.5
F2, N2O, Cl2, H2S

Access for free at OpenStax 9.E.14 https://chem.libretexts.org/@go/page/44130


Q9.4.6
During the discussion of gaseous diffusion for enriching uranium, it was claimed that 235UF6 diffuses 0.4% faster than 238UF6.
Show the calculation that supports this value. The molar mass of 235UF6 = 235.043930 + 6 × 18.998403 = 349.034348 g/mol, and
the molar mass of 238UF6 = 238.050788 + 6 × 18.998403 = 352.041206 g/mol.

Q9.4.7
Calculate the relative rate of diffusion of 1H2 (molar mass 2.0 g/mol) compared to that of 2H2 (molar mass 4.0 g/mol) and the
relative rate of diffusion of O2 (molar mass 32 g/mol) compared to that of O3 (molar mass 48 g/mol).

S9.4.7
1.4; 1.2

Q9.4.8
A gas of unknown identity diffuses at a rate of 83.3 mL/s in a diffusion apparatus in which carbon dioxide diffuses at the rate of
102 mL/s. Calculate the molecular mass of the unknown gas.

Q9.4.9
When two cotton plugs, one moistened with ammonia and the other with hydrochloric acid, are simultaneously inserted into
opposite ends of a glass tube that is 87.0 cm long, a white ring of NH4Cl forms where gaseous NH3 and gaseous HCl first come
into contact. (Hint: Calculate the rates of diffusion for both NH3 and HCl, and find out how much faster NH3 diffuses than HCl.)
NH (g) + HCl(g) ⟶ NH Cl(s) (9.E.6)
3 4

Q9.4.10
At approximately what distance from the ammonia moistened plug does this occur?

S9.4.10
51.7 cm

9.5: The Kinetic-Molecular Theory

Q9.5.1
Using the postulates of the kinetic molecular theory, explain why a gas uniformly fills a container of any shape.

Q9.5.2
Can the speed of a given molecule in a gas double at constant temperature? Explain your answer.

S9.5.2
Yes. At any given instant, there are a range of values of molecular speeds in a sample of gas. Any single molecule can speed up or
slow down as it collides with other molecules. The average velocity of all the molecules is constant at constant temperature.

Q9.5.3
Describe what happens to the average kinetic energy of ideal gas molecules when the conditions are changed as follows:
a. The pressure of the gas is increased by reducing the volume at constant temperature.
b. The pressure of the gas is increased by increasing the temperature at constant volume.
c. The average velocity of the molecules is increased by a factor of 2.

Q9.5.4
The distribution of molecular velocities in a sample of helium is shown in Figure. If the sample is cooled, will the distribution of
velocities look more like that of H2 or of H2O? Explain your answer.

Access for free at OpenStax 9.E.15 https://chem.libretexts.org/@go/page/44130


S9.5.4
H2O. Cooling slows the velocities of the He atoms, causing them to behave as though they were heavier.

Q9.5.5
What is the ratio of the average kinetic energy of a SO2 molecule to that of an O2 molecule in a mixture of two gases? What is the
ratio of the root mean square speeds, urms, of the two gases?

Q9.5.6
A 1-L sample of CO initially at STP is heated to 546 °C, and its volume is increased to 2 L.
a. What effect do these changes have on the number of collisions of the molecules of the gas per unit area of the container wall?
b. What is the effect on the average kinetic energy of the molecules?
c. What is the effect on the root mean square speed of the molecules?

S9.5.6
(a) The number of collisions per unit area of the container wall is constant. (b) The average kinetic energy doubles. (c) The root

mean square speed increases to √2 times its initial value; urms is proportional to KE . avg

Q9.5.7
The root mean square speed of H2 molecules at 25 °C is about 1.6 km/s. What is the root mean square speed of a N2 molecule at 25
°C?

Q9.5.8
Answer the following questions:
a. Is the pressure of the gas in the hot air balloon shown at the opening of this chapter greater than, less than, or equal to that of the
atmosphere outside the balloon?
b. Is the density of the gas in the hot air balloon shown at the opening of this chapter greater than, less than, or equal to that of the
atmosphere outside the balloon?
c. At a pressure of 1 atm and a temperature of 20 °C, dry air has a density of 1.2256 g/L. What is the (average) molar mass of dry
air?
d. The average temperature of the gas in a hot air balloon is 1.30 × 102 °F. Calculate its density, assuming the molar mass equals
that of dry air.
e. The lifting capacity of a hot air balloon is equal to the difference in the mass of the cool air displaced by the balloon and the
mass of the gas in the balloon. What is the difference in the mass of 1.00 L of the cool air in part (c) and the hot air in part (d)?
f. An average balloon has a diameter of 60 feet and a volume of 1.1 × 105 ft3. What is the lifting power of such a balloon? If the
weight of the balloon and its rigging is 500 pounds, what is its capacity for carrying passengers and cargo?
g. A balloon carries 40.0 gallons of liquid propane (density 0.5005 g/L). What volume of CO2 and H2O gas is produced by the
combustion of this propane?
h. A balloon flight can last about 90 minutes. If all of the fuel is burned during this time, what is the approximate rate of heat loss
(in kJ/min) from the hot air in the bag during the flight?

S9.5.1
(a) equal; (b) less than; (c) 29.48 g mol−1; (d) 1.0966 g L−1; (e) 0.129 g/L; (f) 4.01 × 105 g; net lifting capacity = 384 lb; (g) 270 L;
(h) 39.1 kJ min−1

Q9.5.1
R1
Show that the ratio of the rate of diffusion of Gas 1 to the rate of diffusion of Gas 2, , is the same at 0 °C and 100 °C.
R2

9.6: Non-Ideal Gas Behavior

Access for free at OpenStax 9.E.16 https://chem.libretexts.org/@go/page/44130


Q9.6.1
Graphs showing the behavior of several different gases follow. Which of these gases exhibit behavior significantly different from
that expected for ideal gases?

S9.6.1
Gases C, E, and F

Q9.6.3
Explain why the plot of PV for CO2 differs from that of an ideal gas.

Access for free at OpenStax 9.E.17 https://chem.libretexts.org/@go/page/44130


Q9.6.3
Under which of the following sets of conditions does a real gas behave most like an ideal gas, and for which conditions is a real gas
expected to deviate from ideal behavior? Explain.
a. high pressure, small volume
b. high temperature, low pressure
c. low temperature, high pressure

S9.6.3
The gas behavior most like an ideal gas will occur under the conditions in (b). Molecules have high speeds and move through
greater distances between collision; they also have shorter contact times and interactions are less likely. Deviations occur with the
conditions described in (a) and (c). Under conditions of (a), some gases may liquefy. Under conditions of (c), most gases will
liquefy.

Q9.6.4
Describe the factors responsible for the deviation of the behavior of real gases from that of an ideal gas.

Q9.6.5
For which of the following gases should the correction for the molecular volume be largest: CO, CO2, H2, He, NH3, SF6?

S9.6.6
SF6

Q9.6.7
A 0.245-L flask contains 0.467 mol CO2 at 159 °C. Calculate the pressure:
a. using the ideal gas law
b. using the van der Waals equation
c. Explain the reason for the difference.
d. Identify which correction (that for P or V) is dominant and why.

Access for free at OpenStax 9.E.18 https://chem.libretexts.org/@go/page/44130


Q9.6.8
Answer the following questions:
a. If XX behaved as an ideal gas, what would its graph of Z vs. P look like?
b. For most of this chapter, we performed calculations treating gases as ideal. Was this justified?
c. What is the effect of the volume of gas molecules on Z? Under what conditions is this effect small? When is it large? Explain
using an appropriate diagram.
d. What is the effect of intermolecular attractions on the value of Z? Under what conditions is this effect small? When is it large?
Explain using an appropriate diagram.
e. In general, under what temperature conditions would you expect Z to have the largest deviations from the Z for an ideal gas?

S9.6.8
(a) A straight horizontal line at 1.0; (b) When real gases are at low pressures and high temperatures they behave close enough to
ideal gases that they are approximated as such, however, in some cases, we see that at a high pressure and temperature, the ideal
gas approximation breaks down and is significantly different from the pressure calculated by the van der Waals equation (c) The
greater the compressibility, the more the volume matters. At low pressures, the correction factor for intermolecular attractions is
more significant, and the effect of the volume of the gas molecules on Z would be a small lowering compressibility. At higher
pressures, the effect of the volume of the gas molecules themselves on Z would increase compressibility (see Figure) (d) Once
again, at low pressures, the effect of intermolecular attractions on Z would be more important than the correction factor for the
volume of the gas molecules themselves, though perhaps still small. At higher pressures and low temperatures, the effect of
intermolecular attractions would be larger. See Figure. (e) low temperatures

9.6: Non-Ideal Gas Behavior

Exercises
Graphs showing the behavior of several different gases follow. Which of these gases exhibit behavior significantly different from
that expected for ideal gases?

Gases C, E, and F
Explain why the plot of PV for CO2 differs from that of an ideal gas.

Access for free at OpenStax 9.E.19 https://chem.libretexts.org/@go/page/44130


A graph is shown. The horizontal axis is labeled, “P ( a t m ).” Its scale is marked at 0, 1, and 2. The vertical axis is labeled, “P V ( a t m L ).” This scale includes markings at 0, 22.4, 22.5, and 22.6. Two curves and
two lines are drawn of varying colors. One line is a horizontal, blue line extending right from about 22.42 a t m L on the vertical axis, and is labeled, “Ideal gas.” The remaining two curves and one line start at the same
point on the vertical axis. A green line extends up and to the right slightly on the graph, reaching a value of approximately 22.46 a t m L at 2 a t m. This green line is labeled, “H e.” An orange curve dips below the
horizontal ideal gas line initially, then increases to cross the line just past 1 a t m. This curve reaches a value of about 22.52 a t m L at 2 a t m. This curve is labeled, “C H subscript 4.” A purple curve dips below the
horizontal ideal gas line initially, then increases to cross the line at about 0.8 a t m. This curve reaches a value of nearly 22.62 a t m L at nearly 1.2 a t m. This curve is labeled, “C O subscript 2.”

Under which of the following sets of conditions does a real gas behave most like an ideal gas, and for which conditions is a real gas
expected to deviate from ideal behavior? Explain.
(a) high pressure, small volume
(b) high temperature, low pressure
(c) low temperature, high pressure
The gas behavior most like an ideal gas will occur under the conditions in (b). Molecules have high speeds and move through
greater distances between collision; they also have shorter contact times and interactions are less likely. Deviations occur with the
conditions described in (a) and (c). Under conditions of (a), some gases may liquefy. Under conditions of (c), most gases will
liquefy.
Describe the factors responsible for the deviation of the behavior of real gases from that of an
ideal gas.
For which of the following gases should the correction for the molecular volume be largest:
CO, CO2, H2, He, NH3, SF6?
SF6
A 0.245-L flask contains 0.467 mol CO2 at 159 °C. Calculate the pressure:
(a) using the ideal gas law
(b) using the van der Waals equation
(c) Explain the reason for the difference.
(d) Identify which correction (that for P or V) is dominant and why.
Answer the following questions:
(a) If XX behaved as an ideal gas, what would its graph of Z vs. P look like?
(b) For most of this chapter, we performed calculations treating gases as ideal. Was this justified?
(c) What is the effect of the volume of gas molecules on Z? Under what conditions is this effect small? When is it large? Explain
using an appropriate diagram.
(d) What is the effect of intermolecular attractions on the value of Z? Under what conditions is this effect small? When is it large?
Explain using an appropriate diagram.
(e) In general, under what temperature conditions would you expect Z to have the largest deviations from the Z for an ideal gas?
(a) A straight horizontal line at 1.0; (b) When real gases are at low pressures and high temperatures they behave close enough to
ideal gases that they are approximated as such, however, in some cases, we see that at a high pressure and temperature, the ideal
gas approximation breaks down and is significantly different from the pressure calculated by the van der Waals equation (c) The
greater the compressibility, the more the volume matters. At low pressures, the correction factor for intermolecular attractions is
more significant, and the effect of the volume of the gas molecules on Z would be a small lowering compressibility. At higher
pressures, the effect of the volume of the gas molecules themselves on Z would increase compressibility (see Figure) (d) Once
again, at low pressures, the effect of intermolecular attractions on Z would be more important than the correction factor for the
volume of the gas molecules themselves, though perhaps still small. At higher pressures and low temperatures, the effect of
intermolecular attractions would be larger. See Figure. (e) low temperatures

This page titled 9.E: Gases (Exercises) is shared under a CC BY 4.0 license and was authored, remixed, and/or curated by OpenStax via source
content that was edited to the style and standards of the LibreTexts platform; a detailed edit history is available upon request.

Access for free at OpenStax 9.E.20 https://chem.libretexts.org/@go/page/44130


CHAPTER OVERVIEW
10: Liquids and Solids

A general chemistry Libretexts Textbook remixed and remastered from


OpenStax's textbook:
General Chemistry
The great distances between atoms and molecules in a gaseous phase, and the corresponding absence of any significant interactions
between them, allows for simple descriptions of many physical properties that are the same for all gases, regardless of their
chemical identities. As described in the final module of the chapter on gases, this situation changes at high pressures and low
temperatures—conditions that permit the atoms and molecules to interact to a much greater extent. In the liquid and solid states,
these interactions are of considerable strength and play an important role in determining a number of physical properties that do
depend on the chemical identity of the substance. In this chapter, the nature of these interactions and their effects on various
physical properties of liquid and solid phases will be examined.
10.0: Prelude to Liquids and Solids
10.1: Intermolecular Forces
10.2: Properties of Liquids
10.3: Phase Transitions
10.4: Phase Diagrams
10.5: The Solid State of Matter
10.6: Lattice Structures in Crystalline Solids
10.E: Liquids and Solids (Exercises)

This page titled 10: Liquids and Solids is shared under a CC BY 4.0 license and was authored, remixed, and/or curated by OpenStax via source
content that was edited to the style and standards of the LibreTexts platform; a detailed edit history is available upon request.

1
10.0: Prelude to Liquids and Solids
The great distances between atoms and molecules in a gaseous phase, and the corresponding absence of any significant interactions
between them, allows for simple descriptions of many physical properties that are the same for all gases, regardless of their
chemical identities. As described in the final module of the chapter on gases, this situation changes at high pressures and low
temperatures—conditions that permit the atoms and molecules to interact to a much greater extent.

Figure 10.0.1 : Solid carbon dioxide (“dry ice”, left) sublimes vigorously when placed in a liquid (right), cooling the liquid and
generating a fog of condensed water vapor above the cylinder. (credit: modification of work by Paul Flowers)
This figure shows pieces of a white substance which appear to be sublimating. To the right of these pieces are three graduated
cylinders. Each cylinder holds a different color liquid, and above the liquid, the cylinders are filled with a fog-like substance. This
fog-like substance swirls out of the top and around the outside of the cylinders.
In the liquid and solid states, these interactions are of considerable strength and play an important role in determining a number of
physical properties that do depend on the chemical identity of the substance. In this chapter, the nature of these interactions and
their effects on various physical properties of liquid and solid phases will be examined.

This page titled 10.0: Prelude to Liquids and Solids is shared under a CC BY 4.0 license and was authored, remixed, and/or curated by OpenStax
via source content that was edited to the style and standards of the LibreTexts platform; a detailed edit history is available upon request.

Access for free at OpenStax 10.0.1 https://chem.libretexts.org/@go/page/38209


10.1: Intermolecular Forces
 Learning Objectives
Describe the types of intermolecular forces possible between atoms or molecules in condensed phases (dispersion forces,
dipole-dipole attractions, and hydrogen bonding)
Identify the types of intermolecular forces experienced by specific molecules based on their structures
Explain the relation between the intermolecular forces present within a substance and the temperatures associated with
changes in its physical state

As was the case for gaseous substances, the kinetic molecular theory may be used to explain the behavior of solids and liquids. In
the following description, the term particle will be used to refer to an atom, molecule, or ion. Note that we will use the popular
phrase “intermolecular attraction” to refer to attractive forces between the particles of a substance, regardless of whether these
particles are molecules, atoms, or ions.
Consider these two aspects of the molecular-level environments in solid, liquid, and gaseous matter:
Particles in a solid are tightly packed together and often arranged in a regular pattern; in a liquid, they are close together with no
regular arrangement; in a gas, they are far apart with no regular arrangement.
Particles in a solid vibrate about fixed positions and do not generally move in relation to one another; in a liquid, they move
past each other but remain in essentially constant contact; in a gas, they move independently of one another except when they
collide.
The differences in the properties of a solid, liquid, or gas reflect the strengths of the attractive forces between the atoms, molecules,
or ions that make up each phase. The phase in which a substance exists depends on the relative extents of its intermolecular forces
(IMFs) and the kinetic energies (KE) of its molecules. IMFs are the various forces of attraction that may exist between the atoms
and molecules of a substance due to electrostatic phenomena, as will be detailed in this module. These forces serve to hold particles
close together, whereas the particles’ KE provides the energy required to overcome the attractive forces and thus increase the
distance between particles. Figure 10.1.1 illustrates how changes in physical state may be induced by changing the temperature,
hence, the average KE, of a given substance.

Figure 10.1.1 : Transitions between solid, liquid, and gaseous states of a substance occur when conditions of temperature or
pressure favor the associated changes in intermolecular forces. (Note: The space between particles in the gas phase is much greater
than shown.)
Three sealed flasks are labeled, “Crystalline solid,” “Liquid,” and “Gas,” from left to right. The first flask holds a cube composed
of small spheres sitting on the bottom while the second flask shows a lot of small spheres in the bottom that are spaced a small
distance apart from one another and have lines around them to indicate motion. The third flask shows a few spheres spread far from
one another with larger lines to indicate motion. There is a right-facing arrow that spans the top of all three flasks. The arrow is
labeled, “Increasing K E ( temperature ).” There is a left-facing arrow that spans the bottom of all three flasks. The arrow is
labeled, “Increasing I M F.”
As an example of the processes depicted in this figure, consider a sample of water. When gaseous water is cooled sufficiently, the
attractions between H2O molecules will be capable of holding them together when they come into contact with each other; the gas
condenses, forming liquid H2O. For example, liquid water forms on the outside of a cold glass as the water vapor in the air is
cooled by the cold glass, as seen in Figure 10.1.2.

Access for free at OpenStax 10.1.1 https://chem.libretexts.org/@go/page/38210


Figure 10.1.2 : Condensation forms when water vapor in the air is cooled enough to form liquid water, such as (a) on the outside of
a cold beverage glass or (b) in the form of fog. (credit a: modification of work by Jenny Downing; credit b: modification of work
by Cory Zanker)
Image a shows a brown colored beverage in a glass with condensation on the outside. Image b shows a body of water with fog
hovering above the surface of the water.
We can also liquefy many gases by compressing them, if the temperature is not too high. The increased pressure brings the
molecules of a gas closer together, such that the attractions between the molecules become strong relative to their KE.
Consequently, they form liquids. Butane, C4H10, is the fuel used in disposable lighters and is a gas at standard temperature and
pressure. Inside the lighter’s fuel compartment, the butane is compressed to a pressure that results in its condensation to the liquid
state, as shown in Figure 10.1.3.

Figure 10.1.3 : Gaseous butane is compressed within the storage compartment of a disposable lighter, resulting in its condensation
to the liquid state. (credit: modification of work by “Sam-Cat”/Flickr)
Finally, if the temperature of a liquid becomes sufficiently low, or the pressure on the liquid becomes sufficiently high, the
molecules of the liquid no longer have enough KE to overcome the IMF between them, and a solid forms. A more thorough
discussion of these and other changes of state, or phase transitions, is provided in a later module of this chapter.

10.1.1: Forces between Molecules


Under appropriate conditions, the attractions between all gas molecules will cause them to form liquids or solids. This is due to
intermolecular forces, not intramolecular forces. Intramolecular forces are those within the molecule that keep the molecule
together, for example, the bonds between the atoms. Intermolecular forces are the attractions between molecules, which determine
many of the physical properties of a substance. Figure 10.1.4 illustrates these different molecular forces. The strengths of these
attractive forces vary widely, though usually the IMFs between small molecules are weak compared to the intramolecular forces

Access for free at OpenStax 10.1.2 https://chem.libretexts.org/@go/page/38210


that bond atoms together within a molecule. For example, to overcome the IMFs in one mole of liquid HCl and convert it into
gaseous HCl requires only about 17 kilojoules. However, to break the covalent bonds between the hydrogen and chlorine atoms in
one mole of HCl requires about 25 times more energy—430 kilojoules.

Figure 10.1.4 : Intramolecular forces keep a molecule intact. Intermolecular forces hold multiple molecules together and determine
many of a substance’s properties.
An image is shown in which two molecules composed of a green sphere labeled “C l” connected on the right to a white sphere
labeled “H” are near one another with a dotted line labeled “Intermolecular force ( weak )” drawn between them. A line connects
the two spheres in each molecule and the line is labeled “Intramolecular force ( strong ).”
All of the attractive forces between neutral atoms and molecules are known as van der Waals forces, although they are usually
referred to more informally as intermolecular attraction. We will consider the various types of IMFs in the next three sections of
this module.

10.1.2: Dispersion Forces


One of the three van der Waals forces is present in all condensed phases, regardless of the nature of the atoms or molecules
composing the substance. This attractive force is called the London dispersion force in honor of German-born American physicist
Fritz London who, in 1928, first explained it. This force is often referred to as simply the dispersion force. Because the electrons of
an atom or molecule are in constant motion (or, alternatively, the electron’s location is subject to quantum-mechanical variability),
at any moment in time, an atom or molecule can develop a temporary, instantaneous dipole if its electrons are distributed
asymmetrically. The presence of this dipole can, in turn, distort the electrons of a neighboring atom or molecule, producing an
induced dipole. These two rapidly fluctuating, temporary dipoles thus result in a relatively weak electrostatic attraction between the
species—a so-called dispersion force like that illustrated in Figure 10.1.5.

Figure 10.1.5 : Dispersion forces result from the formation of temporary dipoles, as illustrated here for two nonpolar diatomic
molecules.
Two pairs of molecules are shown where each molecule has one larger blue side labeled “delta sign, negative sign” and a smaller
red side labeled “delta sign, positive sign.” Toward the middle of the both molecules, but still on each distinct side, is a black dot.
Between the two images is a dotted line labeled, “Attractive force.” In the first image, the red and blue sides are labeled, “Unequal
distribution of electrons.” Below both images are brackets. The brackets are labeled, “Temporary dipoles.”
Dispersion forces that develop between atoms in different molecules can attract the two molecules to each other. The forces are
relatively weak, however, and become significant only when the molecules are very close. Larger and heavier atoms and molecules
exhibit stronger dispersion forces than do smaller and lighter atoms and molecules. F2 and Cl2 are gases at room temperature
(reflecting weaker attractive forces); Br2 is a liquid, and I2 is a solid (reflecting stronger attractive forces). Trends in observed
melting and boiling points for the halogens clearly demonstrate this effect, as seen in Table 10.1.1.
Table 10.1.1 : Melting and Boiling Points of the Halogens
Halogen Molar Mass Atomic Radius Melting Point Boiling Point

fluorine, F2 38 g/mol 72 pm 53 K 85 K

Access for free at OpenStax 10.1.3 https://chem.libretexts.org/@go/page/38210


Halogen Molar Mass Atomic Radius Melting Point Boiling Point

chlorine, Cl2 71 g/mol 99 pm 172 K 238 K

bromine, Br2 160 g/mol 114 pm 266 K 332 K

iodine, I2 254 g/mol 133 pm 387 K 457 K

astatine, At2 420 g/mol 150 pm 575 K 610 K

The increase in melting and boiling points with increasing atomic/molecular size may be rationalized by considering how the
strength of dispersion forces is affected by the electronic structure of the atoms or molecules in the substance. In a larger atom, the
valence electrons are, on average, farther from the nuclei than in a smaller atom. Thus, they are less tightly held and can more
easily form the temporary dipoles that produce the attraction. The measure of how easy or difficult it is for another electrostatic
charge (for example, a nearby ion or polar molecule) to distort a molecule’s charge distribution (its electron cloud) is known as
polarizability. A molecule that has a charge cloud that is easily distorted is said to be very polarizable and will have large
dispersion forces; one with a charge cloud that is difficult to distort is not very polarizable and will have small dispersion forces.

 Example 10.1.1: London Forces and Their Effects

Order the following compounds of a group 14 element and hydrogen from lowest to highest boiling point: CH4, SiH4, GeH4,
and SnH4. Explain your reasoning.

Solution
Applying the skills acquired in the chapter on chemical bonding and molecular geometry, all of these compounds are predicted
to be nonpolar, so they may experience only dispersion forces: the smaller the molecule, the less polarizable and the weaker the
dispersion forces; the larger the molecule, the larger the dispersion forces. The molar masses of CH4, SiH4, GeH4, and SnH4
are approximately 16 g/mol, 32 g/mol, 77 g/mol, and 123 g/mol, respectively. Therefore, CH4 is expected to have the lowest
boiling point and SnH4 the highest boiling point. The ordering from lowest to highest boiling point is expected to be
CH4 < SiH4 < GeH4 < SnH4
A graph of the actual boiling points of these compounds versus the period of the group 14 elements shows this prediction to be
correct:

A line graph, titled “Carbon Family,” is shown where the y-axis is labeled “Temperature, ( degree sign C )” and has values of
“negative 200” to “negative 40” from bottom to top in increments of 20. The x-axis is labeled “Period” and has values of “0” to
“5” in increments of 1. The first point on the graph is labeled “C H subscript 4” and is at point “2, negative 160.” The second
point on the graph is labeled “S i H subscript 4” and is at point “3, negative 120” while the third point on the graph is labeled
“G e H subscript 4” and is at point “4, negative 100.” The fourth point on the graph is labeled “S n H subscript 4” and is at
point “5, negative 60.”

Access for free at OpenStax 10.1.4 https://chem.libretexts.org/@go/page/38210


 Exercise 10.1.1

Order the following hydrocarbons from lowest to highest boiling point: C2H6, C3H8, and C4H10.

Answer
All of these compounds are nonpolar and only have London dispersion forces: the larger the molecule, the larger the
dispersion forces and the higher the boiling point. The ordering from lowest to highest boiling point is therefore
C2H6 < C3H8 < C4H10.

The shapes of molecules also affect the magnitudes of the dispersion forces between them. For example, boiling points for the
isomers n-pentane, isopentane, and neopentane (shown in Figure 10.1.6) are 36 °C, 27 °C, and 9.5 °C, respectively. Even though
these compounds are composed of molecules with the same chemical formula, C5H12, the difference in boiling points suggests that
dispersion forces in the liquid phase are different, being greatest for n-pentane and least for neopentane. The elongated shape of n-
pentane provides a greater surface area available for contact between molecules, resulting in correspondingly stronger dispersion
forces. The more compact shape of isopentane offers a smaller surface area available for intermolecular contact and, therefore,
weaker dispersion forces. Neopentane molecules are the most compact of the three, offering the least available surface area for
intermolecular contact and, hence, the weakest dispersion forces. This behavior is analogous to the connections that may be formed
between strips of VELCRO brand fasteners: the greater the area of the strip’s contact, the stronger the connection.

Figure 10.1.6 : The strength of the dispersion forces increases with the contact area between molecules, as demonstrated by the
boiling points of these pentane isomers.
Three images of molecules are shown. The first shows a cluster of large, gray spheres each bonded together and to several smaller,
white spheres. There is a gray, jagged line and then the mirror image of the first cluster of spheres is shown. Above these two
clusters is the label, “Small contact area, weakest attraction,” and below is the label, “neopentane boiling point: 9.5 degrees C.” The
second shows a chain of three gray spheres bonded by the middle sphere to a fourth gray sphere. Each gray sphere is bonded to
several smaller, white spheres. There is a jagged, gray line and then the mirror image of the first chain appears. Above these two
chains is the label, “Less surface area, less attraction,” and below is the label, “isopentane boiling point: 27 degrees C.” The third
image shows a chain of five gray spheres bonded together and to several smaller, white spheres. There is a jagged gray line and
then the mirror image of the first chain appears. Above these chains is the label, “Large contact area, strong attraction,” and below
is the label, “n-pentane boiling point 36 degrees C.”

 Applications: Geckos and Intermolecular Forces

Geckos have an amazing ability to adhere to most surfaces. They can quickly run up smooth walls and across ceilings that have
no toe-holds, and they do this without having suction cups or a sticky substance on their toes. And while a gecko can lift its
feet easily as it walks along a surface, if you attempt to pick it up, it sticks to the surface. How are geckos (as well as spiders
and some other insects) able to do this? Although this phenomenon has been investigated for hundreds of years, scientists only
recently uncovered the details of the process that allows geckos’ feet to behave this way.

Access for free at OpenStax 10.1.5 https://chem.libretexts.org/@go/page/38210


Figure 10.1.7 : Geckos’ toes contain large numbers of tiny hairs (setae), which branch into many triangular tips (spatulae).
Geckos adhere to surfaces because of van der Waals attractions between the surface and a gecko’s millions of spatulae. By
changing how the spatulae contact the surface, geckos can turn their stickiness “on” and “off.” (credit photo: modification of
work by “JC*+A!”/Flickr)
Three figures are shown. The first is a photo of the bottom of a gecko’s foot. The second is bigger version which shows the
setae. The third is a bigger version of the setae and shows the spatulae.

Geckos’ toes are covered with hundreds of thousands of tiny hairs known as setae, with each seta, in turn, branching into
hundreds of tiny, flat, triangular tips called spatulae. The huge numbers of spatulae on its setae provide a gecko, shown in
Figure 10.1.7, with a large total surface area for sticking to a surface. In 2000, Kellar Autumn, who leads a multi-institutional
gecko research team, found that geckos adhered equally well to both polar silicon dioxide and nonpolar gallium arsenide. This
proved that geckos stick to surfaces because of dispersion forces—weak intermolecular attractions arising from temporary,
synchronized charge distributions between adjacent molecules. Although dispersion forces are very weak, the total attraction
over millions of spatulae is large enough to support many times the gecko’s weight.
In 2014, two scientists developed a model to explain how geckos can rapidly transition from “sticky” to “non-sticky.” Alex
Greaney and Congcong Hu at Oregon State University described how geckos can achieve this by changing the angle between
their spatulae and the surface. Geckos’ feet, which are normally nonsticky, become sticky when a small shear force is applied.
By curling and uncurling their toes, geckos can alternate between sticking and unsticking from a surface, and thus easily move
across it. Further investigations may eventually lead to the development of better adhesives and other applications.

Smart materials (1 of 5): Gecko Adhes…


Adhes…

Watch this video to learn more about Kellar Autumn’s research that determined that van der Waals forces are responsible for a
gecko’s ability to cling and climb.

10.1.3: Dipole-Dipole Attractions


Recall from the chapter on chemical bonding and molecular geometry that polar molecules have a partial positive charge on one
side and a partial negative charge on the other side of the molecule—a separation of charge called a dipole. Consider a polar
molecule such as hydrogen chloride, HCl. In the HCl molecule, the more electronegative Cl atom bears the partial negative charge,
whereas the less electronegative H atom bears the partial positive charge. An attractive force between HCl molecules results from
the attraction between the positive end of one HCl molecule and the negative end of another. This attractive force is called a dipole-
dipole attraction—the electrostatic force between the partially positive end of one polar molecule and the partially negative end of
another, as illustrated in Figure 10.1.8.

Access for free at OpenStax 10.1.6 https://chem.libretexts.org/@go/page/38210


Figure 10.1.8 : This image shows two arrangements of polar molecules, such as HCl, that allow an attraction between the partial
negative end of one molecule and the partial positive end of another.
Two pairs of molecules are shown where each molecule has one larger blue side labeled “delta sign, negative sign” and a smaller
red side labeled “delta sign, positive sign. In the first pair, the red sides of the two molecules both face to the left and the blue side
to the right. A horizontal dotted line lies in between the two. In the second pair, the molecules face up and down, with the red and
blue ends aligning. A horizontal dotted line lies between the red and blue ends facing upward and another lies between the red and
blue ends facing downward.

The effect of a dipole-dipole attraction is apparent when we compare the properties of HCl molecules to nonpolar F2 molecules.
Both HCl and F2 consist of the same number of atoms and have approximately the same molecular mass. At a temperature of 150
K, molecules of both substances would have the same average KE. However, the dipole-dipole attractions between HCl molecules
are sufficient to cause them to “stick together” to form a liquid, whereas the relatively weaker dispersion forces between nonpolar
F2 molecules are not, and so this substance is gaseous at this temperature. The higher normal boiling point of HCl (188 K)
compared to F2 (85 K) is a reflection of the greater strength of dipole-dipole attractions between HCl molecules, compared to the
attractions between nonpolar F2 molecules. We will often use values such as boiling or freezing points, or enthalpies of
vaporization or fusion, as indicators of the relative strengths of IMFs of attraction present within different substances.

 Example 10.1.2: Dipole-Dipole Forces and Their Effects


Predict which will have the higher boiling point: N2 or CO. Explain your reasoning.

Solution
CO and N2 are both diatomic molecules with masses of about 28 amu, so they experience similar London dispersion forces.
Because CO is a polar molecule, it experiences dipole-dipole attractions. Because N2 is nonpolar, its molecules cannot exhibit
dipole-dipole attractions. The dipole-dipole attractions between CO molecules are comparably stronger than the dispersion
forces between nonpolar N2 molecules, so CO is expected to have the higher boiling point.
A common method for preparing oxygen is the decomposition

 Exercise 10.1.2

Predict which will have the higher boiling point: ICl or Br . Explain your reasoning.
2

Answer
ICl. ICl and Br2 have similar masses (~160 amu) and therefore experience similar London dispersion forces. ICl is polar
and thus also exhibits dipole-dipole attractions; Br2 is nonpolar and does not. The relatively stronger dipole-dipole
attractions require more energy to overcome, so ICl will have the higher boiling point.

10.1.4: Hydrogen Bonding


Nitrosyl fluoride (ONF, molecular mass 49 amu) is a gas at room temperature. Water (H2O, molecular mass 18 amu) is a liquid,
even though it has a lower molecular mass. We clearly cannot attribute this difference between the two compounds to dispersion
forces. Both molecules have about the same shape and ONF is the heavier and larger molecule. It is, therefore, expected to
experience more significant dispersion forces. Additionally, we cannot attribute this difference in boiling points to differences in
the dipole moments of the molecules. Both molecules are polar and exhibit comparable dipole moments. The large difference
between the boiling points is due to a particularly strong dipole-dipole attraction that may occur when a molecule contains a
hydrogen atom bonded to a fluorine, oxygen, or nitrogen atom (the three most electronegative elements). The very large difference
in electronegativity between the H atom (2.1) and the atom to which it is bonded (4.0 for an F atom, 3.5 for an O atom, or 3.0 for a

Access for free at OpenStax 10.1.7 https://chem.libretexts.org/@go/page/38210


N atom), combined with the very small size of a H atom and the relatively small sizes of F, O, or N atoms, leads to highly
concentrated partial charges with these atoms. Molecules with F-H, O-H, or N-H moieties are very strongly attracted to similar
moieties in nearby molecules, a particularly strong type of dipole-dipole attraction called hydrogen bonding. Examples of hydrogen
bonds include HF⋯HF, H2O⋯HOH, and H3N⋯HNH2, in which the hydrogen bonds are denoted by dots. Figure 10.1.9 illustrates
hydrogen bonding between water molecules.

Figure 10.1.9 : Water molecules participate in multiple hydrogen-bonding interactions with nearby water molecules.
Five water molecules are shown near one another, but not touching. A dotted line lies between many of the hydrogen atoms on one
molecule and the oxygen atom on another molecule.
Despite use of the word “bond,” keep in mind that hydrogen bonds are intermolecular attractive forces, not intramolecular
attractive forces (covalent bonds). Hydrogen bonds are much weaker than covalent bonds, only about 5 to 10% as strong, but are
generally much stronger than other dipole-dipole attractions and dispersion forces.
Hydrogen bonds have a pronounced effect on the properties of condensed phases (liquids and solids). For example, consider the
trends in boiling points for the binary hydrides of group 15 (NH3, PH3, AsH3, and SbH3), group 16 hydrides (H2O, H2S, H2Se, and
H2Te), and group 17 hydrides (HF, HCl, HBr, and HI). The boiling points of the heaviest three hydrides for each group are plotted
in Figure 10.1.10. As we progress down any of these groups, the polarities of the molecules decrease slightly, whereas the sizes of
the molecules increase substantially. The effect of increasingly stronger dispersion forces dominates that of increasingly weaker
dipole-dipole attractions, and the boiling points are observed to increase steadily. For the group 15, 16, and 17 hydrides, the boiling
points for each class of compounds increase with increasing molecular mass for elements in periods 3, 4, and 5.

Access for free at OpenStax 10.1.8 https://chem.libretexts.org/@go/page/38210


Figure 10.1.10: In comparison to periods 3−5, the binary hydrides of period 2 elements in groups 17, 16 and 15 (F, O and N,
respectively) exhibit anomalously high boiling points due to hydrogen bonding.
A line graph is shown where the y-axis is labeled “Boiling point (, degree sign, C )” and has values of “ negative 150” to “150”
from bottom to top in increments of 50. The x-axis is labeled “Period” and has values of “0” to “5” in increments of 1. Three lines
are shown on the graph and are labeled in the legend. The red line is labeled as “halogen family,” the blue is “oxygen family” and
the green is “nitrogen family.” The first point on the red line is labeled “question mark” and is at point “2, negative 120”. The
second point on the line is labeled “H C l” and is at point “3, negative 80” while the third point on the line is labeled “H B r” and is
at point “4, negative 60”. The fourth point on the line is labeled “H I” and is at point “5, negative 40.” The first point on the green
line is labeled “question mark” and is at point “2, negative 125.” The second point on the line is labeled “P H, subscript 3” and is at
point “3, negative 80” while the third point on the line is labeled “A s H, subscript 3” and is at point “4, negative 55.” The fourth
point on the line is labeled “S b H, subscript 3” and is at point “5, negative 10.” The first point on the blue line is labeled “question
mark” and is at point “2, negative 80.” The second point on the line is labeled “H, subscript 2, S” and is at point “3, negative 55”
while the third point on the line is labeled “H, subscript 2, S e” and is at point “4, negative 45.” The fourth point on the line is
labeled “H, subscript 2, T e” and is at point “5, negative 3.”
If we use this trend to predict the boiling points for the lightest hydride for each group, we would expect NH3 to boil at about −120
°C, H2O to boil at about −80 °C, and HF to boil at about −110 °C. However, when we measure the boiling points for these
compounds, we find that they are dramatically higher than the trends would predict, as shown in Figure 10.1.10. The stark contrast
between our naïve predictions and reality provides compelling evidence for the strength of hydrogen bonding.

 Example 10.1.3: Effect of Hydrogen Bonding on Boiling Points

Consider the compounds dimethylether (CH3OCH3), ethanol (CH3CH2OH), and propane (CH3CH2CH3). Their boiling points,
not necessarily in order, are −42.1 °C, −24.8 °C, and 78.4 °C. Match each compound with its boiling point. Explain your
reasoning.

Solution
The VSEPR-predicted shapes of CH3OCH3, CH3CH2OH, and CH3CH2CH3 are similar, as are their molar masses (46 g/mol,
46 g/mol, and 44 g/mol, respectively), so they will exhibit similar dispersion forces. Since CH3CH2CH3 is nonpolar, it may
exhibit only dispersion forces. Because CH3OCH3 is polar, it will also experience dipole-dipole attractions. Finally,
CH3CH2OH has an −OH group, and so it will experience the uniquely strong dipole-dipole attraction known as hydrogen
bonding. So the ordering in terms of strength of IMFs, and thus boiling points, is CH3CH2CH3 < CH3OCH3 < CH3CH2OH.
The boiling point of propane is −42.1 °C, the boiling point of dimethylether is −24.8 °C, and the boiling point of ethanol is
78.5 °C.

Access for free at OpenStax 10.1.9 https://chem.libretexts.org/@go/page/38210


 Exercise 10.1.3
Ethane (CH3CH3) has a melting point of −183 °C and a boiling point of −89 °C. Predict the melting and boiling points for
methylamine (CH3NH2). Explain your reasoning.

Answer
The melting point and boiling point for methylamine are predicted to be significantly greater than those of ethane. CH3CH3
and CH3NH2 are similar in size and mass, but methylamine possesses an −NH group and therefore may exhibit hydrogen
bonding. This greatly increases its IMFs, and therefore its melting and boiling points. It is difficult to predict values, but the
known values are a melting point of −93 °C and a boiling point of −6 °C.

10.1.5: Hydrogen Bonding and DNA


Deoxyribonucleic acid (DNA) is found in every living organism and contains the genetic information that determines the
organism’s characteristics, provides the blueprint for making the proteins necessary for life, and serves as a template to pass this
information on to the organism’s offspring. A DNA molecule consists of two (anti-)parallel chains of repeating nucleotides, which
form its well-known double helical structure, as shown in Figure 10.1.10.

Access for free at OpenStax 10.1.10 https://chem.libretexts.org/@go/page/38210


Figure 10.1.11: Two separate DNA molecules form a double-stranded helix in which the molecules are held together via hydrogen
bonding. (credit: modification of work by Jerome Walker, Dennis Myts)
Two images are shown. The first lies on the left side of the page and shows a helical structure like a twisted ladder where the rungs
of the ladder, labeled “Base pair” are red, yellow, green and blue paired bars. The red and yellow bars, which are always paired
together, are labeled in the legend, which is titled “Nitrogenous bases” as “adenine” and “thymine,” respectively. The blue and
green bars, which are always paired together, are labeled in the legend as “guanine” and “cytosine,” respectively. At the top of the
helical structure, the left-hand side rail, or “Sugar, dash, phosphate backbone,” is labeled as “3, prime” while the right is labeled as
“5, prime.” These labels are reversed at the bottom of the helix. To the right of the page is a large Lewis structure. The top left
corner of this structure, labeled “5, prime,” shows a phosphorus atom single bonded to three oxygen atoms, one of which has a
superscripted negative charge, and double bonded to a fourth oxygen atom. One of the single bonded oxygen atoms is single
bonded to the left corner of a five-membered ring with an oxygen atom at its top point and which is single bonded to an oxygen
atom on the bottom left. This oxygen atom is single bonded to a phosphorus atom that is single bonded to two other hydrogen
atoms and double bonded to a fourth oxygen atom. The lower left of these oxygen atoms is single bonded to another oxygen atom
that is single bonded to a five-membered ring with an oxygen in the upper bonding site. The bottom left of this ring has a hydroxyl
group attached to it while the upper right carbon is single bonded to a nitrogen atom that is part of a five-membered ring bonded to
a six-membered ring. Both of these rings have points of unsaturation and nitrogen atoms bonded into their structures. On the right
side of the six-membered ring are two single bonded amine groups and a double bonded oxygen. Three separate dotted lines extend
from these sites to corresponding sites on a second six-membered ring. This ring has points of unsaturation and a nitrogen atom in
the bottom right bonding position that is single bonded to a five-membered ring on the right side of the image. This ring is single
bonded to a carbon that is single bonded to an oxygen that is single bonded to a phosphorus. The phosphorus is single bonded to
two other oxygen atoms and double bonded to a fourth oxygen atom. This group is labeled “5, prime.” The five-membered ring is
also bonded on the top side to an oxygen that is bonded to a phosphorus single bonded to two other oxygen atoms and double
bonded to a fourth oxygen atom. The upper left oxygen of this group is single bonded to a carbon that is single bonded to a five-
membered ring with an oxygen in the bottom bonding position. This ring has a hydroxyl group on its upper right side that is labeled
“3, prime” and is bonded on the left side to a nitrogen that is a member of a five-membered ring. This ring is bonded to a six-
membered ring and both have points of unsaturation. This ring has a nitrogen on the left side, as well as an amine group, that have
two dotted lines leading from them to an oxygen and amine group on a six membered ring. These dotted lines are labeled
“Hydrogen bonds.” The six membered ring also has a double bonded oxygen on its lower side and a nitrogen atom on its left side
that is single bonded to a five-membered ring. This ring connects to the two phosphate groups mentioned at the start of this to form
a large circle. The name “guanine” is written below the lower left side of this image while the name “cytosine” is written on the
lower right. The name “thymine” is written above the right side of the image and “adenine” is written on the top right. Three
sections are indicated below the images where the left is labeled “Sugar, dash, phosphate backbone,” the middle is labeled “Bases”
and the right is labeled “Sugar, dash, phosphate backbone.”
Each nucleotide contains a (deoxyribose) sugar bound to a phosphate group on one side, and one of four nitrogenous bases on the
other. Two of the bases, cytosine (C) and thymine (T), are single-ringed structures known as pyrimidines. The other two, adenine
(A) and guanine (G), are double-ringed structures called purines. These bases form complementary base pairs consisting of one
purine and one pyrimidine, with adenine pairing with thymine, and cytosine with guanine. Each base pair is held together by
hydrogen bonding. A and T share two hydrogen bonds, C and G share three, and both pairings have a similar shape and structure
Figure 10.1.12

Access for free at OpenStax 10.1.11 https://chem.libretexts.org/@go/page/38210


Figure 10.1.12: The geometries of the base molecules result in maximum hydrogen bonding between adenine and thymine (AT)
and between guanine and cytosine (GC), so-called “complementary base pairs.”
A large Lewis structure is shown. The top left corner of this structure, labeled “5, prime,” shows a phosphorus atom single bonded
to three oxygen atoms, one of which has a superscripted negative charge, and double bonded to a fourth oxygen atom. One of the
single bonded oxygen atoms is single bonded to the left corner of a five-membered ring with an oxygen atom at its top point and
which is single bonded to an oxygen atom on the bottom left. This oxygen atom is single bonded to a phosphorus atom that is
single bonded to two other hydrogen atoms and double bonded to a fourth oxygen atom. The lower left of these oxygen atoms is
single bonded to another oxygen atom that is single bonded to a five-membered ring with an oxygen in the upper bonding site. The
bottom left of this ring has a hydroxyl group attached to it while the upper right carbon is single bonded to a nitrogen atom that is
part of a five-membered ring bonded to a six-membered ring. Both of these rings have points of unsaturation and nitrogen atoms
bonded into their structures. On the right side of the six-membered ring are two single bonded amine groups and a double bonded
oxygen. Three separate dotted lines extend from these sites to corresponding sites on a second six-membered ring. This ring has
points of unsaturation and a nitrogen atom in the bottom right bonding position that is single bonded to a five-membered ring on
the right side of the image. This ring is single bonded to a carbon that is single bonded to an oxygen that is single bonded to a
phosphorus. The phosphorus is single bonded to two other oxygen atoms and double bonded to a fourth oxygen atom. This group is
labeled “5, prime.” The five-membered ring is also bonded on the top side to an oxygen that is bonded to a phosphorus single
bonded to two other oxygen atoms and double bonded to a fourth oxygen atom. The upper left oxygen of this group is single
bonded to a carbon that is single bonded to a five-membered ring with an oxygen in the bottom bonding position. This ring has a
hydroxyl group on its upper right side that is labeled “3, prime” and is bonded on the left side to a nitrogen that is a member of a
five-membered ring. This ring is bonded to a six-membered ring and both have points of unsaturation. This ring has a nitrogen on
the left side, as well as an amine group, that have two dotted lines leading from them to an oxygen and amine group on a six
membered ring. These dotted lines are labeled “Hydrogen bonds.” The six membered ring also has a double bonded oxygen on its
lower side and a nitrogen atom on its left side that is single bonded to a five-membered ring. This ring connects to the two
phosphate groups mentioned at the start of this to form a large circle. The name “guanine” is written below the lower left side of
this image while the name “cytosine” is written on the lower right. The name “thymine” is written above the right side of the image
and “adenine” is written on the top right. Three sections are indicated below the images where the left is labeled “Sugar, dash,
phosphate backbone,” the middle is labeled “Bases” and the right is labeled “Sugar, dash, phosphate backbone.”

The cumulative effect of millions of hydrogen bonds effectively holds the two strands of DNA together. Importantly, the two
strands of DNA can relatively easily “unzip” down the middle since hydrogen bonds are relatively weak compared to the covalent
bonds that hold the atoms of the individual DNA molecules together. This allows both strands to function as a template for
replication.

Summary
The physical properties of condensed matter (liquids and solids) can be explained in terms of the kinetic molecular theory. In a
liquid, intermolecular attractive forces hold the molecules in contact, although they still have sufficient kinetic energy to move past
each other. Intermolecular attractive forces, collectively referred to as van der Waals forces, are responsible for the behavior of
liquids and solids and are electrostatic in nature. Dipole-dipole attractions result from the electrostatic attraction of the partial
negative end of one dipolar molecule for the partial positive end of another. The temporary dipole that results from the motion of
the electrons in an atom can induce a dipole in an adjacent atom and give rise to the London dispersion force. London forces
increase with increasing molecular size. Hydrogen bonds are a special type of dipole-dipole attraction that results when hydrogen is
bonded to one of the three most electronegative elements: F, O, or N.

Access for free at OpenStax 10.1.12 https://chem.libretexts.org/@go/page/38210


Glossary
dipole-dipole attraction
intermolecular attraction between two permanent dipoles

dispersion force
(also, London dispersion force) attraction between two rapidly fluctuating, temporary dipoles; significant only when particles
are very close together

hydrogen bonding
occurs when exceptionally strong dipoles attract; bonding that exists when hydrogen is bonded to one of the three most
electronegative elements: F, O, or N

induced dipole
temporary dipole formed when the electrons of an atom or molecule are distorted by the instantaneous dipole of a neighboring
atom or molecule

instantaneous dipole
temporary dipole that occurs for a brief moment in time when the electrons of an atom or molecule are distributed
asymmetrically

intermolecular force
noncovalent attractive force between atoms, molecules, and/or ions

polarizability
measure of the ability of a charge to distort a molecule’s charge distribution (electron cloud)

van der Waals force


attractive or repulsive force between molecules, including dipole-dipole, dipole-induced dipole, and London dispersion forces;
does not include forces due to covalent or ionic bonding, or the attraction between ions and molecules

This page titled 10.1: Intermolecular Forces is shared under a CC BY 4.0 license and was authored, remixed, and/or curated by OpenStax via
source content that was edited to the style and standards of the LibreTexts platform; a detailed edit history is available upon request.

Access for free at OpenStax 10.1.13 https://chem.libretexts.org/@go/page/38210


10.2: Properties of Liquids
 Learning Objectives
Distinguish between adhesive and cohesive forces
Define viscosity, surface tension, and capillary rise
Describe the roles of intermolecular attractive forces in each of these properties/phenomena

When you pour a glass of water, or fill a car with gasoline, you observe that water and gasoline flow freely. But when you pour
syrup on pancakes or add oil to a car engine, you note that syrup and motor oil do not flow as readily. The viscosity of a liquid is a
measure of its resistance to flow. Water, gasoline, and other liquids that flow freely have a low viscosity. Honey, syrup, motor oil,
and other liquids that do not flow freely, like those shown in Figure 10.2.1, have higher viscosities. We can measure viscosity by
measuring the rate at which a metal ball falls through a liquid (the ball falls more slowly through a more viscous liquid) or by
measuring the rate at which a liquid flows through a narrow tube (more viscous liquids flow more slowly).

Figure 10.2.1 : (a) Honey and (b) motor oil are examples of liquids with high viscosities; they flow slowly. (credit a: modification
of work by Scott Bauer; credit b: modification of work by David Nagy)
Two photographs are shown and labeled “a” and “b.” Photo a shows a jar of honey with a dipper drizzling it onto a biscuit. More
biscuits are shown in a basket in the background. Photo b shows the engine of a car and a person adding motor oil to the engine.
The IMFs between the molecules of a liquid, the size and shape of the molecules, and the temperature determine how easily a
liquid flows. As Table 10.2.1 shows, the more structurally complex are the molecules in a liquid and the stronger the IMFs between
them, the more difficult it is for them to move past each other and the greater is the viscosity of the liquid. As the temperature
increases, the molecules move more rapidly and their kinetic energies are better able to overcome the forces that hold them
together; thus, the viscosity of the liquid decreases.
Table 10.2.1 : Viscosities of Common Substances at 25 °C
Substance Formula Viscosity (mPa·s)

water H2O 0.890

mercury Hg 1.526

ethanol C2H5OH 1.074

octane C8H18 0.508

ethylene glycol CH2(OH)CH2(OH) 16.1

honey variable ~2,000–10,000

motor oil variable ~50–500

The various IMFs between identical molecules of a substance are examples of cohesive forces. The molecules within a liquid are
surrounded by other molecules and are attracted equally in all directions by the cohesive forces within the liquid. However, the
molecules on the surface of a liquid are attracted only by about one-half as many molecules. Because of the unbalanced molecular
attractions on the surface molecules, liquids contract to form a shape that minimizes the number of molecules on the surface—that
is, the shape with the minimum surface area. A small drop of liquid tends to assume a spherical shape, as shown in Figure 10.2.2,

Access for free at OpenStax 10.2.1 https://chem.libretexts.org/@go/page/38211


because in a sphere, the ratio of surface area to volume is at a minimum. Larger drops are more greatly affected by gravity, air
resistance, surface interactions, and so on, and as a result, are less spherical.

Figure 10.2.2 : Attractive forces result in a spherical water drop that minimizes surface area; cohesive forces hold the sphere
together; adhesive forces keep the drop attached to the web. (credit photo: modification of work by “OliBac”/Flickr)
A photo of a spider’s web with droplets of water attached to it is shown. Two images are shown the right of the photo and arrows
lead from the photo to the images. The upper image shows twenty eight blue spheres stacked one atop the other in the bottom of a
circular background. Five arrows are drawn pointing to the sides and downward from the sphere in the top middle of the drawing.
The lower image shows another circular background of the same size as the first, but this time the blue spheres fill the image and
are packed closely together. A sphere in the middle has six arrows pointing in all directions away from it.
Surface tension is defined as the energy required to increase the surface area of a liquid, or the force required to increase the length
of a liquid surface by a given amount. This property results from the cohesive forces between molecules at the surface of a liquid,
and it causes the surface of a liquid to behave like a stretched rubber membrane. Surface tensions of several liquids are presented in
Table 10.2.2.
Table 10.2.2 : Surface Tensions of Common Substances at 25 °C
Substance Formula Surface Tension (mN/m)

water H2O 71.99

mercury Hg 458.48

ethanol C2H5OH 21.97

octane C8H18 21.14

ethylene glycol CH2(OH)CH2(OH) 47.99

Among common liquids, water exhibits a distinctly high surface tension due to strong hydrogen bonding between its molecules. As
a result of this high surface tension, the surface of water represents a relatively “tough skin” that can withstand considerable force
without breaking. A steel needle carefully placed on water will float. Some insects, like the one shown in Figure 10.2.3, even
though they are denser than water, move on its surface because they are supported by the surface tension.

Figure 10.2.3 : Surface tension (right) prevents this insect, a “water strider,” from sinking into the water.
A photo and a diagram as shown and a right-facing arrow leads from the photo to the image. The photo shows an insect standing on
the surface of a sample of water. The image shows a square background that is two thirds covered in blue spheres that are closely
packet together. A brown line starts at the upper left corner of the background and rests on top of the first row of spheres. The
sphere directly under this low point of the line has four arrows drawn on it that face to both sides and downward. A sphere in the
bottom center of the image has six arrows drawn on it that all face outward in different directions.
The IMFs of attraction between two different molecules are called adhesive forces. Consider what happens when water comes into
contact with some surface. If the adhesive forces between water molecules and the molecules of the surface are weak compared to
the cohesive forces between the water molecules, the water does not “wet” the surface. For example, water does not wet waxed
surfaces or many plastics such as polyethylene. Water forms drops on these surfaces because the cohesive forces within the drops

Access for free at OpenStax 10.2.2 https://chem.libretexts.org/@go/page/38211


are greater than the adhesive forces between the water and the plastic. Water spreads out on glass because the adhesive force
between water and glass is greater than the cohesive forces within the water. When water is confined in a glass tube, its meniscus
(surface) has a concave shape because the water wets the glass and creeps up the side of the tube. On the other hand, the cohesive
forces between mercury atoms are much greater than the adhesive forces between mercury and glass. Mercury therefore does not
wet glass, and it forms a convex meniscus when confined in a tube because the cohesive forces within the mercury tend to draw it
into a drop (Figure 10.2.4).

Figure 10.2.4 : Differences in the relative strengths of cohesive and adhesive forces result in different meniscus shapes for mercury
(left) and water (right) in glass tubes. (credit: Mark Ott)
This figure shows two test tubes. The test tube on the left contains mercury with a meniscus that rounds up. The test tube on the
right contains water with a meniscus that rounds down.
If you place one end of a paper towel in spilled wine, as shown in Figure 10.2.5, the liquid wicks up the paper towel. A similar
process occurs in a cloth towel when you use it to dry off after a shower. These are examples of capillary action—when a liquid
flows within a porous material due to the attraction of the liquid molecules to the surface of the material and to other liquid
molecules. The adhesive forces between the liquid and the porous material, combined with the cohesive forces within the liquid,
may be strong enough to move the liquid upward against gravity.

Figure 10.2.5 : Wine wicks up a paper towel (left) because of the strong attractions of water (and ethanol) molecules to the −OH
groups on the towel’s cellulose fibers and the strong attractions of water molecules to other water (and ethanol) molecules (right).
(credit photo: modification of work by Mark Blaser)
A photo and a diagram are shown. In the photo, a paper towel is dipped into a bowl full of a red liquid sitting on a countertop. The
red liquid is traveling up the lower part of the paper towel, and this section of the photo has a square drawn around it. A right-
facing arrow leads from this square to the image. The image is square and has a background of two types of molecules, mixed
together. The first type of molecule is composed of two bonded black spheres, one of which is single bonded to three white spheres
and one of which is single bonded to two white spheres and a red sphere that is itself bonded to a white sphere. The other type of
molecule is composed of six black spheres bonded together in a row and bonded to other red and white spheres. Six upward-facing
arrows are drawn on top of this background. They have positive signs on their lower ends and negative signs on their heads. Four
upward-facing arrows are drawn with their signs reversed.
Towels soak up liquids like water because the fibers of a towel are made of molecules that are attracted to water molecules. Most
cloth towels are made of cotton, and paper towels are generally made from paper pulp. Both consist of long molecules of cellulose
that contain many −OH groups. Water molecules are attracted to these −OH groups and form hydrogen bonds with them, which
draws the H2O molecules up the cellulose molecules. The water molecules are also attracted to each other, so large amounts of
water are drawn up the cellulose fibers.
Capillary action can also occur when one end of a small diameter tube is immersed in a liquid, as illustrated in Figure 10.2.6. If the
liquid molecules are strongly attracted to the tube molecules, the liquid creeps up the inside of the tube until the weight of the liquid
and the adhesive forces are in balance. The smaller the diameter of the tube is, the higher the liquid climbs. It is partly by capillary

Access for free at OpenStax 10.2.3 https://chem.libretexts.org/@go/page/38211


action occurring in plant cells called xylem that water and dissolved nutrients are brought from the soil up through the roots and
into a plant. Capillary action is the basis for thin layer chromatography, a laboratory technique commonly used to separate small
quantities of mixtures. You depend on a constant supply of tears to keep your eyes lubricated and on capillary action to pump tear
fluid away.

Figure 10.2.6 : Depending upon the relative strengths of adhesive and cohesive forces, a liquid may rise (such as water) or fall
(such as mercury) in a glass capillary tube. The extent of the rise (or fall) is directly proportional to the surface tension of the liquid
and inversely proportional to the density of the liquid and the radius of the tube.
An image of two beakers and a tube is shown. The first beaker, drawn on the left and labeled “Water,” is drawn half-full of a blue
liquid. Two tubes are placed vertically in the beaker and inserted into the liquid. The liquid is shown higher in the tubes than in the
beaker and is labeled “Capillary attraction.” The second beaker, drawn in the middle and labeled “Mercury,” is drawn half-full of a
gray liquid. Two tubes are placed vertically in the beaker and inserted into the liquid. The liquid is shown lower in the tubes than in
the beaker and is labeled “Capillary repulsion.” Lines point to the vertical tubes and label them “Capillary tubes.” A separate
drawing of one of the vertical tubes from the first beaker is shown on the right. A right-facing arrow leads from the liquid in the
tube to a square call-out box that shows a close-up view of the liquid’s surface. The distance across the tube is labeled “2 r” in this
image.
The height to which a liquid will rise in a capillary tube is determined by several factors as shown in the following equation:
2T cos θ
h = (10.2.1)
rρg

where
h is the height of the liquid inside the capillary tube relative to the surface of the liquid outside the tube,
T is the surface tension of the liquid,
θ is the contact angle between the liquid and the tube,
r is the radius of the tube, ρ is the density of the liquid, and
g is the acceleration due to gravity, 9.8 m/s2.
When the tube is made of a material to which the liquid molecules are strongly attracted, they will spread out completely on the
surface, which corresponds to a contact angle of 0°. This is the situation for water rising in a glass tube.

 Example 10.2.1: Capillary Rise


At 25 °C, how high will water rise in a glass capillary tube with an inner diameter of 0.25 mm?
For water, T = 71.99 mN/m and ρ = 1.0 g/cm3.

Solution
The liquid will rise to a height h given by Equation 10.2.1 :
2T cos θ
h =
rρg

The Newton is defined as a kg m/s2, and so the provided surface tension is equivalent to 0.07199 kg/s2. The provided density
must be converted into units that will cancel appropriately: ρ = 1000 kg/m3. The diameter of the tube in meters is 0.00025 m,
so the radius is 0.000125 m. For a glass tube immersed in water, the contact angle is θ = 0°, so cos θ = 1. Finally, acceleration
due to gravity on the earth is g = 9.8 m/s2. Substituting these values into the equation, and cancelling units, we have:

Access for free at OpenStax 10.2.4 https://chem.libretexts.org/@go/page/38211


2
2(0.07199 kg/ s )
h = = 0.12 m = 12 cm
3 2
(0.000125 m)(1000 kg/ m )(9.8 m/ s )

 Exercise 10.2.1

Water rises in a glass capillary tube to a height of 8.4 cm. What is the diameter of the capillary tube?

Answer
diameter = 0.36 mm

 Applications: Capillary Action is Used to Draw Blood

Many medical tests require drawing a small amount of blood, for example to determine the amount of glucose in someone with
diabetes or the hematocrit level in an athlete. This procedure can be easily done because of capillary action, the ability of a
liquid to flow up a small tube against gravity, as shown in Figure 10.2.7. When your finger is pricked, a drop of blood forms
and holds together due to surface tension—the unbalanced intermolecular attractions at the surface of the drop. Then, when the
open end of a narrow-diameter glass tube touches the drop of blood, the adhesive forces between the molecules in the blood
and those at the glass surface draw the blood up the tube. How far the blood goes up the tube depends on the diameter of the
tube (and the type of fluid). A small tube has a relatively large surface area for a given volume of blood, which results in larger
(relative) attractive forces, allowing the blood to be drawn farther up the tube. The liquid itself is held together by its own
cohesive forces. When the weight of the liquid in the tube generates a downward force equal to the upward force associated
with capillary action, the liquid stops rising.

Figure 10.2.7 :: Blood is collected for medical analysis by capillary action, which draws blood into a small diameter glass tube.
(credit: modification of work by Centers for Disease Control and Prevention)
A photograph shows a person’s hand being held by a person wearing medical gloves. A thin glass tube is pressed against the
persons finger and blood is moving up the tube.

10.2.1: Key Concepts and Summary


The intermolecular forces between molecules in the liquid state vary depending upon their chemical identities and result in
corresponding variations in various physical properties. Cohesive forces between like molecules are responsible for a liquid’s
viscosity (resistance to flow) and surface tension (elasticity of a liquid surface). Adhesive forces between the molecules of a liquid
and different molecules composing a surface in contact with the liquid are responsible for phenomena such as surface wetting and
capillary rise.

10.2.2: Key Equations


2T cos θ
h =
rρg

Glossary
adhesive force
force of attraction between molecules of different chemical identities

capillary action

Access for free at OpenStax 10.2.5 https://chem.libretexts.org/@go/page/38211


flow of liquid within a porous material due to the attraction of the liquid molecules to the surface of the material and to other
liquid molecules

cohesive force
force of attraction between identical molecules

surface tension
energy required to increase the area, or length, of a liquid surface by a given amount

viscosity
measure of a liquid’s resistance to flow

This page titled 10.2: Properties of Liquids is shared under a CC BY 4.0 license and was authored, remixed, and/or curated by OpenStax via
source content that was edited to the style and standards of the LibreTexts platform; a detailed edit history is available upon request.

Access for free at OpenStax 10.2.6 https://chem.libretexts.org/@go/page/38211


10.3: Phase Transitions
 Learning Objectives
Define phase transitions and phase transition temperatures
Explain the relation between phase transition temperatures and intermolecular attractive forces
Describe the processes represented by typical heating and cooling curves, and compute heat flows and enthalpy changes
accompanying these processes

We witness and utilize changes of physical state, or phase transitions, in a great number of ways. As one example of global
significance, consider the evaporation, condensation, freezing, and melting of water. These changes of state are essential aspects of
our earth’s water cycle as well as many other natural phenomena and technological processes of central importance to our lives. In
this module, the essential aspects of phase transitions are explored.

10.3.1: Vaporization and Condensation


When a liquid vaporizes in a closed container, gas molecules cannot escape. As these gas phase molecules move randomly about,
they will occasionally collide with the surface of the condensed phase, and in some cases, these collisions will result in the
molecules re-entering the condensed phase. The change from the gas phase to the liquid is called condensation. When the rate of
condensation becomes equal to the rate of vaporization, neither the amount of the liquid nor the amount of the vapor in the
container changes. The vapor in the container is then said to be in equilibrium with the liquid. Keep in mind that this is not a static
situation, as molecules are continually exchanged between the condensed and gaseous phases. Such is an example of a dynamic
equilibrium, the status of a system in which reciprocal processes (for example, vaporization and condensation) occur at equal rates.
The pressure exerted by the vapor in equilibrium with a liquid in a closed container at a given temperature is called the liquid’s
vapor pressure (or equilibrium vapor pressure). The area of the surface of the liquid in contact with a vapor and the size of the
vessel have no effect on the vapor pressure, although they do affect the time required for the equilibrium to be reached. We can
measure the vapor pressure of a liquid by placing a sample in a closed container, like that illustrated in Figure 10.3.1, and using a
manometer to measure the increase in pressure that is due to the vapor in equilibrium with the condensed phase.

Figure 10.3.1 : In a closed container, dynamic equilibrium is reached when (a) the rate of molecules escaping from the liquid to
become the gas (b) increases and eventually (c) equals the rate of gas molecules entering the liquid. When this equilibrium is
reached, the vapor pressure of the gas is constant, although the vaporization and condensation processes continue.
Three images are shown and labeled “a,” “b,” and “c.” Each image shows a round bulb connected on the right to a tube that is
horizontal, then is bent vertically, curves, and then is vertical again to make a u-shape. A valve is located in the horizontal portion
of the tube. Image a depicts a liquid in the bulb, labeled, “Liquid,” and upward-facing arrows leading away from the surface of the
liquid. The phrase, “Molecules escape surface and form vapor” is written below the bulb, and a gray liquid in the u-shaped portion
of the tube is shown at equal heights on the right and left sides. Image b depicts a liquid in the bulb, labeled, “Liquid,” and upward-
facing arrows leading away from the surface of the liquid to molecules drawn in the upper portion of the bulb. A gray liquid in the
u-shaped portion of the tube is shown slightly higher on the right side than on the left side. Image c depicts a liquid in the bulb,
labeled, “Liquid,” and upward-facing arrows leading away from the surface of the liquid to molecules drawn in the upper portion of
the bulb. There are more molecules present in c than in b. The phrase “Equilibrium reached, vapor pressure determined,” is written
below the bulb and a gray liquid in the u-shaped portion of the tube is shown higher on the right side. A horizontal line is drawn
level with each of these liquid levels and the distance between the lines is labeled with a double-headed arrow. This section is
labeled with the phrase, “Vapor pressure.”
The chemical identities of the molecules in a liquid determine the types (and strengths) of intermolecular attractions possible;
consequently, different substances will exhibit different equilibrium vapor pressures. Relatively strong intermolecular attractive
forces will serve to impede vaporization as well as favoring “recapture” of gas-phase molecules when they collide with the liquid
surface, resulting in a relatively low vapor pressure. Weak intermolecular attractions present less of a barrier to vaporization, and a

Access for free at OpenStax 10.3.1 https://chem.libretexts.org/@go/page/38212


reduced likelihood of gas recapture, yielding relatively high vapor pressures. The following example illustrates this dependence of
vapor pressure on intermolecular attractive forces.

 Example 10.3.1: Explaining Vapor Pressure in Terms of IMFs

Given the shown structural formulas for these four compounds, explain their relative vapor pressures in terms of types and
extents of IMFs:

Solution
Diethyl ether has a very small dipole and most of its intermolecular attractions are London forces. Although this molecule is
the largest of the four under consideration, its IMFs are the weakest and, as a result, its molecules most readily escape from the
liquid. It also has the highest vapor pressure. Due to its smaller size, ethanol exhibits weaker dispersion forces than diethyl
ether. However, ethanol is capable of hydrogen bonding and, therefore, exhibits stronger overall IMFs, which means that fewer
molecules escape from the liquid at any given temperature, and so ethanol has a lower vapor pressure than diethyl ether. Water
is much smaller than either of the previous substances and exhibits weaker dispersion forces, but its extensive hydrogen
bonding provides stronger intermolecular attractions, fewer molecules escaping the liquid, and a lower vapor pressure than for
either diethyl ether or ethanol. Ethylene glycol has two −OH groups, so, like water, it exhibits extensive hydrogen bonding. It
is much larger than water and thus experiences larger London forces. Its overall IMFs are the largest of these four substances,
which means its vaporization rate will be the slowest and, consequently, its vapor pressure the lowest.

 Exercise 10.3.1

At 20 °C, the vapor pressures of several alcohols are given in this table. Explain these vapor pressures in terms of types and
extents of IMFs for these alcohols:
At 20 °C, the vapor pressures of several alcohols
Compound methanol CH3OH ethanol C2H5OH propanol C3H7OH butanol C4H9OH

Vapor Pressure at 20 °C 11.9 kPa 5.95 kPa 2.67 kPa 0.56 kPa

Answer
All these compounds exhibit hydrogen bonding; these strong IMFs are difficult for the molecules to overcome, so the vapor
pressures are relatively low. As the size of molecule increases from methanol to butanol, dispersion forces increase, which
means that the vapor pressures decrease as observed:

Pmethanol > Pethanol > Ppropanol > Pbutanol

As temperature increases, the vapor pressure of a liquid also increases due to the increased average KE of its molecules. Recall that
at any given temperature, the molecules of a substance experience a range of kinetic energies, with a certain fraction of molecules
having a sufficient energy to overcome IMF and escape the liquid (vaporize). At a higher temperature, a greater fraction of
molecules have enough energy to escape from the liquid, as shown in Figure 10.3.2. The escape of more molecules per unit of time
and the greater average speed of the molecules that escape both contribute to the higher vapor pressure.

Access for free at OpenStax 10.3.2 https://chem.libretexts.org/@go/page/38212


Figure 10.3.2 : Temperature affects the distribution of kinetic energies for the molecules in a liquid. At the higher temperature, more
molecules have the necessary kinetic energy, KE, to escape from the liquid into the gas phase.
A graph is shown where the y-axis is labeled “Number of molecules” and the x-axis is labeled “Kinetic Energy.” Two lines are
graphed and a vertical dotted line, labeled “Minimum K E needed to escape,” is drawn halfway across the x-axis. The first line
move sharply upward and has a high peak near the left side of the x-axis. It drops just as steeply and ends about 60 percent of the
way across the x-axis. This line is labeled “Low T.” A second line, labeled “High T,” begins at the same point as the first, but does
not go to such a high point, is wider, and ends slightly further to the right on the x-axis.

10.3.2: Boiling Points


When the vapor pressure increases enough to equal the external atmospheric pressure, the liquid reaches its boiling point. The
boiling point of a liquid is the temperature at which its equilibrium vapor pressure is equal to the pressure exerted on the liquid by
its gaseous surroundings. For liquids in open containers, this pressure is that due to the earth’s atmosphere. The normal boiling
point of a liquid is defined as its boiling point when surrounding pressure is equal to 1 atm (101.3 kPa). Figure 10.3.3 shows the
variation in vapor pressure with temperature for several different substances. Considering the definition of boiling point, these
curves may be seen as depicting the dependence of a liquid’s boiling point on surrounding pressure.

Figure 10.3.3: The boiling points of liquids are the temperatures at which their equilibrium vapor pressures equal the pressure of
the surrounding atmosphere. Normal boiling points are those corresponding to a pressure of 1 atm (101.3 kPa.)
A graph is shown where the x-axis is labeled “Temperature ( degree sign, C )” and has values of 200 to 1000 in increments of 200
and the y-axis is labeled “Pressure ( k P a )” and has values of 20 to 120 in increments of 20. A horizontal dotted line extends
across the graph at point 780 on the y-axis while three vertical dotted lines extend from points 35, 78, and 100 to meet the
horizontal dotted line. Four lines are graphed. The first line, labeled “ethyl ether,” begins at the point “0 , 200” and extends in a
slight curve to point “45, 1000” while the second line, labeled “ethanol”, extends from point “0, 20” to point “88, 1000” in a more
extreme curve. The third line, labeled “water,” begins at the point “0, 0” and extends in a curve to point “108, 1000” while the
fourth line, labeled “ethylene glycol,” extends from point “80, 0” to point “140, 100” in a very shallow curve.

 Example 10.3.2: A Boiling Point at Reduced Pressure

A typical atmospheric pressure in Leadville, Colorado (elevation 10,200 feet) is 68 kPa. Use the graph in Figure 10.3.3 to
determine the boiling point of water at this elevation.

Access for free at OpenStax 10.3.3 https://chem.libretexts.org/@go/page/38212


Solution
The graph of the vapor pressure of water versus temperature in Figure 10.3.3 indicates that the vapor pressure of water is 68
kPa at about 90 °C. Thus, at about 90 °C, the vapor pressure of water will equal the atmospheric pressure in Leadville, and
water will boil.

 Exercise 10.3.2

The boiling point of ethyl ether was measured to be 10 °C at a base camp on the slopes of Mount Everest. Use Figure 10.3.3 to
determine the approximate atmospheric pressure at the camp.

Answer
Approximately 40 kPa (0.4 atm)

The quantitative relation between a substance’s vapor pressure and its temperature is described by the Clausius-Clapeyron
equation:
−Δ Hva p /RT
P = Ae (10.3.1)

where
ΔHvap is the enthalpy of vaporization for the liquid,
R is the gas constant, and
ln A is a constant whose value depends on the chemical identity of the substance.

Equation 10.3.1 is often rearranged into logarithmic form to yield the linear equation:
ΔHvap
ln P = − + ln A (10.3.2)
RT

This linear equation may be expressed in a two-point format that is convenient for use in various computations, as demonstrated in
the examples and exercises that follow. If at temperature T , the vapor pressure is P , and at temperature T , the vapor pressure is
1 1 2

T , the corresponding linear equations are:


2

ΔHvap
ln P1 = − + ln A
RT1

and
ΔHvap
ln P2 = − + ln A (10.3.3)
RT2

Since the constant, ln A, is the same, these two equations may be rearranged to isolate ln A and then set them equal to one another:
ΔHvap ΔHvap
ln P1 + = ln P2 +
RT1 RT2

which can be combined into:

P2 ΔHvap 1 1
ln( ) = ( − ) (10.3.4)
P1 R T1 T2

 Example 10.3.3: Estimating Enthalpy of Vaporization

Isooctane (2,2,4-trimethylpentane) has an octane rating of 100. It is used as one of the standards for the octane-rating system
for gasoline. At 34.0 °C, the vapor pressure of isooctane is 10.0 kPa, and at 98.8 °C, its vapor pressure is 100.0 kPa. Use this
information to estimate the enthalpy of vaporization for isooctane.

Access for free at OpenStax 10.3.4 https://chem.libretexts.org/@go/page/38212


Solution
The enthalpy of vaporization, ΔH vap , can be determined by using the Clausius-Clapeyron equation (Equation 10.3.4):
P2 ΔHvap 1 1
ln( ) = ( − )
P1 R T1 T2

Since we have two vapor pressure-temperature values


o
T1 = 34.0 C = 307.2 K

P1 = 10.0 kP a and
o
T2 = 98.8 C = 372.0 K

P2 = 100 kP a

we can substitute them into this equation and solve for ΔHvap . Rearranging the Clausius-Clapeyron equation and solving for
ΔH vapyields:
P2
R ⋅ ln( )
P1
ΔHvap =
1 1
( − )
T1 T2

100 kP a
(8.3145 J/mol ⋅ K) ⋅ ln( )
10.0 kP a
=
1 1
( − )
307.2 K 372.0 K

= 33, 800 J/mol = 33.8 kJ/mol

Note that the pressure can be in any units, so long as they agree for both P values, but the temperature must be in kelvin for the
Clausius-Clapeyron equation to be valid.

 Exercise 10.3.3

At 20.0 °C, the vapor pressure of ethanol is 5.95 kPa, and at 63.5 °C, its vapor pressure is 53.3 kPa. Use this information to
estimate the enthalpy of vaporization for ethanol.

Answer
47,782 J/mol = 47.8 kJ/mol

 Example 10.3.4: Estimating Temperature (or Vapor Pressure)

For benzene (C6H6), the normal boiling point is 80.1 °C and the enthalpy of vaporization is 30.8 kJ/mol. What is the boiling
point of benzene in Denver, where atmospheric pressure = 83.4 kPa?

Solution
If the temperature and vapor pressure are known at one point, along with the enthalpy of vaporization, ΔHvap, then the
temperature that corresponds to a different vapor pressure (or the vapor pressure that corresponds to a different temperature)
can be determined by using the Clausius-Clapeyron equation (Equation 10.3.1) :
P2 ΔHvap 1 1
ln( ) = ( − )
P1 R T1 T2

Since the normal boiling point is the temperature at which the vapor pressure equals atmospheric pressure at sea level, we
know one vapor pressure-temperature value (T = 80.1 °C = 353.3 K, P = 101.3 kPa, ΔH = 30.8 kJ/mol) and want to find
1 1 vap

Access for free at OpenStax 10.3.5 https://chem.libretexts.org/@go/page/38212


the temperature (T ) that corresponds to vapor pressure P2 = 83.4 kPa. We can substitute these values into the Clausius-
2

Clapeyron equation and then solve for T . Rearranging the Clausius-Clapeyron equation and solving for T yields:
2 2

−1
P2
⎛ ⎞
−R ⋅ ln( )
⎜ P1 1 ⎟
T2 = ⎜ + ⎟
⎜ ΔHvap T1 ⎟

⎝ ⎠

−1
83.4 kPa
⎛ −(8.3145 J/mol ⋅ K) ⋅ ln( ) ⎞

⎜ 101.3 kPa 1 ⎟
= ⎜ + ⎟
⎜ 30, 800 J/mol 353.3 K ⎟
⎝ ⎠


= 346.9 K or 73.8 C

 Exercise 10.3.4

For acetone (CH ) CO, the normal boiling point is 56.5 °C and the enthalpy of vaporization is 31.3 kJ/mol. What is the vapor
3 2

pressure of acetone at 25.0 °C?

Answer
30.1 kPa

10.3.3: Enthalpy of Vaporization


Vaporization is an endothermic process. The cooling effect can be evident when you leave a swimming pool or a shower. When the
water on your skin evaporates, it removes heat from your skin and causes you to feel cold. The energy change associated with the
vaporization process is the enthalpy of vaporization, ΔH . For example, the vaporization of water at standard temperature is
vap

represented by:

H O(l) ⟶ H O(g) ΔHvap = 44.01 kJ/mol


2 2

As described in the chapter on thermochemistry, the reverse of an endothermic process is exothermic. And so, the condensation of
a gas releases heat:

H O(g) ⟶ H O(l) ΔHcon = −ΔHvap = −44.01 kJ/mol


2 2

 Example 10.3.5: Using Enthalpy of Vaporization


One way our body is cooled is by evaporation of the water in sweat (Figure 10.3.4). In very hot climates, we can lose as much
as 1.5 L of sweat per day. Although sweat is not pure water, we can get an approximate value of the amount of heat removed
by evaporation by assuming that it is. How much heat is required to evaporate 1.5 L of water (1.5 kg) at T = 37 °C (normal
body temperature); ΔH = 43.46 kJ/mol at 37 °C.
vap

Access for free at OpenStax 10.3.6 https://chem.libretexts.org/@go/page/38212


Figure 10.3.4 : Evaporation of sweat helps cool the body. (credit: “Kullez”/Flickr)
Solution We start with the known volume of sweat (approximated as just water) and use the given information to convert to the
amount of heat needed:
1000 g 1 mol 43.46 kJ 3
1.5 L × × × = 3.6 × 10 kJ
1 L 18 g 1 mol

Thus, 3600 kJ of heat are removed by the evaporation of 1.5 L of water.

 Exercise 10.3.5: Boiling Ammonia

How much heat is required to evaporate 100.0 g of liquid ammonia, NH , at its boiling point if its enthalpy of vaporization is
3

4.8 kJ/mol?

Answer
28 kJ

10.3.4: Melting and Freezing


When we heat a crystalline solid, we increase the average energy of its atoms, molecules, or ions and the solid gets hotter. At some
point, the added energy becomes large enough to partially overcome the forces holding the molecules or ions of the solid in their
fixed positions, and the solid begins the process of transitioning to the liquid state, or melting. At this point, the temperature of the
solid stops rising, despite the continual input of heat, and it remains constant until all of the solid is melted. Only after all of the
solid has melted will continued heating increase the temperature of the liquid (Figure 10.3.5.

Figure 10.3.5 : (a) This beaker of ice has a temperature of −12.0 °C. (b) After 10 minutes the ice has absorbed enough heat from the
air to warm to 0 °C. A small amount has melted. (c) Thirty minutes later, the ice has absorbed more heat, but its temperature is still
0 °C. The ice melts without changing its temperature. (d) Only after all the ice has melted does the heat absorbed cause the
temperature to increase to 22.2 °C. (credit: modification of work by Mark Ott).
This figure shows four photos each labeled, “a,” “b,” “c,” and, “d.” Each photo shows a beaker with ice and a digital thermometer.
The first photo shows ice cubes in the beaker, and the thermometer reads negative 12.0 degrees C. The second photo shows slightly
melted ice, and the thermometer reads 0.0 degrees C. The third photo shows more water than ice in the beaker. The thermometer
reads 0.0 degrees C. The fourth photo shows the ice completely melted, and the thermometer reads 22.2 degrees C.
If we stop heating during melting and place the mixture of solid and liquid in a perfectly insulated container so no heat can enter or
escape, the solid and liquid phases remain in equilibrium. This is almost the situation with a mixture of ice and water in a very good

Access for free at OpenStax 10.3.7 https://chem.libretexts.org/@go/page/38212


thermos bottle; almost no heat gets in or out, and the mixture of solid ice and liquid water remains for hours. In a mixture of solid
and liquid at equilibrium, the reciprocal processes of melting and freezing occur at equal rates, and the quantities of solid and liquid
therefore remain constant. The temperature at which the solid and liquid phases of a given substance are in equilibrium is called the
melting point of the solid or the freezing point of the liquid. Use of one term or the other is normally dictated by the direction of the
phase transition being considered, for example, solid to liquid (melting) or liquid to solid (freezing).
The enthalpy of fusion and the melting point of a crystalline solid depend on the strength of the attractive forces between the units
present in the crystal. Molecules with weak attractive forces form crystals with low melting points. Crystals consisting of particles
with stronger attractive forces melt at higher temperatures.
The amount of heat required to change one mole of a substance from the solid state to the liquid state is the enthalpy of fusion,
ΔHfus of the substance. The enthalpy of fusion of ice is 6.0 kJ/mol at 0 °C. Fusion (melting) is an endothermic process:
H O(s) → H O(l) ΔHfus = 6.01 kJ/mol (10.3.5)
2 2

The reciprocal process, freezing, is an exothermic process whose enthalpy change is −6.0 kJ/mol at 0 °C:
H O(l) → H O(s) ΔHfrz = −ΔHfus = −6.01 kJ/mol (10.3.6)
2 2

10.3.5: Sublimation and Deposition


Some solids can transition directly into the gaseous state, bypassing the liquid state, via a process known as sublimation. At room
temperature and standard pressure, a piece of dry ice (solid CO2) sublimes, appearing to gradually disappear without ever forming
any liquid. Snow and ice sublime at temperatures below the melting point of water, a slow process that may be accelerated by
winds and the reduced atmospheric pressures at high altitudes. When solid iodine is warmed, the solid sublimes and a vivid purple
vapor forms (Figure 10.3.6). The reverse of sublimation is called deposition, a process in which gaseous substances condense
directly into the solid state, bypassing the liquid state. The formation of frost is an example of deposition.

Figure 10.3.6 : Sublimation of solid iodine in the bottom of the tube produces a purple gas that subsequently deposits as solid
iodine on the colder part of the tube above. (credit: modification of work by Mark Ott)
This figure shows a test tube. In the bottom is a dark substance which breaks up into a purple gas at the top.
Like vaporization, the process of sublimation requires an input of energy to overcome intermolecular attractions. The enthalpy of
sublimation, ΔHsub, is the energy required to convert one mole of a substance from the solid to the gaseous state. For example, the
sublimation of carbon dioxide is represented by:

CO (s) ⟶ CO (g) ΔHsub = 26.1 kJ/mol


2 2

Likewise, the enthalpy change for the reverse process of deposition is equal in magnitude but opposite in sign to that for
sublimation:

CO (g) ⟶ CO (s) ΔHdep = −ΔHsub = −26.1 kJ/mol


2 2

Consider the extent to which intermolecular attractions must be overcome to achieve a given phase transition. Converting a solid
into a liquid requires that these attractions be only partially overcome; transition to the gaseous state requires that they be
completely overcome. As a result, the enthalpy of fusion for a substance is less than its enthalpy of vaporization. This same logic
can be used to derive an approximate relation between the enthalpies of all phase changes for a given substance. Though not an
entirely accurate description, sublimation may be conveniently modeled as a sequential two-step process of melting followed by
vaporization in order to apply Hess’s Law.

Access for free at OpenStax 10.3.8 https://chem.libretexts.org/@go/page/38212


solid ⟶ liquid ΔHfus

liquid ⟶ gas ΔHvap


–––––––––––––––––––––––
solid ⟶ gas ΔHsub = ΔHfus + ΔHvap

Viewed in this manner, the enthalpy of sublimation for a substance may be estimated as the sum of its enthalpies of fusion and
vaporization, as illustrated in Figure 10.3.7. For example:

Figure 10.3.7 : For a given substance, the sum of its enthalpy of fusion and enthalpy of vaporization is approximately equal to its
enthalpy of sublimation.
A diagram is shown with a vertical line drawn on the left side and labeled “Energy” and three horizontal lines drawn near the
bottom, lower third and top of the diagram. These three lines are labeled, from bottom to top, “Solid,” “Liquid” and “Gas.” Near
the middle of the diagram, a vertical, upward-facing arrow is drawn from the solid line to the gas line and labeled “Sublimation,
delta sign, H, subscript sub.” To the right of this arrow is a second vertical, upward-facing arrow that is drawn from the solid line to
the liquid line and labeled “Fusion, delta sign, H, subscript fus.” Above the second arrow is a third arrow drawn from the liquid line
to the gas line and labeled, “Vaporization, delta sign, H, subscript vap.”

10.3.6: Heating and Cooling Curves


In the chapter on thermochemistry, the relation between the amount of heat absorbed or related by a substance, q, and its
accompanying temperature change, ΔT, was introduced:

q = mcΔT

where m is the mass of the substance and c is its specific heat. The relation applies to matter being heated or cooled, but not
undergoing a change in state. When a substance being heated or cooled reaches a temperature corresponding to one of its phase
transitions, further gain or loss of heat is a result of diminishing or enhancing intermolecular attractions, instead of increasing or
decreasing molecular kinetic energies. While a substance is undergoing a change in state, its temperature remains constant. Figure
10.3.8 shows a typical heating curve.

Access for free at OpenStax 10.3.9 https://chem.libretexts.org/@go/page/38212


Figure 10.3.8 : A typical heating curve for a substance depicts changes in temperature that result as the substance absorbs
increasing amounts of heat. Plateaus in the curve (regions of constant temperature) are exhibited when the substance undergoes
phase transitions.
A graph is shown where the x-axis is labeled “Amount of heat added” and the y-axis is labeled “Temperature ( degree sign C )” and
has values of negative 10 to 100 in increments of 20. A right-facing horizontal arrow extends from point “0, 0” to the right side of
the graph. A line graph begins at the lower left of the graph and moves to point “0” on the y-axis. This segment of the line is
labeled “H, subscript 2, O ( s ).” The line then flattens and travels horizontally for a small distance. This segment is labeled “Solid
begins to melt” on its left side and “All solid melted” on its right side. The line then goes steeply upward in a linear fashion until it
hits point “100” on the y-axis. This segment of the line is labeled “H, subscript 2, O,( l ).” The line then flattens and travels
horizontally for a moderate distance. This segment is labeled “Liquid begins to boil” on its left side and “All liquid evaporated” on
its right side. The line then rises to a point above “100” on the y-axis. This segment of the line is labeled “H, subscript 2, O ( g ).”
Consider the example of heating a pot of water to boiling. A stove burner will supply heat at a roughly constant rate; initially, this
heat serves to increase the water’s temperature. When the water reaches its boiling point, the temperature remains constant despite
the continued input of heat from the stove burner. This same temperature is maintained by the water as long as it is boiling. If the
burner setting is increased to provide heat at a greater rate, the water temperature does not rise, but instead the boiling becomes
more vigorous (rapid). This behavior is observed for other phase transitions as well: For example, temperature remains constant
while the change of state is in progress.

 Example 10.3.6: Total Heat Needed to Change Temperature and Phase for a Substance

How much heat is required to convert 135 g of ice at −15 °C into water vapor at 120 °C?

Solution
The transition described involves the following steps:
1. Heat ice from −15 °C to 0 °C
2. Melt ice
3. Heat water from 0 °C to 100 °C
4. Boil water
5. Heat steam from 100 °C to 120 °C
The heat needed to change the temperature of a given substance (with no change in phase) is: q = m × c × ΔT (see previous
chapter on thermochemistry). The heat needed to induce a given change in phase is given by q = n × ΔH.
Using these equations with the appropriate values for specific heat of ice, water, and steam, and enthalpies of fusion and
vaporization, we have:

Access for free at OpenStax 10.3.10 https://chem.libretexts.org/@go/page/38212


qtotal = (m ⋅ c ⋅ ΔT )ice + n ⋅ ΔHfus + (m ⋅ c ⋅ ΔT )water + n ⋅ ΔHvap + (m ⋅ c ⋅ ΔT )steam

1 mol
= (135 g ⋅ 2.09 J/g ⋅ °C ⋅ 15°C) + (135 ⋅ ⋅ 6.01 kJ/mol)
18.02 g

1 mol
+(135 g ⋅ 4.18 J/g ⋅ °C ⋅ 100°C) + (135 g ⋅ ⋅ 40.67 kJ/mol)
18.02 g

+(135 g ⋅ 1.84 J/g ⋅ °C ⋅ 20°C)

= 4230 J + 45.0 kJ + 56, 500 J + 305 kJ + 4970 J

Converting the quantities in J to kJ permits them to be summed, yielding the total heat required:

= 4.23 kJ + 45.0 kJ + 56.5 kJ + 305 kJ + 4.97 kJ = 416 kJ

 Exercise 10.3.6

What is the total amount of heat released when 94.0 g water at 80.0 °C cools to form ice at −30.0 °C?

Answer
40.5 kJ

Summary
Phase transitions are processes that convert matter from one physical state into another. There are six phase transitions between the
three phases of matter. Melting, vaporization, and sublimation are all endothermic processes, requiring an input of heat to overcome
intermolecular attractions. The reciprocal transitions of freezing, condensation, and deposition are all exothermic processes,
involving heat as intermolecular attractive forces are established or strengthened. The temperatures at which phase transitions occur
are determined by the relative strengths of intermolecular attractions and are, therefore, dependent on the chemical identity of the
substance.

10.3.7: Key Equations


−Δ Hv ap /RT
P = Ae

ΔHvap
ln P = − + ln A
RT
P2 ΔHvap 1 1
ln( ) = ( − )
P1 R T1 T2

Glossary
boiling point
temperature at which the vapor pressure of a liquid equals the pressure of the gas above it

Clausius-Clapeyron equation
mathematical relationship between the temperature, vapor pressure, and enthalpy of vaporization for a substance

condensation
change from a gaseous to a liquid state

deposition
change from a gaseous state directly to a solid state

dynamic equilibrium
state of a system in which reciprocal processes are occurring at equal rates

Access for free at OpenStax 10.3.11 https://chem.libretexts.org/@go/page/38212


freezing
change from a liquid state to a solid state

freezing point
temperature at which the solid and liquid phases of a substance are in equilibrium; see also melting point

melting
change from a solid state to a liquid state

melting point
temperature at which the solid and liquid phases of a substance are in equilibrium; see also freezing point

normal boiling point


temperature at which a liquid’s vapor pressure equals 1 atm (760 torr)

sublimation
change from solid state directly to gaseous state

vapor pressure
(also, equilibrium vapor pressure) pressure exerted by a vapor in equilibrium with a solid or a liquid at a given temperature

vaporization
change from liquid state to gaseous state

This page titled 10.3: Phase Transitions is shared under a CC BY 4.0 license and was authored, remixed, and/or curated by OpenStax via source
content that was edited to the style and standards of the LibreTexts platform; a detailed edit history is available upon request.

Access for free at OpenStax 10.3.12 https://chem.libretexts.org/@go/page/38212


10.4: Phase Diagrams
 Learning Objectives
Explain the construction and use of a typical phase diagram
Use phase diagrams to identify stable phases at given temperatures and pressures, and to describe phase transitions
resulting from changes in these properties
Describe the supercritical fluid phase of matter

In the previous module, the variation of a liquid’s equilibrium vapor pressure with temperature was described. Considering the
definition of boiling point, plots of vapor pressure versus temperature represent how the boiling point of the liquid varies with
pressure. Also described was the use of heating and cooling curves to determine a substance’s melting (or freezing) point. Making
such measurements over a wide range of pressures yields data that may be presented graphically as a phase diagram. A phase
diagram combines plots of pressure versus temperature for the liquid-gas, solid-liquid, and solid-gas phase-transition equilibria of a
substance. These diagrams indicate the physical states that exist under specific conditions of pressure and temperature, and also
provide the pressure dependence of the phase-transition temperatures (melting points, sublimation points, boiling points). A typical
phase diagram for a pure substance is shown in Figure 10.4.1.

Figure 10.4.1 : The physical state of a substance and its phase-transition temperatures are represented graphically in a phase
diagram.
A graph is shown where the x-axis is labeled “Temperature” and the y-axis is labeled “Pressure.” A line extends from the lower left
bottom of the graph sharply upward to a point that is a third across the x-axis. A second line begins at the lower third of the first
line at a point labeled “triple point” and extends to the upper right corner of the graph where it is labeled “critical point.” The two
lines bisect the graph area to create three sections, labeled “solid” near the top left, “liquid” in the top middle and “gas” near the
bottom right. A pair of horizontal arrows, one left-facing and labeled “deposition” and one right-facing and labeled” sublimation,”
are drawn on top of the bottom section of the first line. A second pair of horizontal arrows, one left-facing and labeled “freezing”
and one right-facing and labeled “melting”, are drawn on top of the upper section of the first line. A third pair of horizontal arrows,
one left-facing and labeled “condensation” and one right-facing and labeled ”vaporization,” are drawn on top of the middle section
of the second line.
To illustrate the utility of these plots, consider the phase diagram for water shown in Figure 10.4.2.

Access for free at OpenStax 10.4.1 https://chem.libretexts.org/@go/page/38213


Figure 10.4.2 : The pressure and temperature axes on this phase diagram of water are not drawn to constant scale in order to
illustrate several important properties.
A graph is shown where the x-axis is labeled “Temperature in degrees Celsius” and the y-axis is labeled “Pressure ( k P a ).” A line
extends from the origin of the graph which is labeled “A” sharply upward to a point in the bottom third of the diagram labeled “B”
where it branches into a line that slants slightly backward until it hits the highest point on the y-axis labeled “D” and a second line
that extends to the upper right corner of the graph labeled “C”. C is labeled “Critical point, with a dotted line extending downward
to the x-axis labeled 374 degrees Celsius, and another dotted line extending to the y-axis labeled 22,089 k P a. The two lines bisect
the graph area to create three sections, labeled “Ice (solid)” near the middle left, “Water (liquid)” in the top middle and “Water
vapor (gas)” near the bottom middle. Point B is labeled “Triple point” and has a dotted line extending downward to the x-axis
labeled 0.01, and another dotted line extending to the y-axis labeled 0.6. Halfway between points B and C a dotted line extends
from the originally discussed line downward to the point 100 degrees Celsius on the x-axis, and another dotted line extends to the
y-axis at 101 k P a. Another dotted line extends from this dotted line downward at 0 degrees Celsius.
We can use the phase diagram to identify the physical state of a sample of water under specified conditions of pressure and
temperature. For example, a pressure of 50 kPa and a temperature of −10 °C correspond to the region of the diagram labeled “ice.”
Under these conditions, water exists only as a solid (ice). A pressure of 50 kPa and a temperature of 50 °C correspond to the
“water” region—here, water exists only as a liquid. At 25 kPa and 200 °C, water exists only in the gaseous state. Note that on the
H2O phase diagram, the pressure and temperature axes are not drawn to a constant scale in order to permit the illustration of several
important features as described here.
The curve BC in Figure 10.4.2 is the plot of vapor pressure versus temperature as described in the previous module of this chapter.
This “liquid-vapor” curve separates the liquid and gaseous regions of the phase diagram and provides the boiling point for water at
any pressure. For example, at 1 atm, the boiling point is 100 °C. Notice that the liquid-vapor curve terminates at a temperature of
374 °C and a pressure of 218 atm, indicating that water cannot exist as a liquid above this temperature, regardless of the pressure.
The physical properties of water under these conditions are intermediate between those of its liquid and gaseous phases. This
unique state of matter is called a supercritical fluid, a topic that will be described in the next section of this module.
The solid-vapor curve, labeled AB in Figure 10.4.2, indicates the temperatures and pressures at which ice and water vapor are in
equilibrium. These temperature-pressure data pairs correspond to the sublimation, or deposition, points for water. If we could zoom
in on the solid-gas line in Figure 10.4.2, we would see that ice has a vapor pressure of about 0.20 kPa at −10 °C. Thus, if we place
a frozen sample in a vacuum with a pressure less than 0.20 kPa, ice will sublime. This is the basis for the “freeze-drying” process
often used to preserve foods, such as the ice cream shown in Figure 10.4.3.

Access for free at OpenStax 10.4.2 https://chem.libretexts.org/@go/page/38213


Figure 10.4.3 : Freeze-dried foods, like this ice cream, are dehydrated by sublimation at pressures below the triple point for water.
(credit: ʺlwaoʺ/Flickr)
The solid-liquid curve labeled BD shows the temperatures and pressures at which ice and liquid water are in equilibrium,
representing the melting/freezing points for water. Note that this curve exhibits a slight negative slope (greatly exaggerated for
clarity), indicating that the melting point for water decreases slightly as pressure increases. Water is an unusual substance in this
regard, as most substances exhibit an increase in melting point with increasing pressure. This behavior is partly responsible for the
movement of glaciers, like the one shown in Figure 10.4.4. The bottom of a glacier experiences an immense pressure due to its
weight that can melt some of the ice, forming a layer of liquid water on which the glacier may more easily slide.

Figure 10.4.4 : The immense pressures beneath glaciers result in partial melting to produce a layer of water that provides lubrication
to assist glacial movement. This satellite photograph shows the advancing edge of the Perito Moreno glacier in Argentina. (credit:
NASA)
A photograph shows an aerial view of a land mass. The white mass of a glacier is shown near the top left quadrant of the photo and
leads to two branching blue rivers. The open land is shown in brown.
The point of intersection of all three curves is labeled B in Figure 10.4.2. At the pressure and temperature represented by this point,
all three phases of water coexist in equilibrium. This temperature-pressure data pair is called the triple point. At pressures lower
than the triple point, water cannot exist as a liquid, regardless of the temperature.

 Example 10.4.1: Determining the State of Water

Using the phase diagram for water given in Figure 10.4.2, determine the state of water at the following temperatures and
pressures:
a. −10 °C and 50 kPa
b. 25 °C and 90 kPa
c. 50 °C and 40 kPa
d. 80 °C and 5 kPa
e. −10 °C and 0.3 kPa
f. 50 °C and 0.3 kPa

Solution
Using the phase diagram for water, we can determine that the state of water at each temperature and pressure given are as
follows: (a) solid; (b) liquid; (c) liquid; (d) gas; (e) solid; (f) gas.

Access for free at OpenStax 10.4.3 https://chem.libretexts.org/@go/page/38213


 Exercise 10.4.1
What phase changes can water undergo as the temperature changes if the pressure is held at 0.3 kPa? If the pressure is held at
50 kPa?

Answer
At 0.3 kPa: s⟶ g at −58 °C. At 50 kPa: s⟶ l at 0 °C, l ⟶ g at 78 °C

Consider the phase diagram for carbon dioxide shown in Figure 10.4.5 as another example. The solid-liquid curve exhibits a
positive slope, indicating that the melting point for CO2 increases with pressure as it does for most substances (water being a
notable exception as described previously). Notice that the triple point is well above 1 atm, indicating that carbon dioxide cannot
exist as a liquid under ambient pressure conditions. Instead, cooling gaseous carbon dioxide at 1 atm results in its deposition into
the solid state. Likewise, solid carbon dioxide does not melt at 1 atm pressure but instead sublimes to yield gaseous CO2. Finally,
notice that the critical point for carbon dioxide is observed at a relatively modest temperature and pressure in comparison to water.

Figure 10.4.5 : The pressure and temperature axes on this phase diagram of carbon dioxide are not drawn to constant scale in order
to illustrate several important properties.
A graph is shown where the x-axis is labeled “Temperature ( degree sign, C )” and has values of negative 100 to 100 in increments
of 25 and the y-axis is labeled “Pressure ( k P a )” and has values of 10 to 1,000,000. A line extends from the lower left bottom of
the graph upward to a point around“27, 9000,” where it ends. The space under this curve is labeled “Gas.” A second line extends in
a curve from point around “-73, 100” to “27, 1,000,000.” The area to the left of this line and above the first line is labeled “Solid”
while the area to the right is labeled “Liquid.” A section on the graph under the second line and past the point “28” on the x-axis is
labeled “S C F.”

 Example 10.4.2: Determining the State of Carbon Dioxide

Using the phase diagram for carbon dioxide shown in Figure 10.4.5, determine the state of CO2 at the following temperatures
and pressures:
a. −30 °C and 2000 kPa
b. −60 °C and 1000 kPa
c. −60 °C and 100 kPa
d. 20 °C and 1500 kPa
e. 0 °C and 100 kPa
f. 20 °C and 100 kPa

Solution
Using the phase diagram for carbon dioxide provided, we can determine that the state of CO2 at each temperature and pressure
given are as follows: (a) liquid; (b) solid; (c) gas; (d) liquid; (e) gas; (f) gas.

Access for free at OpenStax 10.4.4 https://chem.libretexts.org/@go/page/38213


 Exercise 10.4.2

Determine the phase changes carbon dioxide undergoes when its temperature is varied, thus holding its pressure constant at
1500 kPa? At 500 kPa? At what approximate temperatures do these phase changes occur?

Answer
at 1500 kPa: s⟶ l at −45 °C, l⟶ g at −10 °C; at 500 kPa: s⟶ g at −58 °C

10.4.1: Supercritical Fluids


If we place a sample of water in a sealed container at 25 °C, remove the air, and let the vaporization-condensation equilibrium
establish itself, we are left with a mixture of liquid water and water vapor at a pressure of 0.03 atm. A distinct boundary between
the more dense liquid and the less dense gas is clearly observed. As we increase the temperature, the pressure of the water vapor
increases, as described by the liquid-gas curve in the phase diagram for water (Figure 10.4.2), and a two-phase equilibrium of
liquid and gaseous phases remains. At a temperature of 374 °C, the vapor pressure has risen to 218 atm, and any further increase in
temperature results in the disappearance of the boundary between liquid and vapor phases. All of the water in the container is now
present in a single phase whose physical properties are intermediate between those of the gaseous and liquid states. This phase of
matter is called a supercritical fluid, and the temperature and pressure above which this phase exists is the critical point (Figure
10.4.5). Above its critical temperature, a gas cannot be liquefied no matter how much pressure is applied. The pressure required to

liquefy a gas at its critical temperature is called the critical pressure. The critical temperatures and critical pressures of some
common substances are given in Table 10.4.1.
Table 10.4.1 : Critical Temperatures and Critical Pressures of select substances
Substance Critical Temperature (K) Critical Pressure (atm)

hydrogen 33.2 12.8

nitrogen 126.0 33.5

oxygen 154.3 49.7

carbon dioxide 304.2 73.0

ammonia 405.5 111.5

sulfur dioxide 430.3 77.7

water 647.1 217.7

Like a gas, a supercritical fluid will expand and fill a container, but its density is much greater than typical gas densities, typically
being close to those for liquids. Similar to liquids, these fluids are capable of dissolving nonvolatile solutes. They exhibit
essentially no surface tension and very low viscosities, however, so they can more effectively penetrate very small openings in a
solid mixture and remove soluble components. These properties make supercritical fluids extremely useful solvents for a wide
range of applications. For example, supercritical carbon dioxide has become a very popular solvent in the food industry, being used
to decaffeinate coffee, remove fats from potato chips, and extract flavor and fragrance compounds from citrus oils. It is nontoxic,
relatively inexpensive, and not considered to be a pollutant. After use, the CO2 can be easily recovered by reducing the pressure
and collecting the resulting gas.

Access for free at OpenStax 10.4.5 https://chem.libretexts.org/@go/page/38213


Figure 10.4.6 : (a) A sealed container of liquid carbon dioxide slightly below its critical point is heated, resulting in (b) the
formation of the supercritical fluid phase. Cooling the supercritical fluid lowers its temperature and pressure below the critical
point, resulting in the reestablishment of separate liquid and gaseous phases (c and d). Colored floats illustrate differences in
density between the liquid, gaseous, and supercritical fluid states. (credit: modification of work by “mrmrobin”/YouTube)
Four photographs are shown where each shows a circular container with a green and red float in each. In the left diagram, the
container is half filled with a colorless liquid and the floats sit on the surface of the liquid. In the second photo, the green float is
near the top and the red float lies near the bottom of the container. In the third photo, the fluid is darker and the green float sits
halfway up the container while the red is sitting at the bottom. In the right photo, the liquid is colorless again and the two floats sit
on the surface.

 Example 10.4.3: The Critical Temperature of Carbon Dioxide

If we shake a carbon dioxide fire extinguisher on a cool day (18 °C), we can hear liquid CO2 sloshing around inside the
cylinder. However, the same cylinder appears to contain no liquid on a hot summer day (35 °C). Explain these observations.

Solution
On the cool day, the temperature of the CO2 is below the critical temperature of CO2, 304 K or 31 °C (Table 10.4.1), so liquid
CO2 is present in the cylinder. On the hot day, the temperature of the CO2 is greater than its critical temperature of 31 °C.
Above this temperature no amount of pressure can liquefy CO2 so no liquid CO2 exists in the fire extinguisher.

 Exercise 10.4.3

Ammonia can be liquefied by compression at room temperature; oxygen cannot be liquefied under these conditions. Why do
the two gases exhibit different behavior?

Answer
The critical temperature of ammonia is 405.5 K, which is higher than room temperature. The critical temperature of oxygen
is below room temperature; thus oxygen cannot be liquefied at room temperature.

10.4.2: Decaffeinating Coffee Using Supercritical CO2


Coffee is the world’s second most widely traded commodity, following only petroleum. Across the globe, people love coffee’s
aroma and taste. Many of us also depend on one component of coffee—caffeine—to help us get going in the morning or stay alert
in the afternoon. But late in the day, coffee’s stimulant effect can keep you from sleeping, so you may choose to drink decaffeinated
coffee in the evening.
Since the early 1900s, many methods have been used to decaffeinate coffee. All have advantages and disadvantages, and all depend
on the physical and chemical properties of caffeine. Because caffeine is a somewhat polar molecule, it dissolves well in water, a
polar liquid. However, since many of the other 400-plus compounds that contribute to coffee’s taste and aroma also dissolve in
H2O, hot water decaffeination processes can also remove some of these compounds, adversely affecting the smell and taste of the
decaffeinated coffee. Dichloromethane (CH2Cl2) and ethyl acetate (CH3CO2C2H5) have similar polarity to caffeine, and are
therefore very effective solvents for caffeine extraction, but both also remove some flavor and aroma components, and their use
requires long extraction and cleanup times. Because both of these solvents are toxic, health concerns have been raised regarding the
effect of residual solvent remaining in the decaffeinated coffee.

Access for free at OpenStax 10.4.6 https://chem.libretexts.org/@go/page/38213


Figure 10.4.7 : (a) Caffeine molecules have both polar and nonpolar regions, making it soluble in solvents of varying polarities. (b)
The schematic shows a typical decaffeination process involving supercritical carbon dioxide.

Two images are shown and labeled “a” and “b.” Image a shows a molecule composed of a five member ring composed of two blue
spheres and three black spheres. One of the blue spheres is bonded to a black sphere bonded to three white spheres and one of the
black spheres is bonded to a white sphere. The other two black spheres are double bonded together and make up one side of a six-
membered ring that is also composed of two more black spheres and two blue spheres, both of which are bonded to a black sphere
bonded to three white spheres. The black spheres are each double bonded to red spheres. Image b shows a diagram of two vertical
tubes that lie next to one another. The left-hand tube is labeled “Extraction vessel.” A small tube labeled “Soaked beans” leads into
the top of the tube and a label at the bottom of the tube reads “Decaffeinated beans.” The right tube is labeled “Absorption vessel.”
A tube near the top of this tube is labeled “Water” and another tube leads from the right tube to the left. This tube is labeled with a
left-facing arrow and the phrase “supercritical carbon dioxide.” There is a tube leading away from the bottom which is labeled,
“Caffeine and water.” There is another tube that leads from the extraction vessel to the absorption vessel which is labeled,
“supercritical C O subscript 2 plus caffeine.”

Supercritical fluid extraction using carbon dioxide is now being widely used as a more effective and environmentally friendly
decaffeination method (Figure 10.4.7). At temperatures above 304.2 K and pressures above 7376 kPa, CO2 is a supercritical fluid,
with properties of both gas and liquid. Like a gas, it penetrates deep into the coffee beans; like a liquid, it effectively dissolves
certain substances. Supercritical carbon dioxide extraction of steamed coffee beans removes 97−99% of the caffeine, leaving
coffee’s flavor and aroma compounds intact. Because CO2 is a gas under standard conditions, its removal from the extracted coffee
beans is easily accomplished, as is the recovery of the caffeine from the extract. The caffeine recovered from coffee beans via this
process is a valuable product that can be used subsequently as an additive to other foods or drugs.

Summary
The temperature and pressure conditions at which a substance exists in solid, liquid, and gaseous states are summarized in a phase
diagram for that substance. Phase diagrams are combined plots of three pressure-temperature equilibrium curves: solid-liquid,
liquid-gas, and solid-gas. These curves represent the relationships between phase-transition temperatures and pressures. The point
of intersection of all three curves represents the substance’s triple point—the temperature and pressure at which all three phases are
in equilibrium. At pressures below the triple point, a substance cannot exist in the liquid state, regardless of its temperature. The
terminus of the liquid-gas curve represents the substance’s critical point, the pressure and temperature above which a liquid phase
cannot exist.

Glossary
critical point
temperature and pressure above which a gas cannot be condensed into a liquid

phase diagram
pressure-temperature graph summarizing conditions under which the phases of a substance can exist

supercritical fluid
substance at a temperature and pressure higher than its critical point; exhibits properties intermediate between those of gaseous
and liquid states

Access for free at OpenStax 10.4.7 https://chem.libretexts.org/@go/page/38213


triple point
temperature and pressure at which the vapor, liquid, and solid phases of a substance are in equilibrium

This page titled 10.4: Phase Diagrams is shared under a CC BY 4.0 license and was authored, remixed, and/or curated by OpenStax via source
content that was edited to the style and standards of the LibreTexts platform; a detailed edit history is available upon request.

Access for free at OpenStax 10.4.8 https://chem.libretexts.org/@go/page/38213


10.5: The Solid State of Matter
 Learning Objectives
Define and describe the bonding and properties of ionic, molecular, metallic, and covalent network crystalline solids
Describe the main types of crystalline solids: ionic solids, metallic solids, covalent network solids, and molecular solids
Explain the ways in which crystal defects can occur in a solid

When most liquids are cooled, they eventually freeze and form crystalline solids, solids in which the atoms, ions, or molecules are
arranged in a definite repeating pattern. It is also possible for a liquid to freeze before its molecules become arranged in an orderly
pattern. The resulting materials are called amorphous solids or noncrystalline solids (or, sometimes, glasses). The particles of such
solids lack an ordered internal structure and are randomly arranged (Figure 10.5.1).

Figure 10.5.1 : The entities of a solid phase may be arranged in a regular, repeating pattern (crystalline solids) or randomly
(amorphous).
The crystalline arrangement shows many circles drawn in rows and stacked together tightly. The amorphous arrangement shows
many circles spread slightly apart and in no organized pattern.
Metals and ionic compounds typically form ordered, crystalline solids. Substances that consist of large molecules, or a mixture of
molecules whose movements are more restricted, often form amorphous solids. For examples, candle waxes are amorphous solids
composed of large hydrocarbon molecules. Some substances, such as boron oxide (Figure 10.5.2), can form either crystalline or
amorphous solids, depending on the conditions under which it is produced. Also, amorphous solids may undergo a transition to the
crystalline state under appropriate conditions.

Figure 10.5.2 : (a) Diboron trioxide, B2O3, is normally found as a white, amorphous solid (a glass), which has a high degree of
disorder in its structure. (b) By careful, extended heating, it can be converted into a crystalline form of B2O3, which has a very
ordered arrangement.
The first structure of diboron trioxide shows five identical and separated hexagonal rings. The second structure of diboron trioxide
shows a more interconnected structure with four large rings forming a more stable structure.
Crystalline solids are generally classified according the nature of the forces that hold its particles together. These forces are
primarily responsible for the physical properties exhibited by the bulk solids. The following sections provide descriptions of the

Access for free at OpenStax 10.5.1 https://chem.libretexts.org/@go/page/38214


major types of crystalline solids: ionic, metallic, covalent network, and molecular.

10.5.1: Ionic Solids


Ionic solids, such as sodium chloride and nickel oxide, are composed of positive and negative ions that are held together by
electrostatic attractions, which can be quite strong (Figure 10.5.3). Many ionic crystals also have high melting points. This is due to
the very strong attractions between the ions—in ionic compounds, the attractions between full charges are (much) larger than those
between the partial charges in polar molecular compounds. This will be looked at in more detail in a later discussion of lattice
energies. Although they are hard, they also tend to be brittle, and they shatter rather than bend. Ionic solids do not conduct
electricity; however, they do conduct when molten or dissolved because their ions are free to move. Many simple compounds
formed by the reaction of a metallic element with a nonmetallic element are ionic.

Figure 10.5.3 : Sodium chloride is an ionic solid.


A cube composed of purple and green spheres is shown. The cube has dimensions of three by three spheres. The purple spheres are
slightly larger than the green spheres.

10.5.2: Metallic Solids


Metallic solids such as crystals of copper, aluminum, and iron are formed by metal atoms Figure 10.5.4. The structure of metallic
crystals is often described as a uniform distribution of atomic nuclei within a “sea” of delocalized electrons. The atoms within such
a metallic solid are held together by a unique force known as metallic bonding that gives rise to many useful and varied bulk
properties. All exhibit high thermal and electrical conductivity, metallic luster, and malleability. Many are very hard and quite
strong. Because of their malleability (the ability to deform under pressure or hammering), they do not shatter and, therefore, make
useful construction materials. The melting points of the metals vary widely. Mercury is a liquid at room temperature, and the alkali
metals melt below 200 °C. Several post-transition metals also have low melting points, whereas the transition metals melt at
temperatures above 1000 °C. These differences reflect differences in strengths of metallic bonding among the metals.

Figure 10.5.4 : Copper is a metallic solid.

10.5.3: Covalent Network Solids


Covalent network solids include crystals of diamond, silicon, some other nonmetals, and some covalent compounds such as silicon
dioxide (sand) and silicon carbide (carborundum, the abrasive on sandpaper). Many minerals have networks of covalent bonds. The
atoms in these solids are held together by a network of covalent bonds, as shown in Figure 10.5.5. To break or to melt a covalent
network solid, covalent bonds must be broken. Because covalent bonds are relatively strong, covalent network solids are typically
characterized by hardness, strength, and high melting points. For example, diamond is one of the hardest substances known and
melts above 3500 °C.

Access for free at OpenStax 10.5.2 https://chem.libretexts.org/@go/page/38214


Figure 10.5.5 . A covalent crystal contains a three-dimensional network of covalent bonds, as illustrated by the structures of
diamond, silicon dioxide, silicon carbide, and graphite. Graphite is an exceptional example, composed of planar sheets of covalent
crystals that are held together in layers by noncovalent forces. Unlike typical covalent solids, graphite is very soft and electrically
conductive.
The complex three dimensional structure of diamond, silicon dioxide, silicon carbide and graphite is shown along with a a smaller
repeating unit of the structure shown above each structure.

10.5.4: Molecular Solids


Molecular solids, such as ice, sucrose (table sugar), and iodine, as shown in Figure 10.5.6, are composed of neutral molecules. The
strengths of the attractive forces between the units present in different crystals vary widely, as indicated by the melting points of the
crystals. Small symmetrical molecules (nonpolar molecules), such as H2, N2, O2, and F2, have weak attractive forces and form
molecular solids with very low melting points (below −200 °C). Substances consisting of larger, nonpolar molecules have larger
attractive forces and melt at higher temperatures. Molecular solids composed of molecules with permanent dipole moments (polar
molecules) melt at still higher temperatures. Examples include ice (melting point, 0 °C) and table sugar (melting point, 185 °C).

Figure 10.5.6 : Carbon dioxide (CO2) consists of small, nonpolar molecules and forms a molecular solid with a melting point of
−78 °C. Iodine (I2) consists of larger, nonpolar molecules and forms a molecular solid that melts at 114 °C.
On the left, many red and grey molecules are densely stacked in a 3-D drawing to represent carbon dioxide. On the right, purple
molecules are scattered randomly to represent iodine.

10.5.5: Properties of Solids


A crystalline solid, like those listed in Table 10.5.1 has a precise melting temperature because each atom or molecule of the same
type is held in place with the same forces or energy. Thus, the attractions between the units that make up the crystal all have the
same strength and all require the same amount of energy to be broken. The gradual softening of an amorphous material differs
dramatically from the distinct melting of a crystalline solid. This results from the structural nonequivalence of the molecules in the
amorphous solid. Some forces are weaker than others, and when an amorphous material is heated, the weakest intermolecular
attractions break first. As the temperature is increased further, the stronger attractions are broken. Thus amorphous materials soften
over a range of temperatures.
Table 10.5.1 : Types of Crystalline Solids and Their Properties

Access for free at OpenStax 10.5.3 https://chem.libretexts.org/@go/page/38214


Type of Solid Type of Particles Type of Attractions Properties Examples

hard, brittle, conducts


electricity as a liquid but
ionic ions ionic bonds NaCl, Al2O3
not as a solid, high to very
high melting points

shiny, malleable, ductile,


conducts heat and
atoms of electropositive
metallic metallic bonds electricity well, variable Cu, Fe, Ti, Pb, U
elements
hardness and melting
temperature

atoms of electronegative very hard, not conductive,


covalent network covalent bonds C (diamond), SiO2, SiC
elements very high melting points

variable hardness, variable


molecular molecules (or atoms) IMFs brittleness, not conductive, H2O, CO2, I2, C12H22O11
low melting points

 Graphene: Material of the Future

Carbon is an essential element in our world. The unique properties of carbon atoms allow the existence of carbon-based life
forms such as ourselves. Carbon forms a huge variety of substances that we use on a daily basis, including those shown in
Figure 10.5.7. You may be familiar with diamond and graphite, the two most common allotropes of carbon. (Allotropes are
different structural forms of the same element.) Diamond is one of the hardest-known substances, whereas graphite is soft
enough to be used as pencil lead. These very different properties stem from the different arrangements of the carbon atoms in
the different allotropes.

Figure 10.5.7 : Diamond is extremely hard because of the strong bonding between carbon atoms in all directions. Graphite (in
pencil lead) rubs off onto paper due to the weak attractions between the carbon layers. An image of a graphite surface shows
the distance between the centers of adjacent carbon atoms. (credit left photo: modification of work by Steve Jurvetson; credit
middle photo: modification of work by United States Geological Survey)
A close up of a piece of diamond shows a three dimensional sturcture of a complex network of well bonded carbon atoms. A
close up of a graphite shows several layers of carbon sheets. Each sheet is composed of a repeated and connected hexagonal
structure of carbon atoms. The third diagram shows that the distance between the center of atoms is 1.4 times 10 to the power
of negative 10 meters.
You may be less familiar with a recently discovered form of carbon: graphene. Graphene was first isolated in 2004 by using
tape to peel off thinner and thinner layers from graphite. It is essentially a single sheet (one atom thick) of graphite. Graphene,
illustrated in Figure 10.5.8, is not only strong and lightweight, but it is also an excellent conductor of electricity and heat.

Access for free at OpenStax 10.5.4 https://chem.libretexts.org/@go/page/38214


These properties may prove very useful in a wide range of applications, such as vastly improved computer chips and circuits,
better batteries and solar cells, and stronger and lighter structural materials. The 2010 Nobel Prize in Physics was awarded to
Andre Geim and Konstantin Novoselov for their pioneering work with graphene.

Figure 10.5.8 : Graphene sheets can be formed into buckyballs, nanotubes, and stacked layers.
A sheet of interconnected hexagonal rings is shown at the top. Below it, a bukcyball is shown which is a sphere is composed of
hexagonal rings. In the lower middle image, a nanotube is shown that is made by rolling a graphene sheet into a tube. In the
lower right image, stacked sheets made up of four horizontal sheets composed of joined, hexagonal rings is shown.

10.5.6: Crystal Defects


In a crystalline solid, the atoms, ions, or molecules are arranged in a definite repeating pattern, but occasional defects may occur in
the pattern. Several types of defects are known, as illustrated in Figure 10.5.9. Vacancies are defects that occur when positions that
should contain atoms or ions are vacant. Less commonly, some atoms or ions in a crystal may occupy positions, called interstitial
sites, located between the regular positions for atoms. Other distortions are found in impure crystals, as, for example, when the
cations, anions, or molecules of the impurity are too large to fit into the regular positions without distorting the structure. Trace
amounts of impurities are sometimes added to a crystal (a process known as doping) in order to create defects in the structure that
yield desirable changes in its properties. For example, silicon crystals are doped with varying amounts of different elements to
yield suitable electrical properties for their use in the manufacture of semiconductors and computer chips.

Figure 10.5.9 : Types of crystal defects include vacancies, interstitial atoms, and substitutions impurities.

Access for free at OpenStax 10.5.5 https://chem.libretexts.org/@go/page/38214


Summary
Some substances form crystalline solids consisting of particles in a very organized structure; others form amorphous
(noncrystalline) solids with an internal structure that is not ordered. The main types of crystalline solids are ionic solids, metallic
solids, covalent network solids, and molecular solids. The properties of the different kinds of crystalline solids are due to the types
of particles of which they consist, the arrangements of the particles, and the strengths of the attractions between them. Because
their particles experience identical attractions, crystalline solids have distinct melting temperatures; the particles in amorphous
solids experience a range of interactions, so they soften gradually and melt over a range of temperatures. Some crystalline solids
have defects in the definite repeating pattern of their particles. These defects (which include vacancies, atoms or ions not in the
regular positions, and impurities) change physical properties such as electrical conductivity, which is exploited in the silicon
crystals used to manufacture computer chips.

Glossary
amorphous solid
(also, noncrystalline solid) solid in which the particles lack an ordered internal structure

covalent network solid


solid whose particles are held together by covalent bonds

crystalline solid
solid in which the particles are arranged in a definite repeating pattern

interstitial sites
spaces between the regular particle positions in any array of atoms or ions

ionic solid
solid composed of positive and negative ions held together by strong electrostatic attractions

metallic solid
solid composed of metal atoms

molecular solid
solid composed of neutral molecules held together by intermolecular forces of attraction

vacancy
defect that occurs when a position that should contain an atom or ion is vacant

This page titled 10.5: The Solid State of Matter is shared under a CC BY 4.0 license and was authored, remixed, and/or curated by OpenStax via
source content that was edited to the style and standards of the LibreTexts platform; a detailed edit history is available upon request.

Access for free at OpenStax 10.5.6 https://chem.libretexts.org/@go/page/38214


10.6: Lattice Structures in Crystalline Solids
 Learning Objectives
Describe the arrangement of atoms and ions in crystalline structures
Compute ionic radii using unit cell dimensions
Explain the use of X-ray diffraction measurements in determining crystalline structures

Over 90% of naturally occurring and man-made solids are crystalline. Most solids form with a regular arrangement of their
particles because the overall attractive interactions between particles are maximized, and the total intermolecular energy is
minimized, when the particles pack in the most efficient manner. The regular arrangement at an atomic level is often reflected at a
macroscopic level. In this module, we will explore some of the details about the structures of metallic and ionic crystalline solids,
and learn how these structures are determined experimentally.

10.6.1: The Structures of Metals


We will begin our discussion of crystalline solids by considering elemental metals, which are relatively simple because each
contains only one type of atom. A pure metal is a crystalline solid with metal atoms packed closely together in a repeating pattern.
Some of the properties of metals in general, such as their malleability and ductility, are largely due to having identical atoms
arranged in a regular pattern. The different properties of one metal compared to another partially depend on the sizes of their atoms
and the specifics of their spatial arrangements. We will explore the similarities and differences of four of the most common metal
crystal geometries in the sections that follow.

10.6.2: Unit Cells of Metals


The structure of a crystalline solid, whether a metal or not, is best described by considering its simplest repeating unit, which is
referred to as its unit cell. The unit cell consists of lattice points that represent the locations of atoms or ions. The entire structure
then consists of this unit cell repeating in three dimensions, as illustrated in Figure 10.6.1.

Figure 10.6.1 : A unit cell shows the locations of lattice points repeating in all directions.
A diagram of two images is shown. In the first image, a cube with a sphere at each corner is shown. The cube is labeled “Unit cell”
and the spheres at the corners are labeled “Lattice points.” The second image shows the same cube, but this time it is one cube
amongst eight that make up a larger cube. The original cube is shaded a color while the other cubes are not.
Let us begin our investigation of crystal lattice structure and unit cells with the most straightforward structure and the most basic
unit cell. To visualize this, imagine taking a large number of identical spheres, such as tennis balls, and arranging them uniformly in
a container. The simplest way to do this would be to make layers in which the spheres in one layer are directly above those in the
layer below, as illustrated in Figure 10.6.2. This arrangement is called simple cubic structure, and the unit cell is called the simple
cubic unit cell or primitive cubic unit cell.

Access for free at OpenStax 10.6.1 https://chem.libretexts.org/@go/page/38215


Figure 10.6.2 : .When metal atoms are arranged with spheres in one layer directly above or below spheres in another layer, the
lattice structure is called simple cubic. Note that the spheres are in contact.
A diagram of three images is shown. In the first image, a cube with a sphere at each corner is shown. The spheres at the corners are
circled. The second image shows the same cube, but this time the spheres at the corners are larger and shaded in. In the third image,
the cube is one cube amongst eight that make up a larger cube. The original cube is shaded a color while the other cubes are not.
In a simple cubic structure, the spheres are not packed as closely as they could be, and they only “fill” about 52% of the volume of
the container. This is a relatively inefficient arrangement, and only one metal (polonium, Po) crystallizes in a simple cubic
structure. As shown in Figure 10.6.3, a solid with this type of arrangement consists of planes (or layers) in which each atom
contacts only the four nearest neighbors in its layer; one atom directly above it in the layer above; and one atom directly below it in
the layer below. The number of other particles that each particle in a crystalline solid contacts is known as its coordination number.
For a polonium atom in a simple cubic array, the coordination number is, therefore, six.
<div data-mt-source="1"
&quot;&quot;
" height="200" width="463" src="/@api/deki/files/61021/CNX_Chem_10_06_SimpleCub2.jpg">
Figure 10.6.3 : An atom in a simple cubic lattice structure contacts six other atoms, so it has a coordination number of six.
In a simple cubic lattice, the unit cell that repeats in all directions is a cube defined by the centers of eight atoms, as shown in
Figure 10.6.4. Atoms at adjacent corners of this unit cell contact each other, so the edge length of this cell is equal to two atomic
radii, or one atomic diameter. A cubic unit cell contains only the parts of these atoms that are within it. Since an atom at a corner of
a simple cubic unit cell is contained by a total of eight unit cells, only one-eighth of that atom is within a specific unit cell. And
1
since each simple cubic unit cell has one atom at each of its eight “corners,” there is 8 × =1 atom within one simple cubic unit
8
cell.

Figure 10.6.4 : A simple cubic lattice unit cell contains one-eighth of an atom at each of its eight corners, so it contains one atom
total.
A diagram of two images is shown. In the first image, eight spheres are stacked together to form a cube and dots at the center of
each sphere are connected to form a cube shape. The dots are labeled “Lattice points” while a label under the image reads “Simple
cubic lattice cell.” The second image shows the portion of each sphere that lie inside the cube. The corners of the cube are shown
with small circles labeled “Lattice points” and the phrase “8 corners” is written below the image.

 Example 10.6.1: Calculating Atomic Radius and Density for Metals (Part 1)
The edge length of the unit cell of alpha polonium is 336 pm.
a. Determine the radius of a polonium atom.
b. Determine the density of alpha polonium.

Solution
Alpha polonium crystallizes in a simple cubic unit cell:

Access for free at OpenStax 10.6.2 https://chem.libretexts.org/@go/page/38215


(a) Two adjacent Po atoms contact each other, so the edge length of this cell is equal to two Po atomic radii: l = 2r . Therefore,
the radius of Po is
l 336 pm
r = = = 168 pm
2 2

(b) Density is given by


mass
density = .
volume

The density of polonium can be found by determining the density of its unit cell (the mass contained within a unit cell divided
by the volume of the unit cell). Since a Po unit cell contains one-eighth of a Po atom at each of its eight corners, a unit cell
contains one Po atom.
The mass of a Po unit cell can be found by:
1 Po atom 1 mol Po 208.998 g
−22
1 Po unit cell × × × = 3.47 × 10 g
23
1 Po unit cell 6.022 × 10 Po atoms 1 mol Po

The volume of a Po unit cell can be found by:


3 −10 3 −23 3
V =l = (336 × 10 cm) = 3.79 × 10 cm

(Note that the edge length was converted from pm to cm to get the usual volume units for density.)
Therefore, the density of
−22
3.471 × 10 g
3
Po = = 9.16 g/cm
−23
3.79 × 10 cm3

 Exercise 10.6.1

The edge length of the unit cell for nickel is 0.3524 nm. The density of Ni is 8.90 g/cm3. Does nickel crystallize in a simple
cubic structure? Explain.

Answer
No. If Ni were simple cubic, its density would be given by:
1 mol Ni 58.693 g
−23
1 Ni atom × × = 9.746 × 10 g
23
6.022 × 10 Ni atoms 1 mol Ni

3 −8 3 −23 3
V =l = (3.524 × 10 cm) = 4.376 × 10 cm

Then the density of Ni would be


−23
9.746 × 10 g
3
(= = 2.23 g/cm
−23 3
4.376 × 10 cm

Access for free at OpenStax 10.6.3 https://chem.libretexts.org/@go/page/38215


Since the actual density of Ni is not close to this, Ni does not form a simple cubic structure.

Most metal crystals are one of the four major types of unit cells. For now, we will focus on the three cubic unit cells: simple cubic
(which we have already seen), body-centered cubic unit cell, and face-centered cubic unit cell—all of which are illustrated in
Figure 10.6.5. (Note that there are actually seven different lattice systems, some of which have more than one type of lattice, for a
total of 14 different types of unit cells. We leave the more complicated geometries for later in this module.)

Figure 10.6.5 : Cubic unit cells of metals show (in the upper figures) the locations of lattice points and (in the lower figures) metal
atoms located in the unit cell.
Three pairs of images are shown. The first three images are in a row and are labeled “Lattice point locations” while the second
three images are in a row labeled “Cubic unit cells.” The first image in the top row shows a cube with black dots at each corner
while the first image in the second row is composed of eight spheres that are stacked together to form a cube and dots at the center
of each sphere are connected to form a cube shape. The name under this image reads “Simple cubic.” The second image in the top
row shows a cube with black dots at each corner and a red dot in the center while the second image in the second row is composed
of eight spheres that are stacked together to form a cube with one sphere in the center of the cube and dots at the center of each
corner sphere connected to form a cube shape. The name under this image reads “Body-centered cubic.” The third image in the top
row shows a cube with black dots at each corner and red dots in the center of each face while the third image in the second row is
composed of eight spheres that are stacked together to form a cube with six more spheres located in the center of each face of the
cube. Dots at the center of each corner sphere are connected to form a cube shape. The name under this image reads “Face-centered
cubic.”
Some metals crystallize in an arrangement that has a cubic unit cell with atoms at all of the corners and an atom in the center, as
shown in Figure 10.6.6. This is called a body-centered cubic (BCC) solid. Atoms in the corners of a BCC unit cell do not contact
each other but contact the atom in the center. A BCC unit cell contains two atoms: one-eighth of an atom at each of the eight
1
corners (8 × =1 atom from the corners) plus one atom from the center. Any atom in this structure touches four atoms in the
8
layer above it and four atoms in the layer below it. Thus, an atom in a BCC structure has a coordination number of eight.

Figure 10.6.6 : In a body-centered cubic structure, atoms in a specific layer do not touch each other. Each atom touches four atoms
in the layer above it and four atoms in the layer below it.
Three images are shown. The first image shows a cube with black dots at each corner and a red dot in the center while the second
image is composed of eight spheres that are stacked together to form a cube with one sphere in the center of the cube and dots at
the center of each corner sphere connected to form a cube shape. The name under this image reads “Body-centered cubic
structure.” The third image is the same as the second, but only shows the portions of the spheres that lie inside the cube shape.

Access for free at OpenStax 10.6.4 https://chem.libretexts.org/@go/page/38215


Atoms in BCC arrangements are much more efficiently packed than in a simple cubic structure, occupying about 68% of the total
volume. Isomorphous metals with a BCC structure include K, Ba, Cr, Mo, W, and Fe at room temperature. (Elements or
compounds that crystallize with the same structure are said to be isomorphous.)
Many other metals, such as aluminum, copper, and lead, crystallize in an arrangement that has a cubic unit cell with atoms at all of
the corners and at the centers of each face, as illustrated in Figure 10.6.7. This arrangement is called a face-centered cubic (FCC)
1
solid. A FCC unit cell contains four atoms: one-eighth of an atom at each of the eight corners (8 × =1 atom from the corners)
8
1
and one-half of an atom on each of the six faces (6 × =3 atoms from the faces). The atoms at the corners touch the atoms in the
2
centers of the adjacent faces along the face diagonals of the cube. Because the atoms are on identical lattice points, they have
identical environments.

Figure 10.6.7 :A face-centered cubic solid has atoms at the corners and, as the name implies, at the centers of the faces of its unit
cells.
Three images are shown. The first image shows a cube with black dots at each corner and red dots in the center of each face of the
cube while the second image is composed of eight spheres that are stacked together to form a cube with six more spheres, one
located on each face of the structure. Dots at the center of each corner sphere are connected to form a cube shape. The name under
this image reads “Face-centered cubic structure.” The third image is the same as the second, but only shows the portions of the
spheres that lie inside the cube shape.
Atoms in an FCC arrangement are packed as closely together as possible, with atoms occupying 74% of the volume. This structure
is also called cubic closest packing (CCP). In CCP, there are three repeating layers of hexagonally arranged atoms. Each atom
contacts six atoms in its own layer, three in the layer above, and three in the layer below. In this arrangement, each atom touches 12
near neighbors, and therefore has a coordination number of 12. The fact that FCC and CCP arrangements are equivalent may not be
immediately obvious, but why they are actually the same structure is illustrated in Figure 10.6.8.

Figure 10.6.8 : A CCP arrangement consists of three repeating layers (ABCABC…) of hexagonally arranged atoms. Atoms in a
CCP structure have a coordination number of 12 because they contact six atoms in their layer, plus three atoms in the layer above
and three atoms in the layer below. By rotating our perspective, we can see that a CCP structure has a unit cell with a face
containing an atom from layer A at one corner, atoms from layer B across a diagonal (at two corners and in the middle of the face),
and an atom from layer C at the remaining corner. This is the same as a face-centered cubic arrangement.
Three images are shown. In the first image, a side view shows a layer of blue spheres, labeled “C” stacked on top of, and sitting in
between the gaps in a second layer that is composed of green spheres, labeled “B,” which are sitting atop a purple layer of spheres
labeled “A.” A label below this image reads “Side view.” The second image shows a top view of the same layers of spheres, where
the top layer is “C,” the second layer is “B” and the lowest layer is “C.” This image is labeled “Top view” and written under this is
the phrase “Cubic closest packed structure.” The third image shows an upper view of the side of a cube composed of two sets of the
repeating layers shown in the other images. The layers are arranged “C, B, A, C, B, A, C” and the phrase written under this image
reads “Rotated view.”
Because closer packing maximizes the overall attractions between atoms and minimizes the total intermolecular energy, the atoms
in most metals pack in this manner. We find two types of closest packing in simple metallic crystalline structures: CCP, which we

Access for free at OpenStax 10.6.5 https://chem.libretexts.org/@go/page/38215


have already encountered, and hexagonal closest packing (HCP) shown in Figure 10.6.9. Both consist of repeating layers of
hexagonally arranged atoms. In both types, a second layer (B) is placed on the first layer (A) so that each atom in the second layer
is in contact with three atoms in the first layer. The third layer is positioned in one of two ways. In HCP, atoms in the third layer are
directly above atoms in the first layer (i.e., the third layer is also type A), and the stacking consists of alternating type A and type B
close-packed layers (i.e., ABABAB⋯). In CCP, atoms in the third layer are not above atoms in either of the first two layers (i.e.,
the third layer is type C), and the stacking consists of alternating type A, type B, and type C close-packed layers (i.e.,
ABCABCABC⋯). About two–thirds of all metals crystallize in closest-packed arrays with coordination numbers of 12. Metals
that crystallize in an HCP structure include Cd, Co, Li, Mg, Na, and Zn, and metals that crystallize in a CCP structure include Ag,
Al, Ca, Cu, Ni, Pb, and Pt.

Figure 10.6.9 : In both types of closest packing, atoms are packed as compactly as possible. Hexagonal closest packing consists of
two alternating layers (ABABAB…). Cubic closest packing consists of three alternating layers (ABCABCABC…).
Two images are shown. The first image, labeled “Hexagonal closest packed,” shows seven green spheres arranged in a circular
sheet lying atop another sheet that is the same except the spheres are purple. The second sheet is offset just a bit so that the spheres
of the top sheet lie in the grooves of the second sheet. Two more alternating green and purple layers of spheres lie below the first
pair. The second image shows seven blue spheres, labeled “Layer C,” arranged in a circular sheet laying atop another sheet, labeled
“Layer B” that is the same except the spheres are green. The second sheet is offset just a bit so that the spheres of the top sheet lie
in the grooves of the second sheet. Two more alternating purple and then blue layers of spheres lie below the first pair. The purple
layer is labeled “Layer A” and the phrase written below this image reads “Cubic closest packed.”

 Example 10.6.2: Calculating Atomic Radius and Density for Metals (Part 2)

Calcium crystallizes in a face-centered cubic structure. The edge length of its unit cell is 558.8 pm.
a. What is the atomic radius of Ca in this structure?
b. Calculate the density of Ca.

Solution
(a) In an FCC structure, Ca atoms contact each other across the diagonal of the face, so the length of the diagonal is equal to
four Ca atomic radii (d = 4r).

Two adjacent edges and the diagonal of the face form a right triangle, with the length of each side equal to 558.8 pm and the
length of the hypotenuse equal to four Ca atomic radii:
2 2 2
a +a =d

2 2 2
(558.8 pm) + (558.5 pm) = (4r)

Access for free at OpenStax 10.6.6 https://chem.libretexts.org/@go/page/38215


Solving this gives
−−−−−−−−−−−−−−−−−−−−−
2 2
(558.8 pm) + (558.5 pm)
r =√ = 197.6 pmg for a Ca radius.
16

mass
(b) Density is given by density = . The density of calcium can be found by determining the density of its unit cell:
volume
for example, the mass contained within a unit cell divided by the volume of the unit cell. A face-centered Ca unit cell has one-
1 1
eighth of an atom at each of the eight corners (8 × =1 atom) and one-half of an atom on each of the six faces 6× =3
8 2
atoms), for a total of four atoms in the unit cell.
The mass of the unit cell can be found by:
4 Ca atoms 1 mol Ca 40.078 g
−22
1 Ca unit cell × × × = 2.662 × 10 g
23
1 Ca unit cell 6.022 × 10 Ca atoms 1 mol Ca

The volume of a Ca unit cell can be found by:


3 −10 3 −22 3
V =a = (558.8 × 10 cm) = 1.745 × 10 cm

(Note that the edge length was converted from pm to cm to get the usual volume units for density.)
Then, the density of polonium:
−22
2.662 × 10 g 3
Po = = 1.53 g/cm
−22 3
1.745 × 10 cm

 Exercise 10.6.2

Silver crystallizes in an FCC structure. The edge length of its unit cell is 409 pm.
a. What is the atomic radius of Ag in this structure?
b. Calculate the density of Ag.

Answer a
144 pm
Answer b
10.5 g/cm3

In general, a unit cell is defined by the lengths of three axes (a, b, and c) and the angles (α, β, and γ) between them, as illustrated in
Figure 10.6.10. The axes are defined as being the lengths between points in the space lattice. Consequently, unit cell axes join
points with identical environments.

Figure 10.6.10: A unit cell is defined by the lengths of its three axes (a, b, and c) and the angles (α, β, and γ) between the axes.
A cube is shown where each corner has a black dot drawn on it. A circle in the bottom of the cube is composed of three double-
ended arrows. The left top of this circle is labeled “alpha,” the top right is labeled “beta” and the bottom is labeled “gamma.” The
bottom left corner of the cube is labeled “a” while the bottom of the back face is labeled “b” and the top, back, left corner is labeled
“c.”

Access for free at OpenStax 10.6.7 https://chem.libretexts.org/@go/page/38215


There are seven different lattice systems, some of which have more than one type of lattice, for a total of fourteen different unit
cells, which have the shapes shown in Figure 10.6.11.

Figure 10.6.11:There are seven different lattice systems and 14 different unit cells.
A table is composed of two columns and eight rows. The header row reads “System / Axes / Angles” and “Unit Cells .” The first
column reads “Cubic, a equals b equals c, alpha equals beta equals gamma equals 90 degrees,” “Tetragonal, a equals b does not
equal c, alpha equals beta equals gamma equals 90 degrees,” “Orthorhombic, a does not equal b does not equal c, alpha equals beta
equals gamma equals 90 degrees,” “Monoclinic, a does not equal b does not equal c, alpha equals gamma equals 90 degrees, beta
does not equal 90 degrees,” “Triclinic, a does not equal b does not equal c, alpha does not equal beta does not equal gamma does
not equal 90 degrees,” “Hexagonal, a equals b does not equal c, alpha equals beta equals 90 degrees, gamma equals 120 degrees,”
“Rhombohedral, a equals b equals c, alpha equals beta equals gamma does not equal 90 degrees.” The second column is composed
of diagrams. The first set of diagrams in the first cell show a cube with spheres at each corner labeled “Simple,” a cube with
spheres in each corner and on each face labeled “Face-centered” and a cube with spheres in each corner and one in the center
labeled “Body-centered.” The second set of diagrams in the second cell show a vertical rectangle with spheres at each corner
labeled “Simple” and a vertical rectangle with spheres in each corner and one in the center labeled “Body-centered.” The third set
of diagrams in the third cell show a vertical rectangle with spheres at each corner labeled “Simple,” a vertical rectangle with
spheres in each corner and one in the center labeled “Body-centered,” a vertical rectangle with spheres in each corner and one on
the top and bottom faces labeled “Base-centered,” and a vertical rectangle with spheres in each corner and one on each face labeled
“Face-centered.” The fourth set of diagrams in the fourth cell show a vertical rectangle with spheres at each corner that is slanted to
one side labeled “Simple” and a vertical rectangle with spheres in each corner that is slanted to one side and has two spheres in the
center is labeled “Body-centered.” The fifth diagrams in the fifth cell show a cube that is slanted with spheres at each corner while
the sixth diagram in the sixth cell shows a pair of hexagonal rings that are connected together to form a six-sided shape with
spheres at each corner. The seventh diagram in the seventh cell shows a rectangle that is slanted with spheres at each corner.

Access for free at OpenStax 10.6.8 https://chem.libretexts.org/@go/page/38215


10.6.3: The Structures of Ionic Crystals
Ionic crystals consist of two or more different kinds of ions that usually have different sizes. The packing of these ions into a
crystal structure is more complex than the packing of metal atoms that are the same size. Most monatomic ions behave as charged
spheres, and their attraction for ions of opposite charge is the same in every direction. Consequently, stable structures for ionic
compounds result (1) when ions of one charge are surrounded by as many ions as possible of the opposite charge and (2) when the
cations and anions are in contact with each other. Structures are determined by two principal factors: the relative sizes of the ions
and the ratio of the numbers of positive and negative ions in the compound.
In simple ionic structures, we usually find the anions, which are normally larger than the cations, arranged in a closest-packed
array. (As seen previously, additional electrons attracted to the same nucleus make anions larger and fewer electrons attracted to the
same nucleus make cations smaller when compared to the atoms from which they are formed.) The smaller cations commonly
occupy one of two types of holes (or interstices) remaining between the anions. The smaller of the holes is found between three
anions in one plane and one anion in an adjacent plane. The four anions surrounding this hole are arranged at the corners of a
tetrahedron, so the hole is called a tetrahedral hole. The larger type of hole is found at the center of six anions (three in one layer
and three in an adjacent layer) located at the corners of an octahedron; this is called an octahedral hole. Figure 10.6.12 illustrates
both of these types of holes.

Figure 10.6.12: Cations may occupy two types of holes between anions: octahedral holes or tetrahedral holes.
An image shows a top-view of a layer of blue spheres arranged in a sheet lying atop another sheet that is the same except the
spheres are green. The second sheet is offset just a bit so that the spheres of the top sheet lie in the grooves of the second sheet. A
third sheet composed of purple spheres lies at the bottom. The spaces created between the spheres in each layer are labeled
“Octahedral holes” and “Tetrahedral holes.”
Depending on the relative sizes of the cations and anions, the cations of an ionic compound may occupy tetrahedral or octahedral
holes, as illustrated in Figure 10.6.13. Relatively small cations occupy tetrahedral holes, and larger cations occupy octahedral
holes. If the cations are too large to fit into the octahedral holes, the anions may adopt a more open structure, such as a simple cubic
array. The larger cations can then occupy the larger cubic holes made possible by the more open spacing.

Figure 10.6.13:A cation’s size and the shape of the hole occupied by the compound are directly related.
A diagram of three images is shown. In the first image, eight stacked cubes, with purple spheres at each corner, that make up one
large cube are shown. The bottom left cube is different. It has green spheres at each corner and has four orange and six light purple
spheres located on the faces of the cube. Labels below this structure read “Tetrahedral hole” and “Cation radius is about 22.5 to
41.4 percent of the anion radius. In the second image, eight stacked cubes, with alternating orange and green spheres at each corner,
make up one large cube that is shown. The bottom left cube has darker lines that connect the spheres together. Labels below this
structure read “Octahedral hole” and “Cation radius is about 41.4 to 73.2 percent of the anion radius. In the third image, eight
stacked cubes, with purple spheres at each corner and light purple spheres on their interior faces, make up one large cube that is
shown. Labels below this structure read “Cubic hole” and “Cation radius is about 73.2 to 100 percent of the anion radius.”

Access for free at OpenStax 10.6.9 https://chem.libretexts.org/@go/page/38215


There are two tetrahedral holes for each anion in either an HCP or CCP array of anions. A compound that crystallizes in a closest-
packed array of anions with cations in the tetrahedral holes can have a maximum cation:anion ratio of 2:1; all of the tetrahedral
holes are filled at this ratio. Examples include Li2O, Na2O, Li2S, and Na2S. Compounds with a ratio of less than 2:1 may also
crystallize in a closest-packed array of anions with cations in the tetrahedral holes, if the ionic sizes fit. In these compounds,
however, some of the tetrahedral holes remain vacant.

 Example 10.6.3: Occupancy of Tetrahedral Holes

Zinc sulfide is an important industrial source of zinc and is also used as a white pigment in paint. Zinc sulfide crystallizes with
zinc ions occupying one-half of the tetrahedral holes in a closest-packed array of sulfide ions. What is the formula of zinc
sulfide?

Solution
Because there are two tetrahedral holes per anion (sulfide ion) and one-half of these holes are occupied by zinc ions, there must
1
be ×2 , or 1, zinc ion per sulfide ion. Thus, the formula is ZnS.
2

 Exercise 10.6.3: Lithium selenide


Lithium selenide can be described as a closest-packed array of selenide ions with lithium ions in all of the tetrahedral holes.
What it the formula of lithium selenide?

Answer
Li Se
2

The ratio of octahedral holes to anions in either an HCP or CCP structure is 1:1. Thus, compounds with cations in octahedral holes
in a closest-packed array of anions can have a maximum cation:anion ratio of 1:1. In NiO, MnS, NaCl, and KH, for example, all of
the octahedral holes are filled. Ratios of less than 1:1 are observed when some of the octahedral holes remain empty.

 Example 10.6.4: Stoichiometry of Ionic Compounds Sapphire

Aluminum oxide crystallizes with aluminum ions in two-thirds of the octahedral holes in a closest-packed array of oxide ions.
What is the formula of aluminum oxide?

Solution
Because there is one octahedral hole per anion (oxide ion) and only two-thirds of these holes are occupied, the ratio of
2
aluminum to oxygen must be :1, which would give Al 2/3 O . The simplest whole number ratio is 2:3, so the formula is Al2O3.
3

 Exercise 10.6.4

The white pigment titanium oxide crystallizes with titanium ions in one-half of the octahedral holes in a closest-packed array
of oxide ions. What is the formula of titanium oxide?

Answer
TiO
2

In a simple cubic array of anions, there is one cubic hole that can be occupied by a cation for each anion in the array. In CsCl, and
in other compounds with the same structure, all of the cubic holes are occupied. Half of the cubic holes are occupied in SrH2, UO2,
SrCl2, and CaF2.

Access for free at OpenStax 10.6.10 https://chem.libretexts.org/@go/page/38215


Different types of ionic compounds often crystallize in the same structure when the relative sizes of their ions and their
stoichiometries (the two principal features that determine structure) are similar.

10.6.4: Unit Cells of Ionic Compounds


Many ionic compounds crystallize with cubic unit cells, and we will use these compounds to describe the general features of ionic
structures. When an ionic compound is composed of cations and anions of similar size in a 1:1 ratio, it typically forms a simple
cubic structure. Cesium chloride, CsCl, (Figure 10.6.14) is an example of this, with Cs+ and Cl− having radii of 174 pm and 181
pm, respectively. We can think of this as chloride ions forming a simple cubic unit cell, with a cesium ion in the center; or as
cesium ions forming a unit cell with a chloride ion in the center; or as simple cubic unit cells formed by Cs+ ions overlapping unit
cells formed by Cl− ions. Cesium ions and chloride ions touch along the body diagonals of the unit cells. One cesium ion and one
chloride ion are present per unit cell, giving the l:l stoichiometry required by the formula for cesium chloride. Note that there is no
lattice point in the center of the cell, and CsCl is not a BCC structure because a cesium ion is not identical to a chloride ion.

Figure 10.6.14: Ionic compounds with similar-sized cations and anions, such as CsCl, usually form a simple cubic structure. They
can be described by unit cells with either cations at the corners or anions at the corners.
Three images are shown. The first image shows a cube with black dots at each corner and a red dot in the center. This cube is
stacked with seven others that are not colored to form a larger cube. The second image is composed of eight spheres that are
grouped together to form a cube with one smaller sphere in the center. The name under this image reads “Body-centered simple
cubic structure.” The third image shows five horizontal layers of purple spheres with layers of smaller green spheres in between.
We have said that the location of lattice points is arbitrary. This is illustrated by an alternate description of the CsCl structure in
which the lattice points are located in the centers of the cesium ions. In this description, the cesium ions are located on the lattice
points at the corners of the cell, and the chloride ion is located at the center of the cell. The two unit cells are different, but they
describe identical structures.
When an ionic compound is composed of a 1:1 ratio of cations and anions that differ significantly in size, it typically crystallizes
with an FCC unit cell, like that shown in Figure 10.6.15. Sodium chloride, NaCl, is an example of this, with Na+ and Cl− having
radii of 102 pm and 181 pm, respectively. We can think of this as chloride ions forming an FCC cell, with sodium ions located in
the octahedral holes in the middle of the cell edges and in the center of the cell. The sodium and chloride ions touch each other
along the cell edges. The unit cell contains four sodium ions and four chloride ions, giving the 1:1 stoichiometry required by the
formula, NaCl.

Access for free at OpenStax 10.6.11 https://chem.libretexts.org/@go/page/38215


Figure 10.6.15: Ionic compounds with anions that are much larger than cations, such as NaCl, usually form an FCC structure. They
can be described by FCC unit cells with cations in the octahedral holes.
Three images are shown. The first image shows a cube with black dots at each corner and a red dot in the center. This cube is
stacked with seven others that are not colored to form a larger cube. The second image is composed of eight spheres that are
grouped together to form a cube with one much larger sphere in the center. The name under this image reads “Body-centered
simple cubic structure.” The third image shows seven horizontal layers of alternating purple and green spheres that are slightly
offset with one another and form a large cube.
The cubic form of zinc sulfide, zinc blende, also crystallizes in an FCC unit cell, as illustrated in Figure 10.6.16. This structure
contains sulfide ions on the lattice points of an FCC lattice. (The arrangement of sulfide ions is identical to the arrangement of
chloride ions in sodium chloride.) The radius of a zinc ion is only about 40% of the radius of a sulfide ion, so these small Zn2+ ions
are located in alternating tetrahedral holes, that is, in one half of the tetrahedral holes. There are four zinc ions and four sulfide ions
in the unit cell, giving the empirical formula ZnS.

Figure 10.6.16: ZnS, zinc sulfide (or zinc blende) forms an FCC unit cell with sulfide ions at the lattice points and much smaller
zinc ions occupying half of the tetrahedral holes in the structure.
Two images are shown. The first image shows a cube with black dots at each corner and a red dot in the center of each face of the
cube. This cube is stacked with seven others that are not colored to form a larger cube. The second image is composed of eight
spheres that form the corners of a cube with six other spheres located in the face of the cube. The spheres are connected to one
another by lines. The name under this image reads “Z n S, face-centered unit cell.”
A calcium fluoride unit cell, like that shown in Figure 10.6.17, is also an FCC unit cell, but in this case, the cations are located on
the lattice points; equivalent calcium ions are located on the lattice points of an FCC lattice. All of the tetrahedral sites in the FCC
array of calcium ions are occupied by fluoride ions. There are four calcium ions and eight fluoride ions in a unit cell, giving a
calcium:fluorine ratio of 1:2, as required by the chemical formula, CaF2. Close examination of Figure 10.6.17 will reveal a simple
cubic array of fluoride ions with calcium ions in one half of the cubic holes. The structure cannot be described in terms of a space
lattice of points on the fluoride ions because the fluoride ions do not all have identical environments. The orientation of the four
calcium ions about the fluoride ions differs.

Access for free at OpenStax 10.6.12 https://chem.libretexts.org/@go/page/38215


Figure 10.6.17: Calcium fluoride, CaF2, forms an FCC unit cell with calcium ions (green) at the lattice points and fluoride ions
(red) occupying all of the tetrahedral sites between them.
Two images are shown. The first image shows a cube with black dots at each corner and a red dot in the center of each face of the
cube. This cube is stacked with seven others that are not colored to form a larger cube. The second image is composed of eight
small green spheres that form the corners of a cube with six other small green spheres located in the faces of the cube. Eight larger
green spheres are spaced inside the cube and all of the spheres are connect to one another by lines. The name under this image
reads “C a F, subscript 2, face-centered unit cell.”

10.6.5: Calculation of Ionic Radii


If we know the edge length of a unit cell of an ionic compound and the position of the ions in the cell, we can calculate ionic radii
for the ions in the compound if we make assumptions about individual ionic shapes and contacts.

 Example 10.6.5: Calculation of Ionic Radii

The edge length of the unit cell of LiCl (NaCl-like structure, FCC) is 0.514 nm or 5.14 Å. Assuming that the lithium ion is
small enough so that the chloride ions are in contact, calculate the ionic radius for the chloride ion. Note: The length unit
angstrom, Å, is often used to represent atomic-scale dimensions and is equivalent to 10−10 m.

Solution
On the face of a LiCl unit cell, chloride ions contact each other across the diagonal of the face:

Drawing a right triangle on the face of the unit cell, we see that the length of the diagonal is equal to four chloride radii (one
radius from each corner chloride and one diameter—which equals two radii—from the chloride ion in the center of the face),
so d = 4r . From the Pythagorean theorem, we have:
2 2 2
a +a =d

which yields:
2 2 2 2
(0.514 nm) + (0.514 nm) = (4r) = 16 r

Solving this gives:


−−−−−−−−−−−−−−−−−−−−−
(0.514 nm)2 + (0.514 nm)2
r =√ = 0.182 nm (1.82 Å) f or a −
Cl radius.
16

Access for free at OpenStax 10.6.13 https://chem.libretexts.org/@go/page/38215


 Exercise 10.6.6
The edge length of the unit cell of KCl (NaCl-like structure, FCC) is 6.28 Å. Assuming anion-cation contact along the cell
edge, calculate the radius of the potassium ion. The radius of the chloride ion is 1.82 Å.

Answer
The radius of the potassium ion is 1.33 Å.

It is important to realize that values for ionic radii calculated from the edge lengths of unit cells depend on numerous assumptions,
such as a perfect spherical shape for ions, which are approximations at best. Hence, such calculated values are themselves
approximate and comparisons cannot be pushed too far. Nevertheless, this method has proved useful for calculating ionic radii from
experimental measurements such as X-ray crystallographic determinations.

10.6.6: X-Ray Crystallography


The size of the unit cell and the arrangement of atoms in a crystal may be determined from measurements of the diffraction of X-
rays by the crystal, termed X-ray crystallography. Diffraction is the change in the direction of travel experienced by an
electromagnetic wave when it encounters a physical barrier whose dimensions are comparable to those of the wavelength of the
light. X-rays are electromagnetic radiation with wavelengths about as long as the distance between neighboring atoms in crystals
(on the order of a few Å).
When a beam of monochromatic X-rays strikes a crystal, its rays are scattered in all directions by the atoms within the crystal.
When scattered waves traveling in the same direction encounter one another, they undergo interference, a process by which the
waves combine to yield either an increase or a decrease in amplitude (intensity) depending upon the extent to which the combining
waves’ maxima are separated (Figure 10.6.18).

Figure 10.6.18: Light waves occupying the same space experience interference, combining to yield waves of greater (a) or lesser
(b) intensity, depending upon the separation of their maxima and minima.
A pair of images is shown that has four sections. In the first section, two sinusoidal waves are shown, one drawn above the other,
and a section from the top of one curve to the top of the next curve is labeled “lambda.” The curves align with one another. The
phrase below this reads “Constructive interference.” A right facing arrow leads from the first section to the second, which shows
one larger sinusoidal curve that has higher and lower peaks and troughs. A section from the top of one curve to the top of the next
curve is labeled “lambda” and the phrase below this reads “Maxima and minima reinforce.” In the second section, two sinusoidal
waves are shown, one drawn above the other, and a section from the top of one curve to the top of the next curve is labeled
“lambda.” The curves do not align with one another. The phrase below this reads “Destructive interference.” A right facing arrow
leads from the first section to the second, which shows one flat line. The phrase below this reads “Maxima and minima cancel.”
When X-rays of a certain wavelength, λ, are scattered by atoms in adjacent crystal planes separated by a distance, d, they may
undergo constructive interference when the difference between the distances traveled by the two waves prior to their combination is
an integer factor, n, of the wavelength. This condition is satisfied when the angle of the diffracted beam, θ, is related to the
wavelength and interatomic distance by the equation:
nλ = 2d sin θ (10.6.1)

This relation is known as the Bragg equation in honor of W. H. Bragg, the English physicist who first explained this phenomenon.
Figure 10.6.18 illustrates two examples of diffracted waves from the same two crystal planes. The figure on the left depicts waves
diffracted at the Bragg angle, resulting in constructive interference, while that on the right shows diffraction and a different angle
that does not satisfy the Bragg condition, resulting in destructive interference.

Access for free at OpenStax 10.6.14 https://chem.libretexts.org/@go/page/38215


Figure 10.6.19: The diffraction of X-rays scattered by the atoms within a crystal permits the determination of the distance between
the atoms. The top image depicts constructive interference between two scattered waves and a resultant diffracted wave of high
intensity. The bottom image depicts destructive interference and a low intensity diffracted wave.
Two similar figures are shown. The first figure, labeled “Constructive Interference,” shows two horizontal rows of seven black dots
with a line passing through them. The fourth dots of each row have a vertical line connecting them. The distance between these
rows is labeled “d.” A beam labeled “Incident beam” descends at an angle labeled “theta” until it hits the line connecting the fourth
dots, after which a diffracted beam ascends at the same angle “theta.” A dotted line is drawn across the diffracted beam. The second
figure, labeled “Destructive interference,” is very similar, except that the angles “theta” are far more acute, making the slopes of the
beams more shallow.

An X-ray diffractometer, such as the one illustrated in Figure 10.6.20, may be used to measure the angles at which X-rays are
diffracted when interacting with a crystal as described earlier. From such measurements, the Bragg equation may be used to
compute distances between atoms as demonstrated in the following example exercise.

Figure 10.6.20: (a) In a diffractometer, a beam of X-rays strikes a crystalline material, producing (b) an X-ray diffraction pattern
that can be analyzed to determine the crystal structure.
A diagram, labeled “a” shows a cube on the left with a channel bored into its right side labeled “X dash ray source.” A beam is
leaving from this channel and traveling in a horizontal line toward an oval-shaped, short tube, labeled “Collimator to focus beam”
and “X dash ray diffraction,” where it passes through a cube labeled “Crystalline material” and scatters onto a vertical sheet labeled
“Imaging surface.” A second diagram, labeled “b,” shows a square sheet with a large dot in the center labeled “X dash ray beam,”
that is surrounded by smaller dots arranged in rings and labeled “Diffracted X dash rays.”

Access for free at OpenStax 10.6.15 https://chem.libretexts.org/@go/page/38215


 Example 10.6.6: Using the Bragg Equation
In a diffractometer, X-rays with a wavelength of 0.1315 nm were used to produce a diffraction pattern for copper. The first
order diffraction (n = 1) occurred at an angle θ = 25.25°. Determine the spacing between the diffracting planes in copper.

Solution
The distance between the planes is found by solving the Bragg equation (Equation 10.6.1) for d.
This gives
nλ 1(0.1315 nm)
d = = = 0.154 nm
2 sin θ 2 sin(25.25°)

 Exercise 10.6.6

A crystal with spacing between planes equal to 0.394 nm diffracts X-rays with a wavelength of 0.147 nm. What is the angle for
the first order diffraction?

Answer
21.9°

 X-ray Crystallographer Rosalind Franklin


The discovery of the structure of DNA in 1953 by Francis Crick and James Watson is one of the great achievements in the
history of science. They were awarded the 1962 Nobel Prize in Physiology or Medicine, along with Maurice Wilkins, who
provided experimental proof of DNA’s structure. British chemist Rosalind Franklin made invaluable contributions to this
monumental achievement through her work in measuring X-ray diffraction images of DNA. Early in her career, Franklin’s
research on the structure of coals proved helpful to the British war effort. After shifting her focus to biological systems in the
early 1950s, Franklin and doctoral student Raymond Gosling discovered that DNA consists of two forms: a long, thin fiber
formed when wet (type “B”) and a short, wide fiber formed when dried (type “A”). Her X-ray diffraction images of DNA
provided the crucial information that allowed Watson and Crick to confirm that DNA forms a double helix, and to determine
details of its size and structure. Franklin also conducted pioneering research on viruses and the RNA that contains their genetic
information, uncovering new information that radically changed the body of knowledge in the field. After developing ovarian
cancer, Franklin continued to work until her death in 1958 at age 37. Among many posthumous recognitions of her work, the
Chicago Medical School of Finch University of Health Sciences changed its name to the Rosalind Franklin University of
Medicine and Science in 2004, and adopted an image of her famous X-ray diffraction image of DNA as its official university
logo.

Access for free at OpenStax 10.6.16 https://chem.libretexts.org/@go/page/38215


Figure 10.6.7 .This illustration shows an X-ray diffraction image similar to the one Franklin found in her research. (credit:
National Institutes of Health)

10.6.7: Key Concepts and Summary


The structures of crystalline metals and simple ionic compounds can be described in terms of packing of spheres. Metal atoms can
pack in hexagonal closest-packed structures, cubic closest-packed structures, body-centered structures, and simple cubic structures.
The anions in simple ionic structures commonly adopt one of these structures, and the cations occupy the spaces remaining
between the anions. Small cations usually occupy tetrahedral holes in a closest-packed array of anions. Larger cations usually
occupy octahedral holes. Still larger cations can occupy cubic holes in a simple cubic array of anions. The structure of a solid can
be described by indicating the size and shape of a unit cell and the contents of the cell. The type of structure and dimensions of the
unit cell can be determined by X-ray diffraction measurements.

Glossary
body-centered cubic (BCC) solid
crystalline structure that has a cubic unit cell with lattice points at the corners and in the center of the cell

body-centered cubic unit cell


simplest repeating unit of a body-centered cubic crystal; it is a cube containing lattice points at each corner and in the center of
the cube

Bragg equation
equation that relates the angles at which X-rays are diffracted by the atoms within a crystal

coordination number
number of atoms closest to any given atom in a crystal or to the central metal atom in a complex

cubic closest packing (CCP)


crystalline structure in which planes of closely packed atoms or ions are stacked as a series of three alternating layers of
different relative orientations (ABC)

diffraction
redirection of electromagnetic radiation that occurs when it encounters a physical barrier of appropriate dimensions

face-centered cubic (FCC) solid


crystalline structure consisting of a cubic unit cell with lattice points on the corners and in the center of each face

face-centered cubic unit cell


simplest repeating unit of a face-centered cubic crystal; it is a cube containing lattice points at each corner and in the center of
each face

Access for free at OpenStax 10.6.17 https://chem.libretexts.org/@go/page/38215


hexagonal closest packing (HCP)
crystalline structure in which close packed layers of atoms or ions are stacked as a series of two alternating layers of different
relative orientations (AB)

hole
(also, interstice) space between atoms within a crystal

isomorphous
possessing the same crystalline structure

octahedral hole
open space in a crystal at the center of six particles located at the corners of an octahedron

simple cubic unit cell


(also, primitive cubic unit cell) unit cell in the simple cubic structure

simple cubic structure


crystalline structure with a cubic unit cell with lattice points only at the corners

space lattice
all points within a crystal that have identical environments

tetrahedral hole
tetrahedral space formed by four atoms or ions in a crystal

unit cell
smallest portion of a space lattice that is repeated in three dimensions to form the entire lattice

X-ray crystallography
experimental technique for determining distances between atoms in a crystal by measuring the angles at which X-rays are
diffracted when passing through the crystal

Paul Flowers (University of North Carolina - Pembroke), Klaus Theopold (University of Delaware) and Richard Langley (Stephen
F. Austin State University) with contributing authors. Textbook content produced by OpenStax College is licensed under a Creative
Commons Attribution License 4.0 license. Download for free at http://cnx.org/contents/85abf193-2bd...a7ac8df6@9.110).

This page titled 10.6: Lattice Structures in Crystalline Solids is shared under a CC BY 4.0 license and was authored, remixed, and/or curated by
OpenStax via source content that was edited to the style and standards of the LibreTexts platform; a detailed edit history is available upon request.

Access for free at OpenStax 10.6.18 https://chem.libretexts.org/@go/page/38215


10.E: Liquids and Solids (Exercises)
10.E.1: 10.1: Intermolecular Forces
10.E.1.1: Q10.1.1
In terms of their bulk properties, how do liquids and solids differ? How are they similar?

10.E.1.2: S10.1.1
Liquids and solids are similar in that they are matter composed of atoms, ions, or molecules. They are incompressible and have
similar densities that are both much larger than those of gases. They are different in that liquids have no fixed shape, and solids are
rigid.

10.E.1.3: Q10.1.2
In terms of the kinetic molecular theory, in what ways are liquids similar to solids? In what ways are liquids different from solids?

10.E.1.4: Q10.1.3
In terms of the kinetic molecular theory, in what ways are liquids similar to gases? In what ways are liquids different from gases?

10.E.1.5: S10.1.3
They are similar in that the atoms or molecules are free to move from one position to another. They differ in that the particles of a
liquid are confined to the shape of the vessel in which they are placed. In contrast, a gas will expand without limit to fill the space
into which it is placed.

10.E.1.6: Q10.1.4
Explain why liquids assume the shape of any container into which they are poured, whereas solids are rigid and retain their shape.

10.E.1.7: Q10.1.5
What is the evidence that all neutral atoms and molecules exert attractive forces on each other?

10.E.1.8: S10.1.5
All atoms and molecules will condense into a liquid or solid in which the attractive forces exceed the kinetic energy of the
molecules, at sufficiently low temperature.

10.E.1.9: Q10.1.6
Open the PhET States of Matter Simulation to answer the following questions:
a. Select the Solid, Liquid, Gas tab. Explore by selecting different substances, heating and cooling the systems, and changing the
state. What similarities do you notice between the four substances for each phase (solid, liquid, gas)? What differences do you
notice?
b. For each substance, select each of the states and record the given temperatures. How do the given temperatures for each state
correlate with the strengths of their intermolecular attractions? Explain.
c. Select the Interaction Potential tab, and use the default neon atoms. Move the Ne atom on the right and observe how the
potential energy changes. Select the Total Force button, and move the Ne atom as before. When is the total force on each atom
attractive and large enough to matter? Then select the Component Forces button, and move the Ne atom. When do the attractive
(van der Waals) and repulsive (electron overlap) forces balance? How does this relate to the potential energy versus the distance
between atoms graph? Explain.

10.E.1.10: Q10.1.7
Define the following and give an example of each:
a. dispersion force
b. dipole-dipole attraction
c. hydrogen bond

Access for free at OpenStax 10.E.1 https://chem.libretexts.org/@go/page/44115


10.E.1.11: S10.1.7
a. Dispersion forces occur as an atom develops a temporary dipole moment when its electrons are distributed asymmetrically
about the nucleus. This structure is more prevalent in large atoms such as argon or radon. A second atom can then be distorted
by the appearance of the dipole in the first atom. The electrons of the second atom are attracted toward the positive end of the
first atom, which sets up a dipole in the second atom. The net result is rapidly fluctuating, temporary dipoles that attract one
another (example: Ar).
b. A dipole-dipole attraction is a force that results from an electrostatic attraction of the positive end of one polar molecule for the
negative end of another polar molecule (example: ICI molecules attract one another by dipole-dipole interaction).
c. Hydrogen bonds form whenever a hydrogen atom is bonded to one of the more electronegative atoms, such as a fluorine,
oxygen, nitrogen, or chlorine atom. The electrostatic attraction between the partially positive hydrogen atom in one molecule
and the partially negative atom in another molecule gives rise to a strong dipole-dipole interaction called a hydrogen bond
(example: HF ⋯ HF).

10.E.1.12: Q10.1.8
The types of intermolecular forces in a substance are identical whether it is a solid, a liquid, or a gas. Why then does a substance
change phase from a gas to a liquid or to a solid?

10.E.1.13: Q10.1.9
Why do the boiling points of the noble gases increase in the order He < Ne < Ar < Kr < Xe?

10.E.1.14: S10.1.9
The London forces typically increase as the number of electrons increase.

10.E.1.15: Q10.1.10
Neon and HF have approximately the same molecular masses.
a. Explain why the boiling points of Neon and HF differ.
b. Compare the change in the boiling points of Ne, Ar, Kr, and Xe with the change of the boiling points of HF, HCl, HBr, and HI,
and explain the difference between the changes with increasing atomic or molecular mass.

10.E.1.16: Q10.1.11
Arrange each of the following sets of compounds in order of increasing boiling point temperature:
a. HCl, H2O, SiH4
b. F2, Cl2, Br2
c. CH4, C2H6, C3H8
d. O2, NO, N2

10.E.1.17: S10.1.11
(a) SiH4 < HCl < H2O; (b) F2 < Cl2 < Br2; (c) CH4 < C2H6 < C3H8; (d) N2 < O2 < NO

10.E.1.18: Q10.1.12
The molecular mass of butanol, C4H9OH, is 74.14; that of ethylene glycol, CH2(OH)CH2OH, is 62.08, yet their boiling points are
117.2 °C and 174 °C, respectively. Explain the reason for the difference.

10.E.1.19: Q10.1.13
On the basis of intermolecular attractions, explain the differences in the boiling points of n–butane (−1 °C) and chloroethane (12
°C), which have similar molar masses.

10.E.1.20: S10.1.13
Only rather small dipole-dipole interactions from C-H bonds are available to hold n-butane in the liquid state. Chloroethane,
however, has rather large dipole interactions because of the Cl-C bond; the interaction is therefore stronger, leading to a higher
boiling point.

Access for free at OpenStax 10.E.2 https://chem.libretexts.org/@go/page/44115


10.E.1.21: Q10.1.14
On the basis of dipole moments and/or hydrogen bonding, explain in a qualitative way the differences in the boiling points of
acetone (56.2 °C) and 1-propanol (97.4 °C), which have similar molar masses.

10.E.1.22: Q10.1.15
The melting point of H2O(s) is 0 °C. Would you expect the melting point of H2S(s) to be −85 °C, 0 °C, or 185 °C? Explain your
answer.

10.E.1.23: S10.1.15
−85 °C. Water has stronger hydrogen bonds so it melts at a higher temperature.

10.E.1.24: Q10.1.16
Silane (SiH4), phosphine (PH3), and hydrogen sulfide (H2S) melt at −185 °C, −133 °C, and −85 °C, respectively. What does this
suggest about the polar character and intermolecular attractions of the three compounds?

10.E.1.25: Q10.1.17
Explain why a hydrogen bond between two water molecules is weaker than a hydrogen bond between two hydrogen fluoride
molecules.

10.E.1.26: S10.1.17
The hydrogen bond between two hydrogen fluoride molecules is stronger than that between two water molecules because the
electronegativity of F is greater than that of O. Consequently, the partial negative charge on F is greater than that on O. The
hydrogen bond between the partially positive H and the larger partially negative F will be stronger than that formed between H and
O.

10.E.1.27: Q10.1.18
Under certain conditions, molecules of acetic acid, CH3COOH, form “dimers,” pairs of acetic acid molecules held together by
strong intermolecular attractions:

Draw a dimer of acetic acid, showing how two CH3COOH molecules are held together, and stating the type of IMF that is
responsible.

10.E.1.28: Q10.1.19
Proteins are chains of amino acids that can form in a variety of arrangements, one of which is a helix. What kind of IMF is
responsible for holding the protein strand in this shape? On the protein image, show the locations of the IMFs that hold the protein
together:

10.E.1.29: S10.1.19
H-bonding is the principle IMF holding the DNA strands together. The H-bonding is between the N − H and C = O .

10.E.1.30: Q10.1.20
The density of liquid NH3 is 0.64 g/mL; the density of gaseous NH3 at STP is 0.0007 g/mL. Explain the difference between the
densities of these two phases.

Access for free at OpenStax 10.E.3 https://chem.libretexts.org/@go/page/44115


10.E.1.31: Q10.1.21
Identify the intermolecular forces present in the following solids:
a. CH3CH2OH
b. CH3CH2CH3
c. CH3CH2Cl

10.E.1.32: S10.1.21
(a) hydrogen bonding and dispersion forces; (b) dispersion forces; (c) dipole-dipole attraction and dispersion forces

10.E.2: 10.2: Properties of Liquids


10.E.2.1: Q10.2.1
The test tubes shown here contain equal amounts of the specified motor oils. Identical metal spheres were dropped at the same time
into each of the tubes, and a brief moment later, the spheres had fallen to the heights indicated in the illustration. Rank the motor
oils in order of increasing viscosity, and explain your reasoning:

10.E.2.2: Q10.2.2
Although steel is denser than water, a steel needle or paper clip placed carefully lengthwise on the surface of still water can be
made to float. Explain at a molecular level how this is possible:

(credit: Cory Zanker)

10.E.2.3: S10.2.2
The water molecules have strong intermolecular forces of hydrogen bonding. The water molecules are thus attracted strongly to
one another and exhibit a relatively large surface tension, forming a type of “skin” at its surface. This skin can support a bug or
paper clip if gently placed on the water.

10.E.2.4: Q10.2.3
The surface tension and viscosity values for diethyl ether, acetone, ethanol, and ethylene glycol are shown here.

Access for free at OpenStax 10.E.4 https://chem.libretexts.org/@go/page/44115


a. Explain their differences in viscosity in terms of the size and shape of their molecules and their IMFs.
b. Explain their differences in surface tension in terms of the size and shape of their molecules and their IMFs:

10.E.2.5: Q10.2.4
You may have heard someone use the figure of speech “slower than molasses in winter” to describe a process that occurs slowly.
Explain why this is an apt idiom, using concepts of molecular size and shape, molecular interactions, and the effect of changing
temperature.

10.E.2.6: S10.2.4
Temperature has an effect on intermolecular forces: the higher the temperature, the greater the kinetic energies of the molecules and
the greater the extent to which their intermolecular forces are overcome, and so the more fluid (less viscous) the liquid; the lower
the temperature, the lesser the intermolecular forces are overcome, and so the less viscous the liquid.

10.E.2.7: Q10.2.5
It is often recommended that you let your car engine run idle to warm up before driving, especially on cold winter days. While the
benefit of prolonged idling is dubious, it is certainly true that a warm engine is more fuel efficient than a cold one. Explain the
reason for this.

10.E.2.8: Q10.2.6
The surface tension and viscosity of water at several different temperatures are given in this table.

Water Surface Tension (mN/m) Viscosity (mPa s)

0 °C 75.6 1.79

20 °C 72.8 1.00

60 °C 66.2 0.47

100 °C 58.9 0.28

a. As temperature increases, what happens to the surface tension of water? Explain why this occurs, in terms of molecular
interactions and the effect of changing temperature.
b. As temperature increases, what happens to the viscosity of water? Explain why this occurs, in terms of molecular interactions
and the effect of changing temperature.

10.E.2.9: S10.2.6
(a) As the water reaches higher temperatures, the increased kinetic energies of its molecules are more effective in overcoming
hydrogen bonding, and so its surface tension decreases. Surface tension and intermolecular forces are directly related. (b) The same
trend in viscosity is seen as in surface tension, and for the same reason.

10.E.2.10: Q10.2.7
At 25 °C, how high will water rise in a glass capillary tube with an inner diameter of 0.63 mm? Refer to Example for the required
information.

Access for free at OpenStax 10.E.5 https://chem.libretexts.org/@go/page/44115


10.E.2.11: Q10.2.8
Water rises in a glass capillary tube to a height of 17 cm. What is the diameter of the capillary tube?

10.E.2.12: S10.2.8
9.5 × 10−5 m

10.E.3: 10.3: Phase Transitions


10.E.3.1: Q10.3.1
Heat is added to boiling water. Explain why the temperature of the boiling water does not change. What does change?

10.E.3.2: Q10.3.2
Heat is added to ice at 0 °C. Explain why the temperature of the ice does not change. What does change?

10.E.3.3: S10.3.2
The heat is absorbed by the ice, providing the energy required to partially overcome intermolecular attractive forces in the solid and
causing a phase transition to liquid water. The solution remains at 0 °C until all the ice is melted. Only the amount of water existing
as ice changes until the ice disappears. Then the temperature of the water can rise.

10.E.3.4: Q10.3.3
What feature characterizes the dynamic equilibrium between a liquid and its vapor in a closed container?

10.E.3.5: Q10.3.4
Identify two common observations indicating some liquids have sufficient vapor pressures to noticeably evaporate?

10.E.3.6: S10.3.4
We can see the amount of liquid in an open container decrease and we can smell the vapor of some liquids.

10.E.3.7: Q10.3.5
Identify two common observations indicating some solids, such as dry ice and mothballs, have vapor pressures sufficient to
sublime?

10.E.3.8: Q10.3.6
What is the relationship between the intermolecular forces in a liquid and its vapor pressure?

10.E.3.9: S10.3.7
The vapor pressure of a liquid decreases as the strength of its intermolecular forces increases.

10.E.3.10: Q10.3.7
What is the relationship between the intermolecular forces in a solid and its melting temperature?

10.E.3.11: Q10.3.8
Why does spilled gasoline evaporate more rapidly on a hot day than on a cold day?

10.E.3.12: S10.3.8
As the temperature increases, the average kinetic energy of the molecules of gasoline increases and so a greater fraction of
molecules have sufficient energy to escape from the liquid than at lower temperatures.

10.E.3.13: Q10.3.9
Carbon tetrachloride, CCl4, was once used as a dry cleaning solvent, but is no longer used because it is carcinogenic. At 57.8 °C,
the vapor pressure of CCl4 is 54.0 kPa, and its enthalpy of vaporization is 33.05 kJ/mol. Use this information to estimate the normal
boiling point for CCl4.

Access for free at OpenStax 10.E.6 https://chem.libretexts.org/@go/page/44115


10.E.3.14: Q10.3.10
When is the boiling point of a liquid equal to its normal boiling point?

10.E.3.15: S10.3.10
When the pressure of gas above the liquid is exactly 1 atm

10.E.3.16: Q10.3.11
How does the boiling of a liquid differ from its evaporation?

10.E.3.17: Q10.3.12
Use the information in Figure to estimate the boiling point of water in Denver when the atmospheric pressure is 83.3 kPa.

10.E.3.18: S10.3.12
approximately 95 °C

10.E.3.19: Q10.3.13
A syringe at a temperature of 20 °C is filled with liquid ether in such a way that there is no space for any vapor. If the temperature
is kept constant and the plunger is withdrawn to create a volume that can be occupied by vapor, what would be the approximate
pressure of the vapor produced?

10.E.3.20: Q10.3.14
Explain the following observations:
a. It takes longer to cook an egg in Ft. Davis, Texas (altitude, 5000 feet above sea level) than it does in Boston (at sea level).
b. Perspiring is a mechanism for cooling the body.

10.E.3.21: S10.3.14
(a) At 5000 feet, the atmospheric pressure is lower than at sea level, and water will therefore boil at a lower temperature. This
lower temperature will cause the physical and chemical changes involved in cooking the egg to proceed more slowly, and a longer
time is required to fully cook the egg. (b) As long as the air surrounding the body contains less water vapor than the maximum that
air can hold at that temperature, perspiration will evaporate, thereby cooling the body by removing the heat of vaporization
required to vaporize the water.

10.E.3.22: Q10.3.15
The enthalpy of vaporization of water is larger than its enthalpy of fusion. Explain why.

10.E.3.23: Q10.3.16
Explain why the molar enthalpies of vaporization of the following substances increase in the order CH4 < C2H6 < C3H8, even
though all three substances experience the same dispersion forces when in the liquid state.

10.E.3.24: S10.3.16
Dispersion forces increase with molecular mass or size. As the number of atoms composing the molecules in this homologous
series increases, so does the extent of intermolecular attraction via dispersion forces and, consequently, the energy required to
overcome these forces and vaporize the liquids.

10.E.3.25: Q10.3.17
Explain why the enthalpies of vaporization of the following substances increase in the order CH4 < NH3 < H2O, even though all
three substances have approximately the same molar mass.

10.E.3.26: Q10.3.18
The enthalpy of vaporization of CO2(l) is 9.8 kJ/mol. Would you expect the enthalpy of vaporization of CS2(l) to be 28 kJ/mol, 9.8
kJ/mol, or −8.4 kJ/mol? Discuss the plausibility of each of these answers.

Access for free at OpenStax 10.E.7 https://chem.libretexts.org/@go/page/44115


10.E.3.27: S10.3.18
The boiling point of CS2 is higher than that of CO2 partially because of the higher molecular weight of CS2; consequently, the
attractive forces are stronger in CS2. It would be expected, therefore, that the heat of vaporization would be greater than that of 9.8
kJ/mol for CO2. A value of 28 kJ/mol would seem reasonable. A value of −8.4 kJ/mol would indicate a release of energy upon
vaporization, which is clearly implausible.

10.E.3.28: Q10.3.19
The hydrogen fluoride molecule, HF, is more polar than a water molecule, H2O (for example, has a greater dipole moment), yet the
molar enthalpy of vaporization for liquid hydrogen fluoride is lesser than that for water. Explain.

10.E.3.29: Q10.3.20
Ethyl chloride (boiling point, 13 °C) is used as a local anesthetic. When the liquid is sprayed on the skin, it cools the skin enough to
freeze and numb it. Explain the cooling effect of liquid ethyl chloride.

10.E.3.30: S10.3.20
The thermal energy (heat) needed to evaporate the liquid is removed from the skin.

10.E.3.31: Q10.3.21
Which contains the compounds listed correctly in order of increasing boiling points?
a. N2 < CS2 < H2O < KCl
b. H2O < N2 < CS2 < KCl
c. N2 < KCl < CS2 < H2O
d. CS2 < N2 < KCl < H2O
e. KCl < H2O < CS2 < N2

10.E.3.32: Q10.3.22
How much heat is required to convert 422 g of liquid H2O at 23.5 °C into steam at 150 °C?

10.E.3.33: S10.3.22
1130 kJ

10.E.3.34: Q10.3.22
Evaporation of sweat requires energy and thus take excess heat away from the body. Some of the water that you drink may
eventually be converted into sweat and evaporate. If you drink a 20-ounce bottle of water that had been in the refrigerator at 3.8 °C,
how much heat is needed to convert all of that water into sweat and then to vapor?? (Note: Your body temperature is 36.6 °C. For
the purpose of solving this problem, assume that the thermal properties of sweat are the same as for water.)

10.E.3.35: Q10.3.24
Titanium tetrachloride, TiCl4, has a melting point of −23.2 °C and has a ΔH fusion = 9.37 kJ/mol.
a. How much energy is required to melt 263.1 g TiCl4?
b. For TiCl4, which will likely have the larger magnitude: ΔH fusion or ΔH vaporization? Explain your reasoning.

10.E.3.36: S10.3.24
(a) 13.0 kJ; (b) It is likely that the heat of vaporization will have a larger magnitude since in the case of vaporization the
intermolecular interactions have to be completely overcome, while melting weakens or destroys only some of them.

10.E.4: 10.4: Phase Diagrams


10.E.4.1: Q10.4.1
From the phase diagram for water, determine the state of water at:
a. 35 °C and 85 kPa
b. −15 °C and 40 kPa

Access for free at OpenStax 10.E.8 https://chem.libretexts.org/@go/page/44115


c. −15 °C and 0.1 kPa
d. 75 °C and 3 kPa
e. 40 °C and 0.1 kPa
f. 60 °C and 50 kPa

10.E.4.2: Q10.4.2
What phase changes will take place when water is subjected to varying pressure at a constant temperature of 0.005 °C? At 40 °C?
At −40 °C?

10.E.4.3: S10.4.2
At low pressures and 0.005 °C, the water is a gas. As the pressure increases to 4.6 torr, the water becomes a solid; as the pressure
increases still more, it becomes a liquid. At 40 °C, water at low pressure is a vapor; at pressures higher than about 75 torr, it
converts into a liquid. At −40 °C, water goes from a gas to a solid as the pressure increases above very low values.

10.E.4.4: Q10.4.3
Pressure cookers allow food to cook faster because the higher pressure inside the pressure cooker increases the boiling temperature
of water. A particular pressure cooker has a safety valve that is set to vent steam if the pressure exceeds 3.4 atm. What is the
approximate maximum temperature that can be reached inside this pressure cooker? Explain your reasoning.

10.E.4.5: Q10.4.4
From the phase diagram for carbon dioxide, determine the state of CO2 at:
a. 20 °C and 1000 kPa
b. 10 °C and 2000 kPa
c. 10 °C and 100 kPa
d. −40 °C and 500 kPa
e. −80 °C and 1500 kPa
f. −80 °C and 10 kPa

The pressure and temperature axes on this phase diagram of carbon dioxide are not drawn to constant scale in order to illustrate
several important properties.

Access for free at OpenStax 10.E.9 https://chem.libretexts.org/@go/page/44115


10.E.4.6: S10.4.4
(a) liquid; (b) solid; (c) gas; (d) gas; (e) gas; (f) gas

10.E.4.7: Q10.4.5
Determine the phase changes that carbon dioxide undergoes as the pressure changes if the temperature is held at −50 °C? If the
temperature is held at −40 °C? At 20 °C?

The pressure and temperature axes on this phase diagram of carbon dioxide are not drawn to constant scale in order to illustrate
several important properties.

10.E.4.8: Q10.4.6
Consider a cylinder containing a mixture of liquid carbon dioxide in equilibrium with gaseous carbon dioxide at an initial pressure
of 65 atm and a temperature of 20 °C. Sketch a plot depicting the change in the cylinder pressure with time as gaseous carbon
dioxide is released at constant temperature.

10.E.4.9: S10.4.6

10.E.4.10: Q10.4.7
Dry ice, CO2(s), does not melt at atmospheric pressure. It sublimes at a temperature of −78 °C. What is the lowest pressure at
which CO2(s) will melt to give CO2(l)? At approximately what temperature will this occur? (See Figure for the phase diagram.)

Access for free at OpenStax 10.E.10 https://chem.libretexts.org/@go/page/44115


10.E.4.11: S10.4.7
Dry ice, CO2(s), will melt to give CO2(l) at 5.11 atm at −56.6 °C, the triple point of carbon dioxide.

10.E.4.12: Q10.4.8
If a severe storm results in the loss of electricity, it may be necessary to use a clothesline to dry laundry. In many parts of the
country in the dead of winter, the clothes will quickly freeze when they are hung on the line. If it does not snow, will they dry
anyway? Explain your answer.

10.E.4.13: S10.4.8
Yes, ice will sublime, although it may take it several days. Ice has a small vapor pressure, and some ice molecules form gas and
escape from the ice crystals. As time passes, more and more solid converts to gas until eventually the clothes are dry.

10.E.4.14: Q10.4.9
Is it possible to liquefy nitrogen at room temperature (about 25 °C)? Is it possible to liquefy sulfur dioxide at room temperature?
Explain your answers.

10.E.4.15: Q10.4.10
Elemental carbon has one gas phase, one liquid phase, and three different solid phases, as shown in the phase diagram:

a. On the phase diagram, label the gas and liquid regions.


b. Graphite is the most stable phase of carbon at normal conditions. On the phase diagram, label the graphite phase.
c. If graphite at normal conditions is heated to 2500 K while the pressure is increased to 1010 Pa, it is converted into diamond.
Label the diamond phase.
d. Circle each triple point on the phase diagram.
e. In what phase does carbon exist at 5000 K and 108 Pa?
f. If the temperature of a sample of carbon increases from 3000 K to 5000 K at a constant pressure of 106 Pa, which phase
transition occurs, if any?

10.E.5: 10.5: The Solid State of Matter


10.E.5.1: Q10.5.1
What types of liquids typically form amorphous solids?

10.E.5.2: S10.5.1
Amorphous solids lack an ordered internal structure. Liquid materials that contain large, cumbersome molecules that cannot move
readily into ordered positions generally form such solids.

Access for free at OpenStax 10.E.11 https://chem.libretexts.org/@go/page/44115


10.E.5.3: Q10.5.2
At very low temperatures oxygen, O2, freezes and forms a crystalline solid. Which best describes these crystals?
a. ionic
b. covalent network
c. metallic
d. amorphous
e. molecular crystals

10.E.5.4: S10.5.3
(e) molecular crystals

10.E.5.5: Q10.5.4
As it cools, olive oil slowly solidifies and forms a solid over a range of temperatures. Which best describes the solid?
a. ionic
b. covalent network
c. metallic
d. amorphous
e. molecular crystals

10.E.5.6: S10.5.4
(d) amorphous

10.E.5.7: Q10.5.5
Explain why ice, which is a crystalline solid, has a melting temperature of 0 °C, whereas butter, which is an amorphous solid,
softens over a range of temperatures.

10.E.5.8: S10.5.6
Ice has a crystalline structure stabilized by hydrogen bonding. These intermolecular forces are of comparable strength and thus
require the same amount of energy to overcome. As a result, ice melts at a single temperature and not over a range of temperatures.
The various, very large molecules that compose butter experience varied van der Waals attractions of various strengths that are
overcome at various temperatures, and so the melting process occurs over a wide temperature range.

10.E.5.9: Q10.5.7
Identify the type of crystalline solid (metallic, network covalent, ionic, or molecular) formed by each of the following substances:
a. SiO2
b. KCl
c. Cu
d. CO2
e. C (diamond)
f. BaSO4
g. NH3
h. NH4F
i. C2H5OH

10.E.5.10: S10.5.7
(a) SiO2, covalent network; (b) KCl, ionic; (c) Cu, metallic; (d) CO, molecular; (e) C (diamond), covalent network; (f) BaSO4,
ionic; (g) NH3, molecular; (h) NH4F, ionic; (i) C2H5OH, molecular

10.E.5.11: Q10.5.8
Identify the type of crystalline solid (metallic, network covalent, ionic, or molecular) formed by each of the following substances:
a. CaCl2
b. SiC

Access for free at OpenStax 10.E.12 https://chem.libretexts.org/@go/page/44115


c. N2
d. Fe
e. C (graphite)
f. CH3CH2CH2CH3
g. HCl
h. NH4NO3
i. K3PO4

10.E.5.12: S10.5.8
(a) CaCl2, ionic; (b) SiC, covalent network; (c) N2, molecular; (d) Fe, metallic; (e) C (graphite), covalent network; (f)
CH3CH2CH2CH3, molecular; (g) HCl, molecular; (h) NH4NO3, ionic; (i) K3PO4, ionic

10.E.5.13: Q10.5.9
Classify each substance in the table as either a metallic, ionic, molecular, or covalent network solid:

Substance Appearance Melting Point Electrical Conductivity Solubility in Water

X lustrous, malleable 1500 °C high insoluble

Y soft, yellow 113 °C none insoluble

Z hard, white 800 °C only if melted/dissolved soluble

10.E.5.14: S10.5.9
X = metallic; Y = covalent network; Z = ionic

10.E.5.15: Q10.5.10
Classify each substance in the table as either a metallic, ionic, molecular, or covalent network solid:

Substance Appearance Melting Point Electrical Conductivity Solubility in Water

X brittle, white 800 °C only if melted/dissolved soluble

Y shiny, malleable 1100 °C high insoluble

Z hard, colorless 3550 °C none insoluble

10.E.5.16: S10.5.10
X = ionic; Y = metallic; Z = covalent network

10.E.5.17: Q10.5.11
Identify the following substances as ionic, metallic, covalent network, or molecular solids:
Substance A is malleable, ductile, conducts electricity well, and has a melting point of 1135 °C. Substance B is brittle, does not
conduct electricity as a solid but does when molten, and has a melting point of 2072 °C. Substance C is very hard, does not conduct
electricity, and has a melting point of 3440 °C. Substance D is soft, does not conduct electricity, and has a melting point of 185 °C.

10.E.5.18: S10.5.11
A = metallic; B = ionic; C = covalent network; D = molecular

10.E.5.19: Q10.5.12
Substance A is shiny, conducts electricity well, and melts at 975 °C. Substance A is likely a(n):
a. ionic solid
b. metallic solid
c. molecular solid
d. covalent network solid

Access for free at OpenStax 10.E.13 https://chem.libretexts.org/@go/page/44115


10.E.5.20: S10.5.12
(b) metallic solid

10.E.5.21: Q10.5.13
Substance B is hard, does not conduct electricity, and melts at 1200 °C. Substance B is likely a(n):
a. ionic solid
b. metallic solid
c. molecular solid
d. covalent network solid

10.E.5.22: S10.5.13
(d) covalent network solid

10.E.6: 10.6: Lattice Structures


10.E.6.1: Q10.6.1
Describe the crystal structure of iron, which crystallizes with two equivalent metal atoms in a cubic unit cell.

10.E.6.2: S10.6.1
The structure of this low-temperature form of iron (below 910 °C) is body-centered cubic. There is one-eighth atom at each of the
eight corners of the cube and one atom in the center of the cube.

10.E.6.3: Q10.6.2
Describe the crystal structure of Pt, which crystallizes with four equivalent metal atoms in a cubic unit cell.

10.E.6.4: Q10.6.3
What is the coordination number of a chromium atom in the body-centered cubic structure of chromium?

10.E.6.5: S10.6.3
eight

10.E.6.6: Q10.6.4
What is the coordination number of an aluminum atom in the face-centered cubic structure of aluminum?

10.E.6.7: Q10.6.5
Cobalt metal crystallizes in a hexagonal closest packed structure. What is the coordination number of a cobalt atom?

10.E.6.8: S10.6.5
12

10.E.6.9: Q10.6.6
Nickel metal crystallizes in a cubic closest packed structure. What is the coordination number of a nickel atom?

10.E.6.10: Q10.6.7
Tungsten crystallizes in a body-centered cubic unit cell with an edge length of 3.165 Å.
a. What is the atomic radius of tungsten in this structure?
b. Calculate the density of tungsten.

10.E.6.11: S10.6.7
(a) 1.370 Å; (b) 19.26 g/cm

Access for free at OpenStax 10.E.14 https://chem.libretexts.org/@go/page/44115


10.E.6.12: Q10.6.8
Platinum (atomic radius = 1.38 Å) crystallizes in a cubic closely packed structure. Calculate the edge length of the face-centered
cubic unit cell and the density of platinum.

10.E.6.13: Q10.6.9
Barium crystallizes in a body-centered cubic unit cell with an edge length of 5.025 Å
a. What is the atomic radius of barium in this structure?
b. Calculate the density of barium.

10.E.6.14: S10.6.9
(a) 2.176 Å; (b) 3.595 g/cm3

10.E.6.15: Q10.6.10
Aluminum (atomic radius = 1.43 Å) crystallizes in a cubic closely packed structure. Calculate the edge length of the face-centered
cubic unit cell and the density of aluminum.

10.E.6.16: Q10.6.11
The density of aluminum is 2.7 g/cm3; that of silicon is 2.3 g/cm3. Explain why Si has the lower density even though it has heavier
atoms.

10.E.6.17: S10.6.11
The crystal structure of Si shows that it is less tightly packed (coordination number 4) in the solid than Al (coordination number
12).

10.E.6.18: Q10.6.12
The free space in a metal may be found by subtracting the volume of the atoms in a unit cell from the volume of the cell. Calculate
the percentage of free space in each of the three cubic lattices if all atoms in each are of equal size and touch their nearest
neighbors. Which of these structures represents the most efficient packing? That is, which packs with the least amount of unused
space?

10.E.6.19: Q10.6.13
Cadmium sulfide, sometimes used as a yellow pigment by artists, crystallizes with cadmium, occupying one-half of the tetrahedral
holes in a closest packed array of sulfide ions. What is the formula of cadmium sulfide? Explain your answer.

10.E.6.20: S10.6.13
In a closest-packed array, two tetrahedral holes exist for each anion. If only half the tetrahedral holes are occupied, the numbers of
anions and cations are equal. The formula for cadmium sulfide is CdS.

10.E.6.21: Q10.6.14
A compound of cadmium, tin, and phosphorus is used in the fabrication of some semiconductors. It crystallizes with cadmium
occupying one-fourth of the tetrahedral holes and tin occupying one-fourth of the tetrahedral holes in a closest packed array of
phosphide ions. What is the formula of the compound? Explain your answer.

10.E.6.22: Q10.6.15
What is the formula of the magnetic oxide of cobalt, used in recording tapes, that crystallizes with cobalt atoms occupying one-
eighth of the tetrahedral holes and one-half of the octahedral holes in a closely packed array of oxide ions?

10.E.6.23: S10.6.15
Co3O4

10.E.6.24: Q10.6.16
A compound containing zinc, aluminum, and sulfur crystallizes with a closest-packed array of sulfide ions. Zinc ions are found in
one-eighth of the tetrahedral holes and aluminum ions in one-half of the octahedral holes. What is the empirical formula of the

Access for free at OpenStax 10.E.15 https://chem.libretexts.org/@go/page/44115


compound?

10.E.6.25: Q10.6.17
A compound of thallium and iodine crystallizes in a simple cubic array of iodide ions with thallium ions in all of the cubic holes.
What is the formula of this iodide? Explain your answer.

10.E.6.26: S10.6.17
In a simple cubic array, only one cubic hole can be occupied be a cation for each anion in the array. The ratio of thallium to iodide
must be 1:1; therefore, the formula for thallium is TlI.

10.E.6.27: Q10.6.18
Which of the following elements reacts with sulfur to form a solid in which the sulfur atoms form a closest-packed array with all of
the octahedral holes occupied: Li, Na, Be, Ca, or Al?

10.E.6.28: Q10.6.19
What is the percent by mass of titanium in rutile, a mineral that contains titanium and oxygen, if structure can be described as a
closest packed array of oxide ions with titanium ions in one-half of the octahedral holes? What is the oxidation number of titanium?

10.E.6.29: S10.6.19
59.95%; The oxidation number of titanium is +4.

10.E.6.30: Q10.6.20
Explain why the chemically similar alkali metal chlorides NaCl and CsCl have different structures, whereas the chemically
different NaCl and MnS have the same structure.

10.E.6.31: Q10.6.21
As minerals were formed from the molten magma, different ions occupied the same cites in the crystals. Lithium often occurs along
with magnesium in minerals despite the difference in the charge on their ions. Suggest an explanation.

10.E.6.32: S10.6.21
Both ions are close in size: Mg, 0.65; Li, 0.60. This similarity allows the two to interchange rather easily. The difference in charge
is generally compensated by the switch of Si4+ for Al3+.

10.E.6.33: Q10.6.22
Rubidium iodide crystallizes with a cubic unit cell that contains iodide ions at the corners and a rubidium ion in the center. What is
the formula of the compound?

10.E.6.34: Q10.6.23
One of the various manganese oxides crystallizes with a cubic unit cell that contains manganese ions at the corners and in the
center. Oxide ions are located at the center of each edge of the unit cell. What is the formula of the compound?

10.E.6.35: S10.6.23
Mn2O3

10.E.6.36: Q10.6.24
NaH crystallizes with the same crystal structure as NaCl. The edge length of the cubic unit cell of NaH is 4.880 Å.
a. Calculate the ionic radius of H−. (The ionic radius of Li+ is 0.0.95 Å.)
b. Calculate the density of NaH.

10.E.6.37: Q10.6.25
Thallium(I) iodide crystallizes with the same structure as CsCl. The edge length of the unit cell of TlI is 4.20 Å. Calculate the ionic
radius of TI+. (The ionic radius of I− is 2.16 Å.)

Access for free at OpenStax 10.E.16 https://chem.libretexts.org/@go/page/44115


10.E.6.38: S10.6.25
1.48 Å

10.E.6.39: Q10.6.26
A cubic unit cell contains manganese ions at the corners and fluoride ions at the center of each edge.
a. What is the empirical formula of this compound? Explain your answer.
b. What is the coordination number of the Mn3+ ion?
c. Calculate the edge length of the unit cell if the radius of a Mn3+ ion is 0.65 A.
d. Calculate the density of the compound.

10.E.6.40: Q10.6.27
What is the spacing between crystal planes that diffract X-rays with a wavelength of 1.541 nm at an angle θ of 15.55° (first order
reflection)?

10.E.6.41: S10.6.27
2.874 Å

10.E.6.42: Q10.6.28
A diffractometer using X-rays with a wavelength of 0.2287 nm produced first-order diffraction peak for a crystal angle θ = 16.21°.
Determine the spacing between the diffracting planes in this crystal.

10.E.6.43: Q10.6.29
A metal with spacing between planes equal to 0.4164 nm diffracts X-rays with a wavelength of 0.2879 nm. What is the diffraction
angle for the first order diffraction peak?

10.E.6.44: S10.6.29
20.2°

10.E.6.45: Q10.6.30
Gold crystallizes in a face-centered cubic unit cell. The second-order reflection (n = 2) of X-rays for the planes that make up the
tops and bottoms of the unit cells is at θ = 22.20°. The wavelength of the X-rays is 1.54 Å. What is the density of metallic gold?

10.E.6.46: Q10.6.31
When an electron in an excited molybdenum atom falls from the L to the K shell, an X-ray is emitted. These X-rays are diffracted
at an angle of 7.75° by planes with a separation of 2.64 Å. What is the difference in energy between the K shell and the L shell in
molybdenum assuming a first-order diffraction?

10.E.6.47: S10.6.31
1.74 × 104 eV

This page titled 10.E: Liquids and Solids (Exercises) is shared under a CC BY 4.0 license and was authored, remixed, and/or curated by OpenStax
via source content that was edited to the style and standards of the LibreTexts platform; a detailed edit history is available upon request.

Access for free at OpenStax 10.E.17 https://chem.libretexts.org/@go/page/44115


CHAPTER OVERVIEW
11: Solutions and Colloids

A general chemistry Libretexts Textbook remixed and remastered from


OpenStax's textbook:
General Chemistry
In this chapter, we will consider the nature of solutions, and examine factors that determine whether a solution will form and what
properties it may have. In addition, we will discuss colloids—systems that resemble solutions but consist of dispersions of particles
somewhat larger than ordinary molecules or ions.
11.0: Prelude to Solutions and Colloids
11.1: The Dissolution Process
11.2: Electrolytes
11.3: Solubility
11.4: Colligative Properties
11.5: Colloids
11.E: Solutions and Colloids (Exercises)

Thumbnail: Nile red solution. (CC BY-SA 3.0; Armin Kübelbeck).

This page titled 11: Solutions and Colloids is shared under a CC BY 4.0 license and was authored, remixed, and/or curated by OpenStax via
source content that was edited to the style and standards of the LibreTexts platform; a detailed edit history is available upon request.

1
11.0: Prelude to Solutions and Colloids
Coral reefs are home to about 25% of all marine species. They are being threatened by climate change, oceanic acidification, and
water pollution, all of which change the composition of the solution we know as seawater. Dissolved oxygen in seawater is critical
for sea creatures, but as the oceans warm, oxygen becomes less soluble. As the concentration of carbon dioxide in the atmosphere
increases, the concentration of carbon dioxide in the oceans increases, contributing to oceanic acidification. Coral reefs are
particularly sensitive to the acidification of the ocean, since the exoskeletons of the coral polyps are soluble in acidic solutions.
Humans contribute to the changing of seawater composition by allowing agricultural runoff and other forms of pollution to affect
our oceans.

Figure 11.0.1 : Coral reefs, such as this one at the Palmyra Atoll National Wildlife Refuge, are vital to the ecosystem of earth’s
oceans but are threatened by climate change and dissolved pollution. Marine life depends on the specific chemical composition of
the complex mixture we know as seawater. (credit: modification of work by “USFWS – Pacific Region”/Wikimedia Commons)
Solutions are crucial to the processes that sustain life and to many other processes involving chemical reactions. In this chapter, we
will consider the nature of solutions, and examine factors that determine whether a solution will form and what properties it may
have. In addition, we will discuss colloids—systems that resemble solutions but consist of dispersions of particles somewhat larger
than ordinary molecules or ions.

This page titled 11.0: Prelude to Solutions and Colloids is shared under a CC BY 4.0 license and was authored, remixed, and/or curated by
OpenStax via source content that was edited to the style and standards of the LibreTexts platform; a detailed edit history is available upon request.

Access for free at OpenStax 11.0.1 https://chem.libretexts.org/@go/page/38248


11.1: The Dissolution Process
 Learning Objectives
Describe the basic properties of solutions and how they form.
Predict whether a given mixture will yield a solution based on molecular properties of its components.
Explain why some solutions either produce or absorb heat when they form.

An earlier chapter of this text introduced solutions, defined as homogeneous mixtures of two or more substances. Often, one
component of a solution is present at a significantly greater concentration, in which case it is called the solvent. The other
components of the solution present in relatively lesser concentrations are called solutes. Sugar is a covalent solid composed of
sucrose molecules, C H O . When this compound dissolves in water, its molecules become uniformly distributed among the
12 22 11

molecules of water:
C H O (s) ⟶ C H O (aq) (11.1.1)
12 22 11 12 22 11

The subscript “aq” in the equation signifies that the sucrose molecules are solutes and are therefore individually dispersed
throughout the aqueous solution (water is the solvent). Although sucrose molecules are heavier than water molecules, they remain
dispersed throughout the solution; gravity does not cause them to “settle out” over time.
Potassium dichromate, K Cr O , is an ionic compound composed of colorless potassium ions, K , and orange dichromate ions,
2 2 7
+

Cr O
2 7
. When a small amount of solid potassium dichromate is added to water, the compound dissolves and dissociates to yield
2 −

potassium ions and dichromate ions uniformly distributed throughout the mixture (Figure 11.1.1), as indicated in this equation:
+ 2 −
K Cr O (s) ⟶ 2 K (aq) + Cr O7 (aq) (11.1.2)
2 2 7 2

As with the mixture of sugar and water, this mixture is also an aqueous solution. Its solutes, potassium and dichromate ions, remain
individually dispersed among the solvent (water) molecules.

Figure 11.1.1 : When potassium dichromate (K Cr O ) is mixed with water, it forms a homogeneous orange solution. (credit:
2 2 7

modification of work by Mark Ott).


The first photo shows a small mound of an orange crystalline solid. There is a right-facing arrow. The second photo shows a
translucent, colorless liquid in a clear container. There is a right-facing arrow. The third photo shows a translucent orange liquid in
a clear, covered container.
Water is used so often as a solvent that the word solution has come to imply an aqueous solution to many people. However, almost
any gas, liquid, or solid can act as a solvent. Many alloys are solid solutions of one metal dissolved in another; for example, US
five-cent coins contain nickel dissolved in copper. Air is a gaseous solution, a homogeneous mixture of nitrogen, oxygen, and
several other gases. Oxygen (a gas), alcohol (a liquid), and sugar (a solid) all dissolve in water (a liquid) to form liquid solutions.
Table 11.1.1 gives examples of several different solutions and the phases of the solutes and solvents.
Table 11.1.1 : Different Types of Solutions
Solution Solute Solvent

air O2(g) N2(g)

soft drinks CO2(g) H2O(l)

hydrogen in palladium H2(g) Pd(s)

rubbing alcohol H2O(l) C3H8O(l) (2-propanol)

saltwater NaCl(s) H2O(l)

Access for free at OpenStax 11.1.1 https://chem.libretexts.org/@go/page/38249


Solution Solute Solvent

brass Zn(s) Cu(s)

Solutions exhibit these defining traits:


They are homogeneous; that is, after a solution is mixed, it has the same composition at all points throughout (its composition is
uniform).
The physical state of a solution—solid, liquid, or gas—is typically the same as that of the solvent, as demonstrated by the
examples in Table 11.1.1.
The components of a solution are dispersed on a molecular scale; that is, they consist of a mixture of separated molecules,
atoms, and/or ions.
The dissolved solute in a solution will not settle out or separate from the solvent.
The composition of a solution, or the concentrations of its components, can be varied continuously, within limits.

11.1.1: The Formation of Solutions


The formation of a solution is an example of a spontaneous process, a process that occurs under specified conditions without the
requirement of energy from some external source. Sometimes we stir a mixture to speed up the dissolution process, but this is not
necessary; a homogeneous solution would form if we waited long enough. The topic of spontaneity is critically important to the
study of chemical thermodynamics and is treated more thoroughly in a later chapter of this text. For purposes of this chapter’s
discussion, it will suffice to consider two criteria that favor, but do not guarantee, the spontaneous formation of a solution:
1. a decrease in the internal energy of the system (an exothermic change, as discussed in the previous chapter on thermochemistry)
2. an increase in the disorder in the system (which indicates an increase in the entropy of the system, as you will learn about in the
later chapter on thermodynamics)
In the process of dissolution, an internal energy change often, but not always, occurs as heat is absorbed or evolved. An increase in
disorder always results when a solution forms.
When the strengths of the intermolecular forces of attraction between solute and solvent species in a solution are no different than
those present in the separated components, the solution is formed with no accompanying energy change. Such a solution is called
an ideal solution. A mixture of ideal gases (or gases such as helium and argon, which closely approach ideal behavior) is an
example of an ideal solution, since the entities comprising these gases experience no significant intermolecular attractions.
When containers of helium and argon are connected, the gases spontaneously mix due to diffusion and form a solution (Figure
11.1.2). The formation of this solution clearly involves an increase in disorder, since the helium and argon atoms occupy a volume

twice as large as that which each occupied before mixing.

Figure 11.1.2 : Samples of helium and argon spontaneously mix to give a solution in which the disorder of the atoms of the two
gases is increased.
Two figures are shown. The first contains two spherical containers joined by a closed stopcock. The container to the left is labeled
H e. It holds about thirty evenly dispersed, small, light blue spheres. The container on the right is labeled A r and contains about
thirty slightly larger blue-green spheres. The second, similar figure has an open stopcock between the two spherical containers. The
light blue and green spheres are evenly dispersed and present in both containers.
Ideal solutions may also form when structurally similar liquids are mixed. For example, mixtures of the alcohols methanol
(CH3OH) and ethanol (C2H5OH) form ideal solutions, as do mixtures of the hydrocarbons pentane, C H , and hexane, C H .
5 12 6 14

Placing methanol and ethanol, or pentane and hexane, in the bulbs shown in Figure 11.1.2 will result in the same diffusion and
subsequent mixing of these liquids as is observed for the He and Ar gases (although at a much slower rate), yielding solutions with
no significant change in energy. Unlike a mixture of gases, however, the components of these liquid-liquid solutions do, indeed,

Access for free at OpenStax 11.1.2 https://chem.libretexts.org/@go/page/38249


experience intermolecular attractive forces. But since the molecules of the two substances being mixed are structurally very similar,
the intermolecular attractive forces between like and unlike molecules are essentially the same, and the dissolution process,
therefore, does not entail any appreciable increase or decrease in energy. These examples illustrate how diffusion alone can provide
the driving force required to cause the spontaneous formation of a solution. In some cases, however, the relative magnitudes of
intermolecular forces of attraction between solute and solvent species may prevent dissolution.

Figure 11.1.3 : This schematic representation of dissolution shows a stepwise process involving the endothermic separation of
solute and solvent species (Steps 1 and 2) and exothermic solvation (Step 3).
The top, central region of the figure shows solute particles as seven blue spheres and solvent particles as 16 red spheres in separate,
labeled boxes. The particles in these boxes are touching. An arrow labeled “Step 1” points left of the solute box, and shows the blue
spheres no longer touching in another box labeled “expanded solute.” An arrow labeled “Step 2” points right from the solvent box
and shows the red spheres no longer touching in another box labeled “expanded solvent.” Arrows proceed from the bottom of the
expanded solute and expanded solvent boxes and join at the bottom of the figure where a step 3 label is shown. The joined arrows
point to a box just above in which the red and blue spheres are mixed together and touching. The solute and solvent boxes are
joined by another arrow labeled “direct formation of solution” which points downward at the center of the figure. This arrow also
points to the box containing mixed red and blue spheres near the bottom of the figure.
Three types of intermolecular attractive forces are relevant to the dissolution process: solute-solute, solvent-solvent, and solute-
solvent. As illustrated in Figure 11.1.3, the formation of a solution may be viewed as a stepwise process in which energy is
consumed to overcome solute-solute and solvent-solvent attractions (endothermic processes) and released when solute-solvent
attractions are established (an exothermic process referred to as solvation). The relative magnitudes of the energy changes
associated with these stepwise processes determine whether the dissolution process overall will release or absorb energy. In some
cases, solutions do not form because the energy required to separate solute and solvent species is so much greater than the energy
released by solvation.

Figure 11.1.4 : A mixture of nonpolar cooking oil and polar water does not yield a solution. (credit: Gautam Dogra).
For example, cooking oils and water will not mix to any appreciable extent to yield solutions (Figure 11.1.4). Hydrogen bonding is
the dominant intermolecular attractive force present in liquid water; the nonpolar hydrocarbon molecules of cooking oils are not

Access for free at OpenStax 11.1.3 https://chem.libretexts.org/@go/page/38249


capable of hydrogen bonding, instead being held together by dispersion forces. Forming an oil-water solution would require
overcoming the very strong hydrogen bonding in water, as well as the significantly strong dispersion forces between the relatively
large oil molecules. And, since the polar water molecules and nonpolar oil molecules would not experience very strong
intermolecular attraction, very little energy would be released by solvation.
On the other hand, a mixture of ethanol and water will mix in any proportions to yield a solution. In this case, both substances are
capable of hydrogen bonding, and so the solvation process is sufficiently exothermic to compensate for the endothermic separations
of solute and solvent molecules.

Figure 11.1.5 : An instant cold pack gets cold when certain salts, such as ammonium nitrate, dissolve in water—an endothermic
process.
This figure shows a single use instant cold pack with labels indicating an inner bag of water surrounded by white particulate
ammonium nitrate.
As noted at the beginning of this module, spontaneous solution formation is favored, but not guaranteed, by exothermic dissolution
processes. While many soluble compounds do, indeed, dissolve with the release of heat, some dissolve endothermically.
Ammonium nitrate (NH4NO3) is one such example and is used to make instant cold packs for treating injuries like the one pictured
in Figure 11.1.5. A thin-walled plastic bag of water is sealed inside a larger bag with solid NH4NO3. When the smaller bag is
broken, a solution of NH4NO3 forms, absorbing heat from the surroundings (the injured area to which the pack is applied) and
providing a cold compress that decreases swelling. Endothermic dissolutions such as this one require a greater energy input to
separate the solute species than is recovered when the solutes are solvated, but they are spontaneous nonetheless due to the increase
in disorder that accompanies formation of the solution.

Comparing Temperature Change from …

Video 11.1.1 : Watch this brief video illustrating endothermic and exothermic dissolution processes.

Access for free at OpenStax 11.1.4 https://chem.libretexts.org/@go/page/38249


Summary
A solution forms when two or more substances combine physically to yield a mixture that is homogeneous at the molecular level.
The solvent is the most concentrated component and determines the physical state of the solution. The solutes are the other
components typically present at concentrations less than that of the solvent. Solutions may form endothermically or exothermically,
depending upon the relative magnitudes of solute and solvent intermolecular attractive forces. Ideal solutions form with no
appreciable change in energy.

Footnotes
1. If bubbles of gas are observed within the liquid, the mixture is not homogeneous and, thus, not a solution.

Glossary
alloy
solid mixture of a metallic element and one or more additional elements

ideal solution
solution that forms with no accompanying energy change

solvation
exothermic process in which intermolecular attractive forces between the solute and solvent in a solution are established

spontaneous process
physical or chemical change that occurs without the addition of energy from an external source

This page titled 11.1: The Dissolution Process is shared under a CC BY 4.0 license and was authored, remixed, and/or curated by OpenStax via
source content that was edited to the style and standards of the LibreTexts platform; a detailed edit history is available upon request.

Access for free at OpenStax 11.1.5 https://chem.libretexts.org/@go/page/38249


11.2: Electrolytes
 Learning Objectives
Define and give examples of electrolytes
Distinguish between the physical and chemical changes that accompany dissolution of ionic and covalent electrolytes
Relate electrolyte strength to solute-solvent attractive forces

When some substances are dissolved in water, they undergo either a physical or a chemical change that yields ions in solution.
These substances constitute an important class of compounds called electrolytes. Substances that do not yield ions when dissolved
are called nonelectrolytes. If the physical or chemical process that generates the ions is essentially 100% efficient (all of the
dissolved compound yields ions), then the substance is known as a strong electrolyte. If only a relatively small fraction of the
dissolved substance undergoes the ion-producing process, it is called a weak electrolyte.
Substances may be identified as strong, weak, or nonelectrolytes by measuring the electrical conductance of an aqueous solution
containing the substance. To conduct electricity, a substance must contain freely mobile, charged species. Most familiar is the
conduction of electricity through metallic wires, in which case the mobile, charged entities are electrons. Solutions may also
conduct electricity if they contain dissolved ions, with conductivity increasing as ion concentration increases. Applying a voltage to
electrodes immersed in a solution permits assessment of the relative concentration of dissolved ions, either quantitatively, by
measuring the electrical current flow, or qualitatively, by observing the brightness of a light bulb included in the circuit (Figure
11.2.1).

Figure 11.2.1 : Solutions of nonelectrolytes such as ethanol do not contain dissolved ions and cannot conduct electricity. Solutions
of electrolytes contain ions that permit the passage of electricity. The conductivity of an electrolyte solution is related to the
strength of the electrolyte.
This diagram shows three separate beakers. Each has a wire plugged into a wall outlet. In each case, the wire leads from the wall to
the beaker and is split resulting in two ends. One end leads to a light bulb and continues on to a rectangle labeled with a plus sign.
The other end leads to a rectangle labeled with a minus sign. The rectangles are in a solution. In the first beaker, labeled “Ethanol
No Conductivity,” four pairs of linked small green spheres suspended in the solution between the rectangles. In the second beaker,
labeled “K C l Strong Conductivity,” six individual green spheres, three labeled plus and three labeled minus are suspended in the
solution. Each of the six spheres has an arrow extending from it pointing to the rectangle labeled with the opposite sign. In the third
beaker, labeled “Acetic acid solution Weak conductivity,” two pairs of joined green spheres and two individual spheres, one labeled
plus and one labeled minus are shown suspended between the two rectangles. The plus labeled sphere has an arrow pointing to the
rectangle labeled minus and the minus labeled sphere has an arrow pointing to the rectangle labeled plus.

11.2.1: Ionic Electrolytes


Water and other polar molecules are attracted to ions, as shown in Figure 11.2.2. The electrostatic attraction between an ion and a
molecule with a dipole is called an ion-dipole attraction. These attractions play an important role in the dissolution of ionic
compounds in water.

Access for free at OpenStax 11.2.1 https://chem.libretexts.org/@go/page/38250


Figure 11.2.2 : As potassium chloride (KCl) dissolves in water, the ions are hydrated. The polar water molecules are attracted by
the charges on the K+ and Cl− ions. Water molecules in front of and behind the ions are not shown.
The diagram shows eight purple spheres labeled K superscript plus and eight green spheres labeled C l superscript minus mixed
and touching near the center of the diagram. Outside of this cluster of spheres are seventeen clusters of three spheres, which include
one red and two white spheres. A red sphere in one of these clusters is labeled O. A white sphere is labeled H. Two of the green C l
superscript minus spheres are surrounded by three of the red and white clusters, with the red spheres closer to the green spheres
than the white spheres. One of the K superscript plus purple spheres is surrounded by four of the red and white clusters. The white
spheres of these clusters are closest to the purple spheres.

When ionic compounds dissolve in water, the ions in the solid separate and disperse uniformly throughout the solution because
water molecules surround and solvate the ions, reducing the strong electrostatic forces between them. This process represents a
physical change known as dissociation. Under most conditions, ionic compounds will dissociate nearly completely when dissolved,
and so they are classified as strong electrolytes.
Let us consider what happens at the microscopic level when we add solid KCl to water. Ion-dipole forces attract the positive
(hydrogen) end of the polar water molecules to the negative chloride ions at the surface of the solid, and they attract the negative
(oxygen) ends to the positive potassium ions. The water molecules penetrate between individual K+ and Cl− ions and surround
them, reducing the strong interionic forces that bind the ions together and letting them move off into solution as solvated ions, as
Figure shows. The reduction of the electrostatic attraction permits the independent motion of each hydrated ion in a dilute solution,
resulting in an increase in the disorder of the system as the ions change from their fixed and ordered positions in the crystal to
mobile and much more disordered states in solution. This increased disorder is responsible for the dissolution of many ionic
compounds, including KCl, which dissolve with absorption of heat.
In other cases, the electrostatic attractions between the ions in a crystal are so large, or the ion-dipole attractive forces between the
ions and water molecules are so weak, that the increase in disorder cannot compensate for the energy required to separate the ions,
and the crystal is insoluble. Such is the case for compounds such as calcium carbonate (limestone), calcium phosphate (the
inorganic component of bone), and iron oxide (rust).

11.2.2: Covalent Electrolytes


Pure water is an extremely poor conductor of electricity because it is only very slightly ionized—only about two out of every 1
billion molecules ionize at 25 °C. Water ionizes when one molecule of water gives up a proton to another molecule of water,
yielding hydronium and hydroxide ions.
+ + −
H O(l) H O(l) ⇌ H O (aq) + OH (aq) (11.2.1)
2 2 3

In some cases, we find that solutions prepared from covalent compounds conduct electricity because the solute molecules react
chemically with the solvent to produce ions. For example, pure hydrogen chloride is a gas consisting of covalent HCl molecules.
This gas contains no ions. However, when we dissolve hydrogen chloride in water, we find that the solution is a very good
conductor. The water molecules play an essential part in forming ions: Solutions of hydrogen chloride in many other solvents, such
as benzene, do not conduct electricity and do not contain ions.
Hydrogen chloride is an acid, and so its molecules react with water, transferring H+ ions to form hydronium ions (H 3O
+
) and
chloride ions (Cl−):

Access for free at OpenStax 11.2.2 https://chem.libretexts.org/@go/page/38250


A chemical equation is shown. To the left, two hydrogen atoms are linked, each with a single dash to a central oxygen atom to the
left and below the oxygen symbol, which has two pairs of dots, above and to the right of the atom. A plus sign is shown to the
right, then a hydrogen atom linked to the left side of chlorine atom by a single dash with three pairs of dots, above, to the right, and
below the element symbol. An arrow points to the products which are three hydrogen atoms linked by single dashes to a central
oxygen atom shown in brackets with superscript plus. The oxygen atom has a single pair of dots above the element symbol. This is
followed by a plus and C l superscript minus. This symbol is surrounded by four pairs of dots, above and below and to the left and
right of the element symbol.
This reaction is essentially 100% complete for HCl (i.e., it is a strong acid and, consequently, a strong electrolyte). Likewise, weak
acids and bases that only react partially generate relatively low concentrations of ions when dissolved in water and are classified as
weak electrolytes. The reader may wish to review the discussion of strong and weak acids provided in the earlier chapter of this
text on reaction classes and stoichiometry.

Summary
Substances that dissolve in water to yield ions are called electrolytes. Electrolytes may be covalent compounds that chemically
react with water to produce ions (for example, acids and bases), or they may be ionic compounds that dissociate to yield their
constituent cations and anions, when dissolved. Dissolution of an ionic compound is facilitated by ion-dipole attractions between
the ions of the compound and the polar water molecules. Soluble ionic substances and strong acids ionize completely and are
strong electrolytes, while weak acids and bases ionize to only a small extent and are weak electrolytes. Nonelectrolytes are
substances that do not produce ions when dissolved in water.

Glossary
dissociation
physical process accompanying the dissolution of an ionic compound in which the compound’s constituent ions are solvated
and dispersed throughout the solution

electrolyte
substance that produces ions when dissolved in water

ion-dipole attraction
electrostatic attraction between an ion and a polar molecule

nonelectrolyte
substance that does not produce ions when dissolved in water

strong electrolyte
substance that dissociates or ionizes completely when dissolved in water

weak electrolyte
substance that ionizes only partially when dissolved in water

This page titled 11.2: Electrolytes is shared under a CC BY 4.0 license and was authored, remixed, and/or curated by OpenStax via source content
that was edited to the style and standards of the LibreTexts platform; a detailed edit history is available upon request.

Access for free at OpenStax 11.2.3 https://chem.libretexts.org/@go/page/38250


11.3: Solubility
 Learning Objectives
Describe the effects of temperature and pressure on solubility
State Henry’s law and use it in calculations involving the solubility of a gas in a liquid
Explain the degrees of solubility possible for liquid-liquid solutions

Imagine adding a small amount of salt to a glass of water, stirring until all the salt has dissolved, and then adding a bit more. You
can repeat this process until the salt concentration of the solution reaches its natural limit, a limit determined primarily by the
relative strengths of the solute-solute, solute-solvent, and solvent-solvent attractive forces discussed in the previous two modules of
this chapter. You can be certain that you have reached this limit because, no matter how long you stir the solution, undissolved salt
remains. The concentration of salt in the solution at this point is known as its solubility.
The solubility of a solute in a particular solvent is the maximum concentration that may be achieved under given conditions when
the dissolution process is at equilibrium. Referring to the example of salt in water:
+ −
NaCl(s) ⇌ Na (aq) + Cl (aq) (11.3.1)

When a solute’s concentration is equal to its solubility, the solution is said to be saturated with that solute. If the solute’s
concentration is less than its solubility, the solution is said to be unsaturated. A solution that contains a relatively low concentration
of solute is called dilute, and one with a relatively high concentration is called concentrated.
If we add more salt to a saturated solution of salt, we see it fall to the bottom and no more seems to dissolve. In fact, the added salt
does dissolve, as represented by the forward direction of the dissolution equation. Accompanying this process, dissolved salt will
precipitate, as depicted by the reverse direction of the equation. The system is said to be at equilibrium when these two reciprocal
processes are occurring at equal rates, and so the amount of undissolved and dissolved salt remains constant. Support for the
simultaneous occurrence of the dissolution and precipitation processes is provided by noting that the number and sizes of the
undissolved salt crystals will change over time, though their combined mass will remain the same.

crystallization of sodium acetate (v2)

Video 11.3.1 : Watch this impressive video showing the precipitation of sodium acetate from a supersaturated solution.
Solutions may be prepared in which a solute concentration exceeds its solubility. Such solutions are said to be supersaturated, and
they are interesting examples of nonequilibrium states. For example, the carbonated beverage in an open container that has not yet
“gone flat” is supersaturated with carbon dioxide gas; given time, the CO2 concentration will decrease until it reaches its
equilibrium value.

11.3.1: Solutions of Gases in Liquids


In an earlier module of this chapter, the effect of intermolecular attractive forces on solution formation was discussed. The
chemical structures of the solute and solvent dictate the types of forces possible and, consequently, are important factors in
determining solubility. For example, under similar conditions, the water solubility of oxygen is approximately three times greater
than that of helium, but 100 times less than the solubility of chloromethane, CHCl3. Considering the role of the solvent’s chemical

Access for free at OpenStax 11.3.1 https://chem.libretexts.org/@go/page/38251


structure, note that the solubility of oxygen in the liquid hydrocarbon hexane, C6H14, is approximately 20 times greater than it is in
water.
Other factors also affect the solubility of a given substance in a given solvent. Temperature is one such factor, with gas solubility
typically decreasing as temperature increases (Figure 11.3.1). This is one of the major impacts resulting from the thermal pollution
of natural bodies of water.

Figure 11.3.1 : The solubilities of these gases in water decrease as the temperature increases. All solubilities were measured with a
constant pressure of 101.3 kPa (1 atm) of gas above the solutions.
When the temperature of a river, lake, or stream is raised abnormally high, usually due to the discharge of hot water from some
industrial process, the solubility of oxygen in the water is decreased. Decreased levels of dissolved oxygen may have serious
consequences for the health of the water’s ecosystems and, in severe cases, can result in large-scale fish kills (Figure 11.3.2).

Figure 11.3.2 : (a) The small bubbles of air in this glass of chilled water formed when the water warmed to room temperature and
the solubility of its dissolved air decreased. (b) The decreased solubility of oxygen in natural waters subjected to thermal pollution
can result in large-scale fish kills. (credit a: modification of work by Liz West; credit b: modification of work by U.S. Fish and
Wildlife Service)
The solubility of a gaseous solute is also affected by the partial pressure of solute in the gas to which the solution is exposed. Gas
solubility increases as the pressure of the gas increases. Carbonated beverages provide a nice illustration of this relationship. The
carbonation process involves exposing the beverage to a relatively high pressure of carbon dioxide gas and then sealing the
beverage container, thus saturating the beverage with CO2 at this pressure. When the beverage container is opened, a familiar hiss
is heard as the carbon dioxide gas pressure is released, and some of the dissolved carbon dioxide is typically seen leaving solution
in the form of small bubbles (Figure 11.3.3). At this point, the beverage is supersaturated with carbon dioxide and, with time, the
dissolved carbon dioxide concentration will decrease to its equilibrium value and the beverage will become “flat.”

Access for free at OpenStax 11.3.2 https://chem.libretexts.org/@go/page/38251


Figure 11.3.3 : Opening the bottle of carbonated beverage reduces the pressure of the gaseous carbon dioxide above the beverage.
The solubility of CO2 is thus lowered, and some dissolved carbon dioxide may be seen leaving the solution as small gas bubbles.
(credit: modification of work by Derrick Coetzee)
A dark brown liquid is shown in a clear, colorless container. A thick layer of beige bubbles appear at the surface of the liquid. In
the liquid, thirteen small clusters of single black spheres with two red spheres attached to the left and right are shown. Red spheres
represent oxygen atoms and black represent carbon atoms. Seven white arrows point upward in the container from these clusters to
the bubble layer at the top of the liquid.
For many gaseous solutes, the relation between solubility, C , and partial pressure, P , is a proportional one:
g g

Cg = kPg

where k is a proportionality constant that depends on the identities of the gaseous solute and solvent, and on the solution
temperature. This is a mathematical statement of Henry’s law: The quantity of an ideal gas that dissolves in a definite volume of
liquid is directly proportional to the pressure of the gas.

 Example 11.3.1: Application of Henry’s Law

At 20 °C, the concentration of dissolved oxygen in water exposed to gaseous oxygen at a partial pressure of 101.3 kPa (760
torr) is 1.38 × 10−3 mol L−1. Use Henry’s law to determine the solubility of oxygen when its partial pressure is 20.7 kPa (155
torr), the approximate pressure of oxygen in earth’s atmosphere.

Solution
According to Henry’s law, for an ideal solution the solubility, Cg, of a gas (1.38 × 10−3 mol L−1, in this case) is directly
proportional to the pressure, Pg, of the undissolved gas above the solution (101.3 kPa, or 760 torr, in this case). Because we
know both Cg and Pg, we can rearrange this expression to solve for k.
Cg = kPg

Cg
k =
Pg

−3 −1
1.38 × 10 mol L
=
101.3 kPa

−5 −1 −1
= 1.36 × 10 mol L kPa

−6 −1 −1
(1.82 × 10 mol L torr )

Now we can use k to find the solubility at the lower pressure.

Cg = kPg

−5 −1 −1
1.36 × 10 mol L kPa × 20.7 kPa

−6 −1 −1
(or 1.82 × 10 mol L torr × 155 torr)

−4 −1
= 2.82 × 10 mol L

Note that various units may be used to express the quantities involved in these sorts of computations. Any combination of units
that yield to the constraints of dimensional analysis are acceptable.

Access for free at OpenStax 11.3.3 https://chem.libretexts.org/@go/page/38251


 Exercise 11.3.1

Exposing a 100.0 mL sample of water at 0 °C to an atmosphere containing a gaseous solute at 20.26 kPa (152 torr) resulted in
the dissolution of 1.45 × 10−3 g of the solute. Use Henry’s law to determine the solubility of this gaseous solute when its
pressure is 101.3 kPa (760 torr).

Answer
7.25 × 10−3 g in 100.0 mL or 0.0725 g/L

 Case Study: Decompression Sickness (“The Bends”)

Decompression sickness (DCS), or “the bends,” is an effect of the increased pressure of the air inhaled by scuba divers when
swimming underwater at considerable depths. In addition to the pressure exerted by the atmosphere, divers are subjected to
additional pressure due to the water above them, experiencing an increase of approximately 1 atm for each 10 m of depth.
Therefore, the air inhaled by a diver while submerged contains gases at the corresponding higher ambient pressure, and the
concentrations of the gases dissolved in the diver’s blood are proportionally higher per Henry’s law.

Figure 11.3.4 : (a) US Navy divers undergo training in a recompression chamber. (b) Divers receive hyperbaric oxygen therapy.
Two photos are shown. The first shows two people seated in a steel chamber on benches that run length of the chamber on each
side. The chamber has a couple of small circular windows and an open hatch-type door. One of the two people is giving a
thumbs up gesture. The second image provides a view through a small, circular window. Inside the two people can be seen
with masks over their mouths and noses. The people appear to be reading.
As the diver ascends to the surface of the water, the ambient pressure decreases and the dissolved gases becomes less soluble.
If the ascent is too rapid, the gases escaping from the diver’s blood may form bubbles that can cause a variety of symptoms
ranging from rashes and joint pain to paralysis and death. To avoid DCS, divers must ascend from depths at relatively slow
speeds (10 or 20 m/min) or otherwise make several decompression stops, pausing for several minutes at given depths during
the ascent. When these preventive measures are unsuccessful, divers with DCS are often provided hyperbaric oxygen therapy
in pressurized vessels called decompression (or recompression) chambers (Figure 11.3.4).

Deviations from Henry’s law are observed when a chemical reaction takes place between the gaseous solute and the solvent. Thus,
for example, the solubility of ammonia in water does not increase as rapidly with increasing pressure as predicted by the law
because ammonia, being a base, reacts to some extent with water to form ammonium ions and hydroxide ions.

Gases can form supersaturated solutions. If a solution of a gas in a liquid is prepared either at low temperature or under pressure
(or both), then as the solution warms or as the gas pressure is reduced, the solution may become supersaturated. In 1986, more than
1700 people in Cameroon were killed when a cloud of gas, almost certainly carbon dioxide, bubbled from Lake Nyos (Figure
11.3.5), a deep lake in a volcanic crater. The water at the bottom of Lake Nyos is saturated with carbon dioxide by volcanic activity

beneath the lake. It is believed that the lake underwent a turnover due to gradual heating from below the lake, and the warmer, less-

Access for free at OpenStax 11.3.4 https://chem.libretexts.org/@go/page/38251


dense water saturated with carbon dioxide reached the surface. Consequently, tremendous quantities of dissolved CO2 were
released, and the colorless gas, which is denser than air, flowed down the valley below the lake and suffocated humans and animals
living in the valley.

Figure 11.3.5 : (a) It is believed that the 1986 disaster that killed more than 1700 people near Lake Nyos in Cameroon resulted
when a large volume of carbon dioxide gas was released from the lake. (b) A CO2 vent has since been installed to help outgas the
lake in a slow, controlled fashion and prevent a similar catastrophe from happening in the future. (credit a: modification of work by
Jack Lockwood; credit b: modification of work by Bill Evans)
Two photos are shown. The first is an aerial view of a lake surrounded by green hills. The second shows a large body of water with
a fountain sending liquid up into the air several yards or meters above the surface of the water.

11.3.2: Solutions of Liquids in Liquids


We know that some liquids mix with each other in all proportions; in other words, they have infinite mutual solubility and are said
to be miscible. Ethanol, sulfuric acid, and ethylene glycol (popular for use as antifreeze, pictured in Figure 11.3.6) are examples of
liquids that are completely miscible with water. Two-cycle motor oil is miscible with gasoline.

Figure 11.3.6 : Water and antifreeze are miscible; mixtures of the two are homogeneous in all proportions. (credit:
“dno1967”/Wikimedia commons)
Liquids that mix with water in all proportions are usually polar substances or substances that form hydrogen bonds. For such
liquids, the dipole-dipole attractions (or hydrogen bonding) of the solute molecules with the solvent molecules are at least as strong
as those between molecules in the pure solute or in the pure solvent. Hence, the two kinds of molecules mix easily. Likewise,
nonpolar liquids are miscible with each other because there is no appreciable difference in the strengths of solute-solute, solvent-
solvent, and solute-solvent intermolecular attractions. The solubility of polar molecules in polar solvents and of nonpolar molecules
in nonpolar solvents is, again, an illustration of the chemical axiom “like dissolves like.”
Two liquids that do not mix to an appreciable extent are called immiscible. Layers are formed when we pour immiscible liquids
into the same container. Gasoline, oil (Figure 11.3.7), benzene, carbon tetrachloride, some paints, and many other nonpolar liquids
are immiscible with water. The attraction between the molecules of such nonpolar liquids and polar water molecules is ineffectively
weak. The only strong attractions in such a mixture are between the water molecules, so they effectively squeeze out the molecules
of the nonpolar liquid. The distinction between immiscibility and miscibility is really one of degrees, so that miscible liquids are of
infinite mutual solubility, while liquids said to be immiscible are of very low (though not zero) mutual solubility.

Access for free at OpenStax 11.3.5 https://chem.libretexts.org/@go/page/38251


Figure 11.3.7 : Water and oil are immiscible. Mixtures of these two substances will form two separate layers with the less dense oil
floating on top of the water. (credit: “Yortw”/Flickr)

This is a photo of a clear, colorless martini glass containing a golden colored liquid layer resting on top of a clear, colorless liquid.
Two liquids, such as bromine and water, that are of moderate mutual solubility are said to be partially miscible. Two partially
miscible liquids usually form two layers when mixed. In the case of the bromine and water mixture, the upper layer is water,
saturated with bromine, and the lower layer is bromine saturated with water. Since bromine is nonpolar, and, thus, not very soluble
in water, the water layer is only slightly discolored by the bright orange bromine dissolved in it. Since the solubility of water in
bromine is very low, there is no noticeable effect on the dark color of the bromine layer (Figure 11.3.8).

Figure 11.3.8 : Bromine (the deep orange liquid on the left) and water (the clear liquid in the middle) are partially miscible. The top
layer in the mixture on the right is a saturated solution of bromine in water; the bottom layer is a saturated solution of water in
bromine. (credit: Paul Flowers)
This figure shows three test tubes. The first test tube holds a dark orange-brown substance. The second test tube holds a clear
substance. The amount of substance in both test tubes is the same. The third test tube holds a dark orange-brown substance on the
bottom with a lighter orange substance on top. The amount of substance in the third test tube is almost double of the first two.

11.3.3: Solutions of Solids in Liquids


The dependence of solubility on temperature for a number of inorganic solids in water is shown by the solubility curves in Figure
11.3.9. Reviewing these data indicate a general trend of increasing solubility with temperature, although there are exceptions, as

illustrated by the ionic compound cerium sulfate.

Access for free at OpenStax 11.3.6 https://chem.libretexts.org/@go/page/38251


Figure 11.3.9 : This graph shows how the solubility of several solids changes with temperature.
This shows a graph of the solubility of sugar C subscript 12 H subscript 22 O subscript 11, K N O subscript 3, N a N O subscript 3,
N a B r, K B r, N a subscript 2 S O subscript 4, K C l, and C e subscript 2 left parenthesis S O subscript 4 right parenthesis subscript
3 in g solute per 100 g H subscript 2 O at temperatures ranging from 0 degrees Celsius to 100 degrees Celsius. At 0 degrees
Celsius, solubilities are approximately 180 for sugar C subscript 12 H subscript 22 O subscript 11, 115 for K N O subscript 3, 75
for N a N O subscript 3, 115 for N a B r, 55 for K B r, 7 for N a subscript 2 S O subscript 4, 25 for K C l, and 20 for C e subscript 2
left parenthesis S O subscript 4 right parenthesis subscript 3. At 0 degrees Celsius, solubilities are approximately 180 for sugar C
subscript 12 H subscript 22 O subscript 11, 115 for K N O subscript 3, 75 for N a N O subscript 3, 115 for N a B r, 55 for K B r, 7
for N a subscript 2 S O subscript 4, 25 for K C l, and 20 for C e subscript 2 left parenthesis S O subscript 4 right parenthesis
subscript 3. At 100 degrees Celsius, sugar C subscript 12 H subscript 22 O subscript 11 has exceeded the upper limit of solubility
indicated on the graph, 240 for K N O subscript 3, 178 for N a N O subscript 3, 123 for N a B r, 105 for K B r, 52 for N a subscript
2 S O subscript 4, 58 for K C l, and the graph for C e subscript 2 left parenthesis S O subscript 4 right parenthesis subscript 3 stops
at about 92 degrees Celsius where the solubility is nearly zero. The graph for N a subscript 2 S O subscript 4 is shown in red. All
others substances are shown in blue. The solubility of this substance increases until about 30 degrees Celsius and declines beyond
that point with increasing temperature.
The temperature dependence of solubility can be exploited to prepare supersaturated solutions of certain compounds. A solution
may be saturated with the compound at an elevated temperature (where the solute is more soluble) and subsequently cooled to a
lower temperature without precipitating the solute. The resultant solution contains solute at a concentration greater than its
equilibrium solubility at the lower temperature (i.e., it is supersaturated) and is relatively stable. Precipitation of the excess solute
can be initiated by adding a seed crystal (see the video in the Link to Learning earlier in this module) or by mechanically agitating
the solution. Some hand warmers, such as the one pictured in Figure 11.3.10, take advantage of this behavior.

Figure 11.3.10: This hand warmer produces heat when the sodium acetate in a supersaturated solution precipitates. Precipitation of
the solute is initiated by a mechanical shockwave generated when the flexible metal disk within the solution is “clicked.” (credit:
modification of work by “Velela”/Wikimedia Commons)
Three photos of hand warmers are shown side by side with an arrow pointing from the first photo to the second, and another arrow
pointing from the second photo to the third. The first packet contains a clear colorless liquid and a small metal disc can be seen. In
the second packet, the disc can’t be seen and a dispersion of white liquid is beginning. In the third packet, all of the liquid is white.

Access for free at OpenStax 11.3.7 https://chem.libretexts.org/@go/page/38251


Crystallization of the "Magic" Gel Hand …

Video 11.3.2: This video shows the crystallization process occurring in a hand warmer.

Summary
The extent to which one substance will dissolve in another is determined by several factors, including the types and relative
strengths of intermolecular attractive forces that may exist between the substances’ atoms, ions, or molecules. This tendency to
dissolve is quantified as substance’s solubility, its maximum concentration in a solution at equilibrium under specified conditions.
A saturated solution contains solute at a concentration equal to its solubility. A supersaturated solution is one in which a solute’s
concentration exceeds its solubility—a nonequilibrium (unstable) condition that will result in solute precipitation when the solution
is appropriately perturbed. Miscible liquids are soluble in all proportions, and immiscible liquids exhibit very low mutual solubility.
Solubilities for gaseous solutes decrease with increasing temperature, while those for most, but not all, solid solutes increase with
temperature. The concentration of a gaseous solute in a solution is proportional to the partial pressure of the gas to which the
solution is exposed, a relation known as Henry’s law.

11.3.4: Key Equations


Cg = kPg

Glossary
Henry’s law
law stating the proportional relationship between the concentration of dissolved gas in a solution and the partial pressure of the
gas in contact with the solution

immiscible
of negligible mutual solubility; typically refers to liquid substances

miscible
mutually soluble in all proportions; typically refers to liquid substances

partially miscible
of moderate mutual solubility; typically refers to liquid substances

saturated
of concentration equal to solubility; containing the maximum concentration of solute possible for a given temperature and
pressure

solubility
extent to which a solute may be dissolved in water, or any solvent

supersaturated
of concentration that exceeds solubility; a nonequilibrium state

Access for free at OpenStax 11.3.8 https://chem.libretexts.org/@go/page/38251


unsaturated
of concentration less than solubility

This page titled 11.3: Solubility is shared under a CC BY 4.0 license and was authored, remixed, and/or curated by OpenStax via source content
that was edited to the style and standards of the LibreTexts platform; a detailed edit history is available upon request.

Access for free at OpenStax 11.3.9 https://chem.libretexts.org/@go/page/38251


11.4: Colligative Properties
 Learning Objectives
Express concentrations of solution components using mole fraction and molality
Describe the effect of solute concentration on various solution properties (vapor pressure, boiling point, freezing point, and
osmotic pressure)
Perform calculations using the mathematical equations that describe these various colligative effects
Describe the process of distillation and its practical applications
Explain the process of osmosis and describe how it is applied industrially and in nature

The properties of a solution are different from those of either the pure solute(s) or solvent. Many solution properties are dependent
upon the chemical identity of the solute. Compared to pure water, a solution of hydrogen chloride is more acidic, a solution of
ammonia is more basic, a solution of sodium chloride is more dense, and a solution of sucrose is more viscous. There are a few
solution properties, however, that depend only upon the total concentration of solute species, regardless of their identities. These
colligative properties include vapor pressure lowering, boiling point elevation, freezing point depression, and osmotic pressure.
This small set of properties is of central importance to many natural phenomena and technological applications, as will be
described in this module.

11.4.1: Mole Fraction and Molality


Several units commonly used to express the concentrations of solution components were introduced in an earlier chapter of this
text, each providing certain benefits for use in different applications. For example, molarity (M) is a convenient unit for use in
stoichiometric calculations, since it is defined in terms of the molar amounts of solute species:
mol solute
M = (11.4.1)
L solution

Because solution volumes vary with temperature, molar concentrations will likewise vary. When expressed as molarity, the
concentration of a solution with identical numbers of solute and solvent species will be different at different temperatures, due to
the contraction/expansion of the solution. More appropriate for calculations involving many colligative properties are mole-based
concentration units whose values are not dependent on temperature. Two such units are mole fraction (introduced in the previous
chapter on gases) and molality.
The mole fraction, χ, of a component is the ratio of its molar amount to the total number of moles of all solution components:
mol A
χA = (11.4.2)
total mol of all components

Molality is a concentration unit defined as the ratio of the numbers of moles of solute to the mass of the solvent in kilograms:
mol solute
m = (11.4.3)
kg solvent

Since these units are computed using only masses and molar amounts, they do not vary with temperature and, thus, are better suited
for applications requiring temperature-independent concentrations, including several colligative properties, as will be described in
this chapter module.

 Example 11.4.1: Calculating Mole Fraction and Molality


The antifreeze in most automobile radiators is a mixture of equal volumes of ethylene glycol and water, with minor amounts of
other additives that prevent corrosion. What are the (a) mole fraction and (b) molality of ethylene glycol, C2H4(OH)2, in a
solution prepared from 2.22 × 10 g of ethylene glycol and 2.00 × 10 g of water (approximately 2 L of glycol and 2 L of
3 3

water)?

Solution

Access for free at OpenStax 11.4.1 https://chem.libretexts.org/@go/page/38252


(a) The mole fraction of ethylene glycol may be computed by first deriving molar amounts of both solution components and
then substituting these amounts into the unit definition.
1 mol C2 H4 (OH)2
mol C2 H4 (OH)2 = 2220 g × = 35.8 mol C2 H4 (OH)2
62.07 g C2 H4 (OH)2

1 mol H2 O
mol H2 O = 2000 g × = 111 mol H2 O
18.02 g H2 O

35.8 mol C2 H4 (OH)2


χethylene glycol = = 0.245
(35.8 + 111) mol total

Notice that mole fraction is a dimensionless property, being the ratio of properties with identical units (moles).
(b) To find molality, we need to know the moles of the solute and the mass of the solvent (in kg).
First, use the given mass of ethylene glycol and its molar mass to find the moles of solute:
mol C2 H2 (OH)2
2220 g C2 H4 (OH)2 ( ) = 35.8 mol C2 H4 (OH)2
62.07 g

Then, convert the mass of the water from grams to kilograms:


1 kg
2000 g H2 O ( ) = 2 kg H2 O
1000 g

Finally, calculate molarity per its definition:


mol solute
molality =
kg solvent

35.8 mol C2 H4 (OH)2


molality =
2 kg H2 O

molality = 17.9 m

 Exercise 11.4.1

What are the mole fraction and molality of a solution that contains 0.850 g of ammonia, NH3, dissolved in 125 g of water?

Answer
7.14 × 10−3; 0.399 m

 Example 11.4.2: Converting Mole Fraction and Molal Concentrations

Calculate the mole fraction of solute and solvent in a 3.0 m solution of sodium chloride.

Solution
Converting from one concentration unit to another is accomplished by first comparing the two unit definitions. In this case,
both units have the same numerator (moles of solute) but different denominators. The provided molal concentration may be
written as:
3.0 mol NaCl

1.0 kg H2 O

The numerator for this solution’s mole fraction is, therefore, 3.0 mol NaCl. The denominator may be computed by deriving the
molar amount of water corresponding to 1.0 kg
1000 g mol H2 O
1.0 kg H2 O ( )( ) = 55 mol H2 O
1 kg 18.02 g

Access for free at OpenStax 11.4.2 https://chem.libretexts.org/@go/page/38252


and then substituting these molar amounts into the definition for mole fraction.
mol H2 O
XH O =
2
mol NaCl + mol H2 O

55 mol H2 O
XH2 O =
3.0 mol NaCl + 55 mol H2 O

XH O = 0.95
2

mol NaCl
XNaCl =
mol NaCl + mol H2 O

3.0 mol NaCl


XNaCl =
3.0 mol NaCl + 55 mol H2 O

XNaCl = 0.052

 Exercise 11.4.2

The mole fraction of iodine, I , dissolved in dichloromethane, CH


2 2
Cl
2
, is 0.115. What is the molal concentration, m, of iodine
in this solution?

Answer
1.50 m

11.4.2: Vapor Pressure Lowering


As described in the chapter on liquids and solids, the equilibrium vapor pressure of a liquid is the pressure exerted by its gaseous
phase when vaporization and condensation are occurring at equal rates:
liquid⇌ gas (11.4.4)

Dissolving a nonvolatile substance in a volatile liquid results in a lowering of the liquid’s vapor pressure. This phenomenon can be
rationalized by considering the effect of added solute molecules on the liquid's vaporization and condensation processes. To
vaporize, solvent molecules must be present at the surface of the solution. The presence of solute decreases the surface area
available to solvent molecules and thereby reduces the rate of solvent vaporization. Since the rate of condensation is unaffected by
the presence of solute, the net result is that the vaporization-condensation equilibrium is achieved with fewer solvent molecules in
the vapor phase (i.e., at a lower vapor pressure) (Figure 11.4.1). While this kinetic interpretation is useful, it does not account for
several important aspects of the colligative nature of vapor pressure lowering. A more rigorous explanation involves the property of
entropy, a topic of discussion in a later text chapter on thermodynamics. For purposes of understanding the lowering of a liquid's
vapor pressure, it is adequate to note that the greater entropy of a solution in comparison to its separate solvent and solute serves to
effectively stabilize the solvent molecules and hinder their vaporization. A lower vapor pressure results, and a correspondingly
higher boiling point as described in the next section of this module.

Access for free at OpenStax 11.4.3 https://chem.libretexts.org/@go/page/38252


Figure 11.4.1 : The presence of nonvolatile solutes lowers the vapor pressure of a solution by impeding the evaporation of solvent
molecules.
This figure contains two images. Figure a is labeled “pure water.” It shows a beaker half-filled with liquid. In the liquid, eleven
molecules are evenly dispersed in the liquid each consisting of one central red sphere and two slightly smaller white spheres are
shown. Four molecules near the surface of the liquid have curved arrows drawn from them pointing to the space above the liquid in
the beaker. Above the liquid, twelve molecules are shown, with arrows pointing from three of them into the liquid below. Figure b
is labeled “Aqueous solution.” It is similar to figure a except that eleven blue spheres, slightly larger in size than the molecules, are
dispersed evenly in the liquid. Only four curved arrows appear in this diagram with two from the molecules in the liquid pointing to
the space above and two from molecules in the space above the liquid pointing into the liquid below.
The relationship between the vapor pressures of solution components and the concentrations of those components is described by
Raoult’s law: The partial pressure exerted by any component of an ideal solution is equal to the vapor pressure of the pure
component multiplied by its mole fraction in the solution.

PA = XA P (11.4.5)
A

where PA is the partial pressure exerted by component A in the solution, P is the vapor pressure of pure A, and XA is the mole

A

fraction of A in the solution. (Mole fraction is a concentration unit introduced in the chapter on gases.)
Recalling that the total pressure of a gaseous mixture is equal to the sum of partial pressures for all its components (Dalton’s law of
partial pressures), the total vapor pressure exerted by a solution containing i components is

Psolution = ∑ Pi = ∑ Xi P (11.4.6)
i

i i

A nonvolatile substance is one whose vapor pressure is negligible (P° ≈ 0), and so the vapor pressure above a solution containing
only nonvolatile solutes is due only to the solvent:

Psolution = Xsolvent P (11.4.7)
solvent

 Example 11.4.3: Calculation of a Vapor Pressure

Compute the vapor pressure of an ideal solution containing 92.1 g of glycerin, C3H5(OH)3, and 184.4 g of ethanol, C2H5OH, at
40 °C. The vapor pressure of pure ethanol is 0.178 atm at 40 °C. Glycerin is essentially nonvolatile at this temperature.

Solution
Since the solvent is the only volatile component of this solution, its vapor pressure may be computed per Raoult’s law as:

Psolution = Xsolvent P
solvent

First, calculate the molar amounts of each solution component using the provided mass data.
1 mol C3 H5 (OH)3
92.1 g C3 H5 (OH)3 × = 1.00 mol C3 H5 (OH)3
92.094 g C3 H5 (OH)3

1 mol C2 H5 OH
184.4 g C2 H5 OH × = 4.000 mol C2 H5 OH
46.069 g C2 H5 OH

Next, calculate the mole fraction of the solvent (ethanol) and use Raoult’s law to compute the solution’s vapor pressure.

Access for free at OpenStax 11.4.4 https://chem.libretexts.org/@go/page/38252


4.000 mol
XC2 H5 OH = = 0.800
(1.00 mol + 4.000 mol)


Psolv = XsolvP = 0.800 × 0.178 atm = 0.142 atm
solv

 Exercise 11.4.3

A solution contains 5.00 g of urea, CO(NH2)2 (a nonvolatile solute) and 0.100 kg of water. If the vapor pressure of pure water
at 25 °C is 23.7 torr, what is the vapor pressure of the solution?

Answer
23.4 torr

11.4.3: Elevation of the Boiling Point of a Solvent


As described in the chapter on liquids and solids, the boiling point of a liquid is the temperature at which its vapor pressure is equal
to ambient atmospheric pressure. Since the vapor pressure of a solution is lowered due to the presence of nonvolatile solutes, it
stands to reason that the solution’s boiling point will subsequently be increased. Compared to pure solvent, a solution, therefore,
will require a higher temperature to achieve any given vapor pressure, including one equivalent to that of the surrounding
atmosphere. The increase in boiling point observed when nonvolatile solute is dissolved in a solvent, ΔT , is called boiling point
b

elevation and is directly proportional to the molal concentration of solute species:


ΔTb = Kb m (11.4.8)

where
Kb is the boiling point elevation constant, or the ebullioscopic constant and
m is the molal concentration (molality) of all solute species.
Boiling point elevation constants are characteristic properties that depend on the identity of the solvent. Values of Kb for several
solvents are listed in Table 11.4.1.
Table 11.4.1 : Boiling Point Elevation and Freezing Point Depression Constants for Several Solvents
Freezing Point (°C at 1
Solvent Boiling Point (°C at 1 atm) Kb (Cm−1) Kf (Cm−1)
atm)

water 100.0 0.512 0.0 1.86

hydrogen acetate 118.1 3.07 16.6 3.9

benzene 80.1 2.53 5.5 5.12

chloroform 61.26 3.63 −63.5 4.68

nitrobenzene 210.9 5.24 5.67 8.1

The extent to which the vapor pressure of a solvent is lowered and the boiling point is elevated depends on the total number of
solute particles present in a given amount of solvent, not on the mass or size or chemical identities of the particles. A 1 m aqueous
solution of sucrose (342 g/mol) and a 1 m aqueous solution of ethylene glycol (62 g/mol) will exhibit the same boiling point
because each solution has one mole of solute particles (molecules) per kilogram of solvent.

 Example 11.4.4: Calculating the Boiling Point of a Solution

What is the boiling point of a 0.33 m solution of a nonvolatile solute in benzene?

Solution
Use the equation relating boiling point elevation to solute molality to solve this problem in two steps.

Access for free at OpenStax 11.4.5 https://chem.libretexts.org/@go/page/38252


1. Calculate the change in boiling point.
−1
ΔTb = Kb m = 2.53 °C m × 0.33 m = 0.83 °C

Add the boiling point elevation to the pure solvent’s boiling point.
Boiling temperature = 80.1 °C + 0.83 °C = 80.9 °C

 Exercise 11.4.4

What is the boiling point of the antifreeze described in Example 11.4.4?

Answer
109.2 °C

 Example 11.4.5: The Boiling Point of an Iodine Solution

Find the boiling point of a solution of 92.1 g of iodine, I


2
, in 800.0 g of chloroform, CHCl
3
, assuming that the iodine is
nonvolatile and that the solution is ideal.

Solution
We can solve this problem using four steps.

1. Convert from grams to moles of I using the molar mass of I in the unit conversion factor.
2 2

Result: 0.363 mol


Determine the molality of the solution from the number of moles of solute and the mass of solvent, in kilograms.
Result: 0.454 m
Use the direct proportionality between the change in boiling point and molal concentration to determine how much the boiling
point changes.
Result: 1.65 °C
Determine the new boiling point from the boiling point of the pure solvent and the change.
Result: 62.91 °C
Check each result as a self-assessment.

 Exercise 11.4.5: glycerin:Water Solution


What is the boiling point of a solution of 1.0 g of glycerin, C 3
H (OH)
5 3
, in 47.8 g of water? Assume an ideal solution.

Answer

Access for free at OpenStax 11.4.6 https://chem.libretexts.org/@go/page/38252


100.12 °C

11.4.4: Distillation of Solutions


Distillation is a technique for separating the components of mixtures that is widely applied in both in the laboratory and in
industrial settings. It is used to refine petroleum, to isolate fermentation products, and to purify water. This separation technique
involves the controlled heating of a sample mixture to selectively vaporize, condense, and collect one or more components of
interest. A typical apparatus for laboratory-scale distillations is shown in Figure 11.4.2.

Figure 11.4.2 : A typical laboratory distillation unit is shown in (a) a photograph and (b) a schematic diagram of the components.
(credit a: modification of work by “Rifleman82”/Wikimedia commons; credit b: modification of work by “Slashme”/Wikimedia
Commons)
Figure a contains a photograph of a common laboratory distillation unit. Figure b provides a diagram labeling typical components
of a laboratory distillation unit, including a stirrer/heat plate with heat and stirrer speed control, a heating bath of oil or sand,
stirring means such as boiling chips, a still pot, a still head, a thermometer for boiling point temperature reading, a condenser with a
cool water inlet and outlet, a still receiver with a vacuum or gas inlet, a receiving flask for holding distillate, and a cooling bath.
Oil refineries use large-scale fractional distillation to separate the components of crude oil. The crude oil is heated to high
temperatures at the base of a tall fractionating column, vaporizing many of the components that rise within the column. As
vaporized components reach adequately cool zones during their ascent, they condense and are collected. The collected liquids are
simpler mixtures of hydrocarbons and other petroleum compounds that are of appropriate composition for various applications
(e.g., diesel fuel, kerosene, gasoline), as depicted in Figure 11.4.3.

Access for free at OpenStax 11.4.7 https://chem.libretexts.org/@go/page/38252


Figure 11.4.3 : Crude oil is a complex mixture that is separated by large-scale fractional distillation to isolate various simpler
mixtures.
This figure contains a photo of a refinery, showing large columnar structures. A diagram of a fractional distillation column used in
separating crude oil is also shown. Near the bottom of the column, an arrow pointing into the column shows a point of entry for
heated crude oil. The column contains several layers at which different components are removed. At the very bottom, residue
materials are removed as indicated by an arrow out of the column. At each successive level, different materials are removed
proceeding from the bottom to the top of the column. The materials are fuel oil, followed by diesel oil, kerosene, naptha, gasoline,
and refinery gas at the very top. To the right of the column diagram, a double sided arrow is shown that is blue at the top and
gradually changes color to red moving downward. The blue top of the arrow is labeled, “small molecules: low boiling point, very
volatile, flows easily, ignites easily.” The red bottom of the arrow is labeled, “large molecules: high boiling point, not very volatile,
does not flow easily, does not ignite easily.”

11.4.5: Depression of the Freezing Point of a Solvent


Solutions freeze at lower temperatures than pure liquids. This phenomenon is exploited in “de-icing” schemes that use salt (Figure
11.4.4), calcium chloride, or urea to melt ice on roads and sidewalks, and in the use of ethylene glycol as an “antifreeze” in

automobile radiators. Seawater freezes at a lower temperature than fresh water, and so the Arctic and Antarctic oceans remain
unfrozen even at temperatures below 0 °C (as do the body fluids of fish and other cold-blooded sea animals that live in these
oceans).

Figure 11.4.4 : Rock salt (NaCl), calcium chloride (CaCl2), or a mixture of the two are used to melt ice. (credit: modification of
work by Eddie Welker)
The decrease in freezing point of a dilute solution compared to that of the pure solvent, ΔTf, is called the freezing point depression
and is directly proportional to the molal concentration of the solute

ΔTf = Kf m (11.4.9)

where
mis the molal concentration of the solute in the solvent and
K is called the freezing point depression constant (or cryoscopic constant).
f

Just as for boiling point elevation constants, these are characteristic properties whose values depend on the chemical identity of the
solvent. Values of Kf for several solvents are listed in Table 11.4.1.

Access for free at OpenStax 11.4.8 https://chem.libretexts.org/@go/page/38252


 Example 11.4.5: Calculation of the Freezing Point of a Solution
What is the freezing point of the 0.33 m solution of a nonvolatile nonelectrolyte solute in benzene described in Example
11.4.4?

Solution
Use the equation relating freezing point depression to solute molality to solve this problem in two steps.

1. Calculate the change in freezing point.


−1
ΔTf = Kf m = 5.12 °C m × 0.33 m = 1.7 °C

2. Subtract the freezing point change observed from the pure solvent’s freezing point.

Freezing Temperature = 5.5 °C − 1.7 °C = 3.8 °C

 Exercise 11.4.6

What is the freezing point of a 1.85 m solution of a nonvolatile nonelectrolyte solute in nitrobenzene?

Answer
−9.3 °C

 Colligative Properties and De-Icing

Sodium chloride and its group 2 analogs calcium and magnesium chloride are often used to de-ice roadways and sidewalks,
due to the fact that a solution of any one of these salts will have a freezing point lower than 0 °C, the freezing point of pure
water. The group 2 metal salts are frequently mixed with the cheaper and more readily available sodium chloride (“rock salt”)
for use on roads, since they tend to be somewhat less corrosive than the NaCl, and they provide a larger depression of the
freezing point, since they dissociate to yield three particles per formula unit, rather than two particles like the sodium chloride.
Because these ionic compounds tend to hasten the corrosion of metal, they would not be a wise choice to use in antifreeze for
the radiator in your car or to de-ice a plane prior to takeoff. For these applications, covalent compounds, such as ethylene or
propylene glycol, are often used. The glycols used in radiator fluid not only lower the freezing point of the liquid, but they
elevate the boiling point, making the fluid useful in both winter and summer. Heated glycols are often sprayed onto the surface
of airplanes prior to takeoff in inclement weather in the winter to remove ice that has already formed and prevent the formation
of more ice, which would be particularly dangerous if formed on the control surfaces of the aircraft (Video 11.4.1).

Access for free at OpenStax 11.4.9 https://chem.libretexts.org/@go/page/38252


Aircraft De-Icing - Close Up, Details [HD]

Video 11.4.1 : Freezing point depression is exploited to remove ice from the control surfaces of aircraft.

11.4.6: Phase Diagram for an Aqueous Solution of a Nonelectrolyte


The colligative effects on vapor pressure, boiling point, and freezing point described in the previous section are conveniently
summarized by comparing the phase diagrams for a pure liquid and a solution derived from that liquid. Phase diagrams for water
and an aqueous solution are shown in Figure 11.4.5.

Figure 11.4.5 : These phase diagrams show water (solid curves) and an aqueous solution of nonelectrolyte (dashed curves).
This phase diagram indicates the pressure in atmospheres of water and a solution at various temperatures. The graph shows the
freezing point of water and the freezing point of the solution, with the difference between these two values identified as delta T
subscript f. The graph shows the boiling point of water and the boiling point of the solution, with the difference between these two
values identified as delta T subscript b. Similarly, the difference in the pressure of water and the solution at the boiling point of
water is shown and identified as delta P. This difference in pressure is labeled vapor pressure lowering. The lower level of the vapor
pressure curve for the solution as opposed to that of pure water shows vapor pressure lowering in the solution. Background colors
on the diagram indicate the presence of water and the solution in the solid state to the left, liquid state in the central upper region,
and gas to the right.
The liquid-vapor curve for the solution is located beneath the corresponding curve for the solvent, depicting the vapor pressure
lowering, ΔP, that results from the dissolution of nonvolatile solute. Consequently, at any given pressure, the solution’s boiling
point is observed at a higher temperature than that for the pure solvent, reflecting the boiling point elevation, ΔTb, associated with
the presence of nonvolatile solute. The solid-liquid curve for the solution is displaced left of that for the pure solvent, representing
the freezing point depression, ΔTf, that accompanies solution formation. Finally, notice that the solid-gas curves for the solvent and
its solution are identical. This is the case for many solutions comprising liquid solvents and nonvolatile solutes. Just as for
vaporization, when a solution of this sort is frozen, it is actually just the solvent molecules that undergo the liquid-to-solid
transition, forming pure solid solvent that excludes solute species. The solid and gaseous phases, therefore, are composed solvent
only, and so transitions between these phases are not subject to colligative effects.

Access for free at OpenStax 11.4.10 https://chem.libretexts.org/@go/page/38252


11.4.7: Osmosis and Osmotic Pressure of Solutions
A number of natural and synthetic materials exhibit selective permeation, meaning that only molecules or ions of a certain size,
shape, polarity, charge, and so forth, are capable of passing through (permeating) the material. Biological cell membranes provide
elegant examples of selective permeation in nature, while dialysis tubing used to remove metabolic wastes from blood is a more
simplistic technological example. Regardless of how they may be fabricated, these materials are generally referred to as
semipermeable membranes.
Consider the apparatus illustrated in Figure 11.4.6, in which samples of pure solvent and a solution are separated by a membrane
that only solvent molecules may permeate. Solvent molecules will diffuse across the membrane in both directions. Since the
concentration of solvent is greater in the pure solvent than the solution, these molecules will diffuse from the solvent side of the
membrane to the solution side at a faster rate than they will in the reverse direction. The result is a net transfer of solvent molecules
from the pure solvent to the solution. Diffusion-driven transfer of solvent molecules through a semipermeable membrane is a
process known as osmosis.

Figure 11.4.6 : Osmosis results in the transfer of solvent molecules from a sample of low (or zero) solute concentration to a sample
of higher solute concentration.
The figure shows two U shaped tubes with a semi permeable membrane placed at the base of the U. In figure a, pure solvent is
present and indicated by small yellow spheres to the left of the membrane. To the right, a solution exists with larger blue spheres
intermingled with some small yellow spheres. At the membrane, arrows pointing from three small yellow spheres on both sides of
the membrane cross over the membrane. An arrow drawn from one of the large blue spheres does not cross the membrane, but
rather is reflected back from the surface of the membrane. The levels of liquid in both sides of the U shaped tube are equal. In
figure b, arrows again point from small yellow spheres across the semipermeable membrane from both sides. This diagram shows
the level of liquid in the left, pure solvent, side to be significantly lower than the liquid level on the right. Dashed lines are drawn
from these two liquid levels into the middle of the U-shaped tube and between them is the term osmotic pressure.
When osmosis is carried out in an apparatus like that shown in Figure 11.4.6, the volume of the solution increases as it becomes
diluted by accumulation of solvent. This causes the level of the solution to rise, increasing its hydrostatic pressure (due to the
weight of the column of solution in the tube) and resulting in a faster transfer of solvent molecules back to the pure solvent side.
When the pressure reaches a value that yields a reverse solvent transfer rate equal to the osmosis rate, bulk transfer of solvent
ceases. This pressure is called the osmotic pressure (Π ) of the solution. The osmotic pressure of a dilute solution is related to its
solute molarity, M, and absolute temperature, T, according to the equation

Π = M RT (11.4.10)

where R is the universal gas constant.

 Example 11.4.7: Calculation of Osmotic Pressure

What is the osmotic pressure (atm) of a 0.30 M solution of glucose in water that is used for intravenous infusion at body
temperature, 37 °C?

Access for free at OpenStax 11.4.11 https://chem.libretexts.org/@go/page/38252


Solution
We can find the osmotic pressure, \(Π\), using Equation 11.4.10, where T is on the Kelvin scale (310 K) and the value of R is
expressed in appropriate units (0.08206 L atm/mol K).
Π = M RT

= 0.03 mol/L × 0.08206 L atm/mol K × 310 K

= 7.6 atm

 Exercise 11.4.7

What is the osmotic pressure (atm) a solution with a volume of 0.750 L that contains 5.0 g of methanol, CH3OH, in water at 37
°C?

Answer
5.3 atm

If a solution is placed in an apparatus like the one shown in Figure 11.4.7, applying pressure greater than the osmotic pressure of
the solution reverses the osmosis and pushes solvent molecules from the solution into the pure solvent. This technique of reverse
osmosis is used for large-scale desalination of seawater and on smaller scales to produce high-purity tap water for drinking.

Figure 11.4.7 : Applying a pressure greater than the osmotic pressure of a solution will reverse osmosis. Solvent molecules from the
solution are pushed into the pure solvent.
The figure shows a U shaped tube with a semi permeable membrane placed at the base of the U. Pure solvent is present and
indicated by small yellow spheres to the left of the membrane. To the right, a solution exists with larger blue spheres intermingled
with some small yellow spheres. At the membrane, arrows point from four small yellow spheres to the left of the membrane. On
the right side of the U, there is a disk that is the same width of the tube and appears to block it. The disk is at the same level as the
solution. An arrow points down from the top of the tube to the disk and is labeled, “Pressure greater than Π subscript solution.”
Examples of osmosis are evident in many biological systems because cells are surrounded by semipermeable membranes. Carrots
and celery that have become limp because they have lost water can be made crisp again by placing them in water. Water moves into
the carrot or celery cells by osmosis. A cucumber placed in a concentrated salt solution loses water by osmosis and absorbs some
salt to become a pickle. Osmosis can also affect animal cells. Solute concentrations are particularly important when solutions are
injected into the body. Solutes in body cell fluids and blood serum give these solutions an osmotic pressure of approximately 7.7
atm. Solutions injected into the body must have the same osmotic pressure as blood serum; that is, they should be isotonic with
blood serum. If a less concentrated solution, a hypotonic solution, is injected in sufficient quantity to dilute the blood serum, water
from the diluted serum passes into the blood cells by osmosis, causing the cells to expand and rupture. This process is called

Access for free at OpenStax 11.4.12 https://chem.libretexts.org/@go/page/38252


hemolysis. When a more concentrated solution, a hypertonic solution, is injected, the cells lose water to the more concentrated
solution, shrivel, and possibly die in a process called crenation (Figure 11.5.8).

Figure 11.4.8 : Red blood cell membranes are water permeable and will (a) swell and possibly rupture in a hypotonic solution; (b)
maintain normal volume and shape in an isotonic solution; and (c) shrivel and possibly die in a hypertonic solution. (credit a/b/c:
modifications of work by “LadyofHats”/Wikimedia commons)

This figure shows three scenarios relating to red blood cell membranes. In a, H subscript 2 O has two arrows drawn from it
pointing into a red disk. Beneath it in a circle are eleven similar disks with a bulging appearance, one of which appears to have
burst with blue liquid erupting from it. In b, the image is similar except that rather than having two arrows pointing into the red
disk, one points in and a second points out toward the H subscript 2 O. In the circle beneath, twelve of the red disks are present. In
c, both arrows are drawn from a red shriveled disk toward the H subscript 2 O. In the circle below, twelve shriveled disks are
shown.

11.4.8: Determination of Molar Masses


Osmotic pressure and changes in freezing point, boiling point, and vapor pressure are directly proportional to the concentration of
solute present. Consequently, we can use a measurement of one of these properties to determine the molar mass of the solute from
the measurements.

 Example 11.4.8: Determining Molar Mass from Freezing Point Depression

A solution of 4.00 g of a nonelectrolyte dissolved in 55.0 g of benzene is found to freeze at 2.32 °C. What is the molar mass of
this compound?

Solution
We can solve this problem using the following steps.

1. Determine the change in freezing point from the observed freezing point and the freezing point of pure benzene (Table
11.5.1).

Access for free at OpenStax 11.4.13 https://chem.libretexts.org/@go/page/38252


ΔTf = 5.5 °C − 2.32 °C = 3.2 °C

1. Determine the molal concentration from Kf, the freezing point depression constant for benzene (Table 11.5.1), and ΔTf.
\(ΔT_\ce{f}=K_\ce{f}m\)
ΔTf 3.2 °C
m = = = 0.63 m
−1
Kf 5.12 °Cm

1. Determine the number of moles of compound in the solution from the molal concentration and the mass of solvent used to make
the solution.
0.62 mol solute
Moles of solute = × 0.0550 kg solvent = 0.035 mol
1.00 kg solvent

2. Determine the molar mass from the mass of the solute and the number of moles in that mass.
\(\mathrm{Molar\: mass=\dfrac{4.00\:g}{0.034\:mol}=1.2×10^2\:g/mol}\)

 Exercise 11.4.8

A solution of 35.7 g of a nonelectrolyte in 220.0 g of chloroform has a boiling point of 64.5 °C. What is the molar mass of this
compound?

Answer
1.8 × 102 g/mol

 Example 11.4.9: Determination of a Molar Mass from Osmotic Pressure

A 0.500 L sample of an aqueous solution containing 10.0 g of hemoglobin has an osmotic pressure of 5.9 torr at 22 °C. What is
the molar mass of hemoglobin?

Solution
Here is one set of steps that can be used to solve the problem:

5.9 torr × 1 atm


1. Π = = 7.8 × 10
−3
atm
760 torr

Π = M RT

−3
Π 7.8 × 10 atm
−4
M = = = 3.2 × 10 M
RT (0.08206 L atm/mol K)(295 K)

−4
3.2 × 10 mol
1. moles of hemoglobin = × 0.500 L solution = 1.6 × 10
−4
mol
1 L solution

2. Determine the molar mass from the mass of hemoglobin and the number of moles in that mass.
10.0 g
4
molar mass = = 6.2 × 10 g/mol
−4
1.6 × 10 mol

Access for free at OpenStax 11.4.14 https://chem.libretexts.org/@go/page/38252


 Exercise 11.4.9

What is the molar mass of a protein if a solution of 0.02 g of the protein in 25.0 mL of solution has an osmotic pressure of 0.56
torr at 25 °C?

Answer
2.7 × 104 g/mol

11.4.9: Colligative Properties of Electrolytes


As noted previously in this module, the colligative properties of a solution depend only on the number, not on the kind, of solute
species dissolved. For example, 1 mole of any nonelectrolyte dissolved in 1 kilogram of solvent produces the same lowering of the
freezing point as does 1 mole of any other nonelectrolyte. However, 1 mole of sodium chloride (an electrolyte) forms 2 moles of
ions when dissolved in solution. Each individual ion produces the same effect on the freezing point as a single molecule does.

 Example 11.4.10: The Freezing Point of a Solution of an Electrolyte

The concentration of ions in seawater is approximately the same as that in a solution containing 4.2 g of NaCl dissolved in 125
g of water. Assume that each of the ions in the NaCl solution has the same effect on the freezing point of water as a
nonelectrolyte molecule, and determine the freezing temperature the solution (which is approximately equal to the freezing
temperature of seawater).

Solution
We can solve this problem using the following series of steps.
Convert from grams to moles of NaCl using the molar mass of NaCl in the unit conversion factor. Result: 0.072 mol NaCl
Determine the number of moles of ions present in the solution using the number of moles of ions in 1 mole of NaCl as the
conversion factor (2 mol ions/1 mol NaCl). Result: 0.14 mol ions
Determine the molality of the ions in the solution from the number of moles of ions and the mass of solvent, in kilograms.
Result: 1.1 m
Use the direct proportionality between the change in freezing point and molal concentration to determine how much the
freezing point changes. Result: 2.0 °C
Determine the new freezing point from the freezing point of the pure solvent and the change. Result: −2.0 °C
Check each result as a self-assessment.

 Exercise 11.4.10

Assume that each of the ions in calcium chloride, CaCl2, has the same effect on the freezing point of water as a nonelectrolyte
molecule. Calculate the freezing point of a solution of 0.724 g of CaCl2 in 175 g of water.

Answer
−0.208 °C

Assuming complete dissociation, a 1.0 m aqueous solution of NaCl contains 2.0 mole of ions (1.0 mol Na+ and 1.0 mol Cl−) per
each kilogram of water, and its freezing point depression is expected to be
ΔTf = 2.0 mol ions/kg water × 1.86 °C kg water/mol ion = 3.7 °C. (11.4.11)

When this solution is actually prepared and its freezing point depression measured, however, a value of 3.4 °C is obtained. Similar
discrepancies are observed for other ionic compounds, and the differences between the measured and expected colligative property

Access for free at OpenStax 11.4.15 https://chem.libretexts.org/@go/page/38252


values typically become more significant as solute concentrations increase. These observations suggest that the ions of sodium
chloride (and other strong electrolytes) are not completely dissociated in solution.
To account for this and avoid the errors accompanying the assumption of total dissociation, an experimentally measured parameter
named in honor of Nobel Prize-winning German chemist Jacobus Henricus van’t Hoff is used. The van’t Hoff factor (i) is defined
as the ratio of solute particles in solution to the number of formula units dissolved:
moles of particles in solution
i = (11.4.12)
moles of formula units dissolved

Values for measured van’t Hoff factors for several solutes, along with predicted values assuming complete dissociation, are shown
in Table 11.4.2.
Table 11.4.2 : Expected and Observed van’t Hoff Factors for Several 0.050 m Aqueous Electrolyte Solutions
Electrolyte Particles in Solution i (Predicted) i (Measured)

HCl H+, Cl− 2 1.9

NaCl Na+, Cl− 2 1.9

MgSO4 Mg2+, SO
2−
4
2 1.3

MgCl2 Mg2+, 2Cl− 3 2.7

FeCl3 Fe3+, 3Cl− 4 3.4

glucose (a non-electrolyte) C12H22O11 1 1.0

In 1923, the chemists Peter Debye and Erich Hückel proposed a theory to explain the apparent incomplete ionization of strong
electrolytes. They suggested that although interionic attraction in an aqueous solution is very greatly reduced by solvation of the
ions and the insulating action of the polar solvent, it is not completely nullified. The residual attractions prevent the ions from
behaving as totally independent particles (Figure 11.4.9). In some cases, a positive and negative ion may actually touch, giving a
solvated unit called an ion pair. Thus, the activity, or the effective concentration, of any particular kind of ion is less than that
indicated by the actual concentration. Ions become more and more widely separated the more dilute the solution, and the residual
interionic attractions become less and less. Thus, in extremely dilute solutions, the effective concentrations of the ions (their
activities) are essentially equal to the actual concentrations. Note that the van’t Hoff factors for the electrolytes in Table 11.4.2 are
for 0.05 m solutions, at which concentration the value of i for NaCl is 1.9, as opposed to an ideal value of 2.

Figure 11.4.9 : Ions become more and more widely separated the more dilute the solution, and the residual interionic attractions
become less.
The diagram shows four purple spheres labeled K superscript plus and four green spheres labeled C l superscript minus dispersed in
H subscript 2 O as shown by clusters of single red spheres with two white spheres attached. Red spheres represent oxygen and
white represent hydrogen. In two locations, the purple and green spheres are touching. In these two locations, the diagram is
labeled ion pair. All red and green spheres are surrounded by the white and red H subscript 2 O clusters. The white spheres are
attracted to the purple spheres and the red spheres are attracted to the green spheres.

Access for free at OpenStax 11.4.16 https://chem.libretexts.org/@go/page/38252


Summary
Properties of a solution that depend only on the concentration of solute particles are called colligative properties. They include
changes in the vapor pressure, boiling point, and freezing point of the solvent in the solution. The magnitudes of these properties
depend only on the total concentration of solute particles in solution, not on the type of particles. The total concentration of solute
particles in a solution also determines its osmotic pressure. This is the pressure that must be applied to the solution to prevent
diffusion of molecules of pure solvent through a semipermeable membrane into the solution. Ionic compounds may not completely
dissociate in solution due to activity effects, in which case observed colligative effects may be less than predicted.

11.4.10: Key Equations



(PA = XA P )
A

Psolution = ∑ Pi = ∑ Xi P
i i i

Psolution = Xsolvent P
solvent

ΔTb = Kbm
ΔTf = Kfm
Π = MRT

Footnotes
1. A nonelectrolyte shown for comparison.

Glossary
boiling point elevation
elevation of the boiling point of a liquid by addition of a solute

boiling point elevation constant


the proportionality constant in the equation relating boiling point elevation to solute molality; also known as the ebullioscopic
constant

colligative property
property of a solution that depends only on the concentration of a solute species

crenation
process whereby biological cells become shriveled due to loss of water by osmosis

freezing point depression


lowering of the freezing point of a liquid by addition of a solute

freezing point depression constant


(also, cryoscopic constant) proportionality constant in the equation relating freezing point depression to solute molality

hemolysis
rupture of red blood cells due to the accumulation of excess water by osmosis

hypertonic
of greater osmotic pressure

hypotonic
of less osmotic pressure

ion pair
solvated anion/cation pair held together by moderate electrostatic attraction

isotonic
of equal osmotic pressure

Access for free at OpenStax 11.4.17 https://chem.libretexts.org/@go/page/38252


molality (m)
a concentration unit defined as the ratio of the numbers of moles of solute to the mass of the solvent in kilograms

osmosis
diffusion of solvent molecules through a semipermeable membrane

osmotic pressure (Π)


opposing pressure required to prevent bulk transfer of solvent molecules through a semipermeable membrane

Raoult’s law
the partial pressure exerted by a solution component is equal to the product of the component’s mole fraction in the solution and
its equilibrium vapor pressure in the pure state

semipermeable membrane
a membrane that selectively permits passage of certain ions or molecules

van’t Hoff factor (i)


the ratio of the number of moles of particles in a solution to the number of moles of formula units dissolved in the solution

This page titled 11.4: Colligative Properties is shared under a CC BY 4.0 license and was authored, remixed, and/or curated by OpenStax via
source content that was edited to the style and standards of the LibreTexts platform; a detailed edit history is available upon request.

Access for free at OpenStax 11.4.18 https://chem.libretexts.org/@go/page/38252


11.5: Colloids
 Learning Objectives
Describe the composition and properties of colloidal dispersions
List and explain several technological applications of colloids

As a child, you may have made suspensions such as mixtures of mud and water, flour and water, or a suspension of solid pigments
in water, known as tempera paint. These suspensions are heterogeneous mixtures composed of relatively large particles that are
visible (or that can be seen with a magnifying glass). They are cloudy, and the suspended particles settle out after mixing. On the
other hand, when we make a solution, we prepare a homogeneous mixture in which no settling occurs and in which the dissolved
species are molecules or ions. Solutions exhibit completely different behavior from suspensions. A solution may be colored, but it
is transparent, the molecules or ions are invisible, and they do not settle out on standing. A group of mixtures called colloids (or
colloidal dispersions) exhibit properties intermediate between those of suspensions and solutions (Figure 11.5.1). The particles in a
colloid are larger than most simple molecules; however, colloidal particles are small enough that they do not settle out upon
standing.

Figure 11.5.1 : (a) A solution is a homogeneous mixture that appears clear, such as the saltwater in this aquarium. (b) In a colloid,
such as milk, the particles are much larger but remain dispersed and do not settle. (c) A suspension, such as mud, is a
heterogeneous mixture of suspended particles that appears cloudy and in which the particles can settle. (credit a photo:
modification of work by Adam Wimsatt; credit b photo: modification of work by Melissa Wiese; credit c photo: modification of
work by Peter Burgess)
This figure contains three photos and correponding particle diagrams. In a, a photo of an aquarium containing fish is shown. The
particle diagram beneath it shows 90 tiny red spheres. In b, a photo is shown of milk being poured into a cup. The corresponding
particle diagram shows about 25 medium sized red spheres.In c, a photo is shown of two pairs of sandal clad feet in mud. The
particle diagram below shows 10 fairly large red spheres.
The particles in a colloid are large enough to scatter light, a phenomenon called the Tyndall effect. This can make colloidal
mixtures appear cloudy or opaque, such as the searchlight beams shown in Figure 11.5.2. Clouds are colloidal mixtures. They are
composed of water droplets that are much larger than molecules, but that are small enough that they do not settle out.

Access for free at OpenStax 11.5.1 https://chem.libretexts.org/@go/page/38253


Figure 11.5.2 : The paths of searchlight beams are made visible when light is scattered by colloidal-size particles in the air (fog,
smoke, etc.). (credit: “Bahman”/Wikimedia Commons)
The term “colloid”—from the Greek words kolla, meaning “glue,” and eidos, meaning “like”—was first used in 1861 by Thomas
Graham to classify mixtures such as starch in water and gelatin. Many colloidal particles are aggregates of hundreds or thousands
of molecules, but others (such as proteins and polymer molecules) consist of a single extremely large molecule. The protein and
synthetic polymer molecules that form colloids may have molecular masses ranging from a few thousand to many million atomic
mass units.
Analogous to the identification of solution components as “solute” and “solvent,” the components of a colloid are likewise
classified according to their relative amounts. The particulate component typically present in a relatively minor amount is called the
dispersed phase and the substance or solution throughout which the particulate is dispersed is called the dispersion medium.
Colloids may involve virtually any combination of physical states (gas in liquid, liquid in solid, solid in gas, etc.), as illustrated by
the examples of colloidal systems given in Table 11.5.1.
Table 11.5.2 : Examples of Colloidal Systems
Dispersed Phase Dispersion Medium Common Examples Name

solid gas smoke, dust —

starch in water, some inks, paints,


solid liquid sol
milk of magnesia

solid solid some colored gems, some alloys —

liquid gas clouds, fogs, mists, sprays aerosol

liquid liquid milk, mayonnaise, butter emulsion

jellies, gels, pearl, opal (H2O in


liquid solid gel
SiO2)

foams, whipped cream, beaten egg


gas liquid foam
whites

gas solid pumice, floating soaps —

11.5.1: Preparation of Colloidal Systems


We can prepare a colloidal system by producing particles of colloidal dimensions and distributing these particles throughout a
dispersion medium. Particles of colloidal size are formed by two methods:
1. Dispersion methods: that is, by breaking down larger particles. For example, paint pigments are produced by dispersing large
particles by grinding in special mills.
2. Condensation methods: that is, growth from smaller units, such as molecules or ions. For example, clouds form when water
molecules condense and form very small droplets.

Access for free at OpenStax 11.5.2 https://chem.libretexts.org/@go/page/38253


A few solid substances, when brought into contact with water, disperse spontaneously and form colloidal systems. Gelatin, glue,
starch, and dehydrated milk powder behave in this manner. The particles are already of colloidal size; the water simply disperses
them. Powdered milk particles of colloidal size are produced by dehydrating milk spray. Some atomizers produce colloidal
dispersions of a liquid in air.
We can prepare an emulsion by shaking together or blending two immiscible liquids. This breaks one liquid into droplets of
colloidal size, which then disperse throughout the other liquid. Oil spills in the ocean may be difficult to clean up, partly because
wave action can cause the oil and water to form an emulsion. In many emulsions, however, the dispersed phase tends to coalesce,
form large drops, and separate. Therefore, emulsions are usually stabilized by an emulsifying agent, a substance that inhibits the
coalescence of the dispersed liquid. For example, a little soap will stabilize an emulsion of kerosene in water. Milk is an emulsion
of butterfat in water, with the protein casein as the emulsifying agent. Mayonnaise is an emulsion of oil in vinegar, with egg yolk
components as the emulsifying agents.
Condensation methods form colloidal particles by aggregation of molecules or ions. If the particles grow beyond the colloidal size
range, drops or precipitates form, and no colloidal system results. Clouds form when water molecules aggregate and form colloid-
sized particles. If these water particles coalesce to form adequately large water drops of liquid water or crystals of solid water, they
settle from the sky as rain, sleet, or snow. Many condensation methods involve chemical reactions. We can prepare a red colloidal
suspension of iron(III) hydroxide by mixing a concentrated solution of iron(III) chloride with hot water:
3+ − + −
Fe (aq)
+ 3Cl (aq)
+ 6 H2 O(l) ⟶ Fe(OH) + H3 O (aq)
+ 3Cl (aq)
(11.5.1)
3(s)

A colloidal gold sol results from the reduction of a very dilute solution of gold(III) chloride by a reducing agent such as
formaldehyde, tin(II) chloride, or iron(II) sulfate:
3+ −
Au +3 e → Au (11.5.2)

Some gold sols prepared in 1857 are still intact (the particles have not coalesced and settled), illustrating the long-term stability of
many colloids.

11.5.2: Soaps and Detergents


Pioneers made soap by boiling fats with a strongly basic solution made by leaching potassium carbonate, K CO , from wood 2 3

ashes with hot water. Animal fats contain polyesters of fatty acids (long-chain carboxylic acids). When animal fats are treated with
a base like potassium carbonate or sodium hydroxide, glycerol and salts of fatty acids such as palmitic, oleic, and stearic acid are
formed. The salts of fatty acids are called soaps. The sodium salt of stearic acid, sodium stearate, has the formula C H CO Na 17 35 2

and contains an uncharged nonpolar hydrocarbon chain, the C H − unit, and an ionic carboxylate group, the −CO unit
17 35

2

(Figure 11.5.3).

Figure 11.5.3 : Soaps contain a nonpolar hydrocarbon end (blue) and an ionic end (red). The ionic end is a carboxylate group. The
length of the hydrocarbon end can vary from soap to soap.
This figure shows a structural formula for soap known as sodium stearate. A hydrocarbon chain composed of 18 carbon atoms and
35 hydrogen atoms is shown with an ionic end with 2 oxygen atoms and a negative charge. A positively charged N a superscript
plus is also shown at the ionic end.
Detergents (soap substitutes) also contain nonpolar hydrocarbon chains, such as C H —, and an ionic group, such as a sulfate
12 25

— OSO , or a sulfonate — SO (Figure 11.5.4). Soaps form insoluble calcium and magnesium compounds in hard water;
− −
3 3

detergents form water-soluble products—a definite advantage for detergents.

Figure 11.5.4 : Detergents contain a nonpolar hydrocarbon end (blue) and an ionic end (red). The ionic end can be either a sulfate or
a sulfonate. The length of the hydrocarbon end can vary from detergent to detergent.
This figure shows a structural formula for a detergent known as sodium lauryl sulfate. A hydrocarbon chain composed of 12 carbon
atoms and 25 hydrogen atoms is shown with an ionic end involving a negatively charged sulfur and four oxygen atoms at the ionic
end of the chain. A positively charged N a superscript plus is also shown at the ionic end.

Access for free at OpenStax 11.5.3 https://chem.libretexts.org/@go/page/38253


The cleaning action of soaps and detergents can be explained in terms of the structures of the molecules involved. The hydrocarbon
(nonpolar) end of a soap or detergent molecule dissolves in, or is attracted to, nonpolar substances such as oil, grease, or dirt
particles. The ionic end is attracted by water (polar), illustrated in Figure 11.5.5. As a result, the soap or detergent molecules
become oriented at the interface between the dirt particles and the water so they act as a kind of bridge between two different kinds
of matter, nonpolar and polar. Molecules such as this are termed amphiphilic since they have both a hydrophobic (“water-fearing”)
part and a hydrophilic (“water-loving”) part. As a consequence, dirt particles become suspended as colloidal particles and are
readily washed away.

Figure 11.5.5 : This diagrammatic cross section of an emulsified drop of oil in water shows how soap or detergent acts as an
emulsifier.
This figure shows a drop of oil in which approximately thirty hydrocarbon tails are oriented toward the center of the drop with
ionic ends indicated as tiny red spheres on the surface of the oil drop. Solvated cations are indicated as purple spheres surrounded
by clusters of H subscript 2 subscript O molecules shown as tiny clusters of red central oxygen spheres with two white hydrogen
spheres attached.

 Deepwater Horizon Oil Spill

The blowout of the Deepwater Horizon oil drilling rig on April 20, 2010, in the Gulf of Mexico near Mississippi began the
largest marine oil spill in the history of the petroleum. In the 87 days following the blowout, an estimated 4.9 million barrels
(210 million gallons) of oil flowed from the ruptured well 5000 feet below the water’s surface. The well was finally declared
sealed on September 19, 2010.
Crude oil is immiscible with and less dense than water, so the spilled oil rose to the surface of the water. Floating booms,
skimmer ships, and controlled burns were used to remove oil from the water’s surface in an attempt to protect beaches and
wetlands along the Gulf coast. In addition to removal of the oil, attempts were also made to lessen its environmental impact by
rendering it “soluble” (in the loose sense of the term) and thus allowing it to be diluted to hopefully less harmful levels by the
vast volume of ocean water. This approach used 1.84 million gallons of the oil dispersant Corexit 9527, most of which was
injected underwater at the site of the leak, with small amounts being sprayed on top of the spill. Corexit 9527 contains 2-
butoxyethanol (C6H14O2), an amphiphilic molecule whose polar and nonpolar ends are useful for emulsifying oil into small
droplets, increasing the surface area of the oil and making it more available to marine bacteria for digestion (Figure 11.5.6).
While this approach avoids many of the immediate hazards that bulk oil poses to marine and coastal ecosystems, it introduces
the possibility of long-term effects resulting from the introduction of the complex and potential toxic components of petroleum
into the ocean’s food chain. A number of organizations are involved in monitoring the extended impact of this oil spill,
including the National Oceanic and Atmospheric Administration.

Access for free at OpenStax 11.5.4 https://chem.libretexts.org/@go/page/38253


Figure 11.5.6 : (a) This NASA satellite image shows the oil slick from the Deepwater Horizon spill. (b) A US Air Force plane
sprays Corexit, a dispersant. (c) The molecular structure of 2-butoxyethanol is shown. (credit a: modification of work by
“NASA, FT2, demis.nl”/Wikimedia Commons; credit b: modification of work by “NASA/MODIS Rapid Response
Team”/Wikimedia Commons)

11.5.3: Electrical Properties of Colloidal Particles


Dispersed colloidal particles are often electrically charged. A colloidal particle of iron(III) hydroxide, for example, does not contain
enough hydroxide ions to compensate exactly for the positive charges on the iron(III) ions. Thus, each individual colloidal particle
bears a positive charge, and the colloidal dispersion consists of charged colloidal particles and some free hydroxide ions, which
keep the dispersion electrically neutral. Most metal hydroxide colloids have positive charges, whereas most metals and metal
sulfides form negatively charged dispersions. All colloidal particles in any one system have charges of the same sign. This helps
keep them dispersed because particles containing like charges repel each other.
We can take advantage of the charge on colloidal particles to remove them from a variety of mixtures. If we place a colloidal
dispersion in a container with charged electrodes, positively charged particles, such as iron(III) hydroxide particles, would move to
the negative electrode. There, the colloidal particles lose their charge and coagulate as a precipitate.

Figure 11.5.7 : In a Cottrell precipitator, positively and negatively charged particles are attracted to highly charged electrodes,
where they are neutralized and deposited as dust.
This figure shows a diagram of a Cottrell precipitator. An arrow pointing into a cylindrical chamber shows the path of soot laden
smoke. In the presence of high DC voltage and both point and plate electrodes, soot particles are removed at the bottom of the
chamber and soot free air exits the top. A photo shows the honeycomb electrodes of a modern electrostatic precipitator.
The carbon and dust particles in smoke are often colloidally dispersed and electrically charged. The charged particles are attracted
to highly charged electrodes, where they are neutralized and deposited as dust (Figure 11.5.7. This is one of the important methods
used to clean up the smoke from a variety of industrial processes. The process is also important in the recovery of valuable
products from the smoke and flue dust of smelters, furnaces, and kilns. There are also ionic air filters designed for home use to
improve indoor air quality.

Access for free at OpenStax 11.5.5 https://chem.libretexts.org/@go/page/38253


11.5.4: Gels
When we make gelatin, such as Jell-O, we are making a type of colloid (Figure 11.5.8). Gelatin sets on cooling because the hot
aqueous mixture of gelatin coagulates as it cools and the whole mass, including the liquid, sets to an extremely viscous body
known as a gel, a colloid in which the dispersing medium is a solid and the dispersed phase is a liquid. It appears that the fibers of
the dispersing medium form a complex three-dimensional network, the interstices being filled with the liquid medium or a dilute
solution of the dispersing medium. Because the formation of a gel is accompanied by the taking up of water or some other solvent,
the gel is said to be hydrated or solvated.

Figure 11.5.8 : Gelatin desserts are colloids in which an aqueous solution of sweeteners and flavors is dispersed throughout a
medium of solid proteins. (credit photo: modification of work by Steven Depolo).
In this figure, a large molecular model of gelatin is shown with black carbon atoms, red oxygen atoms, white hydrogen atoms, and
blue nitrogen atoms. A photo is shown of gelatin dessert cut into colorful rectangles.
Pectin, a carbohydrate from fruit juices, is a gel-forming substance important in jelly making. Silica gel, a colloidal dispersion of
hydrated silicon dioxide, is formed when dilute hydrochloric acid is added to a dilute solution of sodium silicate. Canned Heat is a
gel made by mixing alcohol and a saturated aqueous solution of calcium acetate.

Summary
Colloids are mixtures in which one or more substances are dispersed as relatively large solid particles or liquid droplets throughout
a solid, liquid, or gaseous medium. The particles of a colloid remain dispersed and do not settle due to gravity, and they are often
electrically charged. Colloids are widespread in nature and are involved in many technological applications.

Glossary
amphiphilic
molecules possessing both hydrophobic (nonpolar) and a hydrophilic (polar) parts

colloid
(also, colloidal dispersion) mixture in which relatively large solid or liquid particles are dispersed uniformly throughout a gas,
liquid, or solid

dispersion medium
solid, liquid, or gas in which colloidal particles are dispersed

dispersed phase
substance present as relatively large solid or liquid particles in a colloid

emulsifying agent

Access for free at OpenStax 11.5.6 https://chem.libretexts.org/@go/page/38253


amphiphilic substance used to stabilize the particles of some emulsions

emulsion
colloid formed from immiscible liquids

gel
colloidal dispersion of a liquid in a solid

Tyndall effect
scattering of visible light by a colloidal dispersion

This page titled 11.5: Colloids is shared under a CC BY 4.0 license and was authored, remixed, and/or curated by OpenStax via source content
that was edited to the style and standards of the LibreTexts platform; a detailed edit history is available upon request.

Access for free at OpenStax 11.5.7 https://chem.libretexts.org/@go/page/38253


11.E: Solutions and Colloids (Exercises)
11.E.1: 11.2: The Dissolution Process
11.E.1.1: Q11.2.1
How do solutions differ from compounds? From other mixtures?

11.E.1.2: S11.2.1
A solution can vary in composition, while a compound cannot vary in composition. Solutions are homogeneous at the molecular
level, while other mixtures are heterogeneous.

11.E.1.3: Q11.2.2
Which of the principal characteristics of solutions can we see in the solutions of K 2
Cr O
2 7
shown in

Figure: When potassium dichromate (K 2


Cr O
2
) is mixed with water, it forms a homogeneous orange solution. (credit:
7

modification of work by Mark Ott)

11.E.1.4: S11.2.2
The solutions are the same throughout (the color is constant throughout), and the composition of a solution of K2Cr2O7 in water
can vary.

11.E.1.5: Q11.2.3
When KNO3 is dissolved in water, the resulting solution is significantly colder than the water was originally.
a. Is the dissolution of KNO3 an endothermic or an exothermic process?
b. What conclusions can you draw about the intermolecular attractions involved in the process?
c. Is the resulting solution an ideal solution?

11.E.1.6: S11.2.3
(a) The process is endothermic as the solution is consuming heat. (b) Attraction between the K+ and NO ions is stronger than

between the ions and water molecules (the ion-ion interactions have a lower, more negative energy). Therefore, the dissolution
process increases the energy of the molecular interactions, and it consumes the thermal energy of the solution to make up for the
difference. (c) No, an ideal solution is formed with no appreciable heat release or consumption.

11.E.1.7: Q11.2.4
Give an example of each of the following types of solutions:
a. a gas in a liquid
b. a gas in a gas
c. a solid in a solid

11.E.1.8: S11.2.4
(a) CO2 in water; (b) O2 in N2 (air); (c) bronze (solution of tin or other metals in copper)

11.E.1.9: Q11.2.5
Indicate the most important types of intermolecular attractions in each of the following solutions:
a. The solution in Figure.

Access for free at OpenStax 11.E.1 https://chem.libretexts.org/@go/page/44117


b. NO(l) in CO(l)
c. Cl2(g) in Br2(l)
d. HCl(aq) in benzene C6H6(l)
e. Methanol CH3OH(l) in H2O(l)

11.E.1.10: S11.2.5
(a) ion-dipole forces; (b) dipole-dipole forces; (c) dispersion forces; (d) dispersion forces; (e) hydrogen bonding

11.E.1.11: Q11.2.5
Predict whether each of the following substances would be more soluble in water (polar solvent) or in a hydrocarbon such as
heptane (C7H16, nonpolar solvent):
a. vegetable oil (nonpolar)
b. isopropyl alcohol (polar)
c. potassium bromide (ionic)

11.E.1.12: S11.2.5
(a) heptane; (b) water; (c) water

11.E.1.13: Q11.2.6
Heat is released when some solutions form; heat is absorbed when other solutions form. Provide a molecular explanation for the
difference between these two types of spontaneous processes.

11.E.1.14: S11.2.6
Heat is released when the total intermolecular forces (IMFs) between the solute and solvent molecules are stronger than the total
IMFs in the pure solute and in the pure solvent: Breaking weaker IMFs and forming stronger IMFs releases heat. Heat is absorbed
when the total IMFs in the solution are weaker than the total of those in the pure solute and in the pure solvent: Breaking stronger
IMFs and forming weaker IMFs absorbs heat.

11.E.1.15: Q11.2.7
Solutions of hydrogen in palladium may be formed by exposing Pd metal to H2 gas. The concentration of hydrogen in the
palladium depends on the pressure of H2 gas applied, but in a more complex fashion than can be described by Henry’s law. Under
certain conditions, 0.94 g of hydrogen gas is dissolved in 215 g of palladium metal.
a. Determine the molarity of this solution (solution density = 1.8 g/cm3).
b. Determine the molality of this solution (solution density = 1.8 g/cm3).
c. Determine the percent by mass of hydrogen atoms in this solution (solution density = 1.8 g/cm3).

11.E.1.16: S11.2.7
http://cnx.org/contents/mH6aqegx@2/The-Dissolution-Process

11.E.2: 11.3: Electrolytes


11.E.2.1: Q11.3.1
Explain why the ions Na+ and Cl− are strongly solvated in water but not in hexane, a solvent composed of nonpolar molecules.

11.E.2.2: S11.3.2
Crystals of NaCl dissolve in water, a polar liquid with a very large dipole moment, and the individual ions become strongly
solvated. Hexane is a nonpolar liquid with a dipole moment of zero and, therefore, does not significantly interact with the ions of
the NaCl crystals.

11.E.2.3: Q11.3.2
Explain why solutions of HBr in benzene (a nonpolar solvent) are nonconductive, while solutions in water (a polar solvent) are
conductive.

Access for free at OpenStax 11.E.2 https://chem.libretexts.org/@go/page/44117


11.E.2.4: S11.3.2
HBr is an acid and so its molecules react with water molecules to form H3O+ and Br− ions that provide conductivity to the solution.
Though HBr is soluble in benzene, it does not react chemically but remains dissolved as neutral HBr molecules. With no ions
present in the benzene solution, it is electrically nonconductive.

11.E.2.5: Q11.3.3
Consider the solutions presented:
(a) Which of the following sketches best represents the ions in a solution of Fe(NO3)3(aq)?

(b) Write a balanced chemical equation showing the products of the dissolution of Fe(NO3)3.

11.E.2.6: S11.3.3
(a) Fe(NO3)3 is a strong electrolyte, thus it should completely dissociate into Fe3+ and (NO −

3
) ions. Therefore, (z) best represents
the solution. (b) Fe(NO ) (s) ⟶ Fe (aq) + 3 NO (aq)
3 3
3+ −

11.E.2.7: Q11.3.4
Compare the processes that occur when methanol (CH3OH), hydrogen chloride (HCl), and sodium hydroxide (NaOH) dissolve in
water. Write equations and prepare sketches showing the form in which each of these compounds is present in its respective
solution.

11.E.2.8: S11.3.3
Methanol, C H 3 OH , dissolves in water in all proportions, interacting via hydrogen bonding.
Methanol:

C H3 OH(l) + H2 O(l) ⟶ C H3 OH(aq) (11.E.1)

Hydrogen chloride, HCl, dissolves in and reacts with water to yield hydronium cations and chloride anions that are solvated by
strong ion-dipole interactions.
Hydrogen chloride:
+ −
H C l(g) + H2 O(l) → H3 O + Cl (11.E.2)
(aq) (aq)

Sodium hydroxide, NaOH, dissolves in water and dissociates to yield sodium cations and hydroxide anions that are strongly
solvated by ion-dipole interactions and hydrogen bonding, respectively.
Sodium hydroxide:
+ −
N aOH(s) → N a + OH (11.E.3)
(aq) (aq)

Access for free at OpenStax 11.E.3 https://chem.libretexts.org/@go/page/44117


11.E.2.9: Q11.3.5
What is the expected electrical conductivity of the following solutions?
a. NaOH(aq)
b. HCl(aq)
c. C6H12O6(aq) (glucose)
d. NH3(l)

11.E.2.10: S11.3.5
(a) high conductivity (solute is an ionic compound that will dissociate when dissolved); (b) high conductivity (solute is a strong
acid and will ionize completely when dissolved); (c) nonconductive (solute is a covalent compound, neither acid nor base,
unreactive towards water); (d) low conductivity (solute is a weak base and will partially ionize when dissolved)

11.E.2.11: Q11.3.6
Why are most solid ionic compounds electrically nonconductive, whereas aqueous solutions of ionic compounds are good
conductors? Would you expect a liquid (molten) ionic compound to be electrically conductive or nonconductive? Explain.

11.E.2.12: S11.3.6
A medium must contain freely mobile, charged entities to be electrically conductive. The ions present in a typical ionic solid are
immobilized in a crystalline lattice and so the solid is not able to support an electrical current. When the ions are mobilized, either
by melting the solid or dissolving it in water to dissociate the ions, current may flow and these forms of the ionic compound are
conductive.

11.E.2.13: Q11.3.7
Indicate the most important type of intermolecular attraction responsible for solvation in each of the following solutions:
a. the solutions in Figure
b. methanol, CH3OH, dissolved in ethanol, C2H5OH
c. methane, CH4, dissolved in benzene, C6H6
d. the polar halocarbon CF2Cl2 dissolved in the polar halocarbon CF2ClCFCl2
e. O2(l) in N2(l)

11.E.2.14: S11.3.7
(a) ion-dipole; (b) hydrogen bonds; (c) dispersion forces; (d) dipole-dipole attractions; (e) dispersion forces

11.E.3: 11.4: Solubility


11.E.3.1: Q11.4.1
Suppose you are presented with a clear solution of sodium thiosulfate, Na2S2O3. How could you determine whether the solution is
unsaturated, saturated, or supersaturated?

11.E.3.2: S11.4.1
Add a small crystal of N a S O . It will dissolve in an unsaturated solution, remain apparently unchanged in a saturated solution,
2 2 3

or initiate precipitation in a supersaturated solution.

11.E.3.3: Q11.4.2
Supersaturated solutions of most solids in water are prepared by cooling saturated solutions. Supersaturated solutions of most gases
in water are prepared by heating saturated solutions. Explain the reasons for the difference in the two procedures.

11.E.3.4: S11.4.2
The solubility of solids usually decreases upon cooling a solution, while the solubility of gases usually decreases upon heating.

11.E.3.5: Q11.4.3
Suggest an explanation for the observations that ethanol, C2H5OH, is completely miscible with water and that ethanethiol,
C2H5SH, is soluble only to the extent of 1.5 g per 100 mL of water.

Access for free at OpenStax 11.E.4 https://chem.libretexts.org/@go/page/44117


11.E.3.6: S11.4.3
The hydrogen bonds between water and C2H5OH are much stronger than the intermolecular attractions between water and
C2H5SH.

11.E.3.7: Q11.4.4
Calculate the percent by mass of KBr in a saturated solution of KBr in water at 10 °C using the following figure for useful data,
and report the computed percentage to one significant digit.

This graph shows how the solubility of several solids changes with temperature.

11.E.3.8: S11.4.4
At 10 °C, the solubility of KBr in water is approximately 60 g per 100 g of water.
60 g KBr
% KBr = = 40% (11.E.4)
(60 + 100) g solution

11.E.3.9: Q11.4.5
Which of the following gases is expected to be most soluble in water? Explain your reasoning.
a. CH4
b. CCl4
c. CHCl3

11.E.3.10: S11.4.5
(c) CHCl3 is expected to be most soluble in water. Of the three gases, only this one is polar and thus capable of experiencing
relatively strong dipole-dipole attraction to water molecules.

11.E.3.11: Q11.4.6
At 0 °C and 1.00 atm, as much as 0.70 g of O2 can dissolve in 1 L of water. At 0 °C and 4.00 atm, how many grams of O2 dissolve
in 1 L of water?

11.E.3.12: S11.4.6
This problem requires the application of Henry’s law. The governing equation is C g = kPg .
Cg 0.70 g
−1
k = = = 0.70 g atm (11.E.5)
Pg 1.00 atm

Under the new conditions, C g = 0.70 g atm


−1
× 4.00 atm = 2.80 g .

Access for free at OpenStax 11.E.5 https://chem.libretexts.org/@go/page/44117


11.E.3.13: Q11.4.7
Refer to following figure for the following three questions:

a. How did the concentration of dissolved CO2 in the beverage change when the bottle was opened?
b. What caused this change?
c. Is the beverage unsaturated, saturated, or supersaturated with CO2?

11.E.3.14: S11.4.7
(a) It decreased as some of the CO2 gas left the solution (evidenced by effervescence). (b) Opening the bottle released the high-
pressure CO2 gas above the beverage. The reduced CO2 gas pressure, per Henry’s law, lowers the solubility for CO2. (c) The
dissolved CO2 concentration will continue to slowly decrease until equilibrium is reestablished between the beverage and the very
low CO2 gas pressure in the opened bottle. Immediately after opening, the beverage, therefore, contains dissolved CO2 at a
concentration greater than its solubility, a nonequilibrium condition, and is said to be supersaturated.

11.E.3.15: Q11.4.8
The Henry’s law constant for CO2 is 3.4 × 10 M /atm at 25 °C. What pressure of carbon dioxide is needed to maintain a CO2
−2

concentration of 0.10 M in a can of lemon-lime soda?

11.E.3.16: S11.4.8
Cg 0.10 M
Pg = = = 2.9 atm (11.E.6)
−2
k 3.4 × 10 M /atm

11.E.3.17: Q11.4.9
The Henry’s law constant for O2 is 1.3 × 10 M /atm at 25 °C. What mass of oxygen would be dissolved in a 40-L aquarium at
−3

25 °C, assuming an atmospheric pressure of 1.00 atm, and that the partial pressure of O2 is 0.21 atm?

11.E.3.18: S11.4.9
Start with Henry's law
Cg = kPg (11.E.7)

and apply it to O 2

−3 −4
C (O2 ) = (1.3 × 10 M /atm)(0.21 atm) = 2.7 × 10 mol/L (11.E.8)

The total amount is \((2.7 \times 10^{−4}\; mol/L)(40\;L=1.08 \times 10^{−2} \;mol\]

Access for free at OpenStax 11.E.6 https://chem.libretexts.org/@go/page/44117


The mass of oxygen is (1.08 × 10 −2
mol)(32.0 g/mol) = 0.346 g

or, using two significant figures, 0.35 g.

11.E.3.19: Q11.4.10
How many liters of HCl gas, measured at 30.0 °C and 745 torr, are required to prepare 1.25 L of a 3.20-M solution of hydrochloric
acid?

11.E.3.20: S11.4.10
First, calculate the moles of HCl needed. Then use the ideal gas law to find the volume required.
M = mol L−1
3.20M=xmol1.25L
x = 4.00 mol HCl
Before using the ideal gas law, change pressure to atmospheres and convert temperature from °C to kelvin.
\[1\;atmx=760torr745torr
x = 0.9803 atm
V=nRTP=(4.000molHCl)(0.08206LatmK−1mol−1)(303.15K)0.9803atm=102 L HCl
102 L HCl
more http://cnx.org/contents/2488fW6W@2/Solubility

11.E.4: 11.5: Colligative Properties


11.E.4.1: Q11.5.1
Which is/are part of the macroscopic domain of solutions and which is/are part of the microscopic domain: boiling point elevation,
Henry’s law, hydrogen bond, ion-dipole attraction, molarity, nonelectrolyte, nonstoichiometric compound, osmosis, solvated ion?

11.E.4.2: Q11.5.2
What is the microscopic explanation for the macroscopic behavior illustrated in [link]?

11.E.4.3: S11.5.2
The strength of the bonds between like molecules is stronger than the strength between unlike molecules. Therefore, some regions
will exist in which the water molecules will exclude oil molecules and other regions will exist in which oil molecules will exclude
water molecules, forming a heterogeneous region.

11.E.4.4: Q11.5.3
Sketch a qualitative graph of the pressure versus time for water vapor above a sample of pure water and a sugar solution, as the
liquids evaporate to half their original volume.

11.E.4.5: Q11.5.4
A solution of potassium nitrate, an electrolyte, and a solution of glycerin (C3H5(OH)3), a nonelectrolyte, both boil at 100.3 °C.
What other physical properties of the two solutions are identical?

11.E.4.6: S11.5.4
Both form homogeneous solutions; their boiling point elevations are the same, as are their lowering of vapor pressures. Osmotic
pressure and the lowering of the freezing point are also the same for both solutions.

11.E.4.7: Q11.5.5
What are the mole fractions of H3PO4 and water in a solution of 14.5 g of H3PO4 in 125 g of water?

Access for free at OpenStax 11.E.7 https://chem.libretexts.org/@go/page/44117


11.E.4.8: Q11.5.6
What are the mole fractions of HNO3 and water in a concentrated solution of nitric acid (68.0% HNO3 by mass)?

11.E.4.9: S11.5.6
a. Find number of moles of HNO3 and H2O in 100 g of the solution. Find the mole fractions for the components.
b. The mole fraction of HNO3 is 0.378. The mole fraction of H2O is 0.622.

11.E.4.10: Q11.5.7
Calculate the mole fraction of each solute and solvent:
a. 583 g of H2SO4 in 1.50 kg of water—the acid solution used in an automobile battery
b. 0.86 g of NaCl in 1.00 × 102 g of water—a solution of sodium chloride for intravenous injection
c. 46.85 g of codeine, C18H21NO3, in 125.5 g of ethanol, C2H5OH
d. 25 g of I2 in 125 g of ethanol, C2H5OH

11.E.4.11: S11.5.7
1 mole H SO
a. 583 g H SO
2 4
×
2 4
= 5.94 mole H SO
2 4
98.08 g H SO
2 4

1000 g 1 mole H O
2
1.50 kg H O × × = 83.2 moles H O
2 2
1 kg 18.02 g H O
2

11.E.4.12: Q11.5.8
Calculate the mole fraction of each solute and solvent:
a. 0.710 kg of sodium carbonate (washing soda), Na2CO3, in 10.0 kg of water—a saturated solution at 0 °C
b. 125 g of NH4NO3 in 275 g of water—a mixture used to make an instant ice pack
c. 25 g of Cl2 in 125 g of dichloromethane, CH2Cl2
d. 0.372 g of histamine, C5H9N, in 125 g of chloroform, CHCl3

11.E.4.13: S11.5.8
a. X = 0.0119; X
Na2 C O3 H2 O = 0.988 ;
b. XNH4 NO3= 0.9927; X H2 O = 0.907 ;
c. X = 0.192; X
Cl2 C H2 C I2 = 0.808 ;
d. X = 0.00426; X
C5 H9 N CHCl3 = 0.997

11.E.4.14: Q11.5.9
Calculate the mole fractions of methanol, CH3OH; ethanol, C2H5OH; and water in a solution that is 40% methanol, 40% ethanol,
and 20% water by mass. (Assume the data are good to two significant figures.)

11.E.4.15: Q11.5.10
What is the difference between a 1 M solution and a 1 m solution?

11.E.4.16: S11.5.10
In a 1 M solution, the mole is contained in exactly 1 L of solution. In a 1 m solution, the mole is contained in exactly 1 kg of
solvent.

11.E.4.17: Q11.5.11
What is the molality of phosphoric acid, H3PO4, in a solution of 14.5 g of H3PO4 in 125 g of water?

11.E.4.18: Q11.5.12
What is the molality of nitric acid in a concentrated solution of nitric acid (68.0% HNO3 by mass)?

11.E.4.19: S11.5.12
(a) Determine the molar mass of HNO3. Determine the number of moles of acid in the solution. From the number of moles and the
mass of solvent, determine the molality. (b) 33.7 m

Access for free at OpenStax 11.E.8 https://chem.libretexts.org/@go/page/44117


11.E.4.20: Q11.5.13
Calculate the molality of each of the following solutions:
a. 583 g of H2SO4 in 1.50 kg of water—the acid solution used in an automobile battery
b. 0.86 g of NaCl in 1.00 × 102 g of water—a solution of sodium chloride for intravenous injection
c. 46.85 g of codeine, C18H21NO3, in 125.5 g of ethanol, C2H5OH
d. 25 g of I2 in 125 g of ethanol, C2H5OH

11.E.4.21: Q11.5.14
Calculate the molality of each of the following solutions:
a. 0.710 kg of sodium carbonate (washing soda), Na2CO3, in 10.0 kg of water—a saturated solution at 0°C
b. 125 g of NH4NO3 in 275 g of water—a mixture used to make an instant ice pack
c. 25 g of Cl2 in 125 g of dichloromethane, CH2Cl2
d. 0.372 g of histamine, C5H9N, in 125 g of chloroform, CHCl3

11.E.4.22: S11.5.14
(a) 6.70 × 10−1 m; (b) 5.67 m; (c) 2.8 m; (d) 0.0358 m

11.E.4.23: Q11.5.15
75 mg
The concentration of glucose, C6H12O6, in normal spinal fluid is . What is the molality of the solution?
100 g

11.E.4.24: Q11.5.16
A 13.0% solution of K2CO3 by mass has a density of 1.09 g/cm3. Calculate the molality of the solution.

11.E.4.25: S11.5.16
1.08 m

11.E.4.26: Q11.5.17
a. Why does 1 mol of sodium chloride depress the freezing point of 1 kg of water almost twice as much as 1 mol of glycerin?
b. What is the boiling point of a solution of 115.0 g of sucrose, C12H22O11, in 350.0 g of water?

11.E.4.27: S11.5.17
a. Determine the molar mass of sucrose; determine the number of moles of sucrose in the solution; convert the mass of solvent to
units of kilograms; from the number of moles and the mass of solvent, determine the molality; determine the difference between
the boiling point of water and the boiling point of the solution; determine the new boiling point.
b. 100.5 °C

11.E.4.28: Q11.5.18
What is the boiling point of a solution of 9.04 g of I2 in 75.5 g of benzene, assuming the I2 is nonvolatile?

11.E.4.29: Q11.5.19
What is the freezing temperature of a solution of 115.0 g of sucrose, C12H22O11, in 350.0 g of water, which freezes at 0.0 °C when
pure?

11.E.4.30: S11.5.19
(a) Determine the molar mass of sucrose; determine the number of moles of sucrose in the solution; convert the mass of solvent to
units of kilograms; from the number of moles and the mass of solvent, determine the molality; determine the difference between
the freezing temperature of water and the freezing temperature of the solution; determine the new freezing temperature. (b) −1.8 °C

11.E.4.31: Q11.5.20
What is the freezing point of a solution of 9.04 g of I2 in 75.5 g of benzene?

Access for free at OpenStax 11.E.9 https://chem.libretexts.org/@go/page/44117


11.E.4.32: Q11.5.21
What is the osmotic pressure of an aqueous solution of 1.64 g of Ca(NO3)2 in water at 25 °C? The volume of the solution is 275
mL.

11.E.4.33: S11.5.21
(a) Determine the molar mass of Ca(NO3)2; determine the number of moles of Ca(NO3)2 in the solution; determine the number of
moles of ions in the solution; determine the molarity of ions, then the osmotic pressure. (b) 2.67 atm

11.E.4.34: Q11.5.22
What is osmotic pressure of a solution of bovine insulin (molar mass, 5700 g mol−1) at 18 °C if 100.0 mL of the solution contains
0.103 g of the insulin?

11.E.4.35: Q11.5.23
What is the molar mass of a solution of 5.00 g of a compound in 25.00 g of carbon tetrachloride (bp 76.8 °C; Kb = 5.02 °C/m) that
boils at 81.5 °C at 1 atm?

11.E.4.36: S11.5.24
(a) Determine the molal concentration from the change in boiling point and Kb; determine the moles of solute in the solution from
the molal concentration and mass of solvent; determine the molar mass from the number of moles and the mass of solute. (b) 2.1 ×
102 g mol−1

11.E.4.37: Q11.5.25
A sample of an organic compound (a nonelectrolyte) weighing 1.35 g lowered the freezing point of 10.0 g of benzene by 3.66 °C.
Calculate the molar mass of the compound.

11.E.4.38: Q11.5.26
A 1.0 m solution of HCl in benzene has a freezing point of 0.4 °C. Is HCl an electrolyte in benzene? Explain.

11.E.4.39: S11.5.26
No. Pure benzene freezes at 5.5 °C, and so the observed freezing point of this solution is depressed by ΔTf = 5.5 − 0.4 = 5.1 °C.
The value computed, assuming no ionization of HCl, is ΔTf = (1.0 m)(5.14 °C/m) = 5.1 °C. Agreement of these values supports the
assumption that HCl is not ionized.

11.E.4.40: Q11.5.27
A solution contains 5.00 g of urea, CO(NH2)2, a nonvolatile compound, dissolved in 0.100 kg of water. If the vapor pressure of
pure water at 25 °C is 23.7 torr, what is the vapor pressure of the solution?

11.E.4.41: Q11.5.28
A 12.0-g sample of a nonelectrolyte is dissolved in 80.0 g of water. The solution freezes at −1.94 °C. Calculate the molar mass of
the substance.

11.E.4.42: S11.5.28
144 g mol−1

11.E.4.43: Q11.5.29
Arrange the following solutions in order by their decreasing freezing points: 0.1 m Na3PO4, 0.1 m C2H5OH, 0.01 m CO2, 0.15 m
NaCl, and 0.2 m CaCl2.

11.E.4.44: Q11.5.30
Calculate the boiling point elevation of 0.100 kg of water containing 0.010 mol of NaCl, 0.020 mol of Na2SO4, and 0.030 mol of
MgCl2, assuming complete dissociation of these electrolytes.

Access for free at OpenStax 11.E.10 https://chem.libretexts.org/@go/page/44117


11.E.4.45: S11.5.30
0.870 °C

11.E.4.46: Q11.5.31
How could you prepare a 3.08 m aqueous solution of glycerin, C3H8O3? What is the freezing point of this solution?

11.E.4.47: Q11.5.32
A sample of sulfur weighing 0.210 g was dissolved in 17.8 g of carbon disulfide, CS2 (Kb = 2.43 °C/m). If the boiling point
elevation was 0.107 °C, what is the formula of a sulfur molecule in carbon disulfide?

11.E.4.48: S11.5.32
S8

11.E.4.49: Q11.5.33
In a significant experiment performed many years ago, 5.6977 g of cadmium iodide in 44.69 g of water raised the boiling point
0.181 °C. What does this suggest about the nature of a solution of CdI2?

11.E.4.50: Q11.5.34
Lysozyme is an enzyme that cleaves cell walls. A 0.100-L sample of a solution of lysozyme that contains 0.0750 g of the enzyme
exhibits an osmotic pressure of 1.32 × 10−3 atm at 25 °C. What is the molar mass of lysozyme?

11.E.4.51: S11.5.34
1.39 × 104 g mol−1

11.E.4.52: Q11.5.35
The osmotic pressure of a solution containing 7.0 g of insulin per liter is 23 torr at 25 °C. What is the molar mass of insulin?

11.E.4.53: Q11.5.36
The osmotic pressure of human blood is 7.6 atm at 37 °C. What mass of glucose, C6H12O6, is required to make 1.00 L of aqueous
solution for intravenous feeding if the solution must have the same osmotic pressure as blood at body temperature, 37 °C?

11.E.4.54: S11.5.36
54 g

11.E.4.55: Q11.5.37
What is the freezing point of a solution of dibromobenzene, C6H4Br2, in 0.250 kg of benzene, if the solution boils at 83.5 °C?

11.E.4.56: Q11.5.38
What is the boiling point of a solution of NaCl in water if the solution freezes at −0.93 °C?

11.E.4.57: S11.5.38
100.26 °C

11.E.4.58: Q11.5.39
The sugar fructose contains 40.0% C, 6.7% H, and 53.3% O by mass. A solution of 11.7 g of fructose in 325 g of ethanol has a
boiling point of 78.59 °C. The boiling point of ethanol is 78.35 °C, and Kb for ethanol is 1.20 °C/m. What is the molecular formula
of fructose?

11.E.4.59: Q11.5.40
The vapor pressure of methanol, CH3OH, is 94 torr at 20 °C. The vapor pressure of ethanol, C2H5OH, is 44 torr at the same
temperature.
a. Calculate the mole fraction of methanol and of ethanol in a solution of 50.0 g of methanol and 50.0 g of ethanol.
b. Ethanol and methanol form a solution that behaves like an ideal solution. Calculate the vapor pressure of methanol and of
ethanol above the solution at 20 °C.

Access for free at OpenStax 11.E.11 https://chem.libretexts.org/@go/page/44117


c. Calculate the mole fraction of methanol and of ethanol in the vapor above the solution.

11.E.4.60: S11.5.40
(a) XC H3 OH ;
= 0.590 XC2 H5 OH = 0.410 ; (b) Vapor pressures are: CH3OH: 55 torr; C2H5OH: 18 torr; (c) CH3OH: 0.75; C2H5OH:
0.25

11.E.4.61: Q11.5.41
The triple point of air-free water is defined as 273.15 K. Why is it important that the water be free of air?

11.E.4.62: Q11.5.42
Meat can be classified as fresh (not frozen) even though it is stored at −1 °C. Why wouldn’t meat freeze at this temperature?

11.E.4.63: S11.5.42
The ions and compounds present in the water in the beef lower the freezing point of the beef below −1 °C.

11.E.4.64: Q11.5.43
An organic compound has a composition of 93.46% C and 6.54% H by mass. A solution of 0.090 g of this compound in 1.10 g of
camphor melts at 158.4 °C. The melting point of pure camphor is 178.4 °C. Kf for camphor is 37.7 °C/m. What is the molecular
formula of the solute? Show your calculations.

11.E.4.65: Q11.5.44
A sample of HgCl2 weighing 9.41 g is dissolved in 32.75 g of ethanol, C2H5OH (Kb = 1.20 °C/m). The boiling point elevation of
the solution is 1.27 °C. Is HgCl2 an electrolyte in ethanol? Show your calculations.

11.E.4.66: S11.5.44
1 mol HgCl2
⎛ 9.41 g × ⎞

⎜ 271.496 g ⎟
Δbp = Kb m = (1.20 °C/m)⎜ ⎟ = 1.27 °C
⎜ 0.03275 kg ⎟

⎝ ⎠

The observed change equals the theoretical change; therefore, no dissociation occurs.

11.E.4.67: Q11.5.45
A salt is known to be an alkali metal fluoride. A quick approximate determination of freezing point indicates that 4 g of the salt
dissolved in 100 g of water produces a solution that freezes at about −1.4 °C. What is the formula of the salt? Show your
calculations.

11.E.5: 11.6: Colloids


11.E.5.1: Q11.6.1
Identify the dispersed phase and the dispersion medium in each of the following colloidal systems: starch dispersion, smoke, fog,
pearl, whipped cream, floating soap, jelly, milk, and ruby.

11.E.5.2: S11.6.1
Colloidal System Dispersed Phase Dispersion Medium

starch dispersion starch water

smoke solid particles air

fog water air

pearl water calcium carbonate (CaCO3)

whipped cream air cream

floating soap air soap

jelly fruit juice pectin gel

Access for free at OpenStax 11.E.12 https://chem.libretexts.org/@go/page/44117


Colloidal System Dispersed Phase Dispersion Medium

milk butterfat water

ruby chromium(III) oxide (Cr2O3) aluminum oxide (Al2O3)

11.E.5.3: Q11.6.2
Distinguish between dispersion methods and condensation methods for preparing colloidal systems.

11.E.5.4: S11.6.2
Dispersion methods use a grinding device or some other means to bring about the subdivision of larger particles. Condensation
methods bring smaller units together to form a larger unit. For example, water molecules in the vapor state come together to form
very small droplets that we see as clouds.

11.E.5.5: Q11.6.3
How do colloids differ from solutions with regard to dispersed particle size and homogeneity?

11.E.5.6: S11.6.3
Colloidal dispersions consist of particles that are much bigger than the solutes of typical solutions. Colloidal particles are either
very large molecules or aggregates of smaller species that usually are big enough to scatter light. Colloids are homogeneous on a
macroscopic (visual) scale, while solutions are homogeneous on a microscopic (molecular) scale.

11.E.5.7: Q11.6.4
Explain the cleansing action of soap.

11.E.5.8: S11.6.4
Soap molecules have both a hydrophobic and a hydrophilic end. The charged (hydrophilic) end, which is usually associated with an
alkali metal ion, ensures water solubility The hydrophobic end permits attraction to oil, grease, and other similar nonpolar
substances that normally do not dissolve in water but are pulled into the solution by the soap molecules.

11.E.5.9: Q11.6.5
How can it be demonstrated that colloidal particles are electrically charged?

11.E.5.10: S11.6.5
If they are placed in an electrolytic cell, dispersed particles will move toward the electrode that carries a charge opposite to their
own charge. At this electrode, the charged particles will be neutralized and will coagulate as a precipitate.

11.E.6: Contributors and Attributions


Paul Flowers (University of North Carolina - Pembroke), Klaus Theopold (University of Delaware) and Richard Langley
(Stephen F. Austin State University) with contributing authors. Textbook content produced by OpenStax College is licensed
under a Creative Commons Attribution License 4.0 license. Download for free at http://cnx.org/contents/85abf193-
2bd...a7ac8df6@9.110).

This page titled 11.E: Solutions and Colloids (Exercises) is shared under a CC BY 4.0 license and was authored, remixed, and/or curated by
OpenStax via source content that was edited to the style and standards of the LibreTexts platform; a detailed edit history is available upon request.

Access for free at OpenStax 11.E.13 https://chem.libretexts.org/@go/page/44117


CHAPTER OVERVIEW
12: Kinetics

A general chemistry Libretexts Textbook remixed and remastered from


OpenStax's textbook:
General Chemistry
Chemical kinetics is the study of rates of chemical processes and includes investigations of how different experimental conditions
can influence the speed of a chemical reaction and yield information about the reaction's mechanism and transition states, as well as
the construction of mathematical models that can describe the characteristics of a chemical reaction.
12.0: Prelude to Kinetics
12.1: Chemical Reaction Rates
12.2: Factors Affecting Reaction Rates
12.3: Rate Laws
12.4: Integrated Rate Laws
12.5: Collision Theory
12.6: Reaction Mechanisms
12.7: Catalysis
12.E: Kinetics (Exercises)

Thumbnail: Molecular collisions frequency. (Public Domain; Sadi Carnot via Wikipedia)

This page titled 12: Kinetics is shared under a CC BY 4.0 license and was authored, remixed, and/or curated by OpenStax via source content that
was edited to the style and standards of the LibreTexts platform; a detailed edit history is available upon request.

1
12.0: Prelude to Kinetics
The lizard in the photograph is not simply enjoying the sunshine or working on its tan. The heat from the sun’s rays is critical to the
lizard’s survival. A warm lizard can move faster than a cold one because the chemical reactions that allow its muscles to move
occur more rapidly at higher temperatures. In the absence of warmth, the lizard is an easy meal for predators.

Figure 12.0.1 : An agama lizard basks in the sun. As its body warms, the chemical reactions of its metabolism speed up.
From baking a cake to determining the useful lifespan of a bridge, rates of chemical reactions play important roles in our
understanding of processes that involve chemical changes. When planning to run a chemical reaction, we should ask at least two
questions. The first is: “Will the reaction produce the desired products in useful quantities?” The second question is: “How rapidly
will the reaction occur?” A reaction that takes 50 years to produce a product is about as useful as one that never gives a product at
all. A third question is often asked when investigating reactions in greater detail: “What specific molecular-level processes take
place as the reaction occurs?” Knowing the answer to this question is of practical importance when the yield or rate of a reaction
needs to be controlled.
The study of chemical kinetics concerns the second and third questions—that is, the rate at which a reaction yields products and the
molecular-scale means by which a reaction occurs. In this chapter, we will examine the factors that influence the rates of chemical
reactions, the mechanisms by which reactions proceed, and the quantitative techniques used to determine and describe the rate at
which reactions occur.
Paul Flowers (University of North Carolina - Pembroke), Klaus Theopold (University of Delaware) and Richard Langley
(Stephen F. Austin State University) with contributing authors. Textbook content produced by OpenStax College is licensed
under a Creative Commons Attribution License 4.0 license. Download for free at http://cnx.org/contents/85abf193-
2bd...a7ac8df6@9.110).

This page titled 12.0: Prelude to Kinetics is shared under a CC BY 4.0 license and was authored, remixed, and/or curated by OpenStax via source
content that was edited to the style and standards of the LibreTexts platform; a detailed edit history is available upon request.

Access for free at OpenStax 12.0.1 https://chem.libretexts.org/@go/page/38257


12.1: Chemical Reaction Rates
 Learning Objectives
Define chemical reaction rate
Derive rate expressions from the balanced equation for a given chemical reaction
Calculate reaction rates from experimental data

A rate is a measure of how some property varies with time. Speed is a familiar rate that expresses the distance traveled by an object
in a given amount of time. Wage is a rate that represents the amount of money earned by a person working for a given amount of
time. Likewise, the rate of a chemical reaction is a measure of how much reactant is consumed, or how much product is produced,
by the reaction in a given amount of time.
The rate of reaction is the change in the amount of a reactant or product per unit time. Reaction rates are therefore determined by
measuring the time dependence of some property that can be related to reactant or product amounts. Rates of reactions that
consume or produce gaseous substances, for example, are conveniently determined by measuring changes in volume or pressure.
For reactions involving one or more colored substances, rates may be monitored via measurements of light absorption. For
reactions involving aqueous electrolytes, rates may be measured via changes in a solution’s conductivity.
For reactants and products in solution, their relative amounts (concentrations) are conveniently used for purposes of expressing
reaction rates. If we measure the concentration of hydrogen peroxide, H2O2, in an aqueous solution, we find that it changes slowly
over time as the H2O2 decomposes, according to the equation:

2 H O (aq) ⟶ 2 H O(l) + O (g)


2 2 2 2

The rate at which the hydrogen peroxide decomposes can be expressed in terms of the rate of change of its concentration, as shown
here:
change in concentration of reactant
rate of decomposition of H O =−
2 2
time interval

[ H O ]t − [ H O ]t
2 2 2 2 2 1

=−
t2 − t1

Δ[ H O ]
2 2
=−
Δt

This mathematical representation of the change in species concentration over time is the rate expression for the reaction. The
brackets indicate molar concentrations, and the symbol delta (Δ) indicates “change in.” Thus, [H O ] represents the molar
2 2 t1

concentration of hydrogen peroxide at some time t1; likewise, [H O ] represents the molar concentration of hydrogen peroxide at
2 2 t2

a later time t2; and Δ[H2O2] represents the change in molar concentration of hydrogen peroxide during the time interval Δt (that is,
t2 − t1). Since the reactant concentration decreases as the reaction proceeds, Δ[H2O2] is a negative quantity; we place a negative
sign in front of the expression because reaction rates are, by convention, positive quantities. Figure 12.1.1 provides an example of
data collected during the decomposition of H2O2.

Figure 12.1.1 : The rate of decomposition of H2O2 in an aqueous solution decreases as the concentration of H2O2 decreases.
To obtain the tabulated results for this decomposition, the concentration of hydrogen peroxide was measured every 6 hours over the
course of a day at a constant temperature of 40 °C. Reaction rates were computed for each time interval by dividing the change in
concentration by the corresponding time increment, as shown here for the first 6-hour period:

Access for free at OpenStax 12.1.1 https://chem.libretexts.org/@go/page/38258


−Δ[ H O ] −(0.500 mol/L − 1.000 mol/L)
2 2 −1 −1
= = 0.0833 mol L h
Δt (6.00 h − 0.00 h)

Notice that the reaction rates vary with time, decreasing as the reaction proceeds. Results for the last 6-hour period yield a reaction
rate of:
−Δ[ H O ] −(0.0625 mol/L − 0.125 mol/L)
2 2 −1 −1
= = 0.0104 mol L h
Δt (24.00 h − 18.00 h)

This behavior indicates the reaction continually slows with time. Using the concentrations at the beginning and end of a time period
over which the reaction rate is changing results in the calculation of an average rate for the reaction over this time interval. At any
specific time, the rate at which a reaction is proceeding is known as its instantaneous rate. The instantaneous rate of a reaction at
“time zero,” when the reaction commences, is its initial rate. Consider the analogy of a car slowing down as it approaches a stop
sign. The vehicle’s initial rate—analogous to the beginning of a chemical reaction—would be the speedometer reading at the
moment the driver begins pressing the brakes (t0). A few moments later, the instantaneous rate at a specific moment—call it t1—
would be somewhat slower, as indicated by the speedometer reading at that point in time. As time passes, the instantaneous rate
will continue to fall until it reaches zero, when the car (or reaction) stops. Unlike instantaneous speed, the car’s average speed is
not indicated by the speedometer; but it can be calculated as the ratio of the distance traveled to the time required to bring the
vehicle to a complete stop (Δt). Like the decelerating car, the average rate of a chemical reaction will fall somewhere between its
initial and final rates.
The instantaneous rate of a reaction may be determined one of two ways. If experimental conditions permit the measurement of
concentration changes over very short time intervals, then average rates computed as described earlier provide reasonably good
approximations of instantaneous rates. Alternatively, a graphical procedure may be used that, in effect, yields the results that would
be obtained if short time interval measurements were possible. If we plot the concentration of hydrogen peroxide against time, the
instantaneous rate of decomposition of H2O2 at any time t is given by the slope of a straight line that is tangent to the curve at that
time (Figure 12.1.2). We can use calculus to evaluating the slopes of such tangent lines, but the procedure for doing so is beyond
the scope of this chapter.

Figure 12.1.1 : This graph shows a plot of concentration versus time for a 1.000 M solution of H2O2. The rate at any instant is equal
to the opposite of the slope of a line tangential to this curve at that time. Tangents are shown at t = 0 h (“initial rate”) and at t = 10 h
(“instantaneous rate” at that particular time).

 Reaction Rates in Analysis: Test Strips for Urinalysis

Physicians often use disposable test strips to measure the amounts of various substances in a patient’s urine (Figure 12.1.2).
These test strips contain various chemical reagents, embedded in small pads at various locations along the strip, which undergo
changes in color upon exposure to sufficient concentrations of specific substances. The usage instructions for test strips often
stress that proper read time is critical for optimal results. This emphasis on read time suggests that kinetic aspects of the
chemical reactions occurring on the test strip are important considerations.
The test for urinary glucose relies on a two-step process represented by the chemical equations shown here:
C H O +O −−−−→ C H O +H O (12.1.1)
6 12 6 2 6 10 6 2 2
catalyst


2H O +2 I −−−−→ I +2 H O+O (12.1.2)
2 2 2 2 2
catalyst

Access for free at OpenStax 12.1.2 https://chem.libretexts.org/@go/page/38258


Equation 12.1.1 depicts the oxidation of glucose in the urine to yield glucolactone and hydrogen peroxide. The hydrogen
peroxide produced subsequently oxidizes colorless iodide ion to yield brown iodine (Equation 12.1.2), which may be visually
detected. Some strips include an additional substance that reacts with iodine to produce a more distinct color change.
The two test reactions shown above are inherently very slow, but their rates are increased by special enzymes embedded in the
test strip pad. This is an example of catalysis, a topic discussed later in this chapter. A typical glucose test strip for use with
urine requires approximately 30 seconds for completion of the color-forming reactions. Reading the result too soon might lead
one to conclude that the glucose concentration of the urine sample is lower than it actually is (a false-negative result). Waiting
too long to assess the color change can lead to a false positive due to the slower (not catalyzed) oxidation of iodide ion by other
substances found in urine.

Figure 12.1.2 : Test strips are commonly used to detect the presence of specific substances in a person’s urine. Many test strips
have several pads containing various reagents to permit the detection of multiple substances on a single strip. (credit: Iqbal
Osman).

12.1.1: Relative Rates of Reaction


The rate of a reaction may be expressed in terms of the change in the amount of any reactant or product, and may be simply derived
from the stoichiometry of the reaction. Consider the reaction represented by the following equation:

2 NH (g) ⟶ N (g) + 3 H (g)


3 2 2

The stoichiometric factors derived from this equation may be used to relate reaction rates in the same manner that they are used to
related reactant and product amounts. The relation between the reaction rates expressed in terms of nitrogen production and
ammonia consumption, for example, is:
Δmol NH3 1 mol N2 Δmol N2
− × =
Δt 2 mol NH3 Δt

We can express this more simply without showing the stoichiometric factor’s units:
1 Δmol NH3 Δmol N2
− =
2 Δt Δt

Note that a negative sign has been added to account for the opposite signs of the two amount changes (the reactant amount is
decreasing while the product amount is increasing). If the reactants and products are present in the same solution, the molar
amounts may be replaced by concentrations:

1 Δ[ NH3 ] Δ[ N ]
2
− =
2 Δt Δt

Similarly, the rate of formation of H2 is three times the rate of formation of N2 because three moles of H2 form during the time
required for the formation of one mole of N2:

1 Δ[ H2 ] Δ[ N ]
2
=
3 Δt Δt

Access for free at OpenStax 12.1.3 https://chem.libretexts.org/@go/page/38258


Figure 12.1.3 illustrates the change in concentrations over time for the decomposition of ammonia into nitrogen and hydrogen at
1100 °C. We can see from the slopes of the tangents drawn at t = 500 seconds that the instantaneous rates of change in the
concentrations of the reactants and products are related by their stoichiometric factors. The rate of hydrogen production, for
example, is observed to be three times greater than that for nitrogen production:
−6
2.91 × 10 M /s
≈3
−6
9.71 × 10 M /s

Figure 12.1.3 : This graph shows the changes in concentrations of the reactants and products during the reaction
2 NH ⟶ N + 3 H . The rates of change of the three concentrations are related by their stoichiometric factors, as shown by the
3 2 2

different slopes of the tangents at t = 500 s.

 Example 12.1.1: Expressions for Relative Reaction Rates

The first step in the production of nitric acid is the combustion of ammonia:

4 NH (g) + 5 O (g) ⟶ 4 NO(g) + 6 H O(g)


3 2 2

Write the equations that relate the rates of consumption of the reactants and the rates of formation of the products.

Solution
Considering the stoichiometry of this homogeneous reaction, the rates for the consumption of reactants and formation of
products are:

1 Δ[ NH3 ] 1 Δ[ O2 ] 1 Δ[NO] 1 Δ[ H2 O]
− =− = =
4 Δt 5 Δt 4 Δt 6 Δt

 Exercise 12.1.1

The rate of formation of Br2 is 6.0 × 10−6 mol/L/s in a reaction described by the following net ionic equation:
− − +
5 Br + BrO 3 + 6 H ⟶ 3 Br +3 H O
2 2

Write the equations that relate the rates of consumption of the reactants and the rates of formation of the products.

Answer
− − +
1 Δ[ Br ] Δ[ BrO ] 1 Δ[ H ] 1 Δ[ Br 2 ] 1 Δ[ H2 O]
3
− =− =− = =
5 Δt Δt 6 Δt 3 Δt 3 Δt

Access for free at OpenStax 12.1.4 https://chem.libretexts.org/@go/page/38258


 Example 12.1.2: Reaction Rate Expressions for Decomposition of H2O2

The graph in Figure 12.1.3 shows the rate of the decomposition of H2O2 over time:

2H O ⟶ 2 H O+O
2 2 2 2

Based on these data, the instantaneous rate of decomposition of H2O2 at t = 11.1 h is determined to be 3.20 × 10−2 mol/L/h,
that is:
Δ[ H O ]
2 2 −2 −1 −1
− = 3.20 × 10 mol L h
Δt

What is the instantaneous rate of production of H2O and O2?

Solution
Using the stoichiometry of the reaction, we may determine that:

1 Δ[ H2 O2 ] 1 Δ[ H2 O] Δ[ O ]
2
− = =
2 Δt 2 Δt Δt

Therefore:

1 Δ[ O ]
−2 −1 −1 2
× 3.20 × 10 mol L h =
2 Δt

and
Δ[ O ]
2 −2 −1 −1
= 1.60 × 10 mol L h
Δt

 Exercise 12.1.2

If the rate of decomposition of ammonia, NH3, at 1150 K is 2.10 × 10−6 mol/L/s, what is the rate of production of nitrogen and
hydrogen?

Answer
1.05 × 10−6 mol/L/s, N2 and 3.15 × 10−6 mol/L/s, H2.

Summary
The rate of a reaction can be expressed either in terms of the decrease in the amount of a reactant or the increase in the amount of a
product per unit time. Relations between different rate expressions for a given reaction are derived directly from the stoichiometric
coefficients of the equation representing the reaction.

Glossary
average rate
rate of a chemical reaction computed as the ratio of a measured change in amount or concentration of substance to the time
interval over which the change occurred

initial rate
instantaneous rate of a chemical reaction at t = 0 s (immediately after the reaction has begun)

instantaneous rate
rate of a chemical reaction at any instant in time, determined by the slope of the line tangential to a graph of concentration as a
function of time

Access for free at OpenStax 12.1.5 https://chem.libretexts.org/@go/page/38258


rate of reaction
measure of the speed at which a chemical reaction takes place

rate expression
mathematical representation relating reaction rate to changes in amount, concentration, or pressure of reactant or product
species per unit time

This page titled 12.1: Chemical Reaction Rates is shared under a CC BY 4.0 license and was authored, remixed, and/or curated by OpenStax via
source content that was edited to the style and standards of the LibreTexts platform; a detailed edit history is available upon request.

Access for free at OpenStax 12.1.6 https://chem.libretexts.org/@go/page/38258


12.2: Factors Affecting Reaction Rates
 Learning Objectives
Describe the effects of chemical nature, physical state, temperature, concentration, and catalysis on reaction rates

The rates at which reactants are consumed and products are formed during chemical reactions vary greatly. We can identify five factors that affect
the rates of chemical reactions: the chemical nature of the reacting substances, the state of subdivision (one large lump versus many small particles)
of the reactants, the temperature of the reactants, the concentration of the reactants, and the presence of a catalyst.

12.2.1: The Chemical Nature of the Reacting Substances


The rate of a reaction depends on the nature of the participating substances. Reactions that appear similar may have different rates under the same
conditions, depending on the identity of the reactants. For example, when small pieces of the metals iron and sodium are exposed to air, the sodium
reacts completely with air overnight, whereas the iron is barely affected. The active metals calcium and sodium both react with water to form
hydrogen gas and a base. Yet calcium reacts at a moderate rate, whereas sodium reacts so rapidly that the reaction is almost explosive.

12.2.2: The State of Subdivision of the Reactants


Except for substances in the gaseous state or in solution, reactions occur at the boundary, or interface, between two phases. Hence, the rate of a
reaction between two phases depends to a great extent on the surface contact between them. A finely divided solid has more surface area available
for reaction than does one large piece of the same substance. Thus a liquid will react more rapidly with a finely divided solid than with a large piece
of the same solid. For example, large pieces of iron react slowly with acids; finely divided iron reacts much more rapidly (Figure 12.2.1). Large
pieces of wood smolder, smaller pieces burn rapidly, and saw dust burns explosively.

Figure 12.2.1 : (a) Iron powder reacts rapidly with dilute hydrochloric acid and produces bubbles of hydrogen gas because the powder has a large
total area: 2Fe(s) + 6HCl(aq) ⟶ 2FeCl3(aq) + 3H2(g). (surfaceb) An iron nail reacts more slowly.
This figure shows two photos labeled (a) and (b). Photo (a) shows the bottom of a test tube. The test tube is filled with a dark gas, and there is a dark
substance and bubbles in the bottom. Photo (b) shows a rod and bubbles in a test tube similar to photo (a), but the gas in the test tube is not as dark.

Caesium in Water (slow motion) - Periodic Table of Videos

Video 12.2.1 : The reaction of cesium with water in slow motion and a discussion of how the state of reactants and particle size affect reaction
rates.

Access for free at OpenStax 12.2.1 https://chem.libretexts.org/@go/page/38259


12.2.3: Temperature of the Reactants
Chemical reactions typically occur faster at higher temperatures. Food can spoil quickly when left on the kitchen counter. However, the lower
temperature inside of a refrigerator slows that process so that the same food remains fresh for days. We use a burner or a hot plate in the laboratory
to increase the speed of reactions that proceed slowly at ordinary temperatures. In many cases, an increase in temperature of only 10 °C will
approximately double the rate of a reaction in a homogeneous system.

12.2.4: Concentrations of the Reactants


The rates of many reactions depend on the concentrations of the reactants. Rates usually increase when the concentration of one or more of the
reactants increases. For example, calcium carbonate (CaCO ) deteriorates as a result of its reaction with the pollutant sulfur dioxide. The rate of this
3

reaction depends on the amount of sulfur dioxide in the air (Figure 12.2.2). As an acidic oxide, sulfur dioxide combines with water vapor in the air
to produce sulfurous acid in the following reaction:

SO 2(g) + H O(g) ⟶ H SO 3(aq) (12.2.1)


2 2

Calcium carbonate reacts with sulfurous acid as follows:

CaCO3(s) + H SO 3(aq) ⟶ CaSO3(aq) + CO2(g) + H O(l) (12.2.2)


2 2

In a polluted atmosphere where the concentration of sulfur dioxide is high, calcium carbonate deteriorates more rapidly than in less polluted air.
Similarly, phosphorus burns much more rapidly in an atmosphere of pure oxygen than in air, which is only about 20% oxygen.

Figure 12.2.2 : Statues made from carbonate compounds such as limestone and marble typically weather slowly over time due to the actions of water,
and thermal expansion and contraction. However, pollutants like sulfur dioxide can accelerate weathering. As the concentration of air pollutants
increases, deterioration of limestone occurs more rapidly. (credit: James P Fisher III).
A photograph is shown of an angel statue. While some details of the statue, including facial features, are present, effects of weathering appear to be
diminishing these features.

Phosphorus burning in atmosphere of pure oxygen

Video 12.2.2 : Phosphorous burns rapidly in air, but it will burn even more rapidly if the concentration of oxygen in is higher.

Access for free at OpenStax 12.2.2 https://chem.libretexts.org/@go/page/38259


12.2.5: The Presence of a Catalyst
Hydrogen peroxide solutions foam when poured onto an open wound because substances in the exposed tissues act as catalysts, increasing the rate
of hydrogen peroxide’s decomposition. However, in the absence of these catalysts (for example, in the bottle in the medicine cabinet) complete
decomposition can take months. A catalyst is a substance that increases the rate of a chemical reaction by lowering the activation energy without
itself being consumed by the reaction. Activation energy is the minimum amount of energy required for a chemical reaction to proceed in the
forward direction. A catalyst increases the reaction rate by providing an alternative pathway or mechanism for the reaction to follow (Figure 12.2.3).
Catalysis will be discussed in greater detail later in this chapter as it relates to mechanisms of reactions.

Figure 12.2.3 : The presence of a catalyst increases the rate of a reaction by lowering its activation energy.
A graph is shown with the label, “Reaction coordinate,” on the x-axis. The x-axis is depicted as an arrow. The y-axis is also an arrow and is labeled,
“Energy.” There is a horizontal line that runs the width of the graph and appears just above the x-axis. A segment of this line is blue and is labeled,
“Reactants.” From the right end of this line segment, a solid black, concave down curve is shown which reaches the level just below the end of the y-
axis. The curve ends at another short, blue line labeled, “Products.” The “Products” line appears at a higher level than the “Reactants” line. An arrow
extends from the horizontal line to the apex of the curve. The arrow is labeled, “Uncatalyzed reaction activation energy.” A second, black concave
down curve is shown. This curve also meets the reactants and products blue line segments, but only extends to about two-thirds the height of the
initial curve. From the horizontal line is another arrow pointing to the apex of the second curve. This arrow is labeled, “Catalyzed reaction activation
energy.”

Chemical reactions occur when molecules collide with each other and undergo a chemical
transformation. Before physically performing a reaction in a laboratory, scientists can use molecular
modeling simulations to predict how the parameters discussed earlier will influence the rate of a
reaction. Use the PhET Reactions & Rates interactive to explore how temperature, concentration, and
the nature of the reactants affect reaction rates.

Summary
The rate of a chemical reaction is affected by several parameters. Reactions involving two phases proceed more rapidly when there is greater surface
area contact. If temperature or reactant concentration is increased, the rate of a given reaction generally increases as well. A catalyst can increase the
rate of a reaction by providing an alternative pathway that causes the activation energy of the reaction to decrease.

Glossary
catalyst
substance that increases the rate of a reaction without itself being consumed by the reaction

This page titled 12.2: Factors Affecting Reaction Rates is shared under a CC BY 4.0 license and was authored, remixed, and/or curated by OpenStax via source
content that was edited to the style and standards of the LibreTexts platform; a detailed edit history is available upon request.

Access for free at OpenStax 12.2.3 https://chem.libretexts.org/@go/page/38259


12.3: Rate Laws
 Learning Objectives
Explain the form and function of a rate law
Use rate laws to calculate reaction rates
Use rate and concentration data to identify reaction orders and derive rate laws

As described in the previous module, the rate of a reaction is affected by the concentrations of reactants. Rate laws or rate equations
are mathematical expressions that describe the relationship between the rate of a chemical reaction and the concentration of its
reactants. In general, a rate law (or differential rate law, as it is sometimes called) takes this form:
m n p
rate = k[A] [B] [C ] …

in which [A], [B], and [C] represent the molar concentrations of reactants, and k is the rate constant, which is specific for a
particular reaction at a particular temperature. The exponents m, n, and p are usually positive integers (although it is possible for
them to be fractions or negative numbers). The rate constant k and the exponents m, n, and p must be determined experimentally by
observing how the rate of a reaction changes as the concentrations of the reactants are changed. The rate constant k is independent
of the concentration of A, B, or C, but it does vary with temperature and surface area.
The exponents in a rate law describe the effects of the reactant concentrations on the reaction rate and define the reaction order.
Consider a reaction for which the rate law is:
m n
rate = k[A] [B]

If the exponent m is 1, the reaction is first order with respect to A. If m is 2, the reaction is second order with respect to A. If n is 1,
the reaction is first order in B. If n is 2, the reaction is second order in B. If m or n is zero, the reaction is zero order in A or B,
respectively, and the rate of the reaction is not affected by the concentration of that reactant. The overall reaction order is the sum
of the orders with respect to each reactant. If m = 1 and n = 1, the overall order of the reaction is second order (m + n = 1 + 1 = 2).
The rate law:

rate = k[ H O ]
2 2

describes a reaction that is first order in hydrogen peroxide and first order overall. The rate law:
2
rate = k[C H ]
4 6

describes a reaction that is second order in C4H6 and second order overall. The rate law:
+ −
rate = k[ H ][ OH ]

describes a reaction that is first order in H+, first order in OH−, and second order overall.

 Example 12.3.1: Writing Rate Laws from Reaction Orders


An experiment shows that the reaction of nitrogen dioxide with carbon monoxide:

NO (g) + CO(g) ⟶ NO(g) + CO (g)


2 2

is second order in NO2 and zero order in CO at 100 °C. What is the rate law for the reaction?

Solution
The reaction will have the form:
m n
rate = k[ NO ] [CO]
2

The reaction is second order in NO2; thus m = 2. The reaction is zero order in CO; thus n = 0. The rate law is:
2 0 2
rate = k[ NO ] [CO] = k[ NO ]
2 2

Access for free at OpenStax 12.3.1 https://chem.libretexts.org/@go/page/38260


Remember that a number raised to the zero power is equal to 1, thus [CO]0 = 1, which is why we can simply drop the
concentration of CO from the rate equation: the rate of reaction is solely dependent on the concentration of NO2. When we
consider rate mechanisms later in this chapter, we will explain how a reactant’s concentration can have no effect on a reaction
despite being involved in the reaction.

 Exercise 12.3.1A

The rate law for the reaction:

H (g) + 2 NO(g) ⟶ N O(g) + H O(g)


2 2 2

2
has been experimentally determined to be rate = k[NO] [H2]. What are the orders with respect to each reactant, and what is the
overall order of the reaction?

Answer
order in NO = 2;
order in H2 = 1;
overall order = 3

 Exercise 12.3.1B

In a transesterification reaction, a triglyceride reacts with an alcohol to form an ester and glycerol. Many students learn about
the reaction between methanol (CH3OH) and ethyl acetate (CH3CH2OCOCH3) as a sample reaction before studying the
chemical reactions that produce biodiesel:

CH OH + CH CH OCOCH ⟶ CH OCOCH + CH CH OH
3 3 2 3 3 3 3 2

The rate law for the reaction between methanol and ethyl acetate is, under certain conditions, experimentally determined to be:

rate = k[ CH OH]
3

What is the order of reaction with respect to methanol and ethyl acetate, and what is the overall order of reaction?

Answer
order in CH3OH = 1;
order in CH3CH2OCOCH3 = 0;
overall order = 1

It is sometimes helpful to use a more explicit algebraic method, often referred to as the method of initial rates, to determine the
orders in rate laws. To use this method, we select two sets of rate data that differ in the concentration of only one reactant and set
up a ratio of the two rates and the two rate laws. After canceling terms that are equal, we are left with an equation that contains only
one unknown, the coefficient of the concentration that varies. We then solve this equation for the coefficient.

 Example 12.3.2: Determining a Rate Law from Initial Rates

Ozone in the upper atmosphere is depleted when it reacts with nitrogen oxides. The rates of the reactions of nitrogen oxides
with ozone are important factors in deciding how significant these reactions are in the formation of the ozone hole over
Antarctica (Figure 12.3.1). One such reaction is the combination of nitric oxide, NO, with ozone, O3:

Access for free at OpenStax 12.3.2 https://chem.libretexts.org/@go/page/38260


Figure 12.3.1 : Over the past several years, the atmospheric ozone concentration over Antarctica has decreased during the
winter. This map shows the decreased concentration as a purple area. (credit: modification of work by NASA)
A view of Earth’s southern hemisphere is shown. A nearly circular region of approximately half the diameter of the image is
shown in shades of purple, with Antarctica appearing in a slightly lighter color than the surrounding ocean areas. Immediately
outside this region is a narrow bright blue zone followed by a bright green zone. In the top half of the figure, the purple region
extends slightly outward from the circle and the blue zone extends more outward to the right of the center as compared to the
lower half of the image. In the upper half of the image, the majority of the space outside the purple region is shaded green, with
a few small strips of interspersed blue regions. The lower half however shows the majority of the space outside the central
purple zone in yellow, orange, and red. The red zones appear in the lower central and left regions outside the purple zone. To
the lower right of this image is a color scale that is labeled “Total Ozone (Dobsone units).” This scale begins at 0 and increases
by 100’s up to 700. At the left end of the scale, the value 0 shows a very deep purple color, 100 is indigo, 200 is blue, 300 is
green, 400 is a yellow-orange, 500 is red, 600 is pink, and 700 is white.

NO(g) + O (g) ⟶ NO (g) + O (g)


3 2 2

This reaction has been studied in the laboratory, and the following rate data were determined at 25 °C.
Δ[ NO ]
Trial [NO] (mol/L) [O ]
3
(mol/L) 2
(mol L
−1 −1
s )
Δt

1 1.00 × 10−6 3.00 × 10−6 6.60 × 10−5

2 1.00 × 10−6 6.00 × 10−6 1.32 × 10−4

3 1.00 × 10−6 9.00 × 10−6 1.98 × 10−4

4 2.00 × 10−6 9.00 × 10−6 3.96 × 10−4

5 3.00 × 10−6 9.00 × 10−6 5.94 × 10−4

Determine the rate law and the rate constant for the reaction at 25 °C.

Solution
The rate law will have the form:
m n
rate = k[NO] [O ]
3

We can determine the values of m, n, and k from the experimental data using the following three-part process:

Access for free at OpenStax 12.3.3 https://chem.libretexts.org/@go/page/38260


1. Determine the value of m from the data in which [NO] varies and [O3] is constant. In the last three experiments, [NO]
varies while [O3] remains constant. When [NO] doubles from trial 3 to 4, the rate doubles, and when [NO] triples from trial
3 to 5, the rate also triples. Thus, the rate is also directly proportional to [NO], and m in the rate law is equal to 1.
2. Determine the value of n from data in which [O3] varies and [NO] is constant. In the first three experiments, [NO] is
constant and [O3] varies. The reaction rate changes in direct proportion to the change in [O3]. When [O3] doubles from trial
1 to 2, the rate doubles; when [O3] triples from trial 1 to 3, the rate increases also triples. Thus, the rate is directly
proportional to [O3], and n is equal to 1.The rate law is thus:
1 1
rate = k[NO] [ O ] = k[NO][ O ]
3 3

3. Determine the value of k from one set of concentrations and the corresponding rate.
rate
k =
[NO][ O3 ]

−5 −1 −1
6.60 × 10 mol L s
=
−6 −1 −6 −1
(1.00 × 10 mol L )(3.00 × 10 mol L )

7 −1 −1
= 2.20 × 10 L mo l s

The large value of k tells us that this is a fast reaction that could play an important role in ozone depletion if [NO] is large
enough.

 Exercise 12.3.2

Acetaldehyde decomposes when heated to yield methane and carbon monoxide according to the equation:

CH CHO(g) ⟶ CH (g) + CO(g)


3 4

Determine the rate law and the rate constant for the reaction from the following experimental data:
Δ[ CH CHO]
Trial [ CH
3
CHO] (mol/L) −
3
(mol L
−1
s
−1
)
Δt

1 1.75 × 10−3 2.06 × 10−11

2 3.50 × 10−3 8.24 × 10−11

3 7.00 × 10−3 3.30 × 10−10

Answer
rate = k[ CH CHO]
3
2
with k = 6.73 × 10−6 L/mol/s

 Example 12.3.3: Determining Rate Laws from Initial Rates

Using the initial rates method and the experimental data, determine the rate law and the value of the rate constant for this
reaction:

2 NO(g) + Cl (g) ⟶ 2 NOCl(g)


2

Δ[NO]
Trial [NO] (mol/L) [C l2 ] (mol/L) − (mol L
−1
s
−1
)
2Δt

1 0.10 0.10 0.00300

2 0.10 0.15 0.00450

3 0.15 0.10 0.00675

Solution

Access for free at OpenStax 12.3.4 https://chem.libretexts.org/@go/page/38260


The rate law for this reaction will have the form:
m n
rate = k[NO] [ Cl ]
2

As in Example 12.3.2, we can approach this problem in a stepwise fashion, determining the values of m and n from the
experimental data and then using these values to determine the value of k. In this example, however, we will use a different
approach to determine the values of m and n:
Determine the value of m from the data in which [NO] varies and [Cl2] is constant. We can write the ratios with the subscripts
x and y to indicate data from two different trials:
m n
ratex k[NO]x [ Cl ]x
2
=
m n
ratey k[NO]y [ Cl ]y
2

Using the third trial and the first trial, in which [Cl2] does not vary, gives:
m n
rate 3 0.00675 k(0.15 ) (0.10 )
= =
m n
rate 1 0.00300 k(0.10 ) (0.10 )

After canceling equivalent terms in the numerator and denominator, we are left with:
m
0.00675 (0.15)
=
m
0.00300 (0.10)

which simplifies to:


m
2.25 = (1.5)

We can use natural logs to determine the value of the exponent m:


ln(2.25)
ln(2.25) = m ln(1.5) = m2 = m
ln(1.5)

We can confirm the result easily, since:


2
1.5 = 2.25

Determine the value of n from data in which [Cl2] varies and [NO] is constant.
m n
rate 2 0.00450 k(0.10 ) (0.15 )
= =
rate 1 0.00300 k(0.10 )m (0.10 )n

Cancelation gives:
n
0.0045 (0.15)
=
0.0030 (0.10)n

which simplifies to:


n
1.5 = (1.5)

Thus n must be 1, and the form of the rate law is:


m n 2
Rate = k[NO] [ Cl ] = k[NO] [ Cl ]
2 2

Determine the numerical value of the rate constant k with appropriate units. The units for the rate of a reaction are mol/L/s. The
units for k are whatever is needed so that substituting into the rate law expression affords the appropriate units for the rate. In
this example, the concentration units are mol3/L3. The units for k should be mol−2 L2/s so that the rate is in terms of mol/L/s.
To determine the value of k once the rate law expression has been solved, simply plug in values from the first experimental trial
and solve for k:
−1 −1 −1 2 −1 1
0.00300 mol L s = k(0.10 mol L ) (0.10 mol L )

−2 2 −1
k = 3.0 mol L s

Access for free at OpenStax 12.3.5 https://chem.libretexts.org/@go/page/38260


 Exercise 12.3.3

Use the provided initial rate data to derive the rate law for the reaction whose equation is:
− − − −
OCl (aq) + I (aq) ⟶ OI (aq) + Cl (aq)

Trial [OCl−] (mol/L) [I−] (mol/L) Initial Rate (mol/L/s)

1 0.0040 0.0020 0.00184

2 0.0020 0.0040 0.00092

3 0.0020 0.0020 0.00046

Determine the rate law expression and the value of the rate constant k with appropriate units for this reaction.

Answer
x y
rate 2 0.00092 k(0.0020 ) (0.0040 )
= =
x y
rate 3 0.00046 k(0.0020 ) (0.0020 )

2.00 = 2.00y
y=1

x y
rate 1 0.00184 k(0.0040 ) (0.0020 )
= =
x y
rate 2 0.00092 k(0.0020 ) (0.0040 )

x
2
2.00 =
y
2
x
2
2.00 =
1
2
x
4.00 = 2

x =2

Substituting the concentration data from trial 1 and solving for k yields:
− 2 − 1
rate = k[ OCl ] [I ]

2 1
0.00184 = k(0.0040 ) (0.0020 )

4 −2 2 −1
k = 5.75 × 10 mo l L s

12.3.1: Reaction Order and Rate Constant Units


In some of our examples, the reaction orders in the rate law happen to be the same as the coefficients in the chemical equation for
the reaction. This is merely a coincidence and very often not the case. Rate laws may exhibit fractional orders for some reactants,
and negative reaction orders are sometimes observed when an increase in the concentration of one reactant causes a decrease in
reaction rate. A few examples illustrating these points are provided:
2
NO + CO ⟶ NO + CO rate = k[ NO ]
2 2 2

2
CH CHO ⟶ CH + CO rate = k[ CH CHO]
3 4 3

2N O ⟶ 2 NO +O rate = k[ N O ]
2 5 2 2 2 5

2 NO +F ⟶ 2 NO F rate = k[ NO ][ F ]
2 2 2 2 2

2 NO Cl ⟶ 2 NO + Cl rate = k[ NO Cl]
2 2 2 2

It is important to note that rate laws are determined by experiment only and are not
reliably predicted by reaction stoichiometry.
Reaction orders also play a role in determining the units for the rate constant k. In Example 12.3.2, a second-order reaction, we
found the units for k to be L mol s , whereas in Example 12.3.3, a third order reaction, we found the units for k to be mol−2
−1 −1

Access for free at OpenStax 12.3.6 https://chem.libretexts.org/@go/page/38260


L2/s. More generally speaking, the units for the rate constant for a reaction of order (m + n) are mol 1−(m+n) (m+n)−1
L
−1
s . Table
12.3.1 summarizes the rate constant units for common reaction orders.

Table 12.3.1 : Rate Constants for Common Reaction Orders


Reaction Order Units of k
1−(m+n) (m+n)−1 −1
(m + n) mol L s

zero mol/L/s

first s−1

second L/mol/s

third mol−2 L2 s−1

Note that the units in the table can also be expressed in terms of molarity (M) instead of mol/L. Also, units of time other than the
second (such as minutes, hours, days) may be used, depending on the situation.

Summary
Rate laws provide a mathematical description of how changes in the amount of a substance affect the rate of a chemical reaction.
Rate laws are determined experimentally and cannot be predicted by reaction stoichiometry. The order of reaction describes how
much a change in the amount of each substance affects the overall rate, and the overall order of a reaction is the sum of the orders
for each substance present in the reaction. Reaction orders are typically first order, second order, or zero order, but fractional and
even negative orders are possible.

Glossary
method of initial rates
use of a more explicit algebraic method to determine the orders in a rate law

overall reaction order


sum of the reaction orders for each substance represented in the rate law

rate constant (k)


proportionality constant in the relationship between reaction rate and concentrations of reactants

rate law
(also, rate equation) mathematical equation showing the dependence of reaction rate on the rate constant and the concentration
of one or more reactants

reaction order
value of an exponent in a rate law, expressed as an ordinal number (for example, zero order for 0, first order for 1, second order
for 2, and so on)

This page titled 12.3: Rate Laws is shared under a CC BY 4.0 license and was authored, remixed, and/or curated by OpenStax via source content
that was edited to the style and standards of the LibreTexts platform; a detailed edit history is available upon request.

Access for free at OpenStax 12.3.7 https://chem.libretexts.org/@go/page/38260


12.4: Integrated Rate Laws
 Learning Objectives
Explain the form and function of an integrated rate law
Perform integrated rate law calculations for zero-, first-, and second-order reactions
Define half-life and carry out related calculations
Identify the order of a reaction from concentration/time data

The rate laws we have seen thus far relate the rate and the concentrations of reactants. We can also determine a second form of each
rate law that relates the concentrations of reactants and time. These are called integrated rate laws. We can use an integrated rate
law to determine the amount of reactant or product present after a period of time or to estimate the time required for a reaction to
proceed to a certain extent. For example, an integrated rate law is used to determine the length of time a radioactive material must
be stored for its radioactivity to decay to a safe level.
Using calculus, the differential rate law for a chemical reaction can be integrated with respect to time to give an equation that
relates the amount of reactant or product present in a reaction mixture to the elapsed time of the reaction. This process can either be
very straightforward or very complex, depending on the complexity of the differential rate law. For purposes of discussion, we will
focus on the resulting integrated rate laws for first-, second-, and zero-order reactions.

12.4.1: First-Order Reactions


An equation relating the rate constant k to the initial concentration [A] and the concentration [A] present after any given time
0 t t

can be derived for a first-order reaction and shown to be:


[A]t
ln( ) = −kt
[A]0

or alternatively
[A]0
ln( ) = kt
[A]t

or
−kt
[A] = [A]0 e

 Example 12.4.1: The Integrated Rate Law for a First-Order Reaction


The rate constant for the first-order decomposition of cyclobutane, C 4
H
8
at 500 °C is 9.2 × 10−3 s−1:

C H ⟶ 2C H
4 8 2 4

How long will it take for 80.0% of a sample of C4H8 to decompose?

Solution
We use the integrated form of the rate law to answer questions regarding time:
[A]0
ln( ) = kt
[A]

There are four variables in the rate law, so if we know three of them, we can determine the fourth. In this case we know [A]0,
[A], and k, and need to find t.
The initial concentration of C4H8, [A]0, is not provided, but the provision that 80.0% of the sample has decomposed is enough
information to solve this problem. Let x be the initial concentration, in which case the concentration after 80.0% decomposition
is 20.0% of x or 0.200x. Rearranging the rate law to isolate t and substituting the provided quantities yields:

Access for free at OpenStax 12.4.1 https://chem.libretexts.org/@go/page/38261


[x] 1
t = ln ×
[0.200x] k

−1
0.100 mol L 1
= ln ×
−1 −3 −1
0.020 mol L 9.2 × 10 s

1
= 1.609 ×
−3 −1
9.2 × 10 s

2
= 1.7 × 10 s

 Exercise 12.4.1

Iodine-131 is a radioactive isotope that is used to diagnose and treat some forms of thyroid cancer. Iodine-131 decays to xenon-
131 according to the equation:

I-131 ⟶ Xe-131 + electron

The decay is first-order with a rate constant of 0.138 d−1. All radioactive decay is first order. How many days will it take for
90% of the iodine−131 in a 0.500 M solution of this substance to decay to Xe-131?

Answer
16.7 days

We can use integrated rate laws with experimental data that consist of time and concentration information to determine the order
and rate constant of a reaction. The integrated rate law can be rearranged to a standard linear equation format:
ln[A] = (−k)(t) + ln[A]0 (12.4.1)

y = mx + b (12.4.2)

A plot of ln[A] versus t for a first-order reaction is a straight line with a slope of −k and an intercept of ln[A] . If a set of rate data
0

are plotted in this fashion but do not result in a straight line, the reaction is not first order in A .

 Example 12.4.2: Determination of Reaction Order by Graphing

Show that the data in this Figure can be represented by a first-order rate law by graphing ln[H2O2] versus time. Determine the
rate constant for the rate of decomposition of H2O2 from this data.

Solution
The data from this Figure with the addition of values of ln[H2O2] are given in Figure 12.4.1.

Figure 12.4.1 : The linear relationship between the ln[H2O2] and time shows that the decomposition of hydrogen peroxide is a
first-order reaction.
Solutions to Example 12.4.2

Access for free at OpenStax 12.4.2 https://chem.libretexts.org/@go/page/38261


Trial Time (h) [H2O2] (M) ln[H2O2]

1 0 1.000 0.0

2 6.00 0.500 −0.693

3 12.00 0.250 −1.386

4 18.00 0.125 −2.079

5 24.00 0.0625 −2.772

The plot of ln[H2O2] versus time is linear, thus we have verified that the reaction may be described by a first-order rate law.
The rate constant for a first-order reaction is equal to the negative of the slope of the plot of ln[H2O2] versus time where:

change in y Δy Δ ln[ H O ]


2 2
slope = = =
change in x Δx Δt

In order to determine the slope of the line, we need two values of ln[H2O2] at different values of t (one near each end of the
line is preferable). For example, the value of ln[H2O2] when t is 6.00 h is −0.693; the value when t = 12.00 h is −1.386:
−1.386 − (−0.693)
slope =
12.00 h − 6.00 h

−0.693
=
6.00 h

−2 −1
= −1.155 × 10 h

−1 −1 −1 −1
k = −slope = −(−1.155 × 10 h ) = 1.155 × 10 h

 Exercise 12.4.2

Graph the following data to determine whether the reaction A ⟶ B + C is first order.
Data for the reaction A ⟶ B + C
Trial Time (s) [A]

1 4.0 0.220

2 8.0 0.144

3 12.0 0.110

4 16.0 0.088

5 20.0 0.074

Answer
The plot of ln[A] vs. t is not a straight line. The equation is not first order:

Access for free at OpenStax 12.4.3 https://chem.libretexts.org/@go/page/38261


A graph, labeled above as “l n [ A ] vs. Time” is shown. The x-axis is labeled, “Time ( s )” and the y-axis is labeled, “l n [
A ].” The x-axis shows markings at 5, 10, 15, 20, and 25 hours. The y-axis shows markings at negative 3, negative 2,
negative 1, and 0. A slight curve is drawn connecting five points at coordinates of approximately (4, negative 1.5), (8,
negative 2), (12, negative 2.2), (16, negative 2.4), and (20, negative 2.6).

12.4.2: Second-Order Reactions


The equations that relate the concentrations of reactants and the rate constant of second-order reactions are fairly complicated. We
will limit ourselves to the simplest second-order reactions, namely, those with rates that are dependent upon just one reactant’s
concentration and described by the differential rate law:
2
Rate = k[A]

For these second-order reactions, the integrated rate law is:


1 1
= kt + (12.4.3)
[A] [A]0

where the terms in the equation have their usual meanings as defined earlier.

 Example 12.4.3: The Integrated Rate Law for a Second-Order Reaction

The reaction of butadiene gas (C4H6) with itself produces C8H12 gas as follows:

2 C H (g) ⟶ C H (g)
4 6 8 12

The reaction is second order with a rate constant equal to 5.76 × 10−2 L/mol/min under certain conditions. If the initial
concentration of butadiene is 0.200 M, what is the concentration remaining after 10.0 min?

Solution
We use the integrated form of the rate law to answer questions regarding time. For a second-order reaction, we have:
1 1
= kt +
[A] [A]0

We know three variables in this equation: [A]0 = 0.200 mol/L, k = 5.76 × 10−2 L/mol/min, and t = 10.0 min. Therefore, we can
solve for [A], the fourth variable:

Access for free at OpenStax 12.4.4 https://chem.libretexts.org/@go/page/38261


1 −2 −1 −1
1
= (5.76 × 10 L mo l mi n )(10 min) +
[A] 0.200 mol−1

1 −1 −1 −1
= (5.76 × 10 L mo l ) + 5.00 L mo l
[A]

1 −1
= 5.58 L mol
[A]

−1 −1
[A] = 1.79 × 10 mol L

Therefore 0.179 mol/L of butadiene remain at the end of 10.0 min, compared to the 0.200 mol/L that was originally present.

 Exercise 12.4.3

If the initial concentration of butadiene is 0.0200 M, what is the concentration remaining after 20.0 min?

Answer
0.0196 mol/L

The integrated rate law for our second-order reactions has the form of the equation of a straight line:
1 1
= kt +
[A] [A]0

y = mx + b

1 1
A plot of versus t for a second-order reaction is a straight line with a slope of k and an intercept of . If the plot is not a
[A] [A]0

straight line, then the reaction is not second order.

 Example 12.4.4: Determination of Reaction Order by Graphing

Test the data given to show whether the dimerization of C4H6 is a first- or a second-order reaction.

Solution
Solutions to Example 12.4.4
Trial Time (s) [C4H6] (M)

1 0 1.00 × 10−2

2 1600 5.04 × 10−3

3 3200 3.37 × 10−3

4 4800 2.53 × 10−3

5 6200 2.08 × 10−3

In order to distinguish a first-order reaction from a second-order reaction, we plot ln[C4H6] versus t and compare it with a plot
1
of versus t. The values needed for these plots follow.
[ C4 H6 ]

Solutions to Example 12.4.4


1
Time (s) (M
−1
) ln[C4H6]
[C H ]
4 6

0 100 −4.605

1600 198 −5.289

3200 296 −5.692

Access for free at OpenStax 12.4.5 https://chem.libretexts.org/@go/page/38261


1
Time (s) (M
−1
) ln[C4H6]
[C H ]
4 6

4800 395 −5.978

6200 481 −6.175

The plots are shown in Figure 12.4.2. As you can see, the plot of ln[C4H6] versus t is not linear, therefore the reaction is not
1
first order. The plot of versus t is linear, indicating that the reaction is second order.
[C H ]
4 6

Figure 12.4.2 : These two graphs show first- and second-order plots for the dimerization of C4H6. Since the first-order plot
(left) is not linear, we know that the reaction is not first order. The linear trend in the second-order plot (right) indicates that the
reaction follows second-order kinetics.
Two graphs are shown, each with the label “Time ( s )” on the x-axis. The graph on the left is labeled, “l n [ C subscript 4 H
subscript 6 ],” on the y-axis. The graph on the right is labeled “1 divided by [ C subscript 4 H subscript 6 ],” on the y-axis. The
x-axes for both graphs show markings at 3000 and 6000. The y-axis for the graph on the left shows markings at negative 6,
negative 5, and negative 4. A decreasing slightly concave up curve is drawn through five points at coordinates that are (0,
negative 4.605), (1600, negative 5.289), (3200, negative 5.692), (4800, negative 5.978), and (6200, negative 6.175). The y-axis
for the graph on the right shows markings at 100, 300, and 500. An approximately linear increasing curve is drawn through
five points at coordinates that are (0, 100), (1600, 198), (3200, 296), and (4800, 395), and (6200, 481).

 Exercise 12.4.4

Does the following data fit a second-order rate law?


Data for second-order rate law
Trial Time (s) [A] (M)

1 5 0.952

2 10 0.625

3 15 0.465

4 20 0.370

5 25 0.308

6 35 0.230

Answer
1
Yes. The plot of vs. t is linear:
[A]

Access for free at OpenStax 12.4.6 https://chem.libretexts.org/@go/page/38261


A graph, with the title “1 divided by [ A ] vs. Time” is shown, with the label, “Time ( s ),” on the x-axis. The label “1
divided by [ A ]” appears left of the y-axis. The x-axis shows markings beginning at zero and continuing at intervals of 10
up to and including 40. The y-axis on the left shows markings beginning at 0 and increasing by intervals of 1 up to and
including 5. A line with an increasing trend is drawn through six points at approximately (4, 1), (10, 1.5), (15, 2.2), (20,
2.8), (26, 3.4), and (36, 4.4).

12.4.3: Zero-Order Reactions


For zero-order reactions, the differential rate law is:
0
Rate = k[A] =k

A zero-order reaction thus exhibits a constant reaction rate, regardless of the concentration of its reactants.
The integrated rate law for a zero-order reaction also has the form of the equation of a straight line:
[A] = −kt + [A]0

y = mx + b

A plot of [A] versus t for a zero-order reaction is a straight line with a slope of −k and an intercept of [A]0. Figure 12.4.3 shows a
plot of [NH3] versus t for the decomposition of ammonia on a hot tungsten wire and for the decomposition of ammonia on hot
quartz (SiO2). The decomposition of NH3 on hot tungsten is zero order; the plot is a straight line. The decomposition of NH3 on hot
quartz is not zero order (it is first order). From the slope of the line for the zero-order decomposition, we can determine the rate
constant:
−6
slope = −k = 1.3110 mol/L/s

Access for free at OpenStax 12.4.7 https://chem.libretexts.org/@go/page/38261


Figure 12.4.3 : The decomposition of NH3 on a tungsten (W) surface is a zero-order reaction, whereas on a quartz (SiO2) surface,
the reaction is first order.

12.4.4: The Half-Life of a Reaction


The half-life of a reaction (t1/2) is the time required for one-half of a given amount of reactant to be consumed. In each succeeding
half-life, half of the remaining concentration of the reactant is consumed. Using the decomposition of hydrogen peroxide as an
example, we find that during the first half-life (from 0.00 hours to 6.00 hours), the concentration of H2O2 decreases from 1.000 M
to 0.500 M. During the second half-life (from 6.00 hours to 12.00 hours), it decreases from 0.500 M to 0.250 M; during the third
half-life, it decreases from 0.250 M to 0.125 M. The concentration of H2O2 decreases by half during each successive period of 6.00
hours. The decomposition of hydrogen peroxide is a first-order reaction, and, as can be shown, the half-life of a first-order reaction
is independent of the concentration of the reactant. However, half-lives of reactions with other orders depend on the concentrations
of the reactants.

12.4.4.1: First-Order Reactions


We can derive an equation for determining the half-life of a first-order reaction from the alternate form of the integrated rate law as
follows:
[A]0
ln = kt
[A]

[A]0 1
t = ln ×
[A] k

If we set the time t equal to the half-life, t1/2 , the corresponding concentration of A at this time is equal to one-half of its initial
1
concentration. Hence, when t = t 1/2 , [A] = [A]0 .
2

Therefore:
[A]0 1
t1/2 = ln ×
1 k
[A]0
2

1 1
= ln 2 × = 0.693 ×
k k

Thus:
0.693
t1/2 =
k

Access for free at OpenStax 12.4.8 https://chem.libretexts.org/@go/page/38261


We can see that the half-life of a first-order reaction is inversely proportional to the rate constant k. A fast reaction (shorter half-
life) will have a larger k; a slow reaction (longer half-life) will have a smaller k.

 Example 12.4.5: Calculation of a First-order Rate Constant using Half-Life

Calculate the rate constant for the first-order decomposition of hydrogen peroxide in water at 40 °C, using the data given in
Figure 12.4.4.

Figure 12.4.4 : The decomposition of H2O2 (2 H O ⟶ 2 H O + O ) at 40 °C is illustrated. The intensity of the color
2 2 2 2

symbolizes the concentration of H2O2 at the indicated times; H2O2 is actually colorless.
Solution The half-life for the decomposition of H2O2 is 2.16 × 104 s:
0.693
t1/2 =
k

0.693 0.693 −5 −1
k = = = 3.21 × 10 s
4
t1/2 2.16 × 10 s

 Exercise 12.4.1
The first-order radioactive decay of iodine-131 exhibits a rate constant of 0.138 d−1. What is the half-life for this decay?

Answer
5.02 d.

12.4.4.2: Second-Order Reactions


We can derive the equation for calculating the half-life of a second order as follows:
1 1
= kt +
[A] [A]0

or
1 1
− = kt
[A] [A]0

If

t = t1/2

then
1
[A] = [A]0
2

and we can write:

Access for free at OpenStax 12.4.9 https://chem.libretexts.org/@go/page/38261


1 1
− = kt1/2
1 [A]0
[A]0
2

1
2[A]0 − = kt1/2
[A]0

1
= kt1/2
[A]0

Thus:
1
t1/2 =
k[A]0

For a second-order reaction, t is inversely proportional to the concentration of the reactant, and the half-life increases as the
1/2

reaction proceeds because the concentration of reactant decreases. Consequently, we find the use of the half-life concept to be more
complex for second-order reactions than for first-order reactions. Unlike with first-order reactions, the rate constant of a second-
order reaction cannot be calculated directly from the half-life unless the initial concentration is known.

12.4.4.3: Zero-Order Reactions


We can derive an equation for calculating the half-life of a zero order reaction as follows:

[A] = −kt + [A]0

[A]0
When half of the initial amount of reactant has been consumed t = t 1/2 and [A] = . Thus:
2

[A]0
= −kt1/2 + [A]0
2

[A]0
kt1/2 =
2

and
[A]0
t1/2 =
2k

The half-life of a zero-order reaction increases as the initial concentration increases. Equations for both differential and integrated
rate laws and the corresponding half-lives for zero-, first-, and second-order reactions are summarized in Table 12.4.1.
Table 12.4.1 : Summary of Rate Laws for Zero-, First-, and Second-Order Reactions
Zero-Order First-Order Second-Order

rate law rate = k rate = k[A] rate = k[A]2

units of rate constant M s−1 s−1 M−1 s−1


1 1
integrated rate law [A] = −kt + [A]0 ln[A] = −kt + ln[A]0 = kt + ( )
[A] [A]0

plot needed for linear fit of rate 1


[A] vs. t ln[A] vs. t vs. t
data [A]

relationship between slope of


k = −slope k = −slope k = +slope
linear plot and rate constant
[A]0 0.693 1
half-life t1/2 = t1/2 = t1/2 =
2k k [A ]0 k

Access for free at OpenStax 12.4.10 https://chem.libretexts.org/@go/page/38261


Summary
Differential rate laws can be determined by the method of initial rates or other methods. We measure values for the initial rates of a
reaction at different concentrations of the reactants. From these measurements, we determine the order of the reaction in each
reactant. Integrated rate laws are determined by integration of the corresponding differential rate laws. Rate constants for those rate
laws are determined from measurements of concentration at various times during a reaction.
The half-life of a reaction is the time required to decrease the amount of a given reactant by one-half. The half-life of a zero-order
reaction decreases as the initial concentration of the reactant in the reaction decreases. The half-life of a first-order reaction is
independent of concentration, and the half-life of a second-order reaction decreases as the concentration increases.

12.4.5: Key Equations


integrated rate law for zero-order reactions (Equation ??? ):
[A]0
[A] = −kt + [A]0 with t 1/2 =
2k

integrated rate law for first-order reactions (Equation 12.4.1):


0.693
ln[A] = −kt + ln[A]0 with t 1/2 =
k

integrated rate law for second-order reactions (Equation 12.4.3):


1 1 1
= kt + with t
1/2
=
[A] [A]0 [A]0 k

Glossary
half-life of a reaction (tl/2)
time required for half of a given amount of reactant to be consumed

integrated rate law


equation that relates the concentration of a reactant to elapsed time of reaction

This page titled 12.4: Integrated Rate Laws is shared under a CC BY 4.0 license and was authored, remixed, and/or curated by OpenStax via
source content that was edited to the style and standards of the LibreTexts platform; a detailed edit history is available upon request.

Access for free at OpenStax 12.4.11 https://chem.libretexts.org/@go/page/38261


12.5: Collision Theory
 Learning Objectives
Use the postulates of collision theory to explain the effects of physical state, temperature, and concentration on reaction
rates
Define the concepts of activation energy and transition state
Use the Arrhenius equation in calculations relating rate constants to temperature

We should not be surprised that atoms, molecules, or ions must collide before they can react with each other. Atoms must be close
together to form chemical bonds. This simple premise is the basis for a very powerful theory that explains many observations
regarding chemical kinetics, including factors affecting reaction rates. Collision theory is based on the following postulates:

 Postulates of Collision theory


1. The rate of a reaction is proportional to the rate of reactant collisions:
# collisions
reaction rate ∝
time

2. The reacting species must collide in an orientation that allows contact between the atoms that will become bonded together
in the product.
3. The collision must occur with adequate energy to permit mutual penetration of the reacting species’ valence shells so that
the electrons can rearrange and form new bonds (and new chemical species).

We can see the importance of the two physical factors noted in postulates 2 and 3, the orientation and energy of collisions, when we
consider the reaction of carbon monoxide with oxygen:

2 CO(g) + O (g) ⟶ 2 CO (g)


2 2

Carbon monoxide is a pollutant produced by the combustion of hydrocarbon fuels. To reduce this pollutant, automobiles have
catalytic converters that use a catalyst to carry out this reaction. It is also a side reaction of the combustion of gunpowder that
results in muzzle flash for many firearms. If carbon monoxide and oxygen are present in sufficient quantity, the reaction is
spontaneous at high temperature and pressure.
The first step in the gas-phase reaction between carbon monoxide and oxygen is a collision between the two molecules:

CO(g) + O (g) ⟶ CO (g) + O(g)


2 2

Although there are many different possible orientations the two molecules can have relative to each other, consider the two
presented in Figure 12.5.1. In the first case, the oxygen side of the carbon monoxide molecule collides with the oxygen molecule.
In the second case, the carbon side of the carbon monoxide molecule collides with the oxygen molecule. The second case is clearly
more likely to result in the formation of carbon dioxide, which has a central carbon atom bonded to two oxygen atoms (O=C=O) .
This is a rather simple example of how important the orientation of the collision is in terms of creating the desired product of the
reaction.

Figure 12.5.1 : Illustrated are two collisions that might take place between carbon monoxide and oxygen molecules. The orientation
of the colliding molecules partially determines whether a reaction between the two molecules will occur.
If the collision does take place with the correct orientation, there is still no guarantee that the reaction will proceed to form carbon
dioxide. Every reaction requires a certain amount of activation energy for it to proceed in the forward direction, yielding an

Access for free at OpenStax 12.5.1 https://chem.libretexts.org/@go/page/38262


appropriate activated complex along the way. As Figure 12.5.2 demonstrates, even a collision with the correct orientation can fail
to form the reaction product. In the study of reaction mechanisms, each of these three arrangements of atoms is called a proposed
activated complex or transition state.

Figure 12.5.2 : Possible transition states (activated complexes) for carbon monoxide reacting with oxygen to form carbon dioxide.
Solid lines represent covalent bonds, while dotted lines represent unstable orbital overlaps that may, or may not, become covalent
bonds as product is formed. In the first two examples in this figure, the O=O double bond is not impacted; therefore, carbon
dioxide cannot form. The third proposed transition state will result in the formation of carbon dioxide if the third “extra” oxygen
atom separates from the rest of the molecule.
In most circumstances, it is impossible to isolate or identify a transition state or activated complex. In the reaction between carbon
monoxide and oxygen to form carbon dioxide, activated complexes have only been observed spectroscopically in systems that
utilize a heterogeneous catalyst. The gas-phase reaction occurs too rapidly to isolate any such chemical compound.
Collision theory explains why most reaction rates increase as concentrations increase. With an increase in the concentration of any
reacting substance, the chances for collisions between molecules are increased because there are more molecules per unit of
volume. More collisions mean a faster reaction rate, assuming the energy of the collisions is adequate.

12.5.1: Activation Energy and the Arrhenius Equation


The minimum energy necessary to form a product during a collision between reactants is called the activation energy (E ). The a

kinetic energy of reactant molecules plays an important role in a reaction because the energy necessary to form a product is
provided by a collision of a reactant molecule with another reactant molecule. (In single-reactant reactions, activation energy may
be provided by a collision of the reactant molecule with the wall of the reaction vessel or with molecules of an inert contaminant.)
If the activation energy is much larger than the average kinetic energy of the molecules, the reaction will occur slowly: Only a few
fast-moving molecules will have enough energy to react. If the activation energy is much smaller than the average kinetic energy of
the molecules, the fraction of molecules possessing the necessary kinetic energy will be large; most collisions between molecules
will result in reaction, and the reaction will occur rapidly.
Figure 12.5.3 shows the energy relationships for the general reaction of a molecule of A with a molecule of B to form molecules
of C and D:

A+B ⟶ C +D

The figure shows that the energy of the transition state is higher than that of the reactants A and B by an amount equal to E , the a

activation energy. Thus, the sum of the kinetic energies of A and B must be equal to or greater than Ea to reach the transition
state. After the transition state has been reached, and as C and D begin to form, the system loses energy until its total energy is
lower than that of the initial mixture. This lost energy is transferred to other molecules, giving them enough energy to reach the
transition state. The forward reaction (that between molecules A and B ) therefore tends to take place readily once the reaction has
started. In Figure 12.5.3, ΔH represents the difference in enthalpy between the reactants (A and B ) and the products (C and D).
The sum of E and ΔH represents the activation energy for the reverse reaction:
a

C +D ⟶ A+B

Access for free at OpenStax 12.5.2 https://chem.libretexts.org/@go/page/38262


Figure 12.5.3 : This graph shows the potential energy relationships for the reaction A + B ⟶ C + D . The dashed portion of the
curve represents the energy of the system with a molecule of A and a molecule of B present, and the solid portion the energy of the
system with a molecule of C and a molecule of D present. The activation energy for the forward reaction is represented by Ea. The
activation energy for the reverse reaction is greater than that for the forward reaction by an amount equal to ΔH. The curve’s peak
represents the transition state.

We can use the Arrhenius equation to relate the activation energy and the rate constant, k, of a given reaction:
−Ea /RT
k = Ae (12.5.1)

In this equation,
R is the ideal gas constant, which has a value 8.314 J/mol/K,
T is temperature on the Kelvin scale,
E is the activation energy in joules per mole,
a

e is the constant 2.7183, and

A is a constant called the frequency factor, which is related to the frequency of collisions and the orientation of the reacting

molecules.
Both postulates of the collision theory of reaction rates are accommodated in the Arrhenius equation. The frequency factor A is
related to the rate at which collisions having the correct orientation occur. The exponential term, e , is related to the fraction
−Ea /RT

of collisions providing adequate energy to overcome the activation barrier of the reaction.
At one extreme, the system does not contain enough energy for collisions to overcome the activation barrier. In such cases, no
reaction occurs. At the other extreme, the system has so much energy that every collision with the correct orientation can overcome
the activation barrier, causing the reaction to proceed. In such cases, the reaction is nearly instantaneous.
The Arrhenius equation (Equation 12.5.1) describes quantitatively much of what we have already discussed about reaction rates.
For two reactions at the same temperature, the reaction with the higher activation energy has the lower rate constant and the slower
rate. The larger value of E results in a smaller value for e
a , reflecting the smaller fraction of molecules with enough energy
−Ea /RT

to react. Alternatively, the reaction with the smaller E has a larger fraction of molecules with enough energy to react. This will be
a

reflected as a larger value of e , a larger rate constant, and a faster rate for the reaction.
−Ea /RT

Access for free at OpenStax 12.5.3 https://chem.libretexts.org/@go/page/38262


Figure 12.5.4 : (a) As the activation energy of a reaction decreases, the number of molecules with at least this much energy
increases, as shown by the shaded areas. (b) At a higher temperature, T2, more molecules have kinetic energies greater than Ea, as
shown by the yellow shaded area.
An increase in temperature has the same effect as a decrease in activation energy. A larger fraction of molecules has the necessary
energy to react (Figure 12.5.4), as indicated by an increase in the value of e . The rate constant is also directly proportional
−Ea /RT

to the frequency factor, A . Hence a change in conditions or reactants that increases the number of collisions with a favorable
orientation for reaction results in an increase in A and, consequently, an increase in k .
A convenient approach to determining E for a reaction involves the measurement of
a k at different temperatures and using of an
alternate version of the Arrhenius equation that takes the form of linear equation:
−Ea 1
ln k = ( )( ) + ln A
R T

y = mx + b

1 −Ea
Thus, a plot of ln k versus gives a straight line with the slope , from which Ea may be determined. The intercept gives the
T R
value of ln A . This is sometimes call an Arrhenius Plot.

 Example 12.5.1

Determination of Ea The variation of the rate constant with temperature for the decomposition of HI(g) to H2(g) and I2(g) is
given here. What is the activation energy for the reaction?

2 HI(g) ⟶ H (g) + I (g)


2 2

variation of the rate constant with temperature for the decomposition of HI(g) to H2(g) and I2(g)
T (K) k (L/mol/s)

555 3.52 × 10−7

575 1.22 × 10−6

645 8.59 × 10−5

700 1.16 × 10−3

781 3.95 × 10−2

Solution
1
Values of and ln k are:
T

Solutions to Example 12.5.1


1
(K
−1
) ln k
T

Access for free at OpenStax 12.5.4 https://chem.libretexts.org/@go/page/38262


1
(K
−1
) ln k
T

1.80 × 10−3 −14.860

1.74 × 10−3 −13.617

1.55 × 10−3 −9.362

1.43 × 10−3 −6.759

1.28 × 10−3 −3.231

1
Figure 12.5.5 is a graph of ln k versus . To determine the slope of the line, we need two values of ln k, which are determined
T
1
from the line at two values of (one near each end of the line is preferable). For example, the value of ln k determined from
T
1 1
the line when = 1.25 × 10
−3
is −2.593; the value when = 1.78 × 10
−3
is −14.447.
T T

1
Figure 12.5.5 : This graph shows the linear relationship between ln k and for the reaction 2 HI ⟶ H
2
+I
2
according to
T
the Arrhenius equation.
The slope of this line is given by the following expression:
Δ(ln k)
Slope =
1
Δ( )
T

(−14.447) − (−2.593)
=
−3 −1 −3 −1
(1.78 × 10 K ) − (1.25 × 10 K )

−11.854
4
= = 2.2 × 10 K
−3 −1
0.53 × 10 K
Ea
=−
R

Thus:
4 −1 −1
Ea = −slope × R = −(−2.2 × 10 K × 8.314 J mo l K )

5 −1
= 1.8 × 10 J mo l

In many situations, it is possible to obtain a reasonable estimate of the activation energy without going through the entire
process of constructing the Arrhenius plot. The Arrhenius equation:

Access for free at OpenStax 12.5.5 https://chem.libretexts.org/@go/page/38262


−Ea 1
ln k = ( )( ) + ln A
R T

can be rearranged as shown to give:


Δ(ln k) Ea
=−
1 R
Δ( )
T

or
k1 Ea 1 1
ln = ( − )
k2 R T2 T1

This equation can be rearranged to give a one-step calculation to obtain an estimate for the activation energy:

⎛ ⎞

⎜ ln k2 − ln k1 ⎟
Ea = −R ⎜ ⎟
⎜ 1 1 ⎟
( ) −( )
⎝ ⎠
T2 T1

Using the experimental data presented here, we can simply select two data entries. For this example, we select the first entry
and the last entry:
First and Last Entry
1
T (K) k (L/mol/s) (K
−1
) ln k
T

555 3.52 × 10−7 1.80 × 10−3 −14.860

781 3.95 × 10−2 1.28 × 10−3 −3.231

1
After calculating and ln k, we can substitute into the equation:
T

−3.231 − (−14.860)
−1 −1
Ea = −8.314 J mo l K ( )
−3 −3
1.28 × 10 K−1 − 1.80 × 10 K−1

and the result is Ea = 185,900 J/mol.


This method is very effective, especially when a limited number of temperature-dependent rate constants are available for the
reaction of interest.

 Exercise 12.5.1
The rate constant for the rate of decomposition of N2O5 to NO and O2 in the gas phase is 1.66 L/mol/s at 650 K and 7.39
L/mol/s at 700 K:

2 N O (g) ⟶ 4 NO(g) + 3 O (g)


2 5 2

Assuming the kinetics of this reaction are consistent with the Arrhenius equation, calculate the activation energy for this
decomposition.

Answer
113,000 J/mol

Summary
Chemical reactions require collisions between reactant species. These reactant collisions must be of proper orientation and
sufficient energy in order to result in product formation. Collision theory provides a simple but effective explanation for the effect

Access for free at OpenStax 12.5.6 https://chem.libretexts.org/@go/page/38262


of many experimental parameters on reaction rates. The Arrhenius equation describes the relation between a reaction’s rate constant
and its activation energy, temperature, and dependence on collision orientation.

12.5.2: Key Equations


−Ea /RT
k = Ae

−Ea 1
ln k = ( )( ) + ln A
R T

k1 Ea 1 1
ln = ( − )
k2 R T2 T1

Glossary
activated complex
(also, transition state) unstable combination of reactant species representing the highest energy state of a reaction system

activation energy (Ea)


energy necessary in order for a reaction to take place

Arrhenius equation
mathematical relationship between the rate constant and the activation energy of a reaction

collision theory
model that emphasizes the energy and orientation of molecular collisions to explain and predict reaction kinetics

frequency factor (A)


proportionality constant in the Arrhenius equation, related to the relative number of collisions having an orientation capable of
leading to product formation

This page titled 12.5: Collision Theory is shared under a CC BY 4.0 license and was authored, remixed, and/or curated by OpenStax via source
content that was edited to the style and standards of the LibreTexts platform; a detailed edit history is available upon request.

Access for free at OpenStax 12.5.7 https://chem.libretexts.org/@go/page/38262


12.6: Reaction Mechanisms
 Learning Objectives
Distinguish net reactions from elementary reactions (steps)
Identify the molecularity of elementary reactions
Write a balanced chemical equation for a process given its reaction mechanism
Derive the rate law consistent with a given reaction mechanism

A balanced equation for a chemical reaction indicates what is reacting and what is produced, but it reveals nothing about how the reaction actually takes place. The reaction mechanism (or reaction
path) is the process, or pathway, by which a reaction occurs. A chemical reaction often occurs in steps, although it may not always be obvious to an observer. The decomposition of ozone, for
example, appears to follow a mechanism with two steps:
O (g) ⟶ O (g) + O (12.6.1)
3 2

O + O (g) ⟶ 2 O (g)
3 2

We call each step in a reaction mechanism an elementary reaction. Elementary reactions occur exactly as they are written and cannot be broken down into simpler steps. Elementary reactions add up
to the overall reaction, which, for the decomposition, is:
2 O (g) ⟶ 3 O (g) (12.6.2)
3 2

Notice that the oxygen atom produced in the first step of this mechanism is consumed in the second step and therefore does not appear as a product in the overall reaction. Species that are produced
in one step and consumed in a subsequent step are called intermediates.
While the overall reaction equation for the decomposition of ozone indicates that two molecules of ozone react to give three molecules of oxygen, the mechanism of the reaction does not involve
the collision and reaction of two ozone molecules. Rather, it involves a molecule of ozone decomposing to an oxygen molecule and an intermediate oxygen atom; the oxygen atom then reacts with a
second ozone molecule to give two oxygen molecules. These two elementary reactions occur exactly as they are shown in the reaction mechanism.
12.6.0.0.1: Unimolecular Elementary Reactions
The molecularity of an elementary reaction is the number of reactant species (atoms, molecules, or ions). For example, a unimolecular reaction involves the rearrangement of a single reactant
species to produce one or more molecules of product:
A ⟶ products (12.6.3)

The rate equation for a unimolecular reaction is:


rate = k[A] (12.6.4)

A unimolecular reaction may be one of several elementary reactions in a complex mechanism. For example, the reaction:
O ⟶ O +O (12.6.5)
3 2

illustrates a unimolecular elementary reaction that occurs as one part of a two-step reaction mechanism. However, some unimolecular reactions may have only a single reaction in the reaction
mechanism. (In other words, an elementary reaction can also be an overall reaction in some cases.) For example, the gas-phase decomposition of cyclobutane, C4H8, to ethylene, C2H4, occurs via a
unimolecular, single-step mechanism:

In this figure, structural formulas are used to illustrate a chemical reaction. On the left, a structural formula for cyclobutane is shown. This structure is composed of 4 C atoms connected with single
bonds in a square shape. Each C atom is bonded to two other C atoms in the structure, leaving two bonds for H atoms pointing outward above, below, left, and right. An arrow points right to two
identical ethane molecules with a plus symbol between them. Each of these molecules contains two C atoms connected with a double bond oriented vertically between them. The C atom at the top
of these molecules has H atoms bonded above to the right and left. Similarly, the lower C atom has two H atoms bonded below to the right and left.
For these unimolecular reactions to occur, all that is required is the separation of parts of single reactant molecules into products.
Chemical bonds do not simply fall apart during chemical reactions. Energy is required to break chemical bonds. The activation energy for the decomposition of C4H8, for example, is 261 kJ per
mole. This means that it requires 261 kilojoules to distort one mole of these molecules into activated complexes that decompose into products:

In this figure, structural formulas are used to illustrate a chemical reaction, including an intermediate step. On the left, a structural formula for cyclobutane is shown. This structure is composed of 4
C atoms connected with single bonds in a square shape. Each C atom is bonded to two other C atoms in the structure, leaving two bonds for H atoms pointing outward above, below, left, and right.
This structure is labeled, “Cyclohexane.” An arrow points right to a similar structure which has the upper and lower bonds replaced by rows of 4 dots. Similarly, columns of 3 dots appear just inside
the line segments indicating the vertically oriented single bonds in the structure. The label “Activated complex” appears beneath this structure. A second arrow points right to two identical ethane
molecules with a plus symbol between them. Each of these molecules contains two C atoms connected with a double bond oriented vertically between them. The C atom at the top of these
molecules has H atoms bonded above to the right and left. Similarly, the lower C atom has two H atoms bonded below to the right and left. Below these two molecules appears the label “Ethylene.”
In a sample of C4H8, a few of the rapidly moving C4H8 molecules collide with other rapidly moving molecules and pick up additional energy. When the C4H8 molecules gain enough energy, they
can transform into an activated complex, and the formation of ethylene molecules can occur. In effect, a particularly energetic collision knocks a C4H8 molecule into the geometry of the activated
complex. However, only a small fraction of gas molecules travel at sufficiently high speeds with large enough kinetic energies to accomplish this. Hence, at any given moment, only a few molecules
pick up enough energy from collisions to react.
The rate of decomposition of C4H8 is directly proportional to its concentration. Doubling the concentration of C4H8 in a sample gives twice as many molecules per liter. Although the fraction of
molecules with enough energy to react remains the same, the total number of such molecules is twice as great. Consequently, there is twice as much C4H8 per liter, and the reaction rate is twice as
fast:
Δ[ C H ]
4 8
rate = − = k[ C H ] (12.6.6)
4 8
Δt

Access for free at OpenStax 12.6.1 https://chem.libretexts.org/@go/page/38263


A similar relationship applies to any unimolecular elementary reaction; the reaction rate is directly proportional to the concentration of the reactant, and the reaction exhibits first-order behavior.
The proportionality constant is the rate constant for the particular unimolecular reaction.

12.6.1: Bimolecular Elementary Reactions


The collision and combination of two molecules or atoms to form an activated complex in an elementary reaction is called a bimolecular reaction. There are two types of bimolecular elementary
reactions:
A + B ⟶ products (12.6.7)

and
2A ⟶ products (12.6.8)

For the first type, in which the two reactant molecules are different, the rate law is first-order in A and first order in B:

rate = k[A][B] (12.6.9)

For the second type, in which two identical molecules collide and react, the rate law is second order in A:
2
rate = k[A][A] = k[A] (12.6.10)

Some chemical reactions have mechanisms that consist of a single bimolecular elementary reaction. One example is the reaction of nitrogen dioxide with carbon monoxide:

NO (g) + CO(g) ⟶ NO(g) + CO (g) (12.6.11)


2 2

(see Figure 12.6.1)

Figure 12.6.1 : The probable mechanism for the reaction between NO and CO to yield NO and CO .
2 2

This figure provides an illustration of a reaction between two H I molecules using space filling models. H atoms are shown as white spheres, and I atoms are shown as purple spheres. On the left,
two H I molecules are shownwith a small white sphere bonded to a much larger purple sphere. The label, “Two H I molecules,” appears below. An arrow points right to a similar structure in which
the two molecules appear pushed together, so that the purple spheres of the two molecules are touching. Below appears the label, “Transition state.” Following another arrow, two white spheres are
shown vertically oriented and bonded together with the label, “H subscript 2” above. The H subscript 2 molecule is followed by a plus sign and two purple spheres bonded together with the label, “I
subscript 2” above. Below these structures is the label, “Hydrogen iodide molecules decompose to produce hydrogen H subscript 2 and iodine I subscript 2.”
Bimolecular elementary reactions may also be involved as steps in a multistep reaction mechanism. The reaction of atomic oxygen with ozone is one example:
O(g) + O (g) ⟶ 2 O (g) (12.6.12)
3 2

12.6.1.0.1: Termolecular Elementary Reactions

An elementary termolecular reaction involves the simultaneous collision of three atoms, molecules, or ions. Termolecular elementary reactions are uncommon because the probability of three
particles colliding simultaneously is less than one one-thousandth of the probability of two particles colliding. There are, however, a few established termolecular elementary reactions. The reaction
of nitric oxide with oxygen appears to involve termolecular steps:
2 NO + O ⟶ 2 NO (12.6.13)
2 2

2
rate = k[NO] [ O ]
2

Likewise, the reaction of nitric oxide with chlorine appears to involve termolecular steps:
2 NO + Cl ⟶ 2 NOCl (12.6.14)
2

2
rate = k[NO] [ Cl ]
2

12.6.1.0.1: Relating Reaction Mechanisms to Rate Laws


It's often the case that one step in a multistep reaction mechanism is significantly slower than the others. Because a reaction cannot proceed faster than its slowest step, this step will limit the rate at
which the overall reaction occurs. The slowest step is therefore called the rate-limiting step (or rate-determining step) of the reaction Figure 12.6.2.

Figure 12.6.2 : A cattle chute is a nonchemical example of a rate-determining step. Cattle can only be moved from one holding pen to another as quickly as one animal can make its way through the
chute. (credit: Loren Kerns)
A photo is shown of cattle passing through a narrow chute into a holding pen. A person directs them through the gate with a long white and red pole.
As described earlier, rate laws may be derived directly from the chemical equations for elementary reactions. This is not the case, however, for ordinary chemical reactions. The balanced equations
most often encountered represent the overall change for some chemical system, and very often this is the result of some multistep reaction mechanisms. In every case, we must determine the overall
rate law from experimental data and deduce the mechanism from the rate law (and sometimes from other data). The reaction of NO2 and CO provides an illustrative example:

NO (g) + CO(g) ⟶ CO (g) + NO(g)


2 2

For temperatures above 225 °C, the rate law has been found to be:

rate = k[ NO ][CO]
2

The reaction is first order with respect to NO2 and first-order with respect to CO. This is consistent with a single-step bimolecular mechanism and it is possible that this is the mechanism for this
reaction at high temperatures.
At temperatures below 225 °C, the reaction is described by a rate law that is second order with respect to NO2:

Access for free at OpenStax 12.6.2 https://chem.libretexts.org/@go/page/38263


2
rate = k[NO ]
2

This is consistent with a mechanism that involves the following two elementary reactions, the first of which is slower and is therefore the rate-determining step:
NO (g) + NO (g) ⟶ NO (g) + NO(g) (slow)
2 2 3

NO (g) + CO(g) ⟶ NO (g) + CO (g) (fast)


3 2 2

The rate-determining step gives a rate law showing second-order dependence on the NO2 concentration, and the sum of the two equations gives the net overall reaction.
In general, when the rate-determining (slower) step is the first step in a mechanism, the rate law for the overall reaction is the same as the rate law for this step. However, when the rate-determining
step is preceded by a step involving an equilibrium reaction, the rate law for the overall reaction may be more difficult to derive.
An elementary reaction is at equilibrium when it proceeds in both the forward and reverse directions at equal rates. Consider the dimerization of NO to N2O2, with k1 used to represent the rate
constant of the forward reaction and k-1 used to represent the rate constant of the reverse reaction:

NO + NO ⇌ N O
2 2

rate =ratereverse
forward

2
k1 [NO] = k−1 [ N O ]
2 2

If N2O2 was an intermediate in a mechanism, this expression could be rearranged to represent the concentration of N2O2 in the overall rate law expression using algebraic manipulation:
2
k1 [NO]
( ) = [ N2 O2 ]
k−1

However, once again, intermediates cannot be listed as part of the overall rate law expression, though they can be included in an individual elementary reaction of a mechanism. Example 12.6.1

will illustrate how to derive overall rate laws from mechanisms involving equilibrium steps preceding the rate-determining step.

 Example 12.6.1: Deriving the Overall Rate Law Expression for a Multistep Reaction

Mechanism Nitryl chloride (NO2Cl) decomposes to nitrogen dioxide (NO2) and chlorine gas (Cl2) according to the following mechanism:
1. 2 NO Cl(g) ⇌ ClO (g) + N O(g) + ClO(g) (fast, k1 represents the rate constant for the forward reaction and k−1 the rate constant for the reverse reaction)
2 2 2

2. N O(g) + ClO (g) ⇌ NO (g) + NOCl(g) (fast, k2 for the forward reaction, k−2 for the reverse reaction)
2 2 2

3. NOCl + ClO ⟶ NO + Cl (slow, k3 the rate constant for the forward reaction)
2 2

Determine the overall reaction, write the rate law expression for each elementary reaction, identify any intermediates, and determine the overall rate law expression.

Solution
For the overall reaction, simply sum the three steps, cancel intermediates, and combine like formulas:

2 NO Cl(g) ⟶ 2 NO (g) + Cl (g)


2 2 2

Next, write the rate law expression for each elementary reaction. Remember that for elementary reactions that are part of a mechanism, the rate law expression can be derived directly from the
stoichiometry:
k1 [ NO Cl] = k−1 [ ClO ][ N O][ClO]
2 2 2 2

k2 [ N O][ ClO ] = k−2 [ NO ][NOCl]


2 2 2

Rate = k3 [NOCl][ClO]

The third step, which is the slow step, is the rate-determining step. Therefore, the overall rate law expression could be written as Rate = k3 [NOCl][ClO]. However, both NOCl and ClO are
intermediates. Algebraic expressions must be used to represent [NOCl] and [ClO] such that no intermediates remain in the overall rate law expression.
2
k1 [ NO Cl]
2
Using elementary reaction 1, [ClO] = .
k−1 [ ClO ][ N O]
2 2

k2 [ N O][ ClO ]
Using elementary reaction 2, [NOCl] = 2 2
.
k−2 [ NO ]
2

Now substitute these algebraic expressions into the overall rate law expression and simplify:
2
k2 [ N O][ ClO ] k1 [ NO Cl]
2 2 2
rate = k3 ( )( )
k−2 [ NO ] k−1 [ ClO ][ N O]
2 2 2

2
k3 k2 k1 [ NO Cl]
2
rate =
k−2 k−1 [ NO ]
2

Notice that this rate law shows an inverse dependence on the concentration of one of the product species, consistent with the presence of an equilibrium step in the reaction mechanism.

 Exercise 12.6.1
Atomic chlorine in the atmosphere reacts with ozone in the following pair of elementary reactions:
Cl + O (g) ⟶ ClO(g) + O (g) (rate constant k1 )
3 2

ClO(g) + O ⟶ Cl(g) + O (g) (rate constant k2 )


2

Determine the overall reaction, write the rate law expression for each elementary reaction, identify any intermediates, and determine the overall rate law expression.

Answer
overall reaction: O3
(g) + O ⟶ 2 O (g)
2

rate1 = k1[O3][Cl]; rate2 = k2[ClO][O]


intermediate: ClO(g)
overall rate = k2k1[O3][Cl][O]

Summary
The sequence of individual steps, or elementary reactions, by which reactants are converted into products during the course of a reaction is called the reaction mechanism. The overall rate of a
reaction is determined by the rate of the slowest step, called the rate-determining step. Unimolecular elementary reactions have first-order rate laws, while bimolecular elementary reactions have
second-order rate laws. By comparing the rate laws derived from a reaction mechanism to that determined experimentally, the mechanism may be deemed either incorrect or plausible.

Access for free at OpenStax 12.6.3 https://chem.libretexts.org/@go/page/38263


12.6.1.1: Footnotes
1. This question is taken from the Chemistry Advanced Placement Examination and is used with the permission of the Educational Testing Service.

12.6.1.2: Glossary

bimolecular reaction
elementary reaction involving the collision and combination of two reactant species

elementary reaction
reaction that takes place precisely as depicted in its chemical equation

intermediate
molecule or ion produced in one step of a reaction mechanism and consumed in another

molecularity
number of reactant species (atoms, molecules or ions) involved in an elementary reaction

rate-determining step
(also, rate-limiting step) slowest elementary reaction in a reaction mechanism; determines the rate of the overall reaction

reaction mechanism
stepwise sequence of elementary reactions by which a chemical change takes place

termolecular reaction
elementary reaction involving the simultaneous collision and combination of three reactant species

unimolecular reaction
elementary reaction involving the rearrangement of a single reactant species to produce one or more molecules of product

This page titled 12.6: Reaction Mechanisms is shared under a CC BY 4.0 license and was authored, remixed, and/or curated by OpenStax via source content that was edited to the style and standards of the LibreTexts
platform; a detailed edit history is available upon request.

Access for free at OpenStax 12.6.4 https://chem.libretexts.org/@go/page/38263


Access for free at OpenStax 12.6.5 https://chem.libretexts.org/@go/page/38263
12.7: Catalysis
 Learning Objectives
Explain the function of a catalyst in terms of reaction mechanisms and potential energy diagrams
List examples of catalysis in natural and industrial processes

We have seen that the rate of many reactions can be accelerated by catalysts. A catalyst speeds up the rate of a reaction by lowering
the activation energy; in addition, the catalyst is regenerated in the process. Several reactions that are thermodynamically favorable
in the absence of a catalyst only occur at a reasonable rate when a catalyst is present. One such reaction is catalytic hydrogenation,
the process by which hydrogen is added across an alkene C=C bond to afford the saturated alkane product. A comparison of the
reaction coordinate diagrams (also known as energy diagrams) for catalyzed and uncatalyzed alkene hydrogenation is shown in
Figure 12.7.1.

Figure 12.7.1 : This graph compares the reaction coordinates for catalyzed and uncatalyzed alkene hydrogenation.
A graph is shown with the label, “Reaction coordinate,” on the x-axis and the label,“Energy,” on the y-axis. Approximately half-
way up the y-axis, a short portion of a black concave down curve which has a horizontal line extended from it across the graph. The
left end of this line is labeled “H subscript 2 C equals C H subscript 2 plus H subscript 2.” The black concave down curve extends
upward to reach a maximum near the height of the y-axis. The peak of this curve is labeled, “Transition state.” A double sided
arrow extends from the horizontal line to the peak of the curve. This arrow is labeled, “Activation energy of Uncatalyzed reation.”
From the peak, the curve continues downward to a second horizontally flattened region well below the origin of the curve near the
x-axis. This flattened region is shaded in blue and is labeled “H subscript 3 C dash C H subscript 3.” A double sided arrow is drawn
from the lowers part of this curve at the far right of the graph to the line extending across the graph above it. This arrow is labeled,
“capital delta H less than 0 : exothermic.” A second curve is drawn with the same flattened regions at the start and end of the curve.
The height of this curve is about two-thirds the height of the first curve. A double sided arrow is drawn from the horizontal line that
originates at the left side of the graph to the peak of this second curve. This arrow is labeled, “Activation energy of catalyzed
reaction.”
Catalysts function by providing an alternate reaction mechanism that has a lower activation energy than would be found in the
absence of the catalyst. In some cases, the catalyzed mechanism may include additional steps, as depicted in the reaction diagrams
shown in Figure 12.7.2 This lower activation energy results in an increase in rate as described by the Arrhenius equation. Note that
a catalyst decreases the activation energy for both the forward and the reverse reactions and hence accelerates both the forward and
the reverse reactions. Consequently, the presence of a catalyst will permit a system to reach equilibrium more quickly, but it has no
effect on the position of the equilibrium as reflected in the value of its equilibrium constant (see the later chapter on chemical
equilibrium).

Access for free at OpenStax 12.7.1 https://chem.libretexts.org/@go/page/38264


Figure 12.7.2 : This potential energy diagram shows the effect of a catalyst on the activation energy. The catalyst provides a
different reaction path with a lower activation energy. As shown, the catalyzed pathway involves a two-step mechanism (note the
presence of two transition states) and an intermediate species (represented by the valley between the two transitions states).
A graph is shown with the label, “Extent of reaction,” appearing in a right pointing arrow below the x-axis and the label, “Energy,”
in an upward pointing arrow just left of the y-axis. Approximately one-fifth of the way up the y-axis, a very short, somewhat
flattened portion of both a red and a blue curve are shown. This region is labeled “Reactants.” A red concave down curve extends
upward to reach a maximum near the height of the y-axis. This curve is labeled, “Uncatalyzed pathway.” From the peak, the curve
continues downward to a second horizontally flattened region at a height of about one-third the height of the y-axis. This flattened
region is labeled, “Products.” A second curve is drawn in blue with the same flattened regions at the start and end of the curve. The
height of this curve is about two-thirds the height of the first curve and just right of its maximum, the curve dips low, then rises
back and continues a downward trend at a lower height, but similar to that of the red curve. This blue curve is labeled, “Catalyzed
pathway.”

 Example 12.7.1: Using Reaction Diagrams to Compare Catalyzed Reactions

The two reaction diagrams here represent the same reaction: one without a catalyst and one with a catalyst. Identify which
diagram suggests the presence of a catalyst, and determine the activation energy for the catalyzed reaction:

In this figure, two graphs are shown. The x-axes are labeled, “Extent of reaction,” and the y-axes are labeled, “Energy ( k J ).”
The y-axes are marked off from 0 to 50 in intervals of five. In a, a blue curve is shown. It begins with a horizontal segment at
about 6. The curve then rises sharply near the middle to reach a maximum of about 32 and similarly falls to another horizontal
segment at about 10. In b, the curve begins and ends similarly, but the maximum reached near the center of the graph is only
20.

Solution
A catalyst does not affect the energy of reactant or product, so those aspects of the diagrams can be ignored; they are, as we
would expect, identical in that respect. There is, however, a noticeable difference in the transition state, which is distinctly
lower in diagram (b) than it is in (a). This indicates the use of a catalyst in diagram (b). The activation energy is the difference
between the energy of the starting reagents and the transition state—a maximum on the reaction coordinate diagram. The
reagents are at 6 kJ and the transition state is at 20 kJ, so the activation energy can be calculated as follows:

Access for free at OpenStax 12.7.2 https://chem.libretexts.org/@go/page/38264


Ea = 20 kJ − 6 kJ = 14 kJ (12.7.1)

 Exercise 12.7.1

Determine which of the two diagrams here (both for the same reaction) involves a catalyst, and identify the activation energy
for the catalyzed reaction:

In this figure, two graphs are shown. The x-axes are labeled, “Extent of reaction,” and the y-axes are labeledc “Energy (k J).”
The y-axes are marked off from 0 to 100 at intervals of 10. In a, a blue curve is shown. It begins with a horizontal segment at
about 10. The curve then rises sharply near the middle to reach a maximum of about 91, then sharply falls to about 52, again
rises sharply to about 73 and falls to another horizontal segment at about 5. In b, the curve begins and ends similarly, but the
first peak reaches about 81, drops to about 55, then rises to about 77 before falling to the horizontal region at about 5.

Answer
Diagram (b) is a catalyzed reaction with an activation energy of about 70 kJ.

12.7.1: Homogeneous Catalysts


A homogeneous catalyst is present in the same phase as the reactants. It interacts with a reactant to form an intermediate substance,
which then decomposes or reacts with another reactant in one or more steps to regenerate the original catalyst and form product. As
an important illustration of homogeneous catalysis, consider the earth’s ozone layer. Ozone in the upper atmosphere, which protects
the earth from ultraviolet radiation, is formed when oxygen molecules absorb ultraviolet light and undergo the reaction:
hv

3 O (g) −→ 2 O (g) (12.7.2)


2 3

Ozone is a relatively unstable molecule that decomposes to yield diatomic oxygen by the reverse of this equation. This
decomposition reaction is consistent with the following mechanism:
O ⟶ O +O (12.7.3)
3 2

O+O ⟶ 2O
3 2

The presence of nitric oxide, NO, influences the rate of decomposition of ozone. Nitric oxide acts as a catalyst in the following
mechanism:
NO(g) + O (g) ⟶ NO (g) + O (g) (12.7.4)
3 2 2

O (g) ⟶ O (g) + O(g)


3 2

NO (g) + O(g) ⟶ NO(g) + O (g)


2 2

The overall chemical change for the catalyzed mechanism is the same as:

2 O (g) ⟶ 3 O (g) (12.7.5)


3 2

The nitric oxide reacts and is regenerated in these reactions. It is not permanently used up; thus, it acts as a catalyst. The rate of
decomposition of ozone is greater in the presence of nitric oxide because of the catalytic activity of NO. Certain compounds that
contain chlorine also catalyze the decomposition of ozone.

Access for free at OpenStax 12.7.3 https://chem.libretexts.org/@go/page/38264


 Mario J. Molina
The 1995 Nobel Prize in Chemistry was shared by Paul J. Crutzen, Mario J. Molina (Figure 12.7.3), and F. Sherwood Rowland
“for their work in atmospheric chemistry, particularly concerning the formation and decomposition of ozone." Molina, a
Mexican citizen, carried out the majority of his work at the Massachusetts Institute of Technology (MIT).

Figure 12.7.3 : (a) Mexican chemist Mario Molina (1943 –) shared the Nobel Prize in Chemistry in 1995 for his research on (b)
the Antarctic ozone hole. (credit a: courtesy of Mario Molina; credit b: modification of work by NASA)
A photograph is shown of Mario Molina. To the right of the photo, an image of Earth’s southern hemisphere is shown with a
central circular region in purple with a radius of about half that of the entire hemisphere. Just outside this region is a narrow
royal blue band, followed by an outer thin turquoise blue band. The majority of the outermost region is green. Two small bands
of yellow are present in the lower regions of the image.
In 1974, Molina and Rowland published a paper in the journal Nature (one of the major peer-reviewed publications in the field
of science) detailing the threat of chlorofluorocarbon gases to the stability of the ozone layer in earth’s upper atmosphere. The
ozone layer protects earth from solar radiation by absorbing ultraviolet light. As chemical reactions deplete the amount of
ozone in the upper atmosphere, a measurable “hole” forms above Antarctica, and an increase in the amount of solar ultraviolet
radiation— strongly linked to the prevalence of skin cancers—reaches earth’s surface. The work of Molina and Rowland was
instrumental in the adoption of the Montreal Protocol, an international treaty signed in 1987 that successfully began phasing
out production of chemicals linked to ozone destruction.
Molina and Rowland demonstrated that chlorine atoms from human-made chemicals can catalyze ozone destruction in a
process similar to that by which NO accelerates the depletion of ozone. Chlorine atoms are generated when chlorocarbons or
chlorofluorocarbons—once widely used as refrigerants and propellants—are photochemically decomposed by ultraviolet light
or react with hydroxyl radicals. A sample mechanism is shown here using methyl chloride:
\[\ce{CH3Cl + OH ⟶ Cl + other\: products} \nonumber \]

Chlorine radicals break down ozone and are regenerated by the following catalytic cycle:
Cl + O ⟶ ClO + O
3 2

ClO + O ⟶ Cl + O
2

overall Reaction: O +O ⟶ 2 O
3 2

A single monatomic chlorine can break down thousands of ozone molecules. Luckily, the majority of atmospheric chlorine exists as
the catalytically inactive forms Cl2 and ClONO2.

12.7.2: Glucose-6-Phosphate Dehydrogenase Deficiency


Enzymes in the human body act as catalysts for important chemical reactions in cellular metabolism. As such, a deficiency of a
particular enzyme can translate to a life-threatening disease. G6PD (glucose-6-phosphate dehydrogenase) deficiency, a genetic
condition that results in a shortage of the enzyme glucose-6-phosphate dehydrogenase, is the most common enzyme deficiency in
humans. This enzyme, shown in Figure 12.7.4, is the rate-limiting enzyme for the metabolic pathway that supplies NADPH to cells
(FIgure 12.7.5).

Access for free at OpenStax 12.7.4 https://chem.libretexts.org/@go/page/38264


Figure 12.7.4: Glucose-6-phosphate dehydrogenase is a rate-limiting enzyme for the metabolic pathway that supplies NADPH to
cells.
A colorful model of the Glucose-6-phosphate dehydrogenase structure is shown. The molecule has two distinct lobes which are
filled with spiraled ribbon-like regions of yellow, lavender, blue, silver, green, and pink.
A disruption in this pathway can lead to reduced glutathione in red blood cells; once all glutathione is consumed, enzymes and
other proteins such as hemoglobin are susceptible to damage. For example, hemoglobin can be metabolized to bilirubin, which
leads to jaundice, a condition that can become severe. People who suffer from G6PD deficiency must avoid certain foods and
medicines containing chemicals that can trigger damage their glutathione-deficient red blood cells.

Figure 12.7.5 : In the mechanism for the pentose phosphate pathway, G6PD catalyzes the reaction that regulates NAPDH, a co-
enzyme that regulates glutathione, an antioxidant that protects red blood cells and other cells from oxidative damage.
A reaction mechanism is diagrammed in this figure. At the left, the name Glucose is followed by a horizontal, right pointing arrow,
labeled, “Hexokinase.” Below this arrow and to the left is a yellow star shape labeled, “A T P.” A curved arrow extends from this
shape to the right pointing arrow, and down to the right to a small brown oval labeled, “A D P.” To the right of the horizontal arrow
is the name Glucose 6 phosphate, which is followed by another horizontal, right pointing arrow which is labeled, “G 6 P D.” A
small orange rectangle below and left of this arrow is labeled “N A D P superscript plus.” A curved arrow extends from this shape
to the right pointing arrow, and down to the right to a small salmon-colored rectangle labeled “N A P D H.” A curved arrow
extends from this shape below and to the left, back to the orange rectangle labeled, “N A D P superscript plus.” Another curved
arrow extends from a green oval labeled “G S S G” below the orange rectangle, up to the arrow curving back to the orange
rectangle. This last curved arrow continues on to the lower right to a second green oval labeled, “G S H.” The end of this curved
arrow is labeled, “Glutathione reductase.” To the right of the rightmost horizontal arrow appears the name 6 phosphogluconate.

12.7.3: Heterogeneous Catalysts


A heterogeneous catalyst is a catalyst that is present in a different phase (usually a solid) than the reactants. Such catalysts
generally function by furnishing an active surface upon which a reaction can occur. Gas and liquid phase reactions catalyzed by
heterogeneous catalysts occur on the surface of the catalyst rather than within the gas or liquid phase.
Heterogeneous catalysis has at least four steps:
1. Adsorption of the reactant onto the surface of the catalyst
2. Activation of the adsorbed reactant
3. Reaction of the adsorbed reactant
4. Diffusion of the product from the surface into the gas or liquid phase (desorption).
Any one of these steps may be slow and thus may serve as the rate determining step. In general, however, in the presence of the
catalyst, the overall rate of the reaction is faster than it would be if the reactants were in the gas or liquid phase.

Access for free at OpenStax 12.7.5 https://chem.libretexts.org/@go/page/38264


Figure 12.7.6 illustrates the steps that chemists believe to occur in the reaction of compounds containing a carbon–carbon double
bond with hydrogen on a nickel catalyst. Nickel is the catalyst used in the hydrogenation of polyunsaturated fats and oils (which
contain several carbon–carbon double bonds) to produce saturated fats and oils (which contain only carbon–carbon single bonds).

Figure 12.7.6 : There are four steps in the catalysis of the reaction C H + H ⟶ C H by nickel. (a) Hydrogen is adsorbed on
2 4 2 2 6

the surface, breaking the H–H bonds and forming Ni–H bonds. (b) Ethylene is adsorbed on the surface, breaking the π-bond and
forming Ni–C bonds. (c) Atoms diffuse across the surface and form new C–H bonds when they collide. (d) C2H6 molecules escape
from the nickel surface, since they are not strongly attracted to nickel.
In this figure, four diagrams labeled a through d are shown. In each, a green square surface is shown in perspective to provide a
three-dimensional appearance. In a, the label “N i surface” is placed above with a line segment extending to the green square. At
the lower left and upper right, pairs of white spheres bonded tougher together appear as well as white spheres on the green surface.
Black arrows are drawn from each of the white spheres above the surface to the white sphere on the green surface. In b, the white
spheres are still present on the green surface. Near the center of this surface is a molecule with two central black spheres with a
double bond indicated by two horizontal black rods between them. Above and below to the left and right, a total of four white
spheres are connected to the black spheres with white rods. A line segment extends from this structure to the label, “Ethylene
absorbed on surface breaking pi bonds.” Just above this is a nearly identical structure greyed out with three downward pointing
arrows to the black and white structure to indicate downward motion. The label “Ethylene” at the top of the diagram is connected
to the greyed out structure with a line segment. In c, the diagram is very similar to b except that the greyed out structure and labels
are gone and one of the white spheres near the black and white structure in each pair on the green surface is greyed out. Arrows
point from the greyed out white spheres to the double bond between the two black spheres. In d, only a single white sphere remains
from each pair in the green surface. A curved arrow points from the middle of the green surface to a model above with two central
black spheres with a single black rod indicating a single bond between them. Each of the black rods has three small white spheres
bonded as indicated by white rods between the black spheres and the small white spheres. The four bonds around each black sphere
are evenly distributed about the black spheres.
Other significant industrial processes that involve the use of heterogeneous catalysts include the preparation of sulfuric acid, the
preparation of ammonia, the oxidation of ammonia to nitric acid, and the synthesis of methanol, CH3OH. Heterogeneous catalysts
are also used in the catalytic converters found on most gasoline-powered automobiles (Figure 12.7.7).

 Automobile Catalytic Converters

Scientists developed catalytic converters to reduce the amount of toxic emissions produced by burning gasoline in internal
combustion engines. Catalytic converters take advantage of all five factors that affect the speed of chemical reactions to ensure
that exhaust emissions are as safe as possible.
By utilizing a carefully selected blend of catalytically active metals, it is possible to effect complete combustion of all carbon-
containing compounds to carbon dioxide while also reducing the output of nitrogen oxides. This is particularly impressive
when we consider that one step involves adding more oxygen to the molecule and the other involves removing the oxygen
(Figure 12.7.6).

Access for free at OpenStax 12.7.6 https://chem.libretexts.org/@go/page/38264


Figure 12.7.6 : A catalytic converter allows for the combustion of all carbon-containing compounds to carbon dioxide, while at
the same time reducing the output of nitrogen oxide and other pollutants in emissions from gasoline-burning engines.
An image is shown of a catalytic converter. At the upper left, a blue arrow pointing into a pipe that enters a larger, widened
chamber is labeled, “Dirty emissions.” A small black arrow that points to the lower right is positioned along the upper left side
of the widened region. This arrow is labeled, “Additional oxygen from air pump.” The image shows the converter with the
upper surface removed, exposing a red-brown interior. The portion of the converter closest to the dirty emissions inlet shows
small, round components in an interior layer. This layer is labeled “Three-way reduction catalyst.” The middle region shows
closely packed small brown rods that are aligned parallel to the dirty emissions inlet pipe. The final quarter of the interior of
the catalytic converter again shows a layer of closely packed small red brown circles. Two large light grey arrows extend from
this layer to the open region at the lower right of the image to the label “Clean emissions.”
Most modern, three-way catalytic converters possess a surface impregnated with a platinum-rhodium catalyst, which catalyzes
the conversion nitric oxide into dinitrogen and oxygen as well as the conversion of carbon monoxide and hydrocarbons such as
octane into carbon dioxide and water vapor:
2 NO (g) ⟶ N (g) + 2 O (g)
2 2 2

[5pt]2 CO(g) + O (g) ⟶ 2 CO (g)


2 2

[5pt]2 C H (g) + 25 O (g) ⟶ 16 CO (g) + 18 H O(g)


8 18 2 2 2

In order to be as efficient as possible, most catalytic converters are preheated by an electric heater. This ensures that the metals in
the catalyst are fully active even before the automobile exhaust is hot enough to maintain appropriate reaction temperatures.

12.7.4: Enzyme Structure and Function


The study of enzymes is an important interconnection between biology and chemistry. Enzymes are usually proteins (polypeptides)
that help to control the rate of chemical reactions between biologically important compounds, particularly those that are involved in
cellular metabolism. Different classes of enzymes perform a variety of functions, as shown in Table 12.7.1.
ATP hydrolysis.”" data-quail-id="131" data-mt-width="1016">
Table 12.7.1 : Classes of Enzymes and Their Functions
Class Function

oxidoreductases redox reactions

transferases transfer of functional groups

hydrolases hydrolysis reactions

lyases group elimination to form double bonds

isomerases isomerization

ligases bond formation with ATP hydrolysis

Enzyme molecules possess an active site, a part of the molecule with a shape that allows it to bond to a specific substrate (a
reactant molecule), forming an enzyme-substrate complex as a reaction intermediate. There are two models that attempt to explain

Access for free at OpenStax 12.7.7 https://chem.libretexts.org/@go/page/38264


how this active site works. The most simplistic model is referred to as the lock-and-key hypothesis, which suggests that the
molecular shapes of the active site and substrate are complementary, fitting together like a key in a lock. The induced fit
hypothesis, on the other hand, suggests that the enzyme molecule is flexible and changes shape to accommodate a bond with the
substrate. This is not to suggest that an enzyme’s active site is completely malleable, however. Both the lock-and-key model and
the induced fit model account for the fact that enzymes can only bind with specific substrates, since in general a particular enzyme
only catalyzes a particular reaction (Figure 12.7.7).

Figure 12.7.7 : (a) According to the lock-and-key model, the shape of an enzyme’s active site is a perfect fit for the substrate. (b)
According to the induced fit model, the active site is somewhat flexible, and can change shape in order to bond with the substrate.
A diagram is shown of two possible interactions of an enzyme and a substrate. In a, which is labeled “Lock-and-key,” two diagrams
are shown. The first shows a green wedge-like shape with two small depressions in the upper surface of similar size, but the
depression on the left has a curved shape, and the depression on the right has a pointed shape. This green shape is labeled
“Enzyme.” Just above this shape are two smaller, irregular, lavender shapes each with a projection from its lower surface. The
lavender shape on the left has a curved projection which matches the shape of the depression on the left in the green shape below.
This projection is shaded orange and has a curved arrow extending from in to the matching depression in the green shape below.
Similarly, the lavender shape on the right has a projection with a pointed tip which matches the shape of the depression on the right
in the green shape below. This projection is shaded orange and has a curved arrow extending from in to the matching depression in
the green shape below. Two line segments extend from the depressions in the green shape to form an inverted V shape above the
depressions. Above this and between the lavender shapes is the label, “Active site is proper shape.” The label “Substrates” is at the
very top of the diagram with line segments extending to the two lavender shapes. To the right of this diagram is a second diagram
showing the lavender shapes positioned next to each other, fit snugly into the depressions in the green shape, which is labeled
“Enzyme.” Above this diagram is the label, “Substrate complex formed.” In b, which is labeled “Induced fit,” two diagrams are
shown. The first shows a green wedge-like shape with two small depressions in the upper surface of similar size, but irregular
shape. This green shape is labeled “Enzyme.” Just above this shape are two smaller irregular lavender shapes each with a projection
from its lower surface. The lavender shape on the left has a curved projection. This projection is shaded orange and has a curved
arrow extending from it to the irregular depression just below it in the green shape below. Similarly, the lavender shape on the right
has a projection with a pointed tip. This projection is shaded orange and has a curved arrow extending from it to the irregular
depression just below it in the green shape below. Two line segments extend from the depressions in the green shape to form an
inverted V shape above the depressions. Above this and between the lavender shapes is the label, “Active site changes to fit.” The
label, “Substrates” is at the very top of the diagram with line segments extending to the two lavender shapes. To the right of this
diagram is a second diagram showing the purple shapes positioned next to each other, fit snugly into the depressions in the green
shape, which is labeled “Enzyme.” Above this diagram is the label “Substrate complex formed.” The projections from the lavender
shapes match the depression shapes in the green shape, resulting in a proper fit.

Summary
Catalysts affect the rate of a chemical reaction by altering its mechanism to provide a lower activation energy. Catalysts can be
homogenous (in the same phase as the reactants) or heterogeneous (a different phase than the reactants).

Footnotes
1. “The Nobel Prize in Chemistry 1995,” Nobel Prize.org, accessed February 18, 2015, Nobel Prizes Chemistry
[www.nobelprize.org].

Glossary
heterogeneous catalyst
catalyst present in a different phase from the reactants, furnishing a surface at which a reaction can occur

homogeneous catalyst

Access for free at OpenStax 12.7.8 https://chem.libretexts.org/@go/page/38264


catalyst present in the same phase as the reactants

This page titled 12.7: Catalysis is shared under a CC BY 4.0 license and was authored, remixed, and/or curated by OpenStax via source content
that was edited to the style and standards of the LibreTexts platform; a detailed edit history is available upon request.

Access for free at OpenStax 12.7.9 https://chem.libretexts.org/@go/page/38264


12.E: Kinetics (Exercises)
12.E.1: 12.1: Chemical Reaction Rates
12.E.1.1: Q12.1.1
What is the difference between average rate, initial rate, and instantaneous rate?

Solution
First, a general reaction rate must be defined to know what any variation of a rate is. The reaction rate is defined as the measure
of the change in concentration of the reactants or products per unit time. The rate of a chemical reaction is not a constant and
rather changes continuously, and can be influenced by temperature. Rate of a reaction can be defined as the disappearance of
any reactant or appearance of any product. Thus, an average rate is the average reaction rate over a given period of time in the
reaction, the instantaneous rate is the reaction rate at a specific given moment during the reaction, and the initial rate is the
instantaneous rate at the very start of the reaction (when the product begins to form).
The instantaneous rate of a reaction can be denoted as
Δ[concentration]
lim
Δt→0 Δt

12.E.1.2: Q12.1.2
Ozone decomposes to oxygen according to the equation 2 O (g) ⟶ 3 O (g) . Write the equation that relates the rate expressions
3 2

for this reaction in terms of the disappearance of O3 and the formation of oxygen.

Solution
For the general reaction, aA ---> bB, the rate of the reaction can be expressed in terms of the disappearance of A or the
appearance of B over a certain time period as follows.

1 Δ[A] 1 Δ[B] 1 Δ[C ] 1 Δ[D]


− =− = = (12.E.1)
a Δt b Δt c Δt d Δt

We want the rate of a reaction to be positive, but the change in the concentration of a reactant, A, will be negative because it is
being used up to be transformed into product, B. Therefore, when expressing the rate of the reaction in terms of the change in
the concentration of A, it is important to add a negative sign in front to ensure the overall rate positive.
Lastly, the rate must be normalized according to the stoichiometry of the reaction. In the decomposition of ozone to oxygen,
two moles of ozone form three moles of oxygen gas. This means that the increase in oxygen gas will be 1.5 times as great as the
decrease in ozone. Because the rate of the reaction should be able to describe both species, we divide the change in
concentration by its stoichiometric coefficient in the balanced reaction equation to deal with this issue.
Therefore, the rate of the reaction of the decomposition of ozone into oxygen gas can be described as follows:
Δ[O3] Δ[O2]
Rate = − = (12.E.2)
2ΔT 3ΔT

Answer
$$Rate=-\frac{Δ[O3]}{2ΔT}=\frac{Δ[O2]}{3ΔT}\]

12.E.1.3: Q12.1.3
In the nuclear industry, chlorine trifluoride is used to prepare uranium hexafluoride, a volatile compound of uranium used in the
separation of uranium isotopes. Chlorine trifluoride is prepared by the reaction Cl (g) + 3 F (g) ⟶ 2 ClF (g) . Write the
2 2 3

equation that relates the rate expressions for this reaction in terms of the disappearance of Cl2 and F2 and the formation of ClF3.

Solution
In this problem we are asked to write the equation that relates rate expressions in terms of disappearance of the reactants of the
equation and in terms of the formation of the product. A reaction rate gives insight to how rate is affected as a function of

Access for free at OpenStax 12.E.1 https://chem.libretexts.org/@go/page/46044


concentration of the substances in the equation. Rates can often be expressed on graphs of concentration vs time expressed in
change (Δ) of concentration and time and in a short enough time interval, the instantaneous rate can be approximated. If we
were to analyze the reaction given, the graph would demonstrate that Cl2 decreases, that F2 decreases 3 times as quickly, and
then ClF3 increases at a rate doubles. The reactants are being used and converted to product so they decrease while products
increase.
For this problem, we can apply the general formula of a rate to the specific aspects of a problem where the general form
follows:

aA + bB ⟶ cC + dD

.
Δ[A] Δ[B] Δ[C ] Δ[D]
And the rate can then be written as rate = − 1

a
=−
Δt
1

b Δt
=
1

c Δt
=
1

d Δt
. Here the negative signs are used to keep
the convention of expressing rates as positive numbers.
In this specific case we use the stoichiometry to get the specific rates of disappearance and formation (back to what was said in
the first paragraph). So, the problem just involves referring the to the equation and its balanced coefficients. Based upon the
equation we see that Cl2 is a reactant and has no coefficient, F2 has a coefficient of 3 and is also used up, and then ClF3 is a
product that increases two-fold with a coefficient of 2. So, the rate here can be written as:
Δ[C l2 ] 1 Δ[ F2 ] 1 Δ[C lF3 ]
rate = − =− =
Δt 3 Δt 2 Δt

Answer

1 Δ[ CIF3 ] Δ[ Cl ]
2 1 Δ[ F2 ]
rate = + =− =−
2 Δt Δt 3 Δt

12.E.1.4: Q12.1.4
A study of the rate of dimerization of C4H6 gave the data shown in the table:

2C H ⟶ C H
4 6 8 12

Time (s) 0 1600 3200 4800 6200

[C4H6] (M) 1.00 × 10−2 5.04 × 10−3 3.37 × 10−3 2.53 × 10−3 2.08 × 10−3

a. Determine the average rate of dimerization between 0 s and 1600 s, and between 1600 s and 3200 s.
b. Estimate the instantaneous rate of dimerization at 3200 s from a graph of time versus [C4H6]. What are the units of this rate?
c. Determine the average rate of formation of C8H12 at 1600 s and the instantaneous rate of formation at 3200 s from the rates
found in parts (a) and (b).

Solution
1.) The average rate of dimerization is the change in concentration of a reactant per unit time. In this case it would be:
Δ[ C4 H6 ]
rate of dimerization = −
Δt

Rate of dimerization between 0 s and 1600 s:


−3 −2
5.04×10 M−1.00×10 M
rate of dimerization = −
1600s−0s

−6 M
rate of dimerization = 3.10 × 10
s

Rate of dimerization between 1600 s and 3200 s:


−3 −3
3.37×10 M−5.04×10 M
rate of dimerization = −
3200s−1600s

−6 M
rate of dimerization = 1.04 × 10
s

2.) The instantaneous rate of dimerization at 3200 s can be found by graphing time versus [C4H6].

Access for free at OpenStax 12.E.2 https://chem.libretexts.org/@go/page/46044


Because you want to find the rate of dimerization at 3200 s, you need to find the slope between 1600 s and 3200 s and also 3200
s and 4800 s.
For the slope between 1600 s and 3200 s use the points (1600 s, 5.04 x 10-3 M) and (3200 s, 3.37 x 10-3 M)
−3 −3
3.37×10 M−5.04×10 M

3200s−1600s

−0.00167M

1600s

−6 M
−1.04 × 10
s

For the slope between 3200 s and 4800 s use the points (3200s, 3.37 x 10-3 M) and (4800s, 2.53 x 10-3 M)
−3 −3
2.53×10 M−3.37×10 M

4800s−3200s

−4
−8.4×10 M

1600s

−7 M
−5.25 × 10
s

Take the two slopes you just found and find the average of them to get the instantaneous rate of dimerization.
−6 M −7 M
−1.04×10 +−5.25×x10
s s

−6 M
−1.565×10
s

− M
−7.83 × 10 7
s

The instantaneous rate of dimerization is −7.83 × 10 −


7
M

s
and the units of this rate is M

s
.
3.) The average rate of formation of C8H12 at 1600 s and the instantaneous rate of formation at 3200 s can be found by using
our answers from part a and b. If you look back up at the original equation, you could see that C4H6 and C8H12 are related in a
two to one ratio. For every two moles of C4H6 used, there is one mole of C8H12 produced.
For this reaction, the average rate of dimerization and the average rate of formation can be linked through this equation:
−1 Δ[ C4 H6 ] Δ[ C8 H1 2 ]
=
2 Δt Δt

Notice that reactant side is negative because the reactants are being used up in the reaction.
So, for the average rate of formation of C8H12 at 1600 s, use the rate of dimerization between 0 s and 1600 s we found earlier
and plug into the equation:
−1 Δ[ C8 H12 ]
−6 M
× 3.10 × 10 =
2 s Δt

Δ[ C8 H12 ]
−6 M
= 1.55 × 10
Δt s

The average rate of formation for C8H12 at 1600 s is 1.55 × 10


−6 M

s
. The rate of formation will be positive because products
are being formed.
The instantaneous rate of formation for C8H12 can be linked to the instantaneous rate of dimerization by this equation:

Access for free at OpenStax 12.E.3 https://chem.libretexts.org/@go/page/46044


−1 d[ C4 H6 ] d[ C8 H1 2 ]
=
2 dt dt

So, for the instantaneous rate of formation for C8H12 at 3200 s, use the value of instantaneous rate of dimerization at 3200 s
found earlier and plug into the equation:
−1 M d[ C8 H12 ]

× −7.83 × 10 7 =
2 s dt

d[ C8 H12 ]
−7 M
= −3.92 × 10
dt s

The instantaneous rate of formation for C8H12 at 3200 s is −3.92 × 10 −


7
M

Answer
a. 3.10 × 10
−6 M
s
and 1.04 × 10 −6 M
s

b. −7.83 × 10

7
M

s
and M

c. −3.92 × 10

7
M

12.E.1.5: Q12.1.5
A study of the rate of the reaction represented as 2A ⟶ B gave the following data:

Time (s) 0.0 5.0 10.0 15.0 20.0 25.0 35.0

[A] (M) 1.00 0.952 0.625 0.465 0.370 0.308 0.230

a. Determine the average rate of disappearance of A between 0.0 s and 10.0 s, and between 10.0 s and 20.0 s.
b. Estimate the instantaneous rate of disappearance of A at 15.0 s from a graph of time versus [A]. What are the units of this rate?
c. Use the rates found in parts (a) and (b) to determine the average rate of formation of B between 0.00 s and 10.0 s, and the
instantaneous rate of formation of B at 15.0 s.

Solution
Equations: −△A

△time
and Rate= −△A

2△time
=
△B

time

−△A
Solve: 1.)The change in A from 0s to 10s is .625-1=-.375 so △time
=.375/10= 0.0374 M/s

Similarly, the change in A from 10 to 20 seconds is .370-.625=-.255 so −△A

△time
=.255/20-10= 0.0255M/s
2.) We can estimate the rate law graphing the points against different order equations to determine the right order.
Zero Order:
d[A]
= −k
dt

A t

∫ d[A] = −k ∫ dt
A∘ 0

[A] = −kt + [ A∘ ]

First Order:
d[A]
= −k[A]
dt

A
d[A]
∫ = −kdt
A∘ [A]

Ln(A) = −kt + Ln(A∘ )

Second Order:

Access for free at OpenStax 12.E.4 https://chem.libretexts.org/@go/page/46044


d[A]
2
= −k[A]
dt

A t
d[A]
∫ = −k ∫ dt
2
A∘ [A] 0

1 1
= kt +
[A] [ A∘ ]

Now that we have found the linear from of each order we will plot the points vs an [A] y-axis, a Ln(A) y-axis, and a 1/[A]
y-axis. whichever of the plots has the most linear points will give us a good idea of the order and the slope will be the k
value.

Here we notice that the second order is most linear so we conclude the Rate to be..
−d[A]
2
= k[A]
2dt

At 15 seconds [A]=.465 and from the slope of the graph we find k=.116.so if we plug this data in and multiply both sides
by 2 to get rid of the 2 in the denominator on the left side of the equation we find that the rate of disappearance of A is .05
M/s where the units are equivalent to [mol*L-1*s-1]
3.) Using the equation −△A
=
2△time
we divide the rates in part a and b in half to get .0188 M/s from 0 to 10 seconds
△B

time

and .025 M/s for the estimated instantaneous rate at 15s.

Answer
(a) average rate, 0 − 10 s = 0.0375 mol L−1 s−1; average rate, 12 − 18 s = 0.0225 mol L−1 s−1; (b) instantaneous rate, 15 s =
0.0500 mol L−1 s−1; (c) average rate for B formation = 0.0188 mol L−1 s−1; instantaneous rate for B formation = 0.0250 mol L−1
s−1

12.E.1.6: Q12.1.6
Consider the following reaction in aqueous solution:
− − +
5 Br (aq) + BrO 3 (aq) + 6 H (aq) ⟶ 3 Br (aq) + 3 H O(l)
2 2

– −4
If the rate of disappearance of Br (aq) at a particular moment during the reaction is 3.5 × 10 M s−1, what is the rate of appearance
of Br2(aq) at that moment?

Solution
Step 1. Define the rate of the reaction.
Recall:
For the general reaction: aA + bB → cC+ dD
Δ[A] Δ[B] Δ[C ] Δ[D]
rate = − =− = =
aΔt bΔt cΔt dΔt

So, for the reaction: 5Br −


(aq) + BrO

3
(aq) + 6 H
+
→ 3Br2 (aq) + 3 H2 O(l)

− − +
Δ[Br ] Δ[BrO ] Δ[ H ] Δ[Br2 ] H2 O
The rate would be: rate = − 5Δt
=−
Δt
3
=−
6Δt
=
3Δt
=
3Δt

Access for free at OpenStax 12.E.5 https://chem.libretexts.org/@go/page/46044


Step 2. Since we are given the rate for the disappearance of Br (aq) is − −
3.5x 10
−1
4M s , and we want to find the rate of
appearance of Br (aq). Therefore we set the two rates equal to each other.
2


Δ[Br ] Δ[Br2 ]
rate = − =
5Δt 3Δt


Δ[Br ]
And,− Δt
= −3.5x 10
−4
Ms
−1

Δ[Br2 ]
So, 3.5x 10 −4
Ms
−1
= 5

3 Δt

Step 3. Now solve the equation.


−4
(3.5x10 )(3) Δ[Br2 ]
=
5 Δt

Δ[Br2 ]
−4 −1
= 2.1x 10 Ms
Δt

Answer
Δ[Br2 ]
−4 −1
= 2.1x 10 Ms
Δt

12.E.2: 12.2: Factors Affecting Reaction Rates


12.E.2.1: Q12.2.1
Describe the effect of each of the following on the rate of the reaction of magnesium metal with a solution of hydrochloric acid: the
molarity of the hydrochloric acid, the temperature of the solution, and the size of the pieces of magnesium.

Solution
Molarity of Hydrochloric Acid
Reaction rates are affected by the frequency at which molecules collide. High Molarity=High Concentration which means
more molecules are available to collide thus a faster reaction that one with a low molarity of HCl at a fixed volume.
Temperature of Solution
Higher temperatures increase the rate of reaction because molecules move faster thus colliding more frequently
increasing temperatures allows for more particles to move past activation energy barrier to start the reaction
Size of pieces of Magnesium
reaction rate is dependent on solid reactant size; smaller pieces increases the chance of collision because they enable a
greater surface area thus faster reaction rate

12.E.2.2: Q12.2.2
Go to the PhET Reactions & Rates interactive. Use the Single Collision tab to represent how the collision between monatomic
oxygen (O) and carbon monoxide (CO) results in the breaking of one bond and the formation of another. Pull back on the red
plunger to release the atom and observe the results. Then, click on “Reload Launcher” and change to “Angled shot” to see the
difference.
a. What happens when the angle of the collision is changed?
b. Explain how this is relevant to rate of reaction.

Solution
According to the collision theory, there are many factors that cause a reaction to happen, with three of the factors being how
often the molecules or atoms collide, the molecules' or atoms' orientations, and if there is sufficient energy for the reaction to
happen. So, if the angle of the plunger is changed, the atom that is shot (a lone Oxygen atom in this case) will hit the other
molecule (CO in this case) at a different spot and at a different angle, therefore changing the orientation and the number of
proper collisions will most likely not cause for a reaction to happen. Thanks to the simulation, we can see that this is true:
depending on the angle selected, the atom may take a long time to collide with the molecule and, when a collision does occur, it
may not result in the breaking of the bond and the forming of the other (no reaction happens).

Access for free at OpenStax 12.E.6 https://chem.libretexts.org/@go/page/46044


In this particular case, the rate of the reaction will decrease because, by changing the angle, the molecules or atoms won't
collide with the correct orientation or as often with the correct orientation.

12.E.2.3: Q12.2.3
In the PhET Reactions & Rates interactive, use the “Many Collisions” tab to observe how multiple atoms and molecules interact
under varying conditions. Select a molecule to pump into the chamber. Set the initial temperature and select the current amounts of
each reactant. Select “Show bonds” under Options. How is the rate of the reaction affected by concentration and temperature?

12.E.2.4: S12.2.3
Based on the Collision Theory, a reaction will only occur if the molecules collide with proper orientation and with sufficient energy
required for the reaction to occur. The minimum energy the molecules must collide with is called the activation energy (energy of
transition state).
Increasing the concentration of reactants increases the probability that reactants will collide in the correct orientation since there are
more reactants in the same volume of space. Therefore, increasing the concentration of reactants would increase the rate of the
reaction. Decreasing the concentration of reactants would decrease the rate of reaction because the overall number of possible
collisions would decrease.
Temperature is directly related the the kinetic energy of molecules and activation energy E is the minimum energy required for a
a

reaction to occur and doesn't change for a reaction. Increasing the temperature increases the kinetic energy of the reactants meaning
the reactants will move faster and collide with each other more frequently. Therefore, increasing the temperature increase the rate
of the reaction. Decreasing the temperature decreases the rate of reaction since the molecules will have less kinetic energy, move
slower, and therefore collide with each other less frequently.

12.E.2.5: Q12.2.4
In the PhET Reactions & Rates interactive, on the Many Collisions tab, set up a simulation with 15 molecules of A and 10
molecules of BC. Select “Show Bonds” under Options.
a. Leave the Initial Temperature at the default setting. Observe the reaction. Is the rate of reaction fast or slow?
b. Click “Pause” and then “Reset All,” and then enter 15 molecules of A and 10 molecules of BC once again. Select “Show
Bonds” under Options. This time, increase the initial temperature until, on the graph, the total average energy line is completely
above the potential energy curve. Describe what happens to the reaction.

Solution
a. On the simulation, we select the default setting and the reaction A+BC. In the default setting, we see frequent collisions, a
low initial temperature, and a total average energy lower than the energy of activation. The collision theory states that the rate of
a reaction is directly proportional to (the fraction of molecules with required orientation), (fractions of collisions with required
energy), and (collision frequency). Although we see moving and frequently colliding reactants, the rate of the forward reaction
is actually slow because it takes a long time for the products, AB and C, to start appearing. This is mainly because the fractions
of collisions with required energy is low, coming from the average energy of the molecules being lower than the energy of
activation.
b. The reaction proceeds at an even faster rate. Again, the collision theory states that the rate of a reaction is directly
proportional to (the fraction of molecules with required orientation), (fractions of collisions with required energy), and
(collision frequency). Because molecules have a higher amount of energy, they have more kinetic energy. With an increased
kinetic energy, the molecules not only collide more but also increase in the fraction of collision. However, the forward reaction
and the backward reaction both proceed at a fast rate, so both happen almost simultaneously. It takes a shorter time for both
reactions to happen. With both of the reactions adding up together overall, there is eventually a state of equilibrium. The process
at which equilibrium is reached, however, is faster. Therefore, the amount of products of A+BC stays the same after a while.

12.E.3: 12.3: Rate Laws


12.E.3.1: Q12.3.1
How do the rate of a reaction and its rate constant differ?

Access for free at OpenStax 12.E.7 https://chem.libretexts.org/@go/page/46044


12.E.3.2: S12.3.1
The rate of a reaction or reaction rate is the change in the concentration of either the reactant or the product over a period of time. If
the concentrations change, the rate also changes.
Rate for A → B:

The rate constant (k) is a proportionality constant that relates the reaction rates to reactants. If the concentrations change, the rate
constant does not change.
For a reaction with the general equation: aA + bB → cC + dD
the experimentally determined rate law usually has the following form:

12.E.3.3: Q12.3.2
Doubling the concentration of a reactant increases the rate of a reaction four times. With this knowledge, answer the following
questions:
a. What is the order of the reaction with respect to that reactant?
b. Tripling the concentration of a different reactant increases the rate of a reaction three times. What is the order of the reaction
with respect to that reactant?

Solution
(a) 2; (b) 1

12.E.3.4: Q12.3.3
Tripling the concentration of a reactant increases the rate of a reaction nine times. With this knowledge, answer the following
questions:
a. What is the order of the reaction with respect to that reactant?
b. Increasing the concentration of a reactant by a factor of four increases the rate of a reaction four times. What is the order of the
reaction with respect to that reactant?

12.E.3.5: Q12.3.4
How much and in what direction will each of the following affect the rate of the reaction:
CO(g) + NO (g) ⟶ CO
2 2
(g) + NO(g) if the rate law for the reaction is rate = k[NO ] ?
2
2

a. Decreasing the pressure of NO2 from 0.50 atm to 0.250 atm.


b. Increasing the concentration of CO from 0.01 M to 0.03 M.

Solution
(a) The process reduces the rate by a factor of 4. (b) Since CO does not appear in the rate law, the rate is not affected.

12.E.3.6: Q12.3.5
How will each of the following affect the rate of the reaction: CO(g) + NO (g) ⟶ CO (g) + NO(g)
2 2
if the rate law for the
reaction is rate = k[NO ][CO] ?
2

a. Increasing the pressure of NO2 from 0.1 atm to 0.3 atm


b. Increasing the concentration of CO from 0.02 M to 0.06 M.

12.E.3.7: Q12.3.6
Regular flights of supersonic aircraft in the stratosphere are of concern because such aircraft produce nitric oxide, NO, as a
byproduct in the exhaust of their engines. Nitric oxide reacts with ozone, and it has been suggested that this could contribute to
depletion of the ozone layer. The reaction NO + O ⟶ NO + O is first order with respect to both NO and O3 with a rate
3 2 2

Access for free at OpenStax 12.E.8 https://chem.libretexts.org/@go/page/46044


constant of 2.20 × 107 L/mol/s. What is the instantaneous rate of disappearance of NO when [NO] = 3.3 × 10−6 M and [O3] = 5.9 ×
10−7 M?

Solution
4.3 × 10−5 mol/L/s

12.E.3.8: Q12.3.7
Radioactive phosphorus is used in the study of biochemical reaction mechanisms because phosphorus atoms are components of
many biochemical molecules. The location of the phosphorus (and the location of the molecule it is bound in) can be detected from
the electrons (beta particles) it produces:
32 32 −
15
P ⟶ S+e
16

Rate = 4.85 × 10−2 day −1 32


[ P]

What is the instantaneous rate of production of electrons in a sample with a phosphorus concentration of 0.0033 M?

12.E.3.9: Q12.3.8
The rate constant for the radioactive decay of 14C is 1.21 × 10−4 year−1. The products of the decay are nitrogen atoms and electrons
(beta particles):
6 6 −
14
C ⟶ N+e
14

6
rate = k[ C]
14

What is the instantaneous rate of production of N atoms in a sample with a carbon-14 content of 6.5 × 10−9 M?

Solution
7.9 × 10−13 mol/L/year

12.E.3.10: Q12.3.9
What is the instantaneous rate of production of N atoms Q12.3.8 in a sample with a carbon-14 content of 1.5 × 10−9 M?

12.E.3.11: Q12.3.10
The decomposition of acetaldehyde is a second order reaction with a rate constant of 4.71 × 10−8 L/mol/s. What is the
instantaneous rate of decomposition of acetaldehyde in a solution with a concentration of 5.55 × 10−4 M?

12.E.3.12: Q12.3.11
Alcohol is removed from the bloodstream by a series of metabolic reactions. The first reaction produces acetaldehyde; then other
products are formed. The following data have been determined for the rate at which alcohol is removed from the blood of an
average male, although individual rates can vary by 25–30%. Women metabolize alcohol a little more slowly than men:

[C2H5OH] (M) 4.4 × 10−2 3.3 × 10−2 2.2 × 10−2

Rate (mol/L/h) 2.0 × 10−2 2.0 × 10−2 2.0 × 10−2

Determine the rate equation, the rate constant, and the overall order for this reaction.

Solution
rate = k; k = 2.0 × 10−2 mol/L/h (about 0.9 g/L/h for the average male); The reaction is zero order.

12.E.3.13: Q12.3.12
Under certain conditions the decomposition of ammonia on a metal surface gives the following data:

[NH3] (M) 1.0 × 10−3 2.0 × 10−3 3.0 × 10−3

Rate (mol/L/h1) 1.5 × 10−6 1.5 × 10−6 1.5 × 10−6

Access for free at OpenStax 12.E.9 https://chem.libretexts.org/@go/page/46044


Determine the rate equation, the rate constant, and the overall order for this reaction.

12.E.3.14: Q12.3.13
Nitrosyl chloride, NOCl, decomposes to NO and Cl2.

2 NOCl(g) ⟶ 2 NO(g) + Cl (g)


2

Determine the rate equation, the rate constant, and the overall order for this reaction from the following data:

[NOCl] (M) 0.10 0.20 0.30

Rate (mol/L/h) 8.0 × 10−10 3.2 × 10−9 7.2 × 10−9

Solution
Before we can figure out the rate constant first we must first determine the basic rate equation and rate order. The basic rate
equation for this reaction, where n is the rate order of NOCl and k is the rate constant, is
n
rate = k[N OC l]

since NOCl is the reactant in the reaction.


In order to figure out the order of the reaction we must find the order of [NOCl] as it is the only reactant in the reaction. To do
this we must examine how the rate of the reaction changes as the concentration of NOCl changes.
As [NOCl] doubles in concentration from 0.10 M to 0.20 M the rate goes from 8.0 x 10-10 to 3.2 x 10-9
(3.2 x 10-9(mol/L/h))/(8.0 x 10-10(mol/L/h)) = 4
so we conclude that as [NOCl] doubles, the rate goes up by 4. Since 22= 4 we can say that the order of [NOCl] is 2 so our
updated rate law is
2
rate = k[N OC l]

Now that we have the order, we can substitute the first experimental values from the given table to find the rate constant, k
(8.0 x 10-10(mol/L/h)) = k(0.10 M)2 so
−10
8.0 × 10
−8 −1 −1
k = = 8 × 10 M sec
2
(0.10 M )

We were able to find the units of k using rate order, when the rate order is 2 units of k are M-1 x sec-1
So the rate equation is: rate = k[NOCl]2, it is second order, and k = 8 x 10-8 M-1 x sec-1
Overall rate law :
−8 2
rate = (8 × 10 ) [N OC l]

1/(M x sec)

Answer
rate = k[NOCl]2; k = 8.0 × 10−8 L/mol/s; second order

12.E.3.15: Q12.3.14
From the following data, determine the rate equation, the rate constant, and the order with respect to A for the reaction A ⟶ 2C .

[A] (M) 1.33 × 10−2 2.66 × 10−2 3.99 × 10−2

Rate (mol/L/h) 3.80 × 10−7 1.52 × 10−6 3.42 × 10−6

Solution
A. Using the experimental data, we can compare the effects of changing [A] on the rate of reaction by relating ratios of [A] to
ratios of rates

Access for free at OpenStax 12.E.10 https://chem.libretexts.org/@go/page/46044


−2
2.66 × 10
=2
−2
1.33 × 10

and
−6
1.52 × 10
=4
−7
3.8 × 10

B. From this we know that doubling the concentration of A will result in quadrupling the rate of reaction. The order of this
reaction is 2.
C. We can now write the rate equation since we know the order:
2
rate = k[A]

D. By plugging in one set of experimental data into our rate equation we can solve for the rate constant, k:
−7 −2 2
3.8 × 10 = k × (1.33 × 10 )

−7
3.8 × 10
k =
−4
1.769 × 10

−1 −1
k = .00215M s

Answer
−1 −1
k = .00215M s

2nd Order

12.E.3.16: Q12.3.15
Nitrogen(II) oxide reacts with chlorine according to the equation:

2 NO(g) + Cl (g) ⟶ 2 NOCl(g)


2

The following initial rates of reaction have been observed for certain reactant concentrations:

[NO] (mol/L1) [Cl2] (mol/L) Rate (mol/L/h)

0.50 0.50 1.14

1.00 0.50 4.56

1.00 1.00 9.12

What is the rate equation that describes the rate’s dependence on the concentrations of NO and Cl2? What is the rate constant?
What are the orders with respect to each reactant?

Solution
For the general equation,
aA + bB → cC + dD

The rate can be written as


rate = k[A]
m
[B]
n
where k is the rate constant, and m and n are the reaction orders.
For our equation
2N O(g) + C l2 (g) → 2N OC l(g)

the rate = k[N O] m n


[C l2 ]

Now, we need to find the reaction orders. Reaction orders can only be found through experimental values. We can compare two
reactions where one of the reactants has the same concentration for both trials, and solve for the reaction order.

Access for free at OpenStax 12.E.11 https://chem.libretexts.org/@go/page/46044


m n
rate1 [N O] [C l2 ]
1 1
= m n
rate2 [N O] [C l2 ]
2 2

We can use the data in the table provided. If we plug in the values for rows 1 and 2, we see that the values for the concentration
of Cl will cancel, leaving just the rates and the concentrations of NO.
m
[0.5]
1.14
= m
4.56 [1.0]

We can now solve for m, and we find that m =2. This means that the reaction order for [NO] is 2.
Now we must find the value of n. To do so, we can use the same equation but with the values from rows 2 and 3. This time, the
concentration of NO will cancel out.
n
4.56 [0.5]
= n
9.12 [1.0]

When we solve for n, we find that n = 1. This means that the reaction order for [Cl2] is 1.
We are one step closer to finishing our rate equation.
2
rate = k[N O] [C l2 ]

Finally, we can solve for the rate constant. To do this, we can use one of the trials of the experiment, and plug in the values for
the rate, and concentrations of reactants, then solve for k.
2
1.14mol/L/h = k[0.5mol/L] [0.5mol/L]

2 −2 −1
k = 9.12 L mol h

So, our final rate equation is:


2 −2 −1 2
rate = (9.12 L mol h )[N O] [C l2 ]

*A common mistake is forgetting units. Make sure to track your units throughout the process of determining your rate constant.
Be careful because the units will change relative to the reaction order.

Answer
rate = k[NO]2[Cl]2; k = 9.12 L2 mol−2 h−1; second order in NO; first order in Cl2

12.E.3.17: Q12.3.17
Hydrogen reacts with nitrogen monoxide to form dinitrogen monoxide (laughing gas) according to the equation:
H (g) + 2 NO(g) ⟶ N O(g) + H O(g)
2 2 2

Determine the rate equation, the rate constant, and the orders with respect to each reactant from the following data:

[NO] (M) 0.30 0.60 0.60

[H2] (M) 0.35 0.35 0.70

Rate (mol/L/s) 2.835 × 10−3 1.134 × 10−2 2.268 × 10−2

Solution
Determine the rate equation, the rate constant, and the orders with respect to each reactant.
The rate constant and the orders can be determined through the differential rate law. The general form of the differential
rate law is given below:
aA + bB + cC ==> products

where A, B, and C are the concentrations of the reactants, k is the rate constant, and n,m, and p refer to the order of
each reactant.

Access for free at OpenStax 12.E.12 https://chem.libretexts.org/@go/page/46044


To find the orders of each reactant, we see that when [NO] doubles but [H2] doesn't change, the rate quadruples, meaning
that [NO] is a second order reaction ([NO]2). When [H2] doubles but [NO] doesn't change, the rate doubles, meaning that
[H2] is a first order reaction. So the rate law would look something like this:
Rate = k[NO]2[H2]
We can use this rate law to determine the value of the rate constant. Plug in the data for reactant concentration and rate
from one of the trials to solve for k the rate constant. In this case, we chose to use the data from trial 1 from the second
column of the data table.
2.835x10-3 = k[0.3]2[0.35]
k = .09 M-2/s-1

12.E.3.18: Q12.3.18
For the reaction A ⟶ B + C , the following data were obtained at 30 °C:

[A] (M) 0.230 0.356 0.557

Rate (mol/L/s) 4.17 × 10−4 9.99 × 10−4 2.44 × 10−3

a. What is the order of the reaction with respect to [A], and what is the rate equation?
b. What is the rate constant?

Solution
1. The rate equation for an n order reaction is given as dr

dt
= k[A]
n
. Where [A] is the concentration in M, and dr

dt
is the rate in
M/s.
We can then use each set of data points, plug its values into the rate equation and solve for n . Note you can use any of the data
points as long as the concentration corresponds to its rate.
Rate equation 1: 4.17 × 10 −4
= k[0.230 ]
n

Rate equation 2: 9.99 × 10 −4


= k[0.356 ]
n

We divide Rate equation 1 by Rate equation 2 in order to cancel out k, the rate constant.
n
4.17×10
−4 k[0.230]
= n
−4
9.99×10 k[0.356]

n
0.417 = 0.646

Now the only unknown we have is n . Using logarithm rules one can solve for it.
ln 0.417 = n ⋅ ln 0.646

ln 0.417
=n =2
ln 0.646

The rate equation is second order with respect to A and is written as dr

dt
= k[A]
2
.

2. We can solve for k by plugging in any data point into our rate equation dr

dt
= k[A]
2
.

Using the first data points for instance [A] = 0.230


mol

L
and dr

dt
= 4.17 × 10
−4 mol

L⋅s
] we get the equation
−4 mol mol 2
4.17 × 10 = k[0.230 ]
L⋅s L

Which solves for k = 7.88 × 10 −3 L

mol⋅s

Since we know this is a second order reaction the appropriate units for k can also be written as 1

M⋅s

Answer
(a) The rate equation is second order in A and is written as rate = k[A]2. (b) k = 7.88 × 10−13 L mol−1 s−1

Access for free at OpenStax 12.E.13 https://chem.libretexts.org/@go/page/46044


12.E.3.19: Q12.3.19
For the reaction Q ⟶ W + X , the following data were obtained at 30 °C:

[Q]initial (M) 0.170 0.212 0.357

Rate (mol/L/s) 6.68 × 10−3 1.04 × 10−2 2.94 × 10−2

a. What is the order of the reaction with respect to [Q], and what is the rate equation?
b. What is the rate constant?

Solution
What is the order of the reaction with respect to [Q], and what is the rate equation?
Order reaction: 2 because when you use the ratio trial 3:2, it will look like this:
−2 x
2.94 ∗ 10 0.357
( )=( x
)
−2
1.04 ∗ 10 0.212

2.82 = 1.7x
x = 2 so the order of reaction is 2
Rate reaction equation: Rate=k[Q]2
What is the rate constant?
To find the rate constant (k) simply plug and calculate one of the trials into the rate equation
1.04 x 10-2=k[0.212]2
k=0.231 M s −1 −1

Answer
Order: 2
k=0.231 M −1
s
−1

12.E.3.20: Q12.3.20
The rate constant for the first-order decomposition at 45 °C of dinitrogen pentoxide, N2O5, dissolved in chloroform, CHCl3, is 6.2
× 10−4 min−1.

2N O ⟶ 4 NO +O
2 5 2 2

What is the rate of the reaction when [N2O5] = 0.40 M?

Solution
Step 1: The first step is to write the rate law. We know the general formula for for a first-order rate law. It is as follows:
Rate=k[A]
Step 2: We now plug in [N2O5] in for [A] in our general rate law. We also plug in our rate constant (k), which was given to us.
Now our equation looks as follows:
Rate=(6.2x10-4 min-1)[N2O5]
Step 3: We now plug in our given molarity. [N2O5]=0.4 M. Now our equation looks as follows:
Rate=(6.2x10-4 min-1)(0.4 M)
Step 4: We now solve our equation. Rate=(6.2x10-4 min-1)(0.4 M)= 2.48x10-4 M/min.
Step 5: Use significant figures and unit conversion to round 2.48x10-4 M/min to 2.5 × 10−4 (moles)L-1min-1

Answer
(a) 2.5 × 10−4 mol/L/min

Access for free at OpenStax 12.E.14 https://chem.libretexts.org/@go/page/46044


12.E.3.21: Q12.3.21
The annual production of HNO3 in 2013 was 60 million metric tons Most of that was prepared by the following sequence of
reactions, each run in a separate reaction vessel.
a. 4 NH (g) + 5 O (g) ⟶ 4 NO(g) + 6 H O(g)
3 2 2

b. 2 NO(g) + O (g) ⟶ 2 NO (g)


2 2

c. 3 NO (g) + H O(l) ⟶ 2 HNO (aq) + NO(g)


2 2 3

The first reaction is run by burning ammonia in air over a platinum catalyst. This reaction is fast. The reaction in equation (c) is
also fast. The second reaction limits the rate at which nitric acid can be prepared from ammonia. If equation (b) is second order in
NO and first order in O2, what is the rate of formation of NO2 when the oxygen concentration is 0.50 M and the nitric oxide
concentration is 0.75 M? The rate constant for the reaction is 5.8 × 10−6 L2/mol2/s.

Solution
To determine the rate law for an equation we need to look at its slow step. Since both equation a and c are fast, equation b can
be considered the slow step of the reaction. The slow step is also considered the rate determining step of the system.
Hence, The rate determining step is the second step because it's the slow step.
rate of production of N O 2 = k[A]
m n
[B]

2 1
rate = k[N O] [ O2 ]  M /s

−6 2 1
rate = (5.8 ∗ 10 )[0.75 ] [0.5 ]  M /s

−6
rate = 1.6 ∗ 10  M /s

Answer
−6
rate = 1.6 ∗ 10  M /s

12.E.3.22: Q12.3.22
The following data have been determined for the reaction:
− − − −
I + OCl ⟶ IO + Cl

1 2 3

[I

]i ni t i al (M) 0.10 0.20 0.30

[OCl

]i ni t i al (M) 0.050 0.050 0.010

Rate (mol/L/s) 3.05 × 10−4 6.20 × 10−4 1.83 × 10−4

Determine the rate equation and the rate constant for this reaction.

Solution
Using the reactants, we can form the rate law of the reaction: $$ r=k[OCl^-]^n[I^-]^m \]
From there, we need to use the data to determine the order of both [OC l −
] and [I −
. In doing so, we need to compare r to r
] 1 2

such that:
m n −4
r1 (0.10 )(0.050 ) 3.05 × 10
= = (12.E.3)
m n −4
r2 (0.20 )(0.050 ) 6.20 × 10

m
0.5 = 0.5 (12.E.4)

m =1 (12.E.5)

We can "cross out" the concentration of [OC l −


] because it has the same concentration in both of the trials used.
Now that we know m ([I −
) has a first order of 1.
]

Access for free at OpenStax 12.E.15 https://chem.libretexts.org/@go/page/46044


We cannot "cross out" [I

] to find −
[OC l ] because no two trials have the same concentration. In order to solve for n we will
plug in 1 for m.
1 n −4
r1 (0.10 )(0.050 ) 3.05 × 10
= = (12.E.6)
1 n −4
r3 (0.30 )(0.010 ) 1.83 × 10

1 n
(5 ) = 1.6666667 (12.E.7)
3

n
5 =5 (12.E.8)

n =1 (12.E.9)

Since we know that orders of both n and m are equal to one, we can not substitute them into the rate law equation along with the
respective concentrations (from either the first, second, or third reaction) and solve for the rate constant, k.
− n − m
r = k[OC l ] [I ] (12.E.10)

−4 1 1
3.05 ∗ 10 = k[0.05 ] [0.10 ] (12.E.11)

−2
L
k = 6.1 ∗ 10 (12.E.12)
mol × s

Thus the overall rate law is: $$ r = (6.1 * 10^{-2} \frac {L}{mol \times s})[OCl^-][I^-] \]
The units for K depend on the overall order of the reaction. To find the overall order we add m and n together. By doing this we
find an overall order of 2. This is why the units for K are $$ \frac {L}{mol \times s} \]

Answer
rate = k[I−][OCl−1]; k = 6.1 × 10−2 L mol −1 s−1

12.E.3.23: Q12.3.23
In the reaction

2N O + C l2 → 2N OC l

the reactants and products are gases at the temperature of the reaction. The following rate data were measured for three
experiments:

Initial p{NO} Initial p{Cl2} Initial rate

(atm) (atm) (moles of A consumed atm sec-1)

0.50 0.50 5.1 x 10-3

1.0 1.0 4.0 x 10-2

0.50 1.0 1.0 x 10-2

a. From these data, write the rate equation for this gas reaction. What order is the reaction in NO, Cl2, and overall?
b. Calculate the specific rate constant for this reaction.

Solution
a. The rate equation can be determined by designing experiments that measure the concentration(s) of one or more reactants or
products as a function of time. For the reaction A + B → products , for example, we need to determine k and the exponents m
and n in the following equation:
m n
rate = k[A] [B]

To do this, the initial concentration of B can be kept constant while varying the initial concentration of A and calculating the
initial reaction rate. This information would deduce the reaction order with respect to A. The same process can be done to find

Access for free at OpenStax 12.E.16 https://chem.libretexts.org/@go/page/46044


the reaction order with respect to B.
In this particular example,
m n
rate2 k[ A2 ] [ B2 ]
=
m n
rate3 k[ A3 ] [ B3 ]

So taking the values from the table,


−2 m n
4.0 ∗ 10 k[1.0 ] [1.0 ]
=
−2 m n
1.0 ∗ 10 k[0.5 ] [1.0 ]

and by canceling like terms, you are left with


−2 m
4.0 ∗ 10 [1.0]
=
−2 m
1.0 ∗ 10 [0.5]

Now, solve for m

4 =2
m
⟹ m =2 Because m=2, the reaction with respect to N O is 2. N O is second order.

You can repeat the same process to find n.

m n
rate3 k[ A3 ] [ B3 ]
=
m n
rate1 k[ A1 ] [ B1 ]

Taking the values from the table,


−2 m n
1.0 ∗ 10 k[0.5 ] [1.0 ]
=
−3 m n
5.1 ∗ 10 k[0.5 ] [0.5 ]

and by canceling like terms, you are left with


−2 n
1.0 ∗ 10 [1.0]
=
−3
5.1 ∗ 10 [0.5]n

Now this time, solve for n

n
2 =2 ⟹ n =1

Because n=1, the reaction with respect to C l is 1. C l is first order.


2 2

So the rate equation is


2 1
rate = k[N O] [C l2 ]

To find the overall rate order, you simply add the orders together. Second order + first order makes the overall reaction third
order.

Access for free at OpenStax 12.E.17 https://chem.libretexts.org/@go/page/46044


b. The rate constant is calculated by inserting the data from any row of the table into the experimentally determined rate law
and solving for k. For a third order reaction, the units of k are f rac1atm ∗ sec. Using Experiment 1,
2

atm
2 1 −3 2 1
rate = k[N O] [C l2 ] ⟹ 5.1 ∗ 10 = k[0.5matm ] [0.5atm ]
sec

1
k = 0.0408
2
atm ∗ sec

Answer
NO is second order.
C l2 is first order.
Overall reaction order is three.
b)
−2 −1
k = 0.0408 atm ∗ sec

12.E.4: 12.4: Integrated Rate Laws


12.E.4.1: Q12.4.1
Describe how graphical methods can be used to determine the order of a reaction and its rate constant from a series of data that
includes the concentration of A at varying times.

Solution

Access for free at OpenStax 12.E.18 https://chem.libretexts.org/@go/page/46044


To determine the order of a reaction when given the data series, one must graph the data how it is, graph it as natural log of [A],
and graph it as 1/[A]. Whichever method yields a straight line will determine the order. Respective of the methods of graphing
above, if a straight line is yielded by the first graphing method its a 0 order, if by the second method it's a 1st order, and the third
graphing method its a 2nd order. When the order of the graph is known, a series of equations, given in the above image, can be
used with the various points on the graph to determine the value of k. We can see that we need an initial value of A and a final
value of A, and both of these would be given by the data.
Zeroth order when plotting initial concentrating versus final concentration you have a negative linear slope.

[A] = [A]0 − kt

First order when plotting ln[initial concentration] versus ln[ final concentration] you have a negative linear slope.

ln[A] = ln[A]0 − kt

Second order when plotting the 1/[initial concentration] versus 1/[final concentration] you have a positive linear slope.
1 1
= + kt
[A] [A]0

12.E.4.2: Q12.4.2
Use the data provided to graphically determine the order and rate constant of the following reaction: SO 2
Cl
2
⟶ SO
2
+ Cl
2

Time (s) 0 5.00 × 103 1.00 × 104 1.50 × 104 2.50 × 104 3.00 × 104 4.00 × 104

Access for free at OpenStax 12.E.19 https://chem.libretexts.org/@go/page/46044


[SO2Cl2] (M) 0.100 0.0896 0.0802 0.0719 0.0577 0.0517 0.0415

Solution
Use the data to graphically determine the order and rate constant of the following reaction.
In order to determine the rate law for a reaction from a set of data consisting of concentration (or the values of some function of
concentration) versus time, make three graphs of the data based on the integrated rate laws of each order reaction.

[concentration] versus time (linear for a zero order reaction) ln [concentration] versus time (linear for a 1st order reaction) 1 /
[concentration] versus time (linear for a 2nd order reaction)
slope= -2.0 x 10-5
k = 2.0 x 10-5

The graph that is linear indicates the order of the reaction. Then, you can find the correct rate equation:

zero order reaction rate = k (k = - slope of line)

1st order reaction rate = k[A] (k = - slope of line)

2nd order reaction rate = k[A]2 (k = slope of line)

In this graph, ln(concentration) vs time is linear, indicating that the reaction is first order.
k=-slope of line

Access for free at OpenStax 12.E.20 https://chem.libretexts.org/@go/page/46044


Answer
Plotting a graph of ln[SO2Cl2] versus t reveals a linear trend; therefore we know this is a first-order reaction:

k = −2.20 × 105 s−1

12.E.4.3: Q12.4.3
Use the data provided in a graphical method to determine the order and rate constant of the following reaction:

2P ⟶ Q + W

Time (s) 9.0 13.0 18.0 22.0 25.0

[P] (M) 1.077 × 10−3 1.068 × 10−3 1.055 × 10−3 1.046 × 10−3 1.039 × 10−3

Solution
Add texts here. Do not delete this text first.

12.E.4.4: Q12.4.4
Pure ozone decomposes slowly to oxygen, 2 O (g) ⟶ 3 O (g)
3 2
. Use the data provided in a graphical method and determine the
order and rate constant of the reaction.

Time (h) 0 2.0 × 103 7.6 × 103 1.23 × 104 1.70 × 104 1.70 × 104

[O3] (M) 1.00 × 10−5 4.98 × 10−6 2.07 × 10−6 1.39 × 10−6 1.22 × 10−6 1.05 × 10−6

Solution
To determine the order and rate constant, you need to graph the data for zero order, first order, and second order by plotting
concentration versus time- [A] vs. time, natural logarithm (ln) of [A] vs. time, and 1/[A] vs. time respectively. The order of the

Access for free at OpenStax 12.E.21 https://chem.libretexts.org/@go/page/46044


reaction is determined by identifying which of these three graphs produces a straight line. The rate constant k is represented by
the slope of the graph. The graphs with their respective data values would be

Time (s) 9.0 13.0 18.0 22.0 25.0

[P] (M) 1.077 × 10−3 1.068 × 10−3 1.055 × 10−3 1.046 × 10−3 1.039 × 10−3

Time (s) 9.0 13.0 18.0 22.0 25.0

ln [P] (M) -6.83358 -6.84197 -6.85421 -6.86278 -6.8695

Time (s) 9.0 13.0 18.0 22.0 25.0

1/[P] (M) 928.5051 936.3296 947.8673 956.0229 962.4639

Access for free at OpenStax 12.E.22 https://chem.libretexts.org/@go/page/46044


Since each graph yields a straight line the order and rate constant of the reaction cannot be determined.
To identify how the concentrations changes a function of time, requires solving the appropriate differential equation (i.e., the
differential rate law).
The zero-order rate law predicts in a linear decay of concentration with time
The 1st-order rate law predicts in an exponential decay of concentration with time
The 2nd-order rate law predicts in an reciprocal decay of concentration with time

The plot is not linear, so the reaction is not zero order.

The plot is not linear, so the reaction is not first order.

Access for free at OpenStax 12.E.23 https://chem.libretexts.org/@go/page/46044


The plot is nicely linear, so the reaction is second order.
To a second order equation, 1/[A]  = k ∗ t + 1/[A 0]

Thus, the value of K is the slope of the graph Time vs 1

O
,
3

k = 50.3*10^6 L mol−1 h−1

Answer

The plot is nicely linear, so the reaction is second order.


k = 50.1 L mol−1 h−1

12.E.4.5: Q12.4.5
From the given data, use a graphical method to determine the order and rate constant of the following reaction:

Access for free at OpenStax 12.E.24 https://chem.libretexts.org/@go/page/46044


2X ⟶ Y + Z (12.E.13)

Time (s) 5.0 10.0 15.0 20.0 25.0 30.0 35.0 40.0

[X] (M) 0.0990 0.0497 0.0332 0.0249 0.0200 0.0166 0.0143 0.0125

Solution
In order to determine the order of the reaction we need to plot the data using three different graphs. All three graphs will have
time in seconds as the x-axis, but the y-axis is what will differ. One graph will plot concentration versus time, the second will
plot natural log of concentration versus time, and the other will plot 1/concentration versus times. Whichever graph results in a
line, we know that must be the order of the reaction. If we get a line using the first graph, it will be zero order, if it is a line for
the second graph it will be first order, and if it is a line for the third graph it will be a second order reaction. Now lets plot the
data to determine the order.

We can clearly see that the third graph, which plots 1/M versus time, is a straight line while the other two are slightly curved.
Therefore, we can determine that the rate of this reaction is second order. This also tells us that the units of the rate constant
which should be M-2s-1 for a second order reaction.
To determine the rate constant, called k, we simple need to figure out the slope of the third graph since that is the order of this
reaction. To find the slope of the line, we take two points and subtract the y values and then divide them by the difference of the
x values. This is how to do it:
Use the points (5, 10.101) and (40, 80).
Now use these to get the slop, aka the rate constant: (80-10.101)/(40-5) = 1.997 = k
So the rate constant for this second order reaction is 1.997 M-1s-1.

12.E.4.6: Q12.4.6
What is the half-life for the first-order decay of phosphorus-32? ( 32
15
P ⟶
32
16
S+e

) The rate constant for the decay is 4.85 × 10−2
day−1.

Solution
This is a first order reaction, so we can use our half life equation below:
0.693
t1/2 =
k

The rate constant is given to us in units per day. All we have to do, is to plug it into the equation.

Access for free at OpenStax 12.E.25 https://chem.libretexts.org/@go/page/46044


0.693
t1/2 =
−2
4.85 ∗ 10

= 14.3 days

A12.4.6
14.3 d

12.E.4.7: Q12.4.7
What is the half-life for the first-order decay of carbon-14? (
6
14
C ⟶
7
14
N+e

) The rate constant for the decay is 1.21 × 10−4
year−1.

Solution
To find the half life, we need to use the first-order half-life equation. All half life reactions undergo first order reactions.
The half-life equation for first order is

t1/2 = ln2/k

with k being the rate constant. The rate constant for carbon-14 was given as 1.21 × 10 −4
year
−1
.
Plug it in the equation.
−4 −1
t1/2 = ln2/(1.21 × 10 year )

and solve for t1/2


.
When you calculate it, the half life for carbon-14 is 5.73*103

Answer
The half-life for carbon-14 is calculated to be 5.73*103

12.E.4.8: Q12.4.8
What is the half-life for the decomposition of NOCl when the concentration of NOCl is 0.15 M? The rate constant for this second-
order reaction is 8.0 × 10−8 L/mol/s.

Solution
The half-life of a reaction, t1/2, is the amount of time that is required for a reactant concentration to decrease by half compared
to its initial concentration. When solving for the half-life of a reaction, we should first consider the order of reaction to
determine it's rate law. In this case, we are told that this reaction is second-order, so we know that the integrated rate law is
given as:
1 1
= kt +
[A] [A]0

Isolating for time, we find that:


1
t1/2 =
k[A]0

Now it is just a matter of substituting the information we have been given to calculate t 1/2
, where the rate constant, k , is equal
to 8.0 × 10−8 L/mol/s and initial concentration, [A] , is equal to 0.15M:
0

1 7
t1/2 = = 8.33 × 10 seconds
−8
(8.0 × 10 )(0.15)

Answer
8.33 × 107 s

Access for free at OpenStax 12.E.26 https://chem.libretexts.org/@go/page/46044


12.E.4.9: Q12.4.9
What is the half-life for the decomposition of O3 when the concentration of O3 is 2.35 × 10−6 M? The rate constant for this second-
order reaction is 50.4 L/mol/h.

Solution
Add texts here. Do not delete this text first.

Since the reaction is second order, its half-life is

1
t1/2 =
−1 −6
(50.4 M /h)[2.35 × 10 M]

So, half-life is 8443 hours.

12.E.4.10: Q12.4.10
The reaction of compound A to give compounds C and D was found to be second-order in A. The rate constant for the reaction was
determined to be 2.42 L/mol/s. If the initial concentration is 0.500 mol/L, what is the value of t1/2?

Solution
As mentioned in the question the reaction of compound A will result in the formation of compounds C and D. This reaction was
found to be second-order in A. Therefore, we should use the second order equation for half-life which relates the rate constant
and initial concentrations to the half-life:
1
t 1 =
2 k[A]0

Since we were given k (rate constant) and Initial concentration of A, we have everything needed to calculate the half life of A.
L

mol
k = 0.5
s

mol
[A]0 = 2.42
L

When we plug in the given information notice that the units cancel out to seconds.
1
t 1 = = 0.826s

2 2.42Lmol mol
[0.500 ]
s L

Answer
0.826 s

12.E.4.11: Q12.4.11
The half-life of a reaction of compound A to give compounds D and E is 8.50 minutes when the initial concentration of A is 0.150
mol/L. How long will it take for the concentration to drop to 0.0300 mol/L if the reaction is (a) first order with respect to A or (b)
second order with respect to A?

Solution
Organize the given variables:
(half-life of A) t = 8.50min
1/2

(initial concentration of A) [A] = 0.150mol/L


0

(target concentration of A) [A] = 0.0300mol/L

Access for free at OpenStax 12.E.27 https://chem.libretexts.org/@go/page/46044


Find the the rate constant k, using the half-life formulas for each respective order. After finding k, use the integrated rate law
respective to each order and the initial and target concentrations of A to find the time it took for the concentration to drop.
(a) first order with respect to A
ln(2)
(half-life) t 1/2 =
k
=
0.693

(rearranged for k) k = 0.693

t1 / 2

(plug in t1/2 = 8.50 min) k = 0.693

8.50min
= 0.0815mi n
−1

(integrated rate law) ln[A] = −kt + ln[A] 0

[A]
(rearranged for t) ln( [A]
) = −kt
0

[A]
−ln( ) = kt
[A]
0

[A]
−1
ln( ) = kt
[A]
0

[A]0
ln( ) = kt
[A]

[A]
0
ln( )
[A]
t =
k
0 .1 5 0 mol/ L
ln( )
ln(5.00)
(plug in variables) t =
0 .0 3 0 0 mol/ L

= = 19.7min
0.0815min− 1 0.0815min− 1

(b) second order with respect to A


(half-life) t 1/2
=
k[A]
1

(rearranged for k) k = 1

t1 / 2[A]
0

(plug in variables) k = (8.50min)(0.150mol/L)


1
=
1

1.275min⋅mol/L
= 0.784L/mol ⋅ min

(integrated rate law) [A]


1
= kt +
[A]
1

(rearranged for t) 1

[A]

[A]0
1
= kt

1 1 1
t = ( − )
k [A] [A]0

(plug in variables) t = 0.784L/mol⋅min


1
(
1

0.0300mol/L

1

0.150mol/L
) =
1

0.784L/mol⋅min
(
80

3
L/mol) = 34.0min

Answer
a) 19.7 min
b) 34.0 min

12.E.4.12: Q12.4.12
Some bacteria are resistant to the antibiotic penicillin because they produce penicillinase, an enzyme with a molecular weight of 3
× 104 g/mol that converts penicillin into inactive molecules. Although the kinetics of enzyme-catalyzed reactions can be complex,
at low concentrations this reaction can be described by a rate equation that is first order in the catalyst (penicillinase) and that also
involves the concentration of penicillin. From the following data: 1.0 L of a solution containing 0.15 µg (0.15 × 10−6 g) of
penicillinase, determine the order of the reaction with respect to penicillin and the value of the rate constant.

[Penicillin] (M) Rate (mol/L/min)

2.0 × 10−6 1.0 × 10−10

3.0 × 10−6 1.5 × 10−10

4.0 × 10−6 2.0 × 10−10

Solution
The first step is to solve for the order or the reaction. This can be done by setting up two expressions which equate the rate to
the rate constant times the molar concentration of penicillin raised to the power of it's order. Once we have both expressions set

Access for free at OpenStax 12.E.28 https://chem.libretexts.org/@go/page/46044


up, we can divide them to cancel out k (rate constant) and use a basic logarithm to solve for the exponent, which is the order. It
will look like this.
rate(mol/L/min)=k[M]x
(1.0 x 10-10)=k[2.0 x 10-6]x
(1.5 x 10-10)=k[3.0 x 10-6]x

Dividing the two equations results in the expression:


(2/3)=(2/3)x
*A single ratio equation can also be set up to solve for the reaction order:
*
x
rate1 k[P enicillin]
1
=
x
rate2 k[P enicillin]
2

*We then solve for x in a similar fashion.


*
−10 −6 x
1.0x10 [2.0x10 ]
=
−10 −6 x
1.5x10 [3.0x10 ]

We can now use the natural logarithm to solve for x, or simply and intuitively see that in order for
the equation to work, x must be equal to one. Thus, the reaction is of the first order.
Now that we have the order of the reaction, we can proceed to solve for the value of the rate constant. Substituting x=1 into our
first equation yields the expression:
(1 x 10-10)=k[2.0 x 10-6]1
k=(1 x 10-10)/(2 x 10-6)
k= (5 x 10-5) min-1
We have a unit of min-1 because we divided (mol/L/min) by molarity, which is in (mol/L), yielding a unit of min-1.
We were given two important pieces of information to finish the problem. It is stated that the enzyme has a molecular weight of
3 × 104 g/mol, and that we have a one liter solution that contains (0.15 x 10-6 g) of penicillinase. Dividing the amount of grams
by the molecular weight yields 5 x 10-12 moles.
(0.15 x 10-6) g / (3 x 104) g/mol = (5 x 10-12) mol
Now that we have the amount of moles, we can divide our rate constant by this value.
(5 x 10-5) min-1 / (5 x 10-12) mol = (1 x 107) mol-1 min -1

Answer

The reaction is first order with k = 1.0 × 107 mol−1 min−1

12.E.4.13: Q12.4.13
Both technetium-99 and thallium-201 are used to image heart muscle in patients with suspected heart problems. The half-lives are 6
h and 73 h, respectively. What percent of the radioactivity would remain for each of the isotopes after 2 days (48 h)?

Solution
This problem is asking us for the percentage of radioactivity remaining after a certain time for both isotopes after 48 hours. We
must identify an equation that will help us solve this and we can determine that we can determine this information using the
first order equation.

Access for free at OpenStax 12.E.29 https://chem.libretexts.org/@go/page/46044


This equation Ln(N/No)= -kt tells that the Natural log of the fraction remaining is equal to the rate constant times time. To
determine the rate constant, we can also compute .693 over the half-life given in the information.
For Technetium-99 we can determine the rate constant by plugging into the second equation: .693/6 hrs= .1155 h-1
Now that we have the rate constant we can plug in : Ln(N/No)=-(.1155h-1)(48h) so Ln(N/No)=-5.544 and if we take the inverse
of the natural log, we get (N/No)=3.9x10 -3 and if we multiply this by 100, we get .39% remaining.
We can do this same process for Thallium-201 and plugin: .693/73 hrs= .009493151 h-1 and when we plug this into the first
order equation we get:
Ln(N/No)=-(.009493h-1)(48h) so Ln(N/No)=-.45567248 and when we take the inverse of the natural log, we get (N/No)=.6340
and when multiplied by 100, we get 63.40% remaining which makes sense since its half-life is 73 hours and only 48 hours have
passed, half of the amount has yet to be consumed.

Answer
Technetium-99: 0.39%
Thallium-201: 63.40%

12.E.4.14: Q12.4.14
There are two molecules with the formula C3H6. Propene, CH CH=CH , is the monomer of the polymer polypropylene, which is
3 2

used for indoor-outdoor carpets. Cyclopropane is used as an anesthetic:

When heated to 499 °C, cyclopropane rearranges (isomerizes) and forms propene with a rate constant of 5.95 × 10−4 s−1. What is
the half-life of this reaction? What fraction of the cyclopropane remains after 0.75 h at 499 °C?

Solution
Use the equation
ln2
t1 / 2 =
k

since this is a first-order reaction. You can tell that this is a first order reaction due to the units of measurement of the rate
constant, which is s-1. Different orders of reactions lead to different rate constants, and a rate constant of s-1 will always be first
order.
Plug into the equation, and you get half life = 1164.95 seconds. To convert this to hours, we would divide this number by 3600
seconds/hour, to get 0.324 hours.
Use the integrated first order rate law
[A]
ln = −kt
[A]0

. In this equation, [A]0 represents the initial amount of compound present at time 0, while [A] represents the amount of
compound that is left after the reaction has occurred. Therefore, the fraction
[A]

[A]0

is equal to the fraction of cyclopropane that remains after a certain amount of time, in this case, 0.75 hours.
Substitute x for the fraction of
[A]

[A]0

Access for free at OpenStax 12.E.30 https://chem.libretexts.org/@go/page/46044


into the integrated rate law:
[A]
ln = −kt
[A]0

−4
ln(x) = −5.95x 10 (0.75)

(−0.000595)(0.75)
x =e

= 0.20058 = 20%.
So, the half life is 0.324 hours, and 20% of the cyclopropane will remain as 80% will have formed propene.

Answer
0.324 hours. ; 20% remains

12.E.4.15: Q12.4.16
Fluorine-18 is a radioactive isotope that decays by positron emission to form oxygen-18 with a half-life of 109.7 min. (A positron
is a particle with the mass of an electron and a single unit of positive charge; the nuclear equation is F ⟶ O + e .)
18
9
18
8
0

1
+

Physicians use 18F to study the brain by injecting a quantity of fluoro-substituted glucose into the blood of a patient. The glucose
accumulates in the regions where the brain is active and needs nourishment.
a. What is the rate constant for the decomposition of fluorine-18?
b. If a sample of glucose containing radioactive fluorine-18 is injected into the blood, what percent of the radioactivity will remain
after 5.59 h?
c. How long does it takFe for 99.99% of the 18F to decay?

Solution
a) The nuclear decay of an isotope of an element is represented by the first order equation:
ln(N/N0) = −kt
Where t is time, N0 is the initial amount of the substance, N is the amount of the substance after time t, and k is the rate
constant. We can rearrange the equation and isolate k so that we could solve for the rate constant:
k = [-ln(N/N0)] / t
We are given that fluorine-18 has a half-life of 109.7 minutes. Since we have the half-life, we can choose an arbitrary value for
N0 and use half of that value for N. In this case, we choose 100 for N0 and 50 for N. Now we can plug in those values into the
equation above and solve for k.
k = [-ln(50/100)] / 109.7
k = 0.6931 / 109.7 = 0.006319 min-1
The rate constant for this reaction is 0.006319 min-1.
b) For this problem, we are able to use the same equation from part a:
ln(N/N0) = −kt
However, this time we are given the amount of time elapsed instead of the half-life, and we are asked to determine the percent
of fluorine-18 radioactivity remaining after that time. In this problem, we must plug in values for N0, k (determined from part
a), and t.
But first, since we are given the elapsed time in hours, we must convert it into minutes:
5.59 hours x (60 minutes / 1 hours) = 335.4 minutes
This gives us the value for t. We also have values for k (0.006319 min-1) and N0 (again an arbitrary number.) Now we can plug
values into the original equation, giving us:
ln(N/100) = −(0.006319)(335.4)
We solve this equation by taking the exponential of both sides:

Access for free at OpenStax 12.E.31 https://chem.libretexts.org/@go/page/46044


eln(N/100) = e−(0.006319)(335.4)
where eln equals 1 and now we can just solve for N:
N/100 = e−(0.006319)(335.4)
N = [e−(0.006319)(335.4)] x 100 = 12.0
Since 100 was used as the initial amount and 12.0 was determined as the remaining amount, 12.0 can be used as the percentage
of remaining amount of radioactivity of fluorine-18. Thus the percent of fluorine-18 radioactivity remaining after 5.59
hours is 12.0%.
c) This part of the question is much like the previous two parts, but this time we are given the initial amount of radioactivity, the
final amount of radioactivity and we are asked do determine how long it took for that amount of radioactivity to decay. We are
able to use the same equation:
ln(N/N0) = −kt
However, now we are given N and N0 and we have already determined k from before. We are told that 99.99% of the
radioactivity has decayed, so we can use 100 and 0.01 for N0 and N respectively. We plug these values in to the equation, solve
for t, and get
ln(0.01/1000) = −0.006319t
-9.21 = −0.006319t
t = 1458 minutes
Therefore, its takes 1458 minutes for 99.99% of the radioactivity to decay.

Answer
a) 0.006319 min-1
b) 12.0%
c) 1458 minutes

12.E.4.16: Q12.4.17
Suppose that the half-life of steroids taken by an athlete is 42 days. Assuming that the steroids biodegrade by a first-order process,
1
how long would it take for of the initial dose to remain in the athlete’s body?
64

Solution
252 days
for first order reaction: t1/2 = 0.693 / k
k = 0.693 / 42
k = 0.0165
for first order reaction: [A] = [A]0 e-kt
1/64 initial means that: [A] = 1/64 [A]0
therefore: 1/64 [A]0 = [A]0 e-0.0165t
t = 252 days

12.E.4.17: Q12.4.18
Recently, the skeleton of King Richard III was found under a parking lot in England. If tissue samples from the skeleton contain
about 93.79% of the carbon-14 expected in living tissue, what year did King Richard III die? The half-life for carbon-14 is 5730
years.

Solution

Access for free at OpenStax 12.E.32 https://chem.libretexts.org/@go/page/46044


In order to find out what year King Richard III died, set [A]/[A0] (the percent of carbon-14 still contained) equal to 0.5time(t)/half
life (t1/2)
or use the equation N(t) = N0e-rt.
Using the first equation:
A/A0 = 0.5 t/t1 / 2
plug in the given numbers .9379 = 0.5 t/5730
and solve for t.
ln.9379 = (t/5730)(ln0.5)(using the rule of logs)
−.0641 = (t/5730)(−.693)
−367.36 = −.693t
t = 530.1years

Using N (t) = N 0e
−rt
this problem is solved by the following:
−5730r
1/2 = e

r = 0.000121

Now that we know what r is, we can use this value in our original formula and solve for t, the amount of years that have passed.
This time, we use 93.78, the percent of the carbon-14 remaining as N(t) and 100 as the original, N0.
−0.000121t
93.78 = 100e

t = 530.7 years
Another way of doing this is by using these two equations:
0.693 nt
λ= and = -λt
t1/2 n0

nt = concentration at time t (93.79)


n0 = initial concentration (100)
First solve for lambda or the decay constant by plugging in the half life.
Then plug in lambda and the other numbers into the second equation, and solve for t- which should equal to 530.1 years as well.
If we want to find out what year King Richard III died, we take the current year, 2017, and subtract 530 years. Doing this, we
find that King Richard III died in the year 1487.

Answer
King Richard III died in the year 1487

12.E.4.18: Q12.4.19
Nitroglycerine is an extremely sensitive explosive. In a series of carefully controlled experiments, samples of the explosive were
heated to 160 °C and their first-order decomposition studied. Determine the average rate constants for each experiment using the
following data:

Initial
[C3H5N3O9] 4.88 3.52 2.29 1.81 5.33 4.05 2.95 1.72
(M)

t (s) 300 300 300 300 180 180 180 180

%
52.0 52.9 53.2 53.9 34.6 35.9 36.0 35.4
Decomposed

Solution
First we need to understand what the question is asking for: the average rate constant. The average rate constant is the variable
"k" when discussing kinetics and it can be defined as the proportionality constant in the equation that expresses the relationship
between the rate of a chemical reaction and the concentrations of the reacting substances. Knowing that we need to find K in
this first order reaction, we can look to formulas that include "k," initial and final concentrations [A] and[A] , and half life time
o t

Access for free at OpenStax 12.E.33 https://chem.libretexts.org/@go/page/46044


"t." Since this is a first order reaction, we can look to the first order equations, and doing that we find one that includes the
variables given in the question:

ln[A]t = −kt + ln[A]o

For the first reaction, we have an initial concentration of 4.88 M, and a percentage decomposed. To find the final concentration,
we must multiply the initial concentration by the percentage decomposed to know how much decomposed, and subtract that
from the original to find out how much is left: 4.88M x 0.52= 2.54 M and 4.88M-2.54M=2.34M
Now, we have the variables we need, and we plug it into the equation above:
ln[A]t = −kt + ln[A]o

ln[2.34M ] = −k(300s) + ln[4.88M ]

−(ln[2.34M]−ln[4.88M])
k= 300

−3
k = 2.45x10

Since it asks for the rate constant of each experiment, we now must do the same procedure for each data set to find the rate
constant:
Second experiment
ln[A]t = −kt + ln[A]o

ln[1.66M ] = −k(300s) + ln[3.52M ]

−(ln[1.66M]−ln[3.52M])
k= 300

−3
k = 2.51x10

Third experiment
ln[A]t = −kt + ln[A]o

ln[1.07M ] = −k(300s) + ln[2.29M ]

−(ln[1.07M]−ln[2.29M])
k= 300

−3
k = 2.54x10

Fourth experiment
ln[A]t = −kt + ln[A]o

ln[0.834M ] = −k(300s) + ln[1.81M ]

−(ln[0.834M]−ln[1.81M])
k= 300

−3
k = 2.58x10

Fifth Experiment
ln[A]t = −kt + ln[A]o

ln[3.49M ] = −k(180s) + ln[5.33M ]

−(ln[3.49M]−ln[5.33M])
k= 180

−3
k = 2.35x10

Sixth Experiment
ln[A]t = −kt + ln[A]o

ln[2.60M ] = −k(180s) + ln[4.05M ]

−(ln[2.60M]−ln[4.05M])
k= 180

−3
k = 2.46x10

Access for free at OpenStax 12.E.34 https://chem.libretexts.org/@go/page/46044


Seventh Experiment
ln[A]t = −kt + ln[A]o

ln[1.89M ] = −k(180s) + ln[2.95M ]

−(ln[1.89M]−ln[2.95M])
k= 180

−3
k = 2.47x10

Eighth experiment
ln[A]t = −kt + ln[A]o

ln[1.11M ] = −k(180s) + ln[1.72M ]

−(ln[1.11M]−ln[1.72M])
k= 180

−3
k = 2.43x10

Answer
[A]0 (M) k × 103 (s−1)

4.88 2.45

3.52 2.51

2.29 2.54

1.81 2.58

5.33 2.35

4.05 2.44

2.95 2.47

1.72 2.43

12.E.4.19: Q12.4.20
For the past 10 years, the unsaturated hydrocarbon 1,3-butadiene (CH =CH– CH=CH ) has ranked 38th among the top 50
2 2

industrial chemicals. It is used primarily for the manufacture of synthetic rubber. An isomer exists also as cyclobutene:

The isomerization of cyclobutene to butadiene is first-order and the rate constant has been measured as 2.0 × 10−4 s−1 at 150 °C in a
0.53-L flask. Determine the partial pressure of cyclobutene and its concentration after 30.0 minutes if an isomerization reaction is
carried out at 150 °C with an initial pressure of 55 torr.

Solution
Since this is a first order reaction, the integrated rate law is: [A ] = [A ]e t 0
−kt

Partial Pressure: Use the integrated rate law to find the partial pressure at 30 minutes:
Use A = 55 torr, t = 30 min, and k = 2.0 ∗ 10
0
−4 −1
s to solve the integrated rate law equation:
1 6 0 sec
−4
−(2.0x10 )(30min⋅ )
sec
[ A30 ] = (55torr) ∗ e 1 min

Solve this equation to get:


−0.36
[ A30 ] = (55torr) ∗ e

A30 ] = 38.37 torr.

Access for free at OpenStax 12.E.35 https://chem.libretexts.org/@go/page/46044


Initial Concentration: Find the initial concentration using the ideal gas law.
The ideal gas law is given by P V = nRT → n =
PV

RT
. Use this form of the gas law to solve for the initial concentration n.

Use V = 0.53L, R = 0.08206 L∗atm

mol∗L
, T = 423.15 K, and P = 1atm

760
= 0.07237 atm .
Solve the ideal gas equation using these values:
(55torr)(0.53L)
n = L∗ at m
= 0.00110 moles cyclobutene.
(0.08206 )(423.15K)
mol∗ K

Now find the initial concentration of cyclobutene A using the equation [A


0 0] =
n

V
:
n 0.00110moles
A0 = = = 0.00208M
V 0.53L

Concentration at 30 minutes: Find the concentration of cyclobutene at 30 minutes by using the integrated rate law given
above, using time t = 30 minutes, or 1800 seconds.
−0.36
[ A30 ] = (0.00208M )e = 0.00145M

So at 30 minutes, the cyclobutene concentration is 0.00145 M, and the partial pressure is 38.37 torr.

Answer
Partial Pressure: 38.37 torr.
Concentration: 0.00145 M

12.E.5: 12.5: Collision Theory


12.E.5.1: Q12.5.1
Chemical reactions occur when reactants collide. What are two factors that may prevent a collision from producing a chemical
reaction?

Solution
The two factors that may prevent a collision from producing a chemical reaction are:
1. Kinetic energy of the molecule
In order for chemical reactions to occur, molecules require enough velocity to overcome the minimum activation energy needed
to break the old bonds and form new bonds with other molecules. At higher temperatures, the molecules possess the minimum
amount of kinetic energy needed which ensures the collisions will be energetic enough to lead to a reaction.
2. The orientation of molecules during the collision
Two molecules have to collide in the right orientation in order for the reaction to occur. Molecules have to orient properly for
another molecule to collide at the right activation state.

12.E.5.2: Q12.5.2
When every collision between reactants leads to a reaction, what determines the rate at which the reaction occurs?

Solution
There has to be contact between reactants for a reaction to occur. The more the reactants collide, the more often reactions can
occur. Factors that determine reaction rates include concentration of reactants, temperature, physical states of reactants, surface
area, and the use of a catalyst. The reaction rate usually increases as the concentration of a reactant increases. Increasing the
temperature increases the average kinetic energy of molecules, causing them to collide more frequently, which increases the
reaction rate. When two reactants are in the same fluid phase, their particles collide more frequently, which increases the
reaction rate. If the surface area of a reactant is increased, more particles are exposed to the other reactant therefore more
collisions occur and the rate of reaction increases. A catalyst participates in a chemical reaction and increases the reaction rate
without changing itself.

Access for free at OpenStax 12.E.36 https://chem.libretexts.org/@go/page/46044


12.E.5.3: Q12.5.3
What is the activation energy of a reaction, and how is this energy related to the activated complex of the reaction?

Solution

Activation energy is the energy barrier that must be overcome in order for a reaction to occur. To get the molecules into a state
that allows them to break and form bonds, the molecules must be contorted (deformed, or bent) into an unstable state called the
transition state. The transition state is a high-energy state, and some amount of energy – the activation energy – must be added
in order for the molecule reach it. Because the transition state is unstable, reactant molecules don’t stay there long, but quickly
proceed to the next step of the chemical reaction.The activated complex is the highest energy of the transition state of the
reaction.

12.E.5.4: Q12.5.5
Describe how graphical methods can be used to determine the activation energy of a reaction from a series of data that includes the
rate of reaction at varying temperatures.

Solution
This method is based on the Arrhenius equation which can be used to show the effect of a change of temperature on the rate
constant, and therefore on the rate of reaction.
The rate constant is different from reaction rat in that the reaction rate is the measure of how fast or slow a chemical
reaction takes place while a rate constant is a constant that shows the relationship between the reaction rate and the
concentrations of the reactants or products.
For example, for the reaction A + B → C , the rate law would be:
a b
rate = k[A] [B]

k = rate constant
[A] = concentration of reactant A
a = order of reaction with respect to A
[B] = concentration of reactant B
b = order of reaction with respect to B
However, the rate constant remains constant only if you are changing the concentration of the reactants. If you change the
temperature or the catalyst of the reaction, the rate constant will change and this is demonstrated by the Arrhenius
equation:
− Ea

k = Ae RT

k1 −Ea 1 1
ln ( ) =( )( − )
k2 R T1 T2

k = rate constant

Access for free at OpenStax 12.E.37 https://chem.libretexts.org/@go/page/46044


A = frequency factor
Ea = activation energy
e = exponential function, e x

R = gas constant
T = temperature (K)

In other words, the activation energy of a reaction, E , from a series of data that includes the rate of reaction, k, at varying
a

temperatures can be determined by graphing it on a plot of ln k versus . You can then use the slope of the graph you have
1

T
−Ea
plotted to solve for E by setting the slope equal to
a
R
.

12.E.5.5: Q12.5.6
How does an increase in temperature affect rate of reaction? Explain this effect in terms of the collision theory of the reaction rate.

12.E.5.6: S12.5.6
Collision theory states that the rates of chemical reactions depend on the fraction of molecules with the correct orientation, fraction
of collisions with required energy, and the collision frequency. Because the fraction of collisions with required energy is a function
of temperature, as temperature increases, the fraction of collisions with required energy also increases. The kinetic energy of
reactants also increases with temperature which means molecules will collide more often increasing collisions frequency. With
increased fraction of collisions with required energy and collisions frequency, the rate of chemical reaction increases. We see
mathematically, from the Arrhenius equation, that temperature and the rate constant are related.
Ea

k = Ae RT
(12.E.14)

where k is the rate constant, A is a specific constant, R is 8.3145 J/K, Ea is the reaction-specific activation energy in J, and T is
temperature in K. We see from the equation that k is very sensitive to changes in the temperature.

12.E.5.7: Q12.5.7
The rate of a certain reaction doubles for every 10 °C rise in temperature.
a. How much faster does the reaction proceed at 45 °C than at 25 °C?
b. How much faster does the reaction proceed at 95 °C than at 25 °C?

Solution
By finding the difference in temperature, 45 °C - 25 °C, we get 20 °C. Since the rate of the reaction doubles every 10 °C
increase in temperature and the rate of the reaction experienced a 20 °C increase in temperature, we see that the reaction rate
doubled twice (22 = 4). As a result, the reaction proceeds four times faster.
Following the same process as in part a, we get the difference in temperature to be 70 °C. Since the rate of the reaction doubles
every 10 °C increase in temperature and the system experienced a 70 °C change, we see that the reaction doubled seven times
(27 = 128). We can see the reaction proceeds 128 times faster.
(a) 4-times faster (b) 128-times faster

12.E.5.8: Q12.5.8
In an experiment, a sample of NaClO3 was 90% decomposed in 48 min. Approximately how long would this decomposition have
taken if the sample had been heated 20 °C higher?

12.E.5.9: S12.5.8
First off, it is important to recognize that this decomposition reaction is a first-order reaction, which can be written as follows:
2N aC lO3 → 2N aC l + 3 O2

Understanding this, it is important to be able to then be able to recognize which equation would be most useful given the initial
conditions presented by the question. Since we are dealing with time, percentage of material left, and temperature, the only viable

Access for free at OpenStax 12.E.38 https://chem.libretexts.org/@go/page/46044


k2
equation that could relate all of this would be the Arrhenius Equation, which is written as follows: ln( k1
) =
Ea

R
(
1

t1

1

t2
)

However, this problem does not give us enough information such as what the activation energy is or the initial temperature in order
to mathematically solve this problem. Additionally, the problem tells us to approximate how long the decomposition would take,
which means we are asked to answer this question conceptually based on our knowledge of thermodynamics and reaction rates. As
a general rule of thumb, we know that for every 10˚C rise in temperature the rate of reaction doubles. Since the question tells us
that there is a 20˚C rise in temperature we can deduce that the reaction rate doubles twice, as per the general rule mentioned before.
This means the overall reaction rate for this decomposition would quadruple, or would be 4 times faster than the reaction rate
at the initial temperature.
We can gut check this answer by recalling how an increase in the average kinetic energy (temperature) decreases the time it takes
for the reaction to take place and increase the reaction rate. Thus, if we increase the temperature we should have a faster reaction
rate.

12.E.5.10: Q12.5.9
The rate constant at 325 °C for the decomposition reaction C H ⟶ 2 C 4 8 2
H
4
is 6.1 × 10−8 s−1, and the activation energy is 261 kJ
per mole of C4H8. Determine the frequency factor for the reaction.

Solution

12.E.5.11: S12.5.9
Using the Arrhenius equation allows me to find the frequency factor, A.
k=Ae-Ea/RT
k, Ea, R, and T are all known values. k, Ea, and T are given in the problem as 6.1x10-8, 261 kJ, and 598 K, respectively.
So, plugging them into the equation gives:
6.1x10-8 s-1=Ae(-261000 J)/(8.3145 J/mol)(598 K)
Take e(-261000 J)/(8.3145 J/mol)(598) and get 1.59 x 10-23. Divide k, 6.1 x 10-8, by 1.59 x 10-23 and get A=3.9 x 1015s-1

12.E.5.12: A12.5.9
15 −1
3.9 × 10 s

12.E.5.13: Q12.5.10
The rate constant for the decomposition of acetaldehyde (CH3CHO), to methane (CH4), and carbon monoxide (CO), in the gas
phase is 1.1 × 10−2 L/mol/s at 703 K and 4.95 L/mol/s at 865 K. Determine the activation energy for this decomposition.

12.E.5.14: S12.5.10
The equation for relating the rate constant and activation energy of a reaction is the Arrhenius equation:
Ea

k = Ae RT
(12.E.15)

When given two rate constants at two different temperatures but for the same reaction, the Arrhenius equation can be rewritten as:
k2 Ea 1 1
ln( ) = ( − ) (12.E.16)
k1 R T1 T2

In this problem, all the variables are given except for the Ea (activation energy).
k1 = 1.1 × 10−2 L/mol/s
T1 = 703 K
k2 = 4.95 L/mol/s
T2 = 865 K
R = 8.314 J/(mol K) (Ideal Gas Constant)
Now plug in all these values into the equation, and solve for Ea.

Access for free at OpenStax 12.E.39 https://chem.libretexts.org/@go/page/46044


L
4.95 Ea 1 1
mol×s
ln( ) = ( − ) (12.E.17)
−2 L −3 kJ
1.1 × 10 8.314 × 10 703 865
mol×s mol×K

Ea = 190 kJ (2 sig figs)

12.E.5.15: Q12.5.11
An elevated level of the enzyme alkaline phosphatase (ALP) in the serum is an indication of possible liver or bone disorder. The
level of serum ALP is so low that it is very difficult to measure directly. However, ALP catalyzes a number of reactions, and its
relative concentration can be determined by measuring the rate of one of these reactions under controlled conditions. One such
reaction is the conversion of p-nitrophenyl phosphate (PNPP) to p-nitrophenoxide ion (PNP) and phosphate ion. Control of
temperature during the test is very important; the rate of the reaction increases 1.47 times if the temperature changes from 30 °C to
37 °C. What is the activation energy for the ALP–catalyzed conversion of PNPP to PNP and phosphate?

Solution
43.0 kJ/mol

12.E.5.16: Q12.5.12
In terms of collision theory, to which of the following is the rate of a chemical reaction proportional?
a. the change in free energy per second
b. the change in temperature per second
c. the number of collisions per second
d. the number of product molecules

12.E.5.17: Q12.5.13
Hydrogen iodide, HI, decomposes in the gas phase to produce hydrogen, H2, and iodine, I2. The value of the rate constant, k, for the
reaction was measured at several different temperatures and the data are shown here:

Temperature (K) k (M−1 s−1)

555 6.23 × 10−7

575 2.42 × 10−6

645 1.44 × 10−4

700 2.01 × 10−3

What is the value of the activation energy (in kJ/mol) for this reaction?

Solution
177 kJ/mol

12.E.5.18: Q12.5.14
The element Co exists in two oxidation states, Co(II) and Co(III), and the ions form many complexes. The rate at which one of the
complexes of Co(III) was reduced by Fe(II) in water was measured. Determine the activation energy of the reaction from the
following data:

T (K) k (s−1)

293 0.054

298 0.100

12.E.5.19: Q12.5.15
The hydrolysis of the sugar sucrose to the sugars glucose and fructose,

C H O +H O ⟶ C H O +C H O
12 22 11 2 6 12 6 6 12 6

Access for free at OpenStax 12.E.40 https://chem.libretexts.org/@go/page/46044


follows a first-order rate equation for the disappearance of sucrose: Rate = k[C12H22O11] (The products of the reaction, glucose and
fructose, have the same molecular formulas but differ in the arrangement of the atoms in their molecules.)
a. In neutral solution, k = 2.1 × 10−11 s−1 at 27 °C and 8.5 × 10−11 s−1 at 37 °C. Determine the activation energy, the frequency
factor, and the rate constant for this equation at 47 °C (assuming the kinetics remain consistent with the Arrhenius equation at
this temperature).
b. When a solution of sucrose with an initial concentration of 0.150 M reaches equilibrium, the concentration of sucrose is 1.65 ×
10−7 M. How long will it take the solution to reach equilibrium at 27 °C in the absence of a catalyst? Because the concentration
of sucrose at equilibrium is so low, assume that the reaction is irreversible.
c. Why does assuming that the reaction is irreversible simplify the calculation in part (b)?

Solution
Ea = 108 kJ
A = 2.0 × 108 s−1
k = 3.2 × 10−10 s−1
(b) 1.81 × 108 h or 7.6 × 106 day. (c) Assuming that the reaction is irreversible simplifies the calculation because we do not
have to account for any reactant that, having been converted to product, returns to the original state.

12.E.5.20: Q12.5.16
Use the PhET Reactions & Rates interactive simulation to simulate a system. On the “Single collision” tab of the simulation applet,
enable the “Energy view” by clicking the “+” icon. Select the first A + BC ⟶ AB + C reaction (A is yellow, B is purple, and C
is navy blue). Using the “straight shot” default option, try launching the A atom with varying amounts of energy. What changes
when the Total Energy line at launch is below the transition state of the Potential Energy line? Why? What happens when it is
above the transition state? Why?

12.E.5.21: Q12.5.17
Use the PhET Reactions & Rates interactive simulation to simulate a system. On the “Single collision” tab of the simulation applet,
enable the “Energy view” by clicking the “+” icon. Select the first A + BC ⟶ AB + C reaction (A is yellow, B is purple, and C
is navy blue). Using the “angled shot” option, try launching the A atom with varying angles, but with more Total energy than the
transition state. What happens when the A atom hits the BC molecule from different directions? Why?

Solution
The A atom has enough energy to react with BC; however, the different angles at which it bounces off of BC without reacting
indicate that the orientation of the molecule is an important part of the reaction kinetics and determines whether a reaction will
occur.

12.E.6: 12.6: Reaction Mechanisms


12.E.6.1: Q12.6.1
Why are elementary reactions involving three or more reactants very uncommon?

12.E.6.2: Q12.6.2
In general, can we predict the effect of doubling the concentration of A on the rate of the overall reaction A + B ⟶ C ? Can we
predict the effect if the reaction is known to be an elementary reaction?

Solution
Add texts here. Do not delete this text first.

No. In general, for the overall reaction, we cannot predict the effect of changing the concentration without knowing the rate
equation. Yes. If the reaction is an elementary reaction, then doubling the concentration of A doubles the rate.

Access for free at OpenStax 12.E.41 https://chem.libretexts.org/@go/page/46044


12.E.6.3: Q12.6.3
Phosgene, COCl2, one of the poison gases used during World War I, is formed from chlorine and carbon monoxide. The
mechanism is thought to proceed by:

step 1: Cl + CO → COCl

step 2: COCl + Cl2→ COCl2 + Cl

a. Write the overall reaction equation.


b. Identify any reaction intermediates.
c. Identify any intermediates.

12.E.6.4: Q12.6.4
Define these terms:
a. unimolecular reaction
b. bimolecular reaction
c. elementary reaction
d. overall reaction

12.E.6.5: Q12.6.5
What is the rate equation for the elementary termolecular reaction A + 2B ⟶ products ? For 3A ⟶ products ?

Solution
Add texts here. Do not delete this text first.

We are given that both of these reactions are elementary termolecular. The molecularity of a reaction refers to the number of
reactant particles that react together with the proper and energy and orientation. Termolecular reactions have three atoms to collide
simultaneously. As it is termolecular, and there are no additional reactants aside from the three given in each reaction, there are no
intermediate reactions. The rate law for elementary reactions is determined by the stoichiometry of the reaction without needed
experimental data.
The basic rate form for the elementary step is what follows:
rate = k ⋅ reactant 1 ⋅ reactant 2
i ii
⋅. . . Where i and ii are the stochiometric coefficient from reactant 1 and 2 respectively.
For: 3A → products
3
k⋅A = rate

For: A + 2B → products
2
k ⋅ [A] ⋅ [B] = rate

Note that the order of these reactions are both three.

Answer
Add texts here. Do not delete this text first.

Rate = k[A][B]2; Rate = k[A]3

12.E.6.6: Q12.6.6
Given the following reactions and the corresponding rate laws, in which of the reactions might the elementary reaction and the
overall reaction be the same?
(a) Cl 2
+ CO ⟶ Cl CO
2

rate = k[ Cl ] 2
[CO]
2

(b) PCl 3
+ Cl
2
⟶ PCl
5

Access for free at OpenStax 12.E.42 https://chem.libretexts.org/@go/page/46044


rate = k[ PCl ][ Cl ]
3 2

(c) 2 NO + H 2
⟶ N
2
+H O
2

rate = k[NO][ H ]
2

(d) 2 NO + O 2
⟶ 2 NO
2

2
rate = k[NO] [ O ]
2

(e) NO + O 3
⟶ NO
2
+O
2

rate = k[NO][ O ]
3

Solution
Add texts here. Do not delete this text first.

An elementary reaction is a chemical reaction in which the reactants directly form products in a single step. In another words, the
rate law for the overall reaction is same as experimentally found rate law. Out of 5 options, option (b),(d), and (e) are such reactions

12.E.6.7: Q12.6.7
Write the rate equation for each of the following elementary reactions:
sunlight

a. O −−−−→ O + O
3 2

b. O + Cl ⟶ O + ClO
3 2

c. ClO + O ⟶ Cl + O 2

d. O + NO ⟶ NO + O
3 2 2

e. NO + O ⟶ NO + O
2 2

Solution
Add texts here. Do not delete this text first.

Rate equations are dependent on the reactants and not the products.
The rate law of a reaction can be found using a rate constant (which is found experimentally), and the initial concentrations of
reactants.
A general solution for the equation
aA + bB → cC + dD

is rate = k[A] m
[B]
n
where m and n are reaction orders.
However, reaction orders are found experimentally, and since we do not have experimental data for these reactions, we can
disregard that part of the equation.
To find the rate laws, all we have to do is plug the reactants into the rate formula. This is only due to the case that these are
elementary reactions. Further reading on elementary reactions can be found on Libre Texts.
a. O3 ⟶ O2 + O
To write this reaction's rate equation, only focus on the reactant(s) and its/their concentration and multiplying that by a rate
constant, "k".
Rate = k[O3]
b. O3 + Cl ⟶ O2 + ClO
To write this reaction's rate equation, only focus on the reactant(s) and its/their concentration and multiplying that by a rate
constant, "k".
Rate = k[O3][Cl]
c. ClO + O ⟶ Cl + O2

Access for free at OpenStax 12.E.43 https://chem.libretexts.org/@go/page/46044


To write this reaction's rate equation, only focus on the reactant(s) and its/their concentration and multiplying that by a rate
constant, "k".
Rate = k[ClO][O]
d. O3 + NO ⟶ NO2 + O2
To write this reaction's rate equation, only focus on the reactant(s) and its/their concentration and multiplying that by a rate
constant, "k".
Rate = k[O3][NO]
e. NO2 + O ⟶ NO + O2
To write this reaction's rate equation, only focus on the reactant(s) and its/their concentration and multiplying that by a rate
constant, "k".
Rate = k[NO2][O]

Answer
Add texts here. Do not delete this text first.

(a) Rate1 = k[O3]; (b) Rate2 = k[O3][Cl]; (c) Rate3 = k[ClO][O]; (d) Rate2 = k[O3][NO]; (e) Rate3 = k[NO2][O]

12.E.6.8: Q12.6.8
Nitrogen(II) oxide, NO, reacts with hydrogen, H2, according to the following equation:
2 NO + 2 H ⟶ N +2 H O
2 2 2

What would the rate law be if the mechanism for this reaction were:

2 NO + H ⟶ N +H O (slow)
2 2 2 2

H O +H ⟶ 2 H O (fast)
2 2 2 2

The rate law of the mechanism is determined by the slow step of the reaction. Since the slow step is an elementary step, the rate
law can be drawn from the coefficients of the chemical equation. So therefore, the rate law is as follows: rate=k[NO]2[H2]. Since
both NO and H2 are reactants in the overall reaction (therefore are not intermediates in the reaction), no further steps have to be
done to determine the rate law.

12.E.6.9: Q12.6.9
Consider the reaction
CH4 + Cl2 → CH3Cl + HCl (occurs under light)
The mechanism is a chain reaction involving Cl atoms and CH3 radicals. Which of the following steps does not terminate this chain
reaction?
a. CH3 + Cl → CH3CI
b. CH3 + HCl → CH4 + Cl
c. CH3 + CH3 → C2H2
d. Cl + Cl → Cl2

Solution
Add texts here. Do not delete this text first.

Chain reactions involve reactions that create products necessary for more reactions to occur. In this case, a reaction step will
continue the chain reaction if a radical is generated. Radicals are highly reactive particles, so more reactions in the chain will take
place as long as they are present. The chlorine is considered a free radical as it has an unpaired electron; for this reason it is very
reactive and propagates a chain reaction. It does so by taking an electron from a stable molecule and making that molecule reactive,
and that molecule goes on to react with stable species, and in that manner a long series of "chain" reactions are initiated. A chlorine
radical will continue the chain by completing the following reaction:

Access for free at OpenStax 12.E.44 https://chem.libretexts.org/@go/page/46044


C l⋅ + C H4 → C H3 ⋅ + H C l

The C H generated by this reaction can then react with other species, continuing to propagate the chain reaction.
3

Option 1 is incorrect because the only species it produces is C H C l, a product in the overall reaction that is unreactive. This
3

terminates the chain reaction because it fails to produce any C l or C H radicals that are necessary for further propagating the
3

overall reaction.
Option 2 is the correct answer because it produces a Cl radical. This Cl radical can continue the chain by colliding with C H4

molecules.
Option 3 is incorrect because it fails to produce a radical capable of continuing the chain.
Option 4 is incorrect because it produces C l , a molecule that does not react unless additional light is supplied. Therefore, this step
2

breaks the chain.

Answer
Add texts here. Do not delete this text first.

Answer: Option 2: C H 3 + H C l → C H4 + C l

12.E.6.10: Q12.6.10
Experiments were conducted to study the rate of the reaction represented by this equation.

2 NO(g) + 2 H (g) ⟶ N (g) + 2 H O(g)


2 2 2

Initial concentrations and rates of reaction are given here.

Initial Concentration [NO] Initial Concentration, [H2] Initial Rate of Formation of N2


Experiment
(mol/L) (mol/L) (mol/L min)

1 0.0060 0.0010 1.8 × 10−4

2 0.0060 0.0020 3.6 × 10−4

3 0.0010 0.0060 0.30 × 10−4

4 0.0020 0.0060 1.2 × 10−4

Consider the following questions:


a. Determine the order for each of the reactants, NO and H2, from the data given and show your reasoning.
b. Write the overall rate law for the reaction.
c. Calculate the value of the rate constant, k, for the reaction. Include units.
d. For experiment 2, calculate the concentration of NO remaining when exactly one-half of the original amount of H2 had been
consumed.
e. The following sequence of elementary steps is a proposed mechanism for the reaction.
Step 1: NO + NO ⇌ N 2
O
2

Step 2: N 2
O
2
+H
2
⇌ H O+N O
2 2

Step 3: N 2
O+H
2
⇌ N
2
+H O
2

Based on the data presented, which of these is the rate determining step? Show that the mechanism is consistent with the observed
rate law for the reaction and the overall stoichiometry of the reaction.
S12.6.10
1. i) Find the order for [NO] by using experiment 3 and 4 where [H2] is constant
Notice that [NO] doubles from experiment 3 to 4 and the rate quadruples. So the order for [NO] is 2
ii) Find the order for [H2] by using experiment 1 and 2 where [NO] is constant
Notice that [H2] doubles from experiment 1 to 2 and the rate doubles as well. So the order for [H2] is 1

Access for free at OpenStax 12.E.45 https://chem.libretexts.org/@go/page/46044


2. Put in the order for each product as the exponents for the corresponding reactant.
2
rate = k[N O] [ H2 ]

3. Put in the concentrations and the rate from one of the experiments into the rate law and solve for k. (Here, experiement 1 is used
but any of them will work)
2 2 −2 −1
rate = k[N O] [ H2 ] .00018 = k[.006 ] [.001]k = 5000M s

4. Plug in values for experiment 2 into the rate law equation and solve for the concentration of NO
2 2 −5
.00036 = 5000[N O] [.001][N O] = 7.2x 10 [N O] = .0085M

5. Write the rate laws for each step and then see which matches the rate law we found in question 2. The rate determining step (the
slow step) is the one that gives the rate for the overall reaction. Because of this, only those concentrations will influence the overall
reaction, contrary to what we would believe if we just looked at the overall reaction.
Step 1: N O + N O ⇌ N 2 O2

rate = k1 [N O]
2
This rate law is not the same as the one we calculate in question 2 so this can not be the rate determining
step
Step 2: N 2 O2 + H2 ⇌ N2 O + N2 O

rate = k2 [ N2 O2 ][ H2 ]

Since N O is an intermediate you must replace it in the rate law equation. Intermediates can not be in the rate law because
2 2

they do not appear in the overall reaction. Here you can take the reverse of equation 1 (k-1) and substitute the other side (the
reactants of equation 1) for the intermediate in the rate law equation.
rate1 = rate−1

2
k1 [N O] = k−1 [ N2 O2 ]

2
k1 [N O]
[ N2 O2 ] =
k−1

2
k2 k1 [N O] [ H2 ]
rate =
k−1

Overall: rate = k[N O] 2


[ H2 ] This is the same so it is the rate determining step.
So N 2 O2 + H2 ⇌ N2 O + N2 O is the rate determining step step.

Answer
Add texts here. Do not delete this text first.

(a) NO: 2
H
2
:1
(b) Rate = k [NO]2[H2];
(c) k = 5.0 × 103 mol−2 L−2 min−1;
(d) 0.0050 mol/L;
(e) Step II is the rate-determining step.

12.E.6.11: Q12.6.11
The reaction of CO with Cl2 gives phosgene (COCl2), a nerve gas that was used in World War I. Use the mechanism shown here to
complete the following exercises:
Cl (g) ⇌ 2 Cl(g)
2
(fast, k1 represents the forward rate constant, k−1 the reverse rate constant)
CO(g) + Cl(g) ⟶ COCl(g) (slow, k2 the rate constant)
COCl(g) + Cl(g) ⟶ COCl (g) (fast, k3 the rate constant)
2

a. Write the overall reaction.

Access for free at OpenStax 12.E.46 https://chem.libretexts.org/@go/page/46044


b. Identify all intermediates.
c. Write the rate law for each elementary reaction.
d. Write the overall rate law expression.

Solution
Add texts here. Do not delete this text first.

1. To write the overall reaction you have to identify the intermediates and leave them out. The easiest way to do this is to write out
all the products and reactants and cross out anything that is on both sides.

Cl2(g) + CO(g) + 2Cl(g) +COCl(g) ⇒ 2Cl(g) + COCl(g) + COCl2(g)

In this you will cross out the 2Cl(g) molecules and the COCl(g). What is left after that is the overall reaction.

Cl2(g) + CO(g) ⇒ + COCl2(g)

2. For part two you will just list the intermediates that you crossed out.
Cl and COCl are intermediates
3. Each rate law will be the rate equal to the rate constant times the concentrations of the reactants
reaction 1
(forward) rate=k1[Cl2] ( reverse) rate=k-1[Cl]
reaction 2
rate=k2[CO][Cl]
Reaction 3
rate=k3[COCl][Cl]
4. The overall rate law is based off the slowest step (step #2), since it is the rate determining step, but Cl is present in that rate law
so we have to replace it with an equivalent that does not contain an intermediate. To do this you use the equilibrium since the rates
are the same you can set up the rate laws of the forward and reverse equal to each other.
k1[Cl2] = k-1[Cl]
[Cl]= k1[Cl2]/k-1
rate=k2[CO]k1[Cl2]/k-1
rate=k°[CO][Cl2]
Steps to replacing and intermediate
1. Set the forward and reverse reaction equal to each other using separate constants
2. Solve for the intermediate using algebra
3. Plug into the rate determining formula
4. All the k's will be condensed into a K prime constant

12.E.7: 12.7: Catalysis


12.E.7.1: Q12.7.1
Account for the increase in reaction rate brought about by a catalyst.

12.E.7.2: Q12.7.2
Compare the functions of homogeneous and heterogeneous catalysts.

Access for free at OpenStax 12.E.47 https://chem.libretexts.org/@go/page/46044


12.E.7.3: Q12.7.3
Consider this scenario and answer the following questions: Chlorine atoms resulting from decomposition of chlorofluoromethanes,
such as CCl2F2, catalyze the decomposition of ozone in the atmosphere. One simplified mechanism for the decomposition is:
sunlight

O −−−−→O +O
3 2

O + Cl ⟶ O + ClO
3 2

ClO + O ⟶ Cl + O
2

a. Explain why chlorine atoms are catalysts in the gas-phase transformation:

2O ⟶ 3O
3 2

b. Nitric oxide is also involved in the decomposition of ozone by the mechanism:


sunlight

O −−−−→O +O
3 2

O + NO ⟶ NO +O
3 2 2

NO + O ⟶ NO + O
2 2

Is NO a catalyst for the decomposition? Explain your answer.

12.E.7.4: Q12.7.4
For each of the following pairs of reaction diagrams, identify which of the pair is catalyzed:
(a)

12.E.7.5: Q12.7.5
For each of the following pairs of reaction diagrams, identify which of the pairs is catalyzed:
(a)

Access for free at OpenStax 12.E.48 https://chem.libretexts.org/@go/page/46044


(b)

12.E.7.6: Q12.7.6
For each of the following reaction diagrams, estimate the activation energy (Ea) of the reaction:
(a)

(b)

Access for free at OpenStax 12.E.49 https://chem.libretexts.org/@go/page/46044


12.E.7.7: Q12.7.7
For each of the following reaction diagrams, estimate the activation energy (Ea) of the reaction:
(a)

(b)

12.E.7.8: Q12.7.8
1. Based on the diagrams in Question Q12.7.6, which of the reactions has the fastest rate? Which has the slowest rate?
2. Based on the diagrams in Question Q12.7.7, which of the reactions has the fastest rate? Which has the slowest rate?

This page titled 12.E: Kinetics (Exercises) is shared under a CC BY 4.0 license and was authored, remixed, and/or curated by OpenStax via
source content that was edited to the style and standards of the LibreTexts platform; a detailed edit history is available upon request.

Access for free at OpenStax 12.E.50 https://chem.libretexts.org/@go/page/46044


CHAPTER OVERVIEW
13: Fundamental Equilibrium Concepts

A general chemistry Libretexts Textbook remixed and remastered from


OpenStax's textbook:
General Chemistry
In this chapter, you will learn how to predict the position of the balance and the yield of a product of a reaction under specific
conditions, how to change a reaction's conditions to increase or reduce yield, and how to evaluate an equilibrium system's reaction
to disturbances.
13.0: Prelude to Equilibrium
13.1: Chemical Equilibria
13.2: Equilibrium Constants
13.3: Shifting Equilibria - Le Chatelier’s Principle
13.4: Equilibrium Calculations
13.E: Fundamental Equilibrium Concepts (Exercises)

This page titled 13: Fundamental Equilibrium Concepts is shared under a CC BY 4.0 license and was authored, remixed, and/or curated by
OpenStax via source content that was edited to the style and standards of the LibreTexts platform; a detailed edit history is available upon request.

1
13.0: Prelude to Equilibrium
Imagine a beach populated with sunbathers and swimmers. As those basking in the sun get too hot and want to cool off, they head
into the surf to swim. As the swimmers tire, they head to the beach to rest. If these two rates of transfer (sunbathers entering the
water, swimmers leaving the water) are equal, the number of sunbathers and swimmers would be constant, or at equilibrium,
although the identities of the people are constantly changing from sunbather to swimmer and back. An analogous situation occurs
in chemical reactions. Reactions can occur in both directions simultaneously (reactants to products and products to reactants) and
eventually reach a state of balance.

Figure 13.0.1 : Movement of carbon dioxide through tissues and blood cells involves several equilibrium reactions.
An image depicts three tan squares, lying side-by-side along the upper left corner. Two of the same squares also lie side-by-side in
the lower right corner. Each square has a black dot in the center. One of the squares is labeled, “C O subscript 2,” and has a double-
headed arrow pointing from it to a red tube-like structure that runs between the squares across the image from the upper right to the
lower left. This arrow is labeled, “C O subscript 2 dissolved in plasma.” The red tube has two round red shapes in it, and the upper
one is labeled, “C O subscript 2 carried in red blood cells.” The gaps between the squares and the red tube are colored light blue.
One of the squares along the top of the image is labeled, “C O subscript 2,” and is connected by a double-headed arrow to an
equation in the red tube that is labeled, “C O subscript 2, a plus sign, H subscript 2 O, right-facing arrow, H subscript 2 C O
subscript 3, right-facing arrow, H C O subscript 3 superscript negative sign, plus sign, H superscript positive sign.” The compound
“H C O subscript 3 superscript negative sign” is then connected by a double-headed arrow to the space in the red tube and is
labeled, “H C O subscript 3 superscript negative sign dissolved in plasma as carbonic acid.”
These balanced two-way reactions occur all around and even in us. For example, they occur in our blood, where the reaction
between carbon dioxide and water forms carbonic acid (HCO ) (Figure 13.0.1). Human physiology is adapted to the amount of

ionized products produced by this reaction (HCO and H+). In this chapter, you will learn how to predict the position of the

balance and the yield of a product of a reaction under specific conditions, how to change a reaction's conditions to increase or
reduce yield, and how to evaluate an equilibrium system's reaction to disturbances.

This page titled 13.0: Prelude to Equilibrium is shared under a CC BY 4.0 license and was authored, remixed, and/or curated by OpenStax via
source content that was edited to the style and standards of the LibreTexts platform; a detailed edit history is available upon request.

Access for free at OpenStax 13.0.1 https://chem.libretexts.org/@go/page/38266


13.1: Chemical Equilibria
 Learning Objectives
Describe the nature of equilibrium systems
Explain the dynamic nature of a chemical equilibrium

A chemical reaction is usually written in a way that suggests it proceeds in one direction, the direction in which we read, but all
chemical reactions are reversible, and both the forward and reverse reaction occur to one degree or another depending on
conditions. In a chemical equilibrium, the forward and reverse reactions occur at equal rates, and the concentrations of products and
reactants remain constant. If we run a reaction in a closed system so that the products cannot escape, we often find the reaction
does not give a 100% yield of products. Instead, some reactants remain after the concentrations stop changing. At this point, when
there is no further change in concentrations of reactants and products, we say the reaction is at equilibrium. A mixture of reactants
and products is found at equilibrium.
For example, when we place a sample of dinitrogen tetroxide (N 2 O4 , a colorless gas) in a glass tube, it forms nitrogen dioxide (
NO , a brown gas) by the reaction
2

N O (g) ⇌ 2 NO (g) (13.1.1)


2 4 2

The color becomes darker as N 2


O
4
is converted to NO . When the system reaches equilibrium, both N
2 2
O
4
and NO are present
2

(Figure 13.1.1).

Access for free at OpenStax 13.1.1 https://chem.libretexts.org/@go/page/38267


Figure 13.1.1 : A mixture of NO and N O moves toward equilibrium. Colorless N O reacts to form brown
2 2 4 2 4
NO
2
. As the
reaction proceeds toward equilibrium, the color of the mixture darkens due to the increasing concentration of NO . 2

A three-part diagram is shown. At the top of the diagram, three beakers are shown, and each one contains a sealed tube. The tube in
the left beaker is full of a colorless gas which is connected to a zoom-in view of the particles in the tube by a downward-facing
arrow. This particle view shows seven particles, each composed of two connected blue spheres. Each blue sphere is connected to
two red spheres. The tube in the middle beaker is full of a light brown gas which is connected to a zoom-in view of the particles in
the tube by a downward-facing arrow. This particle view shows nine particles, five of which are composed of two connected blue
spheres. Each blue sphere is connected to two red spheres. The remaining four are composed of two red spheres connected to a blue
sphere. The tube in the right beaker is full of a brown gas which is connected to a zoom-in view of the particles in the tube by a
downward-facing arrow. This particle view shows eleven particles, three of which are composed of two connected blue spheres.
Each blue sphere is connected to two red spheres. The remaining eight are composed of two red spheres connected to a blue sphere.
At the bottom of the image are two graphs. The left graph has a y-axis labeled, “Concentration,” and an x-axis labeled, “Time.” A
red line labeled, “N O subscript 2,” begins in the bottom left corner of the graph at a point labeled, “0,” and rises near the highest
point on the y-axis before it levels off and becomes horizontal. A blue line labeled, “N subscript 2 O subscript 4,” begins near the
highest point on the y-axis and drops below the midpoint of the y-axis before leveling off. The right graph has a y-axis labeled,
“Rate,” and an x-axis labeled, “Time.” A red line labeled, “k subscript f, [ N subscript 2 O subscript 4 ],” begins in the bottom left
corner of the graph at a point labeled, “0,” and rises near the middle of the y-axis before it levels off and becomes horizontal. A
blue line labeled, “k subscript f, [ N O subscript 2 ] superscript 2,” begins near the highest point on the y-axis and drops to the same
point on the y-axis as the red line before leveling off. The point where both lines become horizontal is labeled, “Equilibrium
achieved.”
The formation of NO from N O is a reversible reaction, which is identified by the equilibrium arrow (⇌). All reactions are
2 2 4

reversible, but many reactions, for all practical purposes, proceed in one direction until the reactants are exhausted and will reverse
only under certain conditions. Such reactions are often depicted with a one-way arrow from reactants to products. Many other
reactions, such as the formation of NO from N O , are reversible under more easily obtainable conditions and, therefore, are
2 2 4

named as such. In a reversible reaction, the reactants can combine to form products and the products can react to form the reactants.
Thus, not only can N O decompose to form NO , but the NO produced can react to form N O . As soon as the forward
2 4 2 2 2 4

reaction produces any NO , the reverse reaction begins and NO starts to react to form N O . At equilibrium, the concentrations
2 2 2 4

of N O and NO no longer change because the rate of formation of NO is exactly equal to the rate of consumption of NO , and
2 4 2 2 2

the rate of formation of N O is exactly equal to the rate of consumption of N O . Chemical equilibrium is a dynamic process: As
2 4 2 4

with the swimmers and the sunbathers, the numbers of each remain constant, yet there is a flux back and forth between them
(Figure 13.1.2).

Access for free at OpenStax 13.1.2 https://chem.libretexts.org/@go/page/38267


Figure 13.1.2 : These jugglers provide an illustration of dynamic equilibrium. Each throws clubs to the other at the same rate at
which he receives clubs from that person. Because clubs are thrown continuously in both directions, the number of clubs moving in
each direction is constant, and the number of clubs each juggler has at a given time remains (roughly) constant.
In a chemical equilibrium, the forward and reverse reactions do not stop, rather they continue to occur at the same rate, leading to
constant concentrations of the reactants and the products. Plots showing how the reaction rates and concentrations change with
respect to time are shown in Figure 13.1.1.
We can detect a state of equilibrium because the concentrations of reactants and products do not appear to change. However, it is
important that we verify that the absence of change is due to equilibrium and not to a reaction rate that is so slow that changes in
concentration are difficult to detect.
We use a double arrow when writing an equation for a reversible reaction. Such a reaction may or may not be at equilibrium. For
example, Figure 13.1.1 shows the reaction:
N O (g) ⇌ 2 NO (g) (13.1.2)
2 4 2

When we wish to speak about one particular component of a reversible reaction, we use a single arrow. For example, in the
equilibrium shown in Figure 13.1.1, the rate of the forward reaction
2 NO (g) → N O (g) (13.1.3)
2 2 4

is equal to the rate of the backward reaction


N O (g) → 2 NO (g) (13.1.4)
2 4 2

 Equilibrium and Soft Drinks

The connection between chemistry and carbonated soft drinks goes back to 1767, when Joseph Priestley (1733–1804; mostly
known today for his role in the discovery and identification of oxygen) discovered a method of infusing water with carbon
dioxide to make carbonated water. In 1772, Priestly published a paper entitled “Impregnating Water with Fixed Air.” The paper
describes dripping oil of vitriol (today we call this sulfuric acid, but what a great way to describe sulfuric acid: “oil of vitriol”
literally means “liquid nastiness”) onto chalk (calcium carbonate). The resulting C O falls into the container of water beneath
2

the vessel in which the initial reaction takes place; agitation helps the gaseous C O mix into the liquid water.
2

H SO (l) + CaCO (s) → CO (g) + H O(l) + CaSO (aq)


2 4 3 2 2 4

Carbon dioxide is slightly soluble in water. There is an equilibrium reaction that occurs as the carbon dioxide reacts with the
water to form carbonic acid (H C O ). Since carbonic acid is a weak acid, it can dissociate into protons (H ) and hydrogen
2 3
+

carbonate ions (H C O ).

− +
CO (aq) + H O(l) ⇌ H CO (aq) ⇌ HCO (aq) + H (aq)
2 2 2 3 3

Today, CO can be pressurized into soft drinks, establishing the equilibrium shown above. Once you open the beverage
2

container, however, a cascade of equilibrium shifts occurs. First, the CO gas in the air space on top of the bottle escapes,
2

causing the equilibrium between gas-phase CO and dissolved or aqueous CO to shift, lowering the concentration of CO in
2 2 2

the soft drink. Less CO dissolved in the liquid leads to carbonic acid decomposing to dissolved CO and H2O. The lowered
2 2

carbonic acid concentration causes a shift of the final equilibrium. As long as the soft drink is in an open container, the CO 2

Access for free at OpenStax 13.1.3 https://chem.libretexts.org/@go/page/38267


bubbles up out of the beverage, releasing the gas into the air (Figure 13.1.3). With the lid off the bottle, the CO reactions are
2

no longer at equilibrium and will continue until no more of the reactants remain. This results in a soft drink with a much
lowered CO concentration, often referred to as “flat.”
2

Figure 13.1.3 : When a soft drink is opened, several equilibrium shifts occur. (credit: modification of work by “D
Coetzee”/Flickr)
A bottle of soda sitting on the ground is shown with a large amount of fizz-filled liquid spewing out of the top.

 Sublimation of Bromine

Let us consider the evaporation of bromine as a second example of a system at equilibrium.

Br (l) ⇌ Br (g)
2 2

An equilibrium can be established for a physical change—like this liquid to gas transition—as well as for a chemical reaction.
Figure 13.1.4 shows a sample of liquid bromine at equilibrium with bromine vapor in a closed container. When we pour liquid
bromine into an empty bottle in which there is no bromine vapor, some liquid evaporates, the amount of liquid decreases, and
the amount of vapor increases. If we cap the bottle so no vapor escapes, the amount of liquid and vapor will eventually stop
changing and an equilibrium between the liquid and the vapor will be established. If the bottle were not capped, the bromine
vapor would escape and no equilibrium would be reached.

Figure 13.1.4 : An equilibrium is pictured between liquid bromine, Br2(l), the dark liquid, and bromine vapor, Br2(g), the
orange gas. Because the container is sealed, bromine vapor cannot escape and equilibrium is maintained. (credit: Bromine
[images-of-elements.com]).
A glass container is shown that is filled with an orange-brown gas and a small amount of dark orange liquid.

Summary
A reaction is at equilibrium when the amounts of reactants or products no longer change. Chemical equilibrium is a dynamic
process, meaning the rate of formation of products by the forward reaction is equal to the rate at which the products re-form
reactants by the reverse reaction.

Glossary
equilibrium
in chemical reactions, the state in which the conversion of reactants into products and the conversion of products back into
reactants occur simultaneously at the same rate; state of balance

Access for free at OpenStax 13.1.4 https://chem.libretexts.org/@go/page/38267


reversible reaction
chemical reaction that can proceed in both the forward and reverse directions under given conditions

This page titled 13.1: Chemical Equilibria is shared under a CC BY 4.0 license and was authored, remixed, and/or curated by OpenStax via source
content that was edited to the style and standards of the LibreTexts platform; a detailed edit history is available upon request.

Access for free at OpenStax 13.1.5 https://chem.libretexts.org/@go/page/38267


13.2: Equilibrium Constants
 Learning Objectives
Derive reaction quotients from chemical equations representing homogeneous and heterogeneous reactions
Calculate values of reaction quotients and equilibrium constants, using concentrations and pressures
Relate the magnitude of an equilibrium constant to properties of the chemical system

Now that we have a symbol (⇌) to designate reversible reactions, we will need a way to express mathematically how the amounts
of reactants and products affect the equilibrium of the system. A general equation for a reversible reaction may be written as
follows:
mA + nB+ ⇌ xC + yD (13.2.1)

We can write the reaction quotient (Q) for this equation. When evaluated using concentrations, it is called Q . We use brackets to c

indicate molar concentrations of reactants and products.


x y
[C ] [D]
Qc = (13.2.2)
m n
[A] [B]

The reaction quotient is equal to the molar concentrations of the products of the chemical equation (multiplied together) over the
reactants (also multiplied together), with each concentration raised to the power of the coefficient of that substance in the balanced
chemical equation. For example, the reaction quotient for the reversible reaction
2 NO2(g) ⇌ N O4(g) (13.2.3)
2

is given by this expression:


[N O ]
2 4
Qc = (13.2.4)
2
[ NO ]
2

 Example 13.2.1: Writing Reaction Quotient Expressions


Write the expression for the reaction quotient for each of the following reactions:
a. 3 O ⇌ 2O
2(g) 3(g)

b. N +3 H
2(g)
⇌ 2 NH 2(g) 3(g)

c. 4 NH +7 O3(g)⇌ 4 NO 2(g) 2(g) + 6 H O(g)


2

Solution
2
[O ]
3
a. Qc =
[O ]3
2
2
[NH ]
3
b. Q c =
3
[N ][H ]
2 2
4 6
[ NO ] [ H O]
2 2
c. Qc =
4 7
[ NH ] [ O ]
3 2

 Exercise 13.2.1
Write the expression for the reaction quotient for each of the following reactions:
a. 2 SO (g) + O (g) ⇌ 2 SO (g)
2 2 3

b. C H (g) ⇌ 2 C H (g)
4 8 2 4

c. 2 C H (g) + 13 O (g) ⇌ 8 CO
4 10 2 2
(g) + 10 H O(g)
2

Access for free at OpenStax 13.2.1 https://chem.libretexts.org/@go/page/38268


Answer a
2
[SO ]
3
Qc =
2
[ SO ] [ O ]
2 2

Answer b
2
[C H ]
2 4
Qc =
[C H ]
4 8

Answer c
8 10
[ CO ] [ H O]
2 2
Qc =
2 13
[C H ] [O ]
4 10 2

The numeric value of Q for a given reaction varies; it depends on the concentrations of products and reactants present at the time
c

when Q is determined. When pure reactants are mixed, Q is initially zero because there are no products present at that point. As
c c

the reaction proceeds, the value of Q increases as the concentrations of the products increase and the concentrations of the
c

reactants simultaneously decrease (Figure 13.2.1). When the reaction reaches equilibrium, the value of the reaction quotient no
longer changes because the concentrations no longer change.

Figure 13.2.1 : (a) The change in the concentrations of reactants and products is depicted as the 2 SO (g) + O (g) ⇌ 2 SO (g)
2 2 3

reaction approaches equilibrium. (b) The change in concentrations of reactants and products is depicted as the reaction
2 SO (g) ⇌ 2 SO (g) + O (g) approaches equilibrium. (c) The graph shows the change in the value of the reaction quotient as
3 2 2

the reaction approaches equilibrium.


Three graphs are shown and labeled, “a,” “b,” and “c.” All three graphs have a vertical dotted line running through the middle
labeled, “Equilibrium is reached.” The y-axis on graph a is labeled, “Concentration,” and the x-axis is labeled, “Time.” Three
curves are plotted on graph a. The first is labeled, “[ S O subscript 2 ];” this line starts high on the y-axis, ends midway down the y-
axis, has a steep initial slope and a more gradual slope as it approaches the far right on the x-axis. The second curve on this graph is
labeled, “[ O subscript 2 ];” this line mimics the first except that it starts and ends about fifty percent lower on the y-axis. The third
curve is the inverse of the first in shape and is labeled, “[ S O subscript 3 ].” The y-axis on graph b is labeled, “Concentration,” and
the x-axis is labeled, “Time.” Three curves are plotted on graph b. The first is labeled, “[ S O subscript 2 ];” this line starts low on
the y-axis, ends midway up the y-axis, has a steep initial slope and a more gradual slope as it approaches the far right on the x-axis.
The second curve on this graph is labeled, “[ O subscript 2 ];” this line mimics the first except that it ends about fifty percent lower
on the y-axis. The third curve is the inverse of the first in shape and is labeled, “[ S O subscript 3 ].” The y-axis on graph c is
labeled, “Reaction Quotient,” and the x-axis is labeled, “Time.” A single curve is plotted on graph c. This curve begins at the
bottom of the y-axis and rises steeply up near the top of the y-axis, then levels off into a horizontal line. The top point of this line is
labeled, “k.”

Access for free at OpenStax 13.2.2 https://chem.libretexts.org/@go/page/38268


When a mixture of reactants and products of a reaction reaches equilibrium at a given temperature, its reaction quotient always has
the same value. This value is called the equilibrium constant (K ) of the reaction at that temperature. As for the reaction quotient,
when evaluated in terms of concentrations, it is noted as K . c

That a reaction quotient always assumes the same value at equilibrium can be expressed as:
x y
[C ] [D] . . .
Qc  at equilibrium = Kc = (13.2.5)
[A]m [B]n . . .

This equation is a mathematical statement of the law of mass action: When a reaction has attained equilibrium at a given
temperature, the reaction quotient for the reaction always has the same value.

 Example 13.2.2: Evaluating a Reaction Quotient

Gaseous nitrogen dioxide forms dinitrogen tetroxide according to this equation:

2 NO2(g) ⇌ N O4(g)
2

When 0.10 mol NO is added to a 1.0-L flask at 25 °C, the concentration changes so that at equilibrium, [NO2] = 0.016 M and
2

[N2O4] = 0.042 M.
a. What is the value of the reaction quotient before any reaction occurs?
b. What is the value of the equilibrium constant for the reaction?

Solution
0.10 mol
a. Before any product is formed, [NO 2] = = 0.10 M , and [N2O4] = 0 M. Thus,
1.0 L

[N O ] 0
2 4
Qc = = =0
2 2
[ NO ] 0.10
2

b. At equilibrium, the value of the equilibrium constant is equal to the value of the reaction quotient. At equilibrium,
[N O ] 0.042
2 4 2
Kc = Qc = = = 1.6 × 10 .
2 2
[ NO ] 0.016
2

The equilibrium constant is 1.6 × 102.


Note that dimensional analysis would suggest the unit for this K value should be M−1. However, it is common practice to omit
c

units for K values computed as described here, since it is the magnitude of an equilibrium constant that relays useful
c

information. As will be discussed later in this module, the rigorous approach to computing equilibrium constants uses
dimensionless quantities derived from activities instead of molar concentrations, and so K values are truly unitless. c

 Exercise 13.2.2

For the reaction

2 SO (g) + O (g) ⇌ 2 SO (g)


2 2 3

the concentrations at equilibrium are [SO2] = 0.90 M, [O2] = 0.35 M, and [SO3] = 1.1 M. What is the value of the equilibrium
constant, Kc?

Answer
Kc = 4.3

The magnitude of an equilibrium constant is a measure of the yield of a reaction when it reaches equilibrium. A large value for K c

indicates that equilibrium is attained only after the reactants have been largely converted into products. A small value of K — c

Access for free at OpenStax 13.2.3 https://chem.libretexts.org/@go/page/38268


much less than 1—indicates that equilibrium is attained when only a small proportion of the reactants have been converted into
products.
Once a value of K is known for a reaction, it can be used to predict directional shifts when compared to the value of Q . A system
c c

that is not at equilibrium will proceed in the direction that establishes equilibrium. The data in Figure 13.2.2 illustrate this. When
heated to a consistent temperature, 800 °C, different starting mixtures of CO, H O , CO , and H react to reach compositions
2 2 2

adhering to the same equilibrium (the value of Q changes until it equals the value of Kc). This value is 0.640, the equilibrium
c

constant for the reaction under these conditions.

CO(g) + H O(g) ⇌ CO (g) + H (g) Kc = 0.640 T = 800°C (13.2.6)


2 2 2

It is important to recognize that an equilibrium can be established starting either from reactants or from products, or from a mixture
of both. For example, equilibrium was established from Mixture 2 in Figure 13.2.2 when the products of the reaction were heated
in a closed container. In fact, one technique used to determine whether a reaction is truly at equilibrium is to approach equilibrium
starting with reactants in one experiment and starting with products in another. If the same value of the reaction quotient is
observed when the concentrations stop changing in both experiments, then we may be certain that the system has reached
equilibrium.

Figure 13.2.2 : Concentrations of three mixtures are shown before and after reaching equilibrium at 800 °C for the so-called water
gas shift reaction (Equation 13.2.6 ).
Two sets of bar graphs are shown. The left is labeled, “Before reaction,” and the right is labeled, “At equilibrium.” Both graphs
have y-axes labeled, “Concentration ( M ),” and three bars on the x-axes labeled, “Mixture 1,” “Mixture 2,” and “Mixture 3.” The
y-axis has a scale beginning at 0.00 and ending at 0.10, with measurement increments of 0.02. The bars on the graphs are color
coded and a key is provided with a legend. Red is labeled, “C O;” blue is labeled, “H subscript 2 O;” green is labeled, “C O
subscript 2,” and yellow is labeled, “H subscript 2.” The graph on the left shows the red bar for mixture one just above 0.02 and the
blue bar near 0.05. For mixture two, the green bar is near 0.05, and the yellow bar is near 0.09. For mixture 3, the red bar is near
0.01, the blue bar is slightly above that with green and yellow topping it off at 0.02. On the right graph, the bar for mixture one
shows the red bar slightly above 0.01, the blue bar stacked on it rising slightly above 0.02, the green rising near 0.04, and the
yellow bar reaching near 0.05. A label above this bar reads, “Q equals 0.640.” The bar for mixture two shows the red bar slightly
above 0.02, the blue bar stacked on it rising near 0.05, the green rising near 0.07, and the yellow bar reaching near 0.10. A label
above this bar reads “Q equals 0.640.” The bar for mixture three shows the red bar near 0.01, the blue bar stacked on it rising
slightly above 0.01, the green rising near 0.02, and the yellow bar reaching 0.02. A label above this bar reads “Q equals 0.640”.

 Example 13.2.3: Predicting the Direction of Reaction


Given here are the starting concentrations of reactants and products for three experiments involving this reaction:

CO(g) + H O(g) ⇌ CO (g) + H (g)


2 2 2

with K c = 0.64 . Determine in which direction the reaction proceeds as it goes to equilibrium in each of the three experiments
shown.
Reactant/Products for Three Experiments
Reactants/Products Experiment 1 Experiment 2 Experiment 3

[CO]i 0.0203 M 0.011 M 0.0094 M

[H2O]i 0.0203 M 0.0011 M 0.0025 M

[CO2]i 0.0040 M 0.037 M 0.0015 M

[H2]i 0.0040 M 0.046 M 0.0076 M

Access for free at OpenStax 13.2.4 https://chem.libretexts.org/@go/page/38268


Solution
Experiment 1:
[ CO ][ H ] (0.0040)(0.0040)
2 2
Qc = = = 0.039.
[CO][ H O] (0.0203)(0.0203)
2

Qc < K (0.039 < 0.64)


c

The reaction will shift to the right.


Experiment 2:
[ CO ][ H ] (0.037)(0.046)
2 2 2
Qc = = = 1.4 × 10
[CO][ H O] (0.011)(0.0011)
2

Qc > K (140 > 0.64)


c

The reaction will shift to the left.


Experiment 3:
[ CO ][ H ] (0.0015)(0.0076)
2 2
Qc = = = 0.48
[CO][ H O] (0.0094)(0.0025)
2

Qc < K (0.48 < 0.64)


c

The reaction will shift to the right.

 Exercise 13.2.3

Calculate the reaction quotient and determine the direction in which each of the following reactions will proceed to reach
equilibrium.
(a) A 1.00-L flask containing 0.0500 mol of NO(g), 0.0155 mol of Cl2(g), and 0.500 mol of NOCl:
4
2 NO(g) + Cl (g) ⇌ 2 NOCl(g) Kc = 4.6 × 10
2

(b) A 5.0-L flask containing 17 g of NH3, 14 g of N2, and 12 g of H2:

N (g) + 3 H (g) ⇌ 2 NH (g) Kc = 0.060


2 2 3

(c) A 2.00-L flask containing 230 g of SO3(g):

2 SO (g) ⇌ 2 SO (g) + O (g) Kc = 0.230


3 2 2

Answer a
Qc = 6.45 × 103, shifts right.
Answer b
Qc = 0.12, shifts left.
Answer c
Qc = 0, shifts right

In Example 13.2.2, it was mentioned that the common practice is to omit units when evaluating reaction quotients and equilibrium
constants. It should be pointed out that using concentrations in these computations is a convenient but simplified approach that
sometimes leads to results that seemingly conflict with the law of mass action. For example, equilibria involving aqueous ions
often exhibit equilibrium constants that vary quite significantly (are not constant) at high solution concentrations. This may be
avoided by computing K values using the activities of the reactants and products in the equilibrium system instead of their
c

concentrations. The activity of a substance is a measure of its effective concentration under specified conditions. While a detailed

Access for free at OpenStax 13.2.5 https://chem.libretexts.org/@go/page/38268


discussion of this important quantity is beyond the scope of an introductory text, it is necessary to be aware of a few important
aspects:
Activities are dimensionless (unitless) quantities and are in essence “adjusted” concentrations.
For relatively dilute solutions, a solute's activity and its molar concentration are roughly equal.
Activities for pure condensed phases (solids and liquids) are equal to 1.
Activities for solvents in dilute solutions are equal to 1.
As a consequence of this last two aspects, Q and K expressions do not contain terms for solids or liquids or solvents in dilute
c c

solutions (being numerically equal to 1, these terms have no effect on the expression's value). Several examples of equilibria
yielding such expressions will be encountered in this section.

13.2.1: Homogeneous Equilibria


A homogeneous equilibrium is one in which all of the reactants and products are present in a single solution (by definition, a
homogeneous mixture). In this chapter, we will concentrate on the two most common types of homogeneous equilibria: those
occurring in liquid-phase solutions and those involving exclusively gaseous species. Reactions between solutes in liquid solutions
belong to one type of homogeneous equilibria. The chemical species involved can be molecules, ions, or a mixture of both. Several
examples are provided here:
Example 1
C H (aq) + 2 Br (aq) ⇌ C H Br (aq) (13.2.7)
2 2 2 2 2 4

with associated equilibrium constant


[ C H Br ]
2 2 4
Kc = (13.2.8)
2
[ C H ] [ Br ]
2 2 2

Example 2
− −
I (aq) + I (aq) ⇌ I (aq) (13.2.9)
2 3

with associated equilibrium constant



[I ]
3
Kc = (13.2.10)

[ I ][ I ]
2

Example 3
2+ − + 2+
Hg (aq) + NO (aq) + 3 H O (aq) ⇌ 2 Hg (aq) + HNO (aq) + 4 H O(l) (13.2.11)
2 3 3 2 2

with associated equilibrium constant


2
2+
[ Hg ] [ HNO ]
2
Kc = (13.2.12)
2+ − + 3
[ Hg ][ NO ][H O ]
2 3 3

Example 4
+ −
HF(aq) + H O(l) ⇌ H O (aq) + F (aq) (13.2.13)
2 3

with associated equilibrium constant


+ −
[H O ][ F ]
3
Kc = (13.2.14)
[HF]

Example 5
+ −
NH (aq) + H O(l) ⇌ NH (aq) + OH (aq) (13.2.15)
3 2 4

with associated equilibrium constant

Access for free at OpenStax 13.2.6 https://chem.libretexts.org/@go/page/38268


+ −
[ NH ][ OH ]
4
Kc = (13.2.16)
[ NH ]
3

In each of these examples, the equilibrium system is an aqueous solution, as denoted by the aq annotations on the solute formulas.
Since H2O(l) is the solvent for these solutions, it is assigned an activity of 1, and thus does not appear explicitly as a term in the
K expression, as discussed earlier, even though it may also appear as a reactant or product in the chemical equation.
c

Reactions in which all reactants and products are gases represent a second class of homogeneous equilibria. We use molar
concentrations in the following examples, but we will see shortly that partial pressures of the gases may be used as well:
Example 1
C H (g) ⇌ C H (g) + H (g) (13.2.17)
2 6 2 4 2

with associated equilibrium constant


[ C H ][ H ]
2 4 2
Kc = (13.2.18)
[C H ]
2 6

Example 2

3 O (g) ⇌ 2 O (g) (13.2.19)


2 3

with associated equilibrium constant


2
[O ]
3
Kc = (13.2.20)
3
[O ]
2

Example 3

N (g) + 3 H (g) ⇌ 2 NH (g) (13.2.21)


2 2 3

with associated equilibrium constant


2
[ NH ]
3
Kc = (13.2.22)
3
[N ][H ]
2 2

Example 4
C H (g) + 5 O (g) ⇌ 3 CO (g) + 4 H O(g) (13.2.23)
3 8 2 2 2

with associated equilibrium constant


3 4
[ CO ] [ H O]
2 2
Kc = (13.2.24)
5
[C H ][O ]
3 8 2

Note that the concentration of H O has been included in the last example because water is not the solvent in this gas-phase
2 (g)

reaction and its concentration (and activity) changes.


Whenever gases are involved in a reaction, the partial pressure of each gas can be used instead of its concentration in the equation
for the reaction quotient because the partial pressure of a gas is directly proportional to its concentration at constant temperature.
n
This relationship can be derived from the ideal gas equation, where M is the molar concentration of gas, .
V

P V = nRT (13.2.25)

n
P =( ) RT (13.2.26)
V

= M RT (13.2.27)

Thus, at constant temperature, the pressure of a gas is directly proportional to its concentration. Using the partial pressures of the
gases, we can write the reaction quotient for the system
C H (g) ⇌ C H (g) + H (g) (13.2.28)
2 6 2 4 2

Access for free at OpenStax 13.2.7 https://chem.libretexts.org/@go/page/38268


by following the same guidelines for deriving concentration-based expressions:
PC H PH
2 4 2
QP = (13.2.29)
PC H
2 6

In this equation we use QP to indicate a reaction quotient written with partial pressures: P C H
2 6
is the partial pressure of C2H6; P ,H
2

the partial pressure of H2; and P , the partial pressure of C2H4. At equilibrium:
C H
2 6

PC H PH
2 4 2

KP = QP = (13.2.30)
PC H
2 6

The subscript P in the symbol K designates an equilibrium constant derived using partial pressures instead of concentrations.
P

The equilibrium constant, KP, is still a constant, but its numeric value may differ from the equilibrium constant found for the same
reaction by using concentrations.
Conversion between a value for K , an equilibrium constant expressed in terms of concentrations, and a value for K , an
c P

equilibrium constant expressed in terms of pressures, is straightforward (a K or Q without a subscript could be either concentration
or pressure).
The equation relating K and K is derived as follows. For the gas-phase reaction:
c P

mA + nB ⇌ xC + yD (13.2.31)

with
x y
(PC ) (PD )
KP = (13.2.32)
m n
(PA ) (PB )

x y
([C] × RT ) ([D] × RT )
= (13.2.33)
m n
([A] × RT ) ([B] × RT )

x y x+y
[C ] [D] (RT )
= × (13.2.34)
[A]m [B]n (RT )m+n

(x+y)−(m+n)
= Kc (RT ) (13.2.35)

Δn
= Kc (RT ) (13.2.36)

The relationship between K and K is


c P

Δn
KP = Kc (RT ) (13.2.37)

In this equation, Δn is the difference between the sum of the coefficients of the gaseous products and the sum of the coefficients of
the gaseous reactants in the reaction (the change in moles of gas between the reactants and the products). For the gas-phase reaction
mA + nB ⇌ xC + yD , we have

Δn = (x + y) − (m + n) (13.2.38)

 Example 13.2.4: Calculation of KP


Write the equations for the conversion of K to KP for each of the following reactions:
c

a. C H (g) ⇌ C H (g) + H (g)


2 6 2 4 2

b. CO(g) + H O(g) ⇌ CO (g) + H (g)


2 2 2

c. N (g) + 3 H (g) ⇌ 2 NH (g)


2 2 3

d. Kc is equal to 0.28 for the following reaction at 900 °C:

CS (g) + 4 H (g) ⇌ CH (g) + 2 H S(g)


2 2 4 2

What is KP at this temperature?

Solution

Access for free at OpenStax 13.2.8 https://chem.libretexts.org/@go/page/38268


(a) Δn = (2) − (1) = 1
KP = K (RT)Δn = K (RT)1 = K (RT)
c c c

(b) Δn = (2) − (2) = 0


KP = K (RT)Δn = K (RT)0 = Kc
c c

(c) Δn = (2) − (1 + 3) = −2
Kc
KP = K (RT)Δn = K (RT)−2 =
c c
2
(RT )

d) KP = K (RT)Δn = (0.28)[(0.0821)(1173)]−2 = 3.0 × 10−5


c

 Exercise 13.2.4
Write the equations for the conversion of K to KP for each of the following reactions, which occur in the gas phase:
c

a. 2 SO (g) + O (g) ⇌ 2 SO (g)


2 2 3

b. N O (g) ⇌ 2 NO (g)
2 4 2

c. C H (g) + 5 O (g) ⇌ 3 CO (g) + 4 H O(g)


3 8 2 2 2

d. At 227 °C, the following reaction has K = 0.0952:


c

CH OH(g) ⇌ CO(g) + 2 H (g)


3 2

What would be the value of KP at this temperature?

Answer a
KP = K (RT)−1
c

Answer b
KP = K (RT)c

Answer c
KP = K (RT);
c

Answer d
160 or 1.6 × 102

13.2.2: Heterogeneous Equilibria


A heterogeneous equilibrium is a system in which reactants and products are found in two or more phases. The phases may be any
combination of solid, liquid, or gas phases, and solutions. When dealing with these equilibria, remember that solids and pure
liquids do not appear in equilibrium constant expressions (the activities of pure solids, pure liquids, and solvents are 1).
Some heterogeneous equilibria involve chemical changes:
Example 1
2+ −
PbCl (s) ⇌ Pb (aq) + 2 Cl (aq) (13.2.39)
2

with associated equilibrium constant


2+ − 2
Kc = [ Pb ] [ Cl ] (13.2.40)

Example 1
CaO(s) + CO (g) ⇌ CaCO (s) (13.2.41)
2 3

with associated equilibrium constant


1
Kc = (13.2.42)
[ CO ]
2

Access for free at OpenStax 13.2.9 https://chem.libretexts.org/@go/page/38268


Example 1
C(s) + 2 S(g) ⇌ CS (g) (13.2.43)
2

with associated equilibrium constant


[ CS ]
2
Kc = (13.2.44)
2
[S]

Other heterogeneous equilibria involve phase changes, for example, the evaporation of liquid bromine, as shown in the following
equation:
Br (l) ⇌ Br (g) (13.2.45)
2 2

with associated equilibrium constant


Kc = [ Br ] (13.2.46)
2

We can write equations for reaction quotients of heterogeneous equilibria that involve gases, using partial pressures instead of
concentrations. Two examples are:
CaO(s) + CO (g) ⇌ CaCO (s) (13.2.47)
2 3

with associated equilibrium constant


1
KP = (13.2.48)
PCO
2

C(s) + 2 S(g) ⇌ CS (g) (13.2.49)


2

with associated equilibrium constant


PCS
2
KP = (13.2.50)
2
(PS )

Summary
For any reaction that is at equilibrium, the reaction quotient Q is equal to the equilibrium constant K for the reaction. If a reactant or
product is a pure solid, a pure liquid, or the solvent in a dilute solution, the concentration of this component does not appear in the
expression for the equilibrium constant. At equilibrium, the values of the concentrations of the reactants and products are constant.
Their particular values may vary depending on conditions, but the value of the reaction quotient will always equal K (Kc when
using concentrations or KP when using partial pressures).
A homogeneous equilibrium is an equilibrium in which all components are in the same phase. A heterogeneous equilibrium is an
equilibrium in which components are in two or more phases. We can decide whether a reaction is at equilibrium by comparing the
reaction quotient with the equilibrium constant for the reaction.

13.2.3: Key Equations


x y
[C ] [D]
Q = where mA + nB ⇌ xC + yD
m n
[A] [B]
x y
(PC ) (PD )
QP = where mA + nB ⇌ xC + yD
(PA )m (PB )n

P = MRT
KP = K (RT)Δn
c

Glossary
equilibrium constant (K)
value of the reaction quotient for a system at equilibrium

heterogeneous equilibria

Access for free at OpenStax 13.2.10 https://chem.libretexts.org/@go/page/38268


equilibria between reactants and products in different phases

homogeneous equilibria
equilibria within a single phase

Kc
equilibrium constant for reactions based on concentrations of reactants and products

KP
equilibrium constant for gas-phase reactions based on partial pressures of reactants and products

law of mass action


when a reversible reaction has attained equilibrium at a given temperature, the reaction quotient remains constant

reaction quotient (Q)


ratio of the product of molar concentrations (or pressures) of the products to that of the reactants, each concentration (or
pressure) being raised to the power equal to the coefficient in the equation

This page titled 13.2: Equilibrium Constants is shared under a CC BY 4.0 license and was authored, remixed, and/or curated by OpenStax via
source content that was edited to the style and standards of the LibreTexts platform; a detailed edit history is available upon request.

Access for free at OpenStax 13.2.11 https://chem.libretexts.org/@go/page/38268


13.3: Shifting Equilibria - Le Chatelier’s Principle
 Learning Objectives
Describe the ways in which an equilibrium system can be stressed
Predict the response of a stressed equilibrium using Le Chatelier’s principle

As we saw in the previous section, reactions proceed in both directions (reactants go to products and products go to reactants). We
can tell a reaction is at equilibrium if the reaction quotient (Q) is equal to the equilibrium constant (K). We next address what
happens when a system at equilibrium is disturbed so that Q is no longer equal to K . If a system at equilibrium is subjected to a
perturbance or stress (such as a change in concentration) the position of equilibrium changes. Since this stress affects the
concentrations of the reactants and the products, the value of Q will no longer equal the value of K . To re-establish equilibrium,
the system will either shift toward the products (if Q < K ) or the reactants (if Q > K ) until Q returns to the same value as K .
This process is described by Le Chatelier's principle: When a chemical system at equilibrium is disturbed, it returns to equilibrium
by counteracting the disturbance. As described in the previous paragraph, the disturbance causes a change in Q; the reaction will
shift to re-establish Q = K .

13.3.1: Predicting the Direction of a Reversible Reaction


Le Chatelier's principle can be used to predict changes in equilibrium concentrations when a system that is at equilibrium is
subjected to a stress. However, if we have a mixture of reactants and products that have not yet reached equilibrium, the changes
necessary to reach equilibrium may not be so obvious. In such a case, we can compare the values of Q and K for the system to
predict the changes.

A chemical system at equilibrium can be temporarily shifted out of equilibrium by adding or removing one or more of the
reactants or products. The concentrations of both reactants and products then undergo additional changes to return the system
to equilibrium.

The stress on the system in Figure 13.3.1 is the reduction of the equilibrium concentration of SCN− (lowering the concentration of
one of the reactants would cause Q to be larger than K). As a consequence, Le Chatelier's principle leads us to predict that the
concentration of Fe(SCN)2+ should decrease, increasing the concentration of SCN− part way back to its original concentration, and
increasing the concentration of Fe3+ above its initial equilibrium concentration.

Figure 13.3.1: (a) The test tube contains 0.1 M Fe3+. (b) Thiocyanate ion has been added to solution in (a), forming the red
Fe(SCN)2+ ion. Fe (aq) + SCN (aq) ⇌ Fe(SCN) (aq) . (c) Silver nitrate has been added to the solution in (b), precipitating
3+ − 2 +

some of the SCN− as the white solid AgSCN. Ag (aq) + SCN (aq) ⇌ AgSCN(s). The decrease in the SCN− concentration
+ −

shifts the first equilibrium in the solution to the left, decreasing the concentration (and lightening color) of the Fe(SCN)2+. (credit:
modification of work by Mark Ott).
Three capped test tubes held vertically in clamps are shown in pictures labeled, “a,” “b,” and “c.” The test tube in picture a is half
filled with a clear, orange liquid. The test tube in picture b is half filled with a dark, burgundy liquid. The test tube in picture c is
half filled with a slightly cloudy, orange liquid.
The effect of a change in concentration on a system at equilibrium is illustrated further by the equilibrium of this chemical reaction:

H2(g) + I2(g) ⇌ 2 HI(g) (13.3.1)

Kc = 50.0 at 400°C (13.3.2)

Access for free at OpenStax 13.3.1 https://chem.libretexts.org/@go/page/38269


The numeric values for this example have been determined experimentally. A mixture of gases at 400 °C with
[ H ] = [ I ] = 0.221 M and [HI] = 1.563 M is at equilibrium; for this mixture, Q = K = 50.0 . If H
2 2 is introduced into the
c c 2

system so quickly that its concentration doubles before it begins to react (new [H ] = 0.442 M), the reaction will shift so that a
2

new equilibrium is reached, at which


[ H ] = 0.374 M
2
,
[ I ] = 0.153 M
2
, and
[HI] = 1.692 M.

This gives:
2 2
[HI] (1.692)
Qc = = = 50.0 = Kc (13.3.3)
[ H2 ][ I2 ] (0.374)(0.153)

We have stressed this system by introducing additional H . The stress is relieved when the reaction shifts to the right, using up
2

some (but not all) of the excess H , reducing the amount of uncombined I , and forming additional HI .
2 2

13.3.2: Effect of Change in Pressure on Equilibrium


Sometimes we can change the position of equilibrium by changing the pressure of a system. However, changes in pressure have a
measurable effect only in systems in which gases are involved, and then only when the chemical reaction produces a change in the
total number of gas molecules in the system. An easy way to recognize such a system is to look for different numbers of moles of
gas on the reactant and product sides of the equilibrium. While evaluating pressure (as well as related factors like volume), it is
important to remember that equilibrium constants are defined with regard to concentration (for K ) or partial pressure (for K ).
c P

Some changes to total pressure, like adding an inert gas that is not part of the equilibrium, will change the total pressure but not the
partial pressures of the gases in the equilibrium constant expression. Thus, addition of a gas not involved in the equilibrium will not
perturb the equilibrium.
As we increase the pressure of a gaseous system at equilibrium, either by decreasing the volume of the system or by adding more of
one of the components of the equilibrium mixture, we introduce a stress by increasing the partial pressures of one or more of the
components. In accordance with Le Chatelier's principle, a shift in the equilibrium that reduces the total number of molecules per
unit of volume will be favored because this relieves the stress. The reverse reaction would be favored by a decrease in pressure.
Consider what happens when we increase the pressure on a system in which NO, O , and NO are at equilibrium:
2 2

2 NO(g) + O (g) ⇌ 2 NO (g) (13.3.4)


2 2

The formation of additional amounts of NO decreases the total number of molecules in the system because each time two
2

molecules of NO form, a total of three molecules of NO and O are consumed. This reduces the total pressure exerted by the
2 2

system and reduces, but does not completely relieve, the stress of the increased pressure. On the other hand, a decrease in the
pressure on the system favors decomposition of NO into NO and O , which tends to restore the pressure.
2 2

Now consider this reaction:

N (g) + O (g) ⇌ 2 NO(g) (13.3.5)


2 2

Because there is no change in the total number of molecules in the system during reaction, a change in pressure does not favor
either formation or decomposition of gaseous nitrogen monoxide.

13.3.3: Effect of Change in Temperature on Equilibrium


Changing concentration or pressure perturbs an equilibrium because the reaction quotient is shifted away from the equilibrium
value. Changing the temperature of a system at equilibrium has a different effect: A change in temperature actually changes the
value of the equilibrium constant. However, we can qualitatively predict the effect of the temperature change by treating it as a
stress on the system and applying Le Chatelier's principle.
When hydrogen reacts with gaseous iodine, heat is evolved.

H (g) + I (g) ⇌ 2 HI(g)  ΔH = −9.4 kJ (exothermic) (13.3.6)


2 2

Because this reaction is exothermic, we can write it with heat as a product.

Access for free at OpenStax 13.3.2 https://chem.libretexts.org/@go/page/38269


H (g) + I (g) ⇌ 2 HI(g) + heat (13.3.7)
2 2

Increasing the temperature of the reaction increases the internal energy of the system. Thus, increasing the temperature has the
effect of increasing the amount of one of the products of this reaction. The reaction shifts to the left to relieve the stress, and there
is an increase in the concentration of H and I and a reduction in the concentration of HI . Lowering the temperature of this
2 2

system reduces the amount of energy present, favors the production of heat, and favors the formation of hydrogen iodide.
When we change the temperature of a system at equilibrium, the equilibrium constant for the reaction changes. Lowering the
temperature in the HI system increases the equilibrium constant: At the new equilibrium the concentration of HI has increased and
the concentrations of H and I decreased. Raising the temperature decreases the value of the equilibrium constant, from 67.5 at
2 2

357 °C to 50.0 at 400 °C.


Temperature affects the equilibrium between NO and N 2 2
O
4
in this reaction
N O (g) ⇌ 2 NO (g) ΔH = 57.20 kJ (13.3.8)
2 4 2

The positive ΔH value tells us that the reaction is endothermic and could be written
heat + N O (g) ⇌ 2 NO (g) (13.3.9)
2 4 2

At higher temperatures, the gas mixture has a deep brown color, indicative of a significant amount of brown NO molecules. If,
2

however, we put a stress on the system by cooling the mixture (withdrawing energy), the equilibrium shifts to the left to supply
some of the energy lost by cooling. The concentration of colorless N O increases, and the concentration of brown NO
2 4 2

decreases, causing the brown color to fade.

13.3.4: Catalysts Do Not Affect Equilibrium


As we learned during our study of kinetics, a catalyst can speed up the rate of a reaction. Though this increase in reaction rate may
cause a system to reach equilibrium more quickly (by speeding up the forward and reverse reactions), a catalyst has no effect on the
value of an equilibrium constant nor on equilibrium concentrations.
The interplay of changes in concentration or pressure, temperature, and the lack of an influence of a catalyst on a chemical
equilibrium is illustrated in the industrial synthesis of ammonia from nitrogen and hydrogen according to the equation
N (g) + 3 H (g) ⇌ 2 NH (g) (13.3.10)
2 2 3

A large quantity of ammonia is manufactured by this reaction. Each year, ammonia is among the top 10 chemicals, by mass,
manufactured in the world. About 2 billion pounds are manufactured in the United States each year. Ammonia plays a vital role in
our global economy. It is used in the production of fertilizers and is, itself, an important fertilizer for the growth of corn, cotton, and
other crops. Large quantities of ammonia are converted to nitric acid, which plays an important role in the production of fertilizers,
explosives, plastics, dyes, and fibers, and is also used in the steel industry.

 Fritz Haber

Haber was born in Breslau, Prussia (presently Wroclaw, Poland) in December 1868. He went on to study chemistry and, while
at the University of Karlsruhe, he developed what would later be known as the Haber process: the catalytic formation of
ammonia from hydrogen and atmospheric nitrogen under high temperatures and pressures.

The work of Nobel Prize recipient Fritz Haber revolutionized agricultural practices in the early 20th century. His work also
affected wartime strategies, adding chemical weapons to the artillery.

Access for free at OpenStax 13.3.3 https://chem.libretexts.org/@go/page/38269


For this work, Haber was awarded the 1918 Nobel Prize in Chemistry for synthesis of ammonia from its elements. The Haber
process was a boon to agriculture, as it allowed the production of fertilizers to no longer be dependent on mined feed stocks
such as sodium nitrate.

N (g) + 3 H (g) ⇌ 2 NH (g)


2 2 3

Currently, the annual production of synthetic nitrogen fertilizers exceeds 100 million tons and synthetic fertilizer production
has increased the number of humans that arable land can support from 1.9 persons per hectare in 1908 to 4.3 in 2008. The
availability of nitrogen is a strong limiting factor to the growth of plants. Despite accounting for 78% of air, diatomic nitrogen (
N ) is nutritionally unavailable due the tremendous stability of the nitrogen-nitrogen triple bond. For plants to use atmospheric
2

nitrogen, the nitrogen must be converted to a more bioavailable form (this conversion is called nitrogen fixation).
In addition to his work in ammonia production, Haber is also remembered by history as one of the fathers of chemical warfare.
During World War I, he played a major role in the development of poisonous gases used for trench warfare. Regarding his role
in these developments, Haber said, “During peace time a scientist belongs to the World, but during war time he belongs to his
country.”1 Haber defended the use of gas warfare against accusations that it was inhumane, saying that death was death, by
whatever means it was inflicted. He stands as an example of the ethical dilemmas that face scientists in times of war and the
double-edged nature of the sword of science.
Like Haber, the products made from ammonia can be multifaceted. In addition to their value for agriculture, nitrogen
compounds can also be used to achieve destructive ends. Ammonium nitrate has also been used in explosives, including
improvised explosive devices. Ammonium nitrate was one of the components of the bomb used in the attack on the Alfred P.
Murrah Federal Building in downtown Oklahoma City on April 19, 1995.

It has long been known that nitrogen and hydrogen react to form ammonia. However, it became possible to manufacture ammonia
in useful quantities by the reaction of nitrogen and hydrogen only in the early 20th century after the factors that influence its
equilibrium were understood.
To be practical, an industrial process must give a large yield of product relatively quickly. One way to increase the yield of
ammonia is to increase the pressure on the system in which N2, H2, and NH3 are at equilibrium or are coming to equilibrium.
N (g) + 3 H (g) ⇌ 2 NH (g) (13.3.11)
2 2 3

The formation of additional amounts of ammonia reduces the total pressure exerted by the system and somewhat reduces the stress
of the increased pressure.
Although increasing the pressure of a mixture of N2, H2, and NH3 will increase the yield of ammonia, at low temperatures, the rate
of formation of ammonia is slow. At room temperature, for example, the reaction is so slow that if we prepared a mixture of N2 and
H2, no detectable amount of ammonia would form during our lifetime. The formation of ammonia from hydrogen and nitrogen is
an exothermic process:
N (g) + 3 H (g) → 2 NH (g) ΔH = −92.2 kJ (13.3.12)
2 2 3

Thus, increasing the temperature to increase the rate lowers the yield. If we lower the temperature to shift the equilibrium to favor
the formation of more ammonia, equilibrium is reached more slowly because of the large decrease of reaction rate with decreasing
temperature.
Part of the rate of formation lost by operating at lower temperatures can be recovered by using a catalyst. The net effect of the
catalyst on the reaction is to cause equilibrium to be reached more rapidly. In the commercial production of ammonia, conditions of
about 500 °C, 150–900 atm, and the presence of a catalyst are used to give the best compromise among rate, yield, and the cost of
the equipment necessary to produce and contain high-pressure gases at high temperatures (Figure 13.3.2).
<div data-mt-source="1"

Figure 13.3.2 : Commercial production of ammonia requires heavy equipment to handle the high temperatures and pressures
required. This schematic outlines the design of an ammonia plant.

Summary
Systems at equilibrium can be disturbed by changes to temperature, concentration, and, in some cases, volume and pressure;
volume and pressure changes will disturb equilibrium if the number of moles of gas is different on the reactant and product sides of
the reaction. The system's response to these disturbances is described by Le Chatelier's principle: The system will respond in a way

Access for free at OpenStax 13.3.4 https://chem.libretexts.org/@go/page/38269


that counteracts the disturbance. Not all changes to the system result in a disturbance of the equilibrium. Adding a catalyst affects
the rates of the reactions but does not alter the equilibrium, and changing pressure or volume will not significantly disturb systems
with no gases or with equal numbers of moles of gas on the reactant and product side.
Table 13.3.1 : Effects of Disturbances of Equilibrium and K
Observed Change as
Disturbance Direction of Shift Effect on K
Equilibrium is Restored

added reactant is partially


reactant added toward products none
consumed

added product is partially


product added toward reactants none
consumed

decrease in volume/increase in gas toward side with fewer moles of


pressure decreases none
pressure gas

increase in volume/decrease in gas toward side with more moles of


pressure increases none
pressure gas

toward products for endothermic,


temperature increase heat is absorbed changes
toward reactants for exothermic

toward reactants for endothermic,


temperature decrease heat is given off changes
toward products for exothermic

Footnotes
1. Herrlich, P. “The Responsibility of the Scientist: What Can History Teach Us About How Scientists Should Handle Research
That Has the Potential to Create Harm?” EMBO Reports 14 (2013): 759–764.

Glossary
Le Chatelier's principle
when a chemical system at equilibrium is disturbed, it returns to equilibrium by counteracting the disturbance

position of equilibrium
concentrations or partial pressures of components of a reaction at equilibrium (commonly used to describe conditions before a
disturbance)

stress
change to a reaction's conditions that may cause a shift in the equilibrium

This page titled 13.3: Shifting Equilibria - Le Chatelier’s Principle is shared under a CC BY 4.0 license and was authored, remixed, and/or curated
by OpenStax via source content that was edited to the style and standards of the LibreTexts platform; a detailed edit history is available upon
request.

Access for free at OpenStax 13.3.5 https://chem.libretexts.org/@go/page/38269


13.4: Equilibrium Calculations
 Learning Objectives
Write equations representing changes in concentration and pressure for chemical species in equilibrium systems
Use algebra to perform various types of equilibrium calculations

We know that at equilibrium, the value of the reaction quotient of any reaction is equal to its equilibrium constant. Thus, we can
use the mathematical expression for Q to determine a number of quantities associated with a reaction at equilibrium or approaching
equilibrium. While we have learned to identify in which direction a reaction will shift to reach equilibrium, we want to extend that
understanding to quantitative calculations. We do so by evaluating the ways that the concentrations of products and reactants
change as a reaction approaches equilibrium, keeping in mind the stoichiometric ratios of the reaction. This algebraic approach to
equilibrium calculations will be explored in this section.
Changes in concentrations or pressures of reactants and products occur as a reaction system approaches equilibrium. In this section
we will see that we can relate these changes to each other using the coefficients in the balanced chemical equation describing the
system. We use the decomposition of ammonia as an example.
On heating, ammonia reversibly decomposes into nitrogen and hydrogen according to this equation:

2 NH (g) ⇌ N (g) + 3 H (g)


3 2 2

If a sample of ammonia decomposes in a closed system and the concentration of N2 increases by 0.11 M, the change in the N2
concentration, Δ[N2], the final concentration minus the initial concentration, is 0.11 M. The change is positive because the
concentration of N2 increases.
The change in the H2 concentration, Δ[H2], is also positive—the concentration of H2 increases as ammonia decomposes. The
chemical equation tells us that the change in the concentration of H2 is three times the change in the concentration of N2 because
for each mole of N2 produced, 3 moles of H2 are produced.
Δ[ H ] = 3 × Δ[ N ]
2 2

= 3 × (0.11 M )

= 0.33 M

The change in concentration of NH3, Δ[NH3], is twice that of Δ[N2]; the equation indicates that 2 moles of NH3 must decompose
for each mole of N2 formed. However, the change in the NH3 concentration is negative because the concentration of ammonia
decreases as it decomposes.
Δ[ NH ] = −2 × Δ[ N ]
3 2

= −2 × (0.11 M )

= −0.22 M

We can relate these relationships directly to the coefficients in the equation


2 NH (g) ⇌ N (g) + 3 H (g) (13.4.1)
3 2 2

Δ[ NH ] = −2 × Δ[ N ] Δ[ N ] = 0.11 M Δ[ H ] = 3 × Δ[ N ] (13.4.2)
3 2 2 2 2

Note that all the changes on one side of the arrows are of the same sign and that all the changes on the other side of the arrows are
of the opposite sign.
If we did not know the magnitude of the change in the concentration of N2, we could represent it by the symbol x.
Δ[ N ] = x
2

The changes in the other concentrations would then be represented as:


Δ[ H ] = 3 × Δ[ N ] = 3x
2 2

Δ[ NH ] = −2 × Δ[ N ] = −2x
3 2

Access for free at OpenStax 13.4.1 https://chem.libretexts.org/@go/page/38270


The coefficients in the Δ terms are identical to those in the balanced equation for the reaction.
2 NH (g) ⇌ N (g) + 3 H (g) (13.4.3)
3 2 2

− 2x x 3x (13.4.4)

The simplest way for us to find the coefficients for the concentration changes in any reaction is to use the coefficients in the
balanced chemical equation. The sign of the coefficient is positive when the concentration increases; it is negative when the
concentration decreases.

 Example 13.4.1: Determining Relative Changes in Concentration

Complete the changes in concentrations for each of the following reactions.


(a)
C H (g) + 2 Br (g) ⇌ C H Br (g) (13.4.5)
2 2 2 2 2 4

x (13.4.6)
––––– –––––

(b)
− −
I (aq) + I (aq) ⇌ I (aq) (13.4.7)
2 3

x (13.4.8)
––––– –––––

(c)
C H (g) + 5 O (g) ⇌ 3 CO (g) + 4 H O(g) (13.4.9)
3 8 2 2 2

x (13.4.10)
––––– ––––– –––––

Solution
(a)
C H (g) + 2 Br (g) ⇌ C H Br (g) (13.4.11)
2 2 2 2 2 4

x 2x −x (13.4.12)

(b)
− −
I (aq) + I (aq) ⇌ I (aq) (13.4.13)
2 3

−x −x x (13.4.14)

(c)
C H (g) + 5 O (g) ⇌ 3 CO (g) + 4 H O(g) (13.4.15)
3 8 2 2 2

x 5x − 3x − 4x (13.4.16)

 Exercise 13.4.1
Complete the changes in concentrations for each of the following reactions:
(a)
2 SO (g) + O (g) ⇌ 2 SO (g) (13.4.17)
2 2 3

x (13.4.18)
––––– –––––

(b)
C H (g) ⇌ 2 C H (g) (13.4.19)
4 8 2 4

− 2x (13.4.20)
–––––

(c)
4 NH (g) + 7 H O(g) ⇌ 4 NO (g) + 6 H O(g) (13.4.21)
3 2 2 2

(13.4.22)
––––– ––––– ––––– –––––

Answer a
2x, x, −2x
Answer b

Access for free at OpenStax 13.4.2 https://chem.libretexts.org/@go/page/38270


x, −2x
Answer c
4x, 7x, −4x, −6x or −4x, −7x, 4x, 6x

13.4.0.1: Calculations Involving Equilibrium Concentrations


Because the value of the reaction quotient of any reaction at equilibrium is equal to its equilibrium constant, we can use the
mathematical expression for Qc (i.e., the law of mass action) to determine a number of quantities associated with a reaction at
equilibrium. It may help if we keep in mind that Qc = Kc (at equilibrium) in all of these situations and that there are only three basic
types of calculations:
1. Calculation of an equilibrium constant. If concentrations of reactants and products at equilibrium are known, the value of the
equilibrium constant for the reaction can be calculated.
2. Calculation of missing equilibrium concentrations. If the value of the equilibrium constant and all of the equilibrium
concentrations, except one, are known, the remaining concentration can be calculated.
3. Calculation of equilibrium concentrations from initial concentrations. If the value of the equilibrium constant and a set of
concentrations of reactants and products that are not at equilibrium are known, the concentrations at equilibrium can be
calculated.
A similar list could be generated using QP, KP, and partial pressure. We will look at solving each of these cases in sequence.

13.4.1: Calculation of an Equilibrium Constant


Since the law of mass action is the only equation we have to describe the relationship between Kc and the concentrations of
reactants and products, any problem that requires us to solve for Kc must provide enough information to determine the reactant and
product concentrations at equilibrium. Armed with the concentrations, we can solve the equation for Kc, as it will be the only
unknown.
Example 13.4.1 showed us how to determine the equilibrium constant of a reaction if we know the concentrations of reactants and
products at equilibrium. The following example shows how to use the stoichiometry of the reaction and a combination of initial
concentrations and equilibrium concentrations to determine an equilibrium constant. This technique, commonly called an ICE table
—for Initial, Change, and Equilibrium–will be helpful in solving many equilibrium problems. A chart is generated beginning with
the equilibrium reaction in question. Underneath the reaction the initial concentrations of the reactants and products are listed—
these conditions are usually provided in the problem and we consider no shift toward equilibrium to have happened. The next row
of data is the change that occurs as the system shifts toward equilibrium—do not forget to consider the reaction stoichiometry as
described in a previous section of this chapter. The last row contains the concentrations once equilibrium has been reached.

 Example 13.4.2: Calculation of an Equilibrium Constant

Iodine molecules react reversibly with iodide ions to produce triiodide ions.
− −
I (aq) + I (aq) ⇌ I (aq)
2 3

If a solution with the concentrations of I2 and I− both equal to 1.000 × 10−3 M before reaction gives an equilibrium
concentration of I2 of 6.61 × 10−4 M, what is the equilibrium constant for the reaction?

Solution
We will begin this problem by calculating the changes in concentration as the system goes to equilibrium. Then we determine
the equilibrium concentrations and, finally, the equilibrium constant. First, we set up a table with the initial concentrations, the
changes in concentrations, and the equilibrium concentrations using −x as the change in concentration of I2.

Access for free at OpenStax 13.4.3 https://chem.libretexts.org/@go/page/38270


Since the equilibrium concentration of I2 is given, we can solve for x. At equilibrium the concentration of I2 is 6.61 × 10−4 M
so that
−3 −4
1.000 × 10 − x = 6.61 × 10

−3 −4
x = 1.000 × 10 − 6.61 × 10

−4
= 3.39 × 10 M

Now we can fill in the table with the concentrations at equilibrium.

We now calculate the value of the equilibrium constant.



[I ]
3
Kc = Qc =

[ I ][ I ]
2

−4
3.39 × 10 M
= = 776
−4 −4
(6.61 × 10 M )(6.61 × 10 M)

 Exercise 13.4.2

Ethanol and acetic acid react and form water and ethyl acetate, the solvent responsible for the odor of some nail polish
removers.
C H OH + CH CO H ⇌ CH CO C H +H O
2 5 3 2 3 2 2 5 2

When 1 mol each of C2H5OH and CH3CO2H are allowed to react in 1 L of the solvent dioxane, equilibrium is established
1
when mol of each of the reactants remains. Calculate the equilibrium constant for the reaction. (Note: Water is not a solvent
3
in this reaction.)

Answer
Kc = 4

13.4.1.1: Calculation of a Missing Equilibrium Concentration


If we know the equilibrium constant for a reaction and know the concentrations at equilibrium of all reactants and products except
one, we can calculate the missing concentration.

 Example 13.4.3: Calculation of a Missing Equilibrium Concentration


Nitrogen oxides are air pollutants produced by the reaction of nitrogen and oxygen at high temperatures. At 2000 °C, the value
of the equilibrium constant for the reaction, N (g) + O (g) ⇌ 2 NO(g) , is 4.1 × 10−4. Find the concentration of NO(g) in an
2 2

equilibrium mixture with air at 1 atm pressure at this temperature. In air, [N2] = 0.036 mol/L and [O2] 0.0089 mol/L.

Solution
We are given all of the equilibrium concentrations except that of NO. Thus, we can solve for the missing equilibrium
concentration by rearranging the equation for the equilibrium constant.

Access for free at OpenStax 13.4.4 https://chem.libretexts.org/@go/page/38270


2
[NO]
Kc = Qc =
[ N ][ O ]
2 2

2
[NO] = Kc [ N ][ O ]
2 2

−−−−−−−−−
[NO] = √Kc [ N ][ O ]
2 2

−−−−−−−−−−−−−−−−−−−−−−
−4
= √(4.1 × 10 )(0.036)(0.0089)

−−−−−−−− −
−7
= √1.31 × 10

−4
= 3.6 × 10

Thus [NO] is 3.6 × 10−4 mol/L at equilibrium under these conditions.


We can check our answer by substituting all equilibrium concentrations into the expression for the reaction quotient to see
whether it is equal to the equilibrium constant.
2
[NO]
Qc =
[ N ][ O ]
2 2

−4 2
(3.6 × 10 )
=
(0.036)(0.0089)

−4
= 4.0 × 10 = Kc

The answer checks; our calculated value gives the equilibrium constant within the error associated with the significant figures
in the problem.

 Exercise 13.4.3

The equilibrium constant for the reaction of nitrogen and hydrogen to produce ammonia at a certain temperature is 6.00 × 10−2.
Calculate the equilibrium concentration of ammonia if the equilibrium concentrations of nitrogen and hydrogen are 4.26 M and
2.09 M, respectively.

Answer
1.53 mol/L

13.4.1.2: Calculation of Changes in Concentration


If we know the equilibrium constant for a reaction and a set of concentrations of reactants and products that are not at equilibrium,
we can calculate the changes in concentrations as the system comes to equilibrium, as well as the new concentrations at
equilibrium. The typical procedure can be summarized in four steps.
1. Determine the direction the reaction proceeds to come to equilibrium.
Write a balanced chemical equation for the reaction.
If the direction in which the reaction must proceed to reach equilibrium is not obvious, calculate Qc from the initial
concentrations and compare to Kc to determine the direction of change.
2. Determine the relative changes needed to reach equilibrium, then write the equilibrium concentrations in terms of these
changes.
Define the changes in the initial concentrations that are needed for the reaction to reach equilibrium. Generally, we represent
the smallest change with the symbol x and express the other changes in terms of the smallest change.
Define missing equilibrium concentrations in terms of the initial concentrations and the changes in concentration determined
in (a).
3. Solve for the change and the equilibrium concentrations.
Substitute the equilibrium concentrations into the expression for the equilibrium constant, solve for x, and check any
assumptions used to find x.

Access for free at OpenStax 13.4.5 https://chem.libretexts.org/@go/page/38270


Calculate the equilibrium concentrations.
4. Check the arithmetic.
Check the calculated equilibrium concentrations by substituting them into the equilibrium expression and determining
whether they give the equilibrium constant.
Sometimes a particular step may differ from problem to problem—it may be more complex in some problems and less complex in
others. However, every calculation of equilibrium concentrations from a set of initial concentrations will involve these steps.
In solving equilibrium problems that involve changes in concentration, sometimes it is convenient to set up an ICE table, as
described in the previous section.

 Example 13.4.4: Calculation of Concentration as a Reaction Equilibrates

Under certain conditions, the equilibrium constant for the decomposition of PCl5(g) into PCl3(g) and Cl2(g) is 0.0211. What
are the equilibrium concentrations of PCl5, PCl3, and Cl2 if the initial concentration of PCl5 was 1.00 M?

Solution
Use the stepwise process described earlier.
1. The balanced equation for the decomposition of PCl5 is
PCl (g) ⇌ PCl (g) + Cl (g)
5 3 2

Because we have no products initially, Qc = 0 and the reaction will proceed to the right.
Let us represent the increase in concentration of PCl3 by the symbol x. The other changes may be written in terms of x by
considering the coefficients in the chemical equation.
PCl (g) ⇌ PCl (g) + Cl (g) (13.4.23)
5 3 2

−x x x (13.4.24)

The changes in concentration and the expressions for the equilibrium concentrations are:

This table has two main columns and four rows. The first row for the first column does not have a heading and then has the
following in the first column: Initial concentration ( M ), Change ( M ), Equilibrium concentration ( M ). The second column
has the header, “P C l subscript 5 equilibrium arrow P C l subscript 3 plus C l subscript 2.” Under the second column is a
subgroup of three rows and three columns. The first column has the following: 1.00, negative x, 1.00 minus x. The second
column has the following: 0, positive x, 0 plus x equals x. The third column has the following: 0, positive x, 0 plus x equals x.
Substituting the equilibrium concentrations into the equilibrium constant equation gives
[ PCl ][ Cl ]
3 2
Kc = = 0.0211
[ PCl ]
5

(x)(x)
=
(1.00 − x)

This equation contains only one variable, x, the change in concentration. We can write the equation as a quadratic equation and
solve for x using the quadratic formula.
(x)(x)
0.0211 =
(1.00 − x)

Access for free at OpenStax 13.4.6 https://chem.libretexts.org/@go/page/38270


2
0.0211(1.00 − x) = x
2
x + 0.0211x − 0.0211 = 0

An equation of the form ax2 + bx + c = 0 can be rearranged to solve for x:


−− −−−−−
2
−b ± √b − 4ac
x =
2a

In this case, a = 1, b = 0.0211, and c = −0.0211. Substituting the appropriate values for a, b, and c yields:
−−−−−−−−−−−−−−−−−−−− −
2
−0.0211 ± √(0.0211 ) − 4(1)(−0.0211)
x =
2(1)
−−−−−−−−−−−−−−−−−−−−−−−
−4 −2
−0.0211 ± √(4.45 × 10 ) + (8.44 × 10 )

=
2
−0.0211 ± 0.291
=
2

Hence
−0.0211 + 0.291
x = = 0.135
2

or
−0.0211 − 0.291
x = = −0.156
2

Quadratic equations often have two different solutions, one that is physically possible and one that is physically impossible (an
extraneous root). In this case, the second solution (−0.156) is physically impossible because we know the change must be a
positive number (otherwise we would end up with negative values for concentrations of the products). Thus, x = 0.135 M.
The equilibrium concentrations are
[ PCl ] = 1.00 − 0.135 = 0.87 M
5

[ PCl ] = x = 0.135 M
3

[ Cl ] = x = 0.135 M
2

Substitution into the expression for Kc (to check the calculation) gives
[ PCl ][ Cl ] (0.135)(0.135)
3 2
Kc = = = 0.021
[ PCl ] 0.87
5

The equilibrium constant calculated from the equilibrium concentrations is equal to the value of Kc given in the problem (when
rounded to the proper number of significant figures). Thus, the calculated equilibrium concentrations check.

 Exercise 13.4.4A

Acetic acid, CH3CO2H, reacts with ethanol, C2H5OH, to form water and ethyl acetate, CH3CO2C2H5.

CH CO H + C H OH ⇌ CH CO C H +H O
3 2 2 5 3 2 2 5 2

The equilibrium constant for this reaction with dioxane as a solvent is 4.0. What are the equilibrium concentrations when a
mixture that is 0.15 M in CH3CO2H, 0.15 M in C2H5OH, 0.40 M in CH3CO2C2H5, and 0.40 M in H2O are mixed in enough
dioxane to make 1.0 L of solution?

Answer
[CH3CO2H] = 0.36 M, [C2H5OH] = 0.36 M, [CH3CO2C2H5] = 0.17 M, [H2O] = 0.17 M

 Exercise 13.4.4B

A 1.00-L flask is filled with 1.00 moles of H2 and 2.00 moles of I2. The value of the equilibrium constant for the reaction of
hydrogen and iodine reacting to form hydrogen iodide is 50.5 under the given conditions. What are the equilibrium
concentrations of H2, I2, and HI in moles/L?

Access for free at OpenStax 13.4.7 https://chem.libretexts.org/@go/page/38270


H (g) + I (g) ⇌ 2 HI(g)
2 2

Answer
[H2] = 0.06 M, [I2] = 1.06 M, [HI] = 1.88 M

Sometimes it is possible to use chemical insight to find solutions to equilibrium problems without actually solving a quadratic (or
more complicated) equation. First, however, it is useful to verify that equilibrium can be obtained starting from two extremes: all
(or mostly) reactants and all (or mostly) products.
Consider the ionization of 0.150 M HA, a weak acid.
+ − −4
HA(aq) ⇌ H (aq) + A (aq) Kc = 6.80 × 10

The most obvious way to determine the equilibrium concentrations would be to start with only reactants. This could be called the
“all reactant” starting point. Using x for the amount of acid ionized at equilibrium, this is the ICE table and solution.

This table has two main columns and four rows. The first row for the first column does not have a heading and then has the
following: Initial pressure ( M ), Change ( M ), Equilibrium ( M ). The second column has the header, “H A ( a q ) equilibrium
arrow H superscript plus sign ( a q ) plus A subscript negative sign ( a q ).” Under the second column is a subgroup of three
columns and three rows. The first column has the following: 0.150, negative x, 0.150 minus x. The second column has the
following: 0, x, x. The third column has the following: 0, x, x.
Setting up and solving the quadratic equation gives
+ −
[H ][ A ] (x)(x)
−4
Kc = = = 6.80 × 10
[HA] (0.150 − x)
2 −4 −4
x + 6.80 × 10 x − 1.02 × 10 =0
−−−−−−−−−−−−−−−−−−−−−−−−−−−−−−
−4 −4 2 −4
x = −6.80 × 10 ± √(6.80 × 10 ) − (4)(1)(−1.02 × 10 ) (2)(1)

x = 0.00977 M  or  − 0.0104 M

Using the positive (physical) root, the equilibrium concentrations are


[HA] = 0.150 − x = 0.140 M
+ −
[H ] = [A ] = x = 0.00977 M

A less obvious way to solve the problem would be to assume all the HA ionizes first, then the system comes to equilibrium. This
could be called the “all product” starting point. Assuming all of the HA ionizes gives
[HA] = 0.150 − 0.150 = 0 M
+
[H ] = 0 + 0.150 = 0.150 M

[A ] = 0 + 0.150 = 0.150 M

Using these as initial concentrations and “y” to represent the concentration of HA at equilibrium, this is the ICE table for this
starting point.

Access for free at OpenStax 13.4.8 https://chem.libretexts.org/@go/page/38270


This table has two main columns and four rows. The first row for the first column does not have a heading and then has the
following: Initial pressure ( M ), Change ( M ), Equilibrium ( M ). The second column has the header, “H A ( a q ) equilibrium
arrow H superscript plus sign ( a q ) plus A subscript negative sign ( a q ).” Under the second column is a subgroup of three
columns and three rows. The first column has the following: 0, positive y, y. The second column has the following: 0.150, negative
y, 0.150 minus y. The third column has the following: 0.150, negative y, and 0.150 minus y.
Setting up and solving the quadratic equation gives
+ −
[H ][ A ] (0.150 − y)(0.150 − y)
−4
Kc = = = 6.80 × 10
[HA] (y)
−4 2
6.80 × 10 y = 0.0225 − 0.300y + y

Retain a few extra significant figures to minimize rounding problems.


2
y
− 0.30068y + 0.022500 = 0
−−−−−−−−−−−−−−−−−−−−−− −
2
0.30068 ± √(0.30068 ) − (4)(1)(0.022500)
y =
(2)(1)

0.30068 ± 0.020210
y =
2

Rounding each solution to three significant figures gives


y = 0.160 M  or y = 0.140 M

Using the physically significant root (0.140 M) gives the equilibrium concentrations as
[HA] = y = 0.140 M
+
[H ] = 0.150 − y = 0.010 M

[A ] = 0.150 − y = 0.010 M

Thus, the two approaches give the same results (to three decimal places), and show that both starting points lead to the same
equilibrium conditions. The “all reactant” starting point resulted in a relatively small change (x) because the system was close to
equilibrium, while the “all product” starting point had a relatively large change (y) that was nearly the size of the initial
concentrations. It can be said that a system that starts “close” to equilibrium will require only a ”small” change in conditions (x) to
reach equilibrium.
Recall that a small Kc means that very little of the reactants form products and a large Kc means that most of the reactants form
products. If the system can be arranged so it starts “close” to equilibrium, then if the change (x) is small compared to any initial
concentrations, it can be neglected. Small is usually defined as resulting in an error of less than 5%. The following two examples
demonstrate this.

 Example 13.4.5: Approximate Solution Starting Close to Equilibrium

What are the concentrations at equilibrium of a 0.15 M solution of HCN?


+ − −10
HCN(aq) ⇌ H (aq) + CN (aq) Kc = 4.9 × 10

Solution
Using “x” to represent the concentration of each product at equilibrium gives this ICE table.

Access for free at OpenStax 13.4.9 https://chem.libretexts.org/@go/page/38270


This table has two main columns and four rows. The first row for the first column does not have a heading and then has the
following: Initial pressure ( M ), Change ( M ), Equilibrium ( M ). The second column has the header, “H C N ( a q )
equilibrium arrow H superscript plus sign ( a q ) plus C N subscript negative sign ( a q ).” Under the second column is a
subgroup of three columns and three rows. The first column has the following: 0.15, negative x, 0.15 minus x. The second
column has the following: 0, x, x. The third column has the following: 0, x, x.

The exact solution may be obtained using the quadratic formula with
(x)(x)
Kc =
0.15 − x

solving
2 −10 −11
x + 4.9 × 10 − 7.35 × 10 =0

−6 −6
x = 8.56 × 10 M  (3 sig. figs.) = 8.6 × 10 M  (2 sig. figs.)

Thus [H+] = [CN–] = x = 8.6 × 10–6 M and [HCN] = 0.15 – x = 0.15 M.


In this case, chemical intuition can provide a simpler solution. From the equilibrium constant and the initial conditions, x must
be small compared to 0.15 M. More formally, if x ≪ 0.15, then 0.15 – x ≈ 0.15. If this assumption is true, then it simplifies
obtaining x
2
(x)(x) x
Kc = ≈
0.15 − x 0.15

2
−10
x
4.9 × 10 =
0.15

2 −10 −11
x = (0.15)(4.9 × 10 ) = 7.4 × 10

− −−−−−−− −
−11 −6
x = √ 7.4 × 10 = 8.6 × 10 M

In this example, solving the exact (quadratic) equation and using approximations gave the same result to two significant
figures. While most of the time the approximation is a bit different from the exact solution, as long as the error is less than 5%,
the approximate solution is considered valid. In this problem, the 5% applies to IF (0.15 – x) ≈ 0.15 M, so if
−6
x 8.6 × 10
× 100% = × 100% = 0.006%
0.15 0.15

is less than 5%, as it is in this case, the assumption is valid. The approximate solution is thus a valid solution.

 Exercise 13.4.5

What are the equilibrium concentrations in a 0.25 M NH3 solution?


+ − −5
NH (aq) + H O(l) ⇌ NH (aq) + OH (aq) Kc = 1.8 × 10
3 2 4

Assume that x is much less than 0.25 M and calculate the error in your assumption.

Answer
[ OH
− +
] = [ NH
4
] = 0.0021 M; [NH3] = 0.25 M, error = 0.84%

Access for free at OpenStax 13.4.10 https://chem.libretexts.org/@go/page/38270


The second example requires that the original information be processed a bit, but it still can be solved using a small x
approximation.

 Example 13.4.6: Approximate Solution After Shifting Starting Concentration

Copper(II) ions form a complex ion in the presence of ammonia


2+
[Cu(NH ) ]
2+ 2+ 13 3 4
Cu (aq) + 4 NH (aq) ⇌ Cu(NH ) (aq) Kc = 5.0 × 10 =
3 3 4 4
2+
[ Cu (aq)] [ NH ]
3

If 0.010 mol Cu2+ is added to 1.00 L of a solution that is 1.00 M NH3 what are the concentrations when the system comes to
equilibrium?
Solution The initial concentration of copper(II) is 0.010 M. The equilibrium constant is very large so it would be better to start
with as much product as possible because “all products” is much closer to equilibrium than “all reactants.” Note that Cu2+ is
the limiting reactant; if all 0.010 M of it reacts to form product the concentrations would be
2+
[ Cu ] = 0.010 − 0.010 = 0 M

2+
[Cu(NH ) ] = 0.010 M
3 4

[ NH ] = 1.00 − 4 × 0.010 = 0.96 M


3

Using these “shifted” values as initial concentrations with x as the free copper(II) ion concentration at equilibrium gives this
ICE table.

This table has two main columns and four rows. The first row for the first column does not have a heading and then has the
following: Initial pressure ( M ), Change ( M ), Equilibrium ( M ). The second column has the header, “C u superscript 2
positive sign ( a q ) plus 4 N H subscript 3 ( a q ) equilibrium arrow C u ( N H subscript 3 ) subscript 4 superscript 2 positive
sign ( a q ).” Under the second column is a subgroup of three columns and three rows. The first column has the following: 0,
positive x, x. The second column has the following: 0.96, positive 4 x, 0.96 plus 4 x. The third column has the following:
0.010, negative x, 0.010 minus x.
Since we are starting close to equilibrium, x should be small so that
0.96 + 4x ≈ 0.96 M

0.010 − x ≈ 0.010 M

(0.010 − x) (0.010)
13
Kc = ≈ = 5.0 × 10
4 4
x(0.96 − 4x) x(0.96)

(0.010)
−16
x = = 2.4 × 10 M
4
Kc (0.96 )

Select the smallest concentration for the 5% rule.


−16
2.4 × 10 −12
× 100% = 2 × 10 %
0.010

This is much less than 5%, so the assumptions are valid. The concentrations at equilibrium are
2+ −16
[ Cu ] = x = 2.4 × 10 M

[ NH ] = 0.96 − 4x = 0.96 M
3

Access for free at OpenStax 13.4.11 https://chem.libretexts.org/@go/page/38270


2+
[Cu(NH ) ] = 0.010 − x = 0.010 M
3 4

By starting with the maximum amount of product, this system was near equilibrium and the change (x) was very small. With only a
small change required to get to equilibrium, the equation for x was greatly simplified and gave a valid result well within the 5%
error maximum.

 Exercise 13.4.6

What are the equilibrium concentrations when 0.25 mol Ni2+ is added to 1.00 L of 2.00 M NH3 solution?
2+ 2+
Ni (aq) + 6 NH (aq) ⇌ Ni (NH ) (aq)
3 3 6

with K c = 5.5 × 10
8
.
With such a large equilibrium constant, first form as much product as possible, then assume that only a small amount (x) of the
product shifts left. Calculate the error in your assumption.

Answer
, [NH3] = 0.50 M, [Ni2+] = 2.9 × 10–8 M, error = 1.2 × 10–5%
2+
[Ni (NH ) ] = 0.25 M
3 6

Summary
The ratios of the rate of change in concentrations of a reaction are equal to the ratios of the coefficients in the balanced chemical
equation. The sign of the coefficient of X is positive when the concentration increases and negative when it decreases. We learned
to approach three basic types of equilibrium problems. When given the concentrations of the reactants and products at equilibrium,
we can solve for the equilibrium constant; when given the equilibrium constant and some of the concentrations involved, we can
solve for the missing concentrations; and when given the equilibrium constant and the initial concentrations, we can solve for the
concentrations at equilibrium.

This page titled 13.4: Equilibrium Calculations is shared under a CC BY 4.0 license and was authored, remixed, and/or curated by OpenStax via
source content that was edited to the style and standards of the LibreTexts platform; a detailed edit history is available upon request.

Access for free at OpenStax 13.4.12 https://chem.libretexts.org/@go/page/38270


13.E: Fundamental Equilibrium Concepts (Exercises)
13.E.1: 13.1: Chemical Equilibria Exercises
13.E.1.1: Q13.1.1
What does it mean to describe a reaction as “reversible”?

13.E.1.2: S13.1.1
The reaction can proceed in both the forward and reverse directions.

13.E.1.3: Q13.1.2
When writing an equation, how is a reversible reaction distinguished from a nonreversible reaction?

13.E.1.4: Q13.1.3
If a reaction is reversible, when can it be said to have reached equilibrium?

13.E.1.5: S13.1.3
When a system has reached equilibrium, no further changes in the reactant and product concentrations occur; the reactions continue
to occur, but at equivalent rates.

13.E.1.6: Q13.1.4
Is a system at equilibrium if the rate constants of the forward and reverse reactions are equal?

13.E.1.7: Q13.1.5
If the concentrations of products and reactants are equal, is the system at equilibrium?

13.E.1.8: S13.1.5
The concept of equilibrium does not imply equal concentrations, though it is possible.

13.E.2: 13.2: Equilibrium Constant Exercises


13.E.2.1: Q13.2.1
Explain why there may be an infinite number of values for the reaction quotient of a reaction at a given temperature but there can
be only one value for the equilibrium constant at that temperature.

13.E.2.2: Q13.2.2
Explain why an equilibrium between Br2(l) and Br2(g) would not be established if the container were not a closed vessel shown
below:

13.E.2.3: S13.2.2
Equilibrium cannot be established between the liquid and the gas phase if the top is removed from the bottle because the system is
not closed; one of the components of the equilibrium, the Br2 vapor, would escape from the bottle until all liquid disappeared.
Thus, more liquid would evaporate than can condense back from the gas phase to the liquid phase.

Access for free at OpenStax 13.E.1 https://chem.libretexts.org/@go/page/42802


13.E.2.4: Q13.2.3
If you observe the following reaction at equilibrium, is it possible to tell whether the reaction started with pure NO2 or with pure
N2O4?
\[\ce{2NO2}(g) \rightleftharpoons \ce{N2O4}(g)\]

13.E.2.5: Q13.2.4
Among the solubility rules previously discussed is the statement: All chlorides are soluble except Hg2Cl2, AgCl, PbCl2, and CuCl.

13.E.2.6: Q13.2.5
1. (a) Write the expression for the equilibrium constant for the reaction represented by the equation
AgCl(s) ⇌ Ag (aq) + Cl (aq) . Is Kc > 1, < 1, or ≈ 1? Explain your answer.
+ −

2. (b) Write the expression for the equilibrium constant for the reaction represented by the equation
(aq) + 2 Cl (aq) ⇌ PbCl (s) . Is Kc > 1, < 1, or ≈ 1? Explain your answer.
2+ −
Pb
2

13.E.2.7: S13.2.5
1
(a) Kc = [Ag+][Cl−] < 1. AgCl is insoluble; thus, the concentrations of ions are much less than 1 M; (b) K c = >1
2+ − 2
[ Pb ] [ Cl ]

because PbCl2 is insoluble and formation of the solid will reduce the concentration of ions to a low level (<1 M).

13.E.2.8: Q13.2.6
Among the solubility rules previously discussed is the statement: Carbonates, phosphates, borates, and arsenates—except those of
the ammonium ion and the alkali metals—are insoluble.
a. Write the expression for the equilibrium constant for the reaction represented by the equation
(aq) + CO (aq) . Is Kc > 1, < 1, or ≈ 1? Explain your answer.
2+ −
CaCO (s) ⇌ Ca
3 3

b. Write the expression for the equilibrium constant for the reaction represented by the equation
(aq) ⇌ Ba (PO ) (s) . Is Kc > 1, < 1, or ≈ 1? Explain your answer.
2+ 3−
3 Ba (aq) + 2 PO
4 3 4 2

13.E.2.9: Q13.2.7
Benzene is one of the compounds used as octane enhancers in unleaded gasoline. It is manufactured by the catalytic conversion of
acetylene to benzene: 3 C H (g) ⟶ C H (g) . Which value of Kc would make this reaction most useful commercially? Kc ≈
2 2 6 6

0.01, Kc ≈ 1, or Kc ≈ 10. Explain your answer.

13.E.2.10: S13.2.7
[C H ]
6 6
Since Kc =
3
, a value of Kc ≈ 10 means that C6H6 predominates over C2H2. In such a case, the reaction would be
[C H ]
2 2

commercially feasible if the rate to equilibrium is suitable.

13.E.2.11: Q13.2.8
Show that the complete chemical equation, the total ionic equation, and the net ionic equation for the reaction represented by the
equation KI(aq) + I (aq) ⇌ KI (aq) give the same expression for the reaction quotient. KI3 is composed of the ions K+ and I3−.
2 3

13.E.2.12: Q13.2.9
For a titration to be effective, the reaction must be rapid and the yield of the reaction must essentially be 100%. Is Kc > 1, < 1, or ≈
1 for a titration reaction?

13.E.2.13: S13.2.9
Kc > 1

13.E.2.14: Q13.2.10
For a precipitation reaction to be useful in a gravimetric analysis, the product of the reaction must be insoluble. Is Kc > 1, < 1, or ≈
1 for a useful precipitation reaction?

Access for free at OpenStax 13.E.2 https://chem.libretexts.org/@go/page/42802


13.E.2.15: Q13.2.11
Write the mathematical expression for the reaction quotient, Qc, for each of the following reactions:
a. CH (g) + Cl (g) ⇌ CH Cl(g) + HCl(g)
4 2 3

b. N (g) + O (g) ⇌ 2 NO(g)


2 2

c. 2 SO (g) + O (g) ⇌ 2 SO (g)


2 2 3

d. BaSO (s) ⇌ BaO(s) + SO (g)


3 2

e. P (g) + 5 O (g) ⇌ P O (s)


4 2 4 10

f. Br (g) ⇌ 2 Br(g)
2

g. CH (g) + 2 O (g) ⇌ CO (g) + 2 H O(l)


4 2 2 2

h. CuSO ⋅ 5 H O(s) ⇌ CuSO (s) + 5 H O(g)


4 2 4 2

13.E.2.16: S13.2.11
2 2 2
[ CH Cl][HCl] [NO] [ SO ] 1 [Br]
3 3
(a) Qc = ; (b) Q c = ; (c) Q c =
2
; (d) Q = [SO2]; (e) Q
c c =
5
; (f) Q
c = ; (g)
[ CH ][ Cl ] [ N ][ O ] [ SO ] [ O ] [P ][O ] [ Br ]
4 2 2 2 2
2 2 4 2

[ CO ]
Qc =
2

2
; (h) Q = [H2O]5
c

[ CH ] [ O ]
4 2

13.E.2.17: Q13.2.12
Write the mathematical expression for the reaction quotient, Qc, for each of the following reactions:
a. N (g) + 3 H (g) ⇌ 2 NH (g)
2 2 3

b. 4 NH (g) + 5 O (g) ⇌ 4 NO(g) + 6 H O(g)


3 2 2

c. N O (g) ⇌ 2 NO (g)
2 4 2

d. CO (g) + H (g) ⇌ CO(g) + H O(g)


2 2 2

e. NH Cl(s) ⇌ NH (g) + HCl(g)


4 3

f. 2 Pb(NO ) (s) ⇌ 2 PbO(s) + 4 NO (g) + O


3 2 2 2
(g)

g. 2 H (g) + O (g) ⇌ 2 H O(l)


2 2 2

h. S (g) ⇌ 8 S(g)
8

13.E.2.18: S13.2.12
The initial concentrations or pressures of reactants and products are given for each of the following systems. Calculate the reaction
quotient and determine the direction in which each system will proceed to reach equilibrium.
a. 2 NH (g) ⇌ N (g) + 3 H (g) K = 17 ; [NH3] = 0.20 M, [N2] = 1.00 M, [H2] = 1.00 M
3 2 2
c

b. 2 NH (g) ⇌ N (g) + 3 H (g) K = 6.8 × 10 ; initial pressures: NH3 = 3.0 atm, N2 = 2.0 atm, H2 = 1.0 atm
3 2 2 P
4

c. 2 SO (g) ⇌ 2 SO (g) + O (g) K = 0.230 ; [SO3] = 0.00 M, [SO2] = 1.00 M, [O2] = 1.00 M
3 2 2 c

d. 2 SO (g) ⇌ 2 SO (g) + O (g) K = 16.5 ; initial pressures: SO3 = 1.00 atm, SO2 = 1.00 atm, O2 = 1.00 atm
3 2 2 P

e. 2 NO(g) + Cl (g) ⇌ 2 NOCl(g) K = 4.6 × 10 ; [NO] = 1.00 M, [Cl2] = 1.00 M, [NOCl] = 0 M


2 c
4

f. N (g) + O (g) ⇌ 2 NO(g) K = 0.050 ; initial pressures: NO = 10.0 atm, N2 = O2 = 5 atm


2 2 P

13.E.2.19: S13.2.13
(a) Q 25 proceeds left; (b) QP 0.22 proceeds right; (c) Q undefined proceeds left; (d) QP 1.00 proceeds right; (e) QP 0 proceeds
c c

right; (f) Q 4 proceeds left


c

13.E.2.20: Q13.2.14
The initial concentrations or pressures of reactants and products are given for each of the following systems. Calculate the reaction
quotient and determine the direction in which each system will proceed to reach equilibrium.
a. 2 NH (g) ⇌ N (g) + 3 H (g) K = 17 ; [NH3] = 0.50 M, [N2] = 0.15 M, [H2] = 0.12 M
3 2 2 c

b. 2 NH (g) ⇌ N (g) + 3 H (g) K = 6.8 × 10 ; initial pressures: NH3 = 2.00 atm, N2 = 10.00 atm, H2 = 10.00 atm
3 2 2 P
4

c. 2 SO (g) ⇌ 2 SO (g) + O (g) K = 0.230 ; [SO3] = 2.00 M, [SO2] = 2.00 M, [O2] = 2.00 M
3 2 2 c

d. 2 SO (g) ⇌ 2 SO (g) + O (g) K = 6.5 atm ; initial pressures: SO2 = 1.00 atm, O2 = 1.130 atm, SO3 = 0 atm
3 2 2 P

e. 2 NO(g) + Cl (g) ⇌ 2 NOCl(g) K = 2.5 × 10 ; initial pressures: NO = 1.00 atm, Cl2 = 1.00 atm, NOCl = 0 atm
2 P
3

Access for free at OpenStax 13.E.3 https://chem.libretexts.org/@go/page/42802


f. N2
(g) + O (g) ⇌ 2 NO(g)
2
Kc = 0.050 ; [N2] = 0.100 M, [O2] = 0.200 M, [NO] = 1.00 M

13.E.2.21: Q13.2.15
The following reaction has KP = 4.50 × 10−5 at 720 K.
\[\ce{N2}(g)+\ce{3H2}(g) \rightleftharpoons \ce{2NH3}(g)\]
If a reaction vessel is filled with each gas to the partial pressures listed, in which direction will it shift to reach equilibrium? P(NH3)
= 93 atm, P(N2) = 48 atm, and P(H2) = 52

13.E.2.22: S13.2.15
The system will shift toward the reactants to reach equilibrium.

13.E.2.23: Q13.2.16
Determine if the following system is at equilibrium. If not, in which direction will the system need to shift to reach equilibrium?
\(\ce{SO2Cl2}(g) \rightleftharpoons \ce{SO2}(g)+\ce{Cl2}(g)\)
[SO2Cl2] = 0.12 M, [Cl2] = 0.16 M and [SO2] = 0.050 M. Kc for the reaction is 0.078.

13.E.2.24: Q13.2.17
Which of the systems described in Exercise give homogeneous equilibria? Which give heterogeneous equilibria?

13.E.2.25: S13.2.17
(a) homogenous; (b) homogenous; (c) homogenous; (d) heterogeneous; (e) heterogeneous; (f) homogenous; (g) heterogeneous; (h)
heterogeneous

13.E.2.26: Q13.2.18
Which of the systems described in Exercise give homogeneous equilibria? Which give heterogeneous equilibria?

13.E.2.27: Q13.2.19
For which of the reactions in Exercise does Kc (calculated using concentrations) equal KP (calculated using pressures)?

13.E.2.28: S13.2.19
This situation occurs in (a) and (b).

13.E.2.29: Q13.2.19
For which of the reactions in Exercise does Kc (calculated using concentrations) equal KP (calculated using pressures)?

13.E.2.30: Q13.2.20
Convert the values of Kc to values of KP or the values of KP to values of Kc.
a. N (g) + 3 H (g) ⇌ 2 NH (g) K = 0.50 at 400°C
2 2 3 c

b. H + I ⇌ 2 HI K = 50.2 at 448°C
2 2 c

c. Na SO ⋅ 10 H O(s) ⇌ Na SO (s) + 10 H O(g) K


2 4 2 2 4 2 P = 4.08 × 10
−25
 at 25°C

d. H O(l) ⇌ H O(g) K = 0.122 at 50°C


2 2 P

13.E.2.31: S13.2.20
(a) KP = 1.6 × 10−4; (b) KP = 50.2; (c) Kc = 5.31 × 10−39; (d) Kc = 4.60 × 10−3

13.E.2.32: Q13.2.21
Convert the values of Kc to values of KP or the values of KP to values of Kc.
a. Cl (g) + Br (g) ⇌ 2 BrCl(g) K = 4.7 × 10  at 25°C
2 2 c
−2

b. 2 SO (g) + O (g) ⇌ 2 SO (g) K = 48.2 at 500°C


2 2 3 P

c. CaCl ⋅ 6 H O(s) ⇌ CaCl (s) + 6 H O(g) K = 5.09 × 10


2 2 2 2
P
−44
 at 25°C

d. H O(l) ⇌ H O(g) K = 0.196 at 60°C


2 2
P

Access for free at OpenStax 13.E.4 https://chem.libretexts.org/@go/page/42802


13.E.2.33: Q13.2.22
What is the value of the equilibrium constant expression for the change H 2
O(l) ⇌ H O(g)
2
at 30 °C?

13.E.2.34: S13.2.22
KP = PH O = 0.042. (13.E.1)
2

13.E.2.35: Q13.2.23
Write the expression of the reaction quotient for the ionization of HOCN in water.

13.E.2.36: Q13.2.24
Write the reaction quotient expression for the ionization of NH3 in water.

13.E.2.37: S13.2.24
+ −
[ NH ][ OH ]
4
Qc = (13.E.2)
[ HN ]
3

13.E.2.38: Q13.2.25
What is the approximate value of the equilibrium constant KP for the change C H OC H (l) ⇌ C H OC H (g) at 25 °C.
2 5 2 5 2 5 2 5

(Vapor pressure was described in the previous chapter on liquids and solids; refer back to this chapter to find the relevant
information needed to solve this problem.)

13.E.3: 13.3: Shifting Equilbria Exercises


13.E.3.1: Q13.3.1
The following equation represents a reversible decomposition:
CaCO (s) ⇌ CaO(s) + CO (g)
3 2

Under what conditions will decomposition in a closed container proceed to completion so that no CaCO3 remains?

13.E.3.2: S13.3.1
The amount of CaCO3 must be so small that P is less than KP when the CaCO3 has completely decomposed. In other words, the
CO
2

starting amount of CaCO3 cannot completely generate the full P required for equilibrium.
CO
2

13.E.3.3: Q13.3.2
Explain how to recognize the conditions under which changes in pressure would affect systems at equilibrium.

13.E.3.4: Q13.3.3
What property of a reaction can we use to predict the effect of a change in temperature on the value of an equilibrium constant?

13.E.3.5: S13.3.3
The change in enthalpy may be used. If the reaction is exothermic, the heat produced can be thought of as a product. If the reaction
is endothermic the heat added can be thought of as a reactant. Additional heat would shift an exothermic reaction back to the
reactants but would shift an endothermic reaction to the products. Cooling an exothermic reaction causes the reaction to shift
toward the product side; cooling an endothermic reaction would cause it to shift to the reactants' side.

13.E.3.6: Q13.3.4
What would happen to the color of the solution in part (b) of Figure if a small amount of NaOH were added and Fe(OH)3
precipitated? Explain your answer.

13.E.3.7: Q13.3.5
The following reaction occurs when a burner on a gas stove is lit:
\(\ce{CH4}(g)+\ce{2O2}(g)\rightleftharpoons\ce{CO2}(g)+\ce{2H2O}(g)\)
Is an equilibrium among CH4, O2, CO2, and H2O established under these conditions? Explain your answer.

Access for free at OpenStax 13.E.5 https://chem.libretexts.org/@go/page/42802


13.E.3.8: S13.3.5
No, it is not at equilibrium. Because the system is not confined, products continuously escape from the region of the flame;
reactants are also added continuously from the burner and surrounding atmosphere.

13.E.3.9: Q13.3.6
A necessary step in the manufacture of sulfuric acid is the formation of sulfur trioxide, SO3, from sulfur dioxide, SO2, and oxygen,
O2, shown here. At high temperatures, the rate of formation of SO is higher, but the equilibrium amount (concentration or partial
3

pressure) of SO3 is lower than it would be at lower temperatures.


2 SO (g) + O (g) ⟶ 2 SO (g) (13.E.3)
2 2 3

a. (a) Does the equilibrium constant for the reaction increase, decrease, or remain about the same as the temperature increases?
b. (b) Is the reaction endothermic or exothermic?

13.E.3.10: Q13.3.7a
Suggest four ways in which the concentration of hydrazine, N2H4, could be increased in an equilibrium described by the following
equation:
\[\ce{N2}(g)+\ce{2H2}(g)\rightleftharpoons\ce{N2H4}(g) \hspace{20px} ΔH=\ce{95\:kJ}\]

13.E.3.11: S13.3.7a
Add N2; add H2; decrease the container volume; heat the mixture.

13.E.3.12: Q13.3.7b
Suggest four ways in which the concentration of PH3 could be increased in an equilibrium described by the following equation:
P (g) + 6 H (g) ⇌ 4 PH (g) ΔH = 110.5 kJ (13.E.4)
4 2 3

13.E.3.13: Q13.3.8
How will an increase in temperature affect each of the following equilibria? How will a decrease in the volume of the reaction
vessel affect each?
a. 2 NH (g) ⇌ N (g) + 3 H (g) ΔH = 92 kJ
3 2 2

b. N (g) + O (g) ⇌ 2 NO(g) ΔH = 181 kJ


2 2

c. 2 O (g) ⇌ 3 O (g) ΔH = −285 kJ


3 2

d. CaO(s) + CO (g) ⇌ CaCO (s) ΔH = −176 kJ


2 3

13.E.3.14: S13.3.8
(a) ΔT increase = shift right, ΔP increase = shift left; (b) ΔT increase = shift right, ΔP increase = no effect; (c) ΔT increase = shift
left, ΔP increase = shift left; (d) ΔT increase = shift left, ΔP increase = shift right.

13.E.3.15: Q13.3.9
How will an increase in temperature affect each of the following equilibria? How will a decrease in the volume of the reaction
vessel affect each?
a. 2 H O(g) ⇌ 2 H (g) + O (g) ΔH = 484 kJ
2 2 2

b. N (g) + 3 H (g) ⇌ 2 NH (g) ΔH = −92.2 kJ


2 2 3

c. 2 Br(g) ⇌ Br (g) ΔH = −224 kJ


2

d. H (g) + I (s) ⇌ 2 HI(g) ΔH = 53 kJ


2 2

13.E.3.16: Q13.3.10
Water gas is a 1:1 mixture of carbon monoxide and hydrogen gas and is called water gas because it is formed from steam and hot
carbon in the following reaction:
H O(g) + C(s) ⇌ H (g) + CO(g). (13.E.5)
2 2

Methanol, a liquid fuel that could possibly replace gasoline, can be prepared from water gas and hydrogen at high temperature and
pressure in the presence of a suitable catalyst.

Access for free at OpenStax 13.E.6 https://chem.libretexts.org/@go/page/42802


a. Write the expression for the equilibrium constant (K ) for the reversible reaction
c

2 H (g) + CO(g) ⇌ CH OH(g) ΔH = −90.2 kJ (13.E.6)


2 3

b. What will happen to the concentrations of H , CO, and CH OH at equilibrium if more H2 is added?
2 3

c. What will happen to the concentrations of HH , CO, and CH OH at equilibrium if CO is removed?


2 3

d. What will happen to the concentrations of H , CO, and CH OH at equilibrium if CH3OH is added?
2 3

e. What will happen to the concentrations of HH , CO, and CH OH at equilibrium if the temperature of the system is increased?
2 3

f. What will happen to the concentrations of H , CO, and CH OH at equilibrium if more catalyst is added?
2 3

13.E.3.17: S13.3.10
[ CH OH]
a. Kc =
3

2
;
[ H ] [CO]
2

b. [H2] increases, [CO] decreases, [CH3OH] increases;


c. [H2] increases, [CO] decreases, [CH3OH] decreases;
d. [H2] increases, [CO] increases, [CH3OH] increases;
e. [H2] increases, [CO] increases, [CH3OH] decreases;
f. no changes.

13.E.3.18: Q13.3.11
Nitrogen and oxygen react at high temperatures.
a. Write the expression for the equilibrium constant (Kc) for the reversible reaction
N (g) + O (g) ⇌ 2 NO(g) ΔH = 181 kJ (13.E.7)
2 2

b. What will happen to the concentrations of N2, O2, and NO at equilibrium if more O2 is added?
c. What will happen to the concentrations of N2, O2, and NO at equilibrium if N2 is removed?
d. What will happen to the concentrations of N2, O2, and NO at equilibrium if NO is added?
e. What will happen to the concentrations of N2, O2, and NO at equilibrium if the pressure on the system is increased by reducing
the volume of the reaction vessel?
f. What will happen to the concentrations of N2, O2, and NO at equilibrium if the temperature of the system is increased?
g. What will happen to the concentrations of N2, O2, and NO at equilibrium if a catalyst is added?

13.E.3.19: Q13.3.12
Water gas, a mixture of H2 and CO, is an important industrial fuel produced by the reaction of steam with red hot coke, essentially
pure carbon.
a. Write the expression for the equilibrium constant for the reversible reaction
C(s) + H O(g) ⇌ CO(g) + H (g) ΔH = 131.30 kJ (13.E.8)
2 2

b. What will happen to the concentration of each reactant and product at equilibrium if more C is added?
c. What will happen to the concentration of each reactant and product at equilibrium if H2O is removed?
d. What will happen to the concentration of each reactant and product at equilibrium if CO is added?
e. What will happen to the concentration of each reactant and product at equilibrium if the temperature of the system is increased?

13.E.3.20: S13.3.12
[CO][ H ]
2
(a) K c = ; (b) [H2O] no change, [CO] no change, [H2] no change; (c) [H2O] decreases, [CO] decreases, [H2] decreases;
[ H O]
2

(d) [H2O] increases, [CO] increases, [H2] decreases; (f) [H2O] decreases, [CO] increases, [H2] increases. In (b), (c), (d), and (e), the
mass of carbon will change, but its concentration (activity) will not change.

13.E.3.21: Q13.3.13
Pure iron metal can be produced by the reduction of iron(III) oxide with hydrogen gas.
a. Write the expression for the equilibrium constant (Kc) for the reversible reaction

Access for free at OpenStax 13.E.7 https://chem.libretexts.org/@go/page/42802


Fe O (s) + 3 H (g) ⇌ 2 Fe(s) + 3 H O(g) ΔH = 98.7 kJ (13.E.9)
2 3 2 2

b. What will happen to the concentration of each reactant and product at equilibrium if more Fe is added?
c. What will happen to the concentration of each reactant and product at equilibrium if H2O is removed?
d. What will happen to the concentration of each reactant and product at equilibrium if H2 is added?
e. What will happen to the concentration of each reactant and product at equilibrium if the pressure on the system is increased by
reducing the volume of the reaction vessel?
f. What will happen to the concentration of each reactant and product at equilibrium if the temperature of the system is increased?

13.E.3.22: Q13.3.14
Ammonia is a weak base that reacts with water according to this equation:
\[\ce{NH3}(aq)+\ce{H2O}(l)\rightleftharpoons\ce{NH4+}(aq)+\ce{OH-}(aq)\]
Will any of the following increase the percent of ammonia that is converted to the ammonium ion in water and why?
a. Addition of NaOH
b. Addition of HCl
c. Addition of NH4Cl

13.E.3.23: S13.3.14
Only (b)

13.E.3.24: Q13.3.15
Acetic acid is a weak acid that reacts with water according to this equation:
\[\ce{CH3CO2H}(aq)+\ce{H2O}(aq)\rightleftharpoons\ce{H3O+}(aq)+\ce{CH3CO2-}(aq)\]
Will any of the following increase the percent of acetic acid that reacts and produces CH 3

CO
2
ion?
a. Addition of HCl
b. Addition of NaOH
c. Addition of NaCH3CO2

13.E.3.25: Q13.3.16
Suggest two ways in which the equilibrium concentration of Ag+ can be reduced in a solution of Na+, Cl−, Ag+, and NO3

, in
contact with solid AgCl.
\(\ce{Na+}(aq)+\ce{Cl-}(aq)+\ce{Ag+}(aq)+\ce{NO3-}(aq)\rightleftharpoons\ce{AgCl}(s)+\ce{Na+}(aq)+\ce{NO3-}(aq)\)
ΔH = −65.9 kJ

13.E.3.26: S13.3.16
Add NaCl or some other salt that produces Cl− to the solution. Cooling the solution forces the equilibrium to the right, precipitating
more AgCl(s).

13.E.3.27: Q13.3.17
How can the pressure of water vapor be increased in the following equilibrium?
\[\ce{H2O}(l)\rightleftharpoons\ce{H2O}(g) \hspace{20px} ΔH=\ce{41\:kJ}\]

13.E.3.28: Q13.3.18
Additional solid silver sulfate, a slightly soluble solid, is added to a solution of silver ion and sulfate ion at equilibrium with solid
silver sulfate.
\[\ce{2Ag+}(aq)+\ce{SO4^2-}(aq)\rightleftharpoons\ce{Ag2SO4}(s)\]
Which of the following will occur?
a. Ag+ or SO concentrations will not change.
2−

b. The added silver sulfate will dissolve.

Access for free at OpenStax 13.E.8 https://chem.libretexts.org/@go/page/42802


c. Additional silver sulfate will form and precipitate from solution as Ag+ ions and SO ions combine.
2−

d. The Ag+ ion concentration will increase and the SO ion concentration will decrease.
2−
4

13.E.3.29: S13.3.18
(a)

13.E.3.30: Q13.3.19
The amino acid alanine has two isomers, α-alanine and β-alanine. When equal masses of these two compounds are dissolved in
equal amounts of a solvent, the solution of α-alanine freezes at the lowest temperature. Which form, α-alanine or β-alanine, has the
larger equilibrium constant for ionization (HX ⇌ H + X ) ? + −

13.E.4: 13.4: Equilibrium Calculations Exercises


13.E.4.1: Q13.4.1
A reaction is represented by this equation: A(aq) + 2 B(aq) ⇌ 2 C(aq) 3
Kc = 1 × 10

a. Write the mathematical expression for the equilibrium constant.


b. Using concentrations ≤1 M, make up two sets of concentrations that describe a mixture of A, B, and C at equilibrium.

13.E.4.2: S13.4.1
2
[C]
Kc =
2
. [A] = 0.1 M, [B] = 0.1 M, [C] = 1 M; and [A] = 0.01, [B] = 0.250, [C] = 0.791.
[A][B]

13.E.4.3: Q13.4.2
A reaction is represented by this equation: 2 W(aq) ⇌ X(aq) + 2 Y(aq) Kc = 5 × 10
−4

a. Write the mathematical expression for the equilibrium constant.


b. Using concentrations of ≤1 M, make up two sets of concentrations that describe a mixture of W, X, and Y at equilibrium.

13.E.4.4: Q13.4.3
What is the value of the equilibrium constant at 500 °C for the formation of NH3 according to the following equation?
N (g) + 3 H (g) ⇌ 2 NH (g) (13.E.10)
2 2 3

An equilibrium mixture of NH3(g), H2(g), and N2(g) at 500 °C was found to contain 1.35 M H2, 1.15 M N2, and 4.12 × 10−1 M
NH3.

13.E.4.5: S13.4.3
Kc = 6.00 × 10−2

13.E.4.6: Q13.4.4
Hydrogen is prepared commercially by the reaction of methane and water vapor at elevated temperatures.

CH (g) + H O(g) ⇌ 3 H (g) + CO(g) (13.E.11)


4 2 2

What is the equilibrium constant for the reaction if a mixture at equilibrium contains gases with the following concentrations: CH4,
0.126 M; H2O, 0.242 M; CO, 0.126 M; H2 1.15 M, at a temperature of 760 °C?
A 0.72-mol sample of PCl5 is put into a 1.00-L vessel and heated. At equilibrium, the vessel contains 0.40 mol of PCl3(g) and 0.40
mol of Cl2(g). Calculate the value of the equilibrium constant for the decomposition of PCl5 to PCl3 and Cl2 at this temperature.

13.E.4.7: S13.4.4
Kc = 0.50

13.E.4.8: Q13.4.5
At 1 atm and 25 °C, NO2 with an initial concentration of 1.00 M is 3.3 × 10−3% decomposed into NO and O2. Calculate the value
of the equilibrium constant for the reaction.

Access for free at OpenStax 13.E.9 https://chem.libretexts.org/@go/page/42802


2 NO (g) ⇌ 2 NO(g) + O (g) (13.E.12)
2 2

13.E.4.9: Q13.4.6
Calculate the value of the equilibrium constant KP for the reaction 2 NO(g) + Cl (g) ⇌ 2 NOCl(g)
2
from these equilibrium
pressures: NO, 0.050 atm; Cl2, 0.30 atm; NOCl, 1.2 atm.

13.E.4.10: S13.4.6
The equilibrium equation is KP = 1.9 × 103

13.E.4.11: Q13.4.7
When heated, iodine vapor dissociates according to this equation:
I (g) ⇌ 2 I(g) (13.E.13)
2

At 1274 K, a sample exhibits a partial pressure of I2 of 0.1122 atm and a partial pressure due to I atoms of 0.1378 atm. Determine
the value of the equilibrium constant, KP, for the decomposition at 1274 K.

13.E.4.12: Q13.4.8
A sample of ammonium chloride was heated in a closed container.

NH Cl(s) ⇌ NH (g) + HCl(g) (13.E.14)


4 3

At equilibrium, the pressure of NH3(g) was found to be 1.75 atm. What is the value of the equilibrium constant KP for the
decomposition at this temperature?

13.E.4.13: S13.4.8
KP = 3.06

13.E.4.14: Q13.4.9
At a temperature of 60 °C, the vapor pressure of water is 0.196 atm. What is the value of the equilibrium constant KP for the
transformation at 60 °C?
H O(l) ⇌ H O(g) (13.E.15)
2 2

13.E.4.15: Q13.4.10
Complete the changes in concentrations (or pressure, if requested) for each of the following reactions.
(a)
2 SO (g) ⇌ 2 SO (g) + O (g) (13.E.16)
3 2 2

+x (13.E.17)
––––– –––––
0.125 M (13.E.18)
––––– –––––

(b)
4 NH (g) + 3 O (g) ⇌ 2 N (g) + 6 H O(g) (13.E.19)
3 2 2 2

3x (13.E.20)
––––– ––––– –––––
0.24 M (13.E.21)
––––– ––––– –––––

(c) Change in pressure:


2 CH (g) ⇌ C H (g) + 3 H (g) (13.E.22)
4 2 2 2

x (13.E.23)
––––– –––––
25 torr (13.E.24)
––––– –––––

(d) Change in pressure:


CH (g) + H O(g) ⇌ CO(g) + 3 H (g) (13.E.25)
4 2 2

x (13.E.26)
––––– ––––– –––––
5 atm (13.E.27)
––––– ––––– –––––

Access for free at OpenStax 13.E.10 https://chem.libretexts.org/@go/page/42802


(e)
NH Cl(s) ⇌ NH (g) + HCl(g) (13.E.28)
4 3

x (13.E.29)
–––––
−4
1.03 × 10 M (13.E.30)
–––––

(f) change in pressure:


Ni(s) + 4 CO(g) ⇌ Ni (CO) (g) (13.E.31)
4

4x (13.E.32)
–––––
0.40 atm (13.E.33)
–––––

13.E.4.16: S13.4.10
a. −2x, 2x, −0.250 M, 0.250 M;
b. 4x, −2x, −6x, 0.32 M, −0.16 M, −0.48 M;
c. −2x, 3x, −50 torr, 75 torr;
d. x, − x, −3x, 5 atm, −5 atm, −15 atm;
e. x, 1.03 × 10−4 M; (f) x, 0.1 atm.

13.E.4.17: Q13.4.11
Complete the changes in concentrations (or pressure, if requested) for each of the following reactions.
(a)
2 H (g) + O (g) ⇌ 2 H O(g) (13.E.34)
2 2 2

+ 2x (13.E.35)
––––– –––––
1.50 M (13.E.36)
––––– –––––

(b)
CS (g) + 4 H (g) ⇌ CH (g) + 2 H S(g) (13.E.37)
2 2 4 2

x (13.E.38)
––––– ––––– –––––
0.020 M (13.E.39)
––––– ––––– –––––

(c) Change in pressure:


H (g) + Cl (g) ⇌ 2 HCl(g) (13.E.40)
2 2

x (13.E.41)
––––– –––––
1.50 atm (13.E.42)
––––– –––––

(d) Change in pressure:


2 NH (g) + 2 O (g) ⇌ N O(g) + 3 H O(g) (13.E.43)
3 2 2 2

x (13.E.44)
––––– ––––– –––––
60.6 torr (13.E.45)
––––– ––––– –––––

(e)
NH HS(s) ⇌ NH (g) + H S(g) (13.E.46)
4 3 2

x (13.E.47)
–––––
−6
9.8 × 10 M (13.E.48)
–––––

(f) Change in pressure:


Fe(s) + 5 CO(g) ⇌ Fe(CO) (g) (13.E.49)
4

x (13.E.50)
–––––
0.012 atm (13.E.51)
–––––

13.E.4.18: Q13.4.12
Why are there no changes specified for Ni in Exercise, part (f)? What property of Ni does change?

Access for free at OpenStax 13.E.11 https://chem.libretexts.org/@go/page/42802


13.E.4.19: S13.4.12
Activities of pure crystalline solids equal 1 and are constant; however, the mass of Ni does change.

13.E.4.20: Q13.4.13
Why are there no changes specified for NH4HS in Exercise, part (e)? What property of NH4HS does change?

13.E.4.21: Q13.4.14
Analysis of the gases in a sealed reaction vessel containing NH3, N2, and H2 at equilibrium at 400 °C established the concentration
of N2 to be 1.2 M and the concentration of H2 to be 0.24 M.
N (g) + 3 H (g) ⇌ 2 NH (g) Kc = 0.50 at 400 °C (13.E.52)
2 2 3

Calculate the equilibrium molar concentration of NH3.

13.E.4.22: S13.4.14
[NH3] = 9.1 × 10−2 M

13.E.4.23: Q13.4.16
Calculate the number of moles of HI that are at equilibrium with 1.25 mol of H2 and 1.25 mol of I2 in a 5.00−L flask at 448 °C.
H +I ⇌ 2 HI Kc = 50.2 at 448 °C
2 2

13.E.4.24: Q13.4.17
What is the pressure of BrCl in an equilibrium mixture of Cl2, Br2, and BrCl if the pressure of Cl2 in the mixture is 0.115 atm and
the pressure of Br2 in the mixture is 0.450 atm?
−2
Cl (g) + Br (g) ⇌ 2 BrCl(g) KP = 4.7 × 10 (13.E.53)
2 2

13.E.4.25: S13.4.17
PBrCl = 4.9 × 10−2 atm

13.E.4.26: Q13.4.18
What is the pressure of CO2 in a mixture at equilibrium that contains 0.50 atm H2, 2.0 atm of H2O, and 1.0 atm of CO at 990 °C?
H (g) + CO (g) ⇌ H O(g) + CO(g) KP = 1.6 at 990 °C (13.E.54)
2 2 2

13.E.4.27: Q13.4.12
Cobalt metal can be prepared by reducing cobalt(II) oxide with carbon monoxide.
2
CoO(s) + CO(g) ⇌ Co(s) + CO (g) Kc = 4.90 × 10  at 550 °C
2

What concentration of CO remains in an equilibrium mixture with [CO2] = 0.100 M?

13.E.4.28: S13.4.12
[CO] = 2.0 × 10−4 M

13.E.4.29: Q13.4.13
Carbon reacts with water vapor at elevated temperatures.
C(s) + H O(g) ⇌ CO(g) + H (g) Kc = 0.2 at 1000 °C
2 2

What is the concentration of CO in an equilibrium mixture with [H2O] = 0.500 M at 1000 °C?

13.E.4.30: Q13.4.14
Sodium sulfate 10−hydrate, Na 2
SO
4
⋅ 10 H O
2
, dehydrates according to the equation
Na SO ⋅ 10 H O(s) ⇌ Na SO (s) + 10 H O(g) (13.E.55)
2 4 2 2 4 2

with Kp = 4.08 × 10
−25
at 25°C. What is the pressure of water vapor at equilibrium with a mixture of Na SO
2 4
⋅ 10 H O
2
and
NaSO
4
?

Access for free at OpenStax 13.E.12 https://chem.libretexts.org/@go/page/42802


13.E.4.31: S13.4.14
−3
PH O = 3.64 × 10 atm
2

13.E.4.32: Q13.4.15
Calcium chloride 6−hydrate, CaCl2·6H2O, dehydrates according to the equation
−44
CaCl ⋅ 6 H O(s) ⇌ CaCl (s) + 6 H O(g) KP = 5.09 × 10  at 25 °C
2 2 2 2

What is the pressure of water vapor at equilibrium with a mixture of CaCl2·6H2O and CaCl2?

13.E.4.33: Q13.4.16
A student solved the following problem and found the equilibrium concentrations to be [SO2] = 0.590 M, [O2] = 0.0450 M, and
[SO3] = 0.260 M. How could this student check the work without reworking the problem? The problem was: For the following
reaction at 600 °C:
2 SO (g) + O (g) ⇌ 2 SO (g) Kc = 4.32
2 2 3

What are the equilibrium concentrations of all species in a mixture that was prepared with [SO3] = 0.500 M, [SO2] = 0 M, and [O2]
= 0.350 M?

13.E.4.34: S13.4.16
Calculate Q based on the calculated concentrations and see if it is equal to Kc. Because Q does equal 4.32, the system must be at
equilibrium.

13.E.4.35: Q13.4.16
A student solved the following problem and found [N2O4] = 0.16 M at equilibrium. How could this student recognize that the
answer was wrong without reworking the problem? The problem was: What is the equilibrium concentration of N2O4 in a mixture
formed from a sample of NO2 with a concentration of 0.10 M?

2 NO (g) ⇌ N O (g) Kc = 160 (13.E.56)


2 2 4

Assume that the change in concentration of N2O4 is small enough to be neglected in the following problem.
(a) Calculate the equilibrium concentration of both species in 1.00 L of a solution prepared from 0.129 mol of N2O4 with
chloroform as the solvent.
N O (g) ⇌ 2 NO (g)
2 4 2
Kc = 1.07 × 10
−5
in chloroform
(b) Show that the change is small enough to be neglected.

13.E.4.36: S13.4.16
(a)
[NO2] = 1.17 × 10−3 M
[N2O4] = 0.128 M
−4
5.87 × 10
(b) Percent error = × 100% = 0.455% . The change in concentration of N2O4 is far less than the 5% maximum
0.129
allowed.

13.E.4.37: Q13.4.17
Assume that the change in concentration of COCl2 is small enough to be neglected in the following problem.
a. Calculate the equilibrium concentration of all species in an equilibrium mixture that results from the decomposition of COCl2
with an initial concentration of 0.3166 M.
−10
COCl (g) ⇌ CO(g) + Cl (g) Kc = 2.2 × 10 (13.E.57)
2 2

b. Show that the change is small enough to be neglected.

Access for free at OpenStax 13.E.13 https://chem.libretexts.org/@go/page/42802


13.E.4.38: Q13.4.18
Assume that the change in pressure of H2S is small enough to be neglected in the following problem.
(a) Calculate the equilibrium pressures of all species in an equilibrium mixture that results from the decomposition of H2S with an
initial pressure of 0.824 atm.
−6
2 H S(g) ⇌ 2 H (g) + S (g) KP = 2.2 × 10
2 2 2

(b) Show that the change is small enough to be neglected.

13.E.4.39: S13.4.18
(a)
[H2S] = 0.810 atm
[H2] = 0.014 atm
S2] = 0.0072 atm
(b) The 2x is dropped from the equilibrium calculation because 0.014 is negligible when subtracted from 0.824. The percent error
0.014
associated with ignoring 2x is × 100% = 1.7% , which is less than allowed by the “5% test.” The error is, indeed,
0.824
negligible.

13.E.4.40: Q13.4.19
What are all concentrations after a mixture that contains [H2O] = 1.00 M and [Cl2O] = 1.00 M comes to equilibrium at 25 °C?
H O(g) + Cl O(g) ⇌ 2 HOCl(g) Kc = 0.0900 (13.E.58)
2 2

13.E.4.41: Q13.4.20
What are the concentrations of PCl5, PCl3, and Cl2 in an equilibrium mixture produced by the decomposition of a sample of pure
PCl5 with [PCl5] = 2.00 M?
PCl (g) ⇌ PCl (g) + Cl (g) Kc = 0.0211 (13.E.59)
5 3 2

13.E.4.42: S13.4.20
[PCl3] = 1.80 M; [PC3] = 0.195 M; [PCl3] = 0.195 M.

13.E.4.43: Q13.4.21
Calculate the pressures of all species at equilibrium in a mixture of NOCl, NO, and Cl2 produced when a sample of NOCl with a
pressure of 10.0 atm comes to equilibrium according to this reaction:
−4
2 NOCl(g) ⇌ 2 NO(g) + Cl (g) KP = 4.0 × 10 (13.E.60)
2

13.E.4.44: Q13.4.22
Calculate the equilibrium concentrations of NO, O2, and NO2 in a mixture at 250 °C that results from the reaction of 0.20 M NO
and 0.10 M O2. (Hint: K is large; assume the reaction goes to completion then comes back to equilibrium.)
5
2 NO(g) + O (g) ⇌ 2 NO (g) Kc = 2.3 × 10  at 250 °C (13.E.61)
2 2

13.E.4.45: S13.4.22
[NO2] = 0.19 M
[NO] = 0.0070 M
[O2] = 0.0035 M

13.E.4.46: Q13.4.23
Calculate the equilibrium concentrations that result when 0.25 M O2 and 1.0 M HCl react and come to equilibrium.
13
4 HCl(g) + O (g) ⇌ 2 Cl (g) + 2 H O(g) Kc = 3.1 × 10 (13.E.62)
2 2 2

Access for free at OpenStax 13.E.14 https://chem.libretexts.org/@go/page/42802


13.E.4.47: Q13.4.24
One of the important reactions in the formation of smog is represented by the equation
34
O (g) + NO(g) ⇌ NO (g) + O (g) KP = 6.0 × 10 (13.E.63)
3 2 2

What is the pressure of O3 remaining after a mixture of O3 with a pressure of 1.2 × 10−8 atm and NO with a pressure of 1.2 × 10−8
atm comes to equilibrium? (Hint: KP is large; assume the reaction goes to completion then comes back to equilibrium.)

13.E.4.48: S13.4.24
−26
PO = 4.9 × 10 atm
3

13.E.4.49: Q13.4.24
Calculate the pressures of NO, Cl2, and NOCl in an equilibrium mixture produced by the reaction of a starting mixture with 4.0 atm
NO and 2.0 atm Cl2. (Hint: KP is small; assume the reverse reaction goes to completion then comes back to equilibrium.)
3
2 NO(g) + Cl (g) ⇌ 2 NOCl(g) KP = 2.5 × 10
2

13.E.4.50: Q13.4.25
Calculate the number of grams of HI that are at equilibrium with 1.25 mol of H2 and 63.5 g of iodine at 448 °C.
H +I ⇌ 2 HI Kc = 50.2 at 448 °C
2 2

13.E.4.51: S13.4.25
507 g

13.E.4.52: Q13.4.26
Butane exists as two isomers, n−butane and isobutane.

KP = 2.5 at 25 °C
What is the pressure of isobutane in a container of the two isomers at equilibrium with a total pressure of 1.22 atm?

13.E.4.53: Q13.4.27
What is the minimum mass of CaCO3 required to establish equilibrium at a certain temperature in a 6.50-L container if the
equilibrium constant (Kc) is 0.050 for the decomposition reaction of CaCO3 at that temperature?
CaCO (s) ⇌ CaO(s) + CO (g)
3 2

13.E.4.54: S13.4.27
330 g

13.E.4.55: Q13.4.28
The equilibrium constant (Kc) for this reaction is 1.60 at 990 °C:
H (g) + CO (g) ⇌ H O(g) + CO(g) (13.E.64)
2 2 2

Calculate the number of moles of each component in the final equilibrium mixture obtained from adding 1.00 mol of H2, 2.00 mol
of CO2, 0.750 mol of H2O, and 1.00 mol of CO to a 5.00-L container at 990 °C.

13.E.4.56: Q13.4.29
At 25 °C and at 1 atm, the partial pressures in an equilibrium mixture of N2O4 and NO2 are PN
2
O
4
= 0.70 atm and
PNO = 0.30 atm.
2

Access for free at OpenStax 13.E.15 https://chem.libretexts.org/@go/page/42802


a. Predict how the pressures of NO2 and N2O4 will change if the total pressure increases to 9.0 atm. Will they increase, decrease,
or remain the same?
b. Calculate the partial pressures of NO2 and N2O4 when they are at equilibrium at 9.0 atm and 25 °C.

13.E.4.57: S13.4.29
(a) Both gases must increase in pressure.
(b) P N O
2 4
= 8.0 atm and PNO
2
= 1.0 atm

13.E.4.58: Q13.4.30
In a 3.0-L vessel, the following equilibrium partial pressures are measured: N2, 190 torr; H2, 317 torr; NH3, 1.00 × 103 torr.
N (g) + 3 H (g) ⇌ 2 NH (g) (13.E.65)
2 2 3

a. How will the partial pressures of H2, N2, and NH3 change if H2 is removed from the system? Will they increase, decrease, or
remain the same?
b. Hydrogen is removed from the vessel until the partial pressure of nitrogen, at equilibrium, is 250 torr. Calculate the partial
pressures of the other substances under the new conditions.

13.E.4.59: Q13.4.31
The equilibrium constant (Kc) for this reaction is 5.0 at a given temperature.
\[\ce{CO}(g)+\ce{H2O}(g) <=> \ce{CO2}(g)+\ce{H2}(g)\)]
a. On analysis, an equilibrium mixture of the substances present at the given temperature was found to contain 0.20 mol of CO,
0.30 mol of water vapor, and 0.90 mol of H2 in a liter. How many moles of CO2 were there in the equilibrium mixture?
b. Maintaining the same temperature, additional H2 was added to the system, and some water vapor was removed by drying. A
new equilibrium mixture was thereby established containing 0.40 mol of CO, 0.30 mol of water vapor, and 1.2 mol of H2 in a
liter. How many moles of CO2 were in the new equilibrium mixture? Compare this with the quantity in part (a), and discuss
whether the second value is reasonable. Explain how it is possible for the water vapor concentration to be the same in the two
equilibrium solutions even though some vapor was removed before the second equilibrium was established.

13.E.4.60: S13.4.31
(a) 0.33 mol.
(b) [CO]2 = 0.50 M Added H2 forms some water to compensate for the removal of water vapor and as a result of a shift to the left
after H2 is added.

13.E.4.61: Q13.4.32a
Antimony pentachloride decomposes according to this equation:
SbCl (g) ⇌ SbCl (g) + Cl (g)
5 3 2

An equilibrium mixture in a 5.00-L flask at 448 °C contains 3.85 g of SbCl5, 9.14 g of SbCl3, and 2.84 g of Cl2. How many grams
of each will be found if the mixture is transferred into a 2.00-L flask at the same temperature?

13.E.4.62: Q13.4.32b
Consider the reaction between H2 and O2 at 1000 K
\[\ce{2H2}(g)+\ce{O2}(g)⇌\ce{2H2O}(g) \hspace{20px} K_P=\dfrac{(P_{\ce{H2O}})^2}{(P_{\ce{O2}})
(P_{\ce{H2}})^3}=1.33×10^{20}\]
If 0.500 atm of H2 and 0.500 atm of O2 are allowed to come to equilibrium at this temperature, what are the partial pressures of the
components?

13.E.4.63: S13.4.32b
−11
PH = 8.64 × 10 atm
2

\(P_{\ce{O2}}=0.250\:\ce{atm}\)

Access for free at OpenStax 13.E.16 https://chem.libretexts.org/@go/page/42802


\(P_{\ce{H2O}}=0.500\:\ce{atm}\)

13.E.4.64: Q13.4.33
An equilibrium is established according to the following equation
2+ + 2+
Hg (aq) + NO − (aq) + 3 H (aq) ⇌ 2 Hg (aq) + HNO (aq) + H O(l) Kc = 4.6 (13.E.66)
2 3 2 2

What will happen in a solution that is 0.20 M each in Hg 2+


2
, NO 3
, H+, Hg2+, and HNO2?

a. Hg will be oxidized and NO − reduced.


2+
2 3

b. Hg will be reduced and NO − oxidized.


2+
2 3

c. Hg2+ will be oxidized and HNO2 reduced.


d. Hg2+ will be reduced and HNO2 oxidized.
e. There will be no change because all reactants and products have an activity of 1.

13.E.4.65: Q13.4.34
Consider the equilibrium

4 NO (g) + 6 H O(g) ⇌ 4 NH (g) + 7 O (g) (13.E.67)


2 2 3 2

a. What is the expression for the equilibrium constant (Kc) of the reaction?
b. How must the concentration of NH3 change to reach equilibrium if the reaction quotient is less than the equilibrium constant?
c. If the reaction were at equilibrium, how would a decrease in pressure (from an increase in the volume of the reaction vessel)
affect the pressure of NO2?
d. If the change in the pressure of NO2 is 28 torr as a mixture of the four gases reaches equilibrium, how much will the pressure of
O2 change?

13.E.4.66: S13.4.34
4 7
[ NH ] [ O ]
3 2
(a) Kc =
4 6
. (b) [NH3] must increase for Qc to reach Kc. (c) That decrease in pressure would decrease [NO2]. (d)
[ NO ] [ H O]
2 2

PO = 49 torr
2

13.E.4.67: Q13.4.35
The binding of oxygen by hemoglobin (Hb), giving oxyhemoglobin (HbO2), is partially regulated by the concentration of H3O+
and dissolved CO2 in the blood. Although the equilibrium is complicated, it can be summarized as
+ +
HbO (aq) + H O (aq) + CO (g) ⇌ CO − Hb − H + O (g) + H O(l)
2 3 2 2 2 2

1. (a) Write the equilibrium constant expression for this reaction.


2. (b) Explain why the production of lactic acid and CO2 in a muscle during exertion stimulates release of O2 from the
oxyhemoglobin in the blood passing through the muscle.

13.E.4.68: Q13.4.36
The hydrolysis of the sugar sucrose to the sugars glucose and fructose follows a first-order rate equation for the disappearance of
sucrose.
C H O (aq) + H O(l) ⟶ C H O (aq) + C H O (aq)
12 22 11 2 6 12 6 6 12 6

Rate = k[C12H22O11]
In neutral solution, k = 2.1 × 10−11/s at 27 °C. (As indicated by the rate constant, this is a very slow reaction. In the human body,
the rate of this reaction is sped up by a type of catalyst called an enzyme.) (Note: That is not a mistake in the equation—the
products of the reaction, glucose and fructose, have the same molecular formulas, C6H12O6, but differ in the arrangement of the
atoms in their molecules). The equilibrium constant for the reaction is 1.36 × 105 at 27 °C. What are the concentrations of glucose,
fructose, and sucrose after a 0.150 M aqueous solution of sucrose has reached equilibrium? Remember that the activity of a solvent
(the effective concentration) is 1.

Access for free at OpenStax 13.E.17 https://chem.libretexts.org/@go/page/42802


13.E.4.69: S13.4.36
[fructose] = 0.15 M

13.E.4.70: Q13.4.37
The density of trifluoroacetic acid vapor was determined at 118.1 °C and 468.5 torr, and found to be 2.784 g/L. Calculate Kc for the
association of the acid.

Liquid N2O3 is dark blue at low temperatures, but the color fades and becomes greenish at higher temperatures as the compound
decomposes to NO and NO2. At 25 °C, a value of KP = 1.91 has been established for this decomposition. If 0.236 moles of N2O3
are placed in a 1.52-L vessel at 25 °C, calculate the equilibrium partial pressures of N2O3(g), NO2(g), and NO(g).

13.E.4.71: S13.4.37
PN O = 1.90 atm and PNO = PNO = 1.90 atm
2 3 2

13.E.4.72: Q13.4.38
A 1.00-L vessel at 400 °C contains the following equilibrium concentrations: N2, 1.00 M; H2, 0.50 M; and NH3, 0.25 M. How
many moles of hydrogen must be removed from the vessel to increase the concentration of nitrogen to 1.1 M?

13.E.4.73: Q13.4.39
A 0.010 M solution of the weak acid HA has an osmotic pressure (see chapter on solutions and colloids) of 0.293 atm at 25 °C. A
0.010 M solution of the weak acid HB has an osmotic pressure of 0.345 atm under the same conditions.
(a) Which acid has the larger equilibrium constant for ionization
HA [HA(aq) ⇌ A −
(aq) + H
+
(aq)] or HB [HB(aq) ⇌ H +
(aq) + B

(aq)] ?
(b) What are the equilibrium constants for the ionization of these acids?
(Hint: Remember that each solution contains three dissolved species: the weak acid (HA or HB), the conjugate base (A− or B−), and
the hydrogen ion (H+). Remember that osmotic pressure (like all colligative properties) is related to the total number of solute
particles. Specifically for osmotic pressure, those concentrations are described by molarities.)

13.E.4.74: S13.4.39
(a) HB ionizes to a greater degree and has the larger Kc.
(b) Kc(HA) = 5 × 10−4
Kc(HB) = 3 × 10−3

This page titled 13.E: Fundamental Equilibrium Concepts (Exercises) is shared under a CC BY 4.0 license and was authored, remixed, and/or
curated by OpenStax via source content that was edited to the style and standards of the LibreTexts platform; a detailed edit history is available
upon request.

Access for free at OpenStax 13.E.18 https://chem.libretexts.org/@go/page/42802


CHAPTER OVERVIEW
14: Acid-Base Equilibria

A general chemistry Libretexts Textbook remixed and remastered from


OpenStax's textbook:
General Chemistry
This chapter will illustrate the chemistry of acid-base reactions and equilibria, and provide you with tools for quantifying the
concentrations of acids and bases in solutions.
14.1: Brønsted-Lowry Acids and Bases
14.2: pH and pOH
14.3: Relative Strengths of Acids and Bases
14.4: Hydrolysis of Salt Solutions
14.5: Polyprotic Acids
14.6: Buffers
14.7: Acid-Base Titrations
14.E: Acid-Base Equilibria (Exercises)

This page titled 14: Acid-Base Equilibria is shared under a CC BY 4.0 license and was authored, remixed, and/or curated by OpenStax via source
content that was edited to the style and standards of the LibreTexts platform; a detailed edit history is available upon request.

1
14.1: Brønsted-Lowry Acids and Bases
 Learning Objectives
Identify acids, bases, and conjugate acid-base pairs according to the Brønsted-Lowry definition
Write equations for acid and base ionization reactions
Use the ion-product constant for water to calculate hydronium and hydroxide ion concentrations
Describe the acid-base behavior of amphiprotic substances

Acids and bases have been known for a long time. When Robert Boyle characterized them in 1680, he noted that acids dissolve
many substances, change the color of certain natural dyes (for example, they change litmus from blue to red), and lose these
characteristic properties after coming into contact with alkalis (bases). In the eighteenth century, it was recognized that acids have a
sour taste, react with limestone to liberate a gaseous substance (now known to be CO2), and interact with alkalis to form neutral
substances. In 1815, Humphry Davy contributed greatly to the development of the modern acid-base concept by demonstrating that
hydrogen is the essential constituent of acids. Around that same time, Joseph Louis Gay-Lussac concluded that acids are substances
that can neutralize bases and that these two classes of substances can be defined only in terms of each other. The significance of
hydrogen was reemphasized in 1884 when Svante Arrhenius defined an acid as a compound that dissolves in water to yield
hydrogen cations (now recognized to be hydronium ions) and a base as a compound that dissolves in water to yield hydroxide
anions.
In an earlier chapter on chemical reactions, we defined acids and bases as Arrhenius did: We identified an acid as a compound that
dissolves in water to yield hydronium ions (H3O+) and a base as a compound that dissolves in water to yield hydroxide ions (
OH ). This definition is not wrong; it is simply limited.

Later, we extended the definition of an acid or a base using the more general definition proposed in 1923 by the Danish chemist
Johannes Brønsted and the English chemist Thomas Lowry. Their definition centers on the proton, H . A proton is what remains +

when a normal hydrogen atom, H , loses an electron. A compound that donates a proton to another compound is called a Brønsted-
1
1

Lowry acid, and a compound that accepts a proton is called a Brønsted-Lowry base. An acid-base reaction is the transfer of a
proton from a proton donor (acid) to a proton acceptor (base). In a subsequent chapter of this text we will introduce the most
general model of acid-base behavior introduced by the American chemist G. N. Lewis.
Acids may be compounds such as HCl or H2SO4, organic acids like acetic acid (CH COOH ) or ascorbic acid (vitamin C), or H2O.
3
3 +
Anions (such as \(\ce{HSO_4^-}\), H PO , HS , and HCO ) and cations (such as H O , NH , and [Al(H O) ] ) may also
2

4
− −

3 3
+ +

4 2 6

act as acids. Bases fall into the same three categories. Bases may be neutral molecules (such as H O , NH , and CH NH ), anions2 3 3 2

(such as OH , HS , HCO , CO , F , and PO ), or cations (such as [Al(H O) OH] ). The most familiar bases are ionic
− − −

3
2 −

3
− 3 −

4 2 5
2 +

compounds such as NaOH and Ca(OH) , which contain the hydroxide ion, OH . The hydroxide ion in these compounds accepts
2

a proton from acids to form water:


+ −
H + OH → H O (14.1.1)
2

We call the product that remains after an acid donates a proton the conjugate base of the acid. This species is a base because it can
accept a proton (to re-form the acid):
acid ⇌ proton + conjugate base (14.1.2)

+ −
HF ⇌ H +F (14.1.3)

+ −
H SO ⇌ H + HSO (14.1.4)
2 4 4

+ −
H O⇌ H + OH (14.1.5)
2

− + 2 −
HSO 4 ⇌ H + SO 4 (14.1.6)

+ +
NH ⇌ H + NH (14.1.7)
4 3

We call the product that results when a base accepts a proton the base’s conjugate acid. This species is an acid because it can give
up a proton (and thus re-form the base):
base + proton ⇌ conjugate acid (14.1.8)

Access for free at OpenStax 14.1.1 https://chem.libretexts.org/@go/page/38276


− +
OH +H ⇌ H O (14.1.9)
2

+ +
H O+H ⇌ H O (14.1.10)
2 3

+ +
NH +H ⇌ NH (14.1.11)
3 4

2 − + −
S +H ⇌ HS (14.1.12)

2 − + −
CO3 +H ⇌ HCO3 (14.1.13)

− +
F +H ⇌ HF (14.1.14)

In these two sets of equations, the behaviors of acids as proton donors and bases as proton acceptors are represented in isolation. In
reality, all acid-base reactions involve the transfer of protons between acids and bases. For example, consider the acid-base reaction
that takes place when ammonia is dissolved in water. A water molecule (functioning as an acid) transfers a proton to an ammonia
molecule (functioning as a base), yielding the conjugate base of water, OH , and the conjugate acid of ammonia, NH :
− +

This figure has three parts in two rows. In the first row, two diagrams of acid-base pairs are shown. On the left, a space filling
model of H subscript 2 O is shown with a red O atom at the center and two smaller white H atoms attached in a bent shape. Above
this model is the label “H subscript 2 O (acid)” in purple. An arrow points right, which is labeled “Remove H superscript plus.” To
the right is another space filling model with a single red O atom to which a single smaller white H atom is attached. The label in
purple above this model reads, “O H superscript negative (conjugate base).” Above both of these red and white models is an
upward pointing bracket that is labeled “Conjugate acid-base pair.” To the right is a space filling model with a central blue N atom
to which three smaller white H atoms are attached in a triangular pyramid arrangement. A label in green above reads “N H
subscript 3 (base).” An arrow labeled “Add H superscript plus” points right. To the right of the arrow is another space filling model
with a blue central N atom and four smaller white H atoms in a tetrahedral arrangement. The green label above reads “N H
subscript 3 superscript plus (conjugate acid).” Above both of these blue and white models is an upward pointing bracket that is
labeled “Conjugate acid-base pair.” The second row of the figure shows the chemical reaction, H subscript 2 O ( l ) is shown in
purple, and is labeled below in purple as “acid,” plus N H subscript 3 (a q) in green, labeled below in green as “base,” followed by
a double sided arrow arrow and O H superscript negative (a q) in purple, labeled in purple as “conjugate base,” plus N H subscript
4 superscript plus (a q)” in green, which is labeled in green as “conjugate acid.” The acid on the left side of the equation is
connected to the conjugate base on the right with a purple line. Similarly, the base on the left is connected to the conjugate acid on
the right side.
The reaction between a Brønsted-Lowry acid and water is called acid ionization. For example, when hydrogen fluoride dissolves in
water and ionizes, protons are transferred from hydrogen fluoride molecules to water molecules, yielding hydronium ions and
fluoride ions:

Access for free at OpenStax 14.1.2 https://chem.libretexts.org/@go/page/38276


This figure has two rows. In both rows, a chemical reaction is shown. In the first, structural formulas are provided. In this model, in
purple, an H atom is connected to an F atom with a single bond. The F atom has pairs of electron dots at the top, right, and bottom.
This is followed by a plus sign, which is followed in green by an O atom which has H atoms singly bonded above and to the right.
The O atom has pairs of electron dots on its left and lower sides. A double arrow follows. To the right, in brackets is a structure
with a central O atom in green, with green H atoms singly bonded above and to the right. A pair of green electron dots is on the
lower side of the O atom. To the left of the green O atom, a purple H atom is singly bonded. This is followed by a plus sign and an
F atom in purple with pairs of electron dots above, right, below, and to the left. This atom also has a superscript negative sign. The
reaction is written in symbolic form below. H F is labeled in purple below as “Acid subscript 1.” This is followed by plus H
subscript 2 O, which is labeled in green below as “Base subscript 2.” A double sided arrow follows. To the right is H subscript 3 O
superscript plus, which is labeled in green as below in as “Acid subscript 2.” This is followed by plus and F surrounded by 4 pairs
of dots and superscript negative. The label below in purple reads, “Base subscript 1.” To the right of the reactions is the formula, K
subscript a equals left bracket H subscript 3 O superscript plus right bracket left bracket F superscript negative right bracket all
over left bracket H F right bracket.
When we add a base to water, a base ionization reaction occurs in which protons are transferred from water molecules to base
molecules. For example, adding pyridine to water yields hydroxide ions and pyridinium ions:
<div data-mt-source="1"

Notice that both these ionization reactions are represented as equilibrium processes. The relative extent to which these acid and
base ionization reactions proceed is an important topic treated in a later section of this chapter. In the preceding paragraphs we saw
that water can function as either an acid or a base, depending on the nature of the solute dissolved in it. In fact, in pure water or in
any aqueous solution, water acts both as an acid and a base. A very small fraction of water molecules donate protons to other water
molecules to form hydronium ions and hydroxide ions:

This figure has two rows. In both rows, a chemical reaction is shown. In the first, structural formulas are provided. In this model, in
purple, O atom which has H atoms singly bonded above and to the right. The O atom has pairs of electron dots on its left and lower
sides. This is followed by a plus sign, which is followed in green by an O atom which has H atoms singly bonded above and to the
right. The O atom has pairs of electron dots on its left and lower sides. A double arrow follows. To the right, in brackets is a
structure with a central O atom in green, with green H atoms singly bonded above and to the right. A pair of green electron dots is
on the lower side of the O atom. To the left of the green O atom, a purple H atom is singly bonded. Outside the brackets to the right
is a superscript plus. This is followed by a plus sign and an O atom in purple with pairs of electron dots above, left, and below. An
H atom is singly bonded to the right. This atom has a superscript negative sign. The reaction is written in symbolic form below. H
subscript 2 O is labeled in purple below as “Acid subscript 1.” This is followed by plus H subscript 2 O, which is labeled in green
below as “Base subscript 2.” A double sided arrow follows. To the right is H subscript 3 O superscript plus, which is labeled in
green as below in as “Acid subscript 2.” This is followed by plus and O with pairs of dots above, below, and to the left with a
singly bonded H on the right with a superscript negative. The label below in purple reads, “ Base subscript 1.”
This type of reaction, in which a substance ionizes when one molecule of the substance reacts with another molecule of the same
substance, is referred to as autoionization. Pure water undergoes autoionization to a very slight extent. Only about two out of
every 10 molecules in a sample of pure water are ionized at 25 °C. The equilibrium constant for the ionization of water is called
9

the ion-product constant for water (Kw):


+ − + −
H O(l) + H O(l) ⇌ H O + OH Kw = [ H O ][ OH ] (14.1.15)
2 2 3 (aq) (aq) 3

The slight ionization of pure water is reflected in the small value of the equilibrium constant; at 25 °C, Kw has a value of
1.0 × 10
−14
. The process is endothermic, and so the extent of ionization and the resulting concentrations of hydronium ion and
hydroxide ion increase with temperature. For example, at 100 °C, the value for K is approximately 5.1 × 10 , roughly 50-
w
−13

times larger than the value at 25 °C.

Access for free at OpenStax 14.1.3 https://chem.libretexts.org/@go/page/38276


 Example 14.1.1: Ion Concentrations in Pure Water
What are the hydronium ion concentration and the hydroxide ion concentration in pure water at 25 °C?

Solution
The autoionization of water yields the same number of hydronium and hydroxide ions. Therefore, in pure water,
[ H O ] = [ OH ] . At 25 °C:
+ −
3

+ − + 2 − 2 −14
Kw = [ H O ][ OH ] = [H O ] = [ OH ] = 1.0 × 10
3 3

So:
− −−−−−−− −
+ − −14 −7
[H O ] = [ OH ] = √ 1.0 × 10 = 1.0 × 10 M
3

The hydronium ion concentration and the hydroxide ion concentration are the same, and we find that both equal
M .
−7
1.0 × 10

 Exercise 14.1.1

The ion product of water at 80 °C is 2.4 × 10 −13


. What are the concentrations of hydronium and hydroxide ions in pure water
at 80 °C?

Answer
+ − −7
[H O ] = [ OH ] = 4.9 × 10 M
3

It is important to realize that the autoionization equilibrium for water is established in all aqueous solutions. Adding an acid or base
to water will not change the position of the equilibrium. Example 14.12 demonstrates the quantitative aspects of this relation
between hydronium and hydroxide ion concentrations.

 Example 14.1.2: The Inverse Proportionality of [H 3


O
+
] and [OH −
]

The Inverse Proportionality of [H3O+] and [OH-] A solution of carbon dioxide in water has a hydronium ion concentration of
M . What is the concentration of hydroxide ion at 25 °C?
−6
2.0 × 10

Solution
We know the value of the ion-product constant for water at 25 °C:
+ −
2 H O(l) ⇌ H O + OH
2 3 (aq) (aq)

+ − −14
Kw = [ H O ][ OH ] = 1.0 × 10
3

Thus, we can calculate the missing equilibrium concentration.


Rearrangement of the Kw expression yields that [OH −
] is directly proportional to the inverse of [H3O+]:
−14

Kw 1.0 × 10
−9
[ OH ] = = = 5.0 × 10
+ −6
[H O ] 2.0 × 10
3

The hydroxide ion concentration in water is reduced to 5.0 × 10 M as the hydrogen ion concentration increases to
−9

M . This is expected from Le Chatelier’s principle; the autoionization reaction shifts to the left to reduce the stress
−6
2.0 × 10

of the increased hydronium ion concentration and the [OH ] is reduced relative to that in pure water.

A check of these concentrations confirms that our arithmetic is correct:

Access for free at OpenStax 14.1.4 https://chem.libretexts.org/@go/page/38276


+ −
Kw = [ H O ][ OH ]
3

−6 −9
= (2.0 × 10 )(5.0 × 10 )

−14
= 1.0 × 10

 Exercise 14.1.2

What is the hydronium ion concentration in an aqueous solution with a hydroxide ion concentration of 0.001 M at 25 °C?

Answer
+ −11
[H O ] = 1 × 10 M
3

14.1.1: Amphiprotic Species


Like water, many molecules and ions may either gain or lose a proton under the appropriate conditions. Such species are said to be
amphiprotic. Another term used to describe such species is amphoteric, which is a more general term for a species that may act
either as an acid or a base by any definition (not just the Brønsted-Lowry one). Consider for example the bicarbonate ion, which
may either donate or accept a proton as shown here:
− 2 − +
HCO + H O(l) ⇌ CO +H O (14.1.16)
3(aq) 2 3(aq) 3 (aq)

− −
HCO + H O(l) ⇌ H CO3(aq) + OH (14.1.17)
3(aq) 2 2 (aq)

 Example 14.1.3: The Acid-Base Behavior of an Amphoteric Substance

Write separate equations representing the reaction of HSO −


3

a. as an acid with OH −

b. as a base with HI

Solution
a. HSO −
3
(aq) + OH

(aq) ⇌ SO
2−
3
(aq) + H O(l)
2

b. HSO −

3
(aq) + HI(aq) ⇌ H SO (aq) + I
2 3

(aq)

 Exercise 14.1.3

Write separate equations representing the reaction of H 2


PO

a. as a base with HBr


b. as an acid with OH −

Answer a
− −
H PO (aq) + HBr(aq) ⇌ H PO (aq) + Br (aq)
2 4 3 4

Answer b
− − 2−
H PO4 (aq) + OH (aq) ⇌ HPO4 (aq) + H O(l)
2 2

Summary
A compound that can donate a proton (a hydrogen ion) to another compound is called a Brønsted-Lowry acid. The compound that
accepts the proton is called a Brønsted-Lowry base. The species remaining after a Brønsted-Lowry acid has lost a proton is the
conjugate base of the acid. The species formed when a Brønsted-Lowry base gains a proton is the conjugate acid of the base. Thus,
an acid-base reaction occurs when a proton is transferred from an acid to a base, with formation of the conjugate base of the
reactant acid and formation of the conjugate acid of the reactant base. Amphiprotic species can act as both proton donors and

Access for free at OpenStax 14.1.5 https://chem.libretexts.org/@go/page/38276


proton acceptors. Water is the most important amphiprotic species. It can form both the hydronium ion, H3O+, and the hydroxide
ion, OH when it undergoes autoionization:

+ −
2 H O(l) ⇌ H O (aq) + OH (aq)
2 3

The ion product of water, Kw is the equilibrium constant for the autoionization reaction:
+ − −14
Kw = [ H3 O ][OH ] = 1.0 × 10 at 25°C

14.1.2: Key Equations


+ − −14
Kw = [ H O ][ OH ] = 1.0 × 10  (at 25 °C)
3

Glossary
acid ionization
reaction involving the transfer of a proton from an acid to water, yielding hydronium ions and the conjugate base of the acid

amphiprotic
species that may either gain or lose a proton in a reaction

amphoteric
species that can act as either an acid or a base

autoionization
reaction between identical species yielding ionic products; for water, this reaction involves transfer of protons to yield
hydronium and hydroxide ions

base ionization
reaction involving the transfer of a proton from water to a base, yielding hydroxide ions and the conjugate acid of the base

Brønsted-Lowry acid
proton donor

Brønsted-Lowry base
proton acceptor

conjugate acid
substance formed when a base gains a proton

conjugate base
substance formed when an acid loses a proton

ion-product constant for water (Kw)


equilibrium constant for the autoionization of water

This page titled 14.1: Brønsted-Lowry Acids and Bases is shared under a CC BY 4.0 license and was authored, remixed, and/or curated by
OpenStax via source content that was edited to the style and standards of the LibreTexts platform; a detailed edit history is available upon request.

Access for free at OpenStax 14.1.6 https://chem.libretexts.org/@go/page/38276


14.2: pH and pOH
 Learning Objectives
Explain the characterization of aqueous solutions as acidic, basic, or neutral
Express hydronium and hydroxide ion concentrations on the pH and pOH scales
Perform calculations relating pH and pOH

As discussed earlier, hydronium and hydroxide ions are present both in pure water and in all aqueous solutions, and their
concentrations are inversely proportional as determined by the ion product of water (K ). The concentrations of these ions in a
w

solution are often critical determinants of the solution’s properties and the chemical behaviors of its other solutes, and specific
vocabulary has been developed to describe these concentrations in relative terms. A solution is neutral if it contains equal
concentrations of hydronium and hydroxide ions; acidic if it contains a greater concentration of hydronium ions than hydroxide
ions; and basic if it contains a lesser concentration of hydronium ions than hydroxide ions.
A common means of expressing quantities, the values of which may span many orders of magnitude, is to use a logarithmic scale.
One such scale that is very popular for chemical concentrations and equilibrium constants is based on the p-function, defined as
shown where “X” is the quantity of interest and “log” is the base-10 logarithm:

pX = − log X (14.2.1)

+
The pH of a solution is therefore defined as shown here, where [H3O ] is the molar concentration of hydronium ion in the solution:
+
pH = − log[ H3 O ] (14.2.2)

Rearranging this equation to isolate the hydronium ion molarity yields the equivalent expression:
+ −pH
[ H3 O ] = 10 (14.2.3)

Likewise, the hydroxide ion molarity may be expressed as a p-function, or pOH:



pOH = − log[OH ] (14.2.4)

or
− −pOH
[OH ] = 10 (14.2.5)

Finally, the relation between these two ion concentration expressed as p-functions is easily derived from the K expression: w

+ −
Kw = [ H O ][ OH ] (14.2.6)
3

+ − + −
− log Kw = − log([ H3 O ][OH ]) = − log[ H3 O ] + − log[OH ] (14.2.7)

pKw = pH + pOH (14.2.8)

At 25 °C, the value of K is 1.0 × 10


w
−14
, and so:
14.00 = pH + pOH (14.2.9)

The hydronium ion molarity in pure water (or any neutral solution) is 1.0 × 10
−7
M at 25 °C. The pH and pOH of a neutral
solution at this temperature are therefore:
+ −7
pH = − log[ H3 O ] = − log(1.0 × 10 ) = 7.00 (14.2.10)

− −7
pOH = − log[OH ] = − log(1.0 × 10 ) = 7.00 (14.2.11)

And so, at this temperature, acidic solutions are those with hydronium ion molarities greater than 1.0 × 10 M and hydroxide ion −7

molarities less than 1.0 × 10 M (corresponding to pH values less than 7.00 and pOH values greater than 7.00). Basic solutions
−7

are those with hydronium ion molarities less than 1.0 × 10 M and hydroxide ion molarities greater than 1.0 × 10 M
−7 −7

(corresponding to pH values greater than 7.00 and pOH values less than 7.00).

Access for free at OpenStax 14.2.1 https://chem.libretexts.org/@go/page/38277


 When pH = 7 Solutions are not Neutral

Since the autoionization constant K is temperature dependent, these correlations between pH values and the
w

acidic/neutral/basic adjectives will be different at temperatures other than 25 °C. For example, the hydronium molarity of pure
water at 80 °C is 4.9 × 10−7 M, which corresponds to pH and pOH values of:
+
pH = − log[ H O ]
3

−7
= − log(4.9 × 10 )

= 6.31


pOH = − log[ OH ]

−7
= − log(4.9 × 10 )

= 6.31

At this temperature, then, neutral solutions exhibit pH = pOH = 6.31, acidic solutions exhibit pH less than 6.31 and pOH
greater than 6.31, whereas basic solutions exhibit pH greater than 6.31 and pOH less than 6.31. This distinction can be
important when studying certain processes that occur at nonstandard temperatures, such as enzyme reactions in warm-blooded
organisms. Unless otherwise noted, references to pH values are presumed to be those at standard temperature (25 °C) (Table
14.2.1).

Table 14.2.1 : Summary of Relations for Acidic, Basic and Neutral Solutions
Classification Relative Ion Concentrations pH at 25 °C

acidic [H3O+] > [OH−] pH < 7

neutral [H3O+] = [OH−] pH = 7

basic [H3O+] < [OH−] pH > 7

Figure 14.2.1 shows the relationships between [H3O+], [OH−], pH, and pOH, and gives values for these properties at standard
temperatures for some common substances.

Access for free at OpenStax 14.2.2 https://chem.libretexts.org/@go/page/38277


Figure 14.2.1 : The pH and pOH scales represent concentrations of [H3O+] and OH−, respectively. The pH and pOH values of some
common substances at standard temperature (25 °C) are shown in this chart.
A table is provided with 5 columns. The first column is labeled “left bracket H subscript 3 O superscript plus right bracket (M).”
Powers of ten are listed in the column beginning at 10 superscript 1, including 10 superscript 0 or 1, 10 superscript negative 1,
decreasing by single powers of 10 to 10 superscript negative 15. The second column is labeled “left bracket O H superscript
negative right bracket (M).” Powers of ten are listed in the column beginning at 10 superscript negative 15, increasing by single
powers of 10 to including 10 superscript 0 or 1, and 10 superscript 1. The third column is labeled “p H.” Values listed in this
column are integers beginning at negative 1, increasing by ones up to 14. The fourth column is labeled “p O H.” Values in this
column are integers beginning at 15, decreasing by ones up to negative 1. The fifth column is labeled “Sample Solution.” A vertical
line at the left of the column has tick marks corresponding to each p H level in the table. Substances are listed next to this line
segment with line segments connecting them to the line to show approximate p H and p O H values. 1 M H C l is listed at a p H of
0. Gastric juices are listed at a p H of about 1.5. Lime juice is listed at a p H of about 2, followed by 1 M C H subscript 3 C O
subscript 2 H, followed by stomach acid at a p H value of nearly 3. Wine is listed around 3.5. Coffee is listed just past 5. Pure water
is listed at a p H of 7. Pure blood is just beyond 7. Milk of Magnesia is listed just past a p H of 10.5. Household ammonia is listed
just before a pH of 12. 1 M N a O H is listed at a p H of 0. To the right of this labeled arrow is an arrow that points up and down
through the height of the column. A beige strip passes through the table and to this double headed arrow at p H 7. To the left of the
double headed arrow in this beige strip is the label “neutral.” A narrow beige strip runs through the arrow. Just above and below
this region, the arrow is purple. It gradually turns to a bright red as it extends upward. At the top of the arrow, near the head of the
arrow is the label “acidic.” Similarly, the lower region changes color from purple to blue moving to the bottom of the column. The
head at this end of the arrow is labeled “basic.”

 Example 14.2.1: Calculation of pH from [H 3


+
O ]

What is the pH of stomach acid, a solution of HCl with a hydronium ion concentration of 1.2 × 10 −3
M ?

Solution
+
pH = − log[ H3 O ]

−3
= − log(1.2 × 10 )

= −(−2.92)

= 2.92

Access for free at OpenStax 14.2.3 https://chem.libretexts.org/@go/page/38277


 Exercise 14.2.1
Water exposed to air contains carbonic acid, H2CO3, due to the reaction between carbon dioxide and water:

CO (aq) + H O(l) ⇌ H CO (aq)


2 2 2 3

Air-saturated water has a hydronium ion concentration caused by the dissolved CO of 2.0 × 10 2
−6
M , about 20-times larger
than that of pure water. Calculate the pH of the solution at 25 °C.

Answer
5.70

 Example 14.2.2: Calculation of Hydronium Ion Concentration from pH


Calculate the hydronium ion concentration of blood, the pH of which is 7.3 (slightly alkaline).

Solution
+
pH = − log[ H3 O ] = 7.3

+
log[ H3 O ] = −7.3

+ −7.3
[ H3 O ] = 10

or
+
[H O ] = antilog of − 7.3
3

+ −8
[H O ] = 5 × 10 M
3

(On a calculator take the antilog, or the “inverse” log, of −7.3, or calculate 10−7.3.)

 Exercise 14.2.2

Calculate the hydronium ion concentration of a solution with a pH of −1.07.

Answer
12 M

14.2.1: Environmental Science


Normal rainwater has a pH between 5 and 6 due to the presence of dissolved CO2 which forms carbonic acid:
H O(l) + CO (g) ⟶ H CO (aq) (14.2.12)
2 2 2 3

+ −
H CO (aq) ⇌ H (aq) + HCO (aq) (14.2.13)
2 3 3

Acid rain is rainwater that has a pH of less than 5, due to a variety of nonmetal oxides, including CO2, SO2, SO3, NO, and NO2
being dissolved in the water and reacting with it to form not only carbonic acid, but sulfuric acid and nitric acid. The formation and
subsequent ionization of sulfuric acid are shown here:

H O(l) + SO (g) ⟶ H SO (aq) (14.2.14)


2 3 2 4

+ −
H SO (aq) ⟶ H (aq) + HSO (aq) (14.2.15)
2 4 4

Carbon dioxide is naturally present in the atmosphere because we and most other organisms produce it as a waste product of
metabolism. Carbon dioxide is also formed when fires release carbon stored in vegetation or when we burn wood or fossil fuels.
Sulfur trioxide in the atmosphere is naturally produced by volcanic activity, but it also stems from burning fossil fuels, which have
traces of sulfur, and from the process of “roasting” ores of metal sulfides in metal-refining processes. Oxides of nitrogen are

Access for free at OpenStax 14.2.4 https://chem.libretexts.org/@go/page/38277


formed in internal combustion engines where the high temperatures make it possible for the nitrogen and oxygen in air to
chemically combine.
Acid rain is a particular problem in industrial areas where the products of combustion and smelting are released into the air without
being stripped of sulfur and nitrogen oxides. In North America and Europe until the 1980s, it was responsible for the destruction of
forests and freshwater lakes, when the acidity of the rain actually killed trees, damaged soil, and made lakes uninhabitable for all
but the most acid-tolerant species. Acid rain also corrodes statuary and building facades that are made of marble and limestone
(Figure 14.2.2). Regulations limiting the amount of sulfur and nitrogen oxides that can be released into the atmosphere by industry
and automobiles have reduced the severity of acid damage to both natural and manmade environments in North America and
Europe. It is now a growing problem in industrial areas of China and India.

Figure 14.2.2 : (a) Acid rain makes trees more susceptible to drought and insect infestation, and depletes nutrients in the soil. (b) It
also is corrodes statues that are carved from marble or limestone. (credit a: modification of work by Chris M Morris; credit b:
modification of work by “Eden, Janine and Jim”/Flickr)
Two photos are shown. Photograph a on the left shows the upper portion of trees against a bright blue sky. The tops of several trees
at the center of the photograph have bare branches and appear to be dead. Image b shows a statue of a man that appears to from the
revolutionary war era in either marble or limestone.

 Example 14.2.3: Calculation of pOH

What are the pOH and the pH of a 0.0125-M solution of potassium hydroxide, KOH?

Solution
Potassium hydroxide is a highly soluble ionic compound and completely dissociates when dissolved in dilute solution, yielding
[OH−] = 0.0125 M:

pOH = − log[OH ] = − log 0.0125

= −(−1.903) = 1.903

The pH can be found from the pOH :

pH + pOH = 14.00

pH = 14.00 − pOH = 14.00 − 1.903 = 12.10

 Exercise 14.2.3

The hydronium ion concentration of vinegar is approximately 4 × 10


−3
M . What are the corresponding values of pOH and
pH?

Answer
pOH = 11.6,
pH = 14.00 - pOH = 2.4

The acidity of a solution is typically assessed experimentally by measurement of its pH. The pOH of a solution is not usually
measured, as it is easily calculated from an experimentally determined pH value. The pH of a solution can be directly measured
using a pH meter (Figure 14.2.3).

Access for free at OpenStax 14.2.5 https://chem.libretexts.org/@go/page/38277


Figure 14.2.3 : (a) A research-grade pH meter used in a laboratory can have a resolution of 0.001 pH units, an accuracy of ± 0.002
pH units, and may cost in excess of $1000. (b) A portable pH meter has lower resolution (0.01 pH units), lower accuracy (± 0.2 pH
units), and a far lower price tag. (credit b: modification of work by Jacopo Werther)
This figure contains two images. The first, image a, is of an analytical digital p H meter on a laboratory counter. The second, image
b, is of a portable hand held digital p H meter.

The pH of a solution may also be visually estimated using colored indicators (Figure 14.2.3).

Figure 14.2.4 : (a) A universal indicator assumes a different color in solutions of different pH values. Thus, it can be added to a
solution to determine the pH of the solution. The eight vials each contain a universal indicator and 0.1-M solutions of progressively
weaker acids: HCl (pH = l), CH3CO2H (pH = 3), and NH4Cl (pH = 5), deionized water, a neutral substance (pH = 7); and 0.1-M
solutions of the progressively stronger bases: KCl (pH = 7), aniline, C6H5NH2 (pH = 9), NH3 (pH = 11), and NaOH (pH = 13). (b)
pH paper contains a mixture of indicators that give different colors in solutions of differing pH values. (credit: modification of
work by Sahar Atwa).
This figure contains two images. The first shows a variety of colors of solutions in labeled beakers. A red solution in a beaker is
labeled “0.10 M H C l.” An orange solution is labeled “0.10 M C H subscript 3 C O O H.” A yellow-orange solution is labeled “0.1
M N H subscript 4 C l.” A yellow solution is labeled “deionized water.” A second solution beaker is labeled “0.10 M K C l.” A
green solution is labeled “0.10 M aniline.” A blue solution is labeled “0.10 M N H subscript 4 C l (a q).” A final beaker containing
a dark blue solution is labeled “0.10 M N a O H.” Image b shows pHydrion paper that is used for measuring pH in the range of p H
from 1 to 12. The color scale for identifying p H based on color is shown along with several of the test strips used to evaluate p H.

Summary
The concentration of hydronium ion in a solution of an acid in water is greater than 1.0 × 10 M at 25 °C. The concentration of
−7

hydroxide ion in a solution of a base in water is greater than 1.0 × 10 M at 25 °C. The concentration of H3O+ in a solution can
−7

be expressed as the pH of the solution; pH = − log H O . The concentration of OH− can be expressed as the pOH of the solution:
3
+

pOH = − log[ OH ] . In pure water, pH = 7.00 and pOH = 7.00


14.2.2: Key Equations


+
pH = − log[ H O ]
3

pOH = − log[ OH ]

[H3O+] = 10−pH
[OH−] = 10−pOH
pH + pOH = pKw = 14.00 at 25 °C

Glossary
acidic
describes a solution in which [H3O+] > [OH−]

basic

Access for free at OpenStax 14.2.6 https://chem.libretexts.org/@go/page/38277


describes a solution in which [H3O+] < [OH−]

neutral
describes a solution in which [H3O+] = [OH−]

pH
logarithmic measure of the concentration of hydronium ions in a solution

pOH
logarithmic measure of the concentration of hydroxide ions in a solution

This page titled 14.2: pH and pOH is shared under a CC BY 4.0 license and was authored, remixed, and/or curated by OpenStax via source
content that was edited to the style and standards of the LibreTexts platform; a detailed edit history is available upon request.

Access for free at OpenStax 14.2.7 https://chem.libretexts.org/@go/page/38277


14.3: Relative Strengths of Acids and Bases
 Learning Objectives
Assess the relative strengths of acids and bases according to their ionization constants
Rationalize trends in acid–base strength in relation to molecular structure
Carry out equilibrium calculations for weak acid–base systems

We can rank the strengths of acids by the extent to which they ionize in aqueous solution. The reaction of an acid with water is
given by the general expression:
+ −
HA(aq) + H O(l) ⇌ H O (aq) + A (aq)
2 3

Water is the base that reacts with the acid HA, A is the conjugate base of the acid HA, and the hydronium ion is the conjugate

acid of water. A strong acid yields 100% (or very nearly so) of H O and A when the acid ionizes in water; Figure 14.3.1 lists
3
+ −

several strong acids. A weak acid gives small amounts of H O and A . 3


+ −

Figure 14.3.1 : Some of the common strong acids and bases are listed here.
Six Strong Acids Six Strong Bases

HClO
4
perchloric acid LiOH lithium hydroxide

HCl hydrochloric acid NaOH sodium hydroxide

HBr hydrobromic acid KOH potassium hydroxide

HI hydroiodic acid Ca(OH)


2
calcium hydroxide

HNO
3
nitric acid Sr(OH)
2
strontium hydroxide

H SO
2 4
sulfuric acid Ba(OH)
2
barium hydroxide

The relative strengths of acids may be determined by measuring their equilibrium constants in aqueous solutions. In solutions of the
same concentration, stronger acids ionize to a greater extent, and so yield higher concentrations of hydronium ions than do weaker
acids. The equilibrium constant for an acid is called the acid-ionization constant, Ka. For the reaction of an acid HA:
+ −
HA(aq) + H O(l) ⇌ H O (aq) + A (aq)
2 3

we write the equation for the ionization constant as:


+ −
[H O ][ A ]
3
Ka =
[HA]

where the concentrations are those at equilibrium. As noted in the section on equilibrium constants, although water is a reactant in
the reaction, it is the solvent as well, so its activity has a value of 1, which does not change the value of K . a

 Note
It is a common error to claim that the molar concentration of the solvent is in some way involved in the equilibrium law. This
error is a result of a misunderstanding of solution thermodynamics. For example, it is often claimed that Ka = Keq[H2O] for
aqueous solutions. This equation is incorrect because it is an erroneous interpretation of the correct equation Ka = Keq(a ). H2 O

Because a H2 O = 1 for a dilute solution, Ka = Keq(1), or Ka = Keq.

The larger the K of an acid, the larger the concentration of H O and A relative to the concentration of the
a 3
+ −

nonionized acid, HA. Thus a stronger acid has a larger ionization constant than does a weaker acid. The ionization
constants increase as the strengths of the acids increase.

The following data on acid-ionization constants indicate the order of acid strength: CH 3
CO H < HNO
2 2
< HSO

Access for free at OpenStax 14.3.1 https://chem.libretexts.org/@go/page/38278


+ − −5
CH CO H(aq) + H O(l) ⇌ H O (aq) + CH CO (aq) Ka = 1.8 × 10
3 2 2 3 3 2

+ − −4
HNO (aq) + H O(l) ⇌ H O (aq) + NO (aq) Ka = 4.6 × 10
2 2 3 2

− + 2− −2
HSO 4 (aq) + H O(l) ⇌ H O (aq) + SO 4 (aq) Ka = 1.2 × 10
2 3

Another measure of the strength of an acid is its percent ionization. The percent ionization of a weak acid is the ratio of the
concentration of the ionized acid to the initial acid concentration, times 100:
+
[H O ]
3 eq
% ionization = × 100% (14.3.1)
[HA]
0

Because the ratio includes the initial concentration, the percent ionization for a solution of a given weak acid varies depending on
the original concentration of the acid, and actually decreases with increasing acid concentration.

 Example 14.3.1: Calculation of Percent Ionization from pH

Calculate the percent ionization of a 0.125-M solution of nitrous acid (a weak acid), with a pH of 2.09.

Solution
The percent ionization for an acid is:
+
[H O ]
3 eq
× 100
[ HNO ]
2 0

The chemical equation for the dissociation of the nitrous acid is:
− +
HNO (aq) + H O(l) ⇌ NO (aq) + H O (aq).
2 2 2 3

Since 10 −pH
= [H O
3
+
] , we find that 10 −2.09
= 8.1 × 10
−3
M , so that percent ionization (Equation 14.3.1) is:
−3
8.1 × 10
× 100 = 6.5%
0.125

Remember, the logarithm 2.09 indicates a hydronium ion concentration with only two significant figures.

 Exercise 14.3.1
Calculate the percent ionization of a 0.10 M solution of acetic acid with a pH of 2.89.

Answer
1.3% ionized

We can rank the strengths of bases by their tendency to form hydroxide ions in aqueous solution. The reaction of a Brønsted-Lowry
base with water is given by:
+ −
B(aq) + H O(l) ⇌ HB (aq) + OH (aq)
2

Water is the acid that reacts with the base, HB is the conjugate acid of the base B , and the hydroxide ion is the conjugate base of
+

water. A strong base yields 100% (or very nearly so) of OH− and HB+ when it reacts with water; Figure 14.3.1 lists several strong
bases. A weak base yields a small proportion of hydroxide ions. Soluble ionic hydroxides such as NaOH are considered strong
bases because they dissociate completely when dissolved in water.
As we did with acids, we can measure the relative strengths of bases by measuring their base-ionization constant (Kb) in aqueous
solutions. In solutions of the same concentration, stronger bases ionize to a greater extent, and so yield higher hydroxide ion
concentrations than do weaker bases. A stronger base has a larger ionization constant than does a weaker base. For the reaction of a
base, B :

Access for free at OpenStax 14.3.2 https://chem.libretexts.org/@go/page/38278


+ −
B(aq) + H O(l) ⇌ HB (aq) + OH (aq),
2

we write the equation for the ionization constant as:


+ −
[ HB ][ OH ]
Kb =
[B]

where the concentrations are those at equilibrium. Again, we do not see water in the equation because water is the solvent and has
an activity of 1. The chemical reactions and ionization constants of the three bases shown are:
− − −11
NO2 (aq) + H O(l) ⇌ HNO (aq) + OH (aq) Kb = 2.17 × 10
2 2

− − −10
CH CO (aq) + H O(l) ⇌ CH CO H(aq) + OH (aq) Kb = 5.6 × 10
3 2 2 3 2

+ − −5
NH (aq) + H O(l) ⇌ NH (aq) + OH (aq) Kb = 1.8 × 10
3 2 4

A table of ionization constants of weak bases appears in Table E2. As with acids, percent ionization can be measured for basic
solutions, but will vary depending on the base ionization constant and the initial concentration of the solution.
Consider the ionization reactions for a conjugate acid-base pair, HA − A : −

+ −
HA(aq) + H O(l) ⇌ H O (aq) + A (aq)
2 3

+ −
[H O ][ A ]
with K a =
3
.
[HA]

− −
A (aq) + H O(l) ⇌ OH (aq) + HA(aq)
2

[HA][OH]
with K b =

.
[A ]

Adding these two chemical equations yields the equation for the autoionization for water:
− + − −
HA(aq) + H O(l) + A (aq) + H O(l) ⇌ H O (aq) + A (aq) + OH (aq) + HA(aq)
2 2 3

+ −
2 H O(l) ⇌ H O (aq) + OH (aq)
2 3

As shown in the previous chapter on equilibrium, the K expression for a chemical equation derived from adding two or more other
equations is the mathematical product of the other equations’ K expressions. Multiplying the mass-action expressions together and
cancelling common terms, we see that:
+ − −
[H O ][ A ] [HA][ OH ]
3 + −
Ka × Kb = × = [H O ][ OH ] = Kw
− 3
[HA] [A ]

For example, the acid ionization constant of acetic acid (CH3COOH) is 1.8 × 10−5, and the base ionization constant of its conjugate
base, acetate ion (CH COO ), is 5.6 × 10−10. The product of these two constants is indeed equal to K :
3

w

−5 −10 −14
Ka × Kb = (1.8 × 10 ) × (5.6 × 10 ) = 1.0 × 10 = Kw

The extent to which an acid, HA, donates protons to water molecules depends on the strength of the conjugate base, A , of the −

acid. If A is a strong base, any protons that are donated to water molecules are recaptured by A . Thus there is relatively little
− −

A

and H O in solution, and the acid, HA, is weak. If A is a weak base, water binds the protons more strongly, and the
3
+ −

solution contains primarily A and H O —the acid is strong. Strong acids form very weak conjugate bases, and weak acids form

3
+

stronger conjugate bases (Figure 14.3.2).


<div data-mt-source="1"
&quot;&quot;
" style="width: 784px; height: 271px;" width="784px" height="271px"
src="/@api/deki/files/65684/CNX_Chem_14_03_strengths.jpg">
Figure 14.3.2 : This diagram shows the relative strengths of conjugate acid-base pairs, as indicated by their ionization constants in
aqueous solution.
Figure 14.3.3 lists a series of acids and bases in order of the decreasing strengths of the acids and the corresponding increasing
strengths of the bases. The acid and base in a given row are conjugate to each other.

Access for free at OpenStax 14.3.3 https://chem.libretexts.org/@go/page/38278


The first six acids in Figure 14.3.3 are the most common strong acids. These acids are completely dissociated in aqueous solution.
The conjugate bases of these acids are weaker bases than water. When one of these acids dissolves in water, their protons are
completely transferred to water, the stronger base.
Those acids that lie between the hydronium ion and water in Figure 14.3.3 form conjugate bases that can compete with water for
possession of a proton. Both hydronium ions and nonionized acid molecules are present in equilibrium in a solution of one of these
acids. Compounds that are weaker acids than water (those found below water in the column of acids) in Figure 14.3.3 exhibit no
observable acidic behavior when dissolved in water. Their conjugate bases are stronger than the hydroxide ion, and if any conjugate
base were formed, it would react with water to re-form the acid.
<div data-mt-source="1"
&quot;&quot;
" style="width: 798px; height: 715px;" width="798px" height="715px"
src="/@api/deki/files/65685/CNX_Chem_14_03_Corresp.jpg">
Figure 14.3.3 : The chart shows the relative strengths of conjugate acid-base pairs.
The extent to which a base forms hydroxide ion in aqueous solution depends on the strength of the base relative to that of the
hydroxide ion, as shown in the last column in Figure 14.3.3. A strong base, such as one of those lying below hydroxide ion, accepts
protons from water to yield 100% of the conjugate acid and hydroxide ion. Those bases lying between water and hydroxide ion
accept protons from water, but a mixture of the hydroxide ion and the base results. Bases that are weaker than water (those that lie
above water in the column of bases) show no observable basic behavior in aqueous solution.

 Example 14.3.2: The Product Ka × Kb = Kw

Use the K for the nitrite ion, NO , to calculate the K for its conjugate acid.
b

2 a

Solution
Kb for NO is given in this section as 2.17 × 10−11. The conjugate acid of

2
NO

2
is HNO2; Ka for HNO2 can be calculated
using the relationship:
−14
Ka × Kb = 1.0 × 10 = Kw

Solving for Ka, we get:


Kw
Ka =
Kb

−14
1.0 × 10
=
−11
2.17 × 10

−4
= 4.6 × 10

This answer can be verified by finding the Ka for HNO2 in Table E1

 Exercise 14.3.2
We can determine the relative acid strengths of NH and HCN by comparing their ionization constants. The ionization
+

constant of HCN is given in Table E1 as 4.9 × 10−10. The ionization constant of NH is not listed, but the ionization constant
+

of its conjugate base, NH , is listed as 1.8 × 10−5. Determine the ionization constant of NH , and decide which is the stronger
3
+

acid, HCN or NH . +

Answer
NH
+
4
is the slightly stronger acid (Ka for NH = 5.6 × 10−10).
+
4

14.3.1: The Ionization of Weak Acids and Weak Bases


Many acids and bases are weak; that is, they do not ionize fully in aqueous solution. A solution of a weak acid in water is a mixture
of the nonionized acid, hydronium ion, and the conjugate base of the acid, with the nonionized acid present in the greatest

Access for free at OpenStax 14.3.4 https://chem.libretexts.org/@go/page/38278


concentration. Thus, a weak acid increases the hydronium ion concentration in an aqueous solution (but not as much as the same
amount of a strong acid).
Acetic acid (CH 3
CO H
2
) is a weak acid. When we add acetic acid to water, it ionizes to a small extent according to the equation:
+ −
CH CO H(aq) + H O(l) ⇌ H O (aq) + CH CO (aq)
3 2 2 3 3 2

giving an equilibrium mixture with most of the acid present in the nonionized (molecular) form. This equilibrium, like other
equilibria, is dynamic; acetic acid molecules donate hydrogen ions to water molecules and form hydronium ions and acetate ions at
the same rate that hydronium ions donate hydrogen ions to acetate ions to reform acetic acid molecules and water molecules. We
can tell by measuring the pH of an aqueous solution of known concentration that only a fraction of the weak acid is ionized at any
moment (Figure 14.3.4). The remaining weak acid is present in the nonionized form.
For acetic acid, at equilibrium:
+ −
[H O ][ CH CO2 ]
3 3 −5
Ka = = 1.8 × 10
[ CH CO H]
3 2

Figure 14.3.4 : pH paper indicates that a 0.l-M solution of HCl (beaker on left) has a pH of 1. The acid is fully ionized and [H O ] 3
+

= 0.1 M. A 0.1-M solution of CH3CO2H (beaker on right) has a pH of 3 ( [H O ] = 0.001 M) because the weak acid CH3CO2H is
3
+

only partially ionized. In this solution, [H O ] < [CH CO H] . (credit: modification of work by Sahar Atwa)
3
+

3 2

This image shows two bottles containing clear colorless solutions. Each bottle contains a single p H indicator strip. The strip in the
bottle on the left is red, and a similar red strip is placed on a filter paper circle in front of the bottle on surface on which the bottles
are resting. Similarly, the second bottle on the right contains and orange strip and an orange strip is placed in front of it on a filter
paper circle. Between the two bottles is a pack of p Hydrion papers with a p H color scale on its cover.
Table 14.3.1 : Ionization Constants of Some Weak Acids
Ionization Reaction Ka at 25 °C

HSO

4
+H O ⇌ H O
2 3
+
+ SO
2−
4
1.2 × 10−2

HF + H O ⇌ H O
2 3
+
+F

3.5 × 10−4

HNO
2
+H O ⇌ H O
2 3
+
+ NO

2
4.6 × 10−4

HNCO + H O ⇌ H O
2 3
+
+ NCO

2 × 10−4

HCO H + H O ⇌ H O
2 2 3
+
+ HCO

2
1.8 × 10−4

CH CO H + H O ⇌ H O
3 2 2 3
+
+ CH CO
3

2
1.8 × 10−5

HCIO + H O ⇌ H O
2 3
+
+ CIO

2.9 × 10−8

HBrO + H O ⇌ H O
2 3
+
+ BrO

2.8 × 10−9

HCN + H O ⇌ H O
2 3
+
+ CN

4.9 × 10−10

Table 14.3.1 gives the ionization constants for several weak acids; additional ionization constants can be found in Table E1.
At equilibrium, a solution of a weak base in water is a mixture of the nonionized base, the conjugate acid of the weak base, and
hydroxide ion with the nonionized base present in the greatest concentration. Thus, a weak base increases the hydroxide ion
concentration in an aqueous solution (but not as much as the same amount of a strong base).

Access for free at OpenStax 14.3.5 https://chem.libretexts.org/@go/page/38278


For example, a solution of the weak base trimethylamine, (CH3)3N, in water reacts according to the equation:
+ −
(CH ) N(aq) + H O(l) ⇌ (CH ) NH (aq) + OH (aq)
3 3 2 3 3

This gives an equilibrium mixture with most of the base present as the nonionized amine. This equilibrium is analogous to that
described for weak acids.
We can confirm by measuring the pH of an aqueous solution of a weak base of known concentration that only a fraction of the base
reacts with water (Figure 14.4.5). The remaining weak base is present as the unreacted form. The equilibrium constant for the
ionization of a weak base, K , is called the ionization constant of the weak base, and is equal to the reaction quotient when the
b

reaction is at equilibrium. For trimethylamine, at equilibrium:


+ −
[ (CH ) NH ][ OH ]
3 3
Kb =
[ (CH ) N]
3 3

Figure 14.3.5 : pH paper indicates that a 0.1-M solution of NH3 (left) is weakly basic. The solution has a pOH of 3 ([OH−] = 0.001
M) because the weak base NH3 only partially reacts with water. A 0.1-M solution of NaOH (right) has a pOH of 1 because NaOH
is a strong base (credit: modification of work by Sahar Atwa).
The ionization constants of several weak bases are given in Table 14.3.2 and Table E2.
Table 14.3.2 : Ionization Constants of Some Weak Bases
Ionization Reaction Kb at 25 °C

(CH ) NH + H O ⇌ (CH ) NH
3 2 2 3 2
+
2
+ OH

5.9 × 10−4

CH NH
3 2
+ H O ⇌ CH NH
2 3
+
3
+ OH

4.4 × 10−4

(CH ) N + H O ⇌ (CH ) NH
3 3 2 3 3
+
+ OH

6.3 × 10−5

NH
3
+ H O ⇌ NH
2
+
4
+ OH

1.8 × 10−5

C H NH
6 5 2
+ H O ⇌ C N NH
2 6 5
+
3
+ OH

4.3 × 10−10

 Example 14.3.3: Determination of Ka from Equilibrium Concentrations


Acetic acid is the principal ingredient in vinegar; that's why it tastes sour. At equilibrium, a solution contains [CH3CO2H] =
0.0787 M and [H O ] = [CH CO ] = 0.00118 M. What is the value of K for acetic acid?
3
+
3

2 a

Vinegar is a solution of acetic acid, a weak acid. (credit: modification of work by “HomeSpot HQ”/Flickr)

Access for free at OpenStax 14.3.6 https://chem.libretexts.org/@go/page/38278


Solution
We are asked to calculate an equilibrium constant from equilibrium concentrations. At equilibrium, the value of the equilibrium
constant is equal to the reaction quotient for the reaction:
+ −
CH CO H(aq) + H O(l) ⇌ H O (aq) + CH CO (aq)
3 2 2 3 3 2

+ −
[H O ][ CH CO ]
3 3 2
Ka =
[ CH CO H]
3 2

(0.00118)(0.00118)
=
0.0787

−5
= 1.77 × 10

 Exercise 14.3.3

What is the equilibrium constant for the ionization of the HSO ion, the weak acid used in some household cleansers:

− + 2−
HSO 4 (aq) + H O(l) ⇌ H O (aq) + SO 4 (aq)
2 3

In one mixture of NaHSO4 and Na2SO4 at equilibrium, [H 3


O
+
] = 0.027 M; [HSO −
4
] = 0.29 M ; and [SO 2−
4
] = 0.13 M .

Answer
Ka for HSO −

4
= 1.2 × ×10
−2

 Example 14.3.4: Determination of Kb from Equilibrium Concentrations


Caffeine, C8H10N4O2 is a weak base. What is the value of Kb for caffeine if a solution at equilibrium has [C8H10N4O2] = 0.050
M, [C H N O H ] = 5.0 × 10−3 M, and [OH−] = 2.5 × 10−3 M?
8 10 4 2
+

Solution
At equilibrium, the value of the equilibrium constant is equal to the reaction quotient for the reaction:
+ −
C H N O (aq) + H O(l) ⇌ C H N O H (aq) + OH (aq)
8 10 4 2 2 8 10 4 2

so
+ − −3 −3
[C H N O H ][ OH ] (5.0 × 10 )(2.5 × 10 )
8 10 4 2 −4
Kb = = = 2.5 × 10
[C H N O ] 0.050
8 10 4 2

 Exercise 14.3.4

What is the equilibrium constant for the ionization of the HPO 2−

4
ion, a weak base:
2− − −
HPO (aq) + H O(l) ⇌ H PO (aq) + OH (aq)
4 2 2 4

In a solution containing a mixture of NaH 2


PO
4
and Na 2
HPO
4
at equilibrium with:
− −6
[ OH ] = 1.3 × 10 M

[ H PO
2

4
]=0 ⋅ 042 M and
M.
2 −
[ HPO ]=0 ⋅ 341
4

Answer
Kb for HPO 2−

4
= 1.6 × 10
−7

Access for free at OpenStax 14.3.7 https://chem.libretexts.org/@go/page/38278


 Example 14.3.5: Determination of Ka or Kb from pH
The pH of a 0.0516-M solution of nitrous acid, HNO , is 2.34. What is its K ?
2 a

+ −
HNO (aq) + H O(l) ⇌ H O (aq) + NO (aq)
2 2 3 2

Solution
We determine an equilibrium constant starting with the initial concentrations of HNO2, H O , and NO as well as one of the
3
+ −
2

final concentrations, the concentration of hydronium ion at equilibrium. (Remember that pH is simply another way to express
the concentration of hydronium ion.)
We can solve this problem with the following steps in which x is a change in concentration of a species in the reaction:

We can summarize the various concentrations and changes as shown here. Because water is the solvent, it has a fixed activity
equal to 1. Any small amount of water produced or used up during the reaction will not change water's role as the solvent, so
the value of its activity remains equal to 1 throughout the reaction.

To get the various values in the ICE (Initial, Change, Equilibrium) table, we first calculate [H O
3
+
, the equilibrium
]

concentration of H O , from the pH:


3
+

+ −2.34
[H O ] = 10 = 0.0046 M
3

The change in concentration of H O , x


3
+
, is the difference between the equilibrium concentration of H3O+, which we
[H O
3
+
]

determined from the pH, and the initial concentration, [H O ] . The initial concentration of H O is its concentration in pure
3
+
i 3
+

water, which is so much less than the final concentration that we approximate it as zero (~0).
The change in concentration of NO is equal to the change in concentration of [H O ]. For each 1 mol of H O that forms,

2 3
+

3
+

1 mol of NO forms. The equilibrium concentration of HNO2 is equal to its initial concentration plus the change in its

concentration.
Now we can fill in the ICE table with the concentrations at equilibrium, as shown here:

Access for free at OpenStax 14.3.8 https://chem.libretexts.org/@go/page/38278


Finally, we calculate the value of the equilibrium constant using the data in the table:
+ −
[H O ][ NO ] (0.0046)(0.0046)
3 2 −4
Ka = = = 4.5 × 10
[ HNO ] (0.0470)
2

 Exercise 14.3.5

The pH of a solution of household ammonia, a 0.950-M solution of NH3, is 11.612. What is Kb for NH3.

Answer
−5
Kb = 1.8 × 10

 Example 14.3.6: Equilibrium Concentrations in a Solution of a Weak Acid

Formic acid, HCO2H, is the irritant that causes the body’s reaction to ant stings.

The pain of an ant’s sting is caused by formic acid. (credit: John Tann)
What is the concentration of hydronium ion and the pH in a 0.534-M solution of formic acid?
+ − −4
HCO H(aq) + H O(l) ⇌ H O (aq) + HCO (aq) Ka = 1.8 × 10
2 2 3 2

Solution
1. Determine x and equilibrium concentrations. The equilibrium expression is:
+ −
HCO H(aq) + H O(l) ⇌ H O (aq) + HCO (aq)
2 2 3 2

Because water is the solvent, it has a fixed activity equal to 1. Any small amount of water produced or used up during the
reaction will not change water's role as the solvent, so the value of its activity remains equal to 1 throughout the reaction so we
do not need to consider it when setting up the ICE table.
The table shows initial concentrations (concentrations before the acid ionizes), changes in concentration, and equilibrium
concentrations follows (the data given in the problem appear in color):

Access for free at OpenStax 14.3.9 https://chem.libretexts.org/@go/page/38278


2. Solve for x and the equilibrium concentrations. At equilibrium:
+ −
[H O ][ HCO ]
−4 3 2
Ka = 1.8 × 10 =
[ HCO H]
2

(x)(x)
−4
= = 1.8 × 10
0.534 − x

Now solve for x. Because the initial concentration of acid is reasonably large and K is very small, we assume that a

x << 0.534, which permits us to simplify the denominator term as (0.534 − x) = 0.534. This gives:

2
x
−4
Ka = 1.8 × 10 =
0.534

Solve for x as follows:


2 −4
x = 0.534 × (1.8 × 10 )

−5
= 9.6 × 10

−−−−−−− −
−5
x = √ 9.6 × 10

−3
= 9.8 × 10

To check the assumption that x is small compared to 0.534, we calculate:


−3
x 9.8 × 10
=
0.534 0.534

−2
= 1.8 × 10 (1.8% of 0.534)

x is less than 5% of the initial concentration; the assumption is valid.


We find the equilibrium concentration of hydronium ion in this formic acid solution from its initial concentration and the change in
that concentration as indicated in the last line of the table:
+ −3
[H O ] =  0 + x = 0 + 9.8 × 10 M.
3

−3
= 9.8 × 10 M

The pH of the solution can be found by taking the negative log of the [H 3
O
+
], so:
−3
pH = − log(9.8 × 10 ) = 2.01

 Exercise 14.3.6: acetic acid

Only a small fraction of a weak acid ionizes in aqueous solution. What is the percent ionization of acetic acid in a 0.100-M
solution of acetic acid, CH3CO2H?
+ − −5
CH CO H(aq) + H O(l) ⇌ H O (aq) + CH CO (aq) Ka = 1.8 × 10
3 2 2 3 3 2

Hint
Determine [CH 3

CO
2
] at equilibrium.) Recall that the percent ionization is the fraction of acetic acid that is ionized × 100,

[ CH CO2 ]
3
or × 100 .
[ CH CO H]
3 2 initial

Access for free at OpenStax 14.3.10 https://chem.libretexts.org/@go/page/38278


Answer
percent ionization = 1.3%

The following example shows that the concentration of products produced by the ionization of a weak base can be determined by
the same series of steps used with a weak acid.

 Example 14.3.7: Equilibrium Concentrations in a Solution of a Weak Base

Find the concentration of hydroxide ion in a 0.25-M solution of trimethylamine, a weak base:
+ − −5
(CH ) N(aq) + H O(l) ⇌ (CH ) NH (aq) + OH (aq) Kb = 6.3 × 10
3 3 2 3 3

Solution This problem requires that we calculate an equilibrium concentration by determining concentration changes as the
ionization of a base goes to equilibrium. The solution is approached in the same way as that for the ionization of formic acid in
Example 14.3.6. The reactants and products will be different and the numbers will be different, but the logic will be the same:

1. Determine x and equilibrium concentrations. The table shows the changes and concentrations:

2. Solve for x and the equilibrium concentrations. At equilibrium:


+ −
[ (CH ) NH ][ OH ] (x)(x)
3 3 −5
Kb = = 6.3 × 10
[ (CH ) N] 0.25 − x =
3 3

If we assume that x is small relative to 0.25, then we can replace (0.25 − x) in the preceding equation with 0.25. Solving the
simplified equation gives:
−3
x = 4.0 × 10

This change is less than 5% of the initial concentration (0.25), so the assumption is justified.
Recall that, for this computation, x is equal to the equilibrium concentration of hydroxide ion in the solution (see earlier
tabulation):
− −3
([ OH ] =  0 + x = x = 4.0 × 10 M

−3
= 4.0 × 10 M

Then calculate pOH as follows:


−3
pOH = − log(4.3 × 10 ) = 2.40

Using the relation introduced in the previous section of this chapter:

pH + pOH = pKw = 14.00

permits the computation of pH:

pH = 14.00 − pOH = 14.00 − 2.37 = 11.60

Access for free at OpenStax 14.3.11 https://chem.libretexts.org/@go/page/38278


Check the work. A check of our arithmetic shows that K b = 6.3 × 10
−5
.

 Exercise 14.3.7
a. Show that the calculation in Step 2 of this example gives an x of 4.3 × 10−3 and the calculation in Step 3 shows Kb = 6.3 ×
10−5.
b. Find the concentration of hydroxide ion in a 0.0325-M solution of ammonia, a weak base with a Kb of 1.76 × 10−5.
+
[ NH ]
4
Calculate the percent ionization of ammonia, the fraction ionized × 100, or × 100%
[ NH ]
3

Answer a
7.56 × 10
−4
M , 2.33%
Answer b
2.33%

Some weak acids and weak bases ionize to such an extent that the simplifying assumption that x is small relative to the initial
concentration of the acid or base is inappropriate. As we solve for the equilibrium concentrations in such cases, we will see that we
cannot neglect the change in the initial concentration of the acid or base, and we must solve the equilibrium equations by using the
quadratic equation.

 Example 14.3.8: Equilibrium Concentrations in a Solution of a Weak Acid

Sodium bisulfate, NaHSO4, is used in some household cleansers because it contains the −
HSO 4 ion, a weak acid. What is the
pH of a 0.50-M solution of HSO ? −
4

− + 2− −2
HSO (aq) + H O(l) ⇌ H O (aq) + SO (aq) Ka = 1.2 × 10
4 2 3 4

Solution
We need to determine the equilibrium concentration of the hydronium ion that results from the ionization of HSO so that we −
4

can use [H O ] to determine the pH. As in the previous examples, we can approach the solution by the following steps:
3
+

1. Determine x and equilibrium concentrations. This table shows the changes and concentrations:

2. Solve for x and the concentrations.


As we begin solving for x, we will find this is more complicated than in previous examples. As we discuss these complications
we should not lose track of the fact that it is still the purpose of this step to determine the value of x.
At equilibrium:

Access for free at OpenStax 14.3.12 https://chem.libretexts.org/@go/page/38278


+ 2−
[H O ][ SO ] (x)(x)
−2 3 4
Ka = 1.2 × 10 = =

[ HSO ] 0.50 − x
4

If we assume that x is small and approximate (0.50 − x) as 0.50, we find:


−2
x = 7.7 × 10

When we check the assumption, we confirm:


x ?

≤ 0.05

[HSO ]i
4

which for this system is


−2
x 7.7 × 10
= = 0.15(15%)
0.50 0.50

The value of x is not less than 5% of 0.50, so the assumption is not valid. We need the quadratic formula to find x.
The equation:
(x)(x)
−2
Ka = 1.2 × 10 =
0.50 − x

gives
−3 −2 2+
6.0 × 10 − 1.2 × 10 x =x

or
2+ −2 −3
x + 1.2 × 10 x − 6.0 × 10 =0

This equation can be solved using the quadratic formula. For an equation of the form
2+
ax + bx + c = 0,

x is given by the quadratic equation:


− −−−−− −−
−b ± √ b2+ − 4ac
x =
2a

In this problem, a = 1 , b = 1.2 × 10 −3


, and c = −6.0 × 10 −3
.
Solving for x gives a negative root (which cannot be correct since concentration cannot be negative) and a positive root:
−2
x = 7.2 × 10

Now determine the hydronium ion concentration and the pH:


+ −2
[H O ] =  0 + x = 0 + 7.2 × 10 M
3

−2
= 7.2 × 10 M

The pH of this solution is:


+ −2
pH = −log[ H3 O ] = −log7.2 × 10 = 1.14

 Exercise 14.3.8
a. Show that the quadratic formula gives x = 7.2 × 10 . −2

b. Calculate the pH in a 0.010-M solution of caffeine, a weak base:


+ − −4
C H N O (aq) + H O(l) ⇌ C H N O H (aq) + OH (aq) Kb = 2.5 × 10
8 10 4 2 2 8 10 4 2

Hint

Access for free at OpenStax 14.3.13 https://chem.libretexts.org/@go/page/38278


It will be necessary to convert [OH−] to [H 3
+
O ] or pOH to pH toward the end of the calculation.

Answer
pH 11.16

14.3.2: The Relative Strengths of Strong Acids and Bases


Strong acids, such as HCl, HBr , and HI , all exhibit the same strength in water. The water molecule is such a strong base compared
to the conjugate bases Cl−, Br−, and I− that ionization of these strong acids is essentially complete in aqueous solutions. In solvents
less basic than water, we find HCl, HBr , and HI differ markedly in their tendency to give up a proton to the solvent. For example,
when dissolved in ethanol (a weaker base than water), the extent of ionization increases in the order HCl < HBr < HI , and so HI
is demonstrated to be the strongest of these acids. The inability to discern differences in strength among strong acids dissolved in
water is known as the leveling effect of water.
Water also exerts a leveling effect on the strengths of strong bases. For example, the oxide ion, O2−, and the amide ion, NH , are −
2

such strong bases that they react completely with water:


2− − −
O (aq) + H O(l) ⟶ OH (aq) + OH (aq)
2

− −
NH (aq) + H O(l) ⟶ NH (aq) + OH (aq)
2 2 3

Thus, O2− and NH appear to have the same base strength in water; they both give a 100% yield of hydroxide ion.

In the absence of any leveling effect, the acid strength of binary compounds of hydrogen with nonmetals (A) increases as the H-A
bond strength decreases down a group in the periodic table. For group 17, the order of increasing acidity is
HF < HCl < HBr < HI . Likewise, for group 16, the order of increasing acid strength is H2O < H2S < H2Se < H2Te. Across a

row in the periodic table, the acid strength of binary hydrogen compounds increases with increasing electronegativity of the
nonmetal atom because the polarity of the H-A bond increases. Thus, the order of increasing acidity (for removal of one proton)
across the second row is CH < NH < H O < HF ; across the third row, it is SiH < PH < H S < HCl (see Figure 14.3.6).
4 3 2 4 3 2

Access for free at OpenStax 14.3.14 https://chem.libretexts.org/@go/page/38278


Figure 14.3.6 : As you move from left to right and down the periodic table, the acid strength increases. As you move from right to
left and up, the base strength increases.
This diagram has two rows and four columns. Red arrows point left across the bottom of the figure and down at the right side and
are labeled “Increasing acid strength.” Blue arrows point left across the bottom and up at the right side of the figure and are labeled
“Increasing base strength.” The first column is labeled 14 at the top and two white squares are beneath it. The first has the number
6 in the upper left corner and the formula C H subscript 4 in the center along with designation Neither acid nor base. The second
square contains the number 14 in the upper left corner, the formula C H subscript 4 at the center and the designation Neither acid
nor base. The second column is labeled 15 at the top and two blue squares are beneath it. The first has the number 7 in the upper
left corner and the formula N H subscript 3 in the center along with the designation Weak base and K subscript b equals 1.8 times
10 superscript negative 5. The second square contains the number 15 in the upper left corner, the formula P H subscript 3 at the
center and the designation Very weak base and K subscript b equals 4 times 10 superscript negative 28. The third column is labeled
16 at the top and two squares are beneath it. The first is shaded tan and has the number 8 in the upper left corner and the formula H
subscript 2 O in the center along with the designation neutral. The second square is shaded pink, contains the number 16 in the
upper left corner, the formula H subscript 2 S at the center and the designation Weak acid and K subscript a equals 9.5 times 10
superscript negative 8. The fourth column is labeled 17 at the top and two squares are beneath it. The first is shaded pink, has the
number 9 in the upper left corner and the formula H F in the center along with the designation Weak acid and K subscript a equals
6.8 times 10 superscript negative 4. The second square is shaded a deeper pink, contains the number 17 in the upper left corner, the
formula H C l at the center, and the designation Strong acid.
Compounds containing oxygen and one or more hydroxyl (OH) groups can be acidic, basic, or amphoteric, depending on the
position in the periodic table of the central atom E, the atom bonded to the hydroxyl group. Such compounds have the general
formula OnE(OH)m, and include sulfuric acid, O S(OH) , sulfurous acid, OS(OH) , nitric acid, O NOH , perchloric acid,
2 2 2 2

O ClOH, aluminum hydroxide, Al(OH) , calcium hydroxide, Ca(OH) , and potassium hydroxide, KOH :
3 3 2

A diagram is shown that includes a central atom designated with the letter E. Single bonds extend above, below, left, and right of
the E. An O atom is bonded to the right of the E, and an arrow points to the bond labeling it, “Bond a.” An H atom is single bonded
to the right of the O atom. An arrow pointing to this bond connects it to the label, “Bond b.”
If the central atom, E, has a low electronegativity, its attraction for electrons is low. Little tendency exists for the central atom to
form a strong covalent bond with the oxygen atom, and bond a between the element and oxygen is more readily broken than bond b
between oxygen and hydrogen. Hence bond a is ionic, hydroxide ions are released to the solution, and the material behaves as a
base—this is the case with Ca(OH)2 and KOH. Lower electronegativity is characteristic of the more metallic elements; hence, the
metallic elements form ionic hydroxides that are by definition basic compounds.
If, on the other hand, the atom E has a relatively high electronegativity, it strongly attracts the electrons it shares with the oxygen
atom, making bond a relatively strongly covalent. The oxygen-hydrogen bond, bond b, is thereby weakened because electrons are
displaced toward E. Bond b is polar and readily releases hydrogen ions to the solution, so the material behaves as an acid. High
electronegativities are characteristic of the more nonmetallic elements. Thus, nonmetallic elements form covalent compounds
containing acidic −OH groups that are called oxyacids.

Access for free at OpenStax 14.3.15 https://chem.libretexts.org/@go/page/38278


Increasing the oxidation number of the central atom E also increases the acidity of an oxyacid because this increases the attraction
of E for the electrons it shares with oxygen and thereby weakens the O-H bond. Sulfuric acid, H2SO4, or O2S(OH)2 (with a sulfur
oxidation number of +6), is more acidic than sulfurous acid, H2SO3, or OS(OH)2 (with a sulfur oxidation number of +4). Likewise
nitric acid, HNO3, or O2NOH (N oxidation number = +5), is more acidic than nitrous acid, HNO2, or ONOH (N oxidation number
= +3). In each of these pairs, the oxidation number of the central atom is larger for the stronger acid (Figure 14.3.7).

Figure 14.3.7 : As the oxidation number of the central atom E increases, the acidity also increases.
A diagram is shown that includes four structural formulas for acids. A red, right pointing arrow is placed beneath the structures
which is labeled “Increasing acid strength.” At the top left, the structure of Nitrous acid is provided. It includes an H atom to which
an O atom with two unshared electron pairs is connected with a single bond to the right. A single bond extends to the right and
slightly below to a N atom with one unshared electron pair. A double bond extends up and to the right from this N atom to an O
atom which has two unshared electron pairs. To the upper right is a structure for Nitric acid. This structure differs from the previous
structure in that the N atom is directly to the right of the first O atom and a second O atom with three unshared electron pairs is
connected with a single bond below and to the right of the N atom which has no unshared electron pairs. At the lower left, an O
atom with two unshared electron pairs is double bonded to its right to an S atom with a single unshared electron pair. An O atom
with two unshared electron pairs is bonded above and an H atom is single bonded to this O atom. To the right of the S atom is a
single bond to another O atom with two unshared electron pairs to which an H atom is single bonded. This structure is labeled
“Sulfurous acid.” A similar structure which is labeled “Sulfuric acid” is placed in the lower right region of the figure. This structure
differs in that an H atom is single bonded to the left of the first O atom, leaving it with two unshared electron pairs and a fourth O
atom with two unshared electron pairs is double bonded beneath the S atom, leaving it with no unshared electron pairs.
Hydroxy compounds of elements with intermediate electronegativities and relatively high oxidation numbers (for example,
elements near the diagonal line separating the metals from the nonmetals in the periodic table) are usually amphoteric. This means
that the hydroxy compounds act as acids when they react with strong bases and as bases when they react with strong acids. The
amphoterism of aluminum hydroxide, which commonly exists as the hydrate Al(H O) (OH) , is reflected in its solubility in both
2 3 3

strong acids and strong bases. In strong bases, the relatively insoluble hydrated aluminum hydroxide, Al(H O) (OH) , is 2 3 3

converted into the soluble ion, [Al(H O) (OH) ] , by reaction with hydroxide ion:
2 2 4

− −
[Al (H O) (OH) ](aq) + OH (aq) ⇌ H O(l) + [Al (H O) (OH) ] (aq)
2 3 3 2 2 2 4

In this reaction, a proton is transferred from one of the aluminum-bound H2O molecules to a hydroxide ion in solution. The
Al(H O) (OH)
2 3
compound thus acts as an acid under these conditions. On the other hand, when dissolved in strong acids, it is
3

converted to the soluble ion [Al(H O) ] by reaction with hydronium ion:


2 6
3+

+ 3+
3H O (aq) + Al (H O) (OH) (aq) ⇌ Al (H O) (aq) + 3 H O(l)
3 2 3 3 2 6 2

In this case, protons are transferred from hydronium ions in solution to Al(H 2
O) (OH)
3 3
, and the compound functions as a base.

Summary
The strengths of Brønsted-Lowry acids and bases in aqueous solutions can be determined by their acid or base ionization constants.
Stronger acids form weaker conjugate bases, and weaker acids form stronger conjugate bases. Thus strong acids are completely
ionized in aqueous solution because their conjugate bases are weaker bases than water. Weak acids are only partially ionized
because their conjugate bases are strong enough to compete successfully with water for possession of protons. Strong bases react
with water to quantitatively form hydroxide ions. Weak bases give only small amounts of hydroxide ion. The strengths of the
binary acids increase from left to right across a period of the periodic table (CH4 < NH3 < H2O < HF), and they increase down a
group (HF < HCl < HBr < HI). The strengths of oxyacids that contain the same central element increase as the oxidation number of

Access for free at OpenStax 14.3.16 https://chem.libretexts.org/@go/page/38278


the element increases (H2SO3 < H2SO4). The strengths of oxyacids also increase as the electronegativity of the central element
increases [H2SeO4 < H2SO4].

14.3.3: Key Equations


+ −
[H O ][ A ]
3
Ka =
[HA]
+ −
[ HB ][ OH ]
Kb =
[B]
−14
Ka × Kb = 1.0 × 10 = Kw (at room temperature)
+
[H O ]
3 eq
Percent ionization = × 100
[HA]
0

Glossary
acid ionization constant (Ka)
equilibrium constant for the ionization of a weak acid

base ionization constant (Kb)


equilibrium constant for the ionization of a weak base

leveling effect of water


any acid stronger than H O , or any base stronger than OH− will react with water to form H O , or OH−, respectively; water
3
+

3
+

acts as a base to make all strong acids appear equally strong, and it acts as an acid to make all strong bases appear equally
strong

oxyacid
compound containing a nonmetal and one or more hydroxyl groups

percent ionization
ratio of the concentration of the ionized acid to the initial acid concentration, times 100

This page titled 14.3: Relative Strengths of Acids and Bases is shared under a CC BY 4.0 license and was authored, remixed, and/or curated by
OpenStax via source content that was edited to the style and standards of the LibreTexts platform; a detailed edit history is available upon request.

Access for free at OpenStax 14.3.17 https://chem.libretexts.org/@go/page/38278


14.4: Hydrolysis of Salt Solutions
 Learning Objectives
Predict whether a salt solution will be acidic, basic, or neutral
Calculate the concentrations of the various species in a salt solution
Describe the process that causes solutions of certain metal ions to be acidic

As we have seen in the section on chemical reactions, when an acid and base are mixed, they undergo a neutralization reaction. The
word “neutralization” seems to imply that a stoichiometrically equivalent solution of an acid and a base would be neutral. This is
sometimes true, but the salts that are formed in these reactions may have acidic or basic properties of their own, as we shall now
see.

14.4.1: Acid-Base Neutralization


A solution is neutral when it contains equal concentrations of hydronium and hydroxide ions. When we mix solutions of an acid
and a base, an acid-base neutralization reaction occurs. However, even if we mix stoichiometrically equivalent quantities, we may
find that the resulting solution is not neutral. It could contain either an excess of hydronium ions or an excess of hydroxide ions
because the nature of the salt formed determines whether the solution is acidic, neutral, or basic. The following four situations
illustrate how solutions with various pH values can arise following a neutralization reaction using stoichiometrically equivalent
quantities:
1. A strong acid and a strong base, such as HCl(aq) and NaOH(aq) will react to form a neutral solution since the conjugate
partners produced are of negligible strength:

HCl(aq) + NaOH(aq) ⇌ NaCl(aq) + H O(l)


2

2. A strong acid and a weak base yield a weakly acidic solution, not because of the strong acid involved, but because of the
conjugate acid of the weak base.
3. A weak acid and a strong base yield a weakly basic solution. A solution of a weak acid reacts with a solution of a strong base to
form the conjugate base of the weak acid and the conjugate acid of the strong base. The conjugate acid of the strong base is a
weaker acid than water and has no effect on the acidity of the resulting solution. However, the conjugate base of the weak acid
is a weak base and ionizes slightly in water. This increases the amount of hydroxide ion in the solution produced in the reaction
and renders it slightly basic.
4. A weak acid plus a weak base can yield either an acidic, basic, or neutral solution. This is the most complex of the four types of
reactions. When the conjugate acid and the conjugate base are of unequal strengths, the solution can be either acidic or basic,
depending on the relative strengths of the two conjugates. Occasionally the weak acid and the weak base will have the same
strength, so their respective conjugate base and acid will have the same strength, and the solution will be neutral. To predict
whether a particular combination will be acidic, basic or neutral, tabulated K values of the conjugates must be compared.

14.4.2: Stomach Antacids


Our stomachs contain a solution of roughly 0.03 M HCl, which helps us digest the food we eat. The burning sensation associated
with heartburn is a result of the acid of the stomach leaking through the muscular valve at the top of the stomach into the lower
reaches of the esophagus. The lining of the esophagus is not protected from the corrosive effects of stomach acid the way the lining
of the stomach is, and the results can be very painful. When we have heartburn, it feels better if we reduce the excess acid in the
esophagus by taking an antacid. As you may have guessed, antacids are bases. One of the most common antacids is calcium
carbonate, CaCO3. The reaction,

C aC O3 (s) + 2H C l(aq) ⇌ C aC l2 (aq) + H2 O(l) + C O2 (g)

not only neutralizes stomach acid, it also produces CO2(g), which may result in a satisfying belch.
Milk of Magnesia is a suspension of the sparingly soluble base magnesium hydroxide, Mg(OH)2. It works according to the
reaction:
2+ −
M g(OH )2 (s) ⇌ M g (aq) + 2OH (aq)

Access for free at OpenStax 14.4.1 https://chem.libretexts.org/@go/page/38279


The hydroxide ions generated in this equilibrium then go on to react with the hydronium ions from the stomach acid, so that :
+ −
H3 O + OH ⇌ 2 H2 O(l)

This reaction does not produce carbon dioxide, but magnesium-containing antacids can have a laxative effect. Several antacids
have aluminum hydroxide, Al(OH)3, as an active ingredient. The aluminum hydroxide tends to cause constipation, and some
antacids use aluminum hydroxide in concert with magnesium hydroxide to balance the side effects of the two substances.

14.4.3: Culinary Aspects of Chemistry


Cooking is essentially synthetic chemistry that happens to be safe to eat. There are a number of examples of acid-base chemistry in
the culinary world. One example is the use of baking soda, or sodium bicarbonate in baking. NaHCO3 is a base. When it reacts
with an acid such as lemon juice, buttermilk, or sour cream in a batter, bubbles of carbon dioxide gas are formed from
decomposition of the resulting carbonic acid, and the batter “rises.” Baking powder is a combination of sodium bicarbonate, and
one or more acid salts that react when the two chemicals come in contact with water in the batter.
Many people like to put lemon juice or vinegar, both of which are acids, on cooked fish (Figure 14.4.1). It turns out that fish have
volatile amines (bases) in their systems, which are neutralized by the acids to yield involatile ammonium salts. This reduces the
odor of the fish, and also adds a “sour” taste that we seem to enjoy.
<div data-mt-source="1"

Figure 14.4.1 : A neutralization reaction takes place between citric acid in lemons or acetic acid in vinegar, and the bases in the
flesh of fish.
Pickling is a method used to preserve vegetables using a naturally produced acidic environment. The vegetable, such as a
cucumber, is placed in a sealed jar submerged in a brine solution. The brine solution favors the growth of beneficial bacteria and
suppresses the growth of harmful bacteria. The beneficial bacteria feed on starches in the cucumber and produce lactic acid as a
waste product in a process called fermentation. The lactic acid eventually increases the acidity of the brine to a level that kills any
harmful bacteria, which require a basic environment. Without the harmful bacteria consuming the cucumbers they are able to last
much longer than if they were unprotected. A byproduct of the pickling process changes the flavor of the vegetables with the acid
making them taste sour.

14.4.4: Salts of Weak Bases and Strong Acids


When we neutralize a weak base with a strong acid, the product is a salt containing the conjugate acid of the weak base. This
conjugate acid is a weak acid. For example, ammonium chloride, NH4Cl, is a salt formed by the reaction of the weak base
ammonia with the strong acid HCl:

NH (aq) + HCl(aq) ⟶ NH Cl(aq)


3 4

A solution of this salt contains ammonium ions and chloride ions. The chloride ion has no effect on the acidity of the solution since
HCl is a strong acid. Chloride is a very weak base and will not accept a proton to a measurable extent. However, the ammonium
ion, the conjugate acid of ammonia, reacts with water and increases the hydronium ion concentration:
+ +
NH (aq) + H O(l) ⇌ H O (aq) + NH (aq)
4 2 3 3

The equilibrium equation for this reaction is simply the ionization constant. Ka, for the acid NH : +
4

+
[H O ][ NH ]
3 3
= Ka
+
[ NH ]
4

We will not find a value of Ka for the ammonium ion in Table E1. However, it is not difficult to determine Ka for NH from the +
4

value of the ionization constant of water, Kw, and Kb, the ionization constant of its conjugate base, NH3, using the following
relationship:

Kw = Ka × Kb

This relation holds for any base and its conjugate acid or for any acid and its conjugate base.

Access for free at OpenStax 14.4.2 https://chem.libretexts.org/@go/page/38279


 Example 14.4.1: pH of a Solution of a Salt of a Weak Base and a Strong Acid

Aniline is an amine that is used to manufacture dyes. It is isolated as aniline hydrochloride, [C H NH ]Cl, a salt prepared by 6 5
+
3

the reaction of the weak base aniline and hydrochloric acid. What is the pH of a 0.233 M solution of aniline hydrochloride?
+ +
C H NH (aq) + H O(l) ⇌ H O (aq) + C H NH (aq)
6 5 3 2 3 6 5 2

Solution
The new step in this example is to determine Ka for the C H NH ion. The C H NH ion is the conjugate acid of a weak
6 5
+
3 6 5
+
3

base. The value of Ka for this acid is not listed in Table E1, but we can determine it from the value of Kb for aniline, C6H5NH2,
which is given as 4.6 × 10−10 :
+ −14
Ka (f or C6 H5 NH ) × Kb (f or C6 H5 NH2 ) = Kw = 1.0 × 10
3

−14
Kw 1.0 × 10
+ −5
Ka (f or C6 H5 NH ) = = = 2.3 × 10
3 −10
Kb (f or C6 H5 NH2 ) 4.6 × 10

Now we have the ionization constant and the initial concentration of the weak acid, the information necessary to determine the
equilibrium concentration of H3O+, and the pH:

Four tan rectangles are shown that are connected with right pointing arrows. The first is labeled “Determine the direction of
change.” The second is labeled “Determine x and the equilibrium concentrations.” The third is labeled “Solve for x and the
equilibrium concentrations.” The fourth is labeled “Check the math.”
With these steps we find [H3O+] = 2.3 × 10−3 M and pH = 2.64

 Exercise 14.4.1
a. Do the calculations and show that the hydronium ion concentration for a 0.233-M solution of C H NH is 2.3 × 10−3 and 6 5
+
3

the pH is 2.64.
b. What is the hydronium ion concentration in a 0.100-M solution of ammonium nitrate, NH4NO3, a salt composed of the ions
NH and NO . Use the data in Table E1 to determine Kb for the ammonium ion. Which is the stronger acid C H NH or
+

4

3 6 5
+
3

NH ?
+

Answer a
Ka (for NH
+
4
−10
) = 5.6 × 10 , [H3O+] = 7.5 × 10−6 M
Answer b
C H NH
6 5
+

3
is the stronger acid (a) (b) .

14.4.5: Salts of Weak Acids and Strong Bases


When we neutralize a weak acid with a strong base, we get a salt that contains the conjugate base of the weak acid. This conjugate
base is usually a weak base. For example, sodium acetate, NaCH3CO2, is a salt formed by the reaction of the weak acid acetic acid
with the strong base sodium hydroxide:

CH CO H(aq) + NaOH(aq) ⟶ NaCH CO (aq) + H O(aq)


3 2 3 2 2

A solution of this salt contains sodium ions and acetate ions. The sodium ion has no effect on the acidity of the solution. However,
the acetate ion, the conjugate base of acetic acid, reacts with water and increases the concentration of hydroxide ion:

Access for free at OpenStax 14.4.3 https://chem.libretexts.org/@go/page/38279


− −
CH CO (aq) + H O(l) ⇌ CH CO H(aq) + OH (aq)
3 2 2 3 2

The equilibrium equation for this reaction is the ionization constant, Kb, for the base CH CO . The value of Kb can be calculated
3

from the value of the ionization constant of water, Kw, and Ka, the ionization constant of the conjugate acid of the anion using the
equation:

w = Ka × Kb

For the acetate ion and its conjugate acid we have:


−14

Kw 1.0 × 10
−10
Kb (f or CH CO ) = = = 5.6 × 10
3 2 −5
Ka (f or C H3 C O2 H) 1.8 × 10

Some handbooks do not report values of Kb. They only report ionization constants for acids. If we want to determine a Kb value
using one of these handbooks, we must look up the value of Ka for the conjugate acid and convert it to a Kb value.

 Example 14.4.2: Equilibrium of a Salt of a Weak Acid and a Strong Base

Determine the acetic acid concentration in a solution with [ CH CO


3

2
] = 0.050 M and [OH−] = 2.5 × 10−6 M at equilibrium.
The reaction is:
− −
CH CO (aq) + H O(l) ⇌ CH CO H(aq) + OH (aq)
3 2 2 3 2

Solution
We are given two of three equilibrium concentrations and asked to find the missing concentration. If we can find the
equilibrium constant for the reaction, the process is straightforward.
The acetate ion behaves as a base in this reaction; hydroxide ions are a product. We determine Kb as follows:
−14

Kw 1.0 × 10
−10
Kb (f or CH CO ) = = = 5.6 × 10
3 2 −5
Ka (f or C H3 C O2 H) 1.8 × 10

Now find the missing concentration:



[ CH CO H][ OH ]
3 2 −10
Kb = = 5.6 × 10

[ CH CO ]
3 2

−6
[ CH CO H](2.5 × 10 )
3 2 −10
= = 5.6 × 10
(0.050)

Solving this equation we get [CH3CO2H] = 1.1 × 10−5 M.

 Exercise 14.4.2

What is the pH of a 0.083-M solution of CN−? Use 4.9 × 10−10 as Ka for HCN. Hint: We will probably need to convert pOH to
pH or find [H3O+] using [OH−] in the final stages of this problem.

Answer
11.16

14.4.6: Equilibrium in a Solution of a Salt of a Weak Acid and a Weak Base


In a solution of a salt formed by the reaction of a weak acid and a weak base, to predict the pH, we must know both the Ka of the
weak acid and the Kb of the weak base. If Ka > Kb, the solution is acidic, and if Kb > Ka, the solution is basic.

Access for free at OpenStax 14.4.4 https://chem.libretexts.org/@go/page/38279


 Example 14.4.3: Determining the Acidic or Basic Nature of Salts

Determine whether aqueous solutions of the following salts are acidic, basic, or neutral:
a. KBr
b. NaHCO3
c. NH4Cl
d. Na2HPO4
e. NH4F

Solution
Consider each of the ions separately in terms of its effect on the pH of the solution, as shown here:
a. The K+ cation and the Br− anion are both spectators, since they are the cation of a strong base (KOH) and the anion of a
strong acid (HBr), respectively. The solution is neutral.
b. The Na+ cation is a spectator, and will not affect the pH of the solution; the HCO anion is amphiprotic.The Ka of HCO

3

3
−14
1.0 × 10
−11
is 4.7 × 10 , so the Kb of its conjugate base is −11
= 2.1 × 10
−4
. Since Kb >> Ka, the solution is basic.
4.7 × 10

c. The NH ion is acidic and the Cl ion is a spectator. The solution will be acidic.
+

d. The Na+ ion is a spectator and will not affect the pH of the solution, while the HPO 2 −
4
ion is amphiprotic. The Ka of
−14
1.0 × 10
HPO
2 −

4
is 4.2 × 10−13 and its Kb is −13
= 2.4 × 10
−2
. Because Kb >> Ka, the solution is basic.
4.2 × 10

e. The NH ion is listed as being acidic, and the F ion is listed as a base, so we must directly compare the Ka and the Kb of
+
4

the two ions. Ka of NH is 5.6 × 10−10, which seems very small, yet the Kb of F− is 1.4 × 10−11, so the solution is acidic,
+
4

since Ka > Kb.

 Exercise 14.4.3

Determine whether aqueous solutions of the following salts are acidic, basic, or neutral:
a. K2CO3
b. CaCl2
c. KH2PO4
d. (NH4)2CO3
e. AlBr3

Answer a
basic
Answer b
neutral
Answer c
acidic
Answer d
basic
Answer e
acidic

14.4.7: The Ionization of Hydrated Metal Ions


If we measure the pH of the solutions of a variety of metal ions we will find that these ions act as weak acids when in solution. The
aluminum ion is an example. When aluminum nitrate dissolves in water, the aluminum ion reacts with water to give a hydrated

Access for free at OpenStax 14.4.5 https://chem.libretexts.org/@go/page/38279


aluminum ion, Al(H O) , dissolved in bulk water. What this means is that the aluminum ion has the strongest interactions with
2
3+

the six closest water molecules (the so-called first solvation shell), even though it does interact with the other water molecules
surrounding this Al(H O) cluster as well:
2
3+

3+ −
Al (NO ) (s) + 6 H O(l) ⟶ Al (H O) (aq) + 3 NO (aq)
3 3 2 2 6 3

We frequently see the formula of this ion simply as “Al3+(aq)”, without explicitly noting the six water molecules that are the closest
ones to the aluminum ion and just describing the ion as being solvated in water (hydrated). This is similar to the simplification of
the formula of the hydronium ion, H3O+ to H+. However, in this case, the hydrated aluminum ion is a weak acid (Figure 14.4.2)
and donates a proton to a water molecule. Thus, the hydration becomes important and we may use formulas that show the extent of
hydration:
3+ + 2+ −5
Al (H O) (aq) + H O(l) ⇌ H O (aq) + Al (H O) (OH) (aq) Ka = 1.4 × 10
2 6 2 3 2 5

As with other polyprotic acids, the hydrated aluminum ion ionizes in stages, as shown by:
3+ + 2+
Al (H O) (aq) + H O(l) ⇌ H O (aq) + Al (H O) (OH) (aq)
2 6 2 3 2 5

2+ + +
Al (H O) (OH) (aq) + H O(l) ⇌ H O (aq) + Al (H O) (OH) (aq)
2 5 2 3 2 4 2

+ +
Al (H O) (OH) 2 (aq) + H O(l) ⇌ H O (aq) + Al (H O) (OH) (aq)
2 4 2 3 2 3 3

Note that some of these aluminum species are exhibiting amphiprotic behavior, since they are acting as acids when they appear on
the left side of the equilibrium expressions and as bases when they appear on the right side.

Figure 14.4.2 : When an aluminum ion reacts with water, the hydrated aluminum ion becomes a weak acid.
A reaction is shown using ball and stick models. On the left, inside brackets with a superscript of 3 plus outside to the right is
structure labeled “[ A l ( H subscript 2 O ) subscript 6 ] superscript 3 plus.” Inside the brackets is s central grey atom to which 6 red
atoms are bonded in an arrangement that distributes them evenly about the central grey atom. Each red atom has two smaller white
atoms attached in a forked or bent arrangement. Outside the brackets to the right is a space-filling model that includes a red central
sphere with two smaller white spheres attached in a bent arrangement. Beneath this structure is the label “H subscript 2 O.” A
double sided arrow follows. Another set of brackets follows to the right of the arrows which have a superscript of two plus outside
to the right. The structure inside the brackets is similar to that on the left, except a white atom is removed from the structure. The
label below is also changed to “[ A l ( H subscript 2 O ) subscript 5 O H ] superscript 2 plus.” To the right of this structure and
outside the brackets is a space filling model with a central red sphere to which 3 smaller white spheres are attached. This structure
is labeled “H subscript 3 O superscript plus.”
However, the ionization of a cation carrying more than one charge is usually not extensive beyond the first stage. Additional
examples of the first stage in the ionization of hydrated metal ions are:
3+ + 2+
Fe(H O) (aq) + H O(l) ⇌ H O (aq) + Fe(H O) (OH) (aq) Ka = 2.74
2 6 2 3 2 5

2+ + +
Cu(H O) (aq) + H O(l) ⇌ H O (aq) + Cu(H O) (OH) (aq) Ka =  6.3
2 6 2 3 2 5

2+ + +
Zn(H O) (aq) + H O(l) ⇌ H O (aq) + Zn(H O) (OH) (aq) Ka = 9.6
2 4 2 3 2 3

 Example 14.4.4: Hydrolysis of [Al(H2O)6]3+


Calculate the pH of a 0.10-M solution of aluminum chloride, which dissolves completely to give the hydrated aluminum ion
3+
[Al (H O) ]
2 6
in solution.

Solution

Access for free at OpenStax 14.4.6 https://chem.libretexts.org/@go/page/38279


In spite of the unusual appearance of the acid, this is a typical acid ionization problem.

A reaction is shown using ball and stick models. On the left, inside brackets with a superscript of 3 plus outside to the right is
structure labeled “[ A l ( H subscript 2 O ) subscript 6 ] superscript 3 plus.” Inside the brackets is s central grey atom to which
6 red atoms are bonded in an arrangement that distributes them evenly about the central grey atom. Each red atom has two
smaller white atoms attached in a forked or bent arrangement. Outside the brackets to the right is a space-filling model that
includes a red central sphere with two smaller white spheres attached in a bent arrangement. Beneath this structure is the label
“H subscript 2 O.” A double sided arrow follows. Another set of brackets follows to the right of the arrows which have a
superscript of two plus outside to the right. The structure inside the brackets is similar to that on the left, except a white atom is
removed from the structure. The label below is also changed to “[ A l ( H subscript 2 O ) subscript 5 O H ] superscript 2 plus.”
To the right of this structure and outside the brackets is a space filling model with a central red sphere to which 3 smaller white
spheres are attached. This structure is labeled “H subscript 3 O superscript plus.”
1. Determine the direction of change. The equation for the reaction and Ka are:
3+ + 2+ −5
Al (H O) (aq) + H O(l) ⇌ H O (aq) + Al (H O) (OH) (aq) Ka = 1.4 × 10
2 6 2 3 2 5

The reaction shifts to the right to reach equilibrium.


2. Determine x and equilibrium concentrations. Use the table
:

This table has two main columns and four rows. The first row for the first column does not have a heading and then has the
following in the first column: Initial concentration ( M ), Change ( M ), Equilibrium concentration ( M ). The second column
has the header of “A l ( H subscript 2 O ) subscript 6 superscript 3 positive sign plus H subscript 2 O equilibrium arrow H
subscript 3 O superscript positive sign plus A l ( H subscript 2 O ) subscript 5 ( O H ) superscript 2 positive sign.” Under the
second column is a subgroup of four columns and three rows. The first column has the following: 0.10 (which appears in red),
negative x, 0.10 minus x. The second column is blank. The third column has the following: approximately 0, x, x. The fourth
column has the following: 0, x, x.
Solve for x and the equilibrium concentrations. Substituting the expressions for the equilibrium concentrations into the
equation for the ionization constant yields:
+ 2+
[H O ][Al (H O) (OH) ]
3 2 5
1. Ka =
3+
[Al (H O) ]
2 6

(x)(x)
−5
= = 1.4 × 10
0.10 − x

Solving this equation gives:


−3
x = 1.2 × 10 M

From this we find:


+ −3
[H O ] = 0 + x = 1.2 × 10 M
3

+
pH = −log[ H3 O ] = 2.92(an acidic solution)

−3
Check the work. The arithmetic checks; when 1.2 × 10 M is substituted for x, the result = Ka.

Access for free at OpenStax 14.4.7 https://chem.libretexts.org/@go/page/38279


 Exercise 14.4.4

in a 0.15-M solution of Al(NO3)3 that contains enough of the strong acid HNO3 to bring [H3O+]
2+
What is [Al(H 2
O) (OH)
5
]

to 0.10 M?

Answer
2.1 × 10−5 M

The constants for the different stages of ionization are not known for many metal ions, so we cannot calculate the extent of their
ionization. However, practically all hydrated metal ions other than those of the alkali metals ionize to give acidic solutions.
Ionization increases as the charge of the metal ion increases or as the size of the metal ion decreases.

Summary
The characteristic properties of aqueous solutions of Brønsted-Lowry acids are due to the presence of hydronium ions; those of
aqueous solutions of Brønsted-Lowry bases are due to the presence of hydroxide ions. The neutralization that occurs when aqueous
solutions of acids and bases are combined results from the reaction of the hydronium and hydroxide ions to form water. Some salts
formed in neutralization reactions may make the product solutions slightly acidic or slightly basic. Solutions that contain salts or
hydrated metal ions have a pH that is determined by the extent of the hydrolysis of the ions in the solution. The pH of the solutions
may be calculated using familiar equilibrium techniques, or it may be qualitatively determined to be acidic, basic, or neutral
depending on the relative Ka and Kb of the ions involved.

This page titled 14.4: Hydrolysis of Salt Solutions is shared under a CC BY 4.0 license and was authored, remixed, and/or curated by OpenStax
via source content that was edited to the style and standards of the LibreTexts platform; a detailed edit history is available upon request.

Access for free at OpenStax 14.4.8 https://chem.libretexts.org/@go/page/38279


14.5: Polyprotic Acids
 Learning Objectives
Extend previously introduced equilibrium concepts to acids and bases that may donate or accept more than one proton

We can classify acids by the number of protons per molecule that they can give up in a reaction. Acids such as HCl, HNO , and 3

HCN that contain one ionizable hydrogen atom in each molecule are called monoprotic acids. Their reactions with water are:

+ −
HCl(aq) + H O(l) ⟶ H O (aq) + Cl (aq)
2 3

+ −
HNO (aq) + H O(l) ⟶ H O (aq) + NO3 (aq)
3 2 3

+ −
HCN(aq) + H O(l) ⟶ H O (aq) + CN (aq)
2 3

Even though it contains four hydrogen atoms, acetic acid, CH CO H


3 2
, is also monoprotic because only the hydrogen atom from
the carboxyl group (−COOH ) reacts with bases:

This image contains two equilibrium reactions. The first shows a C atom bonded to three H atoms and another C atom. The second
C atom is double bonded to an O atom and also forms a single bond to another O atom. The second O atom is bonded to an H atom.
There is a plus sign and then the molecular formula H subscript 2 O. An equilibrium arrow follows the H subscript 2 O. To the
right of the arrow is H subscript 3 O superscript positive sign. There is a plus sign. The final structure shows a C atom bonded the
three H atoms and another C atom. This second C atom is double bonded to an O atom and single bonded to another O atom. The
entire structure is in brackets and a superscript negative sign appears outside the brackets. The second reaction shows C H subscript
3 C O O H ( a q ) plus H subscript 2 O ( l ) equilibrium arrow H subscript 3 O ( a q ) plus C H subscript 3 C O O superscript
negative sign ( a q ).
Similarly, monoprotic bases are bases that will accept a single proton.

14.5.1: Diprotic Acids


Diprotic acids contain two ionizable hydrogen atoms per molecule; ionization of such acids occurs in two steps. The first ionization
always takes place to a greater extent than the second ionization. For example, sulfuric acid, a strong acid, ionizes as follows:
The first ionization is
+ −
H SO (aq) + H O(l) ⇌ H O (aq) + HSO (aq)
2 4 2 3 4

with K a1
2
> 10 ; complete dissociation .
The second ionization is
− + 2 −
HSO (aq) + H O(l) ⇌ H O (aq) + SO (aq)
4 2 3 4

with K a2 = 1.2 × 10
−2
.
This stepwise ionization process occurs for all polyprotic acids. When we make a solution of a weak diprotic acid, we get a solution
that contains a mixture of acids. Carbonic acid, H CO , is an example of a weak diprotic acid. The first ionization of carbonic acid
2 3

yields hydronium ions and bicarbonate ions in small amounts.


First Ionization
+ −
H CO (aq) + H O(l) ⇌ H O (aq) + HCO (aq)
2 3 2 3 3

with

Access for free at OpenStax 14.5.1 https://chem.libretexts.org/@go/page/38280


+ −
[H O ][ HCO ]
3 3 −7
KH CO = = 4.3 × 10
2 3
[ H CO ]
2 3

The bicarbonate ion can also act as an acid. It ionizes and forms hydronium ions and carbonate ions in even smaller quantities.
Second Ionization
− + 2−
HCO (aq) + H O(l) ⇌ H O (aq) + CO (aq)
3 2 3 3

with
+ 2−
[H O ][ CO ]
3 3 −11
KHCO− = = 4.7 × 10
3 −
[ HCO3 ]

KH
2
CO
3
is larger than K by a factor of 104, so H2CO3 is the dominant producer of hydronium ion in the solution. This means
HCO

that little of the HCO formed by the ionization of H2CO3 ionizes to give hydronium ions (and carbonate ions), and the

concentrations of H3O+ and HCO are practically equal in a pure aqueous solution of H2CO3.

If the first ionization constant of a weak diprotic acid is larger than the second by a factor of at least 20, it is appropriate to treat the
first ionization separately and calculate concentrations resulting from it before calculating concentrations of species resulting from
subsequent ionization. This can simplify our work considerably because we can determine the concentration of H3O+ and the
conjugate base from the first ionization, then determine the concentration of the conjugate base of the second ionization in a
solution with concentrations determined by the first ionization.

 Example 14.5.1: Ionization of a Diprotic Acid

When we buy soda water (carbonated water), we are buying a solution of carbon dioxide in water. The solution is acidic
because CO2 reacts with water to form carbonic acid, H2CO3. What are [H O ], [HCO ], and [CO ] in a saturated solution 3
+ −
3
2−
3

of CO2 with an initial [H2CO3] = 0.033 M?


+ − −7
H CO (aq) + H O(l) ⇌ H O (aq) + HCO (aq) Ka1 = 4.3 × 10 (equilibrium step 1)
2 3 2 3 3

− + 2− −11
HCO (aq) + H O(l) ⇌ H O (aq) + CO (aq) Ka2 = 4.7 × 10 (equilibrium step 2)
3 2 3 3

Solution
As indicated by the ionization constants, H2CO3 is a much stronger acid than HCO , so H −
3 2
CO
3
is the dominant producer of
hydronium ion in solution. Thus there are two parts in the solution of this problem:
1. Using the customary four steps, we determine the concentration of H3O+ and HCO produced by ionization of H2CO3. −

2. Then we determine the concentration of CO in a solution with the concentration of H3O+ and HCO determined in (1).
2−
3

3

To summarize:

Four tan rectangles are shown that are connected with right pointing arrows. The first is labeled “left bracket H subscript 2 C O
subscript 3 right bracket.” The second is labeled “left bracket H subscript 3 O superscript plus right bracket and left bracket H
C O subscript 3 superscript negative right bracket from H subscript 2 C O subscript 3.” The third is labeled “left bracket C O
subscript 3 superscript 2 negative right bracket from H C O subscript 3 superscript negative.”
1. First Ionization: Determine the concentrations of H 3
O
+
and HCO . −
3

Since equilibrium step 1 is has a much bigger K = 4.3 × 10 than K = 4.7 × 10


a1 for equilibrium step 2 , we can
−7
a2
−11

safely ignore the second ionization step and focus only on the first step (but address it in next part of problem).
+ − −7
H CO (aq) + H O(l) ⇌ H O (aq) + HCO3 (aq) Ka1 = 4.3 × 10
2 3 2 3

Access for free at OpenStax 14.5.2 https://chem.libretexts.org/@go/page/38280


As for the ionization of any other weak acid:

Four tan rectangles are shown that are connected with right pointing arrows. The first is labeled “Determine the direction of
change.” The second is labeled “Determine x and the equilibrium concentrations.” The third is labeled “Solve for x and the
equilibrium concentrations.” The fourth is labeled “Check the math.”
An abbreviated table of changes and concentrations shows:
Abbreviated table of changes and concentrations
ICE Table H CO
2 3
(aq) H O(l)
2
H O
3
+
(aq) HCO

3
(aq)

Initial (M) 0.033 M - 0 0

Change (M) −x - +x +x

Equilibrium (M) 0.033 M − x - x x

Substituting the equilibrium concentrations into the equilibrium constant gives us:
+ −
[H O ][ HCO ] (x)(x)
3 3 −7
KH CO = = = 4.3 × 10
2 3
[ H CO ] 0.033 − x
2 3

Solving the preceding equation making our standard assumptions gives:


−4
x = 1.2 × 10

Thus:

[ H CO ] = 0.033 M
2 3

+ − −4
[H O ] = [ HCO ] = 1.2 × 10 M
3 3

2. Second Ionization: Determine the concentration of C O 2−

3
in a solution at equilibrium.
Since the equilibrium step 1 is has a much bigger K than equilibrium step 2 , we can the equilibrium conditions calculated
a

from first part of example as the initial conditions for an ICER Table for the equilibrium step 2 :
− + 2−
HCO (aq) + H O(l) ⇌ H O (aq) + CO (aq)
3 2 3 3

ICER Table for the equilibrium step 2:


ICE Table HCO

3
(aq) H O(l)
2
H O
3
+
(aq) CO
2−
3
(aq)

Initial (M) 1.2 × 10


−4
M - 1.2 × 10
−4
M 0

Change (M) −y - +y +y

Equilibrium (M) 1.2 × 10


−4
M −y - 1.2 × 10
−4
M +y y

+ 2−
[H O ][ CO ]
3 3
KHCO− =
3 −
[ HCO3 ]

−4
(1.2 × 10 M + y)(y)
=
−4
(1.2 × 10 M − y)

To avoid solving a quadratic equation, we can assume y ≪ 1.2 × 10 −4


M so
−4
(1.2 × 10 M )(y)
−11
KHCO− = 4.7 × 10 ≈
3 −4
(1.2 × 10 M)

Access for free at OpenStax 14.5.3 https://chem.libretexts.org/@go/page/38280


Rearranging to solve for y
−11 −4
(4.7 × 10 )(1.2 × 10 M)
y ≈
−4
1.2 × 10 M

2− −11
[ CO ] = y ≈ 4.7 × 10
3

To summarize:
In part 1 of this example, we found that the H 2
CO
3
in a 0.033-M solution ionizes slightly and at equilibrium
[ H CO ] = 0.033 M, , and M . In part 2, we determined that
+ −4 − −4
[ H O ] = 1.2 × 10 [ HCO3 ] = 1.2 × 10
2 3 3

M .
2− −11
[ CO ] = 5.6 × 10
3

 Exercise 14.5.2: Hydrogen Sulfide


The concentration of H S in a saturated aqueous solution at room temperature is approximately 0.1 M. Calculate
2 [H O
3
+
,
]

[ HS ], and [ S ] in the solution:


− 2 −

+ − −8
H S(aq) + H O(l) ⇌ H O (aq) + HS (aq) Ka1 = 8.9 × 10
2 2 3

− + 2− −19
HS (aq) + H O(l) ⇌ H O (aq) + S (aq) Ka2 = 1.0 × 10
2 3

Answer
[ H S] = 0.1M
2
, [H 3
O
+
] = [H S

] = 0.0001 M [ S , 2−
] = 1 × 10
−19
M

We note that the concentration of the sulfide ion is the same as Ka2. This is due to the fact that each subsequent dissociation
occurs to a lesser degree (as acid gets weaker).

14.5.2: Triprotic Acids


A triprotic acid is an acid that has three dissociable protons that undergo stepwise ionization: Phosphoric acid is a typical example:
The first ionization is
+ −
H PO (aq) + H O(l) ⇌ H O (aq) + H PO (aq)
3 4 2 3 2 4

with K a1 = 7.5 × 10
−3
.
The second ionization is
− + 2−
H PO (aq) + H O(l) ⇌ H O (aq) + HPO (aq)
2 4 2 3 4

with K a2 = 6.2 × 10
−8
.
The third ionization is
2− + 3−
HPO (aq) + H O(l) ⇌ H O (aq) + PO (aq)
4 2 3 4

with K a3 = 4.2 × 10
−13
.
As with the diprotic acids, the differences in the ionization constants of these reactions tell us that in each successive step the
degree of ionization is significantly weaker. This is a general characteristic of polyprotic acids and successive ionization constants
often differ by a factor of about 105 to 106. This set of three dissociation reactions may appear to make calculations of equilibrium
concentrations in a solution of H3PO4 complicated. However, because the successive ionization constants differ by a factor of 105
to 106, the calculations can be broken down into a series of parts similar to those for diprotic acids.
Polyprotic bases can accept more than one hydrogen ion in solution. The carbonate ion is an example of a diprotic base, since it can
accept up to two protons. Solutions of alkali metal carbonates are quite alkaline, due to the reactions:
2− − −
H O(l) + CO (aq) ⇌ HCO (aq) + OH (aq)
2 3 3

and

Access for free at OpenStax 14.5.4 https://chem.libretexts.org/@go/page/38280


− −
H O(l) + HCO (aq) ⇌ H CO (aq) + OH (aq)
2 3 2 3

Summary
An acid that contains more than one ionizable proton is a polyprotic acid. The protons of these acids ionize in steps. The
differences in the acid ionization constants for the successive ionizations of the protons in a polyprotic acid usually vary by roughly
five orders of magnitude. As long as the difference between the successive values of Ka of the acid is greater than about a factor of
20, it is appropriate to break down the calculations of the concentrations of the ions in solution into a series of steps.

Glossary
diprotic acid
acid containing two ionizable hydrogen atoms per molecule. A diprotic acid ionizes in two steps

diprotic base
base capable of accepting two protons. The protons are accepted in two steps

monoprotic acid
acid containing one ionizable hydrogen atom per molecule

stepwise ionization
process in which an acid is ionized by losing protons sequentially

triprotic acid
acid that contains three ionizable hydrogen atoms per molecule; ionization of triprotic acids occurs in three steps

This page titled 14.5: Polyprotic Acids is shared under a CC BY 4.0 license and was authored, remixed, and/or curated by OpenStax via source
content that was edited to the style and standards of the LibreTexts platform; a detailed edit history is available upon request.

Access for free at OpenStax 14.5.5 https://chem.libretexts.org/@go/page/38280


14.6: Buffers
 Learning Objectives
Describe the composition and function of acid–base buffers
Calculate the pH of a buffer before and after the addition of added acid or base

A mixture of a weak acid and its conjugate base (or a mixture of a weak base and its conjugate acid) is called a buffer solution, or a
buffer. Buffer solutions resist a change in pH when small amounts of a strong acid or a strong base are added (Figure 14.6.1). A
solution of acetic acid (CH COOH and sodium acetate CH COONa) is an example of a buffer that consists of a weak acid and
3 3

its salt. An example of a buffer that consists of a weak base and its salt is a solution of ammonia (NH (aq)) and ammonium 3

chloride (NH Cl(aq)).


4

Figure 14.6.1 : (a) The unbuffered solution on the left and the buffered solution on the right have the same pH (pH 8); they are
basic, showing the yellow color of the indicator methyl orange at this pH. (b) After the addition of 1 mL of a 0.01-M HCl solution,
the buffered solution has not detectably changed its pH but the unbuffered solution has become acidic, as indicated by the change
in color of the methyl orange, which turns red at a pH of about 4. (credit: modification of work by Mark Ott)

14.6.1: How Buffers Work


A mixture of acetic acid and sodium acetate is acidic because the Ka of acetic acid is greater than the Kb of its conjugate base
acetate. It is a buffer because it contains both the weak acid and its salt. Hence, it acts to keep the hydronium ion concentration (and
the pH) almost constant by the addition of either a small amount of a strong acid or a strong base. If we add a base such as sodium
hydroxide, the hydroxide ions react with the few hydronium ions present. Then more of the acetic acid reacts with water, restoring
the hydronium ion concentration almost to its original value:
+ −
CH CO H(aq) + H O(l) ⟶ H O (aq) + CH CO (aq)
3 2 2 3 3 2

The pH changes very little. If we add an acid such as hydrochloric acid, most of the hydronium ions from the hydrochloric acid
combine with acetate ions, forming acetic acid molecules:
+ −
H O (aq) + CH CO (aq) ⟶ CH CO H(aq) + H O(l)
3 3 2 3 2 2

Thus, there is very little increase in the concentration of the hydronium ion, and the pH remains practically unchanged (Figure
14.6.2).

Access for free at OpenStax 14.6.1 https://chem.libretexts.org/@go/page/38281


Figure 14.6.2 : This diagram shows the buffer action of these reactions.
A mixture of ammonia and ammonium chloride is basic because the Kb for ammonia is greater than the Ka for the ammonium ion.
It is a buffer because it also contains the salt of the weak base. If we add a base (hydroxide ions), ammonium ions in the buffer
react with the hydroxide ions to form ammonia and water and reduce the hydroxide ion concentration almost to its original value:
+ −
NH (aq) + OH (aq) ⟶ NH (aq) + H O(l)
4 3 2

If we add an acid (hydronium ions), ammonia molecules in the buffer mixture react with the hydronium ions to form ammonium
ions and reduce the hydronium ion concentration almost to its original value:
+ +
H O (aq) + NH (aq) ⟶ NH (aq) + H O(l)
3 3 4 2

The three parts of the following example illustrate the change in pH that accompanies the addition of base to a buffered solution of
a weak acid and to an unbuffered solution of a strong acid.

 Example 14.6.1: pH Changes in Buffered and Unbuffered Solutions

Acetate buffers are used in biochemical studies of enzymes and other chemical components of cells to prevent pH changes that
might change the biochemical activity of these compounds.
a. Calculate the pH of an acetate buffer that is a mixture with 0.10 M acetic acid and 0.10 M sodium acetate.
b. Calculate the pH after 1.0 mL of 0.10 M NaOH is added to 100 mL of this buffer, giving a solution with a volume of 101
mL.

Solution
a. Calculate the pH of an acetate buffer that is a mixture with 0.10 M acetic acid and 0.10 M sodium acetate.
To determine the pH of the buffer solution we use a typical equilibrium calculation (as illustrated in earlier Examples):

1. Determine the direction of change. The equilibrium in a mixture of H3O+, CH 3



CO
2
, and CH3CO2H is:
+ −
CH CO H(aq) + H O(l) ⇌ H O (aq) + CH CO (aq)
3 2 2 3 3 2

Access for free at OpenStax 14.6.2 https://chem.libretexts.org/@go/page/38281


The equilibrium constant for CH3CO2H is not given, so we look it up in Table E1: Ka = 1.8 × 10−5. With [CH3CO2H] =
+ +
[ CH CO ] = 0.10 M and [H3O ] = ~0 M, the reaction shifts to the right to form H3O .

3 2

2. Determine x and equilibrium concentrations. A table of changes and concentrations follows:

Solve for x and the equilibrium concentrations. We find:


−5
x = 1.8 × 10 M

and
+ −5
[H O ] = 0 + x = 1.8 × 10 M
3

Thus:
+ −5
pH = −log[ H3 O ] = −log(1.8 × 10 )

= 4.74

4. Check the work. If we calculate all calculated equilibrium concentrations, we find that the equilibrium value of the
reaction coefficient, Q = Ka.
(b) Calculate the pH after 1.0 mL of 0.10 M NaOH is added to 100 mL of this buffer, giving a solution with a volume of 101
mL.
First, we calculate the concentrations of an intermediate mixture resulting from the complete reaction between the acid in the
buffer and the added base. Then we determine the concentrations of the mixture at the new equilibrium:

1. Determine the moles of NaOH. One milliliter (0.0010 L) of 0.10 M NaOH contains:

0.10 mol NaOH


−4
0.0010 L × ( ) = 1.0 × 10 mol NaOH
1 L

2. Determine the moles of CH2CO2H. Before reaction, 0.100 L of the buffer solution contains:

0.100 mol C H3 C O2 H
−2
0.100 L × ( ) = 1.00 × 10 mol C H3 C O2 H
1 L

3. Solve for the amount of NaCH3CO2 produced. The 1.0 × 10−4 mol of NaOH neutralizes 1.0 × 10−4 mol of CH3CO2H,
leaving:

Access for free at OpenStax 14.6.3 https://chem.libretexts.org/@go/page/38281


−2 −2 −2
(1.0 × 10 ) − (0.01 × 10 ) = 0.99 × 10 mol C H3 C O2 H

and producing 1.0 × 10−4 mol of NaCH3CO2. This makes a total of:
[\mathrm{(1.0×10^{−2})+(0.01×10^{−2})=1.01×10^{−2}\:mol\:NaCH_3CO_2} \nonumber \]
4. Find the molarity of the products. After reaction, CH3CO2H and NaCH3CO2 are contained in 101 mL of the intermediate
solution, so:
−3
9.9 × 10 mol
[ CH CO H] = = 0.098 M
3 2
0.101 L

−2
1.01 × 10 mol
[ NaCH CO ] = = 0.100 M
3 2
0.101 L

Now we calculate the pH after the intermediate solution, which is 0.098 M in CH3CO2H and 0.100 M in NaCH3CO2, comes to
equilibrium. The calculation is very similar to that in part (a) of this example:

This series of calculations gives a pH = 4.75. Thus the addition of the base barely changes the pH of the solution.
(c) For comparison, calculate the pH after 1.0 mL of 0.10 M NaOH is added to 100 mL of a solution of an unbuffered solution
with a pH of 4.74 (a 1.8 × 10−5-M solution of HCl). The volume of the final solution is 101 mL.
Solution
This 1.8 × 10−5-M solution of HCl has the same hydronium ion concentration as the 0.10-M solution of acetic acid-sodium
acetate buffer described in part (a) of this example. The solution contains:
−5
1.8 × 10 mol HCl
−6
0.100 L × ( ) = 1.8 × 10 mol HCl
1 L

As shown in part (b), 1 mL of 0.10 M NaOH contains 1.0 × 10−4 mol of NaOH. When the NaOH and HCl solutions are mixed,
the HCl is the limiting reagent in the reaction. All of the HCl reacts, and the amount of NaOH that remains is:
−4 −6 −5
(1.0 × 10 ) − (1.8 × 10 ) = 9.8 × 10 M

The concentration of NaOH is:


−5
9.8 × 10 M NaOH −4
= 9.7 × 10 M
0.101 L

The pOH of this solution is:


− −4
pOH = −log[OH ] = −log(9.7 × 10 ) = 3.01

The pH is:
pH = 14.00 − pOH = 10.99

The pH changes from 4.74 to 10.99 in this unbuffered solution. This compares to the change of 4.74 to 4.75 that occurred when
the same amount of NaOH was added to the buffered solution described in part (b).

 Exercise 14.6.1

Show that adding 1.0 mL of 0.10 M HCl changes the pH of 100 mL of a 1.8 × 10−5 M HCl solution from 4.74 to 3.00.

Answer
Initial pH of 1.8 × 10−5 M HCl; pH = −log[H3O+] = −log[1.8 × 10−5] = 4.74

Access for free at OpenStax 14.6.4 https://chem.libretexts.org/@go/page/38281


Moles of H3O+ added by addition of 1.0 mL of 0.10 M HCl: 0.10 moles/L × 0.0010 L = 1.0 × 10−4 moles; final pH after
addition of 1.0 mL of 0.10 M HCl:

⎛ ⎞
+ −4 −6
+
total moles H3 O ⎜ 1.0 × 10 mol + 1.8 × 10 mol ⎟
pH = −log[ H3 O ] = −log ( ) = −log ⎜ ⎟ = 3.00
total volume ⎜ 1 L ⎟
101 mL ( )
⎝ ⎠
1000 mL

14.6.2: Buffer Capacity


Buffer solutions do not have an unlimited capacity to keep the pH relatively constant (Figure 14.6.3). If we add so much base to
a buffer that the weak acid is exhausted, no more buffering action toward the base is possible. On the other hand, if we add an
excess of acid, the weak base would be exhausted, and no more buffering action toward any additional acid would be possible.
In fact, we do not even need to exhaust all of the acid or base in a buffer to overwhelm it; its buffering action will diminish
rapidly as a given component nears depletion.

Figure 14.6.3 : The indicator color (methyl orange) shows that a small amount of acid added to a buffered solution of pH 8
(beaker on the left) has little affect on the buffered system (middle beaker). However, a large amount of acid exhausts the
buffering capacity of the solution and the pH changes dramatically (beaker on the right). (credit: modification of work by Mark
Ott)
The buffer capacity is the amount of acid or base that can be added to a given volume of a buffer solution before the pH changes
significantly, usually by one unit. Buffer capacity depends on the amounts of the weak acid and its conjugate base that are in a
buffer mixture. For example, 1 L of a solution that is 1.0 M in acetic acid and 1.0 M in sodium acetate has a greater buffer
capacity than 1 L of a solution that is 0.10 M in acetic acid and 0.10 M in sodium acetate even though both solutions have the
same pH. The first solution has more buffer capacity because it contains more acetic acid and acetate ion.

14.6.3: Selection of Suitable Buffer Mixtures


There are two useful rules of thumb for selecting buffer mixtures:
1. A good buffer mixture should have about equal concentrations of both of its components. A buffer solution has generally
lost its usefulness when one component of the buffer pair is less than about 10% of the other. Figure 14.6.4 shows an acetic
acid-acetate ion buffer as base is added. The initial pH is 4.74. A change of 1 pH unit occurs when the acetic acid
concentration is reduced to 11% of the acetate ion concentration.

Access for free at OpenStax 14.6.5 https://chem.libretexts.org/@go/page/38281


Figure 14.6.4 : The graph, an illustration of buffering action, shows change of pH as an increasing amount of a 0.10-M
NaOH solution is added to 100 mL of a buffer solution in which, initially, [CH CO H] = 0.10 M and 3 2

[CH CO ] = 0.10 M .

3 2

2. Weak acids and their salts are better as buffers for pHs less than 7; weak bases and their salts are better as buffers for pHs
greater than 7.
Blood is an important example of a buffered solution, with the principal acid and ion responsible for the buffering action being
carbonic acid, H2CO3, and the bicarbonate ion, HCO . When an excess of hydrogen ion enters the blood stream, it is removed

3

primarily by the reaction:


+ −
H O (aq) + HCO (aq) ⟶ H CO (aq) + H O(l)
3 3 2 3 2

When an excess of the hydroxide ion is present, it is removed by the reaction:


− −
OH (aq) + H CO (aq) ⟶ HCO (aq) + H O(l)
2 3 3 2

The pH of human blood thus remains very near 7.35, that is, slightly basic. Variations are usually less than 0.1 of a pH unit. A
change of 0.4 of a pH unit is likely to be fatal.

14.6.4: The Henderson-Hasselbalch Approximation


The ionization-constant expression for a solution of a weak acid can be written as:
+ −
[H O ][ A ]
3
Ka =
[HA]

Rearranging to solve for [H3O+], we get:

+
[HA]
[H O ] = Ka ×
3 −
[A ]

Taking the negative logarithm of both sides of this equation, we arrive at:
[HA]
+
−log[ H3 O ] = −log Ka − log

[A ]

which can be written as



[A ]
pH = pKa + log
[HA]

where pKa is the negative of the common logarithm of the ionization constant of the weak acid (pKa = −log Ka). This equation
relates the pH, the ionization constant of a weak acid, and the concentrations of the weak acid and its salt in a buffered solution.

Access for free at OpenStax 14.6.6 https://chem.libretexts.org/@go/page/38281


Scientists often use this expression, called the Henderson-Hasselbalch approximation, to calculate the pH of buffer solutions. It
is important to note that the “x is small” assumption must be valid to use this equation.

 Lawrence Joseph Henderson and Karl Albert Hasselbalch

Lawrence Joseph Henderson (1878–1942) was an American physician, biochemist and physiologist, to name only a few of
his many pursuits. He obtained a medical degree from Harvard and then spent 2 years studying in Strasbourg, then a part of
Germany, before returning to take a lecturer position at Harvard. He eventually became a professor at Harvard and worked
there his entire life. He discovered that the acid-base balance in human blood is regulated by a buffer system formed by the
dissolved carbon dioxide in blood. He wrote an equation in 1908 to describe the carbonic acid-carbonate buffer system in
blood. Henderson was broadly knowledgeable; in addition to his important research on the physiology of blood, he also
wrote on the adaptations of organisms and their fit with their environments, on sociology and on university education. He
also founded the Fatigue Laboratory, at the Harvard Business School, which examined human physiology with specific
focus on work in industry, exercise, and nutrition.
In 1916, Karl Albert Hasselbalch (1874–1962), a Danish physician and chemist, shared authorship in a paper with Christian
Bohr in 1904 that described the Bohr effect, which showed that the ability of hemoglobin in the blood to bind with oxygen
was inversely related to the acidity of the blood and the concentration of carbon dioxide. The pH scale was introduced in
1909 by another Dane, Sørensen, and in 1912, Hasselbalch published measurements of the pH of blood. In 1916,
Hasselbalch expressed Henderson’s equation in logarithmic terms, consistent with the logarithmic scale of pH, and thus the
Henderson-Hasselbalch equation was born.

 Medicine: The Buffer System in Blood


The normal pH of human blood is about 7.4. The carbonate buffer system in the blood uses the following equilibrium
reaction:
− +
CO (g) + 2 H O(l) ⇌ H CO (aq) ⇌ HCO (aq) + H O (aq)
2 2 2 3 3 3

The concentration of carbonic acid, H2CO3 is approximately 0.0012 M, and the concentration of the hydrogen carbonate
ion, HCO , is around 0.024 M. Using the Henderson-Hasselbalch equation and the pKa of carbonic acid at body

temperature, we can calculate the pH of blood:


[base] 0.024
pH = pKa + log = 6.1 + log = 7.4
[acid] 0.0012

The fact that the H2CO3 concentration is significantly lower than that of the HCO ion may seem unusual, but this

imbalance is due to the fact that most of the by-products of our metabolism that enter our bloodstream are acidic.
Therefore, there must be a larger proportion of base than acid, so that the capacity of the buffer will not be exceeded.
Lactic acid is produced in our muscles when we exercise. As the lactic acid enters the bloodstream, it is neutralized by the
HCO

3
ion, producing H2CO3. An enzyme then accelerates the breakdown of the excess carbonic acid to carbon dioxide
and water, which can be eliminated by breathing. In fact, in addition to the regulating effects of the carbonate buffering
system on the pH of blood, the body uses breathing to regulate blood pH. If the pH of the blood decreases too far, an
increase in breathing removes CO2 from the blood through the lungs driving the equilibrium reaction such that [H3O+] is
lowered. If the blood is too alkaline, a lower breath rate increases CO2 concentration in the blood, driving the equilibrium
reaction the other way, increasing [H+] and restoring an appropriate pH.

Summary
A solution containing a mixture of an acid and its conjugate base, or of a base and its conjugate acid, is called a buffer solution.
Unlike in the case of an acid, base, or salt solution, the hydronium ion concentration of a buffer solution does not change greatly
when a small amount of acid or base is added to the buffer solution. The base (or acid) in the buffer reacts with the added acid
(or base).

Access for free at OpenStax 14.6.7 https://chem.libretexts.org/@go/page/38281


14.6.5: Key Equations
pKa = −log Ka
pKb = −log Kb

[A ]
pH = pKa + log
[HA]

Glossary
buffer capacity
amount of an acid or base that can be added to a volume of a buffer solution before its pH changes significantly (usually by
one pH unit)

buffer
mixture of a weak acid or a weak base and the salt of its conjugate; the pH of a buffer resists change when small amounts of
acid or base are added

Henderson-Hasselbalch equation
equation used to calculate the pH of buffer solutions

This page titled 14.6: Buffers is shared under a CC BY 4.0 license and was authored, remixed, and/or curated by OpenStax via source content that
was edited to the style and standards of the LibreTexts platform; a detailed edit history is available upon request.

Access for free at OpenStax 14.6.8 https://chem.libretexts.org/@go/page/38281


14.7: Acid-Base Titrations
 Learning Objectives
Interpret titration curves for strong and weak acid-base systems
Compute sample pH at important stages of a titration
Explain the function of acid-base indicators

As seen in the chapter on the stoichiometry of chemical reactions, titrations can be used to quantitatively analyze solutions for their
acid or base concentrations. In this section, we will explore the changes in the concentrations of the acidic and basic species present
in a solution during the process of a titration.

14.7.1: Titration Curves


Previously, when we studied acid-base reactions in solution, we focused only on the point at which the acid and base were
stoichiometrically equivalent. No consideration was given to the pH of the solution before, during, or after the neutralization.

 Example 14.7.1: Calculating pH for Titration Solutions: Strong Acid/Strong Base


A titration is carried out for 25.00 mL of 0.100 M HCl (strong acid) with 0.100 M of a strong base NaOH the titration curve is
shown in Figure 14.7.1. Calculate the pH at these volumes of added base solution:
a. 0.00 mL
b. 12.50 mL
c. 25.00 mL
d. 37.50 mL

Solution
Since HCl is a strong acid, we can assume that all of it dissociates. The initial concentration of H3O+ is [H O ] = 0.100 M .
3
+

When the base solution is added, it also dissociates completely, providing OH− ions. The H3O+ and OH− ions neutralize each
other, so only those of the two that were in excess remain, and their concentration determines the pH. Thus, the solution is
initially acidic (pH < 7), but eventually all the hydronium ions present from the original acid are neutralized, and the solution
becomes neutral. As more base is added, the solution turns basic.
The total initial amount of the hydronium ions is:
+ +
n(H )0 = [ H3 O ]0 × 0.02500 L = 0.002500 mol

Once X mL of the 0.100-M base solution is added, the number of moles of the OH− ions introduced is:


1 L
n(OH )0 = 0.100 M × X mL × ( )
1000 mL

The total volume becomes:


1 L
V = (25.00 mL + X mL) ( )
1000 mL

The number of moles of H3O+ becomes:

+ + −
1 L
n(H ) = n(H )0 − n(OH )0 = 0.002500 mol − 0.100 M × X mL × ( )
1000 mL

The concentration of H3O+ is:

Access for free at OpenStax 14.7.1 https://chem.libretexts.org/@go/page/38282


1 L
0.002500 mol − 0.100 M × X mL × ( )
+
n(H ) 1000 mL
+
[ H3 O ] = =
V 1 L
(25.00 mL + X mL) ( )
1000 mL

1000 mL
0.002500 mol × ( ) − 0.100 M × X mL
1 L
=
25.00 mL + X mL

with the definition of pH :


+
pH = − log([ H3 O ]) (14.7.1)

The preceding calculations work if n(H ) − n(OH ) > 0 and so n(H+) > 0. When n(H ) = n(OH ) , the H3O+ ions from
+
0

0
+
0

0

the acid and the OH− ions from the base mutually neutralize. At this point, the only hydronium ions left are those from the
autoionization of water, and there are no OH− particles to neutralize them. Therefore, in this case:
+ − + −14 + −7
[H O ] = [ OH ], [ H O ] = Kw = 1.0 × 10 ; [H O ] = 1.0 × 10
3 3 3

−7
pH = −log(1.0 × 10 ) = 7.00

Finally, when n(OH



)0 > n(H
+
, there are not enough H3O+ ions to neutralize all the OH− ions, and instead of
)0

n(H
+
) = n(H
+
)0 − n(OH

)0 , we calculate: n(OH ) = n(OH ) − n(H ) − −
0
+
0

In this case:
1 L
0.100 M × X mL × ( ) − 0.002500 mol

n(OH ) 1000 mL

[OH ] = =
V 1 L
(25.00 mL + X mL) ( )
1000 mL

1000 mL
0.100 M × X mL − 0.002500 mol × ( )
1 L
=
25.00 mL + X mL

then using the definition of pOH and its relationship to pH in room temperature aqueous solutios (Equation 14.7.1):
pH = 14 − pOH

= 14 + log([OH ])

Let us now consider the four specific cases presented in this problem:
(a) X = 0 mL
1000 mL
0.002500 mol × ( )
+
n(H ) 1 L
+
[ H3 O ] = = = 0.1 M
V 25.00 mL

then using the definition of pH (Equation 14.7.1):


pH = − log(0.100)

= 1.000

(b) X = 12.50 mL
1000 mL
0.002500 mol × ( ) − 0.100 M × 12.50 mL
+
n(H ) 1 L
+
[ H3 O ] = = = 0.0333 M
V 25.00 mL + 12.50 mL

then using the definition of pH (Equation 14.7.1):


pH = − log(0.0333)

= 1.477

Access for free at OpenStax 14.7.2 https://chem.libretexts.org/@go/page/38282


(c) X = 25.00 mL
Since the volumes and concentrations of the acid and base solutions are the same:
+ −
n(H )0 = n(OH )0

and

pH = 7.000

as described earlier.
(d) X = 37.50 mL
In this case:
− +
n(OH )0 > n(H )0

1000 mL
0.100 M × 35.70 mL − 0.002500 mol × ( )

n(OH ) 1 L

[OH ] = = = 0.0200 M
V 25.00 mL + 37.50 mL

then using the definition of pH (Equation 14.7.1):


[pH = 14 − pOH

= 14 + log([OH ])

= 14 + log(0.0200)

= 12.30

 Exercise 14.7.1

Calculate the pH for the strong acid/strong base titration between 50.0 mL of 0.100 M HNO3(aq) and 0.200 M NaOH (titrant)
at the listed volumes of added base:
a. 0.00 mL,
b. 15.0 mL,
c. 25.0 mL, and
d. 40.0 mL.

Answer a
0.00: 1.000
Answer b
15.0: 1.5111
Answer c
25.0: 7e. Do not delete this text first.
Answer d
40.0: 12.523

In Example 14.7.1, we calculated pH at four points during a titration. Table 14.7.1 shows a detailed sequence of changes in the pH
of a strong acid and a weak acid in a titration with NaOH.
Table 14.7.1 : pH Values in the Titrations of a Strong Acid with a Strong Base and of a Weak Acid with a Strong Base
Volume of 0.100 M NaOH pH Values 0.100 M
Moles of NaOH Added pH Values 0.100 M HCl1 2
Added (mL) CH3 CO2 H

0.0 0.0 1.00 2.87

5.0 0.00050 1.18 4.14

Access for free at OpenStax 14.7.3 https://chem.libretexts.org/@go/page/38282


Volume of 0.100 M NaOH pH Values 0.100 M
Moles of NaOH Added pH Values 0.100 M HCl1 2
Added (mL) CH3 CO2 H

10.0 0.00100 1.37 4.57

15.0 0.00150 1.60 4.92

20.0 0.00200 1.95 5.35

22.0 0.00220 2.20 5.61

24.0 0.00240 2.69 6.13

24.5 0.00245 3.00 6.44

24.9 0.00249 3.70 7.14

25.0 0.00250 7.00 8.72

25.1 0.00251 10.30 10.30

25.5 0.00255 11.00 11.00

26.0 0.00260 11.29 11.29

28.0 0.00280 11.75 11.75

30.0 0.00300 11.96 11.96

35.0 0.00350 12.22 12.22

40.0 0.00400 12.36 12.36

45.0 0.00450 12.46 12.46

50.0 0.00500 12.52 12.52

1. Titration of 25.00 mL of 0.100 M HCl (0.00250 mol of HCI) with 0.100 M NaOH.
2. Titration of 25.00 mL of 0.100 M CH3CO2H (0.00250 mol of CH3CO2H) with 0.100 M NaOH.

The simplest acid-base reactions are those of a strong acid with a strong base. Table 14.7.1 shows data for the titration of a 25.0-
mL sample of 0.100 M hydrochloric acid with 0.100 M sodium hydroxide. The values of the pH measured after successive
additions of small amounts of NaOH are listed in the first column of this table, and are graphed in Figure 14.7.1, in a form that is
called a titration curve. The pH increases slowly at first, increases rapidly in the middle portion of the curve, and then increases
slowly again. The point of inflection (located at the midpoint of the vertical part of the curve) is the equivalence point for the
titration. It indicates when equivalent quantities of acid and base are present. For the titration of a strong acid with a strong base,
the equivalence point occurs at a pH of 7.00 and the points on the titration curve can be calculated using solution stoichiometry
(Table 14.7.1 and Figure 14.7.1).

Access for free at OpenStax 14.7.4 https://chem.libretexts.org/@go/page/38282


Figure 14.7.1 : (a) The titration curve for the titration of 25.00 mL of 0.100 M HCl (strong acid) with 0.100 M NaOH (strong base)
has an equivalence point of 7.00 pH. (b) The titration curve for the titration of 25.00 mL of 0.100 M CH3CO2H (weak acid) with
0.100 M NaOH (strong base) has an equivalence point of 8.72 pH.
Two graphs are shown. The first graph on the left is titled “Titration of Weak Acid.” The horizontal axis is labeled “Volume of
0.100 M N a O H added (m L).” Markings and vertical gridlines are provided every 5 units from 0 to 50. The vertical axis is
labeled “p H” and is marked every 1 unis beginning at 0 extending to 14. A red curve is drawn on the graph which increases
steadily from the point (0, 3) up to about (20, 5.5) after which the graph has a vertical section from (25, 7) up to (25, 11). The graph
then levels off to a value of about 12.5 from about 40 m L up to 50 m L. The midpoint of the vertical segment of the curve is
labeled “Equivalence point p H, 8.72.” The second graph on the right is titled “Titration of Strong Acid.” The horizontal axis is
labeled “Volume of 0.100 M N a O H added (m L).” Markings and vertical gridlines are provided every 5 units from 0 to 50. The
vertical axis is labeled “p H” and is marked every 1 units beginning at 0 extending to 14. A red curve is drawn on the graph which
increases gradually from the point (0, 1) up to about (22.5, 2.2) after which the graph has a vertical section from (25, 4) up to nearly
(25, 11). The graph then levels off to a value of about 12.4 from about 40 m L up to 50 m L. The midpoint of the vertical segment
of the curve is labeled “Equivalence point p H, 7.00.”
The titration of a weak acid with a strong base (or of a weak base with a strong acid) is somewhat more complicated than that just
discussed, but it follows the same general principles. Let us consider the titration of 25.0 mL of 0.100 M acetic acid (a weak acid)
with 0.100 M sodium hydroxide and compare the titration curve with that of the strong acid. Table 14.7.1 gives the pH values
during the titration, Figure 14.7.1b shows the titration curve.
Although the initial volume and molarity of the acids are the same, there are important differences between the two titration curves.
The titration curve for the weak acid begins at a higher value (less acidic) and maintains higher pH values up to the equivalence
point. This is because acetic acid is a weak acid, which is only partially ionized. The pH at the equivalence point is also higher
(8.72 rather than 7.00) due to the hydrolysis of acetate, a weak base that raises the pH:
− −
CH CO2 (aq) + H O(l) ⇌ CH CO H(l) + OH (aq)
3 2 3 2

After the equivalence point, the two curves are identical because the pH is dependent on the excess of hydroxide ion in both cases.

 Example 14.7.2: Titration of a Weak Acid with a Strong Base

The titration curve shown in Figure 14.7.1b is for the titration of 25.00 mL of 0.100 M CH3CO2H with 0.100 M NaOH. The
reaction can be represented as:
− −
CH CO H + OH ⟶ CH CO +H O
3 2 3 2 2

a. What is the initial pH before any amount of the NaOH solution has been added? Ka = 1.8 × 10−5 for CH3CO2H.
b. Find the pH after 25.00 mL of the NaOH solution have been added.
c. Find the pH after 12.50 mL of the NaOH solution has been added.
d. Find the pH after 37.50 mL of the NaOH solution has been added.

Access for free at OpenStax 14.7.5 https://chem.libretexts.org/@go/page/38282


Solution
(a) Assuming that the dissociated amount is small compared to 0.100 M, we find that:
+ − + 2
[H O ][ CH CO2 ] [H O ]
3 3 3
Ka = ≈
[ CH CO H] [ CH CO H]
3 2 3 2 0

and
−−−−−−−−−−−−− − − −−−−−−−−−−−−− −
+ −5 −3
[H O ] = √ Ka × [ CH CO H] = √ 1.8 × 10 × 0.100 = 1.3 × 10
3 3 2

−3
pH = − log(1.3 × 10 ) = 2.87

(b) After 25.00 mL of NaOH are added, the number of moles of NaOH and CH3CO2H are equal because the amounts of the
solutions and their concentrations are the same. All of the CH3CO2H has been converted to CH CO . The concentration of 3

2

the CH CO ion is:


3

2

0.00250 mol −
= 0.0500 MCH CO2
3
0.0500 L

The equilibrium that must be focused on now is the basicity equilibrium for CH 3
CO2

:
− −
CH CO2 (aq) + H O(l) ⇌ CH CO H(aq) + OH (aq)
3 2 3 2

so we must determine Kb for the base by using the ion product constant for water:

[ CH CO H][ OH ]
3 2
Kb =

[ CH CO ]
3 2

− + +
[ CH CO ][ H ] [ CH CO H] [H ]
3 2 3 2
Ka = ,  so  = .

[ CH CO H] [ CH CO2 ] Ka
3 2 3

Since Kw = [H+][OH−]:
+ −
[H ][ OH ]
Kb = (14.7.2)
Ka

Kw
= (14.7.3)
Ka
−14
1.0 × 10
= (14.7.4)
−5
1.8 × 10
−10
= 5.6 × 10 (14.7.5)

Let us denote the concentration of each of the products of this reaction, CH3CO2H and OH−, as x. Using the assumption that x
2
x
is small compared to 0.0500 M, K b = , and then:
0.0500 M

− −6
x = [ OH ] = 5.3 × 10

−6
pOH = − log(5.3 × 10 ) = 5.28

pH = 14.00 − 5.28 = 8.72

Note that the pH at the equivalence point of this titration is significantly greater than 7.
(c) In (a), 25.00 mL of the NaOH solution was added, and so practically all the CH3CO2H was converted into CH CO . In 3

this case, only 12.50 mL of the base solution has been introduced, and so only half of all the CH3CO2H is converted into
CH CO . The total initial number of moles of CH3CO2H is 0.02500L × 0.100 M = 0.00250 mol, and so after adding the

3 2

0.00250 mol
NaOH, the numbers of moles of CH3CO2H and CH 3
CO

2
are both approximately equal to = 0.00125 mol , and
2
their concentrations are the same.

Access for free at OpenStax 14.7.6 https://chem.libretexts.org/@go/page/38282


Since the amount of the added base is smaller than the original amount of the acid, the equivalence point has not been reached,
the solution remains a buffer, and we can use the Henderson-Hasselbalch equation:

[Base] [ CH CO2 ]
3 −5
pH = pKa + log = − log(Ka ) + log = − log(1.8 × 10 ) + log(1)
[Acid] [ CH CO H]
3 2

(as the concentrations of CH 3



CO
2
and CH3CO2H are the same)
Thus:
−5
pH = − log(1.8 × 10 ) = 4.74

(the pH = the pKa at the halfway point in a titration of a weak acid)


(d) After 37.50 mL of NaOH is added, the amount of NaOH is 0.03750 L × 0.100 M = 0.003750 mol NaOH. Since this is past the
equivalence point, the excess hydroxide ions will make the solution basic, and we can again use stoichiometric calculations to
determine the pH:
(0.003750 mol − 0.00250 mol)
− −2
[OH ] = = 2.00 × 10 M
0.06250 L

So:
−2
pOH = − log(2.00 × 10 ) = 1.70, and pH = 14.00 − 1.70 = 12.30

Note that this result is the same as for the strong acid-strong base titration example provided, since the amount of the strong base
added moves the solution past the equivalence point.

 Exercise 14.7.2
Calculate the pH for the weak acid/strong base titration between 50.0 mL of 0.100 M HCOOH(aq) (formic acid) and 0.200 M
NaOH (titrant) at the listed volumes of added base:
a. 0.00 mL,
b. 15.0 mL,
c. 25.0 mL, and
d. 30.0 mL.

Answer a
0.00 mL: 2.37
Answer b
15.0 mL: 3.92
Answer c
25.00 mL: 8.29
Answer d
30.0 mL: 12.097

14.7.2: Acid-Base Indicators


Certain organic substances change color in dilute solution when the hydronium ion concentration reaches a particular value. For
example, phenolphthalein is a colorless substance in any aqueous solution with a hydronium ion concentration greater than 5.0 ×
10−9 M (pH < 8.3). In more basic solutions where the hydronium ion concentration is less than 5.0 × 10−9 M (pH > 8.3), it is red or
pink. Substances such as phenolphthalein, which can be used to determine the pH of a solution, are called acid-base indicators.
Acid-base indicators are either weak organic acids or weak organic bases.
The equilibrium in a solution of the acid-base indicator methyl orange, a weak acid, can be represented by an equation in which we
use HIn as a simple representation for the complex methyl orange molecule:

Access for free at OpenStax 14.7.7 https://chem.libretexts.org/@go/page/38282


+ −
HIn(aq) + H O(l) ⇌ H O (aq) + In (aq)
2 3
 
red yellow

+ −
[H O ][ In ]
3 −4
Ka = = 4.0 × 10
[HIn]

The anion of methyl orange, In , is yellow, and the nonionized form, HIn , is red. When we add acid to a solution of methyl

orange, the increased hydronium ion concentration shifts the equilibrium toward the nonionized red form, in accordance with Le
Chatelier’s principle. If we add base, we shift the equilibrium towards the yellow form. This behavior is completely analogous to
the action of buffers.
An indicator’s color is the visible result of the ratio of the concentrations of the two species In− and HIn . If most of the indicator
(typically about 60−90% or more) is present as In , then we see the color of the In ion, which would be yellow for methyl
− −

orange. If most is present as HIn , then we see the color of the HIn molecule: red for methyl orange. For methyl orange, we can
rearrange the equation for Ka and write:

[In ] [substance with yellow color] Ka
= = (14.7.6)
+
[HIn] [substance with red color] [ H3 O ]


[ In ]
Equation 14.7.6 shows us how the ratio of varies with the concentration of hydronium ion. The above expression describing
[HIn]

the indicator equilibrium can be rearranged:


+
[ H3 O ] [H I n]
= (14.7.7)

Ka [I n ]

+
[ H3 O ] [H I n]
log( ) = log( ) (14.7.8)

Ka [I n ]


[I n ]
+
log([ H3 O ]) − log(Ka ) = − log( ) (14.7.9)
[H I n]


[I n ]
−pH + pKa = − log( ) (14.7.10)
[H I n]


[I n ]
pH = p Ka + log( ) (14.7.11)
[H I n]

or in general terms
[base]
pH = p Ka + log( ) (14.7.12)
[acid]

Equation 14.7.12 is the same as the Henderson-Hasselbalch equation, which can be used to describe the equilibrium of indicators.
When [H3O+] has the same numerical value as Ka, the ratio of [In−] to [HIn] is equal to 1, meaning that 50% of the indicator is
present in the red form (HIn) and 50% is in the yellow ionic form (In−), and the solution appears orange in color. When the
hydronium ion concentration increases to 8 × 10−4 M (a pH of 3.1), the solution turns red. No change in color is visible for any
further increase in the hydronium ion concentration (decrease in pH). At a hydronium ion concentration of 4 × 10−5 M (a pH of
4.4), most of the indicator is in the yellow ionic form, and a further decrease in the hydronium ion concentration (increase in pH)
does not produce a visible color change. The pH range between 3.1 (red) and 4.4 (yellow) is the color-change interval of methyl
orange; the pronounced color change takes place between these pH values.
There are many different acid-base indicators that cover a wide range of pH values and can be used to determine the approximate
pH of an unknown solution by a process of elimination. Universal indicators and pH paper contain a mixture of indicators and
exhibit different colors at different pHs. Figure 14.7.2 presents several indicators, their colors, and their color-change intervals.

Access for free at OpenStax 14.7.8 https://chem.libretexts.org/@go/page/38282


Figure 14.7.2 : This chart illustrates the ranges of color change for several acid-base indicators.
Titration curves help us pick an indicator that will provide a sharp color change at the equivalence point. The best selection would
be an indicator that has a color change interval that brackets the pH at the equivalence point of the titration.
The color change intervals of three indicators are shown in Figure 14.7.3. The equivalence points of both the titration of the strong
acid and of the weak acid are located in the color-change interval of phenolphthalein. We can use it for titrations of either strong
acid with strong base or weak acid with strong base.

Access for free at OpenStax 14.7.9 https://chem.libretexts.org/@go/page/38282


Figure 14.7.3 : The graph shows a titration curve for the titration of 25.00 mL of 0.100 M CH3CO2H (weak acid) with 0.100 M
NaOH (strong base) and the titration curve for the titration of HCl (strong acid) with NaOH (strong base). The pH ranges for the
color change of phenolphthalein, litmus, and methyl orange are indicated by the shaded areas.
A graph is shown which is titled “Titration of Weak Acid.” The horizontal axis is labeled “Volume of 0.100 M N a O H added (m
L)” and begins at 0 with markings every 5 units up to 50. The vertical axis is labeled “p H” and begins at 0 and increases by single
units up to 14. A red curve is drawn on the graph. The curve begins at (0, 3) and passes through the points (5, 4.1), (10, 4.7), (15,
5), (20, 5.5), and (22.5, 6), after which it rapidly increases, forming a vertical section centered at the point (25, 8.7). The rapid
increase of the curve then levels off and the curve passes through the points (30, 12), (35, 12.4), (40, 12.5), (45, 12.6), and (50,
12.6). A brown rectangle extends horizontally across the graph covering the p H of 3 to 4.2 range. To the right, this rectangle is
labeled “Methyl orange p H range.” A blue rectangle extends horizontally across the graph covering the p H of 4.6 to 8 range. To
the right, this rectangle is labeled “Litmus p H range.” A purple rectangle extends horizontally across the graph covering the p H of
8.4 to 10 range. To the right, this rectangle is labeled “Phenolphthalein p H range.” The midpoint of the vertical segment of the
curve is labeled “Equivalence point p H, 8.72.”
Litmus is a suitable indicator for the HCl titration because its color change brackets the equivalence point. However, we should not
use litmus for the CH3CO2H titration because the pH is within the color-change interval of litmus when only about 12 mL of
NaOH has been added, and it does not leave the range until 25 mL has been added. The color change would be very gradual, taking
place during the addition of 13 mL of NaOH, making litmus useless as an indicator of the equivalence point.
We could use methyl orange for the HCl titration, but it would not give very accurate results: (1) It completes its color change
slightly before the equivalence point is reached (but very close to it, so this is not too serious); (2) it changes color, as Figure 14.7.2
demonstrates, during the addition of nearly 0.5 mL of NaOH, which is not so sharp a color change as that of litmus or
phenolphthalein; and (3) it goes from yellow to orange to red, making detection of a precise endpoint much more challenging than
the colorless to pink change of phenolphthalein. Figure 14.7.2 shows us that methyl orange would be completely useless as an
indicator for the CH3CO2H titration. Its color change begins after about 1 mL of NaOH has been added and ends when about 8 mL
has been added. The color change is completed long before the equivalence point (which occurs when 25.0 mL of NaOH has been
added) is reached and hence provides no indication of the equivalence point.
We base our choice of indicator on a calculated pH, the pH at the equivalence point. At the equivalence point, equimolar amounts
of acid and base have been mixed, and the calculation becomes that of the pH of a solution of the salt resulting from the titration.

Summary
A titration curve is a graph that relates the change in pH of an acidic or basic solution to the volume of added titrant. The
characteristics of the titration curve are dependent on the specific solutions being titrated. The pH of the solution at the equivalence
point may be greater than, equal to, or less than 7.00. The choice of an indicator for a given titration depends on the expected pH at
the equivalence point of the titration, and the range of the color change of the indicator.

Glossary
acid-base indicator
organic acid or base whose color changes depending on the pH of the solution it is in

color-change interval

Access for free at OpenStax 14.7.10 https://chem.libretexts.org/@go/page/38282


range in pH over which the color change of an indicator takes place

titration curve
plot of the pH of a solution of acid or base versus the volume of base or acid added during a titration

This page titled 14.7: Acid-Base Titrations is shared under a CC BY 4.0 license and was authored, remixed, and/or curated by OpenStax via
source content that was edited to the style and standards of the LibreTexts platform; a detailed edit history is available upon request.

Access for free at OpenStax 14.7.11 https://chem.libretexts.org/@go/page/38282


14.E: Acid-Base Equilibria (Exercises)
14.E.1: 14.1: Brønsted-Lowry Acids and Bases
14.E.1.1: Q14.1.1
Write equations that show NH3 as both a conjugate acid and a conjugate base.

14.E.1.2: S14.1.1
One example for NH3 as a conjugate acid: −
NH
2
+
+H ⟶ NH
3
; as a conjugate base:
+ −
NH (aq) + OH (aq) ⟶ NH (aq) + H O(l)
4 3 2

14.E.1.3: Q14.1.2
Write equations that show H 2
PO4

acting both as an acid and as a base.

14.E.1.4: Q14.1.3
Show by suitable net ionic equations that each of the following species can act as a Brønsted-Lowry acid:
a. H O3
+

b. HCl
c. NH3
d. CH3CO2H
e. NH +

f. HSO −

14.E.1.5: S14.1.3
a. H O (aq) ⟶ H (aq) + H O ;
3
+ +

2 (l)

b. HCl(l) ⟶ H (aq) + Cl (aq) ;+ −

c. NH (aq) ⟶ H (aq) + NH (aq) ;


3
+ −
2

d. CH CO H(aq) ⟶ H (aq) + CH CO
3 2
+

3

2
(aq) ;
e. NH (aq) ⟶ H (aq) + NH (aq) ;
+

4
+

f. HSO (aq) ⟶ H (aq) + SO (aq)


4
+ 2−

14.E.1.6: Q14.1.4
Show by suitable net ionic equations that each of the following species can act as a Brønsted-Lowry acid:
a. HNO3
b. PH +
4

c. H2S
d. CH3CH2COOH
e. H PO
2

4

f. HS−

14.E.1.7: Q14.1.5
Show by suitable net ionic equations that each of the following species can act as a Brønsted-Lowry base:
a. H2O
b. OH −

c. NH3
d. CN−
e. S2−
f. H PO
2

4

14.E.1.8: S14.1.5
a. H O + H (aq) ⟶ H O
2 (l)
+

3
+
(aq)

b. OH (aq) + H (aq) ⟶ H
− +
2
O(l)

Access for free at OpenStax 14.E.1 https://chem.libretexts.org/@go/page/44119


c. NH (aq) + H (aq) ⟶ NH (aq) ;
3
+ +
4

d. CN (aq) + H (aq) ⟶ HCN(aq)


− +

e. S (aq) + H (aq) ⟶ HS (aq)


2− + −

f. H PO (aq) + H (aq) ⟶ H PO (aq)


2

4
+

3 4

14.E.1.9: Q14.1.6
Show by suitable net ionic equations that each of the following species can act as a Brønsted-Lowry base:
a. HS−
b. PO 3−
4

c. NH −

d. C2H5OH
e. O2−
f. H PO
2

4

14.E.1.10: Q14.1.7
What is the conjugate acid of each of the following? What is the conjugate base of each?
a. OH −

b. H2O
c. HCO −

d. NH3
e. HSO −

f. H2O2
g. HS−
h. H N
5
+

14.E.1.11: S14.1.7
H2O, O2−; H3O+, OH ; H2CO3, CO ; NH , NH ; H2SO4, SO
− 2−
3
+
4

2
2−
4
;H
3
+
O
2
, HO ; H2S; S2−; H

2 6
N
2+
2
, H4N2

14.E.1.12: Q14.1.8
What is the conjugate acid of each of the following? What is the conjugate base of each?
a. H2S
b. H PO
2

4

c. PH3
d. HS−
e. HSO −
3

f. H O
3
+
2

g. H4N2
h. CH3OH

14.E.1.13: Q14.1.9
Identify and label the Brønsted-Lowry acid, its conjugate base, the Brønsted-Lowry base, and its conjugate acid in each of the
following equations:
a. HNO + H O ⟶ H O + NO
3 2 3
+ −
3

b. CN + H O ⟶ HCN + OH

2

c. H SO + Cl ⟶ HCl + HSO
2 4
− −

d. HSO + OH ⟶ SO + H O

4
− 2−
4 2

e. O + H O ⟶ 2 OH
2−

2

+ 3+ 2+ 2+
f. [Cu(H O) (OH)] + [Al(H O) ]
2 3 2 6
⟶ [Cu(H O) ]
2 4
+ [Al (H O) (OH)]
2 5

g. H S + NH ⟶ HS + NH
2

2

Access for free at OpenStax 14.E.2 https://chem.libretexts.org/@go/page/44119


14.E.1.14: S14.1.9
The labels are Brønsted-Lowry acid = BA; its conjugate base = CB; Brønsted-Lowry base = BB; its conjugate acid = CA.
HNO3(BA), H2O(BB), H3O+(CA), NO (CB); CN−(BB), H2O(BA), HCN(CA), OH (CB); H2SO4(BA), Cl−(BB), HCl(CA),

3

- 2−
HSO (CB) ; HSO (BA) , OH (BB), SO

4

4
(CB) , H2O(CA); O (BB), H2O(BA) OH (CB and CA); [Cu(H2O)3(OH)]+(BB),
2−

4

[Al(H2O)6] (BA), [Cu(H2O)4] (CA), [Al(H2O)5(OH)] (CB); H2S(BA), NH (BB) , HS−(CB), NH3(CA)
3+ 2+ 2+ −
2

14.E.1.15: Q14.1.10
Identify and label the Brønsted-Lowry acid, its conjugate base, the Brønsted-Lowry base, and its conjugate acid in each of the
following equations:
a. NO + H O ⟶ HNO + OH

2 2 2

b. HBr + H O ⟶ H O + Br
2 3
+ −

c. HS + H O ⟶ H S + OH

2 2

d. H PO + OH ⟶ HPO + H O
2

4
− 2−

4 2

e. H PO + HCl ⟶ H PO + Cl
2

4 3 4

f. [Fe(H O) (OH)] + [Al(H O) ]


2 5
2+

2 6
3+
⟶ [Fe(H O) ]
2 6
3+
+ [Al (H O) (OH)]
2 5
2+

g. CH OH + H ⟶ CH O + H
3

3

2

14.E.1.16: Q14.1.11
What are amphiprotic species? Illustrate with suitable equations.

14.E.1.17: S14.1.11
Amphiprotic species may either gain or lose a proton in a chemical reaction, thus acting as a base or an acid. An example is H2O.
As an acid: H O(aq) + NH (aq) ⇌ NH
2 3
+
4
(aq) + OH

(aq) .
As a base: H O(aq) + HCl(aq) ⇌ H O
2 3
+
(aq) + Cl

(aq)

14.E.1.18: Q14.1.12
State which of the following species are amphiprotic and write chemical equations illustrating the amphiprotic character of these
species:
a. H2O
b. H PO
2

4

c. S2−
d. CO 2−
3

e. HSO −
4

14.E.1.19: Q14.1.13
State which of the following species are amphiprotic and write chemical equations illustrating the amphiprotic character of these
species.
a. NH3
b. HPO −
4

c. Br−
d. NH +

e. ASO 3−

14.E.1.20: S14.113
amphiprotic: NH
3
+H O
3
+
⟶ NH OH + H O
4 2
, NH + OCH ⟶ NH
3

3

2
+ CH OH
3
; HPO
2−

4
+ OH

⟶ PO
3−

4
+H O
2
,
HPO
2−

4
+ HClO
4
⟶ H PO
2

4
+ ClO

4
; not amphiprotic: Br−; NH ; AsO +

4
3−
4

14.E.1.21: Q14.1.14
Is the self ionization of water endothermic or exothermic? The ionization constant for water (Kw) is 2.9 × 10
−14
at 40 °C and
9.6 × 10 at 60 °C.
−14

Access for free at OpenStax 14.E.3 https://chem.libretexts.org/@go/page/44119


14.E.2: 14.2: pH and pOH
14.E.2.1: Q14.2.1
Explain why a sample of pure water at 40 °C is neutral even though [H3O+] = 1.7 × 10−7 M. Kw is 2.9 × 10−14 at 40 °C.

14.E.2.2: S14.2.1
In a neutral solution [H3O+] = [OH−]. At 40 °C,
[H3O+] = [OH−] = (2.910−14)1/2 = 1.7 × 10−7.

14.E.2.3: Q14.2.2
The ionization constant for water (Kw) is 2.9 × 10−14 at 40 °C. Calculate [H3O+], [OH−], pH, and pOH for pure water at 40 °C.

14.E.2.4: Q14.2.3
The ionization constant for water (Kw) is 9.614 × 10−14 at 60 °C. Calculate [H3O+], [OH−], pH, and pOH for pure water at 60 °C.

14.E.2.5: S14.2.3
x = 3.101 × 10−7 M = [H3O+] = [OH−]
pH = -log 3.101 × 10−7 = −(−6.5085) = 6.5085
pOH = pH = 6.5085

14.E.2.6: Q14.2.4
Calculate the pH and the pOH of each of the following solutions at 25 °C for which the substances ionize completely:
a. 0.200 M HCl
b. 0.0143 M NaOH
c. 3.0 M HNO3
d. 0.0031 M Ca(OH)2

14.E.2.7: Q14.2.5
Calculate the pH and the pOH of each of the following solutions at 25 °C for which the substances ionize completely:
a. 0.000259 M HClO4
b. 0.21 M NaOH
c. 0.000071 M Ba(OH)2
d. 2.5 M KOH

14.E.2.8: S14.2.5
pH = 3.587; pOH = 10.413; pH = 0.68; pOH = 13.32; pOH = 3.85; pH = 10.15; pH = −0.40; pOH = 14.4

14.E.2.9: Q14.2.6
What are the pH and pOH of a solution of 2.0 M HCl, which ionizes completely?

14.E.2.10: Q14.2.6
What are the hydronium and hydroxide ion concentrations in a solution whose pH is 6.52?

14.E.2.11: S14.2.6
[H3O+] = 3.0 × 10−7 M; [OH−] = 3.3 × 10−8 M

14.E.2.12: Q14.2.7
Calculate the hydrogen ion concentration and the hydroxide ion concentration in wine from its pH. See below Figure for useful
information.

Access for free at OpenStax 14.E.4 https://chem.libretexts.org/@go/page/44119


14.E.2.13: Q14.2.8
Calculate the hydronium ion concentration and the hydroxide ion concentration in lime juice from its pH. See Figure for useful
information.

14.E.2.14: S14.2.9
[H3O+] = 1 × 10−2 M; [OH−] = 1 × 10−12 M

14.E.2.15: Q14.2.9
The hydronium ion concentration in a sample of rainwater is found to be 1.7 × 10−6 M at 25 °C. What is the concentration of
hydroxide ions in the rainwater?

14.E.2.16: Q14.2.10
The hydroxide ion concentration in household ammonia is 3.2 × 10−3 M at 25 °C. What is the concentration of hydronium ions in
the solution?

14.E.2.17: S14.2.10
[OH−] = 3.1 × 10−12 M

14.E.3: 14.3: Relative Strengths of Acids and Bases


14.E.3.1: Q14.3.1
Explain why the neutralization reaction of a strong acid and a weak base gives a weakly acidic solution.

14.E.3.2: Q14.3.2
Explain why the neutralization reaction of a weak acid and a strong base gives a weakly basic solution.
The salt ionizes in solution, but the anion slightly reacts with water to form the weak acid. This reaction also forms OH−, which
causes the solution to be basic.

14.E.3.3: Q14.3.3
Use this list of important industrial compounds (and Figure) to answer the following questions regarding: CaO, Ca(OH)2,
CH3CO2H, CO2, HCl, H2CO3, HF, HNO2, HNO3, H3PO4, H2SO4, NH3, NaOH, Na2CO3.
a. Identify the strong Brønsted-Lowry acids and strong Brønsted-Lowry bases.
b. List those compounds in that can behave as Brønsted-Lowry acids with strengths lying between those of H3O+ and H2O.

Access for free at OpenStax 14.E.5 https://chem.libretexts.org/@go/page/44119


c. List those compounds in that can behave as Brønsted-Lowry bases with strengths lying between those of H2O and OH−.

14.E.3.4: Q14.3.4
The odor of vinegar is due to the presence of acetic acid, CH3CO2H, a weak acid. List, in order of descending concentration, all of
the ionic and molecular species present in a 1-M aqueous solution of this acid.

14.E.3.5: S14.3.4
[H2O] > [CH3CO2H] > [H 3
+
O ] ≈ [CH 3

CO2 ] > [OH−]

14.E.3.6: Q14.3.5
Household ammonia is a solution of the weak base NH3 in water. List, in order of descending concentration, all of the ionic and
molecular species present in a 1-M aqueous solution of this base.

14.E.3.7: Q14.3.4
Explain why the ionization constant, Ka, for H2SO4 is larger than the ionization constant for H2SO3.

14.E.3.8: S14.3.4
The oxidation state of the sulfur in H2SO4 is greater than the oxidation state of the sulfur in H2SO3.

14.E.3.9: Q14.3.7
Explain why the ionization constant, Ka, for HI is larger than the ionization constant for HF.

14.E.3.10: Q14.3.8
Gastric juice, the digestive fluid produced in the stomach, contains hydrochloric acid, HCl. Milk of Magnesia, a suspension of solid
Mg(OH)2 in an aqueous medium, is sometimes used to neutralize excess stomach acid. Write a complete balanced equation for the
neutralization reaction, and identify the conjugate acid-base pairs.

14.E.3.11: S14.3.8
2+ −
Mg (OH) (s) + HCl(aq) ⟶ Mg (aq) + 2 Cl (aq) + 2 H O(l)
2 2

BB BA CB CA

14.E.3.12: Q14.3.9
Nitric acid reacts with insoluble copper(II) oxide to form soluble copper(II) nitrate, Cu(NO3)2, a compound that has been used to
prevent the growth of algae in swimming pools. Write the balanced chemical equation for the reaction of an aqueous solution of
HNO3 with CuO.

14.E.3.13: Q14.3.10
What is the ionization constant at 25 °C for the weak acid CH NH
3
+
3
, the conjugate acid of the weak base CH3NH2, Kb = 4.4 ×
10−4.

14.E.3.14: S14.3.10
−11
Ka = 2.3 × 10

14.E.3.15: Q14.3.11
What is the ionization constant at 25 °C for the weak acid (CH 3
) NH
2
+
2
, the conjugate acid of the weak base (CH3)2NH, Kb = 7.4 ×
10−4?

14.E.3.16: Q14.3.12
Which base, CH3NH2 or (CH3)2NH, is the strongest base? Which conjugate acid, (CH 3
) NH
2
+

2
or (CH3)2NH, is the strongest acid?

14.E.3.17: S14.3.12
The strongest base or strongest acid is the one with the larger Kb or Ka, respectively. In these two examples, they are (CH3)2NH and
CH NH .
+

3 3

Access for free at OpenStax 14.E.6 https://chem.libretexts.org/@go/page/44119


14.E.3.18: Q14.3.3
Which is the stronger acid, NH or HBrO? +
4

14.E.3.19: Q14.3.14
Which is the stronger base, (CH3)3N or H 2
BO

3
?

14.E.3.20: S14.3.14
triethylamine.

14.E.3.21: Q14.3.15
Predict which acid in each of the following pairs is the stronger and explain your reasoning for each.
a. H2O or HF
b. B(OH)3 or Al(OH)3
c. HSO or HSO

3

d. NH3 or H2S
e. H2O or H2Te

14.E.3.22: Q14.3.16
Predict which compound in each of the following pairs of compounds is more acidic and explain your reasoning for each.
a. HSO or HSeO

4

4

b. NH3 or H2O
c. PH3 or HI
d. NH3 or PH3
e. H2S or HBr

14.E.3.23: S14.3.16
a. HSO ; higher electronegativity of the central ion. H2O;

4

b. NH3 is a base and water is neutral, or decide on the basis of Ka values. HI;
c. PH3 is weaker than HCl; HCl is weaker than HI. Thus, PH3 is weaker than HI.
d. PH3; in binary compounds of hydrogen with nonmetals, the acidity increases for the element lower in a group.
e. HBr; in a period, the acidity increases from left to right; in a group, it increases from top to bottom. Br is to the left and below
S, so HBr is the stronger acid.

14.E.3.24: Q14.3.17
Rank the compounds in each of the following groups in order of increasing acidity or basicity, as indicated, and explain the order
you assign.
a. acidity: HCl, HBr, HI
b. basicity: H2O, OH−, H−, Cl−
c. basicity: Mg(OH)2, Si(OH)4, ClO3(OH) (Hint: Formula could also be written as HClO4).
d. acidity: HF, H2O, NH3, CH4

14.E.3.25: Q14.3.18
Rank the compounds in each of the following groups in order of increasing acidity or basicity, as indicated, and explain the order
you assign.
a. acidity: NaHSO3, NaHSeO3, NaHSO4
b. basicity: BrO , ClO , IO

2

2

c. acidity: HOCl, HOBr, HOI


d. acidity: HOCl, HOClO, HOClO2, HOClO3
e. basicity: NH , HS−, HTe−, PH

2

f. basicity: BrO−, BrO , BrO , BrO


2

3

Access for free at OpenStax 14.E.7 https://chem.libretexts.org/@go/page/44119


14.E.3.26: S14.3.18
a. NaHSeO3 < NaHSO3 < NaHSO4; in polyoxy acids, the more electronegative central element—S, in this case—forms the
stronger acid. The larger number of oxygen atoms on the central atom (giving it a higher oxidation state) also creates a greater
release of hydrogen atoms, resulting in a stronger acid. As a salt, the acidity increases in the same manner.
b. ClO < BrO < IO ; the basicity of the anions in a series of acids will be the opposite of the acidity in their oxyacids. The

2

2

acidity increases as the electronegativity of the central atom increases. Cl is more electronegative than Br, and I is the least
electronegative of the three.
c. HOI < HOBr < HOCl; in a series of the same form of oxyacids, the acidity increases as the electronegativity of the central atom
increases. Cl is more electronegative than Br, and I is the least electronegative of the three.
d. HOCl < HOClO < HOClO2 < HOClO3; in a series of oxyacids of the same central element, the acidity increases as the number
of oxygen atoms increases (or as the oxidation state of the central atom increases).
e. HTe < HS << PH < NH ; PH and NH are anions of weak bases, so they act as strong bases toward H+. HTe and
− − −

2

2

2

2

HS− are anions of weak acids, so they have less basic character. In a periodic group, the more electronegative element has the
more basic anion.
f. BrO < BrO < BrO < BrO ; with a larger number of oxygen atoms (that is, as the oxidation state of the central ion

4

3

2

increases), the corresponding acid becomes more acidic and the anion consequently less basic.

14.E.3.27: Q14.3.19
Both HF and HCN ionize in water to a limited extent. Which of the conjugate bases, F− or CN−, is the stronger base? See Table.

14.E.3.28: Q14.3.20
The active ingredient formed by aspirin in the body is salicylic acid, C6H4OH(CO2H). The carboxyl group (−CO2H) acts as a weak
acid. The phenol group (an OH group bonded to an aromatic ring) also acts as an acid but a much weaker acid. List, in order of
descending concentration, all of the ionic and molecular species present in a 0.001-M aqueous solution of C6H4OH(CO2H).
+ − − −
[ H O] > [ C H OH(CO H)] > [ H ] > [ C H OH(CO ) ] ≫ [ C H O(CO H) ] > [ OH ]
2 6 4 2 0 6 4 2 6 4 2

What do we represent when we write:


+ −
CH CO H(aq) + H O(l) ⇌ H O (aq) + CH CO2 (aq)? (14.E.1)
3 2 2 3 3

14.E.3.29: Q14.3.21
Explain why equilibrium calculations are not necessary to determine ionic concentrations in solutions of certain strong electrolytes
such as NaOH and HCl. Under what conditions are equilibrium calculations necessary as part of the determination of the
concentrations of all ions of some other strong electrolytes in solution?

14.E.3.30: S14.3.21
Strong electrolytes are 100% ionized, and, as long as the component ions are neither weak acids nor weak bases, the ionic species
present result from the dissociation of the strong electrolyte. Equilibrium calculations are necessary when one (or more) of the ions
is a weak acid or a weak base.

14.E.3.31: Q14.3.22
Are the concentrations of hydronium ion and hydroxide ion in a solution of an acid or a base in water directly proportional or
inversely proportional? Explain your answer.

14.E.3.32: Q14.3.23
What two common assumptions can simplify calculation of equilibrium concentrations in a solution of a weak acid?

14.E.3.33: S14.3.23
1. Assume that the change in initial concentration of the acid as the equilibrium is established can be neglected, so this
concentration can be assumed constant and equal to the initial value of the total acid concentration.
2. Assume we can neglect the contribution of water to the equilibrium concentration of H3O+.

Access for free at OpenStax 14.E.8 https://chem.libretexts.org/@go/page/44119


14.E.3.34: Q14.3.24
What two common assumptions can simplify calculation of equilibrium concentrations in a solution of a weak base?

14.E.3.35: Q14.3.25
Which of the following will increase the percent of NH3 that is converted to the ammonium ion in water (Hint: Use LeChâtelier’s
principle.)?
a. addition of NaOH
b. addition of HCl
c. addition of NH4Cl

14.E.3.36: S14.3.25
The addition of HCl

14.E.3.37: Q14.3.26
Which of the following will increase the percent of HF that is converted to the fluoride ion in water?
a. addition of NaOH
b. addition of HCl
c. addition of NaF

14.E.3.38: Q14.3.27
What is the effect on the concentrations of NO , HNO2, and OH− when the following are added to a solution of KNO2 in water:

2

a. HCl
b. HNO2
c. NaOH
d. NaCl
e. KNO
The equation for the equilibrium is:
− −
NO (aq) + H O(l) ⇌ HNO (aq) + OH (aq) (14.E.2)
2 2 2

14.E.3.39: S14.3.27
a. Adding HCl will add H3O+ ions, which will then react with the OH− ions, lowering their concentration. The equilibrium will
shift to the right, increasing the concentration of HNO2, and decreasing the concentration of NO ions.

2

b. Adding HNO2 increases the concentration of HNO2 and shifts the equilibrium to the left, increasing the concentration of NO −
2

ions and decreasing the concentration of OH− ions.


c. Adding NaOH adds OH− ions, which shifts the equilibrium to the left, increasing the concentration of NO ions and decreasing

2

the concentrations of HNO2.


d. Adding NaCl has no effect on the concentrations of the ions.
e. Adding KNO2 adds NO ions and shifts the equilibrium to the right, increasing the HNO2 and OH− ion concentrations.

2

14.E.3.40: Q14.3.28
What is the effect on the concentration of hydrofluoric acid, hydronium ion, and fluoride ion when the following are added to
separate solutions of hydrofluoric acid?
a. HCl
b. KF
c. NaCl
d. KOH
e. HF
The equation for the equilibrium is:
+ −
HF(aq) + H O(l) ⇌ H O (aq) + F (aq) (14.E.3)
2 3

Access for free at OpenStax 14.E.9 https://chem.libretexts.org/@go/page/44119


14.E.3.41: Q14.3.29
Why is the hydronium ion concentration in a solution that is 0.10 M in HCl and 0.10 M in HCOOH determined by the
concentration of HCl?

14.E.3.42: S14.3.29
This is a case in which the solution contains a mixture of acids of different ionization strengths. In solution, the HCO2H exists
primarily as HCO2H molecules because the ionization of the weak acid is suppressed by the strong acid. Therefore, the HCO2H
contributes a negligible amount of hydronium ions to the solution. The stronger acid, HCl, is the dominant producer of hydronium
ions because it is completely ionized. In such a solution, the stronger acid determines the concentration of hydronium ions, and the
ionization of the weaker acid is fixed by the [H3O+] produced by the stronger acid.

14.E.3.43: Q14.3.30
From the equilibrium concentrations given, calculate Ka for each of the weak acids and Kb for each of the weak bases.
CH3CO2H: [H 3
O
+
] = 1.34 × 10−3 M;
[ CH CO
3

2
] = 1.34 × 10−3 M;
[CH3CO2H] = 9.866 × 10−2 M;
ClO−: [OH−] = 4.0 × 10−4 M;
[HClO] = 2.38 × 10−5 M;
[ClO−] = 0.273 M;
HCO2H: [HCO2H] = 0.524 M;
[H O
3
+
] = 9.8 × 10−3 M; [HCO −
2
] = 9.8 × 10−3 M;
C H NH
6 5
+
3
: [ C H NH
6 5
+
3
] = 0.233 M;
[C6H5NH2] = 2.3 × 10−3 M;
[H O
3
+
] = 2.3 × 10−3 M
From the equilibrium concentrations given, calculate Ka for each of the weak acids and Kb for each of the weak bases.
NH3: [OH−] = 3.1 × 10−3 M;
[ NH
+
4
] = 3.1 × 10−3 M;
[NH3] = 0.533 M;
HNO2: [H 3
O
+
] = 0.011 M;
[ NO

2
] = 0.0438 M;
[HNO2] = 1.07 M;
(CH3)3N: [(CH3)3N] = 0.25 M;
[(CH3)3NH+] = 4.3 × 10−3 M;
[OH−] = 4.3 × 10−3 M;
NH
+

4
: [ NH
+

4
] = 0.100 M;
[NH3] = 7.5 × 10−6 M;
[H3O+] = 7.5 × 10−6 M
a. K b = 1.8 × 10
−5
;

b. K a = 4.5 × 10
−4
;

c. K b = 7.4 × 10
−5
;

d. K a = 5.6 × 10
−10

Access for free at OpenStax 14.E.10 https://chem.libretexts.org/@go/page/44119


14.E.3.44: Q14.3.31
Determine Kb for the nitrite ion, NO . In a 0.10-M solution this base is 0.0015% ionized.

2

14.E.3.45: Q14.3.32
Determine Ka for hydrogen sulfate ion, HSO . In a 0.10-M solution the acid is 29% ionized.

14.E.3.46: S14.3.32
−2
Ka = 1.2 × 10

14.E.3.47: Q14.3.33
Calculate the ionization constant for each of the following acids or bases from the ionization constant of its conjugate base or
conjugate acid:
a. F−
b. NH +
4

c. AsO 3−

d. (CH ) NH
3 2
+
2

e. NO −

f. HC O (as a base)
2

14.E.3.48: Q14.3.52
Calculate the ionization constant for each of the following acids or bases from the ionization constant of its conjugate base or
conjugate acid:
1. HTe− (as a base)
2. (CH ) NH 3 3
+

3. HAsO (as a base)


3−

4. HO (as a base)

2

5. C H NH
6 5
+
3

6. HSO (as a base)



3

14.E.3.49: S14.3.52
a. K = 4.3 × 10 ;
b
−12

b. K = 1.4 × 10 ;
a
−10

c. K = 1 × 10 ;
b
−7

d. K = 4.2 × 10 ;
b
−3

e. K = 4.2 × 10 ;
b
−3

f. K = 8.3 × 10
b
−13

14.E.3.50: Q14.3.53
For which of the following solutions must we consider the ionization of water when calculating the pH or pOH?
a. 3 × 10−8 M HNO3
b. 0.10 g HCl in 1.0 L of solution
c. 0.00080 g NaOH in 0.50 L of solution
d. 1 × 10−7 M Ca(OH)2
e. 0.0245 M KNO3

14.E.3.51: Q14.3.54
Even though both NH3 and C6H5NH2 are weak bases, NH3 is a much stronger acid than C6H5NH2. Which of the following is
correct at equilibrium for a solution that is initially 0.10 M in NH3 and 0.10 M in C6H5NH2?
a. [OH −
] = [ NH
+
4
]

b. [NH +
4
] = [ C H NH
6 5
+
3
]

c. [OH −
] = [ C H NH
6 5
+
3
]

Access for free at OpenStax 14.E.11 https://chem.libretexts.org/@go/page/44119


d. [NH3] = [C6H5NH2]
e. both a and b are correct
is the correct statement.

14.E.3.52: Q14.3.55
Calculate the equilibrium concentration of the nonionized acids and all ions in a solution that is 0.25 M in HCO2H and 0.10 M in
HClO.

14.E.3.53: Q14.3.56
Calculate the equilibrium concentration of the nonionized acids and all ions in a solution that is 0.134 M in HNO2 and 0.120 M in
HBrO.

14.E.3.54: S14.3.56
[H3O+] = 7.5 × 10−3 M
[HNO2] = 0.126 [OH−] = 1.3 × 10−12 M [BrO−] = 3.2 × 10−8 M [HBrO] = 0.120 M

14.E.3.55: Q14.3.57
Calculate the equilibrium concentration of the nonionized bases and all ions in a solution that is 0.25 M in CH3NH2 and 0.10 M in
C5H5N (Kb = 1.7 × 10−9).

14.E.3.56: Q14.3.58
Calculate the equilibrium concentration of the nonionized bases and all ions in a solution that is 0.115 M in NH3 and 0.100 M in
C6H5NH2.

14.E.3.57: S14.3.58
[OH−] = [NO +
4
] = 0.0014 M
[NH3] = 0.144 M [H3O+] = 6.9 × 10−12 M [C 6
H NH
5
+

3
] = 3.9 × 10−8 M [C6H5NH2] = 0.100 M

14.E.3.58: Q14.3.59
Using the Ka values in Appendix H, place Al(H 2
O)
3+

6
in the correct location in Figure.

14.E.3.59: Q14.3.60
Calculate the concentration of all solute species in each of the following solutions of acids or bases. Assume that the ionization of
water can be neglected, and show that the change in the initial concentrations can be neglected. Ionization constants can be found
in Appendix H and Appendix I.
a. 0.0092 M HClO, a weak acid
b. 0.0784 M C6H5NH2, a weak base
c. 0.0810 M HCN, a weak acid
d. 0.11 M (CH3)3N, a weak base
e. 0.120 M Fe(H O) a weak acid, Ka = 1.6 × 10−7
2
2+

14.E.3.60: S14.3.60
+ −
[H O ][ ClO ] (x)(x) (x)(x)
3 −8
= ≈ = 3.5 × 10
[HClO] (0.0092 − x) 0.0092

Solving for x gives 1.79 × 10−5 M. This value is less than 5% of 0.0092, so the assumption that it can be neglected is valid. Thus,
the concentrations of solute species at equilibrium are:
[H3O+] = [ClO] = 1.8 × 10−5 M[HClO] = 0.00092 M [OH−] = 5.6 × 10−10 M;
+ −
[ C H NH ][ OH ] (x)(x) (x)(x)
6 5 3 −10
= ≈ = 4.6 × 10
[ C H NH ] (0.0784 − x) 0.0784
6 5 2

Solving for x gives 6.01 × 10−6 M.

Access for free at OpenStax 14.E.12 https://chem.libretexts.org/@go/page/44119


This value is less than 5% of 0.0784, so the assumption that it can be neglected is valid. Thus, the concentrations of solute species
at equilibrium are: [CH CO ] = [OH−] = 6.0 × 10−6 M [C6H5NH2] = 0.00784 [H3O+] = 1.7× 10−9 M;
3

2
+ −
[H O ][ CN ] (x)(x) (x)(x)
Solving for x gives 5.69 × 10−6 M. This value is less than 5% of
3 −10
= ≈ = 4 × 10
[HCN] (0.0810 − x) 0.0810

0.0810, so the assumption that it can be neglected is valid. Thus, the concentrations of solute species at equilibrium are: [H3O+] =
+ −
[ (CH ) NH ][ OH ] (x)(x) (x)(x)
[CN−] = 5.7 × 10−6 M [HCN] = 0.0810 M [OH−] = 1.8 × 10−9 M;
3 3 −5
= ≈ = 7.4 × 10
[ (CH ) N] (0.11 − x) 0.11
3 3

Solving for x gives 2.85 × 10−3 M. This value is less than 5% of 0.11, so the assumption that it can be neglected is valid. Thus, the
concentrations of solute species at equilibrium are: [(CH3)3NH+] = [OH−] = 2.9 × 10−3 M [(CH3)3N] = 0.11 M [H3O+] = 3.5 ×
+ +
[Fe(H O) (OH) ][ H O ] (x)(x) (x)(x)
10−12 M; Solving for x gives 1.39 × 10−4 M. This value is
2 5 3 −7
= ≈ = 1.6 × 10
2+
[Fe(H O) ] (0.120 − x) 0.120
2 6

less than 5% of 0.120, so the assumption that it can be neglected is valid. Thus, the concentrations of solute species at equilibrium
are: [Fe(H2O)5(OH)+] = [H3O+] = 1.4 × 10−4 M [Fe(H O) ] = 0.120 M [OH−] = 7.2 × 10−11 M 2
2+

14.E.3.61: Q14.3.61
Propionic acid, C2H5CO2H (Ka = 1.34 × 10−5), is used in the manufacture of calcium propionate, a food preservative. What is the
hydronium ion concentration in a 0.698-M solution of C2H5CO2H?

14.E.3.62: Q14.3.62
White vinegar is a 5.0% by mass solution of acetic acid in water. If the density of white vinegar is 1.007 g/cm3, what is the pH?

14.E.3.63: S14.3.62
pH = 2.41

14.E.3.64: Q14.3.63
The ionization constant of lactic acid, CH3CH(OH)CO2H, an acid found in the blood after strenuous exercise, is 1.36 × 10−4. If
20.0 g of lactic acid is used to make a solution with a volume of 1.00 L, what is the concentration of hydronium ion in the solution?

14.E.3.65: Q14.3.64
Nicotine, C10H14N2, is a base that will accept two protons (K1 = 7 × 10−7, K2 = 1.4 × 10−11). What is the concentration of each
species present in a 0.050-M solution of nicotine?

14.E.3.66: S14.3.64
[C10H14N2] = 0.049 M
[C10H14N2H+] = 1.9 × 10−4 M [C 10
H
14
N H
2
2+
2
] = 1.4 × 10−11 M [OH−] = 1.9 × 10−4 M [H3O+] = 5.3 × 10−11 M

14.E.3.67: Q14.3.65
The pH of a 0.20-M solution of HF is 1.92. Determine Ka for HF from these data.

14.E.3.68: Q14.3.66
The pH of a 0.15-M solution of HSO is 1.43. Determine Ka for HSO from these data.

4

14.E.3.69: S14.3.66
−2
Ka = 1.2 × 10

14.E.3.70: Q14.3.67
The pH of a 0.10-M solution of caffeine is 11.16. Determine Kb for caffeine from these data:
+ −
C H N O (aq) + H O(l) ⇌ C H N O H (aq) + OH (aq)
8 10 4 2 2 8 10 4 2

14.E.3.71: Q14.3.68
The pH of a solution of household ammonia, a 0.950 M solution of NH3, is 11.612. Determine Kb for NH3 from these data.

Access for free at OpenStax 14.E.13 https://chem.libretexts.org/@go/page/44119


14.E.3.72: S14.3.68
−5
Kb = 1.77 × 10

14.E.4: 14.4: Hydrolysis of Salt Solutions


14.E.4.1: Q14.4.1
Determine whether aqueous solutions of the following salts are acidic, basic, or neutral:
a. Al(NO3)3
b. RbI
c. KHCO2
d. CH3NH3Br

14.E.4.2: Q14.4.2
Determine whether aqueous solutions of the following salts are acidic, basic, or neutral:
a. FeCl3
b. K2CO3
c. NH4Br
d. KClO4

14.E.4.3: S14.4.2
acidic; basic; acidic; neutral

14.E.4.4: Q14.4.3
Novocaine, C13H21O2N2Cl, is the salt of the base procaine and hydrochloric acid. The ionization constant for procaine is 7 × 10−6.
Is a solution of novocaine acidic or basic? What are [H3O+], [OH−], and pH of a 2.0% solution by mass of novocaine, assuming
that the density of the solution is 1.0 g/mL.

14.E.5: 14.5: Polyprotic Acids


14.E.5.1: Q15.5.1
Which of the following concentrations would be practically equal in a calculation of the equilibrium concentrations in a 0.134-M
solution of H2CO3, a diprotic acid:
[H O
3
+
],

[OH ]

[ H2 C O3 ]

[ HCO3 ]
2−
[ CO ]
3

No calculations are needed to answer this question.

14.E.5.2: S15.5.1
[H3O+] and [HCO −
3
] are equal, H3O+ and HCO are practically equal

3

14.E.5.3: Q15.5.2
Calculate the concentration of each species present in a 0.050-M solution of H2S.

14.E.5.4: Q15.5.3
Calculate the concentration of each species present in a 0.010-M solution of phthalic acid, C6H4(CO2H)2.

14.E.5.5: S15.5.3
+ − −3
C H (CO H) (aq) + H O(l) ⇌ H O (aq) + C H (CO H)(CO ) (aq) Ka = 1.1 × 10
6 4 2 2 2 3 6 4 2 2

+ 2− −6
C H (CO H)(CO )(aq) + H O(l) ⇌ H O (aq) + C H (CO ) (aq) Ka = 3.9 × 10
6 4 2 2 2 3 6 4 2 2

Access for free at OpenStax 14.E.14 https://chem.libretexts.org/@go/page/44119


[C6H4(CO2H)2] 7.2 × 10−3 M, [C6H4(CO2H)(CO2)−] = [H3O+] 2.8 × 10−3 M, [C 6
H (CO )
4 2
2−

2
3.9 × 10−6 M, [OH−] 3.6 × 10−12 M
]

14.E.5.6: Q15.5.4
Salicylic acid, HOC6H4CO2H, and its derivatives have been used as pain relievers for a long time. Salicylic acid occurs in small
amounts in the leaves, bark, and roots of some vegetation (most notably historically in the bark of the willow tree). Extracts of
these plants have been used as medications for centuries. The acid was first isolated in the laboratory in 1838.
1. Both functional groups of salicylic acid ionize in water, with Ka = 1.0 × 10−3 for the—CO2H group and 4.2 × 10−13 for the −OH
group. What is the pH of a saturated solution of the acid (solubility = 1.8 g/L).
2. Aspirin was discovered as a result of efforts to produce a derivative of salicylic acid that would not be irritating to the stomach
lining. Aspirin is acetylsalicylic acid, CH3CO2C6H4CO2H. The −CO2H functional group is still present, but its acidity is
reduced, Ka = 3.0 × 10−4. What is the pH of a solution of aspirin with the same concentration as a saturated solution of salicylic
acid (See Part a).
3. Under some conditions, aspirin reacts with water and forms a solution of salicylic acid and acetic acid:
CH CO C H CO H(aq) + H O(l) ⟶ HOC H CO H(aq) + CH CO H(aq) (14.E.4)
3 2 6 4 2 2 6 4 2 3 2

i. Which of the acids salicylic acid or acetic acid produces more hydronium ions in solution such a solution?
ii. What are the concentrations of molecules and ions in a solution produced by the hydrolysis of 0.50 g of aspirin dissolved in
enough water to give 75 mL of solution?

14.E.5.7: Q15.5.5
The ion HTe− is an amphiprotic species; it can act as either an acid or a base.
a. What is Ka for the acid reaction of HTe− with H2O?
b. What is Kb for the reaction in which HTe− functions as a base in water?
c. Demonstrate whether or not the second ionization of H2Te can be neglected in the calculation of [HTe−] in a 0.10 M solution of
H2Te.

14.E.5.8: S15.5.5
a. K = 1 × 10 ;
a2
−5

b. K = 4.3 × 10
b
−12
;
2− +
[ Te ][ H O ] (x)(0.0141 + x) (x)(0.0141)
. Solving for x gives 1 × 10−5 M. Therefore, compared with
3
c. −
= ≈
−5
= 1 × 10
[ HTe ] (0.0141 − x) 0.0141

0.014 M, this value is negligible (0.071%).

14.E.6: 14.6: Buffers


14.E.6.1: Q14.6.1
Explain why a buffer can be prepared from a mixture of NH4Cl and NaOH but not from NH3 and NaOH.

14.E.6.2: Q14.6.2
Explain why the pH does not change significantly when a small amount of an acid or a base is added to a solution that contains
equal amounts of the acid H3PO4 and a salt of its conjugate base NaH2PO4.

14.E.6.3: S14.6.2
Excess H3O+ is removed primarily by the reaction:
H O
3
+
(aq) + H PO
2

4
(aq) ⟶ H PO (aq) + H O(l)
3 4 2
Excess base is removed by the reaction:
− −
OH (aq) + H PO (aq) ⟶ H PO (aq) + H O(l)
3 4 2 4 2

14.E.6.4: Q14.6.3
Explain why the pH does not change significantly when a small amount of an acid or a base is added to a solution that contains
equal amounts of the base NH3 and a salt of its conjugate acid NH4Cl.

Access for free at OpenStax 14.E.15 https://chem.libretexts.org/@go/page/44119


14.E.6.5: Q14.6.4
What is [H3O+] in a solution of 0.25 M CH3CO2H and 0.030 M NaCH3CO2?
+ − −5
CH CO H(aq) + H O(l) ⇌ H O (aq) + CH CO (aq) Ka = 1.8 × 10
3 2 2 3 3 2

14.E.6.6: S14.6.4
[H3O+] = 1.5 × 10−4 M

14.E.6.7: Q14.6.5
What is [H3O+] in a solution of 0.075 M HNO2 and 0.030 M NaNO2?

14.E.6.8: S14.6.6
+ − −5
HNO (aq) + H O(l) ⇌ H O (aq) + NO (aq) Ka = 4.5 × 10
2 2 3 2

14.E.6.9: Q14.6.6
What is [OH−] in a solution of 0.125 M CH3NH2 and 0.130 M CH3NH3Cl?

14.E.6.10: S14.6.6
+ − −4
CH NH (aq) + H O(l) ⇌ CH NH (aq) + OH (aq) Kb = 4.4 × 10
3 2 2 3 3

[OH−] = 4.2 × 10−4 M

14.E.6.11: Q14.6.7
What is [OH−] in a solution of 1.25 M NH3 and 0.78 M NH4NO3?

14.E.6.12: S14.6.7
+ − −5
NH (aq) + H O(l) ⇌ NH (aq) + OH (aq) Kb = 1.8 × 10
3 2 4

14.E.6.13: Q14.6.8
What concentration of NH4NO3 is required to make [OH−] = 1.0 × 10−5 in a 0.200-M solution of NH3?

14.E.6.14: S14.6.8
[NH4NO3] = 0.36 M

14.E.6.15: Q14.6.9A
What concentration of NaF is required to make [H3O+] = 2.3 × 10−4 in a 0.300-M solution of HF?

14.E.6.16: Q14.6.9B
What is the effect on the concentration of acetic acid, hydronium ion, and acetate ion when the following are added to an acidic
buffer solution of equal concentrations of acetic acid and sodium acetate:
a. HCl
b. KCH3CO2
c. NaCl
d. KOH
e. CH3CO2H

14.E.6.17: S14.6.10
a. The added HCl will increase the concentration of H3O+ slightly, which will react with CH CO and produce CH3CO2H in the
3

process. Thus, [CH CO ] decreases and [CH3CO2H] increases.


3

b. The added KCH3CO2 will increase the concentration of [CH CO ] which will react with H3O+ and produce CH3CO2 H in the
3

process. Thus, [H3O+] decreases slightly and [CH3CO2H] increases.


c. The added NaCl will have no effect on the concentration of the ions.
d. The added KOH will produce OH− ions, which will react with the H3O+, thus reducing [H3O+]. Some additional CH3CO2H will
dissociate, producing [CH CO ] ions in the process. Thus, [CH3CO2H] decreases slightly and [CH CO ] increases.
3

2 3

e. The added CH3CO2H will increase its concentration, causing more of it to dissociate and producing more [CH CO ] and 3

H3O+ in the process. Thus, [H3O+] increases slightly and [CH CO ] increases. 3

Access for free at OpenStax 14.E.16 https://chem.libretexts.org/@go/page/44119


14.E.6.18: Q14.6.11
What is the effect on the concentration of ammonia, hydroxide ion, and ammonium ion when the following are added to a basic
buffer solution of equal concentrations of ammonia and ammonium nitrate:
a. KI
b. NH3
c. HI
d. NaOH
e. NH4Cl
What will be the pH of a buffer solution prepared from 0.20 mol NH3, 0.40 mol NH4NO3, and just enough water to give 1.00 L of
solution?
pH = 8.95
Calculate the pH of a buffer solution prepared from 0.155 mol of phosphoric acid, 0.250 mole of KH2PO4, and enough water to
make 0.500 L of solution.
How much solid NaCH3CO2•3H2O must be added to 0.300 L of a 0.50-M acetic acid solution to give a buffer with a pH of 5.00?
(Hint: Assume a negligible change in volume as the solid is added.)
37 g (0.27 mol)
What mass of NH4Cl must be added to 0.750 L of a 0.100-M solution of NH3 to give a buffer solution with a pH of 9.26? (Hint:
Assume a negligible change in volume as the solid is added.)

14.E.6.19: Q14.6.1
A buffer solution is prepared from equal volumes of 0.200 M acetic acid and 0.600 M sodium acetate. Use 1.80 × 10−5 as Ka for
acetic acid.
1. What is the pH of the solution?
2. Is the solution acidic or basic?

14.E.6.20: Q14.6.1
What is the pH of a solution that results when 3.00 mL of 0.034 M HCl is added to 0.200 L of the original buffer?
a. pH = 5.222;
b. The solution is acidic. (c) pH = 5.221

14.E.6.21: Q14.6.1
A 5.36–g sample of NH4Cl was added to 25.0 mL of 1.00 M NaOH and the resulting solution diluted to 0.100 L.
a. What is the pH of this buffer solution?
b. Is the solution acidic or basic?
c. What is the pH of a solution that results when 3.00 mL of 0.034 M HCl is added to the solution?
Which acid in [link] is most appropriate for preparation of a buffer solution with a pH of 3.1? Explain your choice.
+
[H O ]
3
To prepare the best buffer for a weak acid HA and its salt, the ratio should be as close to 1 as possible for effective buffer
Ka
+ + −3.1
action. The [H3O ] concentration in a buffer of pH 3.1 is [H3O ] = 10 = 7.94 × 10−4 M
We can now solve for Ka of the best acid as follows:
+
[H O ]
3
=1
Ka

+
[H O ]
3 −4
Ka = = 7.94 × 10
1

In [link], the acid with the closest Ka to 7.94 × 10−4 is HF, with a Ka of 7.2 × 10−4.
Which acid in [link] is most appropriate for preparation of a buffer solution with a pH of 3.7? Explain your choice.

Access for free at OpenStax 14.E.17 https://chem.libretexts.org/@go/page/44119


Which base in [link] is most appropriate for preparation of a buffer solution with a pH of 10.65? Explain your choice.

[ OH ]
For buffers with pHs > 7, you should use a weak base and its salt. The most effective buffer will have a ratio that is as
Kb

close to 1 as possible. The pOH of the buffer is 14.00 − 10.65 = 3.35. Therefore, [OH−] is [OH−] = 10−pOH = 10−3.35 = 4.467 × 10−4
M.

[ OH ]
We can now solve for Kb of the best base as follows: =1 Kb = [OH−] = 4.47 × 10−4 In [link], the base with the closest Kb
Kb

to 4.47 × 10−4 is CH3NH2, with a Kb = 4.4 × 10−4.


Which base in [link] is most appropriate for preparation of a buffer solution with a pH of 9.20? Explain your choice.

14.E.6.22: Q14.6.4
Saccharin, C7H4NSO3H, is a weak acid (Ka = 2.1 × 10−2). If 0.250 L of diet cola with a buffered pH of 5.48 was prepared from
2.00 × 10−3 g of sodium saccharide, Na(C7H4NSO3), what are the final concentrations of saccharine and sodium saccharide in the
solution?

14.E.6.23: S14.6.4
The molar mass of sodium saccharide is 205.169 g/mol. Using the abbreviations HA for saccharin and NaA for sodium saccharide
the number of moles of NaA in the solution is:
9.75 × 10−6 mol. This ionizes initially to form saccharin ions, A−, with: [A−] = 3.9 × 10−5 M

14.E.6.24: Q14.6.5
What is the pH of 1.000 L of a solution of 100.0 g of glutamic acid (C5H9NO4, a diprotic acid; K1 = 8.5 × 10−5, K2 = 3.39 × 10−10)
to which has been added 20.0 g of NaOH during the preparation of monosodium glutamate, the flavoring agent? What is the pH
when exactly 1 mol of NaOH per mole of acid has been added?

14.E.7: 14.7: Acid-Base Titrations


14.E.7.1: Q14.7.1
Explain how to choose the appropriate acid-base indicator for the titration of a weak base with a strong acid.

14.E.7.2: S14.7.1
At the equivalence point in the titration of a weak base with a strong acid, the resulting solution is slightly acidic due to the
presence of the conjugate acid. Thus, pick an indicator that changes color in the acidic range and brackets the pH at the equivalence
point. Methyl orange is a good example.

14.E.7.3: Q14.7.2
Explain why an acid-base indicator changes color over a range of pH values rather than at a specific pH.

14.E.7.4: Q14.7.3
Why can we ignore the contribution of water to the concentrations of H3O+ in the solutions of following acids:
0.0092 M HClO, a weak acid
0.0810 M HCN, a weak acid
0.120 M Fe(H O) a weak acid, Ka = 1.6 × 10−7
2+

2 6

but not the contribution of water to the concentration of OH−?

14.E.7.5: S14.7.3
In an acid solution, the only source of OH− ions is water. We use Kw to calculate the concentration. If the contribution from water
was neglected, the concentration of OH− would be zero.

14.E.7.6: Q14.7.4
We can ignore the contribution of water to the concentration of OH− in a solution of the following bases:

Access for free at OpenStax 14.E.18 https://chem.libretexts.org/@go/page/44119


0.0784 M C6H5NH2, a weak base
0.11 M (CH3)3N, a weak base
but not the contribution of water to the concentration of H3O+?

14.E.7.7: Q14.7.5
Draw a curve for a series of solutions of HF. Plot [H3O+]total on the vertical axis and the total concentration of HF (the sum of the
concentrations of both the ionized and nonionized HF molecules) on the horizontal axis. Let the total concentration of HF vary
from 1 × 10−10 M to 1 × 10−2 M.

14.E.7.8: Q14.7.6
Draw a curve similar to that shown in Figure for a series of solutions of NH3. Plot [OH−] on the vertical axis and the total
concentration of NH3 (both ionized and nonionized NH3 molecules) on the horizontal axis. Let the total concentration of NH3 vary
from 1 × 10−10 M to 1 × 10−2 M.

14.E.7.9: Q14.7.7
Calculate the pH at the following points in a titration of 40 mL (0.040 L) of 0.100 M barbituric acid (Ka = 9.8 × 10−5) with 0.100 M
KOH.
1. no KOH added
2. 20 mL of KOH solution added

Access for free at OpenStax 14.E.19 https://chem.libretexts.org/@go/page/44119


3. 39 mL of KOH solution added
4. 40 mL of KOH solution added
5. 41 mL of KOH solution added

14.E.7.10: S14.7.7
1. pH = 2.50;
2. pH = 4.01;
3. pH = 5.60;
4. pH = 8.35;
5. pH = 11.08

14.E.7.11: Q14.7.8
The indicator dinitrophenol is an acid with a Ka of 1.1 × 10−4. In a 1.0 × 10−4-M solution, it is colorless in acid and yellow in base.
Calculate the pH range over which it goes from 10% ionized (colorless) to 90% ionized (yellow).

This page titled 14.E: Acid-Base Equilibria (Exercises) is shared under a CC BY 4.0 license and was authored, remixed, and/or curated by
OpenStax via source content that was edited to the style and standards of the LibreTexts platform; a detailed edit history is available upon request.

Access for free at OpenStax 14.E.20 https://chem.libretexts.org/@go/page/44119


CHAPTER OVERVIEW
15: Equilibria of Other Reaction Classes

A general chemistry Libretexts Textbook remixed and remastered from


OpenStax's textbook:
General Chemistry
We previously learned about aqueous solutions and their importance, as well as about solubility rules. While this gives us a picture
of solubility, that picture is not complete if we look at the rules alone. Solubility equilibrium, which we will explore in this chapter,
is a more complex topic that allows us to determine the extent to which a slightly soluble ionic solid will dissolve, and the
conditions under which precipitation.
15.1: Precipitation and Dissolution
15.2: Lewis Acids and Bases
15.3: Coupled Equilibria
15.E: Equilibria of Other Reaction Classes (Exercises)

Thumbnail: Lead (II) iodide precipitates when potassium iodide is mixed with lead (II) nitrate. (CC BY-SA 3.0
Unported; PRHaney via Wikipedia)

This page titled 15: Equilibria of Other Reaction Classes is shared under a CC BY 4.0 license and was authored, remixed, and/or curated by
OpenStax via source content that was edited to the style and standards of the LibreTexts platform; a detailed edit history is available upon request.

1
15.1: Precipitation and Dissolution
 Learning Objectives
Write chemical equations and equilibrium expressions representing solubility equilibria
Carry out equilibrium computations involving solubility, equilibrium expressions, and solute concentrations

The preservation of medical laboratory blood samples, mining of sea water for magnesium, formulation of over-the-counter
medicines such as Milk of Magnesia and antacids, and treating the presence of hard water in your home’s water supply are just a
few of the many tasks that involve controlling the equilibrium between a slightly soluble ionic solid and an aqueous solution of its
ions.
In some cases, we want to prevent dissolution from occurring. Tooth decay, for example, occurs when the calcium hydroxylapatite,
which has the formula Ca5(PO4)3(OH), in our teeth dissolves. The dissolution process is aided when bacteria in our mouths feast on
the sugars in our diets to produce lactic acid, which reacts with the hydroxide ions in the calcium hydroxylapatite. Preventing the
dissolution prevents the decay. On the other hand, sometimes we want a substance to dissolve. We want the calcium carbonate in a
chewable antacid to dissolve because the CO ions produced in this process help soothe an upset stomach.
2−

In this section, we will find out how we can control the dissolution of a slightly soluble ionic solid by the application of Le
Chatelier’s principle. We will also learn how to use the equilibrium constant of the reaction to determine the concentration of ions
present in a solution.

15.1.1: The Solubility Product Constant


Silver chloride is what’s known as a sparingly soluble ionic solid (Figure 15.1.1). Recall from the solubility rules in an earlier
chapter that halides of Ag+ are not normally soluble. However, when we add an excess of solid AgCl to water, it dissolves to a
small extent and produces a mixture consisting of a very dilute solution of Ag+ and Cl– ions in equilibrium with undissolved silver
chloride:
dissolution
+ −
AgCl(s) ⇌ Ag (aq) + Cl (aq)
precipitation

This equilibrium, like other equilibria, is dynamic; some of the solid AgCl continues to dissolve, but at the same time, Ag+ and Cl–
ions in the solution combine to produce an equal amount of the solid. At equilibrium, the opposing processes have equal rates.

Figure 15.1.1 : Silver chloride is a sparingly soluble ionic solid. When it is added to water, it dissolves slightly and produces a
mixture consisting of a very dilute solution of Ag+ and Cl– ions in equilibrium with undissolved silver chloride.
The equilibrium constant for the equilibrium between a slightly soluble ionic solid and a solution of its ions is called the solubility
product (Ksp) of the solid. Recall from the chapter on solutions and colloids that we use an ion’s concentration as an approximation
of its activity in a dilute solution. For silver chloride, at equilibrium:
+ −
AgCl(s) ⇌ Ag (aq) + Cl (aq)

with
+ −
Ksp = [ Ag (aq)][ Cl (aq)]

When looking at dissolution reactions such as this, the solid is listed as a reactant, whereas the ions are listed as products. The
solubility product constant, as with every equilibrium constant expression, is written as the product of the concentrations of each of

Access for free at OpenStax 15.1.1 https://chem.libretexts.org/@go/page/38285


the ions, raised to the power of their stoichiometric coefficients. Here, the solubility product constant is equal to Ag+ and Cl– when
a solution of silver chloride is in equilibrium with undissolved AgCl. There is no denominator representing the reactants in this
equilibrium expression since the reactant is a pure solid; therefore [AgCl] does not appear in the expression for Ksp.
Table 15.1.1 : Common Solubility Products by Decreasing Equilibrium Constants
Substance Ksp at 25 °C

CuCl 1.2 × 10–6

CuBr 6.27 × 10–9

AgI 1.5 × 10–16

PbS 7 × 10–29

Al(OH)3 2 × 10–32

Fe(OH)3 4 × 10–38

Some common solubility products are listed in Table 15.1.1 according to their Ksp values, whereas a more extensive compilation of
products appears in Table E3. Each of these equilibrium constants is much smaller than 1 because the compounds listed are only
slightly soluble. A small Ksp represents a system in which the equilibrium lies to the left, so that relatively few hydrated ions would
be present in a saturated solution.

 Example 15.1.1: Writing Equations and Solubility Products

Write the ionic equation for the dissolution and the solubility product expression for each of the following slightly soluble ionic
compounds:
a. AgI, silver iodide, a solid with antiseptic properties
b. CaCO3, calcium carbonate, the active ingredient in many over-the-counter chewable antacids
c. Mg(OH)2, magnesium hydroxide, the active ingredient in Milk of Magnesia
d. Mg(NH4)PO4, magnesium ammonium phosphate, an essentially insoluble substance used in tests for magnesium
e. Ca5(PO4)3OH, the mineral apatite, a source of phosphate for fertilizers
(Hint: When determining how to break (d) and (e) up into ions, refer to the list of polyatomic ions in the section on chemical
nomenclature.)

Solution
a. AgI(s) ⇌ Ag (aq) + I (aq)
+ −
Ksp = [ Ag
+ −
][ I ]

b. CaCO (s) ⇌ Ca (aq) + CO


3
2+ 2−

3
(aq) Ksp = [ Ca
2+
][ CO
2−

3
]

− 2
c. Mg(OH) (s) ⇌ Mg (aq) + 2 OH (aq) K = [Mg ][OH
2
2+ −
sp
2+
]

d. Mg(NH )PO (s) ⇌ Mg (aq) + NH (aq) + PO (aq) K


4 4
2+ +

4
3−
4 sp = [ Mg
2+
][ NH
+

4
][ PO4
3−
]
5 3
e. Ca (PO ) OH(s) ⇌ 5 Ca
5 4 3
2+
(aq) + 3 PO
3−

4
(aq) + OH

(aq) Ksp = [ Ca
2+
] [ PO
3−

4
] [ OH

]

 Exercise 15.1.1
Write the ionic equation for the dissolution and the solubility product for each of the following slightly soluble compounds:
a. BaSO4
b. Ag2SO4
c. Al(OH)3
d. Pb(OH)Cl

Answer a
2+ 2− 2+ 2−
BaSO (s) ⇌ Ba (aq) + SO (aq) Ksp = [ Ba ][ SO ]
4 4 4

Answer b

Access for free at OpenStax 15.1.2 https://chem.libretexts.org/@go/page/38285


+ 2− + 2 2−
Ag SO (s) ⇌ 2 Ag (aq) + SO (aq) Ksp = [ Ag ] [ SO ]
2 4 4 4

Answer c
2+ − 3+ − 3
Al (OH) (s) ⇌ Al (aq) + 3 OH (aq) Ksp = [ Al ] [ OH ]
3

Answer d
2+ − − 2+ − −
Pb(OH)Cl(s) ⇌ Pb (aq) + OH (aq) + Cl (aq) Ksp = [ Pb ][ OH ][ Cl ]

Now we will extend the discussion of Ksp and show how the solubility product constant is determined from the solubility of its
ions, as well as how Ksp can be used to determine the molar solubility of a substance.

15.1.2: Ksp and Solubility


Recall that the definition of solubility is the maximum possible concentration of a solute in a solution at a given temperature and
pressure. We can determine the solubility product of a slightly soluble solid from that measure of its solubility at a given
temperature and pressure, provided that the only significant reaction that occurs when the solid dissolves is its dissociation into
solvated ions, that is, the only equilibrium involved is:
m+ n−
Mp Xq (s) ⇌ p M (aq) + q X (aq)

In this case, we calculate the solubility product by taking the solid’s solubility expressed in units of moles per liter (mol/L), known
as its molar solubility.

 Example 15.1.2: Calculation of Ksp from Equilibrium Concentrations

We began the chapter with an informal discussion of how the mineral fluorite is formed. Fluorite, CaF2, is a slightly soluble
solid that dissolves according to the equation:
2+ −
CaF (s) ⇌ Ca (aq) + 2 F (aq)
2

The concentration of Ca2+ in a saturated solution of CaF2 is 2.1 × 10–4 M; therefore, that of F– is 4.2 × 10–4 M, that is, twice the
concentration of Ca2+. What is the solubility product of fluorite?

Solution
First, write out the Ksp expression, then substitute in concentrations and solve for Ksp:
2+ −
CaF (s) ⇌ Ca (aq) + 2 F (aq)
2

A saturated solution is a solution at equilibrium with the solid. Thus:


2+ − 2
Ksp = [ Ca ][F ]

−4 −4 2
= (2.1 × 10 )(4.2 × 10 )

−11
= 3.7 × 10

As with other equilibrium constants, we do not include units with Ksp.

 Exercise 15.1.2

In a saturated solution that is in contact with solid Mg(OH)2, the concentration of Mg2+ is 3.7 × 10–5 M. What is the solubility
product for Mg(OH)2?
2+ −
Mg (OH) (s) ⇌ Mg (aq) + 2 OH (aq)
2

Answer
2.0 × 10–13

Access for free at OpenStax 15.1.3 https://chem.libretexts.org/@go/page/38285


 Example 15.1.3: Determination of Molar Solubility from Ksp
The Ksp of copper(I) bromide, CuBr, is 6.3 × 10–9. Calculate the molar solubility of copper bromide.

Solution
The solubility product constant of copper(I) bromide is 6.3 × 10 . –9

The reaction is:


+ −
CuBr(s) ⇌ Cu (aq) + Br (aq)

First, write out the solubility product equilibrium constant expression:


+ −
Ksp = [ Cu ][ Br ]

Create an ICE table (as introduced in the chapter on fundamental equilibrium concepts), leaving the CuBr column empty as it
is a solid and does not contribute to the Ksp:

At equilibrium:
+ −
Ksp = [ Cu ][ Br ]

−9 2
6.3 × 10 = (x)(x) = x

−−−−−−−−−−
−9 −5
x = √ (6.3 × 10 ) = 7.9 × 10

Therefore, the molar solubility of CuBr is 7.9 × 10–5 M.

 Exercise 15.1.3

The Ksp of AgI is 1.5 × 10–16. Calculate the molar solubility of silver iodide.

Answer
1.2 × 10–8 M

 Example 15.1.4: Determination of Molar Solubility from Ksp, Part II

Determination of Molar Solubility from Ksp, Part II The Ksp of calcium hydroxide, Ca(OH)2, is 8.0 × 10–6. Calculate the molar
solubility of calcium hydroxide.

Solution
The solubility product constant of calcium hydroxide is 1.3 × 10–6.
The reaction is:
2+ −
Ca (OH) (s) ⇌ Ca (aq) + 2 OH (aq)
2

First, write out the solubility product equilibrium constant expression:


2+ − 2
Ksp = [ Ca ] [ OH ]

Access for free at OpenStax 15.1.4 https://chem.libretexts.org/@go/page/38285


Create an ICE table, leaving the Ca(OH)2 column empty as it is a solid and does not contribute to the Ksp:

At equilibrium:
2+ − 2
Ksp = [ Ca ] [ OH ]

−6 2 2 3
1.3 × 10 = (x)(2x ) = (x)(4 x ) = 4 x

−−−−−−−−−
−6
1.3 × 10
3 −3
x =√ = 6.9 × 10
4

Therefore, the molar solubility of Ca(OH)2 is 6.9 × 10–3 M.

 Exercise 15.1.4

The Ksp of PbI2 is 1.4 × 10–8. Calculate the molar solubility of lead(II) iodide.

Answer
1.5 × 10–3 M

Note that solubility is not always given as a molar value. When the solubility of a compound is given in some unit other than moles
per liter, we must convert the solubility into moles per liter (i.e., molarity) in order to use it in the solubility product constant
expression. Example 15.1.5 shows how to perform those unit conversions before determining the solubility product equilibrium.

 Example 15.1.5: Determination of Ksp from Gram Solubility

Many of the pigments used by artists in oil-based paints (Figure 15.1.2) are sparingly soluble in water. For example, the
solubility of the artist’s pigment chrome yellow, PbCrO4, is 4.6 × 10–6 g/L. Determine the solubility product equilibrium
constant for PbCrO4.

Figure 15.1.2 : Oil paints contain pigments that are very slightly soluble in water. In addition to chrome yellow (PbCrO4),
examples include Prussian blue (Fe7(CN)18), the reddish-orange color vermilion (HgS), and green color veridian (Cr2O3).
(credit: Sonny Abesamis)

Solution
We are given the solubility of PbCrO4 in grams per liter. If we convert this solubility into moles per liter, we can find the
equilibrium concentrations of Pb2+ and CrO , then Ksp:
2−
4

Access for free at OpenStax 15.1.5 https://chem.libretexts.org/@go/page/38285


323.2 g
1. Use the molar mass of PbCrO4 ( ) to convert the solubility of PbCrO4 in grams per liter into moles per liter:
1 mol
−6
4.6 × 10 g PbCrO4 1 mol PbCrO4
[PbCrO4 ] = ×
1 L 323.2 g PbCrO4

−8
1.4 × 10 mol PbCrO4
=
1 L

−8
= 1.4 × 10 M

The chemical equation for the dissolution indicates that 1 mol of PbCrO4 gives 1 mol of Pb2+(aq) and 1 mol of CrO 2 −

4
(aq) :
2+ 2−
PbCrO (s) ⇌ Pb (aq) + CrO (aq)
4 4

2+
Thus, both [Pb ] and [CrO 2−

4
] are equal to the molar solubility of PbCrO4:
2+ 2− −8
[ Pb ] = [ CrO ] = 1.4 × 10 M
4
2+
Solve. Ksp = [Pb ] [CrO 2−

4
] = (1.4 × 10–8)(1.4 × 10–8) = 2.0 × 10–16

 Exercise 15.1.5

The solubility of TlCl [thallium(I) chloride], an intermediate formed when thallium is being isolated from ores, is 3.46 grams
per liter at 20 °C. What is its solubility product?

Answer
2.08 × 10–4

 Example 15.1.6: Calculating the Solubility of Hg2Cl2

Calomel, Hg2Cl2, is a compound composed of the diatomic ion of mercury(I), Hg , and chloride ions, Cl–. Although most 2+

mercury compounds are now known to be poisonous, eighteenth-century physicians used calomel as a medication. Their
patients rarely suffered any mercury poisoning from the treatments because calomel is quite insoluble:
2+ − −18
Hg Cl (s) ⇌ Hg (aq) + 2 Cl (aq) Ksp = 1.1 × 10
2 2 2

Calculate the molar solubility of Hg2Cl2.

Solution
The molar solubility of Hg2Cl2 is equal to the concentration of Hg 2+
2
ions because for each 1 mol of Hg2Cl2 that dissolves, 1
mol of Hg forms: 2+

1. Determine the direction of change. Before any Hg2Cl2 dissolves, Q is zero, and the reaction will shift to the right to reach
equilibrium.
2. Determine x and equilibrium concentrations. Concentrations and changes are given in the following ICE table:

Access for free at OpenStax 15.1.6 https://chem.libretexts.org/@go/page/38285


Note that the change in the concentration of Cl– (2x) is twice as large as the change in the concentration of Hg (x) because 2 2+
2

mol of Cl– forms for each 1 mol of Hg that forms. Hg2Cl2 is a pure solid, so it does not appear in the calculation.
2+

3. Solve for x and the equilibrium concentrations. We substitute the equilibrium concentrations into the expression for Ksp and
calculate the value of x:
2+ − 2
Ksp = [ Hg ] [ Cl ]
2

−18 2
1.1 × 10 = (x)(2x)

3 −18
4x = 1.1 × 10

−−−−−−−−−−−−
−18
1.1 × 10 −7
3
x = √( ) = 6.5 × 10 M
4

So the concentrations are


2+ −7 −7
[ Hg ] = 6.5 × 10 M = 6.5 × 10 M
2

− −7 −6
[ Cl ] = 2x = 2(6.5 × 10 ) = 1.3 × 10 M

The molar solubility of Hg2Cl2 is equal to [Hg 2+


2
] , or 6.5 × 10–7 M.

Check the work.


At equilibrium, Q = Ksp:
2+ − 2 −7 −6 2 −18
Q = [ Hg ] [ Cl ] = (6.5 × 10 )(1.3 × 10 ) = 1.1 × 10
2

The calculations check.

 Exercise 15.1.6

Determine the molar solubility of MgF2 from its solubility product: Ksp = 6.4 × 10–9.

Answer
1.2 × 10–3 M

Tabulated Ksp values can also be compared to reaction quotients calculated from experimental data to tell whether a solid will
precipitate in a reaction under specific conditions: Q equals Ksp at equilibrium; if Q is less than Ksp, the solid will dissolve until Q
equals Ksp; if Q is greater than Ksp, precipitation will occur at a given temperature until Q equals Ksp.

 Using Barium Sulfate for Medical Imaging

Various types of medical imaging techniques are used to aid diagnoses of illnesses in a noninvasive manner. One such
technique utilizes the ingestion of a barium compound before taking an X-ray image. A suspension of barium sulfate, a chalky
powder, is ingested by the patient. Since the Ksp of barium sulfate is 1.1 × 10–10, very little of it dissolves as it coats the lining
of the patient’s intestinal tract. Barium-coated areas of the digestive tract then appear on an X-ray as white, allowing for greater
visual detail than a traditional X-ray (Figure 15.1.3).

Access for free at OpenStax 15.1.7 https://chem.libretexts.org/@go/page/38285


Figure 15.1.3 : The suspension of barium sulfate coats the intestinal tract, which allows for greater visual detail than a
traditional X-ray. (credit modification of work by “glitzy queen00”/Wikimedia Commons).

Further diagnostic testing can be done using barium sulfate and fluoroscopy. In fluoroscopy, a continuous X-ray is passed
through the body so the doctor can monitor, on a TV or computer screen, the barium sulfate’s movement as it passes through
the digestive tract. Medical imaging using barium sulfate can be used to diagnose acid reflux disease, Crohn’s disease, and
ulcers in addition to other conditions.

15.1.3: Predicting Precipitation


The equation that describes the equilibrium between solid calcium carbonate and its solvated ions is:
2+ 2−
CaCO (s) ⇌ Ca (aq) + CO3 (aq)
3

We can establish this equilibrium either by adding solid calcium carbonate to water or by mixing a solution that contains calcium
ions with a solution that contains carbonate ions. If we add calcium carbonate to water, the solid will dissolve until the
concentrations are such that the value of the reaction quotient (Q=[Ca ][CO ]) is equal to the solubility product (Ksp = 4.8 ×
2+ 2−
3

10–9). If we mix a solution of calcium nitrate, which contains Ca2+ ions, with a solution of sodium carbonate, which contains CO 2−

ions, the slightly soluble ionic solid CaCO3 will precipitate, provided that the concentrations of Ca2+ and CO ions are such that
2−

Q is greater than Ksp for the mixture. The reaction shifts to the left and the concentrations of the ions are reduced by formation of
the solid until the value of Q equals Ksp. A saturated solution in equilibrium with the undissolved solid will result. If the
concentrations are such that Q is less than Ksp, then the solution is not saturated and no precipitate will form.
We can compare numerical values of Q with Ksp to predict whether precipitation will occur, as Example 15.1.7 shows. (Note: Since
all forms of equilibrium constants are temperature dependent, we will assume a room temperature environment going forward in
this chapter unless a different temperature value is explicitly specified.)

 Example 15.1.7: Precipitation of Mg(OH)2


The first step in the preparation of magnesium metal is the precipitation of Mg(OH)2 from sea water by the addition of lime,
Ca(OH)2, a readily available inexpensive source of OH– ion:
2+ − −12
Mg (OH) (s) ⇌ Mg (aq) + 2 OH (aq) Ksp = 8.9 × 10
2

The concentration of Mg2+(aq) in sea water is 0.0537 M. Will Mg(OH)2 precipitate when enough Ca(OH)2 is added to give a
[OH–] of 0.0010 M?

Solution
This problem asks whether the reaction:
2+ −
Mg (OH) (s) ⇌ Mg (aq) + 2 OH (aq)
2

Access for free at OpenStax 15.1.8 https://chem.libretexts.org/@go/page/38285


shifts to the left and forms solid Mg(OH)2 when [Mg2+] = 0.0537 M and [OH–] = 0.0010 M. The reaction shifts to the left if Q
is greater than Ksp. Calculation of the reaction quotient under these conditions is shown here:
2+ − 2 2 −8
Q = [Mg ][OH ] = (0.0537)(0.0010 ) = 5.4 × 10

Because Q is greater than Ksp (Q = 5.4 × 10–8 is larger than Ksp = 8.9 × 10–12), we can expect the reaction to shift to the left
and form solid magnesium hydroxide. Mg(OH)2(s) forms until the concentrations of magnesium ion and hydroxide ion are
reduced sufficiently so that the value of Q is equal to Ksp.

 Exercise 15.1.7
Use the solubility products in Table E3 to determine whether CaHPO4 will precipitate from a solution with [Ca2+] = 0.0001 M
and [HPO ] = 0.001 M.
2−

Answer
No precipitation of CaHPO4; Q = 1 × 10–7, which is less than Ksp

 Example 15.1.8: Precipitation of AgCl upon Mixing Solutions

Does silver chloride precipitate when equal volumes of a 2.0 × 10–4-M solution of AgNO3 and a 2.0 × 10–4-M solution of NaCl
are mixed?
(Note: The solution also contains Na+ and NO ions, but when referring to solubility rules, one can see that sodium nitrate is

very soluble and cannot form a precipitate.)

Solution
The equation for the equilibrium between solid silver chloride, silver ion, and chloride ion is:
+ −
AgCl(s) ⇌ Ag (aq) + Cl (aq)

The solubility product is 1.8 × 10–10 (Table E3).


AgCl will precipitate if the reaction quotient calculated from the concentrations in the mixture of AgNO3 and NaCl is greater than
Ksp. The volume doubles when we mix equal volumes of AgNO3 and NaCl solutions, so each concentration is reduced to half its
initial value. Consequently, immediately upon mixing, [Ag+] and [Cl–] are both equal to:
1
−4 −4
(2.0 × 10 ) M = 1.0 × 10 M
2

The reaction quotient, Q, is momentarily greater than Ksp for AgCl, so a supersaturated solution is formed:
+ − −4 −4 −8
Q = [ Ag ][ Cl ] = (1.0 × 10 )(1.0 × 10 ) = 1.0 × 10 > Ksp

Since supersaturated solutions are unstable, AgCl will precipitate from the mixture until the solution returns to equilibrium, with Q
equal to Ksp.

 Exercise 15.1.8

Will KClO4 precipitate when 20 mL of a 0.050-M solution of K+ is added to 80 mL of a 0.50-M solution of ClO ? −

(Remember to calculate the new concentration of each ion after mixing the solutions before plugging into the reaction quotient
expression.)

Answer
No, Q = 4.0 × 10–3, which is less than Ksp = 1.07 × 10–2

In the previous two examples, we have seen that Mg(OH)2 or AgCl precipitate when Q is greater than Ksp. In general, when a
solution of a soluble salt of the Mm+ ion is mixed with a solution of a soluble salt of the Xn– ion, the solid, MpXq precipitates if the

Access for free at OpenStax 15.1.9 https://chem.libretexts.org/@go/page/38285


value of Q for the mixture of Mm+ and Xn– is greater than Ksp for MpXq. Thus, if we know the concentration of one of the ions of a
slightly soluble ionic solid and the value for the solubility product of the solid, then we can calculate the concentration that the
other ion must exceed for precipitation to begin. To simplify the calculation, we will assume that precipitation begins when the
reaction quotient becomes equal to the solubility product constant.

 Example 15.1.9: Precipitation of Calcium Oxalate

Blood will not clot if calcium ions are removed from its plasma. Some blood collection tubes contain salts of the oxalate ion,
C O
2
, for this purpose (Figure 15.1.4). At sufficiently high concentrations, the calcium and oxalate ions form solid,
2−
4

CaC2O4•H2O (which also contains water bound in the solid). The concentration of Ca2+ in a sample of blood serum is 2.2 ×
10–3 M. What concentration of C O ion must be established before CaC2O4•H2O begins to precipitate?
2
2−

Figure 15.1.4 : Anticoagulants can be added to blood that will combine with the Ca2+ ions in blood serum and prevent the
blood from clotting. (credit: modification of work by Neeta Lind)

Solution
The equilibrium expression is:
2+ 2−
CaC O (s) ⇌ Ca (aq) + C O4 (aq)
2 4 2

For this reaction (Table E3):


2+ 2− −8
Ksp = [ Ca ][ C O ] = 1.96 × 10
2 4

CaC2O4 does not appear in this expression because it is a solid. Water does not appear because it is the solvent.
Solid CaC2O4 does not begin to form until Q equals Ksp. Because we know Ksp and [Ca2+], we can solve for the concentration
of C O that is necessary to produce the first trace of solid:
2
2−
4

2+ 2− −8
Q = Ksp = [ Ca ][ C O4 ] = 1.96 × 10
2

−3 2− −8
(2.2 × 10 )[ C O ] = 1.96 × 10
2 4
−8
2−
1.96 × 10
−6
[C O ] = = 8.9 × 10
2 4
−3
2.2 × 10

A concentration of [C 2
2−
O
4
] = 8.9 × 10–6 M is necessary to initiate the precipitation of CaC2O4 under these conditions.

 Exercise 15.1.9

If a solution contains 0.0020 mol of CrO per liter, what concentration of Ag+ ion must be reached by adding solid AgNO3
2−
4

before Ag2CrO4 begins to precipitate? Neglect any increase in volume upon adding the solid silver nitrate.

Answer
4.5 × 10–9 M

Access for free at OpenStax 15.1.10 https://chem.libretexts.org/@go/page/38285


It is sometimes useful to know the concentration of an ion that remains in solution after precipitation. We can use the solubility
product for this calculation too: If we know the value of Ksp and the concentration of one ion in solution, we can calculate the
concentration of the second ion remaining in solution. The calculation is of the same type as that in Example 15.1.8—calculation
of the concentration of a species in an equilibrium mixture from the concentrations of the other species and the equilibrium
constant. However, the concentrations are different; we are calculating concentrations after precipitation is complete, rather than at
the start of precipitation.

 Example 15.1.10: Concentrations Following Precipitation

Clothing washed in water that has a manganese [Mn2+(aq)] concentration exceeding 0.1 mg/L (1.8 × 10–6 M) may be stained
by the manganese upon oxidation, but the amount of Mn2+ in the water can be reduced by adding a base. If a person doing
laundry wishes to add a buffer to keep the pH high enough to precipitate the manganese as the hydroxide, Mn(OH)2, what pH
is required to keep [Mn2+] equal to 1.8 × 10–6 M?

Solution
The dissolution of Mn(OH)2 is described by the equation:
2+ − −13
Mn(OH) (s) ⇌ Mn (aq) + 2 OH (aq) Ksp = 2 × 10
2

We need to calculate the concentration of OH– when the concentration of Mn2+ is 1.8 × 10–6 M. From that, we calculate the
pH. At equilibrium:
2+ − 2
Ksp = [ Mn ] [ OH ]

or
−6 − 2 −13
(1.8 × 10 )[ OH ] = 2 × 10

so
− −4
[ OH ] = 3.3 × 10 M

Now we calculate the pH from the pOH:


− −4
pOH = − log[OH ] = − log(3.3 × 10 ) = 3.48

pH = 14.00 − pOH = 14.00 − 3.48 = 10.52

If the person doing laundry adds a base, such as the sodium silicate (Na4SiO4) in some detergents, to the wash water until the pH is
raised to 10.52, the manganese ion will be reduced to a concentration of 1.8 × 10–6 M; at that concentration or less, the ion will not
stain clothing.

 Exercise 15.1.10
The first step in the preparation of magnesium metal is the precipitation of Mg(OH)2 from sea water by the addition of
Ca(OH)2. The concentration of Mg2+(aq) in sea water is 5.37 × 10–2 M. Calculate the pH at which [Mg2+] is diminished to 1.0
× 10–5 M by the addition of Ca(OH)2.

Answer
10.97

Due to their light sensitivity, mixtures of silver halides are used in fiber optics for medical lasers, in photochromic eyeglass lenses
(glass lenses that automatically darken when exposed to sunlight), and—before the advent of digital photography—in photographic
film. Even though AgCl (Ksp = 1.6 × 10–10), AgBr (Ksp = 5.0 × 10–13), and AgI (Ksp = 1.5 × 10–16) are each quite insoluble, we
cannot prepare a homogeneous solid mixture of them by adding Ag+ to a solution of Cl–, Br–, and I–; essentially all of the AgI will
precipitate before any of the other solid halides form because of its smaller value for Ksp. However, we can prepare a homogeneous
mixture of the solids by slowly adding a solution of Cl–, Br–, and I– to a solution of Ag+.

Access for free at OpenStax 15.1.11 https://chem.libretexts.org/@go/page/38285


When two anions form slightly soluble compounds with the same cation, or when two cations form slightly soluble compounds
with the same anion, the less soluble compound (usually, the compound with the smaller Ksp) generally precipitates first when we
add a precipitating agent to a solution containing both anions (or both cations). When the Ksp values of the two compounds differ
by two orders of magnitude or more (e.g., 10–2 vs. 10–4), almost all of the less soluble compound precipitates before any of the
more soluble one does. This is an example of selective precipitation, where a reagent is added to a solution of dissolved ions
causing one of the ions to precipitate out before the rest.

15.1.4: The Role of Precipitation in Wastewater Treatment


Solubility equilibria are useful tools in the treatment of wastewater carried out in facilities that may treat the municipal water in
your city or town (Figure 15.1.5). Specifically, selective precipitation is used to remove contaminants from wastewater before it is
released back into natural bodies of water. For example, phosphate ions (PO ) are often present in the water discharged from
2−
4

manufacturing facilities. An abundance of phosphate causes excess algae to grow, which impacts the amount of oxygen available
for marine life as well as making water unsuitable for human consumption.

Figure 15.1.5 : Wastewater treatment facilities, such as this one, remove contaminants from wastewater before the water is released
back into the natural environment. (credit: “eutrophication&hypoxia”/Wikimedia Commons)
One common way to remove phosphates from water is by the addition of calcium hydroxide, known as lime, Ca(OH)2. The lime is
converted into calcium carbonate, a strong base, in the water. As the water is made more basic, the calcium ions react with
phosphate ions to produce hydroxylapatite, Ca5(PO4)3(OH), which then precipitates out of the solution:
2+ 3− −
5 Ca + 3 PO + OH ⇌ Ca (PO ) ⋅ (OH) (s)
4 10 4 6 2

The precipitate is then removed by filtration and the water is brought back to a neutral pH by the addition of CO2 in a recarbonation
process. Other chemicals can also be used for the removal of phosphates by precipitation, including iron(III) chloride and
aluminum sulfate.
Selective precipitation can also be used in qualitative analysis. In this method, reagents are added to an unknown chemical mixture
in order to induce precipitation. Certain reagents cause specific ions to precipitate out; therefore, the addition of the reagent can be
used to determine whether the ion is present in the solution.

 Example 15.1.11: Precipitation of Silver Halides


A solution contains 0.0010 mol of KI and 0.10 mol of KCl per liter. AgNO3 is gradually added to this solution. Which forms
first, solid AgI or solid AgCl?

Solution
The two equilibria involved are:
+ − −10
AgCl(s) ⇌ Ag (aq) + Cl (aq) Ksp = 1.6 × 10

+ − −16
AgI(s) ⇌ Ag (aq) + I (aq) Ksp = 1.5 × 10

If the solution contained about equal concentrations of Cl– and I–, then the silver salt with the smallest Ksp (AgI) would precipitate
first. The concentrations are not equal, however, so we should find the [Ag+] at which AgCl begins to precipitate and the [Ag+] at
which AgI begins to precipitate. The salt that forms at the lower [Ag+] precipitates first.

Access for free at OpenStax 15.1.12 https://chem.libretexts.org/@go/page/38285


For AgI: AgI precipitates when Q equals Ksp for AgI (1.5 × 10–16). When [I–] = 0.0010 M:
+ − + −16
Q = [ Ag ][ I ] = [ Ag ](0.0010) = 1.5 × 10
−16
+
1.5 × 10
−13
[ Ag ] = = 1.5 × 10
0.0010

AgI begins to precipitate when [Ag+] is 1.5 × 10–13 M.


For AgCl: AgCl precipitates when Q equals Ksp for AgCl (1.6 × 10–10). When [Cl–] = 0.10 M:
+ − + −10
Qsp = [ Ag ][ Cl ] = [ Ag ](0.10) = 1.6 × 10
−10
+
1.6 × 10
−9
[ Ag ] = = 1.6 × 10 M
0.10

AgCl begins to precipitate when [Ag+] is 1.6 × 10–9 M.


AgI begins to precipitate at a lower [Ag+] than AgCl, so AgI begins to precipitate first.

 Exercise 15.1.11

If silver nitrate solution is added to a solution which is 0.050 M in both Cl– and Br– ions, at what [Ag+] would precipitation
begin, and what would be the formula of the precipitate?

Answer
[Ag+] = 1.0 × 10–11 M; AgBr precipitates first

15.1.5: Common Ion Effect


As we saw when we discussed buffer solutions, the hydronium ion concentration of an aqueous solution of acetic acid decreases
when the strong electrolyte sodium acetate, NaCH3CO2, is added. We can explain this effect using Le Chatelier’s principle. The
addition of acetate ions causes the equilibrium to shift to the left, decreasing the concentration of H O to compensate for the
3
+

increased acetate ion concentration. This increases the concentration of CH3CO2H:


\[\ce{CH3CO2H + H2O \rightleftharpoons H3O+ + CH3CO2-}\)]
Because sodium acetate and acetic acid have the acetate ion in common, the influence on the equilibrium is called the common ion
effect. The common ion effect can also have a direct effect on solubility equilibria. Suppose we are looking at the reaction where
silver iodide is dissolved:
+ −
AgI(s) ⇌ Ag (aq) + I (aq)

If we were to add potassium iodide (KI) to this solution, we would be adding a substance that shares a common ion with silver
iodide. Le Chatelier’s principle tells us that when a change is made to a system at equilibrium, the reaction will shift to counteract
that change. In this example, there would be an excess of iodide ions, so the reaction would shift toward the left, causing more
silver iodide to precipitate out of solution.

 Example 15.1.12: Common Ion Effect

Calculate the molar solubility of cadmium sulfide (CdS) in a 0.010-M solution of cadmium bromide (CdBr2). The Ksp of CdS
is 1.0 × 10–28.

Solution
The first thing you should notice is that the cadmium sulfide is dissolved in a solution that contains cadmium ions. We need to
use an ICE table to set up this problem and include the CdBr2 concentration as a contributor of cadmium ions:
2+ 2−
CdS(s) ⇌ Cd (aq) + S (aq)

Access for free at OpenStax 15.1.13 https://chem.libretexts.org/@go/page/38285


This table has two main columns and four rows. The first row for the first column does not have a heading and then has the
following in the first column: Initial concentration ( M ), Change ( M ), and Equilibrium concentration ( M ). The second
column has the header, “C d S equilibrium arrow C d to the second power plus S to the second power superscript negative
sign.” Under the second column is a subgroup of three rows and three columns. The first column is blank. The second column
has the following: 0.010, x, 0.010 plus x. The third column has the following: 0, x, 0 plus x equals x.

2+ 2− −28
Ksp = [ Cd ][ S ] = 1.0 × 10

−28 2 −28
(0.010 + x)(x) = 1.0 × 10 x + 0.010x − 1.0 × 10 =0

We can solve this equation using the quadratic formula, but we can also make an assumption to make this calculation much
simpler. Since the Ksp value is so small compared with the cadmium concentration, we can assume that the change between the
initial concentration and the equilibrium concentration is negligible, so that 0.010 + x ~ 0.010. Going back to our Ksp expression,
we would now get:
2+ 2− −28
Ksp = [ Cd ][ S ] = 1.0 × 10
−28
(0.010)(x) = 1.0 × 10
−26
x = 1.0 × 10

Therefore, the molar solubility of CdS in this solution is 1.0 × 10–26 M.

 Exercise 15.1.12

Calculate the molar solubility of aluminum hydroxide, Al(OH)3, in a 0.015-M solution of aluminum nitrate, Al(NO3)3. The Ksp
of Al(OH)3 is 2 × 10–32.

Answer
4 × 10–11

Summary
The equilibrium constant for an equilibrium involving the precipitation or dissolution of a slightly soluble ionic solid is called the
solubility product, Ksp, of the solid. When we have a heterogeneous equilibrium involving the slightly soluble solid MpXq and its
ions Mm+ and Xn–:
m+ n−
Mp Xq (s) ⇌ p M (aq) + q X (aq)

We write the solubility product expression as:


m+ p n− q
Ksp = [ M ] [X ]

The solubility product of a slightly soluble electrolyte can be calculated from its solubility; conversely, its solubility can be
calculated from its Ksp, provided the only significant reaction that occurs when the solid dissolves is the formation of its ions. A
slightly soluble electrolyte begins to precipitate when the magnitude of the reaction quotient for the dissolution reaction exceeds the
magnitude of the solubility product. Precipitation continues until the reaction quotient equals the solubility product. A reagent can
be added to a solution of ions to allow one ion to selectively precipitate out of solution. The common ion effect can also play a role
in precipitation reactions. In the presence of an ion in common with one of the ions in the solution, Le Chatelier’s principle applies
and more precipitate comes out of solution so that the molar solubility is reduced.

Glossary
common ion effect

Access for free at OpenStax 15.1.14 https://chem.libretexts.org/@go/page/38285


effect on equilibrium when a substance with an ion in common with the dissolved species is added to the solution; causes a
decrease in the solubility of an ionic species, or a decrease in the ionization of a weak acid or base

molar solubility
solubility of a compound expressed in units of moles per liter (mol/L)

selective precipitation
process in which ions are separated using differences in their solubility with a given precipitating reagent

solubility product (Ksp)


equilibrium constant for the dissolution of a slightly soluble electrolyte

This page titled 15.1: Precipitation and Dissolution is shared under a CC BY 4.0 license and was authored, remixed, and/or curated by OpenStax
via source content that was edited to the style and standards of the LibreTexts platform; a detailed edit history is available upon request.

Access for free at OpenStax 15.1.15 https://chem.libretexts.org/@go/page/38285


15.2: Lewis Acids and Bases
 Learning Objectives
Explain the Lewis model of acid-base chemistry
Write equations for the formation of adducts and complex ions
Perform equilibrium calculations involving formation constants

In 1923, G. N. Lewis proposed a generalized definition of acid-base behavior in which acids and bases are identified by their ability
to accept or to donate a pair of electrons and form a coordinate covalent bond.
A coordinate covalent bond (or dative bond) occurs when one of the atoms in the bond provides both bonding electrons. For
example, a coordinate covalent bond occurs when a water molecule combines with a hydrogen ion to form a hydronium ion. A
coordinate covalent bond also results when an ammonia molecule combines with a hydrogen ion to form an ammonium ion. Both
of these equations are shown here.

Definition: Lewis Acids and Bases


A Lewis acid is any species (molecule or ion) that can accept a pair of electrons, and a Lewis base is any species (molecule or
ion) that can donate a pair of electrons.
A Lewis acid-base reaction occurs when a base donates a pair of electrons to an acid. A Lewis acid-base adduct, a compound that
contains a coordinate covalent bond between the Lewis acid and the Lewis base, is formed. The following equations illustrate the
general application of the Lewis concept.
The boron atom in boron trifluoride, BF3, has only six electrons in its valence shell. Being short of the preferred octet, BF3 is a very
good Lewis acid and reacts with many Lewis bases; a fluoride ion is the Lewis base in this reaction, donating one of its lone pairs:

In the following reaction, each of two ammonia molecules, Lewis bases, donates a pair of electrons to a silver ion, the Lewis acid:

Nonmetal oxides act as Lewis acids and react with oxide ions, Lewis bases, to form oxyanions:

Access for free at OpenStax 15.2.1 https://chem.libretexts.org/@go/page/38286


Many Lewis acid-base reactions are displacement reactions in which one Lewis base displaces another Lewis base from an acid-
base adduct, or in which one Lewis acid displaces another Lewis acid:

The last displacement reaction shows how the reaction of a Brønsted-Lowry acid with a base fits into the Lewis concept. A
Brønsted-Lowry acid such as HCl is an acid-base adduct according to the Lewis concept, and proton transfer occurs because a
more stable acid-base adduct is formed. Thus, although the definitions of acids and bases in the two theories are quite different, the
theories overlap considerably.
Many slightly soluble ionic solids dissolve when the concentration of the metal ion in solution is decreased through the formation
of complex (polyatomic) ions in a Lewis acid-base reaction. For example, silver chloride dissolves in a solution of ammonia
+ +
because the silver ion reacts with ammonia to form the complex ion Ag(NH ) . The Lewis structure of the Ag(NH ) ion is:
3 2 3 2

The equations for the dissolution of AgCl in a solution of NH3 are:


+ −
AgCl(s) ⟶ Ag (aq) + Cl (aq)

+ +
Ag (aq) + 2 NH (aq) ⟶ Ag (NH ) (aq)
3 3 2

+ −
Net: AgCl(s) + 2 NH (aq) ⟶ Ag (NH ) (aq) + Cl (aq)
3 3 2

Aluminum hydroxide dissolves in a solution of sodium hydroxide or another strong base because of the formation of the complex
− −
ion Al(OH) . The Lewis structure of the Al(OH) ion is:
4 4

Access for free at OpenStax 15.2.2 https://chem.libretexts.org/@go/page/38286


The equations for the dissolution are:
3+ −
Al (OH) (s) ⟶ Al (aq) + 3 OH (aq)
3

3+ − −
Al (aq) + 4 OH (aq) ⟶ Al (OH) 4 (aq)

− −
Net: Al (OH) (s) + OH (aq) ⟶ Al (OH) (aq)
3 4

Mercury(II) sulfide dissolves in a solution of sodium sulfide because HgS reacts with the S2– ion:
2+ 2−
HgS(s) ⟶ Hg (aq) + S (aq)

2+ 2− 2−
Hg (aq) + 2 S (aq) ⟶ HgS2 (aq)

2− 2−
Net: HgS(s) + S (aq) ⟶ HgS (aq)
2

A complex ion consists of a central atom, typically a transition metal cation, surrounded by ions, or molecules called ligands. These
ligands can be neutral molecules like H2O or NH3, or ions such as CN– or OH–. Often, the ligands act as Lewis bases, donating a
pair of electrons to the central atom. The ligands aggregate themselves around the central atom, creating a new ion with a charge
equal to the sum of the charges and, most often, a transitional metal ion. This more complex arrangement is why the resulting ion is
called a complex ion. The complex ion formed in these reactions cannot be predicted; it must be determined experimentally. The
types of bonds formed in complex ions are called coordinate covalent bonds, as electrons from the ligands are being shared with the
central atom. Because of this, complex ions are sometimes referred to as coordination complexes. This will be studied further in
upcoming chapters.
The equilibrium constant for the reaction of the components of a complex ion to form the complex ion in solution is called a
formation constant (Kf) (sometimes called a stability constant). For example, the complex ion Cu(CN) is shown here: −

It forms by the reaction:


+ − −
Cu (aq) + 2 CN (aq) ⇌ Cu(CN) (aq)
2

At equilibrium:

[Cu(CN)2 ]
Kf = Q =
+ − 2
[ Cu ] [ CN ]

The inverse of the formation constant is the dissociation constant (Kd), the equilibrium constant for the decomposition of a complex
ion into its components in solution. We will work with dissociation constants further in the exercises for this section. Table E4 and
Table 15.2.1 are tables of formation constants. In general, the larger the formation constant, the more stable the complex; however,
as in the case of Ksp values, the stoichiometry of the compound must be considered.
Table 15.2.1 : Common Complex Ions by Decreasing Formation Constants
Substance Kf at 25 °C

[Cd(CN) ]
4
2−
3 × 1018

1.7 × 107
+
Ag(NH )
3 2

[AlF ]
6
3−
7 × 1019

As an example of dissolution by complex ion formation, let us consider what happens when we add aqueous ammonia to a mixture
of silver chloride and water. Silver chloride dissolves slightly in water, giving a small concentration of Ag+ ([Ag+] = 1.3 × 10–5 M):
+ −
AgCl(s) ⇌ Ag (aq) + Cl (aq)

+
However, if NH3 is present in the water, the complex ion, Ag(NH 3
)
2
, can form according to the equation:
+ +
Ag (aq) + 2 NH (aq) ⇌ Ag (NH ) (aq)
3 3 2

with

Access for free at OpenStax 15.2.3 https://chem.libretexts.org/@go/page/38286


+
[Ag (NH ) ]
3 2 7
Kf = = 1.7 × 10
+ 2
[ Ag ] [ NH ]
3

The large size of this formation constant indicates that most of the free silver ions produced by the dissolution of AgCl combine
with NH3 to form Ag(NH ) . As a consequence, the concentration of silver ions, [Ag+], is reduced, and the reaction quotient for
3
+

the dissolution of silver chloride, [Ag+][Cl–], falls below the solubility product of AgCl:
+ −
Q = [ Ag ][ Cl ] < Ksp

More silver chloride then dissolves. If the concentration of ammonia is great enough, all of the silver chloride dissolves.

 Example 15.2.1: Dissociation of a Complex Ion

Calculate the concentration of the silver ion in a solution that initially is 0.10 M with respect to Ag(NH 3
)
+

2
.

Solution
We use the familiar path to solve this problem:

+
1. Determine the direction of change. The complex ion Ag(NH 3
)
2
is in equilibrium with its components, as represented by
the equation:
+ +
Ag (aq) + 2 NH (aq) ⇌ Ag (NH ) (aq)
3 3 2

We write the equilibrium as a formation reaction because Table E4 lists formation constants for complex ions. Before equilibrium,
0.10
the reaction quotient is larger than the equilibrium constant (K_f = 1.7 \times 10^7\), and Q = =∞ (it is infinitely large),
0 ×0
so the reaction shifts to the left to reach equilibrium.

This page titled 15.2: Lewis Acids and Bases is shared under a CC BY 4.0 license and was authored, remixed, and/or curated by OpenStax via
source content that was edited to the style and standards of the LibreTexts platform; a detailed edit history is available upon request.

Access for free at OpenStax 15.2.4 https://chem.libretexts.org/@go/page/38286


15.3: Coupled Equilibria
 Learning Objectives
Describe examples of systems involving two (or more) simultaneous chemical equilibria
Calculate reactant and product concentrations for multiple equilibrium systems
Compare dissolution and weak electrolyte formation

There are times when one equilibrium reaction does not adequately describe the system being studied. Sometimes we have more
than one type of equilibrium occurring at once (for example, an acid-base reaction and a precipitation reaction).
The ocean is a unique example of a system with multiple equilibria, or multiple states of solubility equilibria working
simultaneously. Carbon dioxide in the air dissolves in sea water, forming carbonic acid (H2CO3). The carbonic acid then ionizes to
form hydrogen ions and bicarbonate ions (HCO ) , which can further ionize into more hydrogen ions and carbonate ions (CO ):

3
2−

CO (g) ⇌ CO (aq)
2 2

CO (aq) + H O ⇌ H CO (aq)
2 2 2 3

+ −
H CO (aq) ⇌ H (aq) + HCO (aq)
2 3 3

− + 2−
HCO3 (aq) ⇌ H (aq) + CO3 (aq)

The excess H+ ions make seawater more acidic. Increased ocean acidification can then have negative impacts on reef-building
coral, as they cannot absorb the calcium carbonate they need to grow and maintain their skeletons (Figure 15.3.1). This in turn
disrupts the local biosystem that depends upon the health of the reefs for its survival. If enough local reefs are similarly affected,
the disruptions to sea life can be felt globally. The world’s oceans are presently in the midst of a period of intense acidification,
believed to have begun in the mid-nineteenth century, and which is now accelerating at a rate faster than any change to oceanic pH
in the last 20 million years.

Figure 15.3.1 : Healthy coral reefs (a) support a dense and diverse array of sea life across the ocean food chain. But when coral are
unable to adequately build and maintain their calcium carbonite skeletons because of excess ocean acidification, the unhealthy reef
(b) is only capable of hosting a small fraction of the species as before, and the local food chain starts to collapse. (credit a:
modification of work by NOAA Photo Library; credit b: modification of work by “prilfish”/Flickr).

Learn more about ocean acidification and how it affects other marine creatures.
Slightly soluble solids derived from weak acids generally dissolve in strong acids, unless their solubility products are extremely
small. For example, we can dissolve CuCO3, FeS, and Ca3(PO4)2 in HCl because their basic anions react to form weak acids
(H2CO3, H2S, and H PO ). The resulting decrease in the concentration of the anion results in a shift of the equilibrium
2

concentrations to the right in accordance with Le Chatelier’s principle.

Access for free at OpenStax 15.3.1 https://chem.libretexts.org/@go/page/38287


Figure 15.3.2 : Crystal of the mineral hydroxylapatite, Ca5(PO4)3OH, is shown here. Pure apatite is white, but like many other
minerals, this sample is colored because of the presence of impurities.
Of particular relevance to us is the dissolution of hydroxylapatite, Ca5(PO4)3OH, in acid. Apatites are a class of calcium phosphate
minerals (Figure 15.3.2); a biological form of hydroxylapatite is found as the principal mineral in the enamel of our teeth. A
mixture of hydroxylapatite and water (or saliva) contains an equilibrium mixture of solid Ca5(PO4)3OH and dissolved Ca2+, PO , 3−

and OH– ions:


2+ 3− −
Ca (PO ) OH(s) ⟶ 5 Ca (aq) + 3 PO (aq) + OH (aq)
5 4 3 4

When exposed to acid, phosphate ions react with hydronium ions to form hydrogen phosphate ions and ultimately, phosphoric acid:
3− + 2−
PO (aq) + H O ⇌ H PO +H O
4 3 2 4 2

2− + −
PO (aq) + H O ⇌ H PO +H O
4 3 2 4 2

− +
H PO +H O ⇌ H PO +H O
2 4 3 3 4 2

Hydroxide ion reacts to form water:


− +
OH (aq) + H O ⇌ 2H O
3 2

These reactions decrease the phosphate and hydroxide ion concentrations, and additional hydroxylapatite dissolves in an acidic
solution in accord with Le Chatelier’s principle. Our teeth develop cavities when acid waste produced by bacteria growing on them
causes the hydroxylapatite of the enamel to dissolve. Fluoride toothpastes contain sodium fluoride, NaF, or stannous fluoride [more
properly named tin(II) fluoride], SnF2. They function by replacing the OH– ion in hydroxylapatite with F– ion, producing
fluorapatite, Ca5(PO4)3F:
+ −
NaF + Ca (PO ) OH ⇌ Ca (PO ) F + Na + OH
5 4 3 5 4 3

The resulting Ca5(PO4)3F is slightly less soluble than Ca5(PO4)3OH, and F– is a weaker base than OH–. Both of these factors make
the fluorapatite more resistant to attack by acids than hydroxylapatite. See the Chemistry in Everyday Life feature on the role of
fluoride in preventing tooth decay for more information.

 Role of Fluoride in Preventing Tooth Decay

As we saw previously, fluoride ions help protect our teeth by reacting with hydroxylapatite to form fluorapatite, Ca5(PO4)3F.
Since it lacks a hydroxide ion, fluorapatite is more resistant to attacks by acids in our mouths and is thus less soluble,
protecting our teeth. Scientists discovered that naturally fluorinated water could be beneficial to your teeth, and so it became
common practice to add fluoride to drinking water. Toothpastes and mouthwashes also contain amounts of fluoride (Figure
15.3.3).

Figure 15.3.3 : Fluoride, found in many toothpastes, helps prevent tooth decay (credit: Kerry Ceszyk).
Unfortunately, excess fluoride can negate its advantages. Natural sources of drinking water in various parts of the world have
varying concentrations of fluoride, and places where that concentration is high are prone to certain health risks when there is

Access for free at OpenStax 15.3.2 https://chem.libretexts.org/@go/page/38287


no other source of drinking water. The most serious side effect of excess fluoride is the bone disease, skeletal fluorosis. When
excess fluoride is in the body, it can cause the joints to stiffen and the bones to thicken. It can severely impact mobility and can
negatively affect the thyroid gland. Skeletal fluorosis is a condition that over 2.7 million people suffer from across the world.
So while fluoride can protect our teeth from decay, the US Environmental Protection Agency sets a maximum level of 4 ppm
(4 mg/L) of fluoride in drinking water in the US. Fluoride levels in water are not regulated in all countries, so fluorosis is a
problem in areas with high levels of fluoride in the groundwater.

When acid rain attacks limestone or marble, which are calcium carbonates, a reaction occurs that is similar to the acid attack on
hydroxylapatite. The hydronium ion from the acid rain combines with the carbonate ion from calcium carbonates and forms the
hydrogen carbonate ion, a weak acid:
+ 2− −
H O (aq) + CO (aq) ⟶ HCO (aq) + H O(l)
3 3 3 2

Calcium hydrogen carbonate, Ca(HCO3)2, is soluble, so limestone and marble objects slowly dissolve in acid rain.
If calcium carbonate is added to a concentrated acid, hydronium ion reacts with the carbonate ion according to the equation:
+ 2−
2H O (aq) + CO (aq) ⟶ H CO (aq) + 2 H O(l)
3 3 2 3 2

(Acid rain is usually not sufficiently acidic to cause this reaction; however, laboratory acids are.) The solution may become
saturated with the weak electrolyte carbonic acid, which is unstable, and carbon dioxide gas can be evolved:

H CO (aq) ⟶ CO (g) + H O(l)


2 3 2 2

These reactions decrease the carbonate ion concentration, and additional calcium carbonate dissolves. If enough acid is present, the
concentration of carbonate ion is reduced to such a low level that the reaction quotient for the dissolution of calcium carbonate
remains less than the solubility product of calcium carbonate, even after all of the calcium carbonate has dissolved.

 Example 15.3.1: Prevention of Precipitation of Mg(OH)2

Calculate the concentration of ammonium ion that is required to prevent the precipitation of Mg(OH)2 in a solution with
[Mg2+] = 0.10 M and [NH3] = 0.10 M.

Solution
Two equilibria are involved in this system:
Reaction (1): Mg(OH) (s) ⇌ Mg (aq) + 2 OH (aq); K = 8.9 × 10
2
2+ −
sp
−12

Reaction (2): NH (aq) + H O(l) ⇌ NH (aq) + OH (aq) K = 1.8 × 10


3 2
+

4

sp
−5

To prevent the formation of solid Mg(OH)2, we must adjust the concentration of OH– so that the reaction quotient for Equation
(1), Q = [Mg2+][OH–]2, is less than Ksp for Mg(OH)2. (To simplify the calculation, we determine the concentration of OH–
when Q = Ksp.) [OH–] can be reduced by the addition of NH , which shifts Reaction (2) to the left and reduces [OH–].
+
4

1. We determine the [OH–] at which Q = Ksp when [Mg2+] = 0.10 M:


2+ − 2 − 2 −12
Q = [ Mg ] [ OH ] = (0.10)[ OH ] = 8.9 × 10

− −6
[ OH ] = 9.4 × 10 M

Solid Mg(OH)2 will not form in this solution when [OH ] is less than 9.4 × 10–6 M.

We calculate the [NH +


4
] needed to decrease [OH–] to 9.4 × 10–6 M when [NH3] = 0.10.
+ − + −6
[ NH ][ OH ] [ NH ](9.4 × 10 )
4 4 −5
Kb = = = 1.8 × 10
[ NH ] 0.10
3

+
[ NH ] = 0.19 M
4

When [NH ] equals 0.19 M, [OH–] will be 9.4 × 10–6 M. Any [NH
+
4
+
4
] greater than 0.19 M will reduce [OH–] below 9.4 × 10–6
M and prevent the formation of Mg(OH)2.

Access for free at OpenStax 15.3.3 https://chem.libretexts.org/@go/page/38287


 Exercise 15.3.1

Consider the two equilibria:


2+ 2− −27
ZnS(s) ⇌ Zn (aq) + S (aq) Ksp = 1 × 10

+ 2− −26
2 H O(l) + H S(aq) ⇌ 2 H O (aq) + S (aq) K = 1.0 × 10
2 2 3

and calculate the concentration of hydronium ion required to prevent the precipitation of ZnS in a solution that is 0.050 M in
Zn2+ and saturated with H2S (0.10 M H2S).

Answer
+
[H O ] > 0.2 M
3

([S2–] is less than 2 × 10–26 M and precipitation of ZnS does not occur.)
Therefore, precise calculations of the solubility of solids from the solubility product are limited to cases in which the only
significant reaction occurring when the solid dissolves is the formation of its ions.

 Example 15.3.2: Multiple Equilibria

Unexposed silver halides are removed from photographic film when they react with sodium thiosulfate (Na2S2O3, called hypo)
to form the complex ion Ag(S O ) (Kf = 4.7 × 1013). The reaction with silver bromide is:
3−

2 3 2

What mass of Na2S2O3 is required to prepare 1.00 L of a solution that will dissolve 1.00 g of AgBr by the formation of
3−
Ag(S O )
2 3
? 2

Solution
Two equilibria are involved when AgBr dissolves in a solution containing the S 2
O3
2−
ion:
Reaction (1): AgBr(s) ⇌ Ag +
(aq) + Br

(aq) Ksp = 5.0 × 10
−13

3−
Reaction (2): Ag (aq) + S O
+

2
2−
3
(aq) ⇌ Ag (S O )
2 3 2
(aq) Kf = 4.7 × 10
13

In order for 1.00 g of AgBr to dissolve, the [Ag+] in the solution that results must be low enough for Q for Reaction (1) to be
smaller than Ksp for this reaction. We reduce [Ag+] by adding S O and thus cause Reaction (2) to shift to the right. We need
2
2−
3

the following steps to determine what mass of Na2S2O3 is needed to provide the necessary S O . 2
2−

1. We calculate the [Br–] produced by the complete dissolution of 1.00 g of AgBr (5.33 × 10–3 mol AgBr) in 1.00 L of solution:
− −3
[ Br ] = 5.33 × 10 M

We use [Br–] and Ksp to determine the maximum possible concentration of Ag+ that can be present without causing
reprecipitation of AgBr:
+ −11
[ Ag ] = 9.4 × 10 M

We determine the [S O ] required to make [Ag+] = 9.4 × 10–11 M after the remaining Ag+ ion has reacted with S
2
2−

3
2−
2 O3

according to the equation:


+ 2− 3− 13
Ag +2 S O ⇌ Ag (S O ) Kf = 4.7 × 10
2 3 2 3 2

Because 5.33 × 10–3 mol of AgBr dissolves:

Access for free at OpenStax 15.3.4 https://chem.libretexts.org/@go/page/38287


\[(5.33×10^{−3})−(9.4×10^{−11})=5.33×10^{−3}\ce{\:mol\:Ag(S2O3)2^3-}\ \nonumber ]
= 5.33 × 10–3 M, [Ag+] = 9.4× 10–11 M, and Q = Kf = 4.7 × 1013:
3−
Thus, at equilibrium: [Ag(S 2
O )
3 2
]

3−
[Ag (S O ) ]
2 3 2 13
Kf = = 4.7 × 10
+ 2− 2
[ Ag ][S O ]
2 3

2− −3
[S O ] = 1.1 × 10 M
2 3

When [S 2
2−
O
3
] is 1.1 × 10–3 M, [Ag+] is 9.4 × 10–11 M and all AgBr remains dissolved.
We determine the total number of moles of S O that must be added to the solution. This equals the amount that reacts
2
2−

with Ag+ to form Ag(S O ) plus the amount of free S O in solution at equilibrium. To form 5.33 × 10–3 mol of
2 3
3−

2 2
2−

Ag(S O )
2 3 2
requires 2 × (5.33 × 10–3) mol of S O . In addition, 1.1 × 10–3 mol of unreacted S O is present (Step 3).
3−

2
2−

3 2
2−

Thus, the total amount of S O that must be added is:


2
2−

−3 2− −3 2− −2 2−
2 × (5.33 × 10 mol S2 O ) + 1.1 × 10 mol S2 O = 1.18 × 10 mol S2 O
3 3 3

We determine the mass of Na2S2O3 required to give 1.18 × 10–2 mol S 2−


2 O3 using the molar mass of Na2S2O3:
158.1 g Na2 S2 O3
−2 2−
1.18 × 10 mol S2 O × = 1.9 g Na2 S2 O3
3
1 mol Na2 S2 O3

Thus, 1.00 L of a solution prepared from 1.9 g Na2S2O3 dissolves 1.0 g of AgBr.

 Exercise 15.3.2

AgCl(s), silver chloride, is well known to have a very low solubility: Ag(s) ⇌ Ag (aq) + Cl (aq) , Ksp = 1.6 × 10–10. + −

Adding ammonia significantly increases the solubility of AgCl because a complex ion is formed:
+ 7
Ag (aq) + 2 NH (aq) ⇌ Ag (NH ) (aq) , Kf = 1.7 × 10 . What mass of NH3 is required to prepare 1.00 L of solution that
+

3 3 2
+
will dissolve 2.00 g of AgCl by formation of Ag(NH ) ? 3 2

Answer
1.00 L of a solution prepared with 4.81 g NH3 dissolves 2.0 g of AgCl.

15.3.1: Dissolution versus Weak Electrolyte Formation


We can determine how to shift the concentration of ions in the equilibrium between a slightly soluble solid and a solution of its ions
by applying Le Chatelier’s principle. For example, one way to control the concentration of manganese(II) ion, Mn2+, in a solution
is to adjust the pH of the solution and, consequently, to manipulate the equilibrium between the slightly soluble solid manganese(II)
hydroxide, manganese(II) ion, and hydroxide ion:
2+ − 2+ − 2
Mn(OH) (s) ⇌ Mn (aq) + 2 OH (aq) Ksp = [ Mn ] [ OH ]
2

This could be important to a laundry because clothing washed in water that has a manganese concentration exceeding 0.1 mg per
liter may be stained by the manganese. We can reduce the concentration of manganese by increasing the concentration of hydroxide
ion. We could add, for example, a small amount of NaOH or some other base such as the silicates found in many laundry
detergents. As the concentration of OH– ion increases, the equilibrium responds by shifting to the left and reducing the
concentration of Mn2+ ion while increasing the amount of solid Mn(OH)2 in the equilibrium mixture, as predicted by Le Chatelier’s
principle.

 Example 15.3.3: Solubility Equilibrium of a Slightly Soluble Solid

What is the effect on the amount of solid Mg(OH)2 that dissolves and the concentrations of Mg2+ and OH– when each of the
following are added to a mixture of solid Mg(OH)2 in water at equilibrium?
a. MgCl2
b. KOH

Access for free at OpenStax 15.3.5 https://chem.libretexts.org/@go/page/38287


c. an acid
d. NaNO3
e. Mg(OH)2

Solution
The equilibrium among solid Mg(OH)2 and a solution of Mg2+ and OH– is:
2+ −
Mg (OH) (s) ⇌ Mg (aq) + 2 OH (aq)
2

(a) The reaction shifts to the left to relieve the stress produced by the additional Mg2+ ion, in accordance with Le Chatelier’s
principle. In quantitative terms, the added Mg2+ causes the reaction quotient to be larger than the solubility product (Q > Ksp),
and Mg(OH)2 forms until the reaction quotient again equals Ksp. At the new equilibrium, [OH–] is less and [Mg2+] is greater
than in the solution of Mg(OH)2 in pure water. More solid Mg(OH)2 is present.
(b) The reaction shifts to the left to relieve the stress of the additional OH– ion. Mg(OH)2 forms until the reaction quotient
again equals Ksp. At the new equilibrium, [OH–] is greater and [Mg2+] is less than in the solution of Mg(OH)2 in pure water.
More solid Mg(OH)2 is present.
(c) The concentration of OH– is reduced as the OH– reacts with the acid. The reaction shifts to the right to relieve the stress of
less OH– ion. In quantitative terms, the decrease in the OH– concentration causes the reaction quotient to be smaller than the
solubility product (Q < Ksp), and additional Mg(OH)2 dissolves until the reaction quotient again equals Ksp. At the new
equilibrium, [OH–] is less and [Mg2+] is greater than in the solution of Mg(OH)2 in pure water. More Mg(OH)2 is dissolved.
(d) NaNO3 contains none of the species involved in the equilibrium, so we should expect that it has no appreciable effect on
the concentrations of Mg2+ and OH–. (As we have seen previously, dissolved salts change the activities of the ions of an
electrolyte. However, the salt effect is generally small, and we shall neglect the slight errors that may result from it.)
(e) The addition of solid Mg(OH)2 has no effect on the solubility of Mg(OH)2 or on the concentration of Mg2+ and OH–. The
concentration of Mg(OH)2 does not appear in the equation for the reaction quotient:
2+ − 2
Q = [ Mg ] [ OH ]

Thus, changing the amount of solid magnesium hydroxide in the mixture has no effect on the value of Q, and no shift is
required to restore Q to the value of the equilibrium constant.

 Exercise 15.3.3

What is the effect on the amount of solid NiCO3 that dissolves and the concentrations of Ni2+ and CO
2−

3
when each of the
following are added to a mixture of the slightly soluble solid NiCO3 and water at equilibrium?
a. Ni(NO3)2
b. KClO4
c. NiCO3
d. K2CO3
e. HNO3 (reacts with carbonate giving HCO or H2O and CO2)

3

Answer
(a) mass of NiCO3(s) increases, [Ni2+] increases, [CO ] decreases; (b) no appreciable effect; (c) no effect except to increase
2−
3

the amount of solid NiCO3; (d) mass of NiCO3(s) increases, [Ni2+] decreases, [CO ] increases; (e) mass of NiCO3(s)
2−

decreases, [Ni2+] increases, [CO ] decreases


2−

Summary
Several systems we encounter consist of multiple equilibria, systems where two or more equilibria processes are occurring
simultaneously. Some common examples include acid rain, fluoridation, and dissolution of carbon dioxide in sea water. When

Access for free at OpenStax 15.3.6 https://chem.libretexts.org/@go/page/38287


looking at these systems, we need to consider each equilibrium separately and then combine the individual equilibrium constants
into one solubility product or reaction quotient expression using the tools from the first equilibrium chapter. Le Chatelier’s
principle also must be considered, as each reaction in a multiple equilibria system will shift toward reactants or products based on
what is added to the initial reaction and how it affects each subsequent equilibrium reaction.

Glossary
multiple equilibrium
system characterized by more than one state of balance between a slightly soluble ionic solid and an aqueous solution of ions
working simultaneously

This page titled 15.3: Coupled Equilibria is shared under a CC BY 4.0 license and was authored, remixed, and/or curated by OpenStax via source
content that was edited to the style and standards of the LibreTexts platform; a detailed edit history is available upon request.

Access for free at OpenStax 15.3.7 https://chem.libretexts.org/@go/page/38287


15.E: Equilibria of Other Reaction Classes (Exercises)
15.E.1: 15.1: Precipitation and Dissolution
15.E.1.1: Q15.1.1
Complete the changes in concentrations for each of the following reactions:
+ −
AgI(s) ⟶ Ag (aq) + I (aq) (15.E.1)

x (15.E.2)
––––––
2+ 2−
CaCO (s) ⟶ Ca (aq) + CO3 (aq) (15.E.3)
3

x (15.E.4)
––––––
2+ −
Mg (OH) (s) ⟶ Mg (aq) + 2 OH (aq) (15.E.5)
2

x (15.E.6)
––––––
2+ 3−
Mg (PO ) (s) ⟶ 3 Mg (aq) + 2 PO (aq) (15.E.7)
3 4 2 4

x (15.E.8)
––––––
2+ 3− −
Ca (PO ) OH(s) ⟶ 5 Ca (aq) + 3 PO4 (aq) + OH (aq) (15.E.9)
5 4 3

x (15.E.10)
–––––– ––––––

15.E.1.2: S15.1.1
+ −
AgI(s) ⟶ Ag (aq) + I (aq) (15.E.11)

x x (15.E.12)


2+ 2−
CaCO (s) ⟶ Ca (aq) + CO (aq) (15.E.13)
3 3

x x (15.E.14)


2+ −
Mg (OH) (s) ⟶ Mg (aq) + 2 OH (aq) (15.E.15)
2

x 2x (15.E.16)
–––
2+ 3−
Mg (PO ) (s) ⟶ 3 Mg (aq) + 2 PO4 (aq) (15.E.17)
3 4 2

3x 2x (15.E.18)
–––
2+ 3− −
Ca (PO ) OH(s) ⟶ 5 Ca (aq) + 3 PO (aq) + OH (aq) (15.E.19)
5 4 3 4

5x 3x x (15.E.20)
––– –––

15.E.1.3: Q15.1.2
Complete the changes in concentrations for each of the following reactions:
2+ 2−
BaSO (s) ⟶ Ba (aq) + SO (aq) (15.E.21)
4 4

x (15.E.22)
––––––
+ 2−
Ag SO (s) ⟶ 2 Ag (aq) + SO 4 (aq) (15.E.23)
2 4

x (15.E.24)
––––––
3+ −
Al (OH) (s) ⟶ Al (aq) + 3 OH (aq) (15.E.25)
3

x (15.E.26)
––––––
2+ − −
Pb(OH)Cl(s) ⟶ Pb (aq) + OH (aq) + Cl (aq) (15.E.27)

x (15.E.28)
–––––– ––––––
2+ 3−
Ca (AsO ) (s) ⟶ 3 Ca (aq) + 2 AsO (aq) (15.E.29)
3 4 2 4

3x (15.E.30)
––––––

15.E.1.4: Q15.1.3
How do the concentrations of Ag+ and CrO 2−
4
in a saturated solution above 1.0 g of solid Ag2CrO4 change when 100 g of solid
Ag2CrO4 is added to the system? Explain.

Access for free at OpenStax 15.E.1 https://chem.libretexts.org/@go/page/44123


15.E.1.5: S15.1.3
There is no change. A solid has an activity of 1 whether there is a little or a lot.

15.E.1.6: Q15.1.4
How do the concentrations of Pb2+ and S2– change when K2S is added to a saturated solution of PbS?

15.E.1.7: Q15.1.5
What additional information do we need to answer the following question: How is the equilibrium of solid silver bromide with a
saturated solution of its ions affected when the temperature is raised?

15.E.1.8: S15.1.5
The solubility of silver bromide at the new temperature must be known. Normally the solubility increases and some of the solid
silver bromide will dissolve.

15.E.1.9: Q15.1.6
Which of the following slightly soluble compounds has a solubility greater than that calculated from its solubility product because
of hydrolysis of the anion present: CoSO3, CuI, PbCO3, PbCl2, Tl2S, KClO4?

15.E.1.10: Q15.1.7
Which of the following slightly soluble compounds has a solubility greater than that calculated from its solubility product because
of hydrolysis of the anion present: AgCl, BaSO4, CaF2, Hg2I2, MnCO3, ZnS, PbS?

15.E.1.11: S15.1.7
CaF2, MnCO3, and ZnS

15.E.1.12: Q15.1.8
Write the ionic equation for dissolution and the solubility product (Ksp) expression for each of the following slightly soluble ionic
compounds:
a. PbCl2
b. Ag2S
c. Sr3(PO4)2
d. SrSO4

15.E.1.13: Q15.1.9
Write the ionic equation for the dissolution and the Ksp expression for each of the following slightly soluble ionic compounds:
a. LaF3
b. CaCO3
c. Ag2SO4
d. Pb(OH)2

15.E.1.14: Q15.1.10
− 3
a. LaF (s) ⇌ La (aq) + 3 F (aq) K
3
3+ −
sp = [ La
3+
][F ] ;

b. CaCO (s) ⇌ Ca (aq) + CO (aq)


3
2+ 2−
3
Ksp = [ Ca
2+
][ CO3
2−
];
+ 2
c. Ag SO (s) ⇌ 2 Ag (aq) + SO (aq)
2 4
+ 2−
4
Ksp = [ Ag ] [ SO
2−
4
];

− 2
d. Pb(OH) 2
(s) ⇌ Pb
2+
(aq) + 2 OH

(aq) Ksp = [ Pb
2+
] [ OH ]

15.E.1.15: Q15.1.11
The Handbook of Chemistry and Physics gives solubilities of the following compounds in grams per 100 mL of water. Because
these compounds are only slightly soluble, assume that the volume does not change on dissolution and calculate the solubility
product for each.
a. BaSiF6, 0.026 g/100 mL (contains SiF 2−

6
ions)
b. Ce(IO3)4, 1.5 × 10–2 g/100 mL

Access for free at OpenStax 15.E.2 https://chem.libretexts.org/@go/page/44123


c. Gd2(SO4)3, 3.98 g/100 mL
d. (NH4)2PtBr6, 0.59 g/100 mL (contains PtBr 2−
6
ions)

15.E.1.16: Q15.1.12
The Handbook of Chemistry and Physics gives solubilities of the following compounds in grams per 100 mL of water. Because
these compounds are only slightly soluble, assume that the volume does not change on dissolution and calculate the solubility
product for each.
a. BaSeO4, 0.0118 g/100 mL
b. Ba(BrO3)2•H2O, 0.30 g/100 mL
c. NH4MgAsO4•6H2O, 0.038 g/100 mL
d. La2(MoO4)3, 0.00179 g/100 mL

15.E.1.17: S15.1.12
(a)1.77 × 10–7; 1.6 × 10–6; 2.2 × 10–9; 7.91 × 10–22

15.E.1.18: Q15.1.13
Use solubility products and predict which of the following salts is the most soluble, in terms of moles per liter, in pure water: CaF2,
Hg2Cl2, PbI2, or Sn(OH)2.

15.E.1.19: Q15.1.14
Assuming that no equilibria other than dissolution are involved, calculate the molar solubility of each of the following from its
solubility product:
a. KHC4H4O6
b. PbI2
c. Ag4[Fe(CN)6], a salt containing the Fe(CN) ion−

d. Hg2I2

15.E.1.20: S15.1.15
2 × 10–2 M; 1.3 × 10–3 M; 2.27 × 10–9 M; 2.2 × 10–10 M

15.E.1.21: Q15.1.16
Assuming that no equilibria other than dissolution are involved, calculate the molar solubility of each of the following from its
solubility product:
a. Ag2SO4
b. PbBr2
c. AgI
d. CaC2O4•H2O

15.E.1.22: Q15.1.X
Assuming that no equilibria other than dissolution are involved, calculate the concentration of all solute species in each of the
following solutions of salts in contact with a solution containing a common ion. Show that changes in the initial concentrations of
the common ions can be neglected.
a. AgCl(s) in 0.025 M NaCl
b. CaF2(s) in 0.00133 M KF
c. Ag2SO4(s) in 0.500 L of a solution containing 19.50 g of K2SO4
d. Zn(OH)2(s) in a solution buffered at a pH of 11.45

15.E.1.23: S15.1.X
7.2 × 10−9 M = [Ag+], [Cl−] = 0.025 M
−9
7.2 × 10 M
Check: × 100% = 2.9 × 10
−5
% , an insignificant change;
0.025 M

Access for free at OpenStax 15.E.3 https://chem.libretexts.org/@go/page/44123


2.2 × 10−5 M = [Ca2+], [F−] = 0.0013 M
−5
2.25 × 10 M
Check: × 100% = 1.69% . This value is less than 5% and can be ignored.
0.00133 M

0.2238 M = [SO 2−

4
; [Ag+] = 2.30 × 10–9 M
]

−9
1.15 × 10
Check: × 100% = 5.14 × 10
−7
; the condition is satisfied.
0.2238

[OH–] = 2.8 × 10–3 M; 5.7 × 10−12 M = [Zn2+]


−12
5.7 × 10
Check: −3
× 100% = 2.0 × 10
−7
% ; x is less than 5% of [OH–] and is, therefore, negligible.
2.8 × 10

15.E.1.24: Q15.1.X
Assuming that no equilibria other than dissolution are involved, calculate the concentration of all solute species in each of the
following solutions of salts in contact with a solution containing a common ion. Show that changes in the initial concentrations of
the common ions can be neglected.
a. TlCl(s) in 1.250 M HCl
b. PbI2(s) in 0.0355 M CaI2
c. Ag2CrO4(s) in 0.225 L of a solution containing 0.856 g of K2CrO4
d. Cd(OH)2(s) in a solution buffered at a pH of 10.995
Assuming that no equilibria other than dissolution are involved, calculate the concentration of all solute species in each of the
following solutions of salts in contact with a solution containing a common ion. Show that it is not appropriate to neglect the
changes in the initial concentrations of the common ions.
a. TlCl(s) in 0.025 M TlNO3
b. BaF2(s) in 0.0313 M KF
c. MgC2O4 in 2.250 L of a solution containing 8.156 g of Mg(NO3)2
d. Ca(OH)2(s) in an unbuffered solution initially with a pH of 12.700

15.E.1.25: S15.1.X
[Cl–] = 7.6 × 10−3 M
−3
7.6 × 10
Check: × 100% = 30%
0.025

This value is too large to drop x. Therefore solve by using the quadratic equation:
[Ti+] = 3.1 × 10–2 M
[Cl–] = 6.1 × 10–3
[Ba2+] = 1.7 × 10–3 M
−3
1.7 × 10
Check: × 100% = 5.5%
0.0313

This value is too large to drop x, and the entire equation must be solved.
[Ba2+] = 1.6 × 10–3 M
[F–] = 0.0329 M;
Mg(NO3)2 = 0.02444 M
2− −3
[C O ] = 3.5 × 10
2 4

−3
3.5 × 10
Check: × 100% = 14%
0.02444

This value is greater than 5%, so the quadratic equation must be used:
2− −3
[ C O4 ] = 3.5 × 10 M
2
2+
[Mg ] = 0.0275 M
[OH–] = 0.0501 M

Access for free at OpenStax 15.E.4 https://chem.libretexts.org/@go/page/44123


[Ca2+] = 3.15 × 10–3
−3
3.15 × 10
Check: × 100% = 6.28%
0.050

This value is greater than 5%, so a more exact method, such as successive approximations, must be used.
[Ca2+] = 2.8 × 10–3 M
[OH–] = 0.053 × 10–2 M

15.E.1.26: Q15.1.X
Explain why the changes in concentrations of the common ions in Exercise can be neglected.

15.E.1.27: Q15.1.X
Explain why the changes in concentrations of the common ions in Exercise cannot be neglected.

15.E.1.28: S15.1.X
The changes in concentration are greater than 5% and thus exceed the maximum value for disregarding the change.

15.E.1.29: Q15.1.X
Calculate the solubility of aluminum hydroxide, Al(OH)3, in a solution buffered at pH 11.00.

15.E.1.30: Q15.1.X
Refer to Appendix J for solubility products for calcium salts. Determine which of the calcium salts listed is most soluble in moles
per liter and which is most soluble in grams per liter.

15.E.1.31: S15.1.X
CaSO4·2H2O is the most soluble Ca salt in mol/L, and it is also the most soluble Ca salt in g/L.

15.E.1.32: Q15.1.X
Most barium compounds are very poisonous; however, barium sulfate is often administered internally as an aid in the X-ray
examination of the lower intestinal tract. This use of BaSO4 is possible because of its low solubility. Calculate the molar solubility
of BaSO4 and the mass of barium present in 1.00 L of water saturated with BaSO4.

15.E.1.33: Q15.1.X
Public Health Service standards for drinking water set a maximum of 250 mg/L (2.60 × 10–3 M) of SO because of its cathartic
2−
4

action (it is a laxative). Does natural water that is saturated with CaSO4 (“gyp” water) as a result or passing through soil containing
gypsum, CaSO4•2H2O, meet these standards? What is SO in such water?
2−
4

15.E.1.34: S15.1.X
4.9 × 10–3 M = [SO 2−

4
] = [Ca2+]; Since this concentration is higher than 2.60 × 10–3 M, “gyp” water does not meet the standards.

15.E.1.35: Q15.1.X
Perform the following calculations:
a. Calculate [Ag+] in a saturated aqueous solution of AgBr.
b. What will [Ag+] be when enough KBr has been added to make [Br–] = 0.050 M?
c. What will [Br–] be when enough AgNO3 has been added to make [Ag+] = 0.020 M?
The solubility product of CaSO4•2H2O is 2.4 × 10–5. What mass of this salt will dissolve in 1.0 L of 0.010 M SO 2−

4
?

15.E.1.36: S15.1.X
Mass (CaSO4•2H2O) = 0.34 g/L

15.E.1.37: Q15.1.X
Assuming that no equilibria other than dissolution are involved, calculate the concentrations of ions in a saturated solution of each
of the following (see Table E3 for solubility products).

Access for free at OpenStax 15.E.5 https://chem.libretexts.org/@go/page/44123


a. TlCl
b. BaF2
c. Ag2CrO4
d. CaC2O4•H2O
e. the mineral anglesite, PbSO4

15.E.1.38: Q15.1.X
Assuming that no equilibria other than dissolution are involved, calculate the concentrations of ions in a saturated solution of each
of the following (see Table E3 for solubility products).
a. AgI
b. Ag2SO4
c. Mn(OH)2
d. Sr(OH)2•8H2O
e. the mineral brucite, Mg(OH)2

15.E.1.39: S15.1.X
[Ag+] = [I–] = 1.2 × 10–8 M; [Ag+] = 2.86 × 10–2 M, [SO ] = 1.43 × 10–2 M; [Mn2+] = 2.2 × 10–5 M, [OH–] = 4.5 × 10–5 M; [Sr2+]
2−

= 4.3 × 10–2 M, [OH–] = 8.6 × 10–2 M; [Mg2+] = 1.6 × 10–4 M, [OH–] = 3.1 × 10–4 M.

15.E.1.40: Q15.1.X
The following concentrations are found in mixtures of ions in equilibrium with slightly soluble solids. From the concentrations
given, calculate Ksp for each of the slightly soluble solids indicated:
a. AgBr: [Ag+] = 5.7 × 10–7 M, [Br–] = 5.7 × 10–7 M
b. CaCO3: [Ca2+] = 5.3 × 10–3 M, [CO ] = 9.0 × 10–7 M
2−
3

c. PbF2: [Pb2+] = 2.1 × 10–3 M, [F–] = 4.2 × 10–3 M


d. Ag2CrO4: [Ag+] = 5.3 × 10–5 M, 3.2 × 10–3 M
e. InF3: [In3+] = 2.3 × 10–3 M, [F–] = 7.0 × 10–3 M

15.E.1.41: Q15.1.X
The following concentrations are found in mixtures of ions in equilibrium with slightly soluble solids. From the concentrations
given, calculate Ksp for each of the slightly soluble solids indicated:
a. TlCl: [Tl+] = 1.21 × 10–2 M, [Cl–] = 1.2 × 10–2 M
b. Ce(IO3)4: [Ce4+] = 1.8 × 10–4 M, [IO ] = 2.6 × 10–13 M

3

c. Gd2(SO4)3: [Gd3+] = 0.132 M, [SO ] = 0.198 M


2−
4

d. Ag2SO4: [Ag+] = 2.40 × 10–2 M, [SO ] = 2.05 × 10–2 M


2−
4

e. BaSO4: [Ba2+] = 0.500 M, [SO ] = 2.16 × 10–10 M


2−

15.E.1.42: S15.1.X
2.0 × 10–4; 5.1 × 10–17; 1.35 × 10–4; 1.18 × 10–5; 1.08 × 10–10

15.E.1.43: Q15.1.X
Which of the following compounds precipitates from a solution that has the concentrations indicated? (See Table E3 for Ksp
values.)
a. KClO4: [K+] = 0.01 M, [ClO ] = 0.01 M

b. K2PtCl6: [K+] = 0.01 M, [PtCl ] = 0.01 M


2−
6

c. PbI2: [Pb2+] = 0.003 M, [I–] = 1.3 × 10–3 M


d. Ag2S: [Ag+] = 1 × 10–10 M, [S2–] = 1 × 10–13 M

15.E.1.44: Q15.1.X
Which of the following compounds precipitates from a solution that has the concentrations indicated? (See Table E3 for Ksp
values.)

Access for free at OpenStax 15.E.6 https://chem.libretexts.org/@go/page/44123


a. CaCO3: [Ca2+] = 0.003 M, [CO ] = 0.003 M
2−
3

b. Co(OH)2: [Co2+] = 0.01 M, [OH–] = 1 × 10–7 M


c. CaHPO4: [Ca2+] = 0.01 M, [HPO ] = 2 × 10–6 M
2−
4

d. Pb3(PO4)2: [Pb2+] = 0.01 M, [PO ] = 1 × 10–13 M


3−

15.E.1.45: S15.1.X
a. CaCO3 does precipitate.
b. The compound does not precipitate.
c. The compound does not precipitate.
d. The compound precipitates.

15.E.1.46: Q15.1.X
Calculate the concentration of Tl+ when TlCl just begins to precipitate from a solution that is 0.0250 M in Cl–.

15.E.1.47: Q15.1.X
Calculate the concentration of sulfate ion when BaSO4 just begins to precipitate from a solution that is 0.0758 M in Ba2+.

15.E.1.48: S15.1.X
1.42 × 10−9 M

15.E.1.49: Q15.1.X
Calculate the concentration of Sr2+ when SrF2 starts to precipitate from a solution that is 0.0025 M in F–.

15.E.1.50: Q15.1.X
Calculate the concentration of PO 3−

4
when Ag3PO4 starts to precipitate from a solution that is 0.0125 M in Ag+.

15.E.1.51: S15.1.X
9.2 × 10−13 M

15.E.1.52: Q15.1.X
Calculate the concentration of F– required to begin precipitation of CaF2 in a solution that is 0.010 M in Ca2+.

15.E.1.53: Q15.1.X
Calculate the concentration of Ag+ required to begin precipitation of Ag2CO3 in a solution that is 2.50 × 10–6 M in CO . 2−

15.E.1.54: S15.1.X
[Ag+] = 1.8 × 10–3 M

15.E.1.55: Q15.1.X
What [Ag+] is required to reduce [CO 2−
3
] to 8.2 × 10–4 M by precipitation of Ag2CO3?

15.E.1.56: Q15.1.X
What [F–] is required to reduce [Ca2+] to 1.0 × 10–4 M by precipitation of CaF2?

15.E.1.57: S15.1.X
6.2 × 10–4

15.E.1.58: Q15.1.X
A volume of 0.800 L of a 2 × 10–4-M Ba(NO3)2 solution is added to 0.200 L of 5 × 10–4 M Li2SO4. Does BaSO4 precipitate?
Explain your answer.

15.E.1.59: Q15.1.X
Perform these calculations for nickel(II) carbonate.

Access for free at OpenStax 15.E.7 https://chem.libretexts.org/@go/page/44123


a. With what volume of water must a precipitate containing NiCO3 be washed to dissolve 0.100 g of this compound? Assume that
the wash water becomes saturated with NiCO3 (Ksp = 1.36 × 10–7).
b. If the NiCO3 were a contaminant in a sample of CoCO3 (Ksp = 1.0 × 10–12), what mass of CoCO3 would have been lost? Keep
in mind that both NiCO3 and CoCO3 dissolve in the same solution.

15.E.1.60: S15.1.X
2.28 L; 7.3 × 10–7 g

15.E.1.61: Q15.1.X
Iron concentrations greater than 5.4 × 10–6 M in water used for laundry purposes can cause staining. What [OH–] is required to
reduce [Fe2+] to this level by precipitation of Fe(OH)2?

15.E.1.62: Q15.1.X
A solution is 0.010 M in both Cu2+ and Cd2+. What percentage of Cd2+ remains in the solution when 99.9% of the Cu2+ has been
precipitated as CuS by adding sulfide?

15.E.1.63: S15.1.X
100% of it is dissolved

15.E.1.64: Q15.1.X
A solution is 0.15 M in both Pb2+ and Ag+. If Cl– is added to this solution, what is [Ag+] when PbCl2 begins to precipitate?

15.E.1.65: Q15.1.X
What reagent might be used to separate the ions in each of the following mixtures, which are 0.1 M with respect to each ion? In
some cases it may be necessary to control the pH. (Hint: Consider the Ksp values given in Appendix J.)
a. Hg and Cu2+
2+

b. SO and Cl–
2−
4

c. Hg2+ and Co2+


d. Zn2+ and Sr2+
e. Ba2+ and Mg2+
f. CO and OH–
2−

15.E.1.66: S15.1.X
a. Hg and Cu2+: Add SO .
2+
2
2−

b. SO and Cl–: Add Ba2+.


2−

c. Hg2+ and Co2+: Add S2–.


d. Zn2+ an Sr2+: Add OH– until [OH–] = 0.050 M.
e. Ba2+ and Mg2+: Add SO . 2−
4

f. CO and OH–: Add Ba2+.


2−
3

15.E.1.67: Q15.1.X
A solution contains 1.0 × 10–5 mol of KBr and 0.10 mol of KCl per liter. AgNO3 is gradually added to this solution. Which forms
first, solid AgBr or solid AgCl?

15.E.1.68: Q15.1.X
A solution contains 1.0 × 10–2 mol of KI and 0.10 mol of KCl per liter. AgNO3 is gradually added to this solution. Which forms
first, solid AgI or solid AgCl?

15.E.1.69: S15.1.X
AgI will precipitate first.

Access for free at OpenStax 15.E.8 https://chem.libretexts.org/@go/page/44123


15.E.1.70: Q15.1.X
The calcium ions in human blood serum are necessary for coagulation. Potassium oxalate, K2C2O4, is used as an anticoagulant
when a blood sample is drawn for laboratory tests because it removes the calcium as a precipitate of CaC2O4•H2O. It is necessary
to remove all but 1.0% of the Ca2+ in serum in order to prevent coagulation. If normal blood serum with a buffered pH of 7.40
contains 9.5 mg of Ca2+ per 100 mL of serum, what mass of K2C2O4 is required to prevent the coagulation of a 10 mL blood
sample that is 55% serum by volume? (All volumes are accurate to two significant figures. Note that the volume of serum in a 10-
mL blood sample is 5.5 mL. Assume that the Ksp value for CaC2O4 in serum is the same as in water.)

15.E.1.71: Q15.1.X
About 50% of urinary calculi (kidney stones) consist of calcium phosphate, Ca3(PO4)2. The normal mid range calcium content
excreted in the urine is 0.10 g of Ca2+ per day. The normal mid range amount of urine passed may be taken as 1.4 L per day. What
is the maximum concentration of phosphate ion that urine can contain before a calculus begins to form?

15.E.1.72: S15.1.X
4 × 10−9 M

15.E.1.73: Q15.1.X
The pH of normal urine is 6.30, and the total phosphate concentration ([PO ] + [HPO ] + [H PO ] + [H3PO4]) is 0.020 M.
3−
4
2−
4 2

4

What is the minimum concentration of Ca2+ necessary to induce kidney stone formation? (See Exercise for additional information.)

15.E.1.74: Q15.1.X
Magnesium metal (a component of alloys used in aircraft and a reducing agent used in the production of uranium, titanium, and
other active metals) is isolated from sea water by the following sequence of reactions:
2+ 2+
Mg (aq) + Ca (OH) (aq) ⟶ Mg (OH) (s) + Ca (aq)
2 2

Mg (OH) (s) + 2 HCl(aq) ⟶ MgCl (s) + 2 H O(l)


2 2 2

electrolysis

MgCl (l) −−−−−−→ Mg(s) + Cl (g)


2 2

Sea water has a density of 1.026 g/cm3


and contains 1272 parts per million of magnesium as Mg2+(aq) by mass. What mass, in
kilograms, of Ca(OH)2 is required to precipitate 99.9% of the magnesium in 1.00 × 103 L of sea water?

15.E.1.75: S15.1.X
3.99 kg

15.E.1.76: Q15.1.X
Hydrogen sulfide is bubbled into a solution that is 0.10 M in both Pb2+ and Fe2+ and 0.30 M in HCl. After the solution has come to
equilibrium it is saturated with H2S ([H2S] = 0.10 M). What concentrations of Pb2+ and Fe2+ remain in the solution? For a saturated
solution of H2S we can use the equilibrium:
+ 2− −26
H S(aq) + 2 H O(l) ⇌ 2 H O (aq) + S (aq) K = 1.0 × 10
2 2 3

(Hint: The [H3


O
+
] changes as metal sulfides precipitate.)

15.E.1.77: Q15.1.X
Perform the following calculations involving concentrations of iodate ions:
a. The iodate ion concentration of a saturated solution of La(IO3)3 was found to be 3.1 × 10–3 mol/L. Find the Ksp.
b. Find the concentration of iodate ions in a saturated solution of Cu(IO3)2 (Ksp = 7.4 × 10–8).

15.E.1.78: S15.1.X
3.1 × 10–11; [Cu2+] = 2.6 × 10–3; [IO −
3
] = 5.3 × 10–3

15.E.1.79: Q15.1.X
Calculate the molar solubility of AgBr in 0.035 M NaBr (Ksp = 5 × 10–13).

Access for free at OpenStax 15.E.9 https://chem.libretexts.org/@go/page/44123


15.E.1.80: Q15.1.X
How many grams of Pb(OH)2 will dissolve in 500 mL of a 0.050-M PbCl2 solution (Ksp = 1.2 × 10–15)?

15.E.1.81: S15.1.X
1.8 × 10–5 g Pb(OH)2

15.E.1.82: Q15.1.X
Use the simulation from the earlier Link to Learning to complete the following exercise:. Using 0.01 g CaF2, give the Ksp values
found in a 0.2-M solution of each of the salts. Discuss why the values change as you change soluble salts.

15.E.1.83: Q15.1.X
How many grams of Milk of Magnesia, Mg(OH)2 (s) (58.3 g/mol), would be soluble in 200 mL of water. Ksp = 7.1 × 10–12. Include
the ionic reaction and the expression for Ksp in your answer. (Kw = 1 × 10–14 = [H3O+][OH–])

15.E.1.84: S15.1.X
2+ −
Mg (OH) (s) ⇌ Mg + 2 OH (15.E.31)
2

2+ − 2
Ksp = [ Mg ] [ OH ] (15.E.32)

\[1.14 × 10−3 g Mg(OH)2\]

15.E.1.85: Q15.1.X
Two hypothetical salts, LM2 and LQ, have the same molar solubility in H2O. If Ksp for LM2 is 3.20 × 10–5, what is the Ksp value
for LQ?

15.E.1.86: Q15.1.X
Which of the following carbonates will form first? Which of the following will form last? Explain.
a. MgCO 3
Ksp = 3.5 × 10
−8

b. CaCO 3
Ksp = 4.2 × 10
−7

c. SrCO 3
−9
Ksp = 3.9 × 10

d. BaCO 3
Ksp = 4.4 × 10
−5

e. MnCO 3
Ksp = 5.1 × 10
−9

15.E.1.87: S15.1.X
SrCO3 will form first, since it has the smallest Ksp value it is the least soluble. BaCO3 will be the last to precipitate, it has the
largest Ksp value.

15.E.1.88: Q15.1.X
How many grams of Zn(CN)2(s) (117.44 g/mol) would be soluble in 100 mL of H2O? Include the balanced reaction and the
expression for Ksp in your answer. The Ksp value for Zn(CN)2(s) is 3.0 × 10–16.

15.E.2: 15.2: Lewis Acids and Bases


15.E.2.1: Q15.2.X
Under what circumstances, if any, does a sample of solid AgCl completely dissolve in pure water?

15.E.2.2: S15.2.X
when the amount of solid is so small that a saturated solution is not produced

15.E.2.3: Q15.2.X
Explain why the addition of NH3 or HNO3 to a saturated solution of Ag2CO3 in contact with solid Ag2CO3 increases the solubility
of the solid.

Access for free at OpenStax 15.E.10 https://chem.libretexts.org/@go/page/44123


15.E.2.4: Q15.2.X
Calculate the cadmium ion concentration, [Cd2+], in a solution prepared by mixing 0.100 L of 0.0100 M Cd(NO3)2 with 1.150 L of
0.100 NH3(aq).

15.E.2.5: S15.2.X
2.35 × 10–4 M

15.E.2.6: Q15.2.X
Explain why addition of NH3 or HNO3 to a saturated solution of Cu(OH)2 in contact with solid Cu(OH)2 increases the solubility of
the solid.

15.E.2.7: S15.2.X
Sometimes equilibria for complex ions are described in terms of dissociation constants, Kd. For the complex ion AlF6
3−
the
dissociation reaction is:
3+ − 6
[ Al ][F ]
3− 3+ − −24
AlF6 ⇌ Al +6 F \)and\(Kd = = 2 × 10 (15.E.33)
3−
[ AlF ]
6

15.E.2.8: Q15.2.X
Calculate the value of the formation constant, Kf, for AlF 3−

6
.

15.E.2.9: S15.2.X
5 × 1023

15.E.2.10: Q15.2.X
Using the value of the formation constant for the complex ion Co(NH 3
)
2+

6
, calculate the dissociation constant.

15.E.2.11: Q15.2.X
Using the dissociation constant, Kd = 7.8 × 10–18, calculate the equilibrium concentrations of Cd2+ and CN– in a 0.250-M solution
of Cd(CN) . 2−

15.E.2.12: S15.2.X

[Cd2+] = 9.5 × 10–5 M; [CN–] = 3.8 × 10–4 M

15.E.2.13: Q15.2.X
Using the dissociation constant, Kd = 3.4 × 10–15, calculate the equilibrium concentrations of Zn2+ and OH– in a 0.0465-M solution
2−
of Zn(OH) .4

15.E.2.14: Q15.2.X
Using the dissociation constant, Kd = 2.2 × 10–34, calculate the equilibrium concentrations of Co3+ and NH3 in a 0.500-M solution
of Co(NH ) .
3
3+

15.E.2.15: S15.2.X
[Co3+] = 3.0 × 10–6 M; [NH3] = 1.8 × 10–5 M

Access for free at OpenStax 15.E.11 https://chem.libretexts.org/@go/page/44123


15.E.2.16: Q15.2.X
Using the dissociation constant, Kd = 1 × 10–44, calculate the equilibrium concentrations of Fe3+ and CN– in a 0.333 M solution of
3−
Fe(CN) . 6

15.E.2.17: Q15.2.X
Calculate the mass of potassium cyanide ion that must be added to 100 mL of solution to dissolve 2.0 × 10–2 mol of silver cyanide,
AgCN.

15.E.2.18: S15.2.X
1.3 g

15.E.2.19: Q15.2.X
Calculate the minimum concentration of ammonia needed in 1.0 L of solution to dissolve 3.0 × 10–3 mol of silver bromide.

15.E.2.20: Q15.2.X
A roll of 35-mm black and white photographic film contains about 0.27 g of unexposed AgBr before developing. What mass of
Na2S2O3•5H2O (sodium thiosulfate pentahydrate or hypo) in 1.0 L of developer is required to dissolve the AgBr as Ag(S O ) 2 3
3−

(Kf = 4.7 × 1013)?

15.E.2.21: S15.2.X
0.80 g

15.E.2.22: Q15.2.X
We have seen an introductory definition of an acid: An acid is a compound that reacts with water and increases the amount of
hydronium ion present. In the chapter on acids and bases, we saw two more definitions of acids: a compound that donates a proton
(a hydrogen ion, H+) to another compound is called a Brønsted-Lowry acid, and a Lewis acid is any species that can accept a pair
of electrons. Explain why the introductory definition is a macroscopic definition, while the Brønsted-Lowry definition and the
Lewis definition are microscopic definitions.

15.E.2.23: Q15.2.X
Write the Lewis structures of the reactants and product of each of the following equations, and identify the Lewis acid and the
Lewis base in each:
a. CO + OH ⟶ HCO
2
− −
3

b. B(OH) + OH ⟶ B(OH)3
− −

c. I + I ⟶ I

2

3

d. AlCl + Cl ⟶ AlCl (use Al-Cl single bonds)


3
− −

e. O + SO ⟶ SO
2−

3
2−

15.E.2.24: S15.2.X
(a)

;
(b)

Access for free at OpenStax 15.E.12 https://chem.libretexts.org/@go/page/44123


;
(c)

;
(d)

;
(e)

15.E.2.25: Q15.2.X
Write the Lewis structures of the reactants and product of each of the following equations, and identify the Lewis acid and the
Lewis base in each:
a. CS + SH ⟶ HCS
2
− −

b. BF + F ⟶ BF
3
− −
4

c. I + SnI ⟶ SnI

2

3

d. Al(OH) + OH ⟶ Al(OH)
3

4

e. F + SO ⟶ SFO

3

3

15.E.2.26: Q15.2.X
Using Lewis structures, write balanced equations for the following reactions:
a. HCl(g) + PH (g) ⟶ 3

b. H O + CH ⟶
3
+ −
3

c. CaO + SO ⟶ 3

d. NH + C H O ⟶
+

4 2 5

15.E.2.27: S15.2.X
(a)

Access for free at OpenStax 15.E.13 https://chem.libretexts.org/@go/page/44123


;
+ −
H O + CH3 ⟶ CH +H O
3 4 2

;
CaO + SO ⟶ CaSO
3 4

;
+ −
NH +C H O ⟶ C H OH + NH
4 2 5 2 5 3

15.E.2.28: Q15.2.X
Calculate [HgCl 2−

4
] in a solution prepared by adding 0.0200 mol of NaCl to 0.250 L of a 0.100-M HgCl2 solution.

15.E.2.29: Q15.2.X
In a titration of cyanide ion, 28.72 mL of 0.0100 M AgNO3 is added before precipitation begins. [The reaction of Ag+ with CN–
goes to completion, producing the Ag(CN) complex.] Precipitation of solid AgCN takes place when excess Ag+ is added to the

2

solution, above the amount needed to complete the formation of Ag(CN) . How many grams of NaCN were in the original
2

sample?

15.E.2.30: S15.2.X
0.0281 g

15.E.2.31: Q15.2.X
What are the concentrations of Ag+, CN–, and Ag(CN) in a saturated solution of AgCN?

Access for free at OpenStax 15.E.14 https://chem.libretexts.org/@go/page/44123


15.E.2.32: Q15.2.X
In dilute aqueous solution HF acts as a weak acid. However, pure liquid HF (boiling point = 19.5 °C) is a strong acid. In liquid HF,
HNO3 acts like a base and accepts protons. The acidity of liquid HF can be increased by adding one of several inorganic fluorides
that are Lewis acids and accept F– ion (for example, BF3 or SbF5). Write balanced chemical equations for the reaction of pure
HNO3 with pure HF and of pure HF with BF3.

15.E.2.33: S15.2.X
HNO (l) + HF(l) ⟶ H NO
3 2
+

3
+F

; HF(l) + BF 3
(g) ⟶ H
+
+ BF
4

15.E.2.34: Q15.2.X
The simplest amino acid is glycine, H2NCH2CO2H. The common feature of amino acids is that they contain the functional groups:
an amine group, –NH2, and a carboxylic acid group, –CO2H. An amino acid can function as either an acid or a base. For glycine,
the acid strength of the carboxyl group is about the same as that of acetic acid, CH3CO2H, and the base strength of the amino group
is slightly greater than that of ammonia, NH3.

15.E.2.35: Q15.2.X
Write the Lewis structures of the ions that form when glycine is dissolved in 1 M HCl and in 1 M KOH.

15.E.2.36: Q15.2.X
Write the Lewis structure of glycine when this amino acid is dissolved in water. (Hint: Consider the relative base strengths of the –
NH2 and −CO groups.)

2

15.E.2.37: Q15.2.X
Boric acid, H3BO3, is not a Brønsted-Lowry acid but a Lewis acid.
a. Write an equation for its reaction with water.
b. Predict the shape of the anion thus formed.
c. What is the hybridization on the boron consistent with the shape you have predicted?

15.E.2.38: S15.2.X
H BO
3 3
+ H O ⟶ H BO
2 4

4
+H ; The electronic and molecular shapes are the same—both tetrahedral. The tetrahedral
+

structure is consistent with sp3 hybridization.

15.E.3: 15.3: Multiple Equilibria


15.E.3.1: Q15.3.1
A saturated solution of a slightly soluble electrolyte in contact with some of the solid electrolyte is said to be a system in
equilibrium. Explain. Why is such a system called a heterogeneous equilibrium?

15.E.3.2: Q15.3.2
Calculate the equilibrium concentration of Ni2+ in a 1.0-M solution [Ni(NH3)6](NO3)2.

15.E.3.3: S15.3.2
0.014 M

15.E.3.4: Q15.3.3
Calculate the equilibrium concentration of Zn2+ in a 0.30-M solution of Zn(CN) 2−

4
.

15.E.3.5: Q15.3.4
Calculate the equilibrium concentration of Cu2+ in a solution initially with 0.050 M Cu2+ and 1.00 M NH3.

15.E.3.6: S15.3.2
1.0 × 10–13 M

Access for free at OpenStax 15.E.15 https://chem.libretexts.org/@go/page/44123


15.E.3.7: Q15.3.5
Calculate the equilibrium concentration of Zn2+ in a solution initially with 0.150 M Zn2+ and 2.50 M CN–.

15.E.3.8: Q15.3.6
Calculate the Fe3+ equilibrium concentration when 0.0888 mole of K3[Fe(CN)6] is added to a solution with 0.0.00010 M CN–.

15.E.3.9: S15.3.2
9 × 10−22 M

15.E.3.10: Q15.3.7
Calculate the Co2+ equilibrium concentration when 0.100 mole of [Co(NH3)6](NO3)2 is added to a solution with 0.025 M NH3.
Assume the volume is 1.00 L.

15.E.3.11: Q15.3.8
The equilibrium constant for the reaction Hg (aq) + 2 Cl (aq) ⇌ HgCl (aq) is 1.6 × 1013. Is HgCl2 a strong electrolyte or a
2+ −

weak electrolyte? What are the concentrations of Hg2+ and Cl– in a 0.015-M solution of HgCl2?

15.E.3.12: S15.3.2
6.2 × 10–6 M = [Hg2+]; 1.2 × 10–5 M = [Cl–]; The substance is a weak electrolyte because very little of the initial 0.015 M HgCl2
dissolved.

15.E.3.13: Q15.3.9
Calculate the molar solubility of Sn(OH)2 in a buffer solution containing equal concentrations of NH3 and NH . +

15.E.3.14: Q15.3.X
Calculate the molar solubility of Al(OH)3 in a buffer solution with 0.100 M NH3 and 0.400 M NH . +
4

15.E.3.15: S15.3.2
[OH−] = 4.5 × 10−5; [Al3+] = 2.1 × 10–20 (molar solubility)

15.E.3.16: Q15.3.10 Edit section

What is the molar solubility of CaF2 in a 0.100-M solution of HF? Ka for HF = 7.2 × 10–4.

15.E.3.17: Q15.3.X
What is the molar solubility of BaSO4 in a 0.250-M solution of NaHSO4? Ka for HSO = 1.2 × 10–2.

4

15.E.3.18: S15.3.2
2−
[ SO 4 ] = 0.049 M

[Ba ] = 2.2 × 10–9 (molar solubility)


2+

15.E.3.19: Q15.3.X
What is the molar solubility of Tl(OH)3 in a 0.10-M solution of NH3?

15.E.3.20: Q15.3.X
What is the molar solubility of Pb(OH)2 in a 0.138-M solution of CH3NH2?

15.E.3.21: S15.3.2
[OH–] = 7.6 × 10−3 M
[Pb2+] = 4.8 × 10–12 (molar solubility)

15.E.3.22: Q15.3.X
A solution of 0.075 M CoBr2 is saturated with H2S ([H2S] = 0.10 M). What is the minimum pH at which CoS begins to precipitate?
2+ 2− −27
CoS(s) ⇌ Co (aq) + S (aq) Ksp = 4.5 × 10

Access for free at OpenStax 15.E.16 https://chem.libretexts.org/@go/page/44123


+ 2− −26
H S(aq) + 2 H O(l) ⇌ 2 H O (aq) + S (aq) K = 1.0 × 10
2 2 3

A 0.125-M solution of Mn(NO3)2 is saturated with H2S ([H2S] = 0.10 M). At what pH does MnS begin to precipitate?
2+ 2− −22
MnS(s) ⇌ Mn (aq) + S (aq) Ksp = 4.3 × 10

+ 2− −26
H S(aq) + 2 H O(l) ⇌ 2 H O (aq) + S (aq) K = 1.0 × 10
2 2 3

15.E.3.23: S15.3.2
3.27

15.E.3.24: Q15.3.X
Calculate the molar solubility of BaF2 in a buffer solution containing 0.20 M HF and 0.20 M NaF.

15.E.3.25: Q15.3.X
Calculate the molar solubility of CdCO3 in a buffer solution containing 0.115 M Na2CO3 and 0.120 M NaHCO3

15.E.3.26: S15.3.2
2−
[ CO3 ] = 0.115 M

[Cd ] = 3 × 10−12 M
2+

15.E.3.27: Q15.3.X
To a 0.10-M solution of Pb(NO3)2 is added enough HF(g) to make [HF] = 0.10 M.
a. Does PbF2 precipitate from this solution? Show the calculations that support your conclusion.
b. What is the minimum pH at which PbF2 precipitates?
Calculate the concentration of Cd2+ resulting from the dissolution of CdCO3 in a solution that is 0.010 M in H2CO3.

15.E.3.28: S15.3.2
1 × 10−5 M

15.E.3.29: Q15.3.X
Both AgCl and AgI dissolve in NH3.
a. What mass of AgI dissolves in 1.0 L of 1.0 M NH3?
b. What mass of AgCl dissolves in 1.0 L of 1.0 M NH3?
Calculate the volume of 1.50 M CH3CO2H required to dissolve a precipitate composed of 350 mg each of CaCO3, SrCO3, and
BaCO3.

15.E.3.30: S15.3.2
0.0102 L (10.2 mL)

15.E.3.31: Q15.3.X
Even though Ca(OH)2 is an inexpensive base, its limited solubility restricts its use. What is the pH of a saturated solution of
Ca(OH)2?

15.E.3.32: Q15.3.X
What mass of NaCN must be added to 1 L of 0.010 M Mg(NO3)2 in order to produce the first trace of Mg(OH)2?

15.E.3.33: S15.3.2
5 × 10−3 g

15.E.3.34: Q15.3.X
Magnesium hydroxide and magnesium citrate function as mild laxatives when they reach the small intestine. Why do magnesium
hydroxide and magnesium citrate, two very different substances, have the same effect in your small intestine. (Hint: The contents
of the small intestine are basic.)

Access for free at OpenStax 15.E.17 https://chem.libretexts.org/@go/page/44123


15.E.3.35: Q15.3.X
The following question is taken from a Chemistry Advanced Placement Examination and is used with the permission of the
Educational Testing Service.
Solve the following problem:
2+ −
MgF (s) ⇌ Mg (aq) + 2 F (aq)
2

In a saturated solution of MgF2 at 18 °C, the concentration of Mg2+ is 1.21 × 10–3 M. The equilibrium is represented by the
preceding equation.
a. Write the expression for the solubility-product constant, Ksp, and calculate its value at 18 °C.
b. Calculate the equilibrium concentration of Mg2+ in 1.000 L of saturated MgF2 solution at 18 °C to which 0.100 mol of solid KF
has been added. The KF dissolves completely. Assume the volume change is negligible.
c. Predict whether a precipitate of MgF2 will form when 100.0 mL of a 3.00 × 10–3-M solution of Mg(NO3)2 is mixed with 200.0
mL of a 2.00 × 10–3-M solution of NaF at 18 °C. Show the calculations to support your prediction.
d. At 27 °C the concentration of Mg2+ in a saturated solution of MgF2 is 1.17 × 10–3 M. Is the dissolving of MgF2 in water an
endothermic or an exothermic process? Give an explanation to support your conclusion.

15.E.3.36: S15.3.2
Ksp = [Mg2+][F–]2 = (1.21 × 10–3)(2 × 1.21 × 10–3)2 = 7.09 × 10–9; 7.09 × 10–7 M
Determine the concentration of Mg2+ and F– that will be present in the final volume. Compare the value of the ion product [Mg2+]
[F–]2 with Ksp. If this value is larger than Ksp, precipitation will occur. 0.1000 L × 3.00 × 10–3 M Mg(NO3)2 = 0.3000 L × M
Mg(NO3)2 M Mg(NO3)2 = 1.00 × 10–3 M 0.2000 L × 2.00 × 10–3 M NaF = 0.3000 L × M NaF M NaF = 1.33 × 10–3 M ion product
= (1.00 × 10–3)(1.33 × 10–3)2 = 1.77 × 10–9 This value is smaller than Ksp, so no precipitation will occur. MgF2 is less soluble at 27
°C than at 18 °C. Because added heat acts like an added reagent, when it appears on the product side, the Le Chatelier’s principle
states that the equilibrium will shift to the reactants’ side to counter the stress. Consequently, less reagent will dissolve. This
situation is found in our case. Therefore, the reaction is exothermic.

15.E.3.37: Q15.3.X
Which of the following compounds, when dissolved in a 0.01-M solution of HClO4, has a solubility greater than in pure water:
CuCl, CaCO3, MnS, PbBr2, CaF2? Explain your answer.

15.E.3.38: Q15.3.X
Which of the following compounds, when dissolved in a 0.01-M solution of HClO4, has a solubility greater than in pure water:
AgBr, BaF2, Ca3(PO4)3, ZnS, PbI2? Explain your answer.
BaF2, Ca3(PO4)2, ZnS; each is a salt of a weak acid, and the [H 3
O
+
] from perchloric acid reduces the equilibrium concentration of
the anion, thereby increasing the concentration of the cations

15.E.3.39: Q15.3.X
What is the effect on the amount of solid Mg(OH)2 that dissolves and the concentrations of Mg2+ and OH– when each of the
following are added to a mixture of solid Mg(OH)2 and water at equilibrium?
a. MgCl2
b. KOH
c. HClO4
d. NaNO3
e. Mg(OH)2

15.E.3.40: Q15.3.X
What is the effect on the amount of CaHPO4 that dissolves and the concentrations of Ca2+ and HPO when each of the following

4

are added to a mixture of solid CaHPO4 and water at equilibrium?


a. CaCl2
b. HCl
c. KClO4

Access for free at OpenStax 15.E.18 https://chem.libretexts.org/@go/page/44123


d. NaOH
e. CaHPO4

15.E.3.41: S15.3.X
Effect on amount of solid CaHPO4, [Ca2+], [OH–]: increase, increase, decrease; decrease, increase, decrease; no effect, no effect, no
effect; decrease, increase, decrease; increase, no effect, no effect

15.E.3.42: Q15.3.X
Identify all chemical species present in an aqueous solution of Ca3(PO4)2 and list these species in decreasing order of their
concentrations. (Hint: Remember that the PO ion is a weak base.)
3−
4

15.E.3.43: Q15.3.X
A volume of 50 mL of 1.8 M NH3 is mixed with an equal volume of a solution containing 0.95 g of MgCl2. What mass of NH4Cl
must be added to the resulting solution to prevent the precipitation of Mg(OH)2?

15.E.3.44: S15.3.X
7.1 g

15.E.4:

This page titled 15.E: Equilibria of Other Reaction Classes (Exercises) is shared under a CC BY 4.0 license and was authored, remixed, and/or
curated by OpenStax via source content that was edited to the style and standards of the LibreTexts platform; a detailed edit history is available
upon request.

Access for free at OpenStax 15.E.19 https://chem.libretexts.org/@go/page/44123


CHAPTER OVERVIEW
16: Thermodynamics

A general chemistry Libretexts Textbook remixed and remastered from


OpenStax's textbook:
General Chemistry
Among the many capabilities of chemistry is its ability to predict if a process will occur under specified conditions.
Thermodynamics, the study of relationships between the energy and work associated with chemical and physical processes,
provides this predictive ability. Previous chapters in this text have described various applications of thermochemistry, an important
aspect of thermodynamics concerned with the heat flow accompanying chemical reactions and phase transitions. This chapter will
introduce additional thermodynamic concepts, including those that enable the prediction of any chemical or physical changes under
a given set of conditions.
16.1: Spontaneity
16.2: Entropy
16.3: The Second and Third Laws of Thermodynamics
16.4: Gibbs Energy
16.E: Thermodynamics (Exercises)

This page titled 16: Thermodynamics is shared under a CC BY 4.0 license and was authored, remixed, and/or curated by OpenStax via source
content that was edited to the style and standards of the LibreTexts platform; a detailed edit history is available upon request.

1
16.1: Spontaneity
 Learning Objectives
Distinguish between spontaneous and nonspontaneous processes
Describe the dispersal of matter and energy that accompanies certain spontaneous processes

In this section, consider the differences between two types of changes in a system: Those that occur spontaneously and those that occur only with the continuous input of energy. In doing so,
we’ll gain an understanding as to why some systems are naturally inclined to change in one direction under certain conditions. We’ll also gain insight into how the spontaneity of a process
affects the distribution of energy and matter within the system.

16.1.1: Spontaneous and Nonspontaneous Processes


Processes have a natural tendency to occur in one direction under a given set of conditions. Water will naturally flow downhill, but uphill flow requires outside intervention such as the use of a
pump. A spontaneous process is one that occurs naturally under certain conditions. A nonspontaneous process, on the other hand, will not take place unless it is “driven” by the continual input
of energy from an external source. A process that is spontaneous in one direction under a particular set of conditions is nonspontaneous in the reverse direction. At room temperature and typical
atmospheric pressure, for example, ice will spontaneously melt, but water will not spontaneously freeze.
The spontaneity of a process is not correlated to the speed of the process. A spontaneous change may be so rapid that it is essentially instantaneous or so slow that it cannot be observed over any
practical period of time. To illustrate this concept, consider the decay of radioactive isotopes, a topic more thoroughly treated in the chapter on nuclear chemistry. Radioactive decay is by
definition a spontaneous process in which the nuclei of unstable isotopes emit radiation as they are converted to more stable nuclei. All the decay processes occur spontaneously, but the rates at
which different isotopes decay vary widely. Technetium-99m is a popular radioisotope for medical imaging studies that undergoes relatively rapid decay and exhibits a half-life of about six
hours. Uranium-238 is the most abundant isotope of uranium, and its decay occurs much more slowly, exhibiting a half-life of more than four billion years (Figure 16.1.1).

Figure 16.1.1 : Both U-238 and Tc-99m undergo spontaneous radioactive decay, but at drastically different rates. Over the course of one week, essentially all of a Tc-99m sample and none of a
U-238 sample will have decayed. (CC by 4.0; Morgan Johnson via LibreTexts)
Two curves are shown to represent U-238 and Tc-99m respectively. The vertical axes represents the percentage of isotope remaining and the horizontal axes is the time that has elapsed in days.
As another example, consider the conversion of diamond into graphite (Figure 16.1.2).
C(s, diamond) ⟶ C(s, graphite) (16.1.1)

The phase diagram for carbon indicates that graphite is the stable form of this element under ambient atmospheric pressure, while diamond is the stable allotrope at very high pressures, such as
those present during its geologic formation. Thermodynamic calculations of the sort described in the last section of this chapter indicate that the conversion of diamond to graphite at ambient
pressure occurs spontaneously, yet diamonds are observed to exist, and persist, under these conditions. Though the process is spontaneous under typical ambient conditions, its rate is extremely
slow, and so for all practical purposes diamonds are indeed “forever.” Situations such as these emphasize the important distinction between the thermodynamic and the kinetic aspects of a
process. In this particular case, diamonds are said to be thermodynamically unstable but kinetically stable under ambient conditions.

Figure 16.1.2 : The conversion of carbon from the diamond allotrope to the graphite allotrope is spontaneous at ambient pressure, but its rate is immeasurably slow at low to moderate
temperatures. This process is known as graphitization, and its rate can be increased to easily measurable values at temperatures in the 1000–2000 K range. (credit "diamond" photo: modification
of work by "Fancy Diamonds"/Flickr; credit "graphite" photo: modification of work by images-of-elements.com/carbon.php)
Comparison of diamond and graphite shown in its physical form as well as its molecular arrangement respectively.

16.1.2: Dispersal of Matter and Energy


As we extend our discussion of thermodynamic concepts toward the objective of predicting spontaneity, consider now an isolated system consisting of two flasks connected with a closed valve.
Initially there is an ideal gas on the left and a vacuum on the right (Figure 16.1.3). When the valve is opened, the gas spontaneously expands to fill both flasks. Recalling the definition of
pressure-volume work from the chapter on thermochemistry, note that no work has been done because the pressure in a vacuum is zero.
w = −P ΔV (16.1.2)

=0 (P = 0 in a vaccum) (16.1.3)

Note as well that since the system is isolated, no heat has been exchanged with the surroundings (q = 0). The first law of thermodynamics confirms that there has been no change in the system’s
internal energy as a result of this process.

Access for free at OpenStax 16.1.1 https://chem.libretexts.org/@go/page/38294


ΔU = q + w (First Law of Thermodynamics)

= 0 +0 = 0 (16.1.4)

The spontaneity of this process is therefore not a consequence of any change in energy that accompanies the process. Instead, the movement of the gas appears to be related to the greater, more
uniform dispersal of matter that results when the gas is allowed to expand. Initially, the system was comprised of one flask containing matter and another flask containing nothing. After the
spontaneous process took place, the matter was distributed both more widely (occupying twice its original volume) and more uniformly (present in equal amounts in each flask).

Figure 16.1.3 : An isolated system consists of an ideal gas in one flask that is connected by a closed valve to a second flask containing a vacuum. Once the valve is opened, the gas
spontaneously becomes evenly distributed between the flasks.
When the valve is closed, all of the gas molecules accumlating only in one side of the flask. The diagram with the open valve shows gas being equally distributed among the two flasks. The
dispersion of the gas is labeled as spontaneous while the reverse is labeled as non spontaneous.
Now consider two objects at different temperatures: object X at temperature TX and object Y at temperature TY, with TX > TY (Figure 16.1.4). When these objects come into contact, heat
spontaneously flows from the hotter object (X) to the colder one (Y). This corresponds to a loss of thermal energy by X and a gain of thermal energy by Y.

qX < 0 and qY = −qX > 0 (16.1.5)

From the perspective of this two-object system, there was no net gain or loss of thermal energy, rather the available thermal energy was redistributed among the two objects. This spontaneous
process resulted in a more uniform dispersal of energy.

Figure 16.1.4 :When two objects at different temperatures come in contact, heat spontaneously flows from the hotter to the colder object.
Two separated blocks. One is labeled X and the other labeled Y. The diagram next to it shows the two blocks in contact with one another.
As illustrated by the two processes described, an important factor in determining the spontaneity of a process is the extent to which it changes the dispersal or distribution of matter and/or
energy. In each case, a spontaneous process took place that resulted in a more uniform distribution of matter or energy.

 Example 16.1.1: Redistribution of Matter during a Spontaneous Process

Describe how matter and energy are redistributed when the following spontaneous processes take place:
a. A solid sublimes.
b. A gas condenses.
c. A drop of food coloring added to a glass of water forms a solution with uniform color.

Solution

Figure 16.1.5 :(credit a: modification of work by Jenny Downing; credit b: modification of work by “Fuzzy Gerdes”/Flickr; credit c: modification of work by Sahar Atwa)
This figure has three photos labeled, “a,” “b,” and “c.” Photo a shows a glass with dry ice in water. There is a thick vapor coming from the top of the glass. Photo b shows water forming
outside of a glass containing cold beverage. Photo c shows a sealed container that holds a red liquid.
a. Sublimation is the conversion of a solid (relatively high density) to a gas (much lesser density). This process yields a much greater dispersal of matter, since the molecules will occupy a
much greater volume after the solid-to-gas transition. However, an input of energy from the surroundings ss required for the molecules to leave the solid phase and enter the gas phase.
b. Condensation is the conversion of a gas (relatively low density) to a liquid (much greater density). This process yields a much lesser dispersal of matter, since the molecules will occupy
a much lesser volume after the gas-to-liquid transition. As the gas molecules move together to form the droplets of liquid, they form intermolecular forces and thus release energy to the
surroundings.
c. The process in question is dilution. The food dye molecules initially occupy a much smaller volume (the drop of dye solution) than they occupy once the process is complete (in the full
glass of water). The process therefore entails a greater dispersal of matter. The process may also yield a more uniform dispersal of matter, since the initial state of the system involves
two regions of different dye concentrations (high in the drop, zero in the water), and the final state of the system contains a single dye concentration throughout. This process can occur
with out a change in energy because the molecules have kinetic energy relative to the temperature of the water, and so will be constantly in motion.

 Exercise 16.1.1
Describe how matter and energy are redistributed when you empty a canister of compressed air into a room.

Answer
This process entails both a greater and more uniform dispersal of matter as the compressed air in the canister is permitted to expand into the lower-pressure air of the room. The process
also requires an input of energy to disrupt the intermolecular forces between the closely-spaced gas molecules that are originally compressed into the container. If you were to touch the
nozzle of the canister, you would notice that it is cold because the exiting molecules are taking energy away from their surroundings, and the canister is part of the surroundings.

Summary
Chemical and physical processes have a natural tendency to occur in one direction under certain conditions. A spontaneous process occurs without the need for a continual input of energy from
some external source, while a nonspontaneous process requires such. Systems undergoing a spontaneous process may or may not experience a gain or loss of energy, but they will experience a

Access for free at OpenStax 16.1.2 https://chem.libretexts.org/@go/page/38294


change in the way matter and/or energy is distributed within the system. In this section we have only discussed nuclear decay, physical changes of pure substances, and macroscopic events such
as water flowing downhill. In the following sections we will discuss mixtures and chemical reactions, situations in which the description of sponteneity becomes more challenging.

Glossary
nonspontaneous process
process that requires continual input of energy from an external source

spontaneous change
process that takes place without a continuous input of energy from an external source

This page titled 16.1: Spontaneity is shared under a CC BY 4.0 license and was authored, remixed, and/or curated by OpenStax via source content that was edited to the style and standards of the LibreTexts
platform; a detailed edit history is available upon request.

Access for free at OpenStax 16.1.3 https://chem.libretexts.org/@go/page/38294


Access for free at OpenStax 16.1.4 https://chem.libretexts.org/@go/page/38294
16.2: Entropy
 Learning Objectives
Define entropy
Explain the relationship between entropy and the number of microstates
Predict the sign of the entropy change for chemical and physical processes

In 1824, at the age of 28, Nicolas Léonard Sadi Carnot (Figure 16.2.2) published the results of an extensive study regarding the
efficiency of steam heat engines. In a later review of Carnot’s findings, Rudolf Clausius introduced a new thermodynamic property
that relates the spontaneous heat flow accompanying a process to the temperature at which the process takes place. This new
property was expressed as the ratio of the reversible heat (qrev) and the kelvin temperature (T). The term reversible process refers to
a process that takes place at such a slow rate that it is always at equilibrium and its direction can be changed (it can be “reversed”)
by an infinitesimally small change is some condition. Note that the idea of a reversible process is a formalism required to support
the development of various thermodynamic concepts; no real processes are truly reversible, rather they are classified as
irreversible.

Figure 16.2.1 : (a) Nicholas Léonard Sadi Carnot’s research into steam-powered machinery and (b) Rudolf Clausius’s later study of
those findings led to groundbreaking discoveries about spontaneous heat flow processes.
Similar to other thermodynamic properties, this new quantity is a state function, and so its change depends only upon the initial and
final states of a system. In 1865, Clausius named this property entropy (S) and defined its change for any process as the following:
qrev
ΔS = (16.2.1)
T

The entropy change for a real, irreversible process is then equal to that for the theoretical reversible process that involves the same
initial and final states.

16.2.1: Entropy and Microstates


Following the work of Carnot and Clausius, Ludwig Boltzmann developed a molecular-scale statistical model that related the
entropy of a system to the number of microstates possible for the system. A microstate (Ω) is a specific configuration of the
locations and energies of the atoms or molecules that comprise a system like the following:
S = k ln Ω (16.2.2)

Here k is the Boltzmann constant and has a value of 1.38 × 10 −23


J/K .
As for other state functions, the change in entropy for a process is the difference between its final (Sf) and initial (Si) values:

ΔS = Sf − Si

= k ln Ωf − k ln Ωi

Ωf
= k ln (16.2.3)
Ωi

For processes involving an increase in the number of microstates of the system, Ω > Ω , the entropy of the system increases,
f i

ΔS > 0 . Conversely, processes that reduce the number of microstates in the system, Ω < Ω , yield a decrease in system entropy,
f i

Access for free at OpenStax 16.2.1 https://chem.libretexts.org/@go/page/38295


ΔS < 0 . This molecular-scale interpretation of entropy provides a link to the probability that a process will occur as illustrated in
the next paragraphs.

Figure 16.2.2 : The sixteen microstates associated with placing four particles in two boxes are shown. The microstates are collected
into five distributions—(a), (b), (c), (d), and (e)—based on the numbers of particles in each box.
Five rows of diagrams that look like dominoes are shown and labeled a, b, c, d, and e. Row a has one “domino” that has four dots
on the left side, red, green, blue and yellow in a clockwise pattern from the top left, and no dots on the right. Row b has four
“dominos,” each with three dots on the left and one dot on the right. The first shows a “domino” with green, yellow and blue on the
left and red on the right. The second “domino” has yellow, blue and red on the left and green on the right. The third “domino” has
red, green and yellow on the left and blue on the right while the fourth has red, green and blue on the left and yellow on the right.
Row c has six “dominos”, each with two dots on either side. The first has a red and green on the left and a blue and yellow on the
right. The second has a red and blue on the left and a green and yellow on the right while the third has a yellow and red on the left
and a green and blue on the right. The fourth has a green and blue on the left and a red and yellow on the right. The fifth has a
green and yellow on the left and a red and blue on the right. The sixth has a blue and yellow on the left and a green and red on the
right. Row d has four “dominos,” each with one dot on the left and three on the right. The first “domino” has red on the left and a
blue, green and yellow on the right. The second has a green on the left and a red, yellow and blue on the right. The third has a blue
on the left and a red, green and yellow on the right. The fourth has a yellow on the left and a red, green and blue on the right. Row e
has 1 “domino” with no dots on the left and four dots on the right that are red, green, blue and yellow.
Consider the general case of a system comprised of N particles distributed among n boxes. The number of microstates possible for
such a system is nN. For example, distributing four particles among two boxes will result in 24 = 16 different microstates as
illustrated in Figure 16.2.2. Microstates with equivalent particle arrangements (not considering individual particle identities) are
grouped together and are called distributions (sometimes called macrostates or configurations). The probability that a system will
exist with its components in a given distribution is proportional to the number of microstates within the distribution. Since entropy
increases logarithmically with the number of microstates, the most probable distribution is therefore the one of greatest entropy.
For this system, the most probable configuration is one of the six microstates associated with distribution (c) where the particles are
evenly distributed between the boxes, that is, a configuration of two particles in each box. The probability of finding the system in
this configuration is

6 3
=
16 8

The least probable configuration of the system is one in which all four particles are in one box, corresponding to distributions (a)
and (e), each with a probability of
1

16

The probability of finding all particles in only one box (either the left box or right box) is then
1 1 2 1
( + ) = =
16 16 16 8

Access for free at OpenStax 16.2.2 https://chem.libretexts.org/@go/page/38295


As you add more particles to the system, the number of possible microstates increases exponentially (2N). A macroscopic
(laboratory-sized) system would typically consist of moles of particles (N ~ 1023), and the corresponding number of microstates
would be staggeringly huge. Regardless of the number of particles in the system, however, the distributions in which roughly equal
numbers of particles are found in each box are always the most probable configurations.

The most probable distribution is therefore the one of greatest entropy.


The previous description of an ideal gas expanding into a vacuum is a macroscopic example of this particle-in-a-box model. For
this system, the most probable distribution is confirmed to be the one in which the matter is most uniformly dispersed or distributed
between the two flasks. The spontaneous process whereby the gas contained initially in one flask expands to fill both flasks equally
therefore yields an increase in entropy for the system.

Figure 16.2.3 : This shows a microstate model describing the flow of heat from a hot object to a cold object. (a) Before the heat
flow occurs, the object comprised of particles A and B contains both units of energy and as represented by a distribution of three
microstates. (b) If the heat flow results in an even dispersal of energy (one energy unit transferred), a distribution of four
microstates results. (c) If both energy units are transferred, the resulting distribution has three microstates.
Three rows labeled a, b, and c are shown and each contains rectangles with two sides where the left side is labeled, “A,” and “B,”
and the right is labeled, “C,” and “D.” Row a has three rectangles where the first has a dot above and below the letter A, the second
has a dot above the A and B, and the third which has a dot above and below the letter B. Row b has four rectangles; the first has a
dot above A and C, the second has a dot above A and D, the third has a dot above B and C and the fourth has a dot above B and D.
Row c has three rectangles; the first has a dot above and below the letter C, the second has a dot above C and D and the third has a
dot above and below the letter D.
A similar approach may be used to describe the spontaneous flow of heat. Consider a system consisting of two objects, each
containing two particles, and two units of energy (represented as “*”) in Figure 16.2.3. The hot object is comprised of particles A
and B and initially contains both energy units. The cold object is comprised of particles C and D, which initially has no energy
units. Distribution (a) shows the three microstates possible for the initial state of the system, with both units of energy contained
within the hot object. If one of the two energy units is transferred, the result is distribution (b) consisting of four microstates. If both
energy units are transferred, the result is distribution (c) consisting of three microstates. And so, we may describe this system by a
total of ten microstates. The probability that the heat does not flow when the two objects are brought into contact, that is, that the
system remains in distribution (a), is . More likely is the flow of heat to yield one of the other two distribution, the combined
3

10

probability being . The most likely result is the flow of heat to yield the uniform dispersal of energy represented by distribution
7

10

(b), the probability of this configuration being . As for the previous example of matter dispersal, extrapolating this treatment to
4

10

macroscopic collections of particles dramatically increases the probability of the uniform distribution relative to the other
distributions. This supports the common observation that placing hot and cold objects in contact results in spontaneous heat flow
that ultimately equalizes the objects’ temperatures. And, again, this spontaneous process is also characterized by an increase in
system entropy.

 Example 16.2.1: Determination of ΔS

Consider the system shown here. What is the change in entropy for a process that converts the system from distribution (a) to
(c)?

Access for free at OpenStax 16.2.3 https://chem.libretexts.org/@go/page/38295


A diagram shows one rectangle with two sides that has four dots, red, green, yellow and blue written on the left side. A right-
facing arrow leads to six more two-sided rectangles, each with two dots on the left and right sides. The first rectangle has a red
and green dot on the left and a blue and yellow on the right, while the second shows a red and blue on the left and a green and
yellow on the right. The third rectangle has a red and yellow dot on the left and a blue and green on the right, while the fourth
shows a green and blue on the left and a red and yellow on the right. The fifth rectangle has a yellow and green dot on the left
and a blue and red on the right, while the sixth shows a yellow and blue on the left and a green and red on the right.

Solution
We are interested in the following change:
The initial number of microstates is one, the final six:
Ωc
ΔS = k ln
Ωa

−23
6
= 1.38 × 10 J/K × ln
1

−23
= 2.47 × 10 J/K

The sign of this result is consistent with expectation; since there are more microstates possible for the final state than for the
initial state, the change in entropy should be positive.

 Exercise 16.2.1
Consider the system shown in Figure 16.2.3. What is the change in entropy for the process where all the energy is transferred
from the hot object (AB) to the cold object (CD)?

Answer
0 J/K

16.2.2: Predicting the Sign of ΔS


The relationships between entropy, microstates, and matter/energy dispersal described previously allow us to make generalizations
regarding the relative entropies of substances and to predict the sign of entropy changes for chemical and physical processes.
Consider the phase changes illustrated in Figure 16.2.4. In the solid phase, the atoms or molecules are restricted to nearly fixed
positions with respect to each other and are capable of only modest oscillations about these positions. With essentially fixed
locations for the system’s component particles, the number of microstates is relatively small. In the liquid phase, the atoms or
molecules are free to move over and around each other, though they remain in relatively close proximity to one another. This
increased freedom of motion results in a greater variation in possible particle locations, so the number of microstates is
correspondingly greater than for the solid. As a result, Sliquid > Ssolid and the process of converting a substance from solid to liquid
(melting) is characterized by an increase in entropy, ΔS > 0. By the same logic, the reciprocal process (freezing) exhibits a decrease
in entropy, ΔS < 0.

Access for free at OpenStax 16.2.4 https://chem.libretexts.org/@go/page/38295


Figure 16.2.4 : The entropy of a substance increases (ΔS > 0) as it transforms from a relatively ordered solid, to a less-ordered
liquid, and then to a still less-ordered gas. The entropy decreases (ΔS < 0) as the substance transforms from a gas to a liquid and
then to a solid.
Three stoppered flasks are shown with right and left-facing arrows in between each; the first is labeled above as, “delta S greater
than 0,” and below as, “delta S less than 0,” while the second is labeled above as, “delta S greater than 0,” and below as, “delta S
less than 0.” A long, right-facing arrow is drawn above all the flasks and labeled, “Increasing entropy.” The left flask contains
twenty-seven particles arranged in a cube in the bottom of the flask and is labeled, “Crystalline solid,” below. The middle flask
contains twenty-seven particles dispersed randomly in the bottom of the flask and is labeled, “Liquid,” below. The right flask
contains twenty-seven particles dispersed inside of the flask and moving rapidly and is labeled, “Gas,” below.
Now consider the vapor or gas phase. The atoms or molecules occupy a much greater volume than in the liquid phase; therefore
each atom or molecule can be found in many more locations than in the liquid (or solid) phase. Consequently, for any substance,
Sgas > Sliquid > Ssolid, and the processes of vaporization and sublimation likewise involve increases in entropy, ΔS > 0. Likewise, the
reciprocal phase transitions, condensation and deposition, involve decreases in entropy, ΔS < 0.
According to kinetic-molecular theory, the temperature of a substance is proportional to the average kinetic energy of its particles.
Raising the temperature of a substance will result in more extensive vibrations of the particles in solids and more rapid translations
of the particles in liquids and gases. At higher temperatures, the distribution of kinetic energies among the atoms or molecules of
the substance is also broader (more dispersed) than at lower temperatures. Thus, the entropy for any substance increases with
temperature (Figure 16.2.5 ).

Figure 16.2.5 : Entropy increases as the temperature of a substance is raised, which corresponds to the greater spread of kinetic
energies. When a substance melts or vaporizes, it experiences a significant increase in entropy.
Two graphs are shown. The y-axis of the left graph is labeled, “Fraction of molecules,” while the x-axis is labeled, “Velocity, v ( m
/ s ),” and has values of 0 through 1,500 along the axis with increments of 500. Four lines are plotted on this graph. The first,
labeled, “100 K,” peaks around 200 m / s while the second, labeled, “200 K,” peaks near 300 m / s and is slightly lower on the y-
axis than the first. The third line, labeled, “500 K,” peaks around 550 m / s and is lower than the first two on the y-axis. The fourth
line, labeled, “1000 K,” peaks around 750 m / s and is the lowest of the four on the y-axis. Each line get increasingly broad. The
second graph has a y-axis labeled, “Entropy, S,” with an upward-facing arrow and an x-axis labeled, “Temperature ( K ),” and a
right-facing arrow. The graph has three equally spaced columns in the background, labeled, “Solid,” “Liquid,” and, “Gas,” from
left to right. A line extends slightly upward through the first column in a slight upward direction, then goes straight up in the
transition between the first two columns. In then progresses in a slight upward direction through the second column, then goes up
dramatically between the second and third columns, then continues in a slight upward direction once more. The first vertical region
of this line is labeled, “Melting,” and the second is labeled, “Boiling.”

Access for free at OpenStax 16.2.5 https://chem.libretexts.org/@go/page/38295


The entropy of a substance is influenced by structure of the particles (atoms or molecules) that comprise the substance. With regard
to atomic substances, heavier atoms possess greater entropy at a given temperature than lighter atoms, which is a consequence of
the relation between a particle’s mass and the spacing of quantized translational energy levels (which is a topic beyond the scope of
our treatment). For molecules, greater numbers of atoms (regardless of their masses) increase the ways in which the molecules can
vibrate and thus the number of possible microstates and the system entropy.
Finally, variations in the types of particles affects the entropy of a system. Compared to a pure substance, in which all particles are
identical, the entropy of a mixture of two or more different particle types is greater. This is because of the additional orientations
and interactions that are possible in a system comprised of nonidentical components. For example, when a solid dissolves in a
liquid, the particles of the solid experience both a greater freedom of motion and additional interactions with the solvent particles.
This corresponds to a more uniform dispersal of matter and energy and a greater number of microstates. The process of dissolution
therefore involves an increase in entropy, ΔS > 0.
Considering the various factors that affect entropy allows us to make informed predictions of the sign of ΔS for various chemical
and physical processes as illustrated in Example .

 Example 16.2.2: Predicting the Sign of ∆S

Predict the sign of the entropy change for the following processes. Indicate the reason for each of your predictions.
a. One mole liquid water at room temperature ⟶ one mole liquid water at 50 °C
b. Ag (aq) + Cl (aq) ⟶ AgCl(s)
+ −

15
c. C H (l) +
6 6
O (g) ⟶ 6 CO (g) + 3 H O(l)
2 2 2
2
d. NH 3
(s) ⟶ NH (l)
3

Solution
a. positive, temperature increases
b. negative, reduction in the number of ions (particles) in solution, decreased dispersal of matter
c. negative, net decrease in the amount of gaseous species
d. positive, phase transition from solid to liquid, net increase in dispersal of matter

 Exercise 16.2.2
Predict the sign of the enthalpy change for the following processes. Give a reason for your prediction.
a. NaNO (s) ⟶ Na (aq) + NO (aq)
3
+ −
3

b. the freezing of liquid water


c. CO (s) ⟶ CO (g)
2 2

d. CaCO(s) ⟶ CaO(s) + CO (g) 2

Answer a
Positive; The solid dissolves to give an increase of mobile ions in solution.
Answer b
Negative; The liquid becomes a more ordered solid.
Answer c
Positive; The relatively ordered solid becomes a gas
Answer d
Positive; There is a net production of one mole of gas.

Access for free at OpenStax 16.2.6 https://chem.libretexts.org/@go/page/38295


Summary
Entropy (\(S\)) is a state function that can be related to the number of microstates for a system (the number of ways the system can
be arranged) and to the ratio of reversible heat to kelvin temperature. It may be interpreted as a measure of the dispersal or
distribution of matter and/or energy in a system, and it is often described as representing the “disorder” of the system. For a given
substance, S solid<S liquid≪ S gas in a given physical state at a given temperature, entropy is typically greater for heavier atoms or
more complex molecules. Entropy increases when a system is heated and when solutions form. Using these guidelines, the sign of
entropy changes for some chemical reactions may be reliably predicted.

16.2.3: Key Equations


qrev
ΔS =
T
S = k ln W
Wf
ΔS = k ln
Wi

Glossary
entropy (S)
state function that is a measure of the matter and/or energy dispersal within a system, determined by the number of system
microstates often described as a measure of the disorder of the system
microstate (W)
possible configuration or arrangement of matter and energy within a system
reversible process
process that takes place so slowly as to be capable of reversing direction in response to an infinitesimally small change in
conditions; hypothetical construct that can only be approximated by real processes removed

This page titled 16.2: Entropy is shared under a CC BY 4.0 license and was authored, remixed, and/or curated by OpenStax via source content
that was edited to the style and standards of the LibreTexts platform; a detailed edit history is available upon request.

Access for free at OpenStax 16.2.7 https://chem.libretexts.org/@go/page/38295


16.3: The Second and Third Laws of Thermodynamics
 Learning Objectives
State and explain the second and third laws of thermodynamics
Calculate entropy changes for phase transitions and chemical reactions under standard conditions

16.3.1: The Second Law of Thermodynamics


In the quest to identify a property that may reliably predict the spontaneity of a process, we have identified a very promising
candidate: entropy. Processes that involve an increase in entropy of the system (ΔS > 0) are very often spontaneous; however,
examples to the contrary are plentiful. By expanding consideration of entropy changes to include the surroundings, we may reach a
significant conclusion regarding the relation between this property and spontaneity. In thermodynamic models, the system and
surroundings comprise everything, that is, the universe, and so the following is true:

ΔSuniv = ΔSsys + ΔSsurr (16.3.1)

To illustrate this relation, consider again the process of heat flow between two objects, one identified as the system and the other as
the surroundings. There are three possibilities for such a process:
1. The objects are at different temperatures, and heat flows from the hotter to the cooler object. This is always observed to occur
spontaneously. Designating the hotter object as the system and invoking the definition of entropy yields the following:
−qrev qrev
ΔSsys = and ΔSsurr = (16.3.2)
Tsys Tsurr

The arithmetic signs of qrev denote the loss of heat by the system and the gain of heat by the surroundings. Since Tsys > Tsurr in
this scenario, the magnitude of the entropy change for the surroundings will be greater than that for the system, and so the sum
of ΔSsys and ΔSsurr will yield a positive value for ΔSuniv. This process involves an increase in the entropy of the universe.
2. The objects are at different temperatures, and heat flows from the cooler to the hotter object. This is never observed to occur
spontaneously. Again designating the hotter object as the system and invoking the definition of entropy yields the following:
qrev −qrev
ΔSsys = and ΔSsurr = (16.3.3)
Tsys Tsurr

The arithmetic signs of qrev denote the gain of heat by the system and the loss of heat by the surroundings. The magnitude of the
entropy change for the surroundings will again be greater than that for the system, but in this case, the signs of the heat changes
will yield a negative value for ΔSuniv. This process involves a decrease in the entropy of the universe.
3. The temperature difference between the objects is infinitesimally small, Tsys ≈ Tsurr, and so the heat flow is thermodynamically
reversible. See the previous section’s discussion). In this case, the system and surroundings experience entropy changes that are
equal in magnitude and therefore sum to yield a value of zero for ΔSuniv. This process involves no change in the entropy of the
universe.
These results lead to a profound statement regarding the relation between entropy and spontaneity known as the second law of
thermodynamics: all spontaneous changes cause an increase in the entropy of the universe. A summary of these three relations is
provided in Table 16.3.1.
Table 16.3.1 : The Second Law of Thermodynamics
ΔSuniv > 0 spontaneous

ΔSuniv < 0 nonspontaneous (spontaneous in opposite direction)

ΔSuniv = 0 reversible (system is at equilibrium)

Definition: The Second Law of Thermodynamics


All spontaneous changes cause an increase in the entropy of the universe.
For many realistic applications, the surroundings are vast in comparison to the system. In such cases, the heat gained or lost by the
surroundings as a result of some process represents a very small, nearly infinitesimal, fraction of its total thermal energy. For
example, combustion of a fuel in air involves transfer of heat from a system (the fuel and oxygen molecules undergoing reaction) to

Access for free at OpenStax 16.3.1 https://chem.libretexts.org/@go/page/38296


surroundings that are infinitely more massive (the earth’s atmosphere). As a result, qsurr is a good approximation of qrev , and the
second law may be stated as the following:
qsurr
ΔSuniv = ΔSsys + ΔSsurr = ΔSsys + (16.3.4)
T

We may use this equation to predict the spontaneity of a process as illustrated in Example 16.3.1.

 Example 16.3.1: Will Ice Spontaneously Melt?

The entropy change for the process


H O(s) ⟶ H O(l)
2 2

is 22.1 J/K and requires that the surroundings transfer 6.00 kJ of heat to the system. Is the process spontaneous at −10.00 °C? Is
it spontaneous at +10.00 °C?

Solution
We can assess the spontaneity of the process by calculating the entropy change of the universe. If ΔSuniv is positive, then the
process is spontaneous. At both temperatures, ΔSsys = 22.1 J/K and qsurr = −6.00 kJ.
At −10.00 °C (263.15 K), the following is true:
qsurr
ΔSuniv = ΔSsys + ΔSsurr = ΔSsys +
T
3
−6.00 × 10 J
= 22.1 J/K + = −0.7 J/K
263.15 K

Suniv < 0 , so melting is nonspontaneous (not spontaneous) at −10.0 °C.


At 10.00 °C (283.15 K), the following is true:
qsurr
ΔSuniv = ΔSsys +
T

3
−6.00 × 10 J
= 22.1 J/K + = +0.9 J/K
283.15 K

Suniv > 0 , so melting is spontaneous at 10.00 °C.

 Exercise 16.3.1

Using this information, determine if liquid water will spontaneously freeze at the same temperatures. What can you say about
the values of Suniv?

Answer
Entropy is a state function, and freezing is the opposite of melting. At −10.00 °C spontaneous, +0.7 J/K; at +10.00 °C
nonspontaneous, −0.9 J/K.

16.3.2: The Third Law of Thermodynamics


The previous section described the various contributions of matter and energy dispersal that contribute to the entropy of a system.
With these contributions in mind, consider the entropy of a pure, perfectly crystalline solid possessing no kinetic energy (that is, at
a temperature of absolute zero, 0 K). This system may be described by a single microstate, as its purity, perfect crystallinity and
complete lack of motion means there is but one possible location for each identical atom or molecule comprising the crystal (W =
1). According to the Boltzmann equation, the entropy of this system is zero.
S = k ln W = k ln(1) = 0 (16.3.5)

Access for free at OpenStax 16.3.2 https://chem.libretexts.org/@go/page/38296


This limiting condition for a system’s entropy represents the third law of thermodynamics: the entropy of a pure, perfect crystalline
substance at 0 K is zero.

 Definition: Third Law of Thermodynamics

The entropy of a pure, perfect crystalline substance at 0 K is zero.

We can make careful calorimetric measurements to determine the temperature dependence of a substance’s entropy and to derive
absolute entropy values under specific conditions. Standard entropies are given the label S for values determined for one mole

298

of substance, isolated in its pure form in its own container, at a pressure of 1 bar and a temperature of 298 K.

 Definition: Term

The thermodynamic standard state of a substance refers to an isolated sample of that substance, in its own container, at 1.000
bar (0.9869 atm) pressure. If the substance is a solute, the most common standard state is one in which the concentration of the
solute is 1.000 molal (sometimes approximated with 1.000 M). There is no defined temperature for the standard state, but most
discussions about standard state assume that the temperature is 298.15 K (25ºC) unless otherwise noted.
This may seem like a strange definition, because it requires that each of the reactants and each of the products of a reaction
are kept separate from one another, unmixed. The entropy of mixing must be determined separately.

The standard entropy change (ΔS°) for any process may be computed from the standard entropies of its reactant and product
species like the following:
∘ ∘
ΔS° = ∑ ν S (products) − ∑ ν S (reactants) (16.3.6)
298 298

Here, ν represents stoichiometric coefficients in the balanced equation representing the process. For example, ΔS° for the following
reaction at room temperature
mA + nB ⟶ xC + yD (16.3.7)

is computed as the following:


∘ ∘ ∘ ∘
= [x S (C) + y S (D)] − [m S (A) + nS (B)] (16.3.8)
298 298 298 298

Table 16.3.2 lists some standard entropies at 298.15 K. You can find additional standard entropies in Tables T1 or T2.
Table 16.3.2 : Standard Entropies (at 298.15 K, 1 atm)
J
Substance S

298
mol K

carbon

C(s, graphite) 5.740

C(s, diamond) 2.38

CO(g) 197.7

CO2(g) 213.8

CH4(g) 186.3

C2H4(g) 219.5

C2H6(g) 229.5

CH3OH(l) 126.8

C2H5OH(l) 160.7

hydrogen

H2(g) 130.57

H(g) 114.6

Access for free at OpenStax 16.3.3 https://chem.libretexts.org/@go/page/38296


J
Substance S
298

mol K

H2O(g) 188.71

H2O(l) 69.91

HCI(g) 186.8

H2S(g) 205.7

oxygen

O2(g) 205.03

 Example 16.3.2: Determination of ΔS°

Calculate the standard entropy change for the following process:

H O(g) ⟶ H O(l)
2 2

Solution
The value of the standard entropy change at room temperature, ΔS ∘
298
, is the difference between the standard entropy of the
product, H2O(l), and the standard entropy of the reactant, H2O(g).
∘ ∘ ∘
ΔS =S (H O(l)) − S (H O(g))
298 298 2 298 2

−1 −1 −1 −1 −1 −1
= (70.0 J mol K ) − (188.8 Jmol K ) = −118.8 J mol K

The value for ΔS o


298
is negative, as expected for this phase transition (condensation), which the previous section discussed.

 Exercise 16.3.2

Calculate the standard entropy change for the following process:

H (g) + C H (g) ⟶ C H (g)


2 2 4 2 6

Answer
−120.6 J mol−1 K−1

 Example 16.3.3: Determination of ΔS°

Calculate the standard entropy change for the combustion of methanol, CH3OH at room temperature:

2 CH OH(l) + 3 O (g) ⟶ 2 CO (g) + 4 H O(l)


3 2 2 2

Solution
The value of the standard entropy change is equal to the difference between the standard entropies of the products and the
entropies of the reactants scaled by their stoichiometric coefficients.
∘ ∘ ∘ ∘
ΔS = ΔS = ∑ νS (products) − ∑ ν S (reactants)
298 298 298

∘ ∘ ∘ ∘
= [2 S (CO (g)) + 4 S (H O(l))] − [2 S (CH OH(l)) + 3 S (O (g))]
298 2 298 2 298 3 298 2

= {[2(213.8) + 4 × 70.0] − [2(126.8) + 3(205.03)]} = −161.1 J/mol ⋅ K

Access for free at OpenStax 16.3.4 https://chem.libretexts.org/@go/page/38296


 Exercise 16.3.3

Calculate the standard entropy change for the following reaction:

Ca (OH) (s) ⟶ CaO(s) + H O(l)


2 2

Answer
24.7 J/mol•K

Summary
The second law of thermodynamics states that a spontaneous process increases the entropy of the universe, Suniv > 0. If ΔSuniv < 0,
the process is nonspontaneous, and if ΔSuniv = 0, the system is at equilibrium. The third law of thermodynamics establishes the zero
for entropy as that of a perfect, pure crystalline solid at 0 K. With only one possible microstate, the entropy is zero. We may
compute the standard entropy change for a process by using standard entropy values for the reactants and products involved in the
process.

16.3.3: Key Equations


∘ ∘ ∘ ∘
ΔS = ΔS = ∑ νS (products) − ∑ ν S (reactants)
298 298 298
qrev
ΔS =
T
ΔSuniv = ΔSsys + ΔSsurr
qsurr
ΔSuniv = ΔSsys + ΔSsurr = ΔSsys +
T

Glossary
second law of thermodynamics
entropy of the universe increases for a spontaneous process

standard entropy (S°)


entropy for a substance at 1 bar pressure; tabulated values are usually determined at 298.15 K and denoted S ∘
298

standard entropy change (ΔS°)


change in entropy for a reaction calculated using the standard entropies, usually at room temperature and denoted ΔS ∘
298

third law of thermodynamics


entropy of a perfect crystal at absolute zero (0 K) is zero

This page titled 16.3: The Second and Third Laws of Thermodynamics is shared under a CC BY 4.0 license and was authored, remixed, and/or
curated by OpenStax via source content that was edited to the style and standards of the LibreTexts platform; a detailed edit history is available
upon request.

Access for free at OpenStax 16.3.5 https://chem.libretexts.org/@go/page/38296


16.4: Gibbs Energy
 Learning Objectives
Define Gibbs free energy, and describe its relation to spontaneity
Calculate the standard free energy change for a process using standard free energies of formation for its reactants and
products
Calculate standard free energy change for a process using senthalpies of formation and the entropies for its reactants and
products
Explain how temperature affects the spontaneity of some processes
Relate standard free energy changes to equilibrium constants

One of the challenges of using the second law of thermodynamics to determine if a chemical reaction is spontaneous is that we
must determine the entropy change for the system and the entropy change for the surroundings. A second challenge when working
with a chemical reaction is that we need to take into account the mixing of the substances, an issue that does not occur when
observing the phase change if a pure substance. An alternative approach involving a new thermodynamic property defined in terms
of system properties only was introduced in the late nineteenth century by American mathematician Josiah Willard Gibbs. This new
property is called the Gibbs free energy (G) (or simply the free energy), and it is defined in terms of a system’s enthalpy and
entropy as the following:

G = H −TS

Free energy is a state function, and at constant temperature and pressure, the standard free energy change (ΔG°) may be expressed
as the following:
º º º
ΔGsys = ΔHsys − T ΔSsys

(For simplicity’s sake, the subscript “sys” will be omitted henceforth.) We can understand the relationship between this system
property and the spontaneity of a process by recalling the previously derived second law expression:
qsurr
ΔSuniv = ΔS +
T

The first law requires that qsurr = −qsys , and at constant pressure qsys = ΔH , and so this expression may be rewritten as the
following:
ΔH
ΔSuniv = ΔS −
T

ΔH is the enthalpy change of the system. Multiplying both sides of this equation by −T, and rearranging yields the following:

−T ΔSuniv = ΔH − T ΔS

Comparing this equation to the previous one for free energy change shows the following relation:
ΔG = −T ΔSuniv (16.4.1)

The free energy change is therefore a reliable indicator of the spontaneity of a process, being directly related to the previously
identified spontaneity indicator, ΔS . Table 16.4.1 summarizes the relation between the spontaneity of a process and the
univ

arithmetic signs of these indicators.


Table 16.4.1 : Relation between Process Spontaneity and Signs of Thermodynamic Properties
moves spontaneously in the forward direction,
ΔSuniv > 0 ΔG < 0
as written, to reach equilibrium
nonspontaneous in the forward direction, as
written,
ΔSuniv < 0 ΔG > 0
but moves spontaneously in the reverse
direction, as written, to reach equilibrium

ΔSuniv = 0 ΔG = 0 reversible (at equilibrium)

Access for free at OpenStax 16.4.1 https://chem.libretexts.org/@go/page/38297


16.4.1: Calculating Free Energy Change
Free energy is a state function, so its value depends only on the conditions of the initial and final states of the system that have
undergone some change. A convenient and common approach to the calculation of free energy changes for physical changes and
chemical reactions is by use of widely available compilations of standard state thermodynamic data. One method involves the use
of standard enthalpies and entropies to compute standard free energy changes according to the following relation as demonstrated
in Example 16.4.1.
ΔG° = ΔH ° − T ΔS° (16.4.2)

It is important to understand that for phase changes, ΔGº tells you if the phase change is spontaneous or not; will it happen, or not
happen. For chemical reactions, ΔGº tells you the extent of a reaction. In other words, ΔGº for a reaction tells you how much
product will be present at equilibrium. A reaction with ΔGº < 0 is considered product-favored at equilibrium; there will be more
products than reactants when the reaction reaches equilibrium. A reaction with ΔGº > 0 is considered reactant-favored at
equilibrium; there will be more reactants than products when the reaction reaches equilibrium.

 Example 16.4.1: Evaluation of ΔG° for a Phase Change of a Pure Substance

Use standard enthalpy and entropy data from Tables T1 or T2 to calculate the standard free energy change for the vaporization
of water at room temperature (298 K). What does the computed value for ΔG° say about the spontaneity of this physical
change for a pure substance?

Solution
The process of interest is the following:

H O(l) ⟶ H O(g) (16.4.3)


2 2

The standard change in free energy may be calculated using the following equation:

ΔG = ΔH ° − T ΔS° (16.4.4)
298

From Tables T1 or T2, here are the data:


Data from Tables T1 or T2
Substance ΔH

f
(kJ/mol) S

298
(J/K⋅mol)

H2O(l) −286.83 70.0

H2O(g) −241.82 188.8

Combining at 298 K:
∘ ∘ ∘
ΔH ° = ΔH = ΔH (H O(g)) − ΔH (H O(l))
298 f 2 f 2

= [−241.82 kJ − (−285.83)] kJ/mol

= 44.01 kJ/mol

∘ ∘ ∘
ΔS° = ΔS =S (H O(g)) − S (H O(l))
298 298 2 298 2

= 188.8 J/mol ⋅ K − 70.0 J/K

= 118.8 J/mol ⋅ K

then use Equation 16.4.2:

ΔG° = ΔH ° − T ΔS°

Converting everything into kJ and combining at 298 K:

Access for free at OpenStax 16.4.2 https://chem.libretexts.org/@go/page/38297



ΔG = ΔH ° − T ΔS°
298

1 kJ
= 44.01 kJ/mol − (298 K × 118.8 J/mol ⋅ K) ×
1000 J

44.01 kJ/mol − 35.4 kJ/mol = 8.6 kJ/mol

At 298 K (25 °C) ΔG ∘


298
>0 , and so boiling is nonspontaneous (not spontaneous) at 298 K.

 Exercise 16.4.1: Evaluation of ΔG° for a Chemical Reaction

Use standard enthalpy and entropy data from Tables T1 or T2 to calculate the standard free energy change for the reaction
shown here (298 K). What does the computed value for ΔG° say about the extent of this reaction at 298 K?

C H (g) ⟶ H (g) + C H (g)


2 6 2 2 4

Answer
ΔG

298
= 102.0 kJ/mol ; the reaction is reactant-favored at equilibrium at 25 °C. There will be more C 2
H (g)
6
than H
2
(g)

and C 2
H (g)
4
at equilibrium

Free energy changes may also use the standard free energy of formation (ΔG ), for each of the reactants and products involved in

f

the reaction. The standard free energy of formation is the free energy change that accompanies the formation of one mole of a
substance from its elements in their standard states. Similar to the standard enthalpies of formation, (ΔG ) is by definition zero for ∘
f

elemental substances under standard state conditions. The approach to computing the free energy change for a reaction using this
approach is the same as that demonstrated previously for enthalpy and entropy changes. For the reaction

mA + nB ⟶ xC + yD,

the standard free energy change at room temperature may be calculated as



ΔG = ΔG° (16.4.5)
298

∘ ∘
= ∑ νΔG (products) − ∑ νΔG (reactants) (16.4.6)
298 298

∘ ∘ ∘ ∘
= [xΔG (C) + yΔG (D)] − [mΔG (A) + nΔG (B)]. (16.4.7)
f f f f

 Example 16.4.2: Calculation of ΔG ∘


298

Consider the decomposition of yellow mercury(II) oxide.


1
HgO(s, yellow) ⟶ Hg(l) + O (g)
2 2

Calculate the standard free energy change at room temperature, ΔG ∘


298
, using:
a. standard free energies of formation and
b. standard enthalpies of formation and standard entropies.
Do the results indicate the reaction to be product-favored or reactant-favored at equilibrium?

Solution
The required data are available in Tables T1 or T2 and are shown here.
Required data for Tables T1 or T2
Compound ΔG

f
(kJ/mol) ΔH

f
(kJ/mol) S

298
(J/K⋅mol)

HgO (s, yellow) −58.43 −90.46 71.13

Hg(l) 0 0 75.9

O2(g) 0 0 205.2

Access for free at OpenStax 16.4.3 https://chem.libretexts.org/@go/page/38297


(a) Using free energies of formation:
∘ ∘ ∘
ΔG = ∑ νGS (products) − ∑ νΔG (reactants)
298 298 298

1
∘ ∘ ∘
= [1ΔG Hg(l) + ΔG O (g)] − 1ΔG HgO(s, yellow)
298 298 2 298
2

1
= [1 mol(0 kJ/mol) + mol(0 kJ/mol)] − 1 mol(−58.43 kJ/mol) = 58.43 kJ/mol
2

(b) Using enthalpies and entropies of formation:


∘ ∘ ∘
ΔH = ∑ νΔH (products) − ∑ νΔH (reactants)
298 298 298

1
∘ ∘ ∘
= [1ΔH Hg(l) + ΔH O (g)] − 1ΔH HgO(s, yellow)
298 298 2 298
2

1
= [1 mol(0 kJ/mol) + mol(0 kJ/mol)] − 1 mol(−90.46 kJ/mol) = 90.46 kJ/mol
2

∘ ∘ ∘
ΔS = ∑ νΔS (products) − ∑ νΔS (reactants)
298 298 298

1
∘ ∘ ∘
= [1ΔS Hg(l) + ΔS O (g)] − 1ΔS HgO(s, yellow)
298 298 2 298
2

1
= [1 mol(75.9 J/mol K) + mol(205.2 J/mol K)] − 1 mol(71.13 J/mol K) = 107.4 J/mol K
2

then we can use Equation \ref7} directly:


ΔG° = ΔH ° − T ΔS°

1 kJ
= 90.46 kJ − 298.15 K × 107.4 J/K ⋅ mol ×
1000 J

= (90.46 − 32.01) kJ/mol = 58.45 kJ/mol

Both ways to calculate the standard free energy change at 25 °C give the same numerical value (to three significant figures),
and both predict that the process is reactant-favored at equilibrium at room temperature.

 Exercise 16.4.2

Calculate ΔG° using (a) free energies of formation and (b) enthalpies of formation and entropies (Tables T1 or T2). Do the
results indicate the reaction to be product-favored or reactant-favored at equilibrium at 25 °C?

C H (g) ⟶ H (g) + C H (g)


2 4 2 2 2

Answer
141.5 kJ/mol, reactant-favored at equilibrium

16.4.2: Temperature Dependence of Spontaneity and Extent of Reaction


As was previously demonstrated in this chapter’s section on entropy, the spontaneity of a process may depend upon the temperature
of the system. Phase transitions, for example, will proceed spontaneously in one direction or the other depending upon the
temperature of the substance in question. In a similar, but not identical fashion, some chemical reactions can switch from being
product-favored at equilibrium, to being reactant-favored at equilibrium, depending on the temperature.

 Note

The numerical value of ΔGº is always dependent on the temperature. In this section we are determining whether or not the
sign of ΔGº is dependent on the temperature.

Access for free at OpenStax 16.4.4 https://chem.libretexts.org/@go/page/38297


To illustrate this concept, the equation relating free energy change to the enthalpy and entropy changes for the process is
considered:
º º º
ΔG = ΔH − T ΔS

The extent of a process, as reflected in the arithmetic sign of its standard free energy change, is then determined by the signs of the
enthalpy and entropy changes and, in some cases, the absolute temperature. Since T is the absolute (Kelvin) temperature, it can
only have positive values. Four possibilities therefore exist with regard to the signs of the enthalpy and entropy changes:
1. Both ΔHº and ΔSº are positive. This condition describes an endothermic process that involves an increase in system entropy.
In this case, ΔGº will be negative if the magnitude of the TΔSº term is greater than ΔHº. If the TΔSº term is less than ΔHº, the
free energy change will be positive. Such a process is product-favored at equilibrium at high temperatures and reactant-
favored at equilibrium at low temperatures.
2. Both ΔHº and ΔSº are negative. This condition describes an exothermic process that involves a decrease in system entropy. In
this case, ΔGº will be negative if the magnitude of the TΔSº term is less than ΔHº. If the TΔSº term’s magnitude is greater than
ΔHº, the free energy change will be positive. Such a process is product-favored at equilibrium at low temperatures
and reactant-favored at equilibrium at high temperatures.
3. ΔHº is positive and ΔSº is negative. This condition describes an endothermic process that involves a decrease in system
entropy. In this case, ΔGº will be positive regardless of the temperature. Such a process is reactant-favored at equilibrium at all
temperatures.
4. ΔHº is negative and ΔSº is positive. This condition describes an exothermic process that involves an increase in system
entropy. In this case, ΔGº will be negative regardless of the temperature. Such a process is product-favored at equilibrium at all
temperatures.
These four scenarios are summarized in Table 16.4.1
Table 16.4.1
Sign of ΔH o
Sign of ΔS o
Sign of ΔG o
Temperature Dependence of ΔG o

The sign of ΔG does not depend


o

on the temperature.The reaction is


- + -
product-favored at equilibrium at
all temperatures.
The sign of ΔG does not depend
o

on the temperature.The reaction is


+ - +
reactant-favored at equilibrium at
all temperatures.
The sign of ΔG does depend on
o

the temperature. The reaction will


- - - or +
be product-favored at equilibrium
at lower temperatures.
The sign of ΔG does depend on
o

the temperature. The reaction will


+ + - or +
be product-favored at equilibrium
at lower temperatures.

 Example 16.4.3: Predicting the Temperature Dependence of Spontaneity

The incomplete combustion of carbon is described by the following equation:

2 C(s) + O (g) ⟶ 2 CO(g)


2

º
Does the sign of ΔG of this process depend upon temperature?

Solution

Access for free at OpenStax 16.4.5 https://chem.libretexts.org/@go/page/38297


Combustion processes are exothermic (ΔH º < 0 ). This particular reaction involves an increase in entropy due to the
accompanying increase in the amount of gaseous species (net gain of one mole of gas, ΔS º > 0 ). The reaction is therefore
product-favored at equilibrium (ΔGº < 0 ) at all temperatures.

 Exercise 16.4.3

Popular chemical hand warmers generate heat by the air-oxidation of iron:

4 Fe(s) + 3 O (g) ⟶ 2 Fe O (s)


2 2 3

Does the sign of ΔG of this process depend upon temperature?


o

Answer
ΔHº and ΔSº are both negative; the reaction is product-favored at equilibrium at low temperatures.

When considering the conclusions drawn regarding the temperature dependence of the sign of ΔGº, it is important to keep in mind
what the terms “high” and “low” mean. Since these terms are adjectives, the temperatures in question are deemed high or low
relative to some reference temperature. A process that is reactant-favored at equilibrium at one temperature but product-favored at
equilibrium at another temperature will necessarily undergo a change in “extent” (as reflected by its ΔGº) as temperature varies.
This is clearly illustrated by a graphical presentation of the free energy change equation, in which ΔGº is plotted on the y axis
versus T on the x axis:
º º º
ΔG = ΔH − T ΔS

y = b + mx

Such a plot is shown in Figure 16.4.2. A process whose enthalpy and entropy changes are of the same arithmetic sign will exhibit a
temperature-dependence for the sign of ΔGº as depicted by the two yellow lines in the plot. Each line crosses from one spontaneity
domain (positive or negative ΔGº) to the other at a temperature that is characteristic of the process in question. This temperature is
represented by the x-intercept of the line, that is, the value of T for which ΔGº is zero:
º º º
ΔG = 0 = ΔH − T ΔS

º
ΔH
T =
º
ΔS

Thus, saying a process is product-favored at equilibrium at “high” or “low” temperatures means the temperature is above or below,
respectively, that temperature at which ΔGº for the process is zero.

 Note
In this discussion, we have used two different descriptions for the meaning of the sign of ΔGº. You should be aware of the
meaning of each description.
a) Extent of Reaction: This description is used to predict the ratio of the product and reactant concentrations at equilibrium. In
this description, we use the thermodynamic term ΔGº to tell us the same information as the equilibrium constant, K.
When ΔGº < 0, K > 1, and the reaction will be product-favored at equilibrium. When ΔGº > 0, K< 1, and the reaction is
reactant-favored at equilibrium. When ΔGº = 0, K =1, and the reaction will have roughly equal amounts of products and
reactants at equilibrium. In all cases, the reaction will form a mixture of products and reactants at equilibrium. We use the sign
and magnitude of ΔGº to tell us how much product will be made if the reaction is allowed to reach equilibrium.
b) Spontaneity: This description is much more complicated because it involves two different interpretations of how a reaction
at standard state occurs. One interpretation involves the hypothetical process in which the reaction proceeds from a starting
point of pure reactants to a finishing point of pure products, with all substances isolated in their own containers under standard
state conditions. In the second, more realistic interpretation, the reaction starts with all reactants and all products in their
standard state in one container. We then allow this specific mixture to react an infinitesimally small amount so that we can
obtain a rate of change in free energy with respect to the extent of reaction when all reactants and products are mixed and

Access for free at OpenStax 16.4.6 https://chem.libretexts.org/@go/page/38297


(essentially) in their standard states. Although each interpretation describes a different reaction scenario, the value of the
difference in free energy and the value of the rate of change in free energy are the same number. If ΔGº < 0, we say that the
reaction is spontaneous, meaning that the reaction would proceed in the forward direction, as written, to form pure products in
their standard state. If ΔGº > 0, we say that the reaction is nonspontaneous, meaning that the reaction would proceed in the
reverse direction, as written, to form pure reactants in their standard state. If ΔGº = 0, we say that the neither the reactants nor
the products are favored to be formed.
A detailed treatment of the meaning of ΔGº can be found in the paper, "Free Energy versus Extent of Reaction" by Richard S.
Treptow, Journal of Chemical Education, 1996, Volume 73 (1), 51-54.

Figure 16.4.2 : These plots show the variation in ΔGº with temperature for the four possible combinations of arithmetic sign for
ΔHº and ΔSº. Note that in this graph of ΔGº, "spontaneous" is synonymous with "product-favored at equilibrium", and
"nonspontaneous" is synonymous with "reactant-favored at equilibrium".

 Example 16.4.4: Equilibrium Temperature for a Phase Transition

As defined in the chapter on liquids and solids, the boiling point of a liquid is the temperature at which its solid and liquid
phases are in equilibrium (that is, when vaporization and condensation occur at equal rates). Use the information in Tables T1
or T2 to estimate the boiling point of water.

Solution
The process of interest is the following phase change:
H O(l) ⟶ H O(g)
2 2

When this process is at equilibrium, ΔG = 0, so the following is true:


ΔH °
0 = ΔH ° − T ΔS° or T =
ΔS°

Using the standard thermodynamic data from Tables T1 or T2,

Access for free at OpenStax 16.4.7 https://chem.libretexts.org/@go/page/38297


∘ ∘
ΔH ° = ΔH (H O(g)) − ΔH (H O(l))
f 2 f 2

= −241.82 kJ/mol − (−285.83 kJ/mol) = 44.01 kJ/mol

∘ ∘
ΔS° = ΔS (H O(g)) − ΔS (H O(l))
298 2 298 2

= 188.8 J/K ⋅ mol − 70.0 J/K ⋅ mol = 118.8 J/K ⋅ mol

3
ΔH ° 44.01 × 10 J/mol
T = = = 370.5 K = 97.3 °C
ΔS° 118.8 J/K ⋅ mol

The accepted value for water’s normal boiling point is 373.2 K (100.0 °C), and so this calculation is in reasonable agreement.
Note that the values for enthalpy and entropy changes data used were derived from standard data at 298 K (Tables T1 or T2.).
If desired, you could obtain more accurate results by using enthalpy and entropy changes determined at (or at least closer to)
the actual boiling point.

 Exercise 16.4.4

Use the information in Tables T1 or T2 to estimate the boiling point of CS2.

Answer
313 K (accepted value 319 K).

16.4.3: Free Energy and Equilibrium


The free energy change for a process may be viewed as a measure of its driving force. A negative value for ΔG represents a finite
driving force for the process in the forward direction, while a positive value represents a driving force for the process in the reverse
direction. When ΔG is zero, the forward and reverse driving forces are equal, and so the process occurs in both directions at the
same rate (the system is at equilibrium).
In the chapter on equilibrium the reaction quotient, Q, was introduced as a convenient measure of the status of an equilibrium
system. Recall that Q is the numerical value of the mass action expression for the system, and that you may use its value to identify
the direction in which a reaction will proceed in order to achieve equilibrium. When Q is lesser than the equilibrium constant, K,
the reaction will proceed in the forward direction until equilibrium is reached and Q = K. Conversely, if Q > K, the process will
proceed in the reverse direction until equilibrium is achieved.
The free energy change for a process taking place with reactants and products present under nonstandard conditions, ΔG, is related
to the standard free energy change, ΔG°, according to this equation:
ΔG = ΔG° + RT ln Q (16.4.8)

R is the gas constant (8.314 J/K mol), T is the kelvin or absolute temperature, and Q is the reaction quotient. We may use this
equation to predict the spontaneity for a process under any given set of conditions as illustrated in Example 16.4.1.

 Example 16.4.5: Calculating ΔG under Nonstandard Conditions

What is the free energy change for the process shown here under the specified conditions?
T = 25 °C, P N
2
,
= 0.870 atm PH
2
= 0.250 atm , and P NH
3
= 12.9 atm

2 NH (g) ⟶ 3 H (g) + N (g) ΔG° = 33.0 kJ/mol


3 2 2

Solution
Equation 16.4.8 relates free energy change to standard free energy change and reaction quotient and may be used directly:

Access for free at OpenStax 16.4.8 https://chem.libretexts.org/@go/page/38297


ΔG = ΔG° + RT ln Q

3
kJ J (0.250 ) × 0.870
= 33.0 + (8.314 × 298 K × ln )
2
mol mol K 12.9

J
= 9680 or 9.68 kJ/mol
mol

Since the computed value for ΔG is positive, the reaction is nonspontaneous under these conditions. The reaction will proceed
in the reverse direction to reach equilibrium.

 Exercise 16.4.5

Calculate the free energy change for this same reaction at 875 °C in a 5.00 L mixture containing 0.100 mol of each gas. Is the
reaction spontaneous under these conditions?

Answer
ΔG = −47 kJ; yes, the reaction proceeds in the forward direction, as written, to reach equilibrium.

For a system at equilibrium, Q = K and ΔG = 0, and the Equation 16.4.8 may be written as

0 = ΔG° + RT ln K

at equilibrium

ΔG° = −RT ln K (16.4.9)

or alternatively
Δ G°

K =e RT (16.4.10)

This form of the equation provides a useful link between these two essential thermodynamic properties, and it can be used to derive
equilibrium constants from standard free energy changes and vice versa. The relations between standard free energy changes and
equilibrium constants are summarized in Table 16.4.1.
Table 16.4.1 : Relations between Standard Free Energy Changes and Equilibrium Constants
K ΔG° Comments

<1 >0 Reactants are more abundant at equilibrium.

Reactants and products are equally abundant at


=1 =0
equilibrium.

>1 <0 Products are more abundant at equilibrium.

 Example 16.4.6: Equilibrium Constant using Standard Free Energy Change

Given that the standard free energies of formation of Ag+(aq), Cl−(aq), and AgCl(s) are 77.1 kJ/mol, −131.2 kJ/mol, and
−109.8 kJ/mol, respectively, calculate the solubility product, Ksp, for AgCl.

Solution
The reaction of interest is the following:
+ − + −
AgCl(s) ⇌ Ag (aq) + Cl (aq) Ksp = [ Ag ][ Cl ]

The standard free energy change for this reaction is first computed using standard free energies of formation for its reactants
and products:

Access for free at OpenStax 16.4.9 https://chem.libretexts.org/@go/page/38297


∘ ∘ + ∘ − ∘
ΔG° = ΔG = [ΔG (Ag (aq)) + ΔG (Cl (aq))] − [ΔG (AgCl(s))]
298 f f f

= [77.1 kJ/mol − 131.2 kJ/mol] − [−109.8 kJ/mol]

= 55.7 kJ/mol

The equilibrium constant for the reaction may then be derived from its standard free energy change via Equation 16.4.10:
ΔG°
− ΔG°
Ksp = e RT = exp(− )
RT

3
55.7 × 10 J/mol
= exp(− )
8.314 J/mol ⋅ K × 298.15 K

−22.470 −10
= exp(−22.470) = e = 1.74 × 10

 Exercise 16.4.6: dissociation of dinitrogen tetroxide

Use the thermodynamic data provided in Tables T1 or T2 to calculate the equilibrium constant for the dissociation of
dinitrogen tetroxide at 25 °C.

NO2(g) ⇌ N O4(g)
2

Answer
K = 6.9

To further illustrate the relation between these two essential thermodynamic concepts, consider the observation that reactions
spontaneously proceed in a direction that ultimately establishes equilibrium. As may be shown by plotting the free energy change
versus the extent of the reaction (for example, as reflected in the value of Q), equilibrium is established when the system’s free
energy is minimized (Figure 16.4.3). If a system is present with reactants and products present in nonequilibrium amounts (Q ≠ K),
the reaction will proceed spontaneously in the direction necessary to establish equilibrium.

Figure 16.4.3 : These plots show the free energy versus reaction progress for systems whose standard free changes are (a) negative,
(b) positive, and (c) zero. Nonequilibrium systems will proceed spontaneously in whatever direction is necessary to minimize free
energy and establish equilibrium. Note that reactions with K equal to exactly 1.000 are rare.

Summary
Gibbs free energy (G) is a state function defined with regard to system quantities only and may be used to predict the spontaneity
of a process. A negative value for ΔG indicates that the process will proceed in the forward direction to reach equilibrium; a
positive ΔG indicates that the process will proceed in the reverse direction to reach equilibrium ; and a ΔG of zero indicates that the
system is at equilibrium. A negative value for ΔGº means that the reaction is product-favored at equilibrium. A positive value
for ΔGº means that the reaction is reactant-favored at equilibrium. If ΔGº equals 0 (a rare occurrence), the reaction has roughly

Access for free at OpenStax 16.4.10 https://chem.libretexts.org/@go/page/38297


equal amounts of reactants and products at equilibrium.A number of approaches to the computation of free energy changes are
possible.

16.4.4: Key Equations


ΔG = ΔH − TΔS
ΔG = ΔG° + RT ln Q
ΔG° = −RT ln K

Glossary
Gibbs free energy change (G)
thermodynamic property defined in terms of system enthalpy and entropy; all spontaneous processes involve a decrease in G

standard free energy change (ΔG°)


change in free energy for a process occurring under standard conditions (1 bar pressure for gases, 1 M concentration for
solutions)
standard free energy of formation (ΔG ) ∘
f

change in free energy accompanying the formation of one mole of substance from its elements in their standard states

This page titled 16.4: Gibbs Energy is shared under a CC BY 4.0 license and was authored, remixed, and/or curated by OpenStax via source
content that was edited to the style and standards of the LibreTexts platform; a detailed edit history is available upon request.

Access for free at OpenStax 16.4.11 https://chem.libretexts.org/@go/page/38297


16.E: Thermodynamics (Exercises)
16.E.1: 16.1: Spontaneity Exercises
16.E.1.1: Q16.1.1
What is a spontaneous reaction?

16.E.1.2: S16.1.1
A reaction has a natural tendency to occur and takes place without the continual input of energy from an external source.

16.E.1.3: Q16.1.2
What is a nonspontaneous reaction?

16.E.1.4: Q16.1.3
Indicate whether the following processes are spontaneous or nonspontaneous.
a. Liquid water freezing at a temperature below its freezing point
b. Liquid water freezing at a temperature above its freezing point
c. The combustion of gasoline
d. A ball thrown into the air
e. A raindrop falling to the ground
f. Iron rusting in a moist atmosphere

16.E.1.5: S16.1.2
spontaneous; nonspontaneous; spontaneous; nonspontaneous; spontaneous; spontaneous

16.E.1.6: Q16.1.4
A helium-filled balloon spontaneously deflates overnight as He atoms diffuse through the wall of the balloon. Describe the redistribution of matter and/or energy that accompanies
this process.

16.E.1.7: Q16.1.5
Many plastic materials are organic polymers that contain carbon and hydrogen. The oxidation of these plastics in air to form carbon dioxide and water is a spontaneous process;
however, plastic materials tend to persist in the environment. Explain.

16.E.1.8: S16.1.5
Although the oxidation of plastics is spontaneous, the rate of oxidation is very slow. Plastics are therefore kinetically stable and do not decompose appreciably even over relatively
long periods of time.

16.E.2: 16.2: Entropy Exercises


16.E.2.1: Q16.2.1
In the below Figure all possible distributions and microstates are shown for four different particles shared between two boxes. Determine the entropy change, ΔS, if the particles are
initially evenly distributed between the two boxes, but upon redistribution all end up in Box (b).

16.E.2.2: Q16.2.2
In Figure all of the possible distributions and microstates are shown for four different particles shared between two boxes. Determine the entropy change, ΔS, for the system when
it is converted from distribution to distribution (d).

Access for free at OpenStax 16.E.1 https://chem.libretexts.org/@go/page/42801


16.E.2.3: S16.2.2
There are four initial microstates and four final microstates.
Wf 4
−23
ΔS = k ln = 1.38 × 10 J/K × ln =0 (16.E.1)
Wi 4

16.E.2.4: Q16.2.3
How does the process described in the previous item relate to the system shown in [link]?

16.E.2.5: Q16.2.4
Consider a system similar to the one below, except that it contains six particles instead of four. What is the probability of having all the particles in only one of the two boxes in the
1
case? Compare this with the similar probability for the system of four particles that we have derived to be equal to . What does this comparison tell us about even larger systems?
8

16.E.2.6: S16.2.4
1 1
The probability for all the particles to be on one side is . This probability is noticeably lower than the result for the four-particle system. The conclusion we can make is that
32 8
the probability for all the particles to stay in only one part of the system will decrease rapidly as the number of particles increases, and, for instance, the probability for all
molecules of gas to gather in only one side of a room at room temperature and pressure is negligible since the number of gas molecules in the room is very large.

16.E.2.7: Q16.2.5
Consider the system shown in Figure. What is the change in entropy for the process where the energy is initially associated only with particle A, but in the final state the energy is
distributed between two different particles?

16.E.2.8: Q16.2.6
Consider the system shown in Figure. What is the change in entropy for the process where the energy is initially associated with particles A and B, and the energy is distributed
between two particles in different boxes (one in A-B, the other in C-D)?

16.E.2.9: S16.2.6
There is only one initial state. For the final state, the energy can be contained in pairs A-C, A-D, B-C, or B-D. Thus, there are four final possible states.
Wf 4
−23 −23
ΔS = k ln( ) = 1.38 × 10 J/K × ln( ) = 1.91 × 10 J/K (16.E.2)
Wi 1

16.E.2.10: Q16.2.7
Arrange the following sets of systems in order of increasing entropy. Assume one mole of each substance and the same temperature for each member of a set.
a. H2(g), HBrO4(g), HBr(g)
b. H2O(l), H2O(g), H2O(s)
c. He(g), Cl2(g), P4(g)

16.E.2.11: Q16.2.8
At room temperature, the entropy of the halogens increases from I2 to Br2 to Cl2. Explain.

16.E.2.12: S16.2.8
The masses of these molecules would suggest the opposite trend in their entropies. The observed trend is a result of the more significant variation of entropy with a physical state.
At room temperature, I2 is a solid, Br2 is a liquid, and Cl2 is a gas.

16.E.2.13: Q16.2.9
Consider two processes: sublimation of I2(s) and melting of I2(s) (Note: the latter process can occur at the same temperature but somewhat higher pressure).
I (s) ⟶ I (g) (16.E.3)
2 2

I (s) ⟶ I (l) (16.E.4)


2 2

Is ΔS positive or negative in these processes? In which of the processes will the magnitude of the entropy change be greater?

16.E.2.14: Q16.2.11
Indicate which substance in the given pairs has the higher entropy value. Explain your choices.
a. C2H5OH(l) or C3H7OH(l)
b. C2H5OH(l) or C2H5OH(g)
c. 2Hor H(g)

Access for free at OpenStax 16.E.2 https://chem.libretexts.org/@go/page/42801


16.E.2.15: S16.2.11
C3H7OH(l) as it is a larger molecule (more complex and more massive), and so more microstates describing its motions are available at any given temperature. C2H5OH(g) as it is
in the gaseous state. 2H(g), since entropy is an extensive property, and so two H atoms (or two moles of H atoms) possess twice as much entropy as one atom (or one mole of
atoms).

16.E.2.16: Q16.2.11
Predict the sign of the entropy change for the following processes:
a. An ice cube is warmed to near its melting point.
b. Exhaled breath forms fog on a cold morning.
c. Snow melts.

16.E.2.17: Q16.2.12
Predict the sign of the enthalpy change for the following processes. Give a reason for your prediction.
a. Pb (aq) + S (aq) ⟶ PbS(s)
2+ 2−

b. 2 Fe(s) + 3 O (g) ⟶ Fe O (s) 2 2 3

c. 2 C H (l) + 19 O (g) ⟶ 14 H O(g) + 12 CO


6 14 2 2 2
(g)

16.E.2.18: S16.2.12
Negative. The relatively ordered solid precipitating decreases the number of mobile ions in solution. Negative. There is a net loss of three moles of gas from reactants to products.
Positive. There is a net increase of seven moles of gas from reactants to products.

16.E.2.19: Q16.2.13
Write the balanced chemical equation for the combustion of methane, CH4(g), to give carbon dioxide and water vapor. Explain why it is difficult to predict whether ΔS is positive
or negative for this chemical reaction.

16.E.2.20: Q16.2.14
Write the balanced chemical equation for the combustion of benzene, C6H6(l), to give carbon dioxide and water vapor. Would you expect ΔS to be positive or negative in this
process?

16.E.2.21: S16.2.14
C H (l) + 7.5 O (g) ⟶ 3 H O(g) + 6 CO (g) (16.E.5)
6 6 2 2 2

There are 7.5 moles of gas initially, and 3 + 6 = 9 moles of gas in the end. Therefore, it is likely that the entropy increases as a result of this reaction, and ΔS is positive.

16.E.3: 16.3: The Second and Third law


16.E.3.1: Q16.3.0
What is the difference between ΔS, ΔS°, and ΔS ∘
298
for a chemical change?

16.E.3.2: Q16.3.1
Calculate ΔS ∘
298
for the following changes.
a. SnCl (l) ⟶ SnCl (g)
4 4

b. CS (g) ⟶ CS (l)
2 2

c. Cu(s) ⟶ Cu(g)
d. H O(l) ⟶ H O(g)
2 2

e. 2 H (g) + O (g) ⟶ 2 H O(l)


2 2 2

f. 2 HCl(g) + Pb(s) ⟶ PbCl (s) + H (g) 2 2

g. Zn(s) + CuSO (s) ⟶ Cu(s) + ZnSO (s)


4 4

16.E.3.3: S16.3.1
107 J/K; −86.4 J/K; 133.2 J/K; 118.8 J/K; −326.6 J/K; −171.9 J/K; (g) −7.2 J/K

16.E.3.4: Q16.3.2
Determine the entropy change for the combustion of liquid ethanol, C2H5OH, under standard state conditions to give gaseous carbon dioxide and liquid water.

16.E.3.5: Q16.3.3
Determine the entropy change for the combustion of gaseous propane, C3H8, under standard state conditions to give gaseous carbon dioxide and water.

16.E.3.6: S16.3.3
100.6 J/K

16.E.3.7: Q16.3.4
“Thermite” reactions have been used for welding metal parts such as railway rails and in metal refining. One such thermite reaction is:
Fe O (s) + 2 Al(s) ⟶ Al O (s) + 2 Fe(s) (16.E.6)
2 3 2 3

Is the reaction spontaneous at room temperature under standard conditions? During the reaction, the surroundings absorb 851.8 kJ/mol of heat.

16.E.3.8: Q16.3.5
Using the relevant S ∘
298
values listed in Appendix G, calculate S ∘
298
for the following changes:
a. N (g) + 3 H (g) ⟶ 2 NH (g)
2 2 3

5
b. N 2
(g) + O (g) ⟶ N O (g)
2 2 5
2

Access for free at OpenStax 16.E.3 https://chem.libretexts.org/@go/page/42801


16.E.3.9: S16.3.5
−198.1 J/K; −348.9 J/K

16.E.3.10: Q16.3.6
From the following information, determine ΔS ∘
298
for the following:
a. N(g) + O(g) ⟶ NO(g) ΔS = ? ∘
298

b. N (g) + O (g) ⟶ 2 NO(g) ΔS = 24.8 J/K


2 2

298

c. N (g) ⟶ 2 N(g) ΔS = 115.0 J/K


2 298

d. O (g) ⟶ 2 O(g) ΔS = 117.0 J/K


2

298

16.E.3.11: Q16.3.7
By calculating ΔSuniv at each temperature, determine if the melting of 1 mole of NaCl(s) is spontaneous at 500 °C and at 700 °C.

S^\circ_{\ce{NaCl}(s)}=\mathrm{72.11\:\dfrac{J}{mol⋅K}}\hspace{40px} S^\circ_{\ce{NaCl}(l)}=\mathrm{95.06\:\dfrac{J}{mol⋅K}}\hspace{40px ΔH^\circ_\ce{fusion}=\mathrm{27.95\: kJ/mol}

What assumptions are made about the thermodynamic information (entropy and enthalpy values) used to solve this problem?

16.E.3.12: S16.3.7
As ΔSuniv < 0 at each of these temperatures, melting is not spontaneous at either of them. The given values for entropy and enthalpy are for NaCl at 298 K. It is assumed that these
do not change significantly at the higher temperatures used in the problem.

16.E.3.13: Q16.3.8
Use the standard entropy data in Appendix G to determine the change in entropy for each of the reactions listed in [link]. All are run under standard state conditions and 25 °C.

16.E.3.14: Q16.3.8
2.86 J/K; 24.8 J/K; −113.2 J/K; −24.7 J/K; 15.5 J/K; 290.0 J/K

16.E.4: 16.4: Free Energy


16.E.4.1: Q16.4.1
What is the difference between ΔG, ΔG°, and ΔG ∘
298
for a chemical change?

16.E.4.2: Q16.4.2
A reactions has ΔH

298
= 100 kJ/mol and ΔS
298

= 250 J/mol⋅K . Is the reaction spontaneous at room temperature? If not, under what temperature conditions will it become
spontaneous?

16.E.4.3: S16.4.2
The reaction is nonspontaneous at room temperature. Above 400 K, ΔG will become negative, and the reaction will become spontaneous.

16.E.4.4: Q16.4.3
Explain what happens as a reaction starts with ΔG < 0 (negative) and reaches the point where ΔG = 0.
Use the standard free energy of formation data in Appendix G to determine the free energy change for each of the following reactions, which are run under standard state
conditions and 25 °C. Identify each as either spontaneous or nonspontaneous at these conditions.
a. MnO (s) ⟶ Mn(s) + O (g)
2 2

b. H (g) + Br (l) ⟶ 2 HBr(g)


2 2

c. Cu(s) + S(g) ⟶ CuS(s)


d. 2 LiOH(s) + CO (g) ⟶ Li CO (s) + H O(g)
2 2 3 2

e. CH (g) + O (g) ⟶ C(s, graphite) + 2 H O(g)


4 2 2

f. CS (g) + 3 Cl (g) ⟶ CCl (g) + S Cl (g)


2 2 4 2 2

16.E.4.5: S16.4.3
465.1 kJ nonspontaneous; −106.86 kJ spontaneous; −53.6 kJ spontaneous; −83.4 kJ spontaneous; −406.7 kJ spontaneous; −30.0 kJ spontaneous

16.E.4.6: Q16.4.4
Use the standard free energy data in Appendix G to determine the free energy change for each of the following reactions, which are run under standard state conditions and 25 °C.
Identify each as either spontaneous or nonspontaneous at these conditions.
a. C(s, graphite) + O (g) ⟶ CO (g) 2 2

b. O (g) + N (g) ⟶ 2 NO(g)


2 2

c. 2 Cu(s) + S(g) ⟶ Cu S(s) 2

d. CaO(s) + H O(l) ⟶ Ca(OH) (s)


2 2

e. Fe O (s) + 3 CO(g) ⟶ 2 Fe(s) + 3 CO (g)


2 3 2

f. CaSO ⋅ 2 H O(s) ⟶ CaSO (s) + 2 H O(g)


4 2 4 2

Given:

P (s) + 5 O (g) ⟶ P O (s) ΔG = −2697.0 kJ/mol (16.E.7)
4 2 4 10 298


2 H (g) + O (g) ⟶ 2 H O(g) ΔG = −457.18 kJ/mol (16.E.8)
2 2 2 298


6 H O(g) + P O (g) ⟶ 4 H PO (l) ΔG = −428.66 kJ/mol (16.E.9)
2 4 10 3 4 298

16.E.4.7: Q16.4.5
a. Determine the standard free energy of formation, ΔG , for phosphoric acid. ∘
f

b. How does your calculated result compare to the value in Appendix G? Explain.

Access for free at OpenStax 16.E.4 https://chem.libretexts.org/@go/page/42801


16.E.4.8: S16.4.5
−1124.3 kJ/mol for the standard free energy change. The calculation agrees with the value in Appendix G because free energy is a state function (just like the enthalpy and
entropy), so its change depends only on the initial and final states, not the path between them.

16.E.4.9: Q16.4.6
Is the formation of ozone (O3(g)) from oxygen (O2(g)) spontaneous at room temperature under standard state conditions?

16.E.4.10: Q16.4.7
Consider the decomposition of red mercury(II) oxide under standard state conditions.
2 HgO(s, red) ⟶ 2 Hg(l) + O (g) (16.E.10)
2

a. Is the decomposition spontaneous under standard state conditions?


b. Above what temperature does the reaction become spontaneous?

16.E.4.11: S16.4.7
The reaction is nonspontaneous; Above 566 °C the process is spontaneous.

16.E.4.12: Q16.4.8
Among other things, an ideal fuel for the control thrusters of a space vehicle should decompose in a spontaneous exothermic reaction when exposed to the appropriate catalyst.
Evaluate the following substances under standard state conditions as suitable candidates for fuels.
a. Ammonia: 2 NH (g) ⟶ N (g) + 3 H (g) 3 2 2

b. Diborane: B H (g) ⟶ 2 B(g) + 3 H (g)


2 6 2

c. Hydrazine: N H (g) ⟶ N (g) + 2 H (g)2 4 2 2

1
d. Hydrogen peroxide: H 2
O (l) ⟶ H O(g) +
2 2
O (g)
2
2

16.E.4.13: Q16.4.9
Calculate ΔG° for each of the following reactions from the equilibrium constant at the temperature given.
a. N (g) + O (g) ⟶ 2 NO(g) T = 2000 °C K = 4.1 × 10
2 2 p
−4

b. H (g) + I (g) ⟶ 2 HI(g) T = 400 °C K = 50.0


2 2 p

c. CO (g) + H (g) ⟶ CO(g) + H O(g) T = 980 °C K = 1.67


2 2 2 p

d. CaCO (s) ⟶ CaO(s) + CO (g) T = 900 °C K = 1.04


3 2 p

e. HF(aq) + H O(l) ⟶ H O (aq) + F (aq) T = 25 °C K = 7.2 × 10


2 3
+ −
p
−4

f. AgBr(s) ⟶ Ag (aq) + Br (aq) T = 25 °C K = 3.3 × 10


+ −
p
−13

16.E.4.14: S16.4.9
1.5 × 102 kJ; −21.9 kJ; −5.34 kJ; −0.383 kJ; 18 kJ; 71 kJ

16.E.4.15: Q16.4.10
Calculate ΔG° for each of the following reactions from the equilibrium constant at the temperature given.
a. Cl (g) + Br (g) ⟶ 2 BrCl(g) T = 25 °C K = 4.7 × 10
2 2 p
−2

b. 2 SO (g) + O (g) ⇌ 2 SO (g) T = 500 °C K = 48.2


2 2 3 p

c. H O(l) ⇌ H O(g) T = 60 °C K = 0.196 atm


2 2 p

d. CoO(s) + CO(g) ⇌ Co(s) + CO (g) T = 550 °C K = 4.90 × 10 2 p


2

e. CH NH (aq) + H O(l) ⟶ CH NH (aq) + OH (aq) T = 25 °C


3 2 2 3
+
3
− −4
Kp = 4.4 × 10

f. PbI (s) ⟶ Pb (aq) + 2 I (aq) T = 25 °C K = 8.7 × 10


2
2+ −
p
−9

16.E.4.16: Q16.4.11
Calculate the equilibrium constant at 25 °C for each of the following reactions from the value of ΔG° given.
a. O (g) + 2 F (g) ⟶ 2 OF (g) ΔG° = −9.2 kJ
2 2 2

b. I (s) + Br (l) ⟶ 2 IBr(g) ΔG° = 7.3 kJ


2 2

c. 2 LiOH(s) + CO (g) ⟶ Li CO (s) + H O(g) ΔG° = −79 kJ


2 2 3 2

d. N O (g) ⟶ NO(g) + NO (g) ΔG° = −1.6 kJ


2 3 2

e. SnCl (l) ⟶ SnCl (l) ΔG° = 8.0 kJ


4 4

16.E.4.17: S16.4.11
K = 41; K = 0.053; K = 6.9 × 1013; K = 1.9; K = 0.04

16.E.4.18: Q16.4.2
Calculate the equilibrium constant at 25 °C for each of the following reactions from the value of ΔG° given.
a. I (s) + Cl (g) ⟶ 2 ICl(g) ΔG° = −10.88 kJ
2 2

b. H (g) + I (s) ⟶ 2 HI(g) ΔG° = 3.4 kJ


2 2

c. CS (g) + 3 Cl (g) ⟶ CCl (g) + S Cl (g) ΔG° = −39 kJ


2 2 4 2 2

d. 2 SO (g) + O (g) ⟶ 2 SO (g) ΔG° = −141.82 kJ


2 2 3

e. CS (g) ⟶ CS (l) ΔG° = −1.88 kJ


2 2

16.E.4.19: Q16.4.13
Calculate the equilibrium constant at the temperature given.
a. (a) O (g) + 2 F (g) ⟶ 2 F
2 2 2
O(g) (T = 100 °C)

b. I (s) + Br (l) ⟶ 2 IBr(g)


2 2
(T = 0.0 °C)

Access for free at OpenStax 16.E.5 https://chem.libretexts.org/@go/page/42801


c. 2 LiOH(s) + CO (g) ⟶ Li CO (s) + H O(g) (T = 575 °C)
2 2 3 2

d. N O (g) ⟶ NO(g) + NO (g) (T = −10.0 °C)


2 3 2

e. SnCl (l) ⟶ SnCl (g) (T = 200 °C)


4 4

16.E.4.20: S16.4.13
In each of the following, the value of ΔG is not given at the temperature of the reaction. Therefore, we must calculate ΔG° from the values ΔH° and ΔS° and then calculate ΔG
from the relation ΔG° = ΔH° − TΔS°.
a. K = 1.29
b. K = 2.51 × 10−3
c. K = 4.83 × 103
d. K = 0.219
e. K = 16.1

16.E.4.21: Q16.4.14
Calculate the equilibrium constant at the temperature given.
a. (a) I (s) + Cl (g) ⟶ 2 ICl(g) (T = 100 °C)
2 2

b. H (g) + I (s) ⟶ 2 HI(g) (T = 0.0 °C)


2 2

c. CS (g) + 3 Cl (g) ⟶ CCl (g) + S Cl (g) (T = 125 °C)


2 2 4 2 2

d. 2 SO (g) + O (g) ⟶ 2 SO (g) (T = 675 °C)


2 2 3

e. CS (g) ⟶ CS (l) (T = 90 °C)


2 2

16.E.4.22: Q16.4.15
Consider the following reaction at 298 K:
N O (g) ⇌ 2 NO (g) KP = 0.142 (16.E.11)
2 4 2

What is the standard free energy change at this temperature? Describe what happens to the initial system, where the reactants and products are in standard states, as it approaches
equilibrium.

16.E.4.23: S16.4.16
The standard free energy change is ΔG = −RT ln K = 4.84 kJ/mol . When reactants and products are in their standard states (1 bar or 1 atm), Q = 1. As the reaction proceeds

298

toward equilibrium, the reaction shifts left (the amount of products drops while the amount of reactants increases): Q < 1, and ΔG becomes less positive as it approaches zero.
298

At equilibrium, Q = K, and ΔG = 0.

16.E.4.24: Q16.4.17
Determine the normal boiling point (in kelvin) of dichloroethane, CH2Cl2. Find the actual boiling point using the Internet or some other source, and calculate the percent error in
the temperature. Explain the differences, if any, between the two values.

16.E.4.25: Q16.4.18
Under what conditions is N 2
O (g) ⟶ NO(g) + NO (g)
3 2
spontaneous?

16.E.4.26: S16.4.18
The reaction will be spontaneous at temperatures greater than 287 K.

16.E.4.27: Q16.4.19
At room temperature, the equilibrium constant (Kw) for the self-ionization of water is 1.00 × 10−14. Using this information, calculate the standard free energy change for the
aqueous reaction of hydrogen ion with hydroxide ion to produce water. (Hint: The reaction is the reverse of the self-ionization reaction.)

16.E.4.28: Q16.4.20
3
Hydrogen sulfide is a pollutant found in natural gas. Following its removal, it is converted to sulfur by the reaction 2 H 2
S(g) + SO (g) ⇌
2
S (s, rhombic) + 2 H O(l)
8 2
. What
8
is the equilibrium constant for this reaction? Is the reaction endothermic or exothermic?

16.E.4.29: S16.4.20
K = 5.35 × 1015
The process is exothermic.

16.E.4.30: Q16.4.21
Consider the decomposition of CaCO3(s) into CaO(s) and CO2(g). What is the equilibrium partial pressure of CO2 at room temperature?

16.E.4.31: Q16.4.22
In the laboratory, hydrogen chloride (HCl(g)) and ammonia (NH3(g)) often escape from bottles of their solutions and react to form the ammonium chloride (NH4Cl(s)), the white
glaze often seen on glassware. Assuming that the number of moles of each gas that escapes into the room is the same, what is the maximum partial pressure of HCl and NH3 in the
laboratory at room temperature? (Hint: The partial pressures will be equal and are at their maximum value when at equilibrium.)

16.E.4.32: S16.4.22
1.0 × 10−8 atm. This is the maximum pressure of the gases under the stated conditions.

16.E.4.33: Q16.4.23
Benzene can be prepared from acetylene. 3 C H (g) ⇌ C 2 2 6
H (g)
6
. Determine the equilibrium constant at 25 °C and at 850 °C. Is the reaction spontaneous at either of these
temperatures? Why is all acetylene not found as benzene?

Access for free at OpenStax 16.E.6 https://chem.libretexts.org/@go/page/42801


16.E.4.34: Q16.4.24
Carbon dioxide decomposes into CO and O2 at elevated temperatures. What is the equilibrium partial pressure of oxygen in a sample at 1000 °C for which the initial pressure of
CO2 was 1.15 atm?
−5
x = 1.29 × 10 atm = PO (16.E.12)
2

16.E.4.35: Q16.4.25
Carbon tetrachloride, an important industrial solvent, is prepared by the chlorination of methane at 850 K.
CH (g) + 4 Cl (g) ⟶ CCl (g) + 4 HCl(g) (16.E.13)
4 2 4

What is the equilibrium constant for the reaction at 850 K? Would the reaction vessel need to be heated or cooled to keep the temperature of the reaction constant?

16.E.4.36: Q16.4.25B
Acetic acid, CH3CO2H, can form a dimer, (CH3CO2H)2, in the gas phase.
2 CH CO H(g) ⟶ (CH CO H) (g) (16.E.14)
3 2 3 2 2

The dimer is held together by two hydrogen bonds with a total strength of 66.5 kJ per mole of dimer.

At 25 °C, the equilibrium constant for the dimerization is 1.3 × 103 (pressure in atm). What is ΔS° for the reaction?

16.E.4.37: S16.4.25B
−0.16 kJ

16.E.4.38: Q16.4.26
Nitric acid, HNO3, can be prepared by the following sequence of reactions:
4 NH (g) + 5 O (g) ⟶ 4 NO(g) + 6 H O(g) (16.E.15)
3 2 2

2 NO(g) + O (g) ⟶ 2 NO (g) (16.E.16)


2 2

3 NO (g) + H O(l) ⟶ 2 HNO (l) + NO(g) (16.E.17)


2 2 3

How much heat is evolved when 1 mol of NH3(g) is converted to HNO3(l)? Assume standard states at 25 °C.

16.E.4.39: Q16.4.27A
Determine ΔG for the following reactions.
(a) Antimony pentachloride decomposes at 448 °C. The reaction is:
SbCl (g) ⟶ SbCl (g) + Cl (g) (16.E.18)
5 3 2

An equilibrium mixture in a 5.00 L flask at 448 °C contains 3.85 g of SbCl5, 9.14 g of SbCl3, and 2.84 g of Cl2.
Chlorine molecules dissociate according to this reaction:
Cl (g) ⟶ 2 Cl(g) (16.E.19)
2

1.00% of Cl2 molecules dissociate at 975 K and a pressure of 1.00 atm.

16.E.4.40: S16.4.27A
a. (a) −22.1 kJ;
b. 61.6 kJ/mol

16.E.4.41: Q16.4.27
Given that the ΔG for Pb2+(aq) and Cl−(aq) is −24.3 kJ/mole and −131.2 kJ/mole respectively, determine the solubility product, Ksp, for PbCl2(s).

f

16.E.4.42: Q16.4.28
Determine the standard free energy change, ΔG , for the formation of S2−(aq) given that the ΔG for Ag+(aq) and Ag2S(s) are 77.1 k/mole and −39.5 kJ/mole respectively, and

f

f

the solubility product for Ag2S(s) is 8 × 10−51.

16.E.4.43: S16.4.28
90 kJ/mol

16.E.4.44: Q16.4.29
Determine the standard enthalpy change, entropy change, and free energy change for the conversion of diamond to graphite. Discuss the spontaneity of the conversion with respect
to the enthalpy and entropy changes. Explain why diamond spontaneously changing into graphite is not observed.

16.E.4.45: Q16.4.30
The evaporation of one mole of water at 298 K has a standard free energy change of 8.58 kJ.

H O(l) ⇌ H O(g) ΔG = 8.58 kJ (16.E.20)
2 2 298

a. (a) Is the evaporation of water under standard thermodynamic conditions spontaneous?


b. Determine the equilibrium constant, KP, for this physical process.

Access for free at OpenStax 16.E.7 https://chem.libretexts.org/@go/page/42801


c. By calculating ∆G, determine if the evaporation of water at 298 K is spontaneous when the partial pressure of water, P , is 0.011 atm. H O
2

d. If the evaporation of water were always nonspontaneous at room temperature, wet laundry would never dry when placed outside. In order for laundry to dry, what must be the
value of P in the air?
H O
2

16.E.4.46: S16.4.30
(a) Under standard thermodynamic conditions, the evaporation is nonspontaneous; Kp = 0.031; The evaporation of water is spontaneous; P H O
2
must always be less than Kp or less
than 0.031 atm. 0.031 atm represents air saturated with water vapor at 25 °C, or 100% humidity.

16.E.4.47: Q16.4.31
In glycolysis, the reaction of glucose (Glu) to form glucose-6-phosphate (G6P) requires ATP to be present as described by the following equation:

Glu + ATP ⟶ G6P + ADP ΔG = −17 kJ (16.E.21)
298

In this process, ATP becomes ADP summarized by the following equation:



ATP ⟶ ADP ΔG = −30 kJ (16.E.22)
298

Determine the standard free energy change for the following reaction, and explain why ATP is necessary to drive this process:

Glu ⟶ G6P ΔG =? (16.E.23)
298

16.E.4.48: Q16.4.32
One of the important reactions in the biochemical pathway glycolysis is the reaction of glucose-6-phosphate (G6P) to form fructose-6-phosphate (F6P):

G6P ⇌ F6P ΔG = 1.7 kJ (16.E.24)
298

a. (a) Is the reaction spontaneous or nonspontaneous under standard thermodynamic conditions?


b. Standard thermodynamic conditions imply the concentrations of G6P and F6P to be 1 M, however, in a typical cell, they are not even close to these values. Calculate ΔG when
the concentrations of G6P and F6P are 120 μM and 28 μM respectively, and discuss the spontaneity of the forward reaction under these conditions. Assume the temperature is
37 °C.

16.E.4.49: S16.4.32
28
(a) Nonspontaneous as ΔG ∘
298
>0 ; ΔG∘
298
= −RT ln K, ΔG = 1.7 × 10
3
+ (8.314 × 335 × ln ) = −2.5 kJ . The forward reaction to produce F6P is spontaneous under
128

these conditions.

16.E.4.50: Q16.4.33
Without doing a numerical calculation, determine which of the following will reduce the free energy change for the reaction, that is, make it less positive or more negative, when
the temperature is increased. Explain.
a. (a) N (g) + 3 H (g) ⟶ 2 NH (g)
2 2 3

b. HCl(g) + NH (g) ⟶ NH Cl(s)


3 4

c. (NH ) Cr O (s) ⟶ Cr O (s) + 4 H


4 2 2 7 2 3 2
O(g) + N (g)
2

d. 2 Fe(s) + 3 O (g) ⟶ Fe O (s)


2 2 3

When ammonium chloride is added to water and stirred, it dissolves spontaneously and the resulting solution feels cold. Without doing any calculations, deduce the signs of ΔG,
ΔH, and ΔS for this process, and justify your choices.

16.E.4.51: S16.4.33
ΔG is negative as the process is spontaneous. ΔH is positive as with the solution becoming cold, the dissolving must be endothermic. ΔS must be positive as this drives the process,
and it is expected for the dissolution of any soluble ionic compound.

16.E.4.52: Q16.4.34
An important source of copper is from the copper ore, chalcocite, a form of copper(I) sulfide. When heated, the Cu2S decomposes to form copper and sulfur described by the
following equation:

Cu S(s) ⟶ Cu(s) + S(s) (16.E.25)


2

a. (a) Determine ΔG for the decomposition of Cu2S(s).



298

b. The reaction of sulfur with oxygen yields sulfur dioxide as the only product. Write an equation that describes this reaction, and determine ΔG for the process. ∘
298

c. The production of copper from chalcocite is performed by roasting the Cu2S in air to produce the Cu. By combining the equations from Parts (a) and (b), write the equation that
describes the roasting of the chalcocite, and explain why coupling these reactions together makes for a more efficient process for the production of the copper.

16.E.4.53: Q16.4.35
What happens to ΔG ∘
298
(becomes more negative or more positive) for the following chemical reactions when the partial pressure of oxygen is increased?
a. (a) S(s) + O (g) ⟶ SO (g)
2 2

b. 2 SO (g) + O (g) ⟶ SO (g)


2 2 3

c. HgO(s) ⟶ Hg(l) + O (g) 2

16.E.4.54: S16.4.35
a. (a) Increasing P will shift the equilibrium toward the products, which increases the value of K. ΔG therefore becomes more negative.
O
2

298

b. Increasing P will shift the equilibrium toward the products, which increases the value of K. ΔG therefore becomes more negative.
O
2

298

c. Increasing P will shift the equilibrium the reactants, which decreases the value of K. ΔG therefore becomes more positive.
O
2

298

This page titled 16.E: Thermodynamics (Exercises) is shared under a CC BY 4.0 license and was authored, remixed, and/or curated by OpenStax via source content that was edited to the style and
standards of the LibreTexts platform; a detailed edit history is available upon request.

Access for free at OpenStax 16.E.8 https://chem.libretexts.org/@go/page/42801


CHAPTER OVERVIEW
17: Electrochemistry

A general chemistry Libretexts Textbook remixed and remastered from


OpenStax's textbook:
General Chemistry
Electrochemistry deals with chemical reactions that produce electricity and the changes associated with the passage of electrical
current through matter. The reactions involve electron transfer, and so they are oxidation-reduction (or redox) reactions. Many
metals may be purified or electroplated using electrochemical methods. Devices such as automobiles, smartphones, electronic
tablets, watches, pacemakers, and many others use batteries for power. Batteries use chemical reactions that produce electricity
spontaneously and that can be converted into useful work. All electrochemical systems involve the transfer of electrons in a
reacting system. In many systems, the reactions occur in a region known as the cell, where the transfer of electrons occurs at
electrodes.
17.1: Balancing Oxidation-Reduction Reactions
17.2: Galvanic Cells
17.3: Standard Reduction Potentials
17.4: The Nernst Equation
17.5: Batteries and Fuel Cells
17.6: Corrosion
17.7: Electrolysis
17.E: Electrochemistry (Exercises)

This page titled 17: Electrochemistry is shared under a CC BY 4.0 license and was authored, remixed, and/or curated by OpenStax via source
content that was edited to the style and standards of the LibreTexts platform; a detailed edit history is available upon request.

1
17.1: Balancing Oxidation-Reduction Reactions
 Learning Objectives
Define electrochemistry and a number of important associated terms
Split oxidation-reduction reactions into their oxidation half-reactions and reduction half-reactions
Produce balanced oxidation-reduction equations for reactions in acidic or basic solution
Identify oxidizing agents and reducing agents

Electricity refers to a number of phenomena associated with the presence and flow of electric charge. Electricity includes such
diverse things as lightning, static electricity, the current generated by a battery as it discharges, and many other influences on our
daily lives. The flow or movement of charge is an electric current (Figure 17.1.1). Electrons or ions may carry the charge. The
elementary unit of charge is the charge of a proton, which is equal in magnitude to the charge of an electron. The SI unit of charge
is the coulomb (C) and the charge of a proton is 1.602 × 10−19 C. The presence of an electric charge generates an electric field.
Electric current is the rate of flow of charge.

Figure 17.1.1 : Electricity-related phenomena include lightning, accumulation of static electricity, and current produced by a
battery. (credit left: modification of work by Thomas Bresson; credit middle: modification of work by Chris Darling; credit right:
modification of work by Windell Oskay).
The SI unit for electrical current is the SI base unit called the ampere (A), which is a flow rate of 1 coulomb of charge per second
(1 A = 1 C/s). An electric current flows in a path, called an electric circuit. In most chemical systems, it is necessary to maintain a
closed path for current to flow. The flow of charge is generated by an electrical potential difference, or potential, between two
points in the circuit. Electrical potential is the ability of the electric field to do work on the charge. The SI unit of electrical
potential is the volt (V). When 1 coulomb of charge moves through a potential difference of 1 volt, it gains or loses 1 joule (J) of
energy. Table 17.1.1 summarizes some of this information about electricity.
Table 17.1.1 : Common Electrical Terms
Quantity Definition Measure or Unit

Electric charge Charge on a proton 1.602 × 10−19 C

Electric current The movement of charge ampere = A = 1 C/s

Electric potential The force trying to move the charge volt = V = J/C

The force acting upon other charges in the


Electric field
vicinity

Electrochemistry studies oxidation-reduction reactions, which were first discussed in an earlier chapter, where we learned that
oxidation was the loss of electrons and reduction was the gain of electrons. The reactions discussed tended to be rather simple, and
conservation of mass (atom counting by type) and deriving a correctly balanced chemical equation were relatively simple. In this
section, we will concentrate on the half-reaction method for balancing oxidation-reduction reactions. The use of half-reactions is
important partly for balancing more complicated reactions and partly because many aspects of electrochemistry are easier to
discuss in terms of half-reactions. There are alternate methods of balancing these reactions; however, there are no good alternatives
to half-reactions for discussing what is occurring in many systems. The half-reaction method splits oxidation-reduction reactions
into their oxidation “half” and reduction “half” to make finding the overall equation easier.
Electrochemical reactions frequently occur in solutions, which could be acidic, basic, or neutral. When balancing oxidation-
reduction reactions, the nature of the solution may be important. It helps to see this in an actual problem. Consider the following
unbalanced oxidation-reduction reaction in acidic solution:

Access for free at OpenStax 17.1.1 https://chem.libretexts.org/@go/page/38303


− 2+ 2+ 3+
MnO (aq) + Fe (aq) ⟶ Mn (aq) + Fe (aq)
4

We can start by collecting the species we have so far into an unbalanced oxidation half-reaction and an unbalanced reduction half-
reaction. Each of these half-reactions contain the same element in two different oxidation states. The Fe2+ has lost an electron to
become Fe3+; therefore, the iron underwent oxidation. The reduction is not as obvious; however, the manganese gained five
electrons to change from Mn7+ to Mn2+.
2+ 3+
oxidation (unbalanced): Fe (aq) ⟶ Fe (aq)
− 2+
reduction (unbalanced): MnO (aq) ⟶ Mn (aq)
4

In acidic solution, there are hydrogen ions present, which are often useful in balancing half-reactions. It may be necessary to use
the hydrogen ions directly or as a reactant that may react with oxygen to generate water. Hydrogen ions are very important in acidic
solutions where the reactants or products contain hydrogen and/or oxygen. In this example, the oxidation half-reaction involves
neither hydrogen nor oxygen, so hydrogen ions are not necessary to the balancing. However, the reduction half-reaction does
involve oxygen. It is necessary to use hydrogen ions to convert this oxygen to water.
− + 2+
charge not balanced: MnO (aq) + 8 H (aq) ⟶ Mn (aq) + 4 H O(l)
4 2

The situation is different in basic solution because the hydrogen ion concentration is lower and the hydroxide ion concentration is
higher. After finishing this example, we will examine how basic solutions differ from acidic solutions. A neutral solution may be
treated as acidic or basic, though treating it as acidic is usually easier.
The iron atoms in the oxidation half-reaction are balanced (mass balance); however, the charge is unbalanced, since the charges on
the ions are not equal. It is necessary to use electrons to balance the charge. The way to balance the charge is by adding electrons to
one side of the equation. Adding a single electron on the right side gives a balanced oxidation half-reaction:
2+ 3+ −
oxidation (balanced): Fe (aq) ⟶ Fe (aq) + e

You should check the half-reaction for the number of each atom type and the total charge on each side of the equation. The charges
include the actual charges of the ions times the number of ions and the charge on an electron times the number of electrons.
Fe: Does (1 × 1) = (1 × 1)? Yes.

Charge: Does [1 × (+2)] = [1 × (+3) + 1 × (−1)]? Yes.

If the atoms and charges balance, the half-reaction is balanced. In oxidation half-reactions, electrons appear as products (on the
right). As discussed in the earlier chapter, since iron underwent oxidation, iron is the reducing agent.
Now return to the reduction half-reaction equation:
− + 2+
reduction (unbalanced): MnO (aq) + 8 H (aq) ⟶ Mn (aq) + 4 H O(l)
4 2

The atoms are balanced (mass balance), so it is now necessary to check for charge balance. The total charge on the left of the
reaction arrow is [(−1) × (1) + (8) × (+1)], or +7, while the total charge on the right side is [(1) × (+2) + (4) × (0)], or +2. The
difference between +7 and +2 is five; therefore, it is necessary to add five electrons to the left side to achieve charge balance.
− + − 2+
Reduction (balanced): MnO (aq) + 8 H (aq) + 5 e ⟶ Mn (aq) + 4 H O(l)
4 2

You should check this half-reaction for each atom type and for the charge, as well:
Mn: Does (1 × 1) = (1 × 1)? Yes.

H: Does (8 × 1) = (4 × 2)? Yes.

O: Does (1 × 4) = (4 × 1)? Yes.

Charge: Does [1 × (−1) + 8 × (+1) + 5 × (−1)] = [1 × (+2)]? Yes.

Now that this half-reaction is balanced, it is easy to see it involves reduction because electrons were gained when MnO was −
4

reduced to Mn2+. In all reduction half-reactions, electrons appear as reactants (on the left side). As discussed in the earlier chapter,
the species that was reduced, MnO in this case, is also called the oxidizing agent. We now have two balanced half-reactions.

4

2+ 3+ −
oxidation: Fe (aq) ⟶ Fe (aq) + e
− + − 2+
reduction: MnO (aq) + 8 H (aq) + 5 e ⟶ Mn (aq) + 4 H O(l)
4 2

Access for free at OpenStax 17.1.2 https://chem.libretexts.org/@go/page/38303


It is now necessary to combine the two halves to produce a whole reaction. The key to combining the half-reactions is the electrons.
The electrons lost during oxidation must go somewhere. These electrons go to cause reduction. The number of electrons transferred
from the oxidation half-reaction to the reduction half-reaction must be equal. There can be no missing or excess electrons. In this
example, the oxidation half-reaction generates one electron, while the reduction half-reaction requires five. The lowest common
multiple of one and five is five; therefore, it is necessary to multiply every term in the oxidation half-reaction by five and every
term in the reduction half-reaction by one. (In this case, the multiplication of the reduction half-reaction generates no change;
however, this will not always be the case.) The multiplication of the two half-reactions by the appropriate factor followed by
addition of the two halves gives
2+ 3+ −
oxidation: 5 × (Fe (aq) ⟶ Fe (aq) + e )

− + − 2+
reduction: MnO 4 (aq) + 8 H (aq) + 5 e ⟶ Mn (aq) + 4 H O(l)
2
––––––––––––––––––––––––––––––––––––––––––––––––––––––––––––––––
2+ − + 3+ 2+
overall: 5 Fe (aq) + MnO (aq) + 8 H (aq) ⟶ 5 Fe (aq) + Mn (aq) + 4 H O(l)
4 2

The electrons do not appear in the final answer because the oxidation electrons are the same electrons as the reduction electrons
and they “cancel.” Carefully check each side of the overall equation to verify everything was combined correctly:
Fe: Does (5 × 1) = (5 × 1)? Yes.

Mn: Does (1 × 1) = (1 × 1)? Yes.

H: Does (8 × 1) = (4 × 2)? Yes.

O: Does (1 × 4) = (4 × 1)? Yes.

Charge: Does [5 × (+2) + 1 × (−1) + 8 × (+1)] = [5 × (+3) + 1 × (+2)]? Yes.

Everything checks, so this is the overall equation in acidic solution. If something does not check, the most common error occurs
during the multiplication of the individual half-reactions.
Now suppose we wanted the solution to be basic. Recall that basic solutions have excess hydroxide ions. Some of these hydroxide
ions will react with hydrogen ions to produce water. The simplest way to generate the balanced overall equation in basic solution is
to start with the balanced equation in acidic solution, then “convert” it to the equation for basic solution. However, it is necessary to
exercise caution when doing this, as many reactants behave differently under basic conditions and many metal ions will precipitate
as the metal hydroxide. We just produced the following reaction, which we want to change to a basic reaction:
2+ − + 3+ 2+
5 Fe (aq) + MnO (aq) + 8 H (aq) ⟶ 5 Fe (aq) + Mn (aq) + 4 H O(l)
4 2

However, under basic conditions, MnO normally reduces to MnO2 and iron will be present as either Fe(OH)2 or Fe(OH)3. For

these reasons, under basic conditions, this reaction will be


− −
3 Fe(OH) (s) + MnO (aq) + 2 H O(l) ⟶ 3 Fe(OH) (s) + MnO (s) + OH (aq)
2 4 2 3 2

(Under very basic conditions MnO will reduce to MnO



4
2−
4
, instead of MnO2.)
It is still possible to balance any oxidation-reduction reaction as an acidic reaction and then, when necessary, convert the equation
to a basic reaction. This will work if the acidic and basic reactants and products are the same or if the basic reactants and products
are used before the conversion from acidic or basic. There are very few examples in which the acidic and basic reactions will
involve the same reactants and products. However, balancing a basic reaction as acidic and then converting to basic will work. To
convert to a basic reaction, it is necessary to add the same number of hydroxide ions to each side of the equation so that all the
hydrogen ions (H+) are removed and mass balance is maintained. Hydrogen ion combines with hydroxide ion (OH−) to produce
water.
Let us now try a basic equation. We will start with the following basic reaction:
− − −
Cl (aq) + MnO (aq) ⟶ ClO (aq) + MnO (s)
4 3 2

Balancing this as acid gives


− − + −
Cl (aq) + 2 MnO (aq) + 2 H (aq) ⟶ ClO (aq) + 2 MnO (s) + H O(l)
4 3 2 2

In this case, it is necessary to add two hydroxide ions to each side of the equation to convert the two hydrogen ions on the left into
water:
− − + − − −
Cl (aq) + 2 MnO (aq) + (2 H + 2 OH )(aq) ⟶ ClO (aq) + 2 MnO (s) + H O(l) + 2 OH (aq)
4 3 2 2

Access for free at OpenStax 17.1.3 https://chem.libretexts.org/@go/page/38303


− − − −
Cl (aq) + 2 MnO (aq) + (2 H O)(l) ⟶ ClO (aq) + 2 MnO (s) + H O(l) + 2 OH (aq)
4 2 3 2 2

Note that both sides of the equation show water. Simplifying should be done when necessary, and gives the desired equation. In this
case, it is necessary to remove one H2O from each side of the reaction arrows.
− − − −
Cl (aq) + 2 MnO (aq) + H O(l) ⟶ ClO (aq) + 2 MnO (s) + 2 OH (aq)
4 2 3 2

Again, check each side of the overall equation to make sure there are no errors:
Cl: Does (1 × 1) = (1 × 1)? Yes.

Mn: Does (2 × 1) = (2 × 1)? Yes.

H: Does (1 × 2) = (2 × 1)? Yes.

O: Does (2 × 4 + 1 × 1) = (3 × 1 + 2 × 2 + 2 × 1)? Yes.

Charge: Does [1 × (−1) + 2 × (−1)] = [1 × (−1) + 2 × (−1)]? Yes.

Everything checks, so this is the overall equation in basic solution.

 Example 17.1.1: Balancing Acidic Oxidation-Reduction Reactions

Balance the following reaction equation in acidic solution:


− 3+ 2+ 2−
MnO (aq) + Cr (aq) ⟶ Mn (aq) + Cr O (aq)
4 2 7

Solution
This is an oxidation-reduction reaction, so start by collecting the species given into an unbalanced oxidation half-reaction and
an unbalanced reduction half-reaction.
3+ 2−
oxidation (unbalanced): Cr (aq) ⟶ Cr O (aq)
2 7

− 2+
reduction (unbalanced): MnO (aq) ⟶ Mn (aq)
4

Starting with the oxidation half-reaction, we can balance the chromium


3+ 2−
oxidation (unbalanced): 2 Cr (aq) ⟶ Cr O7 (aq)
2

In acidic solution, we can use or generate hydrogen ions (H+). Adding seven water molecules to the left side provides the necessary
oxygen; the “left over” hydrogen appears as 14 H+ on the right:
3+ 2− +
oxidation (unbalanced): 2 Cr (aq) + 7 H O(l) ⟶ Cr O (aq) + 14 H (aq)
2 2 7

The left side of the equation has a total charge of [2 × (+3) = +6], and the right side a total charge of [−2 + 14 × (+1) = +12]. The
difference is six; adding six electrons to the right side produces a mass- and charge-balanced oxidation half-reaction (in acidic
solution):
3+ 2− + −
oxidation (balanced): 2 Cr (aq) + 7 H O(l) ⟶ Cr O (aq) + 14 H (aq) + 6 e
2 2 7

Checking the half-reaction:


Cr: Does (2 × 1) = (1 × 2)? Yes.

H: Does (7 × 2) = (14 × 1)? Yes.

O: Does (7 × 1) = (1 × 7)? Yes.

Charge: Does [2 × (+3)] = [1 × (−2) + 14 × (+1) + 6 × (−1)]? Yes.

Now work on the reduction. It is necessary to convert the four oxygen atoms in the permanganate into four water molecules. To do
this, add eight H+ to convert the oxygen into four water molecules:
− + 2+
reduction (unbalanced): MnO 4 (aq) + 8 H (aq) ⟶ Mn (aq) + 4 H O(l)
2

Then add five electrons to the left side to balance the charge:

Access for free at OpenStax 17.1.4 https://chem.libretexts.org/@go/page/38303


− + − 2+
reduction (balanced): MnO (aq) + 8 H (aq) + 5 e ⟶ Mn (aq) + 4 H O(l)
4 2

Make sure to check the half-reaction:

Mn: Does (1 × 1) = (1 × 1)? Yes.

H: Does (8 × 1) = (4 × 2)? Yes.

O: Does (1 × 4) = (4 × 1)? Yes.

Charge: Does [1 × (−1) + 8 × (+1) + 5 × (−1)] = [1 × (+2)]? Yes.

Collecting what we have so far:


3+ 2− + −
oxidation: 2 Cr (aq) + 7 H O(l) ⟶ Cr O (aq) + 14 H (aq) + 6 e
2 2 7

− + − 2+
reduction: MnO (aq) + 8 H (aq) + 5 e ⟶ Mn (aq) + 4 H O(l)
4 2

The least common multiple for the electrons is 30, so multiply the oxidation half-reaction by five, the reduction half-reaction by
six, combine, and simplify:
3+ − + 2− + 2+
10 Cr (aq) + 35 H O(l) + 6 MnO 4 (aq) + 48 H (aq) ⟶ 5 Cr O7 (aq) + 70 H (aq) + 6 Mn (aq) + 24 H O(l)
2 2 2

3+ − 2− + 2+
10 Cr (aq) + 11 H O(l) + 6 MnO (aq) ⟶ 5 Cr O (aq) + 22 H (aq) + 6 Mn (aq)
2 4 2 7

Checking each side of the equation:

Mn: Does (6 × 1) = (6 × 1)? Yes.

Cr: Does (10 × 1) = (5 × 2)? Yes.

H: Does (11 × 2) = (22 × 1)? Yes.

O: Does (11 × 1 + 6 × 4) = (5 × 7)? Yes.

Charge: Does [10 × (+3) + 6 × (−1)] = [5 × (−2) + 22 × (+1) + 6 × (+2)]? Yes.

This is the balanced equation in acidic solution.

 Exercise 17.1.1

Balance the following equation in acidic solution:


2+ +
Hg + Ag ⟶ Hg + Ag
2

Answer
2+ +
Hg (aq) + 2 Ag(s) ⟶ 2 Hg(l) + 2 Ag (aq)
2

 Example 17.1.2: Balancing Basic Oxidation-Reduction Reactions

Balance the following reaction equation in basic solution:


− 2−
MnO (aq) + Cr(OH) (s) ⟶ MnO (s) + CrO (aq)
4 3 2 4

Solution
This is an oxidation-reduction reaction, so start by collecting the species given into an unbalanced oxidation half-reaction and
an unbalanced reduction half-reaction
2−
oxidation (unbalanced): Cr(OH) (s) ⟶ CrO (aq)
3 4


reduction (unbalanced): MnO (aq) ⟶ MnO (s)
4 2

Starting with the oxidation half-reaction, we can balance the chromium


2−
oxidation (unbalanced): Cr(OH) (s) ⟶ CrO (aq)
3 4

Access for free at OpenStax 17.1.5 https://chem.libretexts.org/@go/page/38303


In acidic solution, we can use or generate hydrogen ions (H+). Adding one water molecule to the left side provides the
necessary oxygen; the “left over” hydrogen appears as five H+ on the right side:
2− +
oxidation (unbalanced): Cr(OH) (s) + H O(l) ⟶ CrO (aq) + 5 H (aq)
3 2 4

The left side of the equation has a total charge of [0], and the right side a total charge of [−2 + 5 × (+1) = +3]. The difference is
three, adding three electrons to the right side produces a mass- and charge-balanced oxidation half-reaction (in acidic solution):
2− + −
oxidation (balanced): Cr(OH) (s) + H O(l) ⟶ CrO (aq) + 5 H (aq) + 3 e
3 2 4

Checking the half-reaction:

Cr: Does (1 × 1) = (1 × 1)? Yes.

H: Does (1 × 3 + 1 × 2) = (5 × 1)? Yes.

O: Does (1 × 3 + 1 × 1) = (4 × 1)? Yes.

Charge: Does [0 = [1 × (−2) + 5 × (+1) + 3 × (−1)]? Yes.

Now work on the reduction. It is necessary to convert the four O atoms in the MnO4− minus the two O atoms in MnO2 into two
water molecules. To do this, add four H+ to convert the oxygen into two water molecules:
− +
reduction (unbalanced): MnO 4 (aq) + 4 H (aq) ⟶ MnO (s) + 2 H O(l)
2 2

Then add three electrons to the left side to balance the charge:
− + −
reduction (balanced): MnO (aq) + 4 H (aq) + 3 e ⟶ MnO (s) + 2 H O(l)
4 2 2

Make sure to check the half-reaction:

Mn: Does (1 × 1) = (1 × 1)? Yes.

H: Does (4 × 1) = (2 × 2)? Yes.

O: Does (1 × 4) = (1 × 2 + 2 × 1)? Yes.

Charge: Does [1 × (−1) + 4 × (+1) + 3 × (−1)] = [0]? Yes.

Collecting what we have so far:


2− + −
oxidation: Cr(OH) (s) + H O(l) ⟶ CrO (aq) + 5 H (aq) + 3 e
3 2 4

− + −
reduction: MnO (aq) + 4 H (aq) + 3 e ⟶ MnO (s) + 2 H O(l)
4 2 2

In this case, both half reactions involve the same number of electrons; therefore, simply add the two half-reactions together.
− + 2− +
MnO (aq) + 4 H (aq) + Cr(OH) (s) + H O(l) ⟶ CrO (aq) + MnO (s) + 2 H O(l) + 5 H (aq)
4 3 2 4 2 2

− 2− +
MnO (aq) + Cr(OH) (s) ⟶ CrO (aq) + MnO (s) + H O(l) + H (aq)
4 3 4 2 2

Checking each side of the equation:


Mn: Does (1 × 1) = (1 × 1)? Yes.

Cr: Does (1 × 1) = (1 × 1)? Yes.

H: Does (1 × 3) = (2 × 1 + 1 × 1)? Yes.

O: Does (1 × 4 + 1 × 3) = (1 × 4 + 1 × 2 + 1 × 1)? Yes.

Charge: Does [1 × (−1)] = [1 × (−2) + 1 × (+1)]? Yes.

This is the balanced equation in acidic solution. For a basic solution, add one hydroxide ion to each side and simplify:
− − 2− + −
OH (aq) + MnO (aq) + Cr(OH) (s) ⟶ CrO (aq) + MnO (s) + H O(l) + (H + OH )(aq)
4 3 4 2 2

− − 2−
OH (aq) + MnO 4 (aq) + Cr(OH) (s) ⟶ CrO4 (aq) + MnO (s) + 2 H O(l)
3 2 2

Checking each side of the equation:

Access for free at OpenStax 17.1.6 https://chem.libretexts.org/@go/page/38303


Mn: Does (1 × 1) = (1 × 1)? Yes.

Cr: Does (1 × 1) = (1 × 1)? Yes.

H: Does (1 × 1 + 1 × 3) = (2 × 2)? Yes.

O: Does (1 × 1 + 1 × 4 + 1 × 3) = (1 × 4 + 1 × 2 + 2 × 1)? Yes.

Charge: Does [1 × (−1) + 1 × (−1)] = [1 × (−2)]? Yes.

This is the balanced equation in basic solution.

 Exercise 17.1.2
Balance the following in the type of solution indicated.
a. H + Cu ⟶ Cu (acidic solution)
2
2+

b. H + Cu(OH) ⟶ Cu (basic solution)


2 2

c. Fe + Ag ⟶ Fe + Ag
+ 2+

d. Identify the oxidizing agents in reactions (a), (b), and (c).


e. Identify the reducing agents in reactions (a), (b), and (c).

Answer a
2+ +
H (g) + Cu (aq) ⟶ 2 H (aq) + Cu(s)
2

Answer b
H (g) + Cu(OH) (s) ⟶ 2 H O(l) + Cu(s)
2 2 2

Answer c
+ 2+
Fe(s) + 2 Ag (aq) ⟶ Fe (aq) + 2 Ag(s)

Answer d
oxidizing agent = species reduced: Cu2+, Cu(OH)2, Ag+
Answer e
reducing agent = species oxidized: H2, H2, Fe.

Summary
An electric current consists of moving charge. The charge may be in the form of electrons or ions. Current flows through an
unbroken or closed circular path called a circuit. The current flows through a conducting medium as a result of a difference in
electrical potential between two points in a circuit. Electrical potential has the units of energy per charge. In SI units, charge is
C J
measured in coulombs (C), current in amperes (A = ) , and electrical potential in volts (V = ) .
s C

Oxidation is the loss of electrons, and the species that is oxidized is also called the reducing agent. Reduction is the gain of
electrons, and the species that is reduced is also called the oxidizing agent. Oxidation-reduction reactions can be balanced using the
half-reaction method. In this method, the oxidation-reduction reaction is split into an oxidation half-reaction and a reduction half-
reaction. The oxidation half-reaction and reduction half-reaction are then balanced separately. Each of the half-reactions must have
the same number of each type of atom on both sides of the equation and show the same total charge on each side of the equation.
Charge is balanced in oxidation half-reactions by adding electrons as products; in reduction half-reactions, charge is balanced by
adding electrons as reactants. The total number of electrons gained by reduction must exactly equal the number of electrons lost by
oxidation when combining the two half-reactions to give the overall balanced equation. Balancing oxidation-reduction reaction
equations in aqueous solutions frequently requires that oxygen or hydrogen be added or removed from a reactant. In acidic
solution, hydrogen is added by adding hydrogen ion (H+) and removed by producing hydrogen ion; oxygen is removed by adding
hydrogen ion and producing water, and added by adding water and producing hydrogen ion. A balanced equation in basic solution
can be obtained by first balancing the equation in acidic solution, and then adding hydroxide ion to each side of the balanced
equation in such numbers that all the hydrogen ions are converted to water.

Access for free at OpenStax 17.1.7 https://chem.libretexts.org/@go/page/38303


Glossary
circuit
path taken by a current as it flows because of an electrical potential difference

current
C
flow of electrical charge; the SI unit of charge is the coulomb (C) and current is measured in amperes (1 A = 1 )
s

electrical potential
energy per charge; in electrochemical systems, it depends on the way the charges are distributed within the system; the SI unit
J
of electrical potential is the volt (1 V = 1 )
C

half-reaction method
method that produces a balanced overall oxidation-reduction reaction by splitting the reaction into an oxidation “half” and
reduction “half,” balancing the two half-reactions, and then combining the oxidation half-reaction and reduction half-reaction in
such a way that the number of electrons generated by the oxidation is exactly canceled by the number of electrons required by
the reduction

oxidation half-reaction
the “half” of an oxidation-reduction reaction involving oxidation; the half-reaction in which electrons appear as products;
balanced when each atom type, as well as the charge, is balanced

reduction half-reaction
the “half” of an oxidation-reduction reaction involving reduction; the half-reaction in which electrons appear as reactants;
balanced when each atom type, as well as the charge, is balanced

This page titled 17.1: Balancing Oxidation-Reduction Reactions is shared under a CC BY 4.0 license and was authored, remixed, and/or curated
by OpenStax via source content that was edited to the style and standards of the LibreTexts platform; a detailed edit history is available upon
request.

Access for free at OpenStax 17.1.8 https://chem.libretexts.org/@go/page/38303


17.2: Galvanic Cells
 Learning Objectives
Use cell notation to describe galvanic cells
Describe the basic components of galvanic cells

Galvanic cells, also known as voltaic cells, are electrochemical cells in which spontaneous oxidation-reduction reactions produce
electrical energy. In writing the equations, it is often convenient to separate the oxidation-reduction reactions into half-reactions to
facilitate balancing the overall equation and to emphasize the actual chemical transformations.
Consider what happens when a clean piece of copper metal is placed in a solution of silver nitrate (Figure 17.2.1). As soon as the
copper metal is added, silver metal begins to form and copper ions pass into the solution. The blue color of the solution on the far
right indicates the presence of copper ions. The reaction may be split into its two half-reactions. Half-reactions separate the
oxidation from the reduction, so each can be considered individually.
2+ −
oxidation: Cu(s) ⟶ Cu (aq) + 2 e (17.2.1)
+ − + −
reduction: 2 × (Ag (aq) + e ⟶ Ag(s)) or 2 Ag (aq) + 2 e ⟶ 2 Ag(s) (17.2.2)
––––––––––––––––––––––––––––––––––––––––––––––––––––––––––––––––––––––––––––––––
+ 2+
overall: 2 Ag (aq) + Cu(s) ⟶ 2 Ag(s) + Cu (aq) (17.2.3)

The equation for the reduction half-reaction had to be doubled so the number electrons “gained” in the reduction half-reaction
equaled the number of electrons “lost” in the oxidation half-reaction.

Figure 17.2.1 : When a clean piece of copper metal is placed into a clear solution of silver nitrate (a), an oxidation-reduction
reaction occurs that results in the exchange of Cu2+ for Ag+ ions in solution. As the reaction proceeds (b), the solution turns blue
(c) because of the copper ions present, and silver metal is deposited on the copper strip as the silver ions are removed from
solution. (credit: modification of work by Mark Ott)
Galvanic or voltaic cells involve spontaneous electrochemical reactions in which the half-reactions are separated (Figure 17.2.2) so
that current can flow through an external wire. The beaker on the left side of the figure is called a half-cell, and contains a 1 M
solution of copper(II) nitrate [Cu(NO3)2] with a piece of copper metal partially submerged in the solution. The copper metal is an
electrode. The copper is undergoing oxidation; therefore, the copper electrode is the anode. The anode is connected to a voltmeter
with a wire and the other terminal of the voltmeter is connected to a silver electrode by a wire. The silver is undergoing reduction;
therefore, the silver electrode is the cathode. The half-cell on the right side of the figure consists of the silver electrode in a 1 M
solution of silver nitrate (AgNO3). At this point, no current flows—that is, no significant movement of electrons through the wire
occurs because the circuit is open. The circuit is closed using a salt bridge, which transmits the current with moving ions. The salt
bridge consists of a concentrated, nonreactive, electrolyte solution such as the sodium nitrate (NaNO3) solution used in this
example. As electrons flow from left to right through the electrode and wire, nitrate ions (anions) pass through the porous plug on
the left into the copper(II) nitrate solution. This keeps the beaker on the left electrically neutral by neutralizing the charge on the
copper(II) ions that are produced in the solution as the copper metal is oxidized. At the same time, the nitrate ions are moving to the
left, sodium ions (cations) move to the right, through the porous plug, and into the silver nitrate solution on the right. These added
cations “replace” the silver ions that are removed from the solution as they were reduced to silver metal, keeping the beaker on the
right electrically neutral. Without the salt bridge, the compartments would not remain electrically neutral and no significant current
would flow. However, if the two compartments are in direct contact, a salt bridge is not necessary. The instant the circuit is
completed, the voltmeter reads +0.46 V, this is called the cell potential. The cell potential is created when the two dissimilar metals
are connected, and is a measure of the energy per unit charge available from the oxidation-reduction reaction. The volt is the
derived SI unit for electrical potential

Access for free at OpenStax 17.2.1 https://chem.libretexts.org/@go/page/38304


J
volt = V =
C

In this equation, A is the current in amperes and C the charge in coulombs. Note that volts must be multiplied by the charge in
coulombs (C) to obtain the energy in joules (J).

Figure 17.2.2 : In this standard galvanic cell, the half-cells are separated; electrons can flow through an external wire and become
available to do electrical work.
When the electrochemical cell is constructed in this fashion, a positive cell potential indicates a spontaneous reaction and that the
electrons are flowing from the left to the right. There is a lot going on in Figure 17.2.2, so it is useful to summarize things for this
system:
Electrons flow from the anode to the cathode: left to right in the standard galvanic cell in the figure.
The electrode in the left half-cell is the anode because oxidation occurs here. The name refers to the flow of anions in the salt
bridge toward it.
The electrode in the right half-cell is the cathode because reduction occurs here. The name refers to the flow of cations in the
salt bridge toward it.
Oxidation occurs at the anode (the left half-cell in the figure).
Reduction occurs at the cathode (the right half-cell in the figure).
The cell potential, +0.46 V, in this case, results from the inherent differences in the nature of the materials used to make the two
half-cells.
The salt bridge must be present to close (complete) the circuit and both an oxidation and reduction must occur for current to
flow.
There are many possible galvanic cells, so a shorthand notation is usually used to describe them. The cell notation (sometimes
called a cell diagram) provides information about the various species involved in the reaction. This notation also works for other
types of cells. A vertical line, │, denotes a phase boundary and a double line, ‖, the salt bridge. Information about the anode is
written to the left, followed by the anode solution, then the salt bridge (when present), then the cathode solution, and, finally,
information about the cathode to the right. The cell notation for the galvanic cell in Figure 17.2.2 is then
2+ +
Cu(s) │Cu (aq, 1 M )║Ag (aq, 1 M )│Ag(s)

Note that spectator ions are not included and that the simplest form of each half-reaction was used. When known, the initial
concentrations of the various ions are usually included.
One of the simplest cells is the Daniell cell. It is possible to construct this battery by placing a copper electrode at the bottom of a
jar and covering the metal with a copper sulfate solution. A zinc sulfate solution is floated on top of the copper sulfate solution;
then a zinc electrode is placed in the zinc sulfate solution. Connecting the copper electrode to the zinc electrode allows an electric
current to flow. This is an example of a cell without a salt bridge, and ions may flow across the interface between the two solutions.
Some oxidation-reduction reactions involve species that are poor conductors of electricity, and so an electrode is used that does not
participate in the reactions. Frequently, the electrode is platinum, gold, or graphite, all of which are inert to many chemical

Access for free at OpenStax 17.2.2 https://chem.libretexts.org/@go/page/38304


reactions. One such system is shown in Figure 17.2.3. Magnesium undergoes oxidation at the anode on the left in the figure and
hydrogen ions undergo reduction at the cathode on the right. The reaction may be summarized as
2+ −
oxidation: Mg(s) ⟶ Mg (aq) + 2 e (17.2.4)
+ −
reduction: 2 H (aq) + 2 e ⟶ H (g) (17.2.5)
2

¯¯¯¯¯¯¯¯¯¯¯¯¯¯¯¯¯¯¯¯¯¯¯¯¯¯¯¯¯¯¯¯¯¯¯¯¯¯¯¯¯¯¯¯¯¯¯¯¯¯¯¯¯¯¯¯¯¯¯¯¯¯¯¯¯¯¯¯¯¯¯¯¯¯¯¯¯¯¯¯¯¯¯¯¯¯¯¯¯¯¯¯¯¯¯¯¯¯¯¯¯¯
¯
+ 2+
overall: Mg(s) + 2 H (aq) ⟶ Mg (aq) + H (g) (17.2.6)
2

The cell used an inert platinum wire for the cathode, so the cell notation is

Mg(s) │Mg2+ (aq)║H+ (aq)│H2 (g)│Pt(s)

The magnesium electrode is an active electrode because it participates in the oxidation-reduction reaction. Inert electrodes, like the
platinum electrode in Figure 17.2.3, do not participate in the oxidation-reduction reaction and are present so that current can flow
through the cell. Platinum or gold generally make good inert electrodes because they are chemically unreactive.

Figure 17.2.2 : The oxidation of magnesium to magnesium ion occurs in the beaker on the left side in this apparatus; the reduction
of hydrogen ions to hydrogen occurs in the beaker on the right. A nonreactive, or inert, platinum wire allows electrons from the left
beaker to move into the right beaker. The overall reaction is: Mg + 2 H ⟶ Mg + H , which is represented in cell notation
+ 2+

as: Mg(s)│Mg (aq)║H (aq)│H (g)│Pt(s)


2+ +
2
.

 Example 17.2.1: Using Cell Notation

Consider a galvanic cell consisting of


2+ 3+
2 Cr(s) + 3 Cu (aq) ⟶ 2 Cr (aq) + 3 Cu(s)

Write the oxidation and reduction half-reactions and write the reaction using cell notation. Which reaction occurs at the anode?
The cathode?

Solution
By inspection, Cr is oxidized when three electrons are lost to form Cr3+, and Cu2+ is reduced as it gains two electrons to form
Cu. Balancing the charge gives
3+ −
oxidation: 2 Cr(s) ⟶ 2 Cr (aq) + 6 e (17.2.7)

2+ −
reduction: 3 Cu (aq) + 6 e ⟶ 3 Cu(s) (17.2.8)

¯¯¯¯¯¯¯¯¯¯¯¯¯¯¯¯¯¯¯¯¯¯¯¯¯¯¯¯¯¯¯¯¯¯¯¯¯¯¯¯¯¯¯¯¯¯¯¯¯¯¯¯¯¯¯¯¯¯¯¯¯¯¯¯¯¯¯¯¯¯¯¯¯¯¯¯¯¯¯¯¯¯¯¯¯¯¯¯¯¯¯¯¯¯¯¯¯¯¯¯¯¯¯¯¯¯¯¯¯¯¯¯¯¯
¯
2+ 3+
overall: 2 Cr(s) + 3 Cu (aq) ⟶ 2 Cr (aq) + 3 Cu(s) (17.2.9)

Cell notation uses the simplest form of each of the equations, and starts with the reaction at the anode. No concentrations were
specified so:

Access for free at OpenStax 17.2.3 https://chem.libretexts.org/@go/page/38304


3+ 2+
Cr(s) │Cr (aq) ║Cu (aq) │Cu(s).

Oxidation occurs at the anode and reduction at the cathode.

 Example 17.2.2: Using Cell Notation

Consider a galvanic cell consisting of


2+ − + 3+ 2+
5 Fe (aq) + MnO (aq) + 8 H (aq) ⟶ 5 Fe (aq) + Mn (aq) + 4 H O(l)
4 2

Write the oxidation and reduction half-reactions and write the reaction using cell notation. Which reaction occurs at the anode?
The cathode?

Solution
By inspection, Fe2+ undergoes oxidation when one electron is lost to form Fe3+, and MnO4− is reduced as it gains five electrons
to form Mn2+. Balancing the charge gives
2+ 3+ −
oxidation: 5(Fe (aq) ⟶ Fe (aq) + e ) (17.2.10)
− + − 2+
reduction: MnO 4 (aq) + 8 H (aq) + 5 e ⟶ Mn (aq) + 4 H O(l) (17.2.11)
2
––––––––––––––––––––––––––––––––––––––––––––––––––––––––––––––––
2+ − + 3+ 2+
overall: 5 Fe (aq) + MnO (aq) + 8 H (aq) ⟶ 5 Fe (aq) + Mn (aq) + 4 H O(l) (17.2.12)
4 2

Cell notation uses the simplest form of each of the equations, and starts with the reaction at the anode. It is necessary to use an
inert electrode, such as platinum, because there is no metal present to conduct the electrons from the anode to the cathode. No
concentrations were specified so:

Pt(s) │Fe2+ (aq), Fe


3+
(aq) ║MnO −
4
(aq), H
+
(aq), Mn
2+
(aq) │Pt(s).

Oxidation occurs at the anode and reduction at the cathode.

 Exercise 17.2.1
Use cell notation to describe the galvanic cell where copper(II) ions are reduced to copper metal and zinc metal is oxidized to
zinc ions.

Answer
From the information given in the problem:
2+ −
anode (oxidation): Zn(s) ⟶ Zn (aq) + 2 e (17.2.13)

2+ −
cathode (reduction): Cu (aq) + 2 e ⟶ Cu(s) (17.2.14)

¯¯¯¯¯¯¯¯¯¯¯¯¯¯¯¯¯¯¯¯¯¯¯¯¯¯¯¯¯¯¯¯¯¯¯¯¯¯¯¯¯¯¯¯¯¯¯¯¯¯¯¯¯¯¯¯¯¯¯¯¯¯¯¯¯¯¯¯¯¯¯¯¯¯¯¯¯¯¯¯¯¯¯¯¯¯¯¯¯¯¯¯¯¯¯¯¯¯¯¯¯
¯
2+ 2+
overall: Zn(s) + Cu (aq) ⟶ Zn (aq) + Cu(s) (17.2.15)

Using cell notation:

Zn(s) │Zn2+ (aq)║Cu2+ (aq)│Cu(s).

Summary
Electrochemical cells typically consist of two half-cells. The half-cells separate the oxidation half-reaction from the reduction half-
reaction and make it possible for current to flow through an external wire. One half-cell, normally depicted on the left side in a
figure, contains the anode. Oxidation occurs at the anode. The anode is connected to the cathode in the other half-cell, often shown
on the right side in a figure. Reduction occurs at the cathode. Adding a salt bridge completes the circuit allowing current to flow.
Anions in the salt bridge flow toward the anode and cations in the salt bridge flow toward the cathode. The movement of these ions
completes the circuit and keeps each half-cell electrically neutral. Electrochemical cells can be described using cell notation. In this
notation, information about the reaction at the anode appears on the left and information about the reaction at the cathode on the
right. The salt bridge is represented by a double line, ‖. The solid, liquid, or aqueous phases within a half-cell are separated by a

Access for free at OpenStax 17.2.4 https://chem.libretexts.org/@go/page/38304


single line, │. The phase and concentration of the various species is included after the species name. Electrodes that participate in
the oxidation-reduction reaction are called active electrodes. Electrodes that do not participate in the oxidation-reduction reaction
but are there to allow current to flow are inert electrodes. Inert electrodes are often made from platinum or gold, which are
unchanged by many chemical reactions.

Glossary
active electrode
electrode that participates in the oxidation-reduction reaction of an electrochemical cell; the mass of an active electrode changes
during the oxidation-reduction reaction

anode
electrode in an electrochemical cell at which oxidation occurs; information about the anode is recorded on the left side of the
salt bridge in cell notation

cathode
electrode in an electrochemical cell at which reduction occurs; information about the cathode is recorded on the right side of the
salt bridge in cell notation

cell notation
shorthand way to represent the reactions in an electrochemical cell

cell potential
difference in electrical potential that arises when dissimilar metals are connected; the driving force for the flow of charge
(current) in oxidation-reduction reactions

galvanic cell
electrochemical cell that involves a spontaneous oxidation-reduction reaction; electrochemical cells with positive cell
potentials; also called a voltaic cell

inert electrode
electrode that allows current to flow, but that does not otherwise participate in the oxidation-reduction reaction in an
electrochemical cell; the mass of an inert electrode does not change during the oxidation-reduction reaction; inert electrodes are
often made of platinum or gold because these metals are chemically unreactive.

voltaic cell
another name for a galvanic cell

This page titled 17.2: Galvanic Cells is shared under a CC BY 4.0 license and was authored, remixed, and/or curated by OpenStax via source
content that was edited to the style and standards of the LibreTexts platform; a detailed edit history is available upon request.

Access for free at OpenStax 17.2.5 https://chem.libretexts.org/@go/page/38304


17.3: Standard Reduction Potentials
 Learning Objectives
Determine standard cell potentials for oxidation-reduction reactions
Use standard reduction potentials to determine the better oxidizing or reducing agent from among several possible choices

The cell potential results from the difference in the electrical potentials for each electrode. While it is impossible to determine the
electrical potential of a single electrode, we can assign an electrode the value of zero and then use it as a reference. The electrode
chosen as the zero is shown in Figure 17.4.1 and is called the standard hydrogen electrode (SHE). The SHE consists of 1 atm of
hydrogen gas bubbled through a 1 M HCl solution, usually at room temperature. Platinum, which is chemically inert, is used as the
electrode. The reduction half-reaction chosen as the reference is
+ −
2H (aq, 1 M ) + 2 e ⇌ H (g, 1 atm) E° = 0 V
2

E° is the standard reduction potential. The superscript “°” on the E denotes standard conditions (1 bar or 1 atm for gases, 1 M for
solutes). The voltage is defined as zero for all temperatures.

Figure 17.3.1 : Hydrogen gas at 1 atm is bubbled through 1 M HCl solution. Platinum, which is inert to the action of the 1 M HCl,
is used as the electrode. Electrons on the surface of the electrode combine with H+ in solution to produce hydrogen gas.
A galvanic cell consisting of a SHE and Cu2+/Cu half-cell can be used to determine the standard reduction potential for Cu2+
(Figure 17.3.2). In cell notation, the reaction is
+ 2+
Pt(s) │H2 (g, 1 atm) │H (aq, 1 M ) ║Cu (aq, 1 M ) │Cu(s)

Electrons flow from the anode to the cathode. The reactions, which are reversible, are
+ −
Anode (oxidation): H (g) ⟶ 2 H (aq) + 2 e
2

2+ −
Cathode (reduction): Cu (aq) + 2 e ⟶ Cu(s)

¯¯¯¯¯¯¯¯¯¯¯¯¯¯¯¯¯¯¯¯¯¯¯¯¯¯¯¯¯¯¯¯¯¯¯¯¯¯¯¯¯¯¯¯¯¯¯¯¯¯¯¯¯¯¯¯¯¯¯¯¯¯¯¯¯¯¯¯¯¯¯¯¯¯¯¯¯¯¯¯¯¯¯¯¯¯¯¯¯¯¯¯¯¯¯¯¯¯¯¯¯¯
¯
2+ +
Overall: Cu (aq) + H (g) ⟶ 2 H (aq) + Cu(s)
2

The standard reduction potential can be determined by subtracting the standard reduction potential for the reaction occurring at the
anode from the standard reduction potential for the reaction occurring at the cathode. The minus sign is necessary because
oxidation is the reverse of reduction.
∘ ∘ ∘
E =E −E
cell cathode anode

∘ ∘ ∘ ∘
+0.34 V = E 2+
−E +
=E 2+
−0 = E 2+
Cu /Cu H /H Cu /Cu Cu /Cu
2

Access for free at OpenStax 17.3.1 https://chem.libretexts.org/@go/page/38305


Figure 17.3.2 : A galvanic cell can be used to determine the standard reduction potential of Cu2+.
Using the SHE as a reference, other standard reduction potentials can be determined. Consider the cell shown in Figure 17.3.2 ,
where
+
Pt(s) │H2 (g, 1 atm) │H+ (aq, 1 M) ║Ag (aq, 1 M ) │Ag(s)

Electrons flow from left to right, and the reactions are


+ −
anode (oxidation): H (g) ⟶ 2 H (aq) + 2 e
2
+ −
cathode (reduction): 2 Ag (aq) + 2 e ⟶ 2 Ag(s)

¯¯¯¯¯¯¯¯¯¯¯¯¯¯¯¯¯¯¯¯¯¯¯¯¯¯¯¯¯¯¯¯¯¯¯¯¯¯¯¯¯¯¯¯¯¯¯¯¯¯¯¯¯¯¯¯¯¯¯¯¯¯¯¯¯¯¯¯¯¯¯¯¯¯¯¯¯¯¯¯¯¯¯¯¯¯¯¯¯¯¯¯¯¯¯¯¯¯¯¯¯¯¯¯¯
¯
+ +
overall: 2 Ag (aq) + H (g) ⟶ 2 H (aq) + 2 Ag(s)
2

The standard reduction potential can be determined by subtracting the standard reduction potential for the reaction occurring at the
anode from the standard reduction potential for the reaction occurring at the cathode. The minus sign is needed because oxidation is
the reverse of reduction.
∘ ∘ ∘
E =E −E
cell cathode anode

∘ ∘ ∘ ∘
+0.80 V = E +
−E +
=E +
−0 = E +
Ag /Ag H /H Ag /Ag Ag /Ag
2

It is important to note that the potential is not doubled for the cathode reaction.
The SHE is rather dangerous and rarely used in the laboratory. Its main significance is that it established the zero for standard
reduction potentials. Once determined, standard reduction potentials can be used to determine the standard cell potential, E , for ∘
cell

any cell. For example, for the following cell:


2+ +
Cu(s) │Cu (aq, 1 M ) ║Ag (aq, 1 M ) │Ag(s)
2+ −
anode (oxidation): Cu(s) ⟶ Cu (aq) + 2 e
+ −
cathode (reduction): 2 Ag (aq) + 2 e ⟶ 2 Ag(s)

¯¯¯¯¯¯¯¯¯¯¯¯¯¯¯¯¯¯¯¯¯¯¯¯¯¯¯¯¯¯¯¯¯¯¯¯¯¯¯¯¯¯¯¯¯¯¯¯¯¯¯¯¯¯¯¯¯¯¯¯¯¯¯¯¯¯¯¯¯¯¯¯¯¯¯¯¯¯¯¯¯¯¯¯¯¯¯¯¯¯¯¯¯¯¯¯¯¯¯¯¯¯¯¯¯¯¯
¯
+ 2+
overall: Cu(s) + 2 Ag (aq) ⟶ Cu (aq) + 2 Ag(s)

∘ ∘ ∘ ∘ ∘
E =E −E =E +
−E 2+
= 0.80 V − 0.34 V = 0.46 V
cell cathode anode Ag /Ag Cu /Cu

Again, note that when calculating E , standard reduction potentials always remain the same even when a half-reaction is

cell

multiplied by a factor. Standard reduction potentials for selected reduction reactions are shown in Table 17.3.1. A more complete
list is provided in Tables P1 or P2.

Access for free at OpenStax 17.3.2 https://chem.libretexts.org/@go/page/38305


Figure 17.3.3 : A galvanic cell can be used to determine the standard reduction potential of Ag+. The SHE on the left is the anode
and assigned a standard reduction potential of zero.
Table 17.3.1 : Selected Standard Reduction Potentials at 25 °C
Half-Reaction E° (V)

F (g) + 2 e
2

⟶ 2F

(aq) +2.866

PbO (s) + SO
2
2−
4
(aq) + 4 H
+
(aq) + 2 e

⟶ PbSO (s) + 2 H O(l)
4 2
+1.69

MnO

4
(aq) + 8 H
+
(aq) + 5 e

⟶ Mn
2+
(aq) + 4 H O(l)
2
+1.507

Au
3+
(aq) + 3 e

⟶ Au(s) +1.498

Cl (g) + 2 e
2

⟶ 2 Cl

(aq) +1.35827

O (g) + 4 H
2
+
(aq) + 4 e

⟶ 2 H O(l)
2
+1.229

Pt
2+
(aq) + 2 e

⟶ Pt(s) +1.20

Br (aq) + 2 e
2

⟶ 2 Br

(aq) +1.0873

Ag
+
(aq) + e

⟶ Ag(s) +0.7996

Hg
2+
2
(aq) + 2 e

⟶ 2 Hg(l) +0.7973

Fe
3+
(aq) + e

⟶ Fe
2+
(aq) +0.771

MnO

4
(aq) + 2 H O(l) + 3 e
2

⟶ MnO (s) + 4 OH
2

(aq) +0.558

I (s) + 2 e
2

⟶ 2I

(aq) +0.5355

NiO (s) + 2 H O(l) + 2 e


2 2

⟶ Ni(OH) (s) + 2 OH
2

(aq) +0.49

Cu
2+
(aq) + 2 e

⟶ Cu(s) +0.34

Hg Cl (s) + 2 e
2 2

⟶ 2 Hg(l) + 2 Cl

(aq) +0.26808

AgCl(s) + e

⟶ Ag(s) + Cl

(aq) +0.22233

Sn
4+
(aq) + 2 e

⟶ Sn
2+
(aq) +0.151

2H
+
(aq) + 2 e

⟶ H (g)
2
0.00

Pb
2+
(aq) + 2 e

⟶ Pb(s) −0.1262

Sn
2+
(aq) + 2 e

⟶ Sn(s) −0.1375

Ni
2+
(aq) + 2 e

⟶ Ni(s) −0.257

Co
2+
(aq) + 2 e

⟶ Co(s) −0.28

Access for free at OpenStax 17.3.3 https://chem.libretexts.org/@go/page/38305


Half-Reaction E° (V)

PbSO (s) + 2 e
4

⟶ Pb(s) + SO
2−
4
(aq) −0.3505

Cd
2+
(aq) + 2 e

⟶ Cd(s) −0.4030

Fe
2+
(aq) + 2 e

⟶ Fe(s) −0.447

Cr
3+
(aq) + 3 e

⟶ Cr(s) −0.744

Mn
2+
(aq) + 2 e

⟶ Mn(s) −1.185

Zn(OH) (s) + 2 e
2

⟶ Zn(s) + 2 OH

(aq) −1.245

Zn
2+
(aq) + 2 e

⟶ Zn(s) −0.7618

Al
3+
(aq) + 3 e

⟶ Al(s) −1.662

Mg
2+
(aq) + 2 e

⟶ Mg(s) −2.372

Na
+
(aq) + e

⟶ Na(s) −2.71

Ca
2+
(aq) + 2 e

⟶ Ca(s) −2.868

Ba
2+
(aq) + 2 e

⟶ Ba(s) −2.912

K
+
(aq) + e

⟶ K(s) −2.931

Li
+
(aq) + e

⟶ Li(s) −3.04

Tables like this make it possible to determine the standard cell potential for many oxidation-reduction reactions.

 Example 17.3.1: Cell Potentials from Standard Reduction Potentials

What is the standard cell potential for a galvanic cell that consists of Au3+/Au and Ni2+/Ni half-cells? Identify the oxidizing
and reducing agents.

Solution
Using Table 17.3.1, the reactions involved in the galvanic cell, both written as reductions, are
3+ − ∘
Au (aq) + 3 e ⟶ Au(s) E 3+
= +1.498 V
Au /Au

2+ − ∘
Ni (aq) + 2 e ⟶ Ni(s) E 2+
= −0.257 V
Ni /Ni

Galvanic cells have positive cell potentials, and all the reduction reactions are reversible. The reaction at the anode will be the
half-reaction with the smaller or more negative standard reduction potential. Reversing the reaction at the anode (to show the
oxidation) but not its standard reduction potential gives:
2+ − ∘ ∘
Anode (oxidation): Ni(s) ⟶ Ni (aq) + 2 e E =E 2+
= −0.257 V
anode Ni /Ni

3+ − ∘ ∘
Cathode (reduction): Au (aq) + 3 e ⟶ Au(s) E =E 3+
= +1.498 V
cathode Au /Au

The least common factor is six, so the overall reaction is


3+ 2+
3 Ni(s) + 2 Au (aq) ⟶ 3 Ni (aq) + 2 Au(s)

The reduction potentials are not scaled by the stoichiometric coefficients when calculating the cell potential, and the unmodified
standard reduction potentials must be used.
∘ ∘ ∘
E =E −E = 1.498 V − (−0.257 V) = 1.755 V
cell cathode anode

From the half-reactions, Ni is oxidized, so it is the reducing agent, and Au3+ is reduced, so it is the oxidizing agent.

Access for free at OpenStax 17.3.4 https://chem.libretexts.org/@go/page/38305


 Exercise 17.3.1

A galvanic cell consists of a Mg electrode in 1 M Mg(NO3)2 solution and a Ag electrode in 1 M AgNO3 solution. Calculate the
standard cell potential at 25 °C.

Answer
+ 2+ ∘
Mg(s) + 2 Ag (aq) ⟶ Mg (aq) + 2 Ag(s) E = 0.7996 V − (−2.372 V) = 3.172 V
cell

Summary
Assigning the potential of the standard hydrogen electrode (SHE) as zero volts allows the determination of standard reduction
potentials, E°, for half-reactions in electrochemical cells. As the name implies, standard reduction potentials use standard states (1
bar or 1 atm for gases; 1 M for solutes, often at 298.15 K) and are written as reductions (where electrons appear on the left side of
the equation). The reduction reactions are reversible, so standard cell potentials can be calculated by subtracting the standard
reduction potential for the reaction at the anode from the standard reduction for the reaction at the cathode. When calculating the
standard cell potential, the standard reduction potentials are not scaled by the stoichiometric coefficients in the balanced overall
equation.

17.3.1: Key Equations


∘ ∘ ∘
E =E −E
cell cathode anode

Glossary
standard cell potential (E ∘
cell
)

the cell potential when all reactants and products are in their standard states (1 bar or 1 atm or gases; 1 M for solutes), usually at
298.15 K; can be calculated by subtracting the standard reduction potential for the half-reaction at the anode from the standard
reduction potential for the half-reaction occurring at the cathode

standard hydrogen electrode (SHE)


the electrode consists of hydrogen gas bubbling through hydrochloric acid over an inert platinum electrode whose reduction at
standard conditions is assigned a value of 0 V; the reference point for standard reduction potentials

standard reduction potential (E°)


the value of the reduction under standard conditions (1 bar or 1 atm for gases; 1 M for solutes) usually at 298.15 K; tabulated
values used to calculate standard cell potentials

This page titled 17.3: Standard Reduction Potentials is shared under a CC BY 4.0 license and was authored, remixed, and/or curated by OpenStax
via source content that was edited to the style and standards of the LibreTexts platform; a detailed edit history is available upon request.

Access for free at OpenStax 17.3.5 https://chem.libretexts.org/@go/page/38305


17.4: The Nernst Equation
 Learning Objectives
Relate cell potentials to free energy changes
Use the Nernst equation to determine cell potentials at nonstandard conditions
Perform calculations that involve converting between cell potentials, free energy changes, and equilibrium constants

We will now extend electrochemistry by determining the relationship between E and the thermodynamics quantities such as

cell

ΔG° (Gibbs free energy) and K (the equilibrium constant). In galvanic cells, chemical energy is converted into electrical energy,
which can do work. The electrical work is the product of the charge transferred multiplied by the potential difference (voltage):

electrical work = volts × (charge in coulombs) = J

The charge on 1 mole of electrons is given by Faraday’s constant (F)


23 − −19
6.022 × 10 e 1.602 × 10 C
F = ×

mol e

C
4
= 9.648 × 10
mol

4
J
= 9.648 × 10
V ⋅ mol

Therefore

total charge = (number of moles of e ) × F = nF

In this equation, n is the number of moles of electrons for the balanced oxidation-reduction reaction. The measured cell potential is
the maximum potential the cell can produce and is related to the electrical work (wele) by
−wele
Ecell = or wele = −nF Ecell
nF

The negative sign for the work indicates that the electrical work is done by the system (the galvanic cell) on the surroundings. In an
earlier chapter, the free energy was defined as the energy that was available to do work. In particular, the change in free energy was
defined in terms of the maximum work (w max ), which, for electrochemical systems, is w . ele

ΔG = wmax = wele

= −nF Ecell

We can verify the signs are correct when we realize that n and F are positive constants and that galvanic cells, which have positive
cell potentials, involve spontaneous reactions. Thus, spontaneous reactions, which have ΔG < 0 , must have E > 0 . If all the cell

reactants and products are in their standard states, this becomes



ΔG° = −nF E
cell

This provides a way to relate standard cell potentials to equilibrium constants, since

ΔG° = −RT ln K


−nF E = −RT ln K
cell

or
RT

E = ln K
cell
nF

Most of the time, the electrochemical reactions are run at standard temperature (298.15 K). Collecting terms at this temperature
yields

Access for free at OpenStax 17.4.1 https://chem.libretexts.org/@go/page/38306


RT

E = ln K
cell
nF

J
(8.314 ) (298.15 K)
K⋅mol
= ln K
n × 96, 485 C/V⋅mol

0.0257 V
= ln K
n

where n is the number of moles of electrons. The logarithm in equations involving cell potentials is often expressed using base 10
logarithms (i.e., log or just log), which changes the constant by a factor of 2.303:
10

0.0592 V

E = log K
cell
n

Thus, if ΔG°, K, or E is known or can be calculated, the other two quantities can be readily determined. The relationships are

cell

shown graphically in Figure 17.4.1.

Figure 17.4.1 : The relationships between ΔG°, K, and E . Given any one of the three quantities, the other two can be calculated,

cell

so any of the quantities could be used to determine whether a process was spontaneous.
A diagram is shown that involves three double headed arrows positioned in the shape of an equilateral triangle. The vertices are
labeled in red. The top vertex is labeled “K.“ The vertex at the lower left is labeled “delta G superscript degree symbol.” The vertex
at the lower right is labeled “E superscript degree symbol subscript cell.” The right side of the triangle is labeled “E superscript
degree symbol subscript cell equals ( R T divided by n F ) l n K.” The lower side of the triangle is labeled “delta G superscript
degree symbol equals negative n F E superscript degree symbol subscript cell.” The left side of the triangle is labeled “delta G
superscript degree symbol equals negative R T l n K.”
Given any one of the quantities, the other two can be calculated.

 Example 17.4.1: Equilibrium Constants, Potentials, & Free Energy Changes

What is the standard Gibbs free energy change and equilibrium constant for the following reaction at 25 °C?
+ 2+
2 Ag (aq) + Fe(s) ⇌ 2 Ag(s) + Fe (aq)

Solution
The reaction involves an oxidation-reduction reaction, so the standard cell potential can be calculated using the data in Table
P1.
2+ − ∘
anode (oxidation): Fe(s) ⟶ Fe (aq) + 2 e E 2+
= −0.447 V
Fe /Fe

+ − ∘
cathode (reduction): 2 × (Ag (aq) + e ⟶ Ag(s)) E +
= 0.7996 V
Ag /Ag

∘ ∘ ∘ ∘ ∘
E =E −E =E +
−E 2+
= +1.247 V
cell cathode anode Ag /Ag Fe /Fe

Access for free at OpenStax 17.4.2 https://chem.libretexts.org/@go/page/38306


Remember that the cell potential for the cathode is not multiplied by two when determining the standard cell potential. With n
= 2, the equilibrium constant is then
0.0592 V

E = log K
cell
n


n×Ecell /0.0592 V
K = 10

2×1.247 V/0.0592 V
= 10

42.128
= 10

42
= 1.3 × 10

The standard free energy is then



ΔG° = −nF E
cell

J kJ
ΔG° = −2 × 96, 485 × 1.247 V = −240.6
V⋅mol mol

Check your answer: A positive standard cell potential means a spontaneous reaction, so the standard free energy change should
be negative, and an equilibrium constant should be >1.

 Exercise 17.4.1

What is the standard Gibbs free energy change and the equilibrium constant for the following reaction at room temperature? Is
the reaction spontaneous?
2+ 2+ +
Sn(s) + 2 Cu (aq) ⇌ Sn (aq) + 2 Cu (aq)

Answer
kJ
Spontaneous; n = 2; E ∘
cell
;
= +0.291 V ΔG° = −56.2 ; K = 6.8 × 10 . 9

mol

Now that the connection has been made between the free energy and cell potentials, nonstandard concentrations follow. Recall that

ΔG = ΔG° + RT ln Q

where Q is the reaction quotient (see the chapter on equilibrium fundamentals). Converting to cell potentials:

−nF Ecell = −nF E + RT ln Q (17.4.1)
cell

or
RT

Ecell = E − ln Q (17.4.2)
cell
nF

Equation 17.4.2 is the generalized Nernst equation that is applicable at any temperature. However, is can be simplified for reactions
occuring at 25 °C (298.15 K) by rewriting it as
0.0257 V

Ecell = E − ln Q (17.4.3)
cell
n

or
0.0592 V

Ecell = E − log10 Q (17.4.4)
cell
n

If the temperature is not 298.15 K, it is necessary to recalculate the potential with Equation17.4.2. With the Nernst equation, it is
possible to calculate the cell potential at nonstandard conditions. This adjustment is necessary because potentials determined under
different conditions will have different values.

Access for free at OpenStax 17.4.3 https://chem.libretexts.org/@go/page/38306


 Example 17.4.2: Cell Potentials at Nonstandard Conditions
Consider the following reaction at room temperature:
2+ 2+
Co(s) + Fe (aq, 1.94 M ) ⟶ Co (aq, 0.15 M ) + Fe(s)

Is the process spontaneous?

Solution
There are two ways to solve the problem. If the thermodynamic information in Table T1 were available, you could calculate
the free energy change. If the free energy change is negative, the process is spontaneous. The other approach, which we will
use, requires information like that given in Table P1. Using those data, the cell potential can be determined. If the cell potential
is positive, the process is spontaneous. Collecting information from Table P1 and the problem,
2+ − ∘
Anode (oxidation): Co(s) ⟶ Co (aq) + 2 e E 2+
= −0.28 V
Co /Co

2+ − ∘
Cathode (reduction): Fe (aq) + 2 e ⟶ Fe(s) E 2+
= −0.447 V
Fe /Fe

∘ ∘ ∘
E =E −E = −0.447 V − (−0.28 V) = −0.17 V
cell cathode anode

The process is not spontaneous under standard conditions. Using the Nernst equation and the concentrations stated in the
problem and n = 2 ,
2+
[ Co ] 0.15 M
Q = = = 0.077
2+
[ Fe ] 1.94 M

Now we can insert these into the Nernst Equation at room temperature (Equation 17.4.4)
0.0592 V

Ecell = E − log Q
cell
n

0.0592 V
= −0.17 V − log 0.077
2

= −0.17 V + 0.033 V = −0.14 V

The process is (still) nonspontaneous.

 Exercise 17.4.2

What is the cell potential for the following reaction at room temperature?
3+ 2+
│Al
Al(s) (aq, 0.15 M ) ║Cu (aq, 0.025 M ) │Cu(s)

What are the values of n and Q for the overall reaction? Is the reaction spontaneous under these conditions?

Answer
n = 6; Q = 1440; Ecell = +1.97 V, spontaneous.

Finally, we will take a brief look at a special type of cell called a concentration cell. In a concentration cell, the electrodes are the
same material and the half-cells differ only in concentration. Since one or both compartments is not standard, the cell potentials
will be unequal; therefore, there will be a potential difference, which can be determined with the aid of the Nernst equation.

 Example 17.4.3: Concentration Cells

What is the cell potential of the concentration cell described by

Zn(s) │Zn2+ (aq, 0.10 M ) ║Zn2+ (aq, 0.50 M ) │Zn(s)

Access for free at OpenStax 17.4.4 https://chem.libretexts.org/@go/page/38306


Solution
From the information given:
2+ − ∘
Anode: Zn(s) ⟶ Zn (aq, 0.10 M ) + 2 e E = −0.7618 V
anode

2+ − ∘
Cathode: Zn (aq, 0.50 M ) + 2 e ⟶ Zn(s) E = −0.7618 V
cathode

¯¯¯¯¯¯¯¯¯¯¯¯¯¯¯¯¯¯¯¯¯¯¯¯¯¯¯¯¯¯¯¯¯¯¯¯¯¯¯¯¯¯¯¯¯¯¯¯¯¯¯¯¯¯¯¯¯¯¯¯¯¯¯¯¯¯¯¯¯¯¯¯¯¯¯¯¯¯¯¯¯¯¯¯¯¯¯¯¯¯¯¯¯¯¯¯¯¯¯¯¯¯¯¯¯¯¯¯¯¯¯¯¯¯¯¯¯¯¯¯¯¯¯¯¯¯¯¯¯¯¯¯¯¯¯¯¯¯¯¯¯¯¯¯
¯
2+ 2+ ∘
Overall: Zn (aq, 0.50 M ) ⟶ Zn (aq, 0.10 M ) E = 0.000 V
cell

The standard cell potential is zero because the anode and cathode involve the same reaction; only the concentration of Zn2+
changes. Substituting into the Nernst equation,
0.0592 V 0.10
Ecell = 0.000 V − log = +0.021 V
2 0.50

and the process is spontaneous at these conditions.


Check your answer: In a concentration cell, the standard cell potential will always be zero. To get a positive cell potential
(spontaneous process) the reaction quotient Q must be <1. Q < 1 in this case, so the process is spontaneous.

 Exercise 17.4.3

What value of Q for the previous concentration cell would result in a voltage of 0.10 V? If the concentration of zinc ion at the
cathode was 0.50 M, what was the concentration at the anode?

Answer
Q = 0.00042; [Zn2+]cat = 2.1 \times 10−4 M.

Summary
Electrical work (wele) is the negative of the product of the total charge (Q) and the cell potential (Ecell). The total charge can be
calculated as the number of moles of electrons (n) times the Faraday constant (F = 96,485 C/mol e−). Electrical work is the
maximum work that the system can produce and so is equal to the change in free energy. Thus, anything that can be done with or to
a free energy change can also be done to or with a cell potential. The Nernst equation relates the cell potential at nonstandard
conditions to the logarithm of the reaction quotient. Concentration cells exploit this relationship and produce a positive cell
potential using half-cells that differ only in the concentration of their solutes.

17.4.1: Key Equations


RT

E = ln K
cell
nF
0.0257 V 0.0592 V

E = ln K = log K (at 298.15 K)
cell
n n
RT

Ecell = E − ln Q (Nernst equation)
cell
nF
0.0257 V 0.0592 V
∘ ∘
Ecell = E − ln Q = E − log Q (at 298.15 K)
cell cell
n n
ΔG = −nFEcell
∘ ∘
ΔG = −nF E
cell

wele = wmax = −nF Ecell

Glossary
concentration cell
galvanic cell in which the two half-cells are the same except for the concentration of the solutes; spontaneous when the overall
reaction is the dilution of the solute

electrical work (wele)


negative of total charge times the cell potential; equal to wmax for the system, and so equals the free energy change (ΔG)

Access for free at OpenStax 17.4.5 https://chem.libretexts.org/@go/page/38306


Faraday’s constant (F)
charge on 1 mol of electrons; F = 96,485 C/mol e−

Nernst equation
equation that relates the logarithm of the reaction quotient (Q) to nonstandard cell potentials; can be used to relate equilibrium
constants to standard cell potentials

This page titled 17.4: The Nernst Equation is shared under a CC BY 4.0 license and was authored, remixed, and/or curated by OpenStax via
source content that was edited to the style and standards of the LibreTexts platform; a detailed edit history is available upon request.

Access for free at OpenStax 17.4.6 https://chem.libretexts.org/@go/page/38306


17.5: Batteries and Fuel Cells
 Learning Objectives
Classify batteries as primary or secondary
List some of the characteristics and limitations of batteries
Provide a general description of a fuel cell

A battery is an electrochemical cell or series of cells that produces an electric current. In principle, any galvanic cell could be used
as a battery. An ideal battery would never run down, produce an unchanging voltage, and be capable of withstanding environmental
extremes of heat and humidity. Real batteries strike a balance between ideal characteristics and practical limitations. For example,
the mass of a car battery is about 18 kg or about 1% of the mass of an average car or light-duty truck. This type of battery would
supply nearly unlimited energy if used in a smartphone, but would be rejected for this application because of its mass. Thus, no
single battery is “best” and batteries are selected for a particular application, keeping things like the mass of the battery, its cost,
reliability, and current capacity in mind. There are two basic types of batteries: primary and secondary. A few batteries of each type
are described next.

Visit this site to learn more about batteries.


17.5.0.0.1: Primary Batteries
Primary batteries are single-use batteries because they cannot be recharged. A common primary battery is the dry cell (Figure
17.5.1). The dry cell is a zinc-carbon battery. The zinc can serves as both a container and the negative electrode. The positive

electrode is a rod made of carbon that is surrounded by a paste of manganese(IV) oxide, zinc chloride, ammonium chloride, carbon
powder, and a small amount of water. The reaction at the anode can be represented as the ordinary oxidation of zinc:
2+ − ∘
Zn(s) ⟶ Zn (aq) + 2 e E 2+
= −0.7618 V
Zn /Zn

The reaction at the cathode is more complicated, in part because more than one reaction occurs. The series of reactions that occurs
at the cathode is approximately
− −
2 MnO (s) + 2 NH Cl(aq) + 2 e ⟶ Mn O (s) + 2 NH (aq) + H O(l) + 2 Cl
2 4 2 3 3 2

The overall reaction for the zinc–carbon battery can be represented as

2+ −
2 MnO (s) + 2 NH Cl(aq) + Zn(s) ⟶ Zn (aq) + Mn O (s) + 2 NH (aq) + H O(l) + 2 Cl
2 4 2 3 3 2

with an overall cell potential which is initially about 1.5 V, but decreases as the battery is used. It is important to remember that the
voltage delivered by a battery is the same regardless of the size of a battery. For this reason, D, C, A, AA, and AAA batteries all
have the same voltage rating. However, larger batteries can deliver more moles of electrons. As the zinc container oxidizes, its
contents eventually leak out, so this type of battery should not be left in any electrical device for extended periods.

Access for free at OpenStax 17.5.1 https://chem.libretexts.org/@go/page/38307


Figure 17.5.1 : The diagram shows a cross section of a flashlight battery, a zinc-carbon dry cell.
A diagram of a cross section of a dry cell battery is shown. The overall shape of the cell is cylindrical. The lateral surface of the
cylinder, indicated as a thin red line, is labeled “zinc can (electrode).” Just beneath this is a slightly thicker dark grey surface that
covers the lateral surface, top, and bottom of the battery, which is labeled “Porous separator.” Inside is a purple region with many
evenly spaced small darker purple dots, labeled “Paste of M n O subscript 2, N H subscript 4 C l, Z n C l subscript 2, water
(cathode).” A dark grey rod, labeled “Carbon rod (electrode),” extends from the top of the battery, leaving a gap of less than one-
fifth the height of the battery below the rod to the bottom of the cylinder. A thin grey line segment at the very bottom of the
cylinder is labeled “Metal bottom cover (negative).” The very top of the cylinder has a thin grey surface that curves upward at the
center over the top of the carbon electrode at the center of the cylinder. This upper surface is labeled “Metal top cover (positive).”
A thin dark grey line just below this surface is labeled “Insulator.” Below this, above the purple region, and outside of the carbon
electrode at the center is an orange region that is labeled “Seal.”

Alkaline batteries (Figure 17.5.2) were developed in the 1950s partly to address some of the
performance issues with zinc–carbon dry cells. They are manufactured to be exact replacements for zinc-
carbon dry cells. As their name suggests, these types of batteries use alkaline electrolytes, often
potassium hydroxide. The reactions are
− − ∘
anode: Zn(s) + 2 OH (aq) ⟶ ZnO(s) + H O(l) + 2 e E = −1.28 V
2 anode

− − ∘
cathode: 2 MnO (s) + H O(l) + 2 e ⟶ Mn O (s) + 2 OH (aq) E = +0.15 V
2 2 2 3 cathode
––––––––––––––––––––––––––––––––––––––––––––––––––––––––––––––––––––––––––––––––––––––

overall: Zn(s) + 2 MnO (s) ⟶ ZnO(s) + Mn O (s) E = +1.43 V
2 2 3 cell

An alkaline battery can deliver about three to five times the energy of a zinc-carbon dry cell of similar size. Alkaline batteries are
prone to leaking potassium hydroxide, so these should also be removed from devices for long-term storage. While some alkaline
batteries are rechargeable, most are not. Attempts to recharge an alkaline battery that is not rechargeable often leads to rupture of
the battery and leakage of the potassium hydroxide electrolyte.

Access for free at OpenStax 17.5.2 https://chem.libretexts.org/@go/page/38307


Figure 17.5.2 : Alkaline batteries were designed as direct replacements for zinc-carbon (dry cell) batteries.
17.5.0.0.1: Secondary Batteries
Secondary batteries are rechargeable. These are the types of batteries found in devices such as smartphones, electronic tablets, and
automobiles.
Nickel-cadmium, or NiCd, batteries (Figure 17.5.3) consist of a nickel-plated cathode, cadmium-plated anode, and a potassium
hydroxide electrode. The positive and negative plates, which are prevented from shorting by the separator, are rolled together and
put into the case. This is a “jelly-roll” design and allows the NiCd cell to deliver much more current than a similar-sized alkaline
battery. The reactions are
− −
anode: Cd(s) + 2 OH (aq) ⟶ Cd(OH) (s) + 2 e
2

− −
cathode: NiO (s) + 2 H O(l) + 2 e ⟶ Ni (OH) (s) + 2 OH (aq)
2 2 2
–––––––––––––––––––––––––––––––––––––––––––––––––––––––––––––––
overall: Cd(s) + NiO (s) + 2 H O(l) ⟶ Cd(OH) (s) + Ni (OH) (s)
2 2 2 2

The voltage is about 1.2 V to 1.25 V as the battery discharges. When properly treated, a NiCd battery can be recharged about 1000
times. Cadmium is a toxic heavy metal so NiCd batteries should never be opened or put into the regular trash.

Figure 17.5.3 : NiCd batteries use a “jelly-roll” design that significantly increases the amount of current the battery can deliver as
compared to a similar-sized alkaline battery.
Lithium ion batteries (Figure 17.5.4) are among the most popular rechargeable batteries and are used in many portable electronic
devices. The reactions are
+ −
anode: LiCoO ⇌ Li 1−x CoO + x Li + xe
2 2

+ −
cathode: x Li + xe + xC ⇌ x LiC
6 6

¯¯¯¯¯¯¯¯¯¯¯¯¯¯¯¯¯¯¯¯¯¯¯¯¯¯¯¯¯¯¯¯¯¯¯¯¯¯¯¯¯¯¯¯¯¯¯¯¯¯¯¯¯¯¯¯¯¯¯¯¯¯¯¯¯¯¯¯¯¯¯¯¯¯¯¯¯¯¯¯¯¯¯¯¯¯¯¯¯¯¯¯¯¯¯¯
¯
overall: LiCoO + xC ⇌ Li 1−x CoO + x LiC
2 6 2 6

Access for free at OpenStax 17.5.3 https://chem.libretexts.org/@go/page/38307


With the coefficients representing moles, x is no more than about 0.5 moles. The battery voltage is about 3.7 V. Lithium batteries
are popular because they can provide a large amount current, are lighter than comparable batteries of other types, produce a nearly
constant voltage as they discharge, and only slowly lose their charge when stored.

Figure 17.5.4 : In a lithium ion battery, charge flows between the electrodes as the lithium ions move between the anode and
cathode.
The lead acid battery (Figure 17.5.5) is the type of secondary battery used in your automobile. It is inexpensive and capable of
producing the high current required by automobile starter motors. The reactions for a lead acid battery are
− + −
anode: Pb(s) + HSO (aq) ⟶ PbSO (s) + H (aq) + 2 e
4 4

− + −
cathode: PbO (s) + HSO 4 (aq) + 3 H (aq) + 2 e ⟶ PbSO (s) + 2 H O(l)
2 4 2
–––––––––––––––––––––––––––––––––––––––––––––––––––––––––––––––––––––––––
overall: Pb(s) + PbO (s) + 2 H SO (aq) ⟶ 2 PbSO (s) + 2 H O(l)
2 2 4 4 2

Each cell produces 2 V, so six cells are connected in series to produce a 12-V car battery. Lead acid batteries are heavy and contain
a caustic liquid electrolyte, but are often still the battery of choice because of their high current density. Since these batteries
contain a significant amount of lead, they must always be disposed of properly.

Figure 17.5.5 : The lead acid battery in your automobile consists of six cells connected in series to give 12 V. Their low cost and
high current output makes these excellent candidates for providing power for automobile starter motors.

17.5.1: Fuel Cells


A fuel cell is a device that converts chemical energy into electrical energy. Fuel cells are similar to batteries but require a
continuous source of fuel, often hydrogen. They will continue to produce electricity as long as fuel is available. Hydrogen fuel cells
have been used to supply power for satellites, space capsules, automobiles, boats, and submarines (Figure 17.5.6).

Access for free at OpenStax 17.5.4 https://chem.libretexts.org/@go/page/38307


Figure 17.5.6 : In this hydrogen fuel-cell schematic, oxygen from the air reacts with hydrogen, producing water and electricity.
In a hydrogen fuel cell, the reactions are
2− −
anode: 2 H +2 O ⟶ 2 H O+4 e
2 2

− 2−
cathode: O +4 e ⟶ 2O
2
–––––––––––––––––––––––––––––––––––
overall: 2 H +O ⟶ 2H O
2 2 2

The voltage is about 0.9 V. The efficiency of fuel cells is typically about 40% to 60%, which is higher than the typical internal
combustion engine (25% to 35%) and, in the case of the hydrogen fuel cell, produces only water as exhaust. Currently, fuel cells
are rather expensive and contain features that cause them to fail after a relatively short time.

Summary
Batteries are galvanic cells, or a series of cells, that produce an electric current. When cells are combined into batteries, the
potential of the battery is an integer multiple of the potential of a single cell. There are two basic types of batteries: primary and
secondary. Primary batteries are “single use” and cannot be recharged. Dry cells and (most) alkaline batteries are examples of
primary batteries. The second type is rechargeable and is called a secondary battery. Examples of secondary batteries include
nickel-cadmium (NiCd), lead acid, and lithium ion batteries. Fuel cells are similar to batteries in that they generate an electrical
current, but require continuous addition of fuel and oxidizer. The hydrogen fuel cell uses hydrogen and oxygen from the air to
produce water, and is generally more efficient than internal combustion engines.

Summary
alkaline battery
primary battery that uses an alkaline (often potassium hydroxide) electrolyte; designed to be an exact replacement for the dry
cell, but with more energy storage and less electrolyte leakage than typical dry cell

battery
galvanic cell or series of cells that produces a current; in theory, any galvanic cell

dry cell
primary battery, also called a zinc-carbon battery; can be used in any orientation because it uses a paste as the electrolyte; tends
to leak electrolyte when stored

fuel cell
devices that produce an electrical current as long as fuel and oxidizer are continuously added; more efficient than internal
combustion engines

lead acid battery

Access for free at OpenStax 17.5.5 https://chem.libretexts.org/@go/page/38307


secondary battery that consists of multiple cells; the lead acid battery found in automobiles has six cells and a voltage of 12 V

lithium ion battery


very popular secondary battery; uses lithium ions to conduct current and is light, rechargeable, and produces a nearly constant
potential as it discharges

nickel-cadmium battery
(NiCd battery) secondary battery that uses cadmium, which is a toxic heavy metal; heavier than lithium ion batteries, but with
similar performance characteristics

primary battery
single-use nonrechargeable battery

secondary battery
battery that can be recharged

This page titled 17.5: Batteries and Fuel Cells is shared under a CC BY 4.0 license and was authored, remixed, and/or curated by OpenStax via
source content that was edited to the style and standards of the LibreTexts platform; a detailed edit history is available upon request.

Access for free at OpenStax 17.5.6 https://chem.libretexts.org/@go/page/38307


17.6: Corrosion
 Learning Objectives
Define corrosion
List some of the methods used to prevent or slow corrosion

Corrosion is usually defined as the degradation of metals due to an electrochemical process. The formation of rust on iron, tarnish
on silver, and the blue-green patina that develops on copper are all examples of corrosion. The total cost of corrosion in the United
States is significant, with estimates in excess of half a trillion dollars a year.

 Changing Colors

The Statue of Liberty is a landmark every American recognizes. The Statue of Liberty is easily identified by its height, stance,
and unique blue-green color. When this statue was first delivered from France, its appearance was not green. It was brown, the
color of its copper “skin.” So how did the Statue of Liberty change colors? The change in appearance was a direct result of
corrosion. The copper that is the primary component of the statue slowly underwent oxidation from the air. The oxidation-
reduction reactions of copper metal in the environment occur in several steps. Copper metal is oxidized to copper(I) oxide (
Cu O), which is red, and then to copper(II) oxide, which is black.
2

1
2 Cu(s) + O (g) → Cu O(s)
2 2 2
red

1
Cu O(s) + O (g) → 2 CuO(s)
2 2 2
black

Coal, which was often high in sulfur, was burned extensively in the early part of the last century. As a result, sulfur trioxide,
carbon dioxide, and water all reacted with the CuO.

2 CuO(s) + CO (g) + H O(l) → Cu CO (OH) (s)


2 2 2 3 2
green

3 CuO(s) + 2 CO (g) + H O(l) → Cu (CO ) (OH) (s)


2 2 2 3 2 2
blue

4 CuO(s) + SO (g) + 3 H O(l) → Cu SO (OH) (s)


3 2 4 4 6
green

These three compounds are responsible for the characteristic blue-green patina seen today. Fortunately, formation of the patina
created a protective layer on the surface, preventing further corrosion of the copper skin. The formation of the protective layer
is a form of passivation, which is discussed further in a later chapter.
This figure contains two photos of the Statue of Liberty. Photo a appears to be an antique photo which shows the original brown color of the copper covered statue. Photo b shows the blue-green appearance of
the statue today. In both photos, the statue is shown atop a building, with a body of water in the background."

Figure 17.6.1 : (a) The Statue of Liberty is covered with a copper skin, and was originally brown, as shown in this painting. (b)
Exposure to the elements has resulted in the formation of the blue-green patina seen today.

Perhaps the most familiar example of corrosion is the formation of rust on iron. Iron will rust when it is exposed to oxygen and
water. The main steps in the rusting of iron appear to involve the following. Once exposed to the atmosphere, iron rapidly oxidizes.
2 + − ∘
anode: Fe(s) → Fe +2 e E 2 +
= −0.44 V
(aq) Fe /Fe

The electrons reduce oxygen in the air in acidic solutions.


+ − ∘
cathode: O2(g) + 4 H +4 e → 2 H O(l) E = +1.23 V
(aq) 2 O /O
2 2

+ 2 + ∘
overall: 2 Fe(s) + O2(g) + 4 H → 2 Fe + 2 H O(l) E = +1.67 V
(aq) (aq) 2 cell

What we call rust is hydrated iron(III) oxide, which forms when iron(II) ions react further with oxygen.
2 + +
4 Fe + O2(g) + (4 + 2x)H O(l) → 2 Fe O ⋅ x H O(s) + 8 H
(aq) 2 2 3 2 (aq)

Access for free at OpenStax 17.6.1 https://chem.libretexts.org/@go/page/38308


The number of water molecules is variable, so it is represented by x. Unlike the patina on copper, the formation of rust does not
create a protective layer and so corrosion of the iron continues as the rust flakes off and exposes fresh iron to the atmosphere.

Figure 17.6.2 : Once the paint is scratched on a painted iron surface, corrosion occurs and rust begins to form. The speed of the
spontaneous reaction is increased in the presence of electrolytes, such as the sodium chloride used on roads to melt ice and snow or
in salt water.
A grey rectangle, labeled &#8220;iron,&#8221; is shown with thin purple layers, labeled &#8220;Paint layer,&#8221; at its upper
and lower surfaces. A gap in the upper purple layer at the upper left of the diagram is labeled &#8220;Cathodic site.&#8221; A
blue droplet labeled &#8220;water&#8221; is positioned on top of the gap. A curved arrow extends from a space above the droplet
to the surface of the grey region and into the water droplet. The base of the arrow is labeled &#8220;O subscript 2&#8221; and the
tip of the arrow is labeled &#8220;H subscript 2 O.&#8221; A gap to the right and on the bottom side of the grey region shows that
some of the grey region is gone from the region beneath the purple layer. A water droplet covers this gap and extends into the open
space in the grey rectangle. The label &#8220;F e superscript 2 positive&#8221; is at the center of the droplet. A curved arrow
points from the edge of the grey area below to the label. A second curved arrow extends from the F e superscript 2 positive arrow
to a rust brown chunk on the lower surface of the purple layer at the edge of the water droplet. A curved arrow extends from O
subscript 2 outside the droplet into the droplet to the rust brown chunk. The grey region at the lower right portion of the diagram is
labeled &#8220;Anodic site.&#8221; An arrow extends from the anodic site toward the cathodic site, which is labeled &#8220;e
superscript negative.&#8221;
One way to keep iron from corroding is to keep it painted. The layer of paint prevents the water and oxygen necessary for rust
formation from coming into contact with the iron. As long as the paint remains intact, the iron is protected from corrosion.
Other strategies include alloying the iron with other metals. For example, stainless steel is mostly iron with a bit of chromium. The
chromium tends to collect near the surface, where it forms an oxide layer that protects the iron.
Zinc-plated or galvanized iron uses a different strategy. Zinc is more easily oxidized than iron because zinc has a lower reduction
potential. Since zinc has a lower reduction potential, it is a more active metal. Thus, even if the zinc coating is scratched, the zinc
will still oxidize before the iron. This suggests that this approach should work with other active metals.

Figure 17.6.3 : One way to protect an underground iron storage tank is through cathodic protection. Using an active metal like zinc
or magnesium for the anode effectively makes the storage tank the cathode, preventing it from corroding (oxidizing).
A diagram is shown of an underground storage tank system. Underground, to the left end of the diagram is a horizontal grey tank
which is labeled “Object to be protected.” A black line extends upward from the center of the tank above ground. An arrow points
upward at the left side of the line segment. A horizontal black line segment continues right above ground, which is labeled “No
power source is needed.” A line segment extends up and to the right, which is labeled “ Lead wire.” A line segment with a
downward pointing arrow to its right extends downward below the ground to a second metal tank-like structure, labeled “Sacrificial
anode” which is vertically oriented. Five black arrows point left underground toward the first tank. These arrows are collectively
labeled “Protective current.”
Another important way to protect metal is to make it the cathode in a galvanic cell. This is cathodic protection and can be used for
metals other than just iron. For example, the rusting of underground iron storage tanks and pipes can be prevented or greatly
reduced by connecting them to a more active metal such as zinc or magnesium. This is also used to protect the metal parts in water
heaters. The more active metals (lower reduction potential) are called sacrificial anodes because as they get used up as they corrode

Access for free at OpenStax 17.6.2 https://chem.libretexts.org/@go/page/38308


(oxidize) at the anode. The metal being protected serves as the cathode, and so does not oxidize (corrode). When the anodes are
properly monitored and periodically replaced, the useful lifetime of the iron storage tank can be greatly extended.

Summary
Corrosion is the degradation of a metal caused by an electrochemical process. Large sums of money are spent each year repairing
the effects of, or preventing, corrosion. Some metals, such as aluminum and copper, produce a protective layer when they corrode
in air. The thin layer that forms on the surface of the metal prevents oxygen from coming into contact with more of the metal atoms
and thus “protects” the remaining metal from further corrosion. Iron corrodes (forms rust) when exposed to water and oxygen. The
rust that forms on iron metal flakes off, exposing fresh metal, which also corrodes. One way to prevent, or slow, corrosion is by
coating the metal. Coating prevents water and oxygen from contacting the metal. Paint or other coatings will slow corrosion, but
they are not effective once scratched. Zinc-plated or galvanized iron exploits the fact that zinc is more likely to oxidize than iron.
As long as the coating remains, even if scratched, the zinc will oxidize before the iron. Another method for protecting metals is
cathodic protection. In this method, an easily oxidized and inexpensive metal, often zinc or magnesium (the sacrificial anode), is
electrically connected to the metal that must be protected. The more active metal is the sacrificial anode, and is the anode in a
galvanic cell. The “protected” metal is the cathode, and remains unoxidized. One advantage of cathodic protection is that the
sacrificial anode can be monitored and replaced if needed.

Glossary
cathodic protection
method of protecting metal by using a sacrificial anode and effectively making the metal that needs protecting the cathode, thus
preventing its oxidation

corrosion
degradation of metal through an electrochemical process

galvanized iron
method for protecting iron by covering it with zinc, which will oxidize before the iron; zinc-plated iron

sacrificial anode
more active, inexpensive metal used as the anode in cathodic protection; frequently made from magnesium or zinc

This page titled 17.6: Corrosion is shared under a CC BY 4.0 license and was authored, remixed, and/or curated by OpenStax via source content
that was edited to the style and standards of the LibreTexts platform; a detailed edit history is available upon request.

Access for free at OpenStax 17.6.3 https://chem.libretexts.org/@go/page/38308


17.7: Electrolysis
 Learning Objectives
Describe electrolytic cells and their relationship to galvanic cells
Perform various calculations related to electrolysis

In galvanic cells, chemical energy is converted into electrical energy. The opposite is true for electrolytic cells. In electrolytic cells,
electrical energy causes nonspontaneous reactions to occur in a process known as electrolysis. The charging electric barttery shows
one such process. Electrical energy is converted into the chemical energy in the battery as it is charged. Once charged, the battery
can be used to power the automobile. The same principles are involved in electrolytic cells as in galvanic cells. We will look at
three electrolytic cells and the quantitative aspects of electrolysis.
17.7.0.0.1: The Electrolysis of Molten Sodium Chloride
In molten sodium chloride, the ions are free to migrate to the electrodes of an electrolytic cell. A simplified diagram of the cell
commercially used to produce sodium metal and chlorine gas is shown in Figure 17.7.1. Sodium is a strong reducing agent and
chlorine is used to purify water, and is used in antiseptics and in paper production. The reactions are
− − ∘
anode: 2 Cl (l) ⟶ Cl (g) + 2 e E −
= +1.3 V (17.7.1)
2 Cl / Cl
2

+ − ∘
cathode: Na (l) + e ⟶ Na(l) E +
= −2.7 V (17.7.2)
Na /Na

¯¯¯¯¯¯¯¯¯¯¯¯¯¯¯¯¯¯¯¯¯¯¯¯¯¯¯¯¯¯¯¯¯¯¯¯¯¯¯¯¯¯¯¯¯¯¯¯¯¯¯¯¯¯¯¯¯¯¯¯¯¯¯¯¯¯¯¯¯¯¯¯¯¯¯¯¯¯¯¯¯¯¯¯¯¯¯¯¯¯¯¯¯¯¯¯¯¯¯¯¯¯¯¯¯¯¯¯¯¯¯¯¯¯¯¯¯¯¯¯¯¯¯¯¯¯¯¯¯¯¯¯¯¯¯¯
¯
+ − ∘
overall: 2 Na (l) + 2 Cl (l) ⟶ 2 Na(l) + Cl (g) E = −4.0 V (17.7.3)
2 cell

The power supply (battery) must supply a minimum of 4 V, but, in practice, the applied voltages are typically higher because of
inefficiencies in the process itself.

Figure 17.7.1 : Passing an electric current through molten sodium chloride decomposes the material into sodium metal and chlorine
gas. Care must be taken to keep the products separated to prevent the spontaneous formation of sodium chloride.
17.7.0.0.1: The Electrolysis of Water
It is possible to split water into hydrogen and oxygen gas by electrolysis. Acids are typically added to increase the concentration of
hydrogen ion in solution (Figure 17.7.2). The reactions are
+ − ∘
anode: 2 H O(l) ⟶ O (g) + 4 H (aq) + 4 e E = +1.229 V (17.7.4)
2 2 anode

+ − ∘
cathode: 2 H (aq) + 2 e ⟶ H (g) E =0 V (17.7.5)
2 cathode

¯¯¯¯¯¯¯¯¯¯¯¯¯¯¯¯¯¯¯¯¯¯¯¯¯¯¯¯¯¯¯¯¯¯¯¯¯¯¯¯¯¯¯¯¯¯¯¯¯¯¯¯¯¯¯¯¯¯¯¯¯¯¯¯¯¯¯¯¯¯¯¯¯¯¯¯¯¯¯¯¯¯¯¯¯¯¯¯¯¯¯¯¯¯¯¯¯¯¯¯¯¯¯¯¯¯¯¯¯¯¯¯¯¯¯¯¯¯
¯

overall: 2 H O(l) ⟶ 2 H (g) + O (g) E = −1.229 V (17.7.6)
2 2 2 cell

Note that the sulfuric acid is not consumed and that the volume of hydrogen gas produced is twice the volume of oxygen gas
produced. The minimum applied voltage is 1.229 V.

Access for free at OpenStax 17.7.1 https://chem.libretexts.org/@go/page/38309


Figure 17.7.2 : Water decomposes into oxygen and hydrogen gas during electrolysis. Sulfuric acid was added to increase the
concentration of hydrogen ions and the total number of ions in solution, but does not take part in the reaction. The volume of
hydrogen gas collected is twice the volume of oxygen gas collected, due to the stoichiometry of the reaction.
17.7.0.0.1: The Electrolysis of Aqueous Sodium Chloride
The electrolysis of aqueous sodium chloride is the more common example of electrolysis because more than one species can be
oxidized and reduced. Considering the anode first, the possible reactions are
− − ∘
(i) 2 Cl (aq) ⟶ Cl (g) + 2 e E = +1.35827 V (17.7.7)
2 anode

+ − ∘
(ii) 2 H O(l) ⟶ O (g) + 4 H (aq) + 4 e E = +1.229 V (17.7.8)
2 2 anode

These values suggest that water should be oxidized at the anode because a smaller potential would be needed—using reaction (ii)
for the oxidation would give a less-negative cell potential. When the experiment is run, it turns out chlorine, not oxygen, is
produced at the anode. The unexpected process is so common in electrochemistry that it has been given the name overpotential.
The overpotential is the difference between the theoretical cell voltage and the actual voltage that is necessary to cause electrolysis.
It turns out that the overpotential for oxygen is rather high and effectively makes the reduction potential more positive. As a result,
under normal conditions, chlorine gas is what actually forms at the anode.
Now consider the cathode. Three reductions could occur:
+ − ∘
(iii) 2 H (aq) + 2 e ⟶ H (g) E =0 V (17.7.9)
2 cathode

− − ∘
(iv) 2 H O(l) + 2 e ⟶ H (g) + 2 OH (aq) E = −0.8277 V (17.7.10)
2 2 cathode
+ − ∘
(v) Na (aq) + e ⟶ Na(s) E = −2.71 V (17.7.11)
cathode

Reaction (v) is ruled out because it has such a negative reduction potential. Under standard state conditions, reaction (iii) would be
preferred to reaction (iv). However, the pH of a sodium chloride solution is 7, so the concentration of hydrogen ions is only 1× 10−7
M. At such low concentrations, reaction (iii) is unlikely and reaction (iv) occurs. The overall reaction is then
− − ∘
overall: 2 H O(l) + 2 Cl (aq) ⟶ H (g) + Cl (g) + 2 OH (aq) E = −2.186 V
2 2 2 cell

As the reaction proceeds, hydroxide ions replace chloride ions in solution. Thus, sodium hydroxide can be obtained by evaporating
the water after the electrolysis is complete. Sodium hydroxide is valuable in its own right and is used for things like oven cleaner,
drain opener, and in the production of paper, fabrics, and soap.

Access for free at OpenStax 17.7.2 https://chem.libretexts.org/@go/page/38309


17.7.0.0.1: Electroplating
An important use for electrolytic cells is in electroplating. Electroplating results in a thin coating of one metal on top of a
conducting surface. Reasons for electroplating include making the object more corrosion resistant, strengthening the surface,
producing a more attractive finish, or for purifying metal. The metals commonly used in electroplating include cadmium,
chromium, copper, gold, nickel, silver, and tin. Common consumer products include silver-plated or gold-plated tableware,
chrome-plated automobile parts, and jewelry. We can get an idea of how this works by investigating how silver-plated tableware is
produced (Figure 17.7.3).

Figure 17.7.3 : The spoon, which is made of an inexpensive metal, is connected to the negative terminal of the voltage source and
acts as the cathode. The anode is a silver electrode. Both electrodes are immersed in a silver nitrate solution. When a steady current
is passed through the solution, the net result is that silver metal is removed from the anode and deposited on the cathode.
In the figure, the anode consists of a silver electrode, shown on the left. The cathode is located on the right and is the spoon, which
is made from inexpensive metal. Both electrodes are immersed in a solution of silver nitrate. As the potential is increased, current
flows. Silver metal is lost at the anode as it goes into solution.
+ −
anode: Ag(s) ⟶ Ag (aq) + e

The mass of the cathode increases as silver ions from the solution are deposited onto the spoon
+ −
cathode: Ag (aq) + e ⟶ Ag(s)

The net result is the transfer of silver metal from the anode to the cathode. The quality of the object is usually determined by the
thickness of the deposited silver and the rate of deposition.

17.7.1: Quantitative Aspects of Electrolysis


The amount of current that is allowed to flow in an electrolytic cell is related to the number of moles of electrons. The number of
moles of electrons can be related to the reactants and products using stoichiometry. Recall that the SI unit for current (I) is the
C
ampere (A), which is the equivalent of 1 coulomb per second (1 A = 1 ). The total charge (Q, in coulombs) is given by
s

Q = I ×t = n×F

where
t is the time in seconds,
n the number of moles of electrons, and
F is the Faraday constant.
Moles of electrons can be used in stoichiometry problems. The time required to deposit a specified amount of metal might also be
requested, as in the second of the following examples.

Access for free at OpenStax 17.7.3 https://chem.libretexts.org/@go/page/38309


 Example 17.7.1: Converting Current to Moles of Electrons
In one process used for electroplating silver, a current of 10.23 A was passed through an electrolytic cell for exactly 1 hour.
How many moles of electrons passed through the cell? What mass of silver was deposited at the cathode from the silver nitrate
solution?

Solution
Faraday’s constant can be used to convert the charge (Q) into moles of electrons (n). The charge is the current (I) multiplied by
the time
10.23 C 60 min 60 s
× 1 hr × ×
Q s hr min 36, 830 C

n = = = = 0.3817 mol e
− −
F 96, 485 C/mol e 96, 485 C/mol e

From the problem, the solution contains AgNO3, so the reaction at the cathode involves 1 mole of electrons for each mole of
silver
+ −
cathode: Ag (aq) + e ⟶ Ag(s)

The atomic mass of silver is 107.9 g/mol, so


1 mol Ag 107.9 g Ag

mass Ag = 0.3817 mol e × × = 41.19 g Ag

1 mol e 1 mol Ag

Check your answer: From the stoichiometry, 1 mole of electrons would produce 1 mole of silver. Less than one-half a mole of
electrons was involved and less than one-half a mole of silver was produced.

 Exercise 17.7.1

Aluminum metal can be made from aluminum ions by electrolysis. What is the half-reaction at the cathode? What mass of
aluminum metal would be recovered if a current of 2.50 × 103 A passed through the solution for 15.0 minutes? Assume the
yield is 100%.

Answer
Al
3+ −
(aq) + 3 e ⟶ Al(s) ; 7.77 mol Al = 210.0 g Al.

 Example 17.7.2: Time Required for Deposition

In one application, a 0.010-mm layer of chromium must be deposited on a part with a total surface area of 3.3 m2 from a
solution of containing chromium(III) ions. How long would it take to deposit the layer of chromium if the current was 33.46
A? The density of chromium (metal) is 7.19 g/cm3.
Solution This problem brings in a number of topics covered earlier. An outline of what needs to be done is:
If the total charge can be determined, the time required is just the charge divided by the current
The total charge can be obtained from the amount of Cr needed and the stoichiometry
The amount of Cr can be obtained using the density and the volume Cr required
The volume Cr required is the thickness times the area
Solving in steps, and taking care with the units, the volume of Cr required is
2
1 cm 10, 000 cm
2 3
volume = (0.010 mm × ) × (3.3 m ×( )) = 33 cm
2
10 mm 1 m

Cubic centimeters were used because they match the volume unit used for the density. The amount of Cr is then

Access for free at OpenStax 17.7.4 https://chem.libretexts.org/@go/page/38309


3
7.19 g
mass = volume × density = 33 cm × = 237 g Cr
3
cm

1 mol Cr
mol Cr = 237 g Cr × = 4.56 mol Cr
52.00 g Cr

Since the solution contains chromium(III) ions, 3 moles of electrons are required per mole of Cr. The total charge is then

3 mol e 96485 C
6
Q = 4.56 mol Cr × × = 1.32 × 10 C

1 mol Cr mol e

The time required is then


6
Q 1.32 × 10 C 4
t = = = 3.95 × 10 s = 11.0 hr
I 33.46 C/s

Check your answer: In a long problem like this, a single check is probably not enough. Each of the steps gives a reasonable
number, so things are probably correct. Pay careful attention to unit conversions and the stoichiometry.

 Exercise 17.7.2

What mass of zinc is required to galvanize the top of a 3.00 m × 5.50 m sheet of iron to a thickness of 0.100 mm of zinc? If the
zinc comes from a solution of Zn(NO3)2 and the current is 25.5 A, how long will it take to galvanize the top of the iron? The
density of zinc is 7.140 g/cm3.

Answer
231 g Zn required 23,000 minutes.

Summary
Using electricity to force a nonspontaneous process to occur is electrolysis. Electrolytic cells are electrochemical cells with
negative cell potentials (meaning a positive Gibbs free energy), and so are nonspontaneous. Electrolysis can occur in electrolytic
cells by introducing a power supply, which supplies the energy to force the electrons to flow in the nonspontaneous direction.
Electrolysis is done in solutions, which contain enough ions so current can flow. If the solution contains only one material, like the
electrolysis of molten sodium chloride, it is a simple matter to determine what is oxidized and what is reduced. In more
complicated systems, like the electrolysis of aqueous sodium chloride, more than one species can be oxidized or reduced and the
standard reduction potentials are used to determine the most likely oxidation (the half-reaction with the largest [most positive]
standard reduction potential) and reduction (the half-reaction with the smallest [least positive] standard reduction potential).
Sometimes unexpected half-reactions occur because of overpotential. Overpotential is the difference between the theoretical half-
reaction reduction potential and the actual voltage required. When present, the applied potential must be increased, making it
possible for a different reaction to occur in the electrolytic cell. The total charge, Q, that passes through an electrolytic cell can be
expressed as the current (I) multiplied by time (Q = It) or as the moles of electrons (n) multiplied by Faraday’s constant (Q = nF).
These relationships can be used to determine things like the amount of material used or generated during electrolysis, how long the
reaction must proceed, or what value of the current is required.

Summary
electrolysis
process using electrical energy to cause a nonspontaneous process to occur

electrolytic cell
electrochemical cell in which electrolysis is used; electrochemical cell with negative cell potentials

electroplating
depositing a thin layer of one metal on top of a conducting surface

overpotential

Access for free at OpenStax 17.7.5 https://chem.libretexts.org/@go/page/38309


difference between the theoretical potential and actual potential in an electrolytic cell; the “extra” voltage required to make
some nonspontaneous electrochemical reaction to occur

This page titled 17.7: Electrolysis is shared under a CC BY 4.0 license and was authored, remixed, and/or curated by OpenStax via source content
that was edited to the style and standards of the LibreTexts platform; a detailed edit history is available upon request.

Access for free at OpenStax 17.7.6 https://chem.libretexts.org/@go/page/38309


17.E: Electrochemistry (Exercises)
Migrated to ADAPT

This page titled 17.E: Electrochemistry (Exercises) is shared under a CC BY 4.0 license and was authored, remixed, and/or curated by OpenStax
via source content that was edited to the style and standards of the LibreTexts platform; a detailed edit history is available upon request.

Access for free at OpenStax 17.E.1 https://chem.libretexts.org/@go/page/44126


CHAPTER OVERVIEW
18: Representative Metals, Metalloids, and Nonmetals

A general chemistry Libretexts Textbook remixed and remastered from


OpenStax's textbook:
General Chemistry
The development of the periodic table in the mid-1800s came from observations that there was a periodic relationship between the
properties of the elements. Chemists, who have an understanding of the variations of these properties, have been able to use this
knowledge to solve a wide variety of technical challenges. For example, silicon and other semiconductors form the backbone of
modern electronics because of our ability to fine-tune the electrical properties of these materials. This chapter explores important
properties of representative metals, metalloids, and nonmetals in the periodic table.
18.1: Periodicity
18.2: Occurrence and Preparation of the Representative Metals
18.3: Structure and General Properties of the Metalloids
18.4: Structure and General Properties of the Nonmetals
18.5: Occurrence, Preparation, and Compounds of Hydrogen
18.6: Occurrence, Preparation, and Properties of Carbonates
18.7: Occurrence, Preparation, and Properties of Nitrogen
18.8: Occurrence, Preparation, and Properties of Phosphorus
18.9: Occurrence, Preparation, and Compounds of Oxygen
18.10: Occurrence, Preparation, and Properties of Sulfur
18.11: Occurrence, Preparation, and Properties of Halogens
18.12: Occurrence, Preparation, and Properties of the Noble Gases
18.E: Representative Metals, Metalloids, and Nonmetals (Exercises)

This page titled 18: Representative Metals, Metalloids, and Nonmetals is shared under a CC BY 4.0 license and was authored, remixed, and/or
curated by OpenStax via source content that was edited to the style and standards of the LibreTexts platform; a detailed edit history is available
upon request.

1
18.1: Periodicity
 Learning Objectives
Classify elements
Make predictions about the periodicity properties of the representative elements

We begin this section by examining the behaviors of representative metals in relation to their positions in the periodic table. The primary focus of
this section will be the application of periodicity to the representative metals.
It is possible to divide elements into groups according to their electron configurations. The representative elements are elements where the s and p
orbitals are filling. The transition elements are elements where the d orbitals (groups 3–11 on the periodic table) are filling, and the inner transition
metals are the elements where the f orbitals are filling. The d orbitals fill with the elements in group 11; therefore, the elements in group 12 qualify
as representative elements because the last electron enters an s orbital. Metals among the representative elements are the representative metals.
Metallic character results from an element’s ability to lose its outer valence electrons and results in high thermal and electrical conductivity, among
other physical and chemical properties. There are 20 nonradioactive representative metals in groups 1, 2, 3, 12, 13, 14, and 15 of the periodic table
(the elements shaded in yellow in Figure 18.1.1). The radioactive elements copernicium, flerovium, polonium, and livermorium are also metals but
are beyond the scope of this chapter.

Figure 18.1.1 : The location of the representative metals is shown in the periodic table. Nonmetals are shown in green, metalloids in purple, and the
transition metals and inner transition metals in blue.
In addition to the representative metals, some of the representative elements are metalloids. A metalloid is an element that has properties that are
between those of metals and nonmetals; these elements are typically semiconductors. The remaining representative elements are nonmetals. Unlike
metals, which typically form cations and ionic compounds (containing ionic bonds), nonmetals tend to form anions or molecular compounds. In
general, the combination of a metal and a nonmetal produces a salt. A salt is an ionic compound consisting of cations and anions.

A salt is an ionic compound consisting of cations and anions.


Most of the representative metals do not occur naturally in an uncombined state because they readily react with water and oxygen in the air.
However, it is possible to isolate elemental beryllium, magnesium, zinc, cadmium, mercury, aluminum, tin, and lead from their naturally occurring
minerals and use them because they react very slowly with air. Part of the reason why these elements react slowly is that these elements react with
air to form a protective coating. The formation of this protective coating is passivation. The coating is a nonreactive film of oxide or some other
compound. Elemental magnesium, aluminum, zinc, and tin are important in the fabrication of many familiar items, including wire, cookware, foil,
and many household and personal objects. Although beryllium, cadmium, mercury, and lead are readily available, there are limitations in their use
because of their toxicity.

Access for free at OpenStax 18.1.1 https://chem.libretexts.org/@go/page/38312


18.1.1: Group 1: The Alkali Metals
The alkali metals lithium, sodium, potassium, rubidium, cesium, and francium constitute group 1 of the periodic table. Although hydrogen is in
group 1 (and also in group 17), it is a nonmetal and deserves separate consideration later in this chapter. The name alkali metal is in reference to the
fact that these metals and their oxides react with water to form very basic (alkaline) solutions.
The properties of the alkali metals are similar to each other as expected for elements in the same family. The alkali metals have the largest atomic
radii and the lowest first ionization energy in their periods. This combination makes it very easy to remove the single electron in the outermost
(valence) shell of each. The easy loss of this valence electron means that these metals readily form stable cations with a charge of 1+. Their
reactivity increases with increasing atomic number due to the ease of losing the lone valence electron (decreasing ionization energy). Since oxidation
is so easy, the reverse, reduction, is difficult, which explains why it is hard to isolate the elements. The solid alkali metals are very soft; lithium,
shown in Figure 18.1.2, has the lowest density of any metal (0.5 g/cm3).

Figure 18.1.2 : Lithium floats in paraffin oil because its density is less than the density of paraffin oil.
The alkali metals all react vigorously with water to form hydrogen gas and a basic solution of the metal hydroxide. This means they are easier to
oxidize than is hydrogen. As an example, the reaction of lithium with water is:

2 Li(s) + 2 H O(l) ⟶ 2 LiOH(aq) + H (g)


2 2

Alkali metals react directly with all the nonmetals (except the noble gases) to yield binary ionic compounds containing 1+ metal ions. These metals
are so reactive that it is necessary to avoid contact with both moisture and oxygen in the air. Therefore, they are stored in sealed containers under
mineral oil, as shown in Figure 18.1.3, to prevent contact with air and moisture. The pure metals never exist free (uncombined) in nature due to their
high reactivity. In addition, this high reactivity makes it necessary to prepare the metals by electrolysis of alkali metal compounds.

Figure 18.1.3 : To prevent contact with air and water, potassium for laboratory use comes as sticks or beads stored under kerosene or mineral oil, or
in sealed containers. (credit: Potassium [images-of-elements.com])
Unlike many other metals, the reactivity and softness of the alkali metals make these metals unsuitable for structural applications. However, there are
applications where the reactivity of the alkali metals is an advantage. For example, the production of metals such as titanium and zirconium relies, in
part, on the ability of sodium to reduce compounds of these metals. The manufacture of many organic compounds, including certain dyes, drugs, and
perfumes, utilizes reduction by lithium or sodium.

Figure 18.1.4 : Colored flames from strontium, cesium, sodium and lithium (from left to right). Picture courtesy of the Claire Murray and Annabelle
Baker from the Diamond Light Source.
Sodium and its compounds impart a bright yellow color to a flame, as seen in Figure 18.1.4. Passing an electrical discharge through sodium vapor
also produces this color. In both cases, this is an example of an emission spectrum as discussed in the chapter on electronic structure. Streetlights

Access for free at OpenStax 18.1.2 https://chem.libretexts.org/@go/page/38312


sometime employ sodium vapor lights because the sodium vapor penetrates fog better than most other light. This is because the fog does not scatter
yellow light as much as it scatters white light. The other alkali metals and their salts also impart color to a flame. Lithium creates a bright, crimson
color, whereas the others create a pale, violet color.

Reaction of Lithium and Water

Video 18.1.1: This video demonstrates the reactions of the alkali metals with water.

18.1.2: Group 2: The Alkaline Earth Metals


The alkaline earth metals (beryllium, magnesium, calcium, strontium, barium, and radium) constitute group 2 of the periodic table. The name
alkaline metal comes from the fact that the oxides of the heavier members of the group react with water to form alkaline solutions. The nuclear
charge increases when going from group 1 to group 2. Because of this charge increase, the atoms of the alkaline earth metals are smaller and have
higher first ionization energies than the alkali metals within the same period. The higher ionization energy makes the alkaline earth metals less
reactive than the alkali metals; however, they are still very reactive elements. Their reactivity increases, as expected, with increasing size and
decreasing ionization energy. In chemical reactions, these metals readily lose both valence electrons to form compounds in which they exhibit an
oxidation state of 2+. Due to their high reactivity, it is common to produce the alkaline earth metals, like the alkali metals, by electrolysis. Even
though the ionization energies are low, the two metals with the highest ionization energies (beryllium and magnesium) do form compounds that
exhibit some covalent characters. Like the alkali metals, the heavier alkaline earth metals impart color to a flame. As in the case of the alkali metals,
this is part of the emission spectrum of these elements. Calcium and strontium produce shades of red, whereas barium produces a green color.
Magnesium is a silver-white metal that is malleable and ductile at high temperatures. Passivation decreases the reactivity of magnesium metal. Upon
exposure to air, a tightly adhering layer of magnesium oxycarbonate forms on the surface of the metal and inhibits further reaction. (The carbonate
comes from the reaction of carbon dioxide in the atmosphere.) Magnesium is the lightest of the widely used structural metals, which is why most
magnesium production is for lightweight alloys.
Magnesium (Figure 18.1.5), calcium, strontium, and barium react with water and air. At room temperature, barium shows the most vigorous
reaction. The products of the reaction with water are hydrogen and the metal hydroxide. The formation of hydrogen gas indicates that the heavier
alkaline earth metals are better reducing agents (more easily oxidized) than is hydrogen. As expected, these metals react with both acids and
nonmetals to form ionic compounds. Unlike most salts of the alkali metals, many of the common salts of the alkaline earth metals are insoluble in
water because of the high lattice energies of these compounds, containing a divalent metal ion.

Access for free at OpenStax 18.1.3 https://chem.libretexts.org/@go/page/38312


Figure 18.1.5 : From left to right: Mg(s), warm water at pH 7, and the resulting solution with a pH greater than 7, as indicated by the pink color of
the phenolphthalein indicator. (credit: modification of work by Sahar Atwa)
The potent reducing power of hot magnesium is useful in preparing some metals from their oxides. Indeed, magnesium’s affinity for oxygen is so
great that burning magnesium reacts with carbon dioxide, producing elemental carbon:

2 Mg(s) + CO (g) ⟶ 2 MgO(s) + C(s)


2

For this reason, a CO2 fire extinguisher will not extinguish a magnesium fire. Additionally, the brilliant white light emitted by burning magnesium
makes it useful in flares and fireworks.

18.1.3: Group 12 Metals


The elements in group 12 are transition elements; however, the last electron added is not a d electron, but an s electron. Since the last electron added
is an s electron, these elements qualify as representative metals, or post-transition metals. The group 12 elements behave more like the alkaline earth
metals than transition metals. Group 12 contains the four elements zinc, cadmium, mercury, and copernicium. Each of these elements has two
electrons in its outer shell (ns2). When atoms of these metals form cations with a charge of 2+, where the two outer electrons are lost, they have
pseudo-noble gas electron configurations. Mercury is sometimes an exception because it also exhibits an oxidation state of 1+ in compounds that
contain a diatomic Hg ion. In their elemental forms and in compounds, cadmium and mercury are both toxic.
2+

Zinc is the most reactive in group 12, and mercury is the least reactive. (This is the reverse of the reactivity trend of the metals of groups 1 and 2, in
which reactivity increases down a group. The increase in reactivity with increasing atomic number only occurs for the metals in groups 1 and 2.) The
decreasing reactivity is due to the formation of ions with a pseudo-noble gas configuration and to other factors that are beyond the scope of this
discussion. The chemical behaviors of zinc and cadmium are quite similar to each other but differ from that of mercury.
Zinc and cadmium have lower reduction potentials than hydrogen, and, like the alkali metals and alkaline earth metals, they will produce hydrogen
gas when they react with acids. The reaction of zinc with hydrochloric acid, shown in Figure 18.1.6, is:

Figure 18.1.6 : Zinc is an active metal. It dissolves in hydrochloric acid, forming a solution of colorless Zn2+ ions, Cl– ions, and hydrogen gas.
Zinc is a silvery metal that quickly tarnishes to a blue-gray appearance. This change in color is due to an adherent coating of a basic carbonate,
Zn2(OH)2CO3, which passivates the metal to inhibit further corrosion. Dry cell and alkaline batteries contain a zinc anode. Brass (Cu and Zn) and
some bronze (Cu, Sn, and sometimes Zn) are important zinc alloys. About half of zinc production serves to protect iron and other metals from
corrosion. This protection may take the form of a sacrificial anode (also known as a galvanic anode, which is a means of providing cathodic
protection for various metals) or as a thin coating on the protected metal. Galvanized steel is steel with a protective coating of zinc.

18.1.4: Sacrificial Anodes


A sacrificial anode, or galvanic anode, is a means of providing cathodic protection of various metals. Cathodic protection refers to the prevention of
corrosion by converting the corroding metal into a cathode. As a cathode, the metal resists corrosion, which is an oxidation process. Corrosion
occurs at the sacrificial anode instead of at the cathode.
The construction of such a system begins with the attachment of a more active metal (more negative reduction potential) to the metal needing
protection. Attachment may be direct or via a wire. To complete the circuit, a salt bridge is necessary. This salt bridge is often seawater or ground

Access for free at OpenStax 18.1.4 https://chem.libretexts.org/@go/page/38312


water. Once the circuit is complete, oxidation (corrosion) occurs at the anode and not the cathode.
The commonly used sacrificial anodes are magnesium, aluminum, and zinc. Magnesium has the most negative reduction potential of the three and
serves best when the salt bridge is less efficient due to a low electrolyte concentration such as in freshwater. Zinc and aluminum work better in
saltwater than does magnesium. Aluminum is lighter than zinc and has a higher capacity; however, an oxide coating may passivate the aluminum. In
special cases, other materials are useful. For example, iron will protect copper.
Mercury is very different from zinc and cadmium. Mercury is the only metal that is liquid at 25 °C. Many metals dissolve in mercury, forming
solutions called amalgams (see the feature on Amalgams), which are alloys of mercury with one or more other metals. Mercury, shown in Figure
18.1.7, is a nonreactive element that is more difficult to oxidize than hydrogen. Thus, it does not displace hydrogen from acids; however, it will react

with strong oxidizing acids, such as nitric acid:

Hg(l) + HCl(aq) ⟶ no reaction

3 Hg(l) + 8 HNO (aq) ⟶ 3 Hg (NO ) (aq) + 4 H O(l) + 2 NO(g)


3 3 2 2

The clear NO initially formed quickly undergoes further oxidation to the reddish brown NO2.

Figure 18.1.7 : From left to right: Hg(l), Hg + concentrated HCl, Hg + concentrated HNO3. (credit: Sahar Atwa)
Most mercury compounds decompose when heated. Most mercury compounds contain mercury with a 2+-oxidation state. When there is a large
excess of mercury, it is possible to form compounds containing the Hg ion. All mercury compounds are toxic, and it is necessary to exercise great
2+

care in their synthesis.

 Amalgams
An amalgam is an alloy of mercury with one or more other metals. This is similar to considering steel to be an alloy of iron with other metals.
Most metals will form an amalgam with mercury, with the main exceptions being iron, platinum, tungsten, and tantalum. Due to toxicity issues
with mercury, there has been a significant decrease in the use of amalgams. Historically, amalgams were important in electrolytic cells and in the
extraction of gold. Amalgams of the alkali metals still find use because they are strong reducing agents and easier to handle than the pure alkali
metals. Prospectors had a problem when they found finely divided gold. They learned that adding mercury to their pans collected the gold into
the mercury to form an amalgam for easier collection. Unfortunately, losses of small amounts of mercury over the years left many streams in
California polluted with mercury. Dentists use amalgams containing silver and other metals to fill cavities. There are several reasons to use an
amalgam including low cost, ease of manipulation, and longevity compared to alternate materials. Dental amalgams are approximately 50%
mercury by weight, which, in recent years, has become a concern due to the toxicity of mercury.
After reviewing the best available data, the Food and Drug Administration (FDA) considers amalgam-based fillings to be safe for adults and
children over six years of age. Even with multiple fillings, the mercury levels in the patients remain far below the lowest levels associated with
harm. Clinical studies have found no link between dental amalgams and health problems. Health issues may not be the same in cases of children
under six or pregnant women. The FDA conclusions are in line with the opinions of the Environmental Protection Agency (EPA) and Centers
for Disease Control (CDC). The only health consideration noted is that some people are allergic to the amalgam or one of its components.

18.1.5: Group 13
Group 13 contains the metalloid boron and the metals aluminum, gallium, indium, and thallium. The lightest element, boron, is semiconducting, and
its binary compounds tend to be covalent and not ionic. The remaining elements of the group are metals, but their oxides and hydroxides change
characters. The oxides and hydroxides of aluminum and gallium exhibit both acidic and basic behaviors. A substance, such as these two, that will
react with both acids and bases is amphoteric. This characteristic illustrates the combination of nonmetallic and metallic behaviors of these two
elements. Indium and thallium oxides and hydroxides exhibit only basic behavior, in accordance with the clearly metallic character of these two
elements. The melting point of gallium is unusually low (about 30 °C) and will melt in your hand.
Aluminum is amphoteric because it will react with both acids and bases. A typical reaction with an acid is:

2 Al(s) + 6 HCl(aq) ⟶ 2 AlCl (aq) + 3 H (g)


3 2

Access for free at OpenStax 18.1.5 https://chem.libretexts.org/@go/page/38312


The products of the reaction of aluminum with a base depend upon the reaction conditions, with the following being one possibility:

2 Al(s) + 2 NaOH(aq) + 6 H O(l) ⟶ 2 Na[Al (OH) ](aq) + 3 H (g)


2 4 2

With both acids and bases, the reaction with aluminum generates hydrogen gas.
The group 13 elements have a valence shell electron configuration of ns2np1. Aluminum normally uses all of its valence electrons when it reacts,
giving compounds in which it has an oxidation state of 3+. Although many of these compounds are covalent, others, such as AlF3 and Al2(SO4)3, are
ionic. Aqueous solutions of aluminum salts contain the cation [Al(H O) ] , abbreviated as Al3+(aq). Gallium, indium, and thallium also form
3+

2 6

ionic compounds containing M3+ ions. These three elements exhibit not only the expected oxidation state of 3+ from the three valence electrons but
also an oxidation state (in this case, 1+) that is two below the expected value. This phenomenon, the inert pair effect, refers to the formation of a
stable ion with an oxidation state two lower than expected for the group. The pair of electrons is the valence s orbital for those elements. In general,
the inert pair effect is important for the lower p-block elements. In an aqueous solution, the Tl+(aq) ion is more stable than is Tl3+(aq). In general,
these metals will react with air and water to form 3+ ions; however, thallium reacts to give thallium(I) derivatives. The metals of group 13 all react
directly with nonmetals such as sulfur, phosphorus, and the halogens, forming binary compounds.
The metals of group 13 (Al, Ga, In, and Tl) are all reactive. However, passivation occurs as a tough, hard, thin film of the metal oxide forms upon
exposure to air. Disruption of this film may counter the passivation, allowing the metal to react. One way to disrupt the film is to expose the
passivated metal to mercury. Some of the metal dissolves in the mercury to form an amalgam, which sheds the protective oxide layer to expose the
metal to further reaction. The formation of an amalgam allows the metal to react with air and water.

Mercury attacks Aluminum

Video 18.1.2 : Although easily oxidized, the passivation of aluminum makes it very useful as a strong, lightweight building material. Because of the
formation of an amalgam, mercury is corrosive to structural materials made of aluminum. This video demonstrates how the integrity of an aluminum
beam can be destroyed by the addition of a small amount of elemental mercury.
The most important uses of aluminum are in the construction and transportation industries, and in the manufacture of aluminum cans and aluminum
foil. These uses depend on the lightness, toughness, and strength of the metal, as well as its resistance to corrosion. Because aluminum is an
excellent conductor of heat and resists corrosion, it is useful in the manufacture of cooking utensils.
Aluminum is a very good reducing agent and may replace other reducing agents in the isolation of certain metals from their oxides. Although more
expensive than reduction by carbon, aluminum is important in the isolation of Mo, W, and Cr from their oxides.

18.1.6: Group 14
The metallic members of group 14 are tin, lead, and flerovium. Carbon is a typical nonmetal. The remaining elements of the group, silicon and
germanium, are examples of semimetals or metalloids. Tin and lead form the stable divalent cations, Sn2+ and Pb2+, with oxidation states two below
the group oxidation state of 4+. The stability of this oxidation state is a consequence of the inert pair effect. Tin and lead also form covalent
compounds with a formal 4+-oxidation state. For example, SnCl4 and PbCl4 are low-boiling covalent liquids.

Access for free at OpenStax 18.1.6 https://chem.libretexts.org/@go/page/38312


Figure 18.1.8 : (a) Tin(II) chloride is an ionic solid; (b) tin(IV) chloride is a covalent liquid.
Tin reacts readily with nonmetals and acids to form tin(II) compounds (indicating that it is more easily oxidized than hydrogen) and with nonmetals
to form either tin(II) or tin(IV) compounds (Figure 18.1.8), depending on the stoichiometry and reaction conditions. Lead is less reactive. It is only
slightly easier to oxidize than hydrogen, and oxidation normally requires a hot concentrated acid.
Many of these elements exist as allotropes. Allotropes are two or more forms of the same element in the same physical state with different chemical
and physical properties. There are two common allotropes of tin. These allotropes are grey (brittle) tin and white tin. As with other allotropes, the
difference between these forms of tin is in the arrangement of the atoms. White tin is stable above 13.2 °C and is malleable like other metals. At low
temperatures, gray tin is the more stable form. Gray tin is brittle and tends to break down to a powder. Consequently, articles made of tin will
disintegrate in cold weather, particularly if the cold spell is lengthy. The change progresses slowly from the spot of origin, and the gray tin that is
first formed catalyzes further change. In a way, this effect is similar to the spread of an infection in a plant or animal body, leading people to call this
process tin disease or tin pest.
The principal use of tin is in the coating of steel to form tin plate-sheet iron, which constitutes the tin in tin cans. Important tin alloys are bronze (Cu
and Sn) and solder (Sn and Pb). Lead is important in the lead storage batteries in automobiles.

18.1.7: Group 15
Bismuth, the heaviest member of group 15, is a less reactive metal than the other representative metals. It readily gives up three of its five valence
electrons to active nonmetals to form the tri-positive ion, Bi3+. It forms compounds with the group oxidation state of 5+ only when treated with
strong oxidizing agents. The stability of the 3+-oxidation state is another example of the inert pair effect.

Summary
This section focuses on the periodicity of the representative elements. These are the elements where the electrons are entering the s and p orbitals.
The representative elements occur in groups 1, 2, and 12–18. These elements are representative metals, metalloids, and nonmetals. The alkali metals
(group 1) are very reactive, readily form ions with a charge of 1+ to form ionic compounds that are usually soluble in water, and react vigorously
with water to form hydrogen gas and a basic solution of the metal hydroxide. The outermost electrons of the alkaline earth metals (group 2) are more
difficult to remove than the outer electron of the alkali metals, leading to the group 2 metals being less reactive than those in group 1. These
elements easily form compounds in which the metals exhibit an oxidation state of 2+. Zinc, cadmium, and mercury (group 12) commonly exhibit the
group oxidation state of 2+ (although mercury also exhibits an oxidation state of 1+ in compounds that contain Hg ). Aluminum, gallium, indium,
2+

and thallium (group 13) are easier to oxidize than is hydrogen. Aluminum, gallium, and indium occur with an oxidation state 3+ (however, thallium
also commonly occurs as the Tl+ ion). Tin and lead form stable divalent cations and covalent compounds in which the metals exhibit the 4+-
oxidation state.

Glossary
alkaline earth metal
any of the metals (beryllium, magnesium, calcium, strontium, barium, and radium) occupying group 2 of the periodic table; they are reactive,
divalent metals that form basic oxides

allotropes
two or more forms of the same element, in the same physical state, with different chemical structures

bismuth
heaviest member of group 15; a less reactive metal than other representative metals

metal (representative)
atoms of the metallic elements of groups 1, 2, 12, 13, 14, 15, and 16, which form ionic compounds by losing electrons from their outer s or p
orbitals

metalloid
element that has properties that are between those of metals and nonmetals; these elements are typically semiconductors

passivation

Access for free at OpenStax 18.1.7 https://chem.libretexts.org/@go/page/38312


metals with a protective nonreactive film of oxide or other compound that creates a barrier for chemical reactions; physical or chemical removal
of the passivating film allows the metals to demonstrate their expected chemical reactivity

representative element
element where the s and p orbitals are filling

representative metal
metal among the representative elements

This page titled 18.1: Periodicity is shared under a CC BY 4.0 license and was authored, remixed, and/or curated by OpenStax via source content that was edited to
the style and standards of the LibreTexts platform; a detailed edit history is available upon request.

Access for free at OpenStax 18.1.8 https://chem.libretexts.org/@go/page/38312


18.2: Occurrence and Preparation of the Representative Metals
 Learning Objectives
Identify natural sources of representative metals
Describe electrolytic and chemical reduction processes used to prepare these elements from natural sources

Because of their reactivity, we do not find most representative metals as free elements in nature. However, compounds that contain
ions of most representative metals are abundant. In this section, we will consider the two common techniques used to isolate the
metals from these compounds—electrolysis and chemical reduction.
These metals primarily occur in minerals, with lithium found in silicate or phosphate minerals, and sodium and potassium found in
salt deposits from evaporation of ancient seas and in silicates. The alkaline earth metals occur as silicates and, with the exception of
beryllium, as carbonates and sulfates. Beryllium occurs as the mineral beryl, Be3Al2Si6O18, which, with certain impurities, may be
either the gemstone emerald or aquamarine. Magnesium is in seawater and, along with the heavier alkaline earth metals, occurs as
silicates, carbonates, and sulfates. Aluminum occurs abundantly in many types of clay and in bauxite, an impure aluminum oxide
hydroxide. The principle tin ore is the oxide cassiterite, SnO2, and the principle lead and thallium ores are the sulfides or the
products of weathering of the sulfides. The remaining representative metals occur as impurities in zinc or aluminum ores.

18.2.1: Electrolysis
Ions of metals in of groups 1 and 2, along with aluminum, are very difficult to reduce; therefore, it is necessary to prepare these
elements by electrolysis, an important process discussed in the chapter on electrochemistry. Briefly, electrolysis involves using
electrical energy to drive unfavorable chemical reactions to completion; it is useful in the isolation of reactive metals in their pure
forms. Sodium, aluminum, and magnesium are typical examples.

18.2.2: The Preparation of Sodium


The most important method for the production of sodium is the electrolysis of molten sodium chloride; the set-up is a Downs cell,
shown in Figure 18.2.1. The reaction involved in this process is:
electrolysis

2 NaCl(l) −−−−−−→ 2 Na(l) + Cl (g)


2
600 °C

The electrolysis cell contains molten sodium chloride (melting point 801 °C), to which calcium chloride has been added to lower
the melting point to 600 °C (a colligative effect). The passage of a direct current through the cell causes the sodium ions to migrate
to the negatively charged cathode and pick up electrons, reducing the ions to sodium metal. Chloride ions migrate to the positively
charged anode, lose electrons, and undergo oxidation to chlorine gas. The overall cell reaction comes from adding the following
reactions:
+ −
at the cathode: 2 Na +2 e ⟶ 2 Na(l) (18.2.1)
− −
at the anode: 2 Cl ⟶ Cl (g) + 2 e (18.2.2)
2

+ −
overall change: 2 Na + 2 Cl ⟶ 2 Na(l) + Cl (g) (18.2.3)
2

Separation of the molten sodium and chlorine prevents recombination. The liquid sodium, which is less dense than molten sodium
chloride, floats to the surface and flows into a collector. The gaseous chlorine goes to storage tanks. Chlorine is also a valuable
product.

Access for free at OpenStax 18.2.1 https://chem.libretexts.org/@go/page/38313


Figure 18.2.1 : Pure sodium metal is isolated by electrolysis of molten sodium chloride using a Downs cell. It is not possible to
isolate sodium by electrolysis of aqueous solutions of sodium salts because hydrogen ions are more easily reduced than are sodium
ions; as a result, hydrogen gas forms at the cathode instead of the desired sodium metal. The high temperature required to melt
NaCl means that liquid sodium metal forms.

18.2.3: The Preparation of Aluminum


The preparation of aluminum utilizes a process invented in 1886 by Charles M. Hall, who began to work on the problem while a
student at Oberlin College in Ohio. Paul L. T. Héroult discovered the process independently a month or two later in France. In
honor to the two inventors, this electrolysis cell is known as the Hall–Héroult cell. The Hall–Héroult cell is an electrolysis cell for
the production of aluminum. Figure 18.2.2 illustrates the Hall–Héroult cell.
The production of aluminum begins with the purification of bauxite, the most common source of aluminum. The reaction of
bauxite, AlO(OH), with hot sodium hydroxide forms soluble sodium aluminate, while clay and other impurities remain
undissolved:

AlO(OH)(s) + NaOH(aq) + H O(l) ⟶ Na[Al (OH) ](aq)


2 4

After the removal of the impurities by filtration, the addition of acid to the aluminate leads to the reprecipitation of aluminum
hydroxide:
+ +
Na[Al (OH) ](aq) + H O (aq) ⟶ Al (OH) (s) + Na (aq) + 2 H O(l)
4 3 3 2

The next step is to remove the precipitated aluminum hydroxide by filtration. Heating the hydroxide produces aluminum oxide,
Al2O3, which dissolves in a molten mixture of cryolite, Na3AlF6, and calcium fluoride, CaF2. Electrolysis of this solution takes
place in a cell like that shown in Figure 18.2.2. Reduction of aluminum ions to the metal occurs at the cathode, while oxygen,
carbon monoxide, and carbon dioxide form at the anode.

Access for free at OpenStax 18.2.2 https://chem.libretexts.org/@go/page/38313


Figure 18.2.2 : An electrolytic cell is used for the production of aluminum. The electrolysis of a solution of cryolite and calcium
fluoride results in aluminum metal at the cathode, and oxygen, carbon monoxide, and carbon dioxide at the anode.

18.2.3.1: The Preparation of Magnesium


Magnesium is the other metal that is isolated in large quantities by electrolysis. Seawater, which contains approximately 0.5%
magnesium chloride, serves as the major source of magnesium. Addition of calcium hydroxide to seawater precipitates magnesium
hydroxide. The addition of hydrochloric acid to magnesium hydroxide, followed by evaporation of the resultant aqueous solution,
leaves pure magnesium chloride. The electrolysis of molten magnesium chloride forms liquid magnesium and chlorine gas:

MgCl (aq) + Ca (OH) (aq) ⟶ Mg (OH) (s) + CaCl (aq)


2 2 2 2

Mg (OH) (s) + 2 HCl(aq) ⟶ MgCl (aq) + 2 H O(l)


2 2 2

MgCl (l) ⟶ Mg(l) + Cl (g)


2 2

Some production facilities have moved away from electrolysis completely. In the next section, we will see how the Pidgeon process
leads to the chemical reduction of magnesium.

18.2.4: Chemical Reduction


It is possible to isolate many of the representative metals by chemical reduction using other elements as reducing agents. In general,
chemical reduction is much less expensive than electrolysis, and for this reason, chemical reduction is the method of choice for the
isolation of these elements. For example, it is possible to produce potassium, rubidium, and cesium by chemical reduction, as it is
possible to reduce the molten chlorides of these metals with sodium metal. This may be surprising given that these metals are more
reactive than sodium; however, the metals formed are more volatile than sodium and can be distilled for collection. The removal of
the metal vapor leads to a shift in the equilibrium to produce more metal (via the Le Chatelier’s principle). The production of
magnesium, zinc, and tin provide additional examples of chemical reduction.

18.2.4.1: The Preparation of Magnesium


The Pidgeon process involves the reaction of magnesium oxide with elemental silicon at high temperatures to form pure
magnesium:
Δ

Si(s) + 2 MgO(s) −
→ SiO (s) + 2 Mg(g)
2

Although this reaction is unfavorable in terms of thermodynamics, the removal of the magnesium vapor produced takes advantage
of Le Chatelier’s principle to continue the forward progress of the reaction. Over 75% of the world’s production of magnesium,
primarily in China, comes from this process.

18.2.5: The Preparation of Zinc


Zinc ores usually contain zinc sulfide, zinc oxide, or zinc carbonate. After separation of these compounds from the ores, heating in
air converts the ore to zinc oxide by one of the following reactions:
Δ

2 ZnS(s) + 3 O (g) −
→ 2 ZnO(s) + 2 SO (g)
2 2

Access for free at OpenStax 18.2.3 https://chem.libretexts.org/@go/page/38313


Δ

ZnCO (s) −
→ ZnO(s) + CO (g)
3 2

Carbon, in the form of coal, reduces the zinc oxide to form zinc vapor:

ZnO(s) + C(s) ⟶ Zn(g) + CO(g)

The zinc can be distilled (boiling point 907 °C) and condensed. This zinc contains impurities of cadmium (767 °C), iron (2862 °C),
lead (1750 °C), and arsenic (613 °C). Careful redistillation produces pure zinc. Arsenic and cadmium are distilled from the zinc
because they have lower boiling points. At higher temperatures, the zinc is distilled from the other impurities, mainly lead and iron.

18.2.6: The Preparation of Tin


The ready reduction of tin(IV) oxide by the hot coals of a campfire accounts for the knowledge of tin in the ancient world. In the
modern process, the roasting of tin ores containing SnO2 removes contaminants such as arsenic and sulfur as volatile oxides.
Treatment of the remaining material with hydrochloric acid removes the oxides of other metals. Heating the purified ore with
carbon at temperature above 1000 °C produces tin:
Δ

SnO (s) + 2 C(s) −


→ Sn(s) + 2 CO(g)
2

The molten tin collects at the bottom of the furnace and is drawn off and cast into blocks.

Summary
Because of their chemical reactivity, it is necessary to produce the representative metals in their pure forms by reduction from
naturally occurring compounds. Electrolysis is important in the production of sodium, potassium, and aluminum. Chemical
reduction is the primary method for the isolation of magnesium, zinc, and tin. Similar procedures are important for the other
representative metals.

Glossary
chemical reduction
method of preparing a representative metal using a reducing agent

Downs cell
electrochemical cell used for the commercial preparation of metallic sodium (and chlorine) from molten sodium chloride

Hall–Héroult cell
electrolysis apparatus used to isolate pure aluminum metal from a solution of alumina in molten cryolite

Pidgeon process
chemical reduction process used to produce magnesium through the thermal reaction of magnesium oxide with silicon

This page titled 18.2: Occurrence and Preparation of the Representative Metals is shared under a CC BY 4.0 license and was authored, remixed,
and/or curated by OpenStax via source content that was edited to the style and standards of the LibreTexts platform; a detailed edit history is
available upon request.

Access for free at OpenStax 18.2.4 https://chem.libretexts.org/@go/page/38313


18.3: Structure and General Properties of the Metalloids
 Learning Objectives
Describe the general preparation, properties, and uses of the metalloids
Describe the preparation, properties, and compounds of boron and silicon

A series of six elements called the metalloids separate the metals from the nonmetals in the periodic table. The metalloids are boron, silicon,
germanium, arsenic, antimony, and tellurium. These elements look metallic; however, they do not conduct electricity as well as metals so they are
semiconductors. They are semiconductors because their electrons are more tightly bound to their nuclei than are those of metallic conductors. Their
chemical behavior falls between that of metals and nonmetals. For example, the pure metalloids form covalent crystals like the nonmetals, but like
the metals, they generally do not form monatomic anions. This intermediate behavior is in part due to their intermediate electronegativity values. In
this section, we will briefly discuss the chemical behavior of metalloids and deal with two of these elements—boron and silicon—in more detail.
The metalloid boron exhibits many similarities to its neighbor carbon and its diagonal neighbor silicon. All three elements form covalent
compounds. However, boron has one distinct difference in that its 2s22p1 outer electron structure gives it one less valence electron than it has
valence orbitals. Although boron exhibits an oxidation state of 3+ in most of its stable compounds, this electron deficiency provides boron with the
ability to form other, sometimes fractional, oxidation states, which occur, for example, in the boron hydrides.
Silicon has the valence shell electron configuration 3s23p2, and it commonly forms tetrahedral structures in which it is sp3 hybridized with a formal
oxidation state of 4+. The major differences between the chemistry of carbon and silicon result from the relative strength of the carbon-carbon bond,
carbon’s ability to form stable bonds to itself, and the presence of the empty 3d valence-shell orbitals in silicon. Silicon’s empty d orbitals and
boron’s empty p orbital enable tetrahedral silicon compounds and trigonal planar boron compounds to act as Lewis acids. Carbon, on the other hand,
has no available valence shell orbitals; tetrahedral carbon compounds cannot act as Lewis acids. Germanium is very similar to silicon in its chemical
behavior.
Arsenic and antimony generally form compounds in which an oxidation state of 3+ or 5+ is exhibited; however, arsenic can form arsenides with an
oxidation state of 3−. These elements tarnish only slightly in dry air but readily oxidize when warmed.
Tellurium combines directly with most elements. The most stable tellurium compounds are the tellurides—salts of Te2− formed with active metals
and lanthanides—and compounds with oxygen, fluorine, and chlorine, in which tellurium normally exhibits an oxidation state 2+ or 4+. Although
tellurium(VI) compounds are known (for example, TeF6), there is a marked resistance to oxidation to this maximum group oxidation state.

18.3.1: Structures of the Metalloids


Covalent bonding is the key to the crystal structures of the metalloids. In this regard, these elements resemble nonmetals in their behavior.

Figure 18.3.1 : (a) Arsenic and (b) antimony have a layered structure similar to that of (c) graphite, except that the layers are puckered rather than
planar. (d) Elemental tellurium forms spiral chains.
Elemental silicon, germanium, arsenic, antimony, and tellurium are lustrous, metallic-looking solids. Silicon and germanium crystallize with a
diamond structure. Each atom within the crystal has covalent bonds to four neighboring atoms at the corners of a regular tetrahedron. Single crystals
of silicon and germanium are giant, three-dimensional molecules. There are several allotropes of arsenic with the most stable being layer like and
containing puckered sheets of arsenic atoms. Each arsenic atom forms covalent bonds to three other atoms within the sheet. The crystal structure of
antimony is similar to that of arsenic, both shown in Figure 18.3.1. The structures of arsenic and antimony are similar to the structure of graphite,
covered later in this chapter. Tellurium forms crystals that contain infinite spiral chains of tellurium atoms. Each atom in the chain bonds to two
other atoms.

Access for free at OpenStax 18.3.1 https://chem.libretexts.org/@go/page/38314


Face Centered Cubic Structure (diamond)

Video 18.3.1 : Explore a cubic diamond crystal structure.


Pure crystalline boron is transparent. The crystals consist of icosahedra, as shown in Figure 18.3.2, with a boron atom at each corner. In the most
common form of boron, the icosahedra pack together in a manner similar to the cubic closest packing of spheres. All boron-boron bonds within each
icosahedron are identical and are approximately 176 pm in length. In the different forms of boron, there are different arrangements and connections
between the icosahedra.

Figure 18.3.2 : An icosahedron is a symmetrical, solid shape with 20 faces, each of which is an equilateral triangle. The faces meet at 12 corners.
The name silicon is derived from the Latin word for flint, silex. The metalloid silicon readily forms compounds containing Si-O-Si bonds, which are
of prime importance in the mineral world. This bonding capability is in contrast to the nonmetal carbon, whose ability to form carbon-carbon bonds
gives it prime importance in the plant and animal worlds.

18.3.2: Occurrence, Preparation, and Compounds of Boron and Silicon


Boron constitutes less than 0.001% by weight of the earth’s crust. In nature, it only occurs in compounds with oxygen. Boron is widely distributed in
volcanic regions as boric acid, B(OH)3, and in dry lake regions, including the desert areas of California, as borates and salts of boron oxyacids, such
as borax, Na2B4O7⋅10H2O.
Elemental boron is chemically inert at room temperature, reacting with only fluorine and oxygen to form boron trifluoride, BF3, and boric oxide,
B2O3, respectively. At higher temperatures, boron reacts with all nonmetals, except tellurium and the noble gases, and with nearly all metals; it
oxidizes to B2O3 when heated with concentrated nitric or sulfuric acid. Boron does not react with nonoxidizing acids. Many boron compounds react
readily with water to give boric acid, B(OH)3 (sometimes written as H3BO3).
Reduction of boric oxide with magnesium powder forms boron (95–98.5% pure) as a brown, amorphous powder:

B O (s) + 3 Mg(s) ⟶ 2 B(s) + 3 MgO(s)


2 3

An amorphous substance is a material that appears to be a solid, but does not have a long-range order like a true solid. Treatment with hydrochloric
acid removes the magnesium oxide. Further purification of the boron begins with conversion of the impure boron into boron trichloride. The next
step is to heat a mixture of boron trichloride and hydrogen:
1500 °C

2 BCl (g) + 3 H (g) −−−−→ 2 B(s) + 6 HCl(g) ΔH ° = 253.7 kJ


3 2

Access for free at OpenStax 18.3.2 https://chem.libretexts.org/@go/page/38314


Silicon makes up nearly one-fourth of the mass of the earth’s crust—second in abundance only to oxygen. The crust is composed almost entirely of
minerals in which the silicon atoms are at the center of the silicon-oxygen tetrahedron, which connect in a variety of ways to produce, among other
things, chains, layers, and three-dimensional frameworks. These minerals constitute the bulk of most common rocks, soil, and clays. In addition,
materials such as bricks, ceramics, and glasses contain silicon compounds.
It is possible to produce silicon by the high-temperature reduction of silicon dioxide with strong reducing agents, such as carbon and magnesium:
Δ

SiO (s) + 2 C(s) −


→ Si(s) + 2 CO(g)
2

SiO (s) + 2 Mg(s) −


→ Si(s) + 2 MgO(s)
2

Extremely pure silicon is necessary for the manufacture of semiconductor electronic devices. This process begins with the conversion of impure
silicon into silicon tetrahalides, or silane (SiH4), followed by decomposition at high temperatures. Zone refining, illustrated in Figure 18.3.3,
completes the purification. In this method, a rod of silicon is heated at one end by a heat source that produces a thin cross-section of molten silicon.
Slowly lowering the rod through the heat source moves the molten zone from one end of the rod to other. As this thin, molten region moves,
impurities in the silicon dissolve in the liquid silicon and move with the molten region. Ultimately, the impurities move to one end of the rod, which
is then cut off.

Figure 18.3.3 : A zone-refining apparatus used to purify silicon.


This highly purified silicon, containing no more than one part impurity per million parts of silicon, is the most important element in the computer
industry. Pure silicon is necessary in semiconductor electronic devices such as transistors, computer chips, and solar cells.
Like some metals, passivation of silicon occurs due the formation of a very thin film of oxide (primarily silicon dioxide, SiO2). Silicon dioxide is
soluble in hot aqueous base; thus, strong bases destroy the passivation. Removal of the passivation layer allows the base to dissolve the silicon,
forming hydrogen gas and silicate anions. For example:
− 4−
Si(s) + 4 OH (aq) ⟶ SiO (aq) + 2 H (g)
4 2

Silicon reacts with halogens at high temperatures, forming volatile tetrahalides, such as SiF4.
Unlike carbon, silicon does not readily form double or triple bonds. Silicon compounds of the general formula SiX4, where X is a highly
electronegative group, can act as Lewis acids to form six-coordinate silicon. For example, silicon tetrafluoride, SiF4, reacts with sodium fluoride to
yield Na2[SiF6], which contains the octahedral [SiF ] ion in which silicon is sp3d2 hybridized:
6
2−

2 NaF(s) + SiF (g) ⟶ Na SiF (s)


4 2 6

Antimony reacts readily with stoichiometric amounts of fluorine, chlorine, bromine, or iodine, yielding trihalides or, with excess fluorine or chlorine,
forming the pentahalides SbF5 and SbCl5. Depending on the stoichiometry, it forms antimony(III) sulfide, Sb2S3, or antimony(V) sulfide when
heated with sulfur. As expected, the metallic nature of the element is greater than that of arsenic, which lies immediately above it in group 15.

18.3.3: Boron and Silicon Halides


Boron trihalides—BF3, BCl3, BBr3, and BI3—can be prepared by the direct reaction of the elements. These nonpolar molecules contain boron with
sp2 hybridization and a trigonal planar molecular geometry. The fluoride and chloride compounds are colorless gasses, the bromide is a liquid, and
the iodide is a white crystalline solid.

Access for free at OpenStax 18.3.3 https://chem.libretexts.org/@go/page/38314


Except for boron trifluoride, the boron trihalides readily hydrolyze in water to form boric acid and the corresponding hydrohalic acid. Boron
trichloride reacts according to the equation:

BCl (g) + 3 H O(l) ⟶ B(OH) (aq) + 3 HCl(aq)


3 2 3

Boron trifluoride reacts with hydrofluoric acid, to yield a solution of fluoroboric acid, HBF4:
+ −
BF (aq) + HF(aq) + H O(l) ⟶ H O (aq) + BF (aq)
3 2 3 4

In this reaction, the BF3 molecule acts as the Lewis acid (electron pair acceptor) and accepts a pair of electrons from a fluoride ion:

All the tetrahalides of silicon, SiX4, have been prepared. Silicon tetrachloride can be prepared by direct chlorination at elevated temperatures or by
heating silicon dioxide with chlorine and carbon:
Δ

SiO (s) + 2 C(s) + 2 Cl (g) −


→ SiCl (g) + 2 CO(g)
2 2 4

Silicon tetrachloride is a covalent tetrahedral molecule, which is a nonpolar, low-boiling (57 °C), colorless liquid. It is possible to prepare silicon
tetrafluoride by the reaction of silicon dioxide with hydrofluoric acid:

SiO (s) + 4 HF(g) ⟶ SiF (g) + 2 H O(l) ΔH ° = −191.2 kJ


2 4 2

Hydrofluoric acid is the only common acid that will react with silicon dioxide or silicates. This reaction occurs because the silicon-fluorine bond is
the only bond that silicon forms that is stronger than the silicon-oxygen bond. For this reason, it is possible to store all common acids, other than
hydrofluoric acid, in glass containers.
Except for silicon tetrafluoride, silicon halides are extremely sensitive to water. Upon exposure to water, SiCl4 reacts rapidly with hydroxide groups,
replacing all four chlorine atoms to produce unstable orthosilicic acid, Si(OH)4 or H4SiO4, which slowly decomposes into SiO2.

18.3.4: Boron and Silicon Oxides and Derivatives


Boron burns at 700 °C in oxygen, forming boric oxide, B2O3. Boric oxide is necessary for the production of heat-resistant borosilicate glass, like that
shown in Figure 18.3.4 and certain optical glasses. Boric oxide dissolves in hot water to form boric acid, B(OH)3:

B O (s) + 3 H O(l) ⟶ 2 B(OH) (aq)


2 3 2 3

Figure 18.3.4 : Laboratory glassware, such as Pyrex and Kimax, is made of borosilicate glass because it does not break when heated. The inclusion
of borates in the glass helps to mediate the effects of thermal expansion and contraction. This reduces the likelihood of thermal shock, which causes
silicate glass to crack upon rapid heating or cooling. (credit: “Tweenk”/Wikimedia Commons)
The boron atom in B(OH)3 is sp2 hybridized and is located at the center of an equilateral triangle with oxygen atoms at the corners. In solid B(OH)3,
hydrogen bonding holds these triangular units together. Boric acid, shown in Figure 18.3.5, is a very weak acid that does not act as a proton donor
but rather as a Lewis acid, accepting an unshared pair of electrons from the Lewis base OH−:
− + −10
B(OH) (aq) + 2 H O(l) ⇌ B(OH) (aq) + H O (aq) Ka = 5.8 × 10
3 2 4 3

Access for free at OpenStax 18.3.4 https://chem.libretexts.org/@go/page/38314


Figure 18.3.5 : Boric acid has a planar structure with three –OH groups spread out equally at 120° angles from each other.
Heating boric acid to 100 °C causes molecules of water to split out between pairs of adjacent –OH groups to form metaboric acid, HBO2. At about
150 °C, additional B-O-B linkages form, connecting the BO3 groups together with shared oxygen atoms to form tetraboric acid, H2B4O7. Complete
water loss, at still higher temperatures, results in boric oxide.
Borates are salts of the oxyacids of boron. Borates result from the reactions of a base with an oxyacid or from the fusion of boric acid or boric oxide
with a metal oxide or hydroxide. Borate anions range from the simple trigonal planar BO ion to complex species containing chains and rings of
3−
3

three- and four-coordinated boron atoms. The structures of the anions found in CaB2O4, K[B5O6(OH)4]⋅2H2O (commonly written KB5O8⋅4H2O)
and Na2[B4O5(OH)4]⋅8H2O (commonly written Na2B4O7⋅10H2O) are shown in Figure 18.3.6. Commercially, the most important borate is borax,
Na2[B4O5(OH)4]⋅8H2O, which is an important component of some laundry detergents. Most of the supply of borax comes directly from dry lakes,
such as Searles Lake in California, or is prepared from kernite, Na2B4O7⋅4H2O.

Figure 18.3.6 : The borate anions are (a) CaB2O4, (b) KB5O8⋅4H2O, and (c) Na2B4O7⋅10H2O. The anion in CaB2O4 is an “infinite” chain.
Silicon dioxide, silica, occurs in both crystalline and amorphous forms. The usual crystalline form of silicon dioxide is quartz, a hard, brittle, clear,
colorless solid. It is useful in many ways—for architectural decorations, semiprecious jewels, and frequency control in radio transmitters. Silica
takes many crystalline forms, or polymorphs, in nature. Trace amounts of Fe3+ in quartz give amethyst its characteristic purple color. The term
quartz is also used for articles such as tubing and lenses that are manufactured from amorphous silica. Opal is a naturally occurring form of
amorphous silica.
The contrast in structure and physical properties between silicon dioxide and carbon dioxide is interesting, as illustrated in Figure 18.3.7. Solid
carbon dioxide (dry ice) contains single CO2 molecules with each of the two oxygen atoms attached to the carbon atom by double bonds. Very weak
intermolecular forces hold the molecules together in the crystal. The volatility of dry ice reflect these weak forces between molecules. In contrast,
silicon dioxide is a covalent network solid. In silicon dioxide, each silicon atom links to four oxygen atoms by single bonds directed toward the
corners of a regular tetrahedron, and SiO4 tetrahedra share oxygen atoms. This arrangement gives a three dimensional, continuous, silicon-oxygen
network. A quartz crystal is a macromolecule of silicon dioxide. The difference between these two compounds is the ability of the group 14 elements
to form strong π bonds. Second-period elements, such as carbon, form very strong π bonds, which is why carbon dioxide forms small molecules with
strong double bonds. Elements below the second period, such as silicon, do not form π bonds as readily as second-period elements, and when they
do form, the π bonds are weaker than those formed by second-period elements. For this reason, silicon dioxide does not contain π bonds but only σ
bonds.

Access for free at OpenStax 18.3.5 https://chem.libretexts.org/@go/page/38314


Figure 18.3.7 : Because carbon tends to form double and triple bonds and silicon does not, (a) carbon dioxide is a discrete molecule with two C=O
double bonds and (b) silicon dioxide is an infinite network of oxygen atoms bridging between silicon atoms with each silicon atom possessing four
Si-O single bonds. (credit a photo: modification of work by Erica Gerdes; credit b photo: modification of work by Didier Descouens)
At 1600 °C, quartz melts to yield a viscous liquid. When the liquid cools, it does not crystallize readily but usually supercools and forms a glass, also
called silica. The SiO4 tetrahedra in glassy silica have a random arrangement characteristic of supercooled liquids, and the glass has some very
useful properties. Silica is highly transparent to both visible and ultraviolet light. For this reason, it is important in the manufacture of lamps that
give radiation rich in ultraviolet light and in certain optical instruments that operate with ultraviolet light. The coefficient of expansion of silica glass
is very low; therefore, rapid temperature changes do not cause it to fracture. CorningWare and other ceramic cookware contain amorphous silica.
Silicates are salts containing anions composed of silicon and oxygen. In nearly all silicates, sp3-hybridized silicon atoms occur at the centers of
tetrahedra with oxygen at the corners. There is a variation in the silicon-to-oxygen ratio that occurs because silicon-oxygen tetrahedra may exist as
discrete, independent units or may share oxygen atoms at corners in a variety of ways. In addition, the presence of a variety of cations gives rise to
the large number of silicate minerals. Many ceramics are composed of silicates. By including small amounts of other compounds, it is possible to
modify the physical properties of the silicate materials to produce ceramics with useful characteristics.

Summary
The elements boron, silicon, germanium, arsenic, antimony, and tellurium separate the metals from the nonmetals in the periodic table. These
elements, called metalloids or sometimes semimetals, exhibit properties characteristic of both metals and nonmetals. The structures of these
elements are similar in many ways to those of nonmetals, but the elements are electrical semiconductors.

Glossary
amorphous
solid material such as a glass that does not have a regular repeating component to its three-dimensional structure; a solid but not a crystal

borate
compound containing boron-oxygen bonds, typically with clusters or chains as a part of the chemical structure

polymorph
variation in crystalline structure that results in different physical properties for the resulting compound

silicate
compound containing silicon-oxygen bonds, with silicate tetrahedra connected in rings, sheets, or three-dimensional networks, depending on the
other elements involved in the formation of the compounds

This page titled 18.3: Structure and General Properties of the Metalloids is shared under a CC BY 4.0 license and was authored, remixed, and/or curated by
OpenStax via source content that was edited to the style and standards of the LibreTexts platform; a detailed edit history is available upon request.

Access for free at OpenStax 18.3.6 https://chem.libretexts.org/@go/page/38314


18.4: Structure and General Properties of the Nonmetals
 Learning Objectives
Describe structure and properties of nonmetals

The nonmetals are elements located in the upper right portion of the periodic table. Their properties and behavior are quite different
from those of metals on the left side. Under normal conditions, more than half of the nonmetals are gases, one is a liquid, and the
rest include some of the softest and hardest of solids. The nonmetals exhibit a rich variety of chemical behaviors. They include the
most reactive and least reactive of elements, and they form many different ionic and covalent compounds. This section presents an
overview of the properties and chemical behaviors of the nonmetals, as well as the chemistry of specific elements. Many of these
nonmetals are important in biological systems.
In many cases, trends in electronegativity enable us to predict the type of bonding and the physical states in compounds involving
the nonmetals. We know that electronegativity decreases as we move down a given group and increases as we move from left to
right across a period. The nonmetals have higher electronegativities than do metals, and compounds formed between metals and
nonmetals are generally ionic in nature because of the large differences in electronegativity between them. The metals form cations,
the nonmetals form anions, and the resulting compounds are solids under normal conditions. On the other hand, compounds formed
between two or more nonmetals have small differences in electronegativity between the atoms, and covalent bonding—sharing of
electrons—results. These substances tend to be molecular in nature and are gases, liquids, or volatile solids at room temperature
and pressure.
In normal chemical processes, nonmetals do not form monatomic positive ions (cations) because their ionization energies are too
high. All monatomic nonmetal ions are anions; examples include the chloride ion, Cl−, the nitride ion, N3−, and the selenide ion,
Se2−. The common oxidation states that the nonmetals exhibit in their ionic and covalent compounds are shown in Figure 18.4.1.
Remember that an element exhibits a positive oxidation state when combined with a more electronegative element and that it
exhibits a negative oxidation state when combined with a less electronegative element.

Figure 18.4.1 : Nonmetals exhibit these common oxidation states in ionic and covalent compounds.
The first member of each nonmetal group exhibits different behaviors, in many respects, from the other group members. The
reasons for this include smaller size, greater ionization energy, and (most important) the fact that the first member of each group
has only four valence orbitals (one 2s and three 2p) available for bonding, whereas other group members have empty d orbitals in
their valence shells, making possible five, six, or even more bonds around the central atom. For example, nitrogen forms only NF3,
whereas phosphorus forms both PF3 and PF5.

Remember that an element exhibits a positive oxidation state when combined with a more
electronegative element and that it exhibits a negative oxidation state when combined
with a less electronegative element.
Another difference between the first group member and subsequent members is the greater ability of the first member to form π
bonds. This is primarily a function of the smaller size of the first member of each group, which allows better overlap of atomic
orbitals. Nonmetals, other than the first member of each group, rarely form π bonds to nonmetals that are the first member of a
group. For example, sulfur-oxygen π bonds are well known, whereas sulfur does not normally form stable π bonds to itself.
The variety of oxidation states displayed by most of the nonmetals means that many of their chemical reactions involve changes in
oxidation state through oxidation-reduction reactions. There are five general aspects of the oxidation-reduction chemistry:
1. Nonmetals oxidize most metals. The oxidation state of the metal becomes positive as it undergoes oxidation and that of the
nonmetal becomes negative as it undergoes reduction. For example:

Access for free at OpenStax 18.4.1 https://chem.libretexts.org/@go/page/38315


4 Fe (s) + 3 O (g) → 2 Fe O (s)
2 2 3
0
0 +3 −2

2. With the exception of nitrogen and carbon, which are poor oxidizing agents, a more electronegative nonmetal oxidizes a less
electronegative nonmetal or the anion of the nonmetal:
S (s) + O (g) → 2 S O (s)
2 2
0 +4
0 −2

− −
Cl (g) + 2 I (aq) → I (s) + 2 Cl (aq)
2 2
0 0

Fluorine and oxygen are the strongest oxidizing agents within their respective groups; each oxidizes all the elements that lie
below it in the group. Within any period, the strongest oxidizing agent is in group 17. A nonmetal often oxidizes an element that
lies to its left in the same period. For example:

2 As (s) + 3 Br (l) → 2 AsBr (s)


2 3
0 +3
0 −1

The stronger a nonmetal is as an oxidizing agent, the more difficult it is to oxidize the anion formed by the nonmetal. This
means that the most stable negative ions are formed by elements at the top of the group or in group 17 of the period.
Fluorine and oxygen are the strongest oxidizing elements known. Fluorine does not form compounds in which it exhibits
positive oxidation states; oxygen exhibits a positive oxidation state only when combined with fluorine. For example:
− −
2F (g) + 2 OH (aq) → O F (g) + 2 F (aq) + H O (l)
2 2 2
+2
0 −1

With the exception of most of the noble gases, all nonmetals form compounds with oxygen, yielding covalent oxides. Most of these
oxides are acidic, that is, they react with water to form oxyacids. Recall from the acid-base chapter that an oxyacid is an acid
consisting of hydrogen, oxygen, and some other element. Notable exceptions are carbon monoxide, CO, nitrous oxide, N2O, and
nitric oxide, NO. There are three characteristics of these acidic oxides:
1. Oxides such as SO2 and N2O5, in which the nonmetal exhibits one of its common oxidation states, are acid anhydrides and react
with water to form acids with no change in oxidation state. The product is an oxyacid. For example:

SO (g) + H O(l) ⟶ H SO (aq)


2 2 2 3

N O (s) + H O(l) ⟶ 2 HNO (aq)


2 5 2 3

2. Those oxides such as NO2 and ClO2, in which the nonmetal does not exhibit one of its common oxidation states, also react with
water. In these reactions, the nonmetal is both oxidized and reduced. For example:

3 NO (g) + H O (l) → 2H N O (aq) + N O (g)


2 2 3
+4 +5 +2

Reactions in which the same element is both oxidized and reduced are called disproportionation reactions.
3. The acid strength increases as the electronegativity of the central atom increases. To learn more, see the discussion in the
chapter on acid-base chemistry.
The binary hydrogen compounds of the nonmetals also exhibit an acidic behavior in water, although only HCl, HBr, and HI are
strong acids. The acid strength of the nonmetal hydrogen compounds increases from left to right across a period and down a group.
For example, ammonia, NH3, is a weaker acid than is water, H2O, which is weaker than is hydrogen fluoride, HF. Water, H2O, is
also a weaker acid than is hydrogen sulfide, H2S, which is weaker than is hydrogen selenide, H2Se. Weaker acidic character implies
greater basic character.

18.4.1: Structures of the Nonmetals


The structures of the nonmetals differ dramatically from those of metals. Metals crystallize in closely packed arrays that do not
contain molecules or covalent bonds. Nonmetal structures contain covalent bonds, and many nonmetals consist of individual
molecules. The electrons in nonmetals are localized in covalent bonds, whereas in a metal, there is delocalization of the electrons
throughout the solid.

Access for free at OpenStax 18.4.2 https://chem.libretexts.org/@go/page/38315


The noble gases are all monatomic, whereas the other nonmetal gases—hydrogen, nitrogen, oxygen, fluorine, and chlorine—
normally exist as the diatomic molecules H2, N2, O2, F2, and Cl2. The other halogens are also diatomic; Br2 is a liquid and I2 exists
as a solid under normal conditions. The changes in state as one moves down the halogen family offer excellent examples of the
increasing strength of intermolecular London forces with increasing molecular mass and increasing polarizability.
Oxygen has two allotropes: O2, dioxygen, and O3, ozone. Phosphorus has three common allotropes, commonly referred to by their
colors: white, red, and black. Sulfur has several allotropes. There are also many carbon allotropes. Most people know of diamond,
graphite, and charcoal, but fewer people know of the recent discovery of fullerenes, carbon nanotubes, and graphene.
Descriptions of the physical properties of three nonmetals that are characteristic of molecular solids follow.

18.4.2: Carbon
Carbon occurs in the uncombined (elemental) state in many forms, such as diamond, graphite, charcoal, coke, carbon black,
graphene, and fullerene.
Diamond, shown in Figure 18.4.2, is a very hard crystalline material that is colorless and transparent when pure. Each atom forms
four single bonds to four other atoms at the corners of a tetrahedron (sp3 hybridization); this makes the diamond a giant molecule.
Carbon-carbon single bonds are very strong, and, because they extend throughout the crystal to form a three-dimensional network,
the crystals are very hard and have high melting points (~4400 °C).

Figure 18.4.2 : (a) Diamond and (b) graphite are two forms of carbon. (c) In the crystal structure of diamond, the covalent bonds
form three-dimensional tetrahedrons. (d) In the crystal structure of graphite, each planar layer is composed of six-membered rings.
(credit a: modification of work by “Fancy Diamonds”/Flickr; credit b: modification of work from Carbon [images-of-
elements.com])
Graphite, also shown in Figure 18.4.2, is a soft, slippery, grayish-black solid that conducts electricity. These properties relate to its
structure, which consists of layers of carbon atoms, with each atom surrounded by three other carbon atoms in a trigonal planar
arrangement. Each carbon atom in graphite forms three σ bonds, one to each of its nearest neighbors, by means of sp2-hybrid
orbitals. The unhybridized p orbital on each carbon atom will overlap unhybridized orbitals on adjacent carbon atoms in the same
layer to form π bonds. Many resonance forms are necessary to describe the electronic structure of a graphite layer; Figure 18.4.3
illustrates two of these forms.

Figure 18.4.3 : (a) Carbon atoms in graphite have unhybridized p orbitals. Each p orbital is perpendicular to the plane of carbon
atoms. (b) These are two of the many resonance forms of graphite necessary to describe its electronic structure as a resonance
hybrid.
Atoms within a graphite layer are bonded together tightly by the σ and π bonds; however, the forces between layers are weak.
London dispersion forces hold the layers together. To learn more, see the discussion of these weak forces in the chapter on liquids
and solids. The weak forces between layers give graphite the soft, flaky character that makes it useful as the so-called “lead” in
pencils and the slippery character that makes it useful as a lubricant. The loosely held electrons in the resonating π bonds can move
throughout the solid and are responsible for the electrical conductivity of graphite.
Other forms of elemental carbon include carbon black, charcoal, and coke. Carbon black is an amorphous form of carbon prepared
by the incomplete combustion of natural gas, CH4. It is possible to produce charcoal and coke by heating wood and coal,
respectively, at high temperatures in the absence of air.

Access for free at OpenStax 18.4.3 https://chem.libretexts.org/@go/page/38315


Figure 18.4.4 : The molecular structure of C60, buckminsterfullerene, is icosahedral.
Recently, new forms of elemental carbon molecules have been identified in the soot generated by a smoky flame and in the vapor
produced when graphite is heated to very high temperatures in a vacuum or in helium. One of these new forms, first isolated by
Professor Richard Smalley and coworkers at Rice University, consists of icosahedral (soccer-ball-shaped) molecules that contain 60
carbon atoms, C60. This is buckminsterfullerene (often called bucky balls) after the architect Buckminster Fuller, who designed
domed structures, which have a similar appearance (Figure 18.4.4).

18.4.2.1: Nanotubes and Graphene


Graphene and carbon nanotubes are two recently discovered allotropes of carbon. Both of the forms bear some relationship to
graphite. Graphene is a single layer of graphite (one atom thick), as illustrated in Figure 18.4.2d, whereas carbon nanotubes roll the
layer into a small tube, as illustrated in Figure 18.4.5.

Figure 18.4.5 : (a) Graphene and (b) carbon nanotubes are both allotropes of carbon.
Graphene is a very strong, lightweight, and efficient conductor of heat and electricity discovered in 2003. As in graphite, the carbon
atoms form a layer of six-membered rings with sp2-hybridized carbon atoms at the corners. Resonance stabilizes the system and
leads to its conductivity. Unlike graphite, there is no stacking of the layers to give a three-dimensional structure. Andre Geim and
Kostya Novoselov at the University of Manchester won the 2010 Nobel Prize in Physics for their pioneering work characterizing
graphene.
The simplest procedure for preparing graphene is to use a piece of adhesive tape to remove a single layer of graphene from the
surface of a piece of graphite. This method works because there are only weak London dispersion forces between the layers in
graphite. Alternative methods are to deposit a single layer of carbon atoms on the surface of some other material (ruthenium,
iridium, or copper) or to synthesize it at the surface of silicon carbide via the sublimation of silicon.
There currently are no commercial applications of graphene. However, its unusual properties, such as high electron mobility and
thermal conductivity, should make it suitable for the manufacture of many advanced electronic devices and for thermal
management applications.
Carbon nanotubes are carbon allotropes, which have a cylindrical structure. Like graphite and graphene, nanotubes consist of rings
of sp2-hybridized carbon atoms. Unlike graphite and graphene, which occur in layers, the layers wrap into a tube and bond together
to produce a stable structure. The walls of the tube may be one atom or multiple atoms thick.

Access for free at OpenStax 18.4.4 https://chem.libretexts.org/@go/page/38315


Carbon nanotubes are extremely strong materials that are harder than diamond. Depending upon the shape of the nanotube, it may
be a conductor or semiconductor. For some applications, the conducting form is preferable, whereas other applications utilize the
semiconducting form.
The basis for the synthesis of carbon nanotubes is the generation of carbon atoms in a vacuum. It is possible to produce carbon
atoms by an electrical discharge through graphite, vaporization of graphite with a laser, and the decomposition of a carbon
compound.
The strength of carbon nanotubes will eventually lead to some of their most exciting applications, as a thread produced from
several nanotubes will support enormous weight. However, the current applications only employ bulk nanotubes. The addition of
nanotubes to polymers improves the mechanical, thermal, and electrical properties of the bulk material. There are currently
nanotubes in some bicycle parts, skis, baseball bats, fishing rods, and surfboards.

18.4.3: Phosphorus
The name phosphorus comes from the Greek words meaning light bringing. When phosphorus was first isolated, scientists noted
that it glowed in the dark and burned when exposed to air. Phosphorus is the only member of its group that does not occur in the
uncombined state in nature; it exists in many allotropic forms. We will consider two of those forms: white phosphorus and red
phosphorus.
White phosphorus is a white, waxy solid that melts at 44.2 °C and boils at 280 °C. It is insoluble in water, is very soluble in carbon
disulfide, and bursts into flame in air. As a solid, as a liquid, as a gas, and in solution, white phosphorus exists as P4 molecules with
four phosphorus atoms at the corners of a regular tetrahedron (Figure 18.4.5). Each phosphorus atom covalently bonds to the other
three atoms in the molecule by single covalent bonds. White phosphorus is the most reactive allotrope and is very toxic.

Figure 18.4.5 : (a) Because white phosphorus bursts into flame in air, it is stored in water. (b) The structure of white phosphorus
consists of P4 molecules arranged in a tetrahedron. (c) Red phosphorus is much less reactive than is white phosphorus. (d) The
structure of red phosphorus consists of networks of P4 tetrahedra joined by P-P single bonds. (credit a: modification of work from
Phosphorus [images-of-elements.com])
Heating white phosphorus to 270–300 °C in the absence of air yields red phosphorus. Red phosphorus is denser, has a higher
melting point (~600 °C), is much less reactive, is essentially nontoxic, and is easier and safer to handle than is white phosphorus.
Its structure is highly polymeric and appears to contain three-dimensional networks of P4 tetrahedra joined by P-P single bonds.
Red phosphorus is insoluble in solvents that dissolve white phosphorus. When red phosphorus is heated, P4 molecules sublime
from the solid.

18.4.4: Sulfur
The allotropy of sulfur is far greater and more complex than that of any other element. Sulfur is the brimstone referred to in the
Bible and other places, and references to sulfur occur throughout recorded history—right up to the relatively recent discovery that it
is a component of the atmospheres of Venus and of Io, a moon of Jupiter. The most common and most stable allotrope of sulfur is
yellow, rhombic sulfur, so named because of the shape of its crystals. Rhombic sulfur is the form to which all other allotropes
revert at room temperature. Crystals of rhombic sulfur melt at 113 °C. Cooling this liquid gives long needles of monoclinic sulfur.
This form is stable from 96 °C to the melting point, 119 °C. At room temperature, it gradually reverts to the rhombic form.
Both rhombic sulfur and monoclinic sulfur contain S8 molecules in which atoms form eight-membered, puckered rings that
resemble crowns (Figure 18.4.6). Each sulfur atom is bonded to each of its two neighbors in the ring by covalent S-S single bonds.

Access for free at OpenStax 18.4.5 https://chem.libretexts.org/@go/page/38315


Figure 18.4.6 : These four sulfur allotropes show eight-membered, puckered rings. Each sulfur atom bonds to each of its two
neighbors in the ring by covalent S-S single bonds. Here are (a) individual S8 rings, (b) S8 chains formed when the rings open, (c)
longer chains formed by adding sulfur atoms to S8 chains, and (d) part of the very long sulfur chains formed at higher temperatures.
When rhombic sulfur melts, the straw-colored liquid is quite mobile; its viscosity is low because S8 molecules are essentially
spherical and offer relatively little resistance as they move past each other. As the temperature rises, S-S bonds in the rings break,
and polymeric chains of sulfur atoms result. These chains combine end to end, forming still longer chains that tangle with one
another. The liquid gradually darkens in color and becomes so viscous that finally (at about 230 °C) it does not pour easily. The
dangling atoms at the ends of the chains of sulfur atoms are responsible for the dark red color because their electronic structure
differs from those of sulfur atoms that have bonds to two adjacent sulfur atoms. This causes them to absorb light differently and
results in a different visible color. Cooling the liquid rapidly produces a rubberlike amorphous mass, called plastic sulfur.
Sulfur boils at 445 °C and forms a vapor consisting of S2, S6, and S8 molecules; at about 1000 °C, the vapor density corresponds to
the formula S2, which is a paramagnetic molecule like O2 with a similar electronic structure and a weak sulfur-sulfur double bond.
As seen in this discussion, an important feature of the structural behavior of the nonmetals is that the elements usually occur with
eight electrons in their valence shells. If necessary, the elements form enough covalent bonds to supplement the electrons already
present to possess an octet. For example, members of group 15 have five valence electrons and require only three additional
electrons to fill their valence shells. These elements form three covalent bonds in their free state: triple bonds in the N2 molecule or
single bonds to three different atoms in arsenic and phosphorus. The elements of group 16 require only two additional electrons.
Oxygen forms a double bond in the O2 molecule, and sulfur, selenium, and tellurium form two single bonds in various rings and
chains. The halogens form diatomic molecules in which each atom is involved in only one bond. This provides the electron
required necessary to complete the octet on the halogen atom. The noble gases do not form covalent bonds to other noble gas atoms
because they already have a filled outer shell.

Summary
Nonmetals have structures that are very different from those of the metals, primarily because they have greater electronegativity
and electrons that are more tightly bound to individual atoms. Most nonmetal oxides are acid anhydrides, meaning that they react
with water to form acidic solutions. Molecular structures are common for most of the nonmetals, and several have multiple
allotropes with varying physical properties.

Glossary
acid anhydride
compound that reacts with water to form an acid or acidic solution

disproportionation reaction
chemical reaction where a single reactant is simultaneously reduced and oxidized; it is both the reducing agent and the
oxidizing agent

Access for free at OpenStax 18.4.6 https://chem.libretexts.org/@go/page/38315


This page titled 18.4: Structure and General Properties of the Nonmetals is shared under a CC BY 4.0 license and was authored, remixed, and/or
curated by OpenStax via source content that was edited to the style and standards of the LibreTexts platform; a detailed edit history is available
upon request.

Access for free at OpenStax 18.4.7 https://chem.libretexts.org/@go/page/38315


18.5: Occurrence, Preparation, and Compounds of Hydrogen
 Learning Objectives
Describe the properties, preparation, and compounds of hydrogen

Hydrogen is the most abundant element in the universe. The sun and other stars are composed largely of hydrogen. Astronomers
estimate that 90% of the atoms in the universe are hydrogen atoms. Hydrogen is a component of more compounds than any other
element. Water is the most abundant compound of hydrogen found on earth. Hydrogen is an important part of petroleum, many
minerals, cellulose and starch, sugar, fats, oils, alcohols, acids, and thousands of other substances.
At ordinary temperatures, hydrogen is a colorless, odorless, tasteless, and nonpoisonous gas consisting of the diatomic molecule
H2. Hydrogen is composed of three isotopes, and unlike other elements, these isotopes have different names and chemical symbols:
protium, 1H, deuterium, 2H (or “D”), and tritium 3H (or “T”). In a naturally occurring sample of hydrogen, there is one atom of
deuterium for every 7000 H atoms and one atom of radioactive tritium for every 1018 H atoms. The chemical properties of the
different isotopes are very similar because they have identical electron structures, but they differ in some physical properties
because of their differing atomic masses. Elemental deuterium and tritium have lower vapor pressure than ordinary hydrogen.
Consequently, when liquid hydrogen evaporates, the heavier isotopes are concentrated in the last portions to evaporate. Electrolysis
of heavy water, D2O, yields deuterium. Most tritium originates from nuclear reactions.

18.5.1: Preparation of Hydrogen


Elemental hydrogen must be prepared from compounds by breaking chemical bonds. The most common methods of preparing
hydrogen follow.

18.5.1.1: From Steam and Carbon or Hydrocarbons


Water is the cheapest and most abundant source of hydrogen. Passing steam over coke (an impure form of elemental carbon) at
1000 °C produces a mixture of carbon monoxide and hydrogen known as water gas:
1000 °C

C(s) + H O(g) −−−−→ CO(g) + H (g)


2 2

water gas

Water gas is as an industrial fuel. It is possible to produce additional hydrogen by mixing the water gas with steam in the presence
of a catalyst to convert the CO to CO2. This reaction is the water gas shift reaction.
It is also possible to prepare a mixture of hydrogen and carbon monoxide by passing hydrocarbons from natural gas or petroleum
and steam over a nickel-based catalyst. Propane is an example of a hydrocarbon reactant:
900 °C

C H (g) + 3 H O(g) −−−−→ 3 CO(g) + 7 H (g)


3 8 2 2
catalyst

18.5.1.2: Electrolysis
Hydrogen forms when direct current electricity passes through water (electrolysis) containing an electrolyte such as H2SO4, (Figure
18.5.1.) Bubbles of hydrogen form at the cathode, and oxygen evolves at the anode. The net reaction is:

2 H O(l) + electrical energy ⟶ 2 H (g) + O (g)


2 2 2

Access for free at OpenStax 18.5.1 https://chem.libretexts.org/@go/page/38316


Figure 18.5.1 : The electrolysis of water produces hydrogen and oxygen. Because there are twice as many hydrogen atoms as
oxygen atoms and both elements are diatomic, there is twice the volume of hydrogen produced at the cathode as there is oxygen
produced at the anode.

18.5.1.3: Reaction of Metals with Acids


This is the most convenient laboratory method of producing hydrogen. Metals with lower reduction potentials reduce the hydrogen
ion in dilute acids to produce hydrogen gas and metal salts. For example, as shown in Figure 18.5.2, iron in dilute hydrochloric
acid produces hydrogen gas and iron(II) chloride:
+ − 2+ −
Fe(s) + 2 H O (aq) + 2 Cl (aq) ⟶ Fe (aq) + 2 Cl (aq) + H (g) + 2 H O(l)
3 2 2

Figure 18.5.2 : The reaction of iron with an acid produces hydrogen. Here, iron reacts with hydrochloric acid. (credit: Mark Ott)

18.5.1.4: Reaction of Ionic Metal Hydrides with Water


It is possible to produce hydrogen from the reaction of hydrides of the active metals, which contain the very strongly basic H−
anion, with water:
2+ −
CaH (s) + 2 H O(l) ⟶ Ca (aq) + 2 OH (aq) + 2 H (g)
2 2 2

Metal hydrides are expensive but convenient sources of hydrogen, especially where space and weight are important factors. They
are important in the inflation of life jackets, life rafts, and military balloons.

18.5.2: Reactions
Under normal conditions, hydrogen is relatively inactive chemically, but when heated, it enters into many chemical reactions.
Two thirds of the world’s hydrogen production is devoted to the manufacture of ammonia, which is a fertilizer and used in the
manufacture of nitric acid. Large quantities of hydrogen are also important in the process of hydrogenation, discussed in the
chapter on organic chemistry.
It is possible to use hydrogen as a nonpolluting fuel. The reaction of hydrogen with oxygen is a very exothermic reaction, releasing
286 kJ of energy per mole of water formed. Hydrogen burns without explosion under controlled conditions. The oxygen-hydrogen
torch, because of the high heat of combustion of hydrogen, can achieve temperatures up to 2800 °C. The hot flame of this torch is
useful in cutting thick sheets of many metals. Liquid hydrogen is also an important rocket fuel (Figure 18.5.3).

Access for free at OpenStax 18.5.2 https://chem.libretexts.org/@go/page/38316


Figure 18.5.3 : Before the fleet’s retirement in 2011, liquid hydrogen and liquid oxygen were used in the three main engines of a
space shuttle. Two compartments in the large tank held these liquids until the shuttle was launched. (credit: “reynermedia”/Flickr)
An uncombined hydrogen atom consists of a nucleus and one valence electron in the 1s orbital. The n = 1 valence shell has a
capacity for two electrons, and hydrogen can rightfully occupy two locations in the periodic table. It is possible to consider
hydrogen a group 1 element because hydrogen can lose an electron to form the cation, H+. It is also possible to consider hydrogen
to be a group 17 element because it needs only one electron to fill its valence orbital to form a hydride ion, H−, or it can share an
electron to form a single, covalent bond. In reality, hydrogen is a unique element that almost deserves its own location in the
periodic table.

18.5.2.1: Reactions with Elements


When heated, hydrogen reacts with the metals of group 1 and with Ca, Sr, and Ba (the more active metals in group 2). The
compounds formed are crystalline, ionic hydrides that contain the hydride anion, H−, a strong reducing agent and a strong base,
which reacts vigorously with water and other acids to form hydrogen gas.
The reactions of hydrogen with nonmetals generally produce acidic hydrogen compounds with hydrogen in the 1+ oxidation state.
The reactions become more exothermic and vigorous as the electronegativity of the nonmetal increases. Hydrogen reacts with
nitrogen and sulfur only when heated, but it reacts explosively with fluorine (forming HF) and, under some conditions, with
chlorine (forming HCl). A mixture of hydrogen and oxygen explodes if ignited. Because of the explosive nature of the reaction, it
is necessary to exercise caution when handling hydrogen (or any other combustible gas) to avoid the formation of an explosive
mixture in a confined space. Although most hydrides of the nonmetals are acidic, ammonia and phosphine (PH3) are very, very
weak acids and generally function as bases. There is a summary of these reactions of hydrogen with the elements in Table 18.5.1.
Table 18.5.1 : Chemical Reactions of Hydrogen with Other Elements
General Equation Comments

MH or MH
2
⟶ MOH or M(OH)
2
+H
2
ionic hydrides with group 1 and Ca, Sr, and Ba

H + C ⟶ (no reaction)
2

3H
2
+N
2
⟶ 2 NH
3
requires high pressure and temperature; low yield

2H
2
+O
2
⟶ 2H O
2
exothermic and potentially explosive

H
2
+S ⟶ H S
2
requires heating; low yield

H
2
+X
2
⟶ 2 HX X = F, Cl, Br, and I; explosive with F2; low yield with I2

18.5.3: Reaction with Compounds


Hydrogen reduces the heated oxides of many metals, with the formation of the metal and water vapor. For example, passing
hydrogen over heated CuO forms copper and water. Hydrogen may also reduce the metal ions in some metal oxides to lower
oxidation states:
Δ

H (g) + MnO (s) −


→ MnO(s) + H O(g)
2 2 2

18.5.4: Hydrogen Compounds


Other than the noble gases, each of the nonmetals forms compounds with hydrogen. For brevity, we will discuss only a few
hydrogen compounds of the nonmetals here.

Access for free at OpenStax 18.5.3 https://chem.libretexts.org/@go/page/38316


18.5.4.1: Nitrogen Hydrogen Compounds
Ammonia, NH3, forms naturally when any nitrogen-containing organic material decomposes in the absence of air. The laboratory
preparation of ammonia is by the reaction of an ammonium salt with a strong base such as sodium hydroxide. The acid-base
reaction with the weakly acidic ammonium ion gives ammonia, illustrated in Figure 18.5.4. Ammonia also forms when ionic
nitrides react with water. The nitride ion is a much stronger base than the hydroxide ion:

Mg N (s) + 6 H O(l) ⟶ 3 Mg (OH) (s) + 2 NH (g)


3 2 2 2 3

The commercial production of ammonia is by the direct combination of the elements in the Haber process:
catalyst

N (g) + 3 H (g) ⇌ 2 NH (g) ΔH ° = −92 kJ


2 2 3

Figure 18.5.4 : The structure of ammonia is shown with a central nitrogen atom and three hydrogen atoms.
Ammonia is a colorless gas with a sharp, pungent odor. Smelling salts utilize this powerful odor. Gaseous ammonia readily
liquefies to give a colorless liquid that boils at −33 °C. Due to intermolecular hydrogen bonding, the enthalpy of vaporization of
liquid ammonia is higher than that of any other liquid except water, so ammonia is useful as a refrigerant. Ammonia is quite soluble
in water (658 L at STP dissolves in 1 L H2O).
The chemical properties of ammonia are as follows:
1. Ammonia acts as a Brønsted base, as discussed in the chapter on acid-base chemistry. The ammonium ion is similar in size to
the potassium ion; compounds of the two ions exhibit many similarities in their structures and solubilities.
2. Ammonia can display acidic behavior, although it is a much weaker acid than water. Like other acids, ammonia reacts with
metals, although it is so weak that high temperatures are necessary. Hydrogen and (depending on the stoichiometry) amides
(salts of NH ), imides (salts of NH2−), or nitrides (salts of N3−) form.

2

3. The nitrogen atom in ammonia has its lowest possible oxidation state (3−) and thus is not susceptible to reduction. However, it
can be oxidized. Ammonia burns in air, giving NO and water. Hot ammonia and the ammonium ion are active reducing agents.
Of particular interest are the oxidations of ammonium ion by nitrite ion, NO , to yield pure nitrogen and by nitrate ion to yield

nitrous oxide, N2O.


4. There are a number of compounds that we can consider derivatives of ammonia through the replacement of one or more
hydrogen atoms with some other atom or group of atoms. Inorganic derivations include chloramine, NH2Cl, and hydrazine,
N2H4:

Chloramine, NH2Cl, results from the reaction of sodium hypochlorite, NaOCl, with ammonia in basic solution. In the presence of a
large excess of ammonia at low temperature, the chloramine reacts further to produce hydrazine, N2H4:
− −
NH (aq) + OCl (aq) ⟶ NH Cl(aq) + OH (aq)
3 2

− −
NH Cl(aq) + NH (aq) + OH (aq) ⟶ N H (aq) + Cl (aq) + H O(l)
2 3 2 4 2

Anhydrous hydrazine is relatively stable in spite of its positive free energy of formation:
∘ −1
N (g) + 2 H (g) ⟶ N H (l) ΔG = 149.2 kJ mo l
2 2 2 4 f

Hydrazine is a fuming, colorless liquid that has some physical properties remarkably similar to those of H2O (it melts at 2 °C, boils
at 113.5 °C, and has a density at 25 °C of 1.00 g/mL). It burns rapidly and completely in air with substantial evolution of heat:

Access for free at OpenStax 18.5.4 https://chem.libretexts.org/@go/page/38316


−1
N H (l) + O (g) ⟶ N (g) + 2 H O(l) ΔH ° = −621.5 kJ mo l
2 4 2 2 2

Like ammonia, hydrazine is both a Brønsted base and a Lewis base, although it is weaker than ammonia. It reacts with strong acids
and forms two series of salts that contain the N H and N H ions, respectively. Some rockets use hydrazine as a fuel.
2
+

5 2
2+

18.5.5: Phosphorus Hydrogen Compounds


The most important hydride of phosphorus is phosphine, PH3, a gaseous analog of ammonia in terms of both formula and structure.
Unlike ammonia, it is not possible to form phosphine by direct union of the elements. There are two methods for the preparation of
phosphine. One method is by the action of an acid on an ionic phosphide. The other method is the disproportionation of white
phosphorus with hot concentrated base to produce phosphine and the hydrogen phosphite ion:
+ 3+
AlP(s) + 3 H O (aq) ⟶ PH (g) + Al (aq) + 3 H O(l)
3 3 2

− 2−
P (s) + 4 OH (aq) + 2 H O(l) ⟶ 2 HPO (aq) + 2 PH (g)
4 2 3 3

Phosphine is a colorless, very poisonous gas, which has an odor like that of decaying fish. Heat easily decomposes phosphine
(4 PH ⟶ P + 6 H ) , and the compound burns in air. The major uses of phosphine are as a fumigant for grains and in
3 4 2

semiconductor processing. Like ammonia, gaseous phosphine unites with gaseous hydrogen halides, forming phosphonium
compounds like PH4Cl and PH4I. Phosphine is a much weaker base than ammonia; therefore, these compounds decompose in
water, and the insoluble PH3 escapes from solution.

18.5.5.1: Sulfur Hydrogen Compounds


Hydrogen sulfide, H2S, is a colorless gas that is responsible for the offensive odor of rotten eggs and of many hot springs.
Hydrogen sulfide is as toxic as hydrogen cyanide; therefore, it is necessary to exercise great care in handling it. Hydrogen sulfide is
particularly deceptive because it paralyzes the olfactory nerves; after a short exposure, one does not smell it.
The production of hydrogen sulfide by the direct reaction of the elements (H2 + S) is unsatisfactory because the yield is low. A
more effective preparation method is the reaction of a metal sulfide with a dilute acid. For example:
+ 2+
FeS(s) + 2 H O (aq) ⟶ Fe (aq) + H S(g) + 2 H O(l)
3 2 2

It is easy to oxidize the sulfur in metal sulfides and in hydrogen sulfide, making metal sulfides and H2S good reducing agents. In
acidic solutions, hydrogen sulfide reduces Fe3+ to Fe2+, MnO to Mn2+, Cr O to Cr3+, and HNO3 to NO2. The sulfur in H2S

4 2
2−
7

usually oxidizes to elemental sulfur, unless a large excess of the oxidizing agent is present. In which case, the sulfide may oxidize
to SO or SO (or to SO2 or SO3 in the absence of water):
2−
3
2−
4

2 H S(g) + O (g) ⟶ 2 S(s) + 2 H O(l)


2 2 2

This oxidation process leads to the removal of the hydrogen sulfide found in many sources of natural gas. The deposits of sulfur in
volcanic regions may be the result of the oxidation of H2S present in volcanic gases.
Hydrogen sulfide is a weak diprotic acid that dissolves in water to form hydrosulfuric acid. The acid ionizes in two stages, yielding
hydrogen sulfide ions, HS−, in the first stage and sulfide ions, S2−, in the second. Since hydrogen sulfide is a weak acid, aqueous
solutions of soluble sulfides and hydrogen sulfides are basic:
2− − −
S (aq) + H O(l) ⇌ HS (aq) + OH (aq)
2

− −
HS (aq) + H O(l) ⇌ H S(g) + OH (aq)
2 2

18.5.5.2: Halogen Hydrogen Compounds


Binary compounds containing only hydrogen and a halogen are hydrogen halides. At room temperature, the pure hydrogen halides
HF, HCl, HBr, and HI are gases.
In general, it is possible to prepare the halides by the general techniques used to prepare other acids. Fluorine, chlorine, and
bromine react directly with hydrogen to form the respective hydrogen halide. This is a commercially important reaction for
preparing hydrogen chloride and hydrogen bromide.
The acid-base reaction between a nonvolatile strong acid and a metal halide will yield a hydrogen halide. The escape of the gaseous
hydrogen halide drives the reaction to completion. For example, the usual method of preparing hydrogen fluoride is by heating a

Access for free at OpenStax 18.5.5 https://chem.libretexts.org/@go/page/38316


mixture of calcium fluoride, CaF2, and concentrated sulfuric acid:

CaF (s) + H SO (aq) ⟶ CaSO (s) + 2 HF(g)


2 2 4 4

Gaseous hydrogen fluoride is also a by-product in the preparation of phosphate fertilizers by the reaction of fluoroapatite,
Ca5(PO4)3F, with sulfuric acid. The reaction of concentrated sulfuric acid with a chloride salt produces hydrogen chloride both
commercially and in the laboratory.
In most cases, sodium chloride is the chloride of choice because it is the least expensive chloride. Hydrogen bromide and hydrogen
iodide cannot be prepared using sulfuric acid because this acid is an oxidizing agent capable of oxidizing both bromide and iodide.
However, it is possible to prepare both hydrogen bromide and hydrogen iodide using an acid such as phosphoric acid because it is a
weaker oxidizing agent. For example:
− −
H PO (l) + Br (aq) ⟶ HBr(g) + H PO (aq)
3 4 2 4

All of the hydrogen halides are very soluble in water, forming hydrohalic acids. With the exception of hydrogen fluoride, which has
a strong hydrogen-fluoride bond, they are strong acids. Reactions of hydrohalic acids with metals, metal hydroxides, oxides, or
carbonates produce salts of the halides. Most chloride salts are soluble in water. AgCl, PbCl2, and Hg2Cl2 are the commonly
encountered exceptions.
The halide ions give the substances the properties associated with X−(aq). The heavier halide ions (Cl−, Br−, and I−) can act as
reducing agents, and the lighter halogens or other oxidizing agents will oxidize them:
− −
Cl (aq) + 2 e ⟶ 2 Cl (aq) E° = 1.36 V
2

− −
Br (aq) + 2 e ⟶ 2 Br (aq) E° = 1.09 V
2

− −
I (aq) + 2 e ⟶ 2I (aq) E° = 0.54 V
2

For example, bromine oxidizes iodine:


Br (aq) + 2 HI(aq) ⟶ 2 HBr(aq) + I (aq) E° = 0.55 V
2 2

Hydrofluoric acid is unique in its reactions with sand (silicon dioxide) and with glass, which is a mixture of silicates:

SiO (s) + 4 HF(aq) ⟶ SiF (g) + 2 H O(l)


2 4 2

CaSiO (s) + 6 HF(aq) ⟶ CaF (s) + SiF (g) + 3 H O(l)


3 2 4 2

The volatile silicon tetrafluoride escapes from these reactions. Because hydrogen fluoride attacks glass, it can frost or etch glass
and is used to etch markings on thermometers, burets, and other glassware.
The largest use for hydrogen fluoride is in production of hydrochlorofluorocarbons for refrigerants, in plastics, and in propellants.
The second largest use is in the manufacture of cryolite, Na3AlF6, which is important in the production of aluminum. The acid is
also important in the production of other inorganic fluorides (such as BF3), which serve as catalysts in the industrial synthesis of
certain organic compounds.
Hydrochloric acid is relatively inexpensive. It is an important and versatile acid in industry and is important for the manufacture of
metal chlorides, dyes, glue, glucose, and various other chemicals. A considerable amount is also important for the activation of oil
wells and as pickle liquor—an acid used to remove oxide coating from iron or steel that is to be galvanized, tinned, or enameled.
The amounts of hydrobromic acid and hydroiodic acid used commercially are insignificant by comparison.

Summary
Hydrogen is the most abundant element in the universe and its chemistry is truly unique. Although it has some chemical reactivity
that is similar to that of the alkali metals, hydrogen has many of the same chemical properties of a nonmetal with a relatively low
electronegativity. It forms ionic hydrides with active metals, covalent compounds in which it has an oxidation state of 1− with less
electronegative elements, and covalent compounds in which it has an oxidation state of 1+ with more electronegative nonmetals. It
reacts explosively with oxygen, fluorine, and chlorine, less readily with bromine, and much less readily with iodine, sulfur, and
nitrogen. Hydrogen reduces the oxides of metals with lower reduction potentials than chromium to form the metal and water. The
hydrogen halides are all acidic when dissolved in water.

Access for free at OpenStax 18.5.6 https://chem.libretexts.org/@go/page/38316


Glossary
Haber process
main industrial process used to produce ammonia from nitrogen and hydrogen; involves the use of an iron catalyst and elevated
temperatures and pressures

hydrogen halide
binary compound formed between hydrogen and the halogens: HF, HCl, HBr, and HI

hydrogenation
addition of hydrogen (H2) to reduce a compound

This page titled 18.5: Occurrence, Preparation, and Compounds of Hydrogen is shared under a CC BY 4.0 license and was authored, remixed,
and/or curated by OpenStax via source content that was edited to the style and standards of the LibreTexts platform; a detailed edit history is
available upon request.

Access for free at OpenStax 18.5.7 https://chem.libretexts.org/@go/page/38316


18.6: Occurrence, Preparation, and Properties of Carbonates
 Learning Objectives
Describe the preparation, properties, and uses of some representative metal carbonates

The chemistry of carbon is extensive; however, most of this chemistry is not relevant to this chapter. The other aspects of the
chemistry of carbon will appear in the chapter covering organic chemistry. In this chapter, we will focus on the carbonate ion and
related substances. The metals of groups 1 and 2, as well as zinc, cadmium, mercury, and lead(II), form ionic carbonates—
compounds that contain the carbonate anions, CO . The metals of group 1, magnesium, calcium, strontium, and barium also form
2−
3

hydrogen carbonates—compounds that contain the hydrogen carbonate anion, HCO , also known as the bicarbonate anion.

3

With the exception of magnesium carbonate, it is possible to prepare carbonates of the metals of groups 1 and 2 by the reaction of
carbon dioxide with the respective oxide or hydroxide. Examples of such reactions include:
Na O(s) + CO (g) ⟶ Na CO (s) (18.6.1)
2 2 2 3

Ca (OH) (s) + CO (g) ⟶ CaCO (s) + H O(l) (18.6.2)


2 2 3 2

The carbonates of the alkaline earth metals of group 12 and lead(II) are not soluble. These carbonates precipitate upon mixing a
solution of soluble alkali metal carbonate with a solution of soluble salts of these metals. Examples of net ionic equations for the
reactions are:
2+ 2−
Ca (aq) + CO3 (aq) ⟶ CaCO (s) (18.6.3)
3

2+ 2−
Pb (aq) + CO (aq) ⟶ PbCO (s) (18.6.4)
3 3

Pearls and the shells of most mollusks are calcium carbonate. Tin(II) or one of the trivalent or tetravalent ions such as Al3+ or Sn4+
behave differently in this reaction as carbon dioxide and the corresponding oxide form instead of the carbonate.
Alkali metal hydrogen carbonates such as NaHCO3 and CsHCO3 form by saturating a solution of the hydroxides with carbon
dioxide. The net ionic reaction involves hydroxide ion and carbon dioxide:
− −
OH (aq) + CO (aq) ⟶ HCO (aq)
2 3

It is possible to isolate the solids by evaporation of the water from the solution.
Although they are insoluble in pure water, alkaline earth carbonates dissolve readily in water containing carbon dioxide because
hydrogen carbonate salts form. For example, caves and sinkholes form in limestone when CaCO3 dissolves in water containing
dissolved carbon dioxide:
2+ −
CaCO (s) + CO (aq) + H O(l) ⟶ Ca (aq) + 2 HCO (aq)
3 2 2 3

Hydrogen carbonates of the alkaline earth metals remain stable only in solution; evaporation of the solution produces the carbonate.
Stalactites and stalagmites, like those shown in Figure 18.6.1, form in caves when drops of water containing dissolved calcium
hydrogen carbonate evaporate to leave a deposit of calcium carbonate.

Access for free at OpenStax 18.6.1 https://chem.libretexts.org/@go/page/38317


Figure 18.6.1 : (a) Stalactites and (b) stalagmites are cave formations of calcium carbonate. (credit a: modification of work by
Arvind Govindaraj; credit b: modification of work by the National Park Service.)
The two carbonates used commercially in the largest quantities are sodium carbonate and calcium carbonate. In the United States,
sodium carbonate is extracted from the mineral trona, Na3(CO3)(HCO3)(H2O)2. Following recrystallization to remove clay and
other impurities, heating the recrystallized trona produces Na2CO3:
2 Na (CO )(HCO )(H O) (s) ⟶ 3 Na CO (s) + 5 H O(l) + CO (g)
3 3 3 2 2 2 3 2 2

Carbonates are moderately strong bases. Aqueous solutions are basic because the carbonate ion accepts hydrogen ion from water in
this reversible reaction:
2− − −
CO (aq) + H O(l) ⇌ HCO (aq) + OH (aq)
3 2 3

Carbonates react with acids to form salts of the metal, gaseous carbon dioxide, and water. The reaction of calcium carbonate, the
active ingredient of the antacid Tums, with hydrochloric acid (stomach acid), as shown in Figure 18.6.2, illustrates the reaction:

CaCO (s) + 2 HCl(aq) ⟶ CaCl (aq) + CO (g) + H O(l)


3 2 2 2

Figure 18.6.2 : The reaction of calcium carbonate with hydrochloric acid is shown. (credit: Mark Ott)
Other applications of carbonates include glass making—where carbonate ions serve as a source of oxide ions—and synthesis of
oxides.
Hydrogen carbonates are amphoteric because they act as both weak acids and weak bases. Hydrogen carbonate ions act as acids
and react with solutions of soluble hydroxides to form a carbonate and water:

KHCO (aq) + KOH(aq) ⟶ K CO (aq) + H O(l)


3 2 3 2

With acids, hydrogen carbonates form a salt, carbon dioxide, and water. Baking soda (bicarbonate of soda or sodium bicarbonate) is
sodium hydrogen carbonate. Baking powder contains baking soda and a solid acid such as potassium hydrogen tartrate (cream of
tartar), KHC4H4O6. As long as the powder is dry, no reaction occurs; immediately after the addition of water, the acid reacts with
the hydrogen carbonate ions to form carbon dioxide:

Access for free at OpenStax 18.6.2 https://chem.libretexts.org/@go/page/38317


− − 2−
HC H O6 (aq) + HCO3 (aq) ⟶ C H O6 (aq) + CO (g) + H O(l)
4 4 4 4 2 2

Dough will trap the carbon dioxide, causing it to expand during baking, producing the characteristic texture of baked goods.

Summary
The usual method for the preparation of the carbonates of the alkali and alkaline earth metals is by reaction of an oxide or
hydroxide with carbon dioxide. Other carbonates form by precipitation. Metal carbonates or hydrogen carbonates such as limestone
(CaCO3), the antacid Tums (CaCO3), and baking soda (NaHCO3) are common examples. Carbonates and hydrogen carbonates
decompose in the presence of acids and most decompose on heating.

Glossary
bicarbonate anion
salt of the hydrogen carbonate ion, HCO −
3

carbonate
salt of the anion CO 2−
3
; often formed by the reaction of carbon dioxide with bases

hydrogen carbonate
salt of carbonic acid, H2CO3 (containing the anion HCO ) in which one hydrogen atom has been replaced; an acid carbonate;

3

also known as bicarbonate ion

This page titled 18.6: Occurrence, Preparation, and Properties of Carbonates is shared under a CC BY 4.0 license and was authored, remixed,
and/or curated by OpenStax via source content that was edited to the style and standards of the LibreTexts platform; a detailed edit history is
available upon request.

Access for free at OpenStax 18.6.3 https://chem.libretexts.org/@go/page/38317


18.7: Occurrence, Preparation, and Properties of Nitrogen
 Learning Objectives
Describe the properties, preparation, and uses of nitrogen

Most pure nitrogen comes from the fractional distillation of liquid air. The atmosphere consists of 78% nitrogen by volume. This
means there are more than 20 million tons of nitrogen over every square mile of the earth’s surface. Nitrogen is a component of
proteins and of the genetic material (DNA/RNA) of all plants and animals.
Under ordinary conditions, nitrogen is a colorless, odorless, and tasteless gas. It boils at 77 K and freezes at 63 K. Liquid nitrogen
is a useful coolant because it is inexpensive and has a low boiling point. Nitrogen is very unreactive because of the very strong
triple bond between the nitrogen atoms. The only common reactions at room temperature occur with lithium to form Li3N, with
certain transition metal complexes, and with hydrogen or oxygen in nitrogen-fixing bacteria. The general lack of reactivity of
nitrogen makes the remarkable ability of some bacteria to synthesize nitrogen compounds using atmospheric nitrogen gas as the
source one of the most exciting chemical events on our planet. This process is one type of nitrogen fixation. In this case, nitrogen
fixation is the process where organisms convert atmospheric nitrogen into biologically useful chemicals. Nitrogen fixation also
occurs when lightning passes through air, causing molecular nitrogen to react with oxygen to form nitrogen oxides, which are then
carried down to the soil.

18.7.1: Nitrogen Fixation


All living organisms require nitrogen compounds for survival. Unfortunately, most of these organisms cannot absorb nitrogen from
its most abundant source—the atmosphere. Atmospheric nitrogen consists of N2 molecules, which are very unreactive due to the
strong nitrogen-nitrogen triple bond. However, a few organisms can overcome this problem through a process known as nitrogen
fixation, illustrated in Figure 18.7.1.

Access for free at OpenStax 18.7.1 https://chem.libretexts.org/@go/page/38318


Figure 18.7.1 : All living organisms require nitrogen. A few microorganisms are able to process atmospheric nitrogen using
nitrogen fixation. (credit “roots”: modification of work by the United States Department of Agriculture; credit “root nodules”:
modification of work by Louisa Howard)
A flow chart is shown. A cow, grass, and a tree are shown in the center of the diagram. Downward-facing arrows lead from them to
the phrase, “Decomposers ( aerobic and anaerobic bacteria and fungi ).” A downward-facing arrow leads to a space-filing model
with one blue atom bonded to four white atoms. The model is labeled, “Ammonium ( N H subscript 4 ).” A right-facing arrow
leads from this molecule to another molecule that is composed of a blue atom bonded to two red atoms. The model is labeled,
“Nitrites ( N O subscript 2 superscript negative sign ).” Below this arrow is a picture of a circle with two rod-shaped structures. It is
labeled, “Nitrifying bacteria.” Above the nitrites label is an upward-facing arrow leading to a blue atom single-bonded to three red
atoms. The model is labeled, “Nitrates ( N O subscript 3 superscript negative sign ).” Next to this arrow is a picture of a circle with
two rod-shaped structures labeled, “Nitrifying bacteria.” The nitrates label has a double-headed, upward-facing arrow that leads to
two pictures: one of the roots of the tree which is labeled, “Assimilation,” and one leading to a picture of a circle with four oval-
shaped structures labeled, “Denitrifying bacteria.” A left-facing arrow leads from this bacteria to a molecule made up of two atoms
triple-bonded together and labeled, “Atmospheric nitrogen ( N subscript 2 ).” This molecule is connected to a downward-facing,
double-headed arrow that leads to an image showing yellow filaments on a black background and a picture of a circle with four
rod-shaped structures labeled, “Nitrogen-fixing soil bacteria.” An arrow leads from a picture of a plant’s roots to the yellow
filaments and then to a photo of a circle with four oval-shaped structures labeled, “Nitrogen-fixing bacteria in root nodules.”
Nitrogen fixation is the process where organisms convert atmospheric nitrogen into biologically useful chemicals. To date, the only
known kind of biological organisms capable of nitrogen fixation are microorganisms. These organisms employ enzymes called
nitrogenases, which contain iron and molybdenum. Many of these microorganisms live in a symbiotic relationship with plants, with
the best-known example being the presence of rhizobia in the root nodules of legumes.
Large volumes of atmospheric nitrogen are necessary for making ammonia—the principal starting material used for preparation of
large quantities of other nitrogen-containing compounds. Most other uses for elemental nitrogen depend on its inactivity. It is
helpful when a chemical process requires an inert atmosphere. Canned foods and luncheon meats cannot oxidize in a pure nitrogen
atmosphere, so they retain a better flavor and color, and spoil less rapidly, when sealed in nitrogen instead of air. This technology
allows fresh produce to be available year-round, regardless of growing season.
There are compounds with nitrogen in all of its oxidation states from 3− to 5+. Much of the chemistry of nitrogen involves
oxidation-reduction reactions. Some active metals (such as alkali metals and alkaline earth metals) can reduce nitrogen to form
metal nitrides. In the remainder of this section, we will examine nitrogen-oxygen chemistry.
There are well-characterized nitrogen oxides in which nitrogen exhibits each of its positive oxidation numbers from 1+ to 5+.
When ammonium nitrate is carefully heated, nitrous oxide (dinitrogen oxide) and water vapor form. Stronger heating generates
nitrogen gas, oxygen gas, and water vapor. No one should ever attempt this reaction—it can be very explosive. In 1947, there was a
major ammonium nitrate explosion in Texas City, Texas, and, in 2013, there was another major explosion in West, Texas. In the last
100 years, there were nearly 30 similar disasters worldwide, resulting in the loss of numerous lives. In this oxidation-reduction
reaction, the nitrogen in the nitrate ion oxidizes the nitrogen in the ammonium ion. Nitrous oxide, shown in Figure 18.7.2, is a

Access for free at OpenStax 18.7.2 https://chem.libretexts.org/@go/page/38318


colorless gas possessing a mild, pleasing odor and a sweet taste. It finds application as an anesthetic for minor operations,
especially in dentistry, under the name “laughing gas.”

Figure 18.7.2 : Nitrous oxide, N2O, is an anesthetic that has these molecular (left) and resonance (right) structures.
Low yields of nitric oxide, NO, form when heating nitrogen and oxygen together. NO also forms when lightning passes through air
during thunderstorms. Burning ammonia is the commercial method of preparing nitric oxide. In the laboratory, the reduction of
nitric acid is the best method for preparing nitric oxide. When copper reacts with dilute nitric acid, nitric oxide is the principal
reduction product:

3 Cu(s) + 8 HNO (aq) ⟶ 2 NO(g) + 3 Cu(NO ) (aq) + 4 H O(l)


3 3 2 2

Gaseous nitric oxide is the most thermally stable of the nitrogen oxides and is the simplest known thermally stable molecule with
an unpaired electron. It is one of the air pollutants generated by internal combustion engines, resulting from the reaction of
atmospheric nitrogen and oxygen during the combustion process.
At room temperature, nitric oxide is a colorless gas consisting of diatomic molecules. As is often the case with molecules that
contain an unpaired electron, two molecules combine to form a dimer by pairing their unpaired electrons to form a bond. Liquid
and solid NO both contain N2O2 dimers, like that shown in Figure 18.7.3. Most substances with unpaired electrons exhibit color by
absorbing visible light; however, NO is colorless because the absorption of light is not in the visible region of the spectrum.

Figure 18.7.3 : This shows the equilibrium between NO and N2O2. The molecule, N2O2, absorbs light.
Cooling a mixture of equal parts nitric oxide and nitrogen dioxide to −21 °C produces dinitrogen trioxide, a blue liquid consisting
of N2O3 molecules (Figure 18.7.4). Dinitrogen trioxide exists only in the liquid and solid states. When heated, it reverts to a
mixture of NO and NO2.

Figure 18.7.4 : Dinitrogen trioxide, N2O3, only exists in liquid or solid states and has these molecular (left) and resonance (right)
structures.
It is possible to prepare nitrogen dioxide in the laboratory by heating the nitrate of a heavy metal, or by the reduction of
concentrated nitric acid with copper metal, as shown in Figure 18.7.5. Commercially, it is possible to prepare nitrogen dioxide by
oxidizing nitric oxide with air.

Figure 18.7.5 : The reaction of copper metal with concentrated HNO3 produces a solution of Cu(NO3)2 and brown fumes of NO2.
(credit: modification of work by Mark Ott)
The nitrogen dioxide molecule (Figure 18.7.6) contains an unpaired electron, which is responsible for its color and paramagnetism.
It is also responsible for the dimerization of NO2. At low pressures or at high temperatures, nitrogen dioxide has a deep brown

Access for free at OpenStax 18.7.3 https://chem.libretexts.org/@go/page/38318


color that is due to the presence of the NO2 molecule. At low temperatures, the color almost entirely disappears as dinitrogen
tetraoxide, N2O4, forms. At room temperature, an equilibrium exists:

2 NO (g) ⇌ N O (g) KP = 6.86


2 2 4

Figure 18.7.6 : The molecular and resonance structures for nitrogen dioxide (NO2, left) and dinitrogen tetraoxide (N2O4, right) are
shown.
Dinitrogen pentaoxide, N2O5 (Figure 18.7.7 ), is a white solid that is formed by the dehydration of nitric acid by phosphorus(V)
oxide (tetraphosphorus decoxide):

P O (s) + 4 HNO (l) ⟶ 4 HPO (s) + 2 N O (s)


4 10 3 3 2 5

It is unstable above room temperature, decomposing to N2O4 and O2.

Figure 18.7.7 : This image shows the molecular structure and one resonance structure of a molecule of dinitrogen pentaoxide, N2O5.
The oxides of nitrogen(III), nitrogen(IV), and nitrogen(V) react with water and form nitrogen-containing oxyacids. Nitrogen(III)
oxide, N2O3, is the anhydride of nitrous acid; HNO2 forms when N2O3 reacts with water. There are no stable oxyacids containing
nitrogen with an oxidation state of 4+; therefore, nitrogen(IV) oxide, NO2, disproportionates in one of two ways when it reacts with
water. In cold water, a mixture of HNO2 and HNO3 forms. At higher temperatures, HNO3 and NO will form. Nitrogen(V) oxide,
N2O5, is the anhydride of nitric acid; HNO3 is produced when N2O5 reacts with water:

N O (s) + H O(l) ⟶ 2 HNO (aq)


2 5 2 3

The nitrogen oxides exhibit extensive oxidation-reduction behavior. Nitrous oxide resembles oxygen in its behavior when heated
with combustible substances. N2O is a strong oxidizing agent that decomposes when heated to form nitrogen and oxygen. Because
one-third of the gas liberated is oxygen, nitrous oxide supports combustion better than air (one-fifth oxygen). A glowing splinter
bursts into flame when thrust into a bottle of this gas. Nitric oxide acts both as an oxidizing agent and as a reducing agent. For
example:

oxidizing agent: P (s) + 6 NO(g) ⟶ P O (s) + 3 N (g)


4 4 6 2

reducing agent: Cl (g) + 2 NO(g) ⟶ 2 ClNO(g)


2

Nitrogen dioxide (or dinitrogen tetraoxide) is a good oxidizing agent. For example:

NO (g) + CO(g) ⟶ NO(g) + CO (g)


2 2

NO (g) + 2 HCl(aq) ⟶ NO(g) + Cl (g) + H O(l)


2 2 2

Summary
Nitrogen exhibits oxidation states ranging from 3− to 5+. Because of the stability of the N≡N triple bond, it requires a great deal of
energy to make compounds from molecular nitrogen. Active metals such as the alkali metals and alkaline earth metals can reduce
nitrogen to form metal nitrides. Nitrogen oxides and nitrogen hydrides are also important substances.

Access for free at OpenStax 18.7.4 https://chem.libretexts.org/@go/page/38318


Glossary
nitrogen fixation
formation of nitrogen compounds from molecular nitrogen

This page titled 18.7: Occurrence, Preparation, and Properties of Nitrogen is shared under a CC BY 4.0 license and was authored, remixed, and/or
curated by OpenStax via source content that was edited to the style and standards of the LibreTexts platform; a detailed edit history is available
upon request.

Access for free at OpenStax 18.7.5 https://chem.libretexts.org/@go/page/38318


18.8: Occurrence, Preparation, and Properties of Phosphorus
 Learning Objectives
Describe the properties, preparation, and uses of phosphorus

The industrial preparation of phosphorus is by heating calcium phosphate, obtained from phosphate rock, with sand and coke:
Δ

2 Ca (PO ) (s) + 6 SiO (s) + 10 C(s) −


→ 6 CaSiO (l) + 10 CO(g) + P (g)
3 4 2 2 3 4

The phosphorus distills out of the furnace and is condensed into a solid or burned to form P4O10. The preparation of many other
phosphorus compounds begins with P4O10. The acids and phosphates are useful as fertilizers and in the chemical industry. Other
uses are in the manufacture of special alloys such as ferrophosphorus and phosphor bronze. Phosphorus is important in making
pesticides, matches, and some plastics. Phosphorus is an active nonmetal. In compounds, phosphorus usually occurs in oxidation
states of 3−, 3+, and 5+. Phosphorus exhibits oxidation numbers that are unusual for a group 15 element in compounds that contain
phosphorus-phosphorus bonds; examples include diphosphorus tetrahydride, H2P-PH2, and tetraphosphorus trisulfide, P4S3,
illustrated in Figure 18.8.1.

Figure 18.8.1 : P4S3 is a component of the heads of strike-anywhere matches.

18.8.1: Phosphorus Oxygen Compounds


Phosphorus forms two common oxides, phosphorus(III) oxide (or tetraphosphorus hexaoxide), P4O6, and phosphorus(V) oxide (or
tetraphosphorus decaoxide), P4O10, both shown in Figure 18.8.2. Phosphorus(III) oxide is a white crystalline solid with a garlic-
like odor. Its vapor is very poisonous. It oxidizes slowly in air and inflames when heated to 70 °C, forming P4O10. Phosphorus(III)
oxide dissolves slowly in cold water to form phosphorous acid, H3PO3.

Figure 18.8.2 : This image shows the molecular structures of P4O6 (left) and P4O10 (right).
Phosphorus(V) oxide, P4O10, is a white powder that is prepared by burning phosphorus in excess oxygen. Its enthalpy of formation
is very high (−2984 kJ), and it is quite stable and a very poor oxidizing agent. Dropping P4O10 into water produces a hissing sound,
heat, and orthophosphoric acid:

P O (s) + 6 H O(l) ⟶ 4 H PO (aq)


4 10 2 3 4

Because of its great affinity for water, phosphorus(V) oxide is an excellent drying agent for gases and solvents, and for removing
water from many compounds.

Access for free at OpenStax 18.8.1 https://chem.libretexts.org/@go/page/38510


18.8.2: Phosphorus Halogen Compounds
Phosphorus will react directly with the halogens, forming trihalides, PX3, and pentahalides, PX5. The trihalides are much more
stable than the corresponding nitrogen trihalides; nitrogen pentahalides do not form because of nitrogen’s inability to form more
than four bonds.
The chlorides PCl3 and PCl5, both shown in Figure 18.8.3, are the most important halides of phosphorus. Phosphorus trichloride is
a colorless liquid that is prepared by passing chlorine over molten phosphorus. Phosphorus pentachloride is an off-white solid that
is prepared by oxidizing the trichloride with excess chlorine. The pentachloride sublimes when warmed and forms an equilibrium
with the trichloride and chlorine when heated.

Figure 18.8.3: This image shows the molecular structure of PCl3 (left) and PCl5 (right) in the gas phase.
Like most other nonmetal halides, both phosphorus chlorides react with an excess of water and yield hydrogen chloride and an
oxyacid: PCl3 yields phosphorous acid H3PO3 and PCl5 yields phosphoric acid, H3PO4.
The pentahalides of phosphorus are Lewis acids because of the empty valence d orbitals of phosphorus. These compounds readily
react with halide ions (Lewis bases) to give the anion PX . Whereas phosphorus pentafluoride is a molecular compound in all

states, X-ray studies show that solid phosphorus pentachloride is an ionic compound, [PCl ][PCl ], as are phosphorus
+

4

6

pentabromide, [PBr ][Br−], and phosphorus pentaiodide, [PI ][I−].


+

4
+

Summary
Phosphorus (group 15) commonly exhibits oxidation states of 3− with active metals and of 3+ and 5+ with more electronegative
nonmetals. The halogens and oxygen will oxidize phosphorus. The oxides are phosphorus(V) oxide, P4O10, and phosphorus(III)
oxide, P4O6. The two common methods for preparing orthophosphoric acid, H3PO4, are either the reaction of a phosphate with
sulfuric acid or the reaction of water with phosphorus(V) oxide. Orthophosphoric acid is a triprotic acid that forms three types of
salts.

This page titled 18.8: Occurrence, Preparation, and Properties of Phosphorus is shared under a CC BY 4.0 license and was authored, remixed,
and/or curated by OpenStax via source content that was edited to the style and standards of the LibreTexts platform; a detailed edit history is
available upon request.

Access for free at OpenStax 18.8.2 https://chem.libretexts.org/@go/page/38510


18.9: Occurrence, Preparation, and Compounds of Oxygen
 Learning Objectives
Describe the properties, preparation, and compounds of oxygen
Describe the preparation, properties, and uses of some representative metal oxides, peroxides, and hydroxides

Oxygen is the most abundant element on the earth’s crust. The earth’s surface is composed of the crust, atmosphere, and hydrosphere. About 50% of
the mass of the earth’s crust consists of oxygen (combined with other elements, principally silicon). Oxygen occurs as O2 molecules and, to a limited
extent, as O3 (ozone) molecules in air. It forms about 20% of the mass of the air. About 89% of water by mass consists of combined oxygen. In
combination with carbon, hydrogen, and nitrogen, oxygen is a large part of plants and animals.
Oxygen is a colorless, odorless, and tasteless gas at ordinary temperatures. It is slightly denser than air. Although it is only slightly soluble in water
(49 mL of gas dissolves in 1 L at STP), oxygen’s solubility is very important to aquatic life.
Most of the oxygen isolated commercially comes from air and the remainder from the electrolysis of water. The separation of oxygen from air begins
with cooling and compressing the air until it liquefies. As liquid air warms, oxygen with its higher boiling point (90 K) separates from nitrogen,
which has a lower boiling point (77 K). It is possible to separate the other components of air at the same time based on differences in their boiling
points.
Oxygen is essential in combustion processes such as the burning of fuels. Plants and animals use the oxygen from the air in respiration. The
administration of oxygen-enriched air is an important medical practice when a patient is receiving an inadequate supply of oxygen because of shock,
pneumonia, or some other illness.
The chemical industry employs oxygen for oxidizing many substances. A significant amount of oxygen produced commercially is important in the
removal of carbon from iron during steel production. Large quantities of pure oxygen are also necessary in metal fabrication and in the cutting and
welding of metals with oxyhydrogen and oxyacetylene torches.
Liquid oxygen is important to the space industry. It is an oxidizing agent in rocket engines. It is also the source of gaseous oxygen for life support in
space.
As we know, oxygen is very important to life. The energy required for the maintenance of normal body functions in human beings and in other
organisms comes from the slow oxidation of chemical compounds. Oxygen is the final oxidizing agent in these reactions. In humans, oxygen passes
from the lungs into the blood, where it combines with hemoglobin, producing oxyhemoglobin. In this form, blood transports the oxygen to tissues,
where it is transferred to the tissues. The ultimate products are carbon dioxide and water. The blood carries the carbon dioxide through the veins to
the lungs, where the blood releases the carbon dioxide and collects another supply of oxygen. Digestion and assimilation of food regenerate the
materials consumed by oxidation in the body; the energy liberated is the same as if the food burned outside the body.
Green plants continually replenish the oxygen in the atmosphere by a process called photosynthesis. The products of photosynthesis may vary, but,
in general, the process converts carbon dioxide and water into glucose (a sugar) and oxygen using the energy of light:
chlorophyll

6 CO (g) + 6 H O(l) −−−−−−→ C H O (aq) + 6 O (g) (18.9.1)


2 2 6 12 6 2
light

carbon water glucose oxygen (18.9.2)

dioxide

Thus, the oxygen that became carbon dioxide and water by the metabolic processes in plants and animals returns to the atmosphere by
photosynthesis.
When dry oxygen is passed between two electrically charged plates, ozone (O3, illustrated in Figure 18.9.1), an allotrope of oxygen possessing a
distinctive odor, forms. The formation of ozone from oxygen is an endothermic reaction, in which the energy comes from an electrical discharge,
heat, or ultraviolet light:
electric discharge

3 O (g) −−−−−−−−−→ 2 O (g) ΔH ° = 287 kJ


2 3

The sharp odor associated with sparking electrical equipment is due, in part, to ozone.

Figure 18.9.1 : The image shows the bent ozone (O3) molecule and the resonance structures necessary to describe its bonding.

Access for free at OpenStax 18.9.1 https://chem.libretexts.org/@go/page/38534


Ozone forms naturally in the upper atmosphere by the action of ultraviolet light from the sun on the oxygen there. Most atmospheric ozone occurs in
the stratosphere, a layer of the atmosphere extending from about 10 to 50 kilometers above the earth’s surface. This ozone acts as a barrier to
harmful ultraviolet light from the sun by absorbing it via a chemical decomposition reaction:
ultraviolet light

O (g) −−−−−−−−→ O(g) + O (g)


3 2

The reactive oxygen atoms recombine with molecular oxygen to complete the ozone cycle. The presence of stratospheric ozone decreases the
frequency of skin cancer and other damaging effects of ultraviolet radiation. It has been clearly demonstrated that chlorofluorocarbons, CFCs
(known commercially as Freons), which were present as aerosol propellants in spray cans and as refrigerants, caused depletion of ozone in the
stratosphere. This occurred because ultraviolet light also causes CFCs to decompose, producing atomic chlorine. The chlorine atoms react with
ozone molecules, resulting in a net removal of O3 molecules from stratosphere. This process is explored in detail in our coverage of chemical
kinetics. There is a worldwide effort to reduce the amount of CFCs used commercially, and the ozone hole is already beginning to decrease in size as
atmospheric concentrations of atomic chlorine decrease. While ozone in the stratosphere helps protect us, ozone in the troposphere is a problem.
This ozone is a toxic component of photochemical smog.
The uses of ozone depend on its reactivity with other substances. It can be used as a bleaching agent for oils, waxes, fabrics, and starch: It oxidizes
the colored compounds in these substances to colorless compounds. It is an alternative to chlorine as a disinfectant for water.

18.9.1: Reactions
Elemental oxygen is a strong oxidizing agent. It reacts with most other elements and many compounds.

18.9.1.1: Reaction with Elements


Oxygen reacts directly at room temperature or at elevated temperatures with all other elements except the noble gases, the halogens, and few second-
and third-row transition metals of low reactivity (those with higher reduction potentials than copper). Rust is an example of the reaction of oxygen
with iron. The more active metals form peroxides or superoxides. Less active metals and the nonmetals give oxides. Two examples of these reactions
are:

2 Mg(s) + O (g) ⟶ 2 MgO(s)


2

P (s) + 5 O (g) ⟶ P O (s)


4 2 4 10

The oxides of halogens, at least one of the noble gases, and metals with higher reduction potentials than copper do not form by the direct action of
the elements with oxygen.

18.9.2: Reaction with Compounds


Elemental oxygen also reacts with some compounds. If it is possible to oxidize any of the elements in a given compound, further oxidation by
oxygen can occur. For example, hydrogen sulfide, H2S, contains sulfur with an oxidation state of 2−. Because the sulfur does not exhibit its
maximum oxidation state, we would expect H2S to react with oxygen. It does, yielding water and sulfur dioxide. The reaction is:

2 H S(g) + 3 O (g) ⟶ 2 H O(l) + 2 SO (g)


2 2 2 2

It is also possible to oxidize oxides such as CO and P4O6 that contain an element with a lower oxidation state. The ease with which elemental
oxygen picks up electrons is mirrored by the difficulty of removing electrons from oxygen in most oxides. Of the elements, only the very reactive
fluorine can oxidize oxides to form oxygen gas.

18.9.3: Oxides, Peroxides, and Hydroxides


Compounds of the representative metals with oxygen fall into three categories: (1) oxides, containing oxide ions, O2−; (2) peroxides, containing
peroxides ions, O , with oxygen-oxygen covalent single bonds and a very limited number of superoxides, containing superoxide ions, O , with
2−
2

2

oxygen-oxygen covalent bonds that have a bond order of , In addition, there are (3) hydroxides, containing hydroxide ions, OH−. All representative
3

metals form oxides. Some of the metals of group 2 also form peroxides, MO2, and the metals of group 1 also form peroxides, M2O2, and
superoxides, MO2.

18.9.3.1: Oxides
It is possible to produce the oxides of most representative metals by heating the corresponding hydroxides (forming the oxide and gaseous water) or
carbonates (forming the oxide and gaseous CO2). Equations for example reactions are:
Δ

2 Al (OH) (s) −
→ Al O (s) + 3 H O(g)
3 2 3 2

CaCO (s) −
→ CaO(s) + CO (g)
3 2

However, alkali metal salts generally are very stable and do not decompose easily when heated. Alkali metal oxides result from the oxidation-
reduction reactions created by heating nitrates or hydroxides with the metals. Equations for sample reactions are:

Access for free at OpenStax 18.9.2 https://chem.libretexts.org/@go/page/38534


Δ

2 KNO (s) + 10 K(s) −


→ 6 K O(s) + N (g)
3 2 2

2 LiOH(s) + 2 Li(s) −
→ 2 Li O(s) + H (g)
2 2

With the exception of mercury(II) oxide, it is possible to produce the oxides of the metals of groups 2–15 by burning the corresponding metal in air.
The heaviest member of each group, the member for which the inert pair effect is most pronounced, forms an oxide in which the oxidation state of
the metal ion is two less than the group oxidation state (inert pair effect). Thus, Tl2O, PbO, and Bi2O3 form when burning thallium, lead, and
bismuth, respectively. The oxides of the lighter members of each group exhibit the group oxidation state. For example, SnO2 forms from burning tin.
Mercury(II) oxide, HgO, forms slowly when mercury is warmed below 500 °C; it decomposes at higher temperatures.
Burning the members of groups 1 and 2 in air is not a suitable way to form the oxides of these elements. These metals are reactive enough to
combine with nitrogen in the air, so they form mixtures of oxides and ionic nitrides. Several also form peroxides or superoxides when heated in air.
Ionic oxides all contain the oxide ion, a very powerful hydrogen ion acceptor. With the exception of the very insoluble aluminum oxide, Al2O3,
tin(IV), SnO2, and lead(IV), PbO2, the oxides of the representative metals react with acids to form salts. Some equations for these reactions are:

Na O + 2 HNO (aq) ⟶ 2 NaNO (aq) + H O(l)


2 3 3 2

CaO(s) + 2 HCL(aq) ⟶ CaCl (aq) + H O(l)


2 2

SnO(s) + 2 HClO (aq) ⟶ Sn(ClO ) (aq) + H O(l)


4 4 2 2

The oxides of the metals of groups 1 and 2 and of thallium(I) oxide react with water and form hydroxides. Examples of such reactions are:

Na O(s) + H O(l) ⟶ NaOH(aq)


2 2

CaO(s) + H O(l) ⟶ Ca (OH) (aq)


2 2

Tl O(s) + H O(aq) ⟶ 2 TlOH(aq)


2 2

The oxides of the alkali metals have little industrial utility, unlike magnesium oxide, calcium oxide, and aluminum oxide. Magnesium oxide is
important in making firebrick, crucibles, furnace linings, and thermal insulation—applications that require chemical and thermal stability. Calcium
oxide, sometimes called quicklime or lime in the industrial market, is very reactive, and its principal uses reflect its reactivity. Pure calcium oxide
emits an intense white light when heated to a high temperature (as illustrated in Figure 18.9.2:). Blocks of calcium oxide heated by gas flames were
the stage lights in theaters before electricity was available. This is the source of the phrase “in the limelight.”

Figure 18.9.2 : Calcium oxide has many industrial uses. When it is heated at high temperatures, it emits an intense white light.
Calcium oxide and calcium hydroxide are inexpensive bases used extensively in chemical processing, although most of the useful products prepared
from them do not contain calcium. Calcium oxide, CaO, is made by heating calcium carbonate, CaCO3, which is widely and inexpensively available
as limestone or oyster shells:

CaCO (s) ⟶ CaO(s) + CO (g)


3 2

Although this decomposition reaction is reversible, it is possible to obtain a 100% yield of CaO by allowing the CO2 to escape. It is possible to
prepare calcium hydroxide by the familiar acid-base reaction of a soluble metal oxide with water:

CaO(s) + H O(l) ⟶ Ca (OH) (s)


2 2

Both CaO and Ca(OH)2 are useful as bases; they accept protons and neutralize acids.
Alumina (Al2O3) occurs in nature as the mineral corundum, a very hard substance used as an abrasive for grinding and polishing. Corundum is
important to the jewelry trade as ruby and sapphire. The color of ruby is due to the presence of a small amount of chromium; other impurities
produce the wide variety of colors possible for sapphires. Artificial rubies and sapphires are now manufactured by melting aluminum oxide (melting
point = 2050 °C) with small amounts of oxides to produce the desired colors and cooling the melt in such a way as to produce large crystals. Ruby
lasers use synthetic ruby crystals.
Zinc oxide, ZnO, was a useful white paint pigment; however, pollutants tend to discolor the compound. The compound is also important in the
manufacture of automobile tires and other rubber goods, and in the preparation of medicinal ointments. For example, zinc-oxide-based sunscreens,
as shown in Figure 18.9.3, help prevent sunburn. The zinc oxide in these sunscreens is present in the form of very small grains known as

Access for free at OpenStax 18.9.3 https://chem.libretexts.org/@go/page/38534


nanoparticles. Lead dioxide is a constituent of charged lead storage batteries. Lead(IV) tends to revert to the more stable lead(II) ion by gaining two
electrons, so lead dioxide is a powerful oxidizing agent.

Figure 18.9.3 : Zinc oxide protects exposed skin from sunburn. (credit: modification of work by "osseous"/Flickr)

18.9.3.2: Peroxides and Superoxides


Peroxides and superoxides are strong oxidizers and are important in chemical processes. Hydrogen peroxide, H2O2, prepared from metal peroxides,
is an important bleach and disinfectant. Peroxides and superoxides form when the metal or metal oxides of groups 1 and 2 react with pure oxygen at
elevated temperatures. Sodium peroxide and the peroxides of calcium, strontium, and barium form by heating the corresponding metal or metal
oxide in pure oxygen:
Δ

2 Na(s) + O (g) −
→ Na O (s)
2 2 2

2 Na O(s) + O (g) −
→ 2 Na O (s)
2 2 2 2

2 SrO(s) + O (g) −
→ 2 SrO (s)
2 2

The peroxides of potassium, rubidium, and cesium can be prepared by heating the metal or its oxide in a carefully controlled amount of oxygen:
2 K(s) + O (g) ⟶ K O (s) (2 mol K per mol O2 )
2 2 2

With an excess of oxygen, the superoxides KO2, RbO2, and CsO2 form. For example:

K(s) + O (g) ⟶ KO (s) (1 mol K per mol O )


2 2 2

The stability of the peroxides and superoxides of the alkali metals increases as the size of the cation increases.

18.9.3.3: Hydroxides
Hydroxides are compounds that contain the OH− ion. It is possible to prepare these compounds by two general types of reactions. Soluble metal
hydroxides can be produced by the reaction of the metal or metal oxide with water. Insoluble metal hydroxides form when a solution of a soluble salt
of the metal combines with a solution containing hydroxide ions.
With the exception of beryllium and magnesium, the metals of groups 1 and 2 react with water to form hydroxides and hydrogen gas. Examples of
such reactions include:

2 Li(s) + 2 H O(l) ⟶ 2 LiOH(aq) + H (g)


2 2

Ca(s) + 2 H O(l) ⟶ Ca (OH) (aq) + H (g)


2 2 2

However, these reactions can be violent and dangerous; therefore, it is preferable to produce soluble metal hydroxides by the reaction of the
respective oxide with water:

Li O(s) + H O(l) ⟶ 2 LiOH(aq)


2 2

CaO(s) + H O(l) ⟶ Ca (OH) (aq)


2 2

Most metal oxides are base anhydrides. This is obvious for the soluble oxides because they form metal hydroxides. Most other metal oxides are
insoluble and do not form hydroxides in water; however, they are still base anhydrides because they will react with acids.
It is possible to prepare the insoluble hydroxides of beryllium, magnesium, and other representative metals by the addition of sodium hydroxide to a
solution of a salt of the respective metal. The net ionic equations for the reactions involving a magnesium salt, an aluminum salt, and a zinc salt are:
2+ −
Mg (aq) + 2 OH (aq) ⟶ Mg (OH) (s)
2

3+ −
Al (aq) + 3 OH (aq) ⟶ Al (OH) (s)
3

Access for free at OpenStax 18.9.4 https://chem.libretexts.org/@go/page/38534


2+ −
Zn (aq) + 2 OH (aq) ⟶ Zn(OH) (s)
2

An excess of hydroxide must be avoided when preparing aluminum, gallium, zinc, and tin(II) hydroxides, or the hydroxides will dissolve with the
− − 2− −
formation of the corresponding complex ions: Al(OH) , Ga(OH) , Zn(OH) , and Sn(OH) (Figure 18.9.4). The important aspect of complex
4 4 4 3

ions for this chapter is that they form by a Lewis acid-base reaction with the metal being the Lewis acid.

Figure 18.9.4 : (a) Mixing solutions of NaOH and Zn(NO3)2 produces a white precipitate of Zn(OH)2. (b) Addition of an excess of NaOH results in
dissolution of the precipitate. (credit: modification of work by Mark Ott)
Industry uses large quantities of sodium hydroxide as a cheap, strong base. Sodium chloride is the starting material for the production of NaOH
because NaCl is a less expensive starting material than the oxide. Sodium hydroxide is among the top 10 chemicals in production in the United
States, and this production was almost entirely by electrolysis of solutions of sodium chloride. This process is the chlor-alkali process, and it is the
primary method for producing chlorine.
Sodium hydroxide is an ionic compound and melts without decomposition. It is very soluble in water, giving off a great deal of heat and forming
very basic solutions: 40 grams of sodium hydroxide dissolves in only 60 grams of water at 25 °C. Sodium hydroxide is employed in the production
of other sodium compounds and is used to neutralize acidic solutions during the production of other chemicals such as petrochemicals and polymers.
Many of the applications of hydroxides are for the neutralization of acids (such as the antacid shown in Figure 18.9.5) and for the preparation of
oxides by thermal decomposition. An aqueous suspension of magnesium hydroxide constitutes the antacid milk of magnesia. Because of its ready
availability (from the reaction of water with calcium oxide prepared by the decomposition of limestone, CaCO3), low cost, and activity, calcium
hydroxide is used extensively in commercial applications needing a cheap, strong base. The reaction of hydroxides with appropriate acids is also
used to prepare salts.

Figure 18.9.5 : Calcium carbonate, CaCO3, can be consumed in the form of an antacid to neutralize the effects of acid in your stomach. (credit:
“Midnightcomm”/Wikimedia Commons)

18.9.4: The Chlor-Alkali Process


Although they are very different chemically, there is a link between chlorine and sodium hydroxide because there is an important electrochemical
process that produces the two chemicals simultaneously. The process known as the chlor-alkali process, utilizes sodium chloride, which occurs in
large deposits in many parts of the world. This is an electrochemical process to oxidize chloride ion to chlorine and generate sodium hydroxide.
Passing a direct current of electricity through a solution of NaCl causes the chloride ions to migrate to the positive electrode where oxidation to
gaseous chlorine occurs when the ion gives up an electron to the electrode:
− −
2 Cl (aq) ⟶ Cl (g) + 2 e (at the positive electrode)
2

The electrons produced travel through the outside electrical circuit to the negative electrode. Although the positive sodium ions migrate toward this
negative electrode, metallic sodium does not form because sodium ions are too difficult to reduce under the conditions used. (Recall that metallic
sodium is active enough to react with water and hence, even if produced, would immediately react with water to produce sodium ions again.)
Instead, water molecules pick up electrons from the electrode and undergo reduction to form hydrogen gas and hydroxide ions:
− −
2 H O(l) + 2 e  (from the negative electrode) ⟶ H (g) + 2 OH (aq)
2 2

The overall result is the conversion of the aqueous solution of NaCl to an aqueous solution of NaOH, gaseous Cl2, and gaseous H2:

Access for free at OpenStax 18.9.5 https://chem.libretexts.org/@go/page/38534


electrolysis
+ − + −
2 Na (aq) + 2 Cl (aq) + 2 H O(l) −−−−−−→ 2 Na (aq) + 2 OH (aq) + Cl (g) + H (g)
2 2 2

18.9.5: Nonmetal Oxygen Compounds


Most nonmetals react with oxygen to form nonmetal oxides. Depending on the available oxidation states for the element, a variety of oxides might
form. Fluorine will combine with oxygen to form fluorides such as OF2, where the oxygen has a 2+-oxidation state.

18.9.5.1: Sulfur Oxygen Compounds


The two common oxides of sulfur are sulfur dioxide, SO2, and sulfur trioxide, SO3. The odor of burning sulfur comes from sulfur dioxide. Sulfur
dioxide, shown in Figure 18.9.6, occurs in volcanic gases and in the atmosphere near industrial plants that burn fuel containing sulfur compounds.

Figure 18.9.6 : This image shows the molecular structure (left) and resonance forms (right) of sulfur dioxide.
Commercial production of sulfur dioxide is from either burning sulfur or roasting sulfide ores such as ZnS, FeS2, and Cu2S in air. (Roasting, which
forms the metal oxide, is the first step in the separation of many metals from their ores.) A convenient method for preparing sulfur dioxide in the
laboratory is by the action of a strong acid on either sulfite salts containing the SO ion or hydrogen sulfite salts containing HSO . Sulfurous acid,
2−
3

3

H2SO3, forms first, but quickly decomposes into sulfur dioxide and water. Sulfur dioxide also forms when many reducing agents react with hot,
concentrated sulfuric acid. Sulfur trioxide forms slowly when heating sulfur dioxide and oxygen together, and the reaction is exothermic:

2 SO (g) + O (g) ⟶ 2 SO (g) ΔH ° = −197.8 kJ


2 2 3

Sulfur dioxide is a gas at room temperature, and the SO2 molecule is bent. Sulfur trioxide melts at 17 °C and boils at 43 °C. In the vapor state, its
molecules are single SO3 units (Figure 18.9.7), but in the solid state, SO3 exists in several polymeric forms.

Figure 18.9.7 : This image shows the structure (top) of sulfur trioxide in the gas phase and its resonance forms (bottom).
The sulfur oxides react as Lewis acids with many oxides and hydroxides in Lewis acid-base reactions, with the formation of sulfites or hydrogen
sulfites, and sulfates or hydrogen sulfates, respectively.

18.9.5.2: Halogen Oxygen Compounds


The halogens do not react directly with oxygen, but it is possible to prepare binary oxygen-halogen compounds by the reactions of the halogens with
oxygen-containing compounds. Oxygen compounds with chlorine, bromine, and iodine are oxides because oxygen is the more electronegative
element in these compounds. On the other hand, fluorine compounds with oxygen are fluorides because fluorine is the more electronegative element.

Figure 18.9.8 : This image shows the structures of the (a) Cl2O and (b) ClO2 molecules.
As a class, the oxides are extremely reactive and unstable, and their chemistry has little practical importance. Dichlorine oxide, formally called
dichlorine monoxide, and chlorine dioxide, both shown in Figure 18.9.8, are the only commercially important compounds. They are important as
bleaching agents (for use with pulp and flour) and for water treatment.

18.9.6: Nonmetal Oxyacids and Their Salts


Nonmetal oxides form acids when allowed to react with water; these are acid anhydrides. The resulting oxyanions can form salts with various metal
ions.

Access for free at OpenStax 18.9.6 https://chem.libretexts.org/@go/page/38534


18.9.6.1: Nitrogen Oxyacids and Salts
Nitrogen pentaoxide, N2O5, and NO2 react with water to form nitric acid, HNO3. Alchemists, as early as the eighth century, knew nitric acid (shown
in Figure 18.9.9) as aqua fortis (meaning "strong water"). The acid was useful in the separation of gold from silver because it dissolves silver but not
gold. Traces of nitric acid occur in the atmosphere after thunderstorms, and its salts are widely distributed in nature. There are tremendous deposits
of Chile saltpeter, NaNO3, in the desert region near the boundary of Chile and Peru. Bengal saltpeter, KNO3, occurs in India and in other countries of
the Far East.

Figure 18.9.9 : This image shows the molecular structure (left) of nitric acid, HNO3 and its resonance forms (right).
In the laboratory, it is possible to produce nitric acid by heating a nitrate salt (such as sodium or potassium nitrate) with concentrated sulfuric acid:
Δ

NaNO (s) + H SO (l) −


→ NaHSO (s) + HNO (g)
3 2 4 4 3

The Ostwald process is the commercial method for producing nitric acid. This process involves the oxidation of ammonia to nitric oxide, NO;
oxidation of nitric oxide to nitrogen dioxide, NO2; and further oxidation and hydration of nitrogen dioxide to form nitric acid:

4 NH (g) + 5 O (g) ⟶ 4 NO(g) + 6 H O(g)


3 2 2

2 NO(g) + O (g) ⟶ 2 NO (g)


2 2

3 NO (g) + H O(l) ⟶ 2 HNO (aq) + NO(g)


2 2 3

or

4 NO (g) + O (g) + 2 H O(g) ⟶ 4 HNO (l)


2 2 2 3

Pure nitric acid is a colorless liquid. However, it is often yellow or brown in color because NO2 forms as the acid decomposes. Nitric acid is stable in
aqueous solution; solutions containing 68% of the acid are commercially available concentrated nitric acid. It is both a strong oxidizing agent and a
strong acid.
The action of nitric acid on a metal rarely produces H2 (by reduction of H+) in more than small amounts. Instead, the reduction of nitrogen occurs.
The products formed depend on the concentration of the acid, the activity of the metal, and the temperature. Normally, a mixture of nitrates, nitrogen
oxides, and various reduction products form. Less active metals such as copper, silver, and lead reduce concentrated nitric acid primarily to nitrogen
dioxide. The reaction of dilute nitric acid with copper produces NO. In each case, the nitrate salts of the metals crystallize upon evaporation of the
resultant solutions.
Nonmetallic elements, such as sulfur, carbon, iodine, and phosphorus, undergo oxidation by concentrated nitric acid to their oxides or oxyacids, with
the formation of NO2:

S(s) + 6 HNO (aq) ⟶ H SO (aq) + 6 NO (g) + 2 H O(l)


3 2 4 2 2

C(s) + 4 HNO (aq) ⟶ CO (g) + 4 NO (g) + 2 H O(l)


3 2 2 2

Nitric acid oxidizes many compounds; for example, concentrated nitric acid readily oxidizes hydrochloric acid to chlorine and chlorine dioxide. A
mixture of one part concentrated nitric acid and three parts concentrated hydrochloric acid (called aqua regia, which means royal water) reacts
vigorously with metals. This mixture is particularly useful in dissolving gold, platinum, and other metals that are more difficult to oxidize than
hydrogen. A simplified equation to represent the action of aqua regia on gold is:

Au(s) + 4 HCl(aq) + 3 HNO (aq) ⟶ HAuCl (aq) + 3 NO (g) + 3 H O(l)


3 4 2 2

Access for free at OpenStax 18.9.7 https://chem.libretexts.org/@go/page/38534


Gold Coin Dissolving in Acid (Aqua Regia)

Video 18.9.1 : Although gold is generally unreactive, you can watch a video of the complex mixture of compounds present in aqua regia dissolving
it into solution.
Nitrates, salts of nitric acid, form when metals, oxides, hydroxides, or carbonates react with nitric acid. Most nitrates are soluble in water; indeed,
one of the significant uses of nitric acid is to prepare soluble metal nitrates.
Nitric acid finds extensive use in the laboratory and in chemical industries as a strong acid and strong oxidizing agent. It is important in the
manufacture of explosives, dyes, plastics, and drugs. Salts of nitric acid (nitrates) are valuable as fertilizers. Gunpowder is a mixture of potassium
nitrate, sulfur, and charcoal.
The reaction of N2O3 with water gives a pale blue solution of nitrous acid, HNO2. However, HNO2 (shown in Figure 18.9.10) is easier to prepare by
the addition of an acid to a solution of nitrite; nitrous acid is a weak acid, so the nitrite ion is basic in aqueous solution:
− +
NO2 (aq) + H O (aq) ⟶ HNO (aq) + H O(l)
3 2 2

Nitrous acid is very unstable and exists only in solution. It disproportionates slowly at room temperature (rapidly when heated) into nitric acid and
nitric oxide. Nitrous acid is an active oxidizing agent with strong reducing agents, and strong oxidizing agents oxidize it to nitric acid.

Figure 18.9.10: This image shows the molecular structure of a molecule of nitrous acid, HNO2.
Sodium nitrite, NaNO2, is an additive to meats such as hot dogs and cold cuts. The nitrite ion has two functions. It limits the growth of bacteria that
can cause food poisoning, and it prolongs the meat’s retention of its red color. The addition of sodium nitrite to meat products is controversial
because nitrous acid reacts with certain organic compounds to form a class of compounds known as nitrosamines. Nitrosamines produce cancer in
laboratory animals. This has prompted the FDA to limit the amount of NaNO2 in foods.
The nitrites are much more stable than the acid, but nitrites, like nitrates, can explode. Nitrites, like nitrates, are also soluble in water (AgNO2 is only
slightly soluble).

18.9.6.2: Phosphorus Oxyacids and Salts


Pure orthophosphoric acid, H3PO4 (Figure 18.9.11), forms colorless, deliquescent crystals that melt at 42 °C. The common name of this compound
is phosphoric acid, and is commercially available as a viscous 82% solution known as syrupy phosphoric acid. One use of phosphoric acid is as an
additive to many soft drinks.
One commercial method of preparing orthophosphoric acid is to treat calcium phosphate rock with concentrated sulfuric acid:

Ca (PO ) (s) + 3 H SO (aq) ⟶ 2 H PO (aq) + 3 CaSO (s)


3 4 2 2 4 3 4 4

Access for free at OpenStax 18.9.8 https://chem.libretexts.org/@go/page/38534


Figure 18.9.11: Orthophosphoric acid, H3PO4, is colorless when pure and has this molecular (left) and Lewis structure (right).
Dilution of the products with water, followed by filtration to remove calcium sulfate, gives a dilute acid solution contaminated with calcium
dihydrogen phosphate, Ca(H2PO4)2, and other compounds associated with calcium phosphate rock. It is possible to prepare pure orthophosphoric
acid by dissolving P4O10 in water.
The action of water on P4O6, PCl3, PBr3, or PI3 forms phosphorous acid, H3PO3 (shown in Figure 18.9.12 ). The best method for preparing pure
phosphorous acid is by hydrolyzing phosphorus trichloride:

PCl (l) + 3 H O(l) ⟶ H PO (aq) + 3 HCl(g)


3 2 3 3

Heating the resulting solution expels the hydrogen chloride and leads to the evaporation of water. When sufficient water evaporates, white crystals of
phosphorous acid will appear upon cooling. The crystals are deliquescent, very soluble in water, and have an odor like that of garlic. The solid melts
at 70.1 °C and decomposes at about 200 °C by disproportionation into phosphine and orthophosphoric acid:

4 H PO (l) ⟶ PH (g) + 3 H PO (l)


3 3 3 3 4

Figure 18.9.12: In a molecule of phosphorous acid, H3PO3, only the two hydrogen atoms bonded to an oxygen atom are acidic.
A space filling model shows an orange atom labeled, “P,” bonded on three sides to red atoms labeled, “O,” and on the other side to a white atom
labeled, “H.” Two of the red atoms are bonded to white atoms labeled, “H.” A Lewis structure is also shown in which a phosphorus atom is single
bonded to a hydrogen atom and three oxygen atoms, two of which have two lone pairs of electrons and single bonds to hydrogen atoms, and one of
which has three lone pairs of electrons.
Phosphorous acid forms only two series of salts, which contain the dihydrogen phosphite ion, H PO , or the hydrogen phosphate ion,
2

3
HPO
2−

3
,
respectively. It is not possible to replace the third atom of hydrogen because it is not very acidic, as it is not easy to ionize the P-H bond.

18.9.6.3: Sulfur Oxyacids and Salts


The preparation of sulfuric acid, H2SO4 (Figure 18.9.13), begins with the oxidation of sulfur to sulfur trioxide and then converting the trioxide to
sulfuric acid. Pure sulfuric acid is a colorless, oily liquid that freezes at 10.5 °C. It fumes when heated because the acid decomposes to water and
sulfur trioxide. The heating process causes the loss of more sulfur trioxide than water, until reaching a concentration of 98.33% acid. Acid of this
concentration boils at 338 °C without further change in concentration (a constant boiling solution) and is commercially concentrated H2SO4. The
amount of sulfuric acid used in industry exceeds that of any other manufactured compound.

Figure 18.9.13: Sulfuric acid has a tetrahedral molecular structure.


The strong affinity of concentrated sulfuric acid for water makes it a good dehydrating agent. It is possible to dry gases and immiscible liquids that
do not react with the acid by passing them through the acid.
Sulfuric acid is a strong diprotic acid that ionizes in two stages. In aqueous solution, the first stage is essentially complete. The secondary ionization
is not nearly so complete, and HSO is a moderately strong acid (about 25% ionized in solution of a HSO salt: Ka = 1.2 × 10−2).

4

Being a diprotic acid, sulfuric acid forms both sulfates, such as Na2SO4, and hydrogen sulfates, such as NaHSO4. Most sulfates are soluble in water;
however, the sulfates of barium, strontium, calcium, and lead are only slightly soluble in water.

Access for free at OpenStax 18.9.9 https://chem.libretexts.org/@go/page/38534


Among the important sulfates are Na2SO4⋅10H2O and Epsom salts, MgSO4⋅7H2O. Because the HSO ion is an acid, hydrogen sulfates, such as −

NaHSO4, exhibit acidic behavior, and this compound is the primary ingredient in some household cleansers.
Hot, concentrated sulfuric acid is an oxidizing agent. Depending on its concentration, the temperature, and the strength of the reducing agent,
sulfuric acid oxidizes many compounds and, in the process, undergoes reduction to SO2, HSO , SO , S, H2S, or S2−. −

3
2−

Sulfur dioxide dissolves in water to form a solution of sulfurous acid, as expected for the oxide of a nonmetal. Sulfurous acid is unstable, and it is
not possible to isolate anhydrous H2SO3. Heating a solution of sulfurous acid expels the sulfur dioxide. Like other diprotic acids, sulfurous acid
ionizes in two steps: The hydrogen sulfite ion, HSO , and the sulfite ion, SO , form. Sulfurous acid is a moderately strong acid. Ionization is

3
2−
3

about 25% in the first stage, but it is much less in the second (Ka1 = 1.2 × 10−2 and Ka2 = 6.2 × 10−8).
In order to prepare solid sulfite and hydrogen sulfite salts, it is necessary to add a stoichiometric amount of a base to a sulfurous acid solution and
then evaporate the water. These salts also form from the reaction of SO2 with oxides and hydroxides. Heating solid sodium hydrogen sulfite forms
sodium sulfite, sulfur dioxide, and water:
Δ

2 NaHSO (s) −
→ Na SO (s) + SO (g) + H O(l)
3 2 3 2 2

Strong oxidizing agents can oxidize sulfurous acid. Oxygen in the air oxidizes it slowly to the more stable sulfuric acid:
Δ
+ −
2 H SO (aq) + O (g) + 2 H O(l) −
→2H O (aq) + 2 HSO (aq)
2 3 2 2 3 4

Solutions of sulfites are also very susceptible to air oxidation to produce sulfates. Thus, solutions of sulfites always contain sulfates after exposure to
air.

18.9.6.4: Halogen Oxyacids and Their Salts


The compounds HXO, HXO2, HXO3, and HXO4, where X represents Cl, Br, or I, are the hypohalous, halous, halic, and perhalic acids, respectively.
The strengths of these acids increase from the hypohalous acids, which are very weak acids, to the perhalic acids, which are very strong. Table
18.9.1 lists the known acids, and, where known, their pKa values are given in parentheses.

Table 18.9.1 : Oxyacids of the Halogens


Name Fluorine Chlorine Bromine Iodine

hypohalous HOF HOCl (7.5) HOBr (8.7) HOI (11)

halous HClO2 (2.0)

halic HClO3 HBrO3 HIO3 (0.8)

perhalic HClO4 HBrO4 HIO4 (1.6)

paraperhalic H5IO6 (1.6)

The only known oxyacid of fluorine is the very unstable hypofluorous acid, HOF, which is prepared by the reaction of gaseous fluorine with ice:

F (g) + H O(s) ⟶ HOF(g) + HF(g)


2 2

The compound is very unstable and decomposes above −40 °C. This compound does not ionize in water, and there are no known salts. It is uncertain
whether the name hypofluorous acid is even appropriate for HOF; a more appropriate name might be hydrogen hypofluorite.
The reactions of chlorine and bromine with water are analogous to that of fluorine with ice, but these reactions do not go to completion, and
mixtures of the halogen and the respective hypohalous and hydrohalic acids result. Other than HOF, the hypohalous acids only exist in solution. The
hypohalous acids are all very weak acids; however, HOCl is a stronger acid than HOBr, which, in turn, is stronger than HOI.
The addition of base to solutions of the hypohalous acids produces solutions of salts containing the basic hypohalite ions, OX−. It is possible to
isolate these salts as solids. All of the hypohalites are unstable with respect to disproportionation in solution, but the reaction is slow for
hypochlorite. Hypobromite and hypoiodite disproportionate rapidly, even in the cold:
− − −
3 XO (aq) ⟶ 2 X (aq) + XO3 (aq)

Sodium hypochlorite is an inexpensive bleach (Clorox) and germicide. The commercial preparation involves the electrolysis of cold, dilute, aqueous
sodium chloride solutions under conditions where the resulting chlorine and hydroxide ion can react. The net reaction is:
electrical energy
− −
Cl (aq) + H O(l) −−−−−−−−−→ ClO (aq) + H (g)
2 2

The only definitely known halous acid is chlorous acid, HClO2, obtained by the reaction of barium chlorite with dilute sulfuric acid:

Ba (ClO ) (aq) + H SO (aq) ⟶ BaSO (s) + 2 HClO (aq)


2 2 2 4 4 2

Filtering the insoluble barium sulfate leaves a solution of HClO2. Chlorous acid is not stable; it slowly decomposes in solution to yield chlorine
dioxide, hydrochloric acid, and water. Chlorous acid reacts with bases to give salts containing the chlorite ion (shown in Figure 18.9.14). Sodium
chlorite finds an extensive application in the bleaching of paper because it is a strong oxidizing agent and does not damage the paper.

Access for free at OpenStax 18.9.10 https://chem.libretexts.org/@go/page/38534


Figure 18.9.14: Chlorite ions, ClO , are produced when chlorous acid reacts with bases.

Chloric acid, HClO3, and bromic acid, HBrO3, are stable only in solution. The reaction of iodine with concentrated nitric acid produces stable white
iodic acid, HIO3:

I (s) + 10 HNO (aq) ⟶ 2 HIO (s) + 10 NO (g) + 4 H O(l)


2 3 3 2 2

It is possible to obtain the lighter halic acids from their barium salts by reaction with dilute sulfuric acid. The reaction is analogous to that used to
prepare chlorous acid. All of the halic acids are strong acids and very active oxidizing agents. The acids react with bases to form salts containing
chlorate ions (shown in Figure 18.9.15). Another preparative method is the electrochemical oxidation of a hot solution of a metal halide to form the
appropriate metal chlorates. Sodium chlorate is a weed killer; potassium chlorate is used as an oxidizing agent.

Figure 18.9.15: Chlorate ions, ClO , are produced when halic acids react with bases.

Perchloric acid, HClO4, forms when treating a perchlorate, such as potassium perchlorate, with sulfuric acid under reduced pressure. The HClO4 can
be distilled from the mixture:

KClO (s) + H SO (aq) ⟶ HClO (g) + KHSO (s)


4 2 4 4 4

Dilute aqueous solutions of perchloric acid are quite stable thermally, but concentrations above 60% are unstable and dangerous. Perchloric acid and
its salts are powerful oxidizing agents, as the very electronegative chlorine is more stable in a lower oxidation state than 7+. Serious explosions have
occurred when heating concentrated solutions with easily oxidized substances. However, its reactions as an oxidizing agent are slow when perchloric
acid is cold and dilute. The acid is among the strongest of all acids. Most salts containing the perchlorate ion (Figure 18.9.16) are soluble. It is
possible to prepare them from reactions of bases with perchloric acid and, commercially, by the electrolysis of hot solutions of their chlorides.

Figure 18.9.16: Perchlorate ions, ClO



4
, can be produced when perchloric acid reacts with a base or by electrolysis of hot solutions of their
chlorides.
Perbromate salts are difficult to prepare, and the best syntheses currently involve the oxidation of bromates in basic solution with fluorine gas
followed by acidification. There are few, if any, commercial uses of this acid or its salts.
There are several different acids containing iodine in the 7+-oxidation state; they include metaperiodic acid, HIO4, and paraperiodic acid, H5IO6.
These acids are strong oxidizing agents and react with bases to form the appropriate salts.

Summary
Oxygen is one of the most reactive elements. This reactivity, coupled with its abundance, makes the chemistry of oxygen very rich and well
understood. Compounds of the representative metals with oxygen exist in three categories (1) oxides, (2) peroxides and superoxides, and (3)
hydroxides. Heating the corresponding hydroxides, nitrates, or carbonates is the most common method for producing oxides. Heating the metal or
metal oxide in oxygen may lead to the formation of peroxides and superoxides. The soluble oxides dissolve in water to form solutions of hydroxides.
Most metals oxides are base anhydrides and react with acids. The hydroxides of the representative metals react with acids in acid-base reactions to
form salts and water. The hydroxides have many commercial uses.
All nonmetals except fluorine form multiple oxides. Nearly all of the nonmetal oxides are acid anhydrides. The acidity of oxyacids requires that the
hydrogen atoms bond to the oxygen atoms in the molecule rather than to the other nonmetal atom. Generally, the strength of the oxyacid increases
with the number of oxygen atoms bonded to the nonmetal atom and not to a hydrogen.

Access for free at OpenStax 18.9.11 https://chem.libretexts.org/@go/page/38534


Glossary
base anhydride
metal oxide that behaves as a base towards acids

chlor-alkali process
electrolysis process for the synthesis of chlorine and sodium hydroxide

hydrogen sulfate
HSO ion −
4

hydrogen sulfite
HSO ion −
3

hydroxide
compound of a metal with the hydroxide ion OH− or the group −OH

nitrate
NO3

ion; salt of nitric acid

Ostwald process
industrial process used to convert ammonia into nitric acid

oxide
binary compound of oxygen with another element or group, typically containing O2− ions or the group –O– or =O

ozone
allotrope of oxygen; O3

peroxide
molecule containing two oxygen atoms bonded together or as the anion, O 2−
2

photosynthesis
process whereby light energy promotes the reaction of water and carbon dioxide to form carbohydrates and oxygen; this allows photosynthetic
organisms to store energy

sulfate
SO
2−
4
ion

sulfite
SO
2−

3
ion

superoxide
oxide containing the anion O −
2

This page titled 18.9: Occurrence, Preparation, and Compounds of Oxygen is shared under a CC BY 4.0 license and was authored, remixed, and/or curated by
OpenStax via source content that was edited to the style and standards of the LibreTexts platform; a detailed edit history is available upon request.

Access for free at OpenStax 18.9.12 https://chem.libretexts.org/@go/page/38534


18.10: Occurrence, Preparation, and Properties of Sulfur
 Learning Objectives
Describe the properties, preparation, and uses of sulfur

Sulfur exists in nature as elemental deposits as well as sulfides of iron, zinc, lead, and copper, and sulfates of sodium, calcium,
barium, and magnesium. Hydrogen sulfide is often a component of natural gas and occurs in many volcanic gases, like those shown
in Figure 18.10.1. Sulfur is a constituent of many proteins and is essential for life.

Figure 18.10.1: Volcanic gases contain hydrogen sulfide. (credit: Daniel Julie/Wikimedia Commons)
The Frasch process, illustrated in Figure 18.10.2, is important in the mining of free sulfur from enormous underground deposits in
Texas and Louisiana. Superheated water (170 °C and 10 atm pressure) is forced down the outermost of three concentric pipes to the
underground deposit. The hot water melts the sulfur. The innermost pipe conducts compressed air into the liquid sulfur. The air
forces the liquid sulfur, mixed with air, to flow up through the outlet pipe. Transferring the mixture to large settling vats allows the
solid sulfur to separate upon cooling. This sulfur is 99.5% to 99.9% pure and requires no purification for most uses.

Figure 18.10.2: The Frasch process is used to mine sulfur from underground deposits.
Larger amounts of sulfur also come from hydrogen sulfide recovered during the purification of natural gas.
Sulfur exists in several allotropic forms. The stable form at room temperature contains eight-membered rings, and so the true
formula is S8. However, chemists commonly use S to simplify the coefficients in chemical equations; we will follow this practice in
this book.

Access for free at OpenStax 18.10.1 https://chem.libretexts.org/@go/page/38557


Like oxygen, which is also a member of group 16, sulfur exhibits a distinctly nonmetallic behavior. It oxidizes metals, giving a
variety of binary sulfides in which sulfur exhibits a negative oxidation state (2−). Elemental sulfur oxidizes less electronegative
nonmetals, and more electronegative nonmetals, such as oxygen and the halogens, will oxidize it. Other strong oxidizing agents
also oxidize sulfur. For example, concentrated nitric acid oxidizes sulfur to the sulfate ion, with the concurrent formation of
nitrogen(IV) oxide:
+ 2−
S(s) + 6 HNO (aq) ⟶ 2 H O (aq) + SO (aq) + 6 NO (g)
3 3 4 2

The chemistry of sulfur with an oxidation state of 2− is similar to that of oxygen. Unlike oxygen, however, sulfur forms many
compounds in which it exhibits positive oxidation states.

Summary
Sulfur (group 16) reacts with almost all metals and readily forms the sulfide ion, S2−, in which it has as oxidation state of 2−. Sulfur
reacts with most nonmetals.

Glossary
Frasch process
important in the mining of free sulfur from enormous underground deposits

This page titled 18.10: Occurrence, Preparation, and Properties of Sulfur is shared under a CC BY 4.0 license and was authored, remixed, and/or
curated by OpenStax via source content that was edited to the style and standards of the LibreTexts platform; a detailed edit history is available
upon request.

Access for free at OpenStax 18.10.2 https://chem.libretexts.org/@go/page/38557


18.11: Occurrence, Preparation, and Properties of Halogens
 Learning Objectives
Describe the preparation, properties, and uses of halogens
Describe the properties, preparation, and uses of halogen compounds

The elements in group 17 are the halogens. These are the elements fluorine, chlorine, bromine, iodine, and astatine. These elements are too reactive
to occur freely in nature, but their compounds are widely distributed. Chlorides are the most abundant; although fluorides, bromides, and iodides are
less common, they are reasonably available. In this section, we will examine the occurrence, preparation, and properties of halogens. Next, we will
examine halogen compounds with the representative metals followed by an examination of the interhalogens. This section will conclude with some
applications of halogens.

18.11.1: Occurrence and Preparation


All of the halogens occur in seawater as halide ions. The concentration of the chloride ion is 0.54 M; that of the other halides is less than 10–4 M.
Fluoride also occurs in minerals such as CaF2, Ca(PO4)3F, and Na3AlF6. Chloride also occurs in the Great Salt Lake and the Dead Sea, and in
extensive salt beds that contain NaCl, KCl, or MgCl2. Part of the chlorine in your body is present as hydrochloric acid, which is a component of
stomach acid. Bromine compounds occur in the Dead Sea and underground brines. Iodine compounds are found in small quantities in Chile saltpeter,
underground brines, and sea kelp. Iodine is essential to the function of the thyroid gland.
The best sources of halogens (except iodine) are halide salts. It is possible to oxidize the halide ions to free diatomic halogen molecules by various
methods, depending on the ease of oxidation of the halide ion. Fluoride is the most difficult to oxidize, whereas iodide is the easiest.
The major method for preparing fluorine is electrolytic oxidation. The most common electrolysis procedure is to use a molten mixture of potassium
hydrogen fluoride, KHF2, and anhydrous hydrogen fluoride. Electrolysis causes HF to decompose, forming fluorine gas at the anode and hydrogen
at the cathode. It is necessary to keep the two gases separated to prevent their explosive recombination to reform hydrogen fluoride.
Most commercial chlorine comes from the electrolysis of the chloride ion in aqueous solutions of sodium chloride; this is the chlor-alkali process
discussed previously. Chlorine is also a product of the electrolytic production of metals such as sodium, calcium, and magnesium from their fused
chlorides. It is also possible to prepare chlorine by the chemical oxidation of the chloride ion in acid solution with strong oxidizing agents such as
manganese dioxide (MnO ) or sodium dichromate (Na Cr O ). The reaction with manganese dioxide is:
2 2 2 7

− + 2+
MnO 2(s) + 2 Cl +4 H O ⟶ Mn + Cl2(g) + 6 H O(l)
(aq) 3 (aq) (aq) 2

The commercial preparation of bromine involves the oxidation of bromide ion by chlorine:
− −
2 Br + Cl2(g) ⟶ Br 2(l) + 2 Cl
(aq) (aq)

Chlorine is a stronger oxidizing agent than bromine. This method is important for the production of essentially all domestic bromine.
Some iodine comes from the oxidation of iodine chloride, ICl, or iodic acid, HlO . The commercial preparation of iodine utilizes the reduction of
3

sodium iodate, NaIO , an impurity in deposits of Chile saltpeter, with sodium hydrogen sulfite:
3

− − − 2 −
2 IO + 5 HSO ⟶ 3 HSO + 2 SO + H O(l) + I2(s)
3(aq) 3(aq) 4(aq) 4(aq) 2

18.11.2: Properties of the Halogens


Fluorine is a pale yellow gas, chlorine is a greenish-yellow gas, bromine is a deep reddish-brown liquid, and iodine is a grayish-black crystalline
solid. Liquid bromine has a high vapor pressure, and the reddish vapor is readily visible in Figure 18.11.1. Iodine crystals have a noticeable vapor
pressure. When gently heated, these crystals sublime and form a beautiful deep violet vapor.

Figure 18.11.1: Chlorine is a pale yellow-green gas (left), gaseous bromine is deep orange (center), and gaseous iodine is purple (right). (Fluorine is
so reactive that it is too dangerous to handle.) (credit: Sahar Atwa)
Bromine is only slightly soluble in water, but it is miscible in all proportions in less polar (or nonpolar) solvents such as chloroform, carbon
tetrachloride, and carbon disulfide, forming solutions that vary from yellow to reddish-brown, depending on the concentration.

Access for free at OpenStax 18.11.1 https://chem.libretexts.org/@go/page/38558


Iodine is soluble in chloroform, carbon tetrachloride, carbon disulfide, and many hydrocarbons, giving violet solutions of I2 molecules. Iodine
dissolves only slightly in water, giving brown solutions. It is quite soluble in aqueous solutions of iodides, with which it forms brown solutions.
These brown solutions result because iodine molecules have empty valence d orbitals and can act as weak Lewis acids towards the iodide ion. The
equation for the reversible reaction of iodine (Lewis acid) with the iodide ion (Lewis base) to form triiodide ion, I , is: −
3

− −
I2(s) + I ⟶ I (18.11.1)
(aq) 3(aq)

The easier it is to oxidize the halide ion, the more difficult it is for the halogen to act as an oxidizing agent. Fluorine generally oxidizes an element to
its highest oxidation state, whereas the heavier halogens may not. For example, when excess fluorine reacts with sulfur, SF forms. Chlorine gives6

SCl and bromine, S Br . Iodine does not react with sulfur.


2 2 2

Fluorine is the most powerful oxidizing agent of the known elements. It spontaneously oxidizes most other elements; therefore, the reverse reaction,
the oxidation of fluorides, is very difficult to accomplish. Fluorine reacts directly and forms binary fluorides with all of the elements except the
lighter noble gases (He, Ne, and Ar). Fluorine is such a strong oxidizing agent that many substances ignite on contact with it. Drops of water inflame
in fluorine and form O , OF , H O , O , and HF. Wood and asbestos ignite and burn in fluorine gas. Most hot metals burn vigorously in fluorine.
2 2 2 2 3

However, it is possible to handle fluorine in copper, iron, or nickel containers because an adherent film of the fluoride salt passivates their surfaces.
Fluorine is the only element that reacts directly with the noble gas xenon.
Although it is a strong oxidizing agent, chlorine is less active than fluorine. Mixing chlorine and hydrogen in the dark makes the reaction between
them to be imperceptibly slow. Exposure of the mixture to light causes the two to react explosively. Chlorine is also less active towards metals than
fluorine, and oxidation reactions usually require higher temperatures. Molten sodium ignites in chlorine. Chlorine attacks most nonmetals (C, N2,
and O2 are notable exceptions), forming covalent molecular compounds. Chlorine generally reacts with compounds that contain only carbon and
hydrogen (hydrocarbons) by adding to multiple bonds or by substitution.
In cold water, chlorine undergoes a disproportionation reaction:
+ −
Cl2(aq) + 2 H O(l) ⟶ HOCl(aq) + H O + Cl
2 3 (aq) (aq)

Half the chlorine atoms oxidize to the 1+ oxidation state (hypochlorous acid), and the other half reduce to the 1− oxidation state (chloride ion). This
disproportionation is incomplete, so chlorine water is an equilibrium mixture of chlorine molecules, hypochlorous acid molecules, hydronium ions,
and chloride ions. When exposed to light, this solution undergoes a photochemical decomposition:
sunlight
+ −
2 HOCl(aq) + 2 H O(l) −−−−→2H O + 2 Cl + O2(g)
2 3 (aq) (aq)

The nonmetal chlorine is more electronegative than any other element except fluorine, oxygen, and nitrogen. In general, very electronegative
elements are good oxidizing agents; therefore, we would expect elemental chlorine to oxidize all of the other elements except for these three (and the
nonreactive noble gases). Its oxidizing property, in fact, is responsible for its principal use. For example, phosphorus(V) chloride, an important
intermediate in the preparation of insecticides and chemical weapons, is manufactured by oxidizing the phosphorus with chlorine:

P4(s) + 10 Cl2(g) ⟶ 4 PCl5(l)

A great deal of chlorine is also used to oxidize, and thus to destroy, organic or biological materials in water purification and in bleaching.
The chemical properties of bromine are similar to those of chlorine, although bromine is the weaker oxidizing agent and its reactivity is less than that
of chlorine.
Iodine is the least reactive of the halogens. It is the weakest oxidizing agent, and the iodide ion is the most easily oxidized halide ion. Iodine reacts
with metals, but heating is often required. It does not oxidize other halide ions.
Compared with the other halogens, iodine reacts only slightly with water. Traces of iodine in water react with a mixture of starch and iodide ion,
forming a deep blue color. This reaction is a very sensitive test for the presence of iodine in water.

18.11.3: Halides of the Representative Metals


Thousands of salts of the representative metals have been prepared. The binary halides are an important subclass of salts. A salt is an ionic
compound composed of cations and anions, other than hydroxide or oxide ions. In general, it is possible to prepare these salts from the metals or
from oxides, hydroxides, or carbonates. We will illustrate the general types of reactions for preparing salts through reactions used to prepare binary
halides.
The binary compounds of a metal with the halogens are the halides. Most binary halides are ionic. However, mercury, the elements of group 13 with
oxidation states of 3+, tin(IV), and lead(IV) form covalent binary halides. The direct reaction of a metal and a halogen produce the halide of the
metal. Examples of these oxidation-reduction reactions include:

Cd(s) + Cl2(g) ⟶ CdCl2(s)

2 Ga (l) + 3 Br 2(l) ⟶ 2 GaBr 3(s)

Access for free at OpenStax 18.11.2 https://chem.libretexts.org/@go/page/38558


Sodium in Chlorine gas

Video 18.11.1 : Reactions of the alkali metals with elemental halogens are very exothermic and often quite violent. Under controlled conditions,
they provide exciting demonstrations for budding students of chemistry. You can view the initial heating of the sodium that removes the coating of
sodium hydroxide, sodium peroxide, and residual mineral oil to expose the reactive surface. The reaction with chlorine gas then proceeds very
nicely.
If a metal can exhibit two oxidation states, it may be necessary to control the stoichiometry in order to obtain the halide with the lower oxidation
state. For example, preparation of tin(II) chloride requires a 1:1 ratio of Sn to Cl , whereas preparation of tin(IV) chloride requires a 1:2 ratio:
2

Sn (s) + Cl2(g) ⟶ SnCl 2(s)

Sn (s) + 2 Cl2(g) ⟶ SnCl 4(l)

The active representative metals—those that are easier to oxidize than hydrogen—react with gaseous hydrogen halides to produce metal halides and
hydrogen. The reaction of zinc with hydrogen fluoride is:

Zn(s) + 2 HF(g) ⟶ ZnF 2(s) + H2(g)

The active representative metals also react with solutions of hydrogen halides to form hydrogen and solutions of the corresponding halides.
Examples of such reactions include:

Cd(s) + 2 HBr (aq) ⟶ CdBr 2(aq) + H2(g)

Sn (s) + 2 HI(aq) ⟶ SnI 2(aq) + H2(g)

Hydroxides, carbonates, and some oxides react with solutions of the hydrogen halides to form solutions of halide salts. It is possible to prepare
additional salts by the reaction of these hydroxides, carbonates, and oxides with aqueous solution of other acids:

CaCo3(s) + 2 HCl(aq) ⟶ CaCl2(aq) + CO2(g) + H O(l)


2

TlOH(aq) + HF(aq) ⟶ TlF(aq) + H O(l)


2

A few halides and many of the other salts of the representative metals are insoluble. It is possible to prepare these soluble salts by metathesis
reactions that occur when solutions of soluble salts are mixed (Figure 18.11.2). Metathesis reactions are examined in the chapter on the
stoichiometry of chemical reactions.

Access for free at OpenStax 18.11.3 https://chem.libretexts.org/@go/page/38558


Figure 18.11.2: Solid HgI2 forms when solutions of KI and Hg(NO3)2 are mixed. (credit: Sahar Atwa).
Several halides occur in large quantities in nature. The ocean and underground brines contain many halides. For example, magnesium chloride in the
ocean is the source of magnesium ions used in the production of magnesium. Large underground deposits of sodium chloride, like the salt mine
shown in Figure 18.11.3, occur in many parts of the world. These deposits serve as the source of sodium and chlorine in almost all other compounds
containing these elements. The chlor-alkali process is one example.

Figure 18.11.3: Underground deposits of sodium chloride are found throughout the world and are often mined. This is a tunnel in the Kłodawa salt
mine in Poland. (credit: Jarek Zok).

18.11.4: Interhalogens
Compounds formed from two or more different halogens are interhalogens. Interhalogen molecules consist of one atom of the heavier halogen
bonded by single bonds to an odd number of atoms of the lighter halogen. The structures of IF3, IF5, and IF7 are illustrated in Figure 18.11.4
Formulas for other interhalogens, each of which comes from the reaction of the respective halogens, are in Table 18.11.1.

Figure 18.11.4: The structure of IF is T-shaped (left), IF is square pyramidal (center), and IF is pentagonal bipyramidal (right).
3 5 7

Three ball-and-stick models are shown. The left structure, labeled, “I F subscript 3,” shows a purple atom labeled, “I,” bonded to three green atoms
labeled, “F,” and with two lone pairs of electrons. The middle structure, labeled, “I F subscript 5,” shows a purple atom labeled, “I,” bonded to five
green atoms labeled, “F,” and with one lone pair of electrons. The right image, labeled, “I F subscript 7,” shows a purple atom labeled, “I,” bonded
to seven green atoms labeled, “F.”
Note from Table 18.11.1 that fluorine is able to oxidize iodine to its maximum oxidation state, 7+, whereas bromine and chlorine, which are more
difficult to oxidize, achieve only the 5+-oxidation state. A 7+-oxidation state is the limit for the halogens. Because smaller halogens are grouped
about a larger one, the maximum number of smaller atoms possible increases as the radius of the larger atom increases. Many of these compounds
are unstable, and most are extremely reactive. The interhalogens react like their component halides; halogen fluorides, for example, are stronger
oxidizing agents than are halogen chlorides.
Table 18.11.1: Interhalogens
YX YX3 YX5 YX7

BrCl(g)

BrF(g) BrF3(l) BrF5(l)

Access for free at OpenStax 18.11.4 https://chem.libretexts.org/@go/page/38558


YX YX3 YX5 YX7

ClF(g) ClF3(g) ClF5(g)

IBr(s)

ICl(l) ICl3(s)

IF(s) IF3(s) IF5(l) IF7(g)

The ionic polyhalides of the alkali metals, such as KI3, KICl2, KICl4, CsIBr2, and CsBrCl2, which contain an anion composed of at least three
halogen atoms, are closely related to the interhalogens. As seen previously, the formation of the polyhalide anion I is responsible for the solubility

of iodine in aqueous solutions containing an iodide ion.

18.11.5: Applications
The fluoride ion and fluorine compounds have many important uses. Compounds of carbon, hydrogen, and fluorine are replacing Freons
(compounds of carbon, chlorine, and fluorine) as refrigerants. Teflon is a polymer composed of –CF2CF2– units. Fluoride ion is added to water
supplies and to some toothpastes as SnF2 or NaF to fight tooth decay. Fluoride partially converts teeth from Ca5(PO4)3(OH) into Ca5(PO4)3F.
Chlorine is important to bleach wood pulp and cotton cloth. The chlorine reacts with water to form hypochlorous acid, which oxidizes colored
substances to colorless ones. Large quantities of chlorine are important in chlorinating hydrocarbons (replacing hydrogen with chlorine) to produce
compounds such as tetrachloride (CCl4), chloroform (CHCl3), and ethyl chloride (C2H5Cl), and in the production of polyvinyl chloride (PVC) and
other polymers. Chlorine is also important to kill the bacteria in community water supplies.
Bromine is important in the production of certain dyes, and sodium and potassium bromides are used as sedatives. At one time, light-sensitive silver
bromide was a component of photographic film.
Iodine in alcohol solution with potassium iodide is an antiseptic (tincture of iodine). Iodide salts are essential for the proper functioning of the
thyroid gland; an iodine deficiency may lead to the development of a goiter. Iodized table salt contains 0.023% potassium iodide. Silver iodide is
useful in the seeding of clouds to induce rain; it was important in the production of photographic film and iodoform, CHI3, is an antiseptic.

Summary
The halogens form halides with less electronegative elements. Halides of the metals vary from ionic to covalent; halides of nonmetals are covalent.
Interhalogens form by the combination of two or more different halogens. All of the representative metals react directly with elemental halogens or
with solutions of the hydrohalic acids (HF, HCl, HBr, and HI) to produce representative metal halides. Other laboratory preparations involve the
addition of aqueous hydrohalic acids to compounds that contain such basic anions, such as hydroxides, oxides, or carbonates.

This page titled 18.11: Occurrence, Preparation, and Properties of Halogens is shared under a CC BY 4.0 license and was authored, remixed, and/or curated by
OpenStax via source content that was edited to the style and standards of the LibreTexts platform; a detailed edit history is available upon request.

Access for free at OpenStax 18.11.5 https://chem.libretexts.org/@go/page/38558


18.12: Occurrence, Preparation, and Properties of the Noble Gases
 Learning Objectives
Describe the properties, preparation, and uses of the noble gases

The elements in group 18 are the noble gases (helium, neon, argon, krypton, xenon, and radon). They earned the name “noble”
because they were assumed to be nonreactive since they have filled valence shells. In 1962, Dr. Neil Bartlett at the University of
British Columbia proved this assumption to be false.
These elements are present in the atmosphere in small amounts. Some natural gas contains 1–2% helium by mass. Helium is
isolated from natural gas by liquefying the condensable components, leaving only helium as a gas. The United States possesses
most of the world’s commercial supply of this element in its helium-bearing gas fields. Argon, neon, krypton, and xenon come
from the fractional distillation of liquid air. Radon comes from other radioactive elements. More recently, it was observed that this
radioactive gas is present in very small amounts in soils and minerals. Its accumulation in well-insulated, tightly sealed buildings,
however, constitutes a health hazard, primarily lung cancer.
The boiling points and melting points of the noble gases are extremely low relative to those of other substances of comparable
atomic or molecular masses. This is because only weak London dispersion forces are present, and these forces can hold the atoms
together only when molecular motion is very slight, as it is at very low temperatures. Helium is the only substance known that does
not solidify on cooling at normal pressure. It remains liquid close to absolute zero (0.001 K) at ordinary pressures, but it solidifies
under elevated pressure.
Helium is used for filling balloons and lighter-than-air craft because it does not burn, making it safer to use than hydrogen. Helium
at high pressures is not a narcotic like nitrogen. Thus, mixtures of oxygen and helium are important for divers working under high
pressures. Using a helium-oxygen mixture avoids the disoriented mental state known as nitrogen narcosis, the so-called rapture of
the deep. Helium is important as an inert atmosphere for the melting and welding of easily oxidizable metals and for many
chemical processes that are sensitive to air.
Liquid helium (boiling point, 4.2 K) is an important coolant to reach the low temperatures necessary for cryogenic research, and it
is essential for achieving the low temperatures necessary to produce superconduction in traditional superconducting materials used
in powerful magnets and other devices. This cooling ability is necessary for the magnets used for magnetic resonance imaging, a
common medical diagnostic procedure. The other common coolant is liquid nitrogen (boiling point, 77 K), which is significantly
cheaper.
Neon is a component of neon lamps and signs. Passing an electric spark through a tube containing neon at low pressure generates
the familiar red glow of neon. It is possible to change the color of the light by mixing argon or mercury vapor with the neon or by
utilizing glass tubes of a special color.
Argon was useful in the manufacture of gas-filled electric light bulbs, where its lower heat conductivity and chemical inertness
made it preferable to nitrogen for inhibiting the vaporization of the tungsten filament and prolonging the life of the bulb.
Fluorescent tubes commonly contain a mixture of argon and mercury vapor. Argon is the third most abundant gas in dry air.
Krypton-xenon flash tubes are used to take high-speed photographs. An electric discharge through such a tube gives a very intense
1
light that lasts only of a second. Krypton forms a difluoride, KrF2, which is thermally unstable at room temperature.
50, 000

Stable compounds of xenon form when xenon reacts with fluorine. Xenon difluoride, XeF2, forms after heating an excess of xenon
gas with fluorine gas and then cooling. The material forms colorless crystals, which are stable at room temperature in a dry
atmosphere. Xenon tetrafluoride, XeF4, (Figure 18.12.1) and xenon hexafluoride, XeF6, are prepared in an analogous manner, with
a stoichiometric amount of fluorine and an excess of fluorine, respectively. Compounds with oxygen are prepared by replacing
fluorine atoms in the xenon fluorides with oxygen.

Access for free at OpenStax 18.12.1 https://chem.libretexts.org/@go/page/38559


Figure 18.12.1: On Oct. 2, 1962, Argonne announced the creation of xenon tetrafluoride, the first simple compound of xenon, a
noble gas widely thought to be chemically inert. The creation opened a new era for the study of chemical bonds. (Public Domain;
Argonne National Lab)
When XeF6 reacts with water, a solution of XeO3 results and the xenon remains in the 6+-oxidation state:

XeF (s) + 3 H O(l) ⟶ XeO (aq) + 6 HF(aq)


6 2 3

Dry, solid xenon trioxide, XeO3, is extremely explosive—it will spontaneously detonate. Both XeF6 and XeO3 disproportionate in
basic solution, producing xenon, oxygen, and salts of the perxenate ion, XeO , in which xenon reaches its maximum oxidation
4−

sate of 8+.
Radon apparently forms RnF2—evidence of this compound comes from radiochemical tracer techniques.
Unstable compounds of argon form at low temperatures, but stable compounds of helium and neon are not known.

Summary
The most significant property of the noble gases (group 18) is their inactivity. They occur in low concentrations in the atmosphere.
They find uses as inert atmospheres, neon signs, and as coolants. The three heaviest noble gases react with fluorine to form
fluorides. The xenon fluorides are the best characterized as the starting materials for a few other noble gas compounds.

Glossary
halide
compound containing an anion of a group 17 element in the 1− oxidation state (fluoride, F−; chloride, Cl−; bromide, Br−; and
iodide, I−)

interhalogen
compound formed from two or more different halogens

This page titled 18.12: Occurrence, Preparation, and Properties of the Noble Gases is shared under a CC BY 4.0 license and was authored,
remixed, and/or curated by OpenStax via source content that was edited to the style and standards of the LibreTexts platform; a detailed edit
history is available upon request.

Access for free at OpenStax 18.12.2 https://chem.libretexts.org/@go/page/38559


18.E: Representative Metals, Metalloids, and Nonmetals (Exercises)
18.E.1: 18.1: Periodicity
How do alkali metals differ from alkaline earth metals in atomic structure and general properties?
The alkali metals all have a single s electron in their outermost shell. In contrast, the alkaline earth metals have a completed s
subshell in their outermost shell. In general, the alkali metals react faster and are more reactive than the corresponding alkaline
earth metals in the same period.
Why does the reactivity of the alkali metals decrease from cesium to lithium?
Predict the formulas for the nine compounds that may form when each species in column 1 of Table reacts with each species in
column 2.

1 2

Na I

Sr Se

Al O

Na + I ⟶ 2 NaI (18.E.1)
2

2 Na + Se ⟶ Na Se
2

2 Na + O ⟶ Na O
2 2 2

Sr + I ⟶ SrI (18.E.2)
2 2

Sr + Se ⟶ SeSe

2 Sr + O ⟶ 2 SrO
2

2 Al + 3 I ⟶ 2 AlI (18.E.3)
2 3

2 Al + 3 Se ⟶ Al Se
2 3

4 Al + 3 O ⟶ 2 Al O
2 2 3

Predict the best choice in each of the following. You may wish to review the chapter on electronic structure for relevant examples.
1. (a) the most metallic of the elements Al, Be, and Ba
2. (b) the most covalent of the compounds NaCl, CaCl2, and BeCl2
3. (c) the lowest first ionization energy among the elements Rb, K, and Li
4. (d) the smallest among Al, Al+, and Al3+
5. (e) the largest among Cs+, Ba2+, and Xe
Sodium chloride and strontium chloride are both white solids. How could you distinguish one from the other?
The possible ways of distinguishing between the two include infrared spectroscopy by comparison of known compounds, a flame
test that gives the characteristic yellow color for sodium (strontium has a red flame), or comparison of their solubilities in water. At
35.7 g 53.8 g
20 °C, NaCl dissolves to the extent of compared with for SrCl2. Heating to 100 °C provides an easy test, since
100 mL 100 mL
39.12 g 100.8 g
the solubility of NaCl is , but that of SrCl2 is . Density determination on a solid is sometimes difficult, but there
100 mL 100 mL
is enough difference (2.165 g/mL NaCl and 3.052 g/mL SrCl2) that this method would be viable and perhaps the easiest and least
expensive test to perform.
The reaction of quicklime, CaO, with water produces slaked lime, Ca(OH)2, which is widely used in the construction industry to
make mortar and plaster. The reaction of quicklime and water is highly exothermic:
−1
CaO(s) + H O(l) ⟶ Ca (OH) (s) ΔH = −350 kJ mo l (18.E.4)
2 2

1. (a) What is the enthalpy of reaction per gram of quicklime that reacts?
2. (b) How much heat, in kilojoules, is associated with the production of 1 ton of slaked lime?

Access for free at OpenStax 18.E.1 https://chem.libretexts.org/@go/page/44128


Write a balanced equation for the reaction of elemental strontium with each of the following:
1. (a) oxygen
2. (b) hydrogen bromide
3. (c) hydrogen
4. (d) phosphorus
5. (e) water
(a) 2 Sr(s) + O (g) ⟶ 2 SrO(s)
2
; (b) Sr(s) + 2 HBr(g) ⟶ SrBr (s) + H (g) ; (c)2 2
Sr(s) + H (g) ⟶ SrH (s)
2 2
; (d)
6 Sr(s) + P (s) ⟶ 2 Sr P (s)
4 3 2
; (e) Sr(s) + 2 H O(l) ⟶ Sr(OH) (aq) + H (g)
2 2 2

How many moles of ionic species are present in 1.0 L of a solution marked 1.0 M mercury(I) nitrate?
What is the mass of fish, in kilograms, that one would have to consume to obtain a fatal dose of mercury, if the fish contains 30
parts per million of mercury by weight? (Assume that all the mercury from the fish ends up as mercury(II) chloride in the body and
that a fatal dose is 0.20 g of HgCl2.) How many pounds of fish is this?
11 lb
The elements sodium, aluminum, and chlorine are in the same period.
1. (a) Which has the greatest electronegativity?
2. (b) Which of the atoms is smallest?
3. (c) Write the Lewis structure for the simplest covalent compound that can form between aluminum and chlorine.
4. (d) Will the oxide of each element be acidic, basic, or amphoteric?
Does metallic tin react with HCl?
Yes, tin reacts with hydrochloric acid to produce hydrogen gas.
What is tin pest, also known as tin disease?
Compare the nature of the bonds in PbCl2 to that of the bonds in PbCl4.
In PbCl2, the bonding is ionic, as indicated by its melting point of 501 °C. In PbCl4, the bonding is covalent, as evidenced by it
being an unstable liquid at room temperature.
Is the reaction of rubidium with water more or less vigorous than that of sodium? How does the rate of reaction of magnesium
compare?

18.E.2: 18.2: Occurrence and Preparation of the Representative Metals


Write an equation for the reduction of cesium chloride by elemental calcium at high temperature.
countercurrent

f ractionating

tower

2 CsCl(l) + Ca(g) −−−−−−−→ 2 Cs(g) + CaCl (l) (18.E.5)


2

Why is it necessary to keep the chlorine and sodium, resulting from the electrolysis of sodium chloride, separate during the
production of sodium metal?
Give balanced equations for the overall reaction in the electrolysis of molten lithium chloride and for the reactions occurring at the
electrodes. You may wish to review the chapter on electrochemistry for relevant examples.
Cathode (reduction): 2 Li
+
+2 e

⟶ 2 Li(l) ; Anode (oxidation): 2 Cl

⟶ Cl (g) + 2 e
2

; Overall reaction:
+ −
2 Li + 2 Cl ⟶ 2 Li(l) + Cl (g)
2

The electrolysis of molten sodium chloride or of aqueous sodium chloride produces chlorine.
Calculate the mass of chlorine produced from 3.00 kg sodium chloride in each case. You may wish to review the chapter on
electrochemistry for relevant examples.
What mass, in grams, of hydrogen gas forms during the complete reaction of 10.01 g of calcium with water?
0.5035 g H2

Access for free at OpenStax 18.E.2 https://chem.libretexts.org/@go/page/44128


How many grams of oxygen gas are necessary to react completely with 3.01 × 1021 atoms of magnesium to yield magnesium
oxide?
Magnesium is an active metal; it burns in the form of powder, ribbons, and filaments to provide flashes of brilliant light. Why is it
possible to use magnesium in construction?
Despite its reactivity, magnesium can be used in construction even when the magnesium is going to come in contact with a flame
because a protective oxide coating is formed, preventing gross oxidation. Only if the metal is finely subdivided or present in a thin
sheet will a high-intensity flame cause its rapid burning.
Why is it possible for an active metal like aluminum to be useful as a structural metal?
Describe the production of metallic aluminum by electrolytic reduction.
Extract from ore: AlO(OH)(s) + NaOH(aq) + H 2
O(l) ⟶ Na[Al (OH) ](aq)
4

Recover: 2 Na[Al(OH) 4
](s) + H SO (aq) ⟶ 2 Al (OH) (s) + Na SO (aq) + 2 H O(l)
2 4 3 2 4 2

Sinter: 2 Al(OH) 3
(s) ⟶ Al O (s) + 3 H O(g)
2 3 2

Dissolve in Na3AlF6(l) and electrolyze: Al 3+ −


+3 e ⟶ Al(s)

What is the common ore of tin and how is tin separated from it?
A chemist dissolves a 1.497-g sample of a type of metal (an alloy of Sn, Pb, Sb, and Cu) in nitric acid, and metastannic acid,
H2SnO3, is precipitated. She heats the precipitate to drive off the water, which leaves 0.4909 g of tin(IV) oxide. What was the
percentage of tin in the original sample?
25.83%
Consider the production of 100 kg of sodium metal using a current of 50,000 A, assuming a 100% yield.
(a) How long will it take to produce the 100 kg of sodium metal?
(b) What volume of chlorine at 25 °C and 1.00 atm forms?
What mass of magnesium forms when 100,000 A is passed through a MgCl2 melt for 1.00 h if the yield of magnesium is 85% of
the theoretical yield?
39 kg

18.E.3: 18.3: Structure and General Properties of the Metalloids


Give the hybridization of the metalloid and the molecular geometry for each of the following compounds or ions. You may wish to
review the chapters on chemical bonding and advanced covalent bonding for relevant examples.
1. (a) GeH4
2. (b) SbF3
3. (c) Te(OH)6
4. (d) H2Te
5. (e) GeF2
6. (f) TeCl4
7. (g) SiF2−
6

8. (h) SbCl5
9. (i) TeF6
Write a Lewis structure for each of the following molecules or ions. You may wish to review the chapter on chemical bonding.
1. (a) H3BPH3
2. (b) BF−

3. (c) BBr3
4. (d) B(CH3)3
5. (e) B(OH)3
(a) H3BPH3:

Access for free at OpenStax 18.E.3 https://chem.libretexts.org/@go/page/44128


;
(b) BF :

;
(c) BBr3:

;
(d) B(CH3)3:

;
(e) B(OH)3:

Describe the hybridization of boron and the molecular structure about the boron in each of the following:
1. (a) H3BPH3
2. (b) BF −

3. (c) BBr3
4. (d) B(CH3)3
5. (e) B(OH)3
Using only the periodic table, write the complete electron configuration for silicon, including any empty orbitals in the valence
shell. You may wish to review the chapter on electronic structure.

Access for free at OpenStax 18.E.4 https://chem.libretexts.org/@go/page/44128


1s22s22p63s23p23d0.
Write a Lewis structure for each of the following molecules and ions:
1. (a) (CH3)3SiH
2. (b) SiO 4−
4

3. (c) Si2H6
4. (d) Si(OH)4
5. (e) SiF 2−
6

Describe the hybridization of silicon and the molecular structure of the following molecules and ions:
1. (a) (CH3)3SiH
2. (b) SiO 4−

3. (c) Si2H6
4. (d) Si(OH)4
5. (e) SiF 2−
6

(a) (CH3)3SiH: sp3 bonding about Si; the structure is tetrahedral; (b) SiO : sp3 bonding about Si; the structure is tetrahedral; (c)
4−

Si2H6: sp3 bonding about each Si; the structure is linear along the Si-Si bond; (d) Si(OH)4: sp3 bonding about Si; the structure is
tetrahedral; (e) SiF : sp3d2 bonding about Si; the structure is octahedral
2−
6

Describe the hybridization and the bonding of a silicon atom in elemental silicon.
Classify each of the following molecules as polar or nonpolar. You may wish to review the chapter on chemical bonding.
(a) SiH4
(b) Si2H6
(c) SiCl3H
(d) SiF4
(e) SiCl2F2
(a) nonpolar; (b) nonpolar; (c) polar; (d) nonpolar; (e) polar
Silicon reacts with sulfur at elevated temperatures. If 0.0923 g of silicon reacts with sulfur to give 0.3030 g of silicon sulfide,
determine the empirical formula of silicon sulfide.
Name each of the following compounds:
1. (a) TeO2
2. (b) Sb2S3
3. (c) GeF4
4. (d) SiH4
5. (e) GeH4
(a) tellurium dioxide or tellurium(IV) oxide; (b) antimony(III) sulfide; (c) germanium(IV) fluoride; (d) silane or silicon(IV)
hydride; (e) germanium(IV) hydride
Write a balanced equation for the reaction of elemental boron with each of the following (most of these reactions require high
temperature):
1. (a) F2
2. (b) O2
3. (c) S
4. (d) Se
5. (e) Br2
Why is boron limited to a maximum coordination number of four in its compounds?
Boron has only s and p orbitals available, which can accommodate a maximum of four electron pairs. Unlike silicon, no d orbitals
are available in boron.

Access for free at OpenStax 18.E.5 https://chem.libretexts.org/@go/page/44128


Write a formula for each of the following compounds:
1. (a) silicon dioxide
2. (b) silicon tetraiodide
3. (c) silane
4. (d) silicon carbide
5. (e) magnesium silicide
From the data given in Appendix I , determine the standard enthalpy change and the standard free energy change for each of the
following reactions:
1. (a) BF (g) + 3 H O(l) ⟶ B(OH) (s) + 3 HF(g)
3 2 3

2. (b) BCl (g) + 3 H O(l) ⟶ B(OH) (s) + 3 HCl(g)


3 2 3

3. (c) B H (g) + 6 H O(l) ⟶ 2 B(OH) (s) + 6 H (g)


2 6 2 3 2

(a) ΔH° = 87 kJ; ΔG° = 44 kJ; (b) ΔH° = −109.9 kJ; ΔG° = −154.7 kJ; (c) ΔH° = −510 kJ; ΔG° = −601.5 kJ
A hydride of silicon prepared by the reaction of Mg2Si with acid exerted a pressure of 306 torr at 26 °C in a bulb with a volume of
57.0 mL. If the mass of the hydride was 0.0861 g, what is its molecular mass? What is the molecular formula for the hydride?
Suppose you discovered a diamond completely encased in a silicate rock. How would you chemically free the diamond without
harming it?
A mild solution of hydrofluoric acid would dissolve the silicate and would not harm the diamond.

18.E.4: 18.4: Structure and General Properties of the Nonmetals


Carbon forms a number of allotropes, two of which are graphite and diamond. Silicon has a diamond structure. Why is there no
allotrope of silicon with a graphite structure?
Nitrogen in the atmosphere exists as very stable diatomic molecules. Why does phosphorus form less stable P4 molecules instead of
P2 molecules?
In the N2 molecule, the nitrogen atoms have an σ bond and two π bonds holding the two atoms together. The presence of three
strong bonds makes N2 a very stable molecule. Phosphorus is a third-period element, and as such, does not form π bonds
efficiently; therefore, it must fulfill its bonding requirement by forming three σ bonds.
Write balanced chemical equations for the reaction of the following acid anhydrides with water:
1. (a) SO3
2. (b) N2O3
3. (c) Cl2O7
4. (d) P4O10
5. (e) NO2
Determine the oxidation number of each element in each of the following compounds:
1. (a) HCN
2. (b) OF2
3. (c) AsCl3
(a) H = 1+, C = 2+, and N = 3−; (b) O = 2+ and F = 1−; (c) As = 3+ and Cl = 1−
Determine the oxidation state of sulfur in each of the following:
1. (a) SO3
2. (b) SO2
3. (c) SO 2−
3

Arrange the following in order of increasing electronegativity: F; Cl; O; and S.


S < Cl < O < F
Why does white phosphorus consist of tetrahedral P4 molecules while nitrogen consists of diatomic N2 molecules?

Access for free at OpenStax 18.E.6 https://chem.libretexts.org/@go/page/44128


18.E.5: 18.5: Occurrence, Preparation, and Compounds of Hydrogen
Why does hydrogen not exhibit an oxidation state of 1− when bonded to nonmetals?
The electronegativity of the nonmetals is greater than that of hydrogen. Thus, the negative charge is better represented on the
nonmetal, which has the greater tendency to attract electrons in the bond to itself.
The reaction of calcium hydride, CaH2, with water can be characterized as a Lewis acid-base reaction:

CaH (s) + 2 H O(l) ⟶ Ca (OH) (aq) + 2 H (g) (18.E.6)


2 2 2 2

Identify the Lewis acid and the Lewis base among the reactants. The reaction is also an oxidation-reduction reaction. Identify the
oxidizing agent, the reducing agent, and the changes in oxidation number that occur in the reaction.
In drawing Lewis structures, we learn that a hydrogen atom forms only one bond in a covalent compound. Why?
Hydrogen has only one orbital with which to bond to other atoms. Consequently, only one two-electron bond can form.
What mass of CaH2 is necessary to react with water to provide enough hydrogen gas to fill a balloon at 20 °C and 0.8 atm pressure
with a volume of 4.5 L? The balanced equation is:

CaH (s) + 2 H O(l) ⟶ Ca (OH) (aq) + 2 H (g) (18.E.7)


2 2 2 2

What mass of hydrogen gas results from the reaction of 8.5 g of KH with water?
KH + H O ⟶ KOH + H (18.E.8)
2 2

0.43 g H2

18.E.6: 18.6: Occurrence, Preparation, and Properties of Carbonates


Carbon forms the CO 2−

3
ion, yet silicon does not form an analogous SiO 2−

3
ion. Why?
Complete and balance the following chemical equations:
(a) hardening of plaster containing slaked lime
Ca (OH) + CO ⟶ (18.E.9)
2 2

(b) removal of sulfur dioxide from the flue gas of power plants
CaO + SO ⟶ (18.E.10)
2

(c) the reaction of baking powder that produces carbon dioxide gas and causes bread to rise
NaHCO + NaH PO ⟶ (18.E.11)
3 2 4

(a) Ca(OH) 2
(aq) + CO (g) ⟶ CaCO (s) + H O(l)
2 3 2
; (b) CaO(s) + SO 2
(g) ⟶ CaSO (s)
3
;
(c) 2 NaHCO 3
(s) + NaH PO (aq) ⟶ Na PO (aq) + 2 CO (g) + 2 H O(l)
2 4 3 4 2 2

Heating a sample of Na2CO3⋅xH2O weighing 4.640 g until the removal of the water of hydration leaves 1.720 g of anhydrous
Na2CO3. What is the formula of the hydrated compound?

18.E.7: 18.7: Occurrence, Preparation, and Properties of Nitrogen


Write the Lewis structures for each of the following:
1. (a) NH2−
2. (b) N2F4
3. (c) NH −
2

4. (d) NF3
5. (e) N

3

2−
(a) NH :

Access for free at OpenStax 18.E.7 https://chem.libretexts.org/@go/page/44128


; (b) N2F4: ; (c) NH : −

; (d) NF3: ; (e) N :



3

For each of the following, indicate the hybridization of the nitrogen atom (for N , the central nitrogen).

1. (a) N2F4
2. (b) NH −
2

3. (c) NF3
4. (d) N −
3

Explain how ammonia can function both as a Brønsted base and as a Lewis base.
Ammonia acts as a Brønsted base because it readily accepts protons and as a Lewis base in that it has an electron pair to donate.
Brønsted base: NH 3
+H O
3
+
⟶ NH
+
4
+H O
2
Lewis base: 2 NH 3
+
+ Ag ⟶ [ H N − Ag − NH ]
3 3
+

Determine the oxidation state of nitrogen in each of the following. You may wish to review the chapter on chemical bonding for
relevant examples.
1. (a) NCl3
2. (b) ClNO
3. (c) N2O5
4. (d) N2O3
5. (e) NO −

6. (f) N2O4
7. (g) N2O
8. (h) NO −

9. (i) HNO2
10. (j) HNO3
For each of the following, draw the Lewis structure, predict the ONO bond angle, and give the hybridization of the nitrogen. You
may wish to review the chapters on chemical bonding and advanced theories of covalent bonding for relevant examples.
(a) NO2
(b) NO −

(c) NO +
2

(a) NO2:

Nitrogen is sp2 hybridized. The molecule has a bent


geometry with an ONO bond angle of approximately 120°. (b) NO : −

Access for free at OpenStax 18.E.8 https://chem.libretexts.org/@go/page/44128


Nitrogen is sp2 hybridized. The molecule has a bent

geometry with an ONO bond angle slightly less than 120°. (c) NO : +
2

Nitrogen is sp hybridized. The molecule has a linear geometry with an ONO bond angle of 180°.
How many grams of gaseous ammonia will the reaction of 3.0 g hydrogen gas and 3.0 g of nitrogen gas produce?
Although PF5 and AsF5 are stable, nitrogen does not form NF5 molecules. Explain this difference among members of the same
group.
Nitrogen cannot form a NF5 molecule because it does not have d orbitals to bond with the additional two fluorine atoms.
The equivalence point for the titration of a 25.00-mL sample of CsOH solution with 0.1062 M HNO3 is at 35.27 mL. What is the
concentration of the CsOH solution?

18.E.8: 18.8: Occurrence, Preparation, and Properties of Phosphorus


Write the Lewis structure for each of the following. You may wish to review the chapter on chemical bonding and molecular
geometry.
1. (a) PH3
2. (b) PH+

3. (c) P2H4
4. (d) PO3−
4

5. (e) PF5
(a)

;
(b)

;
(c)

;
(d)

;
(e)

Access for free at OpenStax 18.E.9 https://chem.libretexts.org/@go/page/44128


Describe the molecular structure of each of the following molecules or ions listed. You may wish to review the chapter on chemical
bonding and molecular geometry.
1. (a) PH3
2. (b) PH +

3. (c) P2H4
4. (d) PO 3−
4

Complete and balance each of the following chemical equations. (In some cases, there may be more than one correct answer.)
1. (a) P + Al ⟶
4

2. (b) P + Na ⟶
4

3. (c) P + F ⟶
4 2

4. (d) P + Cl ⟶
4 2

5. (e) P + O ⟶
4 2

6. (f) P O + O ⟶
4 6 2

(a) P (s) + 4 Al(s) ⟶ 4 AlP(s)


4
; (b) P (s) + 12 Na(s) ⟶ 4 Na P(s)
4 3
; (c) P (s) + 10 F (g) ⟶ 4 PF (l)
4 2 5
; (d)
P (s) + 6 Cl (g) ⟶ 4 PCl (l)
4 2 3
or P (s) + 10 Cl (g) ⟶ 4 PCl (l)
4 2 5
; (e) P (s) + 3 O (g) ⟶ P O (s)
4 2 4 6
or
P (s) + 5 O (g) ⟶ P O
4 2 4 10
(s) ; (f) P 4
O (s) + 2 O (g) ⟶ P O
6 2 4 10
(s)

Describe the hybridization of phosphorus in each of the following compounds: P4O10, P4O6, PH4I (an ionic compound), PBr3,
H3PO4, H3PO3, PH3, and P2H4. You may wish to review the chapter on advanced theories of covalent bonding.
What volume of 0.200 M NaOH is necessary to neutralize the solution produced by dissolving 2.00 g of PCl3 is an excess of water?
Note that when H3PO3 is titrated under these conditions, only one proton of the acid molecule reacts.
291 mL
How much POCl3 can form from 25.0 g of PCl5 and the appropriate amount of H2O?
How many tons of Ca3(PO4)2 are necessary to prepare 5.0 tons of phosphorus if the yield is 90%?
28 tons
Write equations showing the stepwise ionization of phosphorous acid.
Draw the Lewis structures and describe the geometry for the following:
1. (a) PF +
4

2. (b) PF5
3. (c) PF −
6

4. (d) POF3
(a)

;
(b)

Access for free at OpenStax 18.E.10 https://chem.libretexts.org/@go/page/44128


;
(c)

;
(d)

Why does phosphorous acid form only two series of salts, even though the molecule contains three hydrogen atoms?
Assign an oxidation state to phosphorus in each of the following:
1. (a) NaH2PO3
2. (b) PF5
3. (c) P4O6
4. (d) K3PO4
5. (e) Na3P
6. (f) Na4P2O7
(a) P = 3+; (b) P = 5+; (c) P = 3+; (d) P = 5+; (e) P = 3−; (f) P = 5+
Phosphoric acid, one of the acids used in some cola drinks, is produced by the reaction of phosphorus(V) oxide, an acidic oxide,
with water. Phosphorus(V) oxide is prepared by the combustion of phosphorus.
1. (a) Write the empirical formula of phosphorus(V) oxide.
2. (b) What is the molecular formula of phosphorus(V) oxide if the molar mass is about 280.
3. (c) Write balanced equations for the production of phosphorus(V) oxide and phosphoric acid.
4. (d) Determine the mass of phosphorus required to make 1.00 × 104 kg of phosphoric acid, assuming a yield of 98.85%.

18.E.9: 18.9: Occurrence, Preparation, and Compounds of Oxygen


Predict the product of burning francium in air.
FrO2
Using equations, describe the reaction of water with potassium and with potassium oxide.
Write balanced chemical equations for the following reactions:
1. (a) zinc metal heated in a stream of oxygen gas

Access for free at OpenStax 18.E.11 https://chem.libretexts.org/@go/page/44128


2. (b) zinc carbonate heated until loss of mass stops
3. (c) zinc carbonate added to a solution of acetic acid, CH3CO2H
4. (d) zinc added to a solution of hydrobromic acid
(a) 2 Zn(s) + O (g) ⟶ 2 ZnO(s)
2
; (b) ZnCO (s) ⟶ ZnO(s) + CO (g)
3 2
; (c)
ZnCO (s) + 2 CH COOH(aq) ⟶ Zn(CH COO) (aq) + CO (g) + H O(l)
3 3 3 2 2 2
; (d)
Zn(s) + 2 HBr(aq) ⟶ ZnBr (aq) + H (g)
2 2

Write balanced chemical equations for the following reactions:


1. (a) cadmium burned in air
2. (b) elemental cadmium added to a solution of hydrochloric acid
3. (c) cadmium hydroxide added to a solution of acetic acid, CH3CO2H
Illustrate the amphoteric nature of aluminum hydroxide by citing suitable equations.

Al (OH) (s) + 3 H
3
+
(aq) ⟶ Al
3+
+ 3 H O(l)
2
; Al(OH) 3
(s) + OH

⟶ [Al (OH) ]
4
(aq)

Write balanced chemical equations for the following reactions:


1. (a) metallic aluminum burned in air
2. (b) elemental aluminum heated in an atmosphere of chlorine
3. (c) aluminum heated in hydrogen bromide gas
4. (d) aluminum hydroxide added to a solution of nitric acid
Write balanced chemical equations for the following reactions:
1. (a) sodium oxide added to water
2. (b) cesium carbonate added to an excess of an aqueous solution of HF
3. (c) aluminum oxide added to an aqueous solution of HClO4
4. (d) a solution of sodium carbonate added to solution of barium nitrate
5. (e) titanium metal produced from the reaction of titanium tetrachloride with elemental sodium
(a) Na O(s) + H O(l) ⟶ 2 NaOH(aq)
2 2
; (b) Cs CO (s) + 2 HF(aq) ⟶ 2 CsF(aq) + CO (g) + H O(l)
2 3 2 2
; (c)
Al O (s) + 6 HClO (aq) ⟶ 2 Al (ClO ) (aq) + 3 H O(l)
2 3 4 4 3 2
; (d)
Na CO (aq) + Ba (NO ) (aq) ⟶ 2 NaNO (aq) + BaCO (s)
2 3 3 2 3 3
; (e) TiCl 4
(l) + 4 Na(s) ⟶ Ti(s) + 4 NaCl(s)

What volume of 0.250 M H2SO4 solution is required to neutralize a solution that contains 5.00 g of CaCO3?
Which is the stronger acid, HClO4 or HBrO4? Why?
HClO4 is the stronger acid because, in a series of oxyacids with similar formulas, the higher the electronegativity of the central
atom, the stronger is the attraction of the central atom for the electrons of the oxygen(s). The stronger attraction of the oxygen
electron results in a stronger attraction of oxygen for the electrons in the O-H bond, making the hydrogen more easily released. The
weaker this bond, the stronger the acid.
Write a balanced chemical equation for the reaction of an excess of oxygen with each of the following. Remember that oxygen is a
strong oxidizing agent and tends to oxidize an element to its maximum oxidation state.
1. (a) Mg
2. (b) Rb
3. (c) Ga
4. (d) C2H2
5. (e) CO
Which is the stronger acid, H2SO4 or H2SeO4? Why? You may wish to review the chapter on acid-base equilibria.
As H2SO4 and H2SeO4 are both oxyacids and their central atoms both have the same oxidation number, the acid strength depends
on the relative electronegativity of the central atom. As sulfur is more electronegative than selenium, H2SO4 is the stronger acid.

18.E.10: 18.10: Occurrence, Preparation, and Properties of Sulfur


Explain why hydrogen sulfide is a gas at room temperature, whereas water, which has a lower molecular mass, is a liquid.

Access for free at OpenStax 18.E.12 https://chem.libretexts.org/@go/page/44128


Give the hybridization and oxidation state for sulfur in SO2, in SO3, and in H2SO4.
SO2, sp2 4+; SO3, sp2, 6+; H2SO4, sp3, 6+
Which is the stronger acid, NaHSO3 or NaHSO4?
Determine the oxidation state of sulfur in SF6, SO2F2, and KHS.
SF6: S = 6+; SO2F2: S = 6+; KHS: S = 2−
Which is a stronger acid, sulfurous acid or sulfuric acid? Why?
Oxygen forms double bonds in O2, but sulfur forms single bonds in S8. Why?
Sulfur is able to form double bonds only at high temperatures (substantially endothermic conditions), which is not the case for
oxygen.
Give the Lewis structure of each of the following:
1. (a) SF4
2. (b) K2SO4
3. (c) SO2Cl2
4. (d) H2SO3
5. (e) SO3
Write two balanced chemical equations in which sulfuric acid acts as an oxidizing agent.
There are many possible answers including:

Cu(s) + 2 H SO (l) ⟶ CuSO (aq) + SO (g) + 2 H O(l) (18.E.12)


2 4 4 2 2

C(s) + 2 H SO (l) ⟶ CO (g) + 2 SO (g) + 2 H O(l) (18.E.13)


2 4 2 2 2

Explain why sulfuric acid, H2SO4, which is a covalent molecule, dissolves in water and produces a solution that contains ions.
How many grams of Epsom salts (MgSO4⋅7H2O) will form from 5.0 kg of magnesium?
5.1 × 104 g

18.E.11: 18.11: Occurrence, Preparation, and Properties of Halogens


What does it mean to say that mercury(II) halides are weak electrolytes?
Why is SnCl4 not classified as a salt?
SnCl4 is not a salt because it is covalently bonded. A salt must have ionic bonds.
The following reactions are all similar to those of the industrial chemicals. Complete and balance the equations for these reactions:
(a) reaction of a weak base and a strong acid
NH + HClO ⟶ (18.E.14)
3 4

(b) preparation of a soluble silver salt for silver plating


Ag CO + HNO ⟶ (18.E.15)
2 3 3

(c) preparation of strontium hydroxide by electrolysis of a solution of strontium chloride


electrolysis

SrCl (aq) + H O(l) −−−−−−→ (18.E.16)


2 2

Which is the stronger acid, HClO3 or HBrO3? Why?


In oxyacids with similar formulas, the acid strength increases as the electronegativity of the central atom increases. HClO3 is
stronger than HBrO3; Cl is more electronegative than Br.
What is the hybridization of iodine in IF3 and IF5?
Predict the molecular geometries and draw Lewis structures for each of the following. You may wish to review the chapter on
chemical bonding and molecular geometry.

Access for free at OpenStax 18.E.13 https://chem.libretexts.org/@go/page/44128


(a) IF5
(b) I −
3

(c) PCl5
(d) SeF4
(e) ClF3
(a)

;
(b)

;
(c)

;
(d)

;
(e)

Which halogen has the highest ionization energy? Is this what you would predict based on what you have learned about periodic
properties?
Name each of the following compounds:
(a) BrF3

Access for free at OpenStax 18.E.14 https://chem.libretexts.org/@go/page/44128


(b) NaBrO3
(c) PBr5
(d) NaClO4
(e) KClO
(a) bromine trifluoride; (b) sodium bromate; (c) phosphorus pentabromide; (d) sodium perchlorate; (e) potassium hypochlorite
Explain why, at room temperature, fluorine and chlorine are gases, bromine is a liquid, and iodine is a solid.
What is the oxidation state of the halogen in each of the following?
(a) H5IO6
(b) IO −

(c) ClO2
(d) ICl3
(e) F2
(a) I: 7+; (b) I: 7+; (c) Cl: 4+; (d) I: 3+; Cl: 1−; (e) F: 0
Physiological saline concentration—that is, the sodium chloride concentration in our bodies—is approximately 0.16 M. A saline
solution for contact lenses is prepared to match the physiological concentration. If you purchase 25 mL of contact lens saline
solution, how many grams of sodium chloride have you bought?

18.E.12: 18.12: Occurrence, Preparation, and Properties of the Noble Gases


Give the hybridization of xenon in each of the following. You may wish to review the chapter on the advanced theories of covalent
bonding.
1. (a) XeF2
2. (b) XeF4
3. (c) XeO3
4. (d) XeO4
5. (e) XeOF4
(a) sp3d hybridized; (b) sp3d2 hybridized; (c) sp3 hybridized; (d) sp3 hybridized; (e) sp3d2 hybridized;
What is the molecular structure of each of the following molecules? You may wish to review the chapter on chemical bonding and
molecular geometry.
1. (a) XeF2
2. (b) XeF4
3. (c) XeO3
4. (d) XeO4
5. (e) XeOF4
Indicate whether each of the following molecules is polar or nonpolar. You may wish to review the chapter on chemical bonding
and molecular geometry.
1. (a) XeF2
2. (b) XeF4
3. (c) XeO3
4. (d) XeO4
5. (e) XeOF4
(a) nonpolar; (b) nonpolar; (c) polar; (d) nonpolar; (e) polar
What is the oxidation state of the noble gas in each of the following? You may wish to review the chapter on chemical bonding and
molecular geometry.
1. (a) XeO2F2

Access for free at OpenStax 18.E.15 https://chem.libretexts.org/@go/page/44128


2. (b) KrF2
3. (c) XeF +
3

4. (d) XeO 4−

5. (e) XeO3
A mixture of xenon and fluorine was heated. A sample of the white solid that formed reacted with hydrogen to yield 81 mL of
xenon (at STP) and hydrogen fluoride, which was collected in water, giving a solution of hydrofluoric acid. The hydrofluoric acid
solution was titrated, and 68.43 mL of 0.3172 M sodium hydroxide was required to reach the equivalence point. Determine the
empirical formula for the white solid and write balanced chemical equations for the reactions involving xenon.
The empirical formula is XeF6, and the balanced reactions are:
Δ

Xe(g) + 3 F (g) −
→ XeF (s) (18.E.17)
2 6

XeF (s) + 3 H (g) ⟶ 6 HF(g) + Xe(g) (18.E.18)


6 2

Basic solutions of Na4XeO6 are powerful oxidants. What mass of Mn(NO3)2•6H2O reacts with 125.0 mL of a 0.1717 M basic
solution of Na4XeO6 that contains an excess of sodium hydroxide if the products include Xe and solution of sodium permanganate?

This page titled 18.E: Representative Metals, Metalloids, and Nonmetals (Exercises) is shared under a CC BY 4.0 license and was authored,
remixed, and/or curated by OpenStax via source content that was edited to the style and standards of the LibreTexts platform; a detailed edit
history is available upon request.

Access for free at OpenStax 18.E.16 https://chem.libretexts.org/@go/page/44128


CHAPTER OVERVIEW
19: Transition Metals and Coordination Chemistry

A general chemistry Libretexts Textbook remixed and remastered from


OpenStax's textbook:
General Chemistry
Transition metals are defined as those elements that have (or readily form) partially filled d orbitals. These include the d-block
(groups 3–11) and f-block element elements. The variety of properties exhibited by transition metals is due to their complex
valence shells. Unlike most main group metals where one oxidation state is normally observed, the valence shell structure of
transition metals means that they usually occur in several different stable oxidation states. In addition, electron transitions in these
elements can correspond with absorption of photons in the visible electromagnetic spectrum, leading to colored compounds.
Because of these behaviors, transition metals exhibit a rich and fascinating chemistry.
19.1: Properties of Transition Metals and Their Compounds
19.2: Coordination Chemistry of Transition Metals
19.3: Optical and Magnetic Properties of Coordination Compounds
19.E: Transition Metals and Coordination Chemistry (Exercises)

This page titled 19: Transition Metals and Coordination Chemistry is shared under a CC BY 4.0 license and was authored, remixed, and/or
curated by OpenStax via source content that was edited to the style and standards of the LibreTexts platform; a detailed edit history is available
upon request.

1
19.1: Properties of Transition Metals and Their Compounds
 Learning Objectives
Outline the general approach for the isolation of transition metals from natural sources
Describe typical physical and chemical properties of the transition metals
Identify simple compound classes for transition metals and describe their chemical properties

We have daily contact with many transition metals. Iron occurs everywhere—from the rings in your spiral notebook and the cutlery
in your kitchen to automobiles, ships, buildings, and in the hemoglobin in your blood. Titanium is useful in the manufacture of
lightweight, durable products such as bicycle frames, artificial hips, and jewelry. Chromium is useful as a protective plating on
plumbing fixtures and automotive detailing.
Transition metals are defined as those elements that have (or readily form) partially filled d orbitals. As shown in Figure 19.1.2, the
d-block elements in groups 3–11 are transition elements. The f-block elements, also called inner transition metals (the lanthanides
and actinides), also meet this criterion because the d orbital is partially occupied before the f orbitals. The d orbitals fill with the
copper family (group 11); for this reason, the next family (group 12) are technically not transition elements. However, the group 12
elements do display some of the same chemical properties and are commonly included in discussions of transition metals. Some
chemists do treat the group 12 elements as transition metals.

Figure 19.1.1 : Transition metals often form vibrantly colored complexes. The minerals malachite (green), azurite (blue), and
proustite (red) are some examples. (credit left: modification of work by James St. John; credit middle: modification of work by
Stephanie Clifford; credit right: modification of work by Terry Wallace)
The first crystal is a jade green mineral chunk. The second is a crystalline chunk which is primarily bright royal blue. The third
shows long red crystals.
The d-block elements are divided into the first transition series (the elements Sc through Cu), the second transition series (the
elements Y through Ag), and the third transition series (the element La and the elements Hf through Au). Actinium, Ac, is the first
member of the fourth transition series, which also includes Rf through Rg.

Figure 19.1.2 : The transition metals are located in groups 3–11 of the periodic table. The inner transition metals are in the two rows
below the body of the table.
The f-block elements are the elements Ce through Lu, which constitute the lanthanide series (or lanthanoid series), and the elements
Th through Lr, which constitute the actinide series (or actinoid series). Because lanthanum behaves very much like the lanthanide

Access for free at OpenStax 19.1.1 https://chem.libretexts.org/@go/page/38321


elements, it is considered a lanthanide element, even though its electron configuration makes it the first member of the third
transition series. Similarly, the behavior of actinium means it is part of the actinide series, although its electron configuration makes
it the first member of the fourth transition series.

 Example 19.1.1: Valence Electrons in Transition Metals

Review how to write electron configurations, covered in the chapter on electronic structure and periodic properties of elements.
Recall that for the transition and inner transition metals, it is necessary to remove the s electrons before the d or f electrons.
Then, for each ion, give the electron configuration:
a. cerium(III)
b. lead(II)
c. Ti2+
d. Am3+
e. Pd2+
For the examples that are transition metals, determine to which series they belong.

Solution
For ions, the s-valence electrons are lost prior to the d or f electrons.
a. Ce3+[Xe]4f1; Ce3+ is an inner transition element in the lanthanide series.
b. Pb2+[Xe]6s25d104f14; the electrons are lost from the p orbital. This is a main group element.
c. titanium(II) [Ar]3d2; first transition series
d. americium(III) [Rn]5f6; actinide
e. palladium(II) [Kr]4d8; second transition series

 Exercise 19.1.1

Check Your Learning Give an example of an ion from the first transition series with no d electrons.

Answer
V5+ is one possibility. Other examples include Sc3+, Ti4+, Cr6+, and Mn7+.

 Uses of Lanthanides in Devices

Lanthanides (elements 57–71) are fairly abundant in the earth’s crust, despite their historic characterization as rare earth
elements. Thulium, the rarest naturally occurring lanthanoid, is more common in the earth’s crust than silver (4.5 × 10−5%
versus 0.79 × 10−5% by mass). There are 17 rare earth elements, consisting of the 15 lanthanoids plus scandium and yttrium.
They are called rare because they were once difficult to extract economically, so it was rare to have a pure sample; due to
similar chemical properties, it is difficult to separate any one lanthanide from the others. However, newer separation methods,
such as ion exchange resins similar to those found in home water softeners, make the separation of these elements easier and
more economical. Most ores that contain these elements have low concentrations of all the rare earth elements mixed together.
The commercial applications of lanthanides are growing rapidly. For example, europium is important in flat screen displays
found in computer monitors, cell phones, and televisions. Neodymium is useful in laptop hard drives and in the processes that
convert crude oil into gasoline (Figure 19.1.3). Holmium is found in dental and medical equipment. In addition, many
alternative energy technologies rely heavily on lanthanoids. Neodymium and dysprosium are key components of hybrid vehicle
engines and the magnets used in wind turbines.

Access for free at OpenStax 19.1.2 https://chem.libretexts.org/@go/page/38321


Figure 19.1.3 : (a) Europium is used in display screens for televisions, computer monitors, and cell phones. (b) Neodymium
magnets are commonly found in computer hard drives. (credit b: modification of work by “KUERT Datenrettung”/Flickr)
A. Closeup of a flat screen shows the many individual pixels which are red green and blue in color. B. A computer hard drive
is shown with a metallic component highlighted in a red circle.
As the demand for lanthanide materials has increased faster than supply, prices have also increased. In 2008, dysprosium cost
$110/kg; by 2014, the price had increased to $470/kg. Increasing the supply of lanthanoid elements is one of the most
significant challenges facing the industries that rely on the optical and magnetic properties of these materials.

The transition elements have many properties in common with other metals. They are almost all hard, high-melting solids that
conduct heat and electricity well. They readily form alloys and lose electrons to form stable cations. In addition, transition metals
form a wide variety of stable coordination compounds, in which the central metal atom or ion acts as a Lewis acid and accepts one
or more pairs of electrons. Many different molecules and ions can donate lone pairs to the metal center, serving as Lewis bases. In
this chapter, we shall focus primarily on the chemical behavior of the elements of the first transition series.

19.1.1: Properties of the Transition Elements


Transition metals demonstrate a wide range of chemical behaviors. As can be seen from their reduction potentials (Table P1), some
transition metals are strong reducing agents, whereas others have very low reactivity. For example, the lanthanides all form stable
3+ aqueous cations. The driving force for such oxidations is similar to that of alkaline earth metals such as Be or Mg, forming Be2+
and Mg2+. On the other hand, materials like platinum and gold have much higher reduction potentials. Their ability to resist
oxidation makes them useful materials for constructing circuits and jewelry.
Ions of the lighter d-block elements, such as Cr3+, Fe3+, and Co2+, form colorful hydrated ions that are stable in water. However,
ions in the period just below these (Mo3+, Ru3+, and Ir2+) are unstable and react readily with oxygen from the air. The majority of
simple, water-stable ions formed by the heavier d-block elements are oxyanions such as MoO and ReO . 2−
4

4

Ruthenium, osmium, rhodium, iridium, palladium, and platinum are the platinum metals. With difficulty, they form simple cations
that are stable in water, and, unlike the earlier elements in the second and third transition series, they do not form stable oxyanions.
Both the d- and f-block elements react with nonmetals to form binary compounds; heating is often required. These elements react
with halogens to form a variety of halides ranging in oxidation state from 1+ to 6+. On heating, oxygen reacts with all of the
transition elements except palladium, platinum, silver, and gold. The oxides of these latter metals can be formed using other
reactants, but they decompose upon heating. The f-block elements, the elements of group 3, and the elements of the first transition
series except copper react with aqueous solutions of acids, forming hydrogen gas and solutions of the corresponding salts.
Transition metals can form compounds with a wide range of oxidation states. Some of the observed oxidation states of the elements
of the first transition series are shown in Figure 19.1.4. As we move from left to right across the first transition series, we see that
the number of common oxidation states increases at first to a maximum towards the middle of the table, then decreases. The values
in the table are typical values; there are other known values, and it is possible to synthesize new additions. For example, in 2014,
researchers were successful in synthesizing a new oxidation state of iridium (9+).

Access for free at OpenStax 19.1.3 https://chem.libretexts.org/@go/page/38321


Figure 19.1.4 :Transition metals of the first transition series can form compounds with varying oxidation states.
Scandium has oxidation state of positive 3. Titanium has values of positive 3 and 4. Vanadium has values of positive 2 to 5.
Chromium has values of positive 2,3,4, and 6. Manganese has positive 2,3,4,6, and 7. Iron has positive 2,3, and 6. Cobalt and
Nickel has positive 2 and 3. Copper has positive 1,2, and 3. Zinc only has positive 2.
For the elements scandium through manganese (the first half of the first transition series), the highest oxidation state corresponds to
the loss of all of the electrons in both the s and d orbitals of their valence shells. The titanium(IV) ion, for example, is formed when
the titanium atom loses its two 3d and two 4s electrons. These highest oxidation states are the most stable forms of scandium,
titanium, and vanadium. However, it is not possible to continue to remove all of the valence electrons from metals as we continue
through the series. Iron is known to form oxidation states from 2+ to 6+, with iron(II) and iron(III) being the most common. Most
of the elements of the first transition series form ions with a charge of 2+ or 3+ that are stable in water, although those of the early
members of the series can be readily oxidized by air.
The elements of the second and third transition series generally are more stable in higher oxidation states than are the elements of
the first series. In general, the atomic radius increases down a group, which leads to the ions of the second and third series being
larger than are those in the first series. Removing electrons from orbitals that are located farther from the nucleus is easier than
removing electrons close to the nucleus. For example, molybdenum and tungsten, members of group 6, are limited mostly to an
oxidation state of 6+ in aqueous solution. Chromium, the lightest member of the group, forms stable Cr3+ ions in water and, in the
absence of air, less stable Cr2+ ions. The sulfide with the highest oxidation state for chromium is Cr2S3, which contains the Cr3+
ion. Molybdenum and tungsten form sulfides in which the metals exhibit oxidation states of 4+ and 6+.

 Example 19.1.2: Activity of the Transition Metals

Which is the strongest oxidizing agent in acidic solution: dichromate ion, which contains chromium(VI), permanganate ion,
which contains manganese(VII), or titanium dioxide, which contains titanium(IV)?

Solution
First, we need to look up the reduction half reactions (Table P1) for each oxide in the specified oxidation state:
2− + − 3+
Cr O + 14 H +6 e ⟶ 2 Cr +7 H O +1.33 V
2 7 2

− + − 2+
MnO +8 H +5 e ⟶ Mn +H O +1.51 V
4 2

+ − 2+
TiO +4 H +2 e ⟶ Ti +2 H O −0.50 V
2 2

A larger reduction potential means that it is easier to reduce the reactant. Permanganate, with the largest reduction potential, is
the strongest oxidizer under these conditions. Dichromate is next, followed by titanium dioxide as the weakest oxidizing agent
(the hardest to reduce) of this set.

 Exercise 19.1.2

Predict what reaction (if any) will occur between HCl and Co(s), and between HBr and Pt(s). You will need to use the standard
reduction potentials from (Table P1).

Answer
Co(s) + 2 HCl ⟶ H
2
+ CoCl (aq)
2
; no reaction because Pt(s) will not be oxidized by H+

Access for free at OpenStax 19.1.4 https://chem.libretexts.org/@go/page/38321


19.1.2: Preparation of the Transition Elements
Ancient civilizations knew about iron, copper, silver, and gold. The time periods in human history known as the Bronze Age and
Iron Age mark the advancements in which societies learned to isolate certain metals and use them to make tools and goods.
Naturally occurring ores of copper, silver, and gold can contain high concentrations of these metals in elemental form (Figure
19.1.5). Iron, on the other hand, occurs on earth almost exclusively in oxidized forms, such as rust (Fe2O3). The earliest known

iron implements were made from iron meteorites. Surviving iron artifacts dating from approximately 4000 to 2500 BC are rare, but
all known examples contain specific alloys of iron and nickel that occur only in extraterrestrial objects, not on earth. It took
thousands of years of technological advances before civilizations developed iron smelting, the ability to extract a pure element
from its naturally occurring ores and for iron tools to become common.

Figure 19.1.5 : Transition metals occur in nature in various forms. Examples include (a) a nugget of copper, (b) a deposit of gold,
and (c) an ore containing oxidized iron. (credit a: modification of work by Copper [images-of-elements.com]; credit c: modification
of work by Iron ore [images-of-elements.com])
Generally, the transition elements are extracted from minerals found in a variety of ores. However, the ease of their recovery varies
widely, depending on the concentration of the element in the ore, the identity of the other elements present, and the difficulty of
reducing the element to the free metal.
In general, it is not difficult to reduce ions of the d-block elements to the free element. Carbon is a sufficiently strong reducing
agent in most cases. However, like the ions of the more active main group metals, ions of the f-block elements must be isolated by
electrolysis or by reduction with an active metal such as calcium.
We shall discuss the processes used for the isolation of iron, copper, and silver because these three processes illustrate the principal
means of isolating most of the d-block metals. In general, each of these processes involves three principal steps: preliminary
treatment, smelting, and refining.
1. Preliminary treatment. In general, there is an initial treatment of the ores to make them suitable for the extraction of the metals.
This usually involves crushing or grinding the ore, concentrating the metal-bearing components, and sometimes treating these
substances chemically to convert them into compounds that are easier to reduce to the metal.
2. Smelting. The next step is the extraction of the metal in the molten state, a process called smelting, which includes reduction of
the metallic compound to the metal. Impurities may be removed by the addition of a compound that forms a slag—a substance
with a low melting point that can be readily separated from the molten metal.
3. Refining. The final step in the recovery of a metal is refining the metal. Low boiling metals such as zinc and mercury can be
refined by distillation. When fused on an inclined table, low melting metals like tin flow away from higher-melting impurities.
Electrolysis is another common method for refining metals.

19.1.2.1: Isolation of Iron


The early application of iron to the manufacture of tools and weapons was possible because of the wide distribution of iron ores
and the ease with which iron compounds in the ores could be reduced by carbon. For a long time, charcoal was the form of carbon
used in the reduction process. The production and use of iron became much more widespread about 1620, when coke was
introduced as the reducing agent. Coke is a form of carbon formed by heating coal in the absence of air to remove impurities.
The first step in the metallurgy of iron is usually roasting the ore (heating the ore in air) to remove water, decomposing carbonates
into oxides, and converting sulfides into oxides. The oxides are then reduced in a blast furnace that is 80–100 feet high and about
25 feet in diameter (Figure 19.1.6) in which the roasted ore, coke, and limestone (impure CaCO3) are introduced continuously into
the top. Molten iron and slag are withdrawn at the bottom. The entire stock in a furnace may weigh several hundred tons.

Access for free at OpenStax 19.1.5 https://chem.libretexts.org/@go/page/38321


Figure 19.1.6 :Within a blast furnace, different reactions occur in different temperature zones. Carbon monoxide is generated in the
hotter bottom regions and rises upward to reduce the iron oxides to pure iron through a series of reactions that take place in the
upper regions.
At the right side of the figure, furnace heights are labeled in order of increasing height between the outlet pipes, followed by
temperatures and associated chemical reactions. Just above the pipe labeled, “Outlet,” no chemical equation at 5 f t, 1510 degrees
C. At 15 f t, 1300 degrees C is the eC plus O subscript 2 right pointing arrow C O subscript 2. At 25 f t, 1125 degrees C are the two
reactions, C a O plus S i O subscript 2 right pointing arrow C a S i O subscript 3 and C plus C O subscript 2 right pointing arrow 2
C O. At 35 f t, 945 degrees C, are the two reactions C a C O subscript 3 right pointing arrow C a O plus C O subscript 2, and C plus
C O subscript 2 right pointing arrow 2 C O. At 45 f t, 865 degrees C is C plus C O subscript 2 right pointing arrow 2 C O. At 55 f t,
525 degrees C is the equation F e O plus C O right pointing arrow F e plus C O subscript 2. At 65 f t, 410 degrees C, is F e
subscript 3 O subscript 4 plus C O right pointing arrow 3 F e O plus C O subscript 2. At 75 f t, 230 degrees C, is the equation, 3 F e
subscript 2 O subscript 3 plus C O right pointing arrow 2 F e subscript 3 O subscript 4 plus C O subscript 2.

Near the bottom of a furnace are nozzles through which preheated air is blown into the furnace. As soon as the air enters, the coke
in the region of the nozzles is oxidized to carbon dioxide with the liberation of a great deal of heat. The hot carbon dioxide passes
upward through the overlying layer of white-hot coke, where it is reduced to carbon monoxide:

CO (g) + C(s) ⟶ 2 CO(g)


2

The carbon monoxide serves as the reducing agent in the upper regions of the furnace. The individual reactions are indicated in
Figure 19.1.6.
The iron oxides are reduced in the upper region of the furnace. In the middle region, limestone (calcium carbonate) decomposes,
and the resulting calcium oxide combines with silica and silicates in the ore to form slag. The slag is mostly calcium silicate and
contains most of the commercially unimportant components of the ore:

CaO(s) + SiO (s) ⟶ CaSiO (l)


2 3

Just below the middle of the furnace, the temperature is high enough to melt both the iron and the slag. They collect in layers at the
bottom of the furnace; the less dense slag floats on the iron and protects it from oxidation. Several times a day, the slag and molten
iron are withdrawn from the furnace. The iron is transferred to casting machines or to a steelmaking plant (Figure 19.1.7).

Access for free at OpenStax 19.1.6 https://chem.libretexts.org/@go/page/38321


Figure 19.1.7 :Molten iron is shown being cast as steel. (credit: Clint Budd)
Much of the iron produced is refined and converted into steel. Steel is made from iron by removing impurities and adding
substances such as manganese, chromium, nickel, tungsten, molybdenum, and vanadium to produce alloys with properties that
make the material suitable for specific uses. Most steels also contain small but definite percentages of carbon (0.04%–2.5%).
However, a large part of the carbon contained in iron must be removed in the manufacture of steel; otherwise, the excess carbon
would make the iron brittle.

19.1.2.2: Isolation of Copper


The most important ores of copper contain copper sulfides (such as covellite, CuS), although copper oxides (such as tenorite, CuO)
and copper hydroxycarbonates [such as malachite, Cu2(OH)2CO3] are sometimes found. In the production of copper metal, the
concentrated sulfide ore is roasted to remove part of the sulfur as sulfur dioxide. The remaining mixture, which consists of Cu2S,
FeS, FeO, and SiO2, is mixed with limestone, which serves as a flux (a material that aids in the removal of impurities), and heated.
Molten slag forms as the iron and silica are removed by Lewis acid-base reactions:

CaCO (s) + SiO (s) ⟶ CaSiO (l) + CO (g)


3 2 3 2

FeO(s) + SiO (s) ⟶ FeSiO (l)


2 3

In these reactions, the silicon dioxide behaves as a Lewis acid, which accepts a pair of electrons from the Lewis base (the oxide
ion).
Reduction of the Cu2S that remains after smelting is accomplished by blowing air through the molten material. The air converts
part of the Cu2S into Cu2O. As soon as copper(I) oxide is formed, it is reduced by the remaining copper(I) sulfide to metallic
copper:

2 Cu S(l) + 3 O (g) ⟶ 2 Cu O(l) + 2 SO (g)


2 2 2 2

2 Cu O(l) + Cu S(l) ⟶ 6 Cu(l) + SO (g)


2 2 2

The copper obtained in this way is called blister copper because of its characteristic appearance, which is due to the air blisters it
contains (Figure 19.1.8). This impure copper is cast into large plates, which are used as anodes in the electrolytic refining of the
metal (which is described in the chapter on electrochemistry).

Access for free at OpenStax 19.1.7 https://chem.libretexts.org/@go/page/38321


Figure 19.1.8 : Blister copper is obtained during the conversion of copper-containing ore into pure copper. (credit: “Tortie
tude”/Wikimedia Commons)
This figure shows a dull, black, lumpy mass with small, metallic flecks displayed on a clear, colorless rectangular solid base.

19.1.2.3: Isolation of Silver


Silver sometimes occurs in large nuggets (Figure 19.1.9) but more frequently in veins and related deposits. At one time, panning
was an effective method of isolating both silver and gold nuggets. Due to their low reactivity, these metals, and a few others, occur
in deposits as nuggets. The discovery of platinum was due to Spanish explorers in Central America mistaking platinum nuggets for
silver. When the metal is not in the form of nuggets, it often useful to employ a process called hydrometallurgy to separate silver
from its ores.

Figure 19.1.9 : Naturally occurring free silver may be found as nuggets (a) or in veins (b). (credit a: modification of work by
“Teravolt”/Wikimedia Commons; credit b: modification of work by James St. John)
A. A small clump of bronze-colored metal with a very rough, irregular surface. B. A layer-like region of silver metal embedded in
rock.
Hydrology involves the separation of a metal from a mixture by first converting it into soluble ions and then extracting and
reducing them to precipitate the pure metal. In the presence of air, alkali metal cyanides readily form the soluble
dicyanoargentate(I) ion, [Ag(CN) ] , from silver metal or silver-containing compounds such as Ag2S and AgCl. Representative
2

equations are:
− − −
4 Ag(s) + 8 CN (aq) + O (g) + 2 H O(l) ⟶ 4 [Ag (CN) ] (aq) + 4 OH (aq)
2 2 2

− − −
2 Ag S(s) + 8 CN (aq) + O (g) + 2 H O(l) ⟶ 4 [Ag (CN) ] (aq) + 2 S(s) + 4 OH (aq)
2 2 2 2

− − −
AgCl(s) + 2 CN (aq) ⟶ [Ag (CN) ] (aq) + Cl (aq)
2

The silver is precipitated from the cyanide solution by the addition of either zinc or iron(II) ions, which serves as the reducing
agent:
− 2−
2 [Ag (CN) ] (aq) + Zn(s) ⟶ 2 Ag(s) + [Zn(CN) ] (aq)
2 4

 Example 19.1.3: Refining Redox

One of the steps for refining silver involves converting silver into dicyanoargenate(I) ions:
− − −
4 Ag(s) + 8 CN (aq) + O (g) + 2 H O(l) ⟶ 4 [Ag (CN) ] (aq) + 4 OH (aq)
2 2 2

Explain why oxygen must be present to carry out the reaction. Why does the reaction not occur as:

Access for free at OpenStax 19.1.8 https://chem.libretexts.org/@go/page/38321


− −
4 Ag(s) + 8 CN (aq) ⟶ 4 [Ag (CN) ] (aq)?
2

Solution
The charges, as well as the atoms, must balance in reactions. The silver atom is being oxidized from the 0 oxidation state to the
1+ state. Whenever something loses electrons, something must also gain electrons (be reduced) to balance the equation.
Oxygen is a good oxidizing agent for these reactions because it can gain electrons to go from the 0 oxidation state to the 2−
state.

 Exercise 19.1.3

During the refining of iron, carbon must be present in the blast furnace. Why is carbon necessary to convert iron oxide into
iron?

Answer
The carbon is converted into CO, which is the reducing agent that accepts electrons so that iron(III) can be reduced to
iron(0).

19.1.3: Transition Metal Compounds


The bonding in the simple compounds of the transition elements ranges from ionic to covalent. In their lower oxidation states, the
transition elements form ionic compounds; in their higher oxidation states, they form covalent compounds or polyatomic ions. The
variation in oxidation states exhibited by the transition elements gives these compounds a metal-based, oxidation-reduction
chemistry. The chemistry of several classes of compounds containing elements of the transition series follows.

19.1.3.1: Halides
Anhydrous halides of each of the transition elements can be prepared by the direct reaction of the metal with halogens. For
example:

2 Fe(s) + 3 Cl (g) ⟶ 2 FeCl (s)


2 3

Heating a metal halide with additional metal can be used to form a halide of the metal with a lower oxidation state:

Fe(s) + 2 FeCl (s) ⟶ 3 FeCl (s)


3 2

The stoichiometry of the metal halide that results from the reaction of the metal with a halogen is determined by the relative
amounts of metal and halogen and by the strength of the halogen as an oxidizing agent. Generally, fluorine forms fluoride-
containing metals in their highest oxidation states. The other halogens may not form analogous compounds.
In general, the preparation of stable water solutions of the halides of the metals of the first transition series is by the addition of a
hydrohalic acid to carbonates, hydroxides, oxides, or other compounds that contain basic anions. Sample reactions are:
NiCO (s) + 2 HF(aq) ⟶ NiF (aq) + H O(l) + CO (g)
3 2 2 2

Co (OH) (s) + 2 HBr(aq) ⟶ CoBr (aq) + 2 H O(l)


2 2 2

Most of the first transition series metals also dissolve in acids, forming a solution of the salt and hydrogen gas. For example:

Cr(s) + 2 HCl(aq) ⟶ CrCl (aq) + H (g)


2 2

The polarity of bonds with transition metals varies based not only upon the electronegativities of the atoms involved but also upon
the oxidation state of the transition metal. Remember that bond polarity is a continuous spectrum with electrons being shared
evenly (covalent bonds) at one extreme and electrons being transferred completely (ionic bonds) at the other. No bond is ever 100%
ionic, and the degree to which the electrons are evenly distributed determines many properties of the compound. Transition metal
halides with low oxidation numbers form more ionic bonds. For example, titanium(II) chloride and titanium(III) chloride (TiCl2
and TiCl3) have high melting points that are characteristic of ionic compounds, but titanium(IV) chloride (TiCl4) is a volatile liquid,

Access for free at OpenStax 19.1.9 https://chem.libretexts.org/@go/page/38321


consistent with having covalent titanium-chlorine bonds. All halides of the heavier d-block elements have significant covalent
characteristics.
The covalent behavior of the transition metals with higher oxidation states is exemplified by the reaction of the metal tetrahalides
with water. Like covalent silicon tetrachloride, both the titanium and vanadium tetrahalides react with water to give solutions
containing the corresponding hydrohalic acids and the metal oxides:

SiCl (l) + 2 H O(l) ⟶ SiO (s) + 4 HCl(aq)


4 2 2

TiCl (l) + 2 H O(l) ⟶ TiO (s) + 4 HCl(aq)


4 2 2

19.1.3.2: Oxides
As with the halides, the nature of bonding in oxides of the transition elements is determined by the oxidation state of the metal.
Oxides with low oxidation states tend to be more ionic, whereas those with higher oxidation states are more covalent. These
variations in bonding are because the electronegativities of the elements are not fixed values. The electronegativity of an element
increases with increasing oxidation state. Transition metals in low oxidation states have lower electronegativity values than oxygen;
therefore, these metal oxides are ionic. Transition metals in very high oxidation states have electronegativity values close to that of
oxygen, which leads to these oxides being covalent.
The oxides of the first transition series can be prepared by heating the metals in air. These oxides are Sc2O3, TiO2, V2O5, Cr2O3,
Mn3O4, Fe3O4, Co3O4, NiO, and CuO.
Alternatively, these oxides and other oxides (with the metals in different oxidation states) can be produced by heating the
corresponding hydroxides, carbonates, or oxalates in an inert atmosphere. Iron(II) oxide can be prepared by heating iron(II)
oxalate, and cobalt(II) oxide is produced by heating cobalt(II) hydroxide:

FeC O (s) ⟶ FeO(s) + CO(g) + CO (g)


2 4 2

Co (OH) (s) ⟶ CoO(s) + H O(g)


2 2

With the exception of CrO3 and Mn2O7, transition metal oxides are not soluble in water. They can react with acids and, in a few
cases, with bases. Overall, oxides of transition metals with the lowest oxidation states are basic (and react with acids), the
intermediate ones are amphoteric, and the highest oxidation states are primarily acidic. Basic metal oxides at a low oxidation state
react with aqueous acids to form solutions of salts and water. Examples include the reaction of cobalt(II) oxide accepting protons
from nitric acid, and scandium(III) oxide accepting protons from hydrochloric acid:

CoO(s) + 2 HNO (aq) ⟶ Co (NO ) (aq) + H O(l)


3 3 2 2

Sc O (s) + 6 HCl(aq) ⟶ 2 ScCl (aq) + 3 H O(l)


2 3 3 2

The oxides of metals with oxidation states of 4+ are amphoteric, and most are not soluble in either acids or bases. Vanadium(V)
oxide, chromium(VI) oxide, and manganese(VII) oxide are acidic. They react with solutions of hydroxides to form salts of the
oxyanions VO , CrO , and MnO . For example, the complete ionic equation for the reaction of chromium(VI) oxide with a
3−
4
2−
4

4

strong base is given by:


+ − + 2−
CrO (s) + 2 Na (aq) + 2 OH (aq) ⟶ 2 Na (aq) + CrO (aq) + H O(l)
3 4 2

Chromium(VI) oxide and manganese(VII) oxide react with water to form the acids H2CrO4 and HMnO4, respectively.

19.1.3.3: Hydroxides
When a soluble hydroxide is added to an aqueous solution of a salt of a transition metal of the first transition series, a gelatinous
precipitate forms. For example, adding a solution of sodium hydroxide to a solution of cobalt sulfate produces a gelatinous pink or
blue precipitate of cobalt(II) hydroxide. The net ionic equation is:
2+ −
Co (aq) + 2 OH (aq) ⟶ Co (OH) (s)
2

In this and many other cases, these precipitates are hydroxides containing the transition metal ion, hydroxide ions, and water
coordinated to the transition metal. In other cases, the precipitates are hydrated oxides composed of the metal ion, oxide ions, and
water of hydration:

Access for free at OpenStax 19.1.10 https://chem.libretexts.org/@go/page/38321


3+ −
4 Fe (aq) + 6 OH (aq) + nH O(l) ⟶ 2 Fe O ⋅ (n + 3 )H O(s)
2 2 3 2

These substances do not contain hydroxide ions. However, both the hydroxides and the hydrated oxides react with acids to form
salts and water. When precipitating a metal from solution, it is necessary to avoid an excess of hydroxide ion, as this may lead to
complex ion formation as discussed later in this chapter. The precipitated metal hydroxides can be separated for further processing
or for waste disposal.

19.1.3.4: Carbonates
Many of the elements of the first transition series form insoluble carbonates. It is possible to prepare these carbonates by the
addition of a soluble carbonate salt to a solution of a transition metal salt. For example, nickel carbonate can be prepared from
solutions of nickel nitrate and sodium carbonate according to the following net ionic equation:
2+ 2−
Ni (aq) + CO ⟶ NiCO (s)
3 3

The reactions of the transition metal carbonates are similar to those of the active metal carbonates. They react with acids to form
metals salts, carbon dioxide, and water. Upon heating, they decompose, forming the transition metal oxides.

19.1.3.5: Other Salts


In many respects, the chemical behavior of the elements of the first transition series is very similar to that of the main group metals.
In particular, the same types of reactions that are used to prepare salts of the main group metals can be used to prepare simple ionic
salts of these elements.
A variety of salts can be prepared from metals that are more active than hydrogen by reaction with the corresponding acids:
Scandium metal reacts with hydrobromic acid to form a solution of scandium bromide:

2 Sc(s) + 6 HBr(aq) ⟶ 2 ScBr (aq) + 3 H (g)


3 2

The common compounds that we have just discussed can also be used to prepare salts. The reactions involved include the reactions
of oxides, hydroxides, or carbonates with acids. For example:
+ − 2+ −
Ni (OH) (s) + 2 H O (aq) + 2 ClO (aq) ⟶ Ni (aq) + 2 ClO (aq) + 4 H O(l)
2 3 4 4 2

Substitution reactions involving soluble salts may be used to prepare insoluble salts. For example:
2+ − + 2− + −
Ba (aq) + 2 Cl (aq) + 2 K (aq) + CrO (aq) ⟶ BaCrO (s) + 2 K (aq) + 2 Cl (aq)
4 4

In our discussion of oxides in this section, we have seen that reactions of the covalent oxides of the transition elements with
hydroxides form salts that contain oxyanions of the transition elements.

19.1.4: High Temperature Superconductors


A superconductor is a substance that conducts electricity with no resistance. This lack of resistance means that there is no energy
loss during the transmission of electricity. This would lead to a significant reduction in the cost of electricity.
Most currently used, commercial superconducting materials, such as NbTi and Nb3Sn, do not become superconducting until they
are cooled below 23 K (−250 °C). This requires the use of liquid helium, which has a boiling temperature of 4 K and is expensive
and difficult to handle. The cost of liquid helium has deterred the widespread application of superconductors.
One of the most exciting scientific discoveries of the 1980s was the characterization of compounds that exhibit superconductivity at
temperatures above 90 K. (Compared to liquid helium, 90 K is a high temperature.) Typical among the high-temperature
superconducting materials are oxides containing yttrium (or one of several rare earth elements), barium, and copper in a 1:2:3 ratio.
The formula of the ionic yttrium compound is YBa2Cu3O7.
The new materials become superconducting at temperatures close to 90 K (Figure 19.1.10), temperatures that can be reached by
cooling with liquid nitrogen (boiling temperature of 77 K). Not only are liquid nitrogen-cooled materials easier to handle, but the
cooling costs are also about 1000 times lower than for liquid helium.

Access for free at OpenStax 19.1.11 https://chem.libretexts.org/@go/page/38321


Figure 19.1.10: The resistance of the high-temperature superconductor YBa2Cu3O7 varies with temperature. Note how the
resistance falls to zero below 92 K, when the substance becomes superconducting.
A graph of resistance (ohms) against temperature (Kelvin) is shown. The line remains constant at 0 ohms up until 92 Kelvin in
which there is a sharp increase in resistance which continues increasing with a low slope.
Although the brittle, fragile nature of these materials presently hampers their commercial applications, they have tremendous
potential that researchers are hard at work improving their processes to help realize. Superconducting transmission lines would
carry current for hundreds of miles with no loss of power due to resistance in the wires. This could allow generating stations to be
located in areas remote from population centers and near the natural resources necessary for power production. The first project
demonstrating the viability of high-temperature superconductor power transmission was established in New York in 2008.

Figure 19.1.11: (a) This magnetic levitation train (or maglev) uses superconductor technology to move along its tracks. (b) A
magnet can be levitated using a dish like this as a superconductor. (credit a: modification of work by Alex Needham; credit b:
modification of work by Kevin Jarrett)
A. a white train is on a track with a building standing on a bed of water in the background. B. A magnet is floating in a dish with
frost emerging from the dish.
Researchers are also working on using this technology to develop other applications, such as smaller and more powerful
microchips. In addition, high-temperature superconductors can be used to generate magnetic fields for applications such as medical
devices, magnetic levitation trains, and containment fields for nuclear fusion reactors (Figure 19.1.11).

Levitating Superconductor on a Möbius …

Video 19.1.1 : Watch how a high-temperature superconductor levitates around a magnetic racetrack in the video.

Access for free at OpenStax 19.1.12 https://chem.libretexts.org/@go/page/38321


Summary
The transition metals are elements with partially filled d orbitals, located in the d-block of the periodic table. The reactivity of the
transition elements varies widely from very active metals such as scandium and iron to almost inert elements, such as the platinum
metals. The type of chemistry used in the isolation of the elements from their ores depends upon the concentration of the element in
its ore and the difficulty of reducing ions of the elements to the metals. Metals that are more active are more difficult to reduce.
Transition metals exhibit chemical behavior typical of metals. For example, they oxidize in air upon heating and react with
elemental halogens to form halides. Those elements that lie above hydrogen in the activity series react with acids, producing salts
and hydrogen gas. Oxides, hydroxides, and carbonates of transition metal compounds in low oxidation states are basic. Halides and
other salts are generally stable in water, although oxygen must be excluded in some cases. Most transition metals form a variety of
stable oxidation states, allowing them to demonstrate a wide range of chemical reactivity.

Glossary
actinide series
(also, actinoid series) actinium and the elements in the second row or the f-block, atomic numbers 89–103

coordination compound
stable compound in which the central metal atom or ion acts as a Lewis acid and accepts one or more pairs of electrons

d-block element
one of the elements in groups 3–11 with valence electrons in d orbitals

f-block element
(also, inner transition element) one of the elements with atomic numbers 58–71 or 90–103 that have valence electrons in f
orbitals; they are frequently shown offset below the periodic table

first transition series


transition elements in the fourth period of the periodic table (first row of the d-block), atomic numbers 21–29

fourth transition series


transition elements in the seventh period of the periodic table (fourth row of the d-block), atomic numbers 89 and 104–111

hydrometallurgy
process in which a metal is separated from a mixture by first converting it into soluble ions, extracting the ions, and then
reducing the ions to precipitate the pure metal

lanthanide series
(also, lanthanoid series) lanthanum and the elements in the first row or the f-block, atomic numbers 57–71

platinum metals
group of six transition metals consisting of ruthenium, osmium, rhodium, iridium, palladium, and platinum that tend to occur in
the same minerals and demonstrate similar chemical properties

rare earth element


collection of 17 elements including the lanthanides, scandium, and yttrium that often occur together and have similar chemical
properties, making separation difficult

second transition series


transition elements in the fifth period of the periodic table (second row of the d-block), atomic numbers 39–47

smelting
process of extracting a pure metal from a molten ore

steel

Access for free at OpenStax 19.1.13 https://chem.libretexts.org/@go/page/38321


material made from iron by removing impurities in the iron and adding substances that produce alloys with properties suitable
for specific uses

superconductor
material that conducts electricity with no resistance
third transition series
transition elements in the sixth period of the periodic table (third row of the d-block), atomic numbers 57 and 72–79

This page titled 19.1: Properties of Transition Metals and Their Compounds is shared under a CC BY 4.0 license and was authored, remixed,
and/or curated by OpenStax via source content that was edited to the style and standards of the LibreTexts platform; a detailed edit history is
available upon request.

Access for free at OpenStax 19.1.14 https://chem.libretexts.org/@go/page/38321


19.2: Coordination Chemistry of Transition Metals
 Learning Objectives
List the defining traits of coordination compounds
Describe the structures of complexes containing monodentate and polydentate ligands
Use standard nomenclature rules to name coordination compounds
Explain and provide examples of geometric and optical isomerism
Identify several natural and technological occurrences of coordination compounds

The hemoglobin in your blood, the chlorophyll in green plants, vitamin B , and the catalyst used in the manufacture of polyethylene all contain coordination compounds. Ions of the metals,
12

especially the transition metals, are likely to form complexes. Many of these compounds are highly colored (Figure 19.2.1). In the remainder of this chapter, we will consider the structure and
bonding of these remarkable compounds.

Figure 19.2.1 : Metal ions that contain partially filled d subshell usually form colored complex ions; ions with empty d subshell (d0) or with filled d subshells (d10) usually form colorless complexes.
This figure shows, from left to right, solutions containing [M(H2O)6]n+ ions with M = Sc3+(d0), Cr3+(d3), Co2+(d7), Ni2+(d8), Cu2+(d9), and Zn2+(d10). (credit: Sahar Atwa)
There are six containers that are each filled with a different color liquid. The first is clear, followed by purple, red, teal, blue, and the last one is also clear.
Remember that in most main group element compounds, the valence electrons of the isolated atoms combine to form chemical bonds that satisfy the octet rule. For instance, the four valence
electrons of carbon overlap with electrons from four hydrogen atoms to form CH4. The one valence electron leaves sodium and adds to the seven valence electrons of chlorine to form the ionic
formula unit NaCl (Figure 19.2.2). Transition metals do not normally bond in this fashion. They primarily form coordinate covalent bonds, a form of the Lewis acid-base interaction in which both
of the electrons in the bond are contributed by a donor (Lewis base) to an electron acceptor (Lewis acid). The Lewis acid in coordination complexes, often called a central metal ion (or atom), is
often a transition metal or inner transition metal, although main group elements can also form coordination compounds. The Lewis base donors, called ligands, can be a wide variety of chemicals—
atoms, molecules, or ions. The only requirement is that they have one or more electron pairs, which can be donated to the central metal. Most often, this involves a donor atom with a lone pair of
electrons that can form a coordinate bond to the metal.

Figure 19.2.2 : (a) Covalent bonds involve the sharing of electrons, and ionic bonds involve the transferring of electrons associated with each bonding atom, as indicated by the colored electrons. (b)
However, coordinate covalent bonds involve electrons from a Lewis base being donated to a metal center. The lone pairs from six water molecules form bonds to the scandium ion to form an
octahedral complex. (Only the donated pairs are shown.)
The coordination sphere consists of the central metal ion or atom plus its attached ligands. Brackets in a formula enclose the coordination sphere; species outside the brackets are not part of the
coordination sphere. The coordination number of the central metal ion or atom is the number of donor atoms bonded to it. The coordination number for the silver ion in [Ag(NH3)2]+ is two (Figure
2−
19.2.3). For the copper(II) ion in [CuCl4] , the coordination number is four, whereas for the cobalt(II) ion in [Co(H2O)6]2+ the coordination number is six. Each of these ligands is monodentate,
from the Greek for “one toothed,” meaning that they connect with the central metal through only one atom. In this case, the number of ligands and the coordination number are equal.

Figure 19.2.3 : The complexes (a) [Ag(NH3)2]+, (b) [Cu(Cl)4]2−, and (c) [Co(H2O)6]2+ have coordination numbers of two, four, and six, respectively. The geometries of these complexes are the
same as we have seen with VSEPR theory for main group elements: linear, tetrahedral, and octahedral.
The structural formulas for Ag N H 3 subscript 2, C u C l subscript 4, and C o H 2 O subscript 6 is shown in enclosing brackets showing their individual charges on the top right of positive 1,
negative 2 and positive 2 respectively.
Many other ligands coordinate to the metal in more complex fashions. Bidentate ligands are those in which two atoms coordinate to the metal center. For example, ethylenediamine (en,
H2NCH2CH2NH2) contains two nitrogen atoms, each of which has a lone pair and can serve as a Lewis base (Figure 19.2.4). Both of the atoms can coordinate to a single metal center. In the
complex [Co(en)3]3+, there are three bidentate en ligands, and the coordination number of the cobalt(III) ion is six. The most common coordination numbers are two, four, and six, but examples of
all coordination numbers from 1 to 15 are known.

Figure 19.2.4 : (a) The ethylenediamine (en) ligand contains two atoms with lone pairs that can coordinate to the metal center. (b) The cobalt(III) complex [Co(en) ]
3
3+
contains three of these
ligands, each forming two bonds to the cobalt ion.
Any ligand that bonds to a central metal ion by more than one donor atom is a polydentate ligand (or “many teeth”) because it can bite into the metal center with more than one bond. The term
chelate (pronounced “KEY-late”) from the Greek for “claw” is also used to describe this type of interaction. Many polydentate ligands are chelating ligands, and a complex consisting of one or
more of these ligands and a central metal is a chelate. A chelating ligand is also known as a chelating agent. A chelating ligand holds the metal ion rather like a crab’s claw would hold a marble.
Figure 19.2.4 showed one example of a chelate and the heme complex in hemoglobin is another important example (Figure 19.2.5). It contains a polydentate ligand with four donor atoms that
coordinate to iron.

Access for free at OpenStax 19.2.1 https://chem.libretexts.org/@go/page/38322


Figure 19.2.5 : The single ligand heme contains four nitrogen atoms that coordinate to iron in hemoglobin to form a chelate.
Polydentate ligands are sometimes identified with prefixes that indicate the number of donor atoms in the ligand. As we have seen, ligands with one donor atom, such as NH3, Cl−, and H2O, are
monodentate ligands. Ligands with two donor groups are bidentate ligands. Ethylenediamine, H2NCH2CH2NH2, and the anion of the acid glycine, NH CH CO (Figure 19.2.6) are examples of
2 2

2

bidentate ligands. Tridentate ligands, tetradentate ligands, pentadentate ligands, and hexadentate ligands contain three, four, five, and six donor atoms, respectively. The heme ligand (Figure 19.2.5)
is a tetradentate ligand.

Figure 19.2.6 : Each of the anionic ligands shown attaches in a bidentate fashion to platinum(II), with both a nitrogen and oxygen atom coordinating to the metal.

19.2.1: The Naming of Complexes


The nomenclature of the complexes is patterned after a system suggested by Alfred Werner, a Swiss chemist and Nobel laureate, whose outstanding work more than 100 years ago laid the
foundation for a clearer understanding of these compounds. The following five rules are used for naming complexes:
1. If a coordination compound is ionic, name the cation first and the anion second, in accordance with the usual nomenclature.
2. Name the ligands first, followed by the central metal. Name the ligands alphabetically. Negative ligands (anions) have names formed by adding -o to the stem name of the group (e.g., Table
19.2.1. For most neutral ligands, the name of the molecule is used. The four common exceptions are aqua (H2O), amine (NH3), carbonyl (CO), and nitrosyl (NO). For example, name

[Pt(NH3)2Cl4] as diaminetetrachloroplatinum(IV).
3. If more than one ligand of a given type is present, the number is indicated by the prefixes di- (for two), tri- (for three), tetra- (for four), penta- (for five), and hexa- (for six). Sometimes, the
prefixes bis- (for two), tris- (for three), and tetrakis- (for four) are used when the name of the ligand already includes di-, tri-, or tetra-, or when the ligand name begins with a vowel. For
example, the ion bis(bipyridyl)osmium(II) uses bis- to signify that there are two ligands attached to Os, and each bipyridyl ligand contains two pyridine groups (C5H4N).
Table 19.2.1 : Examples of Anionic Ligands
Anionic Ligand Name

F− fluoro

Cl− chloro

Br− bromo

I− iodo

CN− cyano

NO

3
nitrato

OH− hydroxo

O2– oxo

C O
2
2−
4
oxalato

CO
2−
2
carbonato

When the complex is either a cation or a neutral molecule, the name of the central metal atom is spelled exactly like the name of the element and is followed by a Roman numeral in parentheses to
indicate its oxidation state (Tables 19.2.2, 19.2.3, and 19.2.3). When the complex is an anion, the suffix -ate is added to the stem of the name of the metal, followed by the Roman numeral
designation of its oxidation state.
Table 19.2.2 : Select Coordination Complexes based on total Charge
Examples in Which the Complex Is Cation

[Co(NH3)6]Cl3 hexaaminecobalt(III) chloride

[Pt(NH3)4Cl2]2+ tetraaminedichloroplatinum(IV) ion

[Ag(NH3)2]+ diaminesilver(I) ion

[Cr(H2O)4Cl2]Cl tetraaquadichlorochromium(III) chloride

[Co(H2NCH2CH2NH2)3]2(SO4)3 tris(ethylenediamine)cobalt(III) sulfate

Examples in Which the Complex Is Neutral

[Pt(NH3)2Cl4] diaminetetrachloroplatinum(IV)

[Ni(H2NCH2CH2NH2)2Cl2] dichlorobis(ethylenediamine)nickel(II)

Examples in Which the Complex Is an Anion

[PtCl6]2− hexachloroplatinate(IV) ion

Na2[SnCl6] sodium hexachlorostannate(IV)

Sometimes, the Latin name of the metal is used when the English name is clumsy. For example, ferrate is used instead of ironate, plumbate instead leadate, and stannate instead of tinate. The
oxidation state of the metal is determined based on the charges of each ligand and the overall charge of the coordination compound. For example, in [Cr(H2O)4Cl2]Br, the coordination sphere (in
brackets) has a charge of 1+ to balance the bromide ion. The water ligands are neutral, and the chloride ligands are anionic with a charge of 1− each. To determine the oxidation state of the metal,
we set the overall charge equal to the sum of the ligands and the metal: +1 = −2 + x, so the oxidation state (x) is equal to 3+.

Access for free at OpenStax 19.2.2 https://chem.libretexts.org/@go/page/38322


 Example 19.2.1: Coordination Numbers and Oxidation States
Determine the name of the following complexes and give the coordination number of the central metal atom.
a. Na2[PtCl6]
b. K3[Fe(C2O4)3]
c. [Co(NH3)5Cl]Cl2

Solution
a. There are two Na+ ions, so the coordination sphere has a negative two charge: [PtCl6]2−. There are six anionic chloride ligands, so −2 = −6 + x, and the oxidation state of the platinum is 4+.
The name of the complex is sodium hexachloroplatinate(IV), and the coordination number is six.
b. The coordination sphere has a charge of 3− (based on the potassium) and the oxalate ligands each have a charge of 2−, so the metal oxidation state is given by −3 = −6 + x, and this is an
iron(III) complex. The name is potassium trisoxalatoferrate(III) (note that tris is used instead of tri because the ligand name starts with a vowel). Because oxalate is a bidentate ligand, this
complex has a coordination number of six.
c. In this example, the coordination sphere has a cationic charge of 2+. The NH3 ligand is neutral, but the chloro ligand has a charge of 1−. The oxidation state is found by +2 = −1 + x and is
3+, so the complex is pentaaminechlorocobalt(III) chloride and the coordination number is six.

 Exercise 19.2.1

The complex potassium dicyanoargenate(I) is used to make antiseptic compounds. Give the formula and coordination number.

Answer
K[Ag(CN)2]; coordination number two

19.2.2: The Structures of Complexes


The most common structures of the complexes in coordination compounds are octahedral, tetrahedral, and square planar (Figure 19.2.7). For transition metal complexes, the coordination number
determines the geometry around the central metal ion. Table 19.2.3 compares coordination numbers to the molecular geometry:

Figure 19.2.7 : These are geometries of some complexes with coordination numbers of seven and eight.
Table 19.2.3 : Coordination Numbers and Molecular Geometry
Coordination Number Molecular Geometry Example

2 linear [Ag(NH3)2]+

3 trigonal planar [Cu(CN)3]2−

4 tetrahedral(d0 or d10), low oxidation states for M [Ni(CO)4]

4 square planar (d8) [NiCl4]2−

5 trigonal bipyramidal [CoCl5]2−

5 square pyramidal [VO(CN)4]2−

6 octahedral [CoCl6]3−

7 pentagonal bipyramid [ZrF7]3−

8 square antiprism [ReF8]2−

8 dodecahedron [Mo(CN)8]4−

9 and above more complicated structures [ReH9]2−

Unlike main group atoms in which both the bonding and nonbonding electrons determine the molecular shape, the nonbonding d-electrons do not change the arrangement of the ligands. Octahedral
complexes have a coordination number of six, and the six donor atoms are arranged at the corners of an octahedron around the central metal ion. Examples are shown in Figure 19.2.8. The chloride
and nitrate anions in [Co(H2O)6]Cl2 and [Cr(en)3](NO3)3, and the potassium cations in K2[PtCl6], are outside the brackets and are not bonded to the metal ion.

Figure 19.2.8 : Many transition metal complexes adopt octahedral geometries, with six donor atoms forming bond angles of 90° about the central atom with adjacent ligands. Note that only ligands
within the coordination sphere affect the geometry around the metal center.
The structural formulas for C o H 2 O subscript 6, C r e n subscript 3, and P t c l subscript 6 is shown in enclosing brackets showing their individual charges on the top right. These charges are
positive 2, positive 3 and negative 2 respectively. 2 chloride ions, 3 nitrate ions, and 2 potassium ions are shown outside of the three brackets respectively.
For transition metals with a coordination number of four, two different geometries are possible: tetrahedral or square planar. Unlike main group elements, where these geometries can be predicted
from VSEPR theory, a more detailed discussion of transition metal orbitals (discussed in the section on Crystal Field Theory) is required to predict which complexes will be tetrahedral and which
will be square planar. In tetrahedral complexes such as [Zn(CN)4]2− (Figure 19.2.9), each of the ligand pairs forms an angle of 109.5°. In square planar complexes, such as [Pt(NH3)2Cl2], each
ligand has two other ligands at 90° angles (called the cis positions) and one additional ligand at an 180° angle, in the trans position.

Access for free at OpenStax 19.2.3 https://chem.libretexts.org/@go/page/38322


Figure 19.2.9 : Transition metals with a coordination number of four can adopt a tetrahedral geometry (a) as in K2[Zn(CN)4] or a square planar geometry (b) as shown in [Pt(NH3)2Cl2].
Structural formulas of K subscript 2 Z n C N subscript 4 and P t N H subscript 3 subscript 2 C l subscript 2 are shown with wedge and dashed lines to show a more three dimensional structure.

19.2.3: Isomerism in Complexes


Isomers are different chemical species that have the same chemical formula. Transition metals often form geometric isomers, in which the same atoms are connected through the same types of
bonds but with differences in their orientation in space. Coordination complexes with two different ligands in the cis and trans positions from a ligand of interest form isomers. For example, the
octahedral [Co(NH3)4Cl2]+ ion has two isomers. In the cis configuration, the two chloride ligands are adjacent to each other (Figure 19.2.1). The other isomer, the trans configuration, has the two
chloride ligands directly across from one another.

Figure 19.2.10: The cis and trans isomers of [Co(H2O)4Cl2]+ contain the same ligands attached to the same metal ion, but the spatial arrangement causes these two compounds to have very different
properties.
For the Violet, cis form the two chlorine is adjacent to one another. For the green, trans form, the two chlorine is opposite from one another.
Different geometric isomers of a substance are different chemical compounds. They exhibit different properties, even though they have the same formula. For example, the two isomers of
[Co(NH3)4Cl2]NO3 differ in color; the cis form is violet, and the trans form is green. Furthermore, these isomers have different dipole moments, solubilities, and reactivities. As an example of how
the arrangement in space can influence the molecular properties, consider the polarity of the two [Co(NH3)4Cl2]NO3 isomers. Remember that the polarity of a molecule or ion is determined by the
bond dipoles (which are due to the difference in electronegativity of the bonding atoms) and their arrangement in space. In one isomer, cis chloride ligands cause more electron density on one side
of the molecule than on the other, making it polar. For the trans isomer, each ligand is directly across from an identical ligand, so the bond dipoles cancel out, and the molecule is nonpolar.

 Example 19.2.2: Geometric Isomers

Identify which geometric isomer of [Pt(NH3)2Cl2] is shown in Figure 19.2.9b. Draw the other geometric isomer and give its full name.

Solution
In the Figure 19.2.9b, the two chlorine ligands occupy cis positions. The other form is shown in below. When naming specific isomers, the descriptor is listed in front of the name. Therefore,
this complex is trans-diaminedichloroplatinum(II).

Structural formula of trans-diaminedichloroplatinum(II). The chlorine atoms are opposite each other in the same plane. This is also the case for the two amines.
The trans isomer of [Pt(NH3)2Cl2] has each ligand directly across from an adjacent ligand.

 Exercise 19.2.2

Draw the ion trans-diaqua-trans-dibromo-trans-dichlorocobalt(II).

Answer

Cobalt is located at the center with the left and right wedges connected to bromine and H 2 O respectively. The left and right dash lines are connected to H 2 O and bromine respectively.
Pointing directly upwards and downwards opposite from one another are the chlorine. The entire structural formula is enclosed in square bracket with a superscript of 2 negative.

Another important type of isomers are optical isomers, or enantiomers, in which two objects are exact mirror images of each other but cannot be lined up so that all parts match. This means that
optical isomers are nonsuperimposable mirror images. A classic example of this is a pair of hands, in which the right and left hand are mirror images of one another but cannot be superimposed.
Optical isomers are very important in organic and biochemistry because living systems often incorporate one specific optical isomer and not the other. Unlike geometric isomers, pairs of optical
isomers have identical properties (boiling point, polarity, solubility, etc.). Optical isomers differ only in the way they affect polarized light and how they react with other optical isomers. For
coordination complexes, many coordination compounds such as [M(en)3]n+ [in which Mn+ is a central metal ion such as iron(III) or cobalt(II)] form enantiomers, as shown in Figure 19.2.11. These
two isomers will react differently with other optical isomers. For example, DNA helices are optical isomers, and the form that occurs in nature (right-handed DNA) will bind to only one isomer of
[M(en)3]n+ and not the other.

Figure 19.2.11: The complex [M(en)3]n+ (Mn+ = a metal ion, en = ethylenediamine) has a nonsuperimposable mirror image.
Two structural formulas of open bracket M e n subscript 3 close bracket superscript n positive are shown as mirror images with the dashed vertical line in the center as the mirror plane.
The [Co(en)2Cl2]+ ion exhibits geometric isomerism (cis/trans), and its cis isomer exists as a pair of optical isomers (Figure 19.2.12).

Access for free at OpenStax 19.2.4 https://chem.libretexts.org/@go/page/38322


Figure 19.2.12: Three isomeric forms of [Co(en)2Cl2]+ exist. The trans isomer, formed when the chlorines are positioned at a 180° angle, has very different properties from the cis isomers. The
mirror images of the cis isomer form a pair of optical isomers, which have identical behavior except when reacting with other enantiomers.
For the cis isomers, the structural formula shown are mirror images of one another. The two chlorine atoms are adjacent to one another in each cis structure. The trans form is also shown with the
chlorine directly opposite from one another.
Linkage isomers occur when the coordination compound contains a ligand that can bind to the transition metal center through two different atoms. For example, the CN ligand can bind through the
carbon atom (cyano) or through the nitrogen atom (isocyano). Similarly, SCN− can be bound through the sulfur or nitrogen atom, affording two distinct compounds ([Co(NH3)5SCN]2+ or
[Co(NH3)5NCS]2+).
Ionization isomers (or coordination isomers) occur when one anionic ligand in the inner coordination sphere is replaced with the counter ion from the outer coordination sphere. A simple example
of two ionization isomers are [CoCl6][Br] and [CoCl5Br][Cl].

19.2.4: Coordination Complexes in Nature and Technology


Chlorophyll, the green pigment in plants, is a complex that contains magnesium (Figure 19.2.13). This is an example of a main group element in a coordination complex. Plants appear green
because chlorophyll absorbs red and purple light; the reflected light consequently appears green. The energy resulting from the absorption of light is used in photosynthesis.

Figure 19.2.13: (a) Chlorophyll comes in several different forms, which all have the same basic structure around the magnesium center. (b) Copper phthalocyanine blue, a square planar copper
complex, is present in some blue dyes.

19.2.5: Transition Metal Catalysts


One of the most important applications of transition metals is as industrial catalysts. As you recall from the chapter on kinetics, a catalyst increases the rate of reaction by lowering the activation
energy and is regenerated in the catalytic cycle. Over 90% of all manufactured products are made with the aid of one or more catalysts. The ability to bind ligands and change oxidation states makes
transition metal catalysts well suited for catalytic applications. Vanadium oxide is used to produce 230,000,000 tons of sulfuric acid worldwide each year, which in turn is used to make everything
from fertilizers to cans for food. Plastics are made with the aid of transition metal catalysts, along with detergents, fertilizers, paints, and more (Figure 19.2.14). Very complicated pharmaceuticals
are manufactured with catalysts that are selective, reacting with one specific bond out of a large number of possibilities. Catalysts allow processes to be more economical and more environmentally
friendly. Developing new catalysts and better understanding of existing systems are important areas of current research.

Figure 19.2.14: (a) Detergents, (b) paints, and (c) fertilizers are all made using transition metal catalysts. (credit a: modification of work by “Mr. Brian”/Flickr; credit b: modification of work by
Ewen Roberts; credit c: modification of work by “osseous”/Flickr)
Many other coordination complexes are also brightly colored. The square planar copper(II) complex phthalocyanine blue (from Figure 19.2.13) is one of many complexes used as pigments or dyes.
This complex is used in blue ink, blue jeans, and certain blue paints.
The structure of heme (Figure 19.2.15), the iron-containing complex in hemoglobin, is very similar to that in chlorophyll. In hemoglobin, the red heme complex is bonded to a large protein
molecule (globin) by the attachment of the protein to the heme ligand. Oxygen molecules are transported by hemoglobin in the blood by being bound to the iron center. When the hemoglobin loses
its oxygen, the color changes to a bluish red. Hemoglobin will only transport oxygen if the iron is Fe2+; oxidation of the iron to Fe3+ prevents oxygen transport.

Figure 19.2.15: Hemoglobin contains four protein subunits, each of which has an iron center attached to a heme ligand (shown in red), which is coordinated to a globin protein. Each subunit is
shown in a different color.
Three dimensional structure of hemoglobin has many ribbon like structures forming helices and are entangled with one another in a disordered manner. The four subunits are colored differently.
Around the center of hemoglobin are multiple linear structures composed of red globules.
Complexing agents often are used for water softening because they tie up such ions as Ca2+, Mg2+, and Fe2+, which make water hard. Many metal ions are also undesirable in food products because
these ions can catalyze reactions that change the color of food. Coordination complexes are useful as preservatives. For example, the ligand EDTA, (HO2CCH2)2NCH2CH2N(CH2CO2H)2,
coordinates to metal ions through six donor atoms and prevents the metals from reacting (Figure 19.2.16). This ligand also is used to sequester metal ions in paper production, textiles, and
detergents, and has pharmaceutical uses.

Access for free at OpenStax 19.2.5 https://chem.libretexts.org/@go/page/38322


Figure 19.2.16: The ligand EDTA binds tightly to a variety of metal ions by forming hexadentate complexes.
Complexing agents that tie up metal ions are also used as drugs. British Anti-Lewisite (BAL), HSCH2CH(SH)CH2OH, is a drug developed during World War I as an antidote for the arsenic-based
war gas Lewisite. BAL is now used to treat poisoning by heavy metals, such as arsenic, mercury, thallium, and chromium. The drug is a ligand and functions by making a water-soluble chelate of
the metal; the kidneys eliminate this metal chelate (Figure 19.2.17). Another polydentate ligand, enterobactin, which is isolated from certain bacteria, is used to form complexes of iron and thereby
to control the severe iron buildup found in patients suffering from blood diseases such as Cooley’s anemia, who require frequent transfusions. As the transfused blood breaks down, the usual
metabolic processes that remove iron are overloaded, and excess iron can build up to fatal levels. Enterobactin forms a water-soluble complex with excess iron, and the body can safely eliminate
this complex.

Figure 19.2.17: Coordination complexes are used as drugs. (a) British Anti-Lewisite is used to treat heavy metal poisoning by coordinating metals (M), and enterobactin (b) allows excess iron in the
blood to be removed.

 Example 19.2.3: Chelation Therapy

Ligands like BAL and enterobactin are important in medical treatments for heavy metal poisoning. However, chelation therapies can disrupt the normal concentration of ions in the body,
leading to serious side effects, so researchers are searching for new chelation drugs. One drug that has been developed is dimercaptosuccinic acid (DMSA), shown in Figure 19.2.18. Identify
which atoms in this molecule could act as donor atoms.
Structural formula for <span class=
DMSA
is shown with all of the lone pairs on O and S drawn out. " style="width: 650px; height: 191px;" width="650px" height="191px" data-cke-saved-src="/@api/deki/files/65922/
CNX
_Chem_19_02_DMSA.jpg" src="/@api/deki/files/65922/CNX_Chem_19_02_DMSA.jpg" data-quail-id="607">
Figure 19.2.18: Dimercaptosuccinic acid is used to treat heavy metal poisoning.

Solution
All of the oxygen and sulfur atoms have lone pairs of electrons that can be used to coordinate to a metal center, so there are six possible donor atoms. Geometrically, only two of these atoms
can be coordinated to a metal at once. The most common binding mode involves the coordination of one sulfur atom and one oxygen atom, forming a five-member ring with the metal.

 Exercise 19.2.3

Some alternative medicine practitioners recommend chelation treatments for ailments that are not clearly related to heavy metals, such as cancer and autism, although the practice is discouraged
by many scientific organizations.1 Identify at least two biologically important metals that could be disrupted by chelation therapy.

Answer
Ca, Fe, Zn, and Cu

Ligands are also used in the electroplating industry. When metal ions are reduced to produce thin metal coatings, metals can clump together to form clusters and nanoparticles. When metal
coordination complexes are used, the ligands keep the metal atoms isolated from each other. It has been found that many metals plate out as a smoother, more uniform, better-looking, and more
adherent surface when plated from a bath containing the metal as a complex ion. Thus, complexes such as [Ag(CN)2]− and [Au(CN)2]− are used extensively in the electroplating industry.
In 1965, scientists at Michigan State University discovered that there was a platinum complex that inhibited cell division in certain microorganisms. Later work showed that the complex was cis-
diaminedichloroplatinum(II), [Pt(NH3)2(Cl)2], and that the trans isomer was not effective. The inhibition of cell division indicated that this square planar compound could be an anticancer agent. In
1978, the US Food and Drug Administration approved this compound, known as cisplatin, for use in the treatment of certain forms of cancer. Since that time, many similar platinum compounds
have been developed for the treatment of cancer. In all cases, these are the cis isomers and never the trans isomers. The diamine (NH3)2 portion is retained with other groups, replacing the dichloro
[(Cl)2] portion. The newer drugs include carboplatin, oxaliplatin, and satraplatin.

Summary
The transition elements and main group elements can form coordination compounds, or complexes, in which a central metal atom or ion is bonded to one or more ligands by coordinate covalent
bonds. Ligands with more than one donor atom are called polydentate ligands and form chelates. The common geometries found in complexes are tetrahedral and square planar (both with a
coordination number of four) and octahedral (with a coordination number of six). Cis and trans configurations are possible in some octahedral and square planar complexes. In addition to these
geometrical isomers, optical isomers (molecules or ions that are mirror images but not superimposable) are possible in certain octahedral complexes. Coordination complexes have a wide variety of
uses including oxygen transport in blood, water purification, and pharmaceutical use.

Footnotes
1. National Council against Health Fraud, NCAHF Policy Statement on Chelation Therapy, (Peabody, MA, 2002).

Access for free at OpenStax 19.2.6 https://chem.libretexts.org/@go/page/38322


Glossary
bidentate ligand
ligand that coordinates to one central metal through coordinate bonds from two different atoms

central metal
ion or atom to which one or more ligands is attached through coordinate covalent bonds

chelate
complex formed from a polydentate ligand attached to a central metal

chelating ligand
ligand that attaches to a central metal ion by bonds from two or more donor atoms

cis configuration
configuration of a geometrical isomer in which two similar groups are on the same side of an imaginary reference line on the molecule

coordination compound
substance consisting of atoms, molecules, or ions attached to a central atom through Lewis acid-base interactions

coordination number
number of coordinate covalent bonds to the central metal atom in a complex or the number of closest contacts to an atom in a crystalline form

coordination sphere
central metal atom or ion plus the attached ligands of a complex

donor atom
atom in a ligand with a lone pair of electrons that forms a coordinate covalent bond to a central metal

ionization isomer
(or coordination isomer) isomer in which an anionic ligand is replaced by the counter ion in the inner coordination sphere

ligand
ion or neutral molecule attached to the central metal ion in a coordination compound

linkage isomer
coordination compound that possesses a ligand that can bind to the transition metal in two different ways (CN− vs. NC−)

monodentate
ligand that attaches to a central metal through just one coordinate covalent bond

optical isomer
(also, enantiomer) molecule that is a nonsuperimposable mirror image with identical chemical and physical properties, except when it reacts with other optical isomers

polydentate ligand
ligand that is attached to a central metal ion by bonds from two or more donor atoms, named with prefixes specifying how many donors are present (e.g., hexadentate = six coordinate bonds
formed)

trans configuration
configuration of a geometrical isomer in which two similar groups are on opposite sides of an imaginary reference line on the molecule

This page titled 19.2: Coordination Chemistry of Transition Metals is shared under a CC BY 4.0 license and was authored, remixed, and/or curated by OpenStax via source content that was edited to the style and
standards of the LibreTexts platform; a detailed edit history is available upon request.

Access for free at OpenStax 19.2.7 https://chem.libretexts.org/@go/page/38322


Access for free at OpenStax 19.2.8 https://chem.libretexts.org/@go/page/38322
Access for free at OpenStax 19.2.9 https://chem.libretexts.org/@go/page/38322
Access for free at OpenStax 19.2.10 https://chem.libretexts.org/@go/page/38322
Access for free at OpenStax 19.2.11 https://chem.libretexts.org/@go/page/38322
Access for free at OpenStax 19.2.12 https://chem.libretexts.org/@go/page/38322
19.3: Optical and Magnetic Properties of Coordination Compounds
 Learning Objectives
Outline the basic premise of crystal field theory (CFT)
Identify molecular geometries associated with various d-orbital splitting patterns
Predict electron configurations of split d orbitals for selected transition metal atoms or ions
Explain spectral and magnetic properties in terms of CFT concepts

The behavior of coordination compounds cannot be adequately explained by the same theories used for main group element
chemistry. The observed geometries of coordination complexes are not consistent with hybridized orbitals on the central metal
overlapping with ligand orbitals, as would be predicted by valence bond theory. The observed colors indicate that the d orbitals
often occur at different energy levels rather than all being degenerate, that is, of equal energy, as are the three p orbitals. To explain
the stabilities, structures, colors, and magnetic properties of transition metal complexes, a different bonding model has been
developed. Just as valence bond theory explains many aspects of bonding in main group chemistry, crystal field theory is useful in
understanding and predicting the behavior of transition metal complexes.

19.3.1: Crystal Field Theory


To explain the observed behavior of transition metal complexes (such as how colors arise), a model involving electrostatic
interactions between the electrons from the ligands and the electrons in the unhybridized d orbitals of the central metal atom has
been developed. This electrostatic model is crystal field theory (CFT). It allows us to understand, interpret, and predict the colors,
magnetic behavior, and some structures of coordination compounds of transition metals.
CFT focuses on the nonbonding electrons on the central metal ion in coordination complexes not on the metal-ligand bonds. Like
valence bond theory, CFT tells only part of the story of the behavior of complexes. However, it tells the part that valence bond
theory does not. In its pure form, CFT ignores any covalent bonding between ligands and metal ions. Both the ligand and the metal
are treated as infinitesimally small point charges.
All electrons are negative, so the electrons donated from the ligands will repel the electrons of the central metal. Let us consider the
behavior of the electrons in the unhybridized d orbitals in an octahedral complex. The five d orbitals consist of lobe-shaped regions
and are arranged in space, as shown in Figure 19.3.1. In an octahedral complex, the six ligands coordinate along the axes.

Access for free at OpenStax 19.3.1 https://chem.libretexts.org/@go/page/38323


Figure 19.3.1 : The directional characteristics of the five d orbitals are shown here. The shaded portions indicate the phase of the
orbitals. The ligands (L) coordinate along the axes. For clarity, the ligands have been omitted from the d orbital so that the
x2 − y 2

axis labels could be shown.


This figure includes diagrams of five d orbitals. Each diagram includes three axes. The z-axis is vertical and is denoted with an
upward pointing arrow. It is labeled “z” in the first diagram. Arrows similarly identify the x-axis with an arrow pointing from the
rear left to the right front, diagonally across the figure and the y-axis with an arrow pointing from the left front diagonally across
the figure to the right rear of the diagram. These axes are similarly labeled as “x” and “y.” In this first diagram, four orange
balloon-like shapes extend from a point at the origin out along the x- and y- axes in positive and negative directions covering just
over half the length of the positive and negative x- and y- axes. Beneath the diagram is the label, “d subscript ( x superscript 2
minus y superscript 2 ).” The second diagram just right of the first is similar except the x, y, and z labels have been replaced in each
instance with the letter L. Only a pair of the orange balloon-like shapes are present and extend from the origin above and below
along the vertical axis. An orange toroidal or donut shape is positioned around the origin, oriented through the x- and y- axes. This
shape extends out to about a third of the length of the positive and negative regions of the x- and y- axes. This diagram is labeled,
“d subscript ( z superscript 2 ).” The third through fifth diagrams, similar to the first, show four orange balloon-like shapes. These
diagrams differ however in the orientation of the shapes along the axes and the x-, y-, and z-axis labels have each been replaced
with the letter L. Planes are added to the figures to help show the orientation differences with these diagrams. In the third diagram,
a green plane is oriented vertically through the length of the x-axis and a blue plane is oriented horizontally through the length of
the y-axis. The balloon shapes extend from the origin to the spaces between the positive z- and negative y- axes, positive z- and
positive y- axes, negative z- and negative y- axes, and negative z- and positive y- axes. This diagram is labeled, “d subscript ( y z
).” In the fourth diagram, a green plane is oriented vertically through the x- and y- axes and a blue plane is oriented horizontally
through the length of the x-axis. The balloon shapes extend from the origin to the spaces between the positive z- and negative x-
axes, positive z- and positive x- axes, negative z- and negative x- axes, and negative z- and positive x- axes. This diagram is labeled
“d subscript ( x z ).” In the fifth diagram, a pink plane is oriented vertically through the length of the y-axis and a green plane is
oriented vertically through the length of the x-axis. The balloon shapes extend from the origin to the spaces between the positive x-
and negative y- axes, positive x- and positive y- axes, negative x- and negative y- axes, and negative x- and positive y- axes. This
diagram is labeled, “d subscript ( x y ).”
In an uncomplexed metal ion in the gas phase, the electrons are distributed among the five d orbitals in accord with Hund's rule
because the orbitals all have the same energy. However, when ligands coordinate to a metal ion, the energies of the d orbitals are no
longer the same.
In octahedral complexes, the lobes in two of the five d orbitals, the d and d z
2 2orbitals, point toward the ligands (Figure
x −y
2

19.3.1). These two orbitals are called the eg orbitals (the symbol actually refers to the symmetry of the orbitals, but we will use it as

a convenient name for these two orbitals in an octahedral complex). The other three orbitals, the dxy, dxz, and dyz orbitals, have
lobes that point between the ligands and are called the t2g orbitals (again, the symbol really refers to the symmetry of the orbitals).
As six ligands approach the metal ion along the axes of the octahedron, their point charges repel the electrons in the d orbitals of
the metal ion. However, the repulsions between the electrons in the eg orbitals (the d and d z
2 orbitals) and the ligands are
2
x −y
2

greater than the repulsions between the electrons in the t2g orbitals (the dzy, dxz, and dyz orbitals) and the ligands. This is because the
lobes of the eg orbitals point directly at the ligands, whereas the lobes of the t2g orbitals point between them. Thus, electrons in the
eg orbitals of the metal ion in an octahedral complex have higher potential energies than those of electrons in the t2g orbitals. The
difference in energy may be represented as shown in Figure 19.3.2.

Access for free at OpenStax 19.3.2 https://chem.libretexts.org/@go/page/38323


Figure 19.3.2 : In octahedral complexes, the eg orbitals are destabilized (higher in energy) compared to the t2g orbitals because the
ligands interact more strongly with the d orbitals at which they are pointed directly.
The difference in energy between the eg and the t2g orbitals is called the crystal field splitting and is symbolized by Δoct, where oct
stands for octahedral.
The magnitude of Δoct depends on many factors, including the nature of the six ligands located around the central metal ion, the
charge on the metal, and whether the metal is using 3d, 4d, or 5d orbitals. Different ligands produce different crystal field splittings.
The increasing crystal field splitting produced by ligands is expressed in the spectrochemical series, a short version of which is
given here:
− − − − 2− − −
I < Br < Cl <F < H O< C O < NH <en< NO < CN
2 2 4 3 2

−−−−−−−−−−−−−−−−−−−−−−−−−−−−−−−−−−−−−−−−−−−−−−−−→
a few ligands of the spectrochemical series, in order of increasing field strength of the ligand

In this series, ligands on the left cause small crystal field splittings and are weak-field ligands, whereas those on the right cause
larger splittings and are strong-field ligands. Thus, the Δoct value for an octahedral complex with iodide ligands (I−) is much
smaller than the Δoct value for the same metal with cyanide ligands (CN−).
Electrons in the d orbitals follow the aufbau (“filling up”) principle, which says that the orbitals will be filled to give the lowest
total energy, just as in main group chemistry. When two electrons occupy the same orbital, the like charges repel each other. The
energy needed to pair up two electrons in a single orbital is called the pairing energy (P). Electrons will always singly occupy each
orbital in a degenerate set before pairing. P is similar in magnitude to Δoct. When electrons fill the d orbitals, the relative
magnitudes of Δoct and P determine which orbitals will be occupied.
In [Fe(CN)6]4−, the strong field of six cyanide ligands produces a large Δoct. Under these conditions, the electrons require less
energy to pair than they require to be excited to the eg orbitals (Δoct > P). The six 3d electrons of the Fe2+ ion pair in the three t2g
orbitals (Figure 19.3.3). Complexes in which the electrons are paired because of the large crystal field splitting are called low-spin
complexes because the number of unpaired electrons (spins) is minimized.

Figure 19.3.3 : Iron(II) complexes have six electrons in the 5d orbitals. In the absence of a crystal field, the orbitals are degenerate.
For coordination complexes with strong-field ligands such as [Fe(CN)6]4−, Δoct is greater than P, and the electrons pair in the lower
energy t2g orbitals before occupying the eg orbitals. With weak-field ligands such as H2O, the ligand field splitting is less than the
pairing energy, Δoct less than P, so the electrons occupy all d orbitals singly before any pairing occurs.
In [Fe(H2O)6]2+, on the other hand, the weak field of the water molecules produces only a small crystal field splitting (Δoct < P).
Because it requires less energy for the electrons to occupy the eg orbitals than to pair together, there will be an electron in each of
the five 3d orbitals before pairing occurs. For the six d electrons on the iron(II) center in [Fe(H2O)6]2+, there will be one pair of
electrons and four unpaired electrons (Figure 19.3.3). Complexes such as the [Fe(H2O)6]2+ ion, in which the electrons are unpaired
because the crystal field splitting is not large enough to cause them to pair, are called high-spin complexes because the number of
unpaired electrons (spins) is maximized.

Access for free at OpenStax 19.3.3 https://chem.libretexts.org/@go/page/38323


A similar line of reasoning shows why the [Fe(CN)6]3− ion is a low-spin complex with only one unpaired electron, whereas both
the [Fe(H2O)6]3+ and [FeF6]3− ions are high-spin complexes with five unpaired electrons.

 Example 19.3.1: High- and Low-Spin Complexes

Predict the number of unpaired electrons.


a. K3[CrI6]
b. [Cu(en)2(H2O)2]Cl2
c. Na3[Co(NO2)6]

Solution
The complexes are octahedral.
a. Cr3+ has a d3 configuration. These electrons will all be unpaired.
b. Cu2+ is d9, so there will be one unpaired electron.
c. Co3+ has d6 valence electrons, so the crystal field splitting will determine how many are paired. Nitrite is a strong-field
ligand, so the complex will be low spin. Six electrons will go in the t2g orbitals, leaving 0 unpaired.

 Exercise 19.3.1

The size of the crystal field splitting only influences the arrangement of electrons when there is a choice between pairing
electrons and filling the higher-energy orbitals. For which d-electron configurations will there be a difference between high-
and low-spin configurations in octahedral complexes?

Answer
d4, d5, d6, and d7

 Example 19.3.2: CFT for Other Geometries

CFT is applicable to molecules in geometries other than octahedral. In octahedral complexes, remember that the lobes of the eg
set point directly at the ligands. For tetrahedral complexes, the d orbitals remain in place, but now we have only four ligands
located between the axes (Figure 19.3.4). None of the orbitals points directly at the tetrahedral ligands. However, the eg set
(along the Cartesian axes) overlaps with the ligands less than does the t2g set. By analogy with the octahedral case, predict the
energy diagram for the d orbitals in a tetrahedral crystal field. To avoid confusion, the octahedral eg set becomes a tetrahedral e
set, and the octahedral t2g set becomes a t2 set.

Figure 19.3.4 : This diagram shows the orientation of the tetrahedral ligands with respect to the axis system for the orbitals.

Solution
Since CFT is based on electrostatic repulsion, the orbitals closer to the ligands will be destabilized and raised in energy relative
to the other set of orbitals. The splitting is less than for octahedral complexes because the overlap is less, so Δtet is usually

Access for free at OpenStax 19.3.4 https://chem.libretexts.org/@go/page/38323


4
small (Δ tet = Δoct ) :
9

 Exercise 19.3.2

Explain how many unpaired electrons a tetrahedral d4 ion will have.

Answer
4; because Δtet is small, all tetrahedral complexes are high spin and the electrons go into the t2 orbitals before pairing

The other common geometry is square planar. It is possible to consider a square planar geometry as an octahedral structure with a
pair of trans ligands removed. The removed ligands are assumed to be on the z-axis. This changes the distribution of the d orbitals,
as orbitals on or near the z-axis become more stable, and those on or near the x- or y-axes become less stable. This results in the
octahedral t2g and the eg sets splitting and gives a more complicated pattern with no simple Δoct. The basic pattern is:

19.3.2: Magnetic Moments of Molecules and Ions


Experimental evidence of magnetic measurements supports the theory of high- and low-spin complexes. Remember that molecules
such as O2 that contain unpaired electrons are paramagnetic. Paramagnetic substances are attracted to magnetic fields. Many
transition metal complexes have unpaired electrons and hence are paramagnetic. Molecules such as N2 and ions such as Na+ and
[Fe(CN)6]4− that contain no unpaired electrons are diamagnetic. Diamagnetic substances have a slight tendency to be repelled by
magnetic fields.
When an electron in an atom or ion is unpaired, the magnetic moment due to its spin makes the entire atom or ion paramagnetic.
The size of the magnetic moment of a system containing unpaired electrons is related directly to the number of such electrons: the
greater the number of unpaired electrons, the larger the magnetic moment. Therefore, the observed magnetic moment is used to
determine the number of unpaired electrons present. The measured magnetic moment of low-spin d6 [Fe(CN)6]4− confirms that iron
is diamagnetic, whereas high-spin d6 [Fe(H2O)6]2+ has four unpaired electrons with a magnetic moment that confirms this
arrangement.

19.3.3: Colors of Transition Metal Complexes


When atoms or molecules absorb light at the proper frequency, their electrons are excited to higher-energy orbitals. For many main
group atoms and molecules, the absorbed photons are in the ultraviolet range of the electromagnetic spectrum, which cannot be
detected by the human eye. For coordination compounds, the energy difference between the d orbitals often allows photons in the
visible range to be absorbed.
The human eye perceives a mixture of all the colors, in the proportions present in sunlight, as white light. Complementary colors,
those located across from each other on a color wheel, are also used in color vision. The eye perceives a mixture of two

Access for free at OpenStax 19.3.5 https://chem.libretexts.org/@go/page/38323


complementary colors, in the proper proportions, as white light. Likewise, when a color is missing from white light, the eye sees its
complement. For example, when red photons are absorbed from white light, the eyes see the color green. When violet photons are
removed from white light, the eyes see lemon yellow. The blue color of the [Cu(NH3)4]2+ ion results because this ion absorbs
orange and red light, leaving the complementary colors of blue and green (Figure 19.3.5).

Figure 19.3.5 : (a) An object is black if it absorbs all colors of light. If it reflects all colors of light, it is white. An object has a color
if it absorbs all colors except one, such as this yellow strip. The strip also appears yellow if it absorbs the complementary color
from white light (in this case, indigo). (b) Complementary colors are located directly across from one another on the color wheel.
(c) A solution of [Cu(NH3)4]2+ ions absorbs red and orange light, so the transmitted light appears as the complementary color, blue.

 Example 19.3.3: Colors of Complexes

The octahedral complex [Ti(H2O)6]3+ has a single d electron. To excite this electron from the ground state t2g orbital to the eg
orbital, this complex absorbs light from 450 to 600 nm. The maximum absorbance corresponds to Δoct and occurs at 499 nm.
Calculate the value of Δoct in Joules and predict what color the solution will appear.

Solution
Using Planck's equation (refer to the section on electromagnetic energy), we calculate:
8
c 3.00 × 10 m/s
14
v= so = 6.01 × 10 Hz
λ 499 nm × 1 m

9
10 nm

−34 14 −19
E = hnu so 6.63 × 10 J⋅s × 6.01 × 10 Hz = 3.99 × 10 Joules/ion

Because the complex absorbs 600 nm (orange) through 450 (blue), the indigo, violet, and red wavelengths will be transmitted,
and the complex will appear purple.

 Exercise 19.3.3
A complex that appears green, absorbs photons of what wavelengths?

Answer
red, 620–800 nm

Access for free at OpenStax 19.3.6 https://chem.libretexts.org/@go/page/38323


Small changes in the relative energies of the orbitals that electrons are transitioning between can lead to drastic shifts in the color of
light absorbed. Therefore, the colors of coordination compounds depend on many factors. As shown in Figure 19.3.6, different
aqueous metal ions can have different colors. In addition, different oxidation states of one metal can produce different colors, as
shown for the vanadium complexes in the link below.

Figure 19.3.6 : The partially filled d orbitals of the stable ions Cr3+(aq), Fe3+(aq), and Co2+(aq) (left, center and right, respectively)
give rise to various colors. (credit: Sahar Atwa)
The specific ligands coordinated to the metal center also influence the color of coordination complexes. For example, the iron(II)
complex [Fe(H2O)6]SO4 appears blue-green because the high-spin complex absorbs photons in the red wavelengths (Figure
19.3.7). In contrast, the low-spin iron(II) complex K4[Fe(CN)6] appears pale yellow because it absorbs higher-energy violet

photons.

Figure 19.3.7 : Both (a) hexaaquairon(II) sulfate and (b) potassium hexacyanoferrate(II) contain d6 iron(II) octahedral metal
centers, but they absorb photons in different ranges of the visible spectrum.
In general, strong-field ligands cause a large split in the energies of d orbitals of the central metal atom (large Δoct). Transition
metal coordination compounds with these ligands are yellow, orange, or red because they absorb higher-energy violet or blue light.
On the other hand, coordination compounds of transition metals with weak-field ligands are often blue-green, blue, or indigo
because they absorb lower-energy yellow, orange, or red light.

The oxidation states of vanadium - Che…


Che…

Video 19.3.8 : Watch this video of the reduction of vanadium complexes to observe the colorful effect of changing oxidation states.
A coordination compound of the Cu+ ion has a d10 configuration, and all the eg orbitals are filled. To excite an electron to a higher
level, such as the 4p orbital, photons of very high energy are necessary. This energy corresponds to very short wavelengths in the

Access for free at OpenStax 19.3.7 https://chem.libretexts.org/@go/page/38323


ultraviolet region of the spectrum. No visible light is absorbed, so the eye sees no change, and the compound appears white or
colorless. A solution containing [Cu(CN)2]−, for example, is colorless. On the other hand, octahedral Cu2+ complexes have a
vacancy in the eg orbitals, and electrons can be excited to this level. The wavelength (energy) of the light absorbed corresponds to
the visible part of the spectrum, and Cu2+ complexes are almost always colored—blue, blue-green violet, or yellow (Figure 19.3.8).
Although CFT successfully describes many properties of coordination complexes, molecular orbital explanations (beyond the
introductory scope provided here) are required to understand fully the behavior of coordination complexes.

Figure 19.3.8 : (a) Copper(I) complexes with d10 configurations such as CuI tend to be colorless, whereas (b) d9 copper(II)
complexes such as Cu(NO3)2·5H2O are brightly colored.

Summary
Crystal field theory treats interactions between the electrons on the metal and the ligands as a simple electrostatic effect. The
presence of the ligands near the metal ion changes the energies of the metal d orbitals relative to their energies in the free ion. Both
the color and the magnetic properties of a complex can be attributed to this crystal field splitting. The magnitude of the splitting
(Δoct) depends on the nature of the ligands bonded to the metal. Strong-field ligands produce large splitting and favor low-spin
complexes, in which the t2g orbitals are completely filled before any electrons occupy the eg orbitals. Weak-field ligands favor
formation of high-spin complexes. The t2g and the eg orbitals are singly occupied before any are doubly occupied.

Glossary
crystal field splitting (Δoct)
difference in energy between the t2g and eg sets or t and e sets of orbitals

crystal field theory


model that explains the energies of the orbitals in transition metals in terms of electrostatic interactions with the ligands but
does not include metal ligand bonding

eg orbitals
set of two d orbitals that are oriented on the Cartesian axes for coordination complexes; in octahedral complexes, they are
higher in energy than the t2g orbitals

geometric isomers
isomers that differ in the way in which atoms are oriented in space relative to each other, leading to different physical and
chemical properties

high-spin complex
complex in which the electrons maximize the total electron spin by singly populating all of the orbitals before pairing two
electrons into the lower-energy orbitals

low-spin complex
complex in which the electrons minimize the total electron spin by pairing in the lower-energy orbitals before populating the
higher-energy orbitals

pairing energy (P)


energy required to place two electrons with opposite spins into a single orbital

Access for free at OpenStax 19.3.8 https://chem.libretexts.org/@go/page/38323


spectrochemical series
ranking of ligands according to the magnitude of the crystal field splitting they induce

strong-field ligand
ligand that causes larger crystal field splittings

t2g orbitals
set of three d orbitals aligned between the Cartesian axes for coordination complexes; in octahedral complexes, they are
lowered in energy compared to the eg orbitals according to CFT

weak-field ligand
ligand that causes small crystal field splittings

This page titled 19.3: Optical and Magnetic Properties of Coordination Compounds is shared under a CC BY 4.0 license and was authored,
remixed, and/or curated by OpenStax via source content that was edited to the style and standards of the LibreTexts platform; a detailed edit
history is available upon request.

Access for free at OpenStax 19.3.9 https://chem.libretexts.org/@go/page/38323


19.E: Transition Metals and Coordination Chemistry (Exercises)
19.E.1: 19.1: Occurrence, Preparation, and Properties of Transition Metals and Their Compounds
19.E.1.1: Q19.1.1
Write the electron configurations for each of the following elements:
a. Sc
b. Ti
c. Cr
d. Fe
e. Ru

19.E.1.2: S19.1.1
The electron configuration of an atom is the representation of the arrangement of electrons distributed among the orbital shells and sub-shells.
The electron configuration of each element is unique to its position on the periodic table where the energy level is determined by the period and
the number of electrons is given by the atomic number of the element. There are four different types of orbitals (s, p, d, and f) which have
different shapes and each orbital can hold a maximum of 2 electrons, but the p, d and f orbitals have different sub-levels, meaning that they are
able to hold more electrons.
The periodic table is broken up into groups which we can use to determine orbitals and thus, write electron configurations:
Group 1 & 2: S orbital
Group 13 - 18: P orbital
Group 3 - 12: D orbital
Lanthanide & Actinides: F orbital
Each orbital (s, p, d, f) has a maximum number of electrons it can hold. An easy way to remember the electron maximum of each is to
look at the periodic table and count the number of periods in each collection of groups.
Group 1 & 2: 2 (2 electrons total = 1 orbital x max of 2 electrons = 2 electrons)
Group 13 - 18: 6 (6 electrons total = 3 orbitals x 2 electrons max = 6 electrons)
Group 3 - 12: 10 (10 electrons total = 5 orbitals x 2 electrons max = 10 electrons)
Lanthanide & Actinides: 14 (14 electrons total = 7 orbitals x 2 electrons max = 14 electrons)
Electron fills the orbitals in a specific pattern that affects the order in which the long-hand versions are written:
Electron filling pattern: 1s, 2s, 2p, 3s, 3p, 4s, 3d, 4p, 5s, 4d, 5p, 6s, 4f, 5d, 6p, 7s, 5f
An easier and faster way to write electron configurations is to use noble gas configurations as short-cuts. We are able to do this because
the electron configurations of the noble gases always have all filled orbitals.
He: 1s22s2
Ne: 1s22s22p6
Ar: 1s22s22p63s23p6
Kr: 1s22s22p63s23p64s23d104p6
Xe: 1s22s22p63s23p64s23d104p65s24d105p6
Rn: 1s22s22p63s23p64s23d104p65s24d105p66s24f145d106p6
The most common noble gas configuration used is Ar. When you want to use the noble gas configuration short-cut, you place the
noble gas's symbol inside of brackets:
[Ar]
and then write it preceding the rest of the configuration, which is solely the orbitals the proceed after that of the noble gas.
a. Sc
Let's start off by identifying where Scandium sits on the periodic table: row 4, group 3. This identification is the critical basis we need to
write its electron configuration.

Access for free at OpenStax 19.E.1 https://chem.libretexts.org/@go/page/44104


By looking at Scandium's atomic number, 21, it gives us both the number of protons and the number of electrons. At the end of writing
its electron configuration, the electrons should add up to 21.
At row 4, group 3 Sc, is a transition metal; meaning that its electron configuration will include the D orbital.
Now, we can begin to assign the 21 electrons of Sc to orbitals. As you assign electrons to their orbitals, you move right across the
periodic table.
Its first 2 electrons are in the 1s orbital which is denoted as
1s2
where the "1" preceding the s denotes the fact that it is of row one, and it has an exponent of 2 because it fulfills the s
orbital's maximum electron number. Now we have 21-2=19 more electrons to assign.
Its next 2 electrons are in the 2s orbital which is denoted as
2s2
where the "2" preceding the s indicates that it is of row two, and it has an exponent of 2 because it fulfills the s orbital's
maximum electron number. Now we have 19-2=17 more electrons to assign.
Its next 6 electrons are in the 2p orbital which is denoted as
2p6
where the "2" preceding the p indicates that it is of row two, and it has an exponent of 6 because it fulfills the p orbital's
maximum electron number. Now we have 17-6=11 more electrons to assign.
Its next 2 electrons are in the 3s orbital which is denoted as
3s2
where the "3" preceding the s indicates that it is of row three, and it has an exponent of 2 because it fulfills the s orbital's
maximum electron number. Now we have 11-2=9 more electrons to assign.
Its next 6 electrons are in the 3p orbital which is denoted as
3p6
where the "3" preceding the p indicates that it is of row three, and it has an exponent of 6 because it fulfills the p orbital's
maximum electron number. Now we have 9-6=3 more electrons to assign.
Its next 2 electrons are in the 4s orbital which is denoted as
4s2
where the "4" preceding the s indicates that it is of row four, and it has an exponent of 2 because it fulfills the s orbital's
maximum electron number. Now we have 3-2=1 more electron to assign.
Its last electron would be alone in the 3 d orbital which is denoted as
3d1
where the "3" preceding the d indicates that, even though it is technically of row 4, by disregarding the first row of H and
He, this is the third row and it has an exponent of 1 because there is only 1 electron to be placed in the d orbital. Now we
have assigned all of the electrons to the appropriate orbitals and sub-orbitals, so that the final, entire electron configuration is
written as:
1s22s22p63s23p64s23d1
This is the long-hand version of its electron configuration.
So for Sc, its short-hand version of its electron configuration would therefore be:
[Ar] 4s23d1
b. Ti
Start off by identifying where Titanium sits on the periodic table: row 4, group 4, meaning it has 22 electrons total. Titanium is one
element to the right of the previous problem's Sc, so we will basically use the same method except, in the end, there will be 2 electrons
remaining, so therefore the final orbital will be denoted as:
3d2
If needed, look above to the exact steps for how to do it in detail again; the long-hand electron configuration for Titanium will be:

Access for free at OpenStax 19.E.2 https://chem.libretexts.org/@go/page/44104


1s22s22p63s23p64s23d2
So for Ti, its short-hand version of its electron configuration would therefore be:
[Ar] 4s23d2
c. Cr
Start off by identifying where Chromium sits on the periodic table: row 4, group 6, that means it has a total of 24 electrons. But first, Cr,
along with Mo, Nb, Ru, Rh, Pd, Cu, Sg, Pt and Au, is a special case. You would think that since it has 24 electrons that its configuration
would look like:
1s22s22p63s23p64s23d4
which is how we learned it earlier. However, this electron configuration is very unstable because of the fact that there are 4
electrons in its 3 d orbital. The most stable configurations are half-filled (d5) and full orbitals (d10), so the elements with electrons
resulting in ending with the d4 or d9 are so unstable that we write its stable form instead, where an electron from the preceding s
orbital will be moved to fill the d orbital, resulting in a stable orbital.
If needed, look above to the exact steps for how to do the beginning of the configuration in detail again. However we have to apple the
new rule to attain stability so that the long-hand electron configuration for Chromium will be:
1s22s22p63s23p64s13d5
So for Cr, its short-hand version of its electron configuration would therefore be:
[Ar] 4s13d5
d. Fe
Start off by identifying where Iron sits on the periodic table: row 4, group 8, meaning it has 26 electrons total. This is 5 elements to the
right of the previous problem's Sc, so we will basically use the same method except, in the end, there will be 6 electrons remaining, so
therefore the final orbital will be denoted as:
3d6
If needed, look above to the exact steps for how to do it in detail again; the long-hand electron configuration for Iron will be:
1s22s22p63s23p64s23d6
So for Fe, its short-hand version of its electron configuration would therefore be:
[Ar] 4s23d6
e. Ru
Start off by identifying where Ruthenium sits on the periodic table: row 5, group 8, that means it has a total of 44 electrons. But first, as
stated earlier, Ru, along with Cr, Mo, Nb, Rh, Pd, Cu, Sg, Pt and Au, is a special case. You would think that since it has 44 electrons that
its configuration would look like:
1s22s22p63s23p64s23d104p65s2 4d6
which is how we learned it earlier. However, this electron configuration is very unstable because of the fact that, even though there
are 4 paired electrons, there are also 4 electrons unpaired. This results in a very unstable configuration, so to restore stability, we
have to use a configuration that has the most paired electrons, which would be to take an electron from the s orbital and place it in
the d orbital to create:
5s14d7
If needed, look above to the exact steps for how to do the beginning of the configuration in detail again. However we have to apple the
new rule to attain stability so that the long-hand electron configuration for Ru will be:
1s22s22p63s23p64s23d104p65s14d7
So for Cr, its short-hand version of its electron configuration would therefore be:
[Kr] 5s14d7

19.E.1.3: A19.1.1
a. Sc: [Ar]4s23d1
b. Ti: [Ar]4s23d2
c. Cr: [Ar]4s13d5
d. Fe: [Ar]4s23d6
e. Ru: [Kr]5s14d7 (anomalous configuration)

Access for free at OpenStax 19.E.3 https://chem.libretexts.org/@go/page/44104


19.E.1.4: Q19.1.2
Write the electron configurations for each of the following elements and its ions:
a. Ti
b. Ti2+
c. Ti3+
d. Ti4+

19.E.1.5: S19.1.2
Electrons are distributed into molecular orbitals, the s, p, d, andf blocks. An orbital will have a number in front of it and a letter that
corresponds to the block. The s block holds two electrons, the p block holds six, the d block holds ten, and the f block holds fourteen. So, based
on the number of electrons an atom has, the molecular orbitals are filled up in a certain way. The order of the orbitals is
. An exponent will be put after the letter for each orbital to signify how
1s, 2s, 2p, 3s, 3p, 4s, 3d, 4p, 5s, 4d, 5p, 6s, 4f , 5d, 6p, 7s, 5f , 6d, 7p

many electrons are in that orbital. Noble gas notation can also be used by putting the noble gas prior to the element you are writing the
configuration for, and then proceed by writing the orbitals filled after the noble gas. Metal ions of the d-block will have the two electrons
removed from the s block prior to any electrons being removed from the proceeding d-block.
Solutions:
1. T i
Titanium has an atomic number of 22, meaning it has 22 electrons. The noble gas prior to Titanium is Argon. Looking at row 4 of the periodic
table, Titanium still has 4 electrons to be placed in orbitals since Argon has 18 electrons that are already placed. The remaining electrons will
fill the 4s orbital and the remaining two electrons will go into the 3d orbital. [Ar]4s23d2
2. T i
+2

This is an ion with a plus 2 charge, meaning 2 electrons have been removed. The electrons will be removed from the 4s orbital and the 2
remaining electrons will be placed in the 3d orbital. Like number 1, the prior noble gas is Argon. [Ar]3d2
3. T i
+3

This is an ion with a plus 3 charge, meaning 3 electrons have been removed. The first 2 electrons will be removed from the 4s orbital, and the
third will be taken from the 3d orbital, and the 1 remaining electron will be placed in the 3d orbital. Like number 1, the prior noble gas is
Argon. [Ar]3d1
4. T i
+4

This is an ion with a plus 4 charge, meaning 4 electrons have been removed. The first 2 electrons will be removed from the 4s orbital and the
second 2 will be removed from the 3d orbital. This results in the ion having the same electron configuration as Argon. [Ar]
Answers:
1. [Ar]4s 2
3d
2

2. [Ar]3d 2

3. [Ar]3d 1

4. [Ar]

19.E.1.6: A19.1.2
a. [Ar]4s 3d2 2

b. [Ar]3d 2

c. [Ar]3d 1

d. [Ar]

19.E.1.7: Q19.1.3
Write the electron configurations for each of the following elements and its 3+ ions:
a. La
b. Sm
c. Lu

19.E.1.8: S19.1.3
In order to write the electron configuration, we begin by finding the element on the periodic table. Since La, Sm, and Lu are all a period below
the noble gas Xenon, we can abbreviate 1s 2s 2p 3s 3p 3d 4s 4p 4d 5s 5p as [Xe] when writing the orbital configurations. We then
2 2 6 2 6 10 2 6 10 2 6

find the remaining of the orbital configurations using the Aufbau Principle. For other elements not just those in period 6, the shorthand notation
using noble gases would be the noble gas in the period above the given element.

Access for free at OpenStax 19.E.4 https://chem.libretexts.org/@go/page/44104


1. La has three additional electrons. Two of them fill the 6s shell and the other single electron is placed on the 5d shell.
La : [Xe] 6s 2
5d
1

2. Sm has eight more electrons. The 6s orbital is filled as previously and the 4f orbital receives 6 electrons because pairing electrons requires
lower energy on the 4f shell than on the 5d shell.
Sm : [Xe] 6s 2
4f
6

3. Lu has seventeen more electrons. Two electrons fill the 6s orbital, 14 electrons fill the 4f orbital, and extra single electron goes to the 5d
orbital .
Lu : [Xe] 6s 2
4f
14
5d
1

To find the 3+ ion electron configuration, we remove 3 electrons from the neutral configuration, starting with the 6s orbital.
1. The ionization of La3+ removes the three extra electrons. So it reverts back to the stable Xenon configuration.
La
3+
: [Xe]
2. The ionization of Sm3+ removes two electrons from the 6s shell and one from the outermost (4f) shell
Sm
3+
: [Xe] 4f 5

3. The ionization of Lu3+ removes its two 6s shell and one from the outermost (5d) shell, leaving only a full 4f shell
Lu
3+
: [Xe] 4f 14

19.E.1.9: A19.1.3
La: [Xe]6s 25d 1, La3+: [Xe]; Sm: [Xe]6s 24f 6, Sm3+: [Xe]4f 5; Lu: [Xe]6s 24f 145d 1, Lu3+: [Xe]4f 14

19.E.1.10: Q19.1.4
Why are the lanthanoid elements not found in nature in their elemental forms?

19.E.1.11: A19.1.4
Lanthanides are rarely found in their elemental forms because they readily give their electrons to other more electronegative elements, forming
compounds instead of staying in a pure elemental form. They have very similar chemical properties with one another, are often found deep
within the earth, and difficult to extract. They are the inner transition elements and have partially filled d orbitals that can donate electrons.
Because of this, they are very reactive and electropositive.

19.E.1.12: Q19.1.5
Which of the following elements is most likely to be used to prepare La by the reduction of La2O3: Al, C, or Fe? Why?

19.E.1.13: S19.1.5
An activity series is a list of elements in decreasing order of their reactivity. Elements on the top of the list are good reducing agents because
they easily give up an electron, and elements on the bottom of the series are good oxidizing agents because they are highly electronegative
would really want to accept an electron.
Step 1: Compare Aluminum, Carbon, and Iron on an activity series. Many activity series include carbon and hydrogen as references. An
activity series can be found here
The activity series goes in the order (from top to bottom): Aluminum, Carbon, and Iron.
Step 2: Identify which element is the best reducing agent.
Elements on the top of the list are the best reducing agents, because they give up electrons the best.
Aluminum is the best reducing agent of the options available.
Therefore aluminum will be the best reducing agent to prepare La by the reduction of La2O3 because it is the most reactive in the series
amongst the three elements.

19.E.1.14: A19.1.5
Al is used because it is the strongest reducing agent and the only option listed that can provide sufficient driving force to convert La(III) into
La.

19.E.1.15: Q19.1.6
Which of the following is the strongest oxidizing agent: VO 3 −
4
, CrO
2−
4
, or MnO ?

4

Access for free at OpenStax 19.E.5 https://chem.libretexts.org/@go/page/44104


19.E.1.16: S19.1.6
Oxidizing agents oxidize other substances. In other words, they gain electrons or become reduced. These agents should be in their highest
oxidation state. In order to determine, the strength of the compounds above as oxidizing agents, determine the oxidation numbers of each
constituent elements.
3−
VO
4

We know that O has a -2 oxidation state and the overall charge of the ion is -3. We just need to determine Vanadate's oxidation number in this
compound.
V + −2(4) = −3

V = +5

Vanadate has an oxidation number of +5, which is its highest possible oxidation state.
2−
CrO
4

Like in the previous calculation, O has a -2 oxidation state. The overall charge is -2. So calculate for chromium.
Cr + −2(4) = −2

Cr = +6

Chromium is in its highest possible oxidation state of +6 in this compound.



MnO
4

O has a -2 oxidation state and the overall charge is -1.


Mn + −2(4) = −1

Mn = +7

Manganese is also in its highest oxidation state, +7.


An oxidizing agent has to be able to gain electrons which, in turn, reduces its oxidation state. Here manganese has the greatest oxidation state
which allows it to experience a greater decrease in its oxidation state if needed, meaning it can gain the most electrons. So among the three
compounds, MnO is the strongest oxidizing agent. This method assumes the metals have similar electronegativities.

Alternatively, check a redox table.

19.E.1.17: A19.1.6

M nO
4

19.E.1.18: Q19.1.7
Which of the following elements is most likely to form an oxide with the formula MO3: Zr, Nb, or Mo?

19.E.1.19: S19.1.7
Mo because Zr has an oxidation state of +4 and Nb has a oxidation state of +5 and those would not balance out the charge of 3 oxygens in the
state of -2 which creates a charge of -6. Mo however has multiple oxidation states, the most common being +6 which balances out the -6 charge
created by 3 oxygen ions. This is why its most likely to form an oxide with the formula MO3 or MoO . 3

19.E.1.20: A19.1.7
Mo

19.E.1.21: Q19.1.8
The following reactions all occur in a blast furnace. Which of these are redox reactions?
a. 3 Fe O (s) + CO(g) ⟶ 2 Fe O (s) + CO
2 3 3 4 2
(g)

b. Fe O (s) + CO(g) ⟶ 3 FeO(s) + CO (g)


3 4 2

c. FeO(s) + CO(g) ⟶ Fe(l) + CO (g) 2

d. C(s) + O (g) ⟶ CO (g) 2 2

e. C(s) + CO (g) ⟶ 2 CO(g) 2

f. CaCO (s) ⟶ CaO(s) + CO (g)


3 2

g. CaO(s) + SiO (s) ⟶ CaSiO (l) 2 3

Access for free at OpenStax 19.E.6 https://chem.libretexts.org/@go/page/44104


19.E.1.22: S19.1.8
o identify redox reaction, we have to determine if have to see if the equation is an oxidation-reduction reaction-meaning that the species are
changing oxidation states during the reaction, which involves the transfer of electrons between two species. If a species is losing electrons, then
that species is being oxidized. If a species is gaining electrons, then that species is being reduced. A way to remember this is using the
acronyms OIL RIG. Oxidation Is Loss, and Reduction Is Gain, referring to electrons. Both of these must occur for an equation to be a redox
reaction. Let's see if these equations are redox reactions or not:
a. In the reactants side Fe O , Fe is has an oxidation number of +3. In the product Fe O , Fe has an oxidation number of +2.67. Since Fe
2 3 3 4

changed from +3 to +2.67, we can say that Fe had gained electrons and therefore reduced. In the reactant, CO, carbon has an oxidation number
of +2, and in CO (product) carbon has an oxidation number of +4. Therefore, carbon has lost electrons and it has been oxidized. Since there is
2

oxidation and reduction of species- we can conclude that this is a redox reaction.
b. In the reactant, Fe O , Fe has an oxidation number of +2.67. In the product, FeO, Fe has an oxidation number of +2. Since the oxidation of
3 4

Fe has changed from +2.67 to +2, electrons have been added therefore Fe has been reduced. In the reactant, CO, carbon has an oxidation
number of +2, and in CO (product) carbon has an oxidation number of +4. Therefore, carbon has lost electrons and it has been oxidized. Since
2

there is oxidation and reduction of species- we can conclude that this is a redox reaction.
c. In the reactant side, in FeO, Fe has an oxidation number of +2 and in the products side Fe has an oxidation number of 0. Since the oxidation
number of Fe changed from +2 to 0, electrons have been gained and therefore Fe has been reduced. In the reactant, CO, carbon has an
oxidation number of +2, and in CO (product) carbon has an oxidation number of +4. Therefore, carbon has lost electrons and it has been
2

oxidized. Since there is oxidation and reduction of species- we can conclude that this is a redox reaction.
d. In the reactants C has an oxidation number of 0, and in the products side in CO , C has an oxidation number of +4. Since the oxidation
2

number of C has changed from 0 to +4, we can say that C has been oxidized. In the reactants, in O oxygen has an oxidation number of 0, and
2

in the products CO2, oxygen has an oxidation number of -2. Since the oxidation number of oxygen has changed from 0 to -2, oxygen has been
reduced. Since there is oxidation and reduction of species- we can conclude that this is a redox reaction.
e. In the reactants CO has an oxidation number of +4, and in the products side in CO, C has an oxidation number of +2. Since carbon went
2

from +4 to +2, carbon has been reduced. In the reactants, in CO oxygen has an oxidation number of -4 and in the products CO carbon has an
2

oxidation number of -2. Since oxygen went from -4 to -2, it has been oxidized. Since there is oxidation and reduction of species- we can
conclude that this is a redox reaction.
f. In the reactants, CaCO Ca has an oxidation number of +2, and in products CaO Ca has an oxidation number of +2. Since the oxidization
3

number doesn't change- we can conclude that this equation is not a redox reaction.
g. In the products CaO Ca has an oxidation number of +2, and in the products CaSiO Ca has an oxidation number of +2. Since the oxidization
3

number doesn't change- we can conclude that this equation is not a redox reaction.

19.E.1.23: A19.1.8
a, b, c, d, e

19.E.1.24: Q19.1.9
Why is the formation of slag useful during the smelting of iron?

19.E.1.25: S19.1.9
Slag is a substance formed as a byproduct of iron ore or iron pellets melting together in a blast furnace. Slag is also the byproduct that is formed
when a desired metal has been separated from its raw ore. It is important to note that slag from steel mills is created in a manner that reduces
the loss of the desired iron ore. The CaSiO slag is less dense than the molten iron, so it can easily be separated. Also, the floating slag layer
3

creates a barrier that prevents the molten iron from exposure to O , which would oxidize the Fe back to Fe O . Since Fe has a low reduction
2 2 3

potential of -0.440 this means it has a high oxidation potential so it would easily oxidize in the presence of O2. Creating a barrier between iron
and oxygen allows the maximum product of iron to be obtained in the end of smelting.

19.E.1.26: A19.1.9
The CaSiO3 slag is less dense than the molten iron, so it can easily be separated. Also, the floating slag layer creates a barrier that prevents the
molten iron from exposure to O2, which would oxidize the Fe back to Fe2O3.

19.E.1.27: Q19.1.10
Would you expect an aqueous manganese(VII) oxide solution to have a pH greater or less than 7.0? Justify your answer.

19.E.1.28: S19.1.10
Manganese(VII) oxide, can be written as Mn2O7.

Access for free at OpenStax 19.E.7 https://chem.libretexts.org/@go/page/44104


In relation to the Lewis acid-base theory, a Lewis acid accepts lone pair electrons, and is also known as the electron pair acceptor. Based on this
theory, acidity can be measured by the element's ability to accept electron pairs. By doing the math, we find that Manganese has an oxidation
state of +7 (Oxygen has an oxidation state of -2, and 2x-7=-14 or this can be shown as −7(2) + 2(x) = 0 and x = 7 since the whole
compound has a charge of zero, in order to balance the ion's charge, Mn must be +7). Therefore Mn has high capability of accepting electrons
due to its high positive charge. For most metals, as the oxidation number increases, so does its acidity, because of its increased ability to accept
electrons.

19.E.1.29: A19.1.10
In relation to the Lewis acid-base theory, the Lewis acid accepts lone pair electrons; thus, it is also known as the electron pair acceptor. This
may be any chemical species. Acids are substances that must be lower than 7. Therefore, oxides of manganese is most likely going to become
more acidic in (aq) solutions if the oxidation number increases.

19.E.1.30: Q19.1.11
Iron(II) can be oxidized to iron(III) by dichromate ion, which is reduced to chromium(III) in acid solution. A 2.5000-g sample of iron ore is
dissolved and the iron converted into iron(II). Exactly 19.17 mL of 0.0100 M Na2Cr2O7 is required in the titration. What percentage of the ore
sample was iron?

19.E.1.31: S19.1.11
To answer this question, we must first identify the net ionic equation from the given half-reactions. We can write the oxidation and reduction
half-reactions:
 2+  3+
 oxidation:  Fe →  Fe (19.E.1)

2−  3+
 reduction: Cr O →  Cr (19.E.2)
2 7

We can quickly balance the oxidation half-reaction by adding the appropriate number of electrons to get
 2+  3+ −
 Fe →  Fe +  e (19.E.3)

The first step in balancing the reduction half-reaction is to balance elements in the equation other than O and H. In doing so, we get
2−  3+
Cr O → 2  Cr (19.E.4)
2 7

The second step would be to add enough water molecules to balance the oxygen.
2−  3+
Cr O → 2  Cr +7 H O (19.E.5)
2 7 2

+
Next, we add the correct amount of H to balance the hydrogen atoms.
2− +  3+
Cr O7 + 14  H → 2  Cr +7 H O (19.E.6)
2 2

Finally, we add enough electrons to balance charge.


2− + −  3+
Cr O7 + 14  H + 6  e → 2  Cr +7 H O (19.E.7)
2 2

The electrons involved in both half-reactions must be equal in order for us to combine the two to get the net ionic equation. This can be done by
multiplying each equation by the appropriate coefficient. Scaling the oxidation half-reaction by 6, we get
 2+  3+ −
6  Fe → 6  Fe + 6  e (19.E.8)

Now we can combine both half-reactions to get


 2+ 2− + −  3+ −  3+
6  Fe + Cr O + 14  H + 6  e → 6  Fe + 6  e + 2  Cr +7 H O (19.E.9)
2 7 2

The electrons cancel out, so you get:


 2+ 2− +  3+  3+
6  Fe + Cr O + 14  H → 6  Fe + 2  Cr +7 H O (19.E.10)
2 7 2

From this we can see that the mole ratio of Cr2O72- to Fe2+ is 1:6. Given that 19.17 mL (or 0.01917 L) of 0.01 M Na2Cr2O7 was needed for
titration we know that
−4
0.01917 L × 0.01 M = 1.917 × 10  mol (19.E.11)

of Na2Cr2O7 reacted. Also, since any number of moles of Na2Cr2O7 produces the same number of moles of Cr2O72- in solution
−4 −4 2−
1.917 × 10  mol of Na Cr O = 1.917 × 10  mol of Cr O (19.E.12)
2 2 7 2 7

Access for free at OpenStax 19.E.8 https://chem.libretexts.org/@go/page/44104


We can use the mole ratio of Cr2O72- to Fe2+ to determine how many moles of iron (ii) was in the solution. The number of moles of iron (ii) is
the same as the number of moles of pure iron in the sample since all of the iron was converted into iron (ii).
 2+
6 mol of  Fe
−4 2−  2+
1.917 × 10  mol of Cr O × = 0.0011502 mol of  Fe (19.E.13)
2 7 2−
1 mol of Cr O
2 7

 2+
0.0011502 mol of  Fe = 0.0011502 mol of  Fe (19.E.14)

Now we can find the number of grams of iron that were present in the 2.5 g iron ore sample.
55.847 g
0.0011502 mol of  Fe × = 0.0642352194 g of  Fe (19.E.15)
1 mol

Finally, we can answer the question and find what percentage of the ore sample was iron.
0.0642352194 g
× 100 ≈ 2.57% (19.E.16)
2.5 g

So 2.57% of the ore sample was iron.

19.E.1.32: A19.1.11
2.57%

19.E.1.33: Q19.1.12
How many cubic feet of air at a pressure of 760 torr and 0 °C is required per ton of Fe2O3 to convert that Fe2O3 into iron in a blast furnace? For
this exercise, assume air is 19% oxygen by volume.

19.E.1.34: S19.1.12
This question uses a series of unit conversions and the P V = nRT equation.
The first step is to write out the balanced chemical equation for the conversion of Fe2O3 to pure iron.

2 F e2 O3 (s) → 4 F e(s) + 3 O2 (g) (19.E.17)

Next, we need to analyze the original question to determine the value that we need to solve for. Because the question asks for a value of cubic
feet, we know we need to solve for volume. We can manipulate P V = nRT to solve for volume.

V = nRT /P (19.E.18)

Now determine the known variables and convert into units that will be easy to deal with.
453.592 grams F e2 O3 1 mole F e2 O3 3 moles O2
n = 2000 lbs F e2 O3 (19.E.19)
1 lb F e2 O3 159.69 grams F e2 O3 2 moles F e2 O3

n = 8521 moles of O2 (19.E.20)

Convert to atm for easier calculations


.0821 L atm
R = (19.E.21)
mol K


T = 0 C = 273 K (19.E.22)

P = 760 torr = 1 atm (19.E.23)

Now plug the numbers into the manipulated gas law to get to an answer for V.
V = 190991.8 liters of O2 (19.E.24)

From here we convert liters to cubic feet.


use the conversion
3
1 L = .0353f t (19.E.25)

3
thus we have 6744.811 ft of O2
We then refer back to the initial question and remember that this value is only 19% of the volume of the total air. So use a simple equation to
determine the total volume of air in cubic feet.
3
6744.811f t = .19x (19.E.26)

Access for free at OpenStax 19.E.9 https://chem.libretexts.org/@go/page/44104


x=35499 ft3 of air

19.E.1.35: A19.1.12
35499 ft3 of air

19.E.1.36: Q19.1.13
Find the potentials of the following electrochemical cell:
Cd | Cd2+ (M = 0.10) ‖ Ni2+ (M = 0.50) | Ni

19.E.1.37: S19.1.13
Step 1 Write out your two half reactions and identify which is oxidation and which is reduction using the acronym OIL RIG to remember that
oxidation is loss of electrons and reduction is gain of electrons
Cd(s)⟶Cd2+(aq)+2e- (oxidation)
Ni2+(aq)+2e-⟶Ni(s) (reduction)
Step 2 Write out complete balanced equation
Cd(s)+ Ni2+(aq)⟶Cd2+(aq)+Ni(s)
Step 3 Find Eocell
Image result for reduction potential table

Eocell = Ecathode-Eanode
oxidation: Cd(s)⟶Cd2+(aq)+2e- Eo=-0.40V
reduction: Ni2+(aq)+2e-⟶Ni(s) Eo=-0.26V
* E values come from standard reduction potentials table given above. Also, remember anode is where oxidation happens, and cathode is
where reduction happens.
Eocell=-0.26-(-.40)
Eocell=0.14V
Step 4 Find Q
Q=[products]/[reactants] (look at complete balanced equation) (remember that [x] means the concentration of x typically given in
molarity and that we ignore solids or liquids)
Q=[Cd2+]/[Ni2+]
Q=0.10/0.50
Q=0.2
Step 5 Calculate E using E= Eocell-(.0592/n)logQ (n is number of moles of electrons transferred and in our case the balanced reaction transfers 2
electrons)
E= 0.14-(.0592/2)log(0.2)
E= 0.14-(-.207)
E=0.16 V

19.E.1.38: A19.1.13
0.16 V

19.E.1.39: Q19.1.14
A 2.5624-g sample of a pure solid alkali metal chloride is dissolved in water and treated with excess silver nitrate. The resulting precipitate,
filtered and dried, weighs 3.03707 g. What was the percent by mass of chloride ion in the original compound? What is the identity of the salt?

19.E.1.40: S19.1.14
A 2.5624-g sample of a pure solid alkali metal chloride is dissolved in water and treated with excess silver nitrate. The resulting
precipitate, filtered and dried, weighs 3.03707 g. What was the percent by mass of chloride ion in the original compound? What is the
identity of the salt?
Assuming that metal chloride is XCl

Access for free at OpenStax 19.E.10 https://chem.libretexts.org/@go/page/44104


The balance equation for the reaction would be:
XC l(aq) + AgN O3 (aq) → XN O3 (aq) + AgC l(s) (19.E.27)

The mass of AgCl = 3.03707g


To find the moles of AgCl present:
Next, we can determine the moles of AgCl present in the reaction since 1) the mass of the precipitate is given to us and 2) this value can help us
determine the moles of alkali metal chloride compound present. Given the mass of AgCl is 3.03707g in the problem and the molecular mass of
AgCl per mole is 143.32g, we can solve for how many moles of AgCl is in the reaction:
3.03707g
molesof Agcl = = 0.0211mol (19.E.28)
143.32g/mol

Since the molar ratio of the compounds are 1:1 so the number of moles of XCl used = 0.0211 mol
We can calculate the weight of Cl- with the equation:
0.0211mol × 35.5g/mol = 0.7490g (19.E.29)

the amount of metal present in the original compound is the weight of the compound subtracted by weight of the Cl ion:
(2.5624 − 0.7490)g = 1.8134g (19.E.30)

And the percentage can be calculate by


0.7490
× 100 = 29.23% (19.E.31)
2.5624

the molar ratio of XCl is 1:1 so then


1.8134 g metal
Atomic mass of metal = = 0.0211 mol RbC l
= 85.943g/mol

So the atomic mass is 85.943 g/mol which is of Rb hence the identity of the salt is RbCl

19.E.1.41: Q19.1.15
3+ 2+
The standard reduction potential for the reaction [Co(H O) ] (aq) + e ⟶ [Co(H O) ] (aq) is about 1.8 V. The reduction potential for
2 6

2 6

the reaction [Co(NH ) ] (aq) + e ⟶ [Co(NH ) ] (aq) is +0.1 V. Calculate the cell potentials to show whether the complex ions,
3 6
3+ −
3 6
2+

[Co(H2O)6]2+ and/or [Co(NH3)6]2+, can be oxidized to the corresponding cobalt(III) complex by oxygen.

19.E.1.42: S19.1.15
To calculate the cell potential, we need to know the potentials for each half reaction. After doing so, we need to determine which one is being
oxidized and which one is being reduced. The one that is oxidized is the anode and the one that is reduced is the cathode. To find the cell
potential, you use this formula and the reduction potential values found in a reduction potential table. If E°cell is positive, ΔG is negative and
the reaction is spontaneous.
E°cell= E°cathode - E°anode
Because it states that [C o(H 2 O)6 ]
3+
will be oxidized, this means it is the anode.
O2 (g) + 4 H
+
(aq) + 4 e

→ 2 H2 O +1.229 V
O2 is being reduced, so it is the cathode.
1.229V - 1.8V= -.571 V, or -0.6 V using significant figures. This cannot happen spontaneously because E°cell is negative.
For [C o(N H 3 )6 ]
3+
, it is again being oxidized, meaning it’s the anode.
1.229-.1= 1.129 V or 1.1 V using significant figures. This reaction is spontaneous because E°cell is positive.

19.E.1.43: A19.1.15
a) E ° = −0.6 V, E ° is negative so this reduction is not spontaneous.
b) E ° = +1.1 V, E ° is positive so this reduction is spontaneous.

19.E.1.44: Q19.1.16
Predict the products of each of the following reactions. (Note: In addition to using the information in this chapter, also use the knowledge you
have accumulated at this stage of your study, including information on the prediction of reaction products.)
a. MnCO (s) + HI(aq) ⟶
3

b. CoO(s) + O (g) ⟶ 2

Access for free at OpenStax 19.E.11 https://chem.libretexts.org/@go/page/44104


c. La(s) + O (g) ⟶
2

d. V(s) + VCl (s) ⟶


4

e. Co(s) + xsF (g) ⟶ 2

f. CrO (s) + CsOH(aq) ⟶


3

19.E.1.45: S19.1.16
There is a myriad of reactions that can occur, which include: single replacement, double replacement, combustion, acid-base/neutralization,
decomposition or synthesis. The first step to determine the products of a reaction is to identify the type of reaction. From then on, the next steps
you take to predict the products will vary based on the reaction type.
1. This reaction is a double displacement reaction, in which the cations and anions of the reactants switch places to form new compounds.
Writing out the equation in terms of it's aqueous ions will help you visualize what exactly is getting moved around:
+ − + 2− 2+ −
2H (aq) + 2 I (aq) → 2 H (aq) + C O (aq) + M n (aq) + 2 I (aq) (19.E.32)
3

In this case, the hydrogen cations will recombine with carbonate anions whilst manganese cations will recombine with iodide anions giving
us the following equation:
MnCO (s) + 2 HI(aq) ⟶ MnI (aq) + H CO (aq) (19.E.33)
3 2 2 3

This is still not the final answer however, as carbonic acid is unstable and decomposes to carbon dioxide and water under standard
conditions. Taking this into account, our final equation is:
MnCO (s) + 2 HI(aq) ⟶ MnI (aq) + CO (g) + H O(l) (19.E.34)
3 2 2 2

2. This reaction is a synthesis reaction, in which two or more reactants combine to form a more complex compound. In this case we are also
reacting a metal oxide with oxygen which would result in another metal oxide as the product. The resulting product would be the mixed
valence oxide Co3O4 in which one cobalt atom has a +2 oxidation state whilst the other two have a +3 oxidation state. Now all is left is to
balance the equation:
6 CoO(s) + O (g) ⟶ 2 Co O (s) (19.E.35)
2 3 4

3. Like equation 2, this reaction is also a synthesis reaction involving a metal and oxygen which should result in the formation of a metal
oxide. It is a matter now of balancing the oxidation states to attain a neutral compound. Oxygen will always hold a -2 oxidation state in
compounds whilst lanthanum will always exhibit a +3 oxidation state. As such, a combination of 2 lanthanum atoms with a +3 oxidation
state and 3 oxygen atoms with a -2 oxidation state will give us a molecule with an overall charge of 0 (3(-2)+2(+3)=0). We know our
product now, La2O3, and now just need to balance the overall equation, giving us:
4 La(s) + 3 O (g) ⟶ 2 La O (s) (19.E.36)
2 2 3

4. This reaction is slightly harder to define as it encapsulates both the properties of synthesis and decomposition reactions, wherein vanadium
reacts with vanadium tetrachloride to produce vanadium trichloride. This reaction however is primarily a synthesis reaction since we are
combining two reactants to produce one complex compound. With vanadium trichloride as our product, we can balance the equation:
V(s) + 3 VCl (s) ⟶ 4 VCl (s) (19.E.37)
4 3

5. This is another synthesis reaction combining cobalt and fluorine. This equation includes the "xs" notation in front of fluorine which is short
for 'excess', meaning more fluorine than actually required is present in the reactants, ensuring the reaction goes to completion. Finding the
products if a simple matter of combining the cobalt and fluorine into one molecule, which already leaves us with a balanced equation:
Co(s) + xsF (g) ⟶ CoF (s) (19.E.38)
2 2

6. It may not be obvious here, but the reaction we've been given here is actually an acid-base/neutralization reaction, with chromium
trioxide acting as the acid and cesium hydroxide as the base. Chromium trioxide is referred to as an acidic oxide which means that it will
react with water to form an acid. Note that this reaction can still proceed even if the reactants aren't in the same phases. The basic rule for
acid-base/neutralization reactions is they form a salt (salt being the general term for any ionic compound formed from acid-base reactions)
and water. Since we know water is one of our products, our other product must be a salt composed of cesium, chromium and oxygen. Thus,
our other product should be cesium chromate, and you can now balance the equation accordingly:
CrO (s) + 2 CsOH(aq) ⟶ Cs CrO (aq) + H O (19.E.39)
3 2 4 2

Access for free at OpenStax 19.E.12 https://chem.libretexts.org/@go/page/44104


19.E.1.46: A19.1.16
1. MnCO (s) + 2 HI(aq) ⟶ MnI (aq) + CO (g) + H
3 2 2 2
O(l)

2. 6 CoO(s) + O (g) ⟶ 2 Co O (s)


2 3 4

3. 4 La(s) + 3 O (g) ⟶ 2 La O (s)


2 2 3

4. V(s) + 3 VCl (s) ⟶ 4 VCl (s)


4 3

5. Co(s) + xsF (g) ⟶ CoF (s)


2 2

6. CrO (s) + 2 CsOH(aq) ⟶ Cs CrO (aq) + H O


3 2 4 2

19.E.1.47: Q19.1.17
Predict the products of each of the following reactions. (Note: In addition to using the information in this chapter, also use the knowledge you
have accumulated at this stage of your study, including information on the prediction of reaction products.)
a. Fe(s) + H SO (aq) ⟶2 4

b. FeCl (aq) + NaOH(aq) ⟶


3

c. Mn(OH) (s) + HBr(aq) ⟶


2

d. Cr(s) + O (g) ⟶ 2

e. Mn O (s) + HCl(aq) ⟶
2 3

f. Ti(s) + xsF (g) ⟶ 2

19.E.1.48: S19.1.17
Predict the products of each of the following reactions.
Fe(s)+H2SO4(aq)⟶ ?
Whenever a metal reacts with an acid, the products are salt and hydrogen. Because Fe is lower on the activity series, we know that when
it reacts with an acid it will result in the formation of Hydrogen gas. To simplify the equation is:
M etal + Acid ⟶ Salt + H ydrogen (19.E.40)

The salt produced will depend on the metal and in this case, the metal is iron (Fe) so the resulting equation would be:

Fe(s) + H SO (aq) ⟶ FeSO (aq) + H (g) (19.E.41)


2 4 4 2

This equation works out as the H2 is removed from H2SO2, resulting in a SO42- ion where Fe will take on an oxidation state of Fe+2 to
form FeSO4 which will be the salt in this example.
But since FeSO4 and H2SO4 are aqueous, the reactants and products can also be written as its ions where the overall equation can be:
+ −4 + −4
Fe(s) + 2 H O (aq) + SO 2 (aq) ⟶ Fe (aq) + SO 2 (aq) + H (g) + 2 H O(l) (19.E.42)
3 2 2 2

FeCl3(aq)+NaOH(aq)⟶ ?
In this case, adding a metal hydroxide (NaOH) to a solution with a transition metal ion (Fe) will form a transition metal hydroxide
(XOH). As iron is bonded to three chlorine atoms in the reactants side, it has the oxidation state of +3 where three hydroxide ions (OH-)
are needed to balance out the charges when they are bonded in the products. The remaining ions are Na+ and Cl- where they bond
together in a 1:1 ratio where there are 3 molecules of NaCl once the reaction is balanced.
The overall reaction will be:

FeCl (aq) + NaOH(aq) ⟶ Fe(OH) (s) + 3 NaCl(aq) (19.E.43)


3 3

NOTE: Fe(OH)3(s) is a solid as it is a rule that all all transition metal hydroxides are insoluble and a precipitate is formed.
Since NaOH(aq) and NaCl(aq) are aqueous, we can write them out in their ion forms:
+ − + +
FeCl (aq) + 3 Na (aq) + 3 OH (aq) + Fe(OH) (s) + 3 Na (aq) + 3 Cl (aq) (19.E.44)
3 3

Mn(OH)2(s)+HBr(aq)⟶ ?
This is an example of a metal hydroxide reacting with an acid where a metal salt and water will always be formed:
M etalH ydroxide + Acid ⟶ M etalSalt + W ater (19.E.45)

When this rule is applied to this equation, we will get the following:
Mn(OH) (s) + HBr(aq) ⟶ MnBr (aq) + 2 H O(l) (19.E.46)
2 2 2

But to follow through with this question, the aqueous solutions such as HBr(aq) and MnBr2(aq) can be re-written as:
+ − + −
Mn(OH) (s) + 2 H O (aq) + 2 Br (aq) ⟶ Mn (aq) + 2 Br (aq) + 4 H O(l) (19.E.47)
2 3 2 2

Access for free at OpenStax 19.E.13 https://chem.libretexts.org/@go/page/44104


Cr(s)+O2(g)⟶ ?
This is the general reaction of a metal reacting with oxygen which will always result in a metal oxide. However, the metal oxide is
determined by the oxidation state of the metal so there may be several outcomes of this reaction such as:

4 Cr(s) + 3 O (g) ⟶ 2 Cr O (s) (19.E.48)


2 2 3

Cr(s) + O (g) ⟶ 2 CrO(s) (19.E.49)


2

Cr(s) + O (g) ⟶ CrO (s) (19.E.50)


2 2

2 Cr(s) + 3 O (g) ⟶ CrO (s) (19.E.51)


2 3

However, Cr2O3 is the main oxide of chromium so it can be assumed that this is the general product of this reaction.
Mn2O3(s)+HCl(aq)⟶?
This follows the general reaction of a metal oxide and an acid will always result in a salt and water
M etalOxide + Acid ⟶ Salt + W ater (19.E.52)

Using this general reaction, similar to the general reactions above, the reaction will result in:
Mn O (s) + HCl(aq) ⟶ 2 MnCl (s) + 9 H O(l) (19.E.53)
2 3 3 2

However, since HCl is an aqueous solution, the overall equation can also be re-written as:
+ −
Mn O (s) + 6 H O (aq) + 6 Cl (aq) ⟶ 2 MnCl (s) + 9 H O(l) (19.E.54)
2 3 3 3 2

Ti(s)+xsF2(g)⟶?
Titanium is able to react with the halogens where there are two oxidation state that titanium can be: +3 and +4. The following reactions
follow each oxidation state accordingly:
2 Ti(s) + 3 F (g) ⟶ 2 TiF (s) (19.E.55)
2 3

Ti(s) + 2 F (g) ⟶ TiF (s) (19.E.56)


2 4

However, since there is the symbol "xs", this indicates that F2 is added in excess so the second reaction is favored more as it drives the
reaction to completion.
OVERALL:
\[\ce{Ti}(s)+\ce{xsF2}(g)⟶\ce{TiF4}(g)\]

A19.1.17
a. Fe(s) + 2 H O (aq) + SO (aq) ⟶ Fe (aq) + SO (aq) + H (g) + 2 H O(l) ;
3
+ 2−

4
2+ 2−

4 2 2

b. FeCl (aq) + 3 Na (aq) + 3 OH (aq) + Fe(OH) (s) + 3 Na (aq) + 3 Cl (aq) ;


3
+ −

3
+ +

c. Mn(OH) (s) + 2 H O (aq) + 2 Br (aq) ⟶ Mn (aq) + 2 Br (aq) + 4 H O(l) ;


2 3
+ − 2+ −

d. 4 Cr(s) + 3 O (g) ⟶ 2 Cr O (s) ;


2 2 3

e. Mn O (s) + 6 H O (aq) + 6 Cl (aq) ⟶ 2 MnCl (s) + 9 H O(l) ;


2 3 3
+ −

3 2

f. Ti(s) + xsF (g) ⟶ TiF (g)


2 4

19.E.1.49: Q19.1.18
Describe the electrolytic process for refining copper.

19.E.1.50: S19.1.18
By electrolysis, copper can be refined and purely made. The reason why copper needs to remove the impurities is because it helps increase the
electrical conductivity in electrical wire. You can refine copper and remove the impurities through electrolysis. Pure copper is important in
making electrical wire, because it creates better electrical conductivity when transferring electricity. In order for better electrical conductivity,
the impurities needs to be removed and this can be done by firing the impure copper to remove the impurities, such as sulfur, oxygen, etc. and
shaping them into electrical anodes that can be used in electrolysis. Then the copper electrodes are placed into an electrical cell (into separate
beakers) where electrical current can pass through the beakers and onto the electrodes. Through this process, the copper is stripped off of the
anode and deposited onto the cathode. This process helps remove the impurities and refine copper because all the copper has been deposited
onto the cathode all in one electrode. This process increases the weight of the cathode due to copper being deposited onto the cathode. This is a
prime example of how to tell if an electrode is a cathode or an anode, as stated in Q17.2.9 above.

Access for free at OpenStax 19.E.14 https://chem.libretexts.org/@go/page/44104


19.E.1.51: Q19.1.19
Predict the products of the following reactions and balance the equations.
a. Zn is added to a solution of Cr2(SO4)3 in acid.
b. FeCl2 is added to a solution containing an excess of Cr 2
O
2−

7
in hydrochloric acid.
c. Cr2+ is added to Cr O in acid solution.
2
2−

d. Mn is heated with CrO3.


e. CrO is added to 2HNO3 in water.
f. FeCl3 is added to an aqueous solution of NaOH.

19.E.1.52: S19.1.19
1. is added to a solution of in acid.
Oxidized half-reaction:
Reduction half reaction:
Overall reaction:
Chromium will precipitate out of the solution because it has a higher reduction potential than Zinc; the reaction is a single replacement.
2. is added to a solution containing an excess of in hydrochloric acid.
Dissociation reaction:
Oxidation half-reaction:
Reduction half-reaction:
Overall reaction:
The reduction potential for permanganate is larger so the reaction is still favorable even when the oxidation of is negative.
3. is added to in acid solution.
Reduction half-reaction:
Oxidation half-reaction:
Overall reaction:
The reaction is favorable with a high positive
4. is heated with .
Reduction half-reaction:
Oxidation half-reaction:
Overall reaction:
Heat creates a product with higher energy than both previous reactants.
5. is added to in water.
Strong acid dissociation:
Overall reaction:
This reaction works by exchange of electrons to yield Chromium ions.
6. is added to an aqueous solution of .
Overall reaction:
Iron hydroxide will precipitate because the two metals will exchange anions.

19.E.1.53: A19.1.19
a. Cr (SO ) (aq) + 2 Zn(s) + 2 H O (aq) ⟶ 2 Zn (aq) + H (g) + 2 H O(l) + 2 Cr (aq) + 3 SO (aq) ;
2 4 3 3
+ 2+

2 2
2+ 2−

b. 4 TiCl (s) + CrO (aq) + 8 H (aq) ⟶ 4 Ti (aq) + Cr(s) + 4 H O(l) + 12 Cl (aq) ;


3
2−
4
+ 4+

2

c. In acid solution between pH 2 and pH 6, CrO forms HrCO , which is in equilibrium with dichromate ion. The reaction is
2−
4

4

(aq) + H O(l) . At other acidic pHs, the reaction is


− 2−
2 HCrO (aq) ⟶ Cr O
4 2 7 2

(aq) + 12 H O(l) ;
2+ 2− + 3+
3 Cr (aq) + CrO (aq) + 8 H O (aq) ⟶ 4 Cr
4 3 2

Δ
d. 8 CrO 3
(s) + 9 Mn(s) ⟶ 4 Cr O (s) + 3 Mn O (s)
2 3 3 4
;

Access for free at OpenStax 19.E.15 https://chem.libretexts.org/@go/page/44104


e. CrO(s) + 2 H O (aq) + 2 NO (aq) ⟶ Cr (aq) + 2 NO (aq) + 3 H
3
+ −
3
2+ −
3 2
O(l) ;
f. CrCl (s) + 3 NaOH(aq) ⟶ Cr(OH) (s) + 3 Na (aq) + 3 Cl (aq)
3 3
+ −

19.E.1.54: Q19.1.20
What is the gas produced when iron(II) sulfide is treated with a nonoxidizing acid?

19.E.1.55: S19.1.20
Formula for iron(II) sulfide:

F eS (19.E.57)

Definition of non-oxidizing acid: A non-oxidizing acid is an acid that doesn't act the oxidizing agent. Its anion is a weaker oxidizing agent than
H+, thus it can't be reduced. Examples of non-oxidizing acids:

H C l, H I , H Br, H3 P O4 , H2 S O4 (19.E.58)

Step 2: Choose one of the non-oxidizing acid, in this case HCl, and write the chemical reaction:
F eS(s) + 2H C l(aq) → F eC l2 (s) + H2 S(g) (19.E.59)

The gas produced when iron (II) sulfide treated with a non-oxidizing acid, HCl, is H2S (dihydrogen sulfide) gas.

19.E.1.56: Q19.1.21
Predict the products of each of the following reactions and then balance the chemical equations.
a. Fe is heated in an atmosphere of steam.
b. NaOH is added to a solution of Fe(NO3)3.
c. FeSO4 is added to an acidic solution of KMnO4.
d. Fe is added to a dilute solution of H2SO4.
e. A solution of Fe(NO3)2 and HNO3 is allowed to stand in air.
f. FeCO3 is added to a solution of HClO4.
g. Fe is heated in air.

19.E.1.57: S19.1.21
a. Steam is water (H 2
O )
We can write out the reaction as:
Fe +H 2
O →?
This is a single replacement reaction, so Fe replaces H . So, one of the products is Fe
2 3
O
4
since it is a combination of iron(II) oxide, FeO, and
iron(III) oxide, Fe O .
2 3

The Fe is heated in an atmosphere of steam. H becomes neutrally charged and becomes another product.
2

After balancing the coefficients, the final reaction is:


3 Fe(s) +4H 2
O(g) → Fe 3
O (s)
4
+4H 2
(g)

b. NaOH added to a solution of Fe(NO 3


)
3
is a double replacement and precipitation reaction.
We can write out the reaction as:
NaOH + Fe(NO 3
)
3
→?
The Na and Fe switch to form Fe(OH) 3
(s) and NaNO 3
(aq) .
Fe(OH)
3
is solid because it is insoluble according to solubility rules.
After balancing the coefficients in the reaction, the final reaction is:
Fe(NO ) (aq)
3 3
+ 3 NaOH(aq) → Fe(OH) 3
(s) + NaNO 3
(aq)

c. For instance, the acid used to make the acidic solution is H 2


SO
4
, then the reaction is:
FeSO
4
+ KMnO + H 4 2
SO
4
→ Fe 2
(SO )
4 3
+ MnSO + H 4 2
O +K 2
SO
4

Next, the net ionic reaction has to be written to get rid of the spectator ions in the reaction, this is written as:
Fe
2 +
+ MnO + H → Fe

4
+ 3 +
+ Mn 2 +
+H 2
O

As seen in the net ionic equation above, Fe


2 +
is oxidized to Fe
3 +
and MnO

4
is reduced to Mn
2 +
. These can be written as two half
reactions:

Access for free at OpenStax 19.E.16 https://chem.libretexts.org/@go/page/44104


Fe
2 +
→ Fe 3 +

MnO

4
→ Mn 2 +

To balance the oxidation half reaction, one electron as to be added to the Fe 3 +


, this is shown as:
Fe
2 +
→ Fe 3 +
+e −

The reduction half reaction also has to be balanced, but with H ions and H +
2
O , this is shown as:
MnO

4
+ 8 H → Mn + 2 +
+4H 2
O

After the charge of the Mn atoms are balanced, the overall charge has to be balanced on both sides because on the reactants side, the charge is
7 +, and the charge on the products side is 2 +. The overall charge can be balanced by adding electrons, this is shown as:

MnO

4
+ 8 H + 5 e → Mn + − 2 +
+4H 2
O

Now since both half reactions are balanced, the electrons in both half reactions have to be equal, and then the half reactions are added together.
After this is done, the reaction looks like this:
MnO 4

+ 8 H + 5 Fe + 2 +
+ 5 e → Mn − 2 +
+4H 2
O + 5 Fe 3 +
+5e −

The 5 e on both sides cancel out and the final balanced reaction is:

MnO

4
+ 8 H + 5 Fe
+ 2 +
→Mn 2 +
+4H 2
O + 5 Fe 3 +

d. Fe added to a dilute solution of H 2


SO
4
is a single replacement reaction.
The Fe is added to a dilute solution so the H 2
SO
4
is written as separate ions.
We can write out the reaction as:
2 −
Fe(s) +2H +
(aq) + (SO 4
) (aq) →?
The Fe replaces the H ion, and becomes an Fe
+ 2 +
ion.
H O
2
is also a product because the solution is dilute.
Furthermore, the FeSO also has to be separated into ions as a result of the Fe being added to a dilute solution.
4

After balancing all of the coefficients, the final reaction is:


Fe(s) + (2 H 3
O)
+
(aq) + (SO 4
)
2 −
(aq) → Fe 2 +
(aq) + SO 2 −
4
(aq) +H 2
(g) +2H 2
O(l)

Note: H can also be written as the the hydronium ion, (H


+
3
O)
+
.
e. We initially can initially write out:
4 Fe(NO )
3 2
+ 4 HNO + O → ? 3 2

We write the oxygen term in the reactants because it is stated that the solution is allowed to stand in air.
We just have to analyze the possible products that can be formed and we can see that the hydrogen from nitric acid can combine with oxygen
gas to form water and then combining everything together, we get the final reaction to be:
4 Fe(NO ) (aq)
3 2
+ 4 HNO 3
(aq) +O 2
(g) →2H 2
O(l) + 4 Fe(NO 3
) (aq)
3

f. When FeCO is added to HClO , a double replacement reaction occurs.


3 4

The Fe 2 +
ion switches spots with the H ion to form Fe(ClO +

4
)
2
as a product.
When the H ion is added to the (CO
+
3
)
2 −
ion, H 2
CO
3
is formed.
After balancing the coefficients, the final reaction is:
FeCO (s)
3
+ HClO 4
(aq) → Fe(ClO 4
) (aq)
2
+H 2
O(l) + CO 2
(g)

g. Air is composed of oxygen gas, which is a diatomic molecule, so it is O . 2

Adding Fe to O will cause a synthesis reaction to occur forming Fe


2 2
O
3
.
After balancing coefficients, the final reaction is:
3 Fe(s) +2O 2
(g) → Fe 2
O (s)
3

19.E.1.58: A19.1.21
a. 3 Fe(s) + 4 H O(g) ⟶ Fe
2 3
O (s) + 4 H (g)
4 2
;
H2 O

b. 3 NaOH(aq) + Fe(NO 3
) (aq) −
3
−→ Fe(OH) (s) + 3 Na
3
+
(aq) + 3 NO3 (aq)

;

Access for free at OpenStax 19.E.17 https://chem.libretexts.org/@go/page/44104


c. MnO + 8 H + 5 Fe →Mn + 4 H O + 5 Fe

4
+ 2 + 2 +

2
3 +

+ 2 −
d. Fe(s) + (2 H O) (aq) + (SO ) (aq) → Fe (aq) + SO (aq) + H (g) + 2 H
3 4
2 + 2 −

4 2 2
O(l)

e. 4 Fe(NO ) (aq) + 4 HNO (aq) + O (g) → 2 H O(l) + 4 Fe(NO ) (aq)


3 2 3 2 2 3 3

f. FeCO (s) + HClO (aq) → Fe(ClO ) (aq) + H O(l) + CO (g)


3 4 4 2 2 2

g. 3 Fe(s) + 2 O (g) → Fe O (s)


2 2 3

19.E.1.59: Q19.1.22
Balance the following equations by oxidation-reduction methods; note that three elements change oxidation state.
Co (NO ) (s) ⟶ Co O (s) + NO (g) + O (g) (19.E.60)
3 2 2 3 2 2

19.E.1.60: S19.1.22
Balance the following equations by oxidation-reduction methods; note that three elements change oxidation state.

C o(N O3 )2 (s) ⟶ C o2 O3 (s) + N O2 (g) + O2 (g) (19.E.61)

In this reaction, N changes oxidation states from +5 to +4 (reduced), Co changes oxidation states from +2 to +3 (oxidized), and O changes
oxidation states from -2 to 0 (also oxidized).
First, split this reaction into an oxidation and reduction half reaction set, and balance all of the elements that are not hydrogen or oxygen (we
will deal with these later):
Reduction : 2C o(N O3 )2 → C o2 O3 + 4N O2 (19.E.62)

Now, for the oxidation reaction, we are only dealing with O2 on the products side. In order to balance this, we will need to add water and
hydrogen to both sides:
+
Oxidation : 2 H2 O → O2 + 4 H (19.E.63)

Balance the amount of oxygens on each side by adding the correct number of water molecules (H2O), and balance the amount of hydrogen by
adding the correct number of H+ atoms:
+
Reduction : 2 H + 2C o(N O3 )2 → C o2 O3 + 4N O2 + H2 O (19.E.64)

+
Oxidation : 2 H2 O → O2 + 4 H (19.E.65)

Finally, balance the charges by adding electrons to each side of the equation. For the reduction reaction, we will add 2 electrons to balance out
the 2H+, and to the oxidation reaction, we will add 4 electrons to balance out the 4H+. Remember, the goal of this step is to make sure that the
charges are balanced, so we can cancel them out in the end.
− +
Reduction : 2 e + 2H + 2C o(N O3 )2 → C o2 O3 + 4N O2 + H2 O (19.E.66)

+ −
Oxidation : 2 H2 O → O2 + 4 H + 4e (19.E.67)

Multiply the reduction reaction by two, in order to balance the charges so there are 4 electrons on each side of the reaction.
− +
Reduction : 2(2 e + 2H + 2C o(N O3 )2 → C o2 O3 + 4N O2 + H2 O) (19.E.68)

and combine both reactions which comes out to:


+ +
2 H2 O + 4C o(N O3 )2 + 4 H → 2C O2 O3 + 8N O2 + 2 H2 O + O2 + 4 H (19.E.69)

Cancel out like terms:


4C o(N O3 )2 (s) → 2C O2 O3 (s) + 8N O2 (g) + O2 (g) (19.E.70)

Both sides have overall charges of 0 and can be checked to see if they are balanced.

19.E.1.61: A19.1.22
4C o(N O3 )2 (s) → 2C O2 O3 (s) + 8N O2 (g) + O2 (g) (19.E.71)

19.E.1.62: Q19.1.23
Dilute sodium cyanide solution is slowly dripped into a slowly stirred silver nitrate solution. A white precipitate forms temporarily but
dissolves as the addition of sodium cyanide continues. Use chemical equations to explain this observation. Silver cyanide is similar to silver
chloride in its solubility.

Access for free at OpenStax 19.E.18 https://chem.libretexts.org/@go/page/44104


19.E.1.63: S19.1.23
Dilute sodium cyanide solution is slowly dripped into a slowly stirred silver nitrate solution. A white precipitate forms temporarily but
dissolves as the addition of sodium cyanide continues. Use chemical equations to explain this observation. Silver cyanide is similar to silver
chloride in its solubility.
A: Step 1: look at the question and begin to write out a general product to reactant formula for this reaction.
Step 2: try to reason out why a precipitate will form but only for a finite period of time before reforming in an aqueous substance.
Step 3: With step 2 you should have noticed that the reaction is a multiple step reaction and using the rough formula that you derived in
step 1, you should try and see what the series of steps are that lead to the overall product of liquid AgCN2
In this reaction we see how NaCN is added to AgNO3 .A precipitate forms but then disappears with the addition of even more NaCN, this
must mean that its an intermediate reaction which will not appear as the final product. The silver and the cyanide temporarily bond, but
the bond is too weak to hold them together so they are pulled apart again when NaCN is added because a new, more stronger and stable
compound is formed: [Ag(CN)2]- (aq).
The actual reaction equation when it is first taking place is

AgC l(aq) + N aC N (aq) → AgC N (s) + N aC l(aq) (19.E.72)

− + −
This can be written out in the following way: as CN is added, the silver and the cyanide combine : Ag (aq)+CN (aq)→AgCN (s)
As more CN- is added the silver and two cyanide combine to create a more stable compound: Ag+(aq)+2CN−(aq)→[Ag(CN)2]- (aq)
AgCN(s) + CN- (aq) → [Ag(CN)2]- (aq)

19.E.1.64: A19.1.23
As CN− is added,
+ −
Ag (aq) + CN (aq) ⟶ AgCN(s) (19.E.73)


As more CN is added,
+ − −
Ag (aq) + 2 CN (aq) ⟶ [Ag (CN) ] (aq) (19.E.74)
2

− −
AgCN(s) + CN (aq) ⟶ [Ag (CN) ] (aq) (19.E.75)
2

19.E.1.65: Q19.1.24
Predict which will be more stable, [CrO4]2− or [WO4]2−, and explain.

19.E.1.66: S19.1.24
According to the rules associated with Crystal Field Stabilizing Energies, stable molecules contain more electrons in the lower-energy
molecular orbitals than in the high-energy molecular orbitals. In this case, both complexes have O4 as ligands, and both have a -2 charge.
Therefore, you determine stability by comparing the metals. Chromium is in the 3d orbital, according to the periodic table. Tungsten (W) is in
the 5d orbital. 3d is a lower energy level than 5d.Higher-level orbitals are more easily ionized, and make their base elemental form more stable.
If the elemental form is more stable the oxidized form is less stable. Therefore, [CrO4]2− is more stable than [WO4]2−.

19.E.1.67: A19.1.24
[CrO4]2- is more stable because Chromium is in the 3d orbital while Tungsten is in the 4d orbital, which has a higher energy level and makes it
less stable.

19.E.1.68: Q19.1.25
Give the oxidation state of the metal for each of the following oxides of the first transition series. (Hint: Oxides of formula M3O4 are examples
of mixed valence compounds in which the metal ion is present in more than one oxidation state. It is possible to write these compound formulas
in the equivalent format MO·M2O3, to permit estimation of the metal’s two oxidation states.)
a. Sc2O3
b. TiO2
c. V2O5
d. CrO3
e. MnO2
f. Fe3O4
g. Co3O4
h. NiO

Access for free at OpenStax 19.E.19 https://chem.libretexts.org/@go/page/44104


i. Cu2O

19.E.1.69: S19.1.25
The first step to solving this problem is looking at the rules of Oxidizing states for various elements:
chem.libretexts.org/Core/Analytical_Chemistry/Electrochemistry/Redox_Chemistry/Oxidation_State
The main rules that will be used in these problems will be the oxidation state rule 6 which states that oxidation state for Oxygen is (-2) and rule
2 which is that the total sum of the oxidation state of all atoms in any given species is equal to the net charge on that species. Solving these
problems requires simple algebra. The oxidation states of both elements in the compound is equal to zero, so set the unknown oxidation of the
element that is not oxygen to a variable x, and the oxidation state of Oxygen equal to −2. Then multiply both oxygen states by the number of
atoms of the element present. Add the values together, set the equation equal to zero and solve for x.
1. Sc O = 3(−2) + 2x = 0 ⟶ −6 + 2x = 0 ⟶ x = Sc = +3 Sc
2 3
3+

2. TiO = 2(−2) + x = 0 ⟶ −4 + x = 0 ⟶ x = T i = +4 T i
2
4+

3. V O = 5(−2) + 2x = 0 ⟶ −10 + 2x = 0 ⟶ x = V = +5 V
2 5
5+

4. CrO = 3(−2) + x = 0 ⟶ −6 + x = 0 ⟶ x = C r = +6 C r
3
6+

5. MnO = 2(−2) + x = 0 ⟶ −4 + x = 0 ⟶ x = M n = +4 M n
2
4+

6. Fe O = FeO ⋅ Fe O =
3 4 2 3

2+
FeO = −2 + x = 0 ⟶ x = F e = +2 Fe

3=
Fe O = 3(−2) + 2x = 0 ⟶ −6 + 2x = 0 ⟶ x = F e = +3 Fe
2 3

(One Fe Atom has an oxidation state of +2 and the other 2 Fe atoms have an oxidation state of +3)
7. Co 3
O
4
= CoO ⋅ Co O
2 3
=

2+
CoO = −2 + x = 0 ⟶ x = C o = +2 Co

3+
Co O = 3(−2) + 2x = 0 ⟶ −6 + 2x = 0 ⟶ x = C o = +3 Co
2 3

(One Co Atom has an oxidation state of +2 and the other 2 Co atoms have an oxidation state of +3)
8. NiO = −2 + x = 0 ⟶ x = N i = +2 Ni
2+

9. Cu 2
O = −2 + 2x = 0 ⟶ −2 + 2x = 0 ⟶ x = C u = +1 Cu
1+

19.E.1.70: A19.1.25
Sc3+; Ti4+; V5+; Cr6+; Mn4+; Fe2+ and Fe3+; Co2+ and Co3+; Ni2+; Cu+

19.E.2: 19.2: Coordination Chemistry of Transition Metals


19.E.2.1: Q19.2.1
Indicate the coordination number for the central metal atom in each of the following coordination compounds:
a. [Pt(H2O)2Br2]
b. [Pt(NH3)(py)(Cl)(Br)] (py = pyridine, C5H5N)
c. [Zn(NH3)2Cl2]
d. [Zn(NH3)(py)(Cl)(Br)]
e. [Ni(H2O)4Cl2]
f. [Fe(en)2(CN)2]+ (en = ethylenediamine, C2H8N2)

19.E.2.2: S19.2.1
First we must identify whether or not the ligand has more than one bonded atom (bidentate/polydentate). Using the table below we are able to
do this.

Ligand Number of bonded atoms

Ammine (NH3) monodentate

Aqua (H2O) monodentate

Bromo (Br) monodentate

Chloro (Cl) monodentate

Cyano (CN) monodentate

Pyridine (C5H5N) monodentate

Access for free at OpenStax 19.E.20 https://chem.libretexts.org/@go/page/44104


Ligand Number of bonded atoms

Ethylenediamine (C2H8N2) bidentate

Now that we have identified the number of bonded atoms from each ligand, we can find the total number of atoms bonded to the central metal
ion, giving us the coordination number.
a. : We can identify the metal ion in the complex as Pt, platinum, as the other two are listed as ligands above and are
[P t(H2 O)2 Br2 ]

nonmetallic. We can now use the number of ligands and their bonding atoms to find its coordination number. From the table above we see
that H2O has only one bonding atom and Br as well. So for each Br atom we have one bonding atom, and we have two of these, yielding 2
bonding atoms; this is the same for H2O, giving us a total number of 4 bonding atoms, and therefore a coordination number of 4.
b. [P t(N H )(py)(C l)(Br)] (py = pyridine, C5H5N): The metal ion in this complex, similarly to the first one, can be identified as Pt,
3

platinum. The ligands can be identified as NH3, pyridine, Cl, and Br, which are all monodentate ligands and have one bonding atom each.
Since we have four ligands, each with one bonding atom, the total number of bonding atoms on the metal ion is 4, therefore the complex has
a coordination number of 4.
c. [Zn(N H ) C l ]: The metal ion in this complex can be identified as Zn, zinc, and the ligands can be identified as NH3 and Cl. Since these
3 2 2

two are both monodentate ligands they have one bonding atom each. Since we have a total of two NH3 and two Cl ligands, we get a total of
four monodentate ligands, giving us 4 bonding atoms and a coordination number of 4.
d. [Zn(N H )(py)(C l)(Br)]: The metal ion in this complex can be identified as Zn, zinc, and the ligands can be identified as NH3, pyridine,
3

Cl, and Br, which are all monodentate ligands and have one bonding atom each. Since we have four ligands, each with one bonding atom,
the total number of bonding atoms on the metal ion is 4, therefore the complex has a coordination number of 4.
e. [N i(H O) C l ]: The metal ion in this complex can be identified as Ni, nickel, and we can now use the number of ligands and their
2 4 2

bonding atoms to find its coordination number. From the table above we see that H2O has only one bonding atom and Cl as well. So for
each Cl atom we have one bonding atom, and we have two of these, yielding 2 bonding atoms. H2O is the same, having only one bonding
atom, but there are four of these. So this gives us a total number of 6 bonding atoms, and therefore a coordination number of 6.
f. [F e(en) (C N ) ] (en = ethylenediamine, C2H8N2): The metal ion in this complex can be identified as Fe, iron, and the ligands can be
2 2
+

identified as (en) and CN. Since (en) is bidentate, meaning it has 2 bonding atoms, and there are two of these, the total number of bonding
atoms from (en) is four. Since CN is monodentate, meaning it has one bonding atom, and there are two of these, the total number of bonding
atoms from CN ligand is two. So, the total number of bonding atoms is 6, therefore the complex has a coordination number of 6.

19.E.2.3: A19.2.1
a. The 2 aqua and the 2 bromo ligands form a total of 4 coordinate covalent bonds and as a result the coordination number is 4.
b. The ammine, pyridine, chloro and bromo each form one coordinate covalent bond that gives a total of 4 and hence CN=4.
c. Two ammine and two chloro ligands give a total of 4 coordinate covalent bonds and a CN = 4.
d. One ammine, a pyrimidine, a chloro and a bromo ligand give a total of 4 covalent bonds, resulting in CN = 4.
e. Four aqua ligands and two chloro ligands form a total of 6 coordinate covalent bonds and a CN =6.
f. Ethylenediamine is a bidentate ligand that forms two coordinate covalent bonds; along with two cyano ligands, it forms a total of 6
bonds, and hence has a CN=6.

19.E.2.4: Q19.2.2
Give the coordination numbers and write the formulas for each of the following, including all isomers where appropriate:
a. tetrahydroxozincate(II) ion (tetrahedral)
b. hexacyanopalladate(IV) ion
c. dichloroaurate ion (note that aurum is Latin for "gold")
d. diamminedichloroplatinum(II)
e. potassium diamminetetrachlorochromate(III)
f. hexaamminecobalt(III) hexacyanochromate(III)
g. dibromobis(ethylenediamine) cobalt(III) nitrate

19.E.2.5: S19.2.2
To determine coordination numbers we must count the total number of ligands bonded to the central metal and distinguish monodentate and
polydentate ligands. To determine the formulas, we use the nomenclature rules and work backwards.
1. "tetrahydroxo" = 4 hydroxide ligands; since hydroxide is a monodentate ligand, we have a total of 4 bonds to the central metal.
Coordination Number: 4
We review the basics of nomenclature and see that "tetra" = 4 and "hydroxo" = OH-. Since the charge on zinc is 2+, which is given in the
nomenclature by the Roman numerals, we can calculate the total charge on the complex to be 2-.
Formula: [Zn(OH)4]2−

Access for free at OpenStax 19.E.21 https://chem.libretexts.org/@go/page/44104


2. "hexacyano" = 6 cyanide ligands; since cyanide is a monodentate ligand, we have a total of 6 bonds to the central metal.
Coordination Number: 6
We review the basics of nomenclature and see that "hexa" = 6 and "cyano" = CN-. Since the charge on Pd is 4+, which is given in the
nomenclature by the Roman numerals, we can calculate the total charge on the complex to be 2-.
Formula: [Pd(CN)6]2−
3. "dichloro" = 2 chloride ligands; since chloride is a monodentate ligand, we have a total of 2 bonds to the central metal.
Coordination Number: 2
We review the basics of nomenclature and see that "di" = 2 and "chloro" = Cl-. Since the charge on Au is always 1+, we can calculate the
total charge on the complex to be 1-.
Formula: [AuCl2]−
4. "diammine" = 2 ammonia ligands and "dichloro" = 2 chloride ligands; since both ammonia and chloride ligands are monodentate, we have a
total of 4 bonds to the central metal.
Coordination Number: 4
We review the basics of nomenclature and see that "di" = 2, "chloro" = Cl- and "ammine" = NH3. Since the charge on Pt is 2+, which is
given in the nomenclature by the Roman numerals, we can calculate that the total charge is 0, so the complex is neutral.
Formula: [Pt(NH3)2Cl2]
5. "diammine" = 2 ammonia ligands and "tetrachloro" = 4 chloride ligands; since both ammonia and chloride ligands are monodentate, we
have a total of 6 bonds to the central metal.
Coordination Number: 6
We review the basics of nomenclature and see that "di" = 2, "ammine" = NH3, "tetra" = 4 and "chloro" = Cl-. Since the charge on the central
metal, Cr, is 3+, which is given in the nomenclature by the Roman numerals, we can calculate that the total charge of the complex is 1-. The
"potassium" at the front of the nomenclature indicates that it is the corresponding cation to this anionic complex.
Formula: K[Cr(NH3)2Cl4]
6. Both of the metal complexes have "hexa" monodentate ligands, which means both have coordination numbers of 6.
Coordination Number: 6
We review the basics of nomenclature and see that "hexa" = 6, "ammine" = NH3, and "cyano" = CN-. Since the charge on Cr is 3+ and Co is
3+, which is given in the nomenclature by the Roman numerals, we find that these complexes' charges balance out.
Formula: [Co(NH3)6][Cr(CN)6]
7. "dibromo" = 2 bromide ligands "bis(ethylenediamine)" = 2 (en) ligands; bromide is a monodentate ligand while (en) is a bidentate ligand.
Therefore, we have a coordination number of 6.
Coordination Number: 6
We review the basics of nomenclature and see that "di" = 2, "bromo" = Br-, "bis" = 2, and "ethylenediamine" = en. Since the charge on Co is
3+, which is given in the nomenclature by the Roman numerals, we find that the total charge of the complex is 1+. Nitrate is the
corresponding anion to this cationic complex.
Formula: [Co(en)2Br2]NO3

19.E.2.6: A19.2.2
a. 4, [Zn(OH)4]2−;
b. 6, [Pd(CN)6]2−;
c. 2, [AuCl2]−;
d. 4, [Pt(NH3)2Cl2];
e. 6, K[Cr(NH3)2Cl4];
f. 6, [Co(NH3)6][Cr(CN)6];
g. 6, [Co(en)2Br2]NO3

19.E.2.7: Q19.2.3
Give the coordination number for each metal ion in the following compounds:
a. [Co(CO3)3]3− (note that CO32− is bidentate in this complex)
b. [Cu(NH3)4]2+
c. [Co(NH3)4Br2]2(SO4)3
d. [Pt(NH3)4][PtCl4]
e. [Cr(en)3](NO3)3
f. [Pd(NH3)2Br2] (square planar)
g. K3[Cu(Cl)5]
h. [Zn(NH3)2Cl2]

Access for free at OpenStax 19.E.22 https://chem.libretexts.org/@go/page/44104


19.E.2.8: S19.2.3
You can determine a compound's coordination number based on how many ligands are bound to the central atom.
1) In this compound, Cobalt is the central atom, and it has 3 CO32- molecules attached to it. However, CO32- is a bidentate ligand, which means
it binds to the central atom in two places rather than one. This means that the coordination number of [Co(CO3)3]3- is 6. A coordination number
of 6 means that the structure is most likely octahedral.
2) In this compound, Copper is the central atom. 4 ammonia molecules are attached to it. This means the coordination number is 4, and the
structure is likely tetrahedral.
3) For this compound, we can ignore the (SO4)3 because it is not bound to the central atom. The central atom is cobalt, and it has 4 ammonia
molecules and 2 bromine molecules bound to it. The coordination number is 6.
4) There are two compounds here, indicated by the brackets. The central atom for both is platinum. One of them has 4 ammonia molecules
attached, and the other has 4 chlorine atoms attached. Both complexes have a coordination number of 4.
5) We can ignore (NO3)3 for this compound. The central atom is Chromium. There are 3 ethylenediamine molecules attached to the chromium.
Ethylenediamine is a bidentate ligand, so the coordination number is 6.
6) Palladium is the central atom. 2 ammonia molecules and 2 bromine atoms are bound to the palladium atom. The coordination number is 4.
7) We can ignore the K3 structure. Copper is the central atom, and there are 5 chlorine molecules attached to it. The coordination number is 5,
so the structure is either trigonal bipyramidal or square pyramidal.
8) In this compound, zinc is the central atom. There are 2 ammonia molecules and 2 chlorine atoms attached. This means that the coordination
number is 4.

Q19.2.4
Sketch the structures of the following complexes. Indicate any cis, trans, and optical isomers.
a. [Pt(H2O)2Br2] (square planar)
b. [Pt(NH3)(py)(Cl)(Br)] (square planar, py = pyridine, C5H5N)
c. [Zn(NH3)3Cl]+ (tetrahedral)
d. [Pt(NH3)3Cl]+ (square planar)
e. [Ni(H2O)4Cl2]
f. [Co(C2O4)2Cl2]3− (note that C O is the bidentate oxalate ion, −O
2
2−
4 2

CCO
2
)

19.E.2.9: S19.2.4
Cis and trans are a type of geometric isomer, meaning there is a difference in the orientation in which the ligands are attached to the central
metal. In cis, two of the same ligands are adjacent to one another and in trans, two of the same ligands are directly across from one another.
Optical isomers → have the ability to rotate light, optical isomers are also chiral. Only chiral complexes have optical isomers
Chiral → asymmetric, structure of its mirror image is not superimposable
Enantiomers: chiral optical isomers (compound can have multiple enantiomers)
Tetrahedral complex with 4 distinct ligands → always chiral
For tetrahedral, if 2 ligands are the same, then it cannot be chiral, has a plane of symmetry
Solutions:
a. [P t(H2 O)2 Br2 ] (square planar)
This complex has 2 kinds of ligands. The matching ligands can either be adjacent to each other and be cis, or they can be across from each
other and be trans.
b. [P t(N H 3 )(py)(C l)(Br)] (square planar, py = pyridine, C 5 H5 N )
This complex has 4 different ligands. There is no plane of symmetry in any of the enantiomers, making the structures chiral and therefore has
optical isomers.
c. [Zn(N H 3 )3 C l]
+
(tetrahedral)
There is a plane of symmetry from N H through Zn to the other N H , therefore it is not chiral.
3 3

d. [P t(N H +
3 )3 C l] (square planar)
There is a plane of symmetry from N H through P t to the other N H , therefore it is not chiral.
3 3

Access for free at OpenStax 19.E.23 https://chem.libretexts.org/@go/page/44104


e. [N i(H 2 O)2 C l2 ]

The C l ligands can either be right next to each other, or directly across from one another allowing for both cis and trans geometries.
f. [C o(C2 O4 )2 C l2 ]

3 (note that C
2 O4

2 is the bidentate oxalate ion, −
O2 C C O

There is a plane of symmetry from C l through Co to the other C l in a "trans" chlorine configuration, therefore it is not chiral in a chlorine
"trans" configuration. However, there is no symmetry in the chlorine "cis" configuration, indicating multiple "cis" isomers.

19.E.2.10: A19.2.4
a. [Pt(H2O)2Br2]:

b. [Pt(NH3)(py)(Cl)(Br)]:

c. [Zn(NH3)3Cl]+ :

d. [Pt(NH3)3Cl]+ :

e. [Ni(H2O)4Cl2]:

f. [Co(C2O4)2Cl2]3−:

Access for free at OpenStax 19.E.24 https://chem.libretexts.org/@go/page/44104


19.E.2.11: Q19.2.5
Draw diagrams for any cis, trans, and optical isomers that could exist for the following (en is ethylenediamine):
a. [Co(en)2(NO2)Cl]+
b. [Co(en)2Cl2]+
c. [Pt(NH3)2Cl4]
d. [Cr(en)3]3+
e. [Pt(NH3)2Cl2]

19.E.2.12: S19.2.5
We are instructed to draw all geometric isomers and optical isomers for the specified compound. Optical isomers exist when an isomer
configuration is not superimposable on its mirror image. This means there are two distinct molecular shapes. Often a left and right hand are
cited as an example; if you were to take your right hand and place it upon your left, you cannot make the major parts of your hand align on top
of one another. The basic idea when deciding whether something is optically active is to look for a plane of symmetry--if you are able to bisect
a compound in a manner that establishes symmetry, then the compound does not have an optical isomer.
Cis isomers exist when there are 2 ligands of the same species placed at 90 degree angles from each other. Trans isomers exist when there are 2
ligands of the same species placed at 180 degree angles from each other.
Problem 1
This compound is an octahedral molecule, so the six ligands (atoms in the complex that are not the central transition metal) are placed around
the central atom at 90 degree angles. Two optical isomers exist for [Co(en)2(NO2)Cl]+. The second isomer is drawn by taking the mirror image
of the first.

Problem 2
This compound is also an octahedral molecule. Two cis (optical) isomers and one trans isomer exist for [Co(en)2Cl2]+. The trans isomer can be
drawn by placing the chlorine ligands in positions where they form a 180 degree angle with the central atom. The first cis isomer can be drawn
by placing the chlorine ligands in positions where they form a 90 degree angle with the central atom. The second cis isomer can be found by
mirroring the first cis isomer, like we did in problem 1.

Problem 3
This compound is also an octahedral molecule. One trans isomer and one cis isomer of [Pt(NH3)2Cl4] exist. The trans isomer can be drawn by
placing the ammonia ligands in positions where they form a 180 degree angle with the central atom. The cis isomer can be drawn by placing
the ammonia ligands in positions where they form a 90 degree angle with the central atom.

Problem 4
This compound is also an octahedral molecule. Two optical isomers for [Cr(en)3]3+ exist. The second optical isomer can be drawn by taking the
mirror image of the first optical isomer.

Access for free at OpenStax 19.E.25 https://chem.libretexts.org/@go/page/44104


Problem 5
This compound is a square planar complex, so the ligands are placed around the central atom in a plane, at 90 angles. A trans isomer and a cis
isomer exist for the complex [Pt(NH3)2Cl2]. The trans isomer can be drawn by placing the ammonia ligands in positions where they form a 180
degree angle in the plane with the central atom. The cis isomer can be drawn by placing the ammonia ligands in positions where they form a 90
degree angle in the plane with the central atom.

19.E.2.13: Q19.2.6
Name each of the compounds or ions given in Exercise Q19.2.3, including the oxidation state of the metal.

19.E.2.14: S19.2.6
Rules to follow for coordination complexes
1. Cations are always named before the anions.
2. Ligands are named before the metal atom or ion.
3. Ligand names are modified with an ‐o added to the root name of an anion. For neutral ligands the name of the molecule is used, with the
exception of OH2, NH3, CO and NO.
4. The prefixes mono‐, di‐, tri‐, tetra‐, penta‐, and hexa‐ are used to denote the number of simple ligands.
5. The prefixes bis‐, tris‐, tetrakis‐, etc., are used for more complicated ligands or ones that already contain di‐, tri‐, etc.
6. The oxidation state of the central metal ion is designated by a Roman numeral in parentheses.
7. When more than one type of ligand is present, they are named alphabetically. Prefixes do not affect the order.
8. If the complex ion has a negative charge, the suffix –ate is added to the name of the metal.
9. In the case of complex‐ion isomerism the names cis, trans, fac, or mer may precede the formula of the complex‐ion name to indicate the
spatial arrangement of the ligands. Cis means the ligands occupy adjacent coordination positions, and trans means opposite positions just as
they do for organic compounds. The complexity of octahedral complexes allows for two additional geometric isomers that are peculiar to
coordination complexes. Fac means facial, or that the three like ligands occupy the vertices of one face of the octahedron. Mer means
meridional, or that the three like ligands occupy the vertices of a triangle one side of which includes the central metal atom or ion.

19.E.2.15: A19.2.6
a. tricarbonatocobaltate(III) ion;
b. tetraaminecopper(II) ion;
c. tetraaminedibromocobalt(III) sulfate;
d. tetraamineplatinum(II) tetrachloroplatinate(II);
e. tris-(ethylenediamine)chromium(III) nitrate;
f. diaminedibromopalladium(II);
g. potassium pentachlorocuprate(II);
h. diaminedichlorozinc(II)

19.E.2.16: Q19.2.7
Name each of the compounds or ions given in Exercise Q19.2.5.
S19.2.7
Given:

Access for free at OpenStax 19.E.26 https://chem.libretexts.org/@go/page/44104


1. [Co(en)2(NO2)Cl]+
2. [Co(en)2Cl2]+
3. [Pt(NH3)2Cl4]
4. [Cr(en)3]3+
5. [Pt(NH3)2Cl2]
Wanted:
Names of the above compounds.
1. [Co(en)2(NO2)Cl]+
Step 1: Attain the names of the ligands and metal cation. Names can be found here.
Co: Cobalt
en: Ethylenediamine
NO2: Nitro
Cl: Chloro
Step 2: Add the appropriate pre-fixes to each ligand depending on the number. Pre-fixes can be found here.
(en)2: bis(Ethylenediamine)
Step 3: Find the charges of the ligands. Charges can be found here.
en: 0
NO2: -1
Cl: -1
Step 4: Algebraically attain the charge of the metal cation using the overall charge of the complex ion and the individual ligand charges.
Co + 2(en) + (NO2) + Cl = 1
Co +2(0) + (-1) + (-1) = 1
Co = 3
Step 5: For the name alphabetically place the ligands, pre-fixes should not be accounted, and use roman numerals for the metal cation which
should be placed last.
Chlorobis(ethylenediamine)nitrocobalt(III)
2. [Co(en)2Cl2]+
Step 1: Attain the names of the ligands and metal cation. Names can be found here.
Co: Cobalt
en: Ethylenediamine
Cl: Chloro
Step 2: Add the appropriate pre-fixes to each ligand depending on the number. Pre-fixes can be found here.
(en)2: bis(Ethylenediamine)
Cl2: dichloro
Step 3: Find the charges of the ligands. Charges can be found here.
en: 0
Cl: -1
Step 4: Algebraically attain the charge of the metal cation using the overall charge of the complex ion and the individual ligand charges.
Co + 2(en) +2(Cl) = 1
Co + 2(0) + 2(-1) = 1
Co = 3
Step 5: For the name alphabetically place the ligands, pre-fixes should not be accounted, and use roman numerals for the metal cation which
should be placed last.

Access for free at OpenStax 19.E.27 https://chem.libretexts.org/@go/page/44104


Dichlorobis(Ethylenediamine)cobalt(III)
3. [Pt(NH3)2Cl4]
Step 1: Attain the names of the ligands and metal cation. Names can be found here.
Pt: Platinum
NH3: Ammine
Cl: Chloro
Step 2: Add the appropriate pre-fixes to each ligand depending on the number. Pre-fixes can be found here.
(NH3)2: diammine
Cl4: tetrachloro
Step 3: Find the charges of the ligands. Charges can be found here.
NH3: 0
Cl: -1
Step 4: Algebraically attain the charge of the metal cation using the overall charge of the complex ion and the individual ligand charges.
Pt + 2(NH3) + 4(Cl) = 0
Pt + 2(0) + 4(-1) = 0
Pt = 4
Step 5: For the name alphabetically place the ligands, pre-fixes should not be accounted, and use roman numerals for the metal cation which
should be placed last.
Diamminetetrachloroplatinum(IV)
4. [Cr(en)3]3+
Step 1: Attain the names of the ligands and metal cation. Names can be found here.
Cr: Cromium
en: ethylenediamine
Step 2: Add the appropriate pre-fixes to each ligand depending on the number. Pre-fixes can be found here.
(en)3: tris(ethylenediamine)
Step 3: Find the charges of the ligands. Charges can be found here.
en: 0
Step 4: Algebraically attain the charge of the metal cation using the overall charge of the complex ion and the individual ligand charges.
Cr + 3(en) = 3
Cr + 3(0) = 3
Cr = 3
Step 5: For the name alphabetically place the ligands, pre-fixes should not be accounted, and use roman numerals for the metal cation which
should be placed last.
Tris(ethylenediamine)cromium(III)
5. [Pt(NH3)2Cl2]
Step 1: Attain the names of the ligands and metal cation. Names can be found here.
NH3: Ammine
Cl: Chloro
Pt: Platinum
Step 2: Add the appropriate pre-fixes to each ligand depending on the number.
(NH3)2: diammine
Cl2: dichloro

Access for free at OpenStax 19.E.28 https://chem.libretexts.org/@go/page/44104


Step 3: Find the charges of the ligands. Charges can be found here.
NH3: 0
Cl: -1
Step 4: Algebraically attain the charge of the metal cation using the overall charge of the complex ion and the individual ligand charges.
Pt + 2(NH3) + 2(Cl) = 0
Pt + 2(0) + 2(-1) = 0
Pt = 2
Step 5: For the name alphabetically place the ligands, pre-fixes should not be accounted, and use roman numerals for the metal cation which
should be placed last.
Diamminedichloroplatinum(II)

19.E.2.17: A19.2.7
1. Chlorobis(ethylenediamine)nitrocobalt(III)
2. Dichlorobis(Ethylenediamine)cobalt(III)
3. Diamminetetrachloroplatinum(IV)
4. Tris(ethylenediamine)cromium(III)
5. Diamminedichloroplatinum(II)

19.E.2.18: Q19.2.8
Specify whether the following complexes have isomers.
a. tetrahedral [Ni(CO)2(Cl)2]
b. trigonal bipyramidal [Mn(CO)4NO]
c. [Pt(en)2Cl2]Cl2

19.E.2.19: S19.2.8
Isomers are compounds that have the same number of atoms, but have different structures. Structural isomers (linkage, ionization,
coordination) and stereoisomers (geometric and optical) can occur with several compounds.
1. tetrahedral [Ni(CO) 2 (Cl )2 ]

(Fig 1.) In this model, nickel is the dark green central atom, carbonyl ligands are the pink atom, and chloro ligands are the light green atoms.
Immediately, we can cancel out the possibility of linkage, ionization, and coordination isomers. There are no other coordination complexes for
coordination isomerism, there is no ligand that can bond to the atom in more than one way for it to exhibit linkage isomerism, and there are no
ions outside the coordination sphere for ionization isomerism.
This is a tetrahedral structure which immediately rules out any geometric isomers since they require 90° and/or 180° bond angles. Tetrahedral
structures have 109.5° angles.
To confirm that the structure has no optical isomer, we must determine if there is a plane of symmetry. Structures that have no plane of
symmetry are considered chiral and would have optical isomers.

(Fig 2.) We can rotate the structure and find that there is indeed a plane of symmetry through the two chloro ligands and central atom and
between the carbonyl ligands.
Since there is a plane of symmetry, we can conclude that there are no optical isomers.

Access for free at OpenStax 19.E.29 https://chem.libretexts.org/@go/page/44104


Overall, there are no isomers that exist for this compound.

2. trigonal byprimidal [Mn(CO) 4 (NO)]

(Fig 3.) The central purple atom is manganese, the carbonyl ligands are the pink atoms, and the nitrosyl ligand is the fuschia atom.
There are no ions, other coordination complex, and ambidentate ligands. Therefore, no structural isomers exist for this structure.
Geometric isomers do not exist for this compound because there is only one nitrosyl ligand.

(Fig 4.) Dashed line bisects molecule and shows plane of symmetry. The molecule is rotated in this image.
In the image above, after the structure has been rotated, we can see that there is a plane of symmetry. Thus, there are no optical isomers.
No isomers (the ones mentioned above) exist for this compound.

3. [Pt(en)2 C l2 ]C l2

(Fig 5.) The green atoms are the chloro ligands, the the central atom is platinum, and the grey/blue atoms are ethyldiamine ligands.
Coordination isomerism cannot exist for this complex because there are no other complexes. There are no linkage isomers because there are no
ambidentate ligands. Ionization isomers cannot exist in this complex either, even though there is a neutral molecule outside the coordination
sphere. If we exchange Cl with one ethyldiamine molecule, There would be 5 ligands in the coordination sphere instead of 4. This difference
2

in the ratio of metal atom to ligands means that an ionization isomer cannot exist.

(Fig 6.) Here, one chloro ligand exchanged places with the ethyldiamine so that it can be at a 90° angle with the other chloro ligand.
The image above, shows the chloro and ethyldiamine ligands at a 90° angle with its other identical ligand. This is the cis isomer, while Fig. 5
shows the trans isomer.
Fig 5. shows that there is a plane of symmetry in the trans isomer. Therefore, that structure does not have an optical isomer. On the other hand,
the cis isomer does not have a plane of symmetry and therefore has an optical isomer.

Access for free at OpenStax 19.E.30 https://chem.libretexts.org/@go/page/44104


19.E.2.20: A19.2.8
none; none; The two Cl ligands can be cis or trans. When they are cis, there will also be an optical isomer.

19.E.2.21: Q19.2.9
Predict whether the carbonate ligand CO 2−

3
will coordinate to a metal center as a monodentate, bidentate, or tridentate ligand.

19.E.2.22: S19.2.9

2
CO3 − can be either monodentate or bidentate, since two of its oxygen atoms have lone pairs as shown above and can form covalent bonds
with a transition metal ion. In most cases carbonate is monodentate because of its trigonal planar geometry (there is 120 degrees between the
oxygens so it's hard for both to bind to the same metal). However, in some cases it will bind to two different metals, making it bidentate.

19.E.2.23: A19.2.9
CO3-2 will coordinate to a metal center as a monodentate ligand.

19.E.2.24: Q19.2.10
Draw the geometric, linkage, and ionization isomers for [CoCl5CN][CN].

19.E.2.25: S19.2.10
Isomers are compounds with same formula but different atom arrangement. There are two subcategories: structural isomers, which are
isomers that contain the same number of atoms of each kind but differ in which atoms are bonded to one another, and stereoisomers, isomers
that have the same molecular formula and ligands, but differ in the arrangement of those ligands in 3D space.
There are three subcategories under structural isomers: ionization isomers, which are isomers that are identical except for a ligand has
exchanging places with an anion or neutral molecule that was originally outside the coordination complex; coordination isomers, isomers that
have an interchange of some ligands from the cationic part to the anionic part; and linkage isomers, in two or more coordination compounds in
which the donor atom of at least one of the ligands is different.
There are also two main kinds of stereoisomers: geometric isomers, metal complexes that differ only in which ligands are adjacent to one
another (cis) or directly across from one another (trans) in the coordination sphere of the metal, and optical isomers, which occurs when the
mirror image of an object is non-superimposable on the original object.

Some of the isomers look almost identical, but that is because the CN ligand can be attached by both (but not at the same time) the C or N.

19.E.2.26: A19.2.10

Access for free at OpenStax 19.E.31 https://chem.libretexts.org/@go/page/44104


19.E.3: 19.3: Spectroscopic and Magnetic Properties of Coordination Compounds
19.E.3.1: Q19.3.1
Determine the number of unpaired electrons expected for [Fe(NO2)6]3−and for [FeF6]3− in terms of crystal field theory.

19.E.3.2: S19.3.1
The crystal field theory is is a model that describes the breaking of degeneracies of electron orbital states, usually d or f orbitals, due to a
static electric field produced by a surrounding charge distribution.
The degenerate d-orbitals split into two levels, e and t , in the presence of ligands.
g 2g

The energy difference between the two levels is called the crystal-field splitting energy, Δ . ∘

After 1 electron each has been filled in the three t orbitals, the filling of the fourth electron takes place either in the e orbital or in the t ,
2g g 2g

where the electrons pair up. depending on whether the complex is high spin or low spin.
If the Δ value of a ligand is less than the pairing energy (P), then the electrons enter the e orbital, but if the Δ value of a ligand is more
∘ g ∘

than the pairing energy (P), then the electrons enter the t orbital.
2g

when the is less than the pairing energy, the electrons prefer then eg orbitals because there is not enough energy to pair the electrons
together. It will be high spin
when the is more then the pairing energy, the electrons prefer the t2g because there is enough energy to pair the electrons. It will be low
spin.
Step 1: Determine the oxidation state of the Fe
For [F e(N O ) ] and [F eF ] , both N O and F have a charge of -1. Since there is 6 of them then that means the charge is -6 and in
2 6
3−
6
3−
2 6

order for there to be an overall charge of -3, Fe has to have a +3 charge.


Step 2: Determine type of ligand
Based on the spectrochemical series we can see that N O is a stronger field ligand than F , and therefore is a low spin complex because it has

2

a high Δ unlike F which is a high spin.



Step 3: Draw the crystal field


3−
[F e(N O2 )6 ]

3−
[F eF6 ]

Access for free at OpenStax 19.E.32 https://chem.libretexts.org/@go/page/44104


There is 1 unpaired electron for [F e(N O 2 )6 ]
3−
, and 5 for [F eF
6]
3−
based on the crystal field theory.

19.E.3.3: A19.3.1
[Fe(NO2)6]3−:1 electron
[FeF6]3−:5 electrons

19.E.3.4: Q19.3.2
Draw the crystal field diagrams for [Fe(NO2)6]4− and [FeF6]2−. State whether each complex is high spin or low spin, paramagnetic or
diamagnetic, and compare Δoct to P for each complex.

19.E.3.5: S19.3.2
a)
−4
[F e(N O2 )6 ] (19.E.76)

NO2- has a -1 charge. The overall ion has a -4 charge, therefore Fe must be +2 charge. (The math:
x + (6)(−1) = −4, x + −6 = −4, x = +2 or 2+(6*-1)=-4)

Fe2+ has 6 valence electrons.


Next we look at the ligand bonded to Fe, which is NO2- . Based on the spectrochemical Series, NO2- is a strong field ligand
meaning that it has a large DELTAo large splitting energy in comparison to the pairing energy, P. So the electrons would rather
pair up, as it takes the least amount of energy.
So [Fe(NO2)6]4− is low spin.
All 6 electrons are paired up, so it is diamagnetic.
b)
−3
[F eF6 ] (19.E.77)

F- has a -1 charge. The overall ion has a -3 charge, therefore Fe must be +3 charge. (The math:
x + (−1)(6) = −3, x + −6 = −3, x = +3 or 3+(6*-1)=-3)

F3+ has 5 valence electrons.


Next we look at the ligand bonded to Fe, which is F- . Based on the Spectrochemical Series, F- is a weak field ligand, meaning
that it has a small DELTAo or small splitting energy in comparison to the pairing energy, P. So the electrons would rather split
up and move up to the higher energy level, rather than pairing up, as it takes the least amount of energy.
So [Fe(NO2)6]4− is high spin.
[FeF6]3− is paramagnetic because it has unpaired electrons.

19.E.3.6: A19.3.2

19.E.3.7: Q19.3.3
Give the oxidation state of the metal, number of d electrons, and the number of unpaired electrons predicted for [Co(NH3)6]Cl3.

Access for free at OpenStax 19.E.33 https://chem.libretexts.org/@go/page/44104


19.E.3.8: S19.3.3
The oxidation state of the metal can be found by identifying the charge of one of each molecule in the coordinate compound, multiplying each
molecule's charge by the respective number of molecules present, and adding the products. This final sum represents the charge of the overall
coordination compound. You can then solve for the oxidation state of the metal algebraically. In this case, one chloride anion Cl- has a charge
of -1. So three chloride anions have a total charge of -3. One ammine ligand NH3 has no charge so six ammine ligands have a total charge of
zero. Finally, we are trying to solve for the oxidation state of a cobalt ion. Now we can write the equation that adds the total charges of each
molecule or ion and is equal to the total charge of the overall coordinate compound.

(oxidation state of Co) + (−3) + 0 = 0 (19.E.78)

oxidation state of Co = +3 (19.E.79)

3+
So the oxidation state of Co is +3. Now we need to identify the number of d-electrons in the Co ion. The electron configuration for cobalt
that has no charge is
2 7
[Ar]4 s 3 d (19.E.80)

3+
However, a Co ion has 3 less electrons than its neutral counterpart and has an electron configuration of
6
[Ar]3d (19.E.81)

For transition metals, the s electrons are lost first. So cobalt loses its two 4s electrons first and then loses a single 3d electron meaning Co3+ ion
has 6 d electrons. To predict the number of unpaired electrons, we must first determine if the complex is high spin or low spin. Whether the
complex is high spin or low spin is determined by the ligand in the coordinate complex. Specifically, the ligand must be identified as either a
weak-field ligand or a strong-field ligand based on the spectrochemical series. Weak-field ligands induce high spin while strong-field ligands
induce low spin. We can then construct the energy diagram or crystal field diagram of the designated spin that has the proper electron placings.
The geometric shape of the compound must also be identified to construct the correct diagram. Finally, from this crystal field diagram we can
determine the number of unpaired electrons. The number of electrons in the diagram is equal to the number of d electrons of the metal. The
ligand in this case is NH3, which is a strong field ligand according to the spectrochemical series. This means that the complex is low spin.
Additionally, six monodentate ligands means the ligand field is octahedral. The number of electrons that will be in the diagram is 6 since the
metal ion Co3+ has 6 d electrons. Now the proper crystal field diagram can be constructed.

From the crystal field diagram, we can tell that the complex has no unpaired electrons.

19.E.3.9: A19.3.3
a) 3+
b) 6 d electrons
c) No unpaired electrons

19.E.3.10: Q19.3.4
The solid anhydrous solid CoCl2 is blue in color. Because it readily absorbs water from the air, it is used as a humidity indicator to monitor if
equipment (such as a cell phone) has been exposed to excessive levels of moisture. Predict what product is formed by this reaction, and how
many unpaired electrons this complex will have.

19.E.3.11: S19.3.4
From our knowledge of ligands and coordination compounds (or if you need a refresher Coordination Compounds), we can assume the product
of CoCl2 in water. H2O is a common weak field ligand that forms six ligand bonds around the central Cobalt atom while the Chloride stays on
the outer sphere. We can use this to determine the complex:
[C o(H2 O)6 ]C l2

From this formation, we can use the Crystal Field Theory (CFT)(Crystal Field Theory) to determine the number of unpaired electrons. This
coordination compound has six ligand bonds attached to the central atom which means the CFT model will follow the octahedral splitting.

Access for free at OpenStax 19.E.34 https://chem.libretexts.org/@go/page/44104


Keep in mind that we know H2O is a weak field ligand and will produce a high spin. High spin is when the electrons pairing energy (P) is
greater than the octahedral splitting energy. Thus, the electrons spread out and maximize spin.
In order to fill out our crystal field diagram, we need to determine the charge of cobalt. Because the H2O ligand is neutral, and there are two
chlorine ions, we can deduce the charge of cobalt is plus two in order to make the coordination complex neutral. From here, we can use the
electron configuration of Co2+ is [Ar]4s23d7. The electrons that are taken away from the cobalt atom in order to form the plus two charge will
from the 4s orbital and leave the 3d orbital untouched. Thus, there will be 7 electrons in the crystal field diagram and appear as:

We can see here that there are 3 unpaired electrons.

19.E.3.12: A19.3.4
[Co(H2O)6]Cl2 with three unpaired electrons.

19.E.3.13: Q19.3.5
Is it possible for a complex of a metal in the transition series to have six unpaired electrons? Explain.

19.E.3.14: A19.3.5
Is it possible for a complex of a metal in the transition series to have six unpaired electrons? Explain.
It is not possible for a metal in the transition series to have six unpaired electrons. This is because transition metals have a general electron
configuration of (n-1)d1-10 ns1-2 where n is the quantum number. The last electron will go into the d orbital which has 5 orbitals that can each
contain 2 electrons, yielding 10 electrons total. According to Hund's Rule, electrons prefer to fill each orbital singly before they pair up. This is
more energetically favorable. Since there are only 5 orbitals and due to Hund's Rule, the maximum number of unpaired electrons a transition
metal can have is 5. Therefore, there cannot be a complex of a transition metal that has 6 unpaired electrons.
For example, lets look at iron's electron configuration. Iron has an electron configuration of 1s22s22p63s23p64s23d6. Now the most important
orbital to look at is the d orbital which has 6 electrons in it, but there are only 4 unpaired electrons as you can see by this diagram:
3d: [↿⇂] [↿] [↿][↿][↿]
Each [ ] represents an orbital within the d orbital. This diagram follows Hund's rule and shows why no transition metal can have 6 unpaired
electrons.

19.E.3.15: Q19.3.6
How many unpaired electrons are present in each of the following?
a. [CoF6]3− (high spin)
b. [Mn(CN)6]3− (low spin)
c. [Mn(CN)6]4− (low spin)
d. [MnCl6]4− (high spin)
e. [RhCl6]3− (low spin)

19.E.3.16: S19.3.6
1. For [CoF6]3-, we first found the oxidation state of Co, which is 3+ since F has a 1- charge and since there is 6 F, Co's charge has to be 3+ for
the overall charge to be 3-.

charge of Co + -6 = -6 (19.E.82)

charge of Co = +3 (19.E.83)

After finding the oxidation state, I then go to the periodic table to find its electron configuration: [Ar]3d6
We distribute the 6 d-orbital electrons along the complex and since it is high spin, the electrons is distributed once in each energy level before it
is paired. There is only one pair and the other 4 electrons are unpaired, making the answer 4.

Access for free at OpenStax 19.E.35 https://chem.libretexts.org/@go/page/44104


2. The same process is repeated. We find the charge of Mn, which is 3+, making the electron configuration: [Ar]3d4

charge of Mn + -6 = -3 (19.E.84)

charge of Mn = +3 (19.E.85)

There is a difference between this and number 1. This is low spin so instead of distributing one electron in each level before pairing it, I must
distribute one electron on the bottom and then pair them all up before I'm able to move to the top portion. So since there is 4, there is only a pair
at dyz and the other two electrons are unpaired. Making the number of unpaired electrons 2.

3.The same process as number 2 is applied. The only difference is that the charge of Mn is now 2+ so the electron configuration: [Ar]3d5.
charge of Mn + -6 = -4 (19.E.86)

charge of Mn = +2 (19.E.87)

Since is it low spin like number 2, I only need to add an extra electron to the next level, making that 2 pairs of electron and only 1 electron
unpaired.

4. Since Cl has a -1 charge like CN, Mn's charge is also 2+ with the same electron configuration as number 3, which is 5.

charge of Mn + -6 = -4 (19.E.88)

charge of Mn = +2 (19.E.89)

With 5 electrons, this is high spin instead of low. So as stated in number 1, we pair distribute the electrons on all levels first. Since there are 5
electrons and 5 levels and they are al distributed, there are zero pairs, making that 5 unpaired electrons.

Access for free at OpenStax 19.E.36 https://chem.libretexts.org/@go/page/44104


5. Using the same process as the problems above, Rh's charge is 3+, with the electron configuration: [Kr]4d6.
charge of Rh + -6 = -3 (19.E.90)

charge of Mn = +3 (19.E.91)

With a low spin and 6 electrons, all electrons are paired up, making it 0 electrons that are unpaired.

19.E.3.17: A19.3.6
4; 2; 1; 5; 0

19.E.3.18: Q19.3.7
Explain how the diphosphate ion, [O3P−O−PO3]4−, can function as a water softener that prevents the precipitation of Fe2+ as an insoluble iron
salt.

19.E.3.19: S19.3.7
The diphosphate ion, [O3P−O−PO3]4− can function as a water softener keeping the iron in a water soluble form because of its more negative
electrochemical potential than water's. This is similar to the way plating prevents metals from reacting with oxygen to corrode.
Mineral deposits are formed by ionic reactions. The Fe2+ will form an insoluble iron salt of iron(III) oxide-hydroxide when a salt of ferric iron
hydrolyzes water. However, with the addition of [O3P−O−PO3]4−, the Fe2+ cations are more attracted to the PO3 group, forming a Fe(PO3)
complex.
The excess minerals in this type of water is considered hard thus its name hard water.

19.E.3.20: Q19.3.8
For complexes of the same metal ion with no change in oxidation number, the stability increases as the number of electrons in the t2g orbitals
increases. Which complex in each of the following pairs of complexes is more stable?
a. [Fe(H2O)6]2+ or [Fe(CN)6]4−
b. [Co(NH3)6]3+ or [CoF6]3−
c. [Mn(CN)6]4− or [MnCl6]4−

19.E.3.21: S19.3.8
The Spectrochemical Series is as follows
− − − − − − − − −
I < Br < SC N ≈ Cl <F < OH < ON O < ox < H2 O < SC N < EDT A < N H3 < en < N O (19.E.92)
2

< CN

The strong field ligands (on the right) are low spin which fills in more electrons in the t2g orbitals. The weak field ligands (on the left) are high
spin so it can fill electrons in the t2g orbitals and eg orbitals. In conclusion, more electrons are filled up from the strong field ligands because

Access for free at OpenStax 19.E.37 https://chem.libretexts.org/@go/page/44104


the electrons don't move up to the eg orbitals.

a. [F e(C N ) 6]
4−

CN is a stronger ligand than H 2O so it is low spin, which fills up the t2g orbitals.
b. [C o(N H 3+
3 )6 ]

N H3 is a stronger ligand than F .


c. [M n(C N ) 6]
4−

CN is a stronger ligand than C l .


For more information regarding the shape of the complex and d-electron configuration, libretext provides more information on how to classify
high and low spin complexes.

19.E.3.22: A19.3.8
[Fe(CN)6]4−; [Co(NH3)6]3+; [Mn(CN)6]4−

19.E.3.23: Q19.3.9
Trimethylphosphine, P(CH3)3, can act as a ligand by donating the lone pair of electrons on the phosphorus atom. If trimethylphosphine is added
to a solution of nickel(II) chloride in acetone, a blue compound that has a molecular mass of approximately 270 g and contains 21.5% Ni,
26.0% Cl, and 52.5% P(CH3)3 can be isolated. This blue compound does not have any isomeric forms. What are the geometry and molecular
formula of the blue compound?

19.E.3.24: S19.3.9
1)Find the empirical formula. There is a total of 270 grams. To find out how many grams of each element/compound there are,
multiply the percentage by the mass (270).

(270g)(0.215) = 58.05gN i (19.E.93)

(270g)(0.26) = 70.2gC l (19.E.94)

(270g)(0.525) = 141.75gP (C H3 )3 (19.E.95)

Now that we have the grams of each element/compound, we can convert them to moles by using their molar mass.
1mol.
(58.055gN i)( ) = 0.989mol. N i (19.E.96)
58.69gN i

1mol.
(70.2gC l)( ) = 1.98mol. C l (19.E.97)
35.45gC l

1mol.
(141.75gP (C H3 )3 )( ) = 1.86mol. P (C H3 )3 (19.E.98)
76.07gP (C H3 )3

Now that we have the moles of all elements/compounds, we can find the ratio of all them to each other. To do this, we take the
element/compound with the least amount of moles and divide all element/compound moles by this amount. In this case, Ni has the
least number of moles.
0.989mol. N i
=1 (19.E.99)
0.989mol. N i

1.98mol. C l
= approx.2 (19.E.100)
0.989mol. N i

1.86mol. P (C H3 )3
= approx.2 (19.E.101)
0.989mol. N i

We now know the ratio of all element/compounds in the blue compound.

Access for free at OpenStax 19.E.38 https://chem.libretexts.org/@go/page/44104


The empirical formula is: NiCl(P(CH3)3)2
This formula shows us there are 4 ligands. There are 2 chlorine ligands and 2 trimethylphosphine ligands. This means that the
blue compound has either a tetrahedral or square planar shape, where tetrahedral shapes are capable of different isomeric forms
when all ligands are different (because if not, there is only 1 way for them to be arranged), and square planar shapes are capable
of cis/trans forms. In the problem, it states this compound does not have any isomeric forms, therefore this has a tetrahedral
shape.

19.E.3.25: A19.3.9
a) NiCl(P(CH3)3)2
b) Tetrahedral

19.E.3.26: Q19.3.10
Would you expect the complex [Co(en)3]Cl3 to have any unpaired electrons? Any isomers?

19.E.3.27: S19.3.10
Assign oxidation states to each element. Cl- has a -1 oxidation state. En is neutral, so 0. The entire complex is also neutral, so in order to
balance the charges out, Co must be +3 because there are 3 chlorides, which gives a -3 charge.
STEP 2:
Write the electron configuration for C o3+
. [Ar]3d . There are 6 electrons.
6

STEP 3:

Check where en lies on the spectrochemical series. Does it have a strong field strength? It does, so these electrons will exist at the d-level with
high splitting energy because the magnitude of the pairing energy is less than the crystal field splitting energy in the octahedral field.
You will the notice that there aren't any unpaired electrons when you draw the Crystal Field Theory (CFT) diagram.

This complex does not have any geometric isomers because cis-trans structures cannot be formed. The mirror image is nonsuperimpoasable,
which means the enantiomers are chiral molecules; if the mirror image is placed on top on the original molecule, then they will never be
perfectly aligned to give the same molecule.

19.E.3.28: A19.3.10
The complex does not have any unpaired electrons. The complex does not have any geometric isomers, but the mirror image is
nonsuperimposable, so it has an optical isomer.

19.E.3.29: Q19.3.11
Would you expect the Mg3[Cr(CN)6]2 to be diamagnetic or paramagnetic? Explain your reasoning.

Access for free at OpenStax 19.E.39 https://chem.libretexts.org/@go/page/44104


19.E.3.30: S19.3.11
The first step to determine the magnetism of the complex is to calculate the oxidation state of the transition metal. In this case, the transition
metal is Cr.
Before doing so, we need to find charge of the of the complex ion [Cr(CN)6]2 given that the oxidation state of Mg3 is 2+. Using the subscripts of the
Mg ion and the [Cr(CN)6]2 complex, we find that the oxidation state of [Cr(CN)6]2 , x, to be:
2+

3(+2) + 2(x) = 0

x = 3

Now that we found the charge of the coordination complex, we are able to find the charge of the transition metal Cr given that the charge of CN is -1. Again, using the subscripts we find the
oxidation state of Cr, y, to be:

y + 6(−1) = −3

y = 3

Therefore, the oxidation state of the transition metal Cr is Cr


3+

Next, using the transition metal 3+


Cr and the periodic table as reference, we can determine the electron configuration of 3+
Cr to be [Ar ]d . This means that
3 3+
Cr has 3 unpaired
electrons in the 3d sublevel. Therefore, we find that since at least one electron is unpaired(in this case all 3 electrons are unpaired), Mg3[Cr(CN)6]2 is paramagnetic.

A19.3.11
a) Paramagnetic

19.E.3.31: Q19.3.12
Would you expect salts of the gold ion, Au+, to be colored? Explain.

19.E.3.32: S19.3.12
No. Colored ions have unpaired electrons in their outmost orbital. A partially filled d orbital, for example, can yield various colors. After
completing the noble gas configuration, we see that Au+ has a configuration of [Xe] 4f145d10. Since Au+ has a completely filled d sublevel, we
are certain that any salts of the gold ion, Au+ will be colorless.
*An example of a colored ion would be copper(II). Cu2+ has an electron configuration of [Ar]3d9. It has one unpaired electron. Copper(II)
appears blue.

19.E.3.33: A19.3.12
No. Au+ has a complete 5d sublevel.

19.E.3.34: Q19.3.13
[CuCl4]2− is green. [Cu(H2O)6]2+is blue. Which absorbs higher-energy photons? Which is predicted to have a larger crystal field splitting?

19.E.3.35: S19.3.13

Access for free at OpenStax 19.E.40 https://chem.libretexts.org/@go/page/44104


Although a color might appear a certain way, it actual absorbs a different color, opposite of it on the color wheel.

In this case;
[CuCl4]2- appears green but is opposite of red on the color wheel which is absorbed and is characterized by wavelengths 620-800
nanometers.
[Cu(H2O)6]2+ appears blue but is opposite of orange on the color wheel which is absorbed and is characterized by wavelengths 580-620
nanometers.
When determining which absorbs the higher energy photons, one must look at the complex itself. A higher energy indicates a high
energy photon absorbed and a lower energy indicates a lower energy photon absorbed. How can we determine this? By looking at the
complex and more specifically the ligand attached and its location in the spectrochemical series.

The ligands attached are Water and Chlorine and since Water is a stronger ligand than Chlorine according to the series, it also has larger
energy, indicating a higher energy. This means that the complex [Cu(H2O)6]2+ absorbs a higher energy photon because of its a stronger
ligand than chlorine.
Part 2 of this question also asks which complex is predicted to have a larger crystal field splitting. To determine this you also use the
spectrochemical series and see which ligand is stronger. Since H2O is stronger than Cl- on the spectrochemical series, we can say
[Cu(H2O)6]2+ has a higher crystal field splitting.

19.E.3.36: A19.3.13
a) [Cu(H2O)6]2+
b)
[Cu(H2O)6]2+ has a higher crystal field splitting

Access for free at OpenStax 19.E.41 https://chem.libretexts.org/@go/page/44104


This page titled 19.E: Transition Metals and Coordination Chemistry (Exercises) is shared under a CC BY 4.0 license and was authored, remixed, and/or
curated by OpenStax via source content that was edited to the style and standards of the LibreTexts platform; a detailed edit history is available upon request.

Access for free at OpenStax 19.E.42 https://chem.libretexts.org/@go/page/44104


CHAPTER OVERVIEW
20: Organic Chemistry

A general chemistry Libretexts Textbook remixed and remastered from


OpenStax's textbook:
General Chemistry
Organic chemistry involving the scientific study of the structure, properties, and reactions of organic compounds and organic
materials, i.e., matter in its various forms that contain carbon atoms. Study of structure includes many physical and chemical
methods to determine the chemical composition and the chemical constitution of organic compounds and materials. Study of
properties includes both physical properties and chemical properties, and uses similar methods as well as methods to evaluate
chemical reactivity, with the aim to understand the behavior of the organic matter.
20.0: Prelude to Organic Chemistry
20.1: Hydrocarbons
20.2: Alcohols and Ethers
20.3: Aldehydes, Ketones, Carboxylic Acids, and Esters
20.4: Amines and Amides
20.E: Organic Chemistry (Exercises)

This page titled 20: Organic Chemistry is shared under a CC BY 4.0 license and was authored, remixed, and/or curated by OpenStax via source
content that was edited to the style and standards of the LibreTexts platform; a detailed edit history is available upon request.

1
20.0: Prelude to Organic Chemistry
All living things on earth are formed mostly of carbon compounds. The prevalence of carbon compounds in living things has led to
the epithet “carbon-based” life. The truth is we know of no other kind of life. Early chemists regarded substances isolated from
organisms (plants and animals) as a different type of matter that could not be synthesized artificially, and these substances were
thus known as organic compounds. The widespread belief called vitalism held that organic compounds were formed by a vital force
present only in living organisms. The German chemist Friedrich Wohler was one of the early chemists to refute this aspect of
vitalism, when, in 1828, he reported the synthesis of urea, a component of many body fluids, from nonliving materials. Since then,
it has been recognized that organic molecules obey the same natural laws as inorganic substances, and the category of organic
compounds has evolved to include both natural and synthetic compounds that contain carbon. Some carbon-containing compounds
are not classified as organic, for example, carbonates and cyanides, and simple oxides, such as CO and CO2. Although a single,
precise definition has yet to be identified by the chemistry community, most agree that a defining trait of organic molecules is the
presence of carbon as the principal element, bonded to hydrogen and other carbon atoms.

Figure 20.0.1 : All organic compounds contain carbon and most are formed by living things, although they are also formed by
geological and artificial processes. (credit left: modification of work by Jon Sullivan; credit left middle: modification of work by
Deb Tremper; credit right middle: modification of work by “annszyp”/Wikimedia Commons; credit right: modification of work by
George Shuklin)
Today, organic compounds are key components of plastics, soaps, perfumes, sweeteners, fabrics, pharmaceuticals, and many other
substances that we use every day. The value to us of organic compounds ensures that organic chemistry is an important discipline
within the general field of chemistry. In this chapter, we discuss why the element carbon gives rise to a vast number and variety of
compounds, how those compounds are classified, and the role of organic compounds in representative biological and industrial
settings.

This page titled 20.0: Prelude to Organic Chemistry is shared under a CC BY 4.0 license and was authored, remixed, and/or curated by OpenStax
via source content that was edited to the style and standards of the LibreTexts platform; a detailed edit history is available upon request.

Access for free at OpenStax 20.0.1 https://chem.libretexts.org/@go/page/38329


20.1: Hydrocarbons
 Learning Objectives
Explain the importance of hydrocarbons and the reason for their diversity
Name saturated and unsaturated hydrocarbons, and molecules derived from them
Describe the reactions characteristic of saturated and unsaturated hydrocarbons
Identify structural and geometric isomers of hydrocarbons

The largest database1 of organic compounds lists about 10 million substances, which include compounds originating from living
organisms and those synthesized by chemists. The number of potential organic compounds has been estimated2 at 1060—an
astronomically high number. The existence of so many organic molecules is a consequence of the ability of carbon atoms to form
up to four strong bonds to other carbon atoms, resulting in chains and rings of many different sizes, shapes, and complexities.
The simplest organic compounds contain only the elements carbon and hydrogen, and are called hydrocarbons. Even though they
are composed of only two types of atoms, there is a wide variety of hydrocarbons because they may consist of varying lengths of
chains, branched chains, and rings of carbon atoms, or combinations of these structures. In addition, hydrocarbons may differ in the
types of carbon-carbon bonds present in their molecules. Many hydrocarbons are found in plants, animals, and their fossils; other
hydrocarbons have been prepared in the laboratory. We use hydrocarbons every day, mainly as fuels, such as natural gas, acetylene,
propane, butane, and the principal components of gasoline, diesel fuel, and heating oil. The familiar plastics polyethylene,
polypropylene, and polystyrene are also hydrocarbons. We can distinguish several types of hydrocarbons by differences in the
bonding between carbon atoms. This leads to differences in geometries and in the hybridization of the carbon orbitals.

20.1.1: Alkanes
Alkanes, or saturated hydrocarbons, contain only single covalent bonds between carbon atoms. Each of the carbon atoms in an
alkane has sp3 hybrid orbitals and is bonded to four other atoms, each of which is either carbon or hydrogen. The Lewis structures
and models of methane, ethane, and pentane are illustrated in Figure 20.1.1. Carbon chains are usually drawn as straight lines in
Lewis structures, but one has to remember that Lewis structures are not intended to indicate the geometry of molecules. Notice that
the carbon atoms in the structural models (the ball-and-stick and space-filling models) of the pentane molecule do not lie in a
straight line. Because of the sp3 hybridization, the bond angles in carbon chains are close to 109.5°, giving such chains in an alkane
a zigzag shape.
The structures of alkanes and other organic molecules may also be represented in a less detailed manner by condensed structural
formulas (or simply, condensed formulas). Instead of the usual format for chemical formulas in which each element symbol appears
just once, a condensed formula is written to suggest the bonding in the molecule. These formulas have the appearance of a Lewis
structure from which most or all of the bond symbols have been removed. Condensed structural formulas for ethane and pentane
are shown at the bottom of Figure 20.1.1, and several additional examples are provided in the exercises at the end of this chapter.

Figure 20.1.1 : Pictured are the Lewis structures, ball-and-stick models, and space-filling models for molecules of methane, ethane,
and pentane.
A common method used by organic chemists to simplify the drawings of larger molecules is to use a skeletal structure (also called
a line-angle structure). In this type of structure, carbon atoms are not symbolized with a C, but represented by each end of a line or

Access for free at OpenStax 20.1.1 https://chem.libretexts.org/@go/page/38330


bend in a line. Hydrogen atoms are not drawn if they are attached to a carbon. Other atoms besides carbon and hydrogen are
represented by their elemental symbols. Figure 20.1.2 shows three different ways to draw the same structure.

Figure 20.1.2 : The same structure can be represented three different ways: an expanded formula, a condensed formula, and a
skeletal structure.

 Example 20.1.1

Drawing Skeletal Structures Draw the skeletal structures for these two molecules:

Solution
Each carbon atom is converted into the end of a line or the place where lines intersect. All hydrogen atoms attached to the
carbon atoms are left out of the structure (although we still need to recognize they are there):

Figure a shows a branched skeleton structure that looks like a plus sign with line segments extending up and to the right and
down and to the left of the rightmost point of the plus sign. Figure b appears in a zig zag pattern made with six line segments.
The segments rise, fall, rise, fall, rise, and fall moving left to right across the figure.

 Exercise 20.1.1

Draw the skeletal structures for these two molecules:

Figure a shows five C H subscript 2 groups and one C H group bonded in a hexagonal ring. A C H subscript 3 group appears
above and to the right of the ring, bonded to the ring on the C H group appearing at the upper right portion of the ring. In b, a
straight chain molecule composed of C H subscript 3 C H subscript 2 C H subscript 2 C H subscript 2 C H subscript 3 is
shown.

Answer

Access for free at OpenStax 20.1.2 https://chem.libretexts.org/@go/page/38330


In a, a hexagon with a vertex at the top is shown. The vertex just to the right has a line segment attached that extends up and
to the right. In b, a zig zag pattern is shown in which line segments rise, fall, rise, fall, and rise moving left to right.

 Example 20.1.2

Interpreting Skeletal Structures Identify the chemical formula of the molecule represented here:

This figure shows a pentagon with a vertex pointing right, from which a line segment extends that has two line segments
attached at its right end, one extending up and to the right, and the other extending down and to the right.

Solution
There are eight places where lines intersect or end, meaning that there are eight carbon atoms in the molecule. Since we know
that carbon atoms tend to make four bonds, each carbon atom will have the number of hydrogen atoms that are required for
four bonds. This compound contains 16 hydrogen atoms for a molecular formula of C8H16.
Location of the hydrogen atoms:

In this figure a ring composed of four C H subscript 2 groups and one C H group in a pentagonal shape is shown. From the C H
group, which is at the right side of the pentagon, a C H is bonded. From this C H, a C H subscript 3 group is bonded above and
to the right and a second is bonded below and to the right.

 Exercise 20.1.2

Identify the chemical formula of the molecule represented here:

A skeleton model is shown with a zig zag pattern that rises, falls, rises, and falls again left to right through the center of the
molecule. From the two risen points, line segments extend both up and down, creating four branches.

Answer
C9H20

Access for free at OpenStax 20.1.3 https://chem.libretexts.org/@go/page/38330


All alkanes are composed of carbon and hydrogen atoms, and have similar bonds, structures, and formulas; noncyclic alkanes all
have a formula of CnH2n+2. The number of carbon atoms present in an alkane has no limit. Greater numbers of atoms in the
molecules will lead to stronger intermolecular attractions (dispersion forces) and correspondingly different physical properties of
the molecules. Properties such as melting point and boiling point (Table 20.1.1) usually change smoothly and predictably as the
number of carbon and hydrogen atoms in the molecules change.
Table 20.1.1 : Properties of Some Alkanes
Number of
Alkane Molecular Formula Melting Point (°C) Boiling Point (°C) Phase at STP4
Structural Isomers

methane CH4 –182.5 –161.5 gas 1

ethane C2H6 –183.3 –88.6 gas 1

propane C3H8 –187.7 –42.1 gas 1

butane C4H10 –138.3 –0.5 gas 2

pentane C5H12 –129.7 36.1 liquid 3

hexane C6H14 –95.3 68.7 liquid 5

heptane C7H16 –90.6 98.4 liquid 9

octane C8H18 –56.8 125.7 liquid 18

nonane C9H20 –53.6 150.8 liquid 35

decane C10H22 –29.7 174.0 liquid 75

tetradecane C14H30 5.9 253.5 solid 1858

octadecane C18H38 28.2 316.1 solid 60,523

Hydrocarbons with the same formula, including alkanes, can have different structures. For example, two alkanes have the formula
C4H10: They are called n-butane and 2-methylpropane (or isobutane), and have the following Lewis structures:

The compounds n-butane and 2-methylpropane are structural isomers (the term constitutional isomers is also commonly used).
Constitutional isomers have the same molecular formula but different spatial arrangements of the atoms in their molecules. The n-
butane molecule contains an unbranched chain, meaning that no carbon atom is bonded to more than two other carbon atoms. We
use the term normal, or the prefix n, to refer to a chain of carbon atoms without branching. The compound 2–methylpropane has a
branched chain (the carbon atom in the center of the Lewis structure is bonded to three other carbon atoms)
Identifying isomers from Lewis structures is not as easy as it looks. Lewis structures that look different may actually represent the
same isomers. For example, the three structures in Figure 20.1.3 all represent the same molecule, n-butane, and hence are not
different isomers. They are identical because each contains an unbranched chain of four carbon atoms.

Access for free at OpenStax 20.1.4 https://chem.libretexts.org/@go/page/38330


Figure 20.1.3 : These three representations of the structure of n-butane are not isomers because they all contain the same
arrangement of atoms and bonds.

20.1.2: The Basics of Organic Nomenclature: Naming Alkanes


The International Union of Pure and Applied Chemistry (IUPAC) has devised a system of nomenclature that begins with the names
of the alkanes and can be adjusted from there to account for more complicated structures. The nomenclature for alkanes is based on
two rules:
1. To name an alkane, first identify the longest chain of carbon atoms in its structure. A two-carbon chain is called ethane; a three-
carbon chain, propane; and a four-carbon chain, butane. Longer chains are named as follows: pentane (five-carbon chain),
hexane (6), heptane (7), octane (8), nonane (9), and decane (10). These prefixes can be seen in the names of the alkanes
described in Table 20.1.1.
2. Add prefixes to the name of the longest chain to indicate the positions and names of substituents. Substituents are branches or
functional groups that replace hydrogen atoms on a chain. The position of a substituent or branch is identified by the number of
the carbon atom it is bonded to in the chain. We number the carbon atoms in the chain by counting from the end of the chain
nearest the substituents. Multiple substituents are named individually and placed in alphabetical order at the front of the name.
3.

When more than one substituent is present, either on the same carbon atom or on different carbon atoms, the substituents are listed
alphabetically. Because the carbon atom numbering begins at the end closest to a substituent, the longest chain of carbon atoms is
numbered in such a way as to produce the lowest number for the substituents. The ending -o replaces -ide at the end of the name of
an electronegative substituent (in ionic compounds, the negatively charged ion ends with -ide like chloride; in organic compounds,
such atoms are treated as substituents and the -o ending is used). The number of substituents of the same type is indicated by the
prefixes di- (two), tri- (three), tetra- (four), and so on (for example, difluoro- indicates two fluoride substituents).

 Example 20.1.3: Naming Halogen-substituted Alkanes

Name the molecule whose structure is shown here:

Access for free at OpenStax 20.1.5 https://chem.libretexts.org/@go/page/38330


This structure shows a C atom bonded to the H atoms and another C atom. This second C atom is bonded to two H atoms and
another C atom. This third C atom is bonded to a B r atom and another C atom. This fourth C atom is bonded to two H atoms
and a C l atom.

Solution

This structure shows a C atom bonded to the H atoms and another C atom. This second C atom is bonded to two H atoms and
another C atom. This third C atom is bonded to an H atom, a B r atom, and another C atom. This fourth C atom is bonded to
two H atoms and a C l atom. The C atoms are numbered 4, 3, 2, and 1 from left to right.
The four-carbon chain is numbered from the end with the chlorine atom. This puts the substituents on positions 1 and 2
(numbering from the other end would put the substituents on positions 3 and 4). Four carbon atoms means that the base name
of this compound will be butane. The bromine at position 2 will be described by adding 2-bromo-; this will come at the
beginning of the name, since bromo- comes before chloro- alphabetically. The chlorine at position 1 will be described by
adding 1-chloro-, resulting in the name of the molecule being 2-bromo-1-chlorobutane.

 Exercise 20.1.3

Name the following molecule:

This figure shows a C atom bonded to three H atoms and another C atom. This second C atom is bonded to two H atoms and a
third C atom. The third C atom is bonded to two B r atoms and a fourth C atom. This C atom is bonded to an H atom, and I
atom, and a fifth C atom. This last C atom is bonded to three H atoms.

Answer
3,3-dibromo-2-iodopentane

We call a substituent that contains one less hydrogen than the corresponding alkane an alkyl group. The name of an alkyl group is
obtained by dropping the suffix -ane of the alkane name and adding -yl:

Access for free at OpenStax 20.1.6 https://chem.libretexts.org/@go/page/38330


In this figure, methane is named and represented as C with four H atoms bonded above, below, to the left, and to the right of the C.
The methyl group is shown, which appears like methane without the right most H. A dash remains at the location where the H was
formerly bonded. Ethane is named and represented with two centrally bonded C atoms to which six H atoms are bonded; two above
and below each of the two C atoms and to the left and right ends of the linked C atoms. The ethyl group appears as a similar
structure with the right-most H atom removed. A dash remains at the location where the H atom was formerly bonded.
The open bonds in the methyl and ethyl groups indicate that these alkyl groups are bonded to another atom.

 Example 20.1.4 Naming Substituted Alkanes

Name the molecule whose structure is shown here:

A chain of six carbon atoms, numbered 6, 5, 4, 3, 2, and 1 is shown. Bonded above carbon 3, a chain of two carbons is shown,
numbered 1 and 2 moving upward. H atoms are present directly above, below, left and right of all carbon atoms in positions
not already taken up in bonding to other carbon atoms.

Solution
The longest carbon chain runs horizontally across the page and contains six carbon atoms (this makes the base of the name
hexane, but we will also need to incorporate the name of the branch). In this case, we want to number from right to left (as
shown by the blue numbers) so the branch is connected to carbon 3 (imagine the numbers from left to right—this would put the
branch on carbon 4, violating our rules). The branch attached to position 3 of our chain contains two carbon atoms (numbered
in red)—so we take our name for two carbons eth- and attach -yl at the end to signify we are describing a branch. Putting all
the pieces together, this molecule is 3-ethylhexane.

 Exercise 20.1.4
Name the following molecule:

Access for free at OpenStax 20.1.7 https://chem.libretexts.org/@go/page/38330


This figure shows a C atom bonded to three H atoms and another C atom. This C atom is bonded to two H atoms and third C
atom. The third C atom is bonded to two H atoms and a fourth C atom. The fourth C atom is bonded to two H atoms and a fifth
C atom. This C atom is bonded to an H atom, a sixth C atom in the chain, and another C atom which appears to branch off the
chain. The C atom in the branch is bonded to two H atoms and another C atom. This C atom is bonded to two H atoms and
another C atom. This third C atom appears to the left of the second and is bonded to three H atoms. The sixth C atom in the
chain is bonded to two H atoms and a seventh C atom. The seventh C atom is bonded to two H atoms and an eighth C atom.
The eighth C atom is bonded to three H atoms.

Answer
4-propyloctane

Some hydrocarbons can form more than one type of alkyl group when the hydrogen atoms that would be removed have different
“environments” in the molecule. This diversity of possible alkyl groups can be identified in the following way: The four hydrogen
atoms in a methane molecule are equivalent; they all have the same environment. They are equivalent because each is bonded to a
carbon atom (the same carbon atom) that is bonded to three hydrogen atoms. (It may be easier to see the equivalency in the ball and
stick models in Figure 20.1.3. Removal of any one of the four hydrogen atoms from methane forms a methyl group. Likewise, the
six hydrogen atoms in ethane are equivalent and removing any one of these hydrogen atoms produces an ethyl group. Each of the
six hydrogen atoms is bonded to a carbon atom that is bonded to two other hydrogen atoms and a carbon atom. However, in both
propane and 2–methylpropane, there are hydrogen atoms in two different environments, distinguished by the adjacent atoms or
groups of atoms:

Figure 20.1.4 .

Access for free at OpenStax 20.1.8 https://chem.libretexts.org/@go/page/38330


Figure 20.1.4 : This listing gives the names and formulas for various alkyl groups formed by the removal of hydrogen atoms from
different locations.
Note that alkyl groups do not exist as stable independent entities. They are always a part of some larger molecule. The location of
an alkyl group on a hydrocarbon chain is indicated in the same way as any other substituent:

Alkanes are relatively stable molecules, but heat or light will activate reactions that involve the breaking of C–H or C–C single
bonds. Combustion is one such reaction:

CH (g) + 2 O (g) ⟶ CO (g) + 2 H O(g)


4 2 2 2

Alkanes burn in the presence of oxygen, a highly exothermic oxidation-reduction reaction that produces carbon dioxide and water.
As a consequence, alkanes are excellent fuels. For example, methane, CH4, is the principal component of natural gas. Butane,
C4H10, used in camping stoves and lighters is an alkane. Gasoline is a liquid mixture of continuous- and branched-chain alkanes,
each containing from five to nine carbon atoms, plus various additives to improve its performance as a fuel. Kerosene, diesel oil,
and fuel oil are primarily mixtures of alkanes with higher molecular masses. The main source of these liquid alkane fuels is crude
oil, a complex mixture that is separated by fractional distillation. Fractional distillation takes advantage of differences in the boiling
points of the components of the mixture (Figure 20.1.5). You may recall that boiling point is a function of intermolecular
interactions, which was discussed in the chapter on solutions and colloids.

Access for free at OpenStax 20.1.9 https://chem.libretexts.org/@go/page/38330


Figure 20.1.5 :In a column for the fractional distillation of crude oil, oil heated to about 425 °C in the furnace vaporizes when it
enters the base of the tower. The vapors rise through bubble caps in a series of trays in the tower. As the vapors gradually cool,
fractions of higher, then of lower, boiling points condense to liquids and are drawn off. (credit left: modification of work by Luigi
Chiesa)
In a substitution reaction, another typical reaction of alkanes, one or more of the alkane’s hydrogen atoms is replaced with a
different atom or group of atoms. No carbon-carbon bonds are broken in these reactions, and the hybridization of the carbon atoms
does not change. For example, the reaction between ethane and molecular chlorine depicted here is a substitution reaction:

The C–Cl portion of the chloroethane molecule is an example of a functional group, the part or moiety of a molecule that imparts a
specific chemical reactivity. The types of functional groups present in an organic molecule are major determinants of its chemical
properties and are used as a means of classifying organic compounds as detailed in the remaining sections of this chapter.

20.1.3: Alkenes
Organic compounds that contain one or more double or triple bonds between carbon atoms are described as unsaturated. You have
likely heard of unsaturated fats. These are complex organic molecules with long chains of carbon atoms, which contain at least one
double bond between carbon atoms. Unsaturated hydrocarbon molecules that contain one or more double bonds are called alkenes.
Carbon atoms linked by a double bond are bound together by two bonds, one σ bond and one π bond. Double and triple bonds give
rise to a different geometry around the carbon atom that participates in them, leading to important differences in molecular shape
and properties. The differing geometries are responsible for the different properties of unsaturated versus saturated fats.
Ethene, C2H4, is the simplest alkene. Each carbon atom in ethene, commonly called ethylene, has a trigonal planar structure. The
second member of the series is propene (propylene) (Figure 20.1.6); the butene isomers follow in the series. Four carbon atoms in
the chain of butene allows for the formation of isomers based on the position of the double bond, as well as a new form of
isomerism.

Access for free at OpenStax 20.1.10 https://chem.libretexts.org/@go/page/38330


Figure 20.1.6 : Expanded structures, ball-and-stick structures, and space-filling models for the alkenes ethene, propene, and 1-
butene are shown.
Ethylene (the common industrial name for ethene) is a basic raw material in the production of polyethylene and other important
compounds. Over 135 million tons of ethylene were produced worldwide in 2010 for use in the polymer, petrochemical, and plastic
industries. Ethylene is produced industrially in a process called cracking, in which the long hydrocarbon chains in a petroleum
mixture are broken into smaller molecules.

20.1.4: Recycling Plastics


Polymers (from Greek words poly meaning “many” and mer meaning “parts”) are large molecules made up of repeating units,
referred to as monomers. Polymers can be natural (starch is a polymer of sugar residues and proteins are polymers of amino acids)
or synthetic [like polyethylene, polyvinyl chloride (PVC), and polystyrene]. The variety of structures of polymers translates into a
broad range of properties and uses that make them integral parts of our everyday lives. Adding functional groups to the structure of
a polymer can result in significantly different properties (see the discussion about Kevlar later in this chapter).
An example of a polymerization reaction is shown in Figure 20.1.7. The monomer ethylene (C2H4) is a gas at room temperature,
but when polymerized, using a transition metal catalyst, it is transformed into a solid material made up of long chains of –CH2–
units called polyethylene. Polyethylene is a commodity plastic used primarily for packaging (bags and films).

Figure 20.1.7 : The reaction for the polymerization of ethylene to polyethylene is shown.
This diagram has three rows, showing ethylene reacting to form polyethylene. In the first row, Lewis structural formulas show three
molecules of ethylene being added together, which are each composed of two doubly bonded C atoms, each with two bonded H
atoms. Ellipses, or three dots, are present before and after the molecule structures, which in turn are followed by an arrow pointing
right. On the right side of the arrow, the ellipses or dots again appear to the left of a dash that connects to a chain of 7 C atoms, each
with H atoms connected above and below. A dash appears at the end of the chain, which in turn is followed by ellipses or dots. The
reaction diagram is repeated in the second row using ball-and-stick models for the structures. In these representations, single bonds
are represented with sticks, double bonds are represented with two parallel sticks, and elements are represented with balls. Carbon
atoms are black and hydrogen atoms are white in this image. In the third row, space-filling models are shown. In these models,
atoms are enlarged spheres which are pushed together, without sticks to represent bonds.
Polyethylene is a member of one subset of synthetic polymers classified as plastics. Plastics are synthetic organic solids that can be
molded; they are typically organic polymers with high molecular masses. Most of the monomers that go into common plastics
(ethylene, propylene, vinyl chloride, styrene, and ethylene terephthalate) are derived from petrochemicals and are not very

Access for free at OpenStax 20.1.11 https://chem.libretexts.org/@go/page/38330


biodegradable, making them candidate materials for recycling. Recycling plastics helps minimize the need for using more of the
petrochemical supplies and also minimizes the environmental damage caused by throwing away these nonbiodegradable materials.
Plastic recycling is the process of recovering waste, scrap, or used plastics, and reprocessing the material into useful products. For
example, polyethylene terephthalate (soft drink bottles) can be melted down and used for plastic furniture, in carpets, or for other
applications. Other plastics, like polyethylene (bags) and polypropylene (cups, plastic food containers), can be recycled or
reprocessed to be used again. Many areas of the country have recycling programs that focus on one or more of the commodity
plastics that have been assigned a recycling code (Figure 20.1.8). These operations have been in effect since the 1970s and have
made the production of some plastics among the most efficient industrial operations today.

Figure 20.1.8 : Each type of recyclable plastic is imprinted with a code for easy identification.
This table shows recycling symbols, names, and uses of various types of plastics. Symbols are shown with three arrows in a
triangular shape surrounding a number. Number 1 is labeled P E T E. The related plastic, polyethylene terephthalate (P E T E), is
used in soda bottles and oven-ready food trays. Number 2 is labeled H D P E. The related plastic is high-density polyethylene (H D
P E), which is used in bottles for milk and dishwashing liquids. Number 3 is labeled V. The related plastic is polyvinyl chloride or
(P V C). This plastic is used in food trays, plastic wrap, and bottles for mineral water and shampoo. Number 4 is labeled L D P E.
This plastic is low density polyethylene (L D P E). It is used in shopping bags and garbage bags. Number 5 is labeled P P. The
related plastic is polypropylene (P P). It is used in margarine tubs and microwaveable food trays. Number 6 is labeled P S. The
related plastic is polystyrene (P S). It is used in yogurt tubs, foam meat trays, egg cartons, vending cups, plastic cutlery, and
packaging for electronics and toys. Number 7 is labeled other for any other plastics. Items in this category include those plastic
materials that do not fit any other category. Melamine used in plastic plates and cups is an example.
The name of an alkene is derived from the name of the alkane with the same number of carbon atoms. The presence of the double
bond is signified by replacing the suffix -ane with the suffix -ene. The location of the double bond is identified by naming the
smaller of the numbers of the carbon atoms participating in the double bond:

Access for free at OpenStax 20.1.12 https://chem.libretexts.org/@go/page/38330


Four structural formulas and names are shown. The first shows two red C atoms connected by a red double bond illustrated with
two parallel line segments. H atoms are bonded above and below to the left of the left-most C atom. Two more H atoms are
similarly bonded to the right of the C atom on the right. Beneath this structure the name ethene and alternate name ethylene are
shown. The second shows three C atoms bonded together with a red double bond between the red first and second C atoms moving
left to right across the three-carbon chain. H atoms are bonded above and below to the left of the C atom to the left. A single H is
bonded above the middle C atom. Three more H atoms are bonded above, below, and to the right of the third C atom. Beneath this
structure the name propene and alternate name propylene is shown. The third shows four C atoms bonded together, numbered one
through four moving left to right with a red double bond between the red first and second carbon in the chain. H atoms are bonded
above and below to the left of the C atom to the left. A single H is bonded above the second C atom. H atoms are bonded above and
below the third C atom. Three more H atoms are bonded above, below, and to the right of the fourth C atom. Beneath this structure
the name 1 dash butene is shown. The fourth shows four C atoms bonded together, numbered one through four moving left to right
with a red double bond between the red second and third C atoms in the chain. H atoms are bonded above, below, and to the left of
the left-most C atom. A single H atom is bonded above the second C atom. A single H atom is bonded above the third C atom.
Three more H atoms are bonded above, below, and to the right of the fourth C atom. Beneath this structure the name 2 dash butene
is shown.

20.1.5: Isomers of Alkenes


Molecules of 1-butene and 2-butene are structural isomers; the arrangement of the atoms in these two molecules differs. As an
example of arrangement differences, the first carbon atom in 1-butene is bonded to two hydrogen atoms; the first carbon atom in 2-
butene is bonded to three hydrogen atoms.
The compound 2-butene and some other alkenes also form a second type of isomer called a geometric isomer. In a set of geometric
isomers, the same types of atoms are attached to each other in the same order, but the geometries of the two molecules differ.
Geometric isomers of alkenes differ in the orientation of the groups on either side of a C = C bond.
Carbon atoms are free to rotate around a single bond but not around a double bond; a double bond is rigid. This makes it possible to
have two isomers of 2-butene, one with both methyl groups on the same side of the double bond and one with the methyl groups on
opposite sides. When structures of butene are drawn with 120° bond angles around the sp2-hybridized carbon atoms participating in
the double bond, the isomers are apparent. The 2-butene isomer in which the two methyl groups are on the same side is called a cis-
isomer; the one in which the two methyl groups are on opposite sides is called a trans-isomer (Figure 20.1.9). The different
geometries produce different physical properties, such as boiling point, that may make separation of the isomers possible:

Access for free at OpenStax 20.1.13 https://chem.libretexts.org/@go/page/38330


Figure 20.1.9 : These molecular models show the structural and geometric isomers of butene.
The figure illustrates three ways to represent isomers of butene. In the first row of the figure, Lewis structural formulas show
carbon and hydrogen element symbols and bonds between the atoms. The first structure in this row shows a C atom with a double
bond to another C atom which is bonded down and to the right to C H subscript 2 which, in turn, is bonded to C H subscript 3. The
first C atom, moving from left to right, has two H atoms bonded to it and the second C atom has one H atom bonded to it. The
second structure in the row shows a C atom with a double bond to another C atom. The first C atom is bonded to an H atom up and
to the left and C H subscript 3 down and to the left. The second C atom is bonded to an H atom up and to the right and C H
subscript 3 down and to the right. Both C H subscript 3 structures appear in red. The third structure shows a C atom with a double
bond to another C atom. The first C atom from the left is bonded up to a the left to C H subscript 3 which appears and red. It is also
bonded down and to the left to an H atom. The second C atom is bonded up and to the right to an H atom and down and to the left
to C H subscript 3 which appears in red. In the second row, ball-and-stick models for the structures are shown. In these
representations, single bonds are represented with sticks, double bonds are represented with two parallel sticks, and elements are
represented with balls. C atoms are black and H atoms are white in this image. In the third row, space-filling models are shown. In
these models, atoms are enlarged and pushed together, without sticks to represent bonds. In the final row, names are provided. The
molecule with the double bond between the first and second carbons is named 1 dash butene. The two molecules with the double
bond between the second and third carbon atoms is called 2 dash butene. The first model, which has both C H subscript 3 groups
beneath the double bond is called the cis isomer. The second which has the C H subscript 3 groups on opposite sides of the double
bond is named the trans isomer.
Alkenes are much more reactive than alkanes because the C = C moiety is a reactive functional group. A π bond, being a weaker
bond, is disrupted much more easily than a σ bond. Thus, alkenes undergo a characteristic reaction in which the π bond is broken
and replaced by two σ bonds. This reaction is called an addition reaction. The hybridization of the carbon atoms in the double bond
in an alkene changes from sp2 to sp3 during an addition reaction. For example, halogens add to the double bond in an alkene instead
of replacing hydrogen, as occurs in an alkane:

This diagram illustrates the reaction of ethene and C l subscript 2 to form 1 comma 2 dash dichloroethane. In this reaction, the
structural formula of ethane is shown. It has a double bond between the two C atoms with two H atoms bonded to each C atom plus
C l bonded to C l. This is shown on to the left of an arrow. The two C atoms and the double bond between them are shown in red.
To the right of the arrow, the 1 comma 2 dash dichloroethane molecule is shown. It has only single bonds and each C atom has a C l
with three pairs of electron dots bonded beneath it. The C and C l atoms, single bond between them, and electron pairs are shown in
red. Each C atom also has two H atoms bonded to it.

 Example 20.1.5: Alkene Reactivity and Naming


Provide the IUPAC names for the reactant and product of the halogenation reaction shown here:

Access for free at OpenStax 20.1.14 https://chem.libretexts.org/@go/page/38330


The left side of a reaction and arrow are shown with an empty product side. On the left, C H subscript 3 is bonded down and to
the right to C H which has a double bond to another C H. The second C H is bonded up and to the right to C H subscript 2
which is also bonded to C H subscript 3. A plus sign is shown with a C l atom bonded to a C l atom following it. This is also
followed by a reaction arrow.

Solution
The reactant is a five-carbon chain that contains a carbon-carbon double bond, so the base name will be pentene. We begin
counting at the end of the chain closest to the double bond—in this case, from the left—the double bond spans carbons 2 and 3,
so the name becomes 2-pentene. Since there are two carbon-containing groups attached to the two carbon atoms in the double
bond—and they are on the same side of the double bond—this molecule is the cis-isomer, making the name of the starting
alkene cis-2-pentene. The product of the halogenation reaction will have two chlorine atoms attached to the carbon atoms that
were a part of the carbon-carbon double bond:

C H subscript 3 is bonded down and to the right to C H which is bonded down and to the left to C l. C H is also bonded to
another C H which is bonded down and to the right to C l and up and to the right to C H subscript 2. C H subscript 2 is also
bonded to C H subscript 3.
This molecule is now a substituted alkane and will be named as such. The base of the name will be pentane. We will count
from the end that numbers the carbon atoms where the chlorine atoms are attached as 2 and 3, making the name of the product
2,3-dichloropentane.

 Exercise 20.1.5

Provide names for the reactant and product of the reaction shown:

This shows a C atom bonded to three H atoms and another C atom. This second C atom is bonded to two H atoms and a third C
atom. This third C atom is bonded to one H atom and also forms a double bond with a fourth C atom. This fourth C atom is
bonded to one H atom and a fifth C atom. This fifth C atom is bonded to two H atoms and a sixth C atom. This sixth C atom is
bonded to three H atoms. There is a plus sign followed by a C l atom bonded to another C l atom. There is a reaction arrow. no
products are shown.

Answer
reactant: cis-3-hexene, product: 3,4-dichlorohexane

20.1.6: Alkynes
Hydrocarbon molecules with one or more triple bonds are called alkynes; they make up another series of unsaturated hydrocarbons.
Two carbon atoms joined by a triple bond are bound together by one σ bond and two π bonds. The sp-hybridized carbons involved
in the triple bond have bond angles of 180°, giving these types of bonds a linear, rod-like shape.

Access for free at OpenStax 20.1.15 https://chem.libretexts.org/@go/page/38330


The simplest member of the alkyne series is ethyne, C2H2, commonly called acetylene. The Lewis structure for ethyne, a linear
molecule, is:

The structural formula and name for ethyne, also known as acetylene, are shown. In red, two C atoms are shown with a triple bond
illustrated by three horizontal line segments between them. Shown in black at each end of the structure, a single H atom is bonded.
The IUPAC nomenclature for alkynes is similar to that for alkenes except that the suffix -yne is used to indicate a triple bond in the
chain. For example, C H C H C ≡ CH is called 1-butyne.
3 2

 Example 20.1.6: Structure of Alkynes

Describe the geometry and hybridization of the carbon atoms in the following molecule:

A structural formula is shown with C H subscript 3 bonded to a C atom which is triple bonded to another C atom which is
bonded to C H subscript 3. Each C atom is labeled 1, 2, 3, and 4 from left to right.

Solution
Carbon atoms 1 and 4 have four single bonds and are thus tetrahedral with sp3 hybridization. Carbon atoms 2 and 3 are
involved in the triple bond, so they have linear geometries and would be classified as sp hybrids.

 Exercise 20.1.6

Identify the hybridization and bond angles at the carbon atoms in the molecule shown:

A structural formula is shown with an H atom bonded to a C atom. The C atom has a triple bond with another C atom which is
also bonded to C H. The C H has a double bond with another C H which is also bonded up and to the right to C H subscript 3.
Each C atom is labeled 1, 2, 3, 4, or 5 from left to right.

Answer
carbon 1: sp, 180°; carbon 2: sp, 180°; carbon 3: sp2, 120°; carbon 4: sp2, 120°; carbon 5: sp3, 109.5°

Chemically, the alkynes are similar to the alkenes. Since the C ≡ C functional group has two π bonds, alkynes typically react even
more readily, and react with twice as much reagent in addition reactions. The reaction of acetylene with bromine is a typical
example:

Access for free at OpenStax 20.1.16 https://chem.libretexts.org/@go/page/38330


This diagram illustrates the reaction of ethyne and two molecules of B r subscript 2 to form 1 comma 1 comma 2 comma 2 dash
tetrabromoethane. In this reaction, the structural formula of ethyne, an H atom bonded to a red C atom with a red triple bond to
another red C atom bonded to a black H atom, plus B r bonded to B r plus B r bonded to B r is shown to the left of an arrow. On the
right, the form 1 comma 1 comma 2 comma 2 dash tetrabromoethane molecule is shown. It has an H atom bonded to a C atom
which is bonded to another C atom which is bonded to an H atom. Each C atom is bonded above and below to a B r atom. Each B r
atom has three pairs of electron dots. The C and B r atoms, single bond between them, and electron pairs are shown in red.
Acetylene and the other alkynes also burn readily. An acetylene torch takes advantage of the high heat of combustion for acetylene.

20.1.7: Aromatic Hydrocarbons


Benzene, C6H6, is the simplest member of a large family of hydrocarbons, called aromatic hydrocarbons. These compounds contain
ring structures and exhibit bonding that must be described using the resonance hybrid concept of valence bond theory or the
delocalization concept of molecular orbital theory. (To review these concepts, refer to the earlier chapters on chemical bonding).
The resonance structures for benzene, C6H6, are:

Figure 20.1.10.
This structural formula shows a six carbon hydrocarbon ring. On the left side there are six C atoms. The C atom on top and to the
left forms a single bond to the C atom on the top and to the right. The C atom has a double bond to another C atom which has a
single bond to a C atom. That C atom has a double bond to another C atom which has a single bond to a C atom. That C atom
forms a double bond with another C atom. Each C atom has a single bond to an H atom. There is a double sided arrow and the
structure on the right is almost identical to the structure on the left. The structure on the right shows double bonds where the
structure on the left showed single bonds. The structure on the right shows single bonds where the stucture on the left showed
double bonds.

Figure 20.1.10: This condensed formula shows the unique bonding structure of benzene.
A six carbon hydrocarbon ring structural formula is shown. Each C atom is bonded to only one H atom. A circle is at the center of
the ring.
There are many derivatives of benzene. The hydrogen atoms can be replaced by many different substituents. Aromatic compounds
more readily undergo substitution reactions than addition reactions; replacement of one of the hydrogen atoms with another
substituent will leave the delocalized double bonds intact. The following are typical examples of substituted benzene derivatives:

Access for free at OpenStax 20.1.17 https://chem.libretexts.org/@go/page/38330


Three structural formulas are shown. The first is labeled toluene. This molecule has a six carbon hydrocarbon ring in which five of
the C atoms are each bonded to only one H atom. At the upper right of the ring, the C atom that does not have a bonded H atom has
a red C H subscript 3 group attached. A circle is at the center of the ring. The second is labeled xylene. This molecule has a six
carbon hydrocarbon ring in which four of the C atoms are each bonded to only one H atom. At the upper right and right of the ring,
the two C atoms that do not have bonded H atoms have C H subscript 3 groups attached. These C H subscript 3 groups appear in
red. A circle is at the center of the ring. The third is labeled styrene. This molecule has a six carbon hydrocarbon ring in which five
of the carbon atoms are each bonded to only one H atom. At the upper right of the ring, the carbon that does not have a bonded H
atom has a red C H double bond C H subscript 2 group attached. A circle is at the center of the ring.

Toluene and xylene are important solvents and raw materials in the chemical industry. Styrene is used to produce the polymer
polystyrene.

 Example 20.1.7: Structure of Aromatic Hydrocarbons

One possible isomer created by a substitution reaction that replaces a hydrogen atom attached to the aromatic ring of toluene
with a chlorine atom is shown here. Draw two other possible isomers in which the chlorine atom replaces a different hydrogen
atom attached to the aromatic ring:

Two structural formulas are shown. The first has a six carbon hydrocarbon ring in which four of the carbon atoms are each
bonded to only one H atom. At the upper right of the ring, the carbon that does not have a bonded H atom has a C H subscript 3
group attached. The C to the lower right has a C l atom attached. A circle is at the center of the ring. The second molecule has a
hexagon with a circle inside. From a vertex of the hexagon at the upper right a C H subscript 3 group is attached. From the
vertex at the lower right, a C l atom is attached.

Solution
Since the six-carbon ring with alternating double bonds is necessary for the molecule to be classified as aromatic, appropriate
isomers can be produced only by changing the positions of the chloro-substituent relative to the methyl-substituent:

Two pairs of structural formulas are shown. The first has a six carbon hydrocarbon ring in which four of the C atoms are each
bonded to only one H atom. At the upper right of the ring, the C atom that does not have a bonded H atom has a C H subscript
3 group attached. The C atom to the right has a C l atom attached. A circle is at the center of the ring. The second molecule in
the first pair has a hexagon with a circle inside. From a vertex of the hexagon at the upper right a C H subscript 3 group is
attached. From the vertex at the right, a C l atom is attached. The second pair first shows a six carbon hydrocarbon ring in
which four of the C atoms are each bonded to only one H atom. A C l atom is attached to the left-most C atom and a C H
subscript 3 group is attached to the right-most C atom. A circle is at the center of the ring. The second molecule in the pair has
a hexagon with a circle inside. A C H subscript 3 group is attached to a vertex on the right side of the hexagon and to a vertex
on the left side, a C l atom is bonded.

Access for free at OpenStax 20.1.18 https://chem.libretexts.org/@go/page/38330


 Exercise 20.1.7
Draw three isomers of a six-membered aromatic ring compound substituted with two bromines.

Answer

Three pairs of structural formulas are shown. The first has a six carbon hydrocarbon ring in which four of the C atoms are
each bonded to only one H atom. At the upper right and right of the ring, the two C atoms that do not have bonded H atoms
have one B r atom bonded each. A circle is at the center of the ring. Beneath this structure, a similar structure is shown
which has a hexagon with a circle inside. From vertices of the hexagon at the upper right and right single B r atoms are
attached. The second has a six carbon hydrocarbon ring in which four of the C atoms are each bonded to only one H atom.
At the upper right and lower right of the ring, the two C atoms that do not have bonded H atoms have a single B r atom
bonded each. A circle is at the center of the ring. Beneath this structure, a similar structure is shown which has a hexagon
with a circle inside. From vertices of the hexagon at the upper right and lower right single B r atoms are attached. The third
has a six carbon hydrocarbon ring in which four of the C atoms are each bonded to only one H atom. At the upper right and
lower left of the ring, the two C atoms that do not have bonded H atoms have B r atoms bonded. A circle is at the center of
the ring. Beneath this structure, a similar structure is shown which has a hexagon with a circle inside. From vertices of the
hexagon at the upper right and lower left, single B r atoms are attached.

Summary
Strong, stable bonds between carbon atoms produce complex molecules containing chains, branches, and rings. The chemistry of
these compounds is called organic chemistry. Hydrocarbons are organic compounds composed of only carbon and hydrogen. The
alkanes are saturated hydrocarbons—that is, hydrocarbons that contain only single bonds. Alkenes contain one or more carbon-
carbon double bonds. Alkynes contain one or more carbon-carbon triple bonds. Aromatic hydrocarbons contain ring structures with
delocalized π electron systems.

Footnotes
1. This is the Beilstein database, now available through the Reaxys site (www.elsevier.com/online-tools/reaxys).
2. Peplow, Mark. “Organic Synthesis: The Robo-Chemist,” Nature 512 (2014): 20–2.
3. Physical properties for C4H10 and heavier molecules are those of the normal isomer, n-butane, n-pentane, etc.
4. STP indicates a temperature of 0 °C and a pressure of 1 atm.

Glossary
addition reaction
reaction in which a double carbon-carbon bond forms a single carbon-carbon bond by the addition of a reactant. Typical
reaction for an alkene.

alkane

Access for free at OpenStax 20.1.19 https://chem.libretexts.org/@go/page/38330


molecule consisting of only carbon and hydrogen atoms connected by single (σ) bonds

alkene
molecule consisting of carbon and hydrogen containing at least one carbon-carbon double bond

alkyl group
substituent, consisting of an alkane missing one hydrogen atom, attached to a larger structure

alkyne
molecule consisting of carbon and hydrogen containing at least one carbon-carbon triple bond

aromatic hydrocarbon
cyclic molecule consisting of carbon and hydrogen with delocalized alternating carbon-carbon single and double bonds,
resulting in enhanced stability

functional group
part of an organic molecule that imparts a specific chemical reactivity to the molecule

organic compound
natural or synthetic compound that contains carbon

saturated hydrocarbon
molecule containing carbon and hydrogen that has only single bonds between carbon atoms

skeletal structure
shorthand method of drawing organic molecules in which carbon atoms are represented by the ends of lines and bends in
between lines, and hydrogen atoms attached to the carbon atoms are not shown (but are understood to be present by the context
of the structure)

substituent
branch or functional group that replaces hydrogen atoms in a larger hydrocarbon chain

substitution reaction
reaction in which one atom replaces another in a molecule

This page titled 20.1: Hydrocarbons is shared under a CC BY 4.0 license and was authored, remixed, and/or curated by OpenStax via source
content that was edited to the style and standards of the LibreTexts platform; a detailed edit history is available upon request.

Access for free at OpenStax 20.1.20 https://chem.libretexts.org/@go/page/38330


20.2: Alcohols and Ethers
 Learning Objectives
Describe the structure and properties of alcohols
Describe the structure and properties of ethers
Name and draw structures for alcohols and ethers

In this section, we will learn about alcohols and ethers.

20.2.1: Alcohols
Incorporation of an oxygen atom into carbon- and hydrogen-containing molecules leads to new functional groups and new families
of compounds. When the oxygen atom is attached by single bonds, the molecule is either an alcohol or ether.
Alcohols are derivatives of hydrocarbons in which an –OH group has replaced a hydrogen atom. Although all alcohols have one or
more hydroxyl (–OH) functional groups, they do not behave like bases such as NaOH and KOH. NaOH and KOH are ionic
compounds that contain OH– ions. Alcohols are covalent molecules; the –OH group in an alcohol molecule is attached to a carbon
atom by a covalent bond.
Ethanol, CH3CH2OH, also called ethyl alcohol, is a particularly important alcohol for human use. Ethanol is the alcohol produced
by some species of yeast that is found in wine, beer, and distilled drinks. It has long been prepared by humans harnessing the
metabolic efforts of yeasts in fermenting various sugars:

Large quantities of ethanol are synthesized from the addition reaction of water with ethylene using an acid as a catalyst:

This reaction shows two carbons connected by a double bond, each with two bonded H atoms plus H O H arrow labeled “H
subscript 3 O superscript plus” followed by two carbon atoms connected with a single bond with 5 bonded H atoms and an O H
group shown in red at the right end of the molecule. The O of this group is shown with 2 pairs of electron dots.
Alcohols containing two or more hydroxyl groups can be made. Examples include 1,2-ethanediol (ethylene glycol, used in
antifreeze) and 1,2,3-propanetriol (glycerine, used as a solvent for cosmetics and medicines):

20.2.2: Naming Alcohols


The name of an alcohol comes from the hydrocarbon from which it was derived. The final -e in the name of the hydrocarbon is
replaced by -ol, and the carbon atom to which the –OH group is bonded is indicated by a number placed before the name.1

Access for free at OpenStax 20.2.1 https://chem.libretexts.org/@go/page/38331


 Example 20.2.1: Naming Alcohols

Consider the following example. How should it be named?

Solution
The carbon chain contains five carbon atoms. If the hydroxyl group was not present, we would have named this molecule
pentane. To address the fact that the hydroxyl group is present, we change the ending of the name to -ol. In this case, since the
–OH is attached to carbon 2 in the chain, we would name this molecule 2-pentanol.

 Exercise 20.2.1

Name the following molecule:

The structure shown has a C H subscript 3 group bonded up and to the right to a C atom. The C atom is bonded down and to
the right to a C H subscript 2 group. The C H subscript 2 group is bonded up and to the right to a C H subscript 2 group. The C
H subscript 2 group is bonded down and to the right to a C H subscript 3 group. The second C atom (from left to right) is
bonded to a C H subscript 3 group and an O H group.

Answer
2-methyl-2-pentanol

20.2.3: Ethers
Ethers are compounds that contain the functional group –O–. Ethers do not have a designated suffix like the other types of
molecules we have named so far. In the IUPAC system, the oxygen atom and the smaller carbon branch are named as an alkoxy
substituent and the remainder of the molecule as the base chain, as in alkanes. As shown in the following compound, the red
symbols represent the smaller alkyl group and the oxygen atom, which would be named “methoxy.” The larger carbon branch
would be ethane, making the molecule methoxyethane. Many ethers are referred to with common names instead of the IUPAC
system names. For common names, the two branches connected to the oxygen atom are named separately and followed by “ether.”
The common name for the compound shown in below is ethylmethyl ether:

 Example 20.2.2: Naming Ethers


Provide the IUPAC and common name for the ether shown here:

Solution

Access for free at OpenStax 20.2.2 https://chem.libretexts.org/@go/page/38331


IUPAC name: The molecule is made up of an ethoxy group attached to an ethane chain, so the IUPAC name would be
ethoxyethane.
Common name: The groups attached to the oxygen atom are both ethyl groups, so the common name would be diethyl
ether.

 Exercise 20.2.2

Provide the IUPAC and common name for the ether shown:

A molecular structure shows a C H subscript 3 group bonded up and to the right to an O atom. The O atom is bonded down and
to the right to a C H group. The C H group is bonded up and to the right to a C H subscript 3 group. The C H group is also
bonded down and to the right to another C H subscript 3 group.

Answer
IUPAC: 2-methoxypropane; common: isopropylmethyl ether

Ethers can be obtained from alcohols by the elimination of a molecule of water from two molecules of the alcohol. For example,
when ethanol is treated with a limited amount of sulfuric acid and heated to 140 °C, diethyl ether and water are formed:

In the general formula for ethers, R—O—R, the hydrocarbon groups (R) may be the same or different. Diethyl ether, the most
widely used compound of this class, is a colorless, volatile liquid that is highly flammable. It was first used in 1846 as an
anesthetic, but better anesthetics have now largely taken its place. Diethyl ether and other ethers are presently used primarily as
solvents for gums, fats, waxes, and resins. Tertiary-butyl methyl ether, C4H9OCH3 (abbreviated MTBE—italicized portions of
names are not counted when ranking the groups alphabetically—so butyl comes before methyl in the common name), is used as an
additive for gasoline. MTBE belongs to a group of chemicals known as oxygenates due to their capacity to increase the oxygen
content of gasoline.

 Carbohydrates and Diabetes

Carbohydrates are large biomolecules made up of carbon, hydrogen, and oxygen. The dietary forms of carbohydrates are foods
rich in these types of molecules, like pastas, bread, and candy. The name “carbohydrate” comes from the formula of the
molecules, which can be described by the general formula Cm(H2O)n, which shows that they are in a sense “carbon and water”
or “hydrates of carbon.” In many cases, m and n have the same value, but they can be different. The smaller carbohydrates are
generally referred to as “sugars,” the biochemical term for this group of molecules is “saccharide” from the Greek word for
sugar (Figure 20.2.1). Depending on the number of sugar units joined together, they may be classified as monosaccharides
(one sugar unit), disaccharides (two sugar units), oligosaccharides (a few sugars), or polysaccharides (the polymeric version of
sugars—polymers were described in the feature box earlier in this chapter on recycling plastics). The scientific names of sugars
can be recognized by the suffix -ose at the end of the name (for instance, fruit sugar is a monosaccharide called “fructose” and
milk sugar is a disaccharide called lactose composed of two monosaccharides, glucose and galactose, connected together).
Sugars contain some of the functional groups we have discussed: Note the alcohol groups present in the structures and how
monosaccharide units are linked to form a disaccharide by formation of an ether.

Access for free at OpenStax 20.2.3 https://chem.libretexts.org/@go/page/38331


Figure 20.2.1 : The illustrations show the molecular structures of fructose, a five-carbon monosaccharide, and of lactose, a
disaccharide composed of two isomeric, six-carbon sugars.
Organisms use carbohydrates for a variety of functions. Carbohydrates can store energy, such as the polysaccharides glycogen
in animals or starch in plants. They also provide structural support, such as the polysaccharide cellulose in plants and the
modified polysaccharide chitin in fungi and animals. The sugars ribose and deoxyribose are components of the backbones of
RNA and DNA, respectively. Other sugars play key roles in the function of the immune system, in cell-cell recognition, and in
many other biological roles.
Diabetes is a group of metabolic diseases in which a person has a high sugar concentration in their blood (Figure 20.2.2).
Diabetes may be caused by insufficient insulin production by the pancreas or by the body’s cells not responding properly to the
insulin that is produced. In a healthy person, insulin is produced when it is needed and functions to transport glucose from the
blood into the cells where it can be used for energy. The long-term complications of diabetes can include loss of eyesight, heart
disease, and kidney failure.

Figure 20.2.2 : Diabetes is a disease characterized by high concentrations of glucose in the blood. Treating diabetes involves
making lifestyle changes, monitoring blood-sugar levels, and sometimes insulin injections. (credit: “Blausen Medical
Communications”/Wikimedia Commons)
In 2013, it was estimated that approximately 3.3% of the world’s population (~380 million people) suffered from diabetes,
resulting in over a million deaths annually. Prevention involves eating a healthy diet, getting plenty of exercise, and
maintaining a normal body weight. Treatment involves all of these lifestyle practices and may require injections of insulin.

Summary
Many organic compounds that are not hydrocarbons can be thought of as derivatives of hydrocarbons. A hydrocarbon derivative
can be formed by replacing one or more hydrogen atoms of a hydrocarbon by a functional group, which contains at least one atom
of an element other than carbon or hydrogen. The properties of hydrocarbon derivatives are determined largely by the functional
group. The –OH group is the functional group of an alcohol. The –R–O–R– group is the functional group of an ether.

Access for free at OpenStax 20.2.4 https://chem.libretexts.org/@go/page/38331


Footnotes
1. The IUPAC adopted new nomenclature guidelines in 2013 that require this number to be placed as an “infix” rather than a
prefix. For example, the new name for 2-propanol would be propan-2-ol. Widespread adoption of this new nomenclature will
take some time, and students are encouraged to be familiar with both the old and new naming protocols.

Glossary
alcohol
organic compound with a hydroxyl group (–OH) bonded to a carbon atom

ether
organic compound with an oxygen atom that is bonded to two carbon atoms

This page titled 20.2: Alcohols and Ethers is shared under a CC BY 4.0 license and was authored, remixed, and/or curated by OpenStax via
source content that was edited to the style and standards of the LibreTexts platform; a detailed edit history is available upon request.

Access for free at OpenStax 20.2.5 https://chem.libretexts.org/@go/page/38331


20.3: Aldehydes, Ketones, Carboxylic Acids, and Esters
 Learning Objectives
Describe the structure and properties of aldehydes, ketones, carboxylic acids and esters

Another class of organic molecules contains a carbon atom connected to an oxygen atom by a double bond, commonly called a
carbonyl group. The trigonal planar carbon in the carbonyl group can attach to two other substituents leading to several subfamilies
(aldehydes, ketones, carboxylic acids and esters) described in this section.
20.3.0.0.1: Aldehydes and Ketones

Both aldehydes and ketones contain a carbonyl group, a functional group with a carbon-oxygen double bond. The names for
aldehyde and ketone compounds are derived using similar nomenclature rules as for alkanes and alcohols, and include the class-
identifying suffixes -al and -one, respectively.

Five structures are shown. The first is a C atom with an R group bonded to the left and an H atom to the right. An O atom is double
bonded above the C atom. This structure is labeled, “Functional group of an aldehyde.” The second structure shows a C atom with
R groups bonded to the left and right. An O atom is double bonded above the C atom. This structure is labeled, “Functional group
of a ketone.” The third structure looks exactly like the functional group of a ketone. The fourth structure is labeled C H subscript 3
C H O. It is also labeled, “An aldehyde,” and “ethanal (acetaldehyde).” This structure has a C atom to which 3 H atoms are bonded
above, below, and to the left. In red to the right of this C atom, a C atom is attached which has an O atom double bonded above and
an H atom bonded to the right. The O atom as two sets of electron dots. The fifth structure is labeled C H subscript 3 C O C H
subscript 2 C H subscript 3. It is also labeled, “A ketone,” and “butanone.” This structure has a C atom to which 3 H atoms are
bonded above, below, and to the left. To the right of this in red is a C atom to which an O atom is double bonded above. The O
atom has two sets of electron dots. Attached to the right of this red C atom in black is a two carbon atom chain with H atoms
attached above, below, and to the right.
In an aldehyde, the carbonyl group is bonded to at least one hydrogen atom. In a ketone, the carbonyl group is bonded to two
carbon atoms. As text, an aldehyde group is represented as –CHO; a ketone is represented as –C(O)– or –CO–.

In both aldehydes and ketones, the geometry around the carbon atom in the carbonyl group is trigonal planar; the carbon atom
exhibits sp2 hybridization. Two of the sp2 orbitals on the carbon atom in the carbonyl group are used to form σ bonds to the other
carbon or hydrogen atoms in a molecule. The remaining sp2 hybrid orbital forms a σ bond to the oxygen atom. The unhybridized p
orbital on the carbon atom in the carbonyl group overlaps a p orbital on the oxygen atom to form the π bond in the double bond.
Like the C = O bond in carbon dioxide, the C = O bond of a carbonyl group is polar (recall that oxygen is significantly more
electronegative than carbon, and the shared electrons are pulled toward the oxygen atom and away from the carbon atom). Many of
the reactions of aldehydes and ketones start with the reaction between a Lewis base and the carbon atom at the positive end of the
polar C = O bond to yield an unstable intermediate that subsequently undergoes one or more structural rearrangements to form the
final product (Figure 20.3.1).

Access for free at OpenStax 20.3.1 https://chem.libretexts.org/@go/page/38332


Figure 20.3.1 : The carbonyl group is polar, and the geometry of the bonds around the central carbon is trigonal planar.
This structure shows a central C atom to which an O atom is double bonded above. To the lower left, R superscript 1 is bonded and
to the lower right, R superscript 2 is bonded. A Greek lowercase delta superscript plus appears to the left of the C and just above
the bond with R superscript 1. Similarly, a Greek lowercase delta superscript negative sign appears to the left of the O atom. An arc
is drawn from the double bond that links the C atom and the O atom to the bond that links the C atom to the R superscript 2 group.
This arc is labeled approximately 120 degrees.
The importance of molecular structure in the reactivity of organic compounds is illustrated by the reactions that produce aldehydes
and ketones. We can prepare a carbonyl group by oxidation of an alcohol—for organic molecules, oxidation of a carbon atom is
said to occur when a carbon-hydrogen bond is replaced by a carbon-oxygen bond. The reverse reaction—replacing a carbon-
oxygen bond by a carbon-hydrogen bond—is a reduction of that carbon atom. Recall that oxygen is generally assigned a –2
oxidation number unless it is elemental or attached to a fluorine. Hydrogen is generally assigned an oxidation number of +1 unless
it is attached to a metal. Since carbon does not have a specific rule, its oxidation number is determined algebraically by factoring
the atoms it is attached to and the overall charge of the molecule or ion. In general, a carbon atom attached to an oxygen atom will
have a more positive oxidation number and a carbon atom attached to a hydrogen atom will have a more negative oxidation
number. This should fit nicely with your understanding of the polarity of C–O and C–H bonds. The other reagents and possible
products of these reactions are beyond the scope of this chapter, so we will focus only on the changes to the carbon atoms:

A reaction is shown. On the left appears an alcohol and on the right, a carbonyl group. Above the reaction arrow appears the word
“oxidation.” The alcohol is represented as a C atom with dashes to the left and below, an H atom bonded above, and an O atom
bonded to an H atom in red connected to the right. The O atom has two sets of electron dots. The carbonyl group is indicated in red
with a C atom to which an O atom is double bonded above. Dashes appear left and right of the C atom in black. The O atom has
two sets of electron dots.

 Example 20.3.1: Oxidation and Reduction in Organic Chemistry

Methane represents the completely reduced form of an organic molecule that contains one carbon atom. Sequentially replacing
each of the carbon-hydrogen bonds with a carbon-oxygen bond would lead to an alcohol, then an aldehyde, then a carboxylic
acid (discussed later), and, finally, carbon dioxide:
CH ⟶ CH OH ⟶ CH O ⟶ HCO H ⟶ CO
4 3 2 2 2

What are the oxidation numbers for the carbon atoms in the molecules shown here?

Solution
In this example, we can calculate the oxidation number (review the chapter on oxidation-reduction reactions if necessary) for
the carbon atom in each case (note how this would become difficult for larger molecules with additional carbon atoms and
hydrogen atoms, which is why organic chemists use the definition dealing with replacing C–H bonds with C–O bonds
described).
For CH4, the carbon atom carries a –4 oxidation number (the hydrogen atoms are assigned oxidation numbers of +1 and the
carbon atom balances that by having an oxidation number of –4)
For the alcohol (in this case, methanol), the carbon atom has an oxidation number of –2 (the oxygen atom is assigned –2,
the four hydrogen atoms each are assigned +1, and the carbon atom balances the sum by having an oxidation number of –2;
note that compared to the carbon atom in CH4, this carbon atom has lost two electrons so it was oxidized)

Access for free at OpenStax 20.3.2 https://chem.libretexts.org/@go/page/38332


For the aldehyde, the carbon atom’s oxidation number is 0 (–2 for the oxygen atom and +1 for each hydrogen atom already
balances to 0, so the oxidation number for the carbon atom is 0)
For the carboxylic acid, the carbon atom’s oxidation number is +2 (two oxygen atoms each at –2 and two hydrogen atoms
at +1)
For carbon dioxide, the carbon atom’s oxidation number is +4 (here, the carbon atom needs to balance the –4 sum from the
two oxygen atoms).

 Exercise 20.3.1

Indicate whether the marked carbon atoms in the three molecules here are oxidized or reduced relative to the marked carbon
atom in ethanol:

A molecular structure is shown. A C H subscript 3 group is bonded up and to the right to a C H subscript 2 group. Bonded to
the C H subscript 2 group down and to the right is an O H group.
There is no need to calculate oxidation states in this case; instead, just compare the types of atoms bonded to the marked
carbon atoms:

Three molecular structures are shown, each with a red central C atom. In a, a C H subscript 3 group is bonded to the lower left,
an H atom is bonded above, and H subscript 2 appears to the right of the central C atom. In b, an O atom is double bonded
above the central C atom, a C H subscript 3 group is bonded to the lower left, and an H atom is bonded to the lower right. In c,
an O atom is double bonded above the central C atom, a C H subscript 3 group is bonded to the lower left, and an O H group is
bonded to the lower right.

Answer a
reduced (bond to oxygen atom replaced by bond to hydrogen atom);
Answer b
oxidized (one bond to hydrogen atom replaced by one bond to oxygen atom);
Answer c
oxidized (2 bonds to hydrogen atoms have been replaced by bonds to an oxygen atom)

Aldehydes are commonly prepared by the oxidation of alcohols whose –OH functional group is located on the carbon atom at the
end of the chain of carbon atoms in the alcohol:

A reaction is shown. An alcohol appears on the left and an aldehyde on the right of the reaction arrow. The alcohol is shown as C H
subscript 3 C H subscript 2 C H subscript 2 O H, and the aldehyde is shown as C H subscript 3 C H subscript 2 C H O. The O H
group at the right end of the alcohol structure and the C H O group at the right end of the aldehyde structure are in red.
Alcohols that have their –OH groups in the middle of the chain are necessary to synthesize a ketone, which requires the carbonyl
group to be bonded to two other carbon atoms:

Access for free at OpenStax 20.3.3 https://chem.libretexts.org/@go/page/38332


A reaction is shown. An alcohol appears on the left and a ketone on the right of the reaction arrow. The alcohol is shown as C H
subscript 3 C H ( O H ) C H subscript 3 and the ketone is shown as C H subscript 3 C O C H subscript 3. The O H group in the
alcohol structure and the C O group at the center of the ketone structure are in red.
An alcohol with its –OH group bonded to a carbon atom that is bonded to no or one other carbon atom will form an aldehyde. An
alcohol with its –OH group attached to two other carbon atoms will form a ketone. If three carbons are attached to the carbon
bonded to the –OH, the molecule will not have a C–H bond to be replaced, so it will not be susceptible to oxidation.
Formaldehyde, an aldehyde with the formula HCHO, is a colorless gas with a pungent and irritating odor. It is sold in an aqueous
solution called formalin, which contains about 37% formaldehyde by weight. Formaldehyde causes coagulation of proteins, so it
kills bacteria (and any other living organism) and stops many of the biological processes that cause tissue to decay. Thus,
formaldehyde is used for preserving tissue specimens and embalming bodies. It is also used to sterilize soil or other materials.
Formaldehyde is used in the manufacture of Bakelite, a hard plastic having high chemical and electrical resistance.
Dimethyl ketone, CH3COCH3, commonly called acetone, is the simplest ketone. It is made commercially by fermenting corn or
molasses, or by oxidation of 2-propanol. Acetone is a colorless liquid. Among its many uses are as a solvent for lacquer (including
fingernail polish), cellulose acetate, cellulose nitrate, acetylene, plastics, and varnishes; as a paint and varnish remover; and as a
solvent in the manufacture of pharmaceuticals and chemicals.

20.3.1: Carboxylic Acids and Esters


The odor of vinegar is caused by the presence of acetic acid, a carboxylic acid, in the vinegar. The odor of ripe bananas and many
other fruits is due to the presence of esters, compounds that can be prepared by the reaction of a carboxylic acid with an alcohol.
Because esters do not have hydrogen bonds between molecules, they have lower vapor pressures than the alcohols and carboxylic
acids from which they are derived (Figure 20.3.2).

Access for free at OpenStax 20.3.4 https://chem.libretexts.org/@go/page/38332


Figure 20.3.2 : Esters are responsible for the odors associated with various plants and their fruits.
There are nine structures represented in this figure. The first is labeled, “raspberry,” and, “iso-butyl formate.” It shows an H atom
with a line going up and to the right which then goes down and to the right. It goes up and to the right again and down and to the
right and up and to the right. At the first peak is a double bond to an O atom. At the first trough is an O atom. At the second trough,
there is a line going straight down. The second is labeled, “apple,” and, “butyl acetate.” There is a line that goes up and to the right,
down and to the right, up and to the right, and down and to the right. At the second peak is a double bond to an O atom. At the end,
on the right is O C H subscript 3. The third is labeled, “pineapple,” and, “ethyl butyrate.” It is a line that goes up and to the right,
down and to the right, up and to the right, down and to the right, up and to the right, and down and to the right. At the second peak
is a double bond to an O atom and at the second trough is an O atom. The fourth is labeled, “rum,” and “propyl isobutyrate.” It
shows a line that goes down and to the right, up and to the right, down and to the right, up and to the right, down and to the right
and up and to the right. The first complete peak has a double bond to an O atom and the second trough has an O atom. The fifth is
labeled, “peach,” and “benzyl acetate.” It shows a line that goes up and to the right, down and to the right, up and to the right and
down and to the right. This line connects to a hexagon with a circle inside it. The first peak has a double bond to an O atom and the
first trough has an O atom. The sixth is labeled, “orange,” and, “octyl acetate.” It shows a line that goes up and to the right and
down and to the right and up and to the right and down and to the right and up and to the right and down and to the right and up and
to the right and down and to the right and up and to the right and down and to the right. The first peak has a double bond to an O
atom and the first complete trough has and an O atom. The seventh is labeled, “wintergreen,” and “methyl salicylate.” It shows a
hexagon with a circle inside of it. On the right, is a bond down and to the right to an O H group. On the right is a bond to a line that
goes up and to the right and down and two the right and up and to the right. At the first peak is a double bond to an O atom, the
next trough shows and O atom and at the end of the line is a C H subscript 3 group. The eighth is labeled, “honey,” and “methyl
phenylacetate.” It shows a hexagon with a circle inside of it. It shows it connecting to a line on the right that goes down and to the
right then up and to the right and down and to the right and up and to the right. At the first peak that is not part of the hexagon is a
double bond to an O atom. At the last trough is an O atom. The ninth is labeled, “strawberry,” and “ethyl methylphenylglycidate.”
This shows a hexagon with a circle inside of it. On the right, it connects to a line that goes up and to the right and down and to the
right and up and to the right and down and to the right and up and to the right and down and to the right. At the first peak is a line
that extends above and below. Below, it connects to an O atom. At the next trough, the line extends down and to the left to the same
O atom. At the next peak is a double bond to an O atom and at the next trough is an O atom.
Both carboxylic acids and esters contain a carbonyl group with a second oxygen atom bonded to the carbon atom in the carbonyl
group by a single bond. In a carboxylic acid, the second oxygen atom also bonds to a hydrogen atom. In an ester, the second
oxygen atom bonds to another carbon atom. The names for carboxylic acids and esters include prefixes that denote the lengths of
the carbon chains in the molecules and are derived following nomenclature rules similar to those for inorganic acids and salts (see
these examples):

Access for free at OpenStax 20.3.5 https://chem.libretexts.org/@go/page/38332


Two structures are shown. The first structure is labeled, “ethanoic acid,” and, “acetic acid.” This structure indicates a C atom to
which H atoms are bonded above, below and to the left. To the right of this in red is a bonded group comprised of a C atom to
which an O atom is double bonded above. To the right of the red C atom, an O atom is bonded which has an H atom bonded to its
right. Both O atoms have two sets of electron dots. The second structure is labeled, “methyl ethanoate,” and, “methyl acetate.” This
structure indicates a C atom to which H atoms are bonded above, below and to the left. In red, bonded to the right is a C atom with
a double bonded O atom above and a single bonded O atom to the right. To the right of this last O atom in black is another C atom
to which H atoms are bonded above, below and to the right. Both O atoms have two pairs of electron dots.
The functional groups for an acid and for an ester are shown in red in these formulas.
The hydrogen atom in the functional group of a carboxylic acid will react with a base to form an ionic salt:

A chemical reaction is shown. On the left, a structure of propionic acid is indicated. This structure includes a 2 carbon hydrocarbon
group on the left end in black. Above, below, and to the left, H atoms are bonded. This group is bonded to a red group comprised of
a C atom to which an O atom is double bonded above. To the right of the red C atom, an O atom is connected with a single bond.
To the right of the O atom, an H atom is bonded. To the right of this structure appears a plus and N a O H. Following the reaction
arrow, the propionate ion is shown. This structure is in brackets. Appearing inside the brackets, is a 2 carbon hydrocarbon group on
the left end. Above, below, and to the left, H atoms are bonded. To the right of this group, a group in red is attached comprised of a
C atom to which an O atom is double bonded above and a second O atom is single bonded to the right. Outside the brackets appears
a superscript minus symbol. This is followed by a plus sign, N a superscript plus another plus sign and H subscript 2 O. The singly
bonded O atom in the propionate ion structure has 3 pairs of electron dots. All other O atoms have two pairs of electron dots.
Carboxylic acids are weak acids, meaning they are not 100% ionized in water. Generally only about 1% of the molecules of a
carboxylic acid dissolved in water are ionized at any given time. The remaining molecules are undissociated in solution.
We prepare carboxylic acids by the oxidation of aldehydes or alcohols whose –OH functional group is located on the carbon atom
at the end of the chain of carbon atoms in the alcohol:

A chemical reaction with two arrows is shown. On the left, an alcohol, indicated with a C atom to which an R group is bonded to
the left, H atoms are bonded above and below, and in red, a single bonded O atom with an H atom bonded to the right is shown.
Following the first reaction arrow, an aldehyde is shown. This structure is represented with an R group bonded to a red C atom to
which an H atom is bonded above and to the right, and an O atom is double bonded below and to the right. Appearing to the right
of the second arrow, is a carboxylic acid comprised of an R group bonded to a C atom to which, in red, an O atom is single bonded
with an H atom bonded to its right side. A red O is double bonded below and to the right. All O atoms have two pairs of electron
dots.
Esters are produced by the reaction of acids with alcohols. For example, the ester ethyl acetate, CH3CO2CH2CH3, is formed when
acetic acid reacts with ethanol:

Access for free at OpenStax 20.3.6 https://chem.libretexts.org/@go/page/38332


A chemical reaction is shown. On the left, a C H subscript 3 group bonded to a red C atom. The C atom forms a double bond with
an O atom which is also in red. The C atom is also bonded to an O atom which is bonded to an H atom, also in red. A plus sign is
shown, which is followed by H O C H subscript 2 C H subscript 3. The H O group is in red. Following a reaction arrow, a C H
subscript 3 group is shown which is bonded to a red C atom with a double bonded O atom and a single bonded O. To the right of
this single bonded O atom, a C H subscript 2 C H subscript 3 group is attached and shown in black. This structure is followed by a
plus sign and H subscript 2 O. The O atoms in the first structure on the left and the structure following the reaction arrow have two
pairs of electron dots.

The simplest carboxylic acid is formic acid, HCO2H, known since 1670. Its name comes from the Latin word formicus, which
means “ant”; it was first isolated by the distillation of red ants. It is partially responsible for the pain and irritation of ant and wasp
stings, and is responsible for a characteristic odor of ants that can be sometimes detected in their nests.
Acetic acid, CH3CO2H, constitutes 3–6% vinegar. Cider vinegar is produced by allowing apple juice to ferment without oxygen
present. Yeast cells present in the juice carry out the fermentation reactions. The fermentation reactions change the sugar present in
the juice to ethanol, then to acetic acid. Pure acetic acid has a penetrating odor and produces painful burns. It is an excellent solvent
for many organic and some inorganic compounds, and it is essential in the production of cellulose acetate, a component of many
synthetic fibers such as rayon.
The distinctive and attractive odors and flavors of many flowers, perfumes, and ripe fruits are due to the presence of one or more
esters (Figure 20.3.3). Among the most important of the natural esters are fats (such as lard, tallow, and butter) and oils (such as
linseed, cottonseed, and olive oils), which are esters of the trihydroxyl alcohol glycerine, C3H5(OH)3, with large carboxylic acids,
such as palmitic acid, CH3(CH2)14CO2H, stearic acid, CH3(CH2)16CO2H, and oleic acid, C H (C H ) CH = CH(C H ) C O H.
3 2 7 2 7 2

Oleic acid is an unsaturated acid; it contains a C = C double bond. Palmitic and stearic acids are saturated acids that contain no
double or triple bonds.

Figure 20.3.3 : Over 350 different volatile molecules (many members of the ester family) have been identified in strawberries.
(credit: Rebecca Siegel)

Summary
Functional groups related to the carbonyl group include the –CHO group of an aldehyde, the –CO– group of a ketone, the –CO2H
group of a carboxylic acid, and the –CO2R group of an ester. The carbonyl group, a carbon-oxygen double bond, is the key
structure in these classes of organic molecules: Aldehydes contain at least one hydrogen atom attached to the carbonyl carbon
atom, ketones contain two carbon groups attached to the carbonyl carbon atom, carboxylic acids contain a hydroxyl group attached
to the carbonyl carbon atom, and esters contain an oxygen atom attached to another carbon group connected to the carbonyl carbon
atom. All of these compounds contain oxidized carbon atoms relative to the carbon atom of an alcohol group.

20.3.1.1: Glossary

aldehyde
organic compound containing a carbonyl group bonded to two hydrogen atoms or a hydrogen atom and a carbon substituent

carbonyl group
carbon atom double bonded to an oxygen atom

Access for free at OpenStax 20.3.7 https://chem.libretexts.org/@go/page/38332


carboxylic acid
organic compound containing a carbonyl group with an attached hydroxyl group

ester
organic compound containing a carbonyl group with an attached oxygen atom that is bonded to a carbon substituent

ketone
organic compound containing a carbonyl group with two carbon substituents attached to it

This page titled 20.3: Aldehydes, Ketones, Carboxylic Acids, and Esters is shared under a CC BY 4.0 license and was authored, remixed, and/or
curated by OpenStax via source content that was edited to the style and standards of the LibreTexts platform; a detailed edit history is available
upon request.

Access for free at OpenStax 20.3.8 https://chem.libretexts.org/@go/page/38332


20.4: Amines and Amides
 Learning Objectives
Describe the structure and properties of an amine
Describe the structure and properties of an amide

Amines are molecules that contain carbon-nitrogen bonds. The nitrogen atom in an amine has a lone pair of electrons and three
bonds to other atoms, either carbon or hydrogen. Various nomenclatures are used to derive names for amines, but all involve the
class-identifying suffix –ine as illustrated here for a few simple examples:

Figure 20.4.1 ) is one such heterocyclic amine. A heterocyclic compound contains atoms of two or more different elements in its
ring structure.

Figure 20.4.1 : The illustration shows one of the resonance structures of pyridine.

 DNA in Forensics and Paternity

The genetic material for all living things is a polymer of four different molecules, which are themselves a combination of three
subunits. The genetic information, the code for developing an organism, is contained in the specific sequence of the four
molecules, similar to the way the letters of the alphabet can be sequenced to form words that convey information. The
information in a DNA sequence is used to form two other types of polymers, one of which are proteins. The proteins interact to
form a specific type of organism with individual characteristics.
A genetic molecule is called DNA, which stands for deoxyribonucleic acid. The four molecules that make up DNA are called
nucleotides. Each nucleotide consists of a single- or double-ringed molecule containing nitrogen, carbon, oxygen, and
hydrogen called a nitrogenous base. Each base is bonded to a five-carbon sugar called deoxyribose. The sugar is in turn bonded
to a phosphate group (−PO ) When new DNA is made, a polymerization reaction occurs that binds the phosphate group of
3−

one nucleotide to the sugar group of a second nucleotide. The nitrogenous bases of each nucleotide stick out from this sugar-
phosphate backbone. DNA is actually formed from two such polymers coiled around each other and held together by hydrogen
bonds between the nitrogenous bases. Thus, the two backbones are on the outside of the coiled pair of strands, and the bases
are on the inside. The shape of the two strands wound around each other is called a double helix (Figure 20.4.2).
It probably makes sense that the sequence of nucleotides in the DNA of a cat differs from those of a dog. But it is also true that
the sequences of the DNA in the cells of two individual pugs differ. Likewise, the sequences of DNA in you and a sibling differ
(unless your sibling is an identical twin), as do those between you and an unrelated individual. However, the DNA sequences
of two related individuals are more similar than the sequences of two unrelated individuals, and these similarities in sequence
can be observed in various ways. This is the principle behind DNA fingerprinting, which is a method used to determine
whether two DNA samples came from related (or the same) individuals or unrelated individuals.

Access for free at OpenStax 20.4.1 https://chem.libretexts.org/@go/page/38333


Figure 20.4.2 : DNA is an organic molecule and the genetic material for all living organisms. (a) DNA is a double helix
consisting of two single DNA strands hydrogen bonded together at each nitrogenous base. (b) This detail shows the hydrogen
bonding (dotted lines) between nitrogenous bases on each DNA strand and the way in which each nucleotide is joined to the
next, forming a backbone of sugars and phosphate groups along each strand. (c) This detail shows the structure of one of the
four nucleotides that makes up the DNA polymer. Each nucleotide consists of a nitrogenous base (a double-ring molecule, in
this case), a five-carbon sugar (deoxyribose), and a phosphate group.
Using similarities in sequences, technicians can determine whether a man is the father of a child (the identity of the mother is
rarely in doubt, except in the case of an adopted child and a potential birth mother). Likewise, forensic geneticists can
determine whether a crime scene sample of human tissue, such as blood or skin cells, contains DNA that matches exactly the
DNA of a suspect.

Like ammonia, amines are weak bases due to the lone pair of electrons on their nitrogen atoms:

The basicity of an amine’s nitrogen atom plays an important role in much of the compound’s chemistry. Amine functional groups
are found in a wide variety of compounds, including natural and synthetic dyes, polymers, vitamins, and medications such as
penicillin and codeine. They are also found in many molecules essential to life, such as amino acids, hormones, neurotransmitters,
and DNA.

Access for free at OpenStax 20.4.2 https://chem.libretexts.org/@go/page/38333


20.4.1: Addictive Alkaloids
Since ancient times, plants have been used for medicinal purposes. One class of substances, called alkaloids, found in many of
these plants has been isolated and found to contain cyclic molecules with an amine functional group. These amines are bases. They
can react with H3O+ in a dilute acid to form an ammonium salt, and this property is used to extract them from the plant:
+ − + −
R N+H O + Cl ⟶ [ R NH ] Cl +H O
3 3 3 2

The name alkaloid means “like an alkali.” Thus, an alkaloid reacts with acid. The free compound can be recovered after extraction
by reaction with a base:
+ − − −
[ R NH ] Cl + OH ⟶ R N + H O + Cl
3 3 2

The structures of many naturally occurring alkaloids have profound physiological and psychotropic effects in humans. Examples of
these drugs include nicotine, morphine, codeine, and heroin. The plant produces these substances, collectively called secondary
plant compounds, as chemical defenses against the numerous pests that attempt to feed on the plant:

Figure 20.4.3 ).

Figure 20.4.3 : Poppies can be used in the production of opium, a plant latex that contains morphine from which other opiates, such
as heroin, can be synthesized. (credit: Karen Roe)
Amides are molecules that contain nitrogen atoms connected to the carbon atom of a carbonyl group. Like amines, various
nomenclature rules may be used to name amides, but all include use of the class-specific suffix -amide:

Access for free at OpenStax 20.4.3 https://chem.libretexts.org/@go/page/38333


Amides can be produced when carboxylic acids react with amines or ammonia in a process called amidation. A water molecule is
eliminated from the reaction, and the amide is formed from the remaining pieces of the carboxylic acid and the amine (note the
similarity to formation of an ester from a carboxylic acid and an alcohol discussed in the previous section):

The reaction between amines and carboxylic acids to form amides is biologically important. It is through this reaction that amino
acids (molecules containing both amine and carboxylic acid substituents) link together in a polymer to form proteins.

20.4.2: Proteins and Enzymes


Proteins are large biological molecules made up of long chains of smaller molecules called amino acids. Organisms rely on proteins
for a variety of functions—proteins transport molecules across cell membranes, replicate DNA, and catalyze metabolic reactions, to
name only a few of their functions. The properties of proteins are functions of the combination of amino acids that compose them
and can vary greatly. Interactions between amino acid sequences in the chains of proteins result in the folding of the chain into
specific, three-dimensional structures that determine the protein’s activity.
Amino acids are organic molecules that contain an amine functional group (–NH2), a carboxylic acid functional group (–COOH),
and a side chain (that is specific to each individual amino acid). Most living things build proteins from the same 20 different amino
acids. Amino acids connect by the formation of a peptide bond, which is a covalent bond formed between two amino acids when
the carboxylic acid group of one amino acid reacts with the amine group of the other amino acid. The formation of the bond results
in the production of a molecule of water (in general, reactions that result in the production of water when two other molecules
combine are referred to as condensation reactions). The resulting bond—between the carbonyl group carbon atom and the amine
nitrogen atom is called a peptide link or peptide bond. Since each of the original amino acids has an unreacted group (one has an
unreacted amine and the other an unreacted carboxylic acid), more peptide bonds can form to other amino acids, extending the
structure. (Figure 20.4.4) A chain of connected amino acids is called a polypeptide. Proteins contain at least one long polypeptide
chain.

Figure 20.4.4 : This condensation reaction forms a dipeptide from two amino acids and leads to the formation of water.

Access for free at OpenStax 20.4.4 https://chem.libretexts.org/@go/page/38333


Enzymes are large biological molecules, mostly composed of proteins, which are responsible for the thousands of metabolic
processes that occur in living organisms. Enzymes are highly specific catalysts; they speed up the rates of certain reactions.
Enzymes function by lowering the activation energy of the reaction they are catalyzing, which can dramatically increase the rate of
the reaction. Most reactions catalyzed by enzymes have rates that are millions of times faster than the noncatalyzed version. Like
all catalysts, enzymes are not consumed during the reactions that they catalyze. Enzymes do differ from other catalysts in how
specific they are for their substrates (the molecules that an enzyme will convert into a different product). Each enzyme is only
capable of speeding up one or a few very specific reactions or types of reactions. Since the function of enzymes is so specific, the
lack or malfunctioning of an enzyme can lead to serious health consequences. One disease that is the result of an enzyme
malfunction is phenylketonuria. In this disease, the enzyme that catalyzes the first step in the degradation of the amino acid
phenylalanine is not functional (Figure 20.4.5). Untreated, this can lead to an accumulation of phenylalanine, which can lead to
intellectual disabilities.

Figure 20.4.5 : A computer rendering shows the three-dimensional structure of the enzyme phenylalanine hydroxylase. In the
disease phenylketonuria, a defect in the shape of phenylalanine hydroxylase causes it to lose its function in breaking down
phenylalanine.

20.4.3: Kevlar
Kevlar (Figure 20.4.6) is a synthetic polymer made from two monomers 1,4-phenylene-diamine and terephthaloyl chloride (Kevlar
is a registered trademark of DuPont). Kevlar’s first commercial use was as a replacement for steel in racing tires. Kevlar is
typically spun into ropes or fibers. The material has a high tensile strength-to-weight ratio (it is about 5 times stronger than an equal
weight of steel), making it useful for many applications from bicycle tires to sails to body armor.

Figure 20.4.6 : This illustration shows the formula for polymeric Kevlar.
The material owes much of its strength to hydrogen bonds between polymer chains (refer back to the chapter on intermolecular
interactions). These bonds form between the carbonyl group oxygen atom (which has a partial negative charge due to oxygen’s
electronegativity) on one monomer and the partially positively charged hydrogen atom in the N–H bond of an adjacent monomer in
the polymer structure (dashed lines in Figure 20.4.7). There is additional strength derived from the interaction between the
unhybridized p orbitals in the six-membered rings, called aromatic stacking.

Access for free at OpenStax 20.4.5 https://chem.libretexts.org/@go/page/38333


Figure 20.4.7 : The diagram shows the polymer structure of Kevlar, with hydrogen bonds between polymer chains represented by
dotted lines.
Kevlar may be best known as a component of body armor, combat helmets, and face masks. Since the 1980s, the US military has
used Kevlar as a component of the PASGT (personal armor system for ground troops) helmet and vest. Kevlar is also used to
protect armored fighting vehicles and aircraft carriers. Civilian applications include protective gear for emergency service
personnel such as body armor for police officers and heat-resistant clothing for fire fighters. Kevlar based clothing is considerably
lighter and thinner than equivalent gear made from other materials (Figure 20.4.8).

Figure 20.4.8 : (a) These soldiers are sorting through pieces of a Kevlar helmet that helped absorb a grenade blast. Kevlar is also
used to make (b) canoes and (c) marine mooring lines. (credit a: modification of work by “Cla68”/Wikimedia Commons; credit b:
modification of work by “OakleyOriginals”/Flickr; credit c: modification of work by Casey H. Kyhl)
In addition to its better-known uses, Kevlar is also often used in cryogenics for its very low thermal conductivity (along with its
high strength). Kevlar maintains its high strength when cooled to the temperature of liquid nitrogen (–196 °C).
The table here summarizes the structures discussed in this chapter:

Access for free at OpenStax 20.4.6 https://chem.libretexts.org/@go/page/38333


Summary
The addition of nitrogen into an organic framework leads to two families of molecules. Compounds containing a nitrogen atom
bonded in a hydrocarbon framework are classified as amines. Compounds that have a nitrogen atom bonded to one side of a
carbonyl group are classified as amides. Amines are a basic functional group. Amines and carboxylic acids can combine in a
condensation reaction to form amides.

Glossary
amine
organic molecule in which a nitrogen atom is bonded to one or more alkyl group

amide
organic molecule that features a nitrogen atom connected to the carbon atom in a carbonyl group

This page titled 20.4: Amines and Amides is shared under a CC BY 4.0 license and was authored, remixed, and/or curated by OpenStax via
source content that was edited to the style and standards of the LibreTexts platform; a detailed edit history is available upon request.

Access for free at OpenStax 20.4.7 https://chem.libretexts.org/@go/page/38333


20.E: Organic Chemistry (Exercises)
20.E.1: 20.1: Hydrocarbons
20.E.1.1: Q20.1.1
Write the chemical formula and Lewis structure of the following, each of which contains five carbon atoms:
a. an alkane
b. an alkene
c. an alkyne

20.E.1.2: S20.1.1
There are several sets of answers; one is:
(a) C5H12

(b) C5H10

(c) C5H8

20.E.1.3: Q20.1.2
What is the difference between the hybridization of carbon atoms’ valence orbitals in saturated and unsaturated hydrocarbons?

20.E.1.4: Q20.1.3
On a microscopic level, how does the reaction of bromine with a saturated hydrocarbon differ from its reaction with an unsaturated
hydrocarbon? How are they similar?

20.E.1.5: Q20.1.4
Both reactions result in bromine being incorporated into the structure of the product. The difference is the way in which that
incorporation takes place. In the saturated hydrocarbon, an existing C–H bond is broken, and a bond between the C and the Br can
then be formed. In the unsaturated hydrocarbon, the only bond broken in the hydrocarbon is the π bond whose electrons can be
used to form a bond to one of the bromine atoms in Br2 (the electrons from the Br–Br bond form the other C–Br bond on the other
carbon that was part of the π bond in the starting unsaturated hydrocarbon).

20.E.1.6: Q20.1.5
On a microscopic level, how does the reaction of bromine with an alkene differ from its reaction with an alkyne? How are they
similar?

Access for free at OpenStax 20.E.1 https://chem.libretexts.org/@go/page/45401


20.E.1.7: Q20.1.6
Explain why unbranched alkenes can form geometric isomers while unbranched alkanes cannot. Does this explanation involve the
macroscopic domain or the microscopic domain?

20.E.1.8: S20.1.6
Unbranched alkanes have free rotation about the C–C bonds, yielding all orientations of the substituents about these bonds
equivalent, interchangeable by rotation. In the unbranched alkenes, the inability to rotate about the C = C bond results in fixed
(unchanging) substituent orientations, thus permitting different isomers. Since these concepts pertain to phenomena at the
molecular level, this explanation involves the microscopic domain.
Explain why these two molecules are not isomers:

20.E.1.9: Q20.1.7
Explain why these two molecules are not isomers:

20.E.1.10: S20.1.6
They are the same compound because each is a saturated hydrocarbon containing an unbranched chain of six carbon atoms.

20.E.1.11: Q20.1.7
How does the carbon-atom hybridization change when polyethylene is prepared from ethylene?

20.E.1.12: Q20.1.8
Write the Lewis structure and molecular formula for each of the following hydrocarbons:
a. hexane
b. 3-methylpentane
c. cis-3-hexene
d. 4-methyl-1-pentene
e. 3-hexyne
f. 4-methyl-2-pentyne

20.E.1.13: Q20.1.9
(a) C6H14

Access for free at OpenStax 20.E.2 https://chem.libretexts.org/@go/page/45401


(b) C6H14

(c) C6H12

(d) C6H12

(e) C6H10

(f) C6H10

Access for free at OpenStax 20.E.3 https://chem.libretexts.org/@go/page/45401


20.E.1.14: Q20.1.10
Write the chemical formula, condensed formula, and Lewis structure for each of the following hydrocarbons:
a. heptane
b. 3-methylhexane
c. trans-3-heptene
d. 4-methyl-1-hexene
e. 2-heptyne
f. 3,4-dimethyl-1-pentyne

20.E.1.15: Q20.1.11
Give the complete IUPAC name for each of the following compounds:
a. CH3CH2CBr2CH3
b. (CH3)3CCl

c.
d. \(\mathrm{CH_3CH_2C≡CH\, CH_3CH_2C≡CH}\

e.

f.
g. (C H 3 )2 CHC H2 CH = C H2

20.E.1.16: S20.1.11
(a) 2,2-dibromobutane; (b) 2-chloro-2-methylpropane; (c) 2-methylbutane; (d) 1-butyne; (e) 4-fluoro-4-methyl-1-octyne; (f) trans-
1-chloropropene; (g) 5-methyl-1-pentene

20.E.1.17: Q20.1.12
Give the complete IUPAC name for each of the following compounds:
a. (CH3)2CHF
b. CH3CHClCHClCH3

c.
d. C H3 C H2 CH = CHC H3

Access for free at OpenStax 20.E.4 https://chem.libretexts.org/@go/page/45401


e.
f. (C H3 )3 CC H2 C ≡ CH

20.E.1.18: Q20.1.13
Butane is used as a fuel in disposable lighters. Write the Lewis structure for each isomer of butane.

20.E.1.19: S20.1.13

20.E.1.20: Q20.1.14
Write Lewis structures and name the five structural isomers of hexane.

20.E.1.21: Q20.1.15
Write Lewis structures for the cis–trans isomers of C H 3 CH = CHCl .

20.E.1.22: S20.1.15

20.E.1.23: Q20.1.16
Write structures for the three isomers of the aromatic hydrocarbon xylene, C6H4(CH3)2.

20.E.1.24: Q20.1.17
Isooctane is the common name of the isomer of C8H18 used as the standard of 100 for the gasoline octane rating:

a. What is the IUPAC name for the compound?


b. Name the other isomers that contain a five-carbon chain with three methyl substituents.

20.E.1.25: Q20.1.18
(a) 2,2,4-trimethylpentane; (b) 2,2,3-trimethylpentane, 2,3,4-trimethylpentane, and 2,3,3-trimethylpentane:

Access for free at OpenStax 20.E.5 https://chem.libretexts.org/@go/page/45401


20.E.1.26: Q20.1.19
Write Lewis structures and IUPAC names for the alkyne isomers of C4H6.

20.E.1.27: Q20.1.20
Write Lewis structures and IUPAC names for all isomers of C4H9Cl.

20.E.1.28: S20.1.20

20.E.1.29: Q20.1.21
Name and write the structures of all isomers of the propyl and butyl alkyl groups.

20.E.1.30: Q20.1.22
Write the structures for all the isomers of the –C5H11 alkyl group.

20.E.1.31: Q20.1.23
In the following, the carbon backbone and the appropriate number of hydrogen atoms are shown in condensed form:

Access for free at OpenStax 20.E.6 https://chem.libretexts.org/@go/page/45401


20.E.1.32: Q20.1.24
Write Lewis structures and describe the molecular geometry at each carbon atom in the following compounds:
a. cis-3-hexene
b. cis-1-chloro-2-bromoethene
c. 2-pentyne
d. trans-6-ethyl-7-methyl-2-octene

20.E.1.33: Q20.1.25
Benzene is one of the compounds used as an octane enhancer in unleaded gasoline. It is manufactured by the catalytic conversion
of acetylene to benzene:
3C H ⟶ C H (20.E.1)
2 2 6 6

Draw Lewis structures for these compounds, with resonance structures as appropriate, and determine the hybridization of the
carbon atoms in each.

20.E.1.34: S20.1.25

20.E.1.35: Q20.1.26
In acetylene, the bonding uses sp hybrids on carbon atoms and s orbitals on hydrogen atoms. In benzene, the carbon atoms are sp2
hybridized.

20.E.1.36: Q20.1.27
Teflon is prepared by the polymerization of tetrafluoroethylene. Write the equation that describes the polymerization using Lewis
symbols.

20.E.1.37: Q20.1.28
Write two complete, balanced equations for each of the following reactions, one using condensed formulas and one using Lewis
structures.

Access for free at OpenStax 20.E.7 https://chem.libretexts.org/@go/page/45401


a. 1 mol of 1-butyne reacts with 2 mol of iodine.
b. Pentane is burned in air.

20.E.1.38: S20.1.28
(a) CH = CC H 2 C H3 + 2 I2 ⟶ CHI2 C I2 C H2 C H3

(b) CH 3
CH CH CH CH
2 2 2 3
+8 O
2
⟶ 5 CO
2
+6 H O
2

20.E.1.39: Q20.1.29
Write two complete, balanced equations for each of the following reactions, one using condensed formulas and one using Lewis
structures.
a. 2-butene reacts with chlorine.
b. benzene burns in air.

20.E.1.40: Q20.1.30
What mass of 2-bromopropane could be prepared from 25.5 g of propene? Assume a 100% yield of product.

20.E.1.41: S20.1.30
65.2 g

20.E.1.42: Q20.1.31
Acetylene is a very weak acid; however, it will react with moist silver(I) oxide and form water and a compound composed of silver
and carbon. Addition of a solution of HCl to a 0.2352-g sample of the compound of silver and carbon produced acetylene and
0.2822 g of AgCl.
a. What is the empirical formula of the compound of silver and carbon?
b. The production of acetylene on addition of HCl to the compound of silver and carbon suggests that the carbon is present as the
acetylide ion, C . Write the formula of the compound showing the acetylide ion.
2−

20.E.1.43: Q20.1.32
Ethylene can be produced by the pyrolysis of ethane:
C H ⟶ C H +H
2 6 2 4 2

How many kilograms of ethylene is produced by the pyrolysis of 1.000 × 103 kg of ethane, assuming a 100.0% yield?

20.E.1.44: S20.1.33
9.328 × 102 kg

20.E.2: 20.2: Alcohols and Ethers


Why do the compounds hexane, hexanol, and hexene have such similar names?
Write condensed formulas and provide IUPAC names for the following compounds:

Access for free at OpenStax 20.E.8 https://chem.libretexts.org/@go/page/45401


1. (a) ethyl alcohol (in beverages)
2. (b) methyl alcohol (used as a solvent, for example, in shellac)
3. (c) ethylene glycol (antifreeze)
4. (d) isopropyl alcohol (used in rubbing alcohol)
5. (e) glycerine
(a) ethyl alcohol, ethanol: CH3CH2OH; (b) methyl alcohol, methanol: CH3OH; (c) ethylene glycol, ethanediol: HOCH2CH2OH; (d)
isopropyl alcohol, 2-propanol: CH3CH(OH)CH3; (e) glycerine, l,2,3-trihydroxypropane: HOCH2CH(OH)CH2OH
Give the complete IUPAC name for each of the following compounds:
(a)

(b)

(c)

Give the complete IUPAC name and the common name for each of the following compounds:
(a)

(b)

(c)

(a) 1-ethoxybutane, butyl ethyl ether; (b) 1-ethoxypropane, ethyl propyl ether; (c) 1-methoxypropane, methyl propyl ether
Write the condensed structures of both isomers with the formula C2H6O. Label the functional group of each isomer.
Write the condensed structures of all isomers with the formula C2H6O2. Label the functional group (or groups) of each isomer.

Access for free at OpenStax 20.E.9 https://chem.libretexts.org/@go/page/45401


HOCH2CH2OH, two alcohol groups; CH3OCH2OH, ether and alcohol groups
Draw the condensed formulas for each of the following compounds:
1. (a) dipropyl ether
2. (b) 2,2-dimethyl-3-hexanol
3. (c) 2-ethoxybutane
MTBE, Methyl tert-butyl ether, CH3OC(CH3)3, is used as an oxygen source in oxygenated gasolines. MTBE is manufactured by
reacting 2-methylpropene with methanol.
1. (a) Using Lewis structures, write the chemical equation representing the reaction.
2. (b) What volume of methanol, density 0.7915 g/mL, is required to produce exactly 1000 kg of MTBE, assuming a 100% yield?
(a)

(b) 4.593 × 102 L


Write two complete balanced equations for each of the following reactions, one using condensed formulas and one using Lewis
structures.
(a) propanol is converted to dipropyl ether
(b) propene is treated with water in dilute acid.
Write two complete balanced equations for each of the following reactions, one using condensed formulas and one using Lewis
structures.
1. (a) 2-butene is treated with water in dilute acid
2. (b) ethanol is dehydrated to yield ethene
(a) C H3 CH = CHC H3 + H2 O ⟶ C H3 C H2 CH(OH)C H3

;
(b) C H 3 C H2 OH ⟶ C H2 = C H2 + H2 O

20.E.3: 20.3: Aldehydes, Ketones, Carboxylic Acids, and Esters


20.E.3.1: Q20.3.1
Order the following molecules from least to most oxidized, based on the marked carbon atom:

Access for free at OpenStax 20.E.10 https://chem.libretexts.org/@go/page/45401


20.E.3.2: Q20.3.2
Predict the products of oxidizing the molecules shown in this problem. In each case, identify the product that will result from the
minimal increase in oxidation state for the highlighted carbon atom:
(a)

(b)

(c)

(a)

(b)

(c)

20.E.3.3: Q20.3.3
Predict the products of reducing the following molecules. In each case, identify the product that will result from the minimal
decrease in oxidation state for the highlighted carbon atom:
(a)

Access for free at OpenStax 20.E.11 https://chem.libretexts.org/@go/page/45401


(b)

(c)

20.E.3.4: Q20.3.4
Explain why it is not possible to prepare a ketone that contains only two carbon atoms.

20.E.3.5: S20.3.4
A ketone contains a group bonded to two additional carbon atoms; thus, a minimum of three carbon atoms are needed.

20.E.3.6: Q20.3.5
How does hybridization of the substituted carbon atom change when an alcohol is converted into an aldehyde? An aldehyde to a
carboxylic acid?

20.E.3.7: Q20.3.6
Fatty acids are carboxylic acids that have long hydrocarbon chains attached to a carboxylate group. How does a saturated fatty acid
differ from an unsaturated fatty acid? How are they similar?

20.E.3.8: S20.3.6
Since they are both carboxylic acids, they each contain the –COOH functional group and its characteristics. The difference is the
hydrocarbon chain in a saturated fatty acid contains no double or triple bonds, whereas the hydrocarbon chain in an unsaturated
fatty acid contains one or more multiple bonds.

20.E.3.9: S20.3.7
Write a condensed structural formula, such as CH3CH3, and describe the molecular geometry at each carbon atom.
a. propene
b. 1-butanol
c. ethyl propyl ether
d. cis-4-bromo-2-heptene
e. 2,2,3-trimethylhexane
f. formaldehyde

20.E.3.10: Q20.3.8
Write a condensed structural formula, such as CH3CH3, and describe the molecular geometry at each carbon atom.
a. 2-propanol
b. acetone
c. dimethyl ether
d. acetic acid

Access for free at OpenStax 20.E.12 https://chem.libretexts.org/@go/page/45401


e. 3-methyl-1-hexene

20.E.3.11: S20.3.8
(a) CH3CH(OH)CH3: all carbons are tetrahedral; (b) CH C(==O)CH : the end carbons are tetrahedral and the central carbon is
3 3

trigonal planar; (c) CH3OCH3: all are tetrahedral; (d) CH3COOH: the methyl carbon is tetrahedral and the acid carbon is trigonal
planar; (e) CH3CH2CH2CH(CH3)CHCH2: all are tetrahedral except the right-most two carbons, which are trigonal planar

20.E.3.12: Q20.3.9
The foul odor of rancid butter is caused by butyric acid, CH3CH2CH2CO2H.
a. Draw the Lewis structure and determine the oxidation number and hybridization for each carbon atom in the molecule.
b. The esters formed from butyric acid are pleasant-smelling compounds found in fruits and used in perfumes. Draw the Lewis
structure for the ester formed from the reaction of butyric acid with 2-propanol.

20.E.3.13: Q20.3.10
Write the two-resonance structures for the acetate ion.

20.E.3.14: Q20.3.11
Write two complete, balanced equations for each of the following reactions, one using condensed formulas and one using Lewis
structures:
a. ethanol reacts with propionic acid
b. benzoic acid, C6H5CO2H, is added to a solution of sodium hydroxide

20.E.3.15: Q20.3.12
Write two complete balanced equations for each of the following reactions, one using condensed formulas and one using Lewis
structures.
a. 1-butanol reacts with acetic acid
b. propionic acid is poured onto solid calcium carbonate

20.E.3.16: S20.3.12
(a) CH 3
CH CH CH OH + CH C(O)OH ⟶ CH C(O)OCH CH CH CH
2 2 2 3 3 2 2 2 3
+H O
2
:

(b) 2 CH 3
CH COOH + CaCO
2 3
⟶ (CH CH COO) Ca + CO
3 2 2 2
+H O
2
:

Access for free at OpenStax 20.E.13 https://chem.libretexts.org/@go/page/45401


20.E.3.17: Q20.3.13
Yields in organic reactions are sometimes low. What is the percent yield of a process that produces 13.0 g of ethyl acetate from
10.0 g of CH3CO2H?

20.E.3.18: Q20.3.14
Alcohols A, B, and C all have the composition C4H10O. Molecules of alcohol A contain a branched carbon chain and can be
oxidized to an aldehyde; molecules of alcohol B contain a linear carbon chain and can be oxidized to a ketone; and molecules of
alcohol C can be oxidized to neither an aldehyde nor a ketone. Write the Lewis structures of these molecules.

20.E.3.19: S20.3.15

20.E.4: 20.4: Amines and Amides


20.E.4.1: Q20.4.1
Write the Lewis structures of both isomers with the formula C2H7N.

20.E.4.2: Q20.4.2
What is the molecular structure about the nitrogen atom in trimethyl amine and in the trimethyl ammonium ion, (CH3)3NH+? What
is the hybridization of the nitrogen atom in trimethyl amine and in the trimethyl ammonium ion?

20.E.4.3: S20.4.2
Trimethyl amine: trigonal pyramidal, sp3; trimethyl ammonium ion: tetrahedral, sp3

20.E.4.4: Q20.4.3
Write the two resonance structures for the pyridinium ion, C5H5NH+.

20.E.4.5: Q20.4.4
Draw Lewis structures for pyridine and its conjugate acid, the pyridinium ion, C5H5NH+. What are the geometries and
hybridizations about the nitrogen atoms in pyridine and in the pyridinium ion?

Access for free at OpenStax 20.E.14 https://chem.libretexts.org/@go/page/45401


20.E.4.6: Q20.4.5
Write the Lewis structures of all isomers with the formula C3H7ON that contain an amide linkage.

20.E.4.7: Q20.4.6
Write two complete balanced equations for the following reaction, one using condensed formulas and one using Lewis structures.
Methyl amine is added to a solution of HCl.

20.E.4.8: S20.4.6
+ +
CH NH +H O ⟶ CH NH +H O (20.E.2)
3 2 3 3 3 2

20.E.4.9: Q20.4.7
Write two complete, balanced equations for each of the following reactions, one using condensed formulas and one using Lewis
structures.
Ethylammonium chloride is added to a solution of sodium hydroxide.

20.E.4.10: Q20.4.8
Identify any carbon atoms that change hybridization and the change in hybridization during the reactions in [link].

20.E.4.11: S20.4.8
CH3CH = CHCH3(sp2) + Cl ⟶ CH3CH(Cl)H(Cl)CH3(sp3);
2C6H6(sp2) + 15O2 ⟶ 12CO2(sp) + 6H2O

20.E.4.12: Q20.4.9
Identify any carbon atoms that change hybridization and the change in hybridization during the reactions in [link].

20.E.4.13: Q20.4.10
Identify any carbon atoms that change hybridization and the change in hybridization during the reactions in [link].

20.E.4.14: S20.4.10
the carbon in CO32−, initially at sp2, changes hybridization to sp in CO2

Access for free at OpenStax 20.E.15 https://chem.libretexts.org/@go/page/45401


This page titled 20.E: Organic Chemistry (Exercises) is shared under a CC BY 4.0 license and was authored, remixed, and/or curated by
OpenStax via source content that was edited to the style and standards of the LibreTexts platform; a detailed edit history is available upon request.

Access for free at OpenStax 20.E.16 https://chem.libretexts.org/@go/page/45401


CHAPTER OVERVIEW
21: Nuclear Chemistry

A general chemistry Libretexts Textbook remixed and remastered from


OpenStax's textbook:
General Chemistry
The chemical reactions that we have considered in previous chapters involve changes in the electronic structure of the species
involved, that is, the arrangement of the electrons around atoms, ions, or molecules. Nuclear structure, the numbers of protons and
neutrons within the nuclei of the atoms involved, remains unchanged during chemical reactions. This chapter will introduce the
topic of nuclear chemistry, which began with the discovery of radioactivity in 1896 by French physicist Antoine Becquerel and has
become increasingly important during the twentieth and twenty-first centuries, providing the basis for various technologies related
to energy, medicine, geology, and many other areas.
21.1: Nuclear Structure and Stability
21.2: Nuclear Equations
21.3: Radioactive Decay
21.4: Transmutation and Nuclear Energy
21.5: Uses of Radioisotopes
21.6: Biological Effects of Radiation
21.E: Nuclear Chemistry (Exercises)

This page titled 21: Nuclear Chemistry is shared under a CC BY 4.0 license and was authored, remixed, and/or curated by OpenStax via source
content that was edited to the style and standards of the LibreTexts platform; a detailed edit history is available upon request.

1
21.1: Nuclear Structure and Stability
 Learning Objectives
Describe nuclear structure in terms of protons, neutrons, and electrons
Calculate mass defect and binding energy for nuclei
Explain trends in the relative stability of nuclei

Nuclear chemistry is the study of reactions that involve changes in nuclear structure. The chapter on atoms, molecules, and ions
introduced the basic idea of nuclear structure, that the nucleus of an atom is composed of protons and, with the exception of H , 1
1

neutrons. Recall that the number of protons in the nucleus is called the atomic number (Z ) of the element, and the sum of the
number of protons and the number of neutrons is the mass number (A ). Atoms with the same atomic number but different mass
numbers are isotopes of the same element. When referring to a single type of nucleus, we often use the term nuclide and identify it
by the notation:
A
X (21.1.1)
Z

where
X is the symbol for the element,
A is the mass number, and

Z is the atomic number.

Often a nuclide is referenced by the name of the element followed by a hyphen and the mass number. For example, 14
6
C is called
“carbon-14.”
Protons and neutrons, collectively called nucleons, are packed together tightly in a nucleus. With a radius of about 10−15 meters, a
nucleus is quite small compared to the radius of the entire atom, which is about 10−10 meters. Nuclei are extremely dense compared
to bulk matter, averaging 1.8 × 10 grams per cubic centimeter. For example, water has a density of 1 gram per cubic centimeter,
14

and iridium, one of the densest elements known, has a density of 22.6 g/cm3. If the earth’s density were equal to the average
nuclear density, the earth’s radius would be only about 200 meters (earth’s actual radius is approximately 6.4 × 10 meters, 30,000
6

times larger). Example 21.1.1 demonstrates just how great nuclear densities can be in the natural world.

 Example 21.1.1: Neutron Stars


Density of a Neutron Star Neutron stars form when the core of a very massive star undergoes gravitational collapse, causing
the star’s outer layers to explode in a supernova. Composed almost completely of neutrons, they are the densest-known stars in
the universe, with densities comparable to the average density of an atomic nucleus. A neutron star in a faraway galaxy has a
mass equal to 2.4 solar masses (1 solar mass = M☉ = mass of the sun = 1.99 × 10 kg ) and a diameter of 26 km.
30

a. What is the density ρ of this neutron star?


b. How does this neutron star’s density compare to the density of a uranium nucleus, which has a diameter of about 15 fm (1
fm = 10–15 m)?

Solution
We can treat both the neutron star and the U-235 nucleus as spheres. Then the density for both is given by:
m
ρ =
V

with
4 3
V = πr
3

1 1
(a) The radius of the neutron star is × 26 km = × 2.6 × 10
4 4
m = 1.3 × 10 m so the density of the neutron star is:
2 2

Access for free at OpenStax 21.1.1 https://chem.libretexts.org/@go/page/38339


m
ρ =
V

m
=
4 3
πr
3

30
2.4(1.99 × 10 kg)
=
4 4 3
π(1.3 × 10 m )
3

17 3
= 5.2 × 10 kg/ m

1
(b) The radius of the U-235 nucleus is × 15 × 10
−15
m = 7.5 × 10
−15
m , so the density of the U-235 nucleus is:
2

m
ρ =
V

m
=
4 3
πr
3

−27
1.66×10 kg
235 amu ( )
1 amu

=
4 −15 3
π(7.5 × 10 m)
3

17 3
= 2.2 × 10 kg/ m

These values are fairly similar (same order of magnitude), but the nucleus is more than twice as dense as the neutron star.

 Exercise 21.1.1

Find the density of a neutron star with a mass of 1.97 solar masses and a diameter of 13 km, and compare it to the density of a
hydrogen nucleus, which has a diameter of 1.75 fm (1 f m = 1 × 10 m ). –15

Answer
The density of the neutron star is 3.4 × 10 kg/m . The density of a hydrogen nucleus is 6.0 × 10
18 3 17
kg/ m
3
. The neutron
star is 5.7 times denser than the hydrogen nucleus.

To hold positively charged protons together in the very small volume of a nucleus requires very strong attractive forces because the
positively charged protons repel one another strongly at such short distances. The force of attraction that holds the nucleus together
is the strong nuclear force. (The strong force is one of the four fundamental forces that are known to exist. The others are the
electromagnetic force, the gravitational force, and the nuclear weak force.) This force acts between protons, between neutrons, and
between protons and neutrons. It is very different from the electrostatic force that holds negatively charged electrons around a
positively charged nucleus (the attraction between opposite charges). Over distances less than 10−15 meters and within the nucleus,
the strong nuclear force is much stronger than electrostatic repulsions between protons; over larger distances and outside the
nucleus, it is essentially nonexistent.

21.1.1: Nuclear Binding Energy


As a simple example of the energy associated with the strong nuclear force, consider the helium atom composed of two protons,
two neutrons, and two electrons. The total mass of these six subatomic particles may be calculated as:
(2 × 1.0073 amu) + (2 × 1.0087 amu) + (2 × 0.00055 amu) = 4.0331 amu  (21.1.2)

protons neutrons electrons

However, mass spectrometric measurements reveal that the mass of an He atom is 4.0026 amu, less than the combined masses of
4
2

its six constituent subatomic particles. This difference between the calculated and experimentally measured masses is known as the
mass defect of the atom. In the case of helium, the mass defect indicates a “loss” in mass of 4.0331 amu – 4.0026 amu = 0.0305
amu. The loss in mass accompanying the formation of an atom from protons, neutrons, and electrons is due to the conversion of
that mass into energy that is evolved as the atom forms. The nuclear binding energy is the energy produced when the atoms’
nucleons are bound together; this is also the energy needed to break a nucleus into its constituent protons and neutrons. In

Access for free at OpenStax 21.1.2 https://chem.libretexts.org/@go/page/38339


comparison to chemical bond energies, nuclear binding energies are vastly greater, as we will learn in this section. Consequently,
the energy changes associated with nuclear reactions are vastly greater than are those for chemical reactions.
The conversion between mass and energy is most identifiably represented by the mass-energy equivalence equation as stated by
Albert Einstein:
2
E = mc (21.1.3)

where E is energy, m is mass of the matter being converted, and c is the speed of light in a vacuum. This equation can be used to
find the amount of energy that results when matter is converted into energy. Using this mass-energy equivalence equation, the
nuclear binding energy of a nucleus may be calculated from its mass defect, as demonstrated in Example 21.1.2. A variety of units
are commonly used for nuclear binding energies, including electron volts (eV), with 1 eV equaling the amount of energy necessary
to the move the charge of an electron across an electric potential difference of 1 volt, making 1 eV = 1.602 × 10 J.
−19

 Example 21.1.2: Calculation of Nuclear Binding Energy

Determine the binding energy for the nuclide 4


2
He in:
a. joules per mole of nuclei
b. joules per nucleus
c. MeV per nucleus

Solution
The mass defect for a He nucleus is 0.0305 amu, as shown previously. Determine the binding energy in joules per nuclide
4
2

using the mass-energy equivalence equation. To accommodate the requested energy units, the mass defect must be expressed in
kilograms (recall that 1 J = 1 kg m2/s2).
(a) First, express the mass defect in g/mol. This is easily done considering the numerical equivalence of atomic mass (amu) and
molar mass (g/mol) that results from the definitions of the amu and mole units (refer to the previous discussion in the chapter
on atoms, molecules, and ions if needed). The mass defect is therefore 0.0305 g/mol. To accommodate the units of the other
terms in the mass-energy equation, the mass must be expressed in kg, since 1 J = 1 kg m2/s2. Converting grams into kilograms
yields a mass defect of 3.05 × 10 kg/mol. Substituting this quantity into the mass-energy equivalence equation yields:
–5

\[\begin{align*} E &=mc^2 \\[4pt] &= \dfrac{3.05 \times 10^{-5}\;kg}{mol} \times \left(\dfrac{2.998 \times 10^8\;m}
{s}\right)^2 \\[4pt] &= 2.74×10^{12}\:kg\:m^2s^{-2}mol^{-1} \\[4pt] &=2.74 \times 10^{12}\;J/mol=2.74\: TJ /mol
\end{align*} \nonumber \]
Note that this tremendous amount of energy is associated with the conversion of a very small amount of matter (about 30 mg,
roughly the mass of typical drop of water).

(b) The binding energy for a single nucleus is computed from the molar binding energy using Avogadro’s number:
1 mol
12 −1
E = 2.74 × 10 J mol ×
23
6.022 × 10 nuclei

−12
= 4.55 × 10 J = 4.55 pJ

(c) Recall that 1 −19


eV = 1.602 × 10 J . Using the binding energy computed in part (b):
1 eV
−12
E = 4.55 × 10 J×
−19
1.602 × 10 J

7
= 2.84 × 10 eV = 28.4 M eV

 Exercise 21.1.2

What is the binding energy for the nuclide 19


9
F (atomic mass: 18.9984 amu) in MeV per nucleus?

Answer
148.4 MeV

Access for free at OpenStax 21.1.3 https://chem.libretexts.org/@go/page/38339


Because the energy changes for breaking and forming bonds are so small compared to the energy changes for breaking or forming
nuclei, the changes in mass during all ordinary chemical reactions are virtually undetectable. As described in the chapter on
thermochemistry, the most energetic chemical reactions exhibit enthalpies on the order of thousands of kJ/mol, which is equivalent
to mass differences in the nanogram range (10–9 g). On the other hand, nuclear binding energies are typically on the order of
billions of kJ/mol, corresponding to mass differences in the milligram range (10–3 g).

21.1.2: Nuclear Stability


A nucleus is stable if it cannot be transformed into another configuration without adding energy from the outside. Of the thousands
of nuclides that exist, about 250 are stable. A plot of the number of neutrons versus the number of protons for stable nuclei reveals
that the stable isotopes fall into a narrow band. This region is known as the band of stability (also called the belt, zone, or valley of
stability). The straight line in Figure 21.1.1 represents nuclei that have a 1:1 ratio of protons to neutrons (n:p ratio). Note that the
lighter stable nuclei, in general, have equal numbers of protons and neutrons. For example, nitrogen-14 has seven protons and
seven neutrons. Heavier stable nuclei, however, have increasingly more neutrons than protons. For example: iron-56 has 30
neutrons and 26 protons, an n:p ratio of 1.15, whereas the stable nuclide lead-207 has 125 neutrons and 82 protons, an n:p ratio
equal to 1.52. This is because larger nuclei have more proton-proton repulsions, and require larger numbers of neutrons to provide
compensating strong forces to overcome these electrostatic repulsions and hold the nucleus together.

Figure 21.1.1 : This plot shows the nuclides that are known to exist and those that are stable. The stable nuclides are indicated in
blue, and the unstable nuclides are indicated in green. Note that all isotopes of elements with atomic numbers greater than 83 are
unstable. The solid line is the line where n = Z.
The nuclei that are to the left or to the right of the band of stability are unstable and exhibit radioactivity. They change
spontaneously (decay) into other nuclei that are either in, or closer to, the band of stability. These nuclear decay reactions convert
one unstable isotope (or radioisotope) into another, more stable, isotope. We will discuss the nature and products of this radioactive
decay in subsequent sections of this chapter.
Several observations may be made regarding the relationship between the stability of a nucleus and its structure. Nuclei with even
numbers of protons, neutrons, or both are more likely to be stable (Table 21.1.1). Nuclei with certain numbers of nucleons, known
as magic numbers, are stable against nuclear decay. These numbers of protons or neutrons (2, 8, 20, 28, 50, 82, and 126) make
complete shells in the nucleus. These are similar in concept to the stable electron shells observed for the noble gases. Nuclei that
have magic numbers of both protons and neutrons, such as He, O, Ca, and Pb and are particularly stable. These trends in
4
2
16
8
40
20
208
82

Access for free at OpenStax 21.1.4 https://chem.libretexts.org/@go/page/38339


nuclear stability may be rationalized by considering a quantum mechanical model of nuclear energy states analogous to that used to
describe electronic states earlier in this textbook. The details of this model are beyond the scope of this chapter.
Table 21.1.1 : Stable Nuclear Isotopes
Number of Stable Isotopes Proton Number Neutron Number

157 even even

53 even odd

50 odd even

5 odd odd

The relative stability of a nucleus is correlated with its binding energy per nucleon, the total binding energy for the nucleus divided
by the number or nucleons in the nucleus. For instance, the binding energy for a He nucleus is therefore:
4
2

28.4 MeV
= 7.10 MeV/nucleon (21.1.4)
4 nucleons

The binding energy per nucleon of a nuclide on the curve shown in Figure 21.1.2

Figure 21.1.2 : The binding energy per nucleon is largest for nuclides with mass number of approximately 56.

 Example 21.1.3: Calculation of Binding Energy per Nucleon

The iron nuclide Fe lies near the top of the binding energy curve (Figure 21.1.2) and is one of the most stable nuclides. What
56
26

is the binding energy per nucleon (in MeV) for the nuclide Fe (atomic mass of 55.9349 amu)?
56
26

Solution
As in Example, we first determine the mass defect of the nuclide, which is the difference between the mass of 26 protons, 30
neutrons, and 26 electrons, and the observed mass of an Fe atom:
56
26

Mass def ect = [(26 × 1.0073 amu) + (30 × 1.0087 amu) + (26 × 0.00055 amu)] − 55.9349 amu

= 56.4651 amu − 55.9349 amu

= 0.5302 amu

We next calculate the binding energy for one nucleus from the mass defect using the mass-energy equivalence equation:

Access for free at OpenStax 21.1.5 https://chem.libretexts.org/@go/page/38339


−27
1.6605 × 10 kg
2 8 2
E = mc = 0.5302 amu × × (2.998 × 10 m/s)
1 amu
−11 2
= 7.913 × 10 kg⋅m/ s

−11
= 7.913 × 10 J

We then convert the binding energy in joules per nucleus into units of MeV per nuclide:

−11
1 MeV
7.913 × 10 J× = 493.9 MeV
−13
1.602 × 10 J

Finally, we determine the binding energy per nucleon by dividing the total nuclear binding energy by the number of nucleons
in the atom:
493.9 MeV
Binding energy per nucleon = = 8.820 MeV/nucleon
56

Note that this is almost 25% larger than the binding energy per nucleon for He.(Note also that this is the same process as in
4
2

Example \(\PageIndex{2}\, but with the additional step of dividing the total nuclear binding energy by the number of
nucleons.)

 Exercise 21.1.3

What is the binding energy per nucleon in 19


9
F (atomic mass, 18.9984 amu)?

Answer
7.810 MeV/nucleon

Summary
An atomic nucleus consists of protons and neutrons, collectively called nucleons. Although protons repel each other, the nucleus is
held tightly together by a short-range, but very strong, force called the strong nuclear force. A nucleus has less mass than the total
mass of its constituent nucleons. This “missing” mass is the mass defect, which has been converted into the binding energy that
holds the nucleus together according to Einstein’s mass-energy equivalence equation, E = mc2. Of the many nuclides that exist,
only a small number are stable. Nuclides with even numbers of protons or neutrons, or those with magic numbers of nucleons, are
especially likely to be stable. These stable nuclides occupy a narrow band of stability on a graph of number of protons versus
number of neutrons. The binding energy per nucleon is largest for the elements with mass numbers near 56; these are the most
stable nuclei.

21.1.3: Key Equations


E = mc2

Glossary
band of stability
(also, belt of stability, zone of stability, or valley of stability) region of graph of number of protons versus number of neutrons
containing stable (nonradioactive) nuclides

binding energy per nucleon


total binding energy for the nucleus divided by the number of nucleons in the nucleus

electron volt (eV)


measurement unit of nuclear binding energies, with 1 eV equaling the amount energy due to the moving an electron across an
electric potential difference of 1 volt

magic number
nuclei with specific numbers of nucleons that are within the band of stability

Access for free at OpenStax 21.1.6 https://chem.libretexts.org/@go/page/38339


mass defect
difference between the mass of an atom and the summed mass of its constituent subatomic particles (or the mass “lost” when
nucleons are brought together to form a nucleus)

mass-energy equivalence equation


Albert Einstein’s relationship showing that mass and energy are equivalent

nuclear binding energy


energy lost when an atom’s nucleons are bound together (or the energy needed to break a nucleus into its constituent protons
and neutrons)

nuclear chemistry
study of the structure of atomic nuclei and processes that change nuclear structure

nucleon
collective term for protons and neutrons in a nucleus

nuclide
nucleus of a particular isotope

radioactivity
phenomenon exhibited by an unstable nucleon that spontaneously undergoes change into a nucleon that is more stable; an
unstable nucleon is said to be radioactive

radioisotope
isotope that is unstable and undergoes conversion into a different, more stable isotope

strong nuclear force


force of attraction between nucleons that holds a nucleus together

Paul Flowers (University of North Carolina - Pembroke), Klaus Theopold (University of Delaware) and Richard Langley
(Stephen F. Austin State University) with contributing authors. Textbook content produced by OpenStax College is licensed
under a Creative Commons Attribution License 4.0 license. Download for free at http://cnx.org/contents/85abf193-
2bd...a7ac8df6@9.110).

This page titled 21.1: Nuclear Structure and Stability is shared under a CC BY 4.0 license and was authored, remixed, and/or curated by OpenStax
via source content that was edited to the style and standards of the LibreTexts platform; a detailed edit history is available upon request.

Access for free at OpenStax 21.1.7 https://chem.libretexts.org/@go/page/38339


21.2: Nuclear Equations
 Learning Objectives
Identify common particles and energies involved in nuclear reactions
Write and balance nuclear equations

Changes of nuclei that result in changes in their atomic numbers, mass numbers, or energy states are nuclear reactions. To describe
a nuclear reaction, we use an equation that identifies the nuclides involved in the reaction, their mass numbers and atomic numbers,
and the other particles involved in the reaction.

21.2.1: Types of Particles in Nuclear Reactions


Many entities can be involved in nuclear reactions. The most common are protons, neutrons, alpha particles, beta particles,
positrons, and gamma rays, as shown in Figure 21.2.1. Protons ( p, also represented by the symbol H) and neutrons ( n) are the
1
1
1
1
1
0

constituents of atomic nuclei, and have been described previously. Alpha particles ( He , also represented by the symbol α) are
4
2
4
2

high-energy helium nuclei. Beta particles ( β , also represented by the symbol e) are high-energy electrons, and gamma rays
0
−1
0
−1

are photons of very high-energy electromagnetic radiation. Positrons ( e , also represented by the symbol β) are positively
+1
0
+1
0

charged electrons (“anti-electrons”). The subscripts and superscripts are necessary for balancing nuclear equations, but are usually
optional in other circumstances. For example, an alpha particle is a helium nucleus (He) with a charge of +2 and a mass number of
4, so it is symbolized He. This works because, in general, the ion charge is not important in the balancing of nuclear equations.
4
2

Figure 21.2.1 : Although many species are encountered in nuclear reactions, this table summarizes the names, symbols,
representations, and descriptions of the most common of these.
This table has four columns and seven rows. The first row is a header row and it labels each column: “Name,” “Symbol(s),”
“Representation,” and “Description.” Under the “Name” column are the following: “Alpha particle,” “Beta particle,” “Positron,”
“Proton,” “Neutron,” and “Gamma ray.” Under the “Symbol(s)” column are the following: “ superscript 4 stacked over a subscript
2 H e or lowercase alpha,” “superscript 0 stacked over a subscript 1 e or lowercase beta,” “superscript 0 stacked over a positive
subscript 1 e or lowercase beta superscript positive sign,” “superscript 1 stacked over a subscript 1 H or lowercase rho superscript 1
stacked over a subscript 1 H,” “superscript 1 stacked over a subscript 0 n or lowercase eta superscript 1 stacked over a subscript 0
n,” and a lowercase gamma. Under the “Representation column,” are the following: two white sphere attached to two blue spheres
of about the same size with positive signs in them; a small red sphere with a negative sign in it; a small red sphere with a positive
sign in it; a blue spheres with a positive sign in it; a white sphere; and a purple squiggle ling with an arrow pointing right to a
lowercase gamma. Under the “Description” column are the following: “(High-energy) helium nuclei consisting of two protons and
two neutrons,” “(High-energy) elections,” “Particles with the same mass as an electron but with 1 unit of positive charge,” “Nuclei
of hydrogen atoms,” “Particles with a mass approximately equal to that of a proton but with no charge,” and “Very high-energy
electromagnetic radiation.”

Access for free at OpenStax 21.2.1 https://chem.libretexts.org/@go/page/38340


Note that positrons are exactly like electrons, except they have the opposite charge. They are the most common example of
antimatter, particles with the same mass but the opposite state of another property (for example, charge) than ordinary matter.
When antimatter encounters ordinary matter, both are annihilated and their mass is converted into energy in the form of gamma
rays (γ)—and other much smaller subnuclear particles, which are beyond the scope of this chapter—according to the mass-energy
equivalence equation E = mc , seen in the preceding section. For example, when a positron and an electron collide, both are
2

annihilated and two gamma ray photons are created:


0 0
e+ e → γ +γ (21.2.1)
−1 +1

Gamma rays compose short wavelength, high-energy electromagnetic radiation and are (much) more energetic than better-known
X-rays. Gamma rays are produced when a nucleus undergoes a transition from a higher to a lower energy state, similar to how a
photon is produced by an electronic transition from a higher to a lower energy level. Due to the much larger energy differences
between nuclear energy shells, gamma rays emanating from a nucleus have energies that are typically millions of times larger than
electromagnetic radiation emanating from electronic transitions.

21.2.2: Balancing Nuclear Reactions


A balanced chemical reaction equation reflects the fact that during a chemical reaction, bonds break and form, and atoms are
rearranged, but the total numbers of atoms of each element are conserved and do not change. A balanced nuclear reaction equation
indicates that there is a rearrangement during a nuclear reaction, but of subatomic particles rather than atoms. Nuclear reactions
also follow conservation laws, and they are balanced in two ways:
1. The sum of the mass numbers of the reactants equals the sum of the mass numbers of the products.
2. The sum of the charges of the reactants equals the sum of the charges of the products.
If the atomic number and the mass number of all but one of the particles in a nuclear reaction are known, we can identify the
particle by balancing the reaction. For instance, we could determine that O is a product of the nuclear reaction of N and He if
17
8
14
7
4
2

we knew that a proton, H , was one of the two products. Example 21.2.1 shows how we can identify a nuclide by balancing the
1
1

nuclear reaction.

 Example 21.2.1: Balancing Equations for Nuclear Reactions

The reaction of an α particle with magnesium-25 ( 25


12
Mg ) produces a proton and a nuclide of another element. Identify the new
nuclide produced.

Solution
The nuclear reaction can be written as:
25 4 1 A
12
Mg + 2 He → 1
H+ Z
X

where
A is the mass number and
Z is the atomic number of the new nuclide, X .

Because the sum of the mass numbers of the reactants must equal the sum of the mass numbers of the products:

25 + 4 = A + 1

so

A = 28

Similarly, the charges must balance, so:

12 + 2 = Z + 1

so

Z = 13

Access for free at OpenStax 21.2.2 https://chem.libretexts.org/@go/page/38340


Check the periodic table: The element with nuclear charge = +13 is aluminum. Thus, the product is 28
13
Al .

 Exercise 21.2.1

The nuclide I combines with an electron and produces a new nucleus and no other massive particles. What is the equation
125
53

for this reaction?

Answer
125 0 125
53
I+ e → 52
Te
−1

Following are the equations of several nuclear reactions that have important roles in the history of nuclear chemistry:
The first naturally occurring unstable element that was isolated, polonium, was discovered by the Polish scientist Marie Curie
and her husband Pierre in 1898. It decays, emitting α particles:
212 208 4
Po ⟶ Pb + He
84 82 2

The first nuclide to be prepared by artificial means was an isotope of oxygen, 17O. It was made by Ernest Rutherford in 1919 by
bombarding nitrogen atoms with α particles:
14 4 17 1
7
N+ α ⟶ 8
O + 1H
2

James Chadwick discovered the neutron in 1932, as a previously unknown neutral particle produced along with 12C by the
nuclear reaction between 9Be and 4He:
9 4 12 1
4
Be + 2 He ⟶ 6
C+ n
0

The first element to be prepared that does not occur naturally on the earth, technetium, was created by bombardment of
molybdenum by deuterons (heavy hydrogen, H ), by Emilio Segre and Carlo Perrier in 1937:
2
1

2 97 1 97
1
H + 42 Mo ⟶ 2 n + 43 Tc
0

The first controlled nuclear chain reaction was carried out in a reactor at the University of Chicago in 1942. One of the many
reactions involved was:
235 1 87 146 1
U+ n ⟶ Br + La + 3 n
92 0 35 57 0

Summary
Nuclei can undergo reactions that change their number of protons, number of neutrons, or energy state. Many different particles can
be involved in nuclear reactions. The most common are protons, neutrons, positrons (which are positively charged electrons), alpha
(α) particles (which are high-energy helium nuclei), beta (β) particles (which are high-energy electrons), and gamma (γ) rays
(which compose high-energy electromagnetic radiation). As with chemical reactions, nuclear reactions are always balanced. When
a nuclear reaction occurs, the total mass (number) and the total charge remain unchanged.

Glossary
alpha particle
(α or He or
4
2
4
2
α ) high-energy helium nucleus; a helium atom that has lost two electrons and contains two protons and two
neutrons

antimatter
particles with the same mass but opposite properties (such as charge) of ordinary particles

beta particle
(β or −1
0
e or −1
0
) high-energy electron
β

gamma ray

Access for free at OpenStax 21.2.3 https://chem.libretexts.org/@go/page/38340


(γ or 0
0
γ ) short wavelength, high-energy electromagnetic radiation that exhibits wave-particle duality

nuclear reaction
change to a nucleus resulting in changes in the atomic number, mass number, or energy state

positron ( +1
0
β or 0
+1
)
e

antiparticle to the electron; it has identical properties to an electron, except for having the opposite (positive) charge

This page titled 21.2: Nuclear Equations is shared under a CC BY 4.0 license and was authored, remixed, and/or curated by OpenStax via source
content that was edited to the style and standards of the LibreTexts platform; a detailed edit history is available upon request.

Access for free at OpenStax 21.2.4 https://chem.libretexts.org/@go/page/38340


21.3: Radioactive Decay
 Learning Objectives
Recognize common modes of radioactive decay
Identify common particles and energies involved in nuclear decay reactions
Write and balance nuclear decay equations
Calculate kinetic parameters for decay processes, including half-life
Describe common radiometric dating techniques

Following the somewhat serendipitous discovery of radioactivity by Becquerel, many prominent scientists began to investigate this
new, intriguing phenomenon. Among them were Marie Curie (the first woman to win a Nobel Prize, and the only person to win two
Nobel Prizes in different sciences—chemistry and physics), who was the first to coin the term “radioactivity,” and Ernest
Rutherford (of gold foil experiment fame), who investigated and named three of the most common types of radiation. During the
beginning of the twentieth century, many radioactive substances were discovered, the properties of radiation were investigated and
quantified, and a solid understanding of radiation and nuclear decay was developed.
The spontaneous change of an unstable nuclide into another is radioactive decay. The unstable nuclide is called the parent nuclide;
the nuclide that results from the decay is known as the daughter nuclide. The daughter nuclide may be stable, or it may decay itself.
The radiation produced during radioactive decay is such that the daughter nuclide lies closer to the band of stability than the parent
nuclide, so the location of a nuclide relative to the band of stability can serve as a guide to the kind of decay it will undergo (Figure
21.3.1).

Figure 21.3.1 : A nucleus of uranium-238 (the parent nuclide) undergoes α decay to form thorium-234 (the daughter nuclide). The
alpha particle removes two protons (green) and two neutrons (gray) from the uranium-238 nucleus.
A diagram shows two spheres composed of many smaller white and green spheres connected by a right-facing arrow with another,
down-facing arrow coming off of it. The left sphere, labeled “Parent nucleus uranium dash 238” has two white and two green
spheres that are near one another and are outlined in red. These two green and two white spheres are shown near the tip of the
down-facing arrow and labeled “alpha particle.” The right sphere, labeled “Daughter nucleus radon dash 234,” looks the same as
the left, but has a space for four smaller spheres outlined with a red dotted line.
Although the radioactive decay of a nucleus is too small to see with the naked eye, we can indirectly view radioactive decay in an
environment called a cloud chamber. Click here to learn about cloud chambers and to view an interesting Cloud Chamber
Demonstration from the Jefferson Lab.

Access for free at OpenStax 21.3.1 https://chem.libretexts.org/@go/page/38341


How to Build a Cloud Chamber!

Video 21.3.1 : How to Build a Cloud Chamber!

21.3.1: Types of Radioactive Decay


Ernest Rutherford’s experiments involving the interaction of radiation with a magnetic or electric field (Figure 21.3.2) helped him
determine that one type of radiation consisted of positively charged and relatively massive α particles; a second type was made up
of negatively charged and much less massive β particles; and a third was uncharged electromagnetic waves, γ rays. We now know
that α particles are high-energy helium nuclei, β particles are high-energy electrons, and γ radiation compose high-energy
electromagnetic radiation. We classify different types of radioactive decay by the radiation produced.

Figure 21.3.2 : Alpha particles, which are attracted to the negative plate and deflected by a relatively small amount, must be
positively charged and relatively massive. Beta particles, which are attracted to the positive plate and deflected a relatively large
amount, must be negatively charged and relatively light. Gamma rays, which are unaffected by the electric field, must be
uncharged.
A diagram is shown. A gray box on the left side of the diagram labeled “Lead block” has a chamber hollowed out in the center in
which a sample labeled “Radioactive substance” is placed. A blue beam is coming from the sample, out of the block, and passing
through two horizontally placed plates that are labeled “Electrically charged plates.” The top plate is labeled with a positive sign
while the bottom plate is labeled with a negative sign. The beam is shown to break into three beams as it passes in between the
plates; in order from top to bottom, they are red, labeled “beta rays,” purple labeled “gamma rays” and green labeled “alpha rays.”
The beams are shown to hit a vertical plate labeled “Photographic plate” on the far right side of the diagram.
Alpha (α ) decay is the emission of an α particle from the nucleus. For example, polonium-210 undergoes α decay:
210 4 206 210 4 206
Po ⟶ He + Pb or Po ⟶ α+ Pb
84 2 82 84 2 82

Alpha decay occurs primarily in heavy nuclei (A > 200, Z > 83). Because the loss of an α particle gives a daughter nuclide with a
mass number four units smaller and an atomic number two units smaller than those of the parent nuclide, the daughter nuclide has a
larger n:p ratio than the parent nuclide. If the parent nuclide undergoing α decay lies below the band of stability, the daughter
nuclide will lie closer to the band.
Beta (β) decay is the emission of an electron from a nucleus. Iodine-131 is an example of a nuclide that undergoes β decay:
131 0 131 131 0 131
53
I ⟶ e+ 54
X or 53
I ⟶ −1
β+ 54
Xe
−1

Access for free at OpenStax 21.3.2 https://chem.libretexts.org/@go/page/38341


Beta decay, which can be thought of as the conversion of a neutron into a proton and a β particle, is observed in nuclides with a
large n:p ratio. The beta particle (electron) emitted is from the atomic nucleus and is not one of the electrons surrounding the
nucleus. Such nuclei lie above the band of stability. Emission of an electron does not change the mass number of the nuclide but
does increase the number of its protons and decrease the number of its neutrons. Consequently, the n:p ratio is decreased, and the
daughter nuclide lies closer to the band of stability than did the parent nuclide.
Gamma emission (γ emission) is observed when a nuclide is formed in an excited state and then decays to its ground state with the
emission of a γ ray, a quantum of high-energy electromagnetic radiation. The presence of a nucleus in an excited state is often
indicated by an asterisk (*). Cobalt-60 emits γ radiation and is used in many applications including cancer treatment:
60 ∗ 0 60
Co ⟶ γ+ Co
27 0 27

There is no change in mass number or atomic number during the emission of a γ ray unless the γ emission accompanies one of the
other modes of decay.
Positron emission (β decay) is the emission of a positron from the nucleus. Oxygen-15 is an example of a nuclide that undergoes
+

positron emission:
15 0 15 15 0 15
O ⟶ e+ N or O ⟶ β+ N
8 +1 7 8 +1 7

Positron emission is observed for nuclides in which the n:p ratio is low. These nuclides lie below the band of stability. Positron
decay is the conversion of a proton into a neutron with the emission of a positron. The n:p ratio increases, and the daughter nuclide
lies closer to the band of stability than did the parent nuclide.
Electron capture occurs when one of the inner electrons in an atom is captured by the atom’s nucleus. For example, potassium-40
undergoes electron capture:
40 0 40
K+ e ⟶ Ar
19 −1 18

Electron capture occurs when an inner shell electron combines with a proton and is converted into a neutron. The loss of an inner
shell electron leaves a vacancy that will be filled by one of the outer electrons. As the outer electron drops into the vacancy, it will
emit energy. In most cases, the energy emitted will be in the form of an X-ray. Like positron emission, electron capture occurs for
“proton-rich” nuclei that lie below the band of stability. Electron capture has the same effect on the nucleus as does positron
emission: The atomic number is decreased by one and the mass number does not change. This increases the n:p ratio, and the
daughter nuclide lies closer to the band of stability than did the parent nuclide. Whether electron capture or positron emission
occurs is difficult to predict. The choice is primarily due to kinetic factors, with the one requiring the smaller activation energy
being the one more likely to occur. Figure 21.3.3 summarizes these types of decay, along with their equations and changes in
atomic and mass numbers.

Access for free at OpenStax 21.3.3 https://chem.libretexts.org/@go/page/38341


Figure 21.3.3 : This table summarizes the type, nuclear equation, representation, and any changes in the mass or atomic numbers
for various types of decay.
This table has four columns and six rows. The first row is a header row and it labels each column: “Type,” “Nuclear equation,”
“Representation,” and “Change in mass / atomic numbers.” Under the “Type” column are the following: “Alpha decay,” “Beta
decay,” “Gamma decay,” “Positron emission,” and “Electron capture.” Under the “Nuclear equation” column are several equations.
Each begins with superscript A stacked over subscript Z X. There is a large gap of space and then the following equations:
“superscript 4 stacked over subscript 2 He plus superscript A minus 4 stacked over subscript Z minus 2 Y,” “superscript 0 stacked
over subscript negative 1 e plus superscript A stacked over subscript Z plus 1 Y,” “superscript 0 stacked over subscript 0 lowercase
gamma plus superscript A stacked over subscript Z Y,” “superscript 0 stacked over subscript positive 1 e plus superscript A stacked
over subscript Y minus 1 Y,” and “superscript 0 stacked over subscript negative 1 e plus superscript A stacked over subscript Y
minus 1 Y.” Under the “Representation” column are the five diagrams. The first shows a cluster of green and white spheres. A
section of the cluster containing two white and two green spheres is outlined. There is a right-facing arrow pointing to a similar
cluster as previously described, but the outlined section is missing. From the arrow another arrow branches off and points
downward. The small cluster to two white spheres and two green spheres appear at the end of the arrow. The next diagram shows
the same cluster of white and green spheres. One white sphere is outlined. There is a right-facing arrow to a similar cluster, but the
white sphere is missing. Another arrow branches off the main arrow and a red sphere with a negative sign appears at the end. The
next diagram shows the same cluster of white and green spheres. The whole sphere is outlined and labeled, “excited nuclear state.”
There is a right-facing arrow that points to the same cluster. No spheres are missing. Off the main arrow is another arrow which
points to a purple squiggle arrow which in turn points to a lowercase gamma. The next diagram shows the same cluster of white
and green spheres. One green sphere is outlined. There is a right-facing arrow to a similar cluster, but the green sphere is missing.
Another arrow branches off the main arrow and a red sphere with a positive sign appears at the end. The next diagram shows the
same cluster of white and green spheres. One green sphere is outlined. There is a right-facing arrow to a similar cluster, but the
green sphere is missing. Two other arrows branch off the main arrow. The first shows a gold sphere with a negative sign joining
with the right-facing arrow. The secon points to a blue squiggle arrow labeled, “X-ray.” Under the “Change in mass / atomic
numbers” column are the following: “A: decrease by 4, Z: decrease by 2,” “A: unchanged, Z: increased by 1,” “A: unchanged, Z:
unchanged,” “A: unchanged, Z: unchanged,” “A: unchanged, Z: decrease by 1,” and “A: unchanged, Z: decrease by 1.”

 PET Scan

Positron emission tomography (PET) scans use radiation to diagnose and track health conditions and monitor medical
treatments by revealing how parts of a patient’s body function (Figure 21.3.4). To perform a PET scan, a positron-emitting
radioisotope is produced in a cyclotron and then attached to a substance that is used by the part of the body being investigated.
This “tagged” compound, or radiotracer, is then put into the patient (injected via IV or breathed in as a gas), and how it is used
by the tissue reveals how that organ or other area of the body functions.

Access for free at OpenStax 21.3.4 https://chem.libretexts.org/@go/page/38341


Figure 21.3.4 : A PET scanner (a) uses radiation to provide an image of how part of a patient’s body functions. The scans it
produces can be used to image a healthy brain (b) or can be used for diagnosing medical conditions such as Alzheimer’s
disease (c). (credit a: modification of work by Jens Maus)
Three pictures are shown and labeled “a,” “b” and “c.” Picture a shows a machine with a round opening connected to an
examination table. Picture b is a medical scan of the top of a person’s head and shows large patches of yellow and red and
smaller patches of blue, green and purple highlighting. Picture c also shows a medical scan of the top of a person’s head, but
this image is mostly colored in blue and purple with very small patches of red and yellow.
For example, F-18 is produced by proton bombardment of 18O ( O + p ⟶ F + n) and incorporated into a glucose
18
8
1
1
18
9
1
0

analog called fludeoxyglucose (FDG). How FDG is used by the body provides critical diagnostic information; for example,
since cancers use glucose differently than normal tissues, FDG can reveal cancers. The 18F emits positrons that interact with
nearby electrons, producing a burst of gamma radiation. This energy is detected by the scanner and converted into a detailed,
three-dimensional, color image that shows how that part of the patient’s body functions. Different levels of gamma radiation
produce different amounts of brightness and colors in the image, which can then be interpreted by a radiologist to reveal what
is going on. PET scans can detect heart damage and heart disease, help diagnose Alzheimer’s disease, indicate the part of a
brain that is affected by epilepsy, reveal cancer, show what stage it is, and how much it has spread, and whether treatments are
effective. Unlike magnetic resonance imaging and X-rays, which only show how something looks, the big advantage of PET
scans is that they show how something functions. PET scans are now usually performed in conjunction with a computed
tomography scan.

21.3.2: Radioactive Decay Series


The naturally occurring radioactive isotopes of the heaviest elements fall into chains of successive disintegrations, or decays, and
all the species in one chain constitute a radioactive family, or radioactive decay series. Three of these series include most of the
naturally radioactive elements of the periodic table. They are the uranium series, the actinide series, and the thorium series. The
neptunium series is a fourth series, which is no longer significant on the earth because of the short half-lives of the species
involved. Each series is characterized by a parent (first member) that has a long half-life and a series of daughter nuclides that
ultimately lead to a stable end-product—that is, a nuclide on the band of stability (Figure 21.3.5). In all three series, the end-
product is a stable isotope of lead. The neptunium series, previously thought to terminate with bismuth-209, terminates with
thallium-205.

Access for free at OpenStax 21.3.5 https://chem.libretexts.org/@go/page/38341


Figure 21.3.5 : Uranium-238 undergoes a radioactive decay series consisting of 14 separate steps before producing stable lead-206.
This series consists of eight α decays and six β decays.
A graph is shown where the x-axis is labeled “Number of neutrons, open parenthesis, n, close parenthesis” and has values of 122 to
148 in increments of 2. The y-axis is labeled “Atomic number” and has values of 80 to 92 in increments of 1. Two types of arrows
are used in this graph to connect the points. Green arrows are labeled as “alpha decay” while red arrows are labeled “beta decay.”
Beginning at the point “92, 146” that is labeled “superscript 238, U,” a green arrow connects this point to the second point “90,
144” which is labeled “superscript 234, T h.” A red arrow connect this to the third point “91, 143” which is labeled “superscript
234, P a” which is connected to the fourth point “92, 142” by a red arrow and which is labeled “superscript 234, U.” A green arrow
leads to the next point, “90, 140” which is labeled “superscript 230, T h” and is connected by a green arrow to the sixth point, “88,
138” which is labeled “superscript 226, R a” that is in turn connected by a green arrow to the seventh point “86, 136” which is
labeled “superscript 222, Ra.” The eighth point, at “84, 134” is labeled “superscript 218, P o” and has green arrows leading to it
and away from it to the ninth point “82, 132” which is labeled “superscript 214, Pb” which is connected by a red arrow to the tenth
point, “83, 131” which is labeled “superscript 214, B i.” A red arrow leads to the eleventh point “84, 130” which is labeled
“superscript 214, P o” and a green arrow leads to the twelvth point “82, 128” which is labeled “superscript 210, P b.” A red arrow
leads to the thirteenth point “83, 127” which is labeled “superscript 210, B i” and a red arrow leads to the fourteenth point “84,
126” which is labeled “superscript 210, P o.” The final point is labeled “82, 124” and “superscript 206, P b.”

21.3.3: Radioactive Half-Lives


Radioactive decay follows first-order kinetics. Since first-order reactions have already been covered in detail in the kinetics
chapter, we will now apply those concepts to nuclear decay reactions. Each radioactive nuclide has a characteristic, constant half-
life (t1/2), the time required for half of the atoms in a sample to decay. An isotope’s half-life allows us to determine how long a
sample of a useful isotope will be available, and how long a sample of an undesirable or dangerous isotope must be stored before it
decays to a low-enough radiation level that is no longer a problem.
For example, cobalt-60, an isotope that emits gamma rays used to treat cancer, has a half-life of 5.27 years (Figure 21.3.6). In a
given cobalt-60 source, since half of the Co nuclei decay every 5.27 years, both the amount of material and the intensity of the
60
27

radiation emitted is cut in half every 5.27 years. (Note that for a given substance, the intensity of radiation that it produces is
directly proportional to the rate of decay of the substance and the amount of the substance.) This is as expected for a process
following first-order kinetics. Thus, a cobalt-60 source that is used for cancer treatment must be replaced regularly to continue to be
effective.

Access for free at OpenStax 21.3.6 https://chem.libretexts.org/@go/page/38341


Figure 21.3.6 : For cobalt-60, which has a half-life of 5.27 years, 50% remains after 5.27 years (one half-life), 25% remains after
10.54 years (two half-lives), 12.5% remains after 15.81 years (three half-lives), and so on.
A graph, titled “C o dash 60 Decay,” is shown where the x-axis is labeled “C o dash 60 remaining, open parenthesis, percent sign,
close parenthesis” and has values of 0 to 100 in increments of 25. The y-axis is labeled “Number of half dash lives” and has values
of 0 to 5 in increments of 1. The first point, at “0, 100” has a circle filled with tiny dots drawn near it labeled “10 g.” The second
point, at “1, 50” has a smaller circle filled with tiny dots drawn near it labeled “5 g.” The third point, at “2, 25” has a small circle
filled with tiny dots drawn near it labeled “2.5 g.” The fourth point, at “3, 12.5” has a very small circle filled with tiny dots drawn
near it labeled “1.25 g.” The last point, at “4, 6.35” has a tiny circle filled with tiny dots drawn near it labeled.”625 g.”
Since nuclear decay follows first-order kinetics, we can adapt the mathematical relationships used for first-order chemical
reactions. We generally substitute the number of nuclei, N, for the concentration. If the rate is stated in nuclear decays per second,
we refer to it as the activity of the radioactive sample. The rate for radioactive decay is:

decay rate = λN

with λ is the decay constant for the particular radioisotope.


The decay constant, λ , which is the same as a rate constant discussed in the kinetics chapter. It is possible to express the decay
constant in terms of the half-life, t1/2:
ln 2 0.693 ln 2 0.693
λ = = or t1/2 = =
t1/2 t1/2 λ λ

The first-order equations relating amount, N, and time are:


1 Nt
−kt
Nt = N0 e or t =− ln( )
λ N0

where N0 is the initial number of nuclei or moles of the isotope, and Nt is the number of nuclei/moles remaining at time t. Example
21.3.1 applies these calculations to find the rates of radioactive decay for specific nuclides.

 Example 21.3.1: Rates of Radioactive Decay


60
27
Co decays with a half-life of 5.27 years to produce 60
28
Ni .
a. What is the decay constant for the radioactive disintegration of cobalt-60?
b. Calculate the fraction of a sample of the Co isotope that will remain after 15 years.
60
27

c. How long does it take for a sample of Co to disintegrate to the extent that only 2.0% of the original amount remains?
60
27

Solution
(a) The value of the rate constant is given by:

Access for free at OpenStax 21.3.7 https://chem.libretexts.org/@go/page/38341


ln 2 0.693 −1
λ = = = 0.132 y
t1/2 5.27 y

Nt
(b) The fraction of 60
27
Co that is left after time t is given by . Rearranging the first-order relationship Nt = N0e–λt to solve for
N0

this ratio yields:


Nt
−λt −(0.132/y)(15.0/y)
=e =e = 0.138
N0

The fraction of Co that will remain after 15.0 years is 0.138. Or put another way, 13.8% of the
60
27
60
27
Co originally present will
remain after 15 years.
(c) 2.00% of the original amount of 60
27
Co is equal to 0.0200 × N0. Substituting this into the equation for time for first-order
kinetics, we have:
1 Nt 1 0.0200 × N0
t =− ln( ) =− ln( ) = 29.6 y
−1
λ N0 0.132 y N0

 Exercise 21.3.1

Radon-222, Rn, has a half-life of 3.823 days. How long will it take a sample of radon-222 with a mass of 0.750 g to decay
222
86

into other elements, leaving only 0.100 g of radon-222?

Answer
11.1 days

Because each nuclide has a specific number of nucleons, a particular balance of repulsion and attraction, and its own degree of
stability, the half-lives of radioactive nuclides vary widely. For example: the half-life of Bi is 1.9 × 1019 years; Ra is 24,000
209
83
239
94

years; Rn is 3.82 days; and element-111 (Rg for roentgenium) is 1.5 × 10–3 seconds. The half-lives of a number of radioactive
222
86

isotopes important to medicine are shown in Table 21.3.1, and others are listed in Appendix N1.
Table 21.3.1 : Half-lives of Radioactive Isotopes Important to Medicine
Type Decay Mode Half-Life Uses

F-18 β+ decay 110. minutes PET scans

Co-60 β decay, γ decay 5.27 years cancer treatment

Tc-99m1 γ decay 8.01 hours scans of brain, lung, heart, bone

I-131 β decay 8.02 days thyroid scans and treatment

heart and arteries scans; cardiac


Tl-201 electron capture 73 hours
stress tests
The “m” in Tc-99m stands for “metastable,” indicating that this is an unstable, high-energy state of Tc-99. Metastable isotopes emit γ radiation to
rid themselves of excess energy and become (more) stable.

21.3.4: Radiometric Dating


Several radioisotopes have half-lives and other properties that make them useful for purposes of “dating” the origin of objects such
as archaeological artifacts, formerly living organisms, or geological formations. This process is radiometric dating and has been
responsible for many breakthrough scientific discoveries about the geological history of the earth, the evolution of life, and the
history of human civilization. We will explore some of the most common types of radioactive dating and how the particular
isotopes work for each type.

21.3.4.1: Radioactive Dating Using Carbon-14


The radioactivity of carbon-14 provides a method for dating objects that were a part of a living organism. This method of
radiometric dating, which is also called radiocarbon dating or carbon-14 dating, is accurate for dating carbon-containing substances

Access for free at OpenStax 21.3.8 https://chem.libretexts.org/@go/page/38341


that are up to about 30,000 years old, and can provide reasonably accurate dates up to a maximum of about 50,000 years old.
Naturally occurring carbon consists of three isotopes: C, which constitutes about 99% of the carbon on earth; C, about 1% of
12
6
13
6

the total; and trace amounts of C. Carbon-14 forms in the upper atmosphere by the reaction of nitrogen atoms with neutrons from
14
6

cosmic rays in space:


14 1 14 1
7
N+ n ⟶ 6
C + 1H
0

14 12
All isotopes of carbon react with oxygen to produce CO2 molecules. The ratio of CO to CO depends on the ratio of CO to
6 2 6 2
14
6

CO in the atmosphere. The natural abundance of CO in the atmosphere is approximately 1 part per trillion; until recently, this
12 14
6 6
14 12
has generally been constant over time, as seen is gas samples found trapped in ice. The incorporation of C CO and CO 14
6 6 2 6 2

into plants is a regular part of the photosynthesis process, which means that the C : C ratio found in a living plant is the same
14
6
12
6

as the C : C ratio in the atmosphere. But when the plant dies, it no longer traps carbon through photosynthesis. Because C is
14
6
12
6
12
6

a stable isotope and does not undergo radioactive decay, its concentration in the plant does not change. However, carbon-14 decays
by β emission with a half-life of 5730 years:
14 14 0
6
C ⟶ 7
N+ e
−1

Thus, the C : C ratio gradually decreases after the plant dies. The decrease in the ratio with time provides a measure of the time
14
6
12
6

that has elapsed since the death of the plant (or other organism that ate the plant). Figure 21.3.7 visually depicts this process.

Figure 21.3.7 : Along with stable carbon-12, radioactive carbon-14 is taken in by plants and animals, and remains at a constant
level within them while they are alive. After death, the C-14 decays and the C-14:C-12 ratio in the remains decreases. Comparing
this ratio to the C-14:C-12 ratio in living organisms allows us to determine how long ago the organism lived (and died).
A diagram shows a cow standing on the ground next to a tree. In the upper left of the diagram, where the sky is represented, a
single white sphere is shown and is connected by a downward-facing arrow to a larger sphere composed of green and white spheres
that is labeled “superscript 14, subscript 7, N.” This structure is connected to three other structures by a right-facing arrow. Each of
the three it points to are composed of green and white spheres and all have arrows pointing from them to the ground. The first of
these is labeled “Trace, superscript 14, subscript 6, C,” the second is labeled “1 percent, superscript 13, subscript 6, C” and the last
is labeled “99 percent, superscript 12, subscript 6, C.” Two downward-facing arrows that merge into one arrow lead from the cow
and tree to the ground and are labeled “organism dies” and “superscript 14, subscript 6, C, decay begins.” A right-facing arrow
labeled on top as “Decay” and on bottom as “Time” leads from this to a label of “superscript 14, subscript 6, C, backslash,
superscript 12, subscript 6, C, ratio decreased.” Near the top of the tree is a downward facing arrow with the label “superscript 14,
subscript 6, C, backslash, superscript 12, subscript 6, C, ratio is constant in living organisms” that leads to the last of the lower
statements.
For example, with the half-life of C being 5730 years, if the C : C ratio in a wooden object found in an archaeological dig is
14
6
14
6
12
6

half what it is in a living tree, this indicates that the wooden object is 5730 years old. Highly accurate determinations of C : C 14
6
12
6

ratios can be obtained from very small samples (as little as a milligram) by the use of a mass spectrometer.

Access for free at OpenStax 21.3.9 https://chem.libretexts.org/@go/page/38341


 Example 21.3.2: Radiocarbon Dating
A tiny piece of paper (produced from formerly living plant matter) taken from the Dead Sea Scrolls has an activity of 10.8
disintegrations per minute per gram of carbon. If the initial C-14 activity was 13.6 disintegrations/min/g of C, estimate the age
of the Dead Sea Scrolls.

Solution
The rate of decay (number of disintegrations/minute/gram of carbon) is proportional to the amount of radioactive C-14 left in
the paper, so we can substitute the rates for the amounts, N, in the relationship:
1 Nt 1 Ratet
t =− ln( ) ⟶ t =− ln( )
λ N0 λ Rate0

where the subscript 0 represents the time when the plants were cut to make the paper, and the subscript t represents the current
time.
The decay constant can be determined from the half-life of C-14, 5730 years:
ln 2 0.693
−4 −1
λ = = = 1.21 × 10 y
t1/2 5730 y

Substituting and solving, we have:


1 Ratet 1 10.8 dis/min/g C
t =− ln( ) =− ln( ) = 1910 y
−4 −1
λ Rate0 1.21 × 10 y 13.6 dis/min/g C

Therefore, the Dead Sea Scrolls are approximately 1900 years old (Figure 21.3.8).

Figure 21.3.8 : Carbon-14 dating has shown that these pages from the Dead Sea Scrolls were written or copied on paper made
from plants that died between 100 BC and AD 50.

 Exercise 21.3.2
More accurate dates of the reigns of ancient Egyptian pharaohs have been determined recently using plants that were preserved
in their tombs. Samples of seeds and plant matter from King Tutankhamun’s tomb have a C-14 decay rate of 9.07
disintegrations/min/g of C. How long ago did King Tut’s reign come to an end?

Answer
about 3350 years ago, or approximately 1340 BC

There have been some significant, well-documented changes to the C : C ratio. The accuracy of a straightforward application
14
6
12
6

of this technique depends on the C : C ratio in a living plant being the same now as it was in an earlier era, but this is not
14
6
12
6
12
always valid. Due to the increasing accumulation of CO2 molecules (largely CO ) in the atmosphere caused by combustion of
6 2

fossil fuels (in which essentially all of the C has decayed), the ratio of C : C in the atmosphere may be changing. This
14
6
14
6
12
6
12
manmade increase in CO in the atmosphere causes the C : C ratio to decrease, and this in turn affects the ratio in currently
6 2
14
6
12
6

living organisms on the earth. Fortunately, however, we can use other data, such as tree dating via examination of annual growth
rings, to calculate correction factors. With these correction factors, accurate dates can be determined. In general, radioactive dating
only works for about 10 half-lives; therefore, the limit for carbon-14 dating is about 57,000 years.

Access for free at OpenStax 21.3.10 https://chem.libretexts.org/@go/page/38341


21.3.4.2: Radioactive Dating Using Nuclides Other than Carbon-14
Radioactive dating can also use other radioactive nuclides with longer half-lives to date older events. For example, uranium-238
(which decays in a series of steps into lead-206) can be used for establishing the age of rocks (and the approximate age of the oldest
rocks on earth). Since U-238 has a half-life of 4.5 billion years, it takes that amount of time for half of the original U-238 to decay
into Pb-206. In a sample of rock that does not contain appreciable amounts of Pb-208, the most abundant isotope of lead, we can
assume that lead was not present when the rock was formed. Therefore, by measuring and analyzing the ratio of U-238:Pb-206, we
can determine the age of the rock. This assumes that all of the lead-206 present came from the decay of uranium-238. If there is
additional lead-206 present, which is indicated by the presence of other lead isotopes in the sample, it is necessary to make an
adjustment. Potassium-argon dating uses a similar method. K-40 decays by positron emission and electron capture to form Ar-40
with a half-life of 1.25 billion years. If a rock sample is crushed and the amount of Ar-40 gas that escapes is measured,
determination of the Ar-40:K-40 ratio yields the age of the rock. Other methods, such as rubidium-strontium dating (Rb-87 decays
into Sr-87 with a half-life of 48.8 billion years), operate on the same principle. To estimate the lower limit for the earth’s age,
scientists determine the age of various rocks and minerals, making the assumption that the earth is older than the oldest rocks and
minerals in its crust. As of 2014, the oldest known rocks on earth are the Jack Hills zircons from Australia, found by uranium-lead
dating to be almost 4.4 billion years old.

 Example 21.3.3: Radioactive Dating of Rocks

An igneous rock contains 9.58 × 10–5 g of U-238 and 2.51 × 10–5 g of Pb-206, and much, much smaller amounts of Pb-208.
Determine the approximate time at which the rock formed.

Solution
The sample of rock contains very little Pb-208, the most common isotope of lead, so we can safely assume that all the Pb-206
in the rock was produced by the radioactive decay of U-238. When the rock formed, it contained all of the U-238 currently in
it, plus some U-238 that has since undergone radioactive decay.
The amount of U-238 currently in the rock is:

⎛ 1 mol U ⎞
−5 −7
9.58 × 10 g U × = 4.03 × 10 mol U
⎝ 238 g U ⎠

Because when one mole of U-238 decays, it produces one mole of Pb-206, the amount of U-238 that has undergone radioactive
decay since the rock was formed is:

⎛ 1 mol Pb ⎞ 1 mol U
−5 −7
2.51 × 10 g Pb × ×( ) = 1.22 × 10 mol U
⎝ 206 g Pb ⎠ 1 mol Pb

The total amount of U-238 originally present in the rock is therefore:


−7 −7 −7
4.03 × 10 mol + 1.22 × 10 mol = 5.25 × 10 mol U

The amount of time that has passed since the formation of the rock is given by:
1 Nt
t =− ln( )
λ N0

with N0 representing the original amount of U-238 and Nt representing the present amount of U-238.
U-238 decays into Pb-206 with a half-life of 4.5 × 109 y, so the decay constant λ is:
ln 2 0.693
−10 −1
λ = = = 1.54 × 10 y
9
t1/2 4.5 × 10 y

Substituting and solving, we have:

Access for free at OpenStax 21.3.11 https://chem.libretexts.org/@go/page/38341


−7
1 4.03 × 10 mol U
9
t =− ln( ) = 1.7 × 10 y
−10 −1 −7
1.54 × 10 y 5.25 × 10 mol U

Therefore, the rock is approximately 1.7 billion years old.

 Exercise 21.3.3

A sample of rock contains 6.14 × 10–4 g of Rb-87 and 3.51 × 10–5 g of Sr-87. Calculate the age of the rock. (The half-life of
the β decay of Rb-87 is 4.7 × 1010 y.)

Answer
3.7 × 109 y

Summary
Nuclei that have unstable n:p ratios undergo spontaneous radioactive decay. The most common types of radioactivity are α decay, β
decay, γ emission, positron emission, and electron capture. Nuclear reactions also often involve γ rays, and some nuclei decay by
electron capture. Each of these modes of decay leads to the formation of a new nucleus with a more stable n:p ratio. Some
substances undergo radioactive decay series, proceeding through multiple decays before ending in a stable isotope. All nuclear
decay processes follow first-order kinetics, and each radioisotope has its own characteristic half-life, the time that is required for
half of its atoms to decay. Because of the large differences in stability among nuclides, there is a very wide range of half-lives of
radioactive substances. Many of these substances have found useful applications in medical diagnosis and treatment, determining
the age of archaeological and geological objects, and more.

21.3.5: Key Equations


decay rate = λN
ln 2 0.693
t1/2 = =
λ λ

Glossary
alpha (α) decay
loss of an alpha particle during radioactive decay

beta (β) decay


breakdown of a neutron into a proton, which remains in the nucleus, and an electron, which is emitted as a beta particle

daughter nuclide
nuclide produced by the radioactive decay of another nuclide; may be stable or may decay further

electron capture
combination of a core electron with a proton to yield a neutron within the nucleus

gamma (γ) emission


decay of an excited-state nuclide accompanied by emission of a gamma ray

half-life (t1/2)
time required for half of the atoms in a radioactive sample to decay

parent nuclide
unstable nuclide that changes spontaneously into another (daughter) nuclide

positron emission
(also, β+ decay) conversion of a proton into a neutron, which remains in the nucleus, and a positron, which is emitted

Access for free at OpenStax 21.3.12 https://chem.libretexts.org/@go/page/38341


radioactive decay
spontaneous decay of an unstable nuclide into another nuclide

radioactive decay series


chains of successive disintegrations (radioactive decays) that ultimately lead to a stable end-product

radiocarbon dating
highly accurate means of dating objects 30,000–50,000 years old that were derived from once-living matter; achieved by
calculating the ratio of C : C in the object vs. the ratio of C : C in the present-day atmosphere
14
6
12
6
14
6
12
6

radiometric dating
use of radioisotopes and their properties to date the formation of objects such as archeological artifacts, formerly living
organisms, or geological formations

This page titled 21.3: Radioactive Decay is shared under a CC BY 4.0 license and was authored, remixed, and/or curated by OpenStax via source
content that was edited to the style and standards of the LibreTexts platform; a detailed edit history is available upon request.

Access for free at OpenStax 21.3.13 https://chem.libretexts.org/@go/page/38341


21.4: Transmutation and Nuclear Energy
 Learning Objectives
Describe the synthesis of transuranium nuclides
Explain nuclear fission and fusion processes
Relate the concepts of critical mass and nuclear chain reactions
Summarize basic requirements for nuclear fission and fusion reactors

After the discovery of radioactivity, the field of nuclear chemistry was created and developed rapidly during the early twentieth century. A slew of new discoveries in the 1930s and 1940s, along
with World War II, combined to usher in the Nuclear Age in the mid-twentieth century. Science learned how to create new substances, and certain isotopes of certain elements were found to possess
the capacity to produce unprecedented amounts of energy, with the potential to cause tremendous damage during war, as well as produce enormous amounts of power for society’s needs during
peace.

21.4.1: Synthesis of Nuclides


Nuclear transmutation is the conversion of one nuclide into another. It can occur by the radioactive decay of a nucleus, or the reaction of a nucleus with another particle. The first manmade nucleus
was produced in Ernest Rutherford’s laboratory in 1919 by a transmutation reaction, the bombardment of one type of nuclei with other nuclei or with neutrons. Rutherford bombarded nitrogen
atoms with high-speed α particles from a natural radioactive isotope of radium and observed protons resulting from the reaction:
14 4 17 1
N+ He ⟶ O+ H
7 2 8 1

The 17
8
O and 1
1
H nuclei that are produced are stable, so no further (nuclear) changes occur.
To reach the kinetic energies necessary to produce transmutation reactions, devices called particle accelerators are used. These devices use magnetic and electric fields to increase the speeds of
nuclear particles. In all accelerators, the particles move in a vacuum to avoid collisions with gas molecules. When neutrons are required for transmutation reactions, they are usually obtained from
radioactive decay reactions or from various nuclear reactions occurring in nuclear reactors. The Chemistry in Everyday Life feature that follows discusses a famous particle accelerator that made
worldwide news.

 CERN Particle Accelerator


Located near Geneva, the CERN (“Conseil Européen pour la Recherche Nucléaire,” or European Council for Nuclear Research) Laboratory is the world’s premier center for the investigations
of the fundamental particles that make up matter. It contains the 27-kilometer (17 mile) long, circular Large Hadron Collider (LHC), the largest particle accelerator in the world (Figure 21.4.1).
In the LHC, particles are boosted to high energies and are then made to collide with each other or with stationary targets at nearly the speed of light. Superconducting electromagnets are used to
produce a strong magnetic field that guides the particles around the ring. Specialized, purpose-built detectors observe and record the results of these collisions, which are then analyzed by
CERN scientists using powerful computers.

Figure 21.4.1 : A small section of the LHC is shown with workers traveling along it. (credit: Christophe Delaere)
In 2012, CERN announced that experiments at the LHC showed the first observations of the Higgs boson, an elementary particle that helps explain the origin of mass in fundamental particles.
This long-anticipated discovery made worldwide news and resulted in the awarding of the 2103 Nobel Prize in Physics to François Englert and Peter Higgs, who had predicted the existence of
this particle almost 50 years previously.

Prior to 1940, the heaviest-known element was uranium, whose atomic number is 92. Now, many artificial elements have been synthesized and isolated, including several on such a large scale that
they have had a profound effect on society. One of these—element 93, neptunium (Np)—was first made in 1940 by McMillan and Abelson by bombarding uranium-238 with neutrons. The reaction
creates unstable uranium-239, with a half-life of 23.5 minutes, which then decays into neptunium-239. Neptunium-239 is also radioactive, with a half-life of 2.36 days, and it decays into plutonium-
239. The nuclear reactions are:
238 1 239
U+ n ⟶ U
92 0 92

239 239 0
92
U ⟶ 93
Np + e t 1
half-life = 23.5 min
−1
2

239 239 0
93
Np ⟶ 94
Pu + e t 1
half-life = 2.36 days
−1
2

Plutonium is now mostly formed in nuclear reactors as a byproduct during the decay of uranium. Some of the neutrons that are released during U-235 decay combine with U-238 nuclei to form
uranium-239; this undergoes β decay to form neptunium-239, which in turn undergoes β decay to form plutonium-239 as illustrated in the preceding three equations. It is possible to summarize
these equations as:
− −
β β
238 1 239 239 239
U+ n⟶ U−→ Np −→ Pu
92 0 92 93 94

Heavier isotopes of plutonium—Pu-240, Pu-241, and Pu-242—are also produced when lighter plutonium nuclei capture neutrons. Some of this highly radioactive plutonium is used to produce
military weapons, and the rest presents a serious storage problem because they have half-lives from thousands to hundreds of thousands of years.
Although they have not been prepared in the same quantity as plutonium, many other synthetic nuclei have been produced. Nuclear medicine has developed from the ability to convert atoms of one
type into other types of atoms. Radioactive isotopes of several dozen elements are currently used for medical applications. The radiation produced by their decay is used to image or treat various
organs or portions of the body, among other uses.
The elements beyond element 92 (uranium) are called transuranium elements. As of this writing, 22 transuranium elements have been produced and officially recognized by IUPAC; several other
elements have formation claims that are waiting for approval. Some of these elements are shown in Table 21.4.1.
Table 21.4.1 : Preparation of Some of the Transuranium Elements
Name Symbol Atomic Number Reaction
239 240
americium Am 95 94
Pu +
1
0
n ⟶
95
Am +
0
−1
e

curium Cm 96 239
94
Pu +
4
2 He ⟶
242
96 Cm +
1
0
n

Access for free at OpenStax 21.4.1 https://chem.libretexts.org/@go/page/38342


Name Symbol Atomic Number Reaction

californium Cf 98 242
96 Cm +
4
2 He ⟶
245
98 Cf +
1
0
n

238 253
einsteinium Es 99 92
U + 15
1
0
n ⟶
99
Es + 7
0
−1
e

mendelevium Md 101 253


99
Es +
4
2 He ⟶
256
101 Md +
1
0
n

nobelium No 102 246


96 Cm +
12
6C ⟶
254
102
No + 4
1
0
n

rutherfordium Rf 104 249


98 Cf +
12
6C ⟶
257
104 Rf +4
1
0
n

206 54 257 1
82 Pb + 24 Cr ⟶ 106 Sg +3 n
seaborgium Sg 106 249 18 263
0
1
Cf + O ⟶ Sg + 4 n
98 8 106 0

meitnerium Mt 107 209


83
Bi +
58
26
Fe ⟶
266
109
Mt +
1
0
n

21.4.2: Nuclear Fission


Many heavier elements with smaller binding energies per nucleon can decompose into more stable elements that have intermediate mass numbers and larger binding energies per nucleon—that is,
mass numbers and binding energies per nucleon that are closer to the “peak” of the binding energy graph near 56. Sometimes neutrons are also produced. This decomposition is called fission, the
breaking of a large nucleus into smaller pieces. The breaking is rather random with the formation of a large number of different products. Fission usually does not occur naturally, but is induced by
bombardment with neutrons. The first reported nuclear fission occurred in 1939 when three German scientists, Lise Meitner, Otto Hahn, and Fritz Strassman, bombarded uranium-235 atoms with
slow-moving neutrons that split the U-238 nuclei into smaller fragments that consisted of several neutrons and elements near the middle of the periodic table. Since then, fission has been observed
in many other isotopes, including most actinide isotopes that have an odd number of neutrons. A typical nuclear fission reaction is shown in Figure 21.4.2.

Figure 21.4.2 : When a slow neutron hits a fissionable U-235 nucleus, it is absorbed and forms an unstable U-236 nucleus. The U-236 nucleus then rapidly breaks apart into two smaller nuclei (in
this case, Ba-141 and Kr-92) along with several neutrons (usually two or three), and releases a very large amount of energy.
A diagram is shown which has a white sphere labeled “superscript, 1, subscript 0, n” followed by a right-facing arrow and a large sphere composed of many smaller white and green spheres labeled
“superscript, 235, subscript 92, U.” The single sphere has impacted the larger sphere. A right-facing arrow leads from the larger sphere to a vertical dumbbell shaped collection of the same white
and green spheres labeled “superscript, 236, subscript 92, U, Unstable nucleus.” Two right-facing arrows lead from the top and bottom of this structure to two new spheres that are also composed of
green and white spheres and are slightly smaller than the others. The top sphere is labeled “superscript, 92, subscript 36, K r” while the lower one is labeled “superscript, 141, subscript 56, B a.” A
starburst pattern labeled “Energy” lies between these two spheres and has three right-facing arrows leading from it to three white spheres labeled “3, superscript, 1, subscript 0, n.” A balanced
nuclear equation is written below the diagram and says “superscript, 235, subscript 92, U, plus sign, superscript, 1, subscript 0, n, yield arrow, superscript, 236, subscript 92, U, yield arrow,
superscript, 141, subscript 56, B a, plus sign, superscript, 92, subscript 36, K r, plus sign, 3, superscript, 1, subscript 0, n.”
Among the products of Meitner, Hahn, and Strassman’s fission reaction were barium, krypton, lanthanum, and cerium, all of which have nuclei that are more stable than uranium-235. Since then,
hundreds of different isotopes have been observed among the products of fissionable substances. A few of the many reactions that occur for U-235, and a graph showing the distribution of its
fission products and their yields, are shown in Figure 21.4.3. Similar fission reactions have been observed with other uranium isotopes, as well as with a variety of other isotopes such as those of
plutonium.

Figure 21.4.3 : (a) Nuclear fission of U-235 produces a range of fission products. (b) The larger fission products of U-235 are typically one isotope with a mass number around 85–105, and another
isotope with a mass number that is about 50% larger, that is, about 130–150.
Five nuclear equations and a graph are shown. The first equation is “superscript, 235, subscript 92, U, plus sign, superscript, 1, subscript 0, n, yield arrow, superscript, 236, subscript 92, U, yield
arrow, superscript, 90, subscript 38, S r, plus sign, superscript, 144, subscript 54, X e, plus sign, 2, superscript, 1, subscript 0, n.” The second equation is “superscript, 235, subscript 92, U, plus sign,
superscript, 1, subscript 0, n, yield arrow, superscript, 236, subscript 92, U, yield arrow, superscript, 87, subscript 35, B r, plus sign, superscript, 146, subscript 57, L a, plus sign, 3, superscript, 1,
subscript 0, n.” The third equation is “superscript, 235, subscript 92, U, plus sign, superscript, 1, subscript 0, n, yield arrow, superscript, 236, subscript 92, U, yield arrow, superscript, 97, subscript
37, R b, plus sign, superscript, 137, subscript 55, C s, plus sign, 3, superscript, 1, subscript 0, n.” The fourth equation is “superscript, 235, subscript 92, U, plus sign, superscript, 1, subscript 0, n,
yield arrow, superscript, 236, subscript 92, U, yield arrow, superscript, 137, subscript 52, T e, plus sign, superscript, 97, subscript 40, Z r, plus sign, 2, superscript, 1, subscript 0, n.” The fifth
equation is “superscript, 235, subscript 92, U, plus sign, superscript, 1, subscript 0, n, yield arrow, superscript, 236, subscript 92, U, yield arrow, superscript, 141, subscript 56, B a, plus sign,
superscript, 92, subscript 36, K r, plus sign, 3, superscript, 1, subscript 0, n.” A graph is also shown where the y-axis is labeled “Fission yield, open parenthesis, percent sign, close parenthesis” and
has values of 0 to 9 in increments of 1 while the x-axis is labeled “Mass number” and has values of 60 to 180 in increments of 20. The graph begins near point “65, 0” and rises rapidly to near “92,
6.6,” then drops just as rapidly to “107, 0” and remains there to point “127, 0.” The graph then rises again to near “132, 8,” then goes up and down a bit before falling to a point “153, 0,” and going
horizontal.
A tremendous amount of energy is produced by the fission of heavy elements. For instance, when one mole of U-235 undergoes fission, the products weigh about 0.2 grams less than the reactants;
this “lost” mass is converted into a very large amount of energy, about 1.8 × 1010 kJ per mole of U-235. Nuclear fission reactions produce incredibly large amounts of energy compared to chemical
reactions. The fission of 1 kilogram of uranium-235, for example, produces about 2.5 million times as much energy as is produced by burning 1 kilogram of coal.
As described earlier, when undergoing fission U-235 produces two “medium-sized” nuclei, and two or three neutrons. These neutrons may then cause the fission of other uranium-235 atoms, which
in turn provide more neutrons that can cause fission of even more nuclei, and so on. If this occurs, we have a nuclear chain reaction (Figure 21.4.4). On the other hand, if too many neutrons escape
the bulk material without interacting with a nucleus, then no chain reaction will occur.

Access for free at OpenStax 21.4.2 https://chem.libretexts.org/@go/page/38342


Figure 21.4.4 : The fission of a large nucleus, such as U-235, produces two or three neutrons, each of which is capable of causing fission of another nucleus by the reactions shown. If this process
continues, a nuclear chain reaction occurs.
A diagram is shown which has a white sphere labeled “superscript, 1, subscript 0, n” followed by a right-facing arrow and a large sphere composed of many smaller white and green spheres labeled
“superscript, 235, subscript 92, U.” The single sphere has impacted the larger sphere. A right-facing arrow leads from the larger sphere to a pair of smaller spheres which are collections of the same
white and green spheres. The upper of these two images is labeled “superscript, 93, subscript 36, K r” while the lower of the two is labeled “superscript, 142, subscript 56, B a.” A starburst pattern
labeled “Energy” lies between these two spheres and has three right-facing arrows leading from it to three white spheres labeled “ superscript, 1, subscript 0, n.” An equation below this portion of
the diagram reads ““superscript, 235, subscript 92, U, plus sign, superscript, 1, subscript 0, n, yield arrow, superscript, 140, subscript 56, B a, plus sign, superscript 90, subscript 36, K r, plus sign, 3,
superscript 1, subscript 0, n.” A right-facing arrow leads from each of these white spheres to three larger spheres, each composed of many smaller green and white spheres and labeled, from top to
bottom as “a, superscript,235, subscript 92, U,” “b, superscript,235, subscript 92, U” and “c, superscript,235, subscript 92, U.” Each of these spheres is followed by a right-facing arrow which
points to a pair of smaller spheres composed of the same green and white spheres with starburst patterns in between each pair labeled “Energy.” The spheres of the top pair are labeled, from top to
bottom, “superscript, 96, subscript 37, R b” and “superscript, 137, subscript 55, C s.” The spheres of the middle pair are labeled, from top to bottom, “superscript, 90, subscript 38, S r” and
“superscript, 144, subscript 54, X e.” The spheres of the bottom pair are labeled, from top to bottom, “superscript, 87, subscript 35, B r” and “superscript, 146, subscript 57, L a.” Each pair of
spheres is followed by three right-facing arrows leading to three white spheres labeled “superscript, 1, subscript 0, n.” Below the diagram are three nuclear equations. Equation a reads “superscript,
235, subscript 92, U, plus sign, superscript, 1, subscript 0, n, yield arrow, superscript, 96, subscript 37, R b, plus sign, superscript 137, subscript 55, C s, plus sign, 3, superscript 1, subscript 0, n.”
Equation b reads “superscript, 235, subscript 92, U, plus sign, superscript, 1, subscript 0, n, yield arrow, superscript, 90, subscript 38, S r, plus sign, superscript144, subscript 54, X e, plus sign, 2,
superscript 1, subscript 0, n.” Equation c reads “superscript, 235, subscript 92, U, plus sign, superscript, 1, subscript 0, n, yield arrow, superscript, 87, subscript 35, B r, plus sign, superscript 146,
subscript 57, L a, plus sign, 3, superscript 1, subscript 0, n”
Material that can sustain a nuclear fission chain reaction is said to be fissile or fissionable. (Technically, fissile material can undergo fission with neutrons of any energy, whereas fissionable material
requires high-energy neutrons.) Nuclear fission becomes self-sustaining when the number of neutrons produced by fission equals or exceeds the number of neutrons absorbed by splitting nuclei plus
the number that escape into the surroundings. The amount of a fissionable material that will support a self-sustaining chain reaction is a critical mass. An amount of fissionable material that cannot
sustain a chain reaction is a subcritical mass. An amount of material in which there is an increasing rate of fission is known as a supercritical mass. The critical mass depends on the type of material:
its purity, the temperature, the shape of the sample, and how the neutron reactions are controlled (Figure 21.4.5).

Figure 21.4.5 : (a) In a subcritical mass, the fissile material is too small and allows too many neutrons to escape the material, so a chain reaction does not occur. (b) In a critical mass, a large enough
number of neutrons in the fissile material induce fission to create a chain reaction.
The images are shown and labeled “a,” “b” and “c.” Image a, labeled “Sub-critical mass,” shows a blue circle background with a white sphere near the outer, top, left edge of the circle. A
downward, right-facing arrow indicates that the white sphere enters the circle. Seven small, yellow starbursts are drawn in the blue circle and each has an arrow facing from it to outside the circle,
in seemingly random directions. Image b, labeled “Critical mass,” shows a blue circle background with a white sphere near the outer, top, left edge of the circle. A downward, right-facing arrow
indicates that the white sphere enters the circle. Seventeen small, yellow starbursts are drawn in the blue circle and each has an arrow facing from it to outside the circle, in seemingly random
directions. Image c, labeled “Critical mass from neutron deflection,” shows a blue circle background, lying in a larger purple circle, with a white sphere near the outer, top, left edge of the purple
circle. A downward, right-facing arrow indicates that the white sphere enters both of the circles. Thirteen small, yellow starbursts are drawn in the blue circle and each has an arrow facing from it to
outside the blue circle, and a couple outside of the purple circle, in seemingly random directions.
An atomic bomb (Figure 21.4.6) contains several pounds of fissionable material, U or Pu, a source of neutrons, and an explosive device for compressing it quickly into a small volume. When
235
92
239
94

fissionable material is in small pieces, the proportion of neutrons that escape through the relatively large surface area is great, and a chain reaction does not take place. When the small pieces of
fissionable material are brought together quickly to form a body with a mass larger than the critical mass, the relative number of escaping neutrons decreases, and a chain reaction and explosion
result.

Access for free at OpenStax 21.4.3 https://chem.libretexts.org/@go/page/38342


Figure 21.4.6 : (a) The nuclear fission bomb that destroyed Hiroshima on August 6, 1945, consisted of two subcritical masses of U-235, where conventional explosives were used to fire one of the
subcritical masses into the other, creating the critical mass for the nuclear explosion. (b) The plutonium bomb that destroyed Nagasaki on August 12, 1945, consisted of a hollow sphere of
plutonium that was rapidly compressed by conventional explosives. This led to a concentration of plutonium in the center that was greater than the critical mass necessary for the nuclear explosion.
Two diagrams are shown, each to the left of a photo, and labeled “a” and “b.” Diagram a shows the outer casing of a bomb that has a long, tubular shape with a squared-off tail. Components in the
shell show a tube with a white disk labeled “Detonator” on the left, an orange disk with a bright yellow starburst drawn around it labeled “Conventional explosive” in the middle and a right-facing
arrow leading to a blue disk in the nose of the bomb labeled “uranium 235.” A small blue cone next to the orange disk is shares the label of “uranium 235.” A black and white photo next to this
diagram shows a far-off shot of a rising cloud over a landscape. Diagram b shows the outer casing of a bomb that has a short, rounded shape with a squared-off tail. Components in the shell show a
large orange circle labeled “Conventional explosive” with a series of black dots around its edge, labeled “Detonators,” and a yellow starburst behind it. White arrows face from the outer edge of the
orange circle to a blue circle in the center with a yellow core. The blue circle is labeled “plutonium 239” while the yellow core is labeled “beryllium, dash, polonium initiator.” A black and white
photo next to this diagram shows a far-off shot of a giant rising cloud over a landscape.

21.4.2.1: Fission Reactors


Chain reactions of fissionable materials can be controlled and sustained without an explosion in a nuclear reactor (Figure 21.4.7). Any nuclear reactor that produces power via the fission of uranium
or plutonium by bombardment with neutrons must have at least five components: nuclear fuel consisting of fissionable material, a nuclear moderator, reactor coolant, control rods, and a shield and
containment system. We will discuss these components in greater detail later in the section. The reactor works by separating the fissionable nuclear material such that a critical mass cannot be
formed, controlling both the flux and absorption of neutrons to allow shutting down the fission reactions. In a nuclear reactor used for the production of electricity, the energy released by fission
reactions is trapped as thermal energy and used to boil water and produce steam. The steam is used to turn a turbine, which powers a generator for the production of electricity.

Figure 21.4.7 : (a) The Diablo Canyon Nuclear Power Plant near San Luis Obispo is the only nuclear power plant currently in operation in California. The domes are the containment structures for
the nuclear reactors, and the brown building houses the turbine where electricity is generated. Ocean water is used for cooling. (b) The Diablo Canyon uses a pressurized water reactor, one of a few
different fission reactor designs in use around the world, to produce electricity. Energy from the nuclear fission reactions in the core heats water in a closed, pressurized system. Heat from this
system produces steam that drives a turbine, which in turn produces electricity. (credit a: modification of work by “Mike” Michael L. Baird; credit b: modification of work by the Nuclear
Regulatory Commission)
A photo labeled “a” and a diagram labeled “b” is shown. The photo is of a power plant with two large white domes and many buildings. The diagram shows a cylindrical container with thick walls
labeled “Walls made of concrete and steel” and three main components inside. The first of these components is a pair of tall cylinders labeled “Steam generators” that sit to either side of a shorter
cylinder labeled “Core.” Next to the core is a thin cylinder labeled “Pressurizer.” To the left of the outer walls is a set of pistons labeled “Turbines” that sit above a series of other equipment.

21.4.2.2: Nuclear Fuels


Nuclear fuel consists of a fissionable isotope, such as uranium-235, which must be present in sufficient quantity to provide a self-sustaining chain reaction. In the United States, uranium ores
contain from 0.05–0.3% of the uranium oxide U3O8; the uranium in the ore is about 99.3% nonfissionable U-238 with only 0.7% fissionable U-235. Nuclear reactors require a fuel with a higher
concentration of U-235 than is found in nature; it is normally enriched to have about 5% of uranium mass as U-235. At this concentration, it is not possible to achieve the supercritical mass
necessary for a nuclear explosion. Uranium can be enriched by gaseous diffusion (the only method currently used in the US), using a gas centrifuge, or by laser separation.
In the gaseous diffusion enrichment plant where U-235 fuel is prepared, UF6 (uranium hexafluoride) gas at low pressure moves through barriers that have holes just barely large enough for UF6 to
pass through. The slightly lighter 235UF6 molecules diffuse through the barrier slightly faster than the heavier 238UF6 molecules. This process is repeated through hundreds of barriers, gradually
increasing the concentration of 235UF6 to the level needed by the nuclear reactor. The basis for this process, Graham’s law, is described in the chapter on gases. The enriched UF6 gas is collected,
cooled until it solidifies, and then taken to a fabrication facility where it is made into fuel assemblies. Each fuel assembly consists of fuel rods that contain many thimble-sized, ceramic-encased,
enriched uranium (usually UO2) fuel pellets. Modern nuclear reactors may contain as many as 10 million fuel pellets. The amount of energy in each of these pellets is equal to that in almost a ton of
coal or 150 gallons of oil.

21.4.2.3: Nuclear Moderators


Neutrons produced by nuclear reactions move too fast to cause fission (Figure 21.5.5). They must first be slowed to be absorbed by the fuel and produce additional nuclear reactions. A nuclear
moderator is a substance that slows the neutrons to a speed that is low enough to cause fission. Early reactors used high-purity graphite as a moderator. Modern reactors in the US exclusively use
2
heavy water ( H O) or light water (ordinary H2O), whereas some reactors in other countries use other materials, such as carbon dioxide, beryllium, or graphite.
1 2

21.4.2.4: Reactor Coolants


A nuclear reactor coolant is used to carry the heat produced by the fission reaction to an external boiler and turbine, where it is transformed into electricity. Two overlapping coolant loops are often
used; this counteracts the transfer of radioactivity from the reactor to the primary coolant loop. All nuclear power plants in the US use water as a coolant. Other coolants include molten sodium,
lead, a lead-bismuth mixture, or molten salts.

Access for free at OpenStax 21.4.4 https://chem.libretexts.org/@go/page/38342


21.4.2.5: Control Rods
Nuclear reactors use control rods (Figure 21.4.8) to control the fission rate of the nuclear fuel by adjusting the number of slow neutrons present to keep the rate of the chain reaction at a safe level.
Control rods are made of boron, cadmium, hafnium, or other elements that are able to absorb neutrons. Boron-10, for example, absorbs neutrons by a reaction that produces lithium-7 and alpha
particles:
10 1 7 4
B+ n ⟶ Li + He
5 0 3 2

When control rod assemblies are inserted into the fuel element in the reactor core, they absorb a larger fraction of the slow neutrons, thereby slowing the rate of the fission reaction and decreasing
the power produced. Conversely, if the control rods are removed, fewer neutrons are absorbed, and the fission rate and energy production increase. In an emergency, the chain reaction can be shut
down by fully inserting all of the control rods into the nuclear core between the fuel rods.

Figure 21.4.8 : The nuclear reactor core shown in (a) contains the fuel and control rod assembly shown in (b). (credit: modification of work by E. Generalic,
glossary.periodni.com/glossar...en=control+rod)
Two diagrams are shown and labeled “a” and “b.” Diagram a shows a cut-away view of a vertical tube with a flat, horizontal plate near the bottom that connects to a series of vertical pipes lined up
next to one another and labeled “Fuel rods.” A second horizontal plate labeled “Grid” lies at the top of the pipes and a second set of thinner, vertical pipes, labeled “Control rods,” leads from this
plate to the top of the container. The walls of the container are labeled “Steel pressure vessel.” A blue, right-facing arrow leads from an entry point in the left side of the container and is followed by
a second, down-facing blue arrow and a curved, right-facing arrow that trace along the outer, bottom edge of the container. A blue and red arrow follows these and faces up the right side of the
container to an exit near the right face where a red, right-facing arrow leads out. Diagram b is a cut-away image of a vertical, rectangular, three dimensional set of vertical pipes. The pipes are
labeled “Fuel rods” and are inserted into an upper and lower horizontal plate labeled “Grid.” Four thin rods extend above the pipes and are labeled “Control rods.

21.4.2.6: Shield and Containment System


During its operation, a nuclear reactor produces neutrons and other radiation. Even when shut down, the decay products are radioactive. In addition, an operating reactor is thermally very hot, and
high pressures result from the circulation of water or another coolant through it. Thus, a reactor must withstand high temperatures and pressures, and must protect operating personnel from the
radiation. Reactors are equipped with a containment system (or shield) that consists of three parts:
1. The reactor vessel, a steel shell that is 3–20-centimeters thick and, with the moderator, absorbs much of the radiation produced by the reactor
2. A main shield of 1–3 meters of high-density concrete
3. A personnel shield of lighter materials that protects operators from γ rays and X-rays
In addition, reactors are often covered with a steel or concrete dome that is designed to contain any radioactive materials might be released by a reactor accident.

The Design and Safe Operation of a Nuclear Reactor

Video 21.4.1 : Click here to watch a 3-minute video from the Nuclear Energy Institute on how nuclear reactors work.
Nuclear power plants are designed in such a way that they cannot form a supercritical mass of fissionable material and therefore cannot create a nuclear explosion. But as history has shown, failures
of systems and safeguards can cause catastrophic accidents, including chemical explosions and nuclear meltdowns (damage to the reactor core from overheating). The following Chemistry in
Everyday Life feature explores three infamous meltdown incidents.

 Nuclear Accidents
The importance of cooling and containment are amply illustrated by three major accidents that occurred with the nuclear reactors at nuclear power generating stations in the United States
(Three Mile Island), the former Soviet Union (Chernobyl), and Japan (Fukushima).

Access for free at OpenStax 21.4.5 https://chem.libretexts.org/@go/page/38342


In March 1979, the cooling system of the Unit 2 reactor at Three Mile Island Nuclear Generating Station in Pennsylvania failed, and the cooling water spilled from the reactor onto the floor of
the containment building. After the pumps stopped, the reactors overheated due to the high radioactive decay heat produced in the first few days after the nuclear reactor shut down. The
temperature of the core climbed to at least 2200 °C, and the upper portion of the core began to melt. In addition, the zirconium alloy cladding of the fuel rods began to react with steam and
produced hydrogen:

Zr(s) + 2 H O(g) ⟶ ZrO (s) + 2 H (g)


2 2 2

The hydrogen accumulated in the confinement building, and it was feared that there was danger of an explosion of the mixture of hydrogen and air in the building. Consequently, hydrogen gas
and radioactive gases (primarily krypton and xenon) were vented from the building. Within a week, cooling water circulation was restored and the core began to cool. The plant was closed for
nearly 10 years during the cleanup process.
Although zero discharge of radioactive material is desirable, the discharge of radioactive krypton and xenon, such as occurred at the Three Mile Island plant, is among the most tolerable. These
gases readily disperse in the atmosphere and thus do not produce highly radioactive areas. Moreover, they are noble gases and are not incorporated into plant and animal matter in the food
chain. Effectively none of the heavy elements of the core of the reactor were released into the environment, and no cleanup of the area outside of the containment building was necessary
(Figure 21.4.9).

Figure 21.4.9 : (a) In this 2010 photo of Three Mile Island, the remaining structures from the damaged Unit 2 reactor are seen on the left, whereas the separate Unit 1 reactor, unaffected by the
accident, continues generating power to this day (right). (b) President Jimmy Carter visited the Unit 2 control room a few days after the accident in 1979.
Two photos, labeled “a” and “b” are shown. Photo a is an aerial view of a nuclear power plant. Photo b shows a small group of men walking through a room filled with electronics.
Another major nuclear accident involving a reactor occurred in April 1986, at the Chernobyl Nuclear Power Plant in Ukraine, which was still a part of the former Soviet Union. While operating
at low power during an unauthorized experiment with some of its safety devices shut off, one of the reactors at the plant became unstable. Its chain reaction became uncontrollable and increased
to a level far beyond what the reactor was designed for. The steam pressure in the reactor rose to between 100 and 500 times the full power pressure and ruptured the reactor. Because the reactor
was not enclosed in a containment building, a large amount of radioactive material spewed out, and additional fission products were released, as the graphite (carbon) moderator of the core
ignited and burned. The fire was controlled, but over 200 plant workers and firefighters developed acute radiation sickness and at least 32 soon died from the effects of the radiation. It is
predicted that about 4000 more deaths will occur among emergency workers and former Chernobyl residents from radiation-induced cancer and leukemia. The reactor has since been
encapsulated in steel and concrete, a now-decaying structure known as the sarcophagus. Almost 30 years later, significant radiation problems still persist in the area, and Chernobyl largely
remains a wasteland.
In 2011, the Fukushima Daiichi Nuclear Power Plant in Japan was badly damaged by a 9.0-magnitude earthquake and resulting tsunami. Three reactors up and running at the time were shut
down automatically, and emergency generators came online to power electronics and coolant systems. However, the tsunami quickly flooded the emergency generators and cut power to the
pumps that circulated coolant water through the reactors. High-temperature steam in the reactors reacted with zirconium alloy to produce hydrogen gas. The gas escaped into the containment
building, and the mixture of hydrogen and air exploded. Radioactive material was released from the containment vessels as the result of deliberate venting to reduce the hydrogen pressure,
deliberate discharge of coolant water into the sea, and accidental or uncontrolled events.
An evacuation zone around the damaged plant extended over 12.4 miles away, and an estimated 200,000 people were evacuated from the area. All 48 of Japan’s nuclear power plants were
subsequently shut down, remaining shuttered as of December 2014. Since the disaster, public opinion has shifted from largely favoring to largely opposing increasing the use of nuclear power
plants, and a restart of Japan’s atomic energy program is still stalled (Figure 21.4.10).

Figure 21.4.10: (a) After the accident, contaminated waste had to be removed, and (b) an evacuation zone was set up around the plant in areas that received heavy doses of radioactive fallout.
(credit a: modification of work by “Live Action Hero”/Flickr)
A photo and a map, labeled “a” and “b,” respectively, are shown. Photo a shows a man in a body-covering safety suit working near a series of blue, plastic coated containers. Map b shows a
section of land with the ocean on each side. Near the upper right side of the land is a small red dot, labeled “greater than, 12.5, m R backslash, h r,” that is surrounded by a zone of orange that
extends in the upper left direction labeled “2.17, dash, 12.5, m R backslash, h r.” The orange is surrounded by an outline of yellow labeled “1.19, dash, 2.17, m R backslash, h r” and a wider
outline of green labeled “0.25, dash, 1.19, m R backslash, h r.” A large area of light blue, labeled “0.03, dash, 0.25, m R backslash, h r” surrounds the green area and extends to the lower middle
of the map. A large section of the lower middle and left of the land is covered by dark blue, labeled “less than 0.03, m R backslash, h r.”

The energy produced by a reactor fueled with enriched uranium results from the fission of uranium as well as from the fission of plutonium produced as the reactor operates. As discussed
previously, the plutonium forms from the combination of neutrons and the uranium in the fuel. In any nuclear reactor, only about 0.1% of the mass of the fuel is converted into energy. The other
99.9% remains in the fuel rods as fission products and unused fuel. All of the fission products absorb neutrons, and after a period of several months to a few years, depending on the reactor, the
fission products must be removed by changing the fuel rods. Otherwise, the concentration of these fission products would increase and absorb more neutrons until the reactor could no longer
operate.
Spent fuel rods contain a variety of products, consisting of unstable nuclei ranging in atomic number from 25 to 60, some transuranium elements, including plutonium and americium, and unreacted
uranium isotopes. The unstable nuclei and the transuranium isotopes give the spent fuel a dangerously high level of radioactivity. The long-lived isotopes require thousands of years to decay to a
safe level. The ultimate fate of the nuclear reactor as a significant source of energy in the United States probably rests on whether or not a politically and scientifically satisfactory technique for
processing and storing the components of spent fuel rods can be developed.

Access for free at OpenStax 21.4.6 https://chem.libretexts.org/@go/page/38342


21.4.3: Nuclear Fusion and Fusion Reactors
The process of converting very light nuclei into heavier nuclei is also accompanied by the conversion of mass into large amounts of energy, a process called fusion. The principal source of energy in
the sun is a net fusion reaction in which four hydrogen nuclei fuse and produce one helium nucleus and two positrons. This is a net reaction of a more complicated series of events:
1 4 0
4 H ⟶ 2
He + 2
1 +1

A helium nucleus has a mass that is 0.7% less than that of four hydrogen nuclei; this lost mass is converted into energy during the fusion. This reaction produces about 3.6 × 1011 kJ of energy per
mole of He produced. This is somewhat larger than the energy produced by the nuclear fission of one mole of U-235 (1.8 × 1010 kJ), and over 3 million times larger than the energy produced by
4
2

the (chemical) combustion of one mole of octane (5471 kJ).


It has been determined that the nuclei of the heavy isotopes of hydrogen, a deuteron, 2
1
and a triton, , undergo fusion at extremely high temperatures (thermonuclear fusion). They form a helium
3
1

nucleus and a neutron:


2 3 4 1
1
H+ H ⟶ 2
He + 2 n
1 0

9
This change proceeds with a mass loss of 0.0188 amu, corresponding to the release of 1.69 × 10 kilojoules per mole of He formed. The very high temperature is necessary to give the nuclei
4
2

enough kinetic energy to overcome the very strong repulsive forces resulting from the positive charges on their nuclei so they can collide.
Useful fusion reactions require very high temperatures for their initiation—about 15,000,000 K or more. At these temperatures, all molecules dissociate into atoms, and the atoms ionize, forming
plasma. These conditions occur in an extremely large number of locations throughout the universe—stars are powered by fusion. Humans have already figured out how to create temperatures high
enough to achieve fusion on a large scale in thermonuclear weapons. A thermonuclear weapon such as a hydrogen bomb contains a nuclear fission bomb that, when exploded, gives off enough
energy to produce the extremely high temperatures necessary for fusion to occur.
Another much more beneficial way to create fusion reactions is in a fusion reactor, a nuclear reactor in which fusion reactions of light nuclei are controlled. Because no solid materials are stable at
such high temperatures, mechanical devices cannot contain the plasma in which fusion reactions occur. Two techniques to contain plasma at the density and temperature necessary for a fusion
reaction are currently the focus of intensive research efforts: containment by a magnetic field and by the use of focused laser beams (Figure 21.4.11). A number of large projects are working to
attain one of the biggest goals in science: getting hydrogen fuel to ignite and produce more energy than the amount supplied to achieve the extremely high temperatures and pressures that are
required for fusion. At the time of this writing, there are no self-sustaining fusion reactors operating in the world, although small-scale controlled fusion reactions have been run for very brief
periods.

Figure 21.4.11: (a) This model is of the International Thermonuclear Experimental Reactor (ITER) reactor. Currently under construction in the south of France with an expected completion date of
2027, the ITER will be the world’s largest experimental Tokamak nuclear fusion reactor with a goal of achieving large-scale sustained energy production. (b) In 2012, the National Ignition Facility
at Lawrence Livermore National Laboratory briefly produced over 500,000,000,000 watts (500 terawatts, or 500 TW) of peak power and delivered 1,850,000 joules (1.85 MJ) of energy, the largest
laser energy ever produced and 1000 times the power usage of the entire United States in any given moment. Although lasting only a few billionths of a second, the 192 lasers attained the
conditions needed for nuclear fusion ignition. This image shows the target prior to the laser shot. (credit a: modification of work by Stephan Mosel)
Two photos are shown and labeled “a” and “b.” Photo a shows a model of the ITER reactor made up of colorful components. Photo b shows a close-up view of the end of a long, mechanical arm
made up of many metal components.

Summary
It is possible to produce new atoms by bombarding other atoms with nuclei or high-speed particles. The products of these transmutation reactions can be stable or radioactive. A number of artificial
elements, including technetium, astatine, and the transuranium elements, have been produced in this way.
Nuclear power as well as nuclear weapon detonations can be generated through fission (reactions in which a heavy nucleus is split into two or more lighter nuclei and several neutrons). Because the
neutrons may induce additional fission reactions when they combine with other heavy nuclei, a chain reaction can result. Useful power is obtained if the fission process is carried out in a nuclear
reactor. The conversion of light nuclei into heavier nuclei (fusion) also produces energy. At present, this energy has not been contained adequately and is too expensive to be feasible for commercial
energy production.

Glossary
chain reaction
repeated fission caused when the neutrons released in fission bombard other atoms

containment system
(also, shield) a three-part structure of materials that protects the exterior of a nuclear fission reactor and operating personnel from the high temperatures, pressures, and radiation levels inside the
reactor

control rod
material inserted into the fuel assembly that absorbs neutrons and can be raised or lowered to adjust the rate of a fission reaction

critical mass
amount of fissionable material that will support a self-sustaining (nuclear fission) chain reaction

fissile (or fissionable)


when a material is capable of sustaining a nuclear fission reaction

fission
splitting of a heavier nucleus into two or more lighter nuclei, usually accompanied by the conversion of mass into large amounts of energy

fusion
combination of very light nuclei into heavier nuclei, accompanied by the conversion of mass into large amounts of energy

fusion reactor
nuclear reactor in which fusion reactions of light nuclei are controlled

Access for free at OpenStax 21.4.7 https://chem.libretexts.org/@go/page/38342


nuclear fuel
fissionable isotope present in sufficient quantities to provide a self-sustaining chain reaction in a nuclear reactor

nuclear moderator
substance that slows neutrons to a speed low enough to cause fission

nuclear reactor
environment that produces energy via nuclear fission in which the chain reaction is controlled and sustained without explosion

nuclear transmutation
conversion of one nuclide into another nuclide

particle accelerator
device that uses electric and magnetic fields to increase the kinetic energy of nuclei used in transmutation reactions

reactor coolant
assembly used to carry the heat produced by fission in a reactor to an external boiler and turbine where it is transformed into electricity

subcritical mass
amount of fissionable material that cannot sustain a chain reaction; less than a critical mass

supercritical mass
amount of material in which there is an increasing rate of fission

transmutation reaction
bombardment of one type of nuclei with other nuclei or neutrons

transuranium element
element with an atomic number greater than 92; these elements do not occur in nature

This page titled 21.4: Transmutation and Nuclear Energy is shared under a CC BY 4.0 license and was authored, remixed, and/or curated by OpenStax via source content that was edited to the style and standards of the
LibreTexts platform; a detailed edit history is available upon request.

Access for free at OpenStax 21.4.8 https://chem.libretexts.org/@go/page/38342


Access for free at OpenStax 21.4.9 https://chem.libretexts.org/@go/page/38342
Access for free at OpenStax 21.4.10 https://chem.libretexts.org/@go/page/38342
Access for free at OpenStax 21.4.11 https://chem.libretexts.org/@go/page/38342
Access for free at OpenStax 21.4.12 https://chem.libretexts.org/@go/page/38342
Access for free at OpenStax 21.4.13 https://chem.libretexts.org/@go/page/38342
21.5: Uses of Radioisotopes
 Learning Objectives

List common applications of radioactive isotopes

Radioactive isotopes have the same chemical properties as stable isotopes of the same element, but they emit radiation, which can
be detected. If we replace one (or more) atom(s) with radioisotope(s) in a compound, we can track them by monitoring their
radioactive emissions. This type of compound is called a radioactive tracer (or radioactive label). Radioisotopes are used to follow
the paths of biochemical reactions or to determine how a substance is distributed within an organism. Radioactive tracers are also
used in many medical applications, including both diagnosis and treatment. They are used to measure engine wear, analyze the
geological formation around oil wells, and much more.
Radioisotopes have revolutionized medical practice, where they are used extensively. Over 10 million nuclear medicine procedures
and more than 100 million nuclear medicine tests are performed annually in the United States. Four typical examples of radioactive
tracers used in medicine are technetium-99 ( Tc), thallium-201 ( Tl), iodine-131 ( I), and sodium-24 ( Na). Damaged
99
43
201
81
131
53
24
11

tissues in the heart, liver, and lungs absorb certain compounds of technetium-99 preferentially. After it is injected, the location of
the technetium compound, and hence the damaged tissue, can be determined by detecting the γ rays emitted by the Tc-99 isotope.
Thallium-201 (Figure 21.5.1) becomes concentrated in healthy heart tissue, so the two isotopes, Tc-99 and Tl-201, are used
together to study heart tissue. Iodine-131 concentrates in the thyroid gland, the liver, and some parts of the brain. It can therefore be
used to monitor goiter and treat thyroid conditions, such as Grave’s disease, as well as liver and brain tumors. Salt solutions
containing compounds of sodium-24 are injected into the bloodstream to help locate obstructions to the flow of blood.

Figure 21.5.1 : Administering thallium-201 to a patient and subsequently performing a stress test offer medical professionals an
opportunity to visually analyze heart function and blood flow. (credit: modification of work by “Blue0ctane”/Wikimedia
Commons)
A medical professional is assisting a topless elderly man on a treadmill with sensors and electrical wires connected to his torso.
Radioisotopes used in medicine typically have short half-lives—for example, the ubiquitous Tc-99m has a half-life of 6.01 hours.
This makes Tc-99m essentially impossible to store and prohibitively expensive to transport, so it is made on-site instead. Hospitals
and other medical facilities use Mo-99 (which is primarily extracted from U-235 fission products) to generate Tc-99. Mo-99
undergoes β decay with a half-life of 66 hours, and the Tc-99 is then chemically extracted (Figure 21.5.2). The parent nuclide Mo-
99 is part of a molybdate ion, MoO ; when it decays, it forms the pertechnetate ion, TcO . These two water-soluble ions are
2−
4

4

separated by column chromatography, with the higher charge molybdate ion adsorbing onto the alumina in the column, and the
lower charge pertechnetate ion passing through the column in the solution. A few micrograms of Mo-99 can produce enough Tc-99
to perform as many as 10,000 tests.

Access for free at OpenStax 21.5.1 https://chem.libretexts.org/@go/page/38343


Figure 21.5.2 : (a) The first Tc-99m generator (circa 1958) is used to separate Tc-99 from Mo-99. The MoO is retained by the
2−
4

matrix in the column, whereas the TcO passes through and is collected. (b) Tc-99 was used in this scan of the neck of a patient

4

with Grave’s disease. The scan shows the location of high concentrations of Tc-99. (credit a: modification of work by the
Department of Energy; credit b: modification of work by “MBq”/Wikimedia Commons)
The first image shows a hand pouring a liquid from a measuring cylinder into a column held up by a clamp. Below the column is a
glass tube. The second picture shows red dots on a dark background dispersed everywhere with four spots of heavily concentrated
regions.
Radioisotopes can also be used, typically in higher doses than as a tracer, as treatment. Radiation therapy is the use of high-energy
radiation to damage the DNA of cancer cells, which kills them or keeps them from dividing (Figure 21.5.3). A cancer patient may
receive external beam radiation therapy delivered by a machine outside the body, or internal radiation therapy (brachytherapy) from
a radioactive substance that has been introduced into the body. Note that chemotherapy is similar to internal radiation therapy in
that the cancer treatment is injected into the body, but differs in that chemotherapy uses chemical rather than radioactive substances
to kill the cancer cells.

Figure 21.5.3 : The cartoon in (a) shows a cobalt-60 machine used in the treatment of cancer. The diagram in (b) shows how the
gantry of the Co-60 machine swings through an arc, focusing radiation on the targeted region (tumor) and minimizing the amount
of radiation that passes through nearby regions.
A. A woman lies down as she goes into a dome shaped medical machine. B. A closer view of the women's head shows gamma rays
from radioactive cobalt attacks the target on the woman's head.
Cobalt-60 is a synthetic radioisotope produced by the neutron activation of Co-59, which then undergoes β decay to form Ni-60,
along with the emission of γ radiation. The overall process is:
59 1 60 60 0 0
Co + n ⟶ Co ⟶ Ni + β+2 γ
27 0 27 28 −1 0

The overall decay scheme for this is shown graphically in Figure 21.5.4.

Access for free at OpenStax 21.5.2 https://chem.libretexts.org/@go/page/38343


Figure 21.5.4 : Co-60 undergoes a series of radioactive decays. The γ emissions are used for radiation therapy.
Radioisotopes are used in diverse ways to study the mechanisms of chemical reactions in plants and animals. These include
labeling fertilizers in studies of nutrient uptake by plants and crop growth, investigations of digestive and milk-producing processes
in cows, and studies on the growth and metabolism of animals and plants.
For example, the radioisotope C-14 was used to elucidate the details of how photosynthesis occurs. The overall reaction is:

6 CO (g) + 6 H O(l) ⟶ C H O (s) + 6 O (g),


2 2 6 12 6 2

but the process is much more complex, proceeding through a series of steps in which various organic compounds are produced. In
studies of the pathway of this reaction, plants were exposed to CO2 containing a high concentration of C. At regular intervals, the
14
6

plants were analyzed to determine which organic compounds contained carbon-14 and how much of each compound was present.
From the time sequence in which the compounds appeared and the amount of each present at given time intervals, scientists learned
more about the pathway of the reaction.
Commercial applications of radioactive materials are equally diverse (Figure 21.5.5). They include determining the thickness of
films and thin metal sheets by exploiting the penetration power of various types of radiation. Flaws in metals used for structural
purposes can be detected using high-energy gamma rays from cobalt-60 in a fashion similar to the way X-rays are used to examine
the human body. In one form of pest control, flies are controlled by sterilizing male flies with γ radiation so that females breeding
with them do not produce offspring. Many foods are preserved by radiation that kills microorganisms that cause the foods to spoil.

Figure 21.5.5 : Common commercial uses of radiation include (a) X-ray examination of luggage at an airport and (b) preservation
of food. (credit a: modification of work by the Department of the Navy; credit b: modification of work by the US Department of
Agriculture)
A. A man is observing a monitor which shows the X-ray of luggages. B. Many apples on processing belts.
Americium-241, an α emitter with a half-life of 458 years, is used in tiny amounts in ionization-type smoke detectors (Figure
21.5.6). The α emissions from Am-241 ionize the air between two electrode plates in the ionizing chamber. A battery supplies a

potential that causes movement of the ions, thus creating a small electric current. When smoke enters the chamber, the movement
of the ions is impeded, reducing the conductivity of the air. This causes a marked drop in the current, triggering an alarm.

Access for free at OpenStax 21.5.3 https://chem.libretexts.org/@go/page/38343


Figure 21.5.6 : Inside a smoke detector, Am-241 emits α particles that ionize the air, creating a small electric current. During a fire,
smoke particles impede the flow of ions, reducing the current and triggering an alarm. (credit a: modification of work by
“Muffet”/Wikimedia Commons)
The inside of a smoke detector is shown with the alarm and ionization chamber labeled. In the picture beside it, a schematic shows
the mechanisms of a smoke detector. The two oppositely charged metals plates in the detector are shown along with the Americium
source on the bottom part emitting alpha particles. The schematic is divided into two parts, one to show the presence and the other
for the absence of smoke.

Summary
Compounds known as radioactive tracers can be used to follow reactions, track the distribution of a substance, diagnose and treat
medical conditions, and much more. Other radioactive substances are helpful for controlling pests, visualizing structures, providing
fire warnings, and for many other applications. Hundreds of millions of nuclear medicine tests and procedures, using a wide variety
of radioisotopes with relatively short half-lives, are performed every year in the US. Most of these radioisotopes have relatively
short half-lives; some are short enough that the radioisotope must be made on-site at medical facilities. Radiation therapy uses
high-energy radiation to kill cancer cells by damaging their DNA. The radiation used for this treatment may be delivered externally
or internally.

Glossary
chemotherapy
similar to internal radiation therapy, but chemical rather than radioactive substances are introduced into the body to kill cancer
cells

external beam radiation therapy


radiation delivered by a machine outside the body

internal radiation therapy


(also, brachytherapy) radiation from a radioactive substance introduced into the body to kill cancer cells

radiation therapy
use of high-energy radiation to damage the DNA of cancer cells, which kills them or keeps them from dividing

radioactive tracer
(also, radioactive label) radioisotope used to track or follow a substance by monitoring its radioactive emissions

This page titled 21.5: Uses of Radioisotopes is shared under a CC BY 4.0 license and was authored, remixed, and/or curated by OpenStax via
source content that was edited to the style and standards of the LibreTexts platform; a detailed edit history is available upon request.

Access for free at OpenStax 21.5.4 https://chem.libretexts.org/@go/page/38343


21.6: Biological Effects of Radiation
 Learning Objectives
Describe the biological impact of ionizing radiation.
Define units for measuring radiation exposure.
Explain the operation of common tools for detecting radioactivity.
List common sources of radiation exposure in the US.

The increased use of radioisotopes has led to increased concerns over the effects of these materials on biological systems (such as
humans). All radioactive nuclides emit high-energy particles or electromagnetic waves. When this radiation encounters living cells,
it can cause heating, break chemical bonds, or ionize molecules. The most serious biological damage results when these radioactive
emissions fragment or ionize molecules. For example, alpha and beta particles emitted from nuclear decay reactions possess much
higher energies than ordinary chemical bond energies. When these particles strike and penetrate matter, they produce ions and
molecular fragments that are extremely reactive. The damage this does to biomolecules in living organisms can cause serious
malfunctions in normal cell processes, taxing the organism’s repair mechanisms and possibly causing illness or even death (Figure
21.6.1).

Figure 21.6.1 : Radiation can harm biological systems by damaging the DNA of cells. If this damage is not properly repaired, the
cells may divide in an uncontrolled manner and cause cancer.
A diagram is shown which has a white sphere followed by a right-facing arrow and a large sphere composed of many smaller white
and green spheres. The single sphere has impacted the larger sphere. A right-facing arrow leads from the larger sphere to a pair of
smaller spheres which are collections of the same white and green spheres. A starburst pattern lies between these two spheres and
has three right-facing arrows leading from it to two white spheres and a circle full of ten smaller, peach-colored circles with purple
dots in their centers. An arrow leads downward from this circle to a box that contains a helical shape with a starburst near its top
left side and is labeled “D N A damage.” A right-facing arrow leads from this circle to a second circle, with nine smaller, peach-
colored circles with purple dots in their centers and one fully purple small circle labeled “Cancer cell.” A right-facing arrow leads
to a final circle, this time full of the purple cells, that is labeled “Tumor.”

21.6.1: Ionizing vs. Nonionizing Radiation


There is a large difference in the magnitude of the biological effects of nonionizing radiation (for example, light and microwaves)
and ionizing radiation, emissions energetic enough to knock electrons out of molecules (for example, α and β particles, γ rays, X-
rays, and high-energy ultraviolet radiation) (Figure 21.6.2).
<div data-mt-source="1"

Figure 21.6.2 : Lower frequency, lower-energy electromagnetic radiation is nonionizing, and higher frequency, higher-energy
electromagnetic radiation is ionizing.
Energy absorbed from nonionizing radiation speeds up the movement of atoms and molecules, which is equivalent to heating the
sample. Although biological systems are sensitive to heat (as we might know from touching a hot stove or spending a day at the
beach in the sun), a large amount of nonionizing radiation is necessary before dangerous levels are reached. Ionizing radiation,
however, may cause much more severe damage by breaking bonds or removing electrons in biological molecules, disrupting their
structure and function. The damage can also be done indirectly, by first ionizing H2O (the most abundant molecule in living
organisms), which forms a H2O+ ion that reacts with water, forming a hydronium ion and a hydroxyl radical:

Figure 21.6.3 .

Access for free at OpenStax 21.6.1 https://chem.libretexts.org/@go/page/38344


Figure 21.6.3 : Ionizing radiation can (a) directly damage a biomolecule by ionizing it or breaking its bonds, or (b) create an H2O+
ion, which reacts with H2O to form a hydroxyl radical, which in turn reacts with the biomolecule, causing damage indirectly.

21.6.2: Biological Effects of Exposure to Radiation


Radiation can harm either the whole body (somatic damage) or eggs and sperm (genetic damage). Its effects are more pronounced
in cells that reproduce rapidly, such as the stomach lining, hair follicles, bone marrow, and embryos. This is why patients
undergoing radiation therapy often feel nauseous or sick to their stomach, lose hair, have bone aches, and so on, and why particular
care must be taken when undergoing radiation therapy during pregnancy.
Different types of radiation have differing abilities to pass through material (Figure 21.6.4). A very thin barrier, such as a sheet or
two of paper, or the top layer of skin cells, usually stops alpha particles. Because of this, alpha particle sources are usually not
dangerous if outside the body, but are quite hazardous if ingested or inhaled (see the Chemistry in Everyday Life feature on Radon
Exposure). Beta particles will pass through a hand, or a thin layer of material like paper or wood, but are stopped by a thin layer of
metal. Gamma radiation is very penetrating and can pass through a thick layer of most materials. Some high-energy gamma
radiation is able to pass through a few feet of concrete. Certain dense, high atomic number elements (such as lead) can effectively
attenuate gamma radiation with thinner material and are used for shielding. The ability of various kinds of emissions to cause
ionization varies greatly, and some particles have almost no tendency to produce ionization. Alpha particles have about twice the
ionizing power of fast-moving neutrons, about 10 times that of β particles, and about 20 times that of γ rays and X-rays.

Figure 21.6.4 : The ability of different types of radiation to pass through material is shown. From least to most penetrating, they are
alpha < beta < neutron < gamma.
A diagram shows four particles in a vertical column on the left, followed by an upright sheet of paper, a person’s hand, an upright
sheet of metal, a glass of water, a thick block of concrete and an upright, thick piece of lead. The top particle listed is made up of
two white spheres and two green spheres that are labeled with positive signs and is labeled “Alpha.” A right-facing arrow leads
from this to the paper. The second particle is a red sphere labeled “Beta” and is followed by a right-facing arrow that passes
through the paper and stops at the hand. The third particle is a white sphere labeled “Neutron” and is followed by a right-facing
arrow that passes through the paper, hand and metal but is stopped at the glass of water. The fourth particle is shown by a squiggly
arrow and it passes through all of the substances but stops at the lead. Terms at the bottom read, from left to right, “Paper,”
“Metal,” “Water,” “Concrete” and “Lead.”
For many people, one of the largest sources of exposure to radiation is from radon gas (Rn-222). Radon-222 is an α emitter with a
half–life of 3.82 days. It is one of the products of the radioactive decay series of U-238, which is found in trace amounts in soil and
rocks. The radon gas that is produced slowly escapes from the ground and gradually seeps into homes and other structures above.

Access for free at OpenStax 21.6.2 https://chem.libretexts.org/@go/page/38344


Since it is about eight times more dense than air, radon gas accumulates in basements and lower floors, and slowly diffuses
throughout buildings (Figure 21.6.5).

Figure 21.6.5 : Radon-222 seeps into houses and other buildings from rocks that contain uranium-238, a radon emitter. The radon
enters through cracks in concrete foundations and basement floors, stone or porous cinderblock foundations, and openings for
water and gas pipes.
A cut-away image of the side of a house and four layers of the ground it rests on is shown, as well as a second cut-away image of a
person’s head and chest cavity. The house is shown with a restroom on the second floor and a basement with a water heater as the
first floor. Green arrows lead from the lowest ground layer, labeled “radon in ground water,” from the third ground layer, labeled
“Bedrock” and “Fractured bedrock,” from the second layer, labeled “radon in well water,” and from the top layer, labeled “radon in
soil to the inside of the basement area. In the smaller image of the torso, a green arrow is shown to enter the person’s nasal passage
and travel to the lungs. This is labeled “Inhalation of radon decay products.” A small coiled, helical structure next to the torso is
labeled “alpha particle” on one section where it has a starburst pattern and “Radiation damage to D N A” on another segment.
Radon is found in buildings across the country, with amounts dependent on location. The average concentration of radon inside
houses in the US (1.25 pCi/L) is about three times the level found in outside air, and about one in six houses have radon levels high
enough that remediation efforts to reduce the radon concentration are recommended. Exposure to radon increases one’s risk of
getting cancer (especially lung cancer), and high radon levels can be as bad for health as smoking a carton of cigarettes a day.
Radon is the number one cause of lung cancer in nonsmokers and the second leading cause of lung cancer overall. Radon exposure
is believed to cause over 20,000 deaths in the US per year.

21.6.3: Measuring Radiation Exposure


Several different devices are used to detect and measure radiation, including Geiger counters, scintillation counters (scintillators),
and radiation dosimeters (Figure 21.6.6). Probably the best-known radiation instrument, the Geiger counter (also called the Geiger-
Müller counter) detects and measures radiation. Radiation causes the ionization of the gas in a Geiger-Müller tube. The rate of
ionization is proportional to the amount of radiation. A scintillation counter contains a scintillator—a material that emits light
(luminesces) when excited by ionizing radiation—and a sensor that converts the light into an electric signal. Radiation dosimeters
also measure ionizing radiation and are often used to determine personal radiation exposure. Commonly used types are electronic,
film badge, thermoluminescent, and quartz fiber dosimeters.

Access for free at OpenStax 21.6.3 https://chem.libretexts.org/@go/page/38344


Figure 21.6.6 : Devices such as (a) Geiger counters, (b) scintillators, and (c) dosimeters can be used to measure radiation. (Credit c:
modification of work by “osaMu”/Wikimedia commons.)
Three photographs are shown and labeled “a,” “b” and “c.” Photo a shows a Geiger counter sitting on a table. It is made up of a
metal box with a read-out screen and a wire leading away from the box connected to a sensor wand. Photograph b shows a
collection of tall and short vertical tubes arranged in a grouping while photograph c shows a person’s hand holding a small machine
with a digital readout while standing on the edge of a roadway.
A variety of units are used to measure various aspects of radiation (Table 21.6.1). The SI unit for rate of radioactive decay is the
becquerel (Bq), with 1 Bq = 1 disintegration per second. The curie (Ci) and millicurie (mCi) are much larger units and are
frequently used in medicine (1 curie = 1 Ci = 3.7 × 10 disintegrations per second). The SI unit for measuring radiation dose is
10

the gray (Gy), with 1 Gy = 1 J of energy absorbed per kilogram of tissue. In medical applications, the radiation absorbed dose (rad)
is more often used (1 rad = 0.01 Gy; 1 rad results in the absorption of 0.01 J/kg of tissue). The SI unit measuring tissue damage
caused by radiation is the sievert (Sv). This takes into account both the energy and the biological effects of the type of radiation
involved in the radiation dose.
Table 21.6.1 : Units Used for Measuring Radiation
Measurement Purpose Unit Quantity Measured Description

amount of sample that undergoes 1


becquerel (Bq)
decay/second
activity of source radioactive decays or emissions
amount of sample that undergoes
curie (Ci) 10
3.7 × 10 decays/second

gray (Gy) 1 Gy = 1 J/kg tissue


absorbed dose energy absorbed per kg of tissue
radiation absorbed dose (rad) 1 rad = 0.01 J/kg tissue

sievert (Sv) Sv = RBE × Gy


biologically effective dose tissue damage
roentgen equivalent for man (rem) Rem = RBE × rad

The roentgen equivalent for man (rem) is the unit for radiation damage that is used most frequently in medicine (1 rem = 1 Sv).
Note that the tissue damage units (rem or Sv) includes the energy of the radiation dose (rad or Gy), along with a biological factor
referred to as the RBE (for relative biological effectiveness), that is an approximate measure of the relative damage done by the
radiation. These are related by:

number of rems = RBE × number of rads (21.6.1)

with RBE approximately 10 for α radiation, 2(+) for protons and neutrons, and 1 for β and γ radiation.

Access for free at OpenStax 21.6.4 https://chem.libretexts.org/@go/page/38344


Figure 21.6.7 : Different units are used to measure the rate of emission from a radioactive source, the energy that is absorbed from
the source, and the amount of damage the absorbed radiation does. (CC by 4.0; OpenStax)
Two images are shown. The first, labeled “Rate of radioactive decay measured in becquerels or curies,” shows a red sphere with ten
red squiggly arrows facing away from it in a 360 degree circle. The second image shows the head and torso of a woman wearing
medical scrubs with a badge on her chest. The caption to the badge reads “Film badge or dosimeter measures tissue damage
exposure in rems or sieverts” while a phrase under this image states “Absorbed dose measured in grays or rads.”

 Example 21.6.1: Amount of Radiation

Cobalt-60 (t1/2 = 5.26 y) is used in cancer therapy since the γ rays it emits can be focused in small areas where the cancer is
located. A 5.00-g sample of Co-60 is available for cancer treatment.
a. What is its activity in Bq?
b. What is its activity in Ci?

Solution
The activity is given by:
ln 2 ln 2 g 60
Activity = λN = ( )N =( ) × 5.00 g = 0.659   of   Co that decay
t1/2 5.26 y y

And to convert this to decays per second:


23
g y 1 day 1 h 1 mol 6.02 × 10 atoms 1 decay
0.659 × × × × × ×
y 365 day 24 hours 3, 600 s 59.9 g 1 mol 1 atom

decay
14
= 2.10 × 10
s

decay
(a) Since 1 Bq = 1
s
, the activity in Becquerel (Bq) is:

decay 1 Bq
14 14
2.10 × 10 ×( ) = 2.10 × 10 Bq
decay
s
1
s

decay
(b) Since 1 Ci = 3.7 × 10 11

s
, the activity in curie (Ci) is:

decay 1 Ci
14 2
2.10 × 10 ×( ) = 5.7 × 10 Ci
s 11 decay
3.7 × 10
s

Access for free at OpenStax 21.6.5 https://chem.libretexts.org/@go/page/38344


 Exercise 21.6.1

Tritium is a radioactive isotope of hydrogen (t = 12.32 years) that has several uses, including self-powered lighting, in
1/2

which electrons emitted in tritium radioactive decay cause phosphorus to glow. Its nucleus contains one proton and two
neutrons, and the atomic mass of tritium is 3.016 amu. What is the activity of a sample containing 1.00mg of tritium (a) in Bq
and (b) in Ci?

Answer a
11
3.56 × 10 Bq

Answer b
0.962 Ci

21.6.4: Effects of Long-term Radiation Exposure on the Human Body


The effects of radiation depend on the type, energy, and location of the radiation source, and the length of exposure. As shown in
Figure 21.6.8, the average person is exposed to background radiation, including cosmic rays from the sun and radon from uranium
in the ground (see the Chemistry in Everyday Life feature on Radon Exposure); radiation from medical exposure, including CAT
scans, radioisotope tests, X-rays, and so on; and small amounts of radiation from other human activities, such as airplane flights
(which are bombarded by increased numbers of cosmic rays in the upper atmosphere), radioactivity from consumer products, and a
variety of radionuclides that enter our bodies when we breathe (for example, carbon-14) or through the food chain (for example,
potassium-40, strontium-90, and iodine-131).

Access for free at OpenStax 21.6.6 https://chem.libretexts.org/@go/page/38344


Figure 21.6.8 : The total annual radiation exposure for a person in the US is about 620 mrem. The various sources and their relative
amounts are shown in this bar graph. (source: U.S. Nuclear Regulatory Commission).
A bar graph titled “Radiation Doses and Regulatory Limits, open parenthesis, in Millirems, close parenthesis” is shown. The y-axis
is labeled “Doses in Millirems” and has values from 0 to 5000 with a break between 1000 and 5000 to indicate a different scale to
the top of the graph. The y-axis is labeled corresponding to each bar. The first bar, measured to 5000 on the y-axis, is drawn in red
and is labeled “Annual Nuclear Worker Doses Limit, open parenthesis, N R C, close parenthesis.” The second bar, measured to
1000 on the y-axis, is drawn in blue and is labeled “Whole Body C T” while the third bar, measured to 620 on the y-axis, is drawn
in blue and is labeled “Average U period S period Annual Dose.” The fourth bar, measured to 310 on the y-axis, is drawn in blue
and is labeled “U period S period Natural Background Dose” while the fifth bar, measured to 100 on the y-axis and drawn in red
reads “Annual Public Dose Limit, open parenthesis, N R C, close parenthesis.” The sixth bar, measured to 40 on the y-axis, is
drawn in blue and is labeled “From Your Body” while the seventh bar, measured to 30 on the y-axis and drawn in blue reads
“Cosmic rays.” The eighth bar, measured to 4 on the y-axis, is drawn in blue and is labeled “Safe Drinking Water Limit, open
parenthesis, E P A, close parenthesis” while the ninth bar, measured to 2.5 on the y-axis and drawn in red reads “Trans Atlantic
Flight.” A legend on the graph shows that red means “Dose Limit From N R C dash licensed activity” while blue means “Radiation
Doses.”
A short-term, sudden dose of a large amount of radiation can cause a wide range of health effects, from changes in blood chemistry
to death. Short-term exposure to tens of rems of radiation will likely cause very noticeable symptoms or illness; a dose of about 500
rems is estimated to have a 50% probability of causing the death of the victim within 30 days of exposure. Exposure to radioactive
emissions has a cumulative effect on the body during a person’s lifetime, which is another reason why it is important to avoid any
unnecessary exposure to radiation. Health effects of short-term exposure to radiation are shown in Table 21.6.2.
Table 21.6.2 : Health Effects of Radiation
Exposure (rem) Health Effect Time to Onset (Without Treatment)

5–10 changes in blood chemistry —

50 nausea hours

55 fatigue —

70 vomiting —

75 hair loss 2–3 weeks

90 diarrhea —

100 hemorrhage —

400 possible death within 2 months

1000 destruction of intestinal lining —

internal bleeding —

Access for free at OpenStax 21.6.7 https://chem.libretexts.org/@go/page/38344


Exposure (rem) Health Effect Time to Onset (Without Treatment)

death 1–2 weeks

2000 damage to central nervous system —

loss of consciousness minutes

death hours to days

It is impossible to avoid some exposure to ionizing radiation. We are constantly exposed to background radiation from a variety of
natural sources, including cosmic radiation, rocks, medical procedures, consumer products, and even our own atoms. We can
minimize our exposure by blocking or shielding the radiation, moving farther from the source, and limiting the time of exposure.

Summary
We are constantly exposed to radiation from a variety of naturally occurring and human-produced sources. This radiation can affect
living organisms. Ionizing radiation is the most harmful because it can ionize molecules or break chemical bonds, which damages
the molecule and causes malfunctions in cell processes. It can also create reactive hydroxyl radicals that damage biological
molecules and disrupt physiological processes. Radiation can cause somatic or genetic damage, and is most harmful to rapidly
reproducing cells. Types of radiation differ in their ability to penetrate material and damage tissue, with alpha particles the least
penetrating, but potentially most damaging, and gamma rays the most penetrating.
Various devices, including Geiger counters, scintillators, and dosimeters, are used to detect and measure radiation, and monitor
radiation exposure. We use several units to measure radiation: becquerels or curies for rates of radioactive decay; gray or rads for
energy absorbed; and rems or sieverts for biological effects of radiation. Exposure to radiation can cause a wide range of health
effects, from minor to severe, including death. We can minimize the effects of radiation by shielding with dense materials such as
lead, moving away from the source of radiation, and limiting time of exposure.

Footnotes
1. 1 Source: US Environmental Protection Agency

Glossary
becquerel (Bq)
SI unit for rate of radioactive decay; 1 Bq = 1 disintegration/s.

curie (Ci)
Larger unit for rate of radioactive decay frequently used in medicine; 1 Ci = 3.7 × 1010 disintegrations/s.

Geiger counter
Instrument that detects and measures radiation via the ionization produced in a Geiger-Müller tube.

gray (Gy)
SI unit for measuring radiation dose; 1 Gy = 1 J absorbed/kg tissue.

ionizing radiation
Radiation that can cause a molecule to lose an electron and form an ion.

millicurie (mCi)
Larger unit for rate of radioactive decay frequently used in medicine; 1 Ci = 3.7 × 1010 disintegrations/s.

nonionizing radiation
Radiation that speeds up the movement of atoms and molecules; it is equivalent to heating a sample, but is not energetic enough
to cause the ionization of molecules.

radiation absorbed dose (rad)


SI unit for measuring radiation dose, frequently used in medical applications; 1 rad = 0.01 Gy.

Access for free at OpenStax 21.6.8 https://chem.libretexts.org/@go/page/38344


radiation dosimeter
Device that measures ionizing radiation and is used to determine personal radiation exposure.

relative biological effectiveness (RBE)


Measure of the relative damage done by radiation.

roentgen equivalent man (rem)


Unit for radiation damage, frequently used in medicine; 1 rem = 1 Sv.

scintillation counter
Instrument that uses a scintillator—a material that emits light when excited by ionizing radiation—to detect and measure
radiation.

sievert (Sv)
SI unit measuring tissue damage caused by radiation; takes energy and biological effects of radiation into account.

This page titled 21.6: Biological Effects of Radiation is shared under a CC BY 4.0 license and was authored, remixed, and/or curated by
OpenStax via source content that was edited to the style and standards of the LibreTexts platform; a detailed edit history is available upon request.

Access for free at OpenStax 21.6.9 https://chem.libretexts.org/@go/page/38344


21.E: Nuclear Chemistry (Exercises)
21.E.1: 21.2: Nuclear Structure and Stability
21.E.1.1: Q21.2.1
Write the following isotopes in hyphenated form (e.g., “carbon-14”)
a. 24
11
Na

b. 29
13
Al

c. 73
36
Kr

d. 194
77
Ir

21.E.1.2: Q21.2.2
Write the following isotopes in nuclide notation (e.g., " 14
6
C ")
a. oxygen-14
b. copper-70
c. tantalum-175
d. francium-217

21.E.1.3: Q21.2.3
For the following isotopes that have missing information, fill in the missing information to complete the notation
a. 34
14
X

b. 36
P

c. 57
X
Mn

d. 121
56
X

21.E.1.4: Q21.2.4
For each of the isotopes in Question 21.2.3, determine the numbers of protons, neutrons, and electrons in a neutral atom of the
isotope.

21.E.1.5: Q21.2.5
Write the nuclide notation, including charge if applicable, for atoms with the following characteristics:
a. 25 protons, 20 neutrons, 24 electrons
b. 45 protons, 24 neutrons, 43 electrons
c. 53 protons, 89 neutrons, 54 electrons
d. 97 protons, 146 neutrons, 97 electrons

21.E.1.6: Q21.2.6
Calculate the density of the 24
12
Mg nucleus in g/mL, assuming that it has the typical nuclear diameter of 1 × 10–13 cm and is
spherical in shape.

21.E.1.7: Q21.2.7
What are the two principal differences between nuclear reactions and ordinary chemical changes?

21.E.1.8: Q21.2.8
The mass of the atom 23
11
Na is 22.9898 amu.
a. Calculate its binding energy per atom in millions of electron volts.
b. Calculate its binding energy per nucleon.

21.E.1.9: Q21.2.9
Which of the following nuclei lie within the band of stability?

Access for free at OpenStax 21.E.1 https://chem.libretexts.org/@go/page/44089


a. chlorine-37
b. calcium-40
c. 204Bi
d. 56Fe
e. 206Pb
f. 211Pb
g. 222Rn
h. carbon-14

21.E.1.10: Q21.2.10
Which of the following nuclei lie within the band of stability?
a. argon-40
b. oxygen-16
c. 122Ba
d. 58Ni
e. 205Tl
f. 210Tl
g. 226Ra
h. magnesium-24

21.E.2: 21.3: Nuclear Equations


21.E.2.1: Q21.3.1
Write a brief description or definition of each of the following:
a. nucleon
b. α particle
c. β particle
d. positron
e. γ ray
f. nuclide
g. mass number
h. atomic number

21.E.2.2: Q21.3.2
Which of the various particles (α particles, β particles, and so on) that may be produced in a nuclear reaction are actually nuclei?

21.E.2.3: Q21.3.3
Complete each of the following equations by adding the missing species:
a. 27

13
Al +
4
2
He ⟶ ? +
1
0
n

b. 239
94
Pu+ ? ⟶
242
96
Cm +
1
0
n

c. 14
7
4
N + 2 He ⟶ ? + 1 H
1

d. 235
92
U ⟶ ?+
135
55
Cs + 4
1
0
n

21.E.2.4: Q21.3.4
Complete each of the following equations:
a. 7
3
Li + ? ⟶ 2
4
2
He

b. 14
6
C ⟶
14
7
N+ ?

c. 27
13
4
Al + 2 He ⟶ ? +
1
0
n

d. 250
96
Cm ⟶ ? + 38 Sr + 4
98 1
0
n

Access for free at OpenStax 21.E.2 https://chem.libretexts.org/@go/page/44089


21.E.2.5: Q21.3.5
Write a balanced equation for each of the following nuclear reactions:
a. the production of 17O from 14N by α particle bombardment
b. the production of 14C from 14N by neutron bombardment
c. the production of 233Th from 232Th by neutron bombardment
d. the production of 239U from 238U by H bombardment
2
1

21.E.2.6: Q21.3.6
Technetium-99 is prepared from 98Mo. Molybdenum-98 combines with a neutron to give molybdenum-99, an unstable isotope that
emits a β particle to yield an excited form of technetium-99, represented as 99Tc*. This excited nucleus relaxes to the ground state,
represented as 99Tc, by emitting a γ ray. The ground state of 99Tc then emits a β particle. Write the equations for each of these
nuclear reactions.

21.E.2.7: Q21.3.7
The mass of the atom 19
9
F is 18.99840 amu.
a. Calculate its binding energy per atom in millions of electron volts.
b. Calculate its binding energy per nucleon.

21.E.2.8: Q21.3.8
For the reaction 14

6
C ⟶
14
7
N+ ? , if 100.0 g of carbon reacts, what volume of nitrogen gas (N2) is produced at 273 K and 1 atm?

21.E.3: 21.4: Radioactive Decay


21.E.3.1: Q21.4.1
What are the types of radiation emitted by the nuclei of radioactive elements?

21.E.3.2: Q21.4.2
What changes occur to the atomic number and mass of a nucleus during each of the following decay scenarios?
a. an α particle is emitted
b. a β particle is emitted
c. γ radiation is emitted
d. a positron is emitted
e. an electron is captured

21.E.3.3: Q21.4.3
What is the change in the nucleus that results from the following decay scenarios?
a. emission of a β particle
b. emission of a β+ particle
c. capture of an electron

21.E.3.4: Q21.4.4
Many nuclides with atomic numbers greater than 83 decay by processes such as electron emission. Explain the observation that the
emissions from these unstable nuclides also normally include α particles.

21.E.3.5: Q21.4.5
Why is electron capture accompanied by the emission of an X-ray?

21.E.3.6: Q21.4.6
Explain how unstable heavy nuclides (atomic number > 83) may decompose to form nuclides of greater stability (a) if they are
below the band of stability and (b) if they are above the band of stability.

Access for free at OpenStax 21.E.3 https://chem.libretexts.org/@go/page/44089


21.E.3.7: Q21.4.7
Which of the following nuclei is most likely to decay by positron emission? Explain your choice.
a. chromium-53
b. manganese-51
c. iron-59

21.E.3.8: Q21.4.8
The following nuclei do not lie in the band of stability. How would they be expected to decay? Explain your answer.
a. 34
15
P

b. 239
92
U

c. 38
20
Ca

d. 3
1
H

e. 245
94
Pu

21.E.3.9: Q21.4.9
The following nuclei do not lie in the band of stability. How would they be expected to decay?
a. 28
15
P

b. 235
92
U

c. 37
20 Ca

d. 9
3
Li

e. 245
96
Cm

21.E.3.10: Q21.4.10
Predict by what mode(s) of spontaneous radioactive decay each of the following unstable isotopes might proceed:
a. He6
2

b. Zn60
30

c. 235
Pa
91

d. Np241
94

e. 18F
f. 129Ba
g. 237Pu

21.E.3.11: Q21.4.11
Write a nuclear reaction for each step in the formation of Po from
218
84
238
92
U , which proceeds by a series of decay reactions involving
the step-wise emission of α, β, β, α, α, α, α particles, in that order.

21.E.3.12: Q21.4.12
Write a nuclear reaction for each step in the formation of Pb from
208
82
228
90
, which proceeds by a series of decay reactions
Th

involving the step-wise emission of α, α, α, α, β, β, α particles, in that order.

21.E.3.13: Q21.4.13
Define the term half-life and illustrate it with an example.

21.E.3.14: Q21.4.14
A 1.00 × 10–6-g sample of nobelium, 254
102
No , has a half-life of 55 seconds after it is formed. What is the percentage of 254
102
No

remaining at the following times?


a. 5.0 min after it forms
b. 1.0 h after it forms

Access for free at OpenStax 21.E.4 https://chem.libretexts.org/@go/page/44089


21.E.3.15: Q21.4.15
239Pu is a nuclear waste byproduct with a half-life of 24,000 y. What fraction of the 239Pu present today will be present in 1000 y?

21.E.3.16: Q21.4.16
The isotope 208Tl undergoes β decay with a half-life of 3.1 min.
a. What isotope is produced by the decay?
b. How long will it take for 99.0% of a sample of pure 208Tl to decay?
c. What percentage of a sample of pure 208Tl remains un-decayed after 1.0 h?

21.E.3.17: Q21.4.17
If 1.000 g of Ra produces 0.0001 mL of the gas
226
88
222
86
Rn at STP (standard temperature and pressure) in 24 h, what is the half-life
of 226Ra in years?

21.E.3.18: Q21.4.18
The isotope Sr is one of the extremely hazardous species in the residues from nuclear power generation. The strontium in a
90
38

0.500-g sample diminishes to 0.393 g in 10.0 y. Calculate the half-life.

21.E.3.19: Q21.4.19
Technetium-99 is often used for assessing heart, liver, and lung damage because certain technetium compounds are absorbed by
damaged tissues. It has a half-life of 6.0 h. Calculate the rate constant for the decay of Tc. 99
43

21.E.3.20: Q21.4.20
What is the age of mummified primate skin that contains 8.25% of the original quantity of 14C?

21.E.3.21: Q21.4.21
A sample of rock was found to contain 8.23 mg of rubidium-87 and 0.47 mg of strontium-87.
a. Calculate the age of the rock if the half-life of the decay of rubidium by β emission is 4.7 × 1010 y.
b. If some Sr was initially present in the rock, would the rock be younger, older, or the same age as the age calculated in (a)?
87
38

Explain your answer.

21.E.3.22: Q21.4.22
A laboratory investigation shows that a sample of uranium ore contains 5.37 mg of 238
92
U and 2.52 mg of 206
82
. Calculate the age
Pb

of the ore. The half-life of U is 4.5 × 109 yr.


238
92

21.E.3.23: Q21.4.23
Plutonium was detected in trace amounts in natural uranium deposits by Glenn Seaborg and his associates in 1941. They proposed
that the source of this 239Pu was the capture of neutrons by 238U nuclei. Why is this plutonium not likely to have been trapped at
the time the solar system formed 4.7 × 109 years ago?

21.E.3.24: Q21.4.24
A Be atom (mass = 7.0169 amu) decays into a Li atom (mass = 7.0160 amu) by electron capture. How much energy (in millions
7
4
7
3

of electron volts, MeV) is produced by this reaction?

21.E.3.25: Q21.4.25
A B atom (mass = 8.0246 amu) decays into a Be atom (mass = 8.0053 amu) by loss of a β+ particle (mass = 0.00055 amu) or by
8
5
8
4

electron capture. How much energy (in millions of electron volts) is produced by this reaction?

21.E.3.26: Q21.4.26
Isotopes such as 26Al (half-life: 7.2 × 105 years) are believed to have been present in our solar system as it formed, but have since
decayed and are now called extinct nuclides.
26
a. Al decays by β+ emission or electron capture. Write the equations for these two nuclear transformations.

Access for free at OpenStax 21.E.5 https://chem.libretexts.org/@go/page/44089


b. The earth was formed about 4.7 × 109 (4.7 billion) years ago. How old was the earth when 99.999999% of the 26Al originally
present had decayed?

21.E.3.27: Q21.4.27
Write a balanced equation for each of the following nuclear reactions:
a. bismuth-212 decays into polonium-212
b. beryllium-8 and a positron are produced by the decay of an unstable nucleus
c. neptunium-239 forms from the reaction of uranium-238 with a neutron and then spontaneously converts into plutonium-239
d. strontium-90 decays into yttrium-90

21.E.3.28: Q21.4.28
Write a balanced equation for each of the following nuclear reactions:
a. mercury-180 decays into platinum-176
b. zirconium-90 and an electron are produced by the decay of an unstable nucleus
c. thorium-232 decays and produces an alpha particle and a radium-228 nucleus, which decays into actinium-228 by beta decay
d. neon-19 decays into fluorine-19

21.E.4: 21.5: Transmutation and Nuclear Energy


21.E.4.1: Q21.5.1
Write the balanced nuclear equation for the production of the following transuranium elements:
a. berkelium-244, made by the reaction of Am-241 and He-4
b. fermium-254, made by the reaction of Pu-239 with a large number of neutrons
c. lawrencium-257, made by the reaction of Cf-250 and B-11
d. dubnium-260, made by the reaction of Cf-249 and N-15

21.E.4.2: Q21.5.2
How does nuclear fission differ from nuclear fusion? Why are both of these processes exothermic?

21.E.4.3: Q21.5.3
Both fusion and fission are nuclear reactions. Why is a very high temperature required for fusion, but not for fission?

21.E.4.4: Q21.5.4
Cite the conditions necessary for a nuclear chain reaction to take place. Explain how it can be controlled to produce energy, but not
produce an explosion.

21.E.4.5: Q21.5.5
Describe the components of a nuclear reactor.

21.E.4.6: Q21.5.6
In usual practice, both a moderator and control rods are necessary to operate a nuclear chain reaction safely for the purpose of
energy production. Cite the function of each and explain why both are necessary.

21.E.4.7: Q21.5.7
Describe how the potential energy of uranium is converted into electrical energy in a nuclear power plant.

21.E.4.8: Q21.5.8
The mass of a hydrogen atom ( H) is 1.007825 amu; that of a tritium atom ( H) is 3.01605 amu; and that of an α particle is
1
1
3
1

4.00150 amu. How much energy in kilojoules per mole of He produced is released by the following fusion reaction:
4
2

H + H ⟶ He .
1 3 4
1 1 2

Access for free at OpenStax 21.E.6 https://chem.libretexts.org/@go/page/44089


21.E.5: 21.6: Uses of Radioisotopes
21.E.5.1: Q21.6.1
How can a radioactive nuclide be used to show that the equilibrium:
+ −
AgCl(s) ⇌ Ag (aq) + Cl (aq) (21.E.1)

is a dynamic equilibrium?

21.E.5.2: Q21.6.2
Technetium-99m has a half-life of 6.01 hours. If a patient injected with technetium-99m is safe to leave the hospital once 75% of
the dose has decayed, when is the patient allowed to leave?

21.E.5.3: Q21.6.3
Iodine that enters the body is stored in the thyroid gland from which it is released to control growth and metabolism. The thyroid
can be imaged if iodine-131 is injected into the body. In larger doses, I-131 is also used as a means of treating cancer of the thyroid.
I-131 has a half-life of 8.70 days and decays by β− emission.
a. Write a nuclear equation for the decay.
b. How long will it take for 95.0% of a dose of I-131 to decay?

21.E.6: 21.7: Biological Effects of Radiation


21.E.6.1: Q21.7.1
If a hospital were storing radioisotopes, what is the minimum containment needed to protect against:
a. cobalt-60 (a strong γ emitter used for irradiation)
b. molybdenum-99 (a beta emitter used to produce technetium-99 for imaging)

21.E.6.2: Q21.7.2
Based on what is known about Radon-222’s primary decay method, why is inhalation so dangerous?

21.E.6.3: Q21.7.3
Given specimens uranium-232 (t 1/2
= 68.9 y ) and uranium-233 (t 1/2
= 159, 200 y ) of equal mass, which one would have greater
activity and why?

21.E.6.4: Q21.7.4
A scientist is studying a 2.234 g sample of thorium-229 (t1/2 = 7340 y) in a laboratory.
a. What is its activity in Bq?
b. What is its activity in Ci?

21.E.6.5: Q21.7.5
Given specimens neon-24 (t 1/2 = 3.38 min ) and bismuth-211 (t 1/2 = 2.14 min ) of equal mass, which one would have greater
activity and why?

This page titled 21.E: Nuclear Chemistry (Exercises) is shared under a CC BY 4.0 license and was authored, remixed, and/or curated by
OpenStax via source content that was edited to the style and standards of the LibreTexts platform; a detailed edit history is available upon request.

Access for free at OpenStax 21.E.7 https://chem.libretexts.org/@go/page/44089


CHAPTER OVERVIEW
Appendices

A general chemistry Libretexts Textbook remixed and remastered from


OpenStax's textbook:
General Chemistry
Composition of Commercial Acids and Bases
Essential Mathematics
Formation Constants for Complex Ions
Fundamental Physical Constants
Ionization Constants of Weak Acids
Ionization Constants of Weak Bases
Solubility Products
Standard Electrode (Half-Cell) Potentials
Standard Thermodynamic Properties for Selected Substances
The Periodic Table
Units and Conversion Factors
Water Properties

This page titled Appendices is shared under a CC BY 4.0 license and was authored, remixed, and/or curated by OpenStax via source content that
was edited to the style and standards of the LibreTexts platform; a detailed edit history is available upon request.

1
Composition of Commercial Acids and Bases
Composition of Commercial Acids and Bases
Composition of Commercial Acids and Bases

Acid or Base1 Density (g/mL)2 Percentage by Mass Molarity

acetic acid, glacial 1.05 99.5% 17.4

aqueous ammonia3 0.90 28% 14.8

hydrochloric acid 1.18 36% 11.6

nitric acid 1.42 71% 16.0

perchloric acid 1.67 70% 11.65

phosphoric acid 1.70 85% 14.7

sodium hydroxide 1.53 50% 19.1

sulfuric acid 1.84 96% 18.0

Footnotes
1. 1 Acids and bases are commercially available as aqueous solutions. This table lists properties (densities and concentrations) of
common acid and base solutions. Nominal values are provided in cases where the manufacturer cites a range of concentrations
and densities.
2. 2 This column contains specific gravity data. In the case of this table, specific gravity is the ratio of density of a substance to the
density of pure water at the same conditions. Specific gravity is often cited on commercial labels.
3. 3 This solution is sometimes called “ammonium hydroxide,” although this term is not chemically accurate.

This page titled Composition of Commercial Acids and Bases is shared under a CC BY 4.0 license and was authored, remixed, and/or curated by
OpenStax via source content that was edited to the style and standards of the LibreTexts platform; a detailed edit history is available upon request.

Access for free at OpenStax 1 https://chem.libretexts.org/@go/page/43240


Essential Mathematics
1: Exponential Arithmetic
Exponential notation is used to express very large and very small numbers as a product of two numbers. The first number of the
product, the digit term, is usually a number not less than 1 and not greater than 10. The second number of the product, the
exponential term, is written as 10 with an exponent. Some examples of exponential notation are:
3
1000 = 1 × 10
2
100 = 1 × 10
1
10 = 1 × 10
0
1 = 1 × 10
−1
0.1 = 1 × 10
−3
0.001 = 1 × 10
3
2386 = 2.386 × 1000 = 2.386 × 10
−1
0.123 = 1.23 × 0.1 = 1.23 × 10

The power (exponent) of 10 is equal to the number of places the decimal is shifted to give the digit number. The exponential
method is particularly useful notation for every large and very small numbers. For example, 1,230,000,000 = 1.23 × 109, and
0.00000000036 = 3.6 × 10−10.

1.1: Addition of Exponentials


Convert all numbers to the same power of 10, add the digit terms of the numbers, and if appropriate, convert the digit term back to
a number between 1 and 10 by adjusting the exponential term.

 Exercise 1 : Adding Exponentials


Add 5.00 × 10−5 and 3.00 × 10−3.

Solution
−3 −5
3.00 × 10 = 300 × 10
−5 −5 −5 −3
(5.00 × 10 ) + (300 × 10 ) = 305 × 10 = 3.05 × 10

1.2: Subtraction of Exponentials


Convert all numbers to the same power of 10, take the difference of the digit terms, and if appropriate, convert the digit term back
to a number between 1 and 10 by adjusting the exponential term.

 Exercise 2 : Subtracting Exponentials


Subtract 4.0 × 10−7 from 5.0 × 10−6.

Solution
−7 −6
4.0 × 10 = 0.40 × 10
−6 −6 −6
(5.0 × 10 ) − (0.40 × 10 ) = 4.6 × 10

1.3: Multiplication of Exponentials


Multiply the digit terms in the usual way and add the exponents of the exponential terms.

Access for free at OpenStax 1 https://chem.libretexts.org/@go/page/43233


 Exercise 3 : Multiplying Exponentials

Multiply 4.2 × 10−8 by 2.0 × 103.

Solution
−8 3 (−8)+(+3) −5
(4.2 × 10 ) × (2.0 × 10 ) = (4.2 × 2.0) × 10 = 8.4 × 10

1.4: Division of Exponentials


Divide the digit term of the numerator by the digit term of the denominator and subtract the exponents of the exponential terms.

 Exercise 4 : Dividing Exponentials

Divide 3.6 × 105 by 6.0 × 10−4.

Solution
−5
3.6 × 10 3.6 (−5)−(−4) −1 −2
=( ) × 10 = 0.60 × 10 = 6.0 × 10
−4
6.0 × 10 6.0

1.5: Squaring of Exponentials


Square the digit term in the usual way and multiply the exponent of the exponential term by 2.

 Exercise 5 : Squaring Exponentials

Square the number 4.0 × 10−6.

Solution
−6 2 2×(−6) −12 −11
(4.0 × 10 ) = 4 × 4 × 10 = 16 × 10 = 1.6 × 10

1.6: Cubing of Exponentials


Cube the digit term in the usual way and multiply the exponent of the exponential term by 3.

 Exercise 6 : Cubing Exponentials

Cube the number 2 × 104.

Solution
4 3 3×4 12
(2 × 10 ) = 2 × 2 × 2 × 10 = 8 × 10

1.7: Taking Square Roots of Exponentials


If necessary, decrease or increase the exponential term so that the power of 10 is evenly divisible by 2. Extract the square root of
the digit term and divide the exponential term by 2.

 Exercise 7 : Finding the Square Root of Exponentials

Find the square root of 1.6 × 10−7.

Access for free at OpenStax 2 https://chem.libretexts.org/@go/page/43233


Solution
−7 −8
1.6 × 10 = 16 × 10
− −−−− −−− −− −−−− −−
8

√ 16 × 10−8 = √16 × √10−8 = √16 × 10 2
−4
= 4.0 × 10

2: Significant Figures
A beekeeper reports that he has 525,341 bees. The last three figures of the number are obviously inaccurate, for during the time the
keeper was counting the bees, some of them died and others hatched; this makes it quite difficult to determine the exact number of
bees. It would have been more accurate if the beekeeper had reported the number 525,000. In other words, the last three figures are
not significant, except to set the position of the decimal point. Their exact values have no meaning useful in this situation. In
reporting any information as numbers, use only as many significant figures as the accuracy of the measurement warrants.
The importance of significant figures lies in their application to fundamental computation. In addition and subtraction, the sum or
difference should contain as many digits to the right of the decimal as that in the least certain of the numbers used in the
computation (indicated by underscoring in the following example).

 Exercise 8 : Addition and Subtraction with Significant Figures

Add 4.383 g and 0.0023 g.

Solution
4.38 3 g

0.002 3 g
––
–––––––
4.38 5 g

In multiplication and division, the product or quotient should contain no more digits than that in the factor containing the least
number of significant figures.

 Exercise 9 : Multiplication and Division with Significant Figures

Multiply 0.6238 by 6.6.

Solution
0.623 8 × 6. 6 = 4. 1
– – –

When rounding numbers, increase the retained digit by 1 if it is followed by a number larger than 5 (“round up”). Do not change
the retained digit if the digits that follow are less than 5 (“round down”). If the retained digit is followed by 5, round up if the
retained digit is odd, or round down if it is even (after rounding, the retained digit will thus always be even).

3: The Use of Logarithms and Exponential Numbers


The common logarithm of a number (log) is the power to which 10 must be raised to equal that number. For example, the common
logarithm of 100 is 2, because 10 must be raised to the second power to equal 100. Additional examples follow.
Logarithms and Exponential Numbers
Number Number Expressed Exponentially Common Logarithm

1000 103 3

10 101 1

1 100 0

0.1 10−1 −1

Access for free at OpenStax 3 https://chem.libretexts.org/@go/page/43233


Number Number Expressed Exponentially Common Logarithm

0.001 10−3 −3

What is the common logarithm of 60? Because 60 lies between 10 and 100, which have logarithms of 1 and 2, respectively, the
logarithm of 60 is 1.7782; that is,
1.7782
60 = 10

The common logarithm of a number less than 1 has a negative value. The logarithm of 0.03918 is −1.4069, or
1
−1.4069
0.03918 = 10 =
1.4069
10

To obtain the common logarithm of a number, use the log button on your calculator. To calculate a number from its logarithm, take
the inverse log of the logarithm, or calculate 10x (where x is the logarithm of the number).
The natural logarithm of a number (ln) is the power to which e must be raised to equal the number; e is the constant 2.7182818. For
example, the natural logarithm of 10 is 2.303; that is,
2.303 2.303
10 = e = 2.7182818

To obtain the natural logarithm of a number, use the ln button on your calculator. To calculate a number from its natural logarithm,
enter the natural logarithm and take the inverse ln of the natural logarithm, or calculate ex (where x is the natural logarithm of the
number).
Logarithms are exponents; thus, operations involving logarithms follow the same rules as operations involving exponents.
1. The logarithm of a product of two numbers is the sum of the logarithms of the two numbers. \[\log xy= \log x + \log y, \textrm{
and }\ln xy=\ln x + \ln y \nonumber \]
2. The logarithm of the number resulting from the division of two numbers is the difference between the logarithms of the two
numbers. \[\log\dfrac{x}{y}=\log x-\log y,\textrm{ and } \ln\dfrac{x}{y}=\ln x-\ln y \nonumber \]
3. The logarithm of a number raised to an exponent is the product of the exponent and the logarithm of the number.
n n
log x = n log x and  ln x = n ln x

4: The Solution of Quadratic Equations


Mathematical functions of this form are known as second-order polynomials or, more commonly, quadratic functions.
2
ax + bx + c = 0

The solution or roots for any quadratic equation can be calculated using the following formula:
− − −−−−−
2
−b ± √ b − 4ac
x =
2a

Solving Quadratic Equations Solve the quadratic equation 3x2 + 13x − 10 = 0.


Solution Substituting the values a = 3, b = 13, c = −10 in the formula, we obtain
−−−−−−−−−−−−−−−− −
2
−13 ± √ (13 ) − 4 × 3 × (−10)
x =
2 ×3

− −−−−− −− −−−
−13 ± √ 169 + 120 −13 ± √289 −13 ± 17
x = = =
6 6 6

The two roots are therefore


−13 + 17 2 −13 − 17
x = =  and x = = −5
6 3 6

Quadratic equations constructed on physical data always have real roots, and of these real roots, often only those having positive
values are of any significance.

Access for free at OpenStax 4 https://chem.libretexts.org/@go/page/43233


5: Two-Dimensional (x-y) Graphing
The relationship between any two properties of a system can be represented graphically by a two-dimensional data plot. Such a
graph has two axes: a horizontal one corresponding to the independent variable, or the variable whose value is being controlled (x),
and a vertical axis corresponding to the dependent variable, or the variable whose value is being observed or measured (y).
When the value of y is changing as a function of x (that is, different values of x correspond to different values of y), a graph of this
change can be plotted or sketched. The graph can be produced by using specific values for (x,y) data pairs.

 Exercise 10

Graphing the Dependence of y on x


Table for Exercise 10
x y

1 5

2 10

3 7

4 14

This table contains the following points: (1,5), (2,10), (3,7), and (4,14). Each of these points can be plotted on a graph and
connected to produce a graphical representation of the dependence of y on x.

If the function that describes the dependence of y on x is known, it may be used to compute x,y data pairs that may subsequently be
plotted.

 Exercise 11

Plotting Data Pairs If we know that y = x2 + 2, we can produce a table of a few (x,y) values and then plot the line based on the
data shown here.
Table for Exercise 11
x y = x2 + 2

1 3

2 6

3 11

4 18

Access for free at OpenStax 5 https://chem.libretexts.org/@go/page/43233


This page titled Essential Mathematics is shared under a CC BY 4.0 license and was authored, remixed, and/or curated by OpenStax via source
content that was edited to the style and standards of the LibreTexts platform; a detailed edit history is available upon request.

Access for free at OpenStax 6 https://chem.libretexts.org/@go/page/43233


Formation Constants for Complex Ions
Formation Constants for Complex Ions
Formation Constants for Complex Ions

Equilibrium Kf

Al
3+
+6F

⇌ [AlF ]
6
3−
7 × 1019

Cd
2+
+ 4 NH
3
⇌ [Cd(NH ) ]
3 4
2+
1.3 × 107

Cd
2+
+ 4 CN

⇌ [Cd(CN) ]
4
2−
3 × 1018

Co
2+
+ 6 NH
3
⇌ [Co (NH ) ]
3 6
2+
1.3 × 105

Co
3+
+ 6 NH
3
⇌ [Co (NH ) ]
3 6
3+
2.3 × 1033

1.0 × 1016
+ −
Cu + 2 CN ⇌ [Cu(CN) ]
2

Cu
2+
+ 4 NH
3
⇌ [Cu(NH ) ]
3 4
2+
1.7 × 1013

Fe
2+
+ 6 CN

⇌ [Fe(CN) ]
6
4−
1.5 × 1035

Fe
3+
+ 6 CN

⇌ [Fe(CN) ]
6
3−
2 × 1043

Fe
3+
+ 6 SCN

⇌ [Fe(SCN) ]
6
3−
3.2 × 103

Hg
2+
+ 4 Cl

⇌ [HgCl ]
4
2−
1.1 × 1016

Ni
2+
+ 6 NH
3
⇌ [Ni(NH ) ]
3 6
2+
2.0 × 108

1.8 × 105
+ − −
Ag + 2 Cl ⇌ [AgCl ]
2

1 × 1021
+ − −
Ag + 2 CN ⇌ [Ag (CN) ]
2

1.7 × 107
+ +
Ag + 2 NH ⇌ [Ag (NH ) ]
3 3 2

Zn
2+
+ 4 CN

⇌ [Zn(CN) ]
4
2−
2.1 × 1019

Zn
2+
+ 4 OH

⇌ [Zn(OH) ]
4
2−
2 × 1015

Fe
3+
+ SCN

⇌ [Fe(SCN)]
2+
8.9 × 102

Ag
+
+ 4 SCN

⇌ [Ag (SCN) ]
4
3−
1.2 × 1010

Pb
2+
+4I

⇌ [PbI ]
4
2−
3.0 × 104

Pt
2+
+ 4 Cl

⇌ [PtCl ]
4
2−
1 × 1016

Cu
2+
+ 4 CN ⇌ [Cu(CN) ]
4
2−
1.0 × 1025

Co
2+
+ 4 SCN

⇌ [Co (SCN) ]
4
2−
1 × 103

This page titled Formation Constants for Complex Ions is shared under a CC BY 4.0 license and was authored, remixed, and/or curated by
OpenStax via source content that was edited to the style and standards of the LibreTexts platform; a detailed edit history is available upon request.

Access for free at OpenStax 1 https://chem.libretexts.org/@go/page/43245


Fundamental Physical Constants
Fundamental Physical Constants
Fundamental Physical Constants

Name and Symbol Value

atomic mass unit (amu) 1.6605402 × 10−27 kg

Avogadro’s number 6.0221367 × 1023 mol−1

Boltzmann’s constant (k) 1.380658 × 10−23 J K−1

charge-to-mass ratio for electron (e/me) 1.75881962 × 1011 C kg−1

electron charge (e) 1.60217733 × 10−19 C

electron rest mass (me) 9.1093897 × 10−31 kg

Faraday’s constant (F) 9.6485309 × 104 C mol−1

gas constant (R) 8.205784 × 10−2 L atm mol−1 K−1 = 8.314510 J mol−1 K−1

molar volume of an ideal gas, 1 atm, 0 °C 22.41409 L mol–1

molar volume of an ideal gas, 1 bar, 0 °C 22.71108 L mol–1

neutron rest mass (mn) 1.6749274 × 10−27 kg

Planck’s constant (h) 6.6260755 × 10−34 J s

proton rest mass (mp) 1.6726231 × 10−27 kg

Rydberg constant (R) 1.0973731534 × 107 m−1 = 2.1798736 × 10−18 J

speed of light (in vacuum) (c) 2.99792458 × 108 m s−1

This page titled Fundamental Physical Constants is shared under a CC BY 4.0 license and was authored, remixed, and/or curated by OpenStax via
source content that was edited to the style and standards of the LibreTexts platform; a detailed edit history is available upon request.

Access for free at OpenStax 1 https://chem.libretexts.org/@go/page/43236


Ionization Constants of Weak Acids
Ionization Constants of Weak Acids
Ionization Constants of Weak Acids

Acid Formula Ka at 25 °C Lewis Structure

acetic CH3CO2H 1.8 × 10−5

H3AsO4 5.5 × 10−3

1.7 × 10−7
arsenic

5.1 × 10−12

arsenous H3AsO3 5.1 × 10−10

boric H3BO3 5.4 × 10−10

H2CO3 4.3 × 10−7

carbonic
4.7 × 10−11

cyanic HCNO 2 × 10−4

formic HCO2H 1.8 × 10−4

hydrazoic HN3 2.5 × 10−5

hydrocyanic HCN 4.9 × 10−10

hydrofluoric HF 3.5 × 10−4

hydrogen peroxide H2O2 2.4 × 10−12

Access for free at OpenStax 1 https://chem.libretexts.org/@go/page/43242


Ionization Constants of Weak Acids

Acid Formula Ka at 25 °C Lewis Structure

H2Se 1.29 × 10−4


hydrogen selenide
HSe– 1 × 10−12

hydrogen sulfate ion 1.2 × 10−2

H2S 8.9 × 10−8


hydrogen sulfide
HS– 1.0 × 10−19

H2Te 2.3 × 10−3


hydrogen telluride
HTe– 1.6 × 10−11

hypobromous HBrO 2.8 × 10−9

hypochlorous HClO 2.9 × 10−8

nitrous HNO2 4.6 × 10−4

H2C2O4 6.0 × 10−2


oxalic
6.1 × 10−5

H3PO4 7.5 × 10−3

6.2 × 10−8
phosphoric

4.2 × 10−13

H3PO3 5 × 10−2

phosphorous
2.0 × 10−7

H2SO3 1.6 × 10−2

sulfurous
6.4 × 10−8

This page titled Ionization Constants of Weak Acids is shared under a CC BY 4.0 license and was authored, remixed, and/or curated by OpenStax
via source content that was edited to the style and standards of the LibreTexts platform; a detailed edit history is available upon request.

Access for free at OpenStax 2 https://chem.libretexts.org/@go/page/43242


Ionization Constants of Weak Bases
Ionization Constants of Weak Bases
Ionization Constants of Weak Bases

Base Lewis Structure Kb at 25 °C

ammonia 1.8 × 10−5

dimethylamine 5.9 × 10−4

methylamine 4.4 × 10−4

phenylamine (aniline) 4.3 × 10−10

trimethylamine 6.3 × 10−5

This page titled Ionization Constants of Weak Bases is shared under a CC BY 4.0 license and was authored, remixed, and/or curated by OpenStax
via source content that was edited to the style and standards of the LibreTexts platform; a detailed edit history is available upon request.

Access for free at OpenStax 1 https://chem.libretexts.org/@go/page/43243


Solubility Products
Solubility Products
Solubility Products

Substance Ksp at 25 °C

aluminum

Al(OH)3 2 × 10−32

barium

BaCO3 1.6 × 10−9

BaC2O4·2H2O 1.1 × 10−7

BaSO4 2.3 × 10−8

BaCrO4 8.5 × 10−11

BaF2 2.4 × 10−5

Ba(OH)2·8H2O 5.0 × 10−3

Ba3(PO4)2 6 × 10−39

Ba3(AsO4)2 1.1 × 10−13

bismuth

BiO(OH) 4 × 10−10

BiOCl 1.8 × 10−31

Bi2S3 1 × 10−97

cadmium

Cd(OH)2 5.9 × 10−15

CdS 1.0 × 10−28

CdCO3 5.2 × 10−12

calcium

Ca(OH)2 1.3 × 10−6

CaCO3 8.7 × 10−9

CaSO4·2H2O 6.1 × 10−5

CaC2O4·H2O 1.96 × 10−8

Ca3(PO4)2 1.3 × 10−32

CaHPO4 7 × 10−7

CaF2 4.0 × 10−11

chromium

Cr(OH)3 6.7 × 10−31

cobalt

Co(OH)2 2.5 × 10−16

CoS(α) 5 × 10−22

CoS(β) 3 × 10−26

CoCO3 1.4 × 10−13

Co(OH)3 2.5 × 10−43

Access for free at OpenStax 1 https://chem.libretexts.org/@go/page/43244


Solubility Products

Substance Ksp at 25 °C

copper

CuCl 1.2 × 10−6

CuBr 6.27 × 10−9

CuI 1.27 × 10−12

CuSCN 1.6 × 10−11

Cu2S 2.5 × 10−48

Cu(OH)2 2.2 × 10−20

CuS 8.5 × 10−45

CuCO3 2.5 × 10−10

iron

Fe(OH)2 1.8 × 10−15

FeCO3 2.1 × 10−11

FeS 3.7 × 10−19

Fe(OH)3 4 × 10−38

lead

Pb(OH)2 1.2 × 10−15

PbF2 4 × 10−8

PbCl2 1.6 × 10−5

PbBr2 4.6 × 10−6

PbI2 1.4 × 10−8

PbCO3 1.5 × 10−15

PbS 7 × 10−29

PbCrO4 2 × 10−16

PbSO4 1.3 × 10−8

Pb3(PO4)2 1 × 10−54

magnesium

Mg(OH)2 8.9 × 10−12

MgCO3·3H2O ca 1 × 10−5

MgNH4PO4 3 × 10−13

MgF2 6.4 × 10−9

MgC2O4 7 × 10−7

manganese

Mn(OH)2 2 × 10−13

MnCO3 8.8 × 10−11

MnS 2.3 × 10−13

mercury

Hg2O·H2O 3.6 × 10−26

Access for free at OpenStax 2 https://chem.libretexts.org/@go/page/43244


Solubility Products

Substance Ksp at 25 °C

Hg2Cl2 1.1 × 10−18

Hg2Br2 1.3 × 10−22

Hg2I2 4.5 × 10−29

Hg2CO3 9 × 10−15

Hg2SO4 7.4 × 10−7

Hg2S 1.0 × 10−47

Hg2CrO4 2 × 10−9

HgS 1.6 × 10−54

nickel

Ni(OH)2 1.6 × 10−16

NiCO3 1.4 × 10−7

NiS(α) 4 × 10−20

NiS(β) 1.3 × 10−25

potassium

KClO4 1.05 × 10−2

K2PtCl6 7.48 × 10−6

KHC4H4O6 3 × 10−4

silver
1

2
Ag O(Ag
2
+
+ OH

) 2 × 10−8

AgCl 1.6 × 10−10

AgBr 5.0 × 10−13

AgI 1.5 × 10−16

AgCN 1.2 × 10−16

AgSCN 1.0 × 10−12

Ag2S 1.6 × 10−49

Ag2CO3 8.1 × 10−12

Ag2CrO4 9.0 × 10−12

Ag4Fe(CN)6 1.55 × 10−41

Ag2SO4 1.2 × 10−5

Ag3PO4 1.8 × 10−18

strontium

Sr(OH)2·8H2O 3.2 × 10−4

SrCO3 7 × 10−10

SrCrO4 3.6 × 10−5

SrSO4 3.2 × 10−7

SrC2O4·H2O 4 × 10−7

thallium

Access for free at OpenStax 3 https://chem.libretexts.org/@go/page/43244


Solubility Products

Substance Ksp at 25 °C

TlCl 1.7 × 10−4

TlSCN 1.6 × 10−4

Tl2S 6 × 10−22

Tl(OH)3 6.3 × 10−46

tin

Sn(OH)2 3 × 10−27

SnS 1 × 10−26

Sn(OH)4 1.0 × 10−57

zinc

ZnCO3 2 × 10−10

This page titled Solubility Products is shared under a CC BY 4.0 license and was authored, remixed, and/or curated by OpenStax via source
content that was edited to the style and standards of the LibreTexts platform; a detailed edit history is available upon request.

Access for free at OpenStax 4 https://chem.libretexts.org/@go/page/43244


Standard Electrode (Half-Cell) Potentials
Standard Electrode (Half-Cell) Potentials
Half-Reaction E° (V)

Ag
+
+e

⟶ Ag +0.7996

AgCl + e

⟶ Ag + Cl

+0.22233

[Ag (CN) ]
2
+e

⟶ Ag + 2 CN

−0.31

Ag CrO
2 4
+2e

⟶ 2 Ag + CrO
2−
4
+0.45
+
[Ag (NH ) ]
3 2
+e

⟶ Ag + 2 NH
3
+0.373

[Ag (S O ) ]
2 3 2
3+
+e

⟶ Ag + 2 S O
2
2−
3
+0.017

[AlF ]
6
3−
+3e

⟶ Al + 6 F

−2.07

Al
3+
+3e

⟶ Al −1.662

Am
3+
+3e

⟶ Am −2.048

Au
3+
+3e

⟶ Au +1.498

Au
+
+e

⟶ Au +1.692

Ba
2+
+2e

⟶ Ba −2.912

Be
2+
+2e

⟶ Be −1.847

Br (aq) + 2 e
2

⟶ 2 Br

+1.0873

Ca
2+
+2e

⟶ Ca −2.868

Ce
3
+3e

⟶ Ce −2.483

Ce
4+
+e

⟶ Ce
3+
+1.61

Cd
2+
+2e

⟶ Cd −0.4030

[Cd(CN) ]
4
2−
+2e

⟶ Cd + 4 CN

−1.09

[Cd(NH ) ]
3 4
2+
+2e

⟶ Cd + 4 NH
3
−0.61

CdS + 2 e

⟶ Cd + S
2−
−1.17

Cl
2
+2e

⟶ 2 Cl

+1.35827

ClO

4
+H O+2e
2

⟶ ClO

3
+ 2 OH

+0.36

ClO

3
+H O+2e
2

⟶ ClO

2
+ 2 OH

+0.33

ClO

2
+H O+2e
2

⟶ ClO

+ 2 OH

+0.66

ClO

+H O+2e
2

⟶ Cl

+ 2 OH

+0.89

ClO

4
+2H O
3
+
+2e

⟶ ClO

3
+3H O
2
+1.189

ClO

3
+3H O
3
+
+2e

⟶ HClO
2
+4H O
2
+1.21

HClO + H O
3
+
+2e

⟶ Cl

+2H O
2
+1.482
1
HClO + H O
3
+
+e

⟶ Cl
2
+2H O
2
+1.611
2

HClO
2
+2H O
3
+
+2e

⟶ HClO + 3 H O
2
+1.628

Co
3+
+e

⟶ Co
2+
(2 mol // H SO )
2 4
+1.83

Co
2+
+2e

⟶ Co −0.28

[Co (NH ) ]
3 6
3+
+e

⟶ [Co (NH ) ]
3 6
2+
+0.1

Co (OH)
3
+e

⟶ Co (OH)
2
+ OH

+0.17

Access for free at OpenStax 1 https://chem.libretexts.org/@go/page/43246


Half-Reaction E° (V)

Cr
3
+3e

⟶ Cr −0.744

Cr
3+
+e

⟶ Cr
2+
−0.407

Cr
2+
+2e

⟶ Cr −0.913

[Cu(CN) ]
2
+e

⟶ Cu + 2 CN

−0.43

CrO
2−
4
+4H O+3e
2

⟶ Cr(OH)
3
+ 5 OH

−0.13

Cr O
2
2−
7
+ 14 H O
3
+
+6e

⟶ 2 Cr
3+
+ 21 H O
2
+1.232

[Cr(OH) ]
4
+3e

⟶ Cr + 4 OH

−1.2

Cr(OH)
3
+3e

⟶ Cr + 3 OH

−1.48

Cu
2+
+e

⟶ Cu
+
+0.153

Cu
2+
+2e

⟶ Cu +0.34

Cu
+
+e

⟶ Cu +0.521

F
2
+2e

⟶ 2F

+2.866

Fe
2+
+2e

⟶ Fe −0.447

Fe
3+
+e

⟶ Fe
2+
+0.771

[Fe(CN) ]
6
3−
+e

⟶ [Fe(CN) ]
6
4−
+0.36

Fe(OH)
2
+2e

⟶ Fe + 2 OH

−0.88

FeS + 2 e

⟶ Fe + S
2−
−1.01

Ga
3+
+3e

⟶ Ga −0.549

Gd
3+
+3e

⟶ Gd −2.279
1
H
2
+e

⟶ H

−2.23
2

2H O+2e
2

⟶ H
2
+ 2 OH

−0.8277

H O
2 2
+2H O
3
+
+2e

⟶ 4H O
2
+1.776

2H O
3
+
+2e

⟶ H
2
+2H O
2
0.00

HO

2
+H O+2e
2

⟶ 3 OH

+0.878

Hf
4+
+4e

⟶ Hf −1.55

Hg
2+
+2e

⟶ Hg +0.851

2 Hg
2+
+2e

⟶ Hg
2+
2
+0.92

Hg
2+
2
+2e

⟶ 2 Hg +0.7973

[HgBr ]
4
2−
+2e

⟶ Hg + 4 Br

+0.21

Hg Cl
2 2
+2e

⟶ 2 Hg + 2 Cl

+0.26808

[Hg (CN) ]
4
2−
+2e

⟶ Hg + 4 CN

−0.37

[HgI ]
4
2−
+2e

⟶ Hg + 4 I

−0.04

HgS + 2 e

⟶ Hg + S
2−
−0.70

I
2
+2e

⟶ 2I

+0.5355

In
3+
+3e

⟶ In −0.3382

K
+
+e

⟶ K −2.931

La
3+
+3e

⟶ La −2.52

Access for free at OpenStax 2 https://chem.libretexts.org/@go/page/43246


Half-Reaction E° (V)

Li
+
+e

⟶ Li −3.04

Lu
3+
+3e

⟶ Lu −2.28

Mg
2+
+2e

⟶ Mg −2.372

Mn
2+
+2e

⟶ Mn −1.185

MnO
2
+2H O+2e
2

⟶ Mn(OH)
2
+ 2 OH

−0.05

MnO

4
+2H O+3e
2

⟶ MnO
2
+ 4 OH

+0.558

MnO
2
+4H
+
+2e

⟶ Mn
2+
+2H O
2
+1.23

MnO

4
+8H
+
+5e

⟶ Mn
2+
+4H O
2
+1.507

Na
+
+e

⟶ Na −2.71

Nd
3+
+3e

⟶ Nd −2.323

Ni
2+
+2e

⟶ Ni −0.257

[Ni(NH ) ]
3 6
2+
+2e

⟶ Ni + 6 NH
3
−0.49

NiO
2
+4H
+
+2e

⟶ Ni
2+
+2H O
2
+1.593

NiO
2
+2H O+2e
2

⟶ Ni(OH)
2
+ 2 OH

+0.49

NiS + 2 e

⟶ Ni + S
2−
+0.76

NO

3
+4H
+
+3e

⟶ NO + 2 H O
2
+0.957

NO

3
+3H
+
+2e

⟶ HNO
2
+H O
2
+0.92

NO

3
+H O+2e
2

⟶ NO

2
+ 2 OH

+0.10

Np
3+
+3e

⟶ Np −1.856

O
2
+2H O+4e
2

⟶ 4 OH

+0.401

O
2
+2H
+
+2e

⟶ H O
2 2
+0.695

O
2
+4H
+
+4e

⟶ 2H O
2
+1.229

Pb
2+
+2e

⟶ Pb −0.1262

PbO
2
+ SO
2−
4
+4H
+
+2e

⟶ PbSO
4
+2H O
2
+1.69

PbS + 2 e

⟶ Pb + S
2−
−0.95

PbSO
4
+2e

⟶ Pb + SO
2−
4
−0.3505

Pd
2+
+2e

⟶ Pd +0.987

[PdCl ]
4
2−
+2e

⟶ Pd + 4 Cl

+0.591

Pt
2+
+2e

⟶ Pt +1.20

[PtBr ]
4
2−
+2e

⟶ Pt + 4 Br

+0.58

[PtCl ]
4
2−
+2e

⟶ Pt + 4 Cl

+0.755

[PtCl ]
6
2−
+2e

⟶ [PtCl ]
4
2−
+ 2 Cl

+0.68

Pu
3
+3e

⟶ Pu −2.03

Ra
2+
+2e

⟶ Ra −2.92

Rb
+
+e

⟶ Rb −2.98

[RhCl
6
]
3−
+3e

⟶ Rh + 6 Cl

+0.44

S+2 e

⟶ S
2−
−0.47627

Access for free at OpenStax 3 https://chem.libretexts.org/@go/page/43246


Half-Reaction E° (V)

S+2 H
+
+2e

⟶ H S
2
+0.142

Sc
3+
+3e

⟶ Sc −2.09

Se + 2 H
+
+2e

⟶ H Se
2
−0.399

[SiF ]
6
2−
+4e

⟶ Si + 6 F

−1.2

SiO
2−
3
+3H O+4e
2

⟶ Si + 6 OH

−1.697

SiO
2
+4H
+
+4e

⟶ Si + 2 H O
2
−0.86

Sm
3+
+3e

⟶ Sm −2.304

Sn
4+
+2e

⟶ Sn
2+
+0.151

Sn
2+
+2e

⟶ Sn −0.1375

[SnF ]
6
2−
+4e

⟶ Sn + 6 F

−0.25

SnS + 2 e

⟶ Sn + S
2−
−0.94

Sr
2+
+2e

⟶ Sr −2.89

TeO
2
+4H
+
+4e

⟶ Te + 2 H O
2
+0.593

Th
4+
+4e

⟶ Th −1.90

Ti
2+
+2e

⟶ Ti −1.630

U
3+
+3e

⟶ U −1.79

V
2+
+2e

⟶ V −1.19

Y
3+
+3e

⟶ Y −2.37

Zn
2+
+2e

⟶ Zn −0.7618

[Zn(CN) ]
4
2−
+2e

⟶ Zn + 4 CN

−1.26

[Zn(NH ) ]
3 4
2+
+2e

⟶ Zn + 4 NH
3
−1.04

Zn(OH)
2
+2e

⟶ Zn + 2 OH

−1.245

[Zn(OH) ]
4
2
+2e

⟶ Zn + 4 OH

−1.199

ZnS + 2 e

⟶ Zn + S
2−
−1.40

Zr
4
+4e

⟶ Zr −1.539

This page titled Standard Electrode (Half-Cell) Potentials is shared under a CC BY 4.0 license and was authored, remixed, and/or curated by
OpenStax via source content that was edited to the style and standards of the LibreTexts platform; a detailed edit history is available upon request.

Access for free at OpenStax 4 https://chem.libretexts.org/@go/page/43246


Standard Thermodynamic Properties for Selected Substances
Standard Thermodynamic Properties for Selected Substances
Standard Thermodynamic Properties for Selected Substances

Substance ΔH

f
(kJ mol–) ΔG

f
(kJ mol–1) S
298

(J K–1 mol–1)

aluminum

Al(s) 0 0 28.3

Al(g) 324.4 285.7 164.54

Al
3 +
(aq) -531 -485 -321.7

Al2O3(s) –1676 –1582 50.92

AlF3(s) –1510.4 –1425 66.5

AlCl3(s) –704.2 –628.8 110.67

AlCl3·6H2O(s) –2691.57 –2269.40 376.56

Al2S3(s) –724.0 –492.4 116.9

Al2(SO4)3(s) –3445.06 –3506.61 239.32

antimony

Sb(s) 0 0 45.69

Sb(g) 262.34 222.17 180.16

Sb4O6(s) –1440.55 –1268.17 220.92

SbCl3(g) –313.8 –301.2 337.80

SbCl5(g) –394.34 –334.29 401.94

Sb2S3(s) –174.89 –173.64 182.00

SbCl3(s) –382.17 –323.72 184.10

SbOCl(s) –374.0 — —

arsenic

As(s) 0 0 35.1

As(g) 302.5 261.0 174.21

As4(g) 143.9 92.4 314

As4O6(s) –1313.94 –1152.52 214.22

As2O5(s) –924.87 –782.41 105.44

AsCl3(g) –261.50 –248.95 327.06

As2S3(s) –169.03 –168.62 163.59

AsH3(g) 66.44 68.93 222.78

H3AsO4(s) –906.3 — —

barium

Ba(s) 0 0 62.5

Ba(g) 180 146 170.24

Ba
2 +
(aq) -537.6 -560.8 9.6

BaO(s) –548.0 –520.3 72.1

BaCl2(s) –855.0 –806.7 123.7

Access for free at OpenStax 1 https://chem.libretexts.org/@go/page/43241


Standard Thermodynamic Properties for Selected Substances

Substance ΔH
f

(kJ mol–) ΔG

f
(kJ mol–1) S

298
(J K–1 mol–1)

BaSO4(s) –1473.2 –1362.3 132.2

beryllium

Be(s) 0 0 9.50

Be(g) 324.3 286.6 136.27

BeO(s) –609.4 –580.1 13.8

bismuth

Bi(s) 0 0 56.74

Bi(g) 207.1 168.2 187.00

Bi2O3(s) –573.88 –493.7 151.5

BiCl3(s) –379.07 –315.06 176.98

Bi2S3(s) –143.1 –140.6 200.4

boron

B(s) 0 0 5.86

B(g) 565.0 521.0 153.4

B2O3(s) –1273.5 –1194.3 53.97

B2H6(g) 36.4 87.6 232.1

H3BO3(s) –1094.33 –968.92 88.83

BF3(g) –1136.0 –1119.4 254.4

BCl3(g) –403.8 –388.7 290.1

B3N3H6(l) –540.99 –392.79 199.58

HBO2(s) –794.25 –723.41 37.66

bromine

Br2(l) 0 0 152.23

Br2(g) 30.91 3.142 245.5

Br(g) 111.88 82.429 175.0

Br

(aq) -120.9 -102.82 80.71

BrF3(g) –255.60 –229.45 292.42

HBr(g) –36.3 –53.43 198.7

cadmium

Cd(s) 0 0 51.76

Cd(g) 112.01 77.41 167.75

Cd
2 +
(aq) -75.90 -77.61 -73.2

CdO(s) –258.2 –228.4 54.8

CdCl2(s) –391.5 –343.9 115.3

CdSO4(s) –933.3 –822.7 123.0

CdS(s) –161.9 –156.5 64.9

calcium

Access for free at OpenStax 2 https://chem.libretexts.org/@go/page/43241


Standard Thermodynamic Properties for Selected Substances

Substance ΔH
f

(kJ mol–) ΔG

f
(kJ mol–1) S

298
(J K–1 mol–1)

Ca(s) 0 0 41.6

Ca(g) 178.2 144.3 154.88

Ca
2 +
(aq) -542.96 -553.04 -55.2

CaO(s) –634.9 –603.3 38.1

Ca(OH)2(s) –985.2 –897.5 83.4

CaSO4(s) –1434.5 –1322.0 106.5

CaSO4·2H2O(s) –2022.63 –1797.45 194.14

CaCO3(s) (calcite) –1220.0 –1081.4 110.0

CaSO3·H2O(s) –1752.68 –1555.19 184.10

carbon

C(s) (graphite) 0 0 5.740

C(s) (diamond) 1.89 2.90 2.38

C(g) 716.681 671.2 158.1

CO(g) –110.52 –137.15 197.7

CO2(g) –393.51 –394.36 213.8

CO
2 −
3
(aq) -677.1 -527.8 -56.9

CH4(g) –74.6 –50.5 186.3

CH3OH(l) –239.2 –166.6 126.8

CH3OH(g) –201.0 –162.3 239.9

CCl4(l) –128.2 –62.5 214.4

CCl4(g) –95.7 –58.2 309.7

CHCl3(l) –134.1 –73.7 201.7

CHCl3(g) –103.14 –70.34 295.71

CS2(l) 89.70 65.27 151.34

CS2(g) 116.9 66.8 238.0

C2H2(g) 227.4 209.2 200.9

C2H4(g) 52.4 68.4 219.3

C2H6(g) –84.0 –32.0 229.2

CH3CO2H(l) –484.3 –389.9 159.8

CH3CO2H(g) –434.84 –376.69 282.50

C2H5OH(l) –277.6 –174.8 160.7

C2H5OH(g) –234.8 –167.9 281.6

HCO

3
(aq) -691.11 -587.06 95

C3H8(g) –103.8 –23.4 270.3

C6H6(g) 82.927 129.66 269.2

C6H6(l) 49.1 124.50 173.4

CH2Cl2(l) –124.2 –63.2 177.8

Access for free at OpenStax 3 https://chem.libretexts.org/@go/page/43241


Standard Thermodynamic Properties for Selected Substances

Substance ΔH
f

(kJ mol–) ΔG

f
(kJ mol–1) S

298
(J K–1 mol–1)

CH2Cl2(g) –95.4 –65.90 270.2

CH3Cl(g) –81.9 –60.2 234.6

C2H5Cl(l) –136.52 –59.31 190.79

C2H5Cl(g) –112.17 –60.39 276.00

C2N2(g) 308.98 297.36 241.90

HCN(l) 108.9 125.0 112.8

HCN(g) 135.5 124.7 201.8

cesium

Cs
+
(aq) -248 -282.0 133

chlorine

Cl2(g) 0 0 223.1

Cl(g) 121.3 105.70 165.2

Cl

(aq) -167.2 -131.2 56.5

ClF(g) –54.48 –55.94 217.78

ClF3(g) –158.99 –118.83 281.50

Cl2O(g) 80.3 97.9 266.2

Cl2O7(l) 238.1 — —

Cl2O7(g) 272.0 — —

HCl(g) –92.307 –95.299 186.9

HClO4(l) –40.58 — —

chromium

Cr(s) 0 0 23.77

Cr(g) 396.6 351.8 174.50

CrO
2 −
4
(aq) -881.2 -727.8 50.21

Cr O
2
2 −
7
(aq) -1490.3 -1301.1 261.9

Cr2O3(s) –1139.7 –1058.1 81.2

CrO3(s) –589.5 — —

(NH4)2Cr2O7(s) –1806.7 — —

cobalt

Co(s) 0 0 30.0

Co
2 +
(aq) -67.4 -51.5 -155

Co
3 +
(aq) 92 134 -305.0

CoO(s) –237.9 –214.2 52.97

Co3O4(s) –910.02 –794.98 114.22

Co(NO3)2(s) –420.5 — —

copper

Cu(s) 0 0 33.15

Access for free at OpenStax 4 https://chem.libretexts.org/@go/page/43241


Standard Thermodynamic Properties for Selected Substances

Substance ΔH
f

(kJ mol–) ΔG

f
(kJ mol–1) S

298
(J K–1 mol–1)

Cu(g) 338.32 298.58 166.38

Cu
+
(aq) 51.9 50.2 -26

Cu
2 +
(aq) 64.77 65.49 -99.6

CuO(s) –157.3 –129.7 42.63

Cu2O(s) –168.6 –146.0 93.14

CuS(s) –53.1 –53.6 66.5

Cu2S(s) –79.5 –86.2 120.9

CuSO4(s) –771.36 –662.2 109.2

Cu(NO3)2(s) –302.9 — —

fluorine

F2(g) 0 0 202.8

F(g) 79.4 62.3 158.8

F

(aq) -332.6 -278.8 -13.8

F2O(g) 24.7 41.9 247.43

HF(g) –273.3 –275.4 173.8

hydrogen

H2(g) 0 0 130.7

H(g) 217.97 203.26 114.7

H
+
(aq) 0 0 0

OH

(aq) -230.0 -157.2 -10.75

H O
3
+
(aq) -285.8 69.91

H2O(l) –285.83 –237.1 70.0

H2O(g) –241.82 –228.59 188.8

H2O2(l) –187.78 –120.35 109.6

H2O2(g) –136.3 –105.6 232.7

HF(g) –273.3 –275.4 173.8

HCl(g) –92.307 –95.299 186.9

HBr(g) –36.3 –53.43 198.7

HI(g) 26.48 1.70 206.59

H2S(g) –20.6 –33.4 205.8

H2Se(g) 29.7 15.9 219.0

iodine

I2(s) 0 0 116.14

I2(g) 62.438 19.3 260.7

I(g) 106.84 70.2 180.8

I

(aq) -55.19 -51.57 11.13

IF(g) 95.65 –118.49 236.06

Access for free at OpenStax 5 https://chem.libretexts.org/@go/page/43241


Standard Thermodynamic Properties for Selected Substances

Substance ΔH

f
(kJ mol–) ΔG

f
(kJ mol–1) S

298
(J K–1 mol–1)

ICl(g) 17.78 –5.44 247.44

IBr(g) 40.84 3.72 258.66

IF7(g) –943.91 –818.39 346.44

HI(g) 26.48 1.70 206.59

iron

Fe(s) 0 0 27.3

Fe(g) 416.3 370.7 180.5

Fe
2 +
(aq) -89.1 -78.90 -137.7

Fe
3 +
(aq) -48.5 -4.7 -315.9

Fe2O3(s) –824.2 –742.2 87.40

Fe3O4(s) –1118.4 –1015.4 146.4

Fe(CO)5(l) –774.04 –705.42 338.07

Fe(CO)5(g) –733.87 –697.26 445.18

FeCl2(s) –341.79 –302.30 117.95

FeCl3(s) –399.49 –334.00 142.3

FeO(s) –272.0 –255.2 60.75

Fe(OH)2(s) –569.0 –486.5 88.

Fe(OH)3(s) –823.0 –696.5 106.7

FeS(s) –100.0 –100.4 60.29

Fe3C(s) 25.10 20.08 104.60

lead

Pb(s) 0 0 64.81

Pb(g) 195.2 162. 175.4

Pb
2 +
(aq) -1.7 -24.43 10.5

PbO(s) (yellow) –217.32 –187.89 68.70

PbO(s) (red) –218.99 –188.93 66.5

Pb(OH)2(s) –515.9 — —

PbS(s) –100.4 –98.7 91.2

Pb(NO3)2(s) –451.9 — —

PbO2(s) –277.4 –217.3 68.6

PbCl2(s) –359.4 –314.1 136.0

lithium

Li(s) 0 0 29.1

Li(g) 159.3 126.6 138.8

Li
+
(aq) -278.5 -293.3 13.4

LiH(s) –90.5 –68.3 20.0

Li(OH)(s) –487.5 –441.5 42.8

Access for free at OpenStax 6 https://chem.libretexts.org/@go/page/43241


Standard Thermodynamic Properties for Selected Substances

Substance ΔH
f

(kJ mol–) ΔG

f
(kJ mol–1) S

298
(J K–1 mol–1)

LiF(s) –616.0 –587.5 35.7

Li2CO3(s) –1216.04 –1132.19 90.17

magnesium

Mg
2 +
(aq) -466.9 -454.8 -138.1

manganese

Mn(s) 0 0 32.0

Mn(g) 280.7 238.5 173.7

Mn
2 +
(aq) -220.8 -228.1 -73.6

MnO(s) –385.2 –362.9 59.71

MnO2(s) –520.03 –465.1 53.05

Mn2O3(s) –958.97 –881.15 110.46

Mn3O4(s) –1378.83 –1283.23 155.64

MnO

4
(aq) -541.4 -447.2 191.2

MnO
2 −
4
(aq) -653.0 -500.7 59

mercury

Hg(l) 0 0 75.9

Hg(g) 61.4 31.8 175.0

Hg
2 +
(aq) 164.8

Hg
2 +
2
(aq) 172.4 153.9 84.5

HgO(s) (red) –90.83 –58.5 70.29

HgO(s) (yellow) –90.46 –58.43 71.13

HgCl2(s) –224.3 –178.6 146.0

Hg2Cl2(s) –265.4 –210.7 191.6

HgS(s) (red) –58.16 –50.6 82.4

HgS(s) (black) –53.56 –47.70 88.28

HgSO4(s) –707.51 –594.13 0.00

nitrogen

N2(g) 0 0 191.6

N(g) 472.704 455.5 153.3

NO(g) 90.25 87.6 210.8

NO2(g) 33.2 51.30 240.1

N2O(g) 81.6 103.7 220.0

N2O3(g) 83.72 139.41 312.17

NO

3
(aq) -205.0 -108.7 146.4

N2O4(g) 11.1 99.8 304.4

N2O5(g) 11.3 115.1 355.7

NH3(g) –45.9 –16.5 192.8

Access for free at OpenStax 7 https://chem.libretexts.org/@go/page/43241


Standard Thermodynamic Properties for Selected Substances

Substance ΔH

f
(kJ mol–) ΔG

f
(kJ mol–1) S

298
(J K–1 mol–1)

NH
+
4
(aq) -132.5 -79.31 113.4

N2H4(l) 50.63 149.43 121.21

N2H4(g) 95.4 159.4 238.5

NH4NO3(s) –365.56 –183.87 151.08

NH4Cl(s) –314.43 –202.87 94.6

NH4Br(s) –270.8 –175.2 113.0

NH4I(s) –201.4 –112.5 117.0

NH4NO2(s) –256.5 — —

HNO3(l) –174.1 –80.7 155.6

HNO3(g) –133.9 –73.5 266.9

oxygen

O2(g) 0 0 205.2

O(g) 249.17 231.7 161.1

O3(g) 142.7 163.2 238.9

phosphorus

P4(s) 0 0 164.4

P4(g) 58.91 24.4 280.0

P(g) 314.64 278.25 163.19

PH3(g) 5.4 13.5 210.2

PCl3(g) –287.0 –267.8 311.78

PCl5(g) –374.9 –305.0 364.4

P4O6(s) –1640.1 — —

P4O10(s) –2984.0 –2697.0 228.86

PO
3 −
4
(aq) -1277 -1019 -222

HPO3(s) –948.5 — —

HPO
2 −
4
(aq) -1292.1 -1089.3 -33

H PO
2

4
(aq) -1296.3 -1130.4 90.4

H3PO2(s) –604.6 — —

H3PO3(s) –964.4 — —

H3PO4(s) –1279.0 –1119.1 110.50

H3PO4(l) –1266.9 –1124.3 110.5

H4P2O7(s) –2241.0 — —

POCl3(l) –597.1 –520.8 222.5

POCl3(g) –558.5 –512.9 325.5

potassium

K(s) 0 0 64.7

K(g) 89.0 60.5 160.3

Access for free at OpenStax 8 https://chem.libretexts.org/@go/page/43241


Standard Thermodynamic Properties for Selected Substances

Substance ΔH
f

(kJ mol–) ΔG

f
(kJ mol–1) S

298
(J K–1 mol–1)

K
+
(aq) -252.4 -283.3 102.5

KF(s) –576.27 –537.75 66.57

KCl(s) –436.5 –408.5 82.6

rubidium

Rb
+
(aq) -246 -282.2 124

silicon

Si(s) 0 0 18.8

Si(g) 450.0 405.5 168.0

SiO2(s) –910.7 –856.3 41.5

SiH4(g) 34.3 56.9 204.6

H2SiO3(s) –1188.67 –1092.44 133.89

H4SiO4(s) –1481.14 –1333.02 192.46

SiF4(g) –1615.0 –1572.8 282.8

SiCl4(l) –687.0 –619.8 239.7

SiCl4(g) –662.75 –622.58 330.62

SiC(s, beta cubic) –73.22 –70.71 16.61

SiC(s, alpha hexagonal) –71.55 –69.04 16.48

silver

Ag(s) 0 0 42.55

Ag(g) 284.9 246.0 172.89

Ag
+
(aq) 105.6 77.11 72.68

Ag2O(s) –31.05 –11.20 121.3

AgCl(s) –127.0 –109.8 96.3

Ag2S(s) –32.6 –40.7 144.0

sodium

Na(s) 0 0 51.3

Na(g) 107.5 77.0 153.7

Na
+
(aq) -240.1 -261.9 59

Na2O(s) –414.2 –375.5 75.1

NaCl(s) –411.2 –384.1 72.1

strontium

Sr
2 +
(aq) -545.8 -557.3 -32.6

sulfur

S8(s) (rhombic) 0 0 256.8

S(g) 278.81 238.25 167.82

S
2 −
(aq) 41.8 83.7 22

SO2(g) –296.83 –300.1 248.2

Access for free at OpenStax 9 https://chem.libretexts.org/@go/page/43241


Standard Thermodynamic Properties for Selected Substances

Substance ΔH

f
(kJ mol–) ΔG

f
(kJ mol–1) S

298
(J K–1 mol–1)

SO3(g) –395.72 –371.06 256.76

SO
2 −
4
(aq) -909.3 -744.5 20.1

S O
2
2 −
3
(aq) -648.5 -522.5 67

H2S(g) –20.6 –33.4 205.8

HS

(aq) -17.7 12.6 61.1

H2SO4(l) –813.989 690.00 156.90

HSO

4
(aq) -885.75 -752.87 126.9

H2S2O7(s) –1273.6 — —

SF4(g) –728.43 –684.84 291.12

SF6(g) –1220.5 –1116.5 291.5

SCl2(l) –50 — —

SCl2(g) –19.7 — —

S2Cl2(l) –59.4 — —

S2Cl2(g) –19.50 –29.25 319.45

SOCl2(g) –212.55 –198.32 309.66

SOCl2(l) –245.6 — —

SO2Cl2(l) –394.1 — —

SO2Cl2(g) –354.80 –310.45 311.83

tin

Sn(s) 0 0 51.2

Sn(g) 301.2 266.2 168.5

SnO(s) –285.8 –256.9 56.5

SnO2(s) –577.6 –515.8 49.0

SnCl4(l) –511.3 –440.1 258.6

SnCl4(g) –471.5 –432.2 365.8

titanium

Ti(s) 0 0 30.7

Ti(g) 473.0 428.4 180.3

TiO2(s) –944.0 –888.8 50.6

TiCl4(l) –804.2 –737.2 252.4

TiCl4(g) –763.2 –726.3 353.2

tungsten

W(s) 0 0 32.6

W(g) 849.4 807.1 174.0

WO3(s) –842.9 –764.0 75.9

zinc

Zn(s) 0 0 41.6

Access for free at OpenStax 10 https://chem.libretexts.org/@go/page/43241


Standard Thermodynamic Properties for Selected Substances

Substance ΔH
f

(kJ mol–) ΔG

f
(kJ mol–1) S

298
(J K–1 mol–1)

Zn(g) 130.73 95.14 160.98

Zn
2 +
(aq) -153.9 -147.1 -112.1

ZnO(s) –350.5 –320.5 43.7

ZnCl2(s) –415.1 –369.43 111.5

ZnS(s) –206.0 –201.3 57.7

ZnSO4(s) –982.8 –871.5 110.5

ZnCO3(s) –812.78 –731.57 82.42

complexes

[Co(NH3)4(NO2)2]NO3, cis –898.7 — —

[Co(NH3)4(NO2)2]NO3, trans –896.2 — —

NH4[Co(NH3)2(NO2)4] –837.6 — —

[Co(NH3)6][Co(NH3)2(NO2)4]3 –2733.0 — —

[Co(NH3)4Cl2]Cl, cis –874.9 — —

[Co(NH3)4Cl2]Cl, trans –877.4 — —

[Co(en)2(NO2)2]NO3, cis –689.5 — —

[Co(en)2Cl2]Cl, cis –681.2 — —

[Co(en)2Cl2]Cl, trans –677.4 — —

[Co(en)3](ClO4)3 –762.7 — —

[Co(en)3]Br2 –595.8 — —

[Co(en)3]I2 –475.3 — —

[Co(en)3]I3 –519.2 — —

[Co(NH3)6](ClO4)3 –1034.7 –221.1 615

[Co(NH3)5NO2](NO3)2 –1088.7 –412.9 331

[Co(NH3)6](NO3)3 –1282.0 –524.5 448

[Co(NH3)5Cl]Cl2 –1017.1 –582.5 366.1

[Pt(NH3)4]Cl2 –725.5 — —

[Ni(NH3)6]Cl2 –994.1 — —

[Ni(NH3)6]Br2 –923.8 — —

[Ni(NH3)6]I2 –808.3 — —

This page titled Standard Thermodynamic Properties for Selected Substances is shared under a CC BY 4.0 license and was authored, remixed,
and/or curated by OpenStax via source content that was edited to the style and standards of the LibreTexts platform; a detailed edit history is
available upon request.

Access for free at OpenStax 11 https://chem.libretexts.org/@go/page/43241


The Periodic Table

This page titled The Periodic Table is shared under a CC BY 4.0 license and was authored, remixed, and/or curated by OpenStax via source
content that was edited to the style and standards of the LibreTexts platform; a detailed edit history is available upon request.

Access for free at OpenStax 1 https://chem.libretexts.org/@go/page/43232


Units and Conversion Factors
Units of Length
Units of Length

= 39.37 inches (in.) = 1.094 yards = 10–8 cm (exact, definition) = 10–


meter (m) angstrom (Å) 10
(yd) m (exact, definition)

centimeter (cm) = 0.01 m (exact, definition) yard (yd) = 0.9144 m

millimeter (mm) = 0.001 m (exact, definition) inch (in.) = 2.54 cm (exact, definition)

kilometer (km) = 1000 m (exact, definition) mile (US) = 1.60934 km

Units of Volume
Units of Volume

= 0.001 m3 (exact, definition) = = 32 (US) liquid ounces (exact,


liter (L) 1000 cm3 (exact, definition) = liquid quart (US) definition) = 0.25 (US) gallon
1.057 (US) quarts (exact, definition) = 0.9463 L

= 0.001 L (exact, definition) = 1


milliliter (mL) dry quart = 1.1012 L
cm3 (exact, definition)

= 10–6 L (exact, definition) = 10–3


microliter (μL) cubic foot (US) = 28.316 L
cm3 (exact, definition)

Units of Mass
Units of Mass

gram (g) = 0.001 kg (exact, definition) ounce (oz) (avoirdupois) = 28.35 g

milligram (mg) = 0.001 g (exact, definition) pound (lb) (avoirdupois) = 0.4535924 kg

= 1000 g (exact, definition) = =2000 lb (exact, definition) =


kilogram (kg) ton (short)
2.205 lb 907.185 kg

=1000 kg (exact, definition) = = 2240 lb (exact, definition) =


ton (metric) ton (long)
2204.62 lb 1.016 metric ton

Units of Energy
Units of Energy

4.184 joule (J) = 1 thermochemical calorie (cal)

1 thermochemical calorie (cal) = 4.184 × 107 erg

erg = 10–7 J (exact, definition)

electron-volt (eV) = 1.60218 × 10−19 J = 23.061 kcal mol−1

liter·atmosphere = 24.217 cal = 101.325 J (exact, definition)

nutritional calorie (Cal) = 1000 cal (exact, definition) = 4184 J

British thermal unit (BTU) = 1054.804 J1

Units of Pressure
Units of Pressure

torr = 1 mm Hg (exact, definition)

pascal (Pa) = N m–2 (exact, definition) = kg m–1 s–2 (exact, definition)

= 760 mm Hg (exact, definition) = 760 torr (exact, definition) = 101,325


atmosphere (atm)
N m–2 (exact, definition) = 101,325 Pa (exact, definition)

bar = 105 Pa (exact, definition) = 105 kg m–1 s–2 (exact, definition)

Access for free at OpenStax 1 https://chem.libretexts.org/@go/page/43235


Footnotes
1. 1 BTU is the amount of energy needed to heat one pound of water by one degree Fahrenheit. Therefore, the exact relationship
of BTU to joules and other energy units depends on the temperature at which BTU is measured. 59 °F (15 °C) is the most
widely used reference temperature for BTU definition in the United States. At this temperature, the conversion factor is the one
provided in this table.

This page titled Units and Conversion Factors is shared under a CC BY 4.0 license and was authored, remixed, and/or curated by OpenStax via
source content that was edited to the style and standards of the LibreTexts platform; a detailed edit history is available upon request.

Access for free at OpenStax 2 https://chem.libretexts.org/@go/page/43235


Water Properties
Water Density (kg/m3) at Different Temperatures (°C)
Temperature1 Density

0 999.8395

4 999.9720 (density maximum)

10 999.7026

15 999.1026

20 998.2071

22 997.7735

25 997.0479

30 995.6502

40 992.2

60 983.2

80 971.8

100 958.4

Water Vapor Pressure at Different Temperatures (°C)


Temperature Vapor Pressure (torr) Vapor Pressure (Pa)

0 4.6 613.2812

4 6.1 813.2642

10 9.2 1226.562

15 12.8 1706.522

20 17.5 2333.135

Access for free at OpenStax 1 https://chem.libretexts.org/@go/page/43237


Temperature Vapor Pressure (torr) Vapor Pressure (Pa)

22 19.8 2639.776

25 23.8 3173.064

30 31.8 4239.64

35 42.2 5626.188

40 55.3 7372.707

45 71.9 9585.852

50 92.5 12332.29

55 118.0 15732

60 149.4 19918.31

65 187.5 24997.88

70 233.7 31157.35

75 289.1 38543.39

80 355.1 47342.64

85 433.6 57808.42

90 525.8 70100.71

95 633.9 84512.82

100 760.0 101324.7

Water Kw and pKw at Different Temperatures (°C)


Temperature Kw 10–14 pKw2

0 0.112 14.95

5 0.182 14.74

10 0.288 14.54

15 0.465 14.33

20 0.671 14.17

Access for free at OpenStax 2 https://chem.libretexts.org/@go/page/43237


Temperature Kw 10–14 pKw2

25 0.991 14.00

30 1.432 13.84

35 2.042 13.69

40 2.851 13.55

45 3.917 13.41

50 5.297 13.28

55 7.080 13.15

60 9.311 13.03

75 19.95 12.70

100 56.23 12.25

Specific Heat Capacity for Water


Specific Heat Capacity for Water

C°(H2O(l)) = 4184 J·K−1·kg−1 = 4.184 J·g-1·°C-1


C°(H2O(s)) = 1864 J·K−1·kg−1
C°(H2O(g)) = 2093 J·K−1·kg−1

Standard Water Melting and Boiling Temperatures and Enthalpies of the Transitions
Standard Water Melting and Boiling Temperatures and Enthalpies of the Transitions

Temperature (K) ΔH (kJ/mol)

melting 273.15 6.088

boiling 373.15 40.656 (44.016 at 298 K)

Water Cryoscopic (Freezing Point Depression) and Ebullioscopic (Boiling Point Elevation) Constants
Water Cryoscopic (Freezing Point Depression) and Ebullioscopic (Boiling Point Elevation) Constants

Kf = 1.86°C·kg·mol−1 (cryoscopic constant)


Kb = 0.51°C·kg·mol−1 (ebullioscopic constant)

Access for free at OpenStax 3 https://chem.libretexts.org/@go/page/43237


<figure class="ui-has-child-figcaption" >

Water full-range spectral absorption curve. This curve shows the full-range spectral absorption for water. The y-axis signifies the
absorption in 1/cm. If we divide 1 by this value, we will obtain the length of the path (in cm) after which the intensity of a light
beam passing through water decays by a factor of the base of the natural logarithm e (e = 2.718281828).

Footnotes
1. 1 Data for t < 0 °C are for supercooled water
2. 2 pKw = –log10(Kw)

This page titled Water Properties is shared under a CC BY 4.0 license and was authored, remixed, and/or curated by OpenStax via source content
that was edited to the style and standards of the LibreTexts platform; a detailed edit history is available upon request.

Access for free at OpenStax 4 https://chem.libretexts.org/@go/page/43237


Index
A amide B
absolute zero 20.4: Amines and Amides balanced equation
9.2: Relating Pressure, Volume, Amount, and amine 4.1: Writing and Balancing Chemical Equations
Temperature - The Ideal Gas Law 20.4: Amines and Amides band of stability
accuracy Amonton's law 21.1: Nuclear Structure and Stability
1.5: Measurement Uncertainty, Accuracy, and 9.2: Relating Pressure, Volume, Amount, and bar
Precision Temperature - The Ideal Gas Law
9.1: Gas Pressure
Acid anhydride Amorphous
barometer
18.4: Structure and General Properties of the 18.3: Structure and General Properties of the
Metalloids 9.1: Gas Pressure
Nonmetals
acid ionization amorphous solid base anhydride
10.5: The Solid State of Matter 18.9: Occurrence, Preparation, and Compounds of
14.1: Brønsted-Lowry Acids and Bases
Oxygen
acid ionization constant amphiphilic
base ionization
14.3: Relative Strengths of Acids and Bases 11.5: Colloids
14.1: Brønsted-Lowry Acids and Bases
acidic amphiprotic
base ionization constant
14.2: pH and pOH 14.1: Brønsted-Lowry Acids and Bases
14.3: Relative Strengths of Acids and Bases
Actinide amphoteric
Basic
2.5: The Periodic Table 14.1: Brønsted-Lowry Acids and Bases
14.2: pH and pOH
Actinide series amplitude
battery
19.1: Properties of Transition Metals and Their 6.1: Electromagnetic Energy
17.5: Batteries and Fuel Cells
Compounds analyte
activated complex becquerel (unit)
4.5: Quantitative Chemical Analysis
21.6: Biological Effects of Radiation
12.5: Collision Theory angular momentum quantum number
activation energy beta decay
6.3: Development of Quantum Theory
21.3: Radioactive Decay
12.5: Collision Theory anion
active electrode beta particle
2.3: Atomic Structure and Symbolism
21.2: Nuclear Equations
17.2: Galvanic Cells anode
actual yield bicarbonate anion
17.2: Galvanic Cells
18.6: Occurrence, Preparation, and Properties of
4.4: Reaction Yields antimatter Carbonates
addition reaction 21.2: Nuclear Equations bidentate ligand
20.1: Hydrocarbons aqueous solution 19.2: Coordination Chemistry of Transition Metals
adhesive force 3.3: Molarity bimolecular reaction
10.2: Properties of Liquids aromatic hydrocarbon 12.6: Reaction Mechanisms
alcohol 20.1: Hydrocarbons binary acid
20.2: Alcohols and Ethers Arrhenius equation 2.7: Chemical Nomenclature
aldehyde 12.5: Collision Theory binary compound
20.3: Aldehydes, Ketones, Carboxylic Acids, and atmosphere 2.7: Chemical Nomenclature
Esters
9.1: Gas Pressure binding energy per nucleon
alkali metal atom 21.1: Nuclear Structure and Stability
2.5: The Periodic Table
1.2: Phases and Classification of Matter Bismuth
alkaline battery atomic mass 18.1: Periodicity
17.5: Batteries and Fuel Cells
2.3: Atomic Structure and Symbolism blackbody
alkaline earth metal atomic mass unit 6.1: Electromagnetic Energy
2.5: The Periodic Table
2.3: Atomic Structure and Symbolism Bohr's model of the hydrogen atom
18.1: Periodicity
alkane atomic number 6.2: The Bohr Model
2.3: Atomic Structure and Symbolism boiling point
20.1: Hydrocarbons
alkene atomic orbital 10.3: Phase Transitions
6.3: Development of Quantum Theory boiling point elevation
20.1: Hydrocarbons
alkyl group aufbau principle 11.4: Colligative Properties
6.4: Electronic Structure of Atoms (Electron boiling point elevation constant
20.1: Hydrocarbons
Configurations)
alkyne autoionization
11.4: Colligative Properties
20.1: Hydrocarbons bomb calorimeter
14.1: Brønsted-Lowry Acids and Bases
Allotropes average rate
5.2: Calorimetry
18.1: Periodicity bond angle
12.1: Chemical Reaction Rates
alloy Avogadro's law
7.6: Molecular Structure and Polarity
11.1: The Dissolution Process bond dipole moment
9.2: Relating Pressure, Volume, Amount, and
alpha decay Temperature - The Ideal Gas Law 7.6: Molecular Structure and Polarity
21.3: Radioactive Decay Avogadro's number bond distance
alpha particle 3.1: Formula Mass and the Mole Concept 7.6: Molecular Structure and Polarity
2.2: Evolution of Atomic Theory axial position bond energy
21.2: Nuclear Equations 7.5: Strengths of Ionic and Covalent Bonds
7.6: Molecular Structure and Polarity

1 https://chem.libretexts.org/@go/page/278609
bond length chemical equation coordinate covalent bond
7.2: Covalent Bonding 4.1: Writing and Balancing Chemical Equations 15.2: Lewis Acids and Bases
borate chemical property coordination compound
18.3: Structure and General Properties of the 1.3: Physical and Chemical Properties 19.1: Properties of Transition Metals and Their
Metalloids Chemical Reduction Compounds
Boyle's Law 19.2: Coordination Chemistry of Transition Metals
18.2: Occurrence and Preparation of the
9.2: Relating Pressure, Volume, Amount, and Representative Metals coordination number
Temperature - The Ideal Gas Law chemical symbol 10.6: Lattice Structures in Crystalline Solids
Bragg equation 19.2: Coordination Chemistry of Transition Metals
2.3: Atomic Structure and Symbolism
10.6: Lattice Structures in Crystalline Solids chemical thermodynamics coordination sphere
buffer 19.2: Coordination Chemistry of Transition Metals
5.3: Enthalpy
14.6: Buffers chemistry core electron
Buffer Capacity 6.4: Electronic Structure of Atoms (Electron
1.1: Chemistry in Context
Configurations)
14.6: Buffers Chemotherapy
Buret corrosion
21.5: Uses of Radioisotopes
17.6: Corrosion
4.5: Quantitative Chemical Analysis circuit covalent bond
17.1: Balancing Oxidation-Reduction Reactions
2.6: Molecular and Ionic Compounds
C cis configuration 7.2: Covalent Bonding
calorie 19.2: Coordination Chemistry of Transition Metals covalent compound
5.1: Energy Basics coefficient 2.6: Molecular and Ionic Compounds
calorimeter 4.1: Writing and Balancing Chemical Equations covalent network solid
5.2: Calorimetry cohesive force 10.5: The Solid State of Matter
Calorimetry 10.2: Properties of Liquids covalent radius
5.2: Calorimetry colligative property 6.5: Periodic Variations in Element Properties
capillary action 11.4: Colligative Properties crenation
10.2: Properties of Liquids collision theory 11.4: Colligative Properties
Carbonate 12.5: Collision Theory critical mass
18.6: Occurrence, Preparation, and Properties of colloid 21.4: Transmutation and Nuclear Energy
Carbonates 11.5: Colloids critical point
carbonyl group combustion analysis 10.4: Phase Diagrams
20.3: Aldehydes, Ketones, Carboxylic Acids, and
Esters
4.5: Quantitative Chemical Analysis crystal field splitting
Combustion Reaction 19.3: Optical and Magnetic Properties of
carboxylic acid Coordination Compounds
4.2: Classifying Chemical Reactions
20.3: Aldehydes, Ketones, Carboxylic Acids, and
Esters common ion effect crystal field theory
15.1: Precipitation and Dissolution 19.3: Optical and Magnetic Properties of
catalyst Coordination Compounds
12.2: Factors Affecting Reaction Rates complete ionic equation
4.1: Writing and Balancing Chemical Equations crystalline solid
cathode 10.5: The Solid State of Matter
17.2: Galvanic Cells complex ion
15.2: Lewis Acids and Bases cubic centimeter
cathodic protection 1.4: Measurements
17.6: Corrosion compound
1.2: Phases and Classification of Matter cubic closest packing
cation 10.6: Lattice Structures in Crystalline Solids
2.3: Atomic Structure and Symbolism compressibility factor
9.6: Non-Ideal Gas Behavior cubic meter
Cell Notation 1.4: Measurements
17.2: Galvanic Cells concentrated
3.3: Molarity curie (unit)
cell potential 21.6: Biological Effects of Radiation
17.2: Galvanic Cells concentration
3.3: Molarity Current
Celsius 17.1: Balancing Oxidation-Reduction Reactions
1.4: Measurements concentration cell
central metal 17.4: The Nernst Equation
condensation D
19.2: Coordination Chemistry of Transition Metals
chain reaction 10.3: Phase Transitions d orbital
21.4: Transmutation and Nuclear Energy conjugate acid 6.3: Development of Quantum Theory

chalcogen 14.1: Brønsted-Lowry Acids and Bases Dalton unit


2.5: The Periodic Table conjugate base 2.3: Atomic Structure and Symbolism

Charles's Law 14.1: Brønsted-Lowry Acids and Bases Dalton's atomic theory
9.2: Relating Pressure, Volume, Amount, and containment system 2.1: Early Ideas in Atomic Theory
Temperature - The Ideal Gas Law 21.4: Transmutation and Nuclear Energy Dalton's Law of Partial Pressure
chelate continuous spectrum 9.3: Stoichiometry of Gaseous Substances, Mixtures,
and Reactions
19.2: Coordination Chemistry of Transition Metals 6.1: Electromagnetic Energy
chelating ligand control rod daughter nuclide
21.3: Radioactive Decay
19.2: Coordination Chemistry of Transition Metals 21.4: Transmutation and Nuclear Energy
chemical change conversion factors density
1.4: Measurements
1.3: Physical and Chemical Properties 1.6: Mathematical Treatment of Measurement
Results

2 https://chem.libretexts.org/@go/page/278609
deposition electromagnetic spectrum exothermic process
10.3: Phase Transitions 6.1: Electromagnetic Energy 5.1: Energy Basics
diffraction electron expansion work
10.6: Lattice Structures in Crystalline Solids 2.2: Evolution of Atomic Theory 5.3: Enthalpy
diffusion Electron Affinity extensive property
9.4: Effusion and Diffusion of Gases 6.5: Periodic Variations in Element Properties 1.3: Physical and Chemical Properties
dilution electron capture external beam radiation therapy
3.3: Molarity 21.3: Radioactive Decay 21.5: Uses of Radioisotopes
dimensional analysis electron configuration
1.6: Mathematical Treatment of Measurement 6.4: Electronic Structure of Atoms (Electron F
Results Configurations)
f orbital
dipole moment electron density 6.3: Development of Quantum Theory
7.6: Molecular Structure and Polarity 6.3: Development of Quantum Theory
Fahrenheit
diprotic acid electron volt 1.6: Mathematical Treatment of Measurement
14.5: Polyprotic Acids 21.1: Nuclear Structure and Stability Results
diprotic base electronegativity Faraday's constant
14.5: Polyprotic Acids 7.2: Covalent Bonding 17.4: The Nernst Equation
dispersed phase electroplating first law of thermodynamics
11.5: Colloids 17.7: Electrolysis 5.3: Enthalpy
dispersion element first transition series
10.1: Intermolecular Forces 1.2: Phases and Classification of Matter 19.1: Properties of Transition Metals and Their
dispersion medium elementary reaction Compounds
11.5: Colloids 12.6: Reaction Mechanisms fissile
disproportionation reaction empirical formula 21.4: Transmutation and Nuclear Energy
18.4: Structure and General Properties of the 2.4: Chemical Formulas fission
Nonmetals empirical formula mass 21.4: Transmutation and Nuclear Energy
dissociation 3.2: Determining Empirical and Molecular Formulas fissionable
11.2: Electrolytes emulsifying agent 21.4: Transmutation and Nuclear Energy
dissociation constant 11.5: Colloids formal charge
15.2: Lewis Acids and Bases emulsion 7.4: Formal Charges and Resonance
dissolved 11.5: Colloids formation constant
3.3: Molarity End point 15.2: Lewis Acids and Bases
donor atom 4.5: Quantitative Chemical Analysis formula mass
19.2: Coordination Chemistry of Transition Metals endothermic process 3.1: Formula Mass and the Mole Concept
double bond 5.1: Energy Basics fourth transition series
7.3: Lewis Symbols and Structures Energy 19.1: Properties of Transition Metals and Their
Downs cell 5.1: Energy Basics Compounds
18.2: Occurrence and Preparation of the Enthalpy Frasch process
Representative Metals 18.10: Occurrence, Preparation, and Properties of
5.3: Enthalpy
dry cell Enthalpy change
Sulfur
17.5: Batteries and Fuel Cells Free radical
5.3: Enthalpy
dynamic equilibrium entropy
7.3: Lewis Symbols and Structures
10.3: Phase Transitions freezing
16.2: Entropy
10.3: Phase Transitions
equatorial position
E 7.6: Molecular Structure and Polarity
freezing point
Effective Nuclear Charge 10.3: Phase Transitions
equilibrium
6.5: Periodic Variations in Element Properties freezing point depression
13.1: Chemical Equilibria
effusion 11.4: Colligative Properties
equilibrium constant
9.4: Effusion and Diffusion of Gases freezing point depression constant
13.2: Equilibrium Constants
eg orbitals 11.4: Colligative Properties
equivalence point
19.3: Optical and Magnetic Properties of frequency
4.5: Quantitative Chemical Analysis
Coordination Compounds 6.1: Electromagnetic Energy
ester
electrical potential frequency factor
20.3: Aldehydes, Ketones, Carboxylic Acids, and
17.1: Balancing Oxidation-Reduction Reactions Esters 12.5: Collision Theory
electrical work ether fuel cell
17.4: The Nernst Equation 20.2: Alcohols and Ethers 17.5: Batteries and Fuel Cells
Electrolysis exact number functional group
17.7: Electrolysis 1.5: Measurement Uncertainty, Accuracy, and 20.1: Hydrocarbons
electrolyte Precision fundamental unit of charge
11.2: Electrolytes excess reactant 2.3: Atomic Structure and Symbolism
electrolytic cell 4.4: Reaction Yields fusion
17.7: Electrolysis excited state 21.4: Transmutation and Nuclear Energy
Electromagnetic Radiation 6.2: The Bohr Model fusion reactor
6.1: Electromagnetic Energy 21.4: Transmutation and Nuclear Energy

3 https://chem.libretexts.org/@go/page/278609
G hole indicator
galvanic cell 10.6: Lattice Structures in Crystalline Solids 4.5: Quantitative Chemical Analysis
17.2: Galvanic Cells homogeneous catalyst induced dipole
galvanized iron 12.7: Catalysis 10.1: Intermolecular Forces
17.6: Corrosion homogeneous equilibria inert electrode
gamma emission 13.2: Equilibrium Constants 17.2: Galvanic Cells
21.3: Radioactive Decay homogeneous mixture inert gas
gamma ray 1.2: Phases and Classification of Matter 2.5: The Periodic Table
21.2: Nuclear Equations Hund's rule inert pair effect
gas 6.4: Electronic Structure of Atoms (Electron 7.1: Ionic Bonding
Configurations) initial rate
1.2: Phases and Classification of Matter
hybrid orbital 12.1: Chemical Reaction Rates
Geiger counter
8.2: Hybrid Atomic Orbitals inner transition metal
21.6: Biological Effects of Radiation
hybridization 2.5: The Periodic Table
gel
8.2: Hybrid Atomic Orbitals instantaneous dipole
11.5: Colloids
hydrocarbon 10.1: Intermolecular Forces
geometric isomers
5.3: Enthalpy instantaneous rate
19.3: Optical and Magnetic Properties of
Coordination Compounds hydrogen bonding 12.1: Chemical Reaction Rates
Gibbs Free Energy 10.1: Intermolecular Forces integrated rate law
16.4: Gibbs Energy hydrogen carbonate 12.4: Integrated Rate Laws
goiter 18.6: Occurrence, Preparation, and Properties of intensity
Carbonates
2.3: Atomic Structure and Symbolism 6.1: Electromagnetic Energy
hydrogen halide intensive property
Graham's law
18.5: Occurrence, Preparation, and Compounds of
9.4: Effusion and Diffusion of Gases 1.3: Physical and Chemical Properties
Hydrogen
gravimetric analysis hydrogen sulfate interference pattern
4.5: Quantitative Chemical Analysis 6.1: Electromagnetic Energy
18.9: Occurrence, Preparation, and Compounds of
gray (unit) Oxygen interhalogen
21.6: Biological Effects of Radiation hydrogen sulfite 18.12: Occurrence, Preparation, and Properties of the
ground state Noble Gases
18.9: Occurrence, Preparation, and Compounds of
6.2: The Bohr Model Oxygen intermediate
group hydrogenation 12.6: Reaction Mechanisms
2.5: The Periodic Table 18.5: Occurrence, Preparation, and Compounds of intermolecular force
Hydrogen 10.1: Intermolecular Forces
H hydrometallurgy internal energy
19.1: Properties of Transition Metals and Their 5.3: Enthalpy
Haber Process Compounds
18.5: Occurrence, Preparation, and Compounds of
internal radiation therapy
hydrostatic pressure 21.5: Uses of Radioisotopes
Hydrogen
9.1: Gas Pressure International System of Units
halide
hydroxide 1.4: Measurements
18.12: Occurrence, Preparation, and Properties of the
Noble Gases 18.9: Occurrence, Preparation, and Compounds of interstitial sites
Oxygen
halogen 10.5: The Solid State of Matter
hypertonic
2.5: The Periodic Table ion
11.4: Colligative Properties
heat 2.3: Atomic Structure and Symbolism
hypervalent molecule
5.1: Energy Basics ion pair
7.3: Lewis Symbols and Structures
Heat capacity 11.4: Colligative Properties
hypothesis
5.1: Energy Basics ionic bond
1.1: Chemistry in Context
Heisenberg Uncertainty Principle 2.6: Molecular and Ionic Compounds
hypotonic 7.1: Ionic Bonding
6.3: Development of Quantum Theory
11.4: Colligative Properties ionic compound
hemolysis
11.4: Colligative Properties 2.6: Molecular and Ionic Compounds
Henry's law
I ionic solid
11.3: Solubility
ideal gas 10.5: The Solid State of Matter
hertz 9.2: Relating Pressure, Volume, Amount, and ionic solvent
Temperature - The Ideal Gas Law
6.1: Electromagnetic Energy 17.7: Electrolysis
ideal gas constant Ionization Energy
Hess's law 9.2: Relating Pressure, Volume, Amount, and
5.3: Enthalpy 6.5: Periodic Variations in Element Properties
Temperature - The Ideal Gas Law
heterogeneous catalyst ideal gas law ionization isomer
12.7: Catalysis 19.2: Coordination Chemistry of Transition Metals
9.2: Relating Pressure, Volume, Amount, and
heterogeneous equilibria Temperature - The Ideal Gas Law ionizing radiation
13.2: Equilibrium Constants ideal solution 21.6: Biological Effects of Radiation
heterogeneous mixture 11.1: The Dissolution Process isoelectronic
1.2: Phases and Classification of Matter immiscible 6.5: Periodic Variations in Element Properties
hexagonal closest packing 11.3: Solubility isomers
10.6: Lattice Structures in Crystalline Solids 2.4: Chemical Formulas

4 https://chem.libretexts.org/@go/page/278609
isomorphous ligand miscible
10.6: Lattice Structures in Crystalline Solids 15.2: Lewis Acids and Bases 11.3: Solubility
isotonic 19.2: Coordination Chemistry of Transition Metals mixture
11.4: Colligative Properties limiting reactant 1.2: Phases and Classification of Matter
isotope 4.4: Reaction Yields molality
2.2: Evolution of Atomic Theory line spectrum 11.4: Colligative Properties
6.1: Electromagnetic Energy molar mass
J Linear 3.1: Formula Mass and the Mole Concept
7.6: Molecular Structure and Polarity molar solubility
joule
5.1: Energy Basics
linkage isomer 15.1: Precipitation and Dissolution
19.2: Coordination Chemistry of Transition Metals molarity
K liquid 3.3: Molarity
1.2: Phases and Classification of Matter mole
Kc
liter 3.1: Formula Mass and the Mole Concept
13.2: Equilibrium Constants
1.4: Measurements mole fraction
kelvin
lithium ion battery 9.3: Stoichiometry of Gaseous Substances, Mixtures,
1.4: Measurements
17.5: Batteries and Fuel Cells and Reactions
ketone
lone pair molecular compound
20.3: Aldehydes, Ketones, Carboxylic Acids, and
Esters 7.3: Lewis Symbols and Structures 2.6: Molecular and Ionic Compounds
kilogram molecular equation
1.4: Measurements M 4.1: Writing and Balancing Chemical Equations
Kinetic Energy macroscopic domain molecular formula
5.1: Energy Basics 1.1: Chemistry in Context 2.4: Chemical Formulas
kinetic molecular theory magic number molecular orbital theory
9.5: The Kinetic-Molecular Theory 21.1: Nuclear Structure and Stability 8.4: Molecular Orbital Theory
Kp magnetic quantum number molecular solid
13.2: Equilibrium Constants 6.3: Development of Quantum Theory 10.5: The Solid State of Matter
manometer Molecular Structure
L 9.1: Gas Pressure 7.4: Formal Charges and Resonance
7.6: Molecular Structure and Polarity
lanthanide mass
1.2: Phases and Classification of Matter
molecularity
2.5: The Periodic Table
12.6: Reaction Mechanisms
lanthanide series mass defect
21.1: Nuclear Structure and Stability
molecule
19.1: Properties of Transition Metals and Their
1.2: Phases and Classification of Matter
Compounds mass number
Lattice Energy 2.3: Atomic Structure and Symbolism
monatomic ion
2.6: Molecular and Ionic Compounds
7.5: Strengths of Ionic and Covalent Bonds mass percentage
law 3.4: Other Units for Solution Concentrations
monodentate
19.2: Coordination Chemistry of Transition Metals
1.1: Chemistry in Context mean free path
law of conservation of matter 9.4: Effusion and Diffusion of Gases
monoprotic acid
14.5: Polyprotic Acids
1.2: Phases and Classification of Matter melting
law of constant composition 10.3: Phase Transitions
multiple equilibrium
15.3: Coupled Equilibria
2.1: Early Ideas in Atomic Theory melting point
law of definite proportions 10.3: Phase Transitions
2.1: Early Ideas in Atomic Theory metal N
Law of Mass Action 2.5: The Periodic Table Nernst Equation
13.2: Equilibrium Constants 18.1: Periodicity 17.4: The Nernst Equation
law of multiple proportions metallic solid net ionic equation
2.1: Early Ideas in Atomic Theory 10.5: The Solid State of Matter 4.1: Writing and Balancing Chemical Equations
Le Chatelier's Principle metalloid neutral
13.3: Shifting Equilibria - Le Chatelier’s Principle 2.5: The Periodic Table 14.2: pH and pOH
18.1: Periodicity neutron
lead acid battery
meter 2.2: Evolution of Atomic Theory
17.5: Batteries and Fuel Cells
1.4: Measurements Nitrate
length
Method of Initial Rates 18.9: Occurrence, Preparation, and Compounds of
1.4: Measurements
12.3: Rate Laws Oxygen
leveling effect of water
microscopic domain nitrogen fixation
14.3: Relative Strengths of Acids and Bases
1.1: Chemistry in Context 18.7: Occurrence, Preparation, and Properties of
Lewis Acid Nitrogen
microstates
15.2: Lewis Acids and Bases noble gas
16.2: Entropy
Lewis base 2.5: The Periodic Table
millicurie (unit)
15.2: Lewis Acids and Bases node
21.6: Biological Effects of Radiation
Lewis structure 6.1: Electromagnetic Energy
milliliter
7.3: Lewis Symbols and Structures 8.1: Valence Bond Theory
1.4: Measurements
Lewis symbol nomenclature
7.3: Lewis Symbols and Structures 2.7: Chemical Nomenclature

5 https://chem.libretexts.org/@go/page/278609
nonelectrolyte oxide physical property
11.2: Electrolytes 18.9: Occurrence, Preparation, and Compounds of 1.3: Physical and Chemical Properties
nonionizing radiation Oxygen pi bond
21.6: Biological Effects of Radiation oxidizing agent 8.1: Valence Bond Theory
nonmetal 4.2: Classifying Chemical Reactions Pidgeon process
2.5: The Periodic Table oxyacid 18.2: Occurrence and Preparation of the
nonpolar covalent bond 2.7: Chemical Nomenclature Representative Metals
14.3: Relative Strengths of Acids and Bases plasma
7.2: Covalent Bonding
nonspontaneous process oxyanion 1.2: Phases and Classification of Matter
2.6: Molecular and Ionic Compounds platinum metals
16.1: Spontaneity
normal boiling point Ozone 19.1: Properties of Transition Metals and Their
18.9: Occurrence, Preparation, and Compounds of Compounds
10.3: Phase Transitions
Oxygen pnictogen
nuclear binding energy
2.5: The Periodic Table
21.1: Nuclear Structure and Stability
P pOH
Nuclear chemistry
p orbital 14.2: pH and pOH
21.1: Nuclear Structure and Stability
nuclear fuel
6.3: Development of Quantum Theory polar covalent bond
pairing energy 7.2: Covalent Bonding
21.4: Transmutation and Nuclear Energy
nuclear moderator
19.3: Optical and Magnetic Properties of polar molecule
Coordination Compounds 7.6: Molecular Structure and Polarity
21.4: Transmutation and Nuclear Energy parent nuclide
nuclear reaction polarizability
21.3: Radioactive Decay 10.1: Intermolecular Forces
21.2: Nuclear Equations partial pressure
Nuclear Reactor polyatomic ion
9.3: Stoichiometry of Gaseous Substances, Mixtures, 2.6: Molecular and Ionic Compounds
21.4: Transmutation and Nuclear Energy and Reactions
nuclear transmutation polydentate ligand
partially miscible
19.2: Coordination Chemistry of Transition Metals
21.4: Transmutation and Nuclear Energy 11.3: Solubility
nucleon polymorph
particle accelerator
18.3: Structure and General Properties of the
21.1: Nuclear Structure and Stability 21.4: Transmutation and Nuclear Energy Metalloids
nucleus parts per billion position of equilibrium
2.2: Evolution of Atomic Theory 3.4: Other Units for Solution Concentrations 13.3: Shifting Equilibria - Le Chatelier’s Principle
nutritional calorie Parts per million Positron
5.2: Calorimetry 3.4: Other Units for Solution Concentrations 21.2: Nuclear Equations
pascal positron emission
O 9.1: Gas Pressure 21.3: Radioactive Decay
Octahedral passivation Potential Energy
7.6: Molecular Structure and Polarity 18.1: Periodicity 5.1: Energy Basics
octahedral hole Pauli exclusion principle pounds per square inch
10.6: Lattice Structures in Crystalline Solids 6.3: Development of Quantum Theory 9.1: Gas Pressure
octet rule percent composition precipitate
7.3: Lewis Symbols and Structures 3.2: Determining Empirical and Molecular Formulas 4.2: Classifying Chemical Reactions
optical isomer percent of ionization Precipitation reaction
19.2: Coordination Chemistry of Transition Metals 14.3: Relative Strengths of Acids and Bases 4.2: Classifying Chemical Reactions
orbital diagram percent yield precision
6.4: Electronic Structure of Atoms (Electron 4.4: Reaction Yields 1.5: Measurement Uncertainty, Accuracy, and
Configurations) period Precision
organic compound 2.5: The Periodic Table pressure
20.1: Hydrocarbons periodic law 9.1: Gas Pressure
osmosis 2.5: The Periodic Table primary battery
11.4: Colligative Properties periodic table 17.5: Batteries and Fuel Cells
osmotic pressure 2.5: The Periodic Table principal quantum number
11.4: Colligative Properties Peroxide 6.3: Development of Quantum Theory
Ostwald process 18.9: Occurrence, Preparation, and Compounds of product
18.9: Occurrence, Preparation, and Compounds of Oxygen 4.1: Writing and Balancing Chemical Equations
Oxygen pH proton
overall reaction order 14.2: pH and pOH 2.2: Evolution of Atomic Theory
12.3: Rate Laws phase diagram PSI (Unit)
overlap 10.4: Phase Diagrams 9.1: Gas Pressure
8.1: Valence Bond Theory photon pure covalent bond
overpotential 6.1: Electromagnetic Energy 7.2: Covalent Bonding
17.7: Electrolysis photosynthesis pure substance
oxidation 18.9: Occurrence, Preparation, and Compounds of 1.2: Phases and Classification of Matter
4.2: Classifying Chemical Reactions Oxygen
Physical change
1.3: Physical and Chemical Properties

6 https://chem.libretexts.org/@go/page/278609
Q representative metal simple cubic structure
Quantitative Analysis 18.1: Periodicity 10.6: Lattice Structures in Crystalline Solids
4.5: Quantitative Chemical Analysis resonance simple cubic unit cell
Quantization 7.4: Formal Charges and Resonance 10.6: Lattice Structures in Crystalline Solids
8.3: Multiple Bonds single bond
6.1: Electromagnetic Energy
resonance forms 7.3: Lewis Symbols and Structures
quantum mechanics
7.4: Formal Charges and Resonance skeletal structure
6.3: Development of Quantum Theory
resonance hybrid 20.1: Hydrocarbons
quantum number
7.4: Formal Charges and Resonance smelting
6.2: The Bohr Model
reversible process 19.1: Properties of Transition Metals and Their
16.2: Entropy Compounds
R reversible reaction solid
radiation absorbed dose 13.1: Chemical Equilibria 1.2: Phases and Classification of Matter
21.6: Biological Effects of Radiation
roentgen equivalent man solubility
radiation dosimeter 21.6: Biological Effects of Radiation 11.3: Solubility
21.6: Biological Effects of Radiation
root mean square velocity solubility product
radiation therapy 9.5: The Kinetic-Molecular Theory 15.1: Precipitation and Dissolution
21.5: Uses of Radioisotopes
rounding solute
radioactive decay 1.5: Measurement Uncertainty, Accuracy, and 3.3: Molarity
21.3: Radioactive Decay Precision solvation
radioactive decay series 11.1: The Dissolution Process
21.3: Radioactive Decay S solvent
radioactive tracer s orbital 3.3: Molarity
21.5: Uses of Radioisotopes 6.3: Development of Quantum Theory sp hybrid orbital
radioactivity sacrificial anode 8.2: Hybrid Atomic Orbitals
21.1: Nuclear Structure and Stability 17.6: Corrosion sp2 hybrid orbital
radiocarbon dating saturated 8.2: Hybrid Atomic Orbitals
21.3: Radioactive Decay 11.3: Solubility sp3 hybrid orbital
radioisotope saturated hydrocarbon 8.2: Hybrid Atomic Orbitals
21.1: Nuclear Structure and Stability 20.1: Hydrocarbons sp3d hybrid orbital
Radiometric Dating scientific method 8.2: Hybrid Atomic Orbitals
21.3: Radioactive Decay 1.1: Chemistry in Context sp3d2 hybrid orbital
Raoult's Law scintillation counter 8.2: Hybrid Atomic Orbitals
11.4: Colligative Properties 21.6: Biological Effects of Radiation space lattice
rare earth element second 10.6: Lattice Structures in Crystalline Solids
19.1: Properties of Transition Metals and Their
Compounds
1.4: Measurements spatial isomers
Second Law of Thermodynamics 2.4: Chemical Formulas
rate constant
12.3: Rate Laws
16.3: The Second and Third Laws of specific heat capacity
Thermodynamics 5.1: Energy Basics
rate expression second transition series
12.1: Chemical Reaction Rates
spectator ion
19.1: Properties of Transition Metals and Their 4.1: Writing and Balancing Chemical Equations
rate law Compounds
spectrochemical series
12.3: Rate Laws secondary battery
19.3: Optical and Magnetic Properties of
rate of diffusion 17.5: Batteries and Fuel Cells Coordination Compounds
9.4: Effusion and Diffusion of Gases selective precipitation spin quantum number
rate of reaction 15.1: Precipitation and Dissolution 6.3: Development of Quantum Theory
12.1: Chemical Reaction Rates semipermeable membrane spontaneous change
reactant 11.4: Colligative Properties 16.1: Spontaneity
4.1: Writing and Balancing Chemical Equations series spontaneous process
reaction mechanism 2.5: The Periodic Table 11.1: The Dissolution Process
12.6: Reaction Mechanisms shell standard cell potential
Reaction order 6.3: Development of Quantum Theory 17.3: Standard Reduction Potentials
12.3: Rate Laws SI units standard conditions of temperature and
Reaction Quotient 1.4: Measurements
pressure
13.2: Equilibrium Constants sievert (unit)
9.2: Relating Pressure, Volume, Amount, and
reactor coolant 21.6: Biological Effects of Radiation Temperature - The Ideal Gas Law
21.4: Transmutation and Nuclear Energy sigma bond standard enthalpy of combustion
reduction 8.1: Valence Bond Theory 5.3: Enthalpy
4.2: Classifying Chemical Reactions Significant figures Standard Enthalpy of Formation
relative biological effectiveness 1.5: Measurement Uncertainty, Accuracy, and 5.3: Enthalpy
21.6: Biological Effects of Radiation Precision
standard entropy
representative element silicate
16.3: The Second and Third Laws of
2.5: The Periodic Table 18.3: Structure and General Properties of the Thermodynamics
18.1: Periodicity Metalloids

7 https://chem.libretexts.org/@go/page/278609
standard entropy change superoxide triprotic acid
16.3: The Second and Third Laws of 18.9: Occurrence, Preparation, and Compounds of 14.5: Polyprotic Acids
Thermodynamics Oxygen Tyndall effect
standard free energy change supersaturated 11.5: Colloids
16.4: Gibbs Energy 11.3: Solubility
standard free energy of formation surface tension U
16.4: Gibbs Energy 10.2: Properties of Liquids
uncertainty
standard hydrogen electrode surroundings 1.5: Measurement Uncertainty, Accuracy, and
17.3: Standard Reduction Potentials 5.2: Calorimetry Precision
standard molar volume symbolic domain unified atomic mass unit
9.2: Relating Pressure, Volume, Amount, and 1.1: Chemistry in Context 2.3: Atomic Structure and Symbolism
Temperature - The Ideal Gas Law system unimolecular reaction
standard reduction potential 5.2: Calorimetry 12.6: Reaction Mechanisms
17.3: Standard Reduction Potentials
unit
standard state T 1.4: Measurements
5.3: Enthalpy
t2g orbitals unit cell
standing wave 19.3: Optical and Magnetic Properties of 10.6: Lattice Structures in Crystalline Solids
6.1: Electromagnetic Energy Coordination Compounds unsaturated
state function temperature 11.3: Solubility
5.3: Enthalpy 5.1: Energy Basics
Steel termolecular reaction V
19.1: Properties of Transition Metals and Their 12.6: Reaction Mechanisms
Compounds vacancy
Tetrahedral 10.5: The Solid State of Matter
stepwise ionization 7.6: Molecular Structure and Polarity
14.5: Polyprotic Acids Valence Bond Theory
tetrahedral hole 8.1: Valence Bond Theory
stoichiometric factor 10.6: Lattice Structures in Crystalline Solids
4.3: Reaction Stoichiometry Valence Electrons
theoretical yield 6.4: Electronic Structure of Atoms (Electron
stoichiometry 4.4: Reaction Yields Configurations)
4.3: Reaction Stoichiometry
theory valence shell
STP 1.1: Chemistry in Context 6.4: Electronic Structure of Atoms (Electron
9.2: Relating Pressure, Volume, Amount, and
Temperature - The Ideal Gas Law
thermal energy Configurations)
5.1: Energy Basics valence shell electron pair repulsion
stress
13.3: Shifting Equilibria - Le Chatelier’s Principle
thermochemistry theory
5.1: Energy Basics 7.6: Molecular Structure and Polarity
strong electrolyte
11.2: Electrolytes
Third Law of Thermodynamics van der Waals equation
16.3: The Second and Third Laws of 9.6: Non-Ideal Gas Behavior
strong nuclear force Thermodynamics
21.1: Nuclear Structure and Stability van der Waals force
third transition series 10.1: Intermolecular Forces
structural formula 19.1: Properties of Transition Metals and Their
2.4: Chemical Formulas Compounds
van't Hoff factor
structural isomers 11.4: Colligative Properties
titrant
2.4: Chemical Formulas 4.5: Quantitative Chemical Analysis
vapor pressure
subcritical mass 10.3: Phase Transitions
titration analysis
21.4: Transmutation and Nuclear Energy 4.5: Quantitative Chemical Analysis
vapor pressure of water
sublimation 9.3: Stoichiometry of Gaseous Substances, Mixtures,
titration curve and Reactions
10.3: Phase Transitions 14.7: Acid-Base Titrations
vaporization
subshell torr 10.3: Phase Transitions
6.3: Development of Quantum Theory 9.1: Gas Pressure
viscosity
substituent trans configuration 10.2: Properties of Liquids
20.1: Hydrocarbons 19.2: Coordination Chemistry of Transition Metals
voltaic cell
substitution reaction transition metal 17.2: Galvanic Cells
20.1: Hydrocarbons 2.5: The Periodic Table
volume
sulfate transmutation reaction 1.4: Measurements
18.9: Occurrence, Preparation, and Compounds of 21.4: Transmutation and Nuclear Energy
Oxygen volume percentage
transuranium element 3.4: Other Units for Solution Concentrations
Sulfite 21.4: Transmutation and Nuclear Energy
18.9: Occurrence, Preparation, and Compounds of VSEPR
Oxygen
trigonal bipyramidal 7.6: Molecular Structure and Polarity
7.6: Molecular Structure and Polarity
superconductor
19.1: Properties of Transition Metals and Their
Trigonal Planar W
Compounds 7.6: Molecular Structure and Polarity
water gas shift reaction
supercritical fluid triple bond
13.2: Equilibrium Constants
10.4: Phase Diagrams 7.3: Lewis Symbols and Structures
Wave
supercritical mass triple point
6.1: Electromagnetic Energy
21.4: Transmutation and Nuclear Energy 10.4: Phase Diagrams

8 https://chem.libretexts.org/@go/page/278609
wavefunction weak electrolyte work
6.3: Development of Quantum Theory 11.2: Electrolytes 5.1: Energy Basics
wavelength weight
6.1: Electromagnetic Energy 1.2: Phases and Classification of Matter

9 https://chem.libretexts.org/@go/page/278609
Glossary
Alpha particle | (α or He or
4
α ) high-energy
4
Atomic orbital | mathematical function that
Accuracy | how closely a measurement aligns with a 2 2

describes the behavior of an electron in an atom (also


correct value helium nucleus; a helium atom that has lost two
electrons and contains two protons and two neutrons called the wavefunction), it can be used to find the
Acid anhydride | compound that reacts with water probability of locating an electron in a specific region
to form an acid or acidic solution Alpha particle (α particle) | positively charged around the nucleus, as well as other dynamical
particle consisting of two protons and two neutrons variables
Acid ionization | reaction involving the transfer of
a proton from an acid to water, yielding hydronium Amide | organic molecule that features a nitrogen Aufbau principle | procedure in which the electron
ions and the conjugate base of the acid atom connected to the carbon atom in a carbonyl group configuration of the elements is determined by
Amine | organic molecule in which a nitrogen atom “building” them in order of atomic numbers, adding
Acid ionization constant (Ka) | equilibrium one proton to the nucleus and one electron to the
constant for the ionization of a weak acid is bonded to one or more alkyl group
proper subshell at a time
Acid-base indicator | organic acid or base whose Amorphous | solid material such as a glass that
does not have a regular repeating component to its
Autoionization | reaction between identical species
color changes depending on the pH of the solution it is yielding ionic products; for water, this reaction
in three-dimensional structure; a solid but not a crystal
involves transfer of protons to yield hydronium and
Acidic | describes a solution in which Amorphous solid | (also, noncrystalline solid) hydroxide ions
+ − solid in which the particles lack an ordered internal
[H O
3
] > [ OH ]
structure Average rate | rate of a chemical reaction computed
as the ratio of a measured change in amount or
Actinide | inner transition metal in the bottom of the Amphiphilic |
molecules possessing both concentration of substance to the time interval over
bottom two rows of the periodic table hydrophobic (nonpolar) and a hydrophilic (polar) parts which the change occurred
Actinide series | (also, actinoid series) actinium Amphiprotic | species that may either gain or lose a Avogadro’s number (NA) | experimentally
and the elements in the second row or the f-block,
proton in a reaction determined value of the number of entities comprising
atomic numbers 89–103
1 mole of substance, equal to 6.022 × 1023 mol−1
Amphoteric | species that can act as either an acid
Activated complex | (also, transition state)
unstable combination of reactant species representing
or a base Axial position | location in a trigonal bipyramidal
geometry in which there is another atom at a 180°
the highest energy state of a reaction system Amplitude | extent of the displacement caused by a angle and the equatorial positions are at a 90° angle
wave (for sinusoidal waves, it is one-half the
Activation energy (Ea) | energy necessary in difference from the peak height to the trough depth, Balanced equation | chemical equation with equal
order for a reaction to take place
and the intensity is proportional to the square of the numbers of atoms for each element in the reactant and
Active electrode | electrode that participates in the amplitude) product
oxidation-reduction reaction of an electrochemical
Analyte | chemical species of interest Band of stability | (also, belt of stability, zone of
cell; the mass of an active electrode changes during the
stability, or valley of stability) region of graph of
oxidation-reduction reaction Angular momentum quantum number (l) | number of protons versus number of neutrons
quantum number distinguishing the different shapes of
Actual yield | amount of product formed in a orbitals; it is also a measure of the orbital angular
containing stable (nonradioactive) nuclides
reaction momentum Bar | (bar or b) unit of pressure; 1 bar = 100,000 Pa
Addition reaction | reaction in which a double Anion | negatively charged atom or molecule Barometer | device used to measure atmospheric
carbon-carbon bond forms a single carbon-carbon (contains more electrons than protons) pressure
bond by the addition of a reactant. Typical reaction for
an alkene. Anode | electrode in an electrochemical cell at which Base anhydride | metal oxide that behaves as a
oxidation occurs; information about the anode is base towards acids
Adhesive force | force of attraction between recorded on the left side of the salt bridge in cell
molecules of different chemical identities notation Base ionization | reaction involving the transfer of
a proton from water to a base, yielding hydroxide ions
Alcohol | organic compound with a hydroxyl group Antimatter | particles with the same mass but and the conjugate acid of the base
(–OH) bonded to a carbon atom opposite properties (such as charge) of ordinary
particles Base ionization constant (Kb) | equilibrium
Alkali metal | element in group 1 constant for the ionization of a weak base
Alkaline earth metal | element in group 2 Aqueous solution | solution for which water is the
solvent Basic | describes a solution in which [H3O+] < [OH−]
Alkaline earth metal | any of the metals Becquerel (Bq) | SI unit for rate of radioactive
(beryllium, magnesium, calcium, strontium, barium, Aromatic hydrocarbon | cyclic molecule
consisting of carbon and hydrogen with delocalized decay; 1 Bq = 1 disintegration/s
and radium) occupying group 2 of the periodic table;
they are reactive, divalent metals that form basic alternating carbon-carbon single and double bonds, Beta (β) decay | breakdown of a neutron into a
oxides resulting in enhanced stability proton, which remains in the nucleus, and an electron,
Arrhenius equation | mathematical relationship which is emitted as a beta particle
Alkane | molecule consisting of only carbon and
hydrogen atoms connected by single (σ) bonds between the rate constant and the activation energy of Beta particle | (β or 0
e or
0
) high-energy
β
a reaction −1 −1

Alkene | molecule consisting of carbon and hydrogen electron


containing at least one carbon-carbon double bond Atmosphere (atm) | unit of pressure; 1 atm =
101,325 Pa
Bicarbonate anion | salt of the hydrogen
carbonate ion, HCO

Alkyl group | substituent, consisting of an alkane 3

missing one hydrogen atom, attached to a larger Atom | smallest particle of an element that can enter
structure into a chemical combination Bidentate ligand | ligand that coordinates to one
central metal through coordinate bonds from two
Alkyne | molecule consisting of carbon and Atomic mass | average mass of atoms of an different atoms
hydrogen containing at least one carbon-carbon triple element, expressed in amu
bond
Binary acid | compound that contains hydrogen and
Atomic mass unit (amu) | (also, unified atomic one other element, bonded in a way that imparts acidic
Allotropes | two or more forms of the same element, mass unit, u, or Dalton, Da) unit of mass equal to
1
properties to the compound (ability to release H+ ions
in the same physical state, with different chemical 12 when dissolved in water)
structures of the mass of a 12C atom
Binary compound | compound containing two
Alloy | solid mixture of a metallic element and one or Atomic number (Z) | number of protons in the different elements.
more additional elements nucleus of an atom
Binding energy per nucleon | total binding
Alpha (α) decay | loss of an alpha particle during energy for the nucleus divided by the number of
radioactive decay nucleons in the nucleus

1 https://chem.libretexts.org/@go/page/279192
Bismuth | heaviest member of group 15; a less Calorie (cal) | unit of heat or other energy; the Circuit | path taken by a current as it flows because
reactive metal than other representative metals amount of energy required to raise 1 gram of water by of an electrical potential difference
1 degree Celsius; 1 cal is defined as 4.184 J
Blackbody | idealized perfect absorber of all cis configuration | configuration of a geometrical
incident electromagnetic radiation; such bodies emit Calorimeter | device used to measure the amount of isomer in which two similar groups are on the same
electromagnetic radiation in characteristic continuous heat absorbed or released in a chemical or physical side of an imaginary reference line on the molecule
spectra called blackbody radiation process
Clausius-Clapeyron equation | mathematical
Body-centered cubic (BCC) solid | crystalline Calorimetry | process of measuring the amount of relationship between the temperature, vapor pressure,
structure that has a cubic unit cell with lattice points at heat involved in a chemical or physical process and enthalpy of vaporization for a substance
the corners and in the center of the cell
Capillary action | flow of liquid within a porous Coefficient | number placed in front of symbols or
Body-centered cubic unit cell | simplest material due to the attraction of the liquid molecules to formulas in a chemical equation to indicate their
repeating unit of a body-centered cubic crystal; it is a the surface of the material and to other liquid relative amount
cube containing lattice points at each corner and in the molecules
center of the cube Cohesive force | force of attraction between
Carbonate | salt of the anion CO ; often formed
2−
3
identical molecules
Bohr’s model of the hydrogen atom | by the reaction of carbon dioxide with bases
structural model in which an electron moves around Colligative property | property of a solution that
the nucleus only in circular orbits, each with a specific Catalyst | substance that increases the rate of a depends only on the concentration of a solute species
allowed radius; the orbiting electron does not normally reaction without itself being consumed by the reaction Collision theory | model that emphasizes the
emit electromagnetic radiation, but does so when energy and orientation of molecular collisions to
Cathode | electrode in an electrochemical cell at
changing from one orbit to another. explain and predict reaction kinetics
which reduction occurs; information about the cathode
Boiling point | temperature at which the vapor is recorded on the right side of the salt bridge in cell Colloid | (also, colloidal dispersion) mixture in
pressure of a liquid equals the pressure of the gas notation which relatively large solid or liquid particles are
above it dispersed uniformly throughout a gas, liquid, or solid
Cathodic protection | method of protecting metal
Boiling point elevation | elevation of the boiling by using a sacrificial anode and effectively making the Color-change interval | range in pH over which
point of a liquid by addition of a solute metal that needs protecting the cathode, thus the color change of an indicator takes place
preventing its oxidation
Boiling point elevation constant | the Combustion analysis | gravimetric technique used
proportionality constant in the equation relating Cation | positively charged atom or molecule to determine the elemental composition of a compound
boiling point elevation to solute molality; also known (contains fewer electrons than protons) via the collection and weighing of its gaseous
as the ebullioscopic constant combustion products
Cell notation | shorthand way to represent the
Bomb calorimeter | device designed to measure reactions in an electrochemical cell Common ion effect | effect on equilibrium when a
the energy change for processes occurring under substance with an ion in common with the dissolved
Cell potential | difference in electrical potential that
conditions of constant volume; commonly used for species is added to the solution; causes a decrease in
arises when dissimilar metals are connected; the
reactions involving solid and gaseous reactants or the solubility of an ionic species, or a decrease in the
driving force for the flow of charge (current) in
products ionization of a weak acid or base
oxidation-reduction reactions
Bond angle | angle between any two covalent bonds Celsius (°C) | unit of temperature; water freezes at 0 Complete ionic equation | chemical equation in
that share a common atom which all dissolved ionic reactants and products,
°C and boils at 100 °C on this scale
Bond dipole moment | separation of charge in a including spectator ions, are explicitly represented by
Central metal | ion or atom to which one or more formulas for their dissociated ions
bond that depends on the difference in
ligands is attached through coordinate covalent bonds
electronegativity and the bond distance represented by Compound | pure substance that can be
partial charges or a vector Chain reaction | repeated fission caused when the decomposed into two or more elements
neutrons released in fission bombard other atoms
Bond distance | (also, bond length) distance Compressibility factor (Z) | ratio of the
between the nuclei of two bonded atoms Chalcogen | element in group 16 experimentally measured molar volume for a gas to its
Bond energy | (also, bond dissociation energy) Chelate | complex formed from a polydentate ligand molar volume as computed from the ideal gas equation
energy required to break a covalent bond in a gaseous attached to a central metal Concentrated | qualitative term for a solution
substance containing solute at a relatively high concentration
Chelating ligand | ligand that attaches to a central
Bond length | distance between the nuclei of two metal ion by bonds from two or more donor atoms Concentration | quantitative measure of the
bonded atoms at which the lowest potential energy is relative amounts of solute and solvent present in a
achieved Chemical change | change producing a different
kind of matter from the original kind of matter solution
Borate | compound containing boron-oxygen bonds, Concentration cell | galvanic cell in which the two
typically with clusters or chains as a part of the Chemical equation | symbolic representation of a
half-cells are the same except for the concentration of
chemical structure chemical reaction
the solutes; spontaneous when the overall reaction is
Born-Haber cycle | thermochemical cycle relating Chemical property | behavior that is related to the the dilution of the solute
the various energetic steps involved in the formation of change of one kind of matter into another kind of
matter Condensation | change from a gaseous to a liquid
an ionic solid from the relevant elements state
Bragg equation | equation that relates the angles at Chemical reduction | method of preparing a
representative metal using a reducing agent Conjugate acid | substance formed when a base
which X-rays are diffracted by the atoms within a gains a proton
crystal Chemical symbol | one-, two-, or three-letter
abbreviation used to represent an element or its atoms
Conjugate base | substance formed when an acid
Brønsted-Lowry acid | proton donor loses a proton
Brønsted-Lowry base | proton acceptor Chemical thermodynamics | area of science that
deals with the relationships between heat, work, and Containment system | (also, shield) a three-part
Buffer | mixture of a weak acid or a weak base and all forms of energy associated with chemical and structure of materials that protects the exterior of a
the salt of its conjugate; the pH of a buffer resists physical processes nuclear fission reactor and operating personnel from
change when small amounts of acid or base are added the high temperatures, pressures, and radiation levels
Chemistry | study of the composition, properties, inside the reactor
Buffer capacity | amount of an acid or base that and interactions of matter
can be added to a volume of a buffer solution before its Continuous spectrum | electromagnetic radiation
pH changes significantly (usually by one pH unit) Chemotherapy | similar to internal radiation given off in an unbroken series of wavelengths (e.g.,
therapy, but chemical rather than radioactive white light from the sun)
Buret | device used for the precise delivery of substances are introduced into the body to kill cancer
variable liquid volumes, such as in a titration analysis cells Control rod | material inserted into the fuel
assembly that absorbs neutrons and can be raised or
Chlor-alkali process | electrolysis process for the lowered to adjust the rate of a fission reaction
synthesis of chlorine and sodium hydroxide

2 https://chem.libretexts.org/@go/page/279192
Coordination compound | stable compound in d-block element | one of the elements in groups 3– Effective nuclear charge | charge that leads to
which the central metal atom or ion acts as a Lewis 11 with valence electrons in d orbitals the Coulomb force exerted by the nucleus on an
acid and accepts one or more pairs of electrons electron, calculated as the nuclear charge minus
Dalton (Da) | alternative unit equivalent to the shielding
Coordination compound | substance consisting atomic mass unit
of atoms, molecules, or ions attached to a central atom eg orbitals | set of two d orbitals that are oriented on
through Lewis acid-base interactions Dalton’s atomic theory | set of postulates that the Cartesian axes for coordination complexes; in
established the fundamental properties of atoms octahedral complexes, they are higher in energy than
Coordination number | number of atoms closest the t2g orbitals
to any given atom in a crystal or to the central metal
Daughter nuclide | nuclide produced by the
radioactive decay of another nuclide; may be stable or
atom in a complex Electrical potential | energy per charge; in
may decay further
electrochemical systems, it depends on the way the
Coordination number | number of coordinate charges are distributed within the system; the SI unit of
covalent bonds to the central metal atom in a complex Density | ratio of mass to volume for a substance or
or the number of closest contacts to an atom in a object electrical potential is the volt (1 V = 1
J
)
C
crystalline form Deposition | change from a gaseous state directly to
a solid state Electrical work (wele) | negative of total charge
Coordination sphere | central metal atom or ion
plus the attached ligands of a complex Diffraction | redirection of electromagnetic times the cell potential; equal to wmax for the system,
radiation that occurs when it encounters a physical and so equals the free energy change (ΔG)
Core electron | electron in an atom that occupies
barrier of appropriate dimensions Electrolyte | substance that produces ions when
the orbitals of the inner shells
dissolved in water
Corrosion | degradation of metal through an Dilute | qualitative term for a solution containing
electrochemical process solute at a relatively low concentration Electromagnetic radiation | energy transmitted
Dilution | process of adding solvent to a solution in by waves that have an electric-field component and a
Covalent bond | attractive force between the nuclei magnetic-field component
order to lower the concentration of solutes
of a molecule’s atoms and pairs of electrons between
the atoms Dimensional analysis | (also, factor-label method) Electromagnetic spectrum | range of energies
that electromagnetic radiation can comprise, including
Covalent bond | bond formed when electrons are versatile mathematical approach that can be applied to
radio, microwaves, infrared, visible, ultraviolet, X-
shared between atoms computations ranging from simple unit conversions to
rays, and gamma rays; since electromagnetic radiation
more complex, multi-step calculations involving
energy is proportional to the frequency and inversely
Covalent compound | (also, molecular several different quantities
proportional to the wavelength, the spectrum can also
compound) composed of molecules formed by atoms
of two or more different elements
Dipole moment | property of a molecule that be specified by ranges of frequencies or wavelengths
describes the separation of charge determined by the
Covalent network solid | solid whose particles sum of the individual bond moments based on the Electron | negatively charged, subatomic particle of
are held together by covalent bonds molecular structure relatively low mass located outside the nucleus

Covalent radius | one-half the distance between Dipole-dipole attraction | intermolecular Electron affinity | energy required to add an
the nuclei of two identical atoms when they are joined attraction between two permanent dipoles electron to a gaseous atom to form an anion
by a covalent bond Electron capture | combination of a core electron
Diprotic acid | acid containing two ionizable
with a proton to yield a neutron within the nucleus
Crenation | process whereby biological cells hydrogen atoms per molecule. A diprotic acid ionizes
become shriveled due to loss of water by osmosis in two steps Electron configuration | electronic structure of
Critical mass | amount of fissionable material that Diprotic base | base capable of accepting two an atom in its ground state given as a listing of the
will support a self-sustaining (nuclear fission) chain protons. The protons are accepted in two steps orbitals occupied by the electrons
reaction Electron density | a measure of the probability of
Dispersed phase | substance present as relatively
Critical point | temperature and pressure above large solid or liquid particles in a colloid locating an electron in a particular region of space, it is
which a gas cannot be condensed into a liquid equal to the squared absolute value of the wave
Dispersion force | (also, London dispersion force) function ψ
Crystal field splitting (Δoct) | difference in attraction between two rapidly fluctuating, temporary
energy between the t2g and eg sets or t and e sets of dipoles; significant only when particles are very close
Electron volt (eV) | measurement unit of nuclear
binding energies, with 1 eV equaling the amount
orbitals together
energy due to the moving an electron across an electric
Crystal field theory | model that explains the Dispersion medium | solid, liquid, or gas in which potential difference of 1 volt
energies of the orbitals in transition metals in terms of colloidal particles are dispersed
electrostatic interactions with the ligands but does not Electron-pair geometry | arrangement around a
include metal ligand bonding Disproportionation reaction | chemical reaction central atom of all regions of electron density (bonds,
where a single reactant is simultaneously reduced and lone pairs, or unpaired electrons)
Crystalline solid | solid in which the particles are oxidized; it is both the reducing agent and the
arranged in a definite repeating pattern oxidizing agent
Electronegativity | tendency of an atom to attract
electrons in a bond to itself
Cubic centimeter (cm3 or cc) | volume of a Dissociation | physical process accompanying the
cube with an edge length of exactly 1 cm dissolution of an ionic compound in which the Element | substance that is composed of a single
compound’s constituent ions are solvated and type of atom; a substance that cannot be decomposed
Cubic closest packing (CCP) | crystalline dispersed throughout the solution by a chemical change
structure in which planes of closely packed atoms or
ions are stacked as a series of three alternating layers Dissolved | describes the process by which solute Empirical formula | formula showing the
of different relative orientations (ABC) components are dispersed in a solvent composition of a compound given as the simplest
whole-number ratio of atoms
Cubic meter (m3) | SI unit of volume Donor atom | atom in a ligand with a lone pair of
electrons that forms a coordinate covalent bond to a
Empirical formula mass | sum of average atomic
Curie (Ci) | larger unit for rate of radioactive decay central metal
masses for all atoms represented in an empirical
frequently used in medicine; 1 Ci = 3.7 × 1010 formula
disintegrations/s Double bond | covalent bond in which two pairs of
electrons are shared between two atoms Emulsifying agent | amphiphilic substance used to
Current | flow of electrical charge; the SI unit of stabilize the particles of some emulsions
charge is the coulomb (C) and current is measured in Downs cell | electrochemical cell used for the
C commercial preparation of metallic sodium (and
Emulsion | colloid formed from immiscible liquids
amperes (1 A = 1 )
s chlorine) from molten sodium chloride End point | measured volume of titrant solution that
Dynamic equilibrium | state of a system in which yields the change in sample solution appearance or
d orbital | region of space with high electron density other property expected for stoichiometric equivalence
that is either four lobed or contains a dumbbell and reciprocal processes are occurring at equal rates
(see equivalence point)
torus shape; describes orbitals with l = 2. An electron
in this orbital is called a d electron Endothermic process | chemical reaction or
physical change that absorbs heat

3 https://chem.libretexts.org/@go/page/279192
Energy | capacity to supply heat or do work Fissile (or fissionable) | when a material is Geometric isomers | isomers that differ in the way
capable of sustaining a nuclear fission reaction in which atoms are oriented in space relative to each
Enthalpy (H) | sum of a system’s internal energy other, leading to different physical and chemical
and the mathematical product of its pressure and Fission | splitting of a heavier nucleus into two or properties
volume more lighter nuclei, usually accompanied by the
conversion of mass into large amounts of energy Gibbs free energy change (G) | thermodynamic
Enthalpy change (ΔH) | heat released or property defined in terms of system enthalpy and
absorbed by a system under constant pressure during a Formal charge | charge that would result on an entropy; all spontaneous processes involve a decrease
chemical or physical process atom by taking the number of valence electrons on the in G
neutral atom and subtracting the nonbonding electrons
Entropy (S) | state function that is a measure of the and the number of bonds (one-half of the bonding Gravimetric analysis | quantitative chemical
matter and/or energy dispersal within a system,
electrons) analysis method involving the separation of an analyte
determined by the number of system microstates often
from a sample by a physical or chemical process and
described as a measure of the disorder of the system Formula mass | sum of the average masses for all subsequent mass measurements of the analyte, reaction
atoms represented in a chemical formula; for covalent
Equatorial position | one of the three positions in product, and/or sample
compounds, this is also the molecular mass
a trigonal bipyramidal geometry with 120° angles Gray (Gy) | SI unit for measuring radiation dose; 1
between them; the axial positions are located at a 90° Fourth transition series | transition elements in Gy = 1 J absorbed/kg tissue
angle the seventh period of the periodic table (fourth row of
the d-block), atomic numbers 89 and 104–111 Ground state | state in which the electrons in an
Equilibrium | in chemical reactions, the state in atom, ion, or molecule have the lowest energy possible
which the conversion of reactants into products and the Frasch process | important in the mining of free
conversion of products back into reactants occur sulfur from enormous underground deposits Group | vertical column of the periodic table
simultaneously at the same rate; state of balance
Free radical | molecule that contains an odd Haber process | main industrial process used to
Equilibrium constant (K) | value of the reaction number of electrons produce ammonia from nitrogen and hydrogen;
quotient for a system at equilibrium involves the use of an iron catalyst and elevated
Freezing | change from a liquid state to a solid state temperatures and pressures
Equivalence point | volume of titrant solution
required to react completely with the analyte in a
Freezing point | temperature at which the solid and Half-life (t1/2) | time required for half of the atoms
liquid phases of a substance are in equilibrium; see
titration analysis; provides a stoichiometric amount of in a radioactive sample to decay
also melting point
titrant for the sample’s analyte according to the
titration reaction
Half-life of a reaction (tl/2) | time required for
Freezing point depression | lowering of the half of a given amount of reactant to be consumed
Ether | organic compound with an oxygen atom that freezing point of a liquid by addition of a solute
is bonded to two carbon atoms Half-reaction method | method that produces a
Freezing point depression constant | (also, balanced overall oxidation-reduction reaction by
cryoscopic constant) proportionality constant in the
Exact number | number derived by counting or by splitting the reaction into an oxidation “half” and
equation relating freezing point depression to solute
definition reduction “half,” balancing the two half-reactions, and
molality
then combining the oxidation half-reaction and
Excess reactant | reactant present in an amount reduction half-reaction in such a way that the number
greater than required by the reaction stoichiometry Frequency (ν ) | number of wave cycles (peaks or
troughs) that pass a specified point in space per unit of electrons generated by the oxidation is exactly
Excited state | state having an energy greater than time canceled by the number of electrons required by the
the ground-state energy reduction
Frequency factor (A) | proportionality constant in
Exothermic process | chemical reaction or the Arrhenius equation, related to the relative number Halide | compound containing an anion of a group 17
physical change that releases heat of collisions having an orientation capable of leading element in the 1− oxidation state (fluoride, F−;
to product formation chloride, Cl−; bromide, Br−; and iodide, I−)
Expansion work (pressure-volume work) |
work done as a system expands or contracts against Functional group | part of an organic molecule Hall–Héroult cell | electrolysis apparatus used to
external pressure that imparts a specific chemical reactivity to the isolate pure aluminum metal from a solution of
molecule alumina in molten cryolite
Extensive property | property of a substance that
depends on the amount of the substance Fundamental unit of charge | (also called the Halogen | element in group 17
elementary charge) equals the magnitude of the charge Heat (q) | transfer of thermal energy between two
External beam radiation therapy | radiation
of an electron (e) with e = 1.602 × 10−19 C bodies
delivered by a machine outside the body
f orbital | multilobed region of space with high Fusion | combination of very light nuclei into heavier Heat capacity (C) | extensive property of a body of
electron density, describes orbitals with l = 3. An nuclei, accompanied by the conversion of mass into matter that represents the quantity of heat required to
electron in this orbital is called an f electron large amounts of energy increase its temperature by 1 degree Celsius (or 1
Fusion reactor | nuclear reactor in which fusion kelvin)
f-block element | (also, inner transition element)
reactions of light nuclei are controlled Heisenberg uncertainty principle | rule stating
one of the elements with atomic numbers 58–71 or 90–
103 that have valence electrons in f orbitals; they are Galvanic cell | electrochemical cell that involves a that it is impossible to exactly determine both certain
frequently shown offset below the periodic table spontaneous oxidation-reduction reaction; conjugate dynamical properties such as the momentum
electrochemical cells with positive cell potentials; also and the position of a particle at the same time. The
Face-centered cubic (FCC) solid | crystalline uncertainty principle is a consequence of quantum
structure consisting of a cubic unit cell with lattice called a voltaic cell
particles exhibiting wave–particle duality
points on the corners and in the center of each face Galvanized iron | method for protecting iron by
covering it with zinc, which will oxidize before the Hemolysis | rupture of red blood cells due to the
Face-centered cubic unit cell | simplest accumulation of excess water by osmosis
repeating unit of a face-centered cubic crystal; it is a iron; zinc-plated iron
cube containing lattice points at each corner and in the Gamma (γ) emission | decay of an excited-state Henderson-Hasselbalch equation | equation
center of each face nuclide accompanied by emission of a gamma ray used to calculate the pH of buffer solutions
Fahrenheit | unit of temperature; water freezes at 32 Gamma ray | (γ or ) short wavelength, high-
0
γ
Henry’s law | law stating the proportional
relationship between the concentration of dissolved
0
°F and boils at 212 °F on this scale energy electromagnetic radiation that exhibits wave-
gas in a solution and the partial pressure of the gas in
Faraday’s constant (F) | charge on 1 mol of particle duality
contact with the solution
electrons; F = 96,485 C/mol e− Gas | state in which matter has neither definite
volume nor shape
Hertz (Hz) | the unit of frequency, which is the
First law of thermodynamics | internal energy number of cycles per second, s−1
of a system changes due to heat flow in or out of the Geiger counter | instrument that detects and
system or work done on or by the system measures radiation via the ionization produced in a
Hess’s law | if a process can be represented as the
sum of several steps, the enthalpy change of the
First transition series | transition elements in the Geiger-Müller tube
process equals the sum of the enthalpy changes of the
fourth period of the periodic table (first row of the d- Gel | colloidal dispersion of a liquid in a solid steps
block), atomic numbers 21–29

4 https://chem.libretexts.org/@go/page/279192
Heterogeneous catalyst | catalyst present in a Immiscible | of negligible mutual solubility; Ionic compound | compound composed of cations
different phase from the reactants, furnishing a surface typically refers to liquid substances and anions combined in ratios, yielding an electrically
at which a reaction can occur neutral substance
Indicator | substance added to the sample in a
Heterogeneous equilibria | equilibria between titration analysis to permit visual detection of the end Ionic solid | solid composed of positive and negative
reactants and products in different phases point ions held together by strong electrostatic attractions
Heterogeneous mixture | combination of Induced dipole | temporary dipole formed when Ionization energy | energy required to remove an
substances with a composition that varies from point to the electrons of an atom or molecule are distorted by electron from a gaseous atom or ion. The associated
point the instantaneous dipole of a neighboring atom or number (e.g., second ionization energy) corresponds to
molecule the charge of the ion produced (X2+)
Hexagonal closest packing (HCP) | crystalline
structure in which close packed layers of atoms or ions Inert electrode | electrode that allows current to Ionization isomer | (or coordination isomer)
are stacked as a series of two alternating layers of flow, but that does not otherwise participate in the isomer in which an anionic ligand is replaced by the
different relative orientations (AB) oxidation-reduction reaction in an electrochemical cell; counter ion in the inner coordination sphere
the mass of an inert electrode does not change during
High-spin complex | complex in which the the oxidation-reduction reaction; inert electrodes are Ionizing radiation | radiation that can cause a
electrons maximize the total electron spin by singly often made of platinum or gold because these metals molecule to lose an electron and form an ion
populating all of the orbitals before pairing two are chemically unreactive.
electrons into the lower-energy orbitals
Isoelectronic | group of ions or atoms that have
identical electron configurations
Inert gas | (also, noble gas) element in group 18
Hole | (also, interstice) space between atoms within a
crystal Inert pair effect | tendency of heavy atoms to form Isomers | compounds with the same chemical
formula but different structures
ions in which their valence s electrons are not lost
Homogeneous catalyst | catalyst present in the
same phase as the reactants Initial rate | instantaneous rate of a chemical Isomorphous | possessing the same crystalline
reaction at t = 0 s (immediately after the reaction has structure
Homogeneous equilibria | equilibria within a begun)
single phase
Isotonic | of equal osmotic pressure
Inner transition metal | (also, lanthanide or Isotopes | atoms that contain the same number of
Homogeneous mixture | (also, solution) actinide) element in the bottom two rows; if in the first
combination of substances with a composition that is protons but different numbers of neutrons
row, also called lanthanide, or if in the second row,
uniform throughout Joule (J) | SI unit of energy; 1 joule is the kinetic
also called actinide
energy of an object with a mass of 2 kilograms moving
Hund’s rule | every orbital in a subshell is singly Instantaneous dipole | temporary dipole that with a velocity of 1 meter per second, 1 J = 1 kg m2/s
occupied with one electron before any one orbital is occurs for a brief moment in time when the electrons and 4.184 J = 1 cal
doubly occupied, and all electrons in singly occupied of an atom or molecule are distributed asymmetrically
orbitals have the same spin Kc | equilibrium constant for reactions based on
Instantaneous rate | rate of a chemical reaction at concentrations of reactants and products
Hydrocarbon | compound composed only of any instant in time, determined by the slope of the line
hydrogen and carbon; the major component of fossil Kelvin (K) | SI unit of temperature; 273.15 K = 0 ºC
tangential to a graph of concentration as a function of
fuels
time Kilogram (kg) | standard SI unit of mass; 1 kg =
Hydrogen bonding | occurs when exceptionally Integrated rate law | equation that relates the approximately 2.2 pounds
strong dipoles attract; bonding that exists when
concentration of a reactant to elapsed time of reaction Kinetic energy | energy of a moving body, in
hydrogen is bonded to one of the three most
electronegative elements: F, O, or N Intensity | property of wave-propagated energy joules, equal to
1
mv
2
(where m = mass and v =
related to the amplitude of the wave, such as 2
Hydrogen carbonate | salt of carbonic acid, brightness of light or loudness of sound velocity)
H2CO3 (containing the anion HCO ) in which one

3

hydrogen atom has been replaced; an acid carbonate; Intensive property | property of a substance that is KP | equilibrium constant for gas-phase reactions
independent of the amount of the substance based on partial pressures of reactants and products
also known as bicarbonate ion
Hydrogen halide | binary compound formed Interference pattern | pattern typically consisting Lanthanide | inner transition metal in the top of the
of alternating bright and dark fringes; it results from bottom two rows of the periodic table
between hydrogen and the halogens: HF, HCl, HBr,
and HI constructive and destructive interference of waves Lanthanide series | (also, lanthanoid series)
Interhalogen | compound formed from two or more lanthanum and the elements in the first row or the f-
Hydrogen sulfate | HSO ion −

4
different halogens block, atomic numbers 57–71
Hydrogen sulfite | HSO ion −

Lattice energy (ΔHlattice) | energy required to


3
Intermolecular force | noncovalent attractive
Hydrogenation | addition of hydrogen (H2) to force between atoms, molecules, and/or ions separate one mole of an ionic solid into its component
reduce a compound gaseous ions
Internal energy (U) | total of all possible kinds of
Hydrometallurgy | process in which a metal is energy present in a substance or substances Law | statement that summarizes a vast number of
separated from a mixture by first converting it into experimental observations, and describes or predicts
soluble ions, extracting the ions, and then reducing the
Internal radiation therapy | (also, some aspect of the natural world
brachytherapy) radiation from a radioactive substance
ions to precipitate the pure metal Law of conservation of matter | when matter
introduced into the body to kill cancer cells
Hydrostatic pressure | pressure exerted by a fluid converts from one type to another or changes form,
Interstitial sites | spaces between the regular there is no detectable change in the total amount of
due to gravity
particle positions in any array of atoms or ions matter present
Hydroxide | compound of a metal with the Ion | electrically charged atom or molecule (contains

hydroxide ion OH or the group −OH Law of constant composition | (also, law of
unequal numbers of protons and electrons) definite proportions) all samples of a pure compound
Hypertonic | of greater osmotic pressure Ion pair | solvated anion/cation pair held together by contain the same elements in the same proportions by
mass
Hypervalent molecule | molecule containing at moderate electrostatic attraction
least one main group element that has more than eight
Ion-dipole attraction | electrostatic attraction Law of definite proportions | (also, law of
electrons in its valence shell constant composition) all samples of a pure compound
between an ion and a polar molecule
contain the same elements in the same proportions by
Hypothesis | tentative explanation of observations
that acts as a guide for gathering and checking
Ion-product constant for water (Kw) | mass
equilibrium constant for the autoionization of water
information Law of mass action | when a reversible reaction
Ionic bond | electrostatic forces of attraction has attained equilibrium at a given temperature, the
Hypotonic | of less osmotic pressure between the oppositely charged ions of an ionic reaction quotient remains constant
Ideal solution | solution that forms with no compound
accompanying energy change Ionic bond | strong electrostatic force of attraction
between cations and anions in an ionic compound

5 https://chem.libretexts.org/@go/page/279192
Law of multiple proportions | when two Mass-energy equivalence equation | Albert Molecular structure | arrangement of atoms in a
elements react to form more than one compound, a Einstein’s relationship showing that mass and energy molecule or ion
fixed mass of one element will react with masses of are equivalent
the other element in a ratio of small whole numbers
Molecular structure | structure that includes only
Mass-volume percent | ratio of solute mass to the placement of the atoms in the molecule
Le Chatelier's principle | when a chemical solution volume, expressed as a percentage
system at equilibrium is disturbed, it returns to Molecule | bonded collection of two or more atoms
equilibrium by counteracting the disturbance Matter | anything that occupies space and has mass of the same or different elements

Length | measure of one dimension of an object Melting | change from a solid state to a liquid state Monatomic ion | ion composed of a single atom
Leveling effect of water | any acid stronger than Melting point | temperature at which the solid and Monodentate | ligand that attaches to a central
H O
+
, or any base stronger than OH− will react with liquid phases of a substance are in equilibrium; see metal through just one coordinate covalent bond
3
also freezing point
water to form H O , or OH−, respectively; water acts
3
+
Monoprotic acid | acid containing one ionizable
as a base to make all strong acids appear equally Metal | element that is shiny, malleable, good hydrogen atom per molecule
strong, and it acts as an acid to make all strong bases conductor of heat and electricity
Multiple equilibrium | system characterized by
appear equally strong Metal (representative) | atoms of the metallic more than one state of balance between a slightly
Lewis structure | diagram showing lone pairs and elements of groups 1, 2, 12, 13, 14, 15, and 16, which soluble ionic solid and an aqueous solution of ions
bonding pairs of electrons in a molecule or an ion form ionic compounds by losing electrons from their working simultaneously
outer s or p orbitals
Lewis symbol | symbol for an element or Nernst equation | equation that relates the
monatomic ion that uses a dot to represent each Metallic solid | solid composed of metal atoms logarithm of the reaction quotient (Q) to nonstandard
valence electron in the element or ion cell potentials; can be used to relate equilibrium
Metalloid | element that conducts heat and constants to standard cell potentials
Ligand | ion or neutral molecule attached to the electricity moderately well, and possesses some
central metal ion in a coordination compound properties of metals and some properties of nonmetals Net ionic equation | chemical equation in which
only those dissolved ionic reactants and products that
Limiting reactant | reactant present in an amount Metalloid | element that has properties that are undergo a chemical or physical change are represented
lower than required by the reaction stoichiometry, thus between those of metals and nonmetals; these elements (excludes spectator ions)
limiting the amount of product generated are typically semiconductors
Neutral | describes a solution in which [H3O+] =
Line spectrum | electromagnetic radiation emitted Meter (m) | standard metric and SI unit of length; 1 [OH−]
at discrete wavelengths by a specific atom (or atoms) m = approximately 1.094 yards
in an excited state Neutron | uncharged, subatomic particle located in
Method of initial rates | use of a more explicit the nucleus
Linear | shape in which two outside groups are algebraic method to determine the orders in a rate law
placed on opposite sides of a central atom Nitrate | NO ion; salt of nitric acid

Microscopic domain | realm of things that are 3

Linkage isomer | coordination compound that much too small to be sensed directly Nitrogen fixation | formation of nitrogen
possesses a ligand that can bind to the transition metal Microstate (W) | possible configuration or compounds from molecular nitrogen
in two different ways (CN− vs. NC−) arrangement of matter and energy within a system Noble gas | (also, inert gas) element in group 18
Liquid | state of matter that has a definite volume but Millicurie (mCi) | larger unit for rate of radioactive Node | any point of a standing wave with zero
indefinite shape decay frequently used in medicine; 1 Ci = 3.7 × 1010 amplitude
Liter (L) | (also, cubic decimeter) unit of volume; 1 disintegrations/s
L = 1,000 cm 3 Node | plane separating different lobes of orbitals,
Milliliter (mL) | 1/1,000 of a liter; equal to 1 cm3 where the probability of finding an electron is zero
Lone pair | two (a pair of) valence electrons that are Miscible | mutually soluble in all proportions; Nomenclature | system of rules for naming objects
not used to form a covalent bond typically refers to liquid substances of interest
Low-spin complex | complex in which the Mixture | matter that can be separated into its
electrons minimize the total electron spin by pairing in
Nonelectrolyte | substance that does not produce
components by physical means ions when dissolved in water
the lower-energy orbitals before populating the higher-
energy orbitals Molality (m) | a concentration unit defined as the Nonionizing radiation | radiation that speeds up
ratio of the numbers of moles of solute to the mass of the movement of atoms and molecules; it is equivalent
Macroscopic domain | realm of everyday things the solvent in kilograms to heating a sample, but is not energetic enough to
that are large enough to sense directly by human sight
Molar mass | mass in grams of 1 mole of a cause the ionization of molecules
and touch
substance Nonmetal | element that appears dull, poor
Magic number | nuclei with specific numbers of conductor of heat and electricity
nucleons that are within the band of stability Molar solubility | solubility of a compound
expressed in units of moles per liter (mol/L) Nonspontaneous process | process that requires
Magnetic quantum number (ml) | quantum
Molarity (M) | unit of concentration, defined as the continual input of energy from an external source
number signifying the orientation of an atomic orbital
around the nucleus; orbitals having different values of number of moles of solute dissolved in 1 liter of Normal boiling point | temperature at which a
ml but the same subshell value of l have the same solution liquid’s vapor pressure equals 1 atm (760 torr)
energy (are degenerate), but this degeneracy can be
removed by application of an external magnetic field
Mole | amount of substance containing the same Nuclear binding energy | energy lost when an
number of atoms, molecules, ions, or other entities as atom’s nucleons are bound together (or the energy
Main-group element | (also, representative the number of atoms in exactly 12 grams of 12C needed to break a nucleus into its constituent protons
element) element in columns 1, 2, and 12–18 and neutrons)
Molecular compound | (also, covalent
Manometer | device used to measure the pressure compound) composed of molecules formed by atoms Nuclear chemistry | study of the structure of
of a gas trapped in a container of two or more different elements atomic nuclei and processes that change nuclear
Molecular equation | chemical equation in which structure
Mass | fundamental property indicating amount of
matter all reactants and products are represented as neutral Nuclear fuel | fissionable isotope present in
substances sufficient quantities to provide a self-sustaining chain
Mass defect | difference between the mass of an reaction in a nuclear reactor
atom and the summed mass of its constituent Molecular formula | formula indicating the
subatomic particles (or the mass “lost” when nucleons composition of a molecule of a compound and giving Nuclear moderator | substance that slows
are brought together to form a nucleus) the actual number of atoms of each element in a neutrons to a speed low enough to cause fission
molecule of the compound.
Mass number (A) | sum of the numbers of Nuclear reaction | change to a nucleus resulting in
neutrons and protons in the nucleus of an atom Molecular solid | solid composed of neutral changes in the atomic number, mass number, or energy
molecules held together by intermolecular forces of state
Mass percentage | ratio of solute-to-solution mass attraction
expressed as a percentage

6 https://chem.libretexts.org/@go/page/279192
Nuclear reactor | environment that produces Pairing energy (P) | energy required to place two Platinum metals | group of six transition metals
energy via nuclear fission in which the chain reaction electrons with opposite spins into a single orbital consisting of ruthenium, osmium, rhodium, iridium,
is controlled and sustained without explosion palladium, and platinum that tend to occur in the same
Parent nuclide | unstable nuclide that changes minerals and demonstrate similar chemical properties
Nuclear transmutation | conversion of one spontaneously into another (daughter) nuclide
nuclide into another nuclide Pnictogen | element in group 15
Partially miscible | of moderate mutual solubility;
Nucleon | collective term for protons and neutrons in typically refers to liquid substances POH | logarithmic measure of the concentration of
a nucleus hydroxide ions in a solution
Particle accelerator | device that uses electric and
Nucleus | massive, positively charged center of an magnetic fields to increase the kinetic energy of nuclei Polar covalent bond | covalent bond between
atom made up of protons and neutrons used in transmutation reactions atoms of different electronegativities; a covalent bond
with a positive end and a negative end
Nuclide | nucleus of a particular isotope Parts per billion (ppb) | ratio of solute-to-
solution mass multiplied by 109 Polar molecule | (also, dipole) molecule with an
Nutritional calorie (Calorie) | unit used for overall dipole moment
quantifying energy provided by digestion of foods, Parts per million (ppm) | ratio of solute-to-
defined as 1000 cal or 1 kcal solution mass multiplied by 106 Polarizability | measure of the ability of a charge to
distort a molecule’s charge distribution (electron
Octahedral | shape in which six outside groups are Pascal (Pa) | SI unit of pressure; 1 Pa = 1 N/m2
cloud)
placed around a central atom such that a three-
dimensional shape is generated with four groups Passivation | metals with a protective nonreactive Polyatomic ion | ion composed of more than one
forming a square and the other two forming the apex film of oxide or other compound that creates a barrier atom
of two pyramids, one above and one below the square for chemical reactions; physical or chemical removal
plane of the passivating film allows the metals to Polydentate ligand | ligand that is attached to a
demonstrate their expected chemical reactivity central metal ion by bonds from two or more donor
Octahedral hole | open space in a crystal at the atoms, named with prefixes specifying how many
center of six particles located at the corners of an
Pauli exclusion principle | specifies that no two donors are present (e.g., hexadentate = six coordinate
electrons in an atom can have the same value for all
octahedron bonds formed)
four quantum numbers
Octet rule | guideline that states main group atoms Polymorph | variation in crystalline structure that
will form structures in which eight valence electrons Percent composition | percentage by mass of the results in different physical properties for the resulting
interact with each nucleus, counting bonding electrons various elements in a compound compound
as interacting with both atoms connected by the bond Percent ionization | ratio of the concentration of Position of equilibrium | concentrations or partial
the ionized acid to the initial acid concentration, times
Optical isomer | (also, enantiomer) molecule that is pressures of components of a reaction at equilibrium
100
a nonsuperimposable mirror image with identical (commonly used to describe conditions before a
chemical and physical properties, except when it reacts Percent yield | measure of the efficiency of a disturbance)
with other optical isomers reaction, expressed as a percentage of the theoretical
yield Positron ( 0
β or 0
) | antiparticle to the
e
Orbital diagram | pictorial representation of the
+1 +1

electron; it has identical properties to an electron,


electron configuration showing each orbital as a box Period | (also, series) horizontal row of the periodic except for having the opposite (positive) charge
and each electron as an arrow table
Organic compound | natural or synthetic Positron emission | (also, β+ decay) conversion of
Periodic law | properties of the elements are a proton into a neutron, which remains in the nucleus,
compound that contains carbon periodic function of their atomic numbers. and a positron, which is emitted
Osmosis | diffusion of solvent molecules through a Periodic table | table of the elements that places
semipermeable membrane
Potential energy | energy of a particle or system of
elements with similar chemical properties close
particles derived from relative position, composition,
together
Osmotic pressure (Π) | opposing pressure or condition
required to prevent bulk transfer of solvent molecules Peroxide | molecule containing two oxygen atoms
through a semipermeable membrane
Pounds per square inch (psi) | unit of pressure
bonded together or as the anion, O
2−

2 common in the US
Ostwald process | industrial process used to PH | logarithmic measure of the concentration of
convert ammonia into nitric acid
Precision | how closely a measurement matches the
hydronium ions in a solution same measurement when repeated
Overall reaction order | sum of the reaction Phase diagram | pressure-temperature graph
orders for each substance represented in the rate law
Pressure | force exerted per unit area
summarizing conditions under which the phases of a
Overlap | coexistence of orbitals from two different substance can exist Principal quantum number (n) | quantum
number specifying the shell an electron occupies in an
atoms sharing the same region of space, leading to the Photon | smallest possible packet of electromagnetic atom
formation of a covalent bond radiation, a particle of light
Oxidation half-reaction | the “half” of an Product | substance formed by a chemical or
Photosynthesis | process whereby light energy physical change; shown on the right side of the arrow
oxidation-reduction reaction involving oxidation; the promotes the reaction of water and carbon dioxide to in a chemical equation
half-reaction in which electrons appear as products; form carbohydrates and oxygen; this allows
balanced when each atom type, as well as the charge, photosynthetic organisms to store energy Proton | positively charged, subatomic particle
is balanced located in the nucleus
Physical change | change in the state or properties
Oxide | binary compound of oxygen with another of matter that does not involve a change in its chemical Pure covalent bond | (also, nonpolar covalent
element or group, typically containing O2− ions or the composition bond) covalent bond between atoms of identical
group –O– or =O electronegativities
Physical property | characteristic of matter that is
Oxyacid | compound that contains hydrogen, not associated with any change in its chemical Pure substance | homogeneous substance that has
oxygen, and one other element, bonded in a way that a constant composition
composition
imparts acidic properties to the compound (ability to
release H+ ions when dissolved in water) Pi bond (π bond) | covalent bond formed by side- Quantitative analysis | the determination of the
by-side overlap of atomic orbitals; the electron density amount or concentration of a substance in a sample
Oxyacid | compound containing a nonmetal and one is found on opposite sides of the internuclear axis
or more hydroxyl groups Quantization | occurring only in specific discrete
Pidgeon process | chemical reduction process used values, not continuous
Oxyanion | polyatomic anion composed of a central to produce magnesium through the thermal reaction of
atom bonded to oxygen atoms Quantum mechanics | field of study that includes
magnesium oxide with silicon quantization of energy, wave-particle duality, and the
Ozone | allotrope of oxygen; O3 Plasma | gaseous state of matter containing a large Heisenberg uncertainty principle to describe matter
p orbital | dumbbell-shaped region of space with number of electrically charged atoms and/or molecules Quantum number | integer number having only
high electron density, describes orbitals with l = 1. An specific allowed values and used to characterize the
electron in this orbital is called a p electron arrangement of electrons in an atom

7 https://chem.libretexts.org/@go/page/279192
Radiation absorbed dose (rad) | SI unit for Reduction half-reaction | the “half” of an Series | (also, period) horizontal row of the period
measuring radiation dose, frequently used in medical oxidation-reduction reaction involving reduction; the table
applications; 1 rad = 0.01 Gy half-reaction in which electrons appear as reactants;
balanced when each atom type, as well as the charge,
Shell | set of orbitals with the same principal quantum
Radiation dosimeter | device that measures is balanced
number, n
ionizing radiation and is used to determine personal
radiation exposure Relative biological effectiveness (RBE) | SI units (International System of Units) |
measure of the relative damage done by radiation standards fixed by international agreement in the
Radiation therapy | use of high-energy radiation International System of Units (Le Système
to damage the DNA of cancer cells, which kills them Representative element | (also, main-group International d’Unités)
or keeps them from dividing element) element in columns 1, 2, and 12–18
Sievert (Sv) | SI unit measuring tissue damage
Radioactive decay | spontaneous decay of an Representative element | element where the s caused by radiation; takes into account energy and
unstable nuclide into another nuclide and p orbitals are filling biological effects of radiation
Radioactive decay series | chains of successive Representative metal | metal among the Sigma bond (σ bond) | covalent bond formed by
disintegrations (radioactive decays) that ultimately representative elements overlap of atomic orbitals along the internuclear axis
lead to a stable end-product
Resonance | situation in which one Lewis structure Significant figures | (also, significant digits) all of
Radioactive tracer | (also, radioactive label) is insufficient to describe the bonding in a molecule the measured digits in a determination, including the
radioisotope used to track or follow a substance by and the average of multiple structures is observed uncertain last digit
monitoring its radioactive emissions
Resonance forms | two or more Lewis structures Silicate | compound containing silicon-oxygen
Radioactivity | phenomenon exhibited by an that have the same arrangement of atoms but different bonds, with silicate tetrahedra connected in rings,
unstable nucleon that spontaneously undergoes change arrangements of electrons sheets, or three-dimensional networks, depending on
into a nucleon that is more stable; an unstable nucleon the other elements involved in the formation of the
is said to be radioactive
Resonance hybrid | average of the resonance compounds
forms shown by the individual Lewis structures
Radiocarbon dating | highly accurate means of Simple cubic structure | crystalline structure with
Reversible process | process that takes place so a cubic unit cell with lattice points only at the corners
dating objects 30,000–50,000 years old that were
slowly as to be capable of reversing direction in
derived from once-living matter; achieved by
response to an infinitesimally small change in Simple cubic unit cell | (also, primitive cubic unit
calculating the ratio of C : C in the object vs. the
14 12
6 6
conditions; hypothetical construct that can only be cell) unit cell in the simple cubic structure
ratio of C : C in the present-day atmosphere
14 12
approximated by real processes removed
Single bond | bond in which a single pair of
6 6

Radioisotope | isotope that is unstable and Reversible reaction | chemical reaction that can electrons is shared between two atoms
undergoes conversion into a different, more stable proceed in both the forward and reverse directions
isotope under given conditions
Skeletal structure | shorthand method of drawing
organic molecules in which carbon atoms are
Radiometric dating | use of radioisotopes and Roentgen equivalent man (rem) | unit for represented by the ends of lines and bends in between
their properties to date the formation of objects such as radiation damage, frequently used in medicine; 1 rem lines, and hydrogen atoms attached to the carbon
archeological artifacts, formerly living organisms, or = 1 Sv atoms are not shown (but are understood to be present
geological formations by the context of the structure)
Rounding | procedure used to ensure that calculated
Raoult’s law | the partial pressure exerted by a results properly reflect the uncertainty in the Smelting | process of extracting a pure metal from a
solution component is equal to the product of the measurements used in the calculation molten ore
component’s mole fraction in the solution and its
equilibrium vapor pressure in the pure state s orbital | spherical region of space with high Solid | state of matter that is rigid, has a definite
electron density, describes orbitals with l = 0. An shape, and has a fairly constant volume
Rare earth element | collection of 17 elements electron in this orbital is called an s electron
including the lanthanides, scandium, and yttrium that Solubility | extent to which a solute may be
often occur together and have similar chemical Sacrificial anode | more active, inexpensive metal dissolved in water, or any solvent
properties, making separation difficult used as the anode in cathodic protection; frequently
made from magnesium or zinc
Solubility product (Ksp) | equilibrium constant
Rate constant (k) | proportionality constant in the for the dissolution of a slightly soluble electrolyte
relationship between reaction rate and concentrations Saturated | of concentration equal to solubility;
containing the maximum concentration of solute Solute | solution component present in a
of reactants concentration less than that of the solvent
possible for a given temperature and pressure
Rate expression | mathematical representation Solvation | exothermic process in which
relating reaction rate to changes in amount, Saturated hydrocarbon | molecule containing
intermolecular attractive forces between the solute and
concentration, or pressure of reactant or product carbon and hydrogen that has only single bonds
solvent in a solution are established
species per unit time between carbon atoms
Scientific method | path of discovery that leads Solvent | solution component present in a
Rate law | (also, rate equation) mathematical concentration that is higher relative to other
equation showing the dependence of reaction rate on from question and observation to law or hypothesis to
theory, combined with experimental verification of the components
the rate constant and the concentration of one or more
reactants hypothesis and any necessary modification of the Space lattice | all points within a crystal that have
theory identical environments
Rate of reaction | measure of the speed at which a
chemical reaction takes place Scintillation counter | instrument that uses a Spatial isomers | compounds in which the relative
scintillator—a material that emits light when excited orientations of the atoms in space differ
Reactant | substance undergoing a chemical or by ionizing radiation—to detect and measure radiation
physical change; shown on the left side of the arrow in Specific heat capacity (c) | intensive property of
a chemical equation Second (s) | SI unit of time a substance that represents the quantity of heat
Second law of thermodynamics | entropy of the required to raise the temperature of 1 gram of the
Reaction order | value of an exponent in a rate law, substance by 1 degree Celsius (or 1 kelvin)
expressed as an ordinal number (for example, zero universe increases for a spontaneous process
order for 0, first order for 1, second order for 2, and so Second transition series | transition elements in Spectator ion | ion that does not undergo a
on) chemical or physical change during a reaction, but its
the fifth period of the periodic table (second row of the
presence is required to maintain charge neutrality
Reaction quotient (Q) | ratio of the product of d-block), atomic numbers 39–47
molar concentrations (or pressures) of the products to Selective precipitation | process in which ions are Spectrochemical series | ranking of ligands
that of the reactants, each concentration (or pressure) separated using differences in their solubility with a according to the magnitude of the crystal field splitting
being raised to the power equal to the coefficient in the given precipitating reagent they induce
equation Spin quantum number (ms) | number specifying
Semipermeable membrane | a membrane that
Reactor coolant | assembly used to carry the heat selectively permits passage of certain ions or the electron spin direction, either +
1
or −
1

produced by fission in a reactor to an external boiler molecules 2 2

and turbine where it is transformed into electricity

8 https://chem.libretexts.org/@go/page/279192
Spontaneous change | process that takes place Strong electrolyte | substance that dissociates or Thermal energy | kinetic energy associated with
without a continuous input of energy from an external ionizes completely when dissolved in water the random motion of atoms and molecules
source
Strong nuclear force | force of attraction between Thermochemistry | study of measuring the
Spontaneous process | physical or chemical nucleons that holds a nucleus together amount of heat absorbed or released during a chemical
change that occurs without the addition of energy from reaction or a physical change
an external source Strong-field ligand | ligand that causes larger
crystal field splittings Third law of thermodynamics | entropy of a
Standard cell potential (E

cell
) | the cell Structural formula | shows the atoms in a
perfect crystal at absolute zero (0 K) is zero
potential when all reactants and products are in their molecule and how they are connected Third transition series | transition elements in
standard states (1 bar or 1 atm or gases; 1 M for the sixth period of the periodic table (third row of the
solutes), usually at 298.15 K; can be calculated by Structural isomer | one of two substances that d-block), atomic numbers 57 and 72–79
subtracting the standard reduction potential for the have the same molecular formula but different physical
half-reaction at the anode from the standard reduction and chemical properties because their atoms are Titrant | solution containing a known concentration
potential for the half-reaction occurring at the cathode bonded differently of substance that will react with the analyte in a
titration analysis
Standard enthalpy of combustion ( ∘
ΔHc ) | Subcritical mass | amount of fissionable material
heat released when one mole of a compound that cannot sustain a chain reaction; less than a critical Titration analysis | quantitative chemical analysis
undergoes complete combustion under standard mass method that involves measuring the volume of a
conditions reactant solution required to completely react with the
Sublimation | change from solid state directly to analyte in a sample
gaseous state
Standard enthalpy of formation (ΔH ) | ∘
f Titration curve | plot of the pH of a solution of
enthalpy change of a chemical reaction in which 1 Subshell | set of orbitals in an atom with the same acid or base versus the volume of base or acid added
mole of a pure substance is formed from its elements values of n and l during a titration
in their most stable states under standard state
Substituent | branch or functional group that
conditions Torr | unit of pressure; 1 torr =
1
replaces hydrogen atoms in a larger hydrocarbon chain atm
760
Standard entropy (S°) | entropy for a substance
at 1 bar pressure; tabulated values are usually
Substitution reaction | reaction in which one trans configuration | configuration of a
atom replaces another in a molecule geometrical isomer in which two similar groups are on
determined at 298.15 K and denoted S ∘
298

Sulfate | SO 2−
ion opposite sides of an imaginary reference line on the
Standard entropy change (ΔS°) | change in 4
molecule
entropy for a reaction calculated using the standard Sulfite | SO 2−
ion
entropies, usually at room temperature and denoted
3
Transition metal | element in columns 3–11
ΔS

298
Superconductor | material that conducts electricity Transmutation reaction | bombardment of one
with no resistance type of nuclei with other nuclei or neutrons
Standard free energy change (ΔG°) | change
in free energy for a process occurring under standard Supercritical fluid | substance at a temperature Transuranium element | element with an atomic
conditions (1 bar pressure for gases, 1 M concentration and pressure higher than its critical point; exhibits
number greater than 92; these elements do not occur in
for solutions) properties intermediate between those of gaseous and
nature
liquid states
Standard free energy of formation (ΔG ∘
) | Trigonal bipyramidal | shape in which five
f
Supercritical mass | amount of material in which outside groups are placed around a central atom such
change in free energy accompanying the formation of there is an increasing rate of fission
one mole of substance from its elements in their that three form a flat triangle with 120° angles between
standard states Superoxide | oxide containing the anion O −
each pair and the central atom, and the other two form
2
the apex of two pyramids, one above and one below
Standard hydrogen electrode (SHE) | the Supersaturated | of concentration that exceeds the triangular plane
electrode consists of hydrogen gas bubbling through solubility; a nonequilibrium state
hydrochloric acid over an inert platinum electrode Trigonal planar | shape in which three outside
whose reduction at standard conditions is assigned a Surface tension | energy required to increase the groups are placed in a flat triangle around a central
value of 0 V; the reference point for standard reduction area, or length, of a liquid surface by a given amount atom with 120° angles between each pair and the
potentials central atom
Surroundings | all matter other than the system
Standard reduction potential (E°) | the value being studied Triple bond | bond in which three pairs of electrons
of the reduction under standard conditions (1 bar or 1 are shared between two atoms
Symbolic domain | specialized language used to
atm for gases; 1 M for solutes) usually at 298.15 K; represent components of the macroscopic and Triple point | temperature and pressure at which the
tabulated values used to calculate standard cell microscopic domains, such as chemical symbols, vapor, liquid, and solid phases of a substance are in
potentials chemical formulas, chemical equations, graphs, equilibrium
Standard state | set of physical conditions as drawings, and calculations
Triprotic acid | acid that contains three ionizable
accepted as common reference conditions for reporting System | portion of matter undergoing a chemical or hydrogen atoms per molecule; ionization of triprotic
thermodynamic properties; 1 bar of pressure, and physical change being studied acids occurs in three steps
solutions at 1 molar concentrations, usually at a
temperature of 298.15 K t2g orbitals | set of three d orbitals aligned between Tyndall effect | scattering of visible light by a
the Cartesian axes for coordination complexes; in colloidal dispersion
Standing wave | (also, stationary wave) localized octahedral complexes, they are lowered in energy
wave phenomenon characterized by discrete compared to the eg orbitals according to CFT Uncertainty | estimate of amount by which
wavelengths determined by the boundary conditions measurement differs from true value
used to generate the waves; standing waves are Temperature | intensive property of matter that is a
inherently quantized quantitative measure of “hotness” and “coldness” Unified atomic mass unit (u) | alternative unit
equivalent to the atomic mass unit
State function | property depending only on the Tetrahedral | shape in which four outside groups
state of a system, and not the path taken to reach that are placed around a central atom such that a three- Unit | standard of comparison for measurements
state dimensional shape is generated with four corners and Unit cell | smallest portion of a space lattice that is
109.5° angles between each pair and the central atom repeated in three dimensions to form the entire lattice
Steel | material made from iron by removing
impurities in the iron and adding substances that Tetrahedral hole | tetrahedral space formed by Unit conversion factor | ratio of equivalent
produce alloys with properties suitable for specific four atoms or ions in a crystal quantities expressed with different units; used to
uses convert from one unit to a different unit
Theoretical yield | amount of product that may be
Stepwise ionization | process in which an acid is produced from a given amount of reactant(s) according Unsaturated | of concentration less than solubility
ionized by losing protons sequentially to the reaction stoichiometry
Vacancy | defect that occurs when a position that
Stress | change to a reaction's conditions that may Theory | well-substantiated, comprehensive, testable should contain an atom or ion is vacant
cause a shift in the equilibrium explanation of a particular aspect of nature

9 https://chem.libretexts.org/@go/page/279192
Valence bond theory | description of bonding that Van’t Hoff factor (i) | the ratio of the number of Wavefunction (ψ) | mathematical description of an
involves atomic orbitals overlapping to form σ or π moles of particles in a solution to the number of moles atomic orbital that describes the shape of the orbital; it
bonds, within which pairs of electrons are shared of formula units dissolved in the solution can be used to calculate the probability of finding the
electron at any given location in the orbital, as well as
Valence electrons | electrons in the outermost or Vapor pressure | (also, equilibrium vapor pressure) dynamical variables such as the energy and the angular
valence shell (highest value of n) of a ground-state pressure exerted by a vapor in equilibrium with a solid
momentum
atom; determine how an element reacts or a liquid at a given temperature
Wavelength (λ) | distance between two consecutive
Valence shell | outermost shell of electrons in a Vaporization | change from liquid state to gaseous peaks or troughs in a wave
ground-state atom; for main group elements, the state
orbitals with the highest n level (s and p subshells) are Weak electrolyte | substance that ionizes only
in the valence shell, while for transition metals, the
Vector | quantity having magnitude and direction partially when dissolved in water
highest energy s and d subshells make up the valence Viscosity | measure of a liquid’s resistance to flow
shell and for inner transition elements, the highest s, d,
Weak-field ligand | ligand that causes small
Voltaic cell | another name for a galvanic cell crystal field splittings
and f subshells are included
Valence shell electron-pair repulsion theory Volume | amount of space occupied by an object Weight | force that gravity exerts on an object
(VSEPR) | theory used to predict the bond angles in Volume percentage | ratio of solute-to-solution Work (w) | energy transfer due to changes in
a molecule based on positioning regions of high external, macroscopic variables such as pressure and
volume expressed as a percentage
electron density as far apart as possible to minimize volume; or causing matter to move against an
electrostatic repulsion Wave | oscillation that can transport energy from one opposing force
point to another in space
Van der Waals equation | modified version of X-ray crystallography | experimental technique
the ideal gas equation containing additional terms to Wave-particle duality | term used to describe the for determining distances between atoms in a crystal
account for non-ideal gas behavior fact that elementary particles including matter exhibit by measuring the angles at which X-rays are diffracted
properties of both particles (including localized when passing through the crystal
Van der Waals force | attractive or repulsive force position, momentum) and waves (including
between molecules, including dipole-dipole, dipole-
nonlocalization, wavelength, frequency)
induced dipole, and London dispersion forces; does
not include forces due to covalent or ionic bonding, or
the attraction between ions and molecules

10 https://chem.libretexts.org/@go/page/279192
11 https://chem.libretexts.org/@go/page/279192
12 https://chem.libretexts.org/@go/page/279192
13 https://chem.libretexts.org/@go/page/279192
14 https://chem.libretexts.org/@go/page/279192
15 https://chem.libretexts.org/@go/page/279192
16 https://chem.libretexts.org/@go/page/279192
17 https://chem.libretexts.org/@go/page/279192
18 https://chem.libretexts.org/@go/page/279192
19 https://chem.libretexts.org/@go/page/279192
20 https://chem.libretexts.org/@go/page/279192
21 https://chem.libretexts.org/@go/page/279192
22 https://chem.libretexts.org/@go/page/279192
23 https://chem.libretexts.org/@go/page/279192
24 https://chem.libretexts.org/@go/page/279192
25 https://chem.libretexts.org/@go/page/279192
26 https://chem.libretexts.org/@go/page/279192
27 https://chem.libretexts.org/@go/page/279192
28 https://chem.libretexts.org/@go/page/279192
29 https://chem.libretexts.org/@go/page/279192
30 https://chem.libretexts.org/@go/page/279192
31 https://chem.libretexts.org/@go/page/279192
32 https://chem.libretexts.org/@go/page/279192
33 https://chem.libretexts.org/@go/page/279192
34 https://chem.libretexts.org/@go/page/279192
35 https://chem.libretexts.org/@go/page/279192
36 https://chem.libretexts.org/@go/page/279192
37 https://chem.libretexts.org/@go/page/279192
38 https://chem.libretexts.org/@go/page/279192
39 https://chem.libretexts.org/@go/page/279192
40 https://chem.libretexts.org/@go/page/279192
41 https://chem.libretexts.org/@go/page/279192
42 https://chem.libretexts.org/@go/page/279192
43 https://chem.libretexts.org/@go/page/279192
44 https://chem.libretexts.org/@go/page/279192
45 https://chem.libretexts.org/@go/page/279192
46 https://chem.libretexts.org/@go/page/279192
47 https://chem.libretexts.org/@go/page/279192
48 https://chem.libretexts.org/@go/page/279192
49 https://chem.libretexts.org/@go/page/279192
50 https://chem.libretexts.org/@go/page/279192
51 https://chem.libretexts.org/@go/page/279192
52 https://chem.libretexts.org/@go/page/279192
53 https://chem.libretexts.org/@go/page/279192
54 https://chem.libretexts.org/@go/page/279192
55 https://chem.libretexts.org/@go/page/279192
56 https://chem.libretexts.org/@go/page/279192
57 https://chem.libretexts.org/@go/page/279192
58 https://chem.libretexts.org/@go/page/279192
59 https://chem.libretexts.org/@go/page/279192
60 https://chem.libretexts.org/@go/page/279192
61 https://chem.libretexts.org/@go/page/279192
62 https://chem.libretexts.org/@go/page/279192
63 https://chem.libretexts.org/@go/page/279192
64 https://chem.libretexts.org/@go/page/279192
65 https://chem.libretexts.org/@go/page/279192
66 https://chem.libretexts.org/@go/page/279192
67 https://chem.libretexts.org/@go/page/279192
68 https://chem.libretexts.org/@go/page/279192
69 https://chem.libretexts.org/@go/page/279192
70 https://chem.libretexts.org/@go/page/279192
71 https://chem.libretexts.org/@go/page/279192
72 https://chem.libretexts.org/@go/page/279192
73 https://chem.libretexts.org/@go/page/279192
74 https://chem.libretexts.org/@go/page/279192
75 https://chem.libretexts.org/@go/page/279192
76 https://chem.libretexts.org/@go/page/279192
77 https://chem.libretexts.org/@go/page/279192
78 https://chem.libretexts.org/@go/page/279192
79 https://chem.libretexts.org/@go/page/279192
80 https://chem.libretexts.org/@go/page/279192
81 https://chem.libretexts.org/@go/page/279192
82 https://chem.libretexts.org/@go/page/279192
83 https://chem.libretexts.org/@go/page/279192
84 https://chem.libretexts.org/@go/page/279192
85 https://chem.libretexts.org/@go/page/279192
86 https://chem.libretexts.org/@go/page/279192
87 https://chem.libretexts.org/@go/page/279192
88 https://chem.libretexts.org/@go/page/279192
89 https://chem.libretexts.org/@go/page/279192
90 https://chem.libretexts.org/@go/page/279192
91 https://chem.libretexts.org/@go/page/279192
92 https://chem.libretexts.org/@go/page/279192
93 https://chem.libretexts.org/@go/page/279192
94 https://chem.libretexts.org/@go/page/279192
95 https://chem.libretexts.org/@go/page/279192
96 https://chem.libretexts.org/@go/page/279192
97 https://chem.libretexts.org/@go/page/279192
98 https://chem.libretexts.org/@go/page/279192
99 https://chem.libretexts.org/@go/page/279192
100 https://chem.libretexts.org/@go/page/279192
101 https://chem.libretexts.org/@go/page/279192
102 https://chem.libretexts.org/@go/page/279192
103 https://chem.libretexts.org/@go/page/279192
104 https://chem.libretexts.org/@go/page/279192
105 https://chem.libretexts.org/@go/page/279192
106 https://chem.libretexts.org/@go/page/279192
107 https://chem.libretexts.org/@go/page/279192
108 https://chem.libretexts.org/@go/page/279192
109 https://chem.libretexts.org/@go/page/279192
110 https://chem.libretexts.org/@go/page/279192
111 https://chem.libretexts.org/@go/page/279192
112 https://chem.libretexts.org/@go/page/279192
113 https://chem.libretexts.org/@go/page/279192
114 https://chem.libretexts.org/@go/page/279192
115 https://chem.libretexts.org/@go/page/279192
116 https://chem.libretexts.org/@go/page/279192
117 https://chem.libretexts.org/@go/page/279192
118 https://chem.libretexts.org/@go/page/279192
119 https://chem.libretexts.org/@go/page/279192
120 https://chem.libretexts.org/@go/page/279192
121 https://chem.libretexts.org/@go/page/279192
122 https://chem.libretexts.org/@go/page/279192
123 https://chem.libretexts.org/@go/page/279192
124 https://chem.libretexts.org/@go/page/279192
Detailed Licensing
Overview
Title: Chemistry 1e (OpenSTAX)
Webpages: 189
All licenses found:
CC BY 4.0: 95.2% (180 pages)
Undeclared: 4.8% (9 pages)

By Page
Chemistry 1e (OpenSTAX) - CC BY 4.0 4: Stoichiometry of Chemical Reactions - CC BY 4.0
Front Matter - Undeclared 4.0: Prelude to Stoichiometry - CC BY 4.0
TitlePage - Undeclared 4.1: Writing and Balancing Chemical Equations - CC
InfoPage - Undeclared BY 4.0
Table of Contents - Undeclared 4.2: Classifying Chemical Reactions - CC BY 4.0
Licensing - Undeclared 4.3: Reaction Stoichiometry - CC BY 4.0
1: Essential Ideas of Chemistry - CC BY 4.0 4.4: Reaction Yields - CC BY 4.0
4.5: Quantitative Chemical Analysis - CC BY 4.0
1.1: Chemistry in Context - CC BY 4.0
4.E: Stoichiometry of Chemical Reactions (Exercises)
1.2: Phases and Classification of Matter - CC BY 4.0
- CC BY 4.0
1.3: Physical and Chemical Properties - CC BY 4.0
1.4: Measurements - CC BY 4.0 5: Thermochemistry - CC BY 4.0
1.5: Measurement Uncertainty, Accuracy, and 5.0: Prelude to Thermochemistry - CC BY 4.0
Precision - CC BY 4.0 5.1: Energy Basics - CC BY 4.0
1.6: Mathematical Treatment of Measurement Results 5.2: Calorimetry - CC BY 4.0
- CC BY 4.0 5.3: Enthalpy - CC BY 4.0
1.E: Essential Ideas of Chemistry (Exercises) - CC 5.E: Thermochemistry (Exercises) - CC BY 4.0
BY 4.0 6: Electronic Structure and Periodic Properties - CC BY
2: Atoms, Molecules, and Ions - CC BY 4.0 4.0
2.0: Prelude to Atoms - CC BY 4.0 6.1: Electromagnetic Energy - CC BY 4.0
2.1: Early Ideas in Atomic Theory - CC BY 4.0 6.2: The Bohr Model - CC BY 4.0
2.2: Evolution of Atomic Theory - CC BY 4.0 6.3: Development of Quantum Theory - CC BY 4.0
2.3: Atomic Structure and Symbolism - CC BY 4.0 6.4: Electronic Structure of Atoms (Electron
2.4: Chemical Formulas - CC BY 4.0 Configurations) - CC BY 4.0
2.5: The Periodic Table - CC BY 4.0 6.5: Periodic Variations in Element Properties - CC
2.6: Molecular and Ionic Compounds - CC BY 4.0 BY 4.0
2.7: Chemical Nomenclature - CC BY 4.0 6.E: Electronic Structure and Periodic Properties
2.E: Atoms, Molecules, and Ions (Exercises) - CC BY (Exercises) - CC BY 4.0
4.0 7: Chemical Bonding and Molecular Geometry - CC BY
3: Composition of Substances and Solutions - CC BY 4.0 4.0
3.1: Formula Mass and the Mole Concept - CC BY 7.0: Prelude to Chemical Bonding and Molecular
4.0 Geometry - CC BY 4.0
3.2: Determining Empirical and Molecular Formulas 7.1: Ionic Bonding - CC BY 4.0
- CC BY 4.0 7.2: Covalent Bonding - CC BY 4.0
3.3: Molarity - CC BY 4.0 7.3: Lewis Symbols and Structures - CC BY 4.0
3.4: Other Units for Solution Concentrations - CC BY 7.4: Formal Charges and Resonance - CC BY 4.0
4.0 7.5: Strengths of Ionic and Covalent Bonds - CC BY
3.E: Composition of Substances and Solutions 4.0
(Exercises) - CC BY 4.0 7.6: Molecular Structure and Polarity - CC BY 4.0

1 https://chem.libretexts.org/@go/page/417134
7.E: Chemical Bonding and Molecular Geometry 13.1: Chemical Equilibria - CC BY 4.0
(Exercises) - CC BY 4.0 13.2: Equilibrium Constants - CC BY 4.0
8: Advanced Theories of Covalent Bonding - CC BY 4.0 13.3: Shifting Equilibria - Le Chatelier’s Principle -
8.0: Prelude to Covalent Bonding - CC BY 4.0 CC BY 4.0
8.1: Valence Bond Theory - CC BY 4.0 13.4: Equilibrium Calculations - CC BY 4.0
8.2: Hybrid Atomic Orbitals - CC BY 4.0 13.E: Fundamental Equilibrium Concepts (Exercises)
8.3: Multiple Bonds - CC BY 4.0 - CC BY 4.0
8.4: Molecular Orbital Theory - CC BY 4.0 14: Acid-Base Equilibria - CC BY 4.0
8.E: Advanced Theories of Covalent Bonding 14.1: Brønsted-Lowry Acids and Bases - CC BY 4.0
(Exercises) - CC BY 4.0 14.2: pH and pOH - CC BY 4.0
9: Gases - CC BY 4.0 14.3: Relative Strengths of Acids and Bases - CC BY
4.0
9.1: Gas Pressure - CC BY 4.0
14.4: Hydrolysis of Salt Solutions - CC BY 4.0
9.2: Relating Pressure, Volume, Amount, and
14.5: Polyprotic Acids - CC BY 4.0
Temperature - The Ideal Gas Law - CC BY 4.0
14.6: Buffers - CC BY 4.0
9.3: Stoichiometry of Gaseous Substances, Mixtures,
14.7: Acid-Base Titrations - CC BY 4.0
and Reactions - CC BY 4.0
14.E: Acid-Base Equilibria (Exercises) - CC BY 4.0
9.4: Effusion and Diffusion of Gases - CC BY 4.0
9.5: The Kinetic-Molecular Theory - CC BY 4.0 15: Equilibria of Other Reaction Classes - CC BY 4.0
9.6: Non-Ideal Gas Behavior - CC BY 4.0 15.1: Precipitation and Dissolution - CC BY 4.0
9.E: Gases (Exercises) - CC BY 4.0 15.2: Lewis Acids and Bases - CC BY 4.0
10: Liquids and Solids - CC BY 4.0 15.3: Coupled Equilibria - CC BY 4.0
10.0: Prelude to Liquids and Solids - CC BY 4.0 15.E: Equilibria of Other Reaction Classes
10.1: Intermolecular Forces - CC BY 4.0 (Exercises) - CC BY 4.0
10.2: Properties of Liquids - CC BY 4.0 16: Thermodynamics - CC BY 4.0
10.3: Phase Transitions - CC BY 4.0 16.1: Spontaneity - CC BY 4.0
10.4: Phase Diagrams - CC BY 4.0 16.2: Entropy - CC BY 4.0
10.5: The Solid State of Matter - CC BY 4.0 16.3: The Second and Third Laws of
10.6: Lattice Structures in Crystalline Solids - CC BY Thermodynamics - CC BY 4.0
4.0 16.4: Gibbs Energy - CC BY 4.0
10.E: Liquids and Solids (Exercises) - CC BY 4.0 16.E: Thermodynamics (Exercises) - CC BY 4.0
11: Solutions and Colloids - CC BY 4.0 17: Electrochemistry - CC BY 4.0
11.0: Prelude to Solutions and Colloids - CC BY 4.0 17.1: Balancing Oxidation-Reduction Reactions - CC
11.1: The Dissolution Process - CC BY 4.0 BY 4.0
11.2: Electrolytes - CC BY 4.0 17.2: Galvanic Cells - CC BY 4.0
11.3: Solubility - CC BY 4.0 17.3: Standard Reduction Potentials - CC BY 4.0
11.4: Colligative Properties - CC BY 4.0 17.4: The Nernst Equation - CC BY 4.0
11.5: Colloids - CC BY 4.0 17.5: Batteries and Fuel Cells - CC BY 4.0
11.E: Solutions and Colloids (Exercises) - CC BY 4.0 17.6: Corrosion - CC BY 4.0
12: Kinetics - CC BY 4.0 17.7: Electrolysis - CC BY 4.0
12.0: Prelude to Kinetics - CC BY 4.0 17.E: Electrochemistry (Exercises) - CC BY 4.0
12.1: Chemical Reaction Rates - CC BY 4.0 18: Representative Metals, Metalloids, and Nonmetals -
12.2: Factors Affecting Reaction Rates - CC BY 4.0 CC BY 4.0
12.3: Rate Laws - CC BY 4.0 18.1: Periodicity - CC BY 4.0
12.4: Integrated Rate Laws - CC BY 4.0 18.2: Occurrence and Preparation of the
12.5: Collision Theory - CC BY 4.0 Representative Metals - CC BY 4.0
12.6: Reaction Mechanisms - CC BY 4.0 18.3: Structure and General Properties of the
12.7: Catalysis - CC BY 4.0 Metalloids - CC BY 4.0
12.E: Kinetics (Exercises) - CC BY 4.0 18.4: Structure and General Properties of the
13: Fundamental Equilibrium Concepts - CC BY 4.0 Nonmetals - CC BY 4.0
13.0: Prelude to Equilibrium - CC BY 4.0

2 https://chem.libretexts.org/@go/page/417134
18.5: Occurrence, Preparation, and Compounds of 20.3: Aldehydes, Ketones, Carboxylic Acids, and
Hydrogen - CC BY 4.0 Esters - CC BY 4.0
18.6: Occurrence, Preparation, and Properties of 20.4: Amines and Amides - CC BY 4.0
Carbonates - CC BY 4.0 20.E: Organic Chemistry (Exercises) - CC BY 4.0
18.7: Occurrence, Preparation, and Properties of 21: Nuclear Chemistry - CC BY 4.0
Nitrogen - CC BY 4.0 21.1: Nuclear Structure and Stability - CC BY 4.0
18.8: Occurrence, Preparation, and Properties of 21.2: Nuclear Equations - CC BY 4.0
Phosphorus - CC BY 4.0 21.3: Radioactive Decay - CC BY 4.0
18.9: Occurrence, Preparation, and Compounds of 21.4: Transmutation and Nuclear Energy - CC BY 4.0
Oxygen - CC BY 4.0 21.5: Uses of Radioisotopes - CC BY 4.0
18.10: Occurrence, Preparation, and Properties of 21.6: Biological Effects of Radiation - CC BY 4.0
Sulfur - CC BY 4.0 21.E: Nuclear Chemistry (Exercises) - CC BY 4.0
18.11: Occurrence, Preparation, and Properties of
Appendices - CC BY 4.0
Halogens - CC BY 4.0
18.12: Occurrence, Preparation, and Properties of the Composition of Commercial Acids and Bases - CC
Noble Gases - CC BY 4.0 BY 4.0
18.E: Representative Metals, Metalloids, and Essential Mathematics - CC BY 4.0
Nonmetals (Exercises) - CC BY 4.0 Formation Constants for Complex Ions - CC BY 4.0
Fundamental Physical Constants - CC BY 4.0
19: Transition Metals and Coordination Chemistry - CC
Ionization Constants of Weak Acids - CC BY 4.0
BY 4.0
Ionization Constants of Weak Bases - CC BY 4.0
19.1: Properties of Transition Metals and Their
Solubility Products - CC BY 4.0
Compounds - CC BY 4.0
Standard Electrode (Half-Cell) Potentials - CC BY 4.0
19.2: Coordination Chemistry of Transition Metals -
Standard Thermodynamic Properties for Selected
CC BY 4.0
Substances - CC BY 4.0
19.3: Optical and Magnetic Properties of
The Periodic Table - CC BY 4.0
Coordination Compounds - CC BY 4.0
Units and Conversion Factors - CC BY 4.0
19.E: Transition Metals and Coordination Chemistry
Water Properties - CC BY 4.0
(Exercises) - CC BY 4.0
Back Matter - Undeclared
20: Organic Chemistry - CC BY 4.0
Index - Undeclared
20.0: Prelude to Organic Chemistry - CC BY 4.0 Glossary - Undeclared
20.1: Hydrocarbons - CC BY 4.0 Detailed Licensing - Undeclared
20.2: Alcohols and Ethers - CC BY 4.0

3 https://chem.libretexts.org/@go/page/417134

You might also like